Вы находитесь на странице: 1из 2645

Problem P1.

2-11 (1-4 in text)


Figure P1.2-11(a) illustrates a composite wall. The wall is composed of two materials (A with kA
= 1 W/m-K and B with kB = 5 W/m-K), each has thickness L = 1.0 cm. The surface of the wall at
x = 0 is perfectly insulated. A very thin heater is placed between the insulation and material A;
the heating element provides q ′′ = 5000 W/m 2 of heat. The surface of the wall at x = 2L is
exposed to fluid at Tf,in = 300 K with heat transfer coefficient hin = 100 W/m2-K.
material A
q ′′ = 5000 W/m
2
kA = 1 W/m-K
insulated L = 1 cm

x T f ,in = 300 K
hin = 100 W/m -K
2

L = 1 cm material B
kB = 5 W/m-K
Figure P1.2-11(a): Composite wall with a heater.

You may neglect radiation and contact resistance for parts (a) through (c) of this problem.
a.) Draw a resistance network to represent this problem; clearly indicate what each resistance
represents and calculate the value of each resistance.

The input parameters are entered in EES:

“P1.2-11: Heater"
$UnitSystem SI MASS RAD PA K J
$TABSTOPS 0.2 0.4 0.6 0.8 3.5 in

"Inputs"
q_flux=100 [W/m^2] "heat flux provided by the heater"
L = 1.0 [cm]*convert(cm,m) "thickness of each layer"
k_A=1.0 [W/m-K] "conductivity of material A"
k_B=5.0 [W/m-K] "conductivity of material B"
T_f_in=300 [K] "fluid temperature at inside surface"
h_in=100 [W/m^2-K] "heat transfer on inside surface"
A=1 [m^2] "per unit area"

The resistance network that represents the problem shown in Figure 2 is:

Figure 2: Resistance network.

The resistances due to conduction through materials A and B are:


L
RA = (1)
kA A

L
RB = (2)
kB A

where A is the area of the wall, taken to be 1 m2 in order to carry out the analysis on a per unit
area basis. The resistance due to convection is:

1
Rconv ,in = (3)
hin A

"part (a)"
R_A=L/(k_A*A) "resistance to conduction through A"
R_B=L/(k_B*A) "resistance to conduction through B"
R_conv_in=1/(h_in*A)
"resistance to convection on inner surface"

which leads to RA = 0.01 K/W, RB = 0.002 K/W, and Rconv,in = 0.01 K/W.

b.) Use your resistance network from (a) to determine the temperature of the heating element.

The resistance network for this problem is simple; the temperature drop across each resistor is
equal to the product of the heat transferred through the resistor and its resistance. In this simple
case, all of the heat provided by the heater must pass through materials A, B, and into the fluid
by convection so these resistances are in series. The heater temperature (Thtr) is therefore:

Thtr = T f ,in + ( RA + RB + Rconv ,in ) q ′′ A (4)

T_htr=T_f_in+(R_A+R_B+R_conv_in)*q_flux*A "heater temperature"

which leads to Thtr = 410 K.

c.) Sketch the temperature distribution on the axes provided below. Make sure that the sketch is
consistent with your solution from (b).

The temperatures at x = L and x = 2L can be computed according to:

Tx = L = T f ,in + ( RB + Rconv ,in ) q ′′ A (5)

Tx = 2 L = T f ,in + Rconv ,in q ′′ A (6)

T_L=T_f_in+(R_B+R_conv_in)*q_flux*A "temperature at x=L"


T_2L=T_f_in+R_conv_in*q_flux*A "temperature at x=2L"
which leads to Tx=L = 360 K and Tx=2L = 350 K. The temperature distribution is sketched on the
axes in Figure 3.

Figure 3: Sketch of temperature distribution.

Notice that the temperature drop through the two larger resistances (RA and RB) are much larger
than the temperature drop across the small resistance, RB.

Figure P1.2-11(b) illustrates the same composite wall shown in Figure P1.2-11(a), but there is an
additional layer added to the wall, material C with kC = 2.0 W/m-K and L = 1.0 cm.

q ′′ = 5000 W/m
2
material A
material C kA = 1 W/m-K
kC = 2 W/m-K
L = 1 cm
insulated

x T f ,in = 300 K
hin = 100 W/m -K
2

material B
L = 1 cm L = 1 cm k = 5 W/m-K
B
Figure P1.2-11(b): Composite wall with Material C.

Neglect radiation and contact resistance for parts (d) through (f) of this problem.
d.) Draw a resistance network to represent the problem shown in Figure P1.2-11(b); clearly
indicate what each resistance represents and calculate the value of each resistance.

There is an additional resistor corresponding to conduction through material C, RC, as shown


below:
Notice that the boundary condition at the end of RC corresponds to the insulated wall; that is, no
heat can be transferred through this resistance. The resistance to conduction through material C
is:

L
RC = (7)
kC A

"part (b)"
k_C=2.0 [W/m-K] "conductivity of material C"
R_C=L/(k_C*A) "resistance to conduction through C"

which leads to RC = 0.005 K/W.

e.) Use your resistance network from (d) to determine the temperature of the heating element.

Because there is no heat transferred through RC, all of the heat must still go through materials A
and B and be convected from the inner surface of the wall. Therefore, the answer is not changed
from part (b), Thtr = 410 K.

f.) Sketch the temperature distribution on the axes provided below. Make sure that the sketch is
consistent with your solution from (e).

The answer is unchanged from part (c) except that there is material to the left of the heater.
However, no heat is transferred through material C and therefore there is no temperature gradient
in the material.
Figure P1.2-11(c) illustrates the same composite wall shown in Figure P1.2-11(b), but there is a
contact resistance between materials A and B, Rc′′ = 0.01 K-m 2 /W , and the surface of the wall at
x = -L is exposed to fluid at Tf,out = 400 K with a heat transfer coefficient hout = 10 W/m2-K.

q ′′ = 5000 W/m
2
material A
material C kA = 1 W/m-K
kC = 2 W/m-K
L = 1 cm

T f ,out = 400 K
T f ,in = 300 K
hout = 10 W/m -K x
2

hin = 100 W/m -K


2

material B
L = 1 cm kB = 5 W/m-K
L = 1 cm
Rc′′ = 0.01 K-m /W
2

Figure P1.2-11(c): Composite wall with convection at the outer surface and contact resistance.

Neglect radiation for parts (g) through (i) of this problem.


g.) Draw a resistance network to represent the problem shown in Figure P1.2-11(c); clearly
indicate what each resistance represents and calculate the value of each resistance.

The additional resistances associated with contact resistance and convection to the fluid at the
outer surface are indicated. Notice that the boundary condition has changed; heat provided by
the heater has two paths ( qout and qin ) and so the problem is not as easy to solve.
The additional resistances are computed according to:

1
Rconv ,out = (8)
hout A

Rc′′
Rcontact = (9)
A

"part (c)"
R``_c=0.01 [K-m^2/W] "area specific contact resistance"
h_out=10 [W/m^2-K] "heat transfer coefficient"
T_f_out=400 [K] "fluid temperature on outside surface"
R_contact=R``_c/A "contact resistance"
R_conv_out=1/(h_out*A)
"convection resistance on outer surface"

which leads to Rcontact = 0.01 K/W and Rconv,out = 0.1 K/W.

h.) Use your resistance network from (j) to determine the temperature of the heating element.

It is necessary to carry out an energy balance on the heater:

q ′′ A = qin + qout (10)

The heat transfer rates can be related to Thtr according to:

qin =
(Thtr − T f ,in )
(11)
RA + Rcontact + RB + Rconv ,in

qout =
(Thtr − T f ,out )
(12)
RC + Rconv ,out

These are 3 equations in 3 unknowns, Thtr, qout and qin , and therefore can be solved
simultaneously in EES (note that the previous temperature calculations from part (b) must be
commented out):

{T_htr=T_f_in+(R_A+R_B+R_conv_in)*q_flux*A "heater temperature"


T_L=T_f_in+(R_B+R_conv_in)*q_flux*A "temperature at x=L"
T_2L=T_f_in+R_conv_in*q_flux*A "temperature at x=2L"}
q_flux*A=q_dot_in+q_dot_out "energy balance on the heater"
q_dot_in=(T_htr-T_f_in)/(R_A+R_contact+R_B+R_conv_in) "heat flow to inner fluid"
q_dot_out=(T_htr-T_f_out)/(R_C+R_conv_out) "heat flow to outer fluid"

which leads to Thtr = 446 K. The other intermediate temperatures shown on the resistance
diagram can be computed:

Tx = L − = Thtr − RA qin (13)

Tx = L + = Thtr − ( RA + Rcontact ) qin (14)

Tx = 2 L = Thtr − ( RA + Rcontact + RB ) qin (15)

Tx =− L = Thtr − RC qout (16)

"intermediate temperatures"
T_Lm=T_htr-R_A*q_dot_in
T_Lp=T_htr-(R_A+R_contact)*q_dot_in
T_2L=T_htr-(R_A+R_contact+R_B)*q_dot_in
T_mL=T_htr-R_C*q_dot_out

which leads to Tx=L- = 400.4 K, Tx=L+ = 354.7 K, Tx=2L = 345.6 K, and Tx=-L = 443.8 K.

i.) Sketch the temperature distribution on the axes provided below.


P1.1-1: Viscosity of a dilute gas
Momentum transfer occurs in a fluid due to interactions between molecules that results in a
transfer of momentum. This process is characterized by viscosity, which relates the shear stress
to a velocity gradient in the same way Fourier’s Law relates heat flux to a temperature gradient.
It is not surprising, then, that the viscosity and thermal conductivity of an ideal gas are analogous
transport properties.
a.) Using reasoning similar to that provided in Section 1.1.2 for thermal conductivity, show that
the viscosity of an ideal gas can be estimated according to μ ∝ T MW / σ 2 .

Consider momentum transfer through a fluid in which a velocity gradient has been established in
the x-direction, as shown in Figure 1. We can evaluate the net rate of momentum transferred
through a plane that is located at position x. The flux of molecules passing through the plane
from left-to-right (i.e., in the positive x-direction) is proportional to the number density of the
molecules (nms) and their mean velocity (vms). The molecules that are moving in the positive x-
direction experienced their last interaction at x–Lms (on average), where Lms is the distance
between molecular interactions. The rate of momentum associated with these molecules per unit
area is the product of the rate of molecules passing through the plane (nms vms) momentum and
the momentum per molecule; the momentum per molecule is the product of the mass of the
molecule (M) and its x-velocity at the point where it experienced its last collision, x-Lms
( M u x − Lms ). Therefore, the rate of momentum passing through the plane from left-to-right ( M x′′+ )
is given approximately by:

M x′′+ ≈ nms vms M ums , x − Lms (1)

Similarly, the momentum per unit area passing through the plane from right-to-left ( M x′′− ) is
given by:

M x′′+ ≈ nms vms M u x + Lms (2)

Velocity
M x′′+

M x′′−
Lms

x-Lms x+Lms
Position
x
Figure 1: Momentum flows through a plane in a material.

The net rate of momentum flux passing through the plane per unit area in the positive x-direction
( M ′′ ) is the difference between M x′′+ and M x′′− ,
(
M ′′ ≈ nms vms M u x − Lms − u x + Lms ) (3)

which can be rearranged to yield:

M ′′ ≈ −2 nms vms M Lms


(u x + Lms − u x − Lms ) ≈ −2nvms M Lms
∂u
(4)
Lms 
ms

∂x

∝μ
∂u
∂x

Comparing Eq. (4) with the definition of viscosity shows that the viscosity is proportional to the
product of the number of molecules per unit volume, their average velocity, the mass of each
molecule, and the mean distance between their interactions.

μ ∝ nms vms M Lms (5)

The mass of a molecule is the molecular weight, MW. As noted in Eq. (1-14), kinetic theory
indicates that

Runiv T
vms ∝ (6)
MW

where Runiv is the universal gas constant and T is the absolute temperature. The distance between
molecular interactions was derived in Eq. (1-17) is

1
Lms = (7)
nms π σ 2

where σ is the equivalent radius of the molecule. Substituting Eqs. (6) and (7) into Eq. (5) shows
that:

1
μ∝ T MW (8)
σ2

which is identical to Eq. (1-18) for conductivity if the specific heat capacity is removed.
P1.1-2 (1-1 in text): Conductivity of a dilute gas
Section 1.1.2 provides an approximation for the thermal conductivity of a monatomic gas at ideal
gas conditions. Test the validity of this approximation by comparing the conductivity estimated
using Eq. (1-18) to the value of thermal conductivity for a monotonic ideal gas (e.g., low
pressure argon) provided by the internal function in EES. Note that the molecular radius, σ, is
provided in EES by the Lennard-Jones potential using the function sigma_LJ.
a.) What is the value and units of the proportionality constant required to make Eq. (1-18) an
equality?

Equation (1-18) is repeated below:

cv T
k∝ (1)
σ 2
MW

Equation (1) is written as an equality by including a constant of proportionality (Ck):

cv T
k = Ck (2)
σ 2
MW

Solving for Ck leads to:

kσ 2 MW
Ck = (3)
cv T

which indicates that Ck has units m-kg1.5/s-kgmol05-K0.5.

The inputs are entered in EES for Argon at relatively low pressure (0.1 MPa) and 300 K.

"Problem 1.1-2"
$UnitSystem SI MASS RAD PA K J
$TABSTOPS 0.2 0.4 0.6 0.8 3.5 in

T=300 [K] "temperature"


F$='Argon' "fluid"
P_MPa=0.1 [MPa] "pressure, in MPa"
P=P_MPa*convert(MPa, Pa) "pressure"

The conductivity, specific heat capacity, Lennard-Jones potential, and molecular weight of
Argon (k, cv, σ, and MW) are evaluated using EES' built-in funcions. Equation (3) is used to
evaluate the proportionality constant.

k=conductivity(F$,T=T,P=P) "conductivity"
cv=cv(F$,T=T,P=P) "specific heat capacity at constant volume"
MW=molarMass(F$) "molecular weight"
sigma=sigma_LJ(F$) "Lennard-Jones potential"
C_k=k*sigma^2*sqrt(MW/T)/cv "constant of proportionality"
which leads to Ck = 2.619x10-24 m-kg1.5/s-kgmol0.5-K0.5.

b.) Plot the value of the proportionality constant for 300 K argon at pressures between 0.01 and
100 MPa on a semi-log plot with pressure on the log scale. At what pressure does the
approximation given in Eq. (1-18) begin to fail at 300 K for argon?

Figure 1 illustrates the constant of proportionality as a function of pressure for argon at 300 K.
The approximation provided by Eq. (1-18) breaks down at approximately 1 MPa.

8x10-24

7x10-24
)
0.5

6x10-24
-K
0.5

5x10-24
/s-kgmol

4x10-24

3x10-24
1.5
Ck (m-kg

2x10-24

10-24

0x100
0.001 0.01 0.1 1 10 100
Pressure (MPa)
Figure 1: Constant of proportionality in Eq. (3) as a function of pressure for argon at 300 K.
P1.1-3: Conductivity of a polyatomic gas
Equation (1-18) cannot be used to understand the thermal conductivity of a polyatomic ideal gas,
such as low pressure oxygen, because the ideal gas thermal conductivity is the sum of two terms
corresponding to translational and internal contributions.

k = ktrans + kint (1)

Equation (1-18) only considers the translatonal contribution. Because thermal conductivity and
viscosity are analagous transport properties, the translation term for the thermal conductivity of a
dilute gas can be estimated as a function of the viscosity (μ) of the gas according to:

15Runiv μ
ktrans = (2)
4 MW

where Runiv is the universal gas constant and MW is the molar mass of the of the gas. The
internal contribution for a polyatomic molecule results from the transfer of energy associated
with rotational and vibrational degrees of freedom. An estimate of the internal contribution is
provided by the Eucken1 correlation

μ ⎡ 5Runiv ⎤
kint ≈ cp − (3)
MW ⎢⎣ 2 ⎥⎦

where the viscosity is in units of Pa-s and the constant pressure specific heat and gas constant are
in units of J/kmol-K. The internal contribution is zero for a monotonic gas.

Choose a gas and use the EES viscosity function to determine its viscosity as a function of
pressure and temperature. Then calculate and plot the thermal conductivity as a function of
pressure at several temperatures. Compare the values you obtain from the dilute gas theory
described above with the values provided at the same conditions obtained from the EES
conductivity function. Use your program to answer the following questions.
a.) The thermal conductivity of an ideal gas should only depend on temperature. At what
pressure does this requirement fail for the temperature and gas you have selected?

Hydrogen is selected as the gas and the inputs are entered in EES:

"Problem 1.1-3"
$UnitSystem SI MASS RAD PA K J
$TABSTOPS 0.2 0.4 0.6 0.8 3.5 in

T=300 [K] "temperature"


F$='Hydrogen' "fluid"
P_MPa=0.1 [MPa] "pressure, in MPa"
P=P_MPa*convert(MPa, Pa) "pressure"

1
Hirschfelder, J.L., Curtiss, C.F, and Bird, R,B., “Molecular Theory of Gases and Liquids”, John Wiley and Sons,
1967
The viscosity, specific heat capacity at constant pressure, and molecular weight of the gas (μ, cp,
and MW) are obtained using EES' built-in property function:

mu=viscosity(F$,T=T,P=P) "viscosity"
MW=MolarMass(F$) "molecular weight"
cP=cp(F$,T=T,P=P) "specific heat capacity"

The translation term in the thermal conductivity is estimated using Eq. (2):

k_trans=15*R#*mu/(4*MW) "translational contribution"

The internal term in the thermal conductivity is estimated using Eq. (3):

cP_molar=cP*MW "specific heat capacity on a molar basis"


k_int=(mu/MW)*(cP_molar-5*R#/2) "internal contribution"

The dilute gas estimate of the thermal conductivity (kdilute) is obtained from Eq. (1) and compared
to the value obtained from EES (k):

k_dilute=k_trans+k_int "dilute gas estimate of the thermal conductivity"


k=conductivity(F$,T=T,P=P) "conductivity from EES' internal function"

Figure 1 illustrates the conductivity of hydrogen and the dilute gas estimate as a function of
pressure at several values of temperature. It appears that the conductivity is independent of
pressure up to about 1 MPa for hydrogen, although this value decreases with reduced
temperature.

0.2 300 K
Thermal conductivity (W/m-K)

0.18
0.16
200 K
0.14
0.12
0.1
100 K
0.08
0.06
0.04 dilute gas theory
0.02 EES function

0
0.001 0.01 0.1 1 10 40
Pressure (MPa)
Figure 1: Thermal conductivity as a function of pressure estimated by the dilute gas theory and using EES
internal property routines for several temperatures.

b.) How does thermal conductivity vary with temperature? What causes this behavior?
Thermal conductivity increases with temperature. This is due to higher molecular velocities
(primarily) but also due to more modes of energy storage being activated with temperature.

c.) How does thermal conductivity vary with the choice of gas. Is there are relationship between
the thermal conductivity and the number of atoms per molecule?

Figure 2 illustrates the conductivity of 6 different gases at 300 K and 100 kPa. There does not
appear to be a clear correlation between conductivity and the number of atoms per molecule.

0.036

0.032
Conductivity (W/m-K)

0.028

0.024

0.02

0.016

0.012
n

e
en

ne
e
go

an
ge

d
og

ha
xi
Ar

t
xy

bu
io
itr

et
O

nD

n-
N

M
bo
ar
C

Figure 2: Thermal conductivity for several gases at 300 K and 100 kPa.
PROBLEM 1.2-1: Composite Wall
A plane wall is a composite of a low conductivity material (with thickness L1 and conductivity
k1) and a high conductivity material (with thickness L2 = L1 and conductivity k2). The edge of
the wall at x = 0 is at temperature T1 and the edge at x = L1 + L2 has temperature T2, as shown in
Figure P1.2-1(a). T1 is greater than T2. The wall is at steady-state and the temperature
distribution in the wall is one-dimensional in x.
x x x
k1 k2 k1 k2
k1 k2 q ′′
T1 T2 T
T1

T2
x x
L1 L2 0 L1 L1+L2 0 L1 L1+L2
(a) (b)
Figure P1.2-1: (a) Composite wall with k1 < k2, and (b) sketch of heat flux and temperature.

a.) Sketch the heat flux ( q ′′ ) and temperature (T) as a function of position within the wall on the
axes in Fig. 1.2-1(b). Make sure that your sketch reflects the fact that (1) the wall is at steady
state, and (2) k1 < k2.
If the process is at steady state, then I can draw a control volume that extends from one surface to
any location x in the material, as shown in Figure 2.

T1

q′′x A
q0′′ A

Figure 2: Control volume for solution

An energy balance on the control volume leads to:

q0′′ A = q ′′x A (1)

Equation (1) shows that the heat fux at any location x must be constant. The heat flux associated
with conduction is governed by Fourier’s law:

dT
q ′′x = −k (2)
dx

Solving Eq. (2) for the temperature gradient leads to:


dT q ′′
=− x (3)
dx k

The numerator of Eq. (3), the heat flux, is constant while the denominator changes depending on
whether you are in material 1 or material 2. In the low conductivity material 1, the temperature
gradient will be higher than in the high conductivity material 2. Within each material, the
temperature gradient must be constant (i.e., the temperature must be linear with x). The solution
is shown in Figure 3.
k1 k2 k1 k2
x x

T
q′′
T1

q′′x

T2
x x
L1 L1+L2 L1 L1+L2
(a) (b)
Figure 3: (a) Heat transfer rate and (b) temperature as a function of position within wall.
Problem P1.2-2: Conduction Through a Shape with Varying Cross-sectional Area
The temperature distribution for the shape shown in Figure P1.2-2 can be assumed to be 1-D in
the coordinate s. The problem is at steady state and the area available for conduction changes
with s according to an arbitrary function, A(s). The temperatures of the two ends of the shape are
specified; TH at s1 and TC at s2.

s2
s1
s

TH
TC
adiabatic
T
TH

TC s
Figure P1.2-2: Conduction through a shape in which the cross-sectional area varies according to A(s).

a.) Sketch the temperature distribution through the shape on the axes below the figure.

The rate of conductive heat transfer ( q ) at any position s is given by Fourier’s law:

dT
q = − k A (1)
ds

At steady state, the heat transfer rate must be constant with position and therefore the
temperature gradient is inversely proportional to area:

dT q
=− (2)
ds kA

The temperature gradient will be steepest where the area is smallest, as shown in Figure 2.
TH TC
s1
s2

T
TH

TC s
Figure 2: Temperature distribution.

b.) Derive the governing differential equation for the problem; the governing differential
equation should include only temperature T and its derivatives with respect to s as well as the
area and its derivatives with respect to s.

A differentially small control volume is defined, as shown in Figure 3.

TH TC
qs + ds
s1 qs
s2

Figure 3: Differential control volume.

An energy balance on the control volume leads to:

q s = q s + ds (3)

Expanding the s+ds term in Eq. (3) leads to:

dq
qs = qs + ds (4)
ds

which can be simplified:

dq
=0 (5)
ds
Substituting in Fourier’s law into Eq. (5) leads to:

d ⎡ dT ⎤
⎢ −k A ⎥ = 0 (6)
ds ⎣ ds ⎦

You can divide through by -k to get the governing differential equation:

d ⎡ dT ⎤ dA dT d 2T
A = 0 or + A =0 (7)
ds ⎢⎣ ds ⎥⎦ ds ds ds 2
Problem 1.2-3 (1-2 in text): Conduction through a Wall
Figure P1.2-3 illustrates a plane wall made of a very thin (thw = 0.001 m) and conductive (k =
100 W/m-K) material that separates two fluids, A and fluid B. Fluid A is at TA = 100°C and the
heat transfer coefficient between the fluid and the wall is hA = 10 W/m2-K while fluid B is at TB
= 0°C with hB = 100 W/m2-K.
thw = 0.001 m

TA = 100°C TB = 0°C
hA = 10 W/m -K hB = 100 W/m -K
2 2

k = 100 W/m-K
Figure P1.2-3: Plane wall separating two fluids

a.) Draw a resistance network that represents this situation and calculate the value of each
resistor (assuming a unit area for the wall, A = 1 m2).

Heat flowing from fluid A to fluid B must pass through a fluid A-to-wall convective resistance
(Rconv,A), a resistance to conduction through the wall (Rcond), and a wall-to-fluid B convective
resistance (Rconv,B). These resistors are in series. The network and values of the resistors are
shown in Figure 2.

K K K
0.1 0.0001 0.01
W W W
TA = 100°C TB = 0°C
1 tw 1
Rconv , A = Rcond = Rcond , B =
hA A kA hB A
Figure 2: Thermal resistance network representing the wall.

b.) If you wanted to predict the heat transfer rate from fluid A to B very accurately, then which
parameter (e.g., thw, k, etc.) would you try to understand/measure very carefully and which
parameters are not very important? Justify your answer.

The largest resistance in a series network will control the heat transfer. For the wall above, the
largest resistance is Rconv,A. Therefore, I would focus on predicting this resistance accurately.
This would suggest that hA is the most important parameter and the others do not matter much.
Problem 1.2-4 (1-4 in text): Resistance Network
Figure P1.2-4 illustrates a plane wall that is composed of two materials, A and B. The interface
between the materials is characterized by a contact resistance. The left surface of material A is
held at TH and the right surface of material B radiates to surroundings at TC and is also exposed to
convection to a fluid at TC.

material A material B

TH
convection and
radiation to TC

contact resistance
Figure P1.2-4: Composite wall with contact resistance, convection and radiation

The resistance network that represents the situation in Figure P1.2-4 should include five thermal
resistors; their values are provided below:
Rcond,A = 0.05 K/W, resistance to conduction through material A
Rcontact = 0.01 K/W, contact resistance
Rcond,B = 0.05 K/W, resistance to conduction through material B
Rconv = 1.0 K/W, resistance to convection
Rrad = 10.0 K/W, resistance to radiation
a.) Draw a resistance network that represents the situation in Figure P1.2-4. Each resistance in
the network should be labeled according to Rcond,A, Rcontact, Rcond,B, Rconv, and Rrad. Show
where the temperatures TH and TC appear on your network.

Figure 2: Resistance network that represents Figure P1.2-4.

b.) What is the most important resistor in the network? That is, the heat transfer from TH to TC is
most sensitive to which of the five resistances?

The most important resistor in a series combination is the largest. The largest resistance is the
parallel combination of Rconv and Rrad. The most important resistance in a parallel combination is
the smallest; the smallest of Rconv and Rrad is Rconv. Thus, Rconv is the most important resistance.

c.) What is the least important resistor in the network?

The least important resistance is the contact resistance; it is the smallest in a series of resistors
that are themselves unimportant relative to convection and radiation.
Problem 1.2-5
Figure P1.2-5 illustrates a wafer that is being developed in an optical lithography process.

T∞ = 20°C chuck posts


h = 15 W/m -K Rc′′ = 5x10 K-m /W
2 -4 2

q = 2 W f = 0.1
thp = 0.5 cm ε = 0.7
wafer

thch = 1.5 cm
chuck base
Dw = 4 inch
Tb = 20°C kch = 25 W/m-K
Figure P1.2-5: Wafer being developed in an optical lithography process.

The energy required to develop the resist is deposited at a rate of q = 2 W near the center of the
upper side of the wafer. The wafer has diameter Dw = 4 inch and is made of a conductive
material; therefore, you may assume that the wafer is isothermal. The wafer is cooled by
convection and radiation to the surroundings at T∞ as well as conduction to the chuck. The
surrounding air is at T∞ = 20ºC and the heat transfer coefficient is h = 15 W/m2-K. The
emissivity of the wafer surface is ε = 0.7. The chuck is made out of a single piece of material
with conductivity kch = 25 W/m-K and consists of a base that is thch = 1.5 cm thick and an array
of posts that are thp = 0.5 cm tall. The area of the base of the chuck is the same as the area of the
wafer. The posts occupy f = 10% of the chuck area and the wafer rests on the top of the posts.
There is an area specific contact resistance of Rc′′ = 5x10-4 K-m2/W between the bottom of the
wafer and the top of the posts. The bottom surface of the chuck base is maintained at Tb = 20 ºC.

a.) What is the temperature of the wafer at steady-state?

The inputs are entered in EES:

"Problem 1.2-5"
$UnitSystem SI MASS RAD PA K J
$TABSTOPS 0.2 0.4 0.6 0.8 3.5 in

"Inputs"
D_w=4.0 [inch]*convert(inch,m) "diameter of wafer"
e=0.7 [-] "emissivity of wafer"
h_bar=15 [W/m^2-K] "heat transfer coefficient"
q_dot=2 [W] "power"
th_ch=1.5 [cm]*convert(cm,m) "chuck base thickness"
k_ch=25 [W/m-K] "chuck conductivity"
R``_c=5e-4 [K-m^2/W] "contact resistance"
th_p=0.5 [cm]*convert(cm,m) "post height"
f = 0.1 [-] "fraction of post coverage"
T_infinity_C=20[C] "ambient temperature in C"
T_infinity=converttemp(C,K,T_infinity_C) "ambient temperature"
T_b_C=20 [C] "chuck base temperature in C"
T_b=converttemp(C,K,T_b_C) "chuck base temperature"

Note that the inputs are converted to base SI units and the units for each variable are set in the
Variables Information window.

The resistance network used to represent this problem is shown in Figure P1.2-5-2:

K
K Rrad = 30.58
Rcond , p = 0.247 W
W
q = 2 W
Tb = 20°C
T∞ = 20°C
Tp,b q2 q1

Tw
K
Rcond ,ch = 0.074
W
K
Rc = 0.617 K
W Rconv = 8.223
W
The resistances include:
Rcond,ch = conduction through chuck base
Rcond,p = conduction through posts
Rc = contact resistance
Rrad = radiation resistance
Rconv = convection resistance
Figure P1.2-5-2: Resistance network.

In order to compute the resistance to radiation, it is necessary to guess a value of the wafer
temperature (Tw) and subsequently comment out this guess in order to close up the solution. A
reasonable value is chosen:

T_w=300 [K] "guess for wafer temperature - will be commented out"

The cross-sectional area of the wafer is:

π Dw2
Aw = (1)
4

The resistance to convection from the top surface of the wafer is:

1
Rconv = (2)
Aw h

A_w=pi*D_w^2/4 "wafer area"


R_conv=1/(A_w*h_bar) "convection resistance"

The equations should be solved and the units set as you move through the problem (rather than at
the end); this prevents the accumulation of small errors that are difficult to debug. The resistance
to radiation is:
1
Rrad = (3)
Aw ε (T + T∞2 ) (Tw + T∞ )
w
2

R_rad=1/(A_w*sigma#*e*(T_w^2+T_infinity^2)*(T_w+T_infinity)) "radiation resistance"

The contact resistance is:

Rc′′
Rc = (4)
Aw f

Notice that the factor f in the denominator accounts for the contact area between the posts and
the wafer.

R_c=R``_c/(A_w*f) "contact resistance"

The resistance to conduction through the posts is:

thp
Rcond , p = (5)
kch Aw f

and the resistance to conduction through the base is:

thch
Rcond ,ch = (6)
kch Aw

R_cond_p=th_p/(k_ch*A_w*f) "resistance to conduction through posts"


R_cond_ch=th_ch/(k_ch*A_w) "resistance to conduction through chuck"

The rate of heat transfer by radiation and convection ( q1 ) and through the chuck ( q2 ) are
computed:

q1 =
(Tw − T∞ ) (7)
−1
⎛ 1 1 ⎞
⎜ + ⎟
⎝ Rconv Rrad ⎠

q2 =
(Tw − Tb ) (8)
Rc + Rcond , p + Rcond ,ch

q_dot_1=(T_w-T_infinity)/(1/R_conv+1/R_rad)^(-1) "rate of heat transfer by convection and radiation"


q_dot_2=(T_w-T_b)/(R_c+R_cond_p+R_cond_ch) "rate of heat transfer to chuck"
Because we guessed a value for Tw, it is not likely that q1 and q2 sum to the applied power to the
wafer, as required by an energy balance:

q = q1 + q2 (9)

In order to finish the solution it is necessary to vary Tw until an energy balance is satisfied. EES
automates this process; however, it will work best if it starts from a good set of guess values.
Therefore, select Update Guesses from the Calculate menu. Then comment out the assumed
value of Tw:

{T_w=300 [K]} "guess for wafer temperature - will be commented out"

and enter the energy balance:

q_dot=q_dot_1+q_dot_2 "energy balance"


T_w_C=converttemp(K,C,T_w) "wafer temperature in C"

which leads to Tw = 294.8 K (21.64ºC).

b.) Prepare a plot showing the wafer temperature as a function of the applied power, q .

29

28

27
Temperature (°C)

26

25

24

23

22

21

20
0 1 2 3 4 5 6 7 8 9 10
Heat transfer (W)
Figure P1.2-5-3: Wafer temperature as a function of applied power.

c.) What are the dominant heat transfer mechanisms for this problem? What aspects of the
problem are least important?

The values of the resistances at the nominal conditions given in the problem statement are shown
in Figure P1.2-5-2. The value of the radiation and convection resistances are both large relative
to the sum of resistances between Tw and Tb and therefore these mechanisms are not likely to
play an important role in the problem. The resistance to conduction through the base of the
chuck is small relative to the resistance to conduction through the posts and the contact
resistance; therefore, conduction through the chuck base is not very important. The dominant
resistance in the problem is the contact resistance and the resistance to conduction through the
posts is also important.

d.) Radiation between the underside of the wafer and the top of the chuck base was ignored in
the analysis; is this an important mechanism for heat transfer? Assume that the chuck
surface is black and justify your answer.

The resistance network, modified to include the resistance to radiation from the bottom of the
wafer to the top of the chuck, is shown in Figure P1.2-5-4.

K
Rc = 0.617
K Rrad = 30.58
K W W
Rcond , p = 0.247
W q = 2 W
Tb = 20°C
T∞ = 20°C
Tp,b q2 q1

Tw
K
Rcond ,ch = 0.074
W
K
Rrad , wc = 33.96 Rconv = 8.223
K
W W
The resistances include:
Rcond,ch = conduction through chuck base
Rcond,p = conduction through posts
Rc = contact resistance
Rrad = radiation resistance
Rconv = convection resistance
Rrad,wc = radiation resistance from top of chuck to bottom of wafer
Figure P1.2-5-4: Resistance network, including radiation from the wafer bottom.

The temperature of the top of the chuck is estimated using our previous solution:

Tp ,b = Tw − q1 ( Rc + Rcond , p ) (10)

and used to estimate the resistance to radiation from the top of the chuck to the bottom of the
wafer:

1
Rrad , wc = (11)
(1 − f ) Aw ε (T + Tp2,b ) (Tw + Tp ,b )
w
2

T_p_b=T_w-q_dot_2*(R_c+R_cond_p) "temperature of the top surface of chuck"


R_rad_wc=1/(A_w*(1-f)*sigma#*e*(T_w^2+T_p_b^2)*(T_w+T_p_b))
"radiation resistance between bottom of wafer and top of chuck"

which leads to Rrad,wc = 33.96 K/W. Because Rrad,wc is in series with Rc and Rcond,p and much
larger than the sum of these resistances it is not very important to the problem.
e.) In an effort to maintain the wafer temperature at Tw= 20ºC, you decide to try to reduce and
control the chuck base temperature, Tb. What temperature do you need to reduce Tb to in
order that Tw= 20ºC? If you can only control Tb to within ±0.5 K then how well can you
control Tw?

The specified chuck temperature is commented out and instead the wafer temperature is
specified:

{T_b_C=20 [C]} "chuck base temperature in C"


T_w_C=20 [C] "specified wafer temperature"

which leads to Tb = 291.3 K (18.13ºC). In order to evaluate the impact of a ±0.5 K fluctuation of
Tb on Tw, the required value of Tb is specified and the value of Tw is again commented out:

T_b_C=18.13 [C] "chuck base temperature in C"


{T_w_C=20 [C] "specified wafer temperature"}

which leads to Tw = 293.2 K (20ºC), as expected. Now the value of Tb is elevated by 0.5 K in
order to determine the impact on Tw:

T_b_C=18.13 [C] + 0.5 [K] "chuck base temperature in C"

which leads to Tw = 293.6 K (20.44ºC). Therefore, the ±0.5 K uncertainty in Tb leads to a ±0.44
K uncertainty in Tw.

f.) Perform the same analysis you carried out in (e), but this time evaluate the merit of
controlling the surrounding temperature, T∞, rather than the chuck temperature. What are the
advantages and disadvantages associated with controlling T∞?

The chuck temperature is returned to 20ºC:

T_b_C=20 [C] "chuck base temperature in C"

The specified surrounding temperature is commented out and instead the wafer temperature is
specified:

{T_infinity_C=20[C]} "ambient temperature in C"


T_w_C=20 [C] "specified wafer temperature"

which leads to T∞ = 280.0 K (6.835ºC); clearly the ambient temperature would need to be
reduced by much more than the chuck temperature due to the weaker interaction between the
wafer and the surroundings. This is a disadvantage of using the ambient temperature to control
the wafer temperature.

In order to evaluate the impact of a ±0.5 K fluctuation of T∞ on Tw, the required value of T∞ is
specified and the value of Tw is again commented out:

T_infinity_C=6.835 [C] "ambient temperature in C"


{T_w_C=20 [C] "specified wafer temperature"}

which leads to Tw = 293.2 K (20ºC), as expected. Now the value of T∞ is elevated by 0.5 K in
order to determine the impact on Tw:

T_infinity_C=6.835 [C]+0.5 [K] "ambient temperature in C"

which leads to Tw = 293.2 K (20.06ºC). Therefore, the ±0.5 K uncertainty in T∞ leads to a ±0.06
K uncertainty in Tw. This is an advantage of using T∞ to control the wafer temperature and is also
related to the relatively weak thermal interaction between T∞ and Tw.
P1.2-6: Freezer Wall
You have designed a wall for a freezer. A cross-section of your freezer wall is shown in Figure
P1.2-6. The wall separates the freezer air at Tf = -10°C from air within the room at Tr = 20°C.
The heat transfer coefficient between the freezer air and the inner wall of the freezer is h f = 10
W/m2-K and the heat transfer coefficient between the room air and the outer wall of the freezer is
hr = 10 W/m2-K. The wall is composed of a thb = 1.0 cm thick layer of fiberglass blanket
sandwiched between two thw = 5.0 mm sheets of stainless steel. The thermal conductivity of
fiberglass and stainless steel are kb = 0.06 W/m-K and kw = 15 W/m-K, respectively. Assume
that the cross-sectional area of the wall is Ac = 1 m2. Neglect radiation from either the inner or
outer walls.

thb = 1 cm
thw = 5 mm thw = 5 mm

Tr = 20°C T f = −10°C
hr = 10 W/m -K h f = 10 W/m -K
2 2

stainless steel,
kw = 15 W/m-K fiberglass blanket,
kb = 0.06 W/m-K
Figure P1.2-6: Freezer wall.

a.) Draw a resistance network to illustrate this problem. Be sure to label the resistances in your
network so that it is clear what each resistance is meant to represent.

There are five resistances associated with the problem; convection to the room and the freezer,
Rconv,r and Rconv,f, and conduction through each of the stainless steel walls and the fiberglass
blanket, Rcond,w and Rcond,f. These are placed in series since the heat transfer must pass through all
of them, as shown in Figure P1.2-6-2.

Figure P1.2-6-2: Thermal resistance network.

b.) Enter all of the inputs in the problem into an EES program. Convert each input into the
corresponding base SI unit (i.e., m, kg, K, W, N, etc.) and set the unit for each variable using
the Variable Information window. Using comments, indicate what each variable means.
Make sure that you set and check units of each variable that you use in the remainder of the
solution process.
The inputs are entered in EES and converted to base SI:
$UnitSystem SI MASS RAD PA K J
$TABSTOPS 0.2 0.4 0.6 0.8 3.5 in

"Inputs"
t_w = 5.0 [mm]*convert(mm,m) "SS wall thickness"
t_b = 1.0 [cm]*convert(cm,m) "fiberglass thickness"
T_r = converttemp(C,K,20) "room air temperature"
h_r = 10 [W/m^2-K]
"room air to outer wall heat transfer coefficient"
k_w = 15 [W/m-K] "SS conductivity"
k_b = 0.06 [W/m-K] "fiberglass conductivity"
h_f = 10 [W/m^2-K]
"freezer air to inner wall heat transfer coefficient"
T_f_C=-10 [C] "freezer temperature in C"
T_f = converttemp(C,K,T_f_C) "freezer air temperature in K"
A = 1 [m^2] "freezer area"

The units for each variable are set in the Variable Information window (see Figure P1.2-6-3).

Figure P1.2-6-3: Variable Information window

c.) Calculate the net heat transfer to the freezer (W).

The values of each of the resistances in Figure P1.2-6-2 are calculated. The convection
resistances between the room air and the outer wall of the freezer and the freezer air and the
inner wall are:

1
Rconv ,r = (1)
hr A
1
Rconv , f = (2)
hf A

R_conv_r = 1/(h_r*A) "convection resistance with room air"


R_conv_f=1/(h_f*A) "convection resistance with freezer air"

The units of the two resistances are set in the Variable Information window (to K/W) and the
units are checked to ensure that the equations entered are dimensionally consistent.

The two conduction resistances are:

tw
Rcond , w = (3)
kw A

tb
Rcond ,b = (4)
kb A

R_cond_w=t_w/(k_w*A) "conduction resistance through SS wall"


R_cond_b=t_b/(k_b*A) "conduction resistance through fiberglass wall"

The total heat transfer through the wall ( q ) is:

q =
(T r − Tf ) (5)
Rconv ,r + 2 Rcond , w + Rcond ,b + Rconv , f

q_dot=(T_r-T_f)/(R_conv_r+R_cond_w+R_cond_b+R_cond_w+R_conv_f)
"net heat transfer to freezer"

The Solution Window is shown in Figure P1.2-6-4, the heat load on the freezer is 81.7 W per m2
of wall area.

Figure P1.2-6-4: Solution window.


d.) Your boss wants to make a more energy efficient freezer by reducing the rate of heat transfer
to the freezer. He suggests that you increase the thickness of the stainless steel wall panels in
order to accomplish this. Is this a good idea? Justify your answer briefly.

The value of the resistances are highlighted in Figure P1.2-6.4. Notice that Rcond,w is
approximately 1000x less than the others. Your boss’ idea is not so good because in a series
combination of resistances, it is the large resistances that dominate the problem. The wall is not
important from a heat transfer standpoint.

e.) Prepare a plot showing the heat transfer to the freezer as a function of the thickness of the
stainless steel walls. Prepare a second plot showing the heat transfer to the freezer as a
function of the thickness of the fiberglass. Make sure that your plots are clear (axes are
labeled, etc.)

A parametric table must be created to vary the thickness of the steel walls. Select New
Parametric Table from the Tables menu (Figure P1.2-6-5) and place the variables q_dot and t_w
in the table (highlight these variables from the list in the left hand box and select Add, then hit
OK).

Figure P1.2-6-5: New Parametric Table dialog

Vary the thickness of the stainless steel walls from 0 to 2.0 cm (which corresponds to an
extremely heavy freezer); right-click on the column of the parametric table that contains the
variable t_w and select Alter Values (Figure P1.2-6-6).
Figure P1.2-6-6: Alter values of t_w to carry out the parametric investigation.

A dialog window will open asking what range you would like to vary t_w over; select 0 to 0.02 m
(Figure P1.2-6-7) and hit OK.

Figure P1.2-6-7: Vary t_w from 0 to 0.02 m.

The entries in the t_w column will be automatically filled in. Each time one row of the Table is
solved, the corresponding value of t_w will be used in the Equations Window; therefore, it is
necessary to remove the value of t_w from the Equations Window. In order to do this
temporarily (you will want to go back to the value in the problem statement), you should
highlight the section of the code that specifies the value and right click. Select Comment to
temporarily remove the code (Figure P1.6-2-8); subsequently performing the same operation and
selecting Undo Comment will remove the comment indicators and “reactivate” the assignment.
Figure P1.2-6-8: Comment out the assignment of t_w in the Equation window

Solve the table by selecting Solve Table from the Calculate menu; the corresponding value of
q_dot will be entered in each row of the parametric table (Figure P1.2-6-9).

Figure P1.2-6-9: Parametric table with solution

The solution can be plotted by selecting New Plot Window from the Plots menu and then X-Y
plot to bring up the dialog shown in Figure P1.2-6-10. Select the source of the data (there is only
one source in your EES file which is the single parametric table that exists) and specify that t_w
will be on the x-axis and q_dot on the y-axis.
Figure P1.2-6-10: New Plot Setup window.

Select OK to create the plot and then edit it so that it looks good (include axes with descriptive
names and units, grid line, etc.); the result should be similar to Figure P1.2-6-11.

Figure P1.2-6-11: Heat transfer to the freezer as a function of the freezer wall thickness.

Follow the same steps to generate Figure P1.2-6-12, which shows the freezer load as a function
of the fiberglass thickness. Note that you will need to un-comment the line in the code where
you specify the wall thickness.
Figure P1.2-6-12: Heat transfer to the freezer as a function of the fiberglass thickness.

f.) What design change to your wall would you suggest in order to improve the energy
efficiency of the freezer.

The largest resistance in Figure P1.2-6-4 is the conduction resistance through the fiberglass; I
suggest that the thickness be increased.

g.) One of your design requirements is that no condensation must form on the external surface of
your freezer wall, even if the relative humidity in the room reaches 75%. This implies that
the temperature of the external surface of the freezer wall must be greater than 15°C. Does
your freezer wall satisfy this requirement? Calculate the external surface temperature (°C).

The temperature at the surface of the freezer wall (Ts) corresponds to the node between Rconv,r
and Rcond,w in Figure P1.2-6-2; the value of this temperature can be calculated according to:

Ts = Tr − q Rconv ,r (6)

T_s = T_r-q_dot*(R_conv_r+R_cond_w) "surface temperature"


T_s_C=converttemp(K,C,T_s) "surface temperature in C"

The solution indicates that Ts =11.8°C which is less than 15°C and therefore condensation on the
outside of the freezer is likely.

h.) In order to prevent condensation, you suggest placing a heater between the outer stainless
steel wall and the fiberglass. How much heat would be required to keep condensation from
forming? Assume that the heater is very thin and conductive.

The addition of the heater provides an additional heat input ( qw ) to the resistance network that
enters between Rcond,w and Rcond,b on the air-side of the circuit, as shown in Figure P1.2-6-13.
Figure P1.2-6-13: Heater power added to the resistance network.

The required surface temperature is Ts,rq = 15°C. Therefore, the heat transfer through Rconv,r ( q1 )
is:

q1 =
(T
r − Ts , rq )
(7)
Rconv ,r

"With the heater added"


T_s_rq = converttemp(C,K,15) "required surface temperature"
q_dot_1=(T_r-T_s_rq)/R_conv_r "heat transfer from the room"

The heater temperature (Thtr) is therefore:

Thtr = Ts ,rq − q1 Rcond , w (8)

and the heat transfer to the freezer space ( q2 ) is:

q2 =
(T htr − Tf ) (9)
Rcond ,b + Rcond , w + Rconv , f

The heat transfer required by the heater ( qhtr ) is obtained by an energy balance on the heater
node:

qhtr = q2 − q1 (10)

T_htr=T_s_rq-q_dot_1*R_cond_w "heater temperature"


q_dot_2=(T_htr-T_f)/(R_cond_b+R_cond_w+R_conv_f) "heat transfer to freezer space"
q_dot_htr=q_dot_2-q_dot_1 "heater power"

The solution indicates that qhtr = 43.6 W.

i.) Prepare a plot showing the amount of heat required by the heater as a function of the freezer
air temperature.
The plot is generated following essentially the same steps discussed in part (e) and shown in
Figure P1.2-6-14.

Figure P1.2-6-14: Heater power as a function of the freezer air temperature.


Problem 1.2-7: Measuring Contact Resistance
You have designed the experimental apparatus shown in Figure P1.2-7 to measure contact
resistance. Four thermocouples (labeled TC1 through TC4) are embedded in two sample blocks
at precise locations. The thermocouples are placed L1 = 0.25 inch from the edges of the sample
blocks and L2 = 1.0 inch apart, as shown. Heat is applied to the top of the apparatus and
removed from the bottom using a flow of coolant. The sides of the sample blocks are insulated.
The sample blocks are fabricated from an alloy with a precisely-known and nearly constant
thermal conductivity, ks = 2.5 W/m-K. The apparatus is activated and allowed to reach steady
state. The temperatures recorded by the thermocouples are TC1 = 53.3°C, TC2 = 43.1°C, TC3 =
22.6°C, and TC4 = 12.3°C. The contact resistance of interest is the interface between the sample
blocks.

heat
sample block, L1 = 0.25 inch
ks = 2.5 W/m-K
δL = 0.01 inch
δks = 0.4 W/m-K TC1
interface L2 = 1 inch
TC2 δL = 0.01 inch
insulation TC3
L1 = 0.25 inch
TC4 δL = 0.01 inch

cooled block
Figure P1.2-7: Experimental device to measure contact resistance.

a.) Use the data provided above to compute the measured heat flux in the upper and lower
sample blocks.

The input parameters are entered in EES:

"P1.2-7 "
$UnitSystem SI MASS RAD PA K J
$TABSTOPS 0.2 0.4 0.6 0.8 3.5 in

k_s=2.5 [W/m-K] "conductivity"


L_1=0.25 [inch]*convert(inch,m) "distance between sensor and interface"
L_2=1.0 [inch]*convert(inch,m) "distance between sensors"
TC_1=converttemp(C,K,53.3) "thermocouple 1 measurement"
TC_2=converttemp(C,K,43.1) "thermocouple 2 measurement"
TC_3=converttemp(C,K,22.6) "thermocouple 3 measurement"
TC_4=converttemp(C,K,12.3) "thermocouple 4 measurement"

The heat transfer through the sample blocks is one-dimensional, steady state conduction through
a constant cross-sectional area and therefore the heat flux through the upper and lower sample
blocks are given by:
q1′′ = k s
(TC1 − TC2 ) (1)
L2

q2′′ = k s
(TC3 − TC4 ) (2)
L2

q_flux_1=(TC_1-TC_2)*k_s/L_2 "heat flux in hot block"


q_flux_2=(TC_3-TC_4)*k_s/L_2 "heat flux in cold block"

The heat flux measurements are q1′′ =1004 W/m2 and q2′′ =1014 W/m2. Note that these values
should be the same but are different due to measurement uncertainty or heat loss through the
insulation.

b.) Use the data to compute the temperature on the hot and cold sides of the interface.

Figure 2 illustrates the measured temperatures as a function of position; the temperatures on the
hot and cold sides of the interface (Th and Tc) can be obtained by extrapolating the temperature
gradient to the interface, as shown in Figure 2.

Figure 2: Measured temperatures as a function of position and extrapolated temperatures at the interface.

The temperatures at the hot and cold sides of the interface are estimated according to:

L1
Th = TC2 − (TC1 − TC2 ) (3)
L2
L1
Tc = TC3 + (TC3 − TC4 ) (4)
L2

T_h=TC_2-(TC_1-TC_2)*L_1/L_2 "extrapolated temperature at the hot interface"


T_c=TC_3+(TC_3-TC_4)*L_1/L_2 "extrapolated temperature at the cold interface"

The extrapolated temperatures at the interface are Th = 313.7 K and Tc = 298.3 K.

c.) Use the data to compute the measured contact resistance.

The average of the two heat flux measurements is:

q ′′ =
( q1′′ + q2′′ ) (5)
2

The measured value of the contact resistance is therefore:

Rc′′ =
(Th − Tc ) (6)
q ′′

q_flux=(q_flux_1+q_flux_2)/2 "average of heat flux calculations"


R_contact=(T_h-T_c)/q_flux "measured contact resistance"

The measured contact resistance is Rc′′ = 0.0152 K-m2/W.

It is important to estimate the uncertainty in your measurement. The uncertainty in the distance
measurements is δL= 0.01 inch, the uncertainty in the conductivity of the sample blocks is δks =
0.4 W/m-K, and the uncertainty in the temperature measurements is δT = 0.5 K.
d.) Estimate the uncertainty in the measurement of the heat flux in the upper sample block, the
answer for (a), manually; that is carry out the uncertainty propagation calculations explicitly.

The uncertainties are entered in EES:

dk_s=0.1 [W/m-K] "uncertainty in conductivity"


dL=0.01 [inch]*convert(inch,m) "uncertainty in position measurements"
dT=0.5 [K] "uncertainty in temperature measurement"

The uncertainty in q1′′ is related to the uncertainty in the measured quantities used to calculate
q1′′ :

q1′′ = k s
(TC1 − TC2 ) (7)
L2

′′ 1 ) is obtained according to:


The uncertainty in q1′′ due to TC1 ( δ q1,TC
∂q1′′ δT
δ q1,′′TC = δ T = ks (8)
1
∂TC1 L2

and the uncertainty in q1′′ due to TC2 ( δ q1,TC


′′ 2 ) is also:

δT
δ q1,′′TC = ks (9)
2
L2

The uncertainty in q1′′ due to ks ( δ q1,′′ks ) is:

δ ks
δ q1,′′k = (TC1 − TC2 ) (10)
s
L2

and the uncertainty in q1′′ due to L2 ( δ q1,L


′′ 2 ) is:

δL
δ q1,′′L = ks (TC1 − TC2 ) (11)
2
L22

The total uncertainty in the heat flux is obtained by combining these contributions using the root-
sum-square (RSS) technique:

δ q1′′ = δ q1,′′TC
2
+ δ q1,′′TC
1
2
+ δ q1,′′k2 + δ q1,′′L2
2 s 2
(12)

"Manual calculation of the uncertainty"


dq_flux_1_TC_1=dT*k_s/L_2 "uncertainty in heat flux 1 due to TC_1"
dq_flux_1_TC_2=dT*k_s/L_2 "uncertainty in heat flux 1 due to TC_2"
dq_flux_1_k_s=(TC_1-TC_2)*dk_s/L_2 "uncertainty in heat flux 1 due to k_s"
dq_flux_1_L_2=(TC_1-TC_2)*k_s*dL/L_2^2 "uncertainty in heat flux 1 due to L_2"
dq_flux_1=sqrt(dq_flux_1_TC_1^2+dq_flux_1_TC_2^2+dq_flux_1_k_s^2+dq_flux_1_L_2^2)
"uncertainty in heat flux 1 measurement"

The total uncertainty in the heat flux is δ q1′′ = 81.0 W/m2.

e.) Verify that EES' uncertainty propagation function provides the same answer obtained in (d).

The uncertainty propagation capability of EES is accessed by selecting Uncertainty Propagation


from the Calculate menu (Figure 3).
Figure 3: Uncertainty Propagation Window.

The calculated variable of interest is q_flux_1 and this should be selected from the Calculated
variable list. The measured variables with uncertainty include the variables k_s, L_1, L_2, TC_1,
TC_2, TC_3, and TC_4; these should be selected from the Measured variable list. The uncertainty
associated with these measured variables can be specified by selecting Set uncertainties (Figure
4).

Figure 4: Uncertainties of Measured Variables Window.

The absolute uncertainties of each of the measured variables are assigned using the
corresponding variable names (Figure 4). Select OK twice to see the results of the uncertainty
propagation calculation (Figure 5).

Figure 5: Uncertainty Results Window.


Notice that the heat flux uncertainty calculated by EES is also 81.0 W/m2. The Uncertainty
Results Window also delineates the sources of the uncertainty.

f.) Use EES' uncertainty propagation function to determine the uncertainty in the measured
value of the contact resistance. What is the % uncertainty in your measurement?

Rather than the variable q_flux_1, the variable R_contact is selected in the Uncertainty of
Measured Variables Window. The result of the calculation is shown in Figure 6.

Figure 6: Uncertainty Results Window.

The uncertainty in the contact resistance is 0.0017 K-m2/W or 11%.

g.) Which of the fundamental measurements that are required by your test facility should be
improved in order to improve your measurement of the contact resistance? That is, would
you focus your attention on reducing δks, δL, or δT? Justify your answer.

Examination of Figure 6 suggests that the uncertainty in the contact resistance is due almost
entirely to the temperature measurements. Therefore, I would focus my attention on reducing
δT.
Problem 1.2-8 (1-3 in text): Frozen Gutters
You have a problem with your house. Every spring at some point the snow immediately
adjacent to your roof melts and runs along the roof line until it reaches the gutter. The water in
the gutter is exposed to air at temperature less than 0°C and therefore freezes, blocking the gutter
and causing water to run into your attic. The situation is shown in Figure P1.2-8.
snow melts at this surface
Tout , hout = 15 W/m -K
2

Ls = 2.5 inch
snow, ks = 0.08 W/m-K
insulation, kins = 0.05 W/m-K
Tin = 22°C, hin = 10 W/m -K
2

Lins = 3 inch
plywood,
L p = 0.5 inch, k p = 0.2 W/m-K

Figure P1.2-8: Roof of your house.

The air in the attic is at Tin = 22°C and the heat transfer coefficient between the inside air and the
inner surface of the roof is hin = 10 W/m2-K. The roof is composed of a Lins = 3.0 inch thick
piece of insulation with conductivity kins = 0.05 W/m-K that is sandwiched between two Lp = 0.5
inch thick pieces of plywood with conductivity kp = 0.2 W/m-K. There is an Ls = 2.5 inch thick
layer of snow on the roof with conductivity ks = 0.08 W/m-K. The heat transfer coefficient
between the outside air at temperature Tout and the surface of the snow is hout = 15 W/m2-K.
Neglect radiation and contact resistances for part (a) of this problem.
a.) What is the range of outdoor air temperatures where you should be concerned that your
gutters will become blocked by ice?

The input parameters are entered in EES and converted to base SI units (N, m, J, K) in order to
eliminate any unit conversion errors; note that units should still be checked as you work the
problem but that this is actually a check on the unit consistency of the equations.

"P1.2-8: Frozen Gutters"


$UnitSystem SI MASS RAD PA K J
$TABSTOPS 0.2 0.4 0.6 0.8 3.5 in

T_in=converttemp(C,K,22) "temperature in your attic"


L_ins=3 [inch]*convert(inch,m) "insulation thickness"
L_p=0.5 [inch]*convert(inch,m) "plywood thickness"
k_ins=0.05 [W/m-K] "insulation conductivity"
k_p=0.2 [W/m-K] "plywood conductivity"
k_s=0.08 [W/m-K] "snow conductivity"
L_s=2.5 [inch]*convert(inch,m) "snow thickness"
h_in=10 [W/m^2-K]
"heat transfer coefficient between attic air and inner surface of roof"
h_out=15 [W/m^2-K]
"heat transfer coefficient between outside air and snow"
A=1 [m^2] "per unit area"
The problem may be represented by the resistance network shown in Figure 2.

Figure 2: Resistance network representing the roof of your house.

The network includes resistances that correspond to convection with the inside and outside air:

1
Rconv ,out = (1)
hout A

1
Rconv ,in = (2)
hin A

where A is 1 m2 in order to accomplish the problem on a per unit area basis. There are also
conduction resistances associated with the insulation, plywood and snow:

Lins
Rins = (3)
kins A

Lp
Rp = (4)
kp A

Ls
Rs = (5)
ks A

R_conv_out=1/(h_out*A) "outer convection resistance"


R_s=L_s/(k_s*A) "snow resistance"
R_p=L_p/(k_p*A) "plywood resistance"
R_ins=L_ins/(k_ins*A) "insulation resistance"
R_conv_in=1/(h_in*A) "inner convection resistance"

Which leads to Rconv,out = 0.07 K/W, Rs = 0.79 K/W, Rp = 0.06 K/W, Rins = 1.52 K/W and Rconv,in
= 0.10 K/W. Therefore, the dominant effects for this problem are conduction through the
insulation and the snow; the other effects (convection and the plywood conduction) are not
terribly important since the largest resistances will dominate in a series network.

If the snow at the surface of the room is melting then the temperature at the connection between
Rs and Rp must be Ts = 0°C (see Figure 2). Therefore, the heat transferred through the roof ( q in
Figure 2) must be:
q =
(Tin − Ts ) (6)
Rconv ,in + 2 R p + Rins

The temperature of the outside air must therefore be:

Tout = Ts − q ( Rs + Rconv ,out ) (7)

T_s=converttemp(C,K,0)
"roof-to-snow interface temperature must be melting point of water"
q_dot=(T_in-T_s)/(R_conv_in+2*R_p+R_ins)
"heat transfer from the attic to the snow when melting point is reached"
T_out=T_s-q_dot*(R_s+R_conv_out)
"outside temperature required to reach melting point at roof surface"
T_out_C=converttemp(K,C,T_out) "outside temperature in C"

which leads to Tout = -10.8°C. If the temperature is below this then the roof temperature will be
below freezing and the snow will not melt. If the temperature is above 0°C then the water will
not refreeze upon hitting the gutter. Therefore, the range of temperatures of concern are -10.8°C
< Tout < 0°C.

b.) Would your answer change much if you considered radiation from the outside surface of the
snow to surroundings at Tout? Assume that the emissivity of snow is εs = 0.82.

The modified resistance network that includes radiation is shown in Figure 3.

Figure 3: Resistance network representing the roof of your house and including radiation.

The additional resistance for radiation is in parallel with convection from the surface of the snow
as heat is transferred from the surface by both mechanisms. The radiation resistance can be
calculated approximately according to:

1
Rrad = (8)
4T ε s σ A
3

where T is the average temperature of the surroundings and the snow surface. In order to get a
quick idea of the magnitude of this resistance we can approximate T with its largest possible
value (which will result in the largest possible amount of radiation); the maximum temperature
of the snow is 0°C:

e_s=0.82 [-] "emissivity of snow"


R_rad=1/(4*T_s^3*e_s*sigma#*A) "radiation resistance"

which leads to Rrad = 0.26 K/W. Notice that Rrad is much larger than Rconv,out; the smallest
resistance in a parallel combination dominates and therefore the impact of radiation will be
minimal. Furthermore, Rconv,out is not even a very important resistance in the original series
circuit shown in Figure 2.
Problem 1.2-9: Computer Chip Cooling
Computer chips tend to work better if they are kept cold. You are examining the feasibility of
maintaining the processor of a personal computer at the sub-ambient temperature of Tchip = 0°F.
Assume that the operation of the computer chip itself generates qchip =10 W of power. Model the
processor unit as a box that is a = 2 inch x b = 6 inch x c = 4 inch. Assume that all six sides of
the box is exposed to air at Tair = 70°F with a convection heat transfer coefficient of h =10
W/m2-K. The box experiences a radiation heat transfer with surroundings that are at Tsur = 70°F.
The emissivity of the processor surface is ε =0.7 and all six sides experience the radiation heat
transfer. You are asked to size the refrigeration system required to maintain the temperature of
the processor.
a.) What is the refrigeration load that your refrigeration system must be able to remove to
maintain the processor at a steady-state temperature (W)?

The input parameters are entered in EES; notice that the units of each parameter are immediately
converted into SI and the units of the associated variables are set (by you) in the Variable
Information Window (Figure 2).

$UnitSystem SI MASS RAD PA K J


$TABSTOPS 0.2 0.4 0.6 0.8 3.5 in

"INPUTS"
T_chip = converttemp(F,K,0) "chip temperature"
q_dot_chip = 10 [W] "chip generation"
a = 2 [inch]*convert(inch,m) "dimensions of processor"
b = 6 [inch]*convert(inch,m)
c = 4 [inch]*convert(inch,m)
h = 10 [W/m^2-K] "heat transfer coefficient"
T_air=converttemp(F,K,70) "air temperature"
T_sur=converttemp(F,K,70) "temperature of surroundings"
e = 0.7 "emissivity of surface"

Figure 2:Variable Information window showing the units for each variable set.

A control volume encompasses just the processor and includes the internal generation from
operating the chip ( qchip ) as well as convection ( qconv ) and radiation ( qrad ) and the heat transfer
removed by the refrigeration system ( qload ). The energy balance is:
qchip + qconv + qrad = qload (1)

The convection and radiation heat transfer rates may be evaluated using the associated rate
equations:

qconv = h As (Tair − Tchip ) (2)

qrad = σ ε As (Tsur
4
− Tchip
4
) (3)

where σ is Stefan-Boltzmann’s constant and As is the surface area of the processor:

As = 2 ( a b + b c + a c ) (4)

These equations are programmed in EES:


"part (a)"
A_s=2*(a*b+b*c+a*c) "surface area of processor"
q_dot_conv=h*A_s*(T_air-T_chip) "convective heat transfer"
q_dot_rad=sigma#*e*A_s*(T_sur^4-T_chip^4) "radiation heat transfer"
q_dot_chip+q_dot_conv+q_dot_rad=q_dot_load "energy balance"

The units of the variables that have been added are also entered in the Variable Information
window (Figure 3).

Figure 3: Variable Information window with additional units entered.

You can check that your solution is dimensionally consistent by selecting Check Units from the
Calculate menu (Figure 4).

Figure 4: Check Units message


Solving the problem (Solve from the Calculate menu) will bring up the Solution Window (Figure
5) and shows that the refrigeration load is 39.4 W.

Figure 5: Solution Window

b.) If the coefficient of performance (COP) of the refrigeration system is nominally 3.5, then
how much heat must be rejected to the ambient air (W)? Recall that COP is the ratio of the
amount of refrigeration provided to the amount of input power required.

The definition of COP is:

qload
COP = (5)
w ref

which is programmed in EES:

"part (b)"
COP = 3.5 "specified COP"
COP = q_dot_load/w_dot_ref "refrigeration power"

and solved to show that the refrigeration power will be 11.3 W.

c.) If electricity costs 12¢/kW-hr, how much does it cost to run the refrigeration system for a
year, assuming that the computer is never shut off.

The cost of electricity and time of operation are both converted to SI units and used to evaluate
the cost per year.

"part (c)"
ecost = 12 [cents/kW-hr]*convert(cents/kW-hr,$/J) "cost of electricity"
time=1 [year]*convert(year,s) "time of operation"
cost=time*ecost*w_dot_ref "cost of operating system for 1 year"

The cost of operating the system for 1 year is $11.8.


Problem 1.2-10: Insulation Conductivity Test
You have been contracted by ASHRAE (the American Society of Heating, Refrigeration, and
Air-Conditioning Engineers) to measure the thermal conductivity of various, new materials for
insulating pipes. Your contract specifies that you will measure the thermal conductivity to
within 10%. Your initial design for the test setup is shown in Figure P1.2-10. The test facility
consists of a pipe (with conductivity kpipe = 120 W/m-K) with inner diameter, Di,pipe = 6.0 inch
and thickness thpipe = 0.5 inch that carries a flow of chilled water, Twater = 10°C. The heat
transfer coefficient between the water and the internal surface of the pipe is hwater = 300 W/m2-K.
The pipe is covered by a thins = 2.0 inch thick layer of the insulation (with conductivity kins) that
is being tested. Two thermocouples are embedded in the insulation, one connected to the outer
surface (Tins,out) and the other to the inner surface (Tins,in). The insulation material is surrounded
by a thm = 3.0 inch thick layer of a material with a well-known thermal conductivity, km = 2.0
W/m-K. Two thermocouples are embedded in the material at its inner and outer surface (Tm,in
and Tm,out, respectively). Finally, a band heater is wrapped around the outer surface of the
material. Assume that the thickness of the band heater is negligibly small. The band heater
provides qband = 3 kW/m. The outer surface of the band heater is exposed to ambient air at Tair =
20°C and has a heat transfer coefficient, hair = 10 W/m2-K and emissivity ε = 0.5. A contact
resistance of Rc′′ =1x10-4 m2-K/W is present at all 3 interfaces in the problem (i.e., between the
pipe and the insulation, the insulation and the material, and the material and the band heater).

Tair = 20°C band heater


hair = 10 W/m -K
2
qband = 3 kW
ε = 0.5
Twater = 10°C
hwater = 300 W/m -K
2
Tm,out
Tm,in pipe, kpipe = 120 W/m-K
Tins,out
thpipe = 0.5 inch
Tins,in

Rc′′ = 1x10 K-m /W


-4 2

thins = 2 inch
thm = 3 inch
insulation being tested, kins
Di,pipe = 6 inch
material with known conductivity,
km = 2.0 W/m-K
Figure P1.2-10: Test facility for measuring pipe insulation

You may assume that the problem is 1-D (i.e., there are no variations along the length or
circumference of the pipe) and do the problem on a per unit length of pipe (L=1 m) basis.
a.) Draw a resistance network that represents the test facility. Clearly label each resistance and
indicate what it represents. Be sure to indicate where in the network the heat input from the
band heater will be applied and also the location of the thermocouples mentioned in the
problem statement.
The resistance network is shown in Figure 2 and includes convection with the water and the air
(Rconv,w and Rair), conduction through the pipe, insulation, and material (Rpipe, Rcond,ins, and
Rcond,m), contact resistances between the pipe and insulation (Rc,1), the insulation and material
(Rc,2), and the material and the band heater (Rc,3), and radiation (Rrad).

Figure 2: Resistance network representing the test facility.

b.) If the coefficient of performance (COP) of the refrigeration system is nominally 3.5, then
how much heat must be rejected to the ambient air (W)? Recall that COP is the ratio of the
amount of refrigeration provided to the amount of input power required.

The known information is entered in EES:

$UnitSystem SI MASS RAD PA K J


$TABSTOPS 0.2 0.4 0.6 0.8 3.5 in

"Inputs"
k_pipe=120 [W/m-K] "pipe conductivity"
D_i_pipe=6.0 [inch]*convert(inch,m) "pipe inner diameter"
th_pipe=0.5 [inch]*convert(inch,m) "pipe thickness"
T_water=converttemp(C,K,10 [C]) "water temperature"
h_water=300 [W/m^2-K] "water to pipe heat transfer coefficient"
th_ins=2.0 [inch]*convert(inch,m) "insulation thickness"
k_m=2.0 [W/m-K] "material thermal conductivity"
th_m=3.0 [inch]*convert(inch,m) "material thickness"
T_air=converttemp(C,K,20 [C]) "air temperature"
h_air=10 [W/m^2-K] "air to heater heat transfer coefficient"
e=0.5 [-] "emissivity of band heater surface"
R``_c=1e-4 [m^2-K/W] "contact resistance"
L=1 [m] "length of pipe"
q_dot_htr=3 [kW]*convert(kW,W) "heater power"
k_ins=1.0 [W/m-K] "insulation conductivity"

The values of the convection resistances are computed:

1
Rconv , w = (1)
hwater π L Di , pipe

1
Rconv ,air = (2)
hair π L ( Di , pipe + 2 thpipe + 2 thins + 2 thm )
R_conv_w=1/(h_water*pi*L*D_i_pipe) "pipe-to-water convection"
R_conv_air=1/(h_air*pi*L*(D_i_pipe+2*th_pipe+2*th_ins+2*th_m)) "heater to air convection"

The conduction resistances are calculated according to:

⎛D + 2 thpipe ⎞
ln ⎜ i , pipe ⎟⎟
⎜ Di , pipe
R pipe = ⎝ ⎠ (3)
2 π L k pipe

⎛D + 2 thpipe + 2 thins ⎞
ln ⎜ i , pipe ⎟⎟
⎜ Di , pipe + 2 thpipe
Rcond ,ins = ⎝ ⎠ (4)
2 π L kins

⎛D + 2 thpipe + 2 thins + 2 thm ⎞


ln ⎜ i , pipe ⎟⎟
⎜ Di , pipe + 2 thpipe + 2 thins
Rcond ,m = ⎝ ⎠ (5)
2 π L km

R_pipe=ln((D_i_pipe/2+th_pipe)/(D_i_pipe/2))/(2*pi*L*k_pipe) "pipe conduction resistance"


R_cond_ins=ln((D_i_pipe/2+th_pipe+th_ins)/(D_i_pipe/2+th_pipe))/(2*pi*L*k_ins)
"insulation conduction resistance"
R_cond_m=ln((D_i_pipe/2+th_pipe+th_ins+th_m)/(D_i_pipe/2+th_pipe+th_ins))/(2*pi*L*k_m)
"material conduction resistance"

The contact resistances are calculated according to:

Rc′′
Rc ,1 = (6)
π L ( Di , pipe + 2 thpipe )

Rc′′
Rc ,2 = (7)
π L ( Di , pipe + 2 thpipe + 2 thins )

Rc′′
Rc ,3 = (8)
π L ( Di , pipe + 2 thpipe + 2 thins + 2 thm )

R_c_1=R``_c/(pi*(D_i_pipe+2*th_pipe)*L) "pipe-to-insulation contact resistance"


R_c_2=R``_c/(pi*(D_i_pipe+2*th_pipe+2*th_ins)*L) "insulation-to-material contact resistance"
R_c_3=R``_c/(pi*(D_i_pipe+2*th_pipe+2*th_ins+2*th_m)*L) "material-to-heater contact resistance"

Finally, the radiation resistance is calculated according to:


1
Rrad = (9)
π L ( Di , pipe + 2 thpipe + 2 thins + 2 thm ) σ ε (Thtr2 + Tair2 )(Thtr + Tair )

but Thtr is not known in Eq. (9); therefore, a guess value of Thtr must be used to allow the
calculation of the resistance. This guess value will be removed once a solution is obtained. A
reasonable guess value for the heater temperature is something higher than the ambient
temperature.

T_htr_g=500 [K]
"this is a guess for the heater temperature - it allows me to calculate the radiation resistance"
"this guess will be removed to complete the solution"
R_rad=1/(pi*(D_i_pipe+2*th_pipe+2*th_ins+2*th_m)*L*sigma#*e*(T_htr_g^2+T_air^2)*(T_htr_g+T_air))
"radiation resistance"

The heat transferred to the heater must either pass inwards to the water or outwards to the
ambient air.

qhtr =
(Thtr − Twater ) +
(Thtr − Tair ) (10)
−1
Rconv , w + R pipe + Rc ,1 + Rcond ,ins + Rc ,2 + Rcond ,m + Rc ,3 ⎛ 1 1 ⎞
⎜⎜ + ⎟⎟
⎝ Rconv , air Rrad ⎠

q_dot_htr=(T_htr-T_water)/(R_conv_w+R_pipe+R_c_1+R_cond_ins+R_c_2+R_cond_m+R_c_3)+(T_htr-
T_air)/((1/R_conv_air+1/R_rad)^(-1)) "heater power"

The calculated and guessed values of Thtr will not be the same (unless you are very lucky);
update the guess values for the calculation (select Update Guesses from the Calculate menu) and
then specify that T_htr_g must equal T_htr. You will need to comment the assignment of T_htr_g
to avoid over-specifying the problem.

{T_htr_g=500 [K] "this is a guess for the heater


temperature - it allows me to calculate the radiation resistance"
"this guess will be removed to complete the solution"}
T_htr=T_htr_g

The heater temperature is found to be 394.6 K (about 120°C which is too hot to touch). The heat
transferred through the insulation is:

qins =
(Thtr − Twater ) (11)
Rconv , w + R pipe + Rc ,1 + Rcond ,ins + Rc ,2 + Rcond ,m + Rc ,3

q_dot_ins=(T_htr-T_water)/(R_conv_w+R_pipe+R_c_1+R_cond_ins+R_c_2+R_cond_m+R_c_3)
"heat transferred through insulation"

which leads to qins = 976.3 W (most of the heat is transferred to the surrounding air).
When the test facility is operated, the heater power is not measured nor are the contact
resistances, emissivity, or heat transfer coefficients known with any precision. Also, the
insulation thermal conductivity is not known, but rather must be calculated from the measured
temperatures. The heat transferred through the material with known thermal conductivity is the
same as the heat transferred through the insulation that is being measured. Therefore:

Tm ,out − Tm ,in Tins ,out − Tins ,in


qins = = (12)
Rcond ,m Rcond ,ins

and so the resistance of the insulation can be calculated based on the ratio of the temperature
differences:

Rcond ,ins =
(T ins , out − Tins ,in )
Rcond ,m (13)
(T m , out − Tm ,in )

Equation (13) indicates that the test facility relies on accurately measuring the temperature
differences across the insulation and the temperature difference across the material.
c.) Using your model, predict the temperature difference across the insulation
( ΔTins = Tins ,out − Tins ,in ) and the material ( ΔTm = Tm ,out − Tm,in ).

Using Eq. (12), the two temperature differences are:

ΔTins = qins Rcond ,ins (14)

ΔTm = qins Rcond ,m (15)

DeltaT_ins=q_dot_ins*R_cond_ins "insulation temperature difference"


DeltaT_m=q_dot_ins*R_cond_m "material temperature difference"

which leads to ΔTins= 70.2 K and ΔTm = 33.8 K.

d.) Prepare a plot showing ΔTm as a function of the material thickness (thm) for thicknesses
ranging from 5.0 mm to 50 cm. Explain the shape of your plot.

The plot requested by the problem statement is generated using a parametric table that include
the variables th_m and DeltaT_m. The result is shown in Figure 3.
Figure 3: Temperature difference across the material as a function of the material thickness.

Notice that at low thm the value of ΔTm is small because the resistance of the material is small.
At high values of thm the resistance of the material is large but the heat transferred through the
material becomes small (more of the energy is transferred to the air) and so the value of ΔTm
begins to decrease.

e.) Based on your plot from part (d), what is a reasonable value for thm? Remember that you
need to measure the temperature difference and therefore you would like it to be large.

A value of thm around 10 cm provides a large value of ΔTm; further increases are probably not
justified. A similar plot and design point could be obtained for ΔTins by varying thins.
Problem P1.2-11: Heater
Figure P1.2-11(a) illustrates a composite wall. The wall is composed of two materials (A with kA
= 1 W/m-K and B with kB = 5 W/m-K), each has thickness L = 1.0 cm. The surface of the wall at
x = 0 is perfectly insulated. A very thin heater is placed between the insulation and material A;
the heating element provides q ′′ = 5000 W/m 2 of heat. The surface of the wall at x = 2L is
exposed to fluid at Tf,in = 300 K with heat transfer coefficient hin = 100 W/m2-K.
material A
q ′′ = 5000 W/m
2
kA = 1 W/m-K
insulated L = 1 cm

x T f ,in = 300 K
hin = 100 W/m -K
2

L = 1 cm material B
kB = 5 W/m-K
Figure P1.2-11(a): Composite wall with a heater.

You may neglect radiation and contact resistance for parts (a) through (c) of this problem.
a.) Draw a resistance network to represent this problem; clearly indicate what each resistance
represents and calculate the value of each resistance.

The input parameters are entered in EES:

“P1.2-11: Heater"
$UnitSystem SI MASS RAD PA K J
$TABSTOPS 0.2 0.4 0.6 0.8 3.5 in

"Inputs"
q_flux=100 [W/m^2] "heat flux provided by the heater"
L = 1.0 [cm]*convert(cm,m) "thickness of each layer"
k_A=1.0 [W/m-K] "conductivity of material A"
k_B=5.0 [W/m-K] "conductivity of material B"
T_f_in=300 [K] "fluid temperature at inside surface"
h_in=100 [W/m^2-K] "heat transfer on inside surface"
A=1 [m^2] "per unit area"

The resistance network that represents the problem shown in Figure 2 is:

Figure 2: Resistance network.

The resistances due to conduction through materials A and B are:


L
RA = (1)
kA A

L
RB = (2)
kB A

where A is the area of the wall, taken to be 1 m2 in order to carry out the analysis on a per unit
area basis. The resistance due to convection is:

1
Rconv ,in = (3)
hin A

"part (a)"
R_A=L/(k_A*A) "resistance to conduction through A"
R_B=L/(k_B*A) "resistance to conduction through B"
R_conv_in=1/(h_in*A)
"resistance to convection on inner surface"

which leads to RA = 0.01 K/W, RB = 0.002 K/W, and Rconv,in = 0.01 K/W.

b.) Use your resistance network from (a) to determine the temperature of the heating element.

The resistance network for this problem is simple; the temperature drop across each resistor is
equal to the product of the heat transferred through the resistor and its resistance. In this simple
case, all of the heat provided by the heater must pass through materials A, B, and into the fluid
by convection so these resistances are in series. The heater temperature (Thtr) is therefore:

Thtr = T f ,in + ( RA + RB + Rconv ,in ) q ′′ A (4)

T_htr=T_f_in+(R_A+R_B+R_conv_in)*q_flux*A "heater temperature"

which leads to Thtr = 410 K.

c.) Sketch the temperature distribution on the axes provided below. Make sure that the sketch is
consistent with your solution from (b).

The temperatures at x = L and x = 2L can be computed according to:

Tx = L = T f ,in + ( RB + Rconv ,in ) q ′′ A (5)

Tx = 2 L = T f ,in + Rconv ,in q ′′ A (6)

T_L=T_f_in+(R_B+R_conv_in)*q_flux*A "temperature at x=L"


T_2L=T_f_in+R_conv_in*q_flux*A "temperature at x=2L"
which leads to Tx=L = 360 K and Tx=2L = 350 K. The temperature distribution is sketched on the
axes in Figure 3.

Figure 3: Sketch of temperature distribution.

Notice that the temperature drop through the two larger resistances (RA and RB) are much larger
than the temperature drop across the small resistance, RB.

Figure P1.2-11(b) illustrates the same composite wall shown in Figure P1.2-11(a), but there is an
additional layer added to the wall, material C with kC = 2.0 W/m-K and L = 1.0 cm.

q ′′ = 5000 W/m
2
material A
material C kA = 1 W/m-K
kC = 2 W/m-K
L = 1 cm
insulated

x T f ,in = 300 K
hin = 100 W/m -K
2

material B
L = 1 cm L = 1 cm k = 5 W/m-K
B
Figure P1.2-11(b): Composite wall with Material C.

Neglect radiation and contact resistance for parts (d) through (f) of this problem.
d.) Draw a resistance network to represent the problem shown in Figure P1.2-11(b); clearly
indicate what each resistance represents and calculate the value of each resistance.

There is an additional resistor corresponding to conduction through material C, RC, as shown


below:
Notice that the boundary condition at the end of RC corresponds to the insulated wall; that is, no
heat can be transferred through this resistance. The resistance to conduction through material C
is:

L
RC = (7)
kC A

"part (b)"
k_C=2.0 [W/m-K] "conductivity of material C"
R_C=L/(k_C*A) "resistance to conduction through C"

which leads to RC = 0.005 K/W.

e.) Use your resistance network from (d) to determine the temperature of the heating element.

Because there is no heat transferred through RC, all of the heat must still go through materials A
and B and be convected from the inner surface of the wall. Therefore, the answer is not changed
from part (b), Thtr = 410 K.

f.) Sketch the temperature distribution on the axes provided below. Make sure that the sketch is
consistent with your solution from (e).

The answer is unchanged from part (c) except that there is material to the left of the heater.
However, no heat is transferred through material C and therefore there is no temperature gradient
in the material.
Figure P1.2-11(c) illustrates the same composite wall shown in Figure P1.2-11(b), but there is a
contact resistance between materials A and B, Rc′′ = 0.01 K-m 2 /W , and the surface of the wall at
x = -L is exposed to fluid at Tf,out = 400 K with a heat transfer coefficient hout = 10 W/m2-K.

q ′′ = 5000 W/m
2
material A
material C kA = 1 W/m-K
kC = 2 W/m-K
L = 1 cm

T f ,out = 400 K
T f ,in = 300 K
hout = 10 W/m -K x
2

hin = 100 W/m -K


2

material B
L = 1 cm kB = 5 W/m-K
L = 1 cm
Rc′′ = 0.01 K-m /W
2

Figure P1.2-11(c): Composite wall with convection at the outer surface and contact resistance.

Neglect radiation for parts (g) through (i) of this problem.


g.) Draw a resistance network to represent the problem shown in Figure P1.2-11(c); clearly
indicate what each resistance represents and calculate the value of each resistance.

The additional resistances associated with contact resistance and convection to the fluid at the
outer surface are indicated. Notice that the boundary condition has changed; heat provided by
the heater has two paths ( qout and qin ) and so the problem is not as easy to solve.
The additional resistances are computed according to:

1
Rconv ,out = (8)
hout A

Rc′′
Rcontact = (9)
A

"part (c)"
R``_c=0.01 [K-m^2/W] "area specific contact resistance"
h_out=10 [W/m^2-K] "heat transfer coefficient"
T_f_out=400 [K] "fluid temperature on outside surface"
R_contact=R``_c/A "contact resistance"
R_conv_out=1/(h_out*A)
"convection resistance on outer surface"

which leads to Rcontact = 0.01 K/W and Rconv,out = 0.1 K/W.

h.) Use your resistance network from (j) to determine the temperature of the heating element.

It is necessary to carry out an energy balance on the heater:

q ′′ A = qin + qout (10)

The heat transfer rates can be related to Thtr according to:

qin =
(Thtr − T f ,in )
(11)
RA + Rcontact + RB + Rconv ,in

qout =
(Thtr − T f ,out )
(12)
RC + Rconv ,out

These are 3 equations in 3 unknowns, Thtr, qout and qin , and therefore can be solved
simultaneously in EES (note that the previous temperature calculations from part (b) must be
commented out):

{T_htr=T_f_in+(R_A+R_B+R_conv_in)*q_flux*A "heater temperature"


T_L=T_f_in+(R_B+R_conv_in)*q_flux*A "temperature at x=L"
T_2L=T_f_in+R_conv_in*q_flux*A "temperature at x=2L"}
q_flux*A=q_dot_in+q_dot_out "energy balance on the heater"
q_dot_in=(T_htr-T_f_in)/(R_A+R_contact+R_B+R_conv_in) "heat flow to inner fluid"
q_dot_out=(T_htr-T_f_out)/(R_C+R_conv_out) "heat flow to outer fluid"

which leads to Thtr = 446 K. The other intermediate temperatures shown on the resistance
diagram can be computed:

Tx = L − = Thtr − RA qin (13)

Tx = L + = Thtr − ( RA + Rcontact ) qin (14)

Tx = 2 L = Thtr − ( RA + Rcontact + RB ) qin (15)

Tx =− L = Thtr − RC qout (16)

"intermediate temperatures"
T_Lm=T_htr-R_A*q_dot_in
T_Lp=T_htr-(R_A+R_contact)*q_dot_in
T_2L=T_htr-(R_A+R_contact+R_B)*q_dot_in
T_mL=T_htr-R_C*q_dot_out

which leads to Tx=L- = 400.4 K, Tx=L+ = 354.7 K, Tx=2L = 345.6 K, and Tx=-L = 443.8 K.

i.) Sketch the temperature distribution on the axes provided below.


Problems 1.2-12 (1-5 in text): Floor Heater
You have decided to install a strip heater under the linoleum in your bathroom in order to keep
your feet warm on cold winter mornings. Figure P1.2-12 illustrates a cross-section of the
bathroom floor. The bathroom is located on the first story of your house and is W = 2.5 m wide
by L = 2.5 m long. The linoleum thickness is thL = 5 mm and has conductivity kL = 0.05 W/m-K.
The strip heater under the linoleum is negligibly thin. Beneath the heater is a piece of plywood
with thickness thp = 5 mm and conductivity kp = 0.4 W/m-K. The plywood is supported by ths =
6 cm thick studs that are Ws = 4 cm wide with thermal conductivity ks = 0.4 W/m-K. The center-
to-center distance between studs is ps = 25.0 cm. Between each stud are pockets of air that can
be considered to be stagnant with conductivity kair = 0.025 W/m-K. A sheet of drywall is nailed
to the bottom of the studs. The thickness of the drywall is thd = 9.0 mm and the conductivity of
drywall is kd = 0.1 W/m-K. The air above in the bathroom is at Tair,1 = 15°C while the air in the
basement is at Tair,2 = 5°C. The heat transfer coefficient on both sides of the floor is h = 15
W/m2-K. You may neglect radiation and contact resistance for this problem.

Tair ,1 = 15°C, h = 15 W/m -K


2
linoleum, kL = 0.05 W/m-K
plywood, kp = 0.4 W/m-K
strip heater
thL = 5 mm
thp = 5 mm ps = 25 cm

ths = 6 cm

Ws = 4 cm
thd = 9 mm
studs, ks = 0.4 W/m-K
drywall, kd = 0.1 W/m-K Tair ,2 = 5°C, h = 15 W/m -K
2

air, ka = 0.025 W/m-K


Figure P1.2-12: Bathroom floor with heater.

a.) Draw a thermal resistance network that can be used to represent this situation. Be sure to
label the temperatures of the air above and below the floor (Tair,1 and Tair,2), the temperature
at the surface of the linoleum (TL), the temperature of the strip heater (Th), and the heat input
to the strip heater ( qh ) on your diagram.

The resistance diagram corresponding to this problem is shown in Figure 2.


Figure 2: Resistance diagram representing the bathroom floor.

Starting at the left-hand side of Figure 2 (i.e., from the basement air), the resistances correspond
to convection between the air in the basement and the drywall (Rconv), conduction through the
drywall (Rd), conduction through the air (Rair) and studs (Rs) in parallel, conduction through the
plywood (Rp), conduction through the linoleum (RL), and convection between the air in the
bathroom and the linoleum (Rconv).

b.) Compute the value of each of the resistances from part (a).

The known values for the problem are entered in EES and converted to base SI units:

$UnitSystem SI MASS RAD PA K J


$TABSTOPS 0.2 0.4 0.6 0.8 3.5 in

"Inputs"
W=2.5 [m] "width of bathroom"
L=2.5 [m] "length of bathroom"
T_air_1=converttemp(C,K,15) "air temperature in the bathroom"
T_air_2=converttemp(C,K,5) "air temperature in the basement"
h=15 [W/m^2-K] "heat transfer coefficient"
th_L=5.0 [mm]*convert(mm,m) "linoleum thickness"
k_L=0.05 [W/m-K] "linoleum thermal conductivity"
th_P=5.0 [mm]*convert(mm,m) "plywood thickness"
k_P=0.4 [W/m-K] "plywood thermal conductivity"
th_s=6.0 [cm]*convert(cm,m) "stud thickness"
W_s=4.0 [cm]*convert(cm,m) "stud width"
k_s=0.4 [W/m-K] "stud conductivity"
p_s=25 [cm]*convert(cm,m) "stud center-to-center distance"
k_air=0.025 [W/m-K] "air conductivity"
th_d=9.0 [mm]*convert(mm,m) "drywall thickness"
k_d=0.1 [W/m-K] "drywall conductivity"

The units for each of these variables is set in the Variable Information window (select Variable
Information from the Options menu), Figure 3. The units of each of the additional variables that
are added as you solve the problem should be set in the Variable Information window.
Figure 3: Variable Information window.

The area of the floor is:

A = LW (1)

The convection resistance is computed according to:

1
Rconv = (2)
hA

The conduction resistances of the linoleum, plywood, and drywall are computed:

thL
RL = (3)
kL A

thp
Rp = (4)
kp A

thd
Rd = (5)
kd A

A=L*W "area for conduction through floor"


R_conv=1/(h*A) "convection resistance"
R_L=th_L/(k_L*A) "linoleum resistance"
R_P=th_P/(k_P*A) "plywood resistance"
R_d=th_d/(k_d*A) "drywall resistance"

The conduction resistance of the studs is computed according to:


ths
Rs = (6)
⎛W ⎞
ks A ⎜ s ⎟
⎝ ps ⎠

Note that the area for conduction is the product of the area of the floor and the fraction of the
floor occupied by the studs. The conduction resistance of the air is:

ths
Rair = (7)
⎛ p − Ws ⎞
kair A⎜ s ⎟
⎝ ps ⎠

R_s=th_s/(k_s*A*W_s/p_s) "stud resistance"


R_air=th_s/(k_air*A*(p_s-W_s)/p_s) "air resistance"

These calculations lead to Rconv = 0.011 K/W, RL = 0.016 K/W, Rp = 0.002 K/W, Rd = 0.014
K/W, Rs = 0.15 K/W, and Rair = 0.46 K/W.

c.) How much heat must be added by the heater to raise the temperature of the floor to a
comfortable 20°C?

If Ts in Figure 2 is 20°C then the heat transferred to the bathroom ( q1 ) is:

TL − Tair ,1
q1 = (8)
Rconv

T_L=converttemp(C,K,20) "linoleum temperature"


q_dot_1=(T_L-T_air_1)/R_conv "heat transferred to bathroom"

which leads to q1 = 469 W. The temperature of the heater is therefore:

Th = TL + q1 RL (9)

T_h=T_L+q_dot_1*R_L "strip heater temperature"

which leads to a heater temperature, Th = 300.7 K. The heater must provide q2 (the heat
transferred to the bathroom) as well as q1 (the heat transferred to the basement).

Th − Tair ,2
q1 = −1
(10)
⎡1 1 ⎤
Rconv + Rd + ⎢ + ⎥ + Rp
⎣ Rs Rair ⎦

and the total heater power is:


qh = q1 + q2 (11)

q_dot_2=(T_h-T_air_2)/(R_conv+R_d+(1/R_s+1/R_air)^(-1)+R_P) "heat lost to lower story"


q_dot_h=q_dot_1+q_dot_2 "total heater power"

which leads to q2 = 161 W and qh = 630 W.

d.) What physical quantities are most important to your analysis? What physical quantities are
unimportant to your analysis?

Figure 4 illustrates the values of the resistances on the resistance diagram.

Figure 4: Resistance values.

Examination of Figure 4 shows that Rp, Rd, and Rconv are unimportant relative to the amount of
heat transferred to the basement; these resistances are small in a series combination. Therefore,
you can expect that the conductivity of the drywall and plywood as well as their thickness are not
very important. Furthermore, the resistance of the air is larger than the resistance of the studs; in
a parallel combination, the larger resistance is not important. Therefore, the conductivity of air is
likely not very important. The important quantities include the conductivity of the studs and
their size as well as the thickness and conductivity of the linoleum and its thickness. The heat
transfer coefficient is also important.

e.) Discuss at least one technique that could be used to substantially reduce the amount of heater
power required while still maintaining the floor at 20°C. Note that you have no control over
Tair,1 or h.

The heat transferred to the bathroom is given by Eq. (8); you cannot change h and therefore the
value of Rconv is fixed. The only way for you can reduce the heater power is to reduce the
amount of heat transferred to the basement. This can be done most effectively by increasing the
resistance of the studs, perhaps by increasing their thickness or reducing their width.
Problem 1.2-13: Burner
An electric burner for a stove is formed by taking a cylindrical piece of metal that is D = 0.32
inch in diameter and L = 36 inch long and winding it into a spiral shape. The burner consumes
electrical power at a rate of q = 900 W. The burner surface has an emissivity of ε = 0.80. The
heat transfer coefficient between the burner and the surrounding air ( h ) depends on the surface
temperature of the burner (Ts) according to:

⎡ W ⎤ ⎡ W ⎤
h = 10.7 ⎢ 2 ⎥ + 0.0048 ⎢ 2 2 ⎥ Ts (1)
⎣ m -K ⎦ ⎣ m -K ⎦

where h is in [W/m2-K] and Ts is in [K]. The surroundings and the surrounding air temperature
are at Tsur = 20°C.
a.) Determine the steady state surface temperature of the burner.

The inputs are entered in EES:

$UnitSystem SI MASS RAD PA K J


$TABSTOPS 0.2 0.4 0.6 0.8 3.5 in

"Inputs"
D=0.32 [inch]*convert(inch,m) "diameter of burner"
L=36 [inch]*convert(inch,m) "length of burner"
q_dot=900 [W] "burner power consumption"
e=0.80 [-] "burner emissivity"
T_sur=converttemp(C,K,20) "surrounding temperature"

In order to move logically through the problem solution it is best to initially assume a surface
temperature, calculate the heat transfer rates, and finally adjust the surface temperature until the
heat transfer rates are consistent with the problem statement. Therefore, an initial and reasonable
guess for the surface temperature is made and used to compute the heat transfer coefficient with
Eq. (1).

T_s=1000 [K]
"initial guess for the surface temperature"
h=10.7 [W/m^2-K]+0.0048 [W/m^2-K^2]*T_s "heat transfer coefficient"

The surface area of the burner is:

As = π D L (2)

The rate of convective heat transfer from the burner to the air is:

qconv = h As ( Ts − Tsur ) (3)

and the rate of radiative heat transfer from the burner to the surroundings is:
qrad = σ ε As (Ts − Tsur ) (4)

where σ is Stefan-Boltzmann’s constant.

A_s=pi*D*L "surface area"


q_dot_conv=A_s*h*(T_s-T_sur) "convection"
q_dot_rad=A_s*e*sigma#*(T_s^4-T_sur^4) "radiation"

At this point, it is necessary to adjust the surface temperature assumption so that the sum of the
radiative and convective heat transfer rates are equal to q , the energy provided to the burner.
This could be done manually, adjust the variable T_s up or down as necessary. However, EES
allows you to automate this process by solving the nonlinear set of equations; like any equation
solver, EES begins this process from an initial set of values (guess values) for each of the
variables and iteratively solves the equations over and over to minimize the error (the residual).
The advantage of the solution approach provided above is that you have a “good” starting point
defined (a good set of guess values); therefore, select Update Guesses from the Calculate menu
in order to lock in these guess values. Then, remove the initially assumed value of the variable
T_s (just comment it out – that is, highlight the line and right click on it, select Comment {}
frorm the menu that will appear) and specify instead that the heat transfer rates must sum to q .

q = qrad + qconv (5)

{T_s=1000 [K]
"initial guess for the surface temperature"}
q_dot=q_dot_conv+q_dot_rad "energy balance on the burner"

Solving the EES program will lead to Ts = 900 K.

b.) Prepare a plot showing the surface temperature as a function of the burner input power.

This is done most easily using a Parametric Table. Comment out the specified value of the
burner input power:

{q_dot=900 [W] "burner power consumption"}

and prepare a Parametric Table that includes the variables q_dot and T_s. Vary the value of
q_dot,for example from 250 W to 1500 W, and solve the table. Plot the results to obtain Figure
2.
Figure 2: Burner surface temperature as a function of burner power
Problem 1.2-14: Ice Rink

Ice for an ice skating rink is formed by running refrigerant at Tr = -30°C through a series of cast
iron pipes that are embedded in concrete, as shown in Figure P1.2-14. The cast iron pipes have
an outer diameter of Do,p = 4 cm and an inner diameter of Di,p = 3 cm. The pipes are spaced Lptp
= 8.0 cm apart. The heat transfer coefficient between the refrigerant and the pipe surface is hr =
100 W/m2-K. The concrete slab is Lc = 8 cm thick and the pipes are in the center of the slab.
The bottom of the slab is insulated (assume perfectly). The thermal conductivity of concrete and
iron are kc = 4.5 W/m-K and kiron = 51 W/m-K, respectively.

An Lfill = 1 cm thick layer of water is placed on the top of the concrete slab. The
refrigerant cools the top of the slab and the water turns to ice slowly. Assume that the water is
stagnant and can be treated as a solid. The conductivity of ice and water are kice = 2.2 W/m-K
and kw = 0.6 W/m-K, respectively. The heat transfer coefficient between the top of the water
layer and the surrounding air at Ta = 15°C is ha = 10.0 W/m2-K. The top of the water surface has
an emissivity of ε = 0.90 and radiates to surroundings at Tsur = 15°C.

Ta = 15°C Tsur = 15°C


ha = 10 W/m -K
2
water, kw = 0.6 W/m-K
Lfill = 1 cm ε = 0.9 ice, kice = 2.2 W/m-K Lice

Lptp = 8 cm
Lc = 8 cm
Lp = 4 cm

cast iron tubes insulation concrete, kc = 4.5 W/m-K


kiron = 51 W/m-K Tr = −30°C
hr = 10 W/m -K
2
Do,p = 4 cm
Di,p = 3 cm
Figure P1.2-14: Schematic of ice rink

a.) Draw a network that represents this situation using 1-D resistances. (Some of the resistances
must be approximate since it is not possible to exactly calculate a 1-D resistance to the
conduction heat flow in the concrete). Include an energy term related to the energy that is
added to the system by the generation of ice. Clearly label the resistors.

The resistance network is shown in Figure 1.


Rrad
Tsur Rw Tice Rice Rc Rp Rconv,r
Tr
Rconv,a
Ta

heat removed from


water to make ice
The resistors include:
Rrad = radiation resistance
Rconv,a = convection resistance to air
Rw = conduction through water
Rice= conduction through ice
Rc = conduction through concrete
Rp = conduction through pipe
Rconv,r = convection resistance to refrigerant
Figure 1: Resistance network representing the ice rink

The resistance network interacts with the air temperature (Ta), the surroundings (Ts), and the
refrigerant (Tr). There is a heat transfer rate at the interface between the ice and water related to
the heat removed from the water in order to form more ice. This heat is accepted because there
is more energy removed by the refrigerant than is provided from the surroundings.

b.) Estimate the magnitude of each of the resistances in your network when the ice is 0.5 cm
thick (i.e., Lice = 0.5 cm).

We will deal with a unit cell of the sub-floor structure, as shown in Figure 2:

Figure 2: Unit cell of the sub-floor structure

The solution process will be described in the context of EES. It is assumed that you have already
been exposed to the EES software by carrying out the self-guided tutorial contained in Appendix
A. The first step in preparing a successful solution to any problem with EES is to enter the
inputs to the problem and set their units. Experience has shown that it is generally best to work
exclusively in SI units (m, J, K, kg, Pa, etc.). This unit system is entirely self-consistent. If the
problem statement includes parameters in other units they are converted to SI units within the
“Inputs” section of the code. The upper section of your EES code should look something like:
$UnitSystem SI MASS RAD PA K J
$Tabstops 0.2 0.4 0.6 3.5 in

"Inputs"
T_r=converttemp(C,K,-30 [C]) "refrigerant temperature (K)"
h_r=100 [W/m^-K] "heat transfer coefficient between refrigerant and pipe"
D_i_p=3.0*convert(cm,m) "pipe inner diameter (m)"
D_o_p=4.0*convert(cm,m) "pipe outer diameter (m)"
k_iron=51 [W/m-K] "pipe conductivity (W/m-K)"
L_ptp=8.0*convert(cm,m) "pipe-to-pipe distance (m)"
k_c=4.5 [W/m-K] "concrete thermal conductivity (W/m-K)"
L_c=8.0*convert(cm,m) "thickness of concrete (m)"
L_p=4.0*convert(cm,m) "center of pipe to upper surface of concrete thickness (m)"
L_fill=1.0*convert(cm,m) "thickness of water layer on concrete (m)"
k_ice=2.2 [W/m-K] "conductivity of ice (W/m-K)"
k_w=0.6 [W/m-K] "conductivity of water (W/m-K)"
h_a=10 [W/m^2-K] "water-to-air heat transfer coefficient (W/m^2-K)"
T_a=converttemp(C,K,15 [C]) "temperature of air on top of slab (K)"
T_sur=converttemp(C,K,15 [C]) "temperature of radiation surrounding on top (K)"
e=0.9 [-] "emissivity of water surface (-)"
W=1 [m] "width of surface (m)"
L_ice=0.5*convert(cm,m) "thickness of ice (m)"

The radiation resistance is:

1
Rrad = (1)
4 σ ε T 3 As

where

Tsur + Ts
T = (2)
2

and Ts is the temperature at the surface of the water. The area of water in the unit cell that is
exposed to air is:

As = Lptp W (3)

where W is the width of the unit cell into the page (here, W = 1 m for a solution per unit length of
the floor).

The surface temperature cannot be known until the problem is solved and yet it must be used to
calculate the resistance to radiation, Rrad. One of the nice things about using the Engineering
Equation Solver (EES) software to solve this problem is that the software can deal with this type
of nonlinearity and provide the solution to the implicit equation. It is this capability that
simultaneously makes EES so powerful and yet sometimes, ironically, difficult to use. EES
should be able to solve equations regardless of the order in which they are entered. However,
you should enter equations in a sequence that allows you to solve them as you enter them; this is
exactly what you would be forced to do if you were to solve the problem using a typical
programming language (e.g., MATLAB, FORTRAN, etc.). This technique of entering your
equations in a systematic order provides you with the opportunity to debug each subset of
equations as you move along rather than waiting until you have entered all of your equations and
tried to solve only to find that there are multiple problems. Another benefit of approaching a
problem in this manner is that you can consistently update your guess values associated with the
variables in your problem; EES solves your equations using a nonlinear relaxation technique and
therefore the closer your variables are to “reasonable” values the better this process will go.

To proceed with the solution to this ice rink problem using EES, it is a helpful idea to assume
initially a reasonable surface temperature (e.g., 273 K) so that it is possible to estimate the
radiation resistance and continue with the solution. The next few lines in your EES code should
look something like:
"Resistances"
"Radiation"
A_s=W*L_ptp "surface area"
T_s=273.2 "this is a guess for the surface temperature - eventually we will
comment this out"
T_bar=(T_s+T_sur)/2 "average temperature"
R_rad=1/(4*e*sigma#*T_bar^3*A_s) "radiation resistance"

If you solve the equations that have been entered (Calculate/Solve) you can check that your
answers make sense and you can verify that your equations have a consistent set of units. It
would be good to do this and then update your guess values (Calculate/Update Guesses); this
operation sets the guess values for each of your variables to their current value and therefore
helps EES iterate to the correct solution. Finally, you should set the units for each of your
variables. The best way to do this is to go to the Variable Information window
(Options/Variable Info) and enter the unit for each variable in the Units column. Once you have
done that you can check units (Calculate/Check Units) in order to make sure that all of the units
you set are consistent with the equations that you’ve entered.

The convection resistance to the air is:

1
Rconv ,a = (4)
ha As

The conduction resistances through the water and ice are:

Rw =
(L fill − Lice )
(5)
kw A

Lice
Rw = (6)
kice A

We will learn how to calculate the resistances of the concrete and pipe more exactly when we get
to extended surfaces and 2-D conduction. For now we will estimate them very approximately
using the concept of an effective length and cross-sectional area for conduction. The length for
conduction will be taken to be the distance that the pipe is submerged beneath the surface and the
area will be taken to be the area of the unit cell:

Lp
Rc = (7)
kc As

The pipe resistance is taken to be the resistance of half a cylinder (in fact, the bottom of the pipe
probably also participates if the pipe is very conductive):

⎛D ⎞
ln ⎜ o , p ⎟
⎜ Di , p ⎟
Rp = ⎝ ⎠ (8)
π kiron W

Finally, the convection resistance to the refrigerant is:

2
Rconv ,r = (9)
hr π Do , p W

The resulting EES code is shown below:

R_conv_a=1/(h_a*A_s) "air convection resistance"


R_w=(L_fill-L_ice)/(k_w*A_s) "resistance of water layer"
R_ice=L_ice/(k_ice*A_s) "resistance of ice layer"
R_c=L_p/(k_c*A_s) "concrete resistance (approximate)"
R_p=2*ln(D_o_p/D_i_p)/(2*pi*W*k_iron) "pipe resistance"
R_conv_r=1/(h_r*W*pi*D_i_p/2) "refrigerant convection resistance"

EES will calculate the resistances in the network (although the radiation resistance continues to
be only approximate). These resistances are placed on the network in Figure 3.

dLice
As h fs
dt
K
Rrad = 2.77
W
Tsur = 15°C Tice = 0°C K K
Rc = 0.11 Rconv ,r = 0.21
W W
Tr = -30°C
K K K
Rw = 0.10 Rice = 0.03 R p = 0.002
Ta = 15°C W W W
K
Rconv ,a =1.25
W
Figure 3: Resistances calculated for ice rink

Resistance networks often provide substantial intuition relative to a problem. For example,
Figure 3 shows that the resistances associated with convection and radiation from the surface of
the water are of the same order of magnitude and large relative to others in the circuit; therefore,
both radiation and convection is important for this problem. If the radiation resistance had been
much larger than the convection resistance (as is often the case in forced convection problems
where the convection coefficient is much larger) then radiation could be neglected; the smallest
resistance in a parallel network will dominate the problem because most of the thermal energy
will tend to flow through that resistance.

In a series resistance network, the larger resistors dominate the problem and the smaller ones can
be neglected. Therefore, we could safely neglect the conduction resistance through the water as
it is small relative to the parallel combination of the radiation and convection resistances.
Similarly, conduction through the ice and the pipe are not important to this problem. It is almost
always a good idea to estimate the size of the resistances in a heat transfer problem prior to
solving it; often it is possible to simplify the problem considerably and the size of the resistances
can certainly be used to guide your efforts. For the ice rink problem, a detailed analysis of
conduction through the pipe would be a misguided use of your time whereas a more accurate
simulation of the conduction through the concrete would be very important.

c.) Calculate the rate of change in the thickness of the ice when the ice thickness is 0.5 cm.

The heat transfer from the ice/water interface to the refrigerant ( qr ) is higher than the heat
transfer from the air and surroundings to the ice/water interface ( qa ) and therefore ice will be
formed. These heat transfer rates can be estimated according to:

qr =
(Tice − Tr ) (10)
Rice + Rc + R p + Rconv ,r

qa =
(Ta − Tice ) (11)
−1
⎡ 1 1 ⎤
Rw + ⎢ + ⎥
⎣ Rrad Rconv ,a ⎦

An energy balance at the interface leads to:

dLice
qr − qa = A h fs (12)
dt

dLice
where hfs is the latent heat of fusion for ice and is the rate of ice formation. The additional
dt
EES code needed to solve this problem is:
"Rate of ice formation"
h_fs=Enthalpy_fusion(Water) "enthalpy of fusion of ice"
rho_ice=1000 "density of ice"
T_ice=convertTemp(C,K,0) "temperature at which water freezes"

q_dot_r=(T_ice-T_r)/(R_conv_r+R_p+R_c+R_ice) "heat transfer to refrigerant"


q_dot_a=(T_a-T_ice)/(R_w+(1/R_conv_a+1/R_rad)^(-1)) "heat transfer from air"
dLicedt=(q_dot_r-q_dot_a)/(A_s*rho_ice*h_fs) "rate of change of ice layer"

At this point, we can use the heat transfer rates to recalculate the water surface temperature (as
opposed to assuming it).

Ts = Tice + qa Rw (13)

It is necessary to comment out or delete the equation that provided the assumed surface
temperature and instead calculate the surface temperature correctly.
{T_s=273.2} "this is a guess for the surface temperature - eventually we will comment this out"
T_s=T_ice+q_dot_a*R_w "recalculate the surface temperature to make radiation resistance exact"

The rate of formation of ice is 2.6e-6 m/s (or 0.94 cm/hr). Note that it will take about 1 hr to
freeze all of the water on the rink based on this answer; the rate of ice formation will not be
significantly affected by the amount of ice because the conduction resistances of the ice and
water were found to be relatively insignificant. Once the ice is completely frozen, the surface
temperature of the ice will drop until qr is balanced by qa .
Problem 1.2-15 (1-6 in text): The super ice-auger

You are a fan of ice fishing but don't enjoy the process of augering out your fishing hole in the
ice. Therefore, you want to build a device, the super ice-auger, that melts a hole in the ice. The
device is shown in Figure P1.2-15.

heater, activated with


2
V = 12 V and I = 150 A
h = 50 W/m -K
T∞ = 5 ° C plate,
kp = 10 W/m-K
insulation, ε = 0.9
kins = 2.2 W/m-K D = 10 inch
thins = 0.5 inch
thp = 0.75 inch

thice = 5 inch
ρice = 920 kg/m3
Δifus = 333.6 kJ/kg

Figure P1.2-15: The super ice-auger.

A heater is attached to the back of a D = 10 inch plate and electrically activated by your truck
battery, which is capable of providing V = 12 V and I = 150 A. The plate is thp = 0.75 inch thick
and has conductivity kp = 10 W/m-K. The back of the heater is insulated; the thickness of the
insulation is thins = 0.5 inch and the insulation has conductivity kins = 2.2 W/m-K. The surface of
the insulation experiences convection with surrounding air at T∞ = 5°C and radiation with
surroundings also at T∞ = 5°C. The emissivity of the surface of the insulation is ε = 0.9 and the
heat transfer coefficient between the surface and the air is h = 50 W/m2-K. The super ice-auger
is placed on the ice and activated, resulting in a heat transfer to the plate-ice interface that causes
the ice to melt. Assume that the water under the ice is at Tice = 0°C so that no heat is conducted
away from the plate-ice interface; all of the energy transferred to the plate-ice interface goes into
melting the ice. The thickness of the ice is thice = 5 inch and the ice has density ρice = 920 kg/m3.
The latent heat of fusion for the ice is Δifus = 333.6 kJ/kg.
a.) Determine the heat transfer rate to the plate-ice interface.

The inputs are entered in EES:

"P1.2-15"
$UnitSystem SI MASS RAD PA K J
$Tabstops 0.2 0.4 0.6 0.8 3.5

D=10 [inch]*convert(inch,m) "diameter of ice fishing hole"


th_ins=0.5 [inch]*convert(inch,m) "insulation thickness"
k_ins=2.2 [W/m-K] "insulation conductivity"
th_p=0.75 [inch]*convert(inch,m) "plate thickness"
k_p=10 [W/m-K] "conductivity of plate"
e=0.9 [-] "emissivity"
h_bar=50 [W/m^2-K] "air heat transfer coefficient"
T_infinity=converttemp(C,K,5 [C]) "ambient temperature"
V=12 [V] "battery voltage"
I=150 [A] "current"
T_ice=converttemp(C,K,0 [C]) "temperature of ice-water interface"
th_ice=5 [inch]*convert(inch,m) "thickness of ice"
rho_ice=920 [kg/m^3] "density of ice"
DELTAi_fus=333.6 [kJ/kg]*convert(kJ/kg,J/kg) "enthalpy of fusion of ice"

The power provided to the heater is the product of the voltage and current:

q = V I (1)

q_dot=V*I "power to melting plate"

A resistance network that can be used to represent this problem is shown in Figure P1.2-15-2.

Rrad = 3.56 K/W

q
q2 q1

Tice = 0°C T∞ = 5°C


Ttop
Rcond , p = 0.038 K/W
Rcond ,ins = 0.114 K/W
Rconv = 0.395 K/W

The resistances include:


Rcond,p = conduction through plate
Rcond,ins = conduction through insulation
Rrad = radiation resistance
Rconv = convection resistance
Figure P1.2-15-2: The super ice-auger.

In order to compute the radiation resistance required to solve the problem, it is necessary to
assume a value of Ttop, the temperature at the top of the insulation (this value will eventually be
commented out in order to complete the problem):

T_top=360 [K] "guess for top surface temperature"

The cross-sectional area of the plate is computed:

π D2
Ac = (2)
4

and the radiation resistance is computed according to:

1
Rrad = (3)
ε Ac σ (T + T∞2 ) (Ttop + T∞ )
2
top

A_c=pi*D^2/4 "area of hole"


R_rad=1/(e*A_c*sigma#*(T_top^2+T_infinity^2)*(T_top+T_infinity)) "radiation resistance"
Note that the equations should be entered, the units set, and the EES code solved line by line in
order to debug the code in small segments.

The convection resistance is computed according to:

1
Rconv = (4)
Ac h

and the conduction resistances are computed according to:

thins
Rcond ,ins = (5)
Ac kins

thp
Rcond , p = (6)
Ac k p

R_conv=1/(A_c*h_bar) "air convection resistance"


R_cond_ins=th_ins/(k_ins*A_c) "conduction resistance of insulation"
R_cond_p=th_p/(k_p*A_c) "conduction resistance of plate"

The heat transfer from the heater to the ambient surroundings ( q1 in Figure P1.2-15-2) is:

q1 =
(Th − T∞ ) (7)
−1
⎛ 1 1 ⎞
Rcond ,ins + ⎜ + ⎟
⎝ Rrad Rconv ⎠

and the heat transfer to the ice is:

q2 =
(Th − Tice ) (8)
Rcond , p

where Th is the heater temperature. An energy balance on the heater leads to:

q = q1 + q2 (9)

Equations (7) through (9) are 3 equations in 3 unknowns ( q , q1 , and q2 ) and can be solved using
EES:

q_dot_1=(T_h-T_infinity)/(R_cond_ins+(1/R_rad+1/R_conv)^(-1)) "heat transfer to ambient"


q_dot_2=(T_h-T_ice)/R_cond_p "heat transfer to ice"
q_dot=q_dot_1+q_dot_2 "energy balance"
The temperature at the top of the plate can be computed based on the solution. Update the guess
values for the problem (select Update Guess Values from the Calculate menu) and comment out
the guessed value for Ttop:

{T_top=360 [K]} "guess for top surface temperature"

and calculate Ttop according to the resistance network:

Ttop = Th − q1 Rcond ,ins (10)

T_top=T_h-q_dot_1*R_cond_ins "recalculate top temperature"

The result is q2 = 1676 W.

The values of the resistances are shown in Figure P1.2-15-2; notice that radiation does not play
an important role in the problem because it is a large resistance in parallel with a much smaller
one. The resistance to conduction through the plate is also unimportant since it is so small. The
resistance to conduction through the insulation and convection are dominant.

b.) How long will it take to melt a hole in the ice?

An energy balance on the ice-to-plate interface leads to:

dthice
q2 = Ac Δi fus ρice (11)
dt

dthice
where is the rate at which the thickness of the ice is reduced. Because there is no energy
dt
lost to the water, the rate of ice melting is constant with ice thickness. Therefore the time
required to melt the ice is estimated according to:

dthice
time = thice (12)
dt

q_dot_2=A_c*DELTAi_fus*dth_icedt*rho_ice "energy balance on ice interface"


dth_icedt*time=th_ice "time to melt ice"
time_min=time*convert(s,min) "in min"

which leads to time = 1178 s (19.6 min).

c.) What is the efficiency of the melting process?

The efficiency is defined as the ratio of the energy provided to the plate-to-ice interface to the
energy provided to the heater:
q2
η= (13)
q

eta=q_dot_2/q_dot "efficiency of process"

which leads to η = 0.93.

d.) If your battery is rated at 100 amp-hr at 12 V then what fraction of the battery's charge is
depleted by running the super ice-auger?

The total amount of energy required to melt a hole in the ice is:

Q = q time (14)

The energy stored in the battery (Ebattery) is the product of the voltage and the amp-hr rating. The
fraction of the battery charge required is:

Q
f = (15)
Ebattery

Q=q_dot*time "total energy required"


E_battery=100 [amp-hr]*V*convert(A-V-hr,J) "car battery energy"
f=Q/E_battery "fraction of car battery energy used"

which leads to f = 0.491.


Problem 1.2-16

The temperature distribution across a L = 0.3 m thick wall at a certain instant of time is given by
T = 900 − 900 x − 500 x 2 where T is the temperature in the wall in °C at position x in m. The
density, specific heat capacity and thermal conductivity of the wall are ρ = 2050 kg/m3, c = 0.96
kJ/kg-K and k = 1.13 W/m-K, respectively.
a) Calculate the rate of change of the average wall temperature.

The inputs are entered in EES:

"P1.2-16"
$UnitSystem SI MASS RAD PA K J
$Tabstops 0.2 0.4 0.6 0.8 3.5

"known"
k=1.13 [W/m-K] "conductivity"
rho=2050 [kg/m^3] "density"
c=0.96 [kJ/kg-K]*convert(kJ/kg-K,J/kg-K) "specific heat capacity"
L=0.3 [m] "thickness of the wall"

A control volume that includes the wall is shown in Figure 1.

q x =0 dU q x = L
dt

x
Figure 1: Energy balance on the wall.

The energy balance on the control volume is:

dU
q x =0 = + q x = L (1)
dt

Fourier's law is used to compute the conduction heat transfer at each edge:

dT
q x =0 = − k A (2)
dx x =0

dT
q x = L = − k A (3)
dx x= L

The energy storage is given by:


dU dT
= ALρ c (4)
dt dt

where T is the average temperature in the wall. Substituting Eqs. (2), (3), and (4) into Eq. (1)
leads to:

dT dT dT
−k A = ALρ c −k A (5)
dx x =0 dt dx x= L

Solving for the rate of change of the average temperature leads to:

dT k ⎛ dT dT ⎞
= ⎜ − ⎟ (6)
dt L ρ c ⎝ dx x=L dx x =0 ⎠

where the temperature gradients can be obtained by taking the derivative of the temperature
distribution given in the problem statement:

dTdx0=-900 [C/m] "temperature gradient at x=0"


dTdxL=-900 [C/m] - 2*500 [C/m^2]*L "temperature gradient at x=L"
dTbardt=k*(dTdxL-dTdx0)/(L*rho*c) "rate of change of the average temperature"

dT
which leads to = -0.000574 ºC/s.
dt

b) The left surface of the wall (at x = 0) is exposed to air at T∞ = 1000°C. Determine the
average convection coefficient between the air and the wall surface at x = 0 m.

Newton's law of cooling defines the heat transfer coefficient according to:

∂T
h A (T∞ − Tx =0 ) = − k A (7)
∂x x =0

solving for the heat transfer coefficient:

∂T
−k
∂x x =0
h= (8)
(T∞ − Tx =0 )
T_infinity=1000 [C] "ambient temperature at x=0"
T0=900[C] "temperature at x=0"
h_bar=-k*dTdx0/(T_infinity-T0) "heat transfer coefficient"
Problem 1.2-17
Figure P1.2-17 illustrates the temperature distribution in a plane wall at a particular
instant of time.

x
Figure P1.2-17: Temperature distribution in a plane wall at a certain instant in time.

Select the correct statement from those listed below and justify your answer briefly.
• The heat transfer at the left-hand face of the wall (i.e., at x = 0) is into the wall
(in the positive x direction),
• The heat transfer at the left-hand face of the wall is out of the wall (in the
negative x direction),
• It is not possible to tell the direction of the heat transfer at the left-hand face of
the wall.

Fourier’s law states that conduction is proportional to the negative of the temperature
gradient. At the left-hand face of the wall the temperature gradient is positive; therefore,
the heat transfer must be in the negative x-direction or out of the wall.
Problem 1.2-18
Urea formaldehyde foam with conductivity kfoam = 0.020 Btu/hr-ft-F is commonly used as an
insulation material in building walls. The major advantage of foam insulation is that it can be
installed in existing walls by injection through a small hole. In a particular case, foam is to be
installed in a wall consisting of a thplaster = 5/8 inch thick sheet of plaster board with conductivity
kplaster = 0.028 Btu/hr-ft-F, a thas = 3.5 inch air space and a thbrick = 2.5 inch layer of brick with
conductivity kbrick = 0.038 Btu/hr-ft-F. Before the air gap is filled with foam, there is natural
convection associated with buoyancy induced air motion. The equivalent thermal resistance of
the air gap on a per unit area basis due to the natural convection is Rag ′′ = 0.95 hr-ft2-F/Btu. The
convection coefficients for the inner and outer surface of the wall are hi = 1.5 Btu/hr-ft2-F and
ho = 3.5 Btu/hr-ft2-F, respectively.
a) The R-value of a wall is the resistance of the wall on a per unit area basis, expressed in units
ft2-F-hr/Btu. Calculate the R-value of the wall before the foam insulation is applied.

The inputs are entered in EES:

$UnitSystem SI MASS RAD PA K J


$TABSTOPS 0.2 0.4 0.6 0.8 3.5 in

"known"
k_brick=0.038 [Btu/hr-ft-F]*convert(Btu/hr-ft-F,W/m-K) "brick conductivity"
k_plaster=0.028 [Btu/hr-ft-F]*convert(Btu/hr-ft-F,W/m-K) "plaster conductivity"
k_foam=0.020 [Btu/hr-ft-F]*convert(Btu/hr-ft-F,W/m-K) "formaldehyde foam conductivity"
th_plaster=(5/8) [in]*convert(in,m) "thickness of plaster"
th_as=3.5 [in]*convert(in,m) "thickness of air space"
th_brick=2.5 [in]*convert(in,m) "thickness of brick"
R``_ag=0.95 [hr-ft^2-F/Btu]*convert(hr-ft^2-F/Btu,K-m^2/W)
"resistance of air gap without foam per unit area"
h_bar_i=1.5 [Btu/hr-ft^2-F]*convert(Btu/hr-ft^2-F,W/m^2-K) "inside heat transfer coefficient"
h_bar_o=3.5 [Btu/hr-ft^2-F]*convert(Btu/hr-ft^2-F,W/m^2-K) "outside heat transfer coefficient"

The resistance per unit area without foam is:

1 thplaster th 1
′′ ,nofoam =
Rtotal + ′′ + brick +
+ Rag (1)
hi k plaster kbrick ho

"No foam"
R``_conv_i=1/h_bar_i "resistance to convection on inner surface per unit area"
R``_plaster=th_plaster/k_plaster "resistance to conduction through plaster per unit area"
R``_brick=th_brick/k_brick "resistance to conduction through brick per unit area"
R``_conv_o=1/h_bar_o "resistance to convection on outer surface per unit area"
R``_total_nofoam=R``_conv_i+R``_plaster+R``_ag+R``_brick+R``_conv_o "total resistance per unit area"
R_value_nofoam=R``_total_nofoam*convert(K-m^2/W,ft^2-F-hr/Btu)"R-value of wall without foam"

which leads to an R-value of 9.245 ft2-F-hr/Btu.

b) Calculate the R-value of the wall after insulation is applied. Assume that the insulating foam
completely fills the air gap.
The resistance per unit area with foam is:

1 thplaster thas thbrick 1


′′ , foam =
Rtotal + + + + (2)
hi k plaster k foam kbrick ho

"Foam, no shrinkage"
R``_foam=th_as/k_foam "resistance to conduction through foam per unit area"
R``_total_foam=R``_conv_i+R``_plaster+R``_foam+R``_brick+R``_conv_o "total resistance per unit area"
R_value_foam=R``_total_foam*convert(K-m^2/W,ft^2-F-hr/Btu) "R-value of wall with foam"

which leads to an R-value of 22.88 ft2-F-hr/Btu.

c) Foam insulation ordinarily shrinks after it is installed by an amount dependent upon


conditions such as the outdoor temperature. Calculate the R-value of the wall assuming that
the foam shrinks by 3%. Assume that the air in the gap between the foam and the plaster and
the foam and the brick is stagnant.

The resistance per unit area with the foam considering shrinkage is:

1 thplaster th th th 1
′′ , foam =
Rtotal + + (1 − shrinkage ) as + shrinkage as + brick + (3)
hi k plaster k foam ka kbrick ho

where ka is the thermal conductivity of air (evaluated at 20°C):

"Foam, shrinkage"
Shrinkage=0.03 [-] "amount of shrinkage"
R``_foam_s=(1-Shrinkage)*th_as/k_foam "resistance to conduction through foam per unit area"
k_a=conductivity(Air,T=converttemp(C,K,20 [C])) "conductivity of air"
R``_air_s=Shrinkage*th_as/k_a "resistance to conduction through air per unit area"
R``_total_foam_s=R``_conv_i+R``_plaster+R``_foam_s+R``_air_s+R``_brick+R``_conv_o
"total resistance per unit area"
R_value_foam_s=R``_total_foam_s*convert(K-m^2/W,ft^2-F-hr/Btu) "R-value of wall with foam"

which leads to an R-value of 23.04 ft2-F-hr/Btu. Note that the R-value actually improved with
shrinkage because the stagnant air is less conductive than the foam. However, if the shrinkage is
more extreme then natural convection will cause the resistance of the air to drop and the R-value
to be reduced.
Problem 1.2-19
Figure P1.2-19 illustrates a cross-section of a thermal protection suit that is being designed for an
astronaut.

thins = 4 cm
thliner = 1 cm
thext = 1 mm

tissue Tspace = 4 K

ε = 0.25

Tb = 37°C heater kext = 14.5 W/m-K


kliner = 0.06 W/m-K kins = 0.06 W/m-K
Figure P1.2-19: Cross-section of thermal protection suit.

The suit consists of a liner that is immediately adjacent to the skin. The skin temperature is
maintained at Tb = 37ºC by the flow of blood in the tissue. The liner is thliner = 1 cm thick and
has conductivity kliner = 0.06 W/m-K. A thin heater is installed at the outer surface of the liner.
Outside of the heater is a layer of insulation that is thins = 4 cm with conductivity kins = 0.06
W/m-K. Finally, the outer layer of the suit is thext = 1 mm thick with conductivity kext = 14.5
W/m-K. The outer surface of the external layer has emissivity ε = 0.25 and is exposed by
radiation only to outer space at Tspace = 4 K.
a.) You want to design the heater so that it completely eliminates any heat loss from the skin.
What is the heat transfer per unit area required?

The inputs are entered in EES and the units converted to base SI units:

$UnitSystem SI MASS RAD PA K J


$Tabstops 0.2 0.4 0.6 3.5 in

"Inputs"
T_b=converttemp(C,K,37 [C]) "tissue temperature"
th_ins=4 [cm]*convert(cm,m) "insulation thickness"
k_ins=0.06 [W/m-K] "insulation conductivity"
th_ext=1 [mm]*convert(mm,m) "exterior wall thickness"
k_ext=14.5 [W/m-K] "conductivity of exterior wall"
emm=0.25 [-] "emissivity of exterior wall"
T_space = 4 [K] "temperature of space"
th_liner=1 [cm]*convert(cm,m) "liner thickness"
k_liner=0.06 [W/m-K] "liner conductivity"
A=1 [m^2] "do problem on a unit area basis"

The units of each variable are set by right-clicking on each variable in the Solution Window and
setting the units in the Units dialog (Figure 2):
Figure 2: Set units for variables.

The units are checked by selecting Check Units from the Calculate menu. A resistance diagram
that represents the suit is shown in Figure 3.

q
qskin qout
Text
Tb Tspace
Rliner Rins Rext Rrad
Figure 3: Resistance network representation of space suit.

The resistance to conduction through the liner, insulation and external layer are computed
according to:

thliner
Rliner = (1)
kliner A

thins
Rins = (2)
kins A

thext
Rext = (3)
kext A

where A is taken to be 1 m2 to do the problem on a per unit area basis.

"part (a)"
R_cond_liner=th_liner/(k_liner*A) "resistance to conduction through liner"
R_cond_ins=th_ins/(k_ins*A) "resistance to conduction through the insulation"
R_cond_ext=th_ext/(k_ext*A) "resistance to conduction through exterior wall"

The resistance to radiation can be computed according to:


1
Rrad = (4)
As σ ε (T + T2
ext
2
space )(T ext + Tspace )

However, Text - the external surface of the space suit, is not known. Therefore, we will guess or
assume this temperature and subsequently complete the problem by calculating this value and
removing this assumption. A reasonable guess is Text = 250 K. An energy balance on the heater
(recall that the heater power is to be selected so that qskin = 0 and therefore Thtr = Tb) leads to:

qout =
(T b − Tspace )
(5)
Rins + Rext + Rrad

T_ext=250 [K] "guess for exterior wall outside temperature"


R_rad=1/(emm*A*sigma#*(T_ext^2+T_space^2)*(T_ext+T_space)) "radiation resistance"
q_dot_out=(T_b-T_space)/(R_cond_ins+R_cond_ext+R_rad) "rate of heat transfer"

The problem is solved and the guess values in EES updated (select Update Guesses from the
Calculate menu). The assumed value of Text is commented out and Text is recalculated according
to:

Text = Tb − qout ( Rins + Rext ) (6)

{T_ext=250 [K]} "guess for exterior wall outside temperature"


T_ext=T_b-q_dot_out*(R_cond_ins+R_cond_ext) "recalculate exterior wall outside temperature"

which leads to qout = 69 W.

b.) In order, rate the importance of the following design parameters to your result from (a): kins,
kext, and ε.

The magnitude of the thermal resistances that participate in the process are Rins = 0.67 K/W, Rext
= 6.9x10-5 K/W, and Rrad = 3.77 K/W. In a series resistance circuit, the largest resistors
dominate and therefore the most important parameters are those that dictate Rrad and the least
important are those that determine Rext. In order, the most important parameters are ε and kins.
The value of kext is almost completely unimportant.

c.) Plot the heat transfer per unit area required to eliminate heat loss as a function of the
emissivity, ε.

A parametric table is created (select New Parametric Table from the Tables menu) and the
parameters emm and q_dot_out are added (Figure 4).
Figure 4: New Parametric Table window.

The value of emm is varied from 0.1 to 1.0 by right-clicking on the column heading and selecting
Alter Values (Figure 5)

Figure 5: Alter values to vary emissivity in the parametric table.

The specified value of emissivity is commented out in the program and the table is run (select
Solve Table from the Calculate menu). Select New Plot (X-Y Plot) from the Plots menu to
generate Figure 6.
140

Heat transfer per unit area (W/m )


2
120

100

80

60

40

20

0
0 0.1 0.2 0.3 0.4 0.5 0.6 0.7 0.8 0.9 1
Emissivity
Figure 6: Required heat transfer per unit area as a function of the suit emissivity.

While the average emissivity of the suit's external surface is ε = 0.25, you have found that this
value can change substantially based on how dirty or polished the suit is. You are worried about
these local variations causing the astronaut discomfort due to local hot and cold spots.
d.) Assume that the heater power is kept at the value calculated in (a). Plot the rate of heat
transfer from the skin as a function of the fractional change in the emissivity of the suit
surface.

The code for part (a) is commented out:

{"part (a)"
R_cond_liner=th_liner/(k_liner*A) "resistance to conduction through liner"
R_cond_ins=th_ins/(k_ins*A) "resistance to conduction through the insulation"
R_cond_ext=th_ext/(k_ext*A) "resistance to conduction through exterior wall"
{T_ext=250 [K]} "guess for exterior wall outside temperature"
R_rad=1/(emm*A*sigma#*(T_ext^2+T_space^2)*(T_ext+T_space)) "radiation resistance"
q_dot_out=(T_b-T_space)/(R_cond_ins+R_cond_ext+R_rad) "
T_ext=T_b-q_dot_out*(R_cond_ins+R_cond_ext) "recalculate exterior wall outside temperature"}

and the heat transfer rate is set according to the result calculated in (a). The emissivity is varied
from its nominal value by an amount fct - the fractional change:

ε dirty = fct ε (7)

"part (d)"
fct=1.5 [-] "fractional change in the emissivity"
emm_dirty=emm*fct "emissivity at a location where suit has gotten tarnished"
q_dot_htr=69 [W] "heat transfer rate calculated in (a)"

The resistance of the liner, insulation, and external layer are computed as before:

R_cond_liner=th_liner/(k_liner*A) "resistance to conduction through liner"


R_cond_ins=th_ins/(k_ins*A) "resistance to conduction through the insulation"
R_cond_ext=th_ext/(k_ext*A) "resistance to conduction through exterior wall"

The external surface temperature, Text, is again assumed and the assumed value is used to
compute Rrad:

T_ext=250 [K] "guess for exterior wall outside temperature"


R_rad=1/(emm_dirty*A*sigma#*(T_ext^2+T_space^2)*(T_ext+T_space)) "radiation resistance"

The energy balance on the heater is:

qhtr =
(T htr − Tspace )
+
(Thtr − Tb ) (8)
Rins + Rext + Rrad Rliner

q_dot_htr=(T_htr-T_space)/(R_cond_ins+R_cond_ext+R_rad)+(T_htr-T_b)/R_cond_liner
"energy balance on heater"

The heat transfer rate to space is:

qout =
(Thtr − Tspace )
(9)
Rins + Rext + Rrad

q_dot_out=(T_htr-T_space)/(R_cond_ins+R_cond_ext+R_rad) "heat transfer to space"

The problem is solved and the guess values in EES updated (select Update Guesses from the
Calculate menu). The assumed value of Text is commented out and Text is recalculated according
to:

Text = Thtr − qout ( Rins + Rext ) (10)

{T_ext=250 [K]} "guess for exterior wall outside temperature"


T_ext=T_htr-q_dot_out*(R_cond_ins+R_cond_ext) "recalculate exterior wall outside temperature"

The heat transfer rate from the skin is:

qskin =
(Tb − Thtr ) (11)
Rliner

q_dot_skin=(T_b-T_htr)/R_cond_liner "heat transfer from tissue"

which leads to qskin = 15.6 W. Figure 7 illustrates the rate of heat transfer from the skin as a
function of the fractional change in the emissivity of the suit surface.
2)
20

Heat transfer from skin per unit area (W/m


15

10

-5

-10

-15

-20

-25
0.5 0.6 0.7 0.8 0.9 1 1.1 1.2 1.3 1.4 1.5
Fractional change in emissivity
Figure 7: Heat transfer from the skin as a function of the fractional change in emissivity.
Problem 1.2-20
Figure P1.2-20 illustrates a temperature sensor that is mounted in a pipe and used to measure the
temperature of a flow of air.

Tw = 20°C

support
Ac = 1x10-6 m2 L = 0.01 m
k = 10 W/m-K

h = 30 W/m -K
2

g = 0.1 W
T∞ = 50°C
sensor
As = 0.001 m2
Figure P1.2-20: Temperature sensor.

The operation of the sensor leads to the dissipation of g = 0.1 W of electrical power. This power
is either convected to the air at T∞ = 50°C or conducted along the support to the wall at Tw =
20°C. Treat the support as conduction through a plane wall (i.e., neglect convection from the
edges of the support). The heat transfer coefficient between the air and the sensor is h = 30
W/m2-K. The surface area of the sensor is As = 0.001 m2. The support has cross-sectional area
Ac = 1x10-6 m2, length L = 0.01 m, and conductivity k = 10 W/m-K.
a.) What is the temperature of the temperature sensor?

A resistance network representation of this problem is shown in Figure 2.

Rconv Rcond
T∞ Tw

g
Figure 2: Resistance network.

The resistance to conduction through the support is:

L 0.01 m K-m K
Rcond = = = 1000 (1)
k Ac -6
10 W 1x10 m 2
W

The resistance to convection from the sensor surface is:

1 m 2 -K K
Rconv = = = 33.3 (2)
h As 30 W 0.001 m 2
W

An energy balance on the sensor leads to:


g =
(Ts − T∞ ) + (Ts − Tw ) (3)
Rconv Rcond

Solving for Ts leads to:

T∞ T 323.2 K 293.2 K
g + + w 0.1W+ +
Rconv Rcond 33.3K/W 1000 K/W
Ts = = = 325.4 K (4)
1 1 1 1
+ +
Rconv Rcond 33.3 K/W 1000 K/W

b.) What is the error associated with the sensor measurement (i.e., what is the difference
between the sensor and the air temperature)? Is the error primarily due to self-heating of the
sensor associated with g or due to the thermal communication between the sensor and with
the wall? Justify your answer.

The error is Ts - T∞ = 2.26 K. The error is dominated by self-heating rather than mounting error.
The mounting error would cause the sensor temperature to be less than the fluid temperature
whereas the self-heating causes the temperature of the sensor to be elevated relative to the fluid
temperature.

c.) Radiation has been neglected for this problem. If the emissivity of the sensor surface is ε =
0.02, then assess whether radiation is truly negligible.

The radiation resistance is:

1
Rrad =
As ε σ (T + Tw2 )(Ts + Tw )
s
2

m2 K 4
= (5)
0.001 m 2 0.02 5.67x10-8 W ( 325.42 + 293.22 ) ( 325.4 + 293.2 ) K 3
K
= 7432
W

The radiation occurs in parallel with convection and conduction and is large relative to either of
these resistances, therefore it is probably negligible.
Problem 1.2-21
Figure P1.2-21 illustrates a plane wall made of a material with a temperature-dependent
conductivity. The conductivity of the material is given by:

k = bT (1)

where b = 1 W/m2-K2 and T is the temperature in K.

TH
k = bT
TC
x
L

Figure P1.2-21: Plane wall with temperature-dependent conductivity.

The thickness of the wall is L = 1 m. The left side of the wall (at x= 0) is maintained at TH = 500
K and the right side (at x= L) is kept at TC = 50 K. The problem is steady-state and 1-D.
a.) Sketch the temperature distribution in the wall (i.e., sketch T as a function of x). Make sure
that you get the qualitative features of your sketch right.

The temperatures at x = 0 and x = L are specified. The temperature variation from 0 < x < L will
not be linear. The rate of heat transfer will be constant with x for this problem. Fourier's law
governs the rate of conduction:

dT
q = − k Ac (2)
dx

According to Eq. (1), in regions where the temperature is high, the conductivity will be high;
therefore, the temperature gradient will be small. In regions where the temperature is low, the
conductivity will be low and the temperature gradient higher. Figure 2 reflects these
characteristics.
TH
k = bT
TC
x
L

T
500 K

50 K
x
0 L
Figure 2: Sketch of temperature distribution.

b.) Derive the ordinary differential equation that governs this problem.

The first step towards developing an analytical solution for this, or any, problem involves the
definition of a differential control volume. The control volume must encompass material at a
uniform temperature; therefore, in this case it must be differentially small in the x-direction (i.e.,
it has width dx, see Figure 3) but can extend across the entire cross-sectional area of the wall as
there are no temperature gradients in the y- or z-directions.

Figure 3: Differential control volume.

Next, the energy transfers across the control surfaces must be defined as well as any thermal
energy generation or storage terms. For the steady-state, 1-D case considered here, there are
only two energy transfers, corresponding to the rate of conduction heat transfer into the left side
(i.e., at position x, q x ) and out of the right side (i.e., at position x+dx, q x + dx ) of the control
volume. A steady-state energy balance for the differential control volume is therefore:

q x = q x + dx (3)
A Taylor series expansion of the term at x+dx leads to:

dq d 2 q dx 2 d 3 q dx3
q x + dx = q x + dx + 2 + + ... (4)
dx dx 2! dx3 3!

The analytical solution proceeds by taking the limit as dx goes to zero so that the higher order
terms in Eq. (4) can be neglected:

dq
q x + dx = q x + dx (5)
dx

Substituting Eq. (5) into Eq. (3) leads to:

dq
q x = q x + dx (6)
dx

or

dq
=0 (7)
dx

Equation (7) indicates that the rate of conduction heat transfer is not a function of x. For the
problem in Figure 1, there are no sources or sinks of energy and no energy storage within the
wall; therefore, there is no reason for the rate of heat transfer to vary with position. The final
step in the derivation of the governing equation is to substitute appropriate rate equations that
relate energy transfer rates to temperatures. The rate equation for conduction is Fourier’s law:

dT
q = − k Ac (8)
dx

Substituting Eq. (8) into Eq. (7) leads to:

d ⎡ dT ⎤
⎢ −k Ac =0 (9)
dx ⎣ dx ⎥⎦

The area is constant and can be divided out of Eq. (9). The thermal conductivity is given by Eq.
(1):

d ⎡ dT ⎤
bT =0 (10)
dx ⎢⎣ dx ⎥⎦

c.) What are the boundary conditions for this problem?

The boundary conditions are:


Tx =0 = TH (11)

and

Tx = L = TC (12)

d.) Solve the governing differential equation from (b) - you should end up with a solution that
involves two unknown constants of integration.

Equation (10) is separated:

⎡ dT ⎤
d ⎢bT =0 (13)
⎣ dx ⎥⎦

and integrated:

⎡ dT ⎤
∫ d ⎢⎣bT dx ⎥⎦ = ∫ 0 (14)

which leads to:

dT
bT = C1 (15)
dx

where C1 is a constant of integration. Equation (15) is separated:

C1
T dT = dx (16)
b

and integrated:

C1
∫ T dT = b ∫ dx (17)

which leads to:

T 2 C1
= x + C2 (18)
2 b

e.) Use the boundary conditions from (c) with the solution from (d) in order to obtain two
equations in the two unknown constants.

Equation (18) is substituted into Eqs. (11) and (12):


TH2
= C2 (19)
2

TC2 C1
= L + C2 (20)
2 b

f.) Type the inputs for the problem and the equations from (e) into EES in order to evaluate the
undetermined constants.

The inputs are entered in EES:

$UnitSystem SI MASS RAD PA K J


$Tabstops 0.2 0.4 0.6 3.5 in

"Inputs"
T_H=500 [K] "temperature at left side of wall"
T_C=50 [K] "temperature at right side of wall"
b=1 [W/m-K^2] "coefficient for conductivity function"
L=1 [m] "thickness of wall"

and Eqs. (19) and (20) are entered in EES:

T_H^2/2=C_2 "boundary condition at x=0"


T_C^2/2=C_1*L/b+C_2 "boundary condition at x=L"

which leads to C1 = -123750 W/m2 and C2 = 125000 K2.

g.) Prepare a plot of the temperature as a function of position in the wall using EES.

The solution, Eq. (18), is entered in EES.

T^2/2=C_1*x/b+C_2 "solution"

and a plot is prepared using a parametric table that contains the variables x and T. The result is
shown in Figure 4, which is qualitatively similar to the sketch in Figure 2.
500

450

400

Temperature (K)
350

300

250

200

150

100

50
0 0.1 0.2 0.3 0.4 0.5 0.6 0.7 0.8 0.9 1
Position (m)
Figure 4: Temperature as a function of position.
Problem 1.2-22
You are designing a cubical case that contains electronic components that drive remotely located
instruments. You have been asked to estimate the maximum and minimum operating
temperature limits that should be used to specify the components within the case. The case is W
= 8 inch on a side. The emissivity of the paint used on the case is ε = 0.85. The operation of the
electronic components within the case generates between q = 5 and q = 10 W due to ohmic
heating, depending on the intensity of the operation. The top surface of the case is exposed to a
solar flux q ′′ . All of the surfaces of the case convect (with average heat transfer coefficient h )
and radiate to surroundings at T∞. The case will be deployed in a variety of climates, ranging
from very hot (T∞,max = 110°F) to very cold (T∞,max = -40°F), very sunny ( qmax ′′ = 850 W/m2) to
′′ = 0 W/m2), and very windy ( hmax = 100 W/m2-K) to still ( hmin = 5 W/m2-K). For the
night ( qmin
following questions, assume that the case is at a single, uniform temperature and at steady state.
a.) Come up with an estimate for the maximum operating temperature limit.

The case temperature will be highest when the case generation is maximum, the ambient
temperature is maximum, the solar flux is maximum, and the heat transfer coefficient is
minimum. These inputs are entered in EES:

$UnitSystem SI MASS RAD PA K J


$Tabstops 0.2 0.4 0.6 3.5 in

"Inputs"
W=8 [inch]*convert(inch,m) "side dimension"
e=0.85 [-] "emissivity"
q_dot=10 [W] "dissipation in case"
q``=850 [W/m^2] "solar flux"
h_bar=5 [W/m^2-K] "heat transfer coefficient"
T_infinity=converttemp(F,K,120 [F]) "ambient temperature"

The surface area of the case is:

As = 6W 2 (1)

The resistance to convection from the case is:

1
Rconv = (2)
h As

A_s=6*W^2 "surface area"


R_conv=1/(h_bar*A_s) "convection resistance"

The radiation resistance cannot be calculated without knowing the surface temperature of the
case, T. Therefore, a reasonable value of the surface temperature is assumed. The radiation
resistance is:
1
Rrad = (3)
ε σ As (T + T∞2 )(T + T∞ )
2

T=350 [K] "guess for the case temperature"


R_rad=1/(e*A_s*sigma#*(T^2+T_infinity^2)*(T+T_infinity)) "radiation resistance"

The guess values are updated and the assumed value of the case temperature is commented out.
An energy balance on the case leads to:

qs + q =
(T − T∞ ) + (T − T∞ ) (4)
Rconv Rrad

where qs is the absorbed solar irradiation.

qs = W 2 q′′ (5)

{T=350 [K]} "guess for the case temperature"


q``*W^2+q_dot=(T-T_infinity)/R_conv+(T-T_infinity)/R_rad "energy balance on case"
T_F=converttemp(K,F,T) "case temperature in F"

which leads to a maximum operating temperature limit of T = 147.5°F.

b.) Plot the maximum operating temperature as a function of the case size, W. Explain the shape
of your plot (why does the temperature go up or down with W? if there is an asymptotic
limit, explain why it exists).

The value of W is commented out and a parametric table is generated that includes W and T.
Figure 1 illustrates the maximum operating temperature as a function of the size of the enclosure.
As the size of the enclosure is reduced, the maximum operating temperature increases because
the 10 W of dissipation must be rejected but the area available for convection and radiation is
reduced. As the size is increased, the maximum operating temperature reaches an asymptote.
The limiting value of T is higher than T∞ because the absorbed solar irradiation and the surface
area for convection and radiation both increase in proportion to W2; therefore the limit is
consistent with the situation where the solar flux is exactly balanced by the heat flux associated
with radiation and convection (the dissipation becomes insignificant relative to the solar flux).
230

Maximum operating temperature (°F)


220

210

200

190

180

170

160

150

140
0.05 0.1 0.15 0.2 0.25 0.3 0.35 0.4 0.45 0.5
Size of enclosure (m)
Figure 1: Maximum operating temperature as a function of the size of the enclosure.

c.) Come up with an estimate for the minimum operating temperature limit (with W = 8 inch).

The case temperature will be lowest when the case generation is minimum, the ambient
temperature is minimum, the solar flux is minimum, and the heat transfer coefficient is
maximum. These inputs are entered in EES:

q_dot=5 [W] "dissipation in case"


q``=0 [W/m^2] "solar flux"
h_bar=100 [W/m^2-K] "heat transfer coefficient"
T_infinity=converttemp(F,K,-40 [F]) "ambient temperature"

The solution is run again at the predicted temperature is T= -39.7°F

d.) Do you feel that the emissivity of the case surface is very important for determining the
minimum operating temperature? Justify your answer.

The emissivity is not important because radiation is not important. To see this, look at the
resistance to convection, Rconv= 0.040 K/W, and the resistance to radiation, Rrad= 1.65 K/W.
These two heat transfer mechanisms occur in parallel; the largest resistance in a parallel network
is not important - therefore, radiation is much less important than convection.
Problem 1.2-23
Figure P1.2-23 illustrates a cross-sectional view of a water heater.

heater
T∞ = 20°C q ′′ = 10,000 W/m
2

hout = 15 W/m -K
2
ktube = 12 W/m-K
thins = 0.5 inch Din = 0.75 inch

Dout = 0.875 inch


Rc′′ = 1x10 K-m /W
-4 2

ε = 0.5 T f = 50°C
p f = 18 psi
kins = 0.5 W/m-K
hin = 250 W/m -K
2

Figure P1.2-23: Water heater.

The water flows through a tube with inner diameter Din= 0.75 inch and outer diameter Dout =
0.875 inch. The conductivity of the tube material is ktube = 12 W/m-K. The water in the tube is
at mean temperature Tf = 50°C and pressure pf = 18 psi. The heat transfer coefficient between
the water and the internal surface of the tube is hin = 250 W/m2-K. A very thin heater is wrapped
around the outer surface of the tube. The heater provides a heat transfer rate of q ′′ = 10,000
W/m2. Insulation is wrapped around the heater. The thickness of the insulation is thins = 0.5 inch
and the conductivity is kins= 0.5 W/m-K. There is a contact resistance between the heater and the
tube and the heater and the insulation. The area specific contact resistance for both interfaces is
Rc′′ = 1x10-4 K-m2/W. The outer surface of the insulation radiates and convects to surroundings
at T∞ = 20°C. The heat transfer coefficient between the surface of the insulation and the air is
hout = 15 W/m2-K. The emissivity of the outer surface of the insulation is ε = 0.5.
a.) Draw a resistance network that represents this problem. Label each resistance and clearly
indicate what it represents. Show where the heater power enters your network.

The resistance network is shown in Figure 2.


Rrad

Rcond,tube Rc T Rc Rcond,ins T∞ = 20°C


Tf h

Rconv,in Text
qin qout
q′′π Dout L
Rconv,out

The resistances include:


Rconv,in = convection to water
Rcond,tube = conduction through tube
Rc = contact resistance
Rcond,ins = conduction through insulation
Rconv,out = convection resistance to air
Rrad = radiation resistance
Figure 2: Resistance network.

b.) Using EES, determine the temperature of the heater and the rate of heat transfer to the water.

The inputs are entered in EES; note that the problem is done on a per unit length basis, L= 1 m.

$UnitSystem SI MASS RAD PA K J


$Tabstops 0.2 0.4 0.6 3.5 in

"Inputs"
D_in=0.75 [inch]*convert(inch,m) "inner diameter"
D_out=0.875 [inch]*convert(inch,m) "outer diameter"
k_tube=12 [W/m-K] "conductivity of tube"
T_f=converttemp(C,K,50 [C]) "fluid temperature"
p_f=18 [psi]*convert(psi,Pa) "fluid pressure"
T_infinity=converttemp(C,K,20 [C]) "ambient temperature"
h_bar_out=15 [W/m^2-K] "heat transfer coefficient to ambient air"
h_bar_in=250 [W/m^2-K] "heat transfer coefficient to fluid"
e=0.5 [-] "emissivity of surface of insulation"
k_ins=0.5 [W/m-K] "conductivity of insulation"
th_ins_inch=0.5 [inch] "thickness of insulation, in inch"
th_ins=th_ins_inch*convert(inch,m) "thickness of insulation"
R``=1e-4 [K-m^2/W] "area specific contact resistance"
q``=10000 [W/m^2] "heat flux provided by heater"
L=1 [m] "per unit length basis"

The conduction resistance of the tube and the insulation is:

⎛D ⎞
ln ⎜ out ⎟
D
Rcond ,tube = ⎝ in ⎠ (1)
2 π ktube L
⎛ D + 2 thins ⎞
ln ⎜ out ⎟
⎝ Dout ⎠
Rcond ,ins = (2)
2 π kins L

The contact resistance is:

R′′
Rc = (3)
π Dout L

The convection resistance between the water and the tube surface is:

1
Rconv ,in = (4)
hin π Din L

1
Rconv ,out = (5)
hout π ( Dout + 2 thins ) L

R_cond_tube=ln(D_out/D_in)/(2*pi*k_tube*L) "tube conduction resistance"


R_cond_ins=ln((D_out+2*th_ins)/D_out)/(2*pi*k_ins*L) "insulation conduction resistance"
R_c=R``/(pi*D_out*L) "contact resistance"
R_conv_in=1/(h_bar_in*pi*D_in*L) "internal convection resistance"
R_conv_out=1/(h_bar_out*pi*(D_out+2*th_ins)*L) "external convection resistance"

In order to calculate the radiation resistance, the external surface temperature Text, is assumed.
The radiation resistance is:

1
Rrad = (6)
ε σ π ( Dout + 2 thins ) L (Text2 + T∞2 )(Text + T∞ )

T_ext=T_infinity "guess for external surface temperature"


R_rad=1/(e*sigma#*pi*(D_out+2*th_ins)*L*(T_ext^2+T_infinity^2)*(T_ext+T_infinity))
"radiation resistance"

An energy balance on the heater is:

q ′′ π Dout =
(Th − T∞ ) +
(T h − Tf ) (7)
−1
⎡ 1 1 ⎤ Rc + Rcond ,tube + Rconv ,in
Rc + Rcond ,ins + ⎢ + ⎥
⎣ Rrad Rconv ,out ⎦

q``*pi*D_out*L=(T_h-T_infinity)/(R_c+R_cond_ins+(1/R_rad+1/R_conv_out)^(-1))+&
(T_h-T_f)/(R_c+R_cond_tube+R_conv_in) "energy balance on heater"
The problem is solved and the guess values are updated. The assumed value of Text is
commented out and then recalculated based on the solution. The rate of heat transfer to the
ambient is:

qout =
(Th − T∞ ) (8)
−1
⎡ 1 1 ⎤
Rc + Rcond ,ins + ⎢ + ⎥
⎣ Rrad Rconv ,out ⎦

and the external surface temperature is:

Text = Th − qout ( Rc + Rcond ,ins ) (9)

{T_ext=T_infinity} "guess for external surface temperature"


q_dot_out=(T_h-T_infinity)/(R_c+R_cond_ins+(1/R_rad+1/R_conv_out)^(-1))
"rate of heat transfer to ambient"
T_ext=T_h-q_dot_out*(R_c+R_cond_ins) "recalculate external temperature"
T_h_C=converttemp(K,C,T_h) "heater temperature, in C"

which leads to Th = 90.85ºC. The rate of heat transfer to the water is computed:

qin =
(T h − Tf ) (10)
Rc + Rcond ,tube + Rconv ,in

q_dot_in=(T_h-T_f)/(R_c+R_cond_tube+R_conv_in) "rate of heat transfer to fluid"

which leads to qin = 581 W.

c.) What is the efficiency of the heater (the ratio of the power provided to the water to the power
provided to the heater)?

The efficiency is defined as:

qin
η= (11)
qin + qout

eta=q_dot_in/(q_dot_out+q_dot_in) "efficiency of heater"

which leads to η = 83.2%.

d.) The efficiency of the heater is less than 100% due to heat lost to the atmosphere. Rank the
following parameters in terms of their relative importance with respect to limiting heat loss to
the atmosphere: ε, Rc′′ , kins, hout . Justify your answers using your resistance network and a
discussion of the magnitude of the relevant resistances.
The resistances separating the heater from the ambient include: Rrad = 1.89 K/W, Rc = 0.0014
K/W, Rcond,ins = 0.24 K/W, and Rconv,out = 0.45 K/W. This suggests that convection is more
important than radiation and also more important than either conduction or contact resistance.
Conduction is the next-most important resistance followed by radiation and finally contact
resistance (which is absolutely unimportant). Therefore, the relative importance is: hout , kins, ε,
and Rc′′ .

e.) Plot the efficiency as a function of the insulation thickness for 0 inch < thins < 1.5 inch.
Explain the shape of your plot.

Figure 3 illustrates the efficiency as a function of the insulation thickness. Notice that initially as
the thickness increases the efficiency actually drops; this is because the convection resistance
(which, from (d), is the most important resistance) will decrease with insulation thickness since
the area for convection increases. Eventually, the insulation conduction resistance increases to
the point where it becomes important and the efficiency begins to increase.
0.852

0.848

0.844
Efficiency

0.84

0.836

0.832

0.828
0 0.2 0.4 0.6 0.8 1 1.2 1.4 1.6
Thickness of insulation (inch)
Figure 3: Efficiency as a function of the insulation thickness.

f.) The temperature on the internal surface of the tube must remain below the saturation
temperature of the water in order to prevent any local boiling of the water. Based on this
criteria, determine the maximum possible heat flux that can be applied to the heater (for thins
= 0.5 inch).

The internal surface temperature of the tube is computed:

Ts ,in = Th − qin ( Rc + Rcond ,tube ) (12)

The saturation temperature of the water (Tsat) is obtained from EES' internal thermodynamic
property routines. The prescribed value of the heat flux is cancelled out and, instead, Ts,in is set
equal to Tsat.
{q``=10000 [W/m^2]} "heat flux provided by heater"
T_s_in=T_h-q_dot_in*(R_c+R_cond_tube) "inner surface temperature of tube"
T_sat=temperature(Water,p=p_f,x=1 [-]) "saturation temperature"
T_s_in=T_sat "maximum allowable temperature"

which leads to q ′′ = 14,061 W/m2.


Problem 1.3-1: Composite Wall with Generation
A plane wall is composed of two materials, A and B, with the same thickness, L, as shown in
Figure P1.3-1. The same, spatially uniform, volumetric rate of generation is present in both
materials ( g ′′′ = g ′′′A = g ′′′B ) and the wall is at steady state. The conductivity of material B is twice
that of material A (kB = 2 kA). The left side of the wall is adiabatic and the right side is
maintained at a temperature Tx=2L = To.

L L
material A material B

kA kB = 2kA
g ′′′A g ′′′B = g ′′′A
To
x

Figure P1.3-1: Plane wall composed of materials A and B.

a.) Sketch the rate of heat transfer as a function of position within the wall ranging from x = 0
(the left face of material A) to x = 2L (the right face of material B). Note that the sketch
should be qualitatively correct, but cannot be quantitative as you have not been given any
numbers for the problem.

The heat transfer rate must increase linearly from zero at x=0; to see this, consider the energy
balance on the control volume shown in Figure 2.

Figure 2: Control volume.

The heat transfer rate at any position x must be:

q = g ′′′ A x (1)
Figure 3: Rate of heat transfer as a function of position in the wall

b.) Sketch the temperature as a function of position within the wall. Again, be sure that your
sketch has the correct qualitative features.

The temperature gradient is, according to Fourier’s law:

dT q
=− (2)
dx k

Therefore, the temperature gradient will become increasingly negative as you move towards
positive x; however, there will be a step change in the temperature gradient at the interface
between materials A and B (i.e., at x=L). The conductivity of material B is twice that of A and
so the temperature gradient at x=L+ will be half that of x=L-.

To x
0 L 2L
Figure 4: Temperature as a function of position in the wall
Problem 1.3-2: Composite Wall with Generation
A plane wall is composed of two materials, A and B, with the same conductivity k and thickness
L, as shown in Figure P1.3-2. The left side of material A is adiabatic (i.e., well insulated) and
the right side of material B is held at a temperature TL. There is no volumetric generation in
material A but material B experiences a uniform rate of volumetric generation of thermal energy,
g ′′′ .

L L
material A material B

kA kB = kA
′′′ TL
g A = 0 g ′′′
B
= g ′′′
x

Figure P1.3-2: Plane wall composed of materials A and B.

a.) Sketch the rate of heat transfer ( q ) as a function of position within the wall. Note that the
sketch should be qualitatively correct but cannot be quantitative as you have not been given
any numbers for the problem.

The control volume shown in Figure 2 can be used to evaluate the heat transfer from 0 < x < L.

Figure 2: Control volume.

The heat transfer rate must be zero until x = L:

q = 0 for 0 < x < L (1)

The control volume shown in Figure 3 can be used to evaluate the heat transfer from L < x < 2 L.
Figure 3: Control volume.

The heat transfer rate must increase linearly from x = L to x= 2 L:

q = g ′′′ A ( x − L ) for L < x < 2 L (2)

The heat transfer rate is sketched in Figure 4.

Figure 4: Rate of heat transfer as a function of position in the wall

b.) Sketch the temperature as a function of position within the wall. Again, be sure that your
sketch has the correct qualitative features.

The temperature gradient is given by Fourier’s law:

dT q
=− (3)
dx k

Therefore, the temperature gradient will be zero within material A and then become increasingly
negative as you move towards positive x within material B. Figure 5 shows a sketch of the
temperature distribution.
Figure 5: Temperature as a function of position in the wall
Problem 1.3-3 (1-7 in text): Critical Evaluation of a Solution
One of the engineers that you supervise has been asked to simulate the heat transfer problem
shown in Figure P1.3-3(a). This is a 1-D, plane wall problem (i.e., the temperature varies only in
the x-direction and the area for conduction is constant with x). Material A (from 0 < x < L) has
conductivity kA and experiences a uniform rate of volumetric thermal energy generation, g ′′′ .
The left side of material A (at x = 0) is completely insulated. Material B (from L < x < 2L) has
lower conductivity, kB < kA. The right side of material B (at x= 2L) experiences convection with
fluid at room temperature (20°C). Based on the facts above, critically examine the solution that
has been provided to you by the engineer and is shown in Figure P1.3-3(b). There should be a
few characteristics of the solution that do not agree with your knowledge of heat transfer; list as
many of these characteristics as you can identify and provide a clear reason why you think the
engineer’s solution must be wrong.
250

200

150

Temperature (°C)
100
L L
50
material A material B
0
kA kB < kA
h , T f = 20°C -50
g ′′′A = g ′′′ g B′′′ = 0 Material A Material B
x -100
0 L 2L
Position (m)
(a) (b)
Figure P1.3-3: (a) Heat transfer problem and (b) "solution" provided by the engineer.

1. The left side of material A is insulated; therefore, the temperature gradient should be zero.
2. Material A has a higher conductivity than material B; therefore, at x = L the temperature
gradient should be larger in material B than in material A.
3. Heat is transferred to the fluid at 20°C; therefore the temperature at x = 2 L must be greater
than 20°C.
Problem 1.3-4: Cylinder Boundary Conditions
A cylinder with conductivity k experiences a uniform rate of volumetric generation g ′′′ , as
shown in Figure P1.3-4. The cylinder experiences 1-D, steady state conduction heat transfer in
the radial direction and therefore the general solution to the ordinary differential equation for
temperature (T) is:

g ′′′ r 2
T =− + C1 ln ( r ) + C2 (1)
4k

where r is the radial location and C1 and C2 are undetermined constants. At the inner radius of
the cylinder (r = rin), a heater applies a uniform rate of heat transfer, qin . At the outer radius of
the cylinder (r = rout), the temperature is fixed at Tout. The length of the cylinder is L. Write the
two algebraic equations that can be solved in order to obtain the constants C1 and C2. Your
equations must contain only the following symbols in the problem statement: qin , Tout, k, rin, rout,
L, g ′′′ , C1, and C2. Do not solve these equations.

qin

L
k , g ′′′
Tout

rin
rout
Figure P1.3-4: Cylinder with uniform volumetric generation.

At the outer surface, the temperature is specified and therefore the boundary condition is:

g ′′′ rout
2
Tout = − + C1 ln ( rout ) + C2 (2)
4k

At the inner surface, the temperature is not specified and therefore it is necessary to do an energy
balance on this interface, as shown in Figure 2.
Figure 2: Interface balance at r = rin.

The interface energy balance is:

qin = q@ r = rin (3)

Substituting Fourier's law for q@ r = rin leads to:

dT
qin = − k 2 π rin L (4)
dr r = rin

Substituting the general solution, Eq. (1), into Eq. (4) leads to:

⎡ g ′′′ rin C1 ⎤
qin = −k 2 π rin L ⎢ − + ⎥ (5)
⎣ 2k rin ⎦
Problem 1.3-5: Windings
Figure P1.3-5(a) illustrates a motor that is constructed using windings that surround laminated
iron poles. You have been asked to estimate the maximum temperature that will occur within the
windings. The windings and poles are both approximated as being cylindrical, as shown in
Figure P1.3-5(b).

rout = 2 cm
L = 2 cm rin = 1 cm
pole
r Tair = 25°C
h = 25 W/m -K
2

windings
Tpole = 50°C windings
k = 1 W/m-K
g ′′′ = 1x10 W/m
6 3

(a) (b)
Figure P1.3-5: (a) Concentrated winding for a permanent magnet motor, (b) cylindrical model of the
windings

The windings are a complicated composite formed from copper conductor, insulation and air that
fills the gaps between adjacent wires. However, in most models, the windings are represented by
a solid with equivalent properties that account for this underlying structure. You can therefore
consider the windings in Figure P1.3-5(b) to be a solid. The electrical current in the windings
causes an ohmic dissipation that can be modeled as a uniform volumetric generation rate of
g ′′′ =1x106 W/m3. The conductivity of the windings is k = 1.0 W/m-K. The inner radius of the
windings is rin = 1.0 cm and the outer radius is rout = 2.0 cm. The windings are L = 2.0 cm long
and the upper and lower surfaces may be assumed to be insulated so that the temperature in the
windings varies only in the radial direction. The stator pole is conductive and cooled externally;
therefore, you can assume that the stator tooth has a uniform temperature of Tpole = 50°C.
Neglect any contact resistance between the inner radius of the winding and the pole; therefore,
the temperature of the windings at r = rin is Tpole. The outer radius of the windings is exposed to
air at Tair = 20°C with a heat transfer coefficient of h = 25 W/m2-K.
a.) Derive the governing differential equation for the temperature within the windings (i.e., the
differential equation that is valid from rin < r < rout). You should end up with an ordinary
differential equation for T in terms of the symbols provided in the problem statement.
Clearly show your steps, which should include:
1. define a differentially small control volume,
2. do an energy balance on your control volume,
3. expand the r + dr terms in your energy balance and take the limit as dr → 0,
4. substitute rate equations into your energy balance.

A differential control volume is shown in Figure 3.


qr qr + dr
g
r dr

Figure 3: A differentially small control volume.

The energy balance on the control volume is:

qr + g = qr + dr (1)

or, after expanding the r + dr term:

dqr d 2 q dr 2 d 3 qr dr 3
qr + g = qr + dr + 2r + + ... (2)
dr dr 2! dr 3 3!

Taking the limit as dr approaches zero leads to:

dqr
qr + g = qr + dr (3)
dr

or

dqr
g = dr (4)
dr

The parameter g is the rate of thermal energy generation within the control volume, which can
be expressed as the product of the volume of the control volume and the volumetric rate of
generation:

g = g ′′′ 2 π r L dr (5)

and the conduction term is expressed using Fourier’s law:

dT
q = − k 2 π r L (6)
dr

Substituting Eqs. (5) and (6) into Eq. (4) leads to:

d ⎡ dT ⎤
g ′′′ 2 π r L dr = ⎢ −k 2 π r L dr (7)
dr ⎣ dr ⎥⎦
or

d ⎡ dT ⎤
g ′′′ r = −k r (8)
dr ⎢⎣ dr ⎥⎦

b.) Specify the boundary conditions for your differential equation. This should be easy at the
inner radius, where the temperature is specified, but you will need to carry out an interface
energy balance at r = rout.

The boundary condition at the inner radius is:

Tr = rin = Tpole (9)

An interface energy balance at the outer radius is shown in Figure 4.

qr = r
qconv
out

Figure 4: An interface energy balance at the outer radius.

The interface energy balance leads to:

qr = rout = qconv (10)

Substituting Fourier's law and Newton's law of cooling into Eq. (10) leads to:

− k 2 π rout L
dT
dr
(
= 2 π rout L h Tr = rout − Tair ) (11)
r = rout

or

−k
dT
dr
(
= h Tr = rout − Tair ) (12)
r = rout

c.) Solve the governing differential equation that you derived in part (a) by integrating twice.
You should end up with a solution that involves two constants of integration, C1 and C2.

Equation (8) is rearranged:


⎡ dT ⎤ g ′′′ r
d⎢r = − dr (13)
⎣ dr ⎥⎦ k

and integrated:

⎡ dT ⎤ ⎛ g ′′′ r ⎞
∫ d ⎣⎢ r dr ⎥⎦ = ∫ ⎝⎜ − k ⎠
⎟ dr (14)

to obtain:

dT g ′′′ r 2
r =− + C1 (15)
dr 2k

so the temperature gradient is:

dT g ′′′ r C1
=− + (16)
dr 2k r

Equation (16) is again rearranged:

⎛ g ′′′ r C1 ⎞
∫ dT = ∫ ⎜⎝ − 2k
+ ⎟ dr
r ⎠
(17)

to obtain:

g ′′′ r 2
T =− + C1 ln ( r ) + C2 (18)
4k

d.) Substitute your answer from part (c) into the boundary conditions specified in part (b) to
obtain two equations for your two unknown constants of integration, C1 and C2.

Substituting Eq. (18) into Eq. (9) leads to:

g ′′′ rin2
− + C1 ln ( rin ) + C2 = Tpole (19)
4k

Substituting Eqs. (18) and (16) into Eq. (12) leads to:

⎛ g ′′′ rout C1 ⎞ ⎛ g ′′′ rout


2

−k ⎜ − + ⎟ = h⎜− + C1 ln ( rout ) + C2 − Tair ⎟ (20)
⎝ 2k rout ⎠ ⎝ 4k ⎠
e.) Implement your results from (c) and (d) in EES and prepare a plot of the temperature in the
stator as a function of radius.

The inputs are entered in EES:

$UnitSystem SI MASS RAD PA K J


$TABSTOPS 0.2 0.4 0.6 0.8 3.5 in

"Inputs"
k=1 [W/m-K] "winding conductivity"
gv=1e6 [W/m^3] "winding volumetric generation"
T_air=converttemp(C,K,25) "air temperature"
h=25 [W/m^2-K] "heat transfer coefficient"
r_out=2.0 [cm]*convert(cm,m) "outer radius of windings"
r_in=1.0 [cm]*convert(cm,m) "inner radius of windings"
T_pole=converttemp(C,K,50) "pole temperature"
L=2.0 [cm]*convert(cm,m) "length of windings"

The two algebraic equations for C1 and C2, Eqs. (19) and (20), are entered:

"boundary conditions"
-gv*r_in^2/(4*k)+C_1*ln(r_in)+C_2=T_pole "at r=r_in"
-k*(-gv*r_out/(2*k)+C_1/r_out)=h*(-gv*r_out^2/(4*k)+C_1*ln(r_out)+C_2-T_air) "at r=r_out"

and the solution is obtained using Eq. (18).

"solution"
T=-gv*r^2/(4*k)+C_1*ln(r)+C_2
r_cm=r*convert(m,cm)
T_C=converttemp(K,C,T)

Figure 5 illustrates the temperature as a function of radial position within the windings.

95

90

85
Temperature (°C)

80

75

70

65

60

55

50
1 1.1 1.2 1.3 1.4 1.5 1.6 1.7 1.8 1.9 2
Radius (cm)
Figure 5: Temperature as a function of position within the windings.
PROBLEM 1.3-6: Heating a Pipe
You need to transport water through a pipe from one building to another in an arctic
environment, as shown in Figure P1.3-6. The water leaves the building very close to freezing, at
Tw = 5°C, and is exposed to a high velocity, very cold wind. The temperature of the surrounding
air is Ta = -35°C and the heat transfer coefficient between the outer surface of the pipe and the air
is ha = 50 W/m2-K. The pipe has an inner radius of rh,in = 2 inch and an outer radius of rh,out = 4
inch and is made of a material with a conductivity kh = 5 W/m-K. The heat transfer coefficient
between the water and the inside surface of the pipe is very large and therefore the inside surface
of the pipe can be assumed to be at the water temperature. Neglect radiation from the external
surface of the pipe.

rh,out = 4 inch Ta = −35°C


ha = 50 W/m -K
2
rh,in = 2 inch

water
Tw = 5°C

kh = 5 W/m-K
Figure P1.3-6: Heated pipe transporting near freezing water in an arctic environment.

a.) Determine the rate of heat lost from the water to the air for a unit length, L=1 m, of pipe.

The known information is converted to base SI units and entered in EES. The units for each
variable are entered in the Variable Information window.

$UnitSystem SI MASS RAD PA K J


$TABSTOPS 0.2 0.4 0.6 0.8 3.5 in

"Inputs"
r_h_in=2 [inch]*convert(inch,m) "inner radius of plastic pipe"
r_h_out=4.0 [inch]*convert(inch,m) "outer radius of heater"
T_a=converttemp(C,K,-35) "air temperature"
T_w=converttemp(C,K,5) "water temperature"
h_a=50 [W/m^2-K] "air to heater heat transfer coefficient"
k_h=5 [W/m-K] "heater conductivity"
L=1 [m] "unit length of pipe"

The resistance network that represents the situation includes a conduction resistance through the
pipe (Rcond) and a convection resistance from the outer surface of the pipe (Rconv).

⎛r ⎞
ln ⎜⎜ h ,out ⎟⎟
rh ,in ⎠
Rcond = ⎝ (1)
2 π kh L
and

1
Rconv = (2)
ha 2 π rh ,out L

and the total heat loss is:

q =
(Tw − Ta ) (3)
Rcond + Rconv

These equations are entered in EES:


"Part a"
R_cond=ln(r_h_out/r_h_in)/(2*pi*k_h*L) "conduction through the pipe"
R_conv=1/(h_a*2*pi*r_h_out*L) "convection from outer surface"
q_dot=(T_w-T_a)/(R_cond+R_conv) "heat loss"

The heat loss is 750 W per m of pipe.

b.) Plot the heat lost from the water as a function of the pipe outer radius from 0.06 m to 0.3 m
(keep the same inner radius for this study). Explain the shape of your plot.

The value of r_h_out in the input section is commented (highlight the variable and select
comment). A parametric table is created that includes the variables r_h_out and q_dot; the value
of r_h_out in the table is varied from 0.01 m to 0.3 m (right click on the r_h_out column header
and select Alter Values). The results in the parametric table are plotted in Figure 2.

Figure 2: Heat transfer from the water as a function of the outer radius of the pipe.

The result can be understood by plotting the resistances R_conv, R_cond, and R_conv+R_cond as a
function of the outer radius of the pipe (as shown in Figure 3). Not that you’ll need to define a
new variable, R_total:
R_total=R_cond+R_conv "total resistance"

in order to make the plot.

Figure 3: Thermal resistance as a function of the pipe outer radius.

Initially, the convection resistance drops more rapidly than the conduction resistance increases as
the radius increases; therefore, initially the total resistance is reduced with radius and the heat
loss increases. Eventually, the conduction resistance dominates the convection resistance and
therefore the total resistance rises with outer radius and therefore the heat loss is reduced.

In order to reduce the heat loss from the water and therefore prevent freezing, you run current
through the pipe material so that it generates thermal energy at with a uniform volumetric rate,
g ′′′ = 5x105 W/m3.
c.) Develop an analytical model capable of predicting the temperature distribution within the
pipe. Implement your model in EES.

The general solution for the temperature distribution in a cylindrical system with a constant rate
of generation is:

g ′′′ r 2
T =− + C1 ln ( r ) + C2 (4)
4k

where C1 and C2 are constants of integration that must come from the boundary conditions. The
boundary conditions are a set temperature at rh,in:

g ′′′ rh2,in
Tw = − + C1 ln ( rh ,in ) + C2 (5)
4 kh

and an interface energy balance at rh,out:


− kh
dT
dr r = rh ,out
(
= ha Tr = rh ,out − Ta ) (6)

The derivative of temperature with respect to radius can be found by manipulating Eq. (4) or in
Table 3-1:

dT g ′′′ r C1
=− + (7)
dr 2k r

Substituting Eq. (7) and Eq. (4) into Eq. (6) leads to:

⎡ g ′′′ rh ,out C1 ⎤ ⎛ g ′′′ rh2,out ⎞


− kh ⎢ − + ⎥ = ha ⎜⎜ − + C1 ln ( rh ,out ) + C2 − Ta ⎟⎟ (8)
⎢⎣ 2 kh rh ,out ⎥⎦ ⎝ 4 kh ⎠

Equations (5) and (8) are programmed in EES to obtain the constants C1 and C2.

g```_dot=5e5 [W/m^3] "volumetric generation rate"


"boundary conditions"
T_w=-g```_dot*r_h_in^2/(4*k_h)+C1*ln(r_h_in)+C2
-k_h*(-g```_dot*r_h_out/(2*k_h)+C1/r_h_out)=h_a*(-g```_dot*r_h_out^2/(4*k_h)+C1*ln(r_h_out)+C2-T_a)

Note that the units of the constant should be set to K (Figure 4):

Figure 4: Variable Information window.

However, if you check units now you will obtain a unit error (Figure 5).
Figure 5: Unit error.

because the argument of the natural logarithm has units m. This cannot be helped; if the algebra
associated with explicitly solving for each constant was followed through and these constants
were substituted back into Eq. (4) then you would find that the arguments of the natural
logarithm can be expressed as the ratio of two radii.

The solution, Eq. (4), is programmed in EES:

T=-g```_dot*r^2/(4*k_h)+C1*ln(r)+C2 "solution"

d.) Prepare a plot showing the temperature as a function of position within the pipe.

The radius is expressed in terms of a non-dimensional radius (r_bar) to facilitate making the
parametric table (it is easier to vary r_bar from 0 to 1 than it is to vary r from r_h_in to r_h_out,
particularly if you plan on varying r_h_in or r_h_out).

r_bar=(r-r_h_in)/(r_h_out-r_h_in) "dimensionless radius used to make plots"


{r_bar=0}
T_C=converttemp(K,C,T)

The temperature distribution is shown in Figure 6.

Figure 6: Temperature in pipe as a function of radius.


e.) Calculate the heat transfer from the water when you are heating the pipe.

The heat transfer from the water is obtained by applying Fourier’s law at r = rh,in:

dT
qw = − kh 2 π rh ,in L (9)
dr r = rh ,in

Substituting Eq. (7) into Eq. (9):

⎡ g ′′′ rh ,in C1 ⎤
qw = −kh 2 π rh ,in L ⎢ − + ⎥ (10)
⎢⎣ 2k rh ,in ⎥⎦

q_dot_w=-k_h*(-g```_dot*r_h_in/(2*k_h)+C1/r_h_in)*2*pi*r_h_in*L "heat transfer from the water"

The heat transfer rate from the water is -8336 W per m of pipe; that is, heat is transferred to the
water, which is evident from the temperature distribution. While you don’t want the water to
freeze, you probably also don’t want to heat it and therefore g ′′′ =5e5 W/m3 is probably too high.

f.) Determine the volumetric generation rate that is required so that there is no heat transferred
from the water.

EES can provide this solution. Simply comment out the generation rate that you set and then
specify that the heat transfer rate from the water is 0.

{g```_dot=5e5 [W/m^3]} "volumetric generation rate"


"boundary conditions"
T_w=-g```_dot*r_h_in^2/(4*k_h)+C1*ln(r_h_in)+C2
-k_h*(-g```_dot*r_h_out/(2*k_h)+C1/r_h_out)=h_a*(-g```_dot*r_h_out^2/(4*k_h)+C1*ln(r_h_out)+C2)

T=-g```_dot*r^2/(4*k_h)+C1*ln(r)+C2 "solution"
r_bar=(r-r_h_in)/(r_h_out-r_h_in) "dimensionless radius used to make
plots"
r_bar=0
T_C=converttemp(K,C,T)

q_dot_w=-k_h*(-g```_dot*r_h_in/(2*k_h)+C1/r_h_in)*2*pi*r_h_in*L "heat transfer from the water"


q_dot_w=0

The volumetric generation rate that results in no heat transfer to the water is 4.12x104 W/m3.
Problem 1.3-7: Nuclear Fuel Element
Figure P1.3-7 illustrates a spherical, nuclear fuel element which consists of a sphere of
fissionable material (fuel) with radius rfuel = 5 cm and kfuel = 1 W/m-K that is surrounded by a
spherical shell of metal cladding with outer radius rclad = 7 cm and kclad = 300 W/m-K. The outer
surface of the cladding is exposed to helium gas that is being heated by the reactor. The
convection coefficient between the gas and the cladding surface is hgas = 100 W/m2-K and the
temperature of the gas is Tgas = 500ºC. Neglect radiation heat transfer from the surface.

Inside the fuel element, fission fragments are produced which have high velocities. The products
collide with the atoms of the material and provide the thermal energy for the reactor. This
process can be modeled as a volumetric source of heat generation in the material that is not
uniform throughout the fuel. The volumetric generation ( g ′′′ ) can be approximated by the
function:
b
⎛ r ⎞
g ′′′ = g e′′′ ⎜ ⎟⎟
⎜ rfuel
⎝ ⎠
where g e′′′ = 5x105 W/m3 is the volumetric rate of heat generation at the edge of the sphere and b
= 1.0; note that the parameter b is a dimensionless positive constant that characterizes how
quickly the generation rate increases in the radial direction.

fissionable material
kfuel = 1 W/m-K
rfuel = 5 cm

rclad = 7 cm

hgas = 100 W/m -K


2

g ′′′
Tgas = 500°C

cladding
kclad = 300 W/m-K
Figure P1.3-7: Spherical fuel element surrounded by cladding

a.) Enter the problem inputs into EES; be sure to set the units appropriately.

The inputs are entered according to:

$UnitSystem SI MASS RAD PA K J


$TABSTOPS 0.2 0.4 0.6 0.8 3.5 in

"Inputs"
r_fuel=5[cm]*convert(cm,m) "radius of fuel"
k_fuel=1 [W/m-K] "conductivity of fuel"
r_clad=7[cm]*convert(cm,m) "cladding radius"
k_clad=300 [W/m-K] "cladding conductivity"
h_gas=100 [W/m^2-K] "convection coefficient"
T_gas=converttemp(C,K,500) "gas temperature"
gve=5e5 [W/m^3] "generation at the center"
b=1 [-] "decay constant"

b.) Determine the governing differential equation that applies within the sphere (i.e., your
differential equation should be valid for 0 < r < rfuel). The differential equation should
include only those symbols given in the problem statement. Clearly show your steps.

A differential control volume is shown in Figure 2 and includes conduction at r and r+dr at the
inner and outer surfaces of the spherical shell as well as generation within the enclosed volume.

Figure 2: Differential control volume

The energy balance suggested by Figure 2 is:

qr + g = qr + dr (1)

The term at r + dr can be expanded:

dqr
qr + dr = qr + dr (2)
dr

and substituted into Eq. (1):

dqr
qr + g = qr + dr (3)
dr

and simplified:

dqr
g = dr (4)
dr

The conduction is governed by Fourier’s Law:

dT
qr = −k fuel 4 π r 2 (5)
dr

and the generation is the product of the volume and the local generation rate:
b
⎛ r ⎞
g = 4 π r dr g e′′′⎜
2
⎟⎟ (6)
⎜r
⎝ fuel ⎠

The rate equations are substituted into Eq. (4):

b
⎛ r ⎞ d ⎡ 2 dT ⎤
4 π r dr g e′′′⎜
2
⎟⎟ = ⎢ −k fuel 4 π r dr (7)
⎜r
⎝ fuel ⎠ dr ⎣ dr ⎥⎦

which can be simplified:

b
d ⎡ 2 dT ⎤ 2 g e′′′ ⎛ r ⎞
r +r ⎜ ⎟⎟ = 0 (8)
dr ⎢⎣ dr ⎥⎦ k fuel ⎜⎝ rfuel ⎠

Notice that it is not possible to cancel the r2 term from each side of Eq. (7) because it appears
within the differential on the right side.

c.) Enter the governing differential equation into Maple and use Maple to obtain a solution that
includes two constants of integration.

The generation function and the governing differential equation are entered according to:

> gen:=gve*(r/r_fuel)^b;
b
gen := gve ⎛⎜⎜
r ⎞
⎟⎟
⎝ r_fuel ⎠
> GDE:=diff(r^2*diff(T(r),r),r)+r^2*gen/k_fuel=0;
b
r gve ⎛⎜⎜ ⎞
r
⎟⎟
2
⎛ ⎞
2
GDE := 2 r ⎛⎜⎜ T( r ) ⎞⎟⎟ + r 2 ⎜⎜ 2 T( r ) ⎟⎟ + ⎝ ⎠ =0
d d r_fuel
⎝ d r ⎠ ⎝ dr ⎠ k_fuel

and solved using the dsolve command:

> Tr:=rhs(dsolve(GDE));
b
r 2 gve ⎛⎜⎜
r ⎞
⎟⎟
Tr := −
_C1
− ⎝ r_fuel ⎠ + _C2
r k_fuel ( b2 + 6 + 5 b )

Notice the two constants of integration that must be determined using the boundary conditions.

d.) The boundary condition at the center of the sphere is that the temperature must remain finite;
this should eliminate one of the constants of integration in your Maple solution. Which
constant must be zero?
In order for the temperature to remain finite as r approaches 0, the constant C1 must be zero.

e.) Determine a symbolic equation for the remaining boundary condition (the one at r = rfuel) in
terms of the temperature and temperature gradient evaluated at r = rfuel.

An interface energy balance at r = rfuel includes conduction from the fuel and conduction into the
cladding, as shown in Figure 3.

Figure 3: Interface balance at r = rfuel

The energy balance suggested by Figure 3 is:

qr = rruel = qout (9)

The conduction term on the left side of Eq. (9) is evaluated using Fourier’s law:

dT
qr = rruel = − k fuel 4 π rfuel
2
(10)
dr r = r fuel

while the heat transfer out of the cladding is driven by the difference between the temperature at
interface between the fuel and the cladding and the temperature of the surrounding gas. The heat
transfer is resisted by the sum of the conduction resistance of the cladding (Rclad):

1 ⎡ 1 1 ⎤
Rclad = ⎢ − ⎥ (11)
4 π kclad ⎢⎣ rfuel rclad ⎥⎦

and the convection resistance (Rconv):

1
Rconv = (12)
4π r 2
clad hgas
so that:

Trfuel − Tgas
qout = (13)
Rclad + Rconv

Subsituting Eqs. (13) and (10) into Eq. (9) leads to:

dT Trfuel − Tgas
− k fuel 4 π rfuel
2
= (14)
dr r = r fuel Rclad + Rconv

Equation (14) provides a single equation for the unknown constant of integration, C2.

f.) Use the expressions from Maple to determine the required constant of integration within
EES. Copy the solution for the temperature in the cladding from Maple and paste it into
EES; modify the expression as necessary for compatibility (remember to eliminate the C1
term) and use it to generate plot of temperature vs radius within the cladding.

The solution in Maple is manipulated using the diff and eval commands:

> dTdr_rfuel:=eval(diff(Tr,r),r=r_fuel);
_C1 2 r_fuel gve r_fuel gve b
dTdr_rfuel := − −
r_fuel 2
k_fuel ( b + 6 + 5 b ) k_fuel ( b2 + 6 + 5 b )
2

> T_rfuel=eval(Tr,r=r_fuel);
_C1 r_fuel 2 gve
T_rfuel = − − + _C2
r_fuel k_fuel ( b2 + 6 + 5 b )

The expressions are copied and pasted into EES; the constant C1 is eliminated and the
expressions are modified to be compatible with EES (the := is replaced with = and the _C2 is
replaced with C2). Also, the _C1 portion of the expression for T_rfuel is deleted.

"Boundary condition expressions"


dTdr_rfuel =-2*r_fuel*gve/k_fuel/(b^2+6+5*b)-r_fuel*gve/k_fuel/(b^2+6+5*b)*b "from Maple"
T_rfuel =-r_fuel^2*gve/k_fuel/(b^2+6+5*b)+C2 "from Maple"

The two resistance values must be calculated:

Rst_clad=(1/r_fuel-1/r_clad)/(4*pi*k_clad) "conduction resistance of cladding"


Rst_conv=1/(4*pi*r_clad^2*h_gas) "convection resistance"

Note that the use of R_clad to represent the cladding resistance would have resulted in problems
because of the existence of the variable r_clad; EES is not case-sensitive. Finally, the boundary
condition, Eq. (14), is programmed:

-4*pi*r_fuel^2*k_fuel*dTdr_rfuel=(T_rfuel-T_gas)/(Rst_clad+Rst_conv) "boundary condition"


Solving the problem should provide a solution for C2 = -1916 K; note that the units should also
be set and checked for all of your variables.

The Maple solution is cut and pasted into EES:

"Solution"
T=-r^2*gve/k_fuel/(b^2+6+5*b)*(r/r_fuel)^b+C2 "solution from Maple"

To facilitate plotting, the solution is converted from K to °C and the radial location is defined in
terms of a dimensionless radial position (r_bar) that goes from 0 to 1 (therefore, if the radius of
the fuel sphere changes in parametric studies it is not necessary to reset the parametric table).

r_bar=r/r_fuel
T_C=converttemp(K,C,T)

Figure 4 illustrates the temperature as a function of radius

Figure 4: Temperature distribution in the fuel


PROBLEM 1.3-8 (1-8 in text): Hay Temperature
Freshly cut hay is not really dead; chemical reactions continue in the plant cells and therefore a
small amount of heat is released within the hay bale. This is an example of the conversion of
chemical to thermal energy and can be thought of as thermal energy generation. The amount of
thermal energy generation within a hay bale depends on the moisture content of the hay when it
is baled. Baled hay can become a fire hazard if the rate of volumetric generation is sufficiently
high and the hay bale sufficiently large so that the interior temperature of the bale reaches 170°F,
the temperature at which self-ignition can occur. Here, we will model a round hay bale that is
wrapped in plastic to protect it from the rain. You may assume that the bale is at steady state and
is sufficiently long that it can be treated as a one-dimensional, radial conduction problem. The
radius of the hay bale is Rbale = 5 ft and the bale is wrapped in plastic that is tp = 0.045 inch thick
with conductivity kp = 0.15 W/m-K. The bale is surrounded by air at T∞ = 20°C with h = 10
W/m2-K. You may neglect radiation. The conductivity of the hay is k = 0.04 W/m-K.
a.) If the volumetric rate of thermal energy generation is constant and equal to g ′′′ = 2 W/m3
then determine the maximum temperature in the hay bale.

The inputs are entered in EES:

$UnitSystem SI MASS RAD PA K J


$TABSTOPS 0.2 0.4 0.6 0.8 3.5 in

R_bale=5 [ft]*convert(ft,m) "hay bale radius"


t_p=0.045 [inch]*convert(inch,m) "plastic thickness"
k_p= 0.15 [W/m-K] "plastic conductivity"
h=10 [W/m^2-K] "heat transfer coefficient"
T_infinity=converttemp(C,K,20 [C]) "ambient temperature"
L=1 [m] "per unit length of bale"
k = 0.04 [W/m-K] "conductivity of hay"
g_dot_v=2 [W/m^3] "volumetric heat generation"

This is an example of a one-dimensional steady conduction problem with constant volumetric


generation and therefore the formulae provided in Table 1-3 can used directly. The general
solution is:

g ′′′ r 2
T =− + C1 ln ( r ) + C2 (1)
4k

where C1 and C2 are constants selected to enforce the boundary conditions. The boundary
condition at the center of the bale is either that the temperature remain bounded or that the
temperature gradient be zero; either will lead to C1 = 0. An energy balance at the outer edge of
the hay bale leads to:

dT Tr = Rbale − T∞
− k 2 π Rbale L = (2)
dr r = Rbale R p + Rconv
where Rp and Rconv are the thermal resistances associated with conduction through the plastic and
convection from the outer surface of the bale, respectively:

tp
Rp = (3)
k p 2 π Rbale L

1
Rconv = (4)
h 2 π Rbale L

where L = 1 m for a problem that is done on a unit length basis. The temperature gradient and
temperature at the outer radius of the bale are obtained using Eq. (1) with C1 = 0:

dT g ′′′ Rbale
=− (5)
dr r = Rbale 2k

g ′′′ Rbale
2
Tr = Rbale = − + C2 (6)
4k

Placing equations (2) through (6) into EES allows the constants of integration to be determined:

R_p=t_p/(k_p*2*pi*R_bale*L)
"thermal resistance associated with conduction through plastic"
R_conv=1/(2*pi*R_bale*L*h)
"thermal resistance associated with convection"
dTdr_Rbale=-g_dot_v*R_bale/(2*k) "temperature gradient at outer edge"
T_Rbale=-g_dot_v*R_bale^2/(4*k)+C_2 "temperature at outer edge"
-k*2*pi*R_bale*L*dTdr_Rbale=(T_Rbale-T_infinity)/(R_p+R_conv) "interface energy balance"

The maximum temperature in the bale occurs at the center; according to Eq. (1) with C1 = 0, this
temperature is given by:

g ′′′ r 2
T =− + C1 ln ( r ) + C2 (7)
4k

T_max=C_2 "maximum bale temperature"


T_max_F=converttemp(K,F,T_max) "maximum bale temperature in F"

The maximum temperature in the hay bale is 322.3 K or 120.6°F.

b.) Prepare a plot showing the maximum temperature in the hay bale as a function of the hay
bale radius. How large can the hay bale be before there is a problem with self-ignition?

A parametric table is generated that contains the variables T_max_F and R_bale and used to
generate Figure 1.
Figure 1: Maximum temperature as a function of the bale radius.

Note that a hay bale larger than approximately 2.1 m will result in a hay fire.

Prepare a model that can consider temperature-dependent volumetric generation. Increasing


temperature tends to increase the rate of chemical reaction and therefore increases the rate of
generation of thermal energy according to: g ′′′ = a + bT where a = -1 W/m3 and b = 0.01
W/m3-K and T is in K.
c.) Enter the governing equation into Maple and obtain the general solution (i.e., a solution that
includes two constants).

The governing differential equation is obtained as discussed in Section 1.3:

d ⎛ dT ⎞
g ′′′ r = ⎜ −k r ⎟ (8)
dr ⎝ dr ⎠

This ordinary differential equation is entered in Maple:

> restart;
> ODE:=(a+b*T(r))*r=diff(-k*r*diff(T(r),r),r);
2
⎛ d ⎞ ⎛d ⎞
ODE := ( a + b T( r ) ) r = −k ⎜⎜ T ( r ) ⎟⎟ − k r ⎜ T( r ) ⎟⎟
⎝ d r ⎠ ⎜ 2
⎝ dr ⎠

and solved:

> Ts:=dsolve(ODE);
⎛ b ⎞ ⎛ b ⎞ a
Ts := T( r ) = BesselJ⎜⎜ 0, r ⎟⎟ _C2 + BesselY⎜⎜ 0, r ⎟⎟ _C1 −
⎝ k ⎠ ⎝ k ⎠ b

Note that the solution is given in the form of Bessel functions;

⎛ b ⎞ ⎛ b ⎞ a
T = −C2 BesselJ ⎜⎜ 0, r ⎟⎟ + C1 BesselY ⎜⎜ 0, r ⎟⎟ − (9)
⎝ k ⎠ ⎝ k ⎠ b

Even though we have not yet learned about Bessel functions, we can manipulate this solution
within Maple.

d.) Use the boundary conditions to obtain values for the two constants in your general solution
(hint: one of the two constants must be zero in order to keep the temperature at the center of
the hay bale finite). You should obtain a symbolic expression for the boundary condition in
Maple that can be evaluated in EES.

In part (a) we could not take the natural logarithm of 0 in Eq. (7) and therefore C1 was zero. A
similar thing happens with the Bessel functions. We can evaluate the limits of the two Bessel
functions as r → 0:

> limit(BesselJ(0,r),r=0);
1
> limit(BesselY(0,r),r=0);
−∞

The BesselY function becomes infinite and therefore C1 in Eq. (9) must be 0.

> Ts:=subs(_C1=0,Ts);
⎛ b ⎞ a
Ts := T( r ) = BesselJ⎜⎜ 0, r ⎟⎟ _C2 −
⎝ k ⎠ b

The boundary condition at the outer surface of the hay does not change; the temperature and
temperature gradient at Rbale can be evaluated symbolically using Maple:

> dTdr_Rbale:=eval(diff(rhs(Ts),r),r=R_bale);
⎛ b ⎞ b
dTdr_Rbale := −BesselJ⎜⎜ 1, R_bale ⎟⎟ _C2
⎝ k ⎠ k
> T_Rbale:=eval(rhs(Ts),r=R_bale);
⎛ b ⎞ a
T_Rbale := BesselJ⎜⎜ 0, R_bale ⎟⎟ _C2 −
⎝ k ⎠ b
These symbolic expressions are cut and paste into EES and used to replace Eqs. (5) and (6) and
provide a new constant C2:

{g_dot_v=2 [W/m^3] "volumetric heat generation"}


a=-1 [W/m^3]
"coefficients for volumetric generation function"
b=0.01 [W/m^3-K]

R_p=t_p/(k_p*2*pi*R_bale*L)
"thermal resistance associated with conduction through plastic"
R_conv=1/(2*pi*R_bale*L*h)
"thermal resistance associated with convection"
{dTdr_Rbale=-g_dot_v*R_bale/(2*k) "temperature gradient at outer edge"
T_Rbale=-g_dot_v*R_bale^2/(4*k)+C_2 "temperature at outer edge"}
dTdr_Rbale = -BesselJ(1,(b/k)^(1/2)*R_bale)*(b/k)^(1/2)*C_2
"symbolic expressions cut and paste from Maple"
T_Rbale = BesselJ(0,(b/k)^(1/2)*R_bale)*C_2-1/b*a
-k*2*pi*R_bale*L*dTdr_Rbale=(T_Rbale-T_infinity)/(R_p+R_conv) "interface energy balance"

The maximum temperature is the temperature at the center of the bale; this is evaluated using
Maple:

> T_max=eval(rhs(Ts),r=0);
a
T_max = _C2 −
b

and copied and pasted into EES:

{T_max=C_2 "maximum bale temperature"}


T_max = C_2-1/b*a
"symbolic expression cut and paste from Maple"
T_max_F=converttemp(K,F,T_max) "maximum bale temperature in F"

e.) Overlay on your plot from part (b) a plot of the maximum temperature in the hay bale as a
function of bale radius when the volumetric generation is a function of temperature.

The result is shown in Figure 1.


Problem 1.3-9 (1-9 in text): Mass Flow Meter
Figure P1.3-9 illustrates a simple mass flow meter for use in an industrial refinery.

T∞ = 20°C insulation
hout = 20 W/m -K
2 kins = 1.5 W/m-K

test section
rout = 1 inch
g ′′′ = 1x10 W/m
7 3

rin = 0.75 inch k = 10 W/m-K

m = 0.75kg/s
T f = 18°C

L = 3 inch thins = 0.25


Figure P1.3-9: A simple mass flow meter.

A flow of liquid passes through a test section consisting of an L = 3 inch section of pipe with
inner and outer radii, rin = 0.75 inch and rout = 1.0 inch, respectively. The test section is
uniformly heated by electrical dissipation at a rate g ′′′ = 1x107 W/m3 and has conductivity k = 10
W/m-K. The pipe is surrounded with insulation that is thins = 0.25 inch thick and has
conductivity kins = 1.5 W/m-K. The external surface of the insulation experiences convection
with air at T∞ = 20°C. The heat transfer coefficient on the external surface is hout = 20 W/m2-K.
A thermocouple is embedded at the center of the pipe wall. By measuring the temperature of the
thermocouple, it is possible to infer the mass flow rate of fluid because the heat transfer
coefficient on the inner surface of the pipe ( hin ) is strongly related to mass flow rate ( m ).
Testing has shown that the heat transfer coefficient and mass flow rate are related according to:

0.8
⎛ m ⎞
hin = C ⎜⎜ ⎟⎟
⎝ 1[ kg/s ] ⎠

where C= 2500 W/m2-K. Under nominal conditions, the mass flow rate through the meter is m
= 0.75 kg/s and the fluid temperature is Tf = 18°C. Assume that the ends of the test section are
insulated so that the problem is 1-D. Neglect radiation and assume that the problem is steady-
state.
a.) Develop an analytical model in EES that can predict the temperature distribution in the test
section. Plot the temperature as a function of radial position for the nominal conditions.

The inputs are entered in EES:

$UnitSystem SI MASS RAD PA K J


$TABSTOPS 0.2 0.4 0.6 0.8 3.5 in

"Inputs"
r_out=1.0 [inch]*convert(inch,m) "outer radius of measurement section"
r_in=0.75 [inch]*convert(inch,m) "inner radius of measurement section"
h_bar_out=20 [W/m^2-K] "external convection coefficient"
T_infinity=converttemp(C,K,20 [C]) "ambient temperature"
T_f=converttemp(C,K, 18 [C]) "fluid temperature"
k=10 [W/m-K] "conductivity"
g```=1e7 [W/m^3] "volumetric rate of thermal energy generation"
m_dot=0.75 [kg/s] "mass flow rate"
th_ins=0.25 [inch]*convert(inch,m) "thickness of insulation"
k_ins=1.5 [W/m-K] "insulation conductivity"
L= 3 [inch]*convert(inch,m) "length of test section"

The heat transfer coefficient on the internal surface is computed according to the specified mass
flow rate:

C=2500 [W/m^2-K] "constant for convection relationship"


h_bar_in=C*(m_dot/1 [kg/s])^0.8 "internal convection coefficient"

The general solution to a 1-D problem in cylindrical coordinates with constant volumetric
thermal energy generation was provided in Table 1-3, to within the unknown constants C1 and
C2:

g ′′′ r 2
T =− + C1 ln ( r ) + C2 (1)
4k

dT g ′′′ r C1
=− + (2)
dr 2k r

The boundary condition at the outer edge of the test section is:

⎛ dT ⎞
− k 2 π rout L ⎜ =
(
Tr = rout − T∞ )
⎟ (3)
⎝ dr ⎠ r = rout ( Rins + Rconv ,out )

where Rins is the thermal resistance to conduction through the insulation (provided in Table 1-2):

⎡ ( r + thins ) ⎤
ln ⎢ out ⎥
rout
Rins = ⎣ ⎦ (4)
2 π L kins

and Rconv,out is the resistance to convection from the outer surface of the insulation:

1
Rconv ,out = (5)
2 π ( rout + thins ) L hout

R_ins=ln((r_out+th_ins)/r_out)/(2*pi*L*k_ins) "resistance to conduction through insulation"


R_conv_out=1/(2*pi*(r_out+th_ins)*L*h_bar_out) "resistance to convection from outer surface"
T_r_out=-g```*r_out^2/(4*k)+C_1*ln(r_out)+C_2 "temperature at outer surface of section"
dTdr_r_out=-g```*r_out/(2*k)+C_1/r_out "temperature gradient at outer surface of section"
-k*2*pi*r_out*L*dTdr_r_out=(T_r_out-T_infinity)/(R_ins+R_conv_out)
"boundary condition at r=r_out"

The boundary condition at the inner edge of the test section is:

⎛ dT ⎞
(
hin 2 π rin L T f − Tr = rin = −k 2 π rin L ⎜) ⎟
⎝ dr ⎠ r = rin
(6)

T_r_in=-g```*r_in^2/(4*k)+C_1*ln(r_in)+C_2 "temperature at inner surface of section"


dTdr_r_in=-g```*r_in/(2*k)+C_1/r_in "temperature gradient at inner surface of section"
h_bar_in*2*pi*r_in*L*(T_f-T_r_in)=-k*2*pi*r_in*L*dTdr_r_in
"boundary condition at r=r_in"

The EES code will provide the solution to the constants C1 and C2; note that it is not possible to
eliminate the unit warnings that are associated with the argument of the natural logarithm in Eq.
(1). In fact, if sufficient algebra was carried out, the equations could be placed in a form where
the natural logarithm had a dimensionless argument.

The location at which to evaluate the temperature (r) is specified in terms of a dimensionless
radial position ( r ) that goes from 0 at the inner surface of the test section to 1 at the outer
surface. The temperature is evaluated using Eq. (1):

r_bar=0.5 [-] "dimensionless radial position"


r=r_in+r_bar*(r_out-r_in) "radial position"
T=-g```*r^2/(4*k)+C_1*ln(r)+C_2 "temperature"
T_C=converttemp(K,C,T) "in C"

Figure P1.3-9-2 illustrates the temperature as a function of radial position.


80

75
Temperature (°C)

70

65

60

55

50
0.019 0.02 0.021 0.022 0.023 0.024 0.025 0.026
Radius (m)
Figure 1.3-9-2: Temperature as a function of radius.

b.) Using your model, develop a calibration curve for the meter; that is, prepare a plot of the
mass flow rate as a function of the measured temperature at the mid-point of the pipe. The
range of the instrument is 0.2 kg/s to 2.0 kg/s.
The dimensionless radial position is set to r =0.5, corresponding to the temperature of the center
of the test section. Figure 1.3-9-3 illustrates the mass flow rate through the meter as a function
of the measured temperature.
2
1.8
1.6

Mass flow rate (kg/s)


1.4
1.2
Tf = 28°C
1 Tf = 18°C
0.8 Tf = 8°C
0.6
0.4
0.2
0
40 60 80 100 120 140 160
Temperature (°C)
Figure 1.3-9-3: Mass flow rate as a function of the temperature at the center of the pipe wall for several
values of the fluid temperature.

The meter must be robust to changes in the fluid temperature. That is, the calibration curve
developed in (b) must continue to be valid even as the fluid temperature changes by as much as
10°C.
c.) Overlay on your plot from (b) the mass flow rate as a function of the measured temperature
for Tf = 8°C and Tf = 28°C. Is your meter robust to changes in Tf?

The calibration curves generated at Tf = 8°C and Tf = 28°C are also shown in Figure 1.3-9-3.
Notice that the fluid temperature has a large effect on the device. For example, if the measured
temperature is 80°C then the mass flow rate could be anywhere from 0.45 kg/s to 0.75 kg/s
depending on the fluid temperature. The meter is not robust to changes in Tf.

In order to improve the meters ability to operate over a range of fluid temperature, a temperature
sensor is installed in the fluid in order to measure Tf during operation.
d.) Using your model, develop a calibration curve for the meter in terms of the mass flow rate as
a function of ΔT, the difference between the measured temperatures at the mid-point of the
pipe wall and the fluid.

The temperature difference is calculated according to:

ΔT = Tr =0.5 − T f (7)

DT=T-T_f "measured temperature difference"

Figure 1.3-9-4 illustrates the mass flow rate as a function of the temperature difference:
2
1.8 Tf = 28°C

1.6 Tf = 18°C

Mass flow rate (kg/s)


Tf = 8°C
1.4
1.2
1
0.8
0.6
0.4
0.2
0
30 40 50 60 70 80 90 100 110 120
Temperature difference (K)
Figure 1.3-9-4: Mass flow rate as a function of the temperature difference between the measured temperature
at the center of the pipe wall and the fluid temperature for several values of the fluid temperature.

e.) Overlay on your plot from (d) the mass flow rate as a function of the difference between the
measured temperatures at the mid-point of the pipe wall and the fluid if the fluid temperature
is Tf = 8°C and Tf = 28°C. Is the meter robust to changes in Tf?

The calibration curves for Tf = 8°C and Tf = 28°C are also shown in Figure 1.3-9-4; notice that
the fluid temperature has almost no effect on the calibration curves and so the meter is robust to
changes in the fluid temperature.

f.) If you can measure the temperature difference to within δΔT = 1 K then what is the
uncertainty in the mass flow rate measurement? (Use your plot from part (d) to answer this
question.)

The uncertainty in the measured mass flow rate that corresponds to an uncertainty in the
temperature difference is evaluated according to:

⎛ ∂m ⎞
δ m = ⎜ ⎟ δΔT (8)
⎝ ∂ΔT ⎠

From Figure 1.3-9-4 we see that the partial derivative of mass flow rate with respect to
temperature difference decreases with flow rate. At high flow rates (around 2 kg/s), the partial
derivative is approximately 0.08 kg/s-K which leads to an uncertainty of 0.08 kg/s. At low flow
rates (around 0.2 kg/s), the partial derivative is approximately 0.04 kg/s-K which leads to an
uncertainty of 0.04 kg/s.

You can use the built-in uncertainty propagation feature in EES to assess uncertainty
automatically.

g.) Set the temperature difference to the value you calculated at the nominal conditions and
allow EES to calculate the associated mass flow rate. Now, select Uncertainty Propagation
from the Calculate menu and specify that the mass flow rate is the calculated variable while
the temperature difference is the measured variable. Set the uncertainty in the temperature
difference to 1 K and verify that EES obtains an answer that is approximately consistent with
part (f).
The temperature difference is set to 50 K corresponding to approximately the middle of the range
of the device. The mass flow rate is commented out and EES is used to calculate the mass flow
rate from the temperature difference:

DT=50 [K]
{m_dot=0.75 [kg/s]} "mass flow rate"

Select Uncertainty Propagation from the Calculate menu (Figure P1.3-9-5) and select the
variable m_dot as the calculated variable and the variable DT as the measured variable.

Figure P1.3-9-5: Determine Propagation of Uncertainty dialog.

Select Set uncertainties and indicate that the uncertainty of the measured temperature difference
is 1 K (Figure P1.3-9-6).

Figure P1.3-9-6: Uncertainties of Measured Variables dialog.

Select OK and then then OK again to carry out the calculation. The results are displayed in the
Uncertainty Results tab of the Solution window (Figure P1.3-9-7).

Figure P1.3-9-7: Uncertainties Results tab of the Solution window.


The uncertainty calculated by EES is δ m = 0.031 kg/s, which falls between the bounds
identified in part (e).

h.) The nice thing about using EES to determine the uncertainty is that it becomes easy to assess
the impact of multiple sources of uncertainty. In addition to the uncertainty δΔT, the
constant C has an uncertainty of δC = 5% and the conductivity of the material is only known
to within δk = 3%. Use EES' built-in uncertainty propagation to assess the resulting
uncertainty in the mass flow rate measurement. Which source of uncertainty is the most
important?

Select Uncertainty Propagation from the Calculate menu and select the variable m_dot as the
calculated variable and the variables DT, C, and k as the measured variables. Set the uncertainty
of each of the measured variables according to the problem statement (Figure P1.3-9-8).

Figure P1.3-9-8: Uncertainties of Measured Variables dialog.

The results of the uncertainty calculation are shown in Figure P1.3-9-9.

Figure P1.3-9-9: Uncertainties Results tab of the Solution window.

Notice that the uncertainty has increased to δ m = 0.062 kg/s and that the dominant source of the
uncertainty is related to C. The effect of the uncertainty in the conductivity is small (only 5.8%
of the total).

i.) The meter must be used in areas where the ambient temperature and heat transfer coefficient
may vary substantially. Prepare a plot showing the mass flow rate predicted by your model
for ΔT = 50 K as a function of T∞ for various values of hout . If the operating range of your
meter must include -5°C < T∞ < 35°C then use your plot to determine the range of hout that
can be tolerated without substantial loss of accuracy.

Figure P1.3-9-10 illustrates the mass flow rate as a function of T∞ for various values of hout .

0.9
2
h = 5 W/m -K
2
0.85 10 W/m -K
2
20 W/m -K
2
50 W/m -K

Mass flow rate (kg/s)


0.8
2
100 W/m -K
0.75

0.7

0.65

0.6
-10 -5 0 5 10 15 20 25 30 35 40 45 50 55
Air temperature (°C)
Figure P1.3-9-10: Mass flow rate predicted with ΔT = 50 K as a function of ambient temperature for various
values of the air heat transfer coefficient.

The shaded region in Figure P1.3-9-10 indicates the operating temperature range (in the x-
direction) and the region of acceptable accuracy (based approximately on the results of part (e)).
Figure P1.3-9-10 shows that 5 W/m2-K < hout < 50 W/m2-K will keep you within the shaded
region and therefore this is, approximately, the range of hout that can be tolerated without
substantial loss of accuracy.
Problem 1.3-10
A current of 100 amps passes through a bare stainless-steel wire of D = 1.0 mm diameter. The
thermal conductivity and electrical resistance per unit length of the wire are k = 15 W/m-K and
Re′ = 0.14 ohm/m, respectively. The wire is submerged in an oil that is maintained at T∞ = 30°C.
The steady-state temperature at the center of the wire is measured to be 180°C, independent of
axial position within the oil bath.
a) What is the temperature at the outer surface of the wire?

The inputs are entered in EES:

$UnitSystem SI MASS RAD PA K J


$TABSTOPS 0.2 0.4 0.6 0.8 3.5 in

k=15 [W/m-K] "conductivity of wire"


R\L=0.14 [ohm/m] "resistance per unit length of wire"
i=100 [amp] "current"
D=1[mm]/2*convert(mm,m) "diameter"
T_c=converttemp(C,K,180 [C]) "temperature at center of wire"
T_oil=converttemp(C,K,30 [C]) "temperature of oil"
L=1 [m] "per unit length basis"

The rate of generation per unit volume is:

I 2 Re′ L
g ′′′ = (1)
L π rout
2

where rout is the outer radius of the wire:

D
rout = (2)
2

r_out=D/2 "outer radius"


g_dot```*(pi*r_out^2*L)=i^2*R\L*L "rate of volumetric generation"

The general solution for radial conduction with uniform volumetric generation is provided in
Table 1-3:

g ′′′ r 2
T =− + C1 ln ( r ) + C2 (3)
4k

dT g ′′′ r C1
=− + (4)
dr 2k r

In order for the temperature to remain bounded at r = 0, it is necessary that C1 be zero. The
temperature at the center is therefore:
Tc = C2 (5)

T_c=C_2 "solve for C_2"

The temperature at the surface is given by:

g ′′′ rout
2
Ts = − + C1 ln ( rout ) + C2 (6)
4k

T_s=-g_dot```*r_out^2/(4*k)+C_2 "surface temperature"


T_s_C=converttemp(K,C,T_s) "in C"

which leads to Ts = 172.6ºC.

b) Estimate the convection coefficient between the submerged wire and the oil.

An interface energy balance at r = rout leads to:

dT
h (Toil − Ts ) = k (7)
dr r = rout

where the temperature gradient is evaluated using Eq. (4):

dT g ′′′ rout
=− (8)
dr r = rout 2k

dT\dr_s=-g_dot```*r_out/(2*k) "surface temperature gradient"


h_bar*(T_oil-T_s)=k*dT\dr_s "interface energy balance"

which leads to h = 6250 W/m2-K.

c) A plastic material (kp = 0.05 W/m-K) can be applied to the outer surface of the wire. Can the
insulation result in a reduction of the center temperature? If so, what insulation thickness
should be applied?

The resistance to conduction through the insulation is:

⎛ r + thins ⎞
ln ⎜ out ⎟
⎝ rout ⎠
Rins = (9)
2π k p L

and the resistance to convection from the surface of the insulation is:
1
Rconv = (10)
2 π rout L h

The total resistance between the surface of the wire and the oil is:

Rtotal = Rconv + Rins (11)

k_p=0.05 [W/m-K] "conductivity of plastic"


R_ins=ln((r_out+th_ins)/r_out)/(2*pi*k_p*L) "resistance to conduction through plastic"
R_conv=1/(h_bar*2*pi*L*(r_out+th_ins)) "resistance to convection from external surface"
R_total=R_ins+R_conv "total resistance from surface of wire"

Figure 1 illustrates Rins, Rconv, and Rtotal as a function of the insulation thickness. Notice that
increasing the insulation thickness reduces Rconv because there is more surface area for
convection but increases Rcond because the length for conduction is longer. In some situations,
the reduction in Rconv dominates the problem and therefore the total resistance may be reduced by
adding insulation. However, in this case, there is no such region.

10
Thermal resistance (K/W)

1 total resistance

convection resistance
0.1

0.01
resistance to conduction through insulation

0.001
0.000001 0.00001 0.0001 0.001 0.01
Insulation thickness (m)
Figure 5: Resistance to convection, resistance to conduction through insulation, and total resistance from wire
surface as a function of the insulation thickness.
Problem 1.3-11: Nuclear Fuel Element
Figure P1.3-11 illustrates a spherical, nuclear fuel element which consists of a sphere of
fissionable material (fuel) with radius rfuel = 5 cm and kfuel = 2 W/m-K that is surrounded by a
spherical shell of metal cladding with outer radius rclad = 7 cm and kclad = 0.25 W/m-K. The
outer surface of the cladding is exposed to fluid that is being heated by the reactor. The
convection coefficient between the fluid and the cladding surface is h = 50 W/m2-K and the
temperature of the fluid is T∞ = 500ºC. Neglect radiation heat transfer from the surface.

Inside the fuel element, thermal energy is being generated for the reactor. This process can be
modeled as a volumetric source of heat generation in the material that is not uniform throughout
the fuel. The volumetric generation ( g ′′′ ) can be approximated by the function:
β
g ′′′ =
r
where β = 5x103 W/m2.

fissionable material
kfuel = 2 W/m-K
rfuel = 5 cm

rclad = 7 cm

g ′′′ h = 50 W/m -K
2

T∞ = 500°C

cladding
kclad = 0.25 W/m-K
Figure P1.3-11: Spherical fuel element surrounded by cladding

a.) Determine an analytical solution for the temperature distribution within the fuel element.
Implement your solution in EES and plot the temperature as a function of radius for 0 < r <
rfuel.

The inputs are entered according to:

$UnitSystem SI MASS RAD PA K J


$Tabstops 0.2 0.4 0.6 3.5 in

"Inputs"
r_fuel=5 [cm]*convert(cm,m) "radius of fuel element"
k_fuel=2 [W/m-K] "conductivity of fuel element"
r_clad=7 [cm]*convert(cm,m) "radius of cladding"
k_clad=0.25 [W/m-K] "conductivity of cladding"
h_bar=50 [W/m^2-K] "heat transfer coefficient"
T_infinity=converttemp(C,K,500[C]) "temperature of fluid"
beta=5e3 [W/m^2] "constant for volumetric generation"
A differential control volume is shown in Figure 2 and includes conduction at r and r+dr at the
inner and outer surfaces of the spherical shell as well as generation within the enclosed volume.

Figure 2: Differential control volume

The energy balance suggested by Figure 2 is:

qr + g = qr + dr (1)

The term at r + dr can be expanded:

dqr
qr + dr = qr + dr (2)
dr

and substituted into Eq. (1):

dqr
qr + g = qr + dr (3)
dr

and simplified:

dqr
g = dr (4)
dr

The conduction is governed by Fourier’s Law:

dT
qr = −k fuel 4 π r 2 (5)
dr

and the generation is the product of the volume and the local generation rate:

β
g = 4 π r 2 dr g ′′′= 4 π r 2 dr (6)
r

The rate equations, Eqs. (5) and (6), are substituted into Eq. (4):
d ⎡ dT ⎤
4 π r dr β = ⎢ − k fuel 4 π r 2 dr (7)
dr ⎣ dr ⎥⎦

which can be simplified:

d ⎡ 2 dT ⎤ rβ
r = − (8)
dr ⎢⎣ dr ⎥⎦ k fuel

Equation (8) can be separated and integrated:

⎡ dT ⎤ rβ
∫ d ⎢⎣ r = ∫− dr
2
⎥ (9)
dr ⎦ k fuel

which leads to:

dT r2 β
r2 =− + C1 (10)
dr 2 k fuel

where C1 is a constant of integration. Equation (10) can be separated and integrated:

⎛ βo C1 ⎞
∫ dT = ∫ ⎜⎜ − 2 k + ⎟ dr
r 2 ⎟⎠
(11)
⎝ fuel

which leads to:

β C1
T =− r− + C2 (12)
2 k fuel r

where C2 is the second constant of integration. The boundary condition at r = 0 requires that the
temperature remain finite and therefore C1 = 0.

βo
T =− r + C2 (13)
2 k fuel

The boundary condition at r = rfuel is obtained using an interface balance, as show in Figure 3.
The interface energy balance includes conduction from the fuel and heat transfer into the
cladding.
Figure 3: Interface balance at r = rfuel

The energy balance suggested by Figure 3 is:

qr = rruel = qout (14)

The conduction term on the left side of Eq. (14) is evaluated using Fourier’s law:

dT
qr = rruel = − k fuel 4 π rfuel
2
(15)
dr r = r fuel

while the heat transfer out of the cladding is driven by the difference between the temperature at
interface between the fuel and the cladding and the temperature of the surrounding gas. The heat
transfer is resisted by the sum of the conduction resistance of the cladding (Rcond,clad):

1 ⎡ 1 1 ⎤
Rcond ,clad = ⎢ − ⎥ (16)
4 π kclad ⎢⎣ rfuel rclad ⎥⎦

and the convection resistance (Rconv):

1
Rconv = (17)
4 π rclad
2
h

so that:

Tr = rfuel − T∞
qout = (18)
Rcond ,clad + Rconv

Subsituting Eqs. (18) and (15) into Eq. (14) leads to:
− k fuel 4 π rfuel
2 dT
=
(T r = r fuel − T∞ ) (19)
dr r = r fuel Rcond ,clad + Rconv

Equation (19) provides a single equation for the unknown constant of integration, C2.
Substituting Eq. (13) into Eq. (19) leads to:

⎛ β ⎞
⎜⎜ − rfuel + C2 − T∞ ⎟

⎛ β ⎞ ⎝ 2 k fuel ⎠
− k fuel 4 π rfuel
2
⎜⎜ − ⎟⎟ = (20)
⎝ 2 k fuel ⎠ Rcond , clad + Rconv

The resistances are computed according to Eqs. (16) and (17) and the constant C2 is computed
according to Eq. (20).

R_cond_clad=(1/r_fuel-1/r_clad)/(4*pi*k_clad) "resistance to conduction through cladding"


R_conv=1/(4*pi*r_clad^2*h_bar) "resistance to convection from cladding"
k_fuel*4*pi*r_fuel^2*(beta/(2*k_fuel))=(-beta*r_fuel/(2*k_fuel)+C_2-T_infinity)/(R_cond_clad+R_conv)
"boundary condition"

The temperature distribution is obtained using Eq. (13).

r=0 [m] "radius"


T=-beta*r/(2*k_fuel)+C_2 "temperature distribution"
T_C=converttemp(K,C,T) "in C"

Figure 4 illustrates the temperature in the sphere as a function of position.


740

730

720
Temperature (°C)

710

700

690

680

670

660
0 0.01 0.02 0.03 0.04 0.05
Radius (m)
Figure 4: Temperature as a function of radius.

b.) The maximum allowable temperature in the fuel element is Tmax = 1100ºC. What is the
maximum value of β that can be used? What is the associated total rate that heat is
transferred to the gas?
The maximum temperature occurs at r = 0. According to Eq. (13), the temperature at r = 0 is:

Tr =0 = C2 (21)

The guess values are updated and the specified value of β is commented out. The temperature at
the center is specified to be Tmax:

{beta=5e3 [W/m^2]} "constant for volumetric generation"


T_max_s=converttemp(C,K,1100 [C]) "maximum allowable temperature"
T_max=C_2 "maximum temperature in fuel"
T_max_s=T_max "adjust beta so that center temperature is equal to T_max_s"

which leads to β = 1.3x104 W/m2. The rate of heat transfer is given by Eq. (18):

βo
− rfuel + C2 − T∞
Tr = rfuel − T∞ 2 k fuel
qout = = (22)
Rcond ,clad + Rconv Rcond ,clad + Rconv

q_dot=(-beta*r_fuel/(2*k_fuel)+C_2-T_infinity)/(R_cond_clad+R_conv) "heat transfer from fuel"

which leads to qout = 204.1 W.

c.) You are designing the fuel elements. You can vary rfuel and β. The cladding must always be
2 cm thick (that is rclad = rfuel + 2 cm). The constraint is that the fuel temperature cannot
exceed Tmax = 1100ºC and the design target (the figure of merit to be maximized) is the rate
of heat transfer per unit volume of material (fuel and cladding). What values of rfuel and β are
optimal?

The volume of the fuel and the cladding is:

4 3
V = π rclad (23)
3

and therefore the heat transfer per unit volume can be determined.

V=4*pi*r_clad^3/3 "volume of fuel"


q_dot\V=q_dot/V "heat transfer per volume"

The cladding radius is specified based on the fuel radius:

r_clad=r_fuel+2 [cm]*convert(cm,m) "radius of cladding"


{r_clad=7 [cm]*convert(cm,m) "radius of cladding"}

The fuel radius is varied in a parametric table and the heat transfer per unit volume as a function
of fuel radius is shown in Figure 5.
2.3x10 5

Heat transfer per unit volume (W/m )


3
2.0x10 5

1.8x10 5

1.5x10 5

1.2x10 5

1.0x10 5

7.5x10 4

5.0x10 4

2.5x10 4
0 0.02 0.04 0.06 0.08 0.1
Fuel element radius (m)
Figure 5: Heat transfer per unit volume as a function of fuel element radius.

Figure 5 shows that the optimal value of rfuel is approximately 1.6 cm. A more exact value can
be obtained using the Min/Max feature from the Calculate menu. The optimal design is rfuel =
1.57 cm with β = 2.6x104 W/m2.
Problem 1.3-12
Figure P1.3-12 illustrates a plane wall. The temperature distribution in the wall is 1-D and the
problem is steady state.

qL′′ h , T∞
x
L

g ′′′ = a x, k
Figure P1.3-12: Plane wall.

There is generation of thermal energy in the wall. The generation per unit volume is not uniform
but rather depends on position according to:

g ′′′ = a x (1)

where a is a constant and x is position. The left side of the wall experiences a specified heat flux,
q ′′L . The right side of the wall experiences convection with heat transfer coefficient h to fluid at
temperature T∞. The thickness of the wall is L and the conductivity of the wall material, k, is
constant.
a.) Derive the ordinary differential equation that governs this problem. Clearly show your steps.

A differential control volume is shown in Figure 2 and leads to:

q x + g = q x + dx (2)

q x q x + dx
g
x

dx

Figure 2: Differential control volume with energy terms.

After expanding the x + dx term:


dq
q x + g = q x + dx (3)
dx

The rate of thermal energy generation within the control volume is:

g = g ′′′ Ac dx (4)

where Ac is the cross-sectional area of the wall. The conduction term is expressed using
Fourier’s law:

dT
q = − k Ac (5)
dx

Substituting Eqs. (5) and (4) into Eq. (3) results in

d ⎛ dT ⎞
g ′′′ Ac dx = ⎜ − k Ac ⎟ dx (6)
dx ⎝ dx ⎠

which can be simplified:

d ⎛ dT ⎞ g ′′′
⎜ ⎟=− (7)
dx ⎝ dx ⎠ k

Substituting the position dependent generation into Eq. (7) leads to:

d ⎛ dT ⎞ ax
⎜ ⎟=− (8)
dx ⎝ dx ⎠ k

b.) Solve the differential equation that you obtained in (a). Your solution should include two
undetermined constants.

Equation (7) is separated and integrated:

⎛ dT ⎞ ax
∫ d ⎜⎝ dx ⎟⎠ = ∫ − k
dx (9)

which leads to:

dT a 2
=− x + C1 (10)
dx 2k

where C1 is a constant of integration. Equation (10) is integrated again:


B B
⎛ a ⎞
∫ dT = ∫ ⎜⎝ − 2 k x + C1 ⎟ dx
2
(11)

which leads to:

a 3
T =− x + C1 x + C2 (12)
6k

c.) Specify the boundary conditions for the differential equation that you derived in (a).

An interface energy balance at x = 0 leads to:

dT
q ′′L = − k (13)
dx x =0

An interface energy balance at x = L leads to:

dT
−k = h (Tx = L − T∞ ) (14)
dx x= L

d.) Use the results of (b) and (c) to obtain two equations that can be solved for the two
undetermined constants.

Substituting Eq. (10) into Eq. (13) leads to:

q L′′ = − k C1 (15)

Substituting Eqs. (10) and (12) into Eq. (14) leads to:

⎛ a 2 ⎞ ⎛ a 3 ⎞
−k ⎜ − L + C1 ⎟ = h ⎜ − L + C1 L + C2 − T∞ ⎟ (16)
⎝ 2k ⎠ ⎝ 6k ⎠

Equations (15) and (16) can be solved for C1 and C2.


Problem 1.4-1: A 3-Node Numerical Solution
Figure P1.4.1(a) illustrates a plane wall with thickness L and cross-sectional area A that has a
specified temperature TH on the left side (at x = 0) and a specified temperature TC on the right
side (at x = L). There is no volumetric generation in the wall. However, the conductivity of the
wall material is a function of temperature such that: k = b + cT where a and b are constants.
You would like to model the wall using a finite difference solution; a model with only 3 nodes is
shown in Figure P1.4-1(b).

TH TC

T1 T2 T3
L
Δx

k = b+cT
(a) (b)
Figure P1.4-1: (a) A plane wall and (b) a numerical model with 3 nodes.

The distance between adjacent nodes for the 3 node solution is: Δx = L/2.
a.) Write down the system of equations that could be solved in order to obtain the temperatures
at the three nodes. Your equations should include the temperature of the nodes (T1, T2, and
T3) and the other parameters listed in the problem statement: TH, TC, Δx, A, b, and c.

The equations for T1 and T3 are easy, their temperatures are specified:

T [1] = TH (1)

T [3] = TC (2)

Figure 2 illustrates the control volume for the 2nd node.

Figure 2: Control volume for node 2.


An energy balance for the control volume shown in Fig. 3 leads to:

q RHS [2] + q LHS [2] = 0 (3)

The energy transfer rates must be approximated according to:

q RHS [2] =
(T [3] − T [ 2]) A ⎡b + c ⎛ T [3] + T [ 2] ⎞⎤
⎢ ⎜ ⎟⎥ (4)
Δx ⎣⎢ ⎝ 2 ⎠ ⎦⎥

and

q LHS [2] =
(T [1] − T [ 2]) A ⎡b + c ⎛ T [1] + T [ 2] ⎞ ⎤
⎢ ⎜ ⎟⎥ (5)
Δx ⎢⎣ ⎝ 2 ⎠ ⎥⎦

Notice that the temperature differences agree with the sign convention used in Figure 3 and that
the conductivity is evaluated at the temperature of the interface. Substituting Eqs. (4) and (5)
into Eq. (3) leads to:

(T [3] − T [ 2]) A ⎡b + c ⎛ T [3] + T [ 2] ⎞⎤ + (T [1] − T [ 2]) A ⎡b + c ⎛ T [1] + T [ 2] ⎞ ⎤ = 0


⎢ ⎜ ⎟⎥ ⎢ ⎜ ⎟⎥ (6)
Δx ⎢⎣ ⎝ 2 ⎠ ⎥⎦ Δx ⎢⎣ ⎝ 2 ⎠ ⎥⎦

Equations (1), (2), and (6) together represent a system of three equations in the three unknown
temperatures.
PROBLEM 1.4-2 (1-10 in text): Mass Flow Meter (revisited)
Reconsider the mass flow meter that was investigated in Problem 1.3-9 (1-9 in text). The
conductivity of the material that is used to make the test section is not actually constant as was
assumed in Problem 1-9 but rather depends on temperature according to:

⎡ W ⎤
2 (
T − 300 [ K ])
W
k = 10 + 0.035 ⎢
m-K ⎣ m-K ⎥⎦

a.) Develop a numerical model of the mass flow meter using EES. Plot the temperature as a
function of radial position for the conditions shown in Figure P1.3-9 (Figure P1-9 in the text)
with the temperature-dependent conductivity.

The inputs are entered in EES:

$UnitSystem SI MASS RAD PA K J


$TABSTOPS 0.2 0.4 0.6 0.8 3.5 in

"Inputs"
r_out=1.0 [inch]*convert(inch,m) "outer radius of measurement section"
r_in=0.75 [inch]*convert(inch,m) "inner radius of measurement section"
h_bar_out=10 [W/m^2-K] "external convection coefficient"
T_infinity_C=20 [C] "ambient temperature in C"
T_infinity=converttemp(C,K,T_infinity_C) "ambient temperature"
T_f=converttemp(C,K, 18 [C]) "fluid temperature"
g```=1e7 [W/m^3] "volumetric rate of thermal energy generation"
m_dot=0.75 [kg/s] "mass flow rate"
th_ins=0.25 [inch]*convert(inch,m) "thickness of insulation"
k_ins=1.5 [W/m-K] "insulation conductivity"
L= 3 [inch]*convert(inch,m) "length of test section"
C=2500 [W/m^2-K] "constant for convection relationship"
h_bar_in=C*(m_dot/1 [kg/s])^0.8 "internal convection coefficient"

A function is defined that returns the conductivity of the material:

Function k_t(T)
"This function returns the conductivity of the test section material as a function of temperature"
k_t=10 [W/m-K]+0.035 [W/m-K^2]*(T-300 [K])
end

A uniform distribution of nodes is used, the radial location of each node (ri) is:

( i − 1) r − r
ri = rin + ( ) for i = 1..N (1)
( N − 1) out in
where N is the number of nodes. The radial distance between adjacent nodes (Δr) is:

Δr =
( rout − rin ) (2)
( N − 1)
N=51 [-] "number of nodes"
DELTAr=(r_out-r_in)/(N-1) "distance between adjacent nodes (m)"
"Set up nodes"
duplicate i=1,N "this loop assigns the radial location to each node"
r[i]=r_in+(r_out-r_in)*(i-1)/(N-1)
end

An energy balance is carried out on a control volume surrounding each node. For node 1, placed
at the inner surface (Figure P1.4-2-1):

qconv ,in + qouter + g = 0 (3)

qouter T2
g1
T1
qconv ,in
Figure P1.4-2-1: Control volume around node 1.

The rate equation for convection is:

qconv ,in = hin 2 π rin L (T f − T1 ) (4)

The rate equation for conduction is:

⎛ Δr ⎞ (T2 − T1 )
qouter = kT =(T1 +T2 ) / 2 2 π ⎜ rin + ⎟L (5)
⎝ 2 ⎠ Δr

The rate equation for generation is:

Δr
g = 2 π rin L g ′′′ (6)
2

"Node 1"
q_dot_conv_in=h_bar_in*2*pi*r_in*L*(T_f-T[1]) "convection from fluid"
g_dot[1]=2*pi*r_in*L*DELTAr*g```/2 "generation"
q_dot_outer[1]=k_t((T[1]+T[2])/2)*2*pi*(r[1]+DELTAr/2)*L*(T[2]-T[1])/DELTAr "conduction from node 2"
q_dot_conv_in+q_dot_outer[1]+g_dot[1]=0 "energy balance on node 1"

An energy balance on an internal node is shown in Figure P1.4-2-2:

qinner + qouter + g = 0 (7)


qouter Ti+1
g Ti
qinner Ti-1
Figure P1.4-2-2: Control volume around internal node i.

The rate equations for conduction are:

⎛ Δr ⎞ (Ti +1 − Ti )
qouter = kT =(Ti +Ti+1 ) / 2 2 π ⎜ rin + ⎟L (8)
⎝ 2 ⎠ Δr

⎛ Δr ⎞ ( T − T )
qinner = kT =(Ti +Ti−1 ) / 2 2 π ⎜ rin − ⎟ L i −1 i (9)
⎝ 2 ⎠ Δr

The rate equation for generation is:

g = 2 π ri Δr L g ′′′ (10)

"Internal nodes"
duplicate i=2,(N-1)
q_dot_inner[i]=k_t((T[i]+T[i-1])/2)*2*pi*(r[i]-DELTAr/2)*L*(T[i-1]-T[i])/DELTAr
"conduction from inner node"
q_dot_outer[i]=k_t((T[i]+T[i+1])/2)*2*pi*(r[i]+DELTAr/2)*L*(T[i+1]-T[i])/DELTAr
"conduction from outer node"
g_dot[i]=2*pi*r[i]*L*DELTAr*g``` "generation"
q_dot_inner[i]+q_dot_outer[i]+g_dot[i]=0 "energy balance on node i"
end

An energy balance on node N placed on the outer surface is shown in Figure P1.4-2-3:

qinner + qair + g = 0 (11)

qair
TN
g
qinner TN-1
Figure P1.4-2-3: Control volume around internal node N.

The rate equation for the heat transfer with the air is:

qair =
(T∞ − TN ) (12)
(R ins + Rconv ,out )
where

⎡ ( r + thins ) ⎤
ln ⎢ out ⎥
Rins = ⎣ ⎦
rout
(13)
2 π L kins

1
Rconv ,out = (14)
2 π ( rout + thins ) L hout

The rate equation for conduction is:

⎛ Δr ⎞ (T − T )
qinner = kT =(TN +TN −1 ) / 2 2 π ⎜ rout − ⎟ L N −1 N (15)
⎝ 2 ⎠ Δr

The rate equation for generation is:

Δr
g = 2 π rout L g ′′′ (16)
2

"Node N"
R_ins=ln((r_out+th_ins)/r_out)/(2*pi*L*k_ins) "resistance to conduction through insulation"
R_conv_out=1/(2*pi*(r_out+th_ins)*L*h_bar_out) "resistance to convection from outer surface"
q_dot_air=(T_infinity-T[N])/(R_ins+R_conv_out) "heat transfer from air"
q_dot_inner[N]=k_t((T[N]+T[N-1])/2)*2*pi*(r_out-DELTAr/2)*L*(T[N-1]-T[N])/DELTAr
"conduction from node N-1"
g_dot[N]=2*pi*r_out*L*DELTAr*g```/2 "generation"
q_dot_air+q_dot_inner[N]+g_dot[N]=0 "energy balance on node N"

The solution is converted to degrees Celsius:

duplicate i=1,N
T_C[i]=converttemp(K,C,T[i]) "convert solution to deg. C"
end

The solution is illustrated in Figure P1.4-2-4.


75

72.5

70

Temperature (°C)
67.5

65

62.5

60

57.5

55

52.5
0.019 0.02 0.021 0.022 0.023 0.024 0.025 0.026
Radius (m)
Figure P1.4-2-4: Temperature as a function of radius.

b.) Verify that your numerical solution limits to the analytical solution from Problem 1.3-9 (1-9
in the text) in the limit that the conductivity is constant.

The conductivity function is modified temporarily so that it returns a constant value:

Function k_t(T)
"This function returns the conductivity of the test section material as a function of temperature"
k_t=10 [W/m-K]{+0.035 [W/m-K^2]*(T-300 [K])}
end

The analytical solution from P1.3-9 is programmed and used to compute the analytical solution
at each node:

"Analytical solution from P1.3-9"


k=k_t(300 [K]) "conductivity to use in the solution"
T_r_out=-g```*r_out^2/(4*k)+C_1*ln(r_out)+C_2 "temperature at outer surface of section"
dTdr_r_out=-g```*r_out/(2*k)+C_1/r_out "temperature gradient at outer surface of section"
-k*2*pi*r_out*L*dTdr_r_out=(T_r_out-T_infinity)/(R_ins+R_conv_out) "boundary condition at r=r_out"
T_r_in=-g```*r_in^2/(4*k)+C_1*ln(r_in)+C_2 "temperature at inner surface of section"
dTdr_r_in=-g```*r_in/(2*k)+C_1/r_in "temperature gradient at inner surface of section"
h_bar_in*2*pi*r_in*L*(T_f-T_r_in)=-k*2*pi*r_in*L*dTdr_r_in "boundary condition at r=r_in"

duplicate i=1,N
T_an[i]=-g```*r[i]^2/(4*k)+C_1*ln(r[i])+C_2 "temperature"
T_an_C[i]=converttemp(K,C,T_an[i]) "in C"
end

Figure P1.4-2-5 illustrates the temperature distribution predicted by the numerical and analytical
solutions in the limit that k is constant.
80

75

Temperature (°C)
70

65

60 analytical model
numerical model
55

50
0.019 0.02 0.021 0.022 0.023 0.024 0.025 0.026
Radius (m)
Figure P1.4-2-5: Temperature as a function of radius predicted by the analytical and numerical models in the
limit that k is constant.

c.) What effect does the temperature dependent conductivity have on the calibration curve that
you generated in part (d) of Problem 1.3-9 (1-9)?

The quantity measured by the meter is the difference between the temperature at the center of the
pipe wall (T[26] when 51 nodes are used) and the fluid temperature:

DT=T[26]-T_f "temperature difference"

Figure P1.4-2-6 illustrates the calibration curve (i.e., the relationship between the temperature
difference and the mass flow rate) with and without the temperature dependent conductivity
included.
2
1.8
1.6
Mass flow rate (kg/s)

1.4
1.2 without temperature dependent conductivity
1 with temperature dependent conductivity
0.8
0.6
0.4
0.2
0
30 40 50 60 70 80 90 100 110 120
Temperature difference (K)
Figure P1.4-2-6: Calibration curve generated with and without the temperature dependent conductivity
included.
PROBLEM 1.4-3: Fuel sphere (revisited)
Reconsider Problem 1.3-7 using a numerical model developed in EES.
a.) Plot the temperature as a function of position within the fuel.

The inputs are entered in EES and a function is defined to return the volumetric generation.

$UnitSystem SI MASS RAD PA K J


$TABSTOPS 0.2 0.4 0.6 0.8 3.5 in

function gv(gve,r,r_fuel,b)
" Inputs:
gve - volumetric generation at the edge (W/m^3)
r - radius (m)
r_fuel - radius of fuel element (m)
b - exponent (-)

Outputs:
gv - volumetric rate of thermal energy generation (W/m^3)"

gv=gve*(r/r_fuel)^b

end

"Inputs"
r_fuel=5[cm]*convert(cm,m) "radius of fuel"
k_fuel=1 [W/m-K] "conductivity of fuel"
r_clad=7[cm]*convert(cm,m) "cladding radius"
k_clad=300 [W/m-K] "cladding conductivity"
h_gas=100 [W/m^2-K] "convection coefficient"
T_gas=converttemp(C,K,500) "gas temperature"
gve=5e5 [W/m^3] "generation at the center"
b=1 [-] "decay constant"

The nodal positions are specified:

N=11 [-] "number of nodes"


duplicate i=1,N
r[i]=(i-1)*r_fuel/(N-1) "location of each node"
end
Dr=r_fuel/(N-1) "distance between nodes"

Energy balances on the internal nodes lead to:

"internal nodes"
duplicate i=2,(N-1)
g_dot[i]=4*pi*r[i]^2*Dr*gv(gve,r[i],r_fuel,b) "generation"
q_dot_LHS[i]=4*pi*(r[i]-Dr/2)^2*k_fuel*(T[i-1]-T[i])/Dr "conduction from node i-1"
q_dot_RHS[i]=4*pi*(r[i]+Dr/2)^2*k_fuel*(T[i+1]-T[i])/Dr "conduction from node i+1"
g_dot[i]+q_dot_LHS[i]+q_dot_RHS[i]=0 "energy balance"
end

An energy balance on node N leads to:


"surface node"
Rst_clad=(1/r_fuel-1/r_clad)/(4*pi*k_clad) "conduction resistance of cladding"
Rst_conv=1/(4*pi*r_clad^2*h_gas) "convection resistance"
g_dot[N]=4*pi*r[N]^2*Dr*gv(gve,r[N],r_fuel,b)/2 "generation"
q_dot_LHS[N]=4*pi*(r[N]-Dr/2)^2*k_fuel*(T[N-1]-T[N])/Dr "conduction from node N-1"
q_dot_RHS[N]=(T_gas-T[N])/(Rst_clad+Rst_conv) "heat transfer from gas"
g_dot[N]+q_dot_LHS[N]+q_dot_RHS[N]=0 "energy balance"

An energy balance on node 1 leads to:

"inner node"
g_dot[1]=4*pi*(Dr/2)^3*gv(gve,r[1],r_fuel,b)/2/3 "generation"
q_dot_RHS[1]=4*pi*(r[1]+Dr/2)^2*k_fuel*(T[2]-T[1])/Dr "conduction from node 2"
g_dot[1]+q_dot_RHS[1]=0 "energy balance"

The solution is converted to °C.

duplicate i=1,N
T_C[i]=converttemp(K,C,T[i])
end

The temperature as a function of position is shown in Figure 1.

640

620
Temperature (°C)

600

580

560
numerical solution
analytical solution from P1.3-7
540

520
0 0.01 0.02 0.03 0.04 0.05
Radius (m)
Figure 1: Temperature as a function of radial position, predicted by the numerical model and the analytical
model derived in Problem 1.3-7.

b.) Verify that your answer agrees with the analytical solution obtained in Problem 1.3-7.

The analytical solution from Problem 1.3-7 is evaluated at the same radial locations used in the
numerical model:

"Boundary condition expressions"


dTdr_rfuel =-2*r_fuel*gve/k_fuel/(b^2+6+5*b)-r_fuel*gve/k_fuel/(b^2+6+5*b)*b "from Maple"
T_rfuel =-r_fuel^2*gve/k_fuel/(b^2+6+5*b)+C2 "from Maple"
-4*pi*r_fuel^2*k_fuel*dTdr_rfuel=(T_rfuel-T_gas)/(Rst_clad+Rst_conv)"boundary condition"

"Solution"
duplicate i=1,N
T_an[i]=-r[i]^2*gve/k_fuel/(b^2+6+5*b)*(r[i]/r_fuel)^b+C2 "solution from Maple"
T_an_C[i]=converttemp(K,C,T_an[i])
end

The analytical solution is overlaid onto the numerical result in Figure 1.

c.) Plot some aspect of the solution as a function of the number of nodes used in the numerical
model and determine the number of nodes required for an accurate solution.

Figure 2 illustrates the maximum temperature in the fuel element as a function of the number of
nodes and shows that at least 50 nodes is required to obtain an accurate solution.

640

630
Maximum temperature (°C)

620

610

600

590

580

570

560
1 10 100 500
Number of nodes
Figure 2: Maximum temperature in the fuel element as a function of the number of nodes.
Problem 1.4-4: Storing Hay in a Barn
If you bale hay without allowing it to dry sufficiently then the hay bales will contain a lot of
water. Besides making the bales heavy and therefore difficult to put in the barn, the water in the
hay bails causes an exothermic chemical reaction to occur within the bale (i.e., the hay is
rotting). The chemical reaction proceeds at a rate that is related to temperature and the bales may
be thermally isolated (they are placed in a barn and surrounded by other hay bales); as a result,
the hay can become very hot and even start a barn fire. Figure P1.4-4 illustrates a cross-section
of a barn wall with hay stacked against it.

thw = 1 cm
L=5m

Ta ,in = 20°C Ta ,out = −5°C


ha ,in = 15 W/m -K
2 g ′′′ ha ,out = 45 W/m -K
2

barn wall
kw = 0.11 W/m-K
x hay
kh = 0.05 W/m-K
Figure P1.4-4: Barn wall with hay.

The air within the barn is maintained at Ta,in = 20°C and the heat transfer coefficient between the
air and the inner surface of the hay is ha ,in = 15 W/m2-K. The outside air is at Ta,out = -5°C with
ha ,out = 45 W/m2-K. Neglect radiation from the surfaces in this problem. The barn wall is
composed of wood (kw = 0.11 W/m-K) and is thw = 1 cm thick. The hay has been stacked L = 5
m thick against the wall. Hay is a composite structure composed of plant fiber and air.
However, hay can be modeled as a single material with an effective conductivity kh = 0.05 W/m-
K. The volumetric generation of the hay due to the chemical reaction is given by:

0.5
⎡W ⎤⎡ ⎛ T ⎞⎤
g ′′′ = 1.5 ⎢ 3 ⎥ ⎢exp ⎜⎜ ⎟⎟ ⎥
⎣ m ⎦ ⎣⎢ ⎝ 320 [ K ] ⎠ ⎦⎥

where T is temperature in K.
a.) Develop a numerical model that can predict the temperature distribution within the hay.

The input information is entered in EES and a function is used to define the volumetric
generation:

$UnitSystem SI MASS RAD PA K J


$TABSTOPS 0.2 0.4 0.6 0.8 3.5 in

function gen(T)
"volumetric heat generation in wall"
"Input - T, temperature [K]"
"Output - gen, volumetric rate of heat generation [W/m^3]"
gen=1.5 [W/m^3]*sqrt(exp(T/320 [K]))

end

"Inputs"
T_a_in=converttemp(C,K,20) "temperature of air within barn"
h_a_in=15 [W/m^2-K] "internal heat transfer coefficient"
T_a_out=converttemp(C,K,-5) "temperature of air outside barn"
h_a_out=45 [W/m^2-K] "external heat transfer coefficient"
k_w=0.11 [W/m-K] "conductivity of barn wall"
th_w=1.0 [cm]*convert(cm,m) "barn wall thickness"
L=5.0 [m] "thickness of hay"
k_h=0.05 [W/m-K] "conductivity of hay"
A = 1 [m^2] "per unit area of wall"

Nodes are distributed uniformly throughout the computational domain (which consists only of
the hay, not the barn wall), the location of each node (xi) is:

(i − 1)
xi = L i = 1..N (1)
( N − 1)
where N is the number of nodes used for the simulation. The distance between adjacent nodes
(Δx) is:

L
Δx = (2)
N −1

"Setup grid"
N=10 [-] "number of nodes"
duplicate i=1,N
x[i]=(i-1)*L/(N-1) "position of each node"
end
Deltax=L/(N-1) "distance between adjacent nodes"

A control volume is defined around each node and an energy balance is written for each control
volume. The control volume for an arbitrary, internal node (i.e., a node that is not placed on the
edge of the hay) experiences conduction heat transfer passing through the internal surface ( q LHS ),
conduction heat transfer passing through the external surface ( q RHS ), and heat generation within
the control volume ( g ). A steady-state energy balance for the control volume is shown in Fig. 2
and leads to:

q LHS + q RHS + g = 0 (3)


Figure 2: An internal control volume

Each of the terms in the energy balance in Eq. (3) must be modeled using a rate equation.
Conduction through the inner surface is driven by the temperature difference between nodes i-1
and i through the material that lies between these nodes.

kh A (Ti −1 − Ti )
q LHS = (4)
Δx

where A is the area of the wall (assumed to be 1 m2, corresponding to doing the problem on a per
unit area of wall basis). The conduction into the outer surface is:

kh A (Ti +1 − Ti )
q RHS = (5)
Δx

The generation is the product of the volume of the control volume and the volumetric generation
rate, which is approximately:

g = g T′′′i A Δx (6)

where g ′′′ must be evaluated at the nodal temperature Ti. Substituting Eqs. (4) through (6) into
Eq. (3) leads to:

kh A (Ti −1 − Ti ) kh A (Ti +1 − Ti )
+ + g T′′′i A Δx = 0 for i = 2...( N − 1) (7)
Δx Δx

Figure 3 illustrates the control volume associated with the node that is placed on the outer
surface of the hay (i.e., node N).
Figure 3: Control volume for node N located on hay outer surface

The energy balance for the control volume associated with node N is:

q LHS + g + qout = 0 (8)

where the conduction term is:

kh A (TN −1 − TN )
q LHS = , (9)
Δx

the generation term is:

Δx
g = g T′′′N A , (10)
2

(note the factor of 2 corresponding to half the volume), and the heat transfer to the external air is:

qconv =
(T a , out − TN )
(11)
Rw + Rconv ,out

where

thw
Rw = (12)
kw A

and

1
Rconv ,out = (13)
ha ,out A

Substituting Eqs. (9) through (11) into Eq. (8) leads to:
kh A (TN −1 − TN ) Δx (Ta ,out − TN )
+ g T′′′N A + =0 (14)
Δx 2 Rw + Rconv ,out

A similar procedure applied to the control volume associated with node 1 leads to:

kh A (T2 − T1 ) Δx
+ g T′′′1 A + ha ,in A (Ta ,in − T1 ) = 0 (15)
Δx 2

Equations (7), (14), and (15) represent N equations in an equal number of unknowns; the solution
of these equations provides the numerical solution.

"Internal control volumes"


duplicate i=2,(N-1)
k_h*A*(T[i-1]-T[i])/Deltax+k_h*A*(T[i+1]-T[i])/Deltax+gen(T[i])*A*Deltax=0
end
R_w=th_w/(k_w*A) "conduction resistance of barn wall"
R_conv_out=1/(h_a_out*A) "convection resistance to external air"
k_h*A*(T[N-1]-T[N])/Deltax+gen(T[N])*A*Deltax/2+(T_a_out-T[N])/(R_w+R_conv_out)=0
"Node N"
k_h*A*(T[2]-T[1])/Deltax+gen(T[1])*A*Deltax/2+h_a_in*A*(T_a_in-T[1])=0
"Node 1"

If the EES program is solved then the temperature distribution will be placed in the Arrays
window.

b.) Prepare a plot that shows the temperature distribution as a function of position in the hay.

The information in the Arrays table is used to prepare the plot shown in Figure 4.

Figure 4: Temperature as a function of position in the wall.


c.) Prepare a plot that shows that you are using a sufficient number of nodes in your numerical
solution.

The most relevant result of the calculation is the maximum temperature within the wall.

T_max=MAX(T[1..N]) "Maximum temperature in the wall"

Comment out the number of nodes assignment:

{N=10 [-]}

and prepare a parametric table that contains N and T_max (Figure 5).

Figure 5: Parametric table

The information in the parametric table is used to create Figure 6 which shows the maximum
temperature as a function of the number of nodes; Figure 6 suggests that 50 nodes should be used
to obtain a numerically convergent solution.

Figure 6: Predicted maximum temperature as a function of the number of nodes.


d.) Verify that your solution is correct by comparing it with an analytical solution in an
appropriate limit. Prepare a plot that overlays your numerical solution and the analytical
solution in this limit.

There are a few limits; the easiest one would be to turn the generation off (i.e., set it to zero).
Alternatively, set the generation rate to a constant value (e.g., 1 W/m3) and obtain the analytical
solution. Modify the function:

function gen(T)
"volumetric heat generation in wall"
"Input - T, temperature [K]"
"Output - gen, volumetric rate of heat generation [W/m^3]"

gen=1.0 [W/m^3] {1.5 [W/m^3]*sqrt(exp(T/320 [K]))}

end

The general solution for a plane wall subjected to a constant generation rate was provided in
Table 3-1:

g ′′′ 2
T =− x + C1 x + C2 (16)
2 kh

The boundary condition at x = L is:

− kh A
dT
=
(Tx= L − Ta ,out )
(17)
dx x=L Rw + Rconv ,out

where the temperature gradient can also be obtained from Table 3-1:

dT ⎡ g ′′′ ⎤ g ′′′
= ⎢− x + C1 ⎥ = − L + C1 (18)
dx x= L ⎣ kh ⎦ x=L kh

and

⎡ g ′′′ 2 ⎤ g ′′′ 2
Tx = L = ⎢ − x + C1 x + C2 ⎥ = − L + C1 L + C2 (19)
⎣ 2 k h ⎦ x= L 2 k h

"Analytical solution for constant generation"


g```_dot=gen(300 [K]) "obtain the rate of generation"
-k_h*A*dTdx_L=(T_L-T_a_out)/(R_w+R_conv_out) "boundary condition at x=L"
dTdx_L=-g```_dot*L/k_h+C_1 "temperature gradient at x=L"
T_L=-g```_dot*L^2/(2*k_h)+C_1*L+C_2 "temperature at x=L"

The boundary condition at x = 0 is:


ha ,in A (Ta ,in − Tx =0 ) = −kh A
dT
(20)
dx x =0

where the temperature gradient can also be obtained from Table 3-1:

dT ⎡ g ′′′ ⎤
= ⎢− x + C1 ⎥ = C1 (21)
dx x =0 ⎣ kh ⎦ x =0

and

⎡ g ′′′ 2 ⎤
Tx =0 = ⎢ − x + C1 x + C2 ⎥ = C2 (22)
⎣ 2 kh ⎦ x =0

h_a_in*A*(T_a_in-T_0)=-k_h*A*dTdx_0 "boundary condition at x=0"


dTdx_0=C_1 "temperature gradient at x=0"
T_0=C_2 "temperature at x=0"

Solving the problem shows that C1 = 45.0 K/m and C2 = 293.3 K. The solution at each node is
obtained:

duplicate i=1,N
T_an[i]=-g```_dot*x[i]^2/(2*k_h)+C_1*x[i]+C_2
end

Figure 7 illustrates the analytical and numerical solutions and shows that they agree.

Figure 7: Numerical and analytical solutions in the limit that g ′′′ is constant.
Temperatures above Tfire= 200°F are considered to be a fire hazard and temperatures above Td =
140°F will result in a degradation of the hay to the point where it is not usable.

e.) What is the maximum allowable thickness of hay (Lmax) based on keeping the maximum
temperature below Tfire?

You can either manually adjust L until the variable Tmax is equal to Tfire or simply set Tmax and
comment out the assignment of the variable L and let EES automatically determine the correct
value (note that you need to return the generation function to its original state).

{L=5.0 [m]} "thickness of hay"


T_fire=converttemp(F,K,200) "combustion temperature"
T_max=T_fire

Which leads to Lmax = 3.615 m.

f.) If L = Lmax from (e) then how much of the hay will remain usable (what percent of the hay is
lost to heat degradation)?

Figure 8 illustrates the temperature distribution for L = 3.615 m and shows the extent of the
region of the heat damaged hay.

Figure 8: Temperature distribution for L = 3.615 m.

The region of usable hay extends from 0 to 0.63 m and from 2.73 m to 3.62 m. Therefore, only
43% of the hay will be useable when it is removed from the bar. Note that simple calculations
like this can be done easily using the Calculator function in EES (select Calculator from the
Windows menu). The calculator environment includes all of the variables from the last run of
EES. Therefore, typing ?L returns 3.615 (Figure 9).
Figure 9: Calculator window.

To calculate the efficiency of the storage process, use the Calculator window as shown in Figure
10.

Figure 10: Calculator window.


Problem 1.4-5
Solve the problem stated in EXAMPLE 1.3-2 numerically rather than analytically.
a.) Develop a numerical model that can predict the temperature distribution within the lens.
Prepare a plot of the temperature as a function of position.

The inputs are entered in EES:

"Problem 1.4-5"
$UnitSystem SI MASS RAD PA K J
$TABSTOPS 0.2 0.4 0.6 0.8 3.5 in

"Inputs"
q``_rad=0.1 [W/cm^2]*convert(W/cm^2,W/m^2) "radiation incident on the lens"
L=1.0 [cm]*convert(cm,m) "thickness of lens"
T_a=converttemp(C,K,20) "ambient temperature"
h=20 [W/m^2-K] "heat transfer coefficient"
k=1.5 [W/m-K] "conductivity of lens"
alpha=0.1 [1/mm]*convert(1/mm,1/m) "absorption coefficient"
A=1 [m^2] "per unit area"

Nodes are distributed uniformly throughout the computational domain; the distance between
adjacent nodes is:

L
Δx = (1)
( N − 1)
where N is the number of nodes. The position of each node is:

xi = Δx ( i − 1) for i = 1...N (2)

N=10 [-] "number of nodes"


Dx=L/(N-1) "distance between adjacent nodes"
duplicate i=1,N
x[i]=Dx*(i-1) "position of each node"
end

An energy balance on an internal control volume is shown in Figure 1.

Figure 1: Energy balance on an internal control volume


The energy balance is:

qtop + qbottom + g = 0 (3)

Substituting rate equations into Eq. (3) leads to:

kA kA
(Ti −1 − Ti ) + (Ti +1 − Ti ) + A Δx qrad
′′ α exp ( −α xi ) = 0 (4)
Δx Δx

"internal control volume energy balances"


duplicate i=2,(N-1)
k*A*(T[i-1]-T[i])/Dx+k*A*(T[i+1]-T[i])/Dx+A*Dx*q``_rad*alpha*exp(-alpha*x[i])=0
end

An energy balance on node 1 located at the upper surface is shown in Figure 2.

Figure 2: Energy balance on the upper edge control volume

The energy balance for node 1 is:

qconv + qbottom + g = 0 (5)

Substituting rate equations into Eq. (5) leads to:

kA A Δx
h A (Ta − T1 ) + (T2 − T1 ) + ′′ α exp ( −α x1 ) = 0
qrad (6)
Δx 2

The corresponding energy balance for node N located at the lower surface is:

kA A Δx
h A (Ta − TN ) + (TN −1 − TN ) + ′′ α exp ( −α xN ) = 0
qrad (7)
Δx 2

"upper edge"
h*A*(T_a-T[1])+k*A*(T[2]-T[1])/Dx+A*Dx*q``_rad*alpha*exp(-alpha*x[1])/2=0

"lower edge"
h*A*(T_a-T[N])+k*A*(T[N-1]-T[N])/Dx+A*Dx*q``_rad*alpha*exp(-alpha*x[N])/2=0
The temperature distribution in the lens is shown in Figure 3.

Figure 3: Temperature as a function of position in the lens

b.) Plot some characteristic of your solution as a function of the number of nodes to show that
you are using a sufficient number of nodes.

The maximum temperature in the lens is obtained using the Max command in EES:

T_max=MAX(T[1..N]) "maximum temperature in lens"

The maximum temperature and number of nodes are placed in a parametric table; the number of
nodes is varied and the results are shown in Figure 4.
Figure 4: Maximum temperature as a function of the number of nodes

c.) Think of a sanity check that you can use to gain confidence in your model; that is, can you
change some input parameter and show that the solution behaves as you would expect.
Support your answer with a plot.

As the lens conductivity becomes very large, the temperature rise within the lens should be
reduced. Figure 3 illustrates the predicted result when the conductivity is increased by a factor
of 10, to 15 W/m-K.

d.) Plot the maximum lens temperature as a function of the heat transfer coefficient, h .

Figure 5 illustrates the maximum temperature in the lens as a function of the heat transfer
coefficient.
Figure 5: Maximum temperature as a function of the heat transfer coefficient
Problem 1.4-6
A current lead must be designed to carry current to a cryogenic superconducting magnet, as
shown in Figure P1.4-6.

TH = 300 K
current lead

D = 1 cm
L = 20 cm
Ic = 1000 amp

TC = 100 K
Figure P1.4-6: Current lead.

The current lead carries Ic = 1000 amp and therefore experiences substantial generation of
thermal energy due to ohmic dissipation. The electrical resistivity of the lead material depends
on temperature according to:

⎡ ohm-m ⎤
ρe = 17x10−9 [ ohm-m ] + (T − 300 [ K ]) 5x10−11 ⎢ (1)
⎣ K ⎥⎦

The length of the current lead is L = 20 cm and the diameter is D = 1 cm. The hot end of the
lead (at x = 0) is maintained at Tx=0 = TH = 300 K and the cold end (at x = L) is maintained at Tx=L
= TC = 100 K. The conductivity of the lead material is k = 400 W/m-K. The lead is installed in a
vacuum chamber and therefore you may assume that the external surfaces of the lead (the outer
surface of the cylinder) are adiabatic.
a.) Develop a numerical model in EES that can predict the temperature distribution within the
current lead. Plot the temperature as a function of position.

The input information is entered in EES and a function is used to define the electrical resistivity
according to Eq. (1):

$UnitSystem SI MASS RAD PA K J


$TABSTOPS 0.2 0.4 0.6 0.8 3.5 in

function rho_e(T)
"Input:
T - temperature (K)

Output:
rho_e - electrical resistivity (ohm-m)"
rho_e=17e-9 [ohm-m]+(T-300 [K])*5e-11 [ohm-m/K]
end

"Inputs"
L=20 [cm]*convert(cm,m) "length of current lead"
T_H=300 [K] "hot end temperature"
T_C=100 [K] "cold end temperature"
Ic=1000 [amp] "current"
k=400 [W/m-K] "conductivity of lead"
D_cm=1 [cm] "diameter of lead, in cm"
D=D_cm*convert(cm,m) "diameter of lead"

Nodes are distributed uniformly throughout the computational, the location of each node (xi) is:

(i − 1)
xi = L i = 1..N (2)
( N − 1)
where N is the number of nodes used for the simulation. The distance between adjacent nodes
(Δx) is:

L
Δx = (3)
N −1

N=21 [-] "number of nodes"


duplicate i=1,N
x[i]=L*(i-1)/(N-1) "axial position"
end
Dx=L/(N-1) "distance between adjacent nodes"

A control volume is defined around each node and an energy balance is written for each control
volume. The control volume for an arbitrary, internal node (i.e., a node that is not placed on the
edge of the hay) experiences conduction heat transfer passing through the top surface ( qtop ),
conduction heat transfer passing through the bottom surface ( qbottom ), and heat generation within
the control volume ( g ). A steady-state energy balance for an internal control volume:

q LHS + q RHS + g = 0 (4)

Each of the terms in the energy balance in Eq. (4) must be modeled using a rate equation.
Conduction through the inner surface is driven by the temperature difference between nodes i-1
and i through the material that lies between these nodes.

k Ac (Ti −1 − Ti )
qtop = (5)
Δx

where Ac is the cross-sectional area of the lead:

π D2
Ac = (6)
4

The conduction through the bottom surface is:


k Ac (Ti +1 − Ti )
qbottom = (7)
Δx

The generation is the product of the electrial resistance of the material in the control volume and
the current squared:

Δx 2
g = ρ e,T =Ti Ic (8)
Ac

Substituting Eqs. (5), (7), and (8) into Eq. (4) leads to:

k Ac (Ti −1 − Ti ) k Ac (Ti +1 − Ti ) Δx 2
+ + ρ e,T =Ti Ic = 0 for i = 2...( N − 1) (9)
Δx Δx Ac

A_c=pi*D^2/4 "cross-sectional area of lead"


duplicate i=2,(N-1)
k*A_c*(T[i-1]-T[i])/Dx+k*A_c*(T[i+1]-T[i])/Dx+rho_e(T[i])*Dx*Ic^2/A_c=0
"energy balance on internal nodes"
end

The temperatures of nodes 1 and N are specified:

T1 = TH (10)

TN = TH (11)

T[1]=T_H "hot end temperature"


T[N]=T_C "cold end temperature"

Figure 2 illustrates the temperature as a function of position.


300

280

260

240
Temperature (K)

220

200

180

160

140

120

100
0 0.04 0.08 0.12 0.16 0.2
Position (m)
Figure 2: Temperature as a function of position.
b.) Determine the rate of energy transfer into the superconducting magnet at the cold end of the
current lead. This parasitic must be removed in order to keep the magnet cold and therefore
must be minimized in the design of the current lead.

An energy balance on node N leads to:

k Ac (TN −1 − TN ) Δx 2
qC = + ρe ,T =TN Ic (12)
Δx 2 Ac

q_dot=k*A_c*(T[N-1]-T[N])/Dx+rho_e(T[N])*Dx*Ic^2/(2*A_c) "parasitic to cold end"

which leads to qC = 45.7 W.

c.) Prepare a plot showing the rate of energy transfer into the magnet as a function of the number
of nodes used in your model.

Figure 3 illustrates qC as a function of N.

46
Rate of heat transfer to cold end (W)

45

44

43

42

41

40
2 10 100 200
Number of nodes
Figure 3: Rate of heat transfer to the cold end as a function of the number of nodes.

d.) Plot the rate of heat transfer to the cold end as a function of the diameter of the lead. You
should see a minimum value and therefore an optimal diameter - explain why this occurs.

Figure 4 illustrates the rate of heat transfer to the cold end as a function of the diameter of the
lead. At very low diameters the ohmic dissipation is large because the electrical resistance is
high and therefore the parasitic is large. At very large diameters, the thermal resistance of the
lead is large therefore the parasitic is large. The optimal diameter is around 0.9 cm and balances
these effects.
300

Rate of heat transfer to cold end (W)


250

200

150

100

50

0
0.5 0.75 1 1.25 1.5 1.75 2
Diameter of lead (cm)
Figure 4: Rate of heat transfer to cold end as a function of the diameter of the lead.

e.) Prepare a plot showing the optimal diameter and minimized rate of heat transfer to the cold
end as a function of the current that must be carried by the lead. You may want to use the
Min/Max Table selection from the Calculate menu to accomplish this.

Figure 5 illustrates the minimized rate of heat transfer to the cold end and the optimal diameter
as function of the level of current.
225 2
Rate of heat transfer to the cold end (W)

200
1.8
175
1.6
150
Diameter of lead (cm)
125 D 1.4

100 qC 1.2
75
1
50
0.8
25

0 0.6
500 1000 1500 2000 2500 3000 3500 4000 4500 5000
Current (amp)
Figure 5: Minimized rate of heat transfer and optimal lead diameter as a function of current.
Problem 1.4-7
Figure P1.4-7 illustrates a plane wall. The temperature distribution in the wall is 1-D and the
problem is steady state.

node 1 h , T∞
node 2 node 3
qL′′

x
L
g ′′′ = a x, k
Figure P1.4-7: Three-node model of a plane wall.

There is generation of thermal energy in the wall. The generation per unit volume is not uniform
but rather depends on position according to:

g ′′′ = a x (1)

where a is a constant and x is position. The left side of the wall experiences a specified heat flux,
q ′′L . The right side of the wall experiences convection with heat transfer coefficient h to fluid at
temperature T∞. The thickness of the wall is L and the conductivity of the wall material, k, is
constant. You are going to develop a numerical model with 3 nodes, as shown in Figure P1.4-7.
The nodes are distributed uniformly throughout the domain. Derive the three equations that must
be solved in order to implement the numerical model. Do not solve these equations.

An energy balance on node 1 leads to:

Δx k Ac
q ′′L Ac + g ′′′x =0 Ac + (T2 − T1 ) = 0 (2)
2 Δx

Substituting Eq. (1) into Eq. (2) and dividing through by Ac leads to:

k
q L′′ + (T2 − T1 ) = 0 (3)
Δx

An energy balance on node 2 leads to:

k Ac kA
(T1 − T2 ) + g ′′′x= L / 2 Ac Δx + c (T3 − T2 ) = 0 (4)
Δx Δx

Substituting Eq. (1) into Eq. (4) and dividing through by Ac leads to:
k L k
(T1 − T2 ) + a Δx + (T3 − T2 ) = 0 (5)
Δx 2 Δx

An energy balance on node 3 leads to:

k Ac Δx
(T2 − T3 ) + g ′′′x= L Ac + h Ac (T∞ − T3 ) = 0 (6)
Δx 2

Substituting Eq. (1) into Eq. (6) and dividing through by Ac leads to:

k Δx
(T2 − T3 ) + a L + h (T∞ − T3 ) = 0 (7)
Δx 2

Equations (3), (5), and (7) can be solved to provide T1, T2, and T3.
PROBLEM 1.5-1 (1-11 in text): Hay Temperature (revisited)
Reconsider Problem P1.3-8, but obtain a solution numerically using MATLAB. The description
of the hay bale is provided in Problem P1.3-8. Prepare a model that can consider the effect of
temperature on the volumetric generation. Increasing temperature tends to increase the rate of
reaction and therefore increase the rate of generation of thermal energy; the volumetric rate of
generation can be approximated by: g ′′′ = a + bT where a = -1 W/m3 and b = 0.01 W/m3-K.
Note that at T = 300 K, the generation is 2 W/m3 but that the generation increases with
temperature.
a.) Prepare a numerical model of the hay bale using EES. Plot the temperature as a function
of position within the hay bale.

The input information is entered in EES and a function is used to define the volumetric
generation:

$UnitSystem SI MASS RAD PA K J


$TABSTOPS 0.2 0.4 0.6 0.8 3.5 in

function gen(T)
"volumetric heat generation in wall"
"Input - T, temperature [K]"
"Output - gen, volumetric rate of heat generation [W/m^3]"

a=-1 [W/m^3] "coefficients in generation function"


b=0.01 [W/m^3-K]
gen=a+b*T

end

"Inputs"
L = 1 [m] "per unit length of bale"
R_bale= 5 [ft]*convert(ft,m) "bale radius"
t_p=0.045 [inch]*convert(inch,m) "plastic thickness"
k_p=0.15 [W/m-K] "plastic conductivity"
T_infinity=converttemp(C,K,20) "ambient temperature"
h=10 [W/m^2-K] "heat transfer coefficient"
k=0.04 [W/m-K] "hay conductivity"

Nodes are distributed uniformly throughout the computational domain (which consists only of
the hay, not the plastic), the location of each node (ri) is:

(i − 1)
ri = R i = 1..N (1)
( N − 1) bale
where N is the number of nodes used for the simulation. The distance between adjacent nodes
(Δr) is:

Rbale
Δr = (2)
( N − 1)
"Setup grid"
N=50 [-] "number of nodes"
duplicate i=1,N
r[i]=(i-1)*R_bale/(N-1) "position of each node"
end
Deltar=R_bale/(N-1) "distance between adjacent nodes"

A control volume is defined around each node and an energy balance is written for each control
volume. The control volume for an arbitrary, internal node (i.e., a node that is not placed on the
edge or at the center of the hay) experiences conduction heat transfer passing through the internal
surface ( q LHS ), conduction heat transfer passing through the external surface ( q RHS ), and heat
generation within the control volume ( g ). A steady-state energy balance for the control volume
is shown in Figure 1:

q LHS + q RHS + g = 0 (3)

Figure 1: Internal node energy balance

Each of the terms in the energy balance in Eq. (3) must be modeled using a rate equation.
Conduction through the inner surface is driven by the temperature difference between nodes i-1
and i through the material that lies between these nodes.

⎛ Δr ⎞
k 2 π ⎜ ri − ⎟ L
⎝ 2 ⎠
q LHS = (Ti −1 − Ti ) (4)
Δr

where L is the length of the bale (assumed to be 1 m, corresponding to doing the problem on a
per unit length of bale basis). The conduction into the outer surface of the control volume is:

⎛ Δr ⎞
k 2 π ⎜ ri + ⎟L
⎝ 2 ⎠
q RHS = (Ti +1 − Ti ) (5)
Δr

The generation is the product of the volume of the control volume and the volumetric generation
rate, which is approximately:

⎡⎛ Δr ⎞ ⎛
2
Δr ⎞ ⎤
2

g = g ′′′ (Ti ) π L ⎢⎜ ri + −
⎟ ⎜ ir − ⎟ ⎥ (6)
⎢⎣⎝ 2 ⎠ ⎝ 2 ⎠ ⎥⎦
where g ′′′ (Ti ) is the volumetric rate of generation evaluated at the nodal temperature Ti.
Substituting Eqs. (4) through (6) into Eq. (3) leads to:

⎛ Δr ⎞ ⎛ Δr ⎞
k 2 π ⎜ ri − ⎟ L k 2 π ⎜ ri + ⎟L
⎝ 2 ⎠ ⎝ 2 ⎠ ⎡⎛ Δr ⎞ ⎛
2
Δr ⎞ ⎤
2

( i −1 i )
T − T + ( i +1 i )
T − T + 
g ′′′ ( i ) ⎢⎜ i
T π L r + −
⎟ ⎜ ir − ⎟ ⎥ = 0 (7)
Δr Δr ⎢⎣⎝ 2 ⎠ ⎝ 2 ⎠ ⎥⎦
for i = 2...( N − 1)

Figure 2 illustrates the control volume associated with the node that is placed on the outer
surface of the hay (i.e., node N).

Figure 2: Control volume for node N located on hay outer surface

The energy balance for the control volume associated with node N is:

q LHS + g = qout (8)

where the conduction term is:

⎛ Δr ⎞
k 2 π ⎜ rN − ⎟ L
⎝ 2 ⎠
q LHS = (TN −1 − TN ) , (9)
Δr

the generation term is:

⎡ ⎛ Δr ⎞ ⎤
2

g = g ′′′ (TN ) π L ⎢ rN2 − ⎜ rN − ⎟ ⎥ , (10)


⎣⎢ ⎝ 2 ⎠ ⎦⎥

Note that the volume in Eq. (10) is calculated differently from the volume in Eq. (6) because the
control volume is half as wide radially. The heat transfer to the external air is:

qout =
(TN − T∞ ) (11)
R p + Rconv

where
thp
Rp = (12)
k p 2 π Rbale L

and

1
Rconv ,out = (13)
h 2 π Rbale L

Substituting Eqs. (9) through (11) into Eq. (8) leads to:

⎛ Δr ⎞
k 2 π ⎜ rN − ⎟ L
⎡ 2 ⎛ Δr ⎞ ⎤ (TN − T∞ )
2
⎝ 2 ⎠
( N −1 N )
T − T + 
g ′′′ ( N) ⎢ N ⎜ N
T π L r − r − ⎟ ⎥= (14)
Δr ⎢⎣ ⎝ 2 ⎠ ⎥⎦ R p + Rconv

A similar procedure applied to the control volume associated with node 1 leads to:

⎛ Δr ⎞
k 2 π ⎜ r1 + ⎟L Δr ⎞
2
⎝ 2 ⎠ ⎛
(T2 − T1 ) + g ′′′ (T1 ) π L ⎜ r1 + ⎟ =0 (15)
Δr ⎝ 2 ⎠

Equations (7), (14), and (15) represent N equations in an equal number of unknowns; the solution
of these equations provides the numerical solution.

"Internal control volumes"


duplicate i=2,(N-1)
k*2*pi*(r[i]-Deltar/2)*L*(T[i-1]-T[i])/Deltar+k*2*pi*(r[i]+Deltar/2)*L*(T[i+1]-
T[i])/Deltar+gen(T[i])*pi*L*((r[i]+Deltar/2)^2-(r[i]-Deltar/2)^2)=0
end

"node N"
R_p=t_p/(k_p*2*pi*R_bale*L) "conduction resistance of plastic"
R_conv=1/(h*2*pi*R_bale*L) "convection resistance"
k*2*pi*(r[N]-Deltar/2)*L*(T[N-1]-T[N])/Deltar+gen(T[N])*pi*L*(r[N]^2-(r[N]-Deltar/2)^2)=(T[N]-
T_infinity)/(R_p+R_conv)

"node 1"
k*2*pi*(r[1]+Deltar/2)*L*(T[2]-T[1])/Deltar+gen(T[1])*pi*L*(r[1]+Deltar/2)^2=0

If the EES program is solved then the temperature distribution will be placed in the Arrays
window. The temperature as a function of position is shown in Figure 3.
Figure 3: Temperature as a function of position within the bale

b.) Show that your model has numerically converged; that is, show some aspect of your solution
as a function of the number of nodes in your solution and discuss an appropriate number of
nodes to use.

The maximum temperature (i.e., the temperature at the center of the bale) is shown in Figure 4 as
a function of the number of nodes. The model is numerically converged after approximately N =
20.

Figure 4: Predicted maximum temperature as a function of the number of nodes


c.) Verify your numerical model by comparing your answer to an analytical solution in some,
appropriate limit. The result of this step should be a plot which shows the temperature as a
function of radius predicted by both your numerical solution and the analytical solution and
demonstrates that they agree.

The analytical solution derived in the problem 1.3-8 is used to compute the temperature at each
nodal position:

"Analytical solution from Problem 1.3-8"


a=-1 [W/m^3] "coefficients for volumetric generation function"
b=0.01 [W/m^3-K]
dTdr_Rbale = -BesselJ(1,(b/k)^(1/2)*R_bale)*(b/k)^(1/2)*C_2 "symbolic expressions from Maple"
T_Rbale = BesselJ(0,(b/k)^(1/2)*R_bale)*C_2-1/b*a
-k*2*pi*R_bale*L*dTdr_Rbale=(T_Rbale-T_infinity)/(R_p+R_conv)"interface energy balance"
duplicate i=1,N
T_an[i]=BesselJ(0,sqrt(b/k)*r[i])*C_2-a/b
end

Figure 3 illustrates the analytical solution overlaid on the numerical solution and demonstrates
agreement.

d.) Prepare a numerical model of the hay bale using MATLAB. Plot the temperature as a
function of position within the hay bale.

A new m-file is opened and formatted as a function with a single input (the number of nodes)
and two outputs (vectors containing the radial position and temperature at each node).

function[r,T]=P1p5_1(N)

L = 1; %per unit length of bale (m)


R_bale= 1.524; %bale radius (m)
t_p=0.00114; %plastic thickness (m)
k_p=0.15; %plastic conductivity (W/m-K)
T_infinity=293.2; %ambient temperature (K)
h=10; %heat transfer coefficient (W/m^2-K)
k=0.04; %hay conductivity (W/m-K)

end

A function is defined that returns the volumetric rate of generation as a function of temperature;
the function is placed at the bottom of the same m-file so that it is accessible locally to P1p5_1.

function[gv]=gen(T)

%coefficients of function
a=-1; %(W/m^3)
b=0.01; %(W/m^3-K)
gv=a+b*T;

end
The radial position of each node is stored in the vector r.

Deltar=R_bale/(N-1); %distance between adjacent nodes (m)


for i=1:N
r(i,1)=Deltar*(i-1); %radial location of each node (m)
end

The problem is nonlinear because the generation rate depends on temperature; therefore, the
method of successive substitution is used. An initial guess for the temperature distribution is
stored in the vector T_g:

%initial guess for temperature distribution


for i=1:N
T_g(i,1)=T_infinity;
end

The guess values for temperature are used to setup the matrix A and vector b which contain the
matrix formulation of the equations. The energy balance for node 1 is placed in row 1 of A.

⎛ Δr ⎞
k 2 π ⎜ r1 + ⎟L Δr ⎞
2
⎝ 2 ⎠ ⎛
(T2 − T1 ) + g ′′′ (T1* ) π L ⎜ r1 + ⎟ =0 (16)
Δr ⎝ 2 ⎠

where T1* is the guess value of the temperature or

⎡ ⎛ Δr ⎞ ⎤ ⎡ ⎛ Δr ⎞ ⎤
⎢ k 2 π ⎜ r1 + 2 ⎟ L ⎥ ⎢ k 2 π ⎜ r1 + 2 ⎟ L ⎥
⎠ ⎥ = − g ′′′ T * π L ⎛ r + Δr ⎞
2

T1 ⎢ − ⎝
Δr
⎠ ⎥ +T ⎢
2

Δr
( 1 ) ⎜⎝ 1 2 ⎟⎠ (17)
⎢ ⎥ ⎢ ⎥ 

⎢⎣ ⎥⎦ ⎢⎣ ⎥⎦ b (1)



A (1,1) A (1,2)

The energy balances for the internal nodes are:

⎛ Δr ⎞ ⎛ Δr ⎞
k 2 π ⎜ ri − ⎟ L k 2 π ⎜ ri + ⎟L
⎝ 2 ⎠ ⎝ 2 ⎠ ⎡⎛ Δr ⎞ ⎛
2
Δr ⎞ ⎤
2

(Ti −1 − Ti ) + (Ti +1 − Ti ) + g ′′′ (Ti ) π L ⎢⎜ ri + ⎟ − ⎜ ri − ⎟ ⎥ = 0 (18)


*

Δr Δr ⎢⎣⎝ 2 ⎠ ⎝ 2 ⎠ ⎥⎦
for i = 2...( N − 1)

or
⎡ ⎛ Δr ⎞ ⎛ Δr ⎞ ⎤ ⎡ ⎛ Δr ⎞ ⎤
⎢ k 2 π ⎜ ri − 2 ⎟ L k 2 π ⎜ ri + 2 ⎟ L ⎥ ⎢ k 2 π ⎜ ri − 2 ⎟ L ⎥
Ti ⎢ − ⎝ ⎠ − ⎝ ⎠ ⎥ +T ⎢ ⎝ ⎠ ⎥+
i −1
⎢ Δr Δr ⎥ ⎢ Δr ⎥
⎢⎣ ⎥⎦ ⎢⎣ ⎥



A ( i ,i ) A ( i ,i −1)

⎡ ⎛ Δr ⎞ ⎤
⎢ k 2 π ⎜ ri + 2 ⎟ L ⎥ ⎡⎛ Δr ⎞ ⎛
2
Δr ⎞ ⎤
2

⎢ ⎝ ⎠ ⎥ = − g (Ti ) π L ⎢⎜ ri
′′′ +
 ⎟ − ⎜ ri − ⎟ ⎥
*
Ti +1 (19)
⎢ Δr ⎥ ⎢⎝
⎣ 2 ⎠ ⎝ 2 ⎠ ⎦⎥
⎢⎣ ⎥⎦ 


b (i )
A ( i ,i +1)

for i = 2 .. ( N − 1)

The energy balance for node N is:

⎛ Δr ⎞
k 2 π ⎜ rN − ⎟ L
⎡ 2 ⎛ Δr ⎞ ⎤ (TN − T∞ )
2
⎝ 2 ⎠
(TN −1 − TN ) + g ′′′ (TN ) π L ⎢ rN − ⎜ rN − ⎟ ⎥ =
*
(20)
Δr ⎢⎣ ⎝ 2 ⎠ ⎥⎦ R p + Rconv

or

⎡ ⎛ Δr ⎞ ⎤ ⎡ ⎛ Δr ⎞ ⎤
⎢ k 2 π ⎜ rN − 2 ⎟ L ⎥ ⎢ k 2 π ⎜ rN − ⎟ L ⎥
TN ⎢ − ⎝ ⎠ − 1
⎥ + TN −1 ⎢ ⎝ 2 ⎠
⎥=
⎢ Δr R p + Rconv ⎥ ⎢ Δr ⎥
⎢⎣ ⎥⎦ ⎢⎣ ⎥


⎦ (21)
A( N , N ) A ( N , N −1)

⎡ ⎛ Δr ⎞ ⎤
2

− g ′′′ (TN* ) π L ⎢ rN2 − ⎜ rN − ⎟ ⎥ −


T∞
⎢⎣ ⎝ 2 ⎠ ⎥⎦ R p + Rconv


b( N )

The matrices A and b are initialized and the resistances due to convection and conduction
through the plastic are computed:

A=spalloc(N,N,3*N);
b=zeros(N,1);
R_p=t_p/(k_p*2*pi*R_bale*L); %resistance through plastic
R_conv=1/(h*2*pi*R_bale*L); %resistance due to convection

The matrices A and b are filled in according to Eqs. (17), (19), and (21):

%Node 1
A(1,1)=-k*2*pi*(r(1)+Deltar/2)*L/Deltar;
A(1,2)=k*2*pi*(r(1)+Deltar/2)*L/Deltar;
b(1)=-gen(T_g(i))*pi*L*(r(1)+Deltar/2)^2;
%Nodes 2 to (N-1)
for i=2:(N-1)
A(i,i)=-k*2*pi*(r(i)-Deltar/2)*L/Deltar-
k*2*pi*(r(i)+Deltar/2)*L/Deltar;
A(i,i-1)=k*2*pi*(r(i)-Deltar/2)*L/Deltar;
A(i,i+1)=k*2*pi*(r(i)+Deltar/2)*L/Deltar;
b(i)=-gen(T_g(i))*pi*L*((r(i)+Deltar/2)^2-(r(i)-Deltar/2)^2);
end

%Node N
A(N,N)=-k*2*pi*(r(N)-Deltar/2)*L/Deltar-1/(R_p+R_conv);
A(N,N-1)=k*2*pi*(r(N)-Deltar/2)*L/Deltar;
b(N)=-gen(T_g(N))*pi*L*(r(N)^2-(r(N)-Deltar/2)^2)-
T_infinity/(R_p+R_conv);

The temperature distribution is obtained according to:

T=A\b;

The successive substitution process occurs within a while loop that is terminated when some
convergence error, err, goes below a tolerance, tol. The tolerance is set and the error is
initialized to a value that will ensure that the loop executes at least once. Once the solution is
obtained, it is compared with the guess value to determine an error. The guess values are reset
and, if the error is not sufficiently small then the process is repeated. The code is shown below;
the new lines are shown in bold:

function[r,T]=P1p5_1(N)

L = 1; %per unit length of bale (m)


R_bale= 1.524; %bale radius (m)
t_p=0.00114; %plastic thickness (m)
k_p=0.15; %plastic conductivity (W/m-K)
T_infinity=293.2; %ambient temperature (K)
h=10; %heat transfer coefficient (W/m^2-K)
k=0.04; %hay conductivity (W/m-K)

Deltar=R_bale/(N-1); %distance between adjacent nodes (m)


for i=1:N
r(i,1)=Deltar*(i-1); %radial location of each node (m)
end

%initial guess for temperature distribution


for i=1:N
T_g(i,1)=T_infinity;
end

A=spalloc(N,N,3*N);
b=zeros(N,1);
R_p=t_p/(k_p*2*pi*R_bale*L); %resistance through plastic
R_conv=1/(h*2*pi*R_bale*L); %resistance due to convection

tol=0.1; %tolerance for convergence (K)


err=2*tol; %error initialization
while(err>tol)
%Node 1
A(1,1)=-k*2*pi*(r(1)+Deltar/2)*L/Deltar;
A(1,2)=k*2*pi*(r(1)+Deltar/2)*L/Deltar;
b(1)=-gen(T_g(i))*pi*L*(r(1)+Deltar/2)^2;

%Nodes 2 to (N-1)
for i=2:(N-1)
A(i,i)=-k*2*pi*(r(i)-Deltar/2)*L/Deltar-
k*2*pi*(r(i)+Deltar/2)*L/Deltar;
A(i,i-1)=k*2*pi*(r(i)-Deltar/2)*L/Deltar;
A(i,i+1)=k*2*pi*(r(i)+Deltar/2)*L/Deltar;
b(i)=-gen(T_g(i))*pi*L*((r(i)+Deltar/2)^2-(r(i)-Deltar/2)^2);
end

%Node N
A(N,N)=-k*2*pi*(r(N)-Deltar/2)*L/Deltar-1/(R_p+R_conv);
A(N,N-1)=k*2*pi*(r(N)-Deltar/2)*L/Deltar;
b(N)=-gen(T_g(N))*pi*L*(r(N)^2-(r(N)-Deltar/2)^2)-
T_infinity/(R_p+R_conv);

T=A\b; %obtain temperature distribution


err=sum(abs(T-T_g))/N %calculate error
T_g=T;
end

end

function[gv]=gen(T)

%coefficients of function
a=-1; %(W/m^3)
b=0.01; %(W/m^3-K)
gv=a+b*T;
end

The temperature as a function of radius is shown in Figure 5.


Figure 5: Predicted temperature as a function of radial position
Problem 1.5-2 (1-12 in text): Mass Flow Meter (re-visited)
Reconsider the mass flow meter that was investigated in Problem 1.3-9 (1-9 in text). Assume
that the conductivity of the material that is used to make the test section is not actually constant
as was assumed in Problem 1.3-9 (1-9 in text) but rather depends on temperature according to:

⎡ W ⎤
2 (
T − 300 [ K ])
W
k = 10 + 0.035 ⎢
m-K ⎣ m-K ⎥⎦

a.) Develop a numerical model of the mass flow meter using MATLAB. Plot the temperature as
a function of radial position for the conditions shown in Figure P1.3-9 (P1-9 in text) with the
temperature-dependent conductivity.

The inputs are entered in a MATLAB function that requires as an input the number of nodes (N):

function[r,T_C]=P1p5_2(N)

r_out=0.0254; %outer radius of test section (m)


r_in=0.01905; %inner radius of test section (m)
h_bar_out=10; %external convection coefficient (W/m^2-K)
T_infinity=293.2; %air temperature (K)
T_f=291.2; %fluid temperature (K)
gv=1e7; %rate of generation (W/m^3)
m_dot=0.75; %mass flow rate (kg/s)
th_ins=0.00635; %thickness of the insulation (m)
k_ins=1.5; %insulation conductivity (W/m-K)
L=0.0762; %length of the test section (m)
C=2500; %constant for convection relationship

The convection coefficient on the internal surface is computed:

h_bar_in=C*m_dot^0.8; %internal convection coefficient

A function is defined that returns the conductivity of the material:

function[k]=k_t(T)
%conductivity of the material
%
%Inputs:
% T: temperature (K)
%
%Outputs:
% k: conductivity (W/m-K)

k=10+0.035*(T-300);
end

A uniform distribution of nodes is used, the radial location of each node (ri) is:
( i − 1) r − r
ri = rin + ( ) for i = 1..N (1)
( N − 1) out in

where N is the number of nodes. The radial distance between adjacent nodes (Δr) is:

Δr =
( rout − rin ) (2)
( N − 1)
DELTAr=(r_out-r_in)/(N-1); %distance between adjacent nodes (m)
for i=1:N
r(i)=r_in+(r_out-r_in)*(i-1)/(N-1); %position of each node (m)
end

The system of equations is placed in matrix format.

AX =b (3)

The most logical technique for ordering the unknown temperatures in the vector X is:

⎡ X 1 = T1 ⎤
⎢ X =T ⎥
X =⎢ 2 2 ⎥
(4)
⎢ ... ⎥
⎢ ⎥
⎣ X N = TN ⎦

Equation (4) shows that the unknown temperature at node i (i.e., Ti) corresponds to element i of
vector X (i.e., Xi). The most logical technique for placing the equations into the A matrix is:

⎡ row 1 = control volume 1 equation ⎤


⎢ row 2 = control volume 2 equation ⎥
A=⎢ ⎥ (5)
⎢ ... ⎥
⎢ ⎥
⎣ row N = control volume N equation ⎦

In Eq. (5), the equation for control volume i is placed into row i.

An energy balance is carried out on a control volume surrounding each node. For node 1, placed
at the inner surface (Figure P1.5-2-1):

qconv ,in + qouter + g = 0 (6)


qouter T2
g1
T1
qconv ,in
Figure P1.5-2-1: Control volume around node 1.

The rate equation for convection is:

qconv ,in = hin 2 π rin L (T f − T1 ) (7)

The rate equation for conduction is:

⎛ Δr ⎞ (T2 − T1 )
qouter = kT =(T1 +T2 ) / 2 2 π ⎜ rin + ⎟L (8)
⎝ 2 ⎠ Δr

The rate equation for generation is:

Δr
g = 2 π rin L g ′′′ (9)
2

Substituting Eqs. (7) through (9) into Eq. (6) leads to:

⎛ Δr ⎞ (T2 − T1 )
hin 2 π rin L (T f − T1 ) + kT =(T1 +T2 ) / 2 2 π ⎜ rin + ⎟L + π rin Δr L g ′′′ = 0 (10)
⎝ 2 ⎠ Δr

Equation (10) is rearranged to identify the coefficients that multiply each unknown temperature:

⎡ ⎛ Δr ⎞ L ⎤ ⎡ ⎛ Δr ⎞ L ⎤
T1 ⎢ − hin 2 π rin L − kT =(T1 +T2 ) / 2 2 π ⎜ rin + ⎟ ⎥ + T2 ⎢ kT =(T1 +T2 ) / 2 2 π ⎜ rin + ⎟ =
⎣ ⎝ 2 ⎠ Δr ⎦ ⎣ ⎝ 2 ⎠ Δr ⎥⎦ (11)
−π rin Δr L g ′′′ − hin 2 π rin LT f

An energy balance on an internal node is shown in Figure P1.5-2-2:

qinner + qouter + g = 0 (12)

qouter Ti+1
g Ti
qinner Ti-1
Figure P1.5-2-2: Control volume around internal node i.
The rate equations for conduction are:

⎛ Δr ⎞ (Ti +1 − Ti )
qouter = kT =(Ti +Ti+1 ) / 2 2 π ⎜ rin + ⎟L (13)
⎝ 2 ⎠ Δr

⎛ Δr ⎞ ( T − T )
qinner = kT =(Ti +Ti−1 ) / 2 2 π ⎜ rin − ⎟ L i −1 i (14)
⎝ 2 ⎠ Δr

The rate equation for generation is:

g = 2 π ri Δr L g ′′′ (15)

Substituting Eqs. (13) through (15) into Eq. (12) for all of the internal nodes leads to:

⎛ Δr ⎞ (T − T ) ⎛ Δr ⎞ (Ti +1 − Ti )
kT =(Ti +Ti−1 ) / 2 2 π ⎜ rin − ⎟ L i −1 i + kT =(Ti +Ti+1 ) / 2 2 π ⎜ rin + ⎟L
⎝ 2 ⎠ Δr ⎝ 2 ⎠ Δr (16)
+2 π ri Δr L g ′′′ = 0 for i = 2.. ( N − 1)

Equation (16) is rearranged to identify the coefficients that multiply each unknown temperature:

⎡ ⎛ Δr ⎞ L ⎛ Δr ⎞ L ⎤
Ti ⎢ − kT =(Ti +Ti−1 ) / 2 2 π ⎜ rin − ⎟ − kT =(Ti +Ti+1 ) / 2 2 π ⎜ rin + ⎟
⎣ ⎝ 2 ⎠ Δr ⎝ 2 ⎠ Δr ⎥⎦
⎡ ⎛ Δr ⎞ L ⎤ ⎡ ⎛ Δr ⎞ L ⎤
+Ti −1 ⎢ −kT =(Ti +Ti−1 ) / 2 2 π ⎜ rin − ⎟ ⎥ + Ti +1 ⎢ − kT =(Ti +Ti+1 ) / 2 2 π ⎜ rin + ⎟ (17)
⎣ ⎝ 2 ⎠ Δr ⎦ ⎣ ⎝ 2 ⎠ Δr ⎦⎥
= −2 π ri Δr L g ′′′ for i = 2.. ( N − 1)

An energy balance on node N placed on the outer surface is shown in Figure P1.5-2-3:

qinner + qair + g = 0 (18)

qair
TN
g
qinner TN-1
Figure P1.5-2-3: Control volume around internal node N.

The rate equation for the heat transfer with the air is:

qair =
(T∞ − TN ) (19)
(R ins + Rconv ,out )
where

⎡ ( r + thins ) ⎤
ln ⎢ out ⎥
Rins = ⎣ rout ⎦ (20)
2 π L kins

1
Rconv ,out = (21)
2 π ( rout + thins ) L hout

R_ins=log((r_out+th_ins)/r_out)/(2*pi*L*k_ins);
%resistance to conduction through insulation
R_conv_out=1/(2*pi*(r_out+th_ins)*L*h_bar_out);
%resistance to convection from the outside surface of the insulation

The rate equation for conduction is:

⎛ Δr ⎞ (T − T )
qinner = kT =(TN +TN −1 ) / 2 2 π ⎜ rout − ⎟ L N −1 N (22)
⎝ 2 ⎠ Δr

The rate equation for generation is:

Δr
g = 2 π rout L g ′′′ (23)
2

Substituting Eqs. (19), (22), and (23) into Eq. (18) leads to:

⎛ Δr ⎞ (T − T ) (T∞ − TN ) + 2 π r Δr L g ′′′ = 0
kT =(TN +TN −1 ) / 2 2 π ⎜ rout − ⎟ L N −1 N + (24)
⎝ 2 ⎠ Δr ( Rins + Rconv,out ) out
2

Equation (24) is rearranged to identify the coefficients that multiply each unknown temperature:

⎡ ⎛ Δr ⎞ L 1 ⎤
TN ⎢ − kT =(TN +TN −1 ) / 2 2 π ⎜ rout − ⎟ − ⎥
⎢⎣ ⎝ 2 ⎠ Δr ( Rins + Rconv ,out ) ⎥⎦
⎡ ⎛ Δr ⎞ L ⎤
+TN −1 ⎢ − kT =(TN +TN −1 ) / 2 2 π ⎜ rout − ⎟ ⎥ (25)
⎣ ⎝ 2 ⎠ Δr ⎦
T∞
= − π rout Δr L g ′′′ −
( Rins + Rconv,out )
Equations (11), (17), and (25) are N equations in the N unknown temperatures. Because they are
non-linear, they must be linearized and a successive substitution method used. A guess
temperature distribution ( Tˆi ) is assumed:

%initial guess for temperature distribution


for i=1:N
Tg(i,1)=T_f;
end

The matrix A is defined as a sparse matrix with at most 3N nonzero entries:

%initialize A and b
A=spalloc(N,N,3*N);
b=zeros(N,1);

The solution is placed within a while loop that terminates when the error between the solution
and the guess is less than some tolerance:

err=999; %initial value for error (K), must be larger than tol
tol=0.01; %tolerance for convergence (K)
while(err>tol)

The equation for node 1, Eq. (11), is linearized by using the guess temperature distribution to
compute the conductivity:

⎡ ⎛ Δr ⎞ L ⎤ ⎡ ⎛ Δr ⎞ L ⎤
T1 ⎢ − hin 2 π rin L − kT = Tˆ +Tˆ / 2 2 π ⎜ rin + ⎟ ⎥ + T2 ⎢ kT =(Tˆ1 +Tˆ2 ) / 2 2 π ⎜ rin + ⎟ =
⎣ ( 1 2)
⎝ 2 ⎠ Δr ⎦ ⎣ ⎝ 2 ⎠ Δr ⎥⎦



A1,1 A1,2 (26)


−π rin Δr L g ′′′ − hin 2 π rin LT f


b1

A(1,1)=-h_bar_in*2*pi*r_in*L-...
k_t((Tg(1)+Tg(2))/2)*2*pi*(r_in+DELTAr/2)*L/DELTAr;
A(1,2)=k_t((Tg(1)+Tg(2))/2)*2*pi*(r_in+DELTAr/2)*L/DELTAr;
b(1)=-pi*r_in*DELTAr*L*gv-h_bar_in*2*pi*r_in*L*T_f;

The equations for the internal nodes, Eq. (17), is also linearized:
⎡ ⎛ Δr ⎞ L ⎛ Δr ⎞ L ⎤
Ti ⎢ − kT =(Ti +Ti−1 ) / 2 2 π ⎜ rin − ⎟ − kT =(Ti +Ti+1 ) / 2 2 π ⎜ rin + ⎟
⎣ ⎝ 2 ⎠ Δr ⎝ 2 ⎠ Δr ⎦⎥

Ai ,i

⎡ ⎛ Δr ⎞ L ⎤ ⎡ ⎛ Δr ⎞ L ⎤
+Ti −1 ⎢ −kT =(Ti +Ti−1 ) / 2 2 π ⎜ rin − ⎟ ⎥ + Ti +1 ⎢ − kT =(Ti +Ti+1 ) / 2 2 π ⎜ rin + ⎟ (27)
⎣ ⎝ 2 ⎠ Δr ⎦ ⎣ ⎝ 2 ⎠ Δr ⎥⎦


Ai ,i −1 Ai ,i +1

= −2 π ri Δr L g ′′′ for i = 2.. ( N − 1)




bi

for i=2:(N-1)
A(i,i)=-k_t((Tg(i)+Tg(i-1))/2)*2*pi*(r(i)-DELTAr/2)*L/DELTAr...
-k_t((Tg(i)+Tg(i+1))/2)*2*pi*(r(i)+DELTAr/2)*L/DELTAr;
A(i,i-1)=k_t((Tg(i)+Tg(i-1))/2)*2*pi*(r(i)-DELTAr/2)*L/DELTAr;
A(i,i+1)=k_t((Tg(i)+Tg(i+1))/2)*2*pi*(r(i)+DELTAr/2)*L/DELTAr;
b(i)=-2*pi*r(i)*DELTAr*L*gv;
end

The equation for node N, Eq. (25), is linearized:

⎡ ⎛ Δr ⎞ L 1 ⎤
TN ⎢ − kT =(TN +TN −1 ) / 2 2 π ⎜ rout − ⎟ − ⎥
⎣⎢ ⎝ 2 ⎠ Δr ( Rins + Rconv ,out ) ⎦⎥

AN , N

⎡ ⎛ Δr ⎞ L ⎤
+TN −1 ⎢ − kT =(TN +TN −1 ) / 2 2 π ⎜ rout − ⎟ ⎥ (28)
⎣ ⎝ 2 ⎠ Δr ⎦

AN , N −1

T∞
= − π rout Δr L g ′′′ −
( Rins + Rconv,out )


bN

A(N,N)=-k_t((Tg(N)+Tg(N-1))/2)*2*pi*(r_in-DELTAr/2)*L/DELTAr-...
1/(R_ins+R_conv_out);
A(N,N-1)=k_t((Tg(N)+Tg(N-1))/2)*2*pi*(r_in-DELTAr/2)*L/DELTAr;
b(N)=-pi*r_out*DELTAr*L*gv-T_infinity/(R_ins+R_conv_out);

The solution is obtained:

X=A\b;
T=X;

and used to compute the error between the assumed and calculated solutions is obtained:

∑ (T − Tˆ )
N
1 2
err = i i (29)
N i =1
err=sqrt(sum((T-Tg).^2)/N) %compute rms error

The calculated solution becomes the guess value for the next iteration:

Tg=T; %reset guess values used to setup A and b


end

The solution is converted to degrees Celsius:

T_C=T-273.2; %convert to C
end

The solution is illustrated in Figure P1.5-2-4.


75

72.5

70
Temperature (°C)

67.5

65

62.5

60

57.5

55

52.5
0.019 0.02 0.021 0.022 0.023 0.024 0.025 0.026
Radius (m)
Figure P1.5-2-4: Temperature as a function of radius.

b.) Verify that your numerical solution limits to the analytical solution from Problem 1.3-9 (1-9
in text) in the limit that the conductivity is constant.

The conductivity function is modified temporarily so that it returns a constant value:

function[k]=k_t(T)
%conductivity of the material
%
%Inputs:
% T: temperature (K)
%
%Outputs:
% k: conductivity (W/m-K)

k=10;%+0.035*(T-300);
end

Figure P1.5-2-5 illustrates the temperature distribution predicted by the numerical and analytical
solutions in the limit that k is constant.
80

75

Temperature (°C)
70

65

60 analytical model
numerical model
55

50
0.019 0.02 0.021 0.022 0.023 0.024 0.025 0.026
Radius (m)
Figure P1.5-2-5: Temperature as a function of radius predicted by the analytical and numerical models in the
limit that k is constant.
Problem 1.6-1 (1-13 in text): Temperature Sensor Error
A resistance temperature detector (RTD) utilizes a material that has a resistivity that is a strong
function of temperature. The temperature of the RTD is inferred by measuring its electrical
resistance. Figure P1.6-1 shows an RTD that is mounted at the end of a metal rod and inserted
into a pipe in order to measure the temperature of a flowing liquid. The RTD is monitored by
passing a known current through it and measuring the voltage across it. This process results in a
constant amount of ohmic heating that may tend to cause the RTD temperature to rise relative to
the temperature of the surrounding liquid; this effect is referred to as a self-heating error. Also,
conduction from the wall of the pipe to the temperature sensor through the metal rod can also
result in a temperature difference between the RTD and the liquid; this effect is referred to as a
mounting error.

Tw = 20°C pipe
L = 5.0 cm

D = 0.5 mm
x
h = 150 W/m -K
2
k = 10 W/m-K
T∞ = 5.0°C qsh = 2.5 mW
RTD

Figure P1.6-1: Temperature sensor mounted in a flowing liquid.

The thermal energy generation associated with ohmic heating is q sh = 2.5 mW. All of this ohmic
heating is assumed to be transferred from the RTD into the end of the rod at x = L. The rod has a
thermal conductivity k = 10 W/m-K, diameter D = 0.5 mm, and length L = 5 cm. The end of the
rod that is connected to the pipe wall (at x = 0) is maintained at a temperature of Tw = 20°C. The
liquid is at a uniform temperature, T∞ = 5°C and the heat transfer coefficient between the liquid
and the rod is h = 150 W/m2-K.
a.) Is it appropriate to treat the rod as an extended surface (i.e., can we assume that the
temperature in the rod is a function only of x)? Justify your answer.

The input parameters are entered in EES.

$UnitSystem SI MASS RAD PA K J


$TABSTOPS 0.2 0.4 0.6 0.8 3.5 in

"Inputs"
q_dot_sh=0.0025 [W] "self-heating power"
k=10 [W/m-K] "conductivity of mounting rod"
d=0.5 [mm]*convert(mm,m) "diameter of mounting rod"
L=5.0 [cm]*convert(cm,m) "length of mounting rod"
T_wall=convertTemp(C,K,20) "temperature of wall"
T_f=convertTemp(C,K,5) "temperature of liquid"
h=150 [W/m^2-K] “heat transfer coefficient”
The appropriate Biot number for this case is:

hd
Bi = (1)
2k

"Extended surface approximation"


Bi=h*d/(2*k)

The Biot number calculated by EES is 0.004 which is much less than 1.0 and therefore the
extended surface approximation is justified.

b.) Develop an analytical model of the rod that will predict the temperature distribution in the
rod and therefore the error in the temperature measurement; this error is the difference
between the temperature at the tip of the rod and the liquid. You may find it easiest to use
Maple for this process.

Figure 2 illustrates a differential control volume for the rod.

Figure 2: Differential control volume for the rod.

The energy balance suggested by Figure 2 is:

q x = q x + dx + qconv (2)

or, expanding the x+dx term:

dq
q x = q x + dx + qconv (3)
dx

The rate equations for conduction and convection are:

d 2 dT
q x = − k π (4)
4 dx

and

qconv = h π d dx (T − T f ) (5)
Substituting Eqs. (4) and (5) into Eq. (3) leads to:

d ⎡ d 2 dT ⎤
0= ⎢ − k π ⎥ dx + h π d dx (T − T f ) (6)
dx ⎣ 4 dx ⎦

or

d 2T 4 h

dx 2 k d
(T − T f ) = 0 (7)

which is a non-homogeneous 2nd order differential equation. The general solution to Eq. (7) can
be found in your text as Eq. (3.66) or in the handout on Extended Surfaces as Eq. (6-22). The
easiest thing to do is enter the differential equation into Maple and let it solve it for you:

> GDE:=diff(diff(T(x),x),x)-4*h*(T(x)-T_f)/(k*d)=0;
⎛d ⎞ 4 h ( T( x ) − T_f )
2
GDE := ⎜⎜ 2 T( x ) ⎟⎟ − =0
⎝ dx ⎠ kd
> Ts:=dsolve(GDE);
⎛⎜ 2 h x ⎞⎟ ⎛⎜ 2 h x ⎞⎟
⎜ k d ⎟ ⎜− k d ⎟
⎝ ⎠ ⎝ ⎠
Ts := T( x ) = e _C2 + e _C1 + T_f

The solution can be copied and pasted into EES (don’t forget that you may need to change your
output display to Maple Notation to facilitate the copying process depending on your version of
Maple):

> Ts:=dsolve(GDE);
Ts := T(x) = exp(2*h^(1/2)*x/(k^(1/2)*d^(1/2)))*_C2+exp(-
2*h^(1/2)*x/(k^(1/2)*d^(1/2)))*_C1+T_f

which can be copied to EES:

Ts := T(x) = exp(2*h^(1/2)*x/(k^(1/2)*d^(1/2)))*_C2+exp(-2*h^(1/2)*x/(k^(1/2)*d^(1/2)))*_C1+T_f
"solution copied from Maple"

The solution will need to be modified slightly so that it is compatible with EES (the _C1 must
become C1, _C2 must be C2, Ts:= should be deleted and the T(x) must be just T:

T = exp(2*h^(1/2)*x/(k^(1/2)*d^(1/2)))*C2+exp(-2*h^(1/2)*x/(k^(1/2)*d^(1/2)))*C1+T_f
"solution copied from Maple and modified"

Trying to solve now should give the message that you have 12 variables but only 9 equations –
you need to specify C1, C2, and x to have a completely specified problem. Let’s set x = 0:

x=0
and concentrate on determining symbolic expressions for the boundary conditions. The
temperature at the pipe wall (x=0) is specified to be Twall. Using Maple:

> rhs(eval(Ts,x=0))=T_wall;
_C2+_C1+T_f = T_wall

which can be pasted into EES (and modified):

C2+C1+T_f = T_wall "wall boundary condition"

The boundary condition at the end of the rod with the sensor is associated with an energy balance
on the interface:

d 2 dT
kπ = qsh (8)
4 dx x= L

which can be evaluated symbolically in Maple:

> k*pi*d^2*rhs(eval(diff(Ts,x),x=L))/4=q_dot_sh;
1/4*k*pi*d^2*(2*h^(1/2)*exp(2*h^(1/2)*L/(k^(1/2)*d^(1/2)))*_C2/(k^(1/2)*d^(1/
2))-2*h^(1/2)*exp(-2*h^(1/2)*L/(k^(1/2)*d^(1/2)))*_C1/(k^(1/2)*d^(1/2))) =
q_dot_sh

Aren’t you glad you don’t have to do this by hand? The expression can be copied and pasted
into EES to complete your solution:

1/4*k*pi*d^2*(2*h^(1/2)*exp(2*h^(1/2)*L/(k^(1/2)*d^(1/2)))*C2/(k^(1/2)*d^(1/2))-2*h^(1/2)*exp(-
2*h^(1/2)*L/(k^(1/2)*d^(1/2)))*C1/(k^(1/2)*d^(1/2))) = q_dot_sh "sensor boundary condition"

Check your units (Figure 3 shows the variable information window with the units set) to make
sure that no errors were made.
Figure 3: Variable Information window.

c.) Prepare a plot of the temperature as a function of position and compute the temperature error.

Comment out the specification that x=0 and prepare a parametric table that includes T and x.
Alter x so that it varies from 0 to 0.05 and plot the result. You can convert the temperature to °C
and position to cm for a better looking plot:

x_cm=x*convert(m,cm)
T_C=converttemp(K,C,T)

Figure 4 illustrates the temperature distribution; note that the temperature elevation at the tip
with respect to the fluid is about 3.6 K and it represents the measurement error. For the
conditions in the problem statement, it is clear that the measurement error is primarily due to the
self-heating effect because the effect of the wall (the temperature elevation at the base) has died
off after about 2.0 cm.
Figure 4: Temperature distribution in the mounting rod.

d.) Investigate the effect of thermal conductivity on the temperature measurement error. Identify
the optimal thermal conductivity and explain why an optimal thermal conductivity exists.

The temperature measurement error can be calculated from your solution by setting x = L:

"Part d - temperature measurement error"


x=L
errT=T-T_f

Figure 5 illustrates the temperature measurement error as a function of the thermal conductivity
of the rod material. Figure 5 shows that the optimal thermal conductivity, corresponding to the
minimum measurement error, is around 100 W/m-K. Below the optimal value, the self-heating
error dominates as the local temperature rise at the tip of the rod is large. Above the optimal
value, the conduction from the wall dominates. The inset figures show the temperature
distribution for high and low thermal conductivity in order to illustrate these different behaviors.
Figure 5: Temperature measurement error as a function of rod thermal conductivity. The inset figures show
the temperature distribution at low conductivity and high conductivity.
Problem 1.6-2 (1-14 in text): Optimizing a Heat Sink
Your company has developed a micro-end milling process that allows you to easily fabricate an
array of very small fins in order to make heat sinks for various types of electrical equipment.
The end milling process removes material in order to generate the array of fins. Your initial
design is the array of pin fins shown in Figure P1.6-2. You have been asked to optimize the
design of the fin array for a particular application where the base temperature is Tbase = 120°C
and the air temperature is Tair = 20°C. The heat sink is square; the size of the heat sink is W = 10
cm. The conductivity of the material is k = 70 W/m-K. The distance between the edges of two
adjacent fins is a, the diameter of a fin is D, and the length of each fin is L.

Tair = 20°C, h
array of fins D
k = 70 W/m-K
a
L

W = 10 cm
Tbase = 120°C

Figure P1.6-2: Pin fin array

Air is forced to flow through the heat sink by a fan. The heat transfer coefficient between the air
and the surface of the fins as well as the unfinned region of the base, h , has been measured for
the particular fan that you plan to use and can be calculated according to:

0.4 −0.3
⎡ W ⎤⎛ a ⎞ ⎛ D ⎞
h = 40 ⎢ 2 ⎥ ⎜⎜ ⎟ ⎜⎜ ⎟⎟
⎣ m K ⎦ ⎝ 0.005 [ m ] ⎟⎠ ⎝ 0.01 [ m ] ⎠

Mass is not a concern for this heat sink; you are only interested in maximizing the heat transfer
rate from the heat sink to the air given the operating temperatures. Therefore, you will want to
make the fins as long as possible. However, in order to use the micro-end milling process you
cannot allow the fins to be longer than 10x the distance between two adjacent fins. That is, the
length of the fins may be computed according to: L = 10 a . You must choose the most optimal
value of a and D for this application.
a.) Prepare a model using EES that can predict the heat transfer coefficient for a given value of a
and D. Use this model to predict the heat transfer rate from the heat sink for a = 0.5 cm and
D = 0.75 cm.

The input values are entered in EES:

$UnitSystem SI MASS RAD PA K J


$TABSTOPS 0.2 0.4 0.6 0.8 3.5 in

"Inputs"
T_air=converttemp(C,K,20) "air temperature"
T_base=converttemp(C,K,120) "base temperature"
k=70 [W/m-K] "fin material conductivity"
W=10.0 [cm]*convert(cm,m) "base width"

The optimization parameters, a and D, are set to their initial values:

"Optimization parameters"
a=0.5 [cm]*convert(cm,m) "distance between adjacent fins"
D=0.75 [cm]*convert(cm,m) "diameter of fins"

The length of the fins is computed using the aspect ratio and the number of fins is determined
according to:

2
⎛ W ⎞
N =⎜ ⎟ (1)
⎝a+D⎠

The heat transfer coefficient is computed using the equation provided in the problem statement.

L=10*a "length of fins"


N=(W/(a+D))^2 "number of fins"
h = 40 [W/m^2-K]*(a/0.005 [m])^(0.4)*(D/0.01 [m])^(-0.3) "heat transfer coefficient"

The perimeter and cross-sectional area of each fin are computed according to:

p =π D (2)

D2
Ac = π (3)
4

The EES function for the fin efficiency of a constant cross-sectional area fin is used. The
function is accessed using the Function Information selection from the Options menu and then
selecting Fin Efficiency from the pull-down menu. Scroll to the Circular-Base Rectangular Fin
(Figure 2(a)) and select Info to learn how to access this function (Figure 2(b)).
(a) (b)
Figure 2: (a) Function Information window and (b) Help information for the Circular-Base Rectangular Fin.

The fin constant, mL, is computed according to:

hp
mL = L (4)
k Ac

and used to call the function eta_fin_spine_rect which returns the fin efficiency, ηf.

p=pi*D "perimeter of fin"


Ac=pi*D^2/4 "cross sectional area of fin"
mL=L*sqrt(h*p/(k*Ac)) "fin constant"
eta=eta_fin_spine_rect(mL) "fin efficiency"

The total area of the fins on the heat sink is:

Af = N p L (5)

and so the total resistance of the fins are:

1
Rf = (6)
h Af η f

A_f=p*L*N "finned area"


R_f=1/(h*A_f*eta) "resistance of fins"

The total area of the base of the heat sink that is not finned is:

Auf = W 2 − N Ac (7)
and the thermal resistance from the unfinned base is:

1
Ruf = (8)
h Auf

A_uf=W^2-N*Ac "unfinned area"


R_uf=1/(h*A_uf) "resistance of unfinned area"

The total resistance of the heat sink is the combination of Rf and Ruf in parallel:

−1
⎛ 1 1 ⎞
Rtotal =⎜ + ⎟⎟ (9)
⎜R R
⎝ f uf ⎠

and the total heat transfer rate is:

q =
(Tbase − Tair ) (10)
Rtotal

R_total=(1/R_f+1/R_uf)^(-1)
"total thermal resistance of the heat sink"
q_dot=(T_base-T_air)/R_total "heat transfer"

which leads to q = 291.7 W.

b.) Prepare a plot that shows the heat transfer rate from the heat sink as a function of the distance
between adjacent fins, a, for a fixed value of D = 0.75 cm. Be sure that the fin length is
calculated using L = 10 a . Your plot should exhibit a maximum value, indicating that there
is an optimal value of a.

Figure 3 illustrates the heat transfer rate from the heat sink as a function of a for D = 0.75 cm.
Figure 3: Heat transfer rate as a function of the distance between adjacent fins for D = 0.75 cm.

c.) Prepare a plot that shows the heat transfer rate from the heat sink as a function of the
diameter of the fins, D, for a fixed value of a = 0.5 cm. Be sure that the fin length is
calculated using L = 10 a . Your plot should exhibit a maximum value, indicating that there
is an optimal value of D.

Figure 4 illustrates the heat transfer rate from the heat sink as a function of D for a = 0.5 cm.

Figure 4: Heat transfer rate as a function of the diameter of the fins for a = 0.5 cm.

d.) Determine the optimal value of a and D using EES' built-in optimization capability.
Comment out the optimization parameters (a and D) and access the optimization algorithms from
the Calculate Menu by selecting Min/Max (Figure 5).

Figure 5: Find Minimum or Maximum Window

Select the variable to be minimized or maximized from the list on the left and the independent
variables to be varied from the list on the right. You will need to provide a reasonable initial
guess and bounds for the independent variables by selecting the Bounds button; note that it is not
practical for a or D to be less than 1.0 mm. You can experiment with the different optimization
methods and see which technique is more robust.

I found the genetic optimization algorithm to work the best for this problem; with a sufficient
number of individuals I identified an optimal design consisting of approximately 1500 very small
fins of D = 1.1 mm separated by a = 1.4 mm. The associated rate of heat transfer is q = 352.2
W.
Problem 1.6-3: Finned Tube Water Heater
A water heater consists of a copper tube that carries water through hot gas in a furnace, as shown
in Fig. P1.6-3(a). The copper tube has an outer radius, ro,tube = 0.25 inch and a tube wall
thickness of th = 0.033 inch. The conductivity of the copper is ktube = 300 W/m-K. Water flows
through the pipe at a temperature of Tw = 30ºC. The heat transfer coefficient between the water
and the internal surface of the pipe is hw = 500 W/m2-K. The external surface of the tube is
exposed to hot gas at Tg = 500ºC. The heat transfer coefficient between the gas and the outer
surface of the pipe is hg = 25 W/m2-K. Neglect radiation from the tube surface.

Tg = 500°C
hg = 25 W/m -K
2

ro,tube = 0.25 inch

ktube = 300 W/m-K


th = 0.033 inch Tw = 30°C
hw = 500 W/m -K
2

Figure P1.6-3(a): Copper tube in a water heater.

a.) At what rate is heat is added to the water for a unit length of tube, L = 1 m, for this
configuration (W/m)?

The known information is entered in EES:

$UnitSystem SI MASS RAD PA K J


$TABSTOPS 0.2 0.4 0.6 0.8 3.5 in

"Inputs"
r_o_tube=0.25 [inch]*convert(inch,m) "outer tube radius"
th=0.033 [inch]*convert(inch,m) "tube thickness"
k_tube=300 [W/m-K] "tube material conductivity"
T_w=converttemp(C,K,30) "water temperature"
h_w=500 [W/m^2-K] "water to tube heat transfer coefficient"
T_g=converttemp(C,K,500) "gas temperature"
h_g=25 [W/m^2-K] "gas to tube heat transfer coefficient"
L=1 [m] "unit length of tube"

The resistance network that represents this problem is shown in Figure 2.

Figure 2: Resistance network for unfinned tube.

The resistance network includes convection to the inner surface of the tube (Rconv,w),
1
Rconv , w = , (1)
hw 2 π ( ro ,tube − th ) L

conduction through the tube (Rcond),

⎛ r ⎞
ln ⎜⎜ o ,tube ⎟⎟
ro ,tube − th ⎠
Rcond = ⎝ , (2)
2 π ktube L

and convection from the external surface of the tube (Rconv,g),

1
Rconv , g = (3)
hg 2 π ro ,tube L

The heat transfer is provided by:

Tg − Tw
q = −1
(4)
⎛ 1 1 ⎞
Rconv , w + Rcond +⎜ + ⎟⎟
⎜R
⎝ conv , g Rrad ⎠

"Part a: unfinned tube"


R_conv_w=1/(h_w*2*pi*(r_o_tube-th)*L) "internal convection resistance"
R_cond=ln(r_o_tube/(r_o_tube-th))/(2*pi*k_tube*L) "conduction resistance"
R_conv_g=1/(h_g*2*pi*r_o_tube*L) "external convection"
q_dot=(T_g-T_w)/(R_conv_w+R_cond+R_conv_g) "heat transfer rate"

The heat transfer is q =443 W.

b.) What is the dominant resistance to heat transfer in your water heater?

The solution window is shown in Figure 3:

Figure 3:Solution window.


Notice that the values of Rconv,g is much higher than Rcond or Rconv,w and therefore the convection
from the surface of the tube limits the heat transfer rate.

In order to increase the capacity of the water heater, you decide to slide washers over the tube, as
shown in Fig. P1.6-3(b). The washers are w = 0.06 inch thick with an outer radius of ro,washer =
0.625 inch and have a thermal conductivity of kwasher = 45 W/m-K. The contact resistance
between the washer and the tube is Rc′′ = 5x10-4 m2-K/W. The distance between two adjacent
washers is b = 0.25 inch.

ro,washer = 0.625 inch


b = 0.25 inch
kwasher = 45 W/m-K

w = 0.06 inch

Rc′′ = 5x10 m -K/W


-4 2

Figure P1.6-3(b): Water heater with washers installed.

c.) Can the brass washers be treated as extended surfaces (i.e. can the temperature in the washers
be considered to be only a function of radius)? Justify your answer with a calculation.

The additional information is entered in EES:

"Inputs for finned tube"


w=0.06 [inch]*convert(inch,m) "thickness of washers"
r_o_washer=0.625 [inch]*convert(inch,m) "outer radius of washer"
k_washer=45 [W/m-K] "conductivity of washer"
R``_c=5e-4 [m^2-K/W] "contact resistance"
b=0.25 [inch]*convert(inch,m) "distance between washers"

The Biot number associated with the washer must be the ratio of the resistance to conduction
from the center of the washer to its edge (axially) to the resistance to heat transfer from its
surface:

Rcond , x
Bi = (5)
Rconv , g , x

where the resistances in Eq. (5) are related to heat transfer axially and so:

w
Rcond , x = (6)
2 kwasher Awasher

1
Rconv , g , x = (7)
hg Awasher
and

Awasher = π ( ro2, washer − ro2,tube ) (8)

"Part c"
A_washer=pi*(r_o_washer^2-r_o_tube^2) "exposed area of one side of washer"
R_cond_x=w/(2*k_washer*A_washer) "conduction resistance axially"
R_conv_g_x=1/(h_g*A_washer) "convection resistance axially"
Bi=R_cond_x/R_conv_g_x "Biot number"

The Biot number is 0.0004 which is much less than one and therefore the temperature gradient in
the washer across its thickness is negligible relative to the temperature drop between its surface
and the gas. Therefore, the extended surface model is valid and the washer can be treated as a
fin.

Assume that your answer to (c) showed that the washers can be treated as extended surfaces and
therefore modeled as a fin with the appropriate fin resistance.
d.) Draw a thermal resistance network that can be used to represent this situation. Be sure to
draw and label resistances associated with convection through the water (Rconv,w), conduction
through the copper tube (Rcond), heat transfer through contact resistance (Rcontact), heat transfer
through the washers (Rwashers), and convection to gas from unfinned outer surface
(Rconv,g,unfinned).

The thermal resistance network is shown in Figure 5.

Figure 5: Resistance network representing finned tube.

e.) How much heat is added to the water with the washers installed on the tube for a 1 m length
of tube?

The value of Rconv,w and Rcond do not change from (a). The resistance associated with convection
from the unfinned portion of the tube is:

1
Rconv , g ,unfinned = (9)
⎛ b ⎞
hg 2 π ro ,tube L ⎜ ⎟
⎝b+w⎠
where the last term in the denominator is the fraction of the tube surface that is not occupied by
the fins. The contact resistance is:

Rc′′
Rcontact = (10)
⎛ w ⎞
2 π ro ,tube L ⎜ ⎟
⎝b+w⎠

where the last term in the denominator is the fraction of the tube surface that is occupied by the
fins.

"Part d"
R_conv_g_unfinned=1/(h_g*2*pi*r_o_tube*L*(b/(b+w))) "convection from unfinned surface"
R_contact=R``_c/(2*pi*r_o_tube*L*(w/(b+w))) "contact resistance"

The resistance of the fins is:

1
Rwashers = (11)
⎛ L ⎞
η f hg 2 Awasher ⎜ ⎟
⎝b+w⎠

where the last term in the denominator is the number of washers present on the tube. The fin
efficiency, ηf, can be calculated using EES’ built-in functions for fin efficiency. Select Function
Info from the Options menu and then select the button next to the list at the lower right of the top
box. Select Fin Efficiency and then Dimensional Efficiency and scroll over until you find
eta_fin_annular_rect (Figure 6).

Figure 6: Fin efficiency function information.


Select Paste and the function call will be inserted into your EES program. Note that Info
provides more detailed help about the function. Modify the arguments of the function so that
they match your variable names:

eta_f=eta_fin_annular_rect(w, r_o_tube, r_o_washer, h_g, k_washer) "fin efficiency"

Use the fin efficiency to calculate the total washer resistance:

R_washers=1/(eta_f*h_g*2*A_washer*(L/(b+w))) "washer resistance"

The total heat transfer to the finned tube is:

q finned =
(T g − Tw )
(12)
−1
⎛ 1 1 ⎞
Rconv , w + Rcond +⎜ + ⎟⎟
⎜R
⎝ conv , g ,unfinned Rcontact + Rwashers ⎠

q_dot_finned=(T_g-T_w)/(R_conv_w+R_cond+(1/R_conv_g_unfinned+1/(R_contact+R_washers))^(-1))
"total heat transfer rate from finned tube"

The addition of the fins has increased the heat transfer rate to 1540 W.
Problem 1.6-4: Wire with Ohmic Heating
A wire is subjected to ohmic heating (i.e., a current runs through it) while it is convectively
cooled. The ends of the wire have fixed temperatures. Describe how you would determine
whether the extended surface approximation (i.e., the approximation in which you treat the
temperature of the wire as being one-dimensional along its length and uniform radially) is
appropriate when solving this problem.

The justification of the extended surface approximation is related to computing the Biot number,
the ratio of the resistance that is being neglected (conduction radially within the wire) to the
resistance(s) being considered (convection from the wire surface). For this problem, the Biot
number would be:

rh
(1)
k

where r is the radius of the wire, h is the heat transfer coefficient, and k is the wire conductivity.
Note that the rate of ohmic heating is not important for this calculation because the ohmic
heating must be both conducted radially and convected from the surface.
Problem 1.6-5
A cylindrical bracket that is L = 4 cm long with diameter D = 5 mm extends between a wall at TH
= 100°C (at x= 0) and a wall at TC = 20°C (at x= L). The conductivity of the bracket is k = 25
W/m2-K. The cylinder is surrounded by gas at T∞ = 200°C and the heat transfer coefficient is
h = 250 W/m2-K.
a.) Is an extended surface approximation appropriate for this problem? Justify your answer.

The inputs are entered in EES:

$UnitSystem SI MASS RAD PA K J


$TABSTOPS 0.2 0.4 0.6 0.8 3.5 in

"Inputs"
D=5 [mm]*convert(mm,m) "diameter of strut"
L=4 [cm]*convert(cm,m) "length of strut"
k=25 [W/m-K] "conductivity of strut"
h_bar=250 [W/m^2-K] "heat transfer coefficient"
T_H=converttemp(C,K,100 [C]) "hot end temperature"
T_C=converttemp(C,K,20 [C]) "cold end temperature"
T_infinity=converttemp(C,K,200 [C]) "ambient temperature"

The Biot number is:

hD
Bi = (1)
k

Bi=h_bar*D/k "Biot number"

which leads to Bi = 0.05, justifying the extended surface approximation.

b.) Assume that your answer to (a) was yes. Develop an analytical model in EES. Plot the
temperature as a function of position within the bracket.

The development of the governing differential equation and the derivation of the general solution
proceeds as discussed in Section 1.6.2 and leads to:

T = C1 exp ( m x ) + C2 exp ( − m x ) + T∞ (2)

where C1 and C2 are undetermined constants and m is the fin constant:

per h
m= (3)
k Ac

where per is the perimeter of the bracket and Ac is the cross-sectional area of the bracket.

per = π D (4)
D2
Ac = π (5)
4

per=pi*D "perimeter"
A_c=pi*D^2/4 "cross-sectional area"
m=sqrt(per*h_bar/(k*A_c)) "fin constant"

The boundary conditions at x = 0 and x = L lead to:

TH = C1 + C2 + T∞ (6)

TC = C1 exp ( m L ) + C2 exp ( − m L ) + T∞ (7)

Equations (6) and (7) are entered in EES in order to determine C1 and C2:

T_H=C_1+C_2+T_infinity "boundary condition at x=0"


T_C=C_1*exp(m*L)+C_2*exp(-m*L)+T_infinity "boundary condition at x=L"

and the solution is entered in EES:

x=0 [m] "axial position"


T=C_1*exp(m*x)+C_2*exp(-m*x)+T_infinity "solution"
T_degC=converttemp(K,C,T) "in C"

Figure 1 illustrates the temperature as a function of position in the bracket.

200
2
h = 2500 W/m -K
175

150
Temperature (°C)

2
h = 250 W/m -K
125

100
2
75 h = 25 W/m -K
2
50 h = 2.5 W/m -K

25

0
0 0.005 0.01 0.015 0.02 0.025 0.03 0.035 0.04
Position (m)
Figure 1: Temperature as a function of position for various values of the heat transfer coefficient.

c.) Overlay on your plot from (b) the temperature as a function of position with h = 2.5, 25 and
2500 W/m2-K. Explain the shape of your plots.
The requested plots are shown in Figure 1. As the heat transfer coefficient increases, the
resistance between the bracket and the surrounding gas:

1
Rconv = (8)
h per L

diminishes while the resistance to conduction along the bracket:

L
Rcond = (9)
k Ac

does not change.

R_cond=L/(k*A_c) "conduction resistance from T_H to T_C"


R_conv=1/(h_bar*per*L) "convection resistance from surface to ambient"
R_cond\R_conv=R_cond/R_conv "ratio of conduction to convection resistances"

At h = 2500 W/m2-K, Rcond/Rconv = 128 and therefore the bracket material very quickly
equilibrates with the gas (see Figure 1). At h = 2.5 W/m2-K, Rcond/Rconv = 0.128 and therefore
convection is not very important and the bracket material temperature distribution is nearly linear
(i.e., the situation is close to conduction through a plane wall).

d.) Plot the heat transfer from the wall at TH into the bracket (i.e., the heat transfer into the
bracket at x = 0) as a function of h . Explain the shape of your plot.

The heat transfer into the bracket at x = 0 is:

⎛ dT ⎞
q H = −k Ac ⎜ ⎟ (10)
⎝ dx ⎠ x =0

Substituting Eq. (2) into Eq. (10) leads to:

q H = − k Ac ( m C1 − m C2 ) (11)

q_dot_H=-k*(m*C_1-m*C_2)*A_c "heat transfer rate at x=0"

Figure 2 illustrates the rate of heat transfer as a function of h and shows that as h approaches
zero the rate of heat transfer approaches a constant value, consistent with conduction through a
plane wall:

k Ac
q H ,h →0 = (TH − TC ) (12)
L
As h becomes large, the heat transfer is reduced and eventually changes sign as heat is
transferred into the wall from the warmer gas.
2.5

Heat transfer from hot wall (W)


0

-2.5

-5

-7.5

-10
1 10 100 1000
2
Average heat transfer coefficient (W/m -K)
Figure 2: Heat transfer from hot wall as a function of the heat transfer coefficient.
Problem 1.6-6
Your company has developed a technique for forming very small fins on a plastic substrate. The
diameter of the fins at their base is D = 1 mm. The ratio of the length of the fin to the base
diameter is the aspect ratio, AR = 10. The fins are arranged in a hexagonal close packed pattern.
The ratio of the distance between fin centers and to the base diameter is the pitch ratio, PR = 2.
The conductivity of the plastic material is k = 2.8 W/m-K. The heat transfer coefficient between
the surface of the plastic and the surrounding gas is h = 35 W/m2-K. The base temperature is Tb
= 60°C and the gas temperature is T∞ = 35°C. Do the analysis on a per unit of base area basis (A
= 1 m2).
a.) Determine the number of fins per unit area and the thermal resistance of the unfinned region
of the base.

The inputs are entered in EES. An aspect ratio is chosen to start the problem.

$UnitSystem SI MASS RAD PA K J


$TABSTOPS 0.2 0.4 0.6 0.8 3.5 in

"Inputs"
D=1 [mm]*convert(mm,m) "diameter"
AR=10 [-] "aspect ratio"
L=AR*D "length"
PR=2 [-] "pitch ratio"
p=PR*D "pitch (distance between fin centers)"
h_bar=35 [W/m^2-K] "heat transfer coefficient"
k=2.8 [W/m-K] "thermal conductivity"
T_b=converttemp(C,K,60 [C]) "temperature of base"
T_infinity=converttemp(C,K,35 [C]) "temperature of ambient air"
A=1 [m^2] "per unit area of base"

The Biot number is:

hD
Bi = (1)
k

Bi=h_bar*D/k "Biot number"

which leads to Bi = 0.0125, justifying the extended surface approximation. A unit cell of the
hexagonal close packed pattern is examined, as shown in Figure 1.

Acell

Figure 1: Hexagonal close pack array of fins.


The area of the triangular unit cell is:

⎛ p⎞ ⎛π ⎞1
Acell = ⎜ ⎟ p sin ⎜ ⎟ (2)
⎝2⎠ ⎝ 3⎠2

The number of fins per unit area is:

0.25
N ′′ = (3)
Acell

A_cell=(p/2)*p*sin(pi/3)/2 "area of unit cell"


N``=0.25/A_cell "number of fins per unit area"

which leads to N ′′ = 2.89x105 fins/m2. The number of fins on a 1 m2 base is:

N fin = N ′′ A (4)

The unfinned area of the base is:

D2
Aunfin = A − N fin π (5)
4

and the thermal resistance of the unfinned region is:

1
Runfin = (6)
h Aunfin

N_fin=N``*A "number of fins"


A_unfin=A-N_fin*pi*D^2/4 "unfinned surface area"
R_unfin=1/(h_bar*A_unfin) "thermal resistance of unfinned surface area"

which leads to Runfin = 0.0080 K/W.

b.) You have been asked to evaluate whether triangular, parabolic concave, or parabolic convex
pin fins will provide the best performance. Plot the heat transfer per unit area of base surface
for each of these fin shapes as a function of aspect ratio. Note that the performance of these
fins can be obtained from the EES functions eta_fin_spine_parabolic_ND,
eta_fin_spine_parabolic2_ND, and eta_fin_spine_triangular_ND. Explain the shape of your plot.

The area of the triangular fins is computed according to Eq. (7) (note that formulae for the fin
area can be found in the Help Information for the fin efficiency functions).
2
D 2 ⎛D⎞
Afin = N fin π L +⎜ ⎟ (7)
2 ⎝2⎠

The fin constant is computed according to:

4h
mL = L (8)
kD

The fin efficiency (η) is computed with the eta_fin_spine_triangular_ND.

"triangle spine fin"


A_fin=N_fin*pi*D*sqrt(L^2+(D/2)^2)/2 "finned area"
mL=sqrt(4*h_bar/(k*D))*L "fin constant"
eta=eta_fin_spine_triangular_ND(mL) "fin efficiency"

The fin resistance is computed according to:

1
R fin = (9)
h A fin η

The total resistance to heat transfer is:

−1
⎛ 1 1 ⎞
Rtotal =⎜ + ⎟⎟ (10)
⎜R
⎝ fin Runfin ⎠

and the rate of heat transfer is:

q =
(Tb − T∞ ) (11)
Rtotal

R_fin=1/(h_bar*A_fin*eta) "thermal resistance of finned surface area"


R_total=(1/R_fin+1/R_unfin)^(-1) "total thermal resistance"
q_dot=(T_b-T_infinity)/R_total "total heat transfer"

which leads to q = 3128 W/m2. Figure 2 illustrates the rate of heat transfer for triangular pin fins
as a function of aspect ratio. Note that increasing the aspect ratio increases the heat transfer
because the surface area is larger for longer fins. However, as the aspect (and therefore the
length) increases, the fins become less efficient.
5500
convex pin fin

Heat transfer per unit area (W/m )


2
5000

4500

4000

3500 triangular pin fin

3000 concave pin fin

2500

2000

1500

1000
2 4 6 8 10 12 14 16 18 20
Aspect ratio
Figure 2: Rate of heat transfer per area as a function of the aspect ratio for various fin shapes.

In order to analyze the concave parabolic fins, the formulae for the fin area, fin constant, and
efficiency are commented out and replaced with:

{"triangle spine fin"


A_fin=N_fin*pi*D*sqrt(L^2+(D/2)^2)/2 "finned area"
mL=sqrt(4*h_bar/(k*D))*L "fin constant"
eta=eta_fin_spine_triangular_ND(mL) "fin efficiency"}
"concave parabolic fin"
C_3=1+2*(D/L)^2
C_4=sqrt(1+(D/L)^2)
A_fin=N_fin*pi*L^3*(C_3*C_4-L*ln(2*D*C_4/L+C_3)/(2*D))/(8*D) "finned area"
mL=sqrt(4*h_bar/(k*D))*L "fin constant"
eta=eta_fin_spine_parabolic_ND(mL) "fin efficiency"

The performance of the concave parabolic fins is overlaid onto Figure 2. In order to analyze the
convex parabolic fins, these equations are replaced with:

{"concave parabolic fin"


C_3=1+2*(D/L)^2
C_4=sqrt(1+(D/L)^2)
A_fin=N_fin*pi*L^3*(C_3*C_4-L*ln(2*D*C_4/L+C_3)/(2*D))/(8*D) "finned area"
mL=sqrt(4*h_bar/(k*D))*L "fin constant"
eta=eta_fin_spine_parabolic_ND(mL) "fin efficiency"}
"convex parabolic"
A_fin=N_fin*pi*D^4*((4*L^2/D^2+1)^1.5-1)/(6*L^2) "finned area"
mL=sqrt(4*h_bar/(k*D))*L "fin constant"
eta=eta_fin_spine_parabolic2_ND(mL) "fin efficiency"

The performance of convex parabolic fins is also overlaid onto Figure 2. Figure 2 shows that the
convex pin fin provides the best performance; this is because this type of fin has the most surface
area.

c.) Assume that part (b) indicated that parabolic convex fins are the best. You have been asked
whether it is most useful to spend time working on techniques to improve (increase) the
aspect ratio or improve (reduce) the pitch ratio. Answer this question using a contour plot
that shows contours of the heat transfer per unit area in the parameter space of AR and PR.

The aspect ratio and pitch ratio are commented out and a parametric table is created with these
variables and the total heat transfer. A contour plot is shown in Figure 3 and shows that it is
much more beneficial to reduce the pitch ratio than the aspect ratio.
2

1.9 6451
7839
1.8
9227
1.7
10615
1.6
Pitch ratio

12003
1.5 13391
1.4 14779
1.3 16167

1.2 17555
18943
1.1

1
10 11 12 13 14 15 16 17 18 19 20
Aspect ratio
Figure 3: Contour plot of refrigeration per unit area as a function of pitch ratio and aspect ratio.
Problem 1.7-1: Furnace Manipulator Arm
You are designing a manipulator for use within a furnace. The arm must penetrate the side of the
furnace, as shown in Figure P1.7-1. The arm has a diameter of D = 0.8 cm and protrudes Li = 0.5
m into the furnace, terminating in the actuator that can be assumed to be adiabatic. The portion
of the arm in the manipulator is exposed to flame and hot gas; these effects can be represented by
a heat flux of q ′′ = 1x104 W/m 2 and convection to gas at Tf = 500°C with heat transfer coefficient
h f = 50 W/m2-K. The conductivity of the arm material is k = 150 W/m-K. The arm outside of
the furnace has the same diameter and conductivity, but is exposed to air at Ta = 20°C with heat
transfer coefficient ha = 30 W/m2-K. The length of the arm outside of the furnace is Lo = 0.75 m
and this portion of the arm terminates in the motor system which can also be considered to be
adiabatic.

′′ = 1x104 W/m 2
qrad
k = 150 W/m-K

x D = 0.8 cm

Lo = 0.75 m
Li = 0.5 m
h f = 50 W/m -K ha = 30 W/m -K
2 2

=
T f 500 C° Ta
= 20°C
Figure P1.7-1: Manipulator arm for a furnace.

a.) Is an extended surface model appropriate for this problem? Justify your answer.

The inputs are entered in EES:

$UnitSystem SI MASS RAD PA K J


$TABSTOPS 0.2 0.4 0.6 0.8 3.5 in

"Inputs"
qf_rad=1e4 [W/m^2] "radiant heat flux on arm"
T_f=converttemp(C,K,500) "air temperature within furnace"
h_f=50 [W/m^2-K]
"heat transfer coefficient within furnace"
L_i=0.5 [m] "manipulator arm length within furnace"
D=0.8 [cm]*convert(cm,m) "diameter of arm"
k=150 [W/m-K] "arm conductivity"
T_a=converttemp(C,K,20) "air temperature outside of furnace"
h_a=30 [W/m^2-K]
"heat transfer coefficient outside of furnace"
L_o=0.75 [m]
"manipulator arm length outside of furnace"

The Biot number based on the heat transfer coefficient within the furnace will be largest because
the highest heat transfer coefficient exists within the furnace:
D hf
Bii = (1)
2k

Bi_i=(D/2)*h_f/k "Biot number based on internal heat transfer coefficient"

which leads to Bii = 0.013 which is sufficiently less than unity to allow an extended surface
approximation model to be used.

b.) Develop an analytical model of the manipulator arm; implement your model in EES. Plot the
temperature as a function of axial position x (see Figure P1.7-1) for -Lo < x < Li.

The differential equation that governs the temperature within the furnace (Ti) is derived using the
differential energy balance shown in Figure 2.

Figure 2: Differential energy balance on manipulator arm within furnace.

The energy balance suggested by Figure 2 is:

dq
′′ π D dx = q x +
q x + qrad dx + h f π D (Ti − T f ) dx (2)
dx

Substituting Fourier's law:

π D 2 dTi
q x = −k (3)
4 dx

into Eq. (2) leads to:

d ⎡ π D 2 dTi ⎤
′′ π D dx =
qrad ⎢ −k ⎥ dx + h f π D (Ti − T f ) dx (4)
dx ⎣ 4 dx ⎦

or
d 2Ti 4 h f 4 hf 4 q ′′
2
− Ti = − T f − rad (5)
dx kD kD kD

The solution to the ordinary differential equation is divided into its homogeneous (ui) and
particular (vi) parts:

Ti = ui + vi (6)

and substituted into Eq. (5) in order to obtain:

d 2ui 4 h f d 2 vi 4 h f 4h 4 q ′′
2
− ui + 2
− vi = − f T f − rad (7)
dx kD dx kD kD kD



homogeneous equation particular equation

The solution to the homogeneous equation:

d 2ui 4 h f
− ui = 0 (8)
dx 2 k D

is

ui = C1 exp ( mi x ) + C2 exp ( − mi x ) (9)

where

4 hf
mi = (10)
kD

The solution to the particular equation:

d 2 vi 4 h f 4h 4 q ′′
2
− vi = − f T f − rad (11)
dx kD kD kD

is

′′
qrad
vi = T f + (12)
hf

So the general solution for Ti is:


′′
qrad
Ti = C1 exp ( mi x ) + C2 exp ( − mi x ) + T f + (13)
hf

A similar set of steps leads to the general solution for the temperature outside of the furnace, To,
which is valid for -Lo < x < 0:

To = C3 exp ( mo x ) + C4 exp ( − mo x ) + Ta (14)

where

4 ha
mo = (15)
kD

The fin constants, mi and mo, are computed:

m_i=sqrt(4*h_f/(k*D)) "fin constant inside furnace"


m_o=sqrt(4*h_a/(k*D)) "fin constant outside of furnace"

The boundary conditions for the solution must be obtained at the edges of each of the
computational domains (i.e., at x= -Lo, x=0, and x=Li).

The two ends of the arm are adiabatic; therefore:

dTi
=0 (16)
dx x = Li

dTo
=0 (17)
dx x =− Lo

The temperature at the interface between the two computational domains must be continuous:

Ti , x =0 = To , x =0 (18)

Also, the rate of energy transferred from the furnace to the ambient air must be the same
regardless of which side of the interface between the computational domains you are on; that is,
an interface balance at x = 0 between the two computational domains leads to:

dTi dTo
= (19)
dx x =0 dx x =0
Substituting Eqs. (13) and (14) into Eqs. (16) through (19) leads to 4 equations for the constants
of integration C1 and C4:

C1 mi exp ( mi Li ) − C2 mi exp ( −mi Li ) = 0 (20)

C3 mo exp ( − mo Lo ) − C4 mo exp ( mo Lo ) (21)

′′
qrad
C1 + C2 + T f + = C3 + C4 + Ta (22)
hf
C1 mi − C2 mi = C3 mo − C4 mo (23)

These are entered in EES:

C_1*m_i*exp(m_i*L_i)-C_2*m_i*exp(-m_i*L_i)=0 "adiabatic end at L_i"


C_3*m_o*exp(-m_o*L_o)-C_4*m_o*exp(m_o*L_o)=0 "adiabatic end at -L_o"
C_1+C_2+T_f+qf_rad/h_f=C_3+C_4+T_a "continuity of temperatures at x=0"
C_1*m_i-C_2*m_i=C_3*m_o-C_4*m_o "energy balance at x=0"

The solutions are entered; note that the variable x_bar is varied from 0 to 1 within a parametric
table which corresponds to x_i going from 0 to L_i and x_o going from 0 to -L_o.

x_i=x_bar*L_i
x_o=-x_bar*L_o
T_i=C_1*exp(m_i*x_i)+C_2*exp(-m_i*x_i)+T_f+qf_rad/h_f
T_o=C_3*exp(m_o*x_o)+C_4*exp(-m_o*x_o)+T_a

The temperature distribution in the arm is shown in Figure 3.

Figure 3: Temperature distribution.


c.) Prepare a plot showing the maximum temperature at the end of the arm (within the furnace)
as a function of the internal length of the arm (Li) for various values of the diameter (D).

Figure 4: Maximum temperature as a function of the arm length within the furnace for various values of the
diameter.
Problem 1.7-2: Rotating Ring
Figure P1.7-2 illustrates a metal ring of radius R = 5.0 cm that is rotating with an angular
velocity ω = 0.1 rad/s. During each rotation, the ring material passes from compartment #1
containing hot fluid at Tf,1 = 200ºC to compartment #2 containing fluid at Tf,2 = 20ºC. The heat
transfer coefficient between the ring surface and the fluid in compartments #1 and #2 are h1 = 10
W/m2-K and h2 = 20 W/m2-K, respectively. The ring has a circular cross-section with diameter
d = 1.0 mm. For this problem you can assume that d/R<<1. The metal has conductivity k = 100
W/m-K, density ρ = 2700 kg/m3, and specific heat capacity c = 900 J/kg-K.
compartment #1
T f ,1 = 200°C
h1 = 10 W/m -K
2

R = 5 cm
ω = 0.1 ra d/s

d = 1 mm

ρ = 2700 kg/m 3 compartment #2


c = 900 J /kg-K T f ,2 = 20°C
h2 = 20 W/m -K
2
k = 100 W/m-K
Figure P1.7-2: Metal ring rotates between two compartments.

a.) Is it appropriate to treat the ring as an extended surface?

The input parameters are entered into EES.

$UnitSystem SI MASS RAD PA K J


$Tabstops 0.2 0.4 0.6 3.5 in

"Inputs"
T_f_1=converttemp(C,K,200 [C]) "temperature of fluid in compartment #1"
T_f_2=converttemp(C,K,20 [C]) "temperature of fluid in compartment #2"
h_1=10 [W/m^2-K] "heat transfer coefficient in compartment #1"
h_2=20 [W/m^2-K] "heat transfer coefficient in compartment #2"
R=5.0 [cm]*convert(cm,m) "radius of ring"
d=1.0 [mm]*convert(mm,m) "diameter of ring"
omega=0.1 [rad/s] "rate of ring rotation"
rho=2700 [kg/m^3] "density"
c=900 [J/kg-K] "specific heat capacity"
k=100 [W/m-K] "thermal conductivity"

The Biot number associated with the ring in compartment #2 will be the largest (the heat transfer
coefficient is larger in compartment #2) and therefore limit the applicability of the extended
surface model. An appropriate Biot number is:
h2 d
Bi =
2k

which is calculated in EES:

"Biot number calculation"


Bi=h_2*d/(2*k) "Biot number in compartment #2"

and found to be 0.0001; clearly there will be almost no temperature gradient within the ring
between the center and the surface. The temperature of the ring can be considered to be a
function only of the linear coordinate s that follows the circumference of the ring (see Figure 1).

b.) Develop an analytical model that can predict the temperature distribution in the ring.

A differential control volume for the ring is considered in Figure 2.

qconv

qcond ,s
qcond , s + ds

E t , s E t , s + ds

Figure 2: Differential control volume for ring material.

The energy balance for the differential control volume is:

dq dE
qcond , s + E t , s = qconv + qcond , s + cond ds + E t , s + t ds
ds ds

where qcond , qconv , and Et are the energy transfers due to conduction, convection, and the motion
of the material, respectively; the rate equations for these terms in compartment #1 are:

dT1
qcond = − k Ac
ds

qconv = p h1 ds (T1 − T f ,1 )

E t = ω R Ac ρ cT1
where Ac and p are the cross-sectional area and perimeter of the ring:

π d2
Ac =
4

p =π d

Combining these equations leads to:

d ⎡ dT ⎤
0 = p h1 ds (T1 − T f ,1 ) +
d
⎢ −k Ac 1 ⎥ ds + [ω R Ac ρ cT1 ] ds
ds ⎣ ds ⎦ ds

which can be simplified:

d 2T1
k Ac 2 − ω R Ac ρ c 1 − p h1 (T1 − T f ,1 ) = 0
dT
ds ds

or

d 2θ1 ω R dθ1
− − m12 θ1 = 0 (1)
ds 2 α ds

where α is the thermal diffusivity of the ring material and m1 is the fin parameter for
compartment #1:

p h1
m1 =
k Ac

and θ1 is the ring-to-fluid temperature difference in compartment #1:

θ1 = T1 − T f ,1

Equation (1) is the governing differential equation for the ring material in compartment #1 and
therefore applies from s = πR to s = 2πR. A similar set of steps leads to a governing differential
equation for the temperature of the ring material in compartment #2, T2, that is valid from s = 0
to s = πR:

d 2θ 2 ω R dθ 2
− − m22 θ 2 = 0 (2)
ds 2
α ds

where m2 is the fin parameter for compartment #2:


p h2
m2 =
k Ac

and θ2 is the ring-to-fluid temperature difference in compartment #2:

θ 2 = T2 − T f ,2

The general solutions to the governing differential equations can be obtained using Maple or
simply copied from Section 1.7.3. In compartment #1 the solution is:

T1 = T f ,1 + C1 exp ( λ1 s ) + C2 exp ( λ2 s ) (3)

where

ωR 1⎛ω R⎞
2

λ1 = + ⎜ ⎟ + m1
2

2α 4⎝ α ⎠

ωR 1⎛ω R ⎞
2

λ2 = − ⎜ ⎟ + m1
2

2α 4⎝ α ⎠

and in compartment #2 the solution is:

T2 = T f ,2 + C3 exp ( β1 s ) + C4 exp ( β 2 s ) (4)

where

ωR 1⎛ω R⎞
2

β1 = + ⎜ ⎟ + m2
2

2α 4⎝ α ⎠

ωR 1⎛ω R⎞
2

β2 = − ⎜ ⎟ + m2
2

2α 4⎝ α ⎠

These equations are entered in EES but they cannot be solved until the 4 undetermined constants,
C1 through C4, are determined using the boundary conditions:

"Solution Parameters"
p=pi*d "Perimeter of ring"
A_c=pi*d^2/4 "Cross-sectional area of ring"
m_1=sqrt(p*h_1/(k*A_c)) "Fin constant in compartment #1"
m_2=sqrt(p*h_2/(k*A_c)) "Fin constant in compartment #2"
alpha=k/(rho*c) "Thermal diffusivity of ring material"
lambda_1=omega*R/(2*alpha)+sqrt((omega*R/alpha)^2/4+m_1^2) "solution constants"
lambda_2=omega*R/(2*alpha)-sqrt((omega*R/alpha)^2/4+m_1^2)
beta_1=omega*R/(2*alpha)+sqrt((omega*R/alpha)^2/4+m_2^2)
beta_2=omega*R/(2*alpha)-sqrt((omega*R/alpha)^2/4+m_2^2)

"Solution"
theta_1=C_1*exp(lambda_1*s)+C_2*exp(lambda_2*s) "ring-to-fluid temp. dif. in comp. #1"
T_1=T_f_1+theta_1 "ring temperature in comp. #1"
theta_2=C_3*exp(beta_1*s)+C_4*exp(beta_2*s) "ring-to-fluid temp. dif. in comp. #2"
T_2=T_f_2+theta_2 "ring temperature in comp. #2"

There are 4 constants (2 for each 2nd order differential equation) and therefore 4 boundary
conditions are required to provide a solution. There can be no discontinuity in the temperature at
the interfaces of the compartments; therefore, the temperature of the material leaving
compartment #1 is the same as the temperature of the material entering compartment #2. The
same requirement applies for the material leaving compartment #2 and entering compartment #1.

T1 s =π R = T2 s =π R
(5)

T1 s = 2π R = T2 s =0
(6)

which leads to:

T f ,1 + C1 exp ( λ1 π R ) + C2 exp ( λ2 π R ) = T f ,2 + C3 exp ( β1 π R ) + C4 exp ( β 2 π R )

T f ,1 + C1 exp ( λ1 2 π R ) + C2 exp ( λ2 2 π R ) = T f ,2 + C3 + C4

These boundary conditions are entered in EES:

"Boundary Conditions"
"Temperature continuity"
T_f_1+C_1*exp(lambda_1*pi*R)+C_2*exp(lambda_2*pi*R)=T_f_2+C_3*exp(beta_1*pi*R)+C_4*exp(beta
_2*pi*R)
T_f_1+C_1*exp(lambda_1*2*pi*R)+C_2*exp(lambda_2*2*pi*R)=T_f_2+C_3+C_4

The remaining boundary conditions are determined by an energy balance at the interfaces
between the compartments. The rate of energy leaving compartment #2 at s = πR must be equal
to the rate of energy entering compartment #1 at s = πR and also at s = 0 in compartment #2 and
s = 2πR in compartment #1. Energy is transferred through the ring due to motion of ring
material as well as conduction; therefore, these energy balances become:

dT1 dT2
ω R Ac ρ cT1 s =π R − k A = ω R A ρ cT2 s =π R
−k A
ds s =π R ds s =π R

Equation (5) results in the 1st term on either side canceling. Since the temperature at the
interface is continuous, the rate of energy transfer by ring motion must be the same at the
interface regardless if it is determined from compartment #1 or compartment #2. Therefore, the
temperature gradient at the interface must be continuous:

dT1 dT2
= (7)
ds s =π R ds s =π R

which leads to:

C1 λ1 exp ( λ1 π R ) + C2 λ2 exp ( λ2 π R ) = C3 β1 exp ( β1 π R ) + C4 β 2 exp ( β 2 π R )

The temperature gradient at the other interface must also be continuous:

dT1 dT2
= (8)
ds s = 2π R ds s =0

which leads to:

C1 λ1 exp ( λ1 2 π R ) + C2 λ2 exp ( λ2 2 π R ) = C3 β1 + C4 β 2

These boundary conditions are entered in EES:

"Temperature gradient continuity"


C_1*lambda_1*exp(lambda_1*pi*R)+C_2*lambda_2*exp(lambda_2*pi*R)=C_3*beta_1*exp(beta_1*pi*R)
+C_4*beta_2*exp(beta_2*pi*R)
C_1*lambda_1*exp(lambda_1*2*pi*R)+C_2*lambda_2*exp(lambda_2*2*pi*R)=C_3*beta_1+C_4*beta_2

If the parameter s is set to some value (e.g., s = 0) then there are an equal number of equations
and unknowns and it should be possible obtain the unknown constants; however, it is likely that
solving the problem (Calculate/Solve) will lead to the Error dialog shown in Figure 3.

Figure 3: Error dialog.

The problem is that the initial guess values for the constants are sufficiently far from the actual
values that the process of simultaneously solving the 4 non-linear coupled equations associated
with the boundary conditions does not converge. Select Yes in the Error dialog to view the
residuals, Figure 4. The Residuals window provides the most useful information available in the
event that a calculation is not successful. The first column in the Residuals window indicates the
Block number; a Block is a set of equations that must be solved together because they are
dependent. Equations that are designated to be in block 0 are equations that contain a single
unknown variable. The variable that is being solved for is shown in bold font. The equations in
block 0 are solved in the order that they appear in the Residuals window, regardless of the order
that they were entered in the Equations window. Also shown in the Residuals window are the
relative and absolute values of the residuals for each equation. The absolute residual is the
difference between the left and right side of the equations. The relative residual is the ratio of
the absolute residual to the value terms on the left side of the equations (assuming that they are
not identically zero). If the equation is properly solved, the residual should be zero, but it may
be some small value depending on the stop criteria tolerances that have been set.

Figure 4: Residuals window.

Scanning down the Residuals window in Figure 4, it can be seen that the equations in block 1
were not solved correctly. Note that there are four equations in this block, representing a system
of four equations and four unknowns, C_1 through C_4. (Note that part of the equations in the
figure shown below have been cut off.) These four equations must be solved simultaneously and
that is why EES has placed them in the same block. Armed with the knowledge of where the
calculation is failing, it is often possible to provide better guess values and/or limits in order to
make the calculation succeed. The Variable information window (Options/Variable Info)
provides information about the guess value and limits for each variable (Figure 5); the default
guess value for every variable is 1 and the limits are from –infinity to infinity.
Figure 5: Variable Information window

Approximate values of the constants and their limits are not obvious for this problem, which
makes it difficult to set more appropriate values. One technique for proceeding is simply to set
the constants to a much larger or smaller value and see if that helps the problem to converge. A
better technique might be to divide the problem into several smaller ones. For example, it is
possible to consider only compartment #1 and set the constants so that the temperatures at the
boundaries are equal to the fluid temperature in compartment #2. This is easily done by
commenting out the 2nd set of boundary conditions and the right hand side of the 1st set (do not
delete these boundary conditions as you will need them later.):

"Boundary Conditions"
"Temperature continuity"
T_f_1+C_1*exp(lambda_1*pi*R)+C_2*exp(lambda_2*pi*R)=T_f_2{+C_3*exp(beta_1*pi*R)+C_4*exp(beta
_2*pi*R)}
T_f_1+C_1*exp(lambda_1*2*pi*R)+C_2*exp(lambda_2*2*pi*R)=T_f_2{+C_3+C_4}
"Temperature gradient continuity"
{C_1*lambda_1*exp(lambda_1*pi*R)+C_2*lambda_2*exp(lambda_2*pi*R)=C_3*beta_1*exp(beta_1*pi*R)
+C_4*beta_2*exp(beta_2*pi*R)
C_1*lambda_1*exp(lambda_1*2*pi*R)+C_2*lambda_2*exp(lambda_2*2*pi*R)=C_3*beta_1+C_4*beta_2}

The new problem will solve, providing reasonable values for C_1 and C_2; now update the guess
values for all of the variables (Calculate/Update Guess Values) and the variable information
window will be updated to reflect the most recent values of each variable (Figure 6). Notice that,
as we suspected, the values for C_1 and C_2 are very large and very small, respectively.
Therefore a guess value of 1 was not appropriate for either variable.
Figure 6: Variable Information window after updating guess values.

Uncomment the boundary conditions and solve the problem again; because the guess values for
C_1 and C_2 are more appropriate, the equations will now solve with no problem. The solution
can be plotted by making two parametric tables, the first with the variables s_bar, s and T_1_C
(the solution in compartment #1 converted from K to °C for presentation) where s_bar is runs
from 0 to 1 and the second with s_bar, s and T_2_C where s_bar runs from 1 to 2; the variable
s_bar is defined as a dimensionless position.

s_bar=0 "dimensionless position along ring"


s=s_bar*pi*R "position along ring"
T_1_C=converttemp(K,C,T_1)
T_2_C=converttemp(K,C,T_2)

c.) Plot the temperature distribution in the ring as a function of position along the circumference
of the ring for various values of the ring rotation rate.

The temperature distribution through the ring is shown in Figure 7. Also shown in Figure 7 is
the temperature distribution for several values of ring rotation rate. Notice that the solution for
very low rotation limits to a pair of interacting, adiabatic tip fins and the symmetric nature of the
resulting temperature distribution reflects this behavior. This solution represents a balance
between conduction into the ring at the interface and convection to the surrounding fluids. As
the rotation rate is increased the lines become progressively flatter. The limit of very high
rotation rate would be a flat line; the energy transferred by ring rotation is sufficiently large that
no temperature gradient is required. The ring would reach a uniform temperature where
convection into the ring in the hot compartment is balanced by convection out in the cold
compartment.
Figure 7: Temperature distribution as a function of dimensionless position for ω = 0.1 rad/s and ω = 0 rad/s.

d.) Compute the total rate of energy transferred between the two compartments.

The total rate of energy transferred between the compartments ( E , the energy transferred from
compartment #2 to compartment #1) is related to conduction and energy carried by ring material
motion at the two interfaces.

⎡ ⎤
E = ω R Ac ρ c (T2 ) − k A ⎢ dT dT2
s =π R
− T2 s =0 c
2
− ⎥

dx s =π R dx s =0 ⎦

or

{
E = ω R Ac ρ c C3 ⎡⎣exp ( β1 π R ) − 1⎤⎦ + C4 ⎡⎣exp ( β 2 π R ) − 1⎤⎦ }
{
− k Ac C3 β1 ⎡⎣exp ( β1 π R ) − 1⎤⎦ + C4 β 2 ⎡⎣exp ( β 2 π R ) − 1⎤⎦ }
Figure 8 illustrates the rate of energy carried from compartment #2 to compartment #1 (the
energy transferred is negative as the fluid in compartment #2 is colder). Notice that the two
regimes of behavior previously discussed in the context of Figure 7 are evident; at low rates of
rotation the energy is carried between compartments by conduction whereas at high rates of
rotation energy is carried by ring motion.
Figure 8: Energy transferred from compartment #2 to compartment #1 as a function of the rate of
ring rotation.

e.) Determine the dimensionless parameter that characterizes the relative importance of energy
transfer due to ring rotation and energy transfer by conduction. Plot the heat transfer rate
between the two compartments as a function of this parameter (varied by changing the ring
rotation rate).

The energy transferred between the compartments is ultimately transferred from/to the fluid by
convection. However, the method of transfer between the compartments is both conduction and
energy carried by ring rotation; one of these processes will typically be dominant. The ratio of
the energy transfer caused by rotation to conduction can be used to form a governing
dimensionless parameter that helps understand the problem and guide the solution. The use of
dimensionless parameters based on the underlying physics is an important tool that allows many
problems to be simplified and provides substantial insight into the characteristics of a solution.
For this problem, the dimensionless parameter is:

rotation ω R A ρ c (T f ,1 − T f ,2 ) R ω R2
= =
conduction 1 k A (T f ,1 − T f ,2 ) α

It would have been best to calculate the value of the dimensionless parameter BEFORE we
solved the problem. If ωR2/α is much less than unity (even if ω itself is not zero) then ring
rotation can be neglected as it is dominated by conduction. On the other hand, if ωR2/α is much
larger than unity (even if k is not zero) then the conduction terms in the governing equations
could have been neglected and the problem would be simplified at least to solving 1st order rather
than 2nd order equations. Figure 9 shows the energy transfer rate as a function of ω R2/α and
demonstrates that the delineation between the regimes of behavior is unity for the appropriately
defined dimensionless number.
Figure 9: Energy transferred from compartment #2 to compartment #1 as a function of the
dimensionless number ωR2/α which characterizes the rate of energy carried by ring motion
to conduction.
Problem 1.7-3 (1-15 in text): Material Processing
Figure P1.7-3 illustrates a material processing system.

oven wall temperature varies with x

gap filled with gas


th = 0.6 mm
kg = 0.03 W/m-K
u = 0.75 m/s
Tin = 300 K
D = 5 cm
x

extruded material
k = 40 W/m-K
α = 0.001 m2/s
Figure P1.7-3: Material processing system.

Material is extruded and enters the oven at Tin = 300 K with velocity u = 0.75 m/s. The material
has velocity u = 0.75 m/s and diameter D = 5 cm. The conductivity of the material is k = 40
W/m-K and the thermal diffusivity is α = 0.001 m2/s.

In order to precisely control the temperature of the material, the oven wall is placed very close to
the outer diameter of the extruded material and the oven wall temperature distribution is
carefully controlled. The gap between the oven wall and the material is th = 0.6 mm and the
oven-to-material gap is filled with gas that has conductivity kg = 0.03 W/m-K. Radiation can be
neglected in favor of convection through the gas from the oven wall to the material. For this
situation, the heat flux experienced by the material surface can be approximately modeled
according to:

kg
′′ ≈
qconv (Tw − T )
th

where Tw and T are the oven wall and material temperatures at that position. The oven wall
temperature varies with position x according to:

⎛ x ⎞
Tw = T f − (T f − Tw,0 ) exp ⎜ − ⎟
⎝ Lc ⎠

where Tw,0 is the temperature of the wall at the inlet (at x = 0), Tf = 1000 K is the temperature of
the wall far from the inlet, and Lc is a characteristic length that dictates how quickly the oven
wall temperature approaches Tf. Initially, assume that Tw,0 = 500 K, Tf = 1000 K, and Lc = 1 m.
Assume that the oven can be approximated as being infinitely long.
a.) Is an extended surface model appropriate for this problem?

The inputs are entered in EES:

$UnitSystem SI MASS DEG PA C J


$Tabstops 0.2 0.4 0.6 0.8 3.5

k=40 [W/m-K] "conductivity"


u=0.75 [m/s] "velocity"
T_f=1000 [K] "wall temperature far from the inlet"
T_w_0=500 [K] "wall temperature at the inlet"
L_c=1 [m] "characteristic length which oven wall approaches T_f"
T_in=300 [K] "inlet temperature"
alpha=0.001 [m^2/s] "thermal diffusivity"
k_g=0.03 [W/m-K] "gas conductivity"
th=0.6 [mm]*convert(mm,m) "oven-to-material gap thickness"
D=5 [cm]*convert(cm,m) "diameter"

The Biot number is the ratio of the resistance that is neglected (internal conduction) to the
resistance that is considered (conduction across the gap):

kg D
Bi = (1)
th 2 k

Bi=(k_g/th)*D/(2*k) "Biot number"

which leads to Bi = 0.031. This is sufficiently less than 1 to justify an extended surface model.

b.) Assume that your answer to (a) was yes. Develop an analytical solution that can be used to
predict the temperature of the material as a function of x.

An energy balance on a control volume differential for a differential (in x) segment of the
material is shown in Figure P1.7-3-2.

kg
per dx (T − Tw )
th
(ρ u Ac cT )x (ρ u Ac cT )x+dx

⎛ dT ⎞ ⎛ dT ⎞
⎜ −k Ac ⎟ ⎜ −k Ac ⎟
⎝ dx ⎠ x dx ⎝ dx ⎠ x+ dx
Figure P1.7-3-2: Energy balance on a differential control volume.

The energy balance suggested by Figure P1.7-3-2 is:

( ρ u Ac cT ) x + ⎛⎜ −k Ac
dT ⎞ ⎛ dT ⎞ k
⎟ = ( ρ u Ac cT ) x + dx + ⎜ − k Ac ⎟ + g per dx (T − Tw ) (2)
⎝ dx ⎠ x ⎝ dx ⎠ x + dx th

where c is the specific heat capacity, Ac is the cross-sectional area and per is the perimeter of the
material:
D2
Ac = π (3)
4

per = π D (4)

A_c=pi*D^2/4 "cross-sectional area"


per=pi*D "perimeter"

Expanding the terms in Eq. (2) and simplifying:

dT d 2T k
0 = ρ u Ac c − k Ac 2 + g per (T − Tw ) (5)
dx dx th

Rearranging Eq. (5) and dividing through by k Ac leads to:

d 2T u dT k g per k per
− − T =− g Tw (6)
dx 2
α dx th k Ac th k Ac

Substituting the wall temperature variation into Eq. (6) leads to:

d 2T u dT ⎡ ⎛ x ⎞⎤
− − m 2 T = − m 2 ⎢T f − (T f − Tw,0 ) exp ⎜ − ⎟⎥ (7)
dx 2
α dx ⎣ ⎝ Lc ⎠⎦

where

k g per
m= (8)
th k Ac

m=sqrt(4*k_g/(th*k*D)) "fin parameter"

The boundary conditions are the inlet temperature:

Tx =0 = Tin (9)

and the temperature must approach Tf as x approaches infinity:

Tx →∞ = T f (10)

The solution is broken into a homogeneous and particular component:

T = Th + Tp (11)

and substituted into Eq. (7):


d 2Th u dTh d 2Tp u dTp ⎡ ⎛ x ⎞⎤

dx 2 α
− m 2
Th + −
dx 2 α dx
− m 2
T p = − m 2
⎢T f − ( T f − Tw ,0 ) exp ⎜ − ⎟⎥ (12)
 dx
 ⎣
⎝ Lc

⎠⎦
homogeneous ordinary
differential equation particular ordinary differential equation

The solution to the homogeneous differential equation is:

⎡⎛ u + u 2 + 4 α 2 m 2 ⎞ ⎤ ⎡⎛ u − u 2 + 4 α 2 m 2 ⎞ ⎤
Th = C1 exp ⎢⎜ ⎟ x ⎥ + C2 exp ⎢⎜ ⎟ x⎥ (13)
⎢⎣⎜⎝ 2α ⎟ ⎥
⎠ ⎦ ⎢⎣⎜⎝ 2α ⎟ ⎥
⎠ ⎦

The particular solution is obtained by the method of undetermined coefficients; the assumed
form of the particular solution is:

⎛ x ⎞
Tp = C3 exp ⎜ - ⎟ + C4 (14)
⎝ Lc ⎠

and substituted into the particular differential equation:

⎛ x ⎞ ⎛ x ⎞ ⎛ x⎞ ⎡ ⎛ x ⎞⎤
C3 exp ⎜ - ⎟ − m 2C3 exp ⎜ - ⎟ − m 2 C4 = −m 2 ⎢T f − (T f − Tw,0 ) exp ⎜ − ⎟ ⎥
C3 u
exp ⎜ - ⎟+
⎠ α Lc
2
Lc ⎝ Lc ⎝ Lc ⎠ ⎝ Lc ⎠ ⎣ ⎝ Lc ⎠ ⎦
(15)

Equation (15) provides one equation for C3 that is obtained by considering the exponential terms:

m 2 (T f − Tw,0 )
C3 = (16)
⎛ 1 u ⎞
⎜ 2+ − m2 ⎟
⎝ Lc α Lc ⎠

and another equation for C4 that is obtained by considering the constant terms:

C4 = T f (17)

Substituting Eqs. (13), (14), (16), and (17) leads to:

⎡⎛ u + u 2 + 4 α 2 m 2 ⎞ ⎤ ⎡⎛ u − u 2 + 4 α 2 m 2 ⎞ ⎤

T = C1 exp ⎜ ⎥
⎟ x + C2 exp ⎜ ⎢ ⎟ x⎥
⎢⎣⎜⎝ 2α ⎟ ⎥
⎠ ⎦ ⎢⎣⎜⎝ 2α ⎟ ⎥
⎠ ⎦
m 2 (T f − Tw,0 ) ⎛ x ⎞ (18)
+ exp ⎜ - ⎟ + T f
⎛ 1 u ⎞ ⎝ Lc ⎠
⎜ 2+ − m2 ⎟
⎝ Lc α Lc ⎠
The constants C1 and C2 are obtained by considering the boundary conditions. Substituting Eq.
(18) into Eq. (10) leads to:

⎡⎛ u + u 2 + 4 α 2 m 2 ⎞ ⎤
C1 exp ⎢⎜ ⎟ ∞⎥ + Tf = Tf (19)
⎢⎣⎜⎝ 2α ⎟ ⎥
⎠ ⎦

which can only be true if C1 = 0. Therefore:

⎡⎛ u − u 2 + 4 α 2 m 2 ⎞ ⎤ m2 (T f − Tw,0 ) ⎛ x ⎞
T = C2 exp ⎢⎜ ⎥
⎟x + exp ⎜ - ⎟ + T f (20)
⎢⎣⎜⎝ 2α ⎟ ⎥ ⎛ 1
⎠ ⎦ ⎜ + u − m 2 ⎞⎟ ⎝ Lc ⎠
⎝ Lc α Lc
2

Substituting Eq. (20) into Eq. (9) leads to:

m 2 (T f − Tw,0 )
C2 + + T f = Tin (21)
⎛ 1 u ⎞
⎜ 2+ − m2 ⎟
⎝ Lc α Lc ⎠

or

m 2 (T f − Tw,0 )
C2 = Tin − T f − (22)
⎛ 1 u ⎞
⎜ 2+ − m2 ⎟
⎝ Lc α Lc ⎠

C_2=T_in-T_f-m^2*(T_f-T_w_0)/(1/L_c^2+u/(alpha*L_c)-m^2) "boundary condition at x=0"

The solution for the material temperature and the wall temperature are entered in EES:

x=0.5 [m] "position"


T=C_2*exp(((u-sqrt(u^2+4*alpha^2*m^2))/(2*alpha))*x)+m^2*(T_f-T_w_0)*&
exp(-x/L_c)/(1/L_c^2+u/(alpha*L_c)-m^2)+T_f "temperature of the material"
T_w=T_f-(T_f-T_w_0)*exp(-x/L_c) "wall temperature"

c.) Plot the temperature of the material and the temperature of the wall as a function of position
for 0 < x < 20 m. Plot the temperature gradient experienced by the material as a function of
position for 0 < x < 20 m.

Figure P1.7-3-3 illustrates the temperature of the material and the wall as a function of position.
1000

900
wall

Temperature (K)
800

700
material
600

500

400

300
0 2.5 5 7.5 10 12.5 15 17.5 20
Position (m)
Figure P1.7-3-3: Temperature of the material and the wall as a function of position.

The temperature gradient is evaluated by differentiating Eq. (20):

dT ⎛ u − u 2 + 4 α 2 m2 ⎞ ⎡⎛ u − u 2 + 4 α 2 m 2 ⎞ ⎤ m 2 (T f − Tw,0 ) ⎛ x ⎞
= C2 ⎜ ⎟ exp ⎢⎜ ⎟ x⎥ − exp ⎜ - ⎟ (23)
⎜ 2α ⎟ ⎜ 2α ⎟
⎣⎢⎝ ⎠ ⎦⎥ Lc ⎛⎜ 1 + u − m 2 ⎞⎟ ⎝ Lc ⎠
dx
⎝ ⎠
⎝ Lc α Lc
2

dTdx=C_2*((u-sqrt(u^2+4*alpha^2*m^2))/(2*alpha))*exp(((u-sqrt(u^2+4*alpha^2*m^2))/(2*alpha))*x)&
-m^2*(T_f-T_w_0)*exp(-x/L_c)/(1/L_c^2+u/(alpha*L_c)-m^2)/L_c "temperature gradient"

Figure P1.7-3-4 illustrates the temperature gradient as a function of position.


70

60
Temperature gradient (K/m)

50

40

30

20

10

0
0 2.5 5 7.5 10 12.5 15 17.5 20
Position (m)
Figure P1.7-3-4: Temperature gradient in the material as a function of position.

The parameter Lc can be controlled in order to control the maximum temperature gradient
experienced by the material as it moves through the oven.

d.) Prepare a plot showing the maximum temperature gradient as a function of Lc. Overlay on
your plot the distance required to heat the material to Tp = 800 K (Lp). If the maximum
temperature gradient that is allowed is 60 K/m then what is the appropriate value of Lc and
the corresponding value of Lp.

The value Lp is obtained:


T_p=800 [K]
T_p=C_2*exp(((u-sqrt(u^2+4*alpha^2*m^2))/(2*alpha))*L_p)+&
m^2*(T_f-T_w_0)*exp(-L_p/L_c)/(1/L_c^2+u/(alpha*L_c)-m^2)+T_f

which leads to Lp = 10.18 m.

The maximum temperature gradient can be obtained by using EES' optimization routines. Setup
a parametric table that includes the variables L_c, x, dTdx, L_p, and L_c. The value of L_c that
is set in the Equations window is commented out and the values of L_c in the table are varied
from 0.1 to 5 m. Min/Max Table is selected from the Calculate menu. The value of dTdx is
maximized by varying x with bounds from 0 to some large value. The maximum temperature
gradient and value of Lp are shown Figure P1.7-3-5 as a function of Lc. Figure P1.7-3-5 indicates
that Lc should be equal to 1.8 m in order to control the temperature gradient, which leads to Lp =
11 m.
90 14
Maximum temperature gradient (K/m)

Position at which T = 800 K


80 maximum temperature gradient 13

70 12

60 11

50 10
distance at which T = 800 K

40 9
0 1 2 3 4 5
Lc (m)
Figure P1.7-3-5: Maximum temperature gradient and Lp as a function of Lc.
Problem 1.7-4 (1-16 in text): Solar Collector Tube
The receiver tube of a concentrating solar collector is shown in Figure P1.7-4.

Ta = 25°C
qs′′ ha = 25 W/m -K
2

r = 5 cm
th = 2.5 mm
k = 10 W/m-K

φ
Tw = 80°C
hw = 100 W/m -K
2

Figure P1.7-4: A solar collector

The receiver tube is exposed to solar radiation that has been reflected from a concentrating
mirror. The heat flux received by the tube is related to the position of the sun and the geometry
and efficiency of the concentrating mirrors. For this problem, you may assume that all of the
radiation heat flux is absorbed by the collector and neglect the radiation emitted by the collector
to its surroundings. (Chapter 10 will provide information on the radiation characteristics of
surfaces that will allow a more complete evaluation of solar collectors.) The flux received at the
collector surface ( qs′′ ) is not circumferentially uniform but rather varies with angular position;
the flux is uniform along the top of the collector, π < φ < 2π rad, and varies sinusoidally along
the bottom, 0 < φ < π rad, with a peak at φ = π/2 rad.

⎪⎧qt′′+ ( q ′′p − qt′′) sin (φ ) for 0 < φ < π


qs′′ (φ ) = ⎨
⎪⎩qt′′ for π < φ < 2 π

where qt′′ = 1000 W/m2 is the uniform heat flux along the top of the collector tube and q ′′p =
5000 W/m2 is the peak heat flux along the bottom. The receiver tube has an inner radius of r =
5.0 cm and thickness of th = 2.5 mm (because th/r << 1 it is possible to ignore the small
difference in convection area on the inner and outer surfaces of the tube). The thermal
conductivity of the tube material is k = 10 W/m-K. The solar collector is used to heat water,
which is at Tw = 80°C at the axial position of interest. The average heat transfer coefficient
between the water and the internal surface of the collector is hw = 100 W/m2-K. The external
surface of the collector is exposed to air at Ta = 25°C. The average heat transfer coefficient
between the air and the external surface of the collector is ha = 25 W/m2-K.
a.) Can the collector be treated as an extended surface for this problem (i.e., can the temperature
gradients in the radial direction in the collector material be neglected)?

The input parameters are entered into the EES program:


$UnitSystem SI MASS RAD PA K J
$Tabstops 0.2 0.4 0.6 3.5 in

"Inputs"
qf_t=1000 [W/m^2] "Heat flux on top surface"
qf_p=5000 [W/m^2] "Peak heat flux on bottom surface"
k = 10 [W/m-K] "Conductivity of the collector material"
th=2.5 [mm]*convert(mm,m) "thickness of collector"
r=5.0[cm]*convert(cm,m) "inner radius of collector"
T_a=converttemp(C,K,25[C]) "temperature of surrounding air"
h_bar_a=25 [W/m^2-K] "heat transfer coefficient to surrounding air"
T_w=converttemp(C,K,80 [C]) "temperature of water"
h_bar_w=100 [W/m^2-K] "heat transfer coefficient to water"
L=1 [m] "length of the collector"

The extended surface approximation neglects temperature gradients across the thickness of the
tube but considers the temperature difference between the collector surfaces and the surrounding
water and air. This assumption is equivalent to neglecting the conduction resistance across the
tube as being small relative to the two convection resistances characterizing heat transfer to the
air and the water. Two Biot numbers are calculated based on the air (Bia) and water (Biw)

ha th
Bia =
2k

hw th
Biw =
2k

Bi_a=h_bar_a*th/(2*k) "Biot number based on air side"


Bi_w=h_bar_w*th/(2*k) "Biot number based on water side"

Both Biot numbers are found to be much less than one (0.003 and 0.01, respectively) and
therefore the extended surface approximation is valid.

b.) Develop an analytical model that will allow the temperature distribution in the collector wall
to be determined as a function of circumferential position.

The computational domain (the receiver tube wall) goes from φ = 0 to 2π rad; however, there are
actually two separate computational domains, the top and bottom, each with a different
governing equation due to the different spatial variation in the heat flux. A differential control
volume is used to derive the governing equation in the bottom computational domain. An energy
balance on the control volume leads to:

qφ + qs = qφ + dφ + qconv ,a + qconv , w

The conduction per unit length in the circumferential direction, qφ , is written using Fourier’s
law.
th L dTb
qφ = −k
r dφ

where L is the length of the collector tube and Tb is the temperature along the bottom of the tube.
Note that the temperature gradient along the circumference of the tube (K/m) is written as the
product the derivative of temperature with respect to angle (K/rad) and the inverse of the radius.

The convection to the air and the water are:

qconv ,a = r dφ L ha (Tb − Ta )

qconv , w = r dφ L hw (Tb − Tw )

The energy absorbed due to the solar flux is:

qs = ⎡⎣ qt′′+ ( q ′′p − qt′′) sin (φ ) ⎤⎦ L r dφ

Combining these equations leads to:

⎡ th dTb ⎤
⎡ qt′′+ ( q ′′p − qt′′) sin (φ ) ⎤ r dφ =
d
⎣ ⎦ dφ ⎢ −k r dφ ⎥ dφ + r dφ ha (Tb − Ta ) + r dφ hw (Tb − Tw )
⎣ ⎦

which can be simplified to:

d 2Tb r 2 ha r 2 hw r 2 ha r 2 hw r2
− Tb − Tb = − Ta − Tw − ⎣ qt + ( q p − qt ) sin (φ ) ⎦
⎡ ′′ ′′ ′′ ⎤ (1)
dφ 2 k th k th k th k th k th

Equation (1) is the governing differential equation for the temperature along bottom of the
collector and it is therefore valid from 0 < θ < π. A similar set of steps leads to the governing
equation for the temperature along the top of the collector (Tt) that is valid from π < θ < 2 π.

d 2Tt r 2 ha r 2 hw r 2 ha r 2 hw r2
− T − T = − T − T − 
q ′′ (2)
dφ 2 k th
t t a w t
k th k th k th k th

Equations (1) and (2) are entered into Maple and solved in order to determine their general
solutions.

> restart;
> ODE_b:=diff(diff(T_b(phi),phi),phi)-r^2*h_bar_a*T_b(phi)/(k*th)-r^2*h_bar_w*T_b(phi)/(k*th)=-
r^2*h_bar_a*T_a/(k*th)-r^2*h_bar_w*T_w/(k*th)-(qf_t+(qf_p-qf_t)*sin(phi))*r^2/(k*th);
⎛d ⎞ r h_bar_a T_b( φ ) r h_bar_w T_b( φ )
2 2 2
ODE_b := ⎜ 2 T_b( φ ) ⎟⎟ − − =
⎜ dφ k th k th
⎝ ⎠
r2 h_bar_a T_a r2 h_bar_w T_w ( qf_t + ( qf_p − qf_t ) sin( φ ) ) r2
− − −
k th k th k th
> T_b_s:=dsolve(ODE_b);
⎛ r h_bar_a + h_bar_w φ ⎞ ⎛ r h_bar_a + h_bar_w φ ⎞
⎜ ⎟ ⎜− ⎟
⎜ ⎟ ⎜ ⎟
⎝ k th ⎠ ⎝ k th ⎠
T_b_s := T_b( φ ) = e _C2 + e _C1 + (
−r ( −qf_p + qf_t ) ( h_bar_a + h_bar_w ) sin( φ )
2

+ ( r2 ( h_bar_a + h_bar_w ) + k th ) ( h_bar_a T_a + h_bar_w T_w + qf_t ) ) (


( r2 ( h_bar_a + h_bar_w ) + k th ) ( h_bar_a + h_bar_w ) )
> ODE_t:=diff(diff(T_t(phi),phi),phi)-r^2*h_bar_a*T_t(phi)/(k*th)-r^2*h_bar_w*T_t(phi)/(k*th)=-
r^2*h_bar_a*T_a/(k*th)-r^2*h_bar_w*T_w/(k*th)-qf_t*r^2/(k*th);
⎛d ⎞ r h_bar_a T_t ( φ ) r h_bar_w T_t( φ )
2 2 2
ODE_t := ⎜ 2 T_t( φ ) ⎟⎟ − − =
⎜ dφ k th k th
⎝ ⎠
r h_bar_a T_a r2 h_bar_w T_w qf_t r2
2
− − −
k th k th k th
> T_t_s:=dsolve(ODE_t);
⎛ r h_bar_a + h_bar_w φ ⎞ ⎛ r h_bar_a + h_bar_w φ ⎞
⎜ ⎟ ⎜− ⎟
⎜ ⎟ ⎜ ⎟
⎝ k th ⎠ ⎝ k th ⎠
T_t_s := T_t( φ ) = e _C2 + e _C1
h_bar_a T_a + h_bar_w T_w + qf_t
+
h_bar_a + h_bar_w

Note that although both solutions are given with constants of integration C1 and C2 it is clear that
these constants cannot be the same. Here, the constants for the general solution for Tt will be C3
and C4. The solutions are copied into EES and manipulated slightly to obtain:

"Solutions"
T_b = exp(r/k^(1/2)/th^(1/2)*(h_bar_a+h_bar_w)^(1/2)*phi)*C_2+&
exp(-r/k^(1/2)/th^(1/2)*(h_bar_a+h_bar_w)^(1/2)*phi)*C_1+&
(-r^2*(h_bar_a+h_bar_w)*(-qf_p+qf_t)*sin(phi)+(h_bar_a*T_a+&
T_w*h_bar_w+qf_t)*(r^2*(h_bar_a+h_bar_w)+k*th))/(h_bar_a+&
h_bar_w)/(r^2*(h_bar_a+h_bar_w)+k*th)
T_t = exp(r/k^(1/2)/th^(1/2)*(h_bar_a+h_bar_w)^(1/2)*phi)*C_4+&
exp(-r/k^(1/2)/th^(1/2)*(h_bar_a+h_bar_w)^(1/2)*phi)*C_3+&
(h_bar_a*T_a+h_bar_w*T_w+qf_t)/(h_bar_a+h_bar_w)

There are four unknown constants of integration (C1 through C4). Therefore, 4 boundary
conditions are required to obtain the four constants of integration (two for each solution). The
temperature must be continuous at both of the interfaces between the top and bottom domains:

Tb ,φ =0 = Tt ,φ = 2π (3)
Tb ,φ =π = Tt ,φ =π (4)

Also, the energy flowing between the regions must be conserved. An interface energy balance at
φ = π rad provides:

L th dTb L th dTt
−k = −k
r dφ φ =π
r dφ φ =π

which implies that the temperature gradient at φ = π rad is continuous in both domains:

dTb dTt
= (5)
dφ φ =π
dφ φ =π

A similar equation results for the interface at φ = 0 rad.

dTb dTt
= (6)
dφ φ =0
dφ φ = 2π

Maple can carry out the symbolic manipulation of the solution while EES does the algebra to
determine the constants. To obtain the left and right hand sides of Eq. (3):

> T_b_0:=rhs(eval(T_b_s,phi=0));
h_bar_a T_a + h_bar_w T_w + qf_t
T_b_0 := _C2 + _C1 +
h_bar_a + h_bar_w
> T_t_2pi:=rhs(eval(T_t_s,phi=2*pi));
⎛ 2 r h_bar_a + h_bar_w π ⎞ ⎛ 2 r h_bar_a + h_bar_w π ⎞
⎜ ⎟ ⎜− ⎟
⎜ ⎟ ⎜ ⎟
⎝ k th ⎠ ⎝ k th ⎠
T_t_2pi := e _C2 + e _C1
h_bar_a T_a + h_bar_w T_w + qf_t
+
h_bar_a + h_bar_w

These two expressions can be cut and pasted into EES and, with minimal modification, used to
set the boundary condition associated with Eq. (3). The necessary modifications include
changing _C1 and _C2 to C_1 and C_2 in the equation for T_b_0 and changing _C1 and _C2 to
C_3 and C_4 in the equation for T_t_2pi.

"Boundary conditions"
"Temperature equality at phi=0"
T_b_0 = C_2+ C_1+(h_bar_a*T_a+h_bar_w*T_w+qf_t)/(h_bar_a+h_bar_w)
"temperature in bottom domain at phi=0"
T_t_2pi = exp(2*r/k^(1/2)/th^(1/2)*(h_bar_a+h_bar_w)^(1/2)*pi)*C_4+&
exp(-2*r/k^(1/2)/th^(1/2)*(h_bar_a+h_bar_w)^(1/2)*pi)*C_3+&
(h_bar_a*T_a+h_bar_w*T_w+qf_t)/(h_bar_a+h_bar_w)
"temperature in top domain at phi=2 pi"
T_b_0=T_t_2pi

The EES and Maple text listed above seems long and complicated however very little of it
needed to be entered manually; the process of solving a relatively complex heat transfer problem
is reduced to a relatively straightforward integration of two powerful pieces of software.

The process is repeated for Eq. (4), in Maple:

> T_b_pi:=rhs(eval(T_b_s,phi=pi));
⎛ r h_bar_a + h_bar_w π ⎞ ⎛ r h_bar_a + h_bar_w π ⎞
⎜ ⎟ ⎜− ⎟
⎜ ⎟ ⎜ ⎟
⎝ k th ⎠ ⎝ k th ⎠
T_b_pi := e _C2 + e _C1 + (
−r2 ( −qf_p + qf_t ) ( h_bar_a + h_bar_w ) sin( π )
+ ( r2 ( h_bar_a + h_bar_w ) + k th ) ( h_bar_a T_a + h_bar_w T_w + qf_t ) ) (
( r2 ( h_bar_a + h_bar_w ) + k th ) ( h_bar_a + h_bar_w ) )
> T_t_pi:=rhs(eval(T_t_s,phi=pi));
⎛ r h_bar_a + h_bar_w π ⎞ ⎛ r h_bar_a + h_bar_w π ⎞
⎜ ⎟ ⎜− ⎟
⎜ ⎟ ⎜ ⎟
⎝ k th ⎠ ⎝ k th ⎠
T_t_pi := e _C2 + e _C1
h_bar_a T_a + h_bar_w T_w + qf_t
+
h_bar_a + h_bar_w

The symbolic equations determined by Maple are then entered in EES with the same
modification for _C1 and _C2 noted above:

"Temperature equality at pi"


T_b_pi = exp(r/k^(1/2)/th^(1/2)*(h_bar_a+h_bar_w)^(1/2)*pi)*C_2+&
exp(-r/k^(1/2)/th^(1/2)*(h_bar_a+h_bar_w)^(1/2)*pi)*C_1+&
(-r^2*(-qf_p+qf_t)*(h_bar_a+h_bar_w)*sin(pi)+(r^2*(h_bar_a+h_bar_w)+k*th)&
*(h_bar_a*T_a+h_bar_w*T_w+qf_t))/(r^2*(h_bar_a+h_bar_w)+k*th)/(h_bar_a+h_bar_w)
"temperature in bottom domain at phi = pi"
T_t_pi = exp(r/k^(1/2)/th^(1/2)*(h_bar_a+h_bar_w)^(1/2)*pi)*C_4+&
exp(-r/k^(1/2)/th^(1/2)*(h_bar_a+h_bar_w)^(1/2)*pi)*C_3+&
(h_bar_a*T_a+h_bar_w*T_w+qf_t)/(h_bar_a+h_bar_w)
"temperature in top domain at phi = pi"
T_b_pi=T_t_pi

Equations (5) and (6) are dealt with in the same way. Maple is used to evaluate the symbolic
expressions for the required derivatives:

> dTbdphi_0:=rhs(eval(diff(T_b_s,phi),phi=0));
r h_bar_a + h_bar_w _C2 r h_bar_a + h_bar_w _C1
dTbdphi_0 := −
k th k th
r2 ( −qf_p + qf_t )

r2 ( h_bar_a + h_bar_w ) + k th
> dTtdphi_2pi:=rhs(eval(diff(T_t_s,phi),phi=2*pi));
⎛ 2 r h_bar_a + h_bar_w π ⎞
⎜ ⎟
⎜ ⎟
⎝ k th ⎠
r h_bar_a + h_bar_w e _C2
dTtdphi_2pi :=
k th
⎛ 2 r h_bar_a + h_bar_w π ⎞
⎜− ⎟
⎜ ⎟
⎝ k th ⎠
r h_bar_a + h_bar_w e _C1

k th
> dTbdphi_pi:=rhs(eval(diff(T_b_s,phi),phi=pi));
⎛ r h_bar_a + h_bar_w π ⎞
⎜ ⎟
⎜ ⎟
⎝ k th ⎠
r h_bar_a + h_bar_w e _C2
dTbdphi_pi :=
k th
⎛ r h_bar_a + h_bar_w π ⎞
⎜− ⎟
⎜ ⎟
⎝ k th ⎠
r h_bar_a + h_bar_w e _C1

k th
r ( −qf_p + qf_t ) cos( π )
2
− 2
r ( h_bar_a + h_bar_w ) + k th
> dTtdphi_pi:=rhs(eval(diff(T_t_s,phi),phi=pi));
⎛ r h_bar_a + h_bar_w π ⎞
⎜ ⎟
⎜ ⎟
⎝ k th ⎠
r h_bar_a + h_bar_w e _C2
dTtdphi_pi :=
k th
⎛ r h_bar_a + h_bar_w π ⎞
⎜− ⎟
⎜ ⎟
⎝ k th ⎠
r h_bar_a + h_bar_w e _C1

k th

These expressions are entered in EES (with changes to _C1 and _C2) in order to provide the
final two boundary conditions.

"Temperature gradient equality at 0"


dTbdphi_0 = r/k^(1/2)/th^(1/2)*(h_bar_a+h_bar_w)^(1/2)*C_2-&
r/k^(1/2)/th^(1/2)*(h_bar_a+h_bar_w)^(1/2)*C_1-&
r^2*(-qf_p+qf_t)/(r^2*(h_bar_a+h_bar_w)+k*th)
"gradient in bottom domain at phi =0"
dTtdphi_2pi = r/k^(1/2)/th^(1/2)*(h_bar_a+h_bar_w)^(1/2)*exp(2*r/k^(1/2)/th^(1/2)*&
(h_bar_a+h_bar_w)^(1/2)*pi)*C_4-r/k^(1/2)/th^(1/2)*(h_bar_a+h_bar_w)^(1/2)*&
exp(-2*r/k^(1/2)/th^(1/2)*(h_bar_a+h_bar_w)^(1/2)*pi)*C_3
"gradient in top at phi = 2 pi"
dTbdphi_0=dTtdphi_2pi

"Temperature gradient equality at pi"


dTbdphi_pi = r/k^(1/2)/th^(1/2)*(h_bar_a+h_bar_w)^(1/2)*exp(r/k^(1/2)/th^(1/2)*&
(h_bar_a+h_bar_w)^(1/2)*pi)*C_2-r/k^(1/2)/th^(1/2)*(h_bar_a+h_bar_w)^(1/2)*&
exp(-r/k^(1/2)/th^(1/2)*(h_bar_a+h_bar_w)^(1/2)*pi)*C_1-r^2*(-qf_p+qf_t)*cos(pi)&
/(r^2*(h_bar_a+h_bar_w)+k*th)
"gradient in bottom at phi = pi"
dTtdphi_pi = r/k^(1/2)/th^(1/2)*(h_bar_a+h_bar_w)^(1/2)*exp(r/k^(1/2)/th^(1/2)*&
(h_bar_a+h_bar_w)^(1/2)*pi)*C_4-r/k^(1/2)/th^(1/2)*(h_bar_a+h_bar_w)^(1/2)&
*exp(-r/k^(1/2)/th^(1/2)*(h_bar_a+h_bar_w)^(1/2)*pi)*C_3
"gradient in top at phi = pi"
dTbdphi_pi=dTtdphi_pi

The analytical solution is converted to Celsius:

T_b_C=converttemp(K,C,T_b) "bottom temperature in C"


T_t_C=converttemp(K,C,T_t) "top temperature in C"

and plotted by setting up two parametric tables. The first table contains the variables phi and
T_b_C (where phi is varied from 0 to π) while the second table contains the variables phi and
T_t_C (where phi is varied from π to 2π). The temperature distribution is shown in Figure 2.

Figure 2: Temperature distribution around the circumference of the collector tube for various values
of the tube conductivity.

It is possible to adjust any of the input parameters within EES the solution remains valid because
the constants are evaluated symbolically. For example, Figure 2 also illustrates how the solution
varies as the conductivity of the receiver tube changes.
Problem 1.7-5
A flux meter is illustrated in Figure P1.7-5.

T∞ = 20°C
q ′′ = 900 W/m
2
Tc = 20°C evacuated
space
k = 10 W/m-K
x

th = 1 mm
L = 2 cm

R′′ = 1x10 K-m /W


-4 2

Figure P1.7-5: Flux meter.

A thin plate is clamped on either end to a casing that is maintained at Tc = 20ºC. There is a
contact resistance between the plate and the casing, R′′ = 1x10-4 K-m2/W. The thickness of the
plate is th = 1 mm and the half-width of the plate is L = 2 cm. The plate conductivity is k = 10
W/m-K. The back of the plate is insulated and the front of the plate is mounted within an
evacuated enclosure in order to eliminate convection. You may model radiation loss from the
plate surface using an effective "radiation" heat transfer coefficient, calculated according to:
hrad ≈ 4 ε σ T 3 , where T is the average absolute temperature of the plate and the surroundings
(take T = 300 K for this problem). The plate radiates to surroundings at T∞ = 20ºC. The
nominal flux on the plate is q ′′ = 900 W/m2. You may assume that the plate temperature
distribution is 1-D in the x-direction. You may also assume that the emissivity of the plate, ε, is
one and therefore all of the flux on the plate is absorbed. The flux meter operates by correlating
the difference between the temperature at the center of the plate and the casing with the applied
heat flux.
a.) Derive the governing differential equation that governs the temperature within the plate.
Clearly show your steps.

An energy balance on a differential control volume (see Figure 2) leads to:

q x + q ′′ dxW = q x + dx + hrad dxW (T − T∞ ) (1)

hrad dxW (T − T∞ )
q ′′ dxW

q x q x + dx
Figure 2: Differential energy balance.

Expanding Eq. (1) leads to:


dq
q x + q ′′ dxW = q x + dx + hrad dxW (T − T∞ ) (2)
dx

Substituting Fourier's law into Eq. (2) leads to:

d ⎛ dT ⎞
q ′′ dxW = ⎜ −k W th ⎟ dx + hrad dxW (T − T∞ ) (3)
dx ⎝ dx ⎠

or

d 2T hrad q ′′ hrad
− T =− − T∞ (4)
dx 2
k th k th k th

b.) What are the boundary conditions for the differential equation?

An interface energy balance at x = 0 leads to:

(Tc − Tx=0 ) = −k ⎛ dT ⎞
⎜ ⎟ (5)
R′′ ⎝ dx ⎠ x =0

and an interface energy balance at x = L leads to:

⎛ dT ⎞
⎜ ⎟ =0 (6)
⎝ dx ⎠ x = L

c.) Determine the solution to the differential equation from (a) subject to the boundary
conditions from (b) without using Maple.

The solution to Eq. (4) is split into a homogeneous and particular solution:

T = Th + Tp (7)

Substituting Eq. (7) into Eq. (4) leads to:

d 2Th hrad d 2Tp


h q ′′ hrad
− Th + − rad Tp = − − T∞ (8)
dx 2 k th dx k th
2
k th k th



= 0 for homogeneous equation whatever is left must be the particular differential equation

The solution to the homogeneous differential equation:

d 2Th hrad
− Th = 0 (9)
dx 2 k th
is an exponential:

Th = C exp ( m x ) (10)

Substituting Eq. (10) into Eq. (9) leads to:

hrad
C m 2 exp ( m x ) − C exp ( m x ) = 0 (11)
k th

which is solved by:

hrad
m2 = (12)
k th

Therefore, there are two solutions corresponding to the two roots of Eq. (12):

Th = C1 exp ( m x ) + C2 exp ( − m x ) (13)

where C1 and C2 are undetermined constants and m is:

hrad
m= (14)
k th

The solution to the particular differential equation:

d 2Tp hrad q ′′ hrad


− Tp = − − T∞ (15)
dx 2
k th k th k th

is, by inspection:

q ′′
Tp = + T∞ (16)
hrad

Substituting Eqs. (13) and (16) into Eq. (7) leads to:

q ′′
T = C1 exp ( m x ) + C2 exp ( − m x ) + + T∞ (17)
hrad

Substituting Eq. (17) into Eq. (5) leads to:


⎛ q ′′ ⎞
⎜ Tc − C1 − C2 − − T∞ ⎟
⎝ hrad ⎠ = −k C m − C m
( 1 ) (18)
R′′
2

Substituting Eq. (17) into Eq. (6) leads to:

C1 m exp ( m L ) − C2 m exp ( −m L ) = 0 (19)

d.) Plot the temperature in the plate as a function of position.

The inputs are entered in EES:

"P1.7-5"
$UnitSystem SI MASS DEG PA C J
$Tabstops 0.2 0.4 0.6 0.8 3.5

"Inputs"
q_flux=900 [W/m^2] "solar flux"
th=1 [mm]*convert(mm,m) "thickness of plate"
L=2 [cm]*convert(cm,m) "half-length of plate"
k=10 [W/m-K] "conductivity of plate"
Rc=1e-4 [K-m^2/W] "contact resistance at clamped edges"
T_c=converttemp(C,K,20[C]) "casing temperature"
T_infinity=converttemp(C,K,20[C]) "surrounding temperature"
e=1 [-] "emissivity"
T_bar=300 [K] "average temperature"
h_rad=4*sigma#*e*T_bar^3 "radiation heat transfer coefficient"

Equations (18) and (19) are entered in EES in order to determine the constants C1 and C2:

"boundary conditions"
m=sqrt(h_rad/(k*th)) "fin constant"
(T_c-C_1-C_2-q_flux/h_rad-T_infinity)/Rc=-k*(C_1*m-C_2*m) "at x=0"
C_1*m*exp(m*L)-C_2*m*exp(-m*L)=0 "at x=L"

The solution is programmed in EES:

"solution"
{x_bar=0 [-]} "dimensionless position"
x=x_bar*L "position"
T=C_1*exp(m*x)+C_2*exp(-m*x)+q_flux/h_rad+T_infinity "temperature solution"
T_Celsius=converttemp(K,C,T) "in C"

Figure 3 illustrates the temperature as a function of position in the plate.


38

36

34

Temperature (°C)
32

30

28

26

24

22

20
0 0.1 0.2 0.3 0.4 0.5 0.6 0.7 0.8 0.9 1
Normalized position
Figure 3: Temperature as a function of position in the plate.

e.) Use Maple to solve the differential equation and obtain symbolic expressions for the
boundary conditions. Implement the Maple expressions into EES and show that your answer
is identical to the one determined in (c).

The differential equation, Eq. (4), is entered in Maple:

> restart;
> ODE:=diff(diff(T(x),x),x)-h_rad*T(x)/(k*th)=-q_flux/(k*th)-h_rad*T_infinity/(k*th);
⎛d ⎞ h_rad T( x ) q_flux h_rad T_infinity
2
ODE := ⎜ 2 T( x ) ⎟⎟ − =− −
⎜ dx k th k th k th
⎝ ⎠

and solved:

> Ts:=dsolve(ODE);
⎛ h_rad x ⎞ ⎛ h_rad x ⎞
⎜ ⎟ ⎜− ⎟
⎜ k th ⎟
⎝ ⎠

⎝ k th ⎟⎠ q_flux + h_rad T_infinity
Ts := T( x ) = e _C2 + e _C1 +
h_rad

Symbolic expressions for the boundary conditions are obtained:

> BC1:=(T_c-rhs(eval(Ts,x=0)))/Rc=-k*rhs(eval(diff(Ts,x),x=L));
q_flux + h_rad T_infinity
T_c − _C2 − _C1 −
h_rad
BC1 := =
Rc
⎛ h_rad L ⎞ ⎛ h_rad L ⎞
⎛ ⎜ ⎟ ⎜− ⎟ ⎞
⎜ ⎜ k th ⎟⎠ ⎜ k th ⎟⎠ ⎟
⎜ h_rad e ⎝ _C2 h_rad e

_C1 ⎟⎟

−k ⎜ −
⎜ ⎟⎟
⎝ k th k th ⎠
> BC2:=rhs(eval(diff(Ts,x),x=L))=0;
⎛ h_rad L ⎞ ⎛ h_rad L ⎞
⎜ ⎟ ⎜− ⎟
⎜ k th ⎟ ⎜ k th ⎟⎠
⎝ ⎠ ⎝
h_rad e _C2 h_rad e _C1
BC2 := − =0
k th k th

The original equations in EES are commented out and the expressions from Maple are copied to
EES:

{"boundary conditions"
m=sqrt(h_rad/(k*th)) "fin constant"
(T_c-C_1-C_2-q_flux/h_rad-T_infinity)/Rc=-k*(C_1*m-C_2*m) "at x=0"
C_1*m*exp(m*L)-C_2*m*exp(-m*L)=0 "at x=L"

"solution"
{x_bar=0 [-]} "dimensionless position"
x=x_bar*L "position"
T=C_1*exp(m*x)+C_2*exp(-m*x)+q_flux/h_rad+T_infinity "temperature solution"
T_Celsius=converttemp(K,C,T) "in C"}

"Maple solution"
(T_c-C_2-C_1-1/h_rad*(q_flux+h_rad*T_infinity))/Rc = -k*(1/k^(1/2)/th^(1/2)*h_rad^(1/2)*&
exp(1/k^(1/2)/th^(1/2)*h_rad^(1/2)*L)*C_2-1/k^(1/2)/th^(1/2)*h_rad^(1/2)*exp(-1/k^(1/2)/&
th^(1/2)*h_rad^(1/2)*L)*C_1) "boundary condition at x=0"

1/k^(1/2)/th^(1/2)*h_rad^(1/2)*exp(1/k^(1/2)/th^(1/2)*h_rad^(1/2)*L)*C_2-1/k^(1/2)/th^(1/2)*&
h_rad^(1/2)*exp(-1/k^(1/2)/th^(1/2)*h_rad^(1/2)*L)*C_1 = 0 "boundary condition at x=L"

The solution from Maple is copied into EES:

T = exp(1/k^(1/2)/th^(1/2)*h_rad^(1/2)*x)*C_2+exp(-1/k^(1/2)/th^(1/2)*h_rad^(1/2)*x)*C_1&
+1/h_rad*(q_flux+h_rad*T_infinity) "solution"
x=x_bar*L "position"
T_Celsius=converttemp(K,C,T) "in C"

The solution is overlaid onto the plot from (d) in Figure 4.


37.5

35

32.5
Temperature (°C)

30

27.5

25 solution from (d)


Maple solution
22.5

20

17.5
0 0.1 0.2 0.3 0.4 0.5 0.6 0.7 0.8 0.9 1
Normalized position
Figure 4: Temperature as a function of position in the plate using Maple solution.
f.) Prepare a calibration curve for the flux meter - plot the heat flux as a function of the
difference between the temperature at the center of the plate and the casing.

Figure 5 illustrates the difference between the temperature at the center of the plate as a function
of the applied heat flux.
1400

1200

1000
Heat flux (W/m )
2

800

600

400

200

0
0 2.5 5 7.5 10 12.5 15 17.5 20 22.5 25
Temperature rise (K)
Figure 5: Calibration curve for the flux meter.

g.) If the uncertainty in the measurement of the temperature difference is δΔT = 0.5 K then what
is the uncertainty in the measurement of the heat flux?

The uncertainty in the measurement of the heat flux is approximately:

∂q ′′
δ q ′′ = δΔT (20)
δΔT

∂q ′′
where = 55 W/m2-K according to Figure 5. Therefore, the uncertainty in the heat flux is
δΔT
approximately 22 W/m2.
Problem 1.8-1 (1-17 in text): Disk Brake
Figure P1.8-1 illustrates a disk brake for a rotating machine. The temperature distribution within
the brake can be assumed to be a function of radius only. The brake is divided into two regions.
In the outer region, from Rp = 3.0 cm to Rd = 4.0 cm, the stationary brake pads create frictional
heating and the disk is not exposed to convection. The clamping pressure applied to the pads is
P = 1.0 MPa and the coefficient of friction between the pad and the disk is μ = 0.15. You may
assume that the pads are not conductive and therefore all of the frictional heating is conducted
into the disk. The disk rotates at N = 3600 rev/min and is b = 5.0 mm thick. The conductivity of
the disk is k = 75 W/m-K and you may assume that the outer rim of the disk is adiabatic.

stationary coefficient of friction, μ = 0.15


brake pads

clamping pressure
P = 1 MPa
b = 5 mm Ta = 30°C, h
Rp = 3 cm
Rd = 4 cm
center line
k = 75 W/m-K
disk, rotates at N = 3600 rev/min
Figure P1.8-1: Disk brake.

In the inner region of the disk, from 0 to Rp, is exposed to air at Ta = 30°C. The heat transfer
coefficient between the air and disk surface depends on the angular velocity of the disk, ω,
according to:

1.25
⎡ W ⎤ ⎡ W ⎤⎛ ω ⎞
h = 20 ⎢ 2 ⎥ + 1500 ⎢ 2 ⎥ ⎜⎜ ⎟
⎣ m -K ⎦ ⎣ m -K ⎦ ⎝ 100 [ rad/s ] ⎟⎠

a.) Develop an analytical model of the temperature distribution in the disk brake; prepare a
plot of the temperature as a function of radius for r = 0 to r = Rd.

The inputs are entered in EES and the heat transfer coefficient is computed according to Eq.
Error! Reference source not found..

$UnitSystem SI MASS RAD PA K J


$TABSTOPS 0.2 0.4 0.6 0.8 3.5 in

b=5 [mm]*convert(mm,m) "thickness of disk"


N=3600 [rev/min] "rotational velocity of disk"
omega=N*convert(rev/min,rad/s) "angular velocity of disk"
mu=0.15 [-] "coefficient of friction"
P=1 [MPa]*convert(MPa,Pa) "clamping pressure"
k=75 [W/m-K] "conductivity"
Rd=4.0 [cm]*convert(cm,m) "outer radius of disk"
Rp=3.0 [cm]*convert(cm,m) "inner radius of pad"
Ta=converttemp(C,K,30) "air temperature"
h=20[W/m^2-K]+1500 [W/m^2-K]*(omega/100 [rad/s])^1.25 "heat transfer coefficient"

In the outer region, region 1, the energy balance on a differential control volume is shown in
Figure 2.

Figure 2: Differential energy balance in outer region, (region 1)

The energy balance suggested by Figure 2 is:

qr + q fh = qr + dr (1)

where q fh is the rate of thermal energy generated by frictional heating. After expanding the r +
dr term, Eq. (1) becomes:

dq
q fh = dr (2)
dr

The rate equation for conduction is:

dT1
q = −b 2 π r k (3)
dr

where T1 is the temperature in region 1. The force generated by the pad within the control
volume is the product of the clamping pressure, the area of contact, and the coefficient of
friction:

F = 4 π r dr P μ (4)

Note that the factor of 4 in Eq. (4) is due to their being contact on both sides of the disk. The
rate of frictional heating is the product of the force, the radius, and the angular velocity:

q fh = 4 π r 2 dr P μ ω (5)

Substituting Eqs. (3) and (5) into Eq. (2) leads to:
d ⎡ dT ⎤
4 π r 2 dr P μ ω = ⎢ −b 2 π r k 1 ⎥ dr (6)
dr ⎣ dr ⎦

which can be rearranged:

d ⎡ dT1 ⎤ 2Pμω 2
r = − r (7)
dr ⎢⎣ dr ⎥⎦ bk

or

d ⎡ dT1 ⎤
⎢ r ⎥ = −β r 2 (8)
dr ⎣ dr ⎦

where

2Pμω
β= (9)
bk

Equation (8) can be directly integrated:

⎡ dT1 ⎤
∫ d ⎢⎣ r dr ⎥⎦ = − β ∫ r
2
dr (10)

to achieve:

dT1 r3
r = − β + C1 (11)
dr 3

Equation (11) can be directly integrated again:

⎛ r 2 C1 ⎞
∫ 1 ∫ ⎜⎝ 3 + r ⎟⎠ dr
dT = − β (12)

to achieve:

r3
T1 = − β + C1 ln ( r ) + C2 (13)
9

Equation (13) is the general solution for the temperature in region 1; the constants of integration
will be selected in order to satisfy the boundary conditions.

In the inner region, region 2, the energy balance on a differential control volume is shown in
Figure 3.
Figure 3: Differential energy balance in inner region, (region 2)

The energy balance suggested by Figure 2 is:

qr = qr + dr + qconv (14)

After expanding the r + dr term, Eq. (14) becomes:

dq
0= dr + qconv (15)
dr

The rate equation for conduction remains the same:

dT2
q = −b 2 π r k (16)
dr

where T2 is the temperature in region 2. The rate equation for convection is:

qconv = 4 π r dr h (T2 − Ta ) (17)

Substituting Eqs. (16) and (17) into Eq. (15) leads to:

d ⎡ dT ⎤
⎢ −b 2 π r k 2 ⎥ dr + 4 π r dr h (T2 − Ta ) = 0 (18)
dr ⎣ dr ⎦

or

d ⎡ dT2 ⎤
r − m 2 r T2 = −m 2 r Ta (19)
dr ⎢⎣ dr ⎥⎦

where
2h
m= (20)
bk

The solution to Eq. (19) can be divided into its homogeneous (u2) and particular (v2) parts:

T2 = u2 + v2 (21)

The solution to the particular equation:

d ⎡ dv2 ⎤
⎢ r ⎥ − m 2 r v2 = − m 2 r Ta (22)
dr ⎣ dr ⎦

is

v2 = Ta (23)

The homogeneous equation:

d ⎡ du2 ⎤
r − m 2 r u2 = 0 (24)
dr ⎢⎣ dr ⎥⎦

is a form of Bessel's equation:

d ⎛ p dθ ⎞ 2 s
⎜x ⎟±c x θ =0 (25)
dx ⎝ dx ⎠

where

x=r (26)

θ = u2 (27)

p =1 (28)

c=m (29)

s =1 (30)

and the last term is negative. Following the flow chart provided in Section 1.8.4 of the book
leads to n = 0, a = 1, and therefore the solution is:

u2 = C3 BesselI ( 0, m r ) + C4 BesselK ( 0, m r ) (31)


The general solution for the temperature distribution in region 2 is therefore:

T2 = C3 BesselI ( 0, m r ) + C4 BesselK ( 0, m r ) + Ta (32)

Note that this could be obtained directly from Maple by entering Eq. (19):

> restart;
> ODE:=diff(r*diff(T2(r),r),r)-m^2*r*T2(r)=-m^2*r*Ta;
⎛d ⎞
2
ODE := ⎛⎜⎜ T2( r ) ⎞⎟⎟ + r ⎜⎜ 2 T2( r ) ⎟⎟ − m 2 r T2( r ) = −m 2 r Ta
d
⎝ d r ⎠ ⎝ dr ⎠
> T2s:=dsolve(ODE);
T2s := T2( r ) = BesselI( 0, m r ) _C2 + BesselK( 0, m r ) _C1 + Ta

The constants C1 through C4 in Eqs. (13) and (32) are obtained by applying the correct boundary
conditions. At r = 0, the temperature must remain finite. The figures provided in Section 1.8.4
of the book or the limit capability in Maple show that BesselK(0,m r) will become infinite as r
approaches zero:
> limit(BesselI(0,m*r),r=0);
1
> limit(BesselK(0,m*r),r=0);

therefore:

C4 = 0 (33)

The temperature and temperature gradient at the interface between the regions must be
continuous:

T2, r = Rp = T1, r = Rp (34)

and

dT2 dT1
= (35)
dr r = Rp dr r = Rp

The temperature gradient at the outer rim must be zero:

dT1
=0 (36)
dr r = Rp
Substituting Eqs. (13) and (32) into Eqs. (33) through (36) leads to:

R 3p
C3 BesselI ( 0, m R p ) + Ta = − β + C1 ln ( R p ) + C2 (37)
9

R p2
C3 m BesselI (1, m R p ) = − β
C1
+ (38)
3 Rp

Rd2 C1
−β + =0 (39)
3 Rd

Equations (37) through (39) are 3 equations for the unknown constants and can be solved in
EES.

beta=2*mu*P*omega/(k*b) "generation parameter"


m=sqrt(2*h/(k*b)) "fin parameter"
BesselI(0,m*Rp)*C_3+Ta=-1/9*beta*Rp^3+C_1*ln(Rp)+C_2 "equality of temperature at r=Rp"
BesselI(1,m*Rp)*m*C_3=-1/3*beta*Rp^2+1/Rp*C_1
"equality of temperature gradient at r=Rp"
-1/3*beta*Rd^2+1/Rd*C_1=0 "zero temperature gradient at r=Rd"

The general solutions are entered in EES:

T2 = BesselI(0,m*r2)*C_3+Ta "solution in region 2"


T1 = -1/9*beta*r1^3+C_1*ln(r1)+C_2 "solution in region 1"

A dimensionless radius, the variable rbar, is defined in order to allow a Parametric Table to be
generated where the variable r1 can be easily altered from Rp to Rd and the r2 can be easily
altered from 0 to Rp:

r1=Rp+(Rd-Rp)*rbar
r2=rbar*Rp

Figure 4 illustrates the temperature distribution in the disk.


Figure 4: Temperature distribution in the disk

b.) If the disk material can withstand a maximum safe operating temperature of 750°C then what
is the maximum allowable clamping pressure that can be applied? Plot the temperature
distribution in the disk at this clamping pressure. What is the braking torque that results?

The maximum operating temperature is obtained at r = Rd (see Figure 4). The clamping pressure
that results in T1 at the outer rim reaching the maximum allowable temperature can be
determined by commenting out the originally specified clamping pressure and specifying this
temperature:

{P=1 [MPa]*convert(MPa,Pa)} "clamping pressure"


T_max_allowed=converttemp(C,K,750) "maximum allowable temperature"
rbar=1.0
T1=T_max_allowed

which leads to a clamping pressure of P = 0.57 MPa. The temperature distribution for this
clamping pressure is shown in Figure 4. The torque applied by the pads (Tq) is obtained from
the integral:
Rd

Tq = ∫ 4 π r μ P dr
2
(40)
Rp

or

4
Tq = π μ P ⎡⎣ Rd3 − R 3p ⎤⎦ (41)
3
which leads to Tq = 13.2 N-m.

c.) Assume that you can control the clamping pressure so that as the machine slows down the
maximum temperature is always kept at the maximum allowable temperature, 750°C. Plot
the torque as a function of rotational speed for 100 rev/min to 3600 rev/min.

A parametric table is created that includes the variables N and Tq,; N is varied from 100 rev/min
to 3600 rev/min. The results are shown in Figure 5. Notice that it is possible to dramatically
improve the performance of the brake if you can adjust the clamping pressure with speed.

Figure 5: Clamping pressure and torque as a function of rotational velocity.


Problem 1.8-2: Absorption in a Window
Figure P1.8-2 illustrates a thin, disk-shaped window that is used to provide optical access to a
combustion chamber. The thickness of the window is b and the outer radius of the window is Ro.
The window is composed of material with conductivity k and absorption coefficient α. The
combustion chamber side of the window is exposed to convection with hot gas at Tg and heat
transfer coefficient h. Convection with the air outside of the chamber can be neglected. There is
′′ , that is incident on the combustion chamber side of the glass. The
a radiation heat flux, qrad
′′ α b . The remainder
amount of this radiation that is absorbed by the glass is, approximately, qrad
′′ (1 − α b ) , exits the opposite surface of the glass. The outer edge (at r = Ro)
of this radiation, qrad
of the glass is held at temperature Tedge.

Tf , h combustion chamber

′′
radiant flux, qrad window, k and α

x b

r
Tedge Ro
Tedge
unabsorbed radiant flux, (1 − α b )q rad
′′

outside of chamber
Figure P1.8-2: Disk-shaped window.

You are to develop a 1-D, steady state analytical model that can predict the temperature
distribution in the glass as a function of radial position, r.
a.) How would you justify using a 1-D model of the glass? What number would you calculate in
order to verify that the temperature does not vary substantially in the x direction?

The Biot number should be computed in order to justify the extended surface approximation.
The Biot number is the ratio of conduction resistance in the axial direction to convection
resistance from the inner surface of the window:

Rcond , x b h A bh
Bi = = = (1)
Rconv kA 1 k

Anything within a factor of 2 of Eq. (1) would be sufficient

b.) Derive the ordinary differential equation in r that must be solved. Make sure that your
differential equation includes the effect of conduction, convection with the gas within the
chamber, and generation of thermal energy due to absorption.

A differentially small control volume is shown in Figure 2.


Figure 2: Differentially small control volume.

The energy balance suggested by Figure 2 is:

qr + qrad + qconv = qr + dr + qrad (1 − α b ) (2)

Expanding the r + dr term and simplifying leads to:

dq
qrad + qconv = dr + qrad (1 − α b ) (3)
dr

The rate equations are:

′′ 2 π r dr
qrad = qrad (4)

dT
q = − k 2 π r b (5)
dr

qconv = h 2 π r dr (T f − T ) (6)

Substituting Eqs. (4) through (6) into Eq. (3) leads to:

d ⎡ dT ⎤
′′ 2 π r dr + h 2 π r dr (T f − T ) =
qrad ⎢ −k 2 π r b ′′ 2 π r dr (1 − α b )
dr + qrad (7)
dr ⎣ dr ⎥⎦

Dividing through by (-k 2 π b dr) leads to:

d ⎡ dT ⎤ h ′′ α h
⎛ qrad ⎞
r − r T = − r ⎜ + T ⎟ (8)
dr ⎢⎣ dr ⎥⎦ k b
f
⎝ k kb ⎠

c.) What are the boundary conditions for the ordinary differential equation that you derived in
part (b)?

At r = 0 the temperature must be finite. At the edge, the temperature is specified:

Tr = Ro = Tedge (9)
d.) Solve the ordinary differential equation in order to obtain an expression for the temperature
as a function of radius.

The solution is split into its homogeneous (u) and particular (v) parts:

T =u+v (10)

The particular solution is a constant:

h ⎛ q ′′ α h ⎞
− r v = − r ⎜ rad + Tf ⎟ (11)
kb ⎝ k kb ⎠

or

′′ α b
qrad
v = Tf + (12)
h

The homogeneous form of the differential equation is:

d ⎡ du ⎤
⎢ r ⎥ − m2 r u = 0 (13)
dr ⎣ dr ⎦

where

h
m= (14)
kb

Equation (13) is a form of Bessel's equation:

d ⎛ p dθ ⎞ 2 s
⎜x ⎟±c x θ = 0 (15)
dx ⎝ dx ⎠

where

θ =u (16)

x=r (17)

p =1 (18)

c=m (19)

s =1 (20)

The solution can be obtained by using the chart found in the notes:
1− p 1−1
n= = =0 (21)
s − p + 2 1−1+ 2

2
a= =1 (22)
1−1+ 2

n 1−1
= =0 (23)
a 2

so that

u = C1 BesselI ( 0, m r ) + C2 BesselK ( 0, m r ) (1-24)

and the solution is:

′′ α b
qrad
T = C1 BesselI ( 0, m r ) + C2 BesselK ( 0, m r ) + T f + (1-25)
h

Because BesselK(0,0) becomes infinite, C2 = 0 and:

′′ α b
qrad
T = C1 BesselI ( 0, m r ) + T f + (1-26)
h

The boundary condition associated with Eq. (9) leads to:

′′ α b
qrad
Tedge = C1 BesselI ( 0, m Ro ) + T f + (1-27)
h

so that:

′′ α b
qrad
Tedge − T f −
C1 = h (1-28)
BesselI ( 0, m Ro )

and

⎛ q ′′ α b ⎞ BesselI ( 0, m r ) q ′′ α b
T = ⎜ Tedge − T f − rad ⎟ + T f + rad (1-29)
⎝ h ⎠ BesselI ( 0, m Ro ) h
Problem 1.8-3: Cryogenic Thermal Standoff
It is often necessary to provide a fluid outlet port that will allow very cold gas to escape from a
cryogenic facility; for example, the boil-off of liquid nitrogen or liquid helium from within a
vacuum vessel must be allowed to vent to the atmosphere. The seal between the base of the
flange that contains the fluid passage and the surrounding vessel is usually made with an o-ring;
these seals are convenient in that they are hermetic and easily demountable. However, most
convenient o-ring materials do not retain their ductility at temperatures much below 0°C and
therefore it is important that the o-ring be kept at or above this temperature so that it continues to
provide a good seal; if the o-ring “freezes” then the cryogenic facility will lose its vacuum. The
o-ring temperature is maintained at an appropriate level using a thermal standoff, as shown in
Figure P1.8-3

The fluid passage is attached to the flange via a separate, slightly larger tube made of a stainless
steel with thermal conductivity k = 15 W/m-K. This thermal standoff has an outer radius, rts = 1
cm, thickness tts = 1 mm, and length Lts = 5 cm. The cryogenic gas is at 77 K and you may
assume that the point x = 0 in Figure P1.8-3 is at Tcold = 77 K. The inside of the tube is exposed
to a vacuum and you may assume that it experiences negligible radiation heat transfer and no
convective heat transfer. The outside of the tube is exposed to air at Tair = 20°C with heat
transfer coefficient, h = 7 W/m2-K. The bottom of the tube (x = Lts) is welded to the flange. The
flange has an outer radius rfl = 8 cm and a thickness tfl = 1.5 mm. The inside of the flange is
exposed to vacuum and therefore, for the purposes of this problem, adiabatic. The outside of the
flange is exposed to the same 20°C air with the same 7 W/m2-K heat transfer coefficient.
thermal stand off
flange

vacuum
o-ring seal cryogenic gas
Tcold = 77 K

Lts = 5 cm
Tair = 20°C
h = 7 W/m -K
2
x
tts = 1 mm
k = 15 W/m-K
tfl = 1.5 mm
rts = 1 cm

rfl = 8 cm
Figure P1.8-3: Cryogenic thermal standoff.

a.) Is it appropriate to treat the thermal standoff and the flange as extended surfaces? That is,
can the temperature within the stand-off be treated as being only a function of x and the
temperature in the flange only a function of r?

The input parameters are entered in EES:

$UnitSystem SI MASS RAD PA K J


$Tabstops 0.2 0.4 0.6 3.5 in

"Inputs"
k=15 [W/m-K] "thermal conductivity"
r_ts=1.0 [cm]*convert(cm,m) "outer radius of thermal standoff"
t_ts=1.0 [mm]*convert(mm,m) "thickness of thermal standoff"
L_ts=5.0 [cm]*convert(cm,m) "length of thermal standoff"
T_cold=77 [K] "cryogenic gas temperature"
T_air=converttemp(C,K,20 [C]) "air temperature"
h=7 [W/m^2-K] "heat transfer coefficient"
r_fl=8.0 [cm]*convert(cm,m) "radius of flange"
t_fl=1.5 [mm]*convert(mm,m) "thickness of flange"

The Biot numbers that must be calculated for the standoff and the flange (Bits and Bifl,
respectively) are:
h tts
Bits =
k

and

h t fl
Bi fl =
k

These are computed in EES:

Bi_ts=t_ts*h/k "Biot number for thermal standoff"


Bi_fl=t_fl*h/k "Biot number for flange"

and found to be very small.

b.) Develop an analytical model of the problem that can predict the temperature distribution as a
function of x in the thermal stand-off and as a function of r in the flange.

An energy balance on the differential control volume within the thermal standoff leads to:

q x = q x + dx + qconv

or, after making the usual simplifications:

d ⎡ dTts ⎤
0=− 2 π r t k dx + 2 π rts dx h (Tts − Tair )
dx ⎢⎣ dx ⎥⎦
ts ts

which becomes the governing differential equation for a constant cross-sectional area fin that
was solved in Section 1.6:

d 2θts
− mts2 θts = 0 (1)
dx 2

where

h
mts =
tts k

and

θts = Tts − Tair

Equation (1) is solved by exponentials (or equivalently by sinh and cosh):


θts = C1 exp ( mts x ) + C2 exp ( − mts x ) (2)

where C1 and C2 are determined by the boundary coefficients.

An energy balance on the differential control volume within the flange (see Figure 1) is:

qr = qr + dr + qconv

or, after making the usual simplifications:

d ⎡ dT fl ⎤
0=− ⎢ 2 π r t fl k ⎥ dr + 2 π r dr (T fl − Tair )
dr ⎣ dr ⎦

which becomes Bessel's equation:

d ⎡ dθ fl ⎤
⎥ − m fl r θ fl = 0
2
⎢r (3)
dr ⎣ dr ⎦

where

h
m fl =
t fl k

and

θ fl = T fl − Tair

Equation (3) is solved by 0th order modified Bessel functions:

θ fl = C3 BesselI ( 0, m fl r ) + C4 BesselK ( 0, m fl r ) (4)

where C3 and C4 are determined by the boundary coefficients.

There must be four boundary conditions; two for each of the 2nd order differential equations.
The temperature at the top of the thermal stand-off is specified:

Tts , x =0 = Tcold

or, substituting into Eq. (2):

C1 + C2 = Tcold − Tair
The temperature at the bottom of the thermal standoff must be equal to the temperature at the
inner edge of the flange:

Tts , x = Lts = T fl ,r = rts

or

C1 exp ( mts Lts ) + C2 exp ( −mts Lts ) = C3 BesselI ( 0, m fl rts ) + C4 BesselK ( 0, m fl rts )

The rate of heat transfer out of the bottom of the thermal standoff must be equal to the rate that
heat that is transferred into the inner edge of the flange:

dTts dT fl
− k 2 π rts tts = − k 2 π rts t fl
dx Lts dr rts

or

dθts dθ fl
tts = t fl
dx Lts dr rts

whic leads to:

tts ⎡⎣C1 mts exp ( mts Lts ) − C2 mts exp ( −mts Lts ) ⎤⎦ = t fl ⎡⎣C3 m fl BesselI (1, m fl rts ) − C4 m fl BesselK (1, m fl rts ) ⎤⎦

Finally, the outer edge of the flange is assumed to be adiabatic (the small amount of convection
from the edge can be neglected with little error):

dT fl
− k 2 π rfl t fl =0
dr r fl

or

dθ fl
=0
dr r fl

which leads to:

C3 m fl BesselI (1, m fl rfl ) − C4 m fl BesselK (1, m fl rfl ) = 0

The boundary condition equations are entered into EES:


"Solution parameters"
m_ts=sqrt(h/(t_ts*k))
m_fl=sqrt(h/(t_fl*k))

"Boundary conditions"
C_1+C_2=T_cold-T_air
C_1*exp(m_ts*L_ts)+C_2*exp(-m_ts*L_ts)=C_3*BesselI(0,m_fl*r_ts)+C_4*BesselK(0,m_fl*r_ts)
t_ts*(C_1*m_ts*exp(m_ts*L_ts)-C_2*m_ts*exp(-m_ts*L_ts))=t_fl*(C_3*m_fl*BesselI(1,m_fl*r_ts)-
C_4*m_fl*BesselK(1,m_fl*r_ts))
C_3*m_fl*BesselI(1,m_fl*r_fl)-C_4*m_fl*BesselK(1,m_fl*r_fl)=0

The solution is provided in EES using two parametric tables for the thermal standoff and flange
temperature distributions. The first table is titled ‘Thermal Stand off’ and includes values of
dimensionless position, x , ranging from 0 to 1; where:

x
x=
Lts

The coordinate, s, is also computed; s goes from 0 to Lts + (rfl – rts) as you move from the top of
the thermal stand off to the edge of the flange. The use of s provides a convenient method for
looking at the entire temperature distribution in the thermal standoff and flange.

s=x

The ‘Thermal Stand off’ table includes columns that contain the values of θts and Tts. The
solution for the temperature within the thermal stand off is only calculated if you are running the
parametric table entitled ‘Thermal Stand off’. The selection of equations that are to be solved
with each parametric table is facilitated by using the $IF PARAMETRICTABLE directive to check
which table is being calculated. The equations located between the $IF and $ENDIF in the EES
code below are executed only if the ‘Thermal Stand off’ parametric table is being calculated.

$IF PARAMETRICTABLE='Thermal Stand off'


x_bar=x/L_ts
s=x
theta_ts=C_1*exp(m_ts*x)+C_2*exp(-m_ts*x)
Temp_ts=theta_ts+T_air
Temp_ts_C=converttemp(K,C,Temp_ts)
$ENDIF

A second parametric table entitled ‘Flange’ is includes values of dimensionless position in the
flange, r :

r=
( r − rts )
(r fl − rts )

and the coordinate, s, which is computed in the flange as:


s = r − rts + L

The table ‘Flange’ also includes the values θfl and Tfl. The required commands are included in a
separate $IF statement:

$IF PARAMETRICTABLE='Flange'
r_bar=(r-r_ts)/(r_fl-r_ts)
s=L_ts+r-r_ts
theta_fl=C_3*BesselI(0,m_fl*r)+C_4*BesselK(0,m_fl*r)
Temp_fl=theta_fl+T_air
Temp_fl_C=converttemp(K,C,Temp_fl)
$ENDIF

By sequentially running parametric table ‘Thermal Stand off’ and ‘Flange’ it is possible to
determine the entire temperature distribution; the result is shown in Figure 2.

Figure 2: Temperature as a function of the coordinate s.

Notice that the solution satisfies each of the boundary conditions. The temperature at the top of
the stand off is equal to 77 K and the temperature gradient at the edge of the flange is zero. The
temperature at the intersection of the flange and the thermal stand off is continuous but there is a
discontinuity in the temperature gradient related to the fact that the flange is slightly thicker (and
therefore has a lower temperature gradient for the same conduction heat transfer) than the
thermal stand off.

The value of the thermal stand off is clear. If the o-ring seal is placed towards the outer radius of
the flange then Figure 2 shows that the temperature will remain above freezing and therefore the
o-ring will continue to function. Using the EES model it is possible to evaluate alternative, more
effective designs (i.e., thermal stand off geometries that keep the temperature at the outer edge of
the flange higher). Figure 3 illustrates the temperature at the edge of the flange (i.e., T fl ,r = rfl ) as a
function of the thermal stand-off thickness for various values of its length.
Figure 3: Temperature at the edge of the flange as a function of the thermal stand off thickness for
various values of the thermal stand off length.

Note that either increasing the thermal stand off length or decreasing its thickness will tend to
make it a less efficient fin and therefore increase the temperature gradient due to conduction. It
turns out that a good thermal stand off is a bad fin, isolating the tip of the fin (i.e., the flange)
from the base of the fin (i.e., the cryogenic temperature).
Problem 1.8-4: Circular Fin
Figure P1.8-4 shows a typical fin design that is fabricated by attaching a thin washer to the outer
radius of a tube. The inner and outer radii of the fin are rin and rout, respectively. The thickness
of the fin is th and the fin material has conductivity, k. The fin is surrounded by fluid at T∞ and
the average heat transfer coefficient is h . The base of the fin is maintained at Tb and the tip is
adiabatic.

h , T∞

rout
th

rin Tb k

Figure P1.8-4: Circular fin.

Determine an analytical solution for the temperature distribution in the fin and the fin efficiency.

The differential control volume shown in Figure 2 can be used to derive the governing equation.

Figure 2: Differential control volume.

An energy balance for the control volume is:

qr = qr + dr + qconv

or

dq
0= dr + qconv
dr

The conduction and convection terms are:


dT
q = − k 2 π r th
dr

qconv = 4 π r dr h (T − T∞ )

Combining these equations leads to:

d ⎡ dT ⎤
0= ⎢ −k 2 π r th dr + 4 π r dr h (T − T∞ ) = 0
dr ⎣ dr ⎥⎦

which can be simplified to:

d ⎡ dT ⎤ 2 r h 2r h
⎢ r ⎥ − T=− T∞
dr ⎣ dr ⎦ k th k th

The solution is divided into a homogeneous and particular component:

T = Th + Tp

which leads to:

d ⎡ dTh ⎤ 2 r h d ⎡ dTp ⎤ 2 r h 2r h
⎢ r ⎥ − Th + ⎢ r ⎥ − Tp = − T∞
dr ⎣ dr ⎦ k th dr ⎣ dr ⎦ k th k th



=0 for homogeneous differential equation whatever is left is the particular differential equation

The solution to the particular differential equation:

d ⎡ dTp ⎤ 2 r h 2r h
⎢r ⎥− Tp = − T∞
dr ⎣ dr ⎦ k th k th

is

Tp = T∞

The homogeneous differential equation is:

d ⎡ dTh ⎤
⎢ r ⎥ − m 2 r Th = 0 (1)
dr ⎣ dr ⎦

where m is the fin parameter, defined as:


2h
m=
kb

Equation (1) is a form of Bessel’s equation:

d ⎛ p dθ ⎞ 2 s
⎜x ⎟±c x θ =0 (2)
dx ⎝ dx ⎠

where (by comparing Eqs. (1) and (2)), p= 1, c = m, and s = 1. Referring to the flow chart
presented in Section 1.8.4, the value of s-p+2 is equal to 2 and therefore the solution parameters
n and a must be computed:

1−1
n= =0
1−1+ 2

2
a= =1
1−1+ 2

The last term in Eq. (1) is negative and therefore the solution to Eq. (1) is given by:

n
(
Th = C1 x a BesselI n, c a x
1
a
)+C x 2
n
a
(
BesselK n, c a x
1
a
)
or

Th = C1 BesselI ( 0, m r ) + C2 BesselK ( 0, m r )

The solution to the governing differential equation is:

T = C1 BesselI ( 0, m r ) + C2 BesselK ( 0, m r ) + T∞ (3)

Note that Maple would provide this information as well:

> restart;
> ODE:=diff(r*diff(T(r),r),r)-m^2*r*T(r)=-m^2*r*T_infinity;
d ⎛d ⎞
2
ODE := ⎛⎜⎜ T( r ) ⎞⎟⎟ + r ⎜ 2 T( r ) ⎟⎟ − m 2 r T( r ) = −m 2 r T_infinity

⎝ r
d ⎠ ⎝ dr ⎠
> Ts:=dsolve(ODE);
Ts := T( r ) = BesselI( 0, m r ) _C2 + BesselK ( 0, m r ) _C1 + T_infinity

The boundary conditions must be used to obtain the constants C1 and C2. The base temperature
is specified:
Tr = rin = Tb

or:

C1 BesselI ( 0, m r ) + C2 BesselK ( 0, m r ) + T∞ = Tb (4)

The tip of the fin is adiabatic:

dT
− k 2 π rin =0
dr r = rout

or

d d
C1 ⎡⎣ BesselI ( 0, m r ) ⎤⎦ r = r + C2 ⎡⎣ BesselK ( 0, m r ) ⎤⎦ r = r = 0
dr out dr out

Using the rules for differentiating Bessel functions presented in Section 1.8.4 leads to:

C1 m Bessel_I (1, m rout ) − C2 m Bessel_K (1, m rout ) = 0 (5)

The boundary condition equations, Eqs. (4) and (5), can be obtained using Maple:

> BC1:=rhs(eval(Ts,r=r_in))=T_b;
BC1 := BesselI( 0, m r_in ) _C2 + BesselK ( 0, m r_in ) _C1 + T_infinity = T_b
> BC2:=rhs(eval(diff(Ts,r),r=r_out))=0;
BC2 := BesselI( 1, m r_out ) m _C2 − BesselK ( 1, m r_out ) m _C1 = 0

These equations can be copied into EES in order to obtain the solution for arbitrary conditions:

"Boundary conditions"
theta_b = C_1*BesselI(0, m*r_tube)+C_2*BesselK(0, m*r_tube)
0 = C_1*BesselI(1, m*r_fin)*m-C_2*BesselK(1, m*r_fin)*m

"Temperature distribution"
theta = C_1*BesselI(0, m*r)+C_2*BesselK(0, m*r)

Given arbitrary values of the variables T_b, T_infinity, m, r_in, and r_out, the EES code above
will provide the temperature distribution.

It is convenient to solve for the two constants explicitly and substitute them into the temperature
distribution; we can let Maple accomplish this process and avoid the algebra. The first step is to
solve the two boundary conditions equations simultaneously to obtain the unknown constants;
this is done using the solve command in Maple where the first argument is the set of equations
(BC1 and BC2) and the second are the arguments to be solved for (_C1 and _C2):
> constants:=solve({BC1,BC2},{_C1,_C2});
BesselK ( 1, m r_out ) ( −T_infinity + T_b )
constants := { _C2 =
BesselK ( 1, m r_out ) BesselI( 0, m r_in ) + BesselK ( 0, m r_in ) BesselI(
BesselI( 1, m r_out ) ( −T_infinity + T_b )
, _C1 =
BesselK ( 1, m r_out ) BesselI( 0, m r_in ) + BesselK ( 0, m r_in ) BesselI( 1, m r_o
}

These equations for the constants can be substituted into the solution using the eval command,
where the first argument is the base expression and the second contains the sub-expressions that
must be substituted into the base expression:

> Ts:=eval(Ts,constants);
BesselI( 0, m r ) BesselK ( 1, m r_out ) ( −T_infinity + T_b )
Ts := T( r ) =
BesselK ( 1, m r_out ) BesselI( 0, m r_in ) + BesselK ( 0, m r_in ) BesselI( 1, m r_
BesselK ( 0, m r ) BesselI( 1, m r_out ) ( −T_infinity + T_b )
+
BesselK ( 1, m r_out ) BesselI( 0, m r_in ) + BesselK ( 0, m r_in ) BesselI( 1, m r_out )
+ T_infinity

which can be copied into EES in place of the 3 original equations:

"Explicit solution"
T = BesselI(0,m*r)*BesselK(1,m*r_out)*(-T_infinity+T_b)/(BesselK(1,m*r_out)*BesselI(0,m*r_in)+&
BesselK(0,m*r_in)*BesselI(1,m*r_out))+BesselK(0,m*r)*BesselI(1,m*r_out)*(-T_infinity+T_b)&
/(BesselK(1,m*r_out)*BesselI(0,m*r_in)+BesselK(0,m*r_in)*BesselI(1,m*r_out))+T_infinity

So the temperature distribution through the circular fin is given by:

⎡ BesselK (1, m rout ) BesselI ( 0, m r ) + BesselI (1, m rout ) BesselK ( 0, m r ) ⎤⎦


T = T∞ + (Tb − T∞ ) ⎣ (6)
⎡⎣ BesselI (1, m rout ) BesselK ( 0, m rin ) + BesselI ( 0, m rin ) BesselK (1, m rout ) ⎤⎦

The heat transfer rate to the base of the fin, q fin , is obtained by applying Fourier’s law to
evaluate the conduction heat transfer rate at the base of the fin:

dT
q fin = −k 2 π rin th (7)
dr r = rin

Substituting Eq. (6) into Eq. (7) leads to:


⎧ d ⎫
⎪ BesselK (1, m rout ) ⎡⎣ BesselI ( 0, m r ) ⎤⎦ r = r ⎪
dr in
+⎪

⎪ BesselI (1, m rout ) BesselK ( 0, m rin ) + BesselI ( 0, m rin ) BesselK (1, m rout ) ⎪
q fin = − k 2 π rin th (Tb − T∞ ) ⎨ ⎬
⎪ d ⎪
BesselI (1, m rout ) ⎡⎣ BesselK ( 0, m r ) ⎤⎦ r = r
⎪ dr in ⎪
⎪ BesselI (1, m r ) BesselK ( 0, m r ) + BesselI ( 0, m r ) BesselK (1, m r ) ⎪
⎩ out in in out ⎭

Using the rules for differentiating Bessel functions, presented in Section 1.8.4, to evaluate the
derivatives leads to:

⎡ BesselK (1, m rout ) BesselI (1, m rin ) -BesselI (1, m rout ) BesselK (1, m rin ) ⎤⎦
q fin = − k 2 π rin th (Tb − T∞ ) m ⎣
⎡⎣ BesselI (1, m rout ) BesselK ( 0, m rin ) + BesselI ( 0, m rin ) BesselK (1, m rout ) ⎤⎦
(8)

Maple achieves the same result:

> q_dot_fin:=-k*2*pi*r_in*th*rhs(eval(diff(Ts,r),r=r_in));
q_dot_fin := − 2 k π r_in th ( −BesselI( 1, m r_in ) m BesselK ( 1, m r_out ) T_infinity
+ BesselI( 1, m r_in ) m BesselK ( 1, m r_out ) T_b
+ BesselK ( 1, m r_in ) m BesselI( 1, m r_out ) T_infinity
− BesselK ( 1, m r_in ) m BesselI( 1, m r_out ) T_b )/(
BesselK ( 1, m r_out ) BesselI( 0, m r_in )
+ BesselK ( 0, m r_in ) BesselI ( 1, m r_out ) )

which can be cut and pasted directly into EES:

"Fin heat transfer rate"


q_dot_fin=-2*k*pi*r_in*th*(-BesselI(1,m*r_in)*m*BesselK(1,m*r_out)*T_infinity+&
BesselI(1,m*r_in)*m*BesselK(1,m*r_out)*T_b+BesselK(1,m*r_in)*m*BesselI(1,m*r_out)&
*T_infinity-BesselK(1,m*r_in)*m*BesselI(1,m*r_out)*T_b)/(BesselK(1,m*r_out)*&
BesselI(0,m*r_in)+BesselK(0,m*r_in)*BesselI(1,m*r_out))

Finally, the fin efficiency (ηfin) is the ratio of the heat transfer rate to the heat transfer rate from
an isothermal fin at the base temperature:

q fin
η fin =
2 π ( r − rin2 ) h (Tb − T∞ )
2
out

Substituting Eq. (8) into the definition of the fin efficiency leads to:
2 rin ⎡⎣ BesselI (1, m rout ) BesselK (1, m rin ) -BesselK (1, m rout ) BesselI (1, m rin ) ⎤⎦
η fin =
m ( rout − rin ) ⎡⎣ BesselI (1, m rout ) BesselK ( 0, m rin ) + BesselI ( 0, m rin ) BesselK (1, m rout ) ⎤⎦
2 2

which can be expressed as a function of the tube-to-fin radius ratio (rin/rout) and the product of
the fin parameter and the fin radius (m rout).

⎛r ⎞ ⎡ ⎛ rin ⎞ ⎛ rin ⎞ ⎤
2 ⎜ in ⎟ ⎢ BesselI (1, m rout ) BesselK ⎜ 1, m rout ⎟ -BesselK (1, m rout ) BesselI ⎜ 1, m rout ⎟⎥
⎝ rout ⎠ ⎣ ⎝ rout ⎠ ⎝ rout ⎠ ⎦
η fin =
⎛ ⎛ r ⎞2 ⎞ ⎡ ⎛ rin ⎞ ⎛ rin ⎞ ⎤
m rout ⎜ 1 − ⎜ in ⎟ ⎟ ⎢ BesselI (1, m rout ) BesselK ⎜ 0, m rout ⎟ + BesselI ⎜ 0, m rout ⎟ BesselK (1, m rout ) ⎥
⎜ ⎝ rout ⎠ ⎟ ⎣ ⎝ rout ⎠ ⎝ rout ⎠ ⎦
⎝ ⎠

The fin efficiency for a circular fin is shown in Figure 3 as a function of m rout for various values
of rin/rout. Note that the fin radius can be corrected approximately to account for convection from
the tip by adding the half-thickness of the fin; as previously discussed in Section 1.6.5, this
correction is small and rarely worth considering.

Figure 3: Fin efficiency of a circular fin as a function of m rout for various values of rin/rout.
Problem 1.8-5 (1-18 in text): Optimizing a Fin
Figure P1.8-5 illustrates a fin that is to be used in the evaporator of a space conditioning system
for a space-craft.

h = 120 W/m -K
2

x th = 1 mm
T∞ = 20°C
L = 2 cm k = 50 W/m-K
ρ = 3000 kg/m3

ρb = 8000 kg/m3 Tb = 10°C

thb = 2 mm thg = 2 mm

Wb = 1 cm
Figure P1.8-5: Fin on an evaporator.

The fin is a plate with a triangular shape. The thickness of the plate is th = 1 mm and the width
of the fin at the base is Wb = 1 cm. The length of the fin is L = 2 cm. The fin material has
conductivity k = 50 W/m-K. The average heat transfer coefficient between the fin surface and
the air in the space-craft is h = 120 W/m2-K. The air is at T∞ = 20°C and the base of the fin is at
Tb = 10°C. Assume that the temperature distribution in the fin is 1-D in x. Neglect convection
from the edges of the fin.
a.) Obtain an analytical solution for the temperature distribution in the fin. Plot the temperature
as a function of position.

The inputs are entered in EES:

$UnitSystem SI MASS RAD PA K J


$TABSTOPS 0.2 0.4 0.6 0.8 3.5 in

h_bar=120 [W/m^2-K] "average heat transfer coefficient"


k=50 [W/m-K] "conductivity"
T_infinity=converttemp(C,K,20[C]) "air material"
T_b=converttemp(C,K,10[C]) "base temperature"
th_mm= 1 [mm] "fin thickness in mm"
th=th_mm*convert(mm,m) "fin thickness"
L_cm=2 [cm] "fin length in cm"
L=L_cm*convert(cm,m) "fin length"
W_b=1 [cm]*convert(cm,m) "fin base width"

The differential control volume shown in Figure P1.8-5-2 is used to derive the governing
differential equation:

q x = q x + dx + qconv (1)
x
qx
qconv
q x + dx

Figure P1.8-5-2: Differential control volume.

The rate of conduction and convection are:

dT
q x = − k Ac (2)
dx

qconv = h per (T − T∞ ) dx (3)

where Ac is the cross-sectional area for conduction and per is the perimeter. The width of the fin
is a function of x:

x
W = Wb (4)
L

Therefore, Ac and per are:

x
Ac = Wb th (5)
L

x
per = 2Wb (6)
L

Substituting Eqs. (5) and (6) into Eq. (2) and (3) leads to:

x dT
q x = − k Wb th (7)
L dx

x
qconv = h 2Wb (T − T∞ ) dx (8)
L

Substituting Eqs. (7) and (8) into Eq. (1) leads to:

d ⎡ x dT ⎤ x
0= ⎢ − k Wb th ⎥ dx + h 2Wb (T − T∞ ) dx (9)
dx ⎣ L dx ⎦ L

Simplifying:
d ⎛ dT ⎞
⎟ − m xT = −m xT∞
2 2
⎜x (10)
dx ⎝ dx ⎠

where

2h
m2 = (11)
k th

m=sqrt(2*h_bar/(k*th)) "solution parameter"

Maple is used to identify the solution to Eq. (10):

> restart;
> ODE:=diff(x*diff(T(x),x),x)-m^2*x*T(x)=-m^2*x*T_infinity;
2
d ⎛d ⎞
ODE := ⎛⎜⎜ T( x ) ⎞⎟⎟ + x ⎜⎜ 2 T( x ) ⎟⎟ − m 2 x T( x ) = −m 2 x T_infinity
⎝ dx ⎠ ⎝ dx ⎠
> Ts:=dsolve(ODE);
Ts := T( x ) = BesselI( 0, m x ) _C2 + BesselK( 0, m x ) _C1 + T_infinity

Therefore:

T = C2 BesselI ( 0, m x ) + C1 BesselK ( 0, m x ) + T∞ (12)

The fin temperature at the tip must be bounded:

Tx =0 = C2 BesselI ( 0, m 0 ) + C1 BesselK ( 0, m 0 ) + T∞ < ∞ (13)





1 ∞

The limit of the 0th order modified Bessel functions as x → 0 are evaluated using Maple:

> limit(BesselI(0,m*x),x=0);
1
> limit(BesselK(0,m*x),x=0);

Therefore, C1 must be zero:

T = C2 BesselI ( 0, m x ) + T∞ (14)

The base temperature is specified; therefore:


Tb = C2 BesselI ( 0, m L ) + T∞ (15)

so:

C2 =
(Tb − T∞ ) (16)
BesselI ( 0, m L )

Substituting Eq. (16) into Eq. (14) leads to:

BesselI ( 0, m x )
T = (Tb − T∞ ) + T∞ (17)
BesselI ( 0, m L )

x_bar=0.5 [-] "dimensionless position"


x=x_bar*L "position"
T=(T_b-T_infinity)*BesselI(0,m*x)/BesselI(0,m*L)+T_infinity "temperature"
T_C=converttemp(K,C,T) "in C"

Figure P1.8-5-3 illustrates the temperature as a function of position normalized by the fin length.
14

13.5

13
Temperature (°C)

12.5

12

11.5

11

10.5

10
0 0.1 0.2 0.3 0.4 0.5 0.6 0.7 0.8 0.9 1
Normalized position, x/L
Figure P1.8-5-3: Fin temperature as a function of dimensionless position.

b.) Calculate the rate of heat transfer to the fin.

The rate of heat transfer to the fin is computed according to:

dT
q fin = k Wb th (18)
dx x= L

Equation (18) is evaluated using Maple:

> restart;
> T:=(T_b-T_infinity)*BesselI(0,m*x)/BesselI(0,m*L)+T_infinity;
( T_b − T_infinity ) BesselI( 0, m x )
T := + T_infinity
BesselI( 0, m L )
> q_dot_fin=k*W_b*th*eval(diff(T,x),x=L);
k W_b th ( T_b − T_infinity ) BesselI( 1, m L ) m
q_dot_fin =
BesselI( 0, m L )

Therefore:

BesselI (1, m L )
q fin = k Wb th m (Tb − T∞ ) (19)
BesselI ( 0, m L )

q_dot_fin=k*W_b*th*(T_b-T_infinity)*m*BesselI(1,m*L)/BesselI(0,m*L) "fin heat transfer rate"

which leads to q fin = -0.196 W (the heat transfer is negative because the base temperature is less
than the ambient temperature).

c.) Determine the fin efficiency.

The fin efficiency is defined according to:

q fin
η fin = (20)
h As (Tb − T∞ )

where As is the total surface area of the fin exposed to the fluid:

As = Wb L (21)

Substituting Eqs. (19) and (21) into Eq. (20) leads to:

k Wb th m (Tb − T∞ ) BesselI (1, m L )


η fin = (22)
h Wb L (Tb − T∞ ) BesselI ( 0, m L )

Substituting Eq. (11) into Eq. (22) and simplifying leads to:

k th 2 h BesselI (1, m L ) 2 th k BesselI (1, m L )


η fin = = (23)
h L k th BesselI ( 0, m L ) L 2 h BesselI ( 0, m L )
N
1/ m

or

2 BesselI (1, mL )
η fin = (24)
mL BesselI ( 0, mL )

eta_fin=2*BesselI(1,m*L)/(m*L*BesselI(0,m*L)) "fin efficiency"


which leads to ηfin = 0.8178. Figure P1.8-5-4 illustrates the fin efficiency as a function of the fin
parameter mL.
1

0.9

0.8

0.7

Fin efficiency
0.6

0.5

0.4

0.3
0 0.5 1 1.5 2 2.5 3 3.5 4 4.5 5
Fin parameter, mL
Problem P1.8-5-4: Fin efficiency as a function of the fin parameter, mL.

The fin has density ρ = 3000 kg/m3. The fin is installed on a base material with thickness thb = 2
mm and density ρb = 8000 kg/m3. The half-width between the gap between adjacent fins is thg =
2 mm. Therefore, the volume of the base material associated with each fin is thb Wb (th + 2 thg).
d.) Determine the ratio of the absolute value of the rate of heat transfer to the fin to the total
mass of material (fin and base material associated with the fin).

The additional inputs are entered in EES:

rho=3000 [kg/m^3] "density of fin material"


th_b=2 [mm]*convert(mm,m) "thickness of base material"
th_g=2 [mm]*convert(mm,m) "half-width of gap between adjacent fins"
rho_b=8000 [kg/m^3] "base material density"

The fin mass is given by:

Wb L
M fin = th ρ (25)
2

The mass of the associated base material is:

M b = Wb ( th + 2 thg ) thb ρb (26)

The ratio of rate of the fin heat transfer to mass is:

q fin q fin
= (27)
M (M fin + Mb )

M_fin=W_b*L*th*rho/2 "fin mass"


M_b=W_b*(th+2*th_g)*th_b*rho_b "mass of base material"
q\M=abs(q_dot_fin)/(M_fin+M_b) "ratio of heat transfer to mass"
which leads to q fin /M = 178.4 W/kg.

e.) Prepare a contour plot that shows the ratio of the heat transfer to the fin to the total mass of
material as a function of the length of the fin (L) and the fin thickness (th).

A parametric table is generated that contains the variables L_cm, th_mm and q\M and has 400
rows. The value of the variable L_cm is varied from 1 cm to 10 cm every 20 rows and the value
of th_mm is varied from 0.2 mm to 2 mm in increments of 20 rows. The table is run and used to
generate the contour plot shown in Figure P1.8-5-5.
2
139.3 115.9
1.8
1.6 127.6
162.7
1.4 151
Fin thickness (mm)

1.2 174.4
186.1
1
197.8
0.8
209.5 W/kg
0.6
0.4
0.2
0
1 2 3 4 5 6 7 8 9 10
Fin length (cm)
Figure P1.8-5-5: Contours of heat transfer per mass in the parameter space of fin length and thickness.

f.) What is the optimal value of L and th that maximizes the absolute value of the fin heat
transfer rate to the mass of material?

According to Figure P1.8-5-5, the optimal design is approximately L = 3.3 cm and th = 0.58 mm.
A more precise optimization can be carried out using EES' internal optimization feature.
Maximizing q fin /M by varying L and th leads to q fin /M = 209.6 W/kg at L = 3.25 cm and th =
0.56 mm.
Problem 1.8-6: Cryosurgical Probe
As an alternative to surgery, cancer tumors may be destroyed by placing cylindrically-shaped
cryoprobes into the body, as shown in Figure P1.8-6. The probe surface is cooled causing the
temperature of the surrounding tissue to drop to a lethal level, killing the tumor.

tissue
k = 0.6 W/m-K
β = 40000 W/m3-K

q ′′p = 30000 W/m


2

Tb = 37°C

rp = 5 mm
Figure 1.8-6: Cryosurgical probe.

The probe radius is rp = 5 mm and the heat flux at the surface of the probe (leaving the tissue) is
q ′′p = 30000 W/m2. The tissue has conductivity k = 0.6 W/m-K. The blood flow through the
tissue results in a volumetric heating effect ( g ′′′ ) that is proportional to the difference between
the local temperature and the blood temperature, Tb = 37ºC:

g ′′′ = β (Tb − T )

where β = 40000 W/m3-K. The temperature of the tissue far from the probe is Tb. Assume that
the temperature distribution is 1-D and steady-state.
a.) Develop an analytical model that can be used to predict the temperature distribution in the
tissue. Implement your solution in EES and prepare a plot of the temperature distribution as
a function of radius.

The differential control volume shown in Figure 2 can be used to derive the governing equation.

qr qr + dr
g

dr
Figure 2: Differential control volume.

An energy balance for the control volume is:

g + qr = qr + dr (1)


or

dq
g = dr (2)
dr

The conduction and blood perfusion terms are:

dT
q = − k 2 π r L (3)
dr

g = 2 π r dr L β (Tb − T ) (4)

Combining these equations leads to:

d ⎡ dT ⎤
2 π r dr L β (Tb − T ) = ⎢ −k 2 π r L dr (5)
dr ⎣ dr ⎥⎦

which can be simplified to:

d ⎡ dT ⎤ β β
r − r T = − r Tb (6)
dr ⎢⎣ dr ⎥⎦ k k

The solution is divided into a homogeneous and particular component:

T = Th + Tp (7)

which leads to:

d ⎡ d (Th + Tp ) ⎤ β β
⎢r ⎥ − r (Th + Tp ) = −r Tb (8)
dr ⎢ dr ⎥⎦ k k

or

d ⎡ dTh ⎤ β d ⎡ dT ⎤ β β
⎢ r ⎥ − r Th + ⎢ r p ⎥ − r Tp = −r Tb (9)
dr ⎣
 ⎦
dr k
dr ⎣ dr ⎦ k k


= 0 for homogeneous ODE whatever is left is particular ODE

The solution to the particular differential equation:

d ⎡ dTp ⎤ β β
⎢r ⎥ − r Tp = − r Tb (10)
dr ⎣ dr ⎦ k k
is

Tp = Tb (11)

The homogeneous differential equation is:

d ⎡ dTh ⎤ β
⎢ r ⎥ − r Th = 0 (12)
dr ⎣ dr ⎦ k

Equation (12) is a form of Bessel’s equation:

d ⎛ p dθ ⎞ 2 s
⎜x ⎟±c x θ =0 (13)
dx ⎝ dx ⎠

where, by comparing Eqs. (12) and (13), p = 1, c = β / k , and s = 1. Referring to the flow
chart presented in Section 1.8, the value of s-p+2 is equal to 2 and therefore the solution
parameters n and a must be computed:

1−1
n= =0 (14)
1−1+ 2

2
a= =1 (15)
1−1+ 2

The last term in Eq. (12) is negative; therefore the homogeneous solution is given by:

n
(
Th = C1 x a BesselI n, c a x
1
a
)+C x
2
n
a
(
BesselK n, c a x
1
a
) (16)

or, for this problem:

⎛ β⎞ ⎛ β⎞
Th = C1 BesselI ⎜⎜ 0, r ⎟⎟ + C2 BesselK ⎜⎜ 0, r ⎟ (17)
⎝ k ⎠ ⎝ k ⎟⎠

Substituting Eqs. (11) and (17) into Eq. (7) leads to:

⎛ β⎞ ⎛ β⎞
T = C1 BesselI ⎜⎜ 0, r ⎟⎟ + C2 BesselK ⎜⎜ 0, r ⎟ + Tb (18)
⎝ k ⎠ ⎝ k ⎟⎠
Note that Maple could be used to identify this solution as well; it is necessary to specify that the
parameters β and k are positive so that Maple identifies the solution in terms of modified Bessel
functions (as opposed to Bessel functions with complex arguments):

> restart;
> assume(beta>0);
> assume(k>0);
> ODE:=diff(r*diff(T(r),r),r)-r*beta*T(r)/k=-r*beta*T_b/k;
⎞ r β∼ T( r ) r β∼ T_b
2
⎛d
ODE := ⎛⎜⎜ T( r ) ⎞⎟⎟ + r ⎜ 2 T( r ) ⎟ −
d
=−
⎝ dr ⎠ ⎜ ⎟ k~ k~
⎝ dr ⎠
> Ts:=dsolve(ODE);
⎛ β∼ r ⎞ ⎛ β∼ r ⎞
Ts := T( r ) = BesselI⎜ 0, ⎟ _C2 + BesselK⎜ 0,
⎟ ⎜
⎟ _C1 + T_b

⎝ k~ ⎠ ⎝ k~ ⎟⎠

The boundary conditions must be used to obtain the constants C1 and C2. As radius approaches
infinity, the body temperature is recovered:

Tr →∞ = Tb (19)

Substituting Eq. (18) into Eq. (19) leads to:

⎛ β⎞ ⎛ β⎞
Tr →∞ = C1 BesselI ⎜⎜ 0, ∞ ⎟⎟ + C2 BesselK ⎜⎜ 0, ∞ ⎟ + Tb = Tb (20)
⎝ k ⎠ ⎝ k ⎟⎠

or

⎛ β⎞ ⎛ β⎞
C1 BesselI ⎜⎜ 0, ∞ ⎟⎟ + C2 BesselK ⎜⎜ 0, ∞ ⎟=0 (21)
⎝ k ⎠ ⎝ k ⎟⎠



∞ 0

The 0th order modified Bessel function of the 1st kind, BesselI(0,x), approaches ∞ as x → ∞ and
0th order modified Bessel function of the 2nd kind, BesselK(0,x), approaches 0 as x → ∞:

> limit(BesselI(0,x),x=infinity);

> limit(BesselK(0,x),x=infinity);
0

Therefore, C1 must be zero and Eq. (18) becomes:

⎛ β⎞
T = C2 BesselK ⎜⎜ 0, r ⎟ + Tb (22)
⎝ k ⎟⎠
The heat flux into the probe (i.e., in the negative r-direction) at r = rp is specified, providing the
additional boundary condition:

dT
k = q ′′p (23)
dr r = rp

Substituting Eq. (22) into Eq. (23) leads to:

d ⎡ ⎛ β⎞ ⎤
k ⎢C2 BesselK ⎜⎜ 0, r ⎟⎟ + Tb ⎥ = q ′′p (24)
dr ⎢⎣ ⎝ k ⎠ ⎥⎦ r = rp

Using the rules for differentiating Bessel functions presented in Section 1.8.4 leads to:

β ⎛ β⎞
− k C2 BesselK ⎜⎜1, rp ⎟⎟ = q ′′p (25)
k ⎝ k ⎠

which leads to:

q ′′p
C2 = − (26)
⎛ β⎞
β k BesselK ⎜1, rp ⎟
⎝ k ⎠

Substituting Eq. (26) into Eq. (22) leads to:

⎛ β⎞
BesselK ⎜ 0, r ⎟
q ′′ ⎝ k ⎠
T = Tb − p (27)
βk ⎛ β⎞
BesselK ⎜1, rp ⎟
⎝ k ⎠

The solution can also be identified using Maple. Substitute C2 = 0 into the previously obtained
solution:

> Ts:=subs(_C2=0,Ts);
⎛ β∼ r ⎞
Ts := T( r ) = BesselK⎜⎜ 0, ⎟ _C1 + T_b
⎝ k~ ⎟⎠

Obtain an equation for the boundary condition associated with Eq. (23):

> BC:=k*rhs(eval(diff(Ts,r),r=r_p))=qf_p;
⎛ β∼ r_p ⎞
BC := − k~ BesselK⎜⎜ 1, ⎟ β∼ _C1 = qf_p
⎝ k~ ⎟⎠

Substitute the solution to the boundary condition equation into the general solution:

> subs(_C1=solve(BC,_C1),Ts);
⎛ β∼ r ⎞
BesselK⎜⎜ 0, ⎟ qf_p
⎝ k~ ⎟⎠
T( r ) = − + T_b
⎛ β∼ r_p ⎞
k~ BesselK⎜⎜ 1, ⎟ β∼
⎝ k~ ⎟⎠

The solution is implemented in EES. The inputs are entered:

$UnitSystem SI MASS RAD PA K J


$TABSTOPS 0.2 0.4 0.6 0.8 3.5 in

"Inputs"
r_p_mm=5 [mm] "probe radius, in mm"
r_p=r_p_mm*convert(mm,m) "probe radius"
q``_p=30000 [W/m^2] "probe heat flux"
k=0.6 [W/m-K] "tissue conductivity"
beta=40000 [W/m^3-K] "blood perfusion effect"
T_b=converttemp(C,K,37[C]) "blood temperature"

Equation (27) is implemented in EES; the radius and temperature are converted to mm and ºC,
respectively.

T=T_b-q``_p*BesselK(0,r*sqrt(beta/k))/(sqrt(beta*k)*BesselK(1,r_p*sqrt(beta/k))) "solution"
r_mm=r*convert(m,mm) "radius"
T_C=converttemp(K,C,T) "in C"

Figure 3 illustrates the temperature as a function of radius.


50

25

0
Temperature (°C)

-25

-50

-75

-100

-125
5 10 15 20 25 30
Radius (mm)
Figure 3: Temperature in tissue as a function of radius.
b.) The lethal temperature for cell death is Tlethal = -30ºC. Plot the radius of the cryolesion (i.e.,
the kill radius - all tissue inside of this radius is dead) as a function of the heat flux provided
by the cryoprobe.

The temperature is set to the lethal temperature. The variable r must be constrained to be
positive in the Variable Information Window to avoid convergence errors. The kill radius as a
function of heat flux is shown in Figure 4.
9

7
Kill radius (mm)

2
5.0x103 1.5x104 2.5x104 3.5x104 4.5x104
2
Probe flux (W/m )
Figure 4: Kill radius as a function of the cryoprobe heat flux.
Problem 1.8-7: Bracket
A disk-shaped bracket connects a cylindrical heater to an outer shell, as shown in Figure P1.8-
7(a).

center line shell


rt
rb h , T∞
Tt
q k
h , T∞
th
bracket
r heater
Figure P1.8-7(a): Disk-shaped bracket.

The thickness of the bracket is th and it is made of material with conductivity k. The bracket
extends radially from rb at the heater to rt at the outer shell. The temperature of the bracket
location where it intersects the shell (at r = rt) is Tt. The heater provides a rate of heat transfer to
the bracket, q , at r = rb. Both the upper and lower surfaces of the bracket are exposed to fluid at
T∞ with average heat transfer coefficient h .

a.) What calculation would you do in order to justify treating the bracket as an extended surface
(i.e., justify the assumption that temperature is only a function of r); provide an expression in
terms of the symbols in the problem statement.

The appropriate Biot number is the ratio of the resistance to conduction across the thickness of
the fin to the resistance to convection from the fin surface. The resistance to conduction across
the thickness of the fin (i.e., in the x direction) is:

th
Rcond , x = (1)
2 k π ( rt 2 − rb2 )

The resistance to convection from the fin surface is:

1
Rconv = (2)
h π ( rt 2 − rb2 )

The Biot number is therefore:

Rcond , x th h π ( rt − rb ) th h
2 2

Bi = = = (3)
Rconv 2 k π ( rt 2 − rb2 ) 1 2k

For the remainder of the problem, assume that the bracket can be treated as an extended surface.
center line
rt
rb h , T∞
Tt
q

h , T∞

r
T h →∞

T∞

h →0

Tt
r
rb rt
Figure P1.8-7(b): Qualitative sketch of the temperature distribution expected if h → 0 and h → ∞.

b.) On the axes in Figure P1.8-7(b), sketch the temperature distribution that you would expect if
the heat transfer coefficient h is very low ( h → 0). Note that the qualitative values of Tt
and T∞ are indicated in the plot - your sketch should be consistent with these values.

The sketch is shown in Figure P1.8-7(a) and has the following characteristics.
1. The slope at r = rb should be negative due to the heat transfer at the base.
2. The temperature at r = rt must be Tt.
3. The surfaces of the bracket are insulated if h → 0; therefore, the conduction heat transfer
is constant in the r-direction. Because the area for conduction increases with r, the
temperature gradient must decrease.

c.) On the axes in Figure P1.8-7(c), sketch the temperature distribution that you would expect if
the heat transfer coefficient h is very high ( h → ∞). Note that the qualitative values of Tt
and T∞ are indicated in the plot - your sketch should be consistent with these values.

The sketch is also shown in Figure P1.8-7(a) and has the following characteristics.
1. The slope at r = rb is negative due to the heat transfer at the base and is identical to the
slope of the distribution from (b).
2. The temperature at r = rt must be Tt.
3. The temperature of the bracket otherwise tends to approach T∞ due to the strong coupling
between the surface and the surrounding fluid. The temperature must approach T∞ from
above at the base (because you are transferring heat from the bracket to the fluid) and
from below at the tip (because you are transferring from the fluid to the bracket.
d.) What dimensionless number would you calculate in order to determine whether the actual
temperature distribution is closer to your sketch from (b) or (c)? Provide an expression in
terms of the symbols in the problem statement.

The behavior of the bracket is governed by two resistances: the resistance to convection from the
surface of the bracket, Eq. (2), and the resistance to conduction in the radial direction:

⎛r ⎞
ln ⎜ t ⎟
r
Rcond ,r = ⎝ b⎠ (4)
2 π th k

The ratio of Rcond,r to Rconv governs the behavior:

⎛r ⎞
ln ⎜ t ⎟
rb ⎠ h π ( rt − rb ) ⎛ rt ⎞ h ( rt − rb )
2 2 2 2
Rcond ,r ⎝
= = ln ⎜ ⎟ (5)
Rconv 2 π th k 1 ⎝ rb ⎠ 2 th k

If Rcond,r/Rconv <<1, then the distribution will approach your answer from (b). If Rcond,r/Rconv >>1,
then the distribution will approach your answer from (c).

e.) Derive the governing ordinary differential equation for the bracket.

An energy balance on a differential segment of the bracket is shown in Figure P1.8-7(c).

center line
rt
rb dr Tt
q qr qr + dr

qconv

r
Figure P1.8-7(c): Energy balance on a differential segment of the bracket.

The energy balance in Figure P1.8-7(c) leads to:

qr = qr + dr + qconv (6)

or

dq
0= dr + qconv (7)
dr
Substituting rate equations into Eq. (7) leads to:

d ⎡ dT ⎤
0= ⎢ −k 2 π r th dr + h 4 π r dr (T − T∞ ) (8)
dr ⎣ dr ⎥⎦

or

d ⎡ dT ⎤
⎢ r ⎥ − β 2 r T = − β 2 r T∞ (9)
dr ⎣ dr ⎦

where

2h
β2 = (10)
k th

f.) What are the boundary conditions for the ordinary differential equation from (e)?

The boundary conditions are:

dT
− k 2 π rb th = q (11)
dr r = rb

and

Tr = rt = Tt (12)

g.) Obtain a solution to your ordinary differential equation that includes two undetermined
constants.

The solution is split into a homogeneous and particular component:

T = Th + Tp (13)

which leads to:

d ⎡ dTh ⎤ d ⎡ dTp ⎤
⎢ r ⎥ − β 2
r Th + ⎢ r ⎥ − β 2 r Tp = − β 2 r T∞ (14)
dr ⎣ dr
 ⎦
dr ⎣ dr ⎦



homogeneous ODE particular ODE

The solution to the particular ordinary differential equation is:


Tp = T∞ (15)

Comparing the homogeneous ordinary differential equation to Bessel's equation and following
the flow chart provided in Figure 1-54 of your notes leads to:

Th = C1 BesselI ( 0,β r ) + C2 BesselK ( 0,β r ) (16)

Substituting Eqs. (15) and (16) into Eq. (13) leads to:

T = C1 BesselI ( 0,β r ) + C2 BesselK ( 0,β r ) + T∞ (17)

h.) Write the two algebraic equation that could be solved to provide the undetermined constants
(don't solve these equations).

Substituting Eq. (17) into Eq. (12) leads to:

C1 BesselI ( 0,β rt ) + C2 BesselK ( 0,β rt ) + T∞ = Tt (18)

Substituting Eq. (17) into Eq. (11) leads to:

− k 2 π rb th β ⎡⎣C1 BesselI (1,β rb ) − C2 BesselK (1,β rb ) ⎤⎦ = q (19)


Problem 1.8-8
Figure P1.8-8 illustrates a triangular fin with a circular cross-section.

Figure P1.8-5: Fin on an evaporator.

The fin is surrounded by fluid at T∞ with heat transfer coefficient h . The base of the fin is at Tb
and the fin conductivity is k.
a.) Derive the governing differential equation and the boundary conditions for the problem.

The x-coordinate is defined as starting from the tip of the fin and moving to the base. The cross-
sectional area is therefore:

D2 x2
Ac = π (1)
4 L2

and the perimeter (assuming that L >> D) is:

x
per = π D (2)
L
A differential energy balance leads to:

q x = q x + dx + qconv (3)

The rate of conduction and convection are:

dT
q x = − k Ac (4)
dx

qconv = h per (T − T∞ ) dx (5)

Substituting Eqs. (1), (2), (4), and (5) into Eq. (3) and expanding the x+dx term leads to:

d ⎡ D 2 x 2 dT ⎤ x
0= ⎢ − k π ⎥ + h π D (T − T∞ ) (6)
dx ⎣ 4 L dx ⎦
2
L
Simplifying:

d ⎡ 2 dT ⎤
⎢ x ⎥ − m 2 x (T − T∞ ) = 0 (7)
dx ⎣ dx ⎦

where

4h L
m2 = (8)
kD

The boundary conditions are:

Tx =0 must be bounded (9)

Tx = L = Tb (10)

b.) Normalize the governing differential equation and the boundary conditions. This process
should lead to the identification of a dimensionless fin parameter that governs the solution.
Identify the physical significance of this parameter.

Dimensionless position and temperature difference are defined:

x
x = (11)
L

T − T∞
θ = (12)
Tb − T∞

Substituting Eqs. (11) and (12) into Eq. (7) leads to:

d ⎡ 2 dθ ⎤
(Tb − T∞ ) ⎢ x ⎥ − m 2 L x (Tb − T∞ ) θ = 0 (13)
dx ⎣ dx ⎦

or

d ⎡ 2 dθ ⎤
⎢ x ⎥ − ( m 2 L ) x θ = 0 (14)
dx ⎣ dx ⎦

where

4 h L2
m2 L = (15)
kD
is the dimensionless parameter identified by the process. The dimensionless parameter is
nominally equal to the ratio of the resistance to conduction along the fin (Rcond) to the resistance
to convection from the fin surface (Rconv):

8L
Rcond ≈ (16)
π D2 k

2
Rconv ≈ (17)
π D Lh

Rcond 8 L π D L h 4 L2 h
= = = m2 L (18)
Rconv π D 2 k 2 Dk

The normalized boundary conditions are:

θx =0 must be bounded (19)

θx =1 = 1 (20)

c.) Solve the differential equation subject to the boundary conditions.

The differential equation:

d ⎡ 2 dθ ⎤
⎢ x  ⎥ − ( m L ) x θ = 0
2  (21)

dx ⎣ dx ⎦

is homogeneous and a form of the Bessel's equation:

d ⎛ p dθ ⎞ 2 s
⎜x ⎟±c x θ = 0 (22)
dx ⎝ dx ⎠

where p = 2, c = m L , and s = 1. The parameter s - p + 2 is 1; therefore the solution parameters


are:

1− 2
n= = −1 (23)
1− 2 + 2

2
a= =2 (24)
1− 2 + 2
n 1− 2 1
= =− (25)
a 2 2

The solution is therefore:

θ = C1
(
BesselI −1, 2 m L x
1
2
) + C BesselK ( −1, 2 m
2
L x
1
2
) (26)
x x

The same solution can be identified in Maple:

> restart;
> assume(m2L,positive);
> ODE:=diff(x^2*diff(q(x),x),x)-m2L*x*q(x)=0;
⎛ d ⎞ 2⎛ d
2

ODE := 2 x ⎜⎜ q( x ) ⎟⎟ + x ⎜ 2 q( x ) ⎟ − m2L~ x q( x ) = 0
⎝ dx ⎜ ⎟
⎠ ⎝ dx ⎠
> qs:=dsolve(ODE);
_C1 BesselI( 1, 2 m2L~ x) _C2 BesselK( 1, 2 m2L~ x)
qs := q( x ) = +
x x

Note that BesselI ( −1, x ) is equal to BesselI (1, x ) and so Maple has identified the same solution
that we found manually. The boundary condition:

θx =0 must be bounded (27)

requires that C2 in Eq. (26) must be zero. To see this, take the limit of the second term as x goes
to zero.

> limit(BesselK(-1,x)/x,x=0);

Therefore:

θ = C1
(
BesselI −1, 2 m L x
1
2
) (28)
x

The second boundary condition:

θx =1 = 1 (29)

leads to:
(
1 = C1 BesselI −1, 2 m L ) (30)

Substituting Eq. (30) into Eq. (28) leads to:

θ =
(
BesselI −1, 2 m L x
1
2
) (31)
(
BesselI −1, 2 m L ) x

d.) Prepare a plot of dimensionless temperature as a function of dimensionless position for


various values of the remaining dimensionless parameter, identified in (b).

The solution is programmed in EES:

"P1.8-8"
$UnitSystem SI MASS RAD PA K J
$TABSTOPS 0.2 0.4 0.6 0.8 3.5 in

msqrtL=10 [-] "fin parameter"


theta_bar=BesselI(-1,2*msqrtL*sqrt(x_hat))/(BesselI(-1,2*msqrtL)*sqrt(x_hat)) "temperature solution"

and used to generate Figure 2.


1
0.1 0.2
0.9 0.5
Dimensionless temperature

0.8
1
0.7

0.6

0.5 2

0.4

0.3 5

0.2
1/2
mL = 10
0.1

0
0 0.1 0.2 0.3 0.4 0.5 0.6 0.7 0.8 0.9 1
Dimensionless position
Figure 2: Dimensionless temperature as a function of dimensionless position for various values of the fin
parameter.

e.) The fin efficiency is defined as the ratio of the heat transfer into the base of the fin to the heat
transfer that would occur if the entire fin were isothermal and at the base temperature (i.e., if
the fin material were infinitely conductive). Develop an equation for the fin efficiency and
plot the fin efficiency as a function of the dimensionless fin parameter identified in (b).

The fin heat transfer rate is:


π D 2 ⎛ dT ⎞
q fin = k ⎜ ⎟ (32)
4 ⎝ dx ⎠ x = L

or

π D 2 (Tb − T∞ ) ⎛ dθ ⎞
q fin = k ⎜ ⎟ (33)
4L ⎝ dx ⎠ x =1

The fin efficiency is therefore:

2 q fin 2 π D 2 (Tb − T∞ ) ⎛ dθ ⎞


η fin = =k ⎜ ⎟ (34)
π D L h (Tb − T∞ ) 4 π D L h (Tb − T∞ ) L ⎝ dx ⎠ x =1

which can be simplified to:

2 k D ⎛ dθ ⎞ 
2 ⎛ dθ ⎞
η fin = ⎜ ⎟ = 2 m L ⎜ ⎟ (35)
2 2 h L2 ⎝ dx ⎠ x =1 ⎝ dx ⎠ x =1

Substituting Eq. (31) into Eq. (35) leads to:

η fin =
2

d ⎜ BesselI −1, 2 m L x
1
 2 ( ) ⎞⎟ (36)
( ⎜
BesselI −1, 2 m L m 2 L dx ⎜

) x


⎠ x =1

The derivative in Eq. (36) is evaluated in Maple:

> restart;
> eval(diff(BesselI(-1,2*msqrtL*sqrt(x))/sqrt(x),x),x=1);
⎛ BesselI( 0, 2 msqrtL ) − 1 BesselI( 1, 2 msqrtL ) ⎞ msqrtL − 1 BesselI( 1, 2 msqrtL )
⎜⎜ ⎟⎟
⎝ 2 msqrtL ⎠ 2
> simplify(%);
BesselI( 0, 2 msqrtL ) msqrtL − BesselI( 1, 2 msqrtL )

Therefore, the fin efficiency is:

η fin
( )
2 ⎡ BesselI 0, 2 m L m L − BesselI 1, 2 m L ⎤
= ⎣ ⎦ ( ) (37)
BesselI −1, 2 m L m L
2
( )
eta_fin=2*(BesselI(0,2*msqrtL)*msqrtL-BesselI(1,2*msqrtL))/(msqrtL^2*BesselI(-1,2*msqrtL))
"fin efficiency"
Figure 3 illustrates the fin efficiency as a function of the fin parameter m L .

0.9

0.8

0.7
Fin efficiency

0.6

0.5

0.4
0.3

0.2

0.1

0
0.01 0.1 1 10 100 1000
1/2
Fin parameter, m L
Fin efficiency as a function of the fin parameter.
Problem 1.8-9
One side of a thin circular membrane is subjected to a flux of energy that varies according to:

q ′′ = a r 2 (1)

One side of the membrane is exposed to fluid at T∞ with heat transfer coefficient h . The outer
edge of the membrane is held at T∞. The radius of the membrane is ro and the thickness is th.
The conductivity of the membrane material is k. Assume that the temperature distribution in the
membrane is only a function of radius.
a.) Derive the governing differential equation for the temperature in the membrane and the
boundary conditions.

A differential energy balance leads to:

qr + q ′′ 2 π r dr = qr + dr + 2 π r dr h (T − T∞ ) (2)

Expanding the r+dr term leads to:

dq
q ′′ 2 π r dr = dr + 2 π r dr h (T − T∞ ) (3)
dr

The rate of conduction heat transfer is:

dT
q = − k 2 π r th (4)
dr

Substituting Eqs. (1) and (4) into Eq. (3) leads to:

d ⎛ dT ⎞
a 2 π r 3 dr = ⎜ −k 2 π r th ⎟ dr + 2 π r dr h (T − T∞ ) (5)
dr ⎝ dr ⎠

which can be simplified to:

d ⎛ dT ⎞
a r3 = ⎜ −k r th ⎟ + r h (T − T∞ ) (6)
dr ⎝ dr ⎠

or

d ⎛ dT ⎞ h a 3
⎜r ⎟− r (T − T∞ ) = − r (7)
dr ⎝ dr ⎠ k th k th

The boundary conditions are:


Tr = ro = T∞ (8)

Tr =0 must be bounded (9)

b.) Define a dimensionless temperature difference and radius. Use them to non-dimensionalize
the governing differential equation and boundary conditions from (a). This process should
lead to the identification of another dimensionless parameter. Explain the significance of this
parameter.

A dimensionless temperature is defined:

T − T∞
θ = (10)
ΔT

where ΔT is a normalizing temperature difference. The normalizing temperature difference is


defined based on the temperature difference that would be induced if the entire rate of energy
transfer from the heat flux were transferred convectively from the membrane. The total rate of
heat transfer from the flux is:
ro
π a ro4
q = ∫ aN
r 2 2 π r dr = (11)
0 q ′′
2

The reference temperature difference is therefore:

q π a ro4 a ro2
ΔT = = = (12)
π ro2 h 2 π ro2 h 2 h

which leads to:

2 h (T − T∞ )
θ = (13)
a ro2

A dimensionless radius is defined:

r
r = (14)
ro

Substituting Eqs. (13) and (14) into Eq. (7) leads to:

a ro d ⎛ dθ ⎞ h a ro3  a ro3 3


⎜ 
r ⎟ − 
r θ = − r (15)
2 h dr ⎝ dr ⎠ k th 2 h k th
or

d ⎛ dθ ⎞
⎜ r ⎟ − m r θ = −2 m r
2 2 3
(16)
dr ⎝ dr ⎠

where

h ro2
m2 = (17)
k th

The parameter m2 is, approximately, the ratio of the resistance to conduction along the membrane
in the radial direction to the resistance to convection from the membrane surface.

The nondimensional boundary conditions are:

θr =1 = 0 (18)

θr =0 must be bounded (19)

c.) Solve the normalized problem from (b). Prepare a plot of the dimensionless temperature as a
function of the dimensionless radius for various values of the dimensionless parameter
identified in (b).

The solution is split into a homogeneous and nonhomogeneous component:

θ = θh + θp (20)

Equation (20) is substituted into Eq. (16), leading to:

d ⎛ dθh ⎞  d ⎛ dθp ⎞

r − m 
r θ + ⎜ r ⎟ − m r θp = −2 m r
2 2 2 3
⎜ ⎟ (21)
dr ⎜⎝ dr ⎟⎠
h
dr ⎝ dr ⎠



= 0 for homogeneous differential equation whatever is left is the particular differential equation

The solution to the particular differential equation is considered first.

d ⎛ dθp ⎞
⎜ r ⎟⎟ − m r θp = −2 m r
2 2 3
(22)
dr ⎜⎝ dr ⎠

Based on the form of the left side, a second order polynomial is assumed for the particular
solution:
θp = a + b r + c r 2 (23)

Substituting Eq. (23) into Eq. (22) leads to:

d
dr
( r ( b + 2 c r ) ) − m2 r ( a + b r + c r 2 ) = −2 m2 r 3 (24)

or

b + 4 c r − m 2 r ( a + b r + c r 2 ) = −2 m 2 r 3 (25)

Equating like coefficients of r leads to:

b=0 (26)

4 c − m2 a = 0 (27)

−m2 b = 0 (28)

− m 2 c = −2 m 2 (29)

which leads to a= 8/m2, b = 0, and c = 2. Therefore, the particular solution is:

8
θp = + 2 r 2 (30)
m 2

The solution to the homogeneous differential equation:

d ⎛ dθh ⎞
⎜ r ⎟ − m r θh = 0
2
(31)
dr ⎝ dr ⎠

is obtained using the flow chart:

θh = C1 BesselI ( 0, m r ) + C2 BesselK ( 0, m r ) (32)

The solution is:

8
θ = C1 BesselI ( 0, m r ) + C2 BesselK ( 0, m r ) + + 2 r 2 (33)
m2

The same solution can be identified in Maple:

> restart;
> ODE:=diff(r*diff(q(r),r),r)-m^2*r*q(r)=-2*m^2*r^3;
d ⎛d ⎞
2
ODE := ⎛⎜⎜ q( r ) ⎞⎟⎟ + r ⎜⎜ 2 q( r ) ⎟⎟ − m 2 r q( r ) = −2 m 2 r3
⎝ dr ⎠ ⎝ dr ⎠
> qs:=dsolve(ODE);
8 + 2 m 2 r2
qs := q( r ) = BesselI( 0, m r ) _C2 + BesselK( 0, m r ) _C1 +
m2

The boundary condition:

θr =0 must be bounded (34)

is satisfied by evaluating the limits of the two Bessel functions:

> limit(BesselI(0,r),r=0);
1
> limit(BesselK(0,r),r=0);

which means that C2 in Eq. (33) must be zero:

8
θ = C1 BesselI ( 0, m r ) + + 2 r 2 (35)
m2

The boundary condition:

θr =1 = 0 (36)

is enforced:

8
C1 BesselI ( 0, m ) + +2=0 (37)
m2

which leads to:

⎛ 8 ⎞
⎜ 2 + 2⎟
m
C1 = − ⎝ ⎠ (38)
BesselI ( 0, m )

Substituting Eq. (38) into Eq. (35) leads to:


⎛ 8 ⎞
⎜ 2 + 2⎟
m
θ = − ⎝ ⎠ BesselI 0, m r + 8 + 2 r 2
( ) 2 (39)
BesselI ( 0, m ) m

Equation (39) is programmed in EES:

"P1.8-9"
$UnitSystem SI MASS RAD PA K J
$TABSTOPS 0.2 0.4 0.6 0.8 3.5 in

"Inputs"
m=1 [-] "dimensionless parameter - ratio of conduction to convection"
theta_hat=-(8/m^2+2)*BesselI(0,m*r_hat)/BesselI(0,m)+8/m^2+2*r_hat^2 "solution"

and used to generate Figure 1, which shows the dimensionless temperature as a function of
dimensionless radius.
1.5

m = 20
1.25
Dimensionless temperature

1
m = 10

0.75

m=5
0.5
m=2
0.25
m=1
m = 0.5

0
0 0.1 0.2 0.3 0.4 0.5 0.6 0.7 0.8 0.9 1

Dimensionless radius
Figure 1: Dimensionless temperature as a function of dimensionless radius for various values of m.
Problem 1.9-1: A 4-Node Numerical Simulation of a Fuse
A fuse is a long, thin piece of metal that will heat up when current is passed through it. If a large
amount of current is passed through the fuse, then the material will melt and this protects the
electrical components downstream of the fuse. Figure P1.9-1 illustrates a fuse that is composed
of a piece of metal with a square cross-section (a x a) that has length L. The conductivity of the
fuse material is k. The fuse surface experiences convection with air at temperature Ta with heat
transfer coefficient h . Radiation from the surface can be neglected for this problem. The ohmic
heating associated with the current passing through the fuse results in a uniform rate of
volumetric thermal energy generation, g ′′′ . The two ends of the fuse (at x = 0 and x = L) are held
at temperature Tb. You have been asked to generate a numerical model of the fuse. Figure P1.9-
1 also shows a simple numerical model that includes only four nodes which are positioned
uniformly along the length of the fuse.

L
A
x
node 1
node 2
k , g ′′′ A
Ta , h
node 3
node 4
a

a
section A-A
Figure P1.9-1: Fuse and a four node numerical model of the fuse.

a.) How would you determine if the extended surface approximation was appropriate for this
problem?

The Biot number compares the resistance to conduction from the center to the edge of the fuse to
the resistance to convection from the surface of the fuse; for this problem, the Biot number
should be:

ha
Bi = (1)
2k

Any formula that is within a factor of 2 of Eq. (1) is fine.

For the remainder of this problem, assume that you can use the extended surface approximation.
b.) Derive a system of algebraic equations that can be solved in order to predict the temperatures
at each of the four nodes in Figure P1.9-1 (T[1], T[2], T[3], and T[4]). Your equations
should include only those symbols defined in the problem statement. Do not solve these
equations.

An energy balance on a control volume around node 2 is shown in Figure 2.


Figure 2: Energy balance on a control volume around node 2

The energy balance suggested by Figure 2 is:

g [ 2] + q LHS [ 2] + q RHS [ 2] + qconv [ 2] = 0 (2)

The rate equations for the terms in Eq. (2) are:

L a2
g [ 2] = g ′′′ (3)
3

3 k a2
q LHS [ 2] =
L
(T [1] − T [ 2]) (4)

3 k a2
q RHS [ 2] =
L
(T [3] − T [ 2]) (5)

qconv [ 2] =
4a Lh
3
(Ta − T [ 2]) (6)

A similar process for node 3 leads to:

g [3] + q LHS [3] + q RHS [3] + qconv [3] = 0 (7)

where:

L a2
g [3] = g ′′′ (8)
3

3 k a2
q LHS [3] =
L
(T [ 2] − T [3]) (9)

3 k a2
q RHS [3] =
L
(T [ 4] − T [3]) (10)
qconv [3] =
4a Lh
3
(Ta − T [3]) (11)

The temperatures of the edge nodes are specified:

T [1] = Tb (12)

T [ 4] = Tb (13)

Equations (2) through (13) can be solved (using, for example, EES) to provide the temperature at
each node.

c.) How would you determine the amount of heat transferred from the fuse to the wall at x = 0
using your solution from (b)?

In order to determine the amount of heat transferred to the wall at x= 0, it is necessary to do an


energy balance on the control volume that surrounds node 1.

g [1] + q RHS [1] + qconv [1] = qwall (14)

where

L a2
g [1] = g ′′′ (15)
6

3 k a2
q RHS [1] =
L
(T [ 2] − T [1]) (16)

qconv [1] =
2a Lh
3
(Ta − T [1]) (17)

d.) Derive the differential equation and boundary conditions that you would need in order to
solve this problem analytically. Show your steps clearly.

A differential control volume is shown in Figure 3.

Figure 3: Differential control volume.


The energy balance suggested by Figure 3 is:

q x + g = q x + dx + qconv (18)

Expanding the x+dx term leads to:

dq
q x + g = q x + dx + qconv (19)
dx

or

dq
g = dx + qconv (20)
dx

The rate equations are:

g = a 2 g ′′′ dx (21)

dT
q = − k a 2 (22)
dx

qconv = 4 h a dx (T − Ta ) (23)

Substituting Eqs. (21) through (23) into Eq. (20) leads to:

d ⎡ dT ⎤
a 2 g ′′′ dx = ⎢ −k a 2 dx + 4 h a dx (T − Ta ) (24)
dx ⎣ dx ⎥⎦

or

d 2T 4 h g ′′′
− ( T − Ta ) = − (25)
dx 2 k a k

The boundary conditions are:

Tx =0 = Tb (26)

Tx = L = Tb (27)
Problem 1.9-2: Bracket (revisited)
Reconsider the disk-shaped bracket that was discussed in Problem P1.8-7. You have decided to
generate a numerical model of the bracket that has three nodes, positioned as shown in Figure
P1.9-2.

center line
rt
rb node 3

node 1 node 2

r
Figure P1.9-2: A 3-node numerical model of the disk-shaped bracket.

a.) Derive a system of algebraic equations that can be solved in order to predict the temperatures
at each of the three nodes in Figure P1.9-2 (T1, T2, T3). Your equations should include only
those symbols defined in the problem statement as well as the radial locations of the three
nodes (r1, r2, and r3). Do not solve these equations.

The temperature at node 3 is specified:

T3 = Tt (1)

An energy balance on node 2 is shown in Figure P1.9-2(b).

center line

qconv
node 3
q LHS q RHS

node 1 node 2

r
Figure P1.9-2(b): Energy balance on node 2.

and leads to:

q LHS + q RHS = qconv (2)

or
( T1 − T2 ) ( T3 − T2 ) ⎡⎛ r2 + r3 ⎞ 2 ⎛ r2 + r1 ⎞ 2 ⎤
2 π k th + 2 π k th = 2π h ⎢⎜ − ⎟ ⎜ ⎟ ⎥ (T2 − T∞ ) (3)
⎛r ⎞ ⎛r ⎞ ⎢⎣⎝ 2 ⎠ ⎝ 2 ⎠ ⎥⎦
ln ⎜ 2 ⎟ ln ⎜ 3 ⎟
⎝ r1 ⎠ ⎝ r2 ⎠

An energy balance on node 1 is shown in Figure P1.9-2(c).

center line

qconv
node 3
q
qRHS

node 1 node 2

r
Figure P1.9-2(c): Energy balance on node 1.

and leads to:

q + q RHS = qconv (4)

or

( T2 − T1 ) ⎡⎛ r1 + r2 ⎞ 2 2 ⎤
q + 2 π k th = 2π h ⎢⎜ −r ⎟ ⎥ (T1 − T∞ ) (5)
⎛r ⎞
1

ln ⎜ 2 ⎟ ⎣⎢⎝ 2 ⎠ ⎦⎥
⎝ r1 ⎠
Problem 1.9-3 (1-19 in text): Fiber optic bundle
A fiber optic bundle (FOB) is shown in Figure P1.9-3 and used to transmit the light for a
building application.

h = 5 W/m -K
2

T∞ = 20°C rout = 2 cm

q ′′ = 1x10 W/m
5 2

x
fiber optic bundle
Figure P1.9-3: Fiber optic bundle used to transmit light.

The fiber optic bundle is composed of several, small diameter fibers that are each coated with a
thin layer of polymer cladding and packed in approximately a hexagonal close-packed array. The
porosity of the FOB is the ratio of the open area of the FOB face to its total area. The porosity of
the FOB face is an important characteristic because any radiation that does not fall directly upon
the fibers will not be transmitted and instead contributes to a thermal load on the FOB. The
fibers are designed so that any radiation that strikes the face of a fiber is “trapped” by total
internal reflection. However, radiation that strikes the interstitial areas between the fibers will
instead be absorbed in the cladding very close to the FOB face. The volumetric generation of
thermal energy associated with this radiation can be represented by:
φ q ′′ ⎛ x ⎞
g ′′′ = exp ⎜ − ⎟
Lch ⎝ Lch ⎠
where q ′′ = 1x105 W/m2 is the energy flux incident on the face, φ = 0.05 is the porosity of the
FOB, x is the distance from the face, and Lch = 0.025 m is the characteristic length for absorption
of the energy. The outer radius of the FOB is rout = 2 cm. The face of the FOB as well as its
outer surface are exposed to air at T∞ = 20°C with heat transfer coefficient h = 5 W/m2-K. The
FOB is a composite structure and therefore conduction through the FOB is a complicated
problem involving conduction through several different media. Section 2.9 discusses methods
for computing the effective thermal conductivity for a composite. The effective thermal
conductivity of the FOB in the radial direction is keff,r = 2.7 W/m-K. In order to control the
temperature of the FOB near the face where the volumetric generation of thermal energy is
largest, it has been suggested that high conductivity filler material be inserted in the interstitial
regions between the fibers. The result of the filler material is that the effective conductivity of
the FOB in the axial direction varies with position according to:
⎛ x ⎞
keff , x = keff , x ,∞ + Δkeff , x exp ⎜ − ⎟
⎝ Lk ⎠
where keff,x,∞ = 2.0 W/m-K is the effective conductivity of the FOB in the x-direction without
filler material, Δkeff,x = 28 W/m-K is the augmentation of the conductivity near the face, and Lk =
0.05 m is the characteristic length over which the effect of the filler material decays. The length
of the FOB is effectively infinite.
a.) Is it appropriate to use a 1-D model of the FOB?
The inputs are entered in EES and functions are defined to return the volumetric generation and
effective conductivity in the x-direction:

$UnitSystem SI MASS RAD PA K J


$TABSTOPS 0.2 0.4 0.6 0.8 3.5 in

function k_FOB(x)
k_eff_x_infinity=2 [W/m-K] "conductivity far from the face"
L_k=0.05 [m] "characteristic length of elevated conductivity"
Dk_eff_x=28 [W/m-K] "conductivity elevation at the face due to filler material"
k_FOB=k_eff_x_infinity+Dk_eff_x*exp(-x/L_k) "conductivity"
end

function gv_FOB(x)
phi=0.05 [-] "porosity"
q``=1e5 [W/m^2] "incident heat flux"
L_ch=0.025 [m] "characteristic length for absorption"
gv_FOB=phi*q``*exp(-x/L_ch)/L_ch "volumetric rate of thermal energy generation"
end

"Inputs"
k_eff_r=2.7 [W/m-K] "effective conductivity in the radial direction"
r_out=2 [cm]*convert(cm,m) "radius of FOB"
h_bar=5 [W/m^2-K] "heat transfer coefficient"
T_infinity=converttemp(C,K,20[C]) "ambient temperature"

A Biot number is defined according to:

h rout
Bi = (1)
keff , r

which leads to Bi = 0.037, justifying an extended surface model of the FOB.

b.) Assume that your answer to (a) was yes. Develop a numerical model of the FOB.

Nodes are positioned along the FOB. The FOB is infinitely long; however, the first L = 0.75 m
of the bundle is simulated. Examination of the solution shows that this is sufficient to capture
the end effects.

L=0.75 [m] "length of FOB to simulate"


N=41 [-] "number of nodes"
Dx=L/(N-1) "distance between adjacent nodes"
duplicate i=1,N
x[i]=Dx*(i-1) "position of each node"
end

An energy balance on node 1 leads to:

Δx π rout
2
2 Δx
h π r (T∞ − T1 ) + h 2 π rout
2
out (T∞ − T1 ) + keff , x , x =( x1 + x2 ) / 2 (T2 − T1 ) + g ′′′x = x1 π rout =0 (2)
2 Δx 2
h_bar*pi*r_out^2*(T_infinity-T[1])+h_bar*2*pi*r_out*(Dx/2)*(T_infinity-T[1])+&
pi*r_out^2*k_FOB((x[1]+x[2])/2)*(T[2]-T[1])/Dx+gv_FOB(x[1])*pi*r_out^2*Dx/2=0
"energy balance on node 1"

Energy balances on the internal nodes lead to:

π rout
2
h 2 π rout Δx (T∞ − Ti ) + keff , x , x =( xi + xi+1 ) / 2 (Ti +1 − Ti ) +
Δx
π rout
2
keff , x , x =( xi + xi−1 ) / 2 (Ti −1 − Ti ) + g ′′′x = xi π rout
2
Δx = 0 (3)
Δx
i = 2.. ( N − 1)

duplicate i=2,(N-1)
h_bar*2*pi*r_out*Dx*(T_infinity-T[i])+pi*r_out^2*k_FOB((x[i]+x[i+1])/2)*(T[i+1]-T[i])/Dx+&
pi*r_out^2*k_FOB((x[i]+x[i-1])/2)*(T[i-1]-T[i])/Dx+gv_FOB(x[i])*pi*r_out^2*Dx=0
"energy balance on internal nodes"
end

The temperature of the last node is taken to be specified at the ambient temperature:

TN = T∞ (4)

T[N]=T_infinity "node N temperature is specified"

The temperature is converted to Celsius:

duplicate i=1,N
T_C[i]=converttemp(K,C,T[i]) "temperature in C"
end

Figure 2 illustrates the temperature distribution within the FOB.


140

120
without filler material

Temperature (°C)
100

80

with filler material


60

40

20
0 0.1 0.2 0.3 0.4 0.5 0.6 0.7 0.8
Position (m)
Figure 2: Temperature distribution within the FOB for the case where the filler material is filler material is
present (Δkeff,x = 28 W/m-K) and the case where no filler material is present (Δkeff,x = 0).

c.) Overlay on a single plot the temperature distribution within the FOB for the case where the
filler material is present (Δkeff,x = 28 W/m-K) and the case where no filler material is present
(Δkeff,x = 0).

Figure 2 shows the case where filler material is present and is removed. The reduction in the
maximum temperature related to the addition of the filler material is evident in Figure 2.
Problem 1.9-4 (1-20 in text)
An expensive power electronics module normally receives only a moderate current. However,
under certain conditions it is possible that it might experience currents in excess of 100 amps.
The module cannot survive such a high current and therefore, you have been asked to design a
fuse that will protect the module by limiting the current that it can experience, as shown in
Figure P1.9-4.

L = 2.5 cm
ε = 0.9

Tend = 20°C Tend = 20°C


D = 0.9 mm
T∞ = 5°C
h = 5 W/m -K
2
k = 150 W/m-K
ρr = 1x10-7 ohm-m
I = 100 amp
Figure 1.9-4: A fuse that protects a power electronics module from high current.

The space available for the fuse allows a wire that is L = 2.5 cm long to be placed between the
module and the surrounding structure. The surface of the fuse wire is exposed to air at T∞ =
20°C and the heat transfer coefficient between the surface of the fuse and the air is h = 5.0
W/m2-K. The fuse surface has an emissivity of ε = 0.90. The fuse is made of an aluminum alloy
with conductivity k = 150 W/m-K. The electrical resistivity of the aluminum alloy is ρe = 1x10-7
ohm-m and the alloy melts at approximately 500°C. Assume that the properties of the alloy do
not depend on temperature. The ends of the fuse (i.e., at x=0 and x=L) are maintained at Tend
=20°C by contact with the surrounding structure and the module. The current passing through
the fuse, I, results in a uniform volumetric generation within the fuse material. If the fuse
operates properly, then it will melt (i.e., at some location within the fuse, the temperature will
exceed 500°C) when the current reaches 100 amp. Your job will be to select the fuse diameter;
to get your model started you may assume a diameter of D = 0.9 mm. Assume that the
volumetric rate of thermal energy generation due to ohmic dissipation is uniform throughout the
fuse volume.
a.) Prepare a numerical model of the fuse that can predict the steady state temperature
distribution within the fuse material. Plot the temperature as a function of position within the
wire when the current is 100 amp and the diameter is 0.9 mm.

The input parameters are entered in EES and the volumetric generation rate is computed:

$UnitSystem SI MASS RAD PA K J


$TABSTOPS 0.2 0.4 0.6 0.8 3.5 in

"Inputs"
L=2.5 [cm]*convert(cm,m) "length"
d=0.9 [mm]*convert(mm,m) "diameter"
T_a=converttemp(C,K,20) "air temperature"
T_end=converttemp(C,K,20) "end temperature"
h=5.0 [W/m^2-K] "heat transfer coefficient"
e=0.90 [-] "emissivity"
k=150 [W/m-K] "conductivity"
er=1e-7 [ohm-m] "electrical resistivity"
T_melt=converttemp(C,K,500) "melting temperature"
current=100 [amp] "current"

"Volumetric generation"
Ac=pi*d^2/4 "cross-sectional area"
Rst=er*L/Ac "resistance"
w_dot_ohmic=current^2*Rst "total dissipation"
g```_dot=w_dot_ohmic/(Ac*L) "volumetric rate of generation"

The appropriate Biot number for this case is:

hd
Bi = (1)
2k

The Biot number is calculated according to:

"Extended surface approximation"


Bi=h*d/(2*k)

The Biot number calculated by EES is much less than 1.0 and therefore the extended surface
approximation is justified.

The development of the numerical model follows the same steps that were previously discussed
in the context of numerical models of 1-D geometries. Nodes (i.e., locations where the
temperature will be determined) are positioned uniformly along the length of the rod. The
location of each node (xi) is:

xi =
( i − 1) L i = 1..N (2)
( N − 1)
where N is the number of nodes used for the simulation. The distance between adjacent nodes
(Δx) is:

L
Δx = (3)
( N − 1)
This distribution is entered in EES:

"Setup nodes"
N=10 [-] "number of nodes"
duplicate i=1,N
x[i]=(i-1)*L/(N-1) "position of nodes"
end
Dx=L/(N-1) "distance between nodes"
A control volume is defined around each node; the control surface bisects the distance between
the nodes. The control volume shown in Fig. 2 is subject to conduction heat transfer at each
edge ( qtop and qbottom ), convection ( qconv ), radiation ( qrad ), and generation ( g ). The energy
balance is:

qtop + qbottom + qconv + qrad + g = 0 (4)

The conduction terms are approximated as:

kπ d2
qtop = (Ti −1 − Ti ) (5)
4 Δx

kπ d2
qbottom = (Ti +1 − Ti ) (6)
4 Δx

The convection term is modeled according to:

qconv = h π d Δx (Ta − Ti ) (7)

The radiation term is:

qrad = ε σ π d Δx (Ta4 − Ti 4 ) (8)

The generation term is:

d2
g = g ′′′ π Δx (9)
4

Substituting Eqs. (5) through (9) into Eq. (4) leads to:

kπ d2 kπ d2
(Ti −1 − Ti ) + (Ti +1 − Ti ) + ha π d Δx (Ta − Ti )
4 Δx 4 Δx
(10)
d2
+ε σ π d Δx (Ta4 − Ti 4 ) + g ′′′ π Δx = 0 for i = 2.. ( N − 1)
4

The nodes at the edges of the domain must be treated separately; the temperature at both edges of
the fuse are specified:

T1 = Tend (11)

TN = Tend (12)
Equations (10) through (12) are a system of N equations in an equal number of unknown
temperatures which are entered in EES:

"Numerical solution"
T[1]=T_end
T[N]=T_end
duplicate i=2,(N-1)
k*pi*d^2*(T[i-1]-T[i])/(4*dx)+k*pi*d^2*(T[i+1]-T[i])/(4*dx)+pi*d*dx*h*(T_a-
T[i])+pi*d*dx*e*sigma#*(T_a^4-T[i]^4)+g```_dot*pi*d^2*dx/4=0
end
duplicate i=1,N
T_C[i]=converttemp(K,C,T[i])
end

Figure 2 illustrates the temperature distribution in the fuse for N = 100 nodes.

Figure 2: Temperature distribution in the fuse.

b.) Verify that your model has numerically converged by plotting the maximum temperature in
the wire as a function of the number of nodes in your model.

With any numerical simulation it is important to verify that a sufficient number of nodes have
been used so that the numerical solution has converged. The key result of the solution is the
maximum temperature in the wire, which can be obtained using the MAX command:

T_max_C=max(T_C[1..N])

Figure 3 illustrates the maximum temperature as a function of the number of nodes and shows
that the solution has converged for N greater than 100 nodes.
Figure 3: Maximum temperature as a function of the number of nodes.

c.) Prepare a plot of the maximum temperature in the wire as a function of the diameter of the
wire for I=100 amp. Use your plot to select an appropriate fuse diameter.

The number of nodes was set to 100 and the plot shown in Figure 4 was generated:

Figure 4: Maximum temperature as a function of diameter.

The maximum temperature reaches 500°C when the diameter is approximately 1.15 mm; this
would provide a fuse that correctly limited the current.
Problem 1.9-5
An A triangular fin is shown in Figure P1.9-5.

Figure P1.9-5: Wedge fin

The fin infinitely long (in the z-direction) and can be treated as an extended surface. The
thickness of the fin base is th = 1 cm and the length is L = 10 cm. The conductivity of the
material is k = 24 W/m-K. The base temperature is Tb = 140°C and the ambient temperature is
T∞ = 25°C. The heat transfer coefficient is h = 15 W/m2-K. The width of the fin, W, is much
larger than its length.
a.) Develop a numerical model of the fin.

The inputs are entered in EES:

$UnitSystem SI MASS RAD PA K J


$TABSTOPS 0.2 0.4 0.6 0.8 3.5 in

"Inputs"
L=10 [cm]*convert(cm,m) "length of fin"
th=1 [cm]*convert(cm,m) "base thickness"
k=24 [W/m-K] "conductivity"
T_infinity=converttemp(C,K,25 [C]) "ambient temperature"
T_b=converttemp(C,K,140 [C]) "base temperature"
h_bar=15 [W/m^2-K] "heat transfer coefficient"
W=1 [m] "per unit width of fin"

The nodes are positioned along the fin according to:

xi =
( i − 1) L for i = 1..N (1)
( N − 1)
where N is the number of nodes. The distance between adjacent nodes is:

L
Δx = (2)
( N − 1)
The cross-sectional area for conduction at each node is:
thW xi
Ac ,i = for i = 1..N (3)
L

"Let x=0 at the tip"


N=11 [-] "number of nodes"
duplicate i=1,N
x[i]=L*(i-1)/(N-1) "position"
Ac[i]=x[i]*th*W/L "area"
end
Dx=L/(N-1) "distance between nodes"

The total surface area available for convection is:

2
⎛ th ⎞
As = 2 W L + ⎜ ⎟2
(4)
⎝2⎠

A_s=2*sqrt(L^2+(th/2)^2)*W "surface area"

The temperature at the base is fixed:

TN = Tb (5)

Energy balances on the internal nodes are:

Δx
h (T∞ − Ti ) + k
( Ac,i + Ac,i+1 ) T − T + k ( Ac,i + Ac,i−1 ) T − T = 0 for i = 2.. N − 1 (6)
As ( i +1 i ) ( i −1 i ) ( )
L 2 Δx 2 Δx

An energy balance on the node at the tip is:

Δx
h (T∞ − Ti ) + k
( Ac,1 + Ac,2 ) T − T = 0
As ( 2 1) (7)
2L 2 Δx

T[N]=T_b "base temperature"


"internal node energy balances"
duplicate i=2,(N-1)
A_s*Dx/L*h_bar*(T_infinity-T[i])+k*(Ac[i]+Ac[i+1])*(T[i+1]-T[i])/(2*Dx)+k*(Ac[i]+Ac[i-1])*(T[i-1]-
T[i])/(2*Dx)=0
end
A_s*Dx/L*h_bar*(T_infinity-T[1])/2+k*(Ac[1]+Ac[2])*(T[2]-T[1])/(2*Dx)=0

The solution is converted to Celsius.

duplicate i=1,N
T_C[i]=converttemp(K,C,T[i])
end
b.) Plot the temperature distribution within the fin.

Figure 2 illustrates the temperature as a function of position (recall that x is measured from the
tip of the fin).
140

130

120
Temperature (°C)

110

100

90

80

70

60
0 0.01 0.02 0.03 0.04 0.05 0.06 0.07 0.08 0.09 0.1
Position (m)
Figure 2: Temperature distribution in the fin.

c.) Determine the fin efficiency. Compare your answer with the fin efficiency obtained from the
EES function eta_fin_straight_triangular.

The rate of heat transfer to the fin base is obtained by carrying out an energy balance on node N.

Δx
h (TN − T∞ ) + k
( Ac, N + Ac, N −1 ) T − T
q = As ( N N −1 ) (8)
2L 2 Δx

The maximum possible heat transfer is:

qmax = As h (Tb − T∞ ) (9)

The fin efficiency is:

q
η= (10)
qmax

q_dot=A_s*Dx*h_bar*(T[N]-T_infinity)/(2*L)+k*(Ac[N]+Ac[N-1])*(T[N]-T[N-1])/(2*Dx)
"actual heat transfer rate"
q_dot_max=A_s*h_bar*(T_b-T_infinity) "maximum possible heat transfer rate"
eta=q_dot/q_dot_max "fin efficiency"

which leads to η = 0.6561. The EES function eta_fin_straight_triangular is also determined:

eta_EES=eta_fin_straight_triangular(th,L,h_bar,k) "fin efficiency from EES"


which leads to ηEES = 0.6556.

d.) Plot the fin efficiency as a function of the number of nodes used in the solution.

Figure 3 illustrates the predicted efficiency as a function of the number of nodes and suggests
that you must use at least 10 nodes.

0.73

0.72

0.71

0.7
Efficiency

0.69

0.68

0.67

0.66

0.65
2 10 100 500
Number of nodes
Figure 3: Predicted efficiency as a function of number of nodes.
Problem 1.9-6
Your company manufacturers heater wire. Heater wire is applied to surfaces that need to be
heated and then current is passed through the wire in order to develop ohmic dissipation. A key
issue with your product is failures that occur when a length of the wire becomes detached from
the surface and therefore the wire is not well connected thermally to the surface. The wire in the
detached region tends to get very hot and melt.

The wire diameter is D = 0.4 mm and the current passing through the wire is current = 10 amp.
the detached wire is exposed to surroundings at T∞ = 20ºC through convection and radiation.
The average convection heat transfer coefficient is h = 30 W/m2-K and the emissivity of the wire
surface is ε = 0.5. The length of the wire that is detached from the surface is L = 1 cm. The ends
of the wire at x = 0 and x = L are held at Tend = 50ºC. The wire conductivity is k = 10 W/m-K
and the electrical resistivity is ρe = 1x10-7 ohm-m.
a.) Develop a numerical model of the wire and use the model to plot the temperature distribution
within the wire.

The inputs are entered in EES and, looking ahead to parts (d) and (e), functions are defined that
return the conductivity and electrical resistivity:

$UnitSystem SI MASS RAD PA K J


$TABSTOPS 0.2 0.4 0.6 0.8 3.5 in

function k(T)
"Input
T - temperature (K)
Output
k - conductivity (W/m-K)"
k_o=10 [W/m-K]
k=k_o
end

function rho_e(T)
"Input
T - temperature (K)
Output
rho_e - electrical resistivity (ohm-m)"
rho_e_o=1e-7 [ohm-m]
rho_e=rho_e_o
end

"Inputs"
L_cm=1 [cm] "length of detached region, in cm"
L=L_cm*convert(cm,m) "length of detached region"
d=0.4 [mm]*convert(mm,m) "diameter of heater wire"
current=10 [amp] "current"
h_bar=30 [W/m^2-K] "heat transfer coefficient"
e=0.5 [-] "emissivity"
T_end=converttemp(C,K,50 [C]) "end temperatures"
T_infinity=converttemp(C,K,20[C]) "surrounding temperatures"

The nodes are positioned along the wire according to:


xi =
( i − 1) L for i = 1..N (1)
( N − 1)
where N is the number of nodes. The distance between adjacent nodes is:

L
Δx = (2)
( N − 1)
The perimeter and cross-sectional area for conduction along the wire is:

per = π D (3)

D2
Ac = π (4)
4

N=21 [-] "number of nodes"


duplicate i=1,N
x[i]=L*(i-1)/(N-1) "position of nodes"
end
Dx=L/(N-1) "distance between nodes"
per=pi*d "perimeter of wire"
A_c=pi*d^2/4 "cross-sectional area of wire"

The temperatures of the nodes at the ends of the wire are set:

T1 = Tend (5)

TN = Tend (6)

T[1]=T_end
T[N]=T_end

Energy balances on the internal nodes lead to:

k Ac
(Ti −1 − Ti ) + k Ac
(Ti +1 − Ti ) + ρ Δx
current 2 +
(Ti +Ti−1 ) (Ti +Ti+1 ) e ,T =Ti
T=
2
Δx T=
2
Δx Ac
ε per Δx σ (T − T

4
i
4
) + h per Δx (T ∞ − Ti )=0 (7)
i = 2.. ( N − 1)

duplicate i=2,(N-1)
k((T[i]+T[i-1])/2)*A_c*(T[i-1]-T[i])/Dx+k((T[i]+T[i+1])/2)*A_c*(T[i+1]-T[i])/Dx&
+rho_e(T[i])*Dx*current^2/A_c+e*per*Dx*sigma#*(T_infinity^4-T[i]^4)+h_bar*per*Dx*(T_infinity-T[i])=0
end
duplicate i=1,N
T_C[i]=converttemp(K,C,T[i]) "temperature in C"
end

Figure 1 illustrates the temperature as a function of position in the wire.


600

500

400
Temperature (°C)

300

200

100

0
0 0.002 0.004 0.006 0.008 0.01
Position (m)
Figure 1: Temperature as a function of position.

b.) Plot the maximum temperature in the wire as a function of the number of nodes in the
numerical model.

The maximum temperature (Tmax) is identified.

T_max_C=max(T_C[1..N]) "maximum temperature in C"

Figure 2 illustrates the maximum temeprature as a function of number of nodes.


560

555

550
Maximum temperature (°C)

545

540

535

530

525

520

515
2 10 100 450
Number of nodes
Figure 2: Maximum temperature as a function of the number of nodes.
c.) Plot the maximum temperature in the wire as a function of the length of the detached region.
If the maximum temperature of the wire before failure is Tmax = 400ºC, then what is the
maximum allowable length of detached wire?

Figure 3 illustrates the maximum temperature in the wire as a function of the length of the
detached region and shows that the detached region cannot exceed about 0.75 cm in length
without resulting in failure.
900 -7
k = 10 W/m-K, ρe = 1x10 ohm-m
800
Maximum temperature (°C)

700 Material D

600
Material E
500

400

300

200

100

0
0 0.5 1 1.5 2 2.5 3 3.5 4
Length of detached region (m)
Figure 3: Maximum temperature as a function of the detached length for the nominal wire with k = 10 W/m-
K and ρe = 1x10-7 ohm-m as well as for materials D and E.

You are looking at methods to alleviate this problem and have identified an alternative material,
material D, in which the electrical resistivity is ρe = 1x10-7 ohm-m but the conductivity increases
with temperature according to:
⎡ W ⎤ ⎡ W ⎤
2 ⎥(
k = 10 ⎢ ⎥ + 0.05 ⎢ T − 300 [ K ])
⎣mK ⎦ ⎣mK ⎦
d.) Modify your numerical model in order to simulate material D. Overlay on your plot from (c)
the maximum temperature as a function of the length of the detached wire for material D.

The function for the conductivity is modified:

function k(T)
"Input
T - temperature (K)
Output
k - conductivity (W/m-K)"
k_o=10 [W/m-K]
alpha=0.05 [W/m-K^2]
{ k=k_o}
k=k_o+alpha*(T-300 [K])
end
The maximum temperature as a function of detached length is shown in Figure 3. The maximum
length of detached wire has increased to approximately 1.1 cm.

You have identified another alternative material, material E, in which the thermal conductivity is
k = 10 W/m-K but the electrical resistivity decreases with temperature according to:
⎡ ohm m ⎤
ρ = 1x10−7 [ ohm m ] − 1x10-10 ⎢ (T − 300 [ K ])
⎣ K ⎥⎦
e.) Modify your numerical model in order to simulate material E. Overlay on your plot from (c)
the maximum temperature as a function of the length of the detached wire for material E.

The function for the conductivity is restored and the function for electrical resistivity is modified:

function k(T)
"Input
T - temperature (K)
Output
k - conductivity (W/m-K)"
k_o=10 [W/m-K]
alpha=0.05 [W/m-K^2]
k=k_o
{ k=k_o+alpha*(T-300 [K])}
end

function rho_e(T)
"Input
T - temperature (K)
Output
rho_e - electrical resistivity (ohm-m)"
rho_e_o=1e-7 [ohm-m]
beta=1e-10 [ohm-m/K]
{rho_e=rho_e_o}
rho_e=rho_e_o-beta*(T-300 [K])
end

The maximum temperature as a function of detached length is shown in Figure 3. The maximum
length of detached wire has increased to approximately 1.0 cm.
Problem 1.9-7: Flat Plate Solar Collector
Figure P1.9-7 illustrates a flat plate solar collector.

q ′′s = 900 W/m h = 5 W/m -K


2 2

T∞ = 10°C
k = 75 W/m2-K
Tw = 50°C
ε=1 th = 1.5 mm

L = 8 cm
Figure 1.9-7: Flat plate solar collector.

The collector consists of a flat plate that is th = 1.5 mm thick with conductivity k = 75 W/m-K.
The plate is insulated on its back side and experiences a solar flux of qs′′ = 900 W/m2 which is all
absorbed. The surface is exposed to convection and radiation to the surroundings. The
emissivity of the surface is ε = 1. The heat transfer coefficient is h = 5 W/m2-K and the
surrounding temperature is T∞ = 10ºC. The temperature of the water is Tw = 50ºC. The center-
to-center distance between adjacent tubes is 2 L where L = 8 cm.
a.) Develop a numerical model that can predict the rate of energy transfer to the water per unit
length of collector.

The inputs are entered in EES:

$UnitSystem SI MASS RAD PA K J


$Tabstops 0.2 0.4 0.6 3.5 in

"Inputs"
th=1.5 [mm]*convert(mm,m) "thickness"
k=75 [W/m-K] "conductivity of collector plate"
e=1 [-] "emissivity of collector plate"
q``_s=900 [W/m^2] "solar flux"
T_infinity=converttemp(C,K,10[C]) "ambient temperature"
h_bar=5 [W/m^2-K] "heat transfer coefficient"
L=8 [cm]*convert(cm,m) "half-width between tubes"
T_w=converttemp(C,K,50 [C]) "water temperature"
W=1 [m] "per unit length of collector"

Nodes are placed along the length of the collector. Only the region from 0 < x < L is considered
due to the symmetry of the system. Therefore, the position of each node is:

L ( i − 1)
xi = for i = 1..N (1)
( N − 1)
The distance between adjacent nodes is:
L
Δx = (2)
( N − 1)
N=21 [-] "number of nodes"
Dx=L/(N-1)
duplicate i=1,N
x[i]=L*(i-1)/(N-1) "location of each node"
end

Energy balances on each of the internal nodes leads to:

q LHS ,i + q RHS ,i + qs ,i + qconv ,i + qrad ,i = 0 for i = 2.. ( N − 1) (3)

where

k W th
q LHS ,i = (Ti −1 − Ti ) (4)
Δx

k W th
q RHS ,i = (Ti +1 − Ti ) (5)
Δx

qs ,i = W Δx q s′′ (6)

qs ,i = W Δx h (T∞ − Ti ) (7)

qrad ,i = W Δx ε σ (T∞4 − Ti 4 ) (8)

"internal nodes"
duplicate i=2,(N-1)
q_dot_LHS[i]=k*W*th*(T[i-1]-T[i])/Dx "conduction from left hand side node"
q_dot_RHS[i]=k*W*th*(T[i+1]-T[i])/Dx "conduction from right hand side node"
q_dot_s[i]=W*Dx*q``_s "absorbed solar radiation"
q_dot_conv[i]=W*Dx*h_bar*(T_infinity-T[i]) "convection"
q_dot_rad[i]=W*Dx*e*sigma#*(T_infinity^4-T[i]^4) "radiation"
q_dot_LHS[i]+q_dot_RHS[i]+q_dot_s[i]+q_dot_conv[i]+q_dot_rad[i]=0 "energy balance"
end

The temperature of node 1 is assumed to be equal to the water temperature (neglecting any
resistance to convection on the water-side):

T1 = Tw (9)

An energy balance on node N leads to:

q LHS , N + qs , N + qconv , N + qrad , N = 0 (10)


where

k W th
q LHS , N = (TN −1 − TN ) (11)
Δx

W Δx
qs , N = q s′′ (12)
2

W Δx
qs , N = h (T∞ − TN ) (13)
2

W Δx
qrad , N = ε σ (T∞4 − TN4 ) (14)
2

"node 1"
T[1]=T_w

"node N"
q_dot_LHS[N]=k*W*th*(T[N-1]-T[N])/Dx "conduction from left hand side node"
q_dot_s[N]=W*Dx*q``_s/2 "absorbed solar radiation"
q_dot_conv[N]=W*Dx*h_bar*(T_infinity-T[N])/2 "convection"
q_dot_rad[N]=W*Dx*e*sigma#*(T_infinity^4-T[N]^4)/2 "radiation"
q_dot_LHS[N]+q_dot_s[N]+q_dot_conv[N]+q_dot_rad[N]=0 "energy balance"

The temperature distribution within the collector is shown in Figure 2:


333

331
Temperature (K)

329

327

325

323
0 0.02 0.04 0.06 0.08 0.1
Position (m)
Figure 2: Temperature as a function of position in the collector.

An energy balance on node 1 provides the rate of energy transfer to the water:

qwater = q RHS ,1 + qs ,1 + qconv ,1 + qrad ,1 (15)


where

k W th
q RHS ,1 = (T2 − T1 ) (16)
Δx

W Δx
qs ,1 = q s′′ (17)
2

W Δx
qs ,1 = h (T∞ − T1 ) (18)
2

W Δx
qrad ,1 = ε σ (T∞4 − T14 ) (19)
2

"node 1"
q_dot_RHS[1]=k*W*th*(T[2]-T[1])/Dx "conduction from right hand side node"
q_dot_s[1]=W*Dx*q``_s/2 "absorbed solar radiation"
q_dot_conv[1]=W*Dx*h_bar*(T_infinity-T[1])/2 "convection"
q_dot_rad[1]=W*Dx*e*sigma#*(T_infinity^4-T[1]^4)/2 "radiation"
q_dot_water=q_dot_RHS[1]+q_dot_s[1]+q_dot_conv[1]+q_dot_rad[1] "energy balance"

which leads to qwater = 28.9 W.

b.) Determine the efficiency of the collector; efficiency is defined as the ratio of the energy
delivered to the water to the solar energy incident on the collector.

The efficiency is calculated according to:

qwater
η= (20)
LW

eta=q_dot_water/(L*W*q``_s) "efficiency"

which leads to η = 0.402.

c.) Plot the efficiency as a function of the number of nodes used in the solution.

Figure 3 illustrates the efficiency as a function of the number of nodes and shows that at least
20-30 nodes are required for numerical convergence.
0.409

0.408

0.407

0.406

Efficiency (-)
0.405

0.404

0.403

0.402

0.401
2 10 100 225
Number of nodes
Figure 3: Efficiency as a function of the number of nodes.

d.) Plot the efficiency as a function of Tw - T∞. Explain your plot.

Figure 4 illustrates the efficiency as a function of the water-to-ambient temperature difference.


When the water temperature is low then the losses are low (but not zero, because the
temperature of the copper plate is elevated by conduction). As the water temperature increases,
the temperature of the plate increases and therefore the losses increase and efficiency drops.
The drop in efficiency is dramatic for this type of unglazed collector and therefore the collector
may be suitable for providing water heating for swimming pools (at low water temperature) but
probably is not suitable for providing domestic hot water (at high water temperature).
1

0.9

0.8

0.7

0.6
Efficiency

0.5

0.4

0.3

0.2

0.1

0
0 10 20 30 40 50 60 70 80
Water to ambient temperature difference (K)
Figure 4: Solar collector efficiency as a function of the water-to-ambient temperature difference.
P1.1-2 (1-1 in text): Conductivity of a dilute gas
Section 1.1.2 provides an approximation for the thermal conductivity of a monatomic gas at ideal
gas conditions. Test the validity of this approximation by comparing the conductivity estimated
using Eq. (1-18) to the value of thermal conductivity for a monotonic ideal gas (e.g., low
pressure argon) provided by the internal function in EES. Note that the molecular radius, σ, is
provided in EES by the Lennard-Jones potential using the function sigma_LJ.
a.) What is the value and units of the proportionality constant required to make Eq. (1-18) an
equality?

Equation (1-18) is repeated below:

cv T
k∝ (1)
σ 2
MW

Equation (1) is written as an equality by including a constant of proportionality (Ck):

cv T
k = Ck (2)
σ 2
MW

Solving for Ck leads to:

kσ 2 MW
Ck = (3)
cv T

which indicates that Ck has units m-kg1.5/s-kgmol05-K0.5.

The inputs are entered in EES for Argon at relatively low pressure (0.1 MPa) and 300 K.

"Problem 1.1-2"
$UnitSystem SI MASS RAD PA K J
$TABSTOPS 0.2 0.4 0.6 0.8 3.5 in

T=300 [K] "temperature"


F$='Argon' "fluid"
P_MPa=0.1 [MPa] "pressure, in MPa"
P=P_MPa*convert(MPa, Pa) "pressure"

The conductivity, specific heat capacity, Lennard-Jones potential, and molecular weight of
Argon (k, cv, σ, and MW) are evaluated using EES' built-in funcions. Equation (3) is used to
evaluate the proportionality constant.

k=conductivity(F$,T=T,P=P) "conductivity"
cv=cv(F$,T=T,P=P) "specific heat capacity at constant volume"
MW=molarMass(F$) "molecular weight"
sigma=sigma_LJ(F$) "Lennard-Jones potential"
C_k=k*sigma^2*sqrt(MW/T)/cv "constant of proportionality"
which leads to Ck = 2.619x10-24 m-kg1.5/s-kgmol0.5-K0.5.

b.) Plot the value of the proportionality constant for 300 K argon at pressures between 0.01 and
100 MPa on a semi-log plot with pressure on the log scale. At what pressure does the
approximation given in Eq. (1-18) begin to fail at 300 K for argon?

Figure 1 illustrates the constant of proportionality as a function of pressure for argon at 300 K.
The approximation provided by Eq. (1-18) breaks down at approximately 1 MPa.

8x10-24

7x10-24
)
0.5

6x10-24
-K
0.5

5x10-24
/s-kgmol

4x10-24

3x10-24
1.5
Ck (m-kg

2x10-24

10-24

0x100
0.001 0.01 0.1 1 10 100
Pressure (MPa)
Figure 1: Constant of proportionality in Eq. (3) as a function of pressure for argon at 300 K.
Problem 1.2-3 (1-2 in text): Conduction through a Wall
Figure P1.2-3 illustrates a plane wall made of a very thin (thw = 0.001 m) and conductive (k =
100 W/m-K) material that separates two fluids, A and fluid B. Fluid A is at TA = 100°C and the
heat transfer coefficient between the fluid and the wall is hA = 10 W/m2-K while fluid B is at TB
= 0°C with hB = 100 W/m2-K.
thw = 0.001 m

TA = 100°C TB = 0°C
hA = 10 W/m -K hB = 100 W/m -K
2 2

k = 100 W/m-K
Figure P1.2-3: Plane wall separating two fluids

a.) Draw a resistance network that represents this situation and calculate the value of each
resistor (assuming a unit area for the wall, A = 1 m2).

Heat flowing from fluid A to fluid B must pass through a fluid A-to-wall convective resistance
(Rconv,A), a resistance to conduction through the wall (Rcond), and a wall-to-fluid B convective
resistance (Rconv,B). These resistors are in series. The network and values of the resistors are
shown in Figure 2.

K K K
0.1 0.0001 0.01
W W W
TA = 100°C TB = 0°C
1 tw 1
Rconv , A = Rcond = Rcond , B =
hA A kA hB A
Figure 2: Thermal resistance network representing the wall.

b.) If you wanted to predict the heat transfer rate from fluid A to B very accurately, then which
parameter (e.g., thw, k, etc.) would you try to understand/measure very carefully and which
parameters are not very important? Justify your answer.

The largest resistance in a series network will control the heat transfer. For the wall above, the
largest resistance is Rconv,A. Therefore, I would focus on predicting this resistance accurately.
This would suggest that hA is the most important parameter and the others do not matter much.
Problem 1.2-8 (1-3 in text): Frozen Gutters
You have a problem with your house. Every spring at some point the snow immediately
adjacent to your roof melts and runs along the roof line until it reaches the gutter. The water in
the gutter is exposed to air at temperature less than 0°C and therefore freezes, blocking the gutter
and causing water to run into your attic. The situation is shown in Figure P1.2-8.
snow melts at this surface
Tout , hout = 15 W/m -K
2

Ls = 2.5 inch
snow, ks = 0.08 W/m-K
insulation, kins = 0.05 W/m-K
Tin = 22°C, hin = 10 W/m -K
2

Lins = 3 inch
plywood,
L p = 0.5 inch, k p = 0.2 W/m-K

Figure P1.2-8: Roof of your house.

The air in the attic is at Tin = 22°C and the heat transfer coefficient between the inside air and the
inner surface of the roof is hin = 10 W/m2-K. The roof is composed of a Lins = 3.0 inch thick
piece of insulation with conductivity kins = 0.05 W/m-K that is sandwiched between two Lp = 0.5
inch thick pieces of plywood with conductivity kp = 0.2 W/m-K. There is an Ls = 2.5 inch thick
layer of snow on the roof with conductivity ks = 0.08 W/m-K. The heat transfer coefficient
between the outside air at temperature Tout and the surface of the snow is hout = 15 W/m2-K.
Neglect radiation and contact resistances for part (a) of this problem.
a.) What is the range of outdoor air temperatures where you should be concerned that your
gutters will become blocked by ice?

The input parameters are entered in EES and converted to base SI units (N, m, J, K) in order to
eliminate any unit conversion errors; note that units should still be checked as you work the
problem but that this is actually a check on the unit consistency of the equations.

"P1.2-8: Frozen Gutters"


$UnitSystem SI MASS RAD PA K J
$TABSTOPS 0.2 0.4 0.6 0.8 3.5 in

T_in=converttemp(C,K,22) "temperature in your attic"


L_ins=3 [inch]*convert(inch,m) "insulation thickness"
L_p=0.5 [inch]*convert(inch,m) "plywood thickness"
k_ins=0.05 [W/m-K] "insulation conductivity"
k_p=0.2 [W/m-K] "plywood conductivity"
k_s=0.08 [W/m-K] "snow conductivity"
L_s=2.5 [inch]*convert(inch,m) "snow thickness"
h_in=10 [W/m^2-K]
"heat transfer coefficient between attic air and inner surface of roof"
h_out=15 [W/m^2-K]
"heat transfer coefficient between outside air and snow"
A=1 [m^2] "per unit area"
The problem may be represented by the resistance network shown in Figure 2.

Figure 2: Resistance network representing the roof of your house.

The network includes resistances that correspond to convection with the inside and outside air:

1
Rconv ,out = (1)
hout A

1
Rconv ,in = (2)
hin A

where A is 1 m2 in order to accomplish the problem on a per unit area basis. There are also
conduction resistances associated with the insulation, plywood and snow:

Lins
Rins = (3)
kins A

Lp
Rp = (4)
kp A

Ls
Rs = (5)
ks A

R_conv_out=1/(h_out*A) "outer convection resistance"


R_s=L_s/(k_s*A) "snow resistance"
R_p=L_p/(k_p*A) "plywood resistance"
R_ins=L_ins/(k_ins*A) "insulation resistance"
R_conv_in=1/(h_in*A) "inner convection resistance"

Which leads to Rconv,out = 0.07 K/W, Rs = 0.79 K/W, Rp = 0.06 K/W, Rins = 1.52 K/W and Rconv,in
= 0.10 K/W. Therefore, the dominant effects for this problem are conduction through the
insulation and the snow; the other effects (convection and the plywood conduction) are not
terribly important since the largest resistances will dominate in a series network.

If the snow at the surface of the room is melting then the temperature at the connection between
Rs and Rp must be Ts = 0°C (see Figure 2). Therefore, the heat transferred through the roof ( q in
Figure 2) must be:
q =
(Tin − Ts ) (6)
Rconv ,in + 2 R p + Rins

The temperature of the outside air must therefore be:

Tout = Ts − q ( Rs + Rconv ,out ) (7)

T_s=converttemp(C,K,0)
"roof-to-snow interface temperature must be melting point of water"
q_dot=(T_in-T_s)/(R_conv_in+2*R_p+R_ins)
"heat transfer from the attic to the snow when melting point is reached"
T_out=T_s-q_dot*(R_s+R_conv_out)
"outside temperature required to reach melting point at roof surface"
T_out_C=converttemp(K,C,T_out) "outside temperature in C"

which leads to Tout = -10.8°C. If the temperature is below this then the roof temperature will be
below freezing and the snow will not melt. If the temperature is above 0°C then the water will
not refreeze upon hitting the gutter. Therefore, the range of temperatures of concern are -10.8°C
< Tout < 0°C.

b.) Would your answer change much if you considered radiation from the outside surface of the
snow to surroundings at Tout? Assume that the emissivity of snow is εs = 0.82.

The modified resistance network that includes radiation is shown in Figure 3.

Figure 3: Resistance network representing the roof of your house and including radiation.

The additional resistance for radiation is in parallel with convection from the surface of the snow
as heat is transferred from the surface by both mechanisms. The radiation resistance can be
calculated approximately according to:

1
Rrad = (8)
4T ε s σ A
3

where T is the average temperature of the surroundings and the snow surface. In order to get a
quick idea of the magnitude of this resistance we can approximate T with its largest possible
value (which will result in the largest possible amount of radiation); the maximum temperature
of the snow is 0°C:

e_s=0.82 [-] "emissivity of snow"


R_rad=1/(4*T_s^3*e_s*sigma#*A) "radiation resistance"

which leads to Rrad = 0.26 K/W. Notice that Rrad is much larger than Rconv,out; the smallest
resistance in a parallel combination dominates and therefore the impact of radiation will be
minimal. Furthermore, Rconv,out is not even a very important resistance in the original series
circuit shown in Figure 2.
Problem P1.2-11 (1-4 in text)
Figure P1.2-11(a) illustrates a composite wall. The wall is composed of two materials (A with kA
= 1 W/m-K and B with kB = 5 W/m-K), each has thickness L = 1.0 cm. The surface of the wall at
x = 0 is perfectly insulated. A very thin heater is placed between the insulation and material A;
the heating element provides q ′′ = 5000 W/m 2 of heat. The surface of the wall at x = 2L is
exposed to fluid at Tf,in = 300 K with heat transfer coefficient hin = 100 W/m2-K.
material A
q ′′ = 5000 W/m
2
kA = 1 W/m-K
insulated L = 1 cm

x T f ,in = 300 K
hin = 100 W/m -K
2

L = 1 cm material B
kB = 5 W/m-K
Figure P1.2-11(a): Composite wall with a heater.

You may neglect radiation and contact resistance for parts (a) through (c) of this problem.
a.) Draw a resistance network to represent this problem; clearly indicate what each resistance
represents and calculate the value of each resistance.

The input parameters are entered in EES:

“P1.2-11: Heater"
$UnitSystem SI MASS RAD PA K J
$TABSTOPS 0.2 0.4 0.6 0.8 3.5 in

"Inputs"
q_flux=100 [W/m^2] "heat flux provided by the heater"
L = 1.0 [cm]*convert(cm,m) "thickness of each layer"
k_A=1.0 [W/m-K] "conductivity of material A"
k_B=5.0 [W/m-K] "conductivity of material B"
T_f_in=300 [K] "fluid temperature at inside surface"
h_in=100 [W/m^2-K] "heat transfer on inside surface"
A=1 [m^2] "per unit area"

The resistance network that represents the problem shown in Figure 2 is:

Figure 2: Resistance network.

The resistances due to conduction through materials A and B are:


L
RA = (1)
kA A

L
RB = (2)
kB A

where A is the area of the wall, taken to be 1 m2 in order to carry out the analysis on a per unit
area basis. The resistance due to convection is:

1
Rconv ,in = (3)
hin A

"part (a)"
R_A=L/(k_A*A) "resistance to conduction through A"
R_B=L/(k_B*A) "resistance to conduction through B"
R_conv_in=1/(h_in*A)
"resistance to convection on inner surface"

which leads to RA = 0.01 K/W, RB = 0.002 K/W, and Rconv,in = 0.01 K/W.

b.) Use your resistance network from (a) to determine the temperature of the heating element.

The resistance network for this problem is simple; the temperature drop across each resistor is
equal to the product of the heat transferred through the resistor and its resistance. In this simple
case, all of the heat provided by the heater must pass through materials A, B, and into the fluid
by convection so these resistances are in series. The heater temperature (Thtr) is therefore:

Thtr = T f ,in + ( RA + RB + Rconv ,in ) q ′′ A (4)

T_htr=T_f_in+(R_A+R_B+R_conv_in)*q_flux*A "heater temperature"

which leads to Thtr = 410 K.

c.) Sketch the temperature distribution on the axes provided below. Make sure that the sketch is
consistent with your solution from (b).

The temperatures at x = L and x = 2L can be computed according to:

Tx = L = T f ,in + ( RB + Rconv ,in ) q ′′ A (5)

Tx = 2 L = T f ,in + Rconv ,in q ′′ A (6)

T_L=T_f_in+(R_B+R_conv_in)*q_flux*A "temperature at x=L"


T_2L=T_f_in+R_conv_in*q_flux*A "temperature at x=2L"
which leads to Tx=L = 360 K and Tx=2L = 350 K. The temperature distribution is sketched on the
axes in Figure 3.

Figure 3: Sketch of temperature distribution.

Notice that the temperature drop through the two larger resistances (RA and RB) are much larger
than the temperature drop across the small resistance, RB.

Figure P1.2-11(b) illustrates the same composite wall shown in Figure P1.2-11(a), but there is an
additional layer added to the wall, material C with kC = 2.0 W/m-K and L = 1.0 cm.

q ′′ = 5000 W/m
2
material A
material C kA = 1 W/m-K
kC = 2 W/m-K
L = 1 cm
insulated

x T f ,in = 300 K
hin = 100 W/m -K
2

material B
L = 1 cm L = 1 cm k = 5 W/m-K
B
Figure P1.2-11(b): Composite wall with Material C.

Neglect radiation and contact resistance for parts (d) through (f) of this problem.
d.) Draw a resistance network to represent the problem shown in Figure P1.2-11(b); clearly
indicate what each resistance represents and calculate the value of each resistance.

There is an additional resistor corresponding to conduction through material C, RC, as shown


below:
Notice that the boundary condition at the end of RC corresponds to the insulated wall; that is, no
heat can be transferred through this resistance. The resistance to conduction through material C
is:

L
RC = (7)
kC A

"part (b)"
k_C=2.0 [W/m-K] "conductivity of material C"
R_C=L/(k_C*A) "resistance to conduction through C"

which leads to RC = 0.005 K/W.

e.) Use your resistance network from (d) to determine the temperature of the heating element.

Because there is no heat transferred through RC, all of the heat must still go through materials A
and B and be convected from the inner surface of the wall. Therefore, the answer is not changed
from part (b), Thtr = 410 K.

f.) Sketch the temperature distribution on the axes provided below. Make sure that the sketch is
consistent with your solution from (e).

The answer is unchanged from part (c) except that there is material to the left of the heater.
However, no heat is transferred through material C and therefore there is no temperature gradient
in the material.
Figure P1.2-11(c) illustrates the same composite wall shown in Figure P1.2-11(b), but there is a
contact resistance between materials A and B, Rc′′ = 0.01 K-m 2 /W , and the surface of the wall at
x = -L is exposed to fluid at Tf,out = 400 K with a heat transfer coefficient hout = 10 W/m2-K.

q ′′ = 5000 W/m
2
material A
material C kA = 1 W/m-K
kC = 2 W/m-K
L = 1 cm

T f ,out = 400 K
T f ,in = 300 K
hout = 10 W/m -K x
2

hin = 100 W/m -K


2

material B
L = 1 cm kB = 5 W/m-K
L = 1 cm
Rc′′ = 0.01 K-m /W
2

Figure P1.2-11(c): Composite wall with convection at the outer surface and contact resistance.

Neglect radiation for parts (g) through (i) of this problem.


g.) Draw a resistance network to represent the problem shown in Figure P1.2-11(c); clearly
indicate what each resistance represents and calculate the value of each resistance.

The additional resistances associated with contact resistance and convection to the fluid at the
outer surface are indicated. Notice that the boundary condition has changed; heat provided by
the heater has two paths ( qout and qin ) and so the problem is not as easy to solve.
The additional resistances are computed according to:

1
Rconv ,out = (8)
hout A

Rc′′
Rcontact = (9)
A

"part (c)"
R``_c=0.01 [K-m^2/W] "area specific contact resistance"
h_out=10 [W/m^2-K] "heat transfer coefficient"
T_f_out=400 [K] "fluid temperature on outside surface"
R_contact=R``_c/A "contact resistance"
R_conv_out=1/(h_out*A)
"convection resistance on outer surface"

which leads to Rcontact = 0.01 K/W and Rconv,out = 0.1 K/W.

h.) Use your resistance network from (j) to determine the temperature of the heating element.

It is necessary to carry out an energy balance on the heater:

q ′′ A = qin + qout (10)

The heat transfer rates can be related to Thtr according to:

qin =
(Thtr − T f ,in )
(11)
RA + Rcontact + RB + Rconv ,in

qout =
(Thtr − T f ,out )
(12)
RC + Rconv ,out

These are 3 equations in 3 unknowns, Thtr, qout and qin , and therefore can be solved
simultaneously in EES (note that the previous temperature calculations from part (b) must be
commented out):

{T_htr=T_f_in+(R_A+R_B+R_conv_in)*q_flux*A "heater temperature"


T_L=T_f_in+(R_B+R_conv_in)*q_flux*A "temperature at x=L"
T_2L=T_f_in+R_conv_in*q_flux*A "temperature at x=2L"}
q_flux*A=q_dot_in+q_dot_out "energy balance on the heater"
q_dot_in=(T_htr-T_f_in)/(R_A+R_contact+R_B+R_conv_in) "heat flow to inner fluid"
q_dot_out=(T_htr-T_f_out)/(R_C+R_conv_out) "heat flow to outer fluid"

which leads to Thtr = 446 K. The other intermediate temperatures shown on the resistance
diagram can be computed:

Tx = L − = Thtr − RA qin (13)

Tx = L + = Thtr − ( RA + Rcontact ) qin (14)

Tx = 2 L = Thtr − ( RA + Rcontact + RB ) qin (15)

Tx =− L = Thtr − RC qout (16)

"intermediate temperatures"
T_Lm=T_htr-R_A*q_dot_in
T_Lp=T_htr-(R_A+R_contact)*q_dot_in
T_2L=T_htr-(R_A+R_contact+R_B)*q_dot_in
T_mL=T_htr-R_C*q_dot_out

which leads to Tx=L- = 400.4 K, Tx=L+ = 354.7 K, Tx=2L = 345.6 K, and Tx=-L = 443.8 K.

i.) Sketch the temperature distribution on the axes provided below.


Problems 1.2-12 (1-5 in text): Floor Heater
You have decided to install a strip heater under the linoleum in your bathroom in order to keep
your feet warm on cold winter mornings. Figure P1.2-12 illustrates a cross-section of the
bathroom floor. The bathroom is located on the first story of your house and is W = 2.5 m wide
by L = 2.5 m long. The linoleum thickness is thL = 5 mm and has conductivity kL = 0.05 W/m-K.
The strip heater under the linoleum is negligibly thin. Beneath the heater is a piece of plywood
with thickness thp = 5 mm and conductivity kp = 0.4 W/m-K. The plywood is supported by ths =
6 cm thick studs that are Ws = 4 cm wide with thermal conductivity ks = 0.4 W/m-K. The center-
to-center distance between studs is ps = 25.0 cm. Between each stud are pockets of air that can
be considered to be stagnant with conductivity kair = 0.025 W/m-K. A sheet of drywall is nailed
to the bottom of the studs. The thickness of the drywall is thd = 9.0 mm and the conductivity of
drywall is kd = 0.1 W/m-K. The air above in the bathroom is at Tair,1 = 15°C while the air in the
basement is at Tair,2 = 5°C. The heat transfer coefficient on both sides of the floor is h = 15
W/m2-K. You may neglect radiation and contact resistance for this problem.

Tair ,1 = 15°C, h = 15 W/m -K


2
linoleum, kL = 0.05 W/m-K
plywood, kp = 0.4 W/m-K
strip heater
thL = 5 mm
thp = 5 mm ps = 25 cm

ths = 6 cm

Ws = 4 cm
thd = 9 mm
studs, ks = 0.4 W/m-K
drywall, kd = 0.1 W/m-K Tair ,2 = 5°C, h = 15 W/m -K
2

air, ka = 0.025 W/m-K


Figure P1.2-12: Bathroom floor with heater.

a.) Draw a thermal resistance network that can be used to represent this situation. Be sure to
label the temperatures of the air above and below the floor (Tair,1 and Tair,2), the temperature
at the surface of the linoleum (TL), the temperature of the strip heater (Th), and the heat input
to the strip heater ( qh ) on your diagram.

The resistance diagram corresponding to this problem is shown in Figure 2.


Figure 2: Resistance diagram representing the bathroom floor.

Starting at the left-hand side of Figure 2 (i.e., from the basement air), the resistances correspond
to convection between the air in the basement and the drywall (Rconv), conduction through the
drywall (Rd), conduction through the air (Rair) and studs (Rs) in parallel, conduction through the
plywood (Rp), conduction through the linoleum (RL), and convection between the air in the
bathroom and the linoleum (Rconv).

b.) Compute the value of each of the resistances from part (a).

The known values for the problem are entered in EES and converted to base SI units:

$UnitSystem SI MASS RAD PA K J


$TABSTOPS 0.2 0.4 0.6 0.8 3.5 in

"Inputs"
W=2.5 [m] "width of bathroom"
L=2.5 [m] "length of bathroom"
T_air_1=converttemp(C,K,15) "air temperature in the bathroom"
T_air_2=converttemp(C,K,5) "air temperature in the basement"
h=15 [W/m^2-K] "heat transfer coefficient"
th_L=5.0 [mm]*convert(mm,m) "linoleum thickness"
k_L=0.05 [W/m-K] "linoleum thermal conductivity"
th_P=5.0 [mm]*convert(mm,m) "plywood thickness"
k_P=0.4 [W/m-K] "plywood thermal conductivity"
th_s=6.0 [cm]*convert(cm,m) "stud thickness"
W_s=4.0 [cm]*convert(cm,m) "stud width"
k_s=0.4 [W/m-K] "stud conductivity"
p_s=25 [cm]*convert(cm,m) "stud center-to-center distance"
k_air=0.025 [W/m-K] "air conductivity"
th_d=9.0 [mm]*convert(mm,m) "drywall thickness"
k_d=0.1 [W/m-K] "drywall conductivity"

The units for each of these variables is set in the Variable Information window (select Variable
Information from the Options menu), Figure 3. The units of each of the additional variables that
are added as you solve the problem should be set in the Variable Information window.
Figure 3: Variable Information window.

The area of the floor is:

A = LW (1)

The convection resistance is computed according to:

1
Rconv = (2)
hA

The conduction resistances of the linoleum, plywood, and drywall are computed:

thL
RL = (3)
kL A

thp
Rp = (4)
kp A

thd
Rd = (5)
kd A

A=L*W "area for conduction through floor"


R_conv=1/(h*A) "convection resistance"
R_L=th_L/(k_L*A) "linoleum resistance"
R_P=th_P/(k_P*A) "plywood resistance"
R_d=th_d/(k_d*A) "drywall resistance"

The conduction resistance of the studs is computed according to:


ths
Rs = (6)
⎛W ⎞
ks A ⎜ s ⎟
⎝ ps ⎠

Note that the area for conduction is the product of the area of the floor and the fraction of the
floor occupied by the studs. The conduction resistance of the air is:

ths
Rair = (7)
⎛ p − Ws ⎞
kair A⎜ s ⎟
⎝ ps ⎠

R_s=th_s/(k_s*A*W_s/p_s) "stud resistance"


R_air=th_s/(k_air*A*(p_s-W_s)/p_s) "air resistance"

These calculations lead to Rconv = 0.011 K/W, RL = 0.016 K/W, Rp = 0.002 K/W, Rd = 0.014
K/W, Rs = 0.15 K/W, and Rair = 0.46 K/W.

c.) How much heat must be added by the heater to raise the temperature of the floor to a
comfortable 20°C?

If Ts in Figure 2 is 20°C then the heat transferred to the bathroom ( q1 ) is:

TL − Tair ,1
q1 = (8)
Rconv

T_L=converttemp(C,K,20) "linoleum temperature"


q_dot_1=(T_L-T_air_1)/R_conv "heat transferred to bathroom"

which leads to q1 = 469 W. The temperature of the heater is therefore:

Th = TL + q1 RL (9)

T_h=T_L+q_dot_1*R_L "strip heater temperature"

which leads to a heater temperature, Th = 300.7 K. The heater must provide q2 (the heat
transferred to the bathroom) as well as q1 (the heat transferred to the basement).

Th − Tair ,2
q1 = −1
(10)
⎡1 1 ⎤
Rconv + Rd + ⎢ + ⎥ + Rp
⎣ Rs Rair ⎦

and the total heater power is:


qh = q1 + q2 (11)

q_dot_2=(T_h-T_air_2)/(R_conv+R_d+(1/R_s+1/R_air)^(-1)+R_P) "heat lost to lower story"


q_dot_h=q_dot_1+q_dot_2 "total heater power"

which leads to q2 = 161 W and qh = 630 W.

d.) What physical quantities are most important to your analysis? What physical quantities are
unimportant to your analysis?

Figure 4 illustrates the values of the resistances on the resistance diagram.

Figure 4: Resistance values.

Examination of Figure 4 shows that Rp, Rd, and Rconv are unimportant relative to the amount of
heat transferred to the basement; these resistances are small in a series combination. Therefore,
you can expect that the conductivity of the drywall and plywood as well as their thickness are not
very important. Furthermore, the resistance of the air is larger than the resistance of the studs; in
a parallel combination, the larger resistance is not important. Therefore, the conductivity of air is
likely not very important. The important quantities include the conductivity of the studs and
their size as well as the thickness and conductivity of the linoleum and its thickness. The heat
transfer coefficient is also important.

e.) Discuss at least one technique that could be used to substantially reduce the amount of heater
power required while still maintaining the floor at 20°C. Note that you have no control over
Tair,1 or h.

The heat transferred to the bathroom is given by Eq. (8); you cannot change h and therefore the
value of Rconv is fixed. The only way for you can reduce the heater power is to reduce the
amount of heat transferred to the basement. This can be done most effectively by increasing the
resistance of the studs, perhaps by increasing their thickness or reducing their width.
Problem 1.2-15 (1-6 in text): The super ice-auger

You are a fan of ice fishing but don't enjoy the process of augering out your fishing hole in the
ice. Therefore, you want to build a device, the super ice-auger, that melts a hole in the ice. The
device is shown in Figure P1.2-15.

heater, activated with


2
V = 12 V and I = 150 A
h = 50 W/m -K
T∞ = 5 ° C plate,
kp = 10 W/m-K
insulation, ε = 0.9
kins = 2.2 W/m-K D = 10 inch
thins = 0.5 inch
thp = 0.75 inch

thice = 5 inch
ρice = 920 kg/m3
Δifus = 333.6 kJ/kg

Figure P1.2-15: The super ice-auger.

A heater is attached to the back of a D = 10 inch plate and electrically activated by your truck
battery, which is capable of providing V = 12 V and I = 150 A. The plate is thp = 0.75 inch thick
and has conductivity kp = 10 W/m-K. The back of the heater is insulated; the thickness of the
insulation is thins = 0.5 inch and the insulation has conductivity kins = 2.2 W/m-K. The surface of
the insulation experiences convection with surrounding air at T∞ = 5°C and radiation with
surroundings also at T∞ = 5°C. The emissivity of the surface of the insulation is ε = 0.9 and the
heat transfer coefficient between the surface and the air is h = 50 W/m2-K. The super ice-auger
is placed on the ice and activated, resulting in a heat transfer to the plate-ice interface that causes
the ice to melt. Assume that the water under the ice is at Tice = 0°C so that no heat is conducted
away from the plate-ice interface; all of the energy transferred to the plate-ice interface goes into
melting the ice. The thickness of the ice is thice = 5 inch and the ice has density ρice = 920 kg/m3.
The latent heat of fusion for the ice is Δifus = 333.6 kJ/kg.
a.) Determine the heat transfer rate to the plate-ice interface.

The inputs are entered in EES:

"P1.2-15"
$UnitSystem SI MASS RAD PA K J
$Tabstops 0.2 0.4 0.6 0.8 3.5

D=10 [inch]*convert(inch,m) "diameter of ice fishing hole"


th_ins=0.5 [inch]*convert(inch,m) "insulation thickness"
k_ins=2.2 [W/m-K] "insulation conductivity"
th_p=0.75 [inch]*convert(inch,m) "plate thickness"
k_p=10 [W/m-K] "conductivity of plate"
e=0.9 [-] "emissivity"
h_bar=50 [W/m^2-K] "air heat transfer coefficient"
T_infinity=converttemp(C,K,5 [C]) "ambient temperature"
V=12 [V] "battery voltage"
I=150 [A] "current"
T_ice=converttemp(C,K,0 [C]) "temperature of ice-water interface"
th_ice=5 [inch]*convert(inch,m) "thickness of ice"
rho_ice=920 [kg/m^3] "density of ice"
DELTAi_fus=333.6 [kJ/kg]*convert(kJ/kg,J/kg) "enthalpy of fusion of ice"

The power provided to the heater is the product of the voltage and current:

q = V I (1)

q_dot=V*I "power to melting plate"

A resistance network that can be used to represent this problem is shown in Figure P1.2-15-2.

Rrad = 3.56 K/W

q
q2 q1

Tice = 0°C T∞ = 5°C


Ttop
Rcond , p = 0.038 K/W
Rcond ,ins = 0.114 K/W
Rconv = 0.395 K/W

The resistances include:


Rcond,p = conduction through plate
Rcond,ins = conduction through insulation
Rrad = radiation resistance
Rconv = convection resistance
Figure P1.2-15-2: The super ice-auger.

In order to compute the radiation resistance required to solve the problem, it is necessary to
assume a value of Ttop, the temperature at the top of the insulation (this value will eventually be
commented out in order to complete the problem):

T_top=360 [K] "guess for top surface temperature"

The cross-sectional area of the plate is computed:

π D2
Ac = (2)
4

and the radiation resistance is computed according to:

1
Rrad = (3)
ε Ac σ (T + T∞2 ) (Ttop + T∞ )
2
top

A_c=pi*D^2/4 "area of hole"


R_rad=1/(e*A_c*sigma#*(T_top^2+T_infinity^2)*(T_top+T_infinity)) "radiation resistance"
Note that the equations should be entered, the units set, and the EES code solved line by line in
order to debug the code in small segments.

The convection resistance is computed according to:

1
Rconv = (4)
Ac h

and the conduction resistances are computed according to:

thins
Rcond ,ins = (5)
Ac kins

thp
Rcond , p = (6)
Ac k p

R_conv=1/(A_c*h_bar) "air convection resistance"


R_cond_ins=th_ins/(k_ins*A_c) "conduction resistance of insulation"
R_cond_p=th_p/(k_p*A_c) "conduction resistance of plate"

The heat transfer from the heater to the ambient surroundings ( q1 in Figure P1.2-15-2) is:

q1 =
(Th − T∞ ) (7)
−1
⎛ 1 1 ⎞
Rcond ,ins + ⎜ + ⎟
⎝ Rrad Rconv ⎠

and the heat transfer to the ice is:

q2 =
(Th − Tice ) (8)
Rcond , p

where Th is the heater temperature. An energy balance on the heater leads to:

q = q1 + q2 (9)

Equations (7) through (9) are 3 equations in 3 unknowns ( q , q1 , and q2 ) and can be solved using
EES:

q_dot_1=(T_h-T_infinity)/(R_cond_ins+(1/R_rad+1/R_conv)^(-1)) "heat transfer to ambient"


q_dot_2=(T_h-T_ice)/R_cond_p "heat transfer to ice"
q_dot=q_dot_1+q_dot_2 "energy balance"
The temperature at the top of the plate can be computed based on the solution. Update the guess
values for the problem (select Update Guess Values from the Calculate menu) and comment out
the guessed value for Ttop:

{T_top=360 [K]} "guess for top surface temperature"

and calculate Ttop according to the resistance network:

Ttop = Th − q1 Rcond ,ins (10)

T_top=T_h-q_dot_1*R_cond_ins "recalculate top temperature"

The result is q2 = 1676 W.

The values of the resistances are shown in Figure P1.2-15-2; notice that radiation does not play
an important role in the problem because it is a large resistance in parallel with a much smaller
one. The resistance to conduction through the plate is also unimportant since it is so small. The
resistance to conduction through the insulation and convection are dominant.

b.) How long will it take to melt a hole in the ice?

An energy balance on the ice-to-plate interface leads to:

dthice
q2 = Ac Δi fus ρice (11)
dt

dthice
where is the rate at which the thickness of the ice is reduced. Because there is no energy
dt
lost to the water, the rate of ice melting is constant with ice thickness. Therefore the time
required to melt the ice is estimated according to:

dthice
time = thice (12)
dt

q_dot_2=A_c*DELTAi_fus*dth_icedt*rho_ice "energy balance on ice interface"


dth_icedt*time=th_ice "time to melt ice"
time_min=time*convert(s,min) "in min"

which leads to time = 1178 s (19.6 min).

c.) What is the efficiency of the melting process?

The efficiency is defined as the ratio of the energy provided to the plate-to-ice interface to the
energy provided to the heater:
q2
η= (13)
q

eta=q_dot_2/q_dot "efficiency of process"

which leads to η = 0.93.

d.) If your battery is rated at 100 amp-hr at 12 V then what fraction of the battery's charge is
depleted by running the super ice-auger?

The total amount of energy required to melt a hole in the ice is:

Q = q time (14)

The energy stored in the battery (Ebattery) is the product of the voltage and the amp-hr rating. The
fraction of the battery charge required is:

Q
f = (15)
Ebattery

Q=q_dot*time "total energy required"


E_battery=100 [amp-hr]*V*convert(A-V-hr,J) "car battery energy"
f=Q/E_battery "fraction of car battery energy used"

which leads to f = 0.491.


Problem 1.3-3 (1-7 in text): Critical Evaluation of a Solution
One of the engineers that you supervise has been asked to simulate the heat transfer problem
shown in Figure P1.3-3(a). This is a 1-D, plane wall problem (i.e., the temperature varies only in
the x-direction and the area for conduction is constant with x). Material A (from 0 < x < L) has
conductivity kA and experiences a uniform rate of volumetric thermal energy generation, g ′′′ .
The left side of material A (at x = 0) is completely insulated. Material B (from L < x < 2L) has
lower conductivity, kB < kA. The right side of material B (at x= 2L) experiences convection with
fluid at room temperature (20°C). Based on the facts above, critically examine the solution that
has been provided to you by the engineer and is shown in Figure P1.3-3(b). There should be a
few characteristics of the solution that do not agree with your knowledge of heat transfer; list as
many of these characteristics as you can identify and provide a clear reason why you think the
engineer’s solution must be wrong.
250

200

150

Temperature (°C)
100
L L
50
material A material B
0
kA kB < kA
h , T f = 20°C -50
g ′′′A = g ′′′ g B′′′ = 0 Material A Material B
x -100
0 L 2L
Position (m)
(a) (b)
Figure P1.3-3: (a) Heat transfer problem and (b) "solution" provided by the engineer.

1. The left side of material A is insulated; therefore, the temperature gradient should be zero.
2. Material A has a higher conductivity than material B; therefore, at x = L the temperature
gradient should be larger in material B than in material A.
3. Heat is transferred to the fluid at 20°C; therefore the temperature at x = 2 L must be greater
than 20°C.
PROBLEM 1.3-8 (1-8 in text): Hay Temperature
Freshly cut hay is not really dead; chemical reactions continue in the plant cells and therefore a
small amount of heat is released within the hay bale. This is an example of the conversion of
chemical to thermal energy and can be thought of as thermal energy generation. The amount of
thermal energy generation within a hay bale depends on the moisture content of the hay when it
is baled. Baled hay can become a fire hazard if the rate of volumetric generation is sufficiently
high and the hay bale sufficiently large so that the interior temperature of the bale reaches 170°F,
the temperature at which self-ignition can occur. Here, we will model a round hay bale that is
wrapped in plastic to protect it from the rain. You may assume that the bale is at steady state and
is sufficiently long that it can be treated as a one-dimensional, radial conduction problem. The
radius of the hay bale is Rbale = 5 ft and the bale is wrapped in plastic that is tp = 0.045 inch thick
with conductivity kp = 0.15 W/m-K. The bale is surrounded by air at T∞ = 20°C with h = 10
W/m2-K. You may neglect radiation. The conductivity of the hay is k = 0.04 W/m-K.
a.) If the volumetric rate of thermal energy generation is constant and equal to g ′′′ = 2 W/m3
then determine the maximum temperature in the hay bale.

The inputs are entered in EES:

$UnitSystem SI MASS RAD PA K J


$TABSTOPS 0.2 0.4 0.6 0.8 3.5 in

R_bale=5 [ft]*convert(ft,m) "hay bale radius"


t_p=0.045 [inch]*convert(inch,m) "plastic thickness"
k_p= 0.15 [W/m-K] "plastic conductivity"
h=10 [W/m^2-K] "heat transfer coefficient"
T_infinity=converttemp(C,K,20 [C]) "ambient temperature"
L=1 [m] "per unit length of bale"
k = 0.04 [W/m-K] "conductivity of hay"
g_dot_v=2 [W/m^3] "volumetric heat generation"

This is an example of a one-dimensional steady conduction problem with constant volumetric


generation and therefore the formulae provided in Table 1-3 can used directly. The general
solution is:

g ′′′ r 2
T =− + C1 ln ( r ) + C2 (1)
4k

where C1 and C2 are constants selected to enforce the boundary conditions. The boundary
condition at the center of the bale is either that the temperature remain bounded or that the
temperature gradient be zero; either will lead to C1 = 0. An energy balance at the outer edge of
the hay bale leads to:

dT Tr = Rbale − T∞
− k 2 π Rbale L = (2)
dr r = Rbale R p + Rconv
where Rp and Rconv are the thermal resistances associated with conduction through the plastic and
convection from the outer surface of the bale, respectively:

tp
Rp = (3)
k p 2 π Rbale L

1
Rconv = (4)
h 2 π Rbale L

where L = 1 m for a problem that is done on a unit length basis. The temperature gradient and
temperature at the outer radius of the bale are obtained using Eq. (1) with C1 = 0:

dT g ′′′ Rbale
=− (5)
dr r = Rbale 2k

g ′′′ Rbale
2
Tr = Rbale = − + C2 (6)
4k

Placing equations (2) through (6) into EES allows the constants of integration to be determined:

R_p=t_p/(k_p*2*pi*R_bale*L)
"thermal resistance associated with conduction through plastic"
R_conv=1/(2*pi*R_bale*L*h)
"thermal resistance associated with convection"
dTdr_Rbale=-g_dot_v*R_bale/(2*k) "temperature gradient at outer edge"
T_Rbale=-g_dot_v*R_bale^2/(4*k)+C_2 "temperature at outer edge"
-k*2*pi*R_bale*L*dTdr_Rbale=(T_Rbale-T_infinity)/(R_p+R_conv) "interface energy balance"

The maximum temperature in the bale occurs at the center; according to Eq. (1) with C1 = 0, this
temperature is given by:

g ′′′ r 2
T =− + C1 ln ( r ) + C2 (7)
4k

T_max=C_2 "maximum bale temperature"


T_max_F=converttemp(K,F,T_max) "maximum bale temperature in F"

The maximum temperature in the hay bale is 322.3 K or 120.6°F.

b.) Prepare a plot showing the maximum temperature in the hay bale as a function of the hay
bale radius. How large can the hay bale be before there is a problem with self-ignition?

A parametric table is generated that contains the variables T_max_F and R_bale and used to
generate Figure 1.
Figure 1: Maximum temperature as a function of the bale radius.

Note that a hay bale larger than approximately 2.1 m will result in a hay fire.

Prepare a model that can consider temperature-dependent volumetric generation. Increasing


temperature tends to increase the rate of chemical reaction and therefore increases the rate of
generation of thermal energy according to: g ′′′ = a + bT where a = -1 W/m3 and b = 0.01
W/m3-K and T is in K.
c.) Enter the governing equation into Maple and obtain the general solution (i.e., a solution that
includes two constants).

The governing differential equation is obtained as discussed in Section 1.3:

d ⎛ dT ⎞
g ′′′ r = ⎜ −k r ⎟ (8)
dr ⎝ dr ⎠

This ordinary differential equation is entered in Maple:

> restart;
> ODE:=(a+b*T(r))*r=diff(-k*r*diff(T(r),r),r);
2
⎛ d ⎞ ⎛d ⎞
ODE := ( a + b T( r ) ) r = −k ⎜⎜ T ( r ) ⎟⎟ − k r ⎜ T( r ) ⎟⎟
⎝ d r ⎠ ⎜ 2
⎝ dr ⎠

and solved:

> Ts:=dsolve(ODE);
⎛ b ⎞ ⎛ b ⎞ a
Ts := T( r ) = BesselJ⎜⎜ 0, r ⎟⎟ _C2 + BesselY⎜⎜ 0, r ⎟⎟ _C1 −
⎝ k ⎠ ⎝ k ⎠ b

Note that the solution is given in the form of Bessel functions;

⎛ b ⎞ ⎛ b ⎞ a
T = −C2 BesselJ ⎜⎜ 0, r ⎟⎟ + C1 BesselY ⎜⎜ 0, r ⎟⎟ − (9)
⎝ k ⎠ ⎝ k ⎠ b

Even though we have not yet learned about Bessel functions, we can manipulate this solution
within Maple.

d.) Use the boundary conditions to obtain values for the two constants in your general solution
(hint: one of the two constants must be zero in order to keep the temperature at the center of
the hay bale finite). You should obtain a symbolic expression for the boundary condition in
Maple that can be evaluated in EES.

In part (a) we could not take the natural logarithm of 0 in Eq. (7) and therefore C1 was zero. A
similar thing happens with the Bessel functions. We can evaluate the limits of the two Bessel
functions as r → 0:

> limit(BesselJ(0,r),r=0);
1
> limit(BesselY(0,r),r=0);
−∞

The BesselY function becomes infinite and therefore C1 in Eq. (9) must be 0.

> Ts:=subs(_C1=0,Ts);
⎛ b ⎞ a
Ts := T( r ) = BesselJ⎜⎜ 0, r ⎟⎟ _C2 −
⎝ k ⎠ b

The boundary condition at the outer surface of the hay does not change; the temperature and
temperature gradient at Rbale can be evaluated symbolically using Maple:

> dTdr_Rbale:=eval(diff(rhs(Ts),r),r=R_bale);
⎛ b ⎞ b
dTdr_Rbale := −BesselJ⎜⎜ 1, R_bale ⎟⎟ _C2
⎝ k ⎠ k
> T_Rbale:=eval(rhs(Ts),r=R_bale);
⎛ b ⎞ a
T_Rbale := BesselJ⎜⎜ 0, R_bale ⎟⎟ _C2 −
⎝ k ⎠ b
These symbolic expressions are cut and paste into EES and used to replace Eqs. (5) and (6) and
provide a new constant C2:

{g_dot_v=2 [W/m^3] "volumetric heat generation"}


a=-1 [W/m^3]
"coefficients for volumetric generation function"
b=0.01 [W/m^3-K]

R_p=t_p/(k_p*2*pi*R_bale*L)
"thermal resistance associated with conduction through plastic"
R_conv=1/(2*pi*R_bale*L*h)
"thermal resistance associated with convection"
{dTdr_Rbale=-g_dot_v*R_bale/(2*k) "temperature gradient at outer edge"
T_Rbale=-g_dot_v*R_bale^2/(4*k)+C_2 "temperature at outer edge"}
dTdr_Rbale = -BesselJ(1,(b/k)^(1/2)*R_bale)*(b/k)^(1/2)*C_2
"symbolic expressions cut and paste from Maple"
T_Rbale = BesselJ(0,(b/k)^(1/2)*R_bale)*C_2-1/b*a
-k*2*pi*R_bale*L*dTdr_Rbale=(T_Rbale-T_infinity)/(R_p+R_conv) "interface energy balance"

The maximum temperature is the temperature at the center of the bale; this is evaluated using
Maple:

> T_max=eval(rhs(Ts),r=0);
a
T_max = _C2 −
b

and copied and pasted into EES:

{T_max=C_2 "maximum bale temperature"}


T_max = C_2-1/b*a
"symbolic expression cut and paste from Maple"
T_max_F=converttemp(K,F,T_max) "maximum bale temperature in F"

e.) Overlay on your plot from part (b) a plot of the maximum temperature in the hay bale as a
function of bale radius when the volumetric generation is a function of temperature.

The result is shown in Figure 1.


Problem 1.3-9 (1-9 in text): Mass Flow Meter
Figure P1.3-9 illustrates a simple mass flow meter for use in an industrial refinery.

T∞ = 20°C insulation
hout = 20 W/m -K
2 kins = 1.5 W/m-K

test section
rout = 1 inch
g ′′′ = 1x10 W/m
7 3

rin = 0.75 inch k = 10 W/m-K

m = 0.75kg/s
T f = 18°C

L = 3 inch thins = 0.25


Figure P1.3-9: A simple mass flow meter.

A flow of liquid passes through a test section consisting of an L = 3 inch section of pipe with
inner and outer radii, rin = 0.75 inch and rout = 1.0 inch, respectively. The test section is
uniformly heated by electrical dissipation at a rate g ′′′ = 1x107 W/m3 and has conductivity k = 10
W/m-K. The pipe is surrounded with insulation that is thins = 0.25 inch thick and has
conductivity kins = 1.5 W/m-K. The external surface of the insulation experiences convection
with air at T∞ = 20°C. The heat transfer coefficient on the external surface is hout = 20 W/m2-K.
A thermocouple is embedded at the center of the pipe wall. By measuring the temperature of the
thermocouple, it is possible to infer the mass flow rate of fluid because the heat transfer
coefficient on the inner surface of the pipe ( hin ) is strongly related to mass flow rate ( m ).
Testing has shown that the heat transfer coefficient and mass flow rate are related according to:

0.8
⎛ m ⎞
hin = C ⎜⎜ ⎟⎟
⎝ 1[ kg/s ] ⎠

where C= 2500 W/m2-K. Under nominal conditions, the mass flow rate through the meter is m
= 0.75 kg/s and the fluid temperature is Tf = 18°C. Assume that the ends of the test section are
insulated so that the problem is 1-D. Neglect radiation and assume that the problem is steady-
state.
a.) Develop an analytical model in EES that can predict the temperature distribution in the test
section. Plot the temperature as a function of radial position for the nominal conditions.

The inputs are entered in EES:

$UnitSystem SI MASS RAD PA K J


$TABSTOPS 0.2 0.4 0.6 0.8 3.5 in

"Inputs"
r_out=1.0 [inch]*convert(inch,m) "outer radius of measurement section"
r_in=0.75 [inch]*convert(inch,m) "inner radius of measurement section"
h_bar_out=20 [W/m^2-K] "external convection coefficient"
T_infinity=converttemp(C,K,20 [C]) "ambient temperature"
T_f=converttemp(C,K, 18 [C]) "fluid temperature"
k=10 [W/m-K] "conductivity"
g```=1e7 [W/m^3] "volumetric rate of thermal energy generation"
m_dot=0.75 [kg/s] "mass flow rate"
th_ins=0.25 [inch]*convert(inch,m) "thickness of insulation"
k_ins=1.5 [W/m-K] "insulation conductivity"
L= 3 [inch]*convert(inch,m) "length of test section"

The heat transfer coefficient on the internal surface is computed according to the specified mass
flow rate:

C=2500 [W/m^2-K] "constant for convection relationship"


h_bar_in=C*(m_dot/1 [kg/s])^0.8 "internal convection coefficient"

The general solution to a 1-D problem in cylindrical coordinates with constant volumetric
thermal energy generation was provided in Table 1-3, to within the unknown constants C1 and
C2:

g ′′′ r 2
T =− + C1 ln ( r ) + C2 (1)
4k

dT g ′′′ r C1
=− + (2)
dr 2k r

The boundary condition at the outer edge of the test section is:

⎛ dT ⎞
− k 2 π rout L ⎜ =
(
Tr = rout − T∞ )
⎟ (3)
⎝ dr ⎠ r = rout ( Rins + Rconv ,out )

where Rins is the thermal resistance to conduction through the insulation (provided in Table 1-2):

⎡ ( r + thins ) ⎤
ln ⎢ out ⎥
rout
Rins = ⎣ ⎦ (4)
2 π L kins

and Rconv,out is the resistance to convection from the outer surface of the insulation:

1
Rconv ,out = (5)
2 π ( rout + thins ) L hout

R_ins=ln((r_out+th_ins)/r_out)/(2*pi*L*k_ins) "resistance to conduction through insulation"


R_conv_out=1/(2*pi*(r_out+th_ins)*L*h_bar_out) "resistance to convection from outer surface"
T_r_out=-g```*r_out^2/(4*k)+C_1*ln(r_out)+C_2 "temperature at outer surface of section"
dTdr_r_out=-g```*r_out/(2*k)+C_1/r_out "temperature gradient at outer surface of section"
-k*2*pi*r_out*L*dTdr_r_out=(T_r_out-T_infinity)/(R_ins+R_conv_out)
"boundary condition at r=r_out"

The boundary condition at the inner edge of the test section is:

⎛ dT ⎞
(
hin 2 π rin L T f − Tr = rin = −k 2 π rin L ⎜) ⎟
⎝ dr ⎠ r = rin
(6)

T_r_in=-g```*r_in^2/(4*k)+C_1*ln(r_in)+C_2 "temperature at inner surface of section"


dTdr_r_in=-g```*r_in/(2*k)+C_1/r_in "temperature gradient at inner surface of section"
h_bar_in*2*pi*r_in*L*(T_f-T_r_in)=-k*2*pi*r_in*L*dTdr_r_in
"boundary condition at r=r_in"

The EES code will provide the solution to the constants C1 and C2; note that it is not possible to
eliminate the unit warnings that are associated with the argument of the natural logarithm in Eq.
(1). In fact, if sufficient algebra was carried out, the equations could be placed in a form where
the natural logarithm had a dimensionless argument.

The location at which to evaluate the temperature (r) is specified in terms of a dimensionless
radial position ( r ) that goes from 0 at the inner surface of the test section to 1 at the outer
surface. The temperature is evaluated using Eq. (1):

r_bar=0.5 [-] "dimensionless radial position"


r=r_in+r_bar*(r_out-r_in) "radial position"
T=-g```*r^2/(4*k)+C_1*ln(r)+C_2 "temperature"
T_C=converttemp(K,C,T) "in C"

Figure P1.3-9-2 illustrates the temperature as a function of radial position.


80

75
Temperature (°C)

70

65

60

55

50
0.019 0.02 0.021 0.022 0.023 0.024 0.025 0.026
Radius (m)
Figure 1.3-9-2: Temperature as a function of radius.

b.) Using your model, develop a calibration curve for the meter; that is, prepare a plot of the
mass flow rate as a function of the measured temperature at the mid-point of the pipe. The
range of the instrument is 0.2 kg/s to 2.0 kg/s.
The dimensionless radial position is set to r =0.5, corresponding to the temperature of the center
of the test section. Figure 1.3-9-3 illustrates the mass flow rate through the meter as a function
of the measured temperature.
2
1.8
1.6

Mass flow rate (kg/s)


1.4
1.2
Tf = 28°C
1 Tf = 18°C
0.8 Tf = 8°C
0.6
0.4
0.2
0
40 60 80 100 120 140 160
Temperature (°C)
Figure 1.3-9-3: Mass flow rate as a function of the temperature at the center of the pipe wall for several
values of the fluid temperature.

The meter must be robust to changes in the fluid temperature. That is, the calibration curve
developed in (b) must continue to be valid even as the fluid temperature changes by as much as
10°C.
c.) Overlay on your plot from (b) the mass flow rate as a function of the measured temperature
for Tf = 8°C and Tf = 28°C. Is your meter robust to changes in Tf?

The calibration curves generated at Tf = 8°C and Tf = 28°C are also shown in Figure 1.3-9-3.
Notice that the fluid temperature has a large effect on the device. For example, if the measured
temperature is 80°C then the mass flow rate could be anywhere from 0.45 kg/s to 0.75 kg/s
depending on the fluid temperature. The meter is not robust to changes in Tf.

In order to improve the meters ability to operate over a range of fluid temperature, a temperature
sensor is installed in the fluid in order to measure Tf during operation.
d.) Using your model, develop a calibration curve for the meter in terms of the mass flow rate as
a function of ΔT, the difference between the measured temperatures at the mid-point of the
pipe wall and the fluid.

The temperature difference is calculated according to:

ΔT = Tr =0.5 − T f (7)

DT=T-T_f "measured temperature difference"

Figure 1.3-9-4 illustrates the mass flow rate as a function of the temperature difference:
2
1.8 Tf = 28°C

1.6 Tf = 18°C

Mass flow rate (kg/s)


Tf = 8°C
1.4
1.2
1
0.8
0.6
0.4
0.2
0
30 40 50 60 70 80 90 100 110 120
Temperature difference (K)
Figure 1.3-9-4: Mass flow rate as a function of the temperature difference between the measured temperature
at the center of the pipe wall and the fluid temperature for several values of the fluid temperature.

e.) Overlay on your plot from (d) the mass flow rate as a function of the difference between the
measured temperatures at the mid-point of the pipe wall and the fluid if the fluid temperature
is Tf = 8°C and Tf = 28°C. Is the meter robust to changes in Tf?

The calibration curves for Tf = 8°C and Tf = 28°C are also shown in Figure 1.3-9-4; notice that
the fluid temperature has almost no effect on the calibration curves and so the meter is robust to
changes in the fluid temperature.

f.) If you can measure the temperature difference to within δΔT = 1 K then what is the
uncertainty in the mass flow rate measurement? (Use your plot from part (d) to answer this
question.)

The uncertainty in the measured mass flow rate that corresponds to an uncertainty in the
temperature difference is evaluated according to:

⎛ ∂m ⎞
δ m = ⎜ ⎟ δΔT (8)
⎝ ∂ΔT ⎠

From Figure 1.3-9-4 we see that the partial derivative of mass flow rate with respect to
temperature difference decreases with flow rate. At high flow rates (around 2 kg/s), the partial
derivative is approximately 0.08 kg/s-K which leads to an uncertainty of 0.08 kg/s. At low flow
rates (around 0.2 kg/s), the partial derivative is approximately 0.04 kg/s-K which leads to an
uncertainty of 0.04 kg/s.

You can use the built-in uncertainty propagation feature in EES to assess uncertainty
automatically.

g.) Set the temperature difference to the value you calculated at the nominal conditions and
allow EES to calculate the associated mass flow rate. Now, select Uncertainty Propagation
from the Calculate menu and specify that the mass flow rate is the calculated variable while
the temperature difference is the measured variable. Set the uncertainty in the temperature
difference to 1 K and verify that EES obtains an answer that is approximately consistent with
part (f).
The temperature difference is set to 50 K corresponding to approximately the middle of the range
of the device. The mass flow rate is commented out and EES is used to calculate the mass flow
rate from the temperature difference:

DT=50 [K]
{m_dot=0.75 [kg/s]} "mass flow rate"

Select Uncertainty Propagation from the Calculate menu (Figure P1.3-9-5) and select the
variable m_dot as the calculated variable and the variable DT as the measured variable.

Figure P1.3-9-5: Determine Propagation of Uncertainty dialog.

Select Set uncertainties and indicate that the uncertainty of the measured temperature difference
is 1 K (Figure P1.3-9-6).

Figure P1.3-9-6: Uncertainties of Measured Variables dialog.

Select OK and then then OK again to carry out the calculation. The results are displayed in the
Uncertainty Results tab of the Solution window (Figure P1.3-9-7).

Figure P1.3-9-7: Uncertainties Results tab of the Solution window.


The uncertainty calculated by EES is δ m = 0.031 kg/s, which falls between the bounds
identified in part (e).

h.) The nice thing about using EES to determine the uncertainty is that it becomes easy to assess
the impact of multiple sources of uncertainty. In addition to the uncertainty δΔT, the
constant C has an uncertainty of δC = 5% and the conductivity of the material is only known
to within δk = 3%. Use EES' built-in uncertainty propagation to assess the resulting
uncertainty in the mass flow rate measurement. Which source of uncertainty is the most
important?

Select Uncertainty Propagation from the Calculate menu and select the variable m_dot as the
calculated variable and the variables DT, C, and k as the measured variables. Set the uncertainty
of each of the measured variables according to the problem statement (Figure P1.3-9-8).

Figure P1.3-9-8: Uncertainties of Measured Variables dialog.

The results of the uncertainty calculation are shown in Figure P1.3-9-9.

Figure P1.3-9-9: Uncertainties Results tab of the Solution window.

Notice that the uncertainty has increased to δ m = 0.062 kg/s and that the dominant source of the
uncertainty is related to C. The effect of the uncertainty in the conductivity is small (only 5.8%
of the total).

i.) The meter must be used in areas where the ambient temperature and heat transfer coefficient
may vary substantially. Prepare a plot showing the mass flow rate predicted by your model
for ΔT = 50 K as a function of T∞ for various values of hout . If the operating range of your
meter must include -5°C < T∞ < 35°C then use your plot to determine the range of hout that
can be tolerated without substantial loss of accuracy.

Figure P1.3-9-10 illustrates the mass flow rate as a function of T∞ for various values of hout .

0.9
2
h = 5 W/m -K
2
0.85 10 W/m -K
2
20 W/m -K
2
50 W/m -K

Mass flow rate (kg/s)


0.8
2
100 W/m -K
0.75

0.7

0.65

0.6
-10 -5 0 5 10 15 20 25 30 35 40 45 50 55
Air temperature (°C)
Figure P1.3-9-10: Mass flow rate predicted with ΔT = 50 K as a function of ambient temperature for various
values of the air heat transfer coefficient.

The shaded region in Figure P1.3-9-10 indicates the operating temperature range (in the x-
direction) and the region of acceptable accuracy (based approximately on the results of part (e)).
Figure P1.3-9-10 shows that 5 W/m2-K < hout < 50 W/m2-K will keep you within the shaded
region and therefore this is, approximately, the range of hout that can be tolerated without
substantial loss of accuracy.
PROBLEM 1.4-2 (1-10 in text): Mass Flow Meter (revisited)
Reconsider the mass flow meter that was investigated in Problem 1.3-9 (1-9 in text). The
conductivity of the material that is used to make the test section is not actually constant as was
assumed in Problem 1-9 but rather depends on temperature according to:

⎡ W ⎤
2 (
T − 300 [ K ])
W
k = 10 + 0.035 ⎢
m-K ⎣ m-K ⎥⎦

a.) Develop a numerical model of the mass flow meter using EES. Plot the temperature as a
function of radial position for the conditions shown in Figure P1.3-9 (Figure P1-9 in the text)
with the temperature-dependent conductivity.

The inputs are entered in EES:

$UnitSystem SI MASS RAD PA K J


$TABSTOPS 0.2 0.4 0.6 0.8 3.5 in

"Inputs"
r_out=1.0 [inch]*convert(inch,m) "outer radius of measurement section"
r_in=0.75 [inch]*convert(inch,m) "inner radius of measurement section"
h_bar_out=10 [W/m^2-K] "external convection coefficient"
T_infinity_C=20 [C] "ambient temperature in C"
T_infinity=converttemp(C,K,T_infinity_C) "ambient temperature"
T_f=converttemp(C,K, 18 [C]) "fluid temperature"
g```=1e7 [W/m^3] "volumetric rate of thermal energy generation"
m_dot=0.75 [kg/s] "mass flow rate"
th_ins=0.25 [inch]*convert(inch,m) "thickness of insulation"
k_ins=1.5 [W/m-K] "insulation conductivity"
L= 3 [inch]*convert(inch,m) "length of test section"
C=2500 [W/m^2-K] "constant for convection relationship"
h_bar_in=C*(m_dot/1 [kg/s])^0.8 "internal convection coefficient"

A function is defined that returns the conductivity of the material:

Function k_t(T)
"This function returns the conductivity of the test section material as a function of temperature"
k_t=10 [W/m-K]+0.035 [W/m-K^2]*(T-300 [K])
end

A uniform distribution of nodes is used, the radial location of each node (ri) is:

( i − 1) r − r
ri = rin + ( ) for i = 1..N (1)
( N − 1) out in
where N is the number of nodes. The radial distance between adjacent nodes (Δr) is:

Δr =
( rout − rin ) (2)
( N − 1)
N=51 [-] "number of nodes"
DELTAr=(r_out-r_in)/(N-1) "distance between adjacent nodes (m)"
"Set up nodes"
duplicate i=1,N "this loop assigns the radial location to each node"
r[i]=r_in+(r_out-r_in)*(i-1)/(N-1)
end

An energy balance is carried out on a control volume surrounding each node. For node 1, placed
at the inner surface (Figure P1.4-2-1):

qconv ,in + qouter + g = 0 (3)

qouter T2
g1
T1
qconv ,in
Figure P1.4-2-1: Control volume around node 1.

The rate equation for convection is:

qconv ,in = hin 2 π rin L (T f − T1 ) (4)

The rate equation for conduction is:

⎛ Δr ⎞ (T2 − T1 )
qouter = kT =(T1 +T2 ) / 2 2 π ⎜ rin + ⎟L (5)
⎝ 2 ⎠ Δr

The rate equation for generation is:

Δr
g = 2 π rin L g ′′′ (6)
2

"Node 1"
q_dot_conv_in=h_bar_in*2*pi*r_in*L*(T_f-T[1]) "convection from fluid"
g_dot[1]=2*pi*r_in*L*DELTAr*g```/2 "generation"
q_dot_outer[1]=k_t((T[1]+T[2])/2)*2*pi*(r[1]+DELTAr/2)*L*(T[2]-T[1])/DELTAr "conduction from node 2"
q_dot_conv_in+q_dot_outer[1]+g_dot[1]=0 "energy balance on node 1"

An energy balance on an internal node is shown in Figure P1.4-2-2:

qinner + qouter + g = 0 (7)


qouter Ti+1
g Ti
qinner Ti-1
Figure P1.4-2-2: Control volume around internal node i.

The rate equations for conduction are:

⎛ Δr ⎞ (Ti +1 − Ti )
qouter = kT =(Ti +Ti+1 ) / 2 2 π ⎜ rin + ⎟L (8)
⎝ 2 ⎠ Δr

⎛ Δr ⎞ ( T − T )
qinner = kT =(Ti +Ti−1 ) / 2 2 π ⎜ rin − ⎟ L i −1 i (9)
⎝ 2 ⎠ Δr

The rate equation for generation is:

g = 2 π ri Δr L g ′′′ (10)

"Internal nodes"
duplicate i=2,(N-1)
q_dot_inner[i]=k_t((T[i]+T[i-1])/2)*2*pi*(r[i]-DELTAr/2)*L*(T[i-1]-T[i])/DELTAr
"conduction from inner node"
q_dot_outer[i]=k_t((T[i]+T[i+1])/2)*2*pi*(r[i]+DELTAr/2)*L*(T[i+1]-T[i])/DELTAr
"conduction from outer node"
g_dot[i]=2*pi*r[i]*L*DELTAr*g``` "generation"
q_dot_inner[i]+q_dot_outer[i]+g_dot[i]=0 "energy balance on node i"
end

An energy balance on node N placed on the outer surface is shown in Figure P1.4-2-3:

qinner + qair + g = 0 (11)

qair
TN
g
qinner TN-1
Figure P1.4-2-3: Control volume around internal node N.

The rate equation for the heat transfer with the air is:

qair =
(T∞ − TN ) (12)
(R ins + Rconv ,out )
where

⎡ ( r + thins ) ⎤
ln ⎢ out ⎥
Rins = ⎣ ⎦
rout
(13)
2 π L kins

1
Rconv ,out = (14)
2 π ( rout + thins ) L hout

The rate equation for conduction is:

⎛ Δr ⎞ (T − T )
qinner = kT =(TN +TN −1 ) / 2 2 π ⎜ rout − ⎟ L N −1 N (15)
⎝ 2 ⎠ Δr

The rate equation for generation is:

Δr
g = 2 π rout L g ′′′ (16)
2

"Node N"
R_ins=ln((r_out+th_ins)/r_out)/(2*pi*L*k_ins) "resistance to conduction through insulation"
R_conv_out=1/(2*pi*(r_out+th_ins)*L*h_bar_out) "resistance to convection from outer surface"
q_dot_air=(T_infinity-T[N])/(R_ins+R_conv_out) "heat transfer from air"
q_dot_inner[N]=k_t((T[N]+T[N-1])/2)*2*pi*(r_out-DELTAr/2)*L*(T[N-1]-T[N])/DELTAr
"conduction from node N-1"
g_dot[N]=2*pi*r_out*L*DELTAr*g```/2 "generation"
q_dot_air+q_dot_inner[N]+g_dot[N]=0 "energy balance on node N"

The solution is converted to degrees Celsius:

duplicate i=1,N
T_C[i]=converttemp(K,C,T[i]) "convert solution to deg. C"
end

The solution is illustrated in Figure P1.4-2-4.


75

72.5

70

Temperature (°C)
67.5

65

62.5

60

57.5

55

52.5
0.019 0.02 0.021 0.022 0.023 0.024 0.025 0.026
Radius (m)
Figure P1.4-2-4: Temperature as a function of radius.

b.) Verify that your numerical solution limits to the analytical solution from Problem 1.3-9 (1-9
in the text) in the limit that the conductivity is constant.

The conductivity function is modified temporarily so that it returns a constant value:

Function k_t(T)
"This function returns the conductivity of the test section material as a function of temperature"
k_t=10 [W/m-K]{+0.035 [W/m-K^2]*(T-300 [K])}
end

The analytical solution from P1.3-9 is programmed and used to compute the analytical solution
at each node:

"Analytical solution from P1.3-9"


k=k_t(300 [K]) "conductivity to use in the solution"
T_r_out=-g```*r_out^2/(4*k)+C_1*ln(r_out)+C_2 "temperature at outer surface of section"
dTdr_r_out=-g```*r_out/(2*k)+C_1/r_out "temperature gradient at outer surface of section"
-k*2*pi*r_out*L*dTdr_r_out=(T_r_out-T_infinity)/(R_ins+R_conv_out) "boundary condition at r=r_out"
T_r_in=-g```*r_in^2/(4*k)+C_1*ln(r_in)+C_2 "temperature at inner surface of section"
dTdr_r_in=-g```*r_in/(2*k)+C_1/r_in "temperature gradient at inner surface of section"
h_bar_in*2*pi*r_in*L*(T_f-T_r_in)=-k*2*pi*r_in*L*dTdr_r_in "boundary condition at r=r_in"

duplicate i=1,N
T_an[i]=-g```*r[i]^2/(4*k)+C_1*ln(r[i])+C_2 "temperature"
T_an_C[i]=converttemp(K,C,T_an[i]) "in C"
end

Figure P1.4-2-5 illustrates the temperature distribution predicted by the numerical and analytical
solutions in the limit that k is constant.
80

75

Temperature (°C)
70

65

60 analytical model
numerical model
55

50
0.019 0.02 0.021 0.022 0.023 0.024 0.025 0.026
Radius (m)
Figure P1.4-2-5: Temperature as a function of radius predicted by the analytical and numerical models in the
limit that k is constant.

c.) What effect does the temperature dependent conductivity have on the calibration curve that
you generated in part (d) of Problem 1.3-9 (1-9)?

The quantity measured by the meter is the difference between the temperature at the center of the
pipe wall (T[26] when 51 nodes are used) and the fluid temperature:

DT=T[26]-T_f "temperature difference"

Figure P1.4-2-6 illustrates the calibration curve (i.e., the relationship between the temperature
difference and the mass flow rate) with and without the temperature dependent conductivity
included.
2
1.8
1.6
Mass flow rate (kg/s)

1.4
1.2 without temperature dependent conductivity
1 with temperature dependent conductivity
0.8
0.6
0.4
0.2
0
30 40 50 60 70 80 90 100 110 120
Temperature difference (K)
Figure P1.4-2-6: Calibration curve generated with and without the temperature dependent conductivity
included.
PROBLEM 1.5-1 (1-11 in text): Hay Temperature (revisited)
Reconsider Problem P1.3-8, but obtain a solution numerically using MATLAB. The description
of the hay bale is provided in Problem P1.3-8. Prepare a model that can consider the effect of
temperature on the volumetric generation. Increasing temperature tends to increase the rate of
reaction and therefore increase the rate of generation of thermal energy; the volumetric rate of
generation can be approximated by: g ′′′ = a + bT where a = -1 W/m3 and b = 0.01 W/m3-K.
Note that at T = 300 K, the generation is 2 W/m3 but that the generation increases with
temperature.
a.) Prepare a numerical model of the hay bale using EES. Plot the temperature as a function
of position within the hay bale.

The input information is entered in EES and a function is used to define the volumetric
generation:

$UnitSystem SI MASS RAD PA K J


$TABSTOPS 0.2 0.4 0.6 0.8 3.5 in

function gen(T)
"volumetric heat generation in wall"
"Input - T, temperature [K]"
"Output - gen, volumetric rate of heat generation [W/m^3]"

a=-1 [W/m^3] "coefficients in generation function"


b=0.01 [W/m^3-K]
gen=a+b*T

end

"Inputs"
L = 1 [m] "per unit length of bale"
R_bale= 5 [ft]*convert(ft,m) "bale radius"
t_p=0.045 [inch]*convert(inch,m) "plastic thickness"
k_p=0.15 [W/m-K] "plastic conductivity"
T_infinity=converttemp(C,K,20) "ambient temperature"
h=10 [W/m^2-K] "heat transfer coefficient"
k=0.04 [W/m-K] "hay conductivity"

Nodes are distributed uniformly throughout the computational domain (which consists only of
the hay, not the plastic), the location of each node (ri) is:

(i − 1)
ri = R i = 1..N (1)
( N − 1) bale
where N is the number of nodes used for the simulation. The distance between adjacent nodes
(Δr) is:

Rbale
Δr = (2)
( N − 1)
"Setup grid"
N=50 [-] "number of nodes"
duplicate i=1,N
r[i]=(i-1)*R_bale/(N-1) "position of each node"
end
Deltar=R_bale/(N-1) "distance between adjacent nodes"

A control volume is defined around each node and an energy balance is written for each control
volume. The control volume for an arbitrary, internal node (i.e., a node that is not placed on the
edge or at the center of the hay) experiences conduction heat transfer passing through the internal
surface ( q LHS ), conduction heat transfer passing through the external surface ( q RHS ), and heat
generation within the control volume ( g ). A steady-state energy balance for the control volume
is shown in Figure 1:

q LHS + q RHS + g = 0 (3)

Figure 1: Internal node energy balance

Each of the terms in the energy balance in Eq. (3) must be modeled using a rate equation.
Conduction through the inner surface is driven by the temperature difference between nodes i-1
and i through the material that lies between these nodes.

⎛ Δr ⎞
k 2 π ⎜ ri − ⎟ L
⎝ 2 ⎠
q LHS = (Ti −1 − Ti ) (4)
Δr

where L is the length of the bale (assumed to be 1 m, corresponding to doing the problem on a
per unit length of bale basis). The conduction into the outer surface of the control volume is:

⎛ Δr ⎞
k 2 π ⎜ ri + ⎟L
⎝ 2 ⎠
q RHS = (Ti +1 − Ti ) (5)
Δr

The generation is the product of the volume of the control volume and the volumetric generation
rate, which is approximately:

⎡⎛ Δr ⎞ ⎛
2
Δr ⎞ ⎤
2

g = g ′′′ (Ti ) π L ⎢⎜ ri + −
⎟ ⎜ ir − ⎟ ⎥ (6)
⎢⎣⎝ 2 ⎠ ⎝ 2 ⎠ ⎥⎦
where g ′′′ (Ti ) is the volumetric rate of generation evaluated at the nodal temperature Ti.
Substituting Eqs. (4) through (6) into Eq. (3) leads to:

⎛ Δr ⎞ ⎛ Δr ⎞
k 2 π ⎜ ri − ⎟ L k 2 π ⎜ ri + ⎟L
⎝ 2 ⎠ ⎝ 2 ⎠ ⎡⎛ Δr ⎞ ⎛
2
Δr ⎞ ⎤
2

( i −1 i )
T − T + ( i +1 i )
T − T + 
g ′′′ ( i ) ⎢⎜ i
T π L r + −
⎟ ⎜ ir − ⎟ ⎥ = 0 (7)
Δr Δr ⎢⎣⎝ 2 ⎠ ⎝ 2 ⎠ ⎥⎦
for i = 2...( N − 1)

Figure 2 illustrates the control volume associated with the node that is placed on the outer
surface of the hay (i.e., node N).

Figure 2: Control volume for node N located on hay outer surface

The energy balance for the control volume associated with node N is:

q LHS + g = qout (8)

where the conduction term is:

⎛ Δr ⎞
k 2 π ⎜ rN − ⎟ L
⎝ 2 ⎠
q LHS = (TN −1 − TN ) , (9)
Δr

the generation term is:

⎡ ⎛ Δr ⎞ ⎤
2

g = g ′′′ (TN ) π L ⎢ rN2 − ⎜ rN − ⎟ ⎥ , (10)


⎣⎢ ⎝ 2 ⎠ ⎦⎥

Note that the volume in Eq. (10) is calculated differently from the volume in Eq. (6) because the
control volume is half as wide radially. The heat transfer to the external air is:

qout =
(TN − T∞ ) (11)
R p + Rconv

where
thp
Rp = (12)
k p 2 π Rbale L

and

1
Rconv ,out = (13)
h 2 π Rbale L

Substituting Eqs. (9) through (11) into Eq. (8) leads to:

⎛ Δr ⎞
k 2 π ⎜ rN − ⎟ L
⎡ 2 ⎛ Δr ⎞ ⎤ (TN − T∞ )
2
⎝ 2 ⎠
( N −1 N )
T − T + 
g ′′′ ( N) ⎢ N ⎜ N
T π L r − r − ⎟ ⎥= (14)
Δr ⎢⎣ ⎝ 2 ⎠ ⎥⎦ R p + Rconv

A similar procedure applied to the control volume associated with node 1 leads to:

⎛ Δr ⎞
k 2 π ⎜ r1 + ⎟L Δr ⎞
2
⎝ 2 ⎠ ⎛
(T2 − T1 ) + g ′′′ (T1 ) π L ⎜ r1 + ⎟ =0 (15)
Δr ⎝ 2 ⎠

Equations (7), (14), and (15) represent N equations in an equal number of unknowns; the solution
of these equations provides the numerical solution.

"Internal control volumes"


duplicate i=2,(N-1)
k*2*pi*(r[i]-Deltar/2)*L*(T[i-1]-T[i])/Deltar+k*2*pi*(r[i]+Deltar/2)*L*(T[i+1]-
T[i])/Deltar+gen(T[i])*pi*L*((r[i]+Deltar/2)^2-(r[i]-Deltar/2)^2)=0
end

"node N"
R_p=t_p/(k_p*2*pi*R_bale*L) "conduction resistance of plastic"
R_conv=1/(h*2*pi*R_bale*L) "convection resistance"
k*2*pi*(r[N]-Deltar/2)*L*(T[N-1]-T[N])/Deltar+gen(T[N])*pi*L*(r[N]^2-(r[N]-Deltar/2)^2)=(T[N]-
T_infinity)/(R_p+R_conv)

"node 1"
k*2*pi*(r[1]+Deltar/2)*L*(T[2]-T[1])/Deltar+gen(T[1])*pi*L*(r[1]+Deltar/2)^2=0

If the EES program is solved then the temperature distribution will be placed in the Arrays
window. The temperature as a function of position is shown in Figure 3.
Figure 3: Temperature as a function of position within the bale

b.) Show that your model has numerically converged; that is, show some aspect of your solution
as a function of the number of nodes in your solution and discuss an appropriate number of
nodes to use.

The maximum temperature (i.e., the temperature at the center of the bale) is shown in Figure 4 as
a function of the number of nodes. The model is numerically converged after approximately N =
20.

Figure 4: Predicted maximum temperature as a function of the number of nodes


c.) Verify your numerical model by comparing your answer to an analytical solution in some,
appropriate limit. The result of this step should be a plot which shows the temperature as a
function of radius predicted by both your numerical solution and the analytical solution and
demonstrates that they agree.

The analytical solution derived in the problem 1.3-8 is used to compute the temperature at each
nodal position:

"Analytical solution from Problem 1.3-8"


a=-1 [W/m^3] "coefficients for volumetric generation function"
b=0.01 [W/m^3-K]
dTdr_Rbale = -BesselJ(1,(b/k)^(1/2)*R_bale)*(b/k)^(1/2)*C_2 "symbolic expressions from Maple"
T_Rbale = BesselJ(0,(b/k)^(1/2)*R_bale)*C_2-1/b*a
-k*2*pi*R_bale*L*dTdr_Rbale=(T_Rbale-T_infinity)/(R_p+R_conv)"interface energy balance"
duplicate i=1,N
T_an[i]=BesselJ(0,sqrt(b/k)*r[i])*C_2-a/b
end

Figure 3 illustrates the analytical solution overlaid on the numerical solution and demonstrates
agreement.

d.) Prepare a numerical model of the hay bale using MATLAB. Plot the temperature as a
function of position within the hay bale.

A new m-file is opened and formatted as a function with a single input (the number of nodes)
and two outputs (vectors containing the radial position and temperature at each node).

function[r,T]=P1p5_1(N)

L = 1; %per unit length of bale (m)


R_bale= 1.524; %bale radius (m)
t_p=0.00114; %plastic thickness (m)
k_p=0.15; %plastic conductivity (W/m-K)
T_infinity=293.2; %ambient temperature (K)
h=10; %heat transfer coefficient (W/m^2-K)
k=0.04; %hay conductivity (W/m-K)

end

A function is defined that returns the volumetric rate of generation as a function of temperature;
the function is placed at the bottom of the same m-file so that it is accessible locally to P1p5_1.

function[gv]=gen(T)

%coefficients of function
a=-1; %(W/m^3)
b=0.01; %(W/m^3-K)
gv=a+b*T;

end
The radial position of each node is stored in the vector r.

Deltar=R_bale/(N-1); %distance between adjacent nodes (m)


for i=1:N
r(i,1)=Deltar*(i-1); %radial location of each node (m)
end

The problem is nonlinear because the generation rate depends on temperature; therefore, the
method of successive substitution is used. An initial guess for the temperature distribution is
stored in the vector T_g:

%initial guess for temperature distribution


for i=1:N
T_g(i,1)=T_infinity;
end

The guess values for temperature are used to setup the matrix A and vector b which contain the
matrix formulation of the equations. The energy balance for node 1 is placed in row 1 of A.

⎛ Δr ⎞
k 2 π ⎜ r1 + ⎟L Δr ⎞
2
⎝ 2 ⎠ ⎛
(T2 − T1 ) + g ′′′ (T1* ) π L ⎜ r1 + ⎟ =0 (16)
Δr ⎝ 2 ⎠

where T1* is the guess value of the temperature or

⎡ ⎛ Δr ⎞ ⎤ ⎡ ⎛ Δr ⎞ ⎤
⎢ k 2 π ⎜ r1 + 2 ⎟ L ⎥ ⎢ k 2 π ⎜ r1 + 2 ⎟ L ⎥
⎠ ⎥ = − g ′′′ T * π L ⎛ r + Δr ⎞
2

T1 ⎢ − ⎝
Δr
⎠ ⎥ +T ⎢
2

Δr
( 1 ) ⎜⎝ 1 2 ⎟⎠ (17)
⎢ ⎥ ⎢ ⎥ 

⎢⎣ ⎥⎦ ⎢⎣ ⎥⎦ b (1)



A (1,1) A (1,2)

The energy balances for the internal nodes are:

⎛ Δr ⎞ ⎛ Δr ⎞
k 2 π ⎜ ri − ⎟ L k 2 π ⎜ ri + ⎟L
⎝ 2 ⎠ ⎝ 2 ⎠ ⎡⎛ Δr ⎞ ⎛
2
Δr ⎞ ⎤
2

(Ti −1 − Ti ) + (Ti +1 − Ti ) + g ′′′ (Ti ) π L ⎢⎜ ri + ⎟ − ⎜ ri − ⎟ ⎥ = 0 (18)


*

Δr Δr ⎢⎣⎝ 2 ⎠ ⎝ 2 ⎠ ⎥⎦
for i = 2...( N − 1)

or
⎡ ⎛ Δr ⎞ ⎛ Δr ⎞ ⎤ ⎡ ⎛ Δr ⎞ ⎤
⎢ k 2 π ⎜ ri − 2 ⎟ L k 2 π ⎜ ri + 2 ⎟ L ⎥ ⎢ k 2 π ⎜ ri − 2 ⎟ L ⎥
Ti ⎢ − ⎝ ⎠ − ⎝ ⎠ ⎥ +T ⎢ ⎝ ⎠ ⎥+
i −1
⎢ Δr Δr ⎥ ⎢ Δr ⎥
⎢⎣ ⎥⎦ ⎢⎣ ⎥



A ( i ,i ) A ( i ,i −1)

⎡ ⎛ Δr ⎞ ⎤
⎢ k 2 π ⎜ ri + 2 ⎟ L ⎥ ⎡⎛ Δr ⎞ ⎛
2
Δr ⎞ ⎤
2

⎢ ⎝ ⎠ ⎥ = − g (Ti ) π L ⎢⎜ ri
′′′ +
 ⎟ − ⎜ ri − ⎟ ⎥
*
Ti +1 (19)
⎢ Δr ⎥ ⎢⎝
⎣ 2 ⎠ ⎝ 2 ⎠ ⎦⎥
⎢⎣ ⎥⎦ 


b (i )
A ( i ,i +1)

for i = 2 .. ( N − 1)

The energy balance for node N is:

⎛ Δr ⎞
k 2 π ⎜ rN − ⎟ L
⎡ 2 ⎛ Δr ⎞ ⎤ (TN − T∞ )
2
⎝ 2 ⎠
(TN −1 − TN ) + g ′′′ (TN ) π L ⎢ rN − ⎜ rN − ⎟ ⎥ =
*
(20)
Δr ⎢⎣ ⎝ 2 ⎠ ⎥⎦ R p + Rconv

or

⎡ ⎛ Δr ⎞ ⎤ ⎡ ⎛ Δr ⎞ ⎤
⎢ k 2 π ⎜ rN − 2 ⎟ L ⎥ ⎢ k 2 π ⎜ rN − ⎟ L ⎥
TN ⎢ − ⎝ ⎠ − 1
⎥ + TN −1 ⎢ ⎝ 2 ⎠
⎥=
⎢ Δr R p + Rconv ⎥ ⎢ Δr ⎥
⎢⎣ ⎥⎦ ⎢⎣ ⎥


⎦ (21)
A( N , N ) A ( N , N −1)

⎡ ⎛ Δr ⎞ ⎤
2

− g ′′′ (TN* ) π L ⎢ rN2 − ⎜ rN − ⎟ ⎥ −


T∞
⎢⎣ ⎝ 2 ⎠ ⎥⎦ R p + Rconv


b( N )

The matrices A and b are initialized and the resistances due to convection and conduction
through the plastic are computed:

A=spalloc(N,N,3*N);
b=zeros(N,1);
R_p=t_p/(k_p*2*pi*R_bale*L); %resistance through plastic
R_conv=1/(h*2*pi*R_bale*L); %resistance due to convection

The matrices A and b are filled in according to Eqs. (17), (19), and (21):

%Node 1
A(1,1)=-k*2*pi*(r(1)+Deltar/2)*L/Deltar;
A(1,2)=k*2*pi*(r(1)+Deltar/2)*L/Deltar;
b(1)=-gen(T_g(i))*pi*L*(r(1)+Deltar/2)^2;
%Nodes 2 to (N-1)
for i=2:(N-1)
A(i,i)=-k*2*pi*(r(i)-Deltar/2)*L/Deltar-
k*2*pi*(r(i)+Deltar/2)*L/Deltar;
A(i,i-1)=k*2*pi*(r(i)-Deltar/2)*L/Deltar;
A(i,i+1)=k*2*pi*(r(i)+Deltar/2)*L/Deltar;
b(i)=-gen(T_g(i))*pi*L*((r(i)+Deltar/2)^2-(r(i)-Deltar/2)^2);
end

%Node N
A(N,N)=-k*2*pi*(r(N)-Deltar/2)*L/Deltar-1/(R_p+R_conv);
A(N,N-1)=k*2*pi*(r(N)-Deltar/2)*L/Deltar;
b(N)=-gen(T_g(N))*pi*L*(r(N)^2-(r(N)-Deltar/2)^2)-
T_infinity/(R_p+R_conv);

The temperature distribution is obtained according to:

T=A\b;

The successive substitution process occurs within a while loop that is terminated when some
convergence error, err, goes below a tolerance, tol. The tolerance is set and the error is
initialized to a value that will ensure that the loop executes at least once. Once the solution is
obtained, it is compared with the guess value to determine an error. The guess values are reset
and, if the error is not sufficiently small then the process is repeated. The code is shown below;
the new lines are shown in bold:

function[r,T]=P1p5_1(N)

L = 1; %per unit length of bale (m)


R_bale= 1.524; %bale radius (m)
t_p=0.00114; %plastic thickness (m)
k_p=0.15; %plastic conductivity (W/m-K)
T_infinity=293.2; %ambient temperature (K)
h=10; %heat transfer coefficient (W/m^2-K)
k=0.04; %hay conductivity (W/m-K)

Deltar=R_bale/(N-1); %distance between adjacent nodes (m)


for i=1:N
r(i,1)=Deltar*(i-1); %radial location of each node (m)
end

%initial guess for temperature distribution


for i=1:N
T_g(i,1)=T_infinity;
end

A=spalloc(N,N,3*N);
b=zeros(N,1);
R_p=t_p/(k_p*2*pi*R_bale*L); %resistance through plastic
R_conv=1/(h*2*pi*R_bale*L); %resistance due to convection

tol=0.1; %tolerance for convergence (K)


err=2*tol; %error initialization
while(err>tol)
%Node 1
A(1,1)=-k*2*pi*(r(1)+Deltar/2)*L/Deltar;
A(1,2)=k*2*pi*(r(1)+Deltar/2)*L/Deltar;
b(1)=-gen(T_g(i))*pi*L*(r(1)+Deltar/2)^2;

%Nodes 2 to (N-1)
for i=2:(N-1)
A(i,i)=-k*2*pi*(r(i)-Deltar/2)*L/Deltar-
k*2*pi*(r(i)+Deltar/2)*L/Deltar;
A(i,i-1)=k*2*pi*(r(i)-Deltar/2)*L/Deltar;
A(i,i+1)=k*2*pi*(r(i)+Deltar/2)*L/Deltar;
b(i)=-gen(T_g(i))*pi*L*((r(i)+Deltar/2)^2-(r(i)-Deltar/2)^2);
end

%Node N
A(N,N)=-k*2*pi*(r(N)-Deltar/2)*L/Deltar-1/(R_p+R_conv);
A(N,N-1)=k*2*pi*(r(N)-Deltar/2)*L/Deltar;
b(N)=-gen(T_g(N))*pi*L*(r(N)^2-(r(N)-Deltar/2)^2)-
T_infinity/(R_p+R_conv);

T=A\b; %obtain temperature distribution


err=sum(abs(T-T_g))/N %calculate error
T_g=T;
end

end

function[gv]=gen(T)

%coefficients of function
a=-1; %(W/m^3)
b=0.01; %(W/m^3-K)
gv=a+b*T;
end

The temperature as a function of radius is shown in Figure 5.


Figure 5: Predicted temperature as a function of radial position
Problem 1.5-2 (1-12 in text): Mass Flow Meter (re-visited)
Reconsider the mass flow meter that was investigated in Problem 1.3-9 (1-9 in text). Assume
that the conductivity of the material that is used to make the test section is not actually constant
as was assumed in Problem 1.3-9 (1-9 in text) but rather depends on temperature according to:

⎡ W ⎤
2 (
T − 300 [ K ])
W
k = 10 + 0.035 ⎢
m-K ⎣ m-K ⎥⎦

a.) Develop a numerical model of the mass flow meter using MATLAB. Plot the temperature as
a function of radial position for the conditions shown in Figure P1.3-9 (P1-9 in text) with the
temperature-dependent conductivity.

The inputs are entered in a MATLAB function that requires as an input the number of nodes (N):

function[r,T_C]=P1p5_2(N)

r_out=0.0254; %outer radius of test section (m)


r_in=0.01905; %inner radius of test section (m)
h_bar_out=10; %external convection coefficient (W/m^2-K)
T_infinity=293.2; %air temperature (K)
T_f=291.2; %fluid temperature (K)
gv=1e7; %rate of generation (W/m^3)
m_dot=0.75; %mass flow rate (kg/s)
th_ins=0.00635; %thickness of the insulation (m)
k_ins=1.5; %insulation conductivity (W/m-K)
L=0.0762; %length of the test section (m)
C=2500; %constant for convection relationship

The convection coefficient on the internal surface is computed:

h_bar_in=C*m_dot^0.8; %internal convection coefficient

A function is defined that returns the conductivity of the material:

function[k]=k_t(T)
%conductivity of the material
%
%Inputs:
% T: temperature (K)
%
%Outputs:
% k: conductivity (W/m-K)

k=10+0.035*(T-300);
end

A uniform distribution of nodes is used, the radial location of each node (ri) is:
( i − 1) r − r
ri = rin + ( ) for i = 1..N (1)
( N − 1) out in

where N is the number of nodes. The radial distance between adjacent nodes (Δr) is:

Δr =
( rout − rin ) (2)
( N − 1)
DELTAr=(r_out-r_in)/(N-1); %distance between adjacent nodes (m)
for i=1:N
r(i)=r_in+(r_out-r_in)*(i-1)/(N-1); %position of each node (m)
end

The system of equations is placed in matrix format.

AX =b (3)

The most logical technique for ordering the unknown temperatures in the vector X is:

⎡ X 1 = T1 ⎤
⎢ X =T ⎥
X =⎢ 2 2 ⎥
(4)
⎢ ... ⎥
⎢ ⎥
⎣ X N = TN ⎦

Equation (4) shows that the unknown temperature at node i (i.e., Ti) corresponds to element i of
vector X (i.e., Xi). The most logical technique for placing the equations into the A matrix is:

⎡ row 1 = control volume 1 equation ⎤


⎢ row 2 = control volume 2 equation ⎥
A=⎢ ⎥ (5)
⎢ ... ⎥
⎢ ⎥
⎣ row N = control volume N equation ⎦

In Eq. (5), the equation for control volume i is placed into row i.

An energy balance is carried out on a control volume surrounding each node. For node 1, placed
at the inner surface (Figure P1.5-2-1):

qconv ,in + qouter + g = 0 (6)


qouter T2
g1
T1
qconv ,in
Figure P1.5-2-1: Control volume around node 1.

The rate equation for convection is:

qconv ,in = hin 2 π rin L (T f − T1 ) (7)

The rate equation for conduction is:

⎛ Δr ⎞ (T2 − T1 )
qouter = kT =(T1 +T2 ) / 2 2 π ⎜ rin + ⎟L (8)
⎝ 2 ⎠ Δr

The rate equation for generation is:

Δr
g = 2 π rin L g ′′′ (9)
2

Substituting Eqs. (7) through (9) into Eq. (6) leads to:

⎛ Δr ⎞ (T2 − T1 )
hin 2 π rin L (T f − T1 ) + kT =(T1 +T2 ) / 2 2 π ⎜ rin + ⎟L + π rin Δr L g ′′′ = 0 (10)
⎝ 2 ⎠ Δr

Equation (10) is rearranged to identify the coefficients that multiply each unknown temperature:

⎡ ⎛ Δr ⎞ L ⎤ ⎡ ⎛ Δr ⎞ L ⎤
T1 ⎢ − hin 2 π rin L − kT =(T1 +T2 ) / 2 2 π ⎜ rin + ⎟ ⎥ + T2 ⎢ kT =(T1 +T2 ) / 2 2 π ⎜ rin + ⎟ =
⎣ ⎝ 2 ⎠ Δr ⎦ ⎣ ⎝ 2 ⎠ Δr ⎥⎦ (11)
−π rin Δr L g ′′′ − hin 2 π rin LT f

An energy balance on an internal node is shown in Figure P1.5-2-2:

qinner + qouter + g = 0 (12)

qouter Ti+1
g Ti
qinner Ti-1
Figure P1.5-2-2: Control volume around internal node i.
The rate equations for conduction are:

⎛ Δr ⎞ (Ti +1 − Ti )
qouter = kT =(Ti +Ti+1 ) / 2 2 π ⎜ rin + ⎟L (13)
⎝ 2 ⎠ Δr

⎛ Δr ⎞ ( T − T )
qinner = kT =(Ti +Ti−1 ) / 2 2 π ⎜ rin − ⎟ L i −1 i (14)
⎝ 2 ⎠ Δr

The rate equation for generation is:

g = 2 π ri Δr L g ′′′ (15)

Substituting Eqs. (13) through (15) into Eq. (12) for all of the internal nodes leads to:

⎛ Δr ⎞ (T − T ) ⎛ Δr ⎞ (Ti +1 − Ti )
kT =(Ti +Ti−1 ) / 2 2 π ⎜ rin − ⎟ L i −1 i + kT =(Ti +Ti+1 ) / 2 2 π ⎜ rin + ⎟L
⎝ 2 ⎠ Δr ⎝ 2 ⎠ Δr (16)
+2 π ri Δr L g ′′′ = 0 for i = 2.. ( N − 1)

Equation (16) is rearranged to identify the coefficients that multiply each unknown temperature:

⎡ ⎛ Δr ⎞ L ⎛ Δr ⎞ L ⎤
Ti ⎢ − kT =(Ti +Ti−1 ) / 2 2 π ⎜ rin − ⎟ − kT =(Ti +Ti+1 ) / 2 2 π ⎜ rin + ⎟
⎣ ⎝ 2 ⎠ Δr ⎝ 2 ⎠ Δr ⎥⎦
⎡ ⎛ Δr ⎞ L ⎤ ⎡ ⎛ Δr ⎞ L ⎤
+Ti −1 ⎢ −kT =(Ti +Ti−1 ) / 2 2 π ⎜ rin − ⎟ ⎥ + Ti +1 ⎢ − kT =(Ti +Ti+1 ) / 2 2 π ⎜ rin + ⎟ (17)
⎣ ⎝ 2 ⎠ Δr ⎦ ⎣ ⎝ 2 ⎠ Δr ⎦⎥
= −2 π ri Δr L g ′′′ for i = 2.. ( N − 1)

An energy balance on node N placed on the outer surface is shown in Figure P1.5-2-3:

qinner + qair + g = 0 (18)

qair
TN
g
qinner TN-1
Figure P1.5-2-3: Control volume around internal node N.

The rate equation for the heat transfer with the air is:

qair =
(T∞ − TN ) (19)
(R ins + Rconv ,out )
where

⎡ ( r + thins ) ⎤
ln ⎢ out ⎥
Rins = ⎣ rout ⎦ (20)
2 π L kins

1
Rconv ,out = (21)
2 π ( rout + thins ) L hout

R_ins=log((r_out+th_ins)/r_out)/(2*pi*L*k_ins);
%resistance to conduction through insulation
R_conv_out=1/(2*pi*(r_out+th_ins)*L*h_bar_out);
%resistance to convection from the outside surface of the insulation

The rate equation for conduction is:

⎛ Δr ⎞ (T − T )
qinner = kT =(TN +TN −1 ) / 2 2 π ⎜ rout − ⎟ L N −1 N (22)
⎝ 2 ⎠ Δr

The rate equation for generation is:

Δr
g = 2 π rout L g ′′′ (23)
2

Substituting Eqs. (19), (22), and (23) into Eq. (18) leads to:

⎛ Δr ⎞ (T − T ) (T∞ − TN ) + 2 π r Δr L g ′′′ = 0
kT =(TN +TN −1 ) / 2 2 π ⎜ rout − ⎟ L N −1 N + (24)
⎝ 2 ⎠ Δr ( Rins + Rconv,out ) out
2

Equation (24) is rearranged to identify the coefficients that multiply each unknown temperature:

⎡ ⎛ Δr ⎞ L 1 ⎤
TN ⎢ − kT =(TN +TN −1 ) / 2 2 π ⎜ rout − ⎟ − ⎥
⎢⎣ ⎝ 2 ⎠ Δr ( Rins + Rconv ,out ) ⎥⎦
⎡ ⎛ Δr ⎞ L ⎤
+TN −1 ⎢ − kT =(TN +TN −1 ) / 2 2 π ⎜ rout − ⎟ ⎥ (25)
⎣ ⎝ 2 ⎠ Δr ⎦
T∞
= − π rout Δr L g ′′′ −
( Rins + Rconv,out )
Equations (11), (17), and (25) are N equations in the N unknown temperatures. Because they are
non-linear, they must be linearized and a successive substitution method used. A guess
temperature distribution ( Tˆi ) is assumed:

%initial guess for temperature distribution


for i=1:N
Tg(i,1)=T_f;
end

The matrix A is defined as a sparse matrix with at most 3N nonzero entries:

%initialize A and b
A=spalloc(N,N,3*N);
b=zeros(N,1);

The solution is placed within a while loop that terminates when the error between the solution
and the guess is less than some tolerance:

err=999; %initial value for error (K), must be larger than tol
tol=0.01; %tolerance for convergence (K)
while(err>tol)

The equation for node 1, Eq. (11), is linearized by using the guess temperature distribution to
compute the conductivity:

⎡ ⎛ Δr ⎞ L ⎤ ⎡ ⎛ Δr ⎞ L ⎤
T1 ⎢ − hin 2 π rin L − kT = Tˆ +Tˆ / 2 2 π ⎜ rin + ⎟ ⎥ + T2 ⎢ kT =(Tˆ1 +Tˆ2 ) / 2 2 π ⎜ rin + ⎟ =
⎣ ( 1 2)
⎝ 2 ⎠ Δr ⎦ ⎣ ⎝ 2 ⎠ Δr ⎥⎦



A1,1 A1,2 (26)


−π rin Δr L g ′′′ − hin 2 π rin LT f


b1

A(1,1)=-h_bar_in*2*pi*r_in*L-...
k_t((Tg(1)+Tg(2))/2)*2*pi*(r_in+DELTAr/2)*L/DELTAr;
A(1,2)=k_t((Tg(1)+Tg(2))/2)*2*pi*(r_in+DELTAr/2)*L/DELTAr;
b(1)=-pi*r_in*DELTAr*L*gv-h_bar_in*2*pi*r_in*L*T_f;

The equations for the internal nodes, Eq. (17), is also linearized:
⎡ ⎛ Δr ⎞ L ⎛ Δr ⎞ L ⎤
Ti ⎢ − kT =(Ti +Ti−1 ) / 2 2 π ⎜ rin − ⎟ − kT =(Ti +Ti+1 ) / 2 2 π ⎜ rin + ⎟
⎣ ⎝ 2 ⎠ Δr ⎝ 2 ⎠ Δr ⎦⎥

Ai ,i

⎡ ⎛ Δr ⎞ L ⎤ ⎡ ⎛ Δr ⎞ L ⎤
+Ti −1 ⎢ −kT =(Ti +Ti−1 ) / 2 2 π ⎜ rin − ⎟ ⎥ + Ti +1 ⎢ − kT =(Ti +Ti+1 ) / 2 2 π ⎜ rin + ⎟ (27)
⎣ ⎝ 2 ⎠ Δr ⎦ ⎣ ⎝ 2 ⎠ Δr ⎥⎦


Ai ,i −1 Ai ,i +1

= −2 π ri Δr L g ′′′ for i = 2.. ( N − 1)




bi

for i=2:(N-1)
A(i,i)=-k_t((Tg(i)+Tg(i-1))/2)*2*pi*(r(i)-DELTAr/2)*L/DELTAr...
-k_t((Tg(i)+Tg(i+1))/2)*2*pi*(r(i)+DELTAr/2)*L/DELTAr;
A(i,i-1)=k_t((Tg(i)+Tg(i-1))/2)*2*pi*(r(i)-DELTAr/2)*L/DELTAr;
A(i,i+1)=k_t((Tg(i)+Tg(i+1))/2)*2*pi*(r(i)+DELTAr/2)*L/DELTAr;
b(i)=-2*pi*r(i)*DELTAr*L*gv;
end

The equation for node N, Eq. (25), is linearized:

⎡ ⎛ Δr ⎞ L 1 ⎤
TN ⎢ − kT =(TN +TN −1 ) / 2 2 π ⎜ rout − ⎟ − ⎥
⎣⎢ ⎝ 2 ⎠ Δr ( Rins + Rconv ,out ) ⎦⎥

AN , N

⎡ ⎛ Δr ⎞ L ⎤
+TN −1 ⎢ − kT =(TN +TN −1 ) / 2 2 π ⎜ rout − ⎟ ⎥ (28)
⎣ ⎝ 2 ⎠ Δr ⎦

AN , N −1

T∞
= − π rout Δr L g ′′′ −
( Rins + Rconv,out )


bN

A(N,N)=-k_t((Tg(N)+Tg(N-1))/2)*2*pi*(r_in-DELTAr/2)*L/DELTAr-...
1/(R_ins+R_conv_out);
A(N,N-1)=k_t((Tg(N)+Tg(N-1))/2)*2*pi*(r_in-DELTAr/2)*L/DELTAr;
b(N)=-pi*r_out*DELTAr*L*gv-T_infinity/(R_ins+R_conv_out);

The solution is obtained:

X=A\b;
T=X;

and used to compute the error between the assumed and calculated solutions is obtained:

∑ (T − Tˆ )
N
1 2
err = i i (29)
N i =1
err=sqrt(sum((T-Tg).^2)/N) %compute rms error

The calculated solution becomes the guess value for the next iteration:

Tg=T; %reset guess values used to setup A and b


end

The solution is converted to degrees Celsius:

T_C=T-273.2; %convert to C
end

The solution is illustrated in Figure P1.5-2-4.


75

72.5

70
Temperature (°C)

67.5

65

62.5

60

57.5

55

52.5
0.019 0.02 0.021 0.022 0.023 0.024 0.025 0.026
Radius (m)
Figure P1.5-2-4: Temperature as a function of radius.

b.) Verify that your numerical solution limits to the analytical solution from Problem 1.3-9 (1-9
in text) in the limit that the conductivity is constant.

The conductivity function is modified temporarily so that it returns a constant value:

function[k]=k_t(T)
%conductivity of the material
%
%Inputs:
% T: temperature (K)
%
%Outputs:
% k: conductivity (W/m-K)

k=10;%+0.035*(T-300);
end

Figure P1.5-2-5 illustrates the temperature distribution predicted by the numerical and analytical
solutions in the limit that k is constant.
80

75

Temperature (°C)
70

65

60 analytical model
numerical model
55

50
0.019 0.02 0.021 0.022 0.023 0.024 0.025 0.026
Radius (m)
Figure P1.5-2-5: Temperature as a function of radius predicted by the analytical and numerical models in the
limit that k is constant.
Problem 1.6-1 (1-13 in text): Temperature Sensor Error
A resistance temperature detector (RTD) utilizes a material that has a resistivity that is a strong
function of temperature. The temperature of the RTD is inferred by measuring its electrical
resistance. Figure P1.6-1 shows an RTD that is mounted at the end of a metal rod and inserted
into a pipe in order to measure the temperature of a flowing liquid. The RTD is monitored by
passing a known current through it and measuring the voltage across it. This process results in a
constant amount of ohmic heating that may tend to cause the RTD temperature to rise relative to
the temperature of the surrounding liquid; this effect is referred to as a self-heating error. Also,
conduction from the wall of the pipe to the temperature sensor through the metal rod can also
result in a temperature difference between the RTD and the liquid; this effect is referred to as a
mounting error.

Tw = 20°C pipe
L = 5.0 cm

D = 0.5 mm
x
h = 150 W/m -K
2
k = 10 W/m-K
T∞ = 5.0°C qsh = 2.5 mW
RTD

Figure P1.6-1: Temperature sensor mounted in a flowing liquid.

The thermal energy generation associated with ohmic heating is q sh = 2.5 mW. All of this ohmic
heating is assumed to be transferred from the RTD into the end of the rod at x = L. The rod has a
thermal conductivity k = 10 W/m-K, diameter D = 0.5 mm, and length L = 5 cm. The end of the
rod that is connected to the pipe wall (at x = 0) is maintained at a temperature of Tw = 20°C. The
liquid is at a uniform temperature, T∞ = 5°C and the heat transfer coefficient between the liquid
and the rod is h = 150 W/m2-K.
a.) Is it appropriate to treat the rod as an extended surface (i.e., can we assume that the
temperature in the rod is a function only of x)? Justify your answer.

The input parameters are entered in EES.

$UnitSystem SI MASS RAD PA K J


$TABSTOPS 0.2 0.4 0.6 0.8 3.5 in

"Inputs"
q_dot_sh=0.0025 [W] "self-heating power"
k=10 [W/m-K] "conductivity of mounting rod"
d=0.5 [mm]*convert(mm,m) "diameter of mounting rod"
L=5.0 [cm]*convert(cm,m) "length of mounting rod"
T_wall=convertTemp(C,K,20) "temperature of wall"
T_f=convertTemp(C,K,5) "temperature of liquid"
h=150 [W/m^2-K] “heat transfer coefficient”
The appropriate Biot number for this case is:

hd
Bi = (1)
2k

"Extended surface approximation"


Bi=h*d/(2*k)

The Biot number calculated by EES is 0.004 which is much less than 1.0 and therefore the
extended surface approximation is justified.

b.) Develop an analytical model of the rod that will predict the temperature distribution in the
rod and therefore the error in the temperature measurement; this error is the difference
between the temperature at the tip of the rod and the liquid. You may find it easiest to use
Maple for this process.

Figure 2 illustrates a differential control volume for the rod.

Figure 2: Differential control volume for the rod.

The energy balance suggested by Figure 2 is:

q x = q x + dx + qconv (2)

or, expanding the x+dx term:

dq
q x = q x + dx + qconv (3)
dx

The rate equations for conduction and convection are:

d 2 dT
q x = − k π (4)
4 dx

and

qconv = h π d dx (T − T f ) (5)
Substituting Eqs. (4) and (5) into Eq. (3) leads to:

d ⎡ d 2 dT ⎤
0= ⎢ − k π ⎥ dx + h π d dx (T − T f ) (6)
dx ⎣ 4 dx ⎦

or

d 2T 4 h

dx 2 k d
(T − T f ) = 0 (7)

which is a non-homogeneous 2nd order differential equation. The general solution to Eq. (7) can
be found in your text as Eq. (3.66) or in the handout on Extended Surfaces as Eq. (6-22). The
easiest thing to do is enter the differential equation into Maple and let it solve it for you:

> GDE:=diff(diff(T(x),x),x)-4*h*(T(x)-T_f)/(k*d)=0;
⎛d ⎞ 4 h ( T( x ) − T_f )
2
GDE := ⎜⎜ 2 T( x ) ⎟⎟ − =0
⎝ dx ⎠ kd
> Ts:=dsolve(GDE);
⎛⎜ 2 h x ⎞⎟ ⎛⎜ 2 h x ⎞⎟
⎜ k d ⎟ ⎜− k d ⎟
⎝ ⎠ ⎝ ⎠
Ts := T( x ) = e _C2 + e _C1 + T_f

The solution can be copied and pasted into EES (don’t forget that you may need to change your
output display to Maple Notation to facilitate the copying process depending on your version of
Maple):

> Ts:=dsolve(GDE);
Ts := T(x) = exp(2*h^(1/2)*x/(k^(1/2)*d^(1/2)))*_C2+exp(-
2*h^(1/2)*x/(k^(1/2)*d^(1/2)))*_C1+T_f

which can be copied to EES:

Ts := T(x) = exp(2*h^(1/2)*x/(k^(1/2)*d^(1/2)))*_C2+exp(-2*h^(1/2)*x/(k^(1/2)*d^(1/2)))*_C1+T_f
"solution copied from Maple"

The solution will need to be modified slightly so that it is compatible with EES (the _C1 must
become C1, _C2 must be C2, Ts:= should be deleted and the T(x) must be just T:

T = exp(2*h^(1/2)*x/(k^(1/2)*d^(1/2)))*C2+exp(-2*h^(1/2)*x/(k^(1/2)*d^(1/2)))*C1+T_f
"solution copied from Maple and modified"

Trying to solve now should give the message that you have 12 variables but only 9 equations –
you need to specify C1, C2, and x to have a completely specified problem. Let’s set x = 0:

x=0
and concentrate on determining symbolic expressions for the boundary conditions. The
temperature at the pipe wall (x=0) is specified to be Twall. Using Maple:

> rhs(eval(Ts,x=0))=T_wall;
_C2+_C1+T_f = T_wall

which can be pasted into EES (and modified):

C2+C1+T_f = T_wall "wall boundary condition"

The boundary condition at the end of the rod with the sensor is associated with an energy balance
on the interface:

d 2 dT
kπ = qsh (8)
4 dx x= L

which can be evaluated symbolically in Maple:

> k*pi*d^2*rhs(eval(diff(Ts,x),x=L))/4=q_dot_sh;
1/4*k*pi*d^2*(2*h^(1/2)*exp(2*h^(1/2)*L/(k^(1/2)*d^(1/2)))*_C2/(k^(1/2)*d^(1/
2))-2*h^(1/2)*exp(-2*h^(1/2)*L/(k^(1/2)*d^(1/2)))*_C1/(k^(1/2)*d^(1/2))) =
q_dot_sh

Aren’t you glad you don’t have to do this by hand? The expression can be copied and pasted
into EES to complete your solution:

1/4*k*pi*d^2*(2*h^(1/2)*exp(2*h^(1/2)*L/(k^(1/2)*d^(1/2)))*C2/(k^(1/2)*d^(1/2))-2*h^(1/2)*exp(-
2*h^(1/2)*L/(k^(1/2)*d^(1/2)))*C1/(k^(1/2)*d^(1/2))) = q_dot_sh "sensor boundary condition"

Check your units (Figure 3 shows the variable information window with the units set) to make
sure that no errors were made.
Figure 3: Variable Information window.

c.) Prepare a plot of the temperature as a function of position and compute the temperature error.

Comment out the specification that x=0 and prepare a parametric table that includes T and x.
Alter x so that it varies from 0 to 0.05 and plot the result. You can convert the temperature to °C
and position to cm for a better looking plot:

x_cm=x*convert(m,cm)
T_C=converttemp(K,C,T)

Figure 4 illustrates the temperature distribution; note that the temperature elevation at the tip
with respect to the fluid is about 3.6 K and it represents the measurement error. For the
conditions in the problem statement, it is clear that the measurement error is primarily due to the
self-heating effect because the effect of the wall (the temperature elevation at the base) has died
off after about 2.0 cm.
Figure 4: Temperature distribution in the mounting rod.

d.) Investigate the effect of thermal conductivity on the temperature measurement error. Identify
the optimal thermal conductivity and explain why an optimal thermal conductivity exists.

The temperature measurement error can be calculated from your solution by setting x = L:

"Part d - temperature measurement error"


x=L
errT=T-T_f

Figure 5 illustrates the temperature measurement error as a function of the thermal conductivity
of the rod material. Figure 5 shows that the optimal thermal conductivity, corresponding to the
minimum measurement error, is around 100 W/m-K. Below the optimal value, the self-heating
error dominates as the local temperature rise at the tip of the rod is large. Above the optimal
value, the conduction from the wall dominates. The inset figures show the temperature
distribution for high and low thermal conductivity in order to illustrate these different behaviors.
Figure 5: Temperature measurement error as a function of rod thermal conductivity. The inset figures show
the temperature distribution at low conductivity and high conductivity.
Problem 1.6-2 (1-14 in text): Optimizing a Heat Sink
Your company has developed a micro-end milling process that allows you to easily fabricate an
array of very small fins in order to make heat sinks for various types of electrical equipment.
The end milling process removes material in order to generate the array of fins. Your initial
design is the array of pin fins shown in Figure P1.6-2. You have been asked to optimize the
design of the fin array for a particular application where the base temperature is Tbase = 120°C
and the air temperature is Tair = 20°C. The heat sink is square; the size of the heat sink is W = 10
cm. The conductivity of the material is k = 70 W/m-K. The distance between the edges of two
adjacent fins is a, the diameter of a fin is D, and the length of each fin is L.

Tair = 20°C, h
array of fins D
k = 70 W/m-K
a
L

W = 10 cm
Tbase = 120°C

Figure P1.6-2: Pin fin array

Air is forced to flow through the heat sink by a fan. The heat transfer coefficient between the air
and the surface of the fins as well as the unfinned region of the base, h , has been measured for
the particular fan that you plan to use and can be calculated according to:

0.4 −0.3
⎡ W ⎤⎛ a ⎞ ⎛ D ⎞
h = 40 ⎢ 2 ⎥ ⎜⎜ ⎟ ⎜⎜ ⎟⎟
⎣ m K ⎦ ⎝ 0.005 [ m ] ⎟⎠ ⎝ 0.01 [ m ] ⎠

Mass is not a concern for this heat sink; you are only interested in maximizing the heat transfer
rate from the heat sink to the air given the operating temperatures. Therefore, you will want to
make the fins as long as possible. However, in order to use the micro-end milling process you
cannot allow the fins to be longer than 10x the distance between two adjacent fins. That is, the
length of the fins may be computed according to: L = 10 a . You must choose the most optimal
value of a and D for this application.
a.) Prepare a model using EES that can predict the heat transfer coefficient for a given value of a
and D. Use this model to predict the heat transfer rate from the heat sink for a = 0.5 cm and
D = 0.75 cm.

The input values are entered in EES:

$UnitSystem SI MASS RAD PA K J


$TABSTOPS 0.2 0.4 0.6 0.8 3.5 in

"Inputs"
T_air=converttemp(C,K,20) "air temperature"
T_base=converttemp(C,K,120) "base temperature"
k=70 [W/m-K] "fin material conductivity"
W=10.0 [cm]*convert(cm,m) "base width"

The optimization parameters, a and D, are set to their initial values:

"Optimization parameters"
a=0.5 [cm]*convert(cm,m) "distance between adjacent fins"
D=0.75 [cm]*convert(cm,m) "diameter of fins"

The length of the fins is computed using the aspect ratio and the number of fins is determined
according to:

2
⎛ W ⎞
N =⎜ ⎟ (1)
⎝a+D⎠

The heat transfer coefficient is computed using the equation provided in the problem statement.

L=10*a "length of fins"


N=(W/(a+D))^2 "number of fins"
h = 40 [W/m^2-K]*(a/0.005 [m])^(0.4)*(D/0.01 [m])^(-0.3) "heat transfer coefficient"

The perimeter and cross-sectional area of each fin are computed according to:

p =π D (2)

D2
Ac = π (3)
4

The EES function for the fin efficiency of a constant cross-sectional area fin is used. The
function is accessed using the Function Information selection from the Options menu and then
selecting Fin Efficiency from the pull-down menu. Scroll to the Circular-Base Rectangular Fin
(Figure 2(a)) and select Info to learn how to access this function (Figure 2(b)).
(a) (b)
Figure 2: (a) Function Information window and (b) Help information for the Circular-Base Rectangular Fin.

The fin constant, mL, is computed according to:

hp
mL = L (4)
k Ac

and used to call the function eta_fin_spine_rect which returns the fin efficiency, ηf.

p=pi*D "perimeter of fin"


Ac=pi*D^2/4 "cross sectional area of fin"
mL=L*sqrt(h*p/(k*Ac)) "fin constant"
eta=eta_fin_spine_rect(mL) "fin efficiency"

The total area of the fins on the heat sink is:

Af = N p L (5)

and so the total resistance of the fins are:

1
Rf = (6)
h Af η f

A_f=p*L*N "finned area"


R_f=1/(h*A_f*eta) "resistance of fins"

The total area of the base of the heat sink that is not finned is:

Auf = W 2 − N Ac (7)
and the thermal resistance from the unfinned base is:

1
Ruf = (8)
h Auf

A_uf=W^2-N*Ac "unfinned area"


R_uf=1/(h*A_uf) "resistance of unfinned area"

The total resistance of the heat sink is the combination of Rf and Ruf in parallel:

−1
⎛ 1 1 ⎞
Rtotal =⎜ + ⎟⎟ (9)
⎜R R
⎝ f uf ⎠

and the total heat transfer rate is:

q =
(Tbase − Tair ) (10)
Rtotal

R_total=(1/R_f+1/R_uf)^(-1)
"total thermal resistance of the heat sink"
q_dot=(T_base-T_air)/R_total "heat transfer"

which leads to q = 291.7 W.

b.) Prepare a plot that shows the heat transfer rate from the heat sink as a function of the distance
between adjacent fins, a, for a fixed value of D = 0.75 cm. Be sure that the fin length is
calculated using L = 10 a . Your plot should exhibit a maximum value, indicating that there
is an optimal value of a.

Figure 3 illustrates the heat transfer rate from the heat sink as a function of a for D = 0.75 cm.
Figure 3: Heat transfer rate as a function of the distance between adjacent fins for D = 0.75 cm.

c.) Prepare a plot that shows the heat transfer rate from the heat sink as a function of the
diameter of the fins, D, for a fixed value of a = 0.5 cm. Be sure that the fin length is
calculated using L = 10 a . Your plot should exhibit a maximum value, indicating that there
is an optimal value of D.

Figure 4 illustrates the heat transfer rate from the heat sink as a function of D for a = 0.5 cm.

Figure 4: Heat transfer rate as a function of the diameter of the fins for a = 0.5 cm.

d.) Determine the optimal value of a and D using EES' built-in optimization capability.
Comment out the optimization parameters (a and D) and access the optimization algorithms from
the Calculate Menu by selecting Min/Max (Figure 5).

Figure 5: Find Minimum or Maximum Window

Select the variable to be minimized or maximized from the list on the left and the independent
variables to be varied from the list on the right. You will need to provide a reasonable initial
guess and bounds for the independent variables by selecting the Bounds button; note that it is not
practical for a or D to be less than 1.0 mm. You can experiment with the different optimization
methods and see which technique is more robust.

I found the genetic optimization algorithm to work the best for this problem; with a sufficient
number of individuals I identified an optimal design consisting of approximately 1500 very small
fins of D = 1.1 mm separated by a = 1.4 mm. The associated rate of heat transfer is q = 352.2
W.
Problem 1.7-3 (1-15 in text): Material Processing
Figure P1.7-3 illustrates a material processing system.

oven wall temperature varies with x

gap filled with gas


th = 0.6 mm
kg = 0.03 W/m-K
u = 0.75 m/s
Tin = 300 K
D = 5 cm
x

extruded material
k = 40 W/m-K
α = 0.001 m2/s
Figure P1.7-3: Material processing system.

Material is extruded and enters the oven at Tin = 300 K with velocity u = 0.75 m/s. The material
has velocity u = 0.75 m/s and diameter D = 5 cm. The conductivity of the material is k = 40
W/m-K and the thermal diffusivity is α = 0.001 m2/s.

In order to precisely control the temperature of the material, the oven wall is placed very close to
the outer diameter of the extruded material and the oven wall temperature distribution is
carefully controlled. The gap between the oven wall and the material is th = 0.6 mm and the
oven-to-material gap is filled with gas that has conductivity kg = 0.03 W/m-K. Radiation can be
neglected in favor of convection through the gas from the oven wall to the material. For this
situation, the heat flux experienced by the material surface can be approximately modeled
according to:

kg
′′ ≈
qconv (Tw − T )
th

where Tw and T are the oven wall and material temperatures at that position. The oven wall
temperature varies with position x according to:

⎛ x ⎞
Tw = T f − (T f − Tw,0 ) exp ⎜ − ⎟
⎝ Lc ⎠

where Tw,0 is the temperature of the wall at the inlet (at x = 0), Tf = 1000 K is the temperature of
the wall far from the inlet, and Lc is a characteristic length that dictates how quickly the oven
wall temperature approaches Tf. Initially, assume that Tw,0 = 500 K, Tf = 1000 K, and Lc = 1 m.
Assume that the oven can be approximated as being infinitely long.
a.) Is an extended surface model appropriate for this problem?

The inputs are entered in EES:

$UnitSystem SI MASS DEG PA C J


$Tabstops 0.2 0.4 0.6 0.8 3.5

k=40 [W/m-K] "conductivity"


u=0.75 [m/s] "velocity"
T_f=1000 [K] "wall temperature far from the inlet"
T_w_0=500 [K] "wall temperature at the inlet"
L_c=1 [m] "characteristic length which oven wall approaches T_f"
T_in=300 [K] "inlet temperature"
alpha=0.001 [m^2/s] "thermal diffusivity"
k_g=0.03 [W/m-K] "gas conductivity"
th=0.6 [mm]*convert(mm,m) "oven-to-material gap thickness"
D=5 [cm]*convert(cm,m) "diameter"

The Biot number is the ratio of the resistance that is neglected (internal conduction) to the
resistance that is considered (conduction across the gap):

kg D
Bi = (1)
th 2 k

Bi=(k_g/th)*D/(2*k) "Biot number"

which leads to Bi = 0.031. This is sufficiently less than 1 to justify an extended surface model.

b.) Assume that your answer to (a) was yes. Develop an analytical solution that can be used to
predict the temperature of the material as a function of x.

An energy balance on a control volume differential for a differential (in x) segment of the
material is shown in Figure P1.7-3-2.

kg
per dx (T − Tw )
th
(ρ u Ac cT )x (ρ u Ac cT )x+dx

⎛ dT ⎞ ⎛ dT ⎞
⎜ −k Ac ⎟ ⎜ −k Ac ⎟
⎝ dx ⎠ x dx ⎝ dx ⎠ x+ dx
Figure P1.7-3-2: Energy balance on a differential control volume.

The energy balance suggested by Figure P1.7-3-2 is:

( ρ u Ac cT ) x + ⎛⎜ −k Ac
dT ⎞ ⎛ dT ⎞ k
⎟ = ( ρ u Ac cT ) x + dx + ⎜ − k Ac ⎟ + g per dx (T − Tw ) (2)
⎝ dx ⎠ x ⎝ dx ⎠ x + dx th

where c is the specific heat capacity, Ac is the cross-sectional area and per is the perimeter of the
material:
D2
Ac = π (3)
4

per = π D (4)

A_c=pi*D^2/4 "cross-sectional area"


per=pi*D "perimeter"

Expanding the terms in Eq. (2) and simplifying:

dT d 2T k
0 = ρ u Ac c − k Ac 2 + g per (T − Tw ) (5)
dx dx th

Rearranging Eq. (5) and dividing through by k Ac leads to:

d 2T u dT k g per k per
− − T =− g Tw (6)
dx 2
α dx th k Ac th k Ac

Substituting the wall temperature variation into Eq. (6) leads to:

d 2T u dT ⎡ ⎛ x ⎞⎤
− − m 2 T = − m 2 ⎢T f − (T f − Tw,0 ) exp ⎜ − ⎟⎥ (7)
dx 2
α dx ⎣ ⎝ Lc ⎠⎦

where

k g per
m= (8)
th k Ac

m=sqrt(4*k_g/(th*k*D)) "fin parameter"

The boundary conditions are the inlet temperature:

Tx =0 = Tin (9)

and the temperature must approach Tf as x approaches infinity:

Tx →∞ = T f (10)

The solution is broken into a homogeneous and particular component:

T = Th + Tp (11)

and substituted into Eq. (7):


d 2Th u dTh d 2Tp u dTp ⎡ ⎛ x ⎞⎤

dx 2 α
− m 2
Th + −
dx 2 α dx
− m 2
T p = − m 2
⎢T f − ( T f − Tw ,0 ) exp ⎜ − ⎟⎥ (12)
 dx
 ⎣
⎝ Lc

⎠⎦
homogeneous ordinary
differential equation particular ordinary differential equation

The solution to the homogeneous differential equation is:

⎡⎛ u + u 2 + 4 α 2 m 2 ⎞ ⎤ ⎡⎛ u − u 2 + 4 α 2 m 2 ⎞ ⎤
Th = C1 exp ⎢⎜ ⎟ x ⎥ + C2 exp ⎢⎜ ⎟ x⎥ (13)
⎢⎣⎜⎝ 2α ⎟ ⎥
⎠ ⎦ ⎢⎣⎜⎝ 2α ⎟ ⎥
⎠ ⎦

The particular solution is obtained by the method of undetermined coefficients; the assumed
form of the particular solution is:

⎛ x ⎞
Tp = C3 exp ⎜ - ⎟ + C4 (14)
⎝ Lc ⎠

and substituted into the particular differential equation:

⎛ x ⎞ ⎛ x ⎞ ⎛ x⎞ ⎡ ⎛ x ⎞⎤
C3 exp ⎜ - ⎟ − m 2C3 exp ⎜ - ⎟ − m 2 C4 = −m 2 ⎢T f − (T f − Tw,0 ) exp ⎜ − ⎟ ⎥
C3 u
exp ⎜ - ⎟+
⎠ α Lc
2
Lc ⎝ Lc ⎝ Lc ⎠ ⎝ Lc ⎠ ⎣ ⎝ Lc ⎠ ⎦
(15)

Equation (15) provides one equation for C3 that is obtained by considering the exponential terms:

m 2 (T f − Tw,0 )
C3 = (16)
⎛ 1 u ⎞
⎜ 2+ − m2 ⎟
⎝ Lc α Lc ⎠

and another equation for C4 that is obtained by considering the constant terms:

C4 = T f (17)

Substituting Eqs. (13), (14), (16), and (17) leads to:

⎡⎛ u + u 2 + 4 α 2 m 2 ⎞ ⎤ ⎡⎛ u − u 2 + 4 α 2 m 2 ⎞ ⎤

T = C1 exp ⎜ ⎥
⎟ x + C2 exp ⎜ ⎢ ⎟ x⎥
⎢⎣⎜⎝ 2α ⎟ ⎥
⎠ ⎦ ⎢⎣⎜⎝ 2α ⎟ ⎥
⎠ ⎦
m 2 (T f − Tw,0 ) ⎛ x ⎞ (18)
+ exp ⎜ - ⎟ + T f
⎛ 1 u ⎞ ⎝ Lc ⎠
⎜ 2+ − m2 ⎟
⎝ Lc α Lc ⎠
The constants C1 and C2 are obtained by considering the boundary conditions. Substituting Eq.
(18) into Eq. (10) leads to:

⎡⎛ u + u 2 + 4 α 2 m 2 ⎞ ⎤
C1 exp ⎢⎜ ⎟ ∞⎥ + Tf = Tf (19)
⎢⎣⎜⎝ 2α ⎟ ⎥
⎠ ⎦

which can only be true if C1 = 0. Therefore:

⎡⎛ u − u 2 + 4 α 2 m 2 ⎞ ⎤ m2 (T f − Tw,0 ) ⎛ x ⎞
T = C2 exp ⎢⎜ ⎥
⎟x + exp ⎜ - ⎟ + T f (20)
⎢⎣⎜⎝ 2α ⎟ ⎥ ⎛ 1
⎠ ⎦ ⎜ + u − m 2 ⎞⎟ ⎝ Lc ⎠
⎝ Lc α Lc
2

Substituting Eq. (20) into Eq. (9) leads to:

m 2 (T f − Tw,0 )
C2 + + T f = Tin (21)
⎛ 1 u ⎞
⎜ 2+ − m2 ⎟
⎝ Lc α Lc ⎠

or

m 2 (T f − Tw,0 )
C2 = Tin − T f − (22)
⎛ 1 u ⎞
⎜ 2+ − m2 ⎟
⎝ Lc α Lc ⎠

C_2=T_in-T_f-m^2*(T_f-T_w_0)/(1/L_c^2+u/(alpha*L_c)-m^2) "boundary condition at x=0"

The solution for the material temperature and the wall temperature are entered in EES:

x=0.5 [m] "position"


T=C_2*exp(((u-sqrt(u^2+4*alpha^2*m^2))/(2*alpha))*x)+m^2*(T_f-T_w_0)*&
exp(-x/L_c)/(1/L_c^2+u/(alpha*L_c)-m^2)+T_f "temperature of the material"
T_w=T_f-(T_f-T_w_0)*exp(-x/L_c) "wall temperature"

c.) Plot the temperature of the material and the temperature of the wall as a function of position
for 0 < x < 20 m. Plot the temperature gradient experienced by the material as a function of
position for 0 < x < 20 m.

Figure P1.7-3-3 illustrates the temperature of the material and the wall as a function of position.
1000

900
wall

Temperature (K)
800

700
material
600

500

400

300
0 2.5 5 7.5 10 12.5 15 17.5 20
Position (m)
Figure P1.7-3-3: Temperature of the material and the wall as a function of position.

The temperature gradient is evaluated by differentiating Eq. (20):

dT ⎛ u − u 2 + 4 α 2 m2 ⎞ ⎡⎛ u − u 2 + 4 α 2 m 2 ⎞ ⎤ m 2 (T f − Tw,0 ) ⎛ x ⎞
= C2 ⎜ ⎟ exp ⎢⎜ ⎟ x⎥ − exp ⎜ - ⎟ (23)
⎜ 2α ⎟ ⎜ 2α ⎟
⎣⎢⎝ ⎠ ⎦⎥ Lc ⎛⎜ 1 + u − m 2 ⎞⎟ ⎝ Lc ⎠
dx
⎝ ⎠
⎝ Lc α Lc
2

dTdx=C_2*((u-sqrt(u^2+4*alpha^2*m^2))/(2*alpha))*exp(((u-sqrt(u^2+4*alpha^2*m^2))/(2*alpha))*x)&
-m^2*(T_f-T_w_0)*exp(-x/L_c)/(1/L_c^2+u/(alpha*L_c)-m^2)/L_c "temperature gradient"

Figure P1.7-3-4 illustrates the temperature gradient as a function of position.


70

60
Temperature gradient (K/m)

50

40

30

20

10

0
0 2.5 5 7.5 10 12.5 15 17.5 20
Position (m)
Figure P1.7-3-4: Temperature gradient in the material as a function of position.

The parameter Lc can be controlled in order to control the maximum temperature gradient
experienced by the material as it moves through the oven.

d.) Prepare a plot showing the maximum temperature gradient as a function of Lc. Overlay on
your plot the distance required to heat the material to Tp = 800 K (Lp). If the maximum
temperature gradient that is allowed is 60 K/m then what is the appropriate value of Lc and
the corresponding value of Lp.

The value Lp is obtained:


T_p=800 [K]
T_p=C_2*exp(((u-sqrt(u^2+4*alpha^2*m^2))/(2*alpha))*L_p)+&
m^2*(T_f-T_w_0)*exp(-L_p/L_c)/(1/L_c^2+u/(alpha*L_c)-m^2)+T_f

which leads to Lp = 10.18 m.

The maximum temperature gradient can be obtained by using EES' optimization routines. Setup
a parametric table that includes the variables L_c, x, dTdx, L_p, and L_c. The value of L_c that
is set in the Equations window is commented out and the values of L_c in the table are varied
from 0.1 to 5 m. Min/Max Table is selected from the Calculate menu. The value of dTdx is
maximized by varying x with bounds from 0 to some large value. The maximum temperature
gradient and value of Lp are shown Figure P1.7-3-5 as a function of Lc. Figure P1.7-3-5 indicates
that Lc should be equal to 1.8 m in order to control the temperature gradient, which leads to Lp =
11 m.
90 14
Maximum temperature gradient (K/m)

Position at which T = 800 K


80 maximum temperature gradient 13

70 12

60 11

50 10
distance at which T = 800 K

40 9
0 1 2 3 4 5
Lc (m)
Figure P1.7-3-5: Maximum temperature gradient and Lp as a function of Lc.
Problem 1.7-4 (1-16 in text): Solar Collector Tube
The receiver tube of a concentrating solar collector is shown in Figure P1.7-4.

Ta = 25°C
qs′′ ha = 25 W/m -K
2

r = 5 cm
th = 2.5 mm
k = 10 W/m-K

φ
Tw = 80°C
hw = 100 W/m -K
2

Figure P1.7-4: A solar collector

The receiver tube is exposed to solar radiation that has been reflected from a concentrating
mirror. The heat flux received by the tube is related to the position of the sun and the geometry
and efficiency of the concentrating mirrors. For this problem, you may assume that all of the
radiation heat flux is absorbed by the collector and neglect the radiation emitted by the collector
to its surroundings. (Chapter 10 will provide information on the radiation characteristics of
surfaces that will allow a more complete evaluation of solar collectors.) The flux received at the
collector surface ( qs′′ ) is not circumferentially uniform but rather varies with angular position;
the flux is uniform along the top of the collector, π < φ < 2π rad, and varies sinusoidally along
the bottom, 0 < φ < π rad, with a peak at φ = π/2 rad.

⎪⎧qt′′+ ( q ′′p − qt′′) sin (φ ) for 0 < φ < π


qs′′ (φ ) = ⎨
⎪⎩qt′′ for π < φ < 2 π

where qt′′ = 1000 W/m2 is the uniform heat flux along the top of the collector tube and q ′′p =
5000 W/m2 is the peak heat flux along the bottom. The receiver tube has an inner radius of r =
5.0 cm and thickness of th = 2.5 mm (because th/r << 1 it is possible to ignore the small
difference in convection area on the inner and outer surfaces of the tube). The thermal
conductivity of the tube material is k = 10 W/m-K. The solar collector is used to heat water,
which is at Tw = 80°C at the axial position of interest. The average heat transfer coefficient
between the water and the internal surface of the collector is hw = 100 W/m2-K. The external
surface of the collector is exposed to air at Ta = 25°C. The average heat transfer coefficient
between the air and the external surface of the collector is ha = 25 W/m2-K.
a.) Can the collector be treated as an extended surface for this problem (i.e., can the temperature
gradients in the radial direction in the collector material be neglected)?

The input parameters are entered into the EES program:


$UnitSystem SI MASS RAD PA K J
$Tabstops 0.2 0.4 0.6 3.5 in

"Inputs"
qf_t=1000 [W/m^2] "Heat flux on top surface"
qf_p=5000 [W/m^2] "Peak heat flux on bottom surface"
k = 10 [W/m-K] "Conductivity of the collector material"
th=2.5 [mm]*convert(mm,m) "thickness of collector"
r=5.0[cm]*convert(cm,m) "inner radius of collector"
T_a=converttemp(C,K,25[C]) "temperature of surrounding air"
h_bar_a=25 [W/m^2-K] "heat transfer coefficient to surrounding air"
T_w=converttemp(C,K,80 [C]) "temperature of water"
h_bar_w=100 [W/m^2-K] "heat transfer coefficient to water"
L=1 [m] "length of the collector"

The extended surface approximation neglects temperature gradients across the thickness of the
tube but considers the temperature difference between the collector surfaces and the surrounding
water and air. This assumption is equivalent to neglecting the conduction resistance across the
tube as being small relative to the two convection resistances characterizing heat transfer to the
air and the water. Two Biot numbers are calculated based on the air (Bia) and water (Biw)

ha th
Bia =
2k

hw th
Biw =
2k

Bi_a=h_bar_a*th/(2*k) "Biot number based on air side"


Bi_w=h_bar_w*th/(2*k) "Biot number based on water side"

Both Biot numbers are found to be much less than one (0.003 and 0.01, respectively) and
therefore the extended surface approximation is valid.

b.) Develop an analytical model that will allow the temperature distribution in the collector wall
to be determined as a function of circumferential position.

The computational domain (the receiver tube wall) goes from φ = 0 to 2π rad; however, there are
actually two separate computational domains, the top and bottom, each with a different
governing equation due to the different spatial variation in the heat flux. A differential control
volume is used to derive the governing equation in the bottom computational domain. An energy
balance on the control volume leads to:

qφ + qs = qφ + dφ + qconv ,a + qconv , w

The conduction per unit length in the circumferential direction, qφ , is written using Fourier’s
law.
th L dTb
qφ = −k
r dφ

where L is the length of the collector tube and Tb is the temperature along the bottom of the tube.
Note that the temperature gradient along the circumference of the tube (K/m) is written as the
product the derivative of temperature with respect to angle (K/rad) and the inverse of the radius.

The convection to the air and the water are:

qconv ,a = r dφ L ha (Tb − Ta )

qconv , w = r dφ L hw (Tb − Tw )

The energy absorbed due to the solar flux is:

qs = ⎡⎣ qt′′+ ( q ′′p − qt′′) sin (φ ) ⎤⎦ L r dφ

Combining these equations leads to:

⎡ th dTb ⎤
⎡ qt′′+ ( q ′′p − qt′′) sin (φ ) ⎤ r dφ =
d
⎣ ⎦ dφ ⎢ −k r dφ ⎥ dφ + r dφ ha (Tb − Ta ) + r dφ hw (Tb − Tw )
⎣ ⎦

which can be simplified to:

d 2Tb r 2 ha r 2 hw r 2 ha r 2 hw r2
− Tb − Tb = − Ta − Tw − ⎣ qt + ( q p − qt ) sin (φ ) ⎦
⎡ ′′ ′′ ′′ ⎤ (1)
dφ 2 k th k th k th k th k th

Equation (1) is the governing differential equation for the temperature along bottom of the
collector and it is therefore valid from 0 < θ < π. A similar set of steps leads to the governing
equation for the temperature along the top of the collector (Tt) that is valid from π < θ < 2 π.

d 2Tt r 2 ha r 2 hw r 2 ha r 2 hw r2
− T − T = − T − T − 
q ′′ (2)
dφ 2 k th
t t a w t
k th k th k th k th

Equations (1) and (2) are entered into Maple and solved in order to determine their general
solutions.

> restart;
> ODE_b:=diff(diff(T_b(phi),phi),phi)-r^2*h_bar_a*T_b(phi)/(k*th)-r^2*h_bar_w*T_b(phi)/(k*th)=-
r^2*h_bar_a*T_a/(k*th)-r^2*h_bar_w*T_w/(k*th)-(qf_t+(qf_p-qf_t)*sin(phi))*r^2/(k*th);
⎛d ⎞ r h_bar_a T_b( φ ) r h_bar_w T_b( φ )
2 2 2
ODE_b := ⎜ 2 T_b( φ ) ⎟⎟ − − =
⎜ dφ k th k th
⎝ ⎠
r2 h_bar_a T_a r2 h_bar_w T_w ( qf_t + ( qf_p − qf_t ) sin( φ ) ) r2
− − −
k th k th k th
> T_b_s:=dsolve(ODE_b);
⎛ r h_bar_a + h_bar_w φ ⎞ ⎛ r h_bar_a + h_bar_w φ ⎞
⎜ ⎟ ⎜− ⎟
⎜ ⎟ ⎜ ⎟
⎝ k th ⎠ ⎝ k th ⎠
T_b_s := T_b( φ ) = e _C2 + e _C1 + (
−r ( −qf_p + qf_t ) ( h_bar_a + h_bar_w ) sin( φ )
2

+ ( r2 ( h_bar_a + h_bar_w ) + k th ) ( h_bar_a T_a + h_bar_w T_w + qf_t ) ) (


( r2 ( h_bar_a + h_bar_w ) + k th ) ( h_bar_a + h_bar_w ) )
> ODE_t:=diff(diff(T_t(phi),phi),phi)-r^2*h_bar_a*T_t(phi)/(k*th)-r^2*h_bar_w*T_t(phi)/(k*th)=-
r^2*h_bar_a*T_a/(k*th)-r^2*h_bar_w*T_w/(k*th)-qf_t*r^2/(k*th);
⎛d ⎞ r h_bar_a T_t ( φ ) r h_bar_w T_t( φ )
2 2 2
ODE_t := ⎜ 2 T_t( φ ) ⎟⎟ − − =
⎜ dφ k th k th
⎝ ⎠
r h_bar_a T_a r2 h_bar_w T_w qf_t r2
2
− − −
k th k th k th
> T_t_s:=dsolve(ODE_t);
⎛ r h_bar_a + h_bar_w φ ⎞ ⎛ r h_bar_a + h_bar_w φ ⎞
⎜ ⎟ ⎜− ⎟
⎜ ⎟ ⎜ ⎟
⎝ k th ⎠ ⎝ k th ⎠
T_t_s := T_t( φ ) = e _C2 + e _C1
h_bar_a T_a + h_bar_w T_w + qf_t
+
h_bar_a + h_bar_w

Note that although both solutions are given with constants of integration C1 and C2 it is clear that
these constants cannot be the same. Here, the constants for the general solution for Tt will be C3
and C4. The solutions are copied into EES and manipulated slightly to obtain:

"Solutions"
T_b = exp(r/k^(1/2)/th^(1/2)*(h_bar_a+h_bar_w)^(1/2)*phi)*C_2+&
exp(-r/k^(1/2)/th^(1/2)*(h_bar_a+h_bar_w)^(1/2)*phi)*C_1+&
(-r^2*(h_bar_a+h_bar_w)*(-qf_p+qf_t)*sin(phi)+(h_bar_a*T_a+&
T_w*h_bar_w+qf_t)*(r^2*(h_bar_a+h_bar_w)+k*th))/(h_bar_a+&
h_bar_w)/(r^2*(h_bar_a+h_bar_w)+k*th)
T_t = exp(r/k^(1/2)/th^(1/2)*(h_bar_a+h_bar_w)^(1/2)*phi)*C_4+&
exp(-r/k^(1/2)/th^(1/2)*(h_bar_a+h_bar_w)^(1/2)*phi)*C_3+&
(h_bar_a*T_a+h_bar_w*T_w+qf_t)/(h_bar_a+h_bar_w)

There are four unknown constants of integration (C1 through C4). Therefore, 4 boundary
conditions are required to obtain the four constants of integration (two for each solution). The
temperature must be continuous at both of the interfaces between the top and bottom domains:

Tb ,φ =0 = Tt ,φ = 2π (3)
Tb ,φ =π = Tt ,φ =π (4)

Also, the energy flowing between the regions must be conserved. An interface energy balance at
φ = π rad provides:

L th dTb L th dTt
−k = −k
r dφ φ =π
r dφ φ =π

which implies that the temperature gradient at φ = π rad is continuous in both domains:

dTb dTt
= (5)
dφ φ =π
dφ φ =π

A similar equation results for the interface at φ = 0 rad.

dTb dTt
= (6)
dφ φ =0
dφ φ = 2π

Maple can carry out the symbolic manipulation of the solution while EES does the algebra to
determine the constants. To obtain the left and right hand sides of Eq. (3):

> T_b_0:=rhs(eval(T_b_s,phi=0));
h_bar_a T_a + h_bar_w T_w + qf_t
T_b_0 := _C2 + _C1 +
h_bar_a + h_bar_w
> T_t_2pi:=rhs(eval(T_t_s,phi=2*pi));
⎛ 2 r h_bar_a + h_bar_w π ⎞ ⎛ 2 r h_bar_a + h_bar_w π ⎞
⎜ ⎟ ⎜− ⎟
⎜ ⎟ ⎜ ⎟
⎝ k th ⎠ ⎝ k th ⎠
T_t_2pi := e _C2 + e _C1
h_bar_a T_a + h_bar_w T_w + qf_t
+
h_bar_a + h_bar_w

These two expressions can be cut and pasted into EES and, with minimal modification, used to
set the boundary condition associated with Eq. (3). The necessary modifications include
changing _C1 and _C2 to C_1 and C_2 in the equation for T_b_0 and changing _C1 and _C2 to
C_3 and C_4 in the equation for T_t_2pi.

"Boundary conditions"
"Temperature equality at phi=0"
T_b_0 = C_2+ C_1+(h_bar_a*T_a+h_bar_w*T_w+qf_t)/(h_bar_a+h_bar_w)
"temperature in bottom domain at phi=0"
T_t_2pi = exp(2*r/k^(1/2)/th^(1/2)*(h_bar_a+h_bar_w)^(1/2)*pi)*C_4+&
exp(-2*r/k^(1/2)/th^(1/2)*(h_bar_a+h_bar_w)^(1/2)*pi)*C_3+&
(h_bar_a*T_a+h_bar_w*T_w+qf_t)/(h_bar_a+h_bar_w)
"temperature in top domain at phi=2 pi"
T_b_0=T_t_2pi

The EES and Maple text listed above seems long and complicated however very little of it
needed to be entered manually; the process of solving a relatively complex heat transfer problem
is reduced to a relatively straightforward integration of two powerful pieces of software.

The process is repeated for Eq. (4), in Maple:

> T_b_pi:=rhs(eval(T_b_s,phi=pi));
⎛ r h_bar_a + h_bar_w π ⎞ ⎛ r h_bar_a + h_bar_w π ⎞
⎜ ⎟ ⎜− ⎟
⎜ ⎟ ⎜ ⎟
⎝ k th ⎠ ⎝ k th ⎠
T_b_pi := e _C2 + e _C1 + (
−r2 ( −qf_p + qf_t ) ( h_bar_a + h_bar_w ) sin( π )
+ ( r2 ( h_bar_a + h_bar_w ) + k th ) ( h_bar_a T_a + h_bar_w T_w + qf_t ) ) (
( r2 ( h_bar_a + h_bar_w ) + k th ) ( h_bar_a + h_bar_w ) )
> T_t_pi:=rhs(eval(T_t_s,phi=pi));
⎛ r h_bar_a + h_bar_w π ⎞ ⎛ r h_bar_a + h_bar_w π ⎞
⎜ ⎟ ⎜− ⎟
⎜ ⎟ ⎜ ⎟
⎝ k th ⎠ ⎝ k th ⎠
T_t_pi := e _C2 + e _C1
h_bar_a T_a + h_bar_w T_w + qf_t
+
h_bar_a + h_bar_w

The symbolic equations determined by Maple are then entered in EES with the same
modification for _C1 and _C2 noted above:

"Temperature equality at pi"


T_b_pi = exp(r/k^(1/2)/th^(1/2)*(h_bar_a+h_bar_w)^(1/2)*pi)*C_2+&
exp(-r/k^(1/2)/th^(1/2)*(h_bar_a+h_bar_w)^(1/2)*pi)*C_1+&
(-r^2*(-qf_p+qf_t)*(h_bar_a+h_bar_w)*sin(pi)+(r^2*(h_bar_a+h_bar_w)+k*th)&
*(h_bar_a*T_a+h_bar_w*T_w+qf_t))/(r^2*(h_bar_a+h_bar_w)+k*th)/(h_bar_a+h_bar_w)
"temperature in bottom domain at phi = pi"
T_t_pi = exp(r/k^(1/2)/th^(1/2)*(h_bar_a+h_bar_w)^(1/2)*pi)*C_4+&
exp(-r/k^(1/2)/th^(1/2)*(h_bar_a+h_bar_w)^(1/2)*pi)*C_3+&
(h_bar_a*T_a+h_bar_w*T_w+qf_t)/(h_bar_a+h_bar_w)
"temperature in top domain at phi = pi"
T_b_pi=T_t_pi

Equations (5) and (6) are dealt with in the same way. Maple is used to evaluate the symbolic
expressions for the required derivatives:

> dTbdphi_0:=rhs(eval(diff(T_b_s,phi),phi=0));
r h_bar_a + h_bar_w _C2 r h_bar_a + h_bar_w _C1
dTbdphi_0 := −
k th k th
r2 ( −qf_p + qf_t )

r2 ( h_bar_a + h_bar_w ) + k th
> dTtdphi_2pi:=rhs(eval(diff(T_t_s,phi),phi=2*pi));
⎛ 2 r h_bar_a + h_bar_w π ⎞
⎜ ⎟
⎜ ⎟
⎝ k th ⎠
r h_bar_a + h_bar_w e _C2
dTtdphi_2pi :=
k th
⎛ 2 r h_bar_a + h_bar_w π ⎞
⎜− ⎟
⎜ ⎟
⎝ k th ⎠
r h_bar_a + h_bar_w e _C1

k th
> dTbdphi_pi:=rhs(eval(diff(T_b_s,phi),phi=pi));
⎛ r h_bar_a + h_bar_w π ⎞
⎜ ⎟
⎜ ⎟
⎝ k th ⎠
r h_bar_a + h_bar_w e _C2
dTbdphi_pi :=
k th
⎛ r h_bar_a + h_bar_w π ⎞
⎜− ⎟
⎜ ⎟
⎝ k th ⎠
r h_bar_a + h_bar_w e _C1

k th
r ( −qf_p + qf_t ) cos( π )
2
− 2
r ( h_bar_a + h_bar_w ) + k th
> dTtdphi_pi:=rhs(eval(diff(T_t_s,phi),phi=pi));
⎛ r h_bar_a + h_bar_w π ⎞
⎜ ⎟
⎜ ⎟
⎝ k th ⎠
r h_bar_a + h_bar_w e _C2
dTtdphi_pi :=
k th
⎛ r h_bar_a + h_bar_w π ⎞
⎜− ⎟
⎜ ⎟
⎝ k th ⎠
r h_bar_a + h_bar_w e _C1

k th

These expressions are entered in EES (with changes to _C1 and _C2) in order to provide the
final two boundary conditions.

"Temperature gradient equality at 0"


dTbdphi_0 = r/k^(1/2)/th^(1/2)*(h_bar_a+h_bar_w)^(1/2)*C_2-&
r/k^(1/2)/th^(1/2)*(h_bar_a+h_bar_w)^(1/2)*C_1-&
r^2*(-qf_p+qf_t)/(r^2*(h_bar_a+h_bar_w)+k*th)
"gradient in bottom domain at phi =0"
dTtdphi_2pi = r/k^(1/2)/th^(1/2)*(h_bar_a+h_bar_w)^(1/2)*exp(2*r/k^(1/2)/th^(1/2)*&
(h_bar_a+h_bar_w)^(1/2)*pi)*C_4-r/k^(1/2)/th^(1/2)*(h_bar_a+h_bar_w)^(1/2)*&
exp(-2*r/k^(1/2)/th^(1/2)*(h_bar_a+h_bar_w)^(1/2)*pi)*C_3
"gradient in top at phi = 2 pi"
dTbdphi_0=dTtdphi_2pi

"Temperature gradient equality at pi"


dTbdphi_pi = r/k^(1/2)/th^(1/2)*(h_bar_a+h_bar_w)^(1/2)*exp(r/k^(1/2)/th^(1/2)*&
(h_bar_a+h_bar_w)^(1/2)*pi)*C_2-r/k^(1/2)/th^(1/2)*(h_bar_a+h_bar_w)^(1/2)*&
exp(-r/k^(1/2)/th^(1/2)*(h_bar_a+h_bar_w)^(1/2)*pi)*C_1-r^2*(-qf_p+qf_t)*cos(pi)&
/(r^2*(h_bar_a+h_bar_w)+k*th)
"gradient in bottom at phi = pi"
dTtdphi_pi = r/k^(1/2)/th^(1/2)*(h_bar_a+h_bar_w)^(1/2)*exp(r/k^(1/2)/th^(1/2)*&
(h_bar_a+h_bar_w)^(1/2)*pi)*C_4-r/k^(1/2)/th^(1/2)*(h_bar_a+h_bar_w)^(1/2)&
*exp(-r/k^(1/2)/th^(1/2)*(h_bar_a+h_bar_w)^(1/2)*pi)*C_3
"gradient in top at phi = pi"
dTbdphi_pi=dTtdphi_pi

The analytical solution is converted to Celsius:

T_b_C=converttemp(K,C,T_b) "bottom temperature in C"


T_t_C=converttemp(K,C,T_t) "top temperature in C"

and plotted by setting up two parametric tables. The first table contains the variables phi and
T_b_C (where phi is varied from 0 to π) while the second table contains the variables phi and
T_t_C (where phi is varied from π to 2π). The temperature distribution is shown in Figure 2.

Figure 2: Temperature distribution around the circumference of the collector tube for various values
of the tube conductivity.

It is possible to adjust any of the input parameters within EES the solution remains valid because
the constants are evaluated symbolically. For example, Figure 2 also illustrates how the solution
varies as the conductivity of the receiver tube changes.
Problem 1.8-1 (1-17 in text): Disk Brake
Figure P1.8-1 illustrates a disk brake for a rotating machine. The temperature distribution within
the brake can be assumed to be a function of radius only. The brake is divided into two regions.
In the outer region, from Rp = 3.0 cm to Rd = 4.0 cm, the stationary brake pads create frictional
heating and the disk is not exposed to convection. The clamping pressure applied to the pads is
P = 1.0 MPa and the coefficient of friction between the pad and the disk is μ = 0.15. You may
assume that the pads are not conductive and therefore all of the frictional heating is conducted
into the disk. The disk rotates at N = 3600 rev/min and is b = 5.0 mm thick. The conductivity of
the disk is k = 75 W/m-K and you may assume that the outer rim of the disk is adiabatic.

stationary coefficient of friction, μ = 0.15


brake pads

clamping pressure
P = 1 MPa
b = 5 mm Ta = 30°C, h
Rp = 3 cm
Rd = 4 cm
center line
k = 75 W/m-K
disk, rotates at N = 3600 rev/min
Figure P1.8-1: Disk brake.

In the inner region of the disk, from 0 to Rp, is exposed to air at Ta = 30°C. The heat transfer
coefficient between the air and disk surface depends on the angular velocity of the disk, ω,
according to:

1.25
⎡ W ⎤ ⎡ W ⎤⎛ ω ⎞
h = 20 ⎢ 2 ⎥ + 1500 ⎢ 2 ⎥ ⎜⎜ ⎟
⎣ m -K ⎦ ⎣ m -K ⎦ ⎝ 100 [ rad/s ] ⎟⎠

a.) Develop an analytical model of the temperature distribution in the disk brake; prepare a
plot of the temperature as a function of radius for r = 0 to r = Rd.

The inputs are entered in EES and the heat transfer coefficient is computed according to Eq.
Error! Reference source not found..

$UnitSystem SI MASS RAD PA K J


$TABSTOPS 0.2 0.4 0.6 0.8 3.5 in

b=5 [mm]*convert(mm,m) "thickness of disk"


N=3600 [rev/min] "rotational velocity of disk"
omega=N*convert(rev/min,rad/s) "angular velocity of disk"
mu=0.15 [-] "coefficient of friction"
P=1 [MPa]*convert(MPa,Pa) "clamping pressure"
k=75 [W/m-K] "conductivity"
Rd=4.0 [cm]*convert(cm,m) "outer radius of disk"
Rp=3.0 [cm]*convert(cm,m) "inner radius of pad"
Ta=converttemp(C,K,30) "air temperature"
h=20[W/m^2-K]+1500 [W/m^2-K]*(omega/100 [rad/s])^1.25 "heat transfer coefficient"

In the outer region, region 1, the energy balance on a differential control volume is shown in
Figure 2.

Figure 2: Differential energy balance in outer region, (region 1)

The energy balance suggested by Figure 2 is:

qr + q fh = qr + dr (1)

where q fh is the rate of thermal energy generated by frictional heating. After expanding the r +
dr term, Eq. (1) becomes:

dq
q fh = dr (2)
dr

The rate equation for conduction is:

dT1
q = −b 2 π r k (3)
dr

where T1 is the temperature in region 1. The force generated by the pad within the control
volume is the product of the clamping pressure, the area of contact, and the coefficient of
friction:

F = 4 π r dr P μ (4)

Note that the factor of 4 in Eq. (4) is due to their being contact on both sides of the disk. The
rate of frictional heating is the product of the force, the radius, and the angular velocity:

q fh = 4 π r 2 dr P μ ω (5)

Substituting Eqs. (3) and (5) into Eq. (2) leads to:
d ⎡ dT ⎤
4 π r 2 dr P μ ω = ⎢ −b 2 π r k 1 ⎥ dr (6)
dr ⎣ dr ⎦

which can be rearranged:

d ⎡ dT1 ⎤ 2Pμω 2
r = − r (7)
dr ⎢⎣ dr ⎥⎦ bk

or

d ⎡ dT1 ⎤
⎢ r ⎥ = −β r 2 (8)
dr ⎣ dr ⎦

where

2Pμω
β= (9)
bk

Equation (8) can be directly integrated:

⎡ dT1 ⎤
∫ d ⎢⎣ r dr ⎥⎦ = − β ∫ r
2
dr (10)

to achieve:

dT1 r3
r = − β + C1 (11)
dr 3

Equation (11) can be directly integrated again:

⎛ r 2 C1 ⎞
∫ 1 ∫ ⎜⎝ 3 + r ⎟⎠ dr
dT = − β (12)

to achieve:

r3
T1 = − β + C1 ln ( r ) + C2 (13)
9

Equation (13) is the general solution for the temperature in region 1; the constants of integration
will be selected in order to satisfy the boundary conditions.

In the inner region, region 2, the energy balance on a differential control volume is shown in
Figure 3.
Figure 3: Differential energy balance in inner region, (region 2)

The energy balance suggested by Figure 2 is:

qr = qr + dr + qconv (14)

After expanding the r + dr term, Eq. (14) becomes:

dq
0= dr + qconv (15)
dr

The rate equation for conduction remains the same:

dT2
q = −b 2 π r k (16)
dr

where T2 is the temperature in region 2. The rate equation for convection is:

qconv = 4 π r dr h (T2 − Ta ) (17)

Substituting Eqs. (16) and (17) into Eq. (15) leads to:

d ⎡ dT ⎤
⎢ −b 2 π r k 2 ⎥ dr + 4 π r dr h (T2 − Ta ) = 0 (18)
dr ⎣ dr ⎦

or

d ⎡ dT2 ⎤
r − m 2 r T2 = −m 2 r Ta (19)
dr ⎢⎣ dr ⎥⎦

where
2h
m= (20)
bk

The solution to Eq. (19) can be divided into its homogeneous (u2) and particular (v2) parts:

T2 = u2 + v2 (21)

The solution to the particular equation:

d ⎡ dv2 ⎤
⎢ r ⎥ − m 2 r v2 = − m 2 r Ta (22)
dr ⎣ dr ⎦

is

v2 = Ta (23)

The homogeneous equation:

d ⎡ du2 ⎤
r − m 2 r u2 = 0 (24)
dr ⎢⎣ dr ⎥⎦

is a form of Bessel's equation:

d ⎛ p dθ ⎞ 2 s
⎜x ⎟±c x θ =0 (25)
dx ⎝ dx ⎠

where

x=r (26)

θ = u2 (27)

p =1 (28)

c=m (29)

s =1 (30)

and the last term is negative. Following the flow chart provided in Section 1.8.4 of the book
leads to n = 0, a = 1, and therefore the solution is:

u2 = C3 BesselI ( 0, m r ) + C4 BesselK ( 0, m r ) (31)


The general solution for the temperature distribution in region 2 is therefore:

T2 = C3 BesselI ( 0, m r ) + C4 BesselK ( 0, m r ) + Ta (32)

Note that this could be obtained directly from Maple by entering Eq. (19):

> restart;
> ODE:=diff(r*diff(T2(r),r),r)-m^2*r*T2(r)=-m^2*r*Ta;
⎛d ⎞
2
ODE := ⎛⎜⎜ T2( r ) ⎞⎟⎟ + r ⎜⎜ 2 T2( r ) ⎟⎟ − m 2 r T2( r ) = −m 2 r Ta
d
⎝ d r ⎠ ⎝ dr ⎠
> T2s:=dsolve(ODE);
T2s := T2( r ) = BesselI( 0, m r ) _C2 + BesselK( 0, m r ) _C1 + Ta

The constants C1 through C4 in Eqs. (13) and (32) are obtained by applying the correct boundary
conditions. At r = 0, the temperature must remain finite. The figures provided in Section 1.8.4
of the book or the limit capability in Maple show that BesselK(0,m r) will become infinite as r
approaches zero:
> limit(BesselI(0,m*r),r=0);
1
> limit(BesselK(0,m*r),r=0);

therefore:

C4 = 0 (33)

The temperature and temperature gradient at the interface between the regions must be
continuous:

T2, r = Rp = T1, r = Rp (34)

and

dT2 dT1
= (35)
dr r = Rp dr r = Rp

The temperature gradient at the outer rim must be zero:

dT1
=0 (36)
dr r = Rp
Substituting Eqs. (13) and (32) into Eqs. (33) through (36) leads to:

R 3p
C3 BesselI ( 0, m R p ) + Ta = − β + C1 ln ( R p ) + C2 (37)
9

R p2
C3 m BesselI (1, m R p ) = − β
C1
+ (38)
3 Rp

Rd2 C1
−β + =0 (39)
3 Rd

Equations (37) through (39) are 3 equations for the unknown constants and can be solved in
EES.

beta=2*mu*P*omega/(k*b) "generation parameter"


m=sqrt(2*h/(k*b)) "fin parameter"
BesselI(0,m*Rp)*C_3+Ta=-1/9*beta*Rp^3+C_1*ln(Rp)+C_2 "equality of temperature at r=Rp"
BesselI(1,m*Rp)*m*C_3=-1/3*beta*Rp^2+1/Rp*C_1
"equality of temperature gradient at r=Rp"
-1/3*beta*Rd^2+1/Rd*C_1=0 "zero temperature gradient at r=Rd"

The general solutions are entered in EES:

T2 = BesselI(0,m*r2)*C_3+Ta "solution in region 2"


T1 = -1/9*beta*r1^3+C_1*ln(r1)+C_2 "solution in region 1"

A dimensionless radius, the variable rbar, is defined in order to allow a Parametric Table to be
generated where the variable r1 can be easily altered from Rp to Rd and the r2 can be easily
altered from 0 to Rp:

r1=Rp+(Rd-Rp)*rbar
r2=rbar*Rp

Figure 4 illustrates the temperature distribution in the disk.


Figure 4: Temperature distribution in the disk

b.) If the disk material can withstand a maximum safe operating temperature of 750°C then what
is the maximum allowable clamping pressure that can be applied? Plot the temperature
distribution in the disk at this clamping pressure. What is the braking torque that results?

The maximum operating temperature is obtained at r = Rd (see Figure 4). The clamping pressure
that results in T1 at the outer rim reaching the maximum allowable temperature can be
determined by commenting out the originally specified clamping pressure and specifying this
temperature:

{P=1 [MPa]*convert(MPa,Pa)} "clamping pressure"


T_max_allowed=converttemp(C,K,750) "maximum allowable temperature"
rbar=1.0
T1=T_max_allowed

which leads to a clamping pressure of P = 0.57 MPa. The temperature distribution for this
clamping pressure is shown in Figure 4. The torque applied by the pads (Tq) is obtained from
the integral:
Rd

Tq = ∫ 4 π r μ P dr
2
(40)
Rp

or

4
Tq = π μ P ⎡⎣ Rd3 − R 3p ⎤⎦ (41)
3
which leads to Tq = 13.2 N-m.

c.) Assume that you can control the clamping pressure so that as the machine slows down the
maximum temperature is always kept at the maximum allowable temperature, 750°C. Plot
the torque as a function of rotational speed for 100 rev/min to 3600 rev/min.

A parametric table is created that includes the variables N and Tq,; N is varied from 100 rev/min
to 3600 rev/min. The results are shown in Figure 5. Notice that it is possible to dramatically
improve the performance of the brake if you can adjust the clamping pressure with speed.

Figure 5: Clamping pressure and torque as a function of rotational velocity.


Problem 1.8-5 (1-18 in text): Optimizing a Fin
Figure P1.8-5 illustrates a fin that is to be used in the evaporator of a space conditioning system
for a space-craft.

h = 120 W/m -K
2

x th = 1 mm
T∞ = 20°C
L = 2 cm k = 50 W/m-K
ρ = 3000 kg/m3

ρb = 8000 kg/m3 Tb = 10°C

thb = 2 mm thg = 2 mm

Wb = 1 cm
Figure P1.8-5: Fin on an evaporator.

The fin is a plate with a triangular shape. The thickness of the plate is th = 1 mm and the width
of the fin at the base is Wb = 1 cm. The length of the fin is L = 2 cm. The fin material has
conductivity k = 50 W/m-K. The average heat transfer coefficient between the fin surface and
the air in the space-craft is h = 120 W/m2-K. The air is at T∞ = 20°C and the base of the fin is at
Tb = 10°C. Assume that the temperature distribution in the fin is 1-D in x. Neglect convection
from the edges of the fin.
a.) Obtain an analytical solution for the temperature distribution in the fin. Plot the temperature
as a function of position.

The inputs are entered in EES:

$UnitSystem SI MASS RAD PA K J


$TABSTOPS 0.2 0.4 0.6 0.8 3.5 in

h_bar=120 [W/m^2-K] "average heat transfer coefficient"


k=50 [W/m-K] "conductivity"
T_infinity=converttemp(C,K,20[C]) "air material"
T_b=converttemp(C,K,10[C]) "base temperature"
th_mm= 1 [mm] "fin thickness in mm"
th=th_mm*convert(mm,m) "fin thickness"
L_cm=2 [cm] "fin length in cm"
L=L_cm*convert(cm,m) "fin length"
W_b=1 [cm]*convert(cm,m) "fin base width"

The differential control volume shown in Figure P1.8-5-2 is used to derive the governing
differential equation:

q x = q x + dx + qconv (1)
x
qx
qconv
q x + dx

Figure P1.8-5-2: Differential control volume.

The rate of conduction and convection are:

dT
q x = − k Ac (2)
dx

qconv = h per (T − T∞ ) dx (3)

where Ac is the cross-sectional area for conduction and per is the perimeter. The width of the fin
is a function of x:

x
W = Wb (4)
L

Therefore, Ac and per are:

x
Ac = Wb th (5)
L

x
per = 2Wb (6)
L

Substituting Eqs. (5) and (6) into Eq. (2) and (3) leads to:

x dT
q x = − k Wb th (7)
L dx

x
qconv = h 2Wb (T − T∞ ) dx (8)
L

Substituting Eqs. (7) and (8) into Eq. (1) leads to:

d ⎡ x dT ⎤ x
0= ⎢ − k Wb th ⎥ dx + h 2Wb (T − T∞ ) dx (9)
dx ⎣ L dx ⎦ L

Simplifying:
d ⎛ dT ⎞
⎟ − m xT = −m xT∞
2 2
⎜x (10)
dx ⎝ dx ⎠

where

2h
m2 = (11)
k th

m=sqrt(2*h_bar/(k*th)) "solution parameter"

Maple is used to identify the solution to Eq. (10):

> restart;
> ODE:=diff(x*diff(T(x),x),x)-m^2*x*T(x)=-m^2*x*T_infinity;
2
d ⎛d ⎞
ODE := ⎛⎜⎜ T( x ) ⎞⎟⎟ + x ⎜⎜ 2 T( x ) ⎟⎟ − m 2 x T( x ) = −m 2 x T_infinity
⎝ dx ⎠ ⎝ dx ⎠
> Ts:=dsolve(ODE);
Ts := T( x ) = BesselI( 0, m x ) _C2 + BesselK( 0, m x ) _C1 + T_infinity

Therefore:

T = C2 BesselI ( 0, m x ) + C1 BesselK ( 0, m x ) + T∞ (12)

The fin temperature at the tip must be bounded:

Tx =0 = C2 BesselI ( 0, m 0 ) + C1 BesselK ( 0, m 0 ) + T∞ < ∞ (13)





1 ∞

The limit of the 0th order modified Bessel functions as x → 0 are evaluated using Maple:

> limit(BesselI(0,m*x),x=0);
1
> limit(BesselK(0,m*x),x=0);

Therefore, C1 must be zero:

T = C2 BesselI ( 0, m x ) + T∞ (14)

The base temperature is specified; therefore:


Tb = C2 BesselI ( 0, m L ) + T∞ (15)

so:

C2 =
(Tb − T∞ ) (16)
BesselI ( 0, m L )

Substituting Eq. (16) into Eq. (14) leads to:

BesselI ( 0, m x )
T = (Tb − T∞ ) + T∞ (17)
BesselI ( 0, m L )

x_bar=0.5 [-] "dimensionless position"


x=x_bar*L "position"
T=(T_b-T_infinity)*BesselI(0,m*x)/BesselI(0,m*L)+T_infinity "temperature"
T_C=converttemp(K,C,T) "in C"

Figure P1.8-5-3 illustrates the temperature as a function of position normalized by the fin length.
14

13.5

13
Temperature (°C)

12.5

12

11.5

11

10.5

10
0 0.1 0.2 0.3 0.4 0.5 0.6 0.7 0.8 0.9 1
Normalized position, x/L
Figure P1.8-5-3: Fin temperature as a function of dimensionless position.

b.) Calculate the rate of heat transfer to the fin.

The rate of heat transfer to the fin is computed according to:

dT
q fin = k Wb th (18)
dx x= L

Equation (18) is evaluated using Maple:

> restart;
> T:=(T_b-T_infinity)*BesselI(0,m*x)/BesselI(0,m*L)+T_infinity;
( T_b − T_infinity ) BesselI( 0, m x )
T := + T_infinity
BesselI( 0, m L )
> q_dot_fin=k*W_b*th*eval(diff(T,x),x=L);
k W_b th ( T_b − T_infinity ) BesselI( 1, m L ) m
q_dot_fin =
BesselI( 0, m L )

Therefore:

BesselI (1, m L )
q fin = k Wb th m (Tb − T∞ ) (19)
BesselI ( 0, m L )

q_dot_fin=k*W_b*th*(T_b-T_infinity)*m*BesselI(1,m*L)/BesselI(0,m*L) "fin heat transfer rate"

which leads to q fin = -0.196 W (the heat transfer is negative because the base temperature is less
than the ambient temperature).

c.) Determine the fin efficiency.

The fin efficiency is defined according to:

q fin
η fin = (20)
h As (Tb − T∞ )

where As is the total surface area of the fin exposed to the fluid:

As = Wb L (21)

Substituting Eqs. (19) and (21) into Eq. (20) leads to:

k Wb th m (Tb − T∞ ) BesselI (1, m L )


η fin = (22)
h Wb L (Tb − T∞ ) BesselI ( 0, m L )

Substituting Eq. (11) into Eq. (22) and simplifying leads to:

k th 2 h BesselI (1, m L ) 2 th k BesselI (1, m L )


η fin = = (23)
h L k th BesselI ( 0, m L ) L 2 h BesselI ( 0, m L )
N
1/ m

or

2 BesselI (1, mL )
η fin = (24)
mL BesselI ( 0, mL )

eta_fin=2*BesselI(1,m*L)/(m*L*BesselI(0,m*L)) "fin efficiency"


which leads to ηfin = 0.8178. Figure P1.8-5-4 illustrates the fin efficiency as a function of the fin
parameter mL.
1

0.9

0.8

0.7

Fin efficiency
0.6

0.5

0.4

0.3
0 0.5 1 1.5 2 2.5 3 3.5 4 4.5 5
Fin parameter, mL
Problem P1.8-5-4: Fin efficiency as a function of the fin parameter, mL.

The fin has density ρ = 3000 kg/m3. The fin is installed on a base material with thickness thb = 2
mm and density ρb = 8000 kg/m3. The half-width between the gap between adjacent fins is thg =
2 mm. Therefore, the volume of the base material associated with each fin is thb Wb (th + 2 thg).
d.) Determine the ratio of the absolute value of the rate of heat transfer to the fin to the total
mass of material (fin and base material associated with the fin).

The additional inputs are entered in EES:

rho=3000 [kg/m^3] "density of fin material"


th_b=2 [mm]*convert(mm,m) "thickness of base material"
th_g=2 [mm]*convert(mm,m) "half-width of gap between adjacent fins"
rho_b=8000 [kg/m^3] "base material density"

The fin mass is given by:

Wb L
M fin = th ρ (25)
2

The mass of the associated base material is:

M b = Wb ( th + 2 thg ) thb ρb (26)

The ratio of rate of the fin heat transfer to mass is:

q fin q fin
= (27)
M (M fin + Mb )

M_fin=W_b*L*th*rho/2 "fin mass"


M_b=W_b*(th+2*th_g)*th_b*rho_b "mass of base material"
q\M=abs(q_dot_fin)/(M_fin+M_b) "ratio of heat transfer to mass"
which leads to q fin /M = 178.4 W/kg.

e.) Prepare a contour plot that shows the ratio of the heat transfer to the fin to the total mass of
material as a function of the length of the fin (L) and the fin thickness (th).

A parametric table is generated that contains the variables L_cm, th_mm and q\M and has 400
rows. The value of the variable L_cm is varied from 1 cm to 10 cm every 20 rows and the value
of th_mm is varied from 0.2 mm to 2 mm in increments of 20 rows. The table is run and used to
generate the contour plot shown in Figure P1.8-5-5.
2
139.3 115.9
1.8
1.6 127.6
162.7
1.4 151
Fin thickness (mm)

1.2 174.4
186.1
1
197.8
0.8
209.5 W/kg
0.6
0.4
0.2
0
1 2 3 4 5 6 7 8 9 10
Fin length (cm)
Figure P1.8-5-5: Contours of heat transfer per mass in the parameter space of fin length and thickness.

f.) What is the optimal value of L and th that maximizes the absolute value of the fin heat
transfer rate to the mass of material?

According to Figure P1.8-5-5, the optimal design is approximately L = 3.3 cm and th = 0.58 mm.
A more precise optimization can be carried out using EES' internal optimization feature.
Maximizing q fin /M by varying L and th leads to q fin /M = 209.6 W/kg at L = 3.25 cm and th =
0.56 mm.
Problem 1.9-3 (1-19 in text): Fiber optic bundle
A fiber optic bundle (FOB) is shown in Figure P1.9-3 and used to transmit the light for a
building application.

h = 5 W/m -K
2

T∞ = 20°C rout = 2 cm

q ′′ = 1x10 W/m
5 2

x
fiber optic bundle
Figure P1.9-3: Fiber optic bundle used to transmit light.

The fiber optic bundle is composed of several, small diameter fibers that are each coated with a
thin layer of polymer cladding and packed in approximately a hexagonal close-packed array. The
porosity of the FOB is the ratio of the open area of the FOB face to its total area. The porosity of
the FOB face is an important characteristic because any radiation that does not fall directly upon
the fibers will not be transmitted and instead contributes to a thermal load on the FOB. The
fibers are designed so that any radiation that strikes the face of a fiber is “trapped” by total
internal reflection. However, radiation that strikes the interstitial areas between the fibers will
instead be absorbed in the cladding very close to the FOB face. The volumetric generation of
thermal energy associated with this radiation can be represented by:
φ q ′′ ⎛ x ⎞
g ′′′ = exp ⎜ − ⎟
Lch ⎝ Lch ⎠
where q ′′ = 1x105 W/m2 is the energy flux incident on the face, φ = 0.05 is the porosity of the
FOB, x is the distance from the face, and Lch = 0.025 m is the characteristic length for absorption
of the energy. The outer radius of the FOB is rout = 2 cm. The face of the FOB as well as its
outer surface are exposed to air at T∞ = 20°C with heat transfer coefficient h = 5 W/m2-K. The
FOB is a composite structure and therefore conduction through the FOB is a complicated
problem involving conduction through several different media. Section 2.9 discusses methods
for computing the effective thermal conductivity for a composite. The effective thermal
conductivity of the FOB in the radial direction is keff,r = 2.7 W/m-K. In order to control the
temperature of the FOB near the face where the volumetric generation of thermal energy is
largest, it has been suggested that high conductivity filler material be inserted in the interstitial
regions between the fibers. The result of the filler material is that the effective conductivity of
the FOB in the axial direction varies with position according to:
⎛ x ⎞
keff , x = keff , x ,∞ + Δkeff , x exp ⎜ − ⎟
⎝ Lk ⎠
where keff,x,∞ = 2.0 W/m-K is the effective conductivity of the FOB in the x-direction without
filler material, Δkeff,x = 28 W/m-K is the augmentation of the conductivity near the face, and Lk =
0.05 m is the characteristic length over which the effect of the filler material decays. The length
of the FOB is effectively infinite.
a.) Is it appropriate to use a 1-D model of the FOB?
The inputs are entered in EES and functions are defined to return the volumetric generation and
effective conductivity in the x-direction:

$UnitSystem SI MASS RAD PA K J


$TABSTOPS 0.2 0.4 0.6 0.8 3.5 in

function k_FOB(x)
k_eff_x_infinity=2 [W/m-K] "conductivity far from the face"
L_k=0.05 [m] "characteristic length of elevated conductivity"
Dk_eff_x=28 [W/m-K] "conductivity elevation at the face due to filler material"
k_FOB=k_eff_x_infinity+Dk_eff_x*exp(-x/L_k) "conductivity"
end

function gv_FOB(x)
phi=0.05 [-] "porosity"
q``=1e5 [W/m^2] "incident heat flux"
L_ch=0.025 [m] "characteristic length for absorption"
gv_FOB=phi*q``*exp(-x/L_ch)/L_ch "volumetric rate of thermal energy generation"
end

"Inputs"
k_eff_r=2.7 [W/m-K] "effective conductivity in the radial direction"
r_out=2 [cm]*convert(cm,m) "radius of FOB"
h_bar=5 [W/m^2-K] "heat transfer coefficient"
T_infinity=converttemp(C,K,20[C]) "ambient temperature"

A Biot number is defined according to:

h rout
Bi = (1)
keff , r

which leads to Bi = 0.037, justifying an extended surface model of the FOB.

b.) Assume that your answer to (a) was yes. Develop a numerical model of the FOB.

Nodes are positioned along the FOB. The FOB is infinitely long; however, the first L = 0.75 m
of the bundle is simulated. Examination of the solution shows that this is sufficient to capture
the end effects.

L=0.75 [m] "length of FOB to simulate"


N=41 [-] "number of nodes"
Dx=L/(N-1) "distance between adjacent nodes"
duplicate i=1,N
x[i]=Dx*(i-1) "position of each node"
end

An energy balance on node 1 leads to:

Δx π rout
2
2 Δx
h π r (T∞ − T1 ) + h 2 π rout
2
out (T∞ − T1 ) + keff , x , x =( x1 + x2 ) / 2 (T2 − T1 ) + g ′′′x = x1 π rout =0 (2)
2 Δx 2
h_bar*pi*r_out^2*(T_infinity-T[1])+h_bar*2*pi*r_out*(Dx/2)*(T_infinity-T[1])+&
pi*r_out^2*k_FOB((x[1]+x[2])/2)*(T[2]-T[1])/Dx+gv_FOB(x[1])*pi*r_out^2*Dx/2=0
"energy balance on node 1"

Energy balances on the internal nodes lead to:

π rout
2
h 2 π rout Δx (T∞ − Ti ) + keff , x , x =( xi + xi+1 ) / 2 (Ti +1 − Ti ) +
Δx
π rout
2
keff , x , x =( xi + xi−1 ) / 2 (Ti −1 − Ti ) + g ′′′x = xi π rout
2
Δx = 0 (3)
Δx
i = 2.. ( N − 1)

duplicate i=2,(N-1)
h_bar*2*pi*r_out*Dx*(T_infinity-T[i])+pi*r_out^2*k_FOB((x[i]+x[i+1])/2)*(T[i+1]-T[i])/Dx+&
pi*r_out^2*k_FOB((x[i]+x[i-1])/2)*(T[i-1]-T[i])/Dx+gv_FOB(x[i])*pi*r_out^2*Dx=0
"energy balance on internal nodes"
end

The temperature of the last node is taken to be specified at the ambient temperature:

TN = T∞ (4)

T[N]=T_infinity "node N temperature is specified"

The temperature is converted to Celsius:

duplicate i=1,N
T_C[i]=converttemp(K,C,T[i]) "temperature in C"
end

Figure 2 illustrates the temperature distribution within the FOB.


140

120
without filler material

Temperature (°C)
100

80

with filler material


60

40

20
0 0.1 0.2 0.3 0.4 0.5 0.6 0.7 0.8
Position (m)
Figure 2: Temperature distribution within the FOB for the case where the filler material is filler material is
present (Δkeff,x = 28 W/m-K) and the case where no filler material is present (Δkeff,x = 0).

c.) Overlay on a single plot the temperature distribution within the FOB for the case where the
filler material is present (Δkeff,x = 28 W/m-K) and the case where no filler material is present
(Δkeff,x = 0).

Figure 2 shows the case where filler material is present and is removed. The reduction in the
maximum temperature related to the addition of the filler material is evident in Figure 2.
Problem 1.9-4 (1-20 in text)
An expensive power electronics module normally receives only a moderate current. However,
under certain conditions it is possible that it might experience currents in excess of 100 amps.
The module cannot survive such a high current and therefore, you have been asked to design a
fuse that will protect the module by limiting the current that it can experience, as shown in
Figure P1.9-4.

L = 2.5 cm
ε = 0.9

Tend = 20°C Tend = 20°C


D = 0.9 mm
T∞ = 5°C
h = 5 W/m -K
2
k = 150 W/m-K
ρr = 1x10-7 ohm-m
I = 100 amp
Figure 1.9-4: A fuse that protects a power electronics module from high current.

The space available for the fuse allows a wire that is L = 2.5 cm long to be placed between the
module and the surrounding structure. The surface of the fuse wire is exposed to air at T∞ =
20°C and the heat transfer coefficient between the surface of the fuse and the air is h = 5.0
W/m2-K. The fuse surface has an emissivity of ε = 0.90. The fuse is made of an aluminum alloy
with conductivity k = 150 W/m-K. The electrical resistivity of the aluminum alloy is ρe = 1x10-7
ohm-m and the alloy melts at approximately 500°C. Assume that the properties of the alloy do
not depend on temperature. The ends of the fuse (i.e., at x=0 and x=L) are maintained at Tend
=20°C by contact with the surrounding structure and the module. The current passing through
the fuse, I, results in a uniform volumetric generation within the fuse material. If the fuse
operates properly, then it will melt (i.e., at some location within the fuse, the temperature will
exceed 500°C) when the current reaches 100 amp. Your job will be to select the fuse diameter;
to get your model started you may assume a diameter of D = 0.9 mm. Assume that the
volumetric rate of thermal energy generation due to ohmic dissipation is uniform throughout the
fuse volume.
a.) Prepare a numerical model of the fuse that can predict the steady state temperature
distribution within the fuse material. Plot the temperature as a function of position within the
wire when the current is 100 amp and the diameter is 0.9 mm.

The input parameters are entered in EES and the volumetric generation rate is computed:

$UnitSystem SI MASS RAD PA K J


$TABSTOPS 0.2 0.4 0.6 0.8 3.5 in

"Inputs"
L=2.5 [cm]*convert(cm,m) "length"
d=0.9 [mm]*convert(mm,m) "diameter"
T_a=converttemp(C,K,20) "air temperature"
T_end=converttemp(C,K,20) "end temperature"
h=5.0 [W/m^2-K] "heat transfer coefficient"
e=0.90 [-] "emissivity"
k=150 [W/m-K] "conductivity"
er=1e-7 [ohm-m] "electrical resistivity"
T_melt=converttemp(C,K,500) "melting temperature"
current=100 [amp] "current"

"Volumetric generation"
Ac=pi*d^2/4 "cross-sectional area"
Rst=er*L/Ac "resistance"
w_dot_ohmic=current^2*Rst "total dissipation"
g```_dot=w_dot_ohmic/(Ac*L) "volumetric rate of generation"

The appropriate Biot number for this case is:

hd
Bi = (1)
2k

The Biot number is calculated according to:

"Extended surface approximation"


Bi=h*d/(2*k)

The Biot number calculated by EES is much less than 1.0 and therefore the extended surface
approximation is justified.

The development of the numerical model follows the same steps that were previously discussed
in the context of numerical models of 1-D geometries. Nodes (i.e., locations where the
temperature will be determined) are positioned uniformly along the length of the rod. The
location of each node (xi) is:

xi =
( i − 1) L i = 1..N (2)
( N − 1)
where N is the number of nodes used for the simulation. The distance between adjacent nodes
(Δx) is:

L
Δx = (3)
( N − 1)
This distribution is entered in EES:

"Setup nodes"
N=10 [-] "number of nodes"
duplicate i=1,N
x[i]=(i-1)*L/(N-1) "position of nodes"
end
Dx=L/(N-1) "distance between nodes"
A control volume is defined around each node; the control surface bisects the distance between
the nodes. The control volume shown in Fig. 2 is subject to conduction heat transfer at each
edge ( qtop and qbottom ), convection ( qconv ), radiation ( qrad ), and generation ( g ). The energy
balance is:

qtop + qbottom + qconv + qrad + g = 0 (4)

The conduction terms are approximated as:

kπ d2
qtop = (Ti −1 − Ti ) (5)
4 Δx

kπ d2
qbottom = (Ti +1 − Ti ) (6)
4 Δx

The convection term is modeled according to:

qconv = h π d Δx (Ta − Ti ) (7)

The radiation term is:

qrad = ε σ π d Δx (Ta4 − Ti 4 ) (8)

The generation term is:

d2
g = g ′′′ π Δx (9)
4

Substituting Eqs. (5) through (9) into Eq. (4) leads to:

kπ d2 kπ d2
(Ti −1 − Ti ) + (Ti +1 − Ti ) + ha π d Δx (Ta − Ti )
4 Δx 4 Δx
(10)
d2
+ε σ π d Δx (Ta4 − Ti 4 ) + g ′′′ π Δx = 0 for i = 2.. ( N − 1)
4

The nodes at the edges of the domain must be treated separately; the temperature at both edges of
the fuse are specified:

T1 = Tend (11)

TN = Tend (12)
Equations (10) through (12) are a system of N equations in an equal number of unknown
temperatures which are entered in EES:

"Numerical solution"
T[1]=T_end
T[N]=T_end
duplicate i=2,(N-1)
k*pi*d^2*(T[i-1]-T[i])/(4*dx)+k*pi*d^2*(T[i+1]-T[i])/(4*dx)+pi*d*dx*h*(T_a-
T[i])+pi*d*dx*e*sigma#*(T_a^4-T[i]^4)+g```_dot*pi*d^2*dx/4=0
end
duplicate i=1,N
T_C[i]=converttemp(K,C,T[i])
end

Figure 2 illustrates the temperature distribution in the fuse for N = 100 nodes.

Figure 2: Temperature distribution in the fuse.

b.) Verify that your model has numerically converged by plotting the maximum temperature in
the wire as a function of the number of nodes in your model.

With any numerical simulation it is important to verify that a sufficient number of nodes have
been used so that the numerical solution has converged. The key result of the solution is the
maximum temperature in the wire, which can be obtained using the MAX command:

T_max_C=max(T_C[1..N])

Figure 3 illustrates the maximum temperature as a function of the number of nodes and shows
that the solution has converged for N greater than 100 nodes.
Figure 3: Maximum temperature as a function of the number of nodes.

c.) Prepare a plot of the maximum temperature in the wire as a function of the diameter of the
wire for I=100 amp. Use your plot to select an appropriate fuse diameter.

The number of nodes was set to 100 and the plot shown in Figure 4 was generated:

Figure 4: Maximum temperature as a function of diameter.

The maximum temperature reaches 500°C when the diameter is approximately 1.15 mm; this
would provide a fuse that correctly limited the current.
Problem 2.1-1 (2-1 in text): Buried Tubes
Figure P2.1-1 illustrates two tubes that are buried in the ground behind your house that transfer
water to and from a wood burner. The left hand tube carries hot water from the burner back to
your house at Tw,h = 135°F while the right hand tube carries cold water from your house back to
the burner at Tw,c = 70°F. Both tubes have outer diameter Do = 0.75 inch and thickness th =
0.065 inch. The conductivity of the tubing material is kt = 0.22 W/m-K. The heat transfer
coefficient between the water and the tube internal surface (in both tubes) is hw = 250 W/m2-K.
The center to center distance between the tubes is w = 1.25 inch and the length of the tubes is L =
20 ft (into the page). The tubes are buried in soil that has conductivity ks = 0.30 W/m-K.
ks = 0.30 W/m-K

Tw,c = 70°F
th = 0.065 inch kt = 0.22 W/m-K
hw = 250 W/m -K
2

Tw,h = 135°F
hw = 250 W/m -K
2

w = 1.25 inch
Do = 0.75 inch
Figure P2.1-1: Tubes buried in soil.

a.) Estimate the heat transfer from the hot water to the cold water due to their proximity to one
another.

The inputs are entered in EES:

$UnitSystem SI MASS RAD PA K J


$TABSTOPS 0.2 0.4 0.6 0.8 3.5 in

D_o=0.75 [inch]*convert(inch,m) "outer diameter of tube"


th=0.065 [inch]*convert(inch,m) "thickness of tube"
T_hw=converttemp(F,K,135) "hot water temperature"
T_cw=converttemp(F,K,70) "cold water temperature"
L=20 [ft]*convert(ft,m) "length of tubes"
w=1.25 [inch]*convert(inch,m) "center to center distance"
k_t=0.22 [W/m-K] "conductivity of teflon"
k_s=0.30 [W/m-K] "conductivity of sand"
h_w=250 [W/m^2-K]
"heat transfer coefficient between the water and tube inner surfaces"

The heat transfer is resisted by convection within the tube, conduction through the tube, and
conduction in the soil. The convection resistance is calculated according to:

1
Rconv = (1)
hw L π ( Do − 2 th )

The conduction resistance associated with the tube is calculated according to:
⎛ Do ⎞
ln ⎜ ⎟
⎝ Do − 2 th ⎠
Rcond = (2)
2 kt L π

R_conv=1/(h_w*L*pi*(D_o-2*th)) "convection resistance"


R_cond=ln(D_o/(D_o-2*th))/(2*pi*k_t*L) "conduction resistance"

The shape factor for parallel, buried tubes (SF) is obtained using EES’ internal library and used
to compute the resistance due to conduction through the soil:

1
Rtubetotube = (3)
SF ks

SF=SF_4(D_o,D_o,w,L) "shape factor"


R_tubetotube=1/(SF*k_s) "tube to tube resistance"

The heat transfer rate is therefore:

Tw,h − Tw,c
q = (4)
2 Rconv + 2 Rcond + Rtubetotube

q_dot=(T_hw-T_cw)/(2*R_conv+2*R_cond+R_tubetotube) "heat transfer rate"

which leads to q = 137.3 W.

b.) To do part (a) you should have needed to determine a shape factor; calculate an approximate
value of the shape factor and compare it to the accepted value.

The shape factor can be thought of as the ratio of the effective area for conduction, Aeff, to the
effective length required for conduction, Leff. The effective area for conduction can be estimated
according to:

Aeff ≈ w L (5)

while the length for conduction is approximately:

Leff ≈ w − Do (6)

and the approximate value of the shape factor should be:

Aeff
SFapp ≈ (7)
Leff

A_eff=L*w "approximate area"


L_eff=(w-D_o) "approximate length"
SF_app=A_eff/L_eff "approximate shape factor"

The exact value of the shape factor returned by the function is SF = 17.4 m while the
approximate value is SFapp = 15.2 m; these are sufficiently close for a sanity check.

c.) Plot the rate of heat transfer from the hot water to the cold water as a function of the center to
center distance between the tubes.

The heat transfer rate is shown in Figure 2 as a function of the center to center distance between
the tubes.

Figure 2: Heat transfer rate as a function of the center to center distance between the tubes
Problem 2.1-2
Currently, the low-pressure steam exhausted from a steam turbine at the power plant is condensed
by heat transfer to cooling water. An alternative that has been proposed is to transport the steam via
an underground pipe to a large building complex and use the steam for space heating. You have
been asked to evaluate the feasibility of this proposal. The building complex is located 0.2 miles
from the power plant. The pipe is made of uninsulated PVC (thermal conductivity of 0.19 W/m-K)
with an inner diameter of 8.33 in and wall thickness 0.148 in. The pipe will be buried underground
at a depth of 4 ft in soil that has an estimated thermal conductivity of 0.5 W/m-K. The steam leaves
the power plant at 6.5 lbm/min, 8 psia with a 95% quality. The outdoor temperature is 5°F.
Condensate is returned to the power plant in a separate pipe as, approximately, saturated liquid at 8
psia. You may neglect the resistance due to convection to the air and the steam.
a.) Neglecting the inevitable pressure loss, estimate the state of the steam that is provided to the
building complex.

The inputs are entered in EES:

$UnitSytem SI K J Pa
$TabStops 0.2 0.4 3.5 in

"known information"
L=0.2 [mile]*convert(mile,m) "pipe length"
D_i=8.33 [in]*convert(in,m) "pipe inner diameter"
thk=0.148 [in]*convert(in,m) "pipe wall thickness"
k_soil=0.5 [W/m-K] "soil thermal conductivity"
k_PVC=0.19 [W/m-K] "PVC thermal conductivity"
T_air=convertTemp(F,K,5 [F]) "air temperature in K"
P_steam=8 [psia]*convert(psia,Pa) "steam pressure"
T_steam=T_sat(Steam,P=P_steam) "steam temperature in K"
W=4 [ft]*convert(ft,m) "distance of pipe below ground level"
m_dot=6.5 [lb_m/min]*convert(lb_m/min,kg/s) "mass flow rate of the steam"

The outer diameter of the pipe is calculated:

Do = Di + 2 th (1)

where Di is the inner diameter and th is the pipe thickness. The resistance to conduction through the
pipe is:

⎛D ⎞
ln ⎜ o ⎟
R pipe = ⎝ Di ⎠ (2)
2 π k PVC L

where kPVC is the conductivity of the PVC pipe and L is the length of the pipe. The resistance
between the pipe surface and the surface of the soil is obtained using the shape factor (S) obtained
using the function SF_2 in EES.

1
Rsoil = (3)
S k soil
The heat transfer from the pipe to the soil surface is:

q =
(Tsteam − Tair ) (4)
R pipe + Rsoil

where Tsteam is the temperature of the steam and Tair is the temperature of the air.

D_o=D_i+2*thk "pipe outer diameter"


R_pipe=ln(D_o/D_i)/(2*pi*k_pvc*L) "resistance to conduction through pipe wall"
R_soil=1/(k_soil*S) "resistance of soil"
S=SF_2(D_o,W,L) "shape factor for buried pipe"
q_dot_loss=(T_steam-T_air)/(R_pipe+R_soil) "heat transfer rate through pipe wall"

which leads to q = 107.3 kW. An energy balance on the steam leads to:

q = m ( is ,in − is ,out ) (5)

where is,in and is,out are the inlet and outlet enthalpies of the steam, respectively. The outlet enthalpy
is used to determine the outlet quality, xs,out:

i_s_in=enthalpy(Steam,P=p_steam,x=0.95) "enthalpy of inlet steam"


q_dot_loss=m_dot*(i_s_in-i_s_out) "energy balance on steam"
x_out=quality(Steam,P=p_steam,h=i_s_out) "quality of leaving steam"

which leads to xs,out = 0.673.

b.) Are the thermal losses experienced in the underground pipe transport process significant in your
opinion? Do you recommend insulating this pipe?

Yes, the losses are significant. The quality of the steam is reduced from 95% to 67.3% and
therefore you have lost approximately 30% of the heating content of the steam. The pipe should be
insulated.

c.) Provide a sanity check on the shape factor that you used to solve this problem.

The shape factor represents, approximately, the ratio of the cross-sectional area for conduction to
the length for conduction. A crude estimate of the shape factor for this problem is:

⎛ W⎞
π ⎜ Do + ⎟L
S app = ⎝ 2⎠
(6)
W

which leads to Sapp = 687.3 m; this is approximately in line with the value provided by the EES
function, S = 651.8 m.
Problem 2.1-3 (2-2 in text)

A solar electric generation system (SEGS) employs molten salt as both the energy transport and
storage fluid. The molten salt is heated to Tsalt = 500°C and stored in a buried semi-spherical
tank. The top (flat) surface of the tank is at ground level. The diameter of the tank before
insulation is applied Dt = 14 m. The outside surfaces of the tank are insulated with tins = 0.30 m
thick fiberglass having a thermal conductivity of kins = 0.035 W/m-K. Sand having a thermal
conductivity of ksand = 0.27 W/m-K surrounds the tank, except on its top surface. Estimate the
rate of heat loss from this storage unit to the Tair = 25°C surroundings. You may neglect the
resistance due to convection.

The inputs are entered in EES:

$UnitSytem SI K J Pa
$TabStops 0.2 0.4 3.5 in

"known information"
t_ins=0.3 [m] "insulation thickness"
D_t=14 [m] "tank diameter"
k_sand=0.27 [W/m-K] "sand thermal conductivity"
k_ins=0.035 [W/m-K] "insulation thermal conductivity"
T_air=convertTemp(C,K,25 [C]) "air temperature in K"
T_salt=converttemp(C,K,500 [C]) "salt temperature in K"

The resistance between the bottom (buried, hemispherical) surface of the tank and the air is due
to the conduction through the spherical shell of insulation

⎛ 1 1 ⎞
⎜ + ⎟
⎝ Dt / 2 ( Dt / 2 + tins ) ⎠
Rins ,bottom = (1)
2 π kins

and the sand:

1
Rsand = (2)
k sand S

where S is the shape factor, obtained using the EES function SF_22. The resistance from the top,
unburied surface of the tank is only due to conduction through the insulation:

tins
Rins ,top = (3)
D2
kins π t
4

The heat transfer from the bottom and top surfaces are calculated according to:
qbottom =
(Tsalt − Tair ) (4)
Rsoil + Rins ,bottom

and

qtop =
(Tsalt − Tair ) (5)
Rins ,top

The total heat loss is:

qtotal = qtop + qbottom (6)

R_ins_bottom=(1/(D_t/2)-1/(D_t/2+t_ins))/(2*pi*k_ins)
"resistance to conduction through insulation on bottom"
S=SF_22(D_t+2*t_ins) "shape factor"
R_sand=1/(k_sand*S) "resistance of sand"
R_ins_top=t_ins/(k_ins*pi*D_t^2/4)
"resistance to conduction through insulation on top"
q_dot_bottom=(T_salt-T_air)/(R_sand+R_ins_bottom) "heat loss from bottom surface of tank"
q_dot_top=(T_salt-T_air)/(R_ins_top) "heat loss from top surface of tank"
q_dot_total=q_dot_bottom+q_dot_top "total heat loss"

which leads to qtotal = 12.95 kW.


Problem 2.1-4
A square extrusion is L = 1 m long and has outer dimension W = 3 cm. There is a D = 1 cm
diameter hole aligned with the center of the extrusion. The material has conductivity k = 0.5
W/m-K. The external surface of the extrusion is exposed to air at Ta = 20ºC with heat transfer
coefficient ha = 50 W/m2-K. The inner surface of the extrusion is exposed to water at Tw = 80 ºC
with heat transfer coefficient hw = 150 W/m2-K.
a.) Determine the rate of heat transfer between the water and the air.

The inputs are entered in EES:

$UnitSystem SI MASS RAD PA K J


$Tabstops 0.2 0.4 0.6 3.5 in

L=1 [m]
D=1 [cm]*convert(cm,m) "diameter of hole"
W=3 [cm]*convert(cm,m) "width of extrusion"
k=0.5 [W/m-K] "conductivity"
h_bar_a=50 [W/m^2-K] "heat transfer coefficient on air-side"
T_a=converttemp(C,K,20 [C]) "air temperature"
h_bar_w=150 [W/m^2-K] "heat transfer coefficient on water-side"
T_w=converttemp(C,K,80 [C]) "water temperature"

The resistance to convection to air is:

1
Rconv ,a = (1)
ha 4W L

The resistance to convection to water is:

1
Rconv , w = (2)
hw π D L

R_conv_a=1/(h_bar_a*4*W*L) "air side convection resistance"


R_conv_w=1/(h_bar_w*pi*D*L) "water side convection resistance"

The shape factor between the inner and outer surfaces of the extrusion (SF) is obtained from the
SF_12 function in EES. The resistance to conduction is:

1
Rcond = (3)
SF k

The total rate of heat transfer is:

q =
(Tw − Ta ) (4)
Rconv ,a + Rcond + Rconv , w
SF=SF_12(D,W,L) "shape factor"
R_cond=1/(SF*k) "conduction resistance"
q_dot=(T_w-T_a)/(R_conv_a+R_cond+R_conv_w) "heat transfer rate"

which leads to q = 79.7 W.

b.) Carry out a sanity check on the value of the shape factor that you used in (a).

The shape factor can be thought of as the ratio of the effective area to the effective length for
conduction. The area for conduction is smallest at the inner surface and largest at the outer
surface. The approximate area can be taken to be the average of these two:

π D L + 4W L
Aapp = (5)
2

The effective length is largest from the surface of the hole to the corner and smallest from the
surface of the hole to the middle of the edge. The average of these values is used:

Lapp =

1 ⎢ (W − D )
+
( )
2W − D ⎤
⎥ (6)
2⎢ 2 2 ⎥
⎣ ⎦

The approximate shape factor is computed according to:

Aapp
SFapp = (7)
Lapp

A_app=(pi*D*L+4*W*L)/2 "approximate area for conduction"


L_app=((W-D)/2+(sqrt(2)*W-D)/2)/2 "approximate length for conduction"
SF_app=A_app/L_app "approximate shape factor"

which leads to SFapp = 5.78 m which compares well to the shape factor obtained using the EES
library, SF = 5.35 m.
$UnitSystem SI MASS RAD PA K J
$Tabstops 0.2 0.4 0.6 3.5 in

"Inputs"
th=1.5 [mm]*convert(mm,m) "thickness"
k=75 [W/m-K] "conductivity of collector plate"
e=1 [-] "emissivity of collector plate"
q``_s=900 [W/m^2] "solar flux"
T_infinity=converttemp(C,K,10[C]) "ambient temperature"
h_bar=5 [W/m^2-K] "heat transfer coefficient"
L=8 [cm]*convert(cm,m) "half-width between tubes"
T_w=converttemp(C,K,50 [C]) "water temperature"
W=1 [m] "per unit length of collector"

Nodes are placed along the length of the collector. Only the region from 0 < x < L is considered
due to the symmetry of the system. Therefore, the position of each node is:

L ( i − 1)
xi = for i = 1..N (8)
( N − 1)
The distance between adjacent nodes is:

L
Δx = (9)
( N − 1)
N=21 [-] "number of nodes"
Dx=L/(N-1)
duplicate i=1,N
x[i]=L*(i-1)/(N-1) "location of each node"
end

Energy balances on each of the internal nodes leads to:

q LHS ,i + q RHS ,i + qs ,i + qconv ,i + qrad ,i = 0 for i = 2.. ( N − 1) (10)

where

k W th
q LHS ,i = (Ti −1 − Ti ) (11)
Δx
k W th
q RHS ,i = (Ti +1 − Ti ) (12)
Δx

qs ,i = W Δx q s′′ (13)

qs ,i = W Δx h (T∞ − Ti ) (14)

qrad ,i = W Δx ε σ (T∞4 − Ti 4 ) (15)

"internal nodes"
duplicate i=2,(N-1)
q_dot_LHS[i]=k*W*th*(T[i-1]-T[i])/Dx "conduction from left hand side node"
q_dot_RHS[i]=k*W*th*(T[i+1]-T[i])/Dx "conduction from right hand side node"
q_dot_s[i]=W*Dx*q``_s "absorbed solar radiation"
q_dot_conv[i]=W*Dx*h_bar*(T_infinity-T[i]) "convection"
q_dot_rad[i]=W*Dx*e*sigma#*(T_infinity^4-T[i]^4) "radiation"
q_dot_LHS[i]+q_dot_RHS[i]+q_dot_s[i]+q_dot_conv[i]+q_dot_rad[i]=0 "energy balance"
end

The temperature of node 1 is assumed to be equal to the water temperature (neglecting any
resistance to convection on the water-side):

T1 = Tw (16)

An energy balance on node N leads to:

q LHS , N + qs , N + qconv , N + qrad , N = 0 (17)

where

k W th
q LHS , N = (TN −1 − TN ) (18)
Δx

W Δx
qs , N = q s′′ (19)
2

W Δx
qs , N = h (T∞ − TN ) (20)
2

W Δx
qrad , N = ε σ (T∞4 − TN4 ) (21)
2

"node 1"
T[1]=T_w
"node N"
q_dot_LHS[N]=k*W*th*(T[N-1]-T[N])/Dx "conduction from left hand side node"
q_dot_s[N]=W*Dx*q``_s/2 "absorbed solar radiation"
q_dot_conv[N]=W*Dx*h_bar*(T_infinity-T[N])/2 "convection"
q_dot_rad[N]=W*Dx*e*sigma#*(T_infinity^4-T[N]^4)/2 "radiation"
q_dot_LHS[N]+q_dot_s[N]+q_dot_conv[N]+q_dot_rad[N]=0 "energy balance"

The temperature distribution within the collector is shown in Figure 2:


333

331
Temperature (K)

329

327

325

323
0 0.02 0.04 0.06 0.08 0.1
Position (m)
Figure 2: Temperature as a function of position in the collector.

An energy balance on node 1 provides the rate of energy transfer to the water:

qwater = q RHS ,1 + qs ,1 + qconv ,1 + qrad ,1 (22)

where

k W th
q RHS ,1 = (T2 − T1 ) (23)
Δx

W Δx
qs ,1 = q s′′ (24)
2

W Δx
qs ,1 = h (T∞ − T1 ) (25)
2

W Δx
qrad ,1 = ε σ (T∞4 − T14 ) (26)
2

"node 1"
q_dot_RHS[1]=k*W*th*(T[2]-T[1])/Dx "conduction from right hand side node"
q_dot_s[1]=W*Dx*q``_s/2 "absorbed solar radiation"
q_dot_conv[1]=W*Dx*h_bar*(T_infinity-T[1])/2 "convection"
q_dot_rad[1]=W*Dx*e*sigma#*(T_infinity^4-T[1]^4)/2 "radiation"
q_dot_water=q_dot_RHS[1]+q_dot_s[1]+q_dot_conv[1]+q_dot_rad[1] "energy balance"

which leads to qwater = 28.9 W.

b.) Determine the efficiency of the collector; efficiency is defined as the ratio of the energy
delivered to the water to the solar energy incident on the collector.

The efficiency is calculated according to:

qwater
η= (27)
LW

eta=q_dot_water/(L*W*q``_s) "efficiency"

which leads to η = 0.402.

c.) Plot the efficiency as a function of the number of nodes used in the solution.

Figure 3 illustrates the efficiency as a function of the number of nodes and shows that at least
20-30 nodes are required for numerical convergence.
0.409

0.408

0.407

0.406
Efficiency (-)

0.405

0.404

0.403

0.402

0.401
2 10 100 225
Number of nodes
Figure 3: Efficiency as a function of the number of nodes.

d.) Plot the efficiency as a function of Tw - T∞. Explain your plot.

Figure 4 illustrates the efficiency as a function of the water-to-ambient temperature difference.


When the water temperature is low then the losses are low (but not zero, because the
temperature of the copper plate is elevated by conduction). As the water temperature increases,
the temperature of the plate increases and therefore the losses increase and efficiency drops.
The drop in efficiency is dramatic for this type of unglazed collector and therefore the collector
may be suitable for providing water heating for swimming pools (at low water temperature) but
probably is not suitable for providing domestic hot water (at high water temperature).
1

0.9

0.8

0.7

0.6

Efficiency
0.5

0.4

0.3

0.2

0.1

0
0 10 20 30 40 50 60 70 80
Water to ambient temperature difference (K)
Figure 4: Solar collector efficiency as a function of the water-to-ambient temperature difference.
Problem 2.1-5
A pipe carrying water for a ground source heat pump is buried horizontally in soil with
conductivity k = 0.4 W/m-K. The center of the pipe is W = 6 ft below the surface of the ground.
The pipe has inner diameter Di = 1.5 inch and outer diameter Do = 2 inch. The pipe is made of
material with conductivity kp = 1.5 W/m-K. The water flowing through the pipe has temperature
Tw = 35ºF with heat transfer coefficient hw = 200 W/m-K. The temperature of the surface of the
soil is Ts = 0ºF.
a.) Determine the rate of heat transfer between the water and the air per unit length of pipe.

The inputs are entered in EES:

$UnitSystem SI MASS RAD PA K J


$Tabstops 0.2 0.4 0.6 3.5 in

L=1 [m]
D_i=1.5 [inch]*convert(inch,m) "inner diameter of tube"
D_o=2.0 [inch]*convert(inch,m) "outer diameter of tube"
k_p=1.5 [W/m-K] "conductivity of plastic"
k=0.4 [W/m-K] "conductivity of soil"
W=6 [ft]*convert(ft,m) "depth of tube beneath soil"
T_s=converttemp(F,K,0 [F]) "temperature of soil surface"
T_w=converttemp(F,K,35 [F]) "water temperature"
h_bar_w=200 [W/m^2-K] "convection coefficient with water"

The resistance to convection to water is:

1
Rconv , w = (1)
hw π Di L

The resistance to conduction through the pipe is:

⎛D ⎞
ln ⎜ o ⎟
= ⎝ i⎠
D
Rcond , p (2)
2π k p L

R_conv_w=1/(h_bar_w*pi*D_i*L) "resistance to convection with water"


R_cond_p=ln(D_o/D_i)/(2*pi*k_p*L) "resistance to conduction through pipe"

The shape factor between the outer surface of the pipe and the surface of the soil (SF) is
obtained from the SF_2 function in EES. The resistance to conduction through the soil is:

1
Rcond , s = (3)
SF k

The total rate of heat transfer is:


q =
(Tw − Ta ) (4)
Rconv , w + Rcond , p + Rcond , s

SF=SF_2(D_o,W,L) "shape factor between outer surface of tube and surface"


R_cond_s=1/(SF*k) "resistance to conduction through soil"
q_dot=(T_w-T_s)/(R_conv_w+R_cond_p+R_cond_s) "rate of heat transfer"

which leads to q = 9.49 W/m.

b.) Plot the heat transfer as a function of the depth of the pipe.

Figure 1 illustrates the rate of heat transfer as a function of the depth of the pipe.
17

16

15
Rate of heat transfer (W)

14

13

12

11

10

7
0 1 2 3 4 5 6 7 8 9 10 11
Depth of pipe (m)
Figure 1: Rate of heat transfer as a function of the depth of the pipe (per unit length).

c.) Carry out a sanity check on the value of the shape factor that you used in (a).

The shape factor can be thought of as the ratio of the effective area to the effective length for
conduction. Assuming that the heat spreads at an angle of approximately 45º as it moves from
the pipe to the surface, the approximate area for conduction is:

Aapp = 2 W L (5)

The effective length is the average of the length from the pipe to the surface perpendicularly and
at 45º.

2W +W
Lapp = (6)
2

The approximate shape factor is computed according to:


Aapp
SFapp = (7)
Lapp

A_app=sqrt(2)*W*L "approximate area"


L_app=(sqrt(2)*W+W)/2 "approximate length"
SF_app=A_app/L_app "approximate shape factor"

which leads to SFapp = 1.17 m which compares well to the shape factor obtained using the EES
library, SF = 1.26 m.
Problem 2.2-1: Model of Welding Process
You are evaluating a technique for controlling the properties of welded joints by using
aggressive liquid cooling. Figure P2.2-1 illustrates a cut-away view of two plates that are being
welded together. Both edges of the plate are clamped and effectively held at temperatures Ts =
25°C. The top of the plate is exposed to a heat flux that varies with position x, measured from
joint, according to: qm′′ ( x ) = q ′′j exp ( − x / L j ) where q ′′j =1x106 W/m2 is the maximum heat flux
(at the joint, x = 0) and Lj = 2.0 cm is a measure of the extent of the heat flux. The back side of
the plates are exposed to aggressive liquid cooling by a jet of fluid at Tf = -35°C with h = 5000
W/m2-K. A half-symmetry model of the problem is shown in Figure P2.2-1. The thickness of
the plate is b = 3.5 cm and the width of a single plate is W = 8.5 cm. You may assume that the
welding process is steady-state and 2-D. You may neglect convection from the top of the plate.
The conductivity of the plate material is k = 38 W/m-K.
both edges are
clamped and held heat flux
at fixed temperature
joint

impingement cooling with liquid jets

qm′′
k = 38 W/m-K

W = 8.5 cm

y b = 3.5 cm Ts = 25°C

T f = −35°C
h = 500 W/m -K
2

Figure P2-2-1: Welding process and half-symmetry model of the welding process.

a.) Develop a separation of variables solution to the problem. Implement the solution in EES
and prepare a plot of the temperature as a function of x at y = 0, 1.0, 2.0, 3.0, and 3.5 cm.

The problem shown in Figure P2.2-1 is governed by the partial differential equation:

∂ 2T ∂ 2T
+ =0 (1)
∂x 2 ∂y 2

and has the boundary conditions:

∂T
=0 (2)
∂x x =0
Tx =W = Ts (3)

∂T
k = h (Ty =0 − T f ) (4)
∂y y =0

∂T
k = qm′′ (5)
∂y y =b

Notice that the problem has 3 non-homogeneous boundary conditions and cannot, as it is written,
be solved using separation of variables. However, the simple transformation:

θ = T − Ts (6)

transforms the boundary condition at x = W to a homogeneous boundary condition and provides


a homogeneous direction (x). The transformed differential equation and boundary conditions
are:

∂ 2θ ∂ 2θ
+ =0 (7)
∂x 2 ∂y 2

and has the boundary conditions:


=0 (8)
dx x =0

θ x =W = 0 (9)

∂θ
k = h (θ y =0 − θ f ) (10)
∂y y =0

∂θ
k = qm′′ (11)
∂y y =b

where

θ f = T f − Ts (12)

The separated solution is assumed:

T = TX TY (13)
and substituted into Eq. (7); the process leads to two ordinary differential equations:

d 2TX
2
+ λ 2 TX = 0 (14)
dx

d 2TY
2
− λ 2 TY = 0 (15)
dy

The solutions to Eqs. (14) and (15) are:

TX = C1 sin ( λ x ) + C2 cos ( λ x ) (16)

TY = C3 sinh ( λ y ) + C4 cosh ( λ y ) (17)

We will solve the eigenproblem first (i.e., the problem in the homogeneous direction). The
boundary condition at x = 0, Eq. (8), leads to:

dTX
= ⎡⎣C1 λ cos ( λ x ) − C2 λ sin ( λ x ) ⎤⎦ x =0 = C1 λ = 0 (18)
dx x =0

which indicates that C1 = 0 and therefore:

TX = C2 cos ( λ x ) (19)

The boundary condition at x = W, Eq. (9), leads to:

TX x =W = C2 cos ( λ W ) = 0 (20)

Equation (20) provides our eigenvalues:

λi =
(1 + 2 i ) π for i = 1..∞ (21)
2W

and the eigenfunctions are:

TX i = C2,i cos ( λi x ) (22)

The solution for TX is the sum of the eigenfunctions:


TX = ∑ C2,i cos ( λi x ) (23)
i =0
and the solution for the temperature is:


TX = ∑ C2,i cos ( λi x ) ⎡⎣C3,i sinh ( λi y ) + C4,i cosh ( λi y ) ⎤⎦ (24)
i =0

Note that C2,i can be absorbed in the other constants:


θ = ∑ cos ( λi x ) ⎡⎣C3,i sinh ( λi y ) + C4,i cosh ( λi y ) ⎤⎦ (25)
i =0

Equation (25) is substituted into the boundary condition at y = 0, Eq. (10):



k ∑ cos ( λi x ) ⎣⎡C3,i λi cosh ( λi 0 ) + C4,i λi sinh ( λi 0 ) ⎦⎤ =
i =0
(26)
⎧∞ ⎫
h ⎨∑ cos ( λi x ) ⎡⎣C3,i sinh ( λi 0 ) + C4,i cosh ( λi 0 ) ⎤⎦ − θ f ⎬
⎩ i =0 ⎭

or


⎡∞ ⎤
k ∑ cos ( λi x ) C3,i λi = h ⎢ ∑ cos ( λi x ) C4,i − θ f ⎥ (27)
i =0 ⎣ i =0 ⎦

We take advantage of the orthogonality of the eigenfunctions by multiplying Eq. (27) by an


arbitrary eigenfunction and integrating from x = 0 to x = L in order to achieve:

W
⎡ W W

k C3,i λi ∫ cos 2 ( λi x ) dx = h ⎢C4,i ∫ cos 2 ( λi x ) dx − θ f ∫ cos ( λ x ) dx ⎥⎦
i (28)
0 ⎣ 0 0

The integrals in Eq. (28) are evaluated using Maple:

> restart;
> assume(i,integer);
> lambda:=(1+2*i)*Pi/(2*W);
( 1 + 2 i~ ) π
λ :=
2W
> int((cos(lambda*x))^2,x=0..W);
W
2
> int(cos(lambda*x),x=0..W);
2 ( -1 ) i~ W
( 1 + 2 i~ ) π
which leads to:

k C3,i λi W ⎡ C4,i W 2 ( −1) W ⎤


i

= h⎢ −θ f ⎥ (29)
2 ⎢⎣ 2 (1 + 2 i ) π ⎥⎦

or

4 ( −1)
i

θf = C4,i − i C3,i (30)
(1 + 2 i ) π h

which is one equation for each of the two pairs of unknown coefficients.

Equations (25) and Error! Reference source not found. are substituted into the final boundary
condition at y = b, Eq. (11):

∞ ⎛ x ⎞
k ∑ cos ( λi x ) ⎡⎣C3,i λi cosh ( λi b ) + C4,i λi sinh ( λi b ) ⎤⎦ = q ′′j exp ⎜ −
⎟⎟
(31)
⎜ Lj
i =0 ⎝ ⎠

Again, the orthogonality of the eigenfunctions is used to obtain:

W W
⎛ x ⎞
k ⎡⎣C3,i λi cosh ( λi b ) + C4,i λi sinh ( λi b ) ⎤⎦ ∫ cos 2 ( λi x ) dx = q ′′j ∫ exp ⎜ − ⎟⎟ cos ( λi x ) dx (32)
⎜ Lj
0 0 ⎝ ⎠

or

2 q ′′
W
⎛ x ⎞
C3,i λi cosh ( λi b ) + C4,i λi sinh ( λi b ) = ∫ exp ⎜⎜ − L ⎟⎟ cos ( λi x ) dx (33)
Wkj 0 ⎝ j ⎠

which is the 2nd relationship between the coefficient pairs. The integral on the right hand side of
Eq. (32) is accomplished using Maple:

> int(exp(-x/L_j)*cos(lambda*x),x=0..W);
⎛ ⎞
⎛ ⎞ ⎛⎜⎜ − L_j ⎞⎟⎟
W W
⎜⎜ ⎟⎟
⎜⎜ ⎟ ⎝
+ π L_j ( -1 ) + 2 π L_j ( -1 ) i~ ⎟⎠ e
⎝ L_j ⎠ i~ i~ ⎠
2 L_j W ⎝ 2 W e
4 W2 + π 2 L_j2 + 4 π 2 L_j2 i~ + 4 π 2 L_j2 i~ 2

and pasted directly into EES in order to accomplish the implementation.

The inputs are entered in EES:

$UnitSystem SI MASS RAD PA K J


$TABSTOPS 0.2 0.4 0.6 0.8 3.5 in

"Inputs"
b=3.5 [cm]*convert(cm,m) "plate thickness"
W=8.5 [cm]*convert(cm,m) "plate width"
k=38 [W/m-K] "plate conductivity"
T_s=converttemp(C,K,25) "edge temperature"
T_f=converttemp(C,K,-35) "fluid temperature"
h=5000 [W/m^2-K] "heat transfer coefficient"
q``_j=1e6 [W/m^2] "heat flux at x=0"
L_j=2.0 [cm]*convert(cm,m) "extent of the heat flux"

and a position is specified:

"Position"
x_bar=0.5
x=x_bar*W
y_bar=0.5
y=y_bar*b

The solution is implemented::

N=10 [-] "number of terms"


theta_f=(T_f-T_s) "transformed fluid temperature"
duplicate i=0,N
lambda[i]=(1+2*i)*pi/(2*W)
theta_f*4*(-1)^i/((1+2*i)*pi)=C_4[i]-k*lambda[i]*C_3[i]/h
C_3[i]*lambda[i]*cosh(lambda[i]*b)+C_4[i]*lambda[i]*sinh(lambda[i]*b)=(2*q``_j/(W*k))*
(2*W*L_j*(2*W*exp(W/L_j)+Pi*L_j*(-1)^i+2*Pi*L_j*(-1)^i*i)*exp(-W/L_j)/(4*W^2+Pi^2*L_j^2
+4*Pi^2*L_j^2*i+4*Pi^2*L_j^2*i^2))
theta[i]=cos(lambda[i]*x)*(C_3[i]*sinh(lambda[i]*y)+C_4[i]*cosh(lambda[i]*y))
end
T=T_s+sum(theta[0..N])
T_C=converttemp(K,C,T)

Note that the bold portion of the code was pasted from Maple.

Figure 2 illustrates the temperature as a function of dimensionless axial position (x/W) for
various values of y.
Figure 2: Temperature as a function of x/W for various values of y.

b.) Prepare a contour plot of the temperature distribution.

Figure 3 illustrates a contour plot of the temperature distribution in the plate.

Figure 3: Contour plot of the temperature distribution in the plate.


Problem 2.2-2: Derive a Partial Differential Equation
Figure P2.2-2 illustrates a thin plate that is exposed to air on upper and lower surfaces. The heat
transfer coefficient between the top and bottom surfaces is h and the air temperature is Tf. The
thickness of the plate is th and its width and height are a and b, respectively. The conductivity of
the plate is k. The top edge is fixed at a uniform temperature, T1. The right edge is fixed at a
different, uniform temperature, T2. The left edge of the plate is insulated. The bottom edge of
the plate is exposed to a heat flux, q ′′ . This problem should be done on paper.

T1

a
T2
k
y
b
x

q ′′
th
h ,Tf

h ,T f
Figure P.2.2-2: Thin plate exposed to air.

a.) The temperature distribution within the plate can be considered 2-D (i.e., temperature
variations in the z-direction can be neglected) if the plate is thin and conductive. How would
you determine if this approximation is valid?

The Biot number characterizes the ratio of conduction in the z direction (which I want to neglect)
to convection from the plate surface (which I will consider):

Rcond , z th h a b th h
Bi = = = (1)
Rconv 2k ab 1 2k

b.) Derive the partial differential equation and boundary conditions that would need to be solved
in order to obtain an analytical solution to this problem.

A differential control volume has extend dx x dy and extends through the thickness of the plate,
as shown in Figure 2.
Figure 2: Differential control volume used to derive the governing partial differential equation.

The first law balance suggested by Figure 2 is:

q x + q y = q x + dx + q y + dy + qconv (2)

or

∂q x ∂q y
0= dx + dy + qconv (3)
∂x ∂y

Substituting the rate equations into the energy balance leads to:

∂ ⎡ ∂T ⎤ ∂ ⎡ ∂T ⎤
0= ⎢ −k th dy ⎥ dx + ⎢ − k th dx ⎥ dy + 2 dx dy h (T − T f ) (4)
∂x ⎣ ∂x ⎦ ∂y ⎣ ∂y ⎦

or

∂ 2T ∂ 2T 2 h
+ −
∂x 2 ∂y 2 k th
(T − T f ) = 0 (5)

The boundary conditions at the upper and right edges are:

Tx = a = T2 (6)

Ty =b = T1 (7)
The boundary condition at the left edge is obtained from an interface balance which requires that
the conduction heat flux in the x-direction at this insulated edge be zero:

q ′′x , x =0 = 0 (8)

or

∂T
−k =0 (9)
∂x x =0

which leads to:

∂T
=0 (10)
∂x x =0

The boundary condition at the bottom edge is also obtained from an interface energy balance:

∂T
q ′′ = −k (11)
∂y y =0

or

∂T q ′′
=− (12)
∂y y =0
k
Problem 2.2-3 (2-3 in text): Heat Transfer Coefficient Measurement Device
You are the engineer responsible for a simple device that is used to measure heat transfer
coefficient as a function of position within a tank of liquid (Figure P2.2-3). The heat transfer
coefficient can be correlated against vapor quality, fluid composition, and other useful quantities.
The measurement device is composed of many thin plates of low conductivity material that are
interspersed with large, copper interconnects. Heater bars run along both edges of the thin
plates. The heater bars are insulated and can only transfer energy to the plate; the heater bars are
conductive and can therefore be assumed to come to a uniform temperature as a current is
applied. This uniform temperature is assumed to be applied to the top and bottom edges of the
plates. The copper interconnects are thermally well-connected to the fluid; therefore, the
temperature of the left and right edges of each plate are equal to the fluid temperature. This is
convenient because it isolates the effect of adjacent plates from one another which allows each
plate to measure the local heat transfer coefficient. Both surfaces of the plate are exposed to the
fluid temperature via a heat transfer coefficient. It is possible to infer the heat transfer
coefficient by measuring heat transfer required to elevate the heater bar temperature a specified
temperature above the fluid temperature.

top and bottom surfaces exposed to fluid


T∞ = 20°C, h = 50 W/m -K
2

copper interconnet, T∞ = 20°C

a = 20 mm
b = 15 mm

plate: heater bar, Th = 40°C


k = 20 W/m-K
th = 0.5 mm
Figure P2.2-3: Device to measure heat transfer coefficient as a function of position.

The nominal design of an individual heater plate utilizes metal with k = 20 W/m-K, th = 0.5 mm,
a = 20 mm, and b = 15 mm (note that a and b are defined as the half-width and half-height of the
heater plate, respectively, and th is the thickness as shown in Figure P2-3). The heater bar
temperature is maintained at Th = 40ºC and the fluid temperature is T∞ = 20ºC. The nominal
value of the average heat transfer coefficient is h = 50 W/m2-K.
a.) Develop an analytical model that can predict the temperature distribution in the plate under
these nominal conditions.

The problem can be simplified and tackled using the quarter-symmetry model of a single plate,
shown in Figure P2.2-3, is specified mathematically in Figure 2(a).
(a)

(b)
Figure 2: Quarter symmetry model of a plate.

The lines of symmetry (x =0 and y = 0) are adiabatic, the top edge (y = b) is held at the heater
temperature (Th) and the right edge (x = a) is held at the fluid temperature (T∞). These boundary
conditions are expressed below:

∂T
=0
∂x x =0

Tx =a = T∞
∂T
=0
∂y y =0

Ty =b = Th

The differential control volume and associated first law balance are shown in Figure 2(a):

q x + q y = q x + dx + q y + dy + qconv

or, after expanding the x + dx and y + dy terms:

∂q x ∂q y
0= dx + dy + qconv
∂x ∂y

Substituting the rate equations into the energy balance leads to:

∂ ⎡ ∂T ⎤ ∂ ⎡ ∂T ⎤
0= −k th dy dx + ⎢ − k th dx dy + 2 dx dy h ( T − T∞ )

∂x ⎣ ⎥
∂x ⎦ ∂y ⎣ ∂y ⎥⎦

or

∂ 2T ∂ 2T 2 h
+ −
∂x 2 ∂y 2 k th
T − T∞ = 0 ( ) (1)

Equation (1) is not homogeneous; however, a transformation variable can be identified by


inspection:

θ = T − T∞ (2)

Substituting Eq. (2) into Eq. (1) transforms the governing partial differential equation into a
homogeneous equation; i.e., any multiple of θ will satisfy transformed partial differential
equation:

∂ 2θ ∂ 2θ
+ 2 − m2 θ = 0 (3)
∂x 2
∂y

where

2h
m=
k th
Substituting Eq. (2) into the boundary conditions leads to:

∂θ
=0 (4)
∂x x =0

θ x =a = 0 (5)

∂θ
=0 (6)
∂y y =0

θ y =b = θ h (7)

where

θ h = Th − T∞

The transformed problem is shown in Figure 2(b); note that the transformed problem is linear
and consists of a homogeneous partial differential equation and homogeneous boundary
conditions in the x-direction. Therefore, we can apply a separation of variables solution to the
problem and x is our homogeneous direction.

The solution is assumed to be the product of θX which is a function of x and θY which is a


function of y:

θ ( x, y ) = θ X ( x ) θ Y ( y )

Substituting the product solution into the differential equation leads to:

d 2θ X d 2θ Y
θY + θ X − m2 θ X θY = 0
dx 2
dy 2

Dividing by the product θX θY:

d 2θ X d θ Y
2

dx 2 + dy − m 2 = 0
2

X θ Y


−λ 2 λ2

Recall that x is our homogeneous direction; therefore, we need the θX group in the equation to
equal a negative constant (-λ2); the remaining part of the equation must equal the positive value
of the same constant (λ2). Therefore, the two ordinary differential equations that result from the
separation process are:

d 2θ X
+ λ2 θ X = 0 (8)
dx 2

and

d 2θ Y
− ( λ 2 + m2 )θY = 0 (9)
dy 2

The next step is to solve the eigenproblem; the solution to the ordinary differential equation for
θX is:

θ X = C1 cos ( λ x ) + C2 sin ( λ x ) (10)

The 1st boundary conditions in the homogeneous direction, Eq. (4), leads to:

dθ X
=0 (11)
dx x =0

Substituting Eq. (10) into Eq. (11) leads to:

dθ X
= −C1 λ sin ( λ 0 ) + C2 λ cos ( λ 0 ) = 0
dx x =0



=0 =1

which can only be true if C2 = 0. the 2nd boundary condition in the x-direction, Eq. (5), leads to:

θ x =a = C1 cos ( λ a ) = 0

which can only be true if the argument of the cosine is 0. Therefore, the eigenfunctions and
eigenvalues for the problem are:

θ X i = C1,i cos ( λi x ) where λi =


( 2 i − 1) π
2 a

The solution to the ordinary differential equation in the non-homogeneous direction, Eq. (9) can
be obtained using Maple:

> restart;
> assume((lambda^2+m^2),positive);
> ODEy:=diff(diff(thetaY(y),y),y)-(lambda^2+m^2)*thetaY(y)=0;
⎛d ⎞
2
ODEy := ⎜ 2 thetaY( y ) ⎟⎟ − ( λ∼ 2 + m~ 2 ) thetaY( y ) = 0
⎜ dy
⎝ ⎠
> qYs:=dsolve(ODEy);
( λ∼ 2 + m~2 y ) ( − λ∼2 + m~2 y )
qYs := thetaY( y ) = _C1 e + _C2 e

Note that Maple can convert this exponential form to an equivalent form involving hyperbolic
sines and cosines with the convert command.

> convert(qYs,'trigh');
thetaY( y ) = ( _C1 + _C2 ) cosh( λ∼ 2 + m~ 2 y ) + ( _C1 − _C2 ) sinh( λ∼ 2 + m~ 2 y )

θ Yi = C3,i cosh ( )
λi2 + m 2 y + C4,i sinh ( λi2 + m 2 y )
The solution associated with the ith eigenvalue is therefore:

θi = θ X i θ Yi = cos ( λi x ) ⎡⎢C3,i cosh


⎣ ( )
λi2 + m 2 y + C4,i sinh ( )
λi2 + m 2 y ⎤⎥

(12)

where the constant C1,i is absorbed into the constants C3,i and C4,i. It is good practice to verify
that Eq. (12) satisfies both boundary conditions in the x-direction and the partial differential
equation for any value of i:

> restart;
> assume(i,integer);
> lambda:=(2*i-1)*Pi/(2*a);
( 2 i~ − 1 ) π
λ :=
2a
> theta:=(x,y)->cos(lambda*x)*(C_3*cosh(sqrt(lambda^2+m^2)*y)+C_4*sinh(sqrt(lambda^2+m^2)*y));
θ := ( x, y ) → cos( λ x ) ( C_3 cosh( λ 2 + m 2 y ) + C_4 sinh( λ 2 + m 2 y ) )
> eval(diff(theta(x,y),x),x=0);
0
> theta(a,y);
0
> diff(diff(theta(x,y),x),x)+diff(diff(theta(x,y),y),y)-m^2*theta(x,y);
( 2 i~ − 1 ) π x ⎞
− cos⎛⎜⎜
1
⎟⎟ ( 2 i~ − 1 ) π
2 2
4 ⎝ 2a ⎠
⎛ ⎛ ( 2 i~ − 1 ) 2 π 2 ⎞ ⎛ ( 2 i~ − 1 ) 2 π 2 ⎞⎞
⎜ ⎜ + 4 m 2 y ⎟⎟ ⎜ + 4 m 2 y ⎟⎟ ⎟⎟
⎜ ⎜ ⎜
⎜ ⎜ a 2
⎟ ⎜ a 2
⎟⎟
⎜⎜ C_3 cosh⎜⎜ ⎟⎟ + C_4 sinh⎜⎜ ⎟⎟ ⎟⎟
⎝ ⎝ 2 ⎠ ⎝ 2 ⎠⎠


− π ⎜
a 2 + cos⎛⎜⎜
( 2 i~ 1 ) x ⎞ ⎜⎜
⎟⎟ ⎜
⎝ 2a ⎠⎝
⎛ ( 2 i~ − 1 ) π 2 2

⎜ + 4 m 2 y ⎟⎟

1 ⎜ a 2
⎟ ⎛ ( 2 i~ − 1 ) 2 π 2 ⎞
C_3 cosh⎜ ⎜ ⎟⎟ ⎜ + 4 m 2 ⎟⎟
4 ⎝ 2 ⎠⎝⎜ a 2

⎛ ( 2 i~ − 1 ) π
2 2
⎞ ⎞
⎜ + m 2
y ⎟ ⎟
⎜ 4 ⎟ ⎟
1 ⎜ a2 ⎟ ⎛ ( 2 i~ − 1 ) 2 π 2 ⎞ ⎟
+ C_4 sinh⎜⎜ ⎟⎟ ⎜
⎜ + 4 m ⎟⎟ ⎟⎟ − m 2
2
⎝ ⎠⎝ a
4 2 2
⎠⎠
i~ − π x
cos⎛⎜⎜ ⎞
( 2 1 )
⎟⎟
⎝ 2 a ⎠
⎛ ⎛ ( 2 i~ − 1 )2 π2 ⎞ ⎛ ( 2 i~ − 1 ) 2 π 2 ⎞⎞
⎜ ⎜ + m 2
y ⎟ ⎜ + 4 m 2 y ⎟⎟ ⎟⎟
⎜ ⎜ 4 ⎟ ⎜
⎜ ⎜ a 2
⎟ ⎜ a 2
⎟⎟
⎜⎜ C_3 cosh⎜⎜ ⎟⎟ + C_4 sinh⎜⎜ ⎟⎟ ⎟⎟
⎝ ⎝ 2 ⎠ ⎝ 2 ⎠⎠
> simplify(%);
0

The general solution is the sum of the solution for each eigenvalue:

( ) ( )
∞ ∞
θ = ∑θi = ∑ cos ( λi x ) ⎡⎢C3,i cosh λi2 + m 2 y + C4,i sinh λi2 + m 2 y ⎤⎥ (13)
i =1 i =1 ⎣ ⎦

The general solution must satisfy the boundary conditions in the non-homogeneous direction.
Substituting Eq. (13) into Eq. (6) leads to:

⎡ ⎤
∂θ
( ) ( )

= ∑ cos ( λi x ) C3,i λi + m sinh λi + m 0 + C4,i λi + m cosh λi + m 0 ⎥ = 0
⎢ 2 2 2 2 2 2 2 2

∂y i =1
⎢ 


y =0
⎣⎢ =0 =1 ⎦⎥

or

∑C
i =1
4,i λi2 + m 2 cos ( λi x ) = 0

which can only be true if C4,i = 0:

( )

θ = ∑ Ci cosh λi2 + m 2 y cos ( λi x ) (14)
i =1
where the subscript 3 has been removed from C3,i because it is the only remaining undetermined
constant. Substituting Eq. (14) into Eq. (7) leads to:

( )

θ y =b = ∑ Ci cosh λi2 + m 2 b cos ( λi x ) = θ h
i =1

This equation is multiplied by cos(λj x) and integrated from 0 to a:

( )
∞ a a

∑ Ci cosh λi2 + m 2 b ∫ cos ( λi x ) cos ( λ j x ) dx = θ h ∫ cos ( λ j x ) dx


i =1 0 0

Orthogonality causes all of the terms in the summation to integrate to zero except for the one in
which i = j:

( ) ∫ cos (λ x ) dx = θ ∫ cos (λ x ) dx
a a
Ci cosh λ +m b
i
2 2 2
i h i
0 0

Maple is used to carry out the integrations:

> restart;
> assume(i,integer);
> lambda:=(2*i-1)*Pi/(2*a);
( 2 i~ − 1 ) π
λ :=
2a
> int((cos(lambda*x))^2,x=0..a);
a
2
> int(cos(lambda*x),x=0..a);
( 1 + i~ )
2 ( -1 ) a
( 2 i~ − 1 ) π

which leads to an equation for each of the constants.

θ h 4 ( −1)
1+ i

Ci =
cosh ( )
λi2 + m 2 b ( 2 i − 1) π

The inputs are entered in EES:

$UnitSystem SI MASS RAD PA C J


$Tabstops 0.2 0.4 0.6 0.8 3.5

"Inputs"
th_mm=0.5 [mm] "thickness of plate in mm"
th=th_mm * convert(mm,m) "thickness of plate"
k=20 [W/m-K] "conductivity of plate"
a_mm=20 [mm] "half-width of plate in mm"
a= a_mm * convert(mm,m) "half-width of plate"
b= 15 [mm] *convert(mm,m) "half-height of plate"
T_h = converttemp(C,K,40 [C]) "heater temperature"
T_infinity=converttemp(C,K,20 [C]) "fluid temperature"
h_bar=50 [W/m^2-K] "heat transfer coefficient"

The position is specified using dimensionless variables x_bar and y_bar:

"position"
x_bar=0.5
y_bar=0.5
x=x_bar*a
y=y_bar*b

The solution is evaluated:

theta_h=T_h-T_infinity "heater temperature difference"


m=sqrt(2*h_bar/(k*th)) "solution constant"
N=100 "number of terms"
duplicate i=1,N
lambda[i]=(2*i-1)*pi/(2*a) "eigenvalues"
C[i]=theta_h*(4*(-1)^(1+i)/(2*i-1)/Pi)/(cosh(sqrt(lambda[i]^2+m^2)*b))
theta[i]=C[i]*cos(lambda[i]*x)*cosh(sqrt(lambda[i]^2+m^2)*y)
end
T=sum(theta[1..N])+T_infinity
T_C=converttemp(K,C,T)

A parametric table is created and used to generate the contour plot shown in Figure 3.
Figure 3: Contour plot of temperature on the plate.

b.) The measured quantity is the rate of heat transfer to the plate from the heater ( qh ) and
therefore the relationship between qh and h (the quantity that is inferred from the heater
power) determines how useful the instrument is. Determine the heater power.

The heater power can be computed by integrating the conduction heat transfer along the top
surface according to:

∂θ
a
qh = 4 ∫ k th dx (15)
0
∂y y =b

where the factor of 4 comes from the quarter symmetry of the model. Equation (14) is
substituted into Eq. (15):

( ) ∫ cos (λ x ) dx
∞ a
qh = 4 k th ∑ Ci λi2 + m 2 sinh λi2 + m 2 b i
i =1 0

which leads to:

( )

qh = 2 k th a ∑ Ci λi2 + m 2 sinh λi2 + m 2 b (16)
i =1

Equation (16) is evaluated in EES:

"heater power"
duplicate i=1,N
q_dot_h[i]=2*k*th*a*C[i]*sqrt(lambda[i]^2+m^2)*sinh(sqrt(lambda[i]^2+m^2)*b)
end
q_dot_h=sum(q_dot_h[1..N])

c.) If the uncertainty in the measurement of the heater power is δ qh = 0.01 W, estimate the
uncertainty in the measured heat transfer coefficient ( δ h ).

A parametric table is used to explore the relationship between h and qhtr ; the two columns in the
table are the variables h_bar and q_dot_h. Figure 4 illustrates the heat transfer coefficient as a
function of the heater power, all else held constant.

Figure 4: Heat transfer coefficient as a function of the heater power.

Note that a good measuring device would show a strong relationship between the physical
quantity being measured ( h ) and the measurement ( qh ); this would be indicated by a large
partial derivative of heat transfer coefficient with respect to heater power. Given the uncertainty
in the heater power measurement, δ qh (related to, for example, the resolution of the data
acquisition system, noise, etc.), it is possible to estimate the uncertainty in the measurement of
the heat transfer coefficient, δ h , according to:

∂h
δh = δ qh
∂qh

For example, if the uncertainty in the heater power is 0.01 W then Fig. 3 suggests that the partial
derivative of heat transfer coefficient with respect to heater power is approximately 125 W/m2-
K-W and the uncertainty in the heat transfer coefficient measurement would be δ h = 1.8 W/m2-
K.

As an engineer designing this measurement device, you would like to calculate not the heater
power but rather the partial derivative in the heat transfer coefficient with respect to heater power
in order to automate the process of computing δ h and therefore evaluate how design changes
affect the instrument. This would be difficult to accomplish analytically; note that both m and
the constants Ci in Eq. (16) are functions of h . It is more convenient to determine the partial
derivative numerically. That is, set the heat transfer coefficient at its nominal value plus a small
amount ( h + Δh ) and evaluate the heater power ( qh , h +Δh ); note that Δh should be small relative
to the nominal value of the heat transfer coefficient. Then set the heat transfer coefficient at its
nominal value less a small amount ( h − Δh ) and evaluate the heater power ( qh , h −Δh ). The partial
derivative is approximately:

∂h 2 Δh

∂qh qh ,h +Δh − qh ,h −Δh

Since qh needs to be evaluated at several values of the heat transfer coefficient, it is convenient
to have the computation of the heater power occur within a function that is called twice within
the equation window. Because the solution is in the form of an EES code, this is an ideal
problem to use a MODULE. A MODULE is a stand-alone EES program that can be called from
the main EES equation window. The MODULE is provided with inputs and it calculates
outputs. The protocol of a call to a MODULE involves the name of the MODULE followed by a
series of inputs separated by a colon from a series of outputs. For example, we will create a
MODULE Heaterpower that calculates the value of the variable q_dot_h. It is important to note
that all of the variables from the main equation window that are required by the MODULE must
be passed to the MODULE; the MODULE can only access variables that are passed to it as
parameters or using the $COMMON directive.

The MODULE Heaterpower must be placed at the top of the EES code and is a small self-
contained EES program; we create the MODULE by copying those lines of the main EES code
that are required to calculate the variable q_dot_h.

MODULE Heaterpower(th,k,a,b,T_h,T_infinity,h_bar:q_dot_h)
theta_h=T_h-T_infinity "heater temperature difference"
m=sqrt(2*h_bar/(k*th)) "solution constant"
N=100 "number of terms"
duplicate i=1,N
lambda[i]=(2*i-1)*pi/(2*a) "eigenvalues"
C[i]=theta_h*(4*(-1)^(1+i)/(2*i-1)/Pi)/(cosh(sqrt(lambda[i]^2+m^2)*b))
q_dot_h[i]=2*k*th*a*C[i]*sqrt(lambda[i]^2+m^2)*sinh(sqrt(lambda[i]^2+m^2)*b)
end
q_dot_h=sum(q_dot_h[1..N])
end
The calling protocol for the MODULE consists of a series of inputs (the variables d, k, a, b,
T_htr, T_f, and h) that are separated by a series outputs (in this case only the variable q_dot_htr)
by a colon. MODULES are most useful where a certain sequence of code must be executed
multiple times; in this case, the MODULE Heaterpower enables the partial derivative to be easily
computed.

delta_q_dot_htr=0.01 [W] "heater power resolution"


dh=1 [W/m^2-K] "perturbation of heat transfer coefficient"

CALL Q_dot_heater(d,k,a,b,T_htr,T_f,h-dh:q_dot_htr_minus)
CALL Q_dot_heater(d,k,a,b,T_htr,T_f,h+dh:q_dot_htr_plus)
dhdqdot=2*dh/(q_dot_htr_plus-q_dot_htr_minus)
delta_h=dhdqdot*delta_q_dot_htr

The modifications to the EES code verifies that the uncertainty in heat transfer coefficient,
delta_h, is 1.26 W/m2-K and provides a convenient tool for assessing the impact of the various
design parameters on the performance of the measurement system. For example, Figure 5
illustrates the uncertainty in the heat transfer coefficient as a function of the plate thickness (th)
for various values of its half-width (a).

Figure 5: Uncertainty in the measured heat transfer coefficient as a function of the plate thickness
for various values of the plate half-width.
Problem 2.2-4: Local Heat Transfer Coefficient Measurement
Figure P2.2-4(a) illustrates a proposed device to measure the local heat transfer coefficient from
a surface undergoing a boiling heat transfer process. Micro-scale heaters and temperature
sensors are embedded in a substrate in a regularly spaced array, as shown.

evaporating fluid

heaters computational
temperature domain
sensors
Figure P2.2-4(a): An array of micro-scale heaters and temperature sensors embedded in a substrate.

The heaters are activated, producing a heat flux that is removed primarily from the surface of the
substrate exposed to evaporating fluid. The temperature sensors are embedded in sets of two
located very near the surface. Each set of thermocouples are used to infer the local heat flux to
the surface ( qs′′ ) and the surface temperature (Ts); these quantities are sufficient to measure the
heat transfer coefficient. A half-symmetry model of the region of the substrate between two
adjacent heaters (see Figure 2.2-4(a)) is shown in Figure P2.2-4(b).

T∞ = 20°C
y1 = 1.9 mm h = 2500 W/m -K
2

qh′′ = 1x10 W/m


5 2

b = 2 mm
W = 5 mm
y
x

y2 = 1.8 mm
k = 3.5 W/m-K
Figure P2.2-4(b): A half-symmetry model of the region of the substrate between two adjacent
heaters.

The thickness of the substrate is b = 2 mm and the half-width between adjacent heaters is W = 5
mm. The substrate has conductivity k = 3.5 W/m-K and you may assume that the presence of the
temperature sensors does not affect the temperature distribution. The heat flux exposed to the
computational domain at x = W is qh′′ = 1x105 W/m2. The heat transfer coefficient between the
evaporating fluid at T∞ = 20ºC and the surface is h = 2500 W/m2-K.
a.) Develop a separation of variables solution based on the computational domain shown in
Figure P2.2-4(b). Implement your solution in EES.

The known information is entered in EES:

$UnitSystem SI MASS RAD PA C J


$Tabstops 0.2 0.4 0.6 0.8 3.5

"Inputs"
k=3.5 [W/m-K] "conductivity"
T_infinity=converttemp(C,K,20 [C]) "fluid temperature"
h=2500 [W/m^2-K] "heat transfer coefficient"
W=5 [mm]*convert(mm,m) "half-width"
b=2 [mm]*convert(mm,m) "substrate thickness"
y_1=1.9 [mm]*convert(mm,m) "temperature sensor #1 position"
y_2=1.8 [mm]*convert(mm,m) "temperature sensor #2 position"
q_h=1e5 [W/m^2] "edge heat flux"

The problem shown in Figure P2.2-4(b) is governed by the partial differential equation:

∂ 2T ∂ 2T
+ =0 (1)
∂x 2 ∂y 2

and has the boundary conditions:

∂T
=0 (2)
∂x x =0

∂T
k = qh′′ (3)
∂x x =W

∂T
=0 (4)
∂y y =0

∂T
−k = h (Ty =b − T∞ ) (5)
∂y y =b

Neither the x- or y-directions are homogeneous. However, by defining the temperature


difference relative to T∞, both boundary conditions in the y-direction can be made homogeneous:

θ = T − T∞ (6)

The transformed differential equation and boundary conditions are:

∂ 2θ ∂ 2θ
+ =0 (7)
∂x 2 ∂y 2

and has the boundary conditions:


∂θ
=0 (8)
∂x x =0

∂θ
k = qh′′ (9)
∂x x =W

∂θ
=0 (10)
∂y y =0

∂θ
−k = h θ y =b (11)
∂y y =b

The separated solution is assumed:

θ = θ X θY (12)

and substituted into Eq. (7); the process leads to two ordinary differential equations:

d 2θ X
− λ2 θ X = 0 (13)
dx 2

d 2θ Y
+ λ 2 θY = 0 (14)
dy 2

Notice that the sign of the constant is selected so that the solution in the homogeneous direction
(y) is sines and cosines. The solution to Eq. (14) is:

θ Y = C1 sin ( λ y ) + C2 cos ( λ y ) (15)

The boundary condition at y = 0, Eq. (10), leads to:

dθ Y
= C1 λ cos ( λ 0 ) − C2 λ sin ( λ 0 ) = 0 (16)
dy y =0

which indicates that C1 = 0 and therefore:

θ Y = C2 cos ( λ y ) (17)

The boundary condition at y = b, Eq. (11), leads to:

k C2 λ sin ( λ b ) = h C2 cos ( λ b ) (18)


Equation (18) provides the eigencondition for the problem:

hb
tan ( λi b ) = (19)
k ( λi b )

The roots of Eq. (19) occur in regular intervals of the argument of the tangent function. A
residual is defined by rearranging Eq. (19):

hb
Res = tan ( λi b ) − (20)
k ( λi b )

Figure P2.2-4-2 illustrates the residual defined in Eq. (20); the eigenvalues correspond to the
points where the residual is zero.
1

0.75

0.5

0.25
Residual

-0.25

-0.5

-0.75
eigenvalues
-1
0 1.57 3.14 4.71 6.28 7.85 9.42 10.99 12.56 14.13 15.7
λb
Figure P2.2-4-2: Residual as a function of λb.

The eigenvalues lie in regular intervals of λb, as shown in Figure 2.2-4-2 and are identified
automatically in EES by defining arrays and using them as the lower limit, upper limit, and guess
values for the entries in the eigenvalue array in the Variable Information window (Figure P2.2-4-
3).

Figure P2.2-4-3: Variable Information window.

{Res=tan(lambdab)-h*b/(k*lambdab)} "residual of the eigencondition"


N=20 [-]
duplicate i=1,N
upperlimit[i]=(i-1/2)*pi
lowerlimit[i]=(i-1)*pi
guess[i]=(i-3/4)*pi
tan(lambdab[i])-h*b/(k*lambdab[i])=0
lambda[i]=lambdab[i]/b
end

The eigenfunctions are:

θ Yi = C2,i cos ( λi y ) (21)

The solution to Eq. (13) for each eigenvalue is:

θ X i = C3,i sinh ( λi x ) + C4,i cosh ( λi x ) (22)

The general solution for for each eigenvalue is:

θi = θ X i θ Yi = C2,i cos ( λi y ) ⎡⎣C3,i sinh ( λi x ) + C4,i cosh ( λi x ) ⎤⎦ (23)

The sum of these solutions is, itself, a solution (note that the constant C2,i is absorbed into the
other constants):

∞ ∞
θ = ∑ θi = ∑ cos ( λi y ) ⎡⎣C3,i sinh ( λi x ) + C4,i cosh ( λi x ) ⎤⎦ (24)
i =1 i =1

Equation (24) is substituted into the boundary condition at x= 0, Eq. (8):

∂θ ∞
= ∑ cos ( λi y ) ⎡⎣C3,i λi cosh ( λi 0 ) + C4,i λi sinh ( λi 0 ) ⎤⎦ (25)
∂x x =0 i =1

or

∑ cos ( λ y ) C
i =1
i 3,i λi = 0 (26)

which can only be true if C3,i = 0; therefore:


θ = ∑ Ci cos ( λi y ) cosh ( λi x ) (27)
i =1
where the constant C4,i is referred to simply as Ci since it is the only remaining undetermined
constant associated with each eigenvalue. Equation (27) is substituted into the boundary
condition at x = W, Eq. (9):


k ∑ Ci cos ( λi y ) λi cosh ( λi W ) = qh′′ (28)
i =1

The eigenfunctions are orthogonal; therefore, Eq. (28) is multiplied by an arbitary eigenfunction
and integrated from y = 0 to y = b. The result is:
b b
Ci k λi cosh ( λi W ) ∫ cos ( λ y ) dy = q′′ ∫ cos ( λ y ) dy
2
i i (29)
y =0 y =0

The two integrals in Eq. (29) are evaluated in Maple:

> restart;
> int((cos(lambda*y))^2,y=0..b);
1 cos( λ b ) sin( λ b ) + λ b
2 λ
> int(cos(lambda*y),y=0..b);
sin( λ b )
λ

Therefore, the constants can be evaluated according to:

sin ( λi b )
Ci = q ′′ (30)
k λ cosh ( λi W ) Integrali
i
2

where

⎡cos ( λi b ) sin ( λi b ) + λi b ⎤⎦
Integrali = ⎣ (31)
2 λi

duplicate i=1,N
Integral[i]=(cos(lambdab[i])*sin(lambdab[i])+lambdab[i])/(2*lambda[i])
C[i]=q_h*sin(lambdab[i])/(k*lambda[i]^2*cosh(lambdab[i])*Integral[i])
end

The solution is obtained according to Eq. (27) at a particular position:

x_bar=0.5 [-]
y_bar=0.5 [-]
x=x_bar*W
y=y_bar*b
duplicate i=1,N
theta[i]=C[i]*cos(lambda[i]*y)*cosh(lambda[i]*x)
end
T=T_infinity+sum(theta[i],i=1,N)
T_C=converttemp(K,C,T)

b.) Prepare a contour plot of the temperature distribution in the substrate.

Figure P2.2-4-4 illustrates a contour plot of temperature in the substrate.


1
contours of constant temperature (°C)
0.9
0.8
0.7
55.72
0.6
49.9
y/b

0.5 61.54
44.08
0.4 32.45
0.3 38.27 67.36

0.2 73.17
0.1
78.99
0
0 0.1 0.2 0.3 0.4 0.5 0.6 0.7 0.8 0.9 1
x/W
Figure P2.2-4-4: Contour plot of temperature.

The position of temperature sensors #1 and #2 at a particular value of x are y1 = 1.9 mm and y2 =
1.8 mm, respectively (see Figure P2.2-4(b)). The surface temperature measurement extracted
from these measured temperatures is associated with a linear extrapolation to the surface at y = 0:

Ts ,m = T2 + (T1 − T2 )
( b − y2 ) (32)
( y1 − y2 )
The heat flux measurement extracted from these measured temperatures is obtained from
Fourier's law according to:

qs′′,m = k
(T2 − T1 ) (33)
( y1 − y2 )
c.) What is the heat transfer coefficient measured by the device at x/W = 0.5? That is, based on
the temperatures T1 and T2 predicted by your model at x/W = 0.5, calculate the measured heat
transfer coefficient according to:

qs′′,m
hm = (34)
(Ts ,m − T∞ )

and determine the discrepancy of your measurement relative to the actual heat transfer
coefficient.
The temperatures measured by the sensor set at x/W = 0.5 are evaluated:

duplicate i=1,N
theta_1[i]=C[i]*cos(lambda[i]*y_1)*cosh(lambda[i]*x)
theta_2[i]=C[i]*cos(lambda[i]*y_2)*cosh(lambda[i]*x)
end
T_1=T_infinity+sum(theta_1[i],i=1,N)
T_2=T_infinity+sum(theta_2[i],i=1,N)

The surface temperature, heat flux, and heat transfer coefficient are evaluated according to Eqs.
(32) through (34):

T_s_m=T_2+(T_1-T_2)*(b-y_2)/(y_1-y_2)
q_s_m=k*(T_2-T_1)/(y_1-y_2)
h_m=q_s_m/(T_s_m-T_infinity)

which leads to hm = 2358 W/m2-K. This is 5.6% in error relative to the actual heat transfer
coefficient.

d.) Plot the % error associated with the device configuration (i.e., the discrepancy between the
measured and actual heat transfer coefficient from part (c)) as a function of axial position, x.

The % error is calculated according to:

deltah=100*abs(h-h_m)/h

and shown in Figure P2.2-4-5 as a function of x/W.


20
Error in heat transfer coefficient (%)

18
16
14
12
10
8
6
4
2
0
0 0.1 0.2 0.3 0.4 0.5 0.6 0.7 0.8 0.9 1
x/W
Figure P2.2-4-5: Error as a function of axial location.
Problem 2.2-5

Three heater blocks provide heat to the back-side of a fin array that must be tested, as shown in
Figure P2.2-5.

q1′′ = 3x10 W/m


4 2

k = 30 W/m-K
q2′′ = 3x10 W/m
4 2

c = 1 cm d = 4 cm
L = 7 cm
y
x h = 20 W/m -K
2

th = 1 cm T∞ = 20°C
h f = 35 W/m -K
2

T f = 35°C
N fins = 150
As, fin = 8 cm
2

Ab, fin = 0.8 cm


2

η fin = 0.85
Figure P2.2-5: An array of fins installed on a base plate energized by three heater blocks.

The fins are installed on a base plate that has half-width of L= 7 cm, thickness of th = 1 cm, and
width W = 20 cm (into the page). The base plate material has conductivity k = 30 W/m-K. The
edge of the base plate (at x = L) is exposed to air at T∞ = 20°C with heat transfer coefficient h =
20 W/m2-K. The middle of the plate (at x = 0) is a line of symmetry and can be modeled as
being adiabatic. The bottom of the plate (at y = 0) has an array of Nfin = 150 fins installed. Each
fin has surface area As,fin= 8 cm2, base area Ab,fin = 0.8 cm2, and efficiency ηfin = 0.85. The fin
and the base material are exposed to fluid at Tf = 35°C with heat transfer coefficient h f = 20
W/m2-K. The top of the plate (at y = th) is exposed to the heat flux from the heater blocks. The
heat flux is distributed according to:

⎧q1′′ if x < c

q ′′y =th = ⎨0 if c < x < ( d + c )
⎪q ′′ if x > ( d + c )
⎩ 2

where q1′′ = 3x104 W/m2, q2′′ = 3x104 W/m2, c = 1 cm and d = 4 cm.


a.) Determine an effective heat transfer coefficient that can be applied to the surface at y = th in
order to capture the combined effect of the fins and the unfinned base area.

The known information is entered in EES:

"PROBLEM 2.2-5"
$UnitSystem SI MASS RAD PA K J
$TABSTOPS 0.2 0.4 0.6 0.8 3.5 in

"Inputs"
k=30 [W/m-K] "thermal conductivity"
h_bar=20 [W/m^2-K] "heat transfer coefficient to ambient air"
W=20 [cm]*convert(cm,m) "width of base"
L=7 [cm]*convert(cm,m) "half-length of base"
th=1 [cm]*convert(cm,m) "thickness of base"
h_bar_f=35 [W/m^2-K] "heat transfer coefficient to fluid"
A_s_fin=8 [cm^2]*convert(cm^2,m^2) "surface area of each fin"
A_b_fin=0.8 [cm^2]*convert(cm^2,m^2) "base area of each fin"
N_fin=150 [-] "number of fins"
eta_fin=0.85 [-] "fin efficiency"
T_infinity=converttemp(C,K,20[C]) "ambient air temperature"
T_f=converttemp(C,K,35 [C]) "fluid temperature"
c=1 [cm]*convert(cm,m) "width of first heating zone"
d=4 [cm]*convert(cm,m) "space between heating zones"
q``_1=3e4 [W/m^2] "heat flux in 1st heating zone"
q``_2=3e4 [W/m^2] "heat flux in 2nd heating zone"

The effective heat transfer coefficient is defined so that it provides the same rate of heat transfer
as the finned surface:

W L heff = (W L − N fins Ab , fin ) h f + N fins As , fin η fin h f (1)

W*L*h_bar_eff=(W*L-N_fin*A_b_fin)*h_bar_f+eta_fin*N_fin*A_s_fin*h_bar_f
"effective heat transfer coefficient"

which leads to heff = 260 W/m2-K.

b.) Develop a separation of variables solution for the temperature distribution within the fin base
material.

The problem shown in Figure P2.2-5 is governed by the transformed partial differential equation:

∂ 2θ ∂ 2θ
+ =0 (2)
∂x 2 ∂y 2

and has the boundary conditions:

∂θ
=0 (3)
∂x x =0

∂θ
−k = h θ x=L (4)
∂y x=L

⎧q1′′ if x < c
∂θ ⎪⎪
k = ⎨0 if c < x < ( d + c ) (5)
∂y y =th ⎪
⎪⎩q2′′ if x > ( d + c )
∂θ
heff ⎡⎣θ f − θ y =0 ⎤⎦ = − k (6)
∂y y =0

where

θ = T − T∞ (7)

and

θ f = T f − T∞ (8)

The separated solution is assumed:

θ = θ X θY (9)

and substituted into Eq. (2); the process leads to two ordinary differential equations:

d 2θ X
+ λ2 θ X = 0 (10)
dx 2

d 2θ Y
− λ 2 θY = 0 (11)
dy 2

Notice that the sign of the constant is selected so that the solution in the homogeneous direction
(x) is sines and cosines. The solution to Eq. (10) is:

θ X = C1 sin ( λ x ) + C2 cos ( λ x ) (12)

The boundary condition at x = 0, Eq. (3) requires that C1 = 0 and therefore:

θ X = C2 cos ( λ x ) (13)

The boundary condition at x = L, Eq. (4), leads to the eigencondition for the problem:

hL
tan ( λi L ) = (14)
k ( λi L )

The eigenvalues lie in regular intervals of λL and are identified automatically in EES by defining
arrays and using them as the lower limit, upper limit, and guess values for the entries in the
eigenvalue array in the Variable Information window.

Nterm=11 [-] "number of terms to use in the solution"


"Setup guess values and lower and upper bounds for eigenvalues"
duplicate i=1,Nterm
lowerlimit[i]=(i-1)*pi
upperlimit[i]=lowerlimit[i]+pi/2
guess[i]=lowerlimit[i]+pi/4
end
"Identify eigenvalues"
duplicate i=1,Nterm
tan(lambdaL[i])=h_bar*L/(k*lambdaL[i]) "eigencondition"
lambda[i]=lambdaL[i]/L "eigenvalue"
end

The eigenfunctions are:

θ X i = C2,i cos ( λi x ) (15)

The solution to Eq. (11) for each eigenvalue is:

θ Yi = C3,i sinh ( λi y ) + C4,i cosh ( λi y ) (16)

The general solution for for each eigenvalue is:

θi = θ X i θ Yi = C2,i cos ( λi x ) ⎡⎣C3,i sinh ( λi y ) + C4,i cosh ( λi y ) ⎤⎦ (17)

The sum of these solutions is, itself, a solution (note that the constant C2,i is absorbed into the
other constants):

∞ ∞
θ = ∑ θi = ∑ cos ( λi x ) ⎡⎣C3,i sinh ( λi y ) + C4,i cosh ( λi y ) ⎤⎦ (18)
i =1 i =1

Equation (18) is substituted into the boundary condition at y = 0, Eq. (6):

⎡ ∞
⎤ ∞
heff ⎢θ f − ∑ C4,i cos ( λi x ) ⎥ = − k ∑ C3,i λi cos ( λi x ) (19)
⎣ i =1 ⎦ i =1

Equation (19) is multiplied by an eigenfunction and integrated from x = 0 to x = L:


L L L
heff θ f ∫ cos ( λi x ) dx − heff C4,i ∫ cos 2 ( λi x ) dx = − k C3,i λi ∫ cos 2 ( λi x ) dx (20)
0 0 0

or
L L
heff θ f ∫ cos ( λi x ) dx = ⎡⎣ heff C4,i − k C3,i λi ⎤⎦ ∫ cos 2 ( λi x ) dx (21)
0

0

Integral1
The interals in Eq. (21) are evaluated in Maple:

> restart;
> int(cos(lambda[i]*x),x=0..L);
sin( λ i L )
λi

> int((cos(lambda[i]*x))^2,x=0..L);
1 cos( λ i L ) sin( λ i L ) + λ i L
2 λi

The integrals are put into EES, leading to a relationship between C3,i and C4,i :

duplicate i=1,Nterm
Int1[i]=1/2*(cos(lambda[i]*L)*sin(lambda[i]*L)+lambda[i]*L)/lambda[i]
h_bar_eff*(T_f-T_infinity)*sin(lambda[i]*L)/lambda[i]=(h_bar_eff*C4[i]-k*C3[i]*lambda[i])*Int1[i]
end

Equation (18) is substituted into the boundary condition at y = th, Eq. (5):

⎧q1′′ if x < c
∞ ⎪⎪
λi k ∑ cos ( λi x ) ⎣⎡C3,i cosh ( λi th ) + C4,i sinh ( λi th ) ⎦⎤ = ⎨0 if c < x < ( d + c ) (22)
i =1 ⎪
⎪⎩q2′′ if x > ( d + c )

Again, the orthogonality of the eigenfunctions are used to obtain:


L c L
λi k ⎡⎣C3,i cosh ( λi th ) + C4,i sinh ( λi th ) ⎤⎦ ∫ cos 2 ( λi x ) dx = q1′′ ∫ cos 2 ( λi x ) dx + q2′′ ∫ cos ( λ x ) dx (23)
2
i
0 0 d +c

The integrals are carried out in Maple:

> int(cos(lambda[i]*x),x=0..c);
sin( λ i c )
λi

> int(cos(lambda[i]*x),x=(d+e)..L);
−sin( λ i ( d + e ) ) + sin( λ i L )
λi

and used to complete the computation of the constants in EES:

duplicate i=1,Nterm
Int2[i]=(-sin(lambda[i]*(d+c))+sin(lambda[i]*L))/lambda[i]
k*(C3[i]*lambda[i]*cosh(lambda[i]*L)+C4[i]*lambda[i]*sinh(lambda[i]*L))*Int1[i]=&
q``_1*sin(lambda[i]*c)/lambda[i]+q``_2*Int2[i]
end

The solution is evaluated at an arbitary position:

x_bar=0.5 [-]
y_bar=0 [-]
x=x_bar*L
y=y_bar*th
duplicate i=1,Nterm
theta[i]=cos(lambda[i]*x)*(C3[i]*sinh(lambda[i]*y)+C4[i]*cosh(lambda[i]*y))
end
T=sum(theta[1..Nterm])+T_infinity
T_C=converttemp(K,C,T)

c.) Prepare a plot showing the temperature as a function of x at various values of y.

Figure 2 illlustrates the temperature as a function of x/L at various values of y/th.

83
y/th = 1

82
y/th = 0.75
Temperature (°C)

81
y/th = 0.5

80
y/th = 0.25

79
y/th = 0

78
0 0.1 0.2 0.3 0.4 0.5 0.6 0.7 0.8 0.9 1
Dimensionless axial position, x/L
Figure 2: Temperature as a function of x/L for various values of y/th.

d.) The goal of the base plate is to provide an uniform heat flow to each fin. Assess the
performance of the base plate by plotting the rate of heat flux transferred to the fluid as a
function of x at y = 0.

The heat flux to the fluid is computed according to:

q ′′f = heff (Ty =0 − T f ) (24)


q``_f=h_bar_eff*(T-T_f)

Figure 3 shows the heat flux to the fluid as a function of axial location.

19000
3 W/m-K
18000
Heat flux to fluid (W/m )
2

17000

16000

15000

14000 10 W/m-K

13000
30 W/m-K
12000 100 W/m-K

11000
0 0.1 0.2 0.3 0.4 0.5 0.6 0.7 0.8 0.9 1
Dimensionless axial location, x/L
Figure 3: Heat flux to fluid as a function of x/L.

e.) Overlay on your plot for (d) the rate of heat flux transferred to the fluid for various values of
the base plate conductivity.

Figure 3 includes various values of conductivity and shows that as conductivity increases, more
energy is lost to the edge and the heat flux along the bottom is more uniform.
Problem 2.2-6 (2-4 in text)

A laminated composite structure is shown in Figure P2.2-6.

q ′′ = 10000 W/m
2
H = 3 cm

Tset = 20°C
W = 6 cm
Tset = 20°C
kx = 50 W/m-K
ky = 4 W/m-K
Figure P2.2-6: Composite structure exposed to a heat flux.

The structure is anisotropic. The effective conductivity of the composite in the x-direction is kx =
50 W/m-K and in the y-direction it is ky = 4 W/m-K. The top of the structure is exposed to a heat
flux of q ′′ = 10,000 W/m2. The other edges are maintained at Tset = 20°C. The height of the
structure is H = 3 cm and the half-width is W = 6 cm.
a.) Develop a separation of variables solution for the 2-D steady-state temperature distribution in
the composite.

The known information is entered in EES:

$UnitSystem SI MASS RAD PA K J


$TABSTOPS 0.2 0.4 0.6 0.8 3.5 in

"Inputs"
W=6 [cm]*convert(cm,m) "width of laminate"
H=3 [cm]*convert(cm,m) "height of laminate"
q``=10000 [W/m^2] "heat flux"
k_x=50 [W/m-K] "conductivity in the x-direction"
k_y=4 [W/m-K] "conductivity in the y-direction"
T_set=converttemp(C,K,20[C]) "specified edge temperatures"

A half-symmetry model of the problem shown in Figure P2.2-6 is governed by the transformed
partial differential equation:

∂ 2θ ∂ 2θ
kx + k =0 (1)
∂x 2 ∂y 2
y

with the boundary conditions:

∂θ
=0 (2)
∂x x =0

θ x =W = 0 (3)
∂θ
ky = q ′′ (4)
∂y y=H

θ y =0 = 0 (5)

where

θ = T − Tset (6)

The separated solution is assumed:

θ = θ X θY (7)

and substituted into Eq. (1):

d 2θ X d 2θ Y
kx θY + y
k θ X =0 (8)
dx 2 dx 2

Dividing through by θY θX leads to:

d 2θ X d 2θ Y
dx 2 + k y dx 2 = 0 (9)
θX

k θY
x

−λ2 λ2

The process leads to two ordinary differential equations:

d 2θ X
2
+ λ2 θ X = 0 (10)
dx

d 2θ Y
− β 2 λ 2 θY = 0 (11)
dy 2

where

kx
β2 = (12)
ky

beta=sqrt(k_x/k_y)

Notice that the sign of the constant is selected so that the solution in the homogeneous direction
(x) is sines and cosines. The solution to Eq. (10) is:
θ X = C1 sin ( λ x ) + C2 cos ( λ x ) (13)

The boundary condition at x = 0, Eq. (2) requires that C1 = 0 and therefore:

θ X = C2 cos ( λ x ) (14)

The boundary condition at x = L, Eq. (3), leads to the eigencondition for the problem:

cos ( λi W ) = 0 (15)

which requires that:

λi =
(1 + 2 i ) π for i = 0,1,...∞ (16)
2W

N_term=11 [-] "number of terms"


duplicate i=0,N_term
lambda[i]=(1+2*i)*pi/(2*W) "eigenvalues"
end

The eigenfunctions are:

θ X i = C2,i cos ( λi x ) (17)

The solution to Eq. (11) for each eigenvalue is:

θ Yi = C3,i sinh ( β λi y ) + C4,i cosh ( β λi y ) (18)

The general solution for for each eigenvalue is:

θi = θ X i θ Yi = C2,i cos ( λi x ) ⎡⎣C3,i sinh ( β λi y ) + C4,i cosh ( β λi y ) ⎤⎦ (19)

The sum of these solutions is, itself, a solution (note that the constant C2,i is absorbed into the
other constants):

∞ ∞
θ = ∑ θi = ∑ cos ( λi x ) ⎡⎣C3,i sinh ( β λi y ) + C4,i cosh ( β λi y ) ⎤⎦ (20)
i =1 i =1

The boundary condition at y = 0, Eq. (5), leads to C4,i = 0:

∞ ∞
θ = ∑ θi = ∑ Ci cos ( λi x ) sinh ( β λi y ) (21)
i =1 i =1
Equation (21) is substituted into the boundary condition at y = H, Eq. (4):


β λi k y ∑ Ci cos ( λi x ) cosh ( β λi H ) = q ′′ (22)
i =1

Equation (22) is multiplied by an eigenfunction and integrated from x = 0 to x = W:


L L
β λi k y cosh ( β λi H ) Ci ∫ cos 2 ( λi x ) dx = q ′′∫ cos ( λi x ) dx (23)
0 0

The integrals are carried out in Maple:

> restart;
> assume(i,integer);
> lambda:=(1+2*i)*Pi/(2*W);
( 1 + 2 i~ ) π
λ :=
2W
> int((cos(lambda*x))^2,x=0..W);
W
2
> int(cos(lambda*x),x=0..W);
2 ( -1 )i~ W
( 1 + 2 i~ ) π

and used to complete the computation of the constants in EES:

duplicate i=0,N_term
k_y*C[i]*beta*lambda[i]*cosh(beta*lambda[i]*H)*W/2=q``*2*(-1)^i*W/((1+2*i)*Pi)
end

The solution is evaluated at an arbitary position:

x_bar=0.5 [-] "dimensionless x-position"


y_bar=0.5 [-] "dimensionless y-position"
x=x_bar*W "x-position"
y=y_bar*H "y-position"
duplicate i=0,N_term
theta[i]=C[i]*cos(lambda[i]*x)*sinh(beta*lambda[i]*y)
end
theta=sum(theta[0..N_term])
T=theta+T_set
T_C=converttemp(K,C,T)

b.) Prepare a contour plot of the temperature distribution.


Figure 2 illlustrates a contour plot of the temperature distribution.

1
20
23.12
Dimensionless y position, y/H 0.8
26.24
29.36
0.6 32.48
35.6
38.72
0.4
41.84
44.96
0.2 48.08
51.2

0
0 0.2 0.4 0.6 0.8 1

Dimensionless x position, x/W


Figure 2: Contour plot of the temperature distribution.
Problem 2.3-1 (2-5 in text): Cryogen Transfer Pipe
Figure P2.3-1 illustrates a pipe that connects two tanks of liquid oxygen on a spacecraft. The
pipe is subjected to a heat flux, q′′ = 8,000 W/m2, which can be assumed to be uniformly applied
to the outer surface of the pipe and entirely absorbed. Neglect radiation from the surface of the
pipe to space. The inner radius of the pipe is rin = 6 cm, the outer radius of the pipe is rout = 10
cm, and the half-length of the pipe is L = 10 cm. The ends of the pipe are attached to the liquid
oxygen tanks and therefore are at a uniform temperature of TLOx = 125 K. The pipe is made of a
material with a conductivity of k = 10 W/m-K. The pipe is empty and therefore the internal
surface can be assumed to be adiabatic.

rout = 10 cm L = 10 cm
rin = 6 cm TLOx = 125 K

k = 10 W/m-K q′′s = 8,000 W/m


2

Figure P2.3-1: Cryogen transfer pipe connecting two liquid oxygen tanks.

a.) Develop an analytical model that can predict the temperature distribution within the pipe.
Prepare a contour plot of the temperature distribution within the pipe.

A differential control volume leads to the energy balance:

qx + qr = qx + dx + qr + dr

or

∂qx ∂q
0= dx + r dr
∂x ∂r

Substituting the rate equations:

∂T
qx = − k 2 π r dr
∂x

and

∂T
qr = − k 2 π r dx
∂r

into the differential energy balance leads to:


∂ ⎡ ∂T ⎤ ∂ ⎡ ∂T ⎤
0= ⎢ − k 2 π r dr ⎥ dx + ⎢ −k 2 π r dx ⎥ dr
∂x ⎣ ∂x ⎦ ∂r ⎣ ∂r ⎦

or

∂ 2T ∂ ⎡ ∂T ⎤
r + r =0
∂x 2 ∂r ⎢⎣ ∂r ⎥⎦

A half-symmetry model of the pipe will be generated; the boundary conditions are therefore:

∂T
=0
∂x x =0

Tx = L = TLOx

∂T
=0
∂r r = rin

∂T
k = q′′
∂r r = rout

As stated, there are two non-homogeneous boundary conditions; however, the boundary
condition at x = L can be made homogeneous by defining the temperature difference:

θ = T − TLOx

The partial differential equation and boundary conditions are written in terms of θ:

∂ 2θ ∂ ⎡ ∂θ ⎤
r + r =0 (1)
∂x 2 ∂r ⎢⎣ ∂r ⎥⎦

∂θ
=0 (2)
∂x x =0

θ x= L = 0 (3)

∂θ
=0 (4)
∂r r = rin
∂θ
k = q′′ (5)
∂r r = rout

Note that the two homogeneous boundary conditions are in the x-direction and so the
eigenfunctions of the problem will be in this direction. We assume that the solution is separable;
that is, the solution is the product of a function only of x (θX) and r (θR):

θ ( x, y ) = θ X ( x ) θ R ( r )

Substituting the product solution into the governing partial differential equation, Eq. (1), leads to:

d 2θ X d ⎡ dθ R ⎤
rθ R +θ X r =0
dx 2
dr ⎢⎣ dr ⎥⎦

Dividing by the product r θX θR leads to:

d 2θ X d ⎡ dθ R ⎤
⎢r ⎥
dx + dr ⎣ dr ⎦ = 0
2

θX rθ R
±λ2 ∓λ2

Note that the 1st term is a function only of x while the 2nd term is a function only of r; these two
quantities must be equal and opposite constants (±λ2). The choice of the sign is again important;
the eigenfunctions must be in x and therefore the two ordinary differential equations must be:

d 2θ X
+ λ2 θ X = 0 (6)
dx 2

d ⎡ dθ R ⎤
⎢ r ⎥ − λ2 rθ R = 0 (7)
dr ⎣ dr ⎦

The eigenproblem will be solved first; the solution to Eq. (6) is:

θ X = C1 cos ( λ x ) + C2 sin ( λ x )

The boundary condition at x = 0, Eq. (2), eliminates the sine term. The boundary condition at x =
L, Eq. (3), leads to:

C1 cos ( λ L ) = 0

which provides the eigenvalues and the eigenfunctions:


θ X i = C1,i cos ( λi x ) where λi =
( 2 i − 1) π for i = 1, 2,..∞
2L

The ordinary differential equation for θR, Eq. (7), is solved by Bessel functions, as discussed in
Section 1.8. Equation (7) is a form of Bessel's equation:

d ⎛ p dθ ⎞ 2 s
⎜x ⎟±c x θ =0
dx ⎝ dx ⎠

where s = 1 and p = 1; the quantity s – p +2 is therefore not equal to zero and we are directed
toward the left-side of the Bessel function flow chart presented in Section 1.8.4 where the
parameters n = 0, a = 1, and n/a = 0 are computed. The last term is negative and therefore the
general solution to Eq. (7) is:

θ Ri = C3,i BesselI ( 0,λi r ) + C4,i BesselK ( 0,λi r )

This general solution could have been obtained using Maple:

> restart;
> ODEr:=diff(r*diff(thetar(r),r),r)-lambda^2*r*thetar(r)=0;
⎛d ⎞
2
⎛ d ⎞
ODEr := ⎜⎜ thetar( r ) ⎟⎟ + r ⎜ 2 thetar( r ) ⎟⎟ − λ 2 r thetar( r ) = 0

⎝ dr ⎠ ⎝ dr ⎠
> thetars:=dsolve(ODEr);
thetars := thetar( r ) = _C1 BesselI( 0, λ r ) + _C2 BesselK( 0, λ r )

The general solution for each eigenvalue is therefore:

θi = θ X i θ Ri = cos ( λi x ) ⎡⎣C3,i BesselI ( 0,λi r ) + C4,i BesselK ( 0,λi r ) ⎤⎦

The general solution is entered as a function of x and y in Maple:

> restart;
> assume(i,integer);
> lambda:=(2*i-1)*Pi/(2*L);
( 2 i~ − 1 ) π
λ :=
2L
> theta:=(x,r)->cos(lambda*x)*(C3*BesselI(0,lambda*r)+C4*BesselK(0,lambda*r));
θ := ( x, r ) → cos( λ x ) ( C3 BesselI( 0, λ r ) + C4 BesselK( 0, λ r ) )

Verify that it solves both boundary conditions in the x-directions, Eqs. (2) and (3):

> eval(diff(theta(x,r),x),x=0);
0
> theta(L,r);
0

and the partial differential equation, Eq. (1):

> r*diff(diff(theta(x,r),x),x)+diff(r*diff(theta(x,r),r),r);
( 2 i~ − 1 ) π x ⎞
− r cos⎛⎜⎜
1
⎟⎟ ( 2 i~ − 1 ) π
2 2
4 ⎝ 2 L ⎠
⎛ ⎛ ( 2 i~ − 1 ) π r ⎞ + C4 BesselK⎛ 0, ( 2 i~ − 1 ) π r ⎞ ⎞ L 2 +
⎜⎜ C3 BesselI⎜⎜ 0, ⎟⎟ ⎜⎜ ⎟⎟ ⎟⎟
⎝ ⎝ 2L ⎠ ⎝ 2L ⎠⎠
⎛ ⎛ ( 2 i~ − 1 ) π r ⎞ ( 2 i~ − 1 ) π
⎜ C3 BesselI⎜⎜ 1, ⎟⎟
⎛ ( 2 i~ − 1 ) π x ⎞ ⎜ 1 ⎝ 2 L ⎠
cos⎜⎜ ⎟⎟ ⎜⎜
⎝ 2 L ⎠⎝ 2 L


⎛ ( 2 i~ − 1 ) π r ⎞ ⎞ ⎜
C4 BesselK⎜⎜ 1, ⎟⎟ ( 2 i~ − 1 ) π ⎟ ⎜
⎟ − π ⎜

1 ⎝ 2L ⎠ ⎟ + r cos⎜⎛ ( 2 i~ 1 ) x ⎞ ⎜
⎟ ⎜ ⎟ ⎜⎜

2 L ⎠ ⎝ 2 L ⎠⎝
⎛ ( 2 i~ − 1 ) π r ⎞ ⎞
⎜ 2 L BesselI⎛⎜⎜ 1, ⎟⎟ ⎟
⎜ ⎛ ( 2 i~ − 1 ) π r ⎞ ⎝ 2L ⎠ ⎟⎟ ( 2 i~ − 1 ) 2 π 2

C3 ⎜ BesselI⎜⎜ 0, ⎟⎟ −
( 2 i~ − 1 ) π r ⎟
1 ⎝ ⎝ 2L ⎠ ⎠ −
4 2
L
⎛ ( 2 i~ − 1 ) π r ⎞ ⎞
⎜ 2 L BesselK⎛⎜⎜ 1, ⎟⎟ ⎟


⎜ ( 2 i~ − 1 ) π r ⎞ ⎠ ⎟⎟ ( 2 i~ − 1 ) 2 π 2 ⎟
C4 ⎜⎜ −BesselK⎛⎜⎜ 0, ⎝ 2L ⎟
⎟⎟ − ⎟ ⎟
1 ⎝ ⎝ 2L ⎠ ( 2 i~ − 1 ) π r ⎠ ⎟
2 ⎟⎟
4 L ⎠
> simplify(%);
0

The general solution for θ is the series:

∞ ∞
θ = ∑ θi = ∑ cos ( λi x ) ⎡⎣C3,i BesselI ( 0,λi r ) + C4,i BesselK ( 0,λi r ) ⎤⎦ (8)
i =1 i =1

The solution must satisfy the boundary conditions in the non-homogeneous direction; the
boundary condition at r = rin, Eq. (4), leads to:

∂θ ∞
d
= ∑ cos ( λi x ) ⎡C3,i BesselI ( 0,λi r ) + C4,i BesselK ( 0,λi r ) ⎤⎦ r = r = 0 (9)
∂r r = rin i =1 dr ⎣ in

The derivatives of the Bessel functions may either be evaluated using the equations provided in
Section 1.8.4 or, more conveniently, using Maple:
> restart;
> diff(BesselI(0,lambda*r),r);
BesselI( 1, λ r ) λ
> diff(BesselK(0,lambda*r),r);
−BesselK( 1, λ r ) λ

Therefore Eq. (9) can be written as:

∑ cos ( λ x ) ⎡⎣C
i =1
i 3,i λi BesselI (1,λi rin ) − C4,i λi BesselK (1,λi rin ) ⎤⎦ = 0

which can only be true if:

C3,i λi BesselI (1,λi rin ) − C4,i λi BesselK (1,λi rin ) = 0

Unlike most of the problems we have previously encountered, neither of the constants is
eliminated; instead we see that there is a relationship between the two constants:

BesselI (1,λi rin )


C4,i = C3,i (10)
BesselK (1,λi rin )

Substituting Eq. (10) into Eq. (8) leads to:

∞ ∞ ⎡ BesselI (1,λi rin ) ⎤


θ = ∑ θi = ∑ Ci cos ( λi x ) ⎢ BesselI ( 0,λi r ) + BesselK ( 0,λi r ) ⎥ (11)
i =1 i =1 ⎣ BesselK (1,λi rin ) ⎦

Equation (11) is substituted into the remaining non-homogeneous boundary condition at r = rout ,
Eq. (5), in order to obtain:

∂θ ∞ ⎡ BesselI (1,λi rin ) ⎤


k = k ∑ Ci λi cos ( λi x ) ⎢ BesselI (1,λi rout ) − BesselK (1,λi rout ) ⎥ = q′′
∂r r = rout i =1 ⎣ BesselK (1,λi rin ) ⎦

This equation is multiplied by an arbitrary eigenfunction, cos(λj x), and integrated between the
homogeneous boundary conditions (from x = 0 to x= L); using the orthogonality property of the
eigenfunctions we obtain:

⎡ BesselI (1,λi rin ) ⎤L L


k Ci λi ⎢ BesselI (1,λi rout ) − BesselK (1,λi rout ) ⎥ ∫ cos 2 ( λi x ) dx = q′′∫ cos ( λi x ) dx
⎣ BesselK (1,λi rin ) ⎦0 0
which leads to an expression for each constant:

k Ci λi L ⎡ BesselI (1,λi rin ) ⎤ sin ( λi L )


⎢ BesselI (1,λi rout ) − BesselK (1,λi rout ) ⎥ = q′′
2 ⎣ BesselK (1,λi rin ) ⎦ λi

Solving for Ci:

2 q′′ sin ( λi L )
Ci =
⎡ BesselI (1,λi rin ) ⎤
λi2 k L ⎢ BesselI (1,λi rout ) − BesselK (1,λi rout ) ⎥
⎣ BesselK (1,λi rin ) ⎦

The inputs are entered in EES:

$UnitSystem SI MASS RAD PA K J


$Tabstops 0.2 0.4 0.6 0.8 3.5

"Inputs"
q``_dot=8000 [W/m^2] "Heat flux on pipe surface"
r_in=6.0 [cm]*convert(cm,m) "Pipe inner radius"
r_out=10.0 [cm]*convert(cm,m) "Pipe outer radius"
L = 10.0 [cm]*convert(cm,m) "Pipe half-length"
T_LOx=125 [K] "Pipe end temperature"
k=10 [W/m-K] "Pipe conductivity"

A position is specified in terms of dimensionless coordinates:

"dimensionless position"
r_bar=0.5
x_bar=0.5
r=r_in+(r_out-r_in)*r_bar
x=x_bar*L

The solution is implemented for the 1st N terms of the series:

N=100 "Number of terms"


duplicate i=1,N
lambda[i]=(2*i-1)*pi/(2*L)
C[i]=2*q``_dot*sin(lambda[i]*L)/(lambda[i]^2*k*L*(BesselI(1,lambda[i]*r_out)-&
BesselI(1,lambda[i]*r_in)*BesselK(1,lambda[i]*r_out)/BesselK(1,lambda[i]*r_in)))
theta[i]=C[i]*cos(lambda[i]*x)*(BesselI(0,lambda[i]*r)+BesselI(1,lambda[i]*r_in)*&
BesselK(0,lambda[i]*r)/BesselK(1,lambda[i]*r_in))
end
T=sum(theta[1..N])+T_LOx

The temperature distribution is computed over a range of positions and the results are used to
generate the contour plot shown in Figure 2.
Figure 2: Contour plot of temperature.
Problem 2.3-2 (2-6 in text)
Figure P2.3-2 illustrates a cylinder that is exposed to a concentrated heat flux at one end.

extends to infinity

k = 168 W/m-K

rout = 200 μm

Ts = 20°C

rexp = 21 μm

q ′′ = 1500 W/cm
2 adiabatic
Figure P2.3-2: Cylinder exposed to a concentrated heat flux at one end.

The cylinder extends infinitely in the x-direction. The surface at x = 0 experiences a uniform
heat flux of q′′ = 1500 W/cm2 for r < rexp = 21 μm and is adiabatic for rexp < r < rout where rout =
200 μm is the outer radius of the cylinder. The outer surface of the cylinder is maintained at a
uniform temperature of Ts = 20ºC. The conductivity of the cylinder material is k = 168 W/m-K.
a.) Develop a separation of variables solution for the temperature distribution within the
cylinder. Plot the temperature as a function of radius for various values of x.

The inputs are entered in EES:

$UnitSystem SI MASS RAD PA K J


$Tabstops 0.2 0.4 0.6 0.8 3.5

"Inputs"
r_out=200 [micron]*convert(micron,m) "outer radius of domain"
q``_dot=1500 [W/cm^2]*convert(W/cm^2,W/m^2) "exposure flux"
k=168 [W/m-K] "conductivity of work piece"
r_exp=21 [micron]*convert(micron,m) "radius of exposure zone"

A differential control volume leads to the energy balance:

qx + qr = qx + dx + qr + dr

or

∂qx ∂q
0= dx + r dr
∂x ∂r

Substituting the rate equations:


∂T
qx = − k 2 π r dr
∂x

and

∂T
qr = − k 2 π r dx
∂r

into the differential energy balance leads to:

∂ ⎡ ∂T ⎤ ∂ ⎡ ∂T ⎤
0= ⎢ − k 2 π r dr ⎥ dx + ⎢ −k 2 π r dx ⎥ dr
∂x ⎣ ∂x ⎦ ∂r ⎣ ∂r ⎦

or

∂ 2T ∂ ⎡ ∂T ⎤
r + r =0
∂x 2 ∂r ⎢⎣ ∂r ⎥⎦

The boundary conditions are:

∂T ⎧⎪q′′ for r < rexp


−k =⎨
∂x x =0 ⎪⎩0 for r > rexp

Tx →∞ = Ts

Tr =0 must be finite

Tr = rout = Ts

As stated, there are two non-homogeneous boundary conditions; however, the boundary
condition at r = rout can be made homogeneous by defining the temperature difference:

θ = T − Ts

The partial differential equation and boundary conditions are written in terms of θ:

∂ 2θ ∂ ⎡ ∂θ ⎤
r + r =0 (1)
∂x 2 ∂r ⎢⎣ ∂r ⎥⎦

∂θ ⎧⎪q′′ for r < rexp


−k =⎨ (2)
∂x x =0 ⎪⎩0 for r > rexp
θ x →∞ = 0 (3)

θ r =0 must be finite (4)

θ r =r = 0
out
(5)

Note that the two homogeneous boundary conditions are in the x-direction and so the
eigenfunctions of the problem will be in this direction. We assume that the solution is separable;
that is, the solution is the product of a function only of x (θX) and r (θR):

θ ( x, y ) = θ X ( x ) θ R ( r )

Substituting the product solution into the governing partial differential equation, Eq. (1), leads to:

d 2θ X d ⎡ dθ R ⎤
rθ R +θ X r =0
dx 2
dr ⎢⎣ dr ⎥⎦

Dividing by the product r θX θR leads to:

d 2θ X d ⎡ dθ R ⎤
⎢r ⎥
dx + dr ⎣ dr ⎦ = 0
2

θX rθ R
±λ2 ∓λ2

Note that the 1st term is a function only of x while the 2nd term is a function only of r; these two
quantities must be equal and opposite constants (±λ2). The choice of the sign is again important;
the eigenfunctions must be in r and therefore the two ordinary differential equations must be:

d 2θ X
− λ2 θ X = 0 (6)
dx 2

d ⎡ dθ R ⎤
⎢ r ⎥ + λ2 rθ R = 0 (7)
dr ⎣ dr ⎦

The eigenproblem will be solved first; the solution to Eq. (7) is:

θ R = C1 BesselJ ( 0,λ r ) + C2 BesselY ( 0,λ r )

The boundary condition at r = 0, Eq. (4), requires that C2 = 0.

θ R = C1 BesselJ ( 0,λ r )
The boundary condition at r = rout, Eq. (5), leads to:

C1 BesselJ ( 0,λ rout ) = 0 (8)

The 0th order Bessel function of the 1st kind (i.e., Bessel_J(0,x)) oscillates about zero every time
the argument changes by 2π in the same way that sine and cosine do; therefore, there are an
infinite number of eigenvalues λi that will satisfy Eq. (8) associated with an infinite number of
eigenfunctions. The eigencondition for this problem cannot be used to explicitly solve for the
eigenvalues; rather, an implicit equation for the eigenvalues results from Eq. (8):

BesselJ ( 0,λi rout ) = 0 where i = 1, 2,...∞ (9)

The eigenfunctions for this problem are:

θ Ri = C1,i BesselJ ( 0,λi r ) where BesselJ ( 0,λi b ) = 0 for i = 1, 2,...∞ (11)

N=51 "number of terms in solution"


duplicate i=1,N
lowerlimit[i]=(i-1)*pi/r_out "lower limit of eigenvalue"
upperlimit[i]=i*pi/r_out "upper limit of eigenvalue"
guess[i]=lowerlimit[i]+pi/(2*r_out) "guess value for eigenvalue"
end

duplicate i=1,N
BesselJ(0,lambda[i]*r_out)=0 "solve for eigenvalues"
end

The arrays lowerlimit[], upperlimit[], and guess[] are used to constrain the solution for the array
lambda[] in the Variable Information window. The solution to the ordinary differential equation
for θX, Eq. (6), is:

θ X i = C3,i exp ( λi x ) + C4,i exp ( −λi x )

The boundary condition at x → ∞, Eq. (3), requires that C3,i = 0:

θ X i = C4,i exp ( −λi x )

and so the general solution for each eigenvalue is:

θ i = θ Ri θ X i = Ci BesselJ ( 0,λi r ) exp ( −λi x )

The series solution for θ is:


∞ ∞
θ = ∑ θi = ∑ Ci BesselJ ( 0,λi r ) exp ( −λi x ) (10)
i =1 i =1

Substituting Eq. (10) into the boundary condition at x = 0, Eq. (2), leads to:

∞ ⎧⎪q′′ for r < rexp


k ∑ Ci λi BesselJ ( 0,λi r ) = ⎨ (11)
i =1 ⎪⎩0 for r > rexp

Equation (11) is multiplied by r BesselJ ( 0,λi r ) and integrated from r = 0 to r = rout:

rout rexp

k Ci λi ∫ r BesselJ ( 0,λi r ) dr = q′′


2
∫ r BesselJ ( 0,λ r ) dr
i (12)
0 0
Integral 1 Integral 2

The integrals in Eq. (12) are carried out in Maple:

> restart;
> Integral1:=int(r*(BesselJ(0,lambda[i]*r))^2,r=0..r_out);
Integral1 :=
2 2
1 r_out ( π λ i r_out BesselJ( 0, λ i r_out ) + π λ i r_out BesselJ( 1, λ i r_out ) )
2 πλ i

> Integral2:=int(r*BesselJ(0,lambda[i]*r),r=0..r_exp);
r_exp BesselJ( 1, r_exp λ i )
Integral2 :=
λi

and used to complete the solution in EES:

duplicate i=1,N
Integral1[i]=1/2*r_out/Pi^(1/2)/lambda[i]*(Pi^(1/2)*lambda[i]*r_out*BesselJ(0,lambda[i]*r_out)^2+&
Pi^(1/2)*lambda[i]*r_out*BesselJ(1,lambda[i]*r_out)^2)
Integral2[i]= 1/lambda[i]*r_exp*BesselJ(1,r_exp*lambda[i])
k*lambda[i]*C[i]*Integral1[i]=q``_dot*Integral2[i]
end

The solution is obtained at an arbitrary position according to:

x=0 [micron]*convert(micron,m) "x-position"


r=0 [micron]*convert(micron,m) "r-position"
duplicate i=1,N
theta[i]=C[i]*BesselJ(0,lambda[i]*r)*exp(-lambda[i]*x) "i'th term"
end
theta=sum(theta[1..N])
Figure 2 shows the temperature elevation relative to Ts as a function of radius for various values
of x.

1.2

Temperature difference relative to Ts (K) x=0


1
x = 5 μm
x = 10 μ m
0.8 x = 25 μ m
x = 50 μ m
0.6

0.4

0.2

0
0 20 40 60 80 100 120 140 160 180 200
Radius (μm)
Figure 2: Temperature difference as a function of r for various values of x.

b.) Determine the average temperature of the cylinder at the surface exposed to the heat flux.

The average temperature of the cylinder over the region x = 0 and 0 < r < rexp is:
rexp
1
T =
π rexp
2 ∫ 2π r T
0
x =0 dr (13)

or:
rexp
1
T = Ts +
π rexp
2 ∫ 2π r θ
0
x =0 dr (14)

Substitutinge Eq. (10) into Eq. (14) leads to:


rexp

1
T = Ts + ∫ 2 π r ∑ Ci BesselJ ( 0,λi r ) dr (15)
π rexp
2
0 i =1

which can be rearranged:


rexp

2
T = Ts + 2
rexp
∑ C ∫ r BesselJ ( 0,λ r ) dr
i =1
i i (16)
0
Integral 2
"compute average temperature in flux region"
duplicate i=1,N
theta_bar[i]=C[i]*2*Integral2[i]/r_exp^2
end
theta_bar=sum(theta_bar[1..N])

c.) Define a dimensionless thermal resistance between the surface exposed to the heat flux and
Ts. Plot the dimensionless thermal resistance as a function of rout/rin.

A dimensionless thermal resistance is defined by normalizing the actual resistance against the
reistance to axial conduction through a cylinder that is rout in radius and rout long:

k (T − Ts ) π rout k (T − Ts ) ⎛ rout
2
R π rout
2 2 ⎞
R= = = k⎜ ⎟⎟ (17)
rout q′′ π rexp rout
2
q′′ rout ⎜⎝ rexp ⎠
actual resistance

R_bar=theta_bar*k/(q``_dot*r_exp) "dimensionless thermal resistance"

Figure 3 illustrates the dimensionless thermal resistance as a function of rout/rexp.

80
Dimensionless thermal resistance

10

0.1
1 10 100
Ratio of cylinder to exposure radii, rout/rexp
Figure 3: Dimensionless thermal resistance as a function of rout/rexp.

d.) Show that your plot from (c) does not change if the problem parameters (e.g., Ts, k, etc.) are
changed.

The values of various parameters were changed and did not affect Figure 3 (the plots are overlaid
onto Figure 3).
Problem 2.3-3
A disk-shaped window in an experiment is shown in Figure P2.3-3.

h = 50 W/m -K
2

T∞ = 20°C
th = 1 cm k = 1.2 W/m-K

Rw = 3 cm
x
r Tedge = 25°C

′′ = 1000 W/m2
qrad
Figure P2.3-3: Window.

The inside of the window (the surface at x = 0) is exposed to vacuum and therefore does not
experience any convection. However, this surface is exposed to a radiation heat flux qrad ′′ = 1000
2
W/m . The window is assumed to be completely opaque to this radiation and therefore it is
absorbed at x = 0. The edge of the window at Rw = 3 cm is maintained at a constant temperature
Tedge = 25°C. The outside of the window (the surface at x = th) is cooled by air at T∞ = 20°C with
heat transfer coefficient h = 50 W/m2-K. The conductivity of the window material is k = 1.2
W/m-K.
a.) Is the extended surface approximation appropriate for this problem? That is, can the
temperature in the window be approximated as being 1-D in the radial direction? Justify
your answer.

The inputs are entered in EES:

$UnitSystem SI MASS RAD PA K J


$Tabstops 0.2 0.4 0.6 0.8 3.5

"Inputs"
th=1 [cm]*convert(cm,m) "thickness of window"
R_w=3 [cm]*convert(cm,m) "outer radius of window"
q``_rad=1000 [W/m^2] "radiation heat flux"
k=1.2 [W/m-K] "conductivity"
T_edge=converttemp(C,K,25 [C]) "edge temperature"
T_infinity=converttemp(C,K,20 [C]) "ambient temperature"
h_bar=50 [W/m^2-K] "heat transfer coefficient"

A Biot number that characterizes the resistance to conduction in the axial direction to convection
is:

h th
Bi = (1)
k

Bi=h_bar*th/k "Biot number"


which leads to Bi = 0.42; this is not sufficiently small relative to 1 to justify an extended surface
solution.

b.) Assume that your answer to (a) is no. Develop a 2-D separation of variables solution to this
problem.

A differential control volume leads to the energy balance:

qx + qr = qx + dx + qr + dr

or

∂qx ∂q
0= dx + r dr
∂x ∂r

Substituting the rate equations:

∂T
qx = − k 2 π r dr
∂x

and

∂T
qr = − k 2 π r dx
∂r

into the differential energy balance leads to:

∂ ⎡ ∂T ⎤ ∂ ⎡ ∂T ⎤
0= ⎢ − k 2 π r dr ⎥ dx + ⎢ −k 2 π r dx ⎥ dr
∂x ⎣ ∂x ⎦ ∂r ⎣ ∂r ⎦

or

∂ 2T ∂ ⎡ ∂T ⎤
r + r =0
∂x 2 ∂r ⎢⎣ ∂r ⎥⎦

The boundary conditions are:

∂T
−k ′′
= qrad
∂x x =0

∂T
−k = h (Tx =th − T∞ )
∂x x = th
Tr =0 must be finite

Tr = Rw = Tedge

As stated, there are two non-homogeneous boundary conditions; however, the boundary
condition at r = Rw can be made homogeneous by defining the temperature difference:

θ = T − Tedge

The partial differential equation and boundary conditions are written in terms of θ:

∂ 2θ ∂ ⎡ ∂θ ⎤
r 2 + ⎢r =0 (2)
∂x ∂r ⎣ ∂r ⎥⎦

∂θ
−k ′′
= qrad (3)
∂x x =0

∂θ
−k = h (θ x =th − θ ∞ ) (4)
∂x x = th

θ r =0 must be finite (5)

θr=R = 0 w
(6)

where

θ = T − Tedge (7)

and

θ ∞ = T∞ − Tedge (8)

theta_infinity=T_infinity-T_edge

Note that the two homogeneous boundary conditions are in the x-direction and so the
eigenfunctions of the problem will be in this direction. We assume that the solution is separable;
that is, the solution is the product of a function only of x (θX) and r (θR):

θ ( x, y ) = θ X ( x ) θ R ( r )

Substituting the product solution into the governing partial differential equation, Eq. (2), leads to:
d 2θ X d ⎡ dθ R ⎤
rθ R +θ X r =0
dx 2
dr ⎢⎣ dr ⎥⎦

Dividing by the product r θX θR leads to:

d 2θ X d ⎡ dθ R ⎤
⎢r ⎥
dx + dr ⎣ dr ⎦ = 0
2

θX rθ R
±λ2 ∓λ2

Note that the 1st term is a function only of x while the 2nd term is a function only of r; these two
quantities must be equal and opposite constants (±λ2). The choice of the sign is again important;
the eigenfunctions must be in r and therefore the two ordinary differential equations must be:

d 2θ X
− λ2 θ X = 0 (9)
dx 2

d ⎡ dθ R ⎤
⎢ r ⎥ + λ2 rθ R = 0 (10)
dr ⎣ dr ⎦

The eigenproblem will be solved first; the solution to Eq. (10) is:

θ R = C1 BesselJ ( 0,λ r ) + C2 BesselY ( 0,λ r )

The boundary condition at r = 0, Eq. (5), requires that C2 = 0.

θ R = C1 BesselJ ( 0,λ r )

The boundary condition at r = Rw, Eq. (6), leads to:

C1 BesselJ ( 0,λ rout ) = 0 (11)

The 0th order Bessel function of the 1st kind (i.e., Bessel_J(0,x)) oscillates about zero every time
the argument changes by 2π in the same way that sine and cosine do; therefore, there are an
infinite number of eigenvalues λi that will satisfy Eq. (11) associated with an infinite number of
eigenfunctions. The eigencondition for this problem cannot be used to explicitly solve for the
eigenvalues; rather, an implicit equation for the eigenvalues results from Eq. (11):

BesselJ ( 0,λi rout ) = 0 where i = 1, 2,...∞ (12)

The eigenfunctions for this problem are:


θ Ri = C1,i BesselJ ( 0,λi r ) where BesselJ ( 0,λi b ) = 0 for i = 1, 2,...∞ (11)

N=11 "number of terms in solution"


duplicate i=1,N
lowerlimit[i]=(i-1)*pi/R_w "lower limit of eigenvalue"
upperlimit[i]=i*pi/R_w "upper limit of eigenvalue"
guess[i]=lowerlimit[i]+pi/(2*R_w) "guess value for eigenvalue"
end

duplicate i=1,N
BesselJ(0,lambda[i]*R_w)=0 "solve for eigenvalues"
end

The arrays lowerlimit[], upperlimit[], and guess[] are used to constrain the solution for the array
lambda[] in the Variable Information window. The solution to the ordinary differential equation
for θX, Eq. (9), is:

θ X i = C3,i sinh ( λi x ) + C4,i cosh ( λi x )

The series solution for θ is:

∞ ∞
θ = ∑ θ Ri θ X i = ∑ BesselJ ( 0,λi r ) ⎡⎣C3,i sinh ( λi x ) + C4,i cosh ( λi x ) ⎤⎦
i =1 i =1

The boundary condition at x = 0, Eq. (3), leads to:


−k ∑ BesselJ ( 0,λi r ) C3,i λi = qrad
′′ (13)
i =1

Equation (13) is multiplied by r BesselJ ( 0,λi r ) and integrated from r = 0 to r = Rw

Rw Rw

−k C3,i λi ∫ r BesselJ ( 0,λ r ) dr = q′′ ∫ r BesselJ ( 0,λ r ) dr


2
i rad i (14)
0 0
Integral 1 Integral 2

The integrals in Eq. (14) are evaluated using Maple:

> restart;
> Int1[i]:=int(r*(BesselJ(0,lambda[i]*r))^2,r=0..R_w);
2 2
1 R_w ( π λ i R_w BesselJ( 0, λ i R_w ) + π λ i R_w BesselJ( 1, λ i R_w ) )
Int1i :=
2 πλ i

> Int2[i]=int(r*BesselJ(0,lambda[i]*r),r=0..R_w);
R_w BesselJ( 1, λ i R_w )
Int2i =
λi

and pasted into EES in order to obtain the constant C3,i for each eigenvalue:

duplicate i=1,N
Int1[i]=(R_w/Pi^(1/2)/lambda[i]*(Pi^(1/2)*lambda[i]*R_w*BesselJ(0,lambda[i]*R_w)^2+&
Pi^(1/2)*lambda[i]*R_w*BesselJ(1,lambda[i]*R_w)^2))/2
Int2[i] = 1/lambda[i]*R_w*BesselJ(1,lambda[i]*R_w)&
-k*C3[i]*lambda[i]*Int1[i]=q``_rad*Int2[i]
end

The boundary condition at x = th, Eq. (4), leads to:



− k ∑ BesselJ ( 0,λi r ) λi ⎡⎣C3,i cosh ( λi th ) + C4,i sinh ( λi th ) ⎤⎦ =
i =1
(15)
⎛ ∞ ⎞
h ⎜ ∑ BesselJ ( 0,λi r ) ⎡⎣C3,i sinh ( λi th ) + C4,i cosh ( λi th ) ⎤⎦ − θ ∞ ⎟
⎝ i =1 ⎠

Equation (15) is multiplied by r BesselJ ( 0,λi r ) and integrated from r = 0 to r = Rw

Rw

h θ∞ ∫ r BesselJ ( 0,λ r ) dr =
0
i

Integral 2
Rw

C4,i ⎡⎣ k λi sinh ( λi th ) + h cosh ( λi th ) ⎤⎦ ∫ r BesselJ ( 0,λ r ) dr


2
i (16)
0
Integral 1
Rw

+C3,i ⎡⎣ k λi cosh ( λi th ) + h sinh ( λi th ) ⎤⎦ ∫ r BesselJ ( 0,λ r ) dr


2
i
0
Integral 1

where the integrals were previously accomplished in Maple:

duplicate i=1,N
h_bar*theta_infinity*Int2[i]=Int1[i]*(C4[i]*(k*lambda[i]*sinh(lambda[i]*th)+&
h_bar*cosh(lambda[i]*th))+C3[i]*(k*lambda[i]*cosh(lambda[i]*th)+&
h_bar*sinh(lambda[i]*th)))
end

The solution is evaluated at an arbitrary position:

x_bar=0 [-]
r_bar=0.5 [-]
x=x_bar*th
r=r_bar*R_w
duplicate i=1,N
theta[i]=BesselJ(0,lambda[i]*r)*(C3[i]*sinh(lambda[i]*x)+C4[i]*cosh(lambda[i]*x))
end
theta=sum(theta[1..N])
T=theta+T_edge
T_C=converttemp(K,C,T)

c.) Plot the temperature as a function of r for various values of x.

Figure 2 illustrates the temperature as a function or r for various values of x.


40

38 x/th = 0
Temperature (°C)

36 x/th = 0.25

34 x/th = 0.5
x/th = 0.75
32
x/th = 1
30

28

26

24
0 0.1 0.2 0.3 0.4 0.5 0.6 0.7 0.8 0.9 1
Dimensionless radius, r/Rw
Figure 2: Temperature as a function of r for various values of x.

d.) Prepare a contour plot of the temperature in the window.

The requested contour plot is shown in Figure 3.


1

0.9 26.33
Dimensionless x position, x/th

27.66
0.8
28.98
0.7
30.31
0.6
31.64
0.5 32.97
0.4 34.3
0.3 35.62

0.2 36.95
38.28
0.1

0
0 0.1 0.2 0.3 0.4 0.5 0.6 0.7 0.8 0.9 1
Dimensionless radial position, r/Rw
Figure 3: Contour plot of the temperature in the window.
Problem 2.3-4
Reconsider Problem 2.3-3. The window is not opaque to the radiation but does absorb some of
it. The radiation that is absorbed is transformed to thermal energy. The volumetric rate of
′′ α exp ( −α x ) where α = 100 m-1 is the
thermal energy generation is given by: g ′′′ = qrad
absorption coefficient. The radiation that is not absorbed is transmitted. Otherwise the problem
remains the same.
a.) Develop a separation of variables solution to this problem using the techniques discussed in
Section 2.3.

The inputs are entered in EES:

$UnitSystem SI MASS RAD PA K J


$Tabstops 0.2 0.4 0.6 0.8 3.5

"Inputs"
th=1 [cm]*convert(cm,m) "thickness of window"
R_w=3 [cm]*convert(cm,m) "outer radius of window"
q``_rad=1000 [W/m^2] "radiation heat flux"
k=1.2 [W/m-K] "conductivity"
T_edge=converttemp(C,K,25 [C]) "edge temperature"
T_infinity=converttemp(C,K,20 [C]) "ambient temperature"
h_bar=50 [W/m^2-K] "heat transfer coefficient"
alpha=100 [1/m] "absorption coefficient"

A differential control volume leads to the energy balance:

q x + qr + g = q x + dx + qr + dr (1)

where g is the rate of generation of thermal energy. Equation (1) can be expanded:

∂q x ∂q
g = dx + r dr (2)
∂x ∂r

Substituting the rate equations:

∂T
q x = − k 2 π r dr
∂x

∂T
qr = − k 2 π r dx
∂r

and

′′ α exp ( −α x )
g = 2 π r dr dx qrad
into the differential energy balance leads to:

∂ ⎡ ∂T ⎤ ∂ ⎡ ∂T ⎤
′′ α exp ( −α x ) =
2 π r dr dx qrad ⎢ −k 2 π r dr ⎥ dx + ⎢ − k 2 π r dx ⎥ dr
∂x ⎣ ∂x ⎦ ∂r ⎣ ∂r ⎦

or

′′ α
∂ 2T ∂ ⎡ ∂T ⎤ qrad
r + ⎢r ⎥ + exp ( −α x ) r = 0 (3)
∂x 2
∂r ⎣ ∂r ⎦ k

The boundary conditions are:

∂T
−k =0 (4)
∂x x =0

∂T
−k = h (Tx =th − T∞ ) (5)
∂x x = th

Tr =0 must be finite (6)

Tr = Rw = Tedge (7)

Note that the radiant heat flux is no longer absorbed at x = 0 but rather is absorbed volumetrically
throughout the domain; it shows up as the volumetric generation in the partial differential
equation rather than as a heat flux at a boundary. With the simple transformation:

θ = T − T∞ (8)

′′ α
∂ 2θ ∂ ⎡ ∂θ ⎤ qrad
r + ⎢r ⎥ + exp ( −α x ) r = 0 (9)
∂x 2
∂r ⎣ ∂r ⎦ k

The boundary conditions are:

∂θ
−k =0 (10)
∂x x =0

∂θ
−k = h θ x =th (11)
∂x x = th
θ r =0 must be finite (12)

θ r = R = θ edge
w
(13)

where

θ edge = Tedge − T∞ (14)

theta_edge=T_edge-T_infinity

The partial differential equation is non-homogeneous and there is no simple transformation that
can rectify this. Therefore, it is necessary to use the solution technique provided in Section 2.3.
The first step is to split the solution into a homogeneous, 2-D solution that can be solved using
separation of variables and one or more 1-D particular solution:

θ = θh + X + R (15)

Substituting Eq. (15) into Eq. (9) leads to:

∂ 2θ h d 2 X ∂ ⎡ ∂θ h ⎤ d ⎡ dR ⎤ qrad ′′ α
r + r + ⎢r + r + exp ( −α x ) r = 0 (16)
∂x 2
dx 2
∂r ⎣ ∂r ⎥⎦ dr ⎢⎣ ∂r ⎥⎦ k

In order to solve the problem for θh using separation of variables it is necessary that:

∂ 2θ h ∂ ⎡ ∂θ h ⎤
r 2 + ⎢r =0 (17)
∂x ∂r ⎣ ∂r ⎥⎦

Therefore, the ODEs for X and R are:

′′ α
d 2 X qrad
2
+ exp ( −α x ) = 0 (18)
dx k

and

d ⎡ dR ⎤
r =0 (19)
dr ⎢⎣ ∂r ⎥⎦

Solving Eq. (18) leads to:

′′
dX qrad
= exp ( −α x ) + C1 (20)
dx k
′′
qrad
X =− exp ( −α x ) + C1 x + C2 (21)

Solving Eq. (19) leads to:

R = C3 ln ( r ) + C4 (22)

where C1 through C4 are undetermined constants.

In order to be able to solve θh using separation of variables it is necessary that one direction have
homogeneous boundary conditions. This problem only works if x is the homogeneous direction.
Substituting Eq. (15) into Eq. (10) leads to:

∂θ h dX
−k −k =0 (23)
∂x x =0 dx x =0

Therefore:

∂θ h
−k =0 (24)
∂x x =0

and

dX
−k =0 (25)
dx x =0

Substituting Eq. (25) into Eq. (20) leads to:

′′
qrad
+ C1 = 0 (26)
k

or:

′′
qrad
C1 = − (27)
k

C_1=-q``_rad/k

Therefore:

′′
dX qrad q ′′
= exp ( −α x ) − rad (28)
dx k k
′′
qrad q ′′
X =− exp ( −α x ) − rad x + C2 (29)
kα k

Substituting Eq. (15) into Eq. (11) leads to:

∂θ h
= h (θ h , x =th + X x =th + R )
dX
−k −k (30)
∂x x =th dx x = th

Equation (30) requires that:

∂θ h
−k = h θ h , x =th (31)
∂x x = th

as well as:

dX
−k = h X x =th (32)
dx x =th

and

R=0 (33)

Therefore C3 and C4 must both be zero. Substituting Eqs. (28) and (29) into Eq. (32) leads to:

⎡ q ′′ q ′′ ⎤ ⎡ q ′′ q ′′ ⎤
− k ⎢ rad exp ( −α th ) − rad ⎥ = h ⎢ − rad exp ( −α th ) − rad th + C2 ⎥ (34)
⎣ k k ⎦ ⎣ kα k ⎦

which can be solved for C2:

-k*(q``_rad*exp(-alpha*th)/k-q``_rad/k)=h_bar*(-q``_rad*th/k+C_2-q``_rad*exp(-alpha*th)/(k*alpha))

Finally, Eq. (15) is substituted into Eqs. (12) and (13) in order to determine the non-
homogeneous boundary conditions for θh:

θ h,r =0 + X must be finite (35)

θ h,r = R + X = θ edge
w
(36)

Therefore:

θ h,r =0 must be finite (37)

and
′′
qrad q ′′
θ h ,r = R = θ edge − exp ( −α x ) + rad x − C2 (38)
w
kα k

The problem for θh is solved using the typical steps for separation of variables. The
eigencondition is:

Bi
tan ( λi th ) = (39)
λi

where

h th
Bi = (40)
k

The eigenvalues occur in regular intervals of λ th and are automatically identified using EES:

N=11 "number of terms in solution"


duplicate i=1,N
lowerlimit[i]=(i-1)*pi "lower limit of eigenvalue"
upperlimit[i]=lowerlimit[i]+pi/2 "upper limit of eigenvalue"
guess[i]=lowerlimit[i]+pi/4 "guess value for eigenvalue"
end
duplicate i=1,N
tan(lambdath[i])=Bi/lambdath[i] "solve for eigenvalues"
lambda[i]=lambdath[i]/th "eigenvalue"
end

The series solution for θh is:


θ = ∑ cos ( λi x ) ⎡⎣C3,i BesselI ( 0,λi r ) + C4,i BesselK ( 0,λi r ) ⎤⎦ (41)
i =1

The boundary condition at r = 0, Eq. (37), leads to C4,i = 0:


θ = ∑ Ci cos ( λi x ) BesselI ( 0,λi r ) (42)
i =1

Substituting Eq. (42) into the boundary condition at r = Rw, Eq. (38), leads to:


′′
qrad q ′′
∑ C cos ( λ x ) BesselI ( 0,λ R ) = θ
i =1
i i i w edge −

exp ( −α x ) + rad x − C2
k
(43)

Multiplying Eq. (43) by cos(λj x) and integrating from x = 0 to x = th leads to:


th th
Ci BesselI ( 0,λi Rw ) ∫ cos ( λi x ) dx = (θ edge − C2 ) ∫ cos ( λi x ) dx
2


0


0

Integral 1 Integral 2
(44)
q ′′ q ′′
th th
+ rad
k ∫0 x cos ( λi x ) dx − kradα ∫0 exp ( −α x ) cos ( λi x ) dx



Integral 3 Integral 4

The integrals in Eq. (44) are carried out using Maple:

> restart;
> Int1[i]:=int((cos(lambda[i]*x))^2,x=0..th);
1 cos ( λ i th ) sin( λ i th ) + λ i th
Int1i :=
2 λi
> Int2[i]:=int(cos(lambda[i]*x),x=0..th);
sin( λ i th )
Int2i :=
λi
> Int3[i]=int(x*cos(lambda[i]*x),x=0..th);
−1 + cos ( λ i th ) + th sin( λ i th ) λ i
Int3i = 2
λi
> Int4[i]=int(exp(-alpha*x)*cos(lambda[i]*x),x=0..th);
( −α th ) ( −α th )
α−αe cos ( λ i th ) + λ i e sin( λ i th )
Int4i = 2
α 2 + λi

and copied into EES in order to evaluate the constants:

duplicate i=1,N
Int1[i] = 1/2*(cos(lambda[i]*th)*sin(lambda[i]*th)+lambda[i]*th)/lambda[i]
Int2[i] = 1/lambda[i]*sin(lambda[i]*th)
Int3[i] = (-1+cos(lambda[i]*th)+th*sin(lambda[i]*th)*lambda[i])/lambda[i]^2
Int4[i] = (alpha-alpha*exp(-alpha*th)*cos(lambda[i]*th)+&
lambda[i]*exp(-alpha*th)*sin(lambda[i]*th))/(alpha^2+lambda[i]^2)
C[i]*BesselI(0,lambda[i]*R_w)*Int1[i]=(theta_edge-C_2)*Int2[i]-C_1*Int3[i]+q``_rad*Int4[i]/(k*alpha)
end

The solution is obtained at an arbitrary position:

x_bar=0.5 [-]
r_bar=0.5 [-]
x=x_bar*th
r=r_bar*R_w
Xp=C_1*x+C_2-q``_rad*exp(-alpha*x)/(k*alpha)

duplicate i=1,N
theta_h[i]=C[i]*cos(lambda[i]*x)*BesselI(0,lambda[i]*r)
end
theta_h=sum(theta_h[1..N])
theta=theta_h+Xp
T=theta+T_infinity
T_C=converttemp(K,C,T)

b.) Plot the temperature as a function of r for various values of x.

Figure 1 illustrates the requested plot.

31
x/th = 0
30 x/th = 0.25
x/th = 0.5
x/th = 0.75
Temperature (°C)

29
x/th = 1.0
28

27

26

25

24
0 0.1 0.2 0.3 0.4 0.5 0.6 0.7 0.8 0.9 1
Dimensionless radius r/Rw
Figure 2: Temperature as a function of dimensionless radius for various values of x/th.

c.) Show that your solution limits to the solution from Problem 2.3-3 in the limit that α → ∞.

Figure 3 illustrates the temperature at x = 0 predicted by Problem 2.3-3 and by the numerical
model developed here with α → ∞.
35

34 P2.3-4 with α→∞


P2.3-3
33

Temperature (°C)
32

31

30

29

28

27

26

25
0 0.1 0.2 0.3 0.4 0.5 0.6 0.7 0.8 0.9 1
Dimensionless radius r/Rw
Figure 3: Temperature at x = 0 as a function of dimensionless radius predicted by the solution developed in
Problem 2.3-3 overlaid onto the solution developed here with α approaching infinity.
Problem 2.4-1 (2-7 in text)

The plate shown in Figure P2.4-1 is exposed to a uniform heat flux q ′′ = 1x105 W/m2 along its
top surface and is adiabatic at its bottom surface. The left side of the plate is kept at TL = 300 K
and the right side is at TR = 500 K. The height and width of the plate are H = 1 cm and W = 5
cm, respectively. The conductivity of the plate is k = 10 W/m-K.

q ′′ = 1x10 W/m
5 2

TL = 300 K W = 5 cm
TR = 500 K
y H = 1 cm
x

k = 10 W/m-K
Figure P2.4-1: Plate.

a.) Derive an analytical solution for the temperature distribution in the plate.

The inputs are entered in EES:

$UnitSystem SI MASS RAD PA K J


$TABSTOPS 0.2 0.4 0.6 0.8 3.5 in

"Inputs"
H=1 [cm]*convert(cm,m) "height of plate"
W=5 [cm]*convert(cm,m) "thickness of plate"
k=10 [W/m-K] "conductivity"
q``=100000 [W/m^2] "heat flux"
T_R=500 [K] "temperature of right hand surface"
T_L=300 [K] "temperature of left hand surface"

A mathematical statement of the transformed problem is:

∂ 2θ ∂ 2θ
+ =0 (1)
∂x 2 ∂y 2

with boundary conditions:

θ x =0 = 0 (2)

θ x =W = θ R (3)

∂θ
=0 (4)
∂y y =0
∂θ
k = q ′′ (5)
∂y y=H

where

θ = T − TL (6)

and

θ R = TR − TL (7)

The problem has two, non-homogeneous boundary condition and therefore must be solved using
superposition.

θ = θ A + θB (8)

Problem θA retains the non-homogeneous boundary condition in the x-direction:

∂ 2θ A ∂ 2θ A
+ 2 =0 (9)
∂x 2 ∂y

with boundary conditions:

θ A, x = 0 = 0 (10)

θ A, x =W = θ R (11)

∂θ A
=0 (12)
∂y y =0

∂θ A
=0 (13)
∂y y=H

By inspection, the solution for θA is 1-D in x and given by:

x
θ A = θR (14)
L

x_bar=0.5 [-] "dimensionless x location"


x=x_bar*W "x location"
y_bar=0.5 [-] "dimensionless y location"
y=y_bar*H "y location"
"sub-problem A solution"
theta_R=T_R-T_L "temperature difference of right hand
surface"
theta_A=theta_R*x/W "solution for sub-problem A"

Problem θB retains the non-homogeneous boundary condition in the y-direction:

∂ 2θ B ∂ 2θ B
+ 2 =0 (15)
∂x 2 ∂y

with boundary conditions:

θ B , x =0 = 0 (16)

θ B , x =W = 0 (17)

∂θ B
=0 (18)
∂y y =0

∂θ B
k = q ′′ (19)
∂y y=H

The solution for θB is 2-D and can be obtained using separation of variables. The eigenfunctions
are:

θ X B ,i = sin ( λB ,i x ) (20)

where the eigenvalues are:

iW
λB , i = for i = 1, 2,..∞ (21)
π

"sub-problem B solution"
N_term=11 [-] "number of terms"
duplicate i=1,N_term
lambda_B[i]=i*pi/W "i'th eigenvalue"
end

The solution in the non-homogeneous direction is:

θ YB ,i = C2,i cosh ( λB ,i y ) (22)

The series solution for θB is:



θ B = ∑ Ci sin ( λB ,i x ) cosh ( λB ,i y ) (23)
i =1

Subsituting Eq. (23) into Eq. (19) leads to:


k ∑ λB ,i Ci sin ( λB ,i x ) sinh ( λB ,i H ) = q ′′ (24)
i =1

Equation (24) is multiplied by an eigenfunction and integrated from x = 0 to x = W:


W W
k λB ,i Ci sinh ( λB ,i H ) ∫ sin ( λB ,i x ) dx = q ′′ ∫ sin ( λB ,i x ) dx
2
(25)
0 0

The integrals in Eq. (25) are evaluated in Maple:

> restart;
> assume(i,integer);
> lambda_B:=i*Pi/W;
i~ π
lambda_B :=
W
> int((sin(lambda_B*x))^2,x=0..W);
W
2
> int(sin(lambda_B*x),x=0..W);
W ( −1 + ( -1 )i~ )

i~ π

and used to evaluate the constants:

duplicate i=1,N_term
C[i]*k*lambda_B[i]*sinh(lambda_B[i]*H)*W/2=q``*(-W*(-1+(-1)^i)/i/Pi) "i'th constant"
end

The solution for θB is obtained:

duplicate i=1,N_term
theta_B[i]=C[i]*sin(lambda_B[i]*x)*cosh(lambda_B[i]*y) "i'th term"
end
theta_B=sum(theta_B[1..N_term]) "solution to sub-problem B"

and used to obtain the solution for T:

theta=theta_A+theta_B "superposition of solutions"


T=theta+T_L "temperature"
b.) Prepare a contour plot of the temperature.

Figure 2 illustrates a contour plot of the temperature distribution in the plate.


1
300
Dimensionless y-position, y/H

345.5
0.8
390.9
436.4
0.6 481.8
527.3
572.7
0.4
618.2
663.6
0.2 709.1
754.5

0
0 0.1 0.2 0.3 0.4 0.5 0.6 0.7 0.8 0.9 1
Dimensionless x-position, x/W
Figure 2: Contour plot of the temperature distribution.
Problem 2.5-1 (2-8 in text): A Heating Element
Figure P2.5-1 illustrates an electrical heating element that is affixed to the wall of a chemical
reactor. The element is rectangular in cross-section and very long (into the page). The
temperature distribution within the element is therefore two-dimensional, T(x, y). The width of
the element is a = 5.0 cm and the height is b = 10.0 cm. The three edges of the element that are
exposed to the chemical (at x = 0, y = 0, and x = a) are maintained at a temperature Tc = 200°C
while the upper edge (at y = b) is affixed to the well-insulated wall of the reactor and can
therefore be considered adiabatic. The element experiences a uniform volumetric rate of thermal
energy generation, g ′′′ = 1x106 W/m3. The conductivity of the material is k = 0.8 W/m-K.

reactor wall

k = 0.8 W/m-K
Tc = 200°C
g ′′′ = 1x10 W/m
6 3

a = 5 cm
y
Tc = 200°C
x

Tc = 200°C b = 10 cm
Figure P2.5-1: Electrical heating element.

a.) Develop a 2-D numerical model of the element using EES.

The inputs are entered in EES:

$UnitSystem SI MASS RAD PA K J


$TABSTOPS 0.2 0.4 0.6 0.8 3.5 in

"Inputs"
a=5.0 [cm]*convert(cm,m) "width of element"
b=10.0 [cm]*convert(cm,m) "height of element"
k=0.8 [W/m-K] "conductivity"
T_c=converttemp(C,K,200 [C]) "chemical temperature"
g```_dot=1e6 [W/m^3] "rate of volumetric generation"
L=1 [m] "unit length of element into the page"

The computational domain of the element with the regularly spaced grid of nodes is shown in
Figure 2.
Figure 2: The regularly spaced grid used to obtain a numerical solution.

The first step in obtaining a numerical solution is to position the nodes throughout the
computational domain. We will use grid with nodes placed on the edges and distributed
uniformly throughout. The x and y distance between adjacent nodes (Δx and Δy) are:

L
Δx = (1)
( m − 1)
b
Δy = (2)
( n − 1)

and the x and y positions of any node i,j are given by:

xi =
( i − 1) a (3)
( m − 1)

yj =
( j − 1) b (4)
( n − 1)
where m and n are the number of nodes used in the x and y directions.

"Setup nodes"
m=11 [-]
"number of nodes in the x-direction"
n=11 [-]
"number of nodes in the y-direction"
Dx=a/(m-1)
"distance between nodes in the x-direction"
Dy=b/(n-1)
"distance between nodes in the y-direction"
duplicate i=1,m
x[i]=(i-1)*Dx "x-position of each node"
end
duplicate j=1,n
y[j]=(j-1)*Dy "y-position of each node"
end

The next step in the solution is to write an energy balance for each node. Figure 3 illustrates a
control volume and the associated energy transfers for an internal node (see Figure 2) which
include conduction from each side ( q RHS and q LHS ), the top ( qtop ), and the bottom ( qbottom ). Note
that the direction associated with these energy transfers is arbitrary (i.e., they could have been
taken as positive if energy leaves the control volume), but it is important to write the equation in
a manner consistent with the chosen directions.

Figure 3: Energy balance for an internal node

The energy balance suggested by Figure 3 is:

q RHS [i, j ] + q LHS [i, j ] + qtop [i, j ] + qbottom [i, j ] + g [i, j ] = 0 (5)

The next step is to approximate each of the terms in the energy balance; the material separating
the nodes is assumed to behave as a plane wall resistance and therefore:

k Δy L
q RHS [i, j ] =
Δx
(T [i + 1, j ] − T [i, j ]) (6)

where L is the length of the element (assumed to be 1 m in order to do the problem on a unit
length basis); therefore, Δy L is the area for conduction and Δx is the distance over which the
conduction heat transfer occurs. Note that the temperature difference is consistent with the
direction of the arrow in Figure 3; if Ti+1,j is greater than Ti,j then energy is leaving the node and
q RHS is positive. The other heat transfers are approximated using a similar model:
k Δy L
q LHS [i, j ] =
Δx
(T [i − 1, j ] − T [i, j ]) (7)

k ΔxW
qtop [i, j ] =
Δy
(T [i, j + 1] − T [i, j ]) (8)

k ΔxW
qbottom [i, j ] =
Δy
(T [i, j − 1] − T [i, j ]) (9)

The generation is the product of the volume of the control volume and the volumetric rate of
generation:

g [i, j ] = g ′′′ Δx Δy L (10)

These equations are entered in EES using a nested duplicate statement:

"Internal node control volumes"


duplicate i=2,(m-1)
duplicate j=2,(n-1)
q_dot_LHS[i,j]+q_dot_RHS[i,j]+q_dot_top[i,j]+q_dot_bottom[i,j]+gen[i,j]=0
q_dot_LHS[i,j]=k*L*Dy*(T[i-1,j]-T[i,j])/Dx
q_dot_RHS[i,j]=k*L*Dy*(T[i+1,j]-T[i,j])/Dx
q_dot_top[i,j]=k*L*Dx*(T[i,j+1]-T[i,j])/Dy
q_dot_bottom[i,j]=k*L*Dx*(T[i,j-1]-T[i,j])/Dy
gen[i,j]=Dx*Dy*L*g```_dot
end
end

Note that each time the outer duplicate statement iterates once (i.e., i is increased by 1), the inner
duplicate statement iterates (n-1) times (i.e., j runs from 2 to n-1). Therefore, all of the internal
nodes are considered with these two nested duplicate loops. Also note that the unknowns are
placed in an array rather than a vector. The entries in the array T is accessed using two indices
contained in square brackets.

The boundary nodes have to be treated separately. The left, right, and bottom boundaries are
easy as the temperature is specified:

T [1, j ] = Tc for j = 1...n (11)

T [ m, j ] = Tc for j = 1...n (12)


T [i,1] = Tc for i = 1...m (13)

These equations are entered in EES:


"Temperature along right edge"
duplicate j=1,n
T[1,j]=T_c
end

"Temperature along left edge"


duplicate j=1,n
T[m,j]=T_c
end

"Temperature along bottom edge"


duplicate i=2,(m-1)
T[i,1]=T_c
end

The upper boundary nodes must be considered using energy balances. Figure 4 illustrates an
energy balance associated with a node that is located on the top, insulated boundary (see Figure
2).

Figure 4: Energy balance for a node on the top boundary

The energy balance suggested by Figure 4 is:

q RHS [i, n ] + q LHS [i, n ] + qbottom [i, n ] + g [i, n ] = 0 (14)

The conduction terms in the x direction must be approximated slightly differently:

k Δy L
q RHS [i, n ] =
2Δx
(T [i + 1, n] − T [i, n]) (15)

k Δy L
q LHS [i, n ] =
2 Δx
(T [i − 1, n] − T [i, n]) (16)

The factor of 2 in the denominator appears because there is half the available area for conduction
through the sides of the control volume on the top boundary. The other conduction term is
approximated as before:

k ΔxW
qbottom [i, n ] =
Δy
(T [i, n − 1] − T [i, n]) (17)
The generation term is:

Δx Δy L g ′′′
g [i, n ] = (18)
2

These equations are entered in EES using a single duplicate statement:

"Upper edge"
duplicate i=2,(m-1)
q_dot_LHS[i,n]+q_dot_RHS[i,n]+q_dot_bottom[i,n]+gen[i,n]=0
q_dot_LHS[i,n]=k*(Dy/2)*L*(T[i-1,n]-T[i,n])/Dx
q_dot_RHS[i,n]=k*(Dy/2)*L*(T[i+1,n]-T[i,n])/Dx
q_dot_bottom[i,n]=k*Dx*L*(T[i,n-1]-T[i,n])/Dy
gen[i,n]=Dx*Dy*L*g```_dot/2
end

Notice that the control volumes at the top left corner (i.e., i =1, j =n) and the top right corner
(i.e., i =m, j =n) have already been taken addressed by the equations for the left and right
boundaries, Eqs. (11) and (12). Therefore, we have to make sure not to write additional
equations related to this node or the problem will be over-specified and so the equations for the
top boundary can only be written for i = 2...(m-1).

We have derived a total of m x n equations in the m x n unknown temperatures; these equations


completely specify the problem and they have now all been entered in EES. Therefore, a
solution can be obtained by solving the EES code. The solution is contained in the Arrays
Window; each column of the table corresponds to the temperatures associated with one value of i
and all of the value of j (i.e., the temperatures in a column are at a constant value of y and
varying values of x).

b.) Plot the temperature as a function of x at various values of y. What is the maximum
temperature within the element and where is it located?

The solution is obtained for n=11 and the columns Ti,1 (corresponding to y =0) through Ti,11
(corresponding to y = 10 cm) are plotted in Figure 5 as a function of x.
Figure 5: Temperature as a function of x for various values of y.

The hottest spot in the element is at the adiabatic wall (y = 10 cm) and the center (x = 2.5 cm);
the hottest temperature is about 860 K.

c.) Prepare a reality check to show that your solution behaves according to your physical
intuition. That is, change some aspect of your program and show that the results behave as
you would expect (clearly describe the change that you made and show the result).

There are several possible answers to this; I increased the conductivity by a factor of 10 and
examined the temperature distribution. Figure 6 illustrates the temperature as a function of x for
the original conductivity (k = 0.8 W/m-K) and the increased conductivity (k = 8.0 W/m-K);
notice that the increased conductivity has had the expected effect of reducing the temperature
rise.
Figure 6: Temperature as a function of x for y = 10 cm and two values of conductivity.
Problem 2.5-2
Figure P2.5-2 illustrates a composite material that is being machined on a lathe.

composite
thins = 100 μm
kins = 1.5 W/m-K
thm = 200 μm
km = 35 W/m-K h depends on RS
T∞ = 20°C
q′′
l
Tchuck = 20°C

y
x

W = 12 cm
H = 3 cm

Figure P2.5-2: Composite material being machined on a lathe.

The composite is composed of alternating layers of insulating material and metal. The insulating
layers have thickness thins= 100 μm and conductivity kins = 1.5 W/m-K. The metal layers have
thickness thm = 200 μm and conductivity km = 35 W/m-K. The workpiece is actually cylindrical
and rotating. However, because the radius is large relative to it thickness and there are no
circumferential variations we can model the workpiece as a 2-D problem in Cartesian
coordinates, x and y, as shown in Figure 2.5-2. The width of the workpiece is W = 12 cm and the
thickness is H = 3 cm. The left surface of the workpiece at x = 0 is attached to the chuck and
therefore maintained at Tchuck = 20°C. The inner surface at y = 0 is insulated. The outer surface
(at y = H) and right surface (at x = W) are exposed to air at T∞ = 20°C with heat transfer
coefficient h that depends on the rotational speed of the chuck, RS in rev/min, according to:

⎡ W min 2 ⎤ 2
h = 2⎢ 2 2⎥
RS
⎣ m K rev ⎦

In order to extend the life of the tool used for the machining process, the workpiece is preheated
by applying laser power to the outer surface. The heat flux applied by the laser depends on the
rotational speed and position according to:

⎡ ⎛ x − x ⎞2 ⎤
ql′′ = a RS exp ⎢ - ⎜ c
⎟ ⎥
⎢⎣ ⎝ pw ⎠ ⎥⎦

where a = 5000 W-min/m2-rev, xc = 8 cm, and pw = 1 cm.


a.) What is the effective thermal conductivity of the composite in the x- and y-directions?

The inputs are entered in EES and a function is defined in order to provide the heat flux:
$UnitSystem SI MASS RAD PA K J
$TABSTOPS 0.2 0.4 0.6 0.8 3.5 in

function q_f(RS,x)
"Inputs
RS - rotational speed (rpm)
x - position (m)

Output
q_f - laser heat flux (W/m^2)"
a=5000 [W-min/m^2-rev]
x_c=8 [cm]*convert(cm,m)
pw=1 [cm]*convert(cm,m)
q_f=a*RS*exp(-((x-x_c)/pw)^2)
end

"Inputs"
th_ins=100 [micron]*convert(micron,m) "insulation layer thickness"
k_ins=1.5 [W/m-K] "insulation conductivity"
th_m=200 [micron]*convert(micron,m) "metal thickness"
k_m=35 [W/m-K] "metal conductivity"
W=12 [cm]*convert(cm,m) "width of work piece"
H=3 [cm]*convert(cm,m) "thickness of work piece"
T_chuck=20 [C] "chuck temperature"
T_infinity=20 [C] "ambient air temperature"
RS=60 [rev/min] "rotational speed"

The heat transfer coefficient ( h ) is evaluated using the equation provided in the problem
statement.

h_bar=2 [W-min^2/m^2-K-rev^2]*RS^2 "heat transfer coefficient"

The effective conductivities of a lamination stack was derived in Section 2.9.1. The effective
conductivity in the x-direction is:

keff , x =
( km thm + kins thins ) (1)
( thm + thins )
and the conductivity in the y-direction is:

keff , y =
( thm + thins ) (2)
thm thins
+
km kins

"Determine effective conductivities"


k_x=(k_m*th_m+k_ins*th_ins)/(th_m+th_ins) "effective conductivity in the x-direction"
k_y=(th_m+th_ins)/(th_m/k_m+th_ins/k_ins) "effective conductivity in the y-direction"

which leads to keff,x = 23.83 W/m-K and keff,y = 4.15 W/m-K.


b.) Develop a 2-D numerical model of the workpiece in EES. Plot the temperature as a function
of x at various values of y, including at least y = 0, H/2, and H.

A regularly spaced grid of nodes is uniformly distributed with the first and last nodes in each
dimension placed on the boundaries of the domain; the x- and y-positions of any node (i,j) are
given by:

xi = ( i − 1) Δx for i = 1..M (3)

y j = ( j − 1) Δy for j = 1..N (4)

where M and N are the number of nodes used in the x- and y-directions, respectively. The x- and
y-distance between adjacent nodes (Δx and Δy, respectively) are:

W
Δx = (5)
( M − 1)
H
Δy = (6)
( N − 1)
This information is entered in EES:

"Setup nodes"
M=21 [-] "number of nodes in the x-direction"
N=11 [-] "number of nodes in the y-direction"
Dx=W/(m-1) "distance between nodes in the x-direction"
Dy=H/(n-1) "distance between nodes in the y-direction"
duplicate i=1,m
x[i]=(i-1)*Dx "x-position of each node"
end
duplicate j=1,n
y[j]=(j-1)*Dy "y-position of each node"
end

The next step in the solution is to write an energy balance for each node. The energy transfers
for an internal node include conduction from each side ( q RHS and q LHS ), the top ( qtop ), and the
bottom ( qbottom ). The direction associated with these energy transfers is arbitrary but they are all
taken as being into the control volume, as was done in Section 2.5.2.

q RHS + q LHS + qtop + qbottom = 0 (7)

The next step is to approximate each of the terms in the energy balance:
Δy Δy Δx Δx
keff , x
Δx
( Ti +1, j − Ti , j ) + keff , x
Δx
( Ti −1, j − Ti , j ) + keff , y
Δy
( Ti , j +1 − Ti , j ) + keff , y
Δy
(Ti, j −1 − Ti, j ) = 0 (8)
for i = 2... ( M − 1) and j = 2... ( N − 1)

These equations are entered in EES using nested duplicate statements:

"Internal node energy balances"


duplicate i=2,(M-1)
duplicate j=2,(N-1)
Dy*k_x*(T[i+1,j]-T[i,j])/Dx+Dy*k_x*(T[i-1,j]-T[i,j])/Dx+Dx*k_y*(T[i,j+1]-T[i,j])/Dy+&
Dx*k_y*(T[i,j-1]-T[i,j])/Dy=0
end
end

The left boundary (x= 0) has a specified temperature:

T1, j = Tchuck for j = 1...N (2-9)

where Tchuck is the chuck temperature. These equations are entered in EES:

"left boundary"
duplicate j=1,N
T[1,j]=T_chuck
end

The remaining boundary nodes do not have specified temperatures and therefore must be
determined using energy balances. An energy balance on a node that is located on the top
boundary (at y= H) is:

q RHS + q LHS + qbottom + qconv + ql = 0 (2-10)

which leads to:

keff , x Δy keff , x Δy keff , y Δx


2Δx
(Ti +1, N − Ti , N ) +
2 Δx
(T
i −1, N − Ti , N ) +
Δy
(T
i , N −1 − Ti , N )

Δx h
+
k
(T∞ − Ti, N ) + ql′′Δx=0 (2-11)

i = 2... ( M − 1)

These equations are entered in EES using a single duplicate statement:

"top boundary"
duplicate i=2,(M-1)
Dy*k_x*(T[i+1,N]-T[i,N])/(2*Dx)+Dy*k_x*(T[i-1,N]-T[i,N])/(2*Dx)+&
Dx*k_y*(T[i,N-1]-T[i,N])/Dy+Dx*h_bar*(T_infinity-T[i,N])+q_f(RS,x[i])*Dx=0
end
A similar procedure for the nodes on the lower boundary leads to:

keff , x Δy keff , x Δy keff , y Δx


2Δx
(T
i +1,1 − Ti ,1 ) +
2 Δx
(Ti −1,1 − Ti ,1 ) +
Δy
(T i ,2 − Ti ,1 ) =0 for i = 2... ( M − 1) (2-12)

"bottom boundary"
duplicate i=2,(M-1)
Dy*k_x*(T[i+1,1]-T[i,1])/(2*Dx)+Dy*k_x*(T[i-1,1]-T[i,1])/(2*Dx)+Dx*k_y*(T[i,2]-T[i,1])/Dy=0
end

Energy balances for the nodes on the right-hand boundary (x = L) lead to:

keff , y Δx keff , y Δx keff , x Δy


2Δy
(T M , j +1 − TM , j ) +
2 Δy
(T M , j −1 − TM , j ) +
Δx
(T
M −1, j − TM , j )
(2-13)
+Δy h (T∞ − TM , j ) =0 for j = 2... ( N − 1)

"right boundary"
duplicate j=2,(N-1)
Dx*k_y*(T[M,j+1]-T[M,j])/(2*Dy)+Dx*k_y*(T[M,j-1]-T[M,j])/(2*Dy)+&
Dy*k_x*(T[M-1,j]-T[M,j])/Dx+Dy*h_bar*(T_infinity-T[M,j])=0
end

The two corners (right upper and right lower) have to be considered separately. A control
volume and energy balance for node (M,N) leads to:

keff , y Δx keff , x Δy
2 Δy
(T M , N −1 − TM , N ) +
2 Δx
(T
M −1, N − TM , N )
(2-14)
Δx Δy
+h
2
( T∞ − TM , N ) +h
2
(T∞ − TM , N ) =0
"upper right corner"
Dx*k_y*(T[M,N-1]-T[M,N])/(2*Dy)+Dy*k_x*(T[M-1,N]-T[M,N])/(2*Dx)+&
h_bar*Dx*(T_infinity-T[M,N])/2+h_bar*Dy*(T_infinity-T[M,N])/2=0

The energy balance for the right lower boundary, node (M,1), leads to:

Δx Δy Δy
2 Δy
( TM ,2 − TM ,1 ) +
2 Δx
( TM −1,1 − TM ,1 ) + h
2
(T∞ − TM ,1 ) = 0 (2-15)

"lower right corner"


Dx*k_y*(T[M,2]-T[M,1])/(2*Dy)+Dy*k_x*(T[M-1,1]-T[M,1])/(2*Dx)+h_bar*Dy*(T_infinity-T[M,1])/2=0

We have derived a total of M x N equations in the M x N unknown temperatures; these equations


completely specify the problem and they have now all been entered in EES. Therefore, a
solution can be obtained by solving the EES code. The solution is contained in the Arrays
window; each column of the table corresponds to the temperatures associated with one value of i
and all of the value of j (i.e., the temperatures in a column are at a constant value of y and
varying values of x). Figure 2 illustrates the temperature as a function of x at various values of y
and shows the heating caused by the laser energy at the top surface.
250

y = 30 mm
Temperature (°C) 200

150
y = 24 mm
y = 18 mm

100

50
y = 0 mm
y = 6 mm
y = 12 mm
0
0 0.02 0.04 0.06 0.08 0.1 0.12
Axial position (m)
Figure 2: Temperature as a function of x for various values of y.

c.) Plot the maximum temperature in the workpiece as a function of the rotational speed, RS. If
the objective is to preheat the material to its maximum possible temperature, then what is the
optimal rotational speed?

The maximum temperature (Tmax) occurs on the upper boundary (j = N) and is identified
according to:

T_max=max(T[1..M,N])

Figure 3 illustrates the maximum temperature as a function of rotational speed and shows that
the maximum preheating occurs when the rotational speed is approximately 12 rpm.
90
Maximum temperature (°C)

80

70

60

50

40

30
0 10 20 30 40 50 60
Rotational speed (rev/min)
Figure 3: Maximum temperature as a function of the rotational speed.
Problem 2.5-3
In Section 1.6 the constant cross-section, straight fin shown in Figure P2.5-3 was analyzed under
the assumption that it could be treated as an extended surface (i.e., temperature gradients in the y
direction can be neglected). In this example, the 2-D temperature distribution within the fin will
be determined using separation of variables.

T∞ , h
W

insulated tip
y
x
g ′′′
Tb L
th
Figure P2.5-3: Straight, constant cross-sectional area fin.

Assume that the tip of the fin is insulated and that the width (W) is much larger than the
thickness (th) so that convection from the edges can be neglected. The length of the fin is L =
5.0 cm. The fin base temperature is Tb = 20ºC and the fin experiences convection with fluid at
T∞= 100ºC with average heat transfer coefficient, h = 100 W/m2-K. The fin is th = 2.0 cm thick
and has conductivity k = 1.5 W/m-K.
a.) Develop a numerical solution for the temperature distribution in the fin using a finite
difference technique.

The inputs are entered in EES:

$UnitSystem SI MASS RAD PA K J


$Tabstops 0.2 0.4 0.6 3.5 in

L=5 [cm]*convert(cm,m) "length of fin"


T_b=20 [C] "base temperature"
T_infinity=100 [C] "fluid temperature"
h_bar=100 [W/m^2-K] "average heat transfer coefficient"
th=3 [cm]*convert(cm,m) "fin thickness"
k=1.5 [W/m-K] "fin conductivity"
g```_dot=5e5 [W/m^3] "volumetric generation"
W=1 [m] "per unit width into page"

A regularly spaced grid of nodes is uniformly distributed with the first and last nodes in each
dimension placed on the boundaries of the domain; the x- and y-positions of any node (i,j) are
given by:

xi =
( i − 1) L (1)
( M − 1)
yj =
( j − 1) th (2)
2 ( N − 1)

where M and N are the number of nodes used in the x- and y-directions, respectively. The x- and
y-distance between adjacent nodes (Δx and Δy, respectively) are:

L
Δx = (3)
( M − 1)

th
Δy = (4)
2 ( N − 1)

N=7 [-] "number of nodes in the y-direction"


M=11 [-] "number of nodes in the x-direction"
Dx=L/(M-1)
Dy=(th/2)/(N-1) "y-distance between adjacent nodes"
duplicate i=1,M
x[i]=L*(i-1)/(M-1) "x-position of each node"
end
duplicate j=1,N
y[j]=(th/2)*(j-1)/(N-1) "y-position of each node"
end

An energy balance on an internal node is:

q RHS ,i , j + q LHS ,i , j + qtop ,i , j + qbottom ,i , j + g i , j = 0


(5)
for i = 2.. ( M − 1) and j = 2.. ( N − 1)

where

k ΔyW
q RHS ,i , j =
Δx
(Ti+1, j − Ti, j ) (6)

k ΔyW
q LHS ,i , j =
Δx
(Ti −1, j − Ti, j ) (7)

k ΔxW
qtop ,i , j =
Δy
(Ti, j +1 − Ti, j ) (8)

k ΔxW
qbottom ,i , j =
Δy
(Ti, j −1 − Ti, j ) (9)

g i , j = Δx ΔyW g ′′′ (10)


"Internal nodes"
duplicate i=2,(M-1)
duplicate j=2,(N-1)
q_dot_LHS[i,j]+q_dot_RHS[i,j]+q_dot_top[i,j]+q_dot_bottom[i,j]+g_dot[i,j]=0
q_dot_LHS[i,j]=k*W*Dy*(T[i-1,j]-T[i,j])/Dx
q_dot_RHS[i,j]=k*W*Dy*(T[i+1,j]-T[i,j])/Dx
q_dot_top[i,j]=k*W*Dx*(T[i,j+1]-T[i,j])/Dy
q_dot_bottom[i,j]=k*W*Dx*(T[i,j-1]-T[i,j])/Dy
g_dot[i,j]=W*Dx*Dy*g```_dot
end
end

The left boundary (x= 0) has a specified temperature:

T1, j = Tb for j = 1...N (11)

"nodes on left edge"


duplicate j=1,N
T[1,j]=T_b
end

An energy balance on the nodes on the upper edge leads to:

q RHS ,i , N + q LHS ,i , N + qconv ,i , N + qbottom ,i , N + g i , N = 0


(12)
for i = 2.. ( M − 1)

where

k ΔyW
q RHS ,i , N =
2 Δx
(Ti+1, N − Ti, N ) (13)

k ΔyW
q LHS ,i , N =
2 Δx
(Ti−1, N − Ti, N ) (14)

qconv ,i , N = h W Δx (T∞ − Ti , N ) (15)

k ΔxW
qbottom ,i , N =
Δy
(Ti, N −1 − Ti, N ) (16)

Δy
g i , N = Δx W g ′′′ (17)
2

"nodes on upper edge"


duplicate i=2,(M-1)
q_dot_LHS[i,N]+q_dot_RHS[i,N]+q_dot_conv[i,N]+q_dot_bottom[i,N]+g_dot[i,N]=0
q_dot_LHS[i,N]=k*W*(Dy/2)*(T[i-1,N]-T[i,N])/Dx
q_dot_RHS[i,N]=k*W*(Dy/2)*(T[i+1,N]-T[i,N])/Dx
q_dot_conv[i,N]=h_bar*W*Dx*(T_infinity-T[i,N])
q_dot_bottom[i,N]=k*W*Dx*(T[i,N-1]-T[i,N])/Dy
g_dot[i,N]=Dx*(Dy/2)*W*g```_dot
end

This process is continued for all of the edge and corner nodes:

"upper right corner node"


q_dot_LHS[M,N]+q_dot_conv[M,N]+q_dot_bottom[M,N]+g_dot[M,N]=0
q_dot_LHS[M,N]=k*W*(Dy/2)*(T[M-1,N]-T[M,N])/Dx
q_dot_conv[M,N]=h_bar*W*(Dx/2)*(T_infinity-T[M,N])
q_dot_bottom[M,N]=k*W*(Dx/2)*(T[M,N-1]-T[M,N])/Dy
g_dot[M,N]=(Dx/2)*(Dy/2)*W*g```_dot

"nodes on right edge"


duplicate j=2,(N-1)
q_dot_LHS[M,j]+q_dot_top[M,j]+q_dot_bottom[M,j]+g_dot[M,j]=0
q_dot_LHS[M,j]=k*W*Dy*(T[M-1,j]-T[M,j])/Dx
q_dot_top[M,j]=k*W*(Dx/2)*(T[M,j+1]-T[M,j])/Dy
q_dot_bottom[M,j]=k*W*(Dx/2)*(T[M,j-1]-T[M,j])/Dy
g_dot[M,j]=(Dx/2)*Dy*W*g```_dot
end

"lower right corner node"


q_dot_LHS[M,1]+q_dot_top[M,1]+g_dot[M,1]=0
q_dot_LHS[M,1]=k*W*(Dy/2)*(T[M-1,1]-T[M,1])/Dx
q_dot_top[M,1]=k*W*(Dx/2)*(T[M,2]-T[M,1])/Dy
g_dot[M,1]=(Dx/2)*(Dy/2)*W*g```_dot

"nodes on bottom edge"


duplicate i=2,(M-1)
q_dot_LHS[i,1]+q_dot_RHS[i,1]+q_dot_top[i,1]+g_dot[i,1]=0
q_dot_LHS[i,1]=k*W*(Dy/2)*(T[i-1,1]-T[i,1])/Dx
q_dot_RHS[i,1]=k*W*(Dy/2)*(T[i+1,1]-T[i,1])/Dx
q_dot_top[i,1]=k*W*Dx*(T[i,2]-T[i,1])/Dy
g_dot[i,1]=Dx*(Dy/2)*W*g```_dot
end

b.) Use the numerical solution to predict and plot the temperature distribution in the heater.

The temperature as a function of x for various values of y is shown in Figure 2.


200

180
y = 0 (center)
160

Temperature (°C)
140
y = th/2 (edge)
120

100

80

60

40

20
0 0.01 0.02 0.03 0.04 0.05
Axial position (m)
Figure 2: Temperature as a function of x for various values of y.

c.) Use the numerical solution to predict the heater efficiency; the heater efficiency is defined as
the ratio of the rate of heat transfer to the fluid to the total rate of thermal energy generation
in the fin.

The convection heat transfer from node 1,N is computed:

Δx
qconv ,1, N = h W
2
(T∞ − T1, N ) (18)

and the total convective heat transfer from the heater is:

M
qhtr = −2 ∑ qconv ,i , N (19)
i =1

The heater efficiency is:

qhtr
ηhtr = (20)
g ′′′W L th

q_dot_conv[1,N]=h_bar*W*(Dx/2)*(T_infinity-T[1,N]) "convection to upper left corner node"


q_dot_htr=-2*sum(q_dot_conv[1..M,N]) "fin heat transfer rate"
eta_htr=q_dot_htr/(W*L*th*g```_dot) "heater efficiency"

which leads to ηhtr = 0.357.

d.) Plot the heater efficiency as a function of the fin length for various values of the fin
thickness. Explain your plot.

Figure 3 illustrates the heater efficiency as a function of length for various values of thickness.
Note that at small lengths the fin efficiency can become negative because the heater thermally
communicates with the wall and is unable to elevate the temperature above the fluid
temperature. Therefore, the fluid is actually cooled by the heater.
1

0.8
th = 3 cm
0.6 th = 5 cm
Heater efficiency 0.4

0.2 th = 7 cm
0 th = 9 cm
-0.2

-0.4

-0.6

-0.8
0 0.02 0.04 0.06 0.08 0.1 0.12 0.14 0.16 0.18 0.2
Heater length (m)
Figure 3: Heater efficiency as a function of length for various values of thickness.
Problem 2.6-1 (2-9 in text): Model of Welding Process (revisited)
Figure P2.6-1 illustrates a cut-away view of two plates that are being welded together. Both
edges of the plate are clamped and effectively held at temperatures Ts = 25°C. The top of the
plate is exposed to a heat flux that varies with position x, measured from joint, according to:
qm′′ ( x ) = q ′′j exp ( − x / L j ) where q ′′j =1x106 W/m2 is the maximum heat flux (at the joint, x = 0)
and Lj = 2.0 cm is a measure of the extent of the heat flux. The back side of the plates are
exposed to liquid cooling by a jet of fluid at Tf = -35°C with h = 5000 W/m2-K. A half-
symmetry model of the problem is shown in Figure P2.6-1. The thickness of the plate is b = 3.5
cm and the width of a single plate is W = 8.5 cm. You may assume that the welding process is
steady-state and 2-D. You may neglect convection from the top of the plate. The conductivity
of the plate material is k = 38 W/m-K.
both edges are held at fixed temperature
heat flux
joint

qm′′ impingement cooling

k = 38 W/m-K

W = 8.5 cm Ts = 25°C
b = 3.5 cm
y

h = 5000 W/m -K, T f = −35°C


2

Figure P2.6-1: Welding process and half-symmetry model of the welding process.

a.) Develop a separation of variables solution to the problem (note, this was done previously in
Problem 2.2-1). Implement the solution in EES and prepare a plot of the temperature as a
function of x at y = 0, 1.0, 2.0, 3.0, and 3.5 cm.
b.) Prepare a contour plot of the temperature distribution.

See the solution for Problem 2.2-1 for parts (a) and (b).

c.) Develop a numerical model of the problem. Implement the solution in MATLAB and
prepare a contour or surface plot of the temperature in the plate.

The input parameters are entered in the MATLAB function P2p2_1; the input arguments are m
and n, the number of nodes in the x and y coordinates while the output arguments are the x and y
positions of each node and the predicted temperature at each node.

function[xm,ym,T]=P2p6_1(m,n)

W=0.085; %width of plate (m)


b=0.035; %thickness of plate (m)
k=38; %conductivity of plate material (W/m-K)
T_s=298.1; %side temperature (K)
T_f=238.2; %fluid temperature (K)
h=5000; %heat transfer coefficient (W/m^2-K)
L=1; %per unit length (m)

end

A sub-function is defined to provide the heat flux on the upper surface:

function[qflux]=qf(x)

L_j=0.02; %length scale (m)


qf_j=1e6; %heat flux at center (W/m^2)

qflux=qf_j*exp(-x/L_j);

end

A 2-D numerical model will be generated using a grid in which the x and y coordinates of each
node are:

xi =
( i − 1)W for i = 1..m (1)
( m − 1)

yi =
( j − 1)W for j = 1..n (2)
( m − 1)
The distance between adjacent nodes is:

W
Δx = (3)
( m − 1)
W
Δy = (4)
( n − 1)
%Setup grid
for i=1:m
x(i,1)=(i-1)*W/(m-1);
end
Dx=W/(m-1);
for j=1:n
y(j,1)=(j-1)*W/(n-1);
end
Dy=b/(n-1);
The problem will be solved by setting up and inverting a matrix containing the algebraic
equations that enforce the conservation of energy for each control volume. A control volume for
an internal node includes conduction from the left and right sides ( q LHS and q RHS ) and top and
bottom ( qtop and qbottom ). The energy balance is:

q RHS + q LHS + qtop + qbottom = 0

The conduction terms are approximated according to:

k Δy L
q RHS =
Δx
(Ti−1, j − Ti, j )
k Δy L
q LHS =
Δx
(Ti+1, j − Ti, j )
k Δx L
qbottom =
Δy
(Ti, j −1 − Ti, j )

k Δx L
qtop =
Δy
(Ti, j +1 − Ti, j )
where L is the depth of the plate (into the page); L is set to 1.0 m which is consistent with doing
the problem on a per unit length basis. Combining these equations leads to:

k Δy L k Δy L k Δx L k Δx L
Δx
( Ti −1, j − Ti , j ) +
Δx
( Ti +1, j − Ti , j ) +
Δy
( Ti , j −1 − Ti , j ) +
Δy
(Ti, j +1 − Ti, j ) = 0 (5)
for i = 2.. ( m − 1) and j = 2.. ( n − 1)

The equation is rearranged to make it clear what the coefficient is for each unknown
temperature:

⎡ k Δy L k Δx L ⎤ ⎡ k Δy L ⎤ ⎡ k Δy L ⎤ ⎡ k Δx L ⎤ ⎡ k Δx L ⎤
Ti , j ⎢ −2 −2 ⎥ + Ti −1, j ⎢ ⎥ + Ti +1, j ⎢ ⎥ + Ti , j −1 ⎢ ⎥ + Ti , j +1 ⎢ ⎥=0
⎣ Δx Δy ⎦ ⎣ Δx ⎦ ⎣ Δx ⎦ ⎣ Δy ⎦ ⎣ Δy ⎦
for i = 2.. ( m − 1) and j = 2.. ( n − 1)
(6)

The control volume equations must be placed into the matrix equation:

AX =b
where the equation for control volume i,j is placed into row m(j-1)+i of A and Ti,j corresponds to
element Xm (j-1)+i in the vector X . Therefore, each coefficient in Eq. (6) (i.e., each term
multiplying an unknown temperature on the left side of the equation) must be placed in the row
of A corresponding to the control volume being examined and the column of A corresponding
to the unknown in X . The matrix assignments consistent with Eq. (6) are:

k Δy L k Δx L
Am( j −1)+i ,m( j −1)+i = −2 −2 for i = 2.. ( m − 1) and j = 2.. ( n − 1) (7)
Δx Δy

k Δy L
Am( j −1)+i ,m( j −1)+i −1 = for i = 2.. ( m − 1) and j = 2.. ( n − 1) (8)
Δx

k Δy L
Am( j −1)+i ,m( j −1)+i +1 = for i = 2.. ( m − 1) and j = 2.. ( n − 1) (9)
Δx

k Δx L
Am( j −1)+i ,m( j −1−1)+i = for i = 2.. ( m − 1) and j = 2.. ( n − 1) (10)
Δy

k Δx L
Am( j −1)+i ,m( j +1−1)+i = for i = 2.. ( m − 1) and j = 2.. ( n − 1) (11)
Δy

A sparse matrix is allocated in MATLAB for A and the equations derived above are
implemented using a nested for loop. The spalloc command requires three arguments, which are
the number of rows and columns and the maximum number of elements in the matrix. Note that
there are at most 5 non-zero entries in each row of A , corresponding to Eqs. (7) through (11);
thus the last argument in the spalloc command which corresponds to the maximum number of
non-zero entries in the sparse matrix.

A=spalloc(m*n,m*n,5*m*n); %allocate a sparse matrix for A


bm=zeros(m*n,1); %allocate a matrix for b
%energy balances for internal nodes
for i=2:(m-1)
for j=2:(n-1)
A(m*(j-1)+i,m*(j-1)+i)=-2*k*Dy*L/Dx-2*k*Dx*L/Dy;
A(m*(j-1)+i,m*(j-1)+i-1)=k*Dy*L/Dx;
A(m*(j-1)+i,m*(j-1)+i+1)=k*Dy*L/Dx;
A(m*(j-1)+i,m*(j-1-1)+i)=k*Dx*L/Dy;
A(m*(j-1)+i,m*(j+1-1)+i)=k*Dx*L/Dy;
end
end

The nodes on the right side have a specified temperature:

Tm, j = Ts for j = 1..n


The matrix assignments suggested by these equations are:

Am( j −1)+ m, m( j −1)+ m = 1 for j = 1..n

bm( j −1)+ m = Tst for j = 1..n

These assignments are implemented in MATLAB:

%right side temperature is specified


for j=1:n
A(m*(j-1)+m,m*(j-1)+m)=1;
bm(m*(j-1)+m,1)=T_s;
end

The nodes along the upper edge must be considered separately, leading to:

⎡ k Δy L k Δx L ⎤ ⎡ k Δy L ⎤ ⎡ k Δy L ⎤ ⎡ k Δx L ⎤
Ti ,n ⎢ −
Δx

Δy ⎦ ⎥ + Ti −1, n ⎢ ⎥ + Ti +1, n ⎢ ⎥ + Ti ,n −1 ⎢ ⎥ = − qm′′ Δx L
⎣ ⎣ 2 Δx ⎦ ⎣ 2 Δx ⎦ ⎣ Δy ⎦ (12)
for i = 2.. ( m − 1)

which is expressed in matrix form as:

k Δy L k Δx L
Am( n −1)+i ,m( n −1)+i = − − for i = 2.. ( m − 1) (13)
Δx Δy

k Δy L
Am( n −1)+i ,m( n −1)+i −1 = for i = 2.. ( m − 1) (14)
2 Δx

k Δy L
Am( n −1)+i ,m( n −1)+i +1 = for i = 2.. ( m − 1) (15)
2 Δx

k Δx L
Am( n −1)+i , m( n −1−1)+i = for i = 2.. ( m − 1) (16)
Δy

bm( n −1)+i ,1 = −qm′′ Δx L for i = 2.. ( m − 1) (17)

%upper edge
for i=2:(m-1)
A(m*(n-1)+i,m*(n-1)+i)=-k*Dy*L/Dx-k*Dx*L/Dy;
A(m*(n-1)+i,m*(n-1)+i-1)=k*Dy*L/(2*Dx);
A(m*(n-1)+i,m*(n-1)+i+1)=k*Dy*L/(2*Dx);
A(m*(n-1)+i,m*(n-1-1)+i)=k*Dx*L/Dy;
bm(m*(n-1)+i,1)=-qf(x(i))*Dx*L;
end
The node at the upper left corner must be considered separately, leading to:

⎡ k Δy L k Δx L ⎤ ⎡ k Δy L ⎤ ⎡ k Δx L ⎤ Δx L
T1,n ⎢ − − ⎥ + T2,n ⎢ ⎥ + T1,n −1 ⎢ ⎥ = − qm′′ (18)
⎣ 2 Δx 2 Δy ⎦ ⎣ 2 Δx ⎦ ⎣ 2 Δy ⎦ 2

which is expressed in matrix form as:

k Δy L k Δx L
Am( n −1)+1,m( n −1)+1 = − − (19)
2 Δx 2 Δy

k Δy L
Am( n −1) +1,m( n −1)+1+1 = (20)
2 Δx

k Δx L
Am( n −1)+1, m( n −1−1)+1 = (21)
2 Δy

Δx L
bm( n −1)+1,1 = −qm′′ (22)
2

%upper left corner


A(m*(n-1)+1,m*(n-1)+1)=-k*Dy*L/(2*Dx)-k*Dx*L/(2*Dy);
A(m*(n-1)+1,m*(n-1)+1+1)=k*Dy*L/(2*Dx);
A(m*(n-1)+1,m*(n-1-1)+1)=k*Dx*L/(2*Dy);
bm(m*(n-1)+1,1)=-qf(x(1))*Dx*L/2;

The node along the left edge must be considered separately, leading to:

⎡ k Δy L k Δx L ⎤ ⎡ k Δy L ⎤ ⎡ k Δx L ⎤ ⎡ k Δx L ⎤
T1, j ⎢ − − ⎥ + T2 +1, j ⎢ ⎥ + T1, j −1 ⎢ ⎥ + T1, j +1 ⎢ ⎥=0
⎣ Δx Δy ⎦ ⎣ 2 Δx ⎦ ⎣ 2 Δy ⎦ ⎣ 2 Δy ⎦ (23)
for j = 2.. ( n − 1)

which is expressed in matrix form as:

k Δy L k Δx L
Am( j −1)+1,m( j −1)+1 = − − for j = 2.. ( n − 1) (24)
Δx Δy

k Δy L
Am( j −1)+1,m( j −1)+1+1 = for j = 2.. ( n − 1) (25)
Δx

k Δx L
Am( j −1) +1,m( j −1−1)+1 = for j = 2.. ( n − 1) (26)
2 Δy
k Δx L
Am( j −1)+1,m( j +1−1)+1 = for j = 2.. ( n − 1) (27)
2 Δy

%left edge
for j=2:(n-1)
A(m*(j-1)+1,m*(j-1)+1)=-k*Dy*L/Dx-k*Dx*L/Dy;
A(m*(j-1)+1,m*(j-1)+1+1)=k*Dy*L/Dx;
A(m*(j-1)+1,m*(j-1-1)+1)=k*Dx*L/(2*Dy);
A(m*(j-1)+1,m*(j+1-1)+1)=k*Dx*L/(2*Dy);
end

The node at the lower left corner must be considered separately, leading to:

⎡ k Δy L k Δ x L Δx L ⎤ ⎡ k Δy L ⎤ ⎡ k Δx L ⎤ Δx L
T1,1 ⎢ − − −h ⎥ + T2,1 ⎢ ⎥ + T1,2 ⎢ ⎥ = −h Tf (28)
⎣ 2 Δx 2 Δy 2 ⎦ ⎣ 2 Δx ⎦ ⎣ 2 Δy ⎦ 2

which is expressed in matrix form as:

k Δy L k Δx L Δx L
Am(1−1) +1,m(1−1)+1 = − − −h (29)
2 Δx 2 Δy 2

k Δy L
Am(1−1)+1, m(1−1)+1+1 = (30)
2 Δx

k Δx L
Am(1−1)+1, m(1+1−1)+1 = (31)
2 Δy

Δx L
bm(1−1)+1,1 = −h Tf (32)
2

%lower left corner


A(m*(1-1)+1,m*(1-1)+1)=-k*Dy*L/(2*Dx)-k*Dx*L/(2*Dy)-h*L*Dx/2;
A(m*(1-1)+1,m*(1-1)+1+1)=k*Dy*L/(2*Dx);
A(m*(1-1)+1,m*(1+1-1)+1)=k*Dx*L/(2*Dy);
bm(m*(1-1)+1,1)=-h*L*Dx*T_f/2;

The nodes along the bottom edge must be considered separately, leading to:

⎡ k Δy L k Δx L ⎤ ⎡ k Δy L ⎤ ⎡ k Δy L ⎤ ⎡ k Δx L ⎤
Ti ,1 ⎢ − − − h Δx L ⎥ + Ti −1,1 ⎢ ⎥ + Ti +1,1 ⎢ ⎥ + Ti ,1+1 ⎢ ⎥ = − h Δx LT f
⎣ Δx Δy ⎦ ⎣ 2 Δx ⎦ ⎣ 2 Δx ⎦ ⎣ Δy ⎦ (33)
for i = 2.. ( m − 1)

which is expressed in matrix form as:


k Δy L k Δx L
Am(1−1)+i ,m(1−1)+i = − − - h Δx L for i = 2.. ( m − 1) (34)
Δx Δy

k Δy L
Am(1−1) +i , m(1−1)+i −1 = for i = 2.. ( m − 1) (35)
2 Δx

k Δy L
Am(1−1) +i , m(1−1)+i +1 = for i = 2.. ( m − 1) (36)
2 Δx

k Δx L
Am(1−1) +i , m(1+1−1)+i = for i = 2.. ( m − 1) (37)
Δy

bm(1−1)+i ,1 = − h Δx LT f for i = 2.. ( m − 1) (38)

%lower edge
for i=2:(m-1)
A(m*(1-1)+i,m*(1-1)+i)=-k*Dy*L/Dx-k*Dx*L/Dy-h*Dx*L;
A(m*(1-1)+i,m*(1-1)+i-1)=k*Dy*L/(2*Dx);
A(m*(1-1)+i,m*(1-1)+i+1)=k*Dy*L/(2*Dx);
A(m*(1-1)+i,m*(1+1-1)+i)=k*Dx*L/Dy;
bm(m*(1-1)+i,1)=-h*Dx*L*T_f;
end

The vector of unknowns X is obtained through matrix manipulation and then placed into matrix
format. Matrices for the x and y positions of each node are also created.

X=A\bm;
for i=1:m
for j=1:n
xm(i,j)=x(i);
ym(i,j)=y(j);
T(i,j)=X(m*(j-1)+i);
end
end

A surface plot of the result is obtained by typing:

>> [x,y,T]=P2p6_1(20,40);
>> surf(x,y,T);

The plot is shown in Figure 3.


Figure 3: Surface plot of temperature in the plate.

b.) Plot the temperature as a function of x at y = 0, b/2, and b and overlay on this plot the
separation of variables solution obtained in part (a) evaluated at the same locations.

The comparison is shown in Figure 4.

Figure 4: Temperature as a function of axial position for y = 0, b/2, and b predicted using the
separation of variables solution and the numerical solution.
Problem 2.6-2
Prepare a solution to Problem 2.3-3 using a finite difference technique.
a.) Plot the temperature as a function of r for various values of x.

The input parameters are entered in the MATLAB script P2p2_2.

clear all;
th=0.01; % thickness of window (m)
h_bar=50; % heat transfer coefficient (W/m^2-K)
T_infinity=20; % ambient temperature (C)
T_edge=25; % edge temperature (C)
k=1.2; % conductivity (W/m-K)
R_w=0.03; % window radius (m)
qf_rad=1000; % radiation heat flux (W/m^2)

A 2-D numerical model will be generated using a grid in which the r and x coordinates of each
node are:

ri =
( i − 1) RW for i = 1..M (1)
( M − 1)

xj =
( j − 1) th for j = 1..N (2)
( N − 1)
The distance between adjacent nodes is:

RW
Δr = (3)
( M − 1)
th
Δx = (4)
( N − 1)
% Setup grid
M=51; % number of r-nodes
for i=1:M
r(i,1)=(i-1)*R_w/(M-1);
end
Dr=R_w/(M-1); % distance between r-nodes
N=101; % number of x-nodes
for j=1:N
x(j,1)=(j-1)*th/(N-1);
end
Dx=th/(N-1); % distance between x-nodes

The problem will be solved by setting up and inverting a matrix containing the algebraic
equations that enforce the conservation of energy for each control volume. A control volume for
an internal node includes conduction from the left and right sides ( q LHS and q RHS ) and top and
bottom ( qtop and qbottom ). The energy balance is:

q RHS + q LHS + qtop + qbottom = 0

The conduction terms are approximated according to:

⎛ Δr ⎞
k 2 π ⎜ ri + ⎟ Δx
⎝ 2 ⎠
q RHS =
Δr
(Ti +1, j − Ti, j )

⎛ Δr ⎞
k 2 π ⎜ ri − ⎟ Δx
⎝ 2 ⎠
q LHS =
Δr
(Ti−1, j − Ti, j )
k 2 π ri Δr
qbottom =
Δx
(Ti, j −1 − Ti, j )
k 2 π ri Δr
qtop =
Δx
(Ti, j +1 − Ti, j )
Combining these equations leads to:
⎛ Δr ⎞ ⎛ Δr ⎞
k 2 π ⎜ ri + ⎟ Δx k 2 π ⎜ ri − ⎟ Δx

Δr
2 ⎠
( Ti +1, j − Ti , j ) + ⎝
Δr
2 ⎠
(Ti−1, j − Ti, j )
k 2 π ri Δr k 2 π ri Δr
+
Δx
( Ti , j −1 − Ti , j ) +
Δx
(Ti, j +1 − Ti, j ) = 0 (5)

for i = 2.. ( M − 1) and j = 2.. ( N − 1)

The equation is rearranged to make it clear what the coefficient is for each unknown
temperature:
⎡ ⎛ Δr ⎞ ⎛ Δr ⎞ ⎤
⎢ k 2 π ⎜ ri − 2 ⎟ Δx k 2 π ⎜ ri + 2 ⎟ Δx k 4 π r Δr ⎥
Ti , j ⎢ − ⎝ ⎠ − ⎝ ⎠ − i
⎥+
⎢ Δr Δr Δx ⎥
⎢⎣ ⎥⎦
⎡ ⎛ Δr ⎞ ⎤ ⎡ ⎛ Δr ⎞ ⎤
⎢ k 2 π ⎜ ri − 2 ⎟ Δx ⎥ ⎢ k 2 π ⎜ ri + 2 ⎟ Δx ⎥
Ti −1, j ⎢ ⎝ ⎠ ⎥ +T ⎢ ⎝ ⎠ ⎥+ (6)
i +1, j
⎢ Δ r ⎥ ⎢ Δr ⎥
⎢⎣ ⎥⎦ ⎢⎣ ⎥⎦
⎡ k 2 π ri Δr ⎤ ⎡ k 2 π ri Δr ⎤
Ti , j −1 ⎢ ⎥ + Ti , j +1 ⎢ ⎥=0
⎣ Δx ⎦ ⎣ Δx ⎦
for i = 2.. ( M − 1) and j = 2.. ( N − 1)

The control volume equations must be placed into the matrix equation:

AX =b

where the equation for control volume i,j is placed into row M (j-1) + i of A and Ti,j corresponds
to element XM(j-1)+i in the vector X . Therefore, each coefficient in Eq. (6) (i.e., each term
multiplying an unknown temperature on the left side of the equation) must be placed in the row
of A corresponding to the control volume being examined and the column of A corresponding
to the unknown in X . The matrix assignments consistent with Eq. (6) are:

⎛ Δr ⎞ ⎛ Δr ⎞
k 2 π ⎜ ri − ⎟ Δx k 2 π ⎜ ri + ⎟ Δx k 4 π r Δr
AM ( j −1)+i , M ( j −1)+i =− ⎝ 2 ⎠
− ⎝ 2 ⎠
− i
(7)
Δr Δr Δx
for i = 2.. ( M − 1) and j = 2.. ( N − 1)

⎛ Δr ⎞
k 2 π ⎜ ri − ⎟ Δx
AM ( j −1)+i , M ( j −1)+i −1 = ⎝ 2 ⎠
for i = 2.. ( M − 1) and j = 2.. ( N − 1) (8)
Δr

⎛ Δr ⎞
k 2 π ⎜ ri + ⎟ Δx
AM ( j −1)+i , M ( j −1)+i +1 = ⎝ 2 ⎠
for i = 2.. ( M − 1) and j = 2.. ( N − 1) (9)
Δr

k 2 π ri Δr
AM ( j −1)+i , M ( j −1−1)+i = for i = 2.. ( M − 1) and j = 2.. ( N − 1) (10)
Δx

k 2 π ri Δr
AM ( j −1)+i , M ( j +1−1)+i = for i = 2.. ( M − 1) and j = 2.. ( N − 1) (11)
Δx
A sparse matrix is allocated in MATLAB for A and the equations derived above are
implemented using a nested for loop. The spalloc command requires three arguments, which are
the number of rows and columns and the maximum number of elements in the matrix. Note that
there are at most 5 non-zero entries in each row of A , corresponding to Eqs. (7) through (11);
thus the last argument in the spalloc command which corresponds to the maximum number of
non-zero entries in the sparse matrix.

A=spalloc(M*N,M*N,5*M*N);
b=zeros(M*N,1);
for i=2:(M-1)
for j=2:(N-1)
A(M*(j-1)+i,M*(j-1)+i)=-k*2*pi*Dx*(r(i)-Dr/2)/Dr-...
k*2*pi*Dx*(r(i)+Dr/2)/Dr-k*4*pi*r(i)*Dr/Dx;
A(M*(j-1)+i,M*(j-1)+i-1)=k*2*pi*Dx*(r(i)-Dr/2)/Dr;
A(M*(j-1)+i,M*(j-1)+i+1)=k*2*pi*Dx*(r(i)+Dr/2)/Dr;
A(M*(j-1)+i,M*(j+1-1)+i)=k*2*pi*r(i)*Dr/Dx;
A(M*(j-1)+i,M*(j-1-1)+i)=k*2*pi*r(i)*Dr/Dx;
end
end

The nodes on the right side have a specified temperature:

TM , j = Tedge for j = 1..N

The matrix assignments suggested by these equations are:

AM ( j −1)+ M , M ( j −1)+ M = 1 for j = 1..N

bM ( j −1)+ M = Tedge for j = 1..N

These assignments are implemented in MATLAB:

for j=1:N
A(M*(j-1)+M,M*(j-1)+M)=1;
b(M*(j-1)+M,1)=T_edge;
end

The nodes along the bottom edge must be considered separately, leading to:

⎡ k π Δx ⎛ Δr ⎞ k π Δ x ⎛ Δr ⎞ k 2 π ri Δr ⎤
Ti ,1 ⎢ − ⎜ ri − ⎟ − ⎜ ri + ⎟ −
⎣ Δr ⎝ 2 ⎠ Δr ⎝ 2 ⎠ Δx ⎥⎦
⎡ k π Δx ⎛ Δr ⎞ ⎤ ⎡ k π Δx ⎛ Δr ⎞ ⎤ ⎡ k 2 π ri Δr ⎤
+Ti −1,1 ⎢ ⎜ ri − ⎟ ⎥ + Ti +1,1 ⎢ ⎜ ri + ⎟ ⎥ + Ti ,2 ⎢ ′′ 2 π ri Δr
⎥ = − qrad (12)
⎣ Δr ⎝ 2 ⎠⎦ ⎣ Δr ⎝ 2 ⎠⎦ ⎣ Δx ⎦
for i = 2.. ( M − 1)
for i=2:(M-1)
A(M*(1-1)+i,M*(1-1)+i)=-k*pi*Dx*(r(i)-Dr/2)/Dr-...
k*pi*Dx*(r(i)+Dr/2)/Dr-k*2*pi*r(i)*Dr/Dx;
A(M*(1-1)+i,M*(1-1)+i-1)=k*pi*Dx*(r(i)-Dr/2)/Dr;
A(M*(1-1)+i,M*(1-1)+i+1)=k*pi*Dx*(r(i)+Dr/2)/Dr;
A(M*(1-1)+i,M*(2-1)+i)=k*2*pi*r(i)*Dr/Dx;
b(M*(1-1)+i,1)=-qf_rad*2*pi*r(i)*Dr;
end

The nodes along the top edge must be considered separately, leading to:

⎡ k π Δx ⎛ Δr ⎞ k π Δ x ⎛ Δr ⎞ k 2 π ri Δr ⎤
Ti , N ⎢ − ⎜ ri − ⎟ − ⎜ ri + ⎟ −
⎣ Δr ⎝ 2 ⎠ Δr ⎝ 2 ⎠ Δx ⎥⎦
⎡ k π Δx ⎛ Δr ⎞ ⎤ ⎡ k π Δx ⎛ Δr ⎞ ⎤ ⎡ k 2 π ri Δr ⎤
+Ti −1, N ⎢ Δr ⎜ ri − 2 ⎟ ⎥ + Ti +1, N ⎢ Δr ⎜ ri + 2 ⎟ ⎥ + Ti , N −1 ⎢ Δx ⎥ = − h 2 π ri Δr T∞ (13)
⎣ ⎝ ⎠⎦ ⎣ ⎝ ⎠⎦ ⎣ ⎦
for i = 2.. ( M − 1)

for i=2:(M-1)
A(M*(N-1)+i,M*(N-1)+i)=-k*pi*Dx*(r(i)-Dr/2)/Dr-k*pi*Dx*(r(i)+Dr/2)/Dr-
k*2*pi*r(i)*Dr/Dx-h_bar*2*pi*r(i)*Dr;
A(M*(N-1)+i,M*(N-1)+i-1)=k*pi*Dx*(r(i)-Dr/2)/Dr;
A(M*(N-1)+i,M*(N-1)+i+1)=k*pi*Dx*(r(i)+Dr/2)/Dr;
A(M*(N-1)+i,M*(N-1-1)+i)=k*2*pi*r(i)*Dr/Dx;
b(M*(N-1)+i,1)=-h_bar*2*pi*r(i)*Dr*T_infinity;
end

The nodes along the left side leads to:

⎡ k 2 π Δx ⎛ Δr ⎞ ⎤ ⎡ k 2 π Δx ⎛ Δr ⎞ ⎤
T1, j ⎢ − ⎜ r1 + ⎟ ⎥ + T2, j ⎢ ⎜ r1 + ⎟ =0
⎣ Δr ⎝ 2 ⎠⎦ ⎣ Δr ⎝ 2 ⎠ ⎦⎥ (14)
for j = 2.. ( N − 1)

for j=2:(N-1)
A(M*(j-1)+1,M*(j-1)+1)=-k*2*pi*(r(1)+Dr/2)*Dx/Dr;
A(M*(j-1)+1,M*(j-1)+2)=k*2*pi*(r(1)+Dr/2)*Dx/Dr;
end

The node at the lower and upper left corners lead to:

⎡ k π Δx ⎛ Δr ⎞ ⎤ ⎡ k π Δx ⎛ Δr ⎞ ⎤
T1,1 ⎢ − ⎜ r1 + ⎟ ⎥ + T2,1 ⎢ ⎜ r1 + ⎟ =0 (15)
⎣ Δr ⎝ 2 ⎠⎦ ⎣ Δr ⎝ 2 ⎠ ⎥⎦

⎡ k π Δx ⎛ Δr ⎞ ⎤ ⎡ k π Δx ⎛ Δr ⎞ ⎤
T1, N ⎢ − ⎜ r1 + ⎟ ⎥ + T2, N ⎢ ⎜ r1 + ⎟ =0 (16)
⎣ Δr ⎝ 2 ⎠⎦ ⎣ Δr ⎝ 2 ⎠ ⎥⎦

A(M*(1-1)+1,M*(1-1)+1)=-k*pi*(r(1)+Dr/2)*Dx/Dr;
A(M*(1-1)+1,M*(1-1)+2)=k*pi*(r(1)+Dr/2)*Dx/Dr;
A(M*(N-1)+1,M*(N-1)+1)=-k*pi*(r(1)+Dr/2)*Dx/Dr;
A(M*(N-1)+1,M*(N-1)+2)=k*pi*(r(1)+Dr/2)*Dx/Dr;

The vector of unknowns X is obtained through matrix manipulation and then placed into matrix
format.

X=A\b;
for i=1:M
for j=1:N
T(i,j)=X(M*(j-1)+i);
end
end

The temperature as a function of radius at various values of x is shown in Figure 1.

40
x/th = 0
38
x/th = 0.25
36 x/th = 0.5
Temperature (°C)

34

32

30
x/th = 0.75
28 x/th = 1

26 Problem 2.6-2
Problem 2.3-3
24
0 0.005 0.01 0.015 0.02 0.025 0.03
Radius (m)
Figure 1: Temperature as a function of radius at various values of axial position.

b.) Prepare a contour plot of the temperature in the window.

A contour plot of the temperature in the window is shown in Figure 2.


0.03

36
0.025

34
0.02
Axial position (m)

32

0.015

30

0.01

28

0.005

26

0
0 0.001 0.002 0.003 0.004 0.005 0.006 0.007 0.008 0.009 0.01
Radius (m)

Figure 2: Contour plot of the temperature.

c.) Verify that your solution agrees with the analytical solution from Problem 2.3-3.

Figure 1 illustrates the solution obtained from Problem 2.3-3 and shows that the agreement is
nearly exact.
Problem 2.6-3
Prepare a solution to Problem 2.3-4 using a finite difference technique.
a.) Plot the temperature as a function of r for various values of x.

The input parameters are entered in the MATLAB script P2p6d3.

clear all;
th=0.01; % thickness of window (m)
h_bar=50; % heat transfer coefficient (W/m^2-K)
T_infinity=20; % ambient temperature (C)
T_edge=25; % edge temperature (C)
k=1.2; % conductivity (W/m-K)
R_w=0.03; % window radius (m)
qf_rad=1000; % radiation heat flux (W/m^2)
alpha=100; % absorption coefficient (1/m)

A 2-D numerical model will be generated using a grid in which the r and x coordinates of each
node are:

ri =
( i − 1) RW for i = 1..M (1)
( M − 1)

xj =
( j − 1) th for j = 1..N (2)
( N − 1)
The distance between adjacent nodes is:

RW
Δr = (3)
( M − 1)
th
Δx = (4)
( N − 1)
% Setup grid
M=51; % number of r-nodes
for i=1:M
r(i,1)=(i-1)*R_w/(M-1);
end
Dr=R_w/(M-1); % distance between r-nodes
N=101; % number of x-nodes
for j=1:N
x(j,1)=(j-1)*th/(N-1);
end
Dx=th/(N-1); % distance between x-nodes

The problem will be solved by setting up and inverting a matrix containing the algebraic
equations that enforce the conservation of energy for each control volume. A control volume for
an internal node includes conduction from the left and right sides ( q LHS and q RHS ) and top and
bottom ( qtop and qbottom ). The energy balance is:

q RHS + q LHS + qtop + qbottom + g = 0

The conduction terms are approximated according to:

⎛ Δr ⎞
k 2 π ⎜ ri + ⎟ Δx
⎝ 2 ⎠
q RHS =
Δr
(Ti +1, j − Ti, j )

⎛ Δr ⎞
k 2 π ⎜ ri − ⎟ Δx
⎝ 2 ⎠
q LHS =
Δr
(Ti−1, j − Ti, j )
k 2 π ri Δr
qbottom =
Δx
(Ti, j −1 − Ti, j )
k 2 π ri Δr
qtop =
Δx
(Ti, j +1 − Ti, j )

′′ α exp ( −α x j )
g = 2 π ri Δr Δx qrad

Combining these equations leads to:

⎛ Δr ⎞ ⎛ Δr ⎞
k 2 π ⎜ ri + ⎟ Δx k 2 π ⎜ ri − ⎟ Δx
⎝ 2 ⎠
Δr
( Ti +1, j − Ti , j ) + ⎝
Δr
2 ⎠
(Ti−1, j − Ti, j )
k 2 π ri Δr k 2 π ri Δr
+
Δx
( Ti , j −1 − Ti , j ) +
Δx
(Ti, j +1 − Ti, j ) = −2 π ri Δr Δx qrad
′′ α exp ( −α x j ) (5)

for i = 2.. ( M − 1) and j = 2.. ( N − 1)

The equation is rearranged to make it clear what the coefficient is for each unknown
temperature:
⎡ ⎛ Δr ⎞ ⎛ Δr ⎞ ⎤
⎢ k 2 π ⎜ ri − 2 ⎟ Δx k 2 π ⎜ ri + 2 ⎟ Δx k 4 π r Δr ⎥
Ti , j ⎢ − ⎝ ⎠ − ⎝ ⎠ − i
⎥+
⎢ Δr Δr Δx ⎥
⎢⎣ ⎥⎦
⎡ ⎛ Δr ⎞ ⎤ ⎡ ⎛ Δr ⎞ ⎤
⎢ k 2 π ⎜ ri − 2 ⎟ Δx ⎥ ⎢ k 2 π ⎜ ri + 2 ⎟ Δx ⎥
Ti −1, j ⎢ ⎝ ⎠ ⎥ +T ⎢ ⎝ ⎠ ⎥+ (6)
i +1, j
⎢ Δ r ⎥ ⎢ Δr ⎥
⎢⎣ ⎥⎦ ⎢⎣ ⎥⎦
⎡ k 2 π ri Δr ⎤ ⎡ k 2 π ri Δr ⎤
Ti , j −1 ⎢ ⎥ + Ti , j +1 ⎢ ⎥ = −2 π ri Δr Δx qrad ′′ α exp ( −α x j )
⎣ Δx ⎦ ⎣ Δx ⎦
for i = 2.. ( M − 1) and j = 2.. ( N − 1)

The control volume equations must be placed into the matrix equation:

AX =b

where the equation for control volume i,j is placed into row M (j-1) + i of A and Ti,j corresponds
to element XM(j-1)+i in the vector X . Therefore, each coefficient in Eq. (6) (i.e., each term
multiplying an unknown temperature on the left side of the equation) must be placed in the row
of A corresponding to the control volume being examined and the column of A corresponding
to the unknown in X . The matrix assignments consistent with Eq. (6) are:

⎛ Δr ⎞ ⎛ Δr ⎞
k 2 π ⎜ ri − ⎟ Δx k 2 π ⎜ ri + ⎟ Δx k 4 π r Δr
AM ( j −1)+i , M ( j −1)+i =− ⎝ 2 ⎠
− ⎝ 2 ⎠
− i
(7)
Δr Δr Δx
for i = 2.. ( M − 1) and j = 2.. ( N − 1)

⎛ Δr ⎞
k 2 π ⎜ ri − ⎟ Δx
AM ( j −1)+i , M ( j −1)+i −1 = ⎝ 2 ⎠
for i = 2.. ( M − 1) and j = 2.. ( N − 1) (8)
Δr

⎛ Δr ⎞
k 2 π ⎜ ri + ⎟ Δx
AM ( j −1)+i , M ( j −1)+i +1 = ⎝ 2 ⎠
for i = 2.. ( M − 1) and j = 2.. ( N − 1) (9)
Δr

k 2 π ri Δr
AM ( j −1)+i , M ( j −1−1)+i = for i = 2.. ( M − 1) and j = 2.. ( N − 1) (10)
Δx

k 2 π ri Δr
AM ( j −1)+i , M ( j +1−1)+i = for i = 2.. ( M − 1) and j = 2.. ( N − 1) (11)
Δx
′′ α exp ( −α x j ) for i = 2.. ( M − 1) and j = 2.. ( N − 1)
bM ( j −1)+i = −2 π ri Δr Δx qrad (12)

A sparse matrix is allocated in MATLAB for A and the equations derived above are
implemented using a nested for loop. The spalloc command requires three arguments, which are
the number of rows and columns and the maximum number of elements in the matrix. Note that
there are at most 5 non-zero entries in each row of A , corresponding to Eqs. (7) through (11);
thus the last argument in the spalloc command which corresponds to the maximum number of
non-zero entries in the sparse matrix.

A=spalloc(M*N,M*N,5*M*N);
b=zeros(M*N,1);
for i=2:(M-1)
for j=2:(N-1)
A(M*(j-1)+i,M*(j-1)+i)=-k*2*pi*Dx*(r(i)-Dr/2)/Dr-
k*2*pi*Dx*(r(i)+Dr/2)/Dr-k*4*pi*r(i)*Dr/Dx;
A(M*(j-1)+i,M*(j-1)+i-1)=k*2*pi*Dx*(r(i)-Dr/2)/Dr;
A(M*(j-1)+i,M*(j-1)+i+1)=k*2*pi*Dx*(r(i)+Dr/2)/Dr;
A(M*(j-1)+i,M*(j+1-1)+i)=k*2*pi*r(i)*Dr/Dx;
A(M*(j-1)+i,M*(j-1-1)+i)=k*2*pi*r(i)*Dr/Dx;
b(M*(j-1)+i,1)=-2*pi*r(i)*Dr*Dx*qf_rad*alpha*exp(-alpha*x(j));
end
end

The nodes on the right side have a specified temperature:

TM , j = Tedge for j = 1..N

The matrix assignments suggested by these equations are:

AM ( j −1)+ M , M ( j −1)+ M = 1 for j = 1..N

bM ( j −1)+ M = Tedge for j = 1..N

These assignments are implemented in MATLAB:

for j=1:N
A(M*(j-1)+M,M*(j-1)+M)=1;
b(M*(j-1)+M,1)=T_edge;
end

The nodes along the bottom edge must be considered separately, leading to:
⎡ k π Δx ⎛ Δr ⎞ k π Δ x ⎛ Δr ⎞ k 2 π ri Δr ⎤
Ti ,1 ⎢ − ⎜ ri − ⎟ − ⎜ ri + ⎟−
⎣ Δr ⎝ 2 ⎠ Δr ⎝ 2 ⎠ Δx ⎥⎦
⎡ k π Δx ⎛ Δr ⎞ ⎤ ⎡ k π Δx ⎛ Δr ⎞ ⎤ ⎡ k 2 π ri Δr ⎤
+Ti −1,1 ⎢ ⎜ ri − ⎟ ⎥ + Ti +1,1 ⎢ ⎜ ri + ⎟ ⎥ + Ti ,2 ⎢ ⎥= (13)
⎣ Δr ⎝ 2 ⎠⎦ ⎣ Δr ⎝ 2 ⎠⎦ ⎣ Δx ⎦
′′ α exp ( −α x1 )
−π ri Δr Δx qrad
for i = 2.. ( M − 1)

for i=2:(M-1)
A(M*(1-1)+i,M*(1-1)+i)=-k*pi*Dx*(r(i)-Dr/2)/Dr-k*pi*Dx*(r(i)+Dr/2)/Dr-...
k*2*pi*r(i)*Dr/Dx;
A(M*(1-1)+i,M*(1-1)+i-1)=k*pi*Dx*(r(i)-Dr/2)/Dr;
A(M*(1-1)+i,M*(1-1)+i+1)=k*pi*Dx*(r(i)+Dr/2)/Dr;
A(M*(1-1)+i,M*(2-1)+i)=k*2*pi*r(i)*Dr/Dx;
b(M*(1-1)+i,1)=-pi*r(i)*Dr*Dx*qf_rad*alpha*exp(-alpha*x(1));
end

The nodes along the top edge must be considered separately, leading to:

⎡ k π Δx ⎛ Δr ⎞ k π Δ x ⎛ Δr ⎞ k 2 π ri Δr ⎤
Ti , N ⎢ − ⎜ ri − ⎟ − ⎜ ri + ⎟−
⎣ Δr ⎝ 2 ⎠ Δr ⎝ 2 ⎠ Δx ⎥⎦
⎡ k π Δx ⎛ Δr ⎞ ⎤ ⎡ k π Δx ⎛ Δr ⎞ ⎤ ⎡ k 2 π ri Δr ⎤
+Ti −1, N ⎢ Δr ⎜ ri − 2 ⎟ ⎥ + Ti +1, N ⎢ Δr ⎜ ri + 2 ⎟ ⎥ + Ti , N −1 ⎢ Δx ⎥ = (14)
⎣ ⎝ ⎠⎦ ⎣ ⎝ ⎠⎦ ⎣ ⎦
′′ α exp ( −α xN )
−h 2 π ri Δr T∞ − π ri Δr Δx qrad
for i = 2.. ( M − 1)

for i=2:(M-1)
A(M*(N-1)+i,M*(N-1)+i)=-k*pi*Dx*(r(i)-Dr/2)/Dr-k*pi*Dx*(r(i)+Dr/2)/Dr-...
k*2*pi*r(i)*Dr/Dx-h_bar*2*pi*r(i)*Dr;
A(M*(N-1)+i,M*(N-1)+i-1)=k*pi*Dx*(r(i)-Dr/2)/Dr;
A(M*(N-1)+i,M*(N-1)+i+1)=k*pi*Dx*(r(i)+Dr/2)/Dr;
A(M*(N-1)+i,M*(N-1-1)+i)=k*2*pi*r(i)*Dr/Dx;
b(M*(N-1)+i,1)=-h_bar*2*pi*r(i)*Dr*T_infinity-...
pi*r(i)*Dr*Dx*qf_rad*alpha*exp(-alpha*x(N));
end

The nodes along the left side leads to:

⎡ k 2 π Δx ⎛ Δr ⎞ ⎤ ⎡ k 2 π Δx ⎛ Δr ⎞ ⎤
T1, j ⎢ − ⎜ r1 + ⎟ ⎥ + T2, j ⎢ ⎜ r1 + ⎟ =0
⎣ Δr ⎝ 2 ⎠⎦ ⎣ Δr ⎝ 2 ⎠ ⎥⎦ (15)
for j = 2.. ( N − 1)

for j=2:(N-1)
A(M*(j-1)+1,M*(j-1)+1)=-k*2*pi*(r(1)+Dr/2)*Dx/Dr;
A(M*(j-1)+1,M*(j-1)+2)=k*2*pi*(r(1)+Dr/2)*Dx/Dr;
end
The node at the lower and upper left corners lead to:

⎡ k π Δx ⎛ Δr ⎞ ⎤ ⎡ k π Δx ⎛ Δr ⎞ ⎤
T1,1 ⎢ − ⎜ r1 + ⎟ ⎥ + T2,1 ⎢ ⎜ r1 + ⎟ =0 (16)
⎣ Δr ⎝ 2 ⎠⎦ ⎣ Δr ⎝ 2 ⎠ ⎥⎦

⎡ k π Δx ⎛ Δr ⎞ ⎤ ⎡ k π Δx ⎛ Δr ⎞ ⎤
T1, N ⎢ − ⎜ r1 + ⎟ ⎥ + T2, N ⎢ ⎜ r1 + ⎟ =0 (17)
⎣ Δr ⎝ 2 ⎠⎦ ⎣ Δr ⎝ 2 ⎠ ⎥⎦

A(M*(1-1)+1,M*(1-1)+1)=-k*pi*(r(1)+Dr/2)*Dx/Dr;
A(M*(1-1)+1,M*(1-1)+2)=k*pi*(r(1)+Dr/2)*Dx/Dr;
A(M*(N-1)+1,M*(N-1)+1)=-k*pi*(r(1)+Dr/2)*Dx/Dr;
A(M*(N-1)+1,M*(N-1)+2)=k*pi*(r(1)+Dr/2)*Dx/Dr;

The vector of unknowns X is obtained through matrix manipulation and then placed into matrix
format.

X=A\b;
for i=1:M
for j=1:N
T(i,j)=X(M*(j-1)+i);
end
end

The temperature as a function of radius at various values of x is shown in Figure 1.

31
x/th = 0
30 x/th = 0.25
x/th = 0.5
x/th = 0.75
Temperature (°C)

29
x/th = 1.0
28

27

26
Problem 2.3-4
25 Problem 2.6-3

24
0 0.005 0.01 0.015 0.02 0.025 0.03
Radius (m)
Figure 1: Temperature as a function of radius at various values of axial position.

b.) Prepare a contour plot of the temperature in the window.

A contour plot of the temperature in the window is shown in Figure 2.


0.03

30

0.025 29.5

29

0.02 28.5
Axial position (m)

28

0.015
27.5

27
0.01

26.5

26
0.005

25.5

0 25
0 0.001 0.002 0.003 0.004 0.005 0.006 0.007 0.008 0.009 0.01

Radius (m)
Figure 2: Contour plot of the temperature.

c.) Verify that your solution agrees with the analytical solution from Problem 2.3-4.

Figure 1 illustrates the solution obtained from Problem 2.3-4 and shows that the agreement is
nearly exact.
Problem 2.7-1 (2-10 in text): A Double Paned Window
Figure P2.7-1(a) illustrates a double paned window. The window consists of two panes of glass
each of which is tg = 0.95 cm thick and W = 4 ft wide by H = 5 ft high. The glass panes are
separated by an air gap of g = 1.9 cm. You may assume that the air is stagnant with ka = 0.025
W/m-K. The glass has conductivity kg = 1.4 W/m-K. The heat transfer coefficient between the
inner surface of the inner pane and the indoor air is hin = 10 W/m2-K and the heat transfer
coefficient between the outer surface of the outer pane and the outdoor air is hout = 25 W/m2-K.
You keep your house heated to Tin = 70°F.
width of window, W = 4 ft

tg = 0.95cm
Tin = 70°F
tg = 0.95 cm
hin = 10 W/m -K
2
g = 1.9 cm

H = 5 ft Tout = 23°F
hout = 25 W/m -K
2

ka = 0.025 W/m-K
kg = 1.4 W/m-K

casing shown in P2.10(b)


Figure P2.7-1(a): Double paned window.

The average heating season in Madison lasts about time = 130 days and the average outdoor
temperature during this time is Tout = 23°F. You heat with natural gas and pay, on average, ec =
1.415 $/therm (a therm is an energy unit =1.055x108 J).
a.) Calculate the average rate of heat transfer through the double paned window during the
heating season.

The inputs are entered in EES:

$UnitSystem SI MASS RAD PA K J


$TABSTOPS 0.2 0.4 0.6 0.8 3.5 in

"Inputs"
tg=0.375*convert(inch,m) "glass thickness"
g=0.75*convert(inch,m) "air gap"
k_g=1.4 [W/m-K] "glass conductivity"
k_a=0.025 [W/m-K] "air conductivity"
H=5 [ft]*convert(ft,m) "height of window"
W=4[ft]*convert(ft,m) "width of window"
T_in=converttemp(F,K,70 [F]) "indoor air temperature"
h_in=10 [W/m^2-K] "heat transfer coefficient on inside of window"
T_out=converttemp(F,K,23 [F]) "outdoor air temperature"
h_out=25 [W/m^2-K] "heat transfer coefficient on outside of window"
time=130 [day]*convert(day,s) "heating season duration"
ec=1.415 [$/therm]*convert($/therm,$/J) "cost of energy"

The heat transfer is resisted by convection on the inner and outer surfaces:
1
Rconv ,in = (1)
hin W H

1
Rconv ,out = (2)
hout W H

and conduction through the glass panes and the air:

tg
Rcond , g = (3)
kg W H

g
Rcond ,a = (4)
ka W H

R_conv_in=1/(h_in*W*H) "convection resistance on inside of window"


R_cond_g=tg/(k_g*W*H) "conduction resistance of glass pane"
R_cond_a=g/(k_a*W*H) "conduction resistance of air gap"
R_conv_out=1/(h_out*W*H) "convection resistance on outside of window"

The heat transfer rate through the window is:

Tin − Tout
q = (5)
Rconv ,in + 2 Rcond , g + Rcond ,a + Rconv ,out

q_dot=(T_in-T_out)/(R_conv_in+2*R_cond_g+R_cond_a+R_conv_out)
"rate of heat transfer through the window"

which leads to q = 53.0 W.

b.) How much does the energy lost through the window cost during a single heating season?

The total amount of energy lost over the course of a heating season is:

Q = q time (6)

and the associated cost is:

cost = ec Q (7)

Q=q_dot*time "total energy loss"


cost=ec*Q "cost to heat house per window"

which leads to cost = $7.98/heating season.


There is a metal casing that holds the panes of glass and connects them to the surrounding wall,
as shown in Figure P2.7-1(b). Because the metal casing is high conductivity, it seems likely that
you could lose a substantial amount of heat by conduction through the casing (potentially
negating the advantage of using a double paned window). The geometry of the casing is shown
in Figure P2.7-1(b); note that the casing is symmetric about the center of the window.
glass panes

1.9 cm
Tin = 70°F 0.95 cm Tout = 23°F
hout = 25 W/m -K
2
hin = 10 W/m -K
2
air 2 cm

4 cm
0.5 cm 3 cm
metal casing
km = 25 W/m-K 0.4 cm
wood

Figure P2-10(b) Metal casing.

All surfaces of the casing that are adjacent to glass, wood, or the air between the glass panes can
be assumed to be adiabatic. The other surfaces are exposed to either the indoor or outdoor air.
c.) Prepare a 2-D thermal analysis of the casing using FEHT. Turn in a print out of your
geometry as well as a contour plot of the temperature distribution. What is the rate of energy
lost via conduction through the casing per unit length (W/m)?

The geometry from Figure 3 is entered approximately, as shown in Figure 4.

Figure 4: Approximate geometry.

Each of the points are selected individually and their exact coordinates are entered, which leads
to the more precise geometry shown in Figure 4.
Figure 5: Geometry.

The material properties are specified by selecting the outline and selecting Material Properties
from the Specify menu. The boundary conditions are specified by selecting each type of
boundary and then selecting Boundary Conditions from the Specify menu. A crude mesh is
generated, as shown in Figure 6.

Figure 6: Crude Mesh.

The temperature contours are shown in Figure 7.

Figure 7: Temperature Contours.


The heat transfer per unit length is obtained by selecting Heat Flow from the View menu and
selecting all of the boundaries that are exposed to either the indoor or outdoor air. The heat flow
is 10.0 W/m.

d.) Show that your numerical model has converged by recording the rate of heat transfer per
length for several values of the number of nodes.

The mesh is refined several times and each time the heat transfer rate per unit length of casing is
recorded; the results are shown in Figure 8.

Figure 8: Heat transfer through the casing per unit length as a function of the number of nodes.

e.) How much does the casing add to the cost of heating your house?

′ ) is entered in EES:
The result from FEHT, the heat transfer per unit length ( qcasing

q`_dot_casing=10.0 [W/m] "casing heat transfer per unit meter"

The total heat transfer through the casing is:


qcasing = qcasing 2 (W + H ) (8)

The total heat lost through the casing is:

Qcasing = qcasing time (9)

and the associated cost is:


costcasing = ec Qcasing (10)

q_dot_casing=q`_dot_casing*2*(W+H) "heat transfer rate through casing"


Q_casing=q_dot_casing*time "total energy flow through casing"
cost_casing=ec*Q_casing
"cost to heat house per window due to casing"

which leads to an additional cost of $8.27 per heating season.


Problem 2.7-2: Energy recovery system
Relatively hot gas flows out of the stack of a power plant. The energy associated with these
combustion products is useful for providing hot water or other low grade energy. The system
shown in Figure P2.7-2(a) has been developed to recover some of this energy.
water flows hot gas
through holes

stack structure

Figure P2.7-2(a): Energy recovery system for hot exhaust gas.

The system is fabricated from a high temperature material in the form of a ring; 16 fluid channels
are integrated with the stack in a circular array. The water to be heated flows through these
channels. The inner surface of the liner is finned to increase its surface area and is exposed to
the hot gas while the outer surface is cooled externally by ambient air. A cut-away view of the
stack is shown in Figure P2.7-2(b).

unit cell
Tair = 20°C
hair = 20 W/m -K
2

Tw = 30°C
hw = 500 W/m -K
2

Tgas = 800°C
hgas = 50 W/m -K
2
k = 25 W/m-K
Figure P2.7-2(b): Problem specification for the energy recovery problem.

At a particular section, the water can be modeled as being at a uniform temperature of Tw = 30°C
with a heat transfer coefficient, hw = 500 W/m2-K. The hot gas is at Tgas = 800°C with a heat
transfer coefficient, hgas = 50 W/m2-K. The ambient air external to the stack is at Tair = 20°C
and hair = 20 W/m2-K. The stack material has conductivity k = 25 W/m-K.

The liner geometry is relatively complex and includes curved segments as well as straight
sections. Only a single unit cell of the structure (see Figure P2.7-2(b)) needs to be simulated.
FEHT does not allow curved sections to be simulated; rather, a curved section must be
approximated as a polygon. Rather than attempting to draw the geometry manually, it is
preferable to import a drawing (e.g., from a computer aided drawing package) and trace the
drawing in FEHT. More advanced finite element tools will have automated processes for
importing geometry from various sources. A drawing of the unit cell with a scale can be copied
onto the clipboard (from the website for this text, using Microsoft Powerpoint) and pasted into
FEHT.
a.) Use FEHT to develop a finite element model of the stack and determine the heat transfer to
the water per unit length of stack.
Open FEHT and specify that you are doing a steady state heat transfer problem in cartesian
coordinates using the C temperature scale using the Setup menu (Figure 3).

Figure 3: Specify menu.

Open the program that contains the unit cell drawing and copy it to the clip board. In FEHT,
select Paste from Clipboard from the File menu; the drawing should appear in the drawing space
(Figure 4).

Figure 4: Drawing pasted onto drawing area.

Move the template to the center of the screen and select Size/Move Template from the draw
menu; this will allow the grid to show through the template (Figure 5).
Figure 5: Template and drawing grid.

Finally, resize your drawing grid so that 1 cm on your template corresponds to 1 cm on your
model. First, identify the size of the square that is contained on the template for this purpose.
Select Scale and Size from the Setup menu and then adjust the Grid Spacing option until the size
of grid matches the size of the square (Figures 6 and 7).

Figure 6: Adjust Grid Spacing until a grid matches the scale on the template.
Figure 7: Grid spacing adjusted to match the template scale.

In Figure 7, the grid spacing is 1.45 cm x 1.45 cm; therefore, the 1.45 cm of screen is equivalent
to 1 cm of drawing distance. Select Scale and Size from the Setup menu and set the scale such
that 1 cm of screen is equal to 1/1.45 cm (or 0.69 cm) of drawing distance (Figure 8).

Figure 8: Adjust scale to match template.

Now it is possible to draw the geometry by tracing the template. You need to specify the outer
region and then “punch” holes in it (rather than drawing the flow passage and then overwriting it
with the outer region). Select Outline from the Draw menu and “trace” the outside edge of the
drawing. It is important to recognize that the curved surfaces must be represented by a discrete
number of straight lines that are defined by the nodes. The more nodes you use, the better you
can simulate the curve but the more difficult it will be to generate a mesh. Start with just a few
nodes, as shown in Figure 9.
Figure 9: Adjust scale to match template.

Select Hide Patterns from the Display menu so that the material is transparent (Figure 10).

Figure 10: Pattern removed to make the template visible.

Select Outline from the Draw menu and trace the circle (use an octagon to represent the curve),
as shown in Figure 11.
Figure 11: Circular flow passage defined in FEHT.

Select Hide Template from the Display menu and the geometry will remain. Save your file.
Having defined the geometry it is necessary to define the material properties, boundary
conditions, and mesh in order to solve the problem. The liquid within the flow passage can be
represented conveniently as a single temperature. In this case it is possible to mesh the liquid
and specify a very high conductivity; however, this is a waste of computational resources given
that we know the answer. Instead, FEHT allows you to specify that a region is a Lumped/Fluid
element which has a single temperature. This is a nice feature for many problems that reduces
the complexity of the model without affecting the accuracy of the predictions. Select the outline
of the fluid passage and then select Material Properties from the Specify menu (Figure 12).
Select not specified from the list and then give the element the name Lumped Water, click on
Type until Fluid/Lump appears.

Figure 12: Specify Properties dialog.

Select Show Patterns from the Display menu to see the water. Next select the outline of the
stack material and specify the conductivity.
The edges that define the unit cell are adiabatic; select these edges and then select Boundary
Conditions from the Specify menu. Set the heat flux to 0 W/m2 (Figure 13).

Figure 13: Specify Boundary Conditions dialog.

Set the boundary conditions on the internal surface (exposed to combustion gas) and external
surface (exposed to ambient air) in the normal way. You also need to specify a heat transfer
coefficient between the water and the fluid passage; select the octagon that defines this boundary
and then select Boundary Conditions from the Specify menu. Set the heat transfer coefficient to
hwater (Figure 14).

Figure 14: Specify Boundary Conditions for the boundary of the lumped element.

You will need to specify the temperature of the lumped element; click on the lumped element
and select Lumped Information from the Specify menu (Figure 15). Set the temperature to be
Twater
Figure 15: Specify Lumped Information dialog.

It is necessary to specify a crude mesh; you do not need to mesh the lumped element because the
governing equations will not be solved within this element (it is just assigned a temperature).
The mesh is specified by selecting Element Lines from the Draw menu; all of the mesh elements
must be triangular. A reasonable initial mesh is shown in Figure 16.

Figure 16: Mesh for the stack material.

Select Calculate from the Run menu and then select Temperature Contours from the View menu
(Figure 17).
Figure 17: Solution with crude mesh.

It is possible to determine the total heat transferred to the water by selecting Heat Flows from the
View menu and then selecting the 8 boundaries that define the water passage (Figure 18). Notice
that the sum of the total heat flow is displayed on the toolbar.

Figure 18: Heat flows.

The total heat flow to a single tube is 3076 W/m; therefore, the total heat transfer to the 16
passages filled with water will be 49.2 kW.

b.) Verify that your solution has converged numerically.


In order to evaluate how many nodes are required, refine the mesh (select Input from the View
menu and then Reduce Mesh from the Draw menu) and solve the problem again. Record the
number of nodes or elements (the number of elements is displayed by selecting Check from the
Run menu) and the heat flow to the water as the mesh is progressively refined. The result is
shown in Figure 19 and suggests that at least 400 nodes should be used.

Figure 19: Predicted heat flows per passage per unit length as a function of the number of elements.

c.) Sanity check your results against a simple model.

The geometry is too irregular to allow an analytical solution. Therefore, it is possible to compare
the answer to a very rough resistance calculation. The known information is entered in EES:

$UnitSystem SI MASS RAD PA K J

"Inputs"
T_gas=converttemp(C,K,800) "hot gas"
h_gas=50 [W/m^2-K] "hot gas heat transfer coefficient"
T_water=converttemp(C,K,30) "water temperature"
h_water=500 [W/m^2-K] "water heat transfer coefficient"
k=25 [W/m-K] "material conductivity"
L=1 [m] "per unit length of stack"

There are, approximately, three resistances between the hot gas and the water. The resistance to
convection with the flue gas, Rconv,gas, is:

1
Rconv , gas = (1)
hgas L pgas

where pgas is the perimeter of the stack material that is exposed to gas - approximately 7.0 cm.
p_gas=7.0 [cm]*convert(cm,m) "perimeter of stack exposed to gas"
R_conv_gas=1/(h_gas*L*p_gas) "resistance due to convection to the gas"

The resistance to convection with the water, Rconv,water, is:

1
Rconv , water = (2)
hwater L π Dwater

where Dwater is the diameter of the water passage, approximately 1.5 cm.

D_water=1.5 [cm]*convert(cm,m) "diameter of water passage"


R_conv_water=1/(h_water*L*pi*D_water) "resistance due to convection to water"

Finally, the resistance due to conduction within the stack material, Rcond, is approximately:

Lc
Rcond = (3)
k L wc

where Lc is the distance heat must be conducted and L wc is the area for conduction; Lc is
approximately 3.0 cm and wc is approximately 2.5 cm.

L_c=3.0 [cm]*convert(cm,m) "distance for conduction"


w_c=2.5 [cm]*convert(cm,m) "width for conduction"
R_cond=L_c/(k*w_c*L) "resistance for conduction"

The total heat transfer rate ( q ) should be approximately:

q ≈
(T gas − Twater )
(4)
Rconv , gas + Rconv , water + Rcond

q_dot=(T_gas-T_water)/(R_conv_gas+R_conv_water+R_cond) "heat transfer rate"

The approximate model predicts that Rconv,gas= 0.29 K/W, Rconv,water = 0.04 K/W, Rcond = 0.05
K/W which leads to q =2050 W; this is 30% less than the FEHT model prediction. The
agreement is not expected to be substantially better than this given the extremely crude
approximations involved in the resistance model. However, the calculations suggest that no
substantial errors were made in the finite element model. Furthermore, the magnitude of the
three resistances provides some insight into what thermal resistance controls the problem (the
convection with the air) and therefore how the design could be most effectively improved.
P2.7-3 (2-11 in text): A Spacecraft Radiator
A radiator panel extends from a spacecraft; both surfaces of the radiator are exposed to space (for
the purposes of this problem it is acceptable to assume that space is at 0 K); the emittance of the
surface is ε = 1.0. The plate is made of aluminum (k = 200 W/m-K and ρ = 2700 kg/m3) and has
a fluid line attached to it, as shown in Figure 2.7-3(a). The half-width of the plate is a=0.5 m
wide while the height of the plate is b=0.75m. The thickness of the plate is a design variable and
will be varied in this analysis; begin by assuming that the thickness is th = 1.0 cm. The fluid
lines carry coolant at Tc = 320 K. Assume that the fluid temperature is constant although the fluid
temperature will actually decrease as it transfers heat to the radiator. The combination of
convection and conduction through the panel-to-fluid line mounting leads to an effective heat
transfer coefficient of h = 1,000 W/m2-K over the 3.0 cm strip occupied by the fluid line.
k = 200 W/m-K
space at 0 K
ρ = 2700 kg/m3
ε = 1.0
a = 0.5 m 3 cm
th = 1 cm

b = 0.75 m
fluid at Tc = 320 K

half-symmetry model of panel, Figure P2-11(b)


Figure 2.7-3(a): Radiator panel

The radiator panel is symmetric about its half-width and the critical dimensions that are required
to develop a half-symmetry model of the radiator are shown in Figure 2.7-3(b). There are three
regions associated with the problem that must be defined separately so that the surface conditions
can be set differently. Regions 1 and 3 are exposed to space on both sides while Region 2 is
exposed to the coolant fluid one side and space on the other; for the purposes of this problem, the
effect of radiation to space on the back side of Region 2 is neglected.
Region 1 (both sides exposed to space)
Region 2 (exposed to fluid - neglect radiation to space)
Region 3 (both sides exposed to space)

(0.50,0.75)

(0.50,0.55)

(0.50,0.52)

y
x
(0,0) (0.50,0)
(0.25,0) line of symmetry
(0.22,0)
Figure 2.7-3(b): Half-symmetry model.

a.) Prepare a FEHT model that can predict the temperature distribution over the radiator panel.
The radiator panel can be modeled as 2-D problem because it is thin and has high conductivity.
The Biot number compares the ratio of the internal conduction resistance to the external
resistance; in this case due to radiation.

Rcond
Bi = (1)
Rrad

where

th
Rcond = (2)
2 k As

Using the concept of a radiation resistance:

1
Rrad = (3)
As σ ε s (T + Tsur
s
2 2
)(Ts + Tsur )
The Biot number is therefore:

th σ ε s (Ts2 + Tsur
2
)(Ts + Tsur )
Bi = (4)
2k

The surrounding temperature in Eq. (4), Tsur corresponds to space which is essentially 0 K and
therefore Eq. (4) reduces to:

th σ ε s Ts3
Bi = (5)
2k

We don’t know the value of the panel surface temperature, Ts in Eq. (5), but we can assume that
it will be near the fluid temperature for any well-designed radiator. Using Ts = Tf in Eq. (5)
results in a Biot number of 4.6x10-5 which is very small; clearly for any reasonable value of Ts
the problem will be 2-D.

FEHT can simulate 2-D problems of this type using the Extended Surface mode. Start FEHT
and select Extended Surface from the Subject menu. Each of the three regions must be drawn
separately using the Outline option from the Draw menu. It is easiest to set an appropriate grid
using the Scale and Size selection from the Setup menu (Figure 2).
Figure 2: Scale and Size dialog window

Generate outlines for the 3 regions that are close to the correct scale and then double-click on
each node and position it exactly (Figure 3).

Figure 3: Geometry definition.

The material properties for all three regions can be set by selecting each while holding down the
shift key and then selecting Specify Properties from the Specify menu. Select Aluminum from
the list of materials and then modify the conductivity to be 200 W/m-K and the thickness to be
0.01 m to match the problem statement (Figure 4).
Figure 4: Specify material properties.

The surface conditions for Regions 1 and 3 can be set by selecting these regions with the shift
key held down and then selecting Surface Conditions from the Specify menu. These regions are
radiating to space but there is no option listed in the Extended Surface Conditions for a radiative
surface condition. The radiation heat flux is:

′′ = ε σ T 4
qrad (6)

Equation (6) can be rewritten according to:

′′ = ε σ T 3 (T − 0 )
qrad (7)

Equation (7) is similar to a convection equation:

′′ = hrad (T − T f )
qrad (8)

where fluid temperature (Tf) is 0 K and the convection coefficient (hrad) is a function of
temperature:

hrad = ε σ T 3 (9)

FEHT allows the specification of the surface conditions in terms of position (X and Y) as well as
temperature (T); therefore, the radiative surface condition can be modeled as shown in Figure 5.
Figure 5: Extended Surface Conditions Dialog window for Regions 1 and 3.

Region 2 experiences a convection boundary condition with the coolant, but only from one side
(as opposed to the double sided condition assumed by FEHT). The heat transfer from a
differential element in Region 2 is given by:

q = h dA (T − Tc ) (10)

whereas the convection surface condition in FEHT assumes convection from both sides of the
plate and therefore:

q = hF 2 dA (T − Tc ) (11)

where hF is the heat transfer coefficient that should be set in FEHT in order to simulate the
single-sided convection coefficient represented by Eq. (10). Comparing Eqs. (10) and (11) leads
to:

h
hF = (12)
2

Click on Region 2 and select Surface Condition from the Specify menu; specify the surface
conditions as shown in Figure 6.

Figure 6: Extended Surface dialog window for Region 2.

Finally, the boundary conditions along each edge of the computational domain must be specified;
the line of symmetry is adiabatic and the remaining edges are also assumed to be adiabatic.
Generate a reasonable but crude mesh, as shown in Figure 7(a) and then refine it. Note that the
mesh within Region 2 will start relatively refined due to its small width and need not be refined
as much as the mesh in Regions 1 and 3. However, it is possible to refine the mesh in a single
region by selecting that region and then selecting Reduce Mesh from the Draw menu. The result
should be similar to Figure 7(b).

(a) (b)
Figure 7: (a) coarse and (b) refined mesh.

Solve the problem by selecting Calculate from the Run menu; the problem is non-linear due to
the temperature dependent heat transfer coefficient and therefore the solution process will be
iterative. Plot the temperature distribution by selecting Temperature Contours from the View
menu (Figure 8).
Figure 8: Temperature distribution for a 1 cm thick plate.

b.) Export the solution to EES and calculate the total heat transferred from the radiator and the
radiator efficiency (defined as the ratio of the radiator heat transfer to the heat transfer from
the radiator if it were isothermal and at the coolant temperature).

Select Tabular Output from the View menu and then Select All and Save As. Save the data as a
file called ‘1 cm’. Open EES and select Open Lookup Table from the Tables menu; navigate the
file ‘1 cm’ and open it; the solution (the temperature at each node together with the locations of
the nodes) is contained in the lookup table. Using the technique discussed in EXAMPLE 16-1 it
is possible to use the Interpolate2D function to obtain the temperature at an arbitrary x and y
location on the radiator plate.

$UnitSystem SI MASS RAD PA K J


$TABSTOPS 0.2 0.4 0.6 0.8 3.5 in

"INPUTS"
e=1.0 [-] "emittance of panel surface"
a=0.5 [m] "half-width of panel"
b=0.75 [m] "height of panel"
T_f=320 [K] "fluid temperature"
th=0.01 [m] "thickness"
rho=2700 [kg/m^3] "density"
k=200 [W/m-K] "conductivity"

"location on plate"
x=0.1 [m]
y=0.5 [m]
"2-D interpolation from table of nodal data"
T=Interpolate2D('1 cm',X,Y,T,X=x,Y=y) "temperature interpolated from data"

In order to obtain the total heat transferred from the panel it is necessary to integrate the heat flux
over the entire area of the plate. This process can be accomplished manually by dividing the
plate into many small integration areas, calculating the heat flux within each area, and summing
the result. The plate is divided into Nx segments in the x direction, each with width:

a
Δx = (13)
Nx

The x position of each segment is:

xi = ( i − 0.5 ) Δx (14)

The plate is divided into Ny segments in the y direction, each with height:

b
Δy = (15)
Ny

The y position of each segment is:

y j = ( j − 0.5 ) Δy (16)

The corresponding EES code is:

"Double integration manually"


Nx=25 [-] "number of integration areas in x"
Ny=10 [-] "number of integration areas in y"
"size of an integration area"
Dx=a/Nx
Dy=b/Ny
"setup position of x and y areas"
duplicate i=1,Nx
x[i]=(i-0.5)*Dx
end
duplicate j=1,Ny
y[j]=(j-0.5)*Dy
end

The total heat transferred from the plate ( q ) is calculated using the double integral:

b a
q = ∫ ∫ q ′′x , y dx dy (17)
0 0

where the heat flux is:


q ′′x , y = ε σ Tx4, y (18)

The numerical summation that approximates the integration in Eq. (17) is:

N y Nx

q = ∑∑ q ′′xi , y j Δx Δy (19)
j =1 i =1

where

q ′′xi , y j = ε σ Tx4i , y j (20)

The corresponding EES code is:

duplicate i=1,Nx
duplicate j=1,Ny
"2-D interpolation from table of nodal data"
T[i,j]=Interpolate2D('1 cm',X,Y,T,X=x[i],Y=y[j]) "temperature interpolated from data"
q``_dot[i,j]=2*e*sigma#*T[i,j]^4 "heat flux"
end
end
q_dot=sum(q``_dot[1..Nx,1..Ny])*Dx*Dy

Note that the same process can be accomplished using EES’ native Integral command. The
Integral command carries out numerical integration using a more sophisticated algorithm than
simply assuming a constant value over each step.

EES’ Integral command requires 4 arguments and allows a 5th, optional argument; the protocol
for calling the function is:

F=Integral(Integrand,VarName,LowerLimit,UpperLimit,StepSize)

where Integrand is the EES variable or expression that must be integrated, VarName is the
integration variable, and LowerLimit and UpperLimit define the limits of integration. StepSize
is optional and defines the numerical step that is used to accomplish the integration; a small
value of StepSize will lead to more accurate results but take longer to calculate.

The double integral in Eq. (17) can be accomplished by calling the Integral function twice; Eq.
(17) is rewritten as:
b
q = ∫ q′y dy (21)
0

where
a
q ′y = ∫ q ′′x , y dx (22)
0

Equation (22) is evaluated using the Integral command as shown in the EES code below:

"Double integration using EES' Integral command"


"2-D interpolation from table of nodal data"
T=Interpolate2D('1 cm',X,Y,T,X=x,Y=y) "temperature interpolated from data"
q``_dot=2*e*sigma#*T^4 "heat flux"
y=0
q`_dot=INTEGRAL(q``_dot,x,0,a,0.02) "heat transfer per unit length"

Note that the value of y was set as q ′y is a function of y but the value of x is not set as x is the
integration variable. To evaluate Eq. (21), integrate q ′y from y=0 to y=b:

"Double integration using EES' Integral command"


"2-D interpolation from table of nodal data"
T=Interpolate2D('1 cm',X,Y,T,X=x,Y=y) "temperature interpolated from data"
q``_dot=2*e*sigma#*T^4 "heat flux"
q`_dot=INTEGRAL(q``_dot,x,0,a,0.02) "heat transfer per unit length"
q_dot=INTEGRAL(q`_dot,y,0,b,0.02) "total heat transfer rate"

Note that the value of y is no longer set as y is the integration variable in the 2nd integral. The
results of the manual integration should agree with the use of the Integral function; either should
yield 339 W. The radiator efficiency (η) and mass (M) are computed according to:

q
η= (23)
2 a b ε σ T f4

and

M = a b th ρ (24)

c.) Explore the effect of thickness on the radiator efficiency and mass.

The solution procedure described above is repeated for several values of the radiator thickness.
Figure 10 shows the predicted temperature distribution for several values of the thickness. Note
that the same contour levels were selected for each plot by selecting User in the Temperature
Contour Information dialog window and specifying the range from 200 K to 320 K (Figure 9);
this allows the different cases to be compared directly and shows clearly that the reduced
thickness increases the resistance to conduction along the plate and therefore leads to
progressively larger temperature gradients through the plate.
Figure 9: Temperature Contour Information dialog window.

Figure 10: Temperature distribution in the panel for thicknesses of (a) 1.0 cm, (b) 0.75 cm, (c) 0.5 cm, (d) 0.3
cm, (e) 0.2 cm, and (f) 0.1 cm.

Figure 11 illustrates the radiator efficiency and mass as a function of the thickness. As the
thickness is reduced, the efficiency drops but so does the mass; clearly there must be a trade-off
between these effects.
Figure 11: Efficiency and mass as a function of thickness.

Figure 12 shows the efficiency as a function of mass and makes the trade-off clearer; above a
panel with a mass of nominally 2 kg there is a region of diminishing return where additional
mass provides only a small gain in efficiency.

Figure 12: Efficiency as a function of mass.

A complete analysis would require more information then is given; specifically, how should the
radiator performance be compared with its mass to determine a true optimal thickness. Barring
additional constraints related to, for example, structural stability, Figure 12 suggests that any
optimization process will result in a panel that is approximately 0.2 cm thick with a mass of 2 kg
and an efficiency of 60%.
Problem 2.7-4
Gas turbine power cycles are used for the generation of power; the size of these systems can
range from 10’s of kWs for the microturbines that are being installed on-site at some commercial
and industrial locations to 100’s of MWs for natural gas fired power plants. The efficiency of a
gas turbine power plant increases with the temperature of the gas entering from the combustion
chamber; this temperature is constrained by the material limitations of the turbine blades which
tend to creep (i.e., slowly grow over time) in the high temperature environment in their high
centrifugal stress state. One technique for achieving high gas temperatures is to cool the blades
internally; often the air is bled through the blade surface using a technique called transpiration.
A simplified version of a turbine blade that will be analyzed in this problem is shown in Figure
P2.7-4.
combustion gas
k = 15 W/m-K
Tg = 1800 K
hg= 850 W/m2-K

coordinates (in cm):


point 1 (0, 0),
point 2 (2, 0), cooling air
point 3 (3.5, 0.25), Tc = 500 K
point 4 (4, 0),
point 5 (5, 0.75), hc = 250 W/m2-K
point 6 (5.5, 0),
point 7 (6.25, 1.25),
point 8 (7, 0),
point 9 (7, 2)

Figure 2.7-4: A simplified schematic of an air cooled blade.

The high temperature combustion gas is at Tg = 1800 K and the heat transfer coefficient between
the gas and blade external surface is hg = 850 W/m2-K. The blades are cooled by three internal
air passages. The cooling air in the passages is at Tc = 500 K and the air-to-blade heat transfer
coefficient is ha= 250 W/m2-K. The blade material has conductivity k = 15 W/m-K. The
coordinates of the points required to define the geometry are indicated in Figure P2.7-4.
a.) Generate a ½ symmetry model of the blade in FEHT. Generate a figure showing the
temperature distribution in the blade predicted using a very crude mesh.

The half-symmetry model is generated by selecting Scale and Size from the Setup menu and
setting up a convenient grid to approximately place the outer nodes; the scale was set to 1 cm =
0.5 cm with grid spacing 0.5 cm x 0.5 cm (Figure 1). The temperatures are set to K and the
problem set to a steady-state problem in cartesian coordinates by making the appropriate
selections in the Setup menu.
Figure 2: Scale and Size window.

The nodes required to define the shape are approximately positioned using the grid by selecting
Outline from the Draw menu (Figure 3(a)); each node is subsequently double-clicked on and the
exact coordinates are entered (Figure 3(b)).

(a)

(b)
Figure 3: FEHT model with (a) nodes approximately placed on the grid and (b) nodes exactly positioned.

The material properties are specified by clicking on the outline and selecting Material Properties
from the Specify menu (Figure 4). It is best to set a lighter color for the material so that the grid
can be seen.
Figure 4: Specifying material properties.

The boundary conditions along the external surfaces are specified by clicking on these two lines
and selecting Boundary Conditions from the Specify menu (Figure 5).

Figure 5: External surface boundary condition specification.

The remaining boundary conditions are specified using the same process (notice that the
boundaries along the line of symmetry are adiabatic); the result is shown in Figure 6.

Figure 6: Boundary conditions

A very crude grid is generated (Figure 7); recall that each element has to be a triangle.
Figure 7: Crude mesh.

The mesh can be checked by selecting Check from the Run menu. If there are no errors then
select Calculate from the Run menu and view the temperature distribution by selecting
Temperature Contours from the View menu; the result is shown in Figure 8.

Figure 8: Temperature distribution with crude mesh.

b.) Refine your mesh and keep track of the temperature experienced at the trailing edge of the
blade (i.e., at position 9 in Figure P2.7-4) as a function of the number of nodes in your mesh.
Prepare a plot of this data that can be used to establish that your model has converged to the
correct solution.

The nodal data can be viewed by selecting either Tabular Output or Temperatures from the View
menu. The number of entries in the Tabular Output table leads to the number of nodes and the
temperature at the trailing edge can be read from the model. The number of unknown
temperatures is also displayed each time the model is calculated. The mesh is refined over an
over again (by selecting Reduce Mesh from the Draw menu) in order to obtain the results listed
in Table 1 and plotted in Figure 9.

Table 1: Mesh convergence data


Number of nodes Temperature at trailing edge of blade
20 1689 K
57 1686 K
185 1685 K
657 1685 K

Table 1 and Figure 9 suggest that the solution has converged at 657 nodes. The temperature
distribution for 657 nodes is shown in Figure 10.
Figure 9: Trailing edge temperature as a function of the number of nodes.

Figure 10: Temperature distribution in the turbine blade with 657 nodes.

c.) Do your results make sense? Use a very simple, order-of-magnitude analysis based on
thermal resistances to decide whether your predicted blade surface temperature is reasonable
(hint – there are three thermal resistances that govern the behavior of the blade, estimate each
one and show that your results are approximately correct given these thermal resistances).

From a very approximate standpoint, the problem is governed by the thermal resistance to
convection on the internal and external surfaces (Rconv,in and Rconv,ext) and conduction through the
blade (Rcond). These resistances are estimated here:

1
Rconv ,c = (1)
hc pc L

where pc is the perimeter of the cooling passages (7.75 cm, from Figure 2.7-4) and L is a unit
length of the turbine blade.
1
Rconv , g = (2)
hg pg L

where pg is the perimeter of the blade exposed to the combustion gas


( 7 cm ) + ( 2 cm ) + 2 cm ) from Figure 2.7-4.
2 2
(

Lcond
Rcond = (3)
k Acond

where Lcond is the approximate length that must be conducted through (about 0.5 cm from Figure
2.7-4) and Acond is the approximate area for conduction (about 5.0 cm x L from Figure 2.7-4).
The calculations are carried out in EES:

$UnitSystem SI MASS RAD PA K J


$TABSTOPS 0.2 0.4 0.6 0.8 3.5 in

"Inputs"
T_c=500 [K] "cooling air temperature"
h_c=250 [W/m^2-K] "cooling air heat transfer coefficient"
T_g=1800 [K] "combustion gas temperature"
h_g=850 [W/m^2-K] "combustion gas heat transfer coefficient"
k=15 [W/m-K] "conductivity"
L=1 [m] "calculations are per meter"

p_c=7.75 [cm]*convert(cm,m) "perimeter of cooling channels"


p_g=(sqrt((7 [cm])^2+(2 [cm])^2)+2[cm])*convert(cm,m) "perimeter of blade"
L_cond=0.5 [cm]*convert(cm,m) "conduction length"
A_cond=5.0 [cm]*convert(cm,m)*L "conduction area"

The heat transfer per unit length of blade is:

Tg − Tc
q = (4)
Rconv , g + Rcond + Rconv ,c

and can be used to estimate the surface temperature of the blade (Ts,g):

Ts , g = Tg − q Rconv , g (5)

q_dot=(T_g-T_c)/(R_conv_g+R_cond+R_conv_c) "heat transfer per unit length per half-blade"


T_s_g=T_g-q_dot*R_conv_g "estimated surface temperature"

The result is 1570 K which is approximately consistent with the results shown in Figure 10.
Problem 2.7-5
a.) Show how the construction of the finite element problem changes with the addition of
volumetric generation.

The governing differential equation, including volumetric generation is:

∂ ⎛ ∂T ⎞ ∂ ⎛ ∂T ⎞
⎜ −k ⎟ + ⎜ −k ⎟ − g ′′′ = 0 (1)
∂x ⎝ ∂x ⎠ ∂y ⎝ ∂y ⎠

The average weighted residual equation is:

∂ ⎛ ∂Tˆ ⎞ ∂ ⎛ ∂Tˆ ⎞
∫∫ f ∂x ⎜⎝ −k ∂x ⎟⎠ dx dy + ∫∫ f ∂y ⎜⎝ −k ∂y ⎟⎠ dx dy − ∫∫ f g ′′′ dx dy = weighted average residual (2)


A


A A
Integral 1 Integral 2

and the weak form of the equation is:

⎡ ∂f ∂Tˆ ∂f ∂Tˆ ⎤
∫∫A ⎢⎣ ∂x ∂x ∂y ∂y ⎥⎦ dA − ∫∫A f g ′′′ dx dy = ∫B f qn′′ ds
k + k (3)

Substituting the approximate temperature distribution:

Tˆ ( x, y ) = wT Tˆ (4)

into Eq. (3) leads to:

⎡ ∂f ∂ ∂f ∂ ⎤
∫∫ ⎢⎣k ∂x ∂x ( w Tˆ ) + k ∂y ∂y ( w Tˆ )⎥⎦ dA − ∫∫ f g ′′′ dx dy = ∫ f qn′′ ds
T T
(5)
A A B

Using the Galerkin technique for the weighting functions leads to:

⎡ ⎛ ∂w1 ∂w1 ∂w ∂w ⎞ ⎤ ⎡ ⎛ ∂w ∂w ∂w ∂w ⎞ ⎤
⎢ ∫∫ ⎜ k + k 1 1 ⎟ dA⎥ Tˆ1 + ⎢ ∫∫ ⎜ k 1 2 + k 1 2 ⎟ dA⎥ Tˆ2 + ...
⎣ A ⎝ ∂x ∂x ∂y ∂y ⎠ ⎦ ⎣ A ⎝ ∂x ∂x ∂y ∂y ⎠ ⎦
⎡ ⎛ ∂w ∂w ∂w ∂w ⎞ ⎤
+ ⎢ ∫∫ ⎜ k 1 N + k 1 N ⎟ dA⎥ TˆN − ∫∫ w1 g ′′′ dx dy = ∫ w1 qn′′ ds
⎣ A ⎝ ∂x ∂x ∂y ∂y ⎠ ⎦ A B
⎡ ⎛ ∂w2 ∂w1 ∂w ∂w ⎞ ⎤ ⎡ ⎛ ∂w ∂w ∂w ∂w ⎞ ⎤
⎢ ∫∫ ⎜ k + k 2 1 ⎟ dA⎥ Tˆ1 + ⎢ ∫∫ ⎜ k 2 2 + k 2 2 ⎟ dA⎥ Tˆ2 + ...
⎣ A ⎝ ∂x ∂x ∂y ∂y ⎠ ⎦ ⎣ A ⎝ ∂x ∂x ∂y ∂y ⎠ ⎦
(6)
⎡ ⎛ ∂w ∂wN ∂w ∂wN ⎞ ⎤ ˆ
+ ⎢ ∫∫ ⎜ k 2 +k 2 ⎟ dA⎥ TN − ∫∫ w2 g ′′′ dx dy = ∫ w2 qn′′ ds
⎣ A ⎝ ∂x ∂x ∂y ∂y ⎠ ⎦ A B

...

⎡ ⎛ ∂wN ∂w1 ∂w ∂w ⎞ ⎤ ⎡ ⎛ ∂w ∂w2 ∂w ∂w2 ⎞ ⎤ ˆ


⎢ ∫∫ ⎜ k + k N 1 ⎟ dA⎥ Tˆ1 + ⎢ ∫∫ ⎜ k N +k N ⎟ dA⎥ T2 + ...
⎣A ⎝ ∂x ∂x ∂y ∂y ⎠ ⎦ ⎣A ⎝ ∂x ∂ x ∂ y ∂ y ⎠ ⎦
⎡ ⎛ ∂w ∂wN ∂w ∂wN ⎞ ⎤ ˆ
+ ⎢ ∫∫ ⎜ k N +k N ⎟ dA⎥ TN − ∫∫ wN g ′′′ dx dy = ∫ wN qn′′ ds
⎣A ⎝ ∂x ∂x ∂y ∂y ⎠ ⎦ A B

Equation (6) can be written more concisely in vector notation:

⎡ ⎛ ∂ w ∂ wT ∂ w ∂ wT ⎞ ⎤ ˆ
⎢ ∫∫ k ⎜ + ⎟ dA⎥ T − ∫∫ w g ′′′ dx dy = ∫ w qn′′ ds (7)
⎣⎢ A ⎝ ∂ x ∂x ∂y ∂y ⎠
⎦⎥
 
A

 B

K g q

The construction of the matrix K and the vector q are discussed in Section 2.7.2. This problem
focuses on the construction of the generation matrix g . The generation vector is assembled by
the summation of element generation vectors according to:

Ne
g = ∑ ge (8)
i =1

where the element generation vector is given by:

g e = ∫∫ w g e′′′dx dy (9)
Ae

where g e′′′ is the volumetric generation within element e. Within element e, only the weighting
functions wi, wj, and wk are nonzero (where i, j, and k are the indices of the nodes that define the
element):

w = ⎡⎣ 0 ... 0 wi 0 ... 0 w j 0 ... 0 wk 0 ... 0 ⎤⎦


T
(10)

Substituting Eq. (10) into Eq. (9) leads to:


⎡0 ⎤
⎢ ⎥
⎢... ⎥
⎢0 ⎥
⎢ ⎥
⎢ wi ⎥
⎢0 ⎥
⎢ ⎥
⎢... ⎥
⎢0 ⎥
⎢ ⎥
g e = g e′′′∫∫ ⎢ w j ⎥ dx dy
Ae ⎢ ⎥
⎢0 ⎥
⎢... ⎥
⎢ ⎥
⎢0 ⎥
⎢w ⎥
⎢ k⎥
⎢0 ⎥
⎢ ⎥
⎢... ⎥
⎢⎣0 ⎥⎦ (11)

The integral of a weighting function over the element is obtained from Eq. (S2.7.2-55):

a !b !c !
∫∫ w wbj wkc dA =
a
2A (S2.7.2-55)
Ae
i
( a + b + c + 2 )! e

or

1!0!0! 1
∫∫ w
Ae
i dA = ∫∫ wi1 w0j wk0 dA =
Ae (1 + 0 + 0 + 2 )!
2 Ae = Ae
3
(12)

Therefore, Eq. (11) can be written as:

⎡. ⎤
⎢⎥ row i
⎢1⎥
⎢. ⎥
 e′′′ Ae ⎢ ⎥
g
ge = row j
3 ⎢⎢ ⎥⎥
1
.
⎢⎥
⎢1⎥ row k
⎢⎣. ⎥⎦
(13)

The overall matrix equation becomes:


( K + H ) Tˆ = Q + g (14)

b.) Re-solve the problem discussed in Section 2.7.2 assuming that the material experiences a
volumetric generation rate of g ′′′ = 1x103 W/m3, as shown in Figure P2.7-5.

ht = 100 W/m -K
2

T∞,t = 300 K
(0,0.75) (0.5,0.75)

k = 0.25 W/m-K
q ′′ = 100 W/m
2
g ′′′ = 1500 W/m
3

(0.5,0.25)

hb = 50 W/m -K
2
(0,0) (0.25,0)
T∞,b = 350 K
Figure P2.7-5: Two-dimensional conduction problem used to illustrate the finite element solution with
generation. The coordinates of points are shown in m.

The solution proceeds as discussed in Section 2.7.2. The information about the sub-domains is
contained in a matrix sd . The number of columns in the matrix sd is equal to the number of
sub-domains in the computational domain. The information about the sub-domains is contained
in the column. The first row of each column contains the conductivity of the sub-domain and the
second row the generation. The matrix sd is setup in the MATLAB script S2p7d5 according to:

clear all;

% setup details of computational domain


sdm=[0.25 1500]'; % conductivity and generation of each sub-domain
bcm=[0,0,0; 50,350,0; 0,0,0; 100,300,0; 0,0,100]'; % boundary condition for each boundary

The boundary condition matrix is unchanged:

bcm=[0,0,0; 50,350,0; 0,0,0; 100,300,0; 0,0,100]'; % boundary condition for each boundary

The same mesh is used, shown in Figure 2:


n11 b6 n10 b5 n9
e8 e11
b7 b4
e9 e10
n6 n8
n7
b8 e6 e5 b3
e7 e4
n5 n3
n4
e1
b9 e3 n = node#
e2 b2 e = element #
n1 b1 n2 b = boundary segment #
Figure 2: A crude mesh.

The point, edge, and triangle matrices remain unchanged:

% setup details of the grid


N_n=11; % number of nodes
% setup point matrix - coordinates of each node
pm=[0,0; 0.25,0; 0.5,0.25; 0.25,0.25; 0,0.25; 0,0.5; 0.25,0.5; 0.5,0.5; 0.5,0.75; 0.25,0.75; 0,0.75]';

N_b=9; % number of boundary segments


% setup edge matrix - nodes that define each segment and boundary
em=[1,2,1; 2,3,2; 3,8,3; 8,9,3; 9,10,4; 10,11,4; 11,6,5; 6,5,5; 5,1,5]';

N_e=11; % number of elements


% setup triangle matrix - nodes that define each element and subdomain
tm=[5,1,4,1; 1,2,4,1; 4,2,3,1; 4,3,8,1; 7,4,8,1; 6,4,7,1; 6,5,4,1; 11,6,10,1; 6,7,10,1; 10,7,8,1; 10,8,9,1]';

The global conduction matrix and generation vector are setup together, element-by-element.
Both are initialized:

K=zeros(N_n,N_n); % global conduction matrix initialization


g=zeros(N_n,1); % global generation matrix initialization

These for loop cycles through all of the elements:

for e=1:N_e

The indices of the three nodes that define the element and the subdomain that the element
belongs to are obtained from the triangle matrix:

i=tm(1,e); % index of node i


j=tm(2,e); % index of node j
k=tm(3,e); % index of node k
d=tm(4,e); % subdomain of element

The x- and y-distances separating the nodes (xij, xik, etc.) are computed; the coordinates of the
nodes are obtained from the point matrix:

xij=pm(1,j)-pm(1,i); % x-distance between nodes j and i in element e


xik=pm(1,k)-pm(1,i); % x-distance between nodes k and i in element e
xjk=pm(1,k)-pm(1,j); % x-distance between nodes k and j in element e
yij=pm(2,j)-pm(2,i); % y-distance between nodes j and i in element e
yik=pm(2,k)-pm(2,i); % y-distance between nodes k and i in element e
yjk=pm(2,k)-pm(2,j); % y-distance between nodes k and j in element e

The parameter bijk and the area of the element are computed:

bijk=xij*yjk-xjk*yij;
Ae=abs(bijk)/2; % area of element e

The conductivity of the element is obtained from the matrix sd and the elements of the element
conduction matrix are added to the global conduction matrix.

% add element conduction matrix to global conduction matrix


ke=sdm(1,d); % conductivity of element
K(i,i)=K(i,i)+Ae*ke*(xjk^2+yjk^2)/bijk^2;
K(i,j)=K(i,j)-Ae*ke*(xjk*xik+yjk*yik)/bijk^2;
K(i,k)=K(i,k)+Ae*ke*(xjk*xij+yjk*yij)/bijk^2;
K(j,i)=K(j,i)-Ae*ke*(xik*xjk+yik*yjk)/bijk^2;
K(j,j)=K(j,j)+Ae*ke*(xik^2+yik^2)/bijk^2;
K(j,k)=K(j,k)-Ae*ke*(xik*xij+yik*yij)/bijk^2;
K(k,i)=K(k,i)+Ae*ke*(xij*xjk+yij*yjk)/bijk^2;
K(k,j)=K(k,j)-Ae*ke*(xij*xik+yij*yik)/bijk^2;
K(k,k)=K(k,k)+Ae*ke*(xij^2+yij^2)/bijk^2;

The generation within the element is obtained from the matrix sd and the elements of the
element conduction matrix are added to the global conduction matrix according to Eq. (13).

% add element generation matrix to global generation matrix


ge=sdm(2,d); % generation of element
g(i,1)=g(i,1)+Ae*ge/3;
g(j,1)=g(j,1)+Ae*ge/3;
g(k,1)=g(k,1)+Ae*ge/3;
end

The vector Q and matrix H are unchanged:

Q=zeros(N_n,1); % global boundary vector initialization


H=zeros(N_n,N_n); % global convection matrix initialization
for b=1:N_b
i=em(1,b); % index of node i
j=em(2,b); % index of node j
bndry=em(3,b); % boundary of boundary segment
xij=pm(1,j)-pm(1,i); % x-distance between nodes j and i on boundary segment b
yij=pm(2,j)-pm(2,i); % y-distance between nodes j and i on boundary segment b
sij=sqrt(xij^2+yij^2); % length of boundary segment b

qfsb=bcm(3,bndry); % specified heat flux (W/m^2)


hb=bcm(1,bndry); % heat transfer coefficient (W/m^2-K)
Tinfb=bcm(2,bndry); % ambient temperature (K)

% add segment boundary vector to global boundary vector


Q(i,1)=Q(i,1)+qfsb*sij/2+hb*Tinfb*sij/2;
Q(j,1)=Q(j,1)+qfsb*sij/2+hb*Tinfb*sij/2;

% add segment convection matrix to global convective matrix


H(i,i)=H(i,i)+hb*sij/3;
H(i,j)=H(i,j)+hb*sij/6;
H(j,i)=H(j,i)+hb*sij/6;
H(j,j)=H(j,j)+hb*sij/3;
end

Equation (14) is solved and the result is plotted:

T=(K+H)\(Q+g); % obtain temperatures at each node


pdeplot(pm,em,tm,'xydata',T,'contour','on');

The solution is shown in Figure 3.

Figure 3: Solution to the problem shown in Figure 2.7-5 with the mesh shown in Figure 2.

Note that the PDE toolbox can be used to generate a more refined mesh, as shown in Figure 4.
Figure 4: Refined mesh generated by the PDE toolbox.

The mesh is exported to the workspace and the script is altered as shown below (changes are
highlighted in bold):

%clear all;

% setup details of computational domain


sdm=[0.25 1500]'; % conductivity and generation of each sub-domain
bcm=[0,0,0; 50,350,0; 0,0,0; 100,300,0; 0,0,100]'; % boundary condition for each boundary

% % setup details of the grid


% N_n=11; % number of nodes
% % setup point matrix - coordinates of each node
% pm=[0,0; 0.25,0; 0.5,0.25; 0.25,0.25; 0,0.25; 0,0.5; 0.25,0.5; 0.5,0.5; 0.5,0.75; 0.25,0.75; 0,0.75]';
%
% N_b=9; % number of boundary segments
% % setup edge matrix - nodes that define each segment and boundary
% em=[1,2,1; 2,3,2; 3,8,3; 8,9,3; 9,10,4; 10,11,4; 11,6,5; 6,5,5; 5,1,5]';
%
% N_e=11; % number of elements
% % setup triangle matrix - nodes that define each element and subdomain
% tm=[5,1,4,1; 1,2,4,1; 4,2,3,1; 4,3,8,1; 7,4,8,1; 6,4,7,1; 6,5,4,1; 11,6,10,1; 6,7,10,1; 10,7,8,1; 10,8,9,1]';

% % setup details of the grid


% N_n=11; % number of nodes
% % setup point matrix - coordinates of each node
% pm=[0,0; 0.25,0; 0.5,0.25; 0.25,0.25; 0,0.25; 0,0.5; 0.25,0.5; 0.5,0.5; 0.5,0.75; 0.25,0.75; 0,0.75]';
%
% N_b=9; % number of boundary segments
% % setup edge matrix - nodes that define each segment and boundary
% em=[1,2,1; 2,3,2; 3,8,3; 8,9,3; 9,10,4; 10,11,4; 11,6,5; 6,5,5; 5,1,5]';
%
% N_e=11; % number of elements
% % setup triangle matrix - nodes that define each element and subdomain
% tm=[5,1,4,1; 1,2,4,1; 4,2,3,1; 4,3,8,1; 7,4,8,1; 6,4,7,1; 6,5,4,1,; 11,6,10,1; 6,7,10,1; 10,7,8,1;
10,8,9,1]';

[g,N_n]=size(p);% number of nodes


pm=p; % point matrix is equal to the one produced by pdetool
[g,N_e]=size(t);% number of elements
tm=t; % triangle matrix is equal to the one produced by pdetool

[g,N_b]=size(e);% number of boundary segments


em=zeros(3,N_b);% initialize edge matrix
em(1,:)=e(1,:); % index of node i
em(2,:)=e(2,:); % index of node j
em(3,:)=e(5,:); % boundary

K=zeros(N_n,N_n); % global conduction matrix initialization


g=zeros(N_n,1); % global generation matrix initialization
for e=1:N_e
i=tm(1,e); % index of node i
j=tm(2,e); % index of node j
k=tm(3,e); % index of node k
d=tm(4,e); % subdomain of element
xij=pm(1,j)-pm(1,i); % x-distance between nodes j and i in element e
xik=pm(1,k)-pm(1,i); % x-distance between nodes k and i in element e
xjk=pm(1,k)-pm(1,j); % x-distance between nodes k and j in element e
yij=pm(2,j)-pm(2,i); % y-distance between nodes j and i in element e
yik=pm(2,k)-pm(2,i); % y-distance between nodes k and i in element e
yjk=pm(2,k)-pm(2,j); % y-distance between nodes k and j in element e
bijk=xij*yjk-xjk*yij;
Ae=abs(bijk)/2; % area of element e

% add element conduction matrix to global conduction matrix


ke=sdm(1,d); % conductivity of element
K(i,i)=K(i,i)+Ae*ke*(xjk^2+yjk^2)/bijk^2;
K(i,j)=K(i,j)-Ae*ke*(xjk*xik+yjk*yik)/bijk^2;
K(i,k)=K(i,k)+Ae*ke*(xjk*xij+yjk*yij)/bijk^2;
K(j,i)=K(j,i)-Ae*ke*(xik*xjk+yik*yjk)/bijk^2;
K(j,j)=K(j,j)+Ae*ke*(xik^2+yik^2)/bijk^2;
K(j,k)=K(j,k)-Ae*ke*(xik*xij+yik*yij)/bijk^2;
K(k,i)=K(k,i)+Ae*ke*(xij*xjk+yij*yjk)/bijk^2;
K(k,j)=K(k,j)-Ae*ke*(xij*xik+yij*yik)/bijk^2;
K(k,k)=K(k,k)+Ae*ke*(xij^2+yij^2)/bijk^2;

% add element generation matrix to global generation matrix


ge=sdm(2,d); % generation of element
g(i,1)=g(i,1)+Ae*ge/3;
g(j,1)=g(j,1)+Ae*ge/3;
g(k,1)=g(k,1)+Ae*ge/3;
end

Q=zeros(N_n,1); % global boundary vector initialization


H=zeros(N_n,N_n); % global convection matrix initialization
for b=1:N_b
i=em(1,b); % index of node i
j=em(2,b); % index of node j
bndry=em(3,b); % boundary of boundary segment
xij=pm(1,j)-pm(1,i); % x-distance between nodes j and i on boundary segment b
yij=pm(2,j)-pm(2,i); % y-distance between nodes j and i on boundary segment b
sij=sqrt(xij^2+yij^2); % length of boundary segment b

qfsb=bcm(3,bndry); % specified heat flux (W/m^2)


hb=bcm(1,bndry); % heat transfer coefficient (W/m^2-K)
Tinfb=bcm(2,bndry); % ambient temperature (K)

% add segment boundary vector to global boundary vector


Q(i,1)=Q(i,1)+qfsb*sij/2+hb*Tinfb*sij/2;
Q(j,1)=Q(j,1)+qfsb*sij/2+hb*Tinfb*sij/2;

% add segment convection matrix to global convective matrix


H(i,i)=H(i,i)+hb*sij/3;
H(i,j)=H(i,j)+hb*sij/6;
H(j,i)=H(j,i)+hb*sij/6;
H(j,j)=H(j,j)+hb*sij/3;
end

T=(K+H)\(Q+g); % obtain temperatures at each node


pdeplot(pm,em,tm,'xydata',T,'contour','on');

The solution with the refined mesh is shown in Figure 5.

Figure 5: Solution to the problem shown in Figure 2.7-5 with the mesh shown in Figure 4.
Problem 2.7-6
Figure P2.7-6 illustrates a tube in a water-to-air heat exchanger with a layer of polymer coating
that can be easily etched away in order to form an array of fin-like structures that increase the
surface area exposed to air.

ha = 10 W/m -K
2

polymer, kp = 1.5 W/m-K Ta = 290 K

hw = 250 W/m -K
2

Tw = 320 K
tube, kt = 15 W/m-K unit cell of fin structure
Figure P2.7-6: Tube coated with polymer and a unit cell showing fin-like structures etched into polymer.

The water flowing through the tube has temperature Tw = 320 K and heat transfer coefficient hw =
250 W/m2-K. The air has temperature Ta = 290 K and heat transfer coefficient ha = 10 W/m2-K.
The thermal conductivity of the polymer and tube material is kp = 1.5 W/m-K and kt = 15 W/m-
K, respectively.
a.) Generate a finite element solution for the temperature distribution within the unit cell shown
in Figure 2.7-6 using the mesh shown in Figure 2.7-6(b). The coordinates of the nodes are
listed in Table P2.7-6.

13 14

12
11
10
7 8 9

6 5 4

1 2 3
Figure P2.7-6(b): Mesh for finite element solution.

Table 2.7-6: Coordinates of nodes in Figure 2.7-6(b).


Node x-coord. (m) y-coord. (m) Node x-coord. (m) y-coord. (m)
1 0 0 8 0.01 0.01
2 0.01 0 9 0.015 0.01
3 0.015 0 10 0 0.015
4 0.015 0.008 11 0.007 0.02
5 0.005 0.008 12 0.005 0.025
6 0 0.008 13 0 0.03
7 0.006 0.01 14 0.009 0.03
The first step is to define the sub-domains and boundaries that specify the problem. The
information about the sub-domains is contained in a matrix sd . The number of columns in the
matrix sd is equal to the number of sub-domains in the computational domain. The information
about the sub-domains is contained in the column. For this problem, only the conductivity is
required. There are two sub-domains, as shown in Figure 3, and therefore the matrix sd is setup
in the MATLAB function S2p7p6 according to:

function[]=P2p7d6(h_a)

% Input:
% h_a - heat transfer coefficient on the air-side (W/m^2-K)
%

% setup details of computational domain


sdm=[15; 1.5]'; % conductivity and generation of each sub-domain

subdomain 2 boundary 3

boundary 4

boundary 2
subdomain 1

boundary 1
Figure 3:Sub-domains and boundaries.

The information about the boundary conditions is contained in a matrix bc . The number of
columns in the matrix bc is equal to the number of boundaries in the computational domain.
The information about the boundaries is contained in the column: the first row holds the heat
transfer coefficient, the second row holds the adjacent fluid temperature, and the third row holds
the specified heat flux into the computational domain. There are four boundaries for the
problem, as shown in Figure 3, and therefore the matrix bc is setup according to:

bcm=[250,320,0; 0,0,0; h_a,290,0; 0,0,0]'; % boundary condition for each boundary

The mesh shown in Figure P2.7-6(b) is defined by Nn = 14 nodes. Information about the nodes
is stored in the point matrix, p . There are Nn columns in the point matrix. The first row of each
column contains the x-coordinate of the corresponding node and the second row contains the y-
coordinate. The point matrix is setup:

% setup details of the grid


N_n=14; % number of nodes
% setup point matrix - coordinates of each node
pm=[0,0; 0.01,0; 0.015,0; 0.015,0.008; 0.005,0.008; 0,0.008; 0.006,0.01; 0.01,0.01; 0.015,0.01;...
0,0.015; 0.007,0.02; 0.005,0.025; 0,0.03; 0.009,0.03]';

The mesh is defined by Nb = 12 boundary segments. Information about the boundary segments
is stored in the edge matrix, e . There are Nb columns in the edge matrix. The first two rows of
each column contain the indices of the nodes that define the boundary segment. The third row
contains the index of the boundary that the boundary segment lies on.

N_b=12; % number of boundary segments


% setup edge matrix - nodes that define each segment and boundary
em=[1,2,1; 2,3,1; 3,4,2; 4,9,2; 9,8,3; 8,7,3; 7,11,3; 11,14,3; 14,13,3; 13,10,4; 10,6,4; 6,1,4]';

The mesh is defined by Ne = 14 elements. Information about the edge is stored in the triangle
matrix, t . There are Ne columns in the triangle matrix. The first three rows contain the indices
of the three nodes that define the elements, given in clockwise order. The fourth row contains
the number of the subdomain that the element lies in.

N_e=14; % number of elements


% setup triangle matrix - nodes that define each element and subdomain
tm=[6,1,5,1; 1,2,5,1; 5,2,4,1; 2,3,4,1; 8,4,9,2; 5,4,8,2; 7,5,8,2;...
7,6,5,2; 10,6,7,2; 10,7,11,2; 12,10,11,2; 13,10,12,2; 12,11,14,2;...
13,12,14,2]';

The element conduction matrix was derived in Eq. (S2.7.2-66), which is repeated below:

⎡. . . . . . .⎤
⎢ ⎥
(x jk + y jk ) . − (x jk xik + y jk yik ) . (x xij + y jk yij )
2 2
⎢. jk .⎥ row i
⎢ ⎥
⎢. . . . . . .⎥
K e = 2 ⎢. − (xik x jk + yik y jk ) (x + yik2 ) . − (xik xij + yik yijk ) .⎥ row j
Ae ke 2
.
bijk ⎢ ⎥
ik

⎢. . . . . . .⎥
⎢ ⎥
⎢. (xij x jk + yij y jk ) − (xij xik + yij yik ) . (x + yij2 )
2
. ij .⎥ row k
⎢ ⎥
⎣. . . . . . .⎦

column i column j column k


(S2.7.2-66)

The elements associated with the element conduction matrix are added, element by element, to
the global conduction matrix according to:

Ne
K = ∑ Ke (S2.7.2-1)
i =1
The global conduction matrix is initialized:

K=zeros(N_n,N_n); % global conduction matrix initialization

The global conduction matrix is constructed element-by-element within a for loop:

for e=1:N_e

The indices of the three nodes that define the element and the subdomain that the element
belongs to are obtained from the triangle matrix:

i=tm(1,e); % index of node i


j=tm(2,e); % index of node j
k=tm(3,e); % index of node k
d=tm(4,e); % subdomain of element

The x- and y-distances separating the nodes (xij, xik, etc.) are computed; the coordinates of the
nodes are obtained from the point matrix:

xij=pm(1,j)-pm(1,i); % x-distance between nodes j and i in element e


xik=pm(1,k)-pm(1,i); % x-distance between nodes k and i in element e
xjk=pm(1,k)-pm(1,j); % x-distance between nodes k and j in element e
yij=pm(2,j)-pm(2,i); % y-distance between nodes j and i in element e
yik=pm(2,k)-pm(2,i); % y-distance between nodes k and i in element e
yjk=pm(2,k)-pm(2,j); % y-distance between nodes k and j in element e

The parameter bijk and the area of the element are computed:

bijk=xij*yjk-xjk*yij;
Ae=abs(bijk)/2; % area of element e

The conductivity of the element is obtained from the matrix sd and the elements of the element
conduction matrix are added to the global conduction matrix according to Eq. (S2.2-66)..

% add element conduction matrix to global conduction matrix


ke=sdm(1,d); % conductivity of element
K(i,i)=K(i,i)+Ae*ke*(xjk^2+yjk^2)/bijk^2;
K(i,j)=K(i,j)-Ae*ke*(xjk*xik+yjk*yik)/bijk^2;
K(i,k)=K(i,k)+Ae*ke*(xjk*xij+yjk*yij)/bijk^2;
K(j,i)=K(j,i)-Ae*ke*(xik*xjk+yik*yjk)/bijk^2;
K(j,j)=K(j,j)+Ae*ke*(xik^2+yik^2)/bijk^2;
K(j,k)=K(j,k)-Ae*ke*(xik*xij+yik*yij)/bijk^2;
K(k,i)=K(k,i)+Ae*ke*(xij*xjk+yij*yjk)/bijk^2;
K(k,j)=K(k,j)-Ae*ke*(xij*xik+yij*yik)/bijk^2;
K(k,k)=K(k,k)+Ae*ke*(xij^2+yij^2)/bijk^2;
end

The vector Qb and matrix H b were derived in Eq. (S2.7.2-84), which is repeated below:
row i ⎡. . . . .⎤
⎡.⎤ ⎡.⎤ ⎢ ⎥
⎢. 1 1
⎢1⎥ ⎢1⎥ . .⎥ row i
qs′′,b sij ⎢ ⎥ hb T∞,b sij ⎢ ⎥ ⎢ 3 6 ⎥
qb = ⎢.⎥ + ⎢ . ⎥ − hb sij ⎢. . . . .⎥ Tˆ
2 ⎢⎥ 2 ⎢⎥ ⎢ ⎥
⎢⎥1 ⎢⎥1 ⎢. 1
.
1
.⎥ row j
⎢⎣ . ⎥⎦ ⎢⎣ . ⎥⎦ ⎢ 6 3 ⎥
⎢ ⎥
row j ⎣. . . . .⎦

Qb Hb column i
column j (S2.7.2-84)

These variables are initialized:

Q=zeros(N_n,1); % global boundary vector initialization


H=zeros(N_n,N_n); % global convection matrix initialization

and constructed boundary segment-by-boundary segment using a for loop:

for b=1:N_b

The indices of the two nodes that define the boundary segment and the boundary that the
segment belongs to are obtained from the edge matrix:

i=em(1,b); % index of node i


j=em(2,b); % index of node j
bndry=em(3,b); % boundary of boundary segment

The x- and y-distance separating the nodes and the linear distance between the nodes, sij, are
computed:

xij=pm(1,j)-pm(1,i); % x-distance between nodes j and i on boundary segment b


yij=pm(2,j)-pm(2,i); % y-distance between nodes j and i on boundary segment b
sij=sqrt(xij^2+yij^2); % length of boundary segment b

The specified heat flux, heat transfer coefficient, and ambient temperature associated with the
boundary segment is obtained from the matrix bc and the elements of Qb and H b are added to
the global vector Q and matrix H according to Eq. (S2.7.2-84).

qfsb=bcm(3,bndry); % specified heat flux (W/m^2)


hb=bcm(1,bndry); % heat transfer coefficient (W/m^2-K)
Tinfb=bcm(2,bndry); % ambient temperature (K)

% add segment boundary vector to global boundary vector


Q(i,1)=Q(i,1)+qfsb*sij/2+hb*Tinfb*sij/2;
Q(j,1)=Q(j,1)+qfsb*sij/2+hb*Tinfb*sij/2;

% add segment convection matrix to global convective matrix


H(i,i)=H(i,i)+hb*sij/3;
H(i,j)=H(i,j)+hb*sij/6;
H(j,i)=H(j,i)+hb*sij/6;
H(j,j)=H(j,j)+hb*sij/3;
end

The temperatures at each node are obtained:

T=(K+H)\Q; % obtain temperatures at each node

and plotted according to:

pdeplot(pm,em,tm,'xydata',T,'contour','on');

which leads to Figure 4.

Figure 4: Contour plot of the finite element solution.

b.) Plot the average heat flux at the internal surface of the tube as a function of the air-side heat
transfer coefficient with and without the polymer coating. You should see a cross-over point
where it becomes disadvantageous to use the polymer coating; explain this.

The heat transfer into the different boundaries can be evaluated by summing the heat transfer to
the boundary segments that lie on that boundary. The heat transfer from the water and the air is
set to 0 and then each boundary segment is considered one by one:

q_dot_w=0;
q_dot_a=0;
for b=1:N_b

The indices of the nodes that define the boundary segment and the boundary that the segment lies
on are obtained from the edge matrix:

i=em(1,b); % index of node i


j=em(2,b); % index of node j
bndry=em(3,b); % boundary of boundary segment

The length of the segment is computed:

xij=pm(1,j)-pm(1,i); % x-distance between nodes j and i on boundary segment b


yij=pm(2,j)-pm(2,i); % y-distance between nodes j and i on boundary segment b
sij=sqrt(xij^2+yij^2); % length of boundary segment b

If the boundary segment lies on boundary 1 then it is exposed to water and the heat transfer into
that boundary is added to the total heat transfer to the water. The heat transfer is the product of
the length of the segment, the heat transfer coefficient and the difference between the adjacent
water temperature and the average temperature on the boundary.

if(bndry==1)
hb=bcm(1,bndry); % heat transfer coefficient (W/m^2-K)
Tinfb=bcm(2,bndry); % ambient temperature (K)
T_b=(T(i)+T(j))/2; % average temperature on boundary (K)
q_dot_w=q_dot_w+sij*hb*(Tinfb-T_b); % rate of heat transfer per unit length (W/m)
end

If the boundary segment lies on boundary 3 then it is exposed to air and the heat transfer into that
boundary is added to the total heat transfer to the air.

if(bndry==3)
hb=bcm(1,bndry); % heat transfer coefficient (W/m^2-K)
Tinfb=bcm(2,bndry); % ambient temperature (K)
T_b=(T(i)+T(j))/2; % average temperature on boundary (K)
q_dot_a=q_dot_a+sij*hb*(Tinfb-T_b); % rate of heat transfer per unit length (W/m)
end
end

The heat flux from the water is the heat transfer over the area:

q_dot_flux=q_dot_w/0.015; % heat flux


end

which leads to 570.6 W/m2. The function P2p7d6_makefig runs the finite element solution over a
range of air-side heat transfer coefficients:

h_a=1;
for i=1:101
h_av(i)=h_a;
[q_dot_fluxv(i)]=P2p7d6(h_a);
h_a=h_a*1.08;
end

Figure 5 illustrates the heat flux as a function of air-side heat transfer coefficient predicted by the
finite element solution.

7000

with polymer fins


1000
Heat flux (W/m )
2

without polymer fins

100

10
1 10 100 1000 2250
2
Air-side heat transfer coefficient (W/m -K)
Figure 5: Heat flux as a function of air-side heat transfer coefficient with the polymer fins and without the
polymer (bare tube).

If the polymer is removed, then the problem is a 1-D conduction problem that can be solved in
EES. The inputs are entered:

$UnitSystem SI MASS RAD PA K J


$TABSTOPS 0.2 0.4 0.6 0.8 3.5 in

"Inputs"
{h_bar_a=10 [W/m^2-K]} "heat transfer coefficient on air-side"
s=0.015 [m] "width of tube"
h_bar_w=250 [W/m^2-K] "heat transfer coefficient on water-side"
k_t=15 [W/m-K] "conductivity of tube"
th_t=0.008 [m] "tube thickness"
T_w=320 [K] "water temperature"
T_a=290 [K] "air temperature"

The resistance to conduction through the tube, per unit area, is:

tht
Rtube =
kt

The resistance to convection on the air- and water-sides, per unit area, are computed according
to:
1
Rconv ,a =
ha

1
Rconv , w =
hw

The total resistance per unit area is:

Rtotal = Rconv ,a + Rtube + Rconv , w

and the heat flux associated with the bare tube is:

q ′′ =
(Tw − Ta )
Rtotal

The heat flux associated with the bare tube (without the polymer fins) as a function of the air-
side heat transfer coefficient is shown in Figure 5. The addition of the polymer fins adds a
resistance to conduction through the polymer but reduces the resistance to air-side convection. If
the resistance to air-side convection is sufficiently small, then the reduction of the air-side
convetion resistance is not sufficient to make up for the additional resistance due to conduction
through the polymer. Therefore, there is a cross-over point in Figure 5.
Problem 2.7-7
Figure P2.7-7 illustrates a power electronics chip that is used to control the current to a winding
of a motor.

ha = 10 W/m -K
silicon chip 2

ks = 80 W/m-K Ta = 20°C
spreader
k = 45 W/m-K
0.5 cm sp
0.2 cm
0.5 cm

0.1 cm 1.4 cm
dielectric layer
hw = 1000 W/m -K
2
kd = 1 W/m-K
Tw = 10°C
Figure P2.7-7: Power electronics chip.

The silicon chip has dimensions 0.2 cm by 0.5 cm and conductivity ks = 80 W/m-K. A
generation of thermal energy occurs due to losses in the chip; the thermal energy generation can
be modeled as being uniformly distributed with a value of g ′′′ = 1x108 W/m3 in the upper 50% of
the silicon. The chip is thermally isolated from the spreader by a dielectric layer with thickness
0.1 cm and conductivity kd = 1 W/m-K. The spreader has dimension 0.5 cm by 1.4 cm and
conductivity ksp = 45 W/m-K. The external surfaces are all air cooled with ha = 10 W/m 2 -K and
Ta = 20ºC except for the bottom surface of the spreader which is water cooled with
hw = 1000 W/m 2 -K and Tw = 10ºC.
a.) Develop a numerical model of the system using FEHT.

FEHT is opened and Heat Transfer is selected from the Subject menu. Steady-state and
Temperatures in C are selected from the setup menu and a grid of dimension 0.2 cm x 0.2 cm is
specified. A rough geometry is obtained using the Outline selection from the Draw menu, as
shown in Figure 2.

Figure 2: Rough geometry.

The precise coordinates of each point are specified by double-clicking on each one in turn. The
result is shown in Figure 3.
Figure 3: Geometry completely specified.

The material properties are specified by selecting an outline and then selecting Material
Properties from the Specify menu. The boundary conditions and generation are specified in the
same way. A very crude mesh is obtained by selecting Element Lines from the Draw menu.
The result is shown in Figure 4.

Figure 4: Crude mesh.

The problem is solved by selecting Calculate from the Run menu. The temperatures are shown
at each node by selecting Temperatures from the View menu, as shown in Figure 5. The
maximum temperature is 153ºC.
Figure 5: Temperature predicted at each node.

b.) Plot the maximum temperature in the system as a function of the number of nodes.

The mesh is successively refined (by selecting Reduce Mesh from the Draw menu. Figure 6
illustrates a highly reduced mesh and Figure 7 illustrates the maximum temperature in the
silicon as a function of the number of nodes in the mesh.

Figure 6: Highly reduced mesh.


154.4

154.2

Maximum temperature (°C)


154

153.8

153.6

153.4

153.2

153
0 100 200 300 400 500 600
Number of nodes
Figure 7: Maximum temperature as a function of the number of nodes.

c.) Develop a simple sanity check of your results using a resistance network.

Figure 8 illustrates the temperature contours predicted by the FEHT model.

Figure 8: Temperature contours predicted by the FEHT model.

Figure 8 shows that the temperature is reduced primarily by conduction across the dielectric and
convection from the surface of the spreader to the water. The temperature gradients caused by
conduction within the spreader and silicon are small. These observations are consistent with a
simple resistance network in which the heat transfer related to generation:

1x108 W 1 m 0.001 m 0.005 m


q = =500 W (1)
m3
must pass through a resistance due to conduction through the silicon:

0.002 m mK K
Rcond , s = = 0.005 (2)
0.005 m 1 m 80 W W

a resistance due to conduction through the dielectric layer:

0.001 m mK K
Rcond ,d = = 0.2 (3)
0.005 m 1 m 1 W W

a resistance due to conduction through the spreader:

0.005 m mK K
Rcond , sp = = 0.008 (4)
0.014 m 1 m 45 W W

and a resistance due to convection to the water:

m2 K K
Rconv , w = = 0.071 (5)
1000 W 0.014 m 1 m W

The resistance to conduction through the dielectric and convection to the water are important,
the others are not. The temperature difference between the silicon and the spreader is
approximately:

0.2 K 500 W
ΔTd = Rcond , d q = = 100 K (6)
W

and the temperature difference between the water and the spreader is approximately:

0.071 K 500 W
ΔTconv ,v = Rconv , w q = = 35.7 K (7)
W

These values are consistent with the FEHT predictions, providing some confidence in the
results.
Problem 2.7-8
Figure P2.7-8(a) illustrates a heat exchanger in which hot fluid and cold fluid flows through
alternating rows of square channels that are installed in a piece of material.

C C C C C C

H H H H H H
unit cell of
C C C C C C
heat exchanger
H H H H H H

H = channels carrying hot fluid


C = channels carrying hot fluid
Figure P2.7-8(a): Heat exchanger.

You are analyzing this heat exchanger and will develop a model of the unit cell shown in Figure
P2.7-8(a) and illustrated in more detail in Figure 2.7-8(b).

L = 6 mm h = 150 W/m -K
2

Th = 80°C
th/2 =0.4 mm

p/2 = 4 mm
thb = 0.8 mm

h = 150 W/m -K
2

Tc = 25°C k = 12 W/m-K
Figure 2.7-8(b): Details of unit cell shown in Figure 2.7-8(a).

The metal struts separating the square channels form fins. The length of the fin (the half-width
of the channel) is L = 6 mm and the fin thickness is th = 0.8 mm. The thickness of the material
separating the channels is thb = 0.8 mm. The distance between adjacent fins is p = 8 mm. The
channel structure for both sides (hot and cold) are identical. The conductivity of the metal is k =
12 W/m-K. The hot fluid has temperature Th = 80ºC and heat transfer coefficient h = 150
W/m2-K. The cold fluid has temperature Tc = 25ºC and the same heat transfer coefficient.
a.) Prepare a numerical model of the unit cell shown in Figure 2.7-8(b) using FEHT.

The geometry is entered in FEHT using the Outline selection from the draw menu. Each point is
precisely positioned by double-clicking on it and entering its coordinates. The result is shown in
Figure 3.
Figure 3: Geometry entered in FEHT.

The material properties and boundary conditions are entered. Note that the lines of symmetry
(i.e., the lines at the ends and centers of the fins) are adiabatic. A crude mesh is drawn. The
result is shown in Figure 4.

Figure 4: FEHT model with crude mesh.

The problem is solved, leading to the temperature contours shown in Figure 5.


Figure 5: Temperature distribution.

b.) Plot the rate of heat transfer from the hot fluid to the cold fluid within the unit cell as a
function of the number of nodes.

The boundary adjacent to the cold fluid is selected and Heat Flows is selected from the View
menu. The mesh is successively refined by selecting Reduce Mesh from the Draw menu and the
heat flow as a function of the number of nodes is shown in Figure 6.
32.2

32

Heat transfer rate (W/m)


31.8

31.6

31.4

31.2

31
0 100 200 300 400 500 600 700
Number of nodes
Figure 6: Heat transfer rate (per length of heat exchanger) as a function of the number of nodes.

c.) Develop a simple model of the unit cell using a resistance network and show that your result
from (b) makes sense.

The inputs are entered in EES:

$UnitSystem SI MASS RAD PA K J


$Tabstops 0.2 0.4 0.6 3.5 in

L=6 [mm]*convert(mm,m) "length of hot-side fins"


th=0.8 [mm]*convert(mm,m) "thickness of hot-side fins"
p=8 [mm]*convert(mm,m) "fin-to-fin distance"
th_b=0.8 [mm]*convert(mm,m) "base thickness"
k=12 [W/m-K] "conductivity of material"
h_bar=150 [W/m^2-K] "heat transfer coefficient"
T_h=80 [C] "temperature of hot fluid"
T_c=25 [C] "temperature of cold fluid"
W=1 [m] "per unit length of heat exchanger"

The fin efficiency (ηf) for an adiabatic tipped constant cross-section fin is obtained using the
function eta_fin_straight_rect in EES. The resistance of each of the fins is:

1
Rf = (1)
h LW η f

where W is a unit length of heat exchanger and only the area of the fin within the unit cell is
used. The resistance of the unfinned region of the base is:

2
Ruf = (2)
h ( p − th ) W

The total resistance is:


−1
⎛ 1 1 ⎞
Rtotal = 2⎜ + ⎟⎟ (3)
⎜R
⎝ f Ruf ⎠

The rate of heat transfer is:

q =
(Th − Tc ) (4)
Rtotal

eta_f=eta_fin_straight_rect(th,L,h_bar,k) "fin efficiency"


R_f=1/(h_bar*W*L*eta_f) "fin resistance"
R_uf=1/(h_bar*W*(p-th)/2) "unfinned surface resistance"
R_total=2*(1/R_f+1/R_uf)^(-1) "total resistance between the streams"
q_dot=(T_h-T_c)/R_total "total heat transfer rate"

which leads to q = 32.6. Note that this is somewhat larger than the prediction from (b) because
the conduction resistance of the base (both across the base and along the base) is not considered.
Problem 2.8-1 (2-12 in text): Cryogenic Thermal Switch
There are several cryogenic systems that require a “thermal switch”, a device that can be
used to control the thermal resistance between two objects. One class of thermal switch
is activated mechanically and an attractive method of providing mechanical actuation at
cryogenic temperatures is with a piezoelectric stack; unfortunately, the displacement
provided by a piezoelectric stack is very small, typically on the order of 10 microns. A
company has proposed an innovative design for a thermal switch, shown in Figure P2.8-
1(a). Two blocks are composed of th = 10 μm laminations that are alternately copper (kCu
= 400 W/m-K) and plastic (kp = 0.5 W/m-K). The thickness of each block is L = 2.0 cm
in the direction of the heat flow. One edge of each block is carefully polished and these
edges are pressed together; the contact resistance associated with this joint is Rc′′ = 5x10-4
K-m2/W.

th = 10 μm plastic laminations
kp = 0.5 W/m-K
L = 2 cm L = 2 cm direction of actuation

TH TC

Figure P2.8-1(b)
“on” position “off” position
contact resistance, Rc′′ = 5x10 m -K/W
-4 2

th = 10 μm copper laminations
kCu = 400 W/m-K
Figure P2.8-1(a): Thermal switch in the “on” and “off” positions.

Figure P2.8-1(a) shows the orientation of the two blocks when the switch is in the “on”
position; notice that the copper laminations are aligned with one another in this
configuration which provides a continuous path for heat through high conductivity
copper (with the exception of the contact resistance at the interface). The vertical
location of the right-hand block is shifted by 10 μm to turn the switch "off". In the “off”
position, the copper laminations are aligned with the plastic laminations; therefore, the
heat transfer is inhibited by low conductivity plastic. Figure P2.8-1(b) illustrates a closer
view of half (in the vertical direction) of two adjacent laminations in the “on” and “off”
configurations. Note that the repeating nature of the geometry means that it is sufficient
to analyze a single lamination set and assume that the upper and lower boundaries are
adiabatic.
L = 2 cm L = 2 cm

TH TC

th/2 = 5 μm kp = 0.5 W/m-K


th/2 = 5 μm kCu = 400 W/m-K
Rc′′ = 5x10 m -K/W
-4 2

“on” position

TH TC

“off” position
Figure P2.8-1(b): A single set consisting of half of two adjacent laminations in the “on” and "off”
positions.

The key parameter that characterizes a thermal switch is the resistance ratio (RR) which is
defined as the ratio of the resistance of the switch in the “off” position to its resistance in
the “on” position. The company claims that they can achieve a resistance ratio of more
than 100 for this switch.
a) Estimate upper and lower bounds for the resistance ratio for the proposed thermal
switch using 1-D conduction network approximations. Be sure to draw and clearly
label the resistance networks that are used to provide the estimates. Use your results
to assess the company’s claim of a resistance ratio of 100.

The inputs are entered in EES:

$UnitSystem SI MASS RAD PA K J


$Tabstops 0.2 0.4 0.6 3.5 in

"Inputs"
th = 10 [micron]*convert(micron,m) "thickness of laminations"
k_Cu=400 [W/m-K] "conductivity of copper laminations"
k_p=0.5 [W/m-K] "conductivity of plastic laminations"
L = 2.0 [cm]*convert(cm,m) "lengt of laminations"
R``_c=5e-4 [K-m^2/W] "area specific contact resistance of interface"
W=1 [m] "unit depth into page"

The isothermal and adiabatic models provide two limiting cases. Both result in a 1-D
problem that can be represented using a resistance network. The “adiabatic”
approximation does not allow heat transfer in the y-direction (i.e., perpendicular to the
laminations). The resistance network for the adiabatic approximation is shown in Figure
3(a) for the “on” position and in Figure 3(b) for the “off” position.
(a) (b)
Figure 3: Resistance network for the “adiabatic” approximation in the (a) “on” and (b) “off”
positions.

Note that the parameter Alam in Figure 3 is the cross-sectional area associated a single
lamination:

Alam = W th

where W is the depth into the page (assumed to be 1 m). The resistance associated with
the adiabatic approximation in the “on” state, Rad,on from Figure 3(a) is:

−1
⎡ ⎤

2 ⎢ 1 1 ⎥
Rad ,on = + ⎥
Alam ⎢ 2 L + R′′ 2 L + R′′ ⎥
⎢k c
kCu
c ⎥
⎣ p ⎦

A_lam=W*th "area of a lamination"


R_ad_on=(2/A_lam)*(1/(2*L/k_p+R``_c)+1/(2*L/k_Cu+R``_c))^(-1) "ad. limit in the on position"

which leads to Rad,on = 119.1 K/W. The resistance associated with the adiabatic
approximation in the “off” state, Rad,off from Figure 3(b) is:

−1
⎡ ⎤ L L
⎢ ⎥ + + Rc′′
2 ⎢ 1 1 k p kCu
Rad ,off = + ⎥ =
Alam ⎢ L + L + R′′ L + L + R′′ ⎥ Alam
⎢k c c ⎥
⎣ p kCu kCu k p ⎦

R_ad_off=(L/k_p+L/k_Cu+R``_c)/A_lam "ad. limit in the off position"

The resistance ratio for the adiabatic limit is:

Rad ,off
RRad =
Rad ,on
RR_ad=R_ad_off/R_ad_on "resistance ratio estimated using adiabatic limit"

which leads to RRad = 34.0.

The “isothermal” approximation provides no resistance to heat transfer in the y-direction


(i.e., perpendicular to the laminations). Therefore, the heat can spread without penalty at
the interface and the resistance network for this approximation is shown in Figure 4(a) for
the “on” position and Figure 4(b) “off” position.

(a) (b)
Figure 4: Resistance network for the “isothermal” approximation in the (a) “on” and (b) “off”
positions.

By inspection, the two resistance networks shown in Figures 4(a) and 4(b) will yield the
same resistance and therefore the “isothermal” assumption will predict a resistance ratio,
RRiso, of 1.0. Clearly the company’s claim of a resistance ratio of 100 is not possible as it
does not lie between the two bounding quantities associated with the isothermal and
adiabatic approximations.

b) Provide one or more suggestions for design changes that would improve the
performance of the switch (i.e., increase the resistance ratio). Justify your
suggestions.

The resistance ratio would be increased by any change that causes the two resistance
networks in Figure 3 to be more different. Possible improvements include using
materials with a larger ratio of conductivities (i.e., lower conductivity plastic and, to a
lesser degree, higher conductivity copper) or eliminating the contact resistance. From a
more practical standpoint, any design change that causes the actual the device to behave
more like the adiabatic approximation in Figure 3 than the isothermal approximation in
Figure 4 will improve the performance; for example, increasing the contact resistance
between adjacent laminations would be important.

c.) Sketch the temperature distribution through the two parallel paths associated with the
adiabatic limit of the switch’s operation in the “off” position. Do not worry about the
quantitative details of the sketch, just make sure that the qualitative features are
correct.
Figure 5: Qualitative temperature distribution through paths 1 and 2 consistent with the adiabatic
approximation when the switch is in the “off” position.

d.) Sketch the temperature distribution through the two parallel paths associated with the
adiabatic limit in the “on” position. Again, do not worry about the quantitative details
of your sketch, just make sure that the qualitative features are correct.

Figure 6: Qualitative temperature distribution through paths 1 and 2 consistent with the adiabatic
approximation when the switch is in the “on” position.
Problem 2.8-2 (2-13 in text): Resistance of a Bus Bar
Figure P2.8-2 illustrates a thermal bus bar that has width W = 2 cm (into the page).

H1 = 5 cm L2 = 7 cm

TH = 80°C h = 10 W/m -K
2

T∞ = 20°C

k = 1 W/m-K
L1 = 3 cm H2 = 1 cm
Figure P2.8-2: Thermal bus bar.

The bus bar is made of a material with conductivity k = 1 W/m-K. The middle section is L2 = 7
cm long with thickness H2 = 1 cm. The two ends are each L1 = 3 cm long with thickness H1 = 3
cm. One end of the bar is held at TH = 80ºC and the other is exposed to air at T∞ = 20ºC with h
= 10 W/m2-K.
a.) Use FEHT to predict the rate of heat transfer through the bus bar.

The geometry is entered in FEHT, a mesh is generated, and the boundary conditions and material
properties are specified (Figure 2).

Figure 2: FEHT model.

The mesh is refined several times and then the heat transfer at the convective boundary is
obtained; this leads to q ′ = 5.40 W/m or q = 0.108 W.

b.) Obtain upper and lower bounds for the rate of heat transfer through the bus bar using
appropriately defined resistance approximations.

The inputs are entered in EES:

$UnitSystem SI MASS RAD PA K J


$TABSTOPS 0.2 0.4 0.6 0.8 3.5 in

"Inputs"
L_1=3 [cm]*convert(cm,m) "length of edge pieces"
H_1=5 [cm]*convert(cm,m) "height of edge pieces"
L_2=7 [cm]*convert(cm,m) "length of center piece"
H_2=1 [cm]*convert(cm,m) "height of center piece"
W=2 [cm]*convert(cm,m) "width"
k=1 [W/m-K] "conductivity"
h_bar=10 [W/m^2-K] "heat transfer coefficient at right side"
T_infinity=20 [C] "ambient temperature at right side"
T_H=80 [C] "left side temperature"

An upper bound on the heat transfer rate is obtained using the isothermal limit. The isothermal
limit is calculated according to:

qiso =
(TH − T∞ )
1 L1 L2
+2 +
h H1 W k H1 W k H 2 W






Rconv ,iso Rcond 1,iso Rcond 2,iso

"Isothermal limit"
R_conv_iso=1/(h_bar*W*H_1)
R_cond_1_iso=L_1/(k*W*H_1)
R_cond_2_iso=L_2/(k*W*H_2)
R_total_iso=(R_conv_iso+2*R_cond_1_iso+R_cond_2_iso)
q_dot_iso=(T_H-T_infinity)/R_total_iso

which leads to qiso = 0.118 W.

A lower bound on the heat transfer rate is obtained using the adiabatic limit. The adiabatic limit
is calculated according to:

qad =
(TH − T∞ )
1 L1 L2
+2 +
h H2 W k H2 W k H2 W






Rconv ,ad Rcond 1,ad Rcond 2,ad

"Adiabatic limit"
R_conv_ad=1/(h_bar*W*H_2)
R_cond_1_ad=L_1/(k*W*H_2)
R_cond_2_ad=L_2/(k*W*H_2)
R_total_ad=R_conv_ad+2*R_cond_1_ad+R_cond_2_ad
q_dot_ad=(T_H-T_infinity)/R_total_ad

which leads to qad = 0.052 W.


Problem 2.8-3: Heat Switch

Figure P2.8-3 illustrates a design for a superconducting heat switch.

superconducting strands,
km,normal = 50 W/m-K
km,sc = 0.2 W/m-K
magnet

a = 5 mm TC b = 1 mm

TH
b = 1 mm polymer matrix
kp = 2.5 W/m-K
Figure P2.8-3: Superconducting heat switch.

The heat switch is made by embedding eight square superconducting strands in a polymer
matrix. The width of switch is W = 10 mm (into the page). The size of the strands are a = 5 mm
and the width of polymer that surrounds each strand is b = 1 mm. The conductivity of the
polymer is kp = 2.5 W/m-K. The heat switch is surrounded by a magnet. When the heat switch
is on (i.e., the thermal resistance through the switch in the x-direction is low, allowing heat flow
from TH to TC), the magnet is on. Therefore, the magnetic field tends to drive the
superconductors to their normal state where they have a high thermal conductivity, km,normal = 50
W/m-K. To turn the heat switch off (i.e., to make the thermal resistance through the switch high,
preventing heat transfer from TH to TC), the magnet is deactivated. The superconductors return to
their superconducting state, where they have a low thermal conductivity, km,sc = 0.2 W/m-K. The
edges of the switch are insulated.
a.) Develop a model using a 1-D resistance network that provides a lower bound on the
resistance of the switch when it is in its off state (i.e., km = km,sc).

The inputs are entered in EES:

$UnitSystem SI MASS RAD PA C J


$Tabstops 0.2 0.4 0.6 0.8 3.5

"Inputs"
W=10 [mm]*convert(mm,m) "width of switch"
a=5 [mm]*convert(mm,m) "size of strands"
b=1 [mm]*convert(mm,m) "width of polymer layer"
k_p=2.5 [W/m-K] "polymer conductivity"
k_m_normal=50 [W/m-K] "metal conductivity in its normal state"
k_m_sc=0.2 [W/m-K] "metal conductivity in its superconducting state"

The conductivity of the material is set to its superconducting value:


k_m=k_m_sc "metal conductivity"

The isothermal limit provides a lower bound on the resistance. The isothermal limit is shown in
Figure P2.8-3-2.

Riso,2

Riso,1
TH TC

Riso,3
Figure P2.8-3-2: Isothermal limit resistance network.

The resistance Riso,1 represents the conduction through the polymer layers that stretch across the
width of the switch:

5b
Riso ,1 = (1)
k p W ( 2 a + 3b)

The resistance Riso,2 represents the conduction through the polymer layers that stretch between
adjacent superconducting fibers:

4a
Riso ,2 = (2)
k p W 3b

The resistance Riso,3 represents the conduction through the superconducting fibers:

4a
Riso ,3 = (3)
km W 2 a

"isothermal model"
R_iso_1=5*b/(k_p*W*(2*a+3*b)) "conduction through polymer layers"
R_iso_2=4*a/(k_p*W*3*b) "conduction through polymer between superconducting strands"
R_iso_3=4*a/(k_m*W*2*a) "conduction through superconducting strands"

The resistance predicted in the isothermal limit is:


−1
⎛ 1 1 ⎞
Riso = Riso ,1 + ⎜ + ⎟⎟ (4)
⎜R R
⎝ iso ,2 iso ,3 ⎠

R_iso=R_iso_1+(1/R_iso_2+1/R_iso_3)^(-1) "isothermal limit"

which leads to Riso = 225.9 K/W.


b.) Develop a model using a 1-D resistance network that provides an upper bound on the
resistance of the switch when it is in its off state (i.e., km = km,sc).

The adiabatic limit provides an upper bound on the resistance. The adiabatic limit is shown in
Figure P2.8-3-3.

Rad,2 Rad,3

TH TC

Rad,1
Figure P2.8-3-3: Adiabatic limit resistance network.

The resistance Rad,1 represents the conduction through the polymer layers that stretch across the
entire length of the switch in the x-direction:

Rad ,1 =
(4 a + 5b) (5)
k p W 3b

The resistance Rad,2 represents the conduction through the polymer layers that lie between
adjacent superconducting fibers in the x-direction:

5b
Rad ,2 = (6)
kp W 2 a

The resistance Rad,3 represents the conduction through the superconducting fibers:

4a
Rad ,3 = (7)
km W 2 a

"adiabatic model"
R_ad_1=(4*a+5*b)/(k_p*W*3*b) "conduction through polymer layers in x-direction"
R_ad_2=5*b/(k_p*W*2*a) "conduction through polymer between strands"
R_ad_3=4*a/(k_m*W*2*a) "conduction through superconducting strands"

The resistance predicted in the adiabatic limit is:

−1
⎡ 1 1 ⎤
Rad = ⎢ + ⎥ (8)
⎢⎣ Rad ,1 ( Rad ,2 + Rad ,3 ) ⎥⎦

R_ad=(1/R_ad_1+1/(R_ad_2+R_ad_3))^(-1) "adiabatic limit"

which leads to Rad = 251.2 K/W.


c.) Plot your answers from parts (a) and (b) as a function of km for km,sc < km < km,normal.

Figure P2.8-3-4 illustrates Riso and Rad as a function of km. Notice that Rad is always larger than
Riso and that regardless of which model you use, the resistance of the switch decreases
substantially when the superconducting strands go from their superconducting to their normal
state.
500

adiabatic limit
Thermal resistance (K/W)

100

isothermal limit

10
0.2 1 10 50
Conductivity of strands (W/m-K)
Figure P2.8-3-4: Thermal resistance as a function of km.

d.) The performance of a heat switch is provided by the resistance ratio; the ratio of the
resistance of the switch in its off state to its resistance in the on state. Use your model to
provide an upper and lower bound on the resistance ratio of the switch.

Setting km = km,sc leads to Riso = 225.9 K/W and Rad = 251.2 K/W. Setting km = km,normal leads Riso
= 19.33 K/W and Rad = 22.39 K/W. Therefore, the maximum possible value of the resistance
ratio is 251.2/19.33 = 12.995 and the minimum possible value of the resistance ratio is
225.9/22.39 = 10.089.

e.) Plot the ratio of your answer from part (b) to your answer from part (a) as a function of km for
km,sc < km < km,normal. Explain the shape of your plot.

Figure P2.8-3-5 illustrates Rad/Riso as a function of km.


1.18

Adiabatic to isothermal resistance


1.16

1.14

1.12

1.1

1.08

1.06

1.04

1.02

1
0.2 1 10 70
Strand conductivity (W/m-K)
Figure P2.8-3-5: Thermal resistance as a function of km.

Notice that the adiabatic and isothermal models predict the same value when km = kp (i.e., Rad/Riso
= 1 at km = 2.5 W/m-K in Figure P2.8-3-5). This is because the problem is 1-D in this limit and
therefore both resistance network representations are exactly correct.
Problem 2.9-1: Composite Equivalent Conductivity
A composite material is formed from laminations of high conductivity material (khigh = 100
W/m-K) and low conductivity material (klow = 1 W/m-K) as shown in Figure P2.9-1. Both
laminations have the same thickness, th.
low conductivity laminations
klow = 1 W/m-K

th
th
y

high conductivity laminations


x khigh = 100 W/m-K
Figure P2.9-1: Composite material formed from high and low conductivity laminations.

a.) Do you expect the equivalent conductivity of the composite to be higher in the x or y
directions? Note from Figure P2.9-1 that the x-direction is parallel to the laminations while
the y direction is perpendicular to the laminations.

The heat can flow entirely through high conductivity laminations in the x direction whereas it
must pass through the low conductivity laminations in order to travel in the y direction.
Therefore, the equivalent conductivity of the composite will be much higher in the x direction.

b.) Estimate the equivalent conductivity of the composite in the x direction. You should not
need to any calculations to come up with a good estimate for this quantity.

You are essentially forcing the heat to flow through a block of high conductivity material with
half the area of the composite in order to move in the x direction. Therefore, the equivalent
conductivity of the composite in the x direction will be approximately half the conductivity of
the high conductivity laminations, keff,x ~ 50 W/m-K.
Problem 2.9-2 (2-14 in text)
A laminated stator is shown in Figure P2.9-2. The stator is composed of laminations with
conductivity klam = 10 W/m-K that are coated with a very thin layer of epoxy with conductivity
kepoxy = 2.0 W/m-K in order to prevent eddy current losses. The laminations are thlam = 0.5 mm
thick and the epoxy coating is 0.1 mm thick (the total amount of epoxy separating each
lamination is thepoxy = 0.2 mm). The inner radius of the laminations is rin= 8.0 mm and the outer
radius of the laminations is ro,lam = 20 mm. The laminations are surrounded by a cylinder of
plastic with conductivity kp = 1.5 W/m-K that has an outer radius of ro,p = 25 mm. The motor
casing surrounds the plastic. The motor casing has an outer radius of ro,c = 35 mm and is
composed of aluminum with conductivity kc = 200 W/m-K.

laminations, thlam = 0.5 mm, klam = 10 W/m-K


epoxy coating, thepoxy = 0.2 mm, kepoxy = 2.0 W/m-K
kp = 1.5 W/m-K
kc = 200 W/m-K

q ′′ = 5x10 W/m T∞ = 20°C


4 2

h = 40 W/m -K
2

Rc′′ = 1x10 K-m /W


-4 2

rin = 8 mm
ro,lam = 20 mm
ro,p = 25 mm
ro,c = 35 mm
Figure P2.9-2: Laminated stator.

A heat flux associated with the windage loss associated with the drag on the shaft is q ′′ = 5x104
W/m2 is imposed on the internal surface of the laminations. The outer surface of the motor is
exposed to air at T∞ = 20°C with a heat transfer coefficient h = 40 W/m2-K. There is a contact
resistance Rc′′ = 1x10-4 K-m2/W between the outer surface of the laminations and the inner
surface of the plastic and the outer surface of the plastic and the inner surface of the motor
housing.
a.) Determine an upper and lower bound for the temperature at the inner surface of the
laminations (Tin).

The inputs are entered in EES:

$UnitSystem SI MASS RAD PA C J


$Tabstops 0.2 0.4 0.6 0.8 3.5

klam=10 [W/m-K] "conductivity of laminations"


kepoxy=2 [W/m-K] "conductivity of epoxy"
kp=1.5 [W/m-K] "conductiity of plastic"
kc=200 [W/m-K] "conductivity of casing"
tlam=0.5 [mm]*convert(mm,m) "thickness of lamination"
tepoxy=0.2 [mm]*convert(mm,m) "thickness of epoxy"
rin=8 [mm]*convert(mm,m) "inner radius of laminations"
rolam=20 [mm]*convert(mm,m) "outer radius of laminations"
rop=25 [mm]*convert(mm,m) "outer radius of plastic"
roc=35 [mm]*convert(mm,m) "outer radius of casing"
qflux=5e4 [W/m^2] "heat flux"
Rc=1e-4 [K-m^2/W] "contact resistance"
Tair=converttemp(C,K,20[C]) "air temperature"
h=40 [W/m^2-K] "heat transfer coefficient"

An upper bound on the temperature allows no heat spreading; therefore, the heat must take two
parallel paths that pass through the iron and the epoxy. The total resistance associated with one
lamination/epoxy pair is:

1 1
=
Rtotal ,ad ⎛r ⎞ ⎛ r ⎞ ⎛r ⎞
ln ⎜ o ,lam ⎟ ln ⎜⎜ o , p ⎟⎟ ln ⎜ o ,c ⎟
⎜ ⎟
⎝ rin ⎠ + Rc′′
+ ⎝
ro ,lam ⎠
+
Rc′′
+ ⎝ ro , p ⎠ + 1
2 π klam thlam 2 π ro ,lam thlam 2 π k p thlam 2 π ro , p thlam 2 π kc thlam 2 π ro, p thlam h
1
+
⎛r ⎞ ⎛ r ⎞ ⎛r ⎞
ln ⎜ o ,lam ⎟ ln ⎜⎜ o , p ⎟⎟ ln ⎜ o ,c ⎟
⎜r ⎟
⎝ rin ⎠ + Rc′′ ⎝ ro ,lam ⎠ Rc′′ ⎝ o, p ⎠ + 1
+ + +
2 π kepoxy thepoxy 2 π ro ,lam thepoxy 2 π k p thepoxy 2 π ro , p thepoxy 2 π kc thepoxy 2 π ro, p thepoxy h
(1)

and the upper bound on the air temperature is:

Tupper = T∞ + Rtotal ,ad q ′′ 2 π rin ( thlam + thepoxy ) (2)

"Upper bound on temperature"


Rlam=ln(rolam/rin)/(2*pi*tlam*klam) "resistance of laminations"
Repoxy=ln(rolam/rin)/(2*pi*tepoxy*kepoxy) "resistance of epoxy"
Rci1=Rc/(2*pi*rolam*tlam) "contact resistance"
Rci2=Rc/(2*pi*rolam*tepoxy)
Rp1=ln(rop/rolam)/(2*pi*tlam*kp) "resistance of plastic"
Rp2=ln(rop/rolam)/(2*pi*tepoxy*kp)
Rco1=Rc/(2*pi*rop*tlam) "contact resistance"
Rco2=Rc/(2*pi*rop*tepoxy)
Rcs1=ln(roc/rop)/(2*pi*tlam*kc) "resistance of casing"
Rcs2=ln(roc/rop)/(2*pi*tepoxy*kc)
Rcv1=1/(h*2*pi*roc*tlam) "convection resistance"
Rcv2=1/(h*2*pi*roc*tepoxy)
1/Rtotal1=1/(Rlam+Rci1+Rp1+Rco1+Rcs1+Rcv1)+1/(Repoxy+Rci2+Rp2+Rco2+Rcs2+Rcv2)
"total resistance"
qdot=qflux*pi*rin*(tlam+tepoxy) "heat transfer"
Tin1=Tair+qdot*Rtotal1 "upper bound on temperature"

which leads to Tupper = 502.7 K.


A lower bound on the temperature allows heat spreading; therefore, the heat must take two
parallel paths that pass through the iron and the epoxy but then a single path to the air. The total
resistance associated with one lamination/epoxy pair is:
−1
⎡ ⎤
⎢ ⎥
⎢ ⎥
⎢ 1 ⎥
Rtotal ,iso =⎢ ⎥
⎢ ln ⎛ ro ,lam ⎞ ⎛r ⎞
ln ⎜ o ,lam ⎟ ⎥
⎢ ⎜ ⎟ ⎥
r ⎝ rin ⎠
⎢ ⎝ in ⎠ + ⎥
⎣⎢ 2 π klam thlam 2 π kepoxy thepoxy ⎦⎥
⎛ r ⎞
ln ⎜⎜ o , p ⎟⎟
Rc′′ ⎝ ro ,lam ⎠
+ + + (3)
2 π ro ,lam ( thlam + thepoxy ) 2 π k p ( thlam + thepoxy )
⎛r ⎞
ln ⎜ o ,c ⎟
Rc′′ ⎜r ⎟
+ ⎝ o, p ⎠ +
1
2 π ro , p ( thlam + thepoxy ) 2 π kc ( thlam + thepoxy ) 2 π ro , p ( thlam + thepoxy ) h

and the lower bound on the air temperature is:

Tlower = T∞ + Rtotal ,iso q ′′ 2 π rin ( thlam + thepoxy ) (4)

"Lower bound on temperature"


Rci=Rc/(2*pi*rolam*(tlam+tepoxy)) "contact resistance"
Rp=ln(rop/rolam)/(2*pi*(tlam+tepoxy)*kp) "plastic resistance"
Rco=Rc/(2*pi*rop*(tlam+tepoxy)) "contact resistance"
Rcs=ln(roc/rop)/(2*pi*(tlam+tepoxy)*kc) "casing resistance"
Rcv=1/(h*2*pi*roc*(tlam+tepoxy)) "convection resistance"
Rtotal2=(1/Rlam+1/Repoxy)^(-1)+Rci+Rp+Rco+Rcs+Rcv "total resistance"
Tin2=Tair+qdot*Rtotal2 "lower bound on temperature"

which leads to Tlower = 491.7 K.

b.) You need to reduce the internal surface temperature of the laminations and there are a few
design options available, including: (1) increase the lamination thickness (up to 0.7 mm), (2)
reduce the epoxy thickness (down to 0.05 mm), (3) increase the epoxy conductivity (up to 2.5
W/m-K), or (4) increase the heat transfer coefficient (up to 100 W/m-K). Which of these
options do you suggest and why?

Examination of the Solution Window (Figure 2) shows that the resistance of the lamination is
much less than the resistance of the epoxy; therefore, the resistance of the lamination dominates
the resistance of the epoxy. The resistance due to convection is much larger than the resistance
of the lamination, contact resistance, or conduction through the plastic and casing. Therefore,
the resistance to convection dominates the problem and the most effective mechanism for
reducing the temperature is to increase the heat transfer coefficient.

Figure 2: Solution Window


Problem 2.9-3
Your company manufactures a product that consists of many small metal bars that run through a
polymer matrix, as shown in Figure P2.9-3. The material can be used as a thermal path, allowing
heat to transfer efficiently in the z-direction (the direction that the metal bars run) because the
heat can travel without interruption through the metal bars. However, the material blocks heat
flow in the x- and y-directions because the energy must be conducted through the low
conductivity polymer. Because the scale of the metal bars is small relative to the size of the
composite structure, it is appropriate to model the material as a composite with an effective
conductivity that depends on direction.

w = 0.2 mm

w = 0.2 mm
s = 1.0 mm
L = 10 cm

s = 1.0 mm
km = 100 W/m-K
kp = 2 W/m-K

z
b = 2 cm
x
y
Figure P2.9-3: Composite material.

The metal bars are square with edge width w = 0.2 mm and are aligned with the z-direction. The
bars are arrayed in a regularly spaced matrix with a center-to-center distance of s = 1.0 mm. The
conductivity of the metal is km = 100 W/m-K. The length of the material in the z direction is L =
10 cm. The polymer fills the space between the bars and has a thermal conductivity kp = 2.0
W/m-K. The cross-section of the material in the x-y plane is square with edge width b = 2.0 cm.
a.) Determine the effective conductivity in the x- , y- , and z-directions.

The known information is entered in EES:


$UnitSystem SI MASS RAD PA C J
$Tabstops 0.2 0.4 0.6 0.8 3.5

"Inputs"
w = 0.2 [mm]*convert(mm,m) "size of metal bars"
s = 1.0 [mm]*convert(mm,m) "center-to-center distance between bars"
k_m=100 [W/m-K] "metal conductivity"
L = 10 [cm]*convert(cm,m) "length of composite"
k_p=2.0 [W/m-K] "polymer conductivity"
b=2.0 [cm]*convert(cm,m) "size of composite"

The effective conductivity in the x and y directions will be the same as the energy must be
transferred through the same structure. The effective conductivity is calculated by considering a
unit cell of the structure, shown in Figure 2.
Figure 2: Unit cell of the structure in the x or y direction

The conduction through the unit cell can be approximated using three resistances, shown in
Figure 2. These resistances are:

w
Rm, x = (1)
km L w

R p ,1, x =
( s − w) (2)
kp L w

s
R p ,2, x = (3)
k p L ( s − w)

The total resistance of the composite is:

−1
⎡ 1 1 ⎤
Reff , x =⎢ + ⎥ (4)
⎣⎢ Rm, x + R p ,1, x R p ,2, x ⎦⎥

The resistance must be equal to the resistance of a homogeneous material with the same size that
has the effective thermal conductivity, keff,x:
s
= Reff , x (5)
keff , x L s

These calculations are entered in EES:

"Effective conductivity in the x or y directions"


R_m_x=w/(k_m*L*w)
R_p_1_x=(s-w)/(k_p*L*w)
R_p_2_x=s/(k_p*L*(s-w))
R_eff_x=(1/(R_m_x+R_p_1_x)+1/R_p_2_x)^(-1)
R_eff_x=s/(s*L*k_eff_x)

and indicate that keff,x= keff,y = 2.1 W/m-K (the matrix behaves essentially like a polymer with
respect to conduction in the x or y directions because the energy must transfer, primarily, through
the polymer).

The same type of analysis is accomplished for conduction in the z direction using the same unit
cell shown in Figure 2. Conduction in the z direction occurs through the metal bar and
surrounding polymer in parallel. The resistances of these two paths are:

L
Rm , z = (6)
k m w2

L
Rp, z = (7)
k p ( s − w2 )
2

The total resistance of the composite in the z direction is:

−1
⎡ 1 1 ⎤
Reff , z =⎢ + ⎥ (8)
⎣⎢ Rm, z R p , z ⎦⎥

The resistance must be equal to the resistance of a homogeneous material with the same size that
has the effective thermal conductivity, keff,z:

L
= Reff , z (9)
keff , z s 2

These calculations are entered in EES:

"Effective conductivity in the z direction"


R_m_z=L/(k_m*w^2)
R_p_z=L/(k_p*(s^2-w^2))
R_eff_z=(1/R_m_z+1/R_p_z)^(-1)
R_eff_z=L/(k_eff_z*s^2)
and indicate that keff,z = 5.9 W/m-K (which is approximately equal to the conductivity of the
metal multiplied by the ratio of the metal area to the total area).

b.) The outer edges of the material are insulated and the faces of the material at z = 0 and z = L
are exposed to a convective boundary condition with h = 10 W/m2-K. Is it appropriate to
treat this problem as a lumped capacitance problem?

The Biot number is the ratio of the conductive resistance associated with getting energy from
within the structure to the surface (Rcond) to the convective resistance associated with transferring
the energy from the surface to the surrounding fluid (Rconv). The conduction resistance is
associated with transferring energy from z = L/2 to z = L:

L/2
Rcond , z = (10)
keff , z b 2

The convection resistance is:

1
Rconv , z = (11)
h b2

The Biot number is therefore:

Rcond , z
Bi = (12)
Rconv , z

"Biot number in z-direction"


h=10 [W/m^2-K] "heat transfer coefficient"
R_cond_z=(L/2)/(k_eff_z*b^2)
R_conv_z=1/(h*b^2)
Bi=R_cond_z/R_conv_z

which leads to a Biot number of 0.08; the lumped capacitance assumption is justified for this
problem.
Problem 2.1-1 (2-1 in text): Buried Tubes
Figure P2.1-1 illustrates two tubes that are buried in the ground behind your house that transfer
water to and from a wood burner. The left hand tube carries hot water from the burner back to
your house at Tw,h = 135°F while the right hand tube carries cold water from your house back to
the burner at Tw,c = 70°F. Both tubes have outer diameter Do = 0.75 inch and thickness th =
0.065 inch. The conductivity of the tubing material is kt = 0.22 W/m-K. The heat transfer
coefficient between the water and the tube internal surface (in both tubes) is hw = 250 W/m2-K.
The center to center distance between the tubes is w = 1.25 inch and the length of the tubes is L =
20 ft (into the page). The tubes are buried in soil that has conductivity ks = 0.30 W/m-K.
ks = 0.30 W/m-K

Tw,c = 70°F
th = 0.065 inch kt = 0.22 W/m-K
hw = 250 W/m -K
2

Tw,h = 135°F
hw = 250 W/m -K
2

w = 1.25 inch
Do = 0.75 inch
Figure P2.1-1: Tubes buried in soil.

a.) Estimate the heat transfer from the hot water to the cold water due to their proximity to one
another.

The inputs are entered in EES:

$UnitSystem SI MASS RAD PA K J


$TABSTOPS 0.2 0.4 0.6 0.8 3.5 in

D_o=0.75 [inch]*convert(inch,m) "outer diameter of tube"


th=0.065 [inch]*convert(inch,m) "thickness of tube"
T_hw=converttemp(F,K,135) "hot water temperature"
T_cw=converttemp(F,K,70) "cold water temperature"
L=20 [ft]*convert(ft,m) "length of tubes"
w=1.25 [inch]*convert(inch,m) "center to center distance"
k_t=0.22 [W/m-K] "conductivity of teflon"
k_s=0.30 [W/m-K] "conductivity of sand"
h_w=250 [W/m^2-K]
"heat transfer coefficient between the water and tube inner surfaces"

The heat transfer is resisted by convection within the tube, conduction through the tube, and
conduction in the soil. The convection resistance is calculated according to:

1
Rconv = (1)
hw L π ( Do − 2 th )

The conduction resistance associated with the tube is calculated according to:
⎛ Do ⎞
ln ⎜ ⎟
⎝ Do − 2 th ⎠
Rcond = (2)
2 kt L π

R_conv=1/(h_w*L*pi*(D_o-2*th)) "convection resistance"


R_cond=ln(D_o/(D_o-2*th))/(2*pi*k_t*L) "conduction resistance"

The shape factor for parallel, buried tubes (SF) is obtained using EES’ internal library and used
to compute the resistance due to conduction through the soil:

1
Rtubetotube = (3)
SF ks

SF=SF_4(D_o,D_o,w,L) "shape factor"


R_tubetotube=1/(SF*k_s) "tube to tube resistance"

The heat transfer rate is therefore:

Tw,h − Tw,c
q = (4)
2 Rconv + 2 Rcond + Rtubetotube

q_dot=(T_hw-T_cw)/(2*R_conv+2*R_cond+R_tubetotube) "heat transfer rate"

which leads to q = 137.3 W.

b.) To do part (a) you should have needed to determine a shape factor; calculate an approximate
value of the shape factor and compare it to the accepted value.

The shape factor can be thought of as the ratio of the effective area for conduction, Aeff, to the
effective length required for conduction, Leff. The effective area for conduction can be estimated
according to:

Aeff ≈ w L (5)

while the length for conduction is approximately:

Leff ≈ w − Do (6)

and the approximate value of the shape factor should be:

Aeff
SFapp ≈ (7)
Leff

A_eff=L*w "approximate area"


L_eff=(w-D_o) "approximate length"
SF_app=A_eff/L_eff "approximate shape factor"

The exact value of the shape factor returned by the function is SF = 17.4 m while the
approximate value is SFapp = 15.2 m; these are sufficiently close for a sanity check.

c.) Plot the rate of heat transfer from the hot water to the cold water as a function of the center to
center distance between the tubes.

The heat transfer rate is shown in Figure 2 as a function of the center to center distance between
the tubes.

Figure 2: Heat transfer rate as a function of the center to center distance between the tubes
Problem 2.1-3 (2-2 in text)

A solar electric generation system (SEGS) employs molten salt as both the energy transport and
storage fluid. The molten salt is heated to Tsalt = 500°C and stored in a buried semi-spherical
tank. The top (flat) surface of the tank is at ground level. The diameter of the tank before
insulation is applied Dt = 14 m. The outside surfaces of the tank are insulated with tins = 0.30 m
thick fiberglass having a thermal conductivity of kins = 0.035 W/m-K. Sand having a thermal
conductivity of ksand = 0.27 W/m-K surrounds the tank, except on its top surface. Estimate the
rate of heat loss from this storage unit to the Tair = 25°C surroundings. You may neglect the
resistance due to convection.

The inputs are entered in EES:

$UnitSytem SI K J Pa
$TabStops 0.2 0.4 3.5 in

"known information"
t_ins=0.3 [m] "insulation thickness"
D_t=14 [m] "tank diameter"
k_sand=0.27 [W/m-K] "sand thermal conductivity"
k_ins=0.035 [W/m-K] "insulation thermal conductivity"
T_air=convertTemp(C,K,25 [C]) "air temperature in K"
T_salt=converttemp(C,K,500 [C]) "salt temperature in K"

The resistance between the bottom (buried, hemispherical) surface of the tank and the air is due
to the conduction through the spherical shell of insulation

⎛ 1 1 ⎞
⎜ + ⎟
⎝ Dt / 2 ( Dt / 2 + tins ) ⎠
Rins ,bottom = (1)
2 π kins

and the sand:

1
Rsand = (2)
k sand S

where S is the shape factor, obtained using the EES function SF_22. The resistance from the top,
unburied surface of the tank is only due to conduction through the insulation:

tins
Rins ,top = (3)
D2
kins π t
4

The heat transfer from the bottom and top surfaces are calculated according to:
qbottom =
(Tsalt − Tair ) (4)
Rsoil + Rins ,bottom

and

qtop =
(Tsalt − Tair ) (5)
Rins ,top

The total heat loss is:

qtotal = qtop + qbottom (6)

R_ins_bottom=(1/(D_t/2)-1/(D_t/2+t_ins))/(2*pi*k_ins)
"resistance to conduction through insulation on bottom"
S=SF_22(D_t+2*t_ins) "shape factor"
R_sand=1/(k_sand*S) "resistance of sand"
R_ins_top=t_ins/(k_ins*pi*D_t^2/4)
"resistance to conduction through insulation on top"
q_dot_bottom=(T_salt-T_air)/(R_sand+R_ins_bottom) "heat loss from bottom surface of tank"
q_dot_top=(T_salt-T_air)/(R_ins_top) "heat loss from top surface of tank"
q_dot_total=q_dot_bottom+q_dot_top "total heat loss"

which leads to qtotal = 12.95 kW.


Problem 2.2-3 (2-3 in text): Heat Transfer Coefficient Measurement Device
You are the engineer responsible for a simple device that is used to measure heat transfer
coefficient as a function of position within a tank of liquid (Figure P2.2-3). The heat transfer
coefficient can be correlated against vapor quality, fluid composition, and other useful quantities.
The measurement device is composed of many thin plates of low conductivity material that are
interspersed with large, copper interconnects. Heater bars run along both edges of the thin
plates. The heater bars are insulated and can only transfer energy to the plate; the heater bars are
conductive and can therefore be assumed to come to a uniform temperature as a current is
applied. This uniform temperature is assumed to be applied to the top and bottom edges of the
plates. The copper interconnects are thermally well-connected to the fluid; therefore, the
temperature of the left and right edges of each plate are equal to the fluid temperature. This is
convenient because it isolates the effect of adjacent plates from one another which allows each
plate to measure the local heat transfer coefficient. Both surfaces of the plate are exposed to the
fluid temperature via a heat transfer coefficient. It is possible to infer the heat transfer
coefficient by measuring heat transfer required to elevate the heater bar temperature a specified
temperature above the fluid temperature.

top and bottom surfaces exposed to fluid


T∞ = 20°C, h = 50 W/m -K
2

copper interconnet, T∞ = 20°C

a = 20 mm
b = 15 mm

plate: heater bar, Th = 40°C


k = 20 W/m-K
th = 0.5 mm
Figure P2.2-3: Device to measure heat transfer coefficient as a function of position.

The nominal design of an individual heater plate utilizes metal with k = 20 W/m-K, th = 0.5 mm,
a = 20 mm, and b = 15 mm (note that a and b are defined as the half-width and half-height of the
heater plate, respectively, and th is the thickness as shown in Figure P2-3). The heater bar
temperature is maintained at Th = 40ºC and the fluid temperature is T∞ = 20ºC. The nominal
value of the average heat transfer coefficient is h = 50 W/m2-K.
a.) Develop an analytical model that can predict the temperature distribution in the plate under
these nominal conditions.

The problem can be simplified and tackled using the quarter-symmetry model of a single plate,
shown in Figure P2.2-3, is specified mathematically in Figure 2(a).
(a)

(b)
Figure 2: Quarter symmetry model of a plate.

The lines of symmetry (x =0 and y = 0) are adiabatic, the top edge (y = b) is held at the heater
temperature (Th) and the right edge (x = a) is held at the fluid temperature (T∞). These boundary
conditions are expressed below:

∂T
=0
∂x x =0

Tx =a = T∞
∂T
=0
∂y y =0

Ty =b = Th

The differential control volume and associated first law balance are shown in Figure 2(a):

q x + q y = q x + dx + q y + dy + qconv

or, after expanding the x + dx and y + dy terms:

∂q x ∂q y
0= dx + dy + qconv
∂x ∂y

Substituting the rate equations into the energy balance leads to:

∂ ⎡ ∂T ⎤ ∂ ⎡ ∂T ⎤
0= −k th dy dx + ⎢ − k th dx dy + 2 dx dy h ( T − T∞ )

∂x ⎣ ⎥
∂x ⎦ ∂y ⎣ ∂y ⎥⎦

or

∂ 2T ∂ 2T 2 h
+ −
∂x 2 ∂y 2 k th
T − T∞ = 0 ( ) (1)

Equation (1) is not homogeneous; however, a transformation variable can be identified by


inspection:

θ = T − T∞ (2)

Substituting Eq. (2) into Eq. (1) transforms the governing partial differential equation into a
homogeneous equation; i.e., any multiple of θ will satisfy transformed partial differential
equation:

∂ 2θ ∂ 2θ
+ 2 − m2 θ = 0 (3)
∂x 2
∂y

where

2h
m=
k th
Substituting Eq. (2) into the boundary conditions leads to:

∂θ
=0 (4)
∂x x =0

θ x =a = 0 (5)

∂θ
=0 (6)
∂y y =0

θ y =b = θ h (7)

where

θ h = Th − T∞

The transformed problem is shown in Figure 2(b); note that the transformed problem is linear
and consists of a homogeneous partial differential equation and homogeneous boundary
conditions in the x-direction. Therefore, we can apply a separation of variables solution to the
problem and x is our homogeneous direction.

The solution is assumed to be the product of θX which is a function of x and θY which is a


function of y:

θ ( x, y ) = θ X ( x ) θ Y ( y )

Substituting the product solution into the differential equation leads to:

d 2θ X d 2θ Y
θY + θ X − m2 θ X θY = 0
dx 2
dy 2

Dividing by the product θX θY:

d 2θ X d θ Y
2

dx 2 + dy − m 2 = 0
2

X θ Y


−λ 2 λ2

Recall that x is our homogeneous direction; therefore, we need the θX group in the equation to
equal a negative constant (-λ2); the remaining part of the equation must equal the positive value
of the same constant (λ2). Therefore, the two ordinary differential equations that result from the
separation process are:

d 2θ X
+ λ2 θ X = 0 (8)
dx 2

and

d 2θ Y
− ( λ 2 + m2 )θY = 0 (9)
dy 2

The next step is to solve the eigenproblem; the solution to the ordinary differential equation for
θX is:

θ X = C1 cos ( λ x ) + C2 sin ( λ x ) (10)

The 1st boundary conditions in the homogeneous direction, Eq. (4), leads to:

dθ X
=0 (11)
dx x =0

Substituting Eq. (10) into Eq. (11) leads to:

dθ X
= −C1 λ sin ( λ 0 ) + C2 λ cos ( λ 0 ) = 0
dx x =0



=0 =1

which can only be true if C2 = 0. the 2nd boundary condition in the x-direction, Eq. (5), leads to:

θ x =a = C1 cos ( λ a ) = 0

which can only be true if the argument of the cosine is 0. Therefore, the eigenfunctions and
eigenvalues for the problem are:

θ X i = C1,i cos ( λi x ) where λi =


( 2 i − 1) π
2 a

The solution to the ordinary differential equation in the non-homogeneous direction, Eq. (9) can
be obtained using Maple:

> restart;
> assume((lambda^2+m^2),positive);
> ODEy:=diff(diff(thetaY(y),y),y)-(lambda^2+m^2)*thetaY(y)=0;
⎛d ⎞
2
ODEy := ⎜ 2 thetaY( y ) ⎟⎟ − ( λ∼ 2 + m~ 2 ) thetaY( y ) = 0
⎜ dy
⎝ ⎠
> qYs:=dsolve(ODEy);
( λ∼ 2 + m~2 y ) ( − λ∼2 + m~2 y )
qYs := thetaY( y ) = _C1 e + _C2 e

Note that Maple can convert this exponential form to an equivalent form involving hyperbolic
sines and cosines with the convert command.

> convert(qYs,'trigh');
thetaY( y ) = ( _C1 + _C2 ) cosh( λ∼ 2 + m~ 2 y ) + ( _C1 − _C2 ) sinh( λ∼ 2 + m~ 2 y )

θ Yi = C3,i cosh ( )
λi2 + m 2 y + C4,i sinh ( λi2 + m 2 y )
The solution associated with the ith eigenvalue is therefore:

θi = θ X i θ Yi = cos ( λi x ) ⎡⎢C3,i cosh


⎣ ( )
λi2 + m 2 y + C4,i sinh ( )
λi2 + m 2 y ⎤⎥

(12)

where the constant C1,i is absorbed into the constants C3,i and C4,i. It is good practice to verify
that Eq. (12) satisfies both boundary conditions in the x-direction and the partial differential
equation for any value of i:

> restart;
> assume(i,integer);
> lambda:=(2*i-1)*Pi/(2*a);
( 2 i~ − 1 ) π
λ :=
2a
> theta:=(x,y)->cos(lambda*x)*(C_3*cosh(sqrt(lambda^2+m^2)*y)+C_4*sinh(sqrt(lambda^2+m^2)*y));
θ := ( x, y ) → cos( λ x ) ( C_3 cosh( λ 2 + m 2 y ) + C_4 sinh( λ 2 + m 2 y ) )
> eval(diff(theta(x,y),x),x=0);
0
> theta(a,y);
0
> diff(diff(theta(x,y),x),x)+diff(diff(theta(x,y),y),y)-m^2*theta(x,y);
( 2 i~ − 1 ) π x ⎞
− cos⎛⎜⎜
1
⎟⎟ ( 2 i~ − 1 ) π
2 2
4 ⎝ 2a ⎠
⎛ ⎛ ( 2 i~ − 1 ) 2 π 2 ⎞ ⎛ ( 2 i~ − 1 ) 2 π 2 ⎞⎞
⎜ ⎜ + 4 m 2 y ⎟⎟ ⎜ + 4 m 2 y ⎟⎟ ⎟⎟
⎜ ⎜ ⎜
⎜ ⎜ a 2
⎟ ⎜ a 2
⎟⎟
⎜⎜ C_3 cosh⎜⎜ ⎟⎟ + C_4 sinh⎜⎜ ⎟⎟ ⎟⎟
⎝ ⎝ 2 ⎠ ⎝ 2 ⎠⎠


− π ⎜
a 2 + cos⎛⎜⎜
( 2 i~ 1 ) x ⎞ ⎜⎜
⎟⎟ ⎜
⎝ 2a ⎠⎝
⎛ ( 2 i~ − 1 ) π 2 2

⎜ + 4 m 2 y ⎟⎟

1 ⎜ a 2
⎟ ⎛ ( 2 i~ − 1 ) 2 π 2 ⎞
C_3 cosh⎜ ⎜ ⎟⎟ ⎜ + 4 m 2 ⎟⎟
4 ⎝ 2 ⎠⎝⎜ a 2

⎛ ( 2 i~ − 1 ) π
2 2
⎞ ⎞
⎜ + m 2
y ⎟ ⎟
⎜ 4 ⎟ ⎟
1 ⎜ a2 ⎟ ⎛ ( 2 i~ − 1 ) 2 π 2 ⎞ ⎟
+ C_4 sinh⎜⎜ ⎟⎟ ⎜
⎜ + 4 m ⎟⎟ ⎟⎟ − m 2
2
⎝ ⎠⎝ a
4 2 2
⎠⎠
i~ − π x
cos⎛⎜⎜ ⎞
( 2 1 )
⎟⎟
⎝ 2 a ⎠
⎛ ⎛ ( 2 i~ − 1 )2 π2 ⎞ ⎛ ( 2 i~ − 1 ) 2 π 2 ⎞⎞
⎜ ⎜ + m 2
y ⎟ ⎜ + 4 m 2 y ⎟⎟ ⎟⎟
⎜ ⎜ 4 ⎟ ⎜
⎜ ⎜ a 2
⎟ ⎜ a 2
⎟⎟
⎜⎜ C_3 cosh⎜⎜ ⎟⎟ + C_4 sinh⎜⎜ ⎟⎟ ⎟⎟
⎝ ⎝ 2 ⎠ ⎝ 2 ⎠⎠
> simplify(%);
0

The general solution is the sum of the solution for each eigenvalue:

( ) ( )
∞ ∞
θ = ∑θi = ∑ cos ( λi x ) ⎡⎢C3,i cosh λi2 + m 2 y + C4,i sinh λi2 + m 2 y ⎤⎥ (13)
i =1 i =1 ⎣ ⎦

The general solution must satisfy the boundary conditions in the non-homogeneous direction.
Substituting Eq. (13) into Eq. (6) leads to:

⎡ ⎤
∂θ
( ) ( )

= ∑ cos ( λi x ) C3,i λi + m sinh λi + m 0 + C4,i λi + m cosh λi + m 0 ⎥ = 0
⎢ 2 2 2 2 2 2 2 2

∂y i =1
⎢ 


y =0
⎣⎢ =0 =1 ⎦⎥

or

∑C
i =1
4,i λi2 + m 2 cos ( λi x ) = 0

which can only be true if C4,i = 0:

( )

θ = ∑ Ci cosh λi2 + m 2 y cos ( λi x ) (14)
i =1
where the subscript 3 has been removed from C3,i because it is the only remaining undetermined
constant. Substituting Eq. (14) into Eq. (7) leads to:

( )

θ y =b = ∑ Ci cosh λi2 + m 2 b cos ( λi x ) = θ h
i =1

This equation is multiplied by cos(λj x) and integrated from 0 to a:

( )
∞ a a

∑ Ci cosh λi2 + m 2 b ∫ cos ( λi x ) cos ( λ j x ) dx = θ h ∫ cos ( λ j x ) dx


i =1 0 0

Orthogonality causes all of the terms in the summation to integrate to zero except for the one in
which i = j:

( ) ∫ cos (λ x ) dx = θ ∫ cos (λ x ) dx
a a
Ci cosh λ +m b
i
2 2 2
i h i
0 0

Maple is used to carry out the integrations:

> restart;
> assume(i,integer);
> lambda:=(2*i-1)*Pi/(2*a);
( 2 i~ − 1 ) π
λ :=
2a
> int((cos(lambda*x))^2,x=0..a);
a
2
> int(cos(lambda*x),x=0..a);
( 1 + i~ )
2 ( -1 ) a
( 2 i~ − 1 ) π

which leads to an equation for each of the constants.

θ h 4 ( −1)
1+ i

Ci =
cosh ( )
λi2 + m 2 b ( 2 i − 1) π

The inputs are entered in EES:

$UnitSystem SI MASS RAD PA C J


$Tabstops 0.2 0.4 0.6 0.8 3.5

"Inputs"
th_mm=0.5 [mm] "thickness of plate in mm"
th=th_mm * convert(mm,m) "thickness of plate"
k=20 [W/m-K] "conductivity of plate"
a_mm=20 [mm] "half-width of plate in mm"
a= a_mm * convert(mm,m) "half-width of plate"
b= 15 [mm] *convert(mm,m) "half-height of plate"
T_h = converttemp(C,K,40 [C]) "heater temperature"
T_infinity=converttemp(C,K,20 [C]) "fluid temperature"
h_bar=50 [W/m^2-K] "heat transfer coefficient"

The position is specified using dimensionless variables x_bar and y_bar:

"position"
x_bar=0.5
y_bar=0.5
x=x_bar*a
y=y_bar*b

The solution is evaluated:

theta_h=T_h-T_infinity "heater temperature difference"


m=sqrt(2*h_bar/(k*th)) "solution constant"
N=100 "number of terms"
duplicate i=1,N
lambda[i]=(2*i-1)*pi/(2*a) "eigenvalues"
C[i]=theta_h*(4*(-1)^(1+i)/(2*i-1)/Pi)/(cosh(sqrt(lambda[i]^2+m^2)*b))
theta[i]=C[i]*cos(lambda[i]*x)*cosh(sqrt(lambda[i]^2+m^2)*y)
end
T=sum(theta[1..N])+T_infinity
T_C=converttemp(K,C,T)

A parametric table is created and used to generate the contour plot shown in Figure 3.
Figure 3: Contour plot of temperature on the plate.

b.) The measured quantity is the rate of heat transfer to the plate from the heater ( qh ) and
therefore the relationship between qh and h (the quantity that is inferred from the heater
power) determines how useful the instrument is. Determine the heater power.

The heater power can be computed by integrating the conduction heat transfer along the top
surface according to:

∂θ
a
qh = 4 ∫ k th dx (15)
0
∂y y =b

where the factor of 4 comes from the quarter symmetry of the model. Equation (14) is
substituted into Eq. (15):

( ) ∫ cos (λ x ) dx
∞ a
qh = 4 k th ∑ Ci λi2 + m 2 sinh λi2 + m 2 b i
i =1 0

which leads to:

( )

qh = 2 k th a ∑ Ci λi2 + m 2 sinh λi2 + m 2 b (16)
i =1

Equation (16) is evaluated in EES:

"heater power"
duplicate i=1,N
q_dot_h[i]=2*k*th*a*C[i]*sqrt(lambda[i]^2+m^2)*sinh(sqrt(lambda[i]^2+m^2)*b)
end
q_dot_h=sum(q_dot_h[1..N])

c.) If the uncertainty in the measurement of the heater power is δ qh = 0.01 W, estimate the
uncertainty in the measured heat transfer coefficient ( δ h ).

A parametric table is used to explore the relationship between h and qhtr ; the two columns in the
table are the variables h_bar and q_dot_h. Figure 4 illustrates the heat transfer coefficient as a
function of the heater power, all else held constant.

Figure 4: Heat transfer coefficient as a function of the heater power.

Note that a good measuring device would show a strong relationship between the physical
quantity being measured ( h ) and the measurement ( qh ); this would be indicated by a large
partial derivative of heat transfer coefficient with respect to heater power. Given the uncertainty
in the heater power measurement, δ qh (related to, for example, the resolution of the data
acquisition system, noise, etc.), it is possible to estimate the uncertainty in the measurement of
the heat transfer coefficient, δ h , according to:

∂h
δh = δ qh
∂qh

For example, if the uncertainty in the heater power is 0.01 W then Fig. 3 suggests that the partial
derivative of heat transfer coefficient with respect to heater power is approximately 125 W/m2-
K-W and the uncertainty in the heat transfer coefficient measurement would be δ h = 1.8 W/m2-
K.

As an engineer designing this measurement device, you would like to calculate not the heater
power but rather the partial derivative in the heat transfer coefficient with respect to heater power
in order to automate the process of computing δ h and therefore evaluate how design changes
affect the instrument. This would be difficult to accomplish analytically; note that both m and
the constants Ci in Eq. (16) are functions of h . It is more convenient to determine the partial
derivative numerically. That is, set the heat transfer coefficient at its nominal value plus a small
amount ( h + Δh ) and evaluate the heater power ( qh , h +Δh ); note that Δh should be small relative
to the nominal value of the heat transfer coefficient. Then set the heat transfer coefficient at its
nominal value less a small amount ( h − Δh ) and evaluate the heater power ( qh , h −Δh ). The partial
derivative is approximately:

∂h 2 Δh

∂qh qh ,h +Δh − qh ,h −Δh

Since qh needs to be evaluated at several values of the heat transfer coefficient, it is convenient
to have the computation of the heater power occur within a function that is called twice within
the equation window. Because the solution is in the form of an EES code, this is an ideal
problem to use a MODULE. A MODULE is a stand-alone EES program that can be called from
the main EES equation window. The MODULE is provided with inputs and it calculates
outputs. The protocol of a call to a MODULE involves the name of the MODULE followed by a
series of inputs separated by a colon from a series of outputs. For example, we will create a
MODULE Heaterpower that calculates the value of the variable q_dot_h. It is important to note
that all of the variables from the main equation window that are required by the MODULE must
be passed to the MODULE; the MODULE can only access variables that are passed to it as
parameters or using the $COMMON directive.

The MODULE Heaterpower must be placed at the top of the EES code and is a small self-
contained EES program; we create the MODULE by copying those lines of the main EES code
that are required to calculate the variable q_dot_h.

MODULE Heaterpower(th,k,a,b,T_h,T_infinity,h_bar:q_dot_h)
theta_h=T_h-T_infinity "heater temperature difference"
m=sqrt(2*h_bar/(k*th)) "solution constant"
N=100 "number of terms"
duplicate i=1,N
lambda[i]=(2*i-1)*pi/(2*a) "eigenvalues"
C[i]=theta_h*(4*(-1)^(1+i)/(2*i-1)/Pi)/(cosh(sqrt(lambda[i]^2+m^2)*b))
q_dot_h[i]=2*k*th*a*C[i]*sqrt(lambda[i]^2+m^2)*sinh(sqrt(lambda[i]^2+m^2)*b)
end
q_dot_h=sum(q_dot_h[1..N])
end
The calling protocol for the MODULE consists of a series of inputs (the variables d, k, a, b,
T_htr, T_f, and h) that are separated by a series outputs (in this case only the variable q_dot_htr)
by a colon. MODULES are most useful where a certain sequence of code must be executed
multiple times; in this case, the MODULE Heaterpower enables the partial derivative to be easily
computed.

delta_q_dot_htr=0.01 [W] "heater power resolution"


dh=1 [W/m^2-K] "perturbation of heat transfer coefficient"

CALL Q_dot_heater(d,k,a,b,T_htr,T_f,h-dh:q_dot_htr_minus)
CALL Q_dot_heater(d,k,a,b,T_htr,T_f,h+dh:q_dot_htr_plus)
dhdqdot=2*dh/(q_dot_htr_plus-q_dot_htr_minus)
delta_h=dhdqdot*delta_q_dot_htr

The modifications to the EES code verifies that the uncertainty in heat transfer coefficient,
delta_h, is 1.26 W/m2-K and provides a convenient tool for assessing the impact of the various
design parameters on the performance of the measurement system. For example, Figure 5
illustrates the uncertainty in the heat transfer coefficient as a function of the plate thickness (th)
for various values of its half-width (a).

Figure 5: Uncertainty in the measured heat transfer coefficient as a function of the plate thickness
for various values of the plate half-width.
Problem 2.2-6 (2-4 in text)

A laminated composite structure is shown in Figure P2.2-6.

q ′′ = 10000 W/m
2
H = 3 cm

Tset = 20°C
W = 6 cm
Tset = 20°C
kx = 50 W/m-K
ky = 4 W/m-K
Figure P2.2-6: Composite structure exposed to a heat flux.

The structure is anisotropic. The effective conductivity of the composite in the x-direction is kx =
50 W/m-K and in the y-direction it is ky = 4 W/m-K. The top of the structure is exposed to a heat
flux of q ′′ = 10,000 W/m2. The other edges are maintained at Tset = 20°C. The height of the
structure is H = 3 cm and the half-width is W = 6 cm.
a.) Develop a separation of variables solution for the 2-D steady-state temperature distribution in
the composite.

The known information is entered in EES:

$UnitSystem SI MASS RAD PA K J


$TABSTOPS 0.2 0.4 0.6 0.8 3.5 in

"Inputs"
W=6 [cm]*convert(cm,m) "width of laminate"
H=3 [cm]*convert(cm,m) "height of laminate"
q``=10000 [W/m^2] "heat flux"
k_x=50 [W/m-K] "conductivity in the x-direction"
k_y=4 [W/m-K] "conductivity in the y-direction"
T_set=converttemp(C,K,20[C]) "specified edge temperatures"

A half-symmetry model of the problem shown in Figure P2.2-6 is governed by the transformed
partial differential equation:

∂ 2θ ∂ 2θ
kx + k =0 (1)
∂x 2 ∂y 2
y

with the boundary conditions:

∂θ
=0 (2)
∂x x =0

θ x =W = 0 (3)
∂θ
ky = q ′′ (4)
∂y y=H

θ y =0 = 0 (5)

where

θ = T − Tset (6)

The separated solution is assumed:

θ = θ X θY (7)

and substituted into Eq. (1):

d 2θ X d 2θ Y
kx θY + y
k θ X =0 (8)
dx 2 dx 2

Dividing through by θY θX leads to:

d 2θ X d 2θ Y
dx 2 + k y dx 2 = 0 (9)
θX

k θY
x

−λ2 λ2

The process leads to two ordinary differential equations:

d 2θ X
2
+ λ2 θ X = 0 (10)
dx

d 2θ Y
− β 2 λ 2 θY = 0 (11)
dy 2

where

kx
β2 = (12)
ky

beta=sqrt(k_x/k_y)

Notice that the sign of the constant is selected so that the solution in the homogeneous direction
(x) is sines and cosines. The solution to Eq. (10) is:
θ X = C1 sin ( λ x ) + C2 cos ( λ x ) (13)

The boundary condition at x = 0, Eq. (2) requires that C1 = 0 and therefore:

θ X = C2 cos ( λ x ) (14)

The boundary condition at x = L, Eq. (3), leads to the eigencondition for the problem:

cos ( λi W ) = 0 (15)

which requires that:

λi =
(1 + 2 i ) π for i = 0,1,...∞ (16)
2W

N_term=11 [-] "number of terms"


duplicate i=0,N_term
lambda[i]=(1+2*i)*pi/(2*W) "eigenvalues"
end

The eigenfunctions are:

θ X i = C2,i cos ( λi x ) (17)

The solution to Eq. (11) for each eigenvalue is:

θ Yi = C3,i sinh ( β λi y ) + C4,i cosh ( β λi y ) (18)

The general solution for for each eigenvalue is:

θi = θ X i θ Yi = C2,i cos ( λi x ) ⎡⎣C3,i sinh ( β λi y ) + C4,i cosh ( β λi y ) ⎤⎦ (19)

The sum of these solutions is, itself, a solution (note that the constant C2,i is absorbed into the
other constants):

∞ ∞
θ = ∑ θi = ∑ cos ( λi x ) ⎡⎣C3,i sinh ( β λi y ) + C4,i cosh ( β λi y ) ⎤⎦ (20)
i =1 i =1

The boundary condition at y = 0, Eq. (5), leads to C4,i = 0:

∞ ∞
θ = ∑ θi = ∑ Ci cos ( λi x ) sinh ( β λi y ) (21)
i =1 i =1
Equation (21) is substituted into the boundary condition at y = H, Eq. (4):


β λi k y ∑ Ci cos ( λi x ) cosh ( β λi H ) = q ′′ (22)
i =1

Equation (22) is multiplied by an eigenfunction and integrated from x = 0 to x = W:


L L
β λi k y cosh ( β λi H ) Ci ∫ cos 2 ( λi x ) dx = q ′′∫ cos ( λi x ) dx (23)
0 0

The integrals are carried out in Maple:

> restart;
> assume(i,integer);
> lambda:=(1+2*i)*Pi/(2*W);
( 1 + 2 i~ ) π
λ :=
2W
> int((cos(lambda*x))^2,x=0..W);
W
2
> int(cos(lambda*x),x=0..W);
2 ( -1 )i~ W
( 1 + 2 i~ ) π

and used to complete the computation of the constants in EES:

duplicate i=0,N_term
k_y*C[i]*beta*lambda[i]*cosh(beta*lambda[i]*H)*W/2=q``*2*(-1)^i*W/((1+2*i)*Pi)
end

The solution is evaluated at an arbitary position:

x_bar=0.5 [-] "dimensionless x-position"


y_bar=0.5 [-] "dimensionless y-position"
x=x_bar*W "x-position"
y=y_bar*H "y-position"
duplicate i=0,N_term
theta[i]=C[i]*cos(lambda[i]*x)*sinh(beta*lambda[i]*y)
end
theta=sum(theta[0..N_term])
T=theta+T_set
T_C=converttemp(K,C,T)

b.) Prepare a contour plot of the temperature distribution.


Figure 2 illlustrates a contour plot of the temperature distribution.

1
20
23.12
Dimensionless y position, y/H 0.8
26.24
29.36
0.6 32.48
35.6
38.72
0.4
41.84
44.96
0.2 48.08
51.2

0
0 0.2 0.4 0.6 0.8 1

Dimensionless x position, x/W


Figure 2: Contour plot of the temperature distribution.
Problem 2.3-1 (2-5 in text): Cryogen Transfer Pipe
Figure P2.3-1 illustrates a pipe that connects two tanks of liquid oxygen on a spacecraft. The
pipe is subjected to a heat flux, q′′ = 8,000 W/m2, which can be assumed to be uniformly applied
to the outer surface of the pipe and entirely absorbed. Neglect radiation from the surface of the
pipe to space. The inner radius of the pipe is rin = 6 cm, the outer radius of the pipe is rout = 10
cm, and the half-length of the pipe is L = 10 cm. The ends of the pipe are attached to the liquid
oxygen tanks and therefore are at a uniform temperature of TLOx = 125 K. The pipe is made of a
material with a conductivity of k = 10 W/m-K. The pipe is empty and therefore the internal
surface can be assumed to be adiabatic.

rout = 10 cm L = 10 cm
rin = 6 cm TLOx = 125 K

k = 10 W/m-K q′′s = 8,000 W/m


2

Figure P2.3-1: Cryogen transfer pipe connecting two liquid oxygen tanks.

a.) Develop an analytical model that can predict the temperature distribution within the pipe.
Prepare a contour plot of the temperature distribution within the pipe.

A differential control volume leads to the energy balance:

qx + qr = qx + dx + qr + dr

or

∂qx ∂q
0= dx + r dr
∂x ∂r

Substituting the rate equations:

∂T
qx = − k 2 π r dr
∂x

and

∂T
qr = − k 2 π r dx
∂r

into the differential energy balance leads to:


∂ ⎡ ∂T ⎤ ∂ ⎡ ∂T ⎤
0= ⎢ − k 2 π r dr ⎥ dx + ⎢ −k 2 π r dx ⎥ dr
∂x ⎣ ∂x ⎦ ∂r ⎣ ∂r ⎦

or

∂ 2T ∂ ⎡ ∂T ⎤
r + r =0
∂x 2 ∂r ⎢⎣ ∂r ⎥⎦

A half-symmetry model of the pipe will be generated; the boundary conditions are therefore:

∂T
=0
∂x x =0

Tx = L = TLOx

∂T
=0
∂r r = rin

∂T
k = q′′
∂r r = rout

As stated, there are two non-homogeneous boundary conditions; however, the boundary
condition at x = L can be made homogeneous by defining the temperature difference:

θ = T − TLOx

The partial differential equation and boundary conditions are written in terms of θ:

∂ 2θ ∂ ⎡ ∂θ ⎤
r + r =0 (1)
∂x 2 ∂r ⎢⎣ ∂r ⎥⎦

∂θ
=0 (2)
∂x x =0

θ x= L = 0 (3)

∂θ
=0 (4)
∂r r = rin
∂θ
k = q′′ (5)
∂r r = rout

Note that the two homogeneous boundary conditions are in the x-direction and so the
eigenfunctions of the problem will be in this direction. We assume that the solution is separable;
that is, the solution is the product of a function only of x (θX) and r (θR):

θ ( x, y ) = θ X ( x ) θ R ( r )

Substituting the product solution into the governing partial differential equation, Eq. (1), leads to:

d 2θ X d ⎡ dθ R ⎤
rθ R +θ X r =0
dx 2
dr ⎢⎣ dr ⎥⎦

Dividing by the product r θX θR leads to:

d 2θ X d ⎡ dθ R ⎤
⎢r ⎥
dx + dr ⎣ dr ⎦ = 0
2

θX rθ R
±λ2 ∓λ2

Note that the 1st term is a function only of x while the 2nd term is a function only of r; these two
quantities must be equal and opposite constants (±λ2). The choice of the sign is again important;
the eigenfunctions must be in x and therefore the two ordinary differential equations must be:

d 2θ X
+ λ2 θ X = 0 (6)
dx 2

d ⎡ dθ R ⎤
⎢ r ⎥ − λ2 rθ R = 0 (7)
dr ⎣ dr ⎦

The eigenproblem will be solved first; the solution to Eq. (6) is:

θ X = C1 cos ( λ x ) + C2 sin ( λ x )

The boundary condition at x = 0, Eq. (2), eliminates the sine term. The boundary condition at x =
L, Eq. (3), leads to:

C1 cos ( λ L ) = 0

which provides the eigenvalues and the eigenfunctions:


θ X i = C1,i cos ( λi x ) where λi =
( 2 i − 1) π for i = 1, 2,..∞
2L

The ordinary differential equation for θR, Eq. (7), is solved by Bessel functions, as discussed in
Section 1.8. Equation (7) is a form of Bessel's equation:

d ⎛ p dθ ⎞ 2 s
⎜x ⎟±c x θ =0
dx ⎝ dx ⎠

where s = 1 and p = 1; the quantity s – p +2 is therefore not equal to zero and we are directed
toward the left-side of the Bessel function flow chart presented in Section 1.8.4 where the
parameters n = 0, a = 1, and n/a = 0 are computed. The last term is negative and therefore the
general solution to Eq. (7) is:

θ Ri = C3,i BesselI ( 0,λi r ) + C4,i BesselK ( 0,λi r )

This general solution could have been obtained using Maple:

> restart;
> ODEr:=diff(r*diff(thetar(r),r),r)-lambda^2*r*thetar(r)=0;
⎛d ⎞
2
⎛ d ⎞
ODEr := ⎜⎜ thetar( r ) ⎟⎟ + r ⎜ 2 thetar( r ) ⎟⎟ − λ 2 r thetar( r ) = 0

⎝ dr ⎠ ⎝ dr ⎠
> thetars:=dsolve(ODEr);
thetars := thetar( r ) = _C1 BesselI( 0, λ r ) + _C2 BesselK( 0, λ r )

The general solution for each eigenvalue is therefore:

θi = θ X i θ Ri = cos ( λi x ) ⎡⎣C3,i BesselI ( 0,λi r ) + C4,i BesselK ( 0,λi r ) ⎤⎦

The general solution is entered as a function of x and y in Maple:

> restart;
> assume(i,integer);
> lambda:=(2*i-1)*Pi/(2*L);
( 2 i~ − 1 ) π
λ :=
2L
> theta:=(x,r)->cos(lambda*x)*(C3*BesselI(0,lambda*r)+C4*BesselK(0,lambda*r));
θ := ( x, r ) → cos( λ x ) ( C3 BesselI( 0, λ r ) + C4 BesselK( 0, λ r ) )

Verify that it solves both boundary conditions in the x-directions, Eqs. (2) and (3):

> eval(diff(theta(x,r),x),x=0);
0
> theta(L,r);
0

and the partial differential equation, Eq. (1):

> r*diff(diff(theta(x,r),x),x)+diff(r*diff(theta(x,r),r),r);
( 2 i~ − 1 ) π x ⎞
− r cos⎛⎜⎜
1
⎟⎟ ( 2 i~ − 1 ) π
2 2
4 ⎝ 2 L ⎠
⎛ ⎛ ( 2 i~ − 1 ) π r ⎞ + C4 BesselK⎛ 0, ( 2 i~ − 1 ) π r ⎞ ⎞ L 2 +
⎜⎜ C3 BesselI⎜⎜ 0, ⎟⎟ ⎜⎜ ⎟⎟ ⎟⎟
⎝ ⎝ 2L ⎠ ⎝ 2L ⎠⎠
⎛ ⎛ ( 2 i~ − 1 ) π r ⎞ ( 2 i~ − 1 ) π
⎜ C3 BesselI⎜⎜ 1, ⎟⎟
⎛ ( 2 i~ − 1 ) π x ⎞ ⎜ 1 ⎝ 2 L ⎠
cos⎜⎜ ⎟⎟ ⎜⎜
⎝ 2 L ⎠⎝ 2 L


⎛ ( 2 i~ − 1 ) π r ⎞ ⎞ ⎜
C4 BesselK⎜⎜ 1, ⎟⎟ ( 2 i~ − 1 ) π ⎟ ⎜
⎟ − π ⎜

1 ⎝ 2L ⎠ ⎟ + r cos⎜⎛ ( 2 i~ 1 ) x ⎞ ⎜
⎟ ⎜ ⎟ ⎜⎜

2 L ⎠ ⎝ 2 L ⎠⎝
⎛ ( 2 i~ − 1 ) π r ⎞ ⎞
⎜ 2 L BesselI⎛⎜⎜ 1, ⎟⎟ ⎟
⎜ ⎛ ( 2 i~ − 1 ) π r ⎞ ⎝ 2L ⎠ ⎟⎟ ( 2 i~ − 1 ) 2 π 2

C3 ⎜ BesselI⎜⎜ 0, ⎟⎟ −
( 2 i~ − 1 ) π r ⎟
1 ⎝ ⎝ 2L ⎠ ⎠ −
4 2
L
⎛ ( 2 i~ − 1 ) π r ⎞ ⎞
⎜ 2 L BesselK⎛⎜⎜ 1, ⎟⎟ ⎟


⎜ ( 2 i~ − 1 ) π r ⎞ ⎠ ⎟⎟ ( 2 i~ − 1 ) 2 π 2 ⎟
C4 ⎜⎜ −BesselK⎛⎜⎜ 0, ⎝ 2L ⎟
⎟⎟ − ⎟ ⎟
1 ⎝ ⎝ 2L ⎠ ( 2 i~ − 1 ) π r ⎠ ⎟
2 ⎟⎟
4 L ⎠
> simplify(%);
0

The general solution for θ is the series:

∞ ∞
θ = ∑ θi = ∑ cos ( λi x ) ⎡⎣C3,i BesselI ( 0,λi r ) + C4,i BesselK ( 0,λi r ) ⎤⎦ (8)
i =1 i =1

The solution must satisfy the boundary conditions in the non-homogeneous direction; the
boundary condition at r = rin, Eq. (4), leads to:

∂θ ∞
d
= ∑ cos ( λi x ) ⎡C3,i BesselI ( 0,λi r ) + C4,i BesselK ( 0,λi r ) ⎤⎦ r = r = 0 (9)
∂r r = rin i =1 dr ⎣ in

The derivatives of the Bessel functions may either be evaluated using the equations provided in
Section 1.8.4 or, more conveniently, using Maple:
> restart;
> diff(BesselI(0,lambda*r),r);
BesselI( 1, λ r ) λ
> diff(BesselK(0,lambda*r),r);
−BesselK( 1, λ r ) λ

Therefore Eq. (9) can be written as:

∑ cos ( λ x ) ⎡⎣C
i =1
i 3,i λi BesselI (1,λi rin ) − C4,i λi BesselK (1,λi rin ) ⎤⎦ = 0

which can only be true if:

C3,i λi BesselI (1,λi rin ) − C4,i λi BesselK (1,λi rin ) = 0

Unlike most of the problems we have previously encountered, neither of the constants is
eliminated; instead we see that there is a relationship between the two constants:

BesselI (1,λi rin )


C4,i = C3,i (10)
BesselK (1,λi rin )

Substituting Eq. (10) into Eq. (8) leads to:

∞ ∞ ⎡ BesselI (1,λi rin ) ⎤


θ = ∑ θi = ∑ Ci cos ( λi x ) ⎢ BesselI ( 0,λi r ) + BesselK ( 0,λi r ) ⎥ (11)
i =1 i =1 ⎣ BesselK (1,λi rin ) ⎦

Equation (11) is substituted into the remaining non-homogeneous boundary condition at r = rout ,
Eq. (5), in order to obtain:

∂θ ∞ ⎡ BesselI (1,λi rin ) ⎤


k = k ∑ Ci λi cos ( λi x ) ⎢ BesselI (1,λi rout ) − BesselK (1,λi rout ) ⎥ = q′′
∂r r = rout i =1 ⎣ BesselK (1,λi rin ) ⎦

This equation is multiplied by an arbitrary eigenfunction, cos(λj x), and integrated between the
homogeneous boundary conditions (from x = 0 to x= L); using the orthogonality property of the
eigenfunctions we obtain:

⎡ BesselI (1,λi rin ) ⎤L L


k Ci λi ⎢ BesselI (1,λi rout ) − BesselK (1,λi rout ) ⎥ ∫ cos 2 ( λi x ) dx = q′′∫ cos ( λi x ) dx
⎣ BesselK (1,λi rin ) ⎦0 0
which leads to an expression for each constant:

k Ci λi L ⎡ BesselI (1,λi rin ) ⎤ sin ( λi L )


⎢ BesselI (1,λi rout ) − BesselK (1,λi rout ) ⎥ = q′′
2 ⎣ BesselK (1,λi rin ) ⎦ λi

Solving for Ci:

2 q′′ sin ( λi L )
Ci =
⎡ BesselI (1,λi rin ) ⎤
λi2 k L ⎢ BesselI (1,λi rout ) − BesselK (1,λi rout ) ⎥
⎣ BesselK (1,λi rin ) ⎦

The inputs are entered in EES:

$UnitSystem SI MASS RAD PA K J


$Tabstops 0.2 0.4 0.6 0.8 3.5

"Inputs"
q``_dot=8000 [W/m^2] "Heat flux on pipe surface"
r_in=6.0 [cm]*convert(cm,m) "Pipe inner radius"
r_out=10.0 [cm]*convert(cm,m) "Pipe outer radius"
L = 10.0 [cm]*convert(cm,m) "Pipe half-length"
T_LOx=125 [K] "Pipe end temperature"
k=10 [W/m-K] "Pipe conductivity"

A position is specified in terms of dimensionless coordinates:

"dimensionless position"
r_bar=0.5
x_bar=0.5
r=r_in+(r_out-r_in)*r_bar
x=x_bar*L

The solution is implemented for the 1st N terms of the series:

N=100 "Number of terms"


duplicate i=1,N
lambda[i]=(2*i-1)*pi/(2*L)
C[i]=2*q``_dot*sin(lambda[i]*L)/(lambda[i]^2*k*L*(BesselI(1,lambda[i]*r_out)-&
BesselI(1,lambda[i]*r_in)*BesselK(1,lambda[i]*r_out)/BesselK(1,lambda[i]*r_in)))
theta[i]=C[i]*cos(lambda[i]*x)*(BesselI(0,lambda[i]*r)+BesselI(1,lambda[i]*r_in)*&
BesselK(0,lambda[i]*r)/BesselK(1,lambda[i]*r_in))
end
T=sum(theta[1..N])+T_LOx

The temperature distribution is computed over a range of positions and the results are used to
generate the contour plot shown in Figure 2.
Figure 2: Contour plot of temperature.
Problem 2.3-2 (2-6 in text)
Figure P2.3-2 illustrates a cylinder that is exposed to a concentrated heat flux at one end.

extends to infinity

k = 168 W/m-K

rout = 200 μm

Ts = 20°C

rexp = 21 μm

q ′′ = 1500 W/cm
2 adiabatic
Figure P2.3-2: Cylinder exposed to a concentrated heat flux at one end.

The cylinder extends infinitely in the x-direction. The surface at x = 0 experiences a uniform
heat flux of q′′ = 1500 W/cm2 for r < rexp = 21 μm and is adiabatic for rexp < r < rout where rout =
200 μm is the outer radius of the cylinder. The outer surface of the cylinder is maintained at a
uniform temperature of Ts = 20ºC. The conductivity of the cylinder material is k = 168 W/m-K.
a.) Develop a separation of variables solution for the temperature distribution within the
cylinder. Plot the temperature as a function of radius for various values of x.

The inputs are entered in EES:

$UnitSystem SI MASS RAD PA K J


$Tabstops 0.2 0.4 0.6 0.8 3.5

"Inputs"
r_out=200 [micron]*convert(micron,m) "outer radius of domain"
q``_dot=1500 [W/cm^2]*convert(W/cm^2,W/m^2) "exposure flux"
k=168 [W/m-K] "conductivity of work piece"
r_exp=21 [micron]*convert(micron,m) "radius of exposure zone"

A differential control volume leads to the energy balance:

qx + qr = qx + dx + qr + dr

or

∂qx ∂q
0= dx + r dr
∂x ∂r

Substituting the rate equations:


∂T
qx = − k 2 π r dr
∂x

and

∂T
qr = − k 2 π r dx
∂r

into the differential energy balance leads to:

∂ ⎡ ∂T ⎤ ∂ ⎡ ∂T ⎤
0= ⎢ − k 2 π r dr ⎥ dx + ⎢ −k 2 π r dx ⎥ dr
∂x ⎣ ∂x ⎦ ∂r ⎣ ∂r ⎦

or

∂ 2T ∂ ⎡ ∂T ⎤
r + r =0
∂x 2 ∂r ⎢⎣ ∂r ⎥⎦

The boundary conditions are:

∂T ⎧⎪q′′ for r < rexp


−k =⎨
∂x x =0 ⎪⎩0 for r > rexp

Tx →∞ = Ts

Tr =0 must be finite

Tr = rout = Ts

As stated, there are two non-homogeneous boundary conditions; however, the boundary
condition at r = rout can be made homogeneous by defining the temperature difference:

θ = T − Ts

The partial differential equation and boundary conditions are written in terms of θ:

∂ 2θ ∂ ⎡ ∂θ ⎤
r + r =0 (1)
∂x 2 ∂r ⎢⎣ ∂r ⎥⎦

∂θ ⎧⎪q′′ for r < rexp


−k =⎨ (2)
∂x x =0 ⎪⎩0 for r > rexp
θ x →∞ = 0 (3)

θ r =0 must be finite (4)

θ r =r = 0
out
(5)

Note that the two homogeneous boundary conditions are in the x-direction and so the
eigenfunctions of the problem will be in this direction. We assume that the solution is separable;
that is, the solution is the product of a function only of x (θX) and r (θR):

θ ( x, y ) = θ X ( x ) θ R ( r )

Substituting the product solution into the governing partial differential equation, Eq. (1), leads to:

d 2θ X d ⎡ dθ R ⎤
rθ R +θ X r =0
dx 2
dr ⎢⎣ dr ⎥⎦

Dividing by the product r θX θR leads to:

d 2θ X d ⎡ dθ R ⎤
⎢r ⎥
dx + dr ⎣ dr ⎦ = 0
2

θX rθ R
±λ2 ∓λ2

Note that the 1st term is a function only of x while the 2nd term is a function only of r; these two
quantities must be equal and opposite constants (±λ2). The choice of the sign is again important;
the eigenfunctions must be in r and therefore the two ordinary differential equations must be:

d 2θ X
− λ2 θ X = 0 (6)
dx 2

d ⎡ dθ R ⎤
⎢ r ⎥ + λ2 rθ R = 0 (7)
dr ⎣ dr ⎦

The eigenproblem will be solved first; the solution to Eq. (7) is:

θ R = C1 BesselJ ( 0,λ r ) + C2 BesselY ( 0,λ r )

The boundary condition at r = 0, Eq. (4), requires that C2 = 0.

θ R = C1 BesselJ ( 0,λ r )
The boundary condition at r = rout, Eq. (5), leads to:

C1 BesselJ ( 0,λ rout ) = 0 (8)

The 0th order Bessel function of the 1st kind (i.e., Bessel_J(0,x)) oscillates about zero every time
the argument changes by 2π in the same way that sine and cosine do; therefore, there are an
infinite number of eigenvalues λi that will satisfy Eq. (8) associated with an infinite number of
eigenfunctions. The eigencondition for this problem cannot be used to explicitly solve for the
eigenvalues; rather, an implicit equation for the eigenvalues results from Eq. (8):

BesselJ ( 0,λi rout ) = 0 where i = 1, 2,...∞ (9)

The eigenfunctions for this problem are:

θ Ri = C1,i BesselJ ( 0,λi r ) where BesselJ ( 0,λi b ) = 0 for i = 1, 2,...∞ (11)

N=51 "number of terms in solution"


duplicate i=1,N
lowerlimit[i]=(i-1)*pi/r_out "lower limit of eigenvalue"
upperlimit[i]=i*pi/r_out "upper limit of eigenvalue"
guess[i]=lowerlimit[i]+pi/(2*r_out) "guess value for eigenvalue"
end

duplicate i=1,N
BesselJ(0,lambda[i]*r_out)=0 "solve for eigenvalues"
end

The arrays lowerlimit[], upperlimit[], and guess[] are used to constrain the solution for the array
lambda[] in the Variable Information window. The solution to the ordinary differential equation
for θX, Eq. (6), is:

θ X i = C3,i exp ( λi x ) + C4,i exp ( −λi x )

The boundary condition at x → ∞, Eq. (3), requires that C3,i = 0:

θ X i = C4,i exp ( −λi x )

and so the general solution for each eigenvalue is:

θ i = θ Ri θ X i = Ci BesselJ ( 0,λi r ) exp ( −λi x )

The series solution for θ is:


∞ ∞
θ = ∑ θi = ∑ Ci BesselJ ( 0,λi r ) exp ( −λi x ) (10)
i =1 i =1

Substituting Eq. (10) into the boundary condition at x = 0, Eq. (2), leads to:

∞ ⎧⎪q′′ for r < rexp


k ∑ Ci λi BesselJ ( 0,λi r ) = ⎨ (11)
i =1 ⎪⎩0 for r > rexp

Equation (11) is multiplied by r BesselJ ( 0,λi r ) and integrated from r = 0 to r = rout:

rout rexp

k Ci λi ∫ r BesselJ ( 0,λi r ) dr = q′′


2
∫ r BesselJ ( 0,λ r ) dr
i (12)
0 0
Integral 1 Integral 2

The integrals in Eq. (12) are carried out in Maple:

> restart;
> Integral1:=int(r*(BesselJ(0,lambda[i]*r))^2,r=0..r_out);
Integral1 :=
2 2
1 r_out ( π λ i r_out BesselJ( 0, λ i r_out ) + π λ i r_out BesselJ( 1, λ i r_out ) )
2 πλ i

> Integral2:=int(r*BesselJ(0,lambda[i]*r),r=0..r_exp);
r_exp BesselJ( 1, r_exp λ i )
Integral2 :=
λi

and used to complete the solution in EES:

duplicate i=1,N
Integral1[i]=1/2*r_out/Pi^(1/2)/lambda[i]*(Pi^(1/2)*lambda[i]*r_out*BesselJ(0,lambda[i]*r_out)^2+&
Pi^(1/2)*lambda[i]*r_out*BesselJ(1,lambda[i]*r_out)^2)
Integral2[i]= 1/lambda[i]*r_exp*BesselJ(1,r_exp*lambda[i])
k*lambda[i]*C[i]*Integral1[i]=q``_dot*Integral2[i]
end

The solution is obtained at an arbitrary position according to:

x=0 [micron]*convert(micron,m) "x-position"


r=0 [micron]*convert(micron,m) "r-position"
duplicate i=1,N
theta[i]=C[i]*BesselJ(0,lambda[i]*r)*exp(-lambda[i]*x) "i'th term"
end
theta=sum(theta[1..N])
Figure 2 shows the temperature elevation relative to Ts as a function of radius for various values
of x.

1.2

Temperature difference relative to Ts (K) x=0


1
x = 5 μm
x = 10 μ m
0.8 x = 25 μ m
x = 50 μ m
0.6

0.4

0.2

0
0 20 40 60 80 100 120 140 160 180 200
Radius (μm)
Figure 2: Temperature difference as a function of r for various values of x.

b.) Determine the average temperature of the cylinder at the surface exposed to the heat flux.

The average temperature of the cylinder over the region x = 0 and 0 < r < rexp is:
rexp
1
T =
π rexp
2 ∫ 2π r T
0
x =0 dr (13)

or:
rexp
1
T = Ts +
π rexp
2 ∫ 2π r θ
0
x =0 dr (14)

Substitutinge Eq. (10) into Eq. (14) leads to:


rexp

1
T = Ts + ∫ 2 π r ∑ Ci BesselJ ( 0,λi r ) dr (15)
π rexp
2
0 i =1

which can be rearranged:


rexp

2
T = Ts + 2
rexp
∑ C ∫ r BesselJ ( 0,λ r ) dr
i =1
i i (16)
0
Integral 2
"compute average temperature in flux region"
duplicate i=1,N
theta_bar[i]=C[i]*2*Integral2[i]/r_exp^2
end
theta_bar=sum(theta_bar[1..N])

c.) Define a dimensionless thermal resistance between the surface exposed to the heat flux and
Ts. Plot the dimensionless thermal resistance as a function of rout/rin.

A dimensionless thermal resistance is defined by normalizing the actual resistance against the
reistance to axial conduction through a cylinder that is rout in radius and rout long:

k (T − Ts ) π rout k (T − Ts ) ⎛ rout
2
R π rout
2 2 ⎞
R= = = k⎜ ⎟⎟ (17)
rout q′′ π rexp rout
2
q′′ rout ⎜⎝ rexp ⎠
actual resistance

R_bar=theta_bar*k/(q``_dot*r_exp) "dimensionless thermal resistance"

Figure 3 illustrates the dimensionless thermal resistance as a function of rout/rexp.

80
Dimensionless thermal resistance

10

0.1
1 10 100
Ratio of cylinder to exposure radii, rout/rexp
Figure 3: Dimensionless thermal resistance as a function of rout/rexp.

d.) Show that your plot from (c) does not change if the problem parameters (e.g., Ts, k, etc.) are
changed.

The values of various parameters were changed and did not affect Figure 3 (the plots are overlaid
onto Figure 3).
Problem 2.4-1 (2-7 in text)

The plate shown in Figure P2.4-1 is exposed to a uniform heat flux q ′′ = 1x105 W/m2 along its
top surface and is adiabatic at its bottom surface. The left side of the plate is kept at TL = 300 K
and the right side is at TR = 500 K. The height and width of the plate are H = 1 cm and W = 5
cm, respectively. The conductivity of the plate is k = 10 W/m-K.

q ′′ = 1x10 W/m
5 2

TL = 300 K W = 5 cm
TR = 500 K
y H = 1 cm
x

k = 10 W/m-K
Figure P2.4-1: Plate.

a.) Derive an analytical solution for the temperature distribution in the plate.

The inputs are entered in EES:

$UnitSystem SI MASS RAD PA K J


$TABSTOPS 0.2 0.4 0.6 0.8 3.5 in

"Inputs"
H=1 [cm]*convert(cm,m) "height of plate"
W=5 [cm]*convert(cm,m) "thickness of plate"
k=10 [W/m-K] "conductivity"
q``=100000 [W/m^2] "heat flux"
T_R=500 [K] "temperature of right hand surface"
T_L=300 [K] "temperature of left hand surface"

A mathematical statement of the transformed problem is:

∂ 2θ ∂ 2θ
+ =0 (1)
∂x 2 ∂y 2

with boundary conditions:

θ x =0 = 0 (2)

θ x =W = θ R (3)

∂θ
=0 (4)
∂y y =0
∂θ
k = q ′′ (5)
∂y y=H

where

θ = T − TL (6)

and

θ R = TR − TL (7)

The problem has two, non-homogeneous boundary condition and therefore must be solved using
superposition.

θ = θ A + θB (8)

Problem θA retains the non-homogeneous boundary condition in the x-direction:

∂ 2θ A ∂ 2θ A
+ 2 =0 (9)
∂x 2 ∂y

with boundary conditions:

θ A, x = 0 = 0 (10)

θ A, x =W = θ R (11)

∂θ A
=0 (12)
∂y y =0

∂θ A
=0 (13)
∂y y=H

By inspection, the solution for θA is 1-D in x and given by:

x
θ A = θR (14)
L

x_bar=0.5 [-] "dimensionless x location"


x=x_bar*W "x location"
y_bar=0.5 [-] "dimensionless y location"
y=y_bar*H "y location"
"sub-problem A solution"
theta_R=T_R-T_L "temperature difference of right hand
surface"
theta_A=theta_R*x/W "solution for sub-problem A"

Problem θB retains the non-homogeneous boundary condition in the y-direction:

∂ 2θ B ∂ 2θ B
+ 2 =0 (15)
∂x 2 ∂y

with boundary conditions:

θ B , x =0 = 0 (16)

θ B , x =W = 0 (17)

∂θ B
=0 (18)
∂y y =0

∂θ B
k = q ′′ (19)
∂y y=H

The solution for θB is 2-D and can be obtained using separation of variables. The eigenfunctions
are:

θ X B ,i = sin ( λB ,i x ) (20)

where the eigenvalues are:

iW
λB , i = for i = 1, 2,..∞ (21)
π

"sub-problem B solution"
N_term=11 [-] "number of terms"
duplicate i=1,N_term
lambda_B[i]=i*pi/W "i'th eigenvalue"
end

The solution in the non-homogeneous direction is:

θ YB ,i = C2,i cosh ( λB ,i y ) (22)

The series solution for θB is:



θ B = ∑ Ci sin ( λB ,i x ) cosh ( λB ,i y ) (23)
i =1

Subsituting Eq. (23) into Eq. (19) leads to:


k ∑ λB ,i Ci sin ( λB ,i x ) sinh ( λB ,i H ) = q ′′ (24)
i =1

Equation (24) is multiplied by an eigenfunction and integrated from x = 0 to x = W:


W W
k λB ,i Ci sinh ( λB ,i H ) ∫ sin ( λB ,i x ) dx = q ′′ ∫ sin ( λB ,i x ) dx
2
(25)
0 0

The integrals in Eq. (25) are evaluated in Maple:

> restart;
> assume(i,integer);
> lambda_B:=i*Pi/W;
i~ π
lambda_B :=
W
> int((sin(lambda_B*x))^2,x=0..W);
W
2
> int(sin(lambda_B*x),x=0..W);
W ( −1 + ( -1 )i~ )

i~ π

and used to evaluate the constants:

duplicate i=1,N_term
C[i]*k*lambda_B[i]*sinh(lambda_B[i]*H)*W/2=q``*(-W*(-1+(-1)^i)/i/Pi) "i'th constant"
end

The solution for θB is obtained:

duplicate i=1,N_term
theta_B[i]=C[i]*sin(lambda_B[i]*x)*cosh(lambda_B[i]*y) "i'th term"
end
theta_B=sum(theta_B[1..N_term]) "solution to sub-problem B"

and used to obtain the solution for T:

theta=theta_A+theta_B "superposition of solutions"


T=theta+T_L "temperature"
b.) Prepare a contour plot of the temperature.

Figure 2 illustrates a contour plot of the temperature distribution in the plate.


1
300
Dimensionless y-position, y/H

345.5
0.8
390.9
436.4
0.6 481.8
527.3
572.7
0.4
618.2
663.6
0.2 709.1
754.5

0
0 0.1 0.2 0.3 0.4 0.5 0.6 0.7 0.8 0.9 1
Dimensionless x-position, x/W
Figure 2: Contour plot of the temperature distribution.
Problem 2.5-1 (2-8 in text): A Heating Element
Figure P2.5-1 illustrates an electrical heating element that is affixed to the wall of a chemical
reactor. The element is rectangular in cross-section and very long (into the page). The
temperature distribution within the element is therefore two-dimensional, T(x, y). The width of
the element is a = 5.0 cm and the height is b = 10.0 cm. The three edges of the element that are
exposed to the chemical (at x = 0, y = 0, and x = a) are maintained at a temperature Tc = 200°C
while the upper edge (at y = b) is affixed to the well-insulated wall of the reactor and can
therefore be considered adiabatic. The element experiences a uniform volumetric rate of thermal
energy generation, g ′′′ = 1x106 W/m3. The conductivity of the material is k = 0.8 W/m-K.

reactor wall

k = 0.8 W/m-K
Tc = 200°C
g ′′′ = 1x10 W/m
6 3

a = 5 cm
y
Tc = 200°C
x

Tc = 200°C b = 10 cm
Figure P2.5-1: Electrical heating element.

a.) Develop a 2-D numerical model of the element using EES.

The inputs are entered in EES:

$UnitSystem SI MASS RAD PA K J


$TABSTOPS 0.2 0.4 0.6 0.8 3.5 in

"Inputs"
a=5.0 [cm]*convert(cm,m) "width of element"
b=10.0 [cm]*convert(cm,m) "height of element"
k=0.8 [W/m-K] "conductivity"
T_c=converttemp(C,K,200 [C]) "chemical temperature"
g```_dot=1e6 [W/m^3] "rate of volumetric generation"
L=1 [m] "unit length of element into the page"

The computational domain of the element with the regularly spaced grid of nodes is shown in
Figure 2.
Figure 2: The regularly spaced grid used to obtain a numerical solution.

The first step in obtaining a numerical solution is to position the nodes throughout the
computational domain. We will use grid with nodes placed on the edges and distributed
uniformly throughout. The x and y distance between adjacent nodes (Δx and Δy) are:

L
Δx = (1)
( m − 1)
b
Δy = (2)
( n − 1)

and the x and y positions of any node i,j are given by:

xi =
( i − 1) a (3)
( m − 1)

yj =
( j − 1) b (4)
( n − 1)
where m and n are the number of nodes used in the x and y directions.

"Setup nodes"
m=11 [-]
"number of nodes in the x-direction"
n=11 [-]
"number of nodes in the y-direction"
Dx=a/(m-1)
"distance between nodes in the x-direction"
Dy=b/(n-1)
"distance between nodes in the y-direction"
duplicate i=1,m
x[i]=(i-1)*Dx "x-position of each node"
end
duplicate j=1,n
y[j]=(j-1)*Dy "y-position of each node"
end

The next step in the solution is to write an energy balance for each node. Figure 3 illustrates a
control volume and the associated energy transfers for an internal node (see Figure 2) which
include conduction from each side ( q RHS and q LHS ), the top ( qtop ), and the bottom ( qbottom ). Note
that the direction associated with these energy transfers is arbitrary (i.e., they could have been
taken as positive if energy leaves the control volume), but it is important to write the equation in
a manner consistent with the chosen directions.

Figure 3: Energy balance for an internal node

The energy balance suggested by Figure 3 is:

q RHS [i, j ] + q LHS [i, j ] + qtop [i, j ] + qbottom [i, j ] + g [i, j ] = 0 (5)

The next step is to approximate each of the terms in the energy balance; the material separating
the nodes is assumed to behave as a plane wall resistance and therefore:

k Δy L
q RHS [i, j ] =
Δx
(T [i + 1, j ] − T [i, j ]) (6)

where L is the length of the element (assumed to be 1 m in order to do the problem on a unit
length basis); therefore, Δy L is the area for conduction and Δx is the distance over which the
conduction heat transfer occurs. Note that the temperature difference is consistent with the
direction of the arrow in Figure 3; if Ti+1,j is greater than Ti,j then energy is leaving the node and
q RHS is positive. The other heat transfers are approximated using a similar model:
k Δy L
q LHS [i, j ] =
Δx
(T [i − 1, j ] − T [i, j ]) (7)

k ΔxW
qtop [i, j ] =
Δy
(T [i, j + 1] − T [i, j ]) (8)

k ΔxW
qbottom [i, j ] =
Δy
(T [i, j − 1] − T [i, j ]) (9)

The generation is the product of the volume of the control volume and the volumetric rate of
generation:

g [i, j ] = g ′′′ Δx Δy L (10)

These equations are entered in EES using a nested duplicate statement:

"Internal node control volumes"


duplicate i=2,(m-1)
duplicate j=2,(n-1)
q_dot_LHS[i,j]+q_dot_RHS[i,j]+q_dot_top[i,j]+q_dot_bottom[i,j]+gen[i,j]=0
q_dot_LHS[i,j]=k*L*Dy*(T[i-1,j]-T[i,j])/Dx
q_dot_RHS[i,j]=k*L*Dy*(T[i+1,j]-T[i,j])/Dx
q_dot_top[i,j]=k*L*Dx*(T[i,j+1]-T[i,j])/Dy
q_dot_bottom[i,j]=k*L*Dx*(T[i,j-1]-T[i,j])/Dy
gen[i,j]=Dx*Dy*L*g```_dot
end
end

Note that each time the outer duplicate statement iterates once (i.e., i is increased by 1), the inner
duplicate statement iterates (n-1) times (i.e., j runs from 2 to n-1). Therefore, all of the internal
nodes are considered with these two nested duplicate loops. Also note that the unknowns are
placed in an array rather than a vector. The entries in the array T is accessed using two indices
contained in square brackets.

The boundary nodes have to be treated separately. The left, right, and bottom boundaries are
easy as the temperature is specified:

T [1, j ] = Tc for j = 1...n (11)

T [ m, j ] = Tc for j = 1...n (12)


T [i,1] = Tc for i = 1...m (13)

These equations are entered in EES:


"Temperature along right edge"
duplicate j=1,n
T[1,j]=T_c
end

"Temperature along left edge"


duplicate j=1,n
T[m,j]=T_c
end

"Temperature along bottom edge"


duplicate i=2,(m-1)
T[i,1]=T_c
end

The upper boundary nodes must be considered using energy balances. Figure 4 illustrates an
energy balance associated with a node that is located on the top, insulated boundary (see Figure
2).

Figure 4: Energy balance for a node on the top boundary

The energy balance suggested by Figure 4 is:

q RHS [i, n ] + q LHS [i, n ] + qbottom [i, n ] + g [i, n ] = 0 (14)

The conduction terms in the x direction must be approximated slightly differently:

k Δy L
q RHS [i, n ] =
2Δx
(T [i + 1, n] − T [i, n]) (15)

k Δy L
q LHS [i, n ] =
2 Δx
(T [i − 1, n] − T [i, n]) (16)

The factor of 2 in the denominator appears because there is half the available area for conduction
through the sides of the control volume on the top boundary. The other conduction term is
approximated as before:

k ΔxW
qbottom [i, n ] =
Δy
(T [i, n − 1] − T [i, n]) (17)
The generation term is:

Δx Δy L g ′′′
g [i, n ] = (18)
2

These equations are entered in EES using a single duplicate statement:

"Upper edge"
duplicate i=2,(m-1)
q_dot_LHS[i,n]+q_dot_RHS[i,n]+q_dot_bottom[i,n]+gen[i,n]=0
q_dot_LHS[i,n]=k*(Dy/2)*L*(T[i-1,n]-T[i,n])/Dx
q_dot_RHS[i,n]=k*(Dy/2)*L*(T[i+1,n]-T[i,n])/Dx
q_dot_bottom[i,n]=k*Dx*L*(T[i,n-1]-T[i,n])/Dy
gen[i,n]=Dx*Dy*L*g```_dot/2
end

Notice that the control volumes at the top left corner (i.e., i =1, j =n) and the top right corner
(i.e., i =m, j =n) have already been taken addressed by the equations for the left and right
boundaries, Eqs. (11) and (12). Therefore, we have to make sure not to write additional
equations related to this node or the problem will be over-specified and so the equations for the
top boundary can only be written for i = 2...(m-1).

We have derived a total of m x n equations in the m x n unknown temperatures; these equations


completely specify the problem and they have now all been entered in EES. Therefore, a
solution can be obtained by solving the EES code. The solution is contained in the Arrays
Window; each column of the table corresponds to the temperatures associated with one value of i
and all of the value of j (i.e., the temperatures in a column are at a constant value of y and
varying values of x).

b.) Plot the temperature as a function of x at various values of y. What is the maximum
temperature within the element and where is it located?

The solution is obtained for n=11 and the columns Ti,1 (corresponding to y =0) through Ti,11
(corresponding to y = 10 cm) are plotted in Figure 5 as a function of x.
Figure 5: Temperature as a function of x for various values of y.

The hottest spot in the element is at the adiabatic wall (y = 10 cm) and the center (x = 2.5 cm);
the hottest temperature is about 860 K.

c.) Prepare a reality check to show that your solution behaves according to your physical
intuition. That is, change some aspect of your program and show that the results behave as
you would expect (clearly describe the change that you made and show the result).

There are several possible answers to this; I increased the conductivity by a factor of 10 and
examined the temperature distribution. Figure 6 illustrates the temperature as a function of x for
the original conductivity (k = 0.8 W/m-K) and the increased conductivity (k = 8.0 W/m-K);
notice that the increased conductivity has had the expected effect of reducing the temperature
rise.
Figure 6: Temperature as a function of x for y = 10 cm and two values of conductivity.
Problem 2.6-1 (2-9 in text): Model of Welding Process (revisited)
Figure P2.6-1 illustrates a cut-away view of two plates that are being welded together. Both
edges of the plate are clamped and effectively held at temperatures Ts = 25°C. The top of the
plate is exposed to a heat flux that varies with position x, measured from joint, according to:
qm′′ ( x ) = q ′′j exp ( − x / L j ) where q ′′j =1x106 W/m2 is the maximum heat flux (at the joint, x = 0)
and Lj = 2.0 cm is a measure of the extent of the heat flux. The back side of the plates are
exposed to liquid cooling by a jet of fluid at Tf = -35°C with h = 5000 W/m2-K. A half-
symmetry model of the problem is shown in Figure P2.6-1. The thickness of the plate is b = 3.5
cm and the width of a single plate is W = 8.5 cm. You may assume that the welding process is
steady-state and 2-D. You may neglect convection from the top of the plate. The conductivity
of the plate material is k = 38 W/m-K.
both edges are held at fixed temperature
heat flux
joint

qm′′ impingement cooling

k = 38 W/m-K

W = 8.5 cm Ts = 25°C
b = 3.5 cm
y

h = 5000 W/m -K, T f = −35°C


2

Figure P2.6-1: Welding process and half-symmetry model of the welding process.

a.) Develop a separation of variables solution to the problem (note, this was done previously in
Problem 2.2-1). Implement the solution in EES and prepare a plot of the temperature as a
function of x at y = 0, 1.0, 2.0, 3.0, and 3.5 cm.
b.) Prepare a contour plot of the temperature distribution.

See the solution for Problem 2.2-1 for parts (a) and (b).

c.) Develop a numerical model of the problem. Implement the solution in MATLAB and
prepare a contour or surface plot of the temperature in the plate.

The input parameters are entered in the MATLAB function P2p2_1; the input arguments are m
and n, the number of nodes in the x and y coordinates while the output arguments are the x and y
positions of each node and the predicted temperature at each node.

function[xm,ym,T]=P2p6_1(m,n)

W=0.085; %width of plate (m)


b=0.035; %thickness of plate (m)
k=38; %conductivity of plate material (W/m-K)
T_s=298.1; %side temperature (K)
T_f=238.2; %fluid temperature (K)
h=5000; %heat transfer coefficient (W/m^2-K)
L=1; %per unit length (m)

end

A sub-function is defined to provide the heat flux on the upper surface:

function[qflux]=qf(x)

L_j=0.02; %length scale (m)


qf_j=1e6; %heat flux at center (W/m^2)

qflux=qf_j*exp(-x/L_j);

end

A 2-D numerical model will be generated using a grid in which the x and y coordinates of each
node are:

xi =
( i − 1)W for i = 1..m (1)
( m − 1)

yi =
( j − 1)W for j = 1..n (2)
( m − 1)
The distance between adjacent nodes is:

W
Δx = (3)
( m − 1)
W
Δy = (4)
( n − 1)
%Setup grid
for i=1:m
x(i,1)=(i-1)*W/(m-1);
end
Dx=W/(m-1);
for j=1:n
y(j,1)=(j-1)*W/(n-1);
end
Dy=b/(n-1);
The problem will be solved by setting up and inverting a matrix containing the algebraic
equations that enforce the conservation of energy for each control volume. A control volume for
an internal node includes conduction from the left and right sides ( q LHS and q RHS ) and top and
bottom ( qtop and qbottom ). The energy balance is:

q RHS + q LHS + qtop + qbottom = 0

The conduction terms are approximated according to:

k Δy L
q RHS =
Δx
(Ti−1, j − Ti, j )
k Δy L
q LHS =
Δx
(Ti+1, j − Ti, j )
k Δx L
qbottom =
Δy
(Ti, j −1 − Ti, j )

k Δx L
qtop =
Δy
(Ti, j +1 − Ti, j )
where L is the depth of the plate (into the page); L is set to 1.0 m which is consistent with doing
the problem on a per unit length basis. Combining these equations leads to:

k Δy L k Δy L k Δx L k Δx L
Δx
( Ti −1, j − Ti , j ) +
Δx
( Ti +1, j − Ti , j ) +
Δy
( Ti , j −1 − Ti , j ) +
Δy
(Ti, j +1 − Ti, j ) = 0 (5)
for i = 2.. ( m − 1) and j = 2.. ( n − 1)

The equation is rearranged to make it clear what the coefficient is for each unknown
temperature:

⎡ k Δy L k Δx L ⎤ ⎡ k Δy L ⎤ ⎡ k Δy L ⎤ ⎡ k Δx L ⎤ ⎡ k Δx L ⎤
Ti , j ⎢ −2 −2 ⎥ + Ti −1, j ⎢ ⎥ + Ti +1, j ⎢ ⎥ + Ti , j −1 ⎢ ⎥ + Ti , j +1 ⎢ ⎥=0
⎣ Δx Δy ⎦ ⎣ Δx ⎦ ⎣ Δx ⎦ ⎣ Δy ⎦ ⎣ Δy ⎦
for i = 2.. ( m − 1) and j = 2.. ( n − 1)
(6)

The control volume equations must be placed into the matrix equation:

AX =b
where the equation for control volume i,j is placed into row m(j-1)+i of A and Ti,j corresponds to
element Xm (j-1)+i in the vector X . Therefore, each coefficient in Eq. (6) (i.e., each term
multiplying an unknown temperature on the left side of the equation) must be placed in the row
of A corresponding to the control volume being examined and the column of A corresponding
to the unknown in X . The matrix assignments consistent with Eq. (6) are:

k Δy L k Δx L
Am( j −1)+i ,m( j −1)+i = −2 −2 for i = 2.. ( m − 1) and j = 2.. ( n − 1) (7)
Δx Δy

k Δy L
Am( j −1)+i ,m( j −1)+i −1 = for i = 2.. ( m − 1) and j = 2.. ( n − 1) (8)
Δx

k Δy L
Am( j −1)+i ,m( j −1)+i +1 = for i = 2.. ( m − 1) and j = 2.. ( n − 1) (9)
Δx

k Δx L
Am( j −1)+i ,m( j −1−1)+i = for i = 2.. ( m − 1) and j = 2.. ( n − 1) (10)
Δy

k Δx L
Am( j −1)+i ,m( j +1−1)+i = for i = 2.. ( m − 1) and j = 2.. ( n − 1) (11)
Δy

A sparse matrix is allocated in MATLAB for A and the equations derived above are
implemented using a nested for loop. The spalloc command requires three arguments, which are
the number of rows and columns and the maximum number of elements in the matrix. Note that
there are at most 5 non-zero entries in each row of A , corresponding to Eqs. (7) through (11);
thus the last argument in the spalloc command which corresponds to the maximum number of
non-zero entries in the sparse matrix.

A=spalloc(m*n,m*n,5*m*n); %allocate a sparse matrix for A


bm=zeros(m*n,1); %allocate a matrix for b
%energy balances for internal nodes
for i=2:(m-1)
for j=2:(n-1)
A(m*(j-1)+i,m*(j-1)+i)=-2*k*Dy*L/Dx-2*k*Dx*L/Dy;
A(m*(j-1)+i,m*(j-1)+i-1)=k*Dy*L/Dx;
A(m*(j-1)+i,m*(j-1)+i+1)=k*Dy*L/Dx;
A(m*(j-1)+i,m*(j-1-1)+i)=k*Dx*L/Dy;
A(m*(j-1)+i,m*(j+1-1)+i)=k*Dx*L/Dy;
end
end

The nodes on the right side have a specified temperature:

Tm, j = Ts for j = 1..n


The matrix assignments suggested by these equations are:

Am( j −1)+ m, m( j −1)+ m = 1 for j = 1..n

bm( j −1)+ m = Tst for j = 1..n

These assignments are implemented in MATLAB:

%right side temperature is specified


for j=1:n
A(m*(j-1)+m,m*(j-1)+m)=1;
bm(m*(j-1)+m,1)=T_s;
end

The nodes along the upper edge must be considered separately, leading to:

⎡ k Δy L k Δx L ⎤ ⎡ k Δy L ⎤ ⎡ k Δy L ⎤ ⎡ k Δx L ⎤
Ti ,n ⎢ −
Δx

Δy ⎦ ⎥ + Ti −1, n ⎢ ⎥ + Ti +1, n ⎢ ⎥ + Ti ,n −1 ⎢ ⎥ = − qm′′ Δx L
⎣ ⎣ 2 Δx ⎦ ⎣ 2 Δx ⎦ ⎣ Δy ⎦ (12)
for i = 2.. ( m − 1)

which is expressed in matrix form as:

k Δy L k Δx L
Am( n −1)+i ,m( n −1)+i = − − for i = 2.. ( m − 1) (13)
Δx Δy

k Δy L
Am( n −1)+i ,m( n −1)+i −1 = for i = 2.. ( m − 1) (14)
2 Δx

k Δy L
Am( n −1)+i ,m( n −1)+i +1 = for i = 2.. ( m − 1) (15)
2 Δx

k Δx L
Am( n −1)+i , m( n −1−1)+i = for i = 2.. ( m − 1) (16)
Δy

bm( n −1)+i ,1 = −qm′′ Δx L for i = 2.. ( m − 1) (17)

%upper edge
for i=2:(m-1)
A(m*(n-1)+i,m*(n-1)+i)=-k*Dy*L/Dx-k*Dx*L/Dy;
A(m*(n-1)+i,m*(n-1)+i-1)=k*Dy*L/(2*Dx);
A(m*(n-1)+i,m*(n-1)+i+1)=k*Dy*L/(2*Dx);
A(m*(n-1)+i,m*(n-1-1)+i)=k*Dx*L/Dy;
bm(m*(n-1)+i,1)=-qf(x(i))*Dx*L;
end
The node at the upper left corner must be considered separately, leading to:

⎡ k Δy L k Δx L ⎤ ⎡ k Δy L ⎤ ⎡ k Δx L ⎤ Δx L
T1,n ⎢ − − ⎥ + T2,n ⎢ ⎥ + T1,n −1 ⎢ ⎥ = − qm′′ (18)
⎣ 2 Δx 2 Δy ⎦ ⎣ 2 Δx ⎦ ⎣ 2 Δy ⎦ 2

which is expressed in matrix form as:

k Δy L k Δx L
Am( n −1)+1,m( n −1)+1 = − − (19)
2 Δx 2 Δy

k Δy L
Am( n −1) +1,m( n −1)+1+1 = (20)
2 Δx

k Δx L
Am( n −1)+1, m( n −1−1)+1 = (21)
2 Δy

Δx L
bm( n −1)+1,1 = −qm′′ (22)
2

%upper left corner


A(m*(n-1)+1,m*(n-1)+1)=-k*Dy*L/(2*Dx)-k*Dx*L/(2*Dy);
A(m*(n-1)+1,m*(n-1)+1+1)=k*Dy*L/(2*Dx);
A(m*(n-1)+1,m*(n-1-1)+1)=k*Dx*L/(2*Dy);
bm(m*(n-1)+1,1)=-qf(x(1))*Dx*L/2;

The node along the left edge must be considered separately, leading to:

⎡ k Δy L k Δx L ⎤ ⎡ k Δy L ⎤ ⎡ k Δx L ⎤ ⎡ k Δx L ⎤
T1, j ⎢ − − ⎥ + T2 +1, j ⎢ ⎥ + T1, j −1 ⎢ ⎥ + T1, j +1 ⎢ ⎥=0
⎣ Δx Δy ⎦ ⎣ 2 Δx ⎦ ⎣ 2 Δy ⎦ ⎣ 2 Δy ⎦ (23)
for j = 2.. ( n − 1)

which is expressed in matrix form as:

k Δy L k Δx L
Am( j −1)+1,m( j −1)+1 = − − for j = 2.. ( n − 1) (24)
Δx Δy

k Δy L
Am( j −1)+1,m( j −1)+1+1 = for j = 2.. ( n − 1) (25)
Δx

k Δx L
Am( j −1) +1,m( j −1−1)+1 = for j = 2.. ( n − 1) (26)
2 Δy
k Δx L
Am( j −1)+1,m( j +1−1)+1 = for j = 2.. ( n − 1) (27)
2 Δy

%left edge
for j=2:(n-1)
A(m*(j-1)+1,m*(j-1)+1)=-k*Dy*L/Dx-k*Dx*L/Dy;
A(m*(j-1)+1,m*(j-1)+1+1)=k*Dy*L/Dx;
A(m*(j-1)+1,m*(j-1-1)+1)=k*Dx*L/(2*Dy);
A(m*(j-1)+1,m*(j+1-1)+1)=k*Dx*L/(2*Dy);
end

The node at the lower left corner must be considered separately, leading to:

⎡ k Δy L k Δ x L Δx L ⎤ ⎡ k Δy L ⎤ ⎡ k Δx L ⎤ Δx L
T1,1 ⎢ − − −h ⎥ + T2,1 ⎢ ⎥ + T1,2 ⎢ ⎥ = −h Tf (28)
⎣ 2 Δx 2 Δy 2 ⎦ ⎣ 2 Δx ⎦ ⎣ 2 Δy ⎦ 2

which is expressed in matrix form as:

k Δy L k Δx L Δx L
Am(1−1) +1,m(1−1)+1 = − − −h (29)
2 Δx 2 Δy 2

k Δy L
Am(1−1)+1, m(1−1)+1+1 = (30)
2 Δx

k Δx L
Am(1−1)+1, m(1+1−1)+1 = (31)
2 Δy

Δx L
bm(1−1)+1,1 = −h Tf (32)
2

%lower left corner


A(m*(1-1)+1,m*(1-1)+1)=-k*Dy*L/(2*Dx)-k*Dx*L/(2*Dy)-h*L*Dx/2;
A(m*(1-1)+1,m*(1-1)+1+1)=k*Dy*L/(2*Dx);
A(m*(1-1)+1,m*(1+1-1)+1)=k*Dx*L/(2*Dy);
bm(m*(1-1)+1,1)=-h*L*Dx*T_f/2;

The nodes along the bottom edge must be considered separately, leading to:

⎡ k Δy L k Δx L ⎤ ⎡ k Δy L ⎤ ⎡ k Δy L ⎤ ⎡ k Δx L ⎤
Ti ,1 ⎢ − − − h Δx L ⎥ + Ti −1,1 ⎢ ⎥ + Ti +1,1 ⎢ ⎥ + Ti ,1+1 ⎢ ⎥ = − h Δx LT f
⎣ Δx Δy ⎦ ⎣ 2 Δx ⎦ ⎣ 2 Δx ⎦ ⎣ Δy ⎦ (33)
for i = 2.. ( m − 1)

which is expressed in matrix form as:


k Δy L k Δx L
Am(1−1)+i ,m(1−1)+i = − − - h Δx L for i = 2.. ( m − 1) (34)
Δx Δy

k Δy L
Am(1−1) +i , m(1−1)+i −1 = for i = 2.. ( m − 1) (35)
2 Δx

k Δy L
Am(1−1) +i , m(1−1)+i +1 = for i = 2.. ( m − 1) (36)
2 Δx

k Δx L
Am(1−1) +i , m(1+1−1)+i = for i = 2.. ( m − 1) (37)
Δy

bm(1−1)+i ,1 = − h Δx LT f for i = 2.. ( m − 1) (38)

%lower edge
for i=2:(m-1)
A(m*(1-1)+i,m*(1-1)+i)=-k*Dy*L/Dx-k*Dx*L/Dy-h*Dx*L;
A(m*(1-1)+i,m*(1-1)+i-1)=k*Dy*L/(2*Dx);
A(m*(1-1)+i,m*(1-1)+i+1)=k*Dy*L/(2*Dx);
A(m*(1-1)+i,m*(1+1-1)+i)=k*Dx*L/Dy;
bm(m*(1-1)+i,1)=-h*Dx*L*T_f;
end

The vector of unknowns X is obtained through matrix manipulation and then placed into matrix
format. Matrices for the x and y positions of each node are also created.

X=A\bm;
for i=1:m
for j=1:n
xm(i,j)=x(i);
ym(i,j)=y(j);
T(i,j)=X(m*(j-1)+i);
end
end

A surface plot of the result is obtained by typing:

>> [x,y,T]=P2p6_1(20,40);
>> surf(x,y,T);

The plot is shown in Figure 3.


Figure 3: Surface plot of temperature in the plate.

b.) Plot the temperature as a function of x at y = 0, b/2, and b and overlay on this plot the
separation of variables solution obtained in part (a) evaluated at the same locations.

The comparison is shown in Figure 4.

Figure 4: Temperature as a function of axial position for y = 0, b/2, and b predicted using the
separation of variables solution and the numerical solution.
Problem 2.7-1 (2-10 in text): A Double Paned Window
Figure P2.7-1(a) illustrates a double paned window. The window consists of two panes of glass
each of which is tg = 0.95 cm thick and W = 4 ft wide by H = 5 ft high. The glass panes are
separated by an air gap of g = 1.9 cm. You may assume that the air is stagnant with ka = 0.025
W/m-K. The glass has conductivity kg = 1.4 W/m-K. The heat transfer coefficient between the
inner surface of the inner pane and the indoor air is hin = 10 W/m2-K and the heat transfer
coefficient between the outer surface of the outer pane and the outdoor air is hout = 25 W/m2-K.
You keep your house heated to Tin = 70°F.
width of window, W = 4 ft

tg = 0.95cm
Tin = 70°F
tg = 0.95 cm
hin = 10 W/m -K
2
g = 1.9 cm

H = 5 ft Tout = 23°F
hout = 25 W/m -K
2

ka = 0.025 W/m-K
kg = 1.4 W/m-K

casing shown in P2.10(b)


Figure P2.7-1(a): Double paned window.

The average heating season in Madison lasts about time = 130 days and the average outdoor
temperature during this time is Tout = 23°F. You heat with natural gas and pay, on average, ec =
1.415 $/therm (a therm is an energy unit =1.055x108 J).
a.) Calculate the average rate of heat transfer through the double paned window during the
heating season.

The inputs are entered in EES:

$UnitSystem SI MASS RAD PA K J


$TABSTOPS 0.2 0.4 0.6 0.8 3.5 in

"Inputs"
tg=0.375*convert(inch,m) "glass thickness"
g=0.75*convert(inch,m) "air gap"
k_g=1.4 [W/m-K] "glass conductivity"
k_a=0.025 [W/m-K] "air conductivity"
H=5 [ft]*convert(ft,m) "height of window"
W=4[ft]*convert(ft,m) "width of window"
T_in=converttemp(F,K,70 [F]) "indoor air temperature"
h_in=10 [W/m^2-K] "heat transfer coefficient on inside of window"
T_out=converttemp(F,K,23 [F]) "outdoor air temperature"
h_out=25 [W/m^2-K] "heat transfer coefficient on outside of window"
time=130 [day]*convert(day,s) "heating season duration"
ec=1.415 [$/therm]*convert($/therm,$/J) "cost of energy"

The heat transfer is resisted by convection on the inner and outer surfaces:
1
Rconv ,in = (1)
hin W H

1
Rconv ,out = (2)
hout W H

and conduction through the glass panes and the air:

tg
Rcond , g = (3)
kg W H

g
Rcond ,a = (4)
ka W H

R_conv_in=1/(h_in*W*H) "convection resistance on inside of window"


R_cond_g=tg/(k_g*W*H) "conduction resistance of glass pane"
R_cond_a=g/(k_a*W*H) "conduction resistance of air gap"
R_conv_out=1/(h_out*W*H) "convection resistance on outside of window"

The heat transfer rate through the window is:

Tin − Tout
q = (5)
Rconv ,in + 2 Rcond , g + Rcond ,a + Rconv ,out

q_dot=(T_in-T_out)/(R_conv_in+2*R_cond_g+R_cond_a+R_conv_out)
"rate of heat transfer through the window"

which leads to q = 53.0 W.

b.) How much does the energy lost through the window cost during a single heating season?

The total amount of energy lost over the course of a heating season is:

Q = q time (6)

and the associated cost is:

cost = ec Q (7)

Q=q_dot*time "total energy loss"


cost=ec*Q "cost to heat house per window"

which leads to cost = $7.98/heating season.


There is a metal casing that holds the panes of glass and connects them to the surrounding wall,
as shown in Figure P2.7-1(b). Because the metal casing is high conductivity, it seems likely that
you could lose a substantial amount of heat by conduction through the casing (potentially
negating the advantage of using a double paned window). The geometry of the casing is shown
in Figure P2.7-1(b); note that the casing is symmetric about the center of the window.
glass panes

1.9 cm
Tin = 70°F 0.95 cm Tout = 23°F
hout = 25 W/m -K
2
hin = 10 W/m -K
2
air 2 cm

4 cm
0.5 cm 3 cm
metal casing
km = 25 W/m-K 0.4 cm
wood

Figure P2-10(b) Metal casing.

All surfaces of the casing that are adjacent to glass, wood, or the air between the glass panes can
be assumed to be adiabatic. The other surfaces are exposed to either the indoor or outdoor air.
c.) Prepare a 2-D thermal analysis of the casing using FEHT. Turn in a print out of your
geometry as well as a contour plot of the temperature distribution. What is the rate of energy
lost via conduction through the casing per unit length (W/m)?

The geometry from Figure 3 is entered approximately, as shown in Figure 4.

Figure 4: Approximate geometry.

Each of the points are selected individually and their exact coordinates are entered, which leads
to the more precise geometry shown in Figure 4.
Figure 5: Geometry.

The material properties are specified by selecting the outline and selecting Material Properties
from the Specify menu. The boundary conditions are specified by selecting each type of
boundary and then selecting Boundary Conditions from the Specify menu. A crude mesh is
generated, as shown in Figure 6.

Figure 6: Crude Mesh.

The temperature contours are shown in Figure 7.

Figure 7: Temperature Contours.


The heat transfer per unit length is obtained by selecting Heat Flow from the View menu and
selecting all of the boundaries that are exposed to either the indoor or outdoor air. The heat flow
is 10.0 W/m.

d.) Show that your numerical model has converged by recording the rate of heat transfer per
length for several values of the number of nodes.

The mesh is refined several times and each time the heat transfer rate per unit length of casing is
recorded; the results are shown in Figure 8.

Figure 8: Heat transfer through the casing per unit length as a function of the number of nodes.

e.) How much does the casing add to the cost of heating your house?

′ ) is entered in EES:
The result from FEHT, the heat transfer per unit length ( qcasing

q`_dot_casing=10.0 [W/m] "casing heat transfer per unit meter"

The total heat transfer through the casing is:


qcasing = qcasing 2 (W + H ) (8)

The total heat lost through the casing is:

Qcasing = qcasing time (9)

and the associated cost is:


costcasing = ec Qcasing (10)

q_dot_casing=q`_dot_casing*2*(W+H) "heat transfer rate through casing"


Q_casing=q_dot_casing*time "total energy flow through casing"
cost_casing=ec*Q_casing
"cost to heat house per window due to casing"

which leads to an additional cost of $8.27 per heating season.


P2.7-3 (2-11 in text): A Spacecraft Radiator
A radiator panel extends from a spacecraft; both surfaces of the radiator are exposed to space (for
the purposes of this problem it is acceptable to assume that space is at 0 K); the emittance of the
surface is ε = 1.0. The plate is made of aluminum (k = 200 W/m-K and ρ = 2700 kg/m3) and has
a fluid line attached to it, as shown in Figure 2.7-3(a). The half-width of the plate is a=0.5 m
wide while the height of the plate is b=0.75m. The thickness of the plate is a design variable and
will be varied in this analysis; begin by assuming that the thickness is th = 1.0 cm. The fluid
lines carry coolant at Tc = 320 K. Assume that the fluid temperature is constant although the fluid
temperature will actually decrease as it transfers heat to the radiator. The combination of
convection and conduction through the panel-to-fluid line mounting leads to an effective heat
transfer coefficient of h = 1,000 W/m2-K over the 3.0 cm strip occupied by the fluid line.
k = 200 W/m-K
space at 0 K
ρ = 2700 kg/m3
ε = 1.0
a = 0.5 m 3 cm
th = 1 cm

b = 0.75 m
fluid at Tc = 320 K

half-symmetry model of panel, Figure P2-11(b)


Figure 2.7-3(a): Radiator panel

The radiator panel is symmetric about its half-width and the critical dimensions that are required
to develop a half-symmetry model of the radiator are shown in Figure 2.7-3(b). There are three
regions associated with the problem that must be defined separately so that the surface conditions
can be set differently. Regions 1 and 3 are exposed to space on both sides while Region 2 is
exposed to the coolant fluid one side and space on the other; for the purposes of this problem, the
effect of radiation to space on the back side of Region 2 is neglected.
Region 1 (both sides exposed to space)
Region 2 (exposed to fluid - neglect radiation to space)
Region 3 (both sides exposed to space)

(0.50,0.75)

(0.50,0.55)

(0.50,0.52)

y
x
(0,0) (0.50,0)
(0.25,0) line of symmetry
(0.22,0)
Figure 2.7-3(b): Half-symmetry model.

a.) Prepare a FEHT model that can predict the temperature distribution over the radiator panel.
The radiator panel can be modeled as 2-D problem because it is thin and has high conductivity.
The Biot number compares the ratio of the internal conduction resistance to the external
resistance; in this case due to radiation.

Rcond
Bi = (1)
Rrad

where

th
Rcond = (2)
2 k As

Using the concept of a radiation resistance:

1
Rrad = (3)
As σ ε s (T + Tsur
s
2 2
)(Ts + Tsur )
The Biot number is therefore:

th σ ε s (Ts2 + Tsur
2
)(Ts + Tsur )
Bi = (4)
2k

The surrounding temperature in Eq. (4), Tsur corresponds to space which is essentially 0 K and
therefore Eq. (4) reduces to:

th σ ε s Ts3
Bi = (5)
2k

We don’t know the value of the panel surface temperature, Ts in Eq. (5), but we can assume that
it will be near the fluid temperature for any well-designed radiator. Using Ts = Tf in Eq. (5)
results in a Biot number of 4.6x10-5 which is very small; clearly for any reasonable value of Ts
the problem will be 2-D.

FEHT can simulate 2-D problems of this type using the Extended Surface mode. Start FEHT
and select Extended Surface from the Subject menu. Each of the three regions must be drawn
separately using the Outline option from the Draw menu. It is easiest to set an appropriate grid
using the Scale and Size selection from the Setup menu (Figure 2).
Figure 2: Scale and Size dialog window

Generate outlines for the 3 regions that are close to the correct scale and then double-click on
each node and position it exactly (Figure 3).

Figure 3: Geometry definition.

The material properties for all three regions can be set by selecting each while holding down the
shift key and then selecting Specify Properties from the Specify menu. Select Aluminum from
the list of materials and then modify the conductivity to be 200 W/m-K and the thickness to be
0.01 m to match the problem statement (Figure 4).
Figure 4: Specify material properties.

The surface conditions for Regions 1 and 3 can be set by selecting these regions with the shift
key held down and then selecting Surface Conditions from the Specify menu. These regions are
radiating to space but there is no option listed in the Extended Surface Conditions for a radiative
surface condition. The radiation heat flux is:

′′ = ε σ T 4
qrad (6)

Equation (6) can be rewritten according to:

′′ = ε σ T 3 (T − 0 )
qrad (7)

Equation (7) is similar to a convection equation:

′′ = hrad (T − T f )
qrad (8)

where fluid temperature (Tf) is 0 K and the convection coefficient (hrad) is a function of
temperature:

hrad = ε σ T 3 (9)

FEHT allows the specification of the surface conditions in terms of position (X and Y) as well as
temperature (T); therefore, the radiative surface condition can be modeled as shown in Figure 5.
Figure 5: Extended Surface Conditions Dialog window for Regions 1 and 3.

Region 2 experiences a convection boundary condition with the coolant, but only from one side
(as opposed to the double sided condition assumed by FEHT). The heat transfer from a
differential element in Region 2 is given by:

q = h dA (T − Tc ) (10)

whereas the convection surface condition in FEHT assumes convection from both sides of the
plate and therefore:

q = hF 2 dA (T − Tc ) (11)

where hF is the heat transfer coefficient that should be set in FEHT in order to simulate the
single-sided convection coefficient represented by Eq. (10). Comparing Eqs. (10) and (11) leads
to:

h
hF = (12)
2

Click on Region 2 and select Surface Condition from the Specify menu; specify the surface
conditions as shown in Figure 6.

Figure 6: Extended Surface dialog window for Region 2.

Finally, the boundary conditions along each edge of the computational domain must be specified;
the line of symmetry is adiabatic and the remaining edges are also assumed to be adiabatic.
Generate a reasonable but crude mesh, as shown in Figure 7(a) and then refine it. Note that the
mesh within Region 2 will start relatively refined due to its small width and need not be refined
as much as the mesh in Regions 1 and 3. However, it is possible to refine the mesh in a single
region by selecting that region and then selecting Reduce Mesh from the Draw menu. The result
should be similar to Figure 7(b).

(a) (b)
Figure 7: (a) coarse and (b) refined mesh.

Solve the problem by selecting Calculate from the Run menu; the problem is non-linear due to
the temperature dependent heat transfer coefficient and therefore the solution process will be
iterative. Plot the temperature distribution by selecting Temperature Contours from the View
menu (Figure 8).
Figure 8: Temperature distribution for a 1 cm thick plate.

b.) Export the solution to EES and calculate the total heat transferred from the radiator and the
radiator efficiency (defined as the ratio of the radiator heat transfer to the heat transfer from
the radiator if it were isothermal and at the coolant temperature).

Select Tabular Output from the View menu and then Select All and Save As. Save the data as a
file called ‘1 cm’. Open EES and select Open Lookup Table from the Tables menu; navigate the
file ‘1 cm’ and open it; the solution (the temperature at each node together with the locations of
the nodes) is contained in the lookup table. Using the technique discussed in EXAMPLE 16-1 it
is possible to use the Interpolate2D function to obtain the temperature at an arbitrary x and y
location on the radiator plate.

$UnitSystem SI MASS RAD PA K J


$TABSTOPS 0.2 0.4 0.6 0.8 3.5 in

"INPUTS"
e=1.0 [-] "emittance of panel surface"
a=0.5 [m] "half-width of panel"
b=0.75 [m] "height of panel"
T_f=320 [K] "fluid temperature"
th=0.01 [m] "thickness"
rho=2700 [kg/m^3] "density"
k=200 [W/m-K] "conductivity"

"location on plate"
x=0.1 [m]
y=0.5 [m]
"2-D interpolation from table of nodal data"
T=Interpolate2D('1 cm',X,Y,T,X=x,Y=y) "temperature interpolated from data"

In order to obtain the total heat transferred from the panel it is necessary to integrate the heat flux
over the entire area of the plate. This process can be accomplished manually by dividing the
plate into many small integration areas, calculating the heat flux within each area, and summing
the result. The plate is divided into Nx segments in the x direction, each with width:

a
Δx = (13)
Nx

The x position of each segment is:

xi = ( i − 0.5 ) Δx (14)

The plate is divided into Ny segments in the y direction, each with height:

b
Δy = (15)
Ny

The y position of each segment is:

y j = ( j − 0.5 ) Δy (16)

The corresponding EES code is:

"Double integration manually"


Nx=25 [-] "number of integration areas in x"
Ny=10 [-] "number of integration areas in y"
"size of an integration area"
Dx=a/Nx
Dy=b/Ny
"setup position of x and y areas"
duplicate i=1,Nx
x[i]=(i-0.5)*Dx
end
duplicate j=1,Ny
y[j]=(j-0.5)*Dy
end

The total heat transferred from the plate ( q ) is calculated using the double integral:

b a
q = ∫ ∫ q ′′x , y dx dy (17)
0 0

where the heat flux is:


q ′′x , y = ε σ Tx4, y (18)

The numerical summation that approximates the integration in Eq. (17) is:

N y Nx

q = ∑∑ q ′′xi , y j Δx Δy (19)
j =1 i =1

where

q ′′xi , y j = ε σ Tx4i , y j (20)

The corresponding EES code is:

duplicate i=1,Nx
duplicate j=1,Ny
"2-D interpolation from table of nodal data"
T[i,j]=Interpolate2D('1 cm',X,Y,T,X=x[i],Y=y[j]) "temperature interpolated from data"
q``_dot[i,j]=2*e*sigma#*T[i,j]^4 "heat flux"
end
end
q_dot=sum(q``_dot[1..Nx,1..Ny])*Dx*Dy

Note that the same process can be accomplished using EES’ native Integral command. The
Integral command carries out numerical integration using a more sophisticated algorithm than
simply assuming a constant value over each step.

EES’ Integral command requires 4 arguments and allows a 5th, optional argument; the protocol
for calling the function is:

F=Integral(Integrand,VarName,LowerLimit,UpperLimit,StepSize)

where Integrand is the EES variable or expression that must be integrated, VarName is the
integration variable, and LowerLimit and UpperLimit define the limits of integration. StepSize
is optional and defines the numerical step that is used to accomplish the integration; a small
value of StepSize will lead to more accurate results but take longer to calculate.

The double integral in Eq. (17) can be accomplished by calling the Integral function twice; Eq.
(17) is rewritten as:
b
q = ∫ q′y dy (21)
0

where
a
q ′y = ∫ q ′′x , y dx (22)
0

Equation (22) is evaluated using the Integral command as shown in the EES code below:

"Double integration using EES' Integral command"


"2-D interpolation from table of nodal data"
T=Interpolate2D('1 cm',X,Y,T,X=x,Y=y) "temperature interpolated from data"
q``_dot=2*e*sigma#*T^4 "heat flux"
y=0
q`_dot=INTEGRAL(q``_dot,x,0,a,0.02) "heat transfer per unit length"

Note that the value of y was set as q ′y is a function of y but the value of x is not set as x is the
integration variable. To evaluate Eq. (21), integrate q ′y from y=0 to y=b:

"Double integration using EES' Integral command"


"2-D interpolation from table of nodal data"
T=Interpolate2D('1 cm',X,Y,T,X=x,Y=y) "temperature interpolated from data"
q``_dot=2*e*sigma#*T^4 "heat flux"
q`_dot=INTEGRAL(q``_dot,x,0,a,0.02) "heat transfer per unit length"
q_dot=INTEGRAL(q`_dot,y,0,b,0.02) "total heat transfer rate"

Note that the value of y is no longer set as y is the integration variable in the 2nd integral. The
results of the manual integration should agree with the use of the Integral function; either should
yield 339 W. The radiator efficiency (η) and mass (M) are computed according to:

q
η= (23)
2 a b ε σ T f4

and

M = a b th ρ (24)

c.) Explore the effect of thickness on the radiator efficiency and mass.

The solution procedure described above is repeated for several values of the radiator thickness.
Figure 10 shows the predicted temperature distribution for several values of the thickness. Note
that the same contour levels were selected for each plot by selecting User in the Temperature
Contour Information dialog window and specifying the range from 200 K to 320 K (Figure 9);
this allows the different cases to be compared directly and shows clearly that the reduced
thickness increases the resistance to conduction along the plate and therefore leads to
progressively larger temperature gradients through the plate.
Figure 9: Temperature Contour Information dialog window.

Figure 10: Temperature distribution in the panel for thicknesses of (a) 1.0 cm, (b) 0.75 cm, (c) 0.5 cm, (d) 0.3
cm, (e) 0.2 cm, and (f) 0.1 cm.

Figure 11 illustrates the radiator efficiency and mass as a function of the thickness. As the
thickness is reduced, the efficiency drops but so does the mass; clearly there must be a trade-off
between these effects.
Figure 11: Efficiency and mass as a function of thickness.

Figure 12 shows the efficiency as a function of mass and makes the trade-off clearer; above a
panel with a mass of nominally 2 kg there is a region of diminishing return where additional
mass provides only a small gain in efficiency.

Figure 12: Efficiency as a function of mass.

A complete analysis would require more information then is given; specifically, how should the
radiator performance be compared with its mass to determine a true optimal thickness. Barring
additional constraints related to, for example, structural stability, Figure 12 suggests that any
optimization process will result in a panel that is approximately 0.2 cm thick with a mass of 2 kg
and an efficiency of 60%.
Problem 2.8-1 (2-12 in text): Cryogenic Thermal Switch
There are several cryogenic systems that require a “thermal switch”, a device that can be
used to control the thermal resistance between two objects. One class of thermal switch
is activated mechanically and an attractive method of providing mechanical actuation at
cryogenic temperatures is with a piezoelectric stack; unfortunately, the displacement
provided by a piezoelectric stack is very small, typically on the order of 10 microns. A
company has proposed an innovative design for a thermal switch, shown in Figure P2.8-
1(a). Two blocks are composed of th = 10 μm laminations that are alternately copper (kCu
= 400 W/m-K) and plastic (kp = 0.5 W/m-K). The thickness of each block is L = 2.0 cm
in the direction of the heat flow. One edge of each block is carefully polished and these
edges are pressed together; the contact resistance associated with this joint is Rc′′ = 5x10-4
K-m2/W.

th = 10 μm plastic laminations
kp = 0.5 W/m-K
L = 2 cm L = 2 cm direction of actuation

TH TC

Figure P2.8-1(b)
“on” position “off” position
contact resistance, Rc′′ = 5x10 m -K/W
-4 2

th = 10 μm copper laminations
kCu = 400 W/m-K
Figure P2.8-1(a): Thermal switch in the “on” and “off” positions.

Figure P2.8-1(a) shows the orientation of the two blocks when the switch is in the “on”
position; notice that the copper laminations are aligned with one another in this
configuration which provides a continuous path for heat through high conductivity
copper (with the exception of the contact resistance at the interface). The vertical
location of the right-hand block is shifted by 10 μm to turn the switch "off". In the “off”
position, the copper laminations are aligned with the plastic laminations; therefore, the
heat transfer is inhibited by low conductivity plastic. Figure P2.8-1(b) illustrates a closer
view of half (in the vertical direction) of two adjacent laminations in the “on” and “off”
configurations. Note that the repeating nature of the geometry means that it is sufficient
to analyze a single lamination set and assume that the upper and lower boundaries are
adiabatic.
L = 2 cm L = 2 cm

TH TC

th/2 = 5 μm kp = 0.5 W/m-K


th/2 = 5 μm kCu = 400 W/m-K
Rc′′ = 5x10 m -K/W
-4 2

“on” position

TH TC

“off” position
Figure P2.8-1(b): A single set consisting of half of two adjacent laminations in the “on” and "off”
positions.

The key parameter that characterizes a thermal switch is the resistance ratio (RR) which is
defined as the ratio of the resistance of the switch in the “off” position to its resistance in
the “on” position. The company claims that they can achieve a resistance ratio of more
than 100 for this switch.
a) Estimate upper and lower bounds for the resistance ratio for the proposed thermal
switch using 1-D conduction network approximations. Be sure to draw and clearly
label the resistance networks that are used to provide the estimates. Use your results
to assess the company’s claim of a resistance ratio of 100.

The inputs are entered in EES:

$UnitSystem SI MASS RAD PA K J


$Tabstops 0.2 0.4 0.6 3.5 in

"Inputs"
th = 10 [micron]*convert(micron,m) "thickness of laminations"
k_Cu=400 [W/m-K] "conductivity of copper laminations"
k_p=0.5 [W/m-K] "conductivity of plastic laminations"
L = 2.0 [cm]*convert(cm,m) "lengt of laminations"
R``_c=5e-4 [K-m^2/W] "area specific contact resistance of interface"
W=1 [m] "unit depth into page"

The isothermal and adiabatic models provide two limiting cases. Both result in a 1-D
problem that can be represented using a resistance network. The “adiabatic”
approximation does not allow heat transfer in the y-direction (i.e., perpendicular to the
laminations). The resistance network for the adiabatic approximation is shown in Figure
3(a) for the “on” position and in Figure 3(b) for the “off” position.
(a) (b)
Figure 3: Resistance network for the “adiabatic” approximation in the (a) “on” and (b) “off”
positions.

Note that the parameter Alam in Figure 3 is the cross-sectional area associated a single
lamination:

Alam = W th

where W is the depth into the page (assumed to be 1 m). The resistance associated with
the adiabatic approximation in the “on” state, Rad,on from Figure 3(a) is:

−1
⎡ ⎤

2 ⎢ 1 1 ⎥
Rad ,on = + ⎥
Alam ⎢ 2 L + R′′ 2 L + R′′ ⎥
⎢k c
kCu
c ⎥
⎣ p ⎦

A_lam=W*th "area of a lamination"


R_ad_on=(2/A_lam)*(1/(2*L/k_p+R``_c)+1/(2*L/k_Cu+R``_c))^(-1) "ad. limit in the on position"

which leads to Rad,on = 119.1 K/W. The resistance associated with the adiabatic
approximation in the “off” state, Rad,off from Figure 3(b) is:

−1
⎡ ⎤ L L
⎢ ⎥ + + Rc′′
2 ⎢ 1 1 k p kCu
Rad ,off = + ⎥ =
Alam ⎢ L + L + R′′ L + L + R′′ ⎥ Alam
⎢k c c ⎥
⎣ p kCu kCu k p ⎦

R_ad_off=(L/k_p+L/k_Cu+R``_c)/A_lam "ad. limit in the off position"

The resistance ratio for the adiabatic limit is:

Rad ,off
RRad =
Rad ,on
RR_ad=R_ad_off/R_ad_on "resistance ratio estimated using adiabatic limit"

which leads to RRad = 34.0.

The “isothermal” approximation provides no resistance to heat transfer in the y-direction


(i.e., perpendicular to the laminations). Therefore, the heat can spread without penalty at
the interface and the resistance network for this approximation is shown in Figure 4(a) for
the “on” position and Figure 4(b) “off” position.

(a) (b)
Figure 4: Resistance network for the “isothermal” approximation in the (a) “on” and (b) “off”
positions.

By inspection, the two resistance networks shown in Figures 4(a) and 4(b) will yield the
same resistance and therefore the “isothermal” assumption will predict a resistance ratio,
RRiso, of 1.0. Clearly the company’s claim of a resistance ratio of 100 is not possible as it
does not lie between the two bounding quantities associated with the isothermal and
adiabatic approximations.

b) Provide one or more suggestions for design changes that would improve the
performance of the switch (i.e., increase the resistance ratio). Justify your
suggestions.

The resistance ratio would be increased by any change that causes the two resistance
networks in Figure 3 to be more different. Possible improvements include using
materials with a larger ratio of conductivities (i.e., lower conductivity plastic and, to a
lesser degree, higher conductivity copper) or eliminating the contact resistance. From a
more practical standpoint, any design change that causes the actual the device to behave
more like the adiabatic approximation in Figure 3 than the isothermal approximation in
Figure 4 will improve the performance; for example, increasing the contact resistance
between adjacent laminations would be important.

c.) Sketch the temperature distribution through the two parallel paths associated with the
adiabatic limit of the switch’s operation in the “off” position. Do not worry about the
quantitative details of the sketch, just make sure that the qualitative features are
correct.
Figure 5: Qualitative temperature distribution through paths 1 and 2 consistent with the adiabatic
approximation when the switch is in the “off” position.

d.) Sketch the temperature distribution through the two parallel paths associated with the
adiabatic limit in the “on” position. Again, do not worry about the quantitative details
of your sketch, just make sure that the qualitative features are correct.

Figure 6: Qualitative temperature distribution through paths 1 and 2 consistent with the adiabatic
approximation when the switch is in the “on” position.
Problem 2.8-2 (2-13 in text): Resistance of a Bus Bar
Figure P2.8-2 illustrates a thermal bus bar that has width W = 2 cm (into the page).

H1 = 5 cm L2 = 7 cm

TH = 80°C h = 10 W/m -K
2

T∞ = 20°C

k = 1 W/m-K
L1 = 3 cm H2 = 1 cm
Figure P2.8-2: Thermal bus bar.

The bus bar is made of a material with conductivity k = 1 W/m-K. The middle section is L2 = 7
cm long with thickness H2 = 1 cm. The two ends are each L1 = 3 cm long with thickness H1 = 3
cm. One end of the bar is held at TH = 80ºC and the other is exposed to air at T∞ = 20ºC with h
= 10 W/m2-K.
a.) Use FEHT to predict the rate of heat transfer through the bus bar.

The geometry is entered in FEHT, a mesh is generated, and the boundary conditions and material
properties are specified (Figure 2).

Figure 2: FEHT model.

The mesh is refined several times and then the heat transfer at the convective boundary is
obtained; this leads to q ′ = 5.40 W/m or q = 0.108 W.

b.) Obtain upper and lower bounds for the rate of heat transfer through the bus bar using
appropriately defined resistance approximations.

The inputs are entered in EES:

$UnitSystem SI MASS RAD PA K J


$TABSTOPS 0.2 0.4 0.6 0.8 3.5 in

"Inputs"
L_1=3 [cm]*convert(cm,m) "length of edge pieces"
H_1=5 [cm]*convert(cm,m) "height of edge pieces"
L_2=7 [cm]*convert(cm,m) "length of center piece"
H_2=1 [cm]*convert(cm,m) "height of center piece"
W=2 [cm]*convert(cm,m) "width"
k=1 [W/m-K] "conductivity"
h_bar=10 [W/m^2-K] "heat transfer coefficient at right side"
T_infinity=20 [C] "ambient temperature at right side"
T_H=80 [C] "left side temperature"

An upper bound on the heat transfer rate is obtained using the isothermal limit. The isothermal
limit is calculated according to:

qiso =
(TH − T∞ )
1 L1 L2
+2 +
h H1 W k H1 W k H 2 W






Rconv ,iso Rcond 1,iso Rcond 2,iso

"Isothermal limit"
R_conv_iso=1/(h_bar*W*H_1)
R_cond_1_iso=L_1/(k*W*H_1)
R_cond_2_iso=L_2/(k*W*H_2)
R_total_iso=(R_conv_iso+2*R_cond_1_iso+R_cond_2_iso)
q_dot_iso=(T_H-T_infinity)/R_total_iso

which leads to qiso = 0.118 W.

A lower bound on the heat transfer rate is obtained using the adiabatic limit. The adiabatic limit
is calculated according to:

qad =
(TH − T∞ )
1 L1 L2
+2 +
h H2 W k H2 W k H2 W






Rconv ,ad Rcond 1,ad Rcond 2,ad

"Adiabatic limit"
R_conv_ad=1/(h_bar*W*H_2)
R_cond_1_ad=L_1/(k*W*H_2)
R_cond_2_ad=L_2/(k*W*H_2)
R_total_ad=R_conv_ad+2*R_cond_1_ad+R_cond_2_ad
q_dot_ad=(T_H-T_infinity)/R_total_ad

which leads to qad = 0.052 W.


Problem 2.9-2 (2-14 in text)
A laminated stator is shown in Figure P2.9-2. The stator is composed of laminations with
conductivity klam = 10 W/m-K that are coated with a very thin layer of epoxy with conductivity
kepoxy = 2.0 W/m-K in order to prevent eddy current losses. The laminations are thlam = 0.5 mm
thick and the epoxy coating is 0.1 mm thick (the total amount of epoxy separating each
lamination is thepoxy = 0.2 mm). The inner radius of the laminations is rin= 8.0 mm and the outer
radius of the laminations is ro,lam = 20 mm. The laminations are surrounded by a cylinder of
plastic with conductivity kp = 1.5 W/m-K that has an outer radius of ro,p = 25 mm. The motor
casing surrounds the plastic. The motor casing has an outer radius of ro,c = 35 mm and is
composed of aluminum with conductivity kc = 200 W/m-K.

laminations, thlam = 0.5 mm, klam = 10 W/m-K


epoxy coating, thepoxy = 0.2 mm, kepoxy = 2.0 W/m-K
kp = 1.5 W/m-K
kc = 200 W/m-K

q ′′ = 5x10 W/m T∞ = 20°C


4 2

h = 40 W/m -K
2

Rc′′ = 1x10 K-m /W


-4 2

rin = 8 mm
ro,lam = 20 mm
ro,p = 25 mm
ro,c = 35 mm
Figure P2.9-2: Laminated stator.

A heat flux associated with the windage loss associated with the drag on the shaft is q ′′ = 5x104
W/m2 is imposed on the internal surface of the laminations. The outer surface of the motor is
exposed to air at T∞ = 20°C with a heat transfer coefficient h = 40 W/m2-K. There is a contact
resistance Rc′′ = 1x10-4 K-m2/W between the outer surface of the laminations and the inner
surface of the plastic and the outer surface of the plastic and the inner surface of the motor
housing.
a.) Determine an upper and lower bound for the temperature at the inner surface of the
laminations (Tin).

The inputs are entered in EES:

$UnitSystem SI MASS RAD PA C J


$Tabstops 0.2 0.4 0.6 0.8 3.5

klam=10 [W/m-K] "conductivity of laminations"


kepoxy=2 [W/m-K] "conductivity of epoxy"
kp=1.5 [W/m-K] "conductiity of plastic"
kc=200 [W/m-K] "conductivity of casing"
tlam=0.5 [mm]*convert(mm,m) "thickness of lamination"
tepoxy=0.2 [mm]*convert(mm,m) "thickness of epoxy"
rin=8 [mm]*convert(mm,m) "inner radius of laminations"
rolam=20 [mm]*convert(mm,m) "outer radius of laminations"
rop=25 [mm]*convert(mm,m) "outer radius of plastic"
roc=35 [mm]*convert(mm,m) "outer radius of casing"
qflux=5e4 [W/m^2] "heat flux"
Rc=1e-4 [K-m^2/W] "contact resistance"
Tair=converttemp(C,K,20[C]) "air temperature"
h=40 [W/m^2-K] "heat transfer coefficient"

An upper bound on the temperature allows no heat spreading; therefore, the heat must take two
parallel paths that pass through the iron and the epoxy. The total resistance associated with one
lamination/epoxy pair is:

1 1
=
Rtotal ,ad ⎛r ⎞ ⎛ r ⎞ ⎛r ⎞
ln ⎜ o ,lam ⎟ ln ⎜⎜ o , p ⎟⎟ ln ⎜ o ,c ⎟
⎜ ⎟
⎝ rin ⎠ + Rc′′
+ ⎝
ro ,lam ⎠
+
Rc′′
+ ⎝ ro , p ⎠ + 1
2 π klam thlam 2 π ro ,lam thlam 2 π k p thlam 2 π ro , p thlam 2 π kc thlam 2 π ro, p thlam h
1
+
⎛r ⎞ ⎛ r ⎞ ⎛r ⎞
ln ⎜ o ,lam ⎟ ln ⎜⎜ o , p ⎟⎟ ln ⎜ o ,c ⎟
⎜r ⎟
⎝ rin ⎠ + Rc′′ ⎝ ro ,lam ⎠ Rc′′ ⎝ o, p ⎠ + 1
+ + +
2 π kepoxy thepoxy 2 π ro ,lam thepoxy 2 π k p thepoxy 2 π ro , p thepoxy 2 π kc thepoxy 2 π ro, p thepoxy h
(1)

and the upper bound on the air temperature is:

Tupper = T∞ + Rtotal ,ad q ′′ 2 π rin ( thlam + thepoxy ) (2)

"Upper bound on temperature"


Rlam=ln(rolam/rin)/(2*pi*tlam*klam) "resistance of laminations"
Repoxy=ln(rolam/rin)/(2*pi*tepoxy*kepoxy) "resistance of epoxy"
Rci1=Rc/(2*pi*rolam*tlam) "contact resistance"
Rci2=Rc/(2*pi*rolam*tepoxy)
Rp1=ln(rop/rolam)/(2*pi*tlam*kp) "resistance of plastic"
Rp2=ln(rop/rolam)/(2*pi*tepoxy*kp)
Rco1=Rc/(2*pi*rop*tlam) "contact resistance"
Rco2=Rc/(2*pi*rop*tepoxy)
Rcs1=ln(roc/rop)/(2*pi*tlam*kc) "resistance of casing"
Rcs2=ln(roc/rop)/(2*pi*tepoxy*kc)
Rcv1=1/(h*2*pi*roc*tlam) "convection resistance"
Rcv2=1/(h*2*pi*roc*tepoxy)
1/Rtotal1=1/(Rlam+Rci1+Rp1+Rco1+Rcs1+Rcv1)+1/(Repoxy+Rci2+Rp2+Rco2+Rcs2+Rcv2)
"total resistance"
qdot=qflux*pi*rin*(tlam+tepoxy) "heat transfer"
Tin1=Tair+qdot*Rtotal1 "upper bound on temperature"

which leads to Tupper = 502.7 K.


A lower bound on the temperature allows heat spreading; therefore, the heat must take two
parallel paths that pass through the iron and the epoxy but then a single path to the air. The total
resistance associated with one lamination/epoxy pair is:
−1
⎡ ⎤
⎢ ⎥
⎢ ⎥
⎢ 1 ⎥
Rtotal ,iso =⎢ ⎥
⎢ ln ⎛ ro ,lam ⎞ ⎛r ⎞
ln ⎜ o ,lam ⎟ ⎥
⎢ ⎜ ⎟ ⎥
r ⎝ rin ⎠
⎢ ⎝ in ⎠ + ⎥
⎣⎢ 2 π klam thlam 2 π kepoxy thepoxy ⎦⎥
⎛ r ⎞
ln ⎜⎜ o , p ⎟⎟
Rc′′ ⎝ ro ,lam ⎠
+ + + (3)
2 π ro ,lam ( thlam + thepoxy ) 2 π k p ( thlam + thepoxy )
⎛r ⎞
ln ⎜ o ,c ⎟
Rc′′ ⎜r ⎟
+ ⎝ o, p ⎠ +
1
2 π ro , p ( thlam + thepoxy ) 2 π kc ( thlam + thepoxy ) 2 π ro , p ( thlam + thepoxy ) h

and the lower bound on the air temperature is:

Tlower = T∞ + Rtotal ,iso q ′′ 2 π rin ( thlam + thepoxy ) (4)

"Lower bound on temperature"


Rci=Rc/(2*pi*rolam*(tlam+tepoxy)) "contact resistance"
Rp=ln(rop/rolam)/(2*pi*(tlam+tepoxy)*kp) "plastic resistance"
Rco=Rc/(2*pi*rop*(tlam+tepoxy)) "contact resistance"
Rcs=ln(roc/rop)/(2*pi*(tlam+tepoxy)*kc) "casing resistance"
Rcv=1/(h*2*pi*roc*(tlam+tepoxy)) "convection resistance"
Rtotal2=(1/Rlam+1/Repoxy)^(-1)+Rci+Rp+Rco+Rcs+Rcv "total resistance"
Tin2=Tair+qdot*Rtotal2 "lower bound on temperature"

which leads to Tlower = 491.7 K.

b.) You need to reduce the internal surface temperature of the laminations and there are a few
design options available, including: (1) increase the lamination thickness (up to 0.7 mm), (2)
reduce the epoxy thickness (down to 0.05 mm), (3) increase the epoxy conductivity (up to 2.5
W/m-K), or (4) increase the heat transfer coefficient (up to 100 W/m-K). Which of these
options do you suggest and why?

Examination of the Solution Window (Figure 2) shows that the resistance of the lamination is
much less than the resistance of the epoxy; therefore, the resistance of the lamination dominates
the resistance of the epoxy. The resistance due to convection is much larger than the resistance
of the lamination, contact resistance, or conduction through the plastic and casing. Therefore,
the resistance to convection dominates the problem and the most effective mechanism for
reducing the temperature is to increase the heat transfer coefficient.

Figure 2: Solution Window


Problem 3.1-1
A cylindrical shaped temperature sensor is placed in a chemical reactor in order to monitor its
temperature. The sensor has diameter D = 4.0 mm and length L = 10.0 mm. The density of the
sensor is ρ = 16,600 kg/m3, the specific heat capacity of the sensor is c = 160 J/kg-K, and the
conductivity of the sensor is k = 47 W/m-K. The heat transfer coefficient between the sensor and
the surrounding chemicals is h = 100 W/m2-K. The sensor is initially (at t = 0) in thermal
equilibrium with the chemicals at Tin = 20°C. For t > 0, the temperature of the chemicals rises
linearly with time according to: Tchem = Tin + β t where β = 100 K/min.
a.) Is the lumped capacitance model appropriate for this problem?

The inputs are entered in EES:

$UnitSystem SI MASS RAD PA K J


$TABSTOPS 0.2 0.4 0.6 0.8 3.5 in

"Inputs"
D=4.0 [mm]*convert(mm,m) "sensor diameter"
L=10.0 [mm]*convert(mm,m) "sensor length"
h=100 [W/m^2-K] "heat transfer coefficient"
T_in=converttemp(C,K,20) "sensor initial temperature"
rho=16600 [kg/m^3] "density"
k=47 [W/m-K] "conductivity"
c=160 [J/kg-K] "specific heat capacity"
beta=100 [K/min]*convert(K/min,K/s) "rate of temperature rise"

The volume of the sensor is:

π D2 L
V= (1)
4

and the surface area is:

π D2
As = +π D L (2)
2

The Biot number is therefore:

Vh
Bi = (3)
As k

V=pi*D^2*L/4 "sensor volume"


A_s=2*pi*D^2/4+pi*D*L "sensor surface area"
Bi=V*h/(A_s*k) "Biot number"

which leads to Bi = 0.002; this is sufficiently less than one to justify the lumped capacitance
model.
b.) Derive the ordinary differential equation that governs the temperature of the sensor.

A control volume defined around the temperature sensor includes convection to the chemical and
the rate of energy storage:

dU
0 = qconv + (4)
dt

The rate of convection to the chemical is:

qconv = h As (T − Tchem ) (5)

and the rate of energy storage is:

dU dT
= ρV c (6)
dt dt

Substituting Eqs. (5) and (6) into Eq. (4) leads to:

dT
0 = h As (T − Tchem ) + ρ V c (7)
dt

or:

dT h As h As
+ T= T (8)
dt ρ V c ρ V c chem

Note that the time constant, τ, of the sensor is:

ρV c
τ= (9)
h As

Substituting Eqs. Error! Reference source not found. and (9) into Eq. (8) leads to:

dT T Tin + β t
+ = (10)
dt τ τ

c.) Solve the differential equation for part (b) in order to obtain a solution for the temperature of
the sensor as a function of time.

The solution is broken into its particular (v) and homogeneous (u) parts:

T =u+v (11)
The particular solution must solve:

dv v Tin + β t
+ = (12)
dt τ τ

The right hand side of Eq. (12) is 1st order with time and so we will assume a particular solution
of the form:

v = a + bt (13)

Substituting Eq. (13) into Eq. (12) leads to:

a b T β
b+ + t = in + t (14)
τ τ τ τ

which can only be true if:

a Tin
b+ = (15)
τ τ

and

b β
= (16)
τ τ

Equation (16) leads to:

b=β (17)

Substituting Eq. (17) into Eq. (15) leads to:

a = Tin − β τ (18)

Substituting Eqs. (17) and (18) into Eq. (13) leads to:

v = Tin − β τ + β t (19)

The homogeneous form of the differential equation is:

du u
+ =0 (20)
dt τ

The solution to Eq. (20) is an exponential of the form:


u = C exp ( d t ) (21)

Substituting Eq. (21) into Eq. (20) leads to:

C exp ( d t )
C d exp ( d t ) + =0 (22)
τ

or

1
d =− (23)
τ

Therefore the homogeneous solution is:

⎛ t⎞
u = C exp ⎜ − ⎟ (24)
⎝ τ⎠

The solution is therefore:

⎛ t⎞
T = Tin − β τ + β t + C exp ⎜ − ⎟ (25)
⎝ τ⎠

The solution must satisfy the initial condition:

Tt =0 = Tin (26)

Substituting Eq. (26) into Eq. (25) leads to:

⎛ 0⎞
Tin = Tin − β τ + β 0 + C exp ⎜ − ⎟ (27)
⎝ τ⎠

or

C = βτ (28)

Therefore, the solution is:

⎡ ⎛ t ⎞ ⎤
T = Tin + β t + β τ ⎢ exp ⎜ − ⎟ − 1⎥ (29)
⎣ ⎝ τ ⎠ ⎦

d.) Implement your solution in EES and prepare a plot showing the temperature of the chemicals
and the temperature of the sensor as a function of time for 0 < t < 200 s.
The temperature of the chemicals, Eq. Error! Reference source not found., and the sensor, Eq.
(29), are implemented in EES:

tau=V*rho*c/(h*A_s) "time constant"


T_chem=T_in+beta*time "chemical temperature"
T=T_in+beta*time+beta*tau*(exp(-time/tau)-1) "sensor temperature"

The temperature of the chemicals and the sensor are shown in Figure 1 as a function of time.
Notice that the sensor lags the chemicals by about 1 time constant.

Figure 1: Temperature of the chemical and the sensor as a function of time.


Problem 3.1-2 (3-1 in text): Heating your House
Your cabin is located close to a source of geothermal energy and therefore you have decided to
heat it during the winter by lowering spheres of metal into the ground in the morning so that they
are heated to a uniform temperature, Tgt = 300°C during the day. In the evenings you remove the
spheres and carry them to your cabin; this trip requires about τtravel = 30 minutes. The spheres
are placed in your cabin and give off heat during the night as they cool; the night is τnight = 6 hrs
long. The heat transfer coefficient between a sphere and the surrounding air (outdoor or cabin) is
h = 20 W/m2-K (neglect radiation) and the temperature of the surrounding air (outdoor or cabin)
is Tamb = 10°C. You can carry about M = 100 lbm of metal and are trying to decide what radius
of sphere would work the best. You can carry a lot of spheres (as small as rmin = 5.0 mm) or a
single very large sphere. The thermal conductivity of the metal is k = 80 W/m-K, density ρ =
9000 kg/m3, and c = 1000 J/kg-K.
a.) What is the largest sphere you could use, rmax? That is, what it is the size of a sphere with
mass M = 100 lbm?

The known information is entered in EES:


$UnitSystem SI MASS RAD PA K J
$TABSTOPS 0.2 0.4 0.6 0.8 3.5 in

"Inputs"
T_gt=converttemp(C,K,300) "geothermal temperature"
T_amb=converttemp(C,K,10) "ambient temperature"
h=20 [W/m^2-K] "heat transfer coefficient"
tau_travel=30 [min]*convert(min,s) "duration of travel time"
tau_night=6[hr]*convert(hr,s) "duration of the night"
M=100 [lbm]*convert(lbm,kg) "total mass of metal"
k=80 [W/m-K] "conductivity"
rho=9000 [kg/m^3] "density"
c=1000 [J/kg-K] "specific heat capacity"
r_min=5.0 [mm]*convert(mm,m) "minimum sphere size"

The maximum sphere is determined by the sphere that contains the entire volume of metal that
you can carry:

M
V= (1)
ρ

4 3
V = π rmax (2)
3

V=M/rho "volume of material"


V=4*pi*r_max^3/3 "maximum sphere size"

which leads to rmax= 0.106 m (10.6 cm).


b.) What is the Biot number associated with the maximum size sphere from (a)? Is a lumped
capacitance model of the sphere appropriate for this problem?

The maximum Biot number occurs when we model the largest sphere:

h rmax
Bi = (3)
k

Bi=h*r_max/k "Biot number of maximum size sphere"

which leads to Bi= 0.027 indicating that the lumped capacitance model is sufficient regardless of
the size of the sphere being considered.

c.) Prepare a plot showing the amount of energy released from the metal (all of the spheres)
during τtravel, the period of time that is required to transport the metal back to your cabin, as a
function of sphere radius. Explain the shape of your plot (that is, explain why it increases or
decreases).

The solution to a lumped capacitance subjected to a step change in the surrounding fluid
temperature is given by:

⎛ t⎞
T = Tamb + (Tgt − Tamb ) exp ⎜ − ⎟ (4)
⎝ τ⎠

where τ is the time constant of the spheres:

ρ cV 4 1 ρ cr
τ= = ρ c π r3 = (5)
h As 3 h 4π r 2
3h

The energy released to the ambient air during the travel time (ΔEtravel) is therefore:

(
ΔEtravel = M c Tt =0 − Tt =τ travel ) (6)

These equations are programmed in EES:

"Energy released to ambient"


{r_cm=1 [cm] "radius of sphere (in cm)"}
r=r_cm*convert(cm,m) "radius of sphere"
tau=rho*c*r/(3*h) "time constant of sphere"
T_0=T_gt "temperature at time=0"
T_tau_travel=T_amb+(T_gt-T_amb)*exp(-tau_travel/tau) "temperature at time=tau_travel"
DeltaE_travel=M*c*(T_0-T_tau_travel) "energy released during travel"
DeltaE_travel_MJ=DeltaE_travel*convert(J,MJ) "energy released during travel in MJ"

The energy released as a function of radius is shown in Figure 1.


Figure 1: Energy released to ambient during travel as a function of the radius of the spheres.

For small radii, all of the available energy is released because the time constant of the spheres is
substantially less than travel time; that is, the spheres arrive at the cabin cooled to near ambient.
Larger spheres have longer time constants and tend to retain their energy better during the walk
home.

d.) Prepare a plot showing the amount of energy released from the metal to your cabin during the
night (i.e., from t=τtravel to t=τtravel + τnight) as a function of sphere radius. Explain the shape
of your plot (again, why does it look the way it does?).

The energy released to your cabin (ΔEcabin) is:

(
ΔEcabin = M c Tt =τ travel − Tt =τ travel +τ night ) (7)

"Energy released to cabin"


T_tau_travelplusnight=T_amb+(T_gt-T_amb)*exp(-(tau_travel+tau_night)/tau) "temperature at
time=tau_travel+tau_night"
DeltaE_cabin=M*c*(T_tau_travel-T_tau_travelplusnight) "energy released to cabin"
DeltaE_cabin_MJ=DeltaE_cabin*convert(J,MJ) "energy released to cabin in MJ"

The energy released to your cabin as a function of the radius of the metal sphere is shown in
Figure 2.
Figure 2: Energy released to cabin as a function of the radius of the spheres.

The energy released to the cabin initially increases with radius as the spheres arrive from the
travel hotter. However, at larger radii the energy drops because the metal has such a long time
constant that it ends the night still quite hot. A good radius to use would provide spheres with
time constants that are long with respect to the travel time but small relative to a night.

e.) Prepare a plot showing the efficiency of the heating process, η, as a function of the radius of
the metal. The efficiency is defined as the ratio of the amount of energy provided to your
cabin to the maximum possible amount of energy you could get from your mass of metal.
(Note that this limit occurs if the metal is delivered to the cabin at Tgt and removed at Tamb).

The maximum possible energy is:

ΔEmax = M c (Tgt − Tamb ) (8)

and so the efficiency is:

ΔEcabin
η= (9)
ΔEmax

"Efficiency of heating process"


DeltaE_max=M*c*(T_gt-T_amb) "maximum possible energy"
eta=DeltaE_cabin/DeltaE_max

The efficiency as a function of radius is shown in Figure 3.


Figure 3: Efficiency as a function of the radius of the spheres.
Problem 3.1-3
A household iron is made of 3 pounds of aluminum. It has a surface area of 0.5 ft2 and it is
equipped with a 50 W heater. The iron is initially at 70°F (the temperature of the surroundings)
and the heat transfer coefficient between the iron surface and the surrounding air is 2.0 Btu/hr-
ft2-F. How long will it take for the iron surface to reach 220°F after it is turned on?

The known information is entered in EES:


$UnitSystem SI MASS RAD PA K J
$TABSTOPS 0.2 0.4 0.6 0.8 3.5 in

"known information"
m=3 [lbm]*convert(lbm,kg) "mass of iron"
A=0.5 [ft^2]*convert(ft^2,m^2) "surface area"
q_dot=50 [W] "heater power"
T_init=converttemp(F,K,70 [F]) "initial temperature"
T_final=converttemp(F,K,220 [F]) "target temperature"
h_bar=2.0 [Btu/hr-ft^2-F]*convert(Btu/hr-ft^2-F,W/m^2-K) "heat transfer coefficient"
k=k_('Aluminum',T=T_init) "conductivity of aluminum"
rho=rho_('Aluminum',T=T_init) "density"
c=c_('Aluminum',T=T_init) "specific heat capacity"
T_inf=T_init "ambient temperature"

First, we will determine whether a lumped capacitance model of the iron is appropriate. The
characteristic length for conduction is:

V
Lchar = (1)
As

where As is the surface area of the iron and V is the volume, calculated according to:

M
V= (2)
ρ

where M is the mass of the iron and ρ is the density of aluminum. The Biot number is:

h Lchar
Bi = (3)
k

where k is the conductivity of aluminum and h is the heat transfer coefficient.

L_s=V/A "characteristic length"


V=m/rho "volume of iron"
Bi=h_bar*L_s/k "Biot number"

which leads to Bi = 0.0005, justifying the lumped capacitance assumption. The lumped
capacitance time constant for the iron is:
Mc
τ lumped = (4)
h As

where c is the specific heat capacity of aluminum.

tau_lumped=M*c/(h_bar*A) "lumped capacitance time constant"

The differential equation that governs the behavior of the iron is:

dT T T q
+ = ∞ + (5)
dt τ lumped τ lumped M c

where T∞ is the ambient temperature and q is the heater power. The solution is assumed to
consist of a homogeneous and particular solution:

T = Th + Tp (6)

Substituting Eq. (6) into Eq. (5) leads to:

dTh T dT T T q
+ h + p+ p = ∞ + (7)
dt τ lumped dt τ lumped τ lumped M c



=0 for homogeneous particular differential equation


differential equation

The homogeneous differential equation is:

dTh T
+ h =0 (8)
dt τ lumped

The solution to the homogeneous differential equation can be obtained by separating variables
and integrating:

dTh dt
∫ Th
= −∫
τ lumped
(9)

Carrying out the indefinite integral leads to:

t
ln (Th ) = − + C1 (5)
τ lumped

where C1 is a constant of integration. Equation (5) can be rearranged:


⎛ t ⎞ ⎛ t ⎞
Th = exp ( C1 ) exp ⎜ − ⎟⎟ = C1 exp ⎜⎜ −
*
⎟⎟ (10)
⎜ τ lumped
⎝ τ lumped


C1*
⎝ ⎠ ⎠

where C1* is an undetermined constant that will be referred to as C1:

⎛ t ⎞
Th = C1 exp ⎜ − ⎟⎟ (11)
⎜ τ lumped
⎝ ⎠

The particular solution (Tp) is obtained by identifying any function that satisfies the particular
differential equation:

dTp Tp T∞ q
+ = + (12)
dt τ lumped τ lumped Mc

The particular solution is:

q
Tp = T∞ + τ lumped (13)
Mc

Therefore, the solution is:

⎛ t ⎞ q
T = C1 exp ⎜ − ⎟⎟ + T∞ + τ lumped (14)
⎜ τ lumped M c
⎝ ⎠

The constant of integration is obtained by enforcing the initial condition:

q
Tini = C1 + T∞ + τ lumped (15)
Mc

where Tini is the initial temperature.

T_init=C_1+T_inf+q_dot*tau_lumped/(M*c) "constant of integration"

Equation (14) is used to determine the time requested in the problem statement:

T_final=C_1*exp(-time/tau_lumped)+T_inf+q_dot*tau_lumped/(M*c) "solution"
time_hr=time*convert(s,hr) "time in hr"

which leads to 1.36 hr.


Problem 3.1-4: Energy Detector
A calorimetric detector can be generated using MEMS techniques by suspending a plate
containing a temperature sensor from one or more cantilever beams; for example, as shown in
Figure P3.1-4(a).

Figure P3.1-4(a): MEMS calorimetric sensor.

When energy is applied to the plate (for example, from a reaction or an energetic particle) then
the plate temperature rise is measured. The magnitude of the temperature rise indicates the
amount of energy and the time resolution of the temperature provides a signature of the time
evolution of the process being monitored. One design of a calorimetric detector is shown in
Figure P3.1-4(b).

Tsur = 20°C Wd = 100 μm


ε = 1.0
ac = 8 μm
Wd = 100 μm

Lc = 150 μm
top view
Tsur = 20°C energy
thc = 8 μm y
x

thd = 25 μm
k = 1.5 W/m-K side view
ρ = 750 kg/m3
c = 200 J/kg-K
Figure P3.1-4(b): Design of a MEMS calorimetric detector (top and side views).

The cantilever that supports the detector is Lc = 150 μm long, ac = 8 μm wide, and thc = 8 μm
thick. The cantilever is anchored at a structure that remains always at Tsur = 20ºC. The detector
is square with dimension W = 100 μm and thickness thd = 25 μm. The detector and cantilever
surfaces have emissivity ε = 1 and radiates to surroundings at Tsur = 20ºC. The detector is placed
in a vacuum and therefore there is no convection. The properties of the detector and the
cantilever are k = 1.5 W/m-K, ρ = 750 kg/m3, and c = 200 J/kg-K. Assume that the sensor is
detecting energy from some source. The total energy associated with an event is Qp = 0.1x10-6 J.
You may model the interaction with the detector as occuring over tp = 0.02 s and being uniformly
distributed in time. The detector is initially at Tsur.
a.) Can the cantilever be modeled as a conduction thermal resistance for a plane wall between
the detector and the surrounding structure? Justify your answer.
The inputs are entered in EES:

$UnitSystem SI MASS RAD PA K J


$TABSTOPS 0.2 0.4 0.6 0.8 3.5 in

"Inputs"
th_c=8 [micron]*convert(micron,m) "thickness of cantilever"
a_c=8 [micron]*convert(micron,m) "width of cantilever"
L_c=150 [micron]*convert(micron,m) "total length of cantilever"
k=1.5 [W/m-K] "conductivity"
rho=750 [kg/m^3] "density"
c=200 [J/kg-K] "specific heat capacity"
th_d=25 [micron]*convert(micron,m) "thickness of detector plate"
W_d=100 [micron]*convert(micron,m) "width of detector plate"
e=1 [-] "emissivity"
Q_p=0.1e-6 [J] "particle energy"
t_p=0.02 [s] "particle absorption time"
T_sur=converttemp(C,K,20 [C]) "temperature of surroundings"

There are several resistances and thermal capacitances that should be computed in order to help
understand this problem and therefore develop an appropriate model. The resistance to
conduction along the length of the cantilever is:

Lc
Rcond ,c = (1)
k thc ac

The resistance to radiation from the surface of the cantilever is:

1
Rrad ,c = (2)
4 σ ε Lc 2 ( ac + thc ) Tsur
3

R_cond_c=L_c/(k*th_c*a_c) "conduction resistance through cantilevers "


R_rad_c=1/(4*sigma#*e*L_c*2*(a_c+th_c)*T_sur^3) "radiation resistance from surface of cantilevers"
R_cond_c\R_rad_c=R_cond_c/R_rad_c "ratio of conduction to radiation resistances "

which leads to Rcond,c = 1.56x106 K/W and Rrad,c = 3.65x107. Therefore, the radiation resistance
is approximately 20x larger than the conduction resistance and the edges of the cantilever can, to
first order, be considered adiabatic. The steady-state heat transfer from the detector through the
cantilever can be modeled as being through a plane wall conduction resistance.

The heat capacity of the cantilever is:

Cc = ρ c Lc ac thc (3)

and the heat capacity of the detector is:

Cd = ρ cWd2 thd (4)


C_c=rho*c*L_c*a_c*th_c "heat capacity of cantilevers"
C_d=rho*c*W_d^2*th_d "heat capacity of detector"
C_c\C_d=C_c/C_d "heat capacity ratio"

which leads to Cc = 1.44x10-9 J/K and Cd = 3.75x10-8 J/K. Therefore, the heat capacity of the
detector is more than 20x larger than the heat capacity of the cantilevers. The transient behavior
of the cantilever will not substantially affect the problem.

Based on these calculations, the cantilever can be modeled as a conduction thermal resistance for
a plane wall between the detector and the surrounding structure.

b.) Can the detector be modeled as a lumped capacitance? Justify your answer.

The resistance to radiation from the surface of the detector is:

1
Rrad ,d = (5)
4 σ ε 2Wd2 Tsur
3

R_rad_d=1/(4*sigma#*e*2*W_d^2*T_sur^3) "resistance related to radiation"

which leads to Rrad,d = 8.75x106 K/W. The radiation resistance from the detector is
approximately 5x larger than the conduction resistance through the cantilever. Therefore, while
most of the energy from the detector will pass through the cantilever, radiation is also important.
The total resistance between the detector and the surroundings is:
−1
⎛ 1 1 ⎞
Rtot = ⎜ + ⎟⎟ (6)
⎜R
⎝ rad ,d Rcond ,c ⎠

R_tot=(1/R_cond_c+1/R_rad_d)^(-1) "total thermal resistance from detector to surroundings"

Therefore Rtot = 1.33x106 K/W.

In order to neglect temperature gradients within the detector, it is necessary that energy can be
conducted across the thickness of the detector (in the y-direction) in order to be radiated from the
surface without causing significant temperature gradients. The resistance to conduction in the y-
direction is, approximately:

thd
Rcond ,d , y = (7)
2 k Wd2

and so one Biot number that must be calculated is:

Rcond ,d , y
Bi1 = (8)
Rrad ,d
R_cond_d_y=th_d/(2*k*W_d^2) "internal resistance to conduction across thickness"
Bi1=R_cond_d_y/R_rad_d "Biot number relative to conduction across detector"

which leads to Bi1 = 9.5x10-5. Therefore, internal temperature gradients in the y-direction are
negligible.

In order to neglect temperature gradients within the detector, it is also necessary that energy can
be conducted along the length of the detector (in the x-direction) in order to be conducted into
the cantilever without causing significant temperature gradients. The resistance to conduction in
the x-direction is, approximately:

Wd
Rcond ,d , x = (9)
2 k thd Wd

and so the other Biot number that must be calculated is:

Rcond ,d , x
Bi2 = (10)
Rcond ,c

R_cond_d_x=W_d/(2*W_d*th_d*k) "internal resistance to conduction from center to cantilever"


Bi2=R_cond_d_x/R_cond_c "Biot number relative to conduction through detector"

which leads to Bi2 = 0.0085. Therefore, internal temperature gradients in the x-direction are also
negligible and a lumped capacitance model is justified.

c.) Assume that your answers to (a) and (b) are both yes. Prepare an analytical lumped
capacitance model of the detector during the time that the energy is applied (0 < t < tp) and
after the time that the energy is applied (t > tp).

An energy balance on the detector while energy is applied is shown in Figure P3.1-4(c).

q p q rad
qcond

dU
dt
Figure P3.1-4(c): Energy balance on detector.

The energy balance is:

dU
q p = qrad + qcond + (11)
dt

or
Qp
=
(T1 − Tsur ) + C dT1
(12)
d
tp Rtot dt

where T1 is the temperature during the first part of the process (when energy is applied, from 0 <
t < tp). Equation (12) can be rearranged:

dT1 T T Q
+ 1 = sur + p (13)
dt τ lumped τ lumped Cd t p

where

τ lumped = Rtot Cd (14)

tau_lumped=C_d*R_tot "lumped time constant of the detector"

which leads to τlumped = 0.050 s.

The solution is divided into a homogeneous and particular component:

T1 = T1,h + T1, p (15)

and substituted into Eq. (13):

dT1,h T dT T T Q
+ 1,h + 1, p + 1, p = sur + p (16)
dt τ lumped dt τ lumped τ lumped Cd t p



= 0, homogeneous ODE particular ODE

The particular ODE is solved by a constant,

Qp
T1, p = Tsur + τ lumped (17)
Cd t p

The homogeneous ODE is solved by:

⎛ t ⎞
T1,h = C1 exp ⎜ − ⎟⎟ (18)
⎜ τ lumped
⎝ ⎠

which leads to:

⎛ t ⎞ Qp
T1 = C1 exp ⎜ − ⎟⎟ + Tsur + τ lumped (19)
⎜ τ lumped Cd t p
⎝ ⎠
The initial condition is:

T1,t =0 = Tsur (20)

Substituting Eq. (20) into Eq. (19) leads to:

Qp
Tsur = C1 + Tsur + τ lumped (21)
Cd t p

Therefore:

Qp
C1 = − τ lumped (22)
Cd t p

time=0 [s] "time"


C_1=-Q_p*tau_lumped/(C_d*t_p) "constant of integration for solution during particle detection"
T_1=C_1*exp(-time/tau_lumped)+T_sur+Q_p*tau_lumped/(C_d*t_p) "solution during particle detection"

The differential equation during the time after the energy is applied is:

dT2 T T
+ 2 = sur (23)
dt τ lumped τ lumped

where T2 is the solution for t > tp. The solution is divided into a homogeneous and particular
component and solved, leading to:

⎛ t ⎞
T2 = C2 exp ⎜ − ⎟⎟ + Tsur (24)
⎜ τ lumped
⎝ ⎠

The initial condition for solution T2 is:

T2,t =t p = T1,t =t p (25)

Substituting Eq. (25) into Eq. (24) leads to:

⎛ t ⎞
T1,t =t p = C2 exp ⎜ − p ⎟⎟ + Tsur (26)
⎜ τ lumped
⎝ ⎠

Therefore:
C2 =
(T
1,t = t p − Tsur ) (27)
⎛ t ⎞
exp ⎜ − p ⎟⎟
⎜ τ lumped
⎝ ⎠

T_1_tp=C_1*exp(-t_p/tau_lumped)+T_sur+Q_p*tau_lumped/(C_d*t_p)
"temperature at the end of particle detection"
C_2=(T_1_tp-T_sur)/exp(-t_p/tau_lumped) "constant of integration for solution after particle detection"
T_2=C_2*exp(-time/tau_lumped)+T_sur "solution after particle detection"

The solution is obtained according to:

⎧⎪T1 if t < t p
T =⎨ (28)
⎪⎩T2 if t > t p

T=IF(time,t_p,T_1,T_1,T_2) "solution for all time"


T_C=converttemp(K,C,T) "in C"

d.) Prepare a plot showing the temperature of the detector as a function of time for 0 < t < 0.01 s.

The temperature as a function of time is shown in Figure P3.1-4(d).


22.25

22

21.75
Temperature (°C)

21.5

21.25

21

20.75

20.5

20.25

20
0 0.01 0.02 0.03 0.04 0.05 0.06 0.07 0.08 0.09 0.1
Time (s)
Figure P3.1-4(d): Temperature of detector as a function of time.

e.) If the resolution of the temperature detector is δT = 0.1 K then what is the smallest amount of
energy that you can detect?

In order for the detector to register that an event has happened, the temperature at t = tp must
change by at least δT. The energy is commented out and the temperature at t = tp is specified:

{Q_p=0.1e-6 [J]} "particle energy"


DT_r=0.1 [K] "temperature sensor resolution"
T_1_tp-T_sur=DT_r "energy resolution"

which leads to Qp = 4.6x10-9 J.


f.) For Qp = 0.1x10-6 J, what is the time resolution of the detector (i.e., your detector will
indicate that an event happens some time after the actual event happens - what is this time
lag?).

The time resolution is the time at which the detector changes by δT. The time is commented out
and the temperature is set (note that the lower limit of the variable time must be set to 0).

Q_p=0.1e-6 [J] "particle energy"


{time=0 [s]} "time"
{T_1_tp-T_sur=DT_r "energy resolution"}
T_1-T_sur=DT_r "time resolution"

which leads to a time resolution of 757 μs.


PROBLEM 3.1-5 (3-2 in text): Cooldown of an instrument
An instrument on a spacecraft must be cooled to cryogenic temperatures in order to function.
The instrument has mass M = 0.05 kg and specific heat capacity c = 300 J/kg-K. The surface
area of the instrument is As = 0.02 m2 and the emissivity of its surface is ε = 0.35. The
instrument is exposed to a radiative heat transfer from surroundings at Tsur = 300 K. It is
connected to a cryocooler that can provide qcooler = 5 W. The instrument is exposed to a solar
flux that oscillates according to: qs′′ = q s′′ + Δqs′′ sin (ω t ) where qs′′ = 100 W/m2, Δq s′′ = 100 W/m2,
and ω = 0.02094 rad/s. The initial temperature of the instrument is Tini = 300 K. Assume that the
instrument can be treated as a lumped capacitance.
a.) Develop an analytical model of the cool-down process and implement your model in EES.

The inputs are entered in EES:

$UnitSystem SI MASS RAD PA K J


$TABSTOPS 0.2 0.4 0.6 0.8 3.5 in

"Inputs"
A_s=0.02 [m^2] "surface area"
Mass=0.05 [kg] "mass"
c = 300 [J/kg-K] "specific heat capacity"
e = 0.35 [-] "emissivity"
T_sur = 300 [K] "temperature of surroundings"
T_ini=300 [K] "initial temperature of instrument"
qfs_bar=100 [W/m^2] "average solar flux"
Dqfs=100 [W/m^2] "variation in the solar flux"
omega=0.02094 [rad/s] "angular frequency of solar flux variation"
q_cooler=5 [W] "cooler power"

An energy balance on the instrument leads to:

dU
qs′′ As + qrad = qcooler + (1)
dt

where qrad is the radiation heat transfer to the instrument. Equation (1) can be expressed as:

⎡ q s′′ + Δqs′′ sin (ω t ) ⎤ As +


(Tsur − T ) = q +M c
dT
(2)
⎣ ⎦ Rrad
cooler
dt

where Rrad is the resistance to radiation based on an average temperature of T = 200 K (this
value can be adjusted based on the results).

1
Rrad = (3)
4 As ε σ T 3

T_bar=200 [K] " estimate of the avg. temp. to use for the radiation resistance"
R_rad=1/(4*A_s*e*sigma#*T_bar^3) "radiation resistance"
Rearranging Eq. (2) leads to:

dT
+
T T
= sur +
q s′′ As − qcooler (
Δq ′′ A
+ s s sin (ω t )
) (4)
dt τ lumped τ lumped Mc Mc

where the lumped capacitance time constant is:

τ lumped = Rrad M c (5)

tau_lumped=R_rad*Mass*c "lumped capacitance time constant"

The solution to Eq. (4) is split into a homogeneous and particular component:

T = Th + Tp (6)

Equation (6) is substituted into Eq. (4):

dTh T
+ h +
dTp
+
Tp T
= sur +
qs′′ As − qcooler Δq ′′ A(
+ s s sin (ω t )
) (7)
dt τ lumped dt τ lumped τ lumped Mc Mc



homogeneous ODE particular ODE

The solution to the homogeneous ordinary differential equation is:

⎛ t ⎞
Th = C1 exp ⎜ − ⎟⎟ (8)
⎜ τ lumped
⎝ ⎠

The solution to the particular ordinary differential equation is more difficult.

dTp
+
Tp
=
Tsur
+
( q′′ A − q ) + Δq′′ A sin (ω t )
s s cooler s s
(9)
dt τ lumped τ lumped Mc Mc

The right side of Eq. (9) includes constants and a sine term. Therefore, it is reasonable to expect
that the solution will include these terms and their derivatives (i.e., 0 and a cosine term):

Tp = C2 + C3 sin (ω t ) + C4 cos (ω t ) (10)

Substituting Eq. (10) into Eq. (9) leads to:


C2 C3 C4
C3 ω cos (ω t ) − C4 ω sin (ω t ) + + sin (ω t ) + cos (ω t ) =
τ lumped τ lumped τ lumped
Tsur
+
( q′′ A − q ) + Δq′′ A sin (ω t )
s s cooler s s
(11)

τ lumped Mc Mc

In order for Eq. (11) to be valid at arbitary time, it is necessary that all of the constant, sine, and
cosine terms separately add to zero. The constant terms in Eq. (11) lead to:

C2
=
Tsur
+
( q′′ A − q )
s s cooler
(12)
τ lumped τ lumped Mc

The coefficients multiplying the sine terms in Eq. (11) provide:

C3 Δqs′′ As
−C 4 ω + = (13)
τ lumped Mc

The coefficients multiplying the cosine terms in Eq. (11) provide:

C4
C3 ω + =0 (14)
τ lumped

Equations (12) through (14) lead to three equations in three unknowns that can be solved to
provide the particular solution:

C_2/tau_lumped=T_sur/tau_lumped+(qfs_bar*A_s-q_cooler)/(Mass*c) "equation 1 for particular solution"


-C_4*omega+C_3/tau_lumped=Dqfs*A_s/(Mass*c) "equation 2 for particular solution"
C_3*omega+C_4/tau_lumped=0 "equation 3 for particular solution"

The solution is obtained by substituting Eqs. (10) and (8) into Eq. (6):

⎛ t ⎞
T = C1 exp ⎜ −
⎜ τ lumped ⎟⎟ + C2 + C3 sin (ω t ) + C4 cos (ω t ) (15)
⎝ ⎠

The undetermined constant in the homogeneous solution, C1, can be obtained by enforcing the
initial condition:

Tt =0 = Tini (16)

Substituting Eq. (15) into Eq. (16) leads to:

Tini = C1 + C2 + C4 (17)
T_ini=C_1+C_2+C_4 "initial condition"

The solution, Eq. (15), is programmed in EES:

time=0 [s] "time"


T=C_1*exp(-time/tau_lumped)+C_2+C_3*sin(omega*time)+C_4*cos(omega*time) "solution"

b.) Plot the temperature as a function of time.

Figure P3.1-5 illustrates the temperature of the instrument as a function of time.


300

250
Temperature (K)

200

150

100

50
0 1000 2000 3000 4000 5000
Time (s)
Figure P3.1-5: Temperature of the instrument as a function of time.
Problem 3.1-6 (3-3 in text): Particle ablation
One technique for detecting chemical threats uses a laser to ablate small particles so that they can
subsequently be analyzed using ion mobility spectroscopy. The laser pulse provides energy to a
⎡ ( t − t )2 ⎤
particle according to: qlaser = qmax exp ⎢ − ⎥ where qmax = 0.22 W is the maximum value of
p

⎢ 2 td ⎥
2

⎣ ⎦
the laser power, tp = 2 μs is the time at which the peak laser power occurs, and td = 0.5 μs is a
measure of the duration of the pulse. The particle has radius rp = 5 μm and has properties c =
1500 J/kg-K, k = 2.0 W/m-K, and ρ = 800 kg/m3. The particle is surrounded by air at T∞ = 20°C.
The heat transfer coefficient is h = 60000 W/m2-K. The particle is initially at T∞.
a.) Is a lumped capacitance model of the particle justified?

The inputs are entered in EES:

$UnitSystem SI MASS RAD PA K J


$TABSTOPS 0.2 0.4 0.6 0.8 3.5 in

"Inputs"
r_p=5 [micron]*convert(micron,m) "particle radius"
c=1500 [J/kg-K] "specific heat capacity"
k=2 [W/m-K] "conductivity"
rho=800 [kg/m^3] "density"
h_bar=60000 [W/m^2-K] "heat transfer coefficient"
t_p=2e-6 [s] "time of the pulse"
t_d=0.5e-6 [s] "duration of pulse"
q_dot_max=0.22 [W] "maximum laser power"
T_infinity=converttemp(C,K,20[C]) "air temperature"

The volume of the particle is:

4
V = π rp3 (1)
3

The surface area of the particle is:

As = 4 π rp2 (2)

The Biot number for the particle is:

hV
Bi = (3)
As k

V=4*pi*r_p^3/3 "volume"
A_s=4*pi*r_p^2 "surface area"
Bi=h_bar*V/(A_s*k) "Biot number"

which leads to Bi = 0.05, justifying a lumped capacitance model.


b.) Assume that your answer to (a) is yes; develop an analytical model of the particle using
Maple and EES. Plot the temperature of the particle as a function of time. Overlay on your
plot (on a secondary axis) the laser power.

An energy balance on the particle leads to:

dT
qlaser = h As (T − T∞ ) + C (4)
dt

where

C =V ρ c (5)

C_tot=rho*V*c "total heat capacity"

Equation (4) is rearranged:

dT T T∞ qlaser
+ = + (6)
dt τ τ C

where τ is the lumped capacitance time constant:

C
τ= (7)
h As

tau=C_tot/(h_bar*A_s) "lumped capacitance time constant"

The ordinary differential is entered in Maple:

> restart;
> q_dot_laser:=q_dot_max*exp(-(t-t_p)^2/(2*t_d^2));
⎛ ( t − t_p )2 ⎞
⎜− ⎟
⎜ ⎟
⎜⎜ 2 t_d 2 ⎟⎟⎠

q_dot_laser := q_dot_max e
> ODE:=diff(T(t),t)+T(t)/tau=T_infinity/tau+q_dot_laser/C_tot;
⎛ ( t − t_p )2 ⎞
⎜− ⎟
⎜ ⎟
⎜⎜ 2 t_d 2 ⎟⎟⎠

d T( t ) T_infinity q_dot_max e
ODE := ⎜⎜⎛ T( t ) ⎞⎟⎟ + = +
⎝ dt ⎠ τ τ C_tot

The ordinary differential equation is solved:

> Ts:=dsolve(ODE);

⎛− t ⎞
⎜⎜ ⎟⎟ ⎜
⎝ τ⎠ 1⎜
Ts := T( t ) = e _C1 + ⎜⎜ 2 T_infinity C_tot
2⎝
⎛ 2 τ t − t_d 2 − 2 τ t_p ⎞
⎜− ⎟⎞
⎜ ⎟⎟
⎛ 2 ( −t_d + τ t − τ t_p ) ⎞ ⎜⎜ ⎟⎟
⎠⎟
2 2
⎝ 2 τ
+ q_dot_max π 2 t_d erf⎜⎜ ⎟⎟ e ⎟⎟/C_tot
⎝ 2 t_d τ ⎠ ⎠

The constant of integration is evaluated using the initial condition:

Tt =0 = T∞ (8)

Equation (8) is entered in Maple:

> rhs(eval(Ts,t=0))=T_infinity;
1
_C1 +
2
⎛ −t_d 2 − 2 τ t_p ⎞
⎜− ⎟
⎜ ⎟
⎛ 2 ( −t_d − τ t_p ) ⎞ ⎜⎜ ⎟⎟
2
⎝ 2 τ2 ⎠
2 T_infinity C_tot + q_dot_max π 2 t_d erf⎜⎜ ⎟⎟ e
⎝ 2 t_d τ ⎠
C_tot
= T_infinity

The result is copied and pasted into EES; the variable _C1 is changed to C_1:

"initial condition"
C_1+1/2*(2*T_infinity*C_tot+q_dot_max*Pi^(1/2)*2^(1/2)*t_d*erf(1/2*2^(1/2)*(-t_d^2-tau*t_p)/tau/t_d)*&
exp(-1/2*(-t_d^2-2*tau*t_p)/tau^2))/C_tot = T_infinity

The solution is copied from Maple and pasted into EES:

"solution"
time=0 [s]
T = exp(-1/tau*time)*C_1+1/2*(2*T_infinity*C_tot+q_dot_max*Pi^(1/2)*2^(1/2)*t_d*erf(1/2*2^(1/2)*&
(-t_d^2+tau*time-tau*t_p)/tau/t_d)*exp(-1/2*(2*tau*time-t_d^2-2*tau*t_p)/tau^2))/C_tot
T_C=converttemp(K,C,T)

The laser power is also programmed in EES:

q_dot=q_dot_max*exp(-(time-t_p)^2/(2*t_d^2)) "laser power"

The plot requested in the problem statement is shown in Figure P3.1-6.


800 0.225

temperature 0.2
700
0.175

Laser power (W)


Temperature (K)
600 0.15

0.125
500
0.1

400 0.075
laser power
0.05
300
0.025

200 0
0.0x100 2.0x10-6 4.0x10-6 6.0x10-6 8.0x10-6 1.0x10-5
Time (s)
Figure P3.1-6: Temperature and laser power as a function of time.
Problem 3.1-7
You are building an instrument for measuring the heat transfer coefficient ( h ) between a sphere
and a flowing fluid. The instrument is a spherical temperature sensor with diameter D = 3 mm
that is initially in equilibrium with the fluid at T∞. The sensor has density ρ = 7500 kg/m3,
specific heat capacity c = 820 J/kg-K, and conductivity k = 75 W/m-K. The sensor is heated
with a constant rate of thermal energy generation of g = 0.1 W and the time required for the
sensor temperature to increase by ΔT = 10 K is recorded. The range of the instrument is
expected to be from h = 30 W/m2-K to h = 300 W/m2-K.
a.) Can the sensor be treated as a lumped capacitance? Justify your answer.

The inputs are entered in EES:

$UnitSystem SI MASS RAD PA K J


$TABSTOPS 0.2 0.4 0.6 0.8 3.5 in

"Inputs"
D=3 [mm]*convert(mm,m) "diameter of sensor"
g_dot=0.1 [W] "power applied to sensor"
k=75 [W/m-K] "conductivity"
rho=7500 [kg/m^3] "density"
c=820 [J/kg-K] "specific heat capacity"
DT=10 [K] "temperature rise"

The volume of the sensor is:

3
4 ⎛D⎞
V = π⎜ ⎟ (1)
3 ⎝2⎠

The surface area of the sensor is:

As = π D 2 (2)

The Biot number for the sensor is:

hV
Bi = (3)
As k

The Biot number is highest for the largest heat transfer coefficient:

V=4*pi*(D/2)^3/3 "volume"
A_s=pi*D^2 "surface area"
h_bar=300 [W/m^2-K] "heat transfer coefficient"
Bi=h_bar*V/(A_s*k) "Biot number"

which leads to Bi = 0.002; the sensor can be treated as a lumped capacitance.


b.) Assume that your answer from (a) is yes. Develop an equation that relates the measured time
to the heat transfer coefficient.

An energy balance on the sensor leads to:

dT
g = h As (T − T∞ ) + ρ cV (4)
dt

which can be rearranged:

dT h As g + h As T∞
+ T= (5)
dt ρ cV ρ cV

Equation (4) is broken into its homogeneous and particular parts. The solution to the
homogeneous differential equation is:

⎛ t⎞
Th = C1 exp ⎜ − ⎟ (6)
⎝ τ⎠

where τ is the lumped capacitance time constant:

ρ cV
τ= (7)
h As

tau=rho*V*c/(h_bar*A_s) "lumped capacitance time constant"

The particular solution is:

g
Tp = T∞ + (8)
h As

The solution is therefore:

⎛ t ⎞ g
T = C1 exp ⎜ − ⎟ + T∞ + (9)
⎝ τ ⎠ h As

The undetermined constant is obtained by enforcing the initial condition:

g
Tt =0 = T∞ = C1 + T∞ + (10)
h As

Solving for C1 leads to:


g
C1 = − (11)
h As

Substituting Eq. (11) into Eq. (9) leads to:

g ⎡ ⎛ t ⎞⎤
T= ⎢1 − exp ⎜ − τ ⎟ ⎥ + T∞ (12)
h As ⎣ ⎝ ⎠⎦

The measured time (tmeas) corresponds to the time that the temperature is T∞ + ΔT:

g ⎡ ⎛ tmeas ⎞⎤
ΔT = ⎢1 − exp ⎜ − τ ⎟⎥ (13)
h As ⎣ ⎝ ⎠⎦

DT=g_dot*(1-exp(-time/tau))/(h_bar*A_s) "solution"

c.) Plot the heat transfer coefficient as a function of measured time.

Figure 1 illustrates the heat transfer coefficient as a function of measured time.


400
Heat transfer coefficient (W/m -K)

350
2

300

250

200

150

100

50

0
0 5 10 15 20 25 30 35 40 45 50 55 60
Measured time (s)
Figure 1: Heat transfer coefficient as a function of measured time.

d.) Assume that you can measure time with an uncertainty of δtmeas = 0.1 s. Use your plot from
(c) to estimate the uncertainty of your measurement of the heat transfer coefficient over the
range of the sensor.

The uncertainty in the measurement of the heat transfer coefficient associated with an
uncertainty in the measured time can be estimated according to:

∂h
δh = δ tmeas (14)
∂tmeas
The partial derivative of heat transfer coefficient with respect to measured time is lowest at high
heat transfer coefficient and therefore the uncertainty in the measurement will be lowest when
the sensor is operated at high h . At h = 300 W/m2-K, the uncertainty is approximately:

⎡ W ⎤ ⎡ W ⎤
δ h = 10 ⎢ 2 ⎥ 0.1[s ] = 1 ⎢ 2 ⎥ (15)
⎣m K s⎦ ⎣m K⎦

At h = 30 W/m2-K, the uncertainty increases to approximately:

⎡ W ⎤ ⎡ W ⎤
δ h = 50 ⎢2 ⎥ 0.1[s ] = 5 ⎢ 2 ⎥ (16)
⎣m K s⎦ ⎣m K⎦
Problem 3.1-8
During normal operation, an electrical component experiences a constant rate of ohmic
dissipation g ini = 0.01 W that causes the conversion of electrical to thermal energy. The
component has conductivity k = 10 W/m-K, density ρ = 1000 kg/m3, and specific heat capacity c
= 100 J/kg-K. The surface of the component is cooled by convection with air at T∞ = 20ºC and
heat transfer coefficient h = 10 W/m2-K. The volume of the component is V = 1x10-7 m3 and its
surface area is As = 1x10-5 m2.
a.) Is a lumped capacitance model of the component appropriate? Justify your answer.

The Biot number is given by:

h V 10 W 1x10-7 m3 m-K
Bi = = 2 = 0.01 (1)
As k m -K -5 2
1x10 m 10 W

which is sufficiently less than one to justify a lumped capacitance model.

b.) Assume that your answer to (a) is yes. What is the steady state temperature of the
component?

An energy balance on the component leads to:

g ini = h As (Tss − T∞ ) (2)

Solving for Tss leads to:

g ini 0.01 W m 2 -K
Tss = T∞ + = 20°C + = 120°C (3)
h As 10 W 1x10-5 m 2

c.) Assume that your answer to (a) is yes. What is the lumped capacitance time constant of the
component?

The time constant is:

ρ cV 1000 kg 100 J 1x10-7 m3 m 2 -K


τ= = = 100 s (4)
h As m3 kg-K 10 W 1x10-5 m 2

At time t = 0, the power to the circuit is shut off and the ohmic dissipation in the component
decays to zero according to:

⎛ t ⎞
g = g ini exp ⎜ - ⎟ (5)
⎝ τe ⎠
where τe = 25 s is the electrical time constant of the circuit.

d.) Sketch the temperature of the component as a function of time after the circuit is shut off. Be
sure to indicate where the temperature starts, where it ends up, and approximately how long
it takes to get there.

The ohmic dissipation decays in about 25 s but the time constant of the object is 100 s.
Therefore, the temperature will drop towards T∞ nearly exponentially with a time constant of 100
s as shown in Figure 1.

Generation
0.01 W

0W Time
25 s 100 s 200 s

Temperature
120°C

20°C Time
25 s 100 s 200 s
Figure 1: Sketch of generation and temperature as a function of time.

e.) Derive the ordinary differential equation for this problem.

An energy balance on the component leads to:

⎛ t ⎞ dT
g ini exp ⎜ - ⎟ = h As (T − T∞ ) + ρ cV (6)
⎝ τe ⎠ dt

which can be rearranged:

dT T T∞ g ⎛ t ⎞
+ = + ini exp ⎜ - ⎟ (7)
dt τ τ ρ cV ⎝ τe ⎠

f.) What is the initial condition for this problem?


At time t = 0, the temperature of the component is the steady state temperature determined in (b).

Tt =0 = Tss (8)

g.) Determine the homogeneous solution for the problem.

The homogeneous solution is:

⎛ t⎞
Th = C1 exp ⎜ − ⎟ (9)
⎝ τ⎠

h.) Determine the particular solution for the problem.

The particular solution is assumed to be of the form:

⎛ t ⎞
Tp = a + b exp ⎜ − ⎟ (10)
⎝ τe ⎠

based on inspection of Eq. (7). Substituting Eq. (10) into Eq. (7) leads to:

b ⎛ t ⎞ a b ⎛ t ⎞ T g ⎛ t ⎞
− exp ⎜ − ⎟ + + exp ⎜ − ⎟ = ∞ + ini exp ⎜ - ⎟ (11)
τe ⎝ τe ⎠ τ τ ⎝ τ e ⎠ τ ρ cV ⎝ τe ⎠

Equation (11) requires that:

b b g
− + = ini (12)
τ e τ ρ cV

and

a T∞
= (13)
τ τ

Therefore:

g ini
b= (14)
⎛1 1 ⎞
ρ cV ⎜ − ⎟
⎝τ τe ⎠

and

a = T∞ (15)
Substituting Eqs. (14) and (15) into Eq. (10) leads to:

g ini ⎛ t ⎞
Tp = T∞ + exp ⎜ − ⎟ (16)
⎛1 1 ⎞
ρ cV ⎜ − ⎟ ⎝ τe ⎠
⎝τ τe ⎠

i.) Provide an equation that can be solved for the undetermined constant.

The solution is:

⎛ t⎞ g ini ⎛ t ⎞
T = C1 exp ⎜ − ⎟ + T∞ + exp ⎜ − ⎟ (17)
⎝ τ⎠ ⎛1 1 ⎞ ⎝ τe ⎠
ρ cV ⎜ − ⎟
⎝τ τe ⎠

At t = 0, the initial condition must be recovered:

g ini g ini
T∞ + = C1 + T∞ + (18)
h As ⎛1 1 ⎞
ρ cV ⎜ − ⎟
⎝τ τe ⎠
which provides an equation for C1.

You have decided to develop a numerical solution for this problem. You are going to use
uniform time steps of duration Δt = 10 s.
j.) Write down the equation to take an Euler step from t = 0 to t = Δt. What is the predicted
temperature at t = Δt?

The Euler method for the first time step is:

dT
Tt =Δt = Tt =0 + Δt (19)
dt T =Tt =0 ,t = 0

Substituting Eq. (7) into Eq. (19) leads to

⎡ (T − T ) g ⎛ 0 ⎞⎤
Tt =Δt = Tt =0 + ⎢ ∞ t =0 + ini exp ⎜ - ⎟ ⎥ Δt (20)
⎣ τ ρ cV ⎝ τ e ⎠⎦

Substituting the numbers from the problem statement into Eq. (20) leads to:

⎡ ( 20°C − 120°C ) 0.01 W m3 kg-K ⎤


Tt =Δt = 120°C + ⎢ + -7 3⎥
10s=120°C (21)
⎣ 100s 1000 kg 100 J 1x10 m ⎦
k.) Will your Euler solution be stable? Justify your answer.

Rearranging Eq. (20) leads to:

⎡ Δt ⎤ ⎡ T g ⎛ 0 ⎞⎤
Tt =Δt = Tt =0 ⎢1 − ⎥ + ⎢ ∞ + ini exp ⎜ - ⎟ ⎥ Δt (22)
⎣ τ ⎦ ⎣ τ ρ cV ⎝ τ e ⎠⎦

Examination of Eq. (22) suggests that the critical timestep is:

Δtcrit = τ (23)

Because Δt is less than Δtcrit, the solution will be stable.


Problem 3.1-9
One technique for detecting chemical threats uses a laser to ablate small particles so that they can
subsequently be analyzed using ion mobility spectroscopy. The laser pulse provides energy to a
⎡ ( t − t )2 ⎤
particle according to: qlaser = qmax exp ⎢ − ⎥ where qmax is the maximum value of the laser
p

⎢ 2 td ⎥
2

⎣ ⎦
power, tp is the time at which the peak laser power occurs, and td is a measure of the duration of
the pulse. The particle has heat capacity C and surface area As. The particle is surrounded by air
at T∞ and the heat transfer coefficient is h . The particle is initially at T∞. Assume that a lumped
capacitance model of the particle is justified.
a.) Derive the dimensional governing differential equation and initial condition for the problem.

An energy balance on the particle leads to:

⎡ ( t − t )2 ⎤
qmax exp ⎢ −
p
⎥ = h As (T − T∞ ) + C dT (1)
⎢ 2 td ⎥
2
dt
⎣ ⎦

The initial condition is:

Tt =0 = T∞ (2)

b.) Define a meaningful dimensionless temperature and a meaningful dimensionless time.

A meaningful dimensionless temperature is:

T =
(T − T∞ ) (3)
qmax
h As

where the denominator of Eq. (3) is the temperature rise that would be produced if the maximum
value of the laser power were applied steadily to the particle. A meaningful dimensionless time
is:

t
t = (4)
td

c.) Substitute your definitions from (b) into your problem from (a). Identify any additional
dimensionless parameters required by your problem and explain their physical significance.

Substituting Eqs. (3) and (4) into Eq. (1) leads to:
⎡ ( t t − t )2 ⎤ 
qmax exp ⎢ −
d p
⎥ = h As T qmax + C dT qmax (5)
⎢ 2 td
2
⎥ h As dt h As td
⎣ ⎦

which can be simplified to:

⎡ ⎛ t p ⎞2 ⎤
⎢ ⎜ t − ⎟ ⎥
⎢ td ⎠ ⎥  dT C
exp ⎢ − ⎝ ⎥ = T + dt h A t (6)
2
⎢ ⎥ s d

⎢⎣ ⎥⎦

One additional dimensionless parameter required by the problem is:

tp
tp = (7)
td

which is the ratio of the time of the laser peak to the duration of the laser. The other
dimensionless parameter is:

h As td
β= (8)
C

which is the ratio of the duration of the laser pulse to the time constant of the object. With these
definitions, Eq. (6) becomes:

⎡ ( t − t )2 ⎤
dT
+ β T = β exp ⎢ − ⎥
p
(9)
dt ⎢ 2 ⎥
⎣ ⎦

with the initial condition:

Tt =0 = 0 (10)

d.) Solve your dimensionless problem from (c) - you may want to use Maple. Implement your
solution in EES and plot the dimensionless temperature as a function of dimensionless time
for various values of the other meaningful dimensionless parameter. Explain the shape of
your plot.

The dimensionless ordinary differential equation, Eq. (9), is entered in Maple:

> restart;
> ODE:=diff(T_bar(time_bar),time_bar)+beta*T_bar(time_bar)=beta*exp(-(time_bar-time_bar_p)^2/2);
⎛ ( time_bar − time_bar_p )2 ⎞
⎜− ⎟
⎜⎜ ⎟⎟
d
ODE := ⎜⎜⎛ T_bar( time_bar ) ⎟⎟⎞ + β T_bar( time_bar ) = β e
⎝ 2 ⎠

⎝ dtime_bar ⎠

and solved:

> T_bar_s:=dsolve(ODE);


T_bar_s := T_bar( time_bar ) = ⎜⎜

⎛ time_bar_p 2 ( time_bar_p + β )2 ⎞⎟

⎜⎜ − + ⎟⎟
1 ⎝ 2 2 ⎠ ⎛ 2 time_bar ( time_bar_p + β ) 2 ⎞
β πe 2 erf⎜⎜ − ⎟⎟
2 ⎝ 2 2 ⎠

⎟ ( −β time_bar )
+ _C1 ⎟⎟ e

The initial condition, Eq. (10), is entered:

> IC:=rhs(eval(T_bar_s,time_bar=0))=0;
⎛ time_bar_p 2 ( time_bar_p + β )2 ⎞
⎜− + ⎟
⎜⎜ ⎟⎟
1 ⎝ 2 2 ⎠ ⎛ ( time_bar_p + β ) 2 ⎞
IC := − β π e 2 erf⎜⎜ ⎟⎟ + _C1 = 0
2 ⎝ 2 ⎠

and copied into EES; with some modification (_C1 is changed to C_1), the result is the
undetermined constant:

$TabStops 3 in

beta=1 [-] "ratio of pulse width to time constant"


time_bar_p=2 [-] "dimensionless pulse peak time"
time_bar=10 [-] "dimensionless time"

-1/2*beta*Pi^(1/2)*exp(-1/2*time_bar_p^2+1/2*(time_bar_p+beta)^2)*2^(1/2)*&
erf(1/2*(time_bar_p+beta)*2^(1/2))+C_1 = 0 "initial condition"

and the solution is copied from Maple and also modified:

T_bar=(1/2*beta*Pi^(1/2)*exp(-1/2*time_bar_p^2+1/2*(time_bar_p+beta)^2)*2^(1/2)*&
erf(1/2*2^(1/2)*time_bar-1/2*(time_bar_p+beta)*2^(1/2))+C_1)*exp(-beta*time_bar) "solution"

Figure 1 illustrates the dimensionless temperature as a function of dimensionless time for various
values of the β.
1

0.9

Dimensionless temperature
0.8

0.7

0.6

0.5

0.4 β=5
β=2 β=1
0.3 β = 0.5
β = 0.25
0.2 β = 0.1
0.1

0
0 2 4 6 8 10 12 14 16
Dimensionless time
Figure 1: Dimensionless temperature as a function of dimensionless time for various values of the ratio of the
pulse duraction to the time constant.

Note that very large value of β correspond to the situation where td >> τ where τ is the lumped
capacitance time constant of the object. Therefore, the object faithfully follows the shape of the
laser pulse and achieves the maximum possible temperature rise. If β << 1 then the time
constant is long relative to the pulse width and therefore the object's temperature rise is mitigated
and the temperature drops more slowly back to equilibrium after the pulse is complete.

e.) Plot the maximum dimensionless temperature achieved by the object as a function of
whatever third dimensionless parameter you've identified.

A symbolic expression for the time at which the object achieves its maximum temperature is
determined by setting the time rate of temperature change to zero in Maple:

> rhs(diff(T_bar_s,time_bar))=0;
⎛ time_bar_p 2 ( time_bar_p + β )2 ⎞ ⎛ 2⎞
⎜− ⎟ ⎜ ⎛ 2 time_bar ( time_bar_p + β ) 2 ⎞ ⎟
⎜ −⎜ ⎟⎟ ⎟⎟ ⎛
⎜⎜ + ⎟⎟ ⎜ ⎜ −
( −β time_bar ) ⎜
− ⎜⎜
⎝ 2 2 ⎠ ⎝ ⎝ 2 2 ⎠ ⎠
βe e e

⎛ time_bar_p 2 ( time_bar_p + β )2 ⎞⎟

⎜⎜ − + ⎟⎟
1 ⎝ 2 2 ⎠ ⎛ 2 time_bar ( time_bar_p + β ) 2 ⎞
β πe 2 erf⎜⎜ − ⎟⎟
2 ⎝ 2 2 ⎠

⎟ ( −β time_bar )
+ _C1 ⎟⎟ β e =0

and copied to EES:

"time at max temperature"


beta*exp(-1/2*time_bar_p^2+1/2*(time_bar_p+beta)^2)*exp(-(1/2*2^(1/2)*time_bar-&
1/2*(time_bar_p+beta)*2^(1/2))^2)*exp(-beta*time_bar)-(1/2*beta*Pi^(1/2)*exp(-1/2*time_bar_p^2&
+1/2*(time_bar_p+beta)^2)*2^(1/2)*erf(1/2*2^(1/2)*time_bar-1/2*(time_bar_p+beta)*2^(1/2))+&
C_1)*beta*exp(-beta*time_bar)=0
Figure 2 illustrates the maximum dimensionless temperature as a function of β.
1

Maximum dimensionless temperature


0.9

0.8

0.7

0.6

0.5

0.4

0.3

0.2
0.1 1 8
Ratio of pulse duration to time constant, β
Figure 2: Maximum dimensionless temperature as a function of the ratio of the pulse with to the time
constant.
Problem 3.2-1: Temperature Sensor in an Oscillating Temperature Environment
A temperature sensor is used to measure the temperature in a chemical reactor that operates in a
cyclic fashion; the temperature of the fluid in the reactor varies in an approximately sinusoidal
manner with a mean temperature T f = 320°C, an amplitude ΔTf = 50°C, and a frequency of f =
0.5 Hz. The sensor can be modeled as a sphere of diameter D = 1.0 mm. The sensor is made of
a material with conductivity ks=50 W/m-K, specific heat capacity cs = 150 J/kg-K, and density ρs
= 16000 kg/m3. In order to provide corrosion resistance, the sensor has been coated with a thin
layer of plastic; the coating is tc = 100 μm thick with conductivity kc = 0.2 W/m-K and has
negligible heat capacity relative to the sensor itself. The heat transfer coefficient between the
surface of the sensor and the fluid is h = 500 W/m2-K.
a.) Is a lumped capacitance model of the temperature sensor appropriate? Your lumped
capacitance model will account for the resistance due to conduction through the plastic and
convection with the fluid but neglect any temperature gradients within the sensor itself.

The Biot number is the ratio of the resistance to internal conduction heat transfer to the resistance
to heat transfer from the surface of the object. In this problem, the resistance to heat transfer
from the surface is due to the series combination of convection (Rconv) and the conduction
resistance of the coating (Rc):

1
Rconv = 2
(1)
⎛D ⎞
h 4 π ⎜ + tc ⎟
⎝2 ⎠

⎡2 2 ⎤
⎢ − ⎥
⎣ D ( D + 2 tc ) ⎦
Rc = (2)
4 π kc

The resistance to internal conduction (Rcond) is approximated according to:

Lc
Rcond = (3)
k s As

where As is the surface area of the sensor:

2
⎛D⎞
As = 4 π ⎜ ⎟ (4)
⎝2⎠

and Lc is the conduction length:

V
Lc = (5)
As
where:

4π ⎛ D ⎞
3

V= ⎜ ⎟ (6)
3 ⎝2⎠

The Biot number that characterizes this problem is therefore:

Rcond
Bi = (7)
Rc + Rconv

These equations are programmed in EES:

$UnitSystem SI MASS RAD PA K J


$TABSTOPS 0.2 0.4 0.6 0.8 3.5 in

"Inputs"
T_f_bar=converttemp(C,K,320 [C]) "average temperature of reactor"
DT_f=50*convert(C,K) "amplitude of reactor temperature change"
f=0.5 [Hz] "frequency of reactor temperature change"
D=1.0 [mm]*convert(mm,m) "diameter of sensor"
k_s=50 [W/m-K] "conductivity of sensor material"
c_s=150 [J/kg-K] "specific heat capacity of sensor material"
rho_s=16000 [kg/m^3] "density of sensor material"
t_c=100 [micron]*convert(micron,m) "thickness of coating"
k_c=0.2 [W/m-K] "conductivity of coating"
h=500 [W/m^2-K] "heat transfer coefficient"

R_conv=1/(h*4*pi*(D/2+t_c)^2) "convective resistance"


R_c=(1/(D/2)-1/(D/2+t_c))/(4*pi*k_c) "conduction resistance of coating"
V=4*pi*(D/2)^3/3 "volume of sensor"
A_s=4*pi*(D/2)^2 "surface area of sensor"
Lc=V/A_s "approximate conduction length"
R_cond=Lc/(k_s*A_s) "internal conduction resistance"
Bi=R_cond/(R_conv+R_c) "Biot number"

A Biot number of 0.0018 is calculated, which is sufficiently less than 1 to justify a lumped
capacitance model.

b.) What is the time constant associated with the sensor? Do you expect there to be a substantial
temperature measurement error related to the dynamic response of the sensor?

The time constant (τ) is the product of the resistance to heat transfer and the thermal mass of the
sensor (C):

τ = ( Rc + Rconv ) C (8)

where
C = V ρ s cs (9)

C=V*rho_s*c_s "capacitance of the sensor"


tau=(R_conv+R_c)*C "time constant of the sensor"

The time constant is 0.72 s and the time per cycle is 2 s; these quantities are on the same order
and therefore it is not likely that the temperature sensor will be able to faithfully follow the
reactor temperature.

c.) Develop a numerical model of the temperature response of the sensor using the Crank-
Nicolson technique; assume that the sensor is initially in thermal equilibrium with the reactor
(i.e. Tt =0 = T f ). Plot the temperature of the fluid and the temperature of the sensor as a
function of time for τsim = 8 s.

The temperature sensor is exposed to a sinusoidally varying temperature:

T f = T f + ΔT f sin ( 2 π f t ) (10)

The governing equation for the sensor balances heat transfer against energy storage:

⎡T − T f ⎤⎦
0= ⎣
dT
+C (11)
Rc + Rconv dt

Solving for the state equation (i.e., the equation that provides the time rate of change of the
sensor temperature):

⎡T − T f + ΔT f sin ( 2 π f t ) ⎤⎦
=−⎣
dT
(12)
dt τ

The total simulation time is broken into time steps of duration:

τ sim
Δt = (13)
( M − 1)
and the time associated with each temperature is:

t j = Δt ( j − 1) (14)

tau_sim=8 [s] "simulation time"


M=501 [-] "number of simulation times"
Dt=tau_sim/(M-1) "duration of time step"
duplicate j=1,M
time[j]=Dt*(j-1) "location of each time step"
end

The initial temperature is equal to the average fluid temperature:

T1 = T f (15)

The 1st Crank-Nicolson step is accomplished according to:

⎡T1 − T f + ΔT f sin ( 2 π f t1 ) ⎤⎦
=−⎣
dT
(16)
dt b ,1 τ

⎡T2 − T f + ΔT f sin ( 2 π f t2 ) ⎤⎦
=−⎣
dT
(17)
dt e ,1 τ

so that:

⎛ dT dT ⎞ Δt
T2 = T1 + ⎜⎜ + ⎟⎟ (18)
⎝ dt b ,1 dt e ,1 ⎠ 2

T[1]=T_f_bar "initial temperature"

"take 1st Crank Nicolson step"


dTdt_b[1]=-(T[1]-T_f_bar-DT_f*sin(2*pi*f*time[1]))/tau "dTdt at the beginning of the 1st time step"
dTdt_e[1]=-(T[2]-T_f_bar-DT_f*sin(2*pi*f*time[2]))/tau "dTdt at the end of the 1st time step"
T[2]=T[1]+(dTdt_b[1]+dTdt_e[1])*Dt/2 "step"

The solution is examined and, once it has been debugged, the process of simulating all of the
time steps is automated; the 1 step code above is placed within a duplicate loop where 1 is
replaced by j and 2 is replaced by j+1:

"take all of the Crank Nicolson steps"


duplicate j=1,(M-1)
dTdt_b[j]=-(T[j]-T_f_bar-DT_f*sin(2*pi*f*time[j]))/tau "dTdt at the beginning of the time step"
dTdt_e[j]=-(T[j+1]-T_f_bar-DT_f*sin(2*pi*f*time[j+1]))/tau "dTdt at the end of the time step"
T[j+1]=T[j]+(dTdt_b[j]+dTdt_e[j])*Dt/2 "step"
end

The fluid temperature is also computed at each time step:

"fluid temperature"
duplicate j=1,M
T_f[j]=T_f_bar+DT_f*sin(2*pi*f*time[j])
end

The temperature of the fluid and the sensor are plotted in Figure 1.
Figure 1: Fluid and sensor temperature as a function of time.
Problem 3.2-2: Soldering Iron
Figure P3.2-2 illustrates a schematic model of an industrial soldering iron tip. The soldering iron
can be approximated as a cylinder of metal with radius rout = 5.0 mm and length L = 20 mm. The
metal is carbon steel; assume that the steel has constant density ρ = 7854 kg/m3 and constant
conductivity k = 50.5 W/m-K, but a specific heat capacity that varies with temperature according
to:
⎡ J ⎤ ⎡ J ⎤ ⎡ J ⎤ 2
c = 374.9 ⎢ ⎥ + 0.0992 ⎢ 2⎥
T + 3.596x10-4 ⎢ 3⎥
T
⎣ kg-K ⎦ ⎣ kg-K ⎦ ⎣ kg-K ⎦
The surface of the iron radiates and convects to surroundings that have temperature Tamb = 20°C.
The heat transfer coefficient is h = 10 W/m2-K and the surface of the iron has an emissivity ε =
1.0. The iron is heated electrically by ohmic dissipation; the rate at which electrical energy is
added to the iron is g . The soldering iron can be used once the tip temperature reaches its
operating temperature, Top = 520°C.

rout = 5 mm

ε=1
L = 20 mm g

h = 20 W/m -K
2

Tamb = 20°C

Figure P3.2-2: Soldering iron.


a.) What is the value of generation, g ss , that is required in order sustain a steady-state tip
temperature of Top?

The known information is entered in EES:


$UnitSystem SI MASS RAD PA K J
$TABSTOPS 0.2 0.4 0.6 0.8 3.5 in

"Inputs"
r_out=5.0 [mm]*convert(mm,m) "radius"
L=20 [mm]*convert(mm,m) "length"
k=50.5 [W/m-K] "conductivity"
rho=7854 [kg/m^3] "density"
e=1.0 [-] "emissivity"
h= 10 [W/m^2-K] "heat transfer coefficient"
T_op=converttemp(C,K,520 [C]) "operating temperature"
T_amb=converttemp(C,K,20 [C]) "ambient temperature"

A steady-state energy balance on the tip at its operating temperature provides:

h As (Top − Tamb ) + As ε σ (Top4 − Tamb


4
) = g ss (1)
where As is the surface area of the iron:

As = 2 π rout
2
+ 2 π rout L (2)

A_s=2*pi*r_out^2+2*pi*r_out*L "surface area"


g_dot_ss=h*A_s*(T_op-T_amb)+A_s*e*sigma#*(T_op^4-T_amb^4)
"steady-state generation"

which leads to g ss = 21.2 W.

b.) Can the soldering iron be treated as a lumped capacitance? Justify your answer.

The Biot number for this problem is defined as:

Rcond
Bi = −1
(3)
⎡ 1 1 ⎤
⎢R + R ⎥
⎣ conv rad ⎦

where Rcond is the resistance to conduction from within the iron to its surface, Rconv is the
resistance due to convection from the surface, and Rrad,min is the minimum resistance due to
radiation from the surface. The Biot number will be largest (and therefore the lumped
capacitance model least justified) when Rrad is smallest and therefore the minimum radiation
resistance should be used; this occurs when the surface temperature, Ts, is largest. Therefore,
we’ll estimate Rrad,min at Ts = Top, which is presumably near the maximum value of the surface
temperature.

1
Rconv = (4)
h As

1
Rrad ,min = (5)
ε σ (T + T
2
op
2
amb )(T op + Tamb ) As

The conduction resistance can be estimated approximately by:

Lc
Rcond ≈ (6)
k As

where Lc is:

V
Lc = (7)
As

and V is the volume of the iron:


V = π rout
2
L (8)

V=pi*r_out^2*L "volume"
L_c=V/A_s "conduction length"
R_cond=L_c/(k*A_s) "conduction resistance"
R_conv=1/(h*A_s) "convection resistance"
R_rad_min=1/(sigma#*e*(T_op^2+T_amb^2)*(T_op+T_amb)*A_s)
"radiation resistance"
Biot=R_cond/(1/R_conv+1/R_rad_min)^(-1) "Biot number"

The result is Bi = 0.002 which indicates that the iron can be treated as a lumped capacitance.

c.) What is the approximate time constant of the soldering iron? That is, about how long would
someone have to wait for it to warm up if it were activated with a constant rate of generation?

The time constant is the product of the heat capacity of the object and its total resistance to its
surroundings:

−1
⎡ 1 1 ⎤
τ =⎢ + ⎥ V ρ cavg (9)
⎢⎣ Rconv Rrad ,avg ⎥⎦

where the radiation resistance and specific heat capacity should computed at the average
temperature:

Tamb + Top
Tavg = (10)
2

1
Rrad ,avg = (11)
ε σ (T2
avg +T 2
amb )(T avg + Tamb ) As

T_avg=(T_op+T_amb)/2 "average temperature"


c_avg=374.9 [J/kg-K]+0.0992 [J/kg-K^2]*T_avg+3.596e-4 [J/kg-K^3]*T_avg^2
"specific heat capacity at the average temperature"
R_rad_avg=1/(sigma#*e*(T_avg^2+T_amb^2)*(T_avg+T_amb)*A_s)
"radiation resistance at the average temperature"
tau=(1/R_conv+1/R_rad_avg)^(-1)*V*c_avg*rho "approximate time constant"

which leads to τ = 300 s (5 minutes).

d.) Develop a numerical model using the Crank-Nicolson technique that is implemented in EES
and can predict the temperature of the soldering iron as a function of time. Assume that it is
activated at ambient temperature with the generation that was calculated in (b).

The governing differential equation for the soldering iron is obtained from an energy balance:
+ h As (T − Tamb ) + ε σ As (T 4 − Tamb )
dT
g ss = V ρ c (T ) 4
(12)
dt

and so the rate of temperature change is:

⎡ g ss − h As (T − Tamb ) − ε σ As (T 4 − Tamb )⎤⎦


dT 1
= ⎣
4
(13)
dt V ρ c (T )

The total simulation time, τsim, is taken to be 3x the time constant computed in (c) and the
simulation is broken into time steps of equal duration:

τ sim
Δt = (14)
( M − 1)

t j = Δt ( j − 1) for j = 1..M (15)

tau_sim=900 [s] "simulation time"


M=101 [-] "number of time steps"
Dt=tau_sim/(M-1)
duplicate j=1,M
time[j]=Dt*(j-1)
end

A function is defined which returns the value of the specific heat capacity.

function c(T)
c = 374.9 [J/kg-K]+0.0992 [J/kg-K^2]*T+3.596e-4 [J/kg-K^3]*T^2
end

The initial temperature of the soldering iron is ambient temperature:

T1 = Tamb (16)

The state equation, Eq. (13), is integrated in time using the Crank-Nicolson technique. The value
of the time rate of change at the beginning and end of any time step is estimated:

⎡ g ss − h As (T j − Tamb ) − ε σ As (T j4 − Tamb )⎤⎦


dT 1
= 4
(17)
dt T =T j ,t = t j V ρ c (T j ) ⎣

⎡ g ss − h As (T j +1 − Tamb ) − ε σ As (T j4+1 − Tamb )⎤⎦


dT 1
= 4
(18)
dt T =T j +1 ,t =t j +1 V ρ c (T j +1 ) ⎣

and their average is used to integrate through the time step:


⎛ dT dT ⎞ Δt
T j +1 = T j + ⎜ + ⎟ (19)
⎜ dt dt ⎟
T =T j +1 ,t = t j +1 ⎠ 2
⎝ T =T j ,t =t j

T[1]=T_amb
duplicate j=1,(M-1)
dTdt_b[j]=(g_dot_ss-h*A_s*(T[j]-T_amb)-e*sigma#*A_s*(T[j]^4-T_amb^4))/(V*rho*c(T[j]))
dTdt_e[j]=(g_dot_ss-h*A_s*(T[j+1]-T_amb)-e*sigma#*A_s*(T[j+1]^4-T_amb^4))/(V*rho*c(T[j+1]))
T[j+1]=T[j]+(dTdt_b[j]+dTdt_e[j])*Dt/2
end

The temperature as a function of time is shown in Figure 2.

Figure 2: Temperature of the soldering iron as a function of time.

e.) Develop an analytical model that can predict the temperature of the soldering iron as a
function of time in the limit that radiation is neglected and the heat capacity is constant (and
equal to its value at the average of the ambient and the operating temperatures). Show that
your numerical model from part (d) agrees with the analytical model in this limit (no
radiation and constant heat capacity) by overlaying a plot of the analytical solution on top of
the numerical solution (suitably modified so that it is consistent with your analytical
solution).

The governing differential equation for the soldering iron under these limiting conditions is
obtained from the energy balance:
dT
g ss = V ρ cavg + h As (T − Tamb ) (20)
dt

Rearranging:

dT h As h As g ss
+ T= Tamb + (21)
dt V ρ cavg V ρ cavg V ρ cavg

The homogeneous solution to this equation is:

⎛ h As ⎞
u = C1 exp ⎜ − t⎟ (22)
⎜ V ρ cavg ⎟
⎝ ⎠

and the particular solution is:

g ss
v = Tamb + (23)
h As

So the total solution is:

⎛ h As ⎞ g
T = C1 exp ⎜ − t ⎟ + Tamb + ss (24)
⎜ V ρ cavg ⎟ h As
⎝ ⎠

The constant is determined using the initial condition:

g ss
Tamb = C1 + Tamb + (25)
h As

so that:

g ss
C1 = − (26)
h As

and

g ss ⎛ h As ⎞ g
T =− exp ⎜ − t ⎟ + Tamb + ss (27)
⎜ ⎟
h As ⎝ V ρ cavg ⎠ h As

The solution is implemented in EES.

"analytical model"
duplicate j=1,M
T_an[j]=-g_dot_ss*exp(-h*A_s*time[j]/(V*rho*c_avg))/(h*A_s)+T_amb+g_dot_ss/(h*A_s)
end

The numerical model is operated in this limit by setting emissivity to 0 and by modifying the
function c(T) so that it returns only the average value of the specific heat capacity (534.9 J/kg-K)
regardless of the temperature provided. The analytical and numerical solutions are shown in
Figure 3.

Figure 3: Temperature of the soldering iron as a function of time predicted by the analytical and
numerical models in the limit of no radiation and a constant specific heat capacity.

Note that the value of g ss will be somewhat different from the value calculated in (a) since
radiation is neglected.

f.) Implement your numerical solution in MATLAB using the ode45 solver. Show that the
result agrees with the result of your numerical model in EES from part (d). Include radiation
and temperature dependent specific heat capacity for this part.

The solution is implemented in a function; the input to the function is the simulation time while
the output is the vector of temperatures and associated times. The remaining inputs for the
problem are set in the function:

function[time,T]=PROBLEM3p2_2(tau_sim)
%Inputs
r_out=0.005; %radius (m)
L=0.02; %length (m)
rho=7854; %density (kg/m^3)
k=50.5; %conductivity (W/m-K)
e=1.0; %emissivity (-)
h= 10; %heat transfer coefficient (W/m^2-K)
T_op=793.2; %operating temperature (K)
T_amb=293.2; %ambient temperature (K)
end

A function is defined that returns the value of the specific heat capacity:

function[c]=cf(T)
c=374.9+0.0992*T+3.596e-4*T^2;
end

A second function is defined while computes the instantaneous rate of the temperature change:

function[dTdt]=dTdt_P3p2_2(time,T,r_out,L,rho,e,h,T_op,T_amb)
sigma=5.67e-8; %Stefan-Boltzman constant
A_s=2*pi*r_out^2+2*pi*r_out*L; %surface area
g_dot_ss=h*A_s*(T_op-T_amb)+A_s*e*sigma*(T_op^4-T_amb^4);
%steady state generation
V=pi*r_out^2*L; %volume
dTdt=(g_dot_ss-h*A_s*(T-T_amb)-e*sigma*A_s*(T^4-T_amb^4))/(V*rho*cf(T));
end

The relative tolerance for the solution is set using the odeset command:

OPTIONS=odeset('RelTol',1e-6);

and the ode45 solver is used to integrate the function dTdt_P3p2_2 through time:

[time,T]=ode15s(@(time,T) dTdt_P3p2_2(time,T,r_out,L,rho,e,h,T_op,...
T_amb),[0,tau_sim],T_amb, OPTIONS);

The function PROBLEM3p2_2 is run from the command line for tau_sim = 900 s; the results are
overlaid on Figure 2.

g.) You want to evaluate methods for accelerating the soldering iron’s heat up process. Assume
that the maximum heater power that can be applied is 100 W and that you can sense the tip
temperature and control the power based upon the instantaneous tip temperature. Modify
your model from (f) so that the power applied to the tip obeys a simple proportional control
algorithm:
g = g ss + K p (Top − T ) with MAX ( g ) = 100 W and MIN ( g ) = 0 W
where Kp = 0.5 W/K is the proportional controller gain and g ss is the generation required at
steady state, computed in part (a). Overlay a plot of the temperature vs time attained using
this controlled soldering iron onto the original, uncontrolled response that was predicted in
parts (d) and (f).

The only portion of the code that must be modified lies within the function dTdt_P3p2_2; the
generation is no longer a constant value of g ss but instead is computed according to the control
law provided by Eq. Error! Reference source not found.:

function[dTdt]=dTdt_P3p2_2(time,T,r_out,L,rho,e,h,T_op,T_amb)
sigma=5.67e-8; %Stefan-Boltzman constant
A_s=2*pi*r_out^2+2*pi*r_out*L; %surface area
g_dot_ss=h*A_s*(T_op-T_amb)+A_s*e*sigma*(T_op^4-T_amb^4); %steady state
generation
Kp=0.5;
g_dot=g_dot_ss+Kp*(T_op-T); %power commanded by controller
if (g_dot<0)
g_dot=0;
end
if (g_dot>100)
g_dot=100;
end
g_dot
V=pi*r_out^2*L; %volume
dTdt=(g_dot-h*A_s*(T-T_amb)-e*sigma*A_s*(T^4-T_amb^4))/(V*rho*cf(T));
% dTdt=(g_dot_ss-h*A_s*(T-T_amb)-e*sigma*A_s*(T^4-T_amb^4))/(V*rho*cf(T));
end

The temperature as a function of time for the controlled and uncontrolled soldering iron is shown
in Figure 4.

Figure 4: Temperature as a function of time for the controlled and uncontrolled soldering iron.
Problem 3.2-3: Laser Target
A disk shaped piece of material is used as the target of a laser, as shown in Figure P3.2-3. The
laser target diameter is D and its thickness is b. The target is made of a material with density ρ,
conductivity k, and specific heat capacity c. The front side of the target is exposed to a time
′′ ( t ) which is given by the function:
varying heat flux from a laser, qlaser

⎛ t ⎞
′′ ( t ) = At 2 exp ⎜ −
qlaser
⎜ t pulse ⎟⎟
⎝ ⎠

where A and tpulse are constants and t is time. The back side of the target is cooled by exposure to
fluid at Tf with heat transfer coefficient h . You may neglect radiation from the target and
convection from the front side of the target.

laser target: ρ, k, c

′′ (t )
qlaser h ,T f

D initial temperature, Tf
Figure P3.2-3: Laser target

a.) How would you determine whether a lumped capacitance model of the laser target is
appropriate? Write down the precise equation that you would need in terms of symbols
given in the problem statement.

The Biot number for the target compares the internal resistance to conduction to the external
resistance to convection:

Rcond b h
Bi = = (1)
Rconv k

Anything similar to Eq. (1) would be accepted.

b.) Assume that a lumped capacitance model is appropriate for the laser target. Derive the
ordinary differential equation that governs the temperature of the laser target (T). You
should end up with an equation for the time rate of change of the target temperature in terms
of the symbols listed in the problem statement.

An energy balance on the laser target leads to:


π D2 π D 2 dT π D2
′′ ( t )
qlaser
4
=ρc
4
b
dt
+h
4
(T − T )
f (2)

or

′′ ( t )
dT qlaser
dt
=
ρ cb

h
ρ cb
(T − T f ) (3)

Substituting Eq. Error! Reference source not found. into Eq. (3) leads to:

At 2 ⎛ t ⎞
⎜ t pulse ⎟ ρ c b ( )
dT h
= exp ⎜ − ⎟ − T − Tf (4)
dt ρ c b ⎝ ⎠

c.) You want to simulate the problem using a numerical method based on the Euler technique.
Write the equation that you would use to move through the first time step (i.e., from T[1] at t
= 0 to T[2] at t = Δt). The duration of the time step is Δt.

⎡ A 02 ⎛ 0 ⎞ ⎤
⎟ ρ cb ( f ) Δt
h
T [2] = T [1] + ⎢ exp ⎜ −
⎜ t ⎟ − T [1] − T ⎥
⎢⎣ ρ c b ⎝ pulse ⎠ ⎥⎦

d.) Will the Euler step in part (c) become unstable if Δt is very large? If so then what is the
critical time step; that is, how large of a time step can you take before the solution becomes
unstable?

Collecting the terms on the right hand side related to T[1]:

⎛ h ⎞ ⎡ A 02 ⎛ 0 ⎞ h ⎤
T [2] = T [1] ⎜1 − Δt ⎟ + ⎢ exp ⎜ − ⎟ + T f ⎥ Δt
⎝ ρ c b ⎠ ⎢⎣ ρ c b ⎜ t ⎟
⎝ pulse ⎠ ρ c b ⎥⎦

The solution goes unstable when the coefficient multiplying T[1] becomes negative:

ρ cb
Δtcrit =
h

e.) You want to simulate the problem using a numerical method based on the fully implicit
technique. Write the equation that you would use to move through the first time step (i.e.,
from T[1] at t = 0 to T[2] at t = Δt).
⎡ A Δt 2 ⎛ Δt ⎞ ⎤
⎟ ρ cb ( [ ] f ) ⎥ Δt
h
T [2] = T [1] + ⎢ exp ⎜ − ⎟ − T 2 −T
⎢⎣ ρ c b ⎜ t ⎥⎦
⎝ pulse ⎠

f.) Will the implicit step in part (e) become unstable if Δt is very large? If so then what is the
critical time step; that is, how large of a time step can you take before the solution becomes
unstable?

The implicit solution will not become unstable.


Problem 3.2-4 (3-4 in text): Cooling down an instrument
Reconsider Problem 3.1-5 (3-2 in text) using a numerical model. The cooling power of the
cryocooler is not constant but is a function of temperature:
⎧ ⎡W⎤ 2 ⎡ W ⎤
⎪−4.995 [ W ] + 0.1013 T ⎢ ⎥ − 0.0001974 T ⎢ 2 ⎥ if T > 55.26 K
qcooler = ⎨ ⎣K⎦ ⎣K ⎦
⎪0 if T < 55.26 K

where T is the temperature of the instrument.


a.) Develop a numerical model in EES using Heun's method. Plot the temperature of the
instrument as a function of time for 2000 s after the cryocooler is activated.

The inputs are entered in EES:

$UnitSystem SI MASS RAD PA K J


$TABSTOPS 0.2 0.4 0.6 0.8 3.5 in

"Inputs"
A_s=0.02 [m^2] "surface area"
Mass=0.05 [kg] "mass"
c = 300 [J/kg-K] "specific heat capacity"
e = 0.35 [-] "emissivity"
T_sur = 300 [K] "temperature of surroundings"
T_ini=300 [K] "initial temperature of instrument"

Functions are defined for the solar flux and load curve of the cryocooler:

function q_dot_load(T)
q_dot_load=-4.995 [W] + 0.1013 [W/K]*T - 0.0001974 [W/K^2]*T^2
end

function qf_dot_solar(time)
qf_dot_solar=100 [W/m^2]+100[W/m^2]*sin(0.02094 [rad/s]*time)
end

The state equation for the system is derived with an energy balance on the instrument:

As qs′′ ( t ) +
(Tsur − T ) = q dT
cooler (T ) + M c (1)
Rrad dt

where

1
Rrad = (2)
As ε σ (T + Tsur
22
) (T + Tsur )
Rearranging Eq. (1) leads to:
dT 1 ⎡
⎢ As qs′′ ( t ) +
(Tsur − T ) − q T ⎤⎥
= cooler ( ) (3)
dt M c ⎢ Rrad ⎥⎦

The computational domain is divided into M steps; the time associated with each time step and
duration of the time step are:

tsim
Δt = (4)
( M − 1)

t j = tsim
( j − 1) for j = 1..M (5)
( M − 1)
t_sim=2000 [s] "simulation time"
M=101 [-] "number of steps"
DELTAtime=t_sim/(M-1) "duration of time step"
duplicate j=1,M
time[j]=(j-1)*t_sim/(M-1)
end

The initial temperature is set:

T[1]=T_ini

For Heun's method, the predictor step computes the temperature rate of change at the beginning
of the time step:

1 ⎡
⎢ As q s′′ ( t j ) +
(Tsur − T j ) − q T ⎤⎥ for j = 1.. M − 1
cooler ( j )
dT
= ( ) (6)
dt Mc⎢ Rrad , j ⎥
j
⎣ ⎦

where

1
Rrad , j = (7)
As ε σ (T + Tsur
j
2 2
) (T j + Tsur )
duplicate j=1,(M-1)
R_rad[j]=1/(A_s*e*sigma#*(T_sur^2+T[j]^2)*(T_sur+T[j]))
dTdt[j]=(qf_dot_solar(time[j])*A_s+(T_sur-T[j])/R_rad[j]-q_dot_load(T[j]))/(Mass*c)

The predictor step is:

dT
Tˆj +1 = T j + Δt for j = 1.. ( M − 1) (8)
dt j
T_hat[j+1]=T[j]+dTdt[j]*DELTAtime "predictor step"

The corrector step computes the temperature rate of change at the end of the time step:

m 1 ⎡⎢ Tsur − Tˆj +1 ( ) ⎤
dT
dt
=
Mc⎢
As qs′′ ( t j +1 ) +
Rˆ rad , j +1
− qcooler Tˆj +1 ( ) ⎥ for j = 1.. ( M − 1)

(9)
j +1 ⎣ ⎦

where

1
Rˆ rad , j +1 = (10)
(
As ε σ Tˆ 2
j +1 + Tsur
2
)(
Tˆj +1 + Tsur )
R_rad_hat[j+1]=1/(A_s*e*sigma#*(T_sur^2+T_hat[j+1]^2)*(T_sur+T_hat[j+1]))
dTdt_hat[j+1]=(qf_dot_solar(time[j+1])*A_s+(T_sur-T_hat[j+1])/R_rad_hat[j+1]-...
q_dot_load(T_hat[j+1]))/(Mass*c)

The corrector step is:

⎛ dT m
dT ⎞ Δt
T j +1 = T j + ⎜ + ⎟ for j = 1.. ( M − 1) (11)
⎜ dt j dt ⎟ 2
⎝ j +1 ⎠

T[j+1]=T[j]+(dTdt[j]+dTdt_hat[j+1])*DELTAtime/2 "corrector step"


end

Figure P3.2-4-2 illustrates the temperature as a function of time.


300

280

260

240
Temperature (K)

220

200

180

160

140

120
0 250 500 750 1000 1250 1500 1750 2000
Time (s)
Figure P3.2-4-2: Temperature as a function of time, predicted using Heun's method.

b.) Verify that your model from (a) limits to the analytical solution developed in Problem 3.1-5
(3-2) in the limit that the cryocooler power is constant and radiation is treated using a
constant, approximate radiation resistance. Overlay on the same plot the temperature of the
instrument as a function of time predicted by the analytical and numerical models.
In order to reach the limiting conditions used in Problem 3.1-5, the cryocooler load curve is
modified:

function q_dot_load(T)
{q_dot_load=-4.995 [W] + 0.1013 [W/K]*T - 0.0001974 [W/K^2]*T^2}
q_dot_load = 5 [W]
end

and the approximate resistance to radiation is computed:

T_bar=200 [K] "approximate average temperature"


R_rad_app=1/(4*e*sigma#*A_s*T_bar^3) "approximate radiation resistance"

and used in place of the actual resistance:

T[1]=T_ini
duplicate j=1,(M-1)
R_rad[j]=R_rad_app{1/(A_s*e*sigma#*(T_sur^2+T[j]^2)*(T_sur+T[j]))}
dTdt[j]=(qf_dot_solar(time[j])*A_s+(T_sur-T[j])/R_rad[j]-q_dot_load(T[j]))/(Mass*c)
T_hat[j+1]=T[j]+dTdt[j]*DELTAtime
R_rad_hat[j+1]=R_rad_app{1/(A_s*e*sigma#*(T_sur^2+T_hat[j+1]^2)*(T_sur+T_hat[j+1]))}
dTdt_hat[j+1]=(qf_dot_solar(time[j+1])*A_s+(T_sur-T_hat[j+1])/R_rad_hat[j+1]-...
q_dot_load(T_hat[j+1]))/(Mass*c)
T[j+1]=T[j]+dTdt_hat[j+1]*DELTAtime
end

Figure P3.2-4-3 illustrates the numerical and analytical results and shows nearly exact
agreement.
300
analytical solution from P3.1-5
numerical solution
250
Temperature (K)

200

150

100

50
0 1000 2000 3000 4000 5000
Time (s)
Figure P3.2-4-3: Temperature as a function of time predicted by the analytical model from P3.1-5 and the
numerical model (1 of every 10 timesteps are shown) under the limiting conditions associated with P3.1-5.

c.) Develop a numerical model in EES using the Integral command. Plot the temperature of the
instrument as a function of time for 2000 s after the cryocooler is activated.

The code from (b) is commented out. The time rate of change, Eq. (3), is evaluated:

dTdt=(qf_dot_solar(time)*A_s+(T_sur-T)/R_rad-q_dot_load(T))/(Mass*c)
R_rad=1/(A_s*e*sigma#*(T_sur^2+T^2)*(T_sur+T))

The Integral command is used:

T=T_ini+Integral(dTdt,time,0,t_sim)

The results are stored in an integral table:

$IntegralTable time:10,T

Figure P3.2-4-3 illustrates the temperature as a function of time predicted using EES' Integral
command.
300

280

260

240
Temperature (K)

220

200

180

160

140

120
0 250 500 750 1000 1250 1500 1750 2000
Time (s)
Figure P3.2-4-3: Temperature as a function of time, predicted using EES' Integral command.

d.) Develop a numerical model in MATLAB using the ode solver. Plot the temperature of the
instrument as a function of time for 2000 s after the cryocooler is activated.

A function dTdt_P3p2d4 is created to return the value of the temperature rate of change given
time, the temperature, and the other required input properties.

function[dTdt]=dTdt_P3p2d4(time,T,A_s,Mass,c,e,T_sur)
sigma=5.67e-8; %Stefan-Boltzman constant (W/m^2-K^4]
dTdt=(qf_dot_solar(time)*A_s+sigma*e*A_s*(T_sur^4-T^4)...
-q_dot_load(T))/(Mass*c);
end

The functions to return the cryocooler load and the solar flux are added to the file dTdt_P3p2d4 as
sub-functions:

function[q_dot]=q_dot_load(T)
q_dot=-4.995+0.1013*T-0.0001974*T^2; %heat load
end

function[qf]=qf_dot_solar(time)
qf=100+100*sin(0.02094*time); %solar flux
end

The integration is carried out from a script named P3p2d4:


clear all;
A_s=0.02; %surface area (m^2)
Mass=0.05; %mass (kg)
c=300; %specific heat capacity (J/kg-K)
e=0.35; %emissivity (-)
T_sur=300; %temperature of surroundings (K)
T_ini=300; %initial temperature of instrument (K)
t_sim=2000; %simulation time (s)

[time,T]=ode45(@(time,T) dTdt_P3p2d4(time,T,A_s,Mass,c,e,T_sur),...
[0, t_sim],T_ini);

Running P3p2d4 from the command line puts the variables time and T into the command space
where they can be manipulated and plotted. Figure P3.2-4-4 illustrates the temperature as a
function of time predicted using MATLAB's ode45 command.

300
Temperature (K)

250

200

150

100
0 500 1000 1500 2000
Time (s)

Figure P3.2-4-3: Temperature as a function of time, predicted using MATLAB's ode45 command.
PROBLEM 3.2-5 (3-5 in text): Particle ablation (revisited)
Reconsider Problem 3.1-6 (3-3 in text).
a.) Develop a numerical model of the particle using the Euler technique implemented in either
EES of MATLAB. Plot the temperature as a function of time and compare your answer with
the analytical solution from Problem 3.1-6 (3-3).

The inputs are entered in EES:

$UnitSystem SI MASS RAD PA K J


$TABSTOPS 0.2 0.4 0.6 0.8 3.5 in

"Inputs"
r_p=5 [micron]*convert(micron,m) "particle radius"
c=1500 [J/kg-K] "specific heat capacity"
rho=800 [kg/m^3] "density"
h_bar=60000 [W/m^2-K] "heat transfer coefficient"
t_p=2e-6 [s] "time of the pulse"
t_d=0.5e-6 [s] "duration of pulse"
q_dot_max=0.22 [W] "maximum laser power"
T_infinity=converttemp(C,K,20[C]) "air temperature"

A function is defined that returns the laser power:

function q_dot_laser(time,t_p,t_d,q_dot_max)
"Inputs:
time - time relative to start of process (s)
t_p - time of laser pulse peak relative to start of process (s)
t_d - pulse duration (s)
q_dot_max - maximum power (W)

Output:
q_dot_laser - instantaneous laser power (W)"
q_dot_laser=q_dot_max*exp(-(time-t_p)^2/(2*t_d^2))
end

The volume of the particle is:

4
V = π rp3 (1)
3

The surface area of the particle is:

As = 4 π rp2 (2)

An energy balance on the particle leads to:

dT
qlaser = h As (T − T∞ ) + C (3)
dt
where

C =V ρ c (4)

C_tot=rho*V*c "total heat capacity"

Equation (3) is rearranged:

dT T T∞ qlaser
+ = + (5)
dt τ τ C

where τ is the lumped capacitance time constant:

C
τ= (6)
h As

tau=C_tot/(h_bar*A_s) "lumped capacitance time constant"

The state equation for this problem is obtained by rearranging Eq. (5):

dT (T∞ − T ) qlaser
= + (7)
dt τ C

where qlaser is a function of time. The simulation time (tsim) is divided into M steps of duration
Δt according to:

tsim
Δt = (8)
( M − 1)

tj =
( j − 1) t (9)
( M − 1) sim
t_sim=10e-6 [s] "total simulation time"
M=21 [-] "number of time steps"
Dt=t_sim/(M-1) "timestep duration"
duplicate j=1,M
time[j]=(j-1)*t_sim/(M-1) "time"
end

The initial temperature is set:

T1 = T∞ (10)

T[1]=T_infinity "initial temperature"


The first step is taken using the Euler technique:

dT
T2 = T1 + Δt (11)
dt t = t1 ,T =T1

Substituting Eq. (7) into Eq. (11):

⎡ (T − T ) qlaser ,t =t1 ⎤
T2 = T1 + ⎢ ∞ 1 + ⎥ Δt (12)
⎣ τ C ⎦

T[1]=T_infinity "initial temperature"


T[2]=T[1]+((T_infinity-T[1])/tau+q_dot_laser(time[1],t_p,t_d,q_dot_max)/C_tot)*Dt "1st Euler step"

All of the time steps are simulated by placing Eq. (12) within a duplicate loop and changing the
subscript 1 to j and 2 to j+1:

{T[2]=T[1]+((T_infinity-T[1])/tau+q_dot_laser(time[1],t_p,t_d,q_dot_max)/C_tot)*Dt "1st Euler step"}


duplicate j=1,(M-1)
T[j+1]=T[j]+((T_infinity-T[j])/tau+q_dot_laser(time[j],t_p,t_d,q_dot_max)/C_tot)*Dt "Euler steps"
end

Figure P3.2-5(a) illustrates the temperature as a function of time; the analytical solution from
Problem 3.1-6 is also shown.
800

700

600
Temperature (K)

500

400

300
numerical solution
analytical solution
200
0x100 2x10-6 4x10-6 6x10-6 8x10-6 10-5
Time (s)
Figure P3.2-5(a): Temperature of the particle as a function of time predicted by the numerical model using
the Euler method and the analytical model from Problem 3.1-6 (3-3).

The solution is obtained in MATLAB using a straightforward translation of the EES code:

function[time,T]=P3p2d5a(M)
%Input:
%M - number of time steps (-)
%Outputs:
%time - vector of times (s)
%T - vector of temperatures (K)
r_p=5e-6; %particle radius (m)
c=1500; %specific heat capacity (J/kg-K)
rho=800; %density (kg/m^3)
h_bar=60000; %heat transfer coefficient (W/m^2-K)
t_p=2e-6; %time of the pulse (s)
t_d=0.5e-6; %duration of pulse (s)
q_dot_max=0.22; %maximum laser power (W)
T_infinity=293.2; %air temperature (K)

V=4*pi*r_p^3/3; %volume (m^3)


A_s=4*pi*r_p^2; %surface area (m^2)
C_tot=rho*V*c; %total heat capacity (J/K)
tau=C_tot/(h_bar*A_s); %lumped capacitance time constant (s)

t_sim=10e-6; %total simulation time (s)


Dt=t_sim/(M-1); %timestep duration (s)
for j=1:M
time(j)=(j-1)*t_sim/(M-1); %time (s)
end

T(1)=T_infinity; %initial temperature


for j=1:(M-1)
T(j+1)=T(j)+((T_infinity-T(j))/tau+...
q_dot_laser(time(j),t_p,t_d,q_dot_max)/C_tot)*Dt;
%all of the Euler steps
end
end

function[q_dot]=q_dot_laser(time,t_p,t_d,q_dot_max)
% Inputs:
% time - time relative to start of process (s)
% t_p - time of laser pulse peak relative to start of process (s)
% t_d - pulse duration (s)
% q_dot_max - maximum power (W)
% Output:
% q_dot_laser - instantaneous laser power (W)

q_dot=q_dot_max*exp(-(time-t_p)^2/(2*t_d^2));

end

b.) Develop a numerical model of the particle using Heun's technique implemented in either EES
of MATLAB. Plot the temperature as a function of time.

The state equation is integrated through the 1st time step using Heun's technique. The predictor
step begins by taking an Euler step to estimate the temperature at the end of the time step:

dT
Tˆ2 = T1 + Δt (13)
dt T =T1 ,t = t1

dTdt_hat[1]=(T_infinity-T[1])/tau+q_dot_laser(time[1],t_p,t_d,q_dot_max)/C_tot
"initial estimate of temperature derivative"
T_hat[2]=T[1]+dTdt_hat[1]*Dt "1st predictor step"

The corrector step uses the estimate of the temperature derivative at the beginning and end of the
time step:

⎛ dT dT ⎞ Δt
T2 = T1 + ⎜ + ⎟⎟ (14)
⎜ dt dt T =Tˆ2 ,t = t2 ⎠ 2
⎝ T =T1 ,t = t1

dTdt[2]=(T_infinity-T_hat[2])/tau+q_dot_laser(time[2],t_p,t_d,q_dot_max)/C_tot
"initial estimate of temperature derivative"
T[2]=T[1]+(dTdt_hat[1]+dTdt[2])*Dt/2 "1st corrector step"

The process of simulating all of the time steps is automated:

{dTdt_hat[1]=(T_infinity-T[1])/tau+q_dot_laser(time[1],t_p,t_d,q_dot_max)/C_tot
"initial estimate of temperature derivative"
T_hat[2]=T[1]+dTdt_hat[1]*Dt "1st predictor step"
dTdt[2]=(T_infinity-T_hat[2])/tau+q_dot_laser(time[2],t_p,t_d,q_dot_max)/C_tot
"initial estimate of temperature derivative"
T[2]=T[1]+(dTdt_hat[1]+dTdt[2])*Dt/2 "1st corrector step"}

duplicate j=1,(M-1)
dTdt_hat[j]=(T_infinity-T[j])/tau+q_dot_laser(time[j],t_p,t_d,q_dot_max)/C_tot
"initial estimate of temperature derivative"
T_hat[j+1]=T[j]+dTdt_hat[j]*Dt "predictor step"
dTdt[j+1]=(T_infinity-T_hat[j+1])/tau+q_dot_laser(time[j+1],t_p,t_d,q_dot_max)/C_tot
"initial estimate of temperature derivative"
T[j+1]=T[j]+(dTdt_hat[j]+dTdt[j+1])*Dt/2 "corrector step"
end

The temperature as a function of time is shown in Figure P3.2-5(b).


800

700

600
Temperature (K)

500

400

300
numerical solution
analytical solution
200
0x100 2x10-6 4x10-6 6x10-6 8x10-6 10-5
Time (s)
Figure P3.2-5(b): Temperature of the particle as a function of time predicted by the numerical model using
the Heun's method and the analytical model from Problem 3.1-6 (3-3).

Because Heun's method is also an explicit technique, the translation to MATLAB is


straightforward:
function[time,T]=P3p2d5b(M)
%Input:
%M - number of time steps (-)
%Outputs:
%time - vector of times (s)
%T - vector of temperatures (K)

r_p=5e-6; %particle radius (m)


c=1500; %specific heat capacity (J/kg-K)
rho=800; %density (kg/m^3)
h_bar=60000; %heat transfer coefficient (W/m^2-K)
t_p=2e-6; %time of the pulse (s)
t_d=0.5e-6; %duration of pulse (s)
q_dot_max=0.22; %maximum laser power (W)
T_infinity=293.2; %air temperature (K)

V=4*pi*r_p^3/3; %volume (m^3)


A_s=4*pi*r_p^2; %surface area (m^2)
C_tot=rho*V*c; %total heat capacity (J/K)
tau=C_tot/(h_bar*A_s); %lumped capacitance time constant (s)

t_sim=10e-6; %total simulation time (s)


Dt=t_sim/(M-1); %timestep duration (s)
for j=1:M
time(j)=(j-1)*t_sim/(M-1); %time (s)
end

T(1)=T_infinity; %initial temperature


for j=1:(M-1)
dTdt_hat=(T_infinity-T(j))/tau+...
q_dot_laser(time(j),t_p,t_d,q_dot_max)/C_tot;
%initial estimate of temperature derivative
T_hat=T(j)+dTdt_hat*Dt; %predictor step
dTdt=(T_infinity-T_hat)/tau+...
q_dot_laser(time(j+1),t_p,t_d,q_dot_max)/C_tot;
%initial estimate of temperature derivative
T(j+1)=T(j)+(dTdt_hat+dTdt)*Dt/2; %corrector step
end
end

function[q_dot]=q_dot_laser(time,t_p,t_d,q_dot_max)
% Inputs:
% time - time relative to start of process (s)
% t_p - time of laser pulse peak relative to start of process (s)
% t_d - pulse duration (s)
% q_dot_max - maximum power (W)
% Output:
% q_dot_laser - instantaneous laser power (W)

q_dot=q_dot_max*exp(-(time-t_p)^2/(2*t_d^2));

end
c.) Develop a numerical model of the particle using the fully implicit technique implemented in
either EES of MATLAB. Plot the temperature as a function of time.

The state equation is integrated through the 1st time step using the fully implicit technique.

dT
T2 = T1 + Δt (15)
dt T =T2 ,t = t2

or, for this problem:

⎡ (T − T ) qlaser ,t =t2 ⎤
T2 = T1 + ⎢ ∞ 2 + ⎥ Δt (16)
⎣ τ C ⎦

T[2]=T[1]+((T_infinity-T[2])/tau+q_dot_laser(time[2],t_p,t_d,q_dot_max)/C_tot )*Dt "1st implicit step"

The solution is extended to all of the time steps according to:

{T[2]=T[1]+((T_infinity-T[2])/tau+q_dot_laser(time[2],t_p,t_d,q_dot_max)/C_tot )*Dt "1st implicit step"}


duplicate j=1,(M-1)
T[j+1]=T[j]+((T_infinity-T[j+1])/tau+q_dot_laser(time[j+1],t_p,t_d,q_dot_max)/C_tot )*Dt "implicit step"
end

The temperature as a function of time predicted using the fully implicit method is shown in
Figure 3.2-5(c).
800

700

600
Temperature (K)

500

400

300
numerical solution
analytical solution
200
0x100 2x10-6 4x10-6 6x10-6 8x10-6 10-5
Time (s)
Figure P3.2-5(c): Temperature of the particle as a function of time predicted by the numerical model using
the fully implicit method and the analytical model from Problem 3.1-6 (3-3).

In order to implement the solution using MATLAB, it is necessary to solve Eq. (16) for T2 in
order to obtain an explicit equation:
⎡T qlaser ,t =t2 ⎤
T1 + ⎢ ∞ + Δt
⎣ τ C ⎥⎦
T2 = (17)
⎛ Δt ⎞
⎜1 + ⎟
⎝ τ ⎠

or in general:

⎡T qlaser ,t =t2 ⎤
Tj + ⎢ ∞ + Δt
⎣ τ C ⎥⎦
T j +1 = (18)
⎛ Δt ⎞
⎜1 + ⎟
⎝ τ ⎠

function[time,T]=P3p2d5c(M)
%Input:
%M - number of time steps (-)
%Outputs:
%time - vector of times (s)
%T - vector of temperatures (K)

r_p=5e-6; %particle radius (m)


c=1500; %specific heat capacity (J/kg-K)
k=1; %conductivity (W/m-K)
rho=800; %density (kg/m^3)
h_bar=60000; %heat transfer coefficient (W/m^2-K)
t_p=2e-6; %time of the pulse (s)
t_d=0.5e-6; %duration of pulse (s)
q_dot_max=0.22; %maximum laser power (W)
T_infinity=293.2; %air temperature (K)

V=4*pi*r_p^3/3; %volume (m^3)


A_s=4*pi*r_p^2; %surface area (m^2)
C_tot=rho*V*c; %total heat capacity (J/K)
tau=C_tot/(h_bar*A_s); %lumped capacitance time constant (s)

t_sim=10e-6; %total simulation time (s)


Dt=t_sim/(M-1); %timestep duration (s)
for j=1:M
time(j)=(j-1)*t_sim/(M-1); %time (s)
end

T(1)=T_infinity; %initial temperature


for j=1:(M-1)

T(j+1)=(T(j)+T_infinity*Dt/tau+q_dot_laser(time(j+1),t_p,t_d,q_dot_max)*Dt/C_
tot)/(1+Dt/tau);
end
end

function[q_dot]=q_dot_laser(time,t_p,t_d,q_dot_max)
% Inputs:
% time - time relative to start of process (s)
% t_p - time of laser pulse peak relative to start of process (s)
% t_d - pulse duration (s)
% q_dot_max - maximum power (W)
% Output:
% q_dot_laser - instantaneous laser power (W)

q_dot=q_dot_max*exp(-(time-t_p)^2/(2*t_d^2));

end

d.) Develop a numerical model of the particle using the Crank-Nicolson technique implemented
in either EES of MATLAB. Plot the temperature as a function of time.

The state equation is integrated through the 1st time step using the Crank-Nicolson technique.

⎛ dT dT ⎞ Δt
T2 = T1 + ⎜ + ⎟⎟ (19)
⎜ dt dt T =T2 ,t = t2 ⎠ 2
⎝ T =T1 ,t = t1

or, for this problem:

⎡ (T − T ) qlaser ,t =t1 (T∞ − T2 ) qlaser ,t =t2 ⎤ Δt


T2 = T1 + ⎢ ∞ 1 + + + ⎥ (20)
⎣ τ C τ C ⎦ 2

T[2]=T[1]+((T_infinity-T[1])/tau+q_dot_laser(time[1],t_p,t_d,q_dot_max)/C_tot+&
(T_infinity-T[2])/tau+q_dot_laser(time[2],t_p,t_d,q_dot_max)/C_tot )*Dt/2 "1st C-N step"

and extended to all of the time steps:

duplicate j=1,(M-1)
T[j+1]=T[j]+((T_infinity-T[j])/tau+q_dot_laser(time[j],t_p,t_d,q_dot_max)/C_tot+&
(T_infinity-T[j+1])/tau+q_dot_laser(time[j+1],t_p,t_d,q_dot_max)/C_tot )*Dt/2 "C-N step"
end

The temperature as a function of time predicted using the Crank-Nicolson method is shown in
Figure 3.2-5(d).
800

700

600

Temperature (K)
500

400

300
numerical solution
analytical solution
200
0x100 2x10-6 4x10-6 6x10-6 8x10-6 10-5
Time (s)
Figure P3.2-5(d): Temperature of the particle as a function of time predicted by the numerical model using
the Crank-Nicolson method and the analytical model from Problem 3.1-6 (3-3).

In order to implement the solution using MATLAB, it is necessary to solve Eq. (20) for T2 in
order to obtain an explicit equation:

⎡ T Δt T Δt qlaser ,t =t1 Δt (T∞ − T2 ) qlaser ,t =t2 Δt ⎤


T1 + ⎢ ∞ − 1 + + + ⎥
⎣ τ 2τ C 2 τ C 2⎦
T2 = (21)
⎛ Δt ⎞
⎜1 + ⎟
⎝ 2⎠

Equation (21) is extended to all of the time steps:

⎡ T Δt T Δt qlaser ,t =t1 Δt (T∞ − T2 ) qlaser ,t =t2 Δt ⎤


Tj + ⎢ ∞ − j + + + ⎥
⎣ τ 2τ C 2 τ C 2⎦
T j +1 = (22)
⎛ Δt ⎞
⎜1 + ⎟
⎝ 2⎠

function[time,T]=P3p2d5d(M)
%Input:
%M - number of time steps (-)
%Outputs:
%time - vector of times (s)
%T - vector of temperatures (K)

r_p=5e-6; %particle radius (m)


c=1500; %specific heat capacity (J/kg-K)
k=1; %conductivity (W/m-K)
rho=800; %density (kg/m^3)
h_bar=60000; %heat transfer coefficient (W/m^2-K)
t_p=2e-6; %time of the pulse (s)
t_d=0.5e-6; %duration of pulse (s)
q_dot_max=0.22; %maximum laser power (W)
T_infinity=293.2; %air temperature (K)

V=4*pi*r_p^3/3; %volume (m^3)


A_s=4*pi*r_p^2; %surface area (m^2)
C_tot=rho*V*c; %total heat capacity (J/K)
tau=C_tot/(h_bar*A_s); %lumped capacitance time constant (s)

t_sim=10e-6; %total simulation time (s)


Dt=t_sim/(M-1); %timestep duration (s)
for j=1:M
time(j)=(j-1)*t_sim/(M-1); %time (s)
end

T(1)=T_infinity; %initial temperature


for j=1:(M-1)
T(j+1)=(T(j)*(1-
Dt/(2*tau))+T_infinity*Dt/tau+(q_dot_laser(time(j),t_p,t_d,q_dot_max)+q_dot_l
aser(time(j+1),t_p,t_d,q_dot_max))*Dt/(2*C_tot))/(1+Dt/(2*tau));
end
end

function[q_dot]=q_dot_laser(time,t_p,t_d,q_dot_max)
% Inputs:
% time - time relative to start of process (s)
% t_p - time of laser pulse peak relative to start of process (s)
% t_d - pulse duration (s)
% q_dot_max - maximum power (W)
% Output:
% q_dot_laser - instantaneous laser power (W)

q_dot=q_dot_max*exp(-(time-t_p)^2/(2*t_d^2));

end

e.) Develop a numerical model of the particle using the Integral command in EES. Plot the
temperature as a function of time.

The state equation is evaluated at an arbitrary value of temperature and time:

t_sim=10e-6 [s] "total simulation time"


time=3e-6 [s] "arbitrary time for evaluation of integrand"
T=500 [K] "arbitary temperature for evaluation of integrand"
dTdt=(T_infinity-T)/tau+q_dot_laser(time,t_p,t_d,q_dot_max)/C_tot "integrand"

The temperature and time are commented out and the integral command is used to integrate the
state equation from 0 to tsim:

{time=3e-6 [s] "arbitrary time for evaluation of integrand"


T=500 [K] "arbitary temperature for evaluation of integrand"}
dTdt=(T_infinity-T)/tau+q_dot_laser(time,t_p,t_d,q_dot_max)/C_tot "integrand"
T=T_infinity+Integral(dTdt,time,0,t_sim)

An integral table is created to hold the results:

$IntegralTable time,T
The temperature as a function of time predicted using the Integral command in EES is shown in
Figure 3.2-5(e).
800

700

Temperature (K)
600

500

400

300

200
0x100 2x10-6 4x10-6 6x10-6 8x10-6 10-5
Time (s)
Figure P3.2-5(e): Temperature of the particle as a function of time predicted using the Integral command in
EES and the analytical model from Problem 3.1-6.

f.) Develop a numerical model of the particle using ode45 solver in MATLAB. Plot the
temperature as a function of time.

A function is created in MATLAB to return the state equation; note that the first two inputs to
the function are time and the value of the state variable, temperature and the function returns the
derivative of the state variable:

function[dTdt]=dTdtP3p2d5f2(time,T,r_p,c,rho,h_bar,t_p,t_d,q_dot_max,T_infini
ty)

V=4*pi*r_p^3/3; %volume (m^3)


A_s=4*pi*r_p^2; %surface area (m^2)
C_tot=rho*V*c; %total heat capacity (J/K)
tau=C_tot/(h_bar*A_s); %lumped capacitance time constant (s)

dTdt=(T_infinity-T)/tau+q_dot_laser(time,t_p,t_d,q_dot_max)/C_tot;
end

function[q_dot]=q_dot_laser(time,t_p,t_d,q_dot_max)
% Inputs:
% time - time relative to start of process (s)
% t_p - time of laser pulse peak relative to start of process (s)
% t_d - pulse duration (s)
% q_dot_max - maximum power (W)
% Output:
% q_dot_laser - instantaneous laser power (W)

q_dot=q_dot_max*exp(-(time-t_p)^2/(2*t_d^2));

end

The function is integrated in a separate function using the ode45 solver:


function[time,T]=P3p2d5f()
r_p=5e-6; %particle radius (m)
c=1500; %specific heat capacity (J/kg-K)
k=1; %conductivity (W/m-K)
rho=800; %density (kg/m^3)
h_bar=60000; %heat transfer coefficient (W/m^2-K)
t_p=2e-6; %time of the pulse (s)
t_d=0.5e-6; %duration of pulse (s)
q_dot_max=0.22; %maximum laser power (W)
T_infinity=293.2; %air temperature (K)

V=4*pi*r_p^3/3; %volume (m^3)


A_s=4*pi*r_p^2; %surface area (m^2)
C_tot=rho*V*c; %total heat capacity (J/K)
tau=C_tot/(h_bar*A_s); %lumped capacitance time constant (s)

t_sim=10e-6; %total simulation time (s)

OPTIONS=odeset('RelTol',1e-6);
[time,T]=ode45(@(time,T) dTdtP3p2d5f2(time,T,r_p,c,rho,h_bar,...
t_p,t_d,q_dot_max,T_infinity),[0,t_sim],T_infinity,OPTIONS);
end

The temperature as a function of time predicted using the ode45 solver in MATLAB is shown in
Figure 3.2-5(f).

750

700

650

600
Temperature (K)

550

500

450

400

350

300

250
0 0.2 0.4 0.6 0.8 1 1.2
Time (s) x 10
-5

Figure P3.2-5(f): Temperature of the particle as a function of time predicted using the ode45 solver in
MATLAB.
PROBLEM 3.2-6 (3-6 in text)
You are interested in using a thermoelectric cooler to quickly reduce the temperature of a small
detector from its original temperature of Tini = 295 K to its operating temperature. As shown in
Figure P3.2-6, the thermoelectric cooler receives power at a rate of w = 5.0 W from a small
battery and rejects heat at a rate of qrej to ambient temperature TH = 305 K. The cooler removes
energy at a rate of qref from the detector which is at temperature T. (The detector temperature T
will change with time, t). The detector has a total heat capacity, C, of 0.5 J/K. Despite your best
efforts to isolate the detector from the ambient, the detector is subjected to a parasitic heat gain,
q p , that can be modeled as occurring through a fixed resistance Rp = 100 K/W; this resistance
represents the combined effect of radiation and conduction.

qrej
TH = 305 K

w
battery
q p
thermoelectric cooler
T qref

detector, C = 0.5 J/K


Figure P3.2-6: Detector cooled by a thermoelectric cooler.

The thermoelectric cooler has a 2nd law efficiency ηc = 10% regardless of its operating
temperatures. That is, the amount of refrigeration provided to the detector can be related to the
input power provided to the thermoelectric cooler and its operating temperatures according to:
w ηc
qref =
⎛ TH ⎞
⎜ − 1⎟
⎝T ⎠
a.) Derive the governing differential equation that describes the temperature of the detector.
Note that the result should be a symbolic equation for the rate of temperature change of the
detector as a function of the quantities given in the problem (i.e., TH, Rp, C, w , ηc) and the
instantaneous value of the detector temperature (T).

An energy balance on the detector leads to:

dT
q p = C + qref (1)
dt

or

(TH − T ) = C dT + w ηc
(2)
Rp dt ⎛ TH ⎞
⎜ − 1⎟
⎝T ⎠
so the state equation for the problem is:

dT (TH − T ) w ηc
= − (3)
dt C Rp ⎛T ⎞
C ⎜ H − 1⎟
⎝T ⎠

b.) Develop an EES program that numerically solves this problem for the values given in the
problem statement using a predictor-corrector technique (e.g., the modified Euler technique).
Using your program, prepare a plot showing the temperature of the detector as a function of
time for 120 sec after the cooler is activated.

The inputs are entered in EES and a function is defined to return the state equation:

$UnitSystem SI MASS RAD PA K J


$TABSTOPS 0.2 0.4 0.6 0.8 3.5 in

function dTdt(time,T,T_H,C,R_p,w_dot,eta_c)
"Inputs:
time - time relative to start of cooldown (s)
T - temperature of detector (K)
T_H - heat rejection temperature (K)
C - heat capacity of detector (J/K)
R_p - resistance to parasitic (K/W)
w_dot - power to cooler (W)
eta_c - 2nd law efficiency of cooler

Output:
dTdt - time rate of change of detector (K/s)"

dTdt=(T_H-T)/(C*R_p)-w_dot*eta_c/(C*(T_H/T-1))
end

"Inputs"
T_ini=295 [K] "initial temperature"
w_dot=30 [W] "power to cooler"
T_H=305 [K] "heat rejection temperature"
C=0.5 [J/K] "heat capacity"
R_p=100 [K/W] "resistance to parasitic"
eta_c=0.1 [-] "cooler efficiency"

The simulation is broken into timesteps, distributed according to:

tsim
Δt = (4)
( M − 1)

where M is the number of times at which the temperature will be evaluated. The time
corresponding to each time step is therefore:
tj =
( j − 1) t for j = 1..M (5)
( M − 1) sim
t_sim=30 [s] "simulation time"
M=91 [-] "number of time steps"
DELTAt=t_sim/(M-1) "duration of time step"
duplicate j=1,M
time[j]=(j-1)*t_sim/(M-1) "time associated with each temperature"
end

Heun's method relies on a predictor step followed by a corrector step:

"Heun's Method"
T[1]=T_ini
duplicate j=1,(M-1)
T_hat[j+1]=T[j]+dTdt(time[j],T[j],T_H,C,R_p,w_dot,eta_c)*DELTAt "predictor step"
T[j+1]=T[j]+dTdt(time[j],T[j],T_H,C,R_p,w_dot,eta_c)*DELTAt/2+&
dTdt(time[j+1],T_hat[j+1],T_H,C,R_p,w_dot,eta_c)*DELTAt/2 "corrector step"
end

Figure 2 illustrates the temperature as a function of time:

300

290

280

270
Temperature (K)

260

250

240

230

220

210

200
0 10 20 30 40 50 60 70 80 90 100 110 120
Time (s)
Figure 2: Temperature of the detector as a function of time.

c.) Modify your program so that it accounts for the fact that your battery only has 100 J of
energy storage capacity; once the 100 J of energy in the battery is depleted then the power
driving the thermoelectric cooler goes to zero. Prepare a plot showing the temperature of the
detector as a function of time for 120 s after the cooler is activated.

The function dTdt is modified so that the refrigeration goes to zero when the power is depleted:

function dTdt(time,T,T_H,C,R_p,w_dot,eta_c)
"Inputs:
time - time relative to start of cooldown (s)
T - temperature of detector (K)
T_H - heat rejection temperature (K)
C - heat capacity of detector (J/K)
R_p - resistance to parasitic (K/W)
w_dot - power to cooler (W)
eta_c - 2nd law efficiency of cooler

Output:
dTdt - time rate of change of detector (K/s)"

Energy = 100 [J] "energy in battery"


if (time<(Energy/w_dot)) then
dTdt=(T_H-T)/(C*R_p)-w_dot*eta_c/(C*(T_H/T-1))
else
dTdt=(T_H-T)/(C*R_p)
endif

{dTdt=(T_H-T)/(C*R_p)-w_dot*eta_c/(C*(T_H/T-1))}
end

Figure 3 illustrates the temperature as a function of time, including the re-equilibration process
that occurs when the battery is discharged at 20 s.

300

290

280

270
Temperature (K)

260

250

240

230

220

210

200
0 10 20 30 40 50 60 70 80 90 100 110 120
Time (s)
Figure 3: Temperature of the detector as a function of time including the limited battery energy.

d.) Assume that the objective of your cooler is to keep the detector at a temperature below 240 K
for as long as possible, given that your battery only has 100 J of energy. What power ( w )
would you use to run the thermoelectric cooler? Justify your answer with plots and an
explanation.

I would turn the power up as high as possible in order to drive the temperature of the detector
down as much as possible very quickly. The impact of the parasitics is related to the product of
the magnitude and time. In order to minimize the effect of parasitics, you want the detector to
cool down quickly. To see this more clearly, consider Figure 4 which shows the temperature as
a function of time for various values of w . When w = 2 W, the detector barely reaches 240 K
when the battery is depleted and therefore it is only active for an instant of time. On the other
hand when w increases beyond about 10 W it remains below 240 K for approximately 21 s
regardless of w because the effect of the parasitic on the cooldown is minimal due to its short
duration.

300

290

280

270
Temperature (K)

260

250
2W
240

230
5W
220
10 W
210 20 W
200
0 10 20 30 40 50 60 70
Time (s)
Figure 4: Temperature of the detector as a function of time for various values of power.
Problem 3.2-7
A pan is placed above a camp fire flame. The pan can be modeled as a circular disk with
thickness th = 0.25 inch and diameter D = 8 inch. The pan is made of material with density ρ =
7870 kg/m3, specific heat capacity c = 450 J/kg-K, and conductivity k = 70 W/m-K. The bottom
side of the plate is exposed to radiation from the flame at Tflame = 650°C. The upper side of the
pan is exposed to radiation with the environment at T∞ = 15°C. The emissivity of the pan surface
is ε = 0.8. The upper side of the pan is also exposed to convection with the environment. The
heat transfer coefficient depends on the surface temperature of the pan (T) according to:

⎡ W ⎤ ⎡ W ⎤
h = 2 ⎢ 2 ⎥ + 0.01 ⎢ 2 2 ⎥ (T − T∞ ) (1)
⎣ m -K ⎦ ⎣ m -K ⎦

a.) What is the steady-state temperature of the pan (Tss)?

The inputs are entered in EES and a function is defined to return the heat transfer coefficient:

$UnitSystem SI MASS RAD PA K J


$TABSTOPS 0.2 0.4 0.6 0.8 3.5 in

function h_bar(T,T_infinity)
"Inputs:
T - surface temperature (K)
T_infinity - ambient temperature (K)
Output:
h_bar - natural convection coefficient (W/m^2-K)"

h_bar=2 [W/m^2-K]+0.01 [W/m^2-K^2]*(T-T_infinity)


end

"Inputs"
T_flame=converttemp(C,K,650 [C]) "flame temperature"
th=0.25 [inch]*convert(inch,m) "thickness of pan"
D=8 [inch]*convert(inch,m) "diameter of pan"
rho=7870 [kg/m^3] "density of iron"
c=450 [J/kg-K] "specific heat capacity of iron"
k=70 [W/m-K] "conductivity of iron"
T_infinity=converttemp(C,K,15 [C]) "ambient temperature"
emm=0.8 [-] "emissivity"

An energy balance on the pan at steady state leads to:

As σ ε (T flame
4
− Tss4 ) = As σ ε (Tss4 − T∞4 ) + As hT =Tss (Tss − T∞ ) (2)

where As is the surface area of one side of the pan:

π
As = D2 (3)
4
A_s=pi*D^2/4 "surface area"
A_s*sigma#*emm*(T_flame^4-T_ss^4)=A_s*sigma#*emm*(T_ss^4-
T_infinity^4)+A_s*h_bar(T_ss,T_infinity)*(T_ss-T_infinity)
"energy balance at steady state"
T_ss_C=converttemp(K,C,T_ss) "steady-state temperature, in C"

which leads to Tss = 485.8°C.

b.) Estimate the lumped capacitance time constant of the pan during the time that it is heated
from T∞ to Tss.

The heat capacity of the pan is:

C = As th (4)

Cap=pi*D^2*th*rho*c/4 "heat capacity of pan"

The average temperature of the pan during the transient process is calculated:

T =
(Tss + T∞ ) (5)
2

and used to determine the resistance to radiation:

1
Rrad = (6)
As σ ε (T + T∞2 )(T + T∞ )
2

and the resistance to convection:

1
Rconv = (7)
As hT =T

The total resistance between the pan and the surroundings is:
−1
⎛ 1 1 ⎞
Rtotal =⎜ + ⎟ (8)
⎝ Rrad Rconv ⎠

The lumped capacitance time constant is:

τ lumped = C Rtotal (9)

T_s_bar=(T_ss+T_infinity)/2 "average surface temperature during transient"


R_rad=1/(A_s*sigma#*emm*(T_s_bar^2+T_infinity^2)*(T_s_bar+T_infinity)) "radiation resistance"
R_conv=1/(A_s*h_bar(T_s_bar,T_infinity)) "convection resistance"
R_total=(1/R_rad+1/R_conv)^(-1) "total resistance"
tau_lumped=Cap*R_total "approximate lumped capacitance time constant"
tau_lumped_min=tau_lumped*convert(s,min) "in min"

which leads to τlumped = 21 min.

c.) Is a lumped capacitance model of the pan appropriate?

The Biot number is the ratio of the resistance to internal conduction heat transfer to the resistance
to heat transfer from the surface of the object. The resistance to internal conduction (Rcond) is
approximated according to:

th
Rcond = (10)
k As

The Biot number that characterizes this problem is therefore:

Rcond
Bi = (11)
Rtotal

R_cond_int=th/(k*A_s) "resistance to internal conduction"


Bi=R_cond_int/R_total "Biot number"

which leads to Bi = 0.002; this is sufficiently less than one to justify the lumped capacitance
model.

d.) Develop a numerical model of the temperature of the pan using the Euler technique; assume
that the pan is initially in thermal equilibrium with the surrounding. Plot the temperature of
the pan as a function of time.

The governing equation for the pan is derived by doing an energy balance on the pan:

dT
=
(
As σ ε (T flame
4
− T 4 ) − As σ ε (T 4 − T∞4 ) − As hT =Tss (T − T∞ ) ) (12)
dt C

The total simulation time is broken into time steps of duration:

τ sim
Δt = (13)
( M − 1)
and the time associated with each temperature is:

t j = Δt ( j − 1) (14)
tau_sim=2*tau_lumped "simulation time"
M=51 [-] "number of simulation times"
Dt=tau_sim/(M-1) "duration of time step"
duplicate j=1,M
time[j]=Dt*(j-1) "location of each time step"
end

The initial temperature is equal to the ambient temperature:

T1 = T∞ (15)

T[1]=T_infinity "initial temperature"

The 1st Euler step is accomplished according to:

dT
=
(
As σ ε (T flame
4
− T14 ) − As σ ε (T14 − T∞4 ) − As hT =Tss (T1 − T∞ ) ) (16)
dt 1 C

so that:

dT
T2 = T1 + Δt (17)
dt 1

"take 1st Euler step"


dTdt[1]=(A_s*sigma#*emm*(T_flame^4-T[1]^4)-A_s*sigma#*emm*(T[1]^4-T_infinity^4)-&
A_s*h_bar(T[1],T_infinity)*(T[1]-T_infinity))/Cap "rate of temperature chage for 1st time step"
T[2]=T[1]+dTdt[1]*Dt "1st Euler step"

The solution is examined and, once it has been debugged, the process of simulating all of the
time steps is automated; the 1-step code above is placed within a duplicate loop where 1 is
replaced by j and 2 is replaced by j+1:

{"take 1st Euler step"


dTdt[1]=(A_s*sigma#*emm*(T_flame^4-T[1]^4)-A_s*sigma#*emm*(T[1]^4-T_infinity^4)-
A_s*h_bar(T[1],T_infinity)*(T[1]-T_infinity))/Cap
"rate of temperature chage for 1st time step"
T[2]=T[1]+dTdt[1]*Dt "1st Euler step"}
"take all of the Euler steps"
duplicate j=1,(M-1)
dTdt[j]=(A_s*sigma#*emm*(T_flame^4-T[j]^4)-A_s*sigma#*emm*(T[j]^4-T_infinity^4)-&
A_s*h_bar(T[j],T_infinity)*(T[j]-T_infinity))/Cap "rate of temperature chage for 1st time step"
T[j+1]=T[j]+dTdt[j]*Dt "1st Euler step"
end

The temperature of the pan is plotted in Figure 1.


800

700

Temperature (K)
600

500

400

300

200
0 250 500 750 1000 1250 1500
Time (s)
Figure 1: Pan temperature as a function of time.
Problem 3.3-1: Approximate Model of the Transient Response of a Wall
A plane wall has an area Ac = 1 m2 and is L = 1 m thick. The wall is composed of a material with
thermal conductivity k = 1 W/m-K, density ρ = 1 kg/m3, and c = 1 J/kg-K, as shown in Figure
P3.3-1. The right side of the wall (at x = L) is exposed to fluid at TC = 0°C with a heat transfer
coefficient h = 1 W/m2-K. The wall is initially at a uniform temperature Tini = 0°C when, at
time t = 0, the left side of the wall is exposed to a hotter fluid, TH = 100°C, with a heat transfer
coefficient h = 1 W/m2-K.

Ac = 1 m2, k = 1 W/m-K, ρ = 1 kg/m3, c = 1 J/kg-K

TH = 100°C
TC = 0°C
h = 1 W/m -K
2

h = 1 W/m -K
2

L=1m

at time t = 0, the material is at Tini = 0°C


Figure P3.3-1: Plane wall.

a.) For some time, the wall can be treated as a semi-infinite body. How long will it take for the
thermal wave associated with the disturbance at the left side of the wall to reach the right side
of the wall? That is, estimate how long it takes for the temperature of the right side of the
wall (at x = L) to begin to rise?

The thermal penetration wave that emanates from the left hand wall will travel according to:

δt = 2 α t (1)

where α is the thermal diffusivity:

k 1 W m3 kg K m2
α= = =1 (2)
ρ c m K 1 kg 1 J s

and so the wave will reach the right side of the wall when:

L = 2 α ta (3)

or:

L2 1 m 2 s
ta = = = 0.25s (4)
4α 4 1 m2
b.) Draw an approximate thermal resistance network that can be used to represent the problem
for times that are less than what you calculated in (a). Label the resistors and show how they
would be calculated using symbols provided in Figure P3.3-1 and, if necessary, time (t).

For times less than 0.25 s, there are two thermal resistances separating TH and Ti; convection
from the surface (Rconv) and conduction through the thermally affected region (Rsemi∞), as shown
in Figure 2.

Figure 2: Resistance network for time less than calculated in (a).

The resistances can be calculated according to:

1
Rconv = (5)
hA

παt
Rsemi∞ = (6)
kA

c.) Sketch the temperature distribution at the time you calculated in (a) and at half of that time.
That is, provide a qualitative sketch of the temperature as a function of position at the instant
that the thermal wave hits the right side of the wall and some time before the thermal wave
hits the right side of the wall.

At t = ta= 0.25 s the thermal wave has just reached the right side of the wall. The temperature
distribution is shown qualitatively in Figure 3 and labeled as t = 0.25 s. For t = 0.125 s, the
thermal has not penetrated as far. Notice that the temperature at the left side of the wall is not
equal to the fluid temperature TH because of the convection resistance. Further note that the heat
transfer rate is decreasing with time because Rsemi∞ is increasing and therefore the surface
temperature is rising with time.
Figure 3: Temperature distribution.

d.) Draw a thermal resistance network that can be used to represent the problem when it reaches
steady state (i.e., for times much larger than you calculated in (a)). Calculate the numerical
value of each resistance in your network.

For times much larger than 0.25s, there are three thermal resistances separating TH and TC;
convection from the left hand surface (Rconv), conduction through the wall (Rwall), and convection
from the right hand surface (Rconv), as shown in Figure 4.

Figure 4: Resistance network for time much larger than calculated in (a).

The resistances can be calculated according to:

1 m2 K K
Rconv = = 2
=1 (7)
hA 1W m W

L 1mmK K
Rcond = = 2
=1 (8)
kA 1W1m W

e.) What is the steady state rate at which heat is conducted into the left hand surface of the wall?
That is, what is the numerical value (W) of the rate of heat conduction at x = 0 as time goes
to infinity?
Using the resistance network from Figure 4:

TH − TC 100 K
q x =0,t →∞ = = = 33.3 W (9)
Rcond + 2 Rconv 3 K/W

f.) What are the steady state temperatures of the left hand and right hand surfaces of the wall?
That is, what are T(x = 0, t → ∞) and T(x = L, t → ∞)?

These temperatures are computed using the resistance network shown in Figure 4:

K
Tx =0,t →∞ = TH − q Rconv = 100°C - ( 33.3 W ) 1 = 66.7°C (10)
W

K
Tx =0,t →∞ = TH − q ( Rconv + Rcond ) = 100°C - ( 33.3 W ) 2 = 33.3°C (11)
W

g.) Sketch the steady state temperature distribution (i.e., the temperature as a function position as
t → ∞).

As t → ∞ the temperature distribution in the material is linear and the surface temperatures are as
calculated in (f); this is shown in Figure 3 and labeled t → ∞.

h.) Sketch the rate of heat conduction at the left hand side of the wall (i.e., at x = 0) as a function
of time. Make sure that you indicate the numerical value of the heat transfer rate (in W) that
you expect at t = 0 s, at the time you calculated in (a) (approximately), and at t → ∞. You
may want to refer to your resistance networks from parts (b) and (d) to help with this
problem.

For times less than t = ta the resistance network shown in Figure 2 can be used to help understand
the problem. Notice that at t = 0 the resistance of the semi-infinite body is zero and so the heat
transfer rate is given by:

TH − TC 100 K
q x =0,t =0 = = = 100 W (12)
Rconv 1 K/W

Therefore, the heat transfer rate will start high (at 100 W) and decrease with time as the
resistance of the semi-infinite body increases. Eventually, the heat transfer rate will decrease to
the 33.3 W value computed in (e). At 0.25 s, the thermal wave has hit the opposite side of the
wall and therefore you would expect:

TH − TC 100 K
q x =0,t =0.25 s = = = 50 W (13)
L 2 K/W
Rconv +
kA
Figure 5: Rate of heat conduction at x = 0 as a function of time.

i.) Sketch the time variation of the temperatures at the left hand and right hand surfaces of the
wall. That is, what are T(x = 0, t) and T(x = L, t)? You may want to refer to your resistance
networks from parts (b) and (d) to help with this problem.

The requested sketch is shown in Figure 6. From part (f) we know that as time becomes large,
the surface temperatures go to T(x = 0, t → ∞) = 66.7°C and T(x = L, t → ∞) = 33.3°C. At time
t= 0, both surfaces are at 0°C. In between we would expect the left hand surface to rise
immediately and the right hand surface to begin to rise only after 0.25 s (that is, only after the
thermal wave has penetrated to the right hand side).

Figure 6: Surface temperature as a function of time.


Problem 3.3-2: Laser Machining Aluminum
A laser machining process uses a very short but very high power heat flux to melt the surface of
an aluminum slab. Initially the aluminum slab is at a uniform temperature, Tin =20ºC. The
aluminum has a density of ρ =2700 kg/m3, a specific heat capacity of c =900 J/kg-K, and a
thermal conductivity of k =240 W/m-K. The heat flux from the laser is applied to the surface of
the aluminum and lasts only τlaser = 0.1 ms. You would like to heat the temperature of the
aluminum at the surface of the slab to its melting temperature, Tmelt =630ºC.
a.) Estimate how thick the slab must be in order to treat it as a semi-infinite solid.

The known inputs are entered in EES:


$UnitSystem SI MASS RAD PA K J
$TABSTOPS 0.2 0.4 0.6 0.8 3.5 in

"Inputs"
T_in=converttemp(C,K,20 [C]) "initial temperature"
T_melt=converttemp(C,K,630 [C]) "melt temperature"
rho=2700 [kg/m^3] "density"
c=900 [J/kg-K] "specific heat capacity"
k=240 [W/m-K] "conductivity"
tau_laser=0.0001 [s] "laser pulse duration"

The size of the thermal penetration wave grows with time approximately according to:

δt ≈ 2 α t (1)

alpha=k/(rho*c) "thermal diffusivity"


delta_t=2*sqrt(alpha*tau_laser) "thermal wave penetration"

which leads to δt = 0.2 mm. If the aluminum slab is much larger than 0.2 mm thick then the
heating process can be treated as occurring in a semi-infinite body.

b.) Estimate the laser heat flux (W/m2) that is required to carry out the machining process (i.e.,
to bring the material to its melting temperature after τlaser). Assume that no convection or
radiation occurs from the surface during the period of time that the laser pulse occurs.

There are two methods for answering this problem. The first is an approximate answer based on
the concept that the thermally affected material (i.e., the material within the thermal wave) acts
as a thermal resistance, Rt:

2 αt
R≈ (2)
kA

where A is the area of the wall. The heat flux required to achieve a surface temperature, Ts, equal
to Tmelt at t=τlaser is therefore:
q ′′ A =
(Tmelt − Tin ) (3)
R@ t =τ laser

or, substituting Eq. (2) into Eq. (3):

k (Tmelt − Tin )
q ′′ ≈ (4)
2 α τ laser

q``_dot_app=k*(T_melt-T_in)/(2*sqrt(alpha*tau_laser)) "approximate answer"

which leads to q ′′ =7.4x108 W/m2.

The more exact answer uses the analytical solution for the problem of a semi-infinite body
subjected to a constant surface heat flux; this analytical solution is provided by the EES function
SemiInf2 (which can be accessed by selecting Function Info from the Options menu and
selecting Transient Conduction from the pull-down menu, as shown in Figure 1).

Figure 1: Accessing the SemiInf2 function.

The temperature at x = 0 and t = τlaser must be equal to Tmelt; this can be determined according to:

T_melt=SemiInf2(T_in,q``_dot,k,alpha,0 [m],tau_laser) "exact answer"

and results in q ′′ =13.1e8 W/m2 which is 44% higher than the approximate calculation.

c.) Assume that the slab is much thicker than your answer from part (a) and that you’ve applied
a heat flux that is consistent with your answer from part (b). Sketch the temperature
distribution in the aluminum at various times. The sketch need not be exact, but should
capture the qualitative features correctly; these include the approximate temperature and
positions. Include sketches for the following times:
• at t = 0 (i.e., the onset of the laser power),
• at t= 0.05 ms (i.e., half-way through the heating process),
• at t = 0.1 ms (i.e., the end of the heating process),
• at t = 0.2 ms (i.e., 0.1 ms after the heating process has ended), and
• at t = 0.4 ms (i.e., 0.3 ms after the heating process has ended).

Figure 2: Temperature distributions (sketched) as a function of position at various times.

The sketches should look approximately like Figure 2; specifically, the following characteristics
should be evident:
• the slope of the curves at x = 0 should be the same for the t = 0.05 s and 0.10 s curves
(because the heat flux at the surface is the same).
• the slope of the curves at x = 0 should be zero (or slightly positive) for the t = 0.2 s and
0.4 s curves (because the flux is zero or, if anything, energy is being transferred back to
ambient by convection in which case the slope would be slightly positive).
• the thermal wave should continue to grow with time even after the laser is de-activated;
however, the total energy deposited in the wall does not grow with time for t > 0.2 s and
therefore the magnitude of the temperature rise should be reduced as the energy is
distributed.
PROBLEM 3.3-3: Aluminum on Glass
A dm = 5 mm thick piece of metal (with density ρm = 2700 kg/m3, specific heat capacity, cm = 900
J/kg-K, and thermal conductivity, km = 240 W/m-K) is attached to a very thick piece of glass
(with density ρg = 2600 kg/m3, specific heat capacity, cg = 800 J/kg-K, and thermal conductivity,
kg = 1.1 W/m-K), as shown in Figure P3.3-3. The interface between the metal and the glass is
characterized by a contact resistance, Rt′′,c = 5.0x10-4 K-m2/W. The face of the metal is exposed
to a constant heat flux q ′′ = 50,000 W/m2. Initially (i.e., at time, t = 0), the metal and glass are at
a uniform temperature, Tini = 20°C.

q ′′ = 50,000 W/m
2
ρm = 2700 kg/m3
km = 240 W/m-K dm = 5 mm
cm = 900 J/kg-K
metal

Rt′′,c = 5x10 K-m /W x


-4 2

ρg = 2600 kg/m3 glass


kg = 1.1 W/m-K
cg= 800 J/kg-K
Figure P3.3-3: Metal layer on a glass substrate.

a.) Calculate the time required for the thermal wave to penetrate to the back of the metal. That
is, at what time after the application of the heat flux will the temperature at the interface between
the metal and the glass begin to rise?

The thermal penetration wave will reach the back of the aluminum in approximately:

d m2
ta = = 0.06 s
4αm

b.) Sketch the temperature as a function of position, x, a times t = 0, t = 0.020 s (20 ms), t =
0.050 s (50 ms), t = 0.2 s, and t = 1.0 s. Use the coordinates shown below. Justify your
sketches as much as possible. Do not worry too much about the magnitude of the
temperatures – concentrate on the shape of the temperature distributions and the relative
temperature drop across the various portions of the system as well as the qualitative features
that the temperature distribution must exhibit.
Things I expect to see: the slope at x = 0 is always the same, at t = 0.02 s the wave has not gone
through the aluminum, at t = 0.05 s the wave has just reached the back of the aluminum. For
times greater than 0.05 s, there is a temperature drop across the contact resistance and the
thermal wave is moving into the glass. The slope on the glass side of the interface is larger than
the slope on the aluminum side because of its lower conductivity.

c.) For times greater than 0.2 s, is it possible to treat the metal as being lumped (i.e., is the
aluminum all at essentially the same temperature)? Justify your answer.

The aluminum can be treated as being lumped if the internal resistance to conduction through the
aluminum is small relative to the external resistance associated with the contact resistance and
the resistance to conduction into the glass. The Biot number is therefore:

dm
km
Bi =
2 α g ( t − ta )
Rtc′′ +
kg

Note that this Biot number will become smaller as time increases. At t = 0.2 s, the Biot number
is 0.02 so the lumped capacitance model of the aluminum is valid.

d.) Is the contact resistance an important parameter in this problem? Are there some periods of
time (e.g., short times or long times or both) where contact resistance does not play a role in
the thermal behavior of the system? Quantify the time period where contact resistance is
important.
For large times the resistance of the glass will be substantially larger than the resistance of the
contact resistance:

2 α g ( t − ta )
>> Rtc′′
kg

This occurs when:

k g2 Rtc′′2
t = ta +
4α g

or time greater than 0.21 s.

e.) Assuming that you answered yes to question (c), derive the governing differential equation
that describes the temperature of the metal as a function of time for times greater than 0.2 s.
Your answer should be a symbolic expression for the rate of temperature change for the
metal in terms of the quantities given in the problem (ρm, cm, km, ρg, cg, kg, dm, Rt′′,c , q ′′ , and
Tin) as well as the time (t) and the instantaneous temperature of the aluminum (T). DO NOT
attempt to explicitly solve the 1-D transient conduction problem associated with the glass –
instead, your solution should be approximate, based on your knowledge of how semi-infinite
bodies behave and use the concept of a penetration depth. Hint: a resistance network might
be a good way to think about this problem.

The lumped capacitance model balances energy in from the laser with energy storage in the
aluminum and energy lost to the interface and the glass.

q ′′ = d m ρ m cm
dTm
+
(T − Tin )
dt 2 α g ( t − ta )
Rtc′′ +
kg
Problem 3.3-4 (3-7 in text): Heater
A thin heater is sandwiched between two materials, A and B, as shown in Figure P3.3-4. Both
materials are very thick and so they may be considered semi-infinite. Initially, both materials are
at a uniform temperature of Tin. The heater is activated at t = 0 and delivers a uniform heat flux,
′′ , to the interface; some of this energy will be conducted into material A ( q ′′A ) and some into
qheater
material B ( q ′′A ). Materials A and B have the same thermal diffusivity, αA = αB = α. and the
same conductivity, kA = kB = k. There is no contact resistance anywhere in this problem and it is
a 1-D, transient conduction problem.

initially, all material is at Tini


thin heater at Theater

q′′A q′′B

material A is semi-infinite material B is semi-infinite


with k, α ′′
qheater with k, α
Figure P3.3-4: Thin heater sandwiched between two semi-infinite bodies.

a.) Draw a thermal resistance network that you could use to model this problem approximately.
Your resistances should be written in terms of time, t, and the symbols in the problem
′′
statement. Clearly indicate on your network where qheater is added to the network and where
the temperatures Tin and Theater are located.

Energy must be conducted from the heater into either material A or B; the appropriate resistance
network is shown in Figure 2.

Figure 2: Resistance network representing the problem.

b.) Use your resistance network from (a) to develop an equation for the heater temperature,
Theater, in terms of the symbols in the problem statement.

The heater temperature is given by:

′′
Theater = Tin + Req qheater (1)
where Req is the parallel combination of resistances RA and RB.

−1
⎡ k k ⎤
Theater = Tin + ⎢ + ′′
⎥ qheater (2)
⎢⎣ 2 α t 2 α t ⎥⎦

or

αt
Theater = Tin + ′′
qheater (3)
k

c.) Sketch on the axes below the temperature distribution at t = 0 (this one is already done) and
two additional times after the heater has been activated (t1 and t2 where t1 > t2). Label your
plots clearly. Focus on getting the qualitative features of your plot correct.

Figure 3: Sketch of temperature distribution at two times, t1 and t2.


Problem 3.3-5 (3-8 in text): Radiant Heating
Figure P3.3-5 shows a slab of material that is L = 5 cm thick and is heated from one side (x = 0)
by a radiant heat flux qs′′ = 7500 W/m2. The material has conductivity k = 2.4 W/m-K and
thermal diffusivity α = 2.2x10-4 m2/s. Both sides of the slab are exposed air at T∞ = 20°C with
heat transfer coefficient h = 15 W/m 2 -K . The initial temperature of the material is Tini = 20°C.

k = 2.4 W/m-K
initial temperature, Tini = 20°C α = 2.2x10-4 m2/s

qs′′ = 7500 W/m h = 15 W/m -K


2 2

T∞ = 20°C

h = 15 W/m -K
2

=
T∞ 20 C° L = 5 cm
x
Figure P3.3-5: Slab of material heated at one surface.

a.) About how long do you expect it to take for the temperature of the material on the unheated
side (x = L) to begin to rise?

The inputs are entered in EES:

$UnitSystem SI MASS RAD PA K J


$TABSTOPS 0.2 0.4 0.6 0.8 3.5 in

"Inputs"
k=2.4 [W/m-K] "thermal conductivity"
alpha=2.2e-4 [m^2/s] "thermal diffusivity"
h_bar=15 [W/m^2-K] "heat transfer coefficient"
q``_s=7500 [W/m^2] "heat flux at left surface"
L=5 [cm]*convert(cm,m) "thickness of slab"
A_c=1 [m^2] "area of wall"
T_ini=converttemp(C,K,20 [C]) "initial temperature"
T_infinity=converttemp(C,K,20 [C]) "ambient temperature"

The time require for the thermal wave associated with the disturbance at the left side to reach the
right side is given approximately by:

2 α t1 = L (1)

2*sqrt(alpha*time_1)=L "time required for thermal wave to reach left side"

which leads to t1 = 2.84 s.

b.) What do you expect the temperature of the material at the heated surface (x = 0) to be
(approximately) at the time identified in (a)?
The resistance network shown in Figure P3.3-5-2 can be used to approximately represent this
problem.

qs′′ As

T∞ = 20°C Tini = 20°C

Rconv Ts Rsemi −∞
Figure P3.3-5-2: Resistance network at time t1.

The convection resistance is:

1
Rconv = (2)
h Ac

and the semi-infinite body resistance at t1 is:

2 α t1
Rsemi −∞ ,t1 = (3)
k Ac

R_conv=1/(h_bar*A_c) "convection resistance"


R_semi_time1=2*sqrt(alpha*time_1)/(k*A_c) "semi-infinite body resistance at time 1"

According to Figure P3.3-5-2, the surface temperature at time t1 is given by:

qs′′ Ac =
(T
s ,t1 − T∞ ) + (T s ,t1 − Tini ) (4)
Rconv Rsemi −∞ ,t1

q``_s*A_c=(T_s_time1-T_infinity)/R_conv+(T_s_time1-T_ini)/R_semi_time1
"surface temperature at time 1"

which leads to Ts ,t1 = 412 K (139°C) .

c.) Develop a simple and approximate model that can predict the temperature at the heated
surface as a function of time for times that are less than the time calculated in (a). Plot the
temperature as a function of time from t = 0 to the time identified in (a).

The resistance network shown in Figure 3.3-5-3 is used.


qs′′ As

T∞ = 20°C Tini = 20°C

Ts
1 2 αt
h Ac k Ac
Figure P3.3-5-3: Resistance network at time t.

According to Figure P3.3-5-3:

qs′′ Ac =
(Ts − T∞ ) + (Ts ,t 1
− Tini ) (5)
1 2 αt
h Ac k Ac

q``_s*A_c=(T_s-T_infinity)/(1/(h_bar*A_c))+(T_s-T_ini)/(2*sqrt(alpha*time)/(k*A_c))
"surface temperature at time less than time 1"
T_s_C=converttemp(K,C,T_s) "in C"

Figure 3.3-5-4 illustrates the surface temperature as a function of time.


140

120
Temperature (°C)

100

80

60

40

20
0 0.5 1 1.5 2 2.5 3
Time (s)
Figure P3.3-5-4: Resistance network at time t.

d.) Sketch the temperature as a function of position in the slab for several times less than the
time identified in (a) and greater than the time identified in (a). Make sure that you get the
qualitative features of the sketch correct. Also sketch the temperature as a function of
position in the slab at steady state (make sure that you get the temperatures at either side
correct).

A thermal wave propagates from the left side of the slab. The temperature gradient at x = 0 is
always negative because energy is conducted into the slab. Further, the boundary condition at x
= 0 is:
∂T
qs′′ = h (Tx =0 − T∞ ) − k (6)
∂x x =0

therefore, the absolute value of the temperature gradient must decrease with time because more
of the energy from the radiant heating goes to convection as Tx=0 increases. After t1, the
temperature at x = L begins to rise. These characteristics are captured in Figure P3.3-5-5.

h = 15 W/m -K
2

qs′′ = 7500 W/m


2 T∞ = 20°C

h = 15 W/m -K
2

T∞ = 20°C
L = 5 cm
x
T
303°C t→∞

236°C
t > t1
139°C
t = t1

20°C
x
t < t1
Figure P3.3-5-5: Sketch of temperature as a function of x for various values of time less than t1 as well as at
steady state.

The steady state solution is obtained using the resistance network shown in Figure P3.3-5-6.

qs′′ As

qw Tx=L
T∞ = 20°C T∞ = 20°C

Ts
1 L 1
h Ac k Ac h Ac
Figure P3.3-5-6: Resistance network used to determine steady state temperature distribution.

The temperature of the surface at x = 0 at steady state is given by:


qs′′ Ac =
(T
x = 0,t →∞ − T∞ )
+
(T x = 0,t →∞ − T∞ )
(7)
1 ⎛ 1 L ⎞
⎜ + ⎟
h Ac ⎝ h Ac k Ac ⎠

The heat transfer to the wall at steady state is given by:

qw =
(T x = 0,t →∞ − T∞ )
(8)
⎛ 1 L ⎞
⎜ + ⎟
⎝ h Ac k Ac ⎠

The temperature of the surface at x = L at steady state is given by:

L
Tx = L ,t →∞ = Tx =0,t →∞ − qw (9)
k Ac

which leads to Tx =0,t →∞ = 576.9 K (303.8°C) and Tx = L ,t →∞ = 509.4 K (236.2°C).


Problem 3.3-6 (3-9 in text): Semi-infinite body exposed to convection
A semi-infinite body has conductivity k = 1.2 W/m-K and thermal diffusivity α = 5x10-4 m2/s.
At time t = 0, the surface is exposed to fluid at T∞ = 90°C with heat transfer coefficient
h = 35 W/m 2 -K . The initial temperature of the material is Tini = 20°C.
a.) Develop an approximate model that can provide the temperature of the surface and the rate of
heat transfer into the surface as a function of time.

k = 1.2 W/m-K
initial temperature, Tini = 20°C α = 5x10-4 m2/s

T∞ = 90°C
h = 35 W/m -K
2

x
Figure P3.3-6(a): Semi-infinite body exposed to a convection boundary condition.

The inputs are entered in EES:

$UnitSystem SI MASS RAD PA K J


$Tabstops 0.2 0.4 0.6 0.8 3.5

"Inputs"
k=1.2 [W/m-K] "thermal conductivity"
alpha=5e-4 [m^2/s] "thermal diffusivity"
h_bar=35 [W/m^2-K] "heat transfer coefficient"
A_c=1 [m^2] "area of wall"
T_ini=converttemp(C,K,20 [C]) "initial temperature"
T_infinity=converttemp(C,K,90 [C]) "ambient temperature"

The resistance network shown in Figure 3.3-6(b) is used to analyze this problem.

T∞ = 90°C q ′′ As Tini = 20°C

Ts
1 2 αt
h Ac k Ac
Figure P3.3-6(b): Resistance network for approximate model.

According to Figure P3.3-6(b), the heat flow into the surface is:

qs′′ Ac ≈
(T∞ − Tini ) (1)
1 2 αt
+
h Ac k Ac

and the surface temperature is:


qs′′
Ts ≈ T∞ − (2)
h

time=0 [s] "time"


q``_s*A_c=(T_infinity-T_ini)/(1/(h_bar*A_c)+2*sqrt(alpha*time)/(k*A_c)) "surface heat flux"
T_s=T_infinity-q``_s/h_bar "surface temperature"
T_s_C=converttemp(K,C,T_s) "in C"

Figure 3.3-6(c) illustrates the predicted surface temperature and heat flux as a function of time.
80 3000
approximate model
exact model
70 2500
Surface temperature (°C)

Surface heat flux (W/m )


2
60 2000

50 surface temperature 1500

40 1000

30 surface heat flux 500

20 0
0 0.5 1 1.5 2 2.5 3 3.5 4 4.5 5
Time (s)
Figure P3.3-6(c): Surface temperature and heat flux predicted by the approximate model and the exact
solution from the EES function SemiInf3 as a function of time.

b.) Based on your model, develop an expression that provides a characteristic time related to
how long will it take for the surface of the solid to approach T∞?

The temperature of the surface will approach the temperature of the fluid when the resistance to
conduction through the thermally affected zone increases to the point where it dominates the
resistance to convection from the surface. A characteristic time is defined as being the time
where these resistances are equal:

1 2 α tchar
= (3)
h Ac k Ac

or

k2
tchar = (4)
4α h 2

t_char=k^2/(4*alpha*h_bar^2) "characteristic time"

c.) Compare the results of your model from (a) with the exact solution programmed in EES and
accessed using the SemiInf3 function.
The surface temperature and heat flux are obtained using the SemiInf3 function and are also
shown in Figure 3.3-6(c).

T_s_exact=SemiInf3(T_ini,T_infinity,h_bar,k,alpha,0 [m],time) "surface temperature"


T_s_exact_C=converttemp(K,C,T_s_exact) "in C"
q``_s_exact=h_bar*(T_infinity-T_s_exact) "surface heat flux"
Problem 3.3-7 (3-10 in text)
A rod with uniform cross-sectional area, Ac = 0.1 m2 and perimeter per = 0.05 m is placed in a
vacuum environment. The length of the rod is L = 0.09 m and the external surfaces of the rod
can be assumed to be adiabatic. For a long time, a heat transfer rate of qh = 100 W is provided to
the end of the rod at x = 0. The tip of the rod at x = L is always maintained at Tt = 20°C. The rod
material has density ρ = 5000 kg/m3, specific heat capacity c = 500 J/kg-K, and conductivity k =
5 W/m-K. The rod is at a steady state operating condition when, at time t = 0, the heat transfer
rate at x = 0 becomes zero.
a.) About how long does it take for the rod to respond to the change in heat transfer?

Ac = 0.1 m2
per = 0.05 m
ρ = 5000 kg/m3
k = 5 W/m-K
c = 500 J/kg-K
Tt = 20°C
qh = 100 W

x
L = 0.09 m
Figure 3.3-7(a): Bracket supporting a heater.

The rod has a uniform cross-sectional area, Ac = 0.1 m2 and perimeter per = 0.05 m. The length
of the rod is L = 0.09 m and the external surfaces of the rod can be assumed to be adiabatic. The
heater provides a heat transfer of qh = 100 W into the end of the rod at x = 0. The tip of the rod
at x = L is always maintained at Tt = 20°C. The rod material has density ρ = 5000 kg/m3,
specific heat capacity c = 500 J/kg-K, and conductivity k = 5 W/m-K. The heater has been
operating for a long time and is at a steady state operating condition when at time t = 0, the
power is shut off so that the heat transfer rate at x = 0 becomes zero.

a.) About how long does it take for the rod to respond to the change in heat transfer?

The time response of the rod is dictated by the time required for a conduction thermal wave to
travel from x = 0 to x = L.

L2
τ diff = (1)

where α is the thermal diffusivity:

k 5W m3 kg-K
α= = = 2.0x10-6 m 2 /s (2)
ρ c m-K 5000 kg 500 J

Therefore the time constant is:


( 0.09 )
2
m2 s
τ diff = = 1013 s (3)
4 2x10-6 m 2

so it will take about 1000 s for the rod to respond.

b.) Sketch the temperature distribution you expect at t = 0 and t → ∞. Make sure that you get
the temperatures at either end of the rod and the shape of the temperature distributions
correct.

At t = 0 the temperature distribution is linear. The resistance to conduction through the rod is:

L 0.09 m m-K
Rcond = = = 0.18 K/W (4)
k Ac 5 W 0.1 m 2

so the temperature of the rod at x = 0 is:

100 W 0.18 K
Tx =0,t =0 = Tt + qh Rh = 20°C + = 38°C (5)
W

As t → ∞, the temperature everywhere in the rod approaches Tt. These characteristics are
reflected in Figure 3.3-7(b).

Tt = 20°C

x
T
t=0
t = 500 s
t = 1000 s
t = 2000 s

t→∞
20°C
x
0 0.09 m
Figure 3.3-7(b): Temperature distribution in the rod at various times.

c.) Overlay on your sketch from (b) the temperature distributions that you expect at the time that
you calculated in (a) as well as half that time and twice that time.

The thermal wave emanates from x = 0 and travels in the positive x-direction. Beyond the extent
of the thermal wave, the rod material does not realize that anything has happened and so the
temperature distribution does not change. The temperature gradient at x = 0 must be zero at all
times greater than 0 because the heat transfer there is zero. These characteristics are reflected in
Figure 3.3-7(b).

d.) Sketch the heat transfer from the rod at x = L (i.e., at the tip) as a function of time. Make sure
that your sketch clearly shows the behavior before and after the time identified in (a). Make
sure that you get the rate of heat transfer at t = 0 and t → ∞ correct.

For times less than 1000 s, the temperature distribution and therefore the temperature gradient at
x = L does not change. Therefore the rate of heat transfer is constant for t < 1000 s. For times
greater than 1000 s, the temperature gradient drops and therefore the heat transfer rate drops.
These characteristics are reflected in Figure 3.3-7(c).

q x = L
100 W

0 t
0 1000 s
Figure 3.3-7(c): Temperature distribution in the rod at various times.
Problem 3.3-8 (3-11 in text)
One technique that is being proposed for measuring the thermal diffusivity of a material is
illustrated schematically in Figure P3.3-8.

Ts = −20°C
L = 10 cm

Tin = 20°C

Figure P3.3-8: Test setup for measuring thermal diffusivity.

The material is placed in a long, insulated container and allowed to come to thermal equilibrium
with its environment Tin = 20°C. A thermocouple is embedded in the material at a distance L =
10 cm below the surface. At time t = 0 the temperature of the surface is changed from Tin to Ts =
-20°C by applying a flow of chilled ethylene glycol to the surface. The time required for the
thermocouple to change from Tin to Ttarget = 0°C is found to be ttarget = 310.2 s.
a.) What is the measured thermal diffusivity?

The known information is entered in EES:


$UnitSystem SI MASS RAD PA C J
$Tabstops 0.2 0.4 0.6 0.8 3.5

"Inputs"
L=10 [cm]*convert(cm,m) "distance between thermocouple and surface"
T_in=converttemp(C,K,20 [C]) "initial temperature"
T_s=converttemp(C,K,-20 [C]) "surface temperature"
tau=310.2 [s] "time required to reach target temperature"
T_target=converttemp(C,K,0 [C]) "target temperature"

The response of the semi-infinite body to the step change in surface temperature is provided by
the function SemiInf1 (accessed through the Function Information window):

"Part a"
T_target=SemiInf1(T_in,T_s,alpha,L,tau) "measurement of alpha"

which leads to α = 3.54e-5 m2/s.

There is some error in your measurement from part (a) due to inaccuracies in your thermocouple
and your measurement of time and position of the thermocouple. Assume that the following
uncertainties characterize your experiment:
• the temperature measurements have an uncertainty of δTin = δTtarget = δTs = 0.2°C
• the position measurement has an uncertainty of δL = 0.1 mm
• the time measurement has an uncertainty of δτtarget = 0.5 s
b.) What is the uncertainty in your measured value of thermal diffusivity from part (a)? You can
answer this question in a number of ways including using the built-in uncertainty propagation
feature in EES.

The uncertainties listed above are entered in EES:

deltaT=0.2 [K] "temperature uncertainty"


deltaL=0.1 [mm]*convert(mm,m) "position uncertainty"
deltatime=0.5 [s] "time uncertainty"

The uncertainty propagation feature in EES is used to compute the effect of the uncertainty in
each measured variable on the calculated variable. Select Uncertainty Propagation from the
Calculate menu. The variable alpha will appear as the Calculated variable; select all of the
temperatures, the length, and time as the measured variables (Figure 2).

Figure 2: Propagation of Uncertainty window.

Select Set uncertainties to bring up the window shown in Figure 3. Each of the uncertainties for
the calculated variables can be specified using variable names; enter the variables that were
defined in the Equations window.

Figure 3: Set uncertainty of measured variables

Select OK and then OK again to carry out the propagation of uncertainty calculation; the solution
will be provided in a Solution Window labeled Uncertainty Results, Figure 4.
Figure 4: Uncertainty Results Solution window.

The uncertainty in the calculated value of α is 1.0e-6 (or 3.0% of the value).
Problem 3.4-1 (3-12 in text): Laser Target
A disk shaped piece of material is used as the target of a laser, as shown in Figure P3.4-1. The
laser target is D = 5.0 mm in diameter and b = 2.5 mm thick. The target is made of a material
with ρ = 2330 kg/m3, k = 500 W/m-K, and c = 400 J/kg-K. The target is mounted on a chuck
with a constant temperature Tc = 20°C. The interface between the target and the chuck is
characterized by a contact resistance, Rc′′ = 1x10-4 K-m2/W. The target is initially in thermal
equilibrium with the chuck. You may neglect radiation and convection from the laser target.
′′ = At 2 exp ( −t / t pulse ) where A = 1x107 W/m2-s2 and
The laser flux is pulsed according to: qlaser
tpulse = 0.10 s.

laser target
ρ = 2330 kg/m3
k = 500 W/m-K chuck, Tc = 20°C
c = 400 J/kg-K

′′
laser flux, qlaser

D = 5 mm
b = 2.5 mm
contact resistance,
Rc′′ = 1x10 K-m /W
-4 2

Figure P3.4-1: Laser target

a.) Is a lumped capacitance model of the laser target appropriate? Justify your answer.

The inputs are entered in EES:

$UnitSystem SI MASS RAD PA K J


$TABSTOPS 0.2 0.4 0.6 0.8 3.5 in

"Inputs"
b=2.5 [mm]*convert(mm,m) "thickness of target"
D=5.0 [mm]*convert(mm,m) "diameter of target"
rho=2330 [kg/m^3] "density"
k=500 [W/m-K] "conductivity"
c=400 [J/kg-K] "specific heat capacity"
Rc=1e-4 [m^2-K/W] "contact resistance"
A=1e7 [W/m^2-s^2]
t_pulse=0.1 [s]
T_c=converttemp(C,K,20[C]) "chuck temperature"

The appropriate Biot number for this problem is:

Rcond 4 b π D2 b
Bi = = = (1)
Rcontact k π D 4 Rc′′ k Rc′′
2
Bi=b/(k*Rc) "Biot number"

which leads to Bi = 0.05.

b.) Use Laplace transforms to determine the temperature of the laser target as a function of time.
Prepare a plot of the temperature of the laser target as a function of time; overlay on this plot
the laser heat flux as a function of time (on a secondary y-axis).

An energy balance on the laser target leads to:

π D2 π D2 π D2 dT
′′
qlaser =
′′
(T − Tc ) + bρ c (2)
4 4 Rc 4 dt

The time constant is defined as:

τ = Rc′′ bρ c (3)

tau=Rc*b*rho*c "time constant"

and substituted into Eq. (2):

′′
dT T Tc qlaser
+ = + Rc′′ (4)
dt τ τ τ

Substituting the laser flux equation into Eq. (4) leads to:

dT T Tc Rc′′ A 2 ⎛ t ⎞
+ = + t exp ⎜ −
⎜ t pulse ⎟⎟
(5)
dt τ τ τ ⎝ ⎠

This governing differential equation is entered in Maple:

> restart;
> with(inttrans):
> GDE:=diff(T(time),time)+T(time)/tau=T_c/tau+A*time^2*exp(-time/t_pulse)*Rc/tau;
⎛⎜ − time ⎞⎟
⎜⎝ t_pulse ⎟⎠
T( time ) T_c A time 2 e
GDE := ⎜⎜⎛ T( time ) ⎟⎟⎞ +
d Rc
= +
⎝ dtime ⎠ τ τ τ

and transformed from the t to the s domain:

> GDEs:=laplace(GDE,time,s);
laplace( T( time ), time, s )
GDEs := s laplace( T( time ), time, s ) − T( 0 ) + =
τ
T_c 2 A Rc
+
τs 3
τ ⎛⎜⎜ s +
1 ⎞
⎟⎟
⎝ t_pulse ⎠

The initial condition (Tt=0 = Tc) is substituted in place of T(0) and the variable T(s) is substituted
in place of laplace(T(time),time,s):

> GDEs:=subs(laplace(T(time),time,s)=T(s),GDEs);
T( s ) T_c 2 A Rc
GDEs := s T( s ) − T( 0 ) + = +
τ τs 3
τ ⎜⎜⎛ s +
1 ⎞

⎝ t_pulse ⎟⎠
> GDEs:=subs(T(0)=T_c,GDEs);
T( s ) T_c 2 A Rc
GDEs := s T( s ) − T_c + = +
τ τs 3
τ ⎜⎜⎛ s +
1 ⎞
⎟⎟
⎝ t_pulse ⎠

The algebraic equation is solved in the s domain:

> Ts:=solve(GDEs,T(s));
Ts := ( T_c τ s4 t_pulse 3 + 3 T_c τ s3 t_pulse 2 + 3 T_c τ s2 t_pulse + T_c τ s
+ T_c s3 t_pulse 3 + 3 T_c s2 t_pulse 2 + 3 T_c s t_pulse + T_c + 2 A Rc t_pulse 3 s ) (
s ( s τ t_pulse + 3 s τ t_pulse + 3 s τ t_pulse + s τ + s t_pulse + 3 s t_pulse
4 3 3 2 2 3 3 2 2

+ 3 s t_pulse + 1 ) )

and transformed back to the time domain:

> Tt:=invlaplace(Ts,s,time);
⎛⎜ ⎛⎜ − time ⎞⎟
⎜ τ ⎟⎠
⎜ 2 2 ⎝
Tt := T_c + ⎝ 2 t_pulse τ e + ( −time 2 t_pulse 2 + 2 time 2 τ t_pulse − time 2 τ2
⎛⎜ − time ⎞⎟ ⎞
⎜ t_pulse ⎟⎠ ⎟
+ 2 t_pulse 2 τ time − 2 t_pulse τ2 time − 2 t_pulse 2 τ2 ) e ⎝ ⎟⎠ t_pulse Rc A
( −t_pulse + τ ) 3

where it can be copied and pasted into EES:

Tt := T_c+(2*t_pulse^2*tau^2*exp(-time/tau)+(-time^2*t_pulse^2+2*time^2*tau*t_pulse-
time^2*tau^2+2*t_pulse^2*tau*time-2*t_pulse*tau^2*time-2*t_pulse^2*tau^2)*exp(-time/t_pulse))/(-
t_pulse+tau)^3*t_pulse*Rc*A

The expression is modified slightly in order to be compatible with EES:


T = T_c+(2*t_pulse^2*tau^2*exp(-time/tau)+(-time^2*t_pulse^2+2*time^2*tau*t_pulse-&
time^2*tau^2+2*t_pulse^2*tau*time-2*t_pulse*tau^2*time-2*t_pulse^2*tau^2)*&
exp(-time/t_pulse))/(-t_pulse+tau)^3*t_pulse*Rc*A

The heat flux from the laser is also calculated:

q_flux=A*time^2*exp(-time/t_pulse)

A parametric table is created that includes the variables time, q_flux, and T. Figure 2 illustrates
the temperature of the laser target and the laser heat flux as a function of time.
296.5 6x104

296
5x104

295.5
Laser target temperature (K)

4x104

Laser flux (W/m2)


295 target temperature
3x104
294.5

2x104
294
laser flux

1x104
293.5

293 0
0 0.2 0.4 0.6 0.8 1 1.2 1.4 1.6 1.8 2
Time (s)
Figure 2: Laser target temperature and laser flux as a function of time
Problem 3.4-2 (3-13 in text): Semi-infinite body with an Increasing Surface Temperature
A semi-infinite piece of material with thermal diffusivity α = 1x10-5 m2/s and conductivity k = 1
W/m-K is initially (at t = 0) at Tini = 300 K when the surface temperature (i.e., the temperature at
x = 0) begins to increase linearly according to: Tx =0,t = Tin + β t where β = 1 K/s.
a.) For part (a), do not solve the problem exactly. Rather, use your conceptual knowledge of
how a thermal wave moves through a semi-infinite body in order to obtain an approximate
model for the heat flux at the surface of the surface as a function of time. Plot the
approximate heat flux as a function of time for t = 0 to 5000 s.

The given information is entered in EES:

$UnitSystem SI MASS RAD PA K J


$TABSTOPS 0.2 0.4 0.6 0.8 3.5 in

"Inputs"
alpha=1e-5 [m^2/s] "thermal diffusivity"
k = 1 [W/m-K] "conductivity of material"
T_in=300 [K]
"initial temperature of material "
beta=1 [K/s]
"rate of change of the surface temperature"

The approximate model of the heat flux at the surface treats the thermally affected region of the
semi-infinite body (of width 2 α t ) as a thermal resistance. Therefore:

q ′′x =0,t ≈
(T x = 0,t − Tin )
(1)
2 αt

or, substituting Eq. Error! Reference source not found. into Eq. (1):

β t
q ′′x =0,t ≈ (2)
2 α

qfs_app=k*beta*sqrt(time/alpha)/2 "approximate heat flux"

Figure 1 illustrates the approximate heat flux at the surface as a function of time.
Figure 1: Approximate and exact solution for the heat flux at the surface of the semi-infinite body.

b.) Use the Laplace transform technique to obtain an analytical solution to this problem.
Implement your solution in EES and prepare a plot showing the temperature as a function of
time (for t = 0 to 5000 s) at locations x = 0, 0.1 m, 0.2 m, and 0.3 m. Prepare another plot
showing the temperature as a function of position (for x = 0 to 0.5 m) for t = 0 s, 300 s, 600 s,
and 900 s.

The definition of the problem in the time domain is:

∂ 2T ∂T
α = (3)
∂x 2 ∂t

with boundary conditions:

Tx =0,t = Tin + β t (4)

Tx →∞ ,t = Tin (5)

Tx ,t =0 = Tin (6)

The governing differential equation is transformed from the x, t domain to the x, s domain using
the Laplace transform.

∂2 T 
α 2 = sT − Tin (7)
∂x
T β
Tˆx =0 = in + 2 (8)
s s

T
Tˆx →∞ = in (9)
s

The same result can be obtained using Maple:

> restart;
> with(inttrans):read(`InvLaplaceTable.m`);
> GDE:=alpha*diff(diff(T(x,t),x),x)=diff(T(x,t),t);
2
⎛∂ ⎞ ∂
GDE := α ⎜ 2 T( x, t ) ⎟⎟ = T( x, t )
⎜ ∂x
⎝ ⎠ ∂t
> GDEs:=laplace(GDE,t,s);
2
⎛∂ ⎞
GDEs := α ⎜⎜ 2 laplace( T( x, t ), t, s ) ⎟⎟ = s laplace( T( x, t ), t, s ) − T( x, 0 )
⎝ ∂x ⎠
> GDEs:=subs(laplace(T(x,t),t,s)=Th(x),GDEs);
2
⎛d ⎞
GDEs := α ⎜ 2 Th( x ) ⎟⎟ = s Th( x ) − T( x, 0 )
⎜ dx
⎝ ⎠
> GDEs:=subs(T(x,0)=T_in,GDEs);
2
⎛d ⎞
GDEs := α ⎜⎜ 2 Th( x ) ⎟⎟ = s Th( x ) − T_in
⎝ dx ⎠
> T_sur_t:=T_in+beta*t;
T_sur_t := T_in + β t
> T_sur_s:=laplace(T_sur_t,t,s);
T_in s + β
T_sur_s :=
s2

The transformed problem is solved in the s domain. Rearranging Eq. (7) and recognizing that it
involves derivatives only of x leads to:

d2T s  T
− T = − in (10)
dx 2
α α

which can be broken into a homogeneous ( û ) and particular ( v̂ ) part. The homogeneous
solution must solve:

d 2 uˆ s
− uˆ = 0 (11)
dx 2 α

which is solved by exponentials (or equivalently sinh and cosh); because there is a boundary
condition at x → ∞, it is natural to represent the solution with exponentials:
⎛ s ⎞ ⎛ s ⎞
uˆ = C1 exp ⎜⎜ x ⎟⎟ + C2 exp ⎜⎜ − x ⎟ (12)
⎝ α⎠ ⎝ α ⎟⎠

The particular solution must solve:

d 2 vˆ s T
− vˆ = − in (13)
dx 2
α α

By inspection , the particular solution is:

Tin
vˆ = (14)
s

and the solution is:

⎛ s ⎞ ⎛ s ⎞ Tin
Tˆ = C1 exp ⎜⎜ x ⎟⎟ + C2 exp ⎜⎜ − x ⎟+ (15)
⎝ α⎠ ⎝ α ⎟⎠ s

The same answer can be achieved using Maple:

> Ts:=dsolve(GDEs);
⎛ s x⎞ ⎛ s x⎞
⎜ ⎟ ⎜− ⎟
⎜ α ⎟ ⎜ α ⎟⎠ T_in
⎝ ⎠ ⎝
Ts := Th( x ) = e _C2 + e _C1 +
s

Applying the boundary condition at x → ∞, Eq. (9), leads to:

Tin ⎛ s ⎞ ⎛ s ⎞ Tin
= C1 exp ⎜⎜ ∞ ⎟⎟ + C2 exp ⎜⎜ −∞ ⎟+ (16)
s ⎝ α⎠ ⎝ α ⎟⎠ s
or

⎛ s ⎞
0 = C1 exp ⎜⎜ ∞ ⎟ (17)
⎝ α ⎟⎠

which can only be true if C1 = 0:

⎛ s ⎞ Tin
Tˆ = C2 exp ⎜⎜ − x ⎟⎟ + (18)
⎝ α ⎠ s

> Ts:=subs(_C2=0,Ts);
⎛ s x⎞
⎜− ⎟
⎜ α ⎟⎠ T_in

Ts := Th( x ) = e _C1 +
s

Applying the boundary condition at x = 0, Eq. (8), leads to:

Tin β T
+ 2 = C2 + in (19)
s s s

which leads to:

β
C2 = (20)
s2

and the solution in the s domain is:

β ⎛ s ⎞ Tin
Tˆ = 2 exp ⎜⎜ − x ⎟+ (21)
s ⎝ α ⎟⎠ s

> Ts:=subs(_C1=solve(rhs(eval(Ts,x=0))=T_sur_s,_C1),Ts);
⎛ s x⎞
⎜− ⎟
⎜ α ⎟⎠

e β T_in
Ts := Th( x ) = 2
+
s s

Equation (21) can be transformed back to the time domain by inspection of the transforms listed
in the table; specifically, the inverse Laplace transform of:

(
exp −C s ) (22)
s2

is

⎛ C2 ⎞ ⎛ C ⎞ C t ⎛ C2 ⎞
⎜t + ⎟ erfc ⎜ ⎟− exp ⎜ − ⎟ (23)
⎝ 2 ⎠ ⎝2 t ⎠ π ⎝ 4t ⎠

Replacing C with x / α leads to:

⎛ x2 ⎞ ⎛ x ⎞ βx t ⎛ x2 ⎞
Tˆ = β ⎜ t + ⎟ erfc ⎜ ⎟ − exp ⎜− ⎟ + Tin (24)
⎝ 2α ⎠ ⎜2 αt ⎟ π α ⎝ 4 α t ⎠
⎝ ⎠

> Tt:=invlaplace(rhs(Ts),s,t);
⎛ 2 ⎞
erfc⎛⎜ ⎞ β ( 2 t α + x2 )
⎜− x ⎟ x
⎜⎜ ⎟⎟
⎝ 4α t⎠ ⎜2 tα ⎟⎟
x te β 1 ⎝ ⎠
Tt := − + T_in +
α π 2 α

Either the solution obtained manually, Eq. (24), or the one from Maple can be implemented in
EES:

T1=-1/alpha^(1/2)*x*time^(1/2)*exp(-1/4/alpha*x^2/time)/pi^(1/2)*beta+T_in&
+1/2*erfc(1/2/(time*alpha)^(1/2)*x)*beta*(2*time*alpha+x^2)/alpha
"solution from Maple"
T2=beta*(time+x^2/(2*alpha))*erfc(x/(2*sqrt(alpha*time)))-beta*x*&
sqrt(time/(pi*alpha))*exp(-x^2/(4*alpha*time))+T_in
"solution from table"

The variables T1 and T2 are identical when evaluated at the same value of time and position.
Figures 2 and 3 are the plots requested in the problem statement.

Figure 2: Temperature as a function of time for various values of position.


Figure 3: Temperature as a function of position for various values of time.

c.) Overlay your exact solution onto the plot of your approximate solution from (a).

The heat flux at the surface can be obtained from the temperature solution according to:

∂T
q ′′x =0,t = − k (25)
∂x x = 0,t

which is evaluated using Maple:

> qf_s:=-k*eval(diff(Tt,x),x=0);
⎛ tβ βt ⎞
qf_s := −k ⎜ − − ⎟

⎜ α π π tα
⎝ ⎠

and copied into EES:

qfs_exact=-k*(-1/alpha^(1/2)*time^(1/2)/pi^(1/2)*beta-1/Pi^(1/2)/(time*alpha)^(1/2)*beta*time)

The exact solution is overlaid on the plot in (a); note that the exact and approximate solutions
differ by a constant factor of 4 / π ; this implies that the thermal penetration depth is always
about 50% as large as would be predicted by our simple model, perhaps because the temperature
at the surface is always rising and therefore the true time relative to the perturbation at the
surface is not really t.
Problem 3.4-3 (3-14 in text): Radiant Heating (revisited)
Solve Problem 3.3-5 (3-8 in text) using the Laplace transform technique for the period of time
where the material can be treated as a semi-infinite body.
a.) Prepare a solution and implement your solution in EES. Plot the temperature as a function of
position for several times.

The inputs are entered in EES:

$UnitSystem SI MASS RAD PA C J


$Tabstops 0.2 0.4 0.6 0.8 3.5

"Inputs"
k=2.4 [W/m-K] "conductivity"
alpha=2.2e-4 [m^2/s] "thermal diffusivity"
h_bar=15 [W/m^2-K] "heat transfer coefficient"
q``_s=7500 [W/m^2] "heat flux"
L=5 [cm]*convert(cm,m) "thickness"
A_c=1 [m^2] "per unit area"
T_ini=converttemp(C,K,20 [C]) "initial temperature"
T_infinity=converttemp(C,K,20 [C]) "ambient temperature"

The governing partial differential equation for the problem is:

∂ 2T ∂T
α 2 = (1)
∂x ∂t

where α is the thermal diffusivity. The boundary conditions are provided by:

∂T
qs′′ = −k + h (Tx =0 − T∞ ) (2)
∂x x =0

Tt =0 = Tini (3)

Tx→∞ = Tini (4)

The governing differential equation is transformed from the x, t domain to the x, s. The first term
in Eq. (1) is transformed:

∂ 2T ( x, t ) ∂ 2 T ( x, s )
α =α (5)
∂x 2 ∂x 2

The second term is transformed:

∂T ( x, t ) 
= sT ( x, s ) − Tini (6)
∂t
The transformed differential equation is:

∂ 2 T ( x, s ) 
α = sT ( x, s ) − Tini (7)
∂x 2

Equation (7) does not involve any derivative with respect to s and therefore it is an ordinary
differential equation in x; the partial differential in Eq. (7) can be changed to an ordinary
differential:

d 2 T ( s, x ) 
α 2
= sT ( s, x ) − Tini (8)
dx

which can be rearranged:



d 2T s  T
− T = − ini (9)
dx 2
α α

Equation (9) is a second order, non-homogeneous equation and therefore requires two boundary
conditions; these are obtained from Eqs. (2) and (4) which must also be transformed to the s
domain.

qs′′ dT ⎛ T ⎞
= −k + h ⎜ Tx =0 − ∞ ⎟ (10)
s dx x =0 ⎝ s ⎠

 T
Tx →∞ = ini (11)
s

The second order differential equation is split into homogeneous and particular components:
  
T = Th + Tp (12)

Equation (12) substituted into Eq. (9):


 2

d 2Th s  d Tp s  T
− Th + − Tp = − ini (13)
dx 2

α 2
dx
 α α

= 0 for homogeneous particular differential equation


differential equation

The homogeneous differential equation is:



d 2Th s 
− Th = 0 (14)
dx 2 α
which has the general solution:

⎛ s ⎞ ⎛ s ⎞
Th = C1 exp ⎜⎜ x ⎟⎟ + C2 exp ⎜⎜ − x ⎟⎟ (15)
⎝ α ⎠ ⎝ α ⎠

where C1 and C2 are undetermined constants. The solution to the particular differential equation

d 2Tp s  T
− Tp = − ini (16)
dx 2 α α

is, by inspection:

 T
Tp = ini (17)
s

Substituting Eqs. (15) and (17) into Eq. (12) leads to:

 ⎛ s ⎞ ⎛ s ⎞ Tini
T = C1 exp ⎜⎜ x ⎟⎟ + C2 exp ⎜⎜ − x ⎟⎟ + (18)
⎝ α ⎠ ⎝ α ⎠ s

The constants C1 and C2 are obtained from the boundary conditions. The boundary condition at
x→ ∞, Eq. (11), leads to:

 ⎛ s ⎞ ⎛ s ⎞ Tini Tini
Tx →∞ = C1 exp ⎜⎜ ∞ ⎟⎟ + C2 exp ⎜⎜ − ∞ ⎟⎟ + = (19)
⎝ α ⎠ ⎝ α ⎠ s s

or

⎛ s ⎞
C1 exp ⎜⎜ ∞ ⎟⎟ = 0 (20)
⎝ α ⎠

which can only be true if C1 = 0; therefore:

 ⎛ s ⎞ Tini
T = C2 exp ⎜⎜ − x ⎟⎟ + (21)
⎝ α ⎠ s

The boundary condition at x = 0, Eq. (10), leads to:

qs′′ s ⎛ T T ⎞
= C2 k + h ⎜ C2 + ini − ∞ ⎟ (22)
s α ⎝ s s ⎠
or

⎡ qs′′ + h (T∞ − Tini ) ⎤⎦


C2 = ⎣ (23)
⎛ s ⎞
s⎜k +h⎟
⎝ α ⎠

Substituting Eq. (23) into Eq. (21) leads to the solution to the problem in the x, s domain:

 ⎡ qs′′ + h (T∞ − Tini ) ⎤⎦ ⎛ s ⎞ Tini


T=⎣ exp ⎜⎜ − x⎟ + (24)
⎛ s ⎞ ⎝ α ⎟⎠ s
s⎜k +h⎟
⎝ α ⎠

The solution in the x, t domain can be obtained using the inverse Laplace transforms contained in
Table 3-3; Eq. (24) is rearranged to make this process clearer:

P a
⎛ P C

h ⎜ 1 ⎟
α exp ⎜ − x s⎟
k ⎜ α ⎟
 ⎡ qs′′ ⎤ ⎝ ⎠ + Tini
T = ⎢ + (T∞ − Tini ) ⎥ (25)
⎣h ⎦ ⎛ ⎞ s

s⎜
h
α + s⎟ ⎟
⎜Nk ⎟
 ⎝ a

a exp( − C s )
s( a + s )

The inverse Laplace transform of Eq. (25) is:

⎡ q ′′ ⎤⎡ ⎛ C ⎞ ⎛ ⎞⎤
⎟ − exp ( a C ) exp ( a t ) erfc ⎜ a t +
C
T = ⎢ s + (T∞ − Tini ) ⎥ ⎢erfc ⎜ 2
⎟ ⎥ + Tini (26)
⎣h ⎦ ⎢⎣ ⎝2 t ⎠ ⎝ 2 t ⎠ ⎥⎦

where

h
a= α (27)
k

1
C=x (28)
α

The solution is programmed in EES:


a=h_bar*sqrt(alpha)/k
C=x/sqrt(alpha)
T=(q``_s/h_bar+T_infinity-T_ini)*(erfc(C/(2*sqrt(time)))-&
exp(a*C)*exp(a^2*time)*erfc(a*sqrt(time)+C/(2*sqrt(time))))+T_ini

and evaluated a particular position and time:

time=2 [s] "time"


x_bar=1 [-] "dimensionless position"
x=x_bar*L "position"
T_C=converttemp(K,C,T) "solution converted to C"

b.) Compare the analytical solution obtained in (a) to the approximate model that you derived in
(c) of Problem 3.3-5 (3-8).

Figure P3.4-3-1 illustrates the temperature as a function of position for various values of time.
Notice that the qualitative characteristics agree with those expected in Problem P3.3-5; the
absolute value of the temperature gradient at x = 0 decreases with time and the thermal wave
moves through the solid.
90

80

70
Temperature (°C)

60
2s
50
1s
40
0.5 s
30
0.25 s
20
0 0.01 0.02 0.03 0.04 0.05
Position (m)
Figure P3.4-3-1: Temperature as a function of position at various times.

Figure P3.4-3-2 illustrates the temperature at the surface of the slab (x = 0) as a function of time
as well as the approximate solution from part (c) of Problem 3.3-5 (3-8).
140
approximate model, P3.3-5(c)
120
Temperature (°C)

100

80
exact model, P3.4-3
60

40

20
0 0.5 1 1.5 2 2.5 3
Time (s)
Figure P3.4-3-2: Temperature as a function of position at various times.
Problem 3.4-4
Reconsider Problem Problem 3.1-1. Prepare an analytical solution using the Laplace transform
technique.
a.) Plot the temperature as a function of time.

The inputs are entered in EES:

$UnitSystem SI MASS RAD PA K J


$TABSTOPS 0.2 0.4 0.6 0.8 3.5 in

"Inputs"
D=4.0 [mm]*convert(mm,m) "sensor diameter"
L=10.0 [mm]*convert(mm,m) "sensor length"
h_bar=100 [W/m^2-K] "heat transfer coefficient"
T_ini=converttemp(C,K,20) "sensor initial temperature"
rho=16600 [kg/m^3] "density"
k=47 [W/m-K] "conductivity"
c=160 [J/kg-K] "specific heat capacity"
beta=100 [K/min]*convert(K/min,K/s) "rate of temperature rise"

The volume of the sensor is:

π D2 L
V= (1)
4

and the surface area is:

π D2
As = +π D L (2)
2

The Biot number is therefore:

Vh
Bi = (3)
As k

V=pi*D^2*L/4 "sensor volume"


A_s=2*pi*D^2/4+pi*D*L "sensor surface area"
Bi=V*h_bar/(A_s*k) "Biot number"

which leads to Bi = 0.002; this is sufficiently less than one to justify the lumped capacitance
model. A control volume defined around the temperature sensor includes convection to the
chemical and the rate of energy storage:

dU
0 = qconv + (4)
dt

The rate of convection to the chemical is:


qconv = h As (T − Tchem ) (5)

and the rate of energy storage is:

dU dT
= ρV c (6)
dt dt

Substituting Eqs. (5) and (6) into Eq. (4) leads to:

dT
0 = h As (T − Tchem ) + ρ V c (7)
dt

or:

dT h As h As
+ T= T (8)
dt ρ V c ρ V c chem

Note that the time constant, τ, of the sensor is:

ρV c
τ= (9)
h As

tau=rho*V*c/(h_bar*A_s) "time constant"

which leads to:

dT T Tini β t
+ = + (10)
dt τ τ τ

In order to solve this problem using the Laplace transform approach it is necessary to transform
the governing differential equation from the t domain to the s domain. The first term in the
governing equation, Eq. (10), is transformed according to:

dT ( t ) 
= sT ( s ) − Tt =0 (3-11)
dt

The initial condition is:

Tt =0 = Tini (3-12)

Substituting Eq. (3-12) into Eq. (3-11) leads to:


dT 
= sT ( s ) − Tini (3-13)
dt

The second term in Eq. (10) becomes:



T (t ) T (s)
= (3-14)
τ τ

The final two terms in Eq. (10) are obtained from Table 3-3:

Tini Tini
= (3-15)
τ lumped sτ

and

βt β
= 2 (3-16)
τ τs

Substituting Eqs. (3-13) through (3-16) into Eq. (10) leads to the transformed governing
equation:

 T ( s ) Tini β
sT ( s ) − Tini + = + 2 (3-17)
τ sτ τ s

Notice that the differential equation in t, Eq. (10), has been transformed to an algebraic equation
in s. The same result can be obtained using Maple. The governing differential equation is
entered:

> resart:with(inttrans):
> ODEt:=diff(T(time),time)+T(time)/tau=T_ini/tau+beta*time/tau;
T( time ) T_ini β time
ODEt := ⎛⎜⎜ T( time ) ⎟⎟⎞ +
d
= +
⎝ dtime ⎠ τ τ τ

and the laplace command is used to obtain the Laplace transform of the entire differential
equation:

> AEs:=laplace(ODEt,time,s);
laplace( T( time ), time, s ) T_ini s + β
AEs := s laplace( T( time ), time, s ) − T( 0 ) + =
τ τ s2
where laplace(T(time),time,s) indicates the Laplace transform of the function T. The transformed
equation can be made more concise by using the subs command to substitute a single variable,
T(s), for the laplace() result.

> AEs:=subs(laplace(T(time),time,s)=T(s),AEs);
T( s ) T_ini s + β
AEs := s T( s ) − T( 0 ) + =
τ τ s2

The solution includes the initial condition, T(0), which can be eliminated using the subs
command again:

> AEs:=subs(T(0)=T_ini,AEs);
T( s ) T_ini s + β
AEs := s T( s ) − T_ini + =
τ τ s2

This result is identical to the result that was obtained manually, Eq. (3-17). Equation (3-17) is
solved to obtain the solution in the s domain:

 T τ s 2 + Tini s + β
T ( s ) = ini 2 (3-18)
s (τ s + 1)

The same result can be obtained using Maple:

> T(s):=solve(AEs,T(s));
T_ini τ s2 + T_ini s + β
T( s ) :=
s2 ( s τ + 1 )

In order to obtain the inverse transform of Eq. (3-18) it is necessary to break it into three
fractions:

 T τ Tini β
T ( s ) = ini + + 2 (3-19)
(τ s + 1) s (τ s + 1) s (τ s + 1)
The second term can be simplified using the method of partial fractions:

Tini C C2
= 1+ (3-20)
s (τ s + 1) s (τ s + 1)

Multiplying Eq. (3-20) by s (τ s + 1) leads to:

Tini = C1 (τ s + 1) + C2 s (3-21)
Therefore:

Tini = C1 (3-22)

and

C2 = −Tini τ (3-23)

Substituting Eqs. (3-22) and (3-23) into Eq. (3-20) leads to:

Tini T T τ
= ini − ini (3-24)
s (τ s + 1) s (τ s + 1)

The third term in Eq. (3-19) can be simplified using the method of partial fractions:

β C C C5
= 3 + 24 + (3-25)
s (τ s + 1) s s
2
(τ s + 1)

Multiplying Eq. (3-25) by s2 (τ s + 1) leads to:

β = C3 s (τ s + 1) + C4 (τ s + 1) + C5 s 2 (3-26)

which leads to:

C4 = β (3-27)

C3 = − β τ (3-28)

C5 = β τ 2 (3-29)

Subsituting Eqs. (3-27) through (3-29) into Eq. (3-25) leads to:

β −β τ β βτ2
= + + (3-30)
s 2 (τ s + 1) s s 2 (τ s + 1)

Substituting Eqs. (3-24) and (3-30) into Eq. (3-19) leads to:

 Tini τ Tini Tini τ βτ β βτ2


T (s) = + − − + + (3-31)
(τ s + 1) s (τ s + 1) s s 2 (τ s + 1)
or
 T −βτ β βτ2
T ( s ) = ini + 2+ (3-32)
s s (τ s + 1)

This expression can be obtained from Maple:

> T(s):=convert(T(s),parfrac,s);
β T_ini − β τ β τ2
T( s ) := + +
s2 s sτ+1

The solution in the time domain can be obtained by using Eq. (3-32) with Table 3-3:

 T −βτ β βτ
T ( s ) = ini + 2+ (3-33)
 s
N s ⎛ 1⎞
⎜ +
τ
⎟⎠
βt
s
Tini − β τ


⎛ t⎞
β τ exp ⎜ − ⎟
⎝ τ⎠

⎛ t⎞
T ( t ) = Tini − β τ + β t + β τ exp ⎜ − ⎟ (3-34)
⎝ τ⎠

The solution can be obtained using the invlaplace command in Maple:

> T(t):=invlaplace(T(s),s,time);
⎛− time ⎞
⎛ ⎛ ⎜⎜ ⎟⎞⎞

⎜ ⎜⎜ ⎝ τ ⎟⎠ ⎟ ⎟
⎟⎟
T( t ) := T_ini + β ⎝ time − τ ⎝ 1 − e ⎠⎠

Equation (3-34) is programmed in EES:

T=T_ini-beta*tau+beta*time+beta*tau*exp(-time/tau) "solution"
T_C=converttemp(K,C,T) "in C"
T_chem=T_ini+beta*time "temperature of chemicals"
T_chem_C=converttemp(K,C,T_chem) "in C"

Figure P3.4-4 illustrates the temperature of the sensor and the chemical as a function of time.
Note that the sensor lags the chemicals by about 20 s, which is consistent with the time constant
of the sensor, τ = 22 s.
400

350

300

Temperature (°C)
250

200 chemicals

150
sensor
100

50

0
0 20 40 60 80 100 120 140 160 180 200
Time (s)
Figure 3.4-4: Temperature of the sensor and the chemical as a function of time.

b.) Compare your answer to the answer obtained in Problem 3.1-1 and demonstrate that they are
identical.

The solution to P3.1-1 obtained by splitting the governing differential equation into its
homogeneous and particular solutions and enforcing the initial condition is:

⎡ ⎛ t ⎞ ⎤
T = Tini + β t + β τ ⎢ exp ⎜ − ⎟ − 1⎥ (35)
⎣ ⎝ τ ⎠ ⎦

which is identical to Eq. (3-34).


Problem 3.4-5 (3-15 in text)
A sphere with radius R = 1 mm is composed of material with density ρ = 9000 kg/m3, specific
heat capacity c = 500 J/kg-K, and conductivity k = 25 W/m-K. The surface is exposed to fluid at
T∞ = 25°C with heat transfer coefficient h = 1000 W/m2-K. The sphere is initially in
equilibrium with the fluid when it experiences a time varying volumetric generation of thermal
energy:
′′′ exp ⎛⎜ - ⎞⎟
t
g ′′′ = g max
⎝ a⎠
′′′ = 1x10 W/m and a = 2 s.
where g max 9 3

a.) Is a lumped capacitance solution appropriate for this problem? Justify your answer.

The inputs are entered in EES:

$UnitSystem SI MASS RAD PA K J


$TABSTOPS 0.2 0.4 0.6 0.8 3.5 in

"Inputs"
R=1 [mm]*convert(mm,m) "radius of sphere"
rho=9000 [kg/m^3] "density"
c=500 [J/kg-K] "specific heat capacity"
k=25 [W/m-K] "thermal conductivity"
T_infinity=converttemp(C,K,25[C]) "ambient temperature"
h_bar=1000 [W/m^2-K] "heat transfer coefficient"
gv_max=1e9 [W/m^3] "maximum volumetric generation"
a=2 [s] "time constant of generation"

The volume of the sensor is:

4 π R3
V= (1)
3

and the surface area is:

As = 4 π R 2 (2)

The Biot number is therefore:

Vh
Bi = (3)
As k

V=4*pi*R^3/3 "volume"
A_s=4*pi*R^2 "surface area"
Bi=V*h_bar/(A_s*k) "Biot number"

which leads to Bi = 0.013; this is sufficiently less than one to justify the lumped capacitance
model.
b.) Assume that your answer to (a) is yes. Determine an expression for the temperature as
function of time using the Laplace transform technique and implement this solution in EES.
Plot temperature as a function of time.

A control volume defined around the sphere leads to:

dU
g = qconv + (4)
dt

The rate of convection is:

qconv = h As (T − T∞ ) (5)

The rate of energy storage is:

dU dT
= ρV c (6)
dt dt

The rate of thermal energy generation is:

′′′ exp ⎛⎜ - ⎞⎟
t
g = V g max (7)
⎝ a⎠

Substituting Eqs. (5) through (7) into Eq. (4) leads to:

′′′ exp ⎛⎜ - ⎞⎟ = h As (T − T∞ ) + ρ V c
t dT
V g max (8)
⎝ a⎠ dt

or:

′′′
dT T T∞ g max ⎛ t⎞
+ = + exp ⎜ - ⎟ (9)
dt τ τ ρc ⎝ a⎠

where the time constant, τ, of the sphere is:

ρV c
τ= (10)
h As

tau=rho*V*c/(h_bar*A_s) "time constant"

The governing differential equation is transformed from the t domain to the s domain.

 T ( s ) T∞ ′′′
g max
sT ( s ) − Tini + = + (11)
τ τs ⎛ 1⎞
ρ c⎜s + ⎟
⎝ a⎠

Equation (11) is solved to obtain the solution in the s domain:

 ⎛ 1⎞ T ′′′
g max
T (s)⎜ s + ⎟ = ∞ + + Tini (12)
⎝ τ ⎠ τ s ρ c⎛ s + 1 ⎞
⎜ ⎟
⎝ a⎠

or

⎛ 1⎞
′′′ τ s + Tini ρ c τ s ⎛⎜ s + ⎞⎟
1
T∞ ρ c ⎜ s + ⎟ + g max
 ⎝ a⎠ ⎝ a⎠
T (s) = (13)
⎛ 1 ⎞⎛ 1⎞
ρ cτ s ⎜ s + ⎟ ⎜ s + ⎟
⎝ a ⎠⎝ τ ⎠

Using the method of partial fractions:

⎛ 1⎞
′′′ τ s + Tini ρ c τ s ⎛⎜ s + ⎞⎟
1
T∞ ρ c ⎜ s + ⎟ + g max
 ⎝ a⎠ ⎝ a ⎠ C1 C2 C3
T (s) = = + + (14)
⎛ 1 ⎞⎛ 1⎞ s ⎛ 1⎞ ⎛ 1⎞
ρ cτ s ⎜ s + ⎟ ⎜ s + ⎟ ⎜s+ ⎟ ⎜s+ ⎟
⎝ a ⎠⎝ τ ⎠ ⎝ a⎠ ⎝ τ ⎠

Multiplying the top and bottom of Eq. (14) through by the denomenator leads to:

⎛ 1⎞
′′′ τ s + Tini ρ c τ s ⎛⎜ s + ⎞⎟ =
1
T∞ ρ c ⎜ s + ⎟ + g max
⎝ a⎠ ⎝ a⎠
(15)
⎛ 1 ⎞⎛ 1⎞ ⎛ 1⎞ ⎛ 1⎞
C1 ρ c τ ⎜ s + ⎟ ⎜ s + ⎟ + C2 ρ c τ s ⎜ s + ⎟ + C3 ρ c τ s ⎜ s + ⎟
⎝ a ⎠⎝ τ ⎠ ⎝ τ⎠ ⎝ a⎠

or:

⎛ T ρ c τ ⎞ T∞ ρ c
Tini ρ c τ s 2 + ⎜ T∞ ρ c + g max
′′′ τ + ini ⎟s+ =
⎝ a ⎠ a
(16)
⎛ ⎛1 1⎞ 1 ⎞ ⎛ 2 s⎞ ⎛ 2 s⎞
C1 ρ c τ ⎜ s 2 + ⎜ + ⎟ s + ⎟ + C2 ρ c τ ⎜ s + ⎟ + C3 ρ c τ ⎜ s + ⎟
⎝ ⎝a τ ⎠ aτ ⎠ ⎝ τ⎠ ⎝ a⎠

Examining the s2 term in Eq. (16) leads to:

Tini = C1 + C2 + C3 (17)
Examining the s1 term in Eq. (16) leads to:

′′′ τ Tini τ
g max ⎛1 1⎞ τ
T∞ + + = C1 τ ⎜ + ⎟ + C2 + C3 (18)
ρc a ⎝a τ ⎠ a

Examining the s0 term in Eq. (16) leads to:

T∞ = C1 (19)

Equations (17) through (19) are solved in EES:

"constants"
C_1=T_infinity
T_infinity+gv_max*tau/(rho*c)+T_ini*tau/a=C_1*tau*(1/a+1/tau)+C_2+C_3*tau/a
T_ini=C_1+C_2+C_3

Taking the inverse transform of Eq. (14) leads to:

⎛ t⎞ ⎛ t⎞
T ( t ) = C1 + C2 exp ⎜ − ⎟ + C3 exp ⎜ − ⎟ (20)
⎝ a⎠ ⎝ τ⎠

"solution"
time=0 [s]
T=C_1+C_2*exp(-time/a)+C_3*exp(-time/tau)
T_C=converttemp(K,C,T)

Figure P3.4-5 illustrates the temperature as a function of time.


180

160

140
Temperature (°C)

120

100

80

60

40

20
0 1 2 3 4 5 6 7 8 9 10
Time (s)
Figure P3.4-5: Temperature as a function of time.
Problem 3.5-1 (3-16 in text): Ice Cream Containers
Ice cream containers are removed from a warehouse and loaded into a refrigerated truck. During
this loading process, the ice cream may sit on the dock for a substantial amount of time. The
dock temperature is substantially higher than the warehouse temperature, which can cause two
problems. First, the temperature of the ice cream near the surface can become elevated, resulting
in a loss of food quality. Second, the energy absorbed by the ice cream on the dock must
subsequently be removed by the equipment on the refrigerated truck, causing a substantial load
on this relatively under-sized and inefficient equipment. The ice cream is placed in cylindrical
cardboard containers. Assume that the containers are very long and therefore, the temperature
distribution of the ice cream is one dimensional), as shown in Figure P3.5-1. The inner radius of
the cardboard ice cream containers is Ro = 10 cm and the thickness of the wall is thcb = 2.0 mm.
The conductivity of cardboard is kcb = 0.08 W/m-K. The ice cream comes out of the warehouse
at Tini = 0°F and is exposed to the dock air at Tdock = 45°F with heat transfer coefficient, h = 20
W/m2-K. The ice cream has properties kic = 0.2 W/m-K, ρic = 720 kg/m3, and cic = 3200 J/kg-K.
(Assume that the ice cream does not melt).

thcb = 2 mm
Tdock = 45°F
h = 20 W/m -K
2
ice cream
kic = 0.2 W/m-K
ρic = 720 kg/m3
cic = 3200 J/kg-K

cardboard, kcb = 0.08 W/m-K

Ro = 10 cm
Figure P3.5-1: Ice cream containers
a.) Determine an effective heat transfer coefficient, heff , that can be used in conjunction with the
analytical solutions for a cylinder subjected to a step change in fluid temperature but includes
the conduction resistance associated with the cardboard as well as the convection to the air.

The inputs are entered in EES:

$UnitSystem SI MASS RAD PA K J


$TABSTOPS 0.2 0.4 0.6 0.8 3.5 in

"Inputs"
R_o=10 [cm]*convert(cm,m) "radius of container"
th_cb=2.0 [mm]*convert(mm,m) "thickness of cardboard"
k_cb=0.08 [W/m-K] "conductivity of cardboard"
k_ic=0.2 [W/m-K] "conductivity of ice cream"
rho_ic=720 [kg/m^3] "density of ice cream"
c_ic=3200 [J/kg-K] "specific heat capacity of ice cream"
h=20 [W/m^2-K] "heat transfer coefficient"
T_in=converttemp(F,K,0) "storage temperature"
T_dock=converttemp(F,K,45) "dock temperature"
L = 1 [m] "per unit length"
The thermal resistance between the surface of the ice cream and the dock air is related to
convection:

1
Rconv = (1)
h 2 π Ro L

and conduction through the cardboard:

thcb
Rcond ,cb = (2)
kcb 2 π Ro L

The effective heat transfer resistance is defined to provide the same total resistance:

1
= Rconv + Rcond ,cb (3)
heff 2 π Ro L

R_conv=1/(h*2*pi*(R_o+th_cb)*L) "convection resistance"


R_cond_cb=th_cb/(2*pi*R_o*L*k_cb) "conduction resistance of cardboard"
1/(h_eff*2*pi*R_o*L)=R_conv+R_cond_cb "effective heat transfer coefficient"

which leads to heff = 13.5 W/m2-K.

b.) If the ice cream remains on the dock for tload = 5 minutes, what will the temperature of the
surface of the ice cream be when it is loaded?

The built-in EES function for a cylinder subjected to a step change in fluid temperature is used:

T_s_load=cylinder_T(R_o, time_load, T_in, T_dock, alpha_ic, k_ic, h_eff, R_o)


"surface temperature upon loading"

which leads to Ts,load = 262.9 K (13.5°F).

c.) How much energy must be removed from the ice cream (per unit length of container) after it
is loaded in order to bring it back to a uniform temperature of Tini = 0°F?

The energy provided to the ice cream is given by the integral of the heat transfer rate to the ice
cream surface over time:
timeload

Q= ∫ 2 π Ro L heff ⎡⎣Tdock − T ( r = Ro , t ) ⎤⎦ dt (4)


0

The integrand is evaluated at a specific value of time:

time=100 [s] "time to evaluate integrand"


T_s=cylinder_T(R_o, time, T_in, T_dock, alpha_ic, k_ic, h_eff, R_o)
"instantaneous surface temperature"
q_dot=2*pi*R_o*L*h_eff*(T_dock-T_s) "instantaneous heat transfer rate to the ice cream"

The specific value of time is commented out and the INTEGRAL command is used to evaluate
the integral in Eq. (4):

{time=100 [s]} "time to evaluate integrand"


T_s=cylinder_T(R_o, time, T_in, T_dock, alpha_ic, k_ic, h_eff, R_o)
"instantaneous surface temperature"
q_dot=2*pi*R_o*L*h_eff*(T_dock-T_s) "instantaneous heat transfer rate to the ice cream"
Q=INTEGRAL(q_dot,time,0,time_load) "total heat transferred to the ice cream"

which leads to Q = 49,900 J.

d.) What is the maximum amount of time that the ice cream can sit on the dock before the ice
cream at the outer surface begins to melt?

The EES' internal function for the analytical solution is employed again; this time, the
temperature is known and the time is solved for:

T_s_max=converttemp(F,K,32.2) "maximum allowable surface


temperature"
T_s_max=cylinder_T(R_o, time_load_max, T_in, T_dock, alpha_ic, k_ic, h_eff, R_o)
"surface temperature upon loading"
time_load_max_min=time_load_max*convert(s,min) "maximum allowable loading time (min)"

which leads to timemax = 5740 s (96 min).


Problem 3.5-2 (3-17 in text)
A wall is exposed to a heat flux for a long time, as shown in Figure P3.5-2. The left side of the
wall is exposed to liquid at Tf = 20°C with a very high heat transfer coefficient; therefore, the left
side of the wall (Tx=0) always has the temperature Tf. The right side of the wall is exposed to the
heat flux and also convects to gas at Tf = 20°C but with a heat transfer coefficient of h = 5000
W/m2-K. The wall is L = 0.5 m thick and composed of a material with k = 1.0 W/m-K, ρ = 4000
kg/m3, and c = 700 J/kg-K. The wall is initially at steady state with the heat flux when, at time t
= 0, the heat flux is suddenly shut off. The wall subsequently equilibrates with the liquid and
gas, eventually it reaches a uniform temperature equal to Tf.

k = 1 W/m-K T f = 20°C
ρ = 4000 kg/m3
h = 5000 W/m -K
2
c = 700 J/kg-K

q ′′ = 5x10 W/m
5 2
T f = 20°C

x
L = 0.5 m
Figure 3.5-2: Wall exposed to a heat flux.

a.) Calculate the temperature of the right hand side of the wall at t = 0 (i.e., determine Tx=L,t=0).

The inputs are entered in EES:

$UnitSystem SI MASS RAD PA K J


$TABSTOPS 0.2 0.4 0.6 0.8 3.5 in

"Inputs"
qf_dot=500000 [W/m^2] "heat flux"
k=1 [W/m-K] "conductivity"
rho=4000 [kg/m^3] "density"
c=700 [J/kg-K] "specific heat capacity"
L=0.5 [m] "thickness of wall"
h=5000 [W/m^2-K] "heat transfer coefficient"
T_f=converttemp(C,K,20) "fluid temperature"
A=1 [m^2] "per unit area"

The steady state problem is represented by a heat flow into a thermal resistance network that
consists of two resistors in parallel; Rcond represents the resistance to heat transfer by conduction
through the wall and Rconv represents the resistance to heat transfer by convection from the right
hand surface.

L
Rcond = (1)
kA
1
Rconv = (2)
hA

The total resistance between the right hand surface of the wall and Tf is:
−1
⎡ 1 1 ⎤
Req = ⎢ + ⎥ (3)
⎣ Rcond Rconv ⎦

The temperature of the right hand side of the wall is:

Tx = L ,t =0 = T f + q ′′ A Req (4)

"Initial temperature distribution"


R_cond=L/(k*A) "conduction resistance through wall"
R_conv=1/(h*A) "convection resistance"
R_eq=(1/R_cond+1/R_conv)^(-1) "total resistance"
T_RHS=T_f+qf_dot*A*R_eq "initial temperature of the RHS of wall"

which leads to Tx=L,t=0 = 393.1 K (120°C).

b.) Sketch the temperature distribution at t = 0 and the temperature distribution as t→ ∞.

The initial temperature distribution is consistent with steady state conduction through a plane
wall; it is a linear temperature distribution from Tf at x = 0 to Tx=L,t=0 at x = L. The temperature
distribution as t→ ∞ is uniform at Tf. These are shown in Figure 2.

Figure 2: Sketch of temperature distribution at various times during the equilibration process.
c.) Sketch the temperature distribution at t = 10 s, 100 s, 1x103 s, 1x104 s, and 1x105 s. Justify
the shape of these sketches by calculating the characteristic time scales that govern the
equilibration process.

The time constant associated with the internal equilibration of the wall by conduction is:

( L / 2)
2

τ int = (5)

where

k
α= (6)
ρc

The time constant associated with the external equilibration of the wall with the surrounding gas
is:

τ ext = ρ c L A Rconv (7)

which leads to τint = 44,000 s and τext = 280 s. Based on these time scales, I would expect that
the right edge would equilibrate with the surrounding gas almost immediately but that internal
equilibration would take much longer. A thermal wave will emanate from the right edge and
travel through the wall; it will take ~100,000 s for the plane wall to finally equilibrate all the way
to its center. The intermediate sketches shown in Figure 2 are consistent with these time scales.

d.) Prepare an analytical solution for the equilibration process using separation of variables.
Implement your solution in EES and prepare a plot showing temperature as a function of
position at the times requested in part (c).

The governing partial differential equation for this situation is:

∂T ∂ 2T
−α 2 = 0 (8)
∂t ∂x

The boundary conditions are:

Tx ,t =0 = T f + (Tx = L ,t =0 − T f ) Lx (9)

Tx =0,t = T f (10)

∂T
−k = h ⎡⎣Tx = L ,t − T f ⎤⎦ (11)
∂x x = L ,t
It is possible to transform the problem so that we obtain the necessary homogeneous boundary
conditions in space:

θ = T − Tf (3-12)

The transformed partial differential equation becomes:

∂ 2θ ∂ θ
α 2 = (3-13)
∂x ∂t

θ x ,t =0 = (Tx = L ,t =0 − T f )
x
(14)
L

θ x =0,t = 0 (15)

∂θ
−k = h θ x = L ,t (16)
∂x x = L ,t

The separation of variables approach assumes that the solution can be expressed as the product
of a function only of time, Tt(t), and a function only of position, TX(x):

θ ( x, t ) = TX ( x ) Tt ( t ) (17)

which leads to the two ordinary differential equations:

d 2TX
2
+ β 2 TX = 0 (18)
dx

dTt
+ β 2 α Tt = 0 (19)
dt

The general solution to the eigenproblem is:

TX = C1 sin ( β x ) + C2 cos ( β x ) (20)

The constants are selected to satisfy the boundary conditions on TX. Substituting Eq. (20) into
Eq. (15) leads to:

C1 sin ( 0 ) + C2 cos ( 0 ) = 0 (21)

and therefore C2 = 0. Substituting C2 = 0 into Eq. (20)


TX = C1 sin ( β x ) (22)

Substituting Eq. (22) into Eq. (16) leads to:

− k C1 β cos ( β L ) = h C1 sin ( β L ) (23)

which provides the eigencondition for the problem:


tan ( β L ) = − (24)
h

or

βL
tan ( β L ) = − (25)
Bi

where Bi is the Biot number:

hL
Bi = (26)
k

Figure 3 illustrates the left and right sides of Eq. (25) as a function of β L for the case where Bi =
1.0.

Figure 3: The left and right sides of the eigencondition equation; the intersections
correspond to eigenvalues for the problem, βi L.
The intersections in Figure 3 correspond to the eigenvalues, βi L (where i = 1, 2, ...); there are an
infinite number of eigenvalues and therefore an infinite number of solutions TXi:

TX i = C1,i sin ( β i x ) (27)

The eigencondition, Eq. (25), cannot be solved explicitly for the eigenvalues. Figure 3 shows
that each successive value of βi L can be found in a well-defined interval; notice that β1 L lies
between π/2 and π, β2 L lies between 3π/2 and 2π, etc. and this will be true regardless of the
value of the Biot number. The number of terms to use in the solution is specified and an array of
appropriate guess values and upper and lower bounds for each eigenvalue is generated.

"identify eigenvalues"
N_term=10 [-] "number of terms"
duplicate i=1,N_term
lowerlimit[i]=i*pi-pi/2 "lower limit of range"
upperlimit[i]=i*pi "upper limit of range"
guess[i]=i*pi-pi/4 "guess"
end

The eigencondition is also programmed using a duplicate loop:

Bi=h*L/k "Biot number"


duplicate i=1,N_term
tan(betaL[i])=-betaL[i]/Bi "eigenvalue equation"
beta[i]=betaL[i]/L "eigenvalue"
end

The interval for each eigenvalue is controlled by selecting Variable Info from the Options menu.
Deselect the Show array variables check box at the upper left so that the arrays are collapsed to a
single entry and use the guess[], upperlimit[], and lowerlimit[] arrays to control the process of
identifying the eigenvalues in the array betaL[].

There is an ODE for Tt corresponding to each eigenvalue:

dTt
+ βi2 α Tt = 0 (28)
dt

which has the solution:

Tti = C3,i exp ( − β i2 α t ) (29)

The solution to the problem is therefore:


θ ( x, t ) = ∑ Ci sin ( βi x ) exp ( − βi2 α t ) (30)
i =1

The undetermined constants are determined by applying the initial condition, Eq. (14):

x
∑ C sin ( β x ) = T
i =1
i i x = L ,t = 0
L
(31)

Each side of Eq. (31) is multiplied by an arbitrary eigenfunction, sin(βj x), and integrated from
x=0 to x=L:

∞ L L

∑ C ∫ sin ( β x ) sin ( β x ) dx = T ∫ L sin ( β x ) dx


x
i i j x = L ,t = 0 j (32)
i =1 0 0

The property of orthogonality ensures that the only term on the left side of Eq. (33) that is not
zero is the one for which j=i:
L L
x
Ci ∫ sin ( β i x ) dx = Tx = L ,t =0 ∫ sin ( β i x ) dx
2
(34)
L

0


0

int1 int 2

The integrals can be evaluated conveniently using Maple:

> restart;
> int1:=int((sin(beta[i]*x))^2,x=0..L);
1 −cos( β i L ) sin( β i L ) + β i L
int1 :=
2 βi

> int2:=int(x*sin(beta[i]*x)/L,x=0..L);
−sin( β i L ) + cos( β i L ) β i L
int2 := − 2
L βi

The result is used to evaluate each constant and the solution in EES:

x=0.1 [m] "position"


time=10 [s] "time"
duplicate i=1,N_term
int1[i]=1/2*(-cos(beta[i]*L)*sin(beta[i]*L)+beta[i]*L)/beta[i]
int2[i]=-(-sin(beta[i]*L)+cos(beta[i]*L)*beta[i]*L)/L/beta[i]^2
C[i]=(T_RHS-T_f)*int2[i]/int1[i]
theta[i]=C[i]*sin(beta[i]*x)*exp(-beta[i]^2*alpha*time)
end
T=T_f+sum(theta[1..N_term])

Figure 4 illustrates the temperature as a function of position at the times requested in the
problem.
Figure 4: Temperature as a function of position at various values of time
Problem 3.5-3: Quenching ball bearings.
Ball bearing made from ANSI 304 stainless steel are hardened by heating them to a uniform
temperature of Tini = 850°C and then quenching in an oil bath that is maintained at T∞ = 50°C.
The diameter of the ball bearings is D = 20 mm and the heat transfer coefficient between the oil
and bearings is estimated to be h = 900 W/m2-K.
a) How much time must the ball bearings spend in the oil bath to ensure that their centers are
cooled to a temperature below Tcenter = 110°C?

The inputs are entered in EES:

$UnitSystem SI MASS RAD PA K J


$TABSTOPS 0.2 0.4 0.6 0.8 3.5 in

"known information"
D=20 [mm]*convert(mm,m) "diameter of ball bearings"
h_bar=900 [W/m^2-K] "heat transfer coefficient"
T_inf=converttemp(C,K,50[C]) "temperature of oil bath"
T_ini=converttemp(C,K,850 [C]) "initial temperature"
T_center=converttemp(C,K,110 [C]) "center temperature"

The properties of the steel (ρ, c, k, and α) are calculated at the average of the initial and target
center temperature using EES' built-in properties.

T_avg=(T_ini+T_center)/2 "temperature to evaluate properties"


rho=rho_('Stainless_AISI304', T=T_avg) "density of steel"
k=k_('Stainless_AISI304', T=T_avg) "conductivity"
c=c_('Stainless_AISI304', T=T_avg) "specific heat capacity"
alpha=k/(rho*c) "thermal diffusivity"

The Biot number that characterizes the equilibration process is:

hD
Bi = (1)
2k

Bi_a=h_bar*D/(2*k) "Biot number"

which leads to Bi = 0.41; this is sufficiently large that the ball bearings cannot be treated as
lumped. The separation of variables solution for the equilibration of a sphere that is discussed in
Section 3.5.2 and programmed in the EES function sphere_T_ND is used for the analysis. The
dimensionless temperature difference associated with Tcenter is:

θ =
(Tcenter − T∞ ) (2)
(Tini − T∞ )
and the dimensionless radius corresponding to the center of the sphere is 0.

Theta=(T_center-T_inf)/(T_ini-T_inf) "dimensionless center temperature"


r_bar=0 "center of sphere"
Theta=sphere_T_ND(r_bar, Fo_a, Bi_a) "access the solution programmed in EES"

which returns the Fourier number; the Fourier number is related to the time according to:

αt
Fo = (3)
( D / 2)
2

Fo_a=alpha*time_a/(D/2)^2 "definition of the Fourier number"

which leads to t = 48.3 s,

b) One type of ball bearing products receives a thin coating of a dielectric material before it is
hardened. This coating adds an additional resistance of 33.1 K/W at the surface of the ball
bearings with negligible effect on the mass or energy storage capacity of the ball bearings.
Does this thin coating affect the time required for quenching? If so, estimate the time to cool
the center of the bearings to 110°C from an initial temperature of 850°C.

The resistance from the surface of the ball bearing to the oil bath is the sum of the convection
resistance and coating resistance in series:

1
Rsurface = Rcoating + (4)
As h

where Rcoating is the resitance of the coating and As is the surface area of the ball bearing:

D2
As = 4 π (5)
4

R_coating=33.1 [K/W] "coating resistance"


A_surface=4*pi*D^2/4 "surface area"
R_surface=1/(h_bar*A_surface)+R_coating "resistance from surface of bearing"

The Biot number is computed according to:

Rcond
Bi = (6)
Rsurface

where Rcond is the resistance to conduction (approximate) between the center and surface of the
ball bearing:

D/2
Rcond = (7)
k Asurface
R_cond=(D/2)/(k*A_surface) "approximate conduction resistance (internal)"
Bi_b=R_cond/R_surface "Biot number"

which leads to Bi = 0.011; this is sufficiently small to justify the use of a lumped capacitance
model. The lumped capacitance time constant is:

τ lumped = M c Rsurface (8)

where M is the mass of the bearing:

M =V ρ (9)

and V is the volume:

3
4 ⎛D⎞
V = π⎜ ⎟ (10)
3 ⎝2⎠

Volume=4/3*pi*(D/2)^3 "volume of ball bearing"


m=rho*Volume "mass of ball bearing"
tau_lumped=m*c*R_surface "lumped capacitance time constant"

The lumped capacitance solution for a step change in the ambient temperature is provided in
Table 3-1:

⎛ t ⎞
T = T∞ + (Tini − T∞ ) exp ⎜ − ⎟⎟ (11)
⎜ τ lumped
⎝ ⎠

T_center=T_inf+(T_ini-T_inf)*exp(-time_b/tau_lumped) "lumped capacitance solution"

which leads to t = 1638 s. Note that the same answer could be obtained using the sphere_T_ND
function:

Theta=sphere_T_ND(r_bar, Fo_b, Bi_b) "access the solution programmed in EES"


Fo_b=alpha*time_bb/(D/2)^2 "definition of the Fourier number"

which leads to t = 1643 s.


Problem 3.5-4 (3-18 in text): Current Lead Deactivation
A current lead carries 1000's of amps of current to a superconducting magnet, as shown in Figure
P3.5-4.
g ′′′ = 3x10 W/m
5 3

h f = 1000 W/m -K
2
h f = 1000 W/m -K
2

T∞ = 20°C T∞ = 20°C

L = 10 cm
ρ = 8000 kg/m3 x
k = 10 W/m-K
c = 700 J/kg-K
Figure P3.5-4: Current lead.

The edges of the current lead are cooled by flowing water at T∞ = 20ºC with heat transfer
coefficient h f = 1000 W/m2-K. The current lead material has density ρ = 8000 kg/m3,
conductivity k = 10 W/m-K, and specific heat capacity c = 700 J/kg-K. The current causes a
uniform rate of volumetric generation of thermal energy, g ′′′ = 3x105 W/m3. The half-width of
the current lead is L = 10 cm.
a.) Determine the steady-state temperature distribution in the current lead, Tss(x). Plot the
temperature distribution.

The inputs are entered in EES:

$UnitSystem SI MASS RAD PA K J


$TABSTOPS 0.2 0.4 0.6 0.8 3.5 in

"Inputs"
gv=3e5 [W/m^3] "volumetric generation"
k=10 [W/m-K] "conductivity"
rho=8000 [kg/m^3] "density"
c=700 [J/kg-K] "specific heat capacity"
L=0.1 [m] "half-thickness of wall"
h_bar_f=1000 [W/m^2-K] "heat transfer coefficient"
T_infinity=converttemp(C,K,20) "fluid temperature"
A=1 [m^2] "per unit area"

The general solution for the 1-D steady-state temperature distribution in a plane wall exposed to
uniform thermal energy generation is provided in Table 1-3:

g ′′′ 2
Tss = − x + C1 x + C2 (1)
2k

dTss g ′′′
=− x + C1 (2)
dx k

The constants C1 and C2 are obtained using the boundary conditions:


dTss
=0 (3)
dx x =0

dTss
−k = h f (Tss , x = L − T∞ ) (4)
dx x=L

Substituting Eqs. (1) and (2) into Eqs. (3) and (4) leads to:

g ′′′ 2 g ′′′ 2 g ′′′ L


Tss = − x + L + + T∞ (5)
2k 2k hf

x_bar=0.1 [-] "dimensionless position"


x=x_bar*L "position"
T_ss=-gv*x^2/(2*k)+gv*L^2/(2*k)+gv*L/h_bar_f+T_infinity "steady-state temperature distribution"
T_ss_C=converttemp(K,C,T_ss) "in C"

The steady-state temperature as a function of position is shown in Figure P3.5-4(b).


140

120
Temperature (°C)

100

80

60

40

20
0 0.01 0.02 0.03 0.04 0.05 0.06 0.07 0.08 0.09 0.1
Position (m)
Figure P3.5-4(b): Steady-state temperature distribution.

At time t = 0 the current is deactivated so that the rate of volumetric generation in the current
lead goes to zero. The cooling water flow is also deactivated at t = 0, causing the heat transfer
coefficient at the surface to be reduced to hs = 100 W/m-K.
b.) Sketch the temperature distribution that you expect within the material at t = 0 s, t = 500 s, t
= 1000 s, t = 5000 s, and t = 10,000 s. Make sure that the qualitative characteristics of your
sketch are correct and justify them if possible.

The time constant associated with the internal equilibration due to conduction is:

L2
τ int ≈ (6)

where α is the thermal diffusivity of the material:

k
α≈ (7)
ρc

The time constant associated with external equilibration with the fluid is:

ρcL
τ ext ≈ (8)
hs

"Time constants"
h_bar_s=100 [W/m^2-K] "heat transfer coefficient after deactivation"
alpha=k/(rho*c) "thermal diffusivity"
tau_int=L^2/(4*alpha) "internal equilibration time constant"
tau_ext=c*rho*L/h_bar_s "external time constant"

which leads to τint = 700 s and τext = 5600 s. Based on these time constants, you should expect
that the material will come to equilibrium internally (i.e., reach a uniform temperature
distribution) much faster than it will come to equilibrium with the cooling water. The sketch in
Figure P3.5-4(c) is consistent with these observations.

Temperature
t=0s
t = 500 s

t = 1000 s
t = 5000 s

t = 10,000 s
T∞ Position
0 L
Figure P3.5-4(c): Sketch of tempeature as a function of position at various values of time.

c.) Sketch the rate of heat transfer per unit area to the cooling water as a function of time. Make
sure that the qualitative characteristics of your sketch are correct and justify them if possible.
Include a rough sense of the scale on the t axis.

Based on the discussion from part (b) and Figure P3.5-4(c), you should expect that initially (i.e.,
for t < τint) the surface temperature will rise as the energy from the center of the current lead
quickly is conducted to the edge. Therefore, for t < τint the rate of heat transfer to the cooling
water will rise. Eventually (i.e., for t > τext) the surface temperature will decay due to the
external equilibration process; this process will be complete at t ~ τext. These characteristics are
reflected in Figure P3.5-4(d).
Heat transfer to cooling water

T∞ τext
Time
0 τint
Figure P3.5-4(d): Sketch of heat transfer to cooling fluid as a function of time.

d.) Develop a separation of variables solution for the process. Prepare the plots requested in
parts (b) and (c) using this model.

The governing partial differential equation for this situation is:

∂T ∂ 2T
−α 2 = 0 (9)
∂t ∂x

The initial condition is the steady state temperature distribution derived in (a):

g ′′′ 2 g ′′′ 2 g ′′′ L


Tt =0 = − x + L + + T∞ (10)
2k 2k hf

At the adiabatic wall, the temperature gradient must be zero:

∂T
=0 (11)
∂x x =0

An interface energy balance at the surface (x = L) balances conduction with convection:

∂T
−k = hs [Tx = L − T∞ ] (12)
∂x x= L

In order to apply separation of variables, it is necessary that the partial differential equation and
both boundary conditions in space be homogeneous. The partial differential equation, Eq. (9) is
homogeneous and the boundary condition associated with the adiabatic wall, Eq. (11), is also
homogeneous. However, the convective boundary condition at x = L, Eq. (12), is not
homogeneous. The temperature difference relative to the fluid temperature is defined:

θ = T − T∞ (13)

The transformed partial differential equation becomes:


∂ 2θ ∂ θ
α = (14)
∂x 2 ∂ t

and the boundary conditions become:

g ′′′ 2 g ′′′ 2 g ′′′ L


θ t =0 = − x + L + (15)
2k 2k hf

∂θ
=0 (16)
∂x x =0

∂θ
−k = h θ x=L (17)
∂x x=L

Notice that both spatial boundary conditions of the transformed problem, Eqs. (16) and (17), are
homogeneous and therefore it will be possible to obtain a set of orthogonal eigenfunctions in x
using the separation of variables technique.

The separation of variables approach assumes that the solution can be expressed as the product
of a function only of time, θt(t), and a function only of position, θX(x):

θ ( x, t ) = θ X ( x ) θ t ( t ) (18)

Substituting Eq. (18) into Eq. (14) leads, eventually, to:

d 2θ X
+ λ2 θ X = 0 (19)
dx 2

and

dθ t
+ λ2 α θt = 0 (20)
dt

The general solution to Eq. (19) is:

θ X = C1 sin ( λ x ) + C2 cos ( λ x ) (21)

where C1 and C2 are unknown constants. Substituting Eq. (18) into the spatial boundary
condition at x = 0, Eq. (16), leads to:
∂θ dθ X ⎡ ⎤
= θt = θ t ⎢C1 λ cos ( λ 0 ) − C2 λ sin ( λ 0 ) ⎥ = 0 (22)
∂x dx ⎢⎣ 


x =0 x =0
=1 =0 ⎦

or C1 = 0:

θ X = C2 cos ( λ x ) (23)

Substituting Eq. (18) into the spatial boundary condition at x = L, Eq. (17), leads to:

dθ X
−k = h θ X x=L (24)
dx x=L

Substituting Eq. (23) into Eq. (24) leads to:

k C2 λ sin ( λ L ) = h C2 cos ( λ L ) (25)

which provides the eigencondition for the problem that defines multiple eigenvalues:

Bi
tan ( λ L ) = (26)
λL

where Bi is the Biot number:

hs L
Bi = (27)
k

As discussed in Section 3.5, successive value of λi L can be found in well-defined intervals. The
number of terms to use in the solution is specified and an array of appropriate guess values and
upper and lower bounds for each eigenvalue is generated.

Nterm=10 [-] "number of terms to use in the solution"


"Setup guess values and lower and upper bounds for eigenvalues"
duplicate i=1,Nterm
lowerlimit[i]=(i-1)*pi
upperlimit[i]=lowerlimit[i]+pi/2
guess[i]=lowerlimit[i]+pi/4
end

The eigencondition is also programmed using a duplicate loop:

Bi=h_bar_s*L/k "Biot number"


"Identify eigenvalues"
duplicate i=1,Nterm
tan(lambdaL[i])=Bi/lambdaL[i] "eigencondition"
lambda[i]=lambdaL[i]/L "eigenvalue"
end

The interval for each eigenvalue is controlled by selecting Variable Info from the Options menu.
Deselect the Show array variables check box at the upper left so that the arrays are collapsed to a
single entry and use the guess[], upperlimit[], and lowerlimit[] arrays to control the process of
identifying the eigenvalues in the array lambdaL[].

At this point, each of the eigenfunctions of the problem have been obtained. The ith
eigenfunction is:

θ X i = C2,i cos ( λi x ) (28)

where λi is the ith eigenvalue, identified by the eigencondition:

Bi
tan ( λi L ) = (29)
λi L

The solution to the non-homogeneous ordinary differential equation corresponding to the ith
eigenvalue, Eq. (20):

dθ ti
+ λi2 α θ ti = 0 (30)
dt

is

θ ti = C3,i exp ( −λi2 α t ) (31)

where C3,i are undetermined constants. According to Eq. (18), the solution associated with the ith
eigenvalue is:

θi = θ X i θ ti = Ci cos ( λi x ) exp ( −λi2 α t ) (32)

where the constants C2,i and C3,i have been combined to a single undetermined constant Ci.
Because the partial differential equation is linear, the sum of the solution θi for each eigenvalue,
Eq. (32), is itself a solution:

∞ ∞
θ = ∑θi = ∑ Ci cos ( λi x ) exp ( −λi2 α t ) (33)
i =1 i =1

The final step of the problem selects the constants so that the series solution satisfies the initial
condition:

g ′′′ 2 g ′′′ 2 g ′′′ L
θt =0 = ∑ Ci cos ( λi x ) = − x + L + (34)
i =1 2k 2k hf

Each side of Eq. (34) is multiplied by cos(λj x) and integrated from x = 0 to x = L. The property
of orthogonality ensures that the only term on the left side of Eq. (34) that is not zero is the one
for which j = i; therefore:

L
g ′′′ 2
L
⎛ g ′′′ 2 g ′′′ L ⎞ L
Ci ∫ cos ( λi x ) dx = − x cos ( λi x ) dx + ⎜ ⎟ cos ( λi x ) dx
2 k ∫0 h f ⎟⎠ ∫0
2
L + (35)
⎜ 2k

0


⎝ 

Integral1i Integral2i Integral3i

The integrals in Eq. (35) are evaluated using Maple:

> restart;
> Integral1:=int((cos(lambda[i]*x))^2,x=0..L);
1 cos ( λ i L ) sin( λ i L ) + λ i L
Integral1 :=
2 λi

> Integral2:=int(x^2*cos(lambda[i]*x),x=0..L);
2
λ i L 2 sin( λ i L ) − 2 sin( λ i L ) + 2 λ i L cos ( λ i L )
Integral2 := 3
λi

> Integral3:=int(cos(lambda[i]*x),x=0..L);
sin( λ i L )
Integral3 :=
λi

The results from Maple are copied and pasted into EES and used with Eq. (35) to determine the
constants associated with each eigenvalue:

duplicate i=1,Nterm
Integral1[i]=1/2*(cos(lambda[i]*L)*sin(lambda[i]*L)+lambda[i]*L)/lambda[i]
Integral2[i]=1/lambda[i]^3*(lambda[i]^2*L^2*sin(lambda[i]*L)-&
2*sin(lambda[i]*L)+2*lambda[i]*L*cos(lambda[i]*L))
Integral3[i]=1/lambda[i]*sin(lambda[i]*L)
C[i]*Integral1[i]=-gv*Integral2[i]/(2*k)+(gv*L^2/(2*k)+gv*L/h_bar_f)*Integral3[i]
end

The solution at a specific time and position is evaluated using Eq. (33).

time=1000 [s] "time"


duplicate i=1,Nterm
theta[i]=C[i]*cos(lambda[i]*x)*exp(-lambda[i]^2*alpha*time)
end
T=T_infinity+sum(theta[1..Nterm])
T_C=converttemp(K,C,T)
Figure P3.5-4(e) illustrates the temperature as a function of position at the same times requested
in (b) and behaves in the manner indicated by the sketch in Figure P3.5-4(c).
140

120
0s

100 500 s
Temperature (°C)
1000 s

80

60 5000 s

40 10000 s

20
0 0.01 0.02 0.03 0.04 0.05 0.06 0.07 0.08 0.09 0.1
Position (m)
Figure P3.5-4(e): Tempeature as a function of position at various values of time, predicted by the separation
of variables model.

The heat transfer rate to the cooling fluid can be computed according to:

dT dθ ∞
q ′′ = − k = −k = k ∑ Ci λ sin ( λi L ) exp ( −λi2 α t ) (36)
dx x= L dx x= L i =1

qf=k*sum(C[i]*lambda[i]*sin(lambda[i]*L)*exp(-lambda[i]^2*alpha*time),i=1,Nterm)

Figure P3.5-4(f) illustrates the heat flux as a function of time and behaves in the manner
indicated by the sketch in Figure P3.5-4(d).
7000
Heat flux to cooling water (W/m )
2

6000

5000

4000

3000

2000

1000
0 1000 2000 3000 4000 5000 6000 7000 8000 9000 10000
Time (s)
Figure P3.5-4(f): Heat flux to the cooling water as a function of time predicted by the separation of variables
model.
Problem 3.5-5: Current Lead Deactivation
A plane wall is initially at Tini = T∞ = 20°C when one side of the wall is exposed to a heat flux,
q ′′ = 1000 W/m2 and the other side is cooled by exposure to a fluid at T∞ = 20°C with heat
transfer coefficient h = 100 W/m2-K. The wall is L = 0.1 m thick and has density ρ = 6000
kg/m3, conductivity k = 10 W/m-K, and specific heat capacity c = 700 J/kg-K.
a.) Develop a separation of variables solution for the temperature distribution in the wall.

The inputs are entered in EES:

$UnitSystem SI MASS RAD PA K J


$TABSTOPS 0.2 0.4 0.6 0.8 3.5 in

"Inputs"
qf=1e3 [W/m^2] "heat flux"
k=10 [W/m-K] "conductivity"
rho=6000 [kg/m^3] "density"
c=700 [J/kg-K] "specific heat capacity"
L=0.1 [m] "half-thickness of wall"
h_bar=100 [W/m^2-K] "heat transfer coefficient"
T_infinity=converttemp(C,K,20) "fluid temperature"
alpha=k/(rho*c) "thermal diffusivity"

The governing partial differential equation for the problem, written in terms of a temperature
difference, θ = T - T∞ is:

∂ 2θ ∂θ
α = (1)
∂x 2 ∂t

The boundary conditions are:

∂θ
−k = h θ x=L (2)
∂x x=L

∂θ
q ′′ = −k (3)
∂x x =0

The initial condition is:

θt =0 = 0 (4)

In order to apply separation of variables it is necessary that both of the boundary conditions be
homogeneous; however, Eq. (3) is not. Therefore, the solution is assumed to be the sum of a
homogeneous and particular solution:

θ = θ h ( x, t ) + X ( x ) (5)
Substituting Eq. (5) into Eq. (1) leads to:

∂ 2θ h d 2 X ∂θ h
α + α = (6)
∂x 2 dx 2 ∂t

which is divided into the partial differential equation for θh and ordinary differential equation for
X:

∂ 2θ h ∂θ h
α = (7)
∂x 2 ∂t

and

d2X
=0 (8)
dx 2

Integrating Eq. (8) twice leads to:

X = C1 x + C2 (9)

Substituting Eq. (5) into the boundary condition at x = L, Eq. (2), leads to:

∂θ h dX
−k −k = h θ x= L + h X x=L (10)
∂x x=L ∂x x=L

which is divided into the homogeneous boundary condition for θh and the boundary condition for
X:

∂θ h
−k = h θ x=L (11)
∂x x= L

dX
−k = h X x=L (12)
∂x x=L

Substituting Eq. (9) into Eq. (12) leads to:

− k C1 = h ( C1 L + C2 ) (13)

Substituting Eq. (5) into the boundary condition at x = 0, Eq. (3), leads to:
∂θ h ∂X
q ′′ = −k −k (14)
∂x x =0 ∂x x =0

which is divided into the homogeneous boundary condition for θh and the boundary condition for
X:

∂θ h
=0 (15)
∂x x =0

∂X
q ′′ = − k (16)
∂x x =0

Substituting Eq. (9) into Eq. (16) leads to:

q ′′ = − k C1 (17)

Equations (13) and (17) are solved for C1 and C2:

"constants for particular solution"


-k*C_1=h_bar*(C_1*L+C_2)
-k*C_1=qf

and the particular solution is evaluated at a specific axial location:

x_bar=0.1 [-] "dimensionless position"


x=x_bar*L "position"
Xs=C_1*x+C_2 "particular solution"

Equation (5) is substituted into Eq. (4):

θ h ,t = 0 + X = 0 (18)

which leads to:

θ h ,t =0 = −C1 x − C2 (19)

The homogeneous solution is solved using separation of variables. The partial differential
equation, Eq. (7), is separated into θX and θt ordinary differential equations:

d 2θ X
+ λ2 θ X = 0 (20)
dx 2

dθ t
+ λ2 α θt = 0 (21)
dt
The general solution to Eq. (20) is:

θ X = C1 sin ( λ x ) + C2 cos ( λ x ) (22)

where C1 and C2 are unknown constants. Substituting Eq. (22) into the spatial boundary
condition at x = 0, Eq. (15), leads to C1 = 0.

θ X = C2 cos ( λ x ) (23)

Substituting Eq. (23)into the spatial boundary condition at x = L, Eq. (11), leads to:

k C2 λ sin ( λ L ) = h C2 cos ( λ L ) (24)

Equation (24) provides the eigencondition for the problem, which defines multiple eigenvalues:

sin ( λ L ) h
= (25)
cos ( λ L ) k λ

or, multiplying and dividing the right side of Eq. (25) by L:

sin ( λ L ) hL
= (26)
cos ( λ L ) k λ L

Writing Eq. (26) in terms of the Biot number leads to:

Bi
tan ( λ L ) = (27)
λL

where

hL
Bi = (28)
k

The eigenvalues are automatically identified in EES by specifying lower and upper limits and
guess values for each one.

Nterm=10 [-] "number of terms to use in the


solution"
"Setup guess values and lower and upper bounds for eigenvalues"
duplicate i=1,Nterm
lowerlimit[i]=(i-1)*pi
upperlimit[i]=lowerlimit[i]+pi/2
guess[i]=lowerlimit[i]+pi/4
end
"Identify eigenvalues"
Bi=h_bar*L/k
duplicate i=1,Nterm
tan(lambdaL[i])=Bi/lambdaL[i] "eigencondition"
lambda[i]=lambdaL[i]/L "eigenvalue"
end

At this point, each of the eigenfunctions of the problem have been obtained. The ith
eigenfunction is:

θ X i = C2,i cos ( λi x ) (29)

where λi is the ith eigenvalue, identified by the eigencondition:

Bi
tan ( λi L ) = (30)
λi L

The solution to the non-homogeneous ordinary differential equation corresponding to the ith
eigenvalue is

θ ti = C3,i exp ( −λi2 α t ) (31)

where C3,i are undetermined constants. The solution associated with the ith eigenvalue is:

θ h,i = θ X i θ ti = Ci cos ( λi x ) exp ( −λi2 α t ) (32)

where the constants C2,i and C3,i have been combined to a single undetermined constant Ci.
Because the partial differential equation is linear, the sum of the solution θi for each eigenvalue,
Eq. (32), is itself a solution:

∞ ∞
θ h = ∑ θi = ∑ Ci cos ( λi x ) exp ( −λi2 α t ) (33)
i =1 i =1

The final step of the problem selects the constants so that the series solution satisfies the initial
condition, Eq. (19):


θt =0 = ∑ Ci cos ( λi x ) = −C1 x − C2 (34)
i =1

Each side of Eq. (34) is multiplied by cos(λj x) and integrated from x = 0 to x = L:


L x x
Ci ∫ cos ( λi x ) dx = −C1 ∫ x cos ( λi x ) dx − C2 ∫ cos ( λi x ) dx
2
(35)

0


0


0

Integral 1 Integral 2 Integral 3

The integrals in Eq. (35) can be evaluated conveniently using Maple:

> restart;
> int((cos(lambda[i]*x))^2,x=0..L);
1 cos ( λ i L ) sin( λ i L ) + λ i L
2 λi
> int(x*cos(lambda[i]*x),x=0..L);
−1 + cos ( λ i L ) + L sin( λ i L ) λ i
2
λi
> int(cos(lambda[i]*x),x=0..L);
sin( λ i L )
λi

Substituting these results into EES provides each of the constants:

duplicate i=1,Nterm
Integral1[i]=1/2*(cos(lambda[i]*L)*sin(lambda[i]*L)+lambda[i]*L)/lambda[i]
Integral2[i]=(-1+cos(lambda[i]*L)+L*sin(lambda[i]*L)*lambda[i])/lambda[i]^2
Integral3[i]=1/lambda[i]*sin(lambda[i]*L)
C[i]*Integral1[i]=-C_1*Integral2[i]-C_2*Integral3[i]
end

The solution is evaluated at a specific time:

time = 100 [s] "time"


duplicate i=1,Nterm
theta_h[i]=C[i]*cos(lambda[i]*x)*exp(-lambda[i]^2*alpha*time)
end
theta_h=sum(theta_h[i],i=1,Nterm)
theta=theta_h+Xs
T=theta+T_infinity
T_C=converttemp(K,C,T)

b.) Overlay on one plot the temperature as a function of position in the wall for time t = 100 s,
200 s, 500 s, 1000 s, 5000 s, and 10,000 s.

The requested plot is shown in Figure P3.5-5.


38

36

34

Temperature (°C)
32 10,000 s

30
100 s 5000 s
28
200 s
26 500 s
1000 s
24

22

20
0 0.1 0.2 0.3 0.4 0.5 0.6 0.7 0.8 0.9 1
Dimensionless axial position, x/L
Figure P3.5-5: Temperature as a function of position for several values of time.
Problem 3.5-6
One side of a plane wall is exposed to fluid at T∞ = 20°C with h = 550 W/m2-K. The other side
is adiabatic. Initially the material is in equilibrium with the fluid. At time t = 0, a uniform rate
of volumetric generation of thermal energy occurs at g ′′′ = 2.5x106 W/m3. The wall material has
conductivity k = 30 W/m-K, specific heat capacity c = 670 J/kg-K, and density ρ = 5000 kg/m3.
The thickness of the wall is L = 2.5 cm.
a.) Develop a separation of variables solution for the 1-D transient situation. Note that you will
need to use the technique discussed in Section 3.5.5.

The inputs are entered in EES:

$UnitSystem SI MASS RAD PA K J


$TABSTOPS 0.2 0.4 0.6 0.8 3.5 in

"Inputs"
k=30 [W/m-K] "thermal conductivity"
rho=5000 [kg/m^3] "density"
c=670 [J/kg-K] "specific heat capacity"
h_bar=550 [W/m^2-K] "heat transfer coefficient"
g```=2.5e6 [W/m^3] "volumetric generation"
L=2.5 [cm]*convert(cm,m) "thickness of slab"
T_ini=converttemp(C,K,20 [C]) "initial temperature"
T_infinity=converttemp(C,K,20 [C]) "ambient temperature"
A=1 [m^2] "per unit area of wall"
alpha=k/(rho*c) "thermal diffusivity"

The governing partial differential equation for the problem, written in terms of a temperature
difference, θ = T - T∞ is:

∂ 2θ g ′′′ 1 ∂θ
+ = (1)
∂x 2 k α ∂t

The boundary conditions are:

∂θ
−k = h θ x=L (2)
∂x x=L

∂θ
=0 (3)
∂x x =0

The initial condition is:

θt =0 = 0 (4)
In order to apply separation of variables it is necessary that the partial differential equation be
homogeneous; however, Eq. (1) is not. Therefore, the solution is assumed to be the sum of a
homogeneous and particular solution:

θ = θ h ( x, t ) + X ( x ) (5)

Substituting Eq. (5) into Eq. (1) leads to:

∂ 2θ h d 2 X g ′′′ 1 ∂θ
+ 2 + = (6)
∂x 2 dx k α ∂t

which is divided into the partial differential equation for θh and ordinary differential equation for
X:

∂ 2θ h ∂θ h
α = (7)
∂x 2 ∂t

and

d2X g ′′′
=− (8)
dx 2
k

Integrating Eq. (8) twice leads to:

g ′′′ 2
X =− x + C1 x + C2 (9)
2k

Substituting Eq. (5) into the boundary condition at x = L, Eq. (2), leads to:

∂θ h dX
−k −k = h θ x= L + h X x=L (10)
∂x x=L ∂x x=L

which is divided into the homogeneous boundary condition for θh and the boundary condition for
X:

∂θ h
−k = h θ x=L (11)
∂x x= L

dX
−k = h X x=L (12)
∂x x=L

Substituting Eq. (9) into Eq. (12) leads to:


⎛ g ′′′ ⎞ ⎛ g ′′′ 2 ⎞
−k ⎜ − L + C1 ⎟ = h ⎜− L + C1 L + C2 ⎟ (13)
⎝ k ⎠ ⎝ 2k ⎠

Substituting Eq. (5) into the boundary condition at x = 0, Eq. (3), leads to:

∂θ h dX
+ =0 (14)
∂x x =0 dx x =0

which is divided into the homogeneous boundary condition for θh and the boundary condition for
X:

∂θ h
=0 (15)
∂x x =0

dX
=0 (16)
dx x =0

Substituting Eq. (9) into Eq. (16) leads to:

C1 = 0 (17)

Substituting Eq. (17) into Eq. (13) leads to:

g ′′′ L g ′′′ 2
C2 = + L (18)
h 2k

C_2=g```*L/h_bar+g```*L^2/k "constant for particular solution"

The particular solution is evaluated at a specific axial location:

x_bar=0 [-] "dimensionless position"


x=x_bar*L "position"
Xp=-g```*x^2/(2*k)+C_2 "particular solution"

Equation (5) is substituted into Eq. (4):

θ h ,t = 0 + X = 0 (19)

which leads to:

g ′′′ 2
θ h ,t = 0 = x − C2 (20)
2k
The homogeneous solution is solved using separation of variables. The partial differential
equation, Eq. (7), is separated into θX and θt ordinary differential equations:

d 2θ X
+ λ2 θ X = 0 (21)
dx 2

dθ t
+ λ2 α θt = 0 (22)
dt

The general solution to Eq. (21) is:

θ X = C1 sin ( λ x ) + C2 cos ( λ x ) (23)

where C1 and C2 are unknown constants. Substituting Eq. (23) into the spatial boundary
condition at x = 0, Eq. (15), leads to C1 = 0.

θ X = C2 cos ( λ x ) (24)

Substituting Eq. (24)into the spatial boundary condition at x = L, Eq. (11), leads to:

k C2 λ sin ( λ L ) = h C2 cos ( λ L ) (25)

Equation (25) provides the eigencondition for the problem, which defines multiple eigenvalues:

sin ( λ L ) h
= (26)
cos ( λ L ) k λ

or, multiplying and dividing the right side of Eq. (26) by L:

sin ( λ L ) hL
= (27)
cos ( λ L ) k λ L

Writing Eq. (27) in terms of the Biot number leads to:

Bi
tan ( λ L ) = (28)
λL

where

hL
Bi = (29)
k
The eigenvalues are automatically identified in EES by specifying lower and upper limits and
guess values for each one.

Bi=h_bar*L/k "Biot number"


Nterm=11 [-] "number of terms to use in the
solution"
"Setup guess values and lower and upper bounds for eigenvalues"
duplicate i=1,Nterm
lowerlimit[i]=(i-1)*pi
upperlimit[i]=lowerlimit[i]+pi/2
guess[i]=lowerlimit[i]+pi/4
end
"Identify eigenvalues"
duplicate i=1,Nterm
tan(lambdaL[i])=Bi/lambdaL[i] "eigencondition"
lambda[i]=lambdaL[i]/L "eigenvalue"
end

At this point, each of the eigenfunctions of the problem have been obtained. The ith
eigenfunction is:

θ X i = C2,i cos ( λi x ) (30)

where λi is the ith eigenvalue, identified by the eigencondition:

Bi
tan ( λi L ) = (31)
λi L

The solution to the non-homogeneous ordinary differential equation corresponding to the ith
eigenvalue is

θ ti = C3,i exp ( −λi2 α t ) (32)

where C3,i are undetermined constants. The solution associated with the ith eigenvalue is:

θ h,i = θ X i θ ti = Ci cos ( λi x ) exp ( −λi2 α t ) (33)

where the constants C2,i and C3,i have been combined to a single undetermined constant Ci.
Because the partial differential equation is linear, the sum of the solution θi for each eigenvalue,
Eq. (33), is itself a solution:

∞ ∞
θ h = ∑ θi = ∑ Ci cos ( λi x ) exp ( −λi2 α t ) (34)
i =1 i =1

The final step of the problem selects the constants so that the series solution satisfies the initial
condition, Eq. (20):

g ′′′ 2
θ t =0 = ∑ Ci cos ( λi x ) = x − C2 (35)
i =1 2k

Each side of Eq. (35) is multiplied by cos(λj x) and integrated from x = 0 to x = L:

g ′′′ 2
L x x
Ci ∫ cos ( λi x ) dx = −C2 ∫ cos ( λi x ) dx + x cos ( λi x ) dx
2 k ∫0
2
(36)

0


0



Integral 1 Integral 2 Integral 3

The integrals in Eq. (36) can be evaluated conveniently using Maple:

> restart;
> int((cos(lambda[i]*x))^2,x=0..L);
1 cos( λ i L ) sin( λ i L ) + λ i L
2 λi

> int(cos(lambda[i]*x),x=0..L);
sin( λ i L )
λi

> int(x^2*cos(lambda[i]*x),x=0..L);
2
λ i L 2 sin( λ i L ) − 2 sin( λ i L ) + 2 λ i L cos( λ i L )
3
λi

Substituting these results into EES provides each of the constants:

"Evaluate constants"
duplicate i=1,Nterm
Int1[i]=1/2*(cos(lambda[i]*L)*sin(lambda[i]*L)+lambda[i]*L)/lambda[i]
Int2[i]=1/lambda[i]*sin(lambda[i]*L)
Int3[i]=1/lambda[i]^3*(lambda[i]^2*L^2*sin(lambda[i]*L)-&
2*sin(lambda[i]*L)+2*lambda[i]*L*cos(lambda[i]*L))
Int1[i]*C[i]=-C_2*Int2[i]+g```*Int3[i]/(2*k)
end

The solution is evaluated at a specific time:

time=60 [s]
duplicate i=1,Nterm
theta_h[i]=C[i]*cos(lambda[i]*x)*exp(-lambda[i]^2*alpha*time)
end
theta=sum(theta_h[1..Nterm])+Xp
T=theta+T_infinity
T_C=converttemp(K,C,T)
b.) Plot the temperature as a function of time at various values of position.

The requested plot is shown in Figure 1.

Figure 1: Temperature as a function of position for several values of time.

The thermal energy generation must be activated for ta= 60 s and during this time the
temperature anywhere within the wall may not exceed 80°C.
c.) Determine the maximum allowable volumetric rate of volumetric thermal energy generation
that can be applied.

The maximum temperature within the wall will occur at x = 0; the volumetric generation that
leads to Tx=0,t = 60 s = 80°C is determined by commenting out the volumetric generation and setting
this temperature. This leads to g ′′′ = 3.5x106 W/m3.

d.) Plot the maximum allowable volumetric rate of thermal energy generation as a function of
the thickness of the wall.

Figure 2 shows requested plot.


Figure 2 Temperature as a function of position for several values of time.
Problem 3.5-7: Photomask Inspection
Figure P3.5-7 shows a wafer that is being inspected by illuminating the bottom side with
radiation while the top side is cooled. The wafer is scanned from right-to-left (in the positive y-
direction) as the inspection progresses.

h = 550 W/m -K
2

T∞ = 20°C
L = 6.5 mm u = 0.025 m/s
Tini = 20°C
y x
q s′′ = 5x10 W/m
4 2

k = 15 W/m-K Lflux = 1 mm
ρ = 2000 kg/m 3
c = 670 K/kg-K
Figure P3.5-7: A scanning wafer being inspected.

The wafer moves with velocity u = 25 mm/s. The wafer is L = 6.5 mm thick and made of
material with density ρ = 2000 kg/m3, specific heat capacity c = 670 J/kg-K, and conductivity k =
15 W/m-K. The top surface is exposed to water at T∞ = 20ºC with convection coefficient h =
550 W/m2-K. The wafer material enters the inspection region (at y = -Lflux) with a uniform
temperature Tini = T∞ = 20ºC. The heat flux applied during inspection is qs′′ = 5x104 W/m2. The
heat flux is applied at the bottom surface (x = 0) over a spatial extent Lflux = 1 mm (-Lflux < y < 0).
The bottom surface is adiabatic outside of the flux region (i.e., for y > 0).
a.) If axial conduction (i.e., conduction in the x-direction) is not important for this problem then
the problem can be re-cast as a 1-D transient problem where time, t, is equivalent to y/u.
Calculate the value of a dimensionless number that characterizes the relative importance of
the energy carried by axial conduction to the energy carried by the wafer motion.

The inputs are entered in a MATLAB script:

clear all;

k=15; % thermal conductivity (W/m-K)


rho=2000; % density (kg/m^3)
c=670; % specific heat capacity (J/kg-K)
h_bar=550; % heat transfer coefficient (W/m^2-K)
L=0.0065; % thickness of wafer (m)
qfs=50000; % surface heat flux (W/m^2)
u=0.025; % velocity (m/s)
L_flux=0.001; % length of flux region (m)

There are different ways of answering this problem. One method is outlined here. The
importance of axial conduction relative to the energy transported by the wafer motion is:

qcond
(1)
E w

where qcond is the approximate scale of the conduction heat transfer:


ΔT
qcond ≈ k LW (2)
Laxial

where W is the width of the wafer (into the page), ΔT is a characteristic temperature change, and
Laxial is a characteristic axial length. The parameter E w is the scale of the energy carried by the
wafer motion:

E w ≈ u ρ c LW ΔT (3)

Substituting Eqs. (2) and (3) into Eq. (1) leads to:

ΔT 1 α
k LW = (4)
Laxial u ρ c LW ΔT u Laxial

The characteristic axial length is the distance that the wafer moves during the time required for a
thermal wave to penetrate from one side of the wafer to the other.

Laxial = u τ diff (5)

where τdiff is the diffusion time constant:

L2
τ diff = (6)

alpha=k/(rho*c); % thermal diffusivity (m^2/s)


tau_diff=L^2/(4*alpha); % diffusive time constant (s)
L_axial=tau_diff*u; % characteristic axial distance (m)
ND=alpha/(u*L_axial) %scale of axial conduction to wafer motion (-)

which leads to qcond / E w = 0.019, which justifies neglecting axial conduction.

Assume for the remainder of this problem that your analysis in (a) showed that axial conduction
can be neglected.
b.) Is it appropriate to treat the material as a semi-infinite body (in x) for the period of time that
it is exposed to the heat flux?

The penetration depth associated with the time that the wafer material is exposed to the heat flux
is:

L flux
δt ≈ 2 α (7)
u

delta_t=2*sqrt(alpha*L_flux/u) %penetration depth in flux region (m)


which leads to δ t = 1.2 mm, which is much less than the thickness of the wafer (6.5 mm);
therefore, the wafer material is effectively semi-infinite as it passes through the flux region.

c.) Assume that your analysis in (b) showed that the material can be treated as being semi-
infinite for -Lflux < y < 0. Determine the temperature distribution in the material at the point
that it leaves the flux region (i.e., at y = 0).

The solution for a semi-infinite body exposed to a constant heat flux is provided in Table 3-2 of
the text:

qs′′ ⎡ 4 α t ⎛ x2 ⎞ ⎛ x ⎞⎤
T − Tini = ⎢ exp ⎜− ⎟ − x erfc ⎜⎜ ⎟⎟ ⎥ (8)
k ⎣⎢ π ⎝ 4α t ⎠ ⎝ 4α t ⎠ ⎦⎥

where t is the time that the material is exposed and x is the distance from the wall. For this
problem, the time that the wafer has been exposed is related to the axial distance from the
beginning of the flux zone:

t=
(y+L ) flux
(9)
u

Therefore, the temperature distribution at y = 0 is:

qs′′ ⎡ 4 α t flux ⎛ x2 ⎞ ⎛ x ⎞⎤
T − Tini = ⎢ exp ⎜ − ⎟⎟ − x erfc ⎜ ⎟⎥ (10)
k ⎢ π ⎜ 4 α t flux ⎜ 4 α t flux ⎟⎥
⎣ ⎝ ⎠ ⎝ ⎠⎦

where tflux is the total time that the wafer is exposed to the heat flux:

L flux
t= (11)
u

The following code provides the temperature as a function of x and y in the region -Lflux < y < 0:

% solution in flux region


Nxf=101;
Nyf=201;
for i=1:Nxf
for j=1:Nyf
xf(i,j)=L*(i-1)/(Nxf-1);
yf(i,j)=-L_flux*(j-1)/(Nyf-1);
time=(yf(i,j)+L_flux)/u;
Tf(i,j)=(qfs/k)*(sqrt(4*alpha*time/pi)*exp(-...
xf(i,j)^2/(4*alpha*time))-...
xf(i,j)*erfc(xf(i,j)/sqrt(4*alpha*time)));
end
end

d.) Develop a separation of variables solution in MATLAB that provides the solution for the
temperature distribution for the material outside of the flux region: 0 < x < L and y > 0. Treat
the problem as a 1-D transient problem where time is related to axial position, y. You will
need to use the function fminbnd to identify each of the eigenvalues and trapz to perform the
numerical integration that will be required to calculate each of the constants.

The transformed partial differential equation that governs this problem is:

∂ 2θ ∂ θ
α 2 = (12)
∂x ∂t

where t is given by:

y
t= (13)
u

and θ is the temperature difference relative to T∞:

θ = T − T∞ (14)

The initial condition is from Eq. (10):

qs′′ ⎡ 4 α t flux ⎛ x2 ⎞ ⎛ x ⎞⎤
θt =0 = ⎢ exp ⎜ − ⎟⎟ − x erfc ⎜ ⎟⎥ (15)
k ⎢ π ⎜ 4 α t flux ⎜ 4 α t flux ⎟⎥
⎣ ⎝ ⎠ ⎝ ⎠⎦

and the boundary conditions are:

∂θ
=0 (16)
∂x x =0

∂θ
−k = h θ x=L (17)
∂x x=L

Notice that both spatial boundary conditions of the transformed problem, Eqs. (16) and (17), are
homogeneous and therefore it will be possible to obtain a set of orthogonal eigenfunctions in x
using the separation of variables technique.

The separation of variables approach assumes that the solution can be expressed as the product
of a function only of time, θt(t), and a function only of position, θX(x):
θ ( x, t ) = θ X ( x ) θ t ( t ) (18)

Substituting Eq. (18) into Eq. (12) leads, eventually, to:

d 2θ X
+ λ2 θ X = 0 (19)
dx 2

and

dθ t
+ λ2 α θt = 0 (20)
dt

The general solution to Eq. (19) is:

θ X = C1 sin ( λ x ) + C2 cos ( λ x ) (21)

where C1 and C2 are unknown constants. Substituting Eq. (18) into the spatial boundary
condition at x = 0, Eq. (16), leads to:

∂θ dθ X ⎡ ⎤
= θt = θ t ⎢C1 λ cos ( λ 0 ) − C2 λ sin ( λ 0 ) ⎥ = 0 (22)
∂x dx ⎢⎣ 


x =0 x =0
=1 =0 ⎦

or C1 = 0:

θ X = C2 cos ( λ x ) (23)

Substituting Eq. (18) into the spatial boundary condition at x = L, Eq. (17), leads to:

dθ X
−k = h θ X x=L (24)
dx x=L

Substituting Eq. (23) into Eq. (24) leads to:

k C2 λ sin ( λ L ) = h C2 cos ( λ L ) (25)

which provides the eigencondition for the problem that defines multiple eigenvalues:

Bi
tan ( λ L ) = (26)
λL

where Bi is the Biot number:


hL
Bi = (27)
k

Bi=h_bar*L/k % Biot number (-)

The eigenvalues are determined using the function fminbnd which minimizes an objective
function within a given region. In our case, the objective function is the residual of Eq. (26):

hL
Bi = (28)
k
and the range applied to each successive call to fminbnd is chosen so that each of the eigenvalues
are solved for:

Nterm=101; % number of terms


% identify eigenvalues
for i=1:Nterm
lambdaL(i)=fminbnd(@(x) (tan(x)-Bi/x)^2,(i-1)*pi,(i-1)*pi+pi/2);
lambda(i)=lambdaL(i)/L;
end

At this point, each of the eigenfunctions of the problem have been obtained. The ith
eigenfunction is:

θ X i = C2,i cos ( λi x ) (29)

where λi is the ith eigenvalue, identified by the eigencondition:

Bi
tan ( λi L ) = (30)
λi L

The solution to the non-homogeneous ordinary differential equation corresponding to the ith
eigenvalue, Eq. (20):

dθ ti
+ λi2 α θ ti = 0 (31)
dt

is

θ ti = C3,i exp ( −λi2 α t ) (32)

where C3,i are undetermined constants. According to Eq. (18), the solution associated with the ith
eigenvalue is:

θi = θ X i θ ti = Ci cos ( λi x ) exp ( −λi2 α t ) (33)


where the constants C2,i and C3,i have been combined to a single undetermined constant Ci.
Because the partial differential equation is linear, the sum of the solution θi for each eigenvalue,
Eq. (33), is itself a solution:

∞ ∞
θ = ∑θi = ∑ Ci cos ( λi x ) exp ( −λi2 α t ) (34)
i =1 i =1

The final step of the problem selects the constants so that the series solution satisfies the initial
condition:


qs′′ ⎡ 4 α t flux ⎛ x2 ⎞ ⎛ x ⎞⎤
θt =0 = ∑ Ci cos ( λi x ) = ⎢ exp ⎜ − ⎟⎟ − x erfc ⎜ ⎟⎥ (35)
k ⎢ π ⎜ 4 α t flux ⎜ 4 α t flux ⎟⎥
i =1
⎣ ⎝ ⎠ ⎝ ⎠⎦

Each side of Eq. (35) is multiplied by cos(λj x) and integrated from x = 0 to x = L. The property
of orthogonality ensures that the only term on the left side of Eq. (35) that is not zero is the one
for which j = i; therefore:

L
q ′′ L ⎡ 4 α t flux ⎛ x2 ⎞ ⎛ x ⎞⎤
Ci ∫ cos 2 ( λi x ) dx = s ∫ ⎢ exp ⎜ − − ⎜ ⎟ ⎥ cos ( λi x ) dx
⎜ 4 α t flux ⎟⎟
x erfc (36)
k ⎢ π ⎝ ⎠ ⎜ 4 α t ⎟
flux ⎠ ⎥

0



0 ⎝

Integral 1
Integral 2

Integral 1 in Eq. (36) can be accomplished analytically. However, Integral 2 is carried out
numerically using the trapz function in MATLAB.

% get constants for no-flux region


x_int=linspace(0,L,1001)';
time_flux=L_flux/u;
for i=1:Nterm
y_int=(qfs/k)*(sqrt(4*alpha*time_flux/pi)*exp(-...
x_int.^2/(4*alpha*time_flux))-...
x_int.*erfc(x_int/sqrt(4*alpha*time_flux))).*cos(lambda(i)*x_int);
Int1=1/2*(cos(lambda(i)*L)*sin(lambda(i)*L)+lambda(i)*L)/lambda(i);
Int2=trapz(x_int,y_int);
C(i)=Int2/Int1;
end

e.) On a single plot, show the temperature distribution in the flux region (-Lflux < y < 0) and the
no-flux region (y > 0).

The MATLAB code below obtains the temperature distribution in the region after the wafer has
been exposed to the flux, y > 0.

Nxnf=101;
Nynf=501;
Lnf=0.02;
for i=1:Nxnf
for j=1:Nynf
xnf(i,j)=L*(i-1)/(Nxnf-1);
ynf(i,j)=Lnf*(j-1)/(Nynf-1);
time=ynf(i,j)/u;
Tnf(i,j)=0;
for k=1:Nterm
Tnf(i,j)=Tnf(i,j)+C(k)*cos(lambda(k)*xnf(i,j))*exp(-
lambda(k)^2*alpha*time);
end
end
end

clf;
hold on;
meshc(xnf,ynf,Tnf);
meshc(xf,yf,Tf);

Figure 2 illustrates the temperature distribution in the wafer.

Figure 2: Temperature distribution in the wafer.


Problem 3.5-8 (3-19 in text)
Reconsider Problem 3.3-7 (3-10 in text) using a separation of variables solution.
a.) Derive the governing differential equation, the boundary conditions, and the initial conditions
for the problem.

dx Tt = 20°C

x ∂T ∂T ∂ ⎡ ∂T ⎤
−k Ac −k Ac + ⎢ −k Ac dx
∂x ∂x ∂x ⎣ ∂x ⎥⎦
∂T
Ac dx ρ c
∂t
Figure P3.5-8(a): Differential energy balance.

A differential energy balance on the rod is shown in Figure 3.5-8(a) and leads to:

∂ 2T ∂T
α = (1)
∂x 2 ∂t

The boundary conditions are:

∂T
=0 (2)
∂x x =0

and

Tx = L = Tt (3)

The initial condition is the linear temperature distribution associated with steady state heat flow
through a constant cross-sectional area resistance:

qh
Tt =0 = Tt + ( L − x) (4)
k Ac

b.) Does the mathematical problem statement derived in (a) satisfy all of the requirements for a
separation of variables solution? If not, provide a simple transformation that can be applied
so that the problem can be solved using separation of variables?

The separation of variables solution requires that the partial differential equation and both spatial
boundary conditions be homogeneous. The partial differential equation, Eq. (1), is
homogeneous. However, the spatial boundary condition at x= L, Eq. (3), is not homogeneous. A
simple transformation:

θ = T − Tt (5)
will lead to a completely homogeneous problem:

∂ 2θ ∂θ
α 2 = (6)
∂x ∂t

∂θ
=0 (7)
∂x x =0

θ x= L = 0 (8)

qh
θt =0 = ( L − x) (9)
k Ac

c.) Prepare a separation of variables solution to the transformed problem from (b) and
implement your solution in EES.

The inputs are entered in EES:

$UnitSystem SI MASS RAD PA C J


$Tabstops 0.2 0.4 0.6 0.8 3.5

"Inputs"
q_dot_h=100 [W] "heater power"
A_c=0.1 [m^2] "cross-sectional area"
per=0.05 [m] "perimeter"
rho=5000 [kg/m^3] "density"
k=5 [W/m-K] "conductivity"
c=500 [J/kg-K] "specific heat capacity"
T_t=converttemp(C,K,20 [C]) "tip temperature"
L=0.09 [m] "length"
alpha=k/(rho*c) "thermal diffusivity"

The separation of variables approach assumes that the solution can be expressed as the product
of a function only of time, θt(t), and a function only of position, θX(x):

θ ( x, t ) = θ X ( x ) θ t ( t ) (10)

Substituting Eq. (10) into Eq. (6) leads, eventually, to:

d 2θ X
2
+ λ2 θ X = 0 (11)
dx

and

dθ t
+ λ2 α θt = 0 (12)
dt
The general solution to Eq. (11) is:

θ X = C1 sin ( λ x ) + C2 cos ( λ x ) (13)

where C1 and C2 are unknown constants. Substituting Eq. (10) into the spatial boundary
condition at x = 0, Eq. (7), leads to:

∂θ dθ X ⎡ ⎤
= θt = θ t ⎢C1 λ cos ( λ 0 ) − C2 λ sin ( λ 0 ) ⎥ = 0 (14)
∂x dx ⎢⎣ 


x =0 x =0
=1 =0 ⎦

or C1 = 0:

θ X = C2 cos ( λ x ) (15)

Substituting Eq. (10) into the spatial boundary condition at x = L, Eq. (8), leads to:

θ X = C2 cos ( λ L ) = 0 (16)

which provides the eigenvalues for the problem:

λi =
( 2 i − 1) π for i = 1, 2,..∞ (17)
2L

N=101 [-] "number of terms"


duplicate i=1,N
lambda[i]=(2*i-1)*pi/(2*L) "eigenvalue"
end

The solution to the non-homogeneous ordinary differential equation corresponding to the ith
eigenvalue, Eq. (12):

dθ ti
+ λi2 α θ ti = 0 (18)
dt

is

θ ti = C3,i exp ( −λi2 α t ) (19)

where C3,i is an undetermined constant. According to Eq. (10), the solution associated with the
ith eigenvalue is:
θi = θ X i θ ti = Ci cos ( λi x ) exp ( −λi2 α t ) (20)

where the constants C2,i and C3,i have been combined to a single undetermined constant Ci.
Because the partial differential equation is linear, the sum of the solution θi for each eigenvalue,
Eq. (20), is itself a solution:

∞ ∞
θ = ∑θi = ∑ Ci cos ( λi x ) exp ( −λi2 α t ) (21)
i =1 i =1

The final step of the problem selects the constants so that the series solution satisfies the initial
condition, Eq. (9):


qh
θt =0 = ∑ Ci cos ( λi x ) = ( L − x) (22)
i =1 k Ac

Each side of Eq. (22) is multiplied by cos(λj x) and integrated from x = 0 to x = L. The property
of orthogonality ensures that the only term on the left side of Eq. (22) that is not zero is the one
for which j = i; therefore:

q L q
L L L
Ci ∫ cos 2 ( λi x ) dx = h ∫ cos ( λi x ) dx − h ∫ x cos ( λi x ) dx (23)
k Ac 0 k Ac 0

0




Integral1i Integral2i Integral3i

The integrals are evaluated in Maple:

> restart;
> assume(i,integer);
> lambda=(2*i-1)*Pi/(2*L);
( 2 i~ − 1 ) π
λ=
2L
> Integral1:=int((cos(lambda*x))^2,x=0..L);
1 cos( λ L ) sin( λ L ) + λ L
Integral1 :=
2 λ
> Integral2:=int(cos(lambda*x),x=0..L);
sin( λ L )
Integral2 :=
λ
> Integral3:=int(x*cos(lambda*x),x=0..L);
−1 + cos( λ L ) + L sin( λ L ) λ
Integral3 :=
λ2

and copied into EES in order to evaluate the constant associated with each eigenvalue:
"Evaluate constants"
duplicate i=1,N
Integral1[i]=1/2*(cos(lambda[i]*L)*sin(lambda[i]*L)+lambda[i]*L)/lambda[i]
Integral2[i]=1/lambda[i]*sin(lambda[i]*L)
Integral3[i]=(-1+cos(lambda[i]*L)+L*sin(lambda[i]*L)*lambda[i])/lambda[i]^2
C[i]*Integral1[i]=q_dot_h*L*Integral2[i]/(k*A_c)-q_dot_h*Integral3[i]/(k*A_c)
end

The solution is evaluated at a particular time and position defined based on a dimensionless time
and position. The dimensionless position is normalized relative to length:

x_bar=0.5 [-] "dimensionless position"


x=x_bar*L "position"

The dimensionless time is normalized relative to the diffusive time constant:

L2
τ diff = (24)

tau_diff=L^2/(4*alpha) "diffusive time constant"


time_bar=0.25 [-] "dimensionless time"
time=time_bar*tau_diff "time"

The solution is evaluated according to:

duplicate i=1,N
theta[i]=C[i]*cos(lambda[i]*x)*exp(-lambda[i]^2*alpha*time)
end
theta=sum(theta[i],i=1,N)
T=theta+T_t
T_C=converttemp(K,C,T)

d.) Prepare a plot of the temperature as a function of position for t = 0 and t → ∞ as well as the
times requested in Problem 3.3-7 (3-10) part (c).

Figure P3.5-8(b) illustrates the temperature distribution at the times requested in the problem
statement.
40

t=0
35

Temperature (°C)
t/τdiff = 0.25

30 t/τdiff = 0.5

t/τdiff = 1

25
t/τdiff = 2

20 t→∞

0 0.1 0.2 0.3 0.4 0.5 0.6 0.7 0.8 0.9 1


Dimensionless position, x/L
Figure 3.5-8(b): Temperature as a function of position for several times.
Problem 3.5-9
Small spheres are injected into flows in order to act as ‘tracers’ for particle-image velocimetry.
The idea is to capture two images of the flow that are closely separated in time; by evaluating the
distance and direction that each particle has traveled between the images it is possible to back out
the velocity of the particle. Your company wants to introduce tracer particles that have optical
properties that are strongly affected by temperature. The intensity of the particle images will
therefore be related to temperature; in this way you can simultaneously measure the velocity and
temperature distribution of a flow. The tracer particles are spherical with radius rout = 1.0 mm.
The material has properties k = 7.0 W/m-K, ρ = 2300 kg/m3, and c=750 J/kg-K. They are to be
used in a liquid flow and the heat transfer coefficient between the fluid and the sphere is
approximately h = 5000 W/m2-K.
a.) What is the Biot number associated with tracers? Can you treat them as a lumped
capacitance?

The known information is entered in EES:


$UnitSystem SI MASS RAD PA K J
$TABSTOPS 0.2 0.4 0.6 0.8 3.5 in

"Inputs"
r_out=1.0 [mm]*convert(mm,m) "radius of tracer"
k = 7 [W/m-K] "conductivity"
rho=2300 [kg/m^3] "density"
c=750 [J/kg-K] "specific heat capacity"
h_bar=5000 [W/m^2-K] "heat transfer coefficient"

The Biot number is given by:

h rout
Bi = (1)
k
Bi=h_bar*r_out/k "Biot number"

which leads to Bi= 0.71; the lumped capacitance solution is not appropriate for this problem.

b.) Estimate a diffusive and lumped capacitance time constant for the sphere.

The diffusive time constant is, approximately, the time required for energy to be conducted from
the center to the edge of the sphere:

rout
2
τ diff = (2)

tau_diff=r_out^2/(4*alpha) "diffusive time constant"

which leads to τdiff = 0.06 s. The lumped capacitance time constant is:
Vρc
τ lumped = (3)
h As

where V is volume:

4 3
V = π rout (4)
3

and As is surface area:

As = 4 π rout
2
(5)

V=4*pi*r_out^3/3 "volume of sphere"


A_s=4*pi*r_out^2 "surface area of sphere"
tau_lumped=V*rho*c/(h_bar*A_s) "lumped capacitance time constant"

which leads to τdiff = 0.12 s.

c.) Assume that your answer from (a) shows that a lumped capacitance solution is NOT
appropriate. Use the exact solution for a sphere exposed to a step change in the convective
surface condition in order to prepare a plot of the surface temperature and center temperature
of the sphere as a function of time for the case where the sphere is initially at a temperature
of Tini = 20°C when at time t=0 the fluid temperature is changed to T∞ = 40°C.

The solution for the surface (Ts) and center (Tc) temperatures are obtained from the sphere_T
function in EES:

T_ini=converttemp(C,K,20[C]) "initial temperature"


T_infinity=converttemp(C,K,40 [C]) "fluid temperature"
time=0.1 [s] "time"
alpha=k/(rho*c) "thermal diffusivity"
T_s=sphere_T(r_out, time, T_ini, T_infinity, alpha, k, h_bar, r_out) "surface temperature"
T_s_C=converttemp(K,C,T_s) "in C"
T_c=sphere_T(0 [m], time, T_ini, T_infinity, alpha, k, h_bar, r_out) "center temperature"
T_c_C=converttemp(K,C,T_c) "in C"

Figure 1 illustrates the center and surface temperature as a function of time.


40

37.5

35
surface

Temperature (°C)
32.5

30
average
27.5 center

25

22.5

20

17.5
0 0.05 0.1 0.15 0.2 0.25 0.3 0.35 0.4 0.45 0.5
Time (s)
Figure 1: Center, surface, and average temperature as a function of time.

d.) Explain how your plot from (c) is consistent with the time constants that you calculated in
(b).

The center temperature lags the surface temperature by about τdiff - the time required for energy
to be conducted to the center. The sphere requires several time τlumped to reach equilibrium with
the environment.

e.) Overlay on your plot from (c) the average temperature of the sphere in the material as a
function of time. Use the Integral command in EES to compute the average temperature.
What is the time constant associated with the temperature measurement?

The average temperature is computed according to:


rout
1
T = ∫ 4π r T dr
2
(6)
V 0

The integrand is evaluated initially at some arbitrary value of r. The temperature is obtained
using the sphere_T function.

r=0.0005 [m] "radius (arbitrary)"


T = sphere_T(r, time, T_ini, T_infinity, alpha, k, h_bar, r_out) "temperature"
Integrand=4*pi*r^2*T "integrand"

The arbitrary value of r is commented out and the Integral command is used to carry out the
integration numerically.

{r=0.0005 [m]} "radius (arbitrary)"


T_bar=Integral(Integrand,r,0,r_out)/V "average temperature"
T_bar_C=converttemp(K,C,T_bar) "in C"

The average temperature as a function of time is shown in Figure 1. The time constant
associated with the temperature measurement is approximately 0.16 s according to Figure 1.
Problem 3.5-10
You are in charge of a processing line that freezes D = 3 inch diameter spheres of dough as they
emerge from a mixing chamber. The dough has conductivity k = 0.5 W/m-K and thermal
diffusivity α = 1.4x10-7 m2/s. The dough balls leave the mixing chamber at a uniform
temperature, Tini = 35ºC. The balls are exposed to very cold air at T∞ = -50ºC with heat transfer
coefficient h = 20 W/m2-K.
a.) Is a lumped capacitance model appropriate for this problem?

The inputs are entered in EES:

$UnitSystem SI MASS RAD PA K J


$TABSTOPS 0.2 0.4 0.6 0.8 3.5 in

"Inputs"
k=0.5 [W/m-K] "conductivity"
alpha=1.4e-7 [m^2/s] "thermal diffusivity"
D=3 [inch]*convert(inch,m) "diameter"
h_bar=20 [W/m^2-K] "heat transfer coefficient"
T_ini=converttemp(C,K,35 [C]) "initial temperature"
T_infinity=converttemp(C,K,-50 [C]) "freezer temperature"

The Biot number is:

Dh
Bi = (1)
2k

Bi=(D/2)*h_bar/k "Biot number"

which leads to Bi = 1.52; therefore, a lumped capacitance model is not appropriate.

b.) Assume that your answer to (a) is no. Use the sphere_T function in EES to plot the center
and surface temperature of the dough as a function of time.

The surface (Ts) and center (Tc) temperatures are obtained according to:

time = 5000 [s] "time"


T_s=sphere_T(D/2, time, T_ini, T_infinity, alpha, k, h_bar, D/2) "surface temperature"
T_s_C=converttemp(K,C,T_s) "in C"
T_c=sphere_T(0 [m], time, T_ini, T_infinity, alpha, k, h_bar, D/2) "center temperature"
T_c_C=converttemp(K,C,T_c) "in C"

A parametric table is created and used to generate Figure 1, which shows the center and surface
temperature of the dough as a function of time.
40

30

20

Temperature (°C)
10 center

-10

-20
surface
-30

-40

-50
0 2000 4000 6000 8000 10000
Time (s)
Figure 1: Center and surface temperature of dough as a function of time.

c.) The processing can be terminated when the center temperature reaches the target temperature
of Tfreeze = -5ºC. Use EES to determine the time required for the processing.

The set time is commented out and the center temperature is set equal to the target temperature:

{time = 5000 [s]} "time"


T_freeze=converttemp(C,K,-5 [C]) "freezing temperature"
T_c=T_freeze "center becomes frozen"
time_min=time*convert(s,min) "time, in min"

which leads to time = 48.8 min.

d.) What is the efficiency of the freezing process? The efficiency is defined as the ratio of the
minimum possible amount of energy that must be removed from the dough ball to bring it to
a uniform temperature of Tfreeze to the actual amount of heat removed from dough ball at the
time identified in (c). You may want to use the function sphere_Q in EES to do this problem.

The actual amount of heat removed from the dough ball (Q) is obtained:

Q=-sphere_Q(time, T_ini, T_infinity, alpha, k, h_bar, D/2) "heat transfer required"

The minimum amount of energy that must be removed to freeze the dough is:

Qmin = V ρ c (Tini − T freeze ) (2)

where V is the volume of the ball:

3
4 ⎛D⎞
V = π⎜ ⎟ (3)
3 ⎝2⎠
and the product of density and specific heat capacity is obtained from the definition of thermal
diffusivity:

k
ρc= (4)
α

V=4*pi*(D/2)^3/3 "volume"
rhoc=k/alpha "density - specific heat capacity product"
Q_min=V*rhoc*(T_ini-T_freeze) "minimum heat transfer required"

The efficiency is:

Qmin
η= (5)
Q

eta=Q_min/Q "efficiency of freezing process"

which leads to η = 0.747.

e.) You would like to improve the processing time and are looking at blowers that will increase
the heat transfer coefficient by increasing the velocity of the air. Plot the processing time
(i.e., the answer to (c)) as a function of the heat transfer coefficient. At high values of h
your plot should asymptotically approach a constant value; explain this behavior.

Figure 2 illustrates the processing time as a function of the heat transfer coefficient. At high
values of heat transfer coefficient the surface temperature immediately approaches the fluid
temperature and so the processing time is entirely related to the time required for the conduction
wave to penetrate to the center of the sphere. The result is therefore insensitive to heat transfer
coefficient above h = 100 W/m2-K.
140 0.9

120 0.85

efficiency
Processing time (min)

0.8
100
Efficiency

0.75
80
0.7
60
0.65

40 0.6
processing time
20 0.55
5 10 100 1000 4500
2
Heat transfer coefficient (W/m -K)
Figure 2: Processing time and efficiency as a function of heat transfer coefficient.
f.) Overlay on your plot from (e) the efficiency as a function of h . Explain the shape of your
plot.

The efficiency as a function of the heat transfer coefficient is also shown in Figure 2. The
reduction in efficiency with increasing heat transfer coefficient is related to the fact that the
edges of the dough ball become increasingly subcooled (i.e., cooled below Tfreeze) as h is
increased. This requires more energy and the efficiency drops.
Problem 3.6-1 (3-20 in text)
An oscillating heat flux is applied to one side of a wall that is exposed to fluid on the other side,
as shown in Figure P3.6-1.

h = 500 W/m -K
2
ρ = 6000 kg/m3
T∞ = 20°C k = 2 W/m-K
L = 0.4 cm c = 700 J/kg-K

q′′ = Δq ′′ ⎡⎣1 − cos (ω t )⎤⎦


Figure P3.6-1: Wall exposed to an oscillating heat flux.

The wall thickness is L = 0.4 cm and the wall material has density ρ = 6000 kg/m3, conductivity
k = 2 W/m-K, and c = 700 J/kg-K. The fluid temperature is T∞ = 20°C and the heat transfer
coefficient is h = 500 W/m2-K. Initially, the wall is in equilibrium with the fluid. The heat flux
varies according to: q ′′ = Δq ′′ ⎡⎣1 − cos (ω t ) ⎤⎦ where Δq ′′ = 1000 W/m2 and ω = 1 rad/s.
a.) Sketch the temperature as a function of time that you expect at x = 0 and x = L for the first 10
oscillations (0 < t < 62.8 s). Try to get the qualitative characteristics of your sketch correct
(e.g., the magnitude of the average temperature rise and temperature oscillations as well as
the time scales involved).

The inputs are entered in EES:

$UnitSystem SI MASS RAD PA K J


$TABSTOPS 0.2 0.4 0.6 0.8 3.5 in

"Inputs"
Dqf=1e3 [W/m^2] "heat flux"
omega=1 [rad/s] "angular frequency"
k=2 [W/m-K] "conductivity"
rho=6000 [kg/m^3] "density"
c=700 [J/kg-K] "specific heat capacity"
L=0.004 [m] "half-thickness of wall"
h_bar=500 [W/m^2-K] "heat transfer coefficient"
T_infinity=converttemp(C,K,20) "fluid temperature"
alpha=k/(rho*c) "thermal diffusivity"

There are a few characteristic resistances the govern this problem. The resistance to convection
from the edge at x = L is:

1
Rconv = (1)
hA

where A = 1 m2 for a per unit area basis analysis. The resistance to conduction across the wall is:
L
Rcond = (2)
kA

R_conv=1/(h_bar*A) "resistance to convection"


R_cond=L/(k*A) "resistance to conduction"

The time available for the heat flux associated with one oscillation to diffuse into the material is,
approximately:

π
tdiff = (3)
ω

and therefore the penetration depth is:

δ diff = 2 α tdiff (4)

t_diff=pi/omega "period of oscillation"


delta_t=2*sqrt(alpha*t_diff) "depth of penetration"

which leads to δdiff = 0.0024 m. This is less than L and therefore I expect that the temperature at
x = 0 will oscillate in response to the heat flux but the temperature at x = L will not. The
resistance to conduction into the penetration depth is:

δt
Rcond ,δt = (5)
kA

R_cond_deltat=delta_t/(k*A) "resistance of penetration depth"

The temperature rise associated with a single oscillation will be on the order of:

ΔTosc ≈ Δq ′′′ A Rcond ,δt (6)

DT_osc=Dqf*A*R_cond_deltat "amplitude of temperature oscillation"

which leads to ΔTosc = 1.2 K. The time required for the wall to thermally equilibrate with the
surroundings is, approximately:

τ lumped = C ( Rconv + Rcond ) (7)

where C is the heat capacity of the wall:

C = ALρ c (8)

Cap=A*L*rho*c "heat capacity"


tau_lump=Cap*(R_conv+R_cond) "lumped time constant"
which leads to τlumped = 33.6 s. Therefore, I would expect that the system will very nearly have
reached equilibrium after 10 cycles (62.8 s). At the point where the system has reached
equilibrium. The average temperature rise at x = 0 will be approximately:

ΔT x =0 ≈ Δq ′′′ A ( Rcond + Rconv ) (9)

and the average temperature rise at x = L will be approximately:

ΔT x = L ≈ Δq ′′′ A Rconv (10)

DT_0=Dqf*A*(R_cond+R_conv) "average temperature rise at x=0"


DT_L=Dqf*A*R_conv "average temperature rise at x=L"

which leads to ΔT x =0 = 4 K and ΔT x = L = 2 K. Figure 2 illustrates a sketch of temperature as a


function of time at x = 0 and x = L that has these characteristics.

~ 1.2 K
Temperature

x=0
~4K
x=L

~2K

Time
Figure 2: Sketch of temperature at x = 0 and x = L as a function of time for ~10 oscillations.

b.) Use Duhamel's Theorem to develop an analytical model of the process. Plot the temperature
as a function of time for the first 10 oscillations at x = 0, x = L/2, and x = L.

The governing partial differential equation for the problem, written in terms of a temperature
difference, θ = T - T∞ is:

∂ 2θ ∂θ
α = (11)
∂x 2 ∂t

The boundary conditions are:

∂θ
−k = h θ x=L (12)
∂x x=L
∂θ
Δq ′′ ⎡⎣1 − cos (ω t ) ⎤⎦ = − k (13)
∂x x =0

The initial condition is:

θt =0 = 0 (14)

In order to apply Duhamel's theorem, it is necessary that the problem have a zero initial
condition and that the only non-homogeneous boundary condition be the time-dependent one.
The problem stated above satisfies this criterion. The next step is to obtain the fundamental
solution. The fundamental solution, θf, is the solution to the isolated sub-problem with the time-
dependent boundary condition replaced by a constant, unit value. For this problem, the time-
dependent heat flux in Eq. (13) is replaced by unity. The mathematical specification of the
fundamental problem is therefore:

∂ 2θ ∂θ
α 2 = (15)
∂x ∂t

The boundary conditions are:

∂θ
−k = h θ x=L (16)
∂x x=L

∂θ
1 = −k (17)
∂x x =0

The initial condition is:

θt =0 = 0 (18)

The fundamental solution was obtained in Problem 3.5-5 and is repeated here. To apply
separation of variables, it is necessary that both of the boundary conditions be homogeneous;
however, Eq. (13) is not. Therefore, the solution is assumed to be the sum of a homogeneous
and particular solution:

θ = θ h ( x, t ) + X ( x ) (19)

Substituting Eq. (19) into Eq. (11) leads to:

∂ 2θ h d 2 X ∂θ h
α + α = (20)
∂x 2 dx 2 ∂t
which is divided into the partial differential equation for θh and ordinary differential equation for
X:

∂ 2θ h ∂θ h
α = (21)
∂x 2 ∂t

and

d2X
=0 (22)
dx 2

Integrating Eq. (22) twice leads to:

X = C1 x + C2 (23)

Substituting Eq. (19) into the boundary condition at x = L, Eq. (12), leads to:

∂θ h dX
−k −k = h θ x= L + h X x=L (24)
∂x x=L ∂x x=L

which is divided into the homogeneous boundary condition for θh and the boundary condition for
X:

∂θ h
−k = h θ x=L (25)
∂x x= L

dX
−k = h X x=L (26)
∂x x=L

Substituting Eq. (23) into Eq. (26) leads to:

− k C1 = h ( C1 L + C2 ) (27)

Substituting Eq. (19) into the boundary condition at x = 0, Eq. (13), leads to:

∂θ h ∂X
q ′′ = −k −k (28)
∂x x =0 ∂x x =0

which is divided into the homogeneous boundary condition for θh and the boundary condition for
X:
∂θ h
=0 (29)
∂x x =0

∂X
q ′′ = − k (30)
∂x x =0

Substituting Eq. (23) into Eq. (30) leads to:

q ′′ = − k C1 (31)

Equations (27) and (31) are solved for C1 and C2:

"constants for particular solution"


-k*C_1=h_bar*(C_1*L+C_2)
-k*C_1=qf

and the particular solution is evaluated at a specific axial location:

x_bar=0.1 [-] "dimensionless position"


x=x_bar*L "position"
Xs=C_1*x+C_2 "particular solution"

Equation (19) is substituted into Eq. (14):

θ h ,t = 0 + X = 0 (32)

which leads to:

θ h ,t =0 = −C1 x − C2 (33)

The homogeneous solution is solved using separation of variables. The partial differential
equation, Eq. (21), is separated into θX and θt ordinary differential equations:

d 2θ X
+ λ2 θ X = 0 (34)
dx 2

dθ t
+ λ2 α θt = 0 (35)
dt

The general solution to Eq. (34) is:

θ X = C1 sin ( λ x ) + C2 cos ( λ x ) (36)


where C1 and C2 are unknown constants. Substituting Eq. (36) into the spatial boundary
condition at x = 0, Eq. (29), leads to C1 = 0.

θ X = C2 cos ( λ x ) (37)

Substituting Eq. (37)into the spatial boundary condition at x = L, Eq. (25), leads to:

k C2 λ sin ( λ L ) = h C2 cos ( λ L ) (38)

Equation (38) provides the eigencondition for the problem, which defines multiple eigenvalues:

sin ( λ L ) h
= (39)
cos ( λ L ) k λ

or, multiplying and dividing the right side of Eq. (39) by L:

sin ( λ L ) hL
= (40)
cos ( λ L ) k λ L

Writing Eq. (40) in terms of the Biot number leads to:

Bi
tan ( λ L ) = (41)
λL

where

hL
Bi = (42)
k

The eigenvalues are automatically identified in EES by specifying lower and upper limits and
guess values for each one.

Nterm=6 [-] "number of terms to use in the


solution"
"Setup guess values and lower and upper bounds for eigenvalues"
duplicate i=1,Nterm
lowerlimit[i]=(i-1)*pi
upperlimit[i]=lowerlimit[i]+pi/2
guess[i]=lowerlimit[i]+pi/4
end
"Identify eigenvalues"
Bi=h_bar*L/k
duplicate i=1,Nterm
tan(lambdaL[i])=Bi/lambdaL[i] "eigencondition"
lambda[i]=lambdaL[i]/L "eigenvalue"
end
At this point, each of the eigenfunctions of the problem have been obtained. The ith
eigenfunction is:

θ X i = C2,i cos ( λi x ) (43)

where λi is the ith eigenvalue, identified by the eigencondition:

Bi
tan ( λi L ) = (44)
λi L

The solution to the non-homogeneous ordinary differential equation corresponding to the ith
eigenvalue is

θ ti = C3,i exp ( −λi2 α t ) (45)

where C3,i are undetermined constants. The solution associated with the ith eigenvalue is:

θ h,i = θ X i θ ti = Ci cos ( λi x ) exp ( −λi2 α t ) (46)

where the constants C2,i and C3,i have been combined to a single undetermined constant Ci.
Because the partial differential equation is linear, the sum of the solution θi for each eigenvalue,
Eq. (46), is itself a solution:

∞ ∞
θ h = ∑ θi = ∑ Ci cos ( λi x ) exp ( −λi2 α t ) (47)
i =1 i =1

The final step of the problem selects the constants so that the series solution satisfies the initial
condition, Eq. (33):


θt =0 = ∑ Ci cos ( λi x ) = −C1 x − C2 (48)
i =1

Each side of Eq. (48) is multiplied by cos(λj x) and integrated from x = 0 to x = L:


L x x
Ci ∫ cos 2 ( λi x ) dx = −C1 ∫ x cos ( λi x ) dx − C2 ∫ cos ( λi x ) dx (49)

0


0


0

Integral 1 Integral 2 Integral 3

The integrals in Eq. (49) can be evaluated conveniently using Maple:

> restart;
> int((cos(lambda[i]*x))^2,x=0..L);
1 cos ( λ i L ) sin( λ i L ) + λ i L
2 λi
> int(x*cos(lambda[i]*x),x=0..L);
−1 + cos ( λ i L ) + L sin( λ i L ) λ i
2
λi
> int(cos(lambda[i]*x),x=0..L);
sin( λ i L )
λi

Substituting these results into EES provides each of the constants:

duplicate i=1,Nterm
Integral1[i]=1/2*(cos(lambda[i]*L)*sin(lambda[i]*L)+lambda[i]*L)/lambda[i]
Integral2[i]=(-1+cos(lambda[i]*L)+L*sin(lambda[i]*L)*lambda[i])/lambda[i]^2
Integral3[i]=1/lambda[i]*sin(lambda[i]*L)
C[i]*Integral1[i]=-C_1*Integral2[i]-C_2*Integral3[i]
end

Duhamel's theorem is restated below, for our problem:

If Tf (x,t) is the response of a linear system with a zero initial condition to a single, constant non-
homogeneous boundary condition with magnitude of unity (referred to as the fundamental
solution), then the response of the same system to a single, time-varying non-homogeneous
boundary condition with magnitude B(t) can be obtained from the fundamental solution
according to:
t
dq ′′ (τ )
T ( x, t ) = ∫ T ( x, t − τ )
f dτ + qt′′=0 T f ( x, t ) (50)
τ=0
dt

where Bt=0 is the value of B at time zero and B must be continuous in time.

We will evaluate the solution at a particular value of time, normalized against the time required
for one oscillation:

t_osc=2*pi/omega "time required for one oscillation"


time_bar=1 [-] "dimensionless time"
time=time_bar*t_osc "time"

An arbitary value of the integration variable, τ, is initially assumed and will eventually be varied
to carry out the integration in Eq. (50).

tau=0 [s] "integration variable"


The fundamental solution is evaluated at xand t - τ:

duplicate i=1,Nterm
theta_f_h[i]=C[i]*cos(lambda[i]*x)*exp(-lambda[i]^2*alpha*(time-tau))
end
theta_f_h=sum(theta_f_h[i],i=1,Nterm)
theta_f=theta_f_h+Xs_f

and multiplied by the time derivative of the heat flux:

dq ′′ (τ )
= Δq ′′ ω sin (ωτ ) (51)
dt

dqfdt=Dqf*omega*sin(omega*tau)

In order to carry out the integration in Eq. (50), the arbitrary value of the integration variable is
commented out and the Integral function in EES is used (note that the value of the heat flux at t =
0 is 0):

{tau=0 [s]} "integration variable"


theta=Integral(theta_f*dqfdt,tau,0,time,0.1)
T=theta+T_infinity
T_C=converttemp(K,C,T)

Figure 3 illustrates the temperature as a function of time for x = 0, L/2, and L:


23.5

23
x/L = 0
22.5 x/L = 0.5
Temperature (°C)

22

21.5 x/L = 1

21

20.5

20

19.5
0 10 20 30 40 50 60 70
Time (s)
Figure 3: Temperature at x/L = 0, 0.5, and 1 as a function of time for 10 oscillations.
Problem 3.6-2
A magnetocaloric material exposed to a time varying magnetic field experiences a conversion of
magnetic entropy to thermal entropy. In some situations, this effect can be modeled as a
volumetric generation of thermal energy that can be both positive (when the magnetic field is
rising) and negative (when the magnetic field is falling). Consider a plate composed of a
magnetocaloric material with thermal diffusivity α. The plate thickness is L and the plate is
exposed on one side (x = L) to fluid at Tc. The heat transfer coefficient is high so that the plate
surface temperature at x = L may be considered to be kept always at Tc. Initially, the material is
in equilibrium with the fluid. A sinusoidally varying magnetic field is applied, leading to a
volumetric generation (or absorption) of thermal energy that varies with time according to:
g ′′′ = Δg ′′′ sin (ω t ) . The surface of the plate at x = 0 is insulated.
a.) Derive the governing partial differential, boundary conditions, and initial condition for the
problem.

The partial differential equation is:

∂ 2T 1 ∂T Δg ′′′ sin (ω t )


= − (1)
∂x 2 α ∂t k

The boundary conditions are:

∂T
=0 (2)
∂x x =0

Tx = L = Tc (3)

The initial condition is:

Tt =0 = Tc (4)

b.) Nondimensionalize your problem from (a). This process should result in the definition of a
dimensionless frequency.

A dimensionless temperature is defined as the temperature difference relative to Tc normalized


by the temperature rise associated with conduction through the wall:

T − Tc
T = (5)
Δg ′′′ L2
k

The dimensionless position and time are defined as:


x
x = (6)
L

t = t ω (7)

Substituting Eqs. (5) through (7) into the problem from (a) leads to:

∂ 2T ∂T
= ω
 − sin ( t ) (8)
∂x 2 ∂t

where ω is the dimensionless frequency:

L2 ω
ω = (9)
α

The boundary conditions and initial condition are:

∂T
=0 (10)
∂x x = 0

Tx = L = 0 (11)

Tt =0 = 0 (12)

c.) Apply Duhamel's theorem to your problem from (b).

The problem from (b) satisfies the conditions required to apply Duhamel's theorem. It is
homogeneous with the exception of the time dependent generation term and has a zero initial
condition. It is first necessary to obtain the solution for the response to a unit step, U. The
problem for U is:

∂ 2U ∂U
= ω −1 (13)
∂x 2
∂t

∂U
=0 (14)
∂x x = 0

U x=L = 0 (15)

U t =0 = 0 (16)
The problem for U cannot be solved directly using separation of variables. It is necessary to split
U into two subproblems, s and a:

U ( x , t ) = s ( x , t ) + a ( x ) (17)

Substituting Eq. (17) into Eqs. (13) through (16) leads to a problem for s:

∂2s ∂s
= ω (18)
∂x 2
∂t

∂s
=0 (19)
∂x x = 0

sx =1 = 0 (20)

st =0 = −a (21)

and a:

d 2a
= −1 (22)
dx 2

da
=0 (23)
dx x = 0

ax =1 = 0 (24)

The solution to subproblem a is:

x 2 1
a=− + (25)
2 2

The subproblem s is divided into an eigenproblem:

d2X
+ λ2 X = 0 (26)
dx 2

with

dX
=0 (27)
∂x x = 0
X x =1 = 0 (28)

which is solved by:

X i = Ai cos ( λi x ) (29)

where the eigenvalues are:

λi =
( 2 i − 1) π (30)
2

The ordinary differential equation for Θi is:

d 2 Θi λ 2
+ Θi = 0 (31)
dt 2 ω

which is solved by:

⎛ λi2 ⎞
Θi = exp ⎜ − t ⎟ (32)
⎝ ω ⎠

Therefore the solution for s is:


⎛ λ2 ⎞
s = ∑ Ai cos ( λi x ) exp ⎜ − i t ⎟ (33)
i =1 ⎝ ω ⎠

The initial condition for the s subproblem is:


x 2 1
st =0 = ∑ Ai cos ( λi x ) = − (34)
i =1 2 2

Applying the orthogonality of the eigenfunctions leads to:

1 1
⎛ x 2 1 ⎞
Ai ∫ cos 2 ( λi x ) dx = ∫ ⎜ − ⎟ cos ( λi x ) dx (35)
0⎝
0
2 2⎠

The integrals in Eq. (35) are evaluated in Maple:

> restart;
> assume(i,integer);
> lambda:=(2*i-1)*Pi/2;
( 2 i~ − 1 ) π
λ :=
2
> int((x^2/2-1/2)*cos(lambda*x),x=0..1);
8 ( -1 )i~
π3 ( 8 i~3 − 12 i~2 + 6 i~ − 1 )
> int((cos(lambda*x))^2,x=0..1);
1
2

and implemented in EES:

$UnitSystem SI MASS RAD PA K J


$TABSTOPS 0.2 0.4 0.6 0.8 3.5 in

"Inputs"
omega_bar=1 [-] "dimensionless frequency"
t_bar=2 "dimensionless time"
x_bar=0.5 [-] "dimensionless position"

N_term=51 [-]
duplicate i=1,N_term
lambda[i]=(2*i-1)*pi/2
A[i]/2=8*(-1)^i/Pi^3/(8*i^3-12*i^2+6*i-1)
end
duplicate i=1,N_term
s[i]=A[i]*cos(lambda[i]*x_bar)*exp(-lambda[i]^2*t_bar/omega_bar)
end
s=sum(s[1..N_term])
a=-x_bar^2/2+1/2
U=s+a

The solution for U is shown as a function of dimensionless position for various values of
dimensionless time in Figure 1. The solution obeys our intuition: the step change in the
generation causes a temperature elevation at the inside of the wall.
0.5
2
0.45

0.4 1

Solution for U (K-m /W)


0.35

3 0.3
0.5
0.25
0.3
0.2
0.2
0.15

0.1 0.1
0.05 t=0
0
0 0.1 0.2 0.3 0.4 0.5 0.6 0.7 0.8 0.9 1
Normalized axial position
Figure 1: Solution for U as a function of x/L for various values of dimensionless time.

Duhamel's theorem states that:

dg ′′′
t
T ( x , t ) = ∫U ( x , t −τ ) dτ (36)
0
dt

Note that there is no jump in g ′′′ at dimensionless time 0. Substituting the solution for U and the
equation for the generation into Eq. (36) leads to:

t ∞
⎛ λ2 ⎞
T ( x , t ) = ∫ ∑ Ai cos ( λi x ) exp ⎜ − i ( t − τ ) ⎟ cos (τ ) dτ (37)
0 i =1 ⎝ ω ⎠

or


⎛ λi2 ⎞ t ⎛ λ2 ⎞
T ( x , t ) = ∑ Ai cos ( λi x ) exp ⎜ − t ⎟ ∫ exp ⎜ i τ ⎟ cos (τ ) dτ (38)
i =1 ⎝ ω ⎠ 0 ⎝ ω ⎠

The integral in Eq. (38) is evaluated in Maple:

> restart;
> int(exp(lambda[i]^2*tau/w)*cos(tau),tau=0..theta);
⎛ ⎞ ⎛ ⎞
⎛ ⎞
2 2
⎜λ θ⎟ ⎜λ θ⎟
⎜ ⎜ i



⎜ i


⎟ ⎟
⎜ 2 ⎜ ⎟ ⎜ ⎟ ⎟
⎜ 2 ⎝ w ⎠
w ⎜ −λ i + λ i e cos ( θ ) + w e ⎝ w ⎠
sin( θ ) ⎟⎟
⎝ ⎠
4
λi + w 2
Therefore:


Ai cos ( λi x ) ω ⎡ 2 ⎛ λi2 ⎞ ⎤
T ( x, t ) = ∑ −
⎢ iλ exp ⎜ −  t ⎟ + λi cos ( t ) + ω sin ( t ) ⎥
2
(39)
i =1 ( λi + ω ) ⎣
4 2
⎝ ω ⎠ ⎦

d.) Plot the dimensionless temperature as a function of dimensionless time at various values of
dimensionless axial position for a dimensionless frequency of 1.
0.2
0
0.25
0.15
Dimensionless temperature

0.5
0.1
0.75
0.05

0 x/L = 1

-0.05

-0.1

-0.15

-0.2
0 2 4 6 8 10 12 14 16
Dimensionless time
Figure 2: Dimensionless temperature as a function of dimensionless time for various values of x/L with a
dimensionless frequency of 1.

e.) Plot the dimensionless temperature as a function of dimensionless time at the insulated edge
of the wall ( x = 0) for various values of dimensionless frequency.
0.6
10
Dimensionless center temperature

5
0.4 2
1
0.2 0.5
0.2

ω = 0.1
-0.2

-0.4

-0.6
0 2 4 6 8 10 12 14 16
Dimensionless time
Figure 3: Dimensionless center temperature as a function of dimensionless time for various values
dimensionless frequency.
Problem 3.7-1 (3-21 in text): Regenerator Matrix
Regenerative heat exchangers are discussed in Section 8.10. The regenerator operates in a cyclic
fashion. Hot fluid passes across the regenerator material for half of a cycle, transferring energy
to the material. Cold fluid passes across the regenerator material for the other half of the cycle,
receiving energy from the material. After a sufficient number of cycles, the temperature
distribution reaches a cyclic steady-state condition. Consider a regenerator matrix that consists
of plates, as shown in Figure P3.7-1.
ρ, k, c

h , T∞ = T∞ + ΔT∞ sin (ω t ) h , T∞ = T∞ + Δ T∞ sin (ω t )


L
x
Figure P3.7-1: Plate regenerator matrix.

The half-thickness of the plate is L and the material properties are ρ, c, and k. The heat transfer
coefficient between the surface of the plate and the fluid is h and the fluid temperature is
assumed to vary sinusoidally with mean temperature T∞ , amplitude ΔT∞ , and frequency ω. In
general, the temperature within the regenerator matrix is a function of both x and t.
a.) Using the method of complex combination, develop a solution for the sustained response of
the temperature within the regenerator.

The governing differential equation for this problem is:

∂ 2T ∂T
α = (1)
∂x 2 ∂t

The spatial boundary conditions are obtained with interface balances at the center (x = 0) and
surface (x = L) of the piston plate:

∂T
−k = h ⎡⎣Tx = L − T∞ − ΔT∞ sin (ω t ) ⎤⎦ (2)
∂x x=L

∂T
−k =0 (3)
∂x x =0

No initial condition is required since only the sustained, periodic solution will be obtained. The
method of complex combination can only be applied if the problem is homogeneous except for
the periodic disturbance. The boundary condition at x = L can be made homogeneous with the
transformation:

θ = T − T∞ (4)
which leads to:

∂ 2θ ∂θ
α = (5)
∂x 2 ∂t

∂θ
−k = h ⎡⎣θ x = L − ΔT∞ sin (ω t ) ⎤⎦ (6)
∂x x= L

∂θ
−k =0 (7)
∂x x =0

The problem for θ90° is constructed by replacing the periodic component of the θ problem with
one that is 90° out of phase (i.e., by replacing the sin(ω t) term in the boundary condition at x = 0
with a cos(ω t) term):

∂ 2θ90° ∂ θ90°
α = (8)
∂x 2 ∂t

∂θ90°
−k = h ⎡⎣θ90°, x = L − ΔT∞ cos (ω t ) ⎤⎦ (9)
∂x x=L

∂θ90°
−k =0 (10)
∂x x =0

The complex conjugate problem for θcc:

θcc = θ + i θ90° (11)

is constructed by multiplying the governing differential equation and boundary conditions for
θ90° by i (the square root of negative one) and adding it to the governing differential equation and
boundary conditions for θ.
∂ 2θ cc ∂ θ cc
α = (12)
∂x 2 ∂t

∂θ ∂θ90°
−k −k i = h ⎡⎣θ x = L − ΔT∞ sin (ω t ) ⎤⎦ + h ⎡⎣θ90°, x= L − ΔT∞ cos (ω t ) ⎤⎦ i (13)
∂x x=L ∂x x= L

or
∂θcc
−k = h θcc, x = L − h ΔT∞ ⎡⎣sin (ω t ) + i cos (ω t ) ⎤⎦ (14)
∂x x= L

∂θcc
−k =0 (15)
∂x x =0

The periodic variation in Eq. (14) must be converted to exponential form; this is most easily
accomplished using Maple:

> restart;
> convert(sin(omega*t)+I*cos(omega*t),exp);
− I ⎛⎜ e − ( ω t I ) ⎟⎞ + ⎛⎜ e
1 (ω t I) 1 1 (ω t I) 1 1 ⎞
+ ⎟I
2 ⎜ ⎟ ⎜2 2 e(ω t I) ⎟
⎝ e ⎠ ⎝ ⎠
> simplify(%);
( −I ω t )
e I

which leads to:

∂θcc
−k = h θcc, x = L − h ΔT∞ i exp ( -i ω t ) (16)
∂x x= L

The complex conjugate solution is assumed in the appropriate exponential form:

θcc = B ( x ) exp ( −i ω t ) (17)

Substituting Eq. (17) into the partial differential equation:

∂2 ∂
α ⎡⎣ B ( x ) exp ( −i ω t ) ⎤⎦ = ⎡⎣ B ( x ) exp ( −i ω t ) ⎤⎦ (18)
∂x 2 ∂t

or

d 2B
α exp ( −i ω t ) = − B i ω exp ( −i ω t ) (19)
dx 2

Note that the time variation must cancel from the complex conjugate problem when expressed in
this manner (a consequence of the problem being homogeneous), leaving an ordinary differential
equation for B:

d 2B
α = −B i ω (20)
dx 2
Substituting Eq. (17) into the boundary condition at x = L:

dB
−k = h Bx = L − h ΔT∞ i (21)
dx x= L

Substituting Eq. (17) into the boundary condition at x = 0, leads to:

dB
=0 (22)
dx x=0

The solution for B is:

⎛ iω ⎞ ⎛ iω ⎞
B = C1 sin ⎜⎜ x ⎟⎟ + C2 cos ⎜⎜ x ⎟⎟ (23)
⎝ α ⎠ ⎝ α ⎠

Substituting Eq. (23) into Eq. (22) leads to:

dB iω ⎛ iω ⎞ iω ⎛ iω ⎞
= C1 cos ⎜⎜ 0 ⎟⎟ − C2 sin ⎜⎜ 0 ⎟⎟ = 0 (24)
dx x =0 α ⎝ α ⎠ α ⎝ α ⎠

which can only be true if C1 = 0:

⎛ iω ⎞
B = C2 cos ⎜⎜ x ⎟⎟ (25)
⎝ α ⎠

Substituting Eq. (25) into Eq. (21) leads to:

iω ⎛ iω ⎞ ⎛ iω ⎞
k C2 sin ⎜⎜ L ⎟⎟ = h C2 cos ⎜⎜ x ⎟⎟ − h ΔT∞ i (26)
α ⎝ α ⎠ ⎝ α ⎠

which can be solved for C2:

− h ΔT∞ i
C2 = (27)
⎡ iω ⎛ iω ⎞ ⎛ i ω ⎞⎤
⎢k sin ⎜ L ⎟ − h cos ⎜ x ⎟⎥
⎣⎢ α ⎝ α ⎠ ⎝ α ⎠ ⎥⎦

Substituting Eqs. (27) and (25) into Eq. (17) leads to:
⎛ iω ⎞
− ΔT∞ i cos ⎜ x ⎟ exp ( −i ω t )
⎝ α ⎠
θcc = (28)
⎡ k iω ⎛ iω ⎞ ⎛ i ω ⎞⎤
⎢ sin ⎜ L ⎟ − cos ⎜ x ⎟⎥
⎣⎢ h α ⎝ α ⎠ ⎝ α ⎠ ⎦⎥

The solution is the real part of Eq. (28).

b.) Identify physically significant dimensionless parameters that can be used to correlate your
solution. You should non-dimensionalize your solution and express it in terms of a
dimensionless position and time as well as the Biot number and an additional dimensionless
parameter that characterizes the frequency of oscillation.

The dimensionless solution is the temperature difference relative to the average fluid temperature
normalized by the fluid temperature oscillation:

T − T∞
θ = (29)
ΔT∞

The dimensionless position is defined relative to the spatial extent of the plate:

x
x = (30)
L

The dimensionless time is defined according to:

t = ω t (31)

The Biot number is the ratio of the internal conduction resistance to external convection
resistance:

hL
Bi = (32)
k

The final dimensionless parameter will be referred to as a dimensionless frequency and is the
ratio of the diffusion time constant to the period of oscillation:

ω L2
ω = (33)
α

Substituting Eqs. (29) through (33) into Eq. (28) leads to:
θcc =
− i cos ( )
i ω x exp ( −i t )
(34)
⎡ i ω ⎤
⎢ sin ( )
i ω − cos ( )
i ω x ⎥
⎢⎣ Bi ⎥⎦

The dimensionless solution, θ , is the real part of Eq. (34). The solution is implemented in EES
according to:

$UnitSystem SI MASS RAD PA K J


$TABSTOPS 0.2 0.4 0.6 0.8 3.5 in
$COMPLEX On i

t_hat=0.1 [-] "dimensionless time"


x_hat=1 [-] "dimensionless position"
omega_hat=2 [-] "dimensionless frequency"
Bi=10 [-] "Biot number"

theta_hat_cc=-i*exp(-i*t_hat)*cos(sqrt(i*omega_hat)*x_hat)/&
(sqrt(i*omega_hat)*sin(sqrt(i*omega_hat))/Bi-cos(sqrt(i*omega_hat)))
"complex conjugate solution"
theta_hat=Real(theta_hat_cc)

c.) Prepare three plots of the dimensionless temperature as a function of dimensionless time for
various values of the dimensionless position. Plot 1 should be for a large Biot number and
large dimensionless frequency, plot 2 should be for a large Biot number and a small
dimensionless frequency, and plot 3 should be for a small Biot number and a small
dimensionless frequency. Explain why the behavior exhibited in each of these plots obeys
your physical intuition.

Figure 2 illustrates θ as a function of t for various values of x when Bi is large and ω is large.
In this limit, the internal conduction resistance is large relative to the convection resistance and
therefore most of the temperature gradient is within the material. Also, the time required for a
single oscillation is much smaller than the time constant for diffusion - therefore the diffusion
wave cannot penetrate to the center of the material. These characteristics are both exhibited by
Figure 2.
1

0.8

Dimensionless temperature
0.6
x/L = 0.25
0.4
x/L = 0
0.2

-0.2

-0.4 x/L = 0.5


-0.6 x/L = 0.75
-0.8 x/L = 1
-1
0 1.57 3.14 4.71 6.28
Dimensionless time
Figure 2: Dimensionless temperature as a function of dimensionless time for various values of dimensionless
position when Bi is large and dimensionless frequencye is large.

Figure 3 illustrates θ as a function of t for various values of x when Bi is large and ω is small.
In this limit, the internal conduction resistance is large relative to the convection resistance and
therefore most of the temperature gradient is within the material. However, the time required for
a single oscillation is much larger than the time constant for diffusion - therefore the diffusion
wave can penetrate to the center of the material and the entire wall participates in the process so
that there are no temperature gradients. The entire wall follows the fluid temperature. These
characteristics are exhibited by Figure 3.

0.8
Dimensionless temperature

0.6
x/L = 0, 0.25, 0.5, 0.75, 1
0.4

0.2

-0.2

-0.4

-0.6

-0.8

-1
0 1.57 3.14 4.71 6.28
Dimensionless time
Figure 3: Dimensionless temperature as a function of dimensionless time for various values of dimensionless
position when Bi is large and dimensionless frequencye is large.
Figure 4 illustrates θ as a function of t for various values of x when Bi is small and ω is small.
In this limit, the internal conduction resistance is small relative to the convection resistance and
therefore most of the temperature gradient is between the surface of the material and the fluid.
Also, the time required for a single oscillation is much larger than the time constant for diffusion
- therefore the diffusion wave can penetrate to the center of the material. These characteristics
are both exhibited by Figure 4.

0.8
Dimensionless temperature

0.6

0.4

0.2

-0.2 x/L = 0, 0.25, 0.5, 0.75, 1


-0.4

-0.6

-0.8

-1
0 1.57 3.14 4.71 6.28
Dimensionless time
Figure 4: Dimensionless temperature as a function of dimensionless time for various values of dimensionless
position when Bi is small and dimensionless frequencye is small.
Problem 3.8-1: Fuse
A power electronics module normally receives a small current; however, under certain
conditions it is possible that it might experience currents as high as 100 amps. The module can
only survive under such a high current for 1.0 s. Therefore, you have been asked to design a fuse
that will protect the module by limiting the current that it can experience.

The design for the fuse is shown in Figure P3.8-1. The fuse is a wire (with no insulation) that
has length L = 8 cm and diameter d = 1.5 mm. The surface of the fuse wire is exposed to air at Tf
= 20°C and the heat transfer coefficient between the surface of the fuse and the air is h = 5
W/m2-K. The fuse is made of an aluminum alloy with k = 150 W/m-K, ρ = 2700 kg/m3, and c =
900 J/kg-K. The electrical resistivity of the aluminum alloy is er = 1x10-7 ohm-m and the alloy
melts at approximately Tmelt = 500°C; you can assume that the properties of the alloy do not
depend on temperature.

The fuse is initially at a uniform temperature of Tf = 20°C when, at time t = 0, it is exposed to a


current of Ic = 100 amp; the current results in a uniform volumetric generation within the fuse
material. If the fuse operates properly, then it will melt (i.e., at some location within the fuse, the
temperature will exceed 500°C) before 1.0 s has passed, protecting the power electronics
module. Both ends of the fuse (x=0 and x=L) are maintained at Tbase = 20°C during the process
(i.e., only the middle of the fuse gets hot).

L = 8 cm
Ic = 100 amp
Tbase = 20°C Tbase = 20°C
x

d = 1.5 mm
T f = 20°C
k = 150 W/m-K
h = 5 W/m -K
2
ρ = 2700 kg/m3
c = 900 J/kg-K
er = 1x10-7 ohm-m
Figure P3.8-1: Fuse

a.) Develop a numerical model of the fuse and implement it in EES (use any method you’d like).
The fuse can be considered to be one-dimensional (that is, the temperature only depends on x
and not on r). The resistance of a wire (Rst) is given by: Rst = er L / Ac where er is the
electrical resistivity, L is the length of the fuse, and Ac is the cross-sectional area. The
amount of energy dissipated by resistive heating ( w ohmic ) is given by: w ohmic = I c2 Rst where Ic
is the current. The volumetric rate of thermal energy generation ( g ′′′ ) is the resistive heating
per unit volume of wire: g ′′′ = w ohmic / ( Ac L ) .

The inputs are entered in EES:

$UnitSystem SI MASS RAD PA K J


$TABSTOPS 0.2 0.4 0.6 0.8 3.5 in
"Inputs"
d=0.0015 [m] "fuse diameter"
L=0.08 [m] "fuse length"
T_base=converttemp(C,K,20) "base temperature"
k=150 [W/m-K] "conductivity"
rho=2700 [kg/m^3] "density"
c=900 [J/kg-K] "specific heat capacity"
er=1e-7 [ohm-m] "fuse resistivity"
T_f=converttemp(C,K,20) "fluid temperature"
h=5 [W/m^2-K] "heat transfer coefficient"
I_c=100 [amp] "fuse current"

The equations provided in the problem statement are used to compute the rate of volumetric
thermal energy generation:

"compute the rate of thermal energy generation"


A_c=pi*d^2/4 "area of fuse"
Rst=er*L/A_c "resistance of fuse"
w_dot_ohmic=I_c^2*Rst "ohmic dissipation"
g_v=w_dot_ohmic/(A_c*L)
"volumetric rate of thermal energy generation"

The nodes are positioned through the fuse according to:

L
Δx = (1)
( N − 1)

xi = Δx ( i − 1) for i = 1..N (2)

The positions in time where the solution will be evaluated are also positioned:

τ sim
Δt = (3)
( M − 1)

ti = Δt ( j − 1) for j = 1..M (4)

where τsim is the total simulation time and M and N are the number of time and space locations at
which we need to evaluate the solution:

tau_sim=1.0 [s] "simulation time"


M=21 [-] "number of time positions"
Dt=tau_sim/(M-1) "time step duration"
duplicate j=1,M
time[j]=Dt*(j-1) "position in time"
end

N=11 [-] "number of space positions"


Dx=L/(N-1) "distance between adjacent nodes"
duplicate i=1,N
x[i]=Dx*(i-1) "position in space"
end

An energy balance on an internal control volume is shown in Figure 2.

Figure 2: Internal control volume

The energy balance shown in Figure 3 is:

dU
q LHS + q RHS + qconv + g = (5)
dt

The rate equations are substituted into Eq. (5):

(Ti −1 − Ti ) + c (Ti +1 − Ti ) + h p Δx (T f − Ti ) + Ac Δx g ′′′ = ρ c Ac Δx i


k Ac kA dT
(6)
Δx Δx dt

which leads to

g ′′′
dTi k
= 2
dt Δx ρ c
(Ti −1 + Ti +1 − 2 Ti ) +
hp
ρ c Ac
( T f − Ti ) +
ρc
for i = 2.. ( N − 1) (7)

The edge temperatures are specified and so no energy balance is required. The initial
temperature of each node is Tf:

Ti ,1 = T f for i = 1..N (8)

Note the 2nd subscript which is used to indicate the time step:

"initial condition"
duplicate i=1,N
T[i,1]=T_f
end

I decided to do a fully implicit model of the fuse; therefore, I evaluate the time rate of change
within each step using the temperature at the end of the time step. I first go from time 1 to time
2:
⎡ k g ′′′ ⎤
Ti ,2 = Ti ,1 + ⎢ 2
Δ ρ
( Ti −1,2 + Ti +1,2 − 2 Ti ,2 ) +
ρ
hp
( T f − Ti ,2 ) +
ρ
⎥ Δt
⎣ x c c Ac c ⎦ (9)
for i = 2.. ( N − 1)

T1,2 = Tbase (10)

T1,N = Tbase (11)

"take a single step"


T[1,2]=T_base
duplicate i=2,(N-1)
T[i,2]=T[i,1]+(k*(T[i-1,2]+T[i+1,2]-2*T[i,2])/(Dx^2*rho*c)+h*p*(T_f-T[i,2])/(rho*c*A_c)+g_v/(rho*c))*Dt
end
T[N,2]=T_base

Once I have the single step working then I copy the code and put it within a duplicate loop to
take all of the steps; change the 2nd subscript from 2 to j+1 and from 1 to j.

"take all of the steps"


duplicate j=1,(M-1)
T[1,j+1]=T_base
duplicate i=2,(N-1)
T[i,j+1]=T[i,j]+(k*(T[i-1,j+1]+T[i+1,j+1]-2*T[i,j+1])/(Dx^2*rho*c)+h*p*(T_f-
T[i,j+1])/(rho*c*A_c)+g_v/(rho*c))*Dt
end
T[N,j+1]=T_base
end

b.) Generate a plot that shows the temperature as a function of axial position within the fuse at
various times, including t = 0 s, 0.2 s, 0.4 s, 0.6 s, 0.8 s, and 1.0 s.

Figure 3 illustrates the temperature as a function of position at the requested times.


Figure 3: Temperature in the fuse as a function of position for various values of time.

c.) Does the fuse meet the design specifications? In other words, based on the graph generated
in (b), does the fuse temperature at any location exceed the melting temperature before 1.0 s?

Examination of Figure 3 indicates that the maximum temperature in the fuse after 1.0 s is only
425 K which is substantially less than the melting point of the alloy. The fuse will not work as
designed.

d.) Generate a plot showing the maximum temperature achieved in the fuse after 1.0 s as a
function of the fuse diameter; based on this plot, suggest a diameter for the fuse.

The diameter is commented out in the equation window and a parametric table is generated
which contains the variables d and T_max_C where T_max_C is the temperature at the center of
the fuse at the end of the final time step (the maximum fuse temperature at 1.0 s).

T_max_C=converttemp(K,C,T[6,M]) "maximum temperature (in C)"

Figure 4 illustrates the maximum temperature in the fuse after 1.0 s as a function of diameter and
suggests that a diameter of about d = 0.0011 m should be selected.
Figure 4: Maximum temperature in the fuse after 1.0 s as a function of diameter.
Problem 3.3-5: Equilibration of Two Fins
Two fins protrude from each side of a very thin wall as shown in Figure P3.3-5(a). One fin is
immersed in hot fluid at TH = 100°C while the other is immersed in cold fluid at TC = 20°C. The
diameter of both fins is D = 0.005 m and their length is L = 0.05 m. The tips of the fins are
adiabatic. The fin material has conductivity k = 200 W/m-K, density ρ = 1000 kg/m3, and
specific heat capacity c = 430 J/kg-K. Initially (at time t = 0), the fins are at steady state and the
temperature distribution along the fins is shown in Figure P3.8-2(a).

TH = 100°C TC = 20°C

L = 0.05 m L = 0.05 m

x
D = 0.005 m k = 200 W/m-K
ρ = 1000 kg/m3
Temperature c = 430 J/kg-K

100°C

t=0

20°C

-L 0 L
Axial position (m)
Figure P3.8-2(a): Two fins protruding from a wall and the initial temperature distribution.

At t = 0, the fluid is removed from both sides of the wall and replaced with gas at Tg = 20°C.
The heat transfer coefficient between the gas and the fin is h = 20 W/m2-K. This problem is
concerned with the equilibration process that occurs after t = 0 (the thermal equilibration of the
fins with the surrounding gas at Tg). You may assume that the temperature distribution during
this process is only a function of axial position (x) and time (t); that is, the Biot number, h D/k, is
much less than 1.0.
a.) Sketch the temperature distribution that you expect to see in the fins (i.e., the temperature as
a function of position from x = -0.05 m to x = +0.05 m) for t = 0.2 s, t = 2.0 s, t = 20 s, t =
200 s, and t → ∞.

In order to understand the equilibration constant it is necessary to compute the time constants for
internal and external equilibration. The thermal diffusivity of the fin material is:

k 200 W m3 kg-K J -4 m
2
α= = = 4.65x10 (1)
ρc m-K 1000 kg 430 J W-s s

The time constant for internal equilibration of the fins due to conduction is:
L2 ( 0.05 m )
2
s
τ int = = = 1.3 s (2)
4α 4 4.65x10-4 m 2

Figure 2: Initial temperature distribution in the fins.

The resistance between the fin and the gas is:

1 m2 K
Rext = = = 32 K/W (3)
2 π D L hg 2 π 0.005 m 0.05 m 20 W

The total capacitance of the fin material is:

π 0.05 m ( 0.005 m ) 1000 kg 430 J


2
D2
C = 2 Lπ ρ c= = 0.84 J/K (4)
4 2 m3 kg-K
The time constant for external equilibration is therefore:

32 K 0.84 J W-s
τ ext = Rext C = = 27 s (5)
W K J

So the time constant for external equilibration is much larger than the time constant for internal
equilibration. The fins will therefore come to a constant temperature (the base temperature,
approximately 60°C) relatively quickly (within a few seconds) and then come to equilibrium
with the external gas more slowly (within 10's of seconds). This is reflected in the sketches in
Figure 2.

b.) You’ve decided to model the equilibration of the fins using an analytical model. Assume that
the equilibration process can be modeled as a 1-D, transient process. Derive the governing
partial differential equation that describes the process; your equation should include only
symbols defined in the problem statement (not their numerical values) as well as temperature
(T), position (x) and time (t).

The partial differential equation is derived by applying an energy balance to a differential control
volume, as shown in Figure 3.

Figure 3: Differential control volume.

The energy balance suggested by Figure 3 is:

∂U
q x = q x + dx + qconv + (6)
∂t

or, after expanding the x + dx term:

∂q ∂U
q x = q x + dx + qconv + (7)
∂x ∂t

Substituting in rate equations leads to:

∂ ⎛ π D2 ∂ T ⎞ ∂ ⎛ π D2 ⎞
0 = ⎜ −k ⎟ dx + π D dx hg (T − Tg ) + ⎜ dx ρ cT ⎟ (8)
∂x ⎝ 4 ∂x ⎠ ∂t ⎝ 4 ⎠
which can be simplified to:

∂ 2 T ρ c ∂T 4 hg 4 hg
− − T = − Tg (9)
∂x 2 k ∂t k D kD

c.) You’ve decided to model the equilibration process using the 5 node numerical model shown
in Figure P3.8-2(b). Derive the state equation (i.e., the equation that provides the time rate of
change of the temperature) for node 1. Your equation should include only symbols defined
in the problem statement (not their numerical values) as well as the temperatures of the nodes
(T1, T2, ...) and time (t).

h , Tg h , Tg

L L

node 1
node 2 node 4
node 3 node 5
Figure P3.8-2(b): 5 node numerical model of the equilibration process.

A control volume around node 1 is shown in Figure 5.

Figure 5: Control volume on node 1.

The energy balance suggested by Figure 5 is:

dU
q RHS + qconv = (10)
dt

Substituting rate equations leads to:

π D2 2 π D2 L
hg (Tg − T1 ) =
L dT
k (T2 − T1 ) + π D ρc 1 (11)
4 L 4 4 4 dt
dT1
which can be solved for :
dt

8k 4 hg
dT1
=
dt ρ c L 2 ( 2
T − T1 ) +
Dρ c
(Tg − T1 ) (12)

d.) Use your equation from part (c) to take a single Euler step for node 1. That is, what equation
would you use to predict the temperature of node 1 at time j+1 (T1,j+1) given all of the nodal
temperatures at time j. Assume that the time step duration for the step is Δt. Your equation
should include only symbols defined in the problem statement (not their numerical values) as
well as the temperatures of the nodes at time j (T1,j, T2,j, ...).

The Euler step assumes that the time rate of change is constant and equal to its value at the
beginning of the time step:

⎡ 8k 4 hg ⎤
2 ( 2, j
T1, j +1 = T1, j + ⎢ T − T1, j ) + ( Tg − T1, j ) ⎥ Δt (13)
⎣ρ cL Dρ c ⎦

e.) Determine the critical time step for the Euler step in part (d). That is, what is the largest time
step that you could take before the solution became unstable? Your equation should include
only symbols defined in the problem statement.

Equation (13) is rearranged so that the coefficient multiplying T1,j is clear:

⎡ 8 k Δt 4 hg Δt ⎤ 8 k Δt 4 hg Δt
T1, j +1 = T1, j ⎢1 − − ⎥ + T + Tg (14)
⎣ ρ c L Dρ c ⎦ ρ c L Dρ c
2 2 2, j

The solution becomes unstable when the coefficient multiplying T1,j becomes negative:

1
Δtcrit = (15)
8k 4 hg
+
ρ c L Dρ c
2
Problem 3.8-3 (3-22 in text)
Prepare a numerical solution for the equilibration process discussed in Problem 3.5-2 (3-18 in
text) using the Crank-Nicolson technique. Implement your solution in MATLAB and prepare a
plot of the temperature as a function of position at t = 10,000 s; overlay the analytical solution
derived in Problem 3.5-2 (3-18) on this plot in order to demonstrate that the analytical and
numerical solutions agree.

The inputs are entered in MATLAB.

clear all;

qf_dot=500000; %heat flux (W/m^2)


k=1; %conductivity (W/m-K)
rho=4000; %density (kg/m^3)
c=700; %specific heat capacity (J/kg-K)
L=0.5; %thickness of wall (m)
h=5000; %heat transfer coefficient (W/m^2-K)
T_f=293.2; %fluid temperature (K)
A=1; %per unit area (m^2)

The initial temperature of the right hand side of the wall is computed as discussed in part (a).

R_cond=L/(k*A); %conduction resistance through wall (K/W)


R_conv=1/(h*A); %convection resistance (K/W)
R_eq=(1/R_cond+1/R_conv)^(-1); %total resistance (K/W)
T_RHS=T_f+qf_dot*A*R_eq; %initial temperature of the RHS of wall (K)

The location of each node (xi) is:

(i − 1)
xi = L for i = 1..N (1)
( N − 1)
where N is the number of nodes used for the simulation. The distance between adjacent nodes
(Δx) is:

L
Δx = (2)
N −1

%SETUP GRID
N=11; %number of nodes (-)
for i=1:N
x(i)=(i-1)*L/(N-1); %position of each node (m)
end
DELTAx=L/(N-1); %distance between adjacent nodes (m)

The total simulation time, τsim, is divided into M time steps; most of the techniques discussed
here will divide the simulation time into time steps of equal duration, Δt:
τ sim
Δt = (3)
( M − 1)
The time associated with any time step is:

t j = ( j − 1) Δt for j = 1...M (4)

%SETUP TIME STEPS


M=1001; %number of time steps (-)
tau_sim=100000; %simulation time (s)
DELTAtime=tau_sim/(M-1); %time step duration (s)
for j=1:M
time(j)=(j-1)*DELTAtime;
end

The initial conditions for this problem are that all of the temperatures at t=0 are equal to Tin.

Ti ,1 = T f + (Tx = L ,t =0 − T f )
xi
for i = 1...N (5)
L

%INITIAL CONDITION
for i=1:N
T(i,1)=T_f+(T_RHS-T_f)*x(i)/L;
end

Control volumes defined around each of the internal nodes lead to the state equations:

dTi k
= 2 (Ti −1 + Ti +1 − 2 Ti ) for i = 2... ( N − 1) (6)
dt Δx ρ c

A control volume defined around node N (at the right hand side of the wall) leads to:

dTN
dt
=
2k
ρ c Δx 2 ( N −1
T − TN ) +
2h
Δx ρ c
(T f − TN ) (7)

The temperature of node 1 is always specified. The Crank-Nicolson technique uses the average
of the time rate of change at the beginning and the end of each time step; therefore, the equations
used to move through a time step for each node are:

k Δt
Ti , j +1 = Ti , j +
2 Δx 2 ρ c
(Ti−1, j + Ti+1, j − 2 Ti, j + Ti−1, j +1 + Ti+1, j +1 − 2 Ti, j +1 ) for i = 2...( N − 1) (8)

k Δt h Δt
2 ( N −1, j
TN , j +1 = TN , j + T − TN , j + TN −1, j +1 − TN , j +1 ) + ( 2 T f − TN , j − TN , j +1 ) (9)
ρ c Δx Δx ρ c
T1, j +1 = T f (10)

Equations (8) through (10) should be placed in matrix format; they are arranged to show the
coefficients and constants more clearly:

⎡ k Δt ⎤ ⎡ k Δt ⎤ ⎡ k Δt ⎤ k Δt
Ti , j +1 ⎢1 + 2 ⎥
Δx ρ c ⎦
+ Ti −1, j +1 ⎢ − ⎥
2 Δx ρ c ⎦
2
+ Ti +1, j +1 ⎢ − ⎥
2 Δx ρ c ⎦
2
= Ti , j +
2 Δx 2 ρ c
(Ti−1, j + Ti+1, j − 2 Ti, j )
⎣






Ai ,i Ai ,i −1 Ai ,i +1 bi

for i = 2... ( N − 1)
(11)

⎡ k Δt h Δt ⎤ ⎡ k Δt ⎤ k Δt h Δt
2 ( N −1, j
TN , j +1 ⎢1 + + ⎥ + TN −1, j +1 ⎢ − 2⎥
= TN , j + T − TN , j ) + ( 2 T f − TN , j )
ρ c Δx Δx ρ c ⎦
⎣

2
ρ c Δx ⎦
⎣
ρ c Δx Δx ρ c


AN , N AN , N −1 bN

(12)

T1, j +1 [1] = T f (13)


N N
A1,1 b1

Notice that the matrix A does not change with time for this problem and need only be setup one
time:

A=spalloc(N,N,3*N); %initialize A
b=zeros(N,1); %initialize b

%SETUP A MATRIX
A(1,1)=1;
for i=2:(N-1)
A(i,i)=1+k*DELTAtime/(rho*c*DELTAx^2);
A(i,i-1)=-k*DELTAtime/(2*rho*c*DELTAx^2);
A(i,i+1)=-k*DELTAtime/(2*rho*c*DELTAx^2);
end
A(N,N)=1+k*DELTAtime/(rho*c*DELTAx^2)+h*DELTAtime/(DELTAx*rho*c);
A(N,N-1)=-k*DELTAtime/(rho*c*DELTAx^2);

The vector b must be setup for each time step since it includes the temperatures at the beginning
of the timestep:

for j=1:(M-1)
%SETUP b MATRIX
b(1)=T_f;
for i=2:(N-1)
b(i)=T(i,j)+k*DELTAtime*(T(i-1,j)+T(i+1,j)-
2*T(i,j))/(2*rho*c*DELTAx^2);
end
b(N)=T(N,j)+k*DELTAtime*(T(N-1,j)-
T(N,j))/(rho*c*DELTAx^2)+h*DELTAtime*(2*T_f-T(N,j))/(DELTAx*rho*c);
%SIMULATE TIME STEP
T(:,j+1)=A\b;
end

Figure 1 illustrates the temperature distribution at t = 10,000 s predicted using the separation of
variables solution in Problem 3.5-2 (3-18 in text) and the numerical solution from this problem.

Figure 1: Temperature as a function of position at t = 10,000 s predicted by the numerical and


analytical solutions.
Problem 3.8-4: Geothermal Heat Pump
Figure P3.8-4(a) illustrates a residence in a warm climate that utilizes a ground-coupled heat
pump.

w qload
qbore qload
fluid at Tf
building
bore hole heat pump

surface of bore
hole is at Tf
qbore
ground
ρc = 2.8x106 J/m3
r k = 1.6 W/m-K
rbore = 5 cm
L = 50 m
Figure 3.8-4(a): Ground-coupled heat pump

The load on the building ( qload ) is shown in Figure 3.8-4(b).

5000

4000

3000
Cooling load (W)

2000

1000

-1000

-2000
Curve fit capturing seasonal variation
-3000
data
-4000
0 1.000x107 2.000x107 3.000x107
Time (s)
Figure 3.8-4(b): Cooling load as a function of time during a year with curve fit capturing the seasonal
variation.

Notice the spikes that occur each day and the lower frequency, seasonal variation in the load.
We are going to focus on the seasonal variation and ignore the daily fluctuations. The curve fit
that captures the yearly variation is shown in Figure 3.8-4(b) and given by:

⎛ ⎡ rad ⎤ ⎞
qload = 1148.78 [ W ] − 1088.8 [ W ] sin ⎜ 1.992x10-7 ⎢ ⎥ t − 425.81⎟
⎝ ⎣ s ⎦ ⎠
The load is removed by a heat pump that requires a work transfer ( w ). The heat pump rejects
heat ( qbore ) to fluid that is circulated through a bore hole. The fluid in the bore hole transfers
energy to the ground. The coefficient of performance of the heat pump, COP, is defined as the
ratio of qload to w . The coefficient of performance, COP, depends on the temperature of the
fluid, Tf, according to:

COP = 4 − 0.05 ⎡⎣ K -1 ⎤⎦ (T f − 285 [ K ])

Note that the COP is reduced as the temperature of the fluid increases (it is harder to provide
cooling when you have to reject heat at a higher temperature). You may neglect any change in
the temperature of the fluid as it is circulated through the bore hole and also neglect the thermal
resistance due to convection between the fluid and the internal surface of the bore hole. The
ground around the bore hole can be treated using a 1-D transient model. The conductivity of the
ground is k = 1.6 W/m-K and the product of the density and specific heat capacity is ρ c =
2.8x106 J/m3. The radius of the bore is rbore = 5 cm. You should develop a 1-D numerical model
of the bore field using the ode solver MATLAB. Set the outer radius of the computational
domain to a value that is large enough that it does not impact your results. The boundary
condition at r = rbore should be the specified heat load, qbore , that is consistent with the fluid
temperature and the building conditions. The length of the bore hole is L = 50 m and the
temperature of the ground initially is Tground = 290 K.
a.) Write down the state equations for the nodes in your numerical model. That is, derive a set
of equations that will provide the derivatives for the temperatures at each of the nodes (Ti for
i = 1..N) if you know t and the instantaneous values of the temperatures at each of the nodes.

The radial location of each of the N nodes is specified according to:

( rout − rbore ) i -1
ri = rbore + ( ) for i = 1.. N (1)
( N − 1)
where rout is taken to be a large number, initially rout = 5 m; the value of rout should be adjusted if
necessary based on inspecting the results. Note that node 1 is on the bore hole surface while
node N is placed far from the bore hole. The distance between adjacent nodes is:

Δr =
( rout − rbore ) (2)
( N − 1)
An energy balance on the internal nodes leads to:

2π k L 2π k L ⎡⎛ Δr ⎞ ⎛
2
Δr ⎞ ⎤
2
dT
(Ti −1 − Ti ) + (Ti +1 − Ti ) = π ⎢⎜ ri + ⎟ − ⎜ ri − ⎟ ⎥ L ρ c i
⎛ r ⎞ ⎛r ⎞ ⎣⎢⎝ 2 ⎠ ⎝ 2 ⎠ ⎦⎥ dt
ln ⎜ i ⎟ ln ⎜ i +1 ⎟ (3)
⎝ ri −1 ⎠ ⎝ ri ⎠
for i = 2.. ( N − 1)
Rearranging Eq. (3) leads to the state equation for each internal node:

⎡ ⎤
⎢ ⎥
2k ⎢ ( Ti −1 − Ti ) ( Ti +1 − Ti ) ⎥
+
⎢ ⎛ ri ⎞ ⎛r ⎞ ⎥
⎢ ln ⎜ ⎟ ln ⎜ i +1 ⎟ ⎥
dTi
= ⎣ ⎝ ri −1 ⎠ ⎝ ri ⎠ ⎦
for i = 2.. ( N − 1) (4)
dt ⎡⎛ Δr ⎞ ⎛
2
Δr ⎞ ⎤
2

⎢⎜ ri + ⎟ − ⎜ ri − ⎟ ⎥ ρ c
⎣⎢⎝ 2 ⎠ ⎝ 2 ⎠ ⎦⎥

The node at r = rout can either be treated as adiabatic or fixed temperature (the value of rout
should be adjusted so that the perturbation at the bore hole surface never reaches this radial
location, therefore the boundary condition doesn’t matter). Here, we will assume that the
temperature remains constant, therefore the state equation for node N is:

dTN
=0 (5)
dt

The state equation for the node placed on the bore hole surface (node 1) is obtained from an
energy balance:

2π k L ⎡⎛ Δr ⎞
2
⎤ dT
qbore + (T2 − T1 ) = π ⎢⎜ r1 + ⎟ − r12 ⎥ L ρ c 1 (6)
⎛r ⎞ ⎣⎢⎝ 2 ⎠ ⎦⎥ dt
ln ⎜ 2 ⎟
⎝ r1 ⎠

Rearranging Eq. (6) leads to:

⎡ ⎤
⎢ ⎥
⎢ qbore + 2 π k L ( T2 − T1 ) ⎥
⎢ ⎛r ⎞ ⎥
⎢ ln ⎜ 2 ⎟ ⎥
dT1 ⎣ ⎝ r1 ⎠ ⎦
= (7)
⎡ Δr
2

π ⎢⎛⎜ r1 + ⎞⎟ − r12 ⎥ L ρ c
dt
⎢⎣ ⎝ 2 ⎠ ⎥⎦

The heat transfer to the bore hole in Eq. (7) depends on the building load and the COP of the
heat pump according to:

qload ⎛ 1 ⎞
qbore = qload + w = qload + = qload ⎜ 1 + ⎟ (8)
COP ⎝ COP ⎠
where qload and COP are given by:

⎛ ⎡ rad ⎤ ⎞
qload = 1148.78 [ W ] − 1088.8 [ W ] sin ⎜ 1.992x10-7 ⎢ ⎥ t − 425.81⎟ (9)
⎝ ⎣ s ⎦ ⎠

COP = 4 − 0.05 ⎡⎣ K -1 ⎤⎦ (T1 − 285[ K ] ) (10)

Equations (4), (5), and (7) through (10) are the state equations for the problem; if I have values
for t and Ti for i = 1..N then these equations provide the derivatives for each of the state
variables.

b.) Implement the state equations in MATLAB and simulate 1 year. Plot the temperature as a
function of time for various values of radial position.

A function dTdtP3p8d4 is generated. The function inputs include the time and vector of
temperatures as well as the other important problem inputs. The function returns the derivatives
of each of the state variables.

function[dTdt]=dTdtP3p8d4(t,T,r_out,r_bore,k,rhoc,L,r,N)

% Inputs
% t - time (s)
% T - vector of nodal temperatures (K)
% r_out - outer radius of computational domain (m)
% r_bore - bore radius (m)
% k - conductivity of ground (W/m-K)
% rhoc - product of density and specific heat capacity (J/m^3-K)
% r - vector of radial locations (m)
% N - number of nodes (-)

Dr=(r_out-r_bore)/(N-1); % distance between nodes


q_dot_load=1148.78-1088.8*sin(1.992e-7*t-425.81);
% load on building
COP=4-0.05*(T(1)-285); % COP of heat pump
w_dot=q_dot_load/COP; % power required by heat pump
q_dot_bore=(q_dot_load+w_dot); % load on bore

%energy balance on node 1


dTdt(1,1)=(q_dot_bore+2*pi*k*L*(T(2)-...
T(1))/log(r(2)/r(1)))/(pi*((r(1)+Dr/2)^2-r(1)^2)*L*rhoc);
%energy balance on nodes 2 through N-1
for i=2:(N-1)
dTdt(i,1)=2*k*((T(i-1)-T(i))/log(r(i)/r(i-1))+(T(i+1)-...
T(i))/log(r(i+1)/r(i)))/(rhoc*(r(i)+Dr/2)^2-(r(i)-Dr/2)^2);
end
%node N never changes temperature
dTdt(N,1)=0;

end
The state equations are integrated through time using the function P3p8d4.

function[t,r,T]=P3p8d4(L)

% Inputs
% L - length of bore field (m)

% Outputs
% t - vector of times at which solution is provided (s)
% r - vector of radial locations at which solution is provided (m)
% T - matrix of temperatures at each time and location (K)

r_out=5; % outer radius of computational domain (m)


k=1.6; % conductivity of ground (W/m-K)
rhoc=2.8e6; % product of density and specific heat capacity (J/m^3-K)
r_bore=0.05; % radius of bore (m)
T_ground=290; % initial ground temperature (K)

N=51; % number of radial locations (-)


for i=1:N
r(i,1)=r_bore+(i-1)*(r_out-r_bore)/(N-1);
% radial locations of nodes (m)
T_ini(i,1)=T_ground;
% initial temperature of ground
end

t_sim=1*365*24*3600; % simulation time


[t,T]=ode45(@(t,T) dTdtP3p8d4(t,T,r_out,r_bore,k,rhoc,L,r,N),...
[0,t_sim], T_ini);
end

Figure P3.8-4(c) illustrates the temperature of the ground at various radial locations. Note that
the temperature at r = rout = 500 cm is not affected by the heat load at the bore hole and therefore
rout has been chosen appropriately.
315

310 r = 5 cm
r = 14.9 cm r = 74.3 cm
Temperature (K)

305 r = 24.8 cm r = 113.9 cm


r = 44.6 cm r = 193.1 cm
r = 292.1 cm
300

295

290

285
0 1.000x107 2.000 x10 7 3.000x107
Time (s)
Figure P3.8-4(c): Temperature of ground as a function of time at various radial locations during the first
year.
c.) Plot the temperature at the bore surface as a function of time for 10 years. You should see
both a seasonal variation related to load as well as a long-term buildup of energy in the
ground (referred to as “annealing” the ground).

The simulation time is increased to 10 years.

t_sim=linspace(0,10*365*24*3600,4000); % simulation time


[t,T]=ode45(@(t,T) dTdtP3p8d4(t,T,r_out,r_bore,k,rhoc,L,r,N), t_sim,...
T_ini);

The temperature at the bore surface (T1) is shown in Figure P3.8-4(d).


315

310
Temperature (K)

305

300

295

290
0 1.000x108 2.000 x10 8 3.000x108
Time (s)
Figure P3.8-4(d): Temperature at the surface of the bore as a function of time for 10 years.

d.) The cost of the electricity required to run the heat pump is ec = 0.1 $/kW-hr and the cost of
installing a bore is bfc = 20 $/m. Integrate the electrical cost in order to determine the
operating cost for 10 years. Neglect the time value of money.

The operating cost is obtained according to:


tsim

OC = ec ∫ w dt (11)
0

This is accomplished by adding the code below to the function P3p8d4.

[M,g]=size(t); % determine number of time steps


for j=1:M
q_dot_load=1148.78-1088.8*sin(1.992e-7*t(j)-425.81); % load (W)
COP=4-0.05*(T(j,1)-285); % COP
w_dot(j)=q_dot_load/COP; % rate of power consumption (W)
end

ec=2.778e-8; %energy cost ($/J)


% integrated power
W=0;
for j=1:(M-1)
W=W+(w_dot(j)+w_dot(j+1))*(t(j+1)-t(j))/2;
end
OC=ec*W; % operating cost

The capital cost is calculated according to:

CC = bfc L (12)

bfc=20; % cost of bore hole ($/m)


CC=bfc*L; % capital cost ($)

The header is changed to output the operating cost:

function[t,r,T,OC,CC]=P3p8d4(L)

% Inputs
% L - length of bore field (m)

% Outputs
% t - vector of times at which solution is provided (s)
% r - vector of radial locations at which solution is provided (m)
% T - matrix of temperatures at each time and location (K)
% OC - operating cost ($)
% CC - capital cost ($)

which leads to OC = $3548 over 10 years and a capital cost of CC = $1000.

e.) Plot the operating cost, capital cost, and total cost (cost of the bore hole plus the operating
cost) as a function of bore length, L, for 10 years of operation (again, neglect the time value
of money). You should see an optimal bore length. Explain why this occurs.

The operating cost, capital cost, and total cost are evaluated at various values of the bore length
using the MATLAB script P3p8d4_makefig:

Lv=[35,40,50,75,100,125,150,175,200,250,300]';
for i=1:11
i
[t,r,T,OC(i),CC(i)]=P3p8d4(Lv(i));
TC(i)=OC(i)+CC(i);
end

The operating cost, capital cost, and total cost as a function bore length is shown in Figure P3.8-
4(e).
8000

7000 total cost

6000

Cost ($)
5000
capital cost
4000

3000
operating cost

2000
0 50 100 150 200 250 300
Bore length (m)
Figure P3.8-4(e): Operating cost, capital cost, and total cost as a function of bore length for 10 years of
operation.

As the bore length is reduced, the operating cost increases because the temperature at the bore
surface is elevated, reducing the COP of the heat pump. As the bore length is increased, the
capital cost increases because a longer bore is more expensive. An optimal length balances these
two effects.
Problem 3.8-5 (3-23 in text)
A pin fin is used as part of a thermal management system for a power electronics system, as
shown in Figure P3.8-5.

h = 50 W/m -K
2

D = 3 mm T∞ = 20°C

k = 10 W/m-K L = 3 cm
ρ = 4000 kg/m3
q
c = 400 J/kg-K

Figure P3.8-5: Pin fin subjected to a transient heat load.

The diameter of the fin is D = 3 mm and the length is L = 3 cm. The fin material has
conductivity k = 10 W/m-K, ρ = 4000 kg/m3, and c = 400 J/kg-K. The surface of the fin is
exposed to air at T∞ = 20ºC with heat transfer coefficient h = 50 W/m2-K. The tip of the fin can
be assumed to be adiabatic. The power electronics system does not operate at steady state;
rather, the load applied at the base of the fin cycles between a high and a low value with some
angular frequency, ω. The average heat transfer rate is q = 0.5 W and the amplitude of the
fluctuation is Δq = 0.1 W. The frequency of oscillation varies. The fin is initially in equilibrium
with T∞.
a.) Develop a 1-D transient model that can be used to analyze the startup and operating behavior
of the pin fin. Use the ode solver in MATLAB.

The inputs are entered in a MATLAB script:

clear all;
D=0.003; % diameter of fin (m)
k=10; % conductivity of fin (W/m-K)
rho=4000; % density (kg/m^3)
c=400; % specific heat capacity (J/kg-K)
L=0.03; % length (m)
h_bar=50; % heat transfer coefficient (W/m^2-K)
T_infinity=293.2; % ambient temperature (K)
omega=0.1; % frequency (rad/s)
Dq_dot=0.1; % amplitude of heat load (W)
q_dot_bar=0.5; % average heat load (W)

Nodes are distributed uniformly throughout the node according to:

(i − 1)
xi = L for i = 1..N (1)
( N − 1)
where N is the number of nodes used for the simulation. The distance between adjacent nodes
(Δx) is:
L
Δx = (2)
( N − 1)
N=21; % number of nodes (-)
for i=1:N
x(i,1)=L*(i-1)/(N-1); % locations of nodes (m)
end

An energy on the internal nodes leads to:

π D2 π D2 π D2 dTi
h Δx π D (T∞ − Ti ) + k (Ti −1 − Ti ) + k (Ti +1 − Ti ) = Δx ρ c for i = 2.. ( N − 1) (3)
4 Δx 4 Δx 4 dt

which can be solved for the time rate of change of Ti:

⎡ πD ⎤
⎢ h Δx π (T∞ − Ti ) + k 4 Δx (Ti −1 + Ti +1 − 2 Ti ) ⎥
dTi ⎣ ⎦
= for i = 2.. ( N − 1) (4)
dt π D
Δx ρ c
4

An energy balance on node 1 leads to:

π D2 Δx π D2 dT
q + k (T2 − T1 ) + h π D (T∞ − T1 ) = Δx ρ c 1 (5)
4 Δx 2 8 dt

which can be solved for the time rate of change of T1:

⎡ π D2 Δx ⎤
⎢ 
q + k (T2 − T1 ) + h π D (T∞ − T1 )⎥
dT1 ⎣ 4 Δx 2 ⎦
= (6)
dt πD 2
Δx ρ c
8

An energy balance on node N leads to:

Δx π D2 π D2 dT
h π D (T∞ − TN ) + k (TN −1 − TN ) = Δx ρ c N (7)
2 4 Δx 8 dt

which can be solved for the time rate of change of TN:


⎡ Δx π D2 ⎤
h
⎢ 2 π D ( T∞ − TN ) + k (TN −1 − TN ) ⎥
dTN ⎣ 4 Δx ⎦
= (8)
dt πD 2
Δx ρ c
8

These state equations, Eqs. (4), (6), and (8), are programmed in a function dTdtP3p8d5.

function[dTdt]=dTdtP3p8d5(t,T,D,k,rho,c,L,h_bar,T_infinity,omega,Dq_dot,...
q_dot_bar,N)

% Inputs
% t - time (s)
% T - vector of nodal temperatures (K)
% D - diameter of fin (m)
% k - conductivity of ground (W/m-K)
% rho - density (kg/m^3)
% c - specific heat capacity (J/kg-K)
% L - length (m)
% h_bar - heat transfer coefficient (W/m^2-K)
% T_infinity - ambient temperature (K)
% omega - frequency of heat transfer oscillation (rad/s)
% Dq_dot - amplitude of heat transfer oscillation (W)
% q_dot_bar - average heat transfer (W)
% N - number of nodes (-)

Dx=L/(N-1); % distance between nodes


q_dot=q_dot_bar+Dq_dot*sign(sin(omega*t)); % load on fin

dTdt=zeros(N,1);

%energy balance on node 1


dTdt(1)=(q_dot+k*pi*D^2*(T(2)-T(1))/(4*Dx)+h_bar*pi*D*Dx*(T_infinity-...
T(1))/2)/(rho*c*Dx*pi*D^2/8);
%energy balance on nodes 2 through N-1
for i=2:(N-1)
dTdt(i)=(h_bar*Dx*(T_infinity-T(i))+k*D*(T(i-1)+T(i+1)-...
2*T(i))/(4*Dx))/(rho*c*Dx*D/4);
end
%energy balance on node N
dTdt(N,1)=(h_bar*Dx*(T_infinity-T(N))/2+k*D*(T(N-1)-...
T(N))/(4*Dx))/(rho*c*Dx*D/8);
end

The ode15s function in MATLAB is used to integrate the state equations forward in time.

for i=1:N
T_ini(i,1)=T_infinity; % initial temperature (K)
end
t_sim=100; % simulation time (s)
OPTIONS=odeset('RelTol',1e-5);
[t,T]=ode15s(@(t,T)
dTdtP3p8d5(t,T,D,k,rho,c,L,h_bar,T_infinity,omega,Dq_dot,q_dot_bar,N),...
[0,t_sim], T_ini,OPTIONS);
b.) Plot the temperature as a function of time at various values of axial position for the start up
assuming a constant heat load (ω = 0).

The value of ω is set to zero:

omega=0; % frequency (rad/s)

and the simulation is used to generate Figure 2.


390

x = 0 cm
370
Temperature (K)

350
x = 0.6 cm

330
x = 1.2 cm

x = 1.8 cm
310

x = 2.4 cm x = 3.0 cm
290
0 20 40 60 80 100
Time (s)
Figure 2: Temperature as a function of time at various values of axial position with a constant heat load.

c.) Calculate a diffusive time constant and a lumped capacitance time constant for the
equilibration process. Is the plot from (b) consistent with these values?

The diffusive time constant is:

L2
τ diff = (9)

The lumped time constant is:

1 π D2 ρcD
τ lumped = R C = ρc L= (10)
hπ DL 4 4h

alpha=k/(rho*c); % thermal diffusivity (m^2/s)


tau_diff=L^2/(4*alpha) % diffusive time constant (s)
tau_lump=(rho*c*D/4)/(h_bar) % lumped time constant (s)

which leads to τdiff = 36 s and τlumped = 24 s; this is consistent with the time required for the fin to
achieve equilibrium as shown in Figure 2.
d.) Adjust the diameter of the fin so that the lumped time constant is much greater than the
diffusive time constant. Plot the temperature as a function of time at various values of axial
position for the start up assuming a constant heat load (ω = 0). Explain your result.

The lumped time constant is proportional to D while the diffusive time constant is independent
of D. The value of the diameter is increased by a factor of 10 to increase the lumped capacitance
time constant relative to the diffusive time constant. The result is shown in Figure 3. Notice that
the diffusive equilibration occurs very quickly (within ~36 s) and therefore the temperature
distribution takes on its equilibrium shape and then subsequently the magnitude of the
temperature everywhere changes according to the lumped capacitance time constant.
297.5
x = 0 cm
297 x = 0.6 cm
x = 1.2 cm
296.5
x = 1.8 cm
Temperature (K)

296 x = 2.4 cm
x = 3.0 cm
295.5

295

294.5

294

293.5

293
0 200 400 600 800 1000
Time (s)
Figure 3: Temperature as a function of time at various values of axial position with a constant heat load and a
diameter adjusted to that τlumped >> τdiff.

e.) Return the diameter of the fin to D = 3 mm and set the oscillation frequency to ω = 1 rad/s.
Prepare a contour plot showing the temperature of the fin as a function of position and time.
You should see that the oscillation of the heat load causes a disturbance that penetrates only
part-way along the axis of the fin. Explain this result.

Figure 4 illustrates the temperature as a function of time for various axial positions and Figure 5
illustrates the contour plot requested by the problem statement. Note that the effect of the
oscillation only extends ≈ 2 α tosc = 1.3 cm into the fin.
390
x = 0 cm

370

Temperature (K)
350 x = 0.6 cm

330 x = 1.2 cm
x = 1.8 cm

310
x = 2.4 cm
x = 3.0 cm
290
0 20 40 60 80 100
Time (s)
Figure 4: Temperature as a function of time at various values of axial position with an oscillating heat load.

100 380

90
370

80
360
70
350
60
Time (s)

340
50

330
40

30 320

20 310

10
300

0
0 0.005 0.01 0.015 0.02 0.025 0.03
Position (m)
Figure 5: Contour plot of temperature as a function of time at various values of axial position with an
oscillating heat load.

f.) Is the maximum temperature experienced by the fin under oscillating conditions at cyclic
steady-state (i.e., after the start-up transient has decayed) greater than or less than the
maximum temperature experienced under steady-state conditions (i.e., with ω = 0)?
Figure 6 illustrates the temperature of the base (x= 0) as a function of time for the steady and
oscillating load cases and shows that the maximum fin temperature is larger when the heat load
oscillating.
390

Base temperature (K) 370


oscillating load
steady load
350

330

310

290
0 20 40 60 80 100
Time (s)
Figure 6: Base temperature as a function of time for the steady and oscillating load cases.

g.) Plot the ratio of the maximum temperature under oscillating conditions to the maximum
temperature under steady-state conditions as a function of frequency.

Figure 7 illustrates the base temperature as a function of time for various values of the frequency.
400
1 rad/s

380
Base temperature (K)

10 rad/s
360
steady load 0.1 rad/s

340

320

300

0 20 40 60 80 100
Time (s)
Figure 7: Base temperature as a function of time for various values of the frequency.

Figure 8 illustrates the ratio of the maximum base-to-fluid temperature difference under an
oscillating condition to the maximum base-to-fluid temperature difference with a steady heat
transfer as a function of the frequency of oscillation.
Maximum temp. to maximum steady temp.
1.05

1.04

1.03

1.02

1.01

1
0.01 0.1 1 10 20
Angular frequency (rad/s)
Figure 8: Ratio of the base temperature difference under oscillating conditions to the base temperature
difference with a steady heat transfer as a function of frequency.

h.) Define a meaningful dimensionless frequency and plot the maximum temperature under
oscillating conditions to the maximum temperature under steady-state conditions as a
function of this dimensionless frequency. Explain the shape of your plot.

The most appropriate dimensionless frequency is the ratio of τdiff to the time required for a single
oscillation.

L2 2 π
ω = (11)
4α ω

Figure 9 illustrates the ratio of the maximum base-to-fluid temperature difference under an
oscillating condition to the maximum base-to-fluid temperature difference with a steady heat
transfer as a function of the dimensionless frequency of oscillation. Note that if ω >>1 then the
ratio approaches 1 because the effect of the oscillation is confined to a relatively small region
near the base and the associated temperature rise is small (see Figure 7). If ω << 1 then the ratio
asymptotes to a number higher than 1 because essentially assumes a steady state condition at the
higher heat load and another steady state condition at the lower heat load (see Figure 7).
1.05

Maximum temp. to maximum steady temp.


1.04

1.03

1.02

1.01

1
0.05 0.1 1 10 100
Dimensionless frequency
Figure 9: Ratio of the base temperature difference under oscillating conditions to the base temperature
difference with a steady heat transfer as a function of dimensionless frequency.
Problem 3.8-6
A refrigerated warehouse is used to store cheese. A very simplified schematic of this situation is
shown in Figure 3.8-6. The ASHRAE handbook recommends a safe storage temperature for milk
products that ranges from 0.6 to 4.4°C.

cheese blocks warehouse wall


Aw = 65,000 m
2
kch = 0.31 W/m-K
Rw′′ = 4.0 K-m /W
2
ρch = 1090 kg/m3
cch = 2102 J/kg-K
ambient air fluctuates
between 16°C and 37°C
Tin with a peak at 2:00 pm

qref qrej

Ach = 50,000m
2

h = 4.0 W/m -K
2

thp = 2.0 mm
k p = 1.5 W/m-K refrigeration system
w with 25% Carnot efficiency
Figure 3.8-6: Schematic of a refrigerated warehouse used to store cheese.

The total resistance of the warehouse wall per unit area is Rw′′ = 4.0 K-m2/W (this includes the
effect of convection on the inside and outside surfaces) and there is 65,000 m2 of warehouse wall
separating the internal air temperature (Tin) from the ambient air temperature (T∞). During a
typical summer day, the temperature of the ambient air oscillates sinusoidally with a 24 hour
period between 16°C and 37°C; the maximum temperature occurs at 2:00 pm. The heat load
through the wall ( qw ) is removed by an industrial refrigeration system with a Carnot efficiency
of 25% (that is, the refrigerator requires four times as much power, w , as a reversible
refrigerator operating between Tin and T∞). The cost of electrical energy to the warehouse during
on-peak hours (9:00 am to 9:00 pm) is 0.060$/kW-hr and drops to only 0.020$/kW-hr during off-
peak hours (9:00 pm to 9:00 am). You can assume that the refrigeration control system and
refrigeration equipment is capable of exactly controlling the internal temperature of the air
within the building. Currently, the warehouse operates the refrigeration system so that it
maintains the internal air temperature at a constant value of Tin = 4.4°C.
a.) Prepare a plot of the outside air temperature as a function of time for a complete day and the
refrigeration load and input power as a function of time for a complete day. What is total
cost required to refrigerate the warehouse using this control scheme ($/day)?

The input parameters are entered in MATLAB:

clear all;
A=65000; % area of warehouse walls (m^2)
Rwdp=4.0; %resistance of warehouse walls per area (K-m^2/W)
Tamin=16; %minimum outside air temp (C)
Tamax=37; %maximum outside air temp (C)
period=24*3600; %period of outside air oscillation (s)
Tz=4.4; %zone temperature (C)
eta_C=0.25; %Carnot efficiency of refrigeration system
costonpeak=0.06; %cost of electricity on-peak ($/kW-hr)
costoffpeak=0.02; %cost of electricity off-peak ($/kW-hr)

The day is divided into N time steps according to:

ti =
( i − 1) period for i = 1..N (1)
( N − 1)
period
Δt = (2)
( N − 1)

where period is the time for 1 day and Δt is the time step duration.

N=3600; %number of timesteps (-)


time=linspace(0,period,N)'; %time (s)
dtime=period/(N-1); %time step duration (s)

The sinusoidal variation of the air temperature is given by:

⎛ 2π t ⎞
Ta = Ta + ΔTa sin ⎜ − 2.094 [ rad ] ⎟ (3)
⎝ period ⎠

where

Ta =
(T
a , min + Ta , max )
(4)
2

and

ΔTa =
(T a , max − Ta ,min )
(5)
2

Tabar=(Tamin+Tamax)/2; %average outdoor air temperature (C)


DTa=(Tamax-Tamin)/2; %amplitude of outdoor air temperature oscillation (C)
Ta=Tabar+DTa*sin(2*pi*time/period-2.0944); %outdoor air temperature (C)

The outdoor air temperature as a function of time of day is shown in Figure 2.


37.5

35

32.5

Outdoor air temperature (°C)


30

27.5

25

22.5

20

17.5

15
0 5 10 15 20 25
Time of day (hr)
Figure 2: Outdoor air temperature as a function of the time of day.

The rate of heat transfer through the walls of the ware house is:

Aw (Ta − Tin )
qw = (6)
Rw′′

An entropy balance on a reversible air conditioning system suggests that the rate of heat rejection
is:

Ta
qh ,rev = q w (7)
Tin

and so the power required by a reversible air conditioning system is:

w rev = qh ,rev − qw (8)

The actual power is higher according to the 2nd law efficiency:

w rev
w = (9)
ηc

qw=(Ta-Tz)*A/Rwdp; %rate of heat transfer through warehouse walls (W)


qh_rev=qw.*(273.2+Ta)./(273.2+Tz); %heat rejection - rev. AC (W)
w_rev=qh_rev-qw; %power - rev. AC (W)
w=w_rev/eta_C; %power - actual AC (W)

The refrigeration load and input power as a function of the time of day.
600

Refrigeration load and power (kW)


500

400 refrigeration load

300

200

input power
100

0
0 5 10 15 20 25
Time of day (hr)
Figure 3: Refrigeration load and power as a function of the time of day.

The total cost is obtained by averaging the input power over each time step and multiplying by
the time step duration and the appropriate cost (on peak or off peak):

totalcost=0;
for i=1:(N-1)
if(time(i)<9*(3600))
totalcost=totalcost+costoffpeak*(w(i)+w(i+1))*dtime/(2*1000*3600);
else
if(time(i)<21*(3600))
totalcost=totalcost+costonpeak*(w(i)+w(i+1))*dtime/(2*1000*3600);
else
totalcost=totalcost+costoffpeak*(w(i)+w(i+1))*dtime/(2*1000*3600);
end
end
end

which leads to a cost of $154.26 per day, which corresponds to roughly $56,000 per year.

You propose that the cheese warehouse take advantage of the thermal capacitance of the cheese
within the factory by operating the refrigeration system at a higher level during off-peak hours in
order to sub-cool the cheese (i.e., cool it to near its lower safe storage temperature of 0.6°C).
The sub-cooled cheese will warm during the on-peak hours and reduce the refrigeration load. To
implement this strategy, you program the refrigeration system to cool the internal air to 0.6°C
during off-peak hours and then switch to 4.4°C during on-peak hours. Again, assume that the
refrigeration system is perfect so that the air temperature within the warehouse can be controlled
exactly. This approach has two potential advantages: (1) you can “shift” your refrigeration load
from on-peak to off-peak hours when electricity is cheaper, and (2) you can take advantage of
the lower ambient air temperatures during the night which translate to higher refrigeration
efficiency. There is a potential disadvantage in that you have to operate your refrigeration
system at a lower temperature some of the time in order to sub-cool the cheese. The cheese has
conductivity kch = 0.31 W/m-K, density ρch = 1090 kg/m3, and specific heat capacity cch = 2102
J/kg-K. You can assume that the cheese behaves like a semi-infinite body That is, we can
ignore the exact shape of the cheese blocks because the thermal wave associated with the
periodically fluctuating temperature of the air within the warehouse never penetrates to the
center of the cheese block. The total surface area of cheese blocks that are exposed to the
internal air temperature Ach = 50,000 m2. The heat transfer coefficient between the cheese
surface and the internal air is h = 4.0 W/m2-K. Each block of cheese is wrapped with a thin
layer of plastic with thickness thp = 2.0 mm and conductivity kp = 1.5 W/m-K.
b.) How big do the cheese blocks need to be (approximately) for the semi-infinite body
assumption to be valid?

The inputs are entered into a new MATLAB script:

clear all;
Aw=65000; %area of warehouse walls (m^2)
Rwdp=6.0; %resistance of warehouse walls per area (K-m^2/W)
Tamin=16; %minimum outside air temp (C)
Tamax=37; %maximum outside air temp (C)
Tzmin=0.6; %minimum zone temperature (C)
Tzmax=4.4; %maximum zone temperature (C)
period=24*3600; %period of outside air oscillation (s)
eta_C=0.25; %Carnot efficiency of refrigeration system
costonpeak=0.044; %cost of electricity on-peak ($/kW-hr)
costoffpeak=0.028; %cost of eelctricity off-peak ($/kW-hr)
kch=0.31; %conductivity of cheese (W/m-K)
rhoch=1090; %density of cheese (kg/m^3)
cch=2102; %specific heat capacity of cheese (J/kg-K)
h=4; %heat transfer coefficient (W/m^2-K)
Ach=50000; %surface area of cheese (m^2)
tp=0.002; %thickness of plastic wrap (m)
kp=1.5; %conductivity of plastic (W/m-K)

The penetration of the thermal wave into the chees block is estimated according to:

δ t = 2 α ch period / 2 (10)

where αch is the thermal diffusivity of the chees:

alphach=kch/(rhoch*cch); %thermal diffusivity of cheese (m^2/s)


delta=2*sqrt(alphach*period/2) %penetration depth (m)

which leads to δt = 0.15 m.

c.) Prepare a 1-D, transient numerical model of the cheese using an implicit technique.
Implement your solution in MATLAB subject to the internal air temperature boundary
condition discussed above (i.e., a stepwise change from 0.6°C to 4.4°C at 9:00 am and back
again at 9:00 pm). Note that your solution must start with the cheese all at some initial
condition and then integrate forward in time until a cyclic steady-state condition is achieved
(that is, the temperature of the cheese at the beginning and end of the day is the same).
Prepare a plot of the cheese temperature at the surface and several positions within the cheese
for this “typical” day.
The day is divided into M time steps and the outdoor air temperature at each time step is
determined:

M=3600; %number of timesteps (-)


time=linspace(0,period,M)'; %time (s)
Dt=period/(M-1); %timestep duration (s)
Tabar=(Tamin+Tamax)/2; %average outdoor temperature (C)
DTa=(Tamax-Tamin)/2; %amplitude of outdoor air temperature oscillation (C)
Ta=Tabar+DTa*sin(2*pi*time/period-2.0944); %outdoor air temperature (C)

The zone temperature is assigned based on the time of day:

for j=1:M
if(time(j)<9*(3600))
Tz(j)=Tzmin;
else
if(time(j)<21*(3600))
Tz(j)=Tzmax;
else
Tz(j)=Tzmin;
end
end
end

The initial temperature distribution in the cheese is uniform and taken to be at the average of the
high and low zone temperatures, Tin,min and Tin,max:

Ti ,1 =
(T
in , min + Tin ,max )
for i = 1..N (11)
2

%setup initial temperature distribution


for i=1:N
T(i,1)=(Tzmin+Tzmax)/2;
end

The cheese is divided into N nodes. The depth of cheese that is simulated is a = 0.4 m, which is
larger than the value of δt determined in part (b).

xi =
( i − 1) a for i = 1..N (12)
( N − 1)
a
Δx = (13)
( N − 1)
a=0.4; %depth of cheese to simulate (m)
N=25; %number of nodes (-)
Dx=a/(N-1); %distance between nodes (m)
for i=1:N
x(i)=(i-1)*a/(N-1); %position of nodes
end

An energy balance on the internal nodes:

kch A (Ti −1 − Ti ) kch A (Ti +1 − Ti ) dT


+ = ρ ch cch A Δx i for i = 2.. ( N − 1) (14)
Δx Δx dt

and energy balances on nodes 1 and N lead to:

kch A (T2 − T1 ) ρ ch cch A Δx dT1


heff A (Tin − T1 ) + = (15)
Δx 2 dt

kch A (TN −1 − TN ) ρch cch A Δx dTN


= (16)
Δx 2 dt

where heff is an equivalent heat transfer coefficient that includes the effect of the plastic:

−1
⎛ 1 th ⎞
heff = ⎜ + p ⎟ (17)
⎜ h kp ⎟
⎝ ⎠

heff=1/(1/h+tp/kp); %effective heat transfer coefficient (W/m^2-K)

This simulation will use a Crank-Nicolson technique. The steps are taken according to:

⎡⎛ dT ⎞ ⎛ dT ⎞ ⎤ Δt
Ti , j +1 = Ti , j + ⎢⎜ ⎟ +⎜ ⎟ ⎥ (18)
⎢⎣⎝ dt ⎠i , j ⎝ dt ⎠i , j +1 ⎥⎦ 2

or, using Eqs. (14) through (16):

kch Δt
Ti , j +1 = Ti , j +
Δx ρch cch
2 (Ti −1, j + Ti +1, j − 2 Ti , j + Ti −1, j +1 + Ti +1, j +1 − 2 Ti , j +1 )
2
for i = 2.. ( N − 1) (19)

⎡ kch (T2, j − T1, j ) kch (T2, j +1 − T1, j +1 ) ⎤ Δt


⎢ heff (Tin − T1, j ) + + h (Tin − T1, j +1 ) +
2
T1, j +1 = T1, j + ⎥ (20)
ρ ch cch Δx ⎢⎣ Δx Δx ⎥⎦ 2

2 kch Δt
TN , j +1 = TN , j +
Δx ρch cch
2 (T N −1, j − TN , j + TN −1, j +1 − TN , j +1 )
2
(21)

Equations (19) through (21) are manipulated and placed into matrix format:
⎡ k Δt ⎤ ⎡ kch Δt ⎤ ⎡ kch Δt ⎤
Ti , j +1 ⎢1 + 2ch ⎥ + Ti −1, j +1 ⎢ − ⎥ + Ti +1, j +1 ⎢ − ⎥=
⎣ Δx ρ ch cch ⎦ ⎣ 2 Δx ρch cch ⎦
2
⎣ 2 Δx 2 ρ ch cch ⎦




Ai ,i Ai ,i −1 Ai ,i +1
(22)
kch Δt
Ti , j + 2
Δx ρch cch
( Ti −1, j + Ti +1, j − 2 Ti , j )
2
for i = 2.. ( N − 1)


bi

⎡ h Δt kch Δt ⎤ ⎡ kch Δt ⎤
T1, j +1 ⎢1 + eff + 2⎥
+ T2, j +1 ⎢ − =
ρch cch Δx ρ ch cch Δx ⎦
⎣ ⎣ ρch cch Δx 2 ⎥⎦



A1,1 A1,2
(23)
⎡ kch (T2, j − T1, j ) ⎤ Δt
⎢ h ( 2 Tin − T1, j ) +
2
T1, j + ⎥
ρ ch cch Δx ⎢⎣ Δx ⎥⎦ 2


b1

⎡ k Δt ⎤ ⎡ kch Δt ⎤ 2 kch Δt
TN , j +1 ⎢1 + 2ch ⎥ + TN −1, j +1 ⎢ − 2
Δx ρch cch ⎦
⎥ = TN , j + 2
Δx ρch cch ⎦ Δx ρ ch cch
( TN −1, j − TN , j ) (24)
⎣
⎣

2

AN , N AN , N −1 bN

The A matrix can be created just once and used for each time step:

A=spalloc(N,N,3*N);
%setup A matrix
A(1,1)=1+heff*Dt/(rhoch*cch*Dx)+kch*Dt/(rhoch*cch*Dx^2);
A(1,2)=-kch*Dt/(rhoch*cch*Dx^2);
for i=2:(N-1)
A(i,i)=1+kch*Dt/(Dx^2*rhoch*cch);
A(i,i-1)=-kch*Dt/(2*Dx^2*rhoch*cch);
A(i,i+1)=-kch*Dt/(2*Dx^2*rhoch*cch);
end
A(N,N)=1+kch*Dt/(Dx^2*rhoch*cch);
A(N,N-1)=-kch*Dt/(Dx^2*rhoch*cch);

The simulation is placed within a while loop that terminates when the cheese reaches a cyclic
steady state (i.e., the temperature distribution at the end of the day matches the temperature
distribution at the beginning of the day).

B=zeros(N,1);
err=999; %deviation from cyclic steady state (K)
tol=0.001; %tolerance for steady state (K)
while(err>tol)

The matrix equation is used to move through each time step; the vector b is constructed for each
time step:
%step through timesteps
for j=1:(M-1)
B(1)=T(1,j)+2*(heff*(2*Tz(j)-T(1,j))+kch*(T(2,j)-...
T(1,j))/Dx)*Dt/(2*rhoch*cch*Dx);
for i=2:(N-1)
B(i)=T(i,j)+kch*(T(i-1,j)+T(i+1,j)-...
2*T(i,j))*Dt/(2*Dx^2*rhoch*cch);
end
B(N)=T(N,j)+2*kch*(T(N-1,j)-T(N,j))*Dt/(2*Dx^2*rhoch*cch);
T(:,j+1)=A\B;
end

The average discrepancy between the temperature distribution at the beginning and end of the
day is computed:

err=max(abs(T(:,M)-T(:,1)))/N

and the temperature distribution at the end of the day is used as the initial temperature for the
subsequent day:

T(:,1)=T(:,M);
end

Figure 4 illustrates the temperature within the cheese as a function of time for various values of
position.

4.5

4
x = 6.7 cm
3.5
x = 10 cm
Temperature (°C)

3 x = 16.7 cm

2.5

1.5 x = 3.3 cm
x = 0 cm
1

0.5
0 5 10 15 20 25
Time of day (hr)
Figure 4: Temperature as a function of time for various values of position.

d.) Prepare a plot that includes the refrigeration load associated with the heat transfer through the
wall, the refrigeration load associated with heat transfer between the air and the cheese (note
that this will be negative and positive depending on the time of the day), and the total
refrigeration load required.
The refrigeration load through the walls ( qw ) is calculated as in part (a). The refrigeration load
imposed by heat transfer from the cheese is:

qch = heff Ach (Tx =0 − Tin ) (25)

The total refrigeration load is:

qtot = qch + qw (26)

qw=(Ta'-Tz)*Aw/Rwdp; %rate of heat transfer through warehouse walls (W)


qch=(T(1,:)-Tz)*Ach*heff; %rate of heat transfer from cheese (W)
qtotal=qw+qch; %total refrigeration load (W)

Figure 5 illustrates the refrigeration load through the walls, the refrigeration load from the
cheese, and the total load as a function of time. Comparing Figure 5 with Figure 3 shows that we
have successfully shifted the load from the middle of the day to the evening.

800 heat transfer through walls


heat transfer from cheese
600 total refrigeration load
Refrigeration load (kW)

400

200

-200

-400

-600
0 5 10 15 20 25
Time of day (hr)
Figure 5: Refrigeration load through the walls, the refrigeration load from the cheese, and the total load as a
function of time.

e.) Prepare a plot of the total refrigeration load and associated input power as a function of time
for a complete day. What is total cost required to refrigerate the warehouse using this
alternative control scheme ($/day)? Does it save money relative to your answer from (a)?

The input power is computed as in part (a); however, the work transfer to the refrigeration
system is not allowed to be negative (i.e., the refrigeration is not allowed to operate as a heat
engine during those portions of the day where the total load is negative):

qh_rev=qtotal'.*(273.2+Ta)./(273.2+Tz'); %heat rejection - rev. AC (W)


w_rev=qh_rev-qtotal'; %power - rev. AC (W)
for j=1:M
if(w_rev(j)>=0)
w(j)=w_rev(j)/eta_C; %power - actual AC (W)
else
w(j)=0;
end
end

The total cost is computed as in part (a):

totalcost=0;
for j=1:(M-1)
if(time(j)<9*(3600))
totalcost=totalcost+costoffpeak*(w(j)+w(j+1))*Dt/(2*1000*3600);
else
if(time(j)<21*(3600))
totalcost=totalcost+costonpeak*(w(j)+w(j+1))*Dt/(2*1000*3600);
else
totalcost=totalcost+costoffpeak*(w(j)+w(j+1))*Dt/(2*1000*3600);
end
end
end

which leads to a total cost of $59.10/day or $21,600 per year - a savings of approximately
$34,000 per year.
Problem 3.8-7
Figure P3.8-7 illustrates the flat plate solar collector that was previously analyzed under steady
state conditions in Problem 1.9-7.

qs′′ h = 5 W/m -K
2

T∞ = 10°C
Tw = 50°C ε=1 th = 1.5 mm

L = 8 cm
k = 75 W/m2-K
ρ = 8800 kg/m3
c = 375 J/kg-K
Figure 3.8-7: Flat plate solar collector.

The collector consists of a flat plate that is th = 1.5 mm thick with conductivity k = 75 W/m-K,
density ρ = 8800 kg/m3, and specific heat capacity c = 375 J/kg-K. The plate is insulated on its
back side and under normal conditions experiences a solar flux of qs′′,max = 900 W/m2 that is all
absorbed. The surface is exposed to convection and radiation to the surroundings. The
emissivity of the surface is ε = 1. The heat transfer coefficient is h = 5 W/m2-K and the
surrounding temperature is T∞ = 10ºC. The temperature of the water is Tw = 50ºC. The center-
to-center distance between adjacent tubes is 2 L where L = 8 cm. This problem deals with the
transient behavior of the collector. Assume that the plate is initially at the water temperature and
that the heat flux from the sun varies due to clouds. Specifically, assume that the solar flux is
given by:

⎧q ′′ 0 < t < 300 s


qs′′ = ⎨ s ,max (1)
⎩0 t > 300 s

so that the initial 300 s are related to the startup of the collector and the subsequent time is
related to the shut down of the collector after the flux is removed.
a.) Develop a numerical model that can predict the temperature distribution in the collector plate
during the transient process associated with the sun going behind a cloud. Plot the
temperature at t = 0 s, 300 s, and 600 s.

The inputs are entered in EES and a function is defined that returns the solar flux as a function of
time:

$UnitSystem SI MASS RAD PA K J


$Tabstops 0.2 0.4 0.6 3.5 in

function q``_s(time,q``_s_max)
if (time<300 [s]) then
q``_s= q``_s_max
else
q``_s=0 [W/m^2]
endif
end

"Inputs"
th=1.5 [mm]*convert(mm,m) "thickness"
k=75 [W/m-K] "conductivity of collector plate"
rho=8800 [kg/m^3] "density of collector plate"
c=375 [J/kg-K] "specific heat capacity of collector
plate"
e=1 [-] "emissivity of collector plate"
q``_s_max=900 [W/m^2] "solar flux"
T_infinity=converttemp(C,K,10[C]) "ambient temperature"
h_bar=5 [W/m^2-K] "heat transfer coefficient"
L=8 [cm]*convert(cm,m) "half-width between tubes"
T_w=converttemp(C,K,50 [C]) "water temperature"
W=1 [m] "per unit length of collector"

The nodes are positioned through the collector according to:

2L
Δx = (2)
( N − 1)

xi = Δx ( i − 1) for i = 1..N (3)

The positions in time where the solution will be evaluated are also positioned:

tsim
Δt = (4)
( M − 1)

ti = Δt ( j − 1) for j = 1..M (5)

where tsim is the total simulation time and M and N are the number of time and space locations,
respectively, at which we need to evaluate the solution:

N=11 [-] "number of nodes"


Dx=2*L/(N-1) "
duplicate i=1,N
x[i]=2*L*(i-1)/(N-1) "location of each node"
end

"Setup time steps"


M=41 [-] "number of time steps"
t_sim=10 [min]*convert(min,s) "simulation time"
Dt=t_sim/(M-1) "time step duration"
duplicate j=1,M
time[j]=(j-1)* Dt
end
The temperature of each node is initially equal to the water temperature:

Ti ,1 = Tw (6)

duplicate i=1,N
T[i,1]=T_w "initial condition"
end

An energy balance on an internal control volume leads to:

dU
q LHS + q RHS + qconv + qrad + qs = (7)
dt

The rate equations are substituted into Eq. (7):

k W th k W th
(Ti −1 − Ti ) + (Ti +1 − Ti ) + h W Δx (T∞ − Ti ) + σ ε W Δx (T∞4 − Ti 4 )
Δx Δx
(8)
dT
+ qs′′W Δx = ρ cW th Δx i
dt

which leads to

⎛ k th k th ⎞
⎜ ( Ti −1 − Ti ) + ( Ti +1 − Ti ) + h Δx (T∞ − Ti ) + σ ε Δx (T∞4 − Ti 4 ) + qs′′ Δx ⎟
dTi ⎝ Δx Δx ⎠
= (9)
dt ρ c th Δx
for i = 2.. ( N − 1)

A single fully implicit step is taken:

dT
Ti ,2 = Ti ,1 + Δt (10)
dt T =Ti ,2 ,t = t2

Substituting Eq. (9) into Eq. (10) leads to:

⎛ k th k th ⎞
⎜ ( Ti −1,2 − Ti ,2 ) + ( Ti +1,2 − Ti ,2 ) + h Δx (T∞ − Ti ,2 ) + σ ε Δx (T∞4 − Ti ,24 ) + qs′′,t =t2 Δx ⎟
Δx Δx
Ti ,2 = Ti ,1 + ⎝ ⎠ Δt
(11)
ρ c th Δx
for j = 2.. ( N − 1)

The temperatures of nodes 1 and N do not change:

T1,2 = Tw (12)
TN ,2 = Tw (13)

"Take a single implicit step"


T[1,2]=T_w "node 1"
duplicate i=2,(N-1)
T[i,2]=T[i,1]+(k*th*(T[i-1,2]+T[i+1,2]-2*T[i,2])/Dx+q``_s(time[2],q``_s_max)*Dx+&
h_bar*Dx*(T_infinity-T[i,2])+sigma#*e*Dx*(T_infinity^4-T[i,2]^4))*Dt/(rho*c*th*Dx)
"internal nodes"
end
T[N,2]=T_w "node N"

The complete set of timesteps is simulated by copying the single step code and pasting it in a
duplicate statement. Change the 2 to j+1 and the 1 to j.

{"Take a single implicit step"


T[1,2]=T_w "node 1"
duplicate i=2,(N-1)
T[i,2]=T[i,1]+(k*th*(T[i-1,2]+T[i+1,2]-2*T[i,2])/Dx+q``_s(time[2],q``_s_max)*Dx+h_bar*Dx*(T_infinity-
T[i,2])+sigma#*e*Dx*(T_infinity^4-T[i,2]^4))*Dt/(rho*c*th*Dx)
"internal nodes"
end
T[N,2]=T_w "node N" }

"Take all of the implicit steps"


duplicate j=1,(M-1)
T[1,j+1]=T_w "node 1"
duplicate i=2,(N-1)
T[i,j+1]=T[i,j]+(k*th*(T[i-1,j+1]+T[i+1,j+1]-2*T[i,j+1])/Dx+q``_s(time[j+1],q``_s_max)*Dx+&
h_bar*Dx*(T_infinity-T[i,j+1])+sigma#*e*Dx*(T_infinity^4-T[i,j+1]^4))*Dt/(rho*c*th*Dx)
"internal nodes"
end
T[N,j+1]=T_w "node N"
end

Figure 2 illustrates the temperature as a function of position at the requested times.


330
t = 300 s
328

326

Temperature (°C)
324
t=0
322

320

318

316

314 t = 600 s

312
0 0.02 0.04 0.06 0.08 0.1 0.12 0.14 0.16
Position (m)
Figure 2: Temperature as a function of position at t = 0, 300 s, and 600 s.

b.) Use your numerical model to compute the rate of heat transfer to the water at each timestep
(per tube per unit length of collector). Plot the rate of heat transfer to the water as a function
of time.

The rate of heat transfer to the water is obtained from an energy balance on node 1:

⎡ k W th Δx Δx Δx
qwater , j = 2 ⎢ ( T2, j − T1, j ) + h W (T∞ − Ti ) + σ ε W (T∞4 − Ti 4 ) + qs′′W ⎤⎥ (14)
⎣ Δx 2 2 2 ⎦

duplicate j=1,M
q_dot_water[j]=2*(k*W*th*(T[2,j]-T[1,j])/Dx+q``_s(time[j],q``_s_max)*W*Dx/2+&
h_bar*W*Dx*(T_infinity-T[1,j])/2+sigma#*e*W*Dx*(T_infinity^4-T[1,j]^4)/2)
"heat transfer to water"
end

Figure 3 illustrates the rate of heat transfer per tube per length to the water as a function of time.
60
Rate of heat transfer to water (W)

40

20

-20

-40

-60
0 100 200 300 400 500 600
Time (s)
Figure 3: Rate of heat transfer to water as a function of time.
Problem 3.9-1
Figure P3.9-1 shows a metal cylinder that is being quenched.

ht = 500 W/m -K
2

T∞ = 10°C
Ro = 5 cm

hs = 100 W/m -K
2
L = 10 cm
T∞ = 10°C
x
r
k = 10 W/m-K
α = 2x10-5 m2/s
Figure P3.9-1: Metal cylinder being quenched.

The metal cylinder has radius Ro = 5 cm and length L = 10 cm. The material has conductivity k
= 10 W/m-K and thermal diffusivity α = 2.0x10-5 m2/s. The cylinder is initially at Tini = 750ºC
when it is sprayed with water at T∞ = 10ºC. The spray directly impacts the top of the cylinder (at
x = L) and therefore the heat transfer coefficient at the top is high, ht = 500 W/m2-K. A lower
heat transfer coefficient is associated with the sides of the cylinder (at r = Ro), hs = 100 W/m2-K.
The bottom surface (at x = 0) is insulated.
a.) Develop a solution by superposition of separation of variables solution.

The inputs are entered in EES:

$UnitSystem SI MASS RAD PA K J


$TABSTOPS 0.2 0.4 0.6 0.8 3.5 in

"Inputs"
R_o=5 [cm]*convert(cm,m) "outer radius of cylinders"
L=10 [cm]*convert(cm,m) "length of cylinders"
k=10 [W/m-K] "conductivity"
T_ini=converttemp(C,K,750 [C]) "initial temperature"
T_infinity=converttemp(C,K,10 [C]) "water temperature"
h_bar_t=500 [W/m^2-K] "heat transfer coefficient on top surface"
h_bar_s=100 [W/m^2-K] "heat transfer coefficient on sides"
alpha=2e-5 [m^2/s] "thermal diffusivity"

The governing partial differential equation, expressed in terms of the temperature difference θ =
T - T∞, is:

∂ ⎛ ∂θ ⎞ ∂ 2θ r ∂θ
⎜r ⎟+r 2 = (1)
∂r ⎝ ∂r ⎠ ∂x α ∂t

and the boundary conditions are:


∂θ
−k = hs θ r = Ro (2)
∂r r = Ro

θ r =0 is finite (3)

∂θ
=0 (4)
∂x x =0

∂θ
−k = ht θ x = L (5)
∂x x= L

The initial condition is:

θt =0 = Tini − T∞ (6)

We assume a product solution:

θ ( x, r , t ) = θ Xt ( x, t )θ Rt ( r , t ) (7)

Equation (7) is substituted into Eq. (1):

∂ ⎛ ∂θ Rt ⎞ ∂ 2θ Xt r ∂θ Xt r ∂θ Rt
θ Xt ⎜ r ⎟ + r θ Rt = θ Rt + θ Xt (8)
∂r ⎝ ∂r ⎠ ∂x 2
α ∂t α ∂t

Equation (8) is divided by the product of θXt θRt and separated into two partial differential
equations:

∂ ⎛ ∂θ Rt ⎞ r ∂θ Rt
⎜r ⎟= (9)
∂r ⎝ ∂r ⎠ α ∂t

∂ 2θ Xt 1 ∂θ Xt
= (10)
∂x 2 α ∂t

Equation (7) is substituted into Eqs. (2) through (5) in order to identify the boundary conditions
for θXt and θRt:

∂θ Rt
−k = hs θ Rtr = Ro (11)
∂r r = Ro

θ Rtr =0 is finite (12)


∂θ Xt
=0 (13)
∂x x =0

∂θ Xt
−k = ht θ x = L (14)
∂x x=L

Finally, Eq. (7) is substituted into Eq. (6) in order to identify the initial conditions for θXt and
θRt:

θ Rtt =0 θ Xtt =0 = Tini − T∞ (15)

which is satisfied if:

θ Rtt =0 = Tini − T∞ (16)

and

θ Xtt =0 = 1 (17)

Equations (9), (11), (12), and (16) correspond exactly to the solution presented in Section 3.5.2
for a cylinder; this solution can be accessed from the cylinder_T function in EES:

x_bar=1 [-] "dimensionless axial position"


x=x_bar*L "axial position"
r_bar=1 [-] "dimensionless radial position"
r=r_bar*R_o "radial position"
time=50 [s] "time"
theta_Rt=cylinder_T(r, time, (T_ini-T_infinity), 0, alpha, k, h_bar_s, R_o) "solution for theta_Rt"

Equations (10), (13), (14), and (17) correspond exactly to the solution presented in Section 3.5.2
for a plane wall; this solution can be accessed from the planewall_T function in EES:

theta_Xt=planewall_T(x, time, 1, 0, alpha, k, h_bar_t, L) "solution for theta_Xt"

The solution is assembled according to Eq. (7):

theta=theta_Rt*theta_Xt "solution for theta"


T=theta+T_infinity "temperature"
T_C=converttemp(K,C,T) "in C"

b.) Plot the highest and lowest temperature within the cylinder as a function of time.

The highest temperature will be at x = 0 and r = 0 while the lowest will be at x = L and r = Ro;
the requested plot is shown in Figure 2.
800

700

600

Temperature (°C)
500

400 x=0, r=0

300

200
x=L, r=R o
100

0
0 50 100 150 200 250 300 350 400 450 500
Time (s)
Figure 2: Highest and lowest temperature as a function of time.

c.) Prepare a contour plot of the temperature distribution within the cylinder at t = 50 s.

Figure 3 illustrates the requested contour plot.


1
151.4
Dimensionless axial position, x/L

193.8
0.8
236.2
278.6
0.6 321
363.4
405.8
0.4
448.2
490.6
0.2 533
575.4

0
0 0.2 0.4 0.6 0.8 1
Dimensionless radial position, r/Ro
Figure 3: Contour plot of the temperature distribution at t = 50 s.
Problem 3.10-1 (3-24 in text)
Figure P3.10-1(a) illustrates a disk brake that is used to bring a piece of rotating machinery to a
smooth stop.

ha = 25 W/m -K
2
pad
Ta = 20°C
q ′′

disk
h jet = 250 W/m -K
2

Ta = 20°C
Figure P3.10-1(a): A disk brake on a rotating machine.

The brake pad engages the disk at its outer edge when the brake is activated. The outer edge and
top surface of the disk (except under the pad) are exposed to air at Ta= 20°C with ha = 25 W/m2-
K. The bottom edge is exposed to air jets in order to control the disk temperature; the air jets
have Ta = 20°C and h jet = 250 W/m2-K. The problem can be modeled as a 2-D, radial problem as
shown in Figure P3.10-1(b).

ha = 25 W/m -K
2

Ta = 20°C q ′′

15 cm
22 cm
ha = 25 W/m -K
2

3 cm Ta = 20°C
ρ = 1000 kg/m3
c = 200 J/kg-K
h jet = 250 W/m -K
2

k = 30 W/m-K
Ta = 20°C
Figure P3.10-1(b): 2-D representation of disk brake

The dimensions of the brake and boundary conditions are shown in Figure P3.10-1(b); the
friction between the disk and the brake causes a spatially uniform heat flux that varies with time
according to:

⎛ ⎛ t [s ] ⎞ 2 ⎞
q ′′ = 200000 ⎡⎣ W/m ⎤⎦ ⎜1 − ⎜⎜
2
⎟ ⎟
⎜ ⎝ 50 [s ] ⎟⎠ ⎟
⎝ ⎠

You may neglect the contribution of the shaft (i.e., assume that the brake is just a disk). The disk
is initially at a uniform temperature of 20°C. The density of the disk material is ρ = 1000 kg/m3,
the conductivity is k = 30 W/m-K, and the specific heat capacity is c = 200 J/kg-K.
a.) Develop a FEHT model that can predict the temperature distribution in the disk as a function
of time during the 50 s that is required for the rotating machine to stop.

The problem is specified as being in cylindrical coordinates, transient with temperature in


Celsius (Setup menu, Figure 3).

Figure 3: Setup menu.

The geometry is setup using a 1 cm x 1 cm grid where 1 cm of screen size = 1 cm of drawing


space. Note that two outlines are required in order to specify the two discrete boundary
conditions along the top surface (Fig. 4). Roughly place the corners of the outline (Fig. 4(a)) and
then double-click on each corner to place it more precisely (Fig. 4(b)).

(a)

(b)
Figure 4: Outlines (a) with nodes roughly placed, and (b) with nodes precisely positioned.

Specify the material properties within each of the outlines (select both outlines using the shift
key and then select Material Properties from the Specify menu, Fig. 5).
Figure 5: Specify material properties

Specify the boundary conditions associated with convection; select the two surfaces exposed to
ambient air and the select Boundary Conditions from the Specify menu (Fig. 6).

Figure 6: Specify convection boundary conditions.

Repeat for the lower surface exposed to the air jet. Finally, select the surface exposed to the
frictional heating and specify the boundary condition as a function of time (Fig. 7).
Figure 7: Specify frictional heating boundary condition.

Setup a crude mesh by selecting Element Lines from the Draw menu; remember that all elements
must be triangular and it is best if they are about the same size. Figure 8 illustrates one
possibility.

Figure 8: A crude mesh for the problem.

Specify the initial temperature by selecting both outlines and then selecting Initial Temperature
from the Specify menu (Fig. 9); make sure to indicate that all nodes in all materials should be set.

Figure 9: Specify initial temperature.

You can check to make sure that the problem is fully specified by selecting Check from the Run
menu. The problem should say that there are no errors found and list the number of elements.
The critical time step should also be provided.

Run the problem by selecting Calculate from the Run menu. The Transient Calculation Setup
dialog will require that you specify the time span to be simulated and the solution technique.
Simulate the entire stopping process (0 to 50 s) and use a Crank-Nicolson technique with a 1
second time step (Fig. 10).

Figure 10: Transient Calculation Setup dialog window.

Select OK and the calculations will proceed; at the conclusion of the calculations a message will
appear specifying the number of unknown temperature and other information. Select Continue
and view the results by selecting Temperature Contours from the View menu. Select from start
to stop rather than a particular time and watch the animation of the temperature distribution (Fig.
11).

Figure 11: Temperature Contour animation setup

b.) Plot the maximum temperature in the disk as a function of time for two values of the number
of nodes in order to demonstrate that your mesh is sufficiently refined. You will need to
generate two plots and the comparison will be qualitative.

You should refine your mesh a few times (Fig. 12 shows the mesh refined twice, select Reduce
Mesh from the Draw menu twice) so that you are close to being adequately refined.
Figure 12: Mesh refined twice

Select Calculate and then select the node at the upper right corner of the model (the hottest
node). The node should blink; select Temperature vs Time from the View menu and the number
of the selected node (5 for my model, Fig. 13) will appear in the Nodes box. You can select
additional nodes as you like.

Figure 13: Temperatures vs Time Plot Setup

Select OK to generate a temperature vs time history for the mesh with 112 nodes, Fig. 14.

Figure 14: Hottest temperature in the disk as a function of time.


Repeat the process after refining the mesh to generate Figure 15.

Figure 15: Hottest temperature in the disk as a function of time for a refined mesh.

The two solutions agree relatively well indicating that the result is insensitive to grid size.
Notice the time step to time step “wiggles” have gotten worse as the grid is resolved; this effect
can be mitigated by choosing a smaller time step; for example, Fig. 16 illustrates the temperature
as a function of time with a time step of 0.01 s.

Figure 15: Hottest temperature in the disk as a function of time for a refined mesh and a 0.01 s time
step.
c.) Prepare a contour plot showing the temperature distribution at t = 10 s, t = 25 s, and t= 50 s.
You may also want to animate your temperature contours by selecting Temperature Contours
from the View menu and selecting From start to stop.

To generate these plots, select Temperature Contours from the View menu and the At time=10 s
(for example) as shown in Fig. 16.

Figure 16: Temperature Contour dialog window.

Figure 17 illustrates the contours at the indicated times; notice that the same scale is used for all
three times so that an accurate comparison can be made.

(a)

(b)
(c)

(d)
Figure 17: Temperature contours at (a) 10 s, (b) 25 s, and (c) 50 s with the color scale shown in (d).

d.) Plot the temperature on the lower surface (the surface exposed to the jets of air) at various
locations as a function of time. Explain the shape of the plot – does the result make physical
sense to you based on any time constants that you can compute?

Figure 18(b) illustrates the temperature as a function of time at the nodes shown in Fig. 18(a).

(a)

(b)
Figure 18: Temperature as a function of time (b) for the nodes shown in (a).
The conduction process within the disk is governed by a diffusion. The nodes along the bottom
surface are approximately s = 3 cm apart; the diffusive time constant associated with heat
moving from one node to the next is:

s2
τ= (1)

where α is the thermal diffusivity of the disk material. Equation (1) indicates that there should
be a time delay of approximately 2 s which is approximately consistent with the lag between
peaks.
Problem 3.4-1 (3-12 in text): Laser Target
A disk shaped piece of material is used as the target of a laser, as shown in Figure P3.4-1. The
laser target is D = 5.0 mm in diameter and b = 2.5 mm thick. The target is made of a material
with ρ = 2330 kg/m3, k = 500 W/m-K, and c = 400 J/kg-K. The target is mounted on a chuck
with a constant temperature Tc = 20°C. The interface between the target and the chuck is
characterized by a contact resistance, Rc′′ = 1x10-4 K-m2/W. The target is initially in thermal
equilibrium with the chuck. You may neglect radiation and convection from the laser target.
′′ = At 2 exp ( −t / t pulse ) where A = 1x107 W/m2-s2 and
The laser flux is pulsed according to: qlaser
tpulse = 0.10 s.

laser target
ρ = 2330 kg/m3
k = 500 W/m-K chuck, Tc = 20°C
c = 400 J/kg-K

′′
laser flux, qlaser

D = 5 mm
b = 2.5 mm
contact resistance,
Rc′′ = 1x10 K-m /W
-4 2

Figure P3.4-1: Laser target

a.) Is a lumped capacitance model of the laser target appropriate? Justify your answer.

The inputs are entered in EES:

$UnitSystem SI MASS RAD PA K J


$TABSTOPS 0.2 0.4 0.6 0.8 3.5 in

"Inputs"
b=2.5 [mm]*convert(mm,m) "thickness of target"
D=5.0 [mm]*convert(mm,m) "diameter of target"
rho=2330 [kg/m^3] "density"
k=500 [W/m-K] "conductivity"
c=400 [J/kg-K] "specific heat capacity"
Rc=1e-4 [m^2-K/W] "contact resistance"
A=1e7 [W/m^2-s^2]
t_pulse=0.1 [s]
T_c=converttemp(C,K,20[C]) "chuck temperature"

The appropriate Biot number for this problem is:

Rcond 4 b π D2 b
Bi = = = (1)
Rcontact k π D 4 Rc′′ k Rc′′
2
Bi=b/(k*Rc) "Biot number"

which leads to Bi = 0.05.

b.) Use Laplace transforms to determine the temperature of the laser target as a function of time.
Prepare a plot of the temperature of the laser target as a function of time; overlay on this plot
the laser heat flux as a function of time (on a secondary y-axis).

An energy balance on the laser target leads to:

π D2 π D2 π D2 dT
′′
qlaser =
′′
(T − Tc ) + bρ c (2)
4 4 Rc 4 dt

The time constant is defined as:

τ = Rc′′ bρ c (3)

tau=Rc*b*rho*c "time constant"

and substituted into Eq. (2):

′′
dT T Tc qlaser
+ = + Rc′′ (4)
dt τ τ τ

Substituting the laser flux equation into Eq. (4) leads to:

dT T Tc Rc′′ A 2 ⎛ t ⎞
+ = + t exp ⎜ −
⎜ t pulse ⎟⎟
(5)
dt τ τ τ ⎝ ⎠

This governing differential equation is entered in Maple:

> restart;
> with(inttrans):
> GDE:=diff(T(time),time)+T(time)/tau=T_c/tau+A*time^2*exp(-time/t_pulse)*Rc/tau;
⎛⎜ − time ⎞⎟
⎜⎝ t_pulse ⎟⎠
T( time ) T_c A time 2 e
GDE := ⎜⎜⎛ T( time ) ⎟⎟⎞ +
d Rc
= +
⎝ dtime ⎠ τ τ τ

and transformed from the t to the s domain:

> GDEs:=laplace(GDE,time,s);
laplace( T( time ), time, s )
GDEs := s laplace( T( time ), time, s ) − T( 0 ) + =
τ
T_c 2 A Rc
+
τs 3
τ ⎛⎜⎜ s +
1 ⎞
⎟⎟
⎝ t_pulse ⎠

The initial condition (Tt=0 = Tc) is substituted in place of T(0) and the variable T(s) is substituted
in place of laplace(T(time),time,s):

> GDEs:=subs(laplace(T(time),time,s)=T(s),GDEs);
T( s ) T_c 2 A Rc
GDEs := s T( s ) − T( 0 ) + = +
τ τs 3
τ ⎜⎜⎛ s +
1 ⎞

⎝ t_pulse ⎟⎠
> GDEs:=subs(T(0)=T_c,GDEs);
T( s ) T_c 2 A Rc
GDEs := s T( s ) − T_c + = +
τ τs 3
τ ⎜⎜⎛ s +
1 ⎞
⎟⎟
⎝ t_pulse ⎠

The algebraic equation is solved in the s domain:

> Ts:=solve(GDEs,T(s));
Ts := ( T_c τ s4 t_pulse 3 + 3 T_c τ s3 t_pulse 2 + 3 T_c τ s2 t_pulse + T_c τ s
+ T_c s3 t_pulse 3 + 3 T_c s2 t_pulse 2 + 3 T_c s t_pulse + T_c + 2 A Rc t_pulse 3 s ) (
s ( s τ t_pulse + 3 s τ t_pulse + 3 s τ t_pulse + s τ + s t_pulse + 3 s t_pulse
4 3 3 2 2 3 3 2 2

+ 3 s t_pulse + 1 ) )

and transformed back to the time domain:

> Tt:=invlaplace(Ts,s,time);
⎛⎜ ⎛⎜ − time ⎞⎟
⎜ τ ⎟⎠
⎜ 2 2 ⎝
Tt := T_c + ⎝ 2 t_pulse τ e + ( −time 2 t_pulse 2 + 2 time 2 τ t_pulse − time 2 τ2
⎛⎜ − time ⎞⎟ ⎞
⎜ t_pulse ⎟⎠ ⎟
+ 2 t_pulse 2 τ time − 2 t_pulse τ2 time − 2 t_pulse 2 τ2 ) e ⎝ ⎟⎠ t_pulse Rc A
( −t_pulse + τ ) 3

where it can be copied and pasted into EES:

Tt := T_c+(2*t_pulse^2*tau^2*exp(-time/tau)+(-time^2*t_pulse^2+2*time^2*tau*t_pulse-
time^2*tau^2+2*t_pulse^2*tau*time-2*t_pulse*tau^2*time-2*t_pulse^2*tau^2)*exp(-time/t_pulse))/(-
t_pulse+tau)^3*t_pulse*Rc*A

The expression is modified slightly in order to be compatible with EES:


T = T_c+(2*t_pulse^2*tau^2*exp(-time/tau)+(-time^2*t_pulse^2+2*time^2*tau*t_pulse-&
time^2*tau^2+2*t_pulse^2*tau*time-2*t_pulse*tau^2*time-2*t_pulse^2*tau^2)*&
exp(-time/t_pulse))/(-t_pulse+tau)^3*t_pulse*Rc*A

The heat flux from the laser is also calculated:

q_flux=A*time^2*exp(-time/t_pulse)

A parametric table is created that includes the variables time, q_flux, and T. Figure 2 illustrates
the temperature of the laser target and the laser heat flux as a function of time.
296.5 6x104

296
5x104

295.5
Laser target temperature (K)

4x104

Laser flux (W/m2)


295 target temperature
3x104
294.5

2x104
294
laser flux

1x104
293.5

293 0
0 0.2 0.4 0.6 0.8 1 1.2 1.4 1.6 1.8 2
Time (s)
Figure 2: Laser target temperature and laser flux as a function of time
PROBLEM 3.1-5 (3-2 in text): Cooldown of an instrument
An instrument on a spacecraft must be cooled to cryogenic temperatures in order to function.
The instrument has mass M = 0.05 kg and specific heat capacity c = 300 J/kg-K. The surface
area of the instrument is As = 0.02 m2 and the emissivity of its surface is ε = 0.35. The
instrument is exposed to a radiative heat transfer from surroundings at Tsur = 300 K. It is
connected to a cryocooler that can provide qcooler = 5 W. The instrument is exposed to a solar
flux that oscillates according to: qs′′ = q s′′ + Δqs′′ sin (ω t ) where qs′′ = 100 W/m2, Δq s′′ = 100 W/m2,
and ω = 0.02094 rad/s. The initial temperature of the instrument is Tini = 300 K. Assume that the
instrument can be treated as a lumped capacitance.
a.) Develop an analytical model of the cool-down process and implement your model in EES.

The inputs are entered in EES:

$UnitSystem SI MASS RAD PA K J


$TABSTOPS 0.2 0.4 0.6 0.8 3.5 in

"Inputs"
A_s=0.02 [m^2] "surface area"
Mass=0.05 [kg] "mass"
c = 300 [J/kg-K] "specific heat capacity"
e = 0.35 [-] "emissivity"
T_sur = 300 [K] "temperature of surroundings"
T_ini=300 [K] "initial temperature of instrument"
qfs_bar=100 [W/m^2] "average solar flux"
Dqfs=100 [W/m^2] "variation in the solar flux"
omega=0.02094 [rad/s] "angular frequency of solar flux variation"
q_cooler=5 [W] "cooler power"

An energy balance on the instrument leads to:

dU
qs′′ As + qrad = qcooler + (1)
dt

where qrad is the radiation heat transfer to the instrument. Equation (1) can be expressed as:

⎡ q s′′ + Δqs′′ sin (ω t ) ⎤ As +


(Tsur − T ) = q +M c
dT
(2)
⎣ ⎦ Rrad
cooler
dt

where Rrad is the resistance to radiation based on an average temperature of T = 200 K (this
value can be adjusted based on the results).

1
Rrad = (3)
4 As ε σ T 3

T_bar=200 [K] " estimate of the avg. temp. to use for the radiation resistance"
R_rad=1/(4*A_s*e*sigma#*T_bar^3) "radiation resistance"
Rearranging Eq. (2) leads to:

dT
+
T T
= sur +
q s′′ As − qcooler (
Δq ′′ A
+ s s sin (ω t )
) (4)
dt τ lumped τ lumped Mc Mc

where the lumped capacitance time constant is:

τ lumped = Rrad M c (5)

tau_lumped=R_rad*Mass*c "lumped capacitance time constant"

The solution to Eq. (4) is split into a homogeneous and particular component:

T = Th + Tp (6)

Equation (6) is substituted into Eq. (4):

dTh T
+ h +
dTp
+
Tp T
= sur +
qs′′ As − qcooler Δq ′′ A(
+ s s sin (ω t )
) (7)
dt τ lumped dt τ lumped τ lumped Mc Mc



homogeneous ODE particular ODE

The solution to the homogeneous ordinary differential equation is:

⎛ t ⎞
Th = C1 exp ⎜ − ⎟⎟ (8)
⎜ τ lumped
⎝ ⎠

The solution to the particular ordinary differential equation is more difficult.

dTp
+
Tp
=
Tsur
+
( q′′ A − q ) + Δq′′ A sin (ω t )
s s cooler s s
(9)
dt τ lumped τ lumped Mc Mc

The right side of Eq. (9) includes constants and a sine term. Therefore, it is reasonable to expect
that the solution will include these terms and their derivatives (i.e., 0 and a cosine term):

Tp = C2 + C3 sin (ω t ) + C4 cos (ω t ) (10)

Substituting Eq. (10) into Eq. (9) leads to:


C2 C3 C4
C3 ω cos (ω t ) − C4 ω sin (ω t ) + + sin (ω t ) + cos (ω t ) =
τ lumped τ lumped τ lumped
Tsur
+
( q′′ A − q ) + Δq′′ A sin (ω t )
s s cooler s s
(11)

τ lumped Mc Mc

In order for Eq. (11) to be valid at arbitary time, it is necessary that all of the constant, sine, and
cosine terms separately add to zero. The constant terms in Eq. (11) lead to:

C2
=
Tsur
+
( q′′ A − q )
s s cooler
(12)
τ lumped τ lumped Mc

The coefficients multiplying the sine terms in Eq. (11) provide:

C3 Δqs′′ As
−C 4 ω + = (13)
τ lumped Mc

The coefficients multiplying the cosine terms in Eq. (11) provide:

C4
C3 ω + =0 (14)
τ lumped

Equations (12) through (14) lead to three equations in three unknowns that can be solved to
provide the particular solution:

C_2/tau_lumped=T_sur/tau_lumped+(qfs_bar*A_s-q_cooler)/(Mass*c) "equation 1 for particular solution"


-C_4*omega+C_3/tau_lumped=Dqfs*A_s/(Mass*c) "equation 2 for particular solution"
C_3*omega+C_4/tau_lumped=0 "equation 3 for particular solution"

The solution is obtained by substituting Eqs. (10) and (8) into Eq. (6):

⎛ t ⎞
T = C1 exp ⎜ −
⎜ τ lumped ⎟⎟ + C2 + C3 sin (ω t ) + C4 cos (ω t ) (15)
⎝ ⎠

The undetermined constant in the homogeneous solution, C1, can be obtained by enforcing the
initial condition:

Tt =0 = Tini (16)

Substituting Eq. (15) into Eq. (16) leads to:

Tini = C1 + C2 + C4 (17)
T_ini=C_1+C_2+C_4 "initial condition"

The solution, Eq. (15), is programmed in EES:

time=0 [s] "time"


T=C_1*exp(-time/tau_lumped)+C_2+C_3*sin(omega*time)+C_4*cos(omega*time) "solution"

b.) Plot the temperature as a function of time.

Figure P3.1-5 illustrates the temperature of the instrument as a function of time.


300

250
Temperature (K)

200

150

100

50
0 1000 2000 3000 4000 5000
Time (s)
Figure P3.1-5: Temperature of the instrument as a function of time.
Problem 3.1-6 (3-3 in text): Particle ablation
One technique for detecting chemical threats uses a laser to ablate small particles so that they can
subsequently be analyzed using ion mobility spectroscopy. The laser pulse provides energy to a
⎡ ( t − t )2 ⎤
particle according to: qlaser = qmax exp ⎢ − ⎥ where qmax = 0.22 W is the maximum value of
p

⎢ 2 td ⎥
2

⎣ ⎦
the laser power, tp = 2 μs is the time at which the peak laser power occurs, and td = 0.5 μs is a
measure of the duration of the pulse. The particle has radius rp = 5 μm and has properties c =
1500 J/kg-K, k = 2.0 W/m-K, and ρ = 800 kg/m3. The particle is surrounded by air at T∞ = 20°C.
The heat transfer coefficient is h = 60000 W/m2-K. The particle is initially at T∞.
a.) Is a lumped capacitance model of the particle justified?

The inputs are entered in EES:

$UnitSystem SI MASS RAD PA K J


$TABSTOPS 0.2 0.4 0.6 0.8 3.5 in

"Inputs"
r_p=5 [micron]*convert(micron,m) "particle radius"
c=1500 [J/kg-K] "specific heat capacity"
k=2 [W/m-K] "conductivity"
rho=800 [kg/m^3] "density"
h_bar=60000 [W/m^2-K] "heat transfer coefficient"
t_p=2e-6 [s] "time of the pulse"
t_d=0.5e-6 [s] "duration of pulse"
q_dot_max=0.22 [W] "maximum laser power"
T_infinity=converttemp(C,K,20[C]) "air temperature"

The volume of the particle is:

4
V = π rp3 (1)
3

The surface area of the particle is:

As = 4 π rp2 (2)

The Biot number for the particle is:

hV
Bi = (3)
As k

V=4*pi*r_p^3/3 "volume"
A_s=4*pi*r_p^2 "surface area"
Bi=h_bar*V/(A_s*k) "Biot number"

which leads to Bi = 0.05, justifying a lumped capacitance model.


b.) Assume that your answer to (a) is yes; develop an analytical model of the particle using
Maple and EES. Plot the temperature of the particle as a function of time. Overlay on your
plot (on a secondary axis) the laser power.

An energy balance on the particle leads to:

dT
qlaser = h As (T − T∞ ) + C (4)
dt

where

C =V ρ c (5)

C_tot=rho*V*c "total heat capacity"

Equation (4) is rearranged:

dT T T∞ qlaser
+ = + (6)
dt τ τ C

where τ is the lumped capacitance time constant:

C
τ= (7)
h As

tau=C_tot/(h_bar*A_s) "lumped capacitance time constant"

The ordinary differential is entered in Maple:

> restart;
> q_dot_laser:=q_dot_max*exp(-(t-t_p)^2/(2*t_d^2));
⎛ ( t − t_p )2 ⎞
⎜− ⎟
⎜ ⎟
⎜⎜ 2 t_d 2 ⎟⎟⎠

q_dot_laser := q_dot_max e
> ODE:=diff(T(t),t)+T(t)/tau=T_infinity/tau+q_dot_laser/C_tot;
⎛ ( t − t_p )2 ⎞
⎜− ⎟
⎜ ⎟
⎜⎜ 2 t_d 2 ⎟⎟⎠

d T( t ) T_infinity q_dot_max e
ODE := ⎜⎜⎛ T( t ) ⎞⎟⎟ + = +
⎝ dt ⎠ τ τ C_tot

The ordinary differential equation is solved:

> Ts:=dsolve(ODE);

⎛− t ⎞
⎜⎜ ⎟⎟ ⎜
⎝ τ⎠ 1⎜
Ts := T( t ) = e _C1 + ⎜⎜ 2 T_infinity C_tot
2⎝
⎛ 2 τ t − t_d 2 − 2 τ t_p ⎞
⎜− ⎟⎞
⎜ ⎟⎟
⎛ 2 ( −t_d + τ t − τ t_p ) ⎞ ⎜⎜ ⎟⎟
⎠⎟
2 2
⎝ 2 τ
+ q_dot_max π 2 t_d erf⎜⎜ ⎟⎟ e ⎟⎟/C_tot
⎝ 2 t_d τ ⎠ ⎠

The constant of integration is evaluated using the initial condition:

Tt =0 = T∞ (8)

Equation (8) is entered in Maple:

> rhs(eval(Ts,t=0))=T_infinity;
1
_C1 +
2
⎛ −t_d 2 − 2 τ t_p ⎞
⎜− ⎟
⎜ ⎟
⎛ 2 ( −t_d − τ t_p ) ⎞ ⎜⎜ ⎟⎟
2
⎝ 2 τ2 ⎠
2 T_infinity C_tot + q_dot_max π 2 t_d erf⎜⎜ ⎟⎟ e
⎝ 2 t_d τ ⎠
C_tot
= T_infinity

The result is copied and pasted into EES; the variable _C1 is changed to C_1:

"initial condition"
C_1+1/2*(2*T_infinity*C_tot+q_dot_max*Pi^(1/2)*2^(1/2)*t_d*erf(1/2*2^(1/2)*(-t_d^2-tau*t_p)/tau/t_d)*&
exp(-1/2*(-t_d^2-2*tau*t_p)/tau^2))/C_tot = T_infinity

The solution is copied from Maple and pasted into EES:

"solution"
time=0 [s]
T = exp(-1/tau*time)*C_1+1/2*(2*T_infinity*C_tot+q_dot_max*Pi^(1/2)*2^(1/2)*t_d*erf(1/2*2^(1/2)*&
(-t_d^2+tau*time-tau*t_p)/tau/t_d)*exp(-1/2*(2*tau*time-t_d^2-2*tau*t_p)/tau^2))/C_tot
T_C=converttemp(K,C,T)

The laser power is also programmed in EES:

q_dot=q_dot_max*exp(-(time-t_p)^2/(2*t_d^2)) "laser power"

The plot requested in the problem statement is shown in Figure P3.1-6.


800 0.225

temperature 0.2
700
0.175

Laser power (W)


Temperature (K)
600 0.15

0.125
500
0.1

400 0.075
laser power
0.05
300
0.025

200 0
0.0x100 2.0x10-6 4.0x10-6 6.0x10-6 8.0x10-6 1.0x10-5
Time (s)
Figure P3.1-6: Temperature and laser power as a function of time.
Problem 3.2-4 (3-4 in text): Cooling down an instrument
Reconsider Problem 3.1-5 (3-2 in text) using a numerical model. The cooling power of the
cryocooler is not constant but is a function of temperature:
⎧ ⎡W⎤ 2 ⎡ W ⎤
⎪−4.995 [ W ] + 0.1013 T ⎢ ⎥ − 0.0001974 T ⎢ 2 ⎥ if T > 55.26 K
qcooler = ⎨ ⎣K⎦ ⎣K ⎦
⎪0 if T < 55.26 K

where T is the temperature of the instrument.


a.) Develop a numerical model in EES using Heun's method. Plot the temperature of the
instrument as a function of time for 2000 s after the cryocooler is activated.

The inputs are entered in EES:

$UnitSystem SI MASS RAD PA K J


$TABSTOPS 0.2 0.4 0.6 0.8 3.5 in

"Inputs"
A_s=0.02 [m^2] "surface area"
Mass=0.05 [kg] "mass"
c = 300 [J/kg-K] "specific heat capacity"
e = 0.35 [-] "emissivity"
T_sur = 300 [K] "temperature of surroundings"
T_ini=300 [K] "initial temperature of instrument"

Functions are defined for the solar flux and load curve of the cryocooler:

function q_dot_load(T)
q_dot_load=-4.995 [W] + 0.1013 [W/K]*T - 0.0001974 [W/K^2]*T^2
end

function qf_dot_solar(time)
qf_dot_solar=100 [W/m^2]+100[W/m^2]*sin(0.02094 [rad/s]*time)
end

The state equation for the system is derived with an energy balance on the instrument:

As qs′′ ( t ) +
(Tsur − T ) = q dT
cooler (T ) + M c (1)
Rrad dt

where

1
Rrad = (2)
As ε σ (T + Tsur
22
) (T + Tsur )
Rearranging Eq. (1) leads to:
dT 1 ⎡
⎢ As qs′′ ( t ) +
(Tsur − T ) − q T ⎤⎥
= cooler ( ) (3)
dt M c ⎢ Rrad ⎥⎦

The computational domain is divided into M steps; the time associated with each time step and
duration of the time step are:

tsim
Δt = (4)
( M − 1)

t j = tsim
( j − 1) for j = 1..M (5)
( M − 1)
t_sim=2000 [s] "simulation time"
M=101 [-] "number of steps"
DELTAtime=t_sim/(M-1) "duration of time step"
duplicate j=1,M
time[j]=(j-1)*t_sim/(M-1)
end

The initial temperature is set:

T[1]=T_ini

For Heun's method, the predictor step computes the temperature rate of change at the beginning
of the time step:

1 ⎡
⎢ As q s′′ ( t j ) +
(Tsur − T j ) − q T ⎤⎥ for j = 1.. M − 1
cooler ( j )
dT
= ( ) (6)
dt Mc⎢ Rrad , j ⎥
j
⎣ ⎦

where

1
Rrad , j = (7)
As ε σ (T + Tsur
j
2 2
) (T j + Tsur )
duplicate j=1,(M-1)
R_rad[j]=1/(A_s*e*sigma#*(T_sur^2+T[j]^2)*(T_sur+T[j]))
dTdt[j]=(qf_dot_solar(time[j])*A_s+(T_sur-T[j])/R_rad[j]-q_dot_load(T[j]))/(Mass*c)

The predictor step is:

dT
Tˆj +1 = T j + Δt for j = 1.. ( M − 1) (8)
dt j
T_hat[j+1]=T[j]+dTdt[j]*DELTAtime "predictor step"

The corrector step computes the temperature rate of change at the end of the time step:

m 1 ⎡⎢ Tsur − Tˆj +1 ( ) ⎤
dT
dt
=
Mc⎢
As qs′′ ( t j +1 ) +
Rˆ rad , j +1
− qcooler Tˆj +1 ( ) ⎥ for j = 1.. ( M − 1)

(9)
j +1 ⎣ ⎦

where

1
Rˆ rad , j +1 = (10)
(
As ε σ Tˆ 2
j +1 + Tsur
2
)(
Tˆj +1 + Tsur )
R_rad_hat[j+1]=1/(A_s*e*sigma#*(T_sur^2+T_hat[j+1]^2)*(T_sur+T_hat[j+1]))
dTdt_hat[j+1]=(qf_dot_solar(time[j+1])*A_s+(T_sur-T_hat[j+1])/R_rad_hat[j+1]-...
q_dot_load(T_hat[j+1]))/(Mass*c)

The corrector step is:

⎛ dT m
dT ⎞ Δt
T j +1 = T j + ⎜ + ⎟ for j = 1.. ( M − 1) (11)
⎜ dt j dt ⎟ 2
⎝ j +1 ⎠

T[j+1]=T[j]+(dTdt[j]+dTdt_hat[j+1])*DELTAtime/2 "corrector step"


end

Figure P3.2-4-2 illustrates the temperature as a function of time.


300

280

260

240
Temperature (K)

220

200

180

160

140

120
0 250 500 750 1000 1250 1500 1750 2000
Time (s)
Figure P3.2-4-2: Temperature as a function of time, predicted using Heun's method.

b.) Verify that your model from (a) limits to the analytical solution developed in Problem 3.1-5
(3-2) in the limit that the cryocooler power is constant and radiation is treated using a
constant, approximate radiation resistance. Overlay on the same plot the temperature of the
instrument as a function of time predicted by the analytical and numerical models.
In order to reach the limiting conditions used in Problem 3.1-5, the cryocooler load curve is
modified:

function q_dot_load(T)
{q_dot_load=-4.995 [W] + 0.1013 [W/K]*T - 0.0001974 [W/K^2]*T^2}
q_dot_load = 5 [W]
end

and the approximate resistance to radiation is computed:

T_bar=200 [K] "approximate average temperature"


R_rad_app=1/(4*e*sigma#*A_s*T_bar^3) "approximate radiation resistance"

and used in place of the actual resistance:

T[1]=T_ini
duplicate j=1,(M-1)
R_rad[j]=R_rad_app{1/(A_s*e*sigma#*(T_sur^2+T[j]^2)*(T_sur+T[j]))}
dTdt[j]=(qf_dot_solar(time[j])*A_s+(T_sur-T[j])/R_rad[j]-q_dot_load(T[j]))/(Mass*c)
T_hat[j+1]=T[j]+dTdt[j]*DELTAtime
R_rad_hat[j+1]=R_rad_app{1/(A_s*e*sigma#*(T_sur^2+T_hat[j+1]^2)*(T_sur+T_hat[j+1]))}
dTdt_hat[j+1]=(qf_dot_solar(time[j+1])*A_s+(T_sur-T_hat[j+1])/R_rad_hat[j+1]-...
q_dot_load(T_hat[j+1]))/(Mass*c)
T[j+1]=T[j]+dTdt_hat[j+1]*DELTAtime
end

Figure P3.2-4-3 illustrates the numerical and analytical results and shows nearly exact
agreement.
300
analytical solution from P3.1-5
numerical solution
250
Temperature (K)

200

150

100

50
0 1000 2000 3000 4000 5000
Time (s)
Figure P3.2-4-3: Temperature as a function of time predicted by the analytical model from P3.1-5 and the
numerical model (1 of every 10 timesteps are shown) under the limiting conditions associated with P3.1-5.

c.) Develop a numerical model in EES using the Integral command. Plot the temperature of the
instrument as a function of time for 2000 s after the cryocooler is activated.

The code from (b) is commented out. The time rate of change, Eq. (3), is evaluated:

dTdt=(qf_dot_solar(time)*A_s+(T_sur-T)/R_rad-q_dot_load(T))/(Mass*c)
R_rad=1/(A_s*e*sigma#*(T_sur^2+T^2)*(T_sur+T))

The Integral command is used:

T=T_ini+Integral(dTdt,time,0,t_sim)

The results are stored in an integral table:

$IntegralTable time:10,T

Figure P3.2-4-3 illustrates the temperature as a function of time predicted using EES' Integral
command.
300

280

260

240
Temperature (K)

220

200

180

160

140

120
0 250 500 750 1000 1250 1500 1750 2000
Time (s)
Figure P3.2-4-3: Temperature as a function of time, predicted using EES' Integral command.

d.) Develop a numerical model in MATLAB using the ode solver. Plot the temperature of the
instrument as a function of time for 2000 s after the cryocooler is activated.

A function dTdt_P3p2d4 is created to return the value of the temperature rate of change given
time, the temperature, and the other required input properties.

function[dTdt]=dTdt_P3p2d4(time,T,A_s,Mass,c,e,T_sur)
sigma=5.67e-8; %Stefan-Boltzman constant (W/m^2-K^4]
dTdt=(qf_dot_solar(time)*A_s+sigma*e*A_s*(T_sur^4-T^4)...
-q_dot_load(T))/(Mass*c);
end

The functions to return the cryocooler load and the solar flux are added to the file dTdt_P3p2d4 as
sub-functions:

function[q_dot]=q_dot_load(T)
q_dot=-4.995+0.1013*T-0.0001974*T^2; %heat load
end

function[qf]=qf_dot_solar(time)
qf=100+100*sin(0.02094*time); %solar flux
end

The integration is carried out from a script named P3p2d4:


clear all;
A_s=0.02; %surface area (m^2)
Mass=0.05; %mass (kg)
c=300; %specific heat capacity (J/kg-K)
e=0.35; %emissivity (-)
T_sur=300; %temperature of surroundings (K)
T_ini=300; %initial temperature of instrument (K)
t_sim=2000; %simulation time (s)

[time,T]=ode45(@(time,T) dTdt_P3p2d4(time,T,A_s,Mass,c,e,T_sur),...
[0, t_sim],T_ini);

Running P3p2d4 from the command line puts the variables time and T into the command space
where they can be manipulated and plotted. Figure P3.2-4-4 illustrates the temperature as a
function of time predicted using MATLAB's ode45 command.

300
Temperature (K)

250

200

150

100
0 500 1000 1500 2000
Time (s)

Figure P3.2-4-3: Temperature as a function of time, predicted using MATLAB's ode45 command.
PROBLEM 3.2-5 (3-5 in text): Particle ablation (revisited)
Reconsider Problem 3.1-6 (3-3 in text).
a.) Develop a numerical model of the particle using the Euler technique implemented in either
EES of MATLAB. Plot the temperature as a function of time and compare your answer with
the analytical solution from Problem 3.1-6 (3-3).

The inputs are entered in EES:

$UnitSystem SI MASS RAD PA K J


$TABSTOPS 0.2 0.4 0.6 0.8 3.5 in

"Inputs"
r_p=5 [micron]*convert(micron,m) "particle radius"
c=1500 [J/kg-K] "specific heat capacity"
rho=800 [kg/m^3] "density"
h_bar=60000 [W/m^2-K] "heat transfer coefficient"
t_p=2e-6 [s] "time of the pulse"
t_d=0.5e-6 [s] "duration of pulse"
q_dot_max=0.22 [W] "maximum laser power"
T_infinity=converttemp(C,K,20[C]) "air temperature"

A function is defined that returns the laser power:

function q_dot_laser(time,t_p,t_d,q_dot_max)
"Inputs:
time - time relative to start of process (s)
t_p - time of laser pulse peak relative to start of process (s)
t_d - pulse duration (s)
q_dot_max - maximum power (W)

Output:
q_dot_laser - instantaneous laser power (W)"
q_dot_laser=q_dot_max*exp(-(time-t_p)^2/(2*t_d^2))
end

The volume of the particle is:

4
V = π rp3 (1)
3

The surface area of the particle is:

As = 4 π rp2 (2)

An energy balance on the particle leads to:

dT
qlaser = h As (T − T∞ ) + C (3)
dt
where

C =V ρ c (4)

C_tot=rho*V*c "total heat capacity"

Equation (3) is rearranged:

dT T T∞ qlaser
+ = + (5)
dt τ τ C

where τ is the lumped capacitance time constant:

C
τ= (6)
h As

tau=C_tot/(h_bar*A_s) "lumped capacitance time constant"

The state equation for this problem is obtained by rearranging Eq. (5):

dT (T∞ − T ) qlaser
= + (7)
dt τ C

where qlaser is a function of time. The simulation time (tsim) is divided into M steps of duration
Δt according to:

tsim
Δt = (8)
( M − 1)

tj =
( j − 1) t (9)
( M − 1) sim
t_sim=10e-6 [s] "total simulation time"
M=21 [-] "number of time steps"
Dt=t_sim/(M-1) "timestep duration"
duplicate j=1,M
time[j]=(j-1)*t_sim/(M-1) "time"
end

The initial temperature is set:

T1 = T∞ (10)

T[1]=T_infinity "initial temperature"


The first step is taken using the Euler technique:

dT
T2 = T1 + Δt (11)
dt t = t1 ,T =T1

Substituting Eq. (7) into Eq. (11):

⎡ (T − T ) qlaser ,t =t1 ⎤
T2 = T1 + ⎢ ∞ 1 + ⎥ Δt (12)
⎣ τ C ⎦

T[1]=T_infinity "initial temperature"


T[2]=T[1]+((T_infinity-T[1])/tau+q_dot_laser(time[1],t_p,t_d,q_dot_max)/C_tot)*Dt "1st Euler step"

All of the time steps are simulated by placing Eq. (12) within a duplicate loop and changing the
subscript 1 to j and 2 to j+1:

{T[2]=T[1]+((T_infinity-T[1])/tau+q_dot_laser(time[1],t_p,t_d,q_dot_max)/C_tot)*Dt "1st Euler step"}


duplicate j=1,(M-1)
T[j+1]=T[j]+((T_infinity-T[j])/tau+q_dot_laser(time[j],t_p,t_d,q_dot_max)/C_tot)*Dt "Euler steps"
end

Figure P3.2-5(a) illustrates the temperature as a function of time; the analytical solution from
Problem 3.1-6 is also shown.
800

700

600
Temperature (K)

500

400

300
numerical solution
analytical solution
200
0x100 2x10-6 4x10-6 6x10-6 8x10-6 10-5
Time (s)
Figure P3.2-5(a): Temperature of the particle as a function of time predicted by the numerical model using
the Euler method and the analytical model from Problem 3.1-6 (3-3).

The solution is obtained in MATLAB using a straightforward translation of the EES code:

function[time,T]=P3p2d5a(M)
%Input:
%M - number of time steps (-)
%Outputs:
%time - vector of times (s)
%T - vector of temperatures (K)
r_p=5e-6; %particle radius (m)
c=1500; %specific heat capacity (J/kg-K)
rho=800; %density (kg/m^3)
h_bar=60000; %heat transfer coefficient (W/m^2-K)
t_p=2e-6; %time of the pulse (s)
t_d=0.5e-6; %duration of pulse (s)
q_dot_max=0.22; %maximum laser power (W)
T_infinity=293.2; %air temperature (K)

V=4*pi*r_p^3/3; %volume (m^3)


A_s=4*pi*r_p^2; %surface area (m^2)
C_tot=rho*V*c; %total heat capacity (J/K)
tau=C_tot/(h_bar*A_s); %lumped capacitance time constant (s)

t_sim=10e-6; %total simulation time (s)


Dt=t_sim/(M-1); %timestep duration (s)
for j=1:M
time(j)=(j-1)*t_sim/(M-1); %time (s)
end

T(1)=T_infinity; %initial temperature


for j=1:(M-1)
T(j+1)=T(j)+((T_infinity-T(j))/tau+...
q_dot_laser(time(j),t_p,t_d,q_dot_max)/C_tot)*Dt;
%all of the Euler steps
end
end

function[q_dot]=q_dot_laser(time,t_p,t_d,q_dot_max)
% Inputs:
% time - time relative to start of process (s)
% t_p - time of laser pulse peak relative to start of process (s)
% t_d - pulse duration (s)
% q_dot_max - maximum power (W)
% Output:
% q_dot_laser - instantaneous laser power (W)

q_dot=q_dot_max*exp(-(time-t_p)^2/(2*t_d^2));

end

b.) Develop a numerical model of the particle using Heun's technique implemented in either EES
of MATLAB. Plot the temperature as a function of time.

The state equation is integrated through the 1st time step using Heun's technique. The predictor
step begins by taking an Euler step to estimate the temperature at the end of the time step:

dT
Tˆ2 = T1 + Δt (13)
dt T =T1 ,t = t1

dTdt_hat[1]=(T_infinity-T[1])/tau+q_dot_laser(time[1],t_p,t_d,q_dot_max)/C_tot
"initial estimate of temperature derivative"
T_hat[2]=T[1]+dTdt_hat[1]*Dt "1st predictor step"

The corrector step uses the estimate of the temperature derivative at the beginning and end of the
time step:

⎛ dT dT ⎞ Δt
T2 = T1 + ⎜ + ⎟⎟ (14)
⎜ dt dt T =Tˆ2 ,t = t2 ⎠ 2
⎝ T =T1 ,t = t1

dTdt[2]=(T_infinity-T_hat[2])/tau+q_dot_laser(time[2],t_p,t_d,q_dot_max)/C_tot
"initial estimate of temperature derivative"
T[2]=T[1]+(dTdt_hat[1]+dTdt[2])*Dt/2 "1st corrector step"

The process of simulating all of the time steps is automated:

{dTdt_hat[1]=(T_infinity-T[1])/tau+q_dot_laser(time[1],t_p,t_d,q_dot_max)/C_tot
"initial estimate of temperature derivative"
T_hat[2]=T[1]+dTdt_hat[1]*Dt "1st predictor step"
dTdt[2]=(T_infinity-T_hat[2])/tau+q_dot_laser(time[2],t_p,t_d,q_dot_max)/C_tot
"initial estimate of temperature derivative"
T[2]=T[1]+(dTdt_hat[1]+dTdt[2])*Dt/2 "1st corrector step"}

duplicate j=1,(M-1)
dTdt_hat[j]=(T_infinity-T[j])/tau+q_dot_laser(time[j],t_p,t_d,q_dot_max)/C_tot
"initial estimate of temperature derivative"
T_hat[j+1]=T[j]+dTdt_hat[j]*Dt "predictor step"
dTdt[j+1]=(T_infinity-T_hat[j+1])/tau+q_dot_laser(time[j+1],t_p,t_d,q_dot_max)/C_tot
"initial estimate of temperature derivative"
T[j+1]=T[j]+(dTdt_hat[j]+dTdt[j+1])*Dt/2 "corrector step"
end

The temperature as a function of time is shown in Figure P3.2-5(b).


800

700

600
Temperature (K)

500

400

300
numerical solution
analytical solution
200
0x100 2x10-6 4x10-6 6x10-6 8x10-6 10-5
Time (s)
Figure P3.2-5(b): Temperature of the particle as a function of time predicted by the numerical model using
the Heun's method and the analytical model from Problem 3.1-6 (3-3).

Because Heun's method is also an explicit technique, the translation to MATLAB is


straightforward:
function[time,T]=P3p2d5b(M)
%Input:
%M - number of time steps (-)
%Outputs:
%time - vector of times (s)
%T - vector of temperatures (K)

r_p=5e-6; %particle radius (m)


c=1500; %specific heat capacity (J/kg-K)
rho=800; %density (kg/m^3)
h_bar=60000; %heat transfer coefficient (W/m^2-K)
t_p=2e-6; %time of the pulse (s)
t_d=0.5e-6; %duration of pulse (s)
q_dot_max=0.22; %maximum laser power (W)
T_infinity=293.2; %air temperature (K)

V=4*pi*r_p^3/3; %volume (m^3)


A_s=4*pi*r_p^2; %surface area (m^2)
C_tot=rho*V*c; %total heat capacity (J/K)
tau=C_tot/(h_bar*A_s); %lumped capacitance time constant (s)

t_sim=10e-6; %total simulation time (s)


Dt=t_sim/(M-1); %timestep duration (s)
for j=1:M
time(j)=(j-1)*t_sim/(M-1); %time (s)
end

T(1)=T_infinity; %initial temperature


for j=1:(M-1)
dTdt_hat=(T_infinity-T(j))/tau+...
q_dot_laser(time(j),t_p,t_d,q_dot_max)/C_tot;
%initial estimate of temperature derivative
T_hat=T(j)+dTdt_hat*Dt; %predictor step
dTdt=(T_infinity-T_hat)/tau+...
q_dot_laser(time(j+1),t_p,t_d,q_dot_max)/C_tot;
%initial estimate of temperature derivative
T(j+1)=T(j)+(dTdt_hat+dTdt)*Dt/2; %corrector step
end
end

function[q_dot]=q_dot_laser(time,t_p,t_d,q_dot_max)
% Inputs:
% time - time relative to start of process (s)
% t_p - time of laser pulse peak relative to start of process (s)
% t_d - pulse duration (s)
% q_dot_max - maximum power (W)
% Output:
% q_dot_laser - instantaneous laser power (W)

q_dot=q_dot_max*exp(-(time-t_p)^2/(2*t_d^2));

end
c.) Develop a numerical model of the particle using the fully implicit technique implemented in
either EES of MATLAB. Plot the temperature as a function of time.

The state equation is integrated through the 1st time step using the fully implicit technique.

dT
T2 = T1 + Δt (15)
dt T =T2 ,t = t2

or, for this problem:

⎡ (T − T ) qlaser ,t =t2 ⎤
T2 = T1 + ⎢ ∞ 2 + ⎥ Δt (16)
⎣ τ C ⎦

T[2]=T[1]+((T_infinity-T[2])/tau+q_dot_laser(time[2],t_p,t_d,q_dot_max)/C_tot )*Dt "1st implicit step"

The solution is extended to all of the time steps according to:

{T[2]=T[1]+((T_infinity-T[2])/tau+q_dot_laser(time[2],t_p,t_d,q_dot_max)/C_tot )*Dt "1st implicit step"}


duplicate j=1,(M-1)
T[j+1]=T[j]+((T_infinity-T[j+1])/tau+q_dot_laser(time[j+1],t_p,t_d,q_dot_max)/C_tot )*Dt "implicit step"
end

The temperature as a function of time predicted using the fully implicit method is shown in
Figure 3.2-5(c).
800

700

600
Temperature (K)

500

400

300
numerical solution
analytical solution
200
0x100 2x10-6 4x10-6 6x10-6 8x10-6 10-5
Time (s)
Figure P3.2-5(c): Temperature of the particle as a function of time predicted by the numerical model using
the fully implicit method and the analytical model from Problem 3.1-6 (3-3).

In order to implement the solution using MATLAB, it is necessary to solve Eq. (16) for T2 in
order to obtain an explicit equation:
⎡T qlaser ,t =t2 ⎤
T1 + ⎢ ∞ + Δt
⎣ τ C ⎥⎦
T2 = (17)
⎛ Δt ⎞
⎜1 + ⎟
⎝ τ ⎠

or in general:

⎡T qlaser ,t =t2 ⎤
Tj + ⎢ ∞ + Δt
⎣ τ C ⎥⎦
T j +1 = (18)
⎛ Δt ⎞
⎜1 + ⎟
⎝ τ ⎠

function[time,T]=P3p2d5c(M)
%Input:
%M - number of time steps (-)
%Outputs:
%time - vector of times (s)
%T - vector of temperatures (K)

r_p=5e-6; %particle radius (m)


c=1500; %specific heat capacity (J/kg-K)
k=1; %conductivity (W/m-K)
rho=800; %density (kg/m^3)
h_bar=60000; %heat transfer coefficient (W/m^2-K)
t_p=2e-6; %time of the pulse (s)
t_d=0.5e-6; %duration of pulse (s)
q_dot_max=0.22; %maximum laser power (W)
T_infinity=293.2; %air temperature (K)

V=4*pi*r_p^3/3; %volume (m^3)


A_s=4*pi*r_p^2; %surface area (m^2)
C_tot=rho*V*c; %total heat capacity (J/K)
tau=C_tot/(h_bar*A_s); %lumped capacitance time constant (s)

t_sim=10e-6; %total simulation time (s)


Dt=t_sim/(M-1); %timestep duration (s)
for j=1:M
time(j)=(j-1)*t_sim/(M-1); %time (s)
end

T(1)=T_infinity; %initial temperature


for j=1:(M-1)

T(j+1)=(T(j)+T_infinity*Dt/tau+q_dot_laser(time(j+1),t_p,t_d,q_dot_max)*Dt/C_
tot)/(1+Dt/tau);
end
end

function[q_dot]=q_dot_laser(time,t_p,t_d,q_dot_max)
% Inputs:
% time - time relative to start of process (s)
% t_p - time of laser pulse peak relative to start of process (s)
% t_d - pulse duration (s)
% q_dot_max - maximum power (W)
% Output:
% q_dot_laser - instantaneous laser power (W)

q_dot=q_dot_max*exp(-(time-t_p)^2/(2*t_d^2));

end

d.) Develop a numerical model of the particle using the Crank-Nicolson technique implemented
in either EES of MATLAB. Plot the temperature as a function of time.

The state equation is integrated through the 1st time step using the Crank-Nicolson technique.

⎛ dT dT ⎞ Δt
T2 = T1 + ⎜ + ⎟⎟ (19)
⎜ dt dt T =T2 ,t = t2 ⎠ 2
⎝ T =T1 ,t = t1

or, for this problem:

⎡ (T − T ) qlaser ,t =t1 (T∞ − T2 ) qlaser ,t =t2 ⎤ Δt


T2 = T1 + ⎢ ∞ 1 + + + ⎥ (20)
⎣ τ C τ C ⎦ 2

T[2]=T[1]+((T_infinity-T[1])/tau+q_dot_laser(time[1],t_p,t_d,q_dot_max)/C_tot+&
(T_infinity-T[2])/tau+q_dot_laser(time[2],t_p,t_d,q_dot_max)/C_tot )*Dt/2 "1st C-N step"

and extended to all of the time steps:

duplicate j=1,(M-1)
T[j+1]=T[j]+((T_infinity-T[j])/tau+q_dot_laser(time[j],t_p,t_d,q_dot_max)/C_tot+&
(T_infinity-T[j+1])/tau+q_dot_laser(time[j+1],t_p,t_d,q_dot_max)/C_tot )*Dt/2 "C-N step"
end

The temperature as a function of time predicted using the Crank-Nicolson method is shown in
Figure 3.2-5(d).
800

700

600

Temperature (K)
500

400

300
numerical solution
analytical solution
200
0x100 2x10-6 4x10-6 6x10-6 8x10-6 10-5
Time (s)
Figure P3.2-5(d): Temperature of the particle as a function of time predicted by the numerical model using
the Crank-Nicolson method and the analytical model from Problem 3.1-6 (3-3).

In order to implement the solution using MATLAB, it is necessary to solve Eq. (20) for T2 in
order to obtain an explicit equation:

⎡ T Δt T Δt qlaser ,t =t1 Δt (T∞ − T2 ) qlaser ,t =t2 Δt ⎤


T1 + ⎢ ∞ − 1 + + + ⎥
⎣ τ 2τ C 2 τ C 2⎦
T2 = (21)
⎛ Δt ⎞
⎜1 + ⎟
⎝ 2⎠

Equation (21) is extended to all of the time steps:

⎡ T Δt T Δt qlaser ,t =t1 Δt (T∞ − T2 ) qlaser ,t =t2 Δt ⎤


Tj + ⎢ ∞ − j + + + ⎥
⎣ τ 2τ C 2 τ C 2⎦
T j +1 = (22)
⎛ Δt ⎞
⎜1 + ⎟
⎝ 2⎠

function[time,T]=P3p2d5d(M)
%Input:
%M - number of time steps (-)
%Outputs:
%time - vector of times (s)
%T - vector of temperatures (K)

r_p=5e-6; %particle radius (m)


c=1500; %specific heat capacity (J/kg-K)
k=1; %conductivity (W/m-K)
rho=800; %density (kg/m^3)
h_bar=60000; %heat transfer coefficient (W/m^2-K)
t_p=2e-6; %time of the pulse (s)
t_d=0.5e-6; %duration of pulse (s)
q_dot_max=0.22; %maximum laser power (W)
T_infinity=293.2; %air temperature (K)

V=4*pi*r_p^3/3; %volume (m^3)


A_s=4*pi*r_p^2; %surface area (m^2)
C_tot=rho*V*c; %total heat capacity (J/K)
tau=C_tot/(h_bar*A_s); %lumped capacitance time constant (s)

t_sim=10e-6; %total simulation time (s)


Dt=t_sim/(M-1); %timestep duration (s)
for j=1:M
time(j)=(j-1)*t_sim/(M-1); %time (s)
end

T(1)=T_infinity; %initial temperature


for j=1:(M-1)
T(j+1)=(T(j)*(1-
Dt/(2*tau))+T_infinity*Dt/tau+(q_dot_laser(time(j),t_p,t_d,q_dot_max)+q_dot_l
aser(time(j+1),t_p,t_d,q_dot_max))*Dt/(2*C_tot))/(1+Dt/(2*tau));
end
end

function[q_dot]=q_dot_laser(time,t_p,t_d,q_dot_max)
% Inputs:
% time - time relative to start of process (s)
% t_p - time of laser pulse peak relative to start of process (s)
% t_d - pulse duration (s)
% q_dot_max - maximum power (W)
% Output:
% q_dot_laser - instantaneous laser power (W)

q_dot=q_dot_max*exp(-(time-t_p)^2/(2*t_d^2));

end

e.) Develop a numerical model of the particle using the Integral command in EES. Plot the
temperature as a function of time.

The state equation is evaluated at an arbitrary value of temperature and time:

t_sim=10e-6 [s] "total simulation time"


time=3e-6 [s] "arbitrary time for evaluation of integrand"
T=500 [K] "arbitary temperature for evaluation of integrand"
dTdt=(T_infinity-T)/tau+q_dot_laser(time,t_p,t_d,q_dot_max)/C_tot "integrand"

The temperature and time are commented out and the integral command is used to integrate the
state equation from 0 to tsim:

{time=3e-6 [s] "arbitrary time for evaluation of integrand"


T=500 [K] "arbitary temperature for evaluation of integrand"}
dTdt=(T_infinity-T)/tau+q_dot_laser(time,t_p,t_d,q_dot_max)/C_tot "integrand"
T=T_infinity+Integral(dTdt,time,0,t_sim)

An integral table is created to hold the results:

$IntegralTable time,T
The temperature as a function of time predicted using the Integral command in EES is shown in
Figure 3.2-5(e).
800

700

Temperature (K)
600

500

400

300

200
0x100 2x10-6 4x10-6 6x10-6 8x10-6 10-5
Time (s)
Figure P3.2-5(e): Temperature of the particle as a function of time predicted using the Integral command in
EES and the analytical model from Problem 3.1-6.

f.) Develop a numerical model of the particle using ode45 solver in MATLAB. Plot the
temperature as a function of time.

A function is created in MATLAB to return the state equation; note that the first two inputs to
the function are time and the value of the state variable, temperature and the function returns the
derivative of the state variable:

function[dTdt]=dTdtP3p2d5f2(time,T,r_p,c,rho,h_bar,t_p,t_d,q_dot_max,T_infini
ty)

V=4*pi*r_p^3/3; %volume (m^3)


A_s=4*pi*r_p^2; %surface area (m^2)
C_tot=rho*V*c; %total heat capacity (J/K)
tau=C_tot/(h_bar*A_s); %lumped capacitance time constant (s)

dTdt=(T_infinity-T)/tau+q_dot_laser(time,t_p,t_d,q_dot_max)/C_tot;
end

function[q_dot]=q_dot_laser(time,t_p,t_d,q_dot_max)
% Inputs:
% time - time relative to start of process (s)
% t_p - time of laser pulse peak relative to start of process (s)
% t_d - pulse duration (s)
% q_dot_max - maximum power (W)
% Output:
% q_dot_laser - instantaneous laser power (W)

q_dot=q_dot_max*exp(-(time-t_p)^2/(2*t_d^2));

end

The function is integrated in a separate function using the ode45 solver:


function[time,T]=P3p2d5f()
r_p=5e-6; %particle radius (m)
c=1500; %specific heat capacity (J/kg-K)
k=1; %conductivity (W/m-K)
rho=800; %density (kg/m^3)
h_bar=60000; %heat transfer coefficient (W/m^2-K)
t_p=2e-6; %time of the pulse (s)
t_d=0.5e-6; %duration of pulse (s)
q_dot_max=0.22; %maximum laser power (W)
T_infinity=293.2; %air temperature (K)

V=4*pi*r_p^3/3; %volume (m^3)


A_s=4*pi*r_p^2; %surface area (m^2)
C_tot=rho*V*c; %total heat capacity (J/K)
tau=C_tot/(h_bar*A_s); %lumped capacitance time constant (s)

t_sim=10e-6; %total simulation time (s)

OPTIONS=odeset('RelTol',1e-6);
[time,T]=ode45(@(time,T) dTdtP3p2d5f2(time,T,r_p,c,rho,h_bar,...
t_p,t_d,q_dot_max,T_infinity),[0,t_sim],T_infinity,OPTIONS);
end

The temperature as a function of time predicted using the ode45 solver in MATLAB is shown in
Figure 3.2-5(f).

750

700

650

600
Temperature (K)

550

500

450

400

350

300

250
0 0.2 0.4 0.6 0.8 1 1.2
Time (s) x 10
-5

Figure P3.2-5(f): Temperature of the particle as a function of time predicted using the ode45 solver in
MATLAB.
PROBLEM 3.2-6 (3-6 in text)
You are interested in using a thermoelectric cooler to quickly reduce the temperature of a small
detector from its original temperature of Tini = 295 K to its operating temperature. As shown in
Figure P3.2-6, the thermoelectric cooler receives power at a rate of w = 5.0 W from a small
battery and rejects heat at a rate of qrej to ambient temperature TH = 305 K. The cooler removes
energy at a rate of qref from the detector which is at temperature T. (The detector temperature T
will change with time, t). The detector has a total heat capacity, C, of 0.5 J/K. Despite your best
efforts to isolate the detector from the ambient, the detector is subjected to a parasitic heat gain,
q p , that can be modeled as occurring through a fixed resistance Rp = 100 K/W; this resistance
represents the combined effect of radiation and conduction.

qrej
TH = 305 K

w
battery
q p
thermoelectric cooler
T qref

detector, C = 0.5 J/K


Figure P3.2-6: Detector cooled by a thermoelectric cooler.

The thermoelectric cooler has a 2nd law efficiency ηc = 10% regardless of its operating
temperatures. That is, the amount of refrigeration provided to the detector can be related to the
input power provided to the thermoelectric cooler and its operating temperatures according to:
w ηc
qref =
⎛ TH ⎞
⎜ − 1⎟
⎝T ⎠
a.) Derive the governing differential equation that describes the temperature of the detector.
Note that the result should be a symbolic equation for the rate of temperature change of the
detector as a function of the quantities given in the problem (i.e., TH, Rp, C, w , ηc) and the
instantaneous value of the detector temperature (T).

An energy balance on the detector leads to:

dT
q p = C + qref (1)
dt

or

(TH − T ) = C dT + w ηc
(2)
Rp dt ⎛ TH ⎞
⎜ − 1⎟
⎝T ⎠
so the state equation for the problem is:

dT (TH − T ) w ηc
= − (3)
dt C Rp ⎛T ⎞
C ⎜ H − 1⎟
⎝T ⎠

b.) Develop an EES program that numerically solves this problem for the values given in the
problem statement using a predictor-corrector technique (e.g., the modified Euler technique).
Using your program, prepare a plot showing the temperature of the detector as a function of
time for 120 sec after the cooler is activated.

The inputs are entered in EES and a function is defined to return the state equation:

$UnitSystem SI MASS RAD PA K J


$TABSTOPS 0.2 0.4 0.6 0.8 3.5 in

function dTdt(time,T,T_H,C,R_p,w_dot,eta_c)
"Inputs:
time - time relative to start of cooldown (s)
T - temperature of detector (K)
T_H - heat rejection temperature (K)
C - heat capacity of detector (J/K)
R_p - resistance to parasitic (K/W)
w_dot - power to cooler (W)
eta_c - 2nd law efficiency of cooler

Output:
dTdt - time rate of change of detector (K/s)"

dTdt=(T_H-T)/(C*R_p)-w_dot*eta_c/(C*(T_H/T-1))
end

"Inputs"
T_ini=295 [K] "initial temperature"
w_dot=30 [W] "power to cooler"
T_H=305 [K] "heat rejection temperature"
C=0.5 [J/K] "heat capacity"
R_p=100 [K/W] "resistance to parasitic"
eta_c=0.1 [-] "cooler efficiency"

The simulation is broken into timesteps, distributed according to:

tsim
Δt = (4)
( M − 1)

where M is the number of times at which the temperature will be evaluated. The time
corresponding to each time step is therefore:
tj =
( j − 1) t for j = 1..M (5)
( M − 1) sim
t_sim=30 [s] "simulation time"
M=91 [-] "number of time steps"
DELTAt=t_sim/(M-1) "duration of time step"
duplicate j=1,M
time[j]=(j-1)*t_sim/(M-1) "time associated with each temperature"
end

Heun's method relies on a predictor step followed by a corrector step:

"Heun's Method"
T[1]=T_ini
duplicate j=1,(M-1)
T_hat[j+1]=T[j]+dTdt(time[j],T[j],T_H,C,R_p,w_dot,eta_c)*DELTAt "predictor step"
T[j+1]=T[j]+dTdt(time[j],T[j],T_H,C,R_p,w_dot,eta_c)*DELTAt/2+&
dTdt(time[j+1],T_hat[j+1],T_H,C,R_p,w_dot,eta_c)*DELTAt/2 "corrector step"
end

Figure 2 illustrates the temperature as a function of time:

300

290

280

270
Temperature (K)

260

250

240

230

220

210

200
0 10 20 30 40 50 60 70 80 90 100 110 120
Time (s)
Figure 2: Temperature of the detector as a function of time.

c.) Modify your program so that it accounts for the fact that your battery only has 100 J of
energy storage capacity; once the 100 J of energy in the battery is depleted then the power
driving the thermoelectric cooler goes to zero. Prepare a plot showing the temperature of the
detector as a function of time for 120 s after the cooler is activated.

The function dTdt is modified so that the refrigeration goes to zero when the power is depleted:

function dTdt(time,T,T_H,C,R_p,w_dot,eta_c)
"Inputs:
time - time relative to start of cooldown (s)
T - temperature of detector (K)
T_H - heat rejection temperature (K)
C - heat capacity of detector (J/K)
R_p - resistance to parasitic (K/W)
w_dot - power to cooler (W)
eta_c - 2nd law efficiency of cooler

Output:
dTdt - time rate of change of detector (K/s)"

Energy = 100 [J] "energy in battery"


if (time<(Energy/w_dot)) then
dTdt=(T_H-T)/(C*R_p)-w_dot*eta_c/(C*(T_H/T-1))
else
dTdt=(T_H-T)/(C*R_p)
endif

{dTdt=(T_H-T)/(C*R_p)-w_dot*eta_c/(C*(T_H/T-1))}
end

Figure 3 illustrates the temperature as a function of time, including the re-equilibration process
that occurs when the battery is discharged at 20 s.

300

290

280

270
Temperature (K)

260

250

240

230

220

210

200
0 10 20 30 40 50 60 70 80 90 100 110 120
Time (s)
Figure 3: Temperature of the detector as a function of time including the limited battery energy.

d.) Assume that the objective of your cooler is to keep the detector at a temperature below 240 K
for as long as possible, given that your battery only has 100 J of energy. What power ( w )
would you use to run the thermoelectric cooler? Justify your answer with plots and an
explanation.

I would turn the power up as high as possible in order to drive the temperature of the detector
down as much as possible very quickly. The impact of the parasitics is related to the product of
the magnitude and time. In order to minimize the effect of parasitics, you want the detector to
cool down quickly. To see this more clearly, consider Figure 4 which shows the temperature as
a function of time for various values of w . When w = 2 W, the detector barely reaches 240 K
when the battery is depleted and therefore it is only active for an instant of time. On the other
hand when w increases beyond about 10 W it remains below 240 K for approximately 21 s
regardless of w because the effect of the parasitic on the cooldown is minimal due to its short
duration.

300

290

280

270
Temperature (K)

260

250
2W
240

230
5W
220
10 W
210 20 W
200
0 10 20 30 40 50 60 70
Time (s)
Figure 4: Temperature of the detector as a function of time for various values of power.
Problem 3.3-4 (3-7 in text): Heater
A thin heater is sandwiched between two materials, A and B, as shown in Figure P3.3-4. Both
materials are very thick and so they may be considered semi-infinite. Initially, both materials are
at a uniform temperature of Tin. The heater is activated at t = 0 and delivers a uniform heat flux,
′′ , to the interface; some of this energy will be conducted into material A ( q ′′A ) and some into
qheater
material B ( q ′′A ). Materials A and B have the same thermal diffusivity, αA = αB = α. and the
same conductivity, kA = kB = k. There is no contact resistance anywhere in this problem and it is
a 1-D, transient conduction problem.

initially, all material is at Tini


thin heater at Theater

q′′A q′′B

material A is semi-infinite material B is semi-infinite


with k, α ′′
qheater with k, α
Figure P3.3-4: Thin heater sandwiched between two semi-infinite bodies.

a.) Draw a thermal resistance network that you could use to model this problem approximately.
Your resistances should be written in terms of time, t, and the symbols in the problem
′′
statement. Clearly indicate on your network where qheater is added to the network and where
the temperatures Tin and Theater are located.

Energy must be conducted from the heater into either material A or B; the appropriate resistance
network is shown in Figure 2.

Figure 2: Resistance network representing the problem.

b.) Use your resistance network from (a) to develop an equation for the heater temperature,
Theater, in terms of the symbols in the problem statement.

The heater temperature is given by:

′′
Theater = Tin + Req qheater (1)
where Req is the parallel combination of resistances RA and RB.

−1
⎡ k k ⎤
Theater = Tin + ⎢ + ′′
⎥ qheater (2)
⎢⎣ 2 α t 2 α t ⎥⎦

or

αt
Theater = Tin + ′′
qheater (3)
k

c.) Sketch on the axes below the temperature distribution at t = 0 (this one is already done) and
two additional times after the heater has been activated (t1 and t2 where t1 > t2). Label your
plots clearly. Focus on getting the qualitative features of your plot correct.

Figure 3: Sketch of temperature distribution at two times, t1 and t2.


Problem 3.3-5 (3-8 in text): Radiant Heating
Figure P3.3-5 shows a slab of material that is L = 5 cm thick and is heated from one side (x = 0)
by a radiant heat flux qs′′ = 7500 W/m2. The material has conductivity k = 2.4 W/m-K and
thermal diffusivity α = 2.2x10-4 m2/s. Both sides of the slab are exposed air at T∞ = 20°C with
heat transfer coefficient h = 15 W/m 2 -K . The initial temperature of the material is Tini = 20°C.

k = 2.4 W/m-K
initial temperature, Tini = 20°C α = 2.2x10-4 m2/s

qs′′ = 7500 W/m h = 15 W/m -K


2 2

T∞ = 20°C

h = 15 W/m -K
2

=
T∞ 20 C° L = 5 cm
x
Figure P3.3-5: Slab of material heated at one surface.

a.) About how long do you expect it to take for the temperature of the material on the unheated
side (x = L) to begin to rise?

The inputs are entered in EES:

$UnitSystem SI MASS RAD PA K J


$TABSTOPS 0.2 0.4 0.6 0.8 3.5 in

"Inputs"
k=2.4 [W/m-K] "thermal conductivity"
alpha=2.2e-4 [m^2/s] "thermal diffusivity"
h_bar=15 [W/m^2-K] "heat transfer coefficient"
q``_s=7500 [W/m^2] "heat flux at left surface"
L=5 [cm]*convert(cm,m) "thickness of slab"
A_c=1 [m^2] "area of wall"
T_ini=converttemp(C,K,20 [C]) "initial temperature"
T_infinity=converttemp(C,K,20 [C]) "ambient temperature"

The time require for the thermal wave associated with the disturbance at the left side to reach the
right side is given approximately by:

2 α t1 = L (1)

2*sqrt(alpha*time_1)=L "time required for thermal wave to reach left side"

which leads to t1 = 2.84 s.

b.) What do you expect the temperature of the material at the heated surface (x = 0) to be
(approximately) at the time identified in (a)?
The resistance network shown in Figure P3.3-5-2 can be used to approximately represent this
problem.

qs′′ As

T∞ = 20°C Tini = 20°C

Rconv Ts Rsemi −∞
Figure P3.3-5-2: Resistance network at time t1.

The convection resistance is:

1
Rconv = (2)
h Ac

and the semi-infinite body resistance at t1 is:

2 α t1
Rsemi −∞ ,t1 = (3)
k Ac

R_conv=1/(h_bar*A_c) "convection resistance"


R_semi_time1=2*sqrt(alpha*time_1)/(k*A_c) "semi-infinite body resistance at time 1"

According to Figure P3.3-5-2, the surface temperature at time t1 is given by:

qs′′ Ac =
(T
s ,t1 − T∞ ) + (T s ,t1 − Tini ) (4)
Rconv Rsemi −∞ ,t1

q``_s*A_c=(T_s_time1-T_infinity)/R_conv+(T_s_time1-T_ini)/R_semi_time1
"surface temperature at time 1"

which leads to Ts ,t1 = 412 K (139°C) .

c.) Develop a simple and approximate model that can predict the temperature at the heated
surface as a function of time for times that are less than the time calculated in (a). Plot the
temperature as a function of time from t = 0 to the time identified in (a).

The resistance network shown in Figure 3.3-5-3 is used.


qs′′ As

T∞ = 20°C Tini = 20°C

Ts
1 2 αt
h Ac k Ac
Figure P3.3-5-3: Resistance network at time t.

According to Figure P3.3-5-3:

qs′′ Ac =
(Ts − T∞ ) + (Ts ,t 1
− Tini ) (5)
1 2 αt
h Ac k Ac

q``_s*A_c=(T_s-T_infinity)/(1/(h_bar*A_c))+(T_s-T_ini)/(2*sqrt(alpha*time)/(k*A_c))
"surface temperature at time less than time 1"
T_s_C=converttemp(K,C,T_s) "in C"

Figure 3.3-5-4 illustrates the surface temperature as a function of time.


140

120
Temperature (°C)

100

80

60

40

20
0 0.5 1 1.5 2 2.5 3
Time (s)
Figure P3.3-5-4: Resistance network at time t.

d.) Sketch the temperature as a function of position in the slab for several times less than the
time identified in (a) and greater than the time identified in (a). Make sure that you get the
qualitative features of the sketch correct. Also sketch the temperature as a function of
position in the slab at steady state (make sure that you get the temperatures at either side
correct).

A thermal wave propagates from the left side of the slab. The temperature gradient at x = 0 is
always negative because energy is conducted into the slab. Further, the boundary condition at x
= 0 is:
∂T
qs′′ = h (Tx =0 − T∞ ) − k (6)
∂x x =0

therefore, the absolute value of the temperature gradient must decrease with time because more
of the energy from the radiant heating goes to convection as Tx=0 increases. After t1, the
temperature at x = L begins to rise. These characteristics are captured in Figure P3.3-5-5.

h = 15 W/m -K
2

qs′′ = 7500 W/m


2 T∞ = 20°C

h = 15 W/m -K
2

T∞ = 20°C
L = 5 cm
x
T
303°C t→∞

236°C
t > t1
139°C
t = t1

20°C
x
t < t1
Figure P3.3-5-5: Sketch of temperature as a function of x for various values of time less than t1 as well as at
steady state.

The steady state solution is obtained using the resistance network shown in Figure P3.3-5-6.

qs′′ As

qw Tx=L
T∞ = 20°C T∞ = 20°C

Ts
1 L 1
h Ac k Ac h Ac
Figure P3.3-5-6: Resistance network used to determine steady state temperature distribution.

The temperature of the surface at x = 0 at steady state is given by:


qs′′ Ac =
(T
x = 0,t →∞ − T∞ )
+
(T x = 0,t →∞ − T∞ )
(7)
1 ⎛ 1 L ⎞
⎜ + ⎟
h Ac ⎝ h Ac k Ac ⎠

The heat transfer to the wall at steady state is given by:

qw =
(T x = 0,t →∞ − T∞ )
(8)
⎛ 1 L ⎞
⎜ + ⎟
⎝ h Ac k Ac ⎠

The temperature of the surface at x = L at steady state is given by:

L
Tx = L ,t →∞ = Tx =0,t →∞ − qw (9)
k Ac

which leads to Tx =0,t →∞ = 576.9 K (303.8°C) and Tx = L ,t →∞ = 509.4 K (236.2°C).


Problem 3.3-6 (3-9 in text): Semi-infinite body exposed to convection
A semi-infinite body has conductivity k = 1.2 W/m-K and thermal diffusivity α = 5x10-4 m2/s.
At time t = 0, the surface is exposed to fluid at T∞ = 90°C with heat transfer coefficient
h = 35 W/m 2 -K . The initial temperature of the material is Tini = 20°C.
a.) Develop an approximate model that can provide the temperature of the surface and the rate of
heat transfer into the surface as a function of time.

k = 1.2 W/m-K
initial temperature, Tini = 20°C α = 5x10-4 m2/s

T∞ = 90°C
h = 35 W/m -K
2

x
Figure P3.3-6(a): Semi-infinite body exposed to a convection boundary condition.

The inputs are entered in EES:

$UnitSystem SI MASS RAD PA K J


$Tabstops 0.2 0.4 0.6 0.8 3.5

"Inputs"
k=1.2 [W/m-K] "thermal conductivity"
alpha=5e-4 [m^2/s] "thermal diffusivity"
h_bar=35 [W/m^2-K] "heat transfer coefficient"
A_c=1 [m^2] "area of wall"
T_ini=converttemp(C,K,20 [C]) "initial temperature"
T_infinity=converttemp(C,K,90 [C]) "ambient temperature"

The resistance network shown in Figure 3.3-6(b) is used to analyze this problem.

T∞ = 90°C q ′′ As Tini = 20°C

Ts
1 2 αt
h Ac k Ac
Figure P3.3-6(b): Resistance network for approximate model.

According to Figure P3.3-6(b), the heat flow into the surface is:

qs′′ Ac ≈
(T∞ − Tini ) (1)
1 2 αt
+
h Ac k Ac

and the surface temperature is:


qs′′
Ts ≈ T∞ − (2)
h

time=0 [s] "time"


q``_s*A_c=(T_infinity-T_ini)/(1/(h_bar*A_c)+2*sqrt(alpha*time)/(k*A_c)) "surface heat flux"
T_s=T_infinity-q``_s/h_bar "surface temperature"
T_s_C=converttemp(K,C,T_s) "in C"

Figure 3.3-6(c) illustrates the predicted surface temperature and heat flux as a function of time.
80 3000
approximate model
exact model
70 2500
Surface temperature (°C)

Surface heat flux (W/m )


2
60 2000

50 surface temperature 1500

40 1000

30 surface heat flux 500

20 0
0 0.5 1 1.5 2 2.5 3 3.5 4 4.5 5
Time (s)
Figure P3.3-6(c): Surface temperature and heat flux predicted by the approximate model and the exact
solution from the EES function SemiInf3 as a function of time.

b.) Based on your model, develop an expression that provides a characteristic time related to
how long will it take for the surface of the solid to approach T∞?

The temperature of the surface will approach the temperature of the fluid when the resistance to
conduction through the thermally affected zone increases to the point where it dominates the
resistance to convection from the surface. A characteristic time is defined as being the time
where these resistances are equal:

1 2 α tchar
= (3)
h Ac k Ac

or

k2
tchar = (4)
4α h 2

t_char=k^2/(4*alpha*h_bar^2) "characteristic time"

c.) Compare the results of your model from (a) with the exact solution programmed in EES and
accessed using the SemiInf3 function.
The surface temperature and heat flux are obtained using the SemiInf3 function and are also
shown in Figure 3.3-6(c).

T_s_exact=SemiInf3(T_ini,T_infinity,h_bar,k,alpha,0 [m],time) "surface temperature"


T_s_exact_C=converttemp(K,C,T_s_exact) "in C"
q``_s_exact=h_bar*(T_infinity-T_s_exact) "surface heat flux"
Problem 3.3-7 (3-10 in text)
A rod with uniform cross-sectional area, Ac = 0.1 m2 and perimeter per = 0.05 m is placed in a
vacuum environment. The length of the rod is L = 0.09 m and the external surfaces of the rod
can be assumed to be adiabatic. For a long time, a heat transfer rate of qh = 100 W is provided to
the end of the rod at x = 0. The tip of the rod at x = L is always maintained at Tt = 20°C. The rod
material has density ρ = 5000 kg/m3, specific heat capacity c = 500 J/kg-K, and conductivity k =
5 W/m-K. The rod is at a steady state operating condition when, at time t = 0, the heat transfer
rate at x = 0 becomes zero.
a.) About how long does it take for the rod to respond to the change in heat transfer?

Ac = 0.1 m2
per = 0.05 m
ρ = 5000 kg/m3
k = 5 W/m-K
c = 500 J/kg-K
Tt = 20°C
qh = 100 W

x
L = 0.09 m
Figure 3.3-7(a): Bracket supporting a heater.

The rod has a uniform cross-sectional area, Ac = 0.1 m2 and perimeter per = 0.05 m. The length
of the rod is L = 0.09 m and the external surfaces of the rod can be assumed to be adiabatic. The
heater provides a heat transfer of qh = 100 W into the end of the rod at x = 0. The tip of the rod
at x = L is always maintained at Tt = 20°C. The rod material has density ρ = 5000 kg/m3,
specific heat capacity c = 500 J/kg-K, and conductivity k = 5 W/m-K. The heater has been
operating for a long time and is at a steady state operating condition when at time t = 0, the
power is shut off so that the heat transfer rate at x = 0 becomes zero.

a.) About how long does it take for the rod to respond to the change in heat transfer?

The time response of the rod is dictated by the time required for a conduction thermal wave to
travel from x = 0 to x = L.

L2
τ diff = (1)

where α is the thermal diffusivity:

k 5W m3 kg-K
α= = = 2.0x10-6 m 2 /s (2)
ρ c m-K 5000 kg 500 J

Therefore the time constant is:


( 0.09 )
2
m2 s
τ diff = = 1013 s (3)
4 2x10-6 m 2

so it will take about 1000 s for the rod to respond.

b.) Sketch the temperature distribution you expect at t = 0 and t → ∞. Make sure that you get
the temperatures at either end of the rod and the shape of the temperature distributions
correct.

At t = 0 the temperature distribution is linear. The resistance to conduction through the rod is:

L 0.09 m m-K
Rcond = = = 0.18 K/W (4)
k Ac 5 W 0.1 m 2

so the temperature of the rod at x = 0 is:

100 W 0.18 K
Tx =0,t =0 = Tt + qh Rh = 20°C + = 38°C (5)
W

As t → ∞, the temperature everywhere in the rod approaches Tt. These characteristics are
reflected in Figure 3.3-7(b).

Tt = 20°C

x
T
t=0
t = 500 s
t = 1000 s
t = 2000 s

t→∞
20°C
x
0 0.09 m
Figure 3.3-7(b): Temperature distribution in the rod at various times.

c.) Overlay on your sketch from (b) the temperature distributions that you expect at the time that
you calculated in (a) as well as half that time and twice that time.

The thermal wave emanates from x = 0 and travels in the positive x-direction. Beyond the extent
of the thermal wave, the rod material does not realize that anything has happened and so the
temperature distribution does not change. The temperature gradient at x = 0 must be zero at all
times greater than 0 because the heat transfer there is zero. These characteristics are reflected in
Figure 3.3-7(b).

d.) Sketch the heat transfer from the rod at x = L (i.e., at the tip) as a function of time. Make sure
that your sketch clearly shows the behavior before and after the time identified in (a). Make
sure that you get the rate of heat transfer at t = 0 and t → ∞ correct.

For times less than 1000 s, the temperature distribution and therefore the temperature gradient at
x = L does not change. Therefore the rate of heat transfer is constant for t < 1000 s. For times
greater than 1000 s, the temperature gradient drops and therefore the heat transfer rate drops.
These characteristics are reflected in Figure 3.3-7(c).

q x = L
100 W

0 t
0 1000 s
Figure 3.3-7(c): Temperature distribution in the rod at various times.
Problem 3.3-8 (3-11 in text)
One technique that is being proposed for measuring the thermal diffusivity of a material is
illustrated schematically in Figure P3.3-8.

Ts = −20°C
L = 10 cm

Tin = 20°C

Figure P3.3-8: Test setup for measuring thermal diffusivity.

The material is placed in a long, insulated container and allowed to come to thermal equilibrium
with its environment Tin = 20°C. A thermocouple is embedded in the material at a distance L =
10 cm below the surface. At time t = 0 the temperature of the surface is changed from Tin to Ts =
-20°C by applying a flow of chilled ethylene glycol to the surface. The time required for the
thermocouple to change from Tin to Ttarget = 0°C is found to be ttarget = 310.2 s.
a.) What is the measured thermal diffusivity?

The known information is entered in EES:


$UnitSystem SI MASS RAD PA C J
$Tabstops 0.2 0.4 0.6 0.8 3.5

"Inputs"
L=10 [cm]*convert(cm,m) "distance between thermocouple and surface"
T_in=converttemp(C,K,20 [C]) "initial temperature"
T_s=converttemp(C,K,-20 [C]) "surface temperature"
tau=310.2 [s] "time required to reach target temperature"
T_target=converttemp(C,K,0 [C]) "target temperature"

The response of the semi-infinite body to the step change in surface temperature is provided by
the function SemiInf1 (accessed through the Function Information window):

"Part a"
T_target=SemiInf1(T_in,T_s,alpha,L,tau) "measurement of alpha"

which leads to α = 3.54e-5 m2/s.

There is some error in your measurement from part (a) due to inaccuracies in your thermocouple
and your measurement of time and position of the thermocouple. Assume that the following
uncertainties characterize your experiment:
• the temperature measurements have an uncertainty of δTin = δTtarget = δTs = 0.2°C
• the position measurement has an uncertainty of δL = 0.1 mm
• the time measurement has an uncertainty of δτtarget = 0.5 s
b.) What is the uncertainty in your measured value of thermal diffusivity from part (a)? You can
answer this question in a number of ways including using the built-in uncertainty propagation
feature in EES.

The uncertainties listed above are entered in EES:

deltaT=0.2 [K] "temperature uncertainty"


deltaL=0.1 [mm]*convert(mm,m) "position uncertainty"
deltatime=0.5 [s] "time uncertainty"

The uncertainty propagation feature in EES is used to compute the effect of the uncertainty in
each measured variable on the calculated variable. Select Uncertainty Propagation from the
Calculate menu. The variable alpha will appear as the Calculated variable; select all of the
temperatures, the length, and time as the measured variables (Figure 2).

Figure 2: Propagation of Uncertainty window.

Select Set uncertainties to bring up the window shown in Figure 3. Each of the uncertainties for
the calculated variables can be specified using variable names; enter the variables that were
defined in the Equations window.

Figure 3: Set uncertainty of measured variables

Select OK and then OK again to carry out the propagation of uncertainty calculation; the solution
will be provided in a Solution Window labeled Uncertainty Results, Figure 4.
Figure 4: Uncertainty Results Solution window.

The uncertainty in the calculated value of α is 1.0e-6 (or 3.0% of the value).
Problem 3.4-1 (3-12 in text): Laser Target
A disk shaped piece of material is used as the target of a laser, as shown in Figure P3.4-1. The
laser target is D = 5.0 mm in diameter and b = 2.5 mm thick. The target is made of a material
with ρ = 2330 kg/m3, k = 500 W/m-K, and c = 400 J/kg-K. The target is mounted on a chuck
with a constant temperature Tc = 20°C. The interface between the target and the chuck is
characterized by a contact resistance, Rc′′ = 1x10-4 K-m2/W. The target is initially in thermal
equilibrium with the chuck. You may neglect radiation and convection from the laser target.
′′ = At 2 exp ( −t / t pulse ) where A = 1x107 W/m2-s2 and
The laser flux is pulsed according to: qlaser
tpulse = 0.10 s.

laser target
ρ = 2330 kg/m3
k = 500 W/m-K chuck, Tc = 20°C
c = 400 J/kg-K

′′
laser flux, qlaser

D = 5 mm
b = 2.5 mm
contact resistance,
Rc′′ = 1x10 K-m /W
-4 2

Figure P3.4-1: Laser target

a.) Is a lumped capacitance model of the laser target appropriate? Justify your answer.

The inputs are entered in EES:

$UnitSystem SI MASS RAD PA K J


$TABSTOPS 0.2 0.4 0.6 0.8 3.5 in

"Inputs"
b=2.5 [mm]*convert(mm,m) "thickness of target"
D=5.0 [mm]*convert(mm,m) "diameter of target"
rho=2330 [kg/m^3] "density"
k=500 [W/m-K] "conductivity"
c=400 [J/kg-K] "specific heat capacity"
Rc=1e-4 [m^2-K/W] "contact resistance"
A=1e7 [W/m^2-s^2]
t_pulse=0.1 [s]
T_c=converttemp(C,K,20[C]) "chuck temperature"

The appropriate Biot number for this problem is:

Rcond 4 b π D2 b
Bi = = = (1)
Rcontact k π D 4 Rc′′ k Rc′′
2
Bi=b/(k*Rc) "Biot number"

which leads to Bi = 0.05.

b.) Use Laplace transforms to determine the temperature of the laser target as a function of time.
Prepare a plot of the temperature of the laser target as a function of time; overlay on this plot
the laser heat flux as a function of time (on a secondary y-axis).

An energy balance on the laser target leads to:

π D2 π D2 π D2 dT
′′
qlaser =
′′
(T − Tc ) + bρ c (2)
4 4 Rc 4 dt

The time constant is defined as:

τ = Rc′′ bρ c (3)

tau=Rc*b*rho*c "time constant"

and substituted into Eq. (2):

′′
dT T Tc qlaser
+ = + Rc′′ (4)
dt τ τ τ

Substituting Eq. Error! Reference source not found. into Eq. (4) leads to:

dT T Tc Rc′′ A 2 ⎛ t ⎞
+ = + t exp ⎜ −
⎜ t pulse ⎟⎟
(5)
dt τ τ τ ⎝ ⎠

This governing differential equation is entered in Maple:

> restart;
> with(inttrans):
> GDE:=diff(T(time),time)+T/tau=T_c/tau+A*time^2*exp(-time/t_pulse)*Rc/tau;
⎛ − time ⎞
⎜⎜ ⎟⎟
⎝ t_pulse ⎠
T T_c A time 2 e
GDE := ⎜⎜⎛ T( time ) ⎞⎟⎟ + =
d Rc
+
⎝ dtime ⎠ τ τ τ

and transformed from the t to the s domain:

> GDEs:=laplace(GDE,time,s);
T T_c 2 A Rc
GDEs := s laplace( T( time ), time, s ) − T( 0 ) + = +
τs τs 3

τ ⎜⎜⎛ s +
1 ⎞

⎝ t_pulse ⎟⎠

The initial condition (Tt=0 = Tc) is substituted in place of T(0) and the variable T(s) is substituted
in place of laplace(T(time),time,s):

> GDEs:=subs(laplace(T(time),time,s)=T(s),GDEs);
T T_c 2 A Rc
GDEs := s T( s ) − T( 0 ) + = +
τs τs 3

τ ⎜⎜⎛ s +
1 ⎞

⎝ t_pulse ⎟⎠
> GDEs:=subs(T(0)=T_c,GDEs);
T T_c 2 A Rc
GDEs := s T( s ) − T_c + = +
τs τs 3

τ ⎜⎜⎛ s +
1 ⎞

⎝ t_pulse ⎟⎠

The algebraic equation is solved in the s domain:

> Ts:=solve(GDEs,T(s));
Ts := ( T_c τ s4 t_pulse 3 + 3 T_c τ s3 t_pulse 2 + 3 T_c τ s2 t_pulse + T_c τ s − T s3 t_pulse 3
− 3 T s2 t_pulse 2 − 3 T s t_pulse − T + T_c s3 t_pulse 3 + 3 T_c s2 t_pulse 2
+ 3 T_c s t_pulse + T_c + 2 A Rc t_pulse 3 s ) ( s2 τ
( s3 t_pulse 3 + 3 s2 t_pulse 2 + 3 s t_pulse + 1 ) )

and transformed back to the time domain:

> Tt:=invlaplace(Ts,s,time);


Tt := ⎜⎝ −time T + T_c ( τ + time )
⎛− ⎞
⎛ ⎞⎞
time
⎜⎜ ⎟⎟
⎜⎜ ⎟⎟
( 2 t_pulse 2 + time 2 + 2 t_pulse time ) ⎟⎠ ⎟⎠/τ
⎝ t_pulse ⎠
+ A Rc t_pulse ⎝ 2 t_pulse − e
2

where it can be copied and pasted into EES:

Tt := (-time*T+T_c*(tau+time)+A*Rc*t_pulse*(2*t_pulse^2-exp(-time/t_pulse)*(2*t_pulse^2+
time^2+2*t_pulse*time)))/tau

The expression is modified slightly in order to be compatible with EES:

T = (-time*T+T_c*(tau+time)+(2*t_pulse^2-exp(-time/t_pulse)*(2*t_pulse^2+2*t_pulse*time+time^2))&
*t_pulse*Rc*A)/tau
The heat flux from the laser is also calculated:

q_flux=A*time^2*exp(-time/t_pulse)

A parametric table is created that includes the variables time, q_flux, and T. Figure 2 illustrates
the temperature of the laser target and the laser heat flux as a function of time.

Figure 2: Laser target temperature and laser flux as a function of time


Problem 3.4-2 (3-13 in text): Semi-infinite body with an Increasing Surface Temperature
A semi-infinite piece of material with thermal diffusivity α = 1x10-5 m2/s and conductivity k = 1
W/m-K is initially (at t = 0) at Tini = 300 K when the surface temperature (i.e., the temperature at
x = 0) begins to increase linearly according to: Tx =0,t = Tin + β t where β = 1 K/s.
a.) For part (a), do not solve the problem exactly. Rather, use your conceptual knowledge of
how a thermal wave moves through a semi-infinite body in order to obtain an approximate
model for the heat flux at the surface of the surface as a function of time. Plot the
approximate heat flux as a function of time for t = 0 to 5000 s.

The given information is entered in EES:

$UnitSystem SI MASS RAD PA K J


$TABSTOPS 0.2 0.4 0.6 0.8 3.5 in

"Inputs"
alpha=1e-5 [m^2/s] "thermal diffusivity"
k = 1 [W/m-K] "conductivity of material"
T_in=300 [K]
"initial temperature of material "
beta=1 [K/s]
"rate of change of the surface temperature"

The approximate model of the heat flux at the surface treats the thermally affected region of the
semi-infinite body (of width 2 α t ) as a thermal resistance. Therefore:

q ′′x =0,t ≈
(T x = 0,t − Tin )
(1)
2 αt

or, substituting Eq. Error! Reference source not found. into Eq. (1):

β t
q ′′x =0,t ≈ (2)
2 α

qfs_app=k*beta*sqrt(time/alpha)/2 "approximate heat flux"

Figure 1 illustrates the approximate heat flux at the surface as a function of time.
Figure 1: Approximate and exact solution for the heat flux at the surface of the semi-infinite body.

b.) Use the Laplace transform technique to obtain an analytical solution to this problem.
Implement your solution in EES and prepare a plot showing the temperature as a function of
time (for t = 0 to 5000 s) at locations x = 0, 0.1 m, 0.2 m, and 0.3 m. Prepare another plot
showing the temperature as a function of position (for x = 0 to 0.5 m) for t = 0 s, 300 s, 600 s,
and 900 s.

The definition of the problem in the time domain is:

∂ 2T ∂T
α = (3)
∂x 2 ∂t

with boundary conditions:

Tx =0,t = Tin + β t (4)

Tx →∞ ,t = Tin (5)

Tx ,t =0 = Tin (6)

The governing differential equation is transformed from the x, t domain to the x, s domain using
the Laplace transform.

∂2 T 
α 2 = sT − Tin (7)
∂x
T β
Tˆx =0 = in + 2 (8)
s s

T
Tˆx →∞ = in (9)
s

The same result can be obtained using Maple:

> restart;
> with(inttrans):read(`InvLaplaceTable.m`);
> GDE:=alpha*diff(diff(T(x,t),x),x)=diff(T(x,t),t);
2
⎛∂ ⎞ ∂
GDE := α ⎜ 2 T( x, t ) ⎟⎟ = T( x, t )
⎜ ∂x
⎝ ⎠ ∂t
> GDEs:=laplace(GDE,t,s);
2
⎛∂ ⎞
GDEs := α ⎜⎜ 2 laplace( T( x, t ), t, s ) ⎟⎟ = s laplace( T( x, t ), t, s ) − T( x, 0 )
⎝ ∂x ⎠
> GDEs:=subs(laplace(T(x,t),t,s)=Th(x),GDEs);
2
⎛d ⎞
GDEs := α ⎜ 2 Th( x ) ⎟⎟ = s Th( x ) − T( x, 0 )
⎜ dx
⎝ ⎠
> GDEs:=subs(T(x,0)=T_in,GDEs);
2
⎛d ⎞
GDEs := α ⎜⎜ 2 Th( x ) ⎟⎟ = s Th( x ) − T_in
⎝ dx ⎠
> T_sur_t:=T_in+beta*t;
T_sur_t := T_in + β t
> T_sur_s:=laplace(T_sur_t,t,s);
T_in s + β
T_sur_s :=
s2

The transformed problem is solved in the s domain. Rearranging Eq. (7) and recognizing that it
involves derivatives only of x leads to:

d2T s  T
− T = − in (10)
dx 2
α α

which can be broken into a homogeneous ( û ) and particular ( v̂ ) part. The homogeneous
solution must solve:

d 2 uˆ s
− uˆ = 0 (11)
dx 2 α

which is solved by exponentials (or equivalently sinh and cosh); because there is a boundary
condition at x → ∞, it is natural to represent the solution with exponentials:
⎛ s ⎞ ⎛ s ⎞
uˆ = C1 exp ⎜⎜ x ⎟⎟ + C2 exp ⎜⎜ − x ⎟ (12)
⎝ α⎠ ⎝ α ⎟⎠

The particular solution must solve:

d 2 vˆ s T
− vˆ = − in (13)
dx 2
α α

By inspection , the particular solution is:

Tin
vˆ = (14)
s

and the solution is:

⎛ s ⎞ ⎛ s ⎞ Tin
Tˆ = C1 exp ⎜⎜ x ⎟⎟ + C2 exp ⎜⎜ − x ⎟+ (15)
⎝ α⎠ ⎝ α ⎟⎠ s

The same answer can be achieved using Maple:

> Ts:=dsolve(GDEs);
⎛ s x⎞ ⎛ s x⎞
⎜ ⎟ ⎜− ⎟
⎜ α ⎟ ⎜ α ⎟⎠ T_in
⎝ ⎠ ⎝
Ts := Th( x ) = e _C2 + e _C1 +
s

Applying the boundary condition at x → ∞, Eq. (9), leads to:

Tin ⎛ s ⎞ ⎛ s ⎞ Tin
= C1 exp ⎜⎜ ∞ ⎟⎟ + C2 exp ⎜⎜ −∞ ⎟+ (16)
s ⎝ α⎠ ⎝ α ⎟⎠ s
or

⎛ s ⎞
0 = C1 exp ⎜⎜ ∞ ⎟ (17)
⎝ α ⎟⎠

which can only be true if C1 = 0:

⎛ s ⎞ Tin
Tˆ = C2 exp ⎜⎜ − x ⎟⎟ + (18)
⎝ α ⎠ s

> Ts:=subs(_C2=0,Ts);
⎛ s x⎞
⎜− ⎟
⎜ α ⎟⎠ T_in

Ts := Th( x ) = e _C1 +
s

Applying the boundary condition at x = 0, Eq. (8), leads to:

Tin β T
+ 2 = C2 + in (19)
s s s

which leads to:

β
C2 = (20)
s2

and the solution in the s domain is:

β ⎛ s ⎞ Tin
Tˆ = 2 exp ⎜⎜ − x ⎟+ (21)
s ⎝ α ⎟⎠ s

> Ts:=subs(_C1=solve(rhs(eval(Ts,x=0))=T_sur_s,_C1),Ts);
⎛ s x⎞
⎜− ⎟
⎜ α ⎟⎠

e β T_in
Ts := Th( x ) = 2
+
s s

Equation (21) can be transformed back to the time domain by inspection of the transforms listed
in the table; specifically, the inverse Laplace transform of:

(
exp −C s ) (22)
s2

is

⎛ C2 ⎞ ⎛ C ⎞ C t ⎛ C2 ⎞
⎜t + ⎟ erfc ⎜ ⎟− exp ⎜ − ⎟ (23)
⎝ 2 ⎠ ⎝2 t ⎠ π ⎝ 4t ⎠

Replacing C with x / α leads to:

⎛ x2 ⎞ ⎛ x ⎞ βx t ⎛ x2 ⎞
Tˆ = β ⎜ t + ⎟ erfc ⎜ ⎟ − exp ⎜− ⎟ + Tin (24)
⎝ 2α ⎠ ⎜2 αt ⎟ π α ⎝ 4 α t ⎠
⎝ ⎠

> Tt:=invlaplace(rhs(Ts),s,t);
⎛ 2 ⎞
erfc⎛⎜ ⎞ β ( 2 t α + x2 )
⎜− x ⎟ x
⎜⎜ ⎟⎟
⎝ 4α t⎠ ⎜2 tα ⎟⎟
x te β 1 ⎝ ⎠
Tt := − + T_in +
α π 2 α

Either the solution obtained manually, Eq. (24), or the one from Maple can be implemented in
EES:

T1=-1/alpha^(1/2)*x*time^(1/2)*exp(-1/4/alpha*x^2/time)/pi^(1/2)*beta+T_in&
+1/2*erfc(1/2/(time*alpha)^(1/2)*x)*beta*(2*time*alpha+x^2)/alpha
"solution from Maple"
T2=beta*(time+x^2/(2*alpha))*erfc(x/(2*sqrt(alpha*time)))-beta*x*&
sqrt(time/(pi*alpha))*exp(-x^2/(4*alpha*time))+T_in
"solution from table"

The variables T1 and T2 are identical when evaluated at the same value of time and position.
Figures 2 and 3 are the plots requested in the problem statement.

Figure 2: Temperature as a function of time for various values of position.


Figure 3: Temperature as a function of position for various values of time.

c.) Overlay your exact solution onto the plot of your approximate solution from (a).

The heat flux at the surface can be obtained from the temperature solution according to:

∂T
q ′′x =0,t = − k (25)
∂x x = 0,t

which is evaluated using Maple:

> qf_s:=-k*eval(diff(Tt,x),x=0);
⎛ tβ βt ⎞
qf_s := −k ⎜ − − ⎟

⎜ α π π tα
⎝ ⎠

and copied into EES:

qfs_exact=-k*(-1/alpha^(1/2)*time^(1/2)/pi^(1/2)*beta-1/Pi^(1/2)/(time*alpha)^(1/2)*beta*time)

The exact solution is overlaid on the plot in (a); note that the exact and approximate solutions
differ by a constant factor of 4 / π ; this implies that the thermal penetration depth is always
about 50% as large as would be predicted by our simple model, perhaps because the temperature
at the surface is always rising and therefore the true time relative to the perturbation at the
surface is not really t.
Problem 3.4-3 (3-14 in text): Radiant Heating (revisited)
Solve Problem 3.3-5 (3-8 in text) using the Laplace transform technique for the period of time
where the material can be treated as a semi-infinite body.
a.) Prepare a solution and implement your solution in EES. Plot the temperature as a function of
position for several times.

The inputs are entered in EES:

$UnitSystem SI MASS RAD PA C J


$Tabstops 0.2 0.4 0.6 0.8 3.5

"Inputs"
k=2.4 [W/m-K] "conductivity"
alpha=2.2e-4 [m^2/s] "thermal diffusivity"
h_bar=15 [W/m^2-K] "heat transfer coefficient"
q``_s=7500 [W/m^2] "heat flux"
L=5 [cm]*convert(cm,m) "thickness"
A_c=1 [m^2] "per unit area"
T_ini=converttemp(C,K,20 [C]) "initial temperature"
T_infinity=converttemp(C,K,20 [C]) "ambient temperature"

The governing partial differential equation for the problem is:

∂ 2T ∂T
α 2 = (1)
∂x ∂t

where α is the thermal diffusivity. The boundary conditions are provided by:

∂T
qs′′ = −k + h (Tx =0 − T∞ ) (2)
∂x x =0

Tt =0 = Tini (3)

Tx→∞ = Tini (4)

The governing differential equation is transformed from the x, t domain to the x, s. The first term
in Eq. (1) is transformed:

∂ 2T ( x, t ) ∂ 2 T ( x, s )
α =α (5)
∂x 2 ∂x 2

The second term is transformed:

∂T ( x, t ) 
= sT ( x, s ) − Tini (6)
∂t
The transformed differential equation is:

∂ 2 T ( x, s ) 
α = sT ( x, s ) − Tini (7)
∂x 2

Equation (7) does not involve any derivative with respect to s and therefore it is an ordinary
differential equation in x; the partial differential in Eq. (7) can be changed to an ordinary
differential:

d 2 T ( s, x ) 
α 2
= sT ( s, x ) − Tini (8)
dx

which can be rearranged:



d 2T s  T
− T = − ini (9)
dx 2
α α

Equation (9) is a second order, non-homogeneous equation and therefore requires two boundary
conditions; these are obtained from Eqs. (2) and (4) which must also be transformed to the s
domain.

qs′′ dT ⎛ T ⎞
= −k + h ⎜ Tx =0 − ∞ ⎟ (10)
s dx x =0 ⎝ s ⎠

 T
Tx →∞ = ini (11)
s

The second order differential equation is split into homogeneous and particular components:
  
T = Th + Tp (12)

Equation (12) substituted into Eq. (9):


 2

d 2Th s  d Tp s  T
− Th + − Tp = − ini (13)
dx 2

α 2
dx
 α α

= 0 for homogeneous particular differential equation


differential equation

The homogeneous differential equation is:



d 2Th s 
− Th = 0 (14)
dx 2 α
which has the general solution:

⎛ s ⎞ ⎛ s ⎞
Th = C1 exp ⎜⎜ x ⎟⎟ + C2 exp ⎜⎜ − x ⎟⎟ (15)
⎝ α ⎠ ⎝ α ⎠

where C1 and C2 are undetermined constants. The solution to the particular differential equation

d 2Tp s  T
− Tp = − ini (16)
dx 2 α α

is, by inspection:

 T
Tp = ini (17)
s

Substituting Eqs. (15) and (17) into Eq. (12) leads to:

 ⎛ s ⎞ ⎛ s ⎞ Tini
T = C1 exp ⎜⎜ x ⎟⎟ + C2 exp ⎜⎜ − x ⎟⎟ + (18)
⎝ α ⎠ ⎝ α ⎠ s

The constants C1 and C2 are obtained from the boundary conditions. The boundary condition at
x→ ∞, Eq. (11), leads to:

 ⎛ s ⎞ ⎛ s ⎞ Tini Tini
Tx →∞ = C1 exp ⎜⎜ ∞ ⎟⎟ + C2 exp ⎜⎜ − ∞ ⎟⎟ + = (19)
⎝ α ⎠ ⎝ α ⎠ s s

or

⎛ s ⎞
C1 exp ⎜⎜ ∞ ⎟⎟ = 0 (20)
⎝ α ⎠

which can only be true if C1 = 0; therefore:

 ⎛ s ⎞ Tini
T = C2 exp ⎜⎜ − x ⎟⎟ + (21)
⎝ α ⎠ s

The boundary condition at x = 0, Eq. (10), leads to:

qs′′ s ⎛ T T ⎞
= C2 k + h ⎜ C2 + ini − ∞ ⎟ (22)
s α ⎝ s s ⎠
or

⎡ qs′′ + h (T∞ − Tini ) ⎤⎦


C2 = ⎣ (23)
⎛ s ⎞
s⎜k +h⎟
⎝ α ⎠

Substituting Eq. (23) into Eq. (21) leads to the solution to the problem in the x, s domain:

 ⎡ qs′′ + h (T∞ − Tini ) ⎤⎦ ⎛ s ⎞ Tini


T=⎣ exp ⎜⎜ − x⎟ + (24)
⎛ s ⎞ ⎝ α ⎟⎠ s
s⎜k +h⎟
⎝ α ⎠

The solution in the x, t domain can be obtained using the inverse Laplace transforms contained in
Table 3-3; Eq. (24) is rearranged to make this process clearer:

P a
⎛ P C

h ⎜ 1 ⎟
α exp ⎜ − x s⎟
k ⎜ α ⎟
 ⎡ qs′′ ⎤ ⎝ ⎠ + Tini
T = ⎢ + (T∞ − Tini ) ⎥ (25)
⎣h ⎦ ⎛ ⎞ s

s⎜
h
α + s⎟ ⎟
⎜Nk ⎟
 ⎝ a

a exp( − C s )
s( a + s )

The inverse Laplace transform of Eq. (25) is:

⎡ q ′′ ⎤⎡ ⎛ C ⎞ ⎛ ⎞⎤
⎟ − exp ( a C ) exp ( a t ) erfc ⎜ a t +
C
T = ⎢ s + (T∞ − Tini ) ⎥ ⎢erfc ⎜ 2
⎟ ⎥ + Tini (26)
⎣h ⎦ ⎢⎣ ⎝2 t ⎠ ⎝ 2 t ⎠ ⎥⎦

where

h
a= α (27)
k

1
C=x (28)
α

The solution is programmed in EES:


a=h_bar*sqrt(alpha)/k
C=x/sqrt(alpha)
T=(q``_s/h_bar+T_infinity-T_ini)*(erfc(C/(2*sqrt(time)))-&
exp(a*C)*exp(a^2*time)*erfc(a*sqrt(time)+C/(2*sqrt(time))))+T_ini

and evaluated a particular position and time:

time=2 [s] "time"


x_bar=1 [-] "dimensionless position"
x=x_bar*L "position"
T_C=converttemp(K,C,T) "solution converted to C"

b.) Compare the analytical solution obtained in (a) to the approximate model that you derived in
(c) of Problem 3.3-5 (3-8).

Figure P3.4-3-1 illustrates the temperature as a function of position for various values of time.
Notice that the qualitative characteristics agree with those expected in Problem P3.3-5; the
absolute value of the temperature gradient at x = 0 decreases with time and the thermal wave
moves through the solid.
90

80

70
Temperature (°C)

60
2s
50
1s
40
0.5 s
30
0.25 s
20
0 0.01 0.02 0.03 0.04 0.05
Position (m)
Figure P3.4-3-1: Temperature as a function of position at various times.

Figure P3.4-3-2 illustrates the temperature at the surface of the slab (x = 0) as a function of time
as well as the approximate solution from part (c) of Problem 3.3-5 (3-8).
140
approximate model, P3.3-5(c)
120
Temperature (°C)

100

80
exact model, P3.4-3
60

40

20
0 0.5 1 1.5 2 2.5 3
Time (s)
Figure P3.4-3-2: Temperature as a function of position at various times.
Problem 3.4-5 (3-15 in text)
A sphere with radius R = 1 mm is composed of material with density ρ = 9000 kg/m3, specific
heat capacity c = 500 J/kg-K, and conductivity k = 25 W/m-K. The surface is exposed to fluid at
T∞ = 25°C with heat transfer coefficient h = 1000 W/m2-K. The sphere is initially in
equilibrium with the fluid when it experiences a time varying volumetric generation of thermal
energy:
′′′ exp ⎛⎜ - ⎞⎟
t
g ′′′ = g max
⎝ a⎠
′′′ = 1x10 W/m and a = 2 s.
where g max 9 3

a.) Is a lumped capacitance solution appropriate for this problem? Justify your answer.

The inputs are entered in EES:

$UnitSystem SI MASS RAD PA K J


$TABSTOPS 0.2 0.4 0.6 0.8 3.5 in

"Inputs"
R=1 [mm]*convert(mm,m) "radius of sphere"
rho=9000 [kg/m^3] "density"
c=500 [J/kg-K] "specific heat capacity"
k=25 [W/m-K] "thermal conductivity"
T_infinity=converttemp(C,K,25[C]) "ambient temperature"
h_bar=1000 [W/m^2-K] "heat transfer coefficient"
gv_max=1e9 [W/m^3] "maximum volumetric generation"
a=2 [s] "time constant of generation"

The volume of the sensor is:

4 π R3
V= (1)
3

and the surface area is:

As = 4 π R 2 (2)

The Biot number is therefore:

Vh
Bi = (3)
As k

V=4*pi*R^3/3 "volume"
A_s=4*pi*R^2 "surface area"
Bi=V*h_bar/(A_s*k) "Biot number"

which leads to Bi = 0.013; this is sufficiently less than one to justify the lumped capacitance
model.
b.) Assume that your answer to (a) is yes. Determine an expression for the temperature as
function of time using the Laplace transform technique and implement this solution in EES.
Plot temperature as a function of time.

A control volume defined around the sphere leads to:

dU
g = qconv + (4)
dt

The rate of convection is:

qconv = h As (T − T∞ ) (5)

The rate of energy storage is:

dU dT
= ρV c (6)
dt dt

The rate of thermal energy generation is:

′′′ exp ⎛⎜ - ⎞⎟
t
g = V g max (7)
⎝ a⎠

Substituting Eqs. (5) through (7) into Eq. (4) leads to:

′′′ exp ⎛⎜ - ⎞⎟ = h As (T − T∞ ) + ρ V c
t dT
V g max (8)
⎝ a⎠ dt

or:

′′′
dT T T∞ g max ⎛ t⎞
+ = + exp ⎜ - ⎟ (9)
dt τ τ ρc ⎝ a⎠

where the time constant, τ, of the sphere is:

ρV c
τ= (10)
h As

tau=rho*V*c/(h_bar*A_s) "time constant"

The governing differential equation is transformed from the t domain to the s domain.

 T ( s ) T∞ ′′′
g max
sT ( s ) − Tini + = + (11)
τ τs ⎛ 1⎞
ρ c⎜s + ⎟
⎝ a⎠

Equation (11) is solved to obtain the solution in the s domain:

 ⎛ 1⎞ T ′′′
g max
T (s)⎜ s + ⎟ = ∞ + + Tini (12)
⎝ τ ⎠ τ s ρ c⎛ s + 1 ⎞
⎜ ⎟
⎝ a⎠

or

⎛ 1⎞
′′′ τ s + Tini ρ c τ s ⎛⎜ s + ⎞⎟
1
T∞ ρ c ⎜ s + ⎟ + g max
 ⎝ a⎠ ⎝ a⎠
T (s) = (13)
⎛ 1 ⎞⎛ 1⎞
ρ cτ s ⎜ s + ⎟ ⎜ s + ⎟
⎝ a ⎠⎝ τ ⎠

Using the method of partial fractions:

⎛ 1⎞
′′′ τ s + Tini ρ c τ s ⎛⎜ s + ⎞⎟
1
T∞ ρ c ⎜ s + ⎟ + g max
 ⎝ a⎠ ⎝ a ⎠ C1 C2 C3
T (s) = = + + (14)
⎛ 1 ⎞⎛ 1⎞ s ⎛ 1⎞ ⎛ 1⎞
ρ cτ s ⎜ s + ⎟ ⎜ s + ⎟ ⎜s+ ⎟ ⎜s+ ⎟
⎝ a ⎠⎝ τ ⎠ ⎝ a⎠ ⎝ τ ⎠

Multiplying the top and bottom of Eq. (14) through by the denomenator leads to:

⎛ 1⎞
′′′ τ s + Tini ρ c τ s ⎛⎜ s + ⎞⎟ =
1
T∞ ρ c ⎜ s + ⎟ + g max
⎝ a⎠ ⎝ a⎠
(15)
⎛ 1 ⎞⎛ 1⎞ ⎛ 1⎞ ⎛ 1⎞
C1 ρ c τ ⎜ s + ⎟ ⎜ s + ⎟ + C2 ρ c τ s ⎜ s + ⎟ + C3 ρ c τ s ⎜ s + ⎟
⎝ a ⎠⎝ τ ⎠ ⎝ τ⎠ ⎝ a⎠

or:

⎛ T ρ c τ ⎞ T∞ ρ c
Tini ρ c τ s 2 + ⎜ T∞ ρ c + g max
′′′ τ + ini ⎟s+ =
⎝ a ⎠ a
(16)
⎛ ⎛1 1⎞ 1 ⎞ ⎛ 2 s⎞ ⎛ 2 s⎞
C1 ρ c τ ⎜ s 2 + ⎜ + ⎟ s + ⎟ + C2 ρ c τ ⎜ s + ⎟ + C3 ρ c τ ⎜ s + ⎟
⎝ ⎝a τ ⎠ aτ ⎠ ⎝ τ⎠ ⎝ a⎠

Examining the s2 term in Eq. (16) leads to:

Tini = C1 + C2 + C3 (17)
Examining the s1 term in Eq. (16) leads to:

′′′ τ Tini τ
g max ⎛1 1⎞ τ
T∞ + + = C1 τ ⎜ + ⎟ + C2 + C3 (18)
ρc a ⎝a τ ⎠ a

Examining the s0 term in Eq. (16) leads to:

T∞ = C1 (19)

Equations (17) through (19) are solved in EES:

"constants"
C_1=T_infinity
T_infinity+gv_max*tau/(rho*c)+T_ini*tau/a=C_1*tau*(1/a+1/tau)+C_2+C_3*tau/a
T_ini=C_1+C_2+C_3

Taking the inverse transform of Eq. (14) leads to:

⎛ t⎞ ⎛ t⎞
T ( t ) = C1 + C2 exp ⎜ − ⎟ + C3 exp ⎜ − ⎟ (20)
⎝ a⎠ ⎝ τ⎠

"solution"
time=0 [s]
T=C_1+C_2*exp(-time/a)+C_3*exp(-time/tau)
T_C=converttemp(K,C,T)

Figure P3.4-5 illustrates the temperature as a function of time.


180

160

140
Temperature (°C)

120

100

80

60

40

20
0 1 2 3 4 5 6 7 8 9 10
Time (s)
Figure P3.4-5: Temperature as a function of time.
Problem 3.5-1 (3-16 in text): Ice Cream Containers
Ice cream containers are removed from a warehouse and loaded into a refrigerated truck. During
this loading process, the ice cream may sit on the dock for a substantial amount of time. The
dock temperature is substantially higher than the warehouse temperature, which can cause two
problems. First, the temperature of the ice cream near the surface can become elevated, resulting
in a loss of food quality. Second, the energy absorbed by the ice cream on the dock must
subsequently be removed by the equipment on the refrigerated truck, causing a substantial load
on this relatively under-sized and inefficient equipment. The ice cream is placed in cylindrical
cardboard containers. Assume that the containers are very long and therefore, the temperature
distribution of the ice cream is one dimensional), as shown in Figure P3.5-1. The inner radius of
the cardboard ice cream containers is Ro = 10 cm and the thickness of the wall is thcb = 2.0 mm.
The conductivity of cardboard is kcb = 0.08 W/m-K. The ice cream comes out of the warehouse
at Tini = 0°F and is exposed to the dock air at Tdock = 45°F with heat transfer coefficient, h = 20
W/m2-K. The ice cream has properties kic = 0.2 W/m-K, ρic = 720 kg/m3, and cic = 3200 J/kg-K.
(Assume that the ice cream does not melt).

thcb = 2 mm
Tdock = 45°F
h = 20 W/m -K
2
ice cream
kic = 0.2 W/m-K
ρic = 720 kg/m3
cic = 3200 J/kg-K

cardboard, kcb = 0.08 W/m-K

Ro = 10 cm
Figure P3.5-1: Ice cream containers
a.) Determine an effective heat transfer coefficient, heff , that can be used in conjunction with the
analytical solutions for a cylinder subjected to a step change in fluid temperature but includes
the conduction resistance associated with the cardboard as well as the convection to the air.

The inputs are entered in EES:

$UnitSystem SI MASS RAD PA K J


$TABSTOPS 0.2 0.4 0.6 0.8 3.5 in

"Inputs"
R_o=10 [cm]*convert(cm,m) "radius of container"
th_cb=2.0 [mm]*convert(mm,m) "thickness of cardboard"
k_cb=0.08 [W/m-K] "conductivity of cardboard"
k_ic=0.2 [W/m-K] "conductivity of ice cream"
rho_ic=720 [kg/m^3] "density of ice cream"
c_ic=3200 [J/kg-K] "specific heat capacity of ice cream"
h=20 [W/m^2-K] "heat transfer coefficient"
T_in=converttemp(F,K,0) "storage temperature"
T_dock=converttemp(F,K,45) "dock temperature"
L = 1 [m] "per unit length"
The thermal resistance between the surface of the ice cream and the dock air is related to
convection:

1
Rconv = (1)
h 2 π Ro L

and conduction through the cardboard:

thcb
Rcond ,cb = (2)
kcb 2 π Ro L

The effective heat transfer resistance is defined to provide the same total resistance:

1
= Rconv + Rcond ,cb (3)
heff 2 π Ro L

R_conv=1/(h*2*pi*(R_o+th_cb)*L) "convection resistance"


R_cond_cb=th_cb/(2*pi*R_o*L*k_cb) "conduction resistance of cardboard"
1/(h_eff*2*pi*R_o*L)=R_conv+R_cond_cb "effective heat transfer coefficient"

which leads to heff = 13.5 W/m2-K.

b.) If the ice cream remains on the dock for tload = 5 minutes, what will the temperature of the
surface of the ice cream be when it is loaded?

The built-in EES function for a cylinder subjected to a step change in fluid temperature is used:

T_s_load=cylinder_T(R_o, time_load, T_in, T_dock, alpha_ic, k_ic, h_eff, R_o)


"surface temperature upon loading"

which leads to Ts,load = 262.9 K (13.5°F).

c.) How much energy must be removed from the ice cream (per unit length of container) after it
is loaded in order to bring it back to a uniform temperature of Tini = 0°F?

The energy provided to the ice cream is given by the integral of the heat transfer rate to the ice
cream surface over time:
timeload

Q= ∫ 2 π Ro L heff ⎡⎣Tdock − T ( r = Ro , t ) ⎤⎦ dt (4)


0

The integrand is evaluated at a specific value of time:

time=100 [s] "time to evaluate integrand"


T_s=cylinder_T(R_o, time, T_in, T_dock, alpha_ic, k_ic, h_eff, R_o)
"instantaneous surface temperature"
q_dot=2*pi*R_o*L*h_eff*(T_dock-T_s) "instantaneous heat transfer rate to the ice cream"

The specific value of time is commented out and the INTEGRAL command is used to evaluate
the integral in Eq. (4):

{time=100 [s]} "time to evaluate integrand"


T_s=cylinder_T(R_o, time, T_in, T_dock, alpha_ic, k_ic, h_eff, R_o)
"instantaneous surface temperature"
q_dot=2*pi*R_o*L*h_eff*(T_dock-T_s) "instantaneous heat transfer rate to the ice cream"
Q=INTEGRAL(q_dot,time,0,time_load) "total heat transferred to the ice cream"

which leads to Q = 49,900 J.

d.) What is the maximum amount of time that the ice cream can sit on the dock before the ice
cream at the outer surface begins to melt?

The EES' internal function for the analytical solution is employed again; this time, the
temperature is known and the time is solved for:

T_s_max=converttemp(F,K,32.2) "maximum allowable surface


temperature"
T_s_max=cylinder_T(R_o, time_load_max, T_in, T_dock, alpha_ic, k_ic, h_eff, R_o)
"surface temperature upon loading"
time_load_max_min=time_load_max*convert(s,min) "maximum allowable loading time (min)"

which leads to timemax = 5740 s (96 min).


Problem 3.5-2 (3-17 in text)
A wall is exposed to a heat flux for a long time, as shown in Figure P3.5-2. The left side of the
wall is exposed to liquid at Tf = 20°C with a very high heat transfer coefficient; therefore, the left
side of the wall (Tx=0) always has the temperature Tf. The right side of the wall is exposed to the
heat flux and also convects to gas at Tf = 20°C but with a heat transfer coefficient of h = 5000
W/m2-K. The wall is L = 0.5 m thick and composed of a material with k = 1.0 W/m-K, ρ = 4000
kg/m3, and c = 700 J/kg-K. The wall is initially at steady state with the heat flux when, at time t
= 0, the heat flux is suddenly shut off. The wall subsequently equilibrates with the liquid and
gas, eventually it reaches a uniform temperature equal to Tf.

k = 1 W/m-K T f = 20°C
ρ = 4000 kg/m3
h = 5000 W/m -K
2
c = 700 J/kg-K

q ′′ = 5x10 W/m
5 2
T f = 20°C

x
L = 0.5 m
Figure 3.5-2: Wall exposed to a heat flux.

a.) Calculate the temperature of the right hand side of the wall at t = 0 (i.e., determine Tx=L,t=0).

The inputs are entered in EES:

$UnitSystem SI MASS RAD PA K J


$TABSTOPS 0.2 0.4 0.6 0.8 3.5 in

"Inputs"
qf_dot=500000 [W/m^2] "heat flux"
k=1 [W/m-K] "conductivity"
rho=4000 [kg/m^3] "density"
c=700 [J/kg-K] "specific heat capacity"
L=0.5 [m] "thickness of wall"
h=5000 [W/m^2-K] "heat transfer coefficient"
T_f=converttemp(C,K,20) "fluid temperature"
A=1 [m^2] "per unit area"

The steady state problem is represented by a heat flow into a thermal resistance network that
consists of two resistors in parallel; Rcond represents the resistance to heat transfer by conduction
through the wall and Rconv represents the resistance to heat transfer by convection from the right
hand surface.

L
Rcond = (1)
kA
1
Rconv = (2)
hA

The total resistance between the right hand surface of the wall and Tf is:
−1
⎡ 1 1 ⎤
Req = ⎢ + ⎥ (3)
⎣ Rcond Rconv ⎦

The temperature of the right hand side of the wall is:

Tx = L ,t =0 = T f + q ′′ A Req (4)

"Initial temperature distribution"


R_cond=L/(k*A) "conduction resistance through wall"
R_conv=1/(h*A) "convection resistance"
R_eq=(1/R_cond+1/R_conv)^(-1) "total resistance"
T_RHS=T_f+qf_dot*A*R_eq "initial temperature of the RHS of wall"

which leads to Tx=L,t=0 = 393.1 K (120°C).

b.) Sketch the temperature distribution at t = 0 and the temperature distribution as t→ ∞.

The initial temperature distribution is consistent with steady state conduction through a plane
wall; it is a linear temperature distribution from Tf at x = 0 to Tx=L,t=0 at x = L. The temperature
distribution as t→ ∞ is uniform at Tf. These are shown in Figure 2.

Figure 2: Sketch of temperature distribution at various times during the equilibration process.
c.) Sketch the temperature distribution at t = 10 s, 100 s, 1x103 s, 1x104 s, and 1x105 s. Justify
the shape of these sketches by calculating the characteristic time scales that govern the
equilibration process.

The time constant associated with the internal equilibration of the wall by conduction is:

( L / 2)
2

τ int = (5)

where

k
α= (6)
ρc

The time constant associated with the external equilibration of the wall with the surrounding gas
is:

τ ext = ρ c L A Rconv (7)

which leads to τint = 44,000 s and τext = 280 s. Based on these time scales, I would expect that
the right edge would equilibrate with the surrounding gas almost immediately but that internal
equilibration would take much longer. A thermal wave will emanate from the right edge and
travel through the wall; it will take ~100,000 s for the plane wall to finally equilibrate all the way
to its center. The intermediate sketches shown in Figure 2 are consistent with these time scales.

d.) Prepare an analytical solution for the equilibration process using separation of variables.
Implement your solution in EES and prepare a plot showing temperature as a function of
position at the times requested in part (c).

The governing partial differential equation for this situation is:

∂T ∂ 2T
−α 2 = 0 (8)
∂t ∂x

The boundary conditions are:

Tx ,t =0 = T f + (Tx = L ,t =0 − T f ) Lx (9)

Tx =0,t = T f (10)

∂T
−k = h ⎡⎣Tx = L ,t − T f ⎤⎦ (11)
∂x x = L ,t
It is possible to transform the problem so that we obtain the necessary homogeneous boundary
conditions in space:

θ = T − Tf (3-12)

The transformed partial differential equation becomes:

∂ 2θ ∂ θ
α 2 = (3-13)
∂x ∂t

θ x ,t =0 = (Tx = L ,t =0 − T f )
x
(14)
L

θ x =0,t = 0 (15)

∂θ
−k = h θ x = L ,t (16)
∂x x = L ,t

The separation of variables approach assumes that the solution can be expressed as the product
of a function only of time, Tt(t), and a function only of position, TX(x):

θ ( x, t ) = TX ( x ) Tt ( t ) (17)

which leads to the two ordinary differential equations:

d 2TX
2
+ β 2 TX = 0 (18)
dx

dTt
+ β 2 α Tt = 0 (19)
dt

The general solution to the eigenproblem is:

TX = C1 sin ( β x ) + C2 cos ( β x ) (20)

The constants are selected to satisfy the boundary conditions on TX. Substituting Eq. (20) into
Eq. (15) leads to:

C1 sin ( 0 ) + C2 cos ( 0 ) = 0 (21)

and therefore C2 = 0. Substituting C2 = 0 into Eq. (20)


TX = C1 sin ( β x ) (22)

Substituting Eq. (22) into Eq. (16) leads to:

− k C1 β cos ( β L ) = h C1 sin ( β L ) (23)

which provides the eigencondition for the problem:


tan ( β L ) = − (24)
h

or

βL
tan ( β L ) = − (25)
Bi

where Bi is the Biot number:

hL
Bi = (26)
k

Figure 3 illustrates the left and right sides of Eq. (25) as a function of β L for the case where Bi =
1.0.

Figure 3: The left and right sides of the eigencondition equation; the intersections
correspond to eigenvalues for the problem, βi L.
The intersections in Figure 3 correspond to the eigenvalues, βi L (where i = 1, 2, ...); there are an
infinite number of eigenvalues and therefore an infinite number of solutions TXi:

TX i = C1,i sin ( β i x ) (27)

The eigencondition, Eq. (25), cannot be solved explicitly for the eigenvalues. Figure 3 shows
that each successive value of βi L can be found in a well-defined interval; notice that β1 L lies
between π/2 and π, β2 L lies between 3π/2 and 2π, etc. and this will be true regardless of the
value of the Biot number. The number of terms to use in the solution is specified and an array of
appropriate guess values and upper and lower bounds for each eigenvalue is generated.

"identify eigenvalues"
N_term=10 [-] "number of terms"
duplicate i=1,N_term
lowerlimit[i]=i*pi-pi/2 "lower limit of range"
upperlimit[i]=i*pi "upper limit of range"
guess[i]=i*pi-pi/4 "guess"
end

The eigencondition is also programmed using a duplicate loop:

Bi=h*L/k "Biot number"


duplicate i=1,N_term
tan(betaL[i])=-betaL[i]/Bi "eigenvalue equation"
beta[i]=betaL[i]/L "eigenvalue"
end

The interval for each eigenvalue is controlled by selecting Variable Info from the Options menu.
Deselect the Show array variables check box at the upper left so that the arrays are collapsed to a
single entry and use the guess[], upperlimit[], and lowerlimit[] arrays to control the process of
identifying the eigenvalues in the array betaL[].

There is an ODE for Tt corresponding to each eigenvalue:

dTt
+ βi2 α Tt = 0 (28)
dt

which has the solution:

Tti = C3,i exp ( − β i2 α t ) (29)

The solution to the problem is therefore:


θ ( x, t ) = ∑ Ci sin ( βi x ) exp ( − βi2 α t ) (30)
i =1

The undetermined constants are determined by applying the initial condition, Eq. (14):

x
∑ C sin ( β x ) = T
i =1
i i x = L ,t = 0
L
(31)

Each side of Eq. (31) is multiplied by an arbitrary eigenfunction, sin(βj x), and integrated from
x=0 to x=L:

∞ L L

∑ C ∫ sin ( β x ) sin ( β x ) dx = T ∫ L sin ( β x ) dx


x
i i j x = L ,t = 0 j (32)
i =1 0 0

The property of orthogonality ensures that the only term on the left side of Eq. (33) that is not
zero is the one for which j=i:
L L
x
Ci ∫ sin ( β i x ) dx = Tx = L ,t =0 ∫ sin ( β i x ) dx
2
(34)
L

0


0

int1 int 2

The integrals can be evaluated conveniently using Maple:

> restart;
> int1:=int((sin(beta[i]*x))^2,x=0..L);
1 −cos( β i L ) sin( β i L ) + β i L
int1 :=
2 βi

> int2:=int(x*sin(beta[i]*x)/L,x=0..L);
−sin( β i L ) + cos( β i L ) β i L
int2 := − 2
L βi

The result is used to evaluate each constant and the solution in EES:

x=0.1 [m] "position"


time=10 [s] "time"
duplicate i=1,N_term
int1[i]=1/2*(-cos(beta[i]*L)*sin(beta[i]*L)+beta[i]*L)/beta[i]
int2[i]=-(-sin(beta[i]*L)+cos(beta[i]*L)*beta[i]*L)/L/beta[i]^2
C[i]=(T_RHS-T_f)*int2[i]/int1[i]
theta[i]=C[i]*sin(beta[i]*x)*exp(-beta[i]^2*alpha*time)
end
T=T_f+sum(theta[1..N_term])

Figure 4 illustrates the temperature as a function of position at the times requested in the
problem.
Figure 4: Temperature as a function of position at various values of time
Problem 3.5-4 (3-18 in text): Current Lead Deactivation
A current lead carries 1000's of amps of current to a superconducting magnet, as shown in Figure
P3.5-4.
g ′′′ = 3x10 W/m
5 3

h f = 1000 W/m -K
2
h f = 1000 W/m -K
2

T∞ = 20°C T∞ = 20°C

L = 10 cm
ρ = 8000 kg/m3 x
k = 10 W/m-K
c = 700 J/kg-K
Figure P3.5-4: Current lead.

The edges of the current lead are cooled by flowing water at T∞ = 20ºC with heat transfer
coefficient h f = 1000 W/m2-K. The current lead material has density ρ = 8000 kg/m3,
conductivity k = 10 W/m-K, and specific heat capacity c = 700 J/kg-K. The current causes a
uniform rate of volumetric generation of thermal energy, g ′′′ = 3x105 W/m3. The half-width of
the current lead is L = 10 cm.
a.) Determine the steady-state temperature distribution in the current lead, Tss(x). Plot the
temperature distribution.

The inputs are entered in EES:

$UnitSystem SI MASS RAD PA K J


$TABSTOPS 0.2 0.4 0.6 0.8 3.5 in

"Inputs"
gv=3e5 [W/m^3] "volumetric generation"
k=10 [W/m-K] "conductivity"
rho=8000 [kg/m^3] "density"
c=700 [J/kg-K] "specific heat capacity"
L=0.1 [m] "half-thickness of wall"
h_bar_f=1000 [W/m^2-K] "heat transfer coefficient"
T_infinity=converttemp(C,K,20) "fluid temperature"
A=1 [m^2] "per unit area"

The general solution for the 1-D steady-state temperature distribution in a plane wall exposed to
uniform thermal energy generation is provided in Table 1-3:

g ′′′ 2
Tss = − x + C1 x + C2 (1)
2k

dTss g ′′′
=− x + C1 (2)
dx k

The constants C1 and C2 are obtained using the boundary conditions:


dTss
=0 (3)
dx x =0

dTss
−k = h f (Tss , x = L − T∞ ) (4)
dx x=L

Substituting Eqs. (1) and (2) into Eqs. (3) and (4) leads to:

g ′′′ 2 g ′′′ 2 g ′′′ L


Tss = − x + L + + T∞ (5)
2k 2k hf

x_bar=0.1 [-] "dimensionless position"


x=x_bar*L "position"
T_ss=-gv*x^2/(2*k)+gv*L^2/(2*k)+gv*L/h_bar_f+T_infinity "steady-state temperature distribution"
T_ss_C=converttemp(K,C,T_ss) "in C"

The steady-state temperature as a function of position is shown in Figure P3.5-4(b).


140

120
Temperature (°C)

100

80

60

40

20
0 0.01 0.02 0.03 0.04 0.05 0.06 0.07 0.08 0.09 0.1
Position (m)
Figure P3.5-4(b): Steady-state temperature distribution.

At time t = 0 the current is deactivated so that the rate of volumetric generation in the current
lead goes to zero. The cooling water flow is also deactivated at t = 0, causing the heat transfer
coefficient at the surface to be reduced to hs = 100 W/m-K.
b.) Sketch the temperature distribution that you expect within the material at t = 0 s, t = 500 s, t
= 1000 s, t = 5000 s, and t = 10,000 s. Make sure that the qualitative characteristics of your
sketch are correct and justify them if possible.

The time constant associated with the internal equilibration due to conduction is:

L2
τ int ≈ (6)

where α is the thermal diffusivity of the material:

k
α≈ (7)
ρc

The time constant associated with external equilibration with the fluid is:

ρcL
τ ext ≈ (8)
hs

"Time constants"
h_bar_s=100 [W/m^2-K] "heat transfer coefficient after deactivation"
alpha=k/(rho*c) "thermal diffusivity"
tau_int=L^2/(4*alpha) "internal equilibration time constant"
tau_ext=c*rho*L/h_bar_s "external time constant"

which leads to τint = 700 s and τext = 5600 s. Based on these time constants, you should expect
that the material will come to equilibrium internally (i.e., reach a uniform temperature
distribution) much faster than it will come to equilibrium with the cooling water. The sketch in
Figure P3.5-4(c) is consistent with these observations.

Temperature
t=0s
t = 500 s

t = 1000 s
t = 5000 s

t = 10,000 s
T∞ Position
0 L
Figure P3.5-4(c): Sketch of tempeature as a function of position at various values of time.

c.) Sketch the rate of heat transfer per unit area to the cooling water as a function of time. Make
sure that the qualitative characteristics of your sketch are correct and justify them if possible.
Include a rough sense of the scale on the t axis.

Based on the discussion from part (b) and Figure P3.5-4(c), you should expect that initially (i.e.,
for t < τint) the surface temperature will rise as the energy from the center of the current lead
quickly is conducted to the edge. Therefore, for t < τint the rate of heat transfer to the cooling
water will rise. Eventually (i.e., for t > τext) the surface temperature will decay due to the
external equilibration process; this process will be complete at t ~ τext. These characteristics are
reflected in Figure P3.5-4(d).
Heat transfer to cooling water

T∞ τext
Time
0 τint
Figure P3.5-4(d): Sketch of heat transfer to cooling fluid as a function of time.

d.) Develop a separation of variables solution for the process. Prepare the plots requested in
parts (b) and (c) using this model.

The governing partial differential equation for this situation is:

∂T ∂ 2T
−α 2 = 0 (9)
∂t ∂x

The initial condition is the steady state temperature distribution derived in (a):

g ′′′ 2 g ′′′ 2 g ′′′ L


Tt =0 = − x + L + + T∞ (10)
2k 2k hf

At the adiabatic wall, the temperature gradient must be zero:

∂T
=0 (11)
∂x x =0

An interface energy balance at the surface (x = L) balances conduction with convection:

∂T
−k = hs [Tx = L − T∞ ] (12)
∂x x= L

In order to apply separation of variables, it is necessary that the partial differential equation and
both boundary conditions in space be homogeneous. The partial differential equation, Eq. (9) is
homogeneous and the boundary condition associated with the adiabatic wall, Eq. (11), is also
homogeneous. However, the convective boundary condition at x = L, Eq. (12), is not
homogeneous. The temperature difference relative to the fluid temperature is defined:

θ = T − T∞ (13)

The transformed partial differential equation becomes:


∂ 2θ ∂ θ
α = (14)
∂x 2 ∂ t

and the boundary conditions become:

g ′′′ 2 g ′′′ 2 g ′′′ L


θ t =0 = − x + L + (15)
2k 2k hf

∂θ
=0 (16)
∂x x =0

∂θ
−k = h θ x=L (17)
∂x x=L

Notice that both spatial boundary conditions of the transformed problem, Eqs. (16) and (17), are
homogeneous and therefore it will be possible to obtain a set of orthogonal eigenfunctions in x
using the separation of variables technique.

The separation of variables approach assumes that the solution can be expressed as the product
of a function only of time, θt(t), and a function only of position, θX(x):

θ ( x, t ) = θ X ( x ) θ t ( t ) (18)

Substituting Eq. (18) into Eq. (14) leads, eventually, to:

d 2θ X
+ λ2 θ X = 0 (19)
dx 2

and

dθ t
+ λ2 α θt = 0 (20)
dt

The general solution to Eq. (19) is:

θ X = C1 sin ( λ x ) + C2 cos ( λ x ) (21)

where C1 and C2 are unknown constants. Substituting Eq. (18) into the spatial boundary
condition at x = 0, Eq. (16), leads to:
∂θ dθ X ⎡ ⎤
= θt = θ t ⎢C1 λ cos ( λ 0 ) − C2 λ sin ( λ 0 ) ⎥ = 0 (22)
∂x dx ⎢⎣ 


x =0 x =0
=1 =0 ⎦

or C1 = 0:

θ X = C2 cos ( λ x ) (23)

Substituting Eq. (18) into the spatial boundary condition at x = L, Eq. (17), leads to:

dθ X
−k = h θ X x=L (24)
dx x=L

Substituting Eq. (23) into Eq. (24) leads to:

k C2 λ sin ( λ L ) = h C2 cos ( λ L ) (25)

which provides the eigencondition for the problem that defines multiple eigenvalues:

Bi
tan ( λ L ) = (26)
λL

where Bi is the Biot number:

hs L
Bi = (27)
k

As discussed in Section 3.5, successive value of λi L can be found in well-defined intervals. The
number of terms to use in the solution is specified and an array of appropriate guess values and
upper and lower bounds for each eigenvalue is generated.

Nterm=10 [-] "number of terms to use in the solution"


"Setup guess values and lower and upper bounds for eigenvalues"
duplicate i=1,Nterm
lowerlimit[i]=(i-1)*pi
upperlimit[i]=lowerlimit[i]+pi/2
guess[i]=lowerlimit[i]+pi/4
end

The eigencondition is also programmed using a duplicate loop:

Bi=h_bar_s*L/k "Biot number"


"Identify eigenvalues"
duplicate i=1,Nterm
tan(lambdaL[i])=Bi/lambdaL[i] "eigencondition"
lambda[i]=lambdaL[i]/L "eigenvalue"
end

The interval for each eigenvalue is controlled by selecting Variable Info from the Options menu.
Deselect the Show array variables check box at the upper left so that the arrays are collapsed to a
single entry and use the guess[], upperlimit[], and lowerlimit[] arrays to control the process of
identifying the eigenvalues in the array lambdaL[].

At this point, each of the eigenfunctions of the problem have been obtained. The ith
eigenfunction is:

θ X i = C2,i cos ( λi x ) (28)

where λi is the ith eigenvalue, identified by the eigencondition:

Bi
tan ( λi L ) = (29)
λi L

The solution to the non-homogeneous ordinary differential equation corresponding to the ith
eigenvalue, Eq. (20):

dθ ti
+ λi2 α θ ti = 0 (30)
dt

is

θ ti = C3,i exp ( −λi2 α t ) (31)

where C3,i are undetermined constants. According to Eq. (18), the solution associated with the ith
eigenvalue is:

θi = θ X i θ ti = Ci cos ( λi x ) exp ( −λi2 α t ) (32)

where the constants C2,i and C3,i have been combined to a single undetermined constant Ci.
Because the partial differential equation is linear, the sum of the solution θi for each eigenvalue,
Eq. (32), is itself a solution:

∞ ∞
θ = ∑θi = ∑ Ci cos ( λi x ) exp ( −λi2 α t ) (33)
i =1 i =1

The final step of the problem selects the constants so that the series solution satisfies the initial
condition:

g ′′′ 2 g ′′′ 2 g ′′′ L
θt =0 = ∑ Ci cos ( λi x ) = − x + L + (34)
i =1 2k 2k hf

Each side of Eq. (34) is multiplied by cos(λj x) and integrated from x = 0 to x = L. The property
of orthogonality ensures that the only term on the left side of Eq. (34) that is not zero is the one
for which j = i; therefore:

L
g ′′′ 2
L
⎛ g ′′′ 2 g ′′′ L ⎞ L
Ci ∫ cos ( λi x ) dx = − x cos ( λi x ) dx + ⎜ ⎟ cos ( λi x ) dx
2 k ∫0 h f ⎟⎠ ∫0
2
L + (35)
⎜ 2k

0


⎝ 

Integral1i Integral2i Integral3i

The integrals in Eq. (35) are evaluated using Maple:

> restart;
> Integral1:=int((cos(lambda[i]*x))^2,x=0..L);
1 cos ( λ i L ) sin( λ i L ) + λ i L
Integral1 :=
2 λi

> Integral2:=int(x^2*cos(lambda[i]*x),x=0..L);
2
λ i L 2 sin( λ i L ) − 2 sin( λ i L ) + 2 λ i L cos ( λ i L )
Integral2 := 3
λi

> Integral3:=int(cos(lambda[i]*x),x=0..L);
sin( λ i L )
Integral3 :=
λi

The results from Maple are copied and pasted into EES and used with Eq. (35) to determine the
constants associated with each eigenvalue:

duplicate i=1,Nterm
Integral1[i]=1/2*(cos(lambda[i]*L)*sin(lambda[i]*L)+lambda[i]*L)/lambda[i]
Integral2[i]=1/lambda[i]^3*(lambda[i]^2*L^2*sin(lambda[i]*L)-&
2*sin(lambda[i]*L)+2*lambda[i]*L*cos(lambda[i]*L))
Integral3[i]=1/lambda[i]*sin(lambda[i]*L)
C[i]*Integral1[i]=-gv*Integral2[i]/(2*k)+(gv*L^2/(2*k)+gv*L/h_bar_f)*Integral3[i]
end

The solution at a specific time and position is evaluated using Eq. (33).

time=1000 [s] "time"


duplicate i=1,Nterm
theta[i]=C[i]*cos(lambda[i]*x)*exp(-lambda[i]^2*alpha*time)
end
T=T_infinity+sum(theta[1..Nterm])
T_C=converttemp(K,C,T)
Figure P3.5-4(e) illustrates the temperature as a function of position at the same times requested
in (b) and behaves in the manner indicated by the sketch in Figure P3.5-4(c).
140

120
0s

100 500 s
Temperature (°C)
1000 s

80

60 5000 s

40 10000 s

20
0 0.01 0.02 0.03 0.04 0.05 0.06 0.07 0.08 0.09 0.1
Position (m)
Figure P3.5-4(e): Tempeature as a function of position at various values of time, predicted by the separation
of variables model.

The heat transfer rate to the cooling fluid can be computed according to:

dT dθ ∞
q ′′ = − k = −k = k ∑ Ci λ sin ( λi L ) exp ( −λi2 α t ) (36)
dx x= L dx x= L i =1

qf=k*sum(C[i]*lambda[i]*sin(lambda[i]*L)*exp(-lambda[i]^2*alpha*time),i=1,Nterm)

Figure P3.5-4(f) illustrates the heat flux as a function of time and behaves in the manner
indicated by the sketch in Figure P3.5-4(d).
7000
Heat flux to cooling water (W/m )
2

6000

5000

4000

3000

2000

1000
0 1000 2000 3000 4000 5000 6000 7000 8000 9000 10000
Time (s)
Figure P3.5-4(f): Heat flux to the cooling water as a function of time predicted by the separation of variables
model.
Problem 3.5-8 (3-19 in text)
Reconsider Problem 3.3-7 (3-10 in text) using a separation of variables solution.
a.) Derive the governing differential equation, the boundary conditions, and the initial conditions
for the problem.

dx Tt = 20°C

x ∂T ∂T ∂ ⎡ ∂T ⎤
−k Ac −k Ac + ⎢ −k Ac dx
∂x ∂x ∂x ⎣ ∂x ⎥⎦
∂T
Ac dx ρ c
∂t
Figure P3.5-8(a): Differential energy balance.

A differential energy balance on the rod is shown in Figure 3.5-8(a) and leads to:

∂ 2T ∂T
α = (1)
∂x 2 ∂t

The boundary conditions are:

∂T
=0 (2)
∂x x =0

and

Tx = L = Tt (3)

The initial condition is the linear temperature distribution associated with steady state heat flow
through a constant cross-sectional area resistance:

qh
Tt =0 = Tt + ( L − x) (4)
k Ac

b.) Does the mathematical problem statement derived in (a) satisfy all of the requirements for a
separation of variables solution? If not, provide a simple transformation that can be applied
so that the problem can be solved using separation of variables?

The separation of variables solution requires that the partial differential equation and both spatial
boundary conditions be homogeneous. The partial differential equation, Eq. (1), is
homogeneous. However, the spatial boundary condition at x= L, Eq. (3), is not homogeneous. A
simple transformation:

θ = T − Tt (5)
will lead to a completely homogeneous problem:

∂ 2θ ∂θ
α 2 = (6)
∂x ∂t

∂θ
=0 (7)
∂x x =0

θ x= L = 0 (8)

qh
θt =0 = ( L − x) (9)
k Ac

c.) Prepare a separation of variables solution to the transformed problem from (b) and
implement your solution in EES.

The inputs are entered in EES:

$UnitSystem SI MASS RAD PA C J


$Tabstops 0.2 0.4 0.6 0.8 3.5

"Inputs"
q_dot_h=100 [W] "heater power"
A_c=0.1 [m^2] "cross-sectional area"
per=0.05 [m] "perimeter"
rho=5000 [kg/m^3] "density"
k=5 [W/m-K] "conductivity"
c=500 [J/kg-K] "specific heat capacity"
T_t=converttemp(C,K,20 [C]) "tip temperature"
L=0.09 [m] "length"
alpha=k/(rho*c) "thermal diffusivity"

The separation of variables approach assumes that the solution can be expressed as the product
of a function only of time, θt(t), and a function only of position, θX(x):

θ ( x, t ) = θ X ( x ) θ t ( t ) (10)

Substituting Eq. (10) into Eq. (6) leads, eventually, to:

d 2θ X
2
+ λ2 θ X = 0 (11)
dx

and

dθ t
+ λ2 α θt = 0 (12)
dt
The general solution to Eq. (11) is:

θ X = C1 sin ( λ x ) + C2 cos ( λ x ) (13)

where C1 and C2 are unknown constants. Substituting Eq. (10) into the spatial boundary
condition at x = 0, Eq. (7), leads to:

∂θ dθ X ⎡ ⎤
= θt = θ t ⎢C1 λ cos ( λ 0 ) − C2 λ sin ( λ 0 ) ⎥ = 0 (14)
∂x dx ⎢⎣ 


x =0 x =0
=1 =0 ⎦

or C1 = 0:

θ X = C2 cos ( λ x ) (15)

Substituting Eq. (10) into the spatial boundary condition at x = L, Eq. (8), leads to:

θ X = C2 cos ( λ L ) = 0 (16)

which provides the eigenvalues for the problem:

λi =
( 2 i − 1) π for i = 1, 2,..∞ (17)
2L

N=101 [-] "number of terms"


duplicate i=1,N
lambda[i]=(2*i-1)*pi/(2*L) "eigenvalue"
end

The solution to the non-homogeneous ordinary differential equation corresponding to the ith
eigenvalue, Eq. (12):

dθ ti
+ λi2 α θ ti = 0 (18)
dt

is

θ ti = C3,i exp ( −λi2 α t ) (19)

where C3,i is an undetermined constant. According to Eq. (10), the solution associated with the
ith eigenvalue is:
θi = θ X i θ ti = Ci cos ( λi x ) exp ( −λi2 α t ) (20)

where the constants C2,i and C3,i have been combined to a single undetermined constant Ci.
Because the partial differential equation is linear, the sum of the solution θi for each eigenvalue,
Eq. (20), is itself a solution:

∞ ∞
θ = ∑θi = ∑ Ci cos ( λi x ) exp ( −λi2 α t ) (21)
i =1 i =1

The final step of the problem selects the constants so that the series solution satisfies the initial
condition, Eq. (9):


qh
θt =0 = ∑ Ci cos ( λi x ) = ( L − x) (22)
i =1 k Ac

Each side of Eq. (22) is multiplied by cos(λj x) and integrated from x = 0 to x = L. The property
of orthogonality ensures that the only term on the left side of Eq. (22) that is not zero is the one
for which j = i; therefore:

q L q
L L L
Ci ∫ cos 2 ( λi x ) dx = h ∫ cos ( λi x ) dx − h ∫ x cos ( λi x ) dx (23)
k Ac 0 k Ac 0

0




Integral1i Integral2i Integral3i

The integrals are evaluated in Maple:

> restart;
> assume(i,integer);
> lambda=(2*i-1)*Pi/(2*L);
( 2 i~ − 1 ) π
λ=
2L
> Integral1:=int((cos(lambda*x))^2,x=0..L);
1 cos( λ L ) sin( λ L ) + λ L
Integral1 :=
2 λ
> Integral2:=int(cos(lambda*x),x=0..L);
sin( λ L )
Integral2 :=
λ
> Integral3:=int(x*cos(lambda*x),x=0..L);
−1 + cos( λ L ) + L sin( λ L ) λ
Integral3 :=
λ2

and copied into EES in order to evaluate the constant associated with each eigenvalue:
"Evaluate constants"
duplicate i=1,N
Integral1[i]=1/2*(cos(lambda[i]*L)*sin(lambda[i]*L)+lambda[i]*L)/lambda[i]
Integral2[i]=1/lambda[i]*sin(lambda[i]*L)
Integral3[i]=(-1+cos(lambda[i]*L)+L*sin(lambda[i]*L)*lambda[i])/lambda[i]^2
C[i]*Integral1[i]=q_dot_h*L*Integral2[i]/(k*A_c)-q_dot_h*Integral3[i]/(k*A_c)
end

The solution is evaluated at a particular time and position defined based on a dimensionless time
and position. The dimensionless position is normalized relative to length:

x_bar=0.5 [-] "dimensionless position"


x=x_bar*L "position"

The dimensionless time is normalized relative to the diffusive time constant:

L2
τ diff = (24)

tau_diff=L^2/(4*alpha) "diffusive time constant"


time_bar=0.25 [-] "dimensionless time"
time=time_bar*tau_diff "time"

The solution is evaluated according to:

duplicate i=1,N
theta[i]=C[i]*cos(lambda[i]*x)*exp(-lambda[i]^2*alpha*time)
end
theta=sum(theta[i],i=1,N)
T=theta+T_t
T_C=converttemp(K,C,T)

d.) Prepare a plot of the temperature as a function of position for t = 0 and t → ∞ as well as the
times requested in Problem 3.3-7 (3-10) part (c).

Figure P3.5-8(b) illustrates the temperature distribution at the times requested in the problem
statement.
40

t=0
35

Temperature (°C)
t/τdiff = 0.25

30 t/τdiff = 0.5

t/τdiff = 1

25
t/τdiff = 2

20 t→∞

0 0.1 0.2 0.3 0.4 0.5 0.6 0.7 0.8 0.9 1


Dimensionless position, x/L
Figure 3.5-8(b): Temperature as a function of position for several times.
Problem 3.6-1 (3-20 in text)
An oscillating heat flux is applied to one side of a wall that is exposed to fluid on the other side,
as shown in Figure P3.6-1.

h = 500 W/m -K
2
ρ = 6000 kg/m3
T∞ = 20°C k = 2 W/m-K
L = 0.4 cm c = 700 J/kg-K

q′′ = Δq ′′ ⎡⎣1 − cos (ω t )⎤⎦


Figure P3.6-1: Wall exposed to an oscillating heat flux.

The wall thickness is L = 0.4 cm and the wall material has density ρ = 6000 kg/m3, conductivity
k = 2 W/m-K, and c = 700 J/kg-K. The fluid temperature is T∞ = 20°C and the heat transfer
coefficient is h = 500 W/m2-K. Initially, the wall is in equilibrium with the fluid. The heat flux
varies according to: q ′′ = Δq ′′ ⎡⎣1 − cos (ω t ) ⎤⎦ where Δq ′′ = 1000 W/m2 and ω = 1 rad/s.
a.) Sketch the temperature as a function of time that you expect at x = 0 and x = L for the first 10
oscillations (0 < t < 62.8 s). Try to get the qualitative characteristics of your sketch correct
(e.g., the magnitude of the average temperature rise and temperature oscillations as well as
the time scales involved).

The inputs are entered in EES:

$UnitSystem SI MASS RAD PA K J


$TABSTOPS 0.2 0.4 0.6 0.8 3.5 in

"Inputs"
Dqf=1e3 [W/m^2] "heat flux"
omega=1 [rad/s] "angular frequency"
k=2 [W/m-K] "conductivity"
rho=6000 [kg/m^3] "density"
c=700 [J/kg-K] "specific heat capacity"
L=0.004 [m] "half-thickness of wall"
h_bar=500 [W/m^2-K] "heat transfer coefficient"
T_infinity=converttemp(C,K,20) "fluid temperature"
alpha=k/(rho*c) "thermal diffusivity"

There are a few characteristic resistances the govern this problem. The resistance to convection
from the edge at x = L is:

1
Rconv = (1)
hA

where A = 1 m2 for a per unit area basis analysis. The resistance to conduction across the wall is:
L
Rcond = (2)
kA

R_conv=1/(h_bar*A) "resistance to convection"


R_cond=L/(k*A) "resistance to conduction"

The time available for the heat flux associated with one oscillation to diffuse into the material is,
approximately:

π
tdiff = (3)
ω

and therefore the penetration depth is:

δ diff = 2 α tdiff (4)

t_diff=pi/omega "period of oscillation"


delta_t=2*sqrt(alpha*t_diff) "depth of penetration"

which leads to δdiff = 0.0024 m. This is less than L and therefore I expect that the temperature at
x = 0 will oscillate in response to the heat flux but the temperature at x = L will not. The
resistance to conduction into the penetration depth is:

δt
Rcond ,δt = (5)
kA

R_cond_deltat=delta_t/(k*A) "resistance of penetration depth"

The temperature rise associated with a single oscillation will be on the order of:

ΔTosc ≈ Δq ′′′ A Rcond ,δt (6)

DT_osc=Dqf*A*R_cond_deltat "amplitude of temperature oscillation"

which leads to ΔTosc = 1.2 K. The time required for the wall to thermally equilibrate with the
surroundings is, approximately:

τ lumped = C ( Rconv + Rcond ) (7)

where C is the heat capacity of the wall:

C = ALρ c (8)

Cap=A*L*rho*c "heat capacity"


tau_lump=Cap*(R_conv+R_cond) "lumped time constant"
which leads to τlumped = 33.6 s. Therefore, I would expect that the system will very nearly have
reached equilibrium after 10 cycles (62.8 s). At the point where the system has reached
equilibrium. The average temperature rise at x = 0 will be approximately:

ΔT x =0 ≈ Δq ′′′ A ( Rcond + Rconv ) (9)

and the average temperature rise at x = L will be approximately:

ΔT x = L ≈ Δq ′′′ A Rconv (10)

DT_0=Dqf*A*(R_cond+R_conv) "average temperature rise at x=0"


DT_L=Dqf*A*R_conv "average temperature rise at x=L"

which leads to ΔT x =0 = 4 K and ΔT x = L = 2 K. Figure 2 illustrates a sketch of temperature as a


function of time at x = 0 and x = L that has these characteristics.

~ 1.2 K
Temperature

x=0
~4K
x=L

~2K

Time
Figure 2: Sketch of temperature at x = 0 and x = L as a function of time for ~10 oscillations.

b.) Use Duhamel's Theorem to develop an analytical model of the process. Plot the temperature
as a function of time for the first 10 oscillations at x = 0, x = L/2, and x = L.

The governing partial differential equation for the problem, written in terms of a temperature
difference, θ = T - T∞ is:

∂ 2θ ∂θ
α = (11)
∂x 2 ∂t

The boundary conditions are:

∂θ
−k = h θ x=L (12)
∂x x=L
∂θ
Δq ′′ ⎡⎣1 − cos (ω t ) ⎤⎦ = − k (13)
∂x x =0

The initial condition is:

θt =0 = 0 (14)

In order to apply Duhamel's theorem, it is necessary that the problem have a zero initial
condition and that the only non-homogeneous boundary condition be the time-dependent one.
The problem stated above satisfies this criterion. The next step is to obtain the fundamental
solution. The fundamental solution, θf, is the solution to the isolated sub-problem with the time-
dependent boundary condition replaced by a constant, unit value. For this problem, the time-
dependent heat flux in Eq. (13) is replaced by unity. The mathematical specification of the
fundamental problem is therefore:

∂ 2θ ∂θ
α 2 = (15)
∂x ∂t

The boundary conditions are:

∂θ
−k = h θ x=L (16)
∂x x=L

∂θ
1 = −k (17)
∂x x =0

The initial condition is:

θt =0 = 0 (18)

The fundamental solution was obtained in Problem 3.5-5 and is repeated here. To apply
separation of variables, it is necessary that both of the boundary conditions be homogeneous;
however, Eq. (13) is not. Therefore, the solution is assumed to be the sum of a homogeneous
and particular solution:

θ = θ h ( x, t ) + X ( x ) (19)

Substituting Eq. (19) into Eq. (11) leads to:

∂ 2θ h d 2 X ∂θ h
α + α = (20)
∂x 2 dx 2 ∂t
which is divided into the partial differential equation for θh and ordinary differential equation for
X:

∂ 2θ h ∂θ h
α = (21)
∂x 2 ∂t

and

d2X
=0 (22)
dx 2

Integrating Eq. (22) twice leads to:

X = C1 x + C2 (23)

Substituting Eq. (19) into the boundary condition at x = L, Eq. (12), leads to:

∂θ h dX
−k −k = h θ x= L + h X x=L (24)
∂x x=L ∂x x=L

which is divided into the homogeneous boundary condition for θh and the boundary condition for
X:

∂θ h
−k = h θ x=L (25)
∂x x= L

dX
−k = h X x=L (26)
∂x x=L

Substituting Eq. (23) into Eq. (26) leads to:

− k C1 = h ( C1 L + C2 ) (27)

Substituting Eq. (19) into the boundary condition at x = 0, Eq. (13), leads to:

∂θ h ∂X
q ′′ = −k −k (28)
∂x x =0 ∂x x =0

which is divided into the homogeneous boundary condition for θh and the boundary condition for
X:
∂θ h
=0 (29)
∂x x =0

∂X
q ′′ = − k (30)
∂x x =0

Substituting Eq. (23) into Eq. (30) leads to:

q ′′ = − k C1 (31)

Equations (27) and (31) are solved for C1 and C2:

"constants for particular solution"


-k*C_1=h_bar*(C_1*L+C_2)
-k*C_1=qf

and the particular solution is evaluated at a specific axial location:

x_bar=0.1 [-] "dimensionless position"


x=x_bar*L "position"
Xs=C_1*x+C_2 "particular solution"

Equation (19) is substituted into Eq. (14):

θ h ,t = 0 + X = 0 (32)

which leads to:

θ h ,t =0 = −C1 x − C2 (33)

The homogeneous solution is solved using separation of variables. The partial differential
equation, Eq. (21), is separated into θX and θt ordinary differential equations:

d 2θ X
+ λ2 θ X = 0 (34)
dx 2

dθ t
+ λ2 α θt = 0 (35)
dt

The general solution to Eq. (34) is:

θ X = C1 sin ( λ x ) + C2 cos ( λ x ) (36)


where C1 and C2 are unknown constants. Substituting Eq. (36) into the spatial boundary
condition at x = 0, Eq. (29), leads to C1 = 0.

θ X = C2 cos ( λ x ) (37)

Substituting Eq. (37)into the spatial boundary condition at x = L, Eq. (25), leads to:

k C2 λ sin ( λ L ) = h C2 cos ( λ L ) (38)

Equation (38) provides the eigencondition for the problem, which defines multiple eigenvalues:

sin ( λ L ) h
= (39)
cos ( λ L ) k λ

or, multiplying and dividing the right side of Eq. (39) by L:

sin ( λ L ) hL
= (40)
cos ( λ L ) k λ L

Writing Eq. (40) in terms of the Biot number leads to:

Bi
tan ( λ L ) = (41)
λL

where

hL
Bi = (42)
k

The eigenvalues are automatically identified in EES by specifying lower and upper limits and
guess values for each one.

Nterm=6 [-] "number of terms to use in the


solution"
"Setup guess values and lower and upper bounds for eigenvalues"
duplicate i=1,Nterm
lowerlimit[i]=(i-1)*pi
upperlimit[i]=lowerlimit[i]+pi/2
guess[i]=lowerlimit[i]+pi/4
end
"Identify eigenvalues"
Bi=h_bar*L/k
duplicate i=1,Nterm
tan(lambdaL[i])=Bi/lambdaL[i] "eigencondition"
lambda[i]=lambdaL[i]/L "eigenvalue"
end
At this point, each of the eigenfunctions of the problem have been obtained. The ith
eigenfunction is:

θ X i = C2,i cos ( λi x ) (43)

where λi is the ith eigenvalue, identified by the eigencondition:

Bi
tan ( λi L ) = (44)
λi L

The solution to the non-homogeneous ordinary differential equation corresponding to the ith
eigenvalue is

θ ti = C3,i exp ( −λi2 α t ) (45)

where C3,i are undetermined constants. The solution associated with the ith eigenvalue is:

θ h,i = θ X i θ ti = Ci cos ( λi x ) exp ( −λi2 α t ) (46)

where the constants C2,i and C3,i have been combined to a single undetermined constant Ci.
Because the partial differential equation is linear, the sum of the solution θi for each eigenvalue,
Eq. (46), is itself a solution:

∞ ∞
θ h = ∑ θi = ∑ Ci cos ( λi x ) exp ( −λi2 α t ) (47)
i =1 i =1

The final step of the problem selects the constants so that the series solution satisfies the initial
condition, Eq. (33):


θt =0 = ∑ Ci cos ( λi x ) = −C1 x − C2 (48)
i =1

Each side of Eq. (48) is multiplied by cos(λj x) and integrated from x = 0 to x = L:


L x x
Ci ∫ cos 2 ( λi x ) dx = −C1 ∫ x cos ( λi x ) dx − C2 ∫ cos ( λi x ) dx (49)

0


0


0

Integral 1 Integral 2 Integral 3

The integrals in Eq. (49) can be evaluated conveniently using Maple:

> restart;
> int((cos(lambda[i]*x))^2,x=0..L);
1 cos ( λ i L ) sin( λ i L ) + λ i L
2 λi
> int(x*cos(lambda[i]*x),x=0..L);
−1 + cos ( λ i L ) + L sin( λ i L ) λ i
2
λi
> int(cos(lambda[i]*x),x=0..L);
sin( λ i L )
λi

Substituting these results into EES provides each of the constants:

duplicate i=1,Nterm
Integral1[i]=1/2*(cos(lambda[i]*L)*sin(lambda[i]*L)+lambda[i]*L)/lambda[i]
Integral2[i]=(-1+cos(lambda[i]*L)+L*sin(lambda[i]*L)*lambda[i])/lambda[i]^2
Integral3[i]=1/lambda[i]*sin(lambda[i]*L)
C[i]*Integral1[i]=-C_1*Integral2[i]-C_2*Integral3[i]
end

Duhamel's theorem is restated below, for our problem:

If Tf (x,t) is the response of a linear system with a zero initial condition to a single, constant non-
homogeneous boundary condition with magnitude of unity (referred to as the fundamental
solution), then the response of the same system to a single, time-varying non-homogeneous
boundary condition with magnitude B(t) can be obtained from the fundamental solution
according to:
t
dq ′′ (τ )
T ( x, t ) = ∫ T ( x, t − τ )
f dτ + qt′′=0 T f ( x, t ) (50)
τ=0
dt

where Bt=0 is the value of B at time zero and B must be continuous in time.

We will evaluate the solution at a particular value of time, normalized against the time required
for one oscillation:

t_osc=2*pi/omega "time required for one oscillation"


time_bar=1 [-] "dimensionless time"
time=time_bar*t_osc "time"

An arbitary value of the integration variable, τ, is initially assumed and will eventually be varied
to carry out the integration in Eq. (50).

tau=0 [s] "integration variable"


The fundamental solution is evaluated at xand t - τ:

duplicate i=1,Nterm
theta_f_h[i]=C[i]*cos(lambda[i]*x)*exp(-lambda[i]^2*alpha*(time-tau))
end
theta_f_h=sum(theta_f_h[i],i=1,Nterm)
theta_f=theta_f_h+Xs_f

and multiplied by the time derivative of the heat flux:

dq ′′ (τ )
= Δq ′′ ω sin (ωτ ) (51)
dt

dqfdt=Dqf*omega*sin(omega*tau)

In order to carry out the integration in Eq. (50), the arbitrary value of the integration variable is
commented out and the Integral function in EES is used (note that the value of the heat flux at t =
0 is 0):

{tau=0 [s]} "integration variable"


theta=Integral(theta_f*dqfdt,tau,0,time,0.1)
T=theta+T_infinity
T_C=converttemp(K,C,T)

Figure 3 illustrates the temperature as a function of time for x = 0, L/2, and L:


23.5

23
x/L = 0
22.5 x/L = 0.5
Temperature (°C)

22

21.5 x/L = 1

21

20.5

20

19.5
0 10 20 30 40 50 60 70
Time (s)
Figure 3: Temperature at x/L = 0, 0.5, and 1 as a function of time for 10 oscillations.
Problem 3.7-1 (3-21 in text): Regenerator Matrix
Regenerative heat exchangers are discussed in Section 8.10. The regenerator operates in a cyclic
fashion. Hot fluid passes across the regenerator material for half of a cycle, transferring energy
to the material. Cold fluid passes across the regenerator material for the other half of the cycle,
receiving energy from the material. After a sufficient number of cycles, the temperature
distribution reaches a cyclic steady-state condition. Consider a regenerator matrix that consists
of plates, as shown in Figure P3.7-1.
ρ, k, c

h , T∞ = T∞ + ΔT∞ sin (ω t ) h , T∞ = T∞ + Δ T∞ sin (ω t )


L
x
Figure P3.7-1: Plate regenerator matrix.

The half-thickness of the plate is L and the material properties are ρ, c, and k. The heat transfer
coefficient between the surface of the plate and the fluid is h and the fluid temperature is
assumed to vary sinusoidally with mean temperature T∞ , amplitude ΔT∞ , and frequency ω. In
general, the temperature within the regenerator matrix is a function of both x and t.
a.) Using the method of complex combination, develop a solution for the sustained response of
the temperature within the regenerator.

The governing differential equation for this problem is:

∂ 2T ∂T
α = (1)
∂x 2 ∂t

The spatial boundary conditions are obtained with interface balances at the center (x = 0) and
surface (x = L) of the piston plate:

∂T
−k = h ⎡⎣Tx = L − T∞ − ΔT∞ sin (ω t ) ⎤⎦ (2)
∂x x=L

∂T
−k =0 (3)
∂x x =0

No initial condition is required since only the sustained, periodic solution will be obtained. The
method of complex combination can only be applied if the problem is homogeneous except for
the periodic disturbance. The boundary condition at x = L can be made homogeneous with the
transformation:

θ = T − T∞ (4)
which leads to:

∂ 2θ ∂θ
α = (5)
∂x 2 ∂t

∂θ
−k = h ⎡⎣θ x = L − ΔT∞ sin (ω t ) ⎤⎦ (6)
∂x x= L

∂θ
−k =0 (7)
∂x x =0

The problem for θ90° is constructed by replacing the periodic component of the θ problem with
one that is 90° out of phase (i.e., by replacing the sin(ω t) term in the boundary condition at x = 0
with a cos(ω t) term):

∂ 2θ90° ∂ θ90°
α = (8)
∂x 2 ∂t

∂θ90°
−k = h ⎡⎣θ90°, x = L − ΔT∞ cos (ω t ) ⎤⎦ (9)
∂x x=L

∂θ90°
−k =0 (10)
∂x x =0

The complex conjugate problem for θcc:

θcc = θ + i θ90° (11)

is constructed by multiplying the governing differential equation and boundary conditions for
θ90° by i (the square root of negative one) and adding it to the governing differential equation and
boundary conditions for θ.
∂ 2θ cc ∂ θ cc
α = (12)
∂x 2 ∂t

∂θ ∂θ90°
−k −k i = h ⎡⎣θ x = L − ΔT∞ sin (ω t ) ⎤⎦ + h ⎡⎣θ90°, x= L − ΔT∞ cos (ω t ) ⎤⎦ i (13)
∂x x=L ∂x x= L

or
∂θcc
−k = h θcc, x = L − h ΔT∞ ⎡⎣sin (ω t ) + i cos (ω t ) ⎤⎦ (14)
∂x x= L

∂θcc
−k =0 (15)
∂x x =0

The periodic variation in Eq. (14) must be converted to exponential form; this is most easily
accomplished using Maple:

> restart;
> convert(sin(omega*t)+I*cos(omega*t),exp);
− I ⎛⎜ e − ( ω t I ) ⎟⎞ + ⎛⎜ e
1 (ω t I) 1 1 (ω t I) 1 1 ⎞
+ ⎟I
2 ⎜ ⎟ ⎜2 2 e(ω t I) ⎟
⎝ e ⎠ ⎝ ⎠
> simplify(%);
( −I ω t )
e I

which leads to:

∂θcc
−k = h θcc, x = L − h ΔT∞ i exp ( -i ω t ) (16)
∂x x= L

The complex conjugate solution is assumed in the appropriate exponential form:

θcc = B ( x ) exp ( −i ω t ) (17)

Substituting Eq. (17) into the partial differential equation:

∂2 ∂
α ⎡⎣ B ( x ) exp ( −i ω t ) ⎤⎦ = ⎡⎣ B ( x ) exp ( −i ω t ) ⎤⎦ (18)
∂x 2 ∂t

or

d 2B
α exp ( −i ω t ) = − B i ω exp ( −i ω t ) (19)
dx 2

Note that the time variation must cancel from the complex conjugate problem when expressed in
this manner (a consequence of the problem being homogeneous), leaving an ordinary differential
equation for B:

d 2B
α = −B i ω (20)
dx 2
Substituting Eq. (17) into the boundary condition at x = L:

dB
−k = h Bx = L − h ΔT∞ i (21)
dx x= L

Substituting Eq. (17) into the boundary condition at x = 0, leads to:

dB
=0 (22)
dx x=0

The solution for B is:

⎛ iω ⎞ ⎛ iω ⎞
B = C1 sin ⎜⎜ x ⎟⎟ + C2 cos ⎜⎜ x ⎟⎟ (23)
⎝ α ⎠ ⎝ α ⎠

Substituting Eq. (23) into Eq. (22) leads to:

dB iω ⎛ iω ⎞ iω ⎛ iω ⎞
= C1 cos ⎜⎜ 0 ⎟⎟ − C2 sin ⎜⎜ 0 ⎟⎟ = 0 (24)
dx x =0 α ⎝ α ⎠ α ⎝ α ⎠

which can only be true if C1 = 0:

⎛ iω ⎞
B = C2 cos ⎜⎜ x ⎟⎟ (25)
⎝ α ⎠

Substituting Eq. (25) into Eq. (21) leads to:

iω ⎛ iω ⎞ ⎛ iω ⎞
k C2 sin ⎜⎜ L ⎟⎟ = h C2 cos ⎜⎜ x ⎟⎟ − h ΔT∞ i (26)
α ⎝ α ⎠ ⎝ α ⎠

which can be solved for C2:

− h ΔT∞ i
C2 = (27)
⎡ iω ⎛ iω ⎞ ⎛ i ω ⎞⎤
⎢k sin ⎜ L ⎟ − h cos ⎜ x ⎟⎥
⎣⎢ α ⎝ α ⎠ ⎝ α ⎠ ⎥⎦

Substituting Eqs. (27) and (25) into Eq. (17) leads to:
⎛ iω ⎞
− ΔT∞ i cos ⎜ x ⎟ exp ( −i ω t )
⎝ α ⎠
θcc = (28)
⎡ k iω ⎛ iω ⎞ ⎛ i ω ⎞⎤
⎢ sin ⎜ L ⎟ − cos ⎜ x ⎟⎥
⎣⎢ h α ⎝ α ⎠ ⎝ α ⎠ ⎦⎥

The solution is the real part of Eq. (28).

b.) Identify physically significant dimensionless parameters that can be used to correlate your
solution. You should non-dimensionalize your solution and express it in terms of a
dimensionless position and time as well as the Biot number and an additional dimensionless
parameter that characterizes the frequency of oscillation.

The dimensionless solution is the temperature difference relative to the average fluid temperature
normalized by the fluid temperature oscillation:

T − T∞
θ = (29)
ΔT∞

The dimensionless position is defined relative to the spatial extent of the plate:

x
x = (30)
L

The dimensionless time is defined according to:

t = ω t (31)

The Biot number is the ratio of the internal conduction resistance to external convection
resistance:

hL
Bi = (32)
k

The final dimensionless parameter will be referred to as a dimensionless frequency and is the
ratio of the diffusion time constant to the period of oscillation:

ω L2
ω = (33)
α

Substituting Eqs. (29) through (33) into Eq. (28) leads to:
θcc =
− i cos ( )
i ω x exp ( −i t )
(34)
⎡ i ω ⎤
⎢ sin ( )
i ω − cos ( )
i ω x ⎥
⎢⎣ Bi ⎥⎦

The dimensionless solution, θ , is the real part of Eq. (34). The solution is implemented in EES
according to:

$UnitSystem SI MASS RAD PA K J


$TABSTOPS 0.2 0.4 0.6 0.8 3.5 in
$COMPLEX On i

t_hat=0.1 [-] "dimensionless time"


x_hat=1 [-] "dimensionless position"
omega_hat=2 [-] "dimensionless frequency"
Bi=10 [-] "Biot number"

theta_hat_cc=-i*exp(-i*t_hat)*cos(sqrt(i*omega_hat)*x_hat)/&
(sqrt(i*omega_hat)*sin(sqrt(i*omega_hat))/Bi-cos(sqrt(i*omega_hat)))
"complex conjugate solution"
theta_hat=Real(theta_hat_cc)

c.) Prepare three plots of the dimensionless temperature as a function of dimensionless time for
various values of the dimensionless position. Plot 1 should be for a large Biot number and
large dimensionless frequency, plot 2 should be for a large Biot number and a small
dimensionless frequency, and plot 3 should be for a small Biot number and a small
dimensionless frequency. Explain why the behavior exhibited in each of these plots obeys
your physical intuition.

Figure 2 illustrates θ as a function of t for various values of x when Bi is large and ω is large.
In this limit, the internal conduction resistance is large relative to the convection resistance and
therefore most of the temperature gradient is within the material. Also, the time required for a
single oscillation is much smaller than the time constant for diffusion - therefore the diffusion
wave cannot penetrate to the center of the material. These characteristics are both exhibited by
Figure 2.
1

0.8

Dimensionless temperature
0.6
x/L = 0.25
0.4
x/L = 0
0.2

-0.2

-0.4 x/L = 0.5


-0.6 x/L = 0.75
-0.8 x/L = 1
-1
0 1.57 3.14 4.71 6.28
Dimensionless time
Figure 2: Dimensionless temperature as a function of dimensionless time for various values of dimensionless
position when Bi is large and dimensionless frequencye is large.

Figure 3 illustrates θ as a function of t for various values of x when Bi is large and ω is small.
In this limit, the internal conduction resistance is large relative to the convection resistance and
therefore most of the temperature gradient is within the material. However, the time required for
a single oscillation is much larger than the time constant for diffusion - therefore the diffusion
wave can penetrate to the center of the material and the entire wall participates in the process so
that there are no temperature gradients. The entire wall follows the fluid temperature. These
characteristics are exhibited by Figure 3.

0.8
Dimensionless temperature

0.6
x/L = 0, 0.25, 0.5, 0.75, 1
0.4

0.2

-0.2

-0.4

-0.6

-0.8

-1
0 1.57 3.14 4.71 6.28
Dimensionless time
Figure 3: Dimensionless temperature as a function of dimensionless time for various values of dimensionless
position when Bi is large and dimensionless frequencye is large.
Figure 4 illustrates θ as a function of t for various values of x when Bi is small and ω is small.
In this limit, the internal conduction resistance is small relative to the convection resistance and
therefore most of the temperature gradient is between the surface of the material and the fluid.
Also, the time required for a single oscillation is much larger than the time constant for diffusion
- therefore the diffusion wave can penetrate to the center of the material. These characteristics
are both exhibited by Figure 4.

0.8
Dimensionless temperature

0.6

0.4

0.2

-0.2 x/L = 0, 0.25, 0.5, 0.75, 1


-0.4

-0.6

-0.8

-1
0 1.57 3.14 4.71 6.28
Dimensionless time
Figure 4: Dimensionless temperature as a function of dimensionless time for various values of dimensionless
position when Bi is small and dimensionless frequencye is small.
Problem 3.8-3 (3-22 in text)
Prepare a numerical solution for the equilibration process discussed in Problem 3.5-2 (3-18 in
text) using the Crank-Nicolson technique. Implement your solution in MATLAB and prepare a
plot of the temperature as a function of position at t = 10,000 s; overlay the analytical solution
derived in Problem 3.5-2 (3-18) on this plot in order to demonstrate that the analytical and
numerical solutions agree.

The inputs are entered in MATLAB.

clear all;

qf_dot=500000; %heat flux (W/m^2)


k=1; %conductivity (W/m-K)
rho=4000; %density (kg/m^3)
c=700; %specific heat capacity (J/kg-K)
L=0.5; %thickness of wall (m)
h=5000; %heat transfer coefficient (W/m^2-K)
T_f=293.2; %fluid temperature (K)
A=1; %per unit area (m^2)

The initial temperature of the right hand side of the wall is computed as discussed in part (a).

R_cond=L/(k*A); %conduction resistance through wall (K/W)


R_conv=1/(h*A); %convection resistance (K/W)
R_eq=(1/R_cond+1/R_conv)^(-1); %total resistance (K/W)
T_RHS=T_f+qf_dot*A*R_eq; %initial temperature of the RHS of wall (K)

The location of each node (xi) is:

(i − 1)
xi = L for i = 1..N (1)
( N − 1)
where N is the number of nodes used for the simulation. The distance between adjacent nodes
(Δx) is:

L
Δx = (2)
N −1

%SETUP GRID
N=11; %number of nodes (-)
for i=1:N
x(i)=(i-1)*L/(N-1); %position of each node (m)
end
DELTAx=L/(N-1); %distance between adjacent nodes (m)

The total simulation time, τsim, is divided into M time steps; most of the techniques discussed
here will divide the simulation time into time steps of equal duration, Δt:
τ sim
Δt = (3)
( M − 1)
The time associated with any time step is:

t j = ( j − 1) Δt for j = 1...M (4)

%SETUP TIME STEPS


M=1001; %number of time steps (-)
tau_sim=100000; %simulation time (s)
DELTAtime=tau_sim/(M-1); %time step duration (s)
for j=1:M
time(j)=(j-1)*DELTAtime;
end

The initial conditions for this problem are that all of the temperatures at t=0 are equal to Tin.

Ti ,1 = T f + (Tx = L ,t =0 − T f )
xi
for i = 1...N (5)
L

%INITIAL CONDITION
for i=1:N
T(i,1)=T_f+(T_RHS-T_f)*x(i)/L;
end

Control volumes defined around each of the internal nodes lead to the state equations:

dTi k
= 2 (Ti −1 + Ti +1 − 2 Ti ) for i = 2... ( N − 1) (6)
dt Δx ρ c

A control volume defined around node N (at the right hand side of the wall) leads to:

dTN
dt
=
2k
ρ c Δx 2 ( N −1
T − TN ) +
2h
Δx ρ c
(T f − TN ) (7)

The temperature of node 1 is always specified. The Crank-Nicolson technique uses the average
of the time rate of change at the beginning and the end of each time step; therefore, the equations
used to move through a time step for each node are:

k Δt
Ti , j +1 = Ti , j +
2 Δx 2 ρ c
(Ti−1, j + Ti+1, j − 2 Ti, j + Ti−1, j +1 + Ti+1, j +1 − 2 Ti, j +1 ) for i = 2...( N − 1) (8)

k Δt h Δt
2 ( N −1, j
TN , j +1 = TN , j + T − TN , j + TN −1, j +1 − TN , j +1 ) + ( 2 T f − TN , j − TN , j +1 ) (9)
ρ c Δx Δx ρ c
T1, j +1 = T f (10)

Equations (8) through (10) should be placed in matrix format; they are arranged to show the
coefficients and constants more clearly:

⎡ k Δt ⎤ ⎡ k Δt ⎤ ⎡ k Δt ⎤ k Δt
Ti , j +1 ⎢1 + 2 ⎥
Δx ρ c ⎦
+ Ti −1, j +1 ⎢ − ⎥
2 Δx ρ c ⎦
2
+ Ti +1, j +1 ⎢ − ⎥
2 Δx ρ c ⎦
2
= Ti , j +
2 Δx 2 ρ c
(Ti−1, j + Ti+1, j − 2 Ti, j )
⎣






Ai ,i Ai ,i −1 Ai ,i +1 bi

for i = 2... ( N − 1)
(11)

⎡ k Δt h Δt ⎤ ⎡ k Δt ⎤ k Δt h Δt
2 ( N −1, j
TN , j +1 ⎢1 + + ⎥ + TN −1, j +1 ⎢ − 2⎥
= TN , j + T − TN , j ) + ( 2 T f − TN , j )
ρ c Δx Δx ρ c ⎦
⎣

2
ρ c Δx ⎦
⎣
ρ c Δx Δx ρ c


AN , N AN , N −1 bN

(12)

T1, j +1 [1] = T f (13)


N N
A1,1 b1

Notice that the matrix A does not change with time for this problem and need only be setup one
time:

A=spalloc(N,N,3*N); %initialize A
b=zeros(N,1); %initialize b

%SETUP A MATRIX
A(1,1)=1;
for i=2:(N-1)
A(i,i)=1+k*DELTAtime/(rho*c*DELTAx^2);
A(i,i-1)=-k*DELTAtime/(2*rho*c*DELTAx^2);
A(i,i+1)=-k*DELTAtime/(2*rho*c*DELTAx^2);
end
A(N,N)=1+k*DELTAtime/(rho*c*DELTAx^2)+h*DELTAtime/(DELTAx*rho*c);
A(N,N-1)=-k*DELTAtime/(rho*c*DELTAx^2);

The vector b must be setup for each time step since it includes the temperatures at the beginning
of the timestep:

for j=1:(M-1)
%SETUP b MATRIX
b(1)=T_f;
for i=2:(N-1)
b(i)=T(i,j)+k*DELTAtime*(T(i-1,j)+T(i+1,j)-
2*T(i,j))/(2*rho*c*DELTAx^2);
end
b(N)=T(N,j)+k*DELTAtime*(T(N-1,j)-
T(N,j))/(rho*c*DELTAx^2)+h*DELTAtime*(2*T_f-T(N,j))/(DELTAx*rho*c);
%SIMULATE TIME STEP
T(:,j+1)=A\b;
end

Figure 1 illustrates the temperature distribution at t = 10,000 s predicted using the separation of
variables solution in Problem 3.5-2 (3-18 in text) and the numerical solution from this problem.

Figure 1: Temperature as a function of position at t = 10,000 s predicted by the numerical and


analytical solutions.
Problem 3.8-5 (3-23 in text)
A pin fin is used as part of a thermal management system for a power electronics system, as
shown in Figure P3.8-5.

h = 50 W/m -K
2

D = 3 mm T∞ = 20°C

k = 10 W/m-K L = 3 cm
ρ = 4000 kg/m3
q
c = 400 J/kg-K

Figure P3.8-5: Pin fin subjected to a transient heat load.

The diameter of the fin is D = 3 mm and the length is L = 3 cm. The fin material has
conductivity k = 10 W/m-K, ρ = 4000 kg/m3, and c = 400 J/kg-K. The surface of the fin is
exposed to air at T∞ = 20ºC with heat transfer coefficient h = 50 W/m2-K. The tip of the fin can
be assumed to be adiabatic. The power electronics system does not operate at steady state;
rather, the load applied at the base of the fin cycles between a high and a low value with some
angular frequency, ω. The average heat transfer rate is q = 0.5 W and the amplitude of the
fluctuation is Δq = 0.1 W. The frequency of oscillation varies. The fin is initially in equilibrium
with T∞.
a.) Develop a 1-D transient model that can be used to analyze the startup and operating behavior
of the pin fin. Use the ode solver in MATLAB.

The inputs are entered in a MATLAB script:

clear all;
D=0.003; % diameter of fin (m)
k=10; % conductivity of fin (W/m-K)
rho=4000; % density (kg/m^3)
c=400; % specific heat capacity (J/kg-K)
L=0.03; % length (m)
h_bar=50; % heat transfer coefficient (W/m^2-K)
T_infinity=293.2; % ambient temperature (K)
omega=0.1; % frequency (rad/s)
Dq_dot=0.1; % amplitude of heat load (W)
q_dot_bar=0.5; % average heat load (W)

Nodes are distributed uniformly throughout the node according to:

(i − 1)
xi = L for i = 1..N (1)
( N − 1)
where N is the number of nodes used for the simulation. The distance between adjacent nodes
(Δx) is:
L
Δx = (2)
( N − 1)
N=21; % number of nodes (-)
for i=1:N
x(i,1)=L*(i-1)/(N-1); % locations of nodes (m)
end

An energy on the internal nodes leads to:

π D2 π D2 π D2 dTi
h Δx π D (T∞ − Ti ) + k (Ti −1 − Ti ) + k (Ti +1 − Ti ) = Δx ρ c for i = 2.. ( N − 1) (3)
4 Δx 4 Δx 4 dt

which can be solved for the time rate of change of Ti:

⎡ πD ⎤
⎢ h Δx π (T∞ − Ti ) + k 4 Δx (Ti −1 + Ti +1 − 2 Ti ) ⎥
dTi ⎣ ⎦
= for i = 2.. ( N − 1) (4)
dt π D
Δx ρ c
4

An energy balance on node 1 leads to:

π D2 Δx π D2 dT
q + k (T2 − T1 ) + h π D (T∞ − T1 ) = Δx ρ c 1 (5)
4 Δx 2 8 dt

which can be solved for the time rate of change of T1:

⎡ π D2 Δx ⎤
⎢ 
q + k (T2 − T1 ) + h π D (T∞ − T1 )⎥
dT1 ⎣ 4 Δx 2 ⎦
= (6)
dt πD 2
Δx ρ c
8

An energy balance on node N leads to:

Δx π D2 π D2 dT
h π D (T∞ − TN ) + k (TN −1 − TN ) = Δx ρ c N (7)
2 4 Δx 8 dt

which can be solved for the time rate of change of TN:


⎡ Δx π D2 ⎤
h
⎢ 2 π D ( T∞ − TN ) + k (TN −1 − TN ) ⎥
dTN ⎣ 4 Δx ⎦
= (8)
dt πD 2
Δx ρ c
8

These state equations, Eqs. (4), (6), and (8), are programmed in a function dTdtP3p8d5.

function[dTdt]=dTdtP3p8d5(t,T,D,k,rho,c,L,h_bar,T_infinity,omega,Dq_dot,...
q_dot_bar,N)

% Inputs
% t - time (s)
% T - vector of nodal temperatures (K)
% D - diameter of fin (m)
% k - conductivity of ground (W/m-K)
% rho - density (kg/m^3)
% c - specific heat capacity (J/kg-K)
% L - length (m)
% h_bar - heat transfer coefficient (W/m^2-K)
% T_infinity - ambient temperature (K)
% omega - frequency of heat transfer oscillation (rad/s)
% Dq_dot - amplitude of heat transfer oscillation (W)
% q_dot_bar - average heat transfer (W)
% N - number of nodes (-)

Dx=L/(N-1); % distance between nodes


q_dot=q_dot_bar+Dq_dot*sign(sin(omega*t)); % load on fin

dTdt=zeros(N,1);

%energy balance on node 1


dTdt(1)=(q_dot+k*pi*D^2*(T(2)-T(1))/(4*Dx)+h_bar*pi*D*Dx*(T_infinity-...
T(1))/2)/(rho*c*Dx*pi*D^2/8);
%energy balance on nodes 2 through N-1
for i=2:(N-1)
dTdt(i)=(h_bar*Dx*(T_infinity-T(i))+k*D*(T(i-1)+T(i+1)-...
2*T(i))/(4*Dx))/(rho*c*Dx*D/4);
end
%energy balance on node N
dTdt(N,1)=(h_bar*Dx*(T_infinity-T(N))/2+k*D*(T(N-1)-...
T(N))/(4*Dx))/(rho*c*Dx*D/8);
end

The ode15s function in MATLAB is used to integrate the state equations forward in time.

for i=1:N
T_ini(i,1)=T_infinity; % initial temperature (K)
end
t_sim=100; % simulation time (s)
OPTIONS=odeset('RelTol',1e-5);
[t,T]=ode15s(@(t,T)
dTdtP3p8d5(t,T,D,k,rho,c,L,h_bar,T_infinity,omega,Dq_dot,q_dot_bar,N),...
[0,t_sim], T_ini,OPTIONS);
b.) Plot the temperature as a function of time at various values of axial position for the start up
assuming a constant heat load (ω = 0).

The value of ω is set to zero:

omega=0; % frequency (rad/s)

and the simulation is used to generate Figure 2.


390

x = 0 cm
370
Temperature (K)

350
x = 0.6 cm

330
x = 1.2 cm

x = 1.8 cm
310

x = 2.4 cm x = 3.0 cm
290
0 20 40 60 80 100
Time (s)
Figure 2: Temperature as a function of time at various values of axial position with a constant heat load.

c.) Calculate a diffusive time constant and a lumped capacitance time constant for the
equilibration process. Is the plot from (b) consistent with these values?

The diffusive time constant is:

L2
τ diff = (9)

The lumped time constant is:

1 π D2 ρcD
τ lumped = R C = ρc L= (10)
hπ DL 4 4h

alpha=k/(rho*c); % thermal diffusivity (m^2/s)


tau_diff=L^2/(4*alpha) % diffusive time constant (s)
tau_lump=(rho*c*D/4)/(h_bar) % lumped time constant (s)

which leads to τdiff = 36 s and τlumped = 24 s; this is consistent with the time required for the fin to
achieve equilibrium as shown in Figure 2.
d.) Adjust the diameter of the fin so that the lumped time constant is much greater than the
diffusive time constant. Plot the temperature as a function of time at various values of axial
position for the start up assuming a constant heat load (ω = 0). Explain your result.

The lumped time constant is proportional to D while the diffusive time constant is independent
of D. The value of the diameter is increased by a factor of 10 to increase the lumped capacitance
time constant relative to the diffusive time constant. The result is shown in Figure 3. Notice that
the diffusive equilibration occurs very quickly (within ~36 s) and therefore the temperature
distribution takes on its equilibrium shape and then subsequently the magnitude of the
temperature everywhere changes according to the lumped capacitance time constant.
297.5
x = 0 cm
297 x = 0.6 cm
x = 1.2 cm
296.5
x = 1.8 cm
Temperature (K)

296 x = 2.4 cm
x = 3.0 cm
295.5

295

294.5

294

293.5

293
0 200 400 600 800 1000
Time (s)
Figure 3: Temperature as a function of time at various values of axial position with a constant heat load and a
diameter adjusted to that τlumped >> τdiff.

e.) Return the diameter of the fin to D = 3 mm and set the oscillation frequency to ω = 1 rad/s.
Prepare a contour plot showing the temperature of the fin as a function of position and time.
You should see that the oscillation of the heat load causes a disturbance that penetrates only
part-way along the axis of the fin. Explain this result.

Figure 4 illustrates the temperature as a function of time for various axial positions and Figure 5
illustrates the contour plot requested by the problem statement. Note that the effect of the
oscillation only extends ≈ 2 α tosc = 1.3 cm into the fin.
390
x = 0 cm

370

Temperature (K)
350 x = 0.6 cm

330 x = 1.2 cm
x = 1.8 cm

310
x = 2.4 cm
x = 3.0 cm
290
0 20 40 60 80 100
Time (s)
Figure 4: Temperature as a function of time at various values of axial position with an oscillating heat load.

100 380

90
370

80
360
70
350
60
Time (s)

340
50

330
40

30 320

20 310

10
300

0
0 0.005 0.01 0.015 0.02 0.025 0.03
Position (m)
Figure 5: Contour plot of temperature as a function of time at various values of axial position with an
oscillating heat load.

f.) Is the maximum temperature experienced by the fin under oscillating conditions at cyclic
steady-state (i.e., after the start-up transient has decayed) greater than or less than the
maximum temperature experienced under steady-state conditions (i.e., with ω = 0)?
Figure 6 illustrates the temperature of the base (x= 0) as a function of time for the steady and
oscillating load cases and shows that the maximum fin temperature is larger when the heat load
oscillating.
390

Base temperature (K) 370


oscillating load
steady load
350

330

310

290
0 20 40 60 80 100
Time (s)
Figure 6: Base temperature as a function of time for the steady and oscillating load cases.

g.) Plot the ratio of the maximum temperature under oscillating conditions to the maximum
temperature under steady-state conditions as a function of frequency.

Figure 7 illustrates the base temperature as a function of time for various values of the frequency.
400
1 rad/s

380
Base temperature (K)

10 rad/s
360
steady load 0.1 rad/s

340

320

300

0 20 40 60 80 100
Time (s)
Figure 7: Base temperature as a function of time for various values of the frequency.

Figure 8 illustrates the ratio of the maximum base-to-fluid temperature difference under an
oscillating condition to the maximum base-to-fluid temperature difference with a steady heat
transfer as a function of the frequency of oscillation.
Maximum temp. to maximum steady temp.
1.05

1.04

1.03

1.02

1.01

1
0.01 0.1 1 10 20
Angular frequency (rad/s)
Figure 8: Ratio of the base temperature difference under oscillating conditions to the base temperature
difference with a steady heat transfer as a function of frequency.

h.) Define a meaningful dimensionless frequency and plot the maximum temperature under
oscillating conditions to the maximum temperature under steady-state conditions as a
function of this dimensionless frequency. Explain the shape of your plot.

The most appropriate dimensionless frequency is the ratio of τdiff to the time required for a single
oscillation.

L2 2 π
ω = (11)
4α ω

Figure 9 illustrates the ratio of the maximum base-to-fluid temperature difference under an
oscillating condition to the maximum base-to-fluid temperature difference with a steady heat
transfer as a function of the dimensionless frequency of oscillation. Note that if ω >>1 then the
ratio approaches 1 because the effect of the oscillation is confined to a relatively small region
near the base and the associated temperature rise is small (see Figure 7). If ω << 1 then the ratio
asymptotes to a number higher than 1 because essentially assumes a steady state condition at the
higher heat load and another steady state condition at the lower heat load (see Figure 7).
1.05

Maximum temp. to maximum steady temp.


1.04

1.03

1.02

1.01

1
0.05 0.1 1 10 100
Dimensionless frequency
Figure 9: Ratio of the base temperature difference under oscillating conditions to the base temperature
difference with a steady heat transfer as a function of dimensionless frequency.
Problem 3.10-1 (3-24 in text)
Figure P3.10-1(a) illustrates a disk brake that is used to bring a piece of rotating machinery to a
smooth stop.

ha = 25 W/m -K
2
pad
Ta = 20°C
q ′′

disk
h jet = 250 W/m -K
2

Ta = 20°C
Figure P3.10-1(a): A disk brake on a rotating machine.

The brake pad engages the disk at its outer edge when the brake is activated. The outer edge and
top surface of the disk (except under the pad) are exposed to air at Ta= 20°C with ha = 25 W/m2-
K. The bottom edge is exposed to air jets in order to control the disk temperature; the air jets
have Ta = 20°C and h jet = 250 W/m2-K. The problem can be modeled as a 2-D, radial problem as
shown in Figure P3.10-1(b).

ha = 25 W/m -K
2

Ta = 20°C q ′′

15 cm
22 cm
ha = 25 W/m -K
2

3 cm Ta = 20°C
ρ = 1000 kg/m3
c = 200 J/kg-K
h jet = 250 W/m -K
2

k = 30 W/m-K
Ta = 20°C
Figure P3.10-1(b): 2-D representation of disk brake

The dimensions of the brake and boundary conditions are shown in Figure P3.10-1(b); the
friction between the disk and the brake causes a spatially uniform heat flux that varies with time
according to:

⎛ ⎛ t [s ] ⎞ 2 ⎞
q ′′ = 200000 ⎡⎣ W/m ⎤⎦ ⎜1 − ⎜⎜
2
⎟ ⎟
⎜ ⎝ 50 [s ] ⎟⎠ ⎟
⎝ ⎠

You may neglect the contribution of the shaft (i.e., assume that the brake is just a disk). The disk
is initially at a uniform temperature of 20°C. The density of the disk material is ρ = 1000 kg/m3,
the conductivity is k = 30 W/m-K, and the specific heat capacity is c = 200 J/kg-K.
a.) Develop a FEHT model that can predict the temperature distribution in the disk as a function
of time during the 50 s that is required for the rotating machine to stop.

The problem is specified as being in cylindrical coordinates, transient with temperature in


Celsius (Setup menu, Figure 3).

Figure 3: Setup menu.

The geometry is setup using a 1 cm x 1 cm grid where 1 cm of screen size = 1 cm of drawing


space. Note that two outlines are required in order to specify the two discrete boundary
conditions along the top surface (Fig. 4). Roughly place the corners of the outline (Fig. 4(a)) and
then double-click on each corner to place it more precisely (Fig. 4(b)).

(a)

(b)
Figure 4: Outlines (a) with nodes roughly placed, and (b) with nodes precisely positioned.

Specify the material properties within each of the outlines (select both outlines using the shift
key and then select Material Properties from the Specify menu, Fig. 5).
Figure 5: Specify material properties

Specify the boundary conditions associated with convection; select the two surfaces exposed to
ambient air and the select Boundary Conditions from the Specify menu (Fig. 6).

Figure 6: Specify convection boundary conditions.

Repeat for the lower surface exposed to the air jet. Finally, select the surface exposed to the
frictional heating and specify the boundary condition as a function of time (Fig. 7).
Figure 7: Specify frictional heating boundary condition.

Setup a crude mesh by selecting Element Lines from the Draw menu; remember that all elements
must be triangular and it is best if they are about the same size. Figure 8 illustrates one
possibility.

Figure 8: A crude mesh for the problem.

Specify the initial temperature by selecting both outlines and then selecting Initial Temperature
from the Specify menu (Fig. 9); make sure to indicate that all nodes in all materials should be set.

Figure 9: Specify initial temperature.

You can check to make sure that the problem is fully specified by selecting Check from the Run
menu. The problem should say that there are no errors found and list the number of elements.
The critical time step should also be provided.

Run the problem by selecting Calculate from the Run menu. The Transient Calculation Setup
dialog will require that you specify the time span to be simulated and the solution technique.
Simulate the entire stopping process (0 to 50 s) and use a Crank-Nicolson technique with a 1
second time step (Fig. 10).

Figure 10: Transient Calculation Setup dialog window.

Select OK and the calculations will proceed; at the conclusion of the calculations a message will
appear specifying the number of unknown temperature and other information. Select Continue
and view the results by selecting Temperature Contours from the View menu. Select from start
to stop rather than a particular time and watch the animation of the temperature distribution (Fig.
11).

Figure 11: Temperature Contour animation setup

b.) Plot the maximum temperature in the disk as a function of time for two values of the number
of nodes in order to demonstrate that your mesh is sufficiently refined. You will need to
generate two plots and the comparison will be qualitative.

You should refine your mesh a few times (Fig. 12 shows the mesh refined twice, select Reduce
Mesh from the Draw menu twice) so that you are close to being adequately refined.
Figure 12: Mesh refined twice

Select Calculate and then select the node at the upper right corner of the model (the hottest
node). The node should blink; select Temperature vs Time from the View menu and the number
of the selected node (5 for my model, Fig. 13) will appear in the Nodes box. You can select
additional nodes as you like.

Figure 13: Temperatures vs Time Plot Setup

Select OK to generate a temperature vs time history for the mesh with 112 nodes, Fig. 14.

Figure 14: Hottest temperature in the disk as a function of time.


Repeat the process after refining the mesh to generate Figure 15.

Figure 15: Hottest temperature in the disk as a function of time for a refined mesh.

The two solutions agree relatively well indicating that the result is insensitive to grid size.
Notice the time step to time step “wiggles” have gotten worse as the grid is resolved; this effect
can be mitigated by choosing a smaller time step; for example, Fig. 16 illustrates the temperature
as a function of time with a time step of 0.01 s.

Figure 15: Hottest temperature in the disk as a function of time for a refined mesh and a 0.01 s time
step.
c.) Prepare a contour plot showing the temperature distribution at t = 10 s, t = 25 s, and t= 50 s.
You may also want to animate your temperature contours by selecting Temperature Contours
from the View menu and selecting From start to stop.

To generate these plots, select Temperature Contours from the View menu and the At time=10 s
(for example) as shown in Fig. 16.

Figure 16: Temperature Contour dialog window.

Figure 17 illustrates the contours at the indicated times; notice that the same scale is used for all
three times so that an accurate comparison can be made.

(a)

(b)
(c)

(d)
Figure 17: Temperature contours at (a) 10 s, (b) 25 s, and (c) 50 s with the color scale shown in (d).

d.) Plot the temperature on the lower surface (the surface exposed to the jets of air) at various
locations as a function of time. Explain the shape of the plot – does the result make physical
sense to you based on any time constants that you can compute?

Figure 18(b) illustrates the temperature as a function of time at the nodes shown in Fig. 18(a).

(a)

(b)
Figure 18: Temperature as a function of time (b) for the nodes shown in (a).
The conduction process within the disk is governed by a diffusion. The nodes along the bottom
surface are approximately s = 3 cm apart; the diffusive time constant associated with heat
moving from one node to the next is:

s2
τ= (1)

where α is the thermal diffusivity of the disk material. Equation (1) indicates that there should
be a time delay of approximately 2 s which is approximately consistent with the lag between
peaks.
Problem 4.1-1: Plate with a Constant Heat Flux
Figure P4.1-1 illustrates a fluid flowing over a flat plate subjected to a constant heat flux, qs′′ .
The free stream temperature is T∞.

y
fluid at T∞

heat flux, qs′′


x

Figure P4.1-1: Fluid flowing over a plate that is subjected to a constant heat flux.

a.) Sketch the thermal boundary layer thickness (δt) as a function of position, x, from the leading
edge of the plate. Assume that the flow is laminar.

Figure 2: Thermal boundary layer thickness as a function of position for a fluid flowing over
a plate with a constant heat flux under laminar conditions.

The thermal boundary layer will grow approximately according to x due to the penetration of
the thermal diffusion wave into the free stream. This behavior is illustrated in Figure 2.

b.) Sketch the surface temperature of the plate (Ts) as a function of position, x, from the leading
edge of the plate. Assume that the flow is laminar.

Figure 3: Plate surface temperature as a function of position for a fluid flowing over a plate
with a constant heat flux under laminar conditions.
The thermal resistance between the plate surface and the free stream temperature is
approximately equal to δt/k where k is the conductivity of the fluid. Therefore, the plate surface
temperature will be given by:

q ′′ δ t
Ts ≈ T∞ + (1)
k

Examination of Figure 2 indicates that the surface temperature will start at the free stream
temperature and the plate-to-free stream temperature difference will increase in proportion to the
thermal boundary layer. This behavior is illustrated in Figure 3.
Problem 4.1-2 (4-1 in text): Water Flowing over a Plate
Water at atmospheric pressure, free stream velocity u∞ = 1.0 m/s and temperature T∞ = 25°C
flows over a flat plate with a surface temperature Ts = 90°C. The plate is L = 0.15 m long.
Assume that the flow is laminar over the entire length of the plate.
a.) Estimate, using your knowledge of how boundary layers grow, the size of the momentum and
thermal boundary layers at the trailing edge of the plate (i.e., at x = L). Do not use a
correlation from your book, instead use the approximate model for boundary layer growth.

The inputs are entered in EES:

$UnitSystem SI MASS RAD PA K J


$TABSTOPS 0.2 0.4 0.6 0.8 3.5 in

T_s=converttemp(C,K,90) "surface temperature"


T_inf=converttemp(C,K,25) "free stream temperature"
P=1 [atm]*convert(atm,Pa) "pressure"
L=0.15 [m] "plate length"
u_inf=1.0 [m/s] "free stream velocity"

The film temperature:

Ts + T∞
T film = (1)
2

is used to evaluate the properties of water (k, μ, ρ, and c) using EES’ built-in property functions:

T_film=(T_s+T_inf)/2 "film temperature"


k=conductivity(Water,T=T_film,P=P) "conductivity"
mu=viscosity(Water,T=T_film,P=P) "viscosity"
rho=density(Water,T=T_film,P=P) "density"
c=cP(Water,T=T_film,P=P) "specific heat capacity"

The kinematic viscosity and thermal diffusivity are computed:

μ
ν= (2)
ρ

k
α= (3)
ρc

nu=mu/rho "kinematic viscosity"


alpha=k/(rho*c) "thermal diffusivity"

The time that the free stream is in contact with the plate is computed:

L
t= (4)
u∞
and used to compute the approximate momentum and thermal boundary layer thicknesses at the
plate’s trailing edge:

δm ≈ 2 ν t (5)

δt ≈ 2 α t (6)

time=L/u_inf "time required to reach back of plate"


delta_m=2*sqrt(nu*time) “approximate size of momentum boundary layer"
delta_t=2*sqrt(alpha*time) "approximate size of thermal boundary layer"

which leads to δm = 0.54 mm and δt = 0.31 mm.

b.) Use your answer from (a) to estimate the shear stress at the trailing edge of the plate and the
heat transfer coefficient at the trailing edge of the plate.

The shear stress at the trailing edge of the plate can be approximated according to:

u∞
τs ≈ μ (7)
δm

and the heat transfer coefficient can be approximated according to:

k
h≈ (8)
δt
tau_app=mu*u_inf/delta_m "approximate value of the shear stress at the end of the plate"
h_app=k/delta_t "approximate value of the heat transfer coefficient at the end of the plate"

which leads to τs = 0.89 Pa and h = 2090 W/m2-K.

c.) You measure a shear stress of τs,meas = 1.0 Pa at the trailing edge of the plate; use the
Modified Reynolds Analogy to predict the heat transfer coefficient at this location.

The Modified Reynolds Analogy relates shear to heat transfer coefficient according to:
kτ 1
h ≈ s ,meas Pr 3 (9)
μ u∞
tau_meas=1.0 [Pa] "measured value of the shear stress"
Pr=nu/alpha "Prandtl number"
h_MRA=k*tau_meas*Pr^(1/3)/(mu*u_inf)
"heat transfer coefficient based on the Modified Reynolds Analogy"

which leads to h = 1940 W/m2-K.


Problem 4.1-3
A sun roof is installed in the roof of a car, as shown in Figure P4.1-3.

ρ = 1.3 kg/m 3
T∞ = −15°C k = 0.023 W/m-K
u∞ = 10m/s μ = 1.65x10 -5 Pa -s
c = 1003 J /kg-K
s un roof with
de fros t he a te r

L1 = 0.3 m
L2 = 0.6 m
Figure P4.1-3: Sun roof.

A defrost heater is integrated with the sun roof glass in order to prevent ice from forming. The
sun roof begins at L1 = 0.3 m from the front of the roof and extends to L2 = 0.6 m. The roof is W
= 1.2 m wide. The car is driving at u∞ = 10 m/s through air at T∞ = -15°C. Assume that the
properties of the air are conductivity k = 0.023 W/m-K, density ρ = 1.3 kg/m3, viscosity μ =
1.65x10-5 Pa-s, and specific heat capacity c = 1003 J/kg-K. The defrost heater is a thin sheet of
electrically resistive material that covers the entire sun roof and transfers a uniform heat flux to
the glass. The convection coefficient between the interior surface of the roof and the cabin air is
sufficiently low that convection heat transfer with the interior can be neglected. Assume that the
flow over the roof remains laminar. Do not use a correlation to solve this problem; instead, use
the conceptual knowledge of boundary layer behavior to estimate the answers.
a.) What is the total power required to keep the sun roof above Tice = 0°C?

The inputs are entered in EES:

$UnitSystem SI MASS RAD PA K J


$TABSTOPS 0.2 0.4 0.6 0.8 3.5 in

L_1=0.3 [m] "location of the beginning of the sun roof"


L_2=0.6 [m] "location of the end of the sun roof"
W=1.2 [m] "width of roof"
T_infinity=converttemp(C,K,-15[C]) "ambient air temperature"
u_infinity=10 [m/s] "car velocity"
rho=1.3 [kg/m^3] "density of air"
mu=1.65e-5 [Pa-s] "viscosity of air"
k=0.023 [W/m-K] "conductivity of air"
c=1003 [J/kg-K] "specific heat capacity of air"

The Reynolds number at the trailing edge of sun roof is calculated:

ρ L2 u∞
ReL2 = (1)
μ

Re_2=u_infinity*L_2*rho/mu "Reynolds number at end of sun roof"


which leads to ReL2 = 4.7x105, this is very near the critical Reynolds number for transition to
turbulence. For this problem, we will assume that the flow remains laminar. Therefore, the
largest heat transfer coefficient will be at the leading edge of the sun roof where the thermal
boundary layer thickness is smallest. The thermal boundary layer thickness at the leading edge
of the sun roof is estimated according to:

L1
δ t ,1 ≈ 2 α (2)
u∞

where α is the thermal diffusivity.

alpha=k/(rho*c) "thermal diffusivity"


delta_t_1=2*sqrt(alpha*L_1/u_infinity) "thermal boundary layer thickness at beginning of sun roof"

The heat flux required to keep the temperature at the leading edge of the sun roof above the
temperature of ice can be estimated according to:

k
q ′′ ≈ (Tice − T= ) (3)
δ t ,1

The total power (assuming that the heat flux is uniform) is:

q = ( L2 − L1 ) W q ′′ (4)

T_ice=converttemp(C,K,0 [C]) "temperature required to keep window free of ice"


q``=(T_ice-T_infinity)*k/delta_t_1 "heat flux required to keep window free of ice"
q_dot_total=q``*(L_2-L_1)*W "total heat transfer required"

which leads to q = 85 W.

b.) Where is the hottest temperature on the sun roof surface? Estimate this temperature.

If the heat flux is uniform and the flow is everywhere laminar, then the hottest temperature will
be at the trailing edge of the sun roof where the thermal boundary layer is largest. The boundary
layer thickness at the trailing edge of the sun roof can be estimated according to:

L2
δ t ,2 ≈ 2 α (5)
u∞

The temperature at the trailing edge of the boundary layer can be estimated according to:

q ′′ δ t ,2
Tx = L2 ≈ T∞ + (6)
k
delta_t_2=2*sqrt(alpha*L_2/u_infinity) "thermal boundary layer thickness at end of sun roof"
T_L_2=T_infinity+q``*delta_t_2/k "temperature at end of sun roof"
T_L_2_C=converttemp(K,C,T_L_2) "in C"

which leads to Tx = L2 = 6.2°C.


Problem 4.1-4
Air flows over two, thin flat plate arrangements (A and B in Figure P4.1-4). The single plate in
arrangement A is twice as long as the two plates in arrangement B. The plates have the same
width (into the page), the same uniform surface temperature, and are exposed to the same free-
stream flow. The flow over the plates is laminar in both arrangements.

Ts
u∞ , T∞
2L
Arrangement A

u∞ , T∞ Ts

Arrangement B
Figure P4.1-4: Air flowing over a single plate (Arrangement A) and two plates that are half the length
(Arrangement B). The width of the plates (into the page), surface temperature, and free stream conditions
are the same for both arrangements.

a.) Do you expect the total rate of heat transfer to the air to be higher for arrangement A or B?
Justify your answer.

The total rate of heat transfer to the air will be higher for arrangement B because the thermal
boundary layer will have less time to develop and therefore will be thinner, on average. The
thermal resistance of the thermal boundary layer will be less for arrangement B and the rate of
heat transfer will be higher.
Problem 4.1-5
Two identical plates are oriented parallel to a flow, as shown in Figure P4.1-5.

u∞ , T∞ L = 2W u∞ , T∞ W
L = 2W
W

Ts Ts
u∞ , T∞ u∞ , T∞

plate A plate B
Figure P4.1-5: Plates A and B.

Plate A is oriented with its long dimension, L, oriented with the flow and its short dimension, W,
oriented perpendicular to the flow. Plate B is oriented with its long dimension perpendicular to
the flow. In both cases, L= 2 W. Assume that the flow remains laminar in both cases. The free
stream velocity and temperature, u∞ and T∞, and the plate temperature, Ts, are the same for both
cases.
a.) Estimate the ratio of the total rate of heat transfer from plate A to the total rate of heat
transfer from plate B. Do not use a correlation to answer this question.

The local heat transfer coefficient can be estimated according to:

k
q ′′ ≈ (Ts − T∞ ) (1)
δt

where δt is the thermal boundary layer thickness, which can be estimated according to:

x
δt ≈ 2 α (2)
u∞

where x is the distance from the leading edge of the plate. Substituting Eq. (2) into Eq. (1) leads
to:

k u
q ′′ ≈ (Ts − T∞ ) α ∞ (3)
2 x

The total rate of heat transfer for plate A is therefore approximately:

W W
k u
q A ≈ ∫ q ′′ 2 W dx = ∫ (Ts − T∞ ) α ∞ 2W dx (4)
0 0
2 x
Carrying out the integral in Eq. (4) leads to:
W
k
q A ≈ (Ts − T∞ ) α u∞ 2 W ∫ x −0.5 dx = (Ts − T∞ ) k α u∞ 2W 1.5 (5)
2 0

The total rate of heat transfer for plate B is approximately:

2W 2W
k u
q B ≈ ∫
0
q ′′W dx = ∫ (T
0
s − T∞ )
2
α ∞ W dx
x
(6)

Carrying out the integral in Eq. (6) leads to:

2W
k
q B ≈ (Ts − T∞ ) α u∞ W ∫x
−0.5
dx = (Ts − T∞ ) k α u∞ W ( 2 W )
0.5
(7)
2 0

The ratio of Eq. (5) to Eq. (7) is:

q A

(Ts − T∞ ) k α u∞ 2W 1.5 = 2 = 2 (8)
q B (Ts − T∞ ) k α u∞ W ( 2 W )0.5 2
Problem 4.1-6 (4-2 in text)
Figure P4.1-6 illustrates the flow of a fluid with T∞ = 0ºC, u∞ = 1 m/s over a flat plate.

fluid at T∞ = 0°C, u∞ = 1 m/s


Pr = 1, k = 1 W/m-K, α = 1x10 m /s
-3 2

y q ′′ = 1000 W/m
2

x
1 2 3 4
adiabatic constant heat flux adiabatic
L=1m L=1m L=1m
Figure P4.1-6: Flow over a flat plate.

The flat plate is made up of three sections, each with length L = 1 m. The first and last sections
are insulated and the middle section is exposed to a constant heat flux, q ′′ = 1000 W/m2. The
properties of the fluid are Prandtl number Pr = 1, conductivity k = 1 W/m-K, and thermal
diffusivity α = 1x10-3 m2/s. Assume that the flow is laminar over the entire surface.
a.) Sketch the momentum and thermal boundary layers as a function of position, x. Do not
worry about the qualitative characteristics of your sketch - get the quantitative characteristics
correct.

boundary layer thickness


δm

δt

adiabatic constant heat flux adiabatic


Figure P4.1-6(b): Sketch of the momentum and thermal boundary layer thickness as a function of position.

The momentum boundary layer develops from the front of the plate and grows according to the
square root of x:

x
δm ≈ 2 υ t = 2 υ (1)
u∞

The thermal boundary layer develops from the front of the heated portion of the plate and also
grows according to the square root of x (measured related to the leading edge of the heated
plate):

δt ≈ 2 α t = 2 α
( x − L) (2)
u∞
Because the Prandtl number is 1, the momentum and thermal boundary layers develop at the
same rate although they start at different positions. Note that the thermal boundary layer
continues to grow downstream of the heated plate because the energy added to the boundary
layer continues to diffuse into the free stream.

b.) Sketch the temperature distribution (the temperature as a function of distance from the plate
y) at the 4 locations indicated in Figure P4.1-6. Location 1 is half-way through the first
adiabatic region, Location 2 is half-way through the heated region, Location 3 is at the
trailing edge of the heated region (in the heated region), and Location 4 is at the trailing edge
of the final adiabatic region. Again, focus on getting as many of the qualitative
characteristics of your sketch correct as you can.

yT y T∞ yT yT
∞ ∞ ∞

δt

T T T T

1 2 3
3 3
4
adiabatic constant heat flux adiabatic
Figure P4.1-6(c): Sketch of the temperature distribution at the four locations shown in Figure P4.1-6(a).

The temperature distribution at location 1, in the initial unheated region, is uniform as there is
nothing to add or subtract energy from the flow. The temperature distribution at locations 2 and
3 are similar in shape. The temperature of the surface is elevated relative to the free stream due
to the resistance of the boundary layer. The temperature gradient at the surface is the same at
both locations 2 and 3 because the heat flux is the same. The surface temperature should be
higher at location 3 because the resistance of the boundary layer is larger. The temperature
distribution at location 4 should extend further out into the free stream due to the increased
thickness of the thermal boundary layer. The slope of the temperature gradient at the plate and
the outer edge of the boundary layer should be zero. By conservation of energy, the average
temperature of the boundary layer should drop at location 4 because the boundary layer has
grown and no additional energy has been added.

c.) Sketch the temperature of the surface of the plate as a function of position, x. Get the
qualitative features of your sketch correct.
surface temperature

T∞
x

adiabatic constant heat flux adiabatic


Figure P4.1-6(d): Sketch of the surface temperature of the plate as a function of position.

The surface temperature is equal to the free stream temperature in the initial unheated region (0 <
x < L). The surface temperature increases approximately proportionally to the thermal boundary
layer thickness according to:

Ts = T∞ +
q ′′δ t q ′′
= T∞ + 2 t α
( x − L) (3)
k k u∞

In the final unheated region, the surface temperature drops as the energy that was added in the
heated region diffuses into the free stream and the boundary layer grows. By conservation of
energy, the average temperature of the boundary layer should change according to:

u∞
q ′′ L ≈ ρ c δ t (T − T∞ ) (4)
2

which leads to:

2 q ′′ L q ′′ L u∞
T ≈ T∞ + = T∞ + (5)
u∞ ρ c δ t u∞ ρ c α ( x − L )

and therefore the surface temperature will drop like x-0.5.

d.) Predict, approximately, the temperature of the surface at locations 1, 2, 3, and 4 in Figure
P4.1-6. Do not use a correlation. Instead, use your conceptual understanding of how
boundary layers behave to come up with very approximate estimates of these temperatures.

The surface temperature at location 1 is Ts,1 = T∞ = 0ºC.

The thermal boundary layer thickness at locations 2, 3 and 4 can be predicted approximately
using Eq. (2):
0.5 L 1x10-3 m 2 0.5 (1 m ) s
δ t ,2 ≈ 2 α =2 = 0.0447 m (6)
u∞ s 1m

L 1x10-3 m 2 1 m s
δ t ,3 ≈ 2 α =2 = 0.0633 m (7)
u∞ s 1m

2L 1x10-3 m 2 2 (1 m ) s
δ t ,4 ≈ 2 α =2 = 0.0894 m (8)
u∞ s 1m

The temperature of the surface at locations 2 and 3 can be predicted approximately using Eq. (3):

1000 W 0.0447 m m-K


Ts ,2 = 0°C + = 44.72°C (9)
m2 1W

1000 W 0.0633 m m-K


Ts ,3 = 0°C + = 63.25°C (10)
m2 1W

The temperature of the surface at location 4 can be predicted approximately using Eq. (5),
substituting ρ c = k/α:

2 q ′′ L α 1000 W 1 m s 1x10-3 m 2 m-K


Ts ,4 ≈ T∞ + = 0°C + 2 = 22.36°C (11)
u∞ k δ t ,4 m2 1m s 1 W 0.0894 m
Problem 4.3-1
You have fabricated a 1/10th scale model of a new type of automobile and placed it in a wind
tunnel. You want to test it at an air velocity that will allow you to compute the drag force on the
car when it travels at 55 miles per hour.
a.) What air velocity should you choose and how is the drag force that you measure on the
model related to the drag force on the automobile?

In order for the results of the test to be applicable, the test and the full scale car must be at the
same Reynolds number. Therefore:

uLρ us Ls ρ
Re = = (1)
μ μ

where u and L are the velocity and length scale of the car and us and Ls are the velocity and
length scale of the test. Therefore, assuming that the density and viscosity of the fluid passing
over the full scale car and the test piece are the same:

L 1 55 mile 1609 m hr m
us = u= = 246 (2)
Ls (1/10 ) hr mile 3600 s s

Once you’ve matched the Reynolds number, the coefficient of drag that you measure should be
the same as it will be for the full-size automobile. Therefore:

F Fs
CD = = (3)
1 1
ρ u 2 L2 ρ us2 L2s
2 2

where F and Fs are the full-scale and test force, respectively. Solving Eq. (3) for the drag force
on the full scale automobile:
2 2
⎛ L⎞ ⎛u⎞
2 2
⎛ 10 ⎞ ⎛ 1 ⎞
F = Fs ⎜ ⎟ ⎜ ⎟ = Fs ⎜ ⎟ ⎜ ⎟ = Fs (4)
⎝ Ls ⎠ ⎝ us ⎠ ⎝ 1 ⎠ ⎝ 10 ⎠
Problem 4.3-2 (4-3 in text)
You have fabricated a 1000x scale model of a microscale feature that is to be used in a
microchip. The device itself is only 1 μm in size and is therefore too small to test accurately.
However, you’d like to know the heat transfer coefficient between the device and an air flow that
has a velocity of 10 m/s.
a.) What velocity should you use for the test and how will the measured heat transfer coefficient
be related to the actual one?

In order for the results of the test to be applicable, the test and the actual device must be at the
same Reynolds number and Prandtl number. Therefore:

uLρ us Ls ρ
Re = = (1)
μ μ

where u and L are the velocity and length scale of the actual device and us and Ls are the velocity
and length scale of the test device. Solving Eq. (1) for the velocity to use in the text leads to:

L 1 m
us = u= 10 = 0.01 (2)
Ls 1000 s

If you use air for both tests then the Prandtl number will also be matched. Once you’ve matched
the Reynolds number and Prandtl number, the Nusselt number that you measure should be the
same as the Nusselt number for the actual device. Therefore:

h L hs Ls
Nu = = (3)
k k

where h and hs are the full-scale and test heat transfer coefficients, respectively. Solving Eq. (3)
for the full-scale heat transfer coefficient:

Ls
h = hs = 1000 hs (4)
L
Problem 4.3-3 (4-4 in text)
Your company has come up with a randomly packed fibrous material that could be used as a
regenerator packing. Currently there are no correlations available that would allow the
prediction of the heat transfer coefficient for the packing. Therefore, you have carried out a
series of tests to measure the heat transfer coefficient. A Dbed = 2 cm diameter bed is filled with
these fibers with diameter dfiber = 200 μm. The nominal temperature and pressure of the testing
is Tnom = 20ºC and pnom = 1 atm, respectively. The mass flow rate of the test fluid, m , is varied
and the heat transfer coefficient is measured. Several fluids, including air, water, and ethanol,
are used for testing. The data are shown in Table 4.3-3; the data can be downloaded from the
website as EES lookup tables (P4p3-3_air.lkt, P4p3-3_ethanol.lkt, and P4p3-3_water.lkt).

Table 4.3-3: Heat transfer data.


Air Water Ethanol
Mass flow Heat transfer Mass flow Heat transfer Mass flow Heat transfer
rate (kg/s) coefficient rate (kg/s) coefficient rate (kg/s) coefficient
(W/m2-K) (W/m2-K) (W/m2-K)
0.0001454 170.7 0.00787 8464 0.009124 4162
0.0004073 311.9 0.02204 15470 0.02555 7607
0.0006691 413.7 0.0362 20515 0.04197 10088
0.0009309 491.8 0.05037 24391 0.05839 11993
0.001193 572.7 0.06454 28399 0.07481 13964
0.001454 631.1 0.0787 31296 0.09124 15388

a.) Plot the heat transfer coefficient as a function of mass flow rate for the three different
different test fluids.

The data is loaded in EES as three lookup tables and used to prepare Figure P4.3-3(a).
35000
water
Heat transfer coefficient (W/m -K)

10000
2

ethanol

1000
air

100

10
0.0001 0.001 0.01 0.08
Mass flow rate (kg/s)
Figure P4.3-3(a): Heat transfer coefficient as a function of the mass flow rate.

b.) Plot the Nusselt number as a function of the Reynolds number for the three different test
fluids. Use the fiber diameter as the characteristic length and the free-flow velocity (i.e., the
velocity in the bed if it were empty) as the characteristic velocity.

The inputs are entered:


$UnitSystem SI MASS RAD PA K J
$Tabstops 0.2 0.4 0.6 0.8 3.5

d_fiber=200 [micron]*convert(micron,m) "Fiber diameter"


D_bed=2 [cm]*convert(cm,m) "Bed diameter"
T_nom=converttemp(C,K,20[C]) "Test temperature"
P_nom=1 [atm]*convert(atm,Pa) "Test pressure"

The data from the lookup tables is processed row by row. The mass flow rate and heat transfer
coefficient are read from the first row of the table and the appropriate fluid is set:

i=1 [-] "Run number"


m_dot=Lookup('Ethanol',i,1) "Mass flow rate"
h=Lookup('Ethanol',i,2) "Mass flow rate"
F$='Ethanol' "Test fluid"

The fluid properties (k, μ, Pr, and ρ) are computed using the internal property routines:

rho=density(F$,T=T_nom,P=P_nom) "density"
k=conductivity(F$,T=T_nom,P=P_nom) "conductivity"
mu=viscosity(F$,T=T_nom,P=P_nom) "viscosity"
Pr=Prandtl(F$,T=T_nom,P=P_nom) "Prandtl number"

The characteristic velocity is calculated according to:

4 m
uchar = (1)
π Dbed
2
ρ

The Reynolds number and Nusselt number are computed according to:

uchar d fiber ρ
Re = (2)
μ

d fiber h
Nu = (3)
k

u_char=m_dot/(rho*pi*D_bed^2/4) "characteristic velocity"


Re=u_char*d_fiber*rho/mu "Reynolds number"
Nusselt=h*d_fiber/k "Nusselt number"

Three parametric tables are set up (one for each fluid); the parametric table for air is shown in
Figure P4.3-3(b). The first column is the row and includes the index i which is varied from 1 to
6 (one for each row of the corresponding lookup table).
Figure P4.3-3(b): Parametric table for the air data reduction.

Figure P4.3-3(c) illustrates the Nusselt number as a function of Reynolds number for various
fluids (i.e., for various values of the Prandtl number).
15

12.5
Nusselt number

ethanol
10
water
7.5

5
air

2.5

0
0 10 20 30 40 50
Reynolds number
Figure P4.3-3(c): Nusselt number as a function of Reynolds number for various fluids.

c.) Correlate the data for all of the fluids using a function of the form: Nu = a Reb Pr c . Note
that you will want to transform the results using a natural logarithm and use the Linear
Regression option from the Tables menu to determine a, b, and c.

In order to use linear regression it is necessary to have a linear equation. Therefore, the proposed
equation is transformed by taking the natural logarithm of both sides:

ln ( Nu ) = ln ( a ) + b ln ( Re ) + c ln ( Pr ) (4)

All of the Nusselt number, Reynolds number, and Prandtl number data are put into one table;
three additional columns are added that contain the natural logarithm of these quantities (note
that this can be accomplished by selecting Alter Values and Enter Equation), as shown in FIgure
P4.3-3(d).
Figure P4.3-3(d): Nusselt number as a function of Reynolds number for various fluids.

The linear regression option is selected from the Tables menu. The dependent variable is the
natural log of the Nusselt number while the independent variables are the natural logarithm of
the Reynolds number and Prandtl number. A first order polynomial is selected, as indicated by
Eq. (4). The dialog should be as shown in Figure P4.3-3(e).

Figure P4.3-3(e): Linear Regression dialog.

The result is:

Nu = 0.6044 Re0.5692 Pr 0.3333 (5)

d.) Use your correlation to estimate the heat transfer coefficient for 20 kg/s of oil passing
through a 50 cm diameter bed composed of fibers with 2 mm diameter. The oil has density
875 kg/m3, viscosity 0.018 Pa-s, conductivity 0.14 W/m-K, and Prandtl number 20.

The inputs are entered in EES:


m_dot=20 [kg/s] "Mass flow rate"
d_fiber=2 [mm]*convert(mm,m) "Fiber diameter"
D_bed=50 [cm]*convert(cm,m) "Bed diameter"
rho=875 [kg/m^3] "density"
k=0.14 [W/m-K] "conductivity"
mu=0.018 [Pa-s] "viscosity"
Pr=20 "Prandtl number"

The characteristic velocity and Reynolds number are calculated:

4 m
uchar = (6)
π Dbed
2
ρ

uchar d fiber ρ
Re = (7)
μ

u_char=m_dot/(rho*pi*D_bed^2/4) "characteristic velocity"


Re=u_char*d_fiber*rho/mu "Reynolds number"

The correlation, Eq. (5), is used to obtain the Nusselt number. The heat transfer coefficient is
computed according to:

k
h = Nu (8)
d fiber

Nusselt=0.6044*Re^0.5692*Pr^0.333 "Nusselt number correlation"


h=Nusselt*k/d_fiber "heat transfer coefficient"

which leads to h = 456.5 W/m2-K.


Problem 4.3-4
The rate equations that govern the shear stress in a Newtonian fluid are given by Bejan (1993)
and elsewhere. The tangential stress and excess normal stress are given by:
⎛ ∂u ∂v ⎞ ∂u
τ yx = μ ⎜ + ⎟ , σ xx = 2 μ
⎝ ∂y ∂x ⎠ ∂x
a.) Show that these rate equations can be substituted into the x-momentum balance and leads to
the x-momentum equation.

The momentum balance in the x-direction on the differential control volume derived in Section
4.2.1 and shown in Figure 4-6 leads to:

∂τ yx ∂σ ∂ (u ) ∂ (u v )
2
∂p ∂u
N
ρ gx + + xx = ρ +ρ + + ρ (1)
∂y ∂x ∂x ∂ y ∂
N x ∂t
N
gravity force 

pressure force rate of
stresses momentum transfer momentum storage

The tangential stress is given by:

⎛ ∂u ∂v ⎞
τ yx = μ ⎜ + ⎟ (2)
⎝ ∂y ∂x ⎠

and the excess normal stress is given by:

∂u
σ xx = 2 μ (3)
∂x

Equations (2) and (3) are substituted into the appropriate terms in Eq. (1):

∂τ yx ∂σ xx ∂ ⎡ ⎛ ∂u ∂v ⎞ ⎤ ∂ ⎡ ∂u ⎤
+ = ⎢μ ⎜ + ⎟⎥ + ⎢2 μ ⎥ (4)
∂y ∂x ∂y ⎣ ⎝ ∂y ∂x ⎠ ⎦ ∂x ⎣ ∂x ⎦

Equation (4) is expanded:

∂τ yx ∂σ xx ∂ 2u ∂ 2v ∂ 2u
+ =μ 2 +μ + 2μ 2 (5)
∂y ∂x ∂y ∂x ∂y ∂x

Equation (5) is rearranged:

∂τ yx ∂σ xx ∂ 2u ∂ 2u ∂ ⎛ ∂u ∂v ⎞
+ =μ 2 +μ 2 +μ ⎜ + ⎟ (6)
∂y ∂x ∂y ∂x ∂x ⎝ ∂x ∂y ⎠


= 0 by continuity
The last term in Eq. (6) must be zero according to continuity. Equation (6) is substituted into Eq.
(1).

∂ 2u ∂ 2u ∂ (u 2 ) ∂ ( u v ) ∂p ∂u
ρ gx + μ 2 + μ 2 = ρ +ρ + +ρ (7)
∂y ∂x ∂x ∂y ∂x ∂t

Equation (7) is rearranged to obtain:

⎡ ∂u ∂ ( u 2 ) ∂ ( u v ) ⎤ ∂p ⎛ ∂ 2u ∂ 2u ⎞
ρ⎢ + + ⎥ = − + μ ⎜ 2 + 2 ⎟ + ρ gx (8)
⎢⎣ ∂t ∂x ∂y ⎥

∂x ⎝ ∂y ∂x ⎠

Typically, the partial differentials on the left side of Eq. (8) are expanded:

⎡ ∂u ∂u ∂v ∂u ⎤ ∂p ⎛ ∂ 2u ∂ 2u ⎞
ρ⎢ + 2u + u + v ⎥ = − + μ ⎜ 2 + 2 ⎟ + ρ gx (9)
⎣ ∂t ∂x ∂y ∂y ⎦ ∂x ⎝ ∂y ∂x ⎠

Equation (9) can be rearranged:

⎡ ⎤
⎢ ∂u ⎛ ∂u ∂v ⎞ ∂u ∂u ⎥ ∂p ⎛ ∂ 2u ∂ 2u ⎞
ρ ⎢ + u ⎜ + ⎟ + u + v ⎥ = − + μ ⎜ 2 + 2 ⎟ + ρ gx (10)
⎢ ∂t ⎝∂ x ∂y ⎠

∂x ∂y ⎥ ∂x ⎝ ∂y ∂x ⎠
⎢⎣ = 0 by continuity ⎥

The second term on the left side Eq. (10) must equal zero according to the continuity equation.
Therefore, the momentum balance in the x-direction is typically written as:

⎡ ∂u ∂u ∂u ⎤ ∂p ⎛ ∂ 2u ∂ 2u ⎞
ρ ⎢ + u + v ⎥ = − + μ ⎜ 2 + 2 ⎟ + ρ gx (11)
⎣ ∂t ∂x ∂y ⎦ ∂x ⎝ ∂y ∂x ⎠

A differential momentum balance in the y-direction leads to a similar equation:

⎡ ∂v ∂v ∂v ⎤ ∂p ⎛ ∂ 2v ∂ 2v ⎞
ρ ⎢ + u + v ⎥ = − + μ ⎜ 2 + 2 ⎟ + ρ gy (12)
⎣ ∂t ∂x ∂y ⎦ ∂y ⎝ ∂y ∂x ⎠

where gy is the component of gravity in the y-direction. Equations (11) and (12) are often
referred to as the Navier-Stokes equation. In cylindrical coordinates, Eqs. (11) and (12) become:

⎡ ∂u ∂u ∂u ⎤ ∂p ⎡ 1 ∂ ⎛ ∂u ⎞ ∂ 2u ⎤
ρ⎢ +u +v ⎥ = − + μ ⎢ ⎜ r ⎟ + 2 ⎥ + ρ gz (13)
⎣ ∂t ∂x ∂r ⎦ ∂x ⎣ r ∂r ⎝ ∂r ⎠ ∂x ⎦
⎡ ∂v ∂v ∂v ⎤ ∂p ⎡ ∂ ⎛ 1 ∂ ( r v ) ⎞ ∂ 2u ⎤
ρ⎢ +u +v ⎥ = − + μ ⎢ ⎜ ⎟+ 2 ⎥ (14)
⎣ ∂t ∂x ∂r ⎦ ∂r ⎢⎣ ∂r ⎝ r ∂r ⎠ ∂r ⎥⎦

where u and v are the velocity components in the x- and r-directions, respectively.
Problem 4.3-5
Figure P4.3-5 illustrates a micro-scale feature that will be fabricated on a micro-electro-
mechanical system (MEMS) in order to sense the characteristics of the flow of a fluid within a
microchannel.

L = μm

air flow

Figure P4.3-5: A MEMS feature for sensing flow characteristics.

In order to design the device it is necessary to understand the heat transfer coefficient associated
with the flow of air across the feature. The device is not a commonly encountered shape (e.g., a
cylinder or sphere) and therefore you are unable to locate appropriate correlations in the
literature. Further, the device only L = 1 μm in size and is therefore too small to test accurately.
Therefore, you have decided to build a scaled-up model of the feature that can be more easily
tested in a wind tunnel. The scaled up model is 10,000x larger than the MEMS feature (i.e., Ltest
= 1 cm and all of the object dimensions scale proportionally). The model is mounted in a wind
tunnel and electrical heaters are embedded in the model and apply a measured, constant heating
of qtest = 0.5 W. The model is composed of a conductive material so that it has approximately a
uniform surface temperature (Ts,test). The wind tunnel is used to provide a uniform flow of air at
T∞,test = 20°C and Ptest = 1 atm over a range of velocities (u∞,test). Table P4.3-5 summarizes the
experimental measurements; note that these data are available from the website as an EES
Lookup Table (P4p3-5 Data.lkt).

Table P4.3-5: Experimental measurements.


Air velocity, Surface
u∞,test (m/s) temperature,
Ts,test (°C)
0.15 84.39
0.30 69.82
0.45 62.88
0.60 58.55
0.75 55.50
1.13 50.55
1.50 47.47
2.25 43.64
3.00 41.25
4.50 38.29
6.00 36.45
7.50 35.14
11.25 33.03

Note that the total surface area of the model is As,test = 6.28x10-4 m2 and the surface area of the
micro-scale device is (10,000)2x smaller.
a.) Use the experimental measurements to prepare a figure that shows the Nusselt number of the
micro-scale device as a function of the Reynolds number for a Prandtl number that is
consistent with air.

The known information is entered in EES:

$UnitSystem SI MASS RAD PA K J


$TABSTOPS 0.2 0.4 0.6 0.8 3.5 in

"Inputs"
L=1 [micron]*convert(micron,m) "length scale of micro-scale device"
SF_test=10000 "scale factor for test"
L_test=SF_test*L "length scale of test device"
A_s_test=6.28e-4 [m^2] "surface area of the test device"
A_s=A_s_test/SF_test^2 "surface area of the micro-scale device"
T_infinity_test=converttemp(C,K,20) "test temperature"
P_test=1[atm]*convert(atm,Pa) "test pressure"
rho=100 [kg/m^3] "density of gas in micro-scale device"
mu=3.2e-5 [Pa-s] "viscosity of gas in micro-scale device"
k=0.05 [W/m-K] "conductivity of gas in micro-scale device"

The properties of the air in the wind tunnel (ρtest, μtest, and ktest) are obtained using EES’ built-in
property routines.

"Air properties"
mu_test=viscosity(Air,T=T_infinity_test) "viscosity of test gas"
rho_test=density(Air,T=T_infinity_test,P=P_test) "density of test gas"
k_test=conductivity(Air,T=T_infinity_test) "conductivity of test gas"

The Lookup Table containing the data is opened in EES; select Open Lookup Table from the
Tables menu and open the table. The values for each run in the table may be accessed by setting
a variable run that corresponds to the row number and using the Lookup command:

Run=1 "row of data table"


u_infinity_test=Lookup('P33-1 Data',Run,'u_infinity_test') "velocity in test"
T_s_test_C=Lookup('P33-1 Data',Run,'T_s_test') "surface temperature in test in C"
T_s_test=converttemp(C,K,T_s_test_C) "surface temperature in test"

The test data may be used to compute the Reynolds number according to:

ρtest Ltest u∞ ,test


Re = (1)
μtest

Re=rho_test*u_infinity_test*L_test/mu_test "Reynolds number associated with test condition"

The test data may also be used to compute a heat transfer coefficient and, from that, a Nusselt
number:
qtest
htest = (2)
As ,test (Ts ,test − T∞ ,test )

htest Ltest
Nu = (3)
ktest

h_test=q_dot_test/(A_s_test*(T_s_test-T_infinity_test)) "heat transfer coefficient"


Nusselt=h_test*L_test/k_test "Nusselt number associated with test condition"

A Parametric Table is setup containing the variables Run, Re, and Nusselt; the variable Run is set
to run from 1 to 13 (the number of data points) and the value of Run in the Equation Window is
commented out. The result of solving the table is shown in Fig. 2.

Figure 2: Parametric Table containing the Reynolds number, Nusselt number results.

Figure 3 illustrates the plot requested in the problem statement; the Nusselt number as a function
of the Reynolds number for a Prandt number of 0.7. Curves for other Prandtl numbers could be
possibly obtained using other fluids than air in the test facility.
Figure 3: Nusselt number as a function of Reynolds number for Pr= 0.7.

b.) Prepare a figure that shows the heat transfer coefficient for the micro-scale device as a
function of the gas velocity across the device for air with T∞ = 80ºC and p = 10 atm.

Each data point corresponds to a unique Reynolds number; the Reynolds number can be used to
compute the velocity of gas in the micro-scale device (u∞) according to:

Re μ
u∞ = (4)
ρL

The Nusselt number at that Reynolds number can be used to compute the heat transfer
coefficient for the micro-scale device:

Nu k
h= (5)
L

u_infinity=Re*mu/(L*rho) "velocity of gas in micro-scale device"


h=Nusselt*k/L "heat transfer coefficient in micro-scale device"

Two columns are added to the Parametric Table corresponding to the variables u_infinity and h
(right click on a column heading in the table and select Add Column). The result of solving the
table is shown in Fig. 4 and the requested plot of heat transfer coefficient as a function of
velocity is shown in Fig. 5.
Figure 4: Parametric Table with the variables u_infinity and h included.

Figure 5: Heat transfer coefficient as a function of gas velocity in the micro-scale device.
Problem 4.3-6 (4-5 in text)
Your company makes an extrusion that can be used as a lightweight structural member; the
extrusion is long and thin and has an odd cross-sectional shape that is optimized for structural
performance. This product has been used primarily in the air-craft industry; however, your
company wants to use the extrusion in an application where it will experience cross-flow of
water rather than air. There is some concern that the drag force experienced by the extrusion will
be larger than it can handle. Because the cross-section of the extrusion is not simple (e.g.,
circular or square) you cannot go look up a correlation for the drag coefficient in the same way
that you could for a cylinder. However, because of the extensive use of the extrusion in the air-
craft industry you have an extensive amount of data relating the drag force on the extrusion to
velocity when it is exposed to a cross-flow of air. These data have been collated and are shown
graphically in Figure P4.3-6.
600
Drag force on 1 m of extrusion (N)

100

10

1
Point A, your boss' estimate of the drag force

0.1

0.01
0 20 40 60 80 100 120 140 160 180 200
Air velocity (m/s)
Figure P4.3-6: Drag force as a function of velocity for the extrusion when it is exposed to a cross-flow of air.

Your boss insists that the drag force for the extrusion exposed to water can be obtained by
looking at Figure P4.3-6 and picking off the data at the point where V = 10 m/s (Pt. A in Figure
P4.3-6); this corresponds to a drag force of about 1.7 N/m of extrusion.
a.) Is your boss correct? Explain why or why not.

Your boss is not correct. The drag force on an object is a function of many parameters including
its shape, size, and the cross-flow velocity (which are constant between Pt. A on Figure P4.3-6
and the proposed water operating condition) as well as the viscosity and density of the fluid
(which are not constant between Pt. A on Figure P4.3-6 and the proposed water operating
condition).

b.) If you think that your boss is not correct, then explain how you could use the data shown in
Figure P4.3-6 to estimate the drag force that will be experienced by the extrusion for a water
cross-flow velocity of 10 m/s.

The nondimensional form of the drag force is the drag coefficient, CD. The drag coefficient for a
specific shape should be only a function of the Reynolds number. Therefore, I would transform
the data from the given dimensional form (drag force vs velocity) to a more useful
nondimensional form (drag coefficient vs Reynolds number). I would calculate the Reynolds
number for the water operating condition and determine the drag coefficient that corresponds to
that Reynolds number; the drag coefficient could be used to compute the drag force.

c.) Use the data in Figure P4.3-6 to estimate the drag force that will be experienced by the
extrusion for a water cross-flow velocity of 10 m/s.

The inputs are entered in EES; both the air and water properties (ρair, μair, ρwater and μwater) are
calculated using the internal property routines in EES assuming room temperature and ambient
pressure.

$UnitSystem SI MASS RAD PA K J


$TABSTOPS 0.2 0.4 0.6 0.8 3.5 in

"Inputs"
T=converttemp(C,K,20 [C]) "room temperature"
P=1 [atm]*convert(atm,Pa) "atmospheric pressure"
V_water=10 [m/s] "frontal velocity"
rho_air=density(Air,T=T,P=P) "air density"
mu_air=viscosity(Air,T=T) "air viscosity"
rho_water=density(Water,T=T,P=P) "water density"
mu_water=viscosity(Water,T=T,P=P) "water viscosity"

It is necessary to identify the point in Figure P4.3-6 where the Reynolds number of the water
operating condition matches the Reynolds number of the air operating condition:

ρ air Lc Vair ρ water Lc Vwater


= (1)
μair μ water

where Lc is the characteristic dimension of the extrusion (which doesn't matter since it cancels
from both sides). Solving Eq. (1) for Vair leads to:

μair ρ water
Vair = V (2)
μ water ρ air water

V_air=mu_air*rho_water*V_water/(mu_water*rho_air) "air velocity that matches Reynolds number"

which leads to Vair = 151 m/s. The correct air operating condition to look at in Fig. 2 therefore
corresponds to the much larger drag force of 300 N/m. The drag coefficient corresponding to
this operating condition must be equivalent to the drag coefficient using water:

2 FD ,air 2 FD , water
= (3)
Ap ρ air V 2
air Ap ρ water Vwater
2

where Ap is the projected area of the extrusion (which again doesn't matter since it cancels).
Solving Eq. (3) for the drag force using water is:
ρ water Vwater
2
FD , water = FD ,air (4)
ρ air Vair2

F_D_air=300 [N/m] "drag force at V_air"


F_D_water=F_D_air*rho_water*V_water^2/(rho_air*V_air^2) "drag force for water"

which leads to FD,water = 1090 N/m of extrusion (this is about 3 orders of magnitude larger than
you boss' estimate).
Problem 4.4-1 (4-6 in text)
The momentum and thermal boundary layer can be substantially affected by either injecting or
removing fluid at the plate surface. For example, Figure P4.4-1 shows the surface of a turbine
blade exposed to the free stream flow of a hot combustion gas with velocity u∞ and temperature
T∞. The surface of the blade is protected by blowing gas through pores in the surface in a
process called transpiration cooling.

u∞ , T∞ y
transpiration flow
x

Ts turbine blade
Figure P4.4-1: Transpiration cooled turbine blade.

u∞ υ
The velocity of the injected gas is a function of x: v y =0 = C , where C is a dimensionless
x
constant and υ is the kinematic viscosity of the fluid. The gas is injected at the same temperature
as the surface of the plate, Ts. The Prandtl number of the combustion gas is Pr = 0.7.
a.) Develop a self-similar solution to the momentum equation for this problem using a Crank-
Nicolson numerical integration implemented in EES.

The development of the self-similar solution proceeds as presented in Section 4.4.2. The
transformed governing differential equation is:

d3 f d2 f
+ 2 f =0 (1)
dη 3 dη 2

Two of the boundary conditions for Eq. (1) due not change with transpiration. The x-velocity at
the plate is zero:

df
u y = 0 = u∞ =0 (2)
dη η =0

or

df
=0 (3)
dη η =0

The x-velocity far from the plate (outside of the boundary layer) is the free stream velocity:

df
u y →∞ = u∞ = u∞ (4)
dη η →∞
or

df
=1 (5)
dη η →∞

The boundary conditions for the y-velocity at the plate surface changes due to transpiration.

⎛ ⎞
1 u∞ υ ⎜ df ⎟ u υ
v y =0 = η − f ⎟ =C ∞
x ⎜⎜ N
(6)
2 dη ⎟ x
⎝ =0 ⎠η =0

or

fη =0 = −2 C (7)

The solution technique discussed in the text is adapted to this solution. The transpiration
constant and Prandtl number are specified:

$UnitSystem SI MASS RAD PA K J


$TabStops 0.2 3.5 in

C=0.1 [-] "constant related to transpiration"


Pr=0.7 [-] "Prandtl number"

df d2 f
For this problem, the state variables are f, , and . The rates of change of the first two
dη dη 2
state variables are obtained from the other state variables:

d df
[f ]= (8)
dη dη

d ⎡ df ⎤ d 2 f
dη ⎢ dη ⎥ = dη 2 (9)
⎣ ⎦

The rate of change of the final state variable is obtained from the governing ordinary differential
equation, Eq. (1):

d ⎡d2 f ⎤ d3 f d2 f
⎢ 2⎥ = = −2 f (10)
dη ⎣ dη ⎦ dη
3
dη 2

These state equations are integrated from η = 0 to η = η∞ where η∞ is selected so that we are
outside of both the thermal and velocity boundary layers:
eta_infinity=Max(10,10/sqrt(Pr)) "outer edge of computational domain"

The nodes for the integration are uniformly spaced according to:

ηi = η ∞
( i − 1) for i = 1...N (11)
( N − 1)
The distance between adjacent nodes is:

η∞
Δη = (12)
( N − 1)
where N is the number of nodes.

N=101 [-] "number of steps in the numerical integration"


DELTAeta=eta_infinity/(N-1) "size of the integration steps"
duplicate i=1,N
eta[i]=(i-1)*eta_infinity/(N-1) "position of integration steps"
end

The values of fη = 0 and dfdηη = 0 are specified according to Eqs. (3) and (7). An initial guess for
the 2nd deriviative of f with respect to η is specified:

f[1]=-C "f at eta = 0"


dfdeta[1]=0 [-] "dfdeta at eta = 0"
d2fdeta2[1]=0.3 [-] "d2fdeta2 at eta = 0, this is a guess"

The Crank-Nicolson technique is used to take the integration steps:

⎡ df df ⎤ Δη
fi +1 = f i + ⎢ + ⎥ for i = 1.. ( N − 1) (13)
⎣ dη i dη i +1 ⎦
2

df df ⎡d2 f d2 f ⎤ Δη
= +⎢ + ⎥ for i = 1.. ( N − 1) (14)
dη i +1
dη i ⎣⎢ dη 2 i dη 2 ⎥ 2
i +1 ⎦

d2 f d2 f ⎡ d2 f d2 f ⎤ Δη
= − ⎢ i
2 f + 2 f i +1 ⎥ for i = 1.. ( N − 1) (15)
dη 2 i +1
dη 2 i ⎢⎣ dη 2 i dη 2 ⎥
i +1 ⎦
2

"Crank-Nicolson integration"
duplicate i=1,(N-1)
f[i+1]=f[i]+(dfdeta[i]+dfdeta[i+1])*DELTAeta/2
dfdeta[i+1]=dfdeta[i]+(d2fdeta2[i]+d2fdeta2[i+1])*DELTAeta/2
d2fdeta2[i+1]=d2fdeta2[i]-(2*f[i]*d2fdeta2[i]+2*f[i+1]*d2fdeta2[i+1])*DELTAeta/2
end
The guess values are updated and the guess for the 2nd derivative of f with respect to η is
commented out. The final boundary condition, Eq. (5), is enforced:

{d2fdeta2[1]=0.3 [-]} "d2fdeta2 at eta = 0, this is a guess"


dfdeta[N]=1 [-] "enforce the free stream velocity"

b.) Plot the dimensionless velocity (u/u∞) as a function of the similarity parameter, η, for various
values of C.

The dimensionless velocity, dfdη, is shown in Figure 2 as a function of η for various values of C.
Note that increasing the amount of transpiration tends to thicken the boundary layer and reduce
the shear stress (the velocity gradient) at the wall.

C = 0 (no transpiration)
1

0.9
Dimensionless velocity, u/u

0.8
0.7
0.6 C = 0.1
C = 0.2
0.5
C = 0.3
0.4
C = 0.4
0.3 C = 0.5
0.2 C = 0.6

0.1
0
0 1 2 3 4 5 6 7
Similarity parameter, η
Figure 2: Dimensionless velocity as a function of the similarity parameter for various values of C.

c.) The boundary layer will "blow-off" of the plate at the point where the shear stress at the plate
surface becomes zero. What is the maximum value of C that can be tolerated before the
boundary layer becomes unstable?

d2 f
Figure 3 illustrates the dimensionless velocity gradient at the plate surface, , as a
dη 2 η =0

function of the transpiration constant, C. Note that the velocity gradient is proportional to the
shear stress and therefore the boundary layer will "blow-off" the plate when C is approximately
0.61.
0.7

Dimensionless velocity gradient at η = 0


0.6

0.5

0.4

0.3

0.2

0.1

0
0 0.1 0.2 0.3 0.4 0.5 0.6 0.7 0.8 0.9 1
Transpiration constant, C
Figure 3: Dimensionless velocity gradient at the plate surface as a function of C.

d.) Plot the ratio of the friction factor experienced by the plate with transpiration to the friction
factor experienced by a plate without transpiration as a function of the parameter C.

The shear stress is given by:

u∞ u∞ d 2 f
τs = μ (16)
2 υ x dη 2 η =0

Therefore, the ratio of the friction factor with transpiration to the friction factor without
transpiration is:

d2 f
Cf dη 2 η =0
= (17)
C f ,C = 0 d f
2

dη 2 η = 0,C = 0

d2 f
where is 0.664.
dη 2 η = 0,C = 0

Cfratio=d2fdeta2[1]/0.664 "ratio of the friction coefficient to the friction coefficient with C=0"

Figure 4 illustrates the friction factor ratio as a function of C.


1

Friction factor/friction factor with C = 0


0.9

0.8

0.7

0.6

0.5

0.4

0.3

0.2

0.1

0
0 0.1 0.2 0.3 0.4 0.5 0.6 0.7 0.8 0.9 1
Transpiration constant, C
Figure 4: Friction factor ratio as a function of C.

e.) Develop a self-similar solution to the thermal energy equation for this problem using a
Crank-Nicolson numerical integration implemented in EES.

The solution is identical to the one laid out in Section 4.4.3 of the text; the governing differential
equation and boundary conditions are unchanged.

theta[1]=0 [-] "theta at eta = 0"


{dthetadeta[1]=0.5 [-]} "dthetadeta at eta = 0, this is a guess"

"Crank-Nicolson integration"
duplicate i=1,(N-1)
theta[i+1]=theta[i]+(dthetadeta[i]+dthetadeta[i+1])*DELTAeta/2
dthetadeta[i+1]=dthetadeta[i]+(-2*f[i]*Pr*dthetadeta[i]-2*f[i+1]*Pr*dthetadeta[i+1])*DELTAeta/2
end
theta[N]=1

f.) Plot the dimensionless temperature difference, (T - Ts)/(T∞ - Ts), as a function of the
similarity parameter, η, for various values of C.

Figure 5 provides the requested plot. Notice that as the transpiration increases, the thermal
boundary layer thickens and the rate of heat transfer at the surface is reduced, as evidenced by
the reduced temperature gradient.
Dimensionless temperature difference
C = 0 (no transpiration)
1
0.9
0.8
0.7
0.6
0.5 C = 0.1
C = 0.2
0.4 C = 0.3
0.3 C = 0.4
C = 0.5
0.2
C = 0.6
0.1
0
0 1 2 3 4 5 6 7
Similarity parameter, η
Figure 5: Dimensionless temperature difference as a function of the similarity parameter for various values of
the transpiration constant, C.

g.) Plot the ratio of the Nusselt number experienced by the plate with transpiration to the Nusselt
number experienced by a plate without transpiration as a function of the parameter C.

The Nusselt number is given by:

1 u∞ x dθ
Nu x = (4-18)
2 υ dη η =0

Therefore the ratio of the Nusselt number to the Nusselt number with no transpiration is given
by:

dθ
Nu x dη η =0
= (4-19)
Nu x ,C =0 dθ
dη η =0,c =0

dθ
where = 0.585 for Pr = 0.7.
dη η =0,c =0

Nusseltratio=dthetadeta[1]/0.5851

Figure 6 illustrates the Nusselt number ratio as a function of C.


Nusselt number/Nusselt number with C = 0
1
0.9
0.8
0.7
0.6
0.5
0.4
0.3
0.2
0.1
0
0 0.1 0.2 0.3 0.4 0.5 0.6 0.7 0.8 0.9 1
Transpiration constant, C
Figure 6: Nusselt number ratio as a function of C.
Problem 4.4-2 (4-7 in text)
Develop a self-similar solution for the flow over a flat plate that includes viscous dissipation.
The ordinary differential equation governing the dimensionless temperature difference should
include an additional term that is related to the Eckert number.
a.) Plot the dimensionless temperature difference, (T - Ts)/(T∞ - Ts), as a function of the
similarity parameter, η, for various values of Ec with Pr = 10.

The development of the self-similar solution for the velocity proceeds as presented in Section
4.4.2.

$UnitSystem SI MASS RAD PA K J


$TabStops 0.2 3.5 in

Pr=10 [-] "Prandtl number"


eta_infinity=Max(10,10/sqrt(Pr)) "outer edge of computational domain"
N=101 [-] "number of steps in the numerical integration"
DELTAeta=eta_infinity/(N-1) "size of the integration steps"
duplicate i=1,N
eta[i]=(i-1)*eta_infinity/(N-1) "position of integration steps"
end

f[1]=0 [-] "f at eta = 0"


dfdeta[1]=0 [-] "dfdeta at eta = 0"
{d2fdeta2[1]=0.3 [-]} "d2fdeta2 at eta = 0, this is a guess"

"Crank-Nicolson integration"
duplicate i=1,(N-1)
f[i+1]=f[i]+(dfdeta[i]+dfdeta[i+1])*DELTAeta/2
dfdeta[i+1]=dfdeta[i]+(d2fdeta2[i]+d2fdeta2[i+1])*DELTAeta/2
d2fdeta2[i+1]=d2fdeta2[i]-(2*f[i]*d2fdeta2[i]+2*f[i+1]*d2fdeta2[i+1])*DELTAeta/2
end
dfdeta[N]=1 [-]

The development of the self-similar for the temperature solution proceeds as presented in Section
4.4.3 but the viscous dissipation term is retained in the thermal energy conservation equation:

2
∂T ∂T ∂ 2T μ ⎛ ∂u ⎞
u +v =α 2 +
ρ c ⎜⎝ ∂y ⎟⎠
(1)
∂x ∂y ∂y

The dimensionless temperature difference is defined as in Section 4.3.2:

T − Ts
θ = (2)
T∞ − Ts

Substituting Eq. (2) into Eq. (1) leads to:

∂θ ∂θ ∂ 2θ


2
μ ⎛ ∂u ⎞
u +v =α 2 +
ρ c (T∞ − Ts ) ⎜⎝ ∂y ⎟⎠
(3)
∂x ∂y ∂y
The boundary conditions expressed in terms of θ are:

θη =0 = 0 (4)

θη →∞ = 1 (5)

The governing differential equation is transformed to the similarity variable.

∂θ η dθ
=− (6)
∂x 2 x dη

∂θ dθ 1 u∞
= (7)
∂y dη 2 υ x

∂ 2θ d 2θ u∞
= (8)
∂y 2 dη 2 4υ x

∂u u∞ d 2 f u∞
= (9)
∂y 2 dη 2 υx

df
u = u∞ (10)

u∞ υ ⎛ df ⎞
v= ⎜η −f⎟ (11)
x ⎝ dη ⎠

Substituting Eqs. (6) through (11) into Eq. (3) leads to:

df ⎛ η dθ ⎞ u∞ υ ⎛ df ⎞ dθ 1 u∞
u∞ ⎜− ⎟+ ⎜η −f⎟ =
N dη ⎝ 2 x dη ⎠ x ⎝ dη ⎠ dη 2 υ x
u




∂θ v ∂θ
∂x ∂y
(12)
d 2θ u
2
μ u∞2 ⎛ d 2 f ⎞ u∞
α 2 ∞ + ⎜ ⎟
dη 4υ x ρ c (T∞ − Ts ) 4 ⎝ dη 2 ⎠ υ x



∂ 2θ ⎛ ∂u ⎞
2

∂y 2 ⎜ ⎟
⎝ ∂y ⎠

Dividing through by u∞/x leads to:


f dθ d 2θ 1
2
u∞2 1 ⎛ d2 f ⎞
− = + ⎜ ⎟ (13)
2 dη dη 2 4 Pr c (T∞ − Ts ) 4 ⎝ dη 2 ⎠


Ec

The Eckert number is defined in Section 4.3.2 as:

u∞2
Ec = (14)
c (T∞ − Ts )

Therefore, the transformed thermal energy equation, including viscous dissipation is:

d 2θ dθ
2
⎛ d2 f ⎞
+ 2 f Pr = Ec Pr ⎜ 2⎟ (15)
dη 2 dη ⎝ dη ⎠

The numerical solution proceeds as discussed in Section 4.4.3 with the modification of the state
equation:

d 2θ dθ
2
⎛ d2 f ⎞
= − 2 f Pr + Ec Pr ⎜ 2⎟ (16)
dη 2 dη ⎝ dη ⎠

The Eckert number is set:

Ec=100 [-] "Eckert number"

and the solution without viscous dissipation ( θnv ) is obtained:

"Solution without including viscous dissipation"


theta_nv[1]=0 [-] "theta at eta = 0"
{dthetadeta_nv[1]=0.5 [-]} "dthetadeta at eta = 0, this is a guess"
"Crank-Nicolson integration"
duplicate i=1,(N-1)
theta_nv[i+1]=theta_nv[i]+(dthetadeta_nv[i]+dthetadeta_nv[i+1])*DELTAeta/2
dthetadeta_nv[i+1]=dthetadeta_nv[i]+(-2*f[i]*Pr*dthetadeta_nv[i]-
2*f[i+1]*Pr*dthetadeta_nv[i+1])*DELTAeta/2
end
theta_nv[N]=1

The solution including viscous dissipation is obtained:

"Solution including viscous dissipation"


theta[1]=0 [-] "theta at eta = 0"
{dthetadeta[1]=0.5 [-]} "dthetadeta at eta = 0, this is a guess"
"Crank-Nicolson integration"
duplicate i=1,(N-1)
theta[i+1]=theta[i]+(dthetadeta[i]+dthetadeta[i+1])*DELTAeta/2
dthetadeta[i+1]=dthetadeta[i]+(-2*f[i]*Pr*dthetadeta[i]-2*f[i+1]*Pr*dthetadeta[i+1]-&
Ec*d2fdeta2[i]^2/4-Ec*d2fdeta2[i+1]/4)*DELTAeta/2
end
theta[N]=1

Figure 1 illustrates the dimensionless temperature difference as a function of η for various values
of Ec with Pr = 10.

2.5
Dimensionless temperature difference

2.25 Ec = 100
Ec = 50
2
Ec = 25
1.75 Ec = 10
1.5

1.25

0.75

0.5 Ec = 0 (no viscous dissipation)

0.25

0
0 0.5 1 1.5 2 2.5 3
Similarity variable
Figure 1: Dimensionless temperature difference as a function of η for various values of Ec with Pr = 10.

b.) Plot the ratio of the Nusselt number to the Nusselt number neglecting viscous dissipation
(i.e., with Ec = 0) as a function of the Eckert number for various values of the Prandtl
number.

The Nusselt number is:

1 u∞ x dθ
Nu x = (17)
2 υ dη η =0

Therefore, the ratio of the Nusselt number including viscous dissipation to the Nusselt number
neglecting viscous dissipation is:

dθ
Nu x dη η =0
= (18)
Nu x , Ec =0 dθnv
dη η =0

Nusselt_ratio=dthetadeta[1]/dthetadeta_nv[1]
Figure 2 illustrates the ratio of the Nusselt number to the Nusselt number neglecting viscous
dissipation as a function of the Eckert number for various values of the Prandtl number.

Nusselt number/Nusselt number with Ec = 0


30

25

Pr = 0.5
20

15 Pr = 1

10 Pr = 2

Pr = 5
5
Pr = 10

0
0 10 20 30 40 50 60 70 80 90 100
Eckert number
Figure 2: Nusselt number ratio as a function of Ec for various values of Pr.
Problem 4.7-1 (4-8 in text)
Use the Spalding model to obtain a velocity and temperature law of the wall (your temperature
law of the wall should be obtained numerically using the EES Integral command). Compare your
result with the Prandtl-Taylor model. Use a molecular Prandtl number of Pr = 0.7 and a
turbulent Prandtl number of Prturb = 0.9.

The Spalding model of the universal velocity distribution is given in Table 4-2:


y + = u + + 0.11408 ⎢exp (κ u + ) − 1 − κ u + −
( κ u )
+ 2


( κ u )
+ 3


( κ u )⎥
+ 4⎤

(1)
⎢ 2 6 24 ⎥
⎣ ⎦

where κ = 0.41. The Prandtl-Taylor model is also provided in Table 4-2:

+
⎧⎪ y + for 0 < y + < 11.5
u =⎨ (2)
⎪⎩2.44 ln ( y ) + 5.5 for y > 11.5
+ +

$UnitSystem SI MASS RAD PA K J


$TabStops 0.2 3.5 in

y_plus=20 [-] "inner position"


K = 0.41 [-] "von Karman constant"
u_plus_PT=IF(y_plus,11.5,y_plus,y_plus,2.44*ln(y_plus)+5.5) "Prandtl-Taylor model"
y_plus = u_plus+0.11408*(exp(K*u_plus)-1-K*u_plus-(K*u_plus)^2/2-(K*u_plus)^3/6-(K*u_plus)^4/24)
"Spalding model"

Figure 1 illustrates the inner velocity as a function of inner position using the Spalding and
Prandtl-Taylor models.

Figure 1: Universal velocity profile from the Prandtl-Taylor and Spalding models.
The inner temperature difference associated with the Prandtl-Taylor model was derived in
Section 4.7.9:

⎛ ⎞
Prturb ⎜
1
+
1
Pr Prturb
( κ y + − 1) ⎟
θ + − 11.5 Pr = ln ⎜ ⎟ for y + > 11.5 (4-3)
κ ⎜ (11.5 κ − 1) ⎟⎟
1 1
⎜ +
⎝ Pr Prturb ⎠

Pr=0.7 "molecular Prandtl number"


Pr_turb=0.9 "turbulent Prandtl number"
theta_plus_PT=IF(y_plus,11.5,Pr*y_plus,Pr*y_plus,11.5*Pr+Pr_turb*ln((1/Pr+(K*y_plus-&
1)/Pr_turb)/(1/Pr+(K*11.5-1)/Pr_turb))/K)
"inner temperature difference predicted using the Prandtl-Taylor model"

The derivative of the inner temperature difference with respect to inner position is given by Eq.
(4-432) in the text:

dθ + 1
+
= (4)
dy ⎛ 1 ε ⎞
⎜ + M ⎟
⎝ Pr Prturb υ ⎠

The ratio of εM/υ associated with the Spalding model is provided in Table 4-3:

εM

= 0.0526 ⎢exp (κ u + ) − 1 − κ u + −
(κ u ) − (κ u )
+ 2 + 3 ⎤
⎥ (5)
υ ⎢ 2 6 ⎥
⎣ ⎦

Equations (4), (1), and (5) together allow the derivative of the inner temperature difference to be
computed at an arbitrary value of y+.

eMovernu=0.0526*(exp(K*u_plus)-1-K*u_plus-(K*u_plus)^2/2-(K*u_plus)^3/6)
dtheta_plusdy_plus=1/(1/Pr+eMovernu/Pr_turb)

The inner temperature distribution associated with the Spalding model is obtained by
numerically integrating Eq. (4) using the Integral function in EES:

y+
dθ + +
θ = ∫ dy + dy
+
(6)
+
y =0

{y_plus=20 [-]} "inner position"


theta_plus=Integral(dtheta_plusdy_plus,y_plus,0.001,330)
$IntegralTable y_plus,theta_plus
Figure 2 illustrates the inner temperature difference as a function of inner position using the
Spalding and Prandtl-Taylor models.

Figure 2: Inner temperature profile from the Prandtl-Taylor and Spalding models.
Problem 4.7-2 (4-9 in text)
Use the van Driest model to obtain a velocity and temperature law of the wall. (Both of these
results should be obtained numerically using the EES Integral command). Compare your result
with the Prandtl-Taylor model. Use a molecular Prandtl number of Pr = 0.7 and a turbulent
Prandtl number of Prturb = 0.9.

The Prandtl-Taylor model is provided in Table 4-2:

+
⎧⎪ y + for 0 < y + < 11.5
u =⎨ (1)
⎪⎩2.44 ln ( y ) + 5.5 for y > 11.5
+ +

$UnitSystem SI MASS RAD PA K J


$TabStops 0.2 3.5 in

y_plus=20 [-] "inner position"


K = 0.41 [-] "von Karman constant"
u_plus_PT=IF(y_plus,11.5,y_plus,y_plus,2.44*ln(y_plus)+5.5) "Prandtl-Taylor model"

Figure 1 illustrates the inner velocity as a function of inner position using the Prandtl-Taylor
model.

Figure 1: Universal velocity profile from the Prandtl-Taylor and van Driest models.

The van Driest model is also given in Table 4-2:

du + 2
= (2)
dy + 2 ⎡
2
⎛ y ⎞⎤
+
1 + 1 + 4 (κ y + ) ⎢1 − exp ⎜ − ⎟ ⎥
⎣ ⎝ 26 ⎠ ⎦
The velocity distribution is obtained by numerically integrating Eq. (2):

y+
du + +

+
u = dy (3)
0
dy +

{y_plus=20 [-]} "inner position"


du_plusdy_plus=2/(1+sqrt(1+4*(K*y_plus)^2*(1-exp(-y_plus/26))^2))
u_plus=Integral(du_plusdy_plus,y_plus,0.001,330)
$IntegralTable y_plus,u_plus

The van Driest velocity distribution is shown in Figure 1.

The inner temperature difference associated with the Prandtl-Taylor model was derived in
Section 4.7.9:

⎛ ⎞
Prturb ⎜
1
+
1
Pr Prturb
( κ y + − 1) ⎟
θ + − 11.5 Pr = ln ⎜ ⎟ for y + > 11.5 (4-4)
κ ⎜ (11.5 κ − 1) ⎟⎟
1 1
⎜ +
⎝ Pr Prturb ⎠

Pr=0.7 "molecular Prandtl number"


Pr_turb=0.9 "turbulent Prandtl number"
theta_plus_PT=IF(y_plus,11.5,Pr*y_plus,Pr*y_plus,11.5*Pr+Pr_turb*ln((1/Pr+(K*y_plus-&
1)/Pr_turb)/(1/Pr+(K*11.5-1)/Pr_turb))/K)

The derivative of the inner temperature difference with respect to inner position is given by Eq.
(4-432) in the text:

dθ + 1
+
= (5)
dy ⎛ 1 ε ⎞
⎜ + M ⎟
⎝ Pr Prturb υ ⎠

The ratio of εM/υ associated with the van Driest model is provided in Table 4-3:

2
ε M ⎧⎪ + ⎡ ⎛ y + ⎞ ⎤ ⎫⎪ du +
= ⎨κ y ⎢1 − exp ⎜ − ⎟⎥ ⎬ (6)
υ ⎪⎩ ⎣ ⎝ 24.7 ⎠ ⎦ ⎪⎭ dy
+

Equations (5), (2), and (6) together allow the derivative of the inner temperature difference to be
computed at an arbitrary value of y+.

eMovernu=(K*y_plus*(1-exp(-y_plus/24.7)))^2*du_plusdy_plus
dtheta_plusdy_plus=1/(1/Pr+eMovernu/Pr_turb)

The inner temperature distribution associated with the Spalding model is obtained by
numerically integrating Eq. (6) using the Integral function in EES:
y+
dθ + +
θ = ∫ dy + dy
+
(7)
y+ =0

theta_plus=Integral(dtheta_plusdy_plus,y_plus,0.001,330)
$IntegralTable y_plus,u_plus,theta_plus

Figure 2 illustrates the inner temperature difference as a function of inner position using the van
Driest and Prandtl-Taylor models.

Figure 2: Inner temperature profile from the Prandtl-Taylor and van Driest models.
Problem 4.7-3 (4-10 in text)
In Section 4.5, a conceptual model of a turbulent flow was justified based on the fact that the
thermal resistance of the viscous sublayer (δvs/k) is larger than the thermal resistance of the
turbulent boundary layer (δturb/kturb). Estimate the magnitude of each of these terms for a flow of
water over a smooth flat plate and evaluate the validity of this simplification. The free stream
velocity is u∞ = 10 m/s and the plate is L = 1 m long. The water is at 20°C and 1 atm.

The inputs are entered in EES:

$UnitSystem SI MASS RAD PA K J


$TABSTOPS 0.2 0.4 0.6 0.8 3.5 in

u_infinity=10 [m/s] "free-stream velocity"


L=1 [m] "length of plate"
T=converttemp(C,K,20[C]) "temperature"
p=1 [atm]*convert(atm,Pa) "pressure"

The properties of water (μ, ρ, c, k, and υ) are obtained using EES' internal property routines:

"Fluid properties"
mu=viscosity(Water,T=T,P=p) "viscosity"
rho=density(Water,T=T,P=p) "density"
c=cP(Water,T=T,P=p) "specific heat capacity"
k=conductivity(Water,T=T,P=p) "conductivity"
nu=mu/rho "kinematic viscosity"

According to Eq. (4-571) in the text, the viscous sublayer thickness at the trailing edge of the
plate can be estimated according to:

34.9 L
δ vs ≈ (1)
ReL0.9

where ReL is the Reynolds number:

ρ u∞ L
ReL ≈ (2)
μ

The thermal resistance of the viscous sublayer per unit area is approximately:

δ vs
Rvs ≈ (3)
k

Re=rho*L*u_infinity/mu "Reynolds number"


delta_vs=34.9*L/Re^0.9 "viscous sublayer thickness"
R_vs=delta_vs/k "thermal resistance of the viscous sublayer"

which leads to Rvs = 3.0x10-5 K-m2/W.


According to Eq. (4-566) of the text, the turbulent boundary layer thickness at the trailing edge is
approximately:

0.19 x
δ m,turb ≈ L (4)
ReL0.1

delta_t_turb=0.19*L/Re^0.1 "turbulent boundary layer thickness"

The effective conductivity of the turbulent boundary layer is almost entirely related to the
turbulent eddies and can be estimated according to:

ρ cεM
kturb ≈ (5)
Prturb

For simplicity, we will use the Prandtl-Taylor model to estimate the eddy diffusivity of
momentum required by Eq. (5):

εM
= κ y+ −1 (6)
υ

We will calculate use a value of y+ that is consistent with being at the mid-point of the boundary
layer:

εM δ
= κ m,turb − 1 (7)
υ 2 Lchar ,vs

where Lchar,vs is defined according to:

μ
Lchar ,vs = (8)
τs ρ

The friction coefficient correlation provided by Eq. (4-588) for turbulent flow over a smooth flat
plate is used to compute the shear stress:

C f = 0.0592 ReL−0.20 (9)

1
τ s = ρ u∞2 C f (10)
2

C_f=0.0592*Re^(-0.2) "friction coefficient"


tau_s=C_f*rho*u_infinity^2/2 "shear stress"

The value of Lchar,vs is computed using Eq. (8) and the value of εM is computed using Eq. (7):
L_char_vs=mu/sqrt(tau_s*rho) "characteristic length of viscous sublayer"
Kappa=0.41 [-] "von Karman constant"
e_M=nu*(Kappa*delta_t_turb/(2*L_char_vs)-1) "eddy diffusivity of momentum"

The turbulent conductivity is estimated according to Eq. (5). The thermal resistance of the
turbulent boundary layer per unit area is estimated according to:

δ m,turb
Rturb ≈ (11)
kturb

Pr_turb=0.9 [-] "turbulent Prandtl number"


k_turb=rho*c*e_M/Pr_turb "effective turbulent conductivity"
R_turb=delta_t_turb/k_turb "thermal resistance of the turbulent boundary layer"

which leads to Rturb = 3.1x10-6 K-m2/W. Note that Rvs/Rturb is approximately 10, justifying the
assumption that the thermal resistance of the viscous sublayer is larger than the thermal
resistance of the turbulent boundary layer.
Problem 4.7-4
A fluid with a low Prandtl number (Pr <<1) flows over a heated flat plate (Ts > T∞). Assume that
the flow velocity, u∞, is low enough that the flow remains laminar at a particular axial location x.
a.) Sketch the velocity and temperature distribution that you expect at location x. Indicate on
your sketches the approximate extent of the momentum and thermal boundary layer
thicknesses (δm and δt); make sure that the relative size of these boundary layer thicknesses is
appropriate given the low Prandtl number of the fluid.

Figure 1 illustrates a fluid with a low Prandtl number (Pr <<1) flowing over a heated flat plate
(Ts > T∞).

(a)

(b)
Figure 1: Fluid with a low Prandtl number flowing over a heated plate; (a) indicates the approximate velocity
distribution and (b) indicates the approximate temperature distribution.

The low Prandtl number indicates that the thermal boundary layer should be larger than the
momentum boundary layer. The laminar flow indicates that there should be a smooth transition
from the surface velocity (0) and temperature (Ts) to the free stream values. These
characteristics are captured in Fig. 1.
b.) Assume that the flow velocity, u∞, is increased sufficiently that the flow transitions to
turbulence. Sketch the temperature distribution that you would expect. Make sure that the
qualitative differences between your answers to (a) and (b) are clear.

Figure 2: Approximate temperature distribution for a fluid flowing over a heated plate under turbulent
conditions.

The temperature distribution for a laminar flow is characterized by a sharp temperature gradient
across the viscous sublayer (where the effective conductivity is low due to the lack of any
turbulent eddies) which is very near the wall and a much lower temperature gradient across the
bulk of the boundary layer (where the effective conductivity is high due to the presence of
turbulent eddies). This is shown in Fig. 2; the differences between Fig. 2 and Fig. 1(b) are clear.
Problem 4.8-1: Flow over a Wedge
Potential flow theory shows that the free stream velocity over a wedge or through an expansion,
shown in Figures P4.8-1(a) and (b), will have a power-law functional form.

u∞ = C1 x m (1)

where the exponent m is related to β according to:

β
m= (2)
2−β

Note that if β >0, as shown in Figure P4.8-1(a), then the flow will be accelerating along the
surface whereas if β < 0, as shown in Figure P4.8-1(b)),then the flow is decelerating.
The surface temperature is constant, Ts, and the free stream temperature is T∞.

x −βπ
βπ

(a) (b)
Figure P4.8-1: Flow (a) over a wedge of with angle β π and (b) through an expansion with angle -β π.

a.) Use an integral technique with a second-order velocity distribution in order to determine a
differential equation relating the momentum boundary layer thickness as a function of axial
flow position.

The appropriate, 2nd order velocity distribution is:

u ⎡ y y2 ⎤
= 2 −
u∞ ⎢⎣ δ m δ m2 ⎥⎦
(3)

which leads to a shear stress at the surface:

u∞
τ s = 2μ (4)
δm

The integral form of the momentum equation is:


d ⎡ ⎤
y =δ m y =δ m
τ

dx ⎣⎢ ∫ ( u 2 − u u∞ ) dy ⎥ + dudx∞ ∫ ( u − u ) dy = − ρ

s
(5)
y =0 ⎦⎥ y =0

Substituting Eqs. (3) and (4) into Eq. (5) leads to:

d ⎡ 2 ⎛ ⎛ y y 2 ⎞2 y y 2 ⎞ ⎤ du∞
y =δ m y =δ m
⎛ y y2 ⎞ 2 μ u∞
⎢ u∞
dx ⎢ ∫ ⎜ ⎜ 2 − 2 ⎟ − 2 + 2 ⎟ dy ⎥ +
⎜ ⎝ δm δm ⎠ δ m δ m ⎟ ⎥ dx
u∞ ∫ ⎜ 2 − 2 − 1⎟ dy = −
y =0 ⎝
δm δm ⎠ ρ δm
(6)
⎣ y =0 ⎝ ⎠ ⎦

or

d ⎡ 2 y y 2 ⎞ ⎤ du∞
y =δ m y =δ m
⎛ y2 y3 y4 ⎛ y y2 ⎞ 2 μ u∞
⎢ u∞
dx ⎢⎣ ∫
y =0
⎜ 2
4
⎝ δm
− 4 +
δm δm
3 4
− 2 + 2 ⎟
dy ⎥
δ m δ m ⎠ ⎥⎦ dx
+ u∞ ∫ ⎜2
y =0 ⎝
− 2 − 1⎟ dy = −
δm δm ⎠ ρ δm
(7)

Carrying out the integrations:

δm δm
d ⎡ 2 ⎛ 4 y3 y4 1 y5 y2 1 y3 ⎞ ⎤ du ⎛ y2 1 y3 ⎞ 2 μ u∞
⎢ u∞ ⎜ − + − + ⎟ ⎥+ ∞
u∞ ⎜ − − y⎟ = − (8)
dx ⎢ ⎝ 3 δ m2 δ m3 5 δ m4 δ m 3 δ m2 ⎠0 ⎥⎦ dx ⎝ δm 3 δm
2
⎠0 ρ δm

Evaluating the limits:

d ⎡ 2 ⎛4 1 1 ⎞ ⎤ du ⎛ 1 ⎞ 2 μ u∞
⎢ u∞ δ m ⎜ − 1 + − 1 + ⎟ ⎥ + ∞ u∞ δ m ⎜ 1 − − 1⎟ = − (9)
dx ⎣ ⎝3 5 3 ⎠ ⎦ dx ⎝ 3 ⎠ ρ δm

or

2 d 2 5 du∞ 2 μ u∞
− ⎡⎣u∞ δ m ⎤⎦ + u∞ δ m = − (10)
15 dx 3 dx ρ δm

Substituting Eq. (1) into Eq. (10) leads to:

2 d 2 μ C1 x m
⎡⎣C12 x 2 m δ m ⎤⎦ + ( C1 m x m −1 ) C1 x m δ m = −
5
− (11)
15 dx 3 ρ δm

which is our differential equation for δm with x. Simplifying Eq. (11) leads to:

dδ m 15μ 21 m δ m
= m + (12)
dx x C1 ρ δ m 2 x

which provides the rate of change of δm and therefore is our state equation.
Problem 4.8-2: Flow over a Plate with varying Heat Flux
A flat plate with a specified heat flux is exposed to a free stream. The flow remains laminar over
the plate. Solve the problem using an integral technique with the following assumptions:
1. a 2nd order velocity profile,
2. a 2nd order temperature distribution,
3. the Prandtl number is greater than 1.0,
4. the heat flux at the surface of the plate ( qs′′ , not the plate surface temperature) is a known
function of x,
5. viscous dissipation is not negligible,
6. the free stream velocity, u∞, is constant,
1
7. the ratio of the momentum to the thermal boundary layer thickness is: δ m / δ t = Pr 3

a.) Derive the ordinary differential equation for the thermal boundary layer thickness.

δ δ
d ⎡t ⎤ qs′′
2
μ t ⎛ ∂u ⎞
⎢ u (T − T∞ ) dy ⎥ =
dx ⎣⎢ ∫0 ∫
+ ⎜ ⎟ dy (1)
⎦⎥ ρ c ρ c 0 ⎝ ∂y ⎠

or

d ⎡ u ⎡⎣T − Ts − (T∞ − Ts ) ⎤⎦ ⎤ qs′′


δt δ 2
μ t ⎛ ∂u ⎞
⎢u∞ (T∞ − Ts ) ∫ dy ⎥ = + ∫ ⎜ ⎟ dy (2)
dx ⎢⎣ 0 ∞
u (T∞ − Ts ) ⎥⎦ ρ c ρ c 0 ⎝ ∂y ⎠

The appropriate form of the 2nd order velocity distribution is:

u ⎡ y y2 ⎤
= ⎢2 − ⎥ (3)
u∞ ⎣ δ m δ m2 ⎦

and the appropriate form of the 2nd order temperature distribution is:

T − Ts ⎡ y y 2 ⎤
= ⎢2 − ⎥ (4)
T∞ − Ts ⎣ δ t δ t2 ⎦

Substituting Eqs. (3) and (4) into Eq. (2) leads to:

δt δ
d ⎡ ⎛ y y 2 ⎞ ⎛ y y 2 ⎞ ⎤ qs′′
2
μ 2 t ⎛ 2 2y ⎞
⎢u∞ (T∞ − Ts ) ∫ ⎜ 2 − 2 ⎟ ⎜ 2 − 2 − 1⎟ dy ⎥ = + u∞ ∫ ⎜ − 2 ⎟ dy (5)
dx ⎣⎢ 0⎝
δ m δ m ⎠ ⎝ δt δt ⎠ ⎦⎥ ρ c ρ c 0 ⎝ δm δm ⎠

Equation (5) can be expanded:


δt
d ⎡ ⎛ y2 y3 y y3 y4 y2 ⎞ ⎤
⎢ ∞ ( ∞ s)∫⎜
u T − T 4 − 2 − 2 − 2 + + 2 ⎟
dy ⎥ =
dx ⎢⎣ 0 ⎝ δ t δ m δ t
2
δ m δ m δ t δ 2
m δ t δ
2 2
m δ m ⎠ ⎥⎦
(6)
δ
qs′′ μ 2 t ⎛ 4 8 y 4 y2 ⎞
ρ c ρ c ∞ ∫0
+ u ⎜ 2 − 3 − 4 ⎟ dy
⎝ δm δm δm ⎠

The integrations are carried out:

δt
d ⎡ ⎛ 4 y3 1 y4 y2 1 y4 1 y5 1 y3 ⎞ ⎤
⎢u∞ (T∞ − Ts ) ⎜ − − − + + 2 ⎟
⎥=
dx ⎢ ⎝ 3 δ t δ m 2 δ t δ m δ m 2 δ t δ m 5 δ t δ m 3 δ m ⎠0 ⎥⎦
2 2 2 2
⎣ (7)
δt
qs′′ μ 2⎛4y 4y 4 y ⎞ 2 3
+ u ⎜ − − ⎟
ρ c ρ c ∞ ⎝ δ m2 δ m3 3 δ m4 ⎠0

Applying the limits:

d ⎡ ⎛ 4 δ t2 1 δ t2 δ t2 1 δ t3 1 δ t3 1 δ t3 ⎞ ⎤
⎢ ∞ ( ∞ s )⎜
u T − T − − − + + 2 ⎟⎥
=
dx ⎣ ⎝ 3 δ m 2 δ m δ m 2 δ m 5 δ m 3 δ m ⎠⎦
2 2

(8)
qs′′ μ 2 ⎛ 4 δ t 4 δ t2 4 δ t3 ⎞
+ u ⎜ − 3 − ⎟
ρ c ρ c ∞ ⎝ δ m2 δ m 3 δ m4 ⎠

or

d ⎡ ⎛ δ t2 δ t3 ⎞ ⎤ qs′′ μ u∞2 ⎛ 4 δ t2 4 δ t3 4 δ t4 ⎞
u ( T −
⎢ ∞ ∞ s ⎜ T ) −0.167 + 0.0333 ⎟⎥ = + ⎜ − 3 − ⎟ (9)
dx ⎣ ⎝ δm δ m2 ⎠ ⎦ ρ c ρ c δ t ⎝ δ m2 δ m 3 δ m4 ⎠

The heat flux at the surface of the plate is given by:

qs′′ = 2k
(Ts − T∞ ) (10)
δt

Since the applied heat flux is specified rather than the plate surface temperature it is useful to
substitute Eq. (10) into Eq. (9) in order to eliminate the plate surface temperature:

d ⎡ qs′′ δ t2 ⎛ δt δ t2 ⎞ ⎤ qs′′ μ u∞2 ⎛ 4 δ t2 4 δ t3 4 δ t4 ⎞



⎢ ∞u ⎜ − 0.167 + 0.0333 ⎟⎥ = + ⎜ − 3 − ⎟ (11)
dx ⎣ 2k ⎝ δm δ m2 ⎠ ⎦ ρ c ρ c δ t ⎝ δ m2 δ m 3 δ m4 ⎠

The ratio of the momentum to the thermal boundary layer thickness is assumed to be related to
the Prandtl number according to assumption 7 in the problem statement:
d ⎡ qs′′ δ t2 ⎛ 0.167 0.0333 ⎞ ⎤ qs′′ μ u∞2 ⎛ 4 4 4 ⎞
⎢ −u∞ ⎜− + ⎟ ⎥ = + ⎜ 2 − − ⎟ (12)
dx ⎣⎢ 2 k ⎝ Pr 3 Pr 3 ⎠ ⎦⎥ ρ c ρ c δ t ⎜⎝ Pr 3 Pr 3 Pr 3 ⎟
1 2 4

or

⎛ 0.167 0.0333 ⎞ d 2 k qs′′ 2 k μ u∞2 ⎛ 4 4 4 ⎞


⎜ − + ⎡ − 
q
⎟ ⎣ s t ⎦ ′′ δ 2
⎤ = + ⎜ 2 − − ⎟ (13)
1
⎝ Pr 3 Pr 3 ⎠ dx
2
ρ c u∞ ρ c δ t ⎜⎝ Pr 3 Pr 3 Pr 4 3 ⎟

b.) Use your result from (a) to determine an equation for the local Nusselt number as a function
of Reynolds number and Prandtl number when the heat flux on the plate surface is constant
in the limit of no viscous dissipation.

Equation (13) becomes:

⎛ 0.167 0.0333 ⎞ d 2k
− ⎟ ⎡⎣δ t ⎤⎦ =
2
⎜ (14)
1
⎝ Pr 3 Pr 3 ⎠ dx
2
ρ c u∞

which can be solved analytically:

⎛ 0.167 0.0333 ⎞ δt 2α
x
− ⎟∫ ( t )
d δ = ∫ dx
2
⎜ (15)
⎝ Pr
1
3
2
Pr ⎠ 0
3 u ∞ 0

or

⎛ 0.167 0.0333 ⎞ 2 2 α x
⎜ − ⎟ δt = (16)
1
⎝ Pr 3
2
Pr 3 ⎠ u∞

Solving for the thermal boundary thickness lead to:

2α x
u∞
δt = 1
(17)
⎛ 0.167 0.0333 ⎞ 2

⎜ 1
− 2 ⎟
⎝ Pr 3 Pr 3 ⎠

The Nusselt number is related to the thermal boundary layer thickness according to:

Nu =
hx
=
q s′′ x (T − T )
= 2k s ∞
x
=2
x
(18)
k (Ts − T∞ ) k δt (Ts − T∞ ) k δ t
Substituting Eq. (17) into Eq. (18) leads to:
1
u∞ ⎛ 0.167 0.0333 ⎞ 2
Nu = 2 x ⎜ − ⎟ (19)
2 α x ⎝ Pr 13 2
Pr 3 ⎠

or
1
2 ⎛ 0.167 0.0333 ⎞ 2
Nu = Rex Pr ⎜ 1
− 2 ⎟ (20)
2 ⎝ Pr 3
Pr 3 ⎠

c.) Compare your result from (b) with the solution provided in Section 4.9.2 by preparing a plot
showing both solutions as a function of Reynolds number for Pr = 1.0 and Pr = 5.0.

The solution provided in Section 4.9.2 is:

1
Nu x = 0.453 Rex0.5 Pr 3
for Pr > 0.6 (21)

Equations (20) and (21) are implemented in EES:

Pr=1.0
Nusselt_s=(2/sqrt(2))*sqrt(Re)*sqrt(Pr)*sqrt(0.167/Pr^(1/3)-0.0333/Pr^(2/3))
Nusselt_l=0.453*sqrt(Re)*Pr^(1/3)

Figure 2 illustrates the local Nusselt number predicted by Eqs. (20) (labeled solution) and (21) as
a function of Reynolds number for Pr = 1.0 and Pr = 5.0.
Figure 2: Local Nusselt number predicted by Eqs. (20) (labeled solution) and Eq. (21) as a function of
Reynolds number for Pr = 1.0 and Pr = 5.0.

d.) Use your result from (a) to determine an equation for the local Nusselt number as a function
of Reynolds number and Prandtl number when the heat flux on the plate surface increases
linearly from 0 at the leading edge in the limit of no viscous dissipation.

⎛ 0.167 0.0333 ⎞ d 2k x
− ⎟ ⎣⎡ x δ t ⎦⎤ =
2
⎜ (22)
1
⎝ Pr 3 Pr 3 ⎠ dx
2
ρ c u∞

which can also be solved analytically:

⎛ 0.167 0.0333 ⎞ x δt
2
x
2k
− ⎟ ∫ ( t )
d x δ = ∫ x dx
2
⎜ (23)
⎝ Pr
1
3
2
Pr ⎠ 0
3 ρ c u ∞ 0

or

⎛ 0.167 0.0333 ⎞ 2 2 k x2
⎜ − x
⎟ tδ = (24)
1
⎝ Pr 3
2
Pr 3 ⎠ ρ c u∞ 2

Solving for δt leads to:


kx
ρ c u∞
δt = 1
(25)
⎛ 0.167 0.0333 ⎞ 2

⎜ 1
− 2 ⎟
⎝ Pr 3 Pr 3 ⎠

Substituting Eq. (25) into Eq. (18) leads to:


1
u∞ ⎛ 0.167 0.0333 ⎞ 2
Nu = 2 x ⎜ − ⎟ (26)
α x ⎝ Pr 13 2
Pr 3 ⎠

or
1
⎛ 0.167 0.0333 ⎞ 2
Nu = 2 Rex Pr ⎜ 1
− 2 ⎟ (27)
⎝ Pr 3
Pr 3 ⎠

e.) Prepare a plot showing the Nusselt number for a constant heat flux (from (b)) and increasing
heat flux (from (d)) as a function of Reynolds number for Pr = 2.0.

Equation (27) is implemented in EES:

Nusselt_s2=2*sqrt(Re)*sqrt(Pr)*sqrt(0.167/Pr^(1/3)-0.0333/Pr^(2/3))

The Nusselt number predicted by Eqs. (20) and (27) are illustrated in Figure 2 as a function of
the Reynolds number for Pr = 2.0.
Figure 2: Local Nusselt number predicted by Eqs. (20) (labeled constant heat flux) and (27) (labeled linearly
increasing heat flux) as a function of Reynolds number for Pr = 2.0.
Problem 4.8-3: Flow over a Heated Region
A laser target is cooled by a flow of water, as shown in Figure P4.8-3.

u∞ = 0.9 m/s
T∞ = 20°C

qs′′ = 1x10 W/m


5 2

Luh = 10 cm Lh = 1 cm
x
Figure P4.8-3: A laser target.

The target is Luh = 10 cm from the edge of the plate and extends Lh = 1 cm in the flow direction.
The heat flux from the laser is uniformly distributed, qs′′ = 1x105 W/m2, and entirely removed by
the fluid (rather than being conducted into the plate). The free stream has velocity u∞ = 0.9 m/s,
temperature T∞ = 20ºC, and pressure p∞ = 1 atm. You are to develop a solution to this problem
using an integral technique. Clearly show your derivation for this homework so that partial
credit can be awarded. Make the following assumptions:
1. the properties of water are constant and equal to their value at the free stream
temperature,
2. the flow remains laminar over the plate,
3. use a 2nd order velocity and temperature profile,
4. viscous dissipation is negligible,
5. the free stream velocity, u∞, is constant.

a.) Derive the ordinary differential equation for the momentum boundary layer thickness.

The second order velocity distribution in Table 4-4 is used and the terms associated with shear at
the edge of the boundary layer are neglected. The resulting velocity distribution and surface
shear stress are:

u ⎡ y y2 ⎤
= ⎢2 − ⎥ (1)
u∞ ⎣ δ m δ m2 ⎦

and

u∞
τ s = 2μ (2)
δm

The momentum integral equation can be simplified for this problem:

d ⎡ m 2 ⎤
y =δ
τ
⎢ ∫ ( u − u u∞ ) dy ⎥ = − s (3)
dx ⎢⎣ y =0 ⎥⎦ ρ
Substituting Eqs. (1) and (2) into Eq. (3) leads to:

d ⎡ m ⎛ ⎛ y y2 ⎞ y y2 ⎞ ⎤
y =δ 2
μu
u2

⎢ ∫ ⎜ ⎜ 2 − 2 ⎟ − 2 + 2 ⎟ dy ⎥ = −2 ∞ (4)
dx ⎢ y =0 ⎜ ⎝ δ m δ m ⎠ δm δm ⎟ ⎥ ρ δm
⎣ ⎝ ⎠ ⎦

or

d ⎡ m⎛ y3 y 4 ⎞ ⎤
y =δ
y y2 μu
u ⎢ ∫ ⎜ −2 + 5 2 − 4 3 + 4 ⎟ dy ⎥ = −2 ∞
2
∞ (5)
dx ⎣⎢ y =0 ⎝ δ m δm δ m δ m ⎠ ⎦⎥ ρ δm

Carrying out the integration leads to:

δm
d ⎡⎛ y 2 5 y 3 y 4 1 y 5 ⎞ ⎤ μu
u2
∞ ⎢⎜ − + − + ⎟ ⎥ = −2 ∞ (6)
dx ⎢⎝ δ m 3 δ m2 δ m3 5 δ m4 ⎠0 ⎥⎦ ρ δm

Applying the limits leads to:

2 2 dδ m μu
− u∞ = −2 ∞ (7)
15 dx ρ δm

Solving for the rate of change of the momentum boundary layer:

dδ m 15 μ
= (8)
dx u∞ ρ δ m

which is the ordinary differential equation that governs the boundary layer growth.

b.) Solve the differential equation from (a) in order to obtain an explicit function for the
momentum boundary layer thickness as a function of x.

The variation of the boundary layer thickness with position is the solution to the ordinary
differential equation, Eq. (8), subject to the initial condition:

δ m, x =0 = 0 (9)

The differential equation, Eq. (8), can be integrated analytically from 0 to x:


δm
μ x

∫0 δ m dδ m = 15 u∞ ρ ∫ dx
0
(10)
which leads to:

30 μ x
δm = (11)
u∞ ρ

c.) Derive the ordinary differential equation for the thermal boundary layer thickness within the
heated region. Note that the heat flux rather than the surface temperature is known.

The energy integral equation is simplified:

δ
d ⎡t ⎤ q s′′
⎢ ∫ u (T − T∞ ) dy ⎥ = (12)
dx ⎢⎣ 0 ⎥⎦ ρ c

and rearranged:

d ⎡ u ⎣⎡T − Ts − (T∞ − Ts ) ⎦⎤ ⎤ qs′′


δt

⎢u∞ (T∞ − Ts ) ∫ dy ⎥ = (13)


dx ⎢⎣ 0 ∞
u (T∞ − Ts ) ⎥⎦ ρ c

The appropriate form of the second order temperature distribution from Table 4-5 is:

T − Ts ⎡ y y 2 ⎤
= ⎢2 − ⎥ (14)
T∞ − Ts ⎣ δ t δ t2 ⎦

Substituting Eqs. (1) and (14) into Eq. (13) leads to:

δt
d ⎡ ⎛ y y 2 ⎞⎛ y y 2 ⎞ ⎤ qs′′
⎢ ∞ ( ∞ s ) ∫ ⎜ 2 − 2 ⎟⎜ 2 − 2 − 1⎟ dy ⎥ =
u T − T (15)
dx ⎢⎣ 0⎝
δ m δ m ⎠⎝ δ t δ t ⎠ ⎥⎦ ρ c

Equation (15) can be expanded:

δt
d ⎡ ⎛ y2 y3 y y3 y4 y 2 ⎞ ⎤ q s′′
⎢u∞ (T∞ − Ts ) ∫ ⎜ 4 −2 2 −2 −2 + + ⎟ dy ⎥ = (16)
dx ⎣⎢ 0⎝
δt δm δt δm δm δ t δ m2 δ t2 δ m2 δ m2 ⎠ ⎦⎥ ρ c

The integrations are carried out:

δt
d ⎡ ⎛ 4 y3 1 y4 y2 1 y4 1 y5 1 y3 ⎞ ⎤ q ′′
⎢u∞ (T∞ − Ts ) ⎜ − − − + + ⎟ ⎥= s (17)
dx ⎢ ⎝ 3 δ t δ m 2 δ t2 δ m δ m 2 δ t δ m2 5 δ t2 δ m2 3 δ m2 ⎠0 ⎥⎦ ρ c

Applying the limits:


d ⎡ ⎛ 4 δ t2 1 δ t2 δ t2 1 δ t3 1 δ t3 1 δ t3 ⎞ ⎤ q s′′
u ( T −
⎢ ∞ ∞ s ⎜ T ) − − − + + 2 ⎟⎥
= (18)
dx ⎣ ⎝ 3 δ m 2 δ m δ m 2 δ m 5 δ m 3 δ m ⎠⎦ ρ c
2 2

or

d ⎡ ⎛ δ t2 δ t3 ⎞ ⎤ qs′′
⎢(T∞ − Ts ) ⎜ −0.167 + 0.0333 2 ⎟ ⎥ = (19)
dx ⎣ ⎝ δm δ m ⎠ ⎦ ρ c u∞

The relationship between heat flux and the surface-to-fluid temperature difference from Table 4-
5 is:

qs′′ = 2k
(Ts − T∞ ) (20)
δt

Substituting Eq. (20) into Eq. (19) leads to:

d ⎡ qs′′ δ t ⎛ δ t2 δ t3 ⎞ ⎤ q s′′
⎢ ⎜ 0.167 − 0.0333 2 ⎟ ⎥ = (21)
dx ⎣ 2 k ⎝ δm δ m ⎠ ⎦ ρ c u∞

The heat flux in the heated region is constant, therefore:

d ⎡ δ t3 δ t4 ⎤ 2 α
⎢ − =
δ m2 ⎥⎦ u∞
0.167 0.0333 (22)
dx ⎣ δm

d.) Substitute the ratio of the thermal to momentum boundary layer thickness, r = δt/δm, into
your solution from (c) to obtain a differential equation for r rather than δt.

Substituting the definition of r into Eq. (22) leads to:

d 2α
⎡δ m2 ( 0.167 r 3 − 0.0333 r 4 ) ⎤ = (23)
dx ⎣ ⎦ u∞

e.) Simplify your equation from (d) by neglecting any terms that are obviously small.

The second term on the left side of Eq. (23) will be small relative to the first term because r is
necessarily small and the coefficient is also small:

d 2 3 11.98α
dx
(δ m r ) = u (24)

which leads to:


d 2 3 11.98α
dx
(δ m r ) = u (25)

f.) Solve your equation from (e) in order to obtain an analytical expression for r as a function of
x.

The ordinary differential equation is entered in Maple:

> restart;
> delta_m:=sqrt(30*mu*x/(u_infinity*rho));
μx
delta_m := 30
u_infinity ρ
> ODE:=diff(delta_m^2*r(x)^2,x)=11.98*alpha/u_infinity;
d
60 μ x r( x ) ⎛⎜⎜ r( x ) ⎞⎟⎟
30 μ r( x ) 2
⎝ dx ⎠ = 11.98 α
ODE := +
u_infinity ρ u_infinity ρ u_infinity

The solution is obtained, subject to the initial condition that rx = Lh = 0 :

> rs:=dsolve({ODE,r(L_uh)=0});
8985 x μ α ρ ( x − L_uh ) 8985 x μ α ρ ( x − L_uh )
rs := r( x ) = , r( x ) = −
150 x μ 150 x μ

g.) Use your solution from (f) to prepare a plot of the surface temperature as a function of x
within the heated region.

The inputs are entered in EES:

$UnitSystem SI MASS RAD PA K J


$TabStops 0.2 0.4 3.5 in

u_infinity=0.9 [m/s] "free stream velocity"


T_infinity=converttemp(C,K,20[C]) "free stream temperature"
p_infinity=1 [atm]*convert(atm,Pa) "pressure"
L_uh=10 [cm]*convert(cm,m) "unheated length"
L_h=1 [cm]*convert(cm,m) "heated length"
q``_s=100000 [W/m^2] "laser flux"

k=conductivity(Water,T=T_infinity,P=p_infinity) "conductivity"
rho=density(Water,T=T_infinity,P=p_infinity) "density"
mu=viscosity(Water,T=T_infinity,P=p_infinity) "viscosity"
c=cP(Water,T=T_infinity,P=p_infinity) "specific heat capacity"
alpha=k/(rho*c) "thermal diffusivity"
nu=mu/rho "kinematic viscosity"
Pr=nu/alpha "Prandtl number"
The momentum boundary layer is calculated according to Eq. (11):

x=0.105 [m]
delta_m=sqrt(30*nu*x/u_infinity) "momentum boundary layer thickness"

The positive root from Maple is copied and pasted into EES:

r=1/150/x/mu*8985^(1/2)*(x*mu*alpha*rho*(x-L_uh))^(1/2)
"thermal-to-momentum boundary layer thickness ratio"

The thermal boundary layer thickness is calculated:

δt = r δ m (26)

and the surface temperature is computed using Eq. (20):

delta_t=r*delta_m "thermal boundary layer thickness"


q``_s=2*k*(T_s-T_infinity)/delta_t "surface temperature"
T_s_C=converttemp(K,C,T_s) "in C"

Figure P4.8-3(b) illustrates the temperature as a function of position predicted using the integral
solution.
120

100 estimate for (h)


Temperature (°C)

80

60

estimate for (i)


40 integral solution

20
0.1 0.102 0.104 0.106 0.108 0.11
Position (m)
Figure P4.8-3(b): Temperature as a function of position in the heated region.

h.) Your co-worker suggests that you could simply have used the solution for a plate with a
constant surface temperature to compute a local heat transfer coefficient and therefore the
surface temperature. Evaluate how well this idea would work by overlaying the surface
temperature predicted using this method on your plot from (g).

The Reynolds number based on the position relative to the front of the plate is computed:

x ρ u∞
Rex ,1 = (27)
μ
The local Nusselt number is computed using the correlation provided in Section 4.9.1:

1 1
0.3387 Rex ,12 Pr 3
Nu x ,1 = 1
(4-28)
⎡ ⎛ 0.0468 ⎞ 2
3 ⎤ 4

⎢1 + ⎜ ⎟ ⎥
⎢⎣ ⎝ Pr ⎠ ⎥⎦

and used to calculate the heat transfer coefficient:

k
h1 = Nu x ,1 (4-29)
x

The surface temperature is computed according to:

qs′′
Ts ,1 = T∞ + (4-30)
h1

Re_x1=x*rho*u_infinity/mu "Reynolds number"


Nusselt_x1_uniform1=0.3387*sqrt(Re_x1)*Pr^(1/3)/(1+(0.0468/Pr)^(2/3))^(1/4)
"Nusselt number based on uniform temperature"
h_uniform1=Nusselt_x1_uniform1*k/x "heat transfer coefficient based on uniform temperature"
T_s_uniform1=T_infinity+q``_s/h_uniform1 "surface temperature based on uniform temperature"
T_s_uniform1_C=converttemp(K,C,T_s_uniform1) "in C"

The solution for Ts,1 is overlaid onto Figure P4.8-3(b). Note that the estimate is not very good
because it substantially over-predicts the thermal boundary layer thickness, the thermal
resistance of the boundary layer and therefore the surface temperature.

i.) Your co-worker is somewhat irritated by how bad his idea from (h) was and has now
proposed an alternative approach. He suggests that you could have used the solution for a
plate with a constant surface temperature to compute a local heat transfer coefficient based
on position with respect to the start of the heated region. Evaluate how well this idea would
work by overlaying the surface temperature predicted using this method on your plot from
(g).

The Reynolds number based on the position relative to the front of the heated region is
computed:

Rex ,2 =
( x − Luh ) ρ u∞ (31)
μ

The local Nusselt number is computed using the correlation provided in Section 4.9.1:
1 1
0.3387 Rex ,22 Pr 3
Nu x ,2 = 1
(4-32)
⎡ ⎛ 0.0468 ⎞ 2 3 ⎤ 4

⎢1 + ⎜ ⎟ ⎥
⎢⎣ ⎝ Pr ⎠ ⎥⎦

and used to calculate the heat transfer coefficient:

k
h2 = Nu x ,2 (4-33)
x

The surface temperature is computed according to:

q s′′
Ts ,2 = T∞ + (4-34)
h2

Re_x2=(x-L_uh)*rho*u_infinity/mu "Reynolds number"


Nusselt_x2_uniform2=0.3387*sqrt(Re_x2)*Pr^(1/3)/(1+(0.0468/Pr)^(2/3))^(1/4)
"Nusselt number based on uniform temperature"
h_uniform2=Nusselt_x2_uniform2*k/(x-L_uh) "heat transfer coefficient based on uniform temperature"
T_s_uniform2=T_infinity+q``_s/h_uniform2 "surface temperature based on uniform temperature"
T_s_uniform2_C=converttemp(K,C,T_s_uniform2) "in C"

The solution for Ts,2 is overlaid onto Figure P4.8-3(b). Note that the estimate is better, but still
not that great.
Problem 4.8-4 (4-11 in text): Plate with Unheated Starting Length
Figure P4.8-4 illustrates a flat plate that has an unheated starting length (ε); the hydrodynamic
boundary layer grows from the leading edge of the plate while the thermal boundary layer grows
from x > Luh. Assume that the plate has a constant surface temperature, Ts, for x > Luh.

free stream, u∞ , T∞
⎛ ∂T ⎞
⎜ ∂ ⎟ =0 Ty=0 = Ts
⎝ y ⎠ y =0

Luh
L
x
Figure P4.8-4: Plate with an unheated starting length.

Determine a correlation for the local Nusselt number in this situation using the integral
technique. Use a third order velocity and temperature distribution. Neglect viscous dissipation.
You may find it useful to solve for the ratio of the thermal to momentum boundary layer
thickness.

The momentum boundary layer associated with a 3rd order polynomial velocity distribution was
found in Eq. (4-486):

μx
δ m = 4.640 (1)
ρ u∞

The differential equation that governs the growth of the thermal boundary layer with a 3rd order
polynomial form of the velocity and temperature distribution was found in Eq. (4-524) and
remains valid:

d ⎡ δ t2 δ t4 ⎤ 3 k
⎢ 0.15 − 0.01 3 ⎥ = (2)
dx ⎣ δm δ m ⎦ 2 δ t ρ c u∞

However, because of the unheated starting length, the momentum and thermal boundary layers
do not develop together and therefore their relative size is not dictated by the Prandtl number.
We will define the variable r as the local ratio of the thermal to momentum boundary layer
thicknesses. Note that r = 0 for x < Luh.

δt
r= (3)
δm

Substituting Eq. (3) into Eq. (2) leads to:

⎡( 0.15 r 2 − 0.01 r 4 ) δ m ⎤ =
d 3 k
⎣ ⎦ (4)
dx 2 r δ m ρ c u∞
Neglecting the 2nd term in Eq. (4) (which is much smaller than the first term) leads to:

d 2 10 k
⎡⎣ r δ m ⎤⎦ = (5)
dx r δ m ρ c u∞

or

d 2 α
r δm ⎡⎣ r δ m ⎤⎦ = 10 (6)
dx u∞

Expanding the derivative:

⎡ dδ dr ⎤ α
r δ m ⎢ r 2 m + 2 δ m r ⎥ = 10 (7)
⎣ dx dx ⎦ u∞

or

dδ m dr α
δm r3 + 2 δ m2 r 2 = 10 (8)
dx dx u∞

The hydrodynamic boundary layer develops from the front of the plate. The derivative of Eq. (1)
leads to:

dδ m υ
= 2.320 (9)
dx x u∞

Substituting Eqs. (1) and (9) into Eq. (8) leads to:

υx υ υx dr α
10.765 r3 + 43.059 r2 = 10 (10)
u∞ x u∞ u∞ dx u∞

which can be simplified to:

dr α
10.765 r 3 + 43.059 x r 2 = 10 (11)
dx υ

Equation (11) is an ordinary differential equation that can be solved either numerically or
analytically. The boundary condition is related to the fact that the thermal boundary layer
develops beginning at x = Luh, therefore r = 0 for x < Luh.

rx = Luh = 0 (12)
Here we will solve the ordinary differential equation analytically. Equation (11) can be
rearranged:

dr α
43.059 x r 2 = 10 − 10.765 r 3 (13)
dx υ

Equation (8) is a separable ordinary differential equation; the terms related to x and r are
separated:

r 2 dr dx
= (14)
10 43.059 x
− 10.765 r 3
Pr

and integrated:
r x
r 2 dr dx
∫ 10 = ∫ 43.059 x (15)
0 − 10.765 r 3 Luh
Pr

An intermediate variable, w, is defined:

10
w= − 10.765 r 3 (16)
Pr

and so

dw = −32.295 r 2 dr (17)

which leads to:

10
−10.765 r 3
Pr x
1 dw dx
−32.395 ∫
10
= ∫
w Luh 43.059 x
(18)
Pr

or:

⎛ 10 ⎞
⎜ − 10.765 r 3 ⎟
1 Pr 1 ⎛ x ⎞
ln ⎜ ⎟= ln ⎜ ⎟ (19)
−32.395 ⎜ 10 ⎟ 43.059 ⎝ Luh ⎠
⎜ ⎟
⎝ Pr ⎠

which can be simplified:


⎛ x ⎞
ln (1 − 1.0765 Pr r 3 ) = −0.752 ln ⎜ ⎟ (20)
⎝ Luh ⎠

Taking the exponential of both sides leads to:


−0.752
⎛ x ⎞
1 − 1.0765 Pr r = ⎜ 3
⎟ (21)
⎝ Luh ⎠

Solving for r:

1
⎡ ⎛ Luh ⎞0.75 ⎤ 3

⎢1 − ⎜ ⎟ ⎥
r=⎢ ⎝
x ⎠ ⎥
(22)
⎢ 1.0765 Pr ⎥
⎢ ⎥
⎣ ⎦

The Nusselt number is defined as:

qs′′ x
Nu x = (23)
(Ts − T∞ ) k
The heat flux at the wall for the 3rd order polynomial form of the temperature distribution is
provided in Table 4-5:

3 (Ts − T∞ )
qs′′ = k (24)
2 δt

Substituting Eq. (11) into Eq. (10) leads to:

x 3 (Ts − T∞ )
Nu x = k (24)
(Ts − T∞ ) k 2 δt
or

3 x
Nu x = (25)
2 δt

Notice that Eq. (25) once again reinforces the concept that the Nusselt number is related to the
ratio of the characteristic length for the problem (x) to the effective conduction length in the fluid
(δt). Substituting the definition of r into Eq. (25) leads to:
3 x
Nu x = (26)
2 r δm

Substituting Eq. (22) for r into Eq. (26) leads to:

x (1.0765 Pr )
1
3
3
Nu x = 1
(27)
2 ⎡ ⎛ L ⎞0.75 ⎤ 3

δ m ⎢1 − ⎜ uh ⎟ ⎥
⎢⎣ ⎝ x ⎠ ⎥⎦

Substituting Eq. (1) for δm into Eq. (27) leads to:

x (1.0765 Pr )
1
3
3
Nu x = 1
(28)
2
υ x ⎡ ⎛ Luh ⎞ ⎤
0.75 3

4.640 ⎢1 − ⎜ ⎟ ⎥
u∞ ⎣⎢ ⎝ x ⎠ ⎦⎥

or
1 1
3
0.331 Rex 2 Pr
Nu x = 1
(29)
⎡ ⎛ Luh ⎞ 0.75
⎤ 3

⎢1 − ⎜ ⎟ ⎥
⎢⎣ ⎝ x ⎠ ⎥⎦

Notice that the numerator of Eq. (29) is the Nusselt number that would have existed at x=Luh in
the absence of an unheated starting length and the denominator is a number that is necessarily
less than one and therefore tends to increase the local Nusselt number due to the under-
developed thermal boundary layer thickness. This solution agrees with the solution provided in
Section 4.9.2 for laminar flow over a plate with an unheated starting length.
Problem 4.8-5 (4-12 in text): Low Prandtl Number Flow
A flow of a liquid metal over a flat plate, shown in Figure 4.8-5, is being considered during the
design of an advanced nuclear reactor.

Pr << 1
u∞ , T∞

Ts

x
Figure P4.8-5: Flow of a low Prandtl number liquid metal over a flat plate.

You are to develop a solution to this problem using an integral technique. Because the Prandtl
number is much less than one, it is appropriate to assume that δm << δt and therefore the velocity
is constant and equal to u∞ throughout the thermal boundary layer. Use a 2nd order temperature
distribution and neglect viscous dissipation.
a.) If the temperature of the plate is constant and equal to Ts then derive a solution for the local
Nusselt number on the plate surface as a function of the Reynolds number and the Prandtl
number.

The energy integral equation is simplified:

δ
d ⎡t ⎤ q s′′
⎢ ∫ u (T − T∞ ) dy ⎥ = (1)
dx ⎣⎢ 0 ⎦⎥ ρ c

The velocity within the boundary layer is assumed to be uniform and equal to u∞:

δ
d ⎡ t ⎤ q s′′
⎢u∞ ∫ (T − T∞ ) dy ⎥ = (2)
dx ⎣⎢ 0 ⎦⎥ ρ c

Equation (2) is rearranged:

d ⎡ ⎣⎡T − Ts − (T∞ − Ts ) ⎦⎤ dy ⎤ = qs′′


δt

⎢ ∞( ∞ s)∫
u T − T ⎥ (3)
dx ⎣⎢ 0 (T∞ − Ts ) ⎦⎥ ρ c

The appropriate form of the second order temperature distribution from Table 4-5 is:

T − Ts ⎡ y y 2 ⎤
= ⎢2 − ⎥ (4)
T∞ − Ts ⎣ δ t δ t2 ⎦

Substituting Eq. (4) into Eq. (3) leads to:


δt
d ⎡ ⎛ y y 2 ⎞ ⎤ q s′′
⎢u∞ (T∞ − Ts ) ∫ ⎜ 2 − 2 − 1⎟ dy ⎥ = (5)
dx ⎣⎢ 0 ⎝
δt δt ⎠ ⎦⎥ ρ c

Equation (5) can be integrated:

δt
d ⎡ ⎛ y 2 1 y3 ⎞ ⎤ q ′′
⎢u∞ (T∞ − Ts ) ⎜ − − y ⎟ dy ⎥ = s (6)
dx ⎢ ⎝ δt 3 δt ⎠0 ⎥⎦ ρ c
2

Applying the limits:

d ⎡ u∞ (T∞ − Ts ) δ t ⎤ qs′′
− ⎢ ⎥= (7)
dx ⎣ 3 ⎦ ρc

The relationship between heat flux and the surface-to-fluid temperature difference from Table 4-
5 is:

qs′′ = 2k
(Ts − T∞ ) (8)
δt

Substituting Eq. (8) into Eq. (7) for the unknown heat flux leads to:

d ⎡ u∞ (T∞ − Ts ) δ t ⎤ k (Ts − T∞ )
− ⎢ ⎥=2 (9)
dx ⎣ 3 ⎦ ρ c δt

The surface to free stream temperature difference is constant, therefore:

dδ t 6α
= (10)
dx δ t u∞

Equation (10) is separated:


δ t dδ t = dx (11)
u∞

and integrated:
δt

x

∫ δ t dδ t =
0
u∞ ∫0
dx (12)

which leads to:


12 α x
δ t2 = (13)
u∞

or:

12 α x
δt = (14)
u∞

The local Nusselt number is defined as:

hx qs′′ x
Nu = = (15)
k (Ts − T∞ ) k
Substituting Eq. (8) into Eq. (15) leads to:

2k x 2 x
Nu = = (16)
δt k δt

Substituting Eq. (14) into Eq. (16) leads to:

u∞
Nu = 2 x (17)
12 α x

or

2
Nu = Re Pr (18)
12

b.) Plot your result from (a) as a function of Re for Pr = 0.001. Overlay on your plot the
correlation for the local number for flow over a constant temperature flat plate found in
Section 4.9.

The solution from Section 4.9 that is accurate at low Prandtl number is:
1 1
0.3387 Re 2 Pr 3
Nu = 1
(19)
⎡ ⎛ 0.0468 ⎞ 2 3 ⎤ 4

⎢1 + ⎜ ⎟ ⎥
⎢⎣ ⎝ Pr ⎠ ⎥⎦

The solution from (a) and the correlation above are entered in EES:
$UnitSystem SI MASS RAD PA K J
$TabStops 0.2 0.4 3.5 in

Pr=0.001 [-] "Prandtl number"


Re=10000 [-] "Reynolds number"

Nusselt=(2/sqrt(12))*sqrt(Re)*sqrt(Pr) "Integral solution for a constant pressure plate"


Nusselt_cor=0.3387*sqrt(Re)*Pr^(1/3)/(1+(0.0468/Pr)^(2/3))^(1/4)
"Local Nusselt number correlation from Section 4.9"

Figure 4.8-5(b) illustrates the local Nusselt number predicted by Eqs. (18) and (19) as a function
of Reynolds number for Pr = 0.001.
7

6
Local Nusselt number

integral solution
5

4
Churchill and Ozoe (1973)
3

0
10 100 1000 10000 100000
Reynolds number
Figure 4.8-5(b): Local Nusselt number as a function of Reynolds number for Pr = 0.001 from the integral
solution in (a) and the correlation in Section 4.9.

c.) Plot your result from (a) and the correlation from Section 4.9 as a function of Pr for Re =
1x104. Your Prandtl number range should be from 0.001 to 100 and it should be clear from
your plot that the solutions begin to diverge as the Prandtl number approaches unity.

Figure 4.8-5(c) illustrates the local Nusselt number predicted by Eqs. (18) and (19) as a function
of Prandtl number for Re = 1x104.
700

integral solution

Local Nusselt number


100

10 Churchill and Ozoe (1973)

1
0.5
0.001 0.01 0.1 1 10 100
Prandtl number
Figure 4.8-5(b): Local Nusselt number as a function of Prandtl number for Re = 1x104 from the integral
solution in (a) and the correlation in Section 4.9. Note the divergence at Pr ≈ 1.

d.) If the heat flux at the surface of the plate is constant and equal to qs′′ then derive a solution
for the local Nusselt number on the plate surface as a function of the Reynolds number and
the Prandtl number.

The solution steps from (a) remain the same up to Eq. (7):

d ⎡ u∞ (T∞ − Ts ) δ t ⎤ qs′′
− ⎢ ⎥= (20)
dx ⎣ 3 ⎦ ρc

The relationship between heat flux and the surface-to-fluid temperature difference from Table 4-
5:

qs′′ = 2k
(Ts − T∞ ) (21)
δt

is substituted for the unknown surface to free stream temperature difference:

d ⎡ qs′′ δ t u∞ δ t ⎤ q s′′
=
dx ⎢⎣ 2 k 3 ⎥⎦ ρ c
(22)

The heat flux is constant, therefore:

d 2 6k
dx
( δt ) =
u∞ ρ c
(23)
Equation (23) is separated:


d (δ t2 ) = dx (24)
u∞

and integrated:

δ t2

x

∫ d (δ t ) = u∞ ∫0
dx
2
(25)
0

which leads to:

6α x
δ t2 = (26)
u∞

or:

6α x
δt = (27)
u∞

The local Nusselt number is calculated according to:

2x
Nu = (28)
δt

Substituting Eq. (27) into Eq. (28) leads to:

u∞
Nu = 2 x (29)
6α x

or

2
Nu = Re Pr (30)
6

e.) If the heat flux at the surface of the plate varies linearly with position then derive a solution
for the local Nusselt number on the plate surface as a function of the Reynolds number and
the Prandtl number.

The solution steps from (d) remain the same up to Eq. (22):
d ⎡ qs′′ δ t u∞ δ t ⎤ q s′′
=
dx ⎢⎣ 2 k 3 ⎥⎦ ρ c
(31)

The heat flux is linear with position:

qs′′ = β x (32)

where β is a constant. Substituting Eq. (32) into Eq. (31) leads to:

d ⎡ β x δ t u∞ δ t ⎤ β x
=
dx ⎢⎣ 2 k 3 ⎥⎦ ρ c
(33)

Therefore:

d 6 xα
⎡⎣ x δ t2 ⎤⎦ = (34)
dx u∞

Equation (34) is separated:

6 xα
d ⎡⎣ x δ t2 ⎤⎦ = dx (35)
u∞

and integrated:

x δ t2

x

∫ d ⎡⎣ x δ ⎤⎦ =
u∞ ∫0
x dx
2
t (36)
0

which leads to:

3α x 2
x δ t2 = (37)
u∞

or:

3α x
δt = (38)
u∞

The local Nusselt number is calculated according to:


2x
Nu = (39)
δt

Substituting Eq. (38) into Eq. (39) leads to:

u∞
Nu = 2 x (40)
3α x

or

2
Nu = Re Pr (41)
3

f.) Plot the solutions from (a), (d), and (e) as a function of Reynolds number for Pr = 0.001.

Figure 4.8-5(d) illustrates the local Nusselt number predicted by Eqs. (18), (30), and (41) as a
function of Re for Pr = 0.001.
14

12
Local Nusselt number

10

linearly varying heat flux


8

6 constant heat flux

constant surface temperature


4

0
10 100 1000 10000 100000
Reynolds number
Figure 4.8-5(d): Local Nusselt number as a function of Reynolds number for Pr = 0.001 for the integral
solutions with a constant surface temperature, constant surface heat flux, and linearly varying surface heat
flux.
Problem 4.8-6 (4-13 in text)
Determine the local friction coefficient as a function of Reynolds number for laminar flow over a
flat plate using the momentum integral technique. Assume a velocity distribution of the form:
u / u∞ = a sin ( b y / δ m + c ) where a, b, and c are undetermined constants. Compare your answer
to the Blasius solution from Section 4.4.

The undetermined coefficients for the velocity distribution function are selected so that:

1. The no-slip condition is satisfied at the plate surface:

u y =0 = 0 (1)

2. The free-stream velocity is recovered at the edge of the boundary layer:

u y =δ m = u∞ (2)

3. The shear stress at the edge of the boundary layer is recovered:

∂u
μ = τ y =δ m (3)
∂y y =δ m

Substituting the velocity distribution into Eq. (1) leads to:

⎛ u ⎞
⎜ ⎟ = a sin ( 0 + c ) = 0 (4)
⎝ u∞ ⎠ y = 0

Substituting the velocity distribution into Eq. (2) leads to:

⎛ u ⎞
⎜ ⎟ = a sin ( b + c ) = 1 (5)
⎝ u∞ ⎠ y =δ m

Substituting the velocity distribution into Eq. (3) leads to:

⎛ ∂u ⎞
⎜ ⎟ = a b cos ( b + c ) = 0 (6)
⎝ ∂y ⎠ y =δ m

Equation (4) can be satisfied if c = 0. Equation (6) can be satisfied if b = π/2. Equation (5) can
be satisfied if a = 1. Therefore, the velocity distribution is:

u ⎛π y ⎞
= sin ⎜ ⎟ (7)
u∞ ⎝ 2 δm ⎠
Equation (7) is substituted into the integral form of the momentum equation, simplified for this
problem:

d ⎡ m 2 ⎤
y =δ
τ
⎢ ∫ ( u − u u∞ ) dy ⎥ = − s (8)
dx ⎢⎣ y =0 ⎥⎦ ρ

The shear stress is given by:

∂u
τs = μ (9)
∂y y =0

Substituting Eq. (7) into Eq. (9) leads to:

π
τ s = μ u∞ (10)
2δm

Substituting Eq. (10) into Eq. (8) leads to:

d ⎡ m 2 ⎤
y =δ
μu π
⎢ ∫ ( u − u u∞ ) dy ⎥ = − ∞ (11)
dx ⎣⎢ y =0 ⎦⎥ ρ 2δm

Substituting Eq. (7) into Eq. (11) leads to:

d ⎡ 2 ⎛ π y ⎞⎤ ⎤
y =δ m
⎡ 2 ⎛π y ⎞ μ u∞ π
⎢ u∞
dx ⎣⎢ ∫ ⎢sin ⎜ ⎟ − sin ⎜
⎝ 2 δm ⎠
⎟ ⎥ dy ⎥ = −
⎝ 2 δ m ⎠ ⎦ ⎦⎥ ρ 2δm
(12)
y =0 ⎣

The integral in Eq. (12) is accomplished using Maple:

> restart;
> int((sin(Pi*y/(2*d)))^2-sin(Pi*y/(2*d)),y=0..d);
d ( −4 + π )

d ⎡ 2 (4 −π ) ⎤ μu π
⎢ −u∞ δm ⎥ = − ∞ (13)
dx ⎣ 2π ⎦ ρ 2δm

Equation (13) can be separated and integrated:


δm
μπ2 x

∫ δ m dδ m = ( 4 − π ) ∫ dx (14)
0
ρ u∞ 0

δ m2μπ2
= (4 −π ) x (15)
2 ρ u∞

Solving for the momentum boundary layer thickness:

μπ 2
δm = 2 (4 −π ) x (16)
ρ u∞

Substituting Eq. (16) into Eq. (10) leads to:

μ u∞ ρ u∞
τs = (17)
2 2μ ( 4 − π ) x

Substituting Eq. (17) into the definition of the friction coefficient:

2τ s 2 μ u∞ ρ u∞
Cf = = (18)
ρ u∞ ρ u∞2 2
2
2μ ( 4 − π ) x

or

1 μ
Cf = (19)
2 ( 4 − π ) ρ u∞ x

which can be rearranged:

1
2(4 −π ) 0.763
Cf = = (20)
Rex Rex

The Blasius solution from Section 4.4 provides:

0.664
Cf = (21)
Rex

and therefore the integral technique is 15% in error relative to the exact solution.
Problem 4.8-7 (4-14 in text)
A flat plate that is L = 0.2 m long experiences a heat flux given by:

⎛π x ⎞
qs′′ = qmax
′′ cos ⎜ ⎟ (1)
⎝ 4L ⎠

′′ = 2x104 W/m2. The free stream velocity is u∞ = 20 m/s and the free stream
where qmax
temperature is T∞ = 20°C. The fluid passing over the plate has thermal diffusivity α = 1x10-4
m2/s, conductivity k = 0.5 W/m-K, and Prandtl number Pr = 2.0.
a.) Use a linear temperature distribution and a linear velocity distribution in order to obtain an
ordinary differential equation for the thermal boundary layer thickness.

The integral form of the energy equation, simplified for this problem, is:

δ
d ⎡t ⎤ q s′′
⎢ ∫ u (T − T∞ ) dy ⎥ = (2)
dx ⎣⎢ 0 ⎦⎥ ρ c

Substituting a linear velocity and temperature distribution (from Tables 4-4 and 4-5) into Eq. (2)
leads to:

δ
d ⎡t ⎛ y ⎞⎛ y ⎞ ⎤ qs′′
⎢ ∫ u∞ (T∞ − Ts ) ⎜ ⎟ ⎜ − 1⎟ dy ⎥ = (3)
dx ⎣⎢ 0 ⎝ δ m ⎠ ⎝ δt ⎠ ⎦⎥ ρ c

Equation (3) is rearranged:

δ
d ⎡ t
⎛ y2 y ⎞ ⎤ q ′′
u (
⎢ ∞ ∞ s ∫
T − T ) ⎜ − ⎟ dy ⎥ = s (4)
dx ⎢⎣ 0 ⎝ δ t δ m δ m ⎠ ⎥⎦ ρ c

and integrated:

d ⎡ ⎛ δ t3 δ t2 ⎞ ⎤ qs′′
⎢ ∞ ( ∞ s )⎜
u T − T − ⎟⎥ = (5)
dx ⎣ δ
⎝ t m
3 δ 2 δ m ⎠⎦ ρc

Equation (5) can be simplified:

d ⎡ δ t2 ⎤ qs′′
⎢u∞ (Ts − T∞ ) ⎥= (6)
dx ⎣ 6δm ⎦ ρ c

According to Table 4-5, the heat flux can be written as:


qs′′ = k
(Ts − T∞ ) (7)
δt

Equation (7) is used to eliminate (Ts - T∞):

d ⎡ u∞ δ t3 qs′′ ⎤ qs′′
⎢ ⎥= (8)
dx ⎣ 6 k δ m ⎦ ρ c

The ratio of the thermal to momentum boundary layer thickness is:

δt −1
= Pr 3 (9)
δm

Substituting Eq. (9) into Eq. (8) leads to:

d ⎡ u∞ δ t2 q s′′ − 13 ⎤ qs′′
⎢ Pr ⎥ = (10)
dx ⎣ 6 k ⎦ ρc

which is rearranged:

d 6 α qs′′ 13
⎡⎣δ t2 q s′′⎤⎦ = Pr (11)
dx u∞

Equation (11) is the governing ordinary differential equation for the problem.

b.) Solve the ordinary differential equation from (a) numerically. At the leading edge of the
plate there is a singularity; obtain an analytical solution in this region and start your
numerical solution at the x position where the analytical solution is no longer valid.

Equation (11) is expanded:

dq s′′ dδ 6 α q s′′ 13


δ t2 + 2 δ t q s′′ t = Pr (12)
dx dx u∞

and the specified heat flux variation, Eq. (1), is substituted:

π ⎛π x ⎞ ⎛ π x ⎞ dδ t 6 α ⎛ π x ⎞ 13
′′
−δ t2 qmax sin ⎜ ′′ cos ⎜
⎟ + 2 δ t qmax ⎟ = ′′ cos ⎜
qmax ⎟ Pr (13)
4L ⎝ 4L ⎠ ⎝ 4 L ⎠ dx u∞ ⎝ 4L ⎠

Equation (13) is solved for the rate of change of the thermal boundary layer thickness:
6α ⎛ π x ⎞ 13 2 π ⎛π x ⎞
cos ⎜ ⎟ Pr + δ t sin ⎜ ⎟
d δ t u∞ ⎝ 4L ⎠ 4L ⎝ 4L ⎠
= (14)
dx ⎛π x ⎞
2 δ t cos ⎜ ⎟
⎝ 4L ⎠

Which can be simplified:

dδ t 3α 1 π ⎛π x ⎞
= Pr 3 + δ t tan ⎜ ⎟ (15)
dx u∞ δ t 8L ⎝ 4 L



1st term 2nd term

Very near the leading edge of the plate (where δt approaches zero), the first term in Eq. (15)
dominates the second term. The ratio of the 2nd term to the 1st term is:

π ⎛π x ⎞
δt tan ⎜ ⎟
8L ⎝ 4 L ⎠ = π δ t u∞ tan ⎛ π x ⎞
2
2nd term
= ⎜ ⎟ (16)
1st term 3α 1 1
24 L α Pr 3 ⎝ 4L ⎠
Pr 3
u∞ δ t

Therefore, very near the leading edge of the plate the differential equation simplifies to:

dδ t 3α 1
= Pr 3 (17)
dx u∞ δ t

which can be separated and integrated:


1
6 α Pr 3
δ =t
2
x (18)
u∞

Equation (18) is valid until the 2nd term in Eq. (15) becomes significant (say 0.01). Substituting
Eq. (18) into Eq. (16) provides a criteria for this location, xini:

π xini ⎛π x ⎞
tan ⎜ ini ⎟ = 0.01 (19)
4 L ⎝ 4L ⎠

The inputs are entered in EES:

$UnitSystem SI MASS RAD PA K J


$TABSTOPS 0.2 0.4 0.6 0.8 3.5 in

"Inputs"
L=0.2 [m] "length of plate"
qf_max=20000 [W/m^2] "maximum heat flux"
alpha=1e-4 [m^2/s] "thermal diffusivity"
Pr=2 [-] "Prandtl number"
u_infinity=20 [m/s] "free stream velocity"
k=0.5 [W/m-K] "conductivity"
T_infinity=converttemp(C,K,20[C]) "free stream temperature"

The numerical solution is initiated at xini, determined according to Eq. (19). The thermal
boundary layer thickness at this location, δt,ini, is obtained using Eq. (18):

pi*x_ini*tan(pi*x_ini/(4*L))/(4*L)=0.01 "position to begin numerical solution"


delta_t_ini=sqrt(6*alpha*Pr^(1/3)*x_ini/u_infinity)
"thermal boundary layer at the beginning of numerical solution"

The boundary layer thickness is obtained by integrating Eq. (15) according to:

dδ t
x
δ t = δ t ,ini + ∫
xini
dx
dx (20)

The integrand is evaluated at an arbitrary set of state variables:

delta_t=delta_t_ini
x=x_ini
ddeltatdx=pi*delta_t*tan(pi*x/(4*L))/(8*L)+3*alpha*Pr^(1/3)/(u_infinity*delta_t)

The state variables are commented out and the Integral command in EES is used to obtain the
solution:

{delta_t=delta_t_ini
x=x_ini}
delta_t=delta_t_ini+Integral(ddeltatdx,x,x_ini,L)
$IntegralTable x,delta_t

c.) Plot the surface temperature of the plate as a function of axial position.

The heat flux is related to the surface temperature according to Eq. (7). Solving for the surface
temperature leads to:

δ t qs′′
Ts = T∞ + (21)
k

where the heat flux is computed according to Eq. (1):

qf_s=qf_max*cos(pi*x/(4*L))
T_s=T_infinity+qf_s*delta_t/k
T_s_C=converttemp(K,C,T_s)
$IntegralTable x,delta_t,qf_s,T_s_C

The plate surface temperature as a function of position is shown in Figure 1.


120

110

100

90

Temperature (°C)
80 numerical solution

70

60

50

40 analytical solution

30
0 0.02 0.04 0.06 0.08 0.1 0.12 0.14 0.16 0.18 0.2

Position (m)
Figure 1: Plate surface temperature as a function of position.

d.) Overlay on your plot from (c) the surface temperature calculated using the correlation for the
local heat transfer coefficient on a flat plate.

The kinematic viscosity is calculated from the Prandtl number and thermal diffusivity:

υ = α Pr (22)

The Reynolds number is computed based on position:

u∞ x
Rex = (23)
υ

The local Nusselt number is computed using the correlation for laminar flow over a flat plate
provided in Section 4.9.2:
1 1
0.3387 Rex 2 Pr 3
Nu x = 1
(4-24)
⎡ ⎛ 0.0468 ⎞ 2 3 ⎤ 4

⎢1 + ⎜ ⎟ ⎥
⎢⎣ ⎝ Pr ⎠ ⎥⎦

The heat transfer coefficient is computed based on the Nusselt number:

k Nu
h= (4-25)
x

and the surface temperature is computed according to:

q s′′
Ts = T∞ + (26)
h
where qs′′ is obtained from Eq. (1).

"Solution using a correlation"


nu=alpha*Pr "kinematic viscosity"
Re_x=u_infinity*x/nu "Reynolds number"
Nusselt=0.3387*sqrt(Re_x)*Pr^0.333/(1+(0.0468/Pr)^0.667)^0.25 "local Nusselt number"
h=Nusselt*k/x "local heat transfer coefficient"
qf_s=qf_max*cos(pi*x/(4*L)) "heat flux"
T_s_an=T_infinity+qf_s/h "surface temperature"
T_s_an_C=converttemp(K,C,T_s_an) "in C"

Figure 2 illustrates the numerical solution using the integral technique and the solution calculated
using the correlation for heat transfer coefficient.

120

110
using correlation
100
Temperature (°C)

90

80 integral technique

70

60

50

40

30
0 0.02 0.04 0.06 0.08 0.1 0.12 0.14 0.16 0.18 0.2
Position (m)
Figure 2: Plate surface temperature as a function of position calculated using the integral technique and the
correlation.
Problem 4.8-8
Figure 4.8-8 illustrates the laminar flow of a very low Prandtl number liquid metal (k = 10 W/m-
K, c = 100 J/kg-K, ρ = 1000 kg/m3) over a heated surface. The length of the surface is L = 0.7
m. The free stream velocity and temperature are u∞ = 10 m/s and T∞ = 400 K, respectively.

u∞ = 10 m/s
T∞ = 400 K
⎛ x π⎞
q′′ = q ′′ + Δq′′ sin ⎜ 2 π − ⎟
x ⎝ b 2⎠
L = 0.7 m

b = 0.1 m

heat sources
Figure P4.8-8: Low Prandtl number liquid metal flowing over a heated wall.

There are discrete heat sources buried within the wall separated by a distance b = 0.1 m so that
the heat flux from the wall to the fluid is not uniform but rather varies sinusoidally according to:

⎛ x π⎞
q ′′ ( x ) = q ′′ + Δq ′′ sin ⎜ 2 π − ⎟ (1)
⎝ b 2⎠

where q ′′ = 1x105 W/m2 is the average heat flux and Δq ′′ = 0.5x105 W/m2 is the amplitude of the
variation. Develop a solution to this problem using the integral technique that will allow you to
predict the temperature of the wall as a function of position (x). Follow the steps outlined below
and clearly indicate your work.
a.) The liquid metal has a low Prandtl number and therefore the thermal boundary layer will be
much larger than the momentum boundary layer. Therefore, it is appropriate to assume that
the velocity is constant within the thermal boundary layer: u ( y ) = u∞ . You should assume a
linear temperature distribution to implement your integral technique. Using this approach,
develop a differential equation that describes the thermal boundary layer thickness.

Starting with the integral form of the energy equation, simplified to eliminate the transpiration
and dissipation terms:
δ
d T qw′′
( )
dx ∫0
u T − T∞ dy = (2)
ρc

A simple, linear temperature distribution is given by:

T − T∞ y
θ= = 1− (3)
Tw − T∞ δT

Substituting Eq. (3) into Eq. (2) and assuming a uniform velocity distribution leads to:
δ
d T ⎛ y ⎞ qw′′
∫ u ∞ ( T − T∞ ) ⎜1 − ⎟ dy = (4)
⎝ δT ρc
w
dx 0 ⎠

The heat flux at the wall can be obtained from the temperature distribution:

⎛ ∂T ⎞
qw′′ = −k ⎜
(Tw − T∞ )
⎟ =k (5)
⎝ ∂y ⎠ y =0 δt

Because the heat flux at the wall is specified (rather than the temperature at the wall), we will use
Eq. (5) to substitute in for the temperature difference in Eq. (4):
δ
d T δ t qw′′ ⎛ y ⎞ qw′′
dx ∫0
u ∞ ⎜ 1 − ⎟ dy = (6)
k ⎝ δT ⎠ ρc

or, after carrying out the integral:

δT
d ⎡ δ t qw′′ ⎛ y2 ⎞⎤ qw′′
u
⎢ ∞ ⎜ y − ⎟⎥ = (7)
dx ⎣ k ⎝ 2δ T ⎠⎦ 0 ρc

δT
d ⎡ δ t2 q w′′ ⎤ qw′′
⎢ u∞ ⎥ = (8)
dx ⎣ 2 k ⎦0 ρc

Note that the wall heat flux is not constant and therefore cannot be canceled from each side.
Expanding the derivative in Eq. (8) leads to:

dδ t δ t2 dq ′′ α qw′′
q w′′ δ t + = (9)
dx 2 dx u∞

where q ′′ is given by Eq. (1) and

dq ′′ 2 π Δq ′′ ⎛ x π⎞
= cos ⎜ 2 π − ⎟ (10)
dx b ⎝ b 2⎠

b.) Using EES, numerically integrate your expression from (a) in order to determine the
boundary layer thickness as a function of x. Prepare a plot of your result and turn in your
EES file.

The inputs are entered in EES:

$UnitSystem SI MASS RAD PA K J


$TabStops 0.2 0.4 3.5 in

"Inputs"
b=0.1 "spacing between sources"
L=0.7 "length of wall"
T_inf=400 [K] "fluid temperature"
qfbar=1e5 [W/m^2] "average heat flux"
Dqf=0.5e5 [W/m^2] "amplitude of heat flux variation"
rho=1000 [kg/m^3] "fluid density"
c=100 [J/kg-K] "specific heat capacity"
k=10 [W/m-K] "conductivity"
u_inf=10 [m/s] "free stream velocity"
alpha=k/(rho*c) "thermal diffusivity"

Nodes are distributed uniformly along the plate according to:

L ( i − 1)
xi = for i = 1..N (11)
( N − 1)
where N is the number of nodes. The distance between adjacent nodes is:

L
Δx = (12)
( N − 1)
N=100 [-] "number of nodes"
duplicate i=1,N
x[i]=L*(i-1)/(N-1) "nodal locations"
end
Dx=L/(N-1) "distance between nodes"

The heat flux and the gradient of the heat flux at each node are computed using Eqs. (1) and (10),
respectively.

duplicate i=1,N
qf[i]=qfbar+Dqf*sin(2*pi*x[i]/b-pi/2) "heat flux"
dqfdx[i]=Dqf*2*pi*cos(2*pi*x[i]/b-pi/2)/b "gradient of heat flux"
end

The initial boundary layer thickness is taken to be small (but not zero) so that the numerical
technique remains stable. The initial wall temperature is T∞.

dt[1]=1e-4 [m] "initial boundary layer thickness"


Tw[1]=T_inf "initial wall temperature"

The state equation, Eq. (9), is integrated using Euler steps. The surface temperature is computed
according to Eq. (5):

duplicate i=1,(N-1)
ddtdx[i]=(alpha*qf[i]/u_inf-dt[i]^2*dqfdx[i]/2)/(qf[i]*dt[i]) "rate of change of the boundary layer thickness"
dt[i+1]=dt[i]+ddtdx[i]*dx "Euler step"
Tw[i+1]=T_inf+qf[i+1]*dt[i+1]/k "surface temperature"
end

Figure 2 shows the thermal boundary layer thickness as a function of position.

Figure 2: Thermal boundary layer thickness as a function of position

c.) Using your EES solution, generate a plot that shows the wall temperature as a function of
position.

Figure 3 illustrates the wall tempeature as a function of position.

Figure 3: Wall temperature as a function of position.


Problem 4.9-1: A Hot Film Anemometer
Figure P4.9-1 illustrates a hot film anemometer used to measure air velocity. The anemometer is
a thin plate of metal that is oriented parallel to the oncoming air flow. The free stream
temperature (T∞) and the plate surface temperature (Ts - assume that the plate is at a constant
temperature) are both measured. The plate temperature is controlled by electrically heating the
material. The plate temperature is always kept ΔT = 5 K higher than the free stream temperature
by varying the electrical power provided, qe . The heat transfer coefficient between the surface
of the plate and the air flow depends on air velocity and therefore it is possible to relate qe to the
free stream velocity, u∞. The length of the plate in the direction of flow is L = 10.0 mm and the
width of the plate is W = 25 mm. The plate is placed in an air flow at pressure p∞ = 1 atm and T∞
= 20°C and the power required by the hot film is qe = 0.20 W. You can neglect radiation for this
problem. Note that both sides of the plate (the top and the bottom in Figure 1) convect to the
free stream.

W = 25 mm
air at u∞
T∞ = 20°C
p∞ = 1 atm
L = 10 mm

plate is electrically heated


air at u∞
T∞ = 20°C
p∞ = 1 atm L = 10 mm
Figure P4.9-1: A hot film anemometer.

a.) What is the air velocity, u∞?

The inputs are entered into EES.

$UnitSystem SI MASS RAD PA K J


$TABSTOPS 0.2 0.4 0.6 0.8 3.5 in

"Inputs"
L = 10 [mm]*convert(mm,m) "length of anemometer"
W = 25 [mm]*convert(mm,m) "width of anemometer"
T_inf=converttemp(C,K,20) "free stream temperature"
DeltaT=5 [K] "temperature difference maintained by heating"
q_dot_e=0.20 [W] "electrical power"
p_inf=1 [atm]*convert(atm,Pa) "operating pressure"

The surface temperature of the plate is computed:

Ts = T∞ + ΔT (1)

The properties of air (ρ, k, c, and μ) are computed at the film temperature:
T film =
(T∞ + Ts ) (2)
2

"get air properties"


T_s=T_inf+DeltaT "plate temperature"
T_film=(T_s+T_inf)/2 "film temperature"
rho=density(Air,T=T_film,p=p_inf) "density"
c=cP(Air,T=T_film) "specific heat capacity"
mu=viscosity(Air,T=T_film) "viscosity"
k=conductivity(Air,T=T_film) "conductivity"

The thermal diffusivity, kinematic viscosity, and Prandtl number are computed:

k
α= (3)
ρc

μ
ν= (4)
ρ

ν
Pr = (5)
α

alpha=k/(rho*c) "thermal diffusivity"


nu=mu/rho "viscosity"
Pr=nu/alpha "Prandtl number"

In order to calculate the Reynolds number

u∞ ρ L
Re = (6)
μ

it is necessary to assume a value of the free stream velocity:

u_inf=11.0 [m/s] "guess for free stream velocity"


Re=rho*L*u_inf/mu "Reynolds number"

The Reynolds number and Prandtl number are used to access the correlation for the average
Nusselt number ( Nu ) and friction factor for a flat plate:

Call External_Flow_Plate_ND(Re,Pr: Nusselt,C_f) "call correlation"

The Nusselt number is used to compute the heat transfer coefficient:

Nu k
h= (7)
L
h=Nusselt*k/L "heat transfer coefficient"

The electrical energy required to keep the plate at Ts is computed:

qe = 2 h LW (Ts − T∞ ) (8)

q_dot_e2=2*h*L*W*(T_s-T_inf) "electrical power, calculated"

which leads to qe = 0.31 W. This answer is not equal to the specified value of qe (because my
guess for u∞ was not correct). Therefore, the guess values are updated (select Update Guesses
from the Calculate menu), the value of the free stream velocity is commented out, and the
calculated and specified values of qe are set equal to one another.

{u_inf=11.0 [m/s]} "guess for free stream velocity"


q_dot_e=q_dot_e2
"set calculated and specified values equal"

which leads to u∞ = 4.45 m/s.

b.) Plot the air velocity as a function of qe . This is the calibration curve for the anemometer; if
you were using the anemometer you would measure qe and use this curve to determine u∞.

A parametric table is set up which includes the variables q_dot_e and u_inf. The value of u_inf is
varied from 0.1 m/s to 50 m/s and the results are shown in Figure 2.

Figure 2: Air velocity as a function of the electrical power provided to the anemometer.
c.) Assess the limits of the operating temperature range for the anemometer. If you change the
operating temperature (T∞) by ±50 K will the calibration curve from (b) still be applicable?
Justify your answer with a plot.

The ambient temperature is increased and decreased by 50 K; the resulting plots for u∞ vs qe are
overlaid on the calibration curve in Figure 3. Note that the change is not very large and so the
anemometer should function well within these temperature limits.

Figure 3: Air velocity as a function of the electrical power provided to the anemometer.
Problem 4.9-2 (4-15 in text): Energy Loss from a High Rise Apartment
You and your friend are looking for an apartment in a high-rise building. You have your choice
of 4 different south-facing units (units #2 through #5) in a city where the wind is predominately
from west-to-east, as shown in Figure P4.9-2. You are responsible for paying the heating bill for
your apartment and you have noticed that the exterior wall is pretty cheaply built. Your friend
has taken heat transfer and therefore is convinced that you should take unit #5 in order to
minimize the cost of heating the unit because the boundary layer will be thickest and heat
transfer coefficient smallest for the exterior wall of that unit. Prepare an analysis that can predict
the cost of heating each of the 4 units so that you can (a) decide whether the difference is worth
considering, and (b) if it is, choose the optimal unit. Assume that the heating season is time = 4
months long (120 days) and the average outdoor air temperature during that time is T∞ = 0°C
and average wind velocity is u∞ = 5 mph. The dimensions of the external walls are provided in
Figure P4-15; assume that no heat loss occurs except through the external walls. Further, assume
that the walls have a total thermal resistance on a unit area basis (not including convection) of
Rw′′ = 1 K-m2/W. The internal heat transfer coefficient is hin = 10 W/m2-K. You like to keep
your apartment at Tin = 22°C and use electric heating at a cost of ec = 0.15$/kW-hr. You may
use the properties of air at T∞ for your analysis and neglect the effect of any windows.

North
top view

wind,
u∞ = 1.2 mph edge of building
T∞ = 0°C

side view H = 12 ft
wind,
u∞ = 1.2 mph
T∞ = 0°C unit #1 #2 #3 #4 #5 #6

L1 = 20 ft
L2 = 40 ft
L3 = 60 ft
L4 = 80 ft
L5 = 80 ft
L6 = 100 ft
Figure P4.9-2: Location of the external wall of units 2 through 5 relative to the wind direction.

a.) Determine the average yearly heating cost for each of the 4 units and discuss which
apartment is best and why.

The known information is entered in EES:

$UnitSystem SI MASS RAD PA K J


$TABSTOPS 0.2 0.4 0.6 0.8 3.5 in

"Inputs"
u_infinity= 5.0 [mile/hr]*convert(mile/hr,m/s) "wind velocity"
T_infinity=converttemp(C,K,0) "wind temperature"
time=120 [day]*convert(day,s) "heating time"
R``_w=1 [K-m^2/W] "wall resistance on a unit area basis"
h_in=10 [W/m^2-K] "internal heat transfer coefficient"
ec=0.15 [$/kW-hr]*convert($/kW-hr,$/J) "electrical energy cost"
T_in=converttemp(C,K,22) "internal temperature"
H=12 [ft]*convert(ft,m) "heigth of apartment"
L2=20 [ft]*convert(ft,m) "position of unit 2 - start"
L3=40 [ft]*convert(ft,m) "position of unit 3 -start"
L4=60 [ft]*convert(ft,m) "position of unit 4 - start"
L5=80 [ft]*convert(ft,m) "position of unit 5 - start"
L6=100 [ft]*convert(ft,m) "end of unit 5"

The properties of air (ρ, k, μ, and Pr) are obtained using EES’ built-in property routines:

"Air properties"
rho=density(Air,T=T_infinity,P=1[atm]*convert(atm,Pa)) "density of air"
mu=viscosity(Air,T=T_infinity) "viscosity of air"
k=conductivity(Air,T=T_infinity) "conductivity of air"
Pr=Prandtl(Air,T=T_infinity) "Prandtl number of air"

We are interested in the heat transferred through each of the sub-sections of the wall outlined by
the red, dashed lines in Figure P4.9-2. However, there is no convenient formula for the average
heat transfer coefficient over a range of position; therefore, the total side of the wall will be split
into 5 plates corresponding to unit#1, units #1&2, units #1 through #3, etc. The heat loss
through each of these 5 plates will be computed and the heat loss through each unit individually
may be obtained from subtraction.

The Reynolds number at the trailing edge of each of these plates is computed according to:

L1 ρ u∞
ReL2 = (1)
μ

L2 ρ u∞
ReL3 = (2)
μ

and so on through ReL6 :

"Reynolds numbers"
Re_L2=rho*u_infinity*L2/mu
Re_L3=rho*u_infinity*L3/mu
Re_L4=rho*u_infinity*L4/mu
Re_L5=rho*u_infinity*L5/mu
Re_L6=rho*u_infinity*L6/mu

Because the leading edge of each of the plates discussed above start at the beginning of the
boundary layer development, the average Nusselt number over each of the plates ( Nu L2 through
Nu L6 ) can be obtained using EES’ built-in External_Flow_Plate_ND function:

"Nusselt numbers"
Call External_Flow_Plate_ND(Re_L2,Pr: Nusselt_bar_L2,C_f_L2)
Call External_Flow_Plate_ND(Re_L3,Pr: Nusselt_bar_L3,C_f_L3)
Call External_Flow_Plate_ND(Re_L4,Pr: Nusselt_bar_L4,C_f_L4)
Call External_Flow_Plate_ND(Re_L5,Pr: Nusselt_bar_L5,C_f_L5)
Call External_Flow_Plate_ND(Re_L6,Pr: Nusselt_bar_L6,C_f_L6)

The average heat transfer coefficient over each of the plates is:

Nu L2 k
hL2 = (3)
L2

Nu L3 k
hL3 = (4)
L3

and so on to hL6 :

"average heat transfer coefficients"


h_bar_L2=Nusselt_bar_L2*k/L2
h_bar_L3=Nusselt_bar_L3*k/L3
h_bar_L4=Nusselt_bar_L4*k/L4
h_bar_L5=Nusselt_bar_L5*k/L5
h_bar_L6=Nusselt_bar_L6*k/L6

The total heat transfer rate for each of the plates is computed. The total resistance to heat
transfer to the plate that includes units #1 through #5 is:

1 R′′ 1
R1to 5 = + w + (5)
hin H L6 H L6 hL6 H L6

and so the total heat transfer rate through the plate is:

q1to 5 =
(Tin − T∞ ) = H L6 (Tin − T∞ ) (6)
R1to 5 1
+ Rw′′ +
1
hin hL6

The heat transfer rate for the plate that includes units #1 through #4 is:

H L5 (Tin − T∞ )
q1to 4 = (7)
1 1
+ Rw′′ +
hin hL5

and so on through q1 :

"heat transfer rates for successively smaller regions"


q_dot_1to5=(T_in-T_infinity)*H*L6/(1/h_in+R``_w+1/h_bar_L6)
q_dot_1to4=(T_in-T_infinity)*H*L5/(1/h_in+R``_w+1/h_bar_L5)
q_dot_1to3=(T_in-T_infinity)*H*L4/(1/h_in+R``_w+1/h_bar_L4)
q_dot_1to2=(T_in-T_infinity)*H*L3/(1/h_in+R``_w+1/h_bar_L3)
q_dot_1=(T_in-T_infinity)*H*L2/(1/h_in+R``_w+1/h_bar_L2)

The heat transfer rate from unit #2 is the difference between the heat transfer rate through the
plate that includes units #1 and #2 ( q1to 2 ) and the plate that includes only unit #1 ( q1 ):

q2 = q1to 2 − q1 (8)

The heat transfer rate from unit #3 is:

q3 = q1to 3 − q1to 2 (9)

and so on through q5 :

"heat transfer rates for units of interest"


q_dot_2=q_dot_1to2-q_dot_1
q_dot_3=q_dot_1to3-q_dot_1to2
q_dot_4=q_dot_1to4-q_dot_1to3
q_dot_5=q_dot_1to5-q_dot_1to4

The heating cost for each unit is the product of its heat transfer rate, the amount of heating tim,
and the cost of energy:

Cost2 = ec time q2 (10)

Cost3 = ec time q3 (11)

through Cost5:

"heating costs for units of interest"


Cost2=ec*q_dot_2*time
Cost3=ec*q_dot_3*time
Cost4=ec*q_dot_4*time
Cost5=ec*q_dot_5*time

The solution is summarized in Table 1.

Table 1: Summary of unit-by-unit heating costs.


Unit Est. heating cost
($/year)
#2 168.0$
#3 165.4$
#4 163.9$
#5 162.8$
There is a minimal difference in the heating cost; not enough to worry about. The Reynolds
number for every unit is greater than 5x105 and therefore the flow is turbulent throughout. The
average heat transfer coefficient for the turbulent flow is not strongly dependent on position and
that is why the difference is not large).

b.) Prepare a plot showing the heating cost for unit #2 and unit #5 as a function of the wind
velocity for the range 0.5 mph to 5.0 mph. Explain any interesting characteristics that you
observe.

A Parametric Table is generated that includes the variables Cost_1 through Cost_5 and
u_infinity_mph where u_infinity_mph is used to set u_infinity in the Equation Window:

u_infinity= u_infinity_mph * convert(mile/hr,m/s) "wind velocity"

The value of the variable u_infinity_mph in the Parametric Table is varied from 0.5 mph to 5.0
mph. The heating cost for units #2 and #5 are shown in Figure 2 as a function of wind velocity.

Figure 2: Yearly heating cost for units #2 and #5 as a function of the wind velocity.

Notice that at high wind velocity it is slightly more economical to live in unit #5 because
essentially the entire face of the building is turbulent and unit #5 is further along the boundary
layer. However, at lower wind velocity (around 1.8 mile/hr), it becomes more economical to live
in unit #2 because the laminar boundary layer (with a substantially lower heat transfer
coefficient) stretches across the face of unit #2, insulating it.
Problem 4.9-3 (4-16 in text): Solar Panel
A solar photovoltaic panel is mounted on a mobile traffic sign in order to provide power without
being connected to the grid. The panel is W = 0.75 m wide by L = 0.5 m long. The wind blows
across the panel with velocity u∞ = 5 miles/hr and temperature T∞ = 90°F, as shown in Figure
P4.9-3. The back side of the panel is insulated. The panel surface has an emissivity of ε = 1.0
and radiates to surroundings at T∞. The PV panel receives a solar flux of qs′′ = 490 W/m2. The
panel produces electricity with an efficiency η (that is, the amount of electrical energy produced
by the panel is the product of the efficiency, the solar flux, and the panel area). The efficiency of
the panel is a function of surface temperature; at 20°C the efficiency is 15% and the efficiency
drops by 0.25%/K as the surface temperature increases (i.e., if the panel surface is at 40°C then
the efficiency has been reduced to 10%). All of the solar radiation absorbed by the panel and not
transformed into electrical energy must be either radiated or convected to its surroundings.
L = 0.5 m

u∞ = 5 mph
T∞ = 90°F W = 0.75 m

top view
qs′′ = 490 W/m
2

u∞ = 5 mph
T∞ = 90°F

side view
Figure P4.9-3: Solar panel for providing power to a remote traffic sign.

a.) Determine the panel surface temperature, Ts, and the amount of electrical energy generated
by the panel.

The inputs are entered in EES:

$UnitSystem SI MASS RAD PA K J


$TABSTOPS 0.2 0.4 0.6 0.8 3.5 in

L = 0.5 [m] "length of the panel"


W=0.75 [m] "width of the panel"
qf_s=490 [W/m^2] "solar flux"
T_inf=converttemp(F,K,90) "ambient temperature"
u_mph=5 [mph] "wind speed in mph"
u_inf=u_mph*convert(mph,m/s) "wind speed"
em=1.0 [-] "emissivity of panel surface"

In order to carry out the calculations required by this problem it is necessary to assume a surface
temperature, Ts.

T_s=315 [K] "guess for surface temperature"


Based on the assumed surface temperature it is possible to compute the efficiency of the panel:

η = 0.15 − (Ts − 293.2 [ K ]) 0.0025 [1/K ] (1)

and the electrical energy produced can be computed:

w e = q s′′W Lη (2)

eta=0.15 [-] - (T_s-293.2 [K])*0.0025 [-/K] "efficiency"


w_dot_e=eta*L*W*qf_s "electrical power generated"

The surface temperature is used to compute the film temperature:

Ts + T∞
T film = (3)
2

and the properties of air (ρ, μ, k, and Pr).

T_film=(T_s+T_inf)/2 "film temperature"


rho=density(Air,T=T_film, P=1 [atm]*convert(atm,Pa)) "density"
mu=viscosity(Air,T=T_film) "viscosity"
k=conductivity(Air,T=T_film) "conductivity"
Pr=Prandtl(Air,T=T_film) "Prandtl number"

The Reynolds number is computed:

ρ u∞ L
Re = (4)
μ

and used to call the procedure that implements the correlations for external flow over a flat plate;
the correlations return the average Nusselt number ( Nu ) and friction coefficient ( C f ). The
average Nusselt number is used to compute the average heat transfer coefficient:

Nu k
h= (5)
L

Re=rho*u_inf*L/mu "Reynolds number"


Call External_Flow_Plate_ND(Re,Pr: Nusselt,C_f) "call correlations"
h_bar=Nusselt*k/L "heat transfer coefficient"

An energy balance on the solar panel includes inflow from the solar flux and outflow to electrical
energy, convection, and radiation:

qs′′W L = w e + h W L (Ts , f − T∞ ) + ε σ W L (Ts4, f − T∞4 ) (6)


where Ts,f is the final calculated value of the surface temperature.

qf_s*W*L=h_bar*W*L*(T_s_f-T_inf)+sigma#*em*W*L*(T_s_f^4-T_inf^4)+w_dot_e
"energy balance on the solar panel"

which leads to Ts,f = 334.1 K. At this point, you can either manually adjust your guess value for
Ts until Ts = Ts,f or, better yet, select Update Guesses from the Calculate Menu and then
comment out your guess for Ts and specify that Ts = Ts,f.

{T_s=315 [K]} "guess for surface temperature"


T_s_f=T_s

which leads to Ts = 335.7 K and w e = 8.1 W.

b.) Prepare a plot of the electrical energy generated by the panel as a function of the solar flux
for qs′′ ranging from 100 W/m2 to 700 W/m2. Your plot should show that there is an optimal
value for the solar flux – explain this result.

The solar flux is commented out and a parametric table is generated that includes the variables
qf_s and w_dot_e. The result is illustrated in Figure 2.

Figure 2: Electrical power as a function of the solar flux.

Notice that there is a maximum in Figure 2; as the solar flux increases the power initially
increases. However, the surface temperature increases as well and so the efficiency decreases. If
the flux becomes very large then the efficiency is so poor that the electrical power will begin to
decrease.
c.) Prepare a plot of the electrical energy generated by the panel as a function of the wind
velocity (with qs′′ = 490 W/m2) for u∞ ranging from 5 mph to 50 mph – explain any
interesting aspects of your plot.

Figure 3 illustrates the electrical power produced by the panel as a function of the wind velocity.
As the wind velocity increases, the heat transfer coefficient increases and this causes the surface
temperature to be reduced. The efficiency therefore is improved and this leads to the
improvement in performance shown in Figure 3. Note that around u∞ = 40 mph the boundary
layer transitions to turbulence which leads to a sharp increase in the heat transfer coefficient and
improvement in the performance.

Figure 3: Electrical power and shear force as a function of the wind velocity.

d.) Prepare a plot of the shear force experienced by the panel due to the wind as a function of
wind velocity for u∞ ranging from 5 mph to 50 mph – explain any interesting aspects of your
plot.

The average friction coefficient is used to compute the total shear force on the panel:

u∞2
F = Cf ρ WL (7)
2

F=C_f*rho*u_inf^2*L*W/2 "frictional force"

The force is overlaid on Figure 3 as a function of the wind velocity. Note the sharp increase
when the boundary layer becomes turbulent.
Problem 4.9-4 (4-17 in text): Wind Chill Temperature
The wind chill temperature is loosely defined as the temperature that it "feels like" outside when
the wind is blowing. More precisely, the wind chill temperature is the temperature of still air
that would produce the same bare skin temperature that you experience on a windy day. If you
are alive then you are always transferring thermal energy (at rate q ) from your skin (at
temperature Tskin). On a windy day, this heat loss is resisted by a convection resistance where the
heat transfer coefficient is related to forced convection (Rconv,fc), as shown in Figure P4.9-4(a).
The skin temperature is therefore greater than the air temperature (Tair). On a still day, this heat
loss is resisted by a lower convection resistance because the heat transfer coefficient is related to
natural convection (Rconv,nc), as shown in Fig. P4.9-4(b). For a given heat loss, air temperature,
and wind velocity, the wind chill temperature (TWC) is the temperature of still air that produces
the same skin temperature.

q q
Tair Tskin TWC Tskin

Rconv,fc Rconv,nc
(a) (b)
Figure P4.9-4: Resistance network for a body losing heat on (a) a windy and (b) a still day.

It is surprisingly complicated to compute the wind chill temperature because it requires that you
know the rate at which the body is losing heat and the heat transfer coefficient between a body
and air on both a windy and still day. At the same time, the wind chill temperature is important
and controversial because it affects winter tourism in many places. The military and other
government agencies that deploy personnel in extreme climates are also very interested in the
wind chill temperature in order to establish allowable exposure limits. This problem looks at the
wind chill temperature using your heat transfer background. It has been shown that the heat
transfer coefficient for most animals can be obtained by treating them as if they were spherical
with an equivalent volume.
a.) What is the diameter of a sphere that has the same volume as a man weighing M = 170 lbm
(assume that the density of human flesh is ρf = 64 lbm/ft3)?

The inputs are entered in EES:

$UnitSystem SI MASS RAD PA K J


$TABSTOPS 0.2 0.4 0.6 0.8 3.5 in

"Inputs"
M = 170 [lbm]*convert(lbm,kg) "mass of human"
rho_f=64 [lbm/ft^3]*convert(lbm/ft^3,kg/m^3) "density of flesh"

The volume of the man is computed according to:

M
V= (1)
ρf
and the diameter of a sphere with the same volume is computed according to:

3
4 ⎛D⎞
V = π⎜ ⎟ (2)
3 ⎝2⎠

V=M/rho_f "volume of human"


V=4*pi*(D/2)^3/3 "equivalent sphere diameter"

which leads to a sphere diameter of D = 0.52 m.

b.) Assuming that the man can be treated as a sphere, compute the skin temperature for the man
on a day when the wind blows at V = 10 mph and the air temperature is Tair = 0°F. Assume
that the metabolic heat generation for the man is q = 150 W.

The additional inputs are entered in EES:

v_wind=10 [mph]*convert(mph,m/s) "wind velocity"


T_air=converttemp(F,K,0) "air temperature"
q_dot=150 [W] "metabolic heat generation"

The properties of air must be computed. The film temperature is not known and is therefore
assumed to be equal to the air temperature to start the process. The properties of air (μa, ka, ρa,
and Pra) are obtained using EES' built-in property routines.

"Air Properties"
T_film=T_air "film temperature (assumed)"
mu_a=viscosity(Air,T=T_film) "viscosity"
k_a=conductivity(Air,T=T_film) "conductivity"
Pr_a=Prandtl(Air,T=T_film) "Prandtl number"
rho_a=density(Air,T=T_film,P=1 [atm]*convert(atm,Pa)) "density"

The Reynolds number is computed according to:

ρ a DV
ReD = (3)
μa

and used to evaluate the average Nusselt number with the function External_Flow_Sphere_ND.
The average Nusselt number is used to compute the forced convection heat transfer coefficient
(hfc).

h fc D
Nu D = (4)
ka

Re_D=rho_a*v_wind*D/mu_a "Reynolds number"


Call External_Flow_Sphere_ND(Re_D,Pr_a :C_d, Nusselt) "access correlations for a sphere"
Nusselt=h_fc*D/k_a
"obtain forced convection heat transfer coefficient"

The resistance to forced convection is:

1
Rconv , fc = 2
(5)
⎛D⎞
h fc 4 π ⎜ ⎟
⎝2⎠

and so the skin temperature is:

Tskin = Tair + Rconv , fc q (6)

R_conv_fc=1/(h_fc*4*pi*(D/2)^2) "forced convection thermal resistance"


T_skin=T_air+q_dot*R_conv_fc "skin temperature"
T_skin_F=converttemp(K,F,T_skin) "skin temperature in F"

The film temperature is recomputed based on the skin temperature:

{T_film=T_air "film temperature (assumed)"}


T_film=(T_air+T_skin)/2 "film temperature"

The skin temperature is 267.5 K (21.8°F).

c.) Assume that the natural convection heat transfer coefficient that would occur on a day with
no wind is hnc = 8.0 W/m2-K. What is the wind chill temperature?

The resistance to natural convection (Rconv,nc) is:

1
Rconv ,nc = 2
(7)
⎛D⎞
hnc 4 π ⎜ ⎟
⎝2⎠

and so the wind chill temperature is:

TWC = Tskin − q Rconv ,nc (8)

h_nc=10 [W/m^2-K]
"natural convection heat transfer coefficient"
R_conv_nc=1/(h_nc*4*pi*(D/2)^2) "natural convection thermal resistance"
T_WC=T_skin-q_dot*R_conv_nc "wind chill temperature"
T_WC_F=converttemp(K,F,T_WC) "wind chill temperature in F"

The wind chill temperature on the day with Tair = 0°F and V = 10 mph is TWC = -17.4°F.

According to the National Weather Service (http://www.weather.gov/om/windchill/), the wind


chill temperature can be computed according to:
TWC = 35.74 + 0.6215 Tair − 35.75V 0.16 + 0.4275 Tair V 0.16 (9)

where Tair is the air temperature in °F and V is the wind velocity in mph.

d.) Use the National Weather Service equation to compute TWC on a day when Tair = 0°F and V =
10 mph.

Equation (9) is programmed in EES:

"from the National Weather Service"


T_WC_NWS_F=35.74 [F] +0.6215*converttemp(K,F,T_air)
-35.75[F/mph^0.16]*(v_wind*convert(m/s,mph))^0.16
+0.4275 [1/mph^0.16]*converttemp(K,F,T_air)*(v_wind*convert(m/s,mph))^0.16

The wind chill temperature predicted by the National Weather Service equation is -15.9°F.

e.) Plot the wind chill temperature on a day with Tair = 0°F as a function of the wind velocity;
show the value predicted by your model and by the National Weather Service equation for
wind velocities ranging from 5 to 30 mph.

Figure 2 illustrates the wind chill temperature as a function of wind velocity predicted by the
model and by Eq. (9) for a day when Tair = 0°F.

Figure 2: Wind chill temperature as a function of the wind velocity predicted by the model
and by the National Weather Service equation on a day when Tair = 0°F.
Problem 4.9-5 (4-18): Soldering Iron
A soldering iron tip can be approximated as a cylinder of metal with radius rout = 5.0 mm and
length L = 20 mm. The metal is carbon steel; assume that the steel has constant density ρ = 7854
kg/m3 and constant conductivity k = 50.5 W/m-K, but a specific heat capacity that varies with
temperature according to:

⎡ J ⎤ ⎡ J ⎤ ⎡ J ⎤ 2
c = 374.9 ⎢ ⎥ + 0.0992 ⎢ 2⎥
T + 3.596x10-4 ⎢ 3⎥
T
⎣ kg-K ⎦ ⎣ kg-K ⎦ ⎣ kg-K ⎦

The surface of the iron radiates and convects to surroundings that have temperature Tamb = 20°C.
Radiation and convection occur from the sides of the cylinder (the top and bottom are insulated).
The soldering iron is exposed to an air flow (across the cylinder) with a velocity V = 3.5 m/s at
Tamb and Pamb = 1 atm. The surface of the iron has an emissivity ε = 1.0. The iron is heated
electrically by ohmic dissipation; the rate at which electrical energy is added to the iron is g = 35
W.
a.) Assume that the soldering iron tip can be treated as a lumped capacitance. Develop a
numerical model using the Euler technique that can predict the temperature of the soldering
iron as a function of time after it is activated. Assume that it is activated when the tip is at
ambient temperature. Be sure to account for the fact that the heat transfer coefficient, the
radiation resistance, and the heat capacity of the soldering iron tip are all a function of the
temperature of the tip.

The inputs are entered in EES:

$UnitSystem SI MASS RAD PA K J


$TABSTOPS 0.2 0.4 0.6 0.8 3.5 in

"Inputs"
L=20 [mm]*convert(mm,m) "length"
r_out=5 [mm]*convert(mm,m) "radius"
rho=7854 [kg/m^3] "density"
k=50.5 [W/m-K] "conductivity"
e=1.0 [-] "emissivity"
g_dot=35 [W] "generation"
T_amb=converttemp(C,K,20) "ambient temperature"
P_amb=1 [atm]*convert(atm,Pa) "ambient pressure"
V=3.5 [m/s] "air velocity"

The volume and surface area (for convection and radiation) of the cylinder are:

V = π rout
2
L (1)

As = 2π rout L (2)

A_s=2*pi*r_out*L
"surface area for convection and radiation"
Vol=pi*r_out^2*L "volume"

The total simulation time, τsim, is broken into time steps of equal duration:

τ sim
Δt = (3)
( M − 1)

t j = Δt ( j − 1) for j = 1..M (4)

where M is the number of time steps used for the simulation:

tau_sim=500 [s] "simulation time"


M=101 [-] "number of time steps"
Delta_time=tau_sim/(M-1) "time step duration"
duplicate j=1,M
time[j]=Delta_time*(j-1)
end

We will focus on moving forward one step in time (from j = 1 to j = 2); once that is complete it is
easy to move through all of the time steps. The initial temperature is ambient:

T1 = Tamb (5)

T[1]=T_amb "initial temperature"

The governing differential equation for the soldering iron is obtained from an energy balance:

+ h As (T − Tamb ) + ε σ As (T 4 − Tamb )
dT
g ss = V ρ c (T ) 4
(6)
dt

and so the rate of temperature change is:

⎡ g ss − h As (T − Tamb ) − ε σ As (T 4 − Tamb )⎤⎦


dT 1
= ⎣
4
(7)
dt V ρ c (T )

In order to compute the time rate of change it is necessary to determine h , the heat transfer
coefficient. The Euler technique will compute each quantity in Eq. (7) using the value of the
temperature at the beginning of the time step (T1 for the 1st time step). The film temperature at
T1 is used to compute the properties of air that are required (ρa,1, μa,1, ka,1, and Pra,1).

Tamb + T1
T film = (8)
2

T_film[1]=(T[1]+T_amb)/2 "film temperature"


rho_a[1]=density(Air,T=T_film[1],P=P_amb) "density"
mu_a[1]=viscosity(Air,T=T_film[1]) "viscosity"
k_a[1]=conductivity(Air,T=T_film[1]) "conductivity"
Pr_a[1]=Prandtl(Air,T=T_film[1]) "Prandtl number"

The Reynolds number is computed:

V ρ a ,1 2 rout
Re1 = (9)
μa ,1

and used to call the correlation for flow over a cylinder which returns the average Nusselt
number ( Nu1 ) and drag coefficient (Cd,1):

Re[1]=rho_a[1]*V*2*r_out/mu_a[1] "Reynolds number"


Call External_Flow_Cylinder_ND(Re[1],Pr_a[1]:Nusselt[1],C_d[1]) "Access correlations"

The average heat transfer coefficient is computed using the average Nusselt number:

Nu1 ka ,1
h1 = (10)
2 rout

h[1]=Nusselt[1]*k_a[1]/(2*r_out) "heat transfer coefficient"

The specific heat capacity is computed at T1:

⎡ J ⎤ ⎡ J ⎤ ⎡ J ⎤ 2
c1 = 374.9 ⎢ ⎥ + 0.0992 ⎢ T + 3.596x10-4 ⎢
2⎥ 1 3⎥ 1
T (11)
⎣ kg-K ⎦ ⎣ kg-K ⎦ ⎣ kg-K ⎦

c[1]=374.9 [J/kg-K]+0.0992 [J/kg-K^2]*T[1]+3.596e-4 [J/kg-K^3]*T[1]^2


"specific heat capacity"

The rate of temperature change is computed at the beginning of the 1st time step, Eq. (7), and
used to compute the temperature at the end of the time step:

dT
T2 = T1 + Δt (12)
dt T1 ,t1

dTdt[1]=(g_dot-h[1]*A_s*(T[1]-T_amb)-e*sigma#*A_s*(T[1]^4-T_amb^4))/(Vol*rho*c[1])
T[2]=T[1]+dTdt[1]*Delta_time

The process of moving through all of the time steps is automated by placing the previous set of
equations within a duplicate loop and changing the index 1 to j and 2 to j+1:

"take all Euler steps"


duplicate j=1,(M-1)
T_film[j]=(T[j]+T_amb)/2 "film temperature"
rho_a[j]=density(Air,T=T_film[j],P=P_amb) "density"
mu_a[j]=viscosity(Air,T=T_film[j]) "viscosity"
k_a[j]=conductivity(Air,T=T_film[j]) "conductivity"
Pr_a[j]=Prandtl(Air,T=T_film[j]) "Prandtl number"
Re[j]=rho_a[j]*V*2*r_out/mu_a[j] "Reynolds number"
Call External_Flow_Cylinder_ND(Re[j],Pr_a[j]:Nusselt[j],C_d[j]) "Access correlations"
h[j]=Nusselt[j]*k_a[j]/(2*r_out) "heat transfer coefficient"
c[j]=374.9 [J/kg-K]+0.0992 [J/kg-K^2]*T[j]+3.596e-4 [J/kg-K^3]*T[j]^2
"specific heat capacity"
dTdt[j]=(g_dot-h[j]*A_s*(T[j]-T_amb)-e*sigma#*A_s*(T[j]^4-T_amb^4))/(Vol*rho*c[j])
T[j+1]=T[j]+dTdt[j]*Delta_time
end

b.) Plot the temperature of the soldering iron as a function of time. Make sure that your plot
covers sufficient time that your soldering iron has reached steady state.

The temperature as a function of time is shown in Figure 2.

Figure 2: Temperature as a function of time

c.) Verify that the soldering iron tip can be treated as a lumped capacitance.

The lumped capacitance model neglects the internal resistance to conduction and considers the
external resistances to radiation and convection; the assumption is therefore justified using a Biot
number:

Rcond
Bi = (13)
Rext

where
−1
⎛ 1 1 ⎞
Rext = ⎜ + ⎟ (14)
⎝ Rconv Rrad ⎠
These resistances vary throughout the process; therefore, we should compute the maximum value
of the Biot number which occurs at the end of the process when temperature is largest and
therefore both Rconv and Rrad are smallest. The minimum value of the convection resistance is
computed:

1
Rconv = (15)
MAX ( h ) As

h_max=max(h[1..M-1]) "maximum heat transfer coefficient"


R_conv_min=1/(A_s*h_max) "minimum convection resistance"

The resistance to radiation is evaluated at the maximum temperature:

1
Rrad = (16)
As ε σ ⎡ MAX (T ) + Tamb
2 ⎤
⎡ MAX (T ) + Tamb ⎤⎦
2
⎣ ⎦⎣

T_max=max(T[1..M]) "maximum temperature"


R_rad_min=1/(A_s*e*sigma#*(T_max^2+T_amb^2)*(T_max+T_amb))
"minimum value of the radiation resistance"

The total external resistance is the parallel combination of the radiation and convection
resistances, according to Eq. (14).

R_ext_min=(1/R_conv_min+1/R_rad_min)^(-1) "total external resistance"

The characteristic length for conduction is:

V
Lchar = (17)
As

and the resistance to conduction is:

Lchar
Rcond = (18)
k As

L_char=Vol/A_s "characteristic length for conduction"


R_cond=L_char/(A_s*k) "resistance to internal conduction"

The maximum value of the Biot number is therefore given by Eq. (13):

Biot_max=R_cond/R_ext_min "maximum value of the Biot number"

which leads to Bi = 0.005; therefore, the lumped capacitance model is justified.


Problem 4.9-6
Air at u∞ = 1.8 m/s and T∞ = 450 K flows over a flat plate in a wind tunnel. The plate is L = 1.6
m in length in the flow direction and 1 m wide. Measurements are made of the air temperature at
various positions above the plate (various values of y) using a small thermocouple that is
designed to minimize the disturbance to the flow. The measurements, made a position x = 0.7 m
from the leading edge of the plate (as shown in Figure P4.9-6), are summarized in the table
below.

y (m) T (K)
air
0 350
u∞ = 1.8 m/s y point of
0.0012 362
T∞ = 450 K measurement
x 0.0045 400
0.008 425
x = 0.7 m 0.012 440
0.015 447
0.020 450
L = 1.6 m
(a) (b)
Figure 4.9-6 Schematic of experiment and experimental measurements

a) Estimate the thermal boundary layer thickness from the experimental data.

The inputs are entered in EES:

$UnitSystem SI MASS RAD PA K J


$TABSTOPS 0.2 0.4 0.6 0.8 3.5 in

T_s=350 [K] "surface temperature"


T_inf=450 [K] "fluid temperature"
P=1 [atm]*convert(atm,Pa) "atmospheric pressure"
u_inf=1.8 [m/s] "free stream velocity"
x=0.7 [m] "location of measurement"

The thermal boundary layer is defined as the location where:

(T − Ts ) = 0.99 (1)
(T∞ − Ts )
(T_s-T)/(T_s-T_inf)=0.99 "temperature at the edge of the boundary layer"

which leads to T = 449 K. The measured boundary layer thickness (δt,m) is estimated by
interpolation of the experimental data:

delta_t_m=0.015 [m]+(T-447 [K])*0.005 [m]/(450 [K]-447 [K])


"interpolation of data to get boundary layer thickness"

which leads to δt,m = 0.0183 m.

b) What is the thermal boundary layer thickness according to accepted correlations?


The properties of air (ρ, Pr, μ, and k) are estimated at the film temperature:

T film =
(Ts + T∞ ) (2)
2

T_film=(T_s+T_inf)/2 "film temperature"


rho=density(Air,T=T_film,P=P) "density"
Pr=Prandtl(Air,T=T_film) "Prandtl number"
mu=viscosity(Air,T=T_film) "viscosity"
k=conductivity(Air,T=T_film) "conductivity"

The Reynolds number at x is computed:

ρ x u∞
Rex = (3)
μ

Re_x=rho*u_inf*x/mu "Reynolds number"

which leads to Rex = 4.9x104; therefore, the formula for the thermal boundary layer thickness for
a laminar flow is used:

4.916 x − 13
δ t ,lam = Pr (4)
Rex

delta_t/x=4.916/(Re_x^0.5*Pr^(1/3)) "thermal boundary layer thickness"

which leads to δt,lam = 0.0175 m.

c) Estimate the local heat transfer coefficient at x = 0.7 m using the experimental data.

The heat flux at the surface of the plate is estimated according to:

⎛ ∂T ⎞
qs′′ = −k ⎜ ⎟ (5)
⎝ ∂y ⎠ y =0

The temperature gradient at the surface of the plate is estimated using the two data points closest
to the plate. The heat transfer coefficient is defined according to:

qs′′ = h (Ts − T∞ ) (6)

dT\dy=(362 [K]-350 [K])/0.0012 [m] "temperature gradient at y=0"


q``_dot=-k*dT\dy "heat flux at surface"
q``_dot=h*(T_s-T_inf) "definition of the heat transfer coefficient"
which leads to h = 3.283 W/m2-K.

d) What it the local heat transfer coefficient according to accepted correlations?

A correlation for the local Nusselt number for laminar flow over a smooth flat plate is provided
in Section 4.9.2:
1 1
0.3387 Rex 2 Pr 3
Nu x = 1
(7)
⎡ ⎛ 0.0468 ⎞ 2 3 ⎤ 4

⎢1 + ⎜ ⎟ ⎥
⎢⎣ ⎝ Pr ⎠ ⎥⎦

The local heat transfer coefficient is obtained according to:

Nu x k
h= (8)
x

Nusselt_x=(0.3387*Re_x^0.5*Pr^(1/3))/(1+(0.04568/Pr)^(2/3))^(1/4) "correlation for local Nusselt number"


h_x=Nusselt_x*k/x "heat transfer coefficient based on correlation"

which leads to h = 3.004 W/m2-K.


Problem 4.9-7 (4-19 in text)
Molten metal droplets must be injected into a plasma for an extreme ultraviolet radiation source,
as shown in Figure P4.9-7.

ρ = 7054 kg/m3
c = 307 J/kg-K
D = 200 μm
Tini = 800 K
uinject = 5 m/s
atmospheric air at T∞ = 20°C
Figure P4.9-7: Injection of molten metal droplets.

The fuel droplets have a diameter of D = 200 μm and are injected at a velocity uinject = 5 m/s with
temperature Tini = 800 K. The density of the droplet ρ = 7054 kg/m3 and the specific heat
capacity is c = 307 J/kg-K. You may assume that the droplet can be treated as a lumped
capacitance. The droplet is exposed to still air at T∞ =20ºC.
a.) Develop a numerical model in EES using the Integral command that can keep track of the
velocity, temperature, and position of the droplet as a function of time.

The inputs are entered in EES:

$UnitSystem SI MASS RAD PA K J


$TABSTOPS 0.2 0.4 0.6 0.8 3.5 in

"Inputs"
D=200 [micron]*convert(micron,m) "diameter of fuel"
T_ini=800 [K] "initial temperature of fuel droplet"
c=307 [J/kg-K] "specific heat capacity of fuel"
rho=7054 [kg/m^3] "density of fuel"
T_infinity=converttemp(C,K,20 [C]) "ambient air temperature"
u_inject=5 [m/s] "injection velocity"

For this problem, the state variables include the distance of the droplet from the nozzle, x, the
velocity of the droplet, u, and T, the temperature of the droplet. Therefore, the state equations
will provide the time rate of change of these variables. In order to develop the program, the
values of the state variables are initially set to arbitrary values:

x=0 [m] "arbitary position to evaluate state equation"


T=T_ini "arbitrary temperature to evaluate state equation"
u=u_inject "arbitrary velocity to evaluate state equation"

The rate of change of the position is the velocity:

dx
=u (1)
dt

dxdt=u "derivative of position"


The state equation for temperature is obtained by carrying out an energy balance on the droplet:

dT
ρV c = h As (T∞ − T ) (2)
dt

where As and V are the surface area and volume of the droplet:

2
⎛D⎞
As = 4 π ⎜ ⎟ (3)
⎝2⎠

3
4 ⎛D⎞
V = π⎜ ⎟ (4)
3 ⎝2⎠

A_s=4*pi*(D/2)^2 "surface area"


V=4*pi*(D/2)^3/3 "volume"

The average heat transfer coefficient is obtained using the correlations that are programmed in
EES. The properties of air (ρa, μa, ca, ka, Pra) are evaluated using the built in property routines
in EES:

T_film=(T+T_infinity)/2 "film temperature for fluid property evaluation"


rho_a=density(Air,T=T_film,p=1 [atm]*convert(atm,Pa)) "density"
mu_a=viscosity(Air,T=T_film) "viscosity"
c_a=cP(Air,T=T_film) "specific heat capacity"
k_a=conductivity(Air,T=T_film) "conductivity"
Pr=mu_a*c_a/k_a "Prandtl number"

The Reynolds number is calculated:

ρa u D
ReD = (5)
μa

and the External_Flow_Sphere_ND function in EES is used to obtain the average Nusselt number
( Nu ) and the drag coefficient (Cd). The heat transfer coefficient is calculated according to:

k
h= Nu (6)
D

and the time rate of change of the temperature is computed using Eq. (2):

Re=rho_a*D*u_inject/mu_a "Reynolds number"


Call External_Flow_Sphere_ND(Re,Pr: Nusselt,C_d) "access correlations"
h=Nusselt*k_a/D "heat transfer coefficient"
rho*c*V*dTdt=h*A_s*(T_infinity-T) "derivative of temperature"
A momentum balance on the droplet leads to the state equation for velocity:

du
ρV = ρ V g − Fd (7)
dt

where g is the acceleration of gravity and Fd is the drag force. The drag force is calculated using
the drag coefficient according to:

1
Fd = Cd ρ a u 2 Ap (8)
2

where Ap is the projected area of the droplet:

2
⎛D⎞
Ap = π ⎜ ⎟ (9)
⎝2⎠

A_p=pi*(D/2)^2 "projected area"


F_d=A_p*rho_a*u^2*C_d/2 "drag force"
rho*V*dudt=rho*V*g#-F_d "derivative of velocity"

Once the state equations have been verified, the arbitrary values of the state variables are
commented out and the Integral function in EES is used to carry out the numerical integrations:
t
du
u = uinject + ∫ dt (10)
0
dt

t
dT
T = Tini + ∫ dt (11)
0
dt

t
dx
x=∫ dt (12)
0
dt

{x=0 [m] "arbitary position to evaluate state equation"


T=T_ini "arbitrary temperature to evaluate state equation"
u=u_inject "arbitrary velocity to evaluate state equation"}
t_sim=2 [s] "simulation time"
u=u_inject+Integral(dudt,time,0,t_sim)
T=T_ini+Integral(dTdt,time,0,t_sim)
x=Integral(dxdt,time,0,t_sim)

$IntegralTable time,x,u,T

b.) Plot the velocity as a function of time and the temperature as a function of time.

Figure 2 illustrates the velocity and temperature of the droplet as a function of time.
5 800

4.8
700
4.6

Droplet velocity (m/s)

Droplet Temperature (K)


600
4.4

4.2 500

4
400
T
3.8
300
3.6
u
3.4 200
0 0.1 0.2 0.3 0.4 0.5 0.6 0.7 0.8 0.9 1
Time (s)
Figure 2: Velocity and temperature as a function of time.

c.) Plot the temperature as a function of position. If the temperature of the droplet must be
greater than 500 K when it reaches the plasma then what is the maximum distance that can
separate the plasma from the injector?

Figure 3 illustrates the temperature as a function of position (distance from the injector). Figure
3 shows that the plasma and the injector cannot be separated by more than about 0.35 m.
800

700
Temperature (K)

600

500

400

300

200
0 0.1 0.2 0.3 0.4 0.5 0.6 0.7 0.8 0.9 1
Distance from injector (m)
Figure 3: Temperature of the droplet as a function of position.
Problem 4.9-8
Figure 4.9-8 illustrates a series of plates inserted into a stream of water in order to provide
cooling to a power electronics system.

L = 0.2 m

s
water
u∞ = 40 m/s
Ts = 50°C
T∞ = 20°C
p = 1 atm

Figure P4.9-8: Plates in a stream of water.

The plates are L = 0.2 m long and W = 0.05 m wide (into the page). The plates are spaced far
enough apart that the boundary layers from adjacent plates do not meet, therefore, the flow over
the plates can be treated as an external flow rather than an internal flow. The free stream
velocity is u∞ = 40 m/s, the free stream temperature and pressure are T∞ = 20°C and p = 1 atm,
respectively. The surface of the plates are maintained at Ts = 50°C. Assume that the plates are
smooth.
a.) What is the total rate of heat transfer from each plate.

The inputs are entered in EES:

$UnitSystem SI MASS RAD PA K J


$TABSTOPS 0.2 0.4 0.6 0.8 3.5 in

"Inputs"
F$='water'
u_infinity=40 [m/s] "free stream velocity"
T_infinity=converttemp(C,K,20[C]) "free stream temperature"
p=1 [atm]*convert(atm,Pa) "pressure"
L=0.2 [m] "length"
W=0.05 [m] "width"
T_s=converttemp(C,K,50 [C]) "surface temperature"

The fluid properties (μ, k, ρ, c, ν, α, and Pr) are evaluated at the film temperature:

T film =
(Ts + T∞ ) (1)
2

using EES internal property routines:

T_film=(T_infinity+T_s)/2 "film temperature"


mu=viscosity(F$,T=T_film,P=p) "viscosity"
k=conductivity(F$,T=T_film,P=p) "conductivity"
rho=density(F$,T=T_film,P=p) "density"
c=cP(F$,T=T_film,P=p) "specific heat capacity"
nu=mu/rho "kinematic viscosity"
alpha=k/(rho*c) "thermal diffusivity"
Pr=nu/alpha "Prandtl number"

The Reynolds number is computed:

ρ u∞ L
Re = (2)
μ

The correlations for flow over a smooth flat plate are accessed using the EES procedure
External_Flow_Plate and returns the average Nusselt number ( Nu ) and average friction factor
( C f ). The average Nusselt number is used to compute the average heat transfer coefficient
according to:

Nu k
h= (3)
L

The total rate of heat transfer from a single plate is:

q = 2 h W L (Ts − T∞ ) (4)

Re=L*rho*u_infinity/mu "Reynolds number"


Call External_Flow_Plate_ND(Re,Pr: Nusselt,C_f) "access correlations"
h_bar=Nusselt*k/L "average heat transfer coefficient"
q_dot=h_bar*2*W*L*(T_s-T_infinity) "heat transfer rate per plate"

which leads to q = 47.0 kW.

b.) Estimate the minimum spacing (s) that can be used before the boundary layers from adjacent
plates meet and the problem becomes an internal flow rather than an external flow problem?

The Reynolds number is Re = 1.11x107 and therefore the flow at the trailing edge of the plate is
turbulent. The turbulent boundary layer thickness at the trailing edge is given approximately by:

0.16 x
δ turb ≈ 1
(5)
Re 7

The minimum plate spacing is twice the boundary layer thickness:

smin = 2 δ turb (6)

delta_turb=0.16*L/Re^(1/7) "turbulent boundary layer thickness"


s_min=2*delta_turb "minimum spacing"

which leads to smin = 0.63 cm.


c.) Surface roughness begins to affect a turbulent flow when the size of the roughness elements
(e) are comparable to the size of the viscous sublayer. Estimate the maximum size of the
roughness elements that can be present on the plate before the plate roughness will begin to
affect your answer from (a).

The viscous sublayer thickness is approximately:

34.9 L
δ vs ≈ (7)
Re0.9

delta_vs=34.9*L/Re^0.9 "viscous sublayer thickness"


e_max=delta_vs "maximum roughness"

which leads to emax = 3.2 μm.


Problem 4.9-9 (4-20 in text)
Figure 4-34 in your text illustrates the drag coefficient for a cylinder as a function of
Reynolds number.
a.) Using Figure 4-34 of your text, discuss briefly (1-2 sentences) why it might make
sense to add dimples to a baseball bat.

100

turbulent boundary layer


-0.5
Cd ≈Re
laminar boundary layer

that separates
that separates
10
Drag coefficient

-1
Cd ≈Re

laminar flow
1
without
separation

0.1
10-1 10 0 101 102 103 104 105 10 6
Reynolds number
Figure 4-34: Drag coefficient for a cylinder as a function of Reynolds number.

The drastic dip in the drag coefficient at very high Reynolds numbers occurs because the
boundary layer that forms on the front face of the cylinder becomes turbulent and
therefore separation is delayed. If you dimple a bat it may be possible to promote
turbulence at lower Reynolds number and therefore reduce the drag experienced by the
batter (i.e., you could move the dip to the left).

b.) Using Figure 4-34 of your text, estimate how fast you would have to be able to swing
a bat in order for it to make sense to think about adding dimples (the estimate can be
rough but should be explained well). Assume that a bat has diameter D = 0.04 m and
air has properties ρ = 1 kg/m3 and μ = 0.00002 Pa-s.

It is not likely that the transition to turbulence can be affected by more than a factor of 2
or so. Therefore, you would need to be able to swing a bat fast enough that the Reynolds
number approached Re = 100000:

ρ air V D
Re = (1)
μair

which leads to V = 50 m/s.


Problem 4.1-2 (4-1 in text): Water Flowing over a Plate
Water at atmospheric pressure, free stream velocity u∞ = 1.0 m/s and temperature T∞ = 25°C
flows over a flat plate with a surface temperature Ts = 90°C. The plate is L = 0.15 m long.
Assume that the flow is laminar over the entire length of the plate.
a.) Estimate, using your knowledge of how boundary layers grow, the size of the momentum and
thermal boundary layers at the trailing edge of the plate (i.e., at x = L). Do not use a
correlation from your book, instead use the approximate model for boundary layer growth.

The inputs are entered in EES:

$UnitSystem SI MASS RAD PA K J


$TABSTOPS 0.2 0.4 0.6 0.8 3.5 in

T_s=converttemp(C,K,90) "surface temperature"


T_inf=converttemp(C,K,25) "free stream temperature"
P=1 [atm]*convert(atm,Pa) "pressure"
L=0.15 [m] "plate length"
u_inf=1.0 [m/s] "free stream velocity"

The film temperature:

Ts + T∞
T film = (1)
2

is used to evaluate the properties of water (k, μ, ρ, and c) using EES’ built-in property functions:

T_film=(T_s+T_inf)/2 "film temperature"


k=conductivity(Water,T=T_film,P=P) "conductivity"
mu=viscosity(Water,T=T_film,P=P) "viscosity"
rho=density(Water,T=T_film,P=P) "density"
c=cP(Water,T=T_film,P=P) "specific heat capacity"

The kinematic viscosity and thermal diffusivity are computed:

μ
ν= (2)
ρ

k
α= (3)
ρc

nu=mu/rho "kinematic viscosity"


alpha=k/(rho*c) "thermal diffusivity"

The time that the free stream is in contact with the plate is computed:

L
t= (4)
u∞
and used to compute the approximate momentum and thermal boundary layer thicknesses at the
plate’s trailing edge:

δm ≈ 2 ν t (5)

δt ≈ 2 α t (6)

time=L/u_inf "time required to reach back of plate"


delta_m=2*sqrt(nu*time) “approximate size of momentum boundary layer"
delta_t=2*sqrt(alpha*time) "approximate size of thermal boundary layer"

which leads to δm = 0.54 mm and δt = 0.31 mm.

b.) Use your answer from (a) to estimate the shear stress at the trailing edge of the plate and the
heat transfer coefficient at the trailing edge of the plate.

The shear stress at the trailing edge of the plate can be approximated according to:

u∞
τs ≈ μ (7)
δm

and the heat transfer coefficient can be approximated according to:

k
h≈ (8)
δt
tau_app=mu*u_inf/delta_m "approximate value of the shear stress at the end of the plate"
h_app=k/delta_t "approximate value of the heat transfer coefficient at the end of the plate"

which leads to τs = 0.89 Pa and h = 2090 W/m2-K.

c.) You measure a shear stress of τs,meas = 1.0 Pa at the trailing edge of the plate; use the
Modified Reynolds Analogy to predict the heat transfer coefficient at this location.

The Modified Reynolds Analogy relates shear to heat transfer coefficient according to:
kτ 1
h ≈ s ,meas Pr 3 (9)
μ u∞
tau_meas=1.0 [Pa] "measured value of the shear stress"
Pr=nu/alpha "Prandtl number"
h_MRA=k*tau_meas*Pr^(1/3)/(mu*u_inf)
"heat transfer coefficient based on the Modified Reynolds Analogy"

which leads to h = 1940 W/m2-K.


Problem 4.1-6 (4-2 in text)
Figure P4.1-6 illustrates the flow of a fluid with T∞ = 0ºC, u∞ = 1 m/s over a flat plate.

fluid at T∞ = 0°C, u∞ = 1 m/s


Pr = 1, k = 1 W/m-K, α = 1x10 m /s
-3 2

y q ′′ = 1000 W/m
2

x
1 2 3 4
adiabatic constant heat flux adiabatic
L=1m L=1m L=1m
Figure P4.1-6: Flow over a flat plate.

The flat plate is made up of three sections, each with length L = 1 m. The first and last sections
are insulated and the middle section is exposed to a constant heat flux, q ′′ = 1000 W/m2. The
properties of the fluid are Prandtl number Pr = 1, conductivity k = 1 W/m-K, and thermal
diffusivity α = 1x10-3 m2/s. Assume that the flow is laminar over the entire surface.
a.) Sketch the momentum and thermal boundary layers as a function of position, x. Do not
worry about the qualitative characteristics of your sketch - get the quantitative characteristics
correct.

boundary layer thickness


δm

δt

adiabatic constant heat flux adiabatic


Figure P4.1-6(b): Sketch of the momentum and thermal boundary layer thickness as a function of position.

The momentum boundary layer develops from the front of the plate and grows according to the
square root of x:

x
δm ≈ 2 υ t = 2 υ (1)
u∞

The thermal boundary layer develops from the front of the heated portion of the plate and also
grows according to the square root of x (measured related to the leading edge of the heated
plate):

δt ≈ 2 α t = 2 α
( x − L) (2)
u∞
Because the Prandtl number is 1, the momentum and thermal boundary layers develop at the
same rate although they start at different positions. Note that the thermal boundary layer
continues to grow downstream of the heated plate because the energy added to the boundary
layer continues to diffuse into the free stream.

b.) Sketch the temperature distribution (the temperature as a function of distance from the plate
y) at the 4 locations indicated in Figure P4.1-6. Location 1 is half-way through the first
adiabatic region, Location 2 is half-way through the heated region, Location 3 is at the
trailing edge of the heated region (in the heated region), and Location 4 is at the trailing edge
of the final adiabatic region. Again, focus on getting as many of the qualitative
characteristics of your sketch correct as you can.

yT y T∞ yT yT
∞ ∞ ∞

δt

T T T T

1 2 3
3 3
4
adiabatic constant heat flux adiabatic
Figure P4.1-6(c): Sketch of the temperature distribution at the four locations shown in Figure P4.1-6(a).

The temperature distribution at location 1, in the initial unheated region, is uniform as there is
nothing to add or subtract energy from the flow. The temperature distribution at locations 2 and
3 are similar in shape. The temperature of the surface is elevated relative to the free stream due
to the resistance of the boundary layer. The temperature gradient at the surface is the same at
both locations 2 and 3 because the heat flux is the same. The surface temperature should be
higher at location 3 because the resistance of the boundary layer is larger. The temperature
distribution at location 4 should extend further out into the free stream due to the increased
thickness of the thermal boundary layer. The slope of the temperature gradient at the plate and
the outer edge of the boundary layer should be zero. By conservation of energy, the average
temperature of the boundary layer should drop at location 4 because the boundary layer has
grown and no additional energy has been added.

c.) Sketch the temperature of the surface of the plate as a function of position, x. Get the
qualitative features of your sketch correct.
surface temperature

T∞
x

adiabatic constant heat flux adiabatic


Figure P4.1-6(d): Sketch of the surface temperature of the plate as a function of position.

The surface temperature is equal to the free stream temperature in the initial unheated region (0 <
x < L). The surface temperature increases approximately proportionally to the thermal boundary
layer thickness according to:

Ts = T∞ +
q ′′δ t q ′′
= T∞ + 2 t α
( x − L) (3)
k k u∞

In the final unheated region, the surface temperature drops as the energy that was added in the
heated region diffuses into the free stream and the boundary layer grows. By conservation of
energy, the average temperature of the boundary layer should change according to:

u∞
q ′′ L ≈ ρ c δ t (T − T∞ ) (4)
2

which leads to:

2 q ′′ L q ′′ L u∞
T ≈ T∞ + = T∞ + (5)
u∞ ρ c δ t u∞ ρ c α ( x − L )

and therefore the surface temperature will drop like x-0.5.

d.) Predict, approximately, the temperature of the surface at locations 1, 2, 3, and 4 in Figure
P4.1-6. Do not use a correlation. Instead, use your conceptual understanding of how
boundary layers behave to come up with very approximate estimates of these temperatures.

The surface temperature at location 1 is Ts,1 = T∞ = 0ºC.

The thermal boundary layer thickness at locations 2, 3 and 4 can be predicted approximately
using Eq. (2):
0.5 L 1x10-3 m 2 0.5 (1 m ) s
δ t ,2 ≈ 2 α =2 = 0.0447 m (6)
u∞ s 1m

L 1x10-3 m 2 1 m s
δ t ,3 ≈ 2 α =2 = 0.0633 m (7)
u∞ s 1m

2L 1x10-3 m 2 2 (1 m ) s
δ t ,4 ≈ 2 α =2 = 0.0894 m (8)
u∞ s 1m

The temperature of the surface at locations 2 and 3 can be predicted approximately using Eq. (3):

1000 W 0.0447 m m-K


Ts ,2 = 0°C + = 44.72°C (9)
m2 1W

1000 W 0.0633 m m-K


Ts ,3 = 0°C + = 63.25°C (10)
m2 1W

The temperature of the surface at location 4 can be predicted approximately using Eq. (5),
substituting ρ c = k/α:

2 q ′′ L α 1000 W 1 m s 1x10-3 m 2 m-K


Ts ,4 ≈ T∞ + = 0°C + 2 = 22.36°C (11)
u∞ k δ t ,4 m2 1m s 1 W 0.0894 m
Problem 4.3-2 (4-3 in text)
You have fabricated a 1000x scale model of a microscale feature that is to be used in a
microchip. The device itself is only 1 μm in size and is therefore too small to test accurately.
However, you’d like to know the heat transfer coefficient between the device and an air flow that
has a velocity of 10 m/s.
a.) What velocity should you use for the test and how will the measured heat transfer coefficient
be related to the actual one?

In order for the results of the test to be applicable, the test and the actual device must be at the
same Reynolds number and Prandtl number. Therefore:

uLρ us Ls ρ
Re = = (1)
μ μ

where u and L are the velocity and length scale of the actual device and us and Ls are the velocity
and length scale of the test device. Solving Eq. (1) for the velocity to use in the text leads to:

L 1 m
us = u= 10 = 0.01 (2)
Ls 1000 s

If you use air for both tests then the Prandtl number will also be matched. Once you’ve matched
the Reynolds number and Prandtl number, the Nusselt number that you measure should be the
same as the Nusselt number for the actual device. Therefore:

h L hs Ls
Nu = = (3)
k k

where h and hs are the full-scale and test heat transfer coefficients, respectively. Solving Eq. (3)
for the full-scale heat transfer coefficient:

Ls
h = hs = 1000 hs (4)
L
Problem 4.3-3 (4-4 in text)
Your company has come up with a randomly packed fibrous material that could be used as a
regenerator packing. Currently there are no correlations available that would allow the
prediction of the heat transfer coefficient for the packing. Therefore, you have carried out a
series of tests to measure the heat transfer coefficient. A Dbed = 2 cm diameter bed is filled with
these fibers with diameter dfiber = 200 μm. The nominal temperature and pressure of the testing
is Tnom = 20ºC and pnom = 1 atm, respectively. The mass flow rate of the test fluid, m , is varied
and the heat transfer coefficient is measured. Several fluids, including air, water, and ethanol,
are used for testing. The data are shown in Table 4.3-3; the data can be downloaded from the
website as EES lookup tables (P4p3-3_air.lkt, P4p3-3_ethanol.lkt, and P4p3-3_water.lkt).

Table 4.3-3: Heat transfer data.


Air Water Ethanol
Mass flow Heat transfer Mass flow Heat transfer Mass flow Heat transfer
rate (kg/s) coefficient rate (kg/s) coefficient rate (kg/s) coefficient
(W/m2-K) (W/m2-K) (W/m2-K)
0.0001454 170.7 0.00787 8464 0.009124 4162
0.0004073 311.9 0.02204 15470 0.02555 7607
0.0006691 413.7 0.0362 20515 0.04197 10088
0.0009309 491.8 0.05037 24391 0.05839 11993
0.001193 572.7 0.06454 28399 0.07481 13964
0.001454 631.1 0.0787 31296 0.09124 15388

a.) Plot the heat transfer coefficient as a function of mass flow rate for the three different
different test fluids.

The data is loaded in EES as three lookup tables and used to prepare Figure P4.3-3(a).
35000
water
Heat transfer coefficient (W/m -K)

10000
2

ethanol

1000
air

100

10
0.0001 0.001 0.01 0.08
Mass flow rate (kg/s)
Figure P4.3-3(a): Heat transfer coefficient as a function of the mass flow rate.

b.) Plot the Nusselt number as a function of the Reynolds number for the three different test
fluids. Use the fiber diameter as the characteristic length and the free-flow velocity (i.e., the
velocity in the bed if it were empty) as the characteristic velocity.

The inputs are entered:


$UnitSystem SI MASS RAD PA K J
$Tabstops 0.2 0.4 0.6 0.8 3.5

d_fiber=200 [micron]*convert(micron,m) "Fiber diameter"


D_bed=2 [cm]*convert(cm,m) "Bed diameter"
T_nom=converttemp(C,K,20[C]) "Test temperature"
P_nom=1 [atm]*convert(atm,Pa) "Test pressure"

The data from the lookup tables is processed row by row. The mass flow rate and heat transfer
coefficient are read from the first row of the table and the appropriate fluid is set:

i=1 [-] "Run number"


m_dot=Lookup('Ethanol',i,1) "Mass flow rate"
h=Lookup('Ethanol',i,2) "Mass flow rate"
F$='Ethanol' "Test fluid"

The fluid properties (k, μ, Pr, and ρ) are computed using the internal property routines:

rho=density(F$,T=T_nom,P=P_nom) "density"
k=conductivity(F$,T=T_nom,P=P_nom) "conductivity"
mu=viscosity(F$,T=T_nom,P=P_nom) "viscosity"
Pr=Prandtl(F$,T=T_nom,P=P_nom) "Prandtl number"

The characteristic velocity is calculated according to:

4 m
uchar = (1)
π Dbed
2
ρ

The Reynolds number and Nusselt number are computed according to:

uchar d fiber ρ
Re = (2)
μ

d fiber h
Nu = (3)
k

u_char=m_dot/(rho*pi*D_bed^2/4) "characteristic velocity"


Re=u_char*d_fiber*rho/mu "Reynolds number"
Nusselt=h*d_fiber/k "Nusselt number"

Three parametric tables are set up (one for each fluid); the parametric table for air is shown in
Figure P4.3-3(b). The first column is the row and includes the index i which is varied from 1 to
6 (one for each row of the corresponding lookup table).
Figure P4.3-3(b): Parametric table for the air data reduction.

Figure P4.3-3(c) illustrates the Nusselt number as a function of Reynolds number for various
fluids (i.e., for various values of the Prandtl number).
15

12.5
Nusselt number

ethanol
10
water
7.5

5
air

2.5

0
0 10 20 30 40 50
Reynolds number
Figure P4.3-3(c): Nusselt number as a function of Reynolds number for various fluids.

c.) Correlate the data for all of the fluids using a function of the form: Nu = a Reb Pr c . Note
that you will want to transform the results using a natural logarithm and use the Linear
Regression option from the Tables menu to determine a, b, and c.

In order to use linear regression it is necessary to have a linear equation. Therefore, the proposed
equation is transformed by taking the natural logarithm of both sides:

ln ( Nu ) = ln ( a ) + b ln ( Re ) + c ln ( Pr ) (4)

All of the Nusselt number, Reynolds number, and Prandtl number data are put into one table;
three additional columns are added that contain the natural logarithm of these quantities (note
that this can be accomplished by selecting Alter Values and Enter Equation), as shown in FIgure
P4.3-3(d).
Figure P4.3-3(d): Nusselt number as a function of Reynolds number for various fluids.

The linear regression option is selected from the Tables menu. The dependent variable is the
natural log of the Nusselt number while the independent variables are the natural logarithm of
the Reynolds number and Prandtl number. A first order polynomial is selected, as indicated by
Eq. (4). The dialog should be as shown in Figure P4.3-3(e).

Figure P4.3-3(e): Linear Regression dialog.

The result is:

Nu = 0.6044 Re0.5692 Pr 0.3333 (5)

d.) Use your correlation to estimate the heat transfer coefficient for 20 kg/s of oil passing
through a 50 cm diameter bed composed of fibers with 2 mm diameter. The oil has density
875 kg/m3, viscosity 0.018 Pa-s, conductivity 0.14 W/m-K, and Prandtl number 20.

The inputs are entered in EES:


m_dot=20 [kg/s] "Mass flow rate"
d_fiber=2 [mm]*convert(mm,m) "Fiber diameter"
D_bed=50 [cm]*convert(cm,m) "Bed diameter"
rho=875 [kg/m^3] "density"
k=0.14 [W/m-K] "conductivity"
mu=0.018 [Pa-s] "viscosity"
Pr=20 "Prandtl number"

The characteristic velocity and Reynolds number are calculated:

4 m
uchar = (6)
π Dbed
2
ρ

uchar d fiber ρ
Re = (7)
μ

u_char=m_dot/(rho*pi*D_bed^2/4) "characteristic velocity"


Re=u_char*d_fiber*rho/mu "Reynolds number"

The correlation, Eq. (5), is used to obtain the Nusselt number. The heat transfer coefficient is
computed according to:

k
h = Nu (8)
d fiber

Nusselt=0.6044*Re^0.5692*Pr^0.333 "Nusselt number correlation"


h=Nusselt*k/d_fiber "heat transfer coefficient"

which leads to h = 456.5 W/m2-K.


Problem 4.3-6 (4-5 in text)
Your company makes an extrusion that can be used as a lightweight structural member; the
extrusion is long and thin and has an odd cross-sectional shape that is optimized for structural
performance. This product has been used primarily in the air-craft industry; however, your
company wants to use the extrusion in an application where it will experience cross-flow of
water rather than air. There is some concern that the drag force experienced by the extrusion will
be larger than it can handle. Because the cross-section of the extrusion is not simple (e.g.,
circular or square) you cannot go look up a correlation for the drag coefficient in the same way
that you could for a cylinder. However, because of the extensive use of the extrusion in the air-
craft industry you have an extensive amount of data relating the drag force on the extrusion to
velocity when it is exposed to a cross-flow of air. These data have been collated and are shown
graphically in Figure P4.3-6.
600
Drag force on 1 m of extrusion (N)

100

10

1
Point A, your boss' estimate of the drag force

0.1

0.01
0 20 40 60 80 100 120 140 160 180 200
Air velocity (m/s)
Figure P4.3-6: Drag force as a function of velocity for the extrusion when it is exposed to a cross-flow of air.

Your boss insists that the drag force for the extrusion exposed to water can be obtained by
looking at Figure P4.3-6 and picking off the data at the point where V = 10 m/s (Pt. A in Figure
P4.3-6); this corresponds to a drag force of about 1.7 N/m of extrusion.
a.) Is your boss correct? Explain why or why not.

Your boss is not correct. The drag force on an object is a function of many parameters including
its shape, size, and the cross-flow velocity (which are constant between Pt. A on Figure P4.3-6
and the proposed water operating condition) as well as the viscosity and density of the fluid
(which are not constant between Pt. A on Figure P4.3-6 and the proposed water operating
condition).

b.) If you think that your boss is not correct, then explain how you could use the data shown in
Figure P4.3-6 to estimate the drag force that will be experienced by the extrusion for a water
cross-flow velocity of 10 m/s.

The nondimensional form of the drag force is the drag coefficient, CD. The drag coefficient for a
specific shape should be only a function of the Reynolds number. Therefore, I would transform
the data from the given dimensional form (drag force vs velocity) to a more useful
nondimensional form (drag coefficient vs Reynolds number). I would calculate the Reynolds
number for the water operating condition and determine the drag coefficient that corresponds to
that Reynolds number; the drag coefficient could be used to compute the drag force.

c.) Use the data in Figure P4.3-6 to estimate the drag force that will be experienced by the
extrusion for a water cross-flow velocity of 10 m/s.

The inputs are entered in EES; both the air and water properties (ρair, μair, ρwater and μwater) are
calculated using the internal property routines in EES assuming room temperature and ambient
pressure.

$UnitSystem SI MASS RAD PA K J


$TABSTOPS 0.2 0.4 0.6 0.8 3.5 in

"Inputs"
T=converttemp(C,K,20 [C]) "room temperature"
P=1 [atm]*convert(atm,Pa) "atmospheric pressure"
V_water=10 [m/s] "frontal velocity"
rho_air=density(Air,T=T,P=P) "air density"
mu_air=viscosity(Air,T=T) "air viscosity"
rho_water=density(Water,T=T,P=P) "water density"
mu_water=viscosity(Water,T=T,P=P) "water viscosity"

It is necessary to identify the point in Figure P4.3-6 where the Reynolds number of the water
operating condition matches the Reynolds number of the air operating condition:

ρ air Lc Vair ρ water Lc Vwater


= (1)
μair μ water

where Lc is the characteristic dimension of the extrusion (which doesn't matter since it cancels
from both sides). Solving Eq. (1) for Vair leads to:

μair ρ water
Vair = V (2)
μ water ρ air water

V_air=mu_air*rho_water*V_water/(mu_water*rho_air) "air velocity that matches Reynolds number"

which leads to Vair = 151 m/s. The correct air operating condition to look at in Fig. 2 therefore
corresponds to the much larger drag force of 300 N/m. The drag coefficient corresponding to
this operating condition must be equivalent to the drag coefficient using water:

2 FD ,air 2 FD , water
= (3)
Ap ρ air V 2
air Ap ρ water Vwater
2

where Ap is the projected area of the extrusion (which again doesn't matter since it cancels).
Solving Eq. (3) for the drag force using water is:
ρ water Vwater
2
FD , water = FD ,air (4)
ρ air Vair2

F_D_air=300 [N/m] "drag force at V_air"


F_D_water=F_D_air*rho_water*V_water^2/(rho_air*V_air^2) "drag force for water"

which leads to FD,water = 1090 N/m of extrusion (this is about 3 orders of magnitude larger than
you boss' estimate).
Problem 4.4-1 (4-6 in text)
The momentum and thermal boundary layer can be substantially affected by either injecting or
removing fluid at the plate surface. For example, Figure P4.4-1 shows the surface of a turbine
blade exposed to the free stream flow of a hot combustion gas with velocity u∞ and temperature
T∞. The surface of the blade is protected by blowing gas through pores in the surface in a
process called transpiration cooling.

u∞ , T∞ y
transpiration flow
x

Ts turbine blade
Figure P4.4-1: Transpiration cooled turbine blade.

u∞ υ
The velocity of the injected gas is a function of x: v y =0 = C , where C is a dimensionless
x
constant and υ is the kinematic viscosity of the fluid. The gas is injected at the same temperature
as the surface of the plate, Ts. The Prandtl number of the combustion gas is Pr = 0.7.
a.) Develop a self-similar solution to the momentum equation for this problem using a Crank-
Nicolson numerical integration implemented in EES.

The development of the self-similar solution proceeds as presented in Section 4.4.2. The
transformed governing differential equation is:

d3 f d2 f
+ 2 f =0 (1)
dη 3 dη 2

Two of the boundary conditions for Eq. (1) due not change with transpiration. The x-velocity at
the plate is zero:

df
u y = 0 = u∞ =0 (2)
dη η =0

or

df
=0 (3)
dη η =0

The x-velocity far from the plate (outside of the boundary layer) is the free stream velocity:

df
u y →∞ = u∞ = u∞ (4)
dη η →∞
or

df
=1 (5)
dη η →∞

The boundary conditions for the y-velocity at the plate surface changes due to transpiration.

⎛ ⎞
1 u∞ υ ⎜ df ⎟ u υ
v y =0 = η − f ⎟ =C ∞
x ⎜⎜ N
(6)
2 dη ⎟ x
⎝ =0 ⎠η =0

or

fη =0 = −2 C (7)

The solution technique discussed in the text is adapted to this solution. The transpiration
constant and Prandtl number are specified:

$UnitSystem SI MASS RAD PA K J


$TabStops 0.2 3.5 in

C=0.1 [-] "constant related to transpiration"


Pr=0.7 [-] "Prandtl number"

df d2 f
For this problem, the state variables are f, , and . The rates of change of the first two
dη dη 2
state variables are obtained from the other state variables:

d df
[f ]= (8)
dη dη

d ⎡ df ⎤ d 2 f
dη ⎢ dη ⎥ = dη 2 (9)
⎣ ⎦

The rate of change of the final state variable is obtained from the governing ordinary differential
equation, Eq. (1):

d ⎡d2 f ⎤ d3 f d2 f
⎢ 2⎥ = = −2 f (10)
dη ⎣ dη ⎦ dη
3
dη 2

These state equations are integrated from η = 0 to η = η∞ where η∞ is selected so that we are
outside of both the thermal and velocity boundary layers:
eta_infinity=Max(10,10/sqrt(Pr)) "outer edge of computational domain"

The nodes for the integration are uniformly spaced according to:

ηi = η ∞
( i − 1) for i = 1...N (11)
( N − 1)
The distance between adjacent nodes is:

η∞
Δη = (12)
( N − 1)
where N is the number of nodes.

N=101 [-] "number of steps in the numerical integration"


DELTAeta=eta_infinity/(N-1) "size of the integration steps"
duplicate i=1,N
eta[i]=(i-1)*eta_infinity/(N-1) "position of integration steps"
end

The values of fη = 0 and dfdηη = 0 are specified according to Eqs. (3) and (7). An initial guess for
the 2nd deriviative of f with respect to η is specified:

f[1]=-C "f at eta = 0"


dfdeta[1]=0 [-] "dfdeta at eta = 0"
d2fdeta2[1]=0.3 [-] "d2fdeta2 at eta = 0, this is a guess"

The Crank-Nicolson technique is used to take the integration steps:

⎡ df df ⎤ Δη
fi +1 = f i + ⎢ + ⎥ for i = 1.. ( N − 1) (13)
⎣ dη i dη i +1 ⎦
2

df df ⎡d2 f d2 f ⎤ Δη
= +⎢ + ⎥ for i = 1.. ( N − 1) (14)
dη i +1
dη i ⎣⎢ dη 2 i dη 2 ⎥ 2
i +1 ⎦

d2 f d2 f ⎡ d2 f d2 f ⎤ Δη
= − ⎢ i
2 f + 2 f i +1 ⎥ for i = 1.. ( N − 1) (15)
dη 2 i +1
dη 2 i ⎢⎣ dη 2 i dη 2 ⎥
i +1 ⎦
2

"Crank-Nicolson integration"
duplicate i=1,(N-1)
f[i+1]=f[i]+(dfdeta[i]+dfdeta[i+1])*DELTAeta/2
dfdeta[i+1]=dfdeta[i]+(d2fdeta2[i]+d2fdeta2[i+1])*DELTAeta/2
d2fdeta2[i+1]=d2fdeta2[i]-(2*f[i]*d2fdeta2[i]+2*f[i+1]*d2fdeta2[i+1])*DELTAeta/2
end
The guess values are updated and the guess for the 2nd derivative of f with respect to η is
commented out. The final boundary condition, Eq. (5), is enforced:

{d2fdeta2[1]=0.3 [-]} "d2fdeta2 at eta = 0, this is a guess"


dfdeta[N]=1 [-] "enforce the free stream velocity"

b.) Plot the dimensionless velocity (u/u∞) as a function of the similarity parameter, η, for various
values of C.

The dimensionless velocity, dfdη, is shown in Figure 2 as a function of η for various values of C.
Note that increasing the amount of transpiration tends to thicken the boundary layer and reduce
the shear stress (the velocity gradient) at the wall.

C = 0 (no transpiration)
1

0.9
Dimensionless velocity, u/u

0.8
0.7
0.6 C = 0.1
C = 0.2
0.5
C = 0.3
0.4
C = 0.4
0.3 C = 0.5
0.2 C = 0.6

0.1
0
0 1 2 3 4 5 6 7
Similarity parameter, η
Figure 2: Dimensionless velocity as a function of the similarity parameter for various values of C.

c.) The boundary layer will "blow-off" of the plate at the point where the shear stress at the plate
surface becomes zero. What is the maximum value of C that can be tolerated before the
boundary layer becomes unstable?

d2 f
Figure 3 illustrates the dimensionless velocity gradient at the plate surface, , as a
dη 2 η =0

function of the transpiration constant, C. Note that the velocity gradient is proportional to the
shear stress and therefore the boundary layer will "blow-off" the plate when C is approximately
0.61.
0.7

Dimensionless velocity gradient at η = 0


0.6

0.5

0.4

0.3

0.2

0.1

0
0 0.1 0.2 0.3 0.4 0.5 0.6 0.7 0.8 0.9 1
Transpiration constant, C
Figure 3: Dimensionless velocity gradient at the plate surface as a function of C.

d.) Plot the ratio of the friction factor experienced by the plate with transpiration to the friction
factor experienced by a plate without transpiration as a function of the parameter C.

The shear stress is given by:

u∞ u∞ d 2 f
τs = μ (16)
2 υ x dη 2 η =0

Therefore, the ratio of the friction factor with transpiration to the friction factor without
transpiration is:

d2 f
Cf dη 2 η =0
= (17)
C f ,C = 0 d f
2

dη 2 η = 0,C = 0

d2 f
where is 0.664.
dη 2 η = 0,C = 0

Cfratio=d2fdeta2[1]/0.664 "ratio of the friction coefficient to the friction coefficient with C=0"

Figure 4 illustrates the friction factor ratio as a function of C.


1

Friction factor/friction factor with C = 0


0.9

0.8

0.7

0.6

0.5

0.4

0.3

0.2

0.1

0
0 0.1 0.2 0.3 0.4 0.5 0.6 0.7 0.8 0.9 1
Transpiration constant, C
Figure 4: Friction factor ratio as a function of C.

e.) Develop a self-similar solution to the thermal energy equation for this problem using a
Crank-Nicolson numerical integration implemented in EES.

The solution is identical to the one laid out in Section 4.4.3 of the text; the governing differential
equation and boundary conditions are unchanged.

theta[1]=0 [-] "theta at eta = 0"


{dthetadeta[1]=0.5 [-]} "dthetadeta at eta = 0, this is a guess"

"Crank-Nicolson integration"
duplicate i=1,(N-1)
theta[i+1]=theta[i]+(dthetadeta[i]+dthetadeta[i+1])*DELTAeta/2
dthetadeta[i+1]=dthetadeta[i]+(-2*f[i]*Pr*dthetadeta[i]-2*f[i+1]*Pr*dthetadeta[i+1])*DELTAeta/2
end
theta[N]=1

f.) Plot the dimensionless temperature difference, (T - Ts)/(T∞ - Ts), as a function of the
similarity parameter, η, for various values of C.

Figure 5 provides the requested plot. Notice that as the transpiration increases, the thermal
boundary layer thickens and the rate of heat transfer at the surface is reduced, as evidenced by
the reduced temperature gradient.
Dimensionless temperature difference
C = 0 (no transpiration)
1
0.9
0.8
0.7
0.6
0.5 C = 0.1
C = 0.2
0.4 C = 0.3
0.3 C = 0.4
C = 0.5
0.2
C = 0.6
0.1
0
0 1 2 3 4 5 6 7
Similarity parameter, η
Figure 5: Dimensionless temperature difference as a function of the similarity parameter for various values of
the transpiration constant, C.

g.) Plot the ratio of the Nusselt number experienced by the plate with transpiration to the Nusselt
number experienced by a plate without transpiration as a function of the parameter C.

The Nusselt number is given by:

1 u∞ x dθ
Nu x = (4-18)
2 υ dη η =0

Therefore the ratio of the Nusselt number to the Nusselt number with no transpiration is given
by:

dθ
Nu x dη η =0
= (4-19)
Nu x ,C =0 dθ
dη η =0,c =0

dθ
where = 0.585 for Pr = 0.7.
dη η =0,c =0

Nusseltratio=dthetadeta[1]/0.5851

Figure 6 illustrates the Nusselt number ratio as a function of C.


Nusselt number/Nusselt number with C = 0
1
0.9
0.8
0.7
0.6
0.5
0.4
0.3
0.2
0.1
0
0 0.1 0.2 0.3 0.4 0.5 0.6 0.7 0.8 0.9 1
Transpiration constant, C
Figure 6: Nusselt number ratio as a function of C.
Problem 4.4-2 (4-7 in text)
Develop a self-similar solution for the flow over a flat plate that includes viscous dissipation.
The ordinary differential equation governing the dimensionless temperature difference should
include an additional term that is related to the Eckert number.
a.) Plot the dimensionless temperature difference, (T - Ts)/(T∞ - Ts), as a function of the
similarity parameter, η, for various values of Ec with Pr = 10.

The development of the self-similar solution for the velocity proceeds as presented in Section
4.4.2.

$UnitSystem SI MASS RAD PA K J


$TabStops 0.2 3.5 in

Pr=10 [-] "Prandtl number"


eta_infinity=Max(10,10/sqrt(Pr)) "outer edge of computational domain"
N=101 [-] "number of steps in the numerical integration"
DELTAeta=eta_infinity/(N-1) "size of the integration steps"
duplicate i=1,N
eta[i]=(i-1)*eta_infinity/(N-1) "position of integration steps"
end

f[1]=0 [-] "f at eta = 0"


dfdeta[1]=0 [-] "dfdeta at eta = 0"
{d2fdeta2[1]=0.3 [-]} "d2fdeta2 at eta = 0, this is a guess"

"Crank-Nicolson integration"
duplicate i=1,(N-1)
f[i+1]=f[i]+(dfdeta[i]+dfdeta[i+1])*DELTAeta/2
dfdeta[i+1]=dfdeta[i]+(d2fdeta2[i]+d2fdeta2[i+1])*DELTAeta/2
d2fdeta2[i+1]=d2fdeta2[i]-(2*f[i]*d2fdeta2[i]+2*f[i+1]*d2fdeta2[i+1])*DELTAeta/2
end
dfdeta[N]=1 [-]

The development of the self-similar for the temperature solution proceeds as presented in Section
4.4.3 but the viscous dissipation term is retained in the thermal energy conservation equation:

2
∂T ∂T ∂ 2T μ ⎛ ∂u ⎞
u +v =α 2 +
ρ c ⎜⎝ ∂y ⎟⎠
(1)
∂x ∂y ∂y

The dimensionless temperature difference is defined as in Section 4.3.2:

T − Ts
θ = (2)
T∞ − Ts

Substituting Eq. (2) into Eq. (1) leads to:

∂θ ∂θ ∂ 2θ


2
μ ⎛ ∂u ⎞
u +v =α 2 +
ρ c (T∞ − Ts ) ⎜⎝ ∂y ⎟⎠
(3)
∂x ∂y ∂y
The boundary conditions expressed in terms of θ are:

θη =0 = 0 (4)

θη →∞ = 1 (5)

The governing differential equation is transformed to the similarity variable.

∂θ η dθ
=− (6)
∂x 2 x dη

∂θ dθ 1 u∞
= (7)
∂y dη 2 υ x

∂ 2θ d 2θ u∞
= (8)
∂y 2 dη 2 4υ x

∂u u∞ d 2 f u∞
= (9)
∂y 2 dη 2 υx

df
u = u∞ (10)

u∞ υ ⎛ df ⎞
v= ⎜η −f⎟ (11)
x ⎝ dη ⎠

Substituting Eqs. (6) through (11) into Eq. (3) leads to:

df ⎛ η dθ ⎞ u∞ υ ⎛ df ⎞ dθ 1 u∞
u∞ ⎜− ⎟+ ⎜η −f⎟ =
N dη ⎝ 2 x dη ⎠ x ⎝ dη ⎠ dη 2 υ x
u




∂θ v ∂θ
∂x ∂y
(12)
d 2θ u
2
μ u∞2 ⎛ d 2 f ⎞ u∞
α 2 ∞ + ⎜ ⎟
dη 4υ x ρ c (T∞ − Ts ) 4 ⎝ dη 2 ⎠ υ x



∂ 2θ ⎛ ∂u ⎞
2

∂y 2 ⎜ ⎟
⎝ ∂y ⎠

Dividing through by u∞/x leads to:


f dθ d 2θ 1
2
u∞2 1 ⎛ d2 f ⎞
− = + ⎜ ⎟ (13)
2 dη dη 2 4 Pr c (T∞ − Ts ) 4 ⎝ dη 2 ⎠


Ec

The Eckert number is defined in Section 4.3.2 as:

u∞2
Ec = (14)
c (T∞ − Ts )

Therefore, the transformed thermal energy equation, including viscous dissipation is:

d 2θ dθ
2
⎛ d2 f ⎞
+ 2 f Pr = Ec Pr ⎜ 2⎟ (15)
dη 2 dη ⎝ dη ⎠

The numerical solution proceeds as discussed in Section 4.4.3 with the modification of the state
equation:

d 2θ dθ
2
⎛ d2 f ⎞
= − 2 f Pr + Ec Pr ⎜ 2⎟ (16)
dη 2 dη ⎝ dη ⎠

The Eckert number is set:

Ec=100 [-] "Eckert number"

and the solution without viscous dissipation ( θnv ) is obtained:

"Solution without including viscous dissipation"


theta_nv[1]=0 [-] "theta at eta = 0"
{dthetadeta_nv[1]=0.5 [-]} "dthetadeta at eta = 0, this is a guess"
"Crank-Nicolson integration"
duplicate i=1,(N-1)
theta_nv[i+1]=theta_nv[i]+(dthetadeta_nv[i]+dthetadeta_nv[i+1])*DELTAeta/2
dthetadeta_nv[i+1]=dthetadeta_nv[i]+(-2*f[i]*Pr*dthetadeta_nv[i]-
2*f[i+1]*Pr*dthetadeta_nv[i+1])*DELTAeta/2
end
theta_nv[N]=1

The solution including viscous dissipation is obtained:

"Solution including viscous dissipation"


theta[1]=0 [-] "theta at eta = 0"
{dthetadeta[1]=0.5 [-]} "dthetadeta at eta = 0, this is a guess"
"Crank-Nicolson integration"
duplicate i=1,(N-1)
theta[i+1]=theta[i]+(dthetadeta[i]+dthetadeta[i+1])*DELTAeta/2
dthetadeta[i+1]=dthetadeta[i]+(-2*f[i]*Pr*dthetadeta[i]-2*f[i+1]*Pr*dthetadeta[i+1]-&
Ec*d2fdeta2[i]^2/4-Ec*d2fdeta2[i+1]/4)*DELTAeta/2
end
theta[N]=1

Figure 1 illustrates the dimensionless temperature difference as a function of η for various values
of Ec with Pr = 10.

2.5
Dimensionless temperature difference

2.25 Ec = 100
Ec = 50
2
Ec = 25
1.75 Ec = 10
1.5

1.25

0.75

0.5 Ec = 0 (no viscous dissipation)

0.25

0
0 0.5 1 1.5 2 2.5 3
Similarity variable
Figure 1: Dimensionless temperature difference as a function of η for various values of Ec with Pr = 10.

b.) Plot the ratio of the Nusselt number to the Nusselt number neglecting viscous dissipation
(i.e., with Ec = 0) as a function of the Eckert number for various values of the Prandtl
number.

The Nusselt number is:

1 u∞ x dθ
Nu x = (17)
2 υ dη η =0

Therefore, the ratio of the Nusselt number including viscous dissipation to the Nusselt number
neglecting viscous dissipation is:

dθ
Nu x dη η =0
= (18)
Nu x , Ec =0 dθnv
dη η =0

Nusselt_ratio=dthetadeta[1]/dthetadeta_nv[1]
Figure 2 illustrates the ratio of the Nusselt number to the Nusselt number neglecting viscous
dissipation as a function of the Eckert number for various values of the Prandtl number.

Nusselt number/Nusselt number with Ec = 0


30

25

Pr = 0.5
20

15 Pr = 1

10 Pr = 2

Pr = 5
5
Pr = 10

0
0 10 20 30 40 50 60 70 80 90 100
Eckert number
Figure 2: Nusselt number ratio as a function of Ec for various values of Pr.
Problem 4.7-1 (4-8 in text)
Use the Spalding model to obtain a velocity and temperature law of the wall (your temperature
law of the wall should be obtained numerically using the EES Integral command). Compare your
result with the Prandtl-Taylor model. Use a molecular Prandtl number of Pr = 0.7 and a
turbulent Prandtl number of Prturb = 0.9.

The Spalding model of the universal velocity distribution is given in Table 4-2:


y + = u + + 0.11408 ⎢exp (κ u + ) − 1 − κ u + −
( κ u )
+ 2


( κ u )
+ 3


( κ u )⎥
+ 4⎤

(1)
⎢ 2 6 24 ⎥
⎣ ⎦

where κ = 0.41. The Prandtl-Taylor model is also provided in Table 4-2:

+
⎧⎪ y + for 0 < y + < 11.5
u =⎨ (2)
⎪⎩2.44 ln ( y ) + 5.5 for y > 11.5
+ +

$UnitSystem SI MASS RAD PA K J


$TabStops 0.2 3.5 in

y_plus=20 [-] "inner position"


K = 0.41 [-] "von Karman constant"
u_plus_PT=IF(y_plus,11.5,y_plus,y_plus,2.44*ln(y_plus)+5.5) "Prandtl-Taylor model"
y_plus = u_plus+0.11408*(exp(K*u_plus)-1-K*u_plus-(K*u_plus)^2/2-(K*u_plus)^3/6-(K*u_plus)^4/24)
"Spalding model"

Figure 1 illustrates the inner velocity as a function of inner position using the Spalding and
Prandtl-Taylor models.

Figure 1: Universal velocity profile from the Prandtl-Taylor and Spalding models.
The inner temperature difference associated with the Prandtl-Taylor model was derived in
Section 4.7.9:

⎛ ⎞
Prturb ⎜
1
+
1
Pr Prturb
( κ y + − 1) ⎟
θ + − 11.5 Pr = ln ⎜ ⎟ for y + > 11.5 (4-3)
κ ⎜ (11.5 κ − 1) ⎟⎟
1 1
⎜ +
⎝ Pr Prturb ⎠

Pr=0.7 "molecular Prandtl number"


Pr_turb=0.9 "turbulent Prandtl number"
theta_plus_PT=IF(y_plus,11.5,Pr*y_plus,Pr*y_plus,11.5*Pr+Pr_turb*ln((1/Pr+(K*y_plus-&
1)/Pr_turb)/(1/Pr+(K*11.5-1)/Pr_turb))/K)
"inner temperature difference predicted using the Prandtl-Taylor model"

The derivative of the inner temperature difference with respect to inner position is given by Eq.
(4-432) in the text:

dθ + 1
+
= (4)
dy ⎛ 1 ε ⎞
⎜ + M ⎟
⎝ Pr Prturb υ ⎠

The ratio of εM/υ associated with the Spalding model is provided in Table 4-3:

εM

= 0.0526 ⎢exp (κ u + ) − 1 − κ u + −
(κ u ) − (κ u )
+ 2 + 3 ⎤
⎥ (5)
υ ⎢ 2 6 ⎥
⎣ ⎦

Equations (4), (1), and (5) together allow the derivative of the inner temperature difference to be
computed at an arbitrary value of y+.

eMovernu=0.0526*(exp(K*u_plus)-1-K*u_plus-(K*u_plus)^2/2-(K*u_plus)^3/6)
dtheta_plusdy_plus=1/(1/Pr+eMovernu/Pr_turb)

The inner temperature distribution associated with the Spalding model is obtained by
numerically integrating Eq. (4) using the Integral function in EES:

y+
dθ + +
θ = ∫ dy + dy
+
(6)
+
y =0

{y_plus=20 [-]} "inner position"


theta_plus=Integral(dtheta_plusdy_plus,y_plus,0.001,330)
$IntegralTable y_plus,theta_plus
Figure 2 illustrates the inner temperature difference as a function of inner position using the
Spalding and Prandtl-Taylor models.

Figure 2: Inner temperature profile from the Prandtl-Taylor and Spalding models.
Problem 4.7-2 (4-9 in text)
Use the van Driest model to obtain a velocity and temperature law of the wall. (Both of these
results should be obtained numerically using the EES Integral command). Compare your result
with the Prandtl-Taylor model. Use a molecular Prandtl number of Pr = 0.7 and a turbulent
Prandtl number of Prturb = 0.9.

The Prandtl-Taylor model is provided in Table 4-2:

+
⎧⎪ y + for 0 < y + < 11.5
u =⎨ (1)
⎪⎩2.44 ln ( y ) + 5.5 for y > 11.5
+ +

$UnitSystem SI MASS RAD PA K J


$TabStops 0.2 3.5 in

y_plus=20 [-] "inner position"


K = 0.41 [-] "von Karman constant"
u_plus_PT=IF(y_plus,11.5,y_plus,y_plus,2.44*ln(y_plus)+5.5) "Prandtl-Taylor model"

Figure 1 illustrates the inner velocity as a function of inner position using the Prandtl-Taylor
model.

Figure 1: Universal velocity profile from the Prandtl-Taylor and van Driest models.

The van Driest model is also given in Table 4-2:

du + 2
= (2)
dy + 2 ⎡
2
⎛ y ⎞⎤
+
1 + 1 + 4 (κ y + ) ⎢1 − exp ⎜ − ⎟ ⎥
⎣ ⎝ 26 ⎠ ⎦
The velocity distribution is obtained by numerically integrating Eq. (2):

y+
du + +

+
u = dy (3)
0
dy +

{y_plus=20 [-]} "inner position"


du_plusdy_plus=2/(1+sqrt(1+4*(K*y_plus)^2*(1-exp(-y_plus/26))^2))
u_plus=Integral(du_plusdy_plus,y_plus,0.001,330)
$IntegralTable y_plus,u_plus

The van Driest velocity distribution is shown in Figure 1.

The inner temperature difference associated with the Prandtl-Taylor model was derived in
Section 4.7.9:

⎛ ⎞
Prturb ⎜
1
+
1
Pr Prturb
( κ y + − 1) ⎟
θ + − 11.5 Pr = ln ⎜ ⎟ for y + > 11.5 (4-4)
κ ⎜ (11.5 κ − 1) ⎟⎟
1 1
⎜ +
⎝ Pr Prturb ⎠

Pr=0.7 "molecular Prandtl number"


Pr_turb=0.9 "turbulent Prandtl number"
theta_plus_PT=IF(y_plus,11.5,Pr*y_plus,Pr*y_plus,11.5*Pr+Pr_turb*ln((1/Pr+(K*y_plus-&
1)/Pr_turb)/(1/Pr+(K*11.5-1)/Pr_turb))/K)

The derivative of the inner temperature difference with respect to inner position is given by Eq.
(4-432) in the text:

dθ + 1
+
= (5)
dy ⎛ 1 ε ⎞
⎜ + M ⎟
⎝ Pr Prturb υ ⎠

The ratio of εM/υ associated with the van Driest model is provided in Table 4-3:

2
ε M ⎧⎪ + ⎡ ⎛ y + ⎞ ⎤ ⎫⎪ du +
= ⎨κ y ⎢1 − exp ⎜ − ⎟⎥ ⎬ (6)
υ ⎪⎩ ⎣ ⎝ 24.7 ⎠ ⎦ ⎪⎭ dy
+

Equations (5), (2), and (6) together allow the derivative of the inner temperature difference to be
computed at an arbitrary value of y+.

eMovernu=(K*y_plus*(1-exp(-y_plus/24.7)))^2*du_plusdy_plus
dtheta_plusdy_plus=1/(1/Pr+eMovernu/Pr_turb)

The inner temperature distribution associated with the Spalding model is obtained by
numerically integrating Eq. (6) using the Integral function in EES:
y+
dθ + +
θ = ∫ dy + dy
+
(7)
y+ =0

theta_plus=Integral(dtheta_plusdy_plus,y_plus,0.001,330)
$IntegralTable y_plus,u_plus,theta_plus

Figure 2 illustrates the inner temperature difference as a function of inner position using the van
Driest and Prandtl-Taylor models.

Figure 2: Inner temperature profile from the Prandtl-Taylor and van Driest models.
Problem 4.7-3 (4-10 in text)
In Section 4.5, a conceptual model of a turbulent flow was justified based on the fact that the
thermal resistance of the viscous sublayer (δvs/k) is larger than the thermal resistance of the
turbulent boundary layer (δturb/kturb). Estimate the magnitude of each of these terms for a flow of
water over a smooth flat plate and evaluate the validity of this simplification. The free stream
velocity is u∞ = 10 m/s and the plate is L = 1 m long. The water is at 20°C and 1 atm.

The inputs are entered in EES:

$UnitSystem SI MASS RAD PA K J


$TABSTOPS 0.2 0.4 0.6 0.8 3.5 in

u_infinity=10 [m/s] "free-stream velocity"


L=1 [m] "length of plate"
T=converttemp(C,K,20[C]) "temperature"
p=1 [atm]*convert(atm,Pa) "pressure"

The properties of water (μ, ρ, c, k, and υ) are obtained using EES' internal property routines:

"Fluid properties"
mu=viscosity(Water,T=T,P=p) "viscosity"
rho=density(Water,T=T,P=p) "density"
c=cP(Water,T=T,P=p) "specific heat capacity"
k=conductivity(Water,T=T,P=p) "conductivity"
nu=mu/rho "kinematic viscosity"

According to Eq. (4-571) in the text, the viscous sublayer thickness at the trailing edge of the
plate can be estimated according to:

34.9 L
δ vs ≈ (1)
ReL0.9

where ReL is the Reynolds number:

ρ u∞ L
ReL ≈ (2)
μ

The thermal resistance of the viscous sublayer per unit area is approximately:

δ vs
Rvs ≈ (3)
k

Re=rho*L*u_infinity/mu "Reynolds number"


delta_vs=34.9*L/Re^0.9 "viscous sublayer thickness"
R_vs=delta_vs/k "thermal resistance of the viscous sublayer"

which leads to Rvs = 3.0x10-5 K-m2/W.


According to Eq. (4-566) of the text, the turbulent boundary layer thickness at the trailing edge is
approximately:

0.19 x
δ m,turb ≈ L (4)
ReL0.1

delta_t_turb=0.19*L/Re^0.1 "turbulent boundary layer thickness"

The effective conductivity of the turbulent boundary layer is almost entirely related to the
turbulent eddies and can be estimated according to:

ρ cεM
kturb ≈ (5)
Prturb

For simplicity, we will use the Prandtl-Taylor model to estimate the eddy diffusivity of
momentum required by Eq. (5):

εM
= κ y+ −1 (6)
υ

We will calculate use a value of y+ that is consistent with being at the mid-point of the boundary
layer:

εM δ
= κ m,turb − 1 (7)
υ 2 Lchar ,vs

where Lchar,vs is defined according to:

μ
Lchar ,vs = (8)
τs ρ

The friction coefficient correlation provided by Eq. (4-588) for turbulent flow over a smooth flat
plate is used to compute the shear stress:

C f = 0.0592 ReL−0.20 (9)

1
τ s = ρ u∞2 C f (10)
2

C_f=0.0592*Re^(-0.2) "friction coefficient"


tau_s=C_f*rho*u_infinity^2/2 "shear stress"

The value of Lchar,vs is computed using Eq. (8) and the value of εM is computed using Eq. (7):
L_char_vs=mu/sqrt(tau_s*rho) "characteristic length of viscous sublayer"
Kappa=0.41 [-] "von Karman constant"
e_M=nu*(Kappa*delta_t_turb/(2*L_char_vs)-1) "eddy diffusivity of momentum"

The turbulent conductivity is estimated according to Eq. (5). The thermal resistance of the
turbulent boundary layer per unit area is estimated according to:

δ m,turb
Rturb ≈ (11)
kturb

Pr_turb=0.9 [-] "turbulent Prandtl number"


k_turb=rho*c*e_M/Pr_turb "effective turbulent conductivity"
R_turb=delta_t_turb/k_turb "thermal resistance of the turbulent boundary layer"

which leads to Rturb = 3.1x10-6 K-m2/W. Note that Rvs/Rturb is approximately 10, justifying the
assumption that the thermal resistance of the viscous sublayer is larger than the thermal
resistance of the turbulent boundary layer.
Problem 4.8-4 (4-11 in text): Plate with Unheated Starting Length
Figure P4.8-4 illustrates a flat plate that has an unheated starting length (ε); the hydrodynamic
boundary layer grows from the leading edge of the plate while the thermal boundary layer grows
from x > Luh. Assume that the plate has a constant surface temperature, Ts, for x > Luh.

free stream, u∞ , T∞
⎛ ∂T ⎞
⎜ ∂ ⎟ =0 Ty=0 = Ts
⎝ y ⎠ y =0

Luh
L
x
Figure P4.8-4: Plate with an unheated starting length.

Determine a correlation for the local Nusselt number in this situation using the integral
technique. Use a third order velocity and temperature distribution. Neglect viscous dissipation.
You may find it useful to solve for the ratio of the thermal to momentum boundary layer
thickness.

The momentum boundary layer associated with a 3rd order polynomial velocity distribution was
found in Eq. (4-486):

μx
δ m = 4.640 (1)
ρ u∞

The differential equation that governs the growth of the thermal boundary layer with a 3rd order
polynomial form of the velocity and temperature distribution was found in Eq. (4-524) and
remains valid:

d ⎡ δ t2 δ t4 ⎤ 3 k
⎢ 0.15 − 0.01 3 ⎥ = (2)
dx ⎣ δm δ m ⎦ 2 δ t ρ c u∞

However, because of the unheated starting length, the momentum and thermal boundary layers
do not develop together and therefore their relative size is not dictated by the Prandtl number.
We will define the variable r as the local ratio of the thermal to momentum boundary layer
thicknesses. Note that r = 0 for x < Luh.

δt
r= (3)
δm

Substituting Eq. (3) into Eq. (2) leads to:

⎡( 0.15 r 2 − 0.01 r 4 ) δ m ⎤ =
d 3 k
⎣ ⎦ (4)
dx 2 r δ m ρ c u∞
Neglecting the 2nd term in Eq. (4) (which is much smaller than the first term) leads to:

d 2 10 k
⎡⎣ r δ m ⎤⎦ = (5)
dx r δ m ρ c u∞

or

d 2 α
r δm ⎡⎣ r δ m ⎤⎦ = 10 (6)
dx u∞

Expanding the derivative:

⎡ dδ dr ⎤ α
r δ m ⎢ r 2 m + 2 δ m r ⎥ = 10 (7)
⎣ dx dx ⎦ u∞

or

dδ m dr α
δm r3 + 2 δ m2 r 2 = 10 (8)
dx dx u∞

The hydrodynamic boundary layer develops from the front of the plate. The derivative of Eq. (1)
leads to:

dδ m υ
= 2.320 (9)
dx x u∞

Substituting Eqs. (1) and (9) into Eq. (8) leads to:

υx υ υx dr α
10.765 r3 + 43.059 r2 = 10 (10)
u∞ x u∞ u∞ dx u∞

which can be simplified to:

dr α
10.765 r 3 + 43.059 x r 2 = 10 (11)
dx υ

Equation (11) is an ordinary differential equation that can be solved either numerically or
analytically. The boundary condition is related to the fact that the thermal boundary layer
develops beginning at x = Luh, therefore r = 0 for x < Luh.

rx = Luh = 0 (12)
Here we will solve the ordinary differential equation analytically. Equation (11) can be
rearranged:

dr α
43.059 x r 2 = 10 − 10.765 r 3 (13)
dx υ

Equation (8) is a separable ordinary differential equation; the terms related to x and r are
separated:

r 2 dr dx
= (14)
10 43.059 x
− 10.765 r 3
Pr

and integrated:
r x
r 2 dr dx
∫ 10 = ∫ 43.059 x (15)
0 − 10.765 r 3 Luh
Pr

An intermediate variable, w, is defined:

10
w= − 10.765 r 3 (16)
Pr

and so

dw = −32.295 r 2 dr (17)

which leads to:

10
−10.765 r 3
Pr x
1 dw dx
−32.395 ∫
10
= ∫
w Luh 43.059 x
(18)
Pr

or:

⎛ 10 ⎞
⎜ − 10.765 r 3 ⎟
1 Pr 1 ⎛ x ⎞
ln ⎜ ⎟= ln ⎜ ⎟ (19)
−32.395 ⎜ 10 ⎟ 43.059 ⎝ Luh ⎠
⎜ ⎟
⎝ Pr ⎠

which can be simplified:


⎛ x ⎞
ln (1 − 1.0765 Pr r 3 ) = −0.752 ln ⎜ ⎟ (20)
⎝ Luh ⎠

Taking the exponential of both sides leads to:


−0.752
⎛ x ⎞
1 − 1.0765 Pr r = ⎜ 3
⎟ (21)
⎝ Luh ⎠

Solving for r:

1
⎡ ⎛ Luh ⎞0.75 ⎤ 3

⎢1 − ⎜ ⎟ ⎥
r=⎢ ⎝
x ⎠ ⎥
(22)
⎢ 1.0765 Pr ⎥
⎢ ⎥
⎣ ⎦

The Nusselt number is defined as:

qs′′ x
Nu x = (23)
(Ts − T∞ ) k
The heat flux at the wall for the 3rd order polynomial form of the temperature distribution is
provided in Table 4-5:

3 (Ts − T∞ )
qs′′ = k (24)
2 δt

Substituting Eq. (11) into Eq. (10) leads to:

x 3 (Ts − T∞ )
Nu x = k (24)
(Ts − T∞ ) k 2 δt
or

3 x
Nu x = (25)
2 δt

Notice that Eq. (25) once again reinforces the concept that the Nusselt number is related to the
ratio of the characteristic length for the problem (x) to the effective conduction length in the fluid
(δt). Substituting the definition of r into Eq. (25) leads to:
3 x
Nu x = (26)
2 r δm

Substituting Eq. (22) for r into Eq. (26) leads to:

x (1.0765 Pr )
1
3
3
Nu x = 1
(27)
2 ⎡ ⎛ L ⎞0.75 ⎤ 3

δ m ⎢1 − ⎜ uh ⎟ ⎥
⎢⎣ ⎝ x ⎠ ⎥⎦

Substituting Eq. (1) for δm into Eq. (27) leads to:

x (1.0765 Pr )
1
3
3
Nu x = 1
(28)
2
υ x ⎡ ⎛ Luh ⎞ ⎤
0.75 3

4.640 ⎢1 − ⎜ ⎟ ⎥
u∞ ⎣⎢ ⎝ x ⎠ ⎦⎥

or
1 1
3
0.331 Rex 2 Pr
Nu x = 1
(29)
⎡ ⎛ Luh ⎞ 0.75
⎤ 3

⎢1 − ⎜ ⎟ ⎥
⎢⎣ ⎝ x ⎠ ⎥⎦

Notice that the numerator of Eq. (29) is the Nusselt number that would have existed at x=Luh in
the absence of an unheated starting length and the denominator is a number that is necessarily
less than one and therefore tends to increase the local Nusselt number due to the under-
developed thermal boundary layer thickness. This solution agrees with the solution provided in
Section 4.9.2 for laminar flow over a plate with an unheated starting length.
Problem 4.8-5 (4-12 in text): Low Prandtl Number Flow
A flow of a liquid metal over a flat plate, shown in Figure 4.8-5, is being considered during the
design of an advanced nuclear reactor.

Pr << 1
u∞ , T∞

Ts

x
Figure P4.8-5: Flow of a low Prandtl number liquid metal over a flat plate.

You are to develop a solution to this problem using an integral technique. Because the Prandtl
number is much less than one, it is appropriate to assume that δm << δt and therefore the velocity
is constant and equal to u∞ throughout the thermal boundary layer. Use a 2nd order temperature
distribution and neglect viscous dissipation.
a.) If the temperature of the plate is constant and equal to Ts then derive a solution for the local
Nusselt number on the plate surface as a function of the Reynolds number and the Prandtl
number.

The energy integral equation is simplified:

δ
d ⎡t ⎤ q s′′
⎢ ∫ u (T − T∞ ) dy ⎥ = (1)
dx ⎣⎢ 0 ⎦⎥ ρ c

The velocity within the boundary layer is assumed to be uniform and equal to u∞:

δ
d ⎡ t ⎤ q s′′
⎢u∞ ∫ (T − T∞ ) dy ⎥ = (2)
dx ⎣⎢ 0 ⎦⎥ ρ c

Equation (2) is rearranged:

d ⎡ ⎣⎡T − Ts − (T∞ − Ts ) ⎦⎤ dy ⎤ = qs′′


δt

⎢ ∞( ∞ s)∫
u T − T ⎥ (3)
dx ⎣⎢ 0 (T∞ − Ts ) ⎦⎥ ρ c

The appropriate form of the second order temperature distribution from Table 4-5 is:

T − Ts ⎡ y y 2 ⎤
= ⎢2 − ⎥ (4)
T∞ − Ts ⎣ δ t δ t2 ⎦

Substituting Eq. (4) into Eq. (3) leads to:


δt
d ⎡ ⎛ y y 2 ⎞ ⎤ q s′′
⎢u∞ (T∞ − Ts ) ∫ ⎜ 2 − 2 − 1⎟ dy ⎥ = (5)
dx ⎣⎢ 0 ⎝
δt δt ⎠ ⎦⎥ ρ c

Equation (5) can be integrated:

δt
d ⎡ ⎛ y 2 1 y3 ⎞ ⎤ q ′′
⎢u∞ (T∞ − Ts ) ⎜ − − y ⎟ dy ⎥ = s (6)
dx ⎢ ⎝ δt 3 δt ⎠0 ⎥⎦ ρ c
2

Applying the limits:

d ⎡ u∞ (T∞ − Ts ) δ t ⎤ qs′′
− ⎢ ⎥= (7)
dx ⎣ 3 ⎦ ρc

The relationship between heat flux and the surface-to-fluid temperature difference from Table 4-
5 is:

qs′′ = 2k
(Ts − T∞ ) (8)
δt

Substituting Eq. (8) into Eq. (7) for the unknown heat flux leads to:

d ⎡ u∞ (T∞ − Ts ) δ t ⎤ k (Ts − T∞ )
− ⎢ ⎥=2 (9)
dx ⎣ 3 ⎦ ρ c δt

The surface to free stream temperature difference is constant, therefore:

dδ t 6α
= (10)
dx δ t u∞

Equation (10) is separated:


δ t dδ t = dx (11)
u∞

and integrated:
δt

x

∫ δ t dδ t =
0
u∞ ∫0
dx (12)

which leads to:


12 α x
δ t2 = (13)
u∞

or:

12 α x
δt = (14)
u∞

The local Nusselt number is defined as:

hx qs′′ x
Nu = = (15)
k (Ts − T∞ ) k
Substituting Eq. (8) into Eq. (15) leads to:

2k x 2 x
Nu = = (16)
δt k δt

Substituting Eq. (14) into Eq. (16) leads to:

u∞
Nu = 2 x (17)
12 α x

or

2
Nu = Re Pr (18)
12

b.) Plot your result from (a) as a function of Re for Pr = 0.001. Overlay on your plot the
correlation for the local number for flow over a constant temperature flat plate found in
Section 4.9.

The solution from Section 4.9 that is accurate at low Prandtl number is:
1 1
0.3387 Re 2 Pr 3
Nu = 1
(19)
⎡ ⎛ 0.0468 ⎞ 2 3 ⎤ 4

⎢1 + ⎜ ⎟ ⎥
⎢⎣ ⎝ Pr ⎠ ⎥⎦

The solution from (a) and the correlation above are entered in EES:
$UnitSystem SI MASS RAD PA K J
$TabStops 0.2 0.4 3.5 in

Pr=0.001 [-] "Prandtl number"


Re=10000 [-] "Reynolds number"

Nusselt=(2/sqrt(12))*sqrt(Re)*sqrt(Pr) "Integral solution for a constant pressure plate"


Nusselt_cor=0.3387*sqrt(Re)*Pr^(1/3)/(1+(0.0468/Pr)^(2/3))^(1/4)
"Local Nusselt number correlation from Section 4.9"

Figure 4.8-5(b) illustrates the local Nusselt number predicted by Eqs. (18) and (19) as a function
of Reynolds number for Pr = 0.001.
7

6
Local Nusselt number

integral solution
5

4
Churchill and Ozoe (1973)
3

0
10 100 1000 10000 100000
Reynolds number
Figure 4.8-5(b): Local Nusselt number as a function of Reynolds number for Pr = 0.001 from the integral
solution in (a) and the correlation in Section 4.9.

c.) Plot your result from (a) and the correlation from Section 4.9 as a function of Pr for Re =
1x104. Your Prandtl number range should be from 0.001 to 100 and it should be clear from
your plot that the solutions begin to diverge as the Prandtl number approaches unity.

Figure 4.8-5(c) illustrates the local Nusselt number predicted by Eqs. (18) and (19) as a function
of Prandtl number for Re = 1x104.
700

integral solution

Local Nusselt number


100

10 Churchill and Ozoe (1973)

1
0.5
0.001 0.01 0.1 1 10 100
Prandtl number
Figure 4.8-5(b): Local Nusselt number as a function of Prandtl number for Re = 1x104 from the integral
solution in (a) and the correlation in Section 4.9. Note the divergence at Pr ≈ 1.

d.) If the heat flux at the surface of the plate is constant and equal to qs′′ then derive a solution
for the local Nusselt number on the plate surface as a function of the Reynolds number and
the Prandtl number.

The solution steps from (a) remain the same up to Eq. (7):

d ⎡ u∞ (T∞ − Ts ) δ t ⎤ qs′′
− ⎢ ⎥= (20)
dx ⎣ 3 ⎦ ρc

The relationship between heat flux and the surface-to-fluid temperature difference from Table 4-
5:

qs′′ = 2k
(Ts − T∞ ) (21)
δt

is substituted for the unknown surface to free stream temperature difference:

d ⎡ qs′′ δ t u∞ δ t ⎤ q s′′
=
dx ⎢⎣ 2 k 3 ⎥⎦ ρ c
(22)

The heat flux is constant, therefore:

d 2 6k
dx
( δt ) =
u∞ ρ c
(23)
Equation (23) is separated:


d (δ t2 ) = dx (24)
u∞

and integrated:

δ t2

x

∫ d (δ t ) = u∞ ∫0
dx
2
(25)
0

which leads to:

6α x
δ t2 = (26)
u∞

or:

6α x
δt = (27)
u∞

The local Nusselt number is calculated according to:

2x
Nu = (28)
δt

Substituting Eq. (27) into Eq. (28) leads to:

u∞
Nu = 2 x (29)
6α x

or

2
Nu = Re Pr (30)
6

e.) If the heat flux at the surface of the plate varies linearly with position then derive a solution
for the local Nusselt number on the plate surface as a function of the Reynolds number and
the Prandtl number.

The solution steps from (d) remain the same up to Eq. (22):
d ⎡ qs′′ δ t u∞ δ t ⎤ q s′′
=
dx ⎢⎣ 2 k 3 ⎥⎦ ρ c
(31)

The heat flux is linear with position:

qs′′ = β x (32)

where β is a constant. Substituting Eq. (32) into Eq. (31) leads to:

d ⎡ β x δ t u∞ δ t ⎤ β x
=
dx ⎢⎣ 2 k 3 ⎥⎦ ρ c
(33)

Therefore:

d 6 xα
⎡⎣ x δ t2 ⎤⎦ = (34)
dx u∞

Equation (34) is separated:

6 xα
d ⎡⎣ x δ t2 ⎤⎦ = dx (35)
u∞

and integrated:

x δ t2

x

∫ d ⎡⎣ x δ ⎤⎦ =
u∞ ∫0
x dx
2
t (36)
0

which leads to:

3α x 2
x δ t2 = (37)
u∞

or:

3α x
δt = (38)
u∞

The local Nusselt number is calculated according to:


2x
Nu = (39)
δt

Substituting Eq. (38) into Eq. (39) leads to:

u∞
Nu = 2 x (40)
3α x

or

2
Nu = Re Pr (41)
3

f.) Plot the solutions from (a), (d), and (e) as a function of Reynolds number for Pr = 0.001.

Figure 4.8-5(d) illustrates the local Nusselt number predicted by Eqs. (18), (30), and (41) as a
function of Re for Pr = 0.001.
14

12
Local Nusselt number

10

linearly varying heat flux


8

6 constant heat flux

constant surface temperature


4

0
10 100 1000 10000 100000
Reynolds number
Figure 4.8-5(d): Local Nusselt number as a function of Reynolds number for Pr = 0.001 for the integral
solutions with a constant surface temperature, constant surface heat flux, and linearly varying surface heat
flux.
Problem 4.8-6 (4-13 in text)
Determine the local friction coefficient as a function of Reynolds number for laminar flow over a
flat plate using the momentum integral technique. Assume a velocity distribution of the form:
u / u∞ = a sin ( b y / δ m + c ) where a, b, and c are undetermined constants. Compare your answer
to the Blasius solution from Section 4.4.

The undetermined coefficients for the velocity distribution function are selected so that:

1. The no-slip condition is satisfied at the plate surface:

u y =0 = 0 (1)

2. The free-stream velocity is recovered at the edge of the boundary layer:

u y =δ m = u∞ (2)

3. The shear stress at the edge of the boundary layer is recovered:

∂u
μ = τ y =δ m (3)
∂y y =δ m

Substituting the velocity distribution into Eq. (1) leads to:

⎛ u ⎞
⎜ ⎟ = a sin ( 0 + c ) = 0 (4)
⎝ u∞ ⎠ y = 0

Substituting the velocity distribution into Eq. (2) leads to:

⎛ u ⎞
⎜ ⎟ = a sin ( b + c ) = 1 (5)
⎝ u∞ ⎠ y =δ m

Substituting the velocity distribution into Eq. (3) leads to:

⎛ ∂u ⎞
⎜ ⎟ = a b cos ( b + c ) = 0 (6)
⎝ ∂y ⎠ y =δ m

Equation (4) can be satisfied if c = 0. Equation (6) can be satisfied if b = π/2. Equation (5) can
be satisfied if a = 1. Therefore, the velocity distribution is:

u ⎛π y ⎞
= sin ⎜ ⎟ (7)
u∞ ⎝ 2 δm ⎠
Equation (7) is substituted into the integral form of the momentum equation, simplified for this
problem:

d ⎡ m 2 ⎤
y =δ
τ
⎢ ∫ ( u − u u∞ ) dy ⎥ = − s (8)
dx ⎢⎣ y =0 ⎥⎦ ρ

The shear stress is given by:

∂u
τs = μ (9)
∂y y =0

Substituting Eq. (7) into Eq. (9) leads to:

π
τ s = μ u∞ (10)
2δm

Substituting Eq. (10) into Eq. (8) leads to:

d ⎡ m 2 ⎤
y =δ
μu π
⎢ ∫ ( u − u u∞ ) dy ⎥ = − ∞ (11)
dx ⎣⎢ y =0 ⎦⎥ ρ 2δm

Substituting Eq. (7) into Eq. (11) leads to:

d ⎡ 2 ⎛ π y ⎞⎤ ⎤
y =δ m
⎡ 2 ⎛π y ⎞ μ u∞ π
⎢ u∞
dx ⎣⎢ ∫ ⎢sin ⎜ ⎟ − sin ⎜
⎝ 2 δm ⎠
⎟ ⎥ dy ⎥ = −
⎝ 2 δ m ⎠ ⎦ ⎦⎥ ρ 2δm
(12)
y =0 ⎣

The integral in Eq. (12) is accomplished using Maple:

> restart;
> int((sin(Pi*y/(2*d)))^2-sin(Pi*y/(2*d)),y=0..d);
d ( −4 + π )

d ⎡ 2 (4 −π ) ⎤ μu π
⎢ −u∞ δm ⎥ = − ∞ (13)
dx ⎣ 2π ⎦ ρ 2δm

Equation (13) can be separated and integrated:


δm
μπ2 x

∫ δ m dδ m = ( 4 − π ) ∫ dx (14)
0
ρ u∞ 0

δ m2μπ2
= (4 −π ) x (15)
2 ρ u∞

Solving for the momentum boundary layer thickness:

μπ 2
δm = 2 (4 −π ) x (16)
ρ u∞

Substituting Eq. (16) into Eq. (10) leads to:

μ u∞ ρ u∞
τs = (17)
2 2μ ( 4 − π ) x

Substituting Eq. (17) into the definition of the friction coefficient:

2τ s 2 μ u∞ ρ u∞
Cf = = (18)
ρ u∞ ρ u∞2 2
2
2μ ( 4 − π ) x

or

1 μ
Cf = (19)
2 ( 4 − π ) ρ u∞ x

which can be rearranged:

1
2(4 −π ) 0.763
Cf = = (20)
Rex Rex

The Blasius solution from Section 4.4 provides:

0.664
Cf = (21)
Rex

and therefore the integral technique is 15% in error relative to the exact solution.
Problem 4.8-7 (4-14 in text)
A flat plate that is L = 0.2 m long experiences a heat flux given by:

⎛π x ⎞
qs′′ = qmax
′′ cos ⎜ ⎟ (1)
⎝ 4L ⎠

′′ = 2x104 W/m2. The free stream velocity is u∞ = 20 m/s and the free stream
where qmax
temperature is T∞ = 20°C. The fluid passing over the plate has thermal diffusivity α = 1x10-4
m2/s, conductivity k = 0.5 W/m-K, and Prandtl number Pr = 2.0.
a.) Use a linear temperature distribution and a linear velocity distribution in order to obtain an
ordinary differential equation for the thermal boundary layer thickness.

The integral form of the energy equation, simplified for this problem, is:

δ
d ⎡t ⎤ q s′′
⎢ ∫ u (T − T∞ ) dy ⎥ = (2)
dx ⎣⎢ 0 ⎦⎥ ρ c

Substituting a linear velocity and temperature distribution (from Tables 4-4 and 4-5) into Eq. (2)
leads to:

δ
d ⎡t ⎛ y ⎞⎛ y ⎞ ⎤ qs′′
⎢ ∫ u∞ (T∞ − Ts ) ⎜ ⎟ ⎜ − 1⎟ dy ⎥ = (3)
dx ⎣⎢ 0 ⎝ δ m ⎠ ⎝ δt ⎠ ⎦⎥ ρ c

Equation (3) is rearranged:

δ
d ⎡ t
⎛ y2 y ⎞ ⎤ q ′′
u (
⎢ ∞ ∞ s ∫
T − T ) ⎜ − ⎟ dy ⎥ = s (4)
dx ⎢⎣ 0 ⎝ δ t δ m δ m ⎠ ⎥⎦ ρ c

and integrated:

d ⎡ ⎛ δ t3 δ t2 ⎞ ⎤ qs′′
⎢ ∞ ( ∞ s )⎜
u T − T − ⎟⎥ = (5)
dx ⎣ δ
⎝ t m
3 δ 2 δ m ⎠⎦ ρc

Equation (5) can be simplified:

d ⎡ δ t2 ⎤ qs′′
⎢u∞ (Ts − T∞ ) ⎥= (6)
dx ⎣ 6δm ⎦ ρ c

According to Table 4-5, the heat flux can be written as:


qs′′ = k
(Ts − T∞ ) (7)
δt

Equation (7) is used to eliminate (Ts - T∞):

d ⎡ u∞ δ t3 qs′′ ⎤ qs′′
⎢ ⎥= (8)
dx ⎣ 6 k δ m ⎦ ρ c

The ratio of the thermal to momentum boundary layer thickness is:

δt −1
= Pr 3 (9)
δm

Substituting Eq. (9) into Eq. (8) leads to:

d ⎡ u∞ δ t2 q s′′ − 13 ⎤ qs′′
⎢ Pr ⎥ = (10)
dx ⎣ 6 k ⎦ ρc

which is rearranged:

d 6 α qs′′ 13
⎡⎣δ t2 q s′′⎤⎦ = Pr (11)
dx u∞

Equation (11) is the governing ordinary differential equation for the problem.

b.) Solve the ordinary differential equation from (a) numerically. At the leading edge of the
plate there is a singularity; obtain an analytical solution in this region and start your
numerical solution at the x position where the analytical solution is no longer valid.

Equation (11) is expanded:

dq s′′ dδ 6 α q s′′ 13


δ t2 + 2 δ t q s′′ t = Pr (12)
dx dx u∞

and the specified heat flux variation, Eq. (1), is substituted:

π ⎛π x ⎞ ⎛ π x ⎞ dδ t 6 α ⎛ π x ⎞ 13
′′
−δ t2 qmax sin ⎜ ′′ cos ⎜
⎟ + 2 δ t qmax ⎟ = ′′ cos ⎜
qmax ⎟ Pr (13)
4L ⎝ 4L ⎠ ⎝ 4 L ⎠ dx u∞ ⎝ 4L ⎠

Equation (13) is solved for the rate of change of the thermal boundary layer thickness:
6α ⎛ π x ⎞ 13 2 π ⎛π x ⎞
cos ⎜ ⎟ Pr + δ t sin ⎜ ⎟
d δ t u∞ ⎝ 4L ⎠ 4L ⎝ 4L ⎠
= (14)
dx ⎛π x ⎞
2 δ t cos ⎜ ⎟
⎝ 4L ⎠

Which can be simplified:

dδ t 3α 1 π ⎛π x ⎞
= Pr 3 + δ t tan ⎜ ⎟ (15)
dx u∞ δ t 8L ⎝ 4 L



1st term 2nd term

Very near the leading edge of the plate (where δt approaches zero), the first term in Eq. (15)
dominates the second term. The ratio of the 2nd term to the 1st term is:

π ⎛π x ⎞
δt tan ⎜ ⎟
8L ⎝ 4 L ⎠ = π δ t u∞ tan ⎛ π x ⎞
2
2nd term
= ⎜ ⎟ (16)
1st term 3α 1 1
24 L α Pr 3 ⎝ 4L ⎠
Pr 3
u∞ δ t

Therefore, very near the leading edge of the plate the differential equation simplifies to:

dδ t 3α 1
= Pr 3 (17)
dx u∞ δ t

which can be separated and integrated:


1
6 α Pr 3
δ =t
2
x (18)
u∞

Equation (18) is valid until the 2nd term in Eq. (15) becomes significant (say 0.01). Substituting
Eq. (18) into Eq. (16) provides a criteria for this location, xini:

π xini ⎛π x ⎞
tan ⎜ ini ⎟ = 0.01 (19)
4 L ⎝ 4L ⎠

The inputs are entered in EES:

$UnitSystem SI MASS RAD PA K J


$TABSTOPS 0.2 0.4 0.6 0.8 3.5 in

"Inputs"
L=0.2 [m] "length of plate"
qf_max=20000 [W/m^2] "maximum heat flux"
alpha=1e-4 [m^2/s] "thermal diffusivity"
Pr=2 [-] "Prandtl number"
u_infinity=20 [m/s] "free stream velocity"
k=0.5 [W/m-K] "conductivity"
T_infinity=converttemp(C,K,20[C]) "free stream temperature"

The numerical solution is initiated at xini, determined according to Eq. (19). The thermal
boundary layer thickness at this location, δt,ini, is obtained using Eq. (18):

pi*x_ini*tan(pi*x_ini/(4*L))/(4*L)=0.01 "position to begin numerical solution"


delta_t_ini=sqrt(6*alpha*Pr^(1/3)*x_ini/u_infinity)
"thermal boundary layer at the beginning of numerical solution"

The boundary layer thickness is obtained by integrating Eq. (15) according to:

dδ t
x
δ t = δ t ,ini + ∫
xini
dx
dx (20)

The integrand is evaluated at an arbitrary set of state variables:

delta_t=delta_t_ini
x=x_ini
ddeltatdx=pi*delta_t*tan(pi*x/(4*L))/(8*L)+3*alpha*Pr^(1/3)/(u_infinity*delta_t)

The state variables are commented out and the Integral command in EES is used to obtain the
solution:

{delta_t=delta_t_ini
x=x_ini}
delta_t=delta_t_ini+Integral(ddeltatdx,x,x_ini,L)
$IntegralTable x,delta_t

c.) Plot the surface temperature of the plate as a function of axial position.

The heat flux is related to the surface temperature according to Eq. (7). Solving for the surface
temperature leads to:

δ t qs′′
Ts = T∞ + (21)
k

where the heat flux is computed according to Eq. (1):

qf_s=qf_max*cos(pi*x/(4*L))
T_s=T_infinity+qf_s*delta_t/k
T_s_C=converttemp(K,C,T_s)
$IntegralTable x,delta_t,qf_s,T_s_C

The plate surface temperature as a function of position is shown in Figure 1.


120

110

100

90

Temperature (°C)
80 numerical solution

70

60

50

40 analytical solution

30
0 0.02 0.04 0.06 0.08 0.1 0.12 0.14 0.16 0.18 0.2

Position (m)
Figure 1: Plate surface temperature as a function of position.

d.) Overlay on your plot from (c) the surface temperature calculated using the correlation for the
local heat transfer coefficient on a flat plate.

The kinematic viscosity is calculated from the Prandtl number and thermal diffusivity:

υ = α Pr (22)

The Reynolds number is computed based on position:

u∞ x
Rex = (23)
υ

The local Nusselt number is computed using the correlation for laminar flow over a flat plate
provided in Section 4.9.2:
1 1
0.3387 Rex 2 Pr 3
Nu x = 1
(4-24)
⎡ ⎛ 0.0468 ⎞ 2 3 ⎤ 4

⎢1 + ⎜ ⎟ ⎥
⎢⎣ ⎝ Pr ⎠ ⎥⎦

The heat transfer coefficient is computed based on the Nusselt number:

k Nu
h= (4-25)
x

and the surface temperature is computed according to:

q s′′
Ts = T∞ + (26)
h
where qs′′ is obtained from Eq. (1).

"Solution using a correlation"


nu=alpha*Pr "kinematic viscosity"
Re_x=u_infinity*x/nu "Reynolds number"
Nusselt=0.3387*sqrt(Re_x)*Pr^0.333/(1+(0.0468/Pr)^0.667)^0.25 "local Nusselt number"
h=Nusselt*k/x "local heat transfer coefficient"
qf_s=qf_max*cos(pi*x/(4*L)) "heat flux"
T_s_an=T_infinity+qf_s/h "surface temperature"
T_s_an_C=converttemp(K,C,T_s_an) "in C"

Figure 2 illustrates the numerical solution using the integral technique and the solution calculated
using the correlation for heat transfer coefficient.

120

110
using correlation
100
Temperature (°C)

90

80 integral technique

70

60

50

40

30
0 0.02 0.04 0.06 0.08 0.1 0.12 0.14 0.16 0.18 0.2
Position (m)
Figure 2: Plate surface temperature as a function of position calculated using the integral technique and the
correlation.
Problem 4.9-2 (4-15 in text): Energy Loss from a High Rise Apartment
You and your friend are looking for an apartment in a high-rise building. You have your choice
of 4 different south-facing units (units #2 through #5) in a city where the wind is predominately
from west-to-east, as shown in Figure P4.9-2. You are responsible for paying the heating bill for
your apartment and you have noticed that the exterior wall is pretty cheaply built. Your friend
has taken heat transfer and therefore is convinced that you should take unit #5 in order to
minimize the cost of heating the unit because the boundary layer will be thickest and heat
transfer coefficient smallest for the exterior wall of that unit. Prepare an analysis that can predict
the cost of heating each of the 4 units so that you can (a) decide whether the difference is worth
considering, and (b) if it is, choose the optimal unit. Assume that the heating season is time = 4
months long (120 days) and the average outdoor air temperature during that time is T∞ = 0°C
and average wind velocity is u∞ = 5 mph. The dimensions of the external walls are provided in
Figure P4-15; assume that no heat loss occurs except through the external walls. Further, assume
that the walls have a total thermal resistance on a unit area basis (not including convection) of
Rw′′ = 1 K-m2/W. The internal heat transfer coefficient is hin = 10 W/m2-K. You like to keep
your apartment at Tin = 22°C and use electric heating at a cost of ec = 0.15$/kW-hr. You may
use the properties of air at T∞ for your analysis and neglect the effect of any windows.

North
top view

wind,
u∞ = 1.2 mph edge of building
T∞ = 0°C

side view H = 12 ft
wind,
u∞ = 1.2 mph
T∞ = 0°C unit #1 #2 #3 #4 #5 #6

L1 = 20 ft
L2 = 40 ft
L3 = 60 ft
L4 = 80 ft
L5 = 80 ft
L6 = 100 ft
Figure P4.9-2: Location of the external wall of units 2 through 5 relative to the wind direction.

a.) Determine the average yearly heating cost for each of the 4 units and discuss which
apartment is best and why.

The known information is entered in EES:

$UnitSystem SI MASS RAD PA K J


$TABSTOPS 0.2 0.4 0.6 0.8 3.5 in

"Inputs"
u_infinity= 5.0 [mile/hr]*convert(mile/hr,m/s) "wind velocity"
T_infinity=converttemp(C,K,0) "wind temperature"
time=120 [day]*convert(day,s) "heating time"
R``_w=1 [K-m^2/W] "wall resistance on a unit area basis"
h_in=10 [W/m^2-K] "internal heat transfer coefficient"
ec=0.15 [$/kW-hr]*convert($/kW-hr,$/J) "electrical energy cost"
T_in=converttemp(C,K,22) "internal temperature"
H=12 [ft]*convert(ft,m) "heigth of apartment"
L2=20 [ft]*convert(ft,m) "position of unit 2 - start"
L3=40 [ft]*convert(ft,m) "position of unit 3 -start"
L4=60 [ft]*convert(ft,m) "position of unit 4 - start"
L5=80 [ft]*convert(ft,m) "position of unit 5 - start"
L6=100 [ft]*convert(ft,m) "end of unit 5"

The properties of air (ρ, k, μ, and Pr) are obtained using EES’ built-in property routines:

"Air properties"
rho=density(Air,T=T_infinity,P=1[atm]*convert(atm,Pa)) "density of air"
mu=viscosity(Air,T=T_infinity) "viscosity of air"
k=conductivity(Air,T=T_infinity) "conductivity of air"
Pr=Prandtl(Air,T=T_infinity) "Prandtl number of air"

We are interested in the heat transferred through each of the sub-sections of the wall outlined by
the red, dashed lines in Figure P4.9-2. However, there is no convenient formula for the average
heat transfer coefficient over a range of position; therefore, the total side of the wall will be split
into 5 plates corresponding to unit#1, units #1&2, units #1 through #3, etc. The heat loss
through each of these 5 plates will be computed and the heat loss through each unit individually
may be obtained from subtraction.

The Reynolds number at the trailing edge of each of these plates is computed according to:

L1 ρ u∞
ReL2 = (1)
μ

L2 ρ u∞
ReL3 = (2)
μ

and so on through ReL6 :

"Reynolds numbers"
Re_L2=rho*u_infinity*L2/mu
Re_L3=rho*u_infinity*L3/mu
Re_L4=rho*u_infinity*L4/mu
Re_L5=rho*u_infinity*L5/mu
Re_L6=rho*u_infinity*L6/mu

Because the leading edge of each of the plates discussed above start at the beginning of the
boundary layer development, the average Nusselt number over each of the plates ( Nu L2 through
Nu L6 ) can be obtained using EES’ built-in External_Flow_Plate_ND function:

"Nusselt numbers"
Call External_Flow_Plate_ND(Re_L2,Pr: Nusselt_bar_L2,C_f_L2)
Call External_Flow_Plate_ND(Re_L3,Pr: Nusselt_bar_L3,C_f_L3)
Call External_Flow_Plate_ND(Re_L4,Pr: Nusselt_bar_L4,C_f_L4)
Call External_Flow_Plate_ND(Re_L5,Pr: Nusselt_bar_L5,C_f_L5)
Call External_Flow_Plate_ND(Re_L6,Pr: Nusselt_bar_L6,C_f_L6)

The average heat transfer coefficient over each of the plates is:

Nu L2 k
hL2 = (3)
L2

Nu L3 k
hL3 = (4)
L3

and so on to hL6 :

"average heat transfer coefficients"


h_bar_L2=Nusselt_bar_L2*k/L2
h_bar_L3=Nusselt_bar_L3*k/L3
h_bar_L4=Nusselt_bar_L4*k/L4
h_bar_L5=Nusselt_bar_L5*k/L5
h_bar_L6=Nusselt_bar_L6*k/L6

The total heat transfer rate for each of the plates is computed. The total resistance to heat
transfer to the plate that includes units #1 through #5 is:

1 R′′ 1
R1to 5 = + w + (5)
hin H L6 H L6 hL6 H L6

and so the total heat transfer rate through the plate is:

q1to 5 =
(Tin − T∞ ) = H L6 (Tin − T∞ ) (6)
R1to 5 1
+ Rw′′ +
1
hin hL6

The heat transfer rate for the plate that includes units #1 through #4 is:

H L5 (Tin − T∞ )
q1to 4 = (7)
1 1
+ Rw′′ +
hin hL5

and so on through q1 :

"heat transfer rates for successively smaller regions"


q_dot_1to5=(T_in-T_infinity)*H*L6/(1/h_in+R``_w+1/h_bar_L6)
q_dot_1to4=(T_in-T_infinity)*H*L5/(1/h_in+R``_w+1/h_bar_L5)
q_dot_1to3=(T_in-T_infinity)*H*L4/(1/h_in+R``_w+1/h_bar_L4)
q_dot_1to2=(T_in-T_infinity)*H*L3/(1/h_in+R``_w+1/h_bar_L3)
q_dot_1=(T_in-T_infinity)*H*L2/(1/h_in+R``_w+1/h_bar_L2)

The heat transfer rate from unit #2 is the difference between the heat transfer rate through the
plate that includes units #1 and #2 ( q1to 2 ) and the plate that includes only unit #1 ( q1 ):

q2 = q1to 2 − q1 (8)

The heat transfer rate from unit #3 is:

q3 = q1to 3 − q1to 2 (9)

and so on through q5 :

"heat transfer rates for units of interest"


q_dot_2=q_dot_1to2-q_dot_1
q_dot_3=q_dot_1to3-q_dot_1to2
q_dot_4=q_dot_1to4-q_dot_1to3
q_dot_5=q_dot_1to5-q_dot_1to4

The heating cost for each unit is the product of its heat transfer rate, the amount of heating tim,
and the cost of energy:

Cost2 = ec time q2 (10)

Cost3 = ec time q3 (11)

through Cost5:

"heating costs for units of interest"


Cost2=ec*q_dot_2*time
Cost3=ec*q_dot_3*time
Cost4=ec*q_dot_4*time
Cost5=ec*q_dot_5*time

The solution is summarized in Table 1.

Table 1: Summary of unit-by-unit heating costs.


Unit Est. heating cost
($/year)
#2 168.0$
#3 165.4$
#4 163.9$
#5 162.8$
There is a minimal difference in the heating cost; not enough to worry about. The Reynolds
number for every unit is greater than 5x105 and therefore the flow is turbulent throughout. The
average heat transfer coefficient for the turbulent flow is not strongly dependent on position and
that is why the difference is not large).

b.) Prepare a plot showing the heating cost for unit #2 and unit #5 as a function of the wind
velocity for the range 0.5 mph to 5.0 mph. Explain any interesting characteristics that you
observe.

A Parametric Table is generated that includes the variables Cost_1 through Cost_5 and
u_infinity_mph where u_infinity_mph is used to set u_infinity in the Equation Window:

u_infinity= u_infinity_mph * convert(mile/hr,m/s) "wind velocity"

The value of the variable u_infinity_mph in the Parametric Table is varied from 0.5 mph to 5.0
mph. The heating cost for units #2 and #5 are shown in Figure 2 as a function of wind velocity.

Figure 2: Yearly heating cost for units #2 and #5 as a function of the wind velocity.

Notice that at high wind velocity it is slightly more economical to live in unit #5 because
essentially the entire face of the building is turbulent and unit #5 is further along the boundary
layer. However, at lower wind velocity (around 1.8 mile/hr), it becomes more economical to live
in unit #2 because the laminar boundary layer (with a substantially lower heat transfer
coefficient) stretches across the face of unit #2, insulating it.
Problem 4.9-3 (4-16 in text): Solar Panel
A solar photovoltaic panel is mounted on a mobile traffic sign in order to provide power without
being connected to the grid. The panel is W = 0.75 m wide by L = 0.5 m long. The wind blows
across the panel with velocity u∞ = 5 miles/hr and temperature T∞ = 90°F, as shown in Figure
P4.9-3. The back side of the panel is insulated. The panel surface has an emissivity of ε = 1.0
and radiates to surroundings at T∞. The PV panel receives a solar flux of qs′′ = 490 W/m2. The
panel produces electricity with an efficiency η (that is, the amount of electrical energy produced
by the panel is the product of the efficiency, the solar flux, and the panel area). The efficiency of
the panel is a function of surface temperature; at 20°C the efficiency is 15% and the efficiency
drops by 0.25%/K as the surface temperature increases (i.e., if the panel surface is at 40°C then
the efficiency has been reduced to 10%). All of the solar radiation absorbed by the panel and not
transformed into electrical energy must be either radiated or convected to its surroundings.
L = 0.5 m

u∞ = 5 mph
T∞ = 90°F W = 0.75 m

top view
qs′′ = 490 W/m
2

u∞ = 5 mph
T∞ = 90°F

side view
Figure P4.9-3: Solar panel for providing power to a remote traffic sign.

a.) Determine the panel surface temperature, Ts, and the amount of electrical energy generated
by the panel.

The inputs are entered in EES:

$UnitSystem SI MASS RAD PA K J


$TABSTOPS 0.2 0.4 0.6 0.8 3.5 in

L = 0.5 [m] "length of the panel"


W=0.75 [m] "width of the panel"
qf_s=490 [W/m^2] "solar flux"
T_inf=converttemp(F,K,90) "ambient temperature"
u_mph=5 [mph] "wind speed in mph"
u_inf=u_mph*convert(mph,m/s) "wind speed"
em=1.0 [-] "emissivity of panel surface"

In order to carry out the calculations required by this problem it is necessary to assume a surface
temperature, Ts.

T_s=315 [K] "guess for surface temperature"


Based on the assumed surface temperature it is possible to compute the efficiency of the panel:

η = 0.15 − (Ts − 293.2 [ K ]) 0.0025 [1/K ] (1)

and the electrical energy produced can be computed:

w e = q s′′W Lη (2)

eta=0.15 [-] - (T_s-293.2 [K])*0.0025 [-/K] "efficiency"


w_dot_e=eta*L*W*qf_s "electrical power generated"

The surface temperature is used to compute the film temperature:

Ts + T∞
T film = (3)
2

and the properties of air (ρ, μ, k, and Pr).

T_film=(T_s+T_inf)/2 "film temperature"


rho=density(Air,T=T_film, P=1 [atm]*convert(atm,Pa)) "density"
mu=viscosity(Air,T=T_film) "viscosity"
k=conductivity(Air,T=T_film) "conductivity"
Pr=Prandtl(Air,T=T_film) "Prandtl number"

The Reynolds number is computed:

ρ u∞ L
Re = (4)
μ

and used to call the procedure that implements the correlations for external flow over a flat plate;
the correlations return the average Nusselt number ( Nu ) and friction coefficient ( C f ). The
average Nusselt number is used to compute the average heat transfer coefficient:

Nu k
h= (5)
L

Re=rho*u_inf*L/mu "Reynolds number"


Call External_Flow_Plate_ND(Re,Pr: Nusselt,C_f) "call correlations"
h_bar=Nusselt*k/L "heat transfer coefficient"

An energy balance on the solar panel includes inflow from the solar flux and outflow to electrical
energy, convection, and radiation:

qs′′W L = w e + h W L (Ts , f − T∞ ) + ε σ W L (Ts4, f − T∞4 ) (6)


where Ts,f is the final calculated value of the surface temperature.

qf_s*W*L=h_bar*W*L*(T_s_f-T_inf)+sigma#*em*W*L*(T_s_f^4-T_inf^4)+w_dot_e
"energy balance on the solar panel"

which leads to Ts,f = 334.1 K. At this point, you can either manually adjust your guess value for
Ts until Ts = Ts,f or, better yet, select Update Guesses from the Calculate Menu and then
comment out your guess for Ts and specify that Ts = Ts,f.

{T_s=315 [K]} "guess for surface temperature"


T_s_f=T_s

which leads to Ts = 335.7 K and w e = 8.1 W.

b.) Prepare a plot of the electrical energy generated by the panel as a function of the solar flux
for qs′′ ranging from 100 W/m2 to 700 W/m2. Your plot should show that there is an optimal
value for the solar flux – explain this result.

The solar flux is commented out and a parametric table is generated that includes the variables
qf_s and w_dot_e. The result is illustrated in Figure 2.

Figure 2: Electrical power as a function of the solar flux.

Notice that there is a maximum in Figure 2; as the solar flux increases the power initially
increases. However, the surface temperature increases as well and so the efficiency decreases. If
the flux becomes very large then the efficiency is so poor that the electrical power will begin to
decrease.
c.) Prepare a plot of the electrical energy generated by the panel as a function of the wind
velocity (with qs′′ = 490 W/m2) for u∞ ranging from 5 mph to 50 mph – explain any
interesting aspects of your plot.

Figure 3 illustrates the electrical power produced by the panel as a function of the wind velocity.
As the wind velocity increases, the heat transfer coefficient increases and this causes the surface
temperature to be reduced. The efficiency therefore is improved and this leads to the
improvement in performance shown in Figure 3. Note that around u∞ = 40 mph the boundary
layer transitions to turbulence which leads to a sharp increase in the heat transfer coefficient and
improvement in the performance.

Figure 3: Electrical power and shear force as a function of the wind velocity.

d.) Prepare a plot of the shear force experienced by the panel due to the wind as a function of
wind velocity for u∞ ranging from 5 mph to 50 mph – explain any interesting aspects of your
plot.

The average friction coefficient is used to compute the total shear force on the panel:

u∞2
F = Cf ρ WL (7)
2

F=C_f*rho*u_inf^2*L*W/2 "frictional force"

The force is overlaid on Figure 3 as a function of the wind velocity. Note the sharp increase
when the boundary layer becomes turbulent.
Problem 4.9-4 (4-17 in text): Wind Chill Temperature
The wind chill temperature is loosely defined as the temperature that it "feels like" outside when
the wind is blowing. More precisely, the wind chill temperature is the temperature of still air
that would produce the same bare skin temperature that you experience on a windy day. If you
are alive then you are always transferring thermal energy (at rate q ) from your skin (at
temperature Tskin). On a windy day, this heat loss is resisted by a convection resistance where the
heat transfer coefficient is related to forced convection (Rconv,fc), as shown in Figure P4.9-4(a).
The skin temperature is therefore greater than the air temperature (Tair). On a still day, this heat
loss is resisted by a lower convection resistance because the heat transfer coefficient is related to
natural convection (Rconv,nc), as shown in Fig. P4.9-4(b). For a given heat loss, air temperature,
and wind velocity, the wind chill temperature (TWC) is the temperature of still air that produces
the same skin temperature.

q q
Tair Tskin TWC Tskin

Rconv,fc Rconv,nc
(a) (b)
Figure P4.9-4: Resistance network for a body losing heat on (a) a windy and (b) a still day.

It is surprisingly complicated to compute the wind chill temperature because it requires that you
know the rate at which the body is losing heat and the heat transfer coefficient between a body
and air on both a windy and still day. At the same time, the wind chill temperature is important
and controversial because it affects winter tourism in many places. The military and other
government agencies that deploy personnel in extreme climates are also very interested in the
wind chill temperature in order to establish allowable exposure limits. This problem looks at the
wind chill temperature using your heat transfer background. It has been shown that the heat
transfer coefficient for most animals can be obtained by treating them as if they were spherical
with an equivalent volume.
a.) What is the diameter of a sphere that has the same volume as a man weighing M = 170 lbm
(assume that the density of human flesh is ρf = 64 lbm/ft3)?

The inputs are entered in EES:

$UnitSystem SI MASS RAD PA K J


$TABSTOPS 0.2 0.4 0.6 0.8 3.5 in

"Inputs"
M = 170 [lbm]*convert(lbm,kg) "mass of human"
rho_f=64 [lbm/ft^3]*convert(lbm/ft^3,kg/m^3) "density of flesh"

The volume of the man is computed according to:

M
V= (1)
ρf
and the diameter of a sphere with the same volume is computed according to:

3
4 ⎛D⎞
V = π⎜ ⎟ (2)
3 ⎝2⎠

V=M/rho_f "volume of human"


V=4*pi*(D/2)^3/3 "equivalent sphere diameter"

which leads to a sphere diameter of D = 0.52 m.

b.) Assuming that the man can be treated as a sphere, compute the skin temperature for the man
on a day when the wind blows at V = 10 mph and the air temperature is Tair = 0°F. Assume
that the metabolic heat generation for the man is q = 150 W.

The additional inputs are entered in EES:

v_wind=10 [mph]*convert(mph,m/s) "wind velocity"


T_air=converttemp(F,K,0) "air temperature"
q_dot=150 [W] "metabolic heat generation"

The properties of air must be computed. The film temperature is not known and is therefore
assumed to be equal to the air temperature to start the process. The properties of air (μa, ka, ρa,
and Pra) are obtained using EES' built-in property routines.

"Air Properties"
T_film=T_air "film temperature (assumed)"
mu_a=viscosity(Air,T=T_film) "viscosity"
k_a=conductivity(Air,T=T_film) "conductivity"
Pr_a=Prandtl(Air,T=T_film) "Prandtl number"
rho_a=density(Air,T=T_film,P=1 [atm]*convert(atm,Pa)) "density"

The Reynolds number is computed according to:

ρ a DV
ReD = (3)
μa

and used to evaluate the average Nusselt number with the function External_Flow_Sphere_ND.
The average Nusselt number is used to compute the forced convection heat transfer coefficient
(hfc).

h fc D
Nu D = (4)
ka

Re_D=rho_a*v_wind*D/mu_a "Reynolds number"


Call External_Flow_Sphere_ND(Re_D,Pr_a :C_d, Nusselt) "access correlations for a sphere"
Nusselt=h_fc*D/k_a
"obtain forced convection heat transfer coefficient"

The resistance to forced convection is:

1
Rconv , fc = 2
(5)
⎛D⎞
h fc 4 π ⎜ ⎟
⎝2⎠

and so the skin temperature is:

Tskin = Tair + Rconv , fc q (6)

R_conv_fc=1/(h_fc*4*pi*(D/2)^2) "forced convection thermal resistance"


T_skin=T_air+q_dot*R_conv_fc "skin temperature"
T_skin_F=converttemp(K,F,T_skin) "skin temperature in F"

The film temperature is recomputed based on the skin temperature:

{T_film=T_air "film temperature (assumed)"}


T_film=(T_air+T_skin)/2 "film temperature"

The skin temperature is 267.5 K (21.8°F).

c.) Assume that the natural convection heat transfer coefficient that would occur on a day with
no wind is hnc = 8.0 W/m2-K. What is the wind chill temperature?

The resistance to natural convection (Rconv,nc) is:

1
Rconv ,nc = 2
(7)
⎛D⎞
hnc 4 π ⎜ ⎟
⎝2⎠

and so the wind chill temperature is:

TWC = Tskin − q Rconv ,nc (8)

h_nc=10 [W/m^2-K]
"natural convection heat transfer coefficient"
R_conv_nc=1/(h_nc*4*pi*(D/2)^2) "natural convection thermal resistance"
T_WC=T_skin-q_dot*R_conv_nc "wind chill temperature"
T_WC_F=converttemp(K,F,T_WC) "wind chill temperature in F"

The wind chill temperature on the day with Tair = 0°F and V = 10 mph is TWC = -17.4°F.

According to the National Weather Service (http://www.weather.gov/om/windchill/), the wind


chill temperature can be computed according to:
TWC = 35.74 + 0.6215 Tair − 35.75V 0.16 + 0.4275 Tair V 0.16 (9)

where Tair is the air temperature in °F and V is the wind velocity in mph.

d.) Use the National Weather Service equation to compute TWC on a day when Tair = 0°F and V =
10 mph.

Equation (9) is programmed in EES:

"from the National Weather Service"


T_WC_NWS_F=35.74 [F] +0.6215*converttemp(K,F,T_air)
-35.75[F/mph^0.16]*(v_wind*convert(m/s,mph))^0.16
+0.4275 [1/mph^0.16]*converttemp(K,F,T_air)*(v_wind*convert(m/s,mph))^0.16

The wind chill temperature predicted by the National Weather Service equation is -15.9°F.

e.) Plot the wind chill temperature on a day with Tair = 0°F as a function of the wind velocity;
show the value predicted by your model and by the National Weather Service equation for
wind velocities ranging from 5 to 30 mph.

Figure 2 illustrates the wind chill temperature as a function of wind velocity predicted by the
model and by Eq. (9) for a day when Tair = 0°F.

Figure 2: Wind chill temperature as a function of the wind velocity predicted by the model
and by the National Weather Service equation on a day when Tair = 0°F.
Problem 4.9-5 (4-18): Soldering Iron
A soldering iron tip can be approximated as a cylinder of metal with radius rout = 5.0 mm and
length L = 20 mm. The metal is carbon steel; assume that the steel has constant density ρ = 7854
kg/m3 and constant conductivity k = 50.5 W/m-K, but a specific heat capacity that varies with
temperature according to:

⎡ J ⎤ ⎡ J ⎤ ⎡ J ⎤ 2
c = 374.9 ⎢ ⎥ + 0.0992 ⎢ 2⎥
T + 3.596x10-4 ⎢ 3⎥
T
⎣ kg-K ⎦ ⎣ kg-K ⎦ ⎣ kg-K ⎦

The surface of the iron radiates and convects to surroundings that have temperature Tamb = 20°C.
Radiation and convection occur from the sides of the cylinder (the top and bottom are insulated).
The soldering iron is exposed to an air flow (across the cylinder) with a velocity V = 3.5 m/s at
Tamb and Pamb = 1 atm. The surface of the iron has an emissivity ε = 1.0. The iron is heated
electrically by ohmic dissipation; the rate at which electrical energy is added to the iron is g = 35
W.
a.) Assume that the soldering iron tip can be treated as a lumped capacitance. Develop a
numerical model using the Euler technique that can predict the temperature of the soldering
iron as a function of time after it is activated. Assume that it is activated when the tip is at
ambient temperature. Be sure to account for the fact that the heat transfer coefficient, the
radiation resistance, and the heat capacity of the soldering iron tip are all a function of the
temperature of the tip.

The inputs are entered in EES:

$UnitSystem SI MASS RAD PA K J


$TABSTOPS 0.2 0.4 0.6 0.8 3.5 in

"Inputs"
L=20 [mm]*convert(mm,m) "length"
r_out=5 [mm]*convert(mm,m) "radius"
rho=7854 [kg/m^3] "density"
k=50.5 [W/m-K] "conductivity"
e=1.0 [-] "emissivity"
g_dot=35 [W] "generation"
T_amb=converttemp(C,K,20) "ambient temperature"
P_amb=1 [atm]*convert(atm,Pa) "ambient pressure"
V=3.5 [m/s] "air velocity"

The volume and surface area (for convection and radiation) of the cylinder are:

V = π rout
2
L (1)

As = 2π rout L (2)

A_s=2*pi*r_out*L
"surface area for convection and radiation"
Vol=pi*r_out^2*L "volume"

The total simulation time, τsim, is broken into time steps of equal duration:

τ sim
Δt = (3)
( M − 1)

t j = Δt ( j − 1) for j = 1..M (4)

where M is the number of time steps used for the simulation:

tau_sim=500 [s] "simulation time"


M=101 [-] "number of time steps"
Delta_time=tau_sim/(M-1) "time step duration"
duplicate j=1,M
time[j]=Delta_time*(j-1)
end

We will focus on moving forward one step in time (from j = 1 to j = 2); once that is complete it is
easy to move through all of the time steps. The initial temperature is ambient:

T1 = Tamb (5)

T[1]=T_amb "initial temperature"

The governing differential equation for the soldering iron is obtained from an energy balance:

+ h As (T − Tamb ) + ε σ As (T 4 − Tamb )
dT
g ss = V ρ c (T ) 4
(6)
dt

and so the rate of temperature change is:

⎡ g ss − h As (T − Tamb ) − ε σ As (T 4 − Tamb )⎤⎦


dT 1
= ⎣
4
(7)
dt V ρ c (T )

In order to compute the time rate of change it is necessary to determine h , the heat transfer
coefficient. The Euler technique will compute each quantity in Eq. (7) using the value of the
temperature at the beginning of the time step (T1 for the 1st time step). The film temperature at
T1 is used to compute the properties of air that are required (ρa,1, μa,1, ka,1, and Pra,1).

Tamb + T1
T film = (8)
2

T_film[1]=(T[1]+T_amb)/2 "film temperature"


rho_a[1]=density(Air,T=T_film[1],P=P_amb) "density"
mu_a[1]=viscosity(Air,T=T_film[1]) "viscosity"
k_a[1]=conductivity(Air,T=T_film[1]) "conductivity"
Pr_a[1]=Prandtl(Air,T=T_film[1]) "Prandtl number"

The Reynolds number is computed:

V ρ a ,1 2 rout
Re1 = (9)
μa ,1

and used to call the correlation for flow over a cylinder which returns the average Nusselt
number ( Nu1 ) and drag coefficient (Cd,1):

Re[1]=rho_a[1]*V*2*r_out/mu_a[1] "Reynolds number"


Call External_Flow_Cylinder_ND(Re[1],Pr_a[1]:Nusselt[1],C_d[1]) "Access correlations"

The average heat transfer coefficient is computed using the average Nusselt number:

Nu1 ka ,1
h1 = (10)
2 rout

h[1]=Nusselt[1]*k_a[1]/(2*r_out) "heat transfer coefficient"

The specific heat capacity is computed at T1:

⎡ J ⎤ ⎡ J ⎤ ⎡ J ⎤ 2
c1 = 374.9 ⎢ ⎥ + 0.0992 ⎢ T + 3.596x10-4 ⎢
2⎥ 1 3⎥ 1
T (11)
⎣ kg-K ⎦ ⎣ kg-K ⎦ ⎣ kg-K ⎦

c[1]=374.9 [J/kg-K]+0.0992 [J/kg-K^2]*T[1]+3.596e-4 [J/kg-K^3]*T[1]^2


"specific heat capacity"

The rate of temperature change is computed at the beginning of the 1st time step, Eq. (7), and
used to compute the temperature at the end of the time step:

dT
T2 = T1 + Δt (12)
dt T1 ,t1

dTdt[1]=(g_dot-h[1]*A_s*(T[1]-T_amb)-e*sigma#*A_s*(T[1]^4-T_amb^4))/(Vol*rho*c[1])
T[2]=T[1]+dTdt[1]*Delta_time

The process of moving through all of the time steps is automated by placing the previous set of
equations within a duplicate loop and changing the index 1 to j and 2 to j+1:

"take all Euler steps"


duplicate j=1,(M-1)
T_film[j]=(T[j]+T_amb)/2 "film temperature"
rho_a[j]=density(Air,T=T_film[j],P=P_amb) "density"
mu_a[j]=viscosity(Air,T=T_film[j]) "viscosity"
k_a[j]=conductivity(Air,T=T_film[j]) "conductivity"
Pr_a[j]=Prandtl(Air,T=T_film[j]) "Prandtl number"
Re[j]=rho_a[j]*V*2*r_out/mu_a[j] "Reynolds number"
Call External_Flow_Cylinder_ND(Re[j],Pr_a[j]:Nusselt[j],C_d[j]) "Access correlations"
h[j]=Nusselt[j]*k_a[j]/(2*r_out) "heat transfer coefficient"
c[j]=374.9 [J/kg-K]+0.0992 [J/kg-K^2]*T[j]+3.596e-4 [J/kg-K^3]*T[j]^2
"specific heat capacity"
dTdt[j]=(g_dot-h[j]*A_s*(T[j]-T_amb)-e*sigma#*A_s*(T[j]^4-T_amb^4))/(Vol*rho*c[j])
T[j+1]=T[j]+dTdt[j]*Delta_time
end

b.) Plot the temperature of the soldering iron as a function of time. Make sure that your plot
covers sufficient time that your soldering iron has reached steady state.

The temperature as a function of time is shown in Figure 2.

Figure 2: Temperature as a function of time

c.) Verify that the soldering iron tip can be treated as a lumped capacitance.

The lumped capacitance model neglects the internal resistance to conduction and considers the
external resistances to radiation and convection; the assumption is therefore justified using a Biot
number:

Rcond
Bi = (13)
Rext

where
−1
⎛ 1 1 ⎞
Rext = ⎜ + ⎟ (14)
⎝ Rconv Rrad ⎠
These resistances vary throughout the process; therefore, we should compute the maximum value
of the Biot number which occurs at the end of the process when temperature is largest and
therefore both Rconv and Rrad are smallest. The minimum value of the convection resistance is
computed:

1
Rconv = (15)
MAX ( h ) As

h_max=max(h[1..M-1]) "maximum heat transfer coefficient"


R_conv_min=1/(A_s*h_max) "minimum convection resistance"

The resistance to radiation is evaluated at the maximum temperature:

1
Rrad = (16)
As ε σ ⎡ MAX (T ) + Tamb
2 ⎤
⎡ MAX (T ) + Tamb ⎤⎦
2
⎣ ⎦⎣

T_max=max(T[1..M]) "maximum temperature"


R_rad_min=1/(A_s*e*sigma#*(T_max^2+T_amb^2)*(T_max+T_amb))
"minimum value of the radiation resistance"

The total external resistance is the parallel combination of the radiation and convection
resistances, according to Eq. (14).

R_ext_min=(1/R_conv_min+1/R_rad_min)^(-1) "total external resistance"

The characteristic length for conduction is:

V
Lchar = (17)
As

and the resistance to conduction is:

Lchar
Rcond = (18)
k As

L_char=Vol/A_s "characteristic length for conduction"


R_cond=L_char/(A_s*k) "resistance to internal conduction"

The maximum value of the Biot number is therefore given by Eq. (13):

Biot_max=R_cond/R_ext_min "maximum value of the Biot number"

which leads to Bi = 0.005; therefore, the lumped capacitance model is justified.


Problem 4.9-7 (4-19 in text)
Molten metal droplets must be injected into a plasma for an extreme ultraviolet radiation source,
as shown in Figure P4.9-7.

ρ = 7054 kg/m3
c = 307 J/kg-K
D = 200 μm
Tini = 800 K
uinject = 5 m/s
atmospheric air at T∞ = 20°C
Figure P4.9-7: Injection of molten metal droplets.

The fuel droplets have a diameter of D = 200 μm and are injected at a velocity uinject = 5 m/s with
temperature Tini = 800 K. The density of the droplet ρ = 7054 kg/m3 and the specific heat
capacity is c = 307 J/kg-K. You may assume that the droplet can be treated as a lumped
capacitance. The droplet is exposed to still air at T∞ =20ºC.
a.) Develop a numerical model in EES using the Integral command that can keep track of the
velocity, temperature, and position of the droplet as a function of time.

The inputs are entered in EES:

$UnitSystem SI MASS RAD PA K J


$TABSTOPS 0.2 0.4 0.6 0.8 3.5 in

"Inputs"
D=200 [micron]*convert(micron,m) "diameter of fuel"
T_ini=800 [K] "initial temperature of fuel droplet"
c=307 [J/kg-K] "specific heat capacity of fuel"
rho=7054 [kg/m^3] "density of fuel"
T_infinity=converttemp(C,K,20 [C]) "ambient air temperature"
u_inject=5 [m/s] "injection velocity"

For this problem, the state variables include the distance of the droplet from the nozzle, x, the
velocity of the droplet, u, and T, the temperature of the droplet. Therefore, the state equations
will provide the time rate of change of these variables. In order to develop the program, the
values of the state variables are initially set to arbitrary values:

x=0 [m] "arbitary position to evaluate state equation"


T=T_ini "arbitrary temperature to evaluate state equation"
u=u_inject "arbitrary velocity to evaluate state equation"

The rate of change of the position is the velocity:

dx
=u (1)
dt

dxdt=u "derivative of position"


The state equation for temperature is obtained by carrying out an energy balance on the droplet:

dT
ρV c = h As (T∞ − T ) (2)
dt

where As and V are the surface area and volume of the droplet:

2
⎛D⎞
As = 4 π ⎜ ⎟ (3)
⎝2⎠

3
4 ⎛D⎞
V = π⎜ ⎟ (4)
3 ⎝2⎠

A_s=4*pi*(D/2)^2 "surface area"


V=4*pi*(D/2)^3/3 "volume"

The average heat transfer coefficient is obtained using the correlations that are programmed in
EES. The properties of air (ρa, μa, ca, ka, Pra) are evaluated using the built in property routines
in EES:

T_film=(T+T_infinity)/2 "film temperature for fluid property evaluation"


rho_a=density(Air,T=T_film,p=1 [atm]*convert(atm,Pa)) "density"
mu_a=viscosity(Air,T=T_film) "viscosity"
c_a=cP(Air,T=T_film) "specific heat capacity"
k_a=conductivity(Air,T=T_film) "conductivity"
Pr=mu_a*c_a/k_a "Prandtl number"

The Reynolds number is calculated:

ρa u D
ReD = (5)
μa

and the External_Flow_Sphere_ND function in EES is used to obtain the average Nusselt number
( Nu ) and the drag coefficient (Cd). The heat transfer coefficient is calculated according to:

k
h= Nu (6)
D

and the time rate of change of the temperature is computed using Eq. (2):

Re=rho_a*D*u_inject/mu_a "Reynolds number"


Call External_Flow_Sphere_ND(Re,Pr: Nusselt,C_d) "access correlations"
h=Nusselt*k_a/D "heat transfer coefficient"
rho*c*V*dTdt=h*A_s*(T_infinity-T) "derivative of temperature"
A momentum balance on the droplet leads to the state equation for velocity:

du
ρV = ρ V g − Fd (7)
dt

where g is the acceleration of gravity and Fd is the drag force. The drag force is calculated using
the drag coefficient according to:

1
Fd = Cd ρ a u 2 Ap (8)
2

where Ap is the projected area of the droplet:

2
⎛D⎞
Ap = π ⎜ ⎟ (9)
⎝2⎠

A_p=pi*(D/2)^2 "projected area"


F_d=A_p*rho_a*u^2*C_d/2 "drag force"
rho*V*dudt=rho*V*g#-F_d "derivative of velocity"

Once the state equations have been verified, the arbitrary values of the state variables are
commented out and the Integral function in EES is used to carry out the numerical integrations:
t
du
u = uinject + ∫ dt (10)
0
dt

t
dT
T = Tini + ∫ dt (11)
0
dt

t
dx
x=∫ dt (12)
0
dt

{x=0 [m] "arbitary position to evaluate state equation"


T=T_ini "arbitrary temperature to evaluate state equation"
u=u_inject "arbitrary velocity to evaluate state equation"}
t_sim=2 [s] "simulation time"
u=u_inject+Integral(dudt,time,0,t_sim)
T=T_ini+Integral(dTdt,time,0,t_sim)
x=Integral(dxdt,time,0,t_sim)

$IntegralTable time,x,u,T

b.) Plot the velocity as a function of time and the temperature as a function of time.

Figure 2 illustrates the velocity and temperature of the droplet as a function of time.
5 800

4.8
700
4.6

Droplet velocity (m/s)

Droplet Temperature (K)


600
4.4

4.2 500

4
400
T
3.8
300
3.6
u
3.4 200
0 0.1 0.2 0.3 0.4 0.5 0.6 0.7 0.8 0.9 1
Time (s)
Figure 2: Velocity and temperature as a function of time.

c.) Plot the temperature as a function of position. If the temperature of the droplet must be
greater than 500 K when it reaches the plasma then what is the maximum distance that can
separate the plasma from the injector?

Figure 3 illustrates the temperature as a function of position (distance from the injector). Figure
3 shows that the plasma and the injector cannot be separated by more than about 0.35 m.
800

700
Temperature (K)

600

500

400

300

200
0 0.1 0.2 0.3 0.4 0.5 0.6 0.7 0.8 0.9 1
Distance from injector (m)
Figure 3: Temperature of the droplet as a function of position.
Problem 4.9-9 (4-20 in text)
Figure 4-34 in your text illustrates the drag coefficient for a cylinder as a function of
Reynolds number.
a.) Using Figure 4-34 of your text, discuss briefly (1-2 sentences) why it might make
sense to add dimples to a baseball bat.

100

turbulent boundary layer


-0.5
Cd ≈Re
laminar boundary layer

that separates
that separates
10
Drag coefficient

-1
Cd ≈Re

laminar flow
1
without
separation

0.1
10-1 10 0 101 102 103 104 105 10 6
Reynolds number
Figure 4-34: Drag coefficient for a cylinder as a function of Reynolds number.

The drastic dip in the drag coefficient at very high Reynolds numbers occurs because the
boundary layer that forms on the front face of the cylinder becomes turbulent and
therefore separation is delayed. If you dimple a bat it may be possible to promote
turbulence at lower Reynolds number and therefore reduce the drag experienced by the
batter (i.e., you could move the dip to the left).

b.) Using Figure 4-34 of your text, estimate how fast you would have to be able to swing
a bat in order for it to make sense to think about adding dimples (the estimate can be
rough but should be explained well). Assume that a bat has diameter D = 0.04 m and
air has properties ρ = 1 kg/m3 and μ = 0.00002 Pa-s.

It is not likely that the transition to turbulence can be affected by more than a factor of 2
or so. Therefore, you would need to be able to swing a bat fast enough that the Reynolds
number approached Re = 100000:

ρ air V D
Re = (1)
μair

which leads to V = 50 m/s.


Problem 5.1-1: Rectangular Duct
Figure P5.1-1 illustrates flow through a rectangular duct. The height of the duct is H = 0.01 m
and the width of the duct is W = 0.05 m, as shown in Figure P5.1-1. The total length of the duct
in the flow direction is Ltotal = 1 m. The duct has two sections of equal length, L = 0.5 m. The
surface temperature of the first section of the duct is maintained at a constant temperature, Ts =
200 K. The second section of the duct is insulated and therefore you may assume that the
surface of the second section is adiabatic. Fluid enters the duct with a uniform temperature, Tin=
100 K. The mass flow rate is m = 0.25 kg/s and the properties of the fluid are ρ = 1000 kg/m3,
c = 100 J/kg-K, μ = 0.01 Pa-s, and k = 1.0 W/m-K.

Ltotal = 0.5 m
L = 0.5 m L = 0.5 m
Tin = 100 K
ρ = 1000 kg/m3
c = 100 J/kg-K y
x H = 0.01 m
k = 1 W/m-K
μ = 0.01 Pa-s
m = 0.25 kg/s the second section of the
duct (x L L) is adiabatic
the first section of the duct (x < L)
has a uniform surface temp. Ts = 200 K

W = 0.05 m H = 0.01 m

view of duct in the direction of flow


Figure P5.1-1(a): Rectangular duct.

a.) Is the flow in the duct laminar or turbulent? Justify your answer.

The character of the flow can be ascertained based on its Reynolds number. The Reynolds
number for an internal flow is based on the hydraulic diameter and mean velocity. The hydraulic
diameter is defined according to:

4 Ac
Dh = (1)
per

where Ac is the cross-sectional area of the channel and per is the wetted perimeter. For the
rectangular channel shown in Figure 1, the hydraulic diameter is:

4W H 2 0.05 m 0.01 m
Dh = = = 0.017 m (2)
2 (W + H ) ( 0.05 + 0.01) m
The mean velocity of the flow is:

m
um = (3)
Ac ρ
or

m 0.25 kg m3
um = = = 0.5 m/s (4)
WHρ s 0.05 m 0.01 m 1000 kg

The Reynolds number based on um and Dh is:

um ρ Dh 0.5 m 1000 kg 0.017 m Pa-m 2 N-s 2


Re = = = 833 (5)
μ s m3 0.01 Pa-s N kg-m

Because Re < 2300, the flow will be laminar.

b.) Estimate the hydrodynamic and thermal entry lengths (xfd,h and xfd,t) for the flow.

According to your text book, the hydrodynamic entry length is given by:

0.017 m 0.05 833


x fd , h = Dh 0.05 Re = = 0.7 m (6)

Alternatively, the hydrodynamic entry length can be estimated according to how the momentum
boundary layer grows:

x
δm ≈ 2 ν (7)
um

The flow will be developed when δm ≈ H/2:

H x
≈ 2 ν fd ,h (8)
2 um

so that:

H 2 um ρ 0.012 m 2 0.25 m 1000 kg Pa-m 2 N-s 2


x fd , h ≈ = = 0.31 m (9)
16 μ 16 s m3 0.01 Pa-s N kg-m

Either answer is acceptable.

The Prandtl number for the fluid is:


μc 0.01Pa-s 100 J m-K N kg-m W-s
Pr = = = 1.0 (10)
k kg-K 1.0 W Pa-m 2 N-s 2 J

Because the Prandtl number is 1.0, the thermal entry length will be approximately the same as
the hydrodynamic entry length.

For the remaining questions, assume that the flow becomes hydrodynamically and thermally
fully developed at exactly half-way through the heated portion of the duct (i.e., xfd,h ≈ xfd,t ≈ L/2);
note that this may or may not be the correct answer to part (b).
c.) Sketch the pressure as a function of position x in the duct.

The friction factor will be constant in the fully developed region and therefore the pressure
gradient will be constant. In the hydrodynamically developing region, the friction factor will
become larger and so will the pressure gradient. These characteristics are reflected in Figure 2.

Figure 2: Sketch of pressure as a function of position.

d.) The average friction factor in the duct is f = 0.13 ; use this value to estimate the pressure
drop across the duct (i.e., the pressure drop from x = 0 to x = Ltotal).

The pressure drop across the channel is:

1000 kg ( 0.25 ) m 0.13 1.0 m


2
ρ um2 L
2
N-s Pa-m 2
Δp = f total = = 828 Pa (11)
2 Dh 2 m3 s2 0.017 m kg-m 2 N

e.) If the roughness of the surface of the duct is increased substantially do you expect that the
pressure drop across the duct would change significantly? Justify your answer.
The characteristics of laminar flow are not strongly affected by roughness because the velocity
gradient extends across the entire channel and is not substantially perturbed by roughness near
the wall (unlike turbulent flow). Therefore, I do not expect a change in the roughness to affect
the pressure drop.

f.) The average Nusselt number in the first section of the duct (i.e., from x=0 to x= L) is
Nu = 5.2 ; use this value to estimate the average heat transfer coefficient in the first section
of the duct.

The average heat transfer coefficient is:

Nu k 5.2 1.0 W W
h= = = 312 2 (12)
Dh m-K 0.017 m m -K

g.) What is the mean temperature of the fluid leaving the first section of the duct?

An energy balance on a flow with a constant surface temperature provides:

⎛ per h L ⎞
Tout = Ts + (Tin − Ts ) exp ⎜ − ⎟ (13)
⎝ m c ⎠

or

⎡ 2(H +W ) h L ⎤
Tout = Ts + (Tin − Ts ) exp ⎢ − ⎥
⎣ m c ⎦
⎡ 2 ( 0.05 + 0.01) m-s 312 W 0.5 m-kg-K J ⎤
Tout = 200 K + (100 K − 200 K ) exp ⎢ − ⎥ (14)
⎢⎣ 0.25 kg m 2 -K 100 J s-W ⎥⎦
Tout = 153 K

For the remainder of this problem, assume that the mean temperature of the fluid leaving the 1st
section of the duct is Tout = 175 K; note that this may or may not be the correct answer to part (g).
h.) Sketch the mean temperature as a function of position x in the duct. Make sure that your
sketch extends all the way to the outlet of the duct (i.e., from x = 0 to x = Ltotal).

The mean temperature increases exponentially in the 1st section of the duct as it is exposed to the
constant surface temperature. In the 2nd section of the duct the mean temperature does not
change because heat is not being added or removed from the flow. These characteristics are
reflected in Figure 3.
Figure 3: Sketch of mean temperature as a function of position.

i.) Sketch the temperature as a function of y at the 5 x-locations in the duct that are indicated in
Figure 5.1-1(b). The x-locations correspond to the following axial positions.
1: inlet to the duct
2: half-way through the thermally developing region (i.e., x2 = xfd,t/2)
3: at the thermal entry length (i.e., x3 = xfd,t)
4: at the end of the 1st section (i.e., x4 = L)
5: at the outlet of the duct (i.e., x5 = Ltotal)

L L

1 2 3 4 5

Figure 5.1-1(b): Five locations in the duct.


Figure 4: Sketch the temperature as a function of y at various values of x.

At location 2 the thermal wave emanating from the surface of the duct will not have reached the
center; therefore, the fluid in the core of the duct will still be at Tin. At location 3, the thermal
wave has just hit the center. At location 4, the surface is at Ts and the temperature distribution is
smooth with a mean temperature of Tout, from part (g). Finally, at location 5 the fluid has
equilibrated with itself and so the temperature distribution will be nearly uniform at Tout. These
characteristics are shown in Figure 4.
Problem 5.1-2
Figure P5.1-2 illustrates flow through a pipe where the surface heat flux decreases linearly with
axial position until it reaches zero at the exit, x = L.

q ′′ decreases linearly to zero

Tin

Figure 5.1-2: Flow through a pipe with a linearly decreasing surface heat flux.

a.) Sketch the mean temperature of the fluid as a function of position, x.


The energy balance on the flow leads to:

dTm
qs′′ per = C (1)
dx

where per is the perimeter of the duct and C is the capacitance rate of the fluid. According to
Eq. (1), the gradient of the mean temperature is proportional to the heat flux. Therefore, the
mean temperature will look qualitatively as shown in Figure 2.

Temperature surface temperature of duct


Ts - Tm is small because heat flux is small

mean temperature of fluid

Position

Ts - Tm is small because h is large


Figure 2: Sketch of the mean fluid temperature and surface temperature as a function of position.

b.) Sketch the surface temperature of the duct as a function of position, x.


The surface temperature is higher than the fluid temperature according to the heat transfer
coefficient:

qs′′
Ts = Tm + (2)
h

The heat transfer coefficient will be high at the inlet and drop off to a constant value as the flow
becomes thermally developed. The sketch of the surface temperature is overlaid onto Figure 2.
Problem 5.1-3
A very long circular tube has a constant surface temperature, Ts = 100°C. During a set of
tests, fluid flows into the tube with an inlet temperature, Tin = 0°C. You have been asked
to help interpret some data from the tests that is puzzling. Figure P5.1-3 illustrates the
measured mean outlet fluid temperature, Tm,out, as a function of the mass flow rate of the
fluid (all other quantities are fixed, including the fluid properties and tube geometry).
100

90

80
Temperature (°C)

70

60

50

40

30

20

10
0.001 0.01 0.1 0.9
Mass flow rate (kg/s)
Figure P5.1-3: Measured mean outlet fluid temperature as a function of the mass flow rate through
the tube (note that all other parameters are fixed, including the fluid properties and the tube
geometry).

a.) Explain (in a few sentences) the sharp transition in Figure P5.1-3 that causes the
outlet temperature to begin to drop dramatically.

The Reynolds number is given by:

ρ D um 4 m
Re = = (1)
μ μπ D

and so, as the mass flow rate increases so does the Reynolds number. Therefore, the
sharp transition must be related to the transition from laminar to turbulent flow.

b.) Explain (in a few sentences) why the outlet temperature at low mass flow rate tends
to increase but then decreases at higher mass flow rates.

The energy equation for fluid flowing through a pipe with a constant surface temperature
is:

⎛ per L h ⎞
Tm,out = Ts − (Ts − Tm ,in ) exp ⎜ − ⎟ (2)
⎝ m c ⎠

At low mass flow the flow is laminar and therefore the Nusselt number for a long tube
will be independent of Reynolds number. Therefore, the heat transfer coefficient is not
substantially affected by the flow and, according to Eq. (2), the outlet temperature must
decrease towards Tm,in as the mass flow rate increases (the argument of the exponential
will become less negative).

At high mass flow rates, both the heat transfer coefficient and the mass flow rate
increases. These effects approximately balance and so the exit temperature is nearly
constant.
Problem 5.1-4 (5-1 in text)
Fluid flows through a tube with inner diameter D. You have been asked to help interpret some
measured data for flow through the tube. Specifically, the heat transfer coefficient, h, has been
measured as a function of the tube diameter. Figure P5.1-4 illustrates the heat transfer
coefficient measured in the thermally fully developed region of the tube as a function of the tube
diameter; note that the mass flow rate of fluid ( m ), the type of fluid, and all other aspects of the
experiment are not changed for these measurements.
Fully developed heat transfer coef. (W/m -K) 6
10
2

5
10

4
10

3
10

2
10

1
10
0 1 2 3 4 5 6 7 8 9 10
Diameter (cm)
Figure P5.1-4: Heat transfer coefficient as a function of diameter (for a constant fluid mass flow rate and
fluid type and all other aspects of the problem held constant.)

a.) Explain in a few sentences the abrupt change in the heat transfer coefficient observed that
occurs at approximately D = 5.5 cm.

The Reynolds number is given by:

ρ um D 4 m
Re = = (1)
μ π Dμ

therefore, for a constant mass flow rate, the Reynolds number increases as the diameter is
reduced. The abrupt change in heat transfer coefficient at D = 4.2 cm must be related to the
transition from laminar to turbulent flow. In laminar flow, the heat must be conducted,
approximately, to the center of the tube whereas the heat must only be conducted across the
viscous sublayer in a turbulent flow; thus, the heat transfer coefficient is much larger in turbulent
as opposed to laminar flow.

b.) Explain in a few sentences why the heat transfer is inversely proportional to diameter for
diameters above about D = 5.5 cm; that is, why is it true that h ∝ D −1 for D > 5.5 cm?

The flow for a diameter above 4.2 cm must be laminar. The heat transfer coefficient for a
laminar flow is approximately equal to:
k
h= (2)
δt

where δt is the thermal boundary layer thickness. In the thermally fully developed region, the
thermal boundary layer thickness is constant and proportional to the tube diameter; therefore:

1
h∝ (3)
D

c.) Sketch your expectation for how Figure P5.1-4 would change if the roughness of the tube
wall is increased dramatically.

The pipe wall roughness affects the heat transfer coefficient for a turbulent flow much more than
laminar flow because the critical conduction length is the viscous sublayer which can approach
the scale of roughness elements. Therefore, you should expect that the heat transfer coefficient
for diameters less than 4.2 cm will increase as the tube roughness increases (see Fig. 2) while the
tube roughness will have essentially no effect for diameters above 4.2 cm.

107
Fully developed heat transfer coef. (W/m -K)
2

106

105

104 with increased roughness

103

102

101
0 1 2 3 4 5 6 7 8 9 10
Diameter (cm)
Figure 2: Sketch of heat transfer coefficient with increased roughness.
Problem 5.1-5
Liquid metal enters a tube in a laminar flow condition. Liquid metal has a very large thermal
diffusivity (α) relative to its kinematic viscosity (ν ).
a.) Sketch the thermal and momentum boundary layers that develop in the tube; make sure that
the relative magnitude of these boundary layers is approximately correct.

Figure 1: Liquid metal with a very large α as compared to ν.

The thermal boundary layer grows like:

αx
δt = 2 (1)
u∞

while the momentum boundary layer grows like:

νx
δm = 2 (2)
u∞

therefore, the thermal boundary layer will grow much more quickly than the momentum
boundary layer, as shown in Figure 1.

b.) Do you expect the hydrodynamic or thermal entry length to be larger?

The thermal entry length (xfd,t) will be much smaller than the hydrodynamic entry length (xfd,h).
Problem 5.1-6 (5-2 in text)
Figure P5.1-6 shows the flow of a fluid with Prandtl number, Pr < 1, through a pipe.

Tin

surface heat flux


b
q′′s

0 x
Figure P5.1-6: Pipe with a surface heat flux that depends on position.

The fluid becomes thermally fully developed at location x = b. The flow of the fluid is laminar.
a.) Sketch the thermal and momentum boundary layer thickness as a function of position (δt and
δm – be sure to clearly show which is which). Label the hydrodynamic and thermal entry
lengths, xfd,t and xfd,h, in your sketch. Show the location x = b in your sketch.

Figure 2: Sketch of thermal and momentum boundary layer thickness as a function of position.

The thermal boundary layers growing from each edge of the pipe will meet at approximately x =
b when the flow becomes thermally fully developed. The momentum boundary layers will grow
more slowly since the fluid has a small Prandtl number (i.e., it is better at transferring energy
than momentum). Therefore, the flow will become hydrodynamically fully developed at x > b.
Figure 2 reflects this behavior.

b.) Sketch the local and average heat transfer coefficient, h and h , as a function of x; indicate on
your sketch the location x = b.
Figure 3: Qualitative sketch of the local and average heat transfer coefficients, h and h , respectively, as a
function of the position from the pipe inlet.

The thermal boundary layer develops from the entrance of the pipe and joins at the center at xfd,t
= b (see Figure 2). The heat transfer coefficient for a laminar flow is inversely proportional to
the boundary layer thickness and will therefore begin high and drop; when the flow becomes
fully developed the heat transfer coefficient will become constant. This is shown in Figure 3.

The average heat transfer coefficient is somewhat larger than the local heat transfer coefficient
because it is integrated from the inlet of the pipe and therefore "remembers" the regions of higher
heat transfer coefficient. This is also shown in Figure 3.

c.) Sketch the local and average friction factor, f and f , as a function of x; indicate on your
sketch the location x = b.
Figure 4: Qualitative sketch of the local and average friction factors, f and f , respectively, as a function of
the position from the pipe inlet.

The momentum boundary layer develops from the entrance of the pipe and joins at the center at
xfd,m > b (see Figure 2). The friction factor for a laminar flow is inversely proportional to the
momentum boundary layer thickness and will therefore begin high and drop; when the flow
becomes fully developed the heat transfer coefficient will become constant. This is shown in
Figure 4.

The average friction factor is somewhat larger than the local friction factor because it is
integrated from the inlet of the pipe and therefore "remembers" the regions of higher pressure
gradient. This is also shown in Figure 4.

Figure P5.1-6 shows that a non-uniform heat flux is applied to the surface of the pipe. The heat
flux decreases linearly from x = 0 to x = b and remains at 0 for all subsequent x. The fluid enters
the pipe with mean temperature, Tin.
d.) Sketch the mean temperature of the fluid as a function of position.
Figure 5: Qualitative sketch of the mean temperature of the fluid (Tm) and the surface temperature of the
pipe (Ts) as a function of the position from the pipe inlet.

A differential energy balance on the fluid leads to:

dTm
m c = qs′′ ( x ) per (1)
dx

so the rate at which the mean temperature rises will be proportional to the rate at which energy is
added by the heat flux. The mean temperature will rise quickly at x = 0 and the rate of
temperature change will drop off to zero at x = b. This is shown in Figure 5.

e.) Sketch the surface temperature of the pipe as a function of position.

The surface temperature of the pipe is greater than the mean temperature of the fluid according
to:

qs′′
Ts = Tm + (2)
h

Figure 5.1-6 shows that the heat flux will be 0 for all x > b and therefore we expect that Ts = Tm
for x > b. Figure 3 shows that the local heat transfer at the inlet to the pipe will be very high
because the thermal boundary layer is so small. Therefore, at x = 0 we expect that Ts = Tm.
Between x = 0 and x = b the temperature difference must first increase to a maximum value and
then decrease. This behavior is shown in Figure 5.
Problem 5.3-1 (5-3 in text): Portable Metabolic Heat Removal System
Dismounted soldiers and emergency response personnel are routinely exposed to high
temperature/humidity environments as well as external energy sources such as flames, motor
heat or solar radiation. The protective apparel required by chemical, laser, biological, and other
threats tend to have limited heat removal capability. These and other factors can lead to severe
heat stress. One solution is a portable, cooling system integrated with an encapsulating garment
to provide metabolic heat removal. A portable metabolic heat removal system that is acceptable
for use by a dismounted soldier or emergency response personnel must satisfy a unique set of
criteria. The key requirements for such a system is that it be extremely low mass very compact
in order to ensure that any gain in performance due to active cooling is not offset by fatigue
related to an increase in pack load. In order to allow operation for an extended period of time, a
system must either be passive (require no consumable energy source), very efficient (require
very little consumable energy), or draw energy from a high energy density power source. One
alternative for providing portable metabolic heat removal is with an ice pack, as shown in Figure
P5.3-1.

ice pack Tout = 30°C vest

Tin = 5°C
W p pump

battery
L = 2.5 m
e=0
Din = 2.5 mm
Figure P5.3-1: Schematic of a portable metabolic heat removal system that utilizes an ice pack.

The pump forces a liquid antifreeze solution to flow through plastic tubes in the vest in order to
transfer the cooling from the ice to the person. Assume that the surface of the plastic is
completely smooth, e = 0, the total length of the tube is L = 2.5 m and the inner diameter of the
tube is Din = 2.5 mm. There are Nb = 20 bends in the vest; the loss coefficient associated with
each bend is Cb = 1.0. The fluid that is being circulated through the vest has properties ρf = 1110
kg/m3, cf = 2415 J/kg-K, μf = 0.0157 Pa-s, and Prf = 151. The fluid enters the vest at Tin= 5.0°C
and leaves the vest at Tout = 30°C. You may assume that the pressure drop associated with the
vest is much greater than the pressure drop associated with any other part of the system.
a.) Assume that the bulk velocity in the tube is um = 1.0 m/s. Determine the pressure drop
required to circulate the fluid through the vest.
The inputs are entered in EES:

$UnitSystem SI MASS RAD PA K J


$TABSTOPS 0.2 0.4 0.6 0.8 3.5 in

"Inputs"
L=2.5 [m] "length of tube"
D_i=2.5 [mm]*convert(mm,m) "inner diameter of tube"
rho_f=1110 [kg/m^3] "density of fluid"
c_f=2415 [J/kg-K] "specific heat capacity of fluid"
mu_f=0.0157 [Pa-s] "viscosity of fluid"
Pr_f=151 [-] "Prandtl number of fluid"
T_out=converttemp(C,K,30) "temperature of fluid leaving vest"
T_in=converttemp(C,K,5) "temperature of fluid entering vest"
N_b=20 [-] "number of bends"
C_b=1.0 [-] "loss coefficient of the bends"
e=0 [m] "roughness of tube"

The Reynolds number in the tube is computed:

ρ f Di um
Re = (1)
μf

and used to access the correlations for internal flow in a tube (via the PipeFlow_N procedure in
EES) in order to obtain the friction factor, f. The friction factor and loss coefficient for the bends
are used to compute the pressure drop:

ρ f um2 ⎛ L ⎞
Δp = ⎜f + N b Cb ⎟ (2)
2 ⎝ Di ⎠

u_m=1.0 [m/s] "initial guess for bulk velocity"


Re=u_m*D_i*rho_f/mu_f "Reynolds number"
call PipeFlow_N(Re,Pr_f,L/D_i,e/D_i: Nusselt_T,Nusselt_H,f) "call correlation"
DeltaP=rho_f*u_m^2*(f*L/D_i+N_b*C_b)/2 "pressure drop"
DeltaP_psig=DeltaP*convert(Pa,psi) "pressure drop in psi"

which leads to Δp = 213 kPa (30.9 psi).

A miniature diaphragm pump is used to circulate the fluid. Assume that the pressure rise
produced by the pump (Δpp) varies linearly from the dead head pressure rise Δpp,dh = 30 psi at no
flow ( Vp = 0) to zero at the maximum unrestricted flow rate Vp ,open = 650 mL/min .

⎛ V ⎞
Δp p = Δp p ,dh ⎜ 1 − p ⎟
⎜ Vp ,open ⎟
⎝ ⎠
b.) Determine the fluid flow rate through the vest that is consistent with the pump curve given by
the equation above; that is, vary the value of the mean velocity, um, until the pressure drop
across the vest and flow rate through the vest falls on the pump curve.

The volume flow rate through the tube is computed:

D2
V = um π i (3)
4

V_dot=u_m*pi*D_i^2/4 "volumetric flow rate"


V_dot_mLmin=V_dot*convert(m^3/s,mL/min) "volumetric flow rate (mL/min)"
The pressure rise that can be provided by the pump at the same volume flow rate is computed
using the linear pump curve:

⎛ V ⎞
Δp p = Δp p ,dh ⎜ 1 −
⎜ Vp ,open ⎟⎟
(4)
⎝ ⎠

DeltaP_pump_dh=30 [psi]*convert(psi,Pa) "dead head pressure rise"


V_dot_open=650 [mL/min]*convert(mL/min,m^3/s) "unrestricted flow rate"
DeltaP_pump=DeltaP_pump_dh*(1-V_dot/V_dot_open)
"pump curve - pressure rise produced by pump"

The guess values for all of the variables are updated and then the assumed value of um is
commented out and the pressure drop across the vest is constrained to be equal to the pressure
rise produced by the pump.

{u_m=1.0 [m/s]} "initial guess for bulk velocity"


DeltaP=DeltaP_pump
"find u_m that is consistent with pump and system resistance curves"

which leads to V = 3.35x10-6 m3/s (201 mL/min).

c.) How much cooling is provided by the vest?

The mass flow through the vest is:

m = V ρ f (5)

and an energy balance on the vest leads to:

q = m c f (Tout − Tin ) (6)

m_dot=V_dot*rho_f "mass flow rate"


q_dot=m_dot*c_f*(T_out-T_in) "cooling provided"

which leads to q = 225 W.

d.) If the pump efficiency is ηp= 0.20 then how much power is consumed by the pump?

The pump power is computed according to:

V Δp
W p = (7)
ηp

eta_p=0.2 [-] "pump efficiency"


W_dot_pump=V_dot*DeltaP/eta_p "pump power"
which leads to W p = 2.4 W.

e.) If the system is run for time = 1 hour then what is the mass of ice that is consumed? (Assume
that the latent heat of fusion associated with melting ice is ifs = 3.33x105 J/kg and that the
only energy transfer to the ice is from the fluid.) What is the mass of batteries that are
consumed, assuming that the energy density of a lead acid battery is edb = 0.05 kW-hr/kg.

The mass of ice required is obtained from an energy balance on the ice pack:

q time = h fs M ice (8)

time=1 [hr]*convert(hr,s) "operating time"


h_fs=333e3 [J/kg] "latent heat of fusion for ice"
q_dot*time=h_fs*M_ice "mass of ice"

which leads to Mice = 2.43 kg. The mass of batteries required is obtained from an energy balance
on the battery:

W p time = edb M b (9)

ed_b=0.05 [kW-hr/kg]*convert(kW-hr/kg,J/kg) "energy density of lithium battery"


W_dot_pump*time=ed_b*M_b "mass of batteries"

which leads to Mb = 0.048 kg.


Problem 5.3-2: Beverage Dispenser
A beverage dispenser such as the one shown in Figure P5.3-2(a) provides carbonated or non-
carbonated drinks that are cooled at the time of dispense by running the fluid through stainless
steel tubes that are embedded in a cast aluminum cold plate that is cooled by ice cubes. The ice
cubes are either manually loaded into the unit or automatically produced by a top-mounted ice
cuber.

ice cuber

drink dispenser

ice dispenser
drain bin
ice cubes on cast
aluminum cold plate
Figure P5.3-2(a): An ice-cooled beverage dispenser.

The cold plate represents the heart of the beverage dispenser; it is the most expensive component
within the system and its performance determines whether the beverage dispenser will be
capable of meeting the standards for temperature/flow performance that are set by the large
distributors (e.g. Coke and Pepsi). The cold plate must provide a high rate of beverage cooling
by channeling the fluid through stainless-steel tubes that are embedded within the heavy
aluminum plate. The cold plate system is shown schematically in Figure P5.3-2(b). The melting
ice cools the aluminum to a spatially uniform temperature of Twall = 0°C. This process requires a
relatively long time (minutes) because the thermal communication between the external surface
of the cylinder and the ice is not very high; fortunately, in most convenient stores there is a long
time between customers dispensing the same beverage. When the customer activates the
dispenser, liquid at Tin = 20°C enters the tubes.
different circuits for
different beverages

out
in
tube bundle top view
ice cold plate

side view
Figure P5.3-2(b): Cold plate system schematic.
The beverage dispenser actually works as a thermal storage unit; the cylinder is cooled to the ice
temperature between dispense processes, storing “cold” or cooling potential that is transferred
very quickly to the beverage during each dispense. The relatively large aluminum cold plate is
necessary so that aluminum doesn’t change temperature much. For this problem, you may
assume that the temperature of the aluminum block does not change at all during the dispense
process and therefore the flowing fluid is exposed to a uniform wall temperature.

A V = 0.35 liter beverage must be dispensed in less than td = 10.0 s (the customer doesn’t want to
wait long to fill his cup). Assume that the beverage has the properties of water. The total length
of the stainless steel tube bundle embedded in the aluminum block is L = 1 m and the inner
diameter of the tube is Di = 3.1 mm. The roughness associated with the tube surface is e = 15
μm. There are Nb = 10, 180 degree bends in the tube bundle, each characterized by a loss
coefficient of Cb = 1.7.
a.) What is the heat transfer coefficient between the fluid and the tube wall?

The inputs are entered in EES:

$UnitSystem SI MASS RAD PA K J


$TABSTOPS 0.2 0.4 0.6 0.8 3.5 in

"Inputs"
T_wall=converttemp(C,K,0 [C]) "wall temperature"
T_in=converttemp(C,K,20 [C]) "inlet temperature"
L=1 [m] "length of tubes in bundle"
D_i=3.1 [mm]*convert(mm,m) "inner diameter of tubes in bundle"
V=0.35 [liter]*convert(liter,m^3) "volume of cup"
t_d=10.0 [s] "dispense time"
e=15 [micron]*convert(micron,m) "roughness"
N_b=10 [-] "number of bends"
C_b=1.7 [-] "loss coefficient of the bends"

The fluid properties (ρ, μ, k, c, and Pr) are calculated at the average of the inlet and wall
temperature:

T_avg=(T_wall+T_in)/2 "average water temperature"


rho=density(Water,T=T_avg,P=1 [atm]*convert(atm,Pa)) "density"
mu=viscosity(Water,T=T_avg,P=1 [atm]*convert(atm,Pa)) "viscosity"
k=conductivity(Water,T=T_avg,P=1 [atm]*convert(atm,Pa)) "conductivity"
c=cP(Water,T=T_avg,P=1 [atm]*convert(atm,Pa)) "specific heat capacity"
Pr=Prandtl(Water,T=T_avg,P=1 [atm]*convert(atm,Pa)) "Prandtl number"

The volumetric flow rate is related to the amount of fluid and dispense time:

V
V = (1)
td

The cross-sectional area of the tube is:


Di2
Ac = π (2)
4

so the mean velocity of the fluid in the tube is:

V
um = (3)
Ac

V_dot=V/t_d "volume flow rate"


A_c=pi*D_i^2/4 "cross sectional area"
u_m=V_dot/A_c "mean velocity"

The Reynolds number of the fluid is:

um ρ Di
Re = (4)
μ

The correlations for flow in a circular tube are accessed using the PipeFlow_N procedure in
order to obtain the average Nusselt number Nu and friction factor f. The average heat transfer
coefficient is obtained from the Nusselt number:

k
h = Nu (5)
Di

Re=u_m*rho*D_i/mu "Reynolds number"


call PipeFlow_N(Re,Pr,L/D_i,e/D_i: Nusselt_T,Nusselt_H,f) "access correlation"
h=Nusselt_T*k/D_i "heat transfer coefficient"

which leads to h = 20480 W/m2-K.

b.) What is the temperature of the fluid as it exits the cold plate?

The mass flow rate of the fluid is:

m = ρ V (6)

The outlet temperature of the fluid is obtained from the integrated energy balance for a tube with
a constant wall temperature:

Twall − Tout ⎛ π Di L h ⎞
= exp ⎜ − ⎟ (7)
Twall − Tin ⎝ m c ⎠

m_dot=V_dot*rho "mass flow rate"


(T_wall-T_out)=(T_wall-T_in)*exp(-pi*D_i*L*h/(m_dot*c)) "outlet temperature"
T_out_C=converttemp(K,C,T_out) "outlet temperature (C)"
which leads to Tout = 278.3 K (5.1°C).

c.) Plot the exit temperature of the fluid as a function of the dispense time for td = 2.5 s to 40 s.
Explain the characteristics that you observe in your plot.

Figure 3 illustrates the outlet temperature of the fluid as a function of the dispense time.

Figure 3: Outlet fluid temperature as a function of the dispense time.

As the dispense time increases the mass flow rate and fluid velocity tends to decrease. Reducing
the fluid velocity and mass flow rate tends to reduce the heat transfer coefficient. Notice that
initially the fluid temperature drops; this is due to the mass flow rate reduction which initially
dominates the reduction in the heat transfer coefficient. However, very quickly the fluid
temperature rises because of the reduction in the heat transfer coefficient. At approximately td =
37 s the flow transitions from turbulent to laminar which causes a very large reduction in the heat
transfer coefficient and therefore a sharp rise in the outlet temperature.

d.) What is the pressure drop of the fluid as it flows through the tube?

The pressure drop is calculated according to:

ρ um2 ⎛
L ⎞
Δp = ⎜f + N b Cb ⎟ (8)
2 ⎝ Di ⎠

DeltaP=(rho*u_m^2/2)*(f*L/D_i+N_b*C_b) "pressure drop"


DeltaP_psi=DeltaP*convert(Pa,psi) "pressure drop (psi)"

which leads to Δp = 3.13x105 Pa (45.4 psi).


Problem 5.3-3 (5-4 in text): Oxygen Fuel Tank
One concept for rapidly launching small satellites involves a rocket boosted, expendable launch
vehicle which is dropped from the cargo bay of a military cargo aircraft. The launch vehicle is
propelled by self-pressurized tanks of liquid oxygen and liquid propane. The liquid oxygen fuel
tank (referred to as the propellant tank) is at elevated pressure and must be kept full while the
aircraft sits on the runway, flies to the launch coordinates, and potentially holds position in order
to wait for a strategically appropriate launch time; the design requires that the propellant tank
remain full for timewait = 12 hours. The propellant tank contains saturated liquid oxygen at ptank =
215 psia. Saturated liquid oxygen at this pressure has a temperature of Ttank = 126.8 K. Because
the tank is so cold, it is subjected to a large heat leak, qtank . Without external cooling, it would
be necessary to vent the liquid oxygen that boils off in order to maintain the proper pressure and
therefore the tank would slowly be emptied. It is not possible to place a cryogenic refrigerator
with 20 kW of cooling capacity in the propellant tanks in order to re-liquefy the oxygen. Rather,
an adjacent dewar of liquid oxygen (referred to as the conditioning tank) is used to remove the
parasitic heat transfer and prevent any oxygen in the propellant tank from boiling away. Figure
P5.3-3 illustrates the proposed system. A pump is used to circulate liquid oxygen from the
propellant tank through a cooling coil that is immersed in the conditioning tank. The pump and
conditioning tank can be quickly removed from the launch vehicle when it is time for launch.
The conditioning tank is maintained at pct = 14.7 psia and contains saturated liquid oxygen; any
oxygen that evaporates due to the heat added by the cooling coil is allowed to escape. The
cooling coil is a coiled up tube with total length L = 10 m, inner diameter Di = 0.8 cm and outer
diameter Do = 1.0 cm. The internal surface of the tube has roughness e = 50 μm and the
conductivity of the tube material is ktube = 2.5 W/m-K. The mass flow rate provided by the pump
is m = 0.25 kg/s and the pump efficiency is ηpump= 0.45. The heat transfer coefficient associated
with the evaporation of the liquid oxygen in the conditioning tank from the external surface of
the tube is ho = 2x104 W/m2-K. You may assume that the liquid oxygen that is pumped through
the cooling coil has constant properties that are consistent with saturated liquid oxygen at the
tank pressure.

heat transfer liquid oxygen


from propane ptank = 215 psia
(tank) Ttank = 126.8 K
(3) pump
vapor boil-off from (2) pump power
conditioning tank

cooling coil liquid oxygen in


(ct)
conditioning tank
pct = 14.7 psia
Figure P5.3-3: Liquid from the propellant tank is pumped through a coil immersed in the conditioning tank.

a.) What is the pressure drop associated with forcing the liquid oxygen through the cooling coil?

The inputs are entered in EES:


$UnitSystem SI MASS RAD PA K J
$TABSTOPS 0.2 0.4 0.6 0.8 3.5 in

"Inputs"
P_tank=215 [psi]*convert(psi,Pa) "pressure of propellant tank"
L=10 [m] "length of cooling coil"
D_i=0.8 [cm]*convert(cm,m) "inner diameter of cooling coil"
e=50 [micron]*convert(micron,m) "roughness of internal surface of
cooling coil"
D_o=1.0 [cm]*convert(cm,m) "outer diameter of cooling coil"
k_tube=2.5 [W/m-K] "conductivity of the tube"
h_o=20e3 [W/m^2-K]
"heat transfer coefficient associated with evaporation on outer surface of the tube"
P_ct=14.7 [psi]*convert(psi,Pa) "conditioning tank pressure"
eta_pump=0.45 [-] "pump efficiency"
m_dot=0.25 [kg/s] "mass flow rate through cooling coil"
time_wait=12 [hr]*convert(hr,s) "hold time"

The properties of the liquid oxygen that is being pumped through the coil (ρtank, μtank, ktank, Prtank,
ctank) are computed using EES’ internal fluid property routines.

T_tank=temperature('Oxygen',P=P_tank,x=0) "temperature of oxygen"


rho_tank=density('Oxygen',P=P_tank,x=0) "density of oxygen"
mu_tank=viscosity('Oxygen',P=P_tank,x=0) "viscosity of oxygen"
k_tank=conductivity('Oxygen',P=P_tank,x=0) "conductivity of oxygen"
Pr_tank=Prandtl('Oxygen',P=P_tank,x=0) "Prandtl number of oxygen"
c_tank=cP('Oxygen',P=P_tank,x=0) "specific heat capacity of oxygen"

The volumetric flow rate of liquid oxygen is:

m
V = (1)
ρtank

The bulk velocity of the liquid oxygen in the cooling coil is:

4 V
um = (2)
π Di2

and used to compute the Reynolds number associated with the internal flow in the tube:

um Di ρtank
Re = (3)
μtank

V_dot=m_dot/rho_tank "volumetric flow rate"


u_m=V_dot/(pi*D_i^2/4) "mean velocity in cooling coil"
Re=u_m*D_i*rho_tank/mu_tank "Reynolds number"
The correlations for internal flow in a circular tube are accessed using the PipeFlow_N procedure
in order to determine the friction factor and average Nusselt number (f and Nu ). The pressure
drop is calculated from the friction factor according to:

ρtank um2 f L
Δp = (4)
2 Di

call PipeFlow_N(Re,Pr_tank,L/D_i,e/D_i: Nusselt_T,Nusselt_H,f) "access correlations"


DeltaP=rho_tank*u_m^2*f*L/(2*D_i) "pressure drop"

which leads to Δp = 5.5x105 Pa.

b.) What is the power required by the pump?

The power required by the pump is computed according to:

V Δp
W pump = (5)
η pump

W_dot_pump=V_dot*DeltaP/eta_pump "pump power"

which leads to W pump = 330 W.

c.) If all of the pump power ultimately is transferred to the liquid oxygen that is being pumped
then what is the temperature of the liquid oxygen leaving the pump (T2 in Figure P5.3-3)?

The temperature of the liquid oxygen leaving the pump is obtained from:

W pump
T2 = T1 +
m ctank
(1 − η pump ) (6)

T_2=T_tank+W_dot_pump*(1-eta_pump)/(m_dot*c_tank) "temperature of fluid leaving pump"

which leads to T2 = 127.1 K.

d.) What is the heat transfer coefficient between the liquid oxygen flowing through the cooling
coil and the internal surface of the tube?

The Nusselt number is used to compute the heat transfer coefficient:

ktank
hi = Nu (7)
Di

h_i=k_tank*Nusselt_T/D_i "internal heat transfer coefficient"


which leads to hi = 2.8x104 W/m2-K.

e.) What is the total conductance associated with the cooling coil?

The total thermal resistance between the evaporating oxygen in the conditioning tank and the
fluid in the cooling coil includes the internal resistance to convection,

1
Rconv ,i = (8)
hi Di π L

conduction through the tube,

⎛D ⎞
ln ⎜ o ⎟
= ⎝ i⎠
D
Rcond (9)
ktube 2 π L

and convection from the external surface of the tube:

1
Rconv ,o = (10)
ho Do π L

R_conv_i=1/(L*pi*D_i*h_i)
"convection resistance on internal surface"
R_cond=ln(D_o/D_i)/(2*pi*k_tube*L) "conduction resistance through tube"
R_conv_o=1/(L*pi*D_o*h_o)
"convection resistance on external surface"

The total resistance is:

Rtotal = Rconv ,i + Rtube + Rconv ,o (11)

and the total conductance is:

1
UA = (12)
Rtotal

R_total=R_conv_i+R_cond+R_conv_o "total resistance"


UA=1/R_total "total conductance"

which leads to UA = 581 W/K.


f.) What is the temperature of the liquid oxygen leaving the cooling coil (T3 in Figure P5.3-3)?

The temperature of the liquid oxygen in the conditioning tank (Tct) is computed using EES
internal thermodynamic property routine. The temperature of the liquid oxygen leaving the
conditioning tank and returned to the propellant tank is computed according to:

⎛ UA ⎞
Tct − T3 = (Tct − T2 ) exp ⎜ − ⎟ (13)
⎝ m ctank ⎠

T_ct=temperature('Oxygen',P=P_ct,x=0)
"temperature of oxygen in conditioning tank"
(T_ct-T_3)=(T_ct-T_2)*exp(-UA/(m_dot*c_tank)) "exit temperature"

which leads to T3 = 102.1 K.

g.) How much cooling is provided to the propellant tank?

The cooling provided to the propellant tank is obtained using an energy balance on the propellant
tank:

qtank = m ctank (Ttank − T3 ) (14)

q_dot_tank=m_dot*c_tank*(T_tank-T_3) "cooling provided to propellant tank"

which leads to qtank = 12600 W.

h.) Plot the cooling provided to the propellant tank and the pump power as a function of the
mass flow rate. If the parasitic heat leak to the propellant tank is qtank = 10 kW then suggest
the best mass flow rate to use for the system.

Figure 2 illustrates the cooling and pump power as a function of the mass flow rate.
Figure 2: Cooling power and pump power as a function of the mass flow rate.

The most attractive operating condition will provide 10 kW with a minimum value of the pump
power; this operating condition is shown in Figure 2 and corresponds to a mass flow rate of
approximately 0.165 kg/s.
Problem 5.3-4: Solar Electric Generating System
A solar electric generating system (SEGS) is uses parabolic solar collectors to heat oil; this
thermal energy is transferred to a power generation plant that uses a steam Rankine cycle. The
collector field shown below is located in the Mojave Desert in southern California. The
parabolic solar collectors are long, parallel rows of curved glass mirrors that focus the sun’s
energy onto absorber pipes that are located at the focal point of the mirror. A heat transfer fluid
(an oil with ρ = 825 kg/m3, μ = 0.0087 Pa-s, k = 0.134 W/m-K, Pr = 152) is pumped through the
absorber pipe. The volumetric flow rate of oil in each pipe is V = 0.012 m3/s. The inner
diameter of the absorber pipe is Di = 0.066 m.

There are a total of N = 50 parallel flow loops in the collector field; a few loops are shown in
Figure P5.3-4. In each loop, the oil is pumped from the power plant through an absorber tube
from the center to the edge of the field and then back from the edge of the field to center where
the oil finally returns to the power generation plant. Therefore, each loop consists of L = 750 m
of absorber tube length that is exposed to a constant surface heat flux due to concentrated solar
radiation. The oil enters the collector field (i.e., leaves the power generation plant) with a mean
temperature of Tin = 500 K and leaves the collector field with an elevated mean temperature.

Figure P5.3-4: Layout of collector field for SEGS plant.

The intensity of solar radiation onto the outer edge of the earth’s atmosphere has a nearly
constant value of 1370 W/m2 (it varies by about 3% during a year). The radiation that is incident
on the collector at the earth’s surface (on a sunny day) is nominally 1000 W/m2 (it is reduced by
absorption in the atmosphere and scattering). The collector concentrates the radiation so that the
heat flux that is incident on the surface of the absorber tube is qs′′ = 15,000 W/m2. You may
assume that the absorber tube is very thin and the heat flux is uniformly distributed.
To restate the problem more concisely; you are going to analyze N = 50 tubes, each with an inner
diameter of Di = 0.066 m and a length of L = 750 m. The outer diameter of the tube is essentially
equal to the inner diameter and the outer surface of these tubes is exposed to a uniform heat flux
of qs′′ = 15,000 W/m2. Each tube has oil flowing through it with a volumetric flow rate (per
tube) of V = 0.012 m3/s. The oil enters the tube with a mean temperature of Tin = 500 K.
a.) Assuming that there are no losses from the tube to the ambient (due to convection or
radiation), what is the mean temperature of the oil leaving the tube?

The inputs are entered in EES:

$UnitSystem SI MASS RAD PA K J


$TABSTOPS 0.2 0.4 0.6 0.8 3.5 in

"Inputs"
rho=825 [kg/m^3] "density of oil"
mu=0.0087 [Pa-s] "viscosity of oil"
k = 0.134 [W/m-K] "conductivity of oil"
Pr = 152 [-] "Prandtl number of oil"
V_dot=0.012 [m^3/s] "volumetric flow rate of oil"
D_i=0.066 [m] "inner diameter of pipe"
N = 50 [-] "number of loops"
L = 750 [m] "length of a single loop"
T_in = 500 [K] "mean temperature of oil entering loop"
qf_dot_s=15000 [W/m^2]
"heat flux incident on the absorber tube surface"

The mass flow rate of the oil in a single tube is:

m = ρ V (1)

An energy balance on a single absorber tube leads to:

m cTin + qs′′ π Di L = m cTout (2)

so that:

qs′′ π Di L
Tout = Tin + (3)
m c

The specific heat capacity is obtained from the Prandtl number according to:

k
c= Pr (4)
μ

m_dot=rho*V_dot "mass flow rate of oil in a single loop"


c=k*Pr/mu "specific heat capacity of oil"
T_out=T_in+qf_dot_s*pi*D_i*L/(m_dot*c) "leaving mean temperature"

which leads to Tout = 601 K.


b.) What is the total rate at which thermal energy delivered to the power plant if there are no
thermal losses from the absorber (i.e., for all of the absorber tubes)?

The total rate of thermal energy is:

qtot , plant = N m c (Tout − Tin ) (5)

q_dot_plant=N*m_dot*c*(T_out-T_in)
"total rate at which thermal energy is delivered to plant"
q_dot_plant_MW=q_dot_plant*convert(W,MW) "in MW"

which leads to qtot , plant = 117 MW.

c.) What is the pressure drop associated with pumping the oil through the collector field? You
may assume that the absorber tubes are perfectly smooth and neglect losses in the bends.

The mean velocity of the oil in a tube is:

4 m
um = (6)
ρ π Di2

and the Reynolds number is:

um ρ Di
Re = (7)
μ

The correlations for internal flow in a round tube are accessed using the procedure PipeFlow_N
in order to evaluate the average Nusselt numbers ( Nu T and Nu H ) and average friction factor
( f ). The average friction factor is used to compute the pressure drop:

ρ um2 ⎛L⎞
Δp = ⎜f ⎟ (8)
2 ⎝ Di ⎠

u_m=V_dot/(pi*D_i^2/4) "mean velocity"


Re=rho*u_m*D_i/mu "Reynolds number"
call PipeFlow_N(Re,Pr,L/D_i,0 [-]: Nusselt_T,Nusselt_H,f) "call correlations"
DeltaP=(rho*u_m^2/2)*(f*L/D_i) "pressure drop"
DeltaP_MPa=DeltaP*convert(Pa,MPa) "in MPa"

which leads to Δp = 1.48 MPa.

d.) What is the total power (i.e., for all of the absorber tubes) that is required to pump the oil
through the field? Assume that your pumps have efficiency ηpump = 0.50 and that the oil is
incompressible.
The pump power is computed according to:

N V Δp
W pump = (9)
η pump

eta_pump=0.5 [-] "pump efficiency"


W_dot_pump=DeltaP*V_dot*N/eta_pump "pump power"
W_dot_pump_MW=W_dot_pump*convert(W,MW) "in MW"

which leads to W pump = 1.77 MW.

e.) If the power plant efficiency is ηpower = 0.38 (relative to converting thermal energy to
electrical energy) then how much electrical power is produced by the plant? Don’t forget to
subtract the pump power that is consumed.

The power produced by the plant is:

W = η power qtot , plant − W pump (10)

W_dot=q_dot_plant*eta_power-W_dot_pump "power produced by plant"


W_dot_MW=W_dot*convert(W,MW) "in MW"

which leads to W = 42.6 MW.

f.) What is the maximum temperature of the surface of the absorber tube? Where does it occur?

The heat transfer coefficient (on average) between the absorber tube and the oil is:

Nu H k
h= (11)
Di

The temperature of the surface of the absorber tube at any location is given by:

qs′′
Ts = Tm + (12)
h

The maximum surface temperature must occur at the exit to the field since the mean temperature
is largest there:

q s′′
Ts ,max = Tout + (13)
h

h=Nusselt_H*k/D_i "heat transfer coefficient"


T_s_max=T_out+qf_dot_s/h "maximum absorber tube temperature"
which leads to Ts,max = 615.7 K.

g.) Estimate the average temperature of the surface of the absorber tube within the collector
field?

The average temperature of the surface of the absorber tube is:

Tin + Tout q s′′


Ts ,avg = + (14)
2 h

T_s_avg=(T_in+T_out)/2+qf_dot_s/h "average absorber tube temperature"

which leads to Ts,avg = 565.4 K.

The absorber tubes are actually vacuum insulated (like a thermos with a transparent outer shell)
in order to reduce the loss of heat to the ambient.
h.) Estimate the total amount of heat that would be lost to ambient if the absorber tubes were
NOT vacuum insulated but rather just exposed to a gentle breeze with u∞ = 5 mph on a day
when the ambient temperature is Tamb = 20°C. Assume that the absorber tube thickness is
small and neglect radiation in your calculation. Use the average tube surface temperature
that you computed in part (g) in this calculation.

The properties of the air (ρa, ka, μa, and Pra) are evaluated at the film temperature:

Ts , avg + Tamb
T film = (15)
2

using EES’ internal property routines.

u_infinity=5 [mph]*convert(mile/hr,m/s) "wind velocity"


T_amb=converttemp(C,K,20) "ambient temperature"
T_film=(T_amb+T_s_avg)/2 "film temperature"
P_amb=1 [atm]*convert(atm,Pa) "ambient pressure"
rho_a=density(Air,T=T_film,P=P_amb) "density of air"
mu_a=viscosity(Air,T=T_film) "viscosity of air"
k_a=conductivity(Air,T=T_film) "conductivity of air"
Pr_a=Prandtl(Air,T=T_film) "Prandtl number of air"

The Reynolds number for the external flow is computed:

ρ a u∞ Di
Reext = (16)
μa

and the procedure External_Flow_Cylinder_ND is used to evaluate the average Nusselt number
( Nu ext ) and drag coefficient (Cd) using the correlations for external flow over a cylinder. The
average Nusselt number is used to evaluate the heat transfer coefficient between the surface of
the tube and the surrounding air:

Nu ext ka
hext = (17)
Di

The thermal loss from the tubes is therefore:

qloss = hext π Di L N (Ts ,avg − Tamb ) (18)

Re_ext=rho_a*u_infinity*D_i/mu_a "Reynolds number for external flow"


Call External_Flow_Cylinder_ND(Re_ext,Pr_a:Nusselt_ext,C_d) "call correlations"
h_ext=Nusselt_ext*k_a/D_i "external flow heat transfer coefficient"
q_dot_loss=pi*D_i*L*N*h_ext*(T_s_avg-T_amb) "thermal loss from tubes"
q_dot_loss_MW=q_dot_loss*convert(W,MW) "in MW"

which leads to qloss = 41.1 MW.

i.) On the same day (i.e., one with a 5 mph breeze), estimate the force per unit meter that is
exerted on the absorber tube due to wind drag?

The force per unit length is computed using the drag coefficient:

Fd ρ a u∞2
= Di (19)
L 2

F_d=(rho_a*u_infinity^2/2)*D_i "force per unit length"

which leads to Fd/L = 0.13 N/m.

j.) What would the electrical output of the plant be considering the thermal losses that you
calculated in (h)?

The power produced by the plant including the losses is:

Ww / loss = η power ( qtot , plant − qloss ) − W pump (20)

W_dot_wloss=(q_dot_plant-q_dot_loss)*eta_power-W_dot_pump
"power produced by plant including losses"
W_dot_wloss_MW=W_dot_wloss*convert(W,MW) "in MW"

which leads to Ww / loss = 26.9 MW.


Problem 5.3-5 (5-5 in text): Water source heat pump
Your father-in-law has asked for your help with the design and installation of a water source heat
pump in his cabin in northern Wisconsin, as shown in Figure P5.3-5. In particular, he would like
you to analyze the system and possibly optimize the design. During the cooling season, the
water source heat pump is, essentially, an air conditioner that rejects heat to a water source rather
than to air. The cabin is located next to a lake and therefore you intend to reject heat by running
a cheap plastic tube through the lake. Currently, your father-in-law has selected a tube with an
outer diameter, Dout = 0.50 inch and a wall thickness th = 0.065 inch. You measure the
temperature of the water in the lake to be Tlake = 50°F and estimate that the heat transfer
coefficient between the external surface of the pipe and the water is ho = 450 W/m2-K. The
conductivity of the tube material is ktube= 1.5 W/m-K.

pump
ηpump = 0.6
V = 4 gal/min
LWT qrej = 30, 000 Btu/hr
qcool
w pump
Tair = 70°F
Tin EWT w hp
Tlake = 50°F
ho = 450 W/m -K
2

L = 100 ft
Dout = 0.5 inch
th = 0.065 inch
Figure P5.3-5: Water source heat pump rejecting heat to a lake.

The manufacturer's sheet for the particular heat pump that has been purchased lists many
characteristics of the heat pump as a function of the entering water temperature, EWT. The
manufacturer recommends a fixed flow rate of water through the pipe of V = 4.0 gal/min, and so
you have found an appropriate fixed displacement pump to provide this constant volumetric flow
rate of water; the pump has an efficiency, ηpump = 0.60. The data from the manufacturer’s sheet
have been used to correlate the heat pump power consumption as a function of the entering water
temperature according to:
⎡ kW ⎤ ⎡ kW ⎤
w [ kW ] = 0.8513 [ kW ] + 1.347x10-3 ⎢ EWT [°F] + 9.901x10-5 ⎢ 2 ⎥ ( EWT [°F])
2

⎣ °F ⎦ ⎣ °F ⎦

You have been asked to determine the length of tube, L, that should be run through the lake in
order to maximize the efficiency of the system (defined as the coefficient of performance, COP,
which is the ratio of the cooling provided to the power consumed by both the pump and the heat
pump). This is not a straightforward problem because it is difficult to see where to start. We'll
tackle it in small steps as discussed here. We'll start by making a couple of assumptions that will
eventually be relaxed; the assumptions are just to get the solution going - it is easier to
accomplish a meaningful analysis when you have a working model. Assume that the leaving
water temperature is LWT = 40°C and that the length of the tube is L = 100 ft.
a.) Calculate the pressure drop required to push the water through the tube in the lake.
The known information and assumptions are entered in EES:

$UnitSystem SI MASS RAD PA K J


$TABSTOPS 0.2 0.4 0.6 0.8 3.5 in

"Inputs"
T_lake=converttemp(F,K,50 [F]) "pond temperature"
eta_pump=0.6 "pump efficiency"
h_o=450 [W/m^2-K] "pond to tube heat transfer coefficient"
k_tube=1.5 [W/m-K] "conductivity of pipe"
D_o=0.5 [inch]*convert(inch,m) "outer diameter of pipe"
th=0.065 [inch]*convert(inch,m) "pipe wall thickness"
e=0.05 [mm]*convert(mm,m) "pipe wall roughness"
T_air=converttemp(F,K,70 [F]) "indoor air temperature"
V_dot=4 [gal/min]*convert(gal/min,m^3/s) "water mass flow rate"
q_dot_rej=30 [MBtu/hr]*convert(MBtu/hr,W) "heat rejection"

"Two assumptions to get started"


T_LWT=converttemp(C,K,40) "leaving water temperature-assumed"
L=100 [ft]*convert(ft,m) "length of pipe"

The water properties (ρ, k, μ, c, α, ν, and Pr) are evaluated using EES' internal property
functions at the average of the lake and the leaving water temperature:

TLWT + Tlake
T film = (1)
2

"Water Properties"
T_film=(T_lake+T_LWT)/2
"assume a film temperature to get properties"
rho=density(Water,T=T_film,P=1 [atm]*convert(atm,Pa)) "density"
k=conductivity(Water,T=T_film,P=1 [atm]*convert(atm,Pa)) "conductivity"
mu=viscosity(Water,T=T_film,P=1 [atm]*convert(atm,Pa)) "viscosity"
c=cP(Water,T=T_film,P=1 [atm]*convert(atm,Pa)) "cP"
alpha=k/(rho*c) "thermal diffusivity"
nu=mu/rho "kinematic viscosity"
Pr=nu/alpha "Prandtl number"

The mass flow rate ( m ) is given by:

m = V ρ (2)

The mean velocity of the water in the tube (um) is:

m
um = (3)
ρ Ac

where Ac is the cross-sectional area for flow within the tube:


π Di2
Ac = (4)
4

and Di is the inner diameter of the tube:

Di = Do − 2 th (5)

The Reynolds number (Re) is:

um ρ Di
Re = (6)
μ

m_dot=V_dot*rho "mass flow rate of water"


D_i=D_o-2*th "inner diameter"
A_c=pi*D_i^2/4 "cross-sectional area"
u_m=m_dot/(rho*A_c) "bulk velocity"
Re=u_m*rho*D_i/mu "Reynolds number"

The internal function PipeFlow_N is used to access the built-in correlations for flow within a
round tube. The pressure drop across the tube (Δp) is computed using the friction factor
according to:

L um2
Δp = f ρ (7)
Di 2

Assuming that the water is incompressible, the minimum power requirement by a perfect (i.e.,
reversible and adiabatic) pump is the product of the volumetric flow rate and the pressure drop.
The actual power required ( w pump ) is larger according to:

V Δp
w = (8)
η pump

call PipeFlow_N(Re,Pr,L/D_i,e/D_i: Nusselt_T,Nusselt_H,f) "correlation for Nusselt and f"


DeltaP=f*(L/D_i)*rho*u_m^2/2 "pressure drop"
w_dot_pump=DeltaP*V_dot/eta_pump "pump power"

The predicted pump power is 300 W.

b.) Predict the temperature of the water leaving the pump (Tin in Figure P5.3-5); assume all of
the pump energy goes into the water.

An energy balance on the pump leads to:

w
Tin = TLWT + (9)
m c
T_in=T_LWT+w_dot_pump/(m_dot*c) "temperature of water entering lake"

The temperature rise across the pump is predicted to be about 0.3°C.

c.) Predict the temperature of the water leaving the lake and entering the heat pump (EWT in
Figure P5.3-5) by considering the heat transfer coefficient associated with the flow of water
in the tube and the energy balance for this flow.

The heat transfer for the flow within the tube (hi) is calculated using the Nusselt number for a
constant wall temperature, returned from the function PipeFlow_N:

NuT k
hi = (10)
Di

The thermal resistance that separates the lake water from the heat pump water is composed to
convection between the flowing water and the internal surface of the tube:

1
Rconv ,i = , (11)
π Di L hi

conduction through the tube:

⎛D ⎞
ln ⎜ i ⎟
= ⎝ o⎠ ,
D
Rcond (12)
2 π L ktube

and convection from the external surface of the tube and the lake water:

1
Rconv ,o = , (13)
π Do L ho

The total conductance that characterizes the thermal communication between the lake water and
the water in the tube is:

1
= Rconv ,i + Rcond + Rconv ,o (14)
UA

The energy balance for a constant wall temperature situation, applied to this problem, leads to:

⎛ UA ⎞
TEWT = Tlake − (Tlake − Tin ) exp ⎜ − ⎟ (15)
⎝ m c ⎠
h_i=Nusselt_T*k/D_i "internal heat transfer coefficient"
R_conv_i=1/(h_i*pi*D_i*L) "resistance to internal convection"
R_conv_o=1/(h_o*pi*D_o*L) "resistance to external convection"
R_cond=ln(D_o/D_i)/(2*pi*k_tube*L) "pipe conductivity"
1/UA=R_conv_i+R_cond+R_conv_o "total conductance"
T_EWT=T_lake-(T_lake-T_in)*exp(-UA/(m_dot*c)) "constant temperature energy balance"
T_EWT_F=converttemp(K,F,T_EWT) "entering water temperature, F"

The entering water temperature predicted by the model is 89.4° F.

d.) Using your model, adjust the leaving water temperature (that you initially assumed to be
40°C) until the heat rejected to the water is equal to the heat rejection required by the heat
pump (i.e., qrej = 30x103 Btu/hr ).

The guess values are updated before this operation. Then, the initial assumption is commented
out and the heat rejection across the heat pump is calculated from an energy balance on the
water:

qrej = m c (TLWT − TEWT ) (16)

{T_LWT=converttemp(C,K,40) "leaving water temperature-assumed"}


q_dot_rej=m_dot*c*(T_LWT-T_EWT) "heat rejection"

The calculated entering water temperature is 90.6° F.

e.) Using the manufacturer's data provided by the curve fit, calculate the power required by the
heat pump and, from that, the cooling provided to the cabin and the total COP (including
both the heat pump and the pump).

The correlation in Eq. Error! Reference source not found. is used to compute the power
consumption (note that the power consumption is provided in kW and the entering water
temperature must be provided in ° F).

w_dot*convert(W,kW)=0.85125564 [kW] + 0.0013466265[kW/F]*converttemp(K,F,T_EWT) +


0.000099013661[kW/F^2]*converttemp(K,F,T_EWT)^2
"power consumption based on catalog data"

An energy balance on the heat pump leads to:

qcool = qrej − w hp (17)

The total COP is therefore:

qcool
COP = (18)
w hp + w pump

The total cooling provided to the cabin for 100 ft of tube is 7010 W at a COP of 3.36.
f.) Use your model to prepare a single plot that shows how the COP and cooling capacity vary
with length of tube. You should see an optimal length of tube that maximizes the COP;
explain why this optimal value exists.

A parametric table is generated that includes the variables L, w_dot, w_dot_pump, q_dot_cool,
and COP. The length of tube is varied from 20 m to 100 m and the cooling capacity and COP
are shown in Figure 2 as a function of the length of tube.

Figure 2: Total COP and cooling capacity as a function of the length of tube.

Note that COP is optimized when a length of tube of approximately 50 m is used. Figure 3
illustrates the power consumed by the pump, the heat pump, and the total system as a function of
the tube length.
Figure 3: Pump power, heat pump power, and total power consumed as a function of the length of tube.

Notice that for small values of L, the pump power is small but the heat pump power is large
because the tube does not do a good job of transferring heat and so the entering water
temperature increases (see Figure 3). At large values of L the situation reverses, the tube is
restrictive and so the pump power increases but the tube has a lot of surface area and so the
entering water temperature is closer to the lake temperature.
Problem 5.3-6: Vapor cooled current leads
Figure P5.3-6(a) illustrates a vapor cooled current lead for a superconducting magnet.

adiabatic coolant current lead carries 1000 amp

Dc,o = 0.5 inch


Dc,i = 0.2 inch

L = 0.2 m

vapor x
Tf,in = 100 K

Tc,x=0 = 100 K
liquid nitrogen
Figure P5.3-6(a): Vapor cooled current leads.

The current lead transfers current = 1000 amps of current to the magnet which is submerged in
liquid nitrogen. The current lead is tubular with an inner diameter of Dc,i = 0.2 inch and an outer
diameter of Dc,o = 0.5 inch. The length of the current lead is L = 0.2 m. The current lead
material has conductivity kc = 250 W/m-K and electrical resistivity ρe,c = 1x10-8 ohm-m. The top
of the lead (x = L) is adiabatic and the bottom of the lead (x = 0) is submerged in liquid cryogen
that keeps the temperature at Tf,in = 100 K. The vapor generated by the heat transfer from the
lead to the liquid cryogen escapes through the center of the lead, providing cooling. The cryogen
is liquid nitrogen at pf = 125 kPa with an enthalpy of vaporization of hfg = 196.4 kJ/kg. The
vapor enters the current lead at x = 0 at Tf,in = 100 K. The external surface of the leads is
adiabatic.
a.) Estimate the temperature rise due to conduction radially within the lead material. Based on
your estimate, is it appropriate to treat the temperature distribution within the lead as being
one-dimensional (i.e., only a function of x and not of r)?

The inputs are entered in EES:

$UnitSystem SI MASS RAD PA K J


$Tabstops 0.2 0.4 0.6 0.8 3.5

D_c_o=0.5 [inch]*convert(inch,m) "outer diameter of current lead"


D_c_i=0.2 [inch]*convert(inch,m) "inner diameter of current lead"
L=0.2 [m] "length of current lead"
k_c=250 [W/m-K] "conductivity of current lead material"
rho_e_c=1e-8 [ohm-m] "resistivity of current lead material"
current=1000 [amp] "current carried by each lead"
F$='Nitrogen' "coolant"
T_f_in=100 [K] "inlet temperature of coolant"
h_fg=196.4 [kJ/kg]*convert(kJ/kg,J/kg) "enthalpy of vaporization"
p_f=125 [kPa]*convert(kPa,Pa) "fluid pressure"

The cross-sectional area of the current lead material is:


π ( Dc2,o − Dc2,i )
Ac ,c = (1)
4

The perimeter and cross-sectional area of the fluid passage is:

π Dc2,i
Ac = (2)
4

per = π Dc ,i (3)

A_c_c=pi*(D_c_o^2-D_c_i^2)/4 "cross-sectional area of current lead material"


A_c=pi*D_c_i^2/4 "cross-sectional area of flow passage"
per=pi*D_c_i "perimeter of flow passage"

The resistance to conduction radially through the current lead is, approximately:

⎛ D + Dc ,i ⎞
ln ⎜⎜ c ,o ⎟⎟
2 Dc ,i
Rcond ,r = ⎝ ⎠ (4)
2 π kc L

The rate of thermal energy generation due to ohmic dissipation is:

ρe ,c L current 2
qohmic = (5)
Ac ,c

The radial temperature elevation is therefore, approximately:

ΔTradial = qohmic Rcond ,r (6)

R_cond_r=ln((D_c_o+D_c_i)/2/D_c_i)/(2*pi*k_c*L) "approximate resistance to radial conduction"


q_dot_ohmic=rho_e_c*L*current^2/A_c_c "amount of energy generated by ohmic dissipation"
DT_radial=q_dot_ohmic*R_cond_r "approximate radial temperature rise"

which leads to ΔTradial = 0.033 K. Therefore, it is appropriate to treat the temperature in the
conductor as being only a function of position, x.

b.) Using a differential energy balance on the conductor material and the vapor coolant, derive
the state equations that govern this problem. For this problem, the state variables include the
fluid temperature (Tf), the conductor temperature (Tc), and the gradient of the conductor
dT
temperature ( c ). Your state equations will include the mass flow rate of coolant, m ,
dx
which is not yet specified.
A differential energy balance on the conductor material leads to:

ρ e,c current 2 d ⎛ dTc ⎞


dx = ⎜ − kc Ac ,c ⎟ dx + h per dx (Tc − T f ) (7)
Ac ,c dx ⎝ dx ⎠

where h is the heat transfer coefficient between the coolant and the conductor. Equation (7) is
dT
solved for the rate of change of c :
dx

d ⎛ dTc ⎞ ρ e,c current 2 h per


⎜ ⎟=−
dx ⎝ dx ⎠ kc Ac2,c
+
kc Ac ,c
(Tc − T f ) (8)

A differential energy balance on the coolant leads to:

h per dx (Tc − T f ) = m c f
dT f
dx (9)
dx

Equation (9) is solved for the rate of change of Tf:

(Tc − T f )
dT f h per
= (10)
dx m c f

The rate of change of the conductor temperature (Tc) is one of the state variables and therefore
the state equation is:

d (Tc ) dTc
= (11)
dx dx

⎛ dT ⎞
c.) Assume that the temperature gradient in the conductor material at x = 0 is ⎜ c ⎟ = 800
⎝ dx ⎠ x =0
K/m. The coolant temperature and conductor temperature at x = 0 are both specified to be
Tf,in. Use the Crank-Nicolson technique to integrate the state equations from x = 0 to x = L.
Do not attempt to enforce the fact that the conductor is adiabatic at x = L during this step.
Use the assumed value of the temperature gradient to compute the mass flow rate of vapor
produced by the heat leak to the liquid nitrogen. Compute the temperature dependent
properties of the coolant and the local heat transfer coefficient assuming that the correlations
for constant heat flux are valid. Your Crank-Nicolson technique should be implicit in the
temperatures but explicit in the heat transfer coefficient (i.e., the heat transfer coefficient can
be calculated at the beginning of the time step). Plot the temperature of the conductor and
the coolant as a function of position.

The initial guess for the temperature gradient is specified and used to compute the heat transfer
rate to the liquid nitrogen:
⎛ dT ⎞
q = kc Ac ,c ⎜ c ⎟ (12)
⎝ dx ⎠ x =0

The mass flow rate of vapor produced by the heat transfer is calculated according to:

q
m = (13)
h fg

dTcdx_0=800 [K/m] "initial guess for temperature gradient at x=0"


q_dot=k_c*A_c_c*dTcdx_0 "total heat transfer to coolant pool from current lead"
m_dot=q_dot/h_fg "mass flow through current lead"

The nodes are setup uniformly along the current lead. The distance between adjacent nodes is:

L
Δx = (14)
( N − 1)
where N is the number of nodes. The position of each node is given by:

xi = ( i − 1) Δx for i = 1..N (15)

"setup nodes"
N=21 [-] "number of nodes"
Dx=L/(N-1) "distance between nodes"
duplicate i=1,N
x[i]=Dx*(i-1) "position of each node"
end

The values of the state variables at node 1 are known:

T_f[1]=T_f_in
T_c[1]=T_f_in
dTcdx[1]=dTcdx_0

The integration proceeds inside a duplicate loop.

duplicate i=1,(N-1)

The properties of the coolant (μf, kf, cf, ρf, and Prf) are computed based on the temperature at the
beginning of the step using EES' internal property routines:

mu_f[i]=viscosity(F$,T=T_f[i],P=p_f) "fluid viscosity"


c_f[i]=cP(F$,T=T_f[i],P=p_f) "fluid specific heat capacity"
rho_f[i]=density(F$,T=T_f[i],P=p_f) "fluid density"
Pr_f[i]=mu_f[i]*c_f[i]/k_f[i] "fluid Prandtl number"
k_f[i]=conductivity(F$,T=T_f[i],P=p_f) "fluid conductivity"
The mean velocity is computed according to:

m
um = (16)
ρ f Ac

and used to compute the Reynolds number:

um ρ f Dc ,i
Re = (17)
μf

The correlations for the local Nusselt number are accessed using the PipeFlow_N_local procedure.
The heat transfer coefficient is computed according to:

Nu k f
h= (18)
Dc ,i

u_m[i]=m_dot/(rho_f[i]*A_c) "mean velocity"


Re[i]=u_m[i]*D_c_i*rho_f[i]/mu_f[i] "Reynolds number"
call PipeFlow_N_local(Re[i],Pr_f[i],x[i]/L,0[-]: Nusselt_T_x[i],Nusselt_H_x[i],f_x[i])
"access correlations for local Nusselt number and friction factor"
h[i]=k_f[i]*Nusselt_H_x[i]/D_c_i "heat transfer coefficient"

The state equations, Eqs. (8), (10), and (11), are used to integrate through the length step using
the Crank-Nicolson algorithm. The fluid temperature is integrated according to:

⎡⎛ dT ⎞ ⎛ dT ⎞ ⎤ Δx
T f ,i +1 = T f ,i + ⎢⎜ f ⎟ + ⎜ f ⎟ ⎥ (19)
⎣⎢⎝ dx ⎠i ⎝ dx ⎠i +1 ⎦⎥ 2

Substituting Eq. (10) into Eq. (19) leads to:

⎡ h per ⎤ Δx
T f ,i +1 = T f ,i + ⎢ ( Tc ,i − T f ,i ) +
h per
( Tc ,i +1 − T f ,i +1 ) ⎥ (20)
⎢⎣ m c f m c f ⎥⎦ 2

The conductor temperature is integrated according to:

⎡⎛ dT ⎞ ⎛ dT ⎞ ⎤ Δx
Tc ,i +1 = Tc ,i + ⎢⎜ c ⎟ + ⎜ c ⎟ ⎥ (21)
⎣⎝ dx ⎠i ⎝ dx ⎠i +1 ⎦ 2

The conductor temperature gradient is integrated according to:


⎛ dTc ⎞ ⎛ dTc ⎞ ⎡⎛ d Tc ⎞ ⎛ d Tc ⎞ ⎤ Δx
2 2

⎜ ⎟ = ⎜ + +
⎟ ⎢⎜ 2 ⎟ ⎜ 2 ⎟ ⎥ (22)
⎝ dx ⎠i +1 ⎝ dx ⎠i ⎢⎣⎝ dx ⎠i ⎝ dx ⎠i +1 ⎥⎦ 2

or, substituting Eq. (8) into Eq. (22) leads to:

⎛ dTc ⎞ ⎛ dTc ⎞
⎜ ⎟ =⎜ ⎟
⎝ dx ⎠i +1 ⎝ dx ⎠i
(23)
⎡ ρ e,c current 2 h per ρ e,c current 2 h per ⎤ Δx
+ ⎢−
kc Ac2,c
+ ( Tc , i − T f , i ) + −
kc Ac2,c
+ ( Tc , i +1 − T f , i +1 ) ⎥
⎣⎢ kc Ac ,c kc Ac ,c ⎦⎥ 2

T_f[i+1]=T_f[i]+((h[i]*per*(T_c[i]-T_f[i])/(m_dot*c_f[i]))+(h[i]*per*(T_c[i+1]-T_f[i+1])/(m_dot*c_f[i])))*Dx/2
T_c[i+1]=T_c[i]+(dTcdx[i]+dTcdx[i+1])*Dx/2
dTcdx[i+1]=dTcdx[i]+(-(rho_e_c*current^2/(k_c*A_c_c^2))+h[i]*per*(T_c[i]-T_f[i])/(k_c*A_c_c))*Dx/2&
+(-(rho_e_c*current^2/(k_c*A_c_c^2))+h[i]*per*(T_c[i+1]-T_f[i+1])/(k_c*A_c_c))*Dx/2
end

Figure P5.3-6(b) illustrates the temperature of the conductor and coolant as a function of
position.
220

200

180
Temperature (K)

160 conductor

140

coolant
120

100
0 0.025 0.05 0.075 0.1 0.125 0.15 0.175 0.2
Position (m)
Figure P5.3-6(b): Conductor and coolant temperature as a function of position.

d.) Adjust the assumed value of the temperature gradient in the conductor material at x = 0,
⎛ dTc ⎞
⎜ ⎟ , until the temperature gradient at x = L is zero (i.e., the end of the conductor is
⎝ dx ⎠ x =0
adiabatic). Plot the temperature of the coolant and the conductor as a function of position.
⎛ dT ⎞
The guess values are updated. The assumed value of ⎜ c ⎟ is commented out and in its
⎝ dx ⎠ x =0
⎛ dT ⎞
place, the value of ⎜ c ⎟ is specified.
⎝ dx ⎠ x = L

{dTcdx_0=800 [K/m]} "initial guess for temperature gradient at x=0"


dTcdx[N]=0

Figure P5.3-6(c) illustrates the temperature of the conductor and the coolant as a function of
position.
160

150

140
Temperature (K)

conductor
130

120 coolant

110

100
0 0.025 0.05 0.075 0.1 0.125 0.15 0.175 0.2
Position (m)
Figure P5.3-6(c): Conductor and coolant temperature as a function of position.

e.) Plot the heat load on the liquid nitrogen as a function of current for the vapor cooled lead.
Overlay on your plot the heat load as a function of current if the lead is not vapor cooled (the
best way to do this is to set the heat transfer coefficient to zero, preventing any thermal
communication between the coolant and the conductor).

The value of current is commented out and a parametric table is used to vary the current. The
heat load as a function of current is shown in Figure P5.3-6(d).
90

Heat load on liquid nitrogen (W)


80

70

60

50
uncooled
40

30

20 vapor cooled
10

0
0 200 400 600 800 1000 1200 1400 1600 1800 2000
Current (amp)
Figure P5.3-6(d): Heat load on the liquid nitrogen as a function of the current for vapor cooled and uncooled
leads.

The heat transfer coefficient is set to zero in order to simulate uncooled leads:

h[i]=0 {k_f[i]*Nusselt_H_x[i]/D_c_i} "heat transfer coefficient"

The heat load as a function of the current for the uncooled lead is also shown in Figure 5.3-6(d).
Problem 5.3-7 (5-6 in text): Cold plate
Figure P5.3-7 illustrates a cold plate that is used as the heat sink for an array of diodes in a power
supply.
q ′′ = 6000 W/m
2

2 Dh Dh = 0.2 cm

Wp = 8 cm
Hp = 2 cm
Figure P5.3-7: Cold plate.

The operation of the diodes provides a uniform heat flux q ′′ = 6000 W/m2 over the top surface of
the cold plate. The plate is cooled by the flow of a coolant with density ρc = 1090 kg/m3,
conductivity kc = 0.8 W/m-K, viscosity μc = 0.01 Pa-s, and specific heat capacity cc = 1500 J/kg-
K. The mass flow rate of coolant is m = 0.1 kg/s and the inlet temperature is Tc,in = 30°C. The
coolant flows along the length of the cold plate through holes that are Dh = 0.2 cm in diameter.
The length of the cold plate (in the flow direction) is Lp = 15 cm, the width is Wp = 8 cm, and the
thickness is Hp = 2 cm. The conductivity of the cold plate is kp = 650 W/m-K. The distance
between the centers of two adjacent holes is twice the hole diameter. All of the surfaces of the
cold plate that are not exposed to the heat flux are adiabatic. Your initial model should assume
that the resistance to conduction through the cold plate from the surface where the heat flux is
applied to the surface of the holes is negligible. Further, your model should assume that the
resistance to conduction along the length of the cold plate is infinite.
a.) Plot the coolant temperature and plate temperature as a function of position, x.

The inputs are entered in EES:

$UnitSystem SI MASS RAD PA K J


$TABSTOPS 0.2 0.4 0.6 0.8 3.5 in

"Inputs"
H_p=2 [cm]*convert(cm,m) "height of plate"
W_p=8 [cm]*convert(cm,m) "width of plate"
L_p=15 [cm]*convert(cm,m) "length of plate in flow direction"
D_h=0.2 [cm]*convert(cm,m) "diameter of holes"
N_h=W_p/(2*D_h) "number of holes"
m_dot=0.1 [kg/s] "mass flow rate of coolant"
T_c_in=converttemp(C,K,30 [C]) "inlet temperature"
q``=6000 [W/m^2] "heat flux"
k_p=650 [W/m-K] "plate conductivity"
rho_c=1090 [kg/m^3] "coolant density"
k_c=0.8 [W/m-K] "coolant conductivity"
mu_c=0.01 [Pa-s] "coolant viscosity"
c_c=1550 [J/kg-K] "coolant specific heat capacity"
Pr_c=mu_c*c_c/k_c "coolant Prandtl number"

With the assumptions listed in the problem statement, the energy balance becomes:
dTm
qs′′ per = m c (1)
dx

where per is the perimeter of the flow passages

per = π Dh N h (2)

and qs′′ is the heat flux at the surface of the passages.

W
qs′′ = q ′′ (3)
per

per=pi*D_h*N_h "perimeter of flow passages"


q``_s=W_p*q``/per "heat flux at surface of flow passages"

Integrating Eq. (1) once leads to:

per qs′′
Tm = Tc ,in + x (4)
m c

The fluid temperature will be evaluated at a uniformly distributed set of nodes:

xi =
( i − 1) L for i = 1..N (5)
( N − 1) p
where N is the number of nodes. The distance between adjacent nodes is:

Lp
Δx = (6)
( N − 1)
N=101 [-] "number of nodes"
Dx=L_p/(N-1) "distance between nodes"
duplicate i=1,N
x[i]=(i-1)*Dx "position of each node"
end

Equation (4) is used to evaluate the mean coolant temperature at each node:

duplicate i=1,N
T_c[i]=T_c_in+q``_s*per*x[i]/(m_dot*c_c) "coolant temperature"
end

The surface temperature is related to the mean temperature according to:


qs′′
Ts = Tm + (7)
h

where h is the local heat transfer coefficient. The local heat transfer coefficient is evaluated
using the correlations programmed in EES. The mean velocity within the passage is:

m
um = (8)
Ac ρc

where Ac is the cross-sectional area of the passages:

Dh2
Ac = N h π (9)
4

The Reynolds number is:

um Dh ρ c
Re = (10)
μc

A_c=N_h*pi*D_h^2/4 "cross-sectional area of flow passages"


u_m=m_dot/(A_c*rho_c) "bulk velocity"
Re=u_m*D_h*rho_c/mu_c "Reynolds number"

The local Nusselt number (Nu) is evaluated at each node using the PipeFlow_N_local function.
The heat transfer coefficient is computed according to:

Nu kc
h= (11)
Dh

The surface temperature is evaluated at each node using Eq. (7).

duplicate i=1,N
call PipeFlow_N_local(Re,Pr_c,x[i]/D_h,0[-]: Nusselt_T_x[i],Nusselt_H_x[i],f_x[i])
"correlation for local heat transfer coefficient"
h[i]=Nusselt_H_x[i]*k_c/D_h "local heat transfer coefficient"
T_p[i]=T_c[i]+q``_s/h[i] "plate temperature"
end

Figure 2 illustrates the mean coolant and plate temperature as a function of position.
305 surface temperature

304.5

Temperature (K)
304
mean coolant temperature

303.5

303
0 0.02 0.04 0.06 0.08 0.1 0.12 0.14 0.16
Position (m)
Figure 2: Mean coolant and plate temperature as a function of position.

b.) Plot the maximum coolant temperature and maximum plate temperature as a function of
mass flow rate for mass flow rates varying between 0.0005 kg/s and 10 kg/s. Use a log-scale
for the mass flow rate. Overlay on your plot the temperature difference between the
maximum plate temperature and maximum coolant temperature. You should see three
distinct types of behavior as you increase the mass flow rate. Explain this behavior.

The maximum coolant and plate temperatures occur at x = Lp.

T_max_p=T_p[N] "maximum plate temperature"


T_max_c=T_c[N] "maximum coolant temperature"
DT_max=T_max_p-T_c[N] "maximum plate to coolant temperature difference"

The maximum coolant temperature and maximum plate temperature as a function of mass flow
rate as well as the temperature difference at the plate exit are shown in Figure 3.
400 2.25

Plate-to-coolant temperature difference (K)


ΔT
2
380 1.75

Temperature (K) 1.5


360
1.25

Tp,x=L 1
340
0.75

320 0.5

Tc,x=L 0.25

300 0
0.0005 0.01 0.1 1 4
Mass flow rate (kg/s)
Figure 3: Maximum plate and coolant temperature and the difference between these values as a function of
mass flow rate.

At low values of m , you have fully developed laminar flow and therefore the Nusselt number at
the plate exit is constant and independent of mass flow rate. This leads to a constant ΔT for m
less than about 0.01 kg/s. For mass flow rates between approximately 0.01 kg/s and 0.7 kg/s,
you have developing laminar flow and therefore the heat transfer coefficient tends to increase,
reducing the value of ΔT. Finally for mass flow rates above about 0.7 kg/s, the flow becomes
turbulent and the value of ΔT drops dramatically.

c.) Assess the validity of neglecting the resistance to conduction through the cold plate from the
surface where the heat flux is applied to the surface of the holes for the nominal mass flow
rate ( m = 0.1 kg/s).

The resistance to conduction laterally between the cold plate surface and the holes is,
approximately:

Hp
Rcond ,lat = (12)
2 k p Lp Wp

R_cond_lat=(H_p/2)/(W_p*L_p*k_p) "approximate resistance laterally to conduction"

which leads to Rcond,lat = 0.00128 K/W. The resistance to convection at the flow passage surface
is:

1
Rconv = (13)
per h

where h is the average heat transfer coefficient, evaluated using the PipeFlow_N function in EES.
call PipeFlow_N(Re,Pr_c,L_p/D_h,0 [-]: Nusselt_bar_T,Nusselt_bar_H,f_bar) "average Nusselt number"
h_bar=Nusselt_bar_H*k_c/D_h "average heat transfer coefficient"
R_conv=1/(h_bar*per*L_p) "approximate resistance to convection"

The validity of neglecting the resistance to conduction laterally across the cold plate is examined
using a Biot number:

Rcond ,lat
Bi = (14)
Rconv

Bi=R_cond_lat/R_conv "Biot number"

which leads to Bi = 0.085; justifying this assumption.

d.) Assess the validity of assuming that the resistance to conduction along the length of the cold
plate is infinite for the nominal mass flow rate ( m = 0.1 kg/s).

The resistance to conduction along the plate is estimated according to:

Lp
Rcond ,ax = (15)
k p Ac , p

where Ac,p is the cross-sectional area of the plate:

Ac , p = H p W p − Ac (16)

A_c_p=W_p*H_p-N_h*pi*D_h^2/4 "cross-sectional area of plate"


R_cond_ax=L_p/((W_p*H_p-A_c)*k_p) "resistance axially to conduction"

The heat transferred along the plate by conduction is, approximately:

qcond ,ax ≈
(T p,x=L − Tp , x = 0 )
(17)
Rcond , ax

The appropriate dimensionless number that characterizes the importance of axial conduction is:

qcond ,ax
(18)
qc

where qc is the heat transferred to the coolant:

qc = q ′′Wp Lp (19)

q_dot_cond_ax=(T_p[N]-T_p[1])/R_cond_ax "axial conduction"


q_dot_c=q``*W_p*L_p "heat transfer"
q_dot_cond_ax\q_dot_c=q_dot_cond_ax/q_dot_c
"dimensionless number that characterizes the importance of axial conduction"

qcond ,ax
which leads to = 0.18. Therefore, axial conduction is likely somewhat important for this
qc
application.

Refine your model so that it includes the effect of conduction along the length of the cold plate.
This refined model should continue to assume that the resistance to conduction through the cold
plate from the surface where the heat flux is applied to the surface of the holes is negligible.
e.) Using differential energy balances on the plate material and the coolant, derive the state
equations that govern this problem. For this problem, the state variables include the coolant
dT
temperature (Tc), the plate temperature (Tp), and the gradient of the plate temperature ( p ).
dx

An energy balance on the fluid leads to:

h per (Tp − Tc ) = m c
dTc
(20)
dx

Notice that if axial conduction is included then it is not necessary that the heat flux at the plate
surface be equal to the heat flux applied to the fluid. An energy balance on a differential section
of the plate leads to:

d 2Tp
q ′′W p = − k p Ac , p + h per (Tp − Tc ) (21)
dx 2

According to Eqs. (20) and (21), the state equations for this problem are:

dTc h per
dx
=
m c
(Tp − Tc ) (22)

d ⎛ dTp ⎞ q ′′W p

dx ⎝ dx
⎟=− +
h per
(Tp − Tc ) (23)
⎠ k p Ac , p k p Ac , p

d ( Tp ) dTp
= (24)
dx dx

f.) Assume that the temperature of the plate material at x = 0 is Tp,x=0 = 310 K. The coolant
temperature and plate temperature gradient at x = 0 are both specified. Use the Crank-
Nicolson technique to integrate the state equations from x = 0 to x = L. Do not attempt to
enforce the fact that the plate is adiabatic at x = L during this step. Your Crank-Nicolson
technique should be implicit in the temperatures but explicit in the heat transfer coefficient
(i.e., the heat transfer coefficient can be calculated at the beginning of the length step). Plot
the temperature of the coolant and the plate as a function of position.

The state variables at x = 0 are set:

T_p_0=310 [K] "guess for the plate temperature at x=0"


T_p[1]=T_p_0
dTpdx[1]=0 [K/m] "plate is adiabatic at x=0"
T_c[1]=T_c_in "coolant temperature at x=0"

The heat transfer coefficient is evaluated at each nodal location:

A_c=N_h*pi*D_h^2/4 "cross-sectional area of flow passages"


u_m=m_dot/(A_c*rho_c) "bulk velocity"
Re=u_m*D_h*rho_c/mu_c "Reynolds number"
duplicate i=1,N
call PipeFlow_N_local(Re,Pr_c,x[i]/D_h,0[-]: Nusselt_T_x[i],Nusselt_H_x[i],f_x[i])
"correlation for local heat transfer coefficient"
h[i]=Nusselt_H_x[i]*k_c/D_h "local heat transfer coefficient"
end

The Crank-Nicolson technique is used to integrate the state equations along the plate:

⎛ dTp ⎞ ⎛ dTp ⎞ ⎛ 2 q ′′W p hi per ⎞ Δx


⎜ ⎟ =⎜ ⎟ + ⎜⎜ − + ( Tp ,i − Tc ,i ) + i
h per
( Tp ,i +1 − Tc ,i +1 ) ⎟
⎟ 2
(25)
⎝ dx ⎠i +1 ⎝ dx ⎠i ⎝ k p Ac , p k p Ac , p k p Ac , p ⎠

⎡ h per ⎤ Δx
Tc ,i +1 = Tc ,i + ⎢ i ( Tp ,i − Tc ,i ) + i +1
h per
( Tp ,i +1 − Tc ,i +1 ) ⎥ (26)
⎣ m c m c ⎦ 2

⎡⎛ dT ⎞ ⎛ dT ⎞ ⎤ Δx
Tp ,i +1 = Tp ,i + ⎢⎜ p ⎟ + ⎜ p ⎟ ⎥ (27)
⎣⎢⎝ dx ⎠i ⎝ dx ⎠i +1 ⎥⎦ 2

A_c_p=W_p*H_p-N_h*pi*D_h^2/4 "cross-sectional area of plate"


duplicate i=1,(N-1)
dTpdx[i+1]=dTpdx[i]+(-2*q``*W_p/(k_p*A_c_p)+h[i]*per*(T_p[i]-&
T_c[i])/(k_p*A_c_p)+h[i+1]*per*(T_p[i+1]-T_c[i+1])/(k_p*A_c_p))*Dx/2
T_p[i+1]=T_p[i]+(dTpdx[i]+dTpdx[i+1])*Dx/2
T_c[i+1]=T_c[i]+(h[i]*per*(T_p[i]-T_c[i])/(m_dot*c_c))*Dx/2+(h[i+1]*per*(T_p[i+1]-&
T_c[i+1])/(m_dot*c_c))*Dx/2
end

Figure 4 illustrates the plate and coolant temperature predicted by the numerical model for the
assumed value of Tp,x=0. Note that the plate temperature distribution does not satisfy the
boundary condition that the plate is adiabatic at x = L and therefore the assumed value of Tp,x=0 is
not correct.
400

390
plate temperature
380

370

Temperature (K)
360

350

340

330

320
coolant temperature
310

300
0 0.02 0.04 0.06 0.08 0.1 0.12 0.14 0.16
Position (m)
Figure 4: Plate and coolant temperature predicted by the model with Tp,x=0 = 310 K.

g.) Adjust the assumed value of the plate temperature at x = 0, Tp,x=0, until the temperature
gradient in the plate material at x = L is zero (i.e., the end of the plate is adiabatic). Overlay
on your plot from (a) the temperature of the coolant and the conductor as a function of
position.

The guess values are updated. The assumed value of Tp,x=0 is commented out and instead the
adiabatic boundary condition at x = L is enforced:

{T_p_0=310 [K]} "guess for the plate temperature at x=0"


dTpdx[N]=0

Figure 5 illustrates the surface and coolant temperature as a function of position overlaid onto the
same plot generated in (a).

surface temperature
305

304.5
Temperature (K)

neglecting axial conduction, from (a)


304 including axial conduction, from (g)

303.5

coolant temperature
303
0 0.02 0.04 0.06 0.08 0.1 0.12 0.14 0.16
Position (m)
Figure 5: Surface and coolant temperature as a function of position, from (a) and (g).
Problem 5.3-8: Point-of-use water heater
Figure P5.3-8(a) illustrates a new design for a point-of-use water heater.

khtr = 25 W/m-K L = 10 cm heater strand


A
Dhtr = 5 mm
water
V = 1.5 gpm Tout = 130°F tube
Tin = 50°F
pin = 65 psia
heater strand A Whtr = 0.3 mm section A-A
tube wall annular gap for flow
Figure P5.3-8(a): Point-of-use water heater.

Heater strands with diameter Dhtr = 5 mm are surrounded by a tube in order to form an annular
flow passage through which the water passes. Ohmic dissipation in the strand material is used to
heat the water. The width of the flow passage is Whtr = 0.3 mm. The tube can be considered to
be perfectly insulated. You can neglect axial conduction within the heater strands and the tube
and assume that the heater strand provides a uniform heat flux at its surface. The initial design
uses N = 2 of these strand/tube assemblies in parallel. Each strand is L = 10 cm long. A total
flow rate of V = 1.5 gal/min is split between the parallel flow passages. The water enters at pin =
65 psia and Tin = 50ºF and must be heated to Tout = 130ºF. The conductivity of the heater strand
is khtr = 25 W/m-K. The surface of the strand and the inner surface of the tube may be assumed
to be smooth, e = 0.
a.) Develop a model of the point-of-use water heater that can predict the pressure drop in the
water as well as the maximum temperature anywhere in the heater material.

The inputs are entered in EES:

$UnitSystem SI MASS RAD PA K J


$TABSTOPS 0.2 0.4 0.6 0.8 3.5 in

"Inputs"
L_cm=10 [cm] "length of heater in cm"
L=L_cm*convert(cm,m) "length of heater"
N=2 [-] "number of heater passages"
D_htr_mm=5 [mm] "diameter of heater strand, in mm"
D_htr=D_htr_mm*convert(mm,m) "heater diameter"
W_htr_mm=0.3 [mm] "width of heater passage in mm"
W_htr=W_htr_mm*convert(mm,m) "width of heater passage in m"
V_dot_gpm=1.5 [gal/min] "volumetric flow rate of water, in gpm"
V_dot=V_dot_gpm*convert(gal/min,m^3/s) "volumetric flow rate of water"
T_in=converttemp(F,K,50 [F]) "inlet water temperature"
T_out=converttemp(F,K,130 [F]) "outlet water temperature"
p_in=65 [psi]*convert(psi,Pa) "inlet pressure"
e=0 [micron]*convert(micron,m) "surface roughness"
k_htr=25 [W/m-K] "heater conductivity"

The properties of water (ρ, μ, c, k, and Pr) are computed at the average water temperature and
the inlet water pressure:
"water properties"
T_bar=(T_in+T_out)/2 "average water temperature"
p_bar=p_in "average water pressure"
rho=density(Water,T=T_bar,p=p_bar) "density"
mu=viscosity(Water,T=T_bar,p=p_bar) "viscosity"
c=cP(Water,T=T_bar,p=p_bar) "specific heat capacity"
k=conductivity(Water,T=T_bar,p=p_bar) "conductivity"
Pr=mu*c/k "Prandtl number"

The mass flow rate is given by:

m = ρ V (1)

The rate of heat transfer required to heat the water is obtained using an energy balance:

q = m c (Tout − Tin ) (2)

The total surface area of the heater strands is:

As = N π Dhtr L (3)

The heat flux at the surface of the heater strand is:

q
q ′′ = (4)
As

m_dot=rho*V_dot "mass flow rate"


q_dot=m_dot*c*(T_out-T_in) "heat transfer rate"
A_s=pi*D_htr*L*N "surface area of heaters"
q``_dot=q_dot/A_s "heat flux"

The total cross-sectional area of the flow passages is:

π ( Do2 − Dhtr
2
)
Ac = N (5)
4

where Do is the outer diameter of the flow passage:

Do = Dhtr + 2Whtr (6)

The total wetted perimeter of the flow passage is:

per = π ( Dhtr + Do ) (7)

Therefore, the hydraulic diameter is:


4 Ac
Dh = (8)
per

and the mean velocity is:

V
um = (9)
Ac

The Reynolds number of the flow is:

ρ um Dh
Re = (10)
μ

D_o=D_htr+2*W_htr "outer diameter of passage"


A_c=N*pi*(D_o^2-D_htr^2)/4 "cross-sectional area for flow"
per=N*pi*(D_o+D_htr) "wetted perimeter for flow"
D_h=4*A_c/per "hydraulic diameter"
u_m=V_dot/A_c "mean velocity"
Re=rho*D_h*u_m/mu "Reynolds number"

In order to calculate the total pressure drop across the water heater, it is necessary to use
correlations for the average friction factor ( f ) which are obtained using the EES procedure
AnnularFlow_N. The total pressure drop is computed according to:

L ρ um2
Δp = f (11)
Dh 2

call AnnularFlow_N(Re, Pr, L/D_h,D_htr/D_o,e/D_h: Nusselt_T_bar,Nusselt_H_bar, f_bar)


"access correlations for average quantities"
Deltap=f_bar*(L/D_h)*rho*u_m^2/2 "pressure drop"

which leads to Δp = 262.2 kPa.

The maximum surface temperature of the heater is at the exit where the fluid temperature is
highest and the heat transfer coefficient is lowest. In order to calculate the surface temperature at
the exit it is necessary to obtain the local Nusselt number (Nu) using the EES procedure
AnnularFlow_N_local. The local heat transfer coefficient is obtained according to:

Nu k
h= (12)
Dh

and the surface temperature at the exit is:


q ′′
Ts = Tout + (13)
h

call AnnularFlow_N_local(Re, Pr, L/D_h, D_htr/D_o, e/D_h: Nusselt_T,Nusselt_H, f)


"access correlations for local quantities at duct exit"
htc=Nusselt_H*k/D_h "heat transfer coefficient at duct exit"
T_htr_s_max=T_out+q``_dot/htc "heater temperature at duct exit"
T_htr_s_max_F=converttemp(K,F,T_htr_s_max) "in F"

The center of the strand will be the hottest location. The volume of the strand material is:

π Dhtr
2
Vhtr = N L (14)
4

so the volumetric rate of thermal energy generation within the strand is:

q
g ′′′ = (15)
Vhtr

Using the solutions listed in Table 1-3, the temperature at the center of the strand is:

g ′′′ Dhtr
2
Tc = Ts + (16)
16 khtr

V_htr=N*pi*D_htr^2*L/4 "volume of heater material"


g```=q_dot/V_htr "volumetric thermal energy generation"
T_htr_c_max=T_htr_s_max+g```*D_htr^2/(16*k_htr) "center temperature of heater at duct exit"
T_htr_c_max_F=converttemp(K,F,T_htr_c_max) "in F"

which leads to Tc = 707.5 K or 813.8ºF.

b.) Plot the pressure drop and the maximum temperature as a function of the water flow rate for
flow rates in the range of 0.1 gal/min < V < 5 gal/min. Explain the shape of your plots and
any distinctive features that you notice.

Figure P5.3-8(b) illustrates the maximum temperature in the strand material as a function of the
volumetric flow rate of the water.
2250

2000

1750 e=0

Temperature (°F)
1500

1250 e = 20 μm

1000

750

500

250
0 0.5 1 1.5 2 2.5 3 3.5 4 4.5 5
Volumetric flow rate (gal/min)
Figure P5.3-8(b): Maximum temperature of the strand material as a function of the volumetric flow rate of
the water with e = 0 and e = 20 μm.

As the volumetric flow rate increases, the heat transfer required increases and therefore in
general the maximum temperature increases with flow rate. At low volumetric flow rates, the
flow is laminar and therefore the heat transfer is nearly unaffected by the flow rate (there is some
small effect depending on whether the flow is thermally fully developed). At V = 0.4 gal/min,
the flow transitions to turbulent flow and therefore the heat transfer coefficient increases leading
to the sudden drop in temperature. For increasing flow rate, the heat transfer increases but so
does the heat transfer coefficient (recall that the heat transfer coefficient is affected by the
velocity in a turbulent flow); the result is that the maximum temperature increases but at a lower
rate than it did in the laminar region.

Figure P5.3-8(c) illustrates the pressure drop across the heater as a function of the volumetric
flow rate. The pressure drop increases with flow and there is a discontinuity associated with the
transition to turbulence.
6
5x10

e = 20 μm
6
10
Pressure drop (Pa)

e=0

5
10

4
10
0 0.5 1 1.5 2 2.5 3 3.5 4 4.5 5
Volumetric flow rate (gal/min)
Figure P5.3-8(c): Pressure drop as a function of the volumetric flow rate of the water with e = 0 and e = 20
μm.
c.) Overlay on your plots from (b) the results obtained if the roughness of the strand and the tube
surface is e = 20 μm. Explain the results.

Figures P5.3-8(b) and (c) illustrate the maximum temperature and pressure drop as a function of
flow rate for the smooth (e = 0) and rough (e = 20 μm) case. The effect of roughness is to
increase the heat transfer coefficient but only in the turbulent region and therefore the maximum
temperature is reduced in the turbulent region. The effect of roughness is also to increase the
pressure drop, but only in the turbulent region.

d.) The maximum allowable pressure drop across the point-of-use water heater is 30 psi and the
maximum allowable temperature of the strand material is 400ºF. With the volumetric flow
rate set to V = 1.5 gpm and the roughness set to e = 20 μm, adjust the values of L and Whtr so
that these two constraints are satisfied.

The guess values are updated and the value of Whtr is commented out. The pressure drop is
specified:

{W_htr_mm=0.3 [mm]} "width of heater passage in mm"


Deltap_max=30 [psi]*convert(psi,Pa) "maximum allowable pressure drop"
Deltap=Deltap_max "specify pressure drop"

The value of Whtr that satisfies the pressure drop constraint is 0.384 mm. The guess values are
again updated and the value of L is commented out. The maximum temperature is specified:

{L_cm=10 [cm]} "length of heater in cm"


T_htr_all_F=400 [F] "maximum allowable temperature of heater strand, in F"
T_htr_all=converttemp(F,K,T_htr_all_F) "maximum allowable temperature of heater strand"
T_htr_c_max=T_htr_all "specify maximum temperature"

The values of Whtr and L that satisfy both the pressure drop and maximum temperature
constraints are Whtr = 0.512 mm and L = 0.551 m, respectively.

e.) Plot the total length of strand material required as a function of the number of strand/tube
assemblies for 2 < N < 11. Be sure that each point on your plot continues to satisfy the
design constraints listed in (d). Explain the shape of your plot.

The total length of strand material is:

Ltot = N L (17)

A parametric table is generated that includes N and Ltot. The total length of strand material as a
function of the number of strands is shown in Figure 5.3-8(d). Adding strands causes the length
of each strand to go down and the velocity within each passage to go down. Initially, the flow is
turbulent and so changing the length has very little effect on the heat transfer coefficient or the
friction factor. Therefore, adding strands does not affect the total length of strand material
required. However, when the flow velocity is sufficiently reduced, the flow transitions to
laminar flow and as a result the total length of strand material increases. Eventually, the total
length is again reduced because the heat transfer coefficient increases with number of strands due
to the fact that the flow does not become fully developed within the channel.
1.6

Total length of strand material (m)


1.4

1.2

0.8

0.6

0.4
2 3 4 5 6 7 8 9 10 11
Number of strands
Figure P5.3-8(d): Total length of strand material as a function of the number of strands.
Problem 5.4-1 (5-7 in text): Thrust bearing
Figure P5.4-1 illustrates a simple slider bearing used to provide support against thrust loads.

ρ = 800 kg/m3
ΔH = 0.05 mm μ = 0.5 Pa-s exit
pamb = 1 atm

y
x
inlet Hmin = 0.2 mm
pamb = 1 atm up = 10 m/s

L = 5 cm
Figure P5.4-1: Thrust bearing.

The slider is a close clearance, converging gap formed between a moving surface (e.g., the
surface of a rotating shaft) and a stationary surface. The velocity of the moving surface is up =
10 m/s. The length of the gap is L = 5.0 cm and the minimum clearance in the gap is at the exit
(i.e., at x = L) is Hmin = 0.2 mm. The maximum clearance of the gap is at the inlet (i.e., at x = 0)
is Hmin + ΔH where ΔH/Hmin = 0.25. The clearance varies linearly with position according to:
H = H min + ΔH
( L − x ) . The pressure at the inlet and exit of the gap is ambient, p = 1 atm.
amb
L
The properties of the oil that flows through the gap are ρ = 800 kg/m3 and μ = 0.5 Pa-s.
a.) Is it appropriate to model the flow through the gap as inertia-free flow using the Reynolds
equation?

The inputs are entered in EES:

$UnitSystem SI MASS RAD PA K J


$Tabstops 0.2 0.4 0.6 3.5 in

"Inputs"
rho=800 [kg/m^3] "density of oil"
mu=0.5 [Pa-s] "viscosity of oil"
u_p=10 [m/s] "velocity of moving surface"
H_min_mm=0.2 [mm] "minimum clearance, in mm"
H_min=H_min_mm*convert(mm,m) "minimum clearance"
DHoverH_min=0.25 [-] "ratio of clearance change to minimum clearance"
DH=DHoverH_min*H_min "clearance change"
L=5.0 [cm]*convert(cm,m) "length of passage"
p_amb=1 [atm]*convert(atm,Pa) "ambient pressure"

The modified Reynolds number is computed using the length of the gap as the characteristic
length:

MR=(rho*u_p*H_min/mu)*(H_min/L) "modified Reynolds number"

which leads to a modified Reynolds number of 0.013; therefore, the Reynolds equation is
appropriate for this problem.
b.) Use the Reynolds equation to obtain an analytical solution for the pressure distribution within
the gap.

The expression for the clearance is entered in Maple:

> restart;
> H:=H_min+DH*(L-x)/L;
DH ( L − x )
H := H_min +
L

The Reynolds equation is modified for this steady-state problem,

d ⎛ dp 3 ⎞ dH
⎜ H ⎟ = 6 μ up (1)
dx ⎝ dx ⎠ dx

and entered in Maple,

> ODE:=diff(diff(p(x),x)*H^3,x)=6*mu*u_p*diff(H,x);
2
DH ( L − x ) ⎞
3 ⎛⎜⎜ p( x ) ⎞⎟⎟ ⎛⎜⎜ H_min +
d
3 ⎟⎟ DH
⎛d
2
⎞⎛ DH ( L − x ) ⎞ ⎝ dx ⎠ ⎝ L ⎠
ODE := ⎜⎜ 2 p( x ) ⎟⎟ ⎜⎜ H_min + ⎟⎟ −
⎝ dx ⎠⎝ L ⎠ L
6 μ u_p DH
=−
L

The solution is obtained:

> ps:=dsolve(ODE);
−6 μ u_p L 3 H_min − 6 μ u_p L 3 DH + _C1
ps := p( x ) =
2 DH ( −H_min L − DH L + DH x ) 2
6 μ u_p L 2
− + _C2
DH ( −H_min L − DH L + DH x )

The solution includes two constants of integration that must be obtained by applying the
boundary conditions at x = 0 and x = L; the pressure is ambient at each of these locations:

px =0 = pamb (2)

px = L = pamb (3)

These boundary conditions are evaluated symbolically in Maple,

> BC1:=(rhs(eval(ps,x=0))=p_amb);
−6 μ u_p L 3 H_min − 6 μ u_p L 3 DH + _C1 6 μ u_p L 2
BC1 := − + _C2 =
2 DH ( −H_min L − DH L ) 2 DH ( −H_min L − DH L )
p_amb
> BC2:=(rhs(eval(ps,x=L))=p_amb);
−6 μ u_p L 3 H_min − 6 μ u_p L 3 DH + _C1 6 μ u_p L
BC2 := + + _C2 = p_amb
2 DH H_min 2 L 2 DH H_min

and solved in order to obtain explicit expressions for C1 and C2:

> solve({BC1,BC2},{_C1,_C2});
−6 μ u_p L + 2 p_amb DH H_min + p_amb DH 2
{ _C2 = ,
DH ( 2 H_min + DH )
6 L 3 μ u_p DH ( H_min + DH )
_C1 = }
2 H_min + DH

The equations for C1 and C2 are copied and pasted into EES with the usual modification:

"constants of integration, from Maple"


C_2 = -1/DH*(6*mu*u_p*L-2*p_amb*DH*H_min-p_amb*DH^2)/(2*H_min+DH)
C_1 = 6*L^3*mu*u_p*DH*(H_min+DH)/(2*H_min+DH)

The solution for the pressure (ps in Maple) is also copied and pasted into EES:

"pressure solution, from Maple"


p = 1/2*(-6*mu*u_p*L^3*H_min-6*mu*u_p*L^3*DH+C_1)/DH/(-H_min*L-DH*L+DH*x)^2&
-6*mu*u_p*L^2/DH/(-H_min*L-DH*L+DH*x)+C_2
x_bar=x/L "dimensionless position"
Dp=p-p_amb "pressure elevation, relative to ambient"
Dp_kPa=Dp*convert(Pa,kPa) "in kPa"

The pressure (relative to ambient) is shown as a function of position in the gap in Figure 2.
Figure 2: Pressure relative to ambient as a function of position.

The pressure is elevated within the gap, providing support to the adjacent surface. This type of
thrust bearing is referred to as a hydrodynamic bearing and operates by dragging fluid through
the gap. In the inlet region of the gap (near x = 0), the clearance is large and therefore the
Couette flow (i.e., the flow dragged along by the moving surface) is large while towards the exit
of the gap (near x = L), the clearance is small and therefore the Couette flow is less. In order to
satisfy continuity (which requires that the flow everywhere in the gap be the same), an adverse
pressure gradient is setup at the inlet, creating a pressure driven flow that opposes the Couette
flow. A favorable pressure gradient is setup at the outlet (i.e., the pressure decreases in the flow
direction), creating a pressure driven flow that augments the Couette flow.

c.) Determine the force per unit width provided by the thrust bearing.

The force is obtained by integrating the pressure elevation over the surface of the thrust bearing:
L
F
= ( p − pamb ) dx
W ∫0
(4)

The integration is carried out numerically using the Integral command in EES:

F`=INTEGRAL(Dp,x,0,L) "force per unit width"

which leads to F/W = 27600 N/m.

d.) Determine an appropriate scaling relation for the force per unit width and use it to define a
non-dimensional thrust force. Plot the dimensionless thrust force as a function of the
parameter ΔH/Hmin.
Equation (5-114) in the text provides some guidance relative to the scaling relationship; for an
inertia-free flow, the scaling relationship becomes:

Δp μ um
≈ (5)
Lchar H2
N
N viscous shear
pressure gradient

which implies that the pressure elevation is, approximately:

μ um
Δp ≈ L (6)
H2

and so the force developed by the bearing will be approximately:

μ um
F≈ 2
L2 W (7)
H

Equation (7) provides a convenient definition for a dimensionless force:

F H2
F = (8)
W μ um L2

which is calculated in EES:

F_bar=F`/(mu*u_p*L^2/H_min^2) "dimensionless force"

Figure 3 illustrates the dimensionless force as a function of the bearing taper, ΔH/Hmin.
Figure 3: Dimensionless thrust force as a function of ΔH/Hmin.
Problem 5.4-2 (5-8 in text)
A very viscous fluid is pumped through a circular tube at a rate of V = 15 liter/min. The tube is
thin wall and made of metal; the thickness of the tube and its resistance to conduction can be
neglected. The tube diameter is D = 0.5 inch. The tube is covered with insulation that is thins =
0.25 inch with conductivity kins= 0.5 W/m-K. The external surface of the tube is exposed to air
at T∞ = 20°C with heat transfer coefficient h = 120 W/m2-K. The viscosity of the fluid is μ = 0.6
Pa-s and its conductivity is k = 0.15 W/m-K.
a.) Prepare an analytical solution for the radial temperature distribution within the fluid at a
location where the fluid temperature is not changing in the x-direction (i.e., in the direction of
the flow). Include the effect of viscous dissipation. You may neglect axial conduction.
Assume that the fluid is hydrodynamically fully developed.

The inputs are entered in EES:

$UnitSystem SI MASS RAD PA K J


$TABSTOPS 0.2 0.4 0.6 0.8 3.5 in

mu=0.6 [Pa-s] "viscosity"


k=0.15 [W/m-K] "conductivity"
V_dot=15 [liter/min]*convert(liter/min,m^3/s) "volumetric flow rate"
D=0.5 [inch]*convert(inch,m) "tube diameter"
th_ins=0.25 [inch]*convert(inch,m) "insulation thickness"
k_ins=0.5 [W/m-K] "insulation conductivity"
h_bar=120 [W/m^2-K] "heat transfer coefficient"
T_infinity=converttemp(C,K,20 [C]) "ambient temperature"
L=1 [m] "length"

The thermal energy conservation equation in this limit is:

2
k d ⎛ dT ⎞ ⎛ du ⎞
0= ⎜r ⎟+μ⎜ ⎟ (1)
r dr ⎝ dr ⎠ ⎝ dr ⎠

where the velocity distribution for fully developed laminar flow in a round tube is:

⎡ ⎛ 2 r ⎞2 ⎤
u = 2 um ⎢1 − ⎜ ⎟ ⎥ (2)
⎢⎣ ⎝ D ⎠ ⎥⎦

where um is the mean velocity:

V
um = (3)
Ac

where

Ac = π rout
2
(4)
and

D
rout = (5)
2

r_out=D/2 "outer radius of tube"


A_c=pi*r_out^2 "cross-sectional area of tube"
u_m=V_dot/A_c "bulk velocity"

The velocity gradient is:

du 16 um r
=− (6)
dr D2

Substituting Eq. (6) into Eq. (1) leads to:

k d ⎛ dT ⎞ μ um2 r 2
0= ⎜ r ⎟ + 256 (7)
r dr ⎝ dr ⎠ D4

Equation (7) is separated:

⎛ dT ⎞ μ um2 r 3
d ⎜r ⎟ = −256 dr (8)
⎝ dr ⎠ k D4

and integrated (note the limits of integration correspond to a 0 temperature gradient at r = 0):
dT
r
dr
⎛ dT ⎞ μ um2 3 r


0
d ⎜r ⎟ = −256
⎝ dr ⎠ k D 4 ∫0
r dr (9)

which leads to:

dT μ u2 r 4
r = −64 m 4 (10)
dr kD

Equation (10) is separated:

μ um2 r 3
dT = −64 dr (11)
k D4

and integrated again:


μ um2
∫ dT = −64 k D ∫ r dr
3
4
(12)

which leads to:

μ um2
T = −16 r 4 + C1 (13)
k D4

where C1 is a constant of integration. The constant of integration is evaluated by enforcing an


energy balance at the outer radius:

− k 2 π rout L
dT
=
(Tr = rout − T∞ ) (14)
dr r = rout Rins + Rconv

where L is the length (taken to be 1 m for a per unit length solution) and Rins is the resistance to
conduction through the insulation:

⎛ D + 2 thins ⎞
ln ⎜ ⎟
D
Rins = ⎝ ⎠ (15)
2 π kins L

and Rconv is the resistance to convection from the outer surface of the insulation:

1
Rconv = (16)
π ( D + 2 thins ) L h

R_ins=ln((D+2*th_ins)/D)/(2*pi*k_ins*L) "resistance of insulation"


R_conv=1/(h_bar*pi*(D+th_ins)*L) "convection resistance"

Equation (13) is used to evaluate the temperature and temperature gradient at r = rout:

μ um2
Tr = rout = −16 rout
4
+ C1 (17)
kD 4

dT μ um2
= −64 rout
3
(18)
dr r = rout k D4

dTdr_out=-64*mu*u_m^2*r_out^3/(k*D^4) "temperature gradient at r=r_out"


T_out=-16*mu*u_m^2*r_out^4/(k*D^4)+C1 "temperature at r=r_out"
-k*2*pi*r_out*L*dTdr_out=(T_out-T_infinity)/(R_ins+R_conv) "boundary condition"

The temperature distribution, Eq. (13), is programmed:


r_bar=0 [-] "dimensionless radius"
r=r_bar*r_out "radius"
T=-16*mu*u_m^2*r^4/(k*D^4)+C1 "temperature"
T_C=converttemp(K,C,T) "in C"

b.) Plot the temperature as a function of radial position. Overlay on your plot the temperature
distribution for V = 5, 10, and 20 liter/min.

The requested plot is shown in Figure 5.4-2.

90
V = 20 lpm
80

70
Temperature (°C)

60
V = 15 lpm
50

40
V = 10 lpm
30
V = 5 lpm
20
0 0.1 0.2 0.3 0.4 0.5 0.6 0.7 0.8 0.9 1
Dimensionless radius, r/rout
Figure 5.4-2: Temperature as a function of normalized radius for various values of volumetric flow rate.
Problem 5.5-1 (5-9 in text): Immersion Lithography
Immersion lithography is a potential technique that will allow optical lithography (the
manufacturing technique used to fabricate computer chips) to create smaller features. A liquid is
inserted into the space between the last optical element (the lens) and the wafer that is being
written in order to increase the index of refraction in this volume (relative to the air that would
otherwise fill this gap). A simplified version of this concept is shown in Figure P5.5-1.

L = 10 cm
ρ = 800 kg/m3
Tlens = 20°C lens c = 500 J/kg-K
Δp = 300 kPa y  ′′ = 2 k = 0.1 W/m-K
= °
Tin 20 C
qexp 500 W/m μ = 0.1 Pa-s
x
wafer
uw = 0.45 m/s
wexp = 2 cm H = 0.5 mm
Figure P5.5-1: An immersion lithography tool.

It is important to predict the temperature distribution in the fluid during this process; even very
small temperature changes will result in imaging problems associated with changes in the
properties of the fluid or thermally induced distortions of the wafer. The fluid is inserted into the
gap at the left hand side (x = 0) and flows from left to right through the lens/wafer gap; the total
length of the gap is L = 10 cm and the height of the gap is H = 0.5 mm. The fluid is driven by
the viscous shear as the wafer is moved under the lens with velocity uw = 0.45 m/s. The fluid is
also driven by a pressure gradient; the pressure at the left hand side of the gap is elevated relative
to the pressure on the right hand side by an amount, Δp = 300 kPa. The fluid enters at the left
hand side of the lens with a uniform temperature, Tin = 20°C. Assume that the flow is laminar
and that the problem is two-dimensional (i.e., the slot extends a long way into the page). In this
case, the liquid has a fully developed velocity distribution when it enters the gap:

⎞ H Δp ⎡⎛ y ⎞ ⎛ y⎞ ⎤
2

2
y
u = uw ⎜ 1 − ⎟ + ⎢⎜ ⎟−⎜ ⎟ ⎥
⎝ H ⎠ 2 μ L ⎢⎣⎝ H ⎠ ⎝ H ⎠ ⎥⎦

where y is the distance from the wafer and μ is the liquid viscosity.

The next generation of immersion lithography tools will use advanced liquids with very high
viscosity and so you have been asked to generate a model that can evaluate the impact of viscous
dissipation on the temperature distribution. The liquid has density ρ = 800 kg/m3, specific heat
capacity c = 500 J/kg-K, thermal conductivity k = 0.1 W/m-K, and viscosity μ = 0.1 Pa-s. The
energy required to develop the resist layer and therefore carry out the lithography process passes
through the lens and the water and is deposited into the wafer; the energy can be modeled as a
heat flux at the wafer surface into the liquid (assume all of the heat flux will go to the liquid
rather than the wafer). The heat flux is concentrated in a small strip (wexp = 2.0 cm long) at the
center of the lens, as shown in Figure P5.5-1 and given by:
⎧0 for x < ( L − wexp ) / 2


′′ for ( L − wexp ) / 2 < x < ( L + wexp ) / 2
qs′′ ( x ) = ⎨qexp

⎪⎩0 for x > ( L + wexp ) / 2

′′ = 500 W/m2. The lens is maintained at a constant temperature, Tlens = 20°C.


where qexp
a.) What is the mean velocity and the Reynolds number that characterizes the flow through the
lens/wafer gap?

The inputs are entered in a MATLAB script, P5p3_1.m:

clear all;

u_w=0.45; %wafer velocity (m/s)


H=0.0005; %gap height (m)
L=0.10; %gap length (m)
DeltaP=300000; %pressure drop (Pa)
T_in=20; %inlet temperature (C)
T_lens=20; %lens temperature (C)
rho=800; %density (kg/m^3)
c=500; %specific heat capacity (J/kg-K)
k=0.1; %conductivity (W/m-K)
mu=0.1; %viscosity (Pa-s)
qf_exp=500; %heat flux in exposure region (W/m^2)
w_exp=0.02; %exposure region width (m)

The mean velocity in the lens/wafer gap is obtained according to:


H
1
u ( y ) dy
H ∫0
um = (1)

Substituting Eq. Error! Reference source not found. into Eq. (1) leads to:

1
H ⎧⎪ ⎛ y ⎞ H 2 Δp ⎡⎛ y ⎞ ⎛ y ⎞ ⎤ ⎫⎪
2

um =
H ∫0 ⎨⎪uw ⎜⎝1 − H ⎟⎠ + 2 μ L ⎢⎢⎜⎝ H ⎟⎠ − ⎜⎝ H ⎟⎠ ⎥⎥ ⎬⎪ dy (2)
⎩ ⎣ ⎦⎭

Carrying out the integration leads to:

H
1 ⎧⎪ ⎛ y2 ⎞ H 2 Δp ⎡⎛ y 2 ⎞ y 3 ⎤ ⎫⎪
um = ⎨uw ⎜ y − ⎟+ ⎢⎜ ⎟ − 2 ⎥⎬
(3)
H ⎩⎪ ⎝ 2H ⎠ 2 μ L ⎣⎝ 2 H ⎠ 3 H ⎦ ⎭⎪0

Applying the limits leads to:


1 ⎪⎧ ⎛ H 2 ⎞ H 2 Δp ⎡⎛ H 2 ⎞ H 3 ⎤ ⎪⎫
um = ⎨uw ⎜ H − ⎟+ ⎢⎜ ⎟− ⎥⎬ (4)
H ⎩⎪ ⎝ 2 H ⎠ 2 μ L ⎣⎝ 2 H ⎠ 3 H 2 ⎦ ⎭⎪

which can be simplified to:

uw Δp H 2
um = + (5)
2 12 μ L

u_m=u_w/2+DeltaP*H^2/(3*mu*L) %mean velocity (m/s)

which leads to um = 0.85 m/s. The hydraulic diameter associated with flow through the gap
between two parallel plates is:

4HW
Dh = = 2H (6)
2W

where W is the length of the duct into the page (note W >> H). The Reynolds number for the
flow in the gap is therefore:

Re=2*H*u_m*rho/mu %Reynolds number (-)

which leads to Re = 6.8.

b.) You’d like to calculate a Brinkman number in order to evaluate the relative impact of viscous
dissipation for the process but you don’t have a convenient reference temperature difference
to use. Use the heat flux to come up with a meaningful reference temperature difference and
from that temperature difference determine a Brinkman number. Comment on the
importance of viscous dissipation for this problem.

A meaningful reference temperature difference might be based on the temperature difference


between the surface of the wafer and the mean temperature in the duct if the heat flux were
applied in a thermally fully developed flow situation; this would be approximately:

qs′′ H
ΔTref = (7)
2k

An alternative might be to estimate the temperature difference between the inlet fluid and the
wafer surface at the trailing edge of the thermal boundary layer that develops over the region of
heat flux:

qs′′ δ t
ΔTref = (8)
k

where δt is an estimate of the thermal boundary layer thickness:


α wexp
δt = 2 (9)
um

Either answer is fine; the reference temperature difference associated with Eq. (7) is computed
according to:

DT_ref=qf_exp*H/(2*k) %reference temperature difference (K)

which leads to ΔTref = 1.25 K. The Brinkman number is:

Br = Ec Pr (10)

where Ec is the Eckert number:

um2
Ec = (11)
c ΔTref

and Pr is the Prandtl number:

ν
Pr = (12)
α

where ν is the kinematic viscosity:

μ
ν= (13)
ρ

and α is the thermal diffusivity:

k
α= (14)
ρc

nu=mu/rho; %kinematic viscosity (m^2/s)


alpha=k/(rho*c); %thermal diffusivity (m^2/s)
Pr=nu/alpha %Prandtl number (-)
Ec=u_m^2/(c*DT_ref) %Eckert number (-)
Br=Pr*Ec %Brinkman number (-)

which leads to Br = 0.58. Because the Brinkman number is larger than unity it is likely that
viscous dissipation is important for this problem.

c.) Is axial conduction important for this problem? Justify your answer.
Axial conduction scales according to Pe-2, where Pe is the Peclet number:

Pe = Re Pr (15)

Pe=Re*Pr %Peclet number (-)

which leads to Pe = 3400. Therefore, axial conduction is not likely to be important.

d.) Develop a numerical model of the thermal behavior of the flow through the gap that accounts
for viscous dissipation but not axial conduction. Use the native ode solver in MATLAB.

There are N nodes distributed uniformly in the y direction across the lens/wafer gap, as shown in
Figure 2.

Figure 2: Node placement and energy balance.

The distance between adjacent nodes


H
Δy = (16)
N

and the location of each of the nodes is given by:

⎛ 1⎞
yi = Δy ⎜ i − ⎟ for i = 1..N (17)
⎝ 2⎠

N=21; %number of nodes (-)


Dy=H/N; %distance between y-nodes (m)
for i=1:N
y(i)=Dy*(i-1/2); %y-position of each node (m)
end

The velocity at each of the nodes is obtained from Eq. Error! Reference source not found.:

⎞ H Δp ⎡⎛ yi ⎞ ⎛ yi ⎞ ⎤
2

2
y
ui = uw ⎜1 − i +
⎟ 2 μ L ⎢⎜ H ⎟ − ⎜ H ⎟ ⎥ for i = 1..N (18)
⎝ H ⎠ ⎢⎣⎝ ⎠ ⎝ ⎠ ⎥⎦

The velocity gradient at each node will also be required in order to compute the viscous
dissipation:

du uw H 2 Δp ⎡ 1 2 yi ⎤
=− + − for i = 1..N (19)
dy i H 2 μ L ⎢⎣ H H 2 ⎥⎦

for i=1:N
u(i)=u_w*(1-y(i)/H)+H^2*DeltaP*(y(i)/H-(y(i)/H)^2)/(2*mu*L); %velocity
dudy(i)=-u_w/H+H^2*DeltaP*(1/H-2*y(i)/H^2)/(2*mu*L);
%velocity gradient
end

The internal nodes are treated separately from the boundary nodes; a control volume for an
internal node is shown in Figure 2. The node experiences conduction in the y direction from
adjacent nodes, volumetric generation (the transformation of the fluid’s mechanical energy into
thermal energy), and enthalpy carried by fluid entering the control volume at x and leaving at x +
dx; the energy balance suggested by the control volume in Figure 2 is:

( ρ c ui ΔyW Ti ) x + qtop + qbottom + g v = ( ρ c ui ΔyW Ti ) x + dx (20)

where W is the depth of the channel into the page. The conduction heat transfer rates are
approximated according to:

k dxW
qtop = (Ti +1 − Ti ) (21)
Δy
k dxW
qbottom = (Ti −1 − Ti ) (22)
Δy

The viscous dissipation is given by:


2
⎛ ∂u ⎞
g v = μ ⎜⎜ ⎟⎟ Δy dxW (23)
∂y
⎝ i⎠

Equations (24) through (23) are substituted into Eq. (20) and the x+dx term is expanded:

2
k dxW k dxW ⎛ ∂u ⎞
( ρ c ui ΔyW Ti ) x + (Ti +1 − Ti ) + (Ti −1 − Ti ) + μ ⎜⎜ ⎟⎟ Δy dxW =
Δy Δy ⎝ ∂y i ⎠
d
( ρ c ui ΔyW Ti ) x + ( ρ c ui ΔyW Ti ) dx (25)
dx
for i = 2.. ( N − 1)

Note that the only term in the derivative that changes with x is the temperature (ui is constant in
the hydrodynamically fully developed region) and therefore Eq. (25) can be rewritten as:
2
k dxW k dxW ⎛ ∂u ⎞ dT
(Ti +1 − Ti ) + (Ti −1 − Ti ) + μ ⎜⎜ ⎟⎟ Δy dxW = ρ c ui ΔyW i dx for i = 2.. ( N − 1)
Δy Δy ⎝ ∂y i ⎠ dx
(26)

Solving for the rate of change of Ti with respect to x:


2
dTi k μ ⎛ ∂u ⎞
= (Ti +1 + Ti −1 − 2 Ti ) + ⎜ ⎟ for i = 2.. ( N − 1) (27)
dx ρ c Δy ui
2
ρ c ui ⎜⎝ ∂y i ⎟⎠

An energy balance for the control volume around node 1 is also shown in Figure 2 and leads to:

dT1
qtop + qwafer + g v = ρ c u1 ΔyW dx (28)
dx

The conductive heat transfer from node 2 is approximated according to:

k dxW
qtop = (T2 − T1 ) (29)
Δy

The heat transfer from the wafer is:


qwafer = qs′′ ( x ) dxW (30)

The viscous dissipation is given by:

2
⎛ ∂u ⎞
g v = μ ⎜⎜ ⎟⎟ Δy dxW (31)
⎝ ∂y 1 ⎠

Substituting Eqs. (29) through (31) into Eq. (28) leads to:

2
k dxW ⎛ ∂u ⎞ dT
(T2 − T1 ) + qs′′ ( x ) dxW + μ ⎜⎜ ⎟⎟ Δy dxW = ρ c u1 ΔyW 1 dx (32)
Δy ⎝ ∂y 1 ⎠ dx

Solving for the rate of change of the temperature of node 1 leads to:

q s′′ ( x )
2
dT1 k μ ⎛ ∂u ⎞
= ( T − T ) + + ⎜ ⎟ (33)
dx ρ c u1 Δy 2
2 1
ρ c u1 Δy ρ c u1 ⎜⎝ ∂y 1 ⎟⎠

An energy balance for the control volume around node 1 is also shown in Figure 2 and leads to:

dTN
qlens + qbottom + g v = ρ c u N ΔyW dx (34)
dx

The conductive heat transfer rates are:

k dxW
qbottom = (TN −1 − TN ) (35)
Δy

2 k dxW
qlens = (Tlens − TN ) (36)
Δy

and the volumetric generation by viscous dissipation is:


2
⎛ du ⎞
g v = μ ⎜⎜ ⎟⎟ W Δy dx (37)
⎝ dy N ⎠

Substituting Eqs. (35) through (37) into Eq. (34) leads to:
2
2 k dxW k dxW ⎛ du ⎞ dTN
(Tlens − TN ) + (TN −1 − TN ) + μ ⎜⎜ ⎟⎟ W Δy dx = ρ c u N ΔyW dx (38)
Δy Δy ⎝ dy N ⎠
dx
Solving for the rate of temperature change of node N:
2
dTN 2k k μ ⎛ du ⎞
2 ( lens
= T − TN ) + 2 ( N −1
T − TN ) + ⎜⎜ ⎟⎟ (39)
dx ρ c u N Δy ρ c u N Δy ρ c uN ⎝ dy N ⎠

A function, dTdx_P5p3_1.m, is defined in order to return the rate of change of each node:

function[dTdx]=dTdx_P5p3_1(x,T,L,w_exp,qf_exp,u,dudy,Dy,k,rho,c,mu,T_lens)

[N,g]=size(T); %determine number of nodes


dTdx=zeros(N,1); %initialize dTdx
%heat flux
if (x<(L-w_exp)/2)
qf=0;
elseif (x<(L+w_exp)/2)
qf=qf_exp;
else
qf=0;
end

dTdx(1)=k*(T(2)-
T(1))/(rho*c*u(1)*Dy^2)+qf/(rho*c*u(1)*Dy)+mu*dudy(1)^2/(rho*c*u(1));
for i=2:(N-1)
dTdx(i)=k*(T(i+1)+T(i-1)-
2*T(i))/(Dy^2*rho*c*u(i))+mu*dudy(i)^2/(rho*c*u(i));
end
dTdx(N)=2*k*(T_lens-T(N))/(rho*c*u(N)*Dy^2)+k*(T(N-1)-
T(N))/(rho*c*u(N)*Dy^2)+mu*dudy(N)^2/(rho*c*u(N));

The ODE solver ode45 is used to integrate the function dTdx_P5p3_1 over the length of the
duct:

OPTIONS=odeset('RelTol',1e-6);
[x,T]=ode45(@(x,T)
dTdx_P5p3_1(x,T,L,w_exp,qf_exp,u,dudy,Dy,k,rho,c,mu,T_lens),...
[0,L],T_in*ones(N,1),OPTIONS);

e.) Prepare a contour plot that shows the temperature distribution in the lens/wafer gap.

The contourf command creates a filled contour plot of the result, shown in Figure 3.

>> contourf(x,y,T',21)
Figure 3: Temperature distribution in the lens/wafer gap.

f.) Prepare a contour plot that shows the temperature distribution in the absence of any applied
heat flux (i.e., what is the heating caused by the viscous dissipation?).

The externally applied heat flux is set to zero:

qf_exp=0; %heat flux in exposure region (W/m^2)

and the temperature distribution is shown in Figure 4.


Figure 4: Temperature distribution in the lens/wafer gap due to viscous dissipation.
Problem 5.5-2
A fluid is used to provide low temperature cooling to a piece of medical equipment. The fluid is
circulated between the device, which adds heat to the fluid, and a bath of coolant maintained at
T∞ = -40°C. The fluid enters a long coiled tube that is submerged in the coolant. The
temperature of the fluid at the inlet is uniform temperature and equal to Tin = -20°C. Due to the
addition of antifreeze to the fluid, the viscosity of the fluid is very high. Therefore, the fluid is
hydrodynamically fully developed throughout the tube and viscous dissipation is important. The
diameter of the tube is D = 3.5 mm and its total length is L = 20 m. The mean velocity of the
fluid is um = 1 m/s. The fluid has conductivity k = 1. 5 W/m-K, viscosity μ = 0.4 Pa-s, density ρ
= 1000 kg/m3, and specific heat capacity c = 4000 J/kg-K. The conduction resistance radially
through the tube is negligible. The heat transfer coefficient between the tube and the coolant is
h = 250 W/m2-K.
a.) Develop a numerical model that can predict the temperature as a function of radius and axial
position within the tube. Neglect axial conduction but include viscous dissipation. Use the
ode solver in MATLAB.

The inputs are entered in a MATLAB script, P5p5d2.m:

clear all;

u_m=1; % mean velocity (m/s)


D=0.0035; % diameter (m)
T_in=273.2-20; % inlet fluid temperature (K)
mu=0.4; % viscosity (Pa-s)
k=1.5; % conductivity (W/m-K)
T_infinity=273.2-40; % bath temperature (K)
rho=1000; % density (kg/m^3)
c=4000; % specific heat capacity (J/kg-K)
L=20; % length of tube (m)
h_bar=250; % heat transfer coefficient (W/m^2-K)

The nodes are distributed radially according to:

⎛ 1⎞
ri = Δr ⎜ i − ⎟ for i = 1..N (1)
⎝ 2⎠

where

D
Δr = (2)
2N

N=21; % number of radial nodes (-)


Dr=(D/2)/N; % distance between nodes (m)
for i=1:N
r(i)=Dr*(i-1/2); % position of each node (m)
end

The velocity distribution for a fully developed flow is:


⎡ ⎛ 2 ri ⎞ 2 ⎤
ui = 2 um ⎢1- ⎜ ⎟ ⎥ for i = 1..N (3)
⎣⎢ ⎝ D ⎠ ⎦⎥

for i=1:N
u(i)=2*u_m*(1-(2*r(i)/D)^2); % velocity at each node (m/s)
end

The volumetric rate of thermal energy generation due to viscous dissipation is a function of
radius:

ri 2
g v′′′,i = 256 μ um2 for i = 1..N (4)
D4

for i=1:N
gv(i)=256*mu*u_m^2*r(i)^2/D^4;
% volumetric rate of thermal energy generation (W/m^3)
end

The cross-sectional area for flow into each node is given by:

⎡⎛ Δr ⎞ ⎛
2
Δr ⎞ ⎤
2

Ac ,i = π ⎢⎜ ri + −
⎟ ⎜ ir − ⎟ ⎥ for i = 1..N (5)
⎢⎣⎝ 2 ⎠ ⎝ 2 ⎠ ⎥⎦

for i=1:N
Ac(i)=pi*((r(i)+Dr/2)^2-(r(i)-Dr/2)^2);
% cross-sectional area for each node (m^2)
end

An energy balance on the internal nodes leads to:

ρ c ui Ac ,i
dTi (T − T ) (T − T )
= 2 π k i +1 i + 2 π k i −1 i + g v′′′,i Ac ,i for i = 2.. ( N − 1) (6)
dx ⎛r ⎞ ⎛ r ⎞
ln ⎜ i +1 ⎟ ln ⎜ i ⎟
⎝ ri ⎠ ⎝ ri −1 ⎠

which is solved for the state equations for the internal nodes:

⎡ ⎤
⎢ ⎥
dTi
=
1 ⎢ 2π k
( Ti +1 − Ti )
+ 2π k
( Ti −1 − Ti )
+ g v′′′,i Ac ,i ⎥ for i = 2.. ( N − 1) (7)
dx ρ c ui Ac ,i ⎢ ⎛ ri +1 ⎞ ⎛ ri ⎞ ⎥
⎢ ln ⎜ ⎟ ln ⎜ ⎟ ⎥
⎢⎣ ⎝ ri ⎠ ⎝ ri −1 ⎠ ⎥⎦

An energy balance on node 1 leads to:


ρ c u1 Ac ,1
dT1 (T − T )
= 2 π k 2 1 + g v′′′,1 Ac ,1 (8)
dx ⎛r ⎞
ln ⎜ 2 ⎟
⎝ r1 ⎠

which is solved for the state equation for node 1:

⎡ ⎤
⎢ ⎥
dT1
=
1 ⎢ 2 π k (T2 − T1 ) + g ′′′ A ⎥ (9)
dx ρ c u1 Ac ,1 ⎢ ⎥
v ,1 c ,1
⎛ r2 ⎞
⎢ ln ⎜ ⎟ ⎥
⎣⎢ ⎝ r1 ⎠ ⎦⎥

An energy balance on node N leads to:

ρ c u N Ac , N
dTN (T − T )
= π D h (T∞ − TN ) + 2 π k N −1 N + g v′′′, N Ac , N (10)
dx ⎛ r ⎞
ln ⎜ N ⎟
⎝ rN −1 ⎠

which is solved for the state equation for node N:

⎡ ⎤
⎢ ⎥
dTN
=
1 ⎢π D h (T − T ) + 2 π k (TN −1 − TN ) + g ′′′ A ⎥ (11)
dx ρ c u N Ac , N ⎢ ∞ N
⎛ rN ⎞
v, N c, N

⎢ ln ⎜ ⎟ ⎥
⎣⎢ ⎝ rN −1 ⎠ ⎦⎥

The state equations are programmed in a function dTdt_p5p5d2:

function[dTdx]=dTdt_P5p5d2(x,T,u,Ac,r,gv,D,mu,k,T_infinity,rho,c,h_bar,N)

% Inputs
% x - position (m)
% T - vector of temperatures (K)
% u - vector of velocities (m/s)
% Ac - vector of cross-sectional areas for each node (m^2)
% r - vector of radii for each node (m)
% gv - vector of volumetric thermal energy generation rates (W/m^3)
% D - diameter of tube (m)
% mu - viscosity (Pa-s)
% k - thermal conductivity (W/m-K)
% T_infinity - ambient temperature (K)
% rho - density (kg/m^3)
% c - specific heat capacity (J/kg-K)
% h_bar - heat transfer coefficient (W/m^2-K)
% N - number of nodes (-)
%
% Output:
% dTdx - rate of temperature change for each node (K/m)

dTdx=zeros(N,1);
dTdx(1)=(2*pi*k*(T(2)-...
T(1))/(log(r(2)/r(1)))+gv(1)*Ac(1))/(rho*c*u(1)*Ac(1));
for i=2:(N-1)
dTdx(i)=(2*pi*k*(T(i+1)-T(i))/log(r(i+1)/r(i))+2*pi*k*(T(i-1)-...
T(i))/log(r(i)/r(i-1))+gv(i)*Ac(i))/(rho*c*u(i)*Ac(i));
end
dTdx(N)=(2*pi*k*(T(N-1)-T(N))/log(r(N)/r(N-1))+...
h_bar*pi*D*(T_infinity-T(N))+gv(N)*Ac(N))/(rho*c*u(N)*Ac(N));
end

and integrated using the ode45 solver:

OPTIONS=odeset('RelTol',1e-6);
[x,T]=ode45(@(x,T) dTdt_P5p5d2(x,T,u,Ac,r,gv,D,mu,k,T_infinity,rho,...
c,h_bar,N),[0,L],T_in*ones(N,1),OPTIONS);

The mean temperature is computed according to:

∑T i, j ui Ac ,i
Tm, j = i =1
(12)
D2
um π
4

b.) Plot the temperature as a function of axial position at various values of radius. Overlay on
your plot the mean temperature as a function of axial position.

Figure 1 illustrates the temperature as a function of axial position at various values of radius as
well as the mean temperature.

254

252

250
Temperature (K)

r=0

248
mean temperature

246

244
r = rout

242

240
0 5 10 15 20
Axial position (m)
Figure 1: Temperature as a function of axial position at various values of radius and mean temperature.
c.) Plot the mean temperature as a function of axial position for various values of viscosity,
including μ = 0.4, 0.6, 0.8, 1.0, 1.5, and 2.0 Pa-s.

Figure 2 illustrates the mean temperature as a function of axial position for the viscosities listed
in the problem statement.

254
2 Pa-s
252

250 1.5 Pa-s


Temperature (K)

248 1 Pa-s

246 0.8 Pa-s

244 0.4 Pa-s 0.6 Pa-s

242

240
0 5 10 15 20
Axial position (m)
Figure 2: Mean temperature as a function of axial position for various values of viscosity.
Problem 5.5-3 (5-10 in text)
Figure P5.5-3 shows a thin-wall tube with radius R = 5.0 mm carrying a flow of liquid with
density ρ = 1000 kg/m3, specific heat capacity c = 1000 J/kg-K, conductivity k = 0.5 W/m-K, and
viscosity μ = 0.017 Pa-s. The fluid is fully developed hydrodynamically with a bulk velocity
um= 0.2 m/s and has a uniform temperature Tini = 80°C when it enters a section of the tube that is
exposed to air at temperature T∞ = 20°C with heat transfer coefficient ha .

um = 0.2 m/s T∞ = 20°C, ha


Tini = 80°C R = 5 mm
μ = 0.017 Pa-s
k = 0.5 W/m-K
c = 1000 J/kg-K
ρ = 1000 kg/m3
L
Figure P5.5-3: Thin-wall tube carrying fluid exposed to air.

The Reynolds number for this flow is around 100 and so this is a laminar flow. Typically, the
heat transfer between the fluid and the air is modeled using a laminar flow heat transfer
coefficient that is calculated using correlations that are based on a constant tube surface
temperature. In fact, the surface temperature of the tube is not constant for this process. The
objective of this homework is to understand how much this approximation affects the solution.
a.) Develop a numerical model of this situation using MATLAB. Prepare a plot showing the
temperature at various radii as a function of axial position for the case where ha = 100 W/m2-
K and L = 5.0 m. Prepare a plot of the Nusselt number as a function of axial position for this
situation.

The inputs are entered in a MATLAB script, P5p5d3.m:

clear all;
L=5; % length of pipe (m)
R=0.005; % radius of pipe (m)
Tin=80; % inlet temperature of water (C)
c=1000; % specific heat capacity of fluid (J/kg-K)
rho=1000; % density of fluid (kg/m^3)
k=0.5; % thermal conductivity (W/m-k)
um=0.2; % bulk velocity of flow (m/s)
mu=0.017; % viscosity (Pa-s)
ha=100; % heat transfer coefficient with surrounding air (W/m^2-K)
Ta=20; % temperature of surrounding air (C)

The nodes are distributed radially according to:

⎛ 1⎞
ri = Δr ⎜ i − ⎟ for i = 1..N (1)
⎝ 2⎠

where
R
Δr = (2)
N

N=21; % number of radial nodes (-)


Dr=R/N; % distance between nodes (m)
for i=1:N
r(i)=Dr*(i-1/2); % position of each node (m)
end

The velocity distribution for a fully developed flow is:

⎡ ⎛ ri ⎞2 ⎤
ui = 2 um ⎢1- ⎜ ⎟ ⎥ for i = 1..N (3)
⎢⎣ ⎝ R ⎠ ⎥⎦

for i=1:N
u(i)=2*um*(1-(r(i)/R)^2); % velocity at each node (m/s)
end

The cross-sectional area for flow into each node is given by:

⎡⎛ Δr ⎞ ⎛
2
Δr ⎞ ⎤
2

Ac ,i = π ⎢⎜ ri + ⎟ − ⎜ ri − ⎟ ⎥ for i = 1..N (4)


⎣⎢⎝ 2 ⎠ ⎝ 2 ⎠ ⎦⎥

for i=1:N
Ac(i)=pi*((r(i)+Dr/2)^2-(r(i)-Dr/2)^2);
% cross-sectional area for each node (m^2)
end

An energy balance on the internal nodes leads to:

ρ c ui Ac ,i
dTi (T − T ) (T − T )
= 2 π k i +1 i + 2 π k i −1 i for i = 2.. ( N − 1) (5)
dx ⎛r ⎞ ⎛ r ⎞
ln ⎜ i +1 ⎟ ln ⎜ i ⎟
⎝ ri ⎠ ⎝ ri −1 ⎠

which is solved for the state equations for the internal nodes:

⎡ ⎤
⎢ ⎥
dTi
=
1 ⎢ 2 π k (Ti +1 − Ti ) + 2 π k (Ti −1 − Ti ) + g ′′′ A ⎥ for i = 2.. ( N − 1) (6)
dx ρ c ui Ac ,i ⎢ ⎥
v ,i c , i
⎛ ri +1 ⎞ ⎛ ri ⎞
⎢ ln ⎜ ⎟ ln ⎜ ⎟ ⎥
⎣⎢ ⎝ ri ⎠ ⎝ ri −1 ⎠ ⎥⎦

An energy balance on node 1 leads to:


ρ c u1 Ac ,1
dT1 (T − T )
= 2π k 2 1 (7)
dx ⎛r ⎞
ln ⎜ 2 ⎟
⎝ r1 ⎠

which is solved for the state equation for node 1:

dT1 2 π k (T2 − T1 )
= (8)
dx ρ c u1 Ac ,1 ⎛ r2 ⎞
ln ⎜ ⎟
⎝ r1 ⎠

An energy balance on node N leads to:

ρ c u N Ac , N
dTN
=
(T∞ − TN ) + 2π k
(TN −1 − TN ) (9)
dx ⎛R⎞ ⎛ r ⎞
ln ⎜ ⎟ ln ⎜ N ⎟
r ⎝ rN −1 ⎠
+ ⎝ N⎠
1
ha 2 π R 2π k

which is solved for the state equation for node N:

⎡ ⎤
⎢ ⎥
⎢ ⎥
dTN
=
1 ⎢ ( ∞ N)
T − T ( T −
+ 2 π k N −1 N ⎥
T )⎥
⎢ (10)
dx ρ c u N Ac , N ⎢ ⎛R⎞ ⎛ r ⎞
ln ⎜ ⎟ ln ⎜ N ⎟ ⎥
⎢ 1 r ⎝ rN −1 ⎠ ⎥
⎢ + ⎝ N⎠ ⎥
⎣ ha 2 π R 2π k ⎦

The state equations are programmed in a function dTdt_p5p5d3:

function[dTdx]=dTdt_P5p5d3(x,T,u,Ac,r,R,k,Ta,rho,c,ha,N)

% Inputs
% x - position (m)
% T - vector of temperatures (K)
% u - vector of velocities (m/s)
% Ac - vector of cross-sectional areas for each node (m^2)
% r - vector of radii for each node (m)
% R - radius of tube (m)
% k - thermal conductivity (W/m-K)
% Ta - ambient temperature (K)
% rho - density (kg/m^3)
% c - specific heat capacity (J/kg-K)
% ha - heat transfer coefficient (W/m^2-K)
% N - number of nodes (-)
%
% Output:
% dTdx - rate of temperature change for each node (K/m)

dTdx=zeros(N,1);
dTdx(1)=(2*pi*k*(T(2)-T(1))/(log(r(2)/r(1))))/(rho*c*u(1)*Ac(1));
for i=2:(N-1)
dTdx(i)=(2*pi*k*(T(i+1)-T(i))/log(r(i+1)/r(i))+2*pi*k*(T(i-1)-...
T(i))/log(r(i)/r(i-1)))/(rho*c*u(i)*Ac(i));
end
dTdx(N)=(2*pi*k*(T(N-1)-T(N))/log(r(N)/r(N-1))+...
(Ta-T(N))/(1/(ha*2*pi*R)+log(R/r(N))/(2*pi*k)))/(rho*c*u(N)*Ac(N));
end

and integrated using the ode45 solver:

OPTIONS=odeset('RelTol',1e-6);
[x,T]=ode45(@(x,T) dTdt_P5p5d3(x,T,u,Ac,r,R,k,Ta,rho,c,ha,N)...
,[0,L],Tin*ones(N,1),OPTIONS);

The mean temperature is computed according to:

∑T i, j ui Ac ,i
Tm, j = i =1
(11)
D2
um π
4

[M,g]=size(x);
for j=1:M
T_mean(j)=0;
for i=1:N
T_mean(j)=T_mean(j)+Ac(i)*u(i)*T(j,i);
end
T_mean(j)=T_mean(j)/(pi*R^2*um);
end

Figure 2 shows the temperature at various radii as a function of axial position for the case where
ha = 100 W/m2-K and L = 5.0 m.
85

80
r=0
75

Temperature (deg. C)
70

65

60

55
r=R
50

45 increasing radius

40
0 1 2 3 4 5
Axial position (m)
Figure 2: Temperature as a function of axial position for various values of radius.

The surface heat flux at any location i is obtained according to:

qs′′, j =
(T∞ − TN , j )
(12)
⎛R⎞
R ln ⎜ ⎟
1
+ ⎝ rN ⎠
ha k

and the surface temperature is given by:

qs′′
Ts , j = T∞ − (13)
ha

for j=1:M
qs(j)=(Ta-T(j,N))/(1/ha+R*log(R/r(N))/k); % surface heat flux (W/m^2)
Ts(j)=Ta-qs(j)/ha; % surface temperature (K)
end

The local heat transfer coefficient on the liquid side and the local Nusselt number are defined
according to:

qs′′
h= (14)
(Ts − Tm )
h2R
Nu = (15)
k

for j=1:M
htc(j)=qs(j)/(Ts(j)-T_mean(j)); % heat transfer coefficient (W/m^2-K)
Nusselt(j)=htc(j)*2*R/k; % Nusselt number (-)
end

Figure 3 illustrates the Nusselt number as a function of position.

20

15
Nusselt number

10

0
0 1 2 3 4 5
Axial position
Figure 3: Nusselt number as a function of axial position.

b.) Verify that your solution by comparing it with an appropriate analytical model.

In the limit that ha → ∞ , the surface temperature of the tube becomes constant and equal to T∞.
The value of ha is set to a large number:

ha=100000; %heat transfer coefficient with surrounding air (W/m^2-K)

and the MATLAB simulation is run to obtain the local Nusselt number. The value is compared
to the correlation for laminar flow in a constant temperature tube programmed in EES and
accessed using the PipeFlow_N_local function:

$UnitSystem SI MASS RAD PA K J


$TABSTOPS 0.2 0.4 0.6 0.8 3.5 in

"Inputs"
L = 5 [m] "length of tube"
R = 0.005 [m] "tube radius"
c=1000 [J/kg-K] "specific heat capacity"
rho=1000 [kg/m^3] "density"
k=0.5 [W/m-K] "conductivity"
um=0.2 [m/s] "mean velocity"
mu=0.017 [Pa-s] "viscosity"

Re=um*2*R*rho/mu "Reynolds number"


Pr=mu*c/k "Prandtl number"
call PipeFlow_N_local(Re,Pr,x/(2*R),0 [-]: Nusselt_T_x,Nusselt_H_x,f_x)
"correlation for local Nusselt number"

Figure 4 illustrates the numerical model and correlation results.

15
correlation in EES
numerical model

10
Local Nusselt number

3
0.02 0.1 1 5
Axial position (m)
Figure 4: Correlation in EES compared to the numerical model solution in the limit that the tube surface
temperature is constant.

c.) Investigate the effect of the external convection coefficient; in the limit that ha is very large
your solution should limit (at long length) to Nu = 3.66. What is the effect of a finite ha ?
Present your conclusions in a logical and systematic manner.

Figure 5 illustrates the fully developed Nusselt number predicted by the numerical model as a
function of the dimensionless parameter:

k
(16)
R ha

which represents approximately the ratio of the thermal resistance on the air-side of the tube to
k
the thermal resistance on the fluid side. Notice that as approaches zero, the solution
R ha
approaches 3.66 which corresponds to fully developed laminar flow through a circular duct with
k
a constant surface temperature. As becomes large, the solution approaches 4.36 which
R ha
corresponds to fully developed laminar flow through a circular duct with a constant surface heat
flux.
4.3

Fully developed Nusselt number


4.2

4.1

3.9

3.8

3.7

3.6
0.01 0.1 1 10 20
k/(R ha)
k
Figure 5: Fully developed Nusselt number as a function of
R ha
Problem 5.5-4 (5-11 in text)
Figure P5.5-4 illustrates a flow of liquid in a passage formed between two parallel plates.

⎡ ⎛ 2π x ⎞⎤
q ′′ = q′′ + Δq ′′ sign ⎢sin ⎜ ⎟⎥
⎣ ⎝ Lh ⎠ ⎦

um = 0.2 m/s
Tin = 300K
μ = 0.05 Pa-s H = 0.5 mm
ρ = 1000 kg/m3
k = 0.25 W/m-K
c = 3800 J/kg-K

Lh = 1 mm Lh = 1 mm
x
Figure P5.5-4: Flow between two parallel plates.

The flow enters the duct having been exposed to a uniform heat flux at qs′′ = 9500 W/m2 for a
long time. Therefore, the flow is both thermally and hydrodynamically fully developed. The
velocity distribution is:
⎛ y y2 ⎞
u = 6 um ⎜ − 2 ⎟
⎝H H ⎠
where um = 0.2 m/s is the bulk velocity and H = 0.5 mm is the plate-to-plate spacing. The
temperature distribution at the inlet is:
qs′′ H ⎡ ⎛ y ⎞ ⎤
4 3
⎛ y⎞ ⎛ y⎞
T = Tin + ⎢ − ⎜ ⎟ + 2 ⎜ ⎟ − ⎜ ⎟ + 0.243⎥
k ⎣⎢ ⎝ H ⎠ ⎝H⎠ ⎝H⎠ ⎦⎥
where Tin = 300 K is the mean temperature of the fluid at the inlet. The properties of the fluid
are density ρ = 1000 kg/m3, viscosity μ = 0.05 Pa-s, conductivity k = 0.25 W/m-K, and specific
heat capacity c = 3800 J/kg-K. The heat flux applied to the surfaces of the channel is non-
uniform and you need to evaluate the impact of the non-uniform heat flux on the surface
temperature of the duct. The heat flux at both the upper and lower surfaces of the duct varies
according to:
⎡ ⎛ π x ⎞⎤
q ′′ = q ′′ + Δq ′′ sign ⎢sin ⎜ ⎟⎥
⎣ ⎝ Lh ⎠ ⎦
where Δq ′′ = 9500 W/m2 is the amplitude of the heat flux variation and Lh = 1 mm is the width
of the heated regions and the function sign returns +1 if the argument is positive and -1 if it is
negative. This equation, with the specified inputs, leads to a wall that alternates between having
a heat flux of 19,000 W/m2 for 1 mm and then being adiabatic for 1 mm.
a.) Is the flow laminar or turbulent?

The inputs are entered in MATLAB as a function:

u_m=0.2; % mean velocity (m/s)


H=0.0005; % duct height (m)
T_in=300; % inlet mean fluid temperature (K)
rho=1000; % density (kg/m^3)
mu=0.05; % viscosity (Pa-s)
k=0.25; % conductivity (Pa-s)
c=3800; % specific heat capacity (J/kg-K)
L=0.01; % length of duct (m)
qf_avg=9500; % average heat flux (W/m^2)
Dqf=9500; % heat flux fluctuation amplitude (W/m^2)
Lh=0.001; % length of heated section (m)
alpha=k/(rho*c) % thermal diffusivity (m^2/s)

The hydraulic diameter of the passage is calculated according to:

Dh = 2 H (1)

The Reynolds number is:

um Dh ρ
Re = (2)
μ

Dh=2*H; % hydraulic diameter


Re=u_m*Dh*rho/mu % Reynolds number (-)

which leads to Re = 4; therefore, the flow is laminar.

b.) Is viscous dissipation important?

The significance of viscous dissipation is quantified by the Brinkman number:

μ um2
Br = (3)
k (Ts − Tm )

The surface-to-mean temperature difference is evaluated at the inlet according to:

q s′′ Dh
Ts − Tm = (4)
Nu fd k

where Nufd is the fully developed Nusselt number for flow between parallel plates with a uniform
heat flux (8.235):

Nu_fd=8.235; % fully developed Nusselt number (-)


DT_uniform=qf_avg*Dh/(Nu_fd*k)
% surface-to-fluid temperature difference for a uniform heat flux
Br=mu*u_m^2/(k*DT_uniform) % Brinkman number

which leads to Br = 0.0017; therefore, viscous dissipation is negligible.


c.) Is axial conduction important?

The significance of axial conduction is quantified according to the inverse of the Peclet number
squared where the Peclet number is:

Pe = Re Pr (5)

The Prandtl number is calculated according to:

μc
Pr = (6)
k

Pr=mu*c/k % Prandtl number (-)


Pe=Re*Pr % Peclet number

which leads to Pe = 3040; therefore, axial conduction is not important.

d.) Develop a 2-D numerical model of the flow in the gap using the Crank-Nicolson solution
technique implemented in MATLAB. Plot the temperature as a function of position x at
various values of y for 0 < x < 1 cm.

The numerical solution proceeds by distributing N nodes uniformly in the y-direction across the
passage, as discussed in Section 5.5. The nodes are positioned in the center of N full sized
control volumes, as discussed in the text. The distance between adjacent nodes is:

H
Δy = (7)
N

and the location of each of the nodes is given by:

⎛ 1⎞
yi = Δy ⎜ i − ⎟ for i = 1..N (8)
⎝ 2⎠

% Setup y grid
N=101; % number of nodes in y direction (-)
Dy=H/N; % distance between nodes (m)
for i=1:N
y(i)=Dy*(i-1/2); % position of each node (m)
end

The velocity distribution in the duct is provided by Eq. Error! Reference source not found.:

% velocity distribution
for i=1:N
u(i)=6*u_m*(y(i)/H-(y(i)/H)^2); % velocity at each node (m/s)
end
The temperature distribution at the inlet to the duct is provided by Eq.
Error! Reference source not found.:

% Initial temperature distribution


for i=1:N
T(i,1)=T_in+(qf_avg*H/k)*(-y(i)^4/H^4+2*y(i)^3/H^3-y(i)/H+0.2429);
end

The axial location of each grid point is given by:

xi = ( j − 1) Δx for i = 1..M (9)

where M is the number of axial positions and Δx is the distance between axial positions, given
by:

L
Δx = (10)
( M − 1)
where L is the length of the duct to be simulated. The heat flux at each axial location is given by
Eq. Error! Reference source not found..

% setup x grid
M=2000; % number of length steps
Dx=L/(M-1); % size of length steps
for j=1:M
x(j)=(j-1)*Dx;
qfs(j)=qfs_f(x(j),qf_avg,Dqf,Lh);
end

A sub-function is defined to return the heat flux:

function[qf]=qfs_f(x,qf_avg,Dqf,Lh)

% Inputs:
% x - position (m)
% qf_avg - average heat flux (W/m^2)
% Dqf - amplitude of heat flux variation (W/m^2)
% Lh - distance of heated regions
%
% Outputs
% qf - heat flux (W/m^2)

qf=qf_avg+Dqf*sign(sin(2*pi*x/Lh));
end

The state equations are derived by defining a control volume around each node that extends a
finite spatial extent in the y-direction (Δy) but is differentially small in the x-direction (dx). The
energy balance for a control volume around an internal node is:
dTi
( ρ c ui ΔyW Ti ) x + qtop + qbottom = ( ρ c ui ΔyW Ti ) x + ρ c ui ΔyW dx (11)
dx

where W is the depth of the channel into the page. The conduction heat transfer rates are
approximated according to:

k dxW
qtop = (Ti +1 − Ti ) (12)
Δy

k dxW
qbottom = (Ti −1 − Ti ) (13)
Δy

Equations (12) and (13) are substituted into Eq. (11):

k dxW k dxW dT
(Ti +1 − Ti ) + (Ti −1 − Ti ) = ρ c ui ΔyW i dx for i = 2.. ( N − 1) (14)
Δy Δy dx

Solving for the rate of change of Ti with respect to x leads to:

dTi k
= (Ti +1 + Ti −1 − 2 Ti ) for i = 2.. ( N − 1) (15)
dx ρ c Δy 2 ui

An energy balance for the control volume around node 1 leads to:

k dxW dT
(T2 − T1 ) + qs′′W dx = ρ c u1 ΔyW 1 dx (16)
Δy dx

Solving for the rate of change of the temperature of node 1 leads to:

dT1 k q s′′
2 ( 2
= T − T1 ) + (17)
dx ρ c u1 Δy ρ c u1 Δy

A similar process applied to node N leads to:

dTN k qs′′
2 ( N −1
= T − TN ) + (18)
dx ρ c u1 Δy ρ c u1 Δy

Equations (16), (17), and (18) are the N state equations for the problem. According to the
problem statement, these equations must be integrated from the entrance of the heated region
downstream using the Crank-Nicolson technique in MATLAB. The formula for taking a Crank-
Nicolson step is:
⎡ dT dT ⎤ Δx
Ti , j +1 = Ti , j + ⎢ + ⎥ for i = 1... N (19)
⎣⎢ dx T =Ti , j , x = x j dx ⎥
T =Ti , j +1 , x = x j +1 ⎦ 2

Substituting Eqs. (16), (17), and (18) into Eq. (19) leads to:

T1, j +1 = T1, j +
k
⎡ ( T − T ) + ( T − T ) ⎤ +
(
Δx q s′′, x j + qs′′, x j+1 Δx ) (20)
ρ c u1 Δy 2 ⎣ 2, j 1, j 2, j +1 1, j +1 ⎦
2 ρ c u1 Δy 2

Δx
⎡(Ti +1, j + Ti −1, j − 2 Ti , j ) + (Ti +1, j +1 + Ti −1, j +1 − 2 Ti , j +1 )⎤
k
Ti , j +1 = Ti , j + ⎣ ⎦
ρ c Δy ui
2
2 (21)
for i = 2... ( N − 1)

TN , j +1 = TN , j +
k
⎡ ( T − T + T − T
Δx qs′′, x j + qs′′, x j+1 Δx
) ( N −1, j +1 N , j +1 )⎦ 2 + ρ c u Δy 2

( ) (22)
ρ c u N Δy 2 ⎣ N −1, j N , j N

Equations (20) through (22) are a set of N linear equations in the unknown temperatures Ti,j+1
where i = 1..N. These equations must be placed into matrix format in order to move forward a
length step:

AX =b (23)

where X is:

⎡ X 1 = T1, j +1 ⎤
⎢ ⎥
⎢ X 2 = T2, j +1 ⎥
X= (24)
⎢... ⎥
⎢ ⎥
⎢⎣ X N = TN , j +1 ⎥⎦

and A is:

⎡ row 1 = control volume 1 equation ⎤


⎢ row 2 = control volume 2 equation ⎥
⎢ ⎥
A = ⎢ row 3 = control volume 3 equation ⎥ (25)
⎢ ⎥
⎢... ⎥
⎢⎣ row N = control volume N equation ⎥⎦
Equations (20) through (22) are rearranged so that the coefficients multiplying the unknowns and
the constants for the linear equations are clear:

⎡ k Δx ⎤ ⎡ k Δx ⎤
T1, j +1 ⎢1 + 2⎥
+ T2, j +1 ⎢ − ⎥=
2 ρ c u1 Δy ⎦
⎣
⎣ 2 ρ c u1 Δy 2 ⎦

A1,1 A1,2

( )
(26)
Δx Δx qs′′, x j + qs′′, x j+1
2 ( 2, j
T − T1, j )
k
T1, j + +
ρ c u1 Δy 2 ρ c u1 Δy 2


b1

⎡ k Δx ⎤ ⎡ k Δx ⎤ ⎡ k Δx ⎤
Ti , j +1 ⎢1 + ⎥ + Ti +1, j +1 ⎢ − ⎥ + Ti −1, j +1 ⎢ − =
⎣ ρ c Δy ui ⎦
2
⎣ 2 ρ c Δy ui ⎦
2
⎣ 2 ρ c Δy 2 ui ⎥⎦




Ai ,i Ai ,i +1 Ai ,i −1

k Δx
Ti , j + ⎡Ti +1, j + Ti −1, j − 2 Ti , j ⎤⎦ (27)
2 ρ c Δy 2 ui ⎣


bi

for i = 2... ( N − 1)

⎡ k Δx ⎤ ⎡ k Δx ⎤
TN , j +1 ⎢1 + 2⎥
+ TN −1, j +1 ⎢ − 2⎥
=
⎣ 2 ρ c u N Δy ⎦ ⎣ 2 ρ c u N Δy ⎦



AN , N AN , N −1

( )
(28)
Δx qs′′, x j + qs′′, x j+1 Δx
TN , j +
k
( T − T )
ρ c u N Δy 2 N −1, j N , j 2
+
ρ c u N Δy 2


bN

The matrix A and vector b are initialized:

A=spalloc(N,N,3*N); % initialize A
b=zeros(N,1); % initialize b

The matrix A can be constructed just once and used without modification to move through each
length step:

% setup A matrix
A(1,1)=1+k*Dx/(2*rho*c*Dy^2*u(1));
A(1,2)=-k*Dx/(2*rho*c*Dy^2*u(1));
for i=2:(N-1)
A(i,i)=1+k*Dx/(rho*c*Dy^2*u(i));
A(i,i+1)=-k*Dx/(2*rho*c*Dy^2*u(i));
A(i,i-1)=-k*Dx/(2*rho*c*Dy^2*u(i));
end
A(N,N)=1+k*Dx/(2*rho*c*Dy^2*u(N));
A(N,N-1)=-k*Dx/(2*rho*c*Dy^2*u(N));

The vector b is reconstructed during each step:

% step through space


for j=1:(M-1)
b(1)=T(1,j)+(k*Dx/(2*rho*c*Dy^2*u(1)))*(T(2,j)-...
T(1,j))+Dx*(qfs(j)+qfs(j+1))/(2*rho*c*u(1)*Dy);
for i=2:(N-1)
b(i)=T(i,j)+k*Dx*(T(i+1,j)+T(i-1,j)-...
2*T(i,j))/(2*rho*c*Dy^2*u(i));
end
b(N)=T(N,j)+(k*Dx/(2*rho*c*Dy^2*u(N)))*(T(N-1,j)-...
T(N,j))+Dx*(qfs(j)+qfs(j+1))/(2*rho*c*u(N)*Dy);
T(:,j+1)=A\b;
end

The temperature as a function of position for various values of y is shown in Figure P5.5-4(b).
307
y = 2.5 μm
306

305

304
Temperature (K)

303 y = 47 μm

302
y = 97 μm
301

300 y = 146 μm
y = 196 μm
299
y = 245 μm
298
0 0.002 0.004 0.006 0.008 0.01
Position (m)
Figure P5.5-4(b): Temperature as a function of position for various values of y.

Notice that the effect of the oscillation extends into the duct approximately 47 μm which is
consistent with the diffusion penetration depth associated with a single heated region:

Lh
δ diff = 2 α t = 2 α (29)
um

diff=2*sqrt(alpha*Lh/u_m) % diffusion distance into stream (m)

which leads to δdiff = 36 μm.

e.) Determine the surface temperature of the duct and the mean temperature of the fluid at each
axial position. Plot Ts and Tm as a function of position x.
The mean temperature is defined as the ratio of the energy carried by the flow to the capacitance
rate of the flow ( m c ):

N N

∑ ui Δy ρ cTi ∑ u Δy T
i i
Tm, j = i =1
= i =1
(30)
um H ρ c um H

The surface temperature of the duct is obtained by extrapolating from nodes 1 and 2:

Ts , j = T2, j + (T1, j − T2, j )


3
(31)
2

for j=1:M
E_dot=0;
for i=1:N
E_dot=E_dot+u(i)*T(i,j)*Dy;
end
T_mean(j)=E_dot/(u_m*H);
T_s(j)=T(2,j)+(T(1,j)-T(2,j))*3/2;
end

Figure 5.5-4(c) shows the mean temperature and the surface temperature as a function of
position.
307

306

305
Temperature (K)

304

surface temperature
303

302

301 mean temperature

300
0 0.002 0.004 0.006 0.008 0.01
Position (m)
Figure P5.5-4(c): Mean and surface temperature as a function of position.

f.) Determine the Nusselt number at each axial position. Plot the Nusselt number as a function
of x. Explain the shape of your plot.

The heat transfer coefficient is defined as:

q ′′
h= (32)
(Ts − Tm )
and the Nusselt number is computed according to:

h Dh
Nu = (33)
k

for j=1:M
htc(j)=qfs(j)/(T_s(j)-T_mean(j));
Nu(j)=htc(j)*Dh/k;
end

Figure 5.5-4(d) illustrates the Nusselt number as a function of position. The Nusselt number
spikes at the entrance to each heated region as the thermal boundary layer because the thermal
boundary layer is partially destroyed in each adiabatic region and so you have some developing
region.
22.5

20

17.5

15
Nusselt number

12.5

10

7.5

2.5

0
0 0.002 0.004 0.006 0.008 0.01
Position (m)
Figure P5.5-4(d): Nusselt number as a function of position.

g.) Verify that your model is working correctly by setting Δq ′′ = 0 and showing that the Nusselt
number in the duct is consistent with the Nusselt number for fully developed flow between
parallel plates subjected to a constant heat flux.

The amplitude of the variation is set to zero, leading to a uniform heat flux:

Dqf=0;%9500; % heat flux fluctuation amplitude (W/m^2)

The Nusselt number as a function of position is shown in Figure 5.5-4(e) and does limit to Nu =
8.235 which is the appropriate value for thermally and hydrodynamically fully developed flow
between parallel plates.
10

Nusselt number
6

0
0 0.002 0.004 0.006 0.008 0.01
Position (m)
Figure P5.5-4(e): Nusselt number as a function of position.

Return the value of Δq ′′ to 9500 W/m2. You are interested in studying the impact of the non-
uniform heat flux on the surface temperature of the duct. There are two natural limits to this
problem.
h.) Calculate the surface-to-mean temperature difference experienced when the average heat flux
is applied at the wall and the flow is fully developed.

The surface-to-mean temperature difference experienced when the average heat flux is applied at
the wall and the flow is fully developed is given by:

q ′′ Dh
ΔTq ′′ = (34)
Nu fd k

where Nufd is the fully developed Nusselt number (8.235):

DT_uniform=qf_avg*Dh/(Nu_fd*k)
% surface-to-fluid temperature difference for fully developed flow with
% average heat flux

which leads to ΔTq ′′ = 4.615 K.

i.) Calculate the surface-to-mean temperature difference experienced when the peak heat flux is
applied at the wall and the flow is fully developed.

The surface-to-mean temperature difference experienced when the peak heat flux is applied at
the wall and the flow is fully developed is given by:

ΔTq ′′+Δq ′′ =
( q′′ + Δq′′) Dh
(35)
Nu fd k
where Nufd is the fully developed Nusselt number.

DT_peak=(qf_avg+Dqf)*Dh/(Nu_fd*k)
% surface-to-fluid temperature difference for fully developed flow with
% peak heat flux

which leads to ΔTq ′′+Δq ′′ = 9.229 K.

j.) Define a meaningful dimensionless spatial period, Lh ,and plot the surface-to-mean
temperature difference as a function of dimensionless spatial period. Show that when Lh is
small, the solution limits to your answer from (h) and when Lh is large then your solution
limits to your solution from (i). Explain this result.

The dimensionless spatial period is the ratio of the period, Lh, to the axial distance required for
the energy to diffuse to the center of the passage, Lchar:

L
Lh = h (36)
Lchar

where

H 2 um
Lchar = (37)

L_char=H^2*u_m/(4*alpha) %characteristic length


Lh_bar=Lh/L_char % dimensionless spatial period

The function is modified so that it has inputs Lh and L and returns the maximum surface-to-
mean temperature difference, ΔTmax, calculated according to:

DT=T_s-T_mean;
DT_max=max(DT);

The function listing is:

function[x,y,T,qfs,T_mean,T_s,htc,Nu,DT_max,Lh_bar]=P5p5d4(Lh,L)

% Inputs:
% Lh - length of heated/adiabatic regions (m)
% L - length of duct to simulate (m)
%
% Outputs:
% x - vector of axial positions (m)
% y - vector of y-positions (m)
% T - matrix of temperature solutions (K)
% qfs - vector heat flux at duct surface (W/m^2)
% T_mean - vector of mean temperature (K)
% T_s - vector of duct surface temperature (K)
% htc - heat transfer coefficient (W/m^2-K)
% Nu - Nusselt number (-)
% T_s_conv - surface temperature predicted using fully developed Nusselt
% number (K)
% DT_max - maximum surface-to-mean temperature difference in duct (K)
% Lh_bar - dimensionless spatial period (-)

u_m=0.2; % mean velocity (m/s)


H=0.0005; % duct height (m)
T_in=300; % inlet mean fluid temperature (K)
rho=1000; % density (kg/m^3)
mu=0.05; % viscosity (Pa-s)
k=0.25; % conductivity (Pa-s)
c=3800; % specific heat capacity (J/kg-K)
%L=0.01; % length of duct (m)
qf_avg=9500; % average heat flux (W/m^2)
Dqf=9500; % heat flux fluctuation amplitude (W/m^2)
%Lh=0.001; % length of heated section (m)
alpha=k/(rho*c) % thermal diffusivity (m^2/s)

Dh=2*H; % hydraulic diameter


Re=u_m*Dh*rho/mu % Reynolds number (-)
Nu_fd=8.235; % fully developed Nusselt number (-)
DT_uniform=qf_avg*Dh/(Nu_fd*k)
% surface-to-fluid temperature difference for fully developed flow with
% average heat flux
DT_peak=(qf_avg+Dqf)*Dh/(Nu_fd*k)
% surface-to-fluid temperature difference for fully developed flow with
% peak heat flux
Br=mu*u_m^2/(k*DT_uniform) % Brinkman number
Pr=mu*c/k % Prandtl number (-)
Pe=Re*Pr % Peclet number

diff=2*sqrt(alpha*Lh/u_m) % diffusion distance into stream (m)


L_char=H^2*u_m/(4*alpha) %characteristic length
Lh_bar=Lh/L_char % dimensionless spatial period

% Setup y grid
N=101; % number of nodes in y direction (-)
Dy=H/N; % distance between nodes (m)
for i=1:N
y(i)=Dy*(i-1/2); % position of each node (m)
end

% velocity distribution
for i=1:N
u(i)=6*u_m*(y(i)/H-(y(i)/H)^2); % velocity at each node (m/s)
end

% Initial temperature distribution


for i=1:N
T(i,1)=T_in+(qf_avg*H/k)*(-y(i)^4/H^4+2*y(i)^3/H^3-y(i)/H+0.2429);
end
% setup x grid
M=2000; % number of length steps
Dx=L/(M-1); % size of length steps
for j=1:M
x(j)=(j-1)*Dx;
qfs(j)=qfs_f(x(j),qf_avg,Dqf,Lh);
end

A=spalloc(N,N,3*N); % initialize A
b=zeros(N,1); % initialize b

% setup A matrix
A(1,1)=1+k*Dx/(2*rho*c*Dy^2*u(1));
A(1,2)=-k*Dx/(2*rho*c*Dy^2*u(1));
for i=2:(N-1)
A(i,i)=1+k*Dx/(rho*c*Dy^2*u(i));
A(i,i+1)=-k*Dx/(2*rho*c*Dy^2*u(i));
A(i,i-1)=-k*Dx/(2*rho*c*Dy^2*u(i));
end
A(N,N)=1+k*Dx/(2*rho*c*Dy^2*u(N));
A(N,N-1)=-k*Dx/(2*rho*c*Dy^2*u(N));

% step through space


for j=1:(M-1)
b(1)=T(1,j)+(k*Dx/(2*rho*c*Dy^2*u(1)))*(T(2,j)-...
T(1,j))+Dx*(qfs(j)+qfs(j+1))/(2*rho*c*u(1)*Dy);
for i=2:(N-1)
b(i)=T(i,j)+k*Dx*(T(i+1,j)+T(i-1,j)-...
2*T(i,j))/(2*rho*c*Dy^2*u(i));
end
b(N)=T(N,j)+(k*Dx/(2*rho*c*Dy^2*u(N)))*(T(N-1,j)-...
T(N,j))+Dx*(qfs(j)+qfs(j+1))/(2*rho*c*u(N)*Dy);
T(:,j+1)=A\b;
end

for j=1:M
E_dot=0;
for i=1:N
E_dot=E_dot+u(i)*T(i,j)*Dy;
end
T_mean(j)=E_dot/(u_m*H);
T_s(j)=T(2,j)+(T(1,j)-T(2,j))*3/2;
end
for j=1:M
htc(j)=qfs(j)/(T_s(j)-T_mean(j));
Nu(j)=htc(j)*Dh/k;
end
DT=T_s-T_mean;
DT_max=max(DT);
end

function[qf]=qfs_f(x,qf_avg,Dqf,Lh)

% Inputs:
% x - position (m)
% qf_avg - average heat flux (W/m^2)
% Dqf - amplitude of heat flux variation (W/m^2)
% Lh - distance of heated regions
%
% Outputs
% qf - heat flux (W/m^2)

qf=qf_avg+Dqf*sign(sin(2*pi*x/Lh));
end

A script is written to call the function for a range of Lh (note that the simulated length of the duct
is always L = 10 Lh in order to capture several heated regions.

Lh=[0.00001,0.00002,0.00004,0.00007,0.0001,0.0002,0.0004,0.0007,0.001,...
0.002,0.004,0.007,0.01,0.02,0.04,0.07,0.1,0.2,0.4,0.7,1];
for i=1:21
i
[x,y,T,qfs,T_mean,T_s,htc,Nu,T_s_conv,DT_max(i),Lh_bar(i)]=...
P5p5d4(Lh(i),10*Lh(i));
end

Figure 5.5-4(f) shows the maximum surface-to-mean temperature difference as a function of the
dimensionless period of the heated regions. Notice that when Lh is small, the energy from each
region does not diffuse substantially into the passage and therefore the answer limits to ΔTq ′′ =
4.615 K (the solution for a uniform heat flux is valid). In the other extreme, if L is large, then h

the reaches a new fully developed state during each heated region and the answer limits to
ΔTq ′′+Δq ′′ = 9.229 K.
Maximum surface-to-mean temp. difference (K)

10

4
0.0001 0.001 0.01 0.1 1 10
Dimensionless period of heated region

Figure P5.5-4(f): Maximum surface-to-mean temperature difference as a function of the dimensionless heated
length.
Problem 5.5-5
Argon at m = 0.002 kg/s flows through a pipe with inner diameter D = 0.375 inch. The argon
has been heated by an electrical heat input that is applied uniformly to the external surface of the
pipe with heat flux q ′′ = 500 W/m2 and is fully developed thermally and hydrodynamically. The
argon has mean temperature Tin = 100 K and pressure p = 100 atm when it leaves the heated
section of the pipe and enters an insulated section. You want to measure the mean temperature
of the argon by placing a temperature sensor at the center of the pipe (assume that the
temperature sensor measures the temperature at r = 0 and does not disturb the flow).
a.) Develop a numerical model of the argon flowing in the insulated section of the tube.
Implement your model in MATLAB using the ode solver.

The inputs are entered in a MATLAB function, P5p5d5.m:

function[x,r,T]=P5p5d5()

% Inputs:
%
% Outputs:
% x - vector of axial positions (m)
% r - vector of r-positions (m)
% T - matrix of temperature solutions (K)

m_dot=0.002; % mass flow rate (kg/s)


rho=1350; % density (kg/m^3)
mu=0.000205; % viscosity (Pa-s)
k=0.116; % conductivity (Pa-s)
c=1093; % specific heat capacity (J/kg-K)
D=0.00953; % inner diameter of duct (m)
T_in=100; % mean temperature leaving the heater (K)
qf=500; % heat flux in heater (W/m^2)
L=1; % length of duct (m)
alpha=k/(rho*c); % thermal diffusivity (m^2/s)

The mean velocity is calculated according to:

4 m
um = (1)
ρ π D2

and the Reynolds number is:

um D ρ
Re = (2)
μ

u_m=m_dot/(rho*pi*D^2/4); % mean velocity (m/s)


Re=u_m*D*rho/mu % Reynolds number (-)

which leads to Re = 1300 which indicates that the flow is likely laminar.
The nodes are distributed radially according to:

⎛ 1⎞
ri = Δr ⎜ i − ⎟ for i = 1..N (3)
⎝ 2⎠

where

D
Δr = (4)
2N

% Setup r grid
N=21; % number of nodes in y direction (-)
Dr=D/(2*N); % distance between nodes (m)
for i=1:N
r(i)=Dr*(i-1/2); % position of each node (m)
end

The velocity distribution for a fully developed laminar flow is:

⎡ ⎛ ri ⎞2 ⎤
ui = 2 um ⎢1- ⎜ ⎟ ⎥ for i = 1..N (5)
⎣⎢ ⎝ R ⎠ ⎦⎥

% velocity distribution
for i=1:N
u(i)=2*u_m*(1-(2*r(i)/D)^2); % velocity at each node (m/s)
end

From Section 5.4.3, the temperature distribution for fully developed laminar flow in a round pipe
exposed to a constant surface heat flux is:

8 qs′′ ⎡ r 2 r4 ⎤ 3 q ′′ D
T= ⎢ − 2⎥
+ Ts − s (6)
k D ⎣ 4 4D ⎦ 8k

where Ts is the surface temperature of the tube. The surface temperature of the tube can be
evaluated according to:

11 D q s′′
Tin = Ts − (7)
48 k

The temperature of each node at the inlet to the unheated portion of the tube is computed
according to Eq. (6):

% Initial temperature distribution


T_s=T_in+11*D*qf/(48*k);
for i=1:N
T_ini(i,1)=T_s+(8*qf/(k*D))*(r(i)^2/4-r(i)^4/(4*D^2))-3*qf*D/(8*k);
end

An energy balance on the internal nodes leads to:

⎡⎛
ρ c ui π ⎢⎜ ri +
Δr ⎞ ⎛
2
Δr ⎞
2
⎤ dTi (T − T ) (T − T )
= 2 π k i +1 i + 2 π k i −1 i for i = 2.. ( N − 1) (8)
⎟ − ⎜ ri − ⎟ ⎥
⎢⎣⎝ 2 ⎠ ⎝ 2 ⎠ ⎥⎦ dx ⎛r ⎞ ⎛ r ⎞
ln ⎜ i +1 ⎟ ln ⎜ i ⎟
⎝ ri ⎠ ⎝ ri −1 ⎠

which is solved for the state equations for the internal nodes:

⎡ ⎤
⎢ ⎥
⎢ 2 k (Ti +1 − Ti ) + 2 k (Ti −1 − Ti ) ⎥
⎢ ⎛r ⎞ ⎛ r ⎞⎥
⎢ ln ⎜ i +1 ⎟ ln ⎜ i ⎟ ⎥
dTi ⎣⎢ ⎝ ri ⎠ ⎝ ri −1 ⎠ ⎦⎥
= for i = 2.. ( N − 1) (9)
dx ⎡⎛ Δr ⎞ ⎛
2
Δr ⎞ ⎤
2

ρ c ui ⎢⎜ ri + ⎟ − ⎜ ri − ⎟ ⎥
⎢⎣⎝ 2 ⎠ ⎝ 2 ⎠ ⎥⎦

An energy balance on node 1 leads to:

⎡⎛ Δr ⎞ ⎛
2
Δr ⎞ ⎤ dT1
ρ c u1 π ⎢⎜ r1 + ⎟ − ⎜ r1 − ⎟ ⎥
2
(T − T )
= 2π k 2 1 (10)
⎣⎢⎝ 2 ⎠ ⎝ 2 ⎠ ⎦⎥ dx ⎛r ⎞
ln ⎜ 2 ⎟
⎝ r1 ⎠

which is solved for the state equation for node 1:

dT1
=
2k (T2 − T1 ) (11)
dx ⎡⎛ Δr ⎞ ⎛
2
Δr ⎞ ⎤ ⎛ r ⎞
2

ρ c u1 ⎢⎜ r1 + ⎟ − ⎜ r1 − ⎟ ⎥ ln ⎜ 2 ⎟
⎣⎢⎝ 2 ⎠ ⎝ 2 ⎠ ⎥⎦ ⎝ r1 ⎠

An energy balance on node N leads to:

⎡⎛ Δr ⎞ ⎛
2
Δr ⎞ ⎤ dTN
ρ c u N π ⎢⎜ rN + ⎟ − ⎜ rN − ⎟ ⎥
2
(T − T )
= 2 π k N −1 N (12)
⎢⎣⎝ 2 ⎠ ⎝ 2 ⎠ ⎥⎦ dx ⎛ r ⎞
ln ⎜ N ⎟
⎝ rN −1 ⎠

which is solved for the state equation for node N:


dTN
=
2k (TN −1 − TN ) (13)
dx ρ c u N ⎡⎛ Δr ⎞ ⎛
2
Δr ⎞ ⎤ ⎛ rN ⎞
2

⎢⎜ rN + ⎟ − ⎜ rN − ⎟ ⎥ ln ⎜ ⎟
⎣⎢⎝ 2 ⎠ ⎝ 2 ⎠ ⎦⎥ ⎝ rN −1 ⎠

The state equations are programmed in a function dTdt_p5p5d3:

function[dTdx]=dTdt_P5p5d5(x,T,u,r,rho,k,c,N,Dr)
% Inputs
% x - axial location (m)
% T - vector of nodal temperatures (K)
% u - vector of nodal velocities (m/s)
% r - vector of nodal radii (m)
% rho - density (kg/m^3)
% k - conductivity (W/m-K)
% c - specific heat capacity (J/kg-K)
% N - number of nodes (-)
% Dr - distance between adjacent nodes (m)
%
% Output
% dTdx - rate of temperature change (K/m)

dTdx=zeros(N,1);
dTdx(1)=2*k*(T(2)-T(1))/(((r(1)+Dr/2)^2-...
(r(1)-Dr/2)^2)*rho*c*u(1)*log(r(2)/r(1)));
for i=2:(N-1)
dTdx(i)=2*k*((T(i-1)-T(i))/log(r(i)/r(i-1))+(T(i+1)-T(i))/...
log(r(i+1)/r(i)))/(((r(i)+Dr/2)^2-(r(i)-Dr/2)^2)*rho*c*u(i));
end
dTdx(N)=2*k*(T(N-1)-T(N))/(((r(N)+Dr/2)^2-(r(N)-Dr/2)^2)*...
rho*c*u(N)*log(r(N)/r(N-1)));
end

and integrated using the ode45 solver:

OPTIONS=odeset('RelTol',1e-6);
[x,T]=ode45(@(x,T) dTdt_P5p5d5(x,T,u,r,rho,k,c,N,Dr),...
[0,L],T_ini,OPTIONS);
end

b.) Plot the temperature as a function of position at various radii.

Figure 1 shows the temperature as a function of axial position for various values of radius.
110

108

106
r = D/2

Temperature (K)
104

102

100

98

96
r=0
94
0 0.2 0.4 0.6 0.8 1
Axial position (m)
Figure 1: Temperature as a function of axial position for various values of radius.

c.) Plot the error between the temperature measured by your sensor and the mean temperature of
the argon as a function of the axial location of the sensor.

The measurement error is the difference between the temperature at r = 0 and the mean
temperature, which is always Tin. Figure 2 illustrates Tr = 0 - Tin as a function of axial position.

0
Temperature measurement error (K)

-1

-2

-3

-4

-5

-6
0 0.2 0.4 0.6 0.8 1
Axial position (m)
Figure 2: Temperature measurement error as a function of axial position.
Problem 5.1-4 (5-1 in text)
Fluid flows through a tube with inner diameter D. You have been asked to help interpret some
measured data for flow through the tube. Specifically, the heat transfer coefficient, h, has been
measured as a function of the tube diameter. Figure P5.1-4 illustrates the heat transfer
coefficient measured in the thermally fully developed region of the tube as a function of the tube
diameter; note that the mass flow rate of fluid ( m ), the type of fluid, and all other aspects of the
experiment are not changed for these measurements.
Fully developed heat transfer coef. (W/m -K) 6
10
2

5
10

4
10

3
10

2
10

1
10
0 1 2 3 4 5 6 7 8 9 10
Diameter (cm)
Figure P5.1-4: Heat transfer coefficient as a function of diameter (for a constant fluid mass flow rate and
fluid type and all other aspects of the problem held constant.)

a.) Explain in a few sentences the abrupt change in the heat transfer coefficient observed that
occurs at approximately D = 5.5 cm.

The Reynolds number is given by:

ρ um D 4 m
Re = = (1)
μ π Dμ

therefore, for a constant mass flow rate, the Reynolds number increases as the diameter is
reduced. The abrupt change in heat transfer coefficient at D = 4.2 cm must be related to the
transition from laminar to turbulent flow. In laminar flow, the heat must be conducted,
approximately, to the center of the tube whereas the heat must only be conducted across the
viscous sublayer in a turbulent flow; thus, the heat transfer coefficient is much larger in turbulent
as opposed to laminar flow.

b.) Explain in a few sentences why the heat transfer is inversely proportional to diameter for
diameters above about D = 5.5 cm; that is, why is it true that h ∝ D −1 for D > 5.5 cm?

The flow for a diameter above 4.2 cm must be laminar. The heat transfer coefficient for a
laminar flow is approximately equal to:
k
h= (2)
δt

where δt is the thermal boundary layer thickness. In the thermally fully developed region, the
thermal boundary layer thickness is constant and proportional to the tube diameter; therefore:

1
h∝ (3)
D

c.) Sketch your expectation for how Figure P5.1-4 would change if the roughness of the tube
wall is increased dramatically.

The pipe wall roughness affects the heat transfer coefficient for a turbulent flow much more than
laminar flow because the critical conduction length is the viscous sublayer which can approach
the scale of roughness elements. Therefore, you should expect that the heat transfer coefficient
for diameters less than 4.2 cm will increase as the tube roughness increases (see Fig. 2) while the
tube roughness will have essentially no effect for diameters above 4.2 cm.

107
Fully developed heat transfer coef. (W/m -K)
2

106

105

104 with increased roughness

103

102

101
0 1 2 3 4 5 6 7 8 9 10
Diameter (cm)
Figure 2: Sketch of heat transfer coefficient with increased roughness.
Problem 5.1-6 (5-2 in text)
Figure P5.1-6 shows the flow of a fluid with Prandtl number, Pr < 1, through a pipe.

Tin

surface heat flux


b
q′′s

0 x
Figure P5.1-6: Pipe with a surface heat flux that depends on position.

The fluid becomes thermally fully developed at location x = b. The flow of the fluid is laminar.
a.) Sketch the thermal and momentum boundary layer thickness as a function of position (δt and
δm – be sure to clearly show which is which). Label the hydrodynamic and thermal entry
lengths, xfd,t and xfd,h, in your sketch. Show the location x = b in your sketch.

Figure 2: Sketch of thermal and momentum boundary layer thickness as a function of position.

The thermal boundary layers growing from each edge of the pipe will meet at approximately x =
b when the flow becomes thermally fully developed. The momentum boundary layers will grow
more slowly since the fluid has a small Prandtl number (i.e., it is better at transferring energy
than momentum). Therefore, the flow will become hydrodynamically fully developed at x > b.
Figure 2 reflects this behavior.

b.) Sketch the local and average heat transfer coefficient, h and h , as a function of x; indicate on
your sketch the location x = b.
Figure 3: Qualitative sketch of the local and average heat transfer coefficients, h and h , respectively, as a
function of the position from the pipe inlet.

The thermal boundary layer develops from the entrance of the pipe and joins at the center at xfd,t
= b (see Figure 2). The heat transfer coefficient for a laminar flow is inversely proportional to
the boundary layer thickness and will therefore begin high and drop; when the flow becomes
fully developed the heat transfer coefficient will become constant. This is shown in Figure 3.

The average heat transfer coefficient is somewhat larger than the local heat transfer coefficient
because it is integrated from the inlet of the pipe and therefore "remembers" the regions of higher
heat transfer coefficient. This is also shown in Figure 3.

c.) Sketch the local and average friction factor, f and f , as a function of x; indicate on your
sketch the location x = b.
Figure 4: Qualitative sketch of the local and average friction factors, f and f , respectively, as a function of
the position from the pipe inlet.

The momentum boundary layer develops from the entrance of the pipe and joins at the center at
xfd,m > b (see Figure 2). The friction factor for a laminar flow is inversely proportional to the
momentum boundary layer thickness and will therefore begin high and drop; when the flow
becomes fully developed the heat transfer coefficient will become constant. This is shown in
Figure 4.

The average friction factor is somewhat larger than the local friction factor because it is
integrated from the inlet of the pipe and therefore "remembers" the regions of higher pressure
gradient. This is also shown in Figure 4.

Figure P5.1-6 shows that a non-uniform heat flux is applied to the surface of the pipe. The heat
flux decreases linearly from x = 0 to x = b and remains at 0 for all subsequent x. The fluid enters
the pipe with mean temperature, Tin.
d.) Sketch the mean temperature of the fluid as a function of position.
Figure 5: Qualitative sketch of the mean temperature of the fluid (Tm) and the surface temperature of the
pipe (Ts) as a function of the position from the pipe inlet.

A differential energy balance on the fluid leads to:

dTm
m c = qs′′ ( x ) per (1)
dx

so the rate at which the mean temperature rises will be proportional to the rate at which energy is
added by the heat flux. The mean temperature will rise quickly at x = 0 and the rate of
temperature change will drop off to zero at x = b. This is shown in Figure 5.

e.) Sketch the surface temperature of the pipe as a function of position.

The surface temperature of the pipe is greater than the mean temperature of the fluid according
to:

qs′′
Ts = Tm + (2)
h

Figure 5.1-6 shows that the heat flux will be 0 for all x > b and therefore we expect that Ts = Tm
for x > b. Figure 3 shows that the local heat transfer at the inlet to the pipe will be very high
because the thermal boundary layer is so small. Therefore, at x = 0 we expect that Ts = Tm.
Between x = 0 and x = b the temperature difference must first increase to a maximum value and
then decrease. This behavior is shown in Figure 5.
Problem 5.3-1 (5-3 in text): Portable Metabolic Heat Removal System
Dismounted soldiers and emergency response personnel are routinely exposed to high
temperature/humidity environments as well as external energy sources such as flames, motor
heat or solar radiation. The protective apparel required by chemical, laser, biological, and other
threats tend to have limited heat removal capability. These and other factors can lead to severe
heat stress. One solution is a portable, cooling system integrated with an encapsulating garment
to provide metabolic heat removal. A portable metabolic heat removal system that is acceptable
for use by a dismounted soldier or emergency response personnel must satisfy a unique set of
criteria. The key requirements for such a system is that it be extremely low mass very compact
in order to ensure that any gain in performance due to active cooling is not offset by fatigue
related to an increase in pack load. In order to allow operation for an extended period of time, a
system must either be passive (require no consumable energy source), very efficient (require
very little consumable energy), or draw energy from a high energy density power source. One
alternative for providing portable metabolic heat removal is with an ice pack, as shown in Figure
P5.3-1.

ice pack Tout = 30°C vest

Tin = 5°C
W p pump

battery
L = 2.5 m
e=0
Din = 2.5 mm
Figure P5.3-1: Schematic of a portable metabolic heat removal system that utilizes an ice pack.

The pump forces a liquid antifreeze solution to flow through plastic tubes in the vest in order to
transfer the cooling from the ice to the person. Assume that the surface of the plastic is
completely smooth, e = 0, the total length of the tube is L = 2.5 m and the inner diameter of the
tube is Din = 2.5 mm. There are Nb = 20 bends in the vest; the loss coefficient associated with
each bend is Cb = 1.0. The fluid that is being circulated through the vest has properties ρf = 1110
kg/m3, cf = 2415 J/kg-K, μf = 0.0157 Pa-s, and Prf = 151. The fluid enters the vest at Tin= 5.0°C
and leaves the vest at Tout = 30°C. You may assume that the pressure drop associated with the
vest is much greater than the pressure drop associated with any other part of the system.
a.) Assume that the bulk velocity in the tube is um = 1.0 m/s. Determine the pressure drop
required to circulate the fluid through the vest.
The inputs are entered in EES:

$UnitSystem SI MASS RAD PA K J


$TABSTOPS 0.2 0.4 0.6 0.8 3.5 in

"Inputs"
L=2.5 [m] "length of tube"
D_i=2.5 [mm]*convert(mm,m) "inner diameter of tube"
rho_f=1110 [kg/m^3] "density of fluid"
c_f=2415 [J/kg-K] "specific heat capacity of fluid"
mu_f=0.0157 [Pa-s] "viscosity of fluid"
Pr_f=151 [-] "Prandtl number of fluid"
T_out=converttemp(C,K,30) "temperature of fluid leaving vest"
T_in=converttemp(C,K,5) "temperature of fluid entering vest"
N_b=20 [-] "number of bends"
C_b=1.0 [-] "loss coefficient of the bends"
e=0 [m] "roughness of tube"

The Reynolds number in the tube is computed:

ρ f Di um
Re = (1)
μf

and used to access the correlations for internal flow in a tube (via the PipeFlow_N procedure in
EES) in order to obtain the friction factor, f. The friction factor and loss coefficient for the bends
are used to compute the pressure drop:

ρ f um2 ⎛ L ⎞
Δp = ⎜f + N b Cb ⎟ (2)
2 ⎝ Di ⎠

u_m=1.0 [m/s] "initial guess for bulk velocity"


Re=u_m*D_i*rho_f/mu_f "Reynolds number"
call PipeFlow_N(Re,Pr_f,L/D_i,e/D_i: Nusselt_T,Nusselt_H,f) "call correlation"
DeltaP=rho_f*u_m^2*(f*L/D_i+N_b*C_b)/2 "pressure drop"
DeltaP_psig=DeltaP*convert(Pa,psi) "pressure drop in psi"

which leads to Δp = 213 kPa (30.9 psi).

A miniature diaphragm pump is used to circulate the fluid. Assume that the pressure rise
produced by the pump (Δpp) varies linearly from the dead head pressure rise Δpp,dh = 30 psi at no
flow ( Vp = 0) to zero at the maximum unrestricted flow rate Vp ,open = 650 mL/min .

⎛ V ⎞
Δp p = Δp p ,dh ⎜ 1 − p ⎟
⎜ Vp ,open ⎟
⎝ ⎠
b.) Determine the fluid flow rate through the vest that is consistent with the pump curve given by
the equation above; that is, vary the value of the mean velocity, um, until the pressure drop
across the vest and flow rate through the vest falls on the pump curve.

The volume flow rate through the tube is computed:

D2
V = um π i (3)
4

V_dot=u_m*pi*D_i^2/4 "volumetric flow rate"


V_dot_mLmin=V_dot*convert(m^3/s,mL/min) "volumetric flow rate (mL/min)"
The pressure rise that can be provided by the pump at the same volume flow rate is computed
using the linear pump curve:

⎛ V ⎞
Δp p = Δp p ,dh ⎜ 1 −
⎜ Vp ,open ⎟⎟
(4)
⎝ ⎠

DeltaP_pump_dh=30 [psi]*convert(psi,Pa) "dead head pressure rise"


V_dot_open=650 [mL/min]*convert(mL/min,m^3/s) "unrestricted flow rate"
DeltaP_pump=DeltaP_pump_dh*(1-V_dot/V_dot_open)
"pump curve - pressure rise produced by pump"

The guess values for all of the variables are updated and then the assumed value of um is
commented out and the pressure drop across the vest is constrained to be equal to the pressure
rise produced by the pump.

{u_m=1.0 [m/s]} "initial guess for bulk velocity"


DeltaP=DeltaP_pump
"find u_m that is consistent with pump and system resistance curves"

which leads to V = 3.35x10-6 m3/s (201 mL/min).

c.) How much cooling is provided by the vest?

The mass flow through the vest is:

m = V ρ f (5)

and an energy balance on the vest leads to:

q = m c f (Tout − Tin ) (6)

m_dot=V_dot*rho_f "mass flow rate"


q_dot=m_dot*c_f*(T_out-T_in) "cooling provided"

which leads to q = 225 W.

d.) If the pump efficiency is ηp= 0.20 then how much power is consumed by the pump?

The pump power is computed according to:

V Δp
W p = (7)
ηp

eta_p=0.2 [-] "pump efficiency"


W_dot_pump=V_dot*DeltaP/eta_p "pump power"
which leads to W p = 2.4 W.

e.) If the system is run for time = 1 hour then what is the mass of ice that is consumed? (Assume
that the latent heat of fusion associated with melting ice is ifs = 3.33x105 J/kg and that the
only energy transfer to the ice is from the fluid.) What is the mass of batteries that are
consumed, assuming that the energy density of a lead acid battery is edb = 0.05 kW-hr/kg.

The mass of ice required is obtained from an energy balance on the ice pack:

q time = h fs M ice (8)

time=1 [hr]*convert(hr,s) "operating time"


h_fs=333e3 [J/kg] "latent heat of fusion for ice"
q_dot*time=h_fs*M_ice "mass of ice"

which leads to Mice = 2.43 kg. The mass of batteries required is obtained from an energy balance
on the battery:

W p time = edb M b (9)

ed_b=0.05 [kW-hr/kg]*convert(kW-hr/kg,J/kg) "energy density of lithium battery"


W_dot_pump*time=ed_b*M_b "mass of batteries"

which leads to Mb = 0.048 kg.


Problem 5.3-3 (5-4 in text): Oxygen Fuel Tank
One concept for rapidly launching small satellites involves a rocket boosted, expendable launch
vehicle which is dropped from the cargo bay of a military cargo aircraft. The launch vehicle is
propelled by self-pressurized tanks of liquid oxygen and liquid propane. The liquid oxygen fuel
tank (referred to as the propellant tank) is at elevated pressure and must be kept full while the
aircraft sits on the runway, flies to the launch coordinates, and potentially holds position in order
to wait for a strategically appropriate launch time; the design requires that the propellant tank
remain full for timewait = 12 hours. The propellant tank contains saturated liquid oxygen at ptank =
215 psia. Saturated liquid oxygen at this pressure has a temperature of Ttank = 126.8 K. Because
the tank is so cold, it is subjected to a large heat leak, qtank . Without external cooling, it would
be necessary to vent the liquid oxygen that boils off in order to maintain the proper pressure and
therefore the tank would slowly be emptied. It is not possible to place a cryogenic refrigerator
with 20 kW of cooling capacity in the propellant tanks in order to re-liquefy the oxygen. Rather,
an adjacent dewar of liquid oxygen (referred to as the conditioning tank) is used to remove the
parasitic heat transfer and prevent any oxygen in the propellant tank from boiling away. Figure
P5.3-3 illustrates the proposed system. A pump is used to circulate liquid oxygen from the
propellant tank through a cooling coil that is immersed in the conditioning tank. The pump and
conditioning tank can be quickly removed from the launch vehicle when it is time for launch.
The conditioning tank is maintained at pct = 14.7 psia and contains saturated liquid oxygen; any
oxygen that evaporates due to the heat added by the cooling coil is allowed to escape. The
cooling coil is a coiled up tube with total length L = 10 m, inner diameter Di = 0.8 cm and outer
diameter Do = 1.0 cm. The internal surface of the tube has roughness e = 50 μm and the
conductivity of the tube material is ktube = 2.5 W/m-K. The mass flow rate provided by the pump
is m = 0.25 kg/s and the pump efficiency is ηpump= 0.45. The heat transfer coefficient associated
with the evaporation of the liquid oxygen in the conditioning tank from the external surface of
the tube is ho = 2x104 W/m2-K. You may assume that the liquid oxygen that is pumped through
the cooling coil has constant properties that are consistent with saturated liquid oxygen at the
tank pressure.

heat transfer liquid oxygen


from propane ptank = 215 psia
(tank) Ttank = 126.8 K
(3) pump
vapor boil-off from (2) pump power
conditioning tank

cooling coil liquid oxygen in


(ct)
conditioning tank
pct = 14.7 psia
Figure P5.3-3: Liquid from the propellant tank is pumped through a coil immersed in the conditioning tank.

a.) What is the pressure drop associated with forcing the liquid oxygen through the cooling coil?

The inputs are entered in EES:


$UnitSystem SI MASS RAD PA K J
$TABSTOPS 0.2 0.4 0.6 0.8 3.5 in

"Inputs"
P_tank=215 [psi]*convert(psi,Pa) "pressure of propellant tank"
L=10 [m] "length of cooling coil"
D_i=0.8 [cm]*convert(cm,m) "inner diameter of cooling coil"
e=50 [micron]*convert(micron,m) "roughness of internal surface of
cooling coil"
D_o=1.0 [cm]*convert(cm,m) "outer diameter of cooling coil"
k_tube=2.5 [W/m-K] "conductivity of the tube"
h_o=20e3 [W/m^2-K]
"heat transfer coefficient associated with evaporation on outer surface of the tube"
P_ct=14.7 [psi]*convert(psi,Pa) "conditioning tank pressure"
eta_pump=0.45 [-] "pump efficiency"
m_dot=0.25 [kg/s] "mass flow rate through cooling coil"
time_wait=12 [hr]*convert(hr,s) "hold time"

The properties of the liquid oxygen that is being pumped through the coil (ρtank, μtank, ktank, Prtank,
ctank) are computed using EES’ internal fluid property routines.

T_tank=temperature('Oxygen',P=P_tank,x=0) "temperature of oxygen"


rho_tank=density('Oxygen',P=P_tank,x=0) "density of oxygen"
mu_tank=viscosity('Oxygen',P=P_tank,x=0) "viscosity of oxygen"
k_tank=conductivity('Oxygen',P=P_tank,x=0) "conductivity of oxygen"
Pr_tank=Prandtl('Oxygen',P=P_tank,x=0) "Prandtl number of oxygen"
c_tank=cP('Oxygen',P=P_tank,x=0) "specific heat capacity of oxygen"

The volumetric flow rate of liquid oxygen is:

m
V = (1)
ρtank

The bulk velocity of the liquid oxygen in the cooling coil is:

4 V
um = (2)
π Di2

and used to compute the Reynolds number associated with the internal flow in the tube:

um Di ρtank
Re = (3)
μtank

V_dot=m_dot/rho_tank "volumetric flow rate"


u_m=V_dot/(pi*D_i^2/4) "mean velocity in cooling coil"
Re=u_m*D_i*rho_tank/mu_tank "Reynolds number"
The correlations for internal flow in a circular tube are accessed using the PipeFlow_N procedure
in order to determine the friction factor and average Nusselt number (f and Nu ). The pressure
drop is calculated from the friction factor according to:

ρtank um2 f L
Δp = (4)
2 Di

call PipeFlow_N(Re,Pr_tank,L/D_i,e/D_i: Nusselt_T,Nusselt_H,f) "access correlations"


DeltaP=rho_tank*u_m^2*f*L/(2*D_i) "pressure drop"

which leads to Δp = 5.5x105 Pa.

b.) What is the power required by the pump?

The power required by the pump is computed according to:

V Δp
W pump = (5)
η pump

W_dot_pump=V_dot*DeltaP/eta_pump "pump power"

which leads to W pump = 330 W.

c.) If all of the pump power ultimately is transferred to the liquid oxygen that is being pumped
then what is the temperature of the liquid oxygen leaving the pump (T2 in Figure P5.3-3)?

The temperature of the liquid oxygen leaving the pump is obtained from:

W pump
T2 = T1 +
m ctank
(1 − η pump ) (6)

T_2=T_tank+W_dot_pump*(1-eta_pump)/(m_dot*c_tank) "temperature of fluid leaving pump"

which leads to T2 = 127.1 K.

d.) What is the heat transfer coefficient between the liquid oxygen flowing through the cooling
coil and the internal surface of the tube?

The Nusselt number is used to compute the heat transfer coefficient:

ktank
hi = Nu (7)
Di

h_i=k_tank*Nusselt_T/D_i "internal heat transfer coefficient"


which leads to hi = 2.8x104 W/m2-K.

e.) What is the total conductance associated with the cooling coil?

The total thermal resistance between the evaporating oxygen in the conditioning tank and the
fluid in the cooling coil includes the internal resistance to convection,

1
Rconv ,i = (8)
hi Di π L

conduction through the tube,

⎛D ⎞
ln ⎜ o ⎟
= ⎝ i⎠
D
Rcond (9)
ktube 2 π L

and convection from the external surface of the tube:

1
Rconv ,o = (10)
ho Do π L

R_conv_i=1/(L*pi*D_i*h_i)
"convection resistance on internal surface"
R_cond=ln(D_o/D_i)/(2*pi*k_tube*L) "conduction resistance through tube"
R_conv_o=1/(L*pi*D_o*h_o)
"convection resistance on external surface"

The total resistance is:

Rtotal = Rconv ,i + Rtube + Rconv ,o (11)

and the total conductance is:

1
UA = (12)
Rtotal

R_total=R_conv_i+R_cond+R_conv_o "total resistance"


UA=1/R_total "total conductance"

which leads to UA = 581 W/K.


f.) What is the temperature of the liquid oxygen leaving the cooling coil (T3 in Figure P5.3-3)?

The temperature of the liquid oxygen in the conditioning tank (Tct) is computed using EES
internal thermodynamic property routine. The temperature of the liquid oxygen leaving the
conditioning tank and returned to the propellant tank is computed according to:

⎛ UA ⎞
Tct − T3 = (Tct − T2 ) exp ⎜ − ⎟ (13)
⎝ m ctank ⎠

T_ct=temperature('Oxygen',P=P_ct,x=0)
"temperature of oxygen in conditioning tank"
(T_ct-T_3)=(T_ct-T_2)*exp(-UA/(m_dot*c_tank)) "exit temperature"

which leads to T3 = 102.1 K.

g.) How much cooling is provided to the propellant tank?

The cooling provided to the propellant tank is obtained using an energy balance on the propellant
tank:

qtank = m ctank (Ttank − T3 ) (14)

q_dot_tank=m_dot*c_tank*(T_tank-T_3) "cooling provided to propellant tank"

which leads to qtank = 12600 W.

h.) Plot the cooling provided to the propellant tank and the pump power as a function of the
mass flow rate. If the parasitic heat leak to the propellant tank is qtank = 10 kW then suggest
the best mass flow rate to use for the system.

Figure 2 illustrates the cooling and pump power as a function of the mass flow rate.
Figure 2: Cooling power and pump power as a function of the mass flow rate.

The most attractive operating condition will provide 10 kW with a minimum value of the pump
power; this operating condition is shown in Figure 2 and corresponds to a mass flow rate of
approximately 0.165 kg/s.
Problem 5.3-5 (5-5 in text): Water source heat pump
Your father-in-law has asked for your help with the design and installation of a water source heat
pump in his cabin in northern Wisconsin, as shown in Figure P5.3-5. In particular, he would like
you to analyze the system and possibly optimize the design. During the cooling season, the
water source heat pump is, essentially, an air conditioner that rejects heat to a water source rather
than to air. The cabin is located next to a lake and therefore you intend to reject heat by running
a cheap plastic tube through the lake. Currently, your father-in-law has selected a tube with an
outer diameter, Dout = 0.50 inch and a wall thickness th = 0.065 inch. You measure the
temperature of the water in the lake to be Tlake = 50°F and estimate that the heat transfer
coefficient between the external surface of the pipe and the water is ho = 450 W/m2-K. The
conductivity of the tube material is ktube= 1.5 W/m-K.

pump
ηpump = 0.6
V = 4 gal/min
LWT qrej = 30, 000 Btu/hr
qcool
w pump
Tair = 70°F
Tin EWT w hp
Tlake = 50°F
ho = 450 W/m -K
2

L = 100 ft
Dout = 0.5 inch
th = 0.065 inch
Figure P5.3-5: Water source heat pump rejecting heat to a lake.

The manufacturer's sheet for the particular heat pump that has been purchased lists many
characteristics of the heat pump as a function of the entering water temperature, EWT. The
manufacturer recommends a fixed flow rate of water through the pipe of V = 4.0 gal/min, and so
you have found an appropriate fixed displacement pump to provide this constant volumetric flow
rate of water; the pump has an efficiency, ηpump = 0.60. The data from the manufacturer’s sheet
have been used to correlate the heat pump power consumption as a function of the entering water
temperature according to:
⎡ kW ⎤ ⎡ kW ⎤
w [ kW ] = 0.8513 [ kW ] + 1.347x10-3 ⎢ EWT [°F] + 9.901x10-5 ⎢ 2 ⎥ ( EWT [°F])
2

⎣ °F ⎦ ⎣ °F ⎦

You have been asked to determine the length of tube, L, that should be run through the lake in
order to maximize the efficiency of the system (defined as the coefficient of performance, COP,
which is the ratio of the cooling provided to the power consumed by both the pump and the heat
pump). This is not a straightforward problem because it is difficult to see where to start. We'll
tackle it in small steps as discussed here. We'll start by making a couple of assumptions that will
eventually be relaxed; the assumptions are just to get the solution going - it is easier to
accomplish a meaningful analysis when you have a working model. Assume that the leaving
water temperature is LWT = 40°C and that the length of the tube is L = 100 ft.
a.) Calculate the pressure drop required to push the water through the tube in the lake.
The known information and assumptions are entered in EES:

$UnitSystem SI MASS RAD PA K J


$TABSTOPS 0.2 0.4 0.6 0.8 3.5 in

"Inputs"
T_lake=converttemp(F,K,50 [F]) "pond temperature"
eta_pump=0.6 "pump efficiency"
h_o=450 [W/m^2-K] "pond to tube heat transfer coefficient"
k_tube=1.5 [W/m-K] "conductivity of pipe"
D_o=0.5 [inch]*convert(inch,m) "outer diameter of pipe"
th=0.065 [inch]*convert(inch,m) "pipe wall thickness"
e=0.05 [mm]*convert(mm,m) "pipe wall roughness"
T_air=converttemp(F,K,70 [F]) "indoor air temperature"
V_dot=4 [gal/min]*convert(gal/min,m^3/s) "water mass flow rate"
q_dot_rej=30 [MBtu/hr]*convert(MBtu/hr,W) "heat rejection"

"Two assumptions to get started"


T_LWT=converttemp(C,K,40) "leaving water temperature-assumed"
L=100 [ft]*convert(ft,m) "length of pipe"

The water properties (ρ, k, μ, c, α, ν, and Pr) are evaluated using EES' internal property
functions at the average of the lake and the leaving water temperature:

TLWT + Tlake
T film = (1)
2

"Water Properties"
T_film=(T_lake+T_LWT)/2
"assume a film temperature to get properties"
rho=density(Water,T=T_film,P=1 [atm]*convert(atm,Pa)) "density"
k=conductivity(Water,T=T_film,P=1 [atm]*convert(atm,Pa)) "conductivity"
mu=viscosity(Water,T=T_film,P=1 [atm]*convert(atm,Pa)) "viscosity"
c=cP(Water,T=T_film,P=1 [atm]*convert(atm,Pa)) "cP"
alpha=k/(rho*c) "thermal diffusivity"
nu=mu/rho "kinematic viscosity"
Pr=nu/alpha "Prandtl number"

The mass flow rate ( m ) is given by:

m = V ρ (2)

The mean velocity of the water in the tube (um) is:

m
um = (3)
ρ Ac

where Ac is the cross-sectional area for flow within the tube:


π Di2
Ac = (4)
4

and Di is the inner diameter of the tube:

Di = Do − 2 th (5)

The Reynolds number (Re) is:

um ρ Di
Re = (6)
μ

m_dot=V_dot*rho "mass flow rate of water"


D_i=D_o-2*th "inner diameter"
A_c=pi*D_i^2/4 "cross-sectional area"
u_m=m_dot/(rho*A_c) "bulk velocity"
Re=u_m*rho*D_i/mu "Reynolds number"

The internal function PipeFlow_N is used to access the built-in correlations for flow within a
round tube. The pressure drop across the tube (Δp) is computed using the friction factor
according to:

L um2
Δp = f ρ (7)
Di 2

Assuming that the water is incompressible, the minimum power requirement by a perfect (i.e.,
reversible and adiabatic) pump is the product of the volumetric flow rate and the pressure drop.
The actual power required ( w pump ) is larger according to:

V Δp
w = (8)
η pump

call PipeFlow_N(Re,Pr,L/D_i,e/D_i: Nusselt_T,Nusselt_H,f) "correlation for Nusselt and f"


DeltaP=f*(L/D_i)*rho*u_m^2/2 "pressure drop"
w_dot_pump=DeltaP*V_dot/eta_pump "pump power"

The predicted pump power is 300 W.

b.) Predict the temperature of the water leaving the pump (Tin in Figure P5.3-5); assume all of
the pump energy goes into the water.

An energy balance on the pump leads to:

w
Tin = TLWT + (9)
m c
T_in=T_LWT+w_dot_pump/(m_dot*c) "temperature of water entering lake"

The temperature rise across the pump is predicted to be about 0.3°C.

c.) Predict the temperature of the water leaving the lake and entering the heat pump (EWT in
Figure P5.3-5) by considering the heat transfer coefficient associated with the flow of water
in the tube and the energy balance for this flow.

The heat transfer for the flow within the tube (hi) is calculated using the Nusselt number for a
constant wall temperature, returned from the function PipeFlow_N:

NuT k
hi = (10)
Di

The thermal resistance that separates the lake water from the heat pump water is composed to
convection between the flowing water and the internal surface of the tube:

1
Rconv ,i = , (11)
π Di L hi

conduction through the tube:

⎛D ⎞
ln ⎜ i ⎟
= ⎝ o⎠ ,
D
Rcond (12)
2 π L ktube

and convection from the external surface of the tube and the lake water:

1
Rconv ,o = , (13)
π Do L ho

The total conductance that characterizes the thermal communication between the lake water and
the water in the tube is:

1
= Rconv ,i + Rcond + Rconv ,o (14)
UA

The energy balance for a constant wall temperature situation, applied to this problem, leads to:

⎛ UA ⎞
TEWT = Tlake − (Tlake − Tin ) exp ⎜ − ⎟ (15)
⎝ m c ⎠
h_i=Nusselt_T*k/D_i "internal heat transfer coefficient"
R_conv_i=1/(h_i*pi*D_i*L) "resistance to internal convection"
R_conv_o=1/(h_o*pi*D_o*L) "resistance to external convection"
R_cond=ln(D_o/D_i)/(2*pi*k_tube*L) "pipe conductivity"
1/UA=R_conv_i+R_cond+R_conv_o "total conductance"
T_EWT=T_lake-(T_lake-T_in)*exp(-UA/(m_dot*c)) "constant temperature energy balance"
T_EWT_F=converttemp(K,F,T_EWT) "entering water temperature, F"

The entering water temperature predicted by the model is 89.4° F.

d.) Using your model, adjust the leaving water temperature (that you initially assumed to be
40°C) until the heat rejected to the water is equal to the heat rejection required by the heat
pump (i.e., qrej = 30x103 Btu/hr ).

The guess values are updated before this operation. Then, the initial assumption is commented
out and the heat rejection across the heat pump is calculated from an energy balance on the
water:

qrej = m c (TLWT − TEWT ) (16)

{T_LWT=converttemp(C,K,40) "leaving water temperature-assumed"}


q_dot_rej=m_dot*c*(T_LWT-T_EWT) "heat rejection"

The calculated entering water temperature is 90.6° F.

e.) Using the manufacturer's data provided by the curve fit, calculate the power required by the
heat pump and, from that, the cooling provided to the cabin and the total COP (including
both the heat pump and the pump).

The correlation in Eq. Error! Reference source not found. is used to compute the power
consumption (note that the power consumption is provided in kW and the entering water
temperature must be provided in ° F).

w_dot*convert(W,kW)=0.85125564 [kW] + 0.0013466265[kW/F]*converttemp(K,F,T_EWT) +


0.000099013661[kW/F^2]*converttemp(K,F,T_EWT)^2
"power consumption based on catalog data"

An energy balance on the heat pump leads to:

qcool = qrej − w hp (17)

The total COP is therefore:

qcool
COP = (18)
w hp + w pump

The total cooling provided to the cabin for 100 ft of tube is 7010 W at a COP of 3.36.
f.) Use your model to prepare a single plot that shows how the COP and cooling capacity vary
with length of tube. You should see an optimal length of tube that maximizes the COP;
explain why this optimal value exists.

A parametric table is generated that includes the variables L, w_dot, w_dot_pump, q_dot_cool,
and COP. The length of tube is varied from 20 m to 100 m and the cooling capacity and COP
are shown in Figure 2 as a function of the length of tube.

Figure 2: Total COP and cooling capacity as a function of the length of tube.

Note that COP is optimized when a length of tube of approximately 50 m is used. Figure 3
illustrates the power consumed by the pump, the heat pump, and the total system as a function of
the tube length.
Figure 3: Pump power, heat pump power, and total power consumed as a function of the length of tube.

Notice that for small values of L, the pump power is small but the heat pump power is large
because the tube does not do a good job of transferring heat and so the entering water
temperature increases (see Figure 3). At large values of L the situation reverses, the tube is
restrictive and so the pump power increases but the tube has a lot of surface area and so the
entering water temperature is closer to the lake temperature.
Problem 5.3-7 (5-6 in text): Cold plate
Figure P5.3-7 illustrates a cold plate that is used as the heat sink for an array of diodes in a power
supply.
q ′′ = 6000 W/m
2

2 Dh Dh = 0.2 cm

Wp = 8 cm
Hp = 2 cm
Figure P5.3-7: Cold plate.

The operation of the diodes provides a uniform heat flux q ′′ = 6000 W/m2 over the top surface of
the cold plate. The plate is cooled by the flow of a coolant with density ρc = 1090 kg/m3,
conductivity kc = 0.8 W/m-K, viscosity μc = 0.01 Pa-s, and specific heat capacity cc = 1500 J/kg-
K. The mass flow rate of coolant is m = 0.1 kg/s and the inlet temperature is Tc,in = 30°C. The
coolant flows along the length of the cold plate through holes that are Dh = 0.2 cm in diameter.
The length of the cold plate (in the flow direction) is Lp = 15 cm, the width is Wp = 8 cm, and the
thickness is Hp = 2 cm. The conductivity of the cold plate is kp = 650 W/m-K. The distance
between the centers of two adjacent holes is twice the hole diameter. All of the surfaces of the
cold plate that are not exposed to the heat flux are adiabatic. Your initial model should assume
that the resistance to conduction through the cold plate from the surface where the heat flux is
applied to the surface of the holes is negligible. Further, your model should assume that the
resistance to conduction along the length of the cold plate is infinite.
a.) Plot the coolant temperature and plate temperature as a function of position, x.

The inputs are entered in EES:

$UnitSystem SI MASS RAD PA K J


$TABSTOPS 0.2 0.4 0.6 0.8 3.5 in

"Inputs"
H_p=2 [cm]*convert(cm,m) "height of plate"
W_p=8 [cm]*convert(cm,m) "width of plate"
L_p=15 [cm]*convert(cm,m) "length of plate in flow direction"
D_h=0.2 [cm]*convert(cm,m) "diameter of holes"
N_h=W_p/(2*D_h) "number of holes"
m_dot=0.1 [kg/s] "mass flow rate of coolant"
T_c_in=converttemp(C,K,30 [C]) "inlet temperature"
q``=6000 [W/m^2] "heat flux"
k_p=650 [W/m-K] "plate conductivity"
rho_c=1090 [kg/m^3] "coolant density"
k_c=0.8 [W/m-K] "coolant conductivity"
mu_c=0.01 [Pa-s] "coolant viscosity"
c_c=1550 [J/kg-K] "coolant specific heat capacity"
Pr_c=mu_c*c_c/k_c "coolant Prandtl number"

With the assumptions listed in the problem statement, the energy balance becomes:
dTm
qs′′ per = m c (1)
dx

where per is the perimeter of the flow passages

per = π Dh N h (2)

and qs′′ is the heat flux at the surface of the passages.

W
qs′′ = q ′′ (3)
per

per=pi*D_h*N_h "perimeter of flow passages"


q``_s=W_p*q``/per "heat flux at surface of flow passages"

Integrating Eq. (1) once leads to:

per qs′′
Tm = Tc ,in + x (4)
m c

The fluid temperature will be evaluated at a uniformly distributed set of nodes:

xi =
( i − 1) L for i = 1..N (5)
( N − 1) p
where N is the number of nodes. The distance between adjacent nodes is:

Lp
Δx = (6)
( N − 1)
N=101 [-] "number of nodes"
Dx=L_p/(N-1) "distance between nodes"
duplicate i=1,N
x[i]=(i-1)*Dx "position of each node"
end

Equation (4) is used to evaluate the mean coolant temperature at each node:

duplicate i=1,N
T_c[i]=T_c_in+q``_s*per*x[i]/(m_dot*c_c) "coolant temperature"
end

The surface temperature is related to the mean temperature according to:


qs′′
Ts = Tm + (7)
h

where h is the local heat transfer coefficient. The local heat transfer coefficient is evaluated
using the correlations programmed in EES. The mean velocity within the passage is:

m
um = (8)
Ac ρc

where Ac is the cross-sectional area of the passages:

Dh2
Ac = N h π (9)
4

The Reynolds number is:

um Dh ρ c
Re = (10)
μc

A_c=N_h*pi*D_h^2/4 "cross-sectional area of flow passages"


u_m=m_dot/(A_c*rho_c) "bulk velocity"
Re=u_m*D_h*rho_c/mu_c "Reynolds number"

The local Nusselt number (Nu) is evaluated at each node using the PipeFlow_N_local function.
The heat transfer coefficient is computed according to:

Nu kc
h= (11)
Dh

The surface temperature is evaluated at each node using Eq. (7).

duplicate i=1,N
call PipeFlow_N_local(Re,Pr_c,x[i]/D_h,0[-]: Nusselt_T_x[i],Nusselt_H_x[i],f_x[i])
"correlation for local heat transfer coefficient"
h[i]=Nusselt_H_x[i]*k_c/D_h "local heat transfer coefficient"
T_p[i]=T_c[i]+q``_s/h[i] "plate temperature"
end

Figure 2 illustrates the mean coolant and plate temperature as a function of position.
305 surface temperature

304.5

Temperature (K)
304
mean coolant temperature

303.5

303
0 0.02 0.04 0.06 0.08 0.1 0.12 0.14 0.16
Position (m)
Figure 2: Mean coolant and plate temperature as a function of position.

b.) Plot the maximum coolant temperature and maximum plate temperature as a function of
mass flow rate for mass flow rates varying between 0.0005 kg/s and 10 kg/s. Use a log-scale
for the mass flow rate. Overlay on your plot the temperature difference between the
maximum plate temperature and maximum coolant temperature. You should see three
distinct types of behavior as you increase the mass flow rate. Explain this behavior.

The maximum coolant and plate temperatures occur at x = Lp.

T_max_p=T_p[N] "maximum plate temperature"


T_max_c=T_c[N] "maximum coolant temperature"
DT_max=T_max_p-T_c[N] "maximum plate to coolant temperature difference"

The maximum coolant temperature and maximum plate temperature as a function of mass flow
rate as well as the temperature difference at the plate exit are shown in Figure 3.
400 2.25

Plate-to-coolant temperature difference (K)


ΔT
2
380 1.75

Temperature (K) 1.5


360
1.25

Tp,x=L 1
340
0.75

320 0.5

Tc,x=L 0.25

300 0
0.0005 0.01 0.1 1 4
Mass flow rate (kg/s)
Figure 3: Maximum plate and coolant temperature and the difference between these values as a function of
mass flow rate.

At low values of m , you have fully developed laminar flow and therefore the Nusselt number at
the plate exit is constant and independent of mass flow rate. This leads to a constant ΔT for m
less than about 0.01 kg/s. For mass flow rates between approximately 0.01 kg/s and 0.7 kg/s,
you have developing laminar flow and therefore the heat transfer coefficient tends to increase,
reducing the value of ΔT. Finally for mass flow rates above about 0.7 kg/s, the flow becomes
turbulent and the value of ΔT drops dramatically.

c.) Assess the validity of neglecting the resistance to conduction through the cold plate from the
surface where the heat flux is applied to the surface of the holes for the nominal mass flow
rate ( m = 0.1 kg/s).

The resistance to conduction laterally between the cold plate surface and the holes is,
approximately:

Hp
Rcond ,lat = (12)
2 k p Lp Wp

R_cond_lat=(H_p/2)/(W_p*L_p*k_p) "approximate resistance laterally to conduction"

which leads to Rcond,lat = 0.00128 K/W. The resistance to convection at the flow passage surface
is:

1
Rconv = (13)
per h

where h is the average heat transfer coefficient, evaluated using the PipeFlow_N function in EES.
call PipeFlow_N(Re,Pr_c,L_p/D_h,0 [-]: Nusselt_bar_T,Nusselt_bar_H,f_bar) "average Nusselt number"
h_bar=Nusselt_bar_H*k_c/D_h "average heat transfer coefficient"
R_conv=1/(h_bar*per*L_p) "approximate resistance to convection"

The validity of neglecting the resistance to conduction laterally across the cold plate is examined
using a Biot number:

Rcond ,lat
Bi = (14)
Rconv

Bi=R_cond_lat/R_conv "Biot number"

which leads to Bi = 0.085; justifying this assumption.

d.) Assess the validity of assuming that the resistance to conduction along the length of the cold
plate is infinite for the nominal mass flow rate ( m = 0.1 kg/s).

The resistance to conduction along the plate is estimated according to:

Lp
Rcond ,ax = (15)
k p Ac , p

where Ac,p is the cross-sectional area of the plate:

Ac , p = H p W p − Ac (16)

A_c_p=W_p*H_p-N_h*pi*D_h^2/4 "cross-sectional area of plate"


R_cond_ax=L_p/((W_p*H_p-A_c)*k_p) "resistance axially to conduction"

The heat transferred along the plate by conduction is, approximately:

qcond ,ax ≈
(T p,x=L − Tp , x = 0 )
(17)
Rcond , ax

The appropriate dimensionless number that characterizes the importance of axial conduction is:

qcond ,ax
(18)
qc

where qc is the heat transferred to the coolant:

qc = q ′′Wp Lp (19)

q_dot_cond_ax=(T_p[N]-T_p[1])/R_cond_ax "axial conduction"


q_dot_c=q``*W_p*L_p "heat transfer"
q_dot_cond_ax\q_dot_c=q_dot_cond_ax/q_dot_c
"dimensionless number that characterizes the importance of axial conduction"

qcond ,ax
which leads to = 0.18. Therefore, axial conduction is likely somewhat important for this
qc
application.

Refine your model so that it includes the effect of conduction along the length of the cold plate.
This refined model should continue to assume that the resistance to conduction through the cold
plate from the surface where the heat flux is applied to the surface of the holes is negligible.
e.) Using differential energy balances on the plate material and the coolant, derive the state
equations that govern this problem. For this problem, the state variables include the coolant
dT
temperature (Tc), the plate temperature (Tp), and the gradient of the plate temperature ( p ).
dx

An energy balance on the fluid leads to:

h per (Tp − Tc ) = m c
dTc
(20)
dx

Notice that if axial conduction is included then it is not necessary that the heat flux at the plate
surface be equal to the heat flux applied to the fluid. An energy balance on a differential section
of the plate leads to:

d 2Tp
q ′′W p = − k p Ac , p + h per (Tp − Tc ) (21)
dx 2

According to Eqs. (20) and (21), the state equations for this problem are:

dTc h per
dx
=
m c
(Tp − Tc ) (22)

d ⎛ dTp ⎞ q ′′W p

dx ⎝ dx
⎟=− +
h per
(Tp − Tc ) (23)
⎠ k p Ac , p k p Ac , p

d ( Tp ) dTp
= (24)
dx dx

f.) Assume that the temperature of the plate material at x = 0 is Tp,x=0 = 310 K. The coolant
temperature and plate temperature gradient at x = 0 are both specified. Use the Crank-
Nicolson technique to integrate the state equations from x = 0 to x = L. Do not attempt to
enforce the fact that the plate is adiabatic at x = L during this step. Your Crank-Nicolson
technique should be implicit in the temperatures but explicit in the heat transfer coefficient
(i.e., the heat transfer coefficient can be calculated at the beginning of the length step). Plot
the temperature of the coolant and the plate as a function of position.

The state variables at x = 0 are set:

T_p_0=310 [K] "guess for the plate temperature at x=0"


T_p[1]=T_p_0
dTpdx[1]=0 [K/m] "plate is adiabatic at x=0"
T_c[1]=T_c_in "coolant temperature at x=0"

The heat transfer coefficient is evaluated at each nodal location:

A_c=N_h*pi*D_h^2/4 "cross-sectional area of flow passages"


u_m=m_dot/(A_c*rho_c) "bulk velocity"
Re=u_m*D_h*rho_c/mu_c "Reynolds number"
duplicate i=1,N
call PipeFlow_N_local(Re,Pr_c,x[i]/D_h,0[-]: Nusselt_T_x[i],Nusselt_H_x[i],f_x[i])
"correlation for local heat transfer coefficient"
h[i]=Nusselt_H_x[i]*k_c/D_h "local heat transfer coefficient"
end

The Crank-Nicolson technique is used to integrate the state equations along the plate:

⎛ dTp ⎞ ⎛ dTp ⎞ ⎛ 2 q ′′W p hi per ⎞ Δx


⎜ ⎟ =⎜ ⎟ + ⎜⎜ − + ( Tp ,i − Tc ,i ) + i
h per
( Tp ,i +1 − Tc ,i +1 ) ⎟
⎟ 2
(25)
⎝ dx ⎠i +1 ⎝ dx ⎠i ⎝ k p Ac , p k p Ac , p k p Ac , p ⎠

⎡ h per ⎤ Δx
Tc ,i +1 = Tc ,i + ⎢ i ( Tp ,i − Tc ,i ) + i +1
h per
( Tp ,i +1 − Tc ,i +1 ) ⎥ (26)
⎣ m c m c ⎦ 2

⎡⎛ dT ⎞ ⎛ dT ⎞ ⎤ Δx
Tp ,i +1 = Tp ,i + ⎢⎜ p ⎟ + ⎜ p ⎟ ⎥ (27)
⎣⎢⎝ dx ⎠i ⎝ dx ⎠i +1 ⎥⎦ 2

A_c_p=W_p*H_p-N_h*pi*D_h^2/4 "cross-sectional area of plate"


duplicate i=1,(N-1)
dTpdx[i+1]=dTpdx[i]+(-2*q``*W_p/(k_p*A_c_p)+h[i]*per*(T_p[i]-&
T_c[i])/(k_p*A_c_p)+h[i+1]*per*(T_p[i+1]-T_c[i+1])/(k_p*A_c_p))*Dx/2
T_p[i+1]=T_p[i]+(dTpdx[i]+dTpdx[i+1])*Dx/2
T_c[i+1]=T_c[i]+(h[i]*per*(T_p[i]-T_c[i])/(m_dot*c_c))*Dx/2+(h[i+1]*per*(T_p[i+1]-&
T_c[i+1])/(m_dot*c_c))*Dx/2
end

Figure 4 illustrates the plate and coolant temperature predicted by the numerical model for the
assumed value of Tp,x=0. Note that the plate temperature distribution does not satisfy the
boundary condition that the plate is adiabatic at x = L and therefore the assumed value of Tp,x=0 is
not correct.
400

390
plate temperature
380

370

Temperature (K)
360

350

340

330

320
coolant temperature
310

300
0 0.02 0.04 0.06 0.08 0.1 0.12 0.14 0.16
Position (m)
Figure 4: Plate and coolant temperature predicted by the model with Tp,x=0 = 310 K.

g.) Adjust the assumed value of the plate temperature at x = 0, Tp,x=0, until the temperature
gradient in the plate material at x = L is zero (i.e., the end of the plate is adiabatic). Overlay
on your plot from (a) the temperature of the coolant and the conductor as a function of
position.

The guess values are updated. The assumed value of Tp,x=0 is commented out and instead the
adiabatic boundary condition at x = L is enforced:

{T_p_0=310 [K]} "guess for the plate temperature at x=0"


dTpdx[N]=0

Figure 5 illustrates the surface and coolant temperature as a function of position overlaid onto the
same plot generated in (a).

surface temperature
305

304.5
Temperature (K)

neglecting axial conduction, from (a)


304 including axial conduction, from (g)

303.5

coolant temperature
303
0 0.02 0.04 0.06 0.08 0.1 0.12 0.14 0.16
Position (m)
Figure 5: Surface and coolant temperature as a function of position, from (a) and (g).
Problem 5.4-1 (5-7 in text): Thrust bearing
Figure P5.4-1 illustrates a simple slider bearing used to provide support against thrust loads.

ρ = 800 kg/m3
ΔH = 0.05 mm μ = 0.5 Pa-s exit
pamb = 1 atm

y
x
inlet Hmin = 0.2 mm
pamb = 1 atm up = 10 m/s

L = 5 cm
Figure P5.4-1: Thrust bearing.

The slider is a close clearance, converging gap formed between a moving surface (e.g., the
surface of a rotating shaft) and a stationary surface. The velocity of the moving surface is up =
10 m/s. The length of the gap is L = 5.0 cm and the minimum clearance in the gap is at the exit
(i.e., at x = L) is Hmin = 0.2 mm. The maximum clearance of the gap is at the inlet (i.e., at x = 0)
is Hmin + ΔH where ΔH/Hmin = 0.25. The clearance varies linearly with position according to:
H = H min + ΔH
( L − x ) . The pressure at the inlet and exit of the gap is ambient, p = 1 atm.
amb
L
The properties of the oil that flows through the gap are ρ = 800 kg/m3 and μ = 0.5 Pa-s.
a.) Is it appropriate to model the flow through the gap as inertia-free flow using the Reynolds
equation?

The inputs are entered in EES:

$UnitSystem SI MASS RAD PA K J


$Tabstops 0.2 0.4 0.6 3.5 in

"Inputs"
rho=800 [kg/m^3] "density of oil"
mu=0.5 [Pa-s] "viscosity of oil"
u_p=10 [m/s] "velocity of moving surface"
H_min_mm=0.2 [mm] "minimum clearance, in mm"
H_min=H_min_mm*convert(mm,m) "minimum clearance"
DHoverH_min=0.25 [-] "ratio of clearance change to minimum clearance"
DH=DHoverH_min*H_min "clearance change"
L=5.0 [cm]*convert(cm,m) "length of passage"
p_amb=1 [atm]*convert(atm,Pa) "ambient pressure"

The modified Reynolds number is computed using the length of the gap as the characteristic
length:

MR=(rho*u_p*H_min/mu)*(H_min/L) "modified Reynolds number"

which leads to a modified Reynolds number of 0.013; therefore, the Reynolds equation is
appropriate for this problem.
b.) Use the Reynolds equation to obtain an analytical solution for the pressure distribution within
the gap.

The expression for the clearance is entered in Maple:

> restart;
> H:=H_min+DH*(L-x)/L;
DH ( L − x )
H := H_min +
L

The Reynolds equation is modified for this steady-state problem,

d ⎛ dp 3 ⎞ dH
⎜ H ⎟ = 6 μ up (1)
dx ⎝ dx ⎠ dx

and entered in Maple,

> ODE:=diff(diff(p(x),x)*H^3,x)=6*mu*u_p*diff(H,x);
2
DH ( L − x ) ⎞
3 ⎛⎜⎜ p( x ) ⎞⎟⎟ ⎛⎜⎜ H_min +
d
3 ⎟⎟ DH
⎛d
2
⎞⎛ DH ( L − x ) ⎞ ⎝ dx ⎠ ⎝ L ⎠
ODE := ⎜⎜ 2 p( x ) ⎟⎟ ⎜⎜ H_min + ⎟⎟ −
⎝ dx ⎠⎝ L ⎠ L
6 μ u_p DH
=−
L

The solution is obtained:

> ps:=dsolve(ODE);
−6 μ u_p L 3 H_min − 6 μ u_p L 3 DH + _C1
ps := p( x ) =
2 DH ( −H_min L − DH L + DH x ) 2
6 μ u_p L 2
− + _C2
DH ( −H_min L − DH L + DH x )

The solution includes two constants of integration that must be obtained by applying the
boundary conditions at x = 0 and x = L; the pressure is ambient at each of these locations:

px =0 = pamb (2)

px = L = pamb (3)

These boundary conditions are evaluated symbolically in Maple,

> BC1:=(rhs(eval(ps,x=0))=p_amb);
−6 μ u_p L 3 H_min − 6 μ u_p L 3 DH + _C1 6 μ u_p L 2
BC1 := − + _C2 =
2 DH ( −H_min L − DH L ) 2 DH ( −H_min L − DH L )
p_amb
> BC2:=(rhs(eval(ps,x=L))=p_amb);
−6 μ u_p L 3 H_min − 6 μ u_p L 3 DH + _C1 6 μ u_p L
BC2 := + + _C2 = p_amb
2 DH H_min 2 L 2 DH H_min

and solved in order to obtain explicit expressions for C1 and C2:

> solve({BC1,BC2},{_C1,_C2});
−6 μ u_p L + 2 p_amb DH H_min + p_amb DH 2
{ _C2 = ,
DH ( 2 H_min + DH )
6 L 3 μ u_p DH ( H_min + DH )
_C1 = }
2 H_min + DH

The equations for C1 and C2 are copied and pasted into EES with the usual modification:

"constants of integration, from Maple"


C_2 = -1/DH*(6*mu*u_p*L-2*p_amb*DH*H_min-p_amb*DH^2)/(2*H_min+DH)
C_1 = 6*L^3*mu*u_p*DH*(H_min+DH)/(2*H_min+DH)

The solution for the pressure (ps in Maple) is also copied and pasted into EES:

"pressure solution, from Maple"


p = 1/2*(-6*mu*u_p*L^3*H_min-6*mu*u_p*L^3*DH+C_1)/DH/(-H_min*L-DH*L+DH*x)^2&
-6*mu*u_p*L^2/DH/(-H_min*L-DH*L+DH*x)+C_2
x_bar=x/L "dimensionless position"
Dp=p-p_amb "pressure elevation, relative to ambient"
Dp_kPa=Dp*convert(Pa,kPa) "in kPa"

The pressure (relative to ambient) is shown as a function of position in the gap in Figure 2.
Figure 2: Pressure relative to ambient as a function of position.

The pressure is elevated within the gap, providing support to the adjacent surface. This type of
thrust bearing is referred to as a hydrodynamic bearing and operates by dragging fluid through
the gap. In the inlet region of the gap (near x = 0), the clearance is large and therefore the
Couette flow (i.e., the flow dragged along by the moving surface) is large while towards the exit
of the gap (near x = L), the clearance is small and therefore the Couette flow is less. In order to
satisfy continuity (which requires that the flow everywhere in the gap be the same), an adverse
pressure gradient is setup at the inlet, creating a pressure driven flow that opposes the Couette
flow. A favorable pressure gradient is setup at the outlet (i.e., the pressure decreases in the flow
direction), creating a pressure driven flow that augments the Couette flow.

c.) Determine the force per unit width provided by the thrust bearing.

The force is obtained by integrating the pressure elevation over the surface of the thrust bearing:
L
F
= ( p − pamb ) dx
W ∫0
(4)

The integration is carried out numerically using the Integral command in EES:

F`=INTEGRAL(Dp,x,0,L) "force per unit width"

which leads to F/W = 27600 N/m.

d.) Determine an appropriate scaling relation for the force per unit width and use it to define a
non-dimensional thrust force. Plot the dimensionless thrust force as a function of the
parameter ΔH/Hmin.
Equation (5-114) in the text provides some guidance relative to the scaling relationship; for an
inertia-free flow, the scaling relationship becomes:

Δp μ um
≈ (5)
Lchar H2
N
N viscous shear
pressure gradient

which implies that the pressure elevation is, approximately:

μ um
Δp ≈ L (6)
H2

and so the force developed by the bearing will be approximately:

μ um
F≈ 2
L2 W (7)
H

Equation (7) provides a convenient definition for a dimensionless force:

F H2
F = (8)
W μ um L2

which is calculated in EES:

F_bar=F`/(mu*u_p*L^2/H_min^2) "dimensionless force"

Figure 3 illustrates the dimensionless force as a function of the bearing taper, ΔH/Hmin.
Figure 3: Dimensionless thrust force as a function of ΔH/Hmin.
Problem 5.4-2 (5-8 in text)
A very viscous fluid is pumped through a circular tube at a rate of V = 15 liter/min. The tube is
thin wall and made of metal; the thickness of the tube and its resistance to conduction can be
neglected. The tube diameter is D = 0.5 inch. The tube is covered with insulation that is thins =
0.25 inch with conductivity kins= 0.5 W/m-K. The external surface of the tube is exposed to air
at T∞ = 20°C with heat transfer coefficient h = 120 W/m2-K. The viscosity of the fluid is μ = 0.6
Pa-s and its conductivity is k = 0.15 W/m-K.
a.) Prepare an analytical solution for the radial temperature distribution within the fluid at a
location where the fluid temperature is not changing in the x-direction (i.e., in the direction of
the flow). Include the effect of viscous dissipation. You may neglect axial conduction.
Assume that the fluid is hydrodynamically fully developed.

The inputs are entered in EES:

$UnitSystem SI MASS RAD PA K J


$TABSTOPS 0.2 0.4 0.6 0.8 3.5 in

mu=0.6 [Pa-s] "viscosity"


k=0.15 [W/m-K] "conductivity"
V_dot=15 [liter/min]*convert(liter/min,m^3/s) "volumetric flow rate"
D=0.5 [inch]*convert(inch,m) "tube diameter"
th_ins=0.25 [inch]*convert(inch,m) "insulation thickness"
k_ins=0.5 [W/m-K] "insulation conductivity"
h_bar=120 [W/m^2-K] "heat transfer coefficient"
T_infinity=converttemp(C,K,20 [C]) "ambient temperature"
L=1 [m] "length"

The thermal energy conservation equation in this limit is:

2
k d ⎛ dT ⎞ ⎛ du ⎞
0= ⎜r ⎟+μ⎜ ⎟ (1)
r dr ⎝ dr ⎠ ⎝ dr ⎠

where the velocity distribution for fully developed laminar flow in a round tube is:

⎡ ⎛ 2 r ⎞2 ⎤
u = 2 um ⎢1 − ⎜ ⎟ ⎥ (2)
⎢⎣ ⎝ D ⎠ ⎥⎦

where um is the mean velocity:

V
um = (3)
Ac

where

Ac = π rout
2
(4)
and

D
rout = (5)
2

r_out=D/2 "outer radius of tube"


A_c=pi*r_out^2 "cross-sectional area of tube"
u_m=V_dot/A_c "bulk velocity"

The velocity gradient is:

du 16 um r
=− (6)
dr D2

Substituting Eq. (6) into Eq. (1) leads to:

k d ⎛ dT ⎞ μ um2 r 2
0= ⎜ r ⎟ + 256 (7)
r dr ⎝ dr ⎠ D4

Equation (7) is separated:

⎛ dT ⎞ μ um2 r 3
d ⎜r ⎟ = −256 dr (8)
⎝ dr ⎠ k D4

and integrated (note the limits of integration correspond to a 0 temperature gradient at r = 0):
dT
r
dr
⎛ dT ⎞ μ um2 3 r


0
d ⎜r ⎟ = −256
⎝ dr ⎠ k D 4 ∫0
r dr (9)

which leads to:

dT μ u2 r 4
r = −64 m 4 (10)
dr kD

Equation (10) is separated:

μ um2 r 3
dT = −64 dr (11)
k D4

and integrated again:


μ um2
∫ dT = −64 k D ∫ r dr
3
4
(12)

which leads to:

μ um2
T = −16 r 4 + C1 (13)
k D4

where C1 is a constant of integration. The constant of integration is evaluated by enforcing an


energy balance at the outer radius:

− k 2 π rout L
dT
=
(Tr = rout − T∞ ) (14)
dr r = rout Rins + Rconv

where L is the length (taken to be 1 m for a per unit length solution) and Rins is the resistance to
conduction through the insulation:

⎛ D + 2 thins ⎞
ln ⎜ ⎟
D
Rins = ⎝ ⎠ (15)
2 π kins L

and Rconv is the resistance to convection from the outer surface of the insulation:

1
Rconv = (16)
π ( D + 2 thins ) L h

R_ins=ln((D+2*th_ins)/D)/(2*pi*k_ins*L) "resistance of insulation"


R_conv=1/(h_bar*pi*(D+th_ins)*L) "convection resistance"

Equation (13) is used to evaluate the temperature and temperature gradient at r = rout:

μ um2
Tr = rout = −16 rout
4
+ C1 (17)
kD 4

dT μ um2
= −64 rout
3
(18)
dr r = rout k D4

dTdr_out=-64*mu*u_m^2*r_out^3/(k*D^4) "temperature gradient at r=r_out"


T_out=-16*mu*u_m^2*r_out^4/(k*D^4)+C1 "temperature at r=r_out"
-k*2*pi*r_out*L*dTdr_out=(T_out-T_infinity)/(R_ins+R_conv) "boundary condition"

The temperature distribution, Eq. (13), is programmed:


r_bar=0 [-] "dimensionless radius"
r=r_bar*r_out "radius"
T=-16*mu*u_m^2*r^4/(k*D^4)+C1 "temperature"
T_C=converttemp(K,C,T) "in C"

b.) Plot the temperature as a function of radial position. Overlay on your plot the temperature
distribution for V = 5, 10, and 20 liter/min.

The requested plot is shown in Figure 5.4-2.

90
V = 20 lpm
80

70
Temperature (°C)

60
V = 15 lpm
50

40
V = 10 lpm
30
V = 5 lpm
20
0 0.1 0.2 0.3 0.4 0.5 0.6 0.7 0.8 0.9 1
Dimensionless radius, r/rout
Figure 5.4-2: Temperature as a function of normalized radius for various values of volumetric flow rate.
Problem 5.5-1 (5-9 in text): Immersion Lithography
Immersion lithography is a potential technique that will allow optical lithography (the
manufacturing technique used to fabricate computer chips) to create smaller features. A liquid is
inserted into the space between the last optical element (the lens) and the wafer that is being
written in order to increase the index of refraction in this volume (relative to the air that would
otherwise fill this gap). A simplified version of this concept is shown in Figure P5.5-1.

L = 10 cm
ρ = 800 kg/m3
Tlens = 20°C lens c = 500 J/kg-K
Δp = 300 kPa y  ′′ = 2 k = 0.1 W/m-K
= °
Tin 20 C
qexp 500 W/m μ = 0.1 Pa-s
x
wafer
uw = 0.45 m/s
wexp = 2 cm H = 0.5 mm
Figure P5.5-1: An immersion lithography tool.

It is important to predict the temperature distribution in the fluid during this process; even very
small temperature changes will result in imaging problems associated with changes in the
properties of the fluid or thermally induced distortions of the wafer. The fluid is inserted into the
gap at the left hand side (x = 0) and flows from left to right through the lens/wafer gap; the total
length of the gap is L = 10 cm and the height of the gap is H = 0.5 mm. The fluid is driven by
the viscous shear as the wafer is moved under the lens with velocity uw = 0.45 m/s. The fluid is
also driven by a pressure gradient; the pressure at the left hand side of the gap is elevated relative
to the pressure on the right hand side by an amount, Δp = 300 kPa. The fluid enters at the left
hand side of the lens with a uniform temperature, Tin = 20°C. Assume that the flow is laminar
and that the problem is two-dimensional (i.e., the slot extends a long way into the page). In this
case, the liquid has a fully developed velocity distribution when it enters the gap:

⎞ H Δp ⎡⎛ y ⎞ ⎛ y⎞ ⎤
2

2
y
u = uw ⎜ 1 − ⎟ + ⎢⎜ ⎟−⎜ ⎟ ⎥
⎝ H ⎠ 2 μ L ⎢⎣⎝ H ⎠ ⎝ H ⎠ ⎥⎦

where y is the distance from the wafer and μ is the liquid viscosity.

The next generation of immersion lithography tools will use advanced liquids with very high
viscosity and so you have been asked to generate a model that can evaluate the impact of viscous
dissipation on the temperature distribution. The liquid has density ρ = 800 kg/m3, specific heat
capacity c = 500 J/kg-K, thermal conductivity k = 0.1 W/m-K, and viscosity μ = 0.1 Pa-s. The
energy required to develop the resist layer and therefore carry out the lithography process passes
through the lens and the water and is deposited into the wafer; the energy can be modeled as a
heat flux at the wafer surface into the liquid (assume all of the heat flux will go to the liquid
rather than the wafer). The heat flux is concentrated in a small strip (wexp = 2.0 cm long) at the
center of the lens, as shown in Figure P5.5-1 and given by:
⎧0 for x < ( L − wexp ) / 2


′′ for ( L − wexp ) / 2 < x < ( L + wexp ) / 2
qs′′ ( x ) = ⎨qexp

⎪⎩0 for x > ( L + wexp ) / 2

′′ = 500 W/m2. The lens is maintained at a constant temperature, Tlens = 20°C.


where qexp
a.) What is the mean velocity and the Reynolds number that characterizes the flow through the
lens/wafer gap?

The inputs are entered in a MATLAB script, P5p3_1.m:

clear all;

u_w=0.45; %wafer velocity (m/s)


H=0.0005; %gap height (m)
L=0.10; %gap length (m)
DeltaP=300000; %pressure drop (Pa)
T_in=20; %inlet temperature (C)
T_lens=20; %lens temperature (C)
rho=800; %density (kg/m^3)
c=500; %specific heat capacity (J/kg-K)
k=0.1; %conductivity (W/m-K)
mu=0.1; %viscosity (Pa-s)
qf_exp=500; %heat flux in exposure region (W/m^2)
w_exp=0.02; %exposure region width (m)

The mean velocity in the lens/wafer gap is obtained according to:


H
1
u ( y ) dy
H ∫0
um = (1)

Substituting Eq. Error! Reference source not found. into Eq. (1) leads to:

1
H ⎧⎪ ⎛ y ⎞ H 2 Δp ⎡⎛ y ⎞ ⎛ y ⎞ ⎤ ⎫⎪
2

um =
H ∫0 ⎨⎪uw ⎜⎝1 − H ⎟⎠ + 2 μ L ⎢⎢⎜⎝ H ⎟⎠ − ⎜⎝ H ⎟⎠ ⎥⎥ ⎬⎪ dy (2)
⎩ ⎣ ⎦⎭

Carrying out the integration leads to:

H
1 ⎧⎪ ⎛ y2 ⎞ H 2 Δp ⎡⎛ y 2 ⎞ y 3 ⎤ ⎫⎪
um = ⎨uw ⎜ y − ⎟+ ⎢⎜ ⎟ − 2 ⎥⎬
(3)
H ⎩⎪ ⎝ 2H ⎠ 2 μ L ⎣⎝ 2 H ⎠ 3 H ⎦ ⎭⎪0

Applying the limits leads to:


1 ⎪⎧ ⎛ H 2 ⎞ H 2 Δp ⎡⎛ H 2 ⎞ H 3 ⎤ ⎪⎫
um = ⎨uw ⎜ H − ⎟+ ⎢⎜ ⎟− ⎥⎬ (4)
H ⎩⎪ ⎝ 2 H ⎠ 2 μ L ⎣⎝ 2 H ⎠ 3 H 2 ⎦ ⎭⎪

which can be simplified to:

uw Δp H 2
um = + (5)
2 12 μ L

u_m=u_w/2+DeltaP*H^2/(3*mu*L) %mean velocity (m/s)

which leads to um = 0.85 m/s. The hydraulic diameter associated with flow through the gap
between two parallel plates is:

4HW
Dh = = 2H (6)
2W

where W is the length of the duct into the page (note W >> H). The Reynolds number for the
flow in the gap is therefore:

Re=2*H*u_m*rho/mu %Reynolds number (-)

which leads to Re = 6.8.

b.) You’d like to calculate a Brinkman number in order to evaluate the relative impact of viscous
dissipation for the process but you don’t have a convenient reference temperature difference
to use. Use the heat flux to come up with a meaningful reference temperature difference and
from that temperature difference determine a Brinkman number. Comment on the
importance of viscous dissipation for this problem.

A meaningful reference temperature difference might be based on the temperature difference


between the surface of the wafer and the mean temperature in the duct if the heat flux were
applied in a thermally fully developed flow situation; this would be approximately:

qs′′ H
ΔTref = (7)
2k

An alternative might be to estimate the temperature difference between the inlet fluid and the
wafer surface at the trailing edge of the thermal boundary layer that develops over the region of
heat flux:

qs′′ δ t
ΔTref = (8)
k

where δt is an estimate of the thermal boundary layer thickness:


α wexp
δt = 2 (9)
um

Either answer is fine; the reference temperature difference associated with Eq. (7) is computed
according to:

DT_ref=qf_exp*H/(2*k) %reference temperature difference (K)

which leads to ΔTref = 1.25 K. The Brinkman number is:

Br = Ec Pr (10)

where Ec is the Eckert number:

um2
Ec = (11)
c ΔTref

and Pr is the Prandtl number:

ν
Pr = (12)
α

where ν is the kinematic viscosity:

μ
ν= (13)
ρ

and α is the thermal diffusivity:

k
α= (14)
ρc

nu=mu/rho; %kinematic viscosity (m^2/s)


alpha=k/(rho*c); %thermal diffusivity (m^2/s)
Pr=nu/alpha %Prandtl number (-)
Ec=u_m^2/(c*DT_ref) %Eckert number (-)
Br=Pr*Ec %Brinkman number (-)

which leads to Br = 0.58. Because the Brinkman number is larger than unity it is likely that
viscous dissipation is important for this problem.

c.) Is axial conduction important for this problem? Justify your answer.
Axial conduction scales according to Pe-2, where Pe is the Peclet number:

Pe = Re Pr (15)

Pe=Re*Pr %Peclet number (-)

which leads to Pe = 3400. Therefore, axial conduction is not likely to be important.

d.) Develop a numerical model of the thermal behavior of the flow through the gap that accounts
for viscous dissipation but not axial conduction. Use the native ode solver in MATLAB.

There are N nodes distributed uniformly in the y direction across the lens/wafer gap, as shown in
Figure 2.

Figure 2: Node placement and energy balance.

The distance between adjacent nodes


H
Δy = (16)
N

and the location of each of the nodes is given by:

⎛ 1⎞
yi = Δy ⎜ i − ⎟ for i = 1..N (17)
⎝ 2⎠

N=21; %number of nodes (-)


Dy=H/N; %distance between y-nodes (m)
for i=1:N
y(i)=Dy*(i-1/2); %y-position of each node (m)
end

The velocity at each of the nodes is obtained from Eq. Error! Reference source not found.:

⎞ H Δp ⎡⎛ yi ⎞ ⎛ yi ⎞ ⎤
2

2
y
ui = uw ⎜1 − i +
⎟ 2 μ L ⎢⎜ H ⎟ − ⎜ H ⎟ ⎥ for i = 1..N (18)
⎝ H ⎠ ⎢⎣⎝ ⎠ ⎝ ⎠ ⎥⎦

The velocity gradient at each node will also be required in order to compute the viscous
dissipation:

du uw H 2 Δp ⎡ 1 2 yi ⎤
=− + − for i = 1..N (19)
dy i H 2 μ L ⎢⎣ H H 2 ⎥⎦

for i=1:N
u(i)=u_w*(1-y(i)/H)+H^2*DeltaP*(y(i)/H-(y(i)/H)^2)/(2*mu*L); %velocity
dudy(i)=-u_w/H+H^2*DeltaP*(1/H-2*y(i)/H^2)/(2*mu*L);
%velocity gradient
end

The internal nodes are treated separately from the boundary nodes; a control volume for an
internal node is shown in Figure 2. The node experiences conduction in the y direction from
adjacent nodes, volumetric generation (the transformation of the fluid’s mechanical energy into
thermal energy), and enthalpy carried by fluid entering the control volume at x and leaving at x +
dx; the energy balance suggested by the control volume in Figure 2 is:

( ρ c ui ΔyW Ti ) x + qtop + qbottom + g v = ( ρ c ui ΔyW Ti ) x + dx (20)

where W is the depth of the channel into the page. The conduction heat transfer rates are
approximated according to:

k dxW
qtop = (Ti +1 − Ti ) (21)
Δy
k dxW
qbottom = (Ti −1 − Ti ) (22)
Δy

The viscous dissipation is given by:


2
⎛ ∂u ⎞
g v = μ ⎜⎜ ⎟⎟ Δy dxW (23)
∂y
⎝ i⎠

Equations (24) through (23) are substituted into Eq. (20) and the x+dx term is expanded:

2
k dxW k dxW ⎛ ∂u ⎞
( ρ c ui ΔyW Ti ) x + (Ti +1 − Ti ) + (Ti −1 − Ti ) + μ ⎜⎜ ⎟⎟ Δy dxW =
Δy Δy ⎝ ∂y i ⎠
d
( ρ c ui ΔyW Ti ) x + ( ρ c ui ΔyW Ti ) dx (25)
dx
for i = 2.. ( N − 1)

Note that the only term in the derivative that changes with x is the temperature (ui is constant in
the hydrodynamically fully developed region) and therefore Eq. (25) can be rewritten as:
2
k dxW k dxW ⎛ ∂u ⎞ dT
(Ti +1 − Ti ) + (Ti −1 − Ti ) + μ ⎜⎜ ⎟⎟ Δy dxW = ρ c ui ΔyW i dx for i = 2.. ( N − 1)
Δy Δy ⎝ ∂y i ⎠ dx
(26)

Solving for the rate of change of Ti with respect to x:


2
dTi k μ ⎛ ∂u ⎞
= (Ti +1 + Ti −1 − 2 Ti ) + ⎜ ⎟ for i = 2.. ( N − 1) (27)
dx ρ c Δy ui
2
ρ c ui ⎜⎝ ∂y i ⎟⎠

An energy balance for the control volume around node 1 is also shown in Figure 2 and leads to:

dT1
qtop + qwafer + g v = ρ c u1 ΔyW dx (28)
dx

The conductive heat transfer from node 2 is approximated according to:

k dxW
qtop = (T2 − T1 ) (29)
Δy

The heat transfer from the wafer is:


qwafer = qs′′ ( x ) dxW (30)

The viscous dissipation is given by:

2
⎛ ∂u ⎞
g v = μ ⎜⎜ ⎟⎟ Δy dxW (31)
⎝ ∂y 1 ⎠

Substituting Eqs. (29) through (31) into Eq. (28) leads to:

2
k dxW ⎛ ∂u ⎞ dT
(T2 − T1 ) + qs′′ ( x ) dxW + μ ⎜⎜ ⎟⎟ Δy dxW = ρ c u1 ΔyW 1 dx (32)
Δy ⎝ ∂y 1 ⎠ dx

Solving for the rate of change of the temperature of node 1 leads to:

q s′′ ( x )
2
dT1 k μ ⎛ ∂u ⎞
= ( T − T ) + + ⎜ ⎟ (33)
dx ρ c u1 Δy 2
2 1
ρ c u1 Δy ρ c u1 ⎜⎝ ∂y 1 ⎟⎠

An energy balance for the control volume around node 1 is also shown in Figure 2 and leads to:

dTN
qlens + qbottom + g v = ρ c u N ΔyW dx (34)
dx

The conductive heat transfer rates are:

k dxW
qbottom = (TN −1 − TN ) (35)
Δy

2 k dxW
qlens = (Tlens − TN ) (36)
Δy

and the volumetric generation by viscous dissipation is:


2
⎛ du ⎞
g v = μ ⎜⎜ ⎟⎟ W Δy dx (37)
⎝ dy N ⎠

Substituting Eqs. (35) through (37) into Eq. (34) leads to:
2
2 k dxW k dxW ⎛ du ⎞ dTN
(Tlens − TN ) + (TN −1 − TN ) + μ ⎜⎜ ⎟⎟ W Δy dx = ρ c u N ΔyW dx (38)
Δy Δy ⎝ dy N ⎠
dx
Solving for the rate of temperature change of node N:
2
dTN 2k k μ ⎛ du ⎞
2 ( lens
= T − TN ) + 2 ( N −1
T − TN ) + ⎜⎜ ⎟⎟ (39)
dx ρ c u N Δy ρ c u N Δy ρ c uN ⎝ dy N ⎠

A function, dTdx_P5p3_1.m, is defined in order to return the rate of change of each node:

function[dTdx]=dTdx_P5p3_1(x,T,L,w_exp,qf_exp,u,dudy,Dy,k,rho,c,mu,T_lens)

[N,g]=size(T); %determine number of nodes


dTdx=zeros(N,1); %initialize dTdx
%heat flux
if (x<(L-w_exp)/2)
qf=0;
elseif (x<(L+w_exp)/2)
qf=qf_exp;
else
qf=0;
end

dTdx(1)=k*(T(2)-
T(1))/(rho*c*u(1)*Dy^2)+qf/(rho*c*u(1)*Dy)+mu*dudy(1)^2/(rho*c*u(1));
for i=2:(N-1)
dTdx(i)=k*(T(i+1)+T(i-1)-
2*T(i))/(Dy^2*rho*c*u(i))+mu*dudy(i)^2/(rho*c*u(i));
end
dTdx(N)=2*k*(T_lens-T(N))/(rho*c*u(N)*Dy^2)+k*(T(N-1)-
T(N))/(rho*c*u(N)*Dy^2)+mu*dudy(N)^2/(rho*c*u(N));

The ODE solver ode45 is used to integrate the function dTdx_P5p3_1 over the length of the
duct:

OPTIONS=odeset('RelTol',1e-6);
[x,T]=ode45(@(x,T)
dTdx_P5p3_1(x,T,L,w_exp,qf_exp,u,dudy,Dy,k,rho,c,mu,T_lens),...
[0,L],T_in*ones(N,1),OPTIONS);

e.) Prepare a contour plot that shows the temperature distribution in the lens/wafer gap.

The contourf command creates a filled contour plot of the result, shown in Figure 3.

>> contourf(x,y,T',21)
Figure 3: Temperature distribution in the lens/wafer gap.

f.) Prepare a contour plot that shows the temperature distribution in the absence of any applied
heat flux (i.e., what is the heating caused by the viscous dissipation?).

The externally applied heat flux is set to zero:

qf_exp=0; %heat flux in exposure region (W/m^2)

and the temperature distribution is shown in Figure 4.


Figure 4: Temperature distribution in the lens/wafer gap due to viscous dissipation.
Problem 5.5-3 (5-10 in text)
Figure P5.5-3 shows a thin-wall tube with radius R = 5.0 mm carrying a flow of liquid with
density ρ = 1000 kg/m3, specific heat capacity c = 1000 J/kg-K, conductivity k = 0.5 W/m-K, and
viscosity μ = 0.017 Pa-s. The fluid is fully developed hydrodynamically with a bulk velocity
um= 0.2 m/s and has a uniform temperature Tini = 80°C when it enters a section of the tube that is
exposed to air at temperature T∞ = 20°C with heat transfer coefficient ha .

um = 0.2 m/s T∞ = 20°C, ha


Tini = 80°C R = 5 mm
μ = 0.017 Pa-s
k = 0.5 W/m-K
c = 1000 J/kg-K
ρ = 1000 kg/m3
L
Figure P5.5-3: Thin-wall tube carrying fluid exposed to air.

The Reynolds number for this flow is around 100 and so this is a laminar flow. Typically, the
heat transfer between the fluid and the air is modeled using a laminar flow heat transfer
coefficient that is calculated using correlations that are based on a constant tube surface
temperature. In fact, the surface temperature of the tube is not constant for this process. The
objective of this homework is to understand how much this approximation affects the solution.
a.) Develop a numerical model of this situation using MATLAB. Prepare a plot showing the
temperature at various radii as a function of axial position for the case where ha = 100 W/m2-
K and L = 5.0 m. Prepare a plot of the Nusselt number as a function of axial position for this
situation.

The inputs are entered in a MATLAB script, P5p5d3.m:

clear all;
L=5; % length of pipe (m)
R=0.005; % radius of pipe (m)
Tin=80; % inlet temperature of water (C)
c=1000; % specific heat capacity of fluid (J/kg-K)
rho=1000; % density of fluid (kg/m^3)
k=0.5; % thermal conductivity (W/m-k)
um=0.2; % bulk velocity of flow (m/s)
mu=0.017; % viscosity (Pa-s)
ha=100; % heat transfer coefficient with surrounding air (W/m^2-K)
Ta=20; % temperature of surrounding air (C)

The nodes are distributed radially according to:

⎛ 1⎞
ri = Δr ⎜ i − ⎟ for i = 1..N (1)
⎝ 2⎠

where
R
Δr = (2)
N

N=21; % number of radial nodes (-)


Dr=R/N; % distance between nodes (m)
for i=1:N
r(i)=Dr*(i-1/2); % position of each node (m)
end

The velocity distribution for a fully developed flow is:

⎡ ⎛ ri ⎞2 ⎤
ui = 2 um ⎢1- ⎜ ⎟ ⎥ for i = 1..N (3)
⎢⎣ ⎝ R ⎠ ⎥⎦

for i=1:N
u(i)=2*um*(1-(r(i)/R)^2); % velocity at each node (m/s)
end

The cross-sectional area for flow into each node is given by:

⎡⎛ Δr ⎞ ⎛
2
Δr ⎞ ⎤
2

Ac ,i = π ⎢⎜ ri + ⎟ − ⎜ ri − ⎟ ⎥ for i = 1..N (4)


⎣⎢⎝ 2 ⎠ ⎝ 2 ⎠ ⎦⎥

for i=1:N
Ac(i)=pi*((r(i)+Dr/2)^2-(r(i)-Dr/2)^2);
% cross-sectional area for each node (m^2)
end

An energy balance on the internal nodes leads to:

ρ c ui Ac ,i
dTi (T − T ) (T − T )
= 2 π k i +1 i + 2 π k i −1 i for i = 2.. ( N − 1) (5)
dx ⎛r ⎞ ⎛ r ⎞
ln ⎜ i +1 ⎟ ln ⎜ i ⎟
⎝ ri ⎠ ⎝ ri −1 ⎠

which is solved for the state equations for the internal nodes:

⎡ ⎤
⎢ ⎥
dTi
=
1 ⎢ 2 π k (Ti +1 − Ti ) + 2 π k (Ti −1 − Ti ) + g ′′′ A ⎥ for i = 2.. ( N − 1) (6)
dx ρ c ui Ac ,i ⎢ ⎥
v ,i c , i
⎛ ri +1 ⎞ ⎛ ri ⎞
⎢ ln ⎜ ⎟ ln ⎜ ⎟ ⎥
⎣⎢ ⎝ ri ⎠ ⎝ ri −1 ⎠ ⎥⎦

An energy balance on node 1 leads to:


ρ c u1 Ac ,1
dT1 (T − T )
= 2π k 2 1 (7)
dx ⎛r ⎞
ln ⎜ 2 ⎟
⎝ r1 ⎠

which is solved for the state equation for node 1:

dT1 2 π k (T2 − T1 )
= (8)
dx ρ c u1 Ac ,1 ⎛ r2 ⎞
ln ⎜ ⎟
⎝ r1 ⎠

An energy balance on node N leads to:

ρ c u N Ac , N
dTN
=
(T∞ − TN ) + 2π k
(TN −1 − TN ) (9)
dx ⎛R⎞ ⎛ r ⎞
ln ⎜ ⎟ ln ⎜ N ⎟
r ⎝ rN −1 ⎠
+ ⎝ N⎠
1
ha 2 π R 2π k

which is solved for the state equation for node N:

⎡ ⎤
⎢ ⎥
⎢ ⎥
dTN
=
1 ⎢ ( ∞ N)
T − T ( T −
+ 2 π k N −1 N ⎥
T )⎥
⎢ (10)
dx ρ c u N Ac , N ⎢ ⎛R⎞ ⎛ r ⎞
ln ⎜ ⎟ ln ⎜ N ⎟ ⎥
⎢ 1 r ⎝ rN −1 ⎠ ⎥
⎢ + ⎝ N⎠ ⎥
⎣ ha 2 π R 2π k ⎦

The state equations are programmed in a function dTdt_p5p5d3:

function[dTdx]=dTdt_P5p5d3(x,T,u,Ac,r,R,k,Ta,rho,c,ha,N)

% Inputs
% x - position (m)
% T - vector of temperatures (K)
% u - vector of velocities (m/s)
% Ac - vector of cross-sectional areas for each node (m^2)
% r - vector of radii for each node (m)
% R - radius of tube (m)
% k - thermal conductivity (W/m-K)
% Ta - ambient temperature (K)
% rho - density (kg/m^3)
% c - specific heat capacity (J/kg-K)
% ha - heat transfer coefficient (W/m^2-K)
% N - number of nodes (-)
%
% Output:
% dTdx - rate of temperature change for each node (K/m)

dTdx=zeros(N,1);
dTdx(1)=(2*pi*k*(T(2)-T(1))/(log(r(2)/r(1))))/(rho*c*u(1)*Ac(1));
for i=2:(N-1)
dTdx(i)=(2*pi*k*(T(i+1)-T(i))/log(r(i+1)/r(i))+2*pi*k*(T(i-1)-...
T(i))/log(r(i)/r(i-1)))/(rho*c*u(i)*Ac(i));
end
dTdx(N)=(2*pi*k*(T(N-1)-T(N))/log(r(N)/r(N-1))+...
(Ta-T(N))/(1/(ha*2*pi*R)+log(R/r(N))/(2*pi*k)))/(rho*c*u(N)*Ac(N));
end

and integrated using the ode45 solver:

OPTIONS=odeset('RelTol',1e-6);
[x,T]=ode45(@(x,T) dTdt_P5p5d3(x,T,u,Ac,r,R,k,Ta,rho,c,ha,N)...
,[0,L],Tin*ones(N,1),OPTIONS);

The mean temperature is computed according to:

∑T i, j ui Ac ,i
Tm, j = i =1
(11)
D2
um π
4

[M,g]=size(x);
for j=1:M
T_mean(j)=0;
for i=1:N
T_mean(j)=T_mean(j)+Ac(i)*u(i)*T(j,i);
end
T_mean(j)=T_mean(j)/(pi*R^2*um);
end

Figure 2 shows the temperature at various radii as a function of axial position for the case where
ha = 100 W/m2-K and L = 5.0 m.
85

80
r=0
75

Temperature (deg. C)
70

65

60

55
r=R
50

45 increasing radius

40
0 1 2 3 4 5
Axial position (m)
Figure 2: Temperature as a function of axial position for various values of radius.

The surface heat flux at any location i is obtained according to:

qs′′, j =
(T∞ − TN , j )
(12)
⎛R⎞
R ln ⎜ ⎟
1
+ ⎝ rN ⎠
ha k

and the surface temperature is given by:

qs′′
Ts , j = T∞ − (13)
ha

for j=1:M
qs(j)=(Ta-T(j,N))/(1/ha+R*log(R/r(N))/k); % surface heat flux (W/m^2)
Ts(j)=Ta-qs(j)/ha; % surface temperature (K)
end

The local heat transfer coefficient on the liquid side and the local Nusselt number are defined
according to:

qs′′
h= (14)
(Ts − Tm )
h2R
Nu = (15)
k

for j=1:M
htc(j)=qs(j)/(Ts(j)-T_mean(j)); % heat transfer coefficient (W/m^2-K)
Nusselt(j)=htc(j)*2*R/k; % Nusselt number (-)
end

Figure 3 illustrates the Nusselt number as a function of position.

20

15
Nusselt number

10

0
0 1 2 3 4 5
Axial position
Figure 3: Nusselt number as a function of axial position.

b.) Verify that your solution by comparing it with an appropriate analytical model.

In the limit that ha → ∞ , the surface temperature of the tube becomes constant and equal to T∞.
The value of ha is set to a large number:

ha=100000; %heat transfer coefficient with surrounding air (W/m^2-K)

and the MATLAB simulation is run to obtain the local Nusselt number. The value is compared
to the correlation for laminar flow in a constant temperature tube programmed in EES and
accessed using the PipeFlow_N_local function:

$UnitSystem SI MASS RAD PA K J


$TABSTOPS 0.2 0.4 0.6 0.8 3.5 in

"Inputs"
L = 5 [m] "length of tube"
R = 0.005 [m] "tube radius"
c=1000 [J/kg-K] "specific heat capacity"
rho=1000 [kg/m^3] "density"
k=0.5 [W/m-K] "conductivity"
um=0.2 [m/s] "mean velocity"
mu=0.017 [Pa-s] "viscosity"

Re=um*2*R*rho/mu "Reynolds number"


Pr=mu*c/k "Prandtl number"
call PipeFlow_N_local(Re,Pr,x/(2*R),0 [-]: Nusselt_T_x,Nusselt_H_x,f_x)
"correlation for local Nusselt number"

Figure 4 illustrates the numerical model and correlation results.

15
correlation in EES
numerical model

10
Local Nusselt number

3
0.02 0.1 1 5
Axial position (m)
Figure 4: Correlation in EES compared to the numerical model solution in the limit that the tube surface
temperature is constant.

c.) Investigate the effect of the external convection coefficient; in the limit that ha is very large
your solution should limit (at long length) to Nu = 3.66. What is the effect of a finite ha ?
Present your conclusions in a logical and systematic manner.

Figure 5 illustrates the fully developed Nusselt number predicted by the numerical model as a
function of the dimensionless parameter:

k
(16)
R ha

which represents approximately the ratio of the thermal resistance on the air-side of the tube to
k
the thermal resistance on the fluid side. Notice that as approaches zero, the solution
R ha
approaches 3.66 which corresponds to fully developed laminar flow through a circular duct with
k
a constant surface temperature. As becomes large, the solution approaches 4.36 which
R ha
corresponds to fully developed laminar flow through a circular duct with a constant surface heat
flux.
4.3

Fully developed Nusselt number


4.2

4.1

3.9

3.8

3.7

3.6
0.01 0.1 1 10 20
k/(R ha)
k
Figure 5: Fully developed Nusselt number as a function of
R ha
Problem 5.5-4 (5-11 in text)
Figure P5.5-4 illustrates a flow of liquid in a passage formed between two parallel plates.

⎡ ⎛ 2π x ⎞⎤
q ′′ = q′′ + Δq ′′ sign ⎢sin ⎜ ⎟⎥
⎣ ⎝ Lh ⎠ ⎦

um = 0.2 m/s
Tin = 300K
μ = 0.05 Pa-s H = 0.5 mm
ρ = 1000 kg/m3
k = 0.25 W/m-K
c = 3800 J/kg-K

Lh = 1 mm Lh = 1 mm
x
Figure P5.5-4: Flow between two parallel plates.

The flow enters the duct having been exposed to a uniform heat flux at qs′′ = 9500 W/m2 for a
long time. Therefore, the flow is both thermally and hydrodynamically fully developed. The
velocity distribution is:
⎛ y y2 ⎞
u = 6 um ⎜ − 2 ⎟
⎝H H ⎠
where um = 0.2 m/s is the bulk velocity and H = 0.5 mm is the plate-to-plate spacing. The
temperature distribution at the inlet is:
qs′′ H ⎡ ⎛ y ⎞ ⎤
4 3
⎛ y⎞ ⎛ y⎞
T = Tin + ⎢ − ⎜ ⎟ + 2 ⎜ ⎟ − ⎜ ⎟ + 0.243⎥
k ⎣⎢ ⎝ H ⎠ ⎝H⎠ ⎝H⎠ ⎦⎥
where Tin = 300 K is the mean temperature of the fluid at the inlet. The properties of the fluid
are density ρ = 1000 kg/m3, viscosity μ = 0.05 Pa-s, conductivity k = 0.25 W/m-K, and specific
heat capacity c = 3800 J/kg-K. The heat flux applied to the surfaces of the channel is non-
uniform and you need to evaluate the impact of the non-uniform heat flux on the surface
temperature of the duct. The heat flux at both the upper and lower surfaces of the duct varies
according to:
⎡ ⎛ π x ⎞⎤
q ′′ = q ′′ + Δq ′′ sign ⎢sin ⎜ ⎟⎥
⎣ ⎝ Lh ⎠ ⎦
where Δq ′′ = 9500 W/m2 is the amplitude of the heat flux variation and Lh = 1 mm is the width
of the heated regions and the function sign returns +1 if the argument is positive and -1 if it is
negative. This equation, with the specified inputs, leads to a wall that alternates between having
a heat flux of 19,000 W/m2 for 1 mm and then being adiabatic for 1 mm.
a.) Is the flow laminar or turbulent?

The inputs are entered in MATLAB as a function:

u_m=0.2; % mean velocity (m/s)


H=0.0005; % duct height (m)
T_in=300; % inlet mean fluid temperature (K)
rho=1000; % density (kg/m^3)
mu=0.05; % viscosity (Pa-s)
k=0.25; % conductivity (Pa-s)
c=3800; % specific heat capacity (J/kg-K)
L=0.01; % length of duct (m)
qf_avg=9500; % average heat flux (W/m^2)
Dqf=9500; % heat flux fluctuation amplitude (W/m^2)
Lh=0.001; % length of heated section (m)
alpha=k/(rho*c) % thermal diffusivity (m^2/s)

The hydraulic diameter of the passage is calculated according to:

Dh = 2 H (1)

The Reynolds number is:

um Dh ρ
Re = (2)
μ

Dh=2*H; % hydraulic diameter


Re=u_m*Dh*rho/mu % Reynolds number (-)

which leads to Re = 4; therefore, the flow is laminar.

b.) Is viscous dissipation important?

The significance of viscous dissipation is quantified by the Brinkman number:

μ um2
Br = (3)
k (Ts − Tm )

The surface-to-mean temperature difference is evaluated at the inlet according to:

q s′′ Dh
Ts − Tm = (4)
Nu fd k

where Nufd is the fully developed Nusselt number for flow between parallel plates with a uniform
heat flux (8.235):

Nu_fd=8.235; % fully developed Nusselt number (-)


DT_uniform=qf_avg*Dh/(Nu_fd*k)
% surface-to-fluid temperature difference for a uniform heat flux
Br=mu*u_m^2/(k*DT_uniform) % Brinkman number

which leads to Br = 0.0017; therefore, viscous dissipation is negligible.


c.) Is axial conduction important?

The significance of axial conduction is quantified according to the inverse of the Peclet number
squared where the Peclet number is:

Pe = Re Pr (5)

The Prandtl number is calculated according to:

μc
Pr = (6)
k

Pr=mu*c/k % Prandtl number (-)


Pe=Re*Pr % Peclet number

which leads to Pe = 3040; therefore, axial conduction is not important.

d.) Develop a 2-D numerical model of the flow in the gap using the Crank-Nicolson solution
technique implemented in MATLAB. Plot the temperature as a function of position x at
various values of y for 0 < x < 1 cm.

The numerical solution proceeds by distributing N nodes uniformly in the y-direction across the
passage, as discussed in Section 5.5. The nodes are positioned in the center of N full sized
control volumes, as discussed in the text. The distance between adjacent nodes is:

H
Δy = (7)
N

and the location of each of the nodes is given by:

⎛ 1⎞
yi = Δy ⎜ i − ⎟ for i = 1..N (8)
⎝ 2⎠

% Setup y grid
N=101; % number of nodes in y direction (-)
Dy=H/N; % distance between nodes (m)
for i=1:N
y(i)=Dy*(i-1/2); % position of each node (m)
end

The velocity distribution in the duct is provided by Eq. Error! Reference source not found.:

% velocity distribution
for i=1:N
u(i)=6*u_m*(y(i)/H-(y(i)/H)^2); % velocity at each node (m/s)
end
The temperature distribution at the inlet to the duct is provided by Eq.
Error! Reference source not found.:

% Initial temperature distribution


for i=1:N
T(i,1)=T_in+(qf_avg*H/k)*(-y(i)^4/H^4+2*y(i)^3/H^3-y(i)/H+0.2429);
end

The axial location of each grid point is given by:

xi = ( j − 1) Δx for i = 1..M (9)

where M is the number of axial positions and Δx is the distance between axial positions, given
by:

L
Δx = (10)
( M − 1)
where L is the length of the duct to be simulated. The heat flux at each axial location is given by
Eq. Error! Reference source not found..

% setup x grid
M=2000; % number of length steps
Dx=L/(M-1); % size of length steps
for j=1:M
x(j)=(j-1)*Dx;
qfs(j)=qfs_f(x(j),qf_avg,Dqf,Lh);
end

A sub-function is defined to return the heat flux:

function[qf]=qfs_f(x,qf_avg,Dqf,Lh)

% Inputs:
% x - position (m)
% qf_avg - average heat flux (W/m^2)
% Dqf - amplitude of heat flux variation (W/m^2)
% Lh - distance of heated regions
%
% Outputs
% qf - heat flux (W/m^2)

qf=qf_avg+Dqf*sign(sin(2*pi*x/Lh));
end

The state equations are derived by defining a control volume around each node that extends a
finite spatial extent in the y-direction (Δy) but is differentially small in the x-direction (dx). The
energy balance for a control volume around an internal node is:
dTi
( ρ c ui ΔyW Ti ) x + qtop + qbottom = ( ρ c ui ΔyW Ti ) x + ρ c ui ΔyW dx (11)
dx

where W is the depth of the channel into the page. The conduction heat transfer rates are
approximated according to:

k dxW
qtop = (Ti +1 − Ti ) (12)
Δy

k dxW
qbottom = (Ti −1 − Ti ) (13)
Δy

Equations (12) and (13) are substituted into Eq. (11):

k dxW k dxW dT
(Ti +1 − Ti ) + (Ti −1 − Ti ) = ρ c ui ΔyW i dx for i = 2.. ( N − 1) (14)
Δy Δy dx

Solving for the rate of change of Ti with respect to x leads to:

dTi k
= (Ti +1 + Ti −1 − 2 Ti ) for i = 2.. ( N − 1) (15)
dx ρ c Δy 2 ui

An energy balance for the control volume around node 1 leads to:

k dxW dT
(T2 − T1 ) + qs′′W dx = ρ c u1 ΔyW 1 dx (16)
Δy dx

Solving for the rate of change of the temperature of node 1 leads to:

dT1 k q s′′
2 ( 2
= T − T1 ) + (17)
dx ρ c u1 Δy ρ c u1 Δy

A similar process applied to node N leads to:

dTN k qs′′
2 ( N −1
= T − TN ) + (18)
dx ρ c u1 Δy ρ c u1 Δy

Equations (16), (17), and (18) are the N state equations for the problem. According to the
problem statement, these equations must be integrated from the entrance of the heated region
downstream using the Crank-Nicolson technique in MATLAB. The formula for taking a Crank-
Nicolson step is:
⎡ dT dT ⎤ Δx
Ti , j +1 = Ti , j + ⎢ + ⎥ for i = 1... N (19)
⎣⎢ dx T =Ti , j , x = x j dx ⎥
T =Ti , j +1 , x = x j +1 ⎦ 2

Substituting Eqs. (16), (17), and (18) into Eq. (19) leads to:

T1, j +1 = T1, j +
k
⎡ ( T − T ) + ( T − T ) ⎤ +
(
Δx q s′′, x j + qs′′, x j+1 Δx ) (20)
ρ c u1 Δy 2 ⎣ 2, j 1, j 2, j +1 1, j +1 ⎦
2 ρ c u1 Δy 2

Δx
⎡(Ti +1, j + Ti −1, j − 2 Ti , j ) + (Ti +1, j +1 + Ti −1, j +1 − 2 Ti , j +1 )⎤
k
Ti , j +1 = Ti , j + ⎣ ⎦
ρ c Δy ui
2
2 (21)
for i = 2... ( N − 1)

TN , j +1 = TN , j +
k
⎡ ( T − T + T − T
Δx qs′′, x j + qs′′, x j+1 Δx
) ( N −1, j +1 N , j +1 )⎦ 2 + ρ c u Δy 2

( ) (22)
ρ c u N Δy 2 ⎣ N −1, j N , j N

Equations (20) through (22) are a set of N linear equations in the unknown temperatures Ti,j+1
where i = 1..N. These equations must be placed into matrix format in order to move forward a
length step:

AX =b (23)

where X is:

⎡ X 1 = T1, j +1 ⎤
⎢ ⎥
⎢ X 2 = T2, j +1 ⎥
X= (24)
⎢... ⎥
⎢ ⎥
⎢⎣ X N = TN , j +1 ⎥⎦

and A is:

⎡ row 1 = control volume 1 equation ⎤


⎢ row 2 = control volume 2 equation ⎥
⎢ ⎥
A = ⎢ row 3 = control volume 3 equation ⎥ (25)
⎢ ⎥
⎢... ⎥
⎢⎣ row N = control volume N equation ⎥⎦
Equations (20) through (22) are rearranged so that the coefficients multiplying the unknowns and
the constants for the linear equations are clear:

⎡ k Δx ⎤ ⎡ k Δx ⎤
T1, j +1 ⎢1 + 2⎥
+ T2, j +1 ⎢ − ⎥=
2 ρ c u1 Δy ⎦
⎣
⎣ 2 ρ c u1 Δy 2 ⎦

A1,1 A1,2

( )
(26)
Δx Δx qs′′, x j + qs′′, x j+1
2 ( 2, j
T − T1, j )
k
T1, j + +
ρ c u1 Δy 2 ρ c u1 Δy 2


b1

⎡ k Δx ⎤ ⎡ k Δx ⎤ ⎡ k Δx ⎤
Ti , j +1 ⎢1 + ⎥ + Ti +1, j +1 ⎢ − ⎥ + Ti −1, j +1 ⎢ − =
⎣ ρ c Δy ui ⎦
2
⎣ 2 ρ c Δy ui ⎦
2
⎣ 2 ρ c Δy 2 ui ⎥⎦




Ai ,i Ai ,i +1 Ai ,i −1

k Δx
Ti , j + ⎡Ti +1, j + Ti −1, j − 2 Ti , j ⎤⎦ (27)
2 ρ c Δy 2 ui ⎣


bi

for i = 2... ( N − 1)

⎡ k Δx ⎤ ⎡ k Δx ⎤
TN , j +1 ⎢1 + 2⎥
+ TN −1, j +1 ⎢ − 2⎥
=
⎣ 2 ρ c u N Δy ⎦ ⎣ 2 ρ c u N Δy ⎦



AN , N AN , N −1

( )
(28)
Δx qs′′, x j + qs′′, x j+1 Δx
TN , j +
k
( T − T )
ρ c u N Δy 2 N −1, j N , j 2
+
ρ c u N Δy 2


bN

The matrix A and vector b are initialized:

A=spalloc(N,N,3*N); % initialize A
b=zeros(N,1); % initialize b

The matrix A can be constructed just once and used without modification to move through each
length step:

% setup A matrix
A(1,1)=1+k*Dx/(2*rho*c*Dy^2*u(1));
A(1,2)=-k*Dx/(2*rho*c*Dy^2*u(1));
for i=2:(N-1)
A(i,i)=1+k*Dx/(rho*c*Dy^2*u(i));
A(i,i+1)=-k*Dx/(2*rho*c*Dy^2*u(i));
A(i,i-1)=-k*Dx/(2*rho*c*Dy^2*u(i));
end
A(N,N)=1+k*Dx/(2*rho*c*Dy^2*u(N));
A(N,N-1)=-k*Dx/(2*rho*c*Dy^2*u(N));

The vector b is reconstructed during each step:

% step through space


for j=1:(M-1)
b(1)=T(1,j)+(k*Dx/(2*rho*c*Dy^2*u(1)))*(T(2,j)-...
T(1,j))+Dx*(qfs(j)+qfs(j+1))/(2*rho*c*u(1)*Dy);
for i=2:(N-1)
b(i)=T(i,j)+k*Dx*(T(i+1,j)+T(i-1,j)-...
2*T(i,j))/(2*rho*c*Dy^2*u(i));
end
b(N)=T(N,j)+(k*Dx/(2*rho*c*Dy^2*u(N)))*(T(N-1,j)-...
T(N,j))+Dx*(qfs(j)+qfs(j+1))/(2*rho*c*u(N)*Dy);
T(:,j+1)=A\b;
end

The temperature as a function of position for various values of y is shown in Figure P5.5-4(b).
307
y = 2.5 μm
306

305

304
Temperature (K)

303 y = 47 μm

302
y = 97 μm
301

300 y = 146 μm
y = 196 μm
299
y = 245 μm
298
0 0.002 0.004 0.006 0.008 0.01
Position (m)
Figure P5.5-4(b): Temperature as a function of position for various values of y.

Notice that the effect of the oscillation extends into the duct approximately 47 μm which is
consistent with the diffusion penetration depth associated with a single heated region:

Lh
δ diff = 2 α t = 2 α (29)
um

diff=2*sqrt(alpha*Lh/u_m) % diffusion distance into stream (m)

which leads to δdiff = 36 μm.

e.) Determine the surface temperature of the duct and the mean temperature of the fluid at each
axial position. Plot Ts and Tm as a function of position x.
The mean temperature is defined as the ratio of the energy carried by the flow to the capacitance
rate of the flow ( m c ):

N N

∑ ui Δy ρ cTi ∑ u Δy T
i i
Tm, j = i =1
= i =1
(30)
um H ρ c um H

The surface temperature of the duct is obtained by extrapolating from nodes 1 and 2:

Ts , j = T2, j + (T1, j − T2, j )


3
(31)
2

for j=1:M
E_dot=0;
for i=1:N
E_dot=E_dot+u(i)*T(i,j)*Dy;
end
T_mean(j)=E_dot/(u_m*H);
T_s(j)=T(2,j)+(T(1,j)-T(2,j))*3/2;
end

Figure 5.5-4(c) shows the mean temperature and the surface temperature as a function of
position.
307

306

305
Temperature (K)

304

surface temperature
303

302

301 mean temperature

300
0 0.002 0.004 0.006 0.008 0.01
Position (m)
Figure P5.5-4(c): Mean and surface temperature as a function of position.

f.) Determine the Nusselt number at each axial position. Plot the Nusselt number as a function
of x. Explain the shape of your plot.

The heat transfer coefficient is defined as:

q ′′
h= (32)
(Ts − Tm )
and the Nusselt number is computed according to:

h Dh
Nu = (33)
k

for j=1:M
htc(j)=qfs(j)/(T_s(j)-T_mean(j));
Nu(j)=htc(j)*Dh/k;
end

Figure 5.5-4(d) illustrates the Nusselt number as a function of position. The Nusselt number
spikes at the entrance to each heated region as the thermal boundary layer because the thermal
boundary layer is partially destroyed in each adiabatic region and so you have some developing
region.
22.5

20

17.5

15
Nusselt number

12.5

10

7.5

2.5

0
0 0.002 0.004 0.006 0.008 0.01
Position (m)
Figure P5.5-4(d): Nusselt number as a function of position.

g.) Verify that your model is working correctly by setting Δq ′′ = 0 and showing that the Nusselt
number in the duct is consistent with the Nusselt number for fully developed flow between
parallel plates subjected to a constant heat flux.

The amplitude of the variation is set to zero, leading to a uniform heat flux:

Dqf=0;%9500; % heat flux fluctuation amplitude (W/m^2)

The Nusselt number as a function of position is shown in Figure 5.5-4(e) and does limit to Nu =
8.235 which is the appropriate value for thermally and hydrodynamically fully developed flow
between parallel plates.
10

Nusselt number
6

0
0 0.002 0.004 0.006 0.008 0.01
Position (m)
Figure P5.5-4(e): Nusselt number as a function of position.

Return the value of Δq ′′ to 9500 W/m2. You are interested in studying the impact of the non-
uniform heat flux on the surface temperature of the duct. There are two natural limits to this
problem.
h.) Calculate the surface-to-mean temperature difference experienced when the average heat flux
is applied at the wall and the flow is fully developed.

The surface-to-mean temperature difference experienced when the average heat flux is applied at
the wall and the flow is fully developed is given by:

q ′′ Dh
ΔTq ′′ = (34)
Nu fd k

where Nufd is the fully developed Nusselt number (8.235):

DT_uniform=qf_avg*Dh/(Nu_fd*k)
% surface-to-fluid temperature difference for fully developed flow with
% average heat flux

which leads to ΔTq ′′ = 4.615 K.

i.) Calculate the surface-to-mean temperature difference experienced when the peak heat flux is
applied at the wall and the flow is fully developed.

The surface-to-mean temperature difference experienced when the peak heat flux is applied at
the wall and the flow is fully developed is given by:

ΔTq ′′+Δq ′′ =
( q′′ + Δq′′) Dh
(35)
Nu fd k
where Nufd is the fully developed Nusselt number.

DT_peak=(qf_avg+Dqf)*Dh/(Nu_fd*k)
% surface-to-fluid temperature difference for fully developed flow with
% peak heat flux

which leads to ΔTq ′′+Δq ′′ = 9.229 K.

j.) Define a meaningful dimensionless spatial period, Lh ,and plot the surface-to-mean
temperature difference as a function of dimensionless spatial period. Show that when Lh is
small, the solution limits to your answer from (h) and when Lh is large then your solution
limits to your solution from (i). Explain this result.

The dimensionless spatial period is the ratio of the period, Lh, to the axial distance required for
the energy to diffuse to the center of the passage, Lchar:

L
Lh = h (36)
Lchar

where

H 2 um
Lchar = (37)

L_char=H^2*u_m/(4*alpha) %characteristic length


Lh_bar=Lh/L_char % dimensionless spatial period

The function is modified so that it has inputs Lh and L and returns the maximum surface-to-
mean temperature difference, ΔTmax, calculated according to:

DT=T_s-T_mean;
DT_max=max(DT);

The function listing is:

function[x,y,T,qfs,T_mean,T_s,htc,Nu,DT_max,Lh_bar]=P5p5d4(Lh,L)

% Inputs:
% Lh - length of heated/adiabatic regions (m)
% L - length of duct to simulate (m)
%
% Outputs:
% x - vector of axial positions (m)
% y - vector of y-positions (m)
% T - matrix of temperature solutions (K)
% qfs - vector heat flux at duct surface (W/m^2)
% T_mean - vector of mean temperature (K)
% T_s - vector of duct surface temperature (K)
% htc - heat transfer coefficient (W/m^2-K)
% Nu - Nusselt number (-)
% T_s_conv - surface temperature predicted using fully developed Nusselt
% number (K)
% DT_max - maximum surface-to-mean temperature difference in duct (K)
% Lh_bar - dimensionless spatial period (-)

u_m=0.2; % mean velocity (m/s)


H=0.0005; % duct height (m)
T_in=300; % inlet mean fluid temperature (K)
rho=1000; % density (kg/m^3)
mu=0.05; % viscosity (Pa-s)
k=0.25; % conductivity (Pa-s)
c=3800; % specific heat capacity (J/kg-K)
%L=0.01; % length of duct (m)
qf_avg=9500; % average heat flux (W/m^2)
Dqf=9500; % heat flux fluctuation amplitude (W/m^2)
%Lh=0.001; % length of heated section (m)
alpha=k/(rho*c) % thermal diffusivity (m^2/s)

Dh=2*H; % hydraulic diameter


Re=u_m*Dh*rho/mu % Reynolds number (-)
Nu_fd=8.235; % fully developed Nusselt number (-)
DT_uniform=qf_avg*Dh/(Nu_fd*k)
% surface-to-fluid temperature difference for fully developed flow with
% average heat flux
DT_peak=(qf_avg+Dqf)*Dh/(Nu_fd*k)
% surface-to-fluid temperature difference for fully developed flow with
% peak heat flux
Br=mu*u_m^2/(k*DT_uniform) % Brinkman number
Pr=mu*c/k % Prandtl number (-)
Pe=Re*Pr % Peclet number

diff=2*sqrt(alpha*Lh/u_m) % diffusion distance into stream (m)


L_char=H^2*u_m/(4*alpha) %characteristic length
Lh_bar=Lh/L_char % dimensionless spatial period

% Setup y grid
N=101; % number of nodes in y direction (-)
Dy=H/N; % distance between nodes (m)
for i=1:N
y(i)=Dy*(i-1/2); % position of each node (m)
end

% velocity distribution
for i=1:N
u(i)=6*u_m*(y(i)/H-(y(i)/H)^2); % velocity at each node (m/s)
end

% Initial temperature distribution


for i=1:N
T(i,1)=T_in+(qf_avg*H/k)*(-y(i)^4/H^4+2*y(i)^3/H^3-y(i)/H+0.2429);
end
% setup x grid
M=2000; % number of length steps
Dx=L/(M-1); % size of length steps
for j=1:M
x(j)=(j-1)*Dx;
qfs(j)=qfs_f(x(j),qf_avg,Dqf,Lh);
end

A=spalloc(N,N,3*N); % initialize A
b=zeros(N,1); % initialize b

% setup A matrix
A(1,1)=1+k*Dx/(2*rho*c*Dy^2*u(1));
A(1,2)=-k*Dx/(2*rho*c*Dy^2*u(1));
for i=2:(N-1)
A(i,i)=1+k*Dx/(rho*c*Dy^2*u(i));
A(i,i+1)=-k*Dx/(2*rho*c*Dy^2*u(i));
A(i,i-1)=-k*Dx/(2*rho*c*Dy^2*u(i));
end
A(N,N)=1+k*Dx/(2*rho*c*Dy^2*u(N));
A(N,N-1)=-k*Dx/(2*rho*c*Dy^2*u(N));

% step through space


for j=1:(M-1)
b(1)=T(1,j)+(k*Dx/(2*rho*c*Dy^2*u(1)))*(T(2,j)-...
T(1,j))+Dx*(qfs(j)+qfs(j+1))/(2*rho*c*u(1)*Dy);
for i=2:(N-1)
b(i)=T(i,j)+k*Dx*(T(i+1,j)+T(i-1,j)-...
2*T(i,j))/(2*rho*c*Dy^2*u(i));
end
b(N)=T(N,j)+(k*Dx/(2*rho*c*Dy^2*u(N)))*(T(N-1,j)-...
T(N,j))+Dx*(qfs(j)+qfs(j+1))/(2*rho*c*u(N)*Dy);
T(:,j+1)=A\b;
end

for j=1:M
E_dot=0;
for i=1:N
E_dot=E_dot+u(i)*T(i,j)*Dy;
end
T_mean(j)=E_dot/(u_m*H);
T_s(j)=T(2,j)+(T(1,j)-T(2,j))*3/2;
end
for j=1:M
htc(j)=qfs(j)/(T_s(j)-T_mean(j));
Nu(j)=htc(j)*Dh/k;
end
DT=T_s-T_mean;
DT_max=max(DT);
end

function[qf]=qfs_f(x,qf_avg,Dqf,Lh)

% Inputs:
% x - position (m)
% qf_avg - average heat flux (W/m^2)
% Dqf - amplitude of heat flux variation (W/m^2)
% Lh - distance of heated regions
%
% Outputs
% qf - heat flux (W/m^2)

qf=qf_avg+Dqf*sign(sin(2*pi*x/Lh));
end

A script is written to call the function for a range of Lh (note that the simulated length of the duct
is always L = 10 Lh in order to capture several heated regions.

Lh=[0.00001,0.00002,0.00004,0.00007,0.0001,0.0002,0.0004,0.0007,0.001,...
0.002,0.004,0.007,0.01,0.02,0.04,0.07,0.1,0.2,0.4,0.7,1];
for i=1:21
i
[x,y,T,qfs,T_mean,T_s,htc,Nu,T_s_conv,DT_max(i),Lh_bar(i)]=...
P5p5d4(Lh(i),10*Lh(i));
end

Figure 5.5-4(f) shows the maximum surface-to-mean temperature difference as a function of the
dimensionless period of the heated regions. Notice that when Lh is small, the energy from each
region does not diffuse substantially into the passage and therefore the answer limits to ΔTq ′′ =
4.615 K (the solution for a uniform heat flux is valid). In the other extreme, if L is large, then h

the reaches a new fully developed state during each heated region and the answer limits to
ΔTq ′′+Δq ′′ = 9.229 K.
Maximum surface-to-mean temp. difference (K)

10

4
0.0001 0.001 0.01 0.1 1 10
Dimensionless period of heated region

Figure P5.5-4(f): Maximum surface-to-mean temperature difference as a function of the dimensionless heated
length.
Problem 6.2-6 (6-5 in text): Double-glazed window
The single-glazed window in problem 6.2-5 is replaced with a double-glazed window. Both
glass panes are 0.25 inch thick and the gap between the panes is 0.5 inch. The gap contains dry
air at atmospheric pressure. All other information is the same as in problem 6.2-5. Neglect heat
transfer by radiation.

a.) Repeat the calculations requested in parts (a) and (b) of problem 6.2-5.

The problem specifications are entered into EES.


$UnitSystem SI MASS RAD PA K J
$TABSTOPS 0.2 0.4 0.6 0.8 3.5 in

"Inputs"
T_air_in_F=70 [F] "outdoor air temperature"
L=6 [ft]*convert(ft,m) "height of window"
W=4 [ft]*convert(ft,m) "width of window"
T_air_out=converttemp(F,K,10) "outside air temperature"
T_air_in=converttemp(F,K,T_air_in_F) "inside air temperature"
t_g=0.25 [inch]*convert(inch,m) "thickness of glass"
k_g=1.4 [W/m-K] "conductivity of glass"
delta=0.5 [in]*convert(in,m) "width of air gap"

The conductive resistance of a glass pane is calculated.


R_cond=t_g/(k_g*L*W) "glass conduction resistance per pane"

The heat transfer coefficient on the outside surface of the outer glass is calculated first. This
calculation requires knowledge of the outside glass surface temperature which is not yet known.
A guess is entered and it will be removed later. Using this guess, the film temperature is
calculated and used to evaluate properties of the air. The Rayleigh and number is calculated and
used with the correlation described in section 6.2.2 (implemented in EES library procedure
FC_plate_vertical_ND) to calculate the Nusselt number for the outside surface of the window,
and thus the heat transfer coefficient for this surface.
"Outside air properties"
T_g2_out=(T_air_out+T_air_in)/2 "an initial guess for the outside surface temperature"
T_film_out=(T_g2_out+T_air_out)/2 "film temperature on the outside surface"
rho_air_out=density(Air,T=T_film_out,P=1 [atm]*convert(atm,Pa)) "density"
mu_air_out=viscosity(Air,T=T_film_out) "viscosity"
k_air_out=conductivity(Air,T=T_film_out) "conductivity"
c_air_out=cP(Air,T=T_film_out) "specific heat capacity"
beta_air_out=VolExpCoef(Air,T=T_film_out) "volumetric expansion coefficient"

"Outside air heat transfer coefficient"


alpha_air_out=k_air_out/(rho_air_out*c_air_out) "thermal diffusivity"
nu_air_out=mu_air_out/rho_air_out "kinematic viscosity"
Pr_air_out=nu_air_out/alpha_air_out "Prandtl number"
Ra_L_out=beta_air_out*g#*L^3*(T_g2_out-T_air_out)/(alpha_air_out*nu_air_out) "Raleigh number"
Call FC_plate_vertical_ND(Ra_L_out, Pr_air_out: Nusselt_out) "access vertical flat plate correlations"
h_bar_out=Nusselt_out*k_air_out/L "average heat transfer coefficient"

The free convection heat transfer coefficient on the indoor side of the window is calculated next.
Properties need to be estimated at the film temperature, which is the average of the indoor
temperature and the inside glass surface temperature, T_g1_in. A guess is provided for the
inside glass temperature, which will later be removed. The properties of air are evaluated at the
film temperature and used with a free convection relation in Section 6.2.2 (implemented in EES
library procedure FC_plate_vertical_ND) to evaluate the inside heat transfer coefficient.
"Inside air properties"
T_g1_in=T_air_in-5 [K] "an initial guess for the inside surface temperature"
T_film_in=(T_g1_in+T_air_out)/2 "film temperature on the inside surface"
rho_air_in=density(Air,T=T_film_in,P=1 [atm]*convert(atm,Pa)) "density"
mu_air_in=viscosity(Air,T=T_film_in) "viscosity"
k_air_in=conductivity(Air,T=T_film_in) "conductivity"
c_air_in=cP(Air,T=T_film_in) "specific heat capacity"
beta_air_in=VolExpCoef(Air,T=T_film_in) "volumetric expansion coefficient"

"Inside air heat transfer coefficient"


alpha_air_in=k_air_in/(rho_air_in*c_air_in) "thermal diffusivity"
nu_air_in=mu_air_in/rho_air_in "kinematic viscosity"
Pr_air_in=nu_air_in/alpha_air_in "Prandtl number"
Ra_L_in=abs(beta_air_in*g#*L^3*(T_air_in-T_g1_in)/(alpha_air_in*nu_air_in)) "Raleigh number"
Call FC_plate_vertical_ND(Ra_L_in, Pr_air_in: Nusselt_in) "access vertical flat plate correlations"
h_bar_in=Nusselt_in*k_air_in/L "average heat transfer coefficient"

Next it is necessary to determine the heat transfer coefficient of the air gap. The temperatures of
the glass surfaces forming the enclosure, T_g1_out and T_g2_in, are needed to evaluate the heat
transfer coefficient, but they are not yet know. Guess values will be again supplied and later
removed.
T_g1_out=(3*T_air_in+T_air_out)/4 "an initial guess for the outside temperature of the inner glass"
T_g2_in=(3*T_air_out+T_air_in)/4 "an initial guess for the insider temperature of the outer glass"

With the temperatures at hand, the average air temperature in the gap can be calculated and used
to determine air properties. These problems are used to calculate the Rayleigh number in the gap
which is used with the Tilted_Rect_Enclosure_ND procedure to evaluate the Nusselt number for
the gap, and thus the heat transfer coefficient.
"Heat transfer coefficient through the air gap"
T_gap=(T_g1_out+T_g2_in)/2 "air gap temperature "
rho_air_gap=density(Air,T=T_gap,P=1 [atm]*convert(atm,Pa)) "density"
mu_air_gap=viscosity(Air,T=T_gap) "viscosity"
k_air_gap=conductivity(Air,T=T_gap) "conductivity"
c_air_gap=cP(Air,T=T_gap) "specific heat capacity"
beta_air_gap=VolExpCoef(Air,T=T_gap) "volumetric expansion coefficient"
alpha_air_gap=k_air_gap/(rho_air_gap*c_air_gap) "thermal diffusivity"
nu_air_gap=mu_air_gap/rho_air_gap "kinematic viscosity"
HoverL=L/delta "height of window divided by gap thickness"
Tau=pi/2 [rad] "tilt"
Ra_gap=g#*Delta^3*beta_air_gap*(T_g1_out-T_g2_in)/(nu_air_gap*alpha_air_gap) "Rayleigh number
for gap"
Call Tilted_Rect_Enclosure_ND(Ra_gap, tau, HoverL : Nusselt_gap) "Nusselt number for heat transfer in
gap"
Nusselt_gap=h_gap*delta/k_air_gap "heat transfer coefficient for gap"

All of the heat transfer coefficients are known (based on the estimated temperatures shown in
yellow). Thermal resistances for convection on the inside surface of the window, convection in
the gap, and convection on the outside surface of the window can be evaluated.
R_gap=1/(h_gap*L*W) "resistance of gap"
R_conv_in=1/(h_bar_in*L*W) "internal convection resistance"
R_conv_out=1/(h_bar_out*L*W) "external convection resistance"

The total resistance to heat transfer is the sum of the three resistances above plus the conduction
resistance of two glazings. The heat transfer rate across the window is the difference between
the indoor and outdoor air temperatures divided by the total thermal resistance.
q_dot=(T_air_in-T_air_out)/(R_conv_in+2*R_cond+R_gap+R_conv_out) "heat transfer rate"

Four temperatures were assumed to get to this point. These temperatures can now be calculated
by recognizing that this is a steady-state problem so that the heat transfer rate through each
component in the window is the same. Before proceeding, solve and update the guess values.
Then remove or comment out the equations that provided guesses that are highlighted. Enter the
following relations.
T_g1_in=T_air_in-q_dot*R_conv_in "inside temperature of internal glass"
T_g1_in-T_g1_out=q_dot*R_cond "determines outside temperature of internal glass"
T_g2_out=T_air_out+q_dot*R_conv_out "outside temperature of external glass "
T_g2_in=T_g2_out+q_dot*R_cond "inside temperature of external glass"

Solving results in a heat transfer rate of 54.7 W.

The cost of this heat loss using the economic information from problem 6.2.5 can be calculated.
t_winter=90 [day]*convert(day,s) "time associated with a winter"
e_cost=0.12 [$/kW-hr]*convert($/kW-hr,$/J) "cost of electricity"
Cost=t_winter*q_dot*e_cost "total heating cost"
N_window=10 "number of windows"
TotalCost=2*N_window*Cost "total house heating cost"

b.) Summarize and explain the benefits of the double-glazed window.

The benefits are directly understood in terms of the costs. The cost with the double glazed
window calculates to be $14.18 per window or 283.6 for the building during the 90 days. With
single glazed windows, the costs from problem 6.2-5 were $30.6 per window and 612.6 for the
building during the same period.

It is noted that the conduction resistance of the glass is very small and it could have been
neglected with no loss in accuracy.
Problem 6.2-7 (6-6 in text): Double-glazed window with inert gas
You have seen an advertisement for argon-filled windows. These windows are similar in
construction to the window described in problem 6.2-6, except that argon, rather than air, is
contained in the gap. Neglect heat transfer by radiation.

a.) Repeat problem 6.2-6 assuming that the gap contains argon.

This solution procedure for this problem is exactly the same as for problem 6.2-6 except for the
calculating the heat transfer coefficient in the gap between the windows. Rather than air, a gas
identified by string variable F$ is used as the gas.
"Heat transfer coefficient through the air gap"
$ifnot ParametricTable
F$='Ar'
$endif

The average temperature of the gas in the gap is determined and used to evaluate its properties.
T_gap=(T_g1_out+T_g2_in)/2 "air gap temperature "
rho_gap=density(F$,T=T_gap,P=1 [atm]*convert(atm,Pa)) "density"
mu_gap=viscosity(F$,T=T_gap) "viscosity"
k_gap=conductivity(F$,T=T_gap) "conductivity"
c_gap=cP(F$,T=T_gap) "specific heat capacity"
beta_gap=VolExpCoef(F$,T=T_gap) "volumetric expansion coefficient"
alpha_gap=k_gap/(rho_gap*c_gap) "thermal diffusivity"
nu_gap=mu_gap/rho_gap "kinematic viscosity"

The free convection relation of Hollands et al. (section 6.2.6) is applied using the
Tilted_Rect_Enclosure procedure.
HoverL=L/delta "height of window divided by gap
thickness"
Tau=pi/2 [rad] "tilt"
Ra_gap=g#*Delta^3*beta_gap*(T_g1_out-T_g2_in)/(nu_gap*alpha_gap) "Rayleigh number for gap"
Call Tilted_Rect_Enclosure_ND(Ra_gap, tau, HoverL : Nusselt_gap) "Nusselt number for heat transfer in
gap"
Nusselt_gap=h_gap*delta/k_gap "heat transfer coefficient for gap"

A Parametric table is constructed with columns for F$ (the gas in the gap), the rate of heat
transfer through the window, and the heating cost per window. Also, to help answer parts b and
c, the thermal conductivity, Rayleigh number and heat transfer coefficient are shown in the table
for the difference gases. Solving produces the interesting results shown in the table.
b.) Are the claims that argon reduces heat loss valid? If so, why does this behavior occur?

The data in the table support the claim that argon-filled windows will reduce heat loss. The
Rayleigh number for argon is higher than for air, but it is still low enough such that limiting
behavior is observed so the Nusselt numbers are about the same. The thermal conductivity of
argon is lower than that of air. With the same Nusselt numbers, that results in a lower heat
transfer coefficient for argon

c.)Would nitrogen (which is cheaper) work as well? Why or why not? Can you suggest another
gas that would work better than argon?

From the data in the table it appears that carbon dioxide would also reduce convective losses in
the gap almost as well as argon.
Problem 6.2-8 (6-7 in text): Solar Collector Design
You are involved in a project to design a solar collector for heating air. Two competing designs
are shown in Figure P6.2-8.
2 2
200 W/m to 800 W/m
5°C, 5 m/s
glass cover plate
air at 25°C, 100 kPa
2
10 liter/s/m of collector 20 mm

black collector plate


2.5 m
(a)
2 2
5°C, 5 m/s 200 W/m to 800 W/m

glass cover plate

black collector plate 20 mm


air at 25°C, 100 kPa 20 mm
2
10 liter/s/m of collector

2.5 m
(b)
Figure P6.2-8: Air heating solar collector with (a) air flowing above the collector plate and (b) air flowing
below the collector plate.

Both designs employ a transparent glass cover plate and a thin metal opaque black collector plate
upon which solar radiation is completely absorbed. The glazing is standard safety glass with a
thickness of 6 mm. In the first design, shown in Figure P6.2-8(a), air is blown through the gap
between the cover and plate. In the second design, shown in Figure P6.2-8(b), the air flows in a
second gap that is below the collector plate and free convection occurs between the collector
plate and the glass cover plate. The collector is 1 meter wide (into the page) and 2.5 m long (in
the air flow direction) and oriented horizontally. In both designs, the gaps are 20 mm wide. Air
at 25°C, 100 kPa enters the flow passage at a flow rate of 10 liters/sec per square meter of
collector area (area exposed to solar radiation) in both cases. The outdoor temperature (above
the glass cover plate) is 5°C and there is a wind that may be represented as a forced convective
flow with a free stream velocity of 5 m/s in the flow direction. Calculate and plot the efficiency
of the two collector designs as a function of the solar radiation absorbed on the plate for values
between 200 and 800 W/m2. Assume that the insulation is adiabatic and neglect radiation in
these calculations.

Known problem specifications can be entered into EES. The absorbed solar radiation is set to
600 W/m2 but will later be varied in a parametric table.
$UnitSystem SI K Pa J rad
$tabstops 0.25 4 in

"known"
thk=6*convert(mm,m) "glass thickness"
V_dot=10 [l/s-m^2]*convert(l,m^3) "volumetric flow rate"
T_air_in=convertTemp(C,K,25 [C]) "temperature of entering air"
P_atm=100 [kPa]*convert(kPa,Pa) "atmospheric pressure"
rho_in=density(Air,T=T_air_in,P=P_atm) "density of entering air"
T_amb=convertTemp(C,K,5 [C]) "outdoor temperature"
Wind=5 [m/s] "wind velocity"
W=1 [m] "width of collector"
delta=20 [mm]*convert(mm,m) "gap width"
L=2.5 [m] "length of collector"
A=L*W "collector area"
m_dot=V_dot*rho_in*A "air mass flow rate"
$ifnot ParametricTable
S=600 [W/m^2] "absorbed solar radiation"
$endif
RelRough=0 "assumed smooth"
tau=0 [rad]

The thermal conductivity and thermal resistance for the glass are found. The Solid-Liquid
Property library provides the thermal conductivity.
k_glass=k_('Glass-soda lime', T=300 [K]) "thermal conductivity of glass at 300 [K]"
R_g=thk/(k_glass*A) "thermal resistance of the glass"

The geometry for case a is analyzed first. An average temperature is defined for determining
properties. The average temperature is the average of the entering and exiting air, but the outlet
temperature of the air is not yet known. A guess value is provided to get started. It will later be
removed.

The heat transfer coefficient in the gap is found using the DuctFlow procedure that implements
the correlation described in section 5.2.4.
T_air_out_a=T_air_in+20 [K] "guess for the air outlet temperature"
T_air_avg_a=(T_air_in+T_air_out_a)/2 "average air temperature"
call DuctFlow('air',T_air_avg_a,P_atm,m_dot, delta ,W,L,RelRough:h_T, h_H ,DELTAP, Nusselt_T, f, Re)
"heat transfer coefficient in flow gap"

The heat transfer coefficient to the plate is assumed to be the same as to the glazing. However,
the plate temperature is not known, so a guess will be provided temporarily to allow the rate of
heat transfer from the plate to the air to be calculated.
T_plate=T_air_in+50 [K] "guess for the plate temperature"
q_dot_air_a=A*h_H*(T_plate-T_air_avg_a) "heat transfer from plate to air"

Since the problem is steady state, the rate of heat transfer to the glazing is equal to the rate of
heat transfer by conduction through the glazing and the rate of heat loss by convection from the
outside surface of the glazing. To get this calculation started, we will assume a temperature for
the inside surface of the glass and later remove it.
T_g_in=T_air_in+10 [K] "guess for the inside glass surface temperature"
q_dot_loss_a=A*h_H*(T_air_avg_a-T_g_in) "heat transfer rate from air to glazing"
q_dot_loss_a =(T_g_in-T_g_out_a)/R_g "rate of conduction through glazing"

The heat transfer coefficient from the outside surface of the glass is found using an external flow
convection relation, External_Flow_Plate. The heat transfer resistance on the glass surface can
then be found.
Call External_Flow_Plate('Air', T_amb, T_g_out_a, P_atm, Wind, L: xx, h_wind_a, C_f, Nusselt_wind,
Re_wind)
R_wind_a=1/(h_wind_a*A) "resistance on top surface of glass"

Solve and update the guesses. Then comment out the guess for the inside glass surface
temperature and enter the relation for convection from the top surface of the glass.
q_dot_loss_a=(T_g_out_a-T_amb)/R_wind_a "rate of energy transfer from top of glass to
surroundings"

Solve and update the guesses. Energy balances on the collector and the airdetermines the plate
and air outlet temperatures. Comment out the guesses made for T_air_out and T_plate and enter
the energy balances.
m_dot*(enthalpy(Air,T=T_air_out_a)-enthalpy(Air,T=T_air_in))=q_dot_air_a-q_dot_loss_a "energy
balance on air"
S*A=q_dot_loss_a+m_dot*(enthalpy(Air,T=T_air_out_a)-enthalpy(Air,T=T_air_in)) "energy balance on
collector"

The collector efficiency is the ratio of the energy transferred to the air to the solar energy
absorbed on the plate. The efficiency is about 0.73.
eta_a=m_dot*(enthalpy(Air,T=T_air_out_a)-enthalpy(Air,T=T_air_in))/(S*A) "collector efficiency for
design a"

A similar analysis is conducted for the design shown in case b of the figure. In this case, the air
flows below the plate and there is a stagnant air gap above the plate. To start, the heat transfer
coefficient on the outside surface of the glass is calculated. A guess for the glass surface
temperature is provided and later removed.
T_g_out_b=T_amb+5 [K] "guess for the outside glass surface temperature"
Call External_Flow_Plate('Air', T_amb, T_g_out_b, P_atm, Wind, L: xxb, h_wind_b, C_fb, Nusselt_windb,
Re_wind_b)
R_wind_b=1/(h_wind_b*A) "resistance on top surface of glass"
q_dot_loss_b=(T_g_out_b-T_amb)/R_wind_b "rate of energy transfer from top of glass to
surroundings"
q_dot_loss_b=(T_g_in_b-T_g_out_b)/R_g "rate of energy transfer through glass"

The heat transfer coefficient in the enclosure is calculated using the Tilted_Rect_Enclosure
procedure in section 6.2.6. An estimate of the plate temperature is provided and later removed.
T_plate_b=T_air_in+50 [K] "guess for the plate temperature"
Call Tilted_Rect_Enclosure('Air', T_plate_b, T_g_in_b, P_atm, L, delta, tau : h_air_gap, Nusselt_gap, Ra)

Solve and update the guesses. The rate of heat transfer through the gap can be determines using
the heat transfer coefficient. Comment out the guess for T_g_out_b and then enter the heat
transfer relation
q_dot_loss_b=A*h_air_gap*(T_plate_b-T_g_in_b) "rate of energy transfer through gap"

Enter a guess for T_air_out_b so that the average air temperature can be calculated and used to
calculate the heat transfer rate to the air.
T_air_out_b=T_air_in+20 [K] "guess for the air outlet temperature"
T_air_avg_b=(T_air_in+T_air_out_b)/2 "average air temperature in gap"
q_dot_air_b=A*h_H*(T_plate-T_air_avg_b) "rate of energy transfer to air"
Solve and update the guess values. Comment out the guesses made for T_air_avg_b and
T_plate_b and enter the following energy balances. The efficiency can also be calculated.

The efficiency of the solar collector in figure b is 0.936 which is significantly better than that for
figure a. The insulation provided by the stagnant air gap is the reason for the improvement.
Problem 6.2-1 (6-1 in text): Convection from an L-shaped pipe
A pipe that is 3 m long has an outer diameter Dout = 0.1 m and is bent in the center to form an
“L” shape, as shown in Figure P6-1. One leg is vertical and the other leg is horizontal. The pipe
is made of thin-walled copper and saturated steam at atmospheric pressure is circulating through
the pipe. The pipe is in a large room and the air temperature far from the pipe is T∞ = 30°C at
atmospheric pressure. The conduction resistance associated with the pipe wall and the
convection resistance associated with steam can be neglected.

two-phase steam
ps = 1.01 bar
g
T∞ = 30°C

L = 1.5 m
Dout = 0.1 m

two-phase steam
L = 1.5 m ps = 1.01 bar
Figure P6-1: An "L"-shaped pipe.

The information provided in the problem statement can be summarized as follows:


$UnitSystem SI K Pa J
$TabStops 0.25 3.5 in
L=1.5 [m] "length of each leg"
d=0.1 [m] "outer diameter"
P_atm=101.3e3 [Pa] "ambient pressure"
T_s=T_sat('Steam',P=P_atm) "surface temperature of the pipe"
T_air=convertTemp(C,K,30 [C]) "ambient air temperature"

The external surface area of each leg is simply the product of the circumference and length of
each leg. It is necessary to calculate thermal properties and these should be evaluated at the
average air temperature. Neglecting conduction resistance through the tube wall and internal
convective resistance, the tube surface is the temperature of the steam, or 100°C
A_leg=pi*d*L "surface area of each leg"
T_avg=(T_air+T_s)/2 "average air temperature"
k_air=conductivity(Air,T=T_avg) "thermal conductivity"
rho=density(Air,T=T_avg,P=P_atm) "density"
mu=viscosity(Air,T=T_avg) "viscosity"
beta=VOLEXPCOEF(Air,T=T_avg) "coefficient of thermal expansion"
Pr=Prandtl(Air,T=T_avg) "Prandtl number"
g=g# "gravitational acceleration"

a) Determine the Grashof, Rayleigh, and Nusselt numbers and the corresponding average heat
transfer coefficient for the horizontal section of the pipe.

The Nusselt – Rayleigh correlations in section 6.2.4 need to be applied. The Grashof number is
evaluated using the tube diameter as the significant length for the horizontal tube. The Rayleigh
number is the product of the Grashof and Prandtl numbers. The FC_horizontal_cylinder_ND
returns the Nusselt number for the given Rayleigh and Prandtl numbers. The heat transfer
coefficient is found by applying the definition of the Nusselt number.
"Horizontal section"
Gr_H=rho^2*g*beta*(T_s-T_air)*d^3/mu^2 "Grashof number uses d as the significant length"
Ra_H=Gr_H*Pr "Rayleigh number"
Call FC_horizontal_cylinder_ND(Ra_H, Pr: Nusselt_H) "correlation from section 6.2.4"
Nusselt_H=h_bar_H*d/k_air "definition of Nusselt number"

b) Determine the Grashof, Rayleigh, and Nusselt numbers and the corresponding average heat
transfer coefficient for the vertical section of the pipe.

Again, the Nusselt – Rayleigh correlations in section 6.2.4 need to be applied but for the vertical
tube, the Grashof number is evaluated using the tube length as the significant lengthe. The
FC_vertical_cylinder_ND returns the Nusselt number for the given Rayleigh and Prandtl
numbers. The heat transfer coefficient is found by applying the definition of the Nusselt number.
"Vertical section"
Gr_V=rho^2*g*beta*(T_s-T_air)*L^3/mu^2 "Grashof number uses L as the significant length"
Ra_V=Gr_V*Pr "Rayleigh number"
LoverD=L/d "length over diameter ratio"
Call FC_vertical_cylinder_ND(Ra_V,Pr, LoverD:Nusselt_V) "correlation from section 6.2.4"
Nusselt_V=h_bar_V*L/k_air "definition of Nusselt number"

c) Calculate the total rate of heat transfer to the air.


The heat transfer rate is found by summing the heat transfer rates due to convection from the
horizontal and vertical legs.
q_dot=(h_bar_H+h_bar_V)*A_leg*(T_s-T_air) "rate of heat transfer from both legs"

The solution is summarized below:

Nusselt_H=18.87 [-] dimensionless groups for the horizontal leg


Gr_H=5.363E+06 [-]
Ra_H=3.855E+06
h_bar_H=5.368 [W/m^2-K] heat transfer coefficient for the horizontal leg|
Gr_V=1.810E+10 dimensionless groups for the vertical leg
Nusselt_V=245.7 [-]
Ra_V=1.301E+10
h_bar_V=4.659 [W/m^2-K] heat transfer coefficient for the vertical leg|
q_dot=330.6 [W] total heat transfer rate from both legs
Problem 6.2-2: Free convection from hot oil
Clean engine oil enters a L = 50 m long thin-walled pipe having an outer diameter Dout = 30 mm.
The mass flow rate is m = 0.25 kg/s and the inlet temperature is Tin = 150°C. The pipe is
suspended in a large room in which the air temperature is T∞ = 20°C. Estimate the heat transfer
rate from the oil to the room and the outlet oil temperature.

Specified problem information can be entered into EES.


$unitSystem SI J K Pa
$TabStops 0.25 3.5 in
"known information"
D=30 [mm]*convert(mm, m) "outer diameter of pipe"
m_dot=0.25 [kg/s] "oil flow rate"
L=50 [m] "pipe length"
T_inf=converttemp(C,K,20 [C]) "air temperature"
T_in=convertTemp(C,K,150 [C]) "inlet oil temperature"
P_air=101.3E3 [Pa] "atmospheric pressure"

It is necessary to determine the overall heat transfer coefficient between the oil and the air in the
room. This coefficient is a function of the thermal resistances to convection on the outside of the
pipe (due to free convection) and the inside of the pipe (due to forced convection). Conduction
through the pipe wall and fouling are neglected.

The heat transfer coefficient on the outside of the pipe is estimated first. It is necessary to
evaluate the air properties at an average temperature. Oil enters at 150°C, but we do not know
the exit temperature of the oil. To get started, a guess is entered for the exit oil temperature,
T_out. This guess will be removed later.
T_out=convertTemp(C,K,100 [C]) "guess - remove later"

We will use the average of the oil inlet and outlet temperatures as a estimate for the average
surface temperature, although the surface temperature will be somewhat lower due to convective
resistance. The average air temperature is take to be the average of the average surface
temperature and the free air temperature.
T_s_avg=(T_in+T_out)/2 "estimated surface temperature - no resistance in pipe wall"
T_avg_air=(T_s_avg+T_inf)/2 "average air temperature"

Air properties are evaluated at this average temperature.


Pr_air=Prandtl(Air,T=T_avg_air) "Prandtl number"
mu_air=Viscosity(Air,T=T_avg_air) "viscosity"
k_air=conductivity(Air,T=T_avg_air) "thermal conductivity"
c_air=cp(Air,T=T_avg_air) "specific heat capacity"
beta=VOLEXPCOEF (Air,T=T_avg_air) "coefficient of thermal expansion"
rho_air=density(Air,T=T_avg_air,P=P_air) "density"
alpha=k_air/(rho_air*c_air) "thermal diffusivity"
upsilon=mu_air/rho_air "dynamic viscosity"

The free convection heat transfer coefficient is found from Nusselt number using a correlation in
section 6.2.4 as a function of Rayleigh and Prandtl numbers. The heat transfer coefficient
determined form the correlation does not include radiative effects. A separate radiation heat
transfer coefficient is calculated, assuming that the pipe is a black surface, and the combined
convective-radiative heat transfer coefficient on the outside of the pipe is determined.
Ra=g#*D^3*beta*(T_s_avg-T_inf)/(alpha*upsilon) "Rayleigh number"
Call FC_horizontal_cylinder_ND(Ra, Pr_air: Nusselt_air) "Correlation from section 6.2.4"
h_bar_air=Nusselt_air*k_air/D "heat transfer coefficient - not including radiation"
h_rad=sigma#*(T_s_avg^2+T_inf^2)*(T_s_avg+T_inf) "radiation heat transfer coefficient"
h_bar_out=h_bar_air+h_rad "estimated heat transfer coefficient on outside surface of
pipe"

Next the heat transfer coefficient due to convection between the oil and the pipe wall is
determined. Oil properties are found at the average oil temperature that was used above to
approximate the average surface temperature. The properties of oil are provided in EES in the
Solid-Liquid property library (Function Info|Options menu).
T_oil_avg=T_s_avg "average oil temperature"
rho_oil=rho_('Engine oil-unused',T=T_oil_avg) "density of oil"
c_oil=c_('Engine oil-unused',T=T_oil_avg) "specific heat of oil"
k_oil=k_('Engine oil-unused',T=T_oil_avg) "thermal conductivity of oil"
Pr_oil=c_oil*mu_oil/k_oil "Prandtl number of oil"
mu_oil=mu_('Engine oil-unused',T=T_oil_avg) "viscosity of oil"

The properties are used to calculate the Reynolds number which is used in a forced internal flow
convection relation provided by the PipeFlow_N procedure. The heat transfer coefficient is
found from the Nusselt number determined by this procedure.
Re=rho_oil*vel*D/mu_oil "Reynolds number"
vel=m_dot/(A*rho_oil) "velocity of oil"
A=pi*D^2/4 "cross-sectional area of the pipe"
RelRough=0 "smooth tube assumed"
call PipeFlow_N(Re,Pr_oil,L/D,RelRough: Nusselt_T,Nusselt_H,f) "convection relation for internal
flow"
h_bar_in=Nusselt_T*k_oil/D "use lower estimate of heat transfer coefficient -
conservative"

The overall heat transfer coefficient – area product (conductance) can be found knowing the heat
transfer coefficients on both the inside and outside of the pipe. The surface area of the pipe is
known. Since the oil temperature varies exponentially as it travels through the pipe, the heat
transfer rate should be expressed in terms of the log-mean temperature difference as explained in
section 8.2.2. However, using the average temperature difference in place of the log-mean
temperature difference provides a reasonable result in this case.
1/UA=1/(h_bar_out*A_s)+1/(h_bar_in*A_s) "UA is the overall conductance"
A_s=pi*D*L "surface area of the pipe"
q_dot=UA*DELTAT_lm "heat transfer relation based on log-mean temperature
difference"
DELTAT_lm=(T_out-T_in)/ln((T_out-T_inf)/(T_in-T_inf)) "log-mean temperature difference"
q_dot_linear=UA*(T_oil_avg-T_inf) "heat transfer rate based on the average temperature
difference"

All of the above results are based on a guess for the oil outlet temperature, T_out. Solve and
then update the guesses. Comment out the specification for T_out as the following energy
balance on the oil determines its temperature.
q_dot=m_dot*c_oil*(T_in-T_out) "energy balance on the oil"
T_out_C=convertTemp(K,C,T_out)
The outlet oil temperature is 138.4 C and the heat transfer rate is 6,975 W.
PROBLEM 6.2-3 (6-2 in text): Self heating error
A resistance temperature detector (RTD) is inserted into a methane pipeline to measure the
temperature. The sensor is spherical with a diameter, D = 5.0 mm, and is exposed to methane at
Pf = 10 atm with a fluid temperature of Tf = 20°C. The resistance of the sensor is related to its
temperature; the resistance is measured by passing a known current through the resistor and
measuring the associated voltage drop. The current causes an ohmic dissipation of q = 5.0
milliW. You have been asked to estimate the associated self-heating error as a function of the
velocity of the methane in the pipe, Vf; focus on the very low velocity operation (e.g., 0 to 0.1
m/s) where self-heating might be large. The self-heating error is the amount that the temperature
sensor surface must rise relative to the surrounding fluid in order to transfer the heat associated
with ohmic dissipation. You may neglect radiation for this problem.

a.) Assume that only forced convection is important and prepare a plot showing the self-heating
error as a function of the methane velocity for velocities ranging from 0 to 0.1 m/s.

The inputs are entered in EES, a value of fluid velocity is used to start the calculation:
$UnitSystem SI MASS RAD PA K J
$TABSTOPS 0.2 0.4 0.6 0.8 3.5 in

T_f=converttemp(C,K,20) "fluid temperature"


P_f=10 [atm]*convert(atm,Pa) "fluid pressure"
V_f=1 [m/s] "fluid velocity"
d=5 [mm]*convert(mm,m) "diameter of sensor"
q_dot=5 [milliW]*convert(milliW,W) "ohmic dissipation"

The temperature of the sensor under forced convection conditions (Ts,fc) is assumed in order to
allow the film temperature to be computed:

Ts , fc + T f
T film, fc = (1)
2

The fluid properties under forced convection conditions (ρfc, kfc, μfc, cfc, αfc, νfc, and Prfc) are
obtained using EES’ built-in property routines evaluated at Tfilm,fc and Pf.
"forced convection"
T_s_fc=300 [K]
"guess for forced convection temperature"
T_film_fc=(T_s_fc+T_f)/2 "film temperature"
rho_fc=density(Methane,T=T_film_fc,P=P_f) "density"
k_fc=conductivity(Methane,T=T_film_fc,P=P_f) "conductivity"
mu_fc=viscosity(Methane,T=T_film_fc,P=P_f) "viscosity"
c_fc=cP(Methane,T=T_film_fc,P=P_f) "cP"
alpha_fc=k_fc/(rho_fc*c_fc) "thermal diffusivity"
nu_fc=mu_fc/rho_fc "kinematic viscosity"
Pr_fc=nu_fc/alpha_fc "Prandtl number"

The Reynolds number is computed:


ρ fc DV f
ReD = (2)
μ fc

and used to call the appropriate correlations for external forced convection heat transfer from a
sphere; these are contained in the procedure External_Flow_Sphere_ND. The correlation
provides the forced convection Nusselt number (Nufc) which is used to compute the forced
convection heat transfer coefficient (hfc):

Nu fc k fc
h fc = (3)
D
Re_D=rho_fc*D*V_f/mu_fc "Reynolds number"
Call External_Flow_Sphere_ND(Re_D,Pr_fc :C_d, Nusselt_fc) "access correlations"
h_fc=Nusselt_fc*k_fc/D “forced convection heat transfer coefficient"

The guess values for the calculation are updated (select Update Guess Values from the Calculate
menu) and the surface temperature assumption is commented out. The surface temperature
under forced convection is computed according to:

q
Ts , fc = T f + 3
(4)
⎛D⎞
h fc 4 π ⎜ ⎟
⎝2⎠

The self-heating error under forced convection conditions is calculated:

δ T fc = Ts , fc − T f (5)
{T_s_fc=300 [K] "guess for forced convection temperature"}
T_s_fc=T_f+q_dot/(4*pi*(D/2)^2*h_fc) "surface temperature"
deltaT_fc=(T_s_fc-T_f) "self heating error"

The value for Vf is commented out and a parametric table is created that includes δTfc and Vf.
The value of Vf is varied from very near zero (0.0001 m/s; the correlations will not work with Vf
= 0 m/s) to 30 m/s. Figure 1 illustrates the predicted self-heating error under forced convection
conditions as a function of the fluid velocity. Notice that the self-heating error becomes very
large as the velocity approaches 0 m/s because the heat transfer coefficient assuming only forced
flow becomes small; it is in this region that the impact of natural convection may become
significant.
Figure 1: Self-heating error as a function of the fluid velocity.

b.) Assume that only natural convection is important and determine the self-heating error in this
limit. Overlay this value on your plot from (a).

A surface temperature for the sensor under natural convection conditions (Ts,nc) is assumed and
used to compute the film temperature for natural convection conditions:

Ts ,nc + T f
T film, nc = (6)
2

The fluid properties under natural convection conditions (ρnc, knc, μnc, cnc, αnc, νnc, βnc, and Prnc)
are obtained using EES’ built-in property routines evaluated at Tfilm,nc and Pf.
"natural convection"
T_s_nc=300 [K]
"guess for natural convection temperature"
T_film_nc=(T_s_nc+T_f)/2 "film temperature"
rho_nc=density(Methane,T=T_film_nc,P=P_f) "density"
k_nc=conductivity(Methane,T=T_film_nc,P=P_f) "conductivity"
mu_nc=viscosity(Methane,T=T_film_nc,P=P_f) "viscosity"
c_nc=cP(Methane,T=T_film_nc,P=P_f) "cP"
beta_nc=VolExpCoef(Methane,T=T_film_nc,P=P_f) "volumetric expansion coefficient"
alpha_nc=k_nc/(rho_nc*c_nc) "thermal diffusivity"
nu_nc=mu_nc/rho_nc "kinematic viscosity"
Pr_nc=nu_nc/alpha_nc "Prandtl number"

The Grashof and Raleigh numbers are computed:

g β nc D 3 (Ts ,nc − T f )
GrD = (7)
υnc2
RaD = GrD Prnc (8)

and used to access the correlations for natural convection from a sphere (contained in the
procedure FC_sphere_ND) which returns the Nusselt number under natural convection
conditions (Nunc). The heat transfer coefficient under natural convection conditions is computed:

Nunc knc
hnc = (9)
D
Gr_D=g#*beta_nc*D^3*(T_s_nc-T_f)/(nu_nc^2) "Grashof number"
Ra_D=Gr_D*Pr_nc "Raleigh number"
Call FC_sphere_ND(Ra_D, Pr_nc: Nusselt_nc) "access correlations"
h_nc=Nusselt_nc*k_nc/D "natural convection heat transfer coefficient"

The guess values for the calculation are updated (select Update Guess Values from the Calculate
menu) and the surface temperature assumption for natural convection is commented out. The
surface temperature under natural convection conditions is computed according to:

q
Ts ,nc = T f + 3
(10)
⎛D⎞
hnc 4 π ⎜ ⎟
⎝2⎠

The self-heating error under natural convection conditions is calculated:

δ Tnc = Ts ,nc − T f (11)

T_s_nc=T_f+q_dot/(4*pi*(D/2)^2*h_nc) "surface temperature"


deltaT_nc=(T_s_nc-T_f) "self heating error"

The self-heating error is predicted to be δTnc =1.81 K; this value is overlaid on Fig. 1. Note that
the natural convection effect will limit that maximum self-heating error at low velocities to less
than 1.81 K.

c.) Prepare a plot that shows your prediction for the self heating error as a function of velocity
considering both natural convection and forced convection effects. Assume that the pipe is
mounted horizontally.

In order to account for the mixed conditions that occur at low velocity it is possible to estimate
the mixed heat transfer coefficient according to:

hmixed = ( h3fc + hnc3 )


1
3
(12)
Note that the natural and forced convection effects are added since the buoyancy induced flow is
not counter to the forced flow (the pipe is horizontal). The surface temperature and self-heating
error considering both natural and forced convection effects are:

q
Ts ,mixed = T f + 3
(13)
⎛D⎞
hmixed 4π ⎜ ⎟
⎝2⎠

δ Tmixed = Ts ,mixed − T f (14)

h_mixed=(h_nc^3+h_fc^3)^(1/3) "mixed mode heat transfer coefficient"


T_s_mixed=T_f+q_dot/(4*pi*(D/2)^2*h_mixed) "surface temperature"
deltaT_mixed=T_s_mixed-T_f "self-heating error"

The self-heating error considering both natural and forced convection effects is shown in Fig. 1;
notice that it limits to the natural convection result at low values of Vf and to the forced
convection result at higher values of Vf.
PROBLEM 6.2-4 (6-3 in text): Heat loss from a solar collector
Figure P6.2-4 illustrates a flat plate solar collector that is mounted at an angle of τ = 45 degrees
on the roof of a house. The collector is used to heat water; a series of tubes are soldered to the
back-side of a black plate. The collector plate is contained in a case with a glass cover.

g T∞ = 20°C
ho = 150 W/m -K
2

H=1m

q′′s = 800 W/m


2

back plate
insulation
Tp = 80°C transparent glass cover
roof
tubes

τ = 45 degree L = 2 cm
Figure P6.2-4: Flat plate solar collector

Assume that the solar collector is H = 1 m wide by W = 1 m long (into the page) and the distance
between the heated plate and the glass covering is L = 2 cm. The collector receives a solar flux
q s′′ =800 W/m2 and the collector plate can be assumed to absorb all of the solar energy. The
collected energy is either transferred to the water in the pipe (in which case the energy is used to
provide useful water heating) or lost due to heat transfer with the environment (either by
radiation, which will be neglected in this problem, or convection). The collector plate
temperature is Tp = 80°C and the ambient temperature is T∞ = 20°C. The heat transfer
coefficient on the external surface of the glass is forced convection (there is a slight breeze) and
equal to ho = 15 W/m2-K. The glass is thin and can be neglected from the standpoint of
providing any thermal resistance between the plate and ambient. Neglect radiative effects.

a.) Determine the rate of heat loss from plate due to convection; you may assume that the
insulation on the back of the tubes is perfect so no heat is conducted to the roof and that
radiation from the plate is negligible.

The inputs are entered in EES:


$UnitSystem SI MASS RAD PA K J
$TABSTOPS 0.2 0.4 0.6 0.8 3.5 in
"Inputs"
tau=45 [degree]*convert(degree,rad) "tilt angle"
Hc=1 [m] "collector width"
W=1 [m] "collector length"
L=2 [cm]*convert(cm,m) "plate to glass spacing"
q``_dot_s=800 [W/m^2] "solar flux"
T_p=converttemp(C,K,80) "plate temperature"
T_amb=converttemp(C,K,20) "ambient temperature"
h_o=15 [W/m^2-K] "external convection coefficient"
The thermal resistance between the plate and the ambient air consists of the series combination
of natural convection through a rectangular enclosure (Renc) and forced convection from the
external surface of the glass (Rconv,o); conduction through the glass is neglected as small in
comparison to these other resistances. The external convection coefficient is given and therefore
Rconv,o can be computed:

1
Rconv ,o = (1)
H W ho

R_conv_o=1/(h_o*Hc*W) "resistance to convection from the external surface"

In order to determine the natural convection heat transfer coefficient for the enclosure it is
necessary to determine the air properties and Rayleigh number that characterizes the problem.
The cold surface temperature (Ts, the temperature of the internal surface of the glass) is not
known apriori; therefore, a reasonable value is guessed and the film temperature is used to obtain
the air properties (ρ, μ, k, β, c, α, ν, and Pr) .

T_s=converttemp(C,K,30) "a 1st guess for the temperature of the internal surface of the glass"
T_film=(T_s+T_amb)/2 "film temperature"
rho=density(Air,T=T_film,P=1 [atm]*convert(atm,Pa)) "density"
mu=viscosity(Air,T=T_film) "viscosity"
k=conductivity(Air,T=T_film) "conductivity"
beta=VolExpCoef(Air,T=T_film) "volumetric thermal expansion coefficient"
c=cP(Air,T=T_film) "specific heat capacity"
alpha=k/(rho*c) "thermal diffusivity"
nu=mu/rho "kinematic viscosity"
Pr=nu/alpha "Prandtl number"

The Rayleigh number for an enclosure is based on the distance between the heated and cooled
surfaces (i.e., the plate and the glass):

g β L3 (Tp − Ts )
RaL = (2)
αυ

The Rayleigh number and tilt are used to call the procedure Tilted_Rect_Enclosure_ND which
returns the Nusselt number, NuL. The Nusselt number is used to compute the heat transfer
coefficient related to natural convection (henc):

henc L
NuL = (3)
k

Ra=beta*g#*L^3*(T_p-T_s)/(nu*alpha) "Raleigh number"


Call Tilted_Rect_Enclosure_ND(Ra, tau, Hc/L : Nusselt) "access correlations"
Nusselt=h_enc*L/k "compute the heat transfer coefficient"

The thermal resistance associated with the enclosure can be computed:


1
Renc = (4)
H W henc

The rate of convective heat loss is:

Tp − Tamb
qconv = (5)
Renc + Rconv ,o

R_enc=1/(h_enc*W*Hc) "thermal resistance associated with natural convection in the enclosure"


q_dot_conv=(T_p-T_amb)/(R_enc+R_conv_o) "rate of convective heat loss"

The solution is obtained and the guess values for the program are updated (Update Guess values
from the Calculate menu). The assumed surface temperature is commented out and the surface
temperature is computed:

Ts = Tp − Renc qconv (6)

{T_s=converttemp(C,K,30) "a 1st guess for the temperature of the internal surface of the glass"}
T_s=T_p-R_enc*q_dot_conv "calculated surface temperature"

The predicted rate of heat loss is 179 W.

b.) What is the efficiency of the solar collector, ηcollector, defined as the ratio of the energy
delivered to the water to the energy received from the sun?

The energy received from the sun is:

qs = q s′′W H (7)

The energy delivered to the water is:

qwater = qs − qconv (8)

and the efficiency is therefore:

qwater
ηcollector = (9)
qs
q_dot_s=q``_dot_s*W*Hc "rate of energy provided by solar flux"
q_dot_water=q_dot_s-q_dot_conv "rate of energy delivered to water"
eta_collector=q_dot_water/q_dot_s "collector efficiency"

The predicted collector efficiency is ηcollector = 0.776.


c.) Prepare a plot showing the collector efficiency as a function of the plate to glass spacing, L.
Explain the shape of the plot.

The value of L is commented out and a parametric table is generated that contains L and ηcollector.
(If convergence difficulties are encountered, try solving the table in reverse order. A check box
in the Solve Table dialog is provided for this purpose.) The results are plotted in Fig. 2 which
illustrates the collector efficiency as a function of the plate to collector spacing.
0.9

0.8
Collector efficiency

0.7

0.6

0.5

0.4
0 0.5 1 1.5 2 2.5 3 3.5 4 4.5 5
Plate to glass spacing (cm)
Figure 2: Collector efficiency as a function plate to glass spacing

The initial rise in collector efficiency with L is related to the increased conduction resistance of
the stagnant air gap. Around L = 0.75 cm, the viscous force is overcome by the buoyancy force
and therefore convective cells are formed so that the convective heat loss actually increases with
L leading to a reduction in collector efficiency.
PROBLEM 6.2-5 (6-4 in text): Heat loss from a window
Figure P6.2-5 illustrates a single-paned glass window that is L = 6 ft high and W = 4 ft wide; the
glass is thg = 0.25 inch thick and has conductivity kg = 1.4 W/m-K.

thg = 0.25 inch


Tair ,in = 70°F L = 6 ft

kg = 1.4 W/m-K Tair ,out = 10°F

Figure P6.2-5: Single-paned glass window.

On a typical winter day, the outdoor temperature is Tair,out =10°F and you keep the indoor
temperature at Tair,in = 70°F.

a.) On a still winter day, estimate the rate of heat loss from the window.

The inputs are entered in EES:


$UnitSystem SI MASS RAD PA K J
$TABSTOPS 0.2 0.4 0.6 0.8 3.5 in

"Inputs"
L=6 [ft]*convert(ft,m) "height of window"
W=4 [ft]*convert(ft,m) "width of window"
T_air_out=converttemp(F,K,10) "outside air temperature"
T_air_in=converttemp(F,K,70) "inside air temperature"
t_g=0.25 [inch]*convert(inch,m) "thickness of glass"
k_g=1.4 [W/m-K] "conductivity of glass"

The thermal resistance network that connects the outdoor air to the indoor air is shown in Fig. 2
and includes a convection resistance between the inner surface of the window and the indoor air
(Rconv,in), a conduction resistance across the gas (Rcond), and a convection resistance between the
outer surface of the window and the outdoor air (Rconv,out).

Figure 2: Resistance network.

The conduction resistance is:

tg
Rcond = (1)
k g LW
R_cond=t_g/(k_g*L*W) "glass conduction resistance"

The convection resistances are:

1
Rconv ,in = (2)
hin LW

1
Rconv ,out = (3)
hout LW

where hin and hout are the average heat transfer coefficients on the internal and external surfaces,
respectively. These are obtained by solving the corresponding external, natural convection
problem for a vertical flat plate. The properties of the external air (ρair,out, kair,out, μair,out, cair,out,
βair,out) are obtained at the film temperature between the external surface and the outside air
(Tfilm,out):

Tair ,out + Tg ,out


T film ,out = (4)
2

where Tg,out is the temperature on the external surface of the glass (see Fig. 2) which is not
known apriori. A reasonable guess is used to start the problem; this value will be adjusted to
finish the problem. The properties of air within the house (ρair,in, kair,in, μair,in, cair,in, βair,in) are
obtained in the same way.

"Outside air properties"


T_g_out=(T_air_out+T_air_in)/2
"an initial guess for the outside surface temperature"
T_film_out=(T_g_out+T_air_out)/2 "film temperature on the outside surface"
rho_air_out=density(Air,T=T_film_out,P=1 [atm]*convert(atm,Pa)) "density"
mu_air_out=viscosity(Air,T=T_film_out) "viscosity"
k_air_out=conductivity(Air,T=T_film_out) "conductivity"
c_air_out=cP(Air,T=T_film_out) "specific heat capacity"
beta_air_out=VolExpCoef(Air,T=T_film_out) "volumetric expansion coefficient"

"Inside air properties"


T_g_in=(T_air_out+T_air_in)/2 "an initial guess for the inside surface temperature"
T_film_in=(T_g_in+T_air_out)/2 "film temperature on the inside surface"
rho_air_in=density(Air,T=T_film_in,P=1 [atm]*convert(atm,Pa)) "density"
mu_air_in=viscosity(Air,T=T_film_in) "viscosity"
k_air_in=conductivity(Air,T=T_film_in) "conductivity"
c_air_in=cP(Air,T=T_film_in) "specific heat capacity"
beta_air_in=VolExpCoef(Air,T=T_film_in) "volumetric expansion coefficient"

The Raleigh number for the external surface (RaL,out) is defined as:

β air ,out g L3 (Tg ,out − Tair ,out )


RaL ,out = (5)
α air ,out ν air ,out
where αair,out and νair,out are the thermal diffusivity and kinematic viscosity of the outside air and
g is the acceleration of gravity. The average Nusselt number for the outside surface of the glass
( Nu L ,out ) is obtained by accessing the EES function FC_plate_vertical_ND with the calculated
Raleigh number and the Prandtl number of the outside air (Prair,out). The Nusselt number is used
to compute the average heat transfer coefficient on the external surface:

Nu L ,out kair ,out


hout = (6)
L

A similar process is used to determine hin ; note that the absolute value of the surface to ambient
temperature difference is used.

"Outside air heat transfer coefficient"


alpha_air_out=k_air_out/(rho_air_out*c_air_out) "thermal diffusivity"
nu_air_out=mu_air_out/rho_air_out "kinematic viscosity"
Pr_air_out=nu_air_out/alpha_air_out "Prandtl number"
Ra_L_out=beta_air_out*g#*L^3*(T_g_out-T_air_out)/(alpha_air_out*nu_air_out)
"Raleigh number"
Call FC_plate_vertical_ND(Ra_L_out, Pr_air_out: Nusselt_out) "access vertical flat plate correlations"
h_bar_out=Nusselt_out*k_air_out/L "average heat transfer coefficient"

"Inside air heat transfer coefficient"


alpha_air_in=k_air_in/(rho_air_in*c_air_in) "thermal diffusivity"
nu_air_in=mu_air_in/rho_air_in "kinematic viscosity"
Pr_air_in=nu_air_in/alpha_air_in "Prandtl number"
Ra_L_in=beta_air_in*g#*L^3*abs(T_g_in-T_air_in)/(alpha_air_in*nu_air_in)
"Raleigh number"
Call FC_plate_vertical_ND(Ra_L_in, Pr_air_in: Nusselt_in) "access vertical flat plate correlations"
h_bar_in=Nusselt_in*k_air_in/L "average heat transfer coefficient"

The resistances to internal and external convection may be computed using Eqs. (2) and (3). The
total heat transfer through the window is therefore:

Tair ,in − Tair ,out


q = (7)
Rconv ,in + Rcond + Rconv ,out

R_conv_in=1/(h_bar_in*L*W) "internal convection resistance"


R_conv_out=1/(h_bar_out*L*W) "external convection resistance"
q_dot=(T_air_in-T_air_out)/(R_conv_in+R_cond+R_conv_out) "heat transfer rate"

The guess values used by EES to start the solution process should be updated (select Update
Guesses from the Calculate menu) and the surface temperatures of the glass calculated:

Tg ,in = Tair ,in − q Rconv ,in (8)

Tg ,out = Tair ,out + q Rconv ,out (9)


and the original guesses are removed:
{T_g_out=(T_air_out+T_air_in)/2 "an initial guess for the outside surface temperature"}
{T_g_in=(T_air_out+T_air_in)/2 "an initial guess for the inside surface temperature"}
T_g_in=T_air_in-q_dot*R_conv_in "internal glass temperature"
T_g_out=T_air_out+q_dot*R_conv_out "external glass temperature"

The result is q = 118 W.

b.) Winter lasts twinter= 90 days and you are heating with electrical resistance heaters. Electricity
costs ecost = $0.12/kW-hr. How much does the heat loss through the window cost you over
the course of 1 winter?

The money required to heat the house is the product of the rate of heat loss, the time associated
with winter, and the cost of the energy (electricity).

Cost = q twinter ecost (10)

t_winter=90 [day]*convert(day,s) "time associated with a winter"


e_cost=0.12 [$/kW-hr]*convert($/kW-hr,$/J) "cost of electricity"
Cost=t_winter*q_dot*e_cost "total heating cost"

The cost associated with 1 window is $30.63 for a winter.

c.) About 40% of your home's heat loss will occur through its windows; more if you have single
paned windows like the one shown in Fig. 1. Assume that 50% of your heat loss in your
house is through your windows and that you have Nwindow = 10 single paned windows in your
house. Prepare a plot showing the cost of heating your house as a function of the thermostat
set point (i.e., the indoor air temperature).

The total cost is twice the product of the number of windows and the cost per window.

Totalcost = 2 N windows Cost (11)

N_window=10 "number of windows"


TotalCost=2*N_window*Cost "total house heating cost"

The indoor air temperature is commented out and varied in a parametric table; the total cost as a
function of outdoor air temperature is shown in Fig. 3.
Figure 3: Total heating cost as a function of the indoor air temperature.
Problem 6.3-2 (6.9 in text): Self-similar Solution for a Vertical Plate
A self-similar solution can be obtained for the free convection problem where a heated vertical
plate has a surface temperature (Ts) that varies with position according to: Ts − T∞ = A x n where x
is measured from the bottom of the plate.
a.) Transform the governing partial differential equations for momentum conservation in the
(vertical) x-direction and thermal energy conservation into ordinary differential equations for
f and θ .

The plate temperature is given by:

Ts − T∞ = A x n (1)

The governing equations are presented in Section 6.1.2. The continuity equation is:

∂u ∂v
+ =0 (2)
∂x ∂y

The x-momentum equation in the boundary layer, including the buoyancy term, which is
represented using the Boussinesq approximation, is:

∂u ∂u ∂ 2u
u +v = g β (T − T∞ ) + υ 2 (3)
∂x ∂y ∂y

Note that x is in the vertical direction as shown in Figure 6-20. The thermal energy equation is:

⎡ ∂T ∂T ⎤ ∂ 2T
ρ c ⎢u +v ⎥ = k 2 (4)
⎣ ∂x ∂y ⎦ ∂y

The similar parameter, is defined in Eq. (6-122)


1
y ⎡ Gr ⎤ 4
η= ⎢ x⎥ (5)
x⎣ 4 ⎦

It is anticipated that the dimensionless velocity and temperature difference:

u
u = (6)
uchar ,nc

T − T∞
θ = (7)
Ts − T∞
at any position x will collapse when expressed in terms of η, as shown in Figure 6-20:

u = u ( x, y ) = u (η ) (8)

θ = θ ( x, y ) = θ (η ) (9)

where the local characteristic velocity, uchar,nc, is:

uchar , nc = g x β (Ts − T∞ ) (10)

The stream function is defined so that the continuity equation, Eq.(2), is automatically satisfied:

⎛∂Ψ ⎞
u =⎜ ⎟ (11)
⎝ ∂ y ⎠x

⎛∂Ψ ⎞
v = −⎜ ⎟ (12)
⎝ ∂ x ⎠y

The stream function is related to the volumetric flow between the surface of the plate and any
position y according to:

V = W Ψ (13)

where W is the width of the plate (into the page). The stream function is related to the similarity
parameter by Eq. (6-32).
1
⎛ Gr ⎞ 4
Ψ = 4υ ⎜ x ⎟ f (η ) (14)
⎝ 4 ⎠

Equations (3) and (4) must be transformed from functions of x and y to a function of η. The
1
mathematical manipulations are facilitated by expressing the parameter Grx 4 / 4 , which appears
in the equations for both η and Ψ, in terms of x rather than Grx:

1 1 1
⎛ Grx ⎞ 4
⎛ g β (Ts − T∞ ) x 3 ⎞ 4
⎛ g β A x n x3 ⎞ 4
1 ( 3+ n )
⎜ ⎟ =⎜ ⎟ =⎜ ⎟ = B 4x 4
(15)
⎝ 4 ⎠ ⎝ 4υ 2 ⎠ ⎝ 4υ 2 ⎠

Substituting Eq.(15) into Eqs. (5) and (14) allows the similarity variables to be written as:

1 ( n −1)
η = yB 4 x 4
(16)
1 ( 3+ n )
Ψ = 4υ B 4
x 4
f (η ) (17)

where
gβ A
B= (18)
4υ 2

The x-velocity is expressed in terms of the similarity variables by substituting Eq. (17) into Eq.
(11):

⎛ ∂Ψ ⎞ ∂ ⎡ 1 ( 3+ n ) ⎤ 1 ( 3+ n ) ∂
u =⎜ ⎟ = ⎢ 4υ B x
4 4
f (η ) ⎥ = 4υ B 4 x 4 ⎡ f (η ) ⎤⎦ x (19)
⎝ ∂y ⎠ x ∂y ⎣ ⎦x ∂y ⎣


df ⎛ ∂η ⎞
⎜ ⎟
dη ⎝ ∂y ⎠ x

The partial derivative of η, Eq. (16) with respect to y at constant x is:

⎛ ∂η ⎞ 1 ( n −1)
⎟ =B x (20)
4 4

⎝ ∂y ⎠ x

Substituting Eq. (20) into Eq. (19) leads to:

1 ( 3+ n ) 1 ( n −1) df 1 ( n +1) df
u = 4υ B 4
x 4
B 4
x 4
= 4υ B 2 x 2 (21)
dη dη

The y-velocity is obtained by substituting Eq. (17) into Eq. (12):

∂ ⎡ ( 3+ n ) ⎤
f (η ( x, y ) ) ⎥
1
v=− ⎢ 4 υ B 4
x 4
(22)
∂x⎣ ⎦y

The similarity parameter, η is a function of x and y and, as a result, f is itself a function of x.


Applicationof the chain rule to Eq. (22) results in:

∂ ⎡ 1 ( 3+ n ) ⎤ 1 ( 3+ n ) df ⎛ ∂η ⎞
v=−f 4 υ B 4
x 4
− 4 υ B 4
x 4
⎜ ⎟ (23)
∂ x ⎢⎣ ⎥
⎦y dη ⎝ ∂x ⎠ y

Substituting Eq. (16) into Eq. (23) leads to:

∂ ⎡ 1 ( 3+ n ) ⎤ 1 ( 3+ n ) df ∂ ⎡ 1 ( n −1) ⎤
v=−f ⎢ 4 υ B 4 x 4 ⎥ − 4υ B 4 x 4
⎢ yB 4 x 4 ⎥ (24)
∂x⎣ ⎦y dη ∂x ⎣ ⎦

or
1 ( n −1) 1 ( n −1) df
v = − f ( 3 + n )υ B 4
x 4
− ( n − 1)υ B 2
x 2
y (25)

In addition to u and v, we will need the partial derivatives of u (see Eq. (3)). The partial
derivative of u with respect to x is:

∂u ∂ ⎡ 1 ( n +1) df ⎤ df ∂ ⎡ 1 ( n +1) ⎤ 1 ( n +1) d 2 f ⎛ ∂η ⎞


= ⎢ 4υ B 2 x 2 = 4 υ B 2
x 2
+ 4 υ B 2
x 2
⎜ ⎟ (26)
∂x ∂x ⎣ dη ⎥⎦ y dη ∂x ⎢⎣ ⎥
⎦y dη 2 ⎝ ∂x ⎠ y

Substituting Eq. (16) into Eq. (26) leads to:

∂u df ∂ ⎡ 1 ( n +1) ⎤ 1 ( n +1) d 2 f ∂ ⎡ 1 ( n −1) ⎤


= 4 υ B 2
x 2
+ 4 υ B 2
x 2
y B 4
x 4
(27)
∂x dη ∂x ⎢⎣ ⎥
⎦y dη 2
∂x ⎢


or

∂u df 1 ( n −1) 1 ( n +1) d 2 f 1 ( n −5)


= 2 ( n + 1)υ B 2 x 2 + υ B 2 x 2 y B 4
( n − 1) x 4
(28)
∂x dη dη 2

which can be rewritten as:

∂u df 1 ( n −1) 3 3 ( n −1)
d2 f
= 2 ( n + 1)υ B 2 x 2 + υ B 4 x 4
y ( n − 1) (29)
∂x dη dη 2

The partial derivative of u with respect to y is:

⎛ ∂u ⎞ ∂ ⎡ 1 ( n +1) df ⎤ 1 ( n +1) d 2 f ⎛ ∂η ⎞
⎜ ⎟ = ⎢ 4 υ B 2
x 2
⎥ = 4 υ B 2
x 2
⎜ ⎟ (30)
∂y
⎝ ⎠x ∂y ⎣ dη ⎦ dη 2 ⎝ ∂y ⎠ x

Substituting Eq. (16) into Eq. (30) leads to:

⎛ ∂u ⎞ ∂ ⎡ 1 ( n +1) df ⎤ 1 ( n +1) d 2 f ⎛ ∂η ⎞
⎜ ⎟ = ⎢ 4υ B x ⎥ = 4υ B x (31)
2 2 2 2
⎜ ⎟
∂y
⎝ ⎠x ∂y ⎣ dη ⎦ dη 2 ⎝ ∂y ⎠ x

⎛ ∂u ⎞ 1 ( n +1) d 2 f ∂ ⎡ 1 ( n −1) ⎤
⎜ ⎟ = 4 υ B 2
x 2
⎢ y B 4
x 4
⎥ (32)
⎝ ∂y ⎠ x dη ∂y ⎣
2

or

⎛ ∂u ⎞ 1 ( n +1) d 2 f 1 ( n −1)
⎜ ⎟ = 4 υ B 2
x 2
B 4
x 4
(33)
∂y
⎝ ⎠x dη 2
which can be rewritten as:

⎛ ∂u ⎞ 3 ( 3 n +1) d 2 f
⎜ ⎟ = 4 υ B 4
x 4
(34)
∂y
⎝ ⎠x dη 2

Finally, the second derivative of u with respect to y is:

∂ 2u ∂ ⎛ 3 ( 3 n +1) d 2 f ⎞ 3 ( 3 n +1) d 3 f ⎛ ∂η ⎞
= ⎜ 4 υ B 4
x 4
⎟ = 4 υ B 4
x 4
⎜ ⎟ (35)
∂y 2 ∂y ⎝ dη 2 ⎠ dη 3 ⎝ ∂y ⎠ x

Substituting Eq. (16) into Eq. (35) leads to:

∂ 2u 3 ( 3 n +1) d 3 f ∂ ⎡ 1 ( n −1) ⎤
= 4 υ B 4
x 4
y B 4
x 4
(36)
∂y 2 dη 3 ∂y ⎢⎣ ⎥

or

∂ 2u 3 ( 3 n +1) d 3 f 1 ( n −1)
= 4 υ B 4
x 4
B 4
x 4
(37)
∂y 2 dη 3

which can be rewritten as:

∂ 2u 3
n d f
= 4 υ B x (38)
∂y 2 dη 3

Substituting Eqs. (30), (34), and (38) into the x-momentum equation, Eq. (3), leads to:

∂u ∂u ∂ 2u
u +v = g β (T − T∞ ) + υ 2 (39)
∂x ∂y ∂y
1 ( n +1) df ⎡ df 1 ( n −1) 3 3 ( n −1)
d2 f ⎤
4υ B 2 x 2 ⎢ 2 ( n + 1) υ B 2
x 2
+ υ B 4
x 4
y ( n − 1) ⎥
dη ⎣ dη dη 2



u ∂u
∂x

⎡ 1 ( n −1) 1 ( n −1) df ⎤ 3 ( 3 n +1) d 2 f


+ ⎢ − f ( 3 + n )υ B 4 x 4 − ( n − 1)υ B 2 x 2 y ⎥ 4υ B 4 x 4
= (40)
⎣
⎦  dη dη 2

v ∂u
∂y

d3 f
g β (Ts − T∞ ) θ + υ 4υ B x n

dη 3
(T −T∞ ) 

∂ 2u
∂y 2

Expanding the terms and substituting the definition of B, Eq. (18), into the buoyancy term leads
to:

1 ( n +1) df df 1 ( n −1) 1 ( n +1) df 3 3 ( n −1)


d2 f
4υ B 2
x 2
2 ( n + 1)υ B 2 x 2 + 4υ B 2 x 2 υB 4 x 4
y ( n − 1)
dη dη dη dη 2
1 ( n −1) 3 ( 3 n +1) d2 f 1 ( n −1) df 3 ( 3 n +1) d 2 f
− f ( 3 + n )υ B 4
x 4
4υ B 4
x 4
− ( n − 1) υ B 2
x 2
y 4 υ B 4
x 4
= (41)
dη 2 dη dη 2
d3 f
4υ 2 B x n θ + 4υ 2 B x n
dη 3

Dividing Eq. (41)through by 4ν 2 B leads to:

2
⎛ df ⎞ d2 f d3 f
⎟ 2 ( n + 1) − f ( 3 + n ) 2 = θ + 3
 (42)

⎝ dη ⎠ dη dη

The differential equation for f is coupled to the differential equation for θ , which will be derived
from the thermal energy equation, Eq. (4). The first derivative of θ with respect to x is obtained
using the chain rule, recognizing that θ is a function only of η, which is a function of x and y:

∂  dθ ⎛ ∂ η ⎞
⎡⎣θ (η ( x, y ) ) ⎦⎤ = ⎜ ⎟ (43)
∂x dη ⎝ ∂ x ⎠ y

Substituting Eq. (16) into Eq. (43) leads to:

∂  dθ ∂ ⎡ 1 4 ( n −1) 4 ⎤
⎡⎣θ (η ( x, y ) ) ⎤⎦ = yB x (44)
∂x dη ∂ x ⎢⎣ ⎥

which can be written as:

∂θ dθ ( n − 1) 1 ( n −5)


= yB 4 x 4 (45)
∂x dη 4

The first derivative of θ with respect to y is obtained in a similar manner:

∂θ dθ ∂ ⎡ 1 ( n −1) ⎤


= y B 4
x 4
(46)
∂y dη ∂ y ⎢⎣ ⎥

or

∂θ dθ 1 4 ( n −1) 4


= B x (47)
∂y dη

The second derivative of θ with respect to y is:

∂ 2θ ∂ ⎛ ∂ θ ⎞ ∂ ⎡ ∂ θ ⎤ ⎛ ∂ η ⎞
= ⎜ ⎟ = ⎢ ⎥⎜ ⎟ (48)
∂y 2 ∂y ⎝ ∂ y ⎠ x ∂η ⎣ ∂ y ⎦ ⎝ ∂ y ⎠ x

Substituting Eq. (47) into Eq. (48) leads to:

∂ 2θ ∂ ⎡ dθ 1 4 ( n −1) 4 ⎤ ⎛ ∂ η ⎞


= ⎢ B x ⎥⎜ ⎟ (49)
∂y 2 ∂η ⎣ dη ⎦ ⎝ ∂ y ⎠x

or

∂ 2θ d 2θ 1 4 ( n −1) 4 ∂ ⎡ 1 4 ( n −1) 4 ⎤


= B x yB x (50)
∂y 2 dη 2 ∂y ⎢⎣ ⎥

∂ 2θ d 2θ 1 2 ( n −1) 2


= B x (51)
∂y 2 dη 2

Subsituting the definition of θ , Eq. (7), into the thermal energy equation, Eq. (4), leads to:

∂θ ∂θ ∂ 2θ


u +v =α 2 (52)
∂x ∂y ∂y

Substituting Eqs. (21), (45), (25), (47), and (51) into Eq. (52) leads to:
1 ( n +1) df dθ ( n − 1) 1 ( n −5 )
4υ B 2 x 2 yB 4 x 4
+
dη dη 4



u ∂θ
∂x

⎡ 1 ( n −1) 1 ( n −1) df ⎤ dθ 1 ( n −1)


⎢ − f ( 3 + n ) υ B 4
x 4
− ( n − 1) υ B 2
x 2
y⎥ B 4x 4 = (53)
⎣
⎦  dη dη

v ∂θ
∂y

d 2θ 1 2 ( n −1) 2
α B x
dη 2


∂ 2θ
α
∂y 2

Expanding the terms in Eq. (53)leads to:

1 df dθ
( n +1) 1 ( n −5) 1 ( n −1) dθ 1 ( n −1)
υB 2x 2
( n − 1) y B 4 x 4 − f ( 3 + n )υ B 4 x 4 B 4 x 4
dη dη dη
(54)
1 ( n −1) df dθ 1 4 ( n −1) 4 d 2θ 1 2 ( n −1) 2
− ( n − 1)υ B 2 x 2 y B x =α B x
dη dη dη 2

which leads to the ordinary differential equation for θ :

dθ d 2θ
− f ( 3 + n ) Pr = (55)
dη dη 2

b.) Transform the boundary conditions for u, v, and T into boundary conditions for f and θ .

The boundary conditions include no-slip at the wall:

u y =0 = 0 (56)

v y =0 = 0 (57)

The plate temperature is specified:

Ty =0 = Ts (58)

As y becomes large, the fluid becomes stagnant:

u y →∞ = 0 (59)
and the ambient temperature is recovered:

Ty →∞ = T∞ (60)

Equation (21) is substituted into Eq. (56):

1 ( n +1) df
u y = 0 = 4υ B 2
x 2
=0
dη η =0

or
df
=0 (61)
dη η =0

Equation (21) is substituted into Eq. (59):

1 ( n +1) df
u y →∞ = 4υ B 2
x 2
=0
dη η →∞

or

df
=0 (62)
dη η →∞

Equation (25) is substituted into Eq. (57):

1 ( n −1) 1 ( n −1) df
v y =0 = − fη =0 ( 3 + n )υ B 4
x 4
− ( n − 1)υ B 2
x 2
0=0 (63)
dη η =0


=0

which can only be true if:

fη =0 = 0 (64)

Equation (7) is substituted into Eqs. (58) and (60):

θη =0 = 1 (65)

θη →∞ = 0 (66)
Equations (42) and (55) together with Eqs. (61), (62), (64), (65)and (66) represent coupled third
order and second order ordinary differential equations with the five required boundary
conditions.

c.) Develop a numerical solution for this problem.

The numerical solution of the problem is more complicated than it would appear and therefore
df d 2 f  dθ
also quite interesting. The five state variables are f, , , θ , and . The state equations
dη dη 2

provide the rate of change of these state variables:

d df
[f ]= (67)
dη dη

d ⎡ df ⎤ d 2 f
dη ⎢ dη ⎥ = dη 2 (68)
⎣ ⎦

2
d ⎡ d 2 f ⎤ ⎛ df ⎞ d2 f 
=
⎢ dη 2 ⎥ ⎜ dη ⎟ 2 ( n + 1) − f ( 3 + n ) −θ (69)
dη ⎣ ⎦ ⎝ ⎠ dη 2

d  dθ
⎡θ ⎤ = (70)
dη ⎣ ⎦ dη

d ⎡ dθ ⎤ dθ
⎢ ⎥ = − f ( 3 + n ) Pr (71)
dη ⎣ dη ⎦ dη

Three of the boundary conditions for the state variables are specified at η = 0 while the
remaining two are specified at η → ∞; therefore, the shooting method is more complex than it
was in Section 4.4 because two boundary conditions must be assumed and then adjusted at η = 0.
This problem can be solved using either EES or MATLAB. Both solutions will be provided.
The EES solution is easy to implement, but it requires more computational time than the
MATLAB solution.

EES Solution
Equations (67) - (71) are implemented with the following EES equations:
f=f_o+integral(df\deta, eta, eta_lo, eta_hi) "integral of df\deta to find f"

df\deta=df\deta_o+integral(d2f\deta2,eta,eta_lo, eta_hi) "integral of d2f\deta2 to find df\deta"

d2f\deta2=d2f\deta2_o+integral(d3f\deta3,eta,eta_lo, eta_hi) "integral of d3f\deta3 to find df2\deta2"

d3f\deta3=df\deta^2*2*(n+1)-f*(3+n)*d2f\deta2-theta "Eq. 42 in solution"


theta=theta_o+integral(dtheta\deta,eta,eta_lo, eta_hi) "integral of dtheta\deta to find theta"

d2theta\deta2=-f*(3+n)*Pr*dtheta\deta "Eq. 55 in solution"

dtheta\deta=dtheta\deta_o+integral(d2theta\deta2,eta,eta_lo,eta_hi) "integral of d2theta\deta2 to find


dtheta\deta"

The boundary conditions at η=0 are:


"Boundary conditions at eta=0"
f_o=0
theta_o=1
df\deta_o=0

The two other boundary conditions (Eqs. (62) and (66)) are at η=infinity. The optimization
capability in EES will be applied to ensure that these boundary conditions are met. EES cannot
integrate to infinity. However, a value of η=10 provides results that are close enough to the
results at infinity. Set Prandtl number =1 and n=0.5 for the numerical solution
Pr=1 "Prandtl number"
n=0.5 "exponent for the equation (T_s-T_inf)=A*x^n"
eta_hi=10 "upper limit for eta - assume to be high enough"
eta_lo=0 "lower limit for eta"

df dθ
Enter guess values for and to test the equations.
dη η =0 dη o
d2f\deta2_o=0.5 "guess for testing"
dtheta\deta_o=-0.5 "guess for testing"

Solve. The boundary conditions required by Eqs. (62) and (66) are not met. We will find the
df dθ
values of and that fulfill the boundary conditions by optimization. The figure of
dη η =0 dη o
merit that we will use is
err=sqrt(df\deta^2+theta^2) "minimize err with respect to d2f\deta2_o and dtheta\deta_o"

Now comment out the guess values above and select the MinMax command (Options menu).
Minimize err with independent variables d2f\deta2_o and dtheta\deta_o
Select the Variable metric method and uncheck the option to Stop if error occurs. It will be
necessary to enter lower and upper bounds for the independent variables. Enter the following
bounds:
Click OK twice. After EES completes the calculations, the values of f and theta will be 0.6064
and -0.5822, respectively. The boundary conditions at η=10 (approximately infinite) are met.
The solution is complete. A MATLAB solution follows:

The Euler technique is implemented in MATLAB in order to integrate from η = 0 to a position


far from the wall. Recall that η is defined as the ratio of the distance from the plate to the
thickness of the boundary layer. Because the temperature rise in the fluid drives the velocity, it
is reasonable to expect that the momentum and thermal boundary layers will have similar
thickness unless the Prandtl number is very different than unity. Therefore, it should be
reasonable to terminate the integration at a value of η that is much larger than unity, for example
out to η∞ = 10, and enforce the boundary conditions given by Eqs. (62) and (66) at η∞.

The computational domain (0 < η < η∞) is divided into steps of size Δη:

η∞
Δη = (6-72)
( N − 1)
where N is the number of nodes. The location of each node is provided by:

ηi = η ∞
( i − 1) for i = 1..N (6-73)
( N − 1)

The solution is implemented in a MATLAB script named NC.

clear all;
Pr=1; % Prandtl number (-)
n=0.5; % exponent on the surface temperature variation (-)

eta_infinity=10; % outer edge of computational domain (-)


N=5001; % number of steps in the numerical integration (-)
Dn=eta_infinity/(N-1); % size of the integration steps (-)
for i=1:N
eta(i)=eta_infinity*(i-1)/(N-1); % location of integration steps (-)
end
d2 f
The initial conditions for the integration process are specified; note that the values of
dη 2 η =0

dθ
and (the variables d2fdn2_0 and dqdn_0) are assumed and must be adjusted to complete
dη η =0
the problem.

d2fdn2_0=0.5; % 2nd derivative of f at the wall (-)


dqdn_0=-0.5; % 1st derivative of q at the wall (-)

%boundary conditions at the wall


f(1)=0;
dfdn(1)=0;
d2fdn2(1)=d2fdn2_0;
q(1)=1;
dqdn(1)=dqdn_0;

Each integration step is taken using Euler's method; the rates of change of each state variable are
evaluated at the beginning of the integration step using Eqs. (67) through
Error! Reference source not found.:

df
fi +1 = fi + Δη (6-74)
dη i

df df d2 f
= + 2 Δη (6-75)
dη i +1
dη i dη i

d2 f d2 f ⎡ d2 f ⎛ df ⎞
2

= + ⎢ − ( 3 + n ) f + 2 ( n + 1) ⎜⎜ ⎟⎟ − θ ⎥
i Δη (6-76)
dη 2 dη i ⎢ dη i ⎝ dη
2 i 2
i⎠ ⎥
i +1 ⎣ ⎦

dθ
θi +1 = θi + Δη (6-77)
dη i

dθ dθ ⎡ dθ ⎤


= + ⎢ − ( 3 + n ) fi Pr ⎥ Δη (6-78)
dη i +1 dη i ⎣⎢ dη i ⎦⎥

for i=1:(N-1)
f(i+1)=f(i)+dfdn(i)*Dn;
dfdn(i+1)=dfdn(i)+d2fdn2(i)*Dn;
d2fdn2(i+1)=d2fdn2(i)+(-(3+n)*f(i)*d2fdn2(i)+2*(n+1)*(dfdn(i))^2-...
q(i))*Dn;
q(i+1)=q(i)+dqdn(i)*Dn;
dqdn(i+1)=dqdn(i)-(3+n)*Pr*f(i)*dqdn(i)*Dn;
end

df
Figure P6.2-1(a) illustrates θ and as a function of η for the case where n = 0.5. Note that

neither of the two boundary conditions at η → ∞ are satisfied and, in fact, the solution grows
d2 f dθ
dramatically and diverges with η even for the reasonable assumed values of and .
dη η =0
2
dη η =0
10

Dimensionless temperature and dfdη


7.5

5
dfdη
2.5

-2.5
dimensionless temperature

-5
0 0.5 1 1.5 2 2.5 3 3.5 4 4.5 5
Dimensionless position, η
df
Figure 6.3-2(a): Dimensionless temperature and as a function of η for Pr = 1.0 and n = 0.5 with

d2 f dθ
= 0.5 and = -0.5 .
dη 2 η =0
dη η =0

df
An error is calculated based on difference between the values of θ and at η∞ and their

required values, based on Eqs. (62) and (66) (i.e., zero).

err=sqrt(dfdn(N)^2+q(N)^2); % error in boundary conditions at n_infinity

d2 f dθ df
The next step in the solution is to adjust and so that = 0 and θη →∞ = 0 ;
dη 2 η =0
dη η =0 dη η →∞
unfortunately, the problem is sufficiently "stiff" (i.e., the values of the outputs are very sensitive
to the values of the inputs) that this is quite difficult to accomplish. Notice that the solution in
Figure 6.3-2(a) grows rapidly as η becomes large. In fact, the solution for dfdn eventually
becomes larger than MATLAB can represent and therefore the final entries in the vector dfdn are
NaN, which is MATLAB shorthand for "not a number". One of the problems with this solution
is that a lot of computational time is wasted finishing the numerical integration for a set of
boundary conditions that are clearly not viable. Anytime the dimensionless temperature
becomes less than 0 or greater than 1 or the dimensionless velocity becomes less than 0 or
d2 f dθ
substantially greater than 1 then it is clear that the assumed values of and are not
dη η =0
2
dη η =0
appropriate and therefore the integration should be terminated. For example, the integration
df
shown in Figure 6.3-2(a) should have been terminated at about η = 1.3 when became

negative. The integration is terminated if the velocity or temperature go out of bounds using the
break command. The break command terminates the execution of a for loop in MATLAB. If the
df
integration is terminated, then the value of θ and at η∞ are assigned to a large value that is

inversely proportional to the point at which the integration went out of bounds.

if((q(i+1)>1)|(q(i+1)<0)|(f(i+1)>2)|(f(i+1)<0))
q(N)=1+1/i;
dfdn(N)=1+1/i;
break;
end

Implementing the solution in MATLAB allows the use of the sophisticated multi-dimensional
optimization functions that are native to MATLAB. In order to use these functions, it is
necessary to convert the script NC into a function. The function takes as input arguments a
vector X that includes the two unknown boundary conditions at η = 0 as well as the Prandtl
number and returns as the first output the scalar argument err, which is related to how well the
boundary conditions at η∞ are met. The three additional outputs are the vectors n, dfdn, and q.

function[err,eta,dfdn,q]=P6p3d2(X,Pr,n)

% Inputs:
% X - vector of unknown boundary conditions at n=0
% X(1) - d2fdn2 at n=0 (-)
% X(2) - dqdn at n=0 (-)
% Pr - Prandtl number (-)
% n - exponent on surface temperature distribution (-)
%
% Outputs:
% err - error associated with mismatch in bc's at n_infinity (-)
% eta - vector of nodal positions (-)
% dfdn - dfdn at each nodal position (-)
% q - q at each nodal position (-)

eta_infinity=10; % outer edge of computational domain (-)


N=5001; % number of steps in the numerical integration (-)
Dn=eta_infinity/(N-1); % size of the integration steps (-)
for i=1:N
eta(i)=eta_infinity*(i-1)/(N-1); % location of integration steps (-)
end

%boundary conditions at the wall


f(1)=0;
dfdn(1)=0;
d2fdn2(1)=X(1);
q(1)=1;
dqdn(1)=X(2);

for i=1:(N-1)
f(i+1)=f(i)+dfdn(i)*Dn;
dfdn(i+1)=dfdn(i)+d2fdn2(i)*Dn;
d2fdn2(i+1)=d2fdn2(i)+(-(3+n)*f(i)*d2fdn2(i)+2*(n+1)*(dfdn(i))^2-...
q(i))*Dn;
q(i+1)=q(i)+dqdn(i)*Dn;
dqdn(i+1)=dqdn(i)-(3+n)*Pr*f(i)*dqdn(i)*Dn;
if((q(i+1)>1)|(q(i+1)<0)|(f(i+1)>2)|(f(i+1)<0))
q(N)=1+1/i;
dfdn(N)=1+1/i;
break;
end
end

err=sqrt(dfdn(N)^2+q(N)^2); % error in boundary conditions at n_infinity


end

The function fminsearch in MATLAB allows multidimensional, unconstrained optimization using


the Nelder-Mead algorithm. The calling protocol for fminsearch is:

X=fminsearch(fun,X0,OPTIONS)

where fun is a handle for the function to be minimized. This function must accept input X that
can be a scalar or a vector of inputs. The vector X0 is the starting values of these inputs that are
used to initiate the minimization and OPTIONS which is a vector of options that can be used to
control the optimization process.

A script called P6p3d2_run is generated to control the optimization. The variable space is cleared
and the Prandtl number set:

clear all;
Pr=1;
n=0.5;

Reasonable values are used to setup X0:

dfdn_0=0.5;
dqdn_0=-0.5;
X0=[dfdn_0,dqdn_0];

The OPTIONS vector is set up using the optimset command; the optimset command is used in the
same way that the odeset command was used to set up the OPTIONS vector for use with
MATLAB's native ode solvers, as discussed in Section 3.2.2. Here, the optimset command is
used to specify that the result of each iteration should be displayed and to set both termination
tolerances to 1x10-6.

OPTIONS=optimset('Display','iter','TolFun',1e-6,'TolX',1e-6);

The fminsearch command is called; notice that the function call is mapped onto a call of the
function P6p3d2, as discussed in Section 3.2.2 and elsewhere.

X = fminsearch(@(X) P6p3d2(X,Pr,n),X0,OPTIONS)

The P6p3d2 function is run again using the boundary conditions identified by the optimization:
[err,eta,dfdn,q]=P6p3d2(X,Pr,n);

The script P6p3d2_run is run from the command line:

>> P6p3d2_run

Iteration Func-count min f(x) Procedure


0 1 1.4155
1 3 1.41537 initial simplex
2 5 1.41527 expand
3 7 1.41499 expand
4 9 0.217854 expand
5 10 0.217854 reflect
6 12 0.217854 contract inside
...
60 116 1.98956e-006 contract inside
61 118 1.98956e-006 contract inside
62 120 1.98956e-006 contract inside

Optimization terminated:
the current x satisfies the termination criteria using OPTIONS.TolX of 1.000000e-006
and F(X) satisfies the convergence criteria using OPTIONS.TolFun of 1.000000e-006

X=

0.6056 -0.5814

The progress of the minimization algorithm is reported after each iteration and the optimization
d2 f dθ
will identify the appropriate boundary conditions, = 0.6056 and ≈ −0.5814 .
dη 2 η =0 dη η =0

df
d.) Plot the dimensionless temperature and velocity ( θ and ) as a function of dimensionless

position (η) for the case where Pr = 1 and n = 0.5.

df
Figure P6.3-2(b) illustrates the dimensionless temperature and (which is proportional to the

dimensionless velocity) as a function of η for these boundary conditions.
Dimensionless temperature and dfdη
1

0.8
dimensionless temperature

0.6

0.4

0.2
dfdη

0
0 1 2 3 4 5
Dimensionless position, η
df
Figure P6.3-2(b): Dimensionless temperature and as a function of η for Pr = 0.7 and n = 0.5 with

d2 f dθ
= 0.6056 and = -0.5814 .
dη η =0
2
dη η =0

e.) Plot the product of the local Nusselt number and the Grashof number based on the local plate
temperature to the -1/4 power as a function of Pr for various values of n.

dθ
The Nusselt number is related to . The local Nusselt number is defined as:
dη η =0

hx qs′′ x
Nu x = = (6-79)
k (Ts − T∞ ) k
The heat flux is given by:

⎛ ∂T ⎞
qs′′ = −k ⎜ ⎟ (6-80)
⎝ ∂y ⎠ y =0

Substituting the definition for dimensionless temperature into Eq. (6-80) leads to:

⎛ ∂θ ⎞
qs′′ = −k (Ts − T∞ ) ⎜ ⎟ (6-81)
⎝ ∂y ⎠ y =0

Substituting Eq. into Eq. (6-81) leads to:


dθ 1 ( n −1)
qs′′ = − k (Ts − T∞ ) B 4x 4 (6-82)
dη η =0

Substituting Eq. (18) into Eq. leads to:

1 1 1
⎛ Grx ⎞ 4
⎛ g β (Ts − T∞ ) x 3 ⎞ 4
⎛ g β A x n x3 ⎞ 4
1 ( 3+ n )
⎜ ⎟ =⎜ ⎟ =⎜ ⎟ = B 4x 4
(6-83)
⎝ 4 ⎠ ⎝ 4υ 2 ⎠ ⎝ 4υ 2 ⎠

gβ A
B= (6-84)
4υ 2

1
⎛ Ts −T∞
P ⎞ 4
dθ ⎜ g β Ax n ⎟
−1
qs′′ = −k (Ts − T∞ ) ⎜ ⎟ x 4 (6-85)
dη η =0 ⎜ 4υ 2
⎟⎟

⎝ ⎠

Substituting Eq. into Eq. (6-79) leads to:

1
⎛ dθ ⎞ ⎡ g β (Ts − T∞ ) ⎤ 4
−1 x
Nu x = − k (Ts − T∞ ) ⎜ ⎟ ⎢ ⎥ x 4
(6-86)
⎝ dη ⎠η =0 ⎣ 4υ 2 ⎦ (Ts − T∞ ) k

which can be simplified to:


1
⎛ dθ ⎞ ⎛ Grx ⎞ 4
Nu x = − ⎜ ⎟ ⎜ ⎟ (6-87)
⎝ dη ⎠η =0 ⎝ 4 ⎠

The product of the local Nusselt number and the local Grashof number to the -1/4 power is related
to the gradient of the dimensionless temperature at the surface of the plate, which is a function of
Pr and n:

1
−1 ⎛ dθ ⎞ ⎛ 1 ⎞ 4
Nu x Gr 4
= −⎜ ⎟ ⎜ ⎟ (6-88)
⎝ dη ⎠η =0 ⎝ 4 ⎠
x

The MATLAB script P6p3d2_run is modified to identify the value of the dimensionless
temperature gradient at the plate over a range of Prandtl number:

clear all;
Prv=[0.01,0.03,0.05,0.07,0.1,0.3,0.5,0.7,1,2,3,4,5,7,10,20,30,40,50,70,100,20
0,300,500,700,1000];
n=-1;
dfdn_0=0.5;
dqdn_0=-0.5;

for i=1:26
i
Pr=Prv(i);
X0=[dfdn_0,dqdn_0];
OPTIONS=optimset('Display','iter','TolFun',1e-6,'TolX',1e-5);
X = fminsearch(@(X) P6p3d2(X,Pr,n),X0,OPTIONS)
[err,eta,dfdn,q]=P6p3d2(X,Pr,n);
dfdn_0=X(1);
dqdn_0=X(2);
dqdnv(i)=-dqdn_0;
end

−1
Figure P6.3-2(c) illustrates the value of Nu x Grx 4
as a function of Pr for various values of the
Nusselt number.
3
n=1
2.5 n = 0.5
n = 0.2
2 n=0
-1/4
Nux Grx

1.5

0.5

0
0.01 0.1 1 10 100 1000
Prandtl number
−1
Figure P6.3-2(c): Nu x Grx 4
as a function of Pr for various values of n.

f.) Plot the product of the average Nusselt number and the Grashof number based on the average
plate temperature to the -1/4 power as a function of Pr for various values of n.

The average Nusselt number is defined as:


L L
1 1 Nu k
hL = ∫ h dx = ∫ x dx (6-89)
L0 L0 x

Substituting Eq. (6-87) into Eq. (6-89) leads to:


1
1
L
⎛ dθ ⎞ ⎛ Grx ⎞ 4
k
hL = ∫ − ⎜ ⎟ dx (6-90)
L 0 ⎝ dη ⎠η =0 ⎜⎝ 4 ⎟⎠ x
Substituting the definition of the Grashof number into Eq. (6-90) leads to:

1
⎛ dθ ⎞ k L ⎛ g β A x n ⎞ 4
−1
hL = − ⎜ ⎟ ∫⎜
⎝ dη ⎠η =0 L 0 ⎝ 4υ
2 ⎟

x 4
dx (6-91)

or

1
⎛ dθ ⎞ k ⎛ g β A ⎞ 4 L ( n −1)
hL = − ⎜ ⎟ ⎜ 2 ⎟
⎝ dη ⎠η =0 L ⎝ 4υ ⎠
∫x
0
4
dx (6-92)

Carrying out the integration leads to:

1
⎛ dθ ⎞ k ⎛ g β A ⎞ 4
4 ( n + 3)
hL = − ⎜ ⎟ ⎜ 2 ⎟
L 4 (6-93)
⎝ dη ⎠η =0 L ⎝ 4υ ⎠ ( n + 3)

The average surface-to-fluid temperature of the plate is:


L
1 A Ln +1 A Ln
L ∫0
Ts − T∞ = A x n
dx = = (6-94)
L ( n + 1) ( n + 1)

Substituting Eq. (6-94) into Eq. (6-93) leads to:

1
⎛ dθ ⎞ k ⎛ g β A Ln ( n + 1) ⎞ 4
4 ( n + 3)
hL = − ⎜ ⎟ ⎜⎜ ⎟⎟ L 4 (6-95)
⎝ dη ⎠η =0 L ⎝ 4υ ( n + 1) L ⎠ ( n + 3)
2 n

or
1
⎛ dθ ⎞ k ⎛ g β (Ts − T∞ ) ( n + 1) ⎞
4
4 ( n + 3)
hL = − ⎜ ⎟ ⎜ ⎟ L 4 (6-96)
⎝ dη ⎠η =0 L ⎜⎝ 4υ 2 Ln ⎟⎠ ( n + 3)

which can be rearranged:


1
⎛ dθ ⎞ k ⎛ g β (Ts − T∞ ) L ⎞ 4 ( n + 1)
3 4 1/ 4

hL = − ⎜ ⎟ ⎜ ⎟ (6-97)
⎝ dη ⎠η =0 L ⎜⎝ 4υ 2 ⎟ ( n + 3)


GrL
4

Substituting Eq. into the definition of the average Nusselt number leads to:
1
⎛ dθ ⎞ ⎛ g β (Ts − T∞ ) L ⎞ 4 ( n + 1)
3 4 1/ 4
hL L
Nu L = = −⎜ ⎟ ⎜ ⎟ (6-98)
k ⎝ dη ⎠η =0 ⎜⎝ 4υ 2 ⎟ ( n + 3)


GrL
4

or

1
4 ( n + 1)
1/ 4
h L ⎛ dθ ⎞ ⎛ GrL ⎞ 4
Nu L = L = − ⎜ ⎟ ⎜ ⎟ (6-99)
k ⎝ dη ⎠η =0 ⎝ 4 ⎠ ( n + 3)

where GrL is the Grashof number based on the average plate temperature:

g β (Ts − T∞ ) L3
GrL = (6-100)
υ2

The product of the average Nusselt number and the average Grashoff number to the -1/4 power is
a function of Prandtl number and n:

4 ( n + 1)
1/ 4
−1 ⎛ dθ ⎞
Nu L GrL 4
= −⎜ ⎟ (6-101)
⎝ dη ⎠η =0 ( n + 3) 4 4
1

The MATLAB code P6p3d2_run is modified to return this parameter. Figure P6.3-2(d) illustrates
−1
4
Nu L GrL as a function of Pr for various values of n.

3.5 n=1
n = 0.5
3 n = 0.2

2.5 n=0
-1/4
NuL GrL

1.5

0.5

0
0.001 0.01 0.1 1 10 100 1000
Prandtl number
−1
4
Figure P6.3-2(d): NuL GrL as a function of Pr for various values of n.
g.) Plot the average Nusselt number as a function of the Grashof number based on the average
plate temperature to the -1/4 power for a plate with a constant heat flux for Pr = 0.7.

The heat flux at the surface of the plate is:

1
⎛ Ts −T∞
P ⎞ 4
dθ ⎜ g β A x n ⎟ −1 4
qs′′ = −k (Ts − T∞ ) ⎜ ⎟ x (6-102)
dη η =0 ⎜ 4υ 2 ⎟
⎜ ⎟
⎝ ⎠

or

1
⎛ Ts −T∞
P ⎞ 4

d θ ⎜ g β A x n ⎟
−1 ( 5 n −1)
qs′′ = −k A x n ⎜ ⎟ x 4
∝x 4
(6-103)
dη η =0 ⎜ 4υ 2 ⎟
⎜ ⎟
⎝ ⎠

Therefore, if n = 0.2 then the solution corresponds to a plate with a uniform heat flux and n = 0
corresponds to a plate with a constant surface temperature. Using the self-similar solution, the
average Nusselt number for a plate with a constant heat flux as a function of the Grashof number
based on the average plate temperature is shown in Figure P6.3-2(e) with Pr = 0.7.

10 3

10 2
NuL

10 1

10 0
102 103 104 105 106 107 108 109 1010 1011 1012
GrL
Figure P6.3-2(d): Average Nusselt number as a function of the Grashof number based on the average plate
temperature for Pr = 0.7 for a plate with a constant heat flux (n = 0.2).
Chapter 6: Natural Convection
Section 6.2: Natural Convection Correlations
(6-1) 6.2-1 A pipe that is 2L = 3 m long has an outer diameter Dout = 0.1 m and is bent in the
center to form an “L” shape, as shown in Figure P6.2-1. One leg is vertical and the other
leg is horizontal. The pipe is made of thin-walled copper and saturated steam at
atmospheric pressure is circulating through the pipe. The pipe is in a large room and the
air temperature far from the pipe is T∞ = 30°C at atmospheric pressure. The pipe carries
two-phase steam at ps = 1.01 bar. The conduction resistance associated with the pipe wall
and the convection resistance associated with steam can be neglected.

two-phase steam
ps = 1.01 bar
g
T∞ = 30°C

L = 1.5 m
Dout = 0.1 m

two-phase steam
L = 1.5 m ps = 1.01 bar
Figure P6.2-1: An L-shaped pipe.

a) Determine the Grashof, Rayleigh, and Nusselt numbers and the corresponding average
heat transfer coefficient for the horizontal section of the pipe.
b) Determine the Grashof, Rayleigh, and Nusselt numbers and the corresponding average
heat transfer coefficient for the vertical section of the pipe.
c) Calculate the total rate of heat transfer to the air.

6.2-2 Clean engine oil enters a L = 50 m long thin-walled pipe having an outer diameter Dout =
30 mm. The mass flow rate is m = 0.25 kg/s and the inlet temperature is Tin = 150°C.
The pipe is suspended in a large room in which the air temperature is T∞ = 20°C.
Estimate the heat transfer rate from the oil to the room and the outlet oil temperature.

(6-2) 6.2-3 A resistance temperature detector (RTD) is inserted into a methane pipeline to
measure the temperature. The sensor is spherical with a diameter, D = 5.0 mm, and is
exposed to methane at Pf = 10 atm with a fluid temperature of Tf = 20°C. The resistance
of the sensor is related to its temperature; the resistance is measured by passing a known
current through the resistor and measuring the associated voltage drop. The current
causes an ohmic dissipation of q = 5.0 milliW. You have been asked to estimate the
associated self-heating error as a function of the velocity of the methane in the pipe, Vf;
focus on the very low velocity operation (e.g., 0 to 0.1 m/s) where self-heating might be
large. The self-heating error is the amount that the temperature sensor surface must rise
relative to the surrounding fluid in order to transfer the heat associated with ohmic
dissipation. You may neglect radiation for this problem.
a.) Assume that only forced convection is important and prepare a plot showing the self-
heating error as a function of the methane velocity for velocities ranging from 0 to 0.1
m/s.
b.) Assume that only natural convection is important and determine the self-heating error
in this limit. Overlay this value on your plot from (a).
c.) Prepare a plot that shows your prediction for the self heating error as a function of
velocity considering both natural convection and forced convection effects. Assume
that the pipe is mounted horizontally.

(6-3) 6.2-4 Figure P6.2-4 illustrates a flat plate solar collector that is mounted at an angle of τ
= 45 degrees on the roof of a house. The collector is used to heat water; a series of tubes
are soldered to the back-side of a black plate. The collector plate is contained in a case
with a glass cover.

g T∞ = 20°C
ho = 150 W/m -K
2

H=1m

q′′s = 800 W/m


2

back plate
insulation
Tp = 80°C transparent glass cover
roof
tubes

τ = 45 degree L = 2 cm
Figure P6.2-4: Flat plate solar collector

Assume that the solar collector is H = 1 m wide by W = 1 m long (into the page) and the
distance between the heated plate and the glass covering is L = 2 cm. The collector
receives a solar flux q s′′ =800 W/m2 and the collector plate can be assumed to absorb all of
the solar energy. The collected energy is either transferred to the water in the pipe (in
which case the energy is used to provide useful water heating) or lost due to heat transfer
with the environment (either by radiation, which will be neglected in this problem, or
convection). The collector plate temperature is Tp = 80°C and the ambient temperature is
T∞ = 20°C. The heat transfer coefficient on the external surface of the glass is forced
convection (there is a slight breeze) and equal to ho = 150 W/m2-K. The glass is thin and
can be neglected from the standpoint of providing any thermal resistance between the
plate and ambient. Neglect radiative effects.
a.) Determine the rate of heat loss from plate due to convection; you may assume that the
insulation on the back of the tubes is perfect so no heat is conducted to the roof and
that radiation from the plate is negligible.
b.) What is the efficiency of the solar collector, ηcollector, defined as the ratio of the energy
delivered to the water to the energy received from the sun?
c.) Prepare a plot showing the collector efficiency as a function of the plate to glass
spacing, L. Explain the shape of the plot.

(6-4) 6.2-5 Figure P6.2-5 illustrates a single-paned glass window that is L = 6 ft high and W =
4 ft wide; the glass is thg = 0.25 inch thick and has conductivity kg = 1.4 W/m-K.

thg = 0.25 inch


Tair ,in = 70°F L = 6 ft

kg = 1.4 W/m-K Tair ,out = 10°F

Figure P6.2-5: Single-paned glass window.

On a typical winter day, the outdoor temperature is Tair,out =10°F and you keep the indoor
temperature at Tair,in = 70°F.
a.) On a still winter day, estimate the rate of heat loss from the window.
b.) Winter lasts twinter= 90 days and you are heating with electrical resistance heaters.
Electricity costs ecost = $0.12/kW-hr. How much does the heat loss through the
window cost you over the course of 1 winter?
c.) About 40% of your home's heat loss will occur through its windows; more if you
have single paned windows like the one shown in Fig. 1. Assume that 50% of your
heat loss in your house is through your windows and that you have Nwindow = 10 single
paned windows in your house. Prepare a plot showing the cost of heating your house
as a function of the thermostat set point (i.e., the indoor air temperature).

(6-5) 6.2-6 The single-glazed window in problem 6.2-5 is replaced with a double-glazed
window. Both glass panes are 0.25 inch thick and the gap between the panes is 0.5 inch.
The gap contains dry air at atmospheric pressure. All other information is the same as in
problem 6.2-5. Neglect heat transfer by radiation.
a.) Repeat the calculations requested in parts (a) and (b) of problem 6.2-5.
b.) Summarize and explain the benefits of the double-glazed window.

(6-6) 6.2-7 You have seen an advertisement for argon-filled windows. These windows are
similar in construction to the window described in problem 6.2-6, except that argon,
rather than air, is contained in the gap. Neglect heat transfer by radiation.
a.) Repeat problem 6.2-6 assuming that the gap contains argon.
b.) Are the claims that argon reduces heat loss valid? If so, why does this behavior
occur?
c.)Would nitrogen (which is cheaper) work as well? Why or why not? Can you suggest
another gas that would work better than argon?

(6-7) 6.2-8 You are involved in a project to design a solar collector for heating air. Two
competing designs are shown in Figure P6.2-8.
2 2
200 W/m to 800 W/m
5°C, 5 m/s
glass cover plate
air at 25°C, 100 kPa
2
10 liter/s/m of collector 20 mm

black collector plate


2.5 m
(a)
2 2
5°C, 5 m/s 200 W/m to 800 W/m

glass cover plate

black collector plate 20 mm


air at 25°C, 100 kPa 20 mm
2
10 liter/s/m of collector

2.5 m
(b)
Figure P6.2-8: Air heating solar collector with (a) air flowing above the collector plate and (b) air
flowing below the collector plate.

Both designs employ a transparent glass cover plate and a thin metal opaque black
collector plate upon which solar radiation is completely absorbed. The glazing is
standard safety glass with a thickness of 6 mm. In the first design, shown in Figure P6.2-
8(a), air is blown through the gap between the cover and plate. In the second design,
shown in Figure P6.2-8(b), the air flows in a second gap that is below the collector plate
and free convection occurs between the collector plate and the glass cover plate. The
collector is 1 meter wide (into the page) and 2.5 m long (in the air flow direction) and
oriented horizontally. In both designs, the gaps are 20 mm wide. Air at 25°C, 100 kPa
enters the flow passage at a flow rate of 10 liters/sec per square meter of collector area
(area exposed to solar radiation) in both cases. The outdoor temperature (above the glass
cover plate) is 5°C and there is a wind that may be represented as a forced convective
flow with a free stream velocity of 5 m/s in the flow direction. Calculate and plot the
efficiency of the two collector designs as a function of the solar radiation absorbed on the
plate for values between 200 and 800 W/m2. Assume that the insulation is adiabatic and
neglect radiation in these calculations.

Section 6.3: Self-Similar Solutions


(6-8) 6.3-1 Reconsider Problem P6.2-5. In Problem P6.2-5, the glass was assumed to be
isothermal and the correlations for the average heat transfer coefficient were used. In this
problem, account for the variation of the local heat transfer coefficient on either side of
the window using the self-similar solution. Neglect conduction along the length of the
glass but allow the glass temperature to vary with position due to the variation of the heat
transfer coefficient with position.
a.) Determine the total rate of heat transfer through the window. Compare your answer
with the solution for P5.2-5.
b.) Plot the inner and outer temperature of the glass as a function of position.
c.) If the relative humidity of the indoor air is 65% then will condensate form on the
window? If so, at what location will the condensate end?

(6-9) 6.3-2 A self-similar solution can be obtained for the free convection problem where a
heated vertical plate has a surface temperature (Ts) that varies with position according to:
Ts − T∞ = A x n where x is measured from the bottom of the plate.
a.) Transform the governing partial differential equations for momentum conservation in
the x-direction and thermal energy conservation into ordinary differential equations
for f and θ .
b.) Transform the boundary conditions for u, v, and T into boundary conditions for f and
θ .
c.) Develop a numerical solution for this problem.
df
d.) Plot the dimensionless temperature and velocity ( θ and ) as a function of

dimensionless position (η) for the case where Pr = 1 and n = 0.5.
e.) Plot the product of the local Nusselt number and the Grashof number based on the
local plate temperature to the -1/4 power as a function of Pr for various values of n.
f.) Plot the product of the average Nusselt number and the Grashof number based on the
average plate temperature to the -1/4 power as a function of Pr for various values of n.
g.) Plot the average Nusselt number as a function of the Grashof number based on the
average plate temperature to the -1/4 power for a plate with a constant heat flux for Pr
= 0.7.
PROBLEM 6.2-3 (6-2 in text): Self heating error
A resistance temperature detector (RTD) is inserted into a methane pipeline to measure the
temperature. The sensor is spherical with a diameter, D = 5.0 mm, and is exposed to methane at
Pf = 10 atm with a fluid temperature of Tf = 20°C. The resistance of the sensor is related to its
temperature; the resistance is measured by passing a known current through the resistor and
measuring the associated voltage drop. The current causes an ohmic dissipation of q = 5.0
milliW. You have been asked to estimate the associated self-heating error as a function of the
velocity of the methane in the pipe, Vf; focus on the very low velocity operation (e.g., 0 to 0.1
m/s) where self-heating might be large. The self-heating error is the amount that the temperature
sensor surface must rise relative to the surrounding fluid in order to transfer the heat associated
with ohmic dissipation. You may neglect radiation for this problem.

a.) Assume that only forced convection is important and prepare a plot showing the self-heating
error as a function of the methane velocity for velocities ranging from 0 to 0.1 m/s.

The inputs are entered in EES, a value of fluid velocity is used to start the calculation:
$UnitSystem SI MASS RAD PA K J
$TABSTOPS 0.2 0.4 0.6 0.8 3.5 in

T_f=converttemp(C,K,20) "fluid temperature"


P_f=10 [atm]*convert(atm,Pa) "fluid pressure"
V_f=1 [m/s] "fluid velocity"
d=5 [mm]*convert(mm,m) "diameter of sensor"
q_dot=5 [milliW]*convert(milliW,W) "ohmic dissipation"

The temperature of the sensor under forced convection conditions (Ts,fc) is assumed in order to
allow the film temperature to be computed:

Ts , fc + T f
T film, fc = (1)
2

The fluid properties under forced convection conditions (ρfc, kfc, μfc, cfc, αfc, νfc, and Prfc) are
obtained using EES’ built-in property routines evaluated at Tfilm,fc and Pf.
"forced convection"
T_s_fc=300 [K]
"guess for forced convection temperature"
T_film_fc=(T_s_fc+T_f)/2 "film temperature"
rho_fc=density(Methane,T=T_film_fc,P=P_f) "density"
k_fc=conductivity(Methane,T=T_film_fc,P=P_f) "conductivity"
mu_fc=viscosity(Methane,T=T_film_fc,P=P_f) "viscosity"
c_fc=cP(Methane,T=T_film_fc,P=P_f) "cP"
alpha_fc=k_fc/(rho_fc*c_fc) "thermal diffusivity"
nu_fc=mu_fc/rho_fc "kinematic viscosity"
Pr_fc=nu_fc/alpha_fc "Prandtl number"

The Reynolds number is computed:


ρ fc DV f
ReD = (2)
μ fc

and used to call the appropriate correlations for external forced convection heat transfer from a
sphere; these are contained in the procedure External_Flow_Sphere_ND. The correlation
provides the forced convection Nusselt number (Nufc) which is used to compute the forced
convection heat transfer coefficient (hfc):

Nu fc k fc
h fc = (3)
D
Re_D=rho_fc*D*V_f/mu_fc "Reynolds number"
Call External_Flow_Sphere_ND(Re_D,Pr_fc :C_d, Nusselt_fc) "access correlations"
h_fc=Nusselt_fc*k_fc/D “forced convection heat transfer coefficient"

The guess values for the calculation are updated (select Update Guess Values from the Calculate
menu) and the surface temperature assumption is commented out. The surface temperature
under forced convection is computed according to:

q
Ts , fc = T f + 3
(4)
⎛D⎞
h fc 4 π ⎜ ⎟
⎝2⎠

The self-heating error under forced convection conditions is calculated:

δ T fc = Ts , fc − T f (5)
{T_s_fc=300 [K] "guess for forced convection temperature"}
T_s_fc=T_f+q_dot/(4*pi*(D/2)^2*h_fc) "surface temperature"
deltaT_fc=(T_s_fc-T_f) "self heating error"

The value for Vf is commented out and a parametric table is created that includes δTfc and Vf.
The value of Vf is varied from very near zero (0.0001 m/s; the correlations will not work with Vf
= 0 m/s) to 30 m/s. Figure 1 illustrates the predicted self-heating error under forced convection
conditions as a function of the fluid velocity. Notice that the self-heating error becomes very
large as the velocity approaches 0 m/s because the heat transfer coefficient assuming only forced
flow becomes small; it is in this region that the impact of natural convection may become
significant.
Figure 1: Self-heating error as a function of the fluid velocity.

b.) Assume that only natural convection is important and determine the self-heating error in this
limit. Overlay this value on your plot from (a).

A surface temperature for the sensor under natural convection conditions (Ts,nc) is assumed and
used to compute the film temperature for natural convection conditions:

Ts ,nc + T f
T film, nc = (6)
2

The fluid properties under natural convection conditions (ρnc, knc, μnc, cnc, αnc, νnc, βnc, and Prnc)
are obtained using EES’ built-in property routines evaluated at Tfilm,nc and Pf.
"natural convection"
T_s_nc=300 [K]
"guess for natural convection temperature"
T_film_nc=(T_s_nc+T_f)/2 "film temperature"
rho_nc=density(Methane,T=T_film_nc,P=P_f) "density"
k_nc=conductivity(Methane,T=T_film_nc,P=P_f) "conductivity"
mu_nc=viscosity(Methane,T=T_film_nc,P=P_f) "viscosity"
c_nc=cP(Methane,T=T_film_nc,P=P_f) "cP"
beta_nc=VolExpCoef(Methane,T=T_film_nc,P=P_f) "volumetric expansion coefficient"
alpha_nc=k_nc/(rho_nc*c_nc) "thermal diffusivity"
nu_nc=mu_nc/rho_nc "kinematic viscosity"
Pr_nc=nu_nc/alpha_nc "Prandtl number"

The Grashof and Raleigh numbers are computed:

g β nc D 3 (Ts ,nc − T f )
GrD = (7)
υnc2
RaD = GrD Prnc (8)

and used to access the correlations for natural convection from a sphere (contained in the
procedure FC_sphere_ND) which returns the Nusselt number under natural convection
conditions (Nunc). The heat transfer coefficient under natural convection conditions is computed:

Nunc knc
hnc = (9)
D
Gr_D=g#*beta_nc*D^3*(T_s_nc-T_f)/(nu_nc^2) "Grashof number"
Ra_D=Gr_D*Pr_nc "Raleigh number"
Call FC_sphere_ND(Ra_D, Pr_nc: Nusselt_nc) "access correlations"
h_nc=Nusselt_nc*k_nc/D "natural convection heat transfer coefficient"

The guess values for the calculation are updated (select Update Guess Values from the Calculate
menu) and the surface temperature assumption for natural convection is commented out. The
surface temperature under natural convection conditions is computed according to:

q
Ts ,nc = T f + 3
(10)
⎛D⎞
hnc 4 π ⎜ ⎟
⎝2⎠

The self-heating error under natural convection conditions is calculated:

δ Tnc = Ts ,nc − T f (11)

T_s_nc=T_f+q_dot/(4*pi*(D/2)^2*h_nc) "surface temperature"


deltaT_nc=(T_s_nc-T_f) "self heating error"

The self-heating error is predicted to be δTnc =1.81 K; this value is overlaid on Fig. 1. Note that
the natural convection effect will limit that maximum self-heating error at low velocities to less
than 1.81 K.

c.) Prepare a plot that shows your prediction for the self heating error as a function of velocity
considering both natural convection and forced convection effects. Assume that the pipe is
mounted horizontally.

In order to account for the mixed conditions that occur at low velocity it is possible to estimate
the mixed heat transfer coefficient according to:

hmixed = ( h3fc + hnc3 )


1
3
(12)
Note that the natural and forced convection effects are added since the buoyancy induced flow is
not counter to the forced flow (the pipe is horizontal). The surface temperature and self-heating
error considering both natural and forced convection effects are:

q
Ts ,mixed = T f + 3
(13)
⎛D⎞
hmixed 4π ⎜ ⎟
⎝2⎠

δ Tmixed = Ts ,mixed − T f (14)

h_mixed=(h_nc^3+h_fc^3)^(1/3) "mixed mode heat transfer coefficient"


T_s_mixed=T_f+q_dot/(4*pi*(D/2)^2*h_mixed) "surface temperature"
deltaT_mixed=T_s_mixed-T_f "self-heating error"

The self-heating error considering both natural and forced convection effects is shown in Fig. 1;
notice that it limits to the natural convection result at low values of Vf and to the forced
convection result at higher values of Vf.
PROBLEM 6.2-4 (6-3 in text): Heat loss from a solar collector
Figure P6.2-4 illustrates a flat plate solar collector that is mounted at an angle of τ = 45 degrees
on the roof of a house. The collector is used to heat water; a series of tubes are soldered to the
back-side of a black plate. The collector plate is contained in a case with a glass cover.

g T∞ = 20°C
ho = 150 W/m -K
2

H=1m

q′′s = 800 W/m


2

back plate
insulation
Tp = 80°C transparent glass cover
roof
tubes

τ = 45 degree L = 2 cm
Figure P6.2-4: Flat plate solar collector

Assume that the solar collector is H = 1 m wide by W = 1 m long (into the page) and the distance
between the heated plate and the glass covering is L = 2 cm. The collector receives a solar flux
q s′′ =800 W/m2 and the collector plate can be assumed to absorb all of the solar energy. The
collected energy is either transferred to the water in the pipe (in which case the energy is used to
provide useful water heating) or lost due to heat transfer with the environment (either by
radiation, which will be neglected in this problem, or convection). The collector plate
temperature is Tp = 80°C and the ambient temperature is T∞ = 20°C. The heat transfer
coefficient on the external surface of the glass is forced convection (there is a slight breeze) and
equal to ho = 15 W/m2-K. The glass is thin and can be neglected from the standpoint of
providing any thermal resistance between the plate and ambient. Neglect radiative effects.

a.) Determine the rate of heat loss from plate due to convection; you may assume that the
insulation on the back of the tubes is perfect so no heat is conducted to the roof and that
radiation from the plate is negligible.

The inputs are entered in EES:


$UnitSystem SI MASS RAD PA K J
$TABSTOPS 0.2 0.4 0.6 0.8 3.5 in
"Inputs"
tau=45 [degree]*convert(degree,rad) "tilt angle"
Hc=1 [m] "collector width"
W=1 [m] "collector length"
L=2 [cm]*convert(cm,m) "plate to glass spacing"
q``_dot_s=800 [W/m^2] "solar flux"
T_p=converttemp(C,K,80) "plate temperature"
T_amb=converttemp(C,K,20) "ambient temperature"
h_o=15 [W/m^2-K] "external convection coefficient"
The thermal resistance between the plate and the ambient air consists of the series combination
of natural convection through a rectangular enclosure (Renc) and forced convection from the
external surface of the glass (Rconv,o); conduction through the glass is neglected as small in
comparison to these other resistances. The external convection coefficient is given and therefore
Rconv,o can be computed:

1
Rconv ,o = (1)
H W ho

R_conv_o=1/(h_o*Hc*W) "resistance to convection from the external surface"

In order to determine the natural convection heat transfer coefficient for the enclosure it is
necessary to determine the air properties and Rayleigh number that characterizes the problem.
The cold surface temperature (Ts, the temperature of the internal surface of the glass) is not
known apriori; therefore, a reasonable value is guessed and the film temperature is used to obtain
the air properties (ρ, μ, k, β, c, α, ν, and Pr) .

T_s=converttemp(C,K,30) "a 1st guess for the temperature of the internal surface of the glass"
T_film=(T_s+T_amb)/2 "film temperature"
rho=density(Air,T=T_film,P=1 [atm]*convert(atm,Pa)) "density"
mu=viscosity(Air,T=T_film) "viscosity"
k=conductivity(Air,T=T_film) "conductivity"
beta=VolExpCoef(Air,T=T_film) "volumetric thermal expansion coefficient"
c=cP(Air,T=T_film) "specific heat capacity"
alpha=k/(rho*c) "thermal diffusivity"
nu=mu/rho "kinematic viscosity"
Pr=nu/alpha "Prandtl number"

The Rayleigh number for an enclosure is based on the distance between the heated and cooled
surfaces (i.e., the plate and the glass):

g β L3 (Tp − Ts )
RaL = (2)
αυ

The Rayleigh number and tilt are used to call the procedure Tilted_Rect_Enclosure_ND which
returns the Nusselt number, NuL. The Nusselt number is used to compute the heat transfer
coefficient related to natural convection (henc):

henc L
NuL = (3)
k

Ra=beta*g#*L^3*(T_p-T_s)/(nu*alpha) "Raleigh number"


Call Tilted_Rect_Enclosure_ND(Ra, tau, Hc/L : Nusselt) "access correlations"
Nusselt=h_enc*L/k "compute the heat transfer coefficient"

The thermal resistance associated with the enclosure can be computed:


1
Renc = (4)
H W henc

The rate of convective heat loss is:

Tp − Tamb
qconv = (5)
Renc + Rconv ,o

R_enc=1/(h_enc*W*Hc) "thermal resistance associated with natural convection in the enclosure"


q_dot_conv=(T_p-T_amb)/(R_enc+R_conv_o) "rate of convective heat loss"

The solution is obtained and the guess values for the program are updated (Update Guess values
from the Calculate menu). The assumed surface temperature is commented out and the surface
temperature is computed:

Ts = Tp − Renc qconv (6)

{T_s=converttemp(C,K,30) "a 1st guess for the temperature of the internal surface of the glass"}
T_s=T_p-R_enc*q_dot_conv "calculated surface temperature"

The predicted rate of heat loss is 179 W.

b.) What is the efficiency of the solar collector, ηcollector, defined as the ratio of the energy
delivered to the water to the energy received from the sun?

The energy received from the sun is:

qs = q s′′W H (7)

The energy delivered to the water is:

qwater = qs − qconv (8)

and the efficiency is therefore:

qwater
ηcollector = (9)
qs
q_dot_s=q``_dot_s*W*Hc "rate of energy provided by solar flux"
q_dot_water=q_dot_s-q_dot_conv "rate of energy delivered to water"
eta_collector=q_dot_water/q_dot_s "collector efficiency"

The predicted collector efficiency is ηcollector = 0.776.


c.) Prepare a plot showing the collector efficiency as a function of the plate to glass spacing, L.
Explain the shape of the plot.

The value of L is commented out and a parametric table is generated that contains L and ηcollector.
(If convergence difficulties are encountered, try solving the table in reverse order. A check box
in the Solve Table dialog is provided for this purpose.) The results are plotted in Fig. 2 which
illustrates the collector efficiency as a function of the plate to collector spacing.
0.9

0.8
Collector efficiency

0.7

0.6

0.5

0.4
0 0.5 1 1.5 2 2.5 3 3.5 4 4.5 5
Plate to glass spacing (cm)
Figure 2: Collector efficiency as a function plate to glass spacing

The initial rise in collector efficiency with L is related to the increased conduction resistance of
the stagnant air gap. Around L = 0.75 cm, the viscous force is overcome by the buoyancy force
and therefore convective cells are formed so that the convective heat loss actually increases with
L leading to a reduction in collector efficiency.
PROBLEM 6.2-5 (6-4 in text): Heat loss from a window
Figure P6.2-5 illustrates a single-paned glass window that is L = 6 ft high and W = 4 ft wide; the
glass is thg = 0.25 inch thick and has conductivity kg = 1.4 W/m-K.

thg = 0.25 inch


Tair ,in = 70°F L = 6 ft

kg = 1.4 W/m-K Tair ,out = 10°F

Figure P6.2-5: Single-paned glass window.

On a typical winter day, the outdoor temperature is Tair,out =10°F and you keep the indoor
temperature at Tair,in = 70°F.

a.) On a still winter day, estimate the rate of heat loss from the window.

The inputs are entered in EES:


$UnitSystem SI MASS RAD PA K J
$TABSTOPS 0.2 0.4 0.6 0.8 3.5 in

"Inputs"
L=6 [ft]*convert(ft,m) "height of window"
W=4 [ft]*convert(ft,m) "width of window"
T_air_out=converttemp(F,K,10) "outside air temperature"
T_air_in=converttemp(F,K,70) "inside air temperature"
t_g=0.25 [inch]*convert(inch,m) "thickness of glass"
k_g=1.4 [W/m-K] "conductivity of glass"

The thermal resistance network that connects the outdoor air to the indoor air is shown in Fig. 2
and includes a convection resistance between the inner surface of the window and the indoor air
(Rconv,in), a conduction resistance across the gas (Rcond), and a convection resistance between the
outer surface of the window and the outdoor air (Rconv,out).

Figure 2: Resistance network.

The conduction resistance is:

tg
Rcond = (1)
k g LW
R_cond=t_g/(k_g*L*W) "glass conduction resistance"

The convection resistances are:

1
Rconv ,in = (2)
hin LW

1
Rconv ,out = (3)
hout LW

where hin and hout are the average heat transfer coefficients on the internal and external surfaces,
respectively. These are obtained by solving the corresponding external, natural convection
problem for a vertical flat plate. The properties of the external air (ρair,out, kair,out, μair,out, cair,out,
βair,out) are obtained at the film temperature between the external surface and the outside air
(Tfilm,out):

Tair ,out + Tg ,out


T film ,out = (4)
2

where Tg,out is the temperature on the external surface of the glass (see Fig. 2) which is not
known apriori. A reasonable guess is used to start the problem; this value will be adjusted to
finish the problem. The properties of air within the house (ρair,in, kair,in, μair,in, cair,in, βair,in) are
obtained in the same way.

"Outside air properties"


T_g_out=(T_air_out+T_air_in)/2
"an initial guess for the outside surface temperature"
T_film_out=(T_g_out+T_air_out)/2 "film temperature on the outside surface"
rho_air_out=density(Air,T=T_film_out,P=1 [atm]*convert(atm,Pa)) "density"
mu_air_out=viscosity(Air,T=T_film_out) "viscosity"
k_air_out=conductivity(Air,T=T_film_out) "conductivity"
c_air_out=cP(Air,T=T_film_out) "specific heat capacity"
beta_air_out=VolExpCoef(Air,T=T_film_out) "volumetric expansion coefficient"

"Inside air properties"


T_g_in=(T_air_out+T_air_in)/2 "an initial guess for the inside surface temperature"
T_film_in=(T_g_in+T_air_out)/2 "film temperature on the inside surface"
rho_air_in=density(Air,T=T_film_in,P=1 [atm]*convert(atm,Pa)) "density"
mu_air_in=viscosity(Air,T=T_film_in) "viscosity"
k_air_in=conductivity(Air,T=T_film_in) "conductivity"
c_air_in=cP(Air,T=T_film_in) "specific heat capacity"
beta_air_in=VolExpCoef(Air,T=T_film_in) "volumetric expansion coefficient"

The Raleigh number for the external surface (RaL,out) is defined as:

β air ,out g L3 (Tg ,out − Tair ,out )


RaL ,out = (5)
α air ,out ν air ,out
where αair,out and νair,out are the thermal diffusivity and kinematic viscosity of the outside air and
g is the acceleration of gravity. The average Nusselt number for the outside surface of the glass
( Nu L ,out ) is obtained by accessing the EES function FC_plate_vertical_ND with the calculated
Raleigh number and the Prandtl number of the outside air (Prair,out). The Nusselt number is used
to compute the average heat transfer coefficient on the external surface:

Nu L ,out kair ,out


hout = (6)
L

A similar process is used to determine hin ; note that the absolute value of the surface to ambient
temperature difference is used.

"Outside air heat transfer coefficient"


alpha_air_out=k_air_out/(rho_air_out*c_air_out) "thermal diffusivity"
nu_air_out=mu_air_out/rho_air_out "kinematic viscosity"
Pr_air_out=nu_air_out/alpha_air_out "Prandtl number"
Ra_L_out=beta_air_out*g#*L^3*(T_g_out-T_air_out)/(alpha_air_out*nu_air_out)
"Raleigh number"
Call FC_plate_vertical_ND(Ra_L_out, Pr_air_out: Nusselt_out) "access vertical flat plate correlations"
h_bar_out=Nusselt_out*k_air_out/L "average heat transfer coefficient"

"Inside air heat transfer coefficient"


alpha_air_in=k_air_in/(rho_air_in*c_air_in) "thermal diffusivity"
nu_air_in=mu_air_in/rho_air_in "kinematic viscosity"
Pr_air_in=nu_air_in/alpha_air_in "Prandtl number"
Ra_L_in=beta_air_in*g#*L^3*abs(T_g_in-T_air_in)/(alpha_air_in*nu_air_in)
"Raleigh number"
Call FC_plate_vertical_ND(Ra_L_in, Pr_air_in: Nusselt_in) "access vertical flat plate correlations"
h_bar_in=Nusselt_in*k_air_in/L "average heat transfer coefficient"

The resistances to internal and external convection may be computed using Eqs. (2) and (3). The
total heat transfer through the window is therefore:

Tair ,in − Tair ,out


q = (7)
Rconv ,in + Rcond + Rconv ,out

R_conv_in=1/(h_bar_in*L*W) "internal convection resistance"


R_conv_out=1/(h_bar_out*L*W) "external convection resistance"
q_dot=(T_air_in-T_air_out)/(R_conv_in+R_cond+R_conv_out) "heat transfer rate"

The guess values used by EES to start the solution process should be updated (select Update
Guesses from the Calculate menu) and the surface temperatures of the glass calculated:

Tg ,in = Tair ,in − q Rconv ,in (8)

Tg ,out = Tair ,out + q Rconv ,out (9)


and the original guesses are removed:
{T_g_out=(T_air_out+T_air_in)/2 "an initial guess for the outside surface temperature"}
{T_g_in=(T_air_out+T_air_in)/2 "an initial guess for the inside surface temperature"}
T_g_in=T_air_in-q_dot*R_conv_in "internal glass temperature"
T_g_out=T_air_out+q_dot*R_conv_out "external glass temperature"

The result is q = 118 W.

b.) Winter lasts twinter= 90 days and you are heating with electrical resistance heaters. Electricity
costs ecost = $0.12/kW-hr. How much does the heat loss through the window cost you over
the course of 1 winter?

The money required to heat the house is the product of the rate of heat loss, the time associated
with winter, and the cost of the energy (electricity).

Cost = q twinter ecost (10)

t_winter=90 [day]*convert(day,s) "time associated with a winter"


e_cost=0.12 [$/kW-hr]*convert($/kW-hr,$/J) "cost of electricity"
Cost=t_winter*q_dot*e_cost "total heating cost"

The cost associated with 1 window is $30.63 for a winter.

c.) About 40% of your home's heat loss will occur through its windows; more if you have single
paned windows like the one shown in Fig. 1. Assume that 50% of your heat loss in your
house is through your windows and that you have Nwindow = 10 single paned windows in your
house. Prepare a plot showing the cost of heating your house as a function of the thermostat
set point (i.e., the indoor air temperature).

The total cost is twice the product of the number of windows and the cost per window.

Totalcost = 2 N windows Cost (11)

N_window=10 "number of windows"


TotalCost=2*N_window*Cost "total house heating cost"

The indoor air temperature is commented out and varied in a parametric table; the total cost as a
function of outdoor air temperature is shown in Fig. 3.
Figure 3: Total heating cost as a function of the indoor air temperature.
Problem 6.2-6 (6-5 in text): Double-glazed window
The single-glazed window in problem 6.2-5 is replaced with a double-glazed window. Both
glass panes are 0.25 inch thick and the gap between the panes is 0.5 inch. The gap contains dry
air at atmospheric pressure. All other information is the same as in problem 6.2-5. Neglect heat
transfer by radiation.

a.) Repeat the calculations requested in parts (a) and (b) of problem 6.2-5.

The problem specifications are entered into EES.


$UnitSystem SI MASS RAD PA K J
$TABSTOPS 0.2 0.4 0.6 0.8 3.5 in

"Inputs"
T_air_in_F=70 [F] "outdoor air temperature"
L=6 [ft]*convert(ft,m) "height of window"
W=4 [ft]*convert(ft,m) "width of window"
T_air_out=converttemp(F,K,10) "outside air temperature"
T_air_in=converttemp(F,K,T_air_in_F) "inside air temperature"
t_g=0.25 [inch]*convert(inch,m) "thickness of glass"
k_g=1.4 [W/m-K] "conductivity of glass"
delta=0.5 [in]*convert(in,m) "width of air gap"

The conductive resistance of a glass pane is calculated.


R_cond=t_g/(k_g*L*W) "glass conduction resistance per pane"

The heat transfer coefficient on the outside surface of the outer glass is calculated first. This
calculation requires knowledge of the outside glass surface temperature which is not yet known.
A guess is entered and it will be removed later. Using this guess, the film temperature is
calculated and used to evaluate properties of the air. The Rayleigh and number is calculated and
used with the correlation described in section 6.2.2 (implemented in EES library procedure
FC_plate_vertical_ND) to calculate the Nusselt number for the outside surface of the window,
and thus the heat transfer coefficient for this surface.
"Outside air properties"
T_g2_out=(T_air_out+T_air_in)/2 "an initial guess for the outside surface temperature"
T_film_out=(T_g2_out+T_air_out)/2 "film temperature on the outside surface"
rho_air_out=density(Air,T=T_film_out,P=1 [atm]*convert(atm,Pa)) "density"
mu_air_out=viscosity(Air,T=T_film_out) "viscosity"
k_air_out=conductivity(Air,T=T_film_out) "conductivity"
c_air_out=cP(Air,T=T_film_out) "specific heat capacity"
beta_air_out=VolExpCoef(Air,T=T_film_out) "volumetric expansion coefficient"

"Outside air heat transfer coefficient"


alpha_air_out=k_air_out/(rho_air_out*c_air_out) "thermal diffusivity"
nu_air_out=mu_air_out/rho_air_out "kinematic viscosity"
Pr_air_out=nu_air_out/alpha_air_out "Prandtl number"
Ra_L_out=beta_air_out*g#*L^3*(T_g2_out-T_air_out)/(alpha_air_out*nu_air_out) "Raleigh number"
Call FC_plate_vertical_ND(Ra_L_out, Pr_air_out: Nusselt_out) "access vertical flat plate correlations"
h_bar_out=Nusselt_out*k_air_out/L "average heat transfer coefficient"

The free convection heat transfer coefficient on the indoor side of the window is calculated next.
Properties need to be estimated at the film temperature, which is the average of the indoor
temperature and the inside glass surface temperature, T_g1_in. A guess is provided for the
inside glass temperature, which will later be removed. The properties of air are evaluated at the
film temperature and used with a free convection relation in Section 6.2.2 (implemented in EES
library procedure FC_plate_vertical_ND) to evaluate the inside heat transfer coefficient.
"Inside air properties"
T_g1_in=T_air_in-5 [K] "an initial guess for the inside surface temperature"
T_film_in=(T_g1_in+T_air_out)/2 "film temperature on the inside surface"
rho_air_in=density(Air,T=T_film_in,P=1 [atm]*convert(atm,Pa)) "density"
mu_air_in=viscosity(Air,T=T_film_in) "viscosity"
k_air_in=conductivity(Air,T=T_film_in) "conductivity"
c_air_in=cP(Air,T=T_film_in) "specific heat capacity"
beta_air_in=VolExpCoef(Air,T=T_film_in) "volumetric expansion coefficient"

"Inside air heat transfer coefficient"


alpha_air_in=k_air_in/(rho_air_in*c_air_in) "thermal diffusivity"
nu_air_in=mu_air_in/rho_air_in "kinematic viscosity"
Pr_air_in=nu_air_in/alpha_air_in "Prandtl number"
Ra_L_in=abs(beta_air_in*g#*L^3*(T_air_in-T_g1_in)/(alpha_air_in*nu_air_in)) "Raleigh number"
Call FC_plate_vertical_ND(Ra_L_in, Pr_air_in: Nusselt_in) "access vertical flat plate correlations"
h_bar_in=Nusselt_in*k_air_in/L "average heat transfer coefficient"

Next it is necessary to determine the heat transfer coefficient of the air gap. The temperatures of
the glass surfaces forming the enclosure, T_g1_out and T_g2_in, are needed to evaluate the heat
transfer coefficient, but they are not yet know. Guess values will be again supplied and later
removed.
T_g1_out=(3*T_air_in+T_air_out)/4 "an initial guess for the outside temperature of the inner glass"
T_g2_in=(3*T_air_out+T_air_in)/4 "an initial guess for the insider temperature of the outer glass"

With the temperatures at hand, the average air temperature in the gap can be calculated and used
to determine air properties. These problems are used to calculate the Rayleigh number in the gap
which is used with the Tilted_Rect_Enclosure_ND procedure to evaluate the Nusselt number for
the gap, and thus the heat transfer coefficient.
"Heat transfer coefficient through the air gap"
T_gap=(T_g1_out+T_g2_in)/2 "air gap temperature "
rho_air_gap=density(Air,T=T_gap,P=1 [atm]*convert(atm,Pa)) "density"
mu_air_gap=viscosity(Air,T=T_gap) "viscosity"
k_air_gap=conductivity(Air,T=T_gap) "conductivity"
c_air_gap=cP(Air,T=T_gap) "specific heat capacity"
beta_air_gap=VolExpCoef(Air,T=T_gap) "volumetric expansion coefficient"
alpha_air_gap=k_air_gap/(rho_air_gap*c_air_gap) "thermal diffusivity"
nu_air_gap=mu_air_gap/rho_air_gap "kinematic viscosity"
HoverL=L/delta "height of window divided by gap thickness"
Tau=pi/2 [rad] "tilt"
Ra_gap=g#*Delta^3*beta_air_gap*(T_g1_out-T_g2_in)/(nu_air_gap*alpha_air_gap) "Rayleigh number
for gap"
Call Tilted_Rect_Enclosure_ND(Ra_gap, tau, HoverL : Nusselt_gap) "Nusselt number for heat transfer in
gap"
Nusselt_gap=h_gap*delta/k_air_gap "heat transfer coefficient for gap"

All of the heat transfer coefficients are known (based on the estimated temperatures shown in
yellow). Thermal resistances for convection on the inside surface of the window, convection in
the gap, and convection on the outside surface of the window can be evaluated.
R_gap=1/(h_gap*L*W) "resistance of gap"
R_conv_in=1/(h_bar_in*L*W) "internal convection resistance"
R_conv_out=1/(h_bar_out*L*W) "external convection resistance"

The total resistance to heat transfer is the sum of the three resistances above plus the conduction
resistance of two glazings. The heat transfer rate across the window is the difference between
the indoor and outdoor air temperatures divided by the total thermal resistance.
q_dot=(T_air_in-T_air_out)/(R_conv_in+2*R_cond+R_gap+R_conv_out) "heat transfer rate"

Four temperatures were assumed to get to this point. These temperatures can now be calculated
by recognizing that this is a steady-state problem so that the heat transfer rate through each
component in the window is the same. Before proceeding, solve and update the guess values.
Then remove or comment out the equations that provided guesses that are highlighted. Enter the
following relations.
T_g1_in=T_air_in-q_dot*R_conv_in "inside temperature of internal glass"
T_g1_in-T_g1_out=q_dot*R_cond "determines outside temperature of internal glass"
T_g2_out=T_air_out+q_dot*R_conv_out "outside temperature of external glass "
T_g2_in=T_g2_out+q_dot*R_cond "inside temperature of external glass"

Solving results in a heat transfer rate of 54.7 W.

The cost of this heat loss using the economic information from problem 6.2.5 can be calculated.
t_winter=90 [day]*convert(day,s) "time associated with a winter"
e_cost=0.12 [$/kW-hr]*convert($/kW-hr,$/J) "cost of electricity"
Cost=t_winter*q_dot*e_cost "total heating cost"
N_window=10 "number of windows"
TotalCost=2*N_window*Cost "total house heating cost"

b.) Summarize and explain the benefits of the double-glazed window.

The benefits are directly understood in terms of the costs. The cost with the double glazed
window calculates to be $14.18 per window or 283.6 for the building during the 90 days. With
single glazed windows, the costs from problem 6.2-5 were $30.6 per window and 612.6 for the
building during the same period.

It is noted that the conduction resistance of the glass is very small and it could have been
neglected with no loss in accuracy.
Problem 6.2-7 (6-6 in text): Double-glazed window with inert gas
You have seen an advertisement for argon-filled windows. These windows are similar in
construction to the window described in problem 6.2-6, except that argon, rather than air, is
contained in the gap. Neglect heat transfer by radiation.

a.) Repeat problem 6.2-6 assuming that the gap contains argon.

This solution procedure for this problem is exactly the same as for problem 6.2-6 except for the
calculating the heat transfer coefficient in the gap between the windows. Rather than air, a gas
identified by string variable F$ is used as the gas.
"Heat transfer coefficient through the air gap"
$ifnot ParametricTable
F$='Ar'
$endif

The average temperature of the gas in the gap is determined and used to evaluate its properties.
T_gap=(T_g1_out+T_g2_in)/2 "air gap temperature "
rho_gap=density(F$,T=T_gap,P=1 [atm]*convert(atm,Pa)) "density"
mu_gap=viscosity(F$,T=T_gap) "viscosity"
k_gap=conductivity(F$,T=T_gap) "conductivity"
c_gap=cP(F$,T=T_gap) "specific heat capacity"
beta_gap=VolExpCoef(F$,T=T_gap) "volumetric expansion coefficient"
alpha_gap=k_gap/(rho_gap*c_gap) "thermal diffusivity"
nu_gap=mu_gap/rho_gap "kinematic viscosity"

The free convection relation of Hollands et al. (section 6.2.6) is applied using the
Tilted_Rect_Enclosure procedure.
HoverL=L/delta "height of window divided by gap
thickness"
Tau=pi/2 [rad] "tilt"
Ra_gap=g#*Delta^3*beta_gap*(T_g1_out-T_g2_in)/(nu_gap*alpha_gap) "Rayleigh number for gap"
Call Tilted_Rect_Enclosure_ND(Ra_gap, tau, HoverL : Nusselt_gap) "Nusselt number for heat transfer in
gap"
Nusselt_gap=h_gap*delta/k_gap "heat transfer coefficient for gap"

A Parametric table is constructed with columns for F$ (the gas in the gap), the rate of heat
transfer through the window, and the heating cost per window. Also, to help answer parts b and
c, the thermal conductivity, Rayleigh number and heat transfer coefficient are shown in the table
for the difference gases. Solving produces the interesting results shown in the table.
b.) Are the claims that argon reduces heat loss valid? If so, why does this behavior occur?

The data in the table support the claim that argon-filled windows will reduce heat loss. The
Rayleigh number for argon is higher than for air, but it is still low enough such that limiting
behavior is observed so the Nusselt numbers are about the same. The thermal conductivity of
argon is lower than that of air. With the same Nusselt numbers, that results in a lower heat
transfer coefficient for argon

c.)Would nitrogen (which is cheaper) work as well? Why or why not? Can you suggest another
gas that would work better than argon?

From the data in the table it appears that carbon dioxide would also reduce convective losses in
the gap almost as well as argon.
Problem 6.2-8 (6-7 in text): Solar Collector Design
You are involved in a project to design a solar collector for heating air. Two competing designs
are shown in Figure P6.2-8.
2 2
200 W/m to 800 W/m
5°C, 5 m/s
glass cover plate
air at 25°C, 100 kPa
2
10 liter/s/m of collector 20 mm

black collector plate


2.5 m
(a)
2 2
5°C, 5 m/s 200 W/m to 800 W/m

glass cover plate

black collector plate 20 mm


air at 25°C, 100 kPa 20 mm
2
10 liter/s/m of collector

2.5 m
(b)
Figure P6.2-8: Air heating solar collector with (a) air flowing above the collector plate and (b) air flowing
below the collector plate.

Both designs employ a transparent glass cover plate and a thin metal opaque black collector plate
upon which solar radiation is completely absorbed. The glazing is standard safety glass with a
thickness of 6 mm. In the first design, shown in Figure P6.2-8(a), air is blown through the gap
between the cover and plate. In the second design, shown in Figure P6.2-8(b), the air flows in a
second gap that is below the collector plate and free convection occurs between the collector
plate and the glass cover plate. The collector is 1 meter wide (into the page) and 2.5 m long (in
the air flow direction) and oriented horizontally. In both designs, the gaps are 20 mm wide. Air
at 25°C, 100 kPa enters the flow passage at a flow rate of 10 liters/sec per square meter of
collector area (area exposed to solar radiation) in both cases. The outdoor temperature (above
the glass cover plate) is 5°C and there is a wind that may be represented as a forced convective
flow with a free stream velocity of 5 m/s in the flow direction. Calculate and plot the efficiency
of the two collector designs as a function of the solar radiation absorbed on the plate for values
between 200 and 800 W/m2. Assume that the insulation is adiabatic and neglect radiation in
these calculations.

Known problem specifications can be entered into EES. The absorbed solar radiation is set to
600 W/m2 but will later be varied in a parametric table.
$UnitSystem SI K Pa J rad
$tabstops 0.25 4 in

"known"
thk=6*convert(mm,m) "glass thickness"
V_dot=10 [l/s-m^2]*convert(l,m^3) "volumetric flow rate"
T_air_in=convertTemp(C,K,25 [C]) "temperature of entering air"
P_atm=100 [kPa]*convert(kPa,Pa) "atmospheric pressure"
rho_in=density(Air,T=T_air_in,P=P_atm) "density of entering air"
T_amb=convertTemp(C,K,5 [C]) "outdoor temperature"
Wind=5 [m/s] "wind velocity"
W=1 [m] "width of collector"
delta=20 [mm]*convert(mm,m) "gap width"
L=2.5 [m] "length of collector"
A=L*W "collector area"
m_dot=V_dot*rho_in*A "air mass flow rate"
$ifnot ParametricTable
S=600 [W/m^2] "absorbed solar radiation"
$endif
RelRough=0 "assumed smooth"
tau=0 [rad]

The thermal conductivity and thermal resistance for the glass are found. The Solid-Liquid
Property library provides the thermal conductivity.
k_glass=k_('Glass-soda lime', T=300 [K]) "thermal conductivity of glass at 300 [K]"
R_g=thk/(k_glass*A) "thermal resistance of the glass"

The geometry for case a is analyzed first. An average temperature is defined for determining
properties. The average temperature is the average of the entering and exiting air, but the outlet
temperature of the air is not yet known. A guess value is provided to get started. It will later be
removed.

The heat transfer coefficient in the gap is found using the DuctFlow procedure that implements
the correlation described in section 5.2.4.
T_air_out_a=T_air_in+20 [K] "guess for the air outlet temperature"
T_air_avg_a=(T_air_in+T_air_out_a)/2 "average air temperature"
call DuctFlow('air',T_air_avg_a,P_atm,m_dot, delta ,W,L,RelRough:h_T, h_H ,DELTAP, Nusselt_T, f, Re)
"heat transfer coefficient in flow gap"

The heat transfer coefficient to the plate is assumed to be the same as to the glazing. However,
the plate temperature is not known, so a guess will be provided temporarily to allow the rate of
heat transfer from the plate to the air to be calculated.
T_plate=T_air_in+50 [K] "guess for the plate temperature"
q_dot_air_a=A*h_H*(T_plate-T_air_avg_a) "heat transfer from plate to air"

Since the problem is steady state, the rate of heat transfer to the glazing is equal to the rate of
heat transfer by conduction through the glazing and the rate of heat loss by convection from the
outside surface of the glazing. To get this calculation started, we will assume a temperature for
the inside surface of the glass and later remove it.
T_g_in=T_air_in+10 [K] "guess for the inside glass surface temperature"
q_dot_loss_a=A*h_H*(T_air_avg_a-T_g_in) "heat transfer rate from air to glazing"
q_dot_loss_a =(T_g_in-T_g_out_a)/R_g "rate of conduction through glazing"

The heat transfer coefficient from the outside surface of the glass is found using an external flow
convection relation, External_Flow_Plate. The heat transfer resistance on the glass surface can
then be found.
Call External_Flow_Plate('Air', T_amb, T_g_out_a, P_atm, Wind, L: xx, h_wind_a, C_f, Nusselt_wind,
Re_wind)
R_wind_a=1/(h_wind_a*A) "resistance on top surface of glass"

Solve and update the guesses. Then comment out the guess for the inside glass surface
temperature and enter the relation for convection from the top surface of the glass.
q_dot_loss_a=(T_g_out_a-T_amb)/R_wind_a "rate of energy transfer from top of glass to
surroundings"

Solve and update the guesses. Energy balances on the collector and the airdetermines the plate
and air outlet temperatures. Comment out the guesses made for T_air_out and T_plate and enter
the energy balances.
m_dot*(enthalpy(Air,T=T_air_out_a)-enthalpy(Air,T=T_air_in))=q_dot_air_a-q_dot_loss_a "energy
balance on air"
S*A=q_dot_loss_a+m_dot*(enthalpy(Air,T=T_air_out_a)-enthalpy(Air,T=T_air_in)) "energy balance on
collector"

The collector efficiency is the ratio of the energy transferred to the air to the solar energy
absorbed on the plate. The efficiency is about 0.73.
eta_a=m_dot*(enthalpy(Air,T=T_air_out_a)-enthalpy(Air,T=T_air_in))/(S*A) "collector efficiency for
design a"

A similar analysis is conducted for the design shown in case b of the figure. In this case, the air
flows below the plate and there is a stagnant air gap above the plate. To start, the heat transfer
coefficient on the outside surface of the glass is calculated. A guess for the glass surface
temperature is provided and later removed.
T_g_out_b=T_amb+5 [K] "guess for the outside glass surface temperature"
Call External_Flow_Plate('Air', T_amb, T_g_out_b, P_atm, Wind, L: xxb, h_wind_b, C_fb, Nusselt_windb,
Re_wind_b)
R_wind_b=1/(h_wind_b*A) "resistance on top surface of glass"
q_dot_loss_b=(T_g_out_b-T_amb)/R_wind_b "rate of energy transfer from top of glass to
surroundings"
q_dot_loss_b=(T_g_in_b-T_g_out_b)/R_g "rate of energy transfer through glass"

The heat transfer coefficient in the enclosure is calculated using the Tilted_Rect_Enclosure
procedure in section 6.2.6. An estimate of the plate temperature is provided and later removed.
T_plate_b=T_air_in+50 [K] "guess for the plate temperature"
Call Tilted_Rect_Enclosure('Air', T_plate_b, T_g_in_b, P_atm, L, delta, tau : h_air_gap, Nusselt_gap, Ra)

Solve and update the guesses. The rate of heat transfer through the gap can be determines using
the heat transfer coefficient. Comment out the guess for T_g_out_b and then enter the heat
transfer relation
q_dot_loss_b=A*h_air_gap*(T_plate_b-T_g_in_b) "rate of energy transfer through gap"

Enter a guess for T_air_out_b so that the average air temperature can be calculated and used to
calculate the heat transfer rate to the air.
T_air_out_b=T_air_in+20 [K] "guess for the air outlet temperature"
T_air_avg_b=(T_air_in+T_air_out_b)/2 "average air temperature in gap"
q_dot_air_b=A*h_H*(T_plate-T_air_avg_b) "rate of energy transfer to air"
Solve and update the guess values. Comment out the guesses made for T_air_avg_b and
T_plate_b and enter the following energy balances. The efficiency can also be calculated.

The efficiency of the solar collector in figure b is 0.936 which is significantly better than that for
figure a. The insulation provided by the stagnant air gap is the reason for the improvement.
Problem 6.3-2 (6.9 in text): Self-similar Solution for a Vertical Plate
A self-similar solution can be obtained for the free convection problem where a heated vertical
plate has a surface temperature (Ts) that varies with position according to: Ts − T∞ = A x n where x
is measured from the bottom of the plate.
a.) Transform the governing partial differential equations for momentum conservation in the
(vertical) x-direction and thermal energy conservation into ordinary differential equations for
f and θ .

The plate temperature is given by:

Ts − T∞ = A x n (1)

The governing equations are presented in Section 6.1.2. The continuity equation is:

∂u ∂v
+ =0 (2)
∂x ∂y

The x-momentum equation in the boundary layer, including the buoyancy term, which is
represented using the Boussinesq approximation, is:

∂u ∂u ∂ 2u
u +v = g β (T − T∞ ) + υ 2 (3)
∂x ∂y ∂y

Note that x is in the vertical direction as shown in Figure 6-20. The thermal energy equation is:

⎡ ∂T ∂T ⎤ ∂ 2T
ρ c ⎢u +v ⎥ = k 2 (4)
⎣ ∂x ∂y ⎦ ∂y

The similar parameter, is defined in Eq. (6-122)


1
y ⎡ Gr ⎤ 4
η= ⎢ x⎥ (5)
x⎣ 4 ⎦

It is anticipated that the dimensionless velocity and temperature difference:

u
u = (6)
uchar ,nc

T − T∞
θ = (7)
Ts − T∞
at any position x will collapse when expressed in terms of η, as shown in Figure 6-20:

u = u ( x, y ) = u (η ) (8)

θ = θ ( x, y ) = θ (η ) (9)

where the local characteristic velocity, uchar,nc, is:

uchar , nc = g x β (Ts − T∞ ) (10)

The stream function is defined so that the continuity equation, Eq.(2), is automatically satisfied:

⎛∂Ψ ⎞
u =⎜ ⎟ (11)
⎝ ∂ y ⎠x

⎛∂Ψ ⎞
v = −⎜ ⎟ (12)
⎝ ∂ x ⎠y

The stream function is related to the volumetric flow between the surface of the plate and any
position y according to:

V = W Ψ (13)

where W is the width of the plate (into the page). The stream function is related to the similarity
parameter by Eq. (6-32).
1
⎛ Gr ⎞ 4
Ψ = 4υ ⎜ x ⎟ f (η ) (14)
⎝ 4 ⎠

Equations (3) and (4) must be transformed from functions of x and y to a function of η. The
1
mathematical manipulations are facilitated by expressing the parameter Grx 4 / 4 , which appears
in the equations for both η and Ψ, in terms of x rather than Grx:

1 1 1
⎛ Grx ⎞ 4
⎛ g β (Ts − T∞ ) x 3 ⎞ 4
⎛ g β A x n x3 ⎞ 4
1 ( 3+ n )
⎜ ⎟ =⎜ ⎟ =⎜ ⎟ = B 4x 4
(15)
⎝ 4 ⎠ ⎝ 4υ 2 ⎠ ⎝ 4υ 2 ⎠

Substituting Eq.(15) into Eqs. (5) and (14) allows the similarity variables to be written as:

1 ( n −1)
η = yB 4 x 4
(16)
1 ( 3+ n )
Ψ = 4υ B 4
x 4
f (η ) (17)

where
gβ A
B= (18)
4υ 2

The x-velocity is expressed in terms of the similarity variables by substituting Eq. (17) into Eq.
(11):

⎛ ∂Ψ ⎞ ∂ ⎡ 1 ( 3+ n ) ⎤ 1 ( 3+ n ) ∂
u =⎜ ⎟ = ⎢ 4υ B x
4 4
f (η ) ⎥ = 4υ B 4 x 4 ⎡ f (η ) ⎤⎦ x (19)
⎝ ∂y ⎠ x ∂y ⎣ ⎦x ∂y ⎣


df ⎛ ∂η ⎞
⎜ ⎟
dη ⎝ ∂y ⎠ x

The partial derivative of η, Eq. (16) with respect to y at constant x is:

⎛ ∂η ⎞ 1 ( n −1)
⎟ =B x (20)
4 4

⎝ ∂y ⎠ x

Substituting Eq. (20) into Eq. (19) leads to:

1 ( 3+ n ) 1 ( n −1) df 1 ( n +1) df
u = 4υ B 4
x 4
B 4
x 4
= 4υ B 2 x 2 (21)
dη dη

The y-velocity is obtained by substituting Eq. (17) into Eq. (12):

∂ ⎡ ( 3+ n ) ⎤
f (η ( x, y ) ) ⎥
1
v=− ⎢ 4 υ B 4
x 4
(22)
∂x⎣ ⎦y

The similarity parameter, η is a function of x and y and, as a result, f is itself a function of x.


Applicationof the chain rule to Eq. (22) results in:

∂ ⎡ 1 ( 3+ n ) ⎤ 1 ( 3+ n ) df ⎛ ∂η ⎞
v=−f 4 υ B 4
x 4
− 4 υ B 4
x 4
⎜ ⎟ (23)
∂ x ⎢⎣ ⎥
⎦y dη ⎝ ∂x ⎠ y

Substituting Eq. (16) into Eq. (23) leads to:

∂ ⎡ 1 ( 3+ n ) ⎤ 1 ( 3+ n ) df ∂ ⎡ 1 ( n −1) ⎤
v=−f ⎢ 4 υ B 4 x 4 ⎥ − 4υ B 4 x 4
⎢ yB 4 x 4 ⎥ (24)
∂x⎣ ⎦y dη ∂x ⎣ ⎦

or
1 ( n −1) 1 ( n −1) df
v = − f ( 3 + n )υ B 4
x 4
− ( n − 1)υ B 2
x 2
y (25)

In addition to u and v, we will need the partial derivatives of u (see Eq. (3)). The partial
derivative of u with respect to x is:

∂u ∂ ⎡ 1 ( n +1) df ⎤ df ∂ ⎡ 1 ( n +1) ⎤ 1 ( n +1) d 2 f ⎛ ∂η ⎞


= ⎢ 4υ B 2 x 2 = 4 υ B 2
x 2
+ 4 υ B 2
x 2
⎜ ⎟ (26)
∂x ∂x ⎣ dη ⎥⎦ y dη ∂x ⎢⎣ ⎥
⎦y dη 2 ⎝ ∂x ⎠ y

Substituting Eq. (16) into Eq. (26) leads to:

∂u df ∂ ⎡ 1 ( n +1) ⎤ 1 ( n +1) d 2 f ∂ ⎡ 1 ( n −1) ⎤


= 4 υ B 2
x 2
+ 4 υ B 2
x 2
y B 4
x 4
(27)
∂x dη ∂x ⎢⎣ ⎥
⎦y dη 2
∂x ⎢


or

∂u df 1 ( n −1) 1 ( n +1) d 2 f 1 ( n −5)


= 2 ( n + 1)υ B 2 x 2 + υ B 2 x 2 y B 4
( n − 1) x 4
(28)
∂x dη dη 2

which can be rewritten as:

∂u df 1 ( n −1) 3 3 ( n −1)
d2 f
= 2 ( n + 1)υ B 2 x 2 + υ B 4 x 4
y ( n − 1) (29)
∂x dη dη 2

The partial derivative of u with respect to y is:

⎛ ∂u ⎞ ∂ ⎡ 1 ( n +1) df ⎤ 1 ( n +1) d 2 f ⎛ ∂η ⎞
⎜ ⎟ = ⎢ 4 υ B 2
x 2
⎥ = 4 υ B 2
x 2
⎜ ⎟ (30)
∂y
⎝ ⎠x ∂y ⎣ dη ⎦ dη 2 ⎝ ∂y ⎠ x

Substituting Eq. (16) into Eq. (30) leads to:

⎛ ∂u ⎞ ∂ ⎡ 1 ( n +1) df ⎤ 1 ( n +1) d 2 f ⎛ ∂η ⎞
⎜ ⎟ = ⎢ 4υ B x ⎥ = 4υ B x (31)
2 2 2 2
⎜ ⎟
∂y
⎝ ⎠x ∂y ⎣ dη ⎦ dη 2 ⎝ ∂y ⎠ x

⎛ ∂u ⎞ 1 ( n +1) d 2 f ∂ ⎡ 1 ( n −1) ⎤
⎜ ⎟ = 4 υ B 2
x 2
⎢ y B 4
x 4
⎥ (32)
⎝ ∂y ⎠ x dη ∂y ⎣
2

or

⎛ ∂u ⎞ 1 ( n +1) d 2 f 1 ( n −1)
⎜ ⎟ = 4 υ B 2
x 2
B 4
x 4
(33)
∂y
⎝ ⎠x dη 2
which can be rewritten as:

⎛ ∂u ⎞ 3 ( 3 n +1) d 2 f
⎜ ⎟ = 4 υ B 4
x 4
(34)
∂y
⎝ ⎠x dη 2

Finally, the second derivative of u with respect to y is:

∂ 2u ∂ ⎛ 3 ( 3 n +1) d 2 f ⎞ 3 ( 3 n +1) d 3 f ⎛ ∂η ⎞
= ⎜ 4 υ B 4
x 4
⎟ = 4 υ B 4
x 4
⎜ ⎟ (35)
∂y 2 ∂y ⎝ dη 2 ⎠ dη 3 ⎝ ∂y ⎠ x

Substituting Eq. (16) into Eq. (35) leads to:

∂ 2u 3 ( 3 n +1) d 3 f ∂ ⎡ 1 ( n −1) ⎤
= 4 υ B 4
x 4
y B 4
x 4
(36)
∂y 2 dη 3 ∂y ⎢⎣ ⎥

or

∂ 2u 3 ( 3 n +1) d 3 f 1 ( n −1)
= 4 υ B 4
x 4
B 4
x 4
(37)
∂y 2 dη 3

which can be rewritten as:

∂ 2u 3
n d f
= 4 υ B x (38)
∂y 2 dη 3

Substituting Eqs. (30), (34), and (38) into the x-momentum equation, Eq. (3), leads to:

∂u ∂u ∂ 2u
u +v = g β (T − T∞ ) + υ 2 (39)
∂x ∂y ∂y
1 ( n +1) df ⎡ df 1 ( n −1) 3 3 ( n −1)
d2 f ⎤
4υ B 2 x 2 ⎢ 2 ( n + 1) υ B 2
x 2
+ υ B 4
x 4
y ( n − 1) ⎥
dη ⎣ dη dη 2



u ∂u
∂x

⎡ 1 ( n −1) 1 ( n −1) df ⎤ 3 ( 3 n +1) d 2 f


+ ⎢ − f ( 3 + n )υ B 4 x 4 − ( n − 1)υ B 2 x 2 y ⎥ 4υ B 4 x 4
= (40)
⎣
⎦  dη dη 2

v ∂u
∂y

d3 f
g β (Ts − T∞ ) θ + υ 4υ B x n

dη 3
(T −T∞ ) 

∂ 2u
∂y 2

Expanding the terms and substituting the definition of B, Eq. (18), into the buoyancy term leads
to:

1 ( n +1) df df 1 ( n −1) 1 ( n +1) df 3 3 ( n −1)


d2 f
4υ B 2
x 2
2 ( n + 1)υ B 2 x 2 + 4υ B 2 x 2 υB 4 x 4
y ( n − 1)
dη dη dη dη 2
1 ( n −1) 3 ( 3 n +1) d2 f 1 ( n −1) df 3 ( 3 n +1) d 2 f
− f ( 3 + n )υ B 4
x 4
4υ B 4
x 4
− ( n − 1) υ B 2
x 2
y 4 υ B 4
x 4
= (41)
dη 2 dη dη 2
d3 f
4υ 2 B x n θ + 4υ 2 B x n
dη 3

Dividing Eq. (41)through by 4ν 2 B leads to:

2
⎛ df ⎞ d2 f d3 f
⎟ 2 ( n + 1) − f ( 3 + n ) 2 = θ + 3
 (42)

⎝ dη ⎠ dη dη

The differential equation for f is coupled to the differential equation for θ , which will be derived
from the thermal energy equation, Eq. (4). The first derivative of θ with respect to x is obtained
using the chain rule, recognizing that θ is a function only of η, which is a function of x and y:

∂  dθ ⎛ ∂ η ⎞
⎡⎣θ (η ( x, y ) ) ⎦⎤ = ⎜ ⎟ (43)
∂x dη ⎝ ∂ x ⎠ y

Substituting Eq. (16) into Eq. (43) leads to:

∂  dθ ∂ ⎡ 1 4 ( n −1) 4 ⎤
⎡⎣θ (η ( x, y ) ) ⎤⎦ = yB x (44)
∂x dη ∂ x ⎢⎣ ⎥

which can be written as:

∂θ dθ ( n − 1) 1 ( n −5)


= yB 4 x 4 (45)
∂x dη 4

The first derivative of θ with respect to y is obtained in a similar manner:

∂θ dθ ∂ ⎡ 1 ( n −1) ⎤


= y B 4
x 4
(46)
∂y dη ∂ y ⎢⎣ ⎥

or

∂θ dθ 1 4 ( n −1) 4


= B x (47)
∂y dη

The second derivative of θ with respect to y is:

∂ 2θ ∂ ⎛ ∂ θ ⎞ ∂ ⎡ ∂ θ ⎤ ⎛ ∂ η ⎞
= ⎜ ⎟ = ⎢ ⎥⎜ ⎟ (48)
∂y 2 ∂y ⎝ ∂ y ⎠ x ∂η ⎣ ∂ y ⎦ ⎝ ∂ y ⎠ x

Substituting Eq. (47) into Eq. (48) leads to:

∂ 2θ ∂ ⎡ dθ 1 4 ( n −1) 4 ⎤ ⎛ ∂ η ⎞


= ⎢ B x ⎥⎜ ⎟ (49)
∂y 2 ∂η ⎣ dη ⎦ ⎝ ∂ y ⎠x

or

∂ 2θ d 2θ 1 4 ( n −1) 4 ∂ ⎡ 1 4 ( n −1) 4 ⎤


= B x yB x (50)
∂y 2 dη 2 ∂y ⎢⎣ ⎥

∂ 2θ d 2θ 1 2 ( n −1) 2


= B x (51)
∂y 2 dη 2

Subsituting the definition of θ , Eq. (7), into the thermal energy equation, Eq. (4), leads to:

∂θ ∂θ ∂ 2θ


u +v =α 2 (52)
∂x ∂y ∂y

Substituting Eqs. (21), (45), (25), (47), and (51) into Eq. (52) leads to:
1 ( n +1) df dθ ( n − 1) 1 ( n −5 )
4υ B 2 x 2 yB 4 x 4
+
dη dη 4



u ∂θ
∂x

⎡ 1 ( n −1) 1 ( n −1) df ⎤ dθ 1 ( n −1)


⎢ − f ( 3 + n ) υ B 4
x 4
− ( n − 1) υ B 2
x 2
y⎥ B 4x 4 = (53)
⎣
⎦  dη dη

v ∂θ
∂y

d 2θ 1 2 ( n −1) 2
α B x
dη 2


∂ 2θ
α
∂y 2

Expanding the terms in Eq. (53)leads to:

1 df dθ
( n +1) 1 ( n −5) 1 ( n −1) dθ 1 ( n −1)
υB 2x 2
( n − 1) y B 4 x 4 − f ( 3 + n )υ B 4 x 4 B 4 x 4
dη dη dη
(54)
1 ( n −1) df dθ 1 4 ( n −1) 4 d 2θ 1 2 ( n −1) 2
− ( n − 1)υ B 2 x 2 y B x =α B x
dη dη dη 2

which leads to the ordinary differential equation for θ :

dθ d 2θ
− f ( 3 + n ) Pr = (55)
dη dη 2

b.) Transform the boundary conditions for u, v, and T into boundary conditions for f and θ .

The boundary conditions include no-slip at the wall:

u y =0 = 0 (56)

v y =0 = 0 (57)

The plate temperature is specified:

Ty =0 = Ts (58)

As y becomes large, the fluid becomes stagnant:

u y →∞ = 0 (59)
and the ambient temperature is recovered:

Ty →∞ = T∞ (60)

Equation (21) is substituted into Eq. (56):

1 ( n +1) df
u y = 0 = 4υ B 2
x 2
=0
dη η =0

or
df
=0 (61)
dη η =0

Equation (21) is substituted into Eq. (59):

1 ( n +1) df
u y →∞ = 4υ B 2
x 2
=0
dη η →∞

or

df
=0 (62)
dη η →∞

Equation (25) is substituted into Eq. (57):

1 ( n −1) 1 ( n −1) df
v y =0 = − fη =0 ( 3 + n )υ B 4
x 4
− ( n − 1)υ B 2
x 2
0=0 (63)
dη η =0


=0

which can only be true if:

fη =0 = 0 (64)

Equation (7) is substituted into Eqs. (58) and (60):

θη =0 = 1 (65)

θη →∞ = 0 (66)
Equations (42) and (55) together with Eqs. (61), (62), (64), (65)and (66) represent coupled third
order and second order ordinary differential equations with the five required boundary
conditions.

c.) Develop a numerical solution for this problem.

The numerical solution of the problem is more complicated than it would appear and therefore
df d 2 f  dθ
also quite interesting. The five state variables are f, , , θ , and . The state equations
dη dη 2

provide the rate of change of these state variables:

d df
[f ]= (67)
dη dη

d ⎡ df ⎤ d 2 f
dη ⎢ dη ⎥ = dη 2 (68)
⎣ ⎦

2
d ⎡ d 2 f ⎤ ⎛ df ⎞ d2 f 
=
⎢ dη 2 ⎥ ⎜ dη ⎟ 2 ( n + 1) − f ( 3 + n ) −θ (69)
dη ⎣ ⎦ ⎝ ⎠ dη 2

d  dθ
⎡θ ⎤ = (70)
dη ⎣ ⎦ dη

d ⎡ dθ ⎤ dθ
⎢ ⎥ = − f ( 3 + n ) Pr (71)
dη ⎣ dη ⎦ dη

Three of the boundary conditions for the state variables are specified at η = 0 while the
remaining two are specified at η → ∞; therefore, the shooting method is more complex than it
was in Section 4.4 because two boundary conditions must be assumed and then adjusted at η = 0.
This problem can be solved using either EES or MATLAB. Both solutions will be provided.
The EES solution is easy to implement, but it requires more computational time than the
MATLAB solution.

EES Solution
Equations (67) - (71) are implemented with the following EES equations:
f=f_o+integral(df\deta, eta, eta_lo, eta_hi) "integral of df\deta to find f"

df\deta=df\deta_o+integral(d2f\deta2,eta,eta_lo, eta_hi) "integral of d2f\deta2 to find df\deta"

d2f\deta2=d2f\deta2_o+integral(d3f\deta3,eta,eta_lo, eta_hi) "integral of d3f\deta3 to find df2\deta2"

d3f\deta3=df\deta^2*2*(n+1)-f*(3+n)*d2f\deta2-theta "Eq. 42 in solution"


theta=theta_o+integral(dtheta\deta,eta,eta_lo, eta_hi) "integral of dtheta\deta to find theta"

d2theta\deta2=-f*(3+n)*Pr*dtheta\deta "Eq. 55 in solution"

dtheta\deta=dtheta\deta_o+integral(d2theta\deta2,eta,eta_lo,eta_hi) "integral of d2theta\deta2 to find


dtheta\deta"

The boundary conditions at η=0 are:


"Boundary conditions at eta=0"
f_o=0
theta_o=1
df\deta_o=0

The two other boundary conditions (Eqs. (62) and (66)) are at η=infinity. The optimization
capability in EES will be applied to ensure that these boundary conditions are met. EES cannot
integrate to infinity. However, a value of η=10 provides results that are close enough to the
results at infinity. Set Prandtl number =1 and n=0.5 for the numerical solution
Pr=1 "Prandtl number"
n=0.5 "exponent for the equation (T_s-T_inf)=A*x^n"
eta_hi=10 "upper limit for eta - assume to be high enough"
eta_lo=0 "lower limit for eta"

df dθ
Enter guess values for and to test the equations.
dη η =0 dη o
d2f\deta2_o=0.5 "guess for testing"
dtheta\deta_o=-0.5 "guess for testing"

Solve. The boundary conditions required by Eqs. (62) and (66) are not met. We will find the
df dθ
values of and that fulfill the boundary conditions by optimization. The figure of
dη η =0 dη o
merit that we will use is
err=sqrt(df\deta^2+theta^2) "minimize err with respect to d2f\deta2_o and dtheta\deta_o"

Now comment out the guess values above and select the MinMax command (Options menu).
Minimize err with independent variables d2f\deta2_o and dtheta\deta_o
Select the Variable metric method and uncheck the option to Stop if error occurs. It will be
necessary to enter lower and upper bounds for the independent variables. Enter the following
bounds:
Click OK twice. After EES completes the calculations, the values of f and theta will be 0.6064
and -0.5822, respectively. The boundary conditions at η=10 (approximately infinite) are met.
The solution is complete. A MATLAB solution follows:

The Euler technique is implemented in MATLAB in order to integrate from η = 0 to a position


far from the wall. Recall that η is defined as the ratio of the distance from the plate to the
thickness of the boundary layer. Because the temperature rise in the fluid drives the velocity, it
is reasonable to expect that the momentum and thermal boundary layers will have similar
thickness unless the Prandtl number is very different than unity. Therefore, it should be
reasonable to terminate the integration at a value of η that is much larger than unity, for example
out to η∞ = 10, and enforce the boundary conditions given by Eqs. (62) and (66) at η∞.

The computational domain (0 < η < η∞) is divided into steps of size Δη:

η∞
Δη = (6-72)
( N − 1)
where N is the number of nodes. The location of each node is provided by:

ηi = η ∞
( i − 1) for i = 1..N (6-73)
( N − 1)

The solution is implemented in a MATLAB script named NC.

clear all;
Pr=1; % Prandtl number (-)
n=0.5; % exponent on the surface temperature variation (-)

eta_infinity=10; % outer edge of computational domain (-)


N=5001; % number of steps in the numerical integration (-)
Dn=eta_infinity/(N-1); % size of the integration steps (-)
for i=1:N
eta(i)=eta_infinity*(i-1)/(N-1); % location of integration steps (-)
end
d2 f
The initial conditions for the integration process are specified; note that the values of
dη 2 η =0

dθ
and (the variables d2fdn2_0 and dqdn_0) are assumed and must be adjusted to complete
dη η =0
the problem.

d2fdn2_0=0.5; % 2nd derivative of f at the wall (-)


dqdn_0=-0.5; % 1st derivative of q at the wall (-)

%boundary conditions at the wall


f(1)=0;
dfdn(1)=0;
d2fdn2(1)=d2fdn2_0;
q(1)=1;
dqdn(1)=dqdn_0;

Each integration step is taken using Euler's method; the rates of change of each state variable are
evaluated at the beginning of the integration step using Eqs. (67) through
Error! Reference source not found.:

df
fi +1 = fi + Δη (6-74)
dη i

df df d2 f
= + 2 Δη (6-75)
dη i +1
dη i dη i

d2 f d2 f ⎡ d2 f ⎛ df ⎞
2

= + ⎢ − ( 3 + n ) f + 2 ( n + 1) ⎜⎜ ⎟⎟ − θ ⎥
i Δη (6-76)
dη 2 dη i ⎢ dη i ⎝ dη
2 i 2
i⎠ ⎥
i +1 ⎣ ⎦

dθ
θi +1 = θi + Δη (6-77)
dη i

dθ dθ ⎡ dθ ⎤


= + ⎢ − ( 3 + n ) fi Pr ⎥ Δη (6-78)
dη i +1 dη i ⎣⎢ dη i ⎦⎥

for i=1:(N-1)
f(i+1)=f(i)+dfdn(i)*Dn;
dfdn(i+1)=dfdn(i)+d2fdn2(i)*Dn;
d2fdn2(i+1)=d2fdn2(i)+(-(3+n)*f(i)*d2fdn2(i)+2*(n+1)*(dfdn(i))^2-...
q(i))*Dn;
q(i+1)=q(i)+dqdn(i)*Dn;
dqdn(i+1)=dqdn(i)-(3+n)*Pr*f(i)*dqdn(i)*Dn;
end

df
Figure P6.2-1(a) illustrates θ and as a function of η for the case where n = 0.5. Note that

neither of the two boundary conditions at η → ∞ are satisfied and, in fact, the solution grows
d2 f dθ
dramatically and diverges with η even for the reasonable assumed values of and .
dη η =0
2
dη η =0
10

Dimensionless temperature and dfdη


7.5

5
dfdη
2.5

-2.5
dimensionless temperature

-5
0 0.5 1 1.5 2 2.5 3 3.5 4 4.5 5
Dimensionless position, η
df
Figure 6.3-2(a): Dimensionless temperature and as a function of η for Pr = 1.0 and n = 0.5 with

d2 f dθ
= 0.5 and = -0.5 .
dη 2 η =0
dη η =0

df
An error is calculated based on difference between the values of θ and at η∞ and their

required values, based on Eqs. (62) and (66) (i.e., zero).

err=sqrt(dfdn(N)^2+q(N)^2); % error in boundary conditions at n_infinity

d2 f dθ df
The next step in the solution is to adjust and so that = 0 and θη →∞ = 0 ;
dη 2 η =0
dη η =0 dη η →∞
unfortunately, the problem is sufficiently "stiff" (i.e., the values of the outputs are very sensitive
to the values of the inputs) that this is quite difficult to accomplish. Notice that the solution in
Figure 6.3-2(a) grows rapidly as η becomes large. In fact, the solution for dfdn eventually
becomes larger than MATLAB can represent and therefore the final entries in the vector dfdn are
NaN, which is MATLAB shorthand for "not a number". One of the problems with this solution
is that a lot of computational time is wasted finishing the numerical integration for a set of
boundary conditions that are clearly not viable. Anytime the dimensionless temperature
becomes less than 0 or greater than 1 or the dimensionless velocity becomes less than 0 or
d2 f dθ
substantially greater than 1 then it is clear that the assumed values of and are not
dη η =0
2
dη η =0
appropriate and therefore the integration should be terminated. For example, the integration
df
shown in Figure 6.3-2(a) should have been terminated at about η = 1.3 when became

negative. The integration is terminated if the velocity or temperature go out of bounds using the
break command. The break command terminates the execution of a for loop in MATLAB. If the
df
integration is terminated, then the value of θ and at η∞ are assigned to a large value that is

inversely proportional to the point at which the integration went out of bounds.

if((q(i+1)>1)|(q(i+1)<0)|(f(i+1)>2)|(f(i+1)<0))
q(N)=1+1/i;
dfdn(N)=1+1/i;
break;
end

Implementing the solution in MATLAB allows the use of the sophisticated multi-dimensional
optimization functions that are native to MATLAB. In order to use these functions, it is
necessary to convert the script NC into a function. The function takes as input arguments a
vector X that includes the two unknown boundary conditions at η = 0 as well as the Prandtl
number and returns as the first output the scalar argument err, which is related to how well the
boundary conditions at η∞ are met. The three additional outputs are the vectors n, dfdn, and q.

function[err,eta,dfdn,q]=P6p3d2(X,Pr,n)

% Inputs:
% X - vector of unknown boundary conditions at n=0
% X(1) - d2fdn2 at n=0 (-)
% X(2) - dqdn at n=0 (-)
% Pr - Prandtl number (-)
% n - exponent on surface temperature distribution (-)
%
% Outputs:
% err - error associated with mismatch in bc's at n_infinity (-)
% eta - vector of nodal positions (-)
% dfdn - dfdn at each nodal position (-)
% q - q at each nodal position (-)

eta_infinity=10; % outer edge of computational domain (-)


N=5001; % number of steps in the numerical integration (-)
Dn=eta_infinity/(N-1); % size of the integration steps (-)
for i=1:N
eta(i)=eta_infinity*(i-1)/(N-1); % location of integration steps (-)
end

%boundary conditions at the wall


f(1)=0;
dfdn(1)=0;
d2fdn2(1)=X(1);
q(1)=1;
dqdn(1)=X(2);

for i=1:(N-1)
f(i+1)=f(i)+dfdn(i)*Dn;
dfdn(i+1)=dfdn(i)+d2fdn2(i)*Dn;
d2fdn2(i+1)=d2fdn2(i)+(-(3+n)*f(i)*d2fdn2(i)+2*(n+1)*(dfdn(i))^2-...
q(i))*Dn;
q(i+1)=q(i)+dqdn(i)*Dn;
dqdn(i+1)=dqdn(i)-(3+n)*Pr*f(i)*dqdn(i)*Dn;
if((q(i+1)>1)|(q(i+1)<0)|(f(i+1)>2)|(f(i+1)<0))
q(N)=1+1/i;
dfdn(N)=1+1/i;
break;
end
end

err=sqrt(dfdn(N)^2+q(N)^2); % error in boundary conditions at n_infinity


end

The function fminsearch in MATLAB allows multidimensional, unconstrained optimization using


the Nelder-Mead algorithm. The calling protocol for fminsearch is:

X=fminsearch(fun,X0,OPTIONS)

where fun is a handle for the function to be minimized. This function must accept input X that
can be a scalar or a vector of inputs. The vector X0 is the starting values of these inputs that are
used to initiate the minimization and OPTIONS which is a vector of options that can be used to
control the optimization process.

A script called P6p3d2_run is generated to control the optimization. The variable space is cleared
and the Prandtl number set:

clear all;
Pr=1;
n=0.5;

Reasonable values are used to setup X0:

dfdn_0=0.5;
dqdn_0=-0.5;
X0=[dfdn_0,dqdn_0];

The OPTIONS vector is set up using the optimset command; the optimset command is used in the
same way that the odeset command was used to set up the OPTIONS vector for use with
MATLAB's native ode solvers, as discussed in Section 3.2.2. Here, the optimset command is
used to specify that the result of each iteration should be displayed and to set both termination
tolerances to 1x10-6.

OPTIONS=optimset('Display','iter','TolFun',1e-6,'TolX',1e-6);

The fminsearch command is called; notice that the function call is mapped onto a call of the
function P6p3d2, as discussed in Section 3.2.2 and elsewhere.

X = fminsearch(@(X) P6p3d2(X,Pr,n),X0,OPTIONS)

The P6p3d2 function is run again using the boundary conditions identified by the optimization:
[err,eta,dfdn,q]=P6p3d2(X,Pr,n);

The script P6p3d2_run is run from the command line:

>> P6p3d2_run

Iteration Func-count min f(x) Procedure


0 1 1.4155
1 3 1.41537 initial simplex
2 5 1.41527 expand
3 7 1.41499 expand
4 9 0.217854 expand
5 10 0.217854 reflect
6 12 0.217854 contract inside
...
60 116 1.98956e-006 contract inside
61 118 1.98956e-006 contract inside
62 120 1.98956e-006 contract inside

Optimization terminated:
the current x satisfies the termination criteria using OPTIONS.TolX of 1.000000e-006
and F(X) satisfies the convergence criteria using OPTIONS.TolFun of 1.000000e-006

X=

0.6056 -0.5814

The progress of the minimization algorithm is reported after each iteration and the optimization
d2 f dθ
will identify the appropriate boundary conditions, = 0.6056 and ≈ −0.5814 .
dη 2 η =0 dη η =0

df
d.) Plot the dimensionless temperature and velocity ( θ and ) as a function of dimensionless

position (η) for the case where Pr = 1 and n = 0.5.

df
Figure P6.3-2(b) illustrates the dimensionless temperature and (which is proportional to the

dimensionless velocity) as a function of η for these boundary conditions.
Dimensionless temperature and dfdη
1

0.8
dimensionless temperature

0.6

0.4

0.2
dfdη

0
0 1 2 3 4 5
Dimensionless position, η
df
Figure P6.3-2(b): Dimensionless temperature and as a function of η for Pr = 0.7 and n = 0.5 with

d2 f dθ
= 0.6056 and = -0.5814 .
dη η =0
2
dη η =0

e.) Plot the product of the local Nusselt number and the Grashof number based on the local plate
temperature to the -1/4 power as a function of Pr for various values of n.

dθ
The Nusselt number is related to . The local Nusselt number is defined as:
dη η =0

hx qs′′ x
Nu x = = (6-79)
k (Ts − T∞ ) k
The heat flux is given by:

⎛ ∂T ⎞
qs′′ = −k ⎜ ⎟ (6-80)
⎝ ∂y ⎠ y =0

Substituting the definition for dimensionless temperature into Eq. (6-80) leads to:

⎛ ∂θ ⎞
qs′′ = −k (Ts − T∞ ) ⎜ ⎟ (6-81)
⎝ ∂y ⎠ y =0

Substituting Eq. into Eq. (6-81) leads to:


dθ 1 ( n −1)
qs′′ = − k (Ts − T∞ ) B 4x 4 (6-82)
dη η =0

Substituting Eq. (18) into Eq. leads to:

1 1 1
⎛ Grx ⎞ 4
⎛ g β (Ts − T∞ ) x 3 ⎞ 4
⎛ g β A x n x3 ⎞ 4
1 ( 3+ n )
⎜ ⎟ =⎜ ⎟ =⎜ ⎟ = B 4x 4
(6-83)
⎝ 4 ⎠ ⎝ 4υ 2 ⎠ ⎝ 4υ 2 ⎠

gβ A
B= (6-84)
4υ 2

1
⎛ Ts −T∞
P ⎞ 4
dθ ⎜ g β Ax n ⎟
−1
qs′′ = −k (Ts − T∞ ) ⎜ ⎟ x 4 (6-85)
dη η =0 ⎜ 4υ 2
⎟⎟

⎝ ⎠

Substituting Eq. into Eq. (6-79) leads to:

1
⎛ dθ ⎞ ⎡ g β (Ts − T∞ ) ⎤ 4
−1 x
Nu x = − k (Ts − T∞ ) ⎜ ⎟ ⎢ ⎥ x 4
(6-86)
⎝ dη ⎠η =0 ⎣ 4υ 2 ⎦ (Ts − T∞ ) k

which can be simplified to:


1
⎛ dθ ⎞ ⎛ Grx ⎞ 4
Nu x = − ⎜ ⎟ ⎜ ⎟ (6-87)
⎝ dη ⎠η =0 ⎝ 4 ⎠

The product of the local Nusselt number and the local Grashof number to the -1/4 power is related
to the gradient of the dimensionless temperature at the surface of the plate, which is a function of
Pr and n:

1
−1 ⎛ dθ ⎞ ⎛ 1 ⎞ 4
Nu x Gr 4
= −⎜ ⎟ ⎜ ⎟ (6-88)
⎝ dη ⎠η =0 ⎝ 4 ⎠
x

The MATLAB script P6p3d2_run is modified to identify the value of the dimensionless
temperature gradient at the plate over a range of Prandtl number:

clear all;
Prv=[0.01,0.03,0.05,0.07,0.1,0.3,0.5,0.7,1,2,3,4,5,7,10,20,30,40,50,70,100,20
0,300,500,700,1000];
n=-1;
dfdn_0=0.5;
dqdn_0=-0.5;

for i=1:26
i
Pr=Prv(i);
X0=[dfdn_0,dqdn_0];
OPTIONS=optimset('Display','iter','TolFun',1e-6,'TolX',1e-5);
X = fminsearch(@(X) P6p3d2(X,Pr,n),X0,OPTIONS)
[err,eta,dfdn,q]=P6p3d2(X,Pr,n);
dfdn_0=X(1);
dqdn_0=X(2);
dqdnv(i)=-dqdn_0;
end

−1
Figure P6.3-2(c) illustrates the value of Nu x Grx 4
as a function of Pr for various values of the
Nusselt number.
3
n=1
2.5 n = 0.5
n = 0.2
2 n=0
-1/4
Nux Grx

1.5

0.5

0
0.01 0.1 1 10 100 1000
Prandtl number
−1
Figure P6.3-2(c): Nu x Grx 4
as a function of Pr for various values of n.

f.) Plot the product of the average Nusselt number and the Grashof number based on the average
plate temperature to the -1/4 power as a function of Pr for various values of n.

The average Nusselt number is defined as:


L L
1 1 Nu k
hL = ∫ h dx = ∫ x dx (6-89)
L0 L0 x

Substituting Eq. (6-87) into Eq. (6-89) leads to:


1
1
L
⎛ dθ ⎞ ⎛ Grx ⎞ 4
k
hL = ∫ − ⎜ ⎟ dx (6-90)
L 0 ⎝ dη ⎠η =0 ⎜⎝ 4 ⎟⎠ x
Substituting the definition of the Grashof number into Eq. (6-90) leads to:

1
⎛ dθ ⎞ k L ⎛ g β A x n ⎞ 4
−1
hL = − ⎜ ⎟ ∫⎜
⎝ dη ⎠η =0 L 0 ⎝ 4υ
2 ⎟

x 4
dx (6-91)

or

1
⎛ dθ ⎞ k ⎛ g β A ⎞ 4 L ( n −1)
hL = − ⎜ ⎟ ⎜ 2 ⎟
⎝ dη ⎠η =0 L ⎝ 4υ ⎠
∫x
0
4
dx (6-92)

Carrying out the integration leads to:

1
⎛ dθ ⎞ k ⎛ g β A ⎞ 4
4 ( n + 3)
hL = − ⎜ ⎟ ⎜ 2 ⎟
L 4 (6-93)
⎝ dη ⎠η =0 L ⎝ 4υ ⎠ ( n + 3)

The average surface-to-fluid temperature of the plate is:


L
1 A Ln +1 A Ln
L ∫0
Ts − T∞ = A x n
dx = = (6-94)
L ( n + 1) ( n + 1)

Substituting Eq. (6-94) into Eq. (6-93) leads to:

1
⎛ dθ ⎞ k ⎛ g β A Ln ( n + 1) ⎞ 4
4 ( n + 3)
hL = − ⎜ ⎟ ⎜⎜ ⎟⎟ L 4 (6-95)
⎝ dη ⎠η =0 L ⎝ 4υ ( n + 1) L ⎠ ( n + 3)
2 n

or
1
⎛ dθ ⎞ k ⎛ g β (Ts − T∞ ) ( n + 1) ⎞
4
4 ( n + 3)
hL = − ⎜ ⎟ ⎜ ⎟ L 4 (6-96)
⎝ dη ⎠η =0 L ⎜⎝ 4υ 2 Ln ⎟⎠ ( n + 3)

which can be rearranged:


1
⎛ dθ ⎞ k ⎛ g β (Ts − T∞ ) L ⎞ 4 ( n + 1)
3 4 1/ 4

hL = − ⎜ ⎟ ⎜ ⎟ (6-97)
⎝ dη ⎠η =0 L ⎜⎝ 4υ 2 ⎟ ( n + 3)


GrL
4

Substituting Eq. into the definition of the average Nusselt number leads to:
1
⎛ dθ ⎞ ⎛ g β (Ts − T∞ ) L ⎞ 4 ( n + 1)
3 4 1/ 4
hL L
Nu L = = −⎜ ⎟ ⎜ ⎟ (6-98)
k ⎝ dη ⎠η =0 ⎜⎝ 4υ 2 ⎟ ( n + 3)


GrL
4

or

1
4 ( n + 1)
1/ 4
h L ⎛ dθ ⎞ ⎛ GrL ⎞ 4
Nu L = L = − ⎜ ⎟ ⎜ ⎟ (6-99)
k ⎝ dη ⎠η =0 ⎝ 4 ⎠ ( n + 3)

where GrL is the Grashof number based on the average plate temperature:

g β (Ts − T∞ ) L3
GrL = (6-100)
υ2

The product of the average Nusselt number and the average Grashoff number to the -1/4 power is
a function of Prandtl number and n:

4 ( n + 1)
1/ 4
−1 ⎛ dθ ⎞
Nu L GrL 4
= −⎜ ⎟ (6-101)
⎝ dη ⎠η =0 ( n + 3) 4 4
1

The MATLAB code P6p3d2_run is modified to return this parameter. Figure P6.3-2(d) illustrates
−1
4
Nu L GrL as a function of Pr for various values of n.

3.5 n=1
n = 0.5
3 n = 0.2

2.5 n=0
-1/4
NuL GrL

1.5

0.5

0
0.001 0.01 0.1 1 10 100 1000
Prandtl number
−1
4
Figure P6.3-2(d): NuL GrL as a function of Pr for various values of n.
g.) Plot the average Nusselt number as a function of the Grashof number based on the average
plate temperature to the -1/4 power for a plate with a constant heat flux for Pr = 0.7.

The heat flux at the surface of the plate is:

1
⎛ Ts −T∞
P ⎞ 4
dθ ⎜ g β A x n ⎟ −1 4
qs′′ = −k (Ts − T∞ ) ⎜ ⎟ x (6-102)
dη η =0 ⎜ 4υ 2 ⎟
⎜ ⎟
⎝ ⎠

or

1
⎛ Ts −T∞
P ⎞ 4

d θ ⎜ g β A x n ⎟
−1 ( 5 n −1)
qs′′ = −k A x n ⎜ ⎟ x 4
∝x 4
(6-103)
dη η =0 ⎜ 4υ 2 ⎟
⎜ ⎟
⎝ ⎠

Therefore, if n = 0.2 then the solution corresponds to a plate with a uniform heat flux and n = 0
corresponds to a plate with a constant surface temperature. Using the self-similar solution, the
average Nusselt number for a plate with a constant heat flux as a function of the Grashof number
based on the average plate temperature is shown in Figure P6.3-2(e) with Pr = 0.7.

10 3

10 2
NuL

10 1

10 0
102 103 104 105 106 107 108 109 1010 1011 1012
GrL
Figure P6.3-2(d): Average Nusselt number as a function of the Grashof number based on the average plate
temperature for Pr = 0.7 for a plate with a constant heat flux (n = 0.2).
Problem 7.2-1 (7-1 in text): Contaminant Removal Method
One method of removing water and other contamination from a gas is to pass it through a cooled
tube so that contaminants with high freezing and liquefaction points (e.g., water) tend to be
collected at the wall. A quick and easy liquid nitrogen trap for methane is constructed by placing
a tube in a Styrofoam cooler that is filled with liquid nitrogen, as shown in Figure P7.2-1.
liquid nitrogen at 1 atm
L=1m

m = 0.01 kg/s
x
T f ,in = 20°C
p f ,in = 400kPa

thins = 0.375 inch Dout = 0.5 inch thtube = 0.065 inch

Figure P7.2-1: Liquid nitrogen trap.

The length of the tube is L = 1 m. The outer diameter of the tube is Dout = 0.5 inch and the tube
thickness is thtube = 0.065 inch. The tube conductivity is ktube = 150 W/m-K. The tube is
wrapped in insulation (to avoid liquefying the methane). The thickness of the insulation is thins =
0.375 inch and the insulation conductivity is kins = 1.5 W/m-K. Methane enters the tube at m =
0.01 kg/s with temperature Tf,in = 20°C and pressure pf,in = 400 kPa. The liquid nitrogen that fills
the container is at 1 atm and is undergoing nucleate boiling on the external surface of the
insulation. You may neglect axial conduction through the tube.
a.) Set up an EES program that can evaluate the state equations for this problem. That is, given
a value of position, x, methane temperature, Tf, and methane pressure, pf, your program
dT f dp f
should be able to compute and .
dx dx

Enter the information in the problem statement into EES.

$UnitSystem SI MASS RAD PA K J


$TABSTOPS 0.2 0.4 0.6 0.8 3.5 in

"Inputs"
L=1 [m] "length of tube"
p_atm=1 [atm]*convert(atm,Pa) "atmospheric pressure"
m_dot=0.01 [kg/s] "mass flow rate of air"
F$='Methane' "fluid"
D_out=0.5 [inch]*convert(inch,m) "outer diameter"
th_tube=0.065 [inch]*convert(inch,m) "tube thickness"
k_tube=150 [W/m-K] "tube conductivity"
h_bar_out=350 [W/m^2-K] "heat transfer coefficient on outer surface"
k_ins=1.5 [W/m-K] "insulation conductivity"
th_ins_inch=0.375 [inch] "insulation thickness, in inch"
th_ins=th_ins_inch*convert(inch,m) "insulation thickness"
T_LN2=temperature('Nitrogen',x=0,p=p_atm) "temperature of liquid nitrogen"
T_f_in=converttemp(C,K,20[C]) "inlet temperature of gas"
p_f_in=400 [kPa]*convert(kPa,Pa) "inlet pressure of gas"
The inner diameter of the tube is:

Din = Dout − 2 thtube (1)

and the outer diameter of the insulation is:

Dout ,ins = Dout + 2 thins (2)

D_in=D_out-2*th_tube "inner diameter of tube"


D_out_ins=D_out+2*th_ins "outer diameter of insulation"

An arbitrary values of position (x), fluid temperature (Tf) and pressure (pf) are specified. These
values will later be commented out when the state equation is integrated.

x=0 [m] "arbitrary value of the position to check state equation"


T_f=T_f_in "arbitrary value of the fluid temperature to check state equation"
p_f=p_f_in "arbitrary value of the fluid pressure to check the state equation"

A guess is made for the temperature of the outside surface of the insulation (Tw,out). This is the
surface in contact with the liquid nitrogen and is necessary to calculate the boiling heat transfer.
The guess will be commented out later.

T_w_out=(T_f+T_LN2)/2 "guess outer temperature of the insulation surface"

The Nucleate_Boiling function in EES returns the heat flux on the outside surface of the insulated
tube ( qs′′,out ).

q``_s_out= Nucleate_Boiling('Nitrogen', T_LN2, T_w_out, 0.0127 [-])"get nucleate boiling heat flux"

The fluid properties of the methane (ρf, μf, kf, cf, and Prf) are obtained at Tf:

rho_f=density(F$,T=T_f,p=p_f) "density of fluid in tube"


mu_f=viscosity(F$,T=T_f,p=p_f) "viscosity"
k_f=conductivity(F$,T=T_f,p=p_f) "conductivity"
c_f=cP(F$,T=T_f,p=p_f) "specific heat capacity"
Pr_f=mu_f*c_f/k_f "Prandtl number"

The mean velocity of the flow is:

4 m
um = (3)
ρ f π Din2

The Reynolds number is computed:

ρ f Din um
Re = (4)
μf
The PipeFlow_N_local procedure is used to obtain the local Nusselt number (Nu) and friction
factor (f). The heat transfer coefficient is computed according to:

Nu k f
hin = (5)
D

u_f=m_dot/(rho_f*pi*D_in^2/4) "bulk velocity"


Re=rho_f*u_f*D_in/mu_f "Reynolds number"
call PipeFlow_N_local(Re,Pr_f,x/D_in,0 [-]: Nusselt_T_x,Nusselt_H_x,f_x) "call correlations"
h_in=Nusselt_T_x*k_f/D_in "local heat transfer coefficient"

The overall resistance per unit length between the outer surface of the tube and the fluid flowing
in the pipe can be found by adding the resistances for convection on the inside surface and
conduction through the wall and insulation:

R′ = Rconv
′ ,in + Rcond
′ ,tube + Rcond
′ ,ins (6)

where

1
′ ,in =
Rconv (7)
hin π D

⎛D ⎞
ln ⎜ out ⎟
= ⎝ in ⎠
D
′ ,tube
Rcond (8)
2 π ktube

⎛D ⎞
ln ⎜ out ,ins ⎟
= ⎝ out ⎠
D
′ ,ins
Rcond (9)
2 π kins

R`=R`_conv_in+R`_cond_tube+R`_cond_ins "resistance per unit length"


R`_conv_in=1/(h_in*pi*D_in) "convection resistance per unit length"
R`_cond_tube=ln(D_out/D_in)/(2*pi*k_tube) "conduction through tube, per unit length"
R`_cond_ins=ln(D_out_ins/D_out)/(2*pi*k_ins) "conduction through insulation, per unit length"

The equations are solved and the guess values updated. The assumed value of Tw,out is
commented out and the heat flux at the outer surface of the insulation is recalculated:

qs′′,out π Dout ,ins =


(T f − Tw,out )
(10)
R′

{T_w_out=(T_f+T_LN2)/2} "guess outer temperature of the insulation surface"


q``_s_out*pi*D_out_ins=(T_f-T_w_out)/R` "recalculate heat flux"
The rate of change of the fluid temperature is obtained using an energy balance:

dT f qs′′,out π Dout ,ins


= (11)
dx m c f

and the rate of change of the fluid pressure is obtained using the friction factor:

dp f ρ f um2
=−f (12)
dx 2 Din

dTfdx=-q``_s_out*pi*D_out_ins/(m_dot*c_f) "obtain temperature derivative"


dpfdx=-f_x*rho_f*u_f^2/(2*D_in) "obtain pressure gradient"

b.) Use the Integral command in EES to integrate the state equations from x = 0 to x = L. Plot the
fluid temperature and pressure as a function of position.

Update the guess values. Then comment out the arbitrary specifications for x, Tf and pf. Use the
Integral command to integrate the two state equations derived in part (a).

{x=0 [m] "arbitrary value of the position to check state equation"


T_f=T_f_in "arbitrary value of the fluid temperature to check state equation"
p_f=p_f_in "arbitrary value of the fluid pressure to check the state equation"}
T_f=T_f_in+Integral(dTfdx,x,0,L,0.01 [m])
p_f=p_f_in+Integral(dpfdx,x,0,L, 0.01 [m])

$IntegralTable x,T_f,p_f

Figure 2 illustrates the fluid temperature and pressure as a function of position.


300 400000

290
398000
Temperature (K)

280
pf
Pressure (Pa)

396000
270
Tf
260
394000

250
392000
240

230 390000
0 0.1 0.2 0.3 0.4 0.5 0.6 0.7 0.8 0.9 1
Position (m)
Figure 2: Fluid temperature and pressure as a function of position.
c.) Plot the heat flux at the insulation surface and the critical heat flux as a function of position.

The properties of the saturated liquid nitrogen (Δivap, ρv,sat, ρl,sat, and σ) are computed using the
internal functions in EES.

"Determine critical heat flux"


Di_vap=enthalpy('Nitrogen',x=1,p=p_atm)-enthalpy('Nitrogen',x=0,p=p_atm) "latent heat of fusion"
rho_v_sat=density('Nitrogen',x=1,p=p_atm) "density of saturated nitrogen vapor"
rho_l_sat=density('Nitrogen',x=0,p=p_atm) "density of saturated nitrogen vapor"
sigma=SurfaceTension('Nitrogen',T=T_LN2) "surface tension"

As discussed in Section 7.2.3, the characteristic length for nucleate boiling is computed
according to:

σ
Lchar , nb = (7-13)
g ( ρl , sat − ρ v , sat )

and the dimensionless length that characterizes the tube is:

D
L = out ,ins (7-14)
2 Lchar ,nb

L_char_nb=sqrt(sigma/(g#*(rho_l_sat-rho_v_sat)))"characteristic length associated with nucleate boiling"


L_hat=(D_out_ins/2)/L_char_nb "dimensionless length"

which leads to L = 15. According to Table 7-2, a cylinder with L > 1.2 is a large cylinder and
should use Ccrit = 0.12 in the correlation:

⎡ σ g ( ρl , sat − ρ v , sat ) ⎤
1
4

qs′′,crit = Ccrit Δivap ρ v , sat ⎢ ⎥ (7-15)


⎢⎣ ρv2, sat ⎥⎦

C_crit=0.12 "critical heat flux constant"


q``_s_crit=C_crit*Di_vap*rho_v_sat*(sigma*g#*(rho_l_sat-rho_v_sat)/rho_v_sat^2)^(1/4)
"critical heat flux"

The Critical_Heat_Flux library function in EES can also be used:

q``_max=Critical_Heat_Flux('Nitrogen','Cylinder', L_char_nb, T_LN2)


"critical heat flux from library routine"

Both provide the same result, qs′′,crit = 148.1 kW/m2. The heat flux calculations are added to the
Integral Table:

$IntegralTable x,T_f,p_f,q``_s_out, ,q``_s_crit, q``_max


Figure 3 illustrates the surface heat flux and the critical heat flux as a function of position and
shows that the heat flux is safely below the critical heat flux.

1.6x105
critical heat flux
1.4x105

1.2x105
Heat flux (W/m )
2

1.0x105

8.0x104

6.0x104

4.0x104

2.0x104
heat flux
0.0x100
0 0.1 0.2 0.3 0.4 0.5 0.6 0.7 0.8 0.9 1
Axial position (m)
Figure 3: Heat flux and critical heat flux as a function of position.

d.) Plot the temperature of the methane at the surface of the tube as a function of position.

The temperature of the inside tube wall surface is calculated according to:

Tw,in = T f − qs′′ Dout ,ins π Rconv


′ ,in (7-16)

T_w_in=T_f-q``_s_out*pi*D_out_ins*R`_conv_in "inner temperature of wall"

The inner wall temperature is added to the $IntegralTable directive.

$IntegralTable x,T_f,p_f,q``_s_out, ,q``_s_crit, q``_max, T_w_out,T_w_in

The inner wall temperature as a function of position is shown in Figure 4.


270

260

Inner wall temperature (K)


250

240

230

220

210

200

190

180
0 0.1 0.2 0.3 0.4 0.5 0.6 0.7 0.8 0.9 1
Position (m)
Figure 4: Inner wall temperature as a function of position.

e.) Plot the lowest temperature experienced by the methane in the trap as a function of the
insulation thickness. If the methane temperature must be maintained at above its liquefaction
point (131.4 K at 400 kPa) then what should the insulation thickness be?

The lowest temperature experienced by the methane is at the wall at the exit (Tw,in,x=L). Figure 5
illustrates the lowest methane temperature as a function of insulation thickness.
200
Lowest methane temperature (K)

180

160

140

120

100

80
0 0.05 0.1 0.15 0.2 0.25 0.3 0.35 0.4 0.45 0.5
Insulation thickness (inch)
Figure 5: Lowest methane temperature as a function of insulation thickness.

Figure 5 suggests that the insulation thickness should be at least 0.1 inch to keep the methane
from liquefying.
Problem 7.2-2 (7-2 in text): Steam Boiler
An industrial boiler generates steam by heat exchanging combustion gases with saturated water
at 125 kPa through mechanically polished AISI 302 stainless steel tubing having an inside
diameter of 5.48 cm with a wall thickness of 2.7 mm and a total submerged length of 10 m. The
combustion gases enter the tubing at 750°C with a mass flow rate of 0.0115 kg/s. The gases
exhaust at ambient pressure. Assume that the combustion gases have the same thermodynamic
properties as air.
a) Identify the state equation for this problem; the differential equation that can be used to
determine the rate of change of the temperature of the combustion gas with respect to
position.

Enter known information into EES.

$UnitSystem SI MASS RAD PA K J


$TABSTOPS 0.2 0.4 0.6 0.8 3.5 in

"known information"
D_i=5.48 [cm]*convert(cm,m) "inside tube diameter"
D_o=D_i+2.7 [mm]*convert(mm,m) "outside tube diameter"
L=10 [m] "tube length"
P_water=125 [kPa]*convert(kPa,Pa) "pressure on water"
T_water=T_sat(Water,P=P_water) "saturation temperature of water"
m_dot_gas=0.0115 [kg/s] "combustion gas flow rate"
T_gas_in=convertTemp(C,K,750 [C]) "combustion gas inlet temperature"
k_steel=k_('Stainless_AISI302', (T_gas_in+T_water)/2) "thermal conductivity of steel"
F$='air' "assume combustion gases behave as air"
P_gas=101.3 [kPa]*convert(kPa,Pa) "air pressure"
RelRough=0 [-] "smooth tube"

Arbitrary values of x and Tgas are set in order to compute the state equations:

x=1 [m] "position"


T_gas=800 [K] "gas temperature"

An energy balance on the combustion gases provides the state equation:

dTgas
m gas c p + qo′′ π Do = 0 (1)
dx

where qo′′ is the heat flux leaving the outer surface of the tube. The PipeFlow procedure
determines the local heat transfer coefficient for the gas flowing in the pipe (hin).

call PipeFlow_local(F$,T_gas,P_gas,m_dot_gas,D_i,x,RelRough:h_in, h_H_x, dPdx)

The heat transfer on the outside surface of the tube is found using the Nucleate_Boiling function in
EES. An assumed value of the wall temperature (Twall) is required and will be commented out:

C_s_f=0.0132 [-] "coefficient for mechanically polished stainless steel"


T_wall=700 [K] "guess for wall temperature"
q``_o= Nucleate_Boiling('Water', T_water, T_wall, C_s_f)
"obtain heat flux as a function of the wall temperature"

The guess values are updated and the value of Twall commented out. The wall temperature is
computed according to:

qo′′ π Do =
(T
gas − Twall )
(2)
R′

where

⎛D ⎞
ln ⎜ o ⎟
+ ⎝ i⎠
1 D
R′ = (3)
hin π Di 2 π ksteel

{T_wall=700 [K]} "guess for wall temperature"


R`=1/(h_in*pi*D_i)+ln(D_o/D_i)/(2*pi*k_steel)
"resistance per unit length between gas and outside surface of tube"
q``_o*pi*D_o=(T_gas-T_wall)/R` "equate steady-state heat flows"

Equation (1) is used to calculate the state equation:

cp=cp(F$,T=T_gas) "specific heat capacity"


m_dot_gas*cp*dT\dx+q``_o*pi*D_o=0 "energy balance"

b) Integrate the state equations developed in part (a) in order to determine the outlet temperature
of the combustion gases.

Solve and update the guess values. Then comment out the values of x and Tgas and use the
Integral command to integrate the state equation. An $IntegralTable directive to include x and Tgas
is used.

T_gas=T_gas_in+integral(dT\dx,x,0,L)
$integralTable, x:0.2 T_gas

Figure 1 illustrates the temperature as a function of position.


1100

1000

Gas temperature (K)


900

800

700

600

500

400

300
0 1 2 3 4 5 6 7 8 9 10
Position (m)
Figure 1: Gas temperature as a function of position.

c) Calculate the rate at which steam is generated in this boiler.

An energy balance on the steam determines the rate at which steam is generated.

(
m steam ( isteam , x =1 − isteam, x =0 ) = m gas igas ,T =Tgas ,in − igas ,T =Tgas ,out ) (4)

"Energy balance to determine steam generation rate"


m_dot_steam*(enthalpy(Water,P=P_water,x=1)-enthalpy(Water,P=P_water,x=0))=&
m_dot_gas*(enthalpy(F$,T=T_gas_in)-enthalpy(F$,T=T_gas))

which leads to m steam = 0.00341 kg/s.


Problem 7.2-3 (7-3 in text): Spaghetti
You are preparing a spaghetti dinner for guests when you realize that your heat transfer training
can be used to answer some fundamental questions about the process. The pot you are using
holds four liters of water. The atmospheric pressure is 101 kPa. When on its high setting, the
electric stove heating unit consumes 1.8 kW of electrical power of which 20% is transferred to
the surroundings, rather than to the water. The pot is made of 4 mm thick polished AISI 304
stainless steel and it has a diameter of 0.25 m. The burner diameter is also 0.25 m.
a) How much time is required to heat the water from 15°C to its boiling temperature?

Enter the known specifications into EES.

$UnitSystem SI MASS RAD PA K J


$TABSTOPS 0.2 0.4 0.6 0.8 3.5 in

"known information"
V_w=4 [liter]*convert(liter,m^3) "volume of water"
P=101.3 [kPa]*convert(kPa,Pa) "atmospheric pressure"
D=0.25 [m] "diameter of the burner and pot"
T_w_i=convertTemp(C,K,15 [C]) "initial water temperature"
T_sat=T_sat(Water,P=P) "boiling temperature"
delta=4 [mm]*convert(mm,m) "thickness of stainless steel"
fraction=0.8 "fraction of the heat flux that enters the water"
q_burner=1.8[kW]*convert(kW,W) "heat flux into water"

The initial mass of water in the pot is calculated:

M w,i = Vw ρ w (1)

where ρw is the density of liquid water computed at the initial condition.

m_w_i=V_w*density(Water,T=T_w_i,P=P) "initial mass of water"

The surface area of the bottom of the pan is computed:

D2
A=π (2)
4

and the initial height of the water in the pan is computed:

V
H= (3)
A

A=pi*D^2/4 "area of the bottom of the pot"


H=V_w/A "height of the water in the pot"

The mass of the pot is computed:


⎛ D2 ⎞
M steel = ⎜ π + π D H ⎟ δ ρ steel (4)
⎝ 4 ⎠

The conductivity and specific heat capacity of steel are determined (ksteel and csteel, respectively).
The total heat capacity of the stell is calculated:

Csteel = M steel csteel (5)

and compared to the sensible heat capacity of the water:

Cwater = M w,i cw (6)

where cw is the specific heat capacity of liquid water:

m_steel=(pi*D^2/4+pi*D*H)*delta*rho_('Stainless_AISI304', T_w_i) "mass of pot in contact with the water"


c_steel=c_('Stainless_AISI304', T_w_i) "specific heat of steel"
k_steel=k_('Stainless_AISI304', T_w_i) "thermal conductivity of steel"
mc_steel=m_steel*c_steel "capacitance of the pot - note that it is negligible relative to the water"
c_water=cp(water,T=(T_w_i+T_sat)/2,P=P) "specific heat of water"
mc_water=m_w_i*c_water "capacitance of water"

Note that Csteel = 1.67 kJ/K whereas Cwater = 16.7 kW/K and therefore it is possible to neglect the
heat capacity of the pan. An energy balance on the pot and its contents determines the time
required for it to start to boil.

qburner Δt fraction = Cwater (Tsat − Tw,i ) (7)

where fraction = 0.8 is the fraction of the burner power that enters the water.

"a)"
q_burner*DELTAt*fraction=mc_water*(T_sat-T_w_i) "energy balance to determine time required to boil"
DELTAt_min=DELTAt*convert(s,min) "in min"

which leads to Δt = 986.2 s (16.4 min).

b) What are the temperatures of the outside and inside surfaces of the bottom of pot while the
water is boiling?

The burner heat flux is:

fraction qburner
′′
qburner = (8)
A

The procedure Nucleate_Boiling in EES is used to solve for the inner temperature of the pan (Ti).
The outer temperature is calculated according to:
k steel
′′
qburner = (To − Ti ) (9)
δ

"b)"
q``_burner=q_burner*fraction/A "heat flux from burner"
C_s_f=0.0132 "coefficient for polished stainless steel"
q``_burner= Nucleate_Boiling('Water', T_sat, T_i, C_s_f) "heat flux to water - solve for T_i"
q``_burner=k_steel*(T_o-T_i)/delta "heat flux through pan bottom"
T_i_C=converttemp(K,C,T_i) "inner temperature, in C"
T_o_C=converttemp(K,C,T_o) "outer temperature, in C"

which leads to Ti = 106.1ºC and To = 114.1ºC.

c) What would the burner electrical power requirement have to be to achieve the critical heat
flux? Compare the actual heat flux during the boiling process to the critical heat flux.

′′ ); the characteristic length for


The Critical_Heat_Flux function returns the critical heat flux ( qmax
the pan bottom is:

Lchar = A (10)

The burner power required to reach critical heat flux is:

′′
qmax
qburner = (11)
fraction A

"c)"
L=sqrt(A) "estimate of the characteric length to use for this circular plate"
q``_max=Critical_Heat_Flux('Water','Plate', L, T_sat) "determine critical heat flux"
q_burner_max=q``_max/fraction*A "required burner capacity - no danger of reaching burn out here"

which leads to qburner ,max = 62.2 kW. It is not likely that we would need to worry about burnout.

d) How much water is vaporized during the 10 minutes required to cook the spaghetti?

An energy balance determines the amount of water vaporized in Δtboil = 10 minutes:

′′
qburner fraction Δtboil
mvaporized = (12)
( iwater , x=1 − iwater , x=0 )
"d)"
DELTAt_boil=10 [min]*convert(min,s)
m_vaporized=q_burner*DELTAt_boil/(enthalpy(Water,T=T_sat,x=1)-enthalpy(Water,T=T_sat,x=0))

which leads to mvaporized = 0.38 kg.


Problem 7.2-4 (7-4 in text): Tungsten Wire
A tungsten wire having a diameter of 1 mm and a length of 0.45 m is suspended in saturated
carbon dioxide liquid maintained at 3.25 MPa. The fluid-surface coefficient needed in the
nucleate boiling relation, Cnb, is estimated to be 0.01 and the emissivity of the tungsten wire is
0.4. Prepare a plot of the electrical power dissipated in the wire versus the excess temperature
for power levels ranging from 10 W to the power corresponding to the critical heat flux for the
nucleate boiling regime. What is your estimate of the excess temperature at the burnout point?

Known information is entered into EES.

$UnitSystem SI MASS RAD PA K J


$TABSTOPS 0.2 0.4 0.6 0.8 3.5 in

"known information"
Fluid$='CarbonDioxide'
P=3.250 [MPa]*convert(MPa,Pa) "saturation pressure"
d=1 [mm]*convert(mm,m) "diameter of wire"
L=0.45 [m] "length of wire"
T_sat=T_sat(Fluid$,P=P) "saturation temperature"

The critical heat flux ( qs′′,max ) is determined using the Critical_Heat_Flux procedure using the radius
of the wire as the characteristic length.

Geom$='CYLINDER'
q``_max=Critical_Heat_Flux(Fluid$,Geom$, d/2, T_sat) "characteristic length is the radius"

The power needed obtain the critical heat flux is calculated based on the heat flux:

w max = q s′′, max π d L (1)

w_dot_max=q``_max*pi*d*L "maximum power dissipation"

which leads to w max = 966 W; this is used as the maximum power for the plot. A specific excess
temperature can be used to determine the wire surface temperature:

Tw = Tsat + ΔTe (2)

The Nucleate_Boiling procedure is used to relate the excess temperature to the wire heat flux
( q s′′,nb ). The heat flux is used to compute the electrical power:

w nb = qs′′,nb π d L (3)

C_s_f=0.01 [-] "nucleate boiling coefficient"


DT_e=10 [K] "excess temperature"
T_w=T_sat+DT_e "excess temperature for nucleate boiling"
q``_s= Nucleate_Boiling(Fluid$, T_sat, T_w, C_s_f) "nucleate boiling correlation"
w_dot_nb=q``_s*pi*d*L "electrical power"
Figure 1 illustrates the electrical power as a function of excess temperature for nucleate boiling;
the critical heat flux is indicated in the plot. The excess temperature at burnout is approximately
10 K.
1000

900 critical heat flux


800
Electrical power (W)

700

600 nucleate boiling


500

400

300 film boiling


200

100

0
0.5 1 10 100 1000
Excess temperature (K)
Figure 1: Electrical power as a function of excess temperature for nucleate and film boiling.

The Film_Boiling procedure is used to compute the heat flux in a film boiling mode ( qs′′, film ) and
therefore the electrical power to the wire.

w nb = qs′′,nb π d L (4)

epsilon=0.4 [-] "emissivity of tungsten"


q``_film= Film_Boiling(Fluid$,Geom$, T_sat, T_w, d, epsilon) "film boiling correlation"
w_dot_film=q``_film*pi*d*L "electrical power"

The electrical power as a function of excess temperature is overlaid onto Figure 1.


Problem 7.2-5 (7-5 in text): Evacuated Tubular Collector
A cross-section of one type of evacuated solar collector is shown in Figure P7.2-5. The collector
consists of a cylindrical glass tube with an outer diameter of 7.5 cm and wall thickness of 5 mm.
In the center of the tube is a heat pipe, which is a copper tube with an outer diameter of 2 cm and
wall thickness of 1.5 mm. The heat pipe contains a small amount of water at a pressure of 200
kPa that experiences nucleate boiling as solar radiation is incident on the outside surface of the
copper tube at a rate of 745 W/m2. You may assume that the glass is transparent to solar
radiation and that the absorptivity of the copper tube with respect to solar radiation is 1.0. The
surface of the copper tube has an emissivity of 0.13 with respect to its radiative interaction with
the inner surface of the glass tube. The glass may be assumed to be opaque to thermal radiation
from the copper tube with an emissivity of 1.0 on both its inner and outer surfaces. The outside
surface of the glass interacts with the 25°C, 101.3 kPa surroundings through radiation and free
convection.
25°C
745 W/m2 2 cm
1.5 mm
glass 5 mm
water
vacuum

7.5 cm
P7.2-5: Cross-section of an evacuated tubular collector

a.) Calculate the net rate of energy transfer to the water per unit length.

Enter problem information into EES:


$UnitSystem SI MASS RAD PA K J
$TABSTOPS 0.2 0.4 0.6 0.8 3.5 in

"known information"
D_g_o=7.5 [cm]*convert(cm,m) "outer glass diameter"
th_g=5 [mm]*convert(mm,m) "glass cylinder thickness"
D_g_i=D_g_o-2*th_g "inner glass diameter"
D_c_o=2 [cm]*convert(cm,m) "outer copper diameter"
th_c=1.5 [mm]*convert(mm,m) "copper tube thickness"
D_c_i=D_c_o-2*th_c "inner copper diameter"
k_c=k_('Copper',T=300 [K]) "thermal conductivity of copper"
k_g=k_('Glass-Pyrex', T=300 [K]) "thermal conductivity of glass"
q``_a=745 [W/m^2] "rate of solar radiation absorbed on copper tube"
P_sat=200 [kPa]*convert(kPa,Pa) "saturation pressure in the tube"
T_w=T_sat(Water,P=P_sat) "saturation temperature"
T_amb=convertTemp(C,K,25 [C]) "ambient temperature"
P_amb=101.3 [kPa]*convert(kPa,Pa) "ambient pressure"
epsilon=0.13 [-] "emittance of the copper surface"
L=1 [m] "per unit length"

The temperatures of the inner and outer surfaces of the glass (Tg,i and Tg,o, respectively) and the
outer surface of the copper tube (Tc,o) must be assumed in order to calculate the resistances to
radiation and natural convection:
T_g_o=310 [K] "guess temperature for outer glass temperature"
T_g_i=315 [K] "guess temperature for inner glass temperature"
T_c_o=320 [K] "guess temperature for outer tube temperature"

The rate at which energy is absorbed by copper tube is:

qsolar = qa′′ Dc ,o L (1)

where qa′′ is the rate at which solar energy is incident on the copper tube per unit area.

q_dot_solar=2*D_c_o*L*q``_a "rate of energy absorption"

The heat loss is related to radiation from the copper tube to the inner surface of the glass tube,
conduction through the glass, and natural convection and radiation from the outer surface of the
glass surface. The natural convection heat transfer coefficient ( hg ,o ) is found from a free
convection heat transfer correlation using the FC_horizontal_cylinder function in EES. The
resistance to natural convection is given by:

1
Rconv = (2)
hg ,o π Dg ,o L

Call FC_horizontal_cylinder('Air', T_g_o, T_amb, P_amb, D_g_o : h_g_o, Nusselt_g_o, Ra_g_o)


"access natural convection correlation for a cylinder"
R_conv=1/(h_g_o*pi*D_g_o*L) "resistance to natural convection"

The resistance to radiation from the outer surface of the glass is:

1
Rrad , g ,o = (3)
π Dg ,o L σ (T 2
g ,o + Tamb
2
)(Tg ,o + Tamb )
The resistance to conduction through the glass is:

⎛D ⎞
ln ⎜ g ,o ⎟
⎜ Dg ,i ⎟
Rg = ⎝ ⎠ (4)
2 π kg L

The resistance to radiation from the copper tube to the glass is:

1
Rrad , gap = (5)
π Dc ,o L σ ε (T + Tg2,i )(Tc ,o + Tg ,i )
2
c ,o

R_rad_g_o=1/(sigma#*pi*D_g_o*L*(T_g_o^2+T_amb^2)*(T_g_o+T_amb))
"resistance to radiation from the outer surface of the glass"
R_g=ln(D_g_o/D_g_i)/(2*pi*k_g*L) "resistance of the glass wall"
R_rad_gap=1/(sigma#*epsilon*pi*D_c_o*L*(T_c_o^2+T_g_i^2)*(T_c_o+T_g_i))
"resistance to radiation from the outer surface of the glass"

The rate of heat loss is:

qloss =
(T c ,o − Tamb )
(6)
−1
⎛ 1 1 ⎞
Rrad , gap + Rg + ⎜ + ⎟⎟
⎜R
⎝ conv Rrad , g ,o ⎠

q_dot_loss=(T_c_o-T_amb)/(R_rad_gap+R_g+(1/R_conv+1/R_rad_g_o)^(-1)) "loss"

The guess values are updated and the guessed values for Tg,o and Tg,i are commented out. These
values are calculated according to:

Tg ,i = Tc ,o − qloss Rrad , gap (7)

Tg ,o = Tg ,i − qloss Rg (8)

{T_g_o=310 [K]} "guess temperature for outer glass temperature"


{T_g_i=315 [K]} "guess temperature for inner glass temperature"
T_g_i=T_c_o-q_dot_loss*R_rad_gap "recalculate inner glass temperature"
T_g_o=T_g_i-q_dot_loss*R_g "recalculate outer glass temperature"

The inner temperature of the copper tube is calculated according to:

Tc ,i = Tc ,o − ( qsolar − qloss ) Rc (9)

where Rc is the resistance to conduction through the copper:

⎛D ⎞
ln ⎜⎜ c ,o ⎟⎟
Dc ,i ⎠
Rc = ⎝ (10)
2 π kc L

R_c=ln(D_c_o/D_c_i)/(2*pi*k_c*L) "conduction resistance of copper"


T_c_i=T_c_o-(q_dot_solar-q_dot_loss)*R_c "inner temperature of copper"

The heat flux experienced at the inner surface of the copper tube is:

qc′′,i =
( qsolar − qloss ) (11)
π Dc ,i L

q``_c_i=(q_dot_solar-q_dot_loss)/(pi*D_c_i*L) "heat flux on inner surface of copper"


The guess values are updated and the assumed value of Tc,o is commented out. The
Nucleate_Boiling procedure is used to determine the value of Tc,o that causes the calculated heat
flux to agree with the heat flux that can be supported at the excess temperature:

{T_c_o=320 [K]} "guess temperature for outer tube temperature"


C_s_f=0.013 [-] "coefficient for nucleate boiling"
q``_c_i=Nucleate_Boiling('Water', T_w, T_c_i, C_s_f) "estimate heat flux"

The rate of energy transfer to the water is:

qwater = qsolar − qloss (12)

q_dot_water=q_dot_solar-q_dot_loss "rate of heat transfer to the water"

which leads to qwater = 22.4 W/m of collector.

b.) Calculate the efficiency of the solar collector, defined as the ratio of the rate of energy
transfer to the water in the heat pipe to the incident solar radiation.

The efficiency is calculated according to:

qwater
η= (13)
qsolar

eta=q_dot_water/q_dot_solar "efficiency"

which leads to η = 0.752 (75.2%).


Problem 7.3-1 (7-6 in text): Effect of Pressure Drop on Saturation Temperature
When one fluid is changing phase in a heat exchanger, it is commonly assumed to be at a
uniform temperature. However, there is a pressure drop in the evaporating fluid, which affects
its saturation temperature. In a particular case, a 2 m long horizontal concentric tube heat
exchanger made of copper is used to evaporate 0.028 kg/s of refrigerant R134a with an entering
state of 300 kPa with a quality of 0.35. Heat transfer is provided by a flow of water that enters
the heat exchanger at 12°C, 1.10 bar with a mass flow rate of 0.20 kg/s. The refrigerant passes
through the central tube of the heat exchanger, which has an inner diameter of 1.25 cm and a
wall thickness of 2 mm. The water flows through the annulus; the inner diameter of the outer
tube is 2.5 cm.
a) Estimate the outlet temperature of the water and the outlet temperature and quality of the
refrigerant.

Enter the known information into EES.

$UnitSystem SI MASS RAD PA K J


$TABSTOPS 0.2 0.4 0.6 0.8 3.5 in

"known information"
R$='R134a'
p_in=300 [kPa]*convert(kPa,Pa) "entering R134a pressure"
x_in=0.35 "entering quality"
d_i=1.25 [cm]*convert(cm,m) "inner diameter of inner tube"
d_o=2.5 [cm]*convert(cm,m) "inner diameter of outer tube"
delta=2 [mm]*convert(mm,m) "wall thickness"
m_dot_R=0.028 [kg/s] "R134a flow rate"
T_w_in=convertTemp(C,K,12 [C]) "water temperature"
P_w=1.1[bar]*convert(bar, Pa) "water pressure"
m_dot_w=0.2 [kg/s] "water temperature"
L=2 [m] "length of concentric tube"
T_r_in=T_sat(R$,P=P_in) "inlet temperature of refrigerant"

The outlet quality of the refrigerant (xout) is guessed in order to get started.

x_out=0.9 [-] "guess for outlet quality"

The mass flux of the refrigerant is computed:

4 m r
G= (1)
π di2

where di is the inner diameter of the inner tube. The function DELTAP_2phase_horiz is used to
estimate the pressure drop of the flowing refrigerant (Δpr). The refrigerant outlet pressure is
estimated according to:

pout = pin − Δpr (2)


Provided that the exiting refrigerant is two-phase, the exit temperature (Tr,out) is the saturation
temperature at pout. The heat transfer rate is computed according to:

(
q = m r ir ,T =Tr ,out , x = xout − ir ,T =Tr ,in , x = xin ) (3)

where ir is the enthalpy of the refrigerant.

G=m_dot_R/(pi*d_i^2/4) "mass velocity of refrigerant"


DELTAP=DELTAP_2phase_horiz(R$,G,P_in,d_i,L,x_in, x_out)
"determine pressure drop for refrigerant"
p_out=p_in-DELTAP "outlet pressure"
T_r_out=T_sat(R$,P=p_out) "outlet temperature of refrigerant"
q_dot=m_dot_R*(enthalpy(R$,T=T_r_out,x=x_out)-enthalpy(R$,T=T_r_in,x=x_in)) "rate of heat transfer"

The inner and outer radii of the annulus are calculated according to:

ri =
( di + 2 δ ) (4)
2

do
ro = (5)
2

The average heat transfer coefficient on the water side ( hw ) is obtained using the AnnularFlow
procedure. The resistance to convection on the water side is:

1
Rconv , w = (6)
2 π ri L hw

r_i=(d_i+2*delta)/2 "inner radius of annulus"


r_o=d_o/2 "outer radius of annulus"
RelRough=0 [-] "relative roughness -assumed"
call AnnularFlow('water',T_w_in,P_w,m_dot_w,r_i,r_o,L,RelRough:h_bar_w, h_H ,&
DELTAP_w, Nusselt_T, f, Re) "determines heat transfer coefficient on water side"
R_conv_w=1/(2*pi*r_i*L*h_bar_w) "resistance to convection on water side"

The resistance to conduction through the tube is:

⎛ d + 2δ ⎞
ln ⎜ i ⎟
Rcond = ⎝ di ⎠
(7)
2 π ktube L

k_tube=k_('Copper', T_r_in) "conductivity of copper"


R_cond=ln((d_i+2*delta)/d_i)/(2*pi*k_tube*L) "resistance to conduction through tube"
The heat transfer coefficient on the refrigerant side ( hr ) depends on the heat flux, which is
computed according to:

q
q ′′ = (8)
π di L

The heat transfer coefficient is computed using the Flow_Boiling_avg function and used to
compute the resistance to convection on the refrigerant side:

1
Rconv ,r = (9)
π di L hr

q``_dot=q_dot/(pi*d_i*L) "heat flux on refrigerant side"


h_bar_r=Flow_Boiling_avg(R$, T_r_in, G, d_i, x_in, x_out, q``_dot, 'Horizontal')
"heat transfer coefficient on refrigerant side"
R_conv_r=1/(pi*d_i*L*h_bar_r) "resistance to convection on refrigerant side"

The overall conductance is computed:

UA = ( Rconv ,r + Rcond + Rconv , w )


−1
(10)

The performance is computed assuming that the water is exposed to a uniform external
temperature that is the average refrigerant temperature:

Tr ,in + Tr ,out
Tr = (11)
2

so the outlet water temperature is:

⎛ UA ⎞
Tw,out = Tr − (Tr − Tw,in ) exp ⎜ − ⎟ (12)
⎝ m w cw ⎠

where cw is the specific heat capacity of the water.

UA=(R_conv_w+R_cond+R_conv_r)^(-1) "overall conductance"


T_r_avg=(T_r_in+T_r_out)/2 "average refrigerant temperature"
c_w=cP(Water,T=T_w_in,P=P_w) "specific heat capacity of water"
T_w_out=T_r_avg-(T_r_avg-T_w_in)*exp(-UA/(m_dot_w*c_w)) "water outlet temperature"

The guess values are updated and the assumed value of xout is commented out. The rate of heat
transfer is computed using an energy balance on the water side:

q = m w cw (Tw,in − Tw,out ) (13)


{x_out=0.9 [-]} "guess for outlet quality"
q_dot=m_dot_w*c_w*(T_w_in-T_w_out) "recalculate heat transfer rate"

which leads to Tw,out = 283.5 K, xout = 0.595, and Tr,out = 273.7 K.


Problem 7.3-2 (7-7 in text): Circular Finned-Tube Evaporator
A circular finned tube evaporator designed for cooling air is made from aluminum. The outer
diameter of the tubes is 10.21 mm with a tube wall thickness of 1 mm. There evaporator is
plumbed such that there are 12 parallel circuits of tubes with each circuit having a length of 0.6
m. Refrigerant R134a enters the evaporator at a mass flowrate of 0.15 kg/s. The refrigerant
enters the throttle valve upstream of the evaporator as 35°C saturated liquid. The pressure in the
evaporator is 240 kPa. The refrigerant exits as a saturated vapor.
a) What is the rate of heat transfer to the air for this evaporator.

Enter the problem information into EES.

$UnitSystem SI MASS RAD PA K J


$TABSTOPS 0.2 0.4 0.6 0.8 3.5 in

"known information"
d_o=10.21 [mm]*convert(mm,m) "outer diameter of the tube"
thickness=1 [mm]*convert(mm,m) "tube thickness"
d_i=d_o-2*thickness "inner diameter of the tube"
m_dot_R=0.15 [kg/s] "refrigerant mass flow rate"
P_evap_i=240 [kPa]*convert(kPa,Pa) "inlet pressure"
T_sat=T_sat(R134a,P=P_evap_i) "saturation temperature"
T_cond=converttemp(C,K,35 [C]) "condenser temperature"
L=0.6 [m] "length"
N_circuits=8 "number of parallel circuits"

The heat transfer rate is found from an energy balance on the refrigerant.

(
q = m r ir ,T =Tcond , x =0 − ir , P = pevap ,i , x =1 ) (1)

"a) Estimate the heat transfer rate"


h_i=enthalpy(R134a,T=T_cond,x=0) "enthalpy of entering refrigerant"
h_e=enthalpy(R134a,P=P_evap_i,x=1) "enthalpy of exiting refrigerant"
q_dot=m_dot_R*(h_e-h_i) "heat transfer rate"

which leads to q = 21.96 kW.

b) Estimate of the average heat transfer coefficient between the R134a and the tube wall

The heat transfer coefficient between the R134a and the tube wall is found using the
Flow_boiling_avg function. The heat flux needed as an input is the heat transfer rate divided by
the surface area of the tubes, considering the 8 parallel circuits.

q
q ′′ = (2)
N π di

and the mass flux is:


m r
G= (3)
N Atube

where Atube is the cross-sectional area of a tube:

di2
Atube =π (4)
4

The inlet quality to the evaporator is the quality leaving the valve, specified by the enthalpy of
the refrigerant entering the valve and the evaporator pressure.

"b) Estimate the heat transfer coefficient"


A_tube=pi*d_i^2/4 "cross-sectional area of tube"
G=m_dot_R/(A_tube*N_circuits) "mass flux"
x_i=quality(R134a,h=h_i,P=P_evap_i) "inlet quality"
x_e=1 [-] "exit quality"
q``=q_dot/(N_circuits*pi*d_i*L) "heat flux"
h_bar=Flow_Boiling_avg('R134a', T_sat, G, d_i, x_i, x_e, q``, 'Horizontal')
"average heat transfer coefficient"

c) Estimate the pressure drop of the R134a as it pass through the evaporator. Does this pressure
drop significantly affect the saturation temperature?

The pressure drop calculation requires use of a 2-phase pressure drop correlation which is
provided in the DELTAP_2phase_horiz function.

"c) estimate the pressure drop"


DELTAP=DELTAP_2phase_horiz('R134a',G,P_evap_i,d_i,L,x_i, x_e) "pressure drop"
T_sat_out=temperature('R134a',P=P_evap_i-DELTAP,x=x_e) "exit saturation temperature"

The pressure drop is 13.86 kPa and the change in the saturation temperature is 1.5 K.
Problem 7.3-3: Solar Electric Generation System (revisited)
Problem 5.3-4 describes a solar electric generating system (SEGS) plant that uses solar collectors
to heat oil. Use of the oil is problematic since a heat exchanger is required between the oil and
the steam that is generated to operate the power cycle. In this problem we wish to investigate the
option of boiling water directly within the tubes of the parabolic collectors. As in problem 5.3-4,
we will analyze N = 50 tubes, each with an inner diameter of Di = 0.066 m and a length of L =
750 m. The outer diameter of the tube is essentially equal to the inner diameter and the outer
surface of these tubes is exposed to a uniform heat flux of qs′′ = 15,000 W/m2. In problem 5.3-4,
each tube has oil (with specific heat of 2341 J/kg-K and a density of 825 kg/m3) flowing through
it with a volumetric flow rate (per tube) of V = 0.012 m3/s. The oil enters the tube with a mean
temperature of Tin = 500 K and exits at 600 K. The pump power for all 50 tubes with a pump
having an efficiency of 0.5 is about 0.9 MW. Assume that, instead of the oil, saturated liquid
water at 600 K enters the tubes and exits as saturated vapor.
a.) Compute the necessary mass flow rate of water so that the rate of energy transfer to the water
is the same as that for the oil.

Enter problem information into EES.

$UnitSystem SI MASS RAD PA K J


$TABSTOPS 0.2 0.4 0.6 0.8 3.5 in

"Inputs"
rho_oil=825 [kg/m^3] "density of oil"
mu=0.0087 [Pa-s] "viscosity of oil"
k = 0.134 [W/m-K] "conductivity of oil"
Pr = 152 [-] "Prandtl number of oil"
V_dot_oil=0.012 [m^3/s] "volumetric flow rate of oil"
D_i=0.066 [m] "inner diameter of pipe"
N = 50 [-] "number of loops"
L = 750 [m] "length of a single loop"
T_in_oil = 500 [K] "mean temperature of oil entering loop"
qf_dot_s=15000 [W/m^2] "heat flux incident on the absorber tube surface"
c_oil=2341 [J/kg-K] "specific heat of the oil"
m_dot_oil=rho_oil*V_dot_oil "mass flow rate of oil in a single loop"
T_out_oil=600 [K] "leaving mean oil temperature"
T_w_in=600 [K] "entering temperature of the water"
T_w_out=T_w_in
"assume that the temperature at the outlet is unchanged by the pressure drop"
x_in=0 [-] "saturated liquid at inlet"
x_out=1 [-] "saturated vapor at exit"
P_in=pressure(Water,T=T_w_in,x=x_in) "pressure at tube inlet"

The total rate of heat transfer delivered to the plant is:

q plant = N m oil coil (Toil ,out − Toil ,in ) (1)

The mass flow rate of water required to deliver the same amout of heat transfer is:
( )
N m water iw,T =Tw ,out , x =1 − iw,T =Tw ,in , x =0 = q plant (2)

q_dot_plant=N*m_dot_oil*c_oil*(T_out_oil-T_in_oil)
"total rate at which thermal energy is delivered to plant"
N*m_dot_water*(enthalpy(Water,T=T_w_out,x=1)-enthalpy(Water,T=T_w_in,x=0))=q_dot_plant
"energy balance to determine water flow per tube"

which leads to m water = 1.98 kg/s per tube.

b.) Calculate the heat transfer coefficient on the inside surface of the tube when water is used as
the heat transfer fluid and compare it to the heat transfer coefficient using oil.

The heat transfer coefficient hw is found using the Flow_boiling_Avg function, which determines
the average value of the 2-phase heat transfer coefficient for qualities ranging from 0 to 1. The
mass flux is:

4 m water
G= (3)
π Di2

G=m_dot_water/(pi*D_i^2/4) "mass flux"


h_bar_water=Flow_Boiling_avg('Water', T_w_in, G, D_i, x_in, x_out, qf_dot_s, 'Horizontal') "heat transfer
coefficient for 2-phase water flow"

which leads to hw = 13830 W/m2-K.

The mean velocity of the oil is:

4 Voil
um,oil = (4)
π Di2

The Reynolds number for flowing oil is:

um ,oil Di ρ oil
Re = (5)
μoil

The PipeFlow_N function is used to obtain the Nusselt number ( Nu ). The heat transfer
coefficient is obtained from:

Nu koil
hoil = (6)
Di

u_m_oil=V_dot_oil/(pi*D_i^2/4) "mean velocity of the oil"


Re_oil=u_m_oil*D_i*rho_oil/mu "Reynolds number of oil"
call PipeFlow_N(Re_oil,Pr,L/D_i,0 [-]: Nusselt_T,Nusselt_H,f) "Nusselt number for oil flow"
h_bar_oil=Nusselt_H*k/D_i "heat transfer coefficient of oil"

which leads to hoil = 998 W/m2-K (more than 10x less than the two-phase flow of water).

c.) Determine the pressure drop across each of the 50 tubes

A 2-phase pressure drop correlation is provided in the EES function DELTAP_2phase_horiz.

Fluid$='Water'
DELTAP=DELTAP_2phase_horiz(Fluid$, G, P_in, D_i, L, x_in, x_out) "pressure drop across each tube"

which leads to Δp = 211.8 kPa.

d.) Calculate the outlet temperature of the steam

Knowing the pressure of the water at the outlet and knowing that it is saturated determines the
outlet temperature.

P_out=P_in-DELTAP "pressure at outlet"


T_w_out_act=temperature(Water,P=P_out,x=1) "temperature at outlet - note how little the temperature
has dropped"

which leads to Tw,out = 598.7 K.

e.) Estimate the minimum pump power required for all 50 tubes and compare the result with the
minimum pump power needed for the oil. Note that the pump that provides the pressure
operates with liquid water at 35°C.

The minimum pump power is the product of the mass flow rate, inlet specific volume and
pressure drop. The actual pump power is the minimum pump power divided by the pump
efficiency.

N m water vin Δpwater


w pump = (7)
η pump

where vin is the specific volume of the water and ηpump is the efficiency of the pump (assumed to
be 50%).

eta_pump=0.5 "pump efficiency"


W_dot_pump=N*m_dot_water*v_in*DELTAP/eta_pump "pump power for all 50 tubes"
v_in=volume(Water,T=convertTemp(C,K,35 [C]),x=0) "specific volume of liquid water"

which leads to w pump = 42.1 kW.


Problem 7.3-4 (7-8 in text)
Repeat EXAMPLE 7.3-1 using the Flow_Boiling_avg function rather than integrating to determine
the average heat transfer coefficient. Compare the two methods of obtaining the average heat
transfer coefficient.

The relevant information from EXAMPLE 7-3-1 is entered into EES.

$UnitSystem SI MASS RAD PA K J


$Tabstops 0.2 0.4 0.6 3.5 in

F$='CarbonDioxide' "fluid type"


D=2.5 [mm]*convert(mm,m) "tube inner diameter"
L=2 [m] "tube length"
p_sat=3.2 [MPa]*convert(MPa,Pa) "boiling saturation pressure"

The mass flux will be varied in a parametric table, but a particular value will be assigned to test
the program.

$ifnot ParametricTable
G=300 [kg/s-m^2] "mass velocity"
$endif

Both the integrated heat transfer coefficient and the result from the Flow_Boiling_avg function are
calculated and compared. The average value of the heat transfer coefficient it found by
calculating the value as a function of quality and integrating for qualities between 0 and 1, as
described in EXAMPLE 7.3.1. The Flow_Boiling_Avg function provides the result directly. The
two method provide essentially identical results, as shown in Figure 1.

T_sat=T_sat(F$,P=p_sat) "saturation temperature"


i_v_sat=enthalpy(F$,T=T_sat,x=1) "specific enthalpy of saturated vapor"
i_l_sat=enthalpy(F$,T=T_sat,x=0) "specific enthalpy of saturated liquid"
DELTAi_vap=i_v_sat-i_l_sat "enthalpy change of vaporization"
q_dot=G*pi*D^2/4*DELTAi_vap "rate of heat transfer"
q``_dot_s=q_dot/(pi*D*L) "heat flux"
call Flow_Boiling(F$, T_sat, G, D, x, q``_dot_s, 'Horizontal': h, T_w) "heat transfer coefficient"
x_in=0
x_out=1
h_bar`=Flow_Boiling_avg(F$, T_sat, G, D, x_in, x_out, q``_dot_s, 'Horizontal')
h_bar=G*D*DELTAi_vap*INTEGRAL(h,x,x_in,x_out)/(4*L*q``_dot_s) "average heat transfer coefficient"
8000

Average heat transfer coefficient (W/m -K)


Flow Boiling Avg function

2
EXAMPLE 7.3-1
7000

6000

5000

4000

3000

2000
1.5x104 2.0x104 2.5x104 3.0x104 3.5x104 4.0x104
2
Heat flux (W/m )
Figure 1: Average heat transfer coefficient computed using the Flow_Boiling_Avg function or using the
integration described in EXAMPLE 7.3-1.
Problem 7.3-5 (7-9 in text): Electronic Cooling Alternative
A computer manufacturer is reviewing alternative ways to remove heat from electronic
components. The electronic circuit board can be assumed to be a thin horizontal plate with a
width of 8 cm and a length of 16 cm. Currently, air is blown over the top of the circuit board at a
velocity of 10 m/s. Additional cooling could be obtained by a higher air velocity, but the
increased noise associated with the larger fan required is judged to be unacceptable. Another
alternative is to immerse the board in a fluid at atmospheric pressure that is undergoing nucleate
boiling. The fluid R245fa has been chosen as a possibility. The surface tension of R245fa at
atmospheric pressure is 0.0153 N/m. Other thermodynamic and transport properties are
available from EES. Prepare a plot that shows the surface temperature of the plate as a function
of the heat flux using air at 10 m/s and nucleate boiling at atmospheric pressure with R245fa for
heat fluxes ranging from 100 to 10000 W/m2.

Known information is entered into EES along with a specified heat flux. The heat flux will later
be varied in a Parametric table.

"known information"
W=8 [cm]*convert(cm,m) "width of plate"
L=16 [cm]*convert(cm,m) "length of plate in flow direction"
u_inf=10 [m/s] "air velocity"
T_air_inf=convertTemp(C,K,25 [C]) "air inlet temperature"
P=101.3 [kPa]*convert(kPa,Pa) "atmospheric pressure"
$ifnot ParametricTable
q``=1000 [W/m^2] "heat flux"
$endif

The heat transfer coefficient if air is used as the coolant is found from the External_Flow_Plate
function that implements a convective flow relation for external flow over a flat horizontal plate
( ha ). Since the heat flux is known, the energy balance determines the surface temperature.

q ′′ = ha (Ts ,air − T∞ ,air ) (1)

Call External_Flow_Plate('Air', T_air_inf, T_s_air, P, u_inf, L: tau, h_air, C_f, Nusselt, Re)
q``=h_air*(T_s_air-T_air_inf) "determine surface temperature with air cooling"

Complete property data are not provided for R234fa in the EES data base, making it necessary to
implement a function that calculates the heat flux according to Eq. (7-3) in the text. This
implementation has been provided as a function. Enter the following function at the top of the
EES equations window.

Function Nucleate_BoilingR245fa(Fluid$, T_sat, T_w, C_s_f)


rho_l=density(Fluid$,T=T_sat,x=0) "density of saturated liquid"
rho_g=density(Fluid$,T=T_sat,x=1) "density of saturated vapor"
k_l=conductivity(Fluid$,T=T_sat,x=0) "thermal conductivity of saturated liquid"
mu_l=viscosity(Fluid$,T=T_sat,x=0) "thermal conductivity of saturated vapor"
c_l=cp(Fluid$,T=T_sat,x=0) "specific heat oof saturated liquid"
Pr_l=Prandtl(Fluid$, T=T_sat, x=0) "Prandtl number for saturated liquid"
h_fg=enthalpy(Fluid$,T=T_sat,x=1)-enthalpy(Fluid$,T=T_sat,x=0) "enthalyp of vaporization"
n=1.7 "exponent on Prandtl number"
sigma_1=0.0153 [N/m] "surface tension for R245fa"
DELTAT_e=T_w-T_sat "excess temperature difference"
g=g# "gravitational acceleration"
Nucleate_BoilingR245fa=mu_l*h_fg*((g*(rho_l-
rho_g))/(sigma_1))^(1/2)*((c_l*DELTAT_e)/(C_s_f*h_fg*Pr_l^n))^3
END

The function is accessed with the following EES statement at the bottom of the Equations
window.

Fluid$='R245fa'
T_sat=T_sat(Fluid$,P=P) "saturation temperature"
C_s_f=0.012 "nucleate boiling coefficient"
q``=Nucleate_BoilingR245fa(Fluid$, T_sat, T_s_R, C_s_f) "determines surface temperature with
nucleate boiling"

Figure 1 illustrates the surface temperatures with air and R245fa cooling.
650

600 air
Surface temperature (K)

550

500

450

400

350
R245fa
300

250
0 2000 4000 6000 8000 10000
2
Heat flux (W/m )
Figure 1: Surface temperature for air and R245fa cooling as a function of heat flux.
Problem 7.3-6
The refrigerant R134a is evaporating as it flows through a pipe with inner diameter D = 0.5 inch.
The mass flow rate is m = 40 lbm/min and the saturation temperature is Tsat = 54°F. The heat
flux at the surface of the tube is qs′′ = 5000 Btu/hr-ft2.
a.) Plot the local heat transfer coefficient as a function of the quality of the R134a.

The inputs are entered in EES:

$UnitSystem English psi F


$TabStops 0.5 3.5 in

D=0.5 [inch]*convert(inch,ft) "diameter of pipe"


m_dot=40 [lbm/min]*convert(lbm/min,lbm/hr) "mass flow rate"
F$='R134a' "fluid"
T_sat=54 [F] "saturation temperature"
q``=5000 [Btu/hr-ft^2] "surface heat flux"

An arbitrary value of quality, x, is set. The mass flux is computed according to:

4 m
G= (1)
π D2

The Flow_Boiling procedure is used to determine the local heat transfer coefficient, h.

x=0.5 [-] "quality"


G=m_dot/(pi*D^2/4) "mass flux"
call Flow_Boiling(F$, T_sat, G, D, x, q``, 'Horizontal': h, T_w) "local heat transfer coefficient"

Figure 1 illustrates the local heat transfer coefficient as a function of quality.


4000
Local heat transfer coefficient (Btu/hr-ft -R)
2

3500

3000

2500

2000

1500

1000

500
0 0.1 0.2 0.3 0.4 0.5 0.6 0.7 0.8 0.9 1
Quality
Figure 1: Local heat transfer coefficient as a function of quality.
b.) Determine the average heat transfer coefficient in the pipe if the R134a enters the pipe as
saturated liquid and leaves as saturated vapor. Use the function Flow_Boiling_avg.

The function Flow_Boiling_avg is used to evaluate the average heat transfer coefficient, h :

h_bar=Flow_Boiling_avg(F$, T_sat, G, D, 0 [-], 1 [-], q``, 'Horizontal') "average heat transfer coefficient"

which leads to h = 2974 Btu/ft2-R-hr.


Problem 7.4-1 (7-10 in text): Evacuated Tubular Solar Collectors with a Heat Pipe
Evacuated tubular solar collectors often employ a heat pipe to transfer collected solar energy for
water heating. Heat transfer between the water that is being heated and the solar collector occurs
at the condenser of the heat pipe, which is a thin-walled cylinder made of copper with a length of
6 cm and a diameter of 1 cm as shown in Figure P7.4-1. Water at 40°C and 1 atm flows past the
condenser at a velocity that can be specified by the flow rate and duct diameter. The fluid inside
the heat pipe is also water and it condenses at a pressure of 100 kPa. The heat transfer situation
of the condensing water within the heat pipe is not known, but will here be assumed that is can
be represented with the same relations as used for film condensation on the inside surface of a
cylinder. This heat transfer coefficient for film condensation is provided by the
Cond_horizontal_Cylinder procedure when the mass flow rate is set to 0. Plot the rate of heat
transfer from the solar collector to the water that is being heated as a function of the flow
velocity of the water for velocities between 1 and 10 m/s.

water at 40°C, 100 kPa, u w

solar collector

6 cm
1 cm
condenser
Figure P7.4-1: Condenser of evacuated solar collector.

The information in the problem statement is entered into EES. A velocity of 5 m/s is selected
initially. After the equations are entered this specification will be replaced with a Parametric
table that varies the velocity from 1 to 10 m/s.

$UnitSystem SI MASS RAD PA K J


$Tabstops 0.2 0.4 0.6 3.5 in

L=6 [cm]*convert(cm,m) "length of condenser"


D=1 [cm]*convert(cm,m) "diameter of condenser"
Fluid$='Water' "fluid in pipe"
P_sat=1 [atm]*convert(atm,Pa) "pressure inside of the heat pipe"
T_sat=T_sat(Fluid$,P=P_sat) "saturation temperature inside the heat pipe"

P=100 [kPa]*convert(kPa,Pa) "atmospheric pressure"


T_inf=convertTemp(C,K,40[C]) "inlet water temperature"
$ifnot ParametricTable
u_inf=5 [m/s] "inlet water velocity"
$endif

The surface area of the cylinder in contact with the water is determined.

A=π DL (1)
A=pi*D*L "surface area of the cylinder"

An initial guess for the wall temperature (Tw) is made in order to estimate the heat transfer
coefficient associated with film condensation and external forced convection.

T_w=330 [K] "guess for wall temperature"

As indicated in the problem statement, the heat transfer coefficient for film condensation ( hi ) can
be found using the Cond_Horizontal_Cylinder procedure with the mass flow rate set to 0. The
resistance to film condensation is:

1
Rconv ,i = (2)
hi A

Call Cond_horizontal_Cylinder(Fluid$, T_sat, T_w, D:h_i, Nusselt_m)


"heat transfer coefficient for film condensation"
R_conv_i=1/(h_i*A) "resistance to convection on inside of tube"

The heat transfer coefficient on the outside of the cylinder ( ho ) is found using an external flow
convection relation provided in the External_Flow_Cylinder procedure. The resistance to
convection on the outer surface of the tube is:

1
Rconv ,o = (3)
ho A

Call External_Flow_Cylinder(Fluid$, T_inf, T_w, P, u_inf, D: F_d\L, h_o, C_d, Nusselt, Re)
"heat transfer coefficient on outer surface"
R_conv_o=1/(h_o*A) "resistance to convection on outside of tube"

The rate of heat transfer is given by:

q =
(Tsat − T∞ ) (4)
Rconv ,i + Rconv ,o

q_dot=(T_sat-T_inf)/(R_conv_i+R_conv_o) "rate of heat transfer"

The guess values are updated and the assumed value of Tw is commented out. Note that this
program works best if limits are placed on the value of Tw (from a lower limit of T∞ to an upper
limit of Tsat). The wall temperature is calculated according to:

Tw = Tsat − Rconv ,i q (5)

{T_w=330 [K]} "guess for wall temperature"


T_w=T_sat-R_conv_i*q_dot "recalculate wall temperature"
A parametric table with columns for u_inf and q_dot is constructed. Figure 2 illustrates the rate of
heat transfer as a function of the water velocity.

950

900

850
Heat transfer rate (W)

800

750

700

650

600

550
1 2 3 4 5 6 7 8 9 10
Water velocity (m/s)
Figure 2: Heat transfer rate as a function of water velocity.
Problem 7.4-2 (7-11 in text): Steam Power Cycle Condenser
The condenser in steam power cycle utilizes a shell and tube heat exchanger that consists of 1200
nominal 1.5 inch schedule 40 tubes made of brass. Each tube is 8 ft long internally smooth.
Cooling water enters each of the tubes at 68°F and exits at 74°F. Saturated steam at 1 psia
having a quality of 91% enters the condenser at a low velocity and is condensed on the tubes.
Estimate the rate of condensate formation and the associated water flow rate at steady state
conditions.

Enter the known information into EES.

$UnitSystem SI MASS RAD PA K J


$Tabstops 0.2 0.4 0.6 3.5 in

"Known"
N=1200 [-] "number of tubes"
L=8 [ft]*convert(ft,m) "length of tubes"
P_sat=1 [psi]*convert(psi,Pa) "saturation temperature of steam"
x_in=0.91 [-] "inlet quality"
T_w_in=converttemp(F,K,68 [F]) "inlet water temperature"
T_w_out=converttemp(F,K,74 [F]) "exit water temperature"
D_i=1.61 [in]*convert(in,m) "inner diameter of 1.5 in Schedule 40 pipe"
delta=0.145 [in]*convert(in,m) "wall thickness"
D_o=D_i+2*delta "outer diameter of 1.5 in Schedule 40 pipe"
T_w_avg=(T_w_in+T_w_out)/2 "average temperature of water in pipe"
k_w=k_('brass',T=(T_sat+T_w_avg)/2) "thermal conductivity of brass"
P_atm=14.7 [psi]*convert(psi,Pa) "atmospheric pressure"

The saturation temperature of the water can be determined using the EES T_sat function.

T_sat=T_sat('Water',P=P_sat) "saturation temperature"

The wall temperature (Twall,o) is needed to determine the condensation heat transfer coefficient,
but it is not known. Provide a guess for this temperature.

T_wall_o=converttemp(F,K,101 [F]) "guess for outer pipe wall temperature"

The Cond_horizontal_N_Cylinders function is used to determine the heat transfer coefficient for
condensation over a bank of cylinders ( hcond ). The resistance to condensation is:

1
Rcond = (1)
hcond π Do L N

Call Cond_horizontal_N_Cylinders('Water', T_sat, T_wall_o, D_i, N:h_cond, Nusselt_cond)


"determine condensation heat transfer coefficient"
R_cond=1/(h_cond*pi*D_o*L*N) "resistance to condensation"

The resistance to conduction through the tube is:


⎛D ⎞
ln ⎜ o ⎟
Rtube = ⎝ Di ⎠ (2)
2 π kw L N

where kw is the conductivity of the tube.

R_tube=ln(D_o/D_i)/(2*pi*k_w*L*N) "resistance to conduction"

The properties of the cooling water should be evaluated at the average water temperature. The
PipeFlow procedure implements an internal flow correlation that determines the heat transfer
coefficient to the cooling water ( hw ). A mass flow rate of water must be assumed to carry out
this calculation. The resistance to convection to the flowing water is:

1
Rconv ,i = (3)
hw π Di L N

m_dot_w=0.1 [kg/s] "guess for total water mass flow rate"


RelRough=1e-8 "relative roughness for smooth tube"
call PipeFlow('Water',T_w_avg,P_atm,m_dot_w/N,D_i,L,RelRough:h_w, h_H ,DELTAP, Nusselt_T, f, Re)
R_conv_i=1/(h_w*pi*D_i*L*N) "resistance to convection on inside of tube"

The overall conductance of the heat exchanger is calculated according to:

UA = ( Rcond + Rtube + Rconv ,i )


−1
(4)

and used to determine the outlet water temperature according to the method discussed in Section
5.3.4.

⎛ UA ⎞
Tw,out = Tsat − (Tsat − Tw,in ) exp ⎜ − ⎟ (5-5)
⎝ m w cw ⎠

where cw is the specific heat capacity of the water.

UA=(R_cond+R_tube+R_conv_i)^(-1) "conductance"
c_w=cp(Water,T=T_w_avg,P=P_atm) "specific heat of water"
T_w_out=T_sat-(T_sat-T_w_in)*exp(-UA/(m_dot_w*c_w)) "outlet water temperature"

The problem is solved and the guess values are updated. The assumed mass flow rate is
commented out and the mass flow rate is recalculated using an energy balance.

m w cw (Tw,out − Tw,in ) = q (6)

{m_dot_w=0.1 [kg/s]} "guess for total water mass flow rate"


m_dot_w*c_w*(T_w_out-T_w_in)=q_dot "energy balance on water"
The problem is solved and the guess values are updated. The assumed wall temperature is
commented out and the wall temperature is estimated according to:

Twall ,o = Tsat − q Rcond (7)

{T_wall_o=converttemp(F,K,101 [F])} "guess for outer pipe wall temperature"


T_wall_o=T_sat-q_dot*R_cond "recalculate wall temperature"

which leads to m w = 55.25 kg/s. The mass flow rate of condensate is estimated using an energy
balance:

q
m condensate = (8)
(i
water ,T =Tsat , x = xin − iwater ,T =Tsat , x =0 )
m_dot_condensate=q_dot/(enthalpy(water,T=T_sat,x=x_in)-enthalpy(Water,T=T_sat,x=0))
"mass flow rate of condensate"

which leads to m condensate = 0.352 kg/s.


Problem 7.4-3 (7-12 in text): R245fa Recovery
Problem 7.3-5 (7-9 in text) described an electronics cooling system that removes the heat
dissipated in an electronic circuit board by submersing the board in R245fa. The circuit board is
maintained at a relatively low and uniform temperature over a range of heat fluxes by boiling
R245fa. However, a problem now arises in dealing with the vapor produced by the evaporation.
One possibility is to condense the vapor on the bottom side of a vertical plate that is cooled by
chilled water on its top side, as shown in Figure P7.4-3 in a sealed container. The top of the
enclosure is made of metal and it can be considered to be isothermal. The chilled water is at 1
atm and has a free stream velocity of 10 m/s and a free stream temperature of 10°C. The circuit
board is 8 cm wide and 16 cm long. The saturation pressure (and thus temperature) of the
R245fa in the enclosure should vary with the heat flux.

water at
10°C, 1 atm, 10 m/s

16 cm
saturated R245fa

circuit board
8 cm x 16 cm
Figure P7.4-3: Sealed container full of evaporating and condensing R245fa.

a.) Prepare a plot of the saturation pressure and circuit board surface temperature as a function
of the heat flux for heat fluxes ranging from 100 to 10000 W/m2.

Enter known information into EES. The heat flux will initially be set to a 8,000 W/m2. It will be
later varied in a Parametric table.

$UnitSystem SI MASS RAD PA K J


$Tabstops 0.2 0.4 0.6 3.5 in

"known"
Fluid$='R245fa'
W=8 [cm]*convert(cm,m) "width of plate"
L=16 [cm]*convert(cm,m) "length of plate in flow direction"
T_w_inf=convertTemp(C,K,10 [C]) "water inlet temperature"
P=101.3 [kPa]*convert(kPa,Pa) "atmospheric pressure"
q``=8000 [W/m^2] "heat flux"
u_inf=10 [m/s] "water velocity"

The heat transfer coefficient for the top plate ( hw ) is found using an external flow relation for
flow over a flat plate provided in the External_Flow_Plate function. The plate temperature, Tplate,
is needed as an input, but it is not yet known. Set Tplate to a reasonable value, e.g., 290 K.
T_plate=290 [K] "guess for plate temperature"
Call External_Flow_Plate('Water', T_w_inf, T_plate, P, u_inf, L: tau, h_bar_w, C_f, Nusselt, Re)

The equations are solved and the guess values updated. The assumed value of Tplate is
commented out and the plate temperature is recalculated using Newton's law of cooling:

q ′′ = hw (Tplate − T∞ ) (1)

{T_plate=290 [K]} "guess for plate temperature"


q``=h_bar_w*(T_plate-T_w_inf) "recalculate plate temperature"

Because the surface tension of R245fa is not available in EES, it is necessary to write a function
Cond_horizontal_downR245fa that is identical to the built-in function Cond_horizontal_down except
that the surface tension is specified:

Procedure Cond_horizontal_downR245fa(Fluid$, T_w, T_sat: h_m, Nusselt_m)


DELTAT=(T_sat-T_w) "temperature difference"
if (DELTAT<0) then DELTAT=0.001 "ensure that the temperature difference is positive"
T_f=(T_sat+T_w)/2
"average temperature of fluid - used in determining fluid properties"
rho_l=density(Fluid$,T=T_f,x=0) "density of saturated liquid"
rho_g=density(Fluid$,T=T_f,x=1) "density of saturated vapor"
k_l=conductivity(Fluid$,T=T_f,x=0) "conductivity of saturated liquid"
mu_l=viscosity(Fluid$,T=T_f,x=0) "viscosity of saturated liquid"
c_l=cp(Fluid$,T=T_f,x=0) "specific heat of saturated liquid"
h_fg=enthalpy(Fluid$,T=T_f,x=1)-enthalpy(Fluid$,T=T_f,x=0) "
sigma=0.0153 [N/m] "surface tension"
Ra=g#*rho_l*(rho_l-rho_g)*h_fg/(mu_l*DELTAT*k_l)*(sigma/(g#*(rho_l-rho_g)))^(3/2)
"Rayleigh number"
If (Ra<1e8) then
Nusselt_m=0.69*Ra^(0.20) "Eq. (7-91)"
else
Nusselt_m=0.81*Ra^(0.193) "Eq. (7-92)"
endif
h_m=Nusselt_m*k_l*(sigma/(g#*(rho_l-rho_g)))^(-1/2) "
END

The saturation temperature is required in order to use the condensation correlation; assume a
reasonable value of Tsat and then call the procedure Cond_horizontal_downR245fa in order to
evaluate the heat transfer coefficient associated with condensation, hcond .

T_sat=300 [K] "guess for saturation temperature"


Call Cond_horizontal_downR245fa(Fluid$, T_plate, T_sat: h_bar_cond, Nusselt_cond)

The equations are solved and the guess values updated. The assumed value of Tsat is commented
out and the saturation temperature is recalculated using Newton's law of cooling:

q ′′ = hcond (Tsat − Tplate ) (2)


The saturation pressure (psat) is obtained using the saturation temperature.

q``=h_bar_cond*(T_sat-T_plate) "recalculate saturation temperature"


p_sat=pressure(Fluid$,T=T_sat,x=0) "saturation pressure"

The function Nucleate_BoilingR245fa is copied from Problem 7.3-5 (7-9 in text)

Function Nucleate_BoilingR245fa(Fluid$, T_sat, T_w, C_s_f)


rho_l=density(Fluid$,T=T_sat,x=0) "density of saturated liquid"
rho_g=density(Fluid$,T=T_sat,x=1) "density of saturated vapor"
k_l=conductivity(Fluid$,T=T_sat,x=0) "thermal conductivity of saturated liquid"
mu_l=viscosity(Fluid$,T=T_sat,x=0) "thermal conductivity of saturated vapor"
c_l=cp(Fluid$,T=T_sat,x=0) "specific heat of saturated liquid"
Pr_l=Prandtl(Fluid$, T=T_sat, x=0) "Prandtl number for saturated liquid"
h_fg=enthalpy(Fluid$,T=T_sat,x=1)-enthalpy(Fluid$,T=T_sat,x=0) "enthalpy of vaporization"
n=1.7 "exponent on Prandtl number"
sigma_1=0.0153 [N/m] "surface tension for R245fa"
DELTAT_e=T_w-T_sat "excess temperature difference"
g=g# "gravitational acceleration"
Nucleate_BoilingR245fa=mu_l*h_fg*((g*(rho_l-&
rho_g))/(sigma_1))^(1/2)*((c_l*DELTAT_e)/(C_s_f*h_fg*Pr_l^n))^3
END

and used to determine the circuit board temperature:

C_s_f=0.012 [-] "nucleate boiling coefficient"


q``=Nucleate_BoilingR245fa(Fluid$, T_sat, T_cb, C_s_f) "circuit board temperature"
T_cb_C=converttemp(K,C,T_cb) "in C"

Figure 2 illustrates the circuit board temperature and saturation pressure as a function of heat
flux.

45 115000
Circuit board surface temperature (°C)

40 110000
Saturation pressure (Pa)

105000
35 Tcb
100000
30
95000
psat
25
90000

20 85000

15 80000
0 2000 4000 6000 8000 10000
2
Heat flux (W/m )
Figure 2: Circuit board surface temperature and saturation pressure as a function of heat flux.
b.) How sensitive are the results to the velocity of the chilled water?

Figure 3 illustrates the circuit board temperature as a function of heat flux for various values of
the chilled water velocity and shows that the results are not very sensitive to this parameter.

45
Circuit board temperature (°C)

40 u = 5 m/s

35
u = 25 m/s

30

25

20

15
0 2000 4000 6000 8000 10000
2
Heat flux (W/m )
Figure 3: Circuit board temperature as a function of heat flux for various values of the water velocity.
Problem 7.4-4 (7-13 in text): Purification of R134a
Recycled refrigerant R134a is purified in a simple distillation process in which a heater,
submerged in the liquid refrigerant heats the liquid, which causes it to vaporize. The distillation
unit is a container with a square base that is 25 cm on a side. Piping and a float valve (not
shown) are provided to maintain a constant liquid level of refrigerant in the container as
condensate is removed. The vapor condenses on the bottom side of a copper plate that is placed
at the top of the device, as shown in Figure P7.4-4. Liquid water at 25°C with free stream
velocity 3 m/s flows over the top of the copper plate. The plate is slightly inclined so that the
condensed refrigerant travels to the left side of the bottom side of the plate and drips into a
collection gutter.

water at
25°C, 3m/s copper plate

25 cm
saturated R134a

heater
Figure P7.4-4: Refrigerant recycling apparatus

a.) Calculate the saturation pressure and temperature of the refrigerant as a function of heater
power for a range of heater powers from 100 W to 1000 W.

Known information is entered into EES. The heater power, W_dot is entered. It will later be
varied in a parametric table.

$UnitSystem SI MASS RAD PA K J


$Tabstops 0.2 0.4 0.6 3.5 in

F$='R134a' "refrigerant"
L=0.25 "length of one side of container"
A=L*L "area of the heat exchange surface"
T_water=convertTemp(C,K,25 [C]) "entering water temperature"
P_atm=101.3 [kPa]*convert(kPa,Pa) "atmopheric pressure"
u_inf=3 [m/s] "water velocity"
$ifnot parametricTable
W_dot=100 [W] "power to the heater"
$endif

The plate temperature, Tplate, is guessed and used with the procedure External_Flow_Plate to
compute the heat transfer coefficient between the water and the copper plate ( h ).

T_plate=300 [K] "plate temperature guess"


Call External_Flow_Plate('water', T_water, T_plate, P_atm, u_inf, L: tau, h, C_f, Nusselt, Re)
"heat transfer coefficient to water"
At steady state, the heater power is transferred as heat to the refrigerant and removed by the
water:

q = w (1)

The guess values are updated and the guess for Tplate is commented out and the plate temperature
is computed according to:

q = h (Tplate − Twater ) (2)

q_dot=w_dot "rate of heat transfer is power to heater"


q_dot=h*A*(T_plate-T_water) "rate of heat transfer to water"

The saturation temperature (Tsat) is guessed and used with the procedure Cond_horizontal_down to
compute the condensate heat transfer coefficient ( hm ).

T_sat=302 [K] "guess for saturation temperature"


Call Cond_horizontal_down(F$, T_plate, T_sat: h_m, Nusselt_m)"condensation coefficient"

The guess values are updated and the guess for Tsat is commented out and the saturation
temperature is computed according to:

q = h (Tsat − Tplate ) (3)

The saturation pressure is computed using the internal property function in EES.

Call Cond_horizontal_down(F$, T_plate, T_sat: h_m, Nusselt_m) "condensation coefficient"


q_dot=A*h_m*(T_sat-T_plate) "rate of heat transfer by condensation"
P_sat=pressure(F$,T=T_sat,x=0) "saturation pressure"

An energy balance determines the rate at which R134a is vaporized and the rate of heat transfer
that must be removed by the cooling water.

( )
m ir ,T =Tsat , x =1 − ir ,T =Tsat , x =0 = w (4)

where ir is the specific enthalpy of the refrigerant.

m_dot*(enthalpy(F$,T=T_sat,x=1)-enthalpy(F$,T=T_sat,x=0))=W_dot "energy balance"

Construct a Parametric table with columns for W_dot, T_sat and m_dot. Solve and plot the
saturation temperature and condensation rate vs the heater power, as shown in Figure 2.
315 0.007

Rate of refrigerant recovery (kg/s)


0.006
Tsat

Saturation temperature (K)


310
m 0.005

0.004
305
0.003

0.002
300

0.001

295 0
100 200 300 400 500 600 700 800 900 1000
Heater power (W)
Figure 2: Saturation temperature and mass flow rate of refrigerant recovered as a function of heater power.
Problem 7.4-5 (7-14 in text): Condensing Steam at 6 kPa.
Calculate the heat flux for a square plate 1 m on each side condensing steam at 6 kPa. Consider
three plate orientations: (1) horizontal facing downward; (2) horizontal facing upward; and (3)
vertical (one-side only is active).
a.) Plot the heat flux for each orientation as a function of plate surface temperature.

The inputs are entered in EES; initially a plate surface temperature is assigned, this value will be
adjusted to generate the plot requested in the problem statement.

$UnitSystem SI MASS RAD PA K J


$Tabstops 0.2 0.4 0.6 3.5 in

Fluid$='steam' "fluid"
P_sat=6 [kPa]*convert(kPa,Pa) "saturation pressure"
T_sat=T_sat(Fluid$,P=P_sat) "saturation temperature"
q``=15000 [W/m^2] "heat flux"
L=1 [m] "
W=1 [m] "width of surface"
A=L*W "area of surface"
T_w=T_sat-10 [K] "plate surface temperature"

This problem is made easy by the availability of the condensation library routines in EES. Call
the Cond_horizontal_down procedure to obtain the heat transfer coefficient on a horizontal plate
facing down ( hhoriz , plate ). The heat transfer rate is given by Newton’s law of cooling.

qhorz = A hhoriz , plate (Tsat − Tw ) (1)

"a) Horizontal downwards"


Call Cond_horizontal_down(Fluid$, T_w, T_sat: h_horiz_plate, Nusselt_horiz_plate)
q_dot_horz=A*h_horiz_plate*(T_sat-T_w)

Call the Cond_horizontal_up procedure to obtain the heat transfer coefficient on a horizontal plate
facing upwards ( hhoriz , plateup ). The heat transfer rate is given by Newton’s law of cooling.

qhorzup = A hhoriz , plateup (Tsat − Tw ) (2)

"b) Horizontal upwards"


Call Cond_horizontal_up(Fluid$, L, T_w, T_sat: h_horiz_plateup, Nusselt_horiz_plateup)
q_dot_horzup=A*h_horiz_plateup*(T_sat-T_w)

Call the Cond_vertical_plate procedure to obtain the heat transfer rate on a vertical plate.

"c) Vertical"
Call Cond_vertical_plate(Fluid$, L, W, T_w, T_sat :h_vert_plate, Re_L, q_dot_vert, m_dot)

Figure 1 illustrates heat transfer rate for these three different cases as a function of the wall
temperature.
70000

60000

Heat transfer rate (W)


50000 horizontal, down

40000

30000
vertical
20000

10000
horizontal, up
0
300 301 302 303 304 305 306 307 308 309
Wall temperature (K)
Figure 1: Rate of heat transfer as a function of wall temperature for the three configurations.

b.) Which geometry provides the highest rate of condensation per unit surface area?

The horizontal, downward facing plate provides the highest rate of condensation per area while
the horizontal, upward facing plate provides the lowest. The performance generally follows
intuition; the best performance occurs in the configuration where gravity most aids the drainage
of the condensate.

c.) How do the answers to (a) and (b) change if the plate dimensions are reduced to 0.5 m per
side?

Change L and W to 0.5 m and replot, as shown in Figure 2.

20000

17500

15000
Heat transfer rate (W)

12500 horizontal, down

10000

7500 vertical

5000

2500
horizontal, up
0
300 301 302 303 304 305 306 307 308 309
Wall temperature (K)
Figure 2: Rate of heat transfer as a function of wall temperature with L = W = 0.5 m.
Problem 7.4-6 (7-15 in text): Heat Pipe
A heat pipe has been instrumented to test its ability to transfer thermal energy. The heat pipe
consists of a sealed vertical thin-walled copper tube that is 1.5 m in length and 2.5 cm in
diameter. The heat pipe contains liquid and vapor toluene. The bottom 5 cm of the tube are
wrapped with heater tape that provides 100 W of heat input to the toluene. The toluene
evaporates at the lower end of the tube and the vapor rises to the top where it is condensed by
contact with the cold top surface of the tube. The top 6 cm of the heat pipe are maintained at
29°C by a flow of liquid water at 25°C and 1 atm. Toluene condensate flows back to the bottom
of the tube; the flow is assisted by surface tension due to the presence of a wicking material on
the inner surface of the copper tube. The heat pipe tube is well-insulated except for the bottom
part that is in contact with the heater and the top part that is in contact with the water.

The known information can be input into EES.

$UnitSystem SI MASS RAD PA K J


$Tabstops 0.2 0.4 0.6 3.5 in

L=1.5 [m] "length of the heat pipe"


D=2.5 [cm]*convert(cm,m) "outside diameter of the heat pipe"
D_c=1.25 [cm]*convert(cm,m) "diameter of center core through which vapor flows up"
F$='Toluene'
L_h=5 [cm]*convert(cm,m) "length of heated section"
A_h=pi*D*L_h "surface area of heated section"
P_atm=101.3 [kPa]*convert(kPa,Pa) "atmospheric pressure"
L_c=6 [cm]*convert(cm,m) "length of cooled section"
A_c=pi*D*L_c "area of cooled section"
T_inf=convertTemp(C,K,25 [C]) "water temperature"
q_dot=100 [W] "rate of heat provided by heater"
T_ts=convertTemp(C,K,29 [C]) "temperature of top surface of tube"

a.) Estimate the saturation temperature and pressure of the toluene in the heat pipe.
b) Estimate the surface temperature of the tube that is in contact with the heater.

Parts (a) and (b) are coupled and best solved together. The Nucleate_Boiling function in EES
implements Eq. (7-3) in the text to characterize the nucleate boiling process. Neither the wall
temperature or saturation temperature are known. Therefore, the wall temperature, Tbs, is
assumed and the function is used to compute the saturation temperature. The saturation pressure
is determined from the temperature.

"hot side of heat pipe"


T_bs=convertTemp(C,K,110 [C]) "guess for the inside surface of the heat pipe - remove later"
C_s_f=0.013 [-] "boiling surface coefficient"
q_dot=Nucleate_Boiling(F$, T_sat, T_bs, C_s_f)*A_h
"heat transfer rate due to nucleate boiling - fixes T_bs"
P_sat=P_sat(F$,T=T_sat) "saturation pressure of heat pipe fluid"

Film condensation occurs at the cold side of the heat pipe. The heat transfer coefficient for this
situation ( hc ) is provided by the Cond_HorizontalTube with the mass flowrate set to 0. Quality is
an input to this procedure, but when there is no flow, its value does not matter.
"cold side of heat pipe"
m_dot=0 "film condensation"
x=0.5 "values is irrelevant for film condensation"
Call Cond_HorizontalTube(F$, m_dot, x, T_sat, T_ts, D : h_c, Regime$)

The guess values are updated and the guess for Tbs is commented out. The heat transfer to the
condensing section is constrained to be the same as the heat input from the heater:

q = Ac hc (Tsat − Tts ) (1)

{T_bs=convertTemp(C,K,110 [C])} "guess for the hot surface of the heat pipe - remove later"
q_dot=A_c*h_c*(T_sat-T_ts) "fixes the surface temperature of the cold side of the heat pipe"

which leads to Tsat = 317.8 K, psat = 9776 Pa, and Tbs = 358.1 K.

c) Estimate the velocity of the cooling water provided at 25°C needed to maintain the top
surface of the heat pipe at 29°C.

The convection coefficient on the outside of the cylinder is computed according to:

q = Ac hconv (Tts − T∞ ) (2)

"calculate heat transfer coefficient outside of cooled surface"


q_dot=h_conv*A_c*(T_ts-T_inf) "definition of convective heat transfer coefficient"

which leads to hconv = 5305 W/m2-K. The required velocity of water (u∞) is obtained from the
External_Flow_Cylinder procedure.

Call External_Flow_Cylinder('water', T_inf, T_ts, P_atm, u_inf, D: F_d\L, h_conv, C_d, Nusselt, Re)
"h_conv is heat transfer coefficient on outside surface of tube to determine u_inf"

which leads to u∞ = 1.01 m/s.

d) Compare the heat transfer rate provided by the heat pipe to the heat transfer rate that would
occur if the tube were replaced with 2.5 cm diameter solid copper rod with the same
temperatures imposed at the hot and cold ends.

Fourier's law is used to obtain the heat transfer through a piece of copper:

D 2 kcu
qcu = π (Tbs − Tts ) (3)
4 L

q_dot_cu=pi*D^2/4*k_cu/L*(T_bs-T_ts) "heat transfer rate of copper rod"


k_cu=k_('Copper',T=(T_bs+T_ts)/2) "thermal conductivity of copper"
which leads to qcu = 7.3 W (much less than the 100 W that can be transferred by the heat pipe
operating under the same conditions).

e) What is the effective thermal conductivity of the heat pipe? What do you see as advantages
of the heat pipe?

The effective conductivity of a material that transfers energy at the same rate as the heat pipe is
defined according to:

D 2 keff
q = π (Tbs − Tts ) (4)
4 L

q_dot=pi*D^2/4*k_eff/L*(T_bs-T_ts) "effective thermal conductivity of the heat pipe"

which leads to keff = 5465 W/m-K. This conductivity is more that an order of magnitude greater
than that for the copper. Heat pipes can be very efficient heat conductors.
Problem 7.4-7 (7-16 in text): Condensation inside a cylinder
A vertical cylindrical container is made of aluminum having a wall thickness of 2.5 mm. The
cylinder is 0.24 m in height and it has an outer diameter of 7.5 cm. Liquid water is placed in the
cylinder and the bottom is heated, evaporating the liquid. The vapor that is produced escapes
through a vent at the top of the cylinder. The flow of vapor drives out air that was originally in
the cylinder. When all of the liquid has been boiled, the heating is stopped and the vent at the
top of the cylinder is closed. The aluminum surfaces are nearly at a uniform temperature of
100°C. The cylinder is allowed to stand in a large room and it transfers energy by free
convection to the 25°C air.
a.) Calculate and plot the pressure inside the cylinder as a function of time for a 5 minute period
after the vent is closed. State and justify any assumptions that you employ. (Note that the
heat transfer coefficient for film condensation on the inside surfaces of the cylinder can be
estimated using the Cond_HorizontalTube procedure with a mass flow rate of zero.)

The inputs are entered in EES:

L=0.24 [m] "length of the cylinder"


D_o=7.5 [cm]*convert(cm,m) "outside diameter of the cylinder"
delta=2.5 [mm]*convert(mm,m) "thickness of the cylinder walls"
D_i=D_o-2*delta "insider diameter of the cylinder"
Fluid$='water'
T_amb=convertTemp(C,K,25 [C]) "temperature of air in the room"
T_boil=convertTemp(C,K,100 [C]) "temperature of the boiling water"
P_atm=101.3 [kPa]*convert(kPa,Pa) "atmospheric air pressure"

The specific heat capacity and density of aluminum are evaluated at the average of the ambient
and initial temperatures (ρal and cal, respectively). The volume of aluminum is:

π D2
Val = π Di L δ + 2 δ (1)
4

and the mass of aluminum is:

Cal = Val ρ al cal (2)

T_avg=(T_amb+T_boil)/2 "average temperature of the aluminum"


rho_al=rho_('Aluminum',T=T_avg) "density of aluminum"
c_al=c_('Aluminum',T=T_avg) "specific heat of aluminum"
V_al=pi*D_i*L*delta+2*pi*D_o^2/4*delta "volume of aluminum"
Cap_al=V_al*rho_al*c_al "specific heat capacity of aluminum"

The mass of water that is contained in the cylinder when it is closed is:

D2 L
Mw = π (3)
4 vw
where vw is the specific volume of the water in the cylinder at the time that the container is
closed (i.e., the specific volume at T = Tboil and x = 1).

v_w=volume(Water,T=T_boil,x=1) "specific volume of water"


m_w=pi*D_i^2/4*L/v_w "mass of water vapor in cylinder when valve is closed"

The state equation for this problem will provide the time rate of change of the temperature of the
water and the cylinder. The time and temperature are initially set and the state equations
developed; these values will be commented out and the state equation integrated using the
Integral command in EES.

"arbitrary value of time and T"


time=1 [s]
T=converttemp(C,K,90 [C])

The pressure in the can follows from the temperature because the water is saturated:

p=pressure(Fluid$,T=T,x=1) "pressure in cylinder"

The surface temperatue is not known so it is assumed in order to evaluate the thermal resistance
between the water and the air.

T_s=T-0.1 [K] "guess for surface temperature"

The thermal resistance is composed of film condensation on the inside surface and natural
convection on the outer surface. The functions FC_vertical_cylinder and FC_plate_horizontal are
used to evaluate the heat transfer coefficient associated with natural convection from the sides
and top, respectively, of the cylinder ( hsides and htop , convection from the bottom is neglected).
The resistance to natural convection is:

−1
⎛ π Do2 htop ⎞
Rconv , nc =⎜ + π Do L hsides ⎟ (4)
⎜ 4 ⎟
⎝ ⎠

Call FC_vertical_cylinder('Air', T_s, T_amb, P_atm, L, D_o : h_sides, Nusselt_sides, Ra_sides)


"estimate convection coefficient on sides of cylinder"
Call FC_plate_horizontal1('Air', T_s, T_amb, P_atm, D_o: h_top, Nusselt_top, Ra_top)
"estimate convection coefficient on top of cylinder"
R_conv_nc=(pi*D_o^2*h_top/4+pi*D_o*L*h_sides)^(-1) "resistance to natural convection"

which leads to Rconv,nc = 2.89 K/W. The film condensation heat transfer coefficient ( h film ) is
obtained from the Cond_HorizontalTube procedure with the mass flow rate set to 0 (note that the
value of quality is arbitrary). The resistance to film condensation is:

1
Rc = (5)
π Di L h film
Call Cond_HorizontalTube(Fluid$, 0 [kg/s], 0.5 [1], T, T_s, D_i : h_film, F$)
"estimate film heat transfer coefficient"
R_c=1/(pi*D_i*L*h_film) "resistance to film condensation"

which leads to Rc = 0.00084 K/W. Because the resistance to condensation is so much less than
the resistance to natural convection, it can be neglected. In this limit, the temperature of the
surface of the cylinder is equal to the temperature of the water:

T_s=T "guess for surface temperature"


{Call Cond_HorizontalTube(Fluid$, 0 [kg/s], 0.5 [1], T, T_s, D_i : h_film, F$)
"estimate film heat transfer coefficient"
R_c=1/(pi*D_i*L*h_film) "resistance to film condensation"}

The rate of heat transfer is given by:

q =
(T − Tamb ) (6)
Rconv , nc

q_dot=(T-T_amb)/R_conv_nc "heat transfer rate"

An energy balance on the water and the cylinder leads to:

⎡ ⎛ ∂u ⎞ ⎤ dT
⎢ mw ⎜ ⎟ + Cal ⎥ + q = 0 (7)
⎣ ⎝ ∂T ⎠v ⎦ dt

⎛ ∂u ⎞
where ⎜ ⎟ is the partial derivative of the water internal energy with respect to temperature at
⎝ ∂T ⎠v
constant specific volume, evaluated numerically according to:

⎛ ∂u ⎞
=
(
uw,T =T +δ T ,v =vw − uw,T =T −δ T ,v =vw )
⎜ ⎟ (8)
⎝ ∂T ⎠v 2δT

where δT is a small change in the temperature.

dT=0.01 [K]
u_p=intenergy(Fluid$,T=T+dT,v=v_w) "internal energy at T+dT"
u_m=intenergy(Fluid$,T=T-dT,v=v_w) "internal energy at T-dT"
dudT=(u_p-u_m)/(2*dT) "partial derivative of internal energy w/respect to T at constant v"
0=(m_w*dudT+Cap_al)*dTdt+q_dot "time rate of temperature change"

The arbitrary values of the state variables are commented out and the integral command is used

{"arbitrary value of time and T"


time=1 [s]
T=converttemp(C,K,100 [C])}
t_sim=5000 [s] "simulation time"
T_ini=T_boil-0.1 [K] "initial temperature"
T=T_ini+Integral(dTdt,time,0,t_sim) "integrate state equations"
T_C=converttemp(K,C,T) "in C"

$integralTable time:1 T_C, p

Figure 1 illustrates the temperature and pressure as a function of time.


100
100000
90

80 80000
Temperature (°C)

Pressure (Pa)
70
60000
60
T
50 40000

40 p
20000
30

20 0
0 1000 2000 3000 4000 5000
Time (s)
Figure 1: Temperature and pressure as a function of time.
Problem 7.4-8: Condenser in an Absorption Cycle
In the condenser in a lithium bromide-water absorption refrigeration system, water vapor at 16
kPa is condensed on the outside surface of horizontal tubes, thereby transferring energy to liquid
water flowing inside of the tubes at atmospheric pressure. The tubes are made of copper with an
inner diameter of 2.5 cm and a wall thickness of 3 mm. The tubes are arranged in banks with N
tubes, stacked vertically, in each bank. The N tubes are plumbed in parallel, with water at 30°C
and a total mass flow rate of 4 kg/s split between the tubes. The cooling water exits at a mean
temperature of 41.4°C.
a) Determine the rate of heat transfer resulting from this condensation process.

Known information is entered into EES. The number of tubes in a vertical bank is set and will
be later investigated with a Parametric table.

$UnitSystem SI MASS RAD PA K J


$Tabstops 0.2 0.4 0.6 3.5 in

D_i=2.5 [cm]*convert(cm,m) "inside diameter of the tubes"


delta=3 [mm]*convert(mm,m) "wall thickness"
D_o=D_i+2*delta "outside diameter of the tubes"
Fluid$='water'
T_w_in=convertTemp(C,K,30 [C]) "temperature of entering cooling water"
T_w_out=convertTemp(C,K,41.4 [C]) "temperature of exiting cooling water"
m_dot_w=4 [kg/s] "flowrate of cooling water"
P_atm=101.3 [kPa]*convert(kPa,Pa) "atmospheric air pressure"
P_sat=16 [kPa]*convert(kPa,Pa) "pressure during condensation"
T_sat=T_sat(Fluid$,P=P_sat) "saturation temperature"
$ifnot ParametricTable
N=4 "number of tubes in vertical bank"
$endif

a) Determine the rate of heat transfer resulting from this condensation process.

The rate of heat transfer is determined by an energy balance on the cooling water.

q = m w cw (Tw,out − Tw,in ) (1)

where cw is the specific heat capacity of the cooling water, evaluated at the average temperature
of the water.

"energy balance on cooling water"


T_avg=(T_w_in+T_w_out)/2 "average cooling water temperature"
c_w=cP(Water,T=T_avg,P=P_atm) "specific heat capacity"
q_dot=m_dot_w*c_w*(T_w_out-T_w_in) "heat transfer rate"

which leads to q = 191 kW.

b) Estimate the flow rate of condensate.

The condensate flow rate is found by an energy balance on the condensing water.
q = m c Δi fg (2)

where Δifg is the latent heat of fusion.

"energy balance on condensing water"


Deltai_fg=enthalpy(Fluid$,P=P_sat,x=1)-enthalpy(Fluid$,P=P_sat,x=0) "latent heat of vaporization"
q_dot=m_dot_c*Deltai_fg "fixes the condensate mass flow rate"

which leads to m c = 0.081 kg/s.

c) Calculate and plot the total length of all tubes required for the condensation process as a
function of the number of vertical tubes plumbed in parallel for N = 2 to 12. Also, calculate
and plot the pressure drop for the cooling water in the tube bank as a function of N.

The length of each tube (L) is assumed in order to determine the heat transfer coefficient and
pressure drop on the cooling water side ( hcw and Δpcw) using the PipeFlow function. The
resistance to convection to the cooling water is:

1
Rconv ,cw = (3)
hcw N π Di L

"consider heat transfer limitations - assuming no temperature difference across the wall"
L=4.5 [m] "guess for length of tubes in each bank"
RelRough=0 [-] "smooth tubes"
call PipeFlow(Fluid$, T_avg, P_atm, m_dot_w/N, D_i, L, RelRough:h_bar_cw, h_H ,DELTAP, &
Nusselt_T, f, Re)
R_conv_cw=1/(h_bar_cw*N*pi*D_i*L) "resistance to convection to the cooling water"

The surface temperature of the tubes (Ts) must be assumed in order to compute the condensation
heat transfer coefficient ( hc ) using the Cond_horizontal_N_Cylinders function. The resistance to
condensation is:

1
Rc = (4)
hc N π Do L

T_s=converttemp(C,K,50 [C]) "guess for surface temperature"


Call Cond_horizontal_N_Cylinders(Fluid$, T_sat, T_s, D_i, N:h_bar_c, Nusselt_m)
R_c=1/(h_bar_c*N*pi*D_o*L) "resistance to condensation"

The problem is solved and the guess values are updated. The guessed value of Ts is commented
out and recalculated according to:

Ts = Tsat − Rc q (5)

{T_s=converttemp(C,K,50 [C])} "guess for surface temperature"


T_s=T_sat-R_c*q_dot "calculate surface temperature"

The conductance of the heat exchanger is computed according to:

UA = ( Rconv ,cw + Rc )
−1
(6)

The problem is solved and the guess values are updated. The guessed value of L is commented
out and the outlet water temperature is computed using the process discussed in Section 5.3.4:

⎛ UA ⎞
Tw,out = Tsat − (Tsat − Tw,in ) exp ⎜ − ⎟ (7)
⎝ m w cw ⎠

The total length of the tubes is computed according to:

Ltotal = N L (8)

{L=4.5 [m]} "guess for length of tubes in each bank"


UA=(R_conv_cw+R_c)^(-1) "conductance"
T_w_out=T_sat-(T_sat-T_w_in)*exp(-UA/(m_dot_w*c_w))
L_total=L*N "total length of tubes required"

Figure 1 illustrates the total tube length and pressure drop as a function of the number of tubes.
50 300000

45
100000
40
Pressure drop (Pa)
Total tube length (m)

35

30
10000
25

20 L Δp

15

10 1000
0 2 4 6 8 10 12
Number of tubes
Figure 1: Total tube length and pressure drop as a function of tubes.
Problem 7.5-1 (7-17 in text): Condenser Tube in a Lake
You have fabricated an inexpensive condenser for your air conditioner by running the refrigerant
through a plastic tube that you have submerged in a lake. The outer diameter of the tube is Do =
7.0 mm and the inner diameter is Di = 5.0 mm. The tube conductivity is ktube = 1.4 W/m-K. The
refrigerant is R134a and enters the tube with quality x = 0.97, temperature Tsat = 35°C and mass
flow rate m r = 0.01 kg/s. The water in the lake has temperature T∞ = 10°C.
a.) Determine the heat transfer rate per unit length from the refrigerant to the lake at the tube
inlet.

The inputs are entered in EES:

$UnitSystem SI MASS RAD PA K J


$Tabstops 0.2 0.4 0.6 3.5 in

"Inputs"
R$='R134a' "refrigerant"
m_dot_r=0.01 [kg/s] "mass flow rate of refrigerant"
x_in=0.97 [-] "inlet quality"
D_i=5.0 [mm]*convert(mm,m) "inner diameter"
D_o=7.0 [mm]*convert(mm,m) "outer diameter"
T_w=converttemp(C,K,10) "temperature of water"
T_sat_C=35 [C] "saturation temperature"
T_sat=converttemp(C,K,T_sat_C) "in K"
p_w=1 [atm]*convert(atm,Pa) "pressure of water"
k_tube=1.4 [W/m-K] "thermal conductivity of tube"

The heat transfer from the R134a to the water in the lake is resisted by convection from the inner
surface of the tube to the refrigerant, conduction through the tube, and convection from the
external surface of the tube to the water. The resistance per unit length associated with
convection to the refrigerant is:

1
Rr′ = (1)
hr π Di

where hr is the heat transfer coefficient associated with the flow condensation of the refrigerant.
The flow condensation heat transfer coefficient can be obtained using the correlation discussed
in Section 7.5.2 and implemented by the Cond_HorizontalTube function. The correlation and
therefore the function requires the surface temperature of the inside of the tube, Ts,r, which is not
given. Therefore, a reasonable value is assumed; this value will be adjusted based on the
solution.

x=x_in "quality"
T_s_r=T_w+(T_sat-T_w)/2 "guess for surface temperature on refrigerant side"

The Cond_HorizontalTube function is accessed by selecting Function Info from the Options menu
and then selecting Boiling and Condensation from the drop down menu. Select Condensation
and then scroll to the appropriate correlation (Figure 1). The function returns the local heat
transfer coefficient, hr , as well as a string that identifies the flow regime. The local heat transfer
coefficient is used to evaluate the thermal resistance per unit length on the refrigerant side
according to Eq. (1).

Call Cond_HorizontalTube(R$, m_dot_r, x, T_sat, T_s_r, D_i : h_r, F$)


"call flow condensation correlation"
R`_r=1/(pi*D_i*h_r) "resistance per unit length on the refrigerant side"

Figure 1: Function Information Window for the Cond_HorizontalTube function

The resistance per unit length associated with conduction through the tube is:

⎛D ⎞
ln ⎜ o ⎟
D

Rtube = ⎝ i⎠ (2)
2 π ktube

R`_tube=ln(D_o/D_i)/(2*pi*k_tube) "resistance per unit length of the tube"

The resistance per unit length associated with convection to the water in the lake is:

1
Rw′ = (3)
hw π Do

where hw is the heat transfer coefficient associated with the natural convection to the water in the
lake. The calculation of the natural convection heat transfer coefficient requires the surface
temperature of the outside of the tube, Ts,w, in order to determine the film temperature and
Raleigh number. The value of Ts,w is not known; therefore, a reasonable value is assumed and
this value will be adjusted based on the solution.

T_s_w=T_w+(T_sat-T_w)/2 "guess for surface temperature on water side"


The film temperature is calculated:

Ts , w + T∞
T film , w = (4)
2

and used to compute the properties of water (ρw, μw, kw, cw, βw, υw, αw, and Prw).

T_film_w=(T_s_w+T_infinity)/2 "film temperature"


mu_w=viscosity(Water,T=T_film_w,p=p_w) "viscosity"
k_w=conductivity(Water,T=T_film_w,p=p_w) "conductivity"
rho_w=density(Water,T=T_film_w,p=p_w) "density"
c_w=cP(Water,T=T_film_w,p=p_w) "specific heat capacity"
beta_w=VolExpCoef(Water,T=T_film_w,p=p_w) "volumetric expansion coefficient"
nu_w=mu_w/rho_w "kinematric viscosity"
alpha_w=k_w/(rho_w*c_w) "thermal diffusivity"
Pr_w=nu_w/alpha_w "Prandtl number"

The Rayleigh number is computed:

g Do3 β w (Ts , w − T∞ )
Ra = (5)
υw α w

Ra=g#*D_o^3*beta_w*(T_s_w-T_infinity)/(nu_w*alpha_w) "Rayleigh number"

and the function FC_horizontal_cylinder_ND is used to access the correlations for natural
convection from a horizontal cylinder in order to obtain the average Nusselt number that
characterizes the heat transfer from the tube surface to the water, Nu w . The heat transfer
coefficient hw is obtained from:

Nu w kw
hw = (6)
Do

The resistance per unit length between the outer surface of the tube and the water is computed
according to Eq. (3).

Call FC_horizontal_cylinder_ND(Ra_w, Pr_w: Nusselt_w) "call natural convection correlation"


h_w=Nusselt_w*k_w/D_o "air side heat transfer coefficient"
R`_w=1/(pi*D_o*h_w) "resistance per unit length on the water side"

The heat transfer per unit length is given by:

q ′ =
(Tsat − T∞ ) (7)
( Rr′ + Rtube
′ + Rw′ )

q`_dot=(T_sat-T_infinity)/(R`_r+R`_tube+R`_w) "heat transfer per unit length"


The surface temperatures that were assumed, Ts,r and Ts,w, should be adjusted based on this
solution. Update Guesses is selected from the Calculate menu and the initial guesses and
commented out. In their place, the surface temperatures are calculated according to:

Ts ,r = Tsat − q ′ Rr′ (8)

Ts , w = T∞ + q ′ Rw′ (9)

{T_s_r=T_w+(T_sat-T_w)/2} "guess for surface temperature on refrigerant side"


{T_s_w=T_w+(T_sat-T_w)/2} "guess for surface temperature on water side"
T_s_r=T_sat-q`_dot*R`_r "surface temperature on refrigerant side"
T_s_w=T_infinity+q`_dot*R`_w "surface temperature on water side"

which leads to q ′ = 224.3 W/m.

b.) If the length of the tube is L = 6 m, then what is the quality of the refrigerant leaving the
tube? Plot the quality and refrigerant heat transfer coefficient as a function of position s in
the tube.

An energy balance on a differential length of the tube (ds) is shown in Figure 2.

Figure 2: Differential energy balance on the tube

The differential energy balance leads to:

⎡ m r ( il , sat + x Δivap ) ⎤ = q ′ ds + ⎡ m r ( il , sat + x Δivap ) ⎤ (3)


⎣ ⎦s ⎣ ⎦ s + ds

where il,sat is the enthalpy of the saturated liquid and Δivap is the latent heat of vaporization,
which is computed using EES' internal property routines:

DELTAi_vap=enthalpy(R$,x=1,T=T_sat)-enthalpy(R$,x=0,T=T_sat) "enthalpy of vaporization"

The s + ds term in Eq. (3) is expanded:

dx
⎣ mr ( il , sat + x Δivap ) ⎦ s = q ds + ⎣ mr ( il , sat + x Δivap ) ⎦ s + mr Δivap ds ds
⎡ ⎤ ′ ⎡ ⎤ 
which can be simplified to provide the rate of change of quality with position:

dx q ′
=−
ds m r Δivap

dxds=-q`_dot/(m_dot_r*DELTAi_vap) "rate of change of quality with position"

The rate of change of quality with respect to position is integrated numerically from the inlet of
the tube (s = 0 where xin = 0.97) to the exit of the tube (s = L) using EES’ INTEGRAL command.
The results, including quality and heat transfer coefficient, are contained in an integral table.

{x=x_in} "quality"
L=6 [m] "length of tube"
x=x_in+INTEGRAL(dxds,s,0,L) "numerical integration"
$IntegralTable s:0.2, x, h_r

The quality and refrigerant side heat transfer coefficient are illustrated as a function of position
in the tube in Figure 3.

Refrigerant heat transfer coefficient (W/m -K)


1 10000

2
0.9 9000

0.8 8000

0.7 7000

0.6 6000
Quality (-)

x hr
0.5 5000

0.4 4000

0.3 3000

0.2 2000

0.1 1000

0 0
0 0.5 1 1.5 2 2.5 3 3.5 4 4.5 5 5.5 6
Position (m)
Figure 3: Quality and refrigerant heat transfer coefficient as a function of position in the tube.
Problem 7.5-2 (7-18 in text): Analogy between condensation and boiling
Condensation and boiling are analogous processes in that both a involve phase change. Heat
exchangers that provide condensation and boiling are often designed in a similar manner. In a
particular case, a phase change of R134a takes place within horizontal tubes having an inner
diameter of 1 cm. The mass velocity is 300 kg/s-m2.
a.) Prepare a plot of the heat transfer coefficient for condensation and boiling as a function of
quality at a saturation temperature of 10°C for heat fluxes of 5,000 and 10,000 W/m2.

The input information is entered:

$UnitSystem SI MASS RAD PA K J


$Tabstops 0.2 0.4 0.6 3.5 in

Fluid$='R134a'
D=1 [cm]*convert(cm,m) "tube inner diameter"
T_sat=convertTemp(C,K,10 [C]) "saturation temperature"
G=300 [kg/m^2-s] "mass velocity"
m_dot=G*A "mass flow rate"
A=pi*D^2/4 "cross-sectional area of the tube"
$ifnot ParametricTable
x=0.5 [-] "quality"
q``=5000 [W/m^2] "heat flux"
$endif

The Flow_Boiling and Cond_HorizontalTube procedures return the boiling and condensation heat
transfer coefficients, respectively. A Parametric table is constructed with columns for quality
(x), the heat flux ( q ′′ ), the heat transfer coefficients and the excess temperatures.

"boiling"
call Flow_Boiling(Fluid$, T_sat, G, D, x, q``, 'Horizontal': h_b, T_w_b)
DELTAT_b=T_w_b-T_sat "excess temperature for boiling"

"condensation"
Call Cond_HorizontalTube(Fluid$, m_dot, x, T_sat, T_w, D : h_c, F$)
DELTAT_c=T_sat-T_w "excess temperature for condensation"
q``=h_c*(T_sat-T_w) "relation between heat flux and wall temperature"

The heat transfer coefficient for boiling and condensation are shown in Figures 1 and 2 as a
function of x for q ′′ = 5,000 W/m2 and q ′′ = 10,000 W/m2.
Figure 1: Heat transfer coefficient for condensation and boiling as a function of quality with 5,000 W/m2 heat
flux.

Figure 2: Heat transfer coefficient for condensation and boiling as a function of quality with 10,000 W/m2
heat flux.

b.) Plot the excess temperature for condensation and boiling as a function of quality.

The excess temperature for condensation and boiling are shown in Figure 3 as a function of
quality for a heat flux of 5,000 W/m2.
Figure 3: Excess temperature for condensation and boiling as a function of quality with 5,000 W/m2 heat flux.

c.) What conclusion can you draw from the results?

Boiling and condensation both exhibit high heat transfer coefficients, but at high quality, the
boiling experiences ‘dry-out’ and the heat transfer coefficient reduces to the value for saturated
vapor. Condensation does not experience this limitation.
Problem 7.5-3 (7-19 in text): Absorption Refrigeration Cycle using Ammonia-Water
Absorption refrigeration cycles are often used to operate small refrigerators in hotel rooms
because they do not require compressors or fans and thus operate quietly. In a particular case, an
absorption refrigeration system uses an ammonia-water mixture. The ammonia is separated from
the water and passes through a condenser where it is isobarically changed from saturated vapor
at 76°C to subcooled liquid at 40°C. The condenser consists of a single unfinned thin-walled
copper tube with a 1 cm inner diameter. The thermal energy released from the ammonia in this
process is transferred to the 25°C room air by free convection. The subcooled ammonia is
throttled to a saturation temperature of -5° and evaporated to saturated vapor to produce a
refrigeration capacity of 110 W.
a) Determine the mass flow rate of ammonia through the evaporator and condenser.

The small amount of water remaining in the ammonia is neglected. The inputs information is:

$UnitSystem SI MASS RAD PA K J


$Tabstops 0.2 0.4 0.6 3.5 in

"known"
T_sat=converttemp(C,K,76[C]) "saturation temperature of the ammonia"
D=1 [cm]*convert(cm,m) "diameter of the condenser pipe"
q_dot_evap=110 [W] "refrigeration effect"
T_air=converttemp(C,K,25 [C]) "air temperature"
T_evap_in=converttemp(C,K,40 [C]) "temperature of refrigerant before throttling"
T_sat_evap=converttemp(C,K,-5 [C]) "evaporation temperature"
P_air=101.3 [kPa]*convert(kPa,Pa) "atmospheric pressure"

The pressure in the condenser (pcond) is obtained from the inlet temperature. An energy balance
on the evaporator leads to:

qevap = m ( ievap ,out − ievap ,in ) (1)

where ievap,out is the specific enthalpy of the ammonia leaving the evaporator at the evaporation
saturation temperature and a quality of one and ievap,in is the enthalpy of the ammonia entering the
evaporator, which is the same as the enthalpy of the subcooled ammonia entering the throttling
valve.

"condensation section"
P_cond=Pressure(ammonia,T=T_sat,x=1 [-]) "pressure of the ammonia in the condenser"
i_evap_in=enthalpy(Ammonia,T=T_evap_in,P=P_cond)
"specific enthalpy of ammonia entering the evaporator"
i_evap_out=enthalpy(Ammonia,T=T_sat_evap,x=1) "specific enthalpy of ammonia exiting the evaporator"
q_dot_evap=m_dot*(i_evap_out-i_evap_in) "energy balance determines ammonia flow rate"

which leads to m = 0.00010 kg/s.

b) Estimate the length of piping required to condense the ammonia from saturated vapor to
saturated liquid at 76°C.
The Cond_HorizontalTube_avg procedure is used to evaluate the average heat transfer coefficient
due to convection within the tube ( hcond , sat ). The surface temperature of the tube in the
condensing region (Tw,sat) is assumed for this purpose.

T_w_sat=converttemp(C,K,30[C]) "guess for the wall temperature in the condensing section"


Call Cond_HorizontalTube_avg('ammonia', m_dot, T_sat, T_w_sat, D, 1 [-], 0 [-] : h_bar_cond_sat)
"determines condensing heat transfer coefficient"

The FC_horizontal_cylinder procedure is used to evaluate the average heat transfer coefficient due
to natural convection from the outer surface of the tube ( hnc , sat ).

Call FC_horizontal_cylinder('air', T_w_sat, T_air, P_air , D : h_bar_nc_sat, Nusselt_cond, Ra_cond)


"determines h_bar_nc"

The rate of heat transfer in the condensing section of the condenser is calculated from an energy
balance on the ammonia:

(
qcond , sat = m iNH 3, x =1,T =Tsat − iNH 3, x =0,T =Tsat ) (2)

q_dot_cond_sat=m_dot*(enthalpy(ammonia,T=T_sat,x=1)-enthalpy(ammonia,T=T_sat,x=0))
"rate of energy transfer during condensation"

The air and ammonia temperatures in the condensing region are both constant and therefore the
heat transfer rate can be written according to:

qcond , sat =
(Tsat − Tair ) (3)
⎛ 1 1 ⎞
⎜⎜ + ⎟⎟
⎝ hcond , sat π Lsat D hnc , sat π Lsat D ⎠

q_dot_cond_sat=(T_sat-T_air)/(1/(h_bar_cond_sat*pi*D*L_sat)+1/(h_bar_nc_sat*pi*D*L_sat))
"length required in saturated section"

which leads to the predicted value of the length of tube required to condense the ammonia. The
program is solved and the guess values are updated. The assumed value of Tw,sat is commented
out and recalculated according to:

qcond , sat
Tw, sat = Tsat − (4)
hcond , sat π D Lsat

{T_w_sat=converttemp(C,K,30[C])} "guess for the wall temperature in the condensing section"


T_w_sat=T_sat-q_dot_cond_sat/(h_bar_cond_sat*pi*D*L_sat) "recalculate wall temperature"

which leads to Lsat = 5.59 m.

c) Estimate the additional length of piping required to subcool the ammonia to 40°C.
The heat transfer rate required in the sub-cooling section is:

(
qcond , sc = m iNH 3, x =0,T =Tsat − ievap ,in ) (5)

"subcooling section"
q_dot_cond_sc=m_dot*(enthalpy(ammonia,T=T_sat,x=0)-i_evap_in)
"rate of energy transfer required to accomplish subcooling"

The wall temperature (Tw,sc) and length of the subcooling section (Lsc) must be assumed in order
to evaluate the heat transfer coefficients.

T_w_sc=T_w_sat "guess for wall temperature in subcooling section"


L_sc=1 [m] "guess for length of tube needed for condensation"

The procedure FC_horizontal_cylinder is used to evaluate the natural convection heat transfer
coefficient ( hnc , sc ).

Call FC_horizontal_cylinder('air', T_w_sc, T_air, P_air , D : h_bar_nc_sc, Nusselt_sc, Ra_sc)


"determines h_bar_nc_sc - for subcooling section"

The procedure PipeFlow is used to evaluate the single-phase forced convection heat transfer
coefficient ( hi , sc ) using the average temperature of the ammonia in the subcooled section.

RelRough=1e-6 [-] "assume smooth tube"


T_avg=(T_sat+T_evap_in)/2 "average temperature during subcooling"
call PipeFlow('Ammonia',T_avg,P_cond,m_dot,D,L_sc,RelRough:h_bar_i_sc, h_H ,DELTAP, &
Nusselt_T, f, Re)

The total conductance of the subcooled section is calculated:


−1
⎛ 1 1 ⎞
UAsc = ⎜ + ⎟⎟ (6)
⎜π DL h π D Lsc hi , sc
⎝ sc nc , sc ⎠

UA_sc=(1/(pi*D*L_sc*h_bar_nc_sc)+1/(pi*D*L_sc*h_bar_i_sc))^(-1) "conductance"

The problem is solved and the guess values updated. The assumed value of Lsc is commented out
and the solution for internal flow exposed to a constant external temperature from Section 5.3.4
is used:

⎛ UA ⎞
Tevap ,in = Tair − (Tair − Tsat ) exp ⎜ − sc ⎟ (7)
⎝ m csc ⎠

where csc is the specific heat capacity of the ammonia in the subcooled section, evaluated at the
average temperature.
{L_sc=1 [m]} "guess for length of tube needed for condensation"
c_sc=cP(ammonia,T=(T_sat+T_evap_in)/2,p=p_cond) "specific heat capacity of subcooled liquid"
T_evap_in=T_air-(T_air-T_sat)*exp(-UA_sc/(m_dot*c_sc)) "recalculate evaporator inlet temperature"

The problem is solved and the guess values updated. The assumed value of Tw,sc is commented
out and recalculated according to:

qcond , sc
Tw, sc = Tair − (8)
hnc , sc π D Lsc

{T_w_sc=T_w_sat} "guess for wall temperature in subcooling section"


T_w_sc=T_air-q_dot_cond_sc/(h_bar_nc_sc*pi*D*L_sc) "recalculate wall temperature"

which leads to Lsc = 2.37 m.


Problem 7.2-1 (7-1 in text): Contaminant Removal Method
One method of removing water and other contamination from a gas is to pass it through a cooled
tube so that contaminants with high freezing and liquefaction points (e.g., water) tend to be
collected at the wall. A quick and easy liquid nitrogen trap for methane is constructed by placing
a tube in a Styrofoam cooler that is filled with liquid nitrogen, as shown in Figure P7.2-1.
liquid nitrogen at 1 atm
L=1m

m = 0.01 kg/s
x
T f ,in = 20°C
p f ,in = 400kPa

thins = 0.375 inch Dout = 0.5 inch thtube = 0.065 inch

Figure P7.2-1: Liquid nitrogen trap.

The length of the tube is L = 1 m. The outer diameter of the tube is Dout = 0.5 inch and the tube
thickness is thtube = 0.065 inch. The tube conductivity is ktube = 150 W/m-K. The tube is
wrapped in insulation (to avoid liquefying the methane). The thickness of the insulation is thins =
0.375 inch and the insulation conductivity is kins = 1.5 W/m-K. Methane enters the tube at m =
0.01 kg/s with temperature Tf,in = 20°C and pressure pf,in = 400 kPa. The liquid nitrogen that fills
the container is at 1 atm and is undergoing nucleate boiling on the external surface of the
insulation. You may neglect axial conduction through the tube.
a.) Set up an EES program that can evaluate the state equations for this problem. That is, given
a value of position, x, methane temperature, Tf, and methane pressure, pf, your program
dT f dp f
should be able to compute and .
dx dx

Enter the information in the problem statement into EES.

$UnitSystem SI MASS RAD PA K J


$TABSTOPS 0.2 0.4 0.6 0.8 3.5 in

"Inputs"
L=1 [m] "length of tube"
p_atm=1 [atm]*convert(atm,Pa) "atmospheric pressure"
m_dot=0.01 [kg/s] "mass flow rate of air"
F$='Methane' "fluid"
D_out=0.5 [inch]*convert(inch,m) "outer diameter"
th_tube=0.065 [inch]*convert(inch,m) "tube thickness"
k_tube=150 [W/m-K] "tube conductivity"
h_bar_out=350 [W/m^2-K] "heat transfer coefficient on outer surface"
k_ins=1.5 [W/m-K] "insulation conductivity"
th_ins_inch=0.375 [inch] "insulation thickness, in inch"
th_ins=th_ins_inch*convert(inch,m) "insulation thickness"
T_LN2=temperature('Nitrogen',x=0,p=p_atm) "temperature of liquid nitrogen"
T_f_in=converttemp(C,K,20[C]) "inlet temperature of gas"
p_f_in=400 [kPa]*convert(kPa,Pa) "inlet pressure of gas"
The inner diameter of the tube is:

Din = Dout − 2 thtube (1)

and the outer diameter of the insulation is:

Dout ,ins = Dout + 2 thins (2)

D_in=D_out-2*th_tube "inner diameter of tube"


D_out_ins=D_out+2*th_ins "outer diameter of insulation"

An arbitrary values of position (x), fluid temperature (Tf) and pressure (pf) are specified. These
values will later be commented out when the state equation is integrated.

x=0 [m] "arbitrary value of the position to check state equation"


T_f=T_f_in "arbitrary value of the fluid temperature to check state equation"
p_f=p_f_in "arbitrary value of the fluid pressure to check the state equation"

A guess is made for the temperature of the outside surface of the insulation (Tw,out). This is the
surface in contact with the liquid nitrogen and is necessary to calculate the boiling heat transfer.
The guess will be commented out later.

T_w_out=(T_f+T_LN2)/2 "guess outer temperature of the insulation surface"

The Nucleate_Boiling function in EES returns the heat flux on the outside surface of the insulated
tube ( qs′′,out ).

q``_s_out= Nucleate_Boiling('Nitrogen', T_LN2, T_w_out, 0.0127 [-])"get nucleate boiling heat flux"

The fluid properties of the methane (ρf, μf, kf, cf, and Prf) are obtained at Tf:

rho_f=density(F$,T=T_f,p=p_f) "density of fluid in tube"


mu_f=viscosity(F$,T=T_f,p=p_f) "viscosity"
k_f=conductivity(F$,T=T_f,p=p_f) "conductivity"
c_f=cP(F$,T=T_f,p=p_f) "specific heat capacity"
Pr_f=mu_f*c_f/k_f "Prandtl number"

The mean velocity of the flow is:

4 m
um = (3)
ρ f π Din2

The Reynolds number is computed:

ρ f Din um
Re = (4)
μf
The PipeFlow_N_local procedure is used to obtain the local Nusselt number (Nu) and friction
factor (f). The heat transfer coefficient is computed according to:

Nu k f
hin = (5)
D

u_f=m_dot/(rho_f*pi*D_in^2/4) "bulk velocity"


Re=rho_f*u_f*D_in/mu_f "Reynolds number"
call PipeFlow_N_local(Re,Pr_f,x/D_in,0 [-]: Nusselt_T_x,Nusselt_H_x,f_x) "call correlations"
h_in=Nusselt_T_x*k_f/D_in "local heat transfer coefficient"

The overall resistance per unit length between the outer surface of the tube and the fluid flowing
in the pipe can be found by adding the resistances for convection on the inside surface and
conduction through the wall and insulation:

R′ = Rconv
′ ,in + Rcond
′ ,tube + Rcond
′ ,ins (6)

where

1
′ ,in =
Rconv (7)
hin π D

⎛D ⎞
ln ⎜ out ⎟
= ⎝ in ⎠
D
′ ,tube
Rcond (8)
2 π ktube

⎛D ⎞
ln ⎜ out ,ins ⎟
= ⎝ out ⎠
D
′ ,ins
Rcond (9)
2 π kins

R`=R`_conv_in+R`_cond_tube+R`_cond_ins "resistance per unit length"


R`_conv_in=1/(h_in*pi*D_in) "convection resistance per unit length"
R`_cond_tube=ln(D_out/D_in)/(2*pi*k_tube) "conduction through tube, per unit length"
R`_cond_ins=ln(D_out_ins/D_out)/(2*pi*k_ins) "conduction through insulation, per unit length"

The equations are solved and the guess values updated. The assumed value of Tw,out is
commented out and the heat flux at the outer surface of the insulation is recalculated:

qs′′,out π Dout ,ins =


(T f − Tw,out )
(10)
R′

{T_w_out=(T_f+T_LN2)/2} "guess outer temperature of the insulation surface"


q``_s_out*pi*D_out_ins=(T_f-T_w_out)/R` "recalculate heat flux"
The rate of change of the fluid temperature is obtained using an energy balance:

dT f qs′′,out π Dout ,ins


= (11)
dx m c f

and the rate of change of the fluid pressure is obtained using the friction factor:

dp f ρ f um2
=−f (12)
dx 2 Din

dTfdx=-q``_s_out*pi*D_out_ins/(m_dot*c_f) "obtain temperature derivative"


dpfdx=-f_x*rho_f*u_f^2/(2*D_in) "obtain pressure gradient"

b.) Use the Integral command in EES to integrate the state equations from x = 0 to x = L. Plot the
fluid temperature and pressure as a function of position.

Update the guess values. Then comment out the arbitrary specifications for x, Tf and pf. Use the
Integral command to integrate the two state equations derived in part (a).

{x=0 [m] "arbitrary value of the position to check state equation"


T_f=T_f_in "arbitrary value of the fluid temperature to check state equation"
p_f=p_f_in "arbitrary value of the fluid pressure to check the state equation"}
T_f=T_f_in+Integral(dTfdx,x,0,L,0.01 [m])
p_f=p_f_in+Integral(dpfdx,x,0,L, 0.01 [m])

$IntegralTable x,T_f,p_f

Figure 2 illustrates the fluid temperature and pressure as a function of position.


300 400000

290
398000
Temperature (K)

280
pf
Pressure (Pa)

396000
270
Tf
260
394000

250
392000
240

230 390000
0 0.1 0.2 0.3 0.4 0.5 0.6 0.7 0.8 0.9 1
Position (m)
Figure 2: Fluid temperature and pressure as a function of position.
c.) Plot the heat flux at the insulation surface and the critical heat flux as a function of position.

The properties of the saturated liquid nitrogen (Δivap, ρv,sat, ρl,sat, and σ) are computed using the
internal functions in EES.

"Determine critical heat flux"


Di_vap=enthalpy('Nitrogen',x=1,p=p_atm)-enthalpy('Nitrogen',x=0,p=p_atm) "latent heat of fusion"
rho_v_sat=density('Nitrogen',x=1,p=p_atm) "density of saturated nitrogen vapor"
rho_l_sat=density('Nitrogen',x=0,p=p_atm) "density of saturated nitrogen vapor"
sigma=SurfaceTension('Nitrogen',T=T_LN2) "surface tension"

As discussed in Section 7.2.3, the characteristic length for nucleate boiling is computed
according to:

σ
Lchar , nb = (7-13)
g ( ρl , sat − ρ v , sat )

and the dimensionless length that characterizes the tube is:

D
L = out ,ins (7-14)
2 Lchar ,nb

L_char_nb=sqrt(sigma/(g#*(rho_l_sat-rho_v_sat)))"characteristic length associated with nucleate boiling"


L_hat=(D_out_ins/2)/L_char_nb "dimensionless length"

which leads to L = 15. According to Table 7-2, a cylinder with L > 1.2 is a large cylinder and
should use Ccrit = 0.12 in the correlation:

⎡ σ g ( ρl , sat − ρ v , sat ) ⎤
1
4

qs′′,crit = Ccrit Δivap ρ v , sat ⎢ ⎥ (7-15)


⎢⎣ ρv2, sat ⎥⎦

C_crit=0.12 "critical heat flux constant"


q``_s_crit=C_crit*Di_vap*rho_v_sat*(sigma*g#*(rho_l_sat-rho_v_sat)/rho_v_sat^2)^(1/4)
"critical heat flux"

The Critical_Heat_Flux library function in EES can also be used:

q``_max=Critical_Heat_Flux('Nitrogen','Cylinder', L_char_nb, T_LN2)


"critical heat flux from library routine"

Both provide the same result, qs′′,crit = 148.1 kW/m2. The heat flux calculations are added to the
Integral Table:

$IntegralTable x,T_f,p_f,q``_s_out, ,q``_s_crit, q``_max


Figure 3 illustrates the surface heat flux and the critical heat flux as a function of position and
shows that the heat flux is safely below the critical heat flux.

1.6x105
critical heat flux
1.4x105

1.2x105
Heat flux (W/m )
2

1.0x105

8.0x104

6.0x104

4.0x104

2.0x104
heat flux
0.0x100
0 0.1 0.2 0.3 0.4 0.5 0.6 0.7 0.8 0.9 1
Axial position (m)
Figure 3: Heat flux and critical heat flux as a function of position.

d.) Plot the temperature of the methane at the surface of the tube as a function of position.

The temperature of the inside tube wall surface is calculated according to:

Tw,in = T f − qs′′ Dout ,ins π Rconv


′ ,in (7-16)

T_w_in=T_f-q``_s_out*pi*D_out_ins*R`_conv_in "inner temperature of wall"

The inner wall temperature is added to the $IntegralTable directive.

$IntegralTable x,T_f,p_f,q``_s_out, ,q``_s_crit, q``_max, T_w_out,T_w_in

The inner wall temperature as a function of position is shown in Figure 4.


270

260

Inner wall temperature (K)


250

240

230

220

210

200

190

180
0 0.1 0.2 0.3 0.4 0.5 0.6 0.7 0.8 0.9 1
Position (m)
Figure 4: Inner wall temperature as a function of position.

e.) Plot the lowest temperature experienced by the methane in the trap as a function of the
insulation thickness. If the methane temperature must be maintained at above its liquefaction
point (131.4 K at 400 kPa) then what should the insulation thickness be?

The lowest temperature experienced by the methane is at the wall at the exit (Tw,in,x=L). Figure 5
illustrates the lowest methane temperature as a function of insulation thickness.
200
Lowest methane temperature (K)

180

160

140

120

100

80
0 0.05 0.1 0.15 0.2 0.25 0.3 0.35 0.4 0.45 0.5
Insulation thickness (inch)
Figure 5: Lowest methane temperature as a function of insulation thickness.

Figure 5 suggests that the insulation thickness should be at least 0.1 inch to keep the methane
from liquefying.
Problem 7.2-2 (7-2 in text): Steam Boiler
An industrial boiler generates steam by heat exchanging combustion gases with saturated water
at 125 kPa through mechanically polished AISI 302 stainless steel tubing having an inside
diameter of 5.48 cm with a wall thickness of 2.7 mm and a total submerged length of 10 m. The
combustion gases enter the tubing at 750°C with a mass flow rate of 0.0115 kg/s. The gases
exhaust at ambient pressure. Assume that the combustion gases have the same thermodynamic
properties as air.
a) Identify the state equation for this problem; the differential equation that can be used to
determine the rate of change of the temperature of the combustion gas with respect to
position.

Enter known information into EES.

$UnitSystem SI MASS RAD PA K J


$TABSTOPS 0.2 0.4 0.6 0.8 3.5 in

"known information"
D_i=5.48 [cm]*convert(cm,m) "inside tube diameter"
D_o=D_i+2.7 [mm]*convert(mm,m) "outside tube diameter"
L=10 [m] "tube length"
P_water=125 [kPa]*convert(kPa,Pa) "pressure on water"
T_water=T_sat(Water,P=P_water) "saturation temperature of water"
m_dot_gas=0.0115 [kg/s] "combustion gas flow rate"
T_gas_in=convertTemp(C,K,750 [C]) "combustion gas inlet temperature"
k_steel=k_('Stainless_AISI302', (T_gas_in+T_water)/2) "thermal conductivity of steel"
F$='air' "assume combustion gases behave as air"
P_gas=101.3 [kPa]*convert(kPa,Pa) "air pressure"
RelRough=0 [-] "smooth tube"

Arbitrary values of x and Tgas are set in order to compute the state equations:

x=1 [m] "position"


T_gas=800 [K] "gas temperature"

An energy balance on the combustion gases provides the state equation:

dTgas
m gas c p + qo′′ π Do = 0 (1)
dx

where qo′′ is the heat flux leaving the outer surface of the tube. The PipeFlow procedure
determines the local heat transfer coefficient for the gas flowing in the pipe (hin).

call PipeFlow_local(F$,T_gas,P_gas,m_dot_gas,D_i,x,RelRough:h_in, h_H_x, dPdx)

The heat transfer on the outside surface of the tube is found using the Nucleate_Boiling function in
EES. An assumed value of the wall temperature (Twall) is required and will be commented out:

C_s_f=0.0132 [-] "coefficient for mechanically polished stainless steel"


T_wall=700 [K] "guess for wall temperature"
q``_o= Nucleate_Boiling('Water', T_water, T_wall, C_s_f)
"obtain heat flux as a function of the wall temperature"

The guess values are updated and the value of Twall commented out. The wall temperature is
computed according to:

qo′′ π Do =
(T
gas − Twall )
(2)
R′

where

⎛D ⎞
ln ⎜ o ⎟
+ ⎝ i⎠
1 D
R′ = (3)
hin π Di 2 π ksteel

{T_wall=700 [K]} "guess for wall temperature"


R`=1/(h_in*pi*D_i)+ln(D_o/D_i)/(2*pi*k_steel)
"resistance per unit length between gas and outside surface of tube"
q``_o*pi*D_o=(T_gas-T_wall)/R` "equate steady-state heat flows"

Equation (1) is used to calculate the state equation:

cp=cp(F$,T=T_gas) "specific heat capacity"


m_dot_gas*cp*dT\dx+q``_o*pi*D_o=0 "energy balance"

b) Integrate the state equations developed in part (a) in order to determine the outlet temperature
of the combustion gases.

Solve and update the guess values. Then comment out the values of x and Tgas and use the
Integral command to integrate the state equation. An $IntegralTable directive to include x and Tgas
is used.

T_gas=T_gas_in+integral(dT\dx,x,0,L)
$integralTable, x:0.2 T_gas

Figure 1 illustrates the temperature as a function of position.


1100

1000

Gas temperature (K)


900

800

700

600

500

400

300
0 1 2 3 4 5 6 7 8 9 10
Position (m)
Figure 1: Gas temperature as a function of position.

c) Calculate the rate at which steam is generated in this boiler.

An energy balance on the steam determines the rate at which steam is generated.

(
m steam ( isteam , x =1 − isteam, x =0 ) = m gas igas ,T =Tgas ,in − igas ,T =Tgas ,out ) (4)

"Energy balance to determine steam generation rate"


m_dot_steam*(enthalpy(Water,P=P_water,x=1)-enthalpy(Water,P=P_water,x=0))=&
m_dot_gas*(enthalpy(F$,T=T_gas_in)-enthalpy(F$,T=T_gas))

which leads to m steam = 0.00341 kg/s.


Problem 7.2-3 (7-3 in text): Spaghetti
You are preparing a spaghetti dinner for guests when you realize that your heat transfer training
can be used to answer some fundamental questions about the process. The pot you are using
holds four liters of water. The atmospheric pressure is 101 kPa. When on its high setting, the
electric stove heating unit consumes 1.8 kW of electrical power of which 20% is transferred to
the surroundings, rather than to the water. The pot is made of 4 mm thick polished AISI 304
stainless steel and it has a diameter of 0.25 m. The burner diameter is also 0.25 m.
a) How much time is required to heat the water from 15°C to its boiling temperature?

Enter the known specifications into EES.

$UnitSystem SI MASS RAD PA K J


$TABSTOPS 0.2 0.4 0.6 0.8 3.5 in

"known information"
V_w=4 [liter]*convert(liter,m^3) "volume of water"
P=101.3 [kPa]*convert(kPa,Pa) "atmospheric pressure"
D=0.25 [m] "diameter of the burner and pot"
T_w_i=convertTemp(C,K,15 [C]) "initial water temperature"
T_sat=T_sat(Water,P=P) "boiling temperature"
delta=4 [mm]*convert(mm,m) "thickness of stainless steel"
fraction=0.8 "fraction of the heat flux that enters the water"
q_burner=1.8[kW]*convert(kW,W) "heat flux into water"

The initial mass of water in the pot is calculated:

M w,i = Vw ρ w (1)

where ρw is the density of liquid water computed at the initial condition.

m_w_i=V_w*density(Water,T=T_w_i,P=P) "initial mass of water"

The surface area of the bottom of the pan is computed:

D2
A=π (2)
4

and the initial height of the water in the pan is computed:

V
H= (3)
A

A=pi*D^2/4 "area of the bottom of the pot"


H=V_w/A "height of the water in the pot"

The mass of the pot is computed:


⎛ D2 ⎞
M steel = ⎜ π + π D H ⎟ δ ρ steel (4)
⎝ 4 ⎠

The conductivity and specific heat capacity of steel are determined (ksteel and csteel, respectively).
The total heat capacity of the stell is calculated:

Csteel = M steel csteel (5)

and compared to the sensible heat capacity of the water:

Cwater = M w,i cw (6)

where cw is the specific heat capacity of liquid water:

m_steel=(pi*D^2/4+pi*D*H)*delta*rho_('Stainless_AISI304', T_w_i) "mass of pot in contact with the water"


c_steel=c_('Stainless_AISI304', T_w_i) "specific heat of steel"
k_steel=k_('Stainless_AISI304', T_w_i) "thermal conductivity of steel"
mc_steel=m_steel*c_steel "capacitance of the pot - note that it is negligible relative to the water"
c_water=cp(water,T=(T_w_i+T_sat)/2,P=P) "specific heat of water"
mc_water=m_w_i*c_water "capacitance of water"

Note that Csteel = 1.67 kJ/K whereas Cwater = 16.7 kW/K and therefore it is possible to neglect the
heat capacity of the pan. An energy balance on the pot and its contents determines the time
required for it to start to boil.

qburner Δt fraction = Cwater (Tsat − Tw,i ) (7)

where fraction = 0.8 is the fraction of the burner power that enters the water.

"a)"
q_burner*DELTAt*fraction=mc_water*(T_sat-T_w_i) "energy balance to determine time required to boil"
DELTAt_min=DELTAt*convert(s,min) "in min"

which leads to Δt = 986.2 s (16.4 min).

b) What are the temperatures of the outside and inside surfaces of the bottom of pot while the
water is boiling?

The burner heat flux is:

fraction qburner
′′
qburner = (8)
A

The procedure Nucleate_Boiling in EES is used to solve for the inner temperature of the pan (Ti).
The outer temperature is calculated according to:
k steel
′′
qburner = (To − Ti ) (9)
δ

"b)"
q``_burner=q_burner*fraction/A "heat flux from burner"
C_s_f=0.0132 "coefficient for polished stainless steel"
q``_burner= Nucleate_Boiling('Water', T_sat, T_i, C_s_f) "heat flux to water - solve for T_i"
q``_burner=k_steel*(T_o-T_i)/delta "heat flux through pan bottom"
T_i_C=converttemp(K,C,T_i) "inner temperature, in C"
T_o_C=converttemp(K,C,T_o) "outer temperature, in C"

which leads to Ti = 106.1ºC and To = 114.1ºC.

c) What would the burner electrical power requirement have to be to achieve the critical heat
flux? Compare the actual heat flux during the boiling process to the critical heat flux.

′′ ); the characteristic length for


The Critical_Heat_Flux function returns the critical heat flux ( qmax
the pan bottom is:

Lchar = A (10)

The burner power required to reach critical heat flux is:

′′
qmax
qburner = (11)
fraction A

"c)"
L=sqrt(A) "estimate of the characteric length to use for this circular plate"
q``_max=Critical_Heat_Flux('Water','Plate', L, T_sat) "determine critical heat flux"
q_burner_max=q``_max/fraction*A "required burner capacity - no danger of reaching burn out here"

which leads to qburner ,max = 62.2 kW. It is not likely that we would need to worry about burnout.

d) How much water is vaporized during the 10 minutes required to cook the spaghetti?

An energy balance determines the amount of water vaporized in Δtboil = 10 minutes:

′′
qburner fraction Δtboil
mvaporized = (12)
( iwater , x=1 − iwater , x=0 )
"d)"
DELTAt_boil=10 [min]*convert(min,s)
m_vaporized=q_burner*DELTAt_boil/(enthalpy(Water,T=T_sat,x=1)-enthalpy(Water,T=T_sat,x=0))

which leads to mvaporized = 0.38 kg.


Problem 7.2-4 (7-4 in text): Tungsten Wire
A tungsten wire having a diameter of 1 mm and a length of 0.45 m is suspended in saturated
carbon dioxide liquid maintained at 3.25 MPa. The fluid-surface coefficient needed in the
nucleate boiling relation, Cnb, is estimated to be 0.01 and the emissivity of the tungsten wire is
0.4. Prepare a plot of the electrical power dissipated in the wire versus the excess temperature
for power levels ranging from 10 W to the power corresponding to the critical heat flux for the
nucleate boiling regime. What is your estimate of the excess temperature at the burnout point?

Known information is entered into EES.

$UnitSystem SI MASS RAD PA K J


$TABSTOPS 0.2 0.4 0.6 0.8 3.5 in

"known information"
Fluid$='CarbonDioxide'
P=3.250 [MPa]*convert(MPa,Pa) "saturation pressure"
d=1 [mm]*convert(mm,m) "diameter of wire"
L=0.45 [m] "length of wire"
T_sat=T_sat(Fluid$,P=P) "saturation temperature"

The critical heat flux ( qs′′,max ) is determined using the Critical_Heat_Flux procedure using the radius
of the wire as the characteristic length.

Geom$='CYLINDER'
q``_max=Critical_Heat_Flux(Fluid$,Geom$, d/2, T_sat) "characteristic length is the radius"

The power needed obtain the critical heat flux is calculated based on the heat flux:

w max = q s′′, max π d L (1)

w_dot_max=q``_max*pi*d*L "maximum power dissipation"

which leads to w max = 966 W; this is used as the maximum power for the plot. A specific excess
temperature can be used to determine the wire surface temperature:

Tw = Tsat + ΔTe (2)

The Nucleate_Boiling procedure is used to relate the excess temperature to the wire heat flux
( q s′′,nb ). The heat flux is used to compute the electrical power:

w nb = qs′′,nb π d L (3)

C_s_f=0.01 [-] "nucleate boiling coefficient"


DT_e=10 [K] "excess temperature"
T_w=T_sat+DT_e "excess temperature for nucleate boiling"
q``_s= Nucleate_Boiling(Fluid$, T_sat, T_w, C_s_f) "nucleate boiling correlation"
w_dot_nb=q``_s*pi*d*L "electrical power"
Figure 1 illustrates the electrical power as a function of excess temperature for nucleate boiling;
the critical heat flux is indicated in the plot. The excess temperature at burnout is approximately
10 K.
1000

900 critical heat flux


800
Electrical power (W)

700

600 nucleate boiling


500

400

300 film boiling


200

100

0
0.5 1 10 100 1000
Excess temperature (K)
Figure 1: Electrical power as a function of excess temperature for nucleate and film boiling.

The Film_Boiling procedure is used to compute the heat flux in a film boiling mode ( qs′′, film ) and
therefore the electrical power to the wire.

w nb = qs′′,nb π d L (4)

epsilon=0.4 [-] "emissivity of tungsten"


q``_film= Film_Boiling(Fluid$,Geom$, T_sat, T_w, d, epsilon) "film boiling correlation"
w_dot_film=q``_film*pi*d*L "electrical power"

The electrical power as a function of excess temperature is overlaid onto Figure 1.


Problem 7.2-5 (7-5 in text): Evacuated Tubular Collector
A cross-section of one type of evacuated solar collector is shown in Figure P7.2-5. The collector
consists of a cylindrical glass tube with an outer diameter of 7.5 cm and wall thickness of 5 mm.
In the center of the tube is a heat pipe, which is a copper tube with an outer diameter of 2 cm and
wall thickness of 1.5 mm. The heat pipe contains a small amount of water at a pressure of 200
kPa that experiences nucleate boiling as solar radiation is incident on the outside surface of the
copper tube at a rate of 745 W/m2. You may assume that the glass is transparent to solar
radiation and that the absorptivity of the copper tube with respect to solar radiation is 1.0. The
surface of the copper tube has an emissivity of 0.13 with respect to its radiative interaction with
the inner surface of the glass tube. The glass may be assumed to be opaque to thermal radiation
from the copper tube with an emissivity of 1.0 on both its inner and outer surfaces. The outside
surface of the glass interacts with the 25°C, 101.3 kPa surroundings through radiation and free
convection.
25°C
745 W/m2 2 cm
1.5 mm
glass 5 mm
water
vacuum

7.5 cm
P7.2-5: Cross-section of an evacuated tubular collector

a.) Calculate the net rate of energy transfer to the water per unit length.

Enter problem information into EES:


$UnitSystem SI MASS RAD PA K J
$TABSTOPS 0.2 0.4 0.6 0.8 3.5 in

"known information"
D_g_o=7.5 [cm]*convert(cm,m) "outer glass diameter"
th_g=5 [mm]*convert(mm,m) "glass cylinder thickness"
D_g_i=D_g_o-2*th_g "inner glass diameter"
D_c_o=2 [cm]*convert(cm,m) "outer copper diameter"
th_c=1.5 [mm]*convert(mm,m) "copper tube thickness"
D_c_i=D_c_o-2*th_c "inner copper diameter"
k_c=k_('Copper',T=300 [K]) "thermal conductivity of copper"
k_g=k_('Glass-Pyrex', T=300 [K]) "thermal conductivity of glass"
q``_a=745 [W/m^2] "rate of solar radiation absorbed on copper tube"
P_sat=200 [kPa]*convert(kPa,Pa) "saturation pressure in the tube"
T_w=T_sat(Water,P=P_sat) "saturation temperature"
T_amb=convertTemp(C,K,25 [C]) "ambient temperature"
P_amb=101.3 [kPa]*convert(kPa,Pa) "ambient pressure"
epsilon=0.13 [-] "emittance of the copper surface"
L=1 [m] "per unit length"

The temperatures of the inner and outer surfaces of the glass (Tg,i and Tg,o, respectively) and the
outer surface of the copper tube (Tc,o) must be assumed in order to calculate the resistances to
radiation and natural convection:
T_g_o=310 [K] "guess temperature for outer glass temperature"
T_g_i=315 [K] "guess temperature for inner glass temperature"
T_c_o=320 [K] "guess temperature for outer tube temperature"

The rate at which energy is absorbed by copper tube is:

qsolar = qa′′ Dc ,o L (1)

where qa′′ is the rate at which solar energy is incident on the copper tube per unit area.

q_dot_solar=2*D_c_o*L*q``_a "rate of energy absorption"

The heat loss is related to radiation from the copper tube to the inner surface of the glass tube,
conduction through the glass, and natural convection and radiation from the outer surface of the
glass surface. The natural convection heat transfer coefficient ( hg ,o ) is found from a free
convection heat transfer correlation using the FC_horizontal_cylinder function in EES. The
resistance to natural convection is given by:

1
Rconv = (2)
hg ,o π Dg ,o L

Call FC_horizontal_cylinder('Air', T_g_o, T_amb, P_amb, D_g_o : h_g_o, Nusselt_g_o, Ra_g_o)


"access natural convection correlation for a cylinder"
R_conv=1/(h_g_o*pi*D_g_o*L) "resistance to natural convection"

The resistance to radiation from the outer surface of the glass is:

1
Rrad , g ,o = (3)
π Dg ,o L σ (T 2
g ,o + Tamb
2
)(Tg ,o + Tamb )
The resistance to conduction through the glass is:

⎛D ⎞
ln ⎜ g ,o ⎟
⎜ Dg ,i ⎟
Rg = ⎝ ⎠ (4)
2 π kg L

The resistance to radiation from the copper tube to the glass is:

1
Rrad , gap = (5)
π Dc ,o L σ ε (T + Tg2,i )(Tc ,o + Tg ,i )
2
c ,o

R_rad_g_o=1/(sigma#*pi*D_g_o*L*(T_g_o^2+T_amb^2)*(T_g_o+T_amb))
"resistance to radiation from the outer surface of the glass"
R_g=ln(D_g_o/D_g_i)/(2*pi*k_g*L) "resistance of the glass wall"
R_rad_gap=1/(sigma#*epsilon*pi*D_c_o*L*(T_c_o^2+T_g_i^2)*(T_c_o+T_g_i))
"resistance to radiation from the outer surface of the glass"

The rate of heat loss is:

qloss =
(T c ,o − Tamb )
(6)
−1
⎛ 1 1 ⎞
Rrad , gap + Rg + ⎜ + ⎟⎟
⎜R
⎝ conv Rrad , g ,o ⎠

q_dot_loss=(T_c_o-T_amb)/(R_rad_gap+R_g+(1/R_conv+1/R_rad_g_o)^(-1)) "loss"

The guess values are updated and the guessed values for Tg,o and Tg,i are commented out. These
values are calculated according to:

Tg ,i = Tc ,o − qloss Rrad , gap (7)

Tg ,o = Tg ,i − qloss Rg (8)

{T_g_o=310 [K]} "guess temperature for outer glass temperature"


{T_g_i=315 [K]} "guess temperature for inner glass temperature"
T_g_i=T_c_o-q_dot_loss*R_rad_gap "recalculate inner glass temperature"
T_g_o=T_g_i-q_dot_loss*R_g "recalculate outer glass temperature"

The inner temperature of the copper tube is calculated according to:

Tc ,i = Tc ,o − ( qsolar − qloss ) Rc (9)

where Rc is the resistance to conduction through the copper:

⎛D ⎞
ln ⎜⎜ c ,o ⎟⎟
Dc ,i ⎠
Rc = ⎝ (10)
2 π kc L

R_c=ln(D_c_o/D_c_i)/(2*pi*k_c*L) "conduction resistance of copper"


T_c_i=T_c_o-(q_dot_solar-q_dot_loss)*R_c "inner temperature of copper"

The heat flux experienced at the inner surface of the copper tube is:

qc′′,i =
( qsolar − qloss ) (11)
π Dc ,i L

q``_c_i=(q_dot_solar-q_dot_loss)/(pi*D_c_i*L) "heat flux on inner surface of copper"


The guess values are updated and the assumed value of Tc,o is commented out. The
Nucleate_Boiling procedure is used to determine the value of Tc,o that causes the calculated heat
flux to agree with the heat flux that can be supported at the excess temperature:

{T_c_o=320 [K]} "guess temperature for outer tube temperature"


C_s_f=0.013 [-] "coefficient for nucleate boiling"
q``_c_i=Nucleate_Boiling('Water', T_w, T_c_i, C_s_f) "estimate heat flux"

The rate of energy transfer to the water is:

qwater = qsolar − qloss (12)

q_dot_water=q_dot_solar-q_dot_loss "rate of heat transfer to the water"

which leads to qwater = 22.4 W/m of collector.

b.) Calculate the efficiency of the solar collector, defined as the ratio of the rate of energy
transfer to the water in the heat pipe to the incident solar radiation.

The efficiency is calculated according to:

qwater
η= (13)
qsolar

eta=q_dot_water/q_dot_solar "efficiency"

which leads to η = 0.752 (75.2%).


Problem 7.3-1 (7-6 in text): Effect of Pressure Drop on Saturation Temperature
When one fluid is changing phase in a heat exchanger, it is commonly assumed to be at a
uniform temperature. However, there is a pressure drop in the evaporating fluid, which affects
its saturation temperature. In a particular case, a 2 m long horizontal concentric tube heat
exchanger made of copper is used to evaporate 0.028 kg/s of refrigerant R134a with an entering
state of 300 kPa with a quality of 0.35. Heat transfer is provided by a flow of water that enters
the heat exchanger at 12°C, 1.10 bar with a mass flow rate of 0.20 kg/s. The refrigerant passes
through the central tube of the heat exchanger, which has an inner diameter of 1.25 cm and a
wall thickness of 2 mm. The water flows through the annulus; the inner diameter of the outer
tube is 2.5 cm.
a) Estimate the outlet temperature of the water and the outlet temperature and quality of the
refrigerant.

Enter the known information into EES.

$UnitSystem SI MASS RAD PA K J


$TABSTOPS 0.2 0.4 0.6 0.8 3.5 in

"known information"
R$='R134a'
p_in=300 [kPa]*convert(kPa,Pa) "entering R134a pressure"
x_in=0.35 "entering quality"
d_i=1.25 [cm]*convert(cm,m) "inner diameter of inner tube"
d_o=2.5 [cm]*convert(cm,m) "inner diameter of outer tube"
delta=2 [mm]*convert(mm,m) "wall thickness"
m_dot_R=0.028 [kg/s] "R134a flow rate"
T_w_in=convertTemp(C,K,12 [C]) "water temperature"
P_w=1.1[bar]*convert(bar, Pa) "water pressure"
m_dot_w=0.2 [kg/s] "water temperature"
L=2 [m] "length of concentric tube"
T_r_in=T_sat(R$,P=P_in) "inlet temperature of refrigerant"

The outlet quality of the refrigerant (xout) is guessed in order to get started.

x_out=0.9 [-] "guess for outlet quality"

The mass flux of the refrigerant is computed:

4 m r
G= (1)
π di2

where di is the inner diameter of the inner tube. The function DELTAP_2phase_horiz is used to
estimate the pressure drop of the flowing refrigerant (Δpr). The refrigerant outlet pressure is
estimated according to:

pout = pin − Δpr (2)


Provided that the exiting refrigerant is two-phase, the exit temperature (Tr,out) is the saturation
temperature at pout. The heat transfer rate is computed according to:

(
q = m r ir ,T =Tr ,out , x = xout − ir ,T =Tr ,in , x = xin ) (3)

where ir is the enthalpy of the refrigerant.

G=m_dot_R/(pi*d_i^2/4) "mass velocity of refrigerant"


DELTAP=DELTAP_2phase_horiz(R$,G,P_in,d_i,L,x_in, x_out)
"determine pressure drop for refrigerant"
p_out=p_in-DELTAP "outlet pressure"
T_r_out=T_sat(R$,P=p_out) "outlet temperature of refrigerant"
q_dot=m_dot_R*(enthalpy(R$,T=T_r_out,x=x_out)-enthalpy(R$,T=T_r_in,x=x_in)) "rate of heat transfer"

The inner and outer radii of the annulus are calculated according to:

ri =
( di + 2 δ ) (4)
2

do
ro = (5)
2

The average heat transfer coefficient on the water side ( hw ) is obtained using the AnnularFlow
procedure. The resistance to convection on the water side is:

1
Rconv , w = (6)
2 π ri L hw

r_i=(d_i+2*delta)/2 "inner radius of annulus"


r_o=d_o/2 "outer radius of annulus"
RelRough=0 [-] "relative roughness -assumed"
call AnnularFlow('water',T_w_in,P_w,m_dot_w,r_i,r_o,L,RelRough:h_bar_w, h_H ,&
DELTAP_w, Nusselt_T, f, Re) "determines heat transfer coefficient on water side"
R_conv_w=1/(2*pi*r_i*L*h_bar_w) "resistance to convection on water side"

The resistance to conduction through the tube is:

⎛ d + 2δ ⎞
ln ⎜ i ⎟
Rcond = ⎝ di ⎠
(7)
2 π ktube L

k_tube=k_('Copper', T_r_in) "conductivity of copper"


R_cond=ln((d_i+2*delta)/d_i)/(2*pi*k_tube*L) "resistance to conduction through tube"
The heat transfer coefficient on the refrigerant side ( hr ) depends on the heat flux, which is
computed according to:

q
q ′′ = (8)
π di L

The heat transfer coefficient is computed using the Flow_Boiling_avg function and used to
compute the resistance to convection on the refrigerant side:

1
Rconv ,r = (9)
π di L hr

q``_dot=q_dot/(pi*d_i*L) "heat flux on refrigerant side"


h_bar_r=Flow_Boiling_avg(R$, T_r_in, G, d_i, x_in, x_out, q``_dot, 'Horizontal')
"heat transfer coefficient on refrigerant side"
R_conv_r=1/(pi*d_i*L*h_bar_r) "resistance to convection on refrigerant side"

The overall conductance is computed:

UA = ( Rconv ,r + Rcond + Rconv , w )


−1
(10)

The performance is computed assuming that the water is exposed to a uniform external
temperature that is the average refrigerant temperature:

Tr ,in + Tr ,out
Tr = (11)
2

so the outlet water temperature is:

⎛ UA ⎞
Tw,out = Tr − (Tr − Tw,in ) exp ⎜ − ⎟ (12)
⎝ m w cw ⎠

where cw is the specific heat capacity of the water.

UA=(R_conv_w+R_cond+R_conv_r)^(-1) "overall conductance"


T_r_avg=(T_r_in+T_r_out)/2 "average refrigerant temperature"
c_w=cP(Water,T=T_w_in,P=P_w) "specific heat capacity of water"
T_w_out=T_r_avg-(T_r_avg-T_w_in)*exp(-UA/(m_dot_w*c_w)) "water outlet temperature"

The guess values are updated and the assumed value of xout is commented out. The rate of heat
transfer is computed using an energy balance on the water side:

q = m w cw (Tw,in − Tw,out ) (13)


{x_out=0.9 [-]} "guess for outlet quality"
q_dot=m_dot_w*c_w*(T_w_in-T_w_out) "recalculate heat transfer rate"

which leads to Tw,out = 283.5 K, xout = 0.595, and Tr,out = 273.7 K.


Problem 7.3-2 (7-7 in text): Circular Finned-Tube Evaporator
A circular finned tube evaporator designed for cooling air is made from aluminum. The outer
diameter of the tubes is 10.21 mm with a tube wall thickness of 1 mm. There evaporator is
plumbed such that there are 12 parallel circuits of tubes with each circuit having a length of 0.6
m. Refrigerant R134a enters the evaporator at a mass flowrate of 0.15 kg/s. The refrigerant
enters the throttle valve upstream of the evaporator as 35°C saturated liquid. The pressure in the
evaporator is 240 kPa. The refrigerant exits as a saturated vapor.
a) What is the rate of heat transfer to the air for this evaporator.

Enter the problem information into EES.

$UnitSystem SI MASS RAD PA K J


$TABSTOPS 0.2 0.4 0.6 0.8 3.5 in

"known information"
d_o=10.21 [mm]*convert(mm,m) "outer diameter of the tube"
thickness=1 [mm]*convert(mm,m) "tube thickness"
d_i=d_o-2*thickness "inner diameter of the tube"
m_dot_R=0.15 [kg/s] "refrigerant mass flow rate"
P_evap_i=240 [kPa]*convert(kPa,Pa) "inlet pressure"
T_sat=T_sat(R134a,P=P_evap_i) "saturation temperature"
T_cond=converttemp(C,K,35 [C]) "condenser temperature"
L=0.6 [m] "length"
N_circuits=8 "number of parallel circuits"

The heat transfer rate is found from an energy balance on the refrigerant.

(
q = m r ir ,T =Tcond , x =0 − ir , P = pevap ,i , x =1 ) (1)

"a) Estimate the heat transfer rate"


h_i=enthalpy(R134a,T=T_cond,x=0) "enthalpy of entering refrigerant"
h_e=enthalpy(R134a,P=P_evap_i,x=1) "enthalpy of exiting refrigerant"
q_dot=m_dot_R*(h_e-h_i) "heat transfer rate"

which leads to q = 21.96 kW.

b) Estimate of the average heat transfer coefficient between the R134a and the tube wall

The heat transfer coefficient between the R134a and the tube wall is found using the
Flow_boiling_avg function. The heat flux needed as an input is the heat transfer rate divided by
the surface area of the tubes, considering the 8 parallel circuits.

q
q ′′ = (2)
N π di

and the mass flux is:


m r
G= (3)
N Atube

where Atube is the cross-sectional area of a tube:

di2
Atube =π (4)
4

The inlet quality to the evaporator is the quality leaving the valve, specified by the enthalpy of
the refrigerant entering the valve and the evaporator pressure.

"b) Estimate the heat transfer coefficient"


A_tube=pi*d_i^2/4 "cross-sectional area of tube"
G=m_dot_R/(A_tube*N_circuits) "mass flux"
x_i=quality(R134a,h=h_i,P=P_evap_i) "inlet quality"
x_e=1 [-] "exit quality"
q``=q_dot/(N_circuits*pi*d_i*L) "heat flux"
h_bar=Flow_Boiling_avg('R134a', T_sat, G, d_i, x_i, x_e, q``, 'Horizontal')
"average heat transfer coefficient"

c) Estimate the pressure drop of the R134a as it pass through the evaporator. Does this pressure
drop significantly affect the saturation temperature?

The pressure drop calculation requires use of a 2-phase pressure drop correlation which is
provided in the DELTAP_2phase_horiz function.

"c) estimate the pressure drop"


DELTAP=DELTAP_2phase_horiz('R134a',G,P_evap_i,d_i,L,x_i, x_e) "pressure drop"
T_sat_out=temperature('R134a',P=P_evap_i-DELTAP,x=x_e) "exit saturation temperature"

The pressure drop is 13.86 kPa and the change in the saturation temperature is 1.5 K.
Problem 7.3-4 (7-8 in text)
Repeat EXAMPLE 7.3-1 using the Flow_Boiling_avg function rather than integrating to determine
the average heat transfer coefficient. Compare the two methods of obtaining the average heat
transfer coefficient.

The relevant information from EXAMPLE 7-3-1 is entered into EES.

$UnitSystem SI MASS RAD PA K J


$Tabstops 0.2 0.4 0.6 3.5 in

F$='CarbonDioxide' "fluid type"


D=2.5 [mm]*convert(mm,m) "tube inner diameter"
L=2 [m] "tube length"
p_sat=3.2 [MPa]*convert(MPa,Pa) "boiling saturation pressure"

The mass flux will be varied in a parametric table, but a particular value will be assigned to test
the program.

$ifnot ParametricTable
G=300 [kg/s-m^2] "mass velocity"
$endif

Both the integrated heat transfer coefficient and the result from the Flow_Boiling_avg function are
calculated and compared. The average value of the heat transfer coefficient it found by
calculating the value as a function of quality and integrating for qualities between 0 and 1, as
described in EXAMPLE 7.3.1. The Flow_Boiling_Avg function provides the result directly. The
two method provide essentially identical results, as shown in Figure 1.

T_sat=T_sat(F$,P=p_sat) "saturation temperature"


i_v_sat=enthalpy(F$,T=T_sat,x=1) "specific enthalpy of saturated vapor"
i_l_sat=enthalpy(F$,T=T_sat,x=0) "specific enthalpy of saturated liquid"
DELTAi_vap=i_v_sat-i_l_sat "enthalpy change of vaporization"
q_dot=G*pi*D^2/4*DELTAi_vap "rate of heat transfer"
q``_dot_s=q_dot/(pi*D*L) "heat flux"
call Flow_Boiling(F$, T_sat, G, D, x, q``_dot_s, 'Horizontal': h, T_w) "heat transfer coefficient"
x_in=0
x_out=1
h_bar`=Flow_Boiling_avg(F$, T_sat, G, D, x_in, x_out, q``_dot_s, 'Horizontal')
h_bar=G*D*DELTAi_vap*INTEGRAL(h,x,x_in,x_out)/(4*L*q``_dot_s) "average heat transfer coefficient"
8000

Average heat transfer coefficient (W/m -K)


Flow Boiling Avg function

2
EXAMPLE 7.3-1
7000

6000

5000

4000

3000

2000
1.5x104 2.0x104 2.5x104 3.0x104 3.5x104 4.0x104
2
Heat flux (W/m )
Figure 1: Average heat transfer coefficient computed using the Flow_Boiling_Avg function or using the
integration described in EXAMPLE 7.3-1.
Problem 7.3-5 (7-9 in text): Electronic Cooling Alternative
A computer manufacturer is reviewing alternative ways to remove heat from electronic
components. The electronic circuit board can be assumed to be a thin horizontal plate with a
width of 8 cm and a length of 16 cm. Currently, air is blown over the top of the circuit board at a
velocity of 10 m/s. Additional cooling could be obtained by a higher air velocity, but the
increased noise associated with the larger fan required is judged to be unacceptable. Another
alternative is to immerse the board in a fluid at atmospheric pressure that is undergoing nucleate
boiling. The fluid R245fa has been chosen as a possibility. The surface tension of R245fa at
atmospheric pressure is 0.0153 N/m. Other thermodynamic and transport properties are
available from EES. Prepare a plot that shows the surface temperature of the plate as a function
of the heat flux using air at 10 m/s and nucleate boiling at atmospheric pressure with R245fa for
heat fluxes ranging from 100 to 10000 W/m2.

Known information is entered into EES along with a specified heat flux. The heat flux will later
be varied in a Parametric table.

"known information"
W=8 [cm]*convert(cm,m) "width of plate"
L=16 [cm]*convert(cm,m) "length of plate in flow direction"
u_inf=10 [m/s] "air velocity"
T_air_inf=convertTemp(C,K,25 [C]) "air inlet temperature"
P=101.3 [kPa]*convert(kPa,Pa) "atmospheric pressure"
$ifnot ParametricTable
q``=1000 [W/m^2] "heat flux"
$endif

The heat transfer coefficient if air is used as the coolant is found from the External_Flow_Plate
function that implements a convective flow relation for external flow over a flat horizontal plate
( ha ). Since the heat flux is known, the energy balance determines the surface temperature.

q ′′ = ha (Ts ,air − T∞ ,air ) (1)

Call External_Flow_Plate('Air', T_air_inf, T_s_air, P, u_inf, L: tau, h_air, C_f, Nusselt, Re)
q``=h_air*(T_s_air-T_air_inf) "determine surface temperature with air cooling"

Complete property data are not provided for R234fa in the EES data base, making it necessary to
implement a function that calculates the heat flux according to Eq. (7-3) in the text. This
implementation has been provided as a function. Enter the following function at the top of the
EES equations window.

Function Nucleate_BoilingR245fa(Fluid$, T_sat, T_w, C_s_f)


rho_l=density(Fluid$,T=T_sat,x=0) "density of saturated liquid"
rho_g=density(Fluid$,T=T_sat,x=1) "density of saturated vapor"
k_l=conductivity(Fluid$,T=T_sat,x=0) "thermal conductivity of saturated liquid"
mu_l=viscosity(Fluid$,T=T_sat,x=0) "thermal conductivity of saturated vapor"
c_l=cp(Fluid$,T=T_sat,x=0) "specific heat oof saturated liquid"
Pr_l=Prandtl(Fluid$, T=T_sat, x=0) "Prandtl number for saturated liquid"
h_fg=enthalpy(Fluid$,T=T_sat,x=1)-enthalpy(Fluid$,T=T_sat,x=0) "enthalyp of vaporization"
n=1.7 "exponent on Prandtl number"
sigma_1=0.0153 [N/m] "surface tension for R245fa"
DELTAT_e=T_w-T_sat "excess temperature difference"
g=g# "gravitational acceleration"
Nucleate_BoilingR245fa=mu_l*h_fg*((g*(rho_l-
rho_g))/(sigma_1))^(1/2)*((c_l*DELTAT_e)/(C_s_f*h_fg*Pr_l^n))^3
END

The function is accessed with the following EES statement at the bottom of the Equations
window.

Fluid$='R245fa'
T_sat=T_sat(Fluid$,P=P) "saturation temperature"
C_s_f=0.012 "nucleate boiling coefficient"
q``=Nucleate_BoilingR245fa(Fluid$, T_sat, T_s_R, C_s_f) "determines surface temperature with
nucleate boiling"

Figure 1 illustrates the surface temperatures with air and R245fa cooling.
650

600 air
Surface temperature (K)

550

500

450

400

350
R245fa
300

250
0 2000 4000 6000 8000 10000
2
Heat flux (W/m )
Figure 1: Surface temperature for air and R245fa cooling as a function of heat flux.
Problem 7.4-1 (7-10 in text): Evacuated Tubular Solar Collectors with a Heat Pipe
Evacuated tubular solar collectors often employ a heat pipe to transfer collected solar energy for
water heating. Heat transfer between the water that is being heated and the solar collector occurs
at the condenser of the heat pipe, which is a thin-walled cylinder made of copper with a length of
6 cm and a diameter of 1 cm as shown in Figure P7.4-1. Water at 40°C and 1 atm flows past the
condenser at a velocity that can be specified by the flow rate and duct diameter. The fluid inside
the heat pipe is also water and it condenses at a pressure of 100 kPa. The heat transfer situation
of the condensing water within the heat pipe is not known, but will here be assumed that is can
be represented with the same relations as used for film condensation on the inside surface of a
cylinder. This heat transfer coefficient for film condensation is provided by the
Cond_horizontal_Cylinder procedure when the mass flow rate is set to 0. Plot the rate of heat
transfer from the solar collector to the water that is being heated as a function of the flow
velocity of the water for velocities between 1 and 10 m/s.

water at 40°C, 100 kPa, u w

solar collector

6 cm
1 cm
condenser
Figure P7.4-1: Condenser of evacuated solar collector.

The information in the problem statement is entered into EES. A velocity of 5 m/s is selected
initially. After the equations are entered this specification will be replaced with a Parametric
table that varies the velocity from 1 to 10 m/s.

$UnitSystem SI MASS RAD PA K J


$Tabstops 0.2 0.4 0.6 3.5 in

L=6 [cm]*convert(cm,m) "length of condenser"


D=1 [cm]*convert(cm,m) "diameter of condenser"
Fluid$='Water' "fluid in pipe"
P_sat=1 [atm]*convert(atm,Pa) "pressure inside of the heat pipe"
T_sat=T_sat(Fluid$,P=P_sat) "saturation temperature inside the heat pipe"

P=100 [kPa]*convert(kPa,Pa) "atmospheric pressure"


T_inf=convertTemp(C,K,40[C]) "inlet water temperature"
$ifnot ParametricTable
u_inf=5 [m/s] "inlet water velocity"
$endif

The surface area of the cylinder in contact with the water is determined.

A=π DL (1)
A=pi*D*L "surface area of the cylinder"

An initial guess for the wall temperature (Tw) is made in order to estimate the heat transfer
coefficient associated with film condensation and external forced convection.

T_w=330 [K] "guess for wall temperature"

As indicated in the problem statement, the heat transfer coefficient for film condensation ( hi ) can
be found using the Cond_Horizontal_Cylinder procedure with the mass flow rate set to 0. The
resistance to film condensation is:

1
Rconv ,i = (2)
hi A

Call Cond_horizontal_Cylinder(Fluid$, T_sat, T_w, D:h_i, Nusselt_m)


"heat transfer coefficient for film condensation"
R_conv_i=1/(h_i*A) "resistance to convection on inside of tube"

The heat transfer coefficient on the outside of the cylinder ( ho ) is found using an external flow
convection relation provided in the External_Flow_Cylinder procedure. The resistance to
convection on the outer surface of the tube is:

1
Rconv ,o = (3)
ho A

Call External_Flow_Cylinder(Fluid$, T_inf, T_w, P, u_inf, D: F_d\L, h_o, C_d, Nusselt, Re)
"heat transfer coefficient on outer surface"
R_conv_o=1/(h_o*A) "resistance to convection on outside of tube"

The rate of heat transfer is given by:

q =
(Tsat − T∞ ) (4)
Rconv ,i + Rconv ,o

q_dot=(T_sat-T_inf)/(R_conv_i+R_conv_o) "rate of heat transfer"

The guess values are updated and the assumed value of Tw is commented out. Note that this
program works best if limits are placed on the value of Tw (from a lower limit of T∞ to an upper
limit of Tsat). The wall temperature is calculated according to:

Tw = Tsat − Rconv ,i q (5)

{T_w=330 [K]} "guess for wall temperature"


T_w=T_sat-R_conv_i*q_dot "recalculate wall temperature"
A parametric table with columns for u_inf and q_dot is constructed. Figure 2 illustrates the rate of
heat transfer as a function of the water velocity.

950

900

850
Heat transfer rate (W)

800

750

700

650

600

550
1 2 3 4 5 6 7 8 9 10
Water velocity (m/s)
Figure 2: Heat transfer rate as a function of water velocity.
Problem 7.4-2 (7-11 in text): Steam Power Cycle Condenser
The condenser in steam power cycle utilizes a shell and tube heat exchanger that consists of 1200
nominal 1.5 inch schedule 40 tubes made of brass. Each tube is 8 ft long internally smooth.
Cooling water enters each of the tubes at 68°F and exits at 74°F. Saturated steam at 1 psia
having a quality of 91% enters the condenser at a low velocity and is condensed on the tubes.
Estimate the rate of condensate formation and the associated water flow rate at steady state
conditions.

Enter the known information into EES.

$UnitSystem SI MASS RAD PA K J


$Tabstops 0.2 0.4 0.6 3.5 in

"Known"
N=1200 [-] "number of tubes"
L=8 [ft]*convert(ft,m) "length of tubes"
P_sat=1 [psi]*convert(psi,Pa) "saturation temperature of steam"
x_in=0.91 [-] "inlet quality"
T_w_in=converttemp(F,K,68 [F]) "inlet water temperature"
T_w_out=converttemp(F,K,74 [F]) "exit water temperature"
D_i=1.61 [in]*convert(in,m) "inner diameter of 1.5 in Schedule 40 pipe"
delta=0.145 [in]*convert(in,m) "wall thickness"
D_o=D_i+2*delta "outer diameter of 1.5 in Schedule 40 pipe"
T_w_avg=(T_w_in+T_w_out)/2 "average temperature of water in pipe"
k_w=k_('brass',T=(T_sat+T_w_avg)/2) "thermal conductivity of brass"
P_atm=14.7 [psi]*convert(psi,Pa) "atmospheric pressure"

The saturation temperature of the water can be determined using the EES T_sat function.

T_sat=T_sat('Water',P=P_sat) "saturation temperature"

The wall temperature (Twall,o) is needed to determine the condensation heat transfer coefficient,
but it is not known. Provide a guess for this temperature.

T_wall_o=converttemp(F,K,101 [F]) "guess for outer pipe wall temperature"

The Cond_horizontal_N_Cylinders function is used to determine the heat transfer coefficient for
condensation over a bank of cylinders ( hcond ). The resistance to condensation is:

1
Rcond = (1)
hcond π Do L N

Call Cond_horizontal_N_Cylinders('Water', T_sat, T_wall_o, D_i, N:h_cond, Nusselt_cond)


"determine condensation heat transfer coefficient"
R_cond=1/(h_cond*pi*D_o*L*N) "resistance to condensation"

The resistance to conduction through the tube is:


⎛D ⎞
ln ⎜ o ⎟
Rtube = ⎝ Di ⎠ (2)
2 π kw L N

where kw is the conductivity of the tube.

R_tube=ln(D_o/D_i)/(2*pi*k_w*L*N) "resistance to conduction"

The properties of the cooling water should be evaluated at the average water temperature. The
PipeFlow procedure implements an internal flow correlation that determines the heat transfer
coefficient to the cooling water ( hw ). A mass flow rate of water must be assumed to carry out
this calculation. The resistance to convection to the flowing water is:

1
Rconv ,i = (3)
hw π Di L N

m_dot_w=0.1 [kg/s] "guess for total water mass flow rate"


RelRough=1e-8 "relative roughness for smooth tube"
call PipeFlow('Water',T_w_avg,P_atm,m_dot_w/N,D_i,L,RelRough:h_w, h_H ,DELTAP, Nusselt_T, f, Re)
R_conv_i=1/(h_w*pi*D_i*L*N) "resistance to convection on inside of tube"

The overall conductance of the heat exchanger is calculated according to:

UA = ( Rcond + Rtube + Rconv ,i )


−1
(4)

and used to determine the outlet water temperature according to the method discussed in Section
5.3.4.

⎛ UA ⎞
Tw,out = Tsat − (Tsat − Tw,in ) exp ⎜ − ⎟ (5-5)
⎝ m w cw ⎠

where cw is the specific heat capacity of the water.

UA=(R_cond+R_tube+R_conv_i)^(-1) "conductance"
c_w=cp(Water,T=T_w_avg,P=P_atm) "specific heat of water"
T_w_out=T_sat-(T_sat-T_w_in)*exp(-UA/(m_dot_w*c_w)) "outlet water temperature"

The problem is solved and the guess values are updated. The assumed mass flow rate is
commented out and the mass flow rate is recalculated using an energy balance.

m w cw (Tw,out − Tw,in ) = q (6)

{m_dot_w=0.1 [kg/s]} "guess for total water mass flow rate"


m_dot_w*c_w*(T_w_out-T_w_in)=q_dot "energy balance on water"
The problem is solved and the guess values are updated. The assumed wall temperature is
commented out and the wall temperature is estimated according to:

Twall ,o = Tsat − q Rcond (7)

{T_wall_o=converttemp(F,K,101 [F])} "guess for outer pipe wall temperature"


T_wall_o=T_sat-q_dot*R_cond "recalculate wall temperature"

which leads to m w = 55.25 kg/s. The mass flow rate of condensate is estimated using an energy
balance:

q
m condensate = (8)
(i
water ,T =Tsat , x = xin − iwater ,T =Tsat , x =0 )
m_dot_condensate=q_dot/(enthalpy(water,T=T_sat,x=x_in)-enthalpy(Water,T=T_sat,x=0))
"mass flow rate of condensate"

which leads to m condensate = 0.352 kg/s.


Problem 7.4-3 (7-12 in text): R245fa Recovery
Problem 7.3-5 (7-9 in text) described an electronics cooling system that removes the heat
dissipated in an electronic circuit board by submersing the board in R245fa. The circuit board is
maintained at a relatively low and uniform temperature over a range of heat fluxes by boiling
R245fa. However, a problem now arises in dealing with the vapor produced by the evaporation.
One possibility is to condense the vapor on the bottom side of a vertical plate that is cooled by
chilled water on its top side, as shown in Figure P7.4-3 in a sealed container. The top of the
enclosure is made of metal and it can be considered to be isothermal. The chilled water is at 1
atm and has a free stream velocity of 10 m/s and a free stream temperature of 10°C. The circuit
board is 8 cm wide and 16 cm long. The saturation pressure (and thus temperature) of the
R245fa in the enclosure should vary with the heat flux.

water at
10°C, 1 atm, 10 m/s

16 cm
saturated R245fa

circuit board
8 cm x 16 cm
Figure P7.4-3: Sealed container full of evaporating and condensing R245fa.

a.) Prepare a plot of the saturation pressure and circuit board surface temperature as a function
of the heat flux for heat fluxes ranging from 100 to 10000 W/m2.

Enter known information into EES. The heat flux will initially be set to a 8,000 W/m2. It will be
later varied in a Parametric table.

$UnitSystem SI MASS RAD PA K J


$Tabstops 0.2 0.4 0.6 3.5 in

"known"
Fluid$='R245fa'
W=8 [cm]*convert(cm,m) "width of plate"
L=16 [cm]*convert(cm,m) "length of plate in flow direction"
T_w_inf=convertTemp(C,K,10 [C]) "water inlet temperature"
P=101.3 [kPa]*convert(kPa,Pa) "atmospheric pressure"
q``=8000 [W/m^2] "heat flux"
u_inf=10 [m/s] "water velocity"

The heat transfer coefficient for the top plate ( hw ) is found using an external flow relation for
flow over a flat plate provided in the External_Flow_Plate function. The plate temperature, Tplate,
is needed as an input, but it is not yet known. Set Tplate to a reasonable value, e.g., 290 K.
T_plate=290 [K] "guess for plate temperature"
Call External_Flow_Plate('Water', T_w_inf, T_plate, P, u_inf, L: tau, h_bar_w, C_f, Nusselt, Re)

The equations are solved and the guess values updated. The assumed value of Tplate is
commented out and the plate temperature is recalculated using Newton's law of cooling:

q ′′ = hw (Tplate − T∞ ) (1)

{T_plate=290 [K]} "guess for plate temperature"


q``=h_bar_w*(T_plate-T_w_inf) "recalculate plate temperature"

Because the surface tension of R245fa is not available in EES, it is necessary to write a function
Cond_horizontal_downR245fa that is identical to the built-in function Cond_horizontal_down except
that the surface tension is specified:

Procedure Cond_horizontal_downR245fa(Fluid$, T_w, T_sat: h_m, Nusselt_m)


DELTAT=(T_sat-T_w) "temperature difference"
if (DELTAT<0) then DELTAT=0.001 "ensure that the temperature difference is positive"
T_f=(T_sat+T_w)/2
"average temperature of fluid - used in determining fluid properties"
rho_l=density(Fluid$,T=T_f,x=0) "density of saturated liquid"
rho_g=density(Fluid$,T=T_f,x=1) "density of saturated vapor"
k_l=conductivity(Fluid$,T=T_f,x=0) "conductivity of saturated liquid"
mu_l=viscosity(Fluid$,T=T_f,x=0) "viscosity of saturated liquid"
c_l=cp(Fluid$,T=T_f,x=0) "specific heat of saturated liquid"
h_fg=enthalpy(Fluid$,T=T_f,x=1)-enthalpy(Fluid$,T=T_f,x=0) "
sigma=0.0153 [N/m] "surface tension"
Ra=g#*rho_l*(rho_l-rho_g)*h_fg/(mu_l*DELTAT*k_l)*(sigma/(g#*(rho_l-rho_g)))^(3/2)
"Rayleigh number"
If (Ra<1e8) then
Nusselt_m=0.69*Ra^(0.20) "Eq. (7-91)"
else
Nusselt_m=0.81*Ra^(0.193) "Eq. (7-92)"
endif
h_m=Nusselt_m*k_l*(sigma/(g#*(rho_l-rho_g)))^(-1/2) "
END

The saturation temperature is required in order to use the condensation correlation; assume a
reasonable value of Tsat and then call the procedure Cond_horizontal_downR245fa in order to
evaluate the heat transfer coefficient associated with condensation, hcond .

T_sat=300 [K] "guess for saturation temperature"


Call Cond_horizontal_downR245fa(Fluid$, T_plate, T_sat: h_bar_cond, Nusselt_cond)

The equations are solved and the guess values updated. The assumed value of Tsat is commented
out and the saturation temperature is recalculated using Newton's law of cooling:

q ′′ = hcond (Tsat − Tplate ) (2)


The saturation pressure (psat) is obtained using the saturation temperature.

q``=h_bar_cond*(T_sat-T_plate) "recalculate saturation temperature"


p_sat=pressure(Fluid$,T=T_sat,x=0) "saturation pressure"

The function Nucleate_BoilingR245fa is copied from Problem 7.3-5 (7-9 in text)

Function Nucleate_BoilingR245fa(Fluid$, T_sat, T_w, C_s_f)


rho_l=density(Fluid$,T=T_sat,x=0) "density of saturated liquid"
rho_g=density(Fluid$,T=T_sat,x=1) "density of saturated vapor"
k_l=conductivity(Fluid$,T=T_sat,x=0) "thermal conductivity of saturated liquid"
mu_l=viscosity(Fluid$,T=T_sat,x=0) "thermal conductivity of saturated vapor"
c_l=cp(Fluid$,T=T_sat,x=0) "specific heat of saturated liquid"
Pr_l=Prandtl(Fluid$, T=T_sat, x=0) "Prandtl number for saturated liquid"
h_fg=enthalpy(Fluid$,T=T_sat,x=1)-enthalpy(Fluid$,T=T_sat,x=0) "enthalpy of vaporization"
n=1.7 "exponent on Prandtl number"
sigma_1=0.0153 [N/m] "surface tension for R245fa"
DELTAT_e=T_w-T_sat "excess temperature difference"
g=g# "gravitational acceleration"
Nucleate_BoilingR245fa=mu_l*h_fg*((g*(rho_l-&
rho_g))/(sigma_1))^(1/2)*((c_l*DELTAT_e)/(C_s_f*h_fg*Pr_l^n))^3
END

and used to determine the circuit board temperature:

C_s_f=0.012 [-] "nucleate boiling coefficient"


q``=Nucleate_BoilingR245fa(Fluid$, T_sat, T_cb, C_s_f) "circuit board temperature"
T_cb_C=converttemp(K,C,T_cb) "in C"

Figure 2 illustrates the circuit board temperature and saturation pressure as a function of heat
flux.

45 115000
Circuit board surface temperature (°C)

40 110000
Saturation pressure (Pa)

105000
35 Tcb
100000
30
95000
psat
25
90000

20 85000

15 80000
0 2000 4000 6000 8000 10000
2
Heat flux (W/m )
Figure 2: Circuit board surface temperature and saturation pressure as a function of heat flux.
b.) How sensitive are the results to the velocity of the chilled water?

Figure 3 illustrates the circuit board temperature as a function of heat flux for various values of
the chilled water velocity and shows that the results are not very sensitive to this parameter.

45
Circuit board temperature (°C)

40 u = 5 m/s

35
u = 25 m/s

30

25

20

15
0 2000 4000 6000 8000 10000
2
Heat flux (W/m )
Figure 3: Circuit board temperature as a function of heat flux for various values of the water velocity.
Problem 7.4-4 (7-13 in text): Purification of R134a
Recycled refrigerant R134a is purified in a simple distillation process in which a heater,
submerged in the liquid refrigerant heats the liquid, which causes it to vaporize. The distillation
unit is a container with a square base that is 25 cm on a side. Piping and a float valve (not
shown) are provided to maintain a constant liquid level of refrigerant in the container as
condensate is removed. The vapor condenses on the bottom side of a copper plate that is placed
at the top of the device, as shown in Figure P7.4-4. Liquid water at 25°C with free stream
velocity 3 m/s flows over the top of the copper plate. The plate is slightly inclined so that the
condensed refrigerant travels to the left side of the bottom side of the plate and drips into a
collection gutter.

water at
25°C, 3m/s copper plate

25 cm
saturated R134a

heater
Figure P7.4-4: Refrigerant recycling apparatus

a.) Calculate the saturation pressure and temperature of the refrigerant as a function of heater
power for a range of heater powers from 100 W to 1000 W.

Known information is entered into EES. The heater power, W_dot is entered. It will later be
varied in a parametric table.

$UnitSystem SI MASS RAD PA K J


$Tabstops 0.2 0.4 0.6 3.5 in

F$='R134a' "refrigerant"
L=0.25 "length of one side of container"
A=L*L "area of the heat exchange surface"
T_water=convertTemp(C,K,25 [C]) "entering water temperature"
P_atm=101.3 [kPa]*convert(kPa,Pa) "atmopheric pressure"
u_inf=3 [m/s] "water velocity"
$ifnot parametricTable
W_dot=100 [W] "power to the heater"
$endif

The plate temperature, Tplate, is guessed and used with the procedure External_Flow_Plate to
compute the heat transfer coefficient between the water and the copper plate ( h ).

T_plate=300 [K] "plate temperature guess"


Call External_Flow_Plate('water', T_water, T_plate, P_atm, u_inf, L: tau, h, C_f, Nusselt, Re)
"heat transfer coefficient to water"
At steady state, the heater power is transferred as heat to the refrigerant and removed by the
water:

q = w (1)

The guess values are updated and the guess for Tplate is commented out and the plate temperature
is computed according to:

q = h (Tplate − Twater ) (2)

q_dot=w_dot "rate of heat transfer is power to heater"


q_dot=h*A*(T_plate-T_water) "rate of heat transfer to water"

The saturation temperature (Tsat) is guessed and used with the procedure Cond_horizontal_down to
compute the condensate heat transfer coefficient ( hm ).

T_sat=302 [K] "guess for saturation temperature"


Call Cond_horizontal_down(F$, T_plate, T_sat: h_m, Nusselt_m)"condensation coefficient"

The guess values are updated and the guess for Tsat is commented out and the saturation
temperature is computed according to:

q = h (Tsat − Tplate ) (3)

The saturation pressure is computed using the internal property function in EES.

Call Cond_horizontal_down(F$, T_plate, T_sat: h_m, Nusselt_m) "condensation coefficient"


q_dot=A*h_m*(T_sat-T_plate) "rate of heat transfer by condensation"
P_sat=pressure(F$,T=T_sat,x=0) "saturation pressure"

An energy balance determines the rate at which R134a is vaporized and the rate of heat transfer
that must be removed by the cooling water.

( )
m ir ,T =Tsat , x =1 − ir ,T =Tsat , x =0 = w (4)

where ir is the specific enthalpy of the refrigerant.

m_dot*(enthalpy(F$,T=T_sat,x=1)-enthalpy(F$,T=T_sat,x=0))=W_dot "energy balance"

Construct a Parametric table with columns for W_dot, T_sat and m_dot. Solve and plot the
saturation temperature and condensation rate vs the heater power, as shown in Figure 2.
315 0.007

Rate of refrigerant recovery (kg/s)


0.006
Tsat

Saturation temperature (K)


310
m 0.005

0.004
305
0.003

0.002
300

0.001

295 0
100 200 300 400 500 600 700 800 900 1000
Heater power (W)
Figure 2: Saturation temperature and mass flow rate of refrigerant recovered as a function of heater power.
Problem 7.4-5 (7-14 in text): Condensing Steam at 6 kPa.
Calculate the heat flux for a square plate 1 m on each side condensing steam at 6 kPa. Consider
three plate orientations: (1) horizontal facing downward; (2) horizontal facing upward; and (3)
vertical (one-side only is active).
a.) Plot the heat flux for each orientation as a function of plate surface temperature.

The inputs are entered in EES; initially a plate surface temperature is assigned, this value will be
adjusted to generate the plot requested in the problem statement.

$UnitSystem SI MASS RAD PA K J


$Tabstops 0.2 0.4 0.6 3.5 in

Fluid$='steam' "fluid"
P_sat=6 [kPa]*convert(kPa,Pa) "saturation pressure"
T_sat=T_sat(Fluid$,P=P_sat) "saturation temperature"
q``=15000 [W/m^2] "heat flux"
L=1 [m] "
W=1 [m] "width of surface"
A=L*W "area of surface"
T_w=T_sat-10 [K] "plate surface temperature"

This problem is made easy by the availability of the condensation library routines in EES. Call
the Cond_horizontal_down procedure to obtain the heat transfer coefficient on a horizontal plate
facing down ( hhoriz , plate ). The heat transfer rate is given by Newton’s law of cooling.

qhorz = A hhoriz , plate (Tsat − Tw ) (1)

"a) Horizontal downwards"


Call Cond_horizontal_down(Fluid$, T_w, T_sat: h_horiz_plate, Nusselt_horiz_plate)
q_dot_horz=A*h_horiz_plate*(T_sat-T_w)

Call the Cond_horizontal_up procedure to obtain the heat transfer coefficient on a horizontal plate
facing upwards ( hhoriz , plateup ). The heat transfer rate is given by Newton’s law of cooling.

qhorzup = A hhoriz , plateup (Tsat − Tw ) (2)

"b) Horizontal upwards"


Call Cond_horizontal_up(Fluid$, L, T_w, T_sat: h_horiz_plateup, Nusselt_horiz_plateup)
q_dot_horzup=A*h_horiz_plateup*(T_sat-T_w)

Call the Cond_vertical_plate procedure to obtain the heat transfer rate on a vertical plate.

"c) Vertical"
Call Cond_vertical_plate(Fluid$, L, W, T_w, T_sat :h_vert_plate, Re_L, q_dot_vert, m_dot)

Figure 1 illustrates heat transfer rate for these three different cases as a function of the wall
temperature.
70000

60000

Heat transfer rate (W)


50000 horizontal, down

40000

30000
vertical
20000

10000
horizontal, up
0
300 301 302 303 304 305 306 307 308 309
Wall temperature (K)
Figure 1: Rate of heat transfer as a function of wall temperature for the three configurations.

b.) Which geometry provides the highest rate of condensation per unit surface area?

The horizontal, downward facing plate provides the highest rate of condensation per area while
the horizontal, upward facing plate provides the lowest. The performance generally follows
intuition; the best performance occurs in the configuration where gravity most aids the drainage
of the condensate.

c.) How do the answers to (a) and (b) change if the plate dimensions are reduced to 0.5 m per
side?

Change L and W to 0.5 m and replot, as shown in Figure 2.

20000

17500

15000
Heat transfer rate (W)

12500 horizontal, down

10000

7500 vertical

5000

2500
horizontal, up
0
300 301 302 303 304 305 306 307 308 309
Wall temperature (K)
Figure 2: Rate of heat transfer as a function of wall temperature with L = W = 0.5 m.
Problem 7.4-6 (7-15 in text): Heat Pipe
A heat pipe has been instrumented to test its ability to transfer thermal energy. The heat pipe
consists of a sealed vertical thin-walled copper tube that is 1.5 m in length and 2.5 cm in
diameter. The heat pipe contains liquid and vapor toluene. The bottom 5 cm of the tube are
wrapped with heater tape that provides 100 W of heat input to the toluene. The toluene
evaporates at the lower end of the tube and the vapor rises to the top where it is condensed by
contact with the cold top surface of the tube. The top 6 cm of the heat pipe are maintained at
29°C by a flow of liquid water at 25°C and 1 atm. Toluene condensate flows back to the bottom
of the tube; the flow is assisted by surface tension due to the presence of a wicking material on
the inner surface of the copper tube. The heat pipe tube is well-insulated except for the bottom
part that is in contact with the heater and the top part that is in contact with the water.

The known information can be input into EES.

$UnitSystem SI MASS RAD PA K J


$Tabstops 0.2 0.4 0.6 3.5 in

L=1.5 [m] "length of the heat pipe"


D=2.5 [cm]*convert(cm,m) "outside diameter of the heat pipe"
D_c=1.25 [cm]*convert(cm,m) "diameter of center core through which vapor flows up"
F$='Toluene'
L_h=5 [cm]*convert(cm,m) "length of heated section"
A_h=pi*D*L_h "surface area of heated section"
P_atm=101.3 [kPa]*convert(kPa,Pa) "atmospheric pressure"
L_c=6 [cm]*convert(cm,m) "length of cooled section"
A_c=pi*D*L_c "area of cooled section"
T_inf=convertTemp(C,K,25 [C]) "water temperature"
q_dot=100 [W] "rate of heat provided by heater"
T_ts=convertTemp(C,K,29 [C]) "temperature of top surface of tube"

a.) Estimate the saturation temperature and pressure of the toluene in the heat pipe.
b) Estimate the surface temperature of the tube that is in contact with the heater.

Parts (a) and (b) are coupled and best solved together. The Nucleate_Boiling function in EES
implements Eq. (7-3) in the text to characterize the nucleate boiling process. Neither the wall
temperature or saturation temperature are known. Therefore, the wall temperature, Tbs, is
assumed and the function is used to compute the saturation temperature. The saturation pressure
is determined from the temperature.

"hot side of heat pipe"


T_bs=convertTemp(C,K,110 [C]) "guess for the inside surface of the heat pipe - remove later"
C_s_f=0.013 [-] "boiling surface coefficient"
q_dot=Nucleate_Boiling(F$, T_sat, T_bs, C_s_f)*A_h
"heat transfer rate due to nucleate boiling - fixes T_bs"
P_sat=P_sat(F$,T=T_sat) "saturation pressure of heat pipe fluid"

Film condensation occurs at the cold side of the heat pipe. The heat transfer coefficient for this
situation ( hc ) is provided by the Cond_HorizontalTube with the mass flowrate set to 0. Quality is
an input to this procedure, but when there is no flow, its value does not matter.
"cold side of heat pipe"
m_dot=0 "film condensation"
x=0.5 "values is irrelevant for film condensation"
Call Cond_HorizontalTube(F$, m_dot, x, T_sat, T_ts, D : h_c, Regime$)

The guess values are updated and the guess for Tbs is commented out. The heat transfer to the
condensing section is constrained to be the same as the heat input from the heater:

q = Ac hc (Tsat − Tts ) (1)

{T_bs=convertTemp(C,K,110 [C])} "guess for the hot surface of the heat pipe - remove later"
q_dot=A_c*h_c*(T_sat-T_ts) "fixes the surface temperature of the cold side of the heat pipe"

which leads to Tsat = 317.8 K, psat = 9776 Pa, and Tbs = 358.1 K.

c) Estimate the velocity of the cooling water provided at 25°C needed to maintain the top
surface of the heat pipe at 29°C.

The convection coefficient on the outside of the cylinder is computed according to:

q = Ac hconv (Tts − T∞ ) (2)

"calculate heat transfer coefficient outside of cooled surface"


q_dot=h_conv*A_c*(T_ts-T_inf) "definition of convective heat transfer coefficient"

which leads to hconv = 5305 W/m2-K. The required velocity of water (u∞) is obtained from the
External_Flow_Cylinder procedure.

Call External_Flow_Cylinder('water', T_inf, T_ts, P_atm, u_inf, D: F_d\L, h_conv, C_d, Nusselt, Re)
"h_conv is heat transfer coefficient on outside surface of tube to determine u_inf"

which leads to u∞ = 1.01 m/s.

d) Compare the heat transfer rate provided by the heat pipe to the heat transfer rate that would
occur if the tube were replaced with 2.5 cm diameter solid copper rod with the same
temperatures imposed at the hot and cold ends.

Fourier's law is used to obtain the heat transfer through a piece of copper:

D 2 kcu
qcu = π (Tbs − Tts ) (3)
4 L

q_dot_cu=pi*D^2/4*k_cu/L*(T_bs-T_ts) "heat transfer rate of copper rod"


k_cu=k_('Copper',T=(T_bs+T_ts)/2) "thermal conductivity of copper"
which leads to qcu = 7.3 W (much less than the 100 W that can be transferred by the heat pipe
operating under the same conditions).

e) What is the effective thermal conductivity of the heat pipe? What do you see as advantages
of the heat pipe?

The effective conductivity of a material that transfers energy at the same rate as the heat pipe is
defined according to:

D 2 keff
q = π (Tbs − Tts ) (4)
4 L

q_dot=pi*D^2/4*k_eff/L*(T_bs-T_ts) "effective thermal conductivity of the heat pipe"

which leads to keff = 5465 W/m-K. This conductivity is more that an order of magnitude greater
than that for the copper. Heat pipes can be very efficient heat conductors.
Problem 7.4-7 (7-16 in text): Condensation inside a cylinder
A vertical cylindrical container is made of aluminum having a wall thickness of 2.5 mm. The
cylinder is 0.24 m in height and it has an outer diameter of 7.5 cm. Liquid water is placed in the
cylinder and the bottom is heated, evaporating the liquid. The vapor that is produced escapes
through a vent at the top of the cylinder. The flow of vapor drives out air that was originally in
the cylinder. When all of the liquid has been boiled, the heating is stopped and the vent at the
top of the cylinder is closed. The aluminum surfaces are nearly at a uniform temperature of
100°C. The cylinder is allowed to stand in a large room and it transfers energy by free
convection to the 25°C air.
a.) Calculate and plot the pressure inside the cylinder as a function of time for a 5 minute period
after the vent is closed. State and justify any assumptions that you employ. (Note that the
heat transfer coefficient for film condensation on the inside surfaces of the cylinder can be
estimated using the Cond_HorizontalTube procedure with a mass flow rate of zero.)

The inputs are entered in EES:

L=0.24 [m] "length of the cylinder"


D_o=7.5 [cm]*convert(cm,m) "outside diameter of the cylinder"
delta=2.5 [mm]*convert(mm,m) "thickness of the cylinder walls"
D_i=D_o-2*delta "insider diameter of the cylinder"
Fluid$='water'
T_amb=convertTemp(C,K,25 [C]) "temperature of air in the room"
T_boil=convertTemp(C,K,100 [C]) "temperature of the boiling water"
P_atm=101.3 [kPa]*convert(kPa,Pa) "atmospheric air pressure"

The specific heat capacity and density of aluminum are evaluated at the average of the ambient
and initial temperatures (ρal and cal, respectively). The volume of aluminum is:

π D2
Val = π Di L δ + 2 δ (1)
4

and the mass of aluminum is:

Cal = Val ρ al cal (2)

T_avg=(T_amb+T_boil)/2 "average temperature of the aluminum"


rho_al=rho_('Aluminum',T=T_avg) "density of aluminum"
c_al=c_('Aluminum',T=T_avg) "specific heat of aluminum"
V_al=pi*D_i*L*delta+2*pi*D_o^2/4*delta "volume of aluminum"
Cap_al=V_al*rho_al*c_al "specific heat capacity of aluminum"

The mass of water that is contained in the cylinder when it is closed is:

D2 L
Mw = π (3)
4 vw
where vw is the specific volume of the water in the cylinder at the time that the container is
closed (i.e., the specific volume at T = Tboil and x = 1).

v_w=volume(Water,T=T_boil,x=1) "specific volume of water"


m_w=pi*D_i^2/4*L/v_w "mass of water vapor in cylinder when valve is closed"

The state equation for this problem will provide the time rate of change of the temperature of the
water and the cylinder. The time and temperature are initially set and the state equations
developed; these values will be commented out and the state equation integrated using the
Integral command in EES.

"arbitrary value of time and T"


time=1 [s]
T=converttemp(C,K,90 [C])

The pressure in the can follows from the temperature because the water is saturated:

p=pressure(Fluid$,T=T,x=1) "pressure in cylinder"

The surface temperatue is not known so it is assumed in order to evaluate the thermal resistance
between the water and the air.

T_s=T-0.1 [K] "guess for surface temperature"

The thermal resistance is composed of film condensation on the inside surface and natural
convection on the outer surface. The functions FC_vertical_cylinder and FC_plate_horizontal are
used to evaluate the heat transfer coefficient associated with natural convection from the sides
and top, respectively, of the cylinder ( hsides and htop , convection from the bottom is neglected).
The resistance to natural convection is:

−1
⎛ π Do2 htop ⎞
Rconv , nc =⎜ + π Do L hsides ⎟ (4)
⎜ 4 ⎟
⎝ ⎠

Call FC_vertical_cylinder('Air', T_s, T_amb, P_atm, L, D_o : h_sides, Nusselt_sides, Ra_sides)


"estimate convection coefficient on sides of cylinder"
Call FC_plate_horizontal1('Air', T_s, T_amb, P_atm, D_o: h_top, Nusselt_top, Ra_top)
"estimate convection coefficient on top of cylinder"
R_conv_nc=(pi*D_o^2*h_top/4+pi*D_o*L*h_sides)^(-1) "resistance to natural convection"

which leads to Rconv,nc = 2.89 K/W. The film condensation heat transfer coefficient ( h film ) is
obtained from the Cond_HorizontalTube procedure with the mass flow rate set to 0 (note that the
value of quality is arbitrary). The resistance to film condensation is:

1
Rc = (5)
π Di L h film
Call Cond_HorizontalTube(Fluid$, 0 [kg/s], 0.5 [1], T, T_s, D_i : h_film, F$)
"estimate film heat transfer coefficient"
R_c=1/(pi*D_i*L*h_film) "resistance to film condensation"

which leads to Rc = 0.00084 K/W. Because the resistance to condensation is so much less than
the resistance to natural convection, it can be neglected. In this limit, the temperature of the
surface of the cylinder is equal to the temperature of the water:

T_s=T "guess for surface temperature"


{Call Cond_HorizontalTube(Fluid$, 0 [kg/s], 0.5 [1], T, T_s, D_i : h_film, F$)
"estimate film heat transfer coefficient"
R_c=1/(pi*D_i*L*h_film) "resistance to film condensation"}

The rate of heat transfer is given by:

q =
(T − Tamb ) (6)
Rconv , nc

q_dot=(T-T_amb)/R_conv_nc "heat transfer rate"

An energy balance on the water and the cylinder leads to:

⎡ ⎛ ∂u ⎞ ⎤ dT
⎢ mw ⎜ ⎟ + Cal ⎥ + q = 0 (7)
⎣ ⎝ ∂T ⎠v ⎦ dt

⎛ ∂u ⎞
where ⎜ ⎟ is the partial derivative of the water internal energy with respect to temperature at
⎝ ∂T ⎠v
constant specific volume, evaluated numerically according to:

⎛ ∂u ⎞
=
(
uw,T =T +δ T ,v =vw − uw,T =T −δ T ,v =vw )
⎜ ⎟ (8)
⎝ ∂T ⎠v 2δT

where δT is a small change in the temperature.

dT=0.01 [K]
u_p=intenergy(Fluid$,T=T+dT,v=v_w) "internal energy at T+dT"
u_m=intenergy(Fluid$,T=T-dT,v=v_w) "internal energy at T-dT"
dudT=(u_p-u_m)/(2*dT) "partial derivative of internal energy w/respect to T at constant v"
0=(m_w*dudT+Cap_al)*dTdt+q_dot "time rate of temperature change"

The arbitrary values of the state variables are commented out and the integral command is used

{"arbitrary value of time and T"


time=1 [s]
T=converttemp(C,K,100 [C])}
t_sim=5000 [s] "simulation time"
T_ini=T_boil-0.1 [K] "initial temperature"
T=T_ini+Integral(dTdt,time,0,t_sim) "integrate state equations"
T_C=converttemp(K,C,T) "in C"

$integralTable time:1 T_C, p

Figure 1 illustrates the temperature and pressure as a function of time.


100
100000
90

80 80000
Temperature (°C)

Pressure (Pa)
70
60000
60
T
50 40000

40 p
20000
30

20 0
0 1000 2000 3000 4000 5000
Time (s)
Figure 1: Temperature and pressure as a function of time.
Problem 7.5-1 (7-17 in text): Condenser Tube in a Lake
You have fabricated an inexpensive condenser for your air conditioner by running the refrigerant
through a plastic tube that you have submerged in a lake. The outer diameter of the tube is Do =
7.0 mm and the inner diameter is Di = 5.0 mm. The tube conductivity is ktube = 1.4 W/m-K. The
refrigerant is R134a and enters the tube with quality x = 0.97, temperature Tsat = 35°C and mass
flow rate m r = 0.01 kg/s. The water in the lake has temperature T∞ = 10°C.
a.) Determine the heat transfer rate per unit length from the refrigerant to the lake at the tube
inlet.

The inputs are entered in EES:

$UnitSystem SI MASS RAD PA K J


$Tabstops 0.2 0.4 0.6 3.5 in

"Inputs"
R$='R134a' "refrigerant"
m_dot_r=0.01 [kg/s] "mass flow rate of refrigerant"
x_in=0.97 [-] "inlet quality"
D_i=5.0 [mm]*convert(mm,m) "inner diameter"
D_o=7.0 [mm]*convert(mm,m) "outer diameter"
T_w=converttemp(C,K,10) "temperature of water"
T_sat_C=35 [C] "saturation temperature"
T_sat=converttemp(C,K,T_sat_C) "in K"
p_w=1 [atm]*convert(atm,Pa) "pressure of water"
k_tube=1.4 [W/m-K] "thermal conductivity of tube"

The heat transfer from the R134a to the water in the lake is resisted by convection from the inner
surface of the tube to the refrigerant, conduction through the tube, and convection from the
external surface of the tube to the water. The resistance per unit length associated with
convection to the refrigerant is:

1
Rr′ = (1)
hr π Di

where hr is the heat transfer coefficient associated with the flow condensation of the refrigerant.
The flow condensation heat transfer coefficient can be obtained using the correlation discussed
in Section 7.5.2 and implemented by the Cond_HorizontalTube function. The correlation and
therefore the function requires the surface temperature of the inside of the tube, Ts,r, which is not
given. Therefore, a reasonable value is assumed; this value will be adjusted based on the
solution.

x=x_in "quality"
T_s_r=T_w+(T_sat-T_w)/2 "guess for surface temperature on refrigerant side"

The Cond_HorizontalTube function is accessed by selecting Function Info from the Options menu
and then selecting Boiling and Condensation from the drop down menu. Select Condensation
and then scroll to the appropriate correlation (Figure 1). The function returns the local heat
transfer coefficient, hr , as well as a string that identifies the flow regime. The local heat transfer
coefficient is used to evaluate the thermal resistance per unit length on the refrigerant side
according to Eq. (1).

Call Cond_HorizontalTube(R$, m_dot_r, x, T_sat, T_s_r, D_i : h_r, F$)


"call flow condensation correlation"
R`_r=1/(pi*D_i*h_r) "resistance per unit length on the refrigerant side"

Figure 1: Function Information Window for the Cond_HorizontalTube function

The resistance per unit length associated with conduction through the tube is:

⎛D ⎞
ln ⎜ o ⎟
D

Rtube = ⎝ i⎠ (2)
2 π ktube

R`_tube=ln(D_o/D_i)/(2*pi*k_tube) "resistance per unit length of the tube"

The resistance per unit length associated with convection to the water in the lake is:

1
Rw′ = (3)
hw π Do

where hw is the heat transfer coefficient associated with the natural convection to the water in the
lake. The calculation of the natural convection heat transfer coefficient requires the surface
temperature of the outside of the tube, Ts,w, in order to determine the film temperature and
Raleigh number. The value of Ts,w is not known; therefore, a reasonable value is assumed and
this value will be adjusted based on the solution.

T_s_w=T_w+(T_sat-T_w)/2 "guess for surface temperature on water side"


The film temperature is calculated:

Ts , w + T∞
T film , w = (4)
2

and used to compute the properties of water (ρw, μw, kw, cw, βw, υw, αw, and Prw).

T_film_w=(T_s_w+T_infinity)/2 "film temperature"


mu_w=viscosity(Water,T=T_film_w,p=p_w) "viscosity"
k_w=conductivity(Water,T=T_film_w,p=p_w) "conductivity"
rho_w=density(Water,T=T_film_w,p=p_w) "density"
c_w=cP(Water,T=T_film_w,p=p_w) "specific heat capacity"
beta_w=VolExpCoef(Water,T=T_film_w,p=p_w) "volumetric expansion coefficient"
nu_w=mu_w/rho_w "kinematric viscosity"
alpha_w=k_w/(rho_w*c_w) "thermal diffusivity"
Pr_w=nu_w/alpha_w "Prandtl number"

The Rayleigh number is computed:

g Do3 β w (Ts , w − T∞ )
Ra = (5)
υw α w

Ra=g#*D_o^3*beta_w*(T_s_w-T_infinity)/(nu_w*alpha_w) "Rayleigh number"

and the function FC_horizontal_cylinder_ND is used to access the correlations for natural
convection from a horizontal cylinder in order to obtain the average Nusselt number that
characterizes the heat transfer from the tube surface to the water, Nu w . The heat transfer
coefficient hw is obtained from:

Nu w kw
hw = (6)
Do

The resistance per unit length between the outer surface of the tube and the water is computed
according to Eq. (3).

Call FC_horizontal_cylinder_ND(Ra_w, Pr_w: Nusselt_w) "call natural convection correlation"


h_w=Nusselt_w*k_w/D_o "air side heat transfer coefficient"
R`_w=1/(pi*D_o*h_w) "resistance per unit length on the water side"

The heat transfer per unit length is given by:

q ′ =
(Tsat − T∞ ) (7)
( Rr′ + Rtube
′ + Rw′ )

q`_dot=(T_sat-T_infinity)/(R`_r+R`_tube+R`_w) "heat transfer per unit length"


The surface temperatures that were assumed, Ts,r and Ts,w, should be adjusted based on this
solution. Update Guesses is selected from the Calculate menu and the initial guesses and
commented out. In their place, the surface temperatures are calculated according to:

Ts ,r = Tsat − q ′ Rr′ (8)

Ts , w = T∞ + q ′ Rw′ (9)

{T_s_r=T_w+(T_sat-T_w)/2} "guess for surface temperature on refrigerant side"


{T_s_w=T_w+(T_sat-T_w)/2} "guess for surface temperature on water side"
T_s_r=T_sat-q`_dot*R`_r "surface temperature on refrigerant side"
T_s_w=T_infinity+q`_dot*R`_w "surface temperature on water side"

which leads to q ′ = 224.3 W/m.

b.) If the length of the tube is L = 6 m, then what is the quality of the refrigerant leaving the
tube? Plot the quality and refrigerant heat transfer coefficient as a function of position s in
the tube.

An energy balance on a differential length of the tube (ds) is shown in Figure 2.

Figure 2: Differential energy balance on the tube

The differential energy balance leads to:

⎡ m r ( il , sat + x Δivap ) ⎤ = q ′ ds + ⎡ m r ( il , sat + x Δivap ) ⎤ (3)


⎣ ⎦s ⎣ ⎦ s + ds

where il,sat is the enthalpy of the saturated liquid and Δivap is the latent heat of vaporization,
which is computed using EES' internal property routines:

DELTAi_vap=enthalpy(R$,x=1,T=T_sat)-enthalpy(R$,x=0,T=T_sat) "enthalpy of vaporization"

The s + ds term in Eq. (3) is expanded:

dx
⎣ mr ( il , sat + x Δivap ) ⎦ s = q ds + ⎣ mr ( il , sat + x Δivap ) ⎦ s + mr Δivap ds ds
⎡ ⎤ ′ ⎡ ⎤ 
which can be simplified to provide the rate of change of quality with position:

dx q ′
=−
ds m r Δivap

dxds=-q`_dot/(m_dot_r*DELTAi_vap) "rate of change of quality with position"

The rate of change of quality with respect to position is integrated numerically from the inlet of
the tube (s = 0 where xin = 0.97) to the exit of the tube (s = L) using EES’ INTEGRAL command.
The results, including quality and heat transfer coefficient, are contained in an integral table.

{x=x_in} "quality"
L=6 [m] "length of tube"
x=x_in+INTEGRAL(dxds,s,0,L) "numerical integration"
$IntegralTable s:0.2, x, h_r

The quality and refrigerant side heat transfer coefficient are illustrated as a function of position
in the tube in Figure 3.

Refrigerant heat transfer coefficient (W/m -K)


1 10000

2
0.9 9000

0.8 8000

0.7 7000

0.6 6000
Quality (-)

x hr
0.5 5000

0.4 4000

0.3 3000

0.2 2000

0.1 1000

0 0
0 0.5 1 1.5 2 2.5 3 3.5 4 4.5 5 5.5 6
Position (m)
Figure 3: Quality and refrigerant heat transfer coefficient as a function of position in the tube.
Problem 7.5-2 (7-18 in text): Analogy between condensation and boiling
Condensation and boiling are analogous processes in that both a involve phase change. Heat
exchangers that provide condensation and boiling are often designed in a similar manner. In a
particular case, a phase change of R134a takes place within horizontal tubes having an inner
diameter of 1 cm. The mass velocity is 300 kg/s-m2.
a.) Prepare a plot of the heat transfer coefficient for condensation and boiling as a function of
quality at a saturation temperature of 10°C for heat fluxes of 5,000 and 10,000 W/m2.

The input information is entered:

$UnitSystem SI MASS RAD PA K J


$Tabstops 0.2 0.4 0.6 3.5 in

Fluid$='R134a'
D=1 [cm]*convert(cm,m) "tube inner diameter"
T_sat=convertTemp(C,K,10 [C]) "saturation temperature"
G=300 [kg/m^2-s] "mass velocity"
m_dot=G*A "mass flow rate"
A=pi*D^2/4 "cross-sectional area of the tube"
$ifnot ParametricTable
x=0.5 [-] "quality"
q``=5000 [W/m^2] "heat flux"
$endif

The Flow_Boiling and Cond_HorizontalTube procedures return the boiling and condensation heat
transfer coefficients, respectively. A Parametric table is constructed with columns for quality
(x), the heat flux ( q ′′ ), the heat transfer coefficients and the excess temperatures.

"boiling"
call Flow_Boiling(Fluid$, T_sat, G, D, x, q``, 'Horizontal': h_b, T_w_b)
DELTAT_b=T_w_b-T_sat "excess temperature for boiling"

"condensation"
Call Cond_HorizontalTube(Fluid$, m_dot, x, T_sat, T_w, D : h_c, F$)
DELTAT_c=T_sat-T_w "excess temperature for condensation"
q``=h_c*(T_sat-T_w) "relation between heat flux and wall temperature"

The heat transfer coefficient for boiling and condensation are shown in Figures 1 and 2 as a
function of x for q ′′ = 5,000 W/m2 and q ′′ = 10,000 W/m2.
Figure 1: Heat transfer coefficient for condensation and boiling as a function of quality with 5,000 W/m2 heat
flux.

Figure 2: Heat transfer coefficient for condensation and boiling as a function of quality with 10,000 W/m2
heat flux.

b.) Plot the excess temperature for condensation and boiling as a function of quality.

The excess temperature for condensation and boiling are shown in Figure 3 as a function of
quality for a heat flux of 5,000 W/m2.
Figure 3: Excess temperature for condensation and boiling as a function of quality with 5,000 W/m2 heat flux.

c.) What conclusion can you draw from the results?

Boiling and condensation both exhibit high heat transfer coefficients, but at high quality, the
boiling experiences ‘dry-out’ and the heat transfer coefficient reduces to the value for saturated
vapor. Condensation does not experience this limitation.
Problem 7.5-3 (7-19 in text): Absorption Refrigeration Cycle using Ammonia-Water
Absorption refrigeration cycles are often used to operate small refrigerators in hotel rooms
because they do not require compressors or fans and thus operate quietly. In a particular case, an
absorption refrigeration system uses an ammonia-water mixture. The ammonia is separated from
the water and passes through a condenser where it is isobarically changed from saturated vapor
at 76°C to subcooled liquid at 40°C. The condenser consists of a single unfinned thin-walled
copper tube with a 1 cm inner diameter. The thermal energy released from the ammonia in this
process is transferred to the 25°C room air by free convection. The subcooled ammonia is
throttled to a saturation temperature of -5° and evaporated to saturated vapor to produce a
refrigeration capacity of 110 W.
a) Determine the mass flow rate of ammonia through the evaporator and condenser.

The small amount of water remaining in the ammonia is neglected. The inputs information is:

$UnitSystem SI MASS RAD PA K J


$Tabstops 0.2 0.4 0.6 3.5 in

"known"
T_sat=converttemp(C,K,76[C]) "saturation temperature of the ammonia"
D=1 [cm]*convert(cm,m) "diameter of the condenser pipe"
q_dot_evap=110 [W] "refrigeration effect"
T_air=converttemp(C,K,25 [C]) "air temperature"
T_evap_in=converttemp(C,K,40 [C]) "temperature of refrigerant before throttling"
T_sat_evap=converttemp(C,K,-5 [C]) "evaporation temperature"
P_air=101.3 [kPa]*convert(kPa,Pa) "atmospheric pressure"

The pressure in the condenser (pcond) is obtained from the inlet temperature. An energy balance
on the evaporator leads to:

qevap = m ( ievap ,out − ievap ,in ) (1)

where ievap,out is the specific enthalpy of the ammonia leaving the evaporator at the evaporation
saturation temperature and a quality of one and ievap,in is the enthalpy of the ammonia entering the
evaporator, which is the same as the enthalpy of the subcooled ammonia entering the throttling
valve.

"condensation section"
P_cond=Pressure(ammonia,T=T_sat,x=1 [-]) "pressure of the ammonia in the condenser"
i_evap_in=enthalpy(Ammonia,T=T_evap_in,P=P_cond)
"specific enthalpy of ammonia entering the evaporator"
i_evap_out=enthalpy(Ammonia,T=T_sat_evap,x=1) "specific enthalpy of ammonia exiting the evaporator"
q_dot_evap=m_dot*(i_evap_out-i_evap_in) "energy balance determines ammonia flow rate"

which leads to m = 0.00010 kg/s.

b) Estimate the length of piping required to condense the ammonia from saturated vapor to
saturated liquid at 76°C.
The Cond_HorizontalTube_avg procedure is used to evaluate the average heat transfer coefficient
due to convection within the tube ( hcond , sat ). The surface temperature of the tube in the
condensing region (Tw,sat) is assumed for this purpose.

T_w_sat=converttemp(C,K,30[C]) "guess for the wall temperature in the condensing section"


Call Cond_HorizontalTube_avg('ammonia', m_dot, T_sat, T_w_sat, D, 1 [-], 0 [-] : h_bar_cond_sat)
"determines condensing heat transfer coefficient"

The FC_horizontal_cylinder procedure is used to evaluate the average heat transfer coefficient due
to natural convection from the outer surface of the tube ( hnc , sat ).

Call FC_horizontal_cylinder('air', T_w_sat, T_air, P_air , D : h_bar_nc_sat, Nusselt_cond, Ra_cond)


"determines h_bar_nc"

The rate of heat transfer in the condensing section of the condenser is calculated from an energy
balance on the ammonia:

(
qcond , sat = m iNH 3, x =1,T =Tsat − iNH 3, x =0,T =Tsat ) (2)

q_dot_cond_sat=m_dot*(enthalpy(ammonia,T=T_sat,x=1)-enthalpy(ammonia,T=T_sat,x=0))
"rate of energy transfer during condensation"

The air and ammonia temperatures in the condensing region are both constant and therefore the
heat transfer rate can be written according to:

qcond , sat =
(Tsat − Tair ) (3)
⎛ 1 1 ⎞
⎜⎜ + ⎟⎟
⎝ hcond , sat π Lsat D hnc , sat π Lsat D ⎠

q_dot_cond_sat=(T_sat-T_air)/(1/(h_bar_cond_sat*pi*D*L_sat)+1/(h_bar_nc_sat*pi*D*L_sat))
"length required in saturated section"

which leads to the predicted value of the length of tube required to condense the ammonia. The
program is solved and the guess values are updated. The assumed value of Tw,sat is commented
out and recalculated according to:

qcond , sat
Tw, sat = Tsat − (4)
hcond , sat π D Lsat

{T_w_sat=converttemp(C,K,30[C])} "guess for the wall temperature in the condensing section"


T_w_sat=T_sat-q_dot_cond_sat/(h_bar_cond_sat*pi*D*L_sat) "recalculate wall temperature"

which leads to Lsat = 5.59 m.

c) Estimate the additional length of piping required to subcool the ammonia to 40°C.
The heat transfer rate required in the sub-cooling section is:

(
qcond , sc = m iNH 3, x =0,T =Tsat − ievap ,in ) (5)

"subcooling section"
q_dot_cond_sc=m_dot*(enthalpy(ammonia,T=T_sat,x=0)-i_evap_in)
"rate of energy transfer required to accomplish subcooling"

The wall temperature (Tw,sc) and length of the subcooling section (Lsc) must be assumed in order
to evaluate the heat transfer coefficients.

T_w_sc=T_w_sat "guess for wall temperature in subcooling section"


L_sc=1 [m] "guess for length of tube needed for condensation"

The procedure FC_horizontal_cylinder is used to evaluate the natural convection heat transfer
coefficient ( hnc , sc ).

Call FC_horizontal_cylinder('air', T_w_sc, T_air, P_air , D : h_bar_nc_sc, Nusselt_sc, Ra_sc)


"determines h_bar_nc_sc - for subcooling section"

The procedure PipeFlow is used to evaluate the single-phase forced convection heat transfer
coefficient ( hi , sc ) using the average temperature of the ammonia in the subcooled section.

RelRough=1e-6 [-] "assume smooth tube"


T_avg=(T_sat+T_evap_in)/2 "average temperature during subcooling"
call PipeFlow('Ammonia',T_avg,P_cond,m_dot,D,L_sc,RelRough:h_bar_i_sc, h_H ,DELTAP, &
Nusselt_T, f, Re)

The total conductance of the subcooled section is calculated:


−1
⎛ 1 1 ⎞
UAsc = ⎜ + ⎟⎟ (6)
⎜π DL h π D Lsc hi , sc
⎝ sc nc , sc ⎠

UA_sc=(1/(pi*D*L_sc*h_bar_nc_sc)+1/(pi*D*L_sc*h_bar_i_sc))^(-1) "conductance"

The problem is solved and the guess values updated. The assumed value of Lsc is commented out
and the solution for internal flow exposed to a constant external temperature from Section 5.3.4
is used:

⎛ UA ⎞
Tevap ,in = Tair − (Tair − Tsat ) exp ⎜ − sc ⎟ (7)
⎝ m csc ⎠

where csc is the specific heat capacity of the ammonia in the subcooled section, evaluated at the
average temperature.
{L_sc=1 [m]} "guess for length of tube needed for condensation"
c_sc=cP(ammonia,T=(T_sat+T_evap_in)/2,p=p_cond) "specific heat capacity of subcooled liquid"
T_evap_in=T_air-(T_air-T_sat)*exp(-UA_sc/(m_dot*c_sc)) "recalculate evaporator inlet temperature"

The problem is solved and the guess values updated. The assumed value of Tw,sc is commented
out and recalculated according to:

qcond , sc
Tw, sc = Tair − (8)
hnc , sc π D Lsc

{T_w_sc=T_w_sat} "guess for wall temperature in subcooling section"


T_w_sc=T_air-q_dot_cond_sc/(h_bar_nc_sc*pi*D*L_sc) "recalculate wall temperature"

which leads to Lsc = 2.37 m.


Problem 8.2-3
You are specifying a plate frame heat exchanger with a counter-flow configuration to transfer
heat from hot water to cooling water. The hot water enters the heat exchanger at TH,in = 120ºF
with a flow rate of VH = 0.85 gal/min. The cooling water enters at TC,in = 35ºF with flow rate
V = 0.65 gal/min. If the hot water must be cooled to TH,out = 75ºF then what is the required
C
conductance of the heat exchanger?

The inputs are entered in EES:

$UnitSystem English psi F


$TabStops 0.5 3.5

"Inputs"
V_dot_H=0.85 [gal/min]*convert(gal/min,ft^3/hr) "volumetric flow rate of hot water"
V_dot_C=0.65 [gal/min]*convert(gal/min,ft^3/hr) "volumetric flow rate of cooling water"
T_H_in=120 [F] "hot inlet temperature"
T_H_out=75 [F] "hot outlet temperature"
T_C_in=35 [F] "cooling water inlet temperature"

The capacitance rates are computed according to:

C H = ρ H cH VH (1)

C C = ρC cC VC (2)

where ρ and c are the density and specific heat capacity of the streams.

"capacitance rates"
rho_H=density(Water,T=T_H_in,P=14.7 [psi]) "density of hot water"
c_H=cP(Water,T=T_H_in,P=14.7 [psi]) "specific heat capacity of hot water"
C_dot_H=V_dot_H*rho_H*c_H "capacitance rate of hot water"
rho_C=density(Water,T=T_C_in,P=14.7 [psi]) "density of cooling water"
c_C=cP(Water,T=T_C_in,P=14.7 [psi]) "specific heat capacity of cooling water"
C_dot_C=V_dot_C*rho_C*c_C "capacitance rate of cooling water"

The heat transfer rate is obtained from an energy balance on the hot water:

q = C H (TH ,in − TH ,out ) (3)

The exit temperature of the cooling water is computed according to:

q = C C (TC ,out − TC ,in ) (4)

q_dot=C_dot_H*(T_H_in-T_H_out) "heat transfer rate"


q_dot=C_dot_C*(T_C_out-T_C_in) "cold outlet temperature"
The log-mean temperature difference is computed according to:

ΔTlm =
(T
H , out − TC ,in ) − (TH ,in − TC ,out )
(8-5)
⎡ (TH ,out − TC ,in ) ⎤
ln ⎢ ⎥
⎢⎣ (TH ,in − TC ,out ) ⎥⎦

The conductance is obtained from:

q = UA ΔTlm (8-6)

DT_lm=((T_H_out-T_C_in)-(T_H_in-T_C_out))/ln((T_H_out-T_C_in)/(T_H_in-T_C_out))
"log-mean temperature difference"
q_dot=UA*DT_lm "conductance"

which leads to UA = 568.7 Btu/hr-R.


Problem 8.1-1 (8-1 in text)
Dry air at Ta,in = 30°C, and atmospheric pressure is blown at ua = 1.0 m3/s through a cross-flow
heat exchanger in which refrigerant R134a is evaporating at a constant pressure of pR = 345 kPa.
The air exits the heat exchanger at Ta,out = 13°C. The tubes and fins of the heat exchanger are
both made of copper. The tubes have an outer diameter of Dout,t = 1.64 cm and tht = 1.5 mm tube
wall thickness. The fins are circular with a spacing that leads to 275 fins per meter, an outer
diameter of Dout,f = 3.1 cm and a thickness of thf = 0.25 mm. The heat transfer coefficient
between the R134a and the inner tube wall is estimated to be hR = 2,500 W/m2-K. The heat
transfer coefficient between the air and tubes and fins is estimated to be ha = 70 W/m2-K. The
total length of finned tubes is L = 110 m.
a) Determine the rate of heat transfer from the air.

Enter the known information into EES.

$UnitSystem SI MASS RAD PA K J


$Tabstops 0.2 0.4 0.6 3.5 in

"known information"
T_air_in=converttemp(C,K,30 [C]) "inlet air temperature"
P=101.3 [kPa]*convert(kPa,Pa) "atmospheric pressure"
V_dot=1.0 [m^3/s] "volumetric flowrate of air"
P_evap=345 [kPa]*convert(kPa,Pa) "pressure of evaporating R134a"
T_air_out=converttemp(C,K,10 [C]) "exit air temperature"
D_o=0.0164 [m] "outer diameter of tube"
WallThk=0.0015 [m] "pipe wall thickness"
D_fin=0.031 [m] "outer diameter of fins"
y_fin=0.00025 [m] "thickness of fins"
Fin_pitch=275 [1/m] "spacing of fins"
L=110 [m] "length of tubing"
h_bar_i=2500 [W/m^2-K] "two-phase heat transfer coefficient is high"
h_bar_o=70 [W/m^2-K] "outside heat transfer coefficient - varies with air velocity"

The mass flow rate of the air can be determined by multiplying the density of the inlet air by the
volumetric flow rate.

m = ρ air ,in V (1)

where ρair,in is the density of the air at the inlet condition. The heat transfer rate is evaluated
using an energy balance on the air:

q = m ( iair ,in − iair ,out ) (2)

where iair,in and iair,out are the specific enthalpy of the air at the inlet and exit temperatures,
respectively.

"Determine the heat transfer rate"


m_dot=density(Air,T=T_air_in,P=P)*V_dot "mass flow rate"
i_air_in=enthalpy(Air,T=T_air_in) "specific enthalpy of entering air"
i_air_out=enthalpy(Air,T=T_air_out) "specific enthalpy of exiting air"
q_dot=m_dot*(i_air_in-i_air_out) "energy balance on air"

b) Determine the value of the heat exchanger conductance for this heat exchanger.

The inner and outer radii of the tube are computed according to:

Do
ro = (3)
2

ri = ro − th (4)

The thermal resistance to convection from the inside surface of the tube is:

1
Ri = (5)
hi 2 π ri L

r_o=D_o/2 "outer radius of tube"


r_i=r_o-WallThk "inner radius of tube"
Res_i=1/(h_bar_i*2*pi*r_i*L) "resistance between evaporating fluid and tube wall"

The resistance to conduction through the tube is:

⎛r ⎞
ln ⎜ o ⎟
Rt = ⎝ i ⎠
r
(6)
2π k L

where k is the thermal conductivity of copper evaluated at the average air temperature.

T_air_avg=(T_air_in+T_air_out)/2 "average air temperature for evaluation of properties"


k=k_('Copper', (T_air_avg)/2) "thermal conductivity of copper tube"
Res_t=ln(r_o/r_i)/(2*pi*k*L) "resistance of copper wall is very small"

The fin efficiency (ηfin) is obtained using the EES function eta_fin_annular_rect. The surface area
of the fins is:

⎡⎛ D fin ⎞ 2 ⎤
Afin = π ⎢⎜ ⎟ − ro ⎥ 2 L FinPitch
2
(7)
⎢⎣⎝ 2 ⎠ ⎥⎦

and the thermal resistance of the fins is:

1
R fin = (8)
η fin Afin ho
eta=eta_fin_annular_rect(y_fin, r_o, D_fin/2, h_bar_o, k) "fin efficiency"
A_fin=pi*((D_fin/2)^2-r_o^2)*2*L*Fin_pitch "finned area"
Res_fin=1/(eta*h_bar_o*A_fin) "resistance of fins"

The area of the unfinned region is:

Aunfin = 2 π ro L (1 − FinPitch y fin ) (9)

and the thermal resistance of the unfinned area is:

1
Runfin = (10)
Aunfin ho

A_unfin=2*pi*r_o*L*(1-Fin_pitch*y_fin) "unfinned area"


Res_unfin=1/(A_unfin*h_bar_o) "resistance of unfinned surface"

The total thermal resistance is:

−1
⎡ 1 1 ⎤
R = Ri + Rt + ⎢ + ⎥ (11)
⎣⎢ R fin Runfin ⎦⎥

The conductance is:

1
UA = (12)
R

Res=Res_i+Res_t+(1/Res_fin+1/Res_unfin)^(-1) "total resistance"


UA=1/Res "conductance"

which leads to UA = 2122 W/K.


Problem 8.1-2 (8-2 in text)
The cross-flow heat exchanger described in Problem 8.1-1 (8-1 in text) has geometry similar to
that for compact heat exchanger 'fc_tubes_sCF-70-58J'. The frontal area of the heat exchanger is
Af = 0.5 m2 and the length of the heat exchanger in the flow direction is W = 0.25 m.
a.) Use the compact heat exchanger library to estimate the air-side conductance and the overall
heat exchanger conductance assuming that the heat transfer coefficient between the R134a
and the inner tube wall is hR = 2,500 W/m2-K.

Enter the known information into EES.

$UnitSystem SI MASS RAD PA K J


$Tabstops 0.2 0.4 0.6 3.5 in

"known information"
T_air_in=converttemp(C,K,30 [C]) "inlet air temperature"
P=101.3 [kPa]*convert(kPa,Pa) "atmospheric pressure"
V_dot=1.0 [m^3/s] "volumetric flowrate of air"
P_evap=345 [kPa]*convert(kPa,Pa) "pressure of evaporating R134a"
T_air_out=converttemp(C,K,10 [C]) "exit air temperature"
h_bar_i=2500 [W/m^2-K] "two-phase heat transfer coefficient is high"
A_f=0.5 [m^2] "frontal area of the heat exchanger"
W=0.25 [m] "length of the heat exchanger"
WallThk=0.0015 [m] "pipe wall thickness"

The Compact Heat Exchanger Library provides a procedure that returns the geometry
specifications for the surface area designated as ‘fc_tubes_sCF-70-58J’. The outer diameter of
the tubes (Do), fin pitch (finpitch), hydraulic diameter of the flow passages (Dh), fin thickness
(finthk), ratio of free flow to frontal area (σ), ratio of surface area to volume (α), and the ratio of
finned to unfinned surface area (Afin/A) are returned by the function.

"Use the compact heat exchanger library to determine the geometry of this heat exchanger"
Call CHX_geom_finned_tube('fc_tubes_sCF-70-58J': D_o, fin_pitch, D_h, fin_thk, sigma, alpha, A_fin\A)
"Note: sigma is the ratio of the minimum flow area to the frontal area and alpha is the surface area per
unit volume"

Air properties (μ, Pr, and c) are evaluated at the average air temperature.

"Air properties at the average temprature"


T_air_avg=(T_air_in+T_air_out)/2 "average air temperature for evaluation of properties"
mu=viscosity(Air,T=T_air_avg) "viscosity"
Pr=Prandtl(Air,T=T_air_avg) "Prandtl number"
c=cp(Air,T=T_air_avg) "specific heat"

The Compact_HX_ND procedure returns the Colburn j factor (jH) and the friction factor (f) given
the Reynolds number. The mass flow rate of the air can be determined by multiplying the
density of the inlet air by the volumetric flow rate.

m = ρ air ,in V (1)


where ρair,in is the density of the air at the inlet condition. The minimum flow area is computed
according to:

Amin = Af σ (2)

The mass flux is:

m
G= (3)
Amin

and the Reynolds number is:

G Dh
Re = (4)
μ

"Calculate the air-side heat transfer coefficient"


m_dot=density(Air,T=T_air_in,P=P)*V_dot "mass flow rate of air"
A_min=A_f*sigma "mininum free flow area"
G=m_dot/A_min "mass velocity"
Re=G*D_h/mu "Reynold's number"
Call Compact_HX_ND('fc_tubes_sCF-70-58J', Re: f,j_H) "determines j_H"

The Stanton number is computed according to:

2
jH = St Pr 3
(5)

and the heat transfer coefficient on the air side is computed according to:

ho
St = (6)
Gc

j_H=St*Pr^(2/3) "determines the Stanton number"


St=h_bar_o/(G*c) "h_bar_o is the heat transfer coefficient on the air side"

which leads to ho = 71.1 W/m2-K. The volume of the heat exchanger is:

V = Af W (7)

so the resistance on the air side (neglecting the fin efficiency) is:

1
Rair = (8)
V α ho

Vol=A_f*W "heat exchanger volume"


Res_air=1/(h_bar_o*Vol*alpha) "air-side heat transfer coefficient"

The heat transfer coefficient can be checked by calling the dimensional version of the compact
heat exchanger procedure, CHX_h_finned_tube.

Call CHX_h_finned_tube('fc_tubes_sCF-70-58J', m_dot, A_f, 'air',T_air_avg, P:h_bar_o_check)


"check result with dimensional call"

which also leads to ho = 71.1 W/m2-K.

The length of tube in the heat exchanger can be estimated based on the unfinned area of the core.
The unfinned area of the core is obtained according to:

⎛ Afin ⎞
Aunfin = V ⎜1 − ⎟ (9)
⎝ A ⎠

The unfinned area is related to the length of tube according to:

⎛ 1 ⎞
Aunfin = Ltube fin pitch ⎜ − finthk ⎟ π Do (10)
⎜ fin pitch ⎟
⎝ ⎠

which leads to Ltube = 119.3 m. The thermal resistance of convection to the flowing refrigerant
is:

1
RR134 a = (11)
hi π ( Do − th ) Ltube

where th is the wall thickness of the tube. The total conductance is:

1
UA = (12)
Rair + RR134 a

"Determine the refrigerant-side and overall heat transfer coefficients"


A_unfin=(1-A_fin\A)*Vol*alpha
"Vol*alpha is the total surface area; (1-A_fin\A) is the surface area of the tubes"
A_unfin=L_tube*fin_pitch*(1/fin_pitch-fin_thk)*pi*D_o "length of tube"
Res_R134a=1/(h_bar_i*pi*(D_o-2*WallThk)*L_tube) "R134a-side resistance"
UA=1/(Res_air+Res_R134a) "conductance"

which leads to UA = 2008 W/K.

b.) Compare the result to the value determined in Problem 8.1-1 (8-1).
The result for Problem 8.1-1, which assumed L = 110 m, is UA = 2122 W/K. If the length of the
tube in Problem 8.1-1 is increased to 119.3 m to match this problem then the conductance
increases to UA = 2302 W/K and the discrepancy increases to 14%.
Problem 8.1-3 (8-3 in text): Cross-Flow Heat Exchanger with Chilled Water
A decision has been made to use chilled water, rather than R134a in the heat exchanger
described in Problems 8.1-1 and 8.1-2 (8-1 and 8-2 in the text). The mass flow rate of chilled
water has been chosen so that the temperature rise of the water is ΔTw = 4°C as it passes through
the heat exchanger. The water-side is configured so that the chilled water flows through Nc = 10
parallel circuits.
a.) Estimate the overall heat transfer conductance and compare the result to your answers from
Problems 8.1-1 and 8.1-2 (8-1 and 8-2).

The known information into EES.

$UnitSystem SI MASS RAD PA K J


$Tabstops 0.2 0.4 0.6 3.5 in

"known information"
T_air_in=converttemp(C,K,30 [C]) "inlet air temperature"
P=101.3 [kPa]*convert(kPa,Pa) "atmospheric pressure"
V_dot=1.0 [m^3/s] "volumetric flowrate of air"
P_evap=345 [kPa]*convert(kPa,Pa) "pressure of evaporating R134a"
T_air_out=converttemp(C,K,10 [C]) "exit air temperature"
h_bar_i=2500 [W/m^2-K] "two-phase heat transfer coefficient is high"
A_f=0.5 [m^2] "frontal area of the heat exchanger"
W=0.25 [m] "length of the heat exchanger"
WallThk=0.0015 [m] "pipe wall thickness"
DELTAT_w=4 [K] "water temperature rise"

The air-side heat thermal resistance (Rair) is computed using the same compact heat exchanger
library, as discussed in Problem 8.1-2.

"Use the compact heat exchanger library to determine the geometry of this heat exchanger"
Call CHX_geom_finned_tube('fc_tubes_sCF-70-58J': D_o, fin_pitch, D_h, fin_thk, sigma, alpha, A_fin\A)
"Note: sigma is the ratio of the minimum flow area to the frontal area and alpha is the surface area per
unit volume"

"Air properties at the average temprature"


T_air_avg=(T_air_in+T_air_out)/2
"average air temperature for evaluation of properties"
mu=viscosity(Air,T=T_air_avg) "viscosity"
Pr=Prandtl(Air,T=T_air_avg) "Prandtl number"
c=cp(Air,T=T_air_avg) "specific heat"

"Calculate the air-side heat transfer coefficient"


m_dot=density(Air,T=T_air_in,P=P)*V_dot "mass flow rate of air"
A_min=A_f*sigma "mininum free flow area"
G=m_dot/A_min "mass velocity"
Re=G*D_h/mu "Reynold's number"
Call Compact_HX_ND('fc_tubes_sCF-70-58J', Re: f,j_H) "determines j_H"
j_H=St*Pr^(2/3) "determines the Stanton number"
St=h_bar_o/(G*c)
"h_bar_o is the heat transfer coefficient on the air side"
Vol=A_f*W "heat exchanger volume"
Res_air=1/(h_bar_o*Vol*alpha) "air-side heat transfer coefficient"
The length of tube in the heat exchanger can be estimated based on the unfinned area of the core.
The unfinned area of the core is obtained according to:

⎛ Afin ⎞
Aunfin = V ⎜1 − ⎟ (1)
⎝ A ⎠

The unfinned area is related to the length of tube according to:

⎛ 1 ⎞
Aunfin = Ltube fin pitch ⎜ − finthk ⎟ π Do (2)
⎜ fin pitch ⎟
⎝ ⎠

which leads to Ltube = 119.3 m.

"Determine the water-side and overall heat transfer coefficients"


A_unfin=(1-A_fin\A)*Vol*alpha
"Vol*alpha is the total surface area; (1-A_fin\A) is the surface area of the tubes"
A_unfin=L_tube*fin_pitch*(1/fin_pitch-fin_thk)*pi*D_o "length of tube"

The mass flow rate of water is calculated according to an energy balance on the heat exchanger:

m w cw ΔTw = m a ca (Ta ,in − Ta ,out ) (3)

where m a is the mass flow rate of air and cw and ca are the specific heat capacities of the water
and the air.

c_w=cP(Water,T=T_air_avg,P=1 [atm]*convert(atm,Pa)) "specific heat capacity of water"


m_dot_w*c_w*DELTAT_w=m_dot*c*(T_air_in-T_air_out) "energy balance on heat exchanger"

The inner diameter of the tube is:

Di = Do − 2 th (4)

where th is the wall thickness of the tube. The procedure PipeFlow is used to compute the heat
transfer coefficient for the flowing water ( hw , note that the mass flow rate per tube is m w / N c ).
The thermal resistance on the water side is:

1
Rw = (5)
hw π Di Ltube

The total conductance, neglecting fouling, is:


1
UA = (6)
( Rair + Rw )
D_i=D_o-2*WallThk "inner diameter of tube"
call PipeFlow('Water',T_air_avg,1 [atm]*convert(atm,Pa),m_dot_w,D_i,L_tube/N_c,0 [-]:h_bar_w, &
h_H_w ,DELTAP_w, Nusselt_T_w, f_w, Re_w)
Res_w=1/(h_bar_w*L_tube*pi*D_i) "water side resistance"
UA=1/(Res_w+Res_air) "conductance"

which leads to UA = 2358 W/K. This value is slightly higher than the conductance computed in
Problem 8.1-2 but very similar because the dominant resistance is on the air-side rather than the
water- or refrigerant-side.

b.) Estimate how much the overall heat transfer coefficient can be expected to drop over time
due to fouling of the closed chilled water loop.

The fouling factor (FF) is estimated assuming city or well water with the EES function
FoulingFactor. The resistance due to fouling is therefore:

FF
Rf = (7)
π Di Ltube

The conductance with fouling is:

1
UAf = (8)
( Rair + R f + Rw )
"with fouling"
FF=FoulingFactor('City or well water') "fouling factor"
Res_f=FF/(L_tube*pi*D_i) "resistance due to fouling"
UA_f=1/(Res_w+Res_f+Res_air) "conductance with fouling"

which leads to UAf = 2178 W/K, a decrease of approximately 8% due to fouling.


Problem 8.1-4
A heat exchanger core consists of finned flat tubes with geometry that is consistent with
ff_tubes_s91-0737-S. The frontal area of the core has width W= 5 ft and height H = 3 ft. The
length of the core in the flow direction is L = 1.5 ft. The fan that is connected to the core
provides a volumetric flow rate of Vopen = 30,000 ft3/min when there is no pressure rise and a
pressure rise of Δpdh = 2 inH2O when there is no flow. The fan curve is linear between these
points. The fan draws in air at Tin = 80°F and passes it across the coil.
a.) Determine the total surface area of the coil that is exposed to air.
b.) Determine the volumetric flow rate of air that will be provided to the coil.
c.) Determine the heat transfer coefficient on the air side.

The inputs are entered in EES:

$UnitSystem Eng F psi


$TabStops 0.5 3.5 in

"Inputs"
W=5 [ft] "width of coil"
H=3 [ft] "height of core"
L=1.5 [ft] "length of core"
T_in=80 [F] "inlet air temperature"
V_dot_open=30000 [ft^3/min] "fan flow at zero DP"
DP_dh=2 [inH2O]*convert(inH2O,psi) "fan pressure rise at zero flow"

The function CHX_geom_finned_tube is used to evaluate the volume specific surface area of the
core, α. The total surface area is:

As ,tot = α W H L (1)

Call CHX_geom_finned_tube('ff_tubes_s91-0737-S': D_o, fin_pitch, D_h, fin_thk, sigma, alpha, A_fin\A)


"geometry"
A_total=alpha*W*H*L "total surface area"

which leads to As,tot = 5041 ft2.

b.) Determine the volumetric flow rate of air that will be provided to the coil.

It is necessary to iterate on the volumetric flow rate until the pressure rise provided by the fan
matches the pressure drop across the coil. Initially, a volumetric flow rate, V , is assumed. The
density of air, ρ, is computed and used to evaluate the mass flow rate of air:

m = ρ V (2)

The function CHX_DELTAp_finned_tube is used to evaluate the pressure drop across the core,
Δpcore. The pressure rise produced by the fan is computed according to:
⎛ V ⎞
Δp fan = Δpdh ⎜ 1 − ⎟⎟ (3)
⎜ Vopen
⎝ ⎠

V_dot=10000 [ft^3/min] "volume flow of air"


rho=density(Air,T=T_in,P=14.7 [psi]) "density"
m_dot=V_dot*rho*convert(lb_m/min,lb_m/hour) "mass flow rate"
Call CHX_DELTAp_finned_tube('ff_tubes_s91-0737-S', m_dot, W*H,L, 'Air', T_in, T_in,&
14.7 [psi]: DP_core) "pressure drop"
DP_fan=DP_dh*(1-V_dot/V_dot_open) "fan pressure rise"

The guess values are updated (select Update Guesses from the Calculate menu) and the assumed
value of V is commented out and the pressure rise from the fan is set equal to the pressure drop
of the core.

{V_dot=10000 [ft^3/min]} "volume flow of air"


DP_fan=DP_core "system operating point"

which leads to V = 14,388 ft3/min.

c.) Determine the heat transfer coefficient on the air side.

The function CHX_h_finned_tube is used to evaluate the heat transfer coefficient, h .

Call CHX_h_finned_tube('ff_tubes_s91-0737-S', m_dot, W*H,'Air',T_in, 14.7 [psi]:h_bar)


"heat transfer coefficient"

which leads to h = 12.65 Btu/hr-ft2-R.


Problem 8.2-1 (8-4 in text)
In Problem 8.1-1 (8-1 in text), the inlet volumetric flowrate and the inlet and outlet temperatures
of the air were known and therefore it was possible to determine the heat transfer rate without a
heat exchanger analysis. However, you have just learned that the outlet air temperature was
measured with a thermocouple in only one location in the duct and it is not necessarily an
accurate measurement of the mixed average outlet air temperature. Use the log-mean
temperature difference method to estimate the average air outlet temperature.

The input information is entered in EES:

$UnitSystem SI MASS RAD PA K J


$Tabstops 0.2 0.4 0.6 3.5 in

"known information"
T_air_in=converttemp(C,K,30 [C]) "inlet air temperature"
P=101.3 [kPa]*convert(kPa,Pa) "atmospheric pressure"
V_dot=1.0 [m^3/s] "volumetric flowrate of air"
P_evap=345 [kPa]*convert(kPa,Pa) "pressure of evaporating R134a"

The mass flow rate of the air can be determined by multiplying the density of the inlet air by the
volumetric flow rate.

m = ρ air ,in V (1)

where ρair,in is the density of the air.

"Determine the heat transfer rate"


rho_air=density(Air,T=T_air_in,P=P) "density of air"
m_dot=rho_air*V_dot "mass flow rate"

The conductance was estimated in Problems 8.1-1 and 8.1-2. Here we will use the value
determined in Problem 8.1-1:

UA=2122 [W/K] "conductance calculated in Problem 8.1-1"

The log-mean temperature difference method is not convenient computationally. It is best to


begin with a guess for the outlet temperature, Tair,out, in order to compute the quantities that are
required to determine the heat transfer and then iterate. The measured value of Tair,out is used as
the guess value:

T_air_out=converttemp(C,K,10 [C]) "exit air temperature - measured"

The temperature of the R134a (TR134a) is obtained from the evaporating pressure:

T_R134a=temperature(R134a,P=P_evap,x=1) "temperature of evaporating R134a"

The log-mean temperature difference for a counter-flow configuration is computed according to:
ΔTlm,cf =
(Tair ,out − TR134 a ) − (Tair ,in − TR134 a )
(2)
⎡ (Tair ,out − TR134 a ) ⎤
ln ⎢ ⎥
⎢⎣ (Tair ,in − TR134 a ) ⎥⎦

The correction factor based on the fact that this is a crossflow heat exchanger with one fluid (the
refrigerant) mixed is a function of P and R, calculated according to:

P=
(T air ,in − Tair ,out )
(3)
(T air ,in − TR134 a )

C T −T
R =  air = R134 a R134 a → 0 (4)
CR134 a Tair ,in − Tair ,out

The correction factor, F, is obtained using the function LMTD_CF and the log-mean temperature
difference is computed according to:

ΔTlm = F ΔTlm,cf (5)

The heat transfer rate is specified by:

q = UA ΔTlm (6)

DELTAT_lmtd_cf=((T_air_out-T_R134a)-(T_air_in-T_R134a))/ln((T_air_out-T_R134a)/&
(T_air_in-T_R134a)) "log-mean temperature difference for a counterflow heat exchanger"
Pp=(T_air_in-T_air_out)/(T_air_in-T_R134a) "P-parameter"
Rp=0.001 [-] "R-parameter"
F=LMTD_CF('crossflow_one_unmixed',Pp,Rp) "correction factor for the LMTD method"
DELTAT_lmtd=DELTAT_lmtd_cf*F "log-mean temperature difference"
q_dot=UA*DELTAT_lmtd "heat transfer rate"

The problem is solved and the guess values are updated. The guess for Tair,out is commented out
and then Tair,out is calculated according to:

q
Tair ,out = Tair ,in − (7)
m cair

where cair is the specific heat capacity of air:

{T_air_out=converttemp(C,K,10 [C])} "exit air temperature - measured"


c_air=cP(Air,T=T_air_in) "specific heat capacity of air"
T_air_out=T_air_in-q_dot/(m_dot*c_air) "recalculate outlet air temperature"
T_air_out_C=converttemp(K,C,T_air_out) "in C"

which leads to Tair,out = 8.74ºC.


Problem 8.2-2 (8-5 in text)
Table P8.2-2 provides heat transfer data from a manufacturer’s catalog for a counterflow oil
cooler. The table provides the heat transfer rate for three different oil flow rates (expressed in
gpm, gallons per minute). The values in the table are the heat transfer rate between the oil and
water in units of Btu/min-ETD where ETD is the entering temperature difference in °F. The
density and specific heat of the oil are ρo = 830 kg/m3 and co = 2.3 kJ/kg-K, respectively. The
water enters at Vw = 35 gallons per minute at Tw,in = 180°F and atmospheric pressure. The oil
enters at To,in = 240°F.

Table P8.2-2: Heat transfer data (heat transfer rate/ETD) for different models and oil flow rates
Oil flow rate
1 gpm 3 gpm 5 gpm
Model 1 2.5 Btu/min-deg. F 4.9 Btu/min-deg. F
Model 2 2.9 Btu/min-deg. F 6.1 Btu/min-deg. F 8.1 Btu/min-deg. F
Model 3 3.1 Btu/min-deg. F 6.8 Btu/min-deg. F 9.7 Btu/min-deg. F

a.) Determine the outlet oil temperature, the log mean temperature difference, and the overall
conductance for Model 2 at oil flow rates 1, 3 and 5 gallons/min.

The input data are entered in EES:

$UnitSystem SI MASS RAD PA K J


$Tabstops 0.2 0.4 0.6 3.5 in

"Inputs"
rho_o=830 [kg/m^3] "density of oil"
c_o=2.3 [kJ/kg-K]*convert(kJ/kg-K,J/kg-K) "specific heat capacity of oil"
V_dot_w=35 [gal/min]*convert(gal/min,m^3/s) "volumetric flow rate of water"
P_w=1 [atm]*convert(atm,Pa) "water pressure"
T_w_in=converttemp(F,K,180 [F]) "inlet water temperature"
T_o_in=converttemp(F,K,240 [F]) "inlet oil temperature"

The first entry in the table for the oil flow rate and performance are entered:

V_dot_o_gpm=1 [gal/min] "oil flow rate, in gpm"


q_dot\ETD_BtuminF=2.5 [Btu/min-F] "performance, in Btu/min-F"
V_dot_o=V_dot_o_gpm*convert(gal/min,m^3/s) "oil flow rate"
q_dot\ETD=q_dot\ETD_BtuminF*convert(Btu/min-F,W/K) "performance"

The mass flow rate of water and oil are calculated according to:

m w = ρ w Vw (1)

m o = ρ o Vo (2)

rho_w=density(Water,T=T_w_in,P=P_w) "density of water"


m_dot_w=rho_w*V_dot_w "mass flow rate of water"
m_dot_o=rho_o*V_dot_o "mass flow rate of oil"
The heat transfer rate is computed from the given value of heat transfer to entering temperature
difference ratio:

q
q =
ETD
(To,in − Tw,in ) (3)

The outlet temperatures of the water and oil are computed from an energy balance:

q
Tw,out = Tw,in + (4)
m w cw

q
To ,out = To ,in − (5)
m o co

q_dot=q_dot\ETD*(T_o_in-T_w_in) "rate of heat transfer"


c_w=cP(Water,T=T_w_in,P=P_w) "specific heat capacity of water"
T_w_out=T_w_in+q_dot/(m_dot_w*c_w) "water outlet temperature"
T_o_out=T_o_in-q_dot/(m_dot_o*c_o) "oil outlet temperature"

The log-mean temperature difference is computed according to:

ΔTlm =
(To , out − Tw,in ) − (To ,in − Tw,out )
(6)
⎡ (To ,out − Tw,in ) ⎤
ln ⎢ ⎥
⎢⎣ (To ,in − Tw,out ) ⎥⎦

and the conductance is given by:

q
UA = (7)
ΔTlm

DELTAT_lmtd=((T_o_out-T_w_in)-(T_o_in-T_w_out))/ln((T_o_out-T_w_in)/(T_o_in-T_w_out))
"log-mean temperature difference for a counterflow heat exchanger"
UA=q_dot/DELTAT_lmtd "conductance"

The calculations are repeated for each of the data points for model 2 by commenting out the
values of oil flow rate and q / ETD in the equations window and setting up a parametric table
with these values set, as shown in Figure 1.
Figure 1: Parametric table.

b) Plot the conductance as a function of the oil flow rate and provide an explanation for the
observed variation.

The conductance as a function of oil flow rate is shown in Figure 2. The conductance increases
with flow rate because the increased flow likely enhances the oil-side heat transfer coefficient.
350

325

300
Conductance (W/K)

275

250

225

200

175

150
1 1.5 2 2.5 3 3.5 4 4.5 5
OIl flow rate (gpm)
Figure 2: Conductance as a function of oil flow rate.

c) Estimate the oil outlet temperature if the oil enters the heat exchanger at To,in = 225°F with a
flow rate of Vo = 4 gpm and the water enters with temperature Tw,in = 180°F at a flow rate of
V = 35 gpm.
w

Part (a) is commented out and the inputs for part (c) are entered:

"Inputs for (c)"


rho_o=830 [kg/m^3] "density of oil"
c_o=2.3 [kJ/kg-K]*convert(kJ/kg-K,J/kg-K) "specific heat capacity of oil"
V_dot_w=35 [gal/min]*convert(gal/min,m^3/s) "volumetric flow rate of water"
P_w=1 [atm]*convert(atm,Pa) "water pressure"
T_w_in=converttemp(F,K,180 [F]) "inlet water temperature"
T_o_in=converttemp(F,K,225 [F]) "inlet oil temperature"
V_dot_o_gpm=4 [gal/min] "oil flow rate, in gpm"
V_dot_o=V_dot_o_gpm*convert(gal/min,m^3/s) "oil flow rate"
According to Figure 2, the conductance of the heat exchanger with an oil flow rate of 4 gal/min
will be approximately 315 W/K.

UA=315 [W/K] "conductance for (c)"

The oil and water mass flow rates are calculated according to Eqs. (1) and (2).

rho_w=density(Water,T=T_w_in,P=P_w) "density of water"


m_dot_w=rho_w*V_dot_w "mass flow rate of water"
m_dot_o=rho_o*V_dot_o "mass flow rate of oil"

In order to use the log-mean temperature difference for a simulation type problem like this one it
is best to assume a heat transfer rate and calculate the water and oil outlet temperatures according
to Eqs. (4) and (5).

q_dot=3000 [W] "this is a guess for the heat transfer rate"


c_w=cP(Water,T=T_w_in,P=P_w) "specific heat capacity of water"
T_w_out=T_w_in+q_dot/(m_dot_w*c_w) "water outlet temperature"
T_o_out=T_o_in-q_dot/(m_dot_o*c_o) "oil outlet temperature"
T_o_out_F=converttemp(K,F,T_o_out) "in F"

The log-mean temperature difference is computed according to Eq. (6).

DELTAT_lmtd=((T_o_out-T_w_in)-(T_o_in-T_w_out))/ln((T_o_out-T_w_in)/(T_o_in-T_w_out))
"log-mean temperature difference for a counterflow heat exchanger"

The problem is solved and the guess values are updated. The assumed value for q is recalculated
according to:

q = UA ΔTlm (8)

{q_dot=3000 [W]} "this is a guess for the heat transfer rate"


q_dot=UA*DELTAT_lmtd "recalculate the heat transfer rate"

which leads to To,out = 203.6°F.


Problem 8.2-3
You are specifying a plate frame heat exchanger with a counter-flow configuration to transfer
heat from hot water to cooling water. The hot water enters the heat exchanger at TH,in = 120ºF
with a flow rate of VH = 0.85 gal/min. The cooling water enters at TC,in = 35ºF with flow rate
V = 0.65 gal/min. If the hot water must be cooled to TH,out = 75ºF then what must is the required
C
conductance of the heat exchanger?

The inputs are entered in EES:

$UnitSystem English psi F


$TabStops 0.5 3.5

"Inputs"
V_dot_H=0.85 [gal/min]*convert(gal/min,ft^3/hr) "volumetric flow rate of hot water"
V_dot_C=0.65 [gal/min]*convert(gal/min,ft^3/hr) "volumetric flow rate of cooling water"
T_H_in=120 [F] "hot inlet temperature"
T_H_out=75 [F] "hot outlet temperature"
T_C_in=35 [F] "cooling water inlet temperature"

The capacitance rates are computed according to:

C H = ρ H cH VH (1)

C C = ρC cC VC (2)

where ρ and c are the density and specific heat capacity of the streams.

"capacitance rates"
rho_H=density(Water,T=T_H_in,P=14.7 [psi]) "density of hot water"
c_H=cP(Water,T=T_H_in,P=14.7 [psi]) "specific heat capacity of hot water"
C_dot_H=V_dot_H*rho_H*c_H "capacitance rate of hot water"
rho_C=density(Water,T=T_C_in,P=14.7 [psi]) "density of cooling water"
c_C=cP(Water,T=T_C_in,P=14.7 [psi]) "specific heat capacity of cooling water"
C_dot_C=V_dot_C*rho_C*c_C "capacitance rate of cooling water"

The heat transfer rate is obtained from an energy balance on the hot water:

q = C H (TH ,in − TH ,out ) (3)

The exit temperature of the cooling water is computed according to:

q = C C (TC ,out − TC ,in ) (4)

q_dot=C_dot_H*(T_H_in-T_H_out) "heat transfer rate"


q_dot=C_dot_C*(T_C_out-T_C_in) "cold outlet temperature"
The log-mean temperature difference is computed according to:

ΔTlm =
(T
H , out − TC ,in ) − (TH ,in − TC ,out )
(8-5)
⎡ (TH ,out − TC ,in ) ⎤
ln ⎢ ⎥
⎢⎣ (TH ,in − TC ,out ) ⎥⎦

The conductance is obtained from:

q = UA ΔTlm (8-6)

DT_lm=((T_H_out-T_C_in)-(T_H_in-T_C_out))/ln((T_H_out-T_C_in)/(T_H_in-T_C_out))
"log-mean temperature difference"
q_dot=UA*DT_lm "conductance"

which leads to UA = 568.7 Btu/hr-R.


PROBLEM 8.3-1 (8-6 in text): Energy Recovery
The plant where you work includes a process that results in a stream of hot combustion products
at moderate temperature Thg,in = 150°C with mass flow rate m hg = 0.25 kg/s. The properties of
the combustion products can be assumed to be the same as those for air. You would like to
recover the energy associated with this flow in order to reduce the cost of heating the plant and
therefore you are evaluating the use of the air-to-air heat exchanger shown in Figure P8.3-1.
close-up of channel structure
top view
hc = 1 mm km = 15 W/m-K
thf = 0.1 mm

pf = 0.1 mm

hot gas
Th ,in = 150°C
cold air
m hg = 0.25 kg/s
thw = 0.2 mm Tcg ,in = −5°C
m cg = 0.5 kg/s
flow passages for cold gas
(flowing into the page) cold air, Tcg,in

flow passages for hot gas hot gas, Th,in Th,out


(flowing into the page) L = 10 cm

L = 10 cm

W = 20 cm Tcg,out

front view side view


Figure P8.3-1: Air-to-air heat exchanger.

The air-to-air heat exchanger is a cross-flow configuration. The length of the heat exchanger
parallel to the two flow directions is L = 10 cm. The width of the heat exchanger in the direction
perpendicular to the two flow directions is W = 20 cm. Cold air enters the heat exchanger from
outdoors at Tcg,in = -5°C with mass flow rate m cg = 0.50 kg/s and is heated by the combustion
products to Tcg,out. The hot and cold air flows through channels that are rectangular (both sides of
the heat exchanger have the same geometry). The width of the channels is hc = 1.0 mm. There
are fins placed in the channel in order to provide structural support and also increase the surface
area for heat transfer. The fins can be assumed to run the complete length of the heat exchanger
and are 100% efficient. The fins are spaced with pitch, pf = 0.5 mm and the fins are thf = 0.10
mm thick. The thickness of the metal separating the cold channels from the hot channels is thw =
0.20 mm and the conductivity of this metal is km = 15 W/m-K. Both the hot and cold flows are at
nominally atmospheric pressure. The fouling factor associated with the flow of combustion gas
through the heat exchanger is R′′f = 0.0018 K-m2/W. There is no fouling associated with the
flow of outdoor air through the heat exchanger.
a.) Compute the heat transfer coefficient between the hot air and the channel walls and the cold
gas and the channel walls. Use the inlet temperatures of the air flows to compute the
properties.

The inputs are entered in EES:

$UnitSystem SI MASS RAD PA K J


$TABSTOPS 0.2 0.4 0.6 0.8 3.5 in

"Inputs"
L=10 [cm]*convert(cm,m) "size of module - parallel to flow directions"
W=20 [cm]*convert(cm,m) "size of module - perpendicular to flow directions"
th_w=0.2 [mm]*convert(mm,m) "channel wall thickness"
th_f=0.1 [mm]*convert(mm,m) "fin thickness"
h_c=1.0 [mm]*convert(mm,m) "channel height"
p_f=0.5 [mm]*convert(mm,m) "fin pitch"
k_m=15 [W/m-K] "metal conductivity"
FF=0.0018 [K-m^2/W] "combustion gas fouling factor"
m_dot_hg=0.25 [kg/s] "mass flow rate of hot gas"
T_hg_in=converttemp(C,K,150 [C]) "hot gas inlet temperature"
m_dot_cg=0.5 [kg/s] "mass flow rate of cold gas"
T_cg_in=converttemp(C,K,-5 [C]) "cold gas inlet temperature"

The properties of the hot and cold flows (chg, khg, μhg, ρhg, ccg, kcg, μcg, and ρcg) are computed
using EES’ internal property routines. The properties should be computed at the average flow
temperature. However, the outlet temperatures are not yet known, so initially, the average
temperature is set to the inlet temperature. This approximation can be corrected after the outlet
temperatures are determined and the guess values are updated.

T_hg_avg=T_hg_in "estimate of the average hot gas temperature"


c_hg=cP(Air,T=T_hg_avg) "specific heat capacity"
k_hg=conductivity(Air,T=T_hg_avg) "conductivity"
mu_hg=viscosity(Air,T=T_hg_avg) "viscosity"
rho_hg=density(Air,T=T_hg_avg,p=1 [atm]*convert(atm,Pa)) "density"

T_cg_avg=T_cg_in "estimate of the average cold gas temperature"


c_cg=cP(Air,T=T_cg_avg) "specific heat capacity"
k_cg=conductivity(Air,T=T_cg_avg) "conductivity"
mu_cg=viscosity(Air,T=T_cg_avg) "viscosity"
rho_cg=density(Air,T=T_cg_avg,p=1 [atm]*convert(atm,Pa)) "density"

The number of hot or cold channels in the heat exchanger is:

W
Nc = (1)
2 ( hc + thw )

The number of fins in each channel is:

L
Nf = (2)
pf
N_c=floor(W/(2*(h_c+th_w))) "number of channels"
N_f=floor(L/p_f) "number of fins per channel"

Note that the floor command rounds the result down to the nearest integer. The total cross-
sectional area for flow (on either side) is:

Ac = N c N f ( p f − th f ) hc (3)

and the total wetted perimeter (on either side) is:

per = N c N f 2 ( hc + p f − th f ) (4)

so the hydraulic diameter of the flow channels is:

Ac
Dh = 4 (5)
per

A_c=N_c*N_f*h_c*(p_f-th_f) "cross-sectional area for flow"


per=N_c*N_f*2*(h_c+p_f-th_f) "wetted perimeter"
D_h=4*A_c/per "hydraulic diameter"

The mean velocity of the hot gas is:

m hg
um, hg = (6)
Ac ρ hg

The Reynolds number associated with this flow is:

um, hg Dh ρ hg
Rehg = (7)
μhg

and the Prandtl number of the hot gas is:

μhg chg
Prhg = (8)
khg

u_m_hg=m_dot_hg/(A_c*rho_hg) "mean velocity"


Re_hg=u_m_hg*D_h*rho_hg/mu_hg "Reynolds number"
Pr_hg=mu_hg*c_hg/k_hg "Prandtl number"

The correlations for flow through a rectangular duct are accessed using the procedure DuctFlow_N
and used to compute the average friction factor ( f hg ) and Nusselt numbers ( Nu T ,hg and Nu H , hg ).
The heat transfer coefficient on the hot gas side is computed:
Nu T ,hg khg
hhg = (9)
Dh

Note that the constant temperature Nusselt number is used in this case because it is a lower
bound and you would like the predictions to be conservative.

call DuctFlow_N(Re_hg,Pr_hg,L/D_h, (p_f-th_f)/h_c,0 [-]: Nusselt_T_hg, Nusselt_H_hg, f_hg)


htc_hg=Nusselt_T_hg*k_hg/D_h "heat transfer coefficient"

The calculations are repeated for the cold gas:

u_m_cg=m_dot_cg/(A_c*rho_cg) "mean velocity"


Re_cg=u_m_cg*D_h*rho_cg/mu_cg "Reynolds number"
Pr_cg=mu_cg*c_cg/k_cg "Prandtl number"
call DuctFlow_N(Re_cg,Pr_cg,L/D_h, (p_f-th_f)/h_c,0 [-]: Nusselt_T_cg, Nusselt_H_cg, f_cg)
htc_cg=Nusselt_T_cg*k_cg/D_h "heat transfer coefficient"

These calculations lead to hhg = 246 W/m2-K and hcg = 235.7 W/m2-K. When the average
(instead of the inlet) temperature is used to evaluate properties, the values are 221.2 and 207.3
W/m2-K, respectively.

b.) Compute the total conductance of the heat exchanger.

The resistance associated with convection to the hot gas is:

1
Rconv ,hg = (10)
per L hhg

The resistance associated with fouling on the hot side is:

FF
R foul = (11)
per L

The resistance associated with conduction through the metal walls is:

thm
Rcond = (12)
km 2 L2 N c

The resistance associated with convection to the cold gas is:

1
Rconv ,cg = (13)
per L hcg

The total resistance between the hot and cold gas is:
Rtotal = Rconv , hg + R foul + Rcond + Rconv ,cg (14)

The conductance is:

1
UA = (15)
Rtotal

R_conv_hg=1/(per*L*htc_hg) "resistance to convection on hot gas side"


R_conv_cg=1/(per*L*htc_cg) "resistance to convection on cold gas side"
R_cond=th_w/(k_m*2*L*L*N_c) "conduction resistance"
R_foul=FF/(per*L) "fouling resistance"
R_total=R_conv_hg+R_foul+R_cond+R_conv_cg "total resistance"
UA=1/R_total "total conductance"

which leads to UA = 458.2 W/K (with properties evaluated at the inlet fluid temperatures) and
415.7 W/K (with properties evaluated at the average fluid temperatures.)

c.) Determine the heat transferred in the heat exchanger and the temperature of the cold gas
leaving the heat exchanger.

The capacity rates of the hot and cold gas flows are computed:

C hg = m hg chg (16)

C cg = m cg ccg (17)

and the minimum and maximum values of the capacity rates ( C min and C max ) are determined.

C_dot_hg=m_dot_hg*c_hg "capacity rate of hot gas"


C_dot_cg=m_dot_cg*c_cg "capacity rate of cold gas"
C_dot_min=MIN(C_dot_hg,C_dot_cg) "minimum capacity rate"
C_dot_max=MAX(C_dot_hg,C_dot_cg) "maximum capacity rate"

The capacity ratio and number of transfer units are computed:

C
CR =  min (18)
Cmax

UA
NTU =  (19)
Cmin

and the function HX is used to determine the effectiveness of the heat exchanger (ε). Note that
the geometry of the passages makes this a clearly a heat exchanger where both sides are
unmixed:
C_R=C_dot_min/C_dot_max "capacity ratio"
NTU=UA/C_dot_min "number of transfer units"
epsilon=HX('crossflow_both_unmixed', NTU, C_dot_cg, C_dot_hg, 'epsilon') "access eff-NTU solution"

The maximum possible rate of heat transfer is computed:

qmax = C min (Thg ,in − Tcg ,in ) (20)

and the actual heat transfer rate is:

q = ε qmax (21)

q_dot_max=C_dot_min*(T_hg_in-T_cg_in) "maximum heat transfer rate"


q_dot=epsilon*q_dot_max "actual heat transfer rate"

which leads to q = 26.8 kW. The temperature of the cold gas leaving the heat exchanger is:

q
Tcg ,out = Tcg ,in +  (22)
Ccg

The temperature of the hot gas leaving the heat exchanger is:

q
Thg ,out = Thg ,in + (23)
C hg

T_hg_out=T_hg_in-q_dot/C_dot_hg "hot gas exit temperature"


T_cg_out=T_cg_in+q_dot/C_dot_cg "cold gas exit temperature"

The initial estimate of the average temperatures are commented out and the actual average
temperatures calculated:

{T_hg_avg=T_hg_in } "estimate of the average hot gas temperature"


T_hg_avg=(T_hg_in +T_hg_out)/2 "average hot gas temperature"
{T_cg_avg=T_cg_in} "estimate of the average cold gas temperature"
T_cg_avg=(T_cg_in+T_cg_out)/2 "average cold gas temperature"

which leads to Thg,out = 317.0 K and Tcg,out = 321.5 K.

d.) Blowers are required to force the hot and cold flows through the heat exchanger. Assume
that you have blowers that are ηblower = 0.65 efficient. Estimate the total blower power
required to operate the energy recovery unit.

The pressure drops on the hot and cold sides of the heat exchanger are:
ρ hg um2 ,hg ⎛ L ⎞
Δphg = ⎜ f hg ⎟ (24)
2 ⎝ Dh ⎠

ρ cg um2 ,cg ⎛ L ⎞
Δpcg = ⎜ f cg ⎟ (25)
2 ⎝ Dh ⎠

The total blower power required is:

Δphg m hg Δpcg m cg
Wblower = + (26)
ρ hg ηblower ρcg ηblower

DeltaP_cg=rho_cg*u_m_cg^2*(f_cg*L/D_h)/2 "cold gas pressure drop"


DeltaP_hg=rho_hg*u_m_hg^2*(f_hg*L/D_h)/2 "cold gas pressure drop"
eta_blower=0.65 [-] "blower efficiency"
W_dot_blower=DeltaP_cg*m_dot_cg/(eta_blower*rho_cg)+DeltaP_hg*m_dot_hg/(eta_blower*rho_hg)
"total blower power"

which leads to Wblower = 13.3 kW.

e.) If you pay ec = 0.08$/kW-hr for electricity (to run the blowers) and 1.50$/therm for gas (to
heat the plant) then estimate the net savings associated with the energy recovery system
(neglect capital investment cost) for time = 1 year; this is the savings associated with the heat
transferred in the heat exchanger less the cost to run the blower for a year. Assume that the
plant runs continuously.

The cost to run the blowers is:

blowercost = Wblower time ec (27)

The savings associated with the heat transferred in the heat exchanger is:

heatsavings = q time hc (28)

The net savings per year is therefore:

netsavings = heatsavings − blowercost (29)

time=365 [day]*convert(day,s) "operating time"


ec=0.08 [$/kW-hr]*convert($/kW-hr,$/J) "cost of electricity"
hc=1.50 [$/therm]*convert($/therm,$/J) "cost of heat"
blowercost=ec*W_dot_blower*time "cost to run blower"
heatsavings=hc*q_dot*time "heating savings"
savingsnet=heatsavings-blowercost "net savings"
which leads to a net savings of 2,811 $/year (inlet temperatures used to evaluate properties) or
2,685$/year (average temperatures used to evaluate properties).

f.) Plot the net savings per year as a function of the mass flow rate of the cold air that is being
heated. Your plot should show a maximum; explain why this maximum exists.

Figure 2 illustrates the net savings as a function of the mass flow rate of the cold air being
heated. Also shown is the savings in heating and the cost to run the blower.

Figure 2: Net savings, blower power, and heating savings per year as a function of the mass flow rate of cold
gas.

Notice that the blower power increases dramatically with mass flow rate, particularly when the
flow becomes turbulent at approximately 0.46 kg/s. The heating savings increases with mass
flow rate as well but at a decreasing rate; this is because the heat exchanger effectiveness
decreases when the mass flow rate of air becomes very large. Note: when using the average
temperatures to evaluate properties, EES may not initially converge at low cold gas flows. A
useful trick to obtain convergence is to start the Parametric Table calculations at a cold gas flow
rate of 0.1 kg/s and do the calculations for flows up to 0.8 kg/s. Then do the calculations in the
Parametric table from the smallest flow rate to 0.1 kg/s in reverse. Be sure that the Update Guess
Values check box is checked. Alternatively evaluate properties at the inlet temperatures rather
than average temperatures.

g.) Determine the optimal values of the mass flow rate of combustion gas and cold gas ( m hg and
m cg ) that maximize the net savings per year. You should use the MIN/MAX capability in
EES to accomplish this - what is the maximum savings/year? This is the most you could
afford to pay for the blowers and heat exchanger if you wanted a 1 year pay-back.
The values of m hg and m cg are commented out:

{m_dot_hg=0.25 [kg/s] "mass flow rate of hot gas"}


{m_dot_cg=0.5 [kg/s] "mass flow rate of cold gas"}

and Min/Max is selected from the Calculate menu. The net savings is maximized by adjusting
the independent variables m_dot_hg and m_dot_cg (Figure 3).

Figure 3: Find Minimum or Maximum window.

Set the bounds on the two independent variables (Figure 4) to reasonable values.

Figure 4: Set bounds on independent variables.


The optimization routine indicates that the most optimal values of mass flow rates are m hg =
0.176 kg/s and m cg = 0.251 kg/s which leads to a net savings of $5748/year (with properties
estimated at the inlet temperatures).
PROBLEM 8.3-2 (8-7 in text): Gas Turbine Engine
A gas turbine engine is used onboard a small ship to drive the propulsion system. The engine
consists of a compressor, turbine, and combustor as shown in Figure P8.3-2(a). Ambient air is
pulled through the gas turbine engine with a mass flow rate of m = 0.1 kg/s and enters the
compressor at T1 = 20°C and P1 = 1 atm. The exit pressure of the compressor is P2 = 3.5 atm and
T2 = 167°C. The air enters a combustor where it is heated to T3 = 810°C with very little loss of
pressure so that P3 = 3.5 atm. The hot air leaving the combustor enters a turbine where it is
expanded to P4 = 1 atm. The temperature of the air leaving the turbine is T4 = 522°C. You may
assume that the turbine and compressor are well-insulated and that the specific heat capacity of
air is constant and equal to c = 1000 J/kg-K. The difference between the power produced by the
turbine and required by the compressor is used to drive the ship.

compressor turbine

net power to ship


fuel
T4 = 522°C
P4 = 1atm
T2 = 167°C T3 = 810°C
P2 = 3.5 atm P3 = 3.5 atm
T1 = 20°C
P1 = 1 atm
m = 0.1 kg/s
Figure P8.3-2(a): Un-recuperated gas turbine engine.

a.) Determine the efficiency of the gas turbine engine (the ratio of the net power to the ship to
the heat transferred in the combustor).

The inputs are entered in EES:

$UnitSystem SI MASS RAD PA K J


$TABSTOPS 0.2 0.4 0.6 0.8 3.5 in

"Inputs"
m_dot=0.1 [kg/s] "mass flow rate of air"
T_1=converttemp(C,K,20) "inlet temperature of air"
P_1=1 [atm]*convert(atm,Pa) "inlet pressure of air"
P_2=3.5 [atm]*convert(atm,Pa) "compressor exit pressure"
T_2=converttemp(C,K,167) "compressor exit temperature"
T_3=converttemp(C,K,810) "combustor exit temperature"
P_3=3.5 [atm]*convert(atm,Pa) "combustor exit pressure"
T_4=converttemp(C,K,522) "turbine exit temperature"
P_4=1.0 [atm]*convert(atm,Pa) "turbine exit pressure"
c=1000 [J/kg-K] "specific heat capacity"

The power required by the compressor is given by:

w c = m c (T2 − T1 ) (1)
The power generated by the turbine is:

w t = m c (T3 − T4 ) (2)

The net power generated by the gas turbine engine is therefore:

w net = w t − w c (3)

The amount of heat transferred in the combustor is:

qcomb,ur = m c (T3 − T2 ) (4)

and the efficiency of the engine is:

w net
η gte ,ur = (5)
qcomb ,ur

w_dot_c=m_dot*c*(T_2-T_1) "compressor power required"


w_dot_t=m_dot*c*(T_3-T_4) "turbine power produced"
w_dot_net=w_dot_t-w_dot_c "net power produced"
q_dot_comb_ur=m_dot*c*(T_3-T_2) "combustor heat required"
eta_gte_ur=w_dot_net/q_dot_comb_ur "GTE efficiency"

which leads to ηgte,ur = 0.22.

b.) The combustor runs on fuel with a heating value of HV = 44x106 J/kg and a mission lasts t =
2 days. What is the mass of fuel that the ship must carry?

The fuel that the ship must carry is:

qcomb ,ur t
M fuel ,ur = (6)
HV

HV=44e6 [J/kg] "heating value of fuel"


time=2 [day]*convert(day,s) "mission duration"
M_fuel_ur=q_dot_comb_ur*time/HV "mass of fuel"
M_fuel_ur_lbm=M_fuel_ur*convert(kg,lbm) "in lbm"

which leads to Mfuel,ur = 253 kg (557 lbm).

In order to reduce the amount of fuel required, you have been asked to look at the option of
adding a recuperative heat exchanger to the gas turbine cycle, as shown in Figure P8.3-2(b).
You are considering the air-to-air heat exchanger that was evaluated in Problem 8.3-1 (8-6 in the
text) and is shown in Figure P8.3-1 (Figure P8-6 in the text). The air-to-air heat exchanger is a
cross-flow configuration. The length of the heat exchanger parallel to the two flow directions is
L = 10 cm. The width of the heat exchanger in the direction perpendicular to the two flow
directions is W = 20 cm but this can easily be adjusted by adding additional plates. Air enters the
heat exchanger from the compressor and is heated by the air leaving the turbine. The hot and
cold air flows through channels that are rectangular (both sides of the heat exchanger have the
same geometry). The width of the channels is hc = 1.0 mm. There are fins placed in the channel
in order to provide structural support and also increase the surface area for heat transfer. The
fins can be assumed to run the complete length of the heat exchanger and are 100% efficient.
The fins are spaced with pitch, pf = 0.5 mm and the fins are thf = 0.10 mm thick. The thickness
of the metal separating the cold channels from the hot channels is thw = 0.20 mm and the
conductivity of this metal is km = 15 W/m-K. The hot and cold flows are at nominally
atmospheric pressure and the compressor discharge pressure, respectively. The fouling factor
associated with the flow of the combustion products leaving the turbine is R′′f = 0.0018 K-m2/W.
There is no fouling associated with the flow of the air leaving the compressor.

compressor turbine

T1 = 20°C net power to ship


P1 = 1 atm fuel
m = 0.1 kg/s
(5) T3 = 810°C
T2 = 167°C P3 = 3.5 atm
P2 = 3.5 atm
T4 = 522°C
recuperative heat exchanger P4 = 1atm
Figure P8.3-2(b): Recuperated gas turbine engine.

c.) Compute the heat transfer coefficient between the hot air from the turbine and the channel
walls and the colder air from the compressor and the channel walls. You may use the inlet
temperatures of the air flows to compute the properties.

The inputs related to the heat exchanger geometry are entered in EES:

"HX Geometry"
L=10 [cm]*convert(cm,m) "size of HX - parallel to flow directions"
W=20 [cm]*convert(cm,m) "size of HX - perpendicular to flow directions"
th_w=0.2 [mm]*convert(mm,m) "channel wall thickness"
th_f=0.1 [mm]*convert(mm,m) "fin thickness"
h_c=1.0 [mm]*convert(mm,m) "channel height"
p_f=0.5 [mm]*convert(mm,m) "fin pitch"
k_m=15 [W/m-K] "stainless conductivity"
FF=0.0018 [K-m^2/W] "combustion gas fouling factor"

The properties of the hot and cold flows (ch, kh, μh, ρh, cc, kc, μc, and ρc) are computed using
EES’ internal property routines:

T_h_avg=T_4 "hot gas temperature"


k_h=conductivity(Air,T=T_h_avg) "conductivity"
mu_h=viscosity(Air,T=T_h_avg) "viscosity"
rho_h=density(Air,T=T_h_avg,P=P_4) "density"

T_c_avg=T_2 "cold gas temperature"


k_c=conductivity(Air,T=T_c_avg) "conductivity"
mu_c=viscosity(Air,T=T_c_avg) "viscosity"
rho_c=density(Air,T=T_c_avg,P=P_2) "density"

The number of hot or cold channels in the heat exchanger is:

W
Nc = (7)
2 ( hc + thw )

The number of fins in each channel is:

L
Nf = (8)
pf

N_c=floor(W/(2*(h_c+th_w))) "number of channels"


N_f=floor(L/p_f) "number of fins per channel"

Note that the floor command rounds the result down to the nearest integer. The total cross-
sectional area for flow (on either side) is:

Ac = N c N f ( p f − th f ) hc (9)

and the total wetted perimeter (on either side) is:

per = N c N f 2 ( hc + p f − th f ) (10)

so the hydraulic diameter of the flow channels is:

Ac
Dh = 4 (11)
per

A_c=N_c*N_f*h_c*(p_f-th_f) "cross-sectional area for flow"


per=N_c*N_f*2*(h_c+p_f-th_f) "wetted perimeter"
D_h=4*A_c/per "hydraulic diameter"

The mean velocity of the hot gas is:

m
um , h = (12)
Ac ρ h

The Reynolds number associated with this flow is:


um ,h Dh ρ h
Reh = (13)
μh

and the Prandtl number of the hot gas is:

μh c
Prhg = (14)
kh

u_m_h=m_dot/(A_c*rho_h) "mean velocity"


Re_h=u_m_h*D_h*rho_h/mu_h "Reynolds number"
Prandtl_h=mu_h*c/k_h "Prandtl number"

The correlations for flow through a rectangular duct are accessed using the procedure DuctFlow_N
and used to compute the average friction factor ( f h ) and Nusselt numbers ( Nu T , h and Nu H ,h ).
The heat transfer coefficient on the hot gas side is computed:

Nu T ,h kh
hh = (15)
Dh

Note that the constant temperature Nusselt number is used in this case because it is a lower
bound and I would like the predictions to be conservative.

call DuctFlow_N(Re_h,Prandtl_h,L/D_h, (p_f-th_f)/h_c,0 [-]: Nusselt_T_h, Nusselt_H_h, f_h)


htc_h=Nusselt_T_h*k_h/D_h "heat transfer coefficient"

The calculations are repeated for the cold side of the heat exchanger:

u_m_c=m_dot/(A_c*rho_c) "mean velocity"


Re_c=u_m_c*D_h*rho_c/mu_c "Reynolds number"
Prandtl_c=mu_c*c/k_c "Prandtl number"
call DuctFlow_N(Re_c,Prandtl_c,L/D_h, (p_f-th_f)/h_c,0 [-]: Nusselt_T_c, Nusselt_H_c, f_c)
htc_c=Nusselt_T_c*k_c/D_h "heat transfer coefficient"

These calculations lead to hh = 381 W/m2-K and hc = 242 W/m2-K.

d.) Compute the total conductance of the heat exchanger.

The resistance associated with convection to the hot air leaving the turbine is:

1
Rconv ,h = (16)
per L hh

The resistance associated with fouling on the hot side is:


FF
R foul = (17)
per L

The resistance associated with conduction through the metal walls is:

thm
Rcond = (18)
km 2 L2 N c

The resistance associated with convection to the air leaving the compressor is:

1
Rconv ,c = (19)
per L hc

The total resistance between the hot and cold air is:

Rtotal = Rconv ,h + R foul + Rcond + Rconv ,c (20)

The conductance is:

1
UA = (21)
Rtotal

R_conv_h=1/(per*L*htc_h)
"resistance to convection on hot gas side"
R_conv_c=1/(per*L*htc_c)
"resistance to convection on cold gas side"
R_cond=th_w/(k_m*2*L*L*N_c) "conduction resistance"
R_foul=FF/(per*L) "fouling resistance"
R_total=R_conv_h+R_foul+R_cond+R_conv_c "total resistance"
UA=1/R_total "total conductance"

which leads to UA = 541 W/K.

e.) Determine the heat transferred in the heat exchanger and the temperature of the air entering
the combustor.

The capacity rates of the hot and cold air flows are computed:

C h = m h c (22)

C c = m c c (23)

and the minimum and maximum values of the capacity rates ( C min and C max ) are determined.

C_dot_h=m_dot*c "capacity rate of hot gas"


C_dot_c=m_dot*c "capacity rate of cold gas"
C_dot_min=MIN(C_dot_h,C_dot_c) "minimum capacity rate"
C_dot_max=MAX(C_dot_h,C_dot_c) "maximum capacity rate"

The capacity ratio and number of transfer units are computed:

C
CR =  min (24)
Cmax

UA
NTU =  (25)
Cmin

and the function HX is used to determine the effectiveness of the heat exchanger (ε). Note that
the geometry of the passages makes this a clearly a heat exchanger where both sides are
unmixed:

C_R=C_dot_min/C_dot_max "capacity ratio"


NTU=UA/C_dot_min "number of transfer units"
epsilon=HX('crossflow_both_unmixed', NTU, C_dot_c, C_dot_h, 'epsilon') "access eff-NTU solution"

The maximum possible rate of heat transfer is computed:

qmax = C min (T4 − T2 ) (26)

and the actual heat transfer rate is:

q = ε qmax (27)

q_dot_max=C_dot_min*(T_4-T_2) "maximum heat transfer rate"


q_dot=epsilon*q_dot_max "actual heat transfer rate"

which leads to q = 26.8 kW. The temperature of the air entering the combustor is:

q
T5 = T2 +  (28)
Cc

T_5=T_2+q_dot/C_dot_c "cold gas exit temperature"

which leads to T5 = 708.5 K.

f.) What is the efficiency of the recuperated gas turbine engine?

The amount of heat transfer required by the combustor in the recuperated gas turbine engine is:

qcomb ,r = m c (T3 − T5 ) (29)


The efficiency associated with the recuperated gas turbine engine is:

w net
η gte,r = (30)
qcomb ,r

q_dot_comb_r=m_dot*c*(T_3-T_5)
"recuperated combustor heat transfer rate"
eta_gte_r=w_dot_net/q_dot_comb_r "recuperated GTE efficiency"

which leads to qcomb ,r = 37.5 kW and ηgte,r = 0.376.

g.) What is the mass of fuel that must be carried by the ship for the 2 day mission if it uses a
recuperated gas turbine engine?

The fuel that the ship must carry is:

qcomb ,r t
M fuel ,r = (31)
HV

M_fuel_r=q_dot_comb_r*time/HV
"Mass of fuel required by recuperated GTE"
M_fuel_r_lbm=M_fuel_r*convert(kg,lbm) "in lbm"

which leads to Mfuel,r = 147 kg (325 lbm).

h.) The density of the metal separating the cold channels from the hot channels is ρm = 8000
kg/m3 and the density of the fins is ρf = 7500 kg/m3. What is the mass of the heat exchanger?

The mass of the heat exchanger is given by:

M HX = L2 thw N c ρ m + 2 N c N f L th f hc ρ f (32)

rho_m=8000 [kg/m^3] "density of stainless steel"


rho_f=7500 [kg/m^3] "density of aluminum"
M_HX=L^2*th_w*N_c*rho_m+2*N_c*N_f*L*th_f*h_c*rho_f "mass of the heat exchanger"

which leads to MHX = 3.8 kg.

i.) What is the net savings in mass associated with using the air-to-air recuperated gas turbine
engine for the 2 day mission.

The net savings in mass if the saving in fuel less the heat exchanger mass:

M savings = M fuel ,ur − M fuel ,r − M HX (33)


M_savings=M_fuel_ur-M_fuel_r-M_HX
"net savings in mass associated with the recuperated gas turbine engine"

which leads to Msavings = 101.6 kg.

j.) Plot the net savings in mass as a function of the width of the heat exchanger (W). Your plot
should show a maximum; explain why.

Figure 2 illustrates the mass savings as a function of the width of the heat exchanger. At very
low width the mass savings is small because the heat exchanger is not very effective. At high
width the mass savings is again small because the heat exchanger mass is large and the
effectiveness is near its maximum value so the fuel savings is not increasing.

Figure 2: Mass savings as a function of the width of the recuperative heat exchanger.
Problem 8.3-3
A parallel-flow heat exchanger is shown schematically in Figure P8.3-3(a). The heat exchanger
is being used to heat air from Ta,in = 10°C using a flow of water at Tw = 80°C. The capacity rate
of the air is C a = 0.1 W/K and the capacity rate of the water is C w = 10 W/K. The conductance
of the heat exchanger is UA = 0.1 W/K.

Ta ,in = 10°C, C a = 0.1W/K Ta,out

q

UA = 0.1 W/K
Tw,in = 80°C, C w = 10 W/K Tw,out
Figure P8.3-3(a): Parallel-flow heat exchanger.

a.) Sketch the temperature distributions in the heat exchanger (i.e., the temperature of the air and
the water as a function of position, x); make sure that the qualitative features of your sketch is
correct based on the operating conditions.

Figure 2 illustrates a sketch of the temperature distributions in the heat exchanger.

Figure 2: Qualitative sketch of the temperature distribution in the heat exchanger.

The capacity rate of the air is much less than the capacity rate of the water and so the water
temperature will not change much. The number of transfer units is near unity and so we expect
the air temperature to change substantially; these characteristics are reflected in Figure 2.
b.) Determine the heat transfer rate in the heat exchanger ( q ) as well as the temperature of the
air leaving the heat exchanger (Ta,out).

The maximum and minimum capacity rates are identified; C min = C a and C max = C w . The number
of transfer units is computed:

UA 0.1 W K
NTU = = = 1.0 (1)
C min K 0.1 W

The capacity ratio is computed:

C 0.1 W K
CR =  min = = 0.01 (2)
Cmax K 10 W

According to Figure 8-15, the effectiveness of a parallel flow heat exchanger with NTU = 1.0 and
CR ~ 0 is ε = 0.64. The maximum possible heat transfer rate in the heat exchanger is:

0.1 W ( 80 − 10 ) °C
qmax = C min (Tw,in − Ta ,in ) = = 7.0 W (3)
K

The actual heat transfer rate is:

0.64 7.0 W
q = ε qmax = = 4.48 W (4)

The temperature of the air leaving the heat exchanger is:

q 4.48 W K
Ta ,out = Ta ,in + = 10°C + = 54.8°C (5)

Ca 0.1 W

c.) Sketch the heat transfer rate ( q ) as a function of the heat exchanger conductance (UA,
assume that all other conditions remain the same as shown in Figure 1). Make sure that you
indicate the heat transfer rate that you expect at UA = 0 and UA → ∞.

Figure 3 is a sketch of the rate of heat transfer as a fuction of conuctance.


Figure 3: Sketch of the heat transfer rate as a function of the heat exchanger conductance.

At UA = 0 you would expect no heat transfer. If UA gets very large (practically, greater than
about 0.5 W/K which corresponds to NTU = 5.0) then the heat transfer will approach the
maximum possible value of 7.0 W. These characteristics are reflected in Figure 3.

d.) Your boss has heard that it is always better to use a counter-flow as opposed to a parallel-
flow heat exchanger and so he insists that you should re-plumb the heat exchanger so that it
is in a counter flow configuration; as shown in Figure P8.3-3(b). Do you expect this change
to substantially improve the performance (i.e., increase the rate of heat transferred in the heat
exchanger)? Clearly justify your result qualitatively.

Ta ,in = 10°C, C a = 0.1W/K Ta,out

q

UA = 0.1 W/K
Tw,out Tw,in = 80°C, C w = 10 W/K
Figure P8.3-3(b): Heat exchanger re-plumbed so that it is in a counter flow configuration.

The capacity ratio of the heat exchanger remains very small; therefore, the temperature
distribution will not be substantially affected by configuration (the water temperature does not
change very much whether it is moving parallel or counter to the air). Thus the effectiveness and
the heat transfer rate will not be substantially improved by the proposed change to a counter flow
configuration.
Problem 8.3-4 (8-8 in text): Heat Exchanger Run-Around Loop
Buildings that have high ventilation rates can significantly reduce their heating load by
recovering energy from the exhaust air stream. One way in which this can be done is by use of a
run-around loop shown in Figure P8.3-4. As shown in the figure, a run-around loop consists of
two conventional liquid to air cross-flow heat exchangers. An ethylene glycol solution with 35%
mass percent glycol is pumped at a rate m g = 1 kg/s through both heat exchangers. The specific
heat of this glycol solution is cg = 3.58 kJ/kg-K. (Note that the properties of glycol solutions can
be determined using the brineprop2 function in EES). During winter operation, the glycol
solution is heated by the warm air exiting in the exhaust duct. The warm glycol solution is then
used to preheat cold air entering from outdoors through the ventilation duct.

ventilation duct exhaust duct


m a = 5 kg/s, T3 = 25°C
(2)
m g = 1 kg/s (3)

(5)
pump

(6)
(4)
(1)
m a = 5 kg/s, T1 = −10°C
Figure P8.3-4: Run-Around loop for energy recovery

In the present case, outdoor air is blown into the building at a rate of m a = 5 kg/s from outdoors.
The outdoor temperature is T1 = -10°C. The building is tightly constructed so the exhaust air
flow rate may be assumed to be equal to the ventilation air flow rate ( m a = 5 kg/s ). The air
leaving the building through the exhaust duct is at T3 = 25°C. The cross-flow heat exchangers in
the exhaust and ventilation streams are identical, each having a finned coil configuration and an
estimated conductance UA = 10 kW/K.
a.) Determine the effectiveness of the ventilation and exhaust heat exchangers.

Known information is entered into EES.

$UnitSystem SI MASS RAD PA K J


$TABSTOPS 0.2 0.4 0.6 0.8 3.5 in

"known information from problem statement"


P=101.3 [kPa]*convert(kPa,Pa) "atmospheric pressure"
T_3=converttemp(C,K,25 [C]) "temperature of air entering exhaust duct"
T_1=converttemp(C,K,-10 [C]) "outdoor air temperature"
m_dot_air=5.0 [kg/s] "ventilation air flowrate"
V_dot=m_dot_air*volume(Air,T=T_1,P=P) "ventilation volumetric flow - just for interest"
m_dot_g=1.0 [kg/s] "mass flow rate of glycol solution"
c_g=3.58 [kJ/kg-K]*convert(kJ/kg-K,J/kg-K) "specific heat capacity of glycol solution"
UA=10 [kW/K]*convert(kW/K,W/K) "conductance"

The capacity rates of the air and glycol are obtained from:
C air = m air cair (1)

C g = m g cg (2)

The minimum capacitance rate ( C min ) is obtained using the Min function in EES. The number of
transfer units is:

UA
NTU = (3)
C min

The effectiveness (ε) is obtained from the HX function in EES.

c_air=cp(Air,T=(T_1+T_3)/2) "ave. specific heat of air"


C_dot_air=m_dot_air*c_air "capacitance rate of the air for both heat exchangers"
C_dot_g=m_dot_g*c_g "capacitance rate of the glycol"
C_dot_min=min(C_dot_air, C_dot_g) "minimum capacitance rate"
NTU=UA/C_dot_min "definition of NTU"
epsilon_hx=HX('crossflow_one_unmixed', NTU, C_dot_air, C_dot_g, 'epsilon') "effectiveness"

which leads to ε = 0.702.

b.) Determine the temperatures of the glycol solution at states (5) and (6).

This part is tricky because it is necessary to solve the energy balances and heat exchanger
relations for the exhaust and ventilation heat exchangers simultaneously. Although it may not be
necessary, it is easier to do this calculation stepwise by guessing the temperature T5.

T_5=converttemp(C,K,10 [C]) "guess - remove this when coupling to ventilation hx"

The heat transfer for the heat exchanger in the exhaust duct is:

qex = ε C min (T3 − T5 ) (4)

The temperatures of the glycol and air leaving the exhaust duct are computed using energy
balances on the glycol and air, respectively:

q
T6 = T5 + ex (5)
Cg

q
T4 = T3 −  ex (6)
Cair

q_dot_HXex=epsilon_hx*C_dot_min*(T_3-T_5) "rate of heat transfer for exhaust hx"


T_6=T_5+q_dot_HXex/C_dot_g "temperature of glycol leaving exhaust duct"
T_4=T_3-q_dot_HXex/C_dot_air "temperature of air leaving exhaust duct"

The heat transfer rate for the heat exchanger in the ventilation duct is:

qv = ε C min (T6 − T1 ) (7)

The temperature of the air leaving the ventilation duct is obtained from an energy balance:

q
T2 = T1 +  v (8)
Cair

q_dot_HXv=epsilon_hx*C_dot_min*(T_6-T_1) "rate of heat transfer for ventilation hx"


T_2=T_1+q_dot_HXv/C_dot_air "temperature of air leaving ventilation duct"

The problem is solved and the guess values updated. The assumed value of T5 is commented out
and T5 is recalculated according to:

q
T5 = T6 − ex (9)
Cg

{T_5=converttemp(C,K,10 [C])} "guess - remove this when coupling to ventilation hx"


T_5=T_6-q_dot_HXv/C_dot_g "recalculate temperature of glycol leaving ventilation duct"

c.) Determine the overall effectiveness of the run-around loop.

The overall effectiveness of the run-around loop is defined as the ratio of the actual to the
maximum rates of heat transfer. The actual rate of heat transfer is qex (or qv , they are identical).
The maximum rate of heat transfer is the product of the minimum air capacitance rate and the
difference between T3 and T1. Since the air flow rate is the same for both heat exchangers:

qex
ε runaround =  (10)
C air ( T3 − T1 )

epsilon_runaround=q_dot_HXv/(C_dot_air*(T_3-T_1)) "definition of run-around loop effectiveness"

which leads to an overall effectiveness of 0.386.

d.) It has been suggested that the performance of the run-around loop can be improved by
optimizing the glycol flow rate. Plot the run-around loop overall effectiveness as a function
of the glycol solution flow rate for 0.1 kg/s < m g < 4 kg/s. Assume that the conductance of
the heat exchangers vary with glycol solution flow rate to the 0.4 power based on a value of
UA = 10 kW/K at m g = 1 kg/s. What flow rate do you recommend?

The conductance is specified according to:


{UA=10 [kW/K]*convert(kW/K,W/K) "conductance"}
UA=10 [kW/K]*convert(kW/K,W/K)*(m_dot_g/1 [kg/s])^0.4 "conductance as a function of glycol flow rate"

Figure 2 illustrates the overall effectiveness of the runaround loop as a function of the glycol
flow rate.

0.6
Runaround loop effectiveness

0.55
0.5
0.45
0.4
0.35
0.3
0.25
0.2
0.15
0.1
0.05
0 0.5 1 1.5 2 2.5 3 3.5 4
Mass flow rate of glycol (kg/s)
Figure 2: Runaround loop effectiveness as a function of glycol mass flow rate.
Problem 8.3-5: Cryosurgical Probe
A cryosurgical probe uses a Joule-Thomson (ienthalpic) expansion of argon gas to produce cold
temperatures, as shown in Figure P8.3-5. The argon is compressed to p2 = 200 atm and cooled at
constant pressure to T3 = 300 K where it enters a recuperative counter-flow heat exchanger. The
gas is cooled to state (4) in the recuperator using the return gas that enters the recuperator at T6 =
140 K. After leaving the recuperator at state (4), the argon is throttled to a low pressure state (5)
where it used to cool tissue. The refrigeration load provided by the cryoprobe ( qload ) warms the
gas to T6 = 140 K. The argon returning to the compressor is at a pressure of p1 = 100 kPa;
pressure losses occur in the recuperator, but the pressure difference between states (5) and (6) is
negligible.

w comp

compressor
(2)

qrej
(1)
(3)
recuperative
heat exchanger
(6)

qload (4)

(5) isenthalpic valve


Figure 8.3-5: Joule-Thomson cycle for a cryosurgical probe.

The recuperator is a smooth thin-walled stainless steel concentric tube heat exchanger. The high
pressure argon passes through the center tube and the low pressure argon returns through the
annulus. The length of the tubes is L = 0.35 m. The inner diameter of the outer tube diameter is
Dout = 1.75 mm. The wall thickness of the tube is th = 180 μm. The inner diameter of the inner
tube is Din = 600 μm. The mass flow rate of argon is m = 0.3 g/s.
a.) Determine the conductance of the recuperative heat exchanger.

The input information is entered in EES:

$UnitSystem SI MASS RAD PA K J


$TABSTOPS 0.2 0.4 0.6 0.8 3.5 in

"known information"
m_dot=0.0003[kg/s] "mass flow rate of Argon"
D_i=6E-4 [m] "inner diameter of the tube"
D_o=1.754E-3 [m] "probe outer diameter"
t=1.8E-4 [m] "tube wall thickness"
L=0.35 [m] "length of heat exchanger"
T[3]=300 [K] "temperature of argon as it enters the recuperator"
P[1]=100 [kPa]*convert(kPa,Pa) "Pressure at exit of recuperator (entering the compressor)"
T[6]=140 [K] "temperature of argon at inlet to recuperator"
P[3]=200 [atm]*convert(atm,Pa) "pressure of argon at inlet to recuperator"

The high pressure exit temperature, T4, is guessed and used to calculate the average temperature
of the fluid on the high-pressure side of the heat exchanger. The properties of the fluid on the
high pressure side (ch, μh, kh, Prh, and ρh) are computed based on this average temperature and
the inlet pressure.

"high pressure Argon stream in center tube recuperator"


T[4]=200 [K] "this is a guess that will be removed later"
T_H_avg=(T[3]+T[4])/2 "average temperature on high pressure side"
cp_h=cp(Argon,T=T_H_avg,P=P[3]) "average specific heat of argon in high pressure stream"
mu_h=viscosity(Argon,T=T_H_avg,P=P[3]) "viscosity for the high-pressure side"
k_h=conductivity(Argon,T=T_H_avg,P=P[3]) "thermal conductivity for the high-pressure side"
Pr_h=Prandtl(Argon,T=T_H_avg,P=P[3]) "Prandtl number for the high-pressure side"
rho_h=density(Argon,T=T_H_avg,P=P[3]) "density for the high-pressure side"

The mean velocity of the high pressure side fluid is:

4 m
um , h = (1)
ρ h π Di2

The Reynolds number of the flow is:

um ,h Di ρ h
Reh = (2)
μh

The procedure PipeFlow_N is used to compute the average Nusselt number ( Nu i ) and average
friction factor ( fi ). The heat transfer coefficient is computed according to:

Nu i kh
hi = (3)
Di

The thermal resistance to convection on the high-pressure side is:

1
Rconv ,i = (4)
π Di L hi

u_m_h=4*m_dot/rho_h/(pi*D_i^2) "velocity of the high-pressure argon"


Re_h=u_m_h*D_i*rho_h/mu_h "Reynold's number for the high pressure steam"
RelRough=0 [-] "smooth tube"
call PipeFlow_N(Re_h,Pr_h,L/D_i,RelRough: Nusselt_bar_T_i,Nusselt_bar_H_i,f_bar_h)
"library function returns Nusselt number"
h_bar_i=Nusselt_bar_T_i*k_h/D_i "average heat transfer coefficient on high-pressure side"
R_conv_i=1/(h_bar_i*pi*D_i*L) "resistance to convection on high-pressure side"
The low pressure exit temperature, T1, is guessed and used to calculate the average temperature
of the fluid on the low-pressure side of the heat exchanger. The properties of the fluid on the
low pressure side (cc, μc, kc, Prc, and ρc) are computed based on this average temperature and the
inlet pressure.

"low pressure Argon stream in recuperator"


T[1]=250 [K] "this guess will be removed later"
T_C_avg=(T[1]+T[6])/2 "average temperature on low pressure side"
cp_c=cp(Argon,T=T_C_avg,P=P[1]) "average specific heat of argon in low pressure stream"
mu_c=viscosity(Argon,T=T_C_avg,P=P[1]) "viscosity for the low-pressure side"
k_c=conductivity(Argon,T=T_C_avg,P=P[1]) "thermal conductivity for the low-pressure side"
Pr_c=Prandtl(Argon,T=T_C_avg,P=P[1]) "Prandtl number for the low-pressure side"
rho_c=density(Argon,T=T_C_avg,P=P[1]) "density for the low-pressure side"

The cross-sectional area and perimeter of the annulus is:

π⎡ 2
Ac ,c = Do − ( Di + 2 th ) ⎤
2
(5)
4⎣ ⎦

perc = π ⎡⎣ Do + ( Di + 2 th ) ⎤⎦ (6)

The mean velocity of the low pressure side fluid is:

m
um ,c = (7)
ρ c Ac ,c

The hydraulic diameter of the annulus is:

4 Ac ,c
Dh ,c = (8)
perc

The Reynolds number of the flow is:

um ,c Dh ,c ρ c
Rec = (9)
μc

The procedure AnnularFlow_N is used to compute the average Nusselt number ( Nu o ) and average
friction factor ( f o ). The heat transfer coefficient is computed according to:

Nu o kc
ho = (10)
Dh ,c

The thermal resistance to convection on the high-pressure side is:


1
Rconv ,o = (11)
π ( Di + 2 th ) L ho

A_c_c=pi*(D_o^2-(D_i+2*t)^2)/4 "cross-sectional area on low pressure side"


per_c=pi*(D_o+(D_i+2*t)) "perimeter on low pressure side"
u_m_c=m_dot/(A_c_c*rho_c) "velocity of the low-pressure argon"
D_h_c=4*A_c_c/per_c "hydraulic diameter on low pressure side"
Re_c=u_m_c*D_h_c*rho_c/mu_c "Reynold's number for the low pressure steam"
call AnnularFlow_N(Re_c, Pr_c, L/D_h_c,(D_i+2*t)/D_o, RelRough: Nusselt_bar_T_o,&
Nusselt_bar_H_o, f_bar_c)
h_bar_o=Nusselt_bar_T_o*k_c/D_h_c "average heat transfer coefficient on high-pressure side"
R_conv_o=1/(h_bar_o*pi*(D_i+2*t)*L) "resistance to convection on low-pressure side"

The overall conductance is:

1
UA = (12)
Rconv ,i + Rconv ,o

UA=1/(R_conv_o+R_conv_i) "total conductance"

which leads to UA = 0.363 W/K.

b.) Determine the heat exchanger effectiveness.

The capacitance rate of the low pressure and high pressure fluids are:

C c = m cc (13)

C h = m ch (14)

The minimum capacitance rate ( C min ) is obtained using the min command in EES. The number
of transfer units is:

UA
NTU =  (15)
Cmin

The heat exchanger effectiveness (ε) is obtained using the HX function in EES.

C_dot_c=m_dot*cp_c "capacitance rate on low pressure side"


C_dot_h=m_dot*cp_h "capacitance rate on high pressure side"
C_dot_min=min(C_dot_c,C_dot_h) "minimum capacitance rate"
NTU=UA/C_dot_min "number of transfer units"
eff=HX('counterflow', NTU, C_dot_c, C_dot_h, 'epsilon') "effectiveness"

which leads to ε = 0.793.


c.) Determine the pressures at states (4) and (6).

The average friction factors are used to compute the pressure drops on both sides:

L ρ h um , h
2

Δph = fi (16)
Di 2

L ρ c um , c
2

Δpc = f o (17)
Dh ,c 2

The pressures at states (4) and (6) are computed according to:

p4 = p3 − Δph (18)

p6 = p1 + Δpc (19)

DELTAP_h=f_bar_h*L/D_i*rho_h*(u_m_h^2/2) "pressure drop on high pressure side"


P[4]=P[3]-DELTAP_h "pressure at state 4"
DELTAP_c=f_bar_c*L/D_h_c*rho_c*(u_m_c^2/2) "pressure drop on low pressure side"
P[6]=P[1]+DELTAP_c "pressure at state 6"

which leads to p4 = 20.24 MPa and p6 = 196.6 kPa.

d.) Determine the temperatures at states (1), (4), (5), and (6).

The maximum heat transfer rate is:

qmax = C min (T3 − T6 ) (20)

The actual heat transfer rate is:

q = ε qmax (21)

The inlet enthalpy of the high pressure stream, i3, is computed according to the inlet temperature
and pressure. The exit enthalpy of the high pressure stream is:

q
i4 = i3 − (22)
m

q_dot_max=C_dot_min*(T[3]-T[6]) "maximum heat transfer rate"


q_dot=q_dot_max*eff "actual heat transfer rate"
i[3]=enthalpy(Argon,T=T[3],P=P[3]) "high pressure inlet enthalpy"
i[4]=i[3]-q_dot/m_dot "high pressure outlet enthalpy"
The problem is solved and the guess values are updated. The assumed value of T4 is commented
out and T4 is recalculated based on the enthalpy and pressure at state (4).

{T[4]=200 [K]} "this is a guess that will be removed later"


T[4]=temperature(Argon,h=i[4],P=P[4]) "recalculate high pressure outlet temperature"

The inlet enthalpy of the low pressure stream, i6, is computed according to the inlet temperature
and pressure. The exit enthalpy of the low pressure stream is:

q
i1 = i6 + (23)
m

i[6]=enthalpy(Argon,T=T[6],P=P[6]) "low pressure inlet enthalpy"


i[1]=i[6]+q_dot/m_dot "low pressure outlet enthalpy"

The problem is solved and the guess values are updated. The assumed value of T1 is commented
out and T1 is recalculated based on the enthalpy and pressure at state (1).

{T[1]=250 [K]} "this guess will be removed later"


T[1]=temperature(Argon,h=i[1],P=P[1]) "recalculate low pressure outlet temperature"

The valve is isenthalpic, therefore:

i5 = i4 (24)

The pressure drop across the load heat exchanger is neglected, therefore:

p5 = p6 (25)

The temperature of the fluid leaving the valve, T5, is obtained from the pressure and enthalpy.

i[5]=i[4] "isenthalpic expansion"


P[5]=P[6] "no pressure drop across load heat exchanger"
T[5]=temperature(Argon,h=i[5],P=P[5]) "temperature of fluid leaving valve"

which leads to T1 = 265.4 K, T4 = 226.4 K, T5 = 112.5 K, and T6 = 140 K.

e.) Determine the refrigeration load provided by the cryoprobe, qload = m ( i6 − i5 ) where i is the
specific enthalpy.

The refrigeration load is:

qload = m ( i6 − i5 ) (26)

q_dot_load=m_dot*(i[6]-i[5]) "refrigeration load"


which leads to qload = 4.50 W.

f.) Plot refrigeration load as a function of mass flow rate. You should see an optimal mass flow
rate, why does such an optimal flow exist?

Figure 2 illustrates the refrigeration load as a function of mass flow rate. The optimal value
exists because as mass flow rate increases the refrigeration potential increases but the
effectiveness of heat exchanger and pressure drops both increase. The optimal mass flow rate
balances these effects.

6.5

6
Refrigeration load (W)

5.5

4.5

3.5

3
0.0002 0.0004 0.0006 0.0008 0.001
Mass flow rate (kg/s)
Figure 2: Refrigeration load as function of mass flow rate.

g.) Plot the recuperator effectiveness and the refrigeration load as a function of the inner
diameter of the inner tube for 400 μm < Din < 1.1 mm and 0.3 g/s mass flow rate. Is there an
optimum inner diameter for the mass flow rate of 0.3 g/s? Why?

The effectiveness and load are shown in Figure 3 as a function of the inner tube diameter. The
optimal value exists because a larger inner tube diameter tends to increase the effectiveness by
improving the low-pressure side thermal resistance but also tends to increase the pressure drop.
7 0.95

Heat exchanger effectiveness


qload
0.9

Refrigeration load (W)


5
ε
0.85
4

3
0.8

2
0.75
1

0 0.7
0.0004 0.0006 0.0008 0.001
Inner diameter of inner tube (m)
Figure 3: Refrigeration load and heat exchanger effectiveness as a function of the inner diameter of the inner
tube.
Problem 8.3-6 Solar water heating system heat exchanger
A schematic of a solar water heating system is shown in Figure P8.3-6. When solar radiation is
available, a 30% (by mass) propylene glycol solution is pumped at volumetric flow rate Vc
through the collectors and through the tubes in a counter-current heat exchanger. The heat
exchanger is tube and shell geometry with 40 tubes each having a diameter of 0.5 cm in a shell
that has a diameter of 4 cm. The copper pipe walls are thin enough to neglect in this analysis.
The tubes are 1.25 m in length. A second pump circulates water at volumetric flow rate Vh from
the storage tank through the shell of the heat exchanger in a counter-flow direction to the glycol
solution flow. A controller is used to control the volumetric flow rate of the glycol and water
solutions so that there is always a 10°C temperature difference across the solar collector when
solar radiation is available. (An average temperature of 50°C can be assumed to evaluate
properties.) The volumetric flow rate of the water pump is always the same as that for the glycol
pump. The purpose of this problem is to determine the effectiveness of the heat exchanger as a
function of the volumetric flowrate.

shell-and-tube
heat exchanger
storage tank
collector

Vc
Vh

P8.3-6 Solar water heating sytem

a.) Calculate and plot the effectiveness of the heat exchanger for volumetric flowrates between
0.1 and 10 l/s. Explain the behavior of observed in this plot.

Enter the known information into EES.

$UnitSystem SI MASS RAD PA K J


$Tabstops 0.2 0.4 0.6 3.5 in

"known information"
V_dot_lps=0.04 [liter/s] "flow rate, in liter/s"
V_dot=V_dot_lps*convert(l/s,m^3/s) "flow rate"
N=40 [-] "number of tubes"
D_i=0.005 [m] "inner diameter of tubes"
L=1.25 [m] "length of tubes"
T_avg=converttemp(C,K,50 [C]) "average temperature to evaluate properties."
P_atm=101.3 [kPa]*convert(kPa,Pa) "atmospheric pressure"
h_bar_o=900 [W/m^2-K] "water-side heat transfer coefficient"

The Brineprop2 procedure is used to determine the properties of the glycol solution (ρg, cg, kg, μg,
and Prg). The specific heat capacity and density of water (cw and ρw) are determined using the
internal property routines in EES.
"determine properties of glycol soluatin and water"
CALL BRINEPROP2('PG',30 [%],converttemp(K,C,T_avg):FP,&
rho_glycol,c_glycol_kJkgK,k_glycol,mu_glycol, Pr_glycol)
c_glycol=c_glycol_kJkgK*convert(kJ/kg-K,J/kg-K) "glycol specific heat capacity"
c_water=SpecHeat(Water,T=T_avg,P=P_atm) "specific heat of water"
rho_water=density(Water,T=T_avg,P=P_atm) "density of water"

The mass flow rate of water and glycol are computed according to:

m w = V ρ w (1)

m g = V ρ g (2)

The capacitance rates are computed:

C w = m w cw (3)

C g = m g cg (4)

The minimum capacitance rate ( C min ) is obtained using the Min function in EES.

"determine mass flow rates and capacitance rates"


m_dot_water=V_dot*rho_water "mass flow rate of water with volumetric flow V_dot"
m_dot_glycol=V_dot*rho_glycol "mass flow rate of glycol solution"
C_dot_glycol=m_dot_glycol*c_glycol "capacitance rate of glycol"
C_dot_water=m_dot_water*c_water "capacitance rate of water"
C_dot_min=min(C_dot_glycol,C_dot_water) "minimum capacitance rate"

The Reynolds number of the glycol flow is computed according to:

4 m g
Reg = (5)
π N Di μ g

and the PipeFlow_N procedure is used to obtain the average Nusselt number ( Nu g ) and the heat
transfer coefficient is computed according to:

Nu g k g
hg = (6)
Di

The resistance to convection to the glycol is:

1
Rconv , g = (7)
π Di L hg
"determine the heat transfer coefficient for the glycol flowing in the tubes"
RelRough=0 [-] "assume smooth tubes"
Re_glycol=4*m_dot_glycol/(pi*N*D_i*mu_glycol) "Reynold's number for glycol solution in tubes"
call PipeFlow_N(Re_glycol,Pr_glycol,L/D_i,RelRough: Nusselt_T_glycol,Nusselt_H_glycol,f_glycol)
"determines Nusselt number and f on glycol side"
h_bar_i=Nusselt_H_glycol*k_glycol/D_i "heat transfer coefficient on glycol side"
R_conv_i=1/(N*pi*D_i*L*h_bar_i) "resistance to convection on the glycol side"

The resistance to convection on the water side is:

1
Rconv ,i = (8)
π Di L hi

The conductance is:

1
UA = (9)
Rconv ,i + Rconv , g

The number of transfer units is:

UA
NTU =  (10)
Cmin

The function HX is used to determine the heat exchanger effectiveness (ε).

"determine the UA and heat exchanger effectiveness"


R_conv_o=1/(N*pi*D_i*L*h_bar_o) "resistance to convection on the water side"
UA=1/(R_conv_i+R_conv_o) "conductance"
NTU=UA/C_dot_min "number of transfer units"
epsilon=HX('counterflow', NTU, C_dot_water, C_dot_glycol, 'epsilon') "access eff-NTU solution"

Figure 2 illustrates the effectiveness as a function of the flow rate. As the mass flow rate
increases, the number of transfer units decreases and the effectiveness drops. There is a
discontinuity where the flow transitions to turbulence.
0.45

0.4

0.35

0.3

Effectiveness
0.25

0.2

0.15

0.1

0.05

0
0.1 1 10 14
Volumetric flow rate (liter/s)
Figure 2: Effectiveness as a function of flow rate.
Problem 8.3-7: Supermarket refrigeration system
Ammonia is an ideal refrigerant in most respects, although it is an irritant at very low exposure
levels and toxic above 1000 ppm. It is also weakly flammable. For these reasons, ammonia has
not been used in supermarket refrigeration systems. It is possible, however, to confine the
ammonia to the equipment room if a heat exchanger is employed to cool a secondary refrigerant.
A 50% propylene glycol solution is heat exchanged with the ammonia, as shown in Figure P8.3-
7. The propylene glycol solution is distributed to food cases in the supermarket. The glycol
returns from the food cases at T5 = -9°C and leaves the heat exchanger at T6 = -15°C. The
anticipated load on the food cases is 45 kW. The propylene glycol solution is non-toxic and non-
flammable, eliminating the concerns with ammonia used in the supermarket. However, the use
of the heat exchanger results in a reduction in the efficiency of the refrigerating equipment and
extra pumping power. The ammonia flow rate is controlled with a thermostatic expansion valve
to provide saturated vapor leaving the heat exchanger at state (1). The condenser provides
saturated liquid at 40°C at state (3). The isentropic efficiency of the compressor is 0.72 and it
operates adiabatically. Neglect pressure losses in the refrigeration piping. A shell and tube heat
exchanger with 2 shell passes is employed, with the ammonia condensing in the shell and the
glycol solution flowing in the tubes.

outdoor air at 35°C

(3) (2)
compressor
condenser
valve
w comp
heat exchanger
(4) (1)

glycol solution to glycol solution from


refrigerated cases refrigerated cases
= − ° T = −9°C
T6 15 C 5

Figure P8.3-7 Supermarket refrigeration system

a.) Prepare a plot of the power required to operate the compressor and the temperature at state
(1) as a function of the conductance of the heat exchanger for values ranging from 1.5 kW/K
< UA < 30 kW/K.

Enter the information from the problem statement.

$UnitSystem SI MASS RAD PA K J


$Tabstops 0.2 0.4 0.6 3.5 in

"known information"
q_dot_load=45 [kW]*convert(kW,W) "required cooling load"
T[3]=converttemp(C,K,40[C]) "temperature at state 3"
T_glycol_in=converttemp(C,K,-9 [C]) "temperature of glycol entering the evaporator"
T_glycol_out=converttemp(C,K,-15 [C]) "temperature of glycol leaving the evaporator"
eta_compressor=0.72 [-] "isentropic efficiency of compressor"
x[1]=1 [-] "saturated vapor at evaporator outlet"
x[3]=0 [-] "saturated liquid at state 3"
UA=1.5 [kW/K]*convert(kW/K,W/K) "conductance"

The thermodynamic state (3) is completely specified by the quality and temperature. The
properties (p3, h3, and s3) are obtained using EES' built-in property routines:

"state 3 - leaving condenser"


P[3]=pressure(Ammonia,T=T[3],x=x[3]) "pressure at state 3 (and 2)"
h[3]=enthalpy(Ammonia,T=T[3],x=x[3]) "specific enthalpy at state 3"
s[3]=entropy(Ammonia,T=T[3],x=x[3]) "specific entropy at state 3"

The temperature of the ammonia in the evaporator is not known; therefore, the temperature of the
fluid leaving the valve, T4, is assumed. The valve is isenthalpic:

h4 = h3 (1)

The remaining properties at state 4 (p4 and s4) are obtained using EES' built-in property routines
based on the known enthalpy and temperature:

"state 4 - entering evaporator"


T[4]=converttemp(C,K,-20[C]) "assumed temperature - will be removed later"
h[4]=h[3] "isenthalpic valve"
P[4]=pressure(Ammonia,T=T[4],h=h[4]) "pressure"
s[4]=entropy(Ammonia,T=T[4],h=h[4]) "entropy"

Pressure loss across the evaporator is neglected:

p1 = p4 (2)

The remaining properties at state 1 (T1, h1, and s1) are obtained using EES' built-in property
routines based on the known pressure and quality. The mass flow rate of ammonia is obtained
from:

qload = m r ( h1 − h4 ) (3)

"state 1 - entering compressor"


P[1]=P[4] "neglect pressure drop in heat exchanger"
T[1]=temperature(Ammonia,P=P[1],x=x[1]) "temperature"
h[1]=enthalpy(Ammonia,P=P[1],x=x[1]) "specific enthalpy at state 1"
s[1]=entropy(Ammonia,P=P[1],x=x[1]) "specific entropy at state 1"
m_dot_R*(h[1]-h[4])=q_dot_load "energy balance on ammonia"

The pressure drop across the condenser is neglected:

p2 = p3 (4)

The entropy leaving a reversible compressor is equal to the inlet entropy:


s2,ideal = s1 (5)

The enthalpy leaving a reversible compressor (h2,ideal) is obtained using the entropy and pressure.
The minimum compressor power is obtained according to:

w ideal = m r ( h2,ideal − h1 ) (6)

The actual compressor power is higher, based on the efficiency:

w ideal
w = (7)
ηcompressor

The enthalpy leaving the actual compressor is obtained from an energy balance:

w
h2 = h1 + (8)
m r

The remaining properties at state 2 (T2 and s2) are obtained from the known pressure and
enthalpy:

"state 2 - leaving compressor"


P[2]=P[3] "neglect pressure loss across condenser"
s2_ideal=s[1] "ideal compressor is isentropic"
h2_ideal=enthalpy(Ammonia,P=P[2],s=s2_ideal) "enthalpy at exit of ideal compressor"
w_dot_ideal=m_dot_R*(h2_ideal-h[1]) "work for ideal compressor"
w_dot=w_dot_ideal/eta_compressor "work for actual compressor"
h[2]=h[1]+w_dot/m_dot_R "specific enthalpy at state 2"
T[2]=temperature(Ammonia,h=h[2],P=P[2]) "temperature at state 2"
s[2]=entropy(Ammonia,h=h[2],P=P[2]) "entropy"

The properties of the glycol solution (ρg, cg, kg, μg, and Prg) are evaluated at the average glycol
temperature using the BrineProp2 procedure. The mass flow rate of the glycol solutions is given
by an energy balance:

m g cg (Tg ,in − Tg ,out ) = qload (9)

The capacitance rate of the glycol is:

C g = m g cg (10)

The number of transfer units is defined according to (note that the glycol solution is the
minimum capacitance rate since the effective capacitance rate of the evaporating refrigerant is
infinite):
UA
NTU =  (11)
Cg

The effectiveness of the evaporator (ε) is obtained from the HX function.

"Calculate the properties and capacitance rates of the glycol solution and the ammonia"
T_avg=(T_glycol_in+T_glycol_out)/2 "average glycol temperature for property evaluation"
Call BRINEPROP2('PG',50 [%],converttemp(K,C,T_avg):FP_glycol, rho_glycol,c_glycol_kJkgK,
k_glycol,mu_glycol, Pr_glycol) "properties of the glycol solution"
c_glycol=c_glycol_kJkgK*convert(kJ/kg-K,J/kg-K) "specific heat capacity"
m_dot_glycol*c_glycol*(T_glycol_in-T_glycol_out)=q_dot_load "determines required glycol flow rate"
C_dot_glycol=m_dot_glycol*c_glycol "capacitance rate of glycol solution"
NTU=UA/C_dot_glycol "definition of NTU"
epsilon=HX('shell&tube_2', NTU, C_dot_glycol, 1e9 [W/K], 'epsilon') "effectiveness"

The problem is solved and the guess values updated. The assumed value of T4 is commented out
and the load is recalculated according to:

qload = ε C g (Tg ,in − T4 ) (12)

The COP of the system is:

qload
COP = (13)
w

q_dot_load=epsilon*C_dot_glycol*(T_glycol_in-T[4]) "recalculate rate of heat transfer"


COP=q_dot_load/w_dot "coefficient of performance for the cycle"

Figure 2 illustrates the temperature entering the compressor (the evaporating temperature) and
the compressor power as a function of conductance.
32500 260

30000
Evaporating temperature (K)

255
Compressor power (W)

27500 T1
250
25000
245
22500
240
20000

17500 wcompressor 235

15000 230
0 5000 10000 15000 20000 25000 30000
Conductance (W/K)
Figure 1: Compressor power and evaporating temperature as a function of UA.
b.) The shell and tube glycol-ammonia heat exchanger considered for this system has 12 parallel
circuits of tubing with an inner diameter of 0.0125 m. The major resistance is due to the heat
transfer coefficient on the glycol side of the heat exchanger. Calculate the required length of
tubing for each circuit and the minimum pump power required to move the glycol solution
through the heat exchanger as a function of the UA. What UA value would you recommend?

The geometry is entered in EES:

"b) Determine length and pump power"


d_tube=0.0125 [m] "inner tube diameter"
N_circuits=12 [-] "number of parallel circuits"

The Reynolds number is calculated according to:

4 m g
Re = (14)
N π dtube μ g

A value of the tube length, L, is assumed and the value of the average Nusselt number ( Nu ) is
obtained using the PipeFlow_N function in EES. The average heat transfer coefficient on the
glycol side is computed:

Nu k g
h= (15)
dtube

Re=4*(m_dot_glycol/N_circuits)/(pi*d_tube*mu_glycol) "Reynold's number"


L=10 [m] "tube length - assumed"
call PipeFlow_N(Re,Pr_glycol,L/d_tube,0 [-]: Nusselt_T_bar,Nusselt_H_bar,f_bar)
"determine Nusselt number; note L/D set to large number for developed flow"
h_bar=Nusselt_T_bar*k_glycol/d_tube "estimate of internal heat transfer coefficient"

The program is solved and the guess values are updated. The length of tube is recalculated
according to:

h N π dtube L = UA (16)

The mean velocity is computed according to:

4 m g
um = (17)
N ρ g π dtube
2

The pressure drop is computed according to:

ρ g um2 L
Δp = f (18)
2 dtube
The minimum pump power (with a 100% efficient pump) is:

π dtube
2
w pump = N um Δp (19)
4

The total power consumption is:

w total = w pump + w (20)

h_bar*N_circuits*pi*d_tube*L=UA "determines required length of flow circuits"


u_m=m_dot_glycol/N_circuits/rho_glycol/(pi/4*D_tube^2) "velocity of glycol in tubes"
DELTAP=f_bar*L/d_tube*(1/2)*rho_glycol*u_m^2 "pressure drop across heat exchanger"
w_dot_pump=N_circuits*u_m*(pi/4*D_tube^2)*DELTAP "required (minimum) pump power"
w_dot_total=w_dot_pump+w_dot "total power"

Figure 3 illustrates the pump, comporessor, and total power as a function of conductance. Figure
3 suggests that a conductance of 10 kW/K provides a compromise between compressor power
and pump power.
32500 7000

Minimum required pump power (W)


30000 6000
Compressor power (W)

27500 5000

25000 4000

22500 wpump 3000


wtotal

20000 wcompressor 2000

17500 1000

15000 0
0 5000 10000 15000 20000 25000 30000
Conductance (W/K)
Figure 3: Pump, compressor, and total power as a function of conductance.
Problem 8.3-8
Air at atmospheric pressure and Ta,in = 55°C flows with a velocity of ua = 15 m/s through a
finned circular tube heat exchanger having a geometry corresponding to compact heat exchanger
core fc_tubes_s80-38T. The fins are made of copper. The length of the heat exchanger matrix
in the flow direction is L = 0.4 m. R134a is flowing through the tubes at a saturation temperature
of Tr = -5°C. The R134a enters the heat exchanger at a quality of xin = 0.45 and exits at a quality
of xout = 0.65. The frontal area of the heat exchanger is Af = 0.2 m2 and the thickness of the tube
wall is thw = 0.9 mm. The tubes are plumbed in N = 10 parallel circuits.
a.) Determine the heat transfer coefficient on the air side

The input information is entered in EES:

$UnitSystem SI MASS RAD PA K J


$Tabstops 0.2 0.4 0.6 3.5 in

"known information"
A_f=0.2 [m^2] "assumed value of frontal area"
T_a_in=converttemp(C,K,55 [C]) "inlet air temperature"
P_atm=101300 [Pa] "atmospheric pressure"
L=0.2 [m] "length in the flow direction"
T_r=converttemp(C,K,-5 [C]) "refrigerant-side temperature"
x_in=0.45 [-] "inlet quality"
x_out=0.65 [-] "outlet quality"
th_w=0.9 [mm]*convert(mm,m) "tube wall thickness"
u_a=15 [m/s] "air velocity"
N=10 [-] "number of parallel circuits"

The geometric details of this heat exchanger (Do, finpitch, Dh, finthk, σ, α, and Afin/A) are
determined by a call to the compact heat exchanger library.

Call CHX_geom_finned_tube('fc_tubes_s80-38T': D_o, fin_pitch, D_h, fin_thk, sigma, alpha, A_fin\A)


"returns geometry factors for the HX"
"Nomenclature:
D_o is the diameter of the tubes
fin_pitch is the number of fins per meter on the tubing [1/m]
D_h is the hydraulic diameter for the air flow [m]
fin_thk is the fin thickness [m]
sigma is the ratio of the mininum free flow area to the frontal area [-]
alpha is the heat transfer surface area per unit volume of matrix [m^2/m^3]
A_fin\A is the ratio of the fin area to total area
"

The outlet air temperature, Ta,out, is assumed. The average air temperature is calculated and used
to compute the air properties (ρa, μa, ca, and Pra). The air density at the inlet and exit state (ρa,in
and ρa,out, respectively) are also computed.

T_a_out=converttemp(C,K,10 [C]) "guess value of air outlet temperature for


evaluation of properties"
T_avg=(T_a_in+T_a_out)/2 "average temperature for evaluation of properties"

"Calculate air properties and the Reynolds number"


rho_a_in=density(Air,T=T_a_in,P=P_atm) "density of air at inlet"
rho_a_out=density(Air,T=T_a_out,P=P_atm) "density of air at outlet"
rho_a=density(Air,T=T_avg,P=P_atm) "average density of the air"
mu_a=viscosity(Air,T=T_avg) "average viscosity of the air"
c_a=cp(Air,T=T_avg) "average specific heat of the air"
Pr_a=Prandtl(Air,T=T_avg) "average Prandtl number of the air"

The mass velocity of the air is:

ρ a ua
G= (1)
σ

The air mass flow rate is:

m a = Ga Af σ (2)

The rate of heat transfer is:

q = m a ca (Ta ,in − Ta ,out ) (3)

The Reynolds number on the air side is:

G Dh
Re = (4)
μa

G_a=rho_a*u_a/sigma "mass velocity"


m_dot_a=G_a*A_f*sigma "mass flow rate of air"
q_dot=m_dot_a*c_a*(T_a_in-T_a_out) "heat transfer rate"
Re_a=G_a*D_h/mu_a "Reynold's number"

The Compact_HX_ND function is used to obtain the friction factor (fa) and Colburn j factor (jH).
The heat transfer coefficient on the air-side is computed according to:

jH Ga ca
ha = 2
(5)
Pra 3

Call Compact_HX_ND('fc_tubes_s80-38T', Re_a: f_a,j_H)


"returns the friction factor and Colburn j factor"
h_bar_a=j_H*G_a*c_a/Pr_a^(2/3) "determine the heat transfer coefficient (Eqns 54-21 and 54-22)"

which leads to ha = 204 W/m2-K. The heat transfer coefficient can be obtained more directly
using the CHX_h_finned_tube procedure.

"Note that the dimensional Compact_HX relation could have also returned h_air, with less effort"
Call CHX_h_finned_tube('fc_tubes_s80-38T', m_dot_a, A_f, 'air', T_avg, P_atm:h_bar_a`)
which also leads to ha = 204 W/m2-K.

b.) Determine the frictional pressure drop on the air side.

The pressure drop is obtained according to:

Ga2 ⎡ 4 L ρ a ,in ⎛ ρ ⎞⎤
Δp = ⎢ fa + (1 + σ 2 ) ⎜ a ,in − 1⎟ ⎥ (6)
2 ρ a ,in ⎣⎢ Dh ρ a ⎜ρ ⎟
⎝ a ,out ⎠ ⎥⎦

DELTAP=G_a^2/(2*rho_a_in)*(f_a*4*L/D_h*rho_a_in/rho_a+(1+sigma^2)*(rho_a_in/rho_a_out-1))
"frictional pressure drop"

which leads to Δp = 1936 Pa. The pressure drop can be calculated approximately according to:

Ga2 4L
Δp = fa (7)
2 ρ a Dh

DELTAP`=G_a^2/2*f_a*4*L/D_h/rho_a "same but neglecting second term"

which leads to Δp = 2023 Pa.

c.) Estimate the fin efficiency.

The conductivity of copper (kf) is evaluated at the average temperature. The total surface area of
the heat exchanger on the air side is:

As = α L Af (8)

The surface area of the fins is:

Afin
Afin = As (9)
A

and the unfinned area is:

Aunfin = As − Afin (10)

"estimate the fin efficiency"


k_f=k_('Copper', T=T_avg) "thermal conductivity of copper"
A_s=alpha*L*A_f "total surface area"
A_fin=A_s*A_fin\A "finned surface area"
A_unfin=A_s-A_fin "unfinned surface area"

The total length of tube can be obtained from the unfinned area according to:
⎛ 1 ⎞
Ltube fin pitch ⎜ − finthk ⎟ π Do = Aunfin (11)
⎜ fin pitch ⎟
⎝ ⎠

The number of fins is:

N fin = Ltube fin pitch (12)

The effective radius of the fins (rfin) are computed in order that the annular fins occupy the same
surface area:

⎡ 2 ⎛ Do ⎞ 2 ⎤
N fin 2 π ⎢ rfin − ⎜ ⎟ ⎥ = Afin (13)
⎣⎢ ⎝ 2 ⎠ ⎦⎥

The fin efficiency (ηfin) is computed using the eta_fin_annular_rect function in EES.

L_tube*fin_pitch*(1/fin_pitch-fin_thk)*pi*D_o=A_unfin "length of tube"


N_fin=L_tube*fin_pitch "number of fins"
N_fin*2*pi*(r_fin^2-(D_o/2)^2)=A_fin "effective fin radius"
eta_fin=eta_fin_annular_rect(fin_thk, D_o/2, r_fin, h_bar_a, k_f) "fin efficiency"

which leads to ηfin = 0.90.

d.) Determine the average heat transfer coefficient on the R134a side

An energy balance on the refrigerant provides the mass flow rate:

m r ( ir ,out − ir ,in ) = q (14)

where ir,in and ir,out are the specific enthalpy of the refrigerant at the inlet and outlet, respectively.
The mass flux of refrigerant is:

m r
Gr = 2
(15)
⎛ th ⎞
N π ⎜ Do − w ⎟
⎝ 2 ⎠

and the heat flux is:

q
q ′′ = (16)
⎛ th ⎞
π ⎜ Do − w ⎟ Ltube
⎝ 2 ⎠
The refrigerant-side heat transfer coefficient ( hr ) is obtained from the Flow_boiling_avg
procedure.

"internal heat transfer coefficient"


i_r_in=enthalpy(R134a,T=T_r,x=x_in) "enthalpy of refrigerant entering heat exchanger"
i_r_out=enthalpy(R134a,T=T_r,x=x_out) "enthalpy of refrigerant exiting heat exchanger"
m_dot_r*(i_r_out-i_r_in)=q_dot "mass flow rate of refrigerant"
G_r=m_dot_r/(N*pi*(D_o-2*th_w)^2/4) "mass flux"
q``=q_dot/(L_tube*pi*(D_o-2*th_w)) "heat flux"
h_bar_r=Flow_Boiling_avg('R134a', T_r, G_r, D_o-2*th_w, x_in, x_out, q``, 'Horizontal')
"average flow boiling heat transfer coefficient"

e.) Determine the effectiveness of the heat exchanger and the rate of heat transfer.

The resistance to convection from the fins is:

1
R fin = (17)
ha Afin η fin

The resistance to convection from the unfinned area is:

1
Runfin = (18)
ha Aunfin

The resistance to convection to the refrigerant is:

1
Rr = (19)
hr π ( Do − 2 thw ) Ltube

The total resistance is:

−1
⎛ 1 1 ⎞
Rtotal = Rr + ⎜ + ⎟⎟ (20)
⎜R R
⎝ fin unfin ⎠

The conductance is:

1
UA = (21)
Rtotal

The number of transfer units is:

UA
NTU = (22)
m a ca
The heat exchanger effectiveness (ε) is computed using the HX function.

Rst_fin=1/(h_bar_a*A_fin*eta_fin) "resistance of fins"


Rst_unfin=1/(h_bar_a*A_unfin) "resistance of unfinned area"
Rst_r=1/(L_tube*pi*(D_o-2*th_w)*h_bar_r) "resistance to refrigerant"
Rst_total=Rst_r+(1/Rst_fin+1/Rst_unfin)^(-1) "total thermal resistance"
UA=1/Rst_total "conductance"
NTU=UA/(m_dot_a*c_a) "number of transfer units"
epsilon=HX('crossflow_one_unmixed', NTU, m_dot_a*c_a, 9e9 [W/K], 'epsilon')
"EES function for epsilon"

The problem is solved and the guess values are updated. The assumed value of Ta,out is
commented out and the heat transfer rate is calculated according to:

q = ε m a ca (Ta ,in − Tr ) (23)

{T_a_out=converttemp(C,K,10 [C])} "guess value of air outlet temperature for evaluation of properties"
q_dot=epsilon*m_dot_a*c_a*(T_a_in-T_r) "effectiveness-NTU method"

which leads to ε = 0.701 and q = 145.8 kW.

An average air temperature is calculated based on a guess for the outlet air temperature. This
guess will be removed later when the calculation loop is closed.

T_o=280 [K] "guess value T_o for evaluation of properties"


T_avg=(T_i+T_o)/2 "average temperature for evaluation of properties"

The properties of air are determined at the average temperature and used to calculated the
Reynolds number.
"Calculate air properties and the Reynolds number"
rho_i=density(Air,T=T_i,P=P_atm) "density of air at inlet"
rho_o=density(Air,T=T_o,P=P_atm) "density of air at outlet"
rho=density(Air,T=T_avg,P=P_atm) "average density of the air"
mu=viscosity(Air,T=T_avg) "average viscosity of the air"
c_p=cp(Air,T=T_avg) "average specific heat of the air"
Pr=Prandtl(Air,T=T_avg) "average Prandtl number of the air"
V_infinity=10 [m/s] "entering velocity"
G_air=rho*V_infinity/sigma "mass velocity"
m_dot_air=G_air*A_fr*sigma "mass flow rate of air"
Q_dot=m_dot_air*c_p*(T_i-T_o) "heat transfer rate"
Re_air=G_air*D_h/mu "Reynold's number"

A call to the compact heat exchanger library returns the friction factor and Colburn j-factor,
which allows the air side heat transfer coefficient and pressure drop to be determined.

Call Compact_HX_ND('fc_tubes_s80-38T', Re_air: f,j_H) "returns the friction factor and Colburn j factor"
h_air=j_H*G_air*c_p/Pr^(2/3) "determine the heat transfer coefficient "
DELTAP=G_air^2/(2*rho_i)*(f*4*L/D_h*rho_i/rho+(1+sigma^2)*(rho_i/rho_o-1)) "frictional pressure drop"
{"Note that the dimensional Compact_HX relation could have also returned h_air, with less effort"
Call CHX_h_finned_tube('fc_tubes_s80-38T', m_dot_air, A_fr, 'air', T_avg, P_atm:h_air)}

The efficiency of the copper fins can be determined from the fin geometry and used to calculate
the UA on the air side. The EES library function is used to estimate the efficiency of an annular
fin.

"estimate the fin efficiency"


k=k_('Copper', T=(T_i+T_o)/2) "thermal conductivity of copper"
eta_fin=eta_fin_annular_rect(fin_thk, D_o/2, r_fin, h_air, k) "fin efficiency from EES library function"
eta_o=1-A_fin\A*(1-eta_fin) "Overall fin efficiency"
UA_air=A_fr*L*alpha*h_air*eta_o "UA on air side"

Next, the heat transfer coefficient on the R134a side is estimated using the Flow_Boiling library.
The quality of the R134a at the entrance and exit is known and that can be used to calculate the
specific enthalpies of the inlet and outlet R134a. Then an energy balance fixes the mass flowrate
of the R134a. The heat transfer coefficient on the R134a side is used along with the coefficient
on the air side to calculate the UA of the heat exchanger.

"Estimate the heat transfer coefficient on the R134a side and the HX UA"
T_sat=convertTemp(C,K,-5 [C]) "saturation temperature of R134a"
x_i=0.45 "entering quality"
x_o=0.65 "exiting quality"
h_i=enthalpy(R134a,T=T_sat,x=x_i) "specific enthalpy of entering R134a"
h_o=enthalpy(R134a,T=T_sat,x=x_o) "specific enthalpy of exiting R134a"
m_dot_R134a*(h_o-h_i)=Q_dot "determines mass flow rate of R134a"
N_tubes=A_fr/(0.022 [m]*0.0254 [m]) "number of tubes in the frontal area - note the tube
spacing infomraiton is from the figure"
TubeWallThickness=0.0009 [m] "tube wall thickness"
D_i=D_o-2*tubeWallThickness "inner tube diameter"
x=0.55 "evaluate the heat transfer coefficient at the average quality"
G_R134a=m_dot_R134a/(N_tubes*pi/4*D_i^2) "liquid mass velocity of R134a - assumes wall
thickness of tube is small"
q``=Q_dot/(N_tubes*pi*D_i*L) "average heat flux"
call Flow_Boiling('R134a', T_sat, G_R134a, D_i, x, q``, 'Horizontal': h_R134a, T_w)
UA=1/(1/UA_air+1/(h_R134a*N_tubes*pi*D_i*L))

Update the guess values at this point. Next we calculate the effectiveness of the heat exchanger.
The heat transfer rate, Q_dot is specified in terms of the heat exchanger effectiveness. Since
Q_dot is already determined from an energy balance to be the product of the air capacitance rate
and the temperature difference between the inlet and outlet air, we now have a relation to
determine the air outlet temperature that was assumed. Comment out the guess for the air outlet
temperature and complete the problem with the heat exchanger relations.

{T_o=280 [K] "guess value T_o for evaluation of properties"}

"Estimate the effectiveness for this cross-flow heat exchanger"


NTU=UA/C_dot_min "NTU for this heat exchanger"
C_dot_min=m_dot_air*c_p "capacitance rate of the air"
C_dot_max=1e99 "the capacitance rate of the R134a is effectively infinite"
epsilon=HX('crossflow_one_unmixed', Ntu, C_dot_min, C_dot_max, 'epsilon') "EES function for epsilon"
Q_dot=epsilon*C_dot_min*(T_i-T_sat) "effectiveness-NTU method"

"Note: The last equation closes the loop in the calculation of T_o as it adds a heat exchanger relation to
the energy balance. Comment out T_o above when this last equation is added. "

Solving produces the following results:

DELTAP=2,907 [Pa] air pressure drop


Q_dot=655,295 [W] heat transfer rate to R134a
h_R134a=1,998 [W/m^2-K] R134a heat transfer coefficient in tubes
epsilon=0.9025 [-] heat exchanger effectiveness
h_air=160 [W/m^2-K] air side heat transfer coefficient
Problem 8.3-9 (8-9 in text)
A concentric tube heat exchanger is built and operated as shown in Figure P8.3-9. The hot
stream is a heat transfer fluid with specific heat capacity cH = 2.5 kJ/kg-K. The hot stream enters
at the center of the annulus at TH,in = 110°C with mass flow rate m H = 0.64 kg/s and then splits
and an equal amount flows in both directions. The cold stream has specific heat capacity cC =
4.0 kJ/kg-K. The cold enters the center pipe at TC,in = 10°C with mass flow rate m C = 0.2 kg/s .
The outlet temperature of the hot-fluid that flows towards the left is TH,out.x=0 = 45°C. The two
sections of the heat exchanger have the same conductance.

TH ,in = 110° C
m H = 0.64 kg/s
TH ,out , x = 0 = 45°C cH = 2.5 kJ/kg-K TH,out,x=L

TC ,in = 10°C
m C = 0.2 kg/s TC,out,x=L
cC = 4 kJ/kg-K TC,out,x=L/2
section 1 section 2
x
Figure P8.3-9: Concentric tube heat exchanger.

a.) Determine the temperature of the cold stream at the midpoint of the center tube and the
temperature of the cold stream leaving the heat exchanger.

The capacitance rates of the hot and cold flows are:

0.64 kg 2.5 kJ kW
C H = m H cH = = 1.6 (1)
s kg-s K

0.2 kg 4.0 kJ kW
C C = m C cC = = 0.8 (2)
s kg-s K

An overall energy balance on section 1 is:

C H
2
(TH ,in − TH ,out , x=0 ) = CC (TC ,out , x= L / 2 − TC ,in ) (3)

which can be solved for the TC,out,x=L/2:


C H
TC ,out , x = L / 2 = TC ,in +
2 C C
(TH ,in − TH ,out , x=0 )
(4)
1.6
TC ,out , x = L / 2 = 10°C + (110°C - 45°C ) = 75°C
2 ( 0.8 )

The effectiveness of section 1 is:

C C (TC ,out , x = L / 2 − TC ,in ) ( 75°C − 10°C ) = 0.65


ε1 = = (5)
(
MIN C , C / 2 (T − T
C H ) H ,in C ,in ) (110°C − 10°C )

The number of transfer units is obtained from the effectiveness-NTU solution in the book for a
counterflow heat exchanger with ε = 0.65 and CR = 1. Figure 8-14 is used to obtain NTU = 1.9.
The number of transfer units for the section 2 is the same as section 1 because the conductance
and capacitance rates are the same. The effectiveness for section 2 is obtained from Figure 8-15
with NTU = 1.9 and CR = 1, ε2 = 0.49. The heat transfer rate in section 2 is computed according
to:

( )
q2 = ε 2 MIN C C , C H / 2 (TH ,in − TC ,out , x = L / 2 )
0.49 0.8 kW (110°C − 75°C ) (6)
= = 13.7 kW
K

An energy balance on the cold fluid leads to:

q2 = C C (TC ,out , x = L − TC ,out , x = L / 2 ) (7)

which can be solved for TC,out,x=L:

q2 13.7 kW K
TC ,out , x = L = TC ,out , x = L / 2 + = 75°C+ = 92.1°C (8)

CC 0.8 kW

An energy balance on the hot fluid leads to:

C H
q2 =
2
(TH ,in − TH ,out , x= L ) (9)

which can be solved for TH,out,x=L:

q2 2 13.7 kW K
TH ,out , x = L = TH ,in − 2 = 110°C- = 92.9°C (10)
C H 1.6 kW

b.) Calculate the overall effectiveness of this heat exchanger considering both sections.
The overall effectiveness is:

C C (TC ,out , x = L − TC ,in )


ε=
(
MIN C , C (T − T
C H ) H ,in C ,in ) (11)
=
( 92.1 − 10 ) = 0.82
(110 − 10 )
c.) How will the overall effectiveness be affected if the inlet temperature is increased to 400°C.
(Assume that the properties of the heat transfer fluid are independent of temperature.)

There will be no change.

d.) Is the overall effectiveness of this heat exchanger higher, lower, or the same as a counter-
flow heat exchanger having the same inlet conditions? Justify your answer.

The overall effectiveness is less than a counter-flow heat exchanger because section 2 is parallel
flow.
Problem 8.3-10: Hot Water System
A factory requires 5000 gallons of hot (95°C) water each day. The water is not consumed;
rather, it is used to provide heating in one part of a process and therefore goes down the drain at
nominally 75°C. The process currently used to supply hot water to the factory is shown in
Figure P8.3-10(a). The factory runs a single, 10-hour shift each day and therefore provides the
hot water using a 5000 gallon supply tank. The supply tank is empty at the end of the shift and is
subsequently filled with city water at 10°C. The cold city water is heated electrically during the
14 hours between the end of the shift and the beginning of the subsequent shift using a 150 kW
heater. Therefore, the tank is full of 95°C water at the start of the shift and is emptied over the
course of the next shift.

to factory supply tank from city


water supply
electrical heater

supply tank is empty at the end of the shift

to factory from city


water supply

supply tank is filled with city water at 10°C

to factory from city


water supply

water in supply tank is heated to 95°C

to factory from city


water supply

supply tank is emptied during shift


Figure P8.3-10(a): Current hot water system for factory.

You have been asked to replace the current hot water system with an alternative system that
utilizes an energy recovery system in order to reduce the cost of the energy required to heat the
water. In the proposed system, a second 5000 gallon tank (the drain tank) will be used to store
the 75°C water that would otherwise go down the drain, as shown in Figure P8.3-10(b). At the
conclusion of the shift, the drain tank will contain 5000 gallons of 75°C water. This water has
been contaminated and therefore cannot be re-used directly. Therefore, a two phase water
preparation process is required. During the first phase, the water in the drain tank is used to pre-
heat the city water using a counterflow heat exchanger. The city water is heated in the
counterflow heat exchanger and enters the supply tank at some intermediate temperature that is
between 10°C and 75°C, depending on the total conductance (UA), of the counterflow heat
exchanger. During the second phase of the water preparation process, the electrical heater is
activated and used to heat the water to its final temperature, 95°C, so that the next shift can start.
counterflow heat exchanger

from factory to factory supply tank from city


water supply
drain tank
150 kW electrical heater
supply tank is empty at end of shift and drain tank is full of 75°C water
counterflow heat exchanger

from factory to factory supply tank from city


water supply
drain tank
150 kW electrical heater
during first phase, water is pumped through a counterflow heat exchanger to pre-heat city water and fills supply tank
counterflow heat exchanger

from factory to factory supply tank from city


water supply
drain tank
150 kW electrical heater
during second phase, water in supply tank is heated to 95°C
Figure P8.3-10(b): Proposed hot water system for factory.

Assume that the total conductance of the counterflow heat exchanger is UA = 6000 W/K and the
duration of the first phase of water preparation is tphaseI = 10 hr. Neglect any energy loss from
either the supply or drain tanks.
a.) What is the volumetric flow rate of city water required to completely fill the supply tank and
empty the drain tank (gal/min) during the first phase of water preparation?

The inputs are entered in EES:

$UnitSystem SI MASS RAD PA K J


$Tabstops 0.2 0.4 0.6 3.5 in

"known information"
Vg=5000 [gal]*convert(gal,m^3) "hot water tank capacity capacity"
T_city=converttemp(C,K,10 [C]) "city water temperature"
T_factory=convettemp(C,K,95 [C]) "temperature of water supplied to the factory"
T_drain=converttemp(C,K,75 [C]) "temperature of water down the drain"
time_offshift=14 [hr]*convert(hr,s) "time off shift available to prepare hot water tank"
q_dot_heater=150 [kW]*convert(kW,W) "electrical heater power"
time_phase1=10 [hr]*convert(hr,s) "time for 1st phase - recuperation"
UA=6000 [W/K] "UA of recuperative heat exchanger"

The density of water (ρ) is computed using EES' built-in property routine. The mass of water in
the tank is:

M = ρV (1)
The mass flow rate of water during phase 1 is:

M
m = (2)
t phaseI

rho=density(Water,T=T_city,P=100000 [Pa]) "density of water"


M=rho*V "mass of water"
m_dot=M/time_phase1 "mass flow rate of water"

The volumetric flow rate is:

m
V = (3)
ρ

V_dot=m_dot/rho "volumetric flow rate"


V_dot_gpm=V_dot*convert(m^3/s,gal/min) "in gpm"

which leads to V = 8.33 gal/min.

b.) What is the temperature of the water in the supply tank at the end of the first phase of water
preparation (°C)?

The number of transfer units is computed according to:

UA
NTU = (4)
m c

where c is the specific heat capacity of water. The heat exchanger effectiveness (ε) is obtained
using the HX command in EES. The intermediate temperature of the water is obtained from:

Tint = Tcity + (Tdrain − Tcity ) ε (5)

c=cP(Water,T=T_city,P=100000) "specific heat capacity of water"


NTU=UA/(m_dot*c) "number of transfer units"
eff=HX('counterflow', NTU, m_dot*c, m_dot*c, 'epsilon')
T_int=T_city+(T_drain-T_city)*eff "cold exit temp"
T_int_C=converttemp(K,C,T_int) "in C"

which leads to Tint = 57.6°C.

c.) If the 150 kW heater is activated at the conclusion of the first phase of water preparation and
remains on until the next shift starts (recall that the time between shifts is 14 hr), then what
will the final temperature of the water in the supply tank be (°C)?

The time remaining for phase 2 of the water preparation process is:
t phaseII = toffshift − t phaseI (6)

so the amount of energy added to the water by the heater (assuming no losses) is:

Qheater = t phaseII qheater (7)

The temperature at the conclusion of the water preparation process is computed using an energy
balance:

Qheater = ( u final − uint ) (8)

where uint is the internal energy of the water at Tint. The temperature of the water at the end of
the process (Tfinal) is obtained from the internal energy.

time_phase2=time_offshift-time_phase1 "time for heater to be on"


Q_heater=time_phase2*q_dot_heater "amount of energy added"
u_int=intenergy(Water,T=T_int,p=1 [atm]*convert(atm,Pa))
"internal energy at the intermediate temperature"
Q_heater=M*(u_final-u_int) "energy balance on the water"
T_final=temperature(Water,u=u_final,p=1 [atm]*convert(atm,Pa)) "final temperature"
T_final_C=converttemp(K,C,T_final) "final temperature in C"

which leads to Tfinal = 84.8°C.

d.) Prepare a plot showing the final temperature of the water in the supply tank (°C) as a
function of duration of the first phase of water preparation (tphaseI). What is the optimal
duration of the first phase of water preparation (i.e., what value of tphaseI provides water at
exactly 95°C for the next shift)?

Figure 3 illustrates the final water temperature as a function of the duration of phase I of the
water preparation process.
110

105

100

Final temperature (°C)


95

90

85

80

75

70

65

60
0 2 4 6 8 10 12 14
Time for phase I (hr)
Figure 3: Final water temperature as a function of the duration of phase I of the water treatment process.

Figure 3 shows that the optimal duration of phase is approximately 8 hr. By commenting out the
value of tphaseI and setting Tfinal = 95°C

{time_phase1_hr=10 [hr]} "time for 1st phase in hr"


T_final=T_factory "uncomment this and comment time_phase1=4 to have EES select the optimal time"

it is possible to determine the exact value, tphaseI = 8.08 hr.

e.) If the cost of electricity is 0.10$/kW-hr, then how much money will the proposed system
save per day as compared with the original system ($/day)? Make sure you set the duration
of the first phase of water preparation to the optimal value you obtained from (d).

The energy cost (ec) is entered in EES. The energy saved per day is:

Esavings = (Tint − Tcity ) M c (9)

and the savings per day is:

Savings = ec Esavings (10)

ecost=0.1 [$/kWhr] "energy cost"


Energyperday=M*c*(T_int-T_city)/1 [day] "energy saved per day"
Energyperday_kWhr=Energyperday*convert(J,kW-hr) "required energy per day"
Savings\day=Energyperday_kWhr*ecost "$ saved per day"

which leads to Savings = $98.4/day.

f.) The cost to install a new tank and pump and to modify the plumbing is estimated to be
$25,000. The cost of the counterflow heat exchanger is proportional to its total conductance;
the constant of proportionality is 10$/(W/K). Therefore, the heat exchanger with a total
conductance of 6,000 W/K will cost $60,000. Determine the simple payback (years) of the
proposed system. That is, neglecting the time value of money, how many years of savings
are required in order to exactly equal the initial outlay of capital cost for the system?

The initial cost (IC) and heat exchanger unit cost (HXcost) are entered in EES. The total cost of
the system is:

Cost = IC + HX cost UA (11)

The payback is computed based on the time required for the savings to equal the cost:

Cost
Payback = (12)
Savings

ICost=25000 [$] "initial fixed cost"


HXCost=10 [$-K/W] "heat exchanger cost per unit of UA"
Cost=ICost+UA*HXCost "initial cost"
payback_day=Cost/(Savings\day) "simple payback time"
payback_year=payback_day*convert(day,year) "simple payback time in years"

which leads to Payback = 863.8 days (2.37 years).

g.) Prepare a plot showing the payback (years) as a function of the total conductance of the heat
exchanger (W/K). Be sure that you are adjusting both the duration of the first phase of the
water preparation process and the cost of the system as you change UA. What is the optimal
value of UA?

Figure 4 illustrates the payback as a function of conductance and shows that the optimal
conductance is approximately 4100 W/K.
7

6
Payback (year)

2
1000 2000 3000 4000 5000 6000 7000 8000 9000 10000
Conductance (W/K)
Figure 4: Payback as a function of conductance.
Problem 8.3-11: Concentric Tube Heat Exchanger
Find the length of a concentric tube, counter-flow HX that is needed in order to cool oil entering
at a flow rate of 0.1 kg/s from 100ºC to 60ºC using water flowing at 0.1 kg/s that enters at 30ºC.
Determine the outlet temperature of the water. The total heat transfer coefficient per unit area,
U, is 60 W/m2-K, based on the inner tube area. The inner tube has a diameter of 2.5 cm. The
specific capacity of the oil is co = 2.13 kJ/kg-K and the specific heat capacity of the water is cw =
4.18 kJ/kg-K.

The known information is entered in EES:

$UnitSystem SI MASS RAD PA K J


$Tabstops 0.2 0.4 0.6 3.5 in

m_dot_o=0.1 [kg/s] "oil flow rate"


T_o_in=converttemp(C,K,100 [C]) "oil inlet temperature"
T_o_out=converttemp(C,K,60 [C]) "oil exit temperature"
m_dot_w=0.1 [kg/s] "water flowrate"
T_w_in=converttemp(C,K,30 [C]) "water inlet temperature"
U=60 [W/m^2-K] "heat transfer coefficient per unit area based on inner tube area"
D_i=2.5 [cm]*convert(cm,m) "inner tube diameter"

The capacitance rates of the oil and water are determined according to:

C o = m o co (1)

C o = m o co (2)

The minimum capacitance rate ( C min ) is identified.

C_dot_o=m_dot_o*c_o "capacitance rate of the oil"


C_dot_w=m_dot_w*c_w "capacitance rate of the water"
C_dot_min=min(C_dot_o,C_dot_w) "minimum capacitance rate"

The required heat transfer rate is:

q = C o (To,in − To ,out ) (3)

The required effectiveness is:

q
ε=  (4)
Co (To ,in − Tw,in )

q_dot=C_dot_o*(T_o_in-T_o_out) "heat transfer rate"


eff=q_dot/(C_dot_min*(T_o_in-T_w_in)) "effectiveness"
The number of transfer units (NTU) is obtained from the HX function in EES. The required
length is obtained from:

U π Di L = NTU C min (5)

NTU=HX('counterflow', eff, C_dot_o, C_dot_w, 'NTU') "get NTU from eff and capacitance rates"
U*pi*D_i*L=NTU*C_dot_min "required length"

which leads to L = 46.4 m.


Problem 8.3-12 (8-10 in text): Engine block and radiator
The power delivered to the wheels of a vehicle ( w ) as a function of vehicle speed (V) is given
⎡ hp ⎤ ⎡ hp ⎤ 2
by: w = −0.3937 [ hp ] + 0.6300 ⎢ ⎥ V + 0.01417 ⎢ 2⎥
V where power is in horsepower and
⎣ mph ⎦ ⎣ mph ⎦
velocity is in mph. The amount of heat rejected from the engine block ( qb ) is approximately
equal to the amount of power delivered to the wheel (the rest of the energy from the fuel leaves
with the exhaust gas). The heat is removed from the engine by pumping water through the
engine block with a mass flow rate of m = 0.80 kg/s. The thermal communication between the
engine block and the cooling water is very good, therefore you may assume that the water will
exit the engine block at the engine block temperature (Tb). For the purpose of this problem you
may model the water as having constant properties that are consistent with liquid water at 70°C.
The heat is rejected from the water to the surrounding air using a radiator, as shown in Figure
P8.3-12. When the car is moving, air is forced through the radiator due to the dynamic pressure
associated with the relative motion of the car with respect to the air. That is, the air is forced
through the radiator by a pressure difference that is equal to equal to ρa V2/2 where ρa is the
density of air. Assume that the temperature of the ambient air is T∞ = 35°C and model the air in
the radiator assuming that it has constant properties consistent with this temperature.

m = 0.8 kg/s
water exits at Tb

L = 10 cm qb
W = 50 cm

engine block

radiator
pf = 1.2 mm

air at T∞ = 35°C
Figure P8.3-12: Engine block and radiator.

The radiator has a plate-fin geometry. There are a series of tubes installed in closely spaced
metal plates that serve as fins. The fin pitch is pf = 1.2 mm and therefore there are W/pf plates
available for heat transfer. The heat exchanger core has overall width W = 50 cm, height H = 30
cm (into the page), and length (in the flow direction) of L = 10 cm. For the purpose of modeling
the air-side of the core you may assume that the air flow is consistent with an internal flow
through rectangular ducts with dimension H x pf. Assume that the fins are 100% efficient and
neglect the thermal resistance between the fluid and the internal surface of the tubes. Also
neglect convection from the external surfaces of the tubes as well as the reduction in the area of
the plates associated with the presence of the tubes.
a.) Using the information above, develop an EES model that will allow you to predict the engine
block temperature as a function of vehicle velocity. Prepare a plot showing Tb vs V and
explain the shape of the plot (if necessary, produce additional plots to help with your
explanation). If the maximum allowable temperature for the engine block is 100°C (in order
to prevent vaporization of the water) then what range of vehicle speeds are allowed? You
should see both a minimum and maximum limit.

The inputs are entered in EES:

$UnitSystem SI MASS RAD PA K J


$Tabstops 0.2 0.4 0.6 3.5 in

V_mph=10 [mph] "Vehicle velocity (mph)"


V=V_mph*convert(mph,m/s) "velocity in m/s"
a_0=-0.3937 [hp]*convert(hp,W) "coefficients for power curve"
a_1=0.63 [hp/mph]*convert(hp/mph,W-s/m)
a_2=0.01417 [hp/mph^2]*convert(hp/mph^2,W-s^2/m^2)
m_dot_w=0.8 [kg/s] "mass flow rate of water"
W=0.50 [m] "width of radiator (m)"
H=0.30 [m] "
L=0.10 [m] "length of radiator (m)"
p_f=0.0012 [m] "fin pitch (m)"
t_f=0.0002 [m] "fin thickness (m)"
T_infinity=converttemp(C,K,35 [C]) "ambient temperature"

The wheel power is computed according to the equation given in the problem statement and set
equal to the heat transfer.

w_dot=a_0+a_1*V+a_2*V^2 "power curve"


q_dot=w_dot "heat removed from engine"

The specific heat capacity of the water (cw) and the properties of the air (ca, ρa, μa, ka, αa, υa, and
Pra) are computed:

T_w=converttemp(C,K,70 [C]) "temperature for properties of water"


P_w=100000 [Pa] "atmospheric pressure"
c_w=cp(Water,T=T_w,P=P_w) "specific heat"

P_a=100000 [Pa] "air pressure"


c_a=cP(Air,T=T_infinity) "specific heat of air"
rho_a=density(Air,T=T_infinity,P=P_a) "density of air"
mu_a=viscosity(Air,T=T_infinity) "viscosity of air"
k_a=conductivity(Air,T=T_infinity) "thermal conductivity of air"
alpha_a=k_a/(rho_a*c_a) "thermal diffusivity of air"
nu_a=mu_a/rho_a "kinematic viscosity of air"
Pr_a=nu_a/alpha_a "Prandtl number of air"

The mass flow rate of the air ( m a ) is assumed. The hydraulic diameter and cross-sectional area
of the air passages are computed according to:

Dhyd = 2 ( p f − t f ) (1)
Ac = W H
(p f −tf ) (2)
pf

The mean velocity of the air in the duct is:

m a
ua = (3)
ρ a Ac

The Reynolds number is:

ua ρ a Dhyd
Re = (4)
μa

The DuctFlow_N procedure in EES is used to obtain the average Nusselt number, Nu , and
average friction factor, f .

m_dot_a=0.5 [kg/s] "guess for mass flow rate of air"


D_hyd=2*(p_f-t_f) "hydraulic diameter"
A_c=W*H*(p_f-t_f)/p_f "cross sectional area"
u_a=m_dot_a/(rho_a*A_c) "air velocity"
Re_a=u_a*rho_a*D_hyd/mu_a "Reynolds number for air"
call DuctFlow_N(Re_a,Pr_a,L/D_hyd, p_f/H,0 [-]: Nusselt_T_bar, Nusselt_H_bar, f_bar)

The pressure drop across the heat exchanger is computed according to:

f ρ a ua2 L
Δpa = (5)
2 Dhyd

The dynamic pressure rise created by the vehicle motion is:

ρa V 2
Δpdyn = (6)
2

DELTAP_a=0.5*f_bar*rho_a*u_a^2*(L/D_hyd) "pressure drop for air"


DELTAP_dyn=0.5*rho_a*V^2 "dynamic pressure produced by vehicle motion"

The problem is solved and the guess values are updated. The assumed value of m a is
commented out and the pressure drop across the heat exchanger is set equal to the dynamic
pressure rise.

{m_dot_a=0.5 [kg/s]} "guess for mass flow rate of air"


DELTAP_a=DELTAP_dyn "set dynamic pressure rise and pressure drop equal"
The average air-side heat transfer coefficient is computed according to:

ka
ha = Nu (7)
Dhyd

The conductance of the heat exchanger is:

W
UA = ha 2H L (8)
pf

h_bar_a=Nusselt_T_bar*k_a/D_hyd "heat transfer coefficient"


UA=h_bar_a*(W/p_f)*H*L*2 "conductance on the heat exchanger"

The capacitance rate of the air and water are computed according to:

C w = m w cw (9)

C a = m a ca (10)

The minimum capacitance rate ( C min ) is identified and the number of transfer units is computed:

UA
NTU =  (11)
Cmin

The effectiveness of the radiator (ε) is computed using the HX function. The engine block
temperature is computed according to:

q = ε C min (Tb − T∞ ) (12)

h_bar_a=Nusselt_T_bar*k_a/D_hyd "heat transfer coefficient"


UA=h_bar_a*(W/p_f)*H*L*2 "conductance on the heat exchanger"
C_dot_w=m_dot_w*c_w "capacitance rate of the water"
C_dot_a=m_dot_a*c_a "capacitance rate of the air"
C_dot_min=Min(C_dot_w,C_dot_a) "minimum capacitance rate"
NTU=UA/C_dot_min "number of transfer units"
eff=HX('crossflow_both_unmixed', NTU, C_dot_a, C_dot_w, 'epsilon') "effectiveness"
q_dot=eff*C_dot_min*(T_b-T_infinity) "solve for engine block temperature"
T_b_C=converttemp(K,C,T_b) "in C"

Figure 2 illustrates the engine block temperature as a function of vehicle speed and shows both a
lower speed limit where the dynamic pressure is insufficient to force a sufficient air flow through
the radiator as well as an upper speed limit where the heat transfer to the radiator is too high.
115
110

Engine block temperature (°C)


105
100
95
90
85
80
75
70
65
60
55
0 10 20 30 40 50 60 70 80
Vehicle velocity (mile/hr)
Figure 2: Engine block temperature as a function of vehicle velocity.

Its not easy to overcome the maximum speed limit identified (a); however, to overcome the
minimum speed limit (so that you can pull up to a stop sign without your car overheating) you
decide to add a fan. The fan can provide at most 500 cfm ( Vo - the open circuit flow) and can
produce at most 2.0 inch H2O (Δpdh - the dead-head pressure). The transition from open circuit to
⎛ V ⎞
dead-head is linear. The fan curve is given by: Δp fan = Δpdh ⎜1 − ⎟

⎝ Vo ⎠
b.) Modify your code to simulate the situation where the air is provided by the fan rather than
the vehicle motion. Overlay a plot showing Tb vs V for this configuration on the one from
(a); have you successfully overcome the lower speed limitation?

The fan characteristics are entered:

"air provided by fan"


V_dot_o_cfm=500 [cfm] "maximum velocity of fan"
V_dot_o=V_dot_o_cfm*convert(ft^3/min,m^3/s) "convert to SI"
DELTAP_dh_inH2O=2 [inH2O] "maximum pressure drop"
DELTAp_dh=DELTAP_dh_inH2O*convert(inH2O,Pa) "convert to SI"

The volumetric flow rate of air is computed:

m
Va = (13)
ρa

and the equation in the problem statement is used to compute the pressure rise caused by the fan:

V_dot_a=m_dot_a/rho_a "volumetric flow rate of air"


DELTAP_f=DELTAP_dh*(1-V_dot_a/V_dot_o) "fan curve"

The pressure drop across the heat exchanger is set equal to the pressure rise produce by the fan
rather than the dynamic pressure rise:
{DELTAP_a=DELTAP_dyn} "set dynamic pressure rise and pressure drop equal"
DELTAP_a=DELTAP_f "set equal to the pressure drop of the air"

Figure 3 illustrates the engine block temperature as a function of vehicle speed using the fan
overlaid onto Figure 2. Note that the addition of the fan maintains the engine at a reasonable
temperature even at very low speed.
115
110
Engine block temperature (°C)

105
100
95
90
85 without fan
80
75 with fan
70
65
60
55
0 10 20 30 40 50 60 70 80
Vehicle velocity (mile/hr)
Figure 3: Engine block temperature as a function of vehicle velocity with fan and without fan.
Problem 8.3-13: Counter-flow Heat Exchanger
A counter-flow heat exchanger is operating with a hot fluid inlet temperature of Th,in = 400 K and
a cold fluid inlet temperature of Tc,in = 300 K. The hot fluid flow has a capacity rate C h = 100
W/K and the cold flow has a capacity rate C = 50 W/K. The total conductance of the heat
c
exchanger is UA = 200 W/K.
a.) What is the heat transfer rate in the heat exchanger, q ?

The number of transfer units in the heat exchanger is:

UA 200 W/K
NTU =  = = 4.0 (1)
Cmin 50 W/K

and the capacity ratio is:

C 50 W/K
CR =  min = = 0.5 (2)
Cmax 100 W/K

according to Figure 8-14 of the text, the effectiveness of a counter-flow heat exchanger at NTU =
4.0 and CR = 0.5 is ε = 0.93. The maximum heat transfer rate in the heat exchanger is:

qmax = C min (Th ,in − Tc ,in ) = 50


W
( 400 - 300 ) K=5000 W (3)
K

so the heat transfer rate is:

q = ε qmax = ( 0.93) 5000 W=4650 W (4)

b.) What is the exit temperature of the cold fluid, Tc,out?

An energy balance on the cold fluid is:

q = C c (Tc ,out − Tc ,in ) (5)

so the exit temperature of the cold fluid is:

q 4650 W
Tc ,out = Tc ,in + = 300 K + = 393 K (6)
C c 50 W/K

c.) Sketch the temperature distribution within the counter-flow heat exchanger.
The temperature of the hot fluid must fall and the temperature of the cold fluid must rise in the
direction that they flow. The hot fluid has twice the heat capacity of the cold fluid and therefore
its temperature must change half as much. These characteristics are reflected in Figure 1.

Figure 1: Sketch of the temperature distribution within the counter-flow heat exchanger.

d.) The heat exchanger is re-plumbed so that the cold and hot fluid flow in the same direction
(i.e., in a parallel-flow configuration), what is the exit temperature of the cold fluid? All
other aspects of the problem remain the same.

The number of transfer units and capacity ratio remain the same. According to Figure 8-15 in the
text, the effectiveness of a parallel-flow heat exchanger at NTU = 4.0 and CR = 0.5 is ε = 0.66.
The maximum heat transfer rate in the heat exchanger is still qmax =5000 W . So the heat transfer
rate is:

q = ε qmax = ( 0.66 ) 5000 W=3300 W (7)

so the exit temperature of the cold fluid is:

q 3300 W
Tc ,out = Tc ,in +  = 300 K + = 366 K (8)
Cc 50 W/K

e.) Sketch the temperature distribution within the parallel-flow heat exchanger.
The temperature of the hot fluid must fall and the temperature of the cold fluid must rise in the
direction that they flow. The hot fluid has twice the heat capacity of the cold fluid and therefore
its temperature must change half as much. These characteristics are reflected in Figure 2.

Figure 2: Sketch of the temperature distribution within the parallel-flow heat exchanger.
Problem 8.3-14 (8-11 in text)
A parallel-flow heat exchanger has a total conductance UA = 10 W/K. The hot fluid enters at
Th,in = 400 K and has a capacity rate C h = 10 W/K. The cold fluid enters at Tc,in = 300 K and has
a capacity rate C = 5 W/K.
c

a.) Determine the number of transfer units (NTU), effectiveness (ε), heat transfer rate ( q ), and
exit temperatures (Th,out and Tc,out) for the heat exchanger.

The number of transfer units is calculated according to:

UA
NTU = = 2.0 (1)
(
MIN C c , C h )
The capacity ratio is defined as:

CR =
(
MIN C c , C h
= 0.5
) (2)
MAX C , C ( c h )
According to Figure 8-15 in the text, the effectiveness is ε = 0.64. The maximum possible heat
transfer rate for the heat exchanger is:

( )
qmax = MIN C c , C h (Th ,in − Tc ,in ) = 500 W (3)

The actual heat transfer is therefore:

q = ε qmax = 315 W (4)

The temperatures leaving the heat exchanger are computed according to:

q
Th ,out = Th ,in −  = 369 K (5)
Ch

q
Tc ,out = Tc ,in + = 363 K (6)
C c

b.) Sketch the temperature distribution within the heat exchanger.


Figure 1: Sketch of the temperature distribution within the heat exchanger.

The capacitance rate of the hot stream is twice that of the cold stream and therefore the
temperature change of the hot stream must be half that of the cold stream. The exit temperatures
were calculated in (a); the sketch is shown in Figure 1.

c.) Sketch the temperature distribution within the heat exchanger if the conductance of the heat
exchanger is very large; that is, what is the temperature distribution in the limit that UA → ∞.

Figure 2: Sketch of the temperature distribution within the heat exchanger as UA → ∞.


In the limit that UA → ∞ the temperatures of the two streams will meet, as shown in Figure 2.
Note that the temperature change of the hot stream is still 1/2 that of the cold stream and so the
effectiveness in this limit must be 2/3.

d.) Sketch how the hot exit temperature will change as the total conductance (UA) is varied, with
all other quantities held constant at the values listed in the problem statement. Be sure to
indicate how your plot behaves as UA approaches zero and as UA approaches infinity.

Figure 3: Sketch of the temperature distribution within the heat exchanger as UA → ∞.

The requested sketch is shown in Figure 3. If UA = 0 then the hot fluid temperature will not
change and so Th,out = 400 K. In the limit that UA → ∞, the hot fluid temperature will change by
its maximum possible amount based on the capacitance ratio. Figure 2 shows that temperature of
the hot fluid must change by an amount that is half that of the temperature change of the cold
fluid and so the hot fluid exit temperature cannot drop below 366.7 K.
Problem 8.3-15 (8-12 in text)
A heat exchanger has a core geometry that corresponds to finned circular tube core
'fc_tubes_s80_38T' in the compact heat exchanger library. The frontal area of the core has
dimensions W = 7.75 inch and H = 7.75 inch. The length of the core is L = 1.5 inch. The core is
integrated with a fan that has a head-flow curve given by: Δp = a − bVa where Δp is the pressure
rise across the fan, V is the volumetric flow rate of air, a = 0.3927 inH2O and b = 0.0021
a
inH2O/cfm are the coefficients of the fan curve. The manufacturer has tested the heat exchanger
with atmospheric air at Ta,in = 20ºC and water at Tw,in = 75ºC, pw = 65 psia flowing through the
tubes. The test data are shown in Table P8.3-15. The tubes are plumbed in series (i.e., all of the
water flows through each tube) and the tube thickness is tht = 0.035 inch.

Table P8.3-15: Manufacturer's data for heat exchanger.


Water flow Water outlet
rate temperature
0.13 gpm 44.3ºC
0.25 gpm 51.1ºC
0.5 gpm 60.1ºC
1 gpm 66.9ºC
2 gpm 70.8ºC
4 gpm 72.9ºC

a.) Develop a model using the effectiveness-NTU technique that can predict the outlet
temperature of the water for a water flow rate of the water Vw .

Enter the input information from the problem statement into EES.

$UnitSystem SI MASS RAD PA K J


$Tabstops 0.2 0.4 0.6 3.5 in

"Inputs"
W=7.75 [inch]*convert(inch,m) "width of core"
H=7.75 [inch]*convert(inch,m) "height of core"
L=1.5 [inch]*convert(inch,m) "length of core"
C$='fc_tubes_s80-38T' "identifier of core geometry"
a=0.3927 [inH2O]*convert(inH2O,Pa) "coefficient on fan curve"
b=0.0021 [inH2O-min/ft^3]*convert(inH2O-min/ft^3,Pa-s/m^3) "coefficient on fan curve"
T_w_in=converttemp(C,K,75 [C]) "inlet water temperature"
T_a_in=converttemp(C,K,20 [C]) "inlet air temperature"
p_w=65 [psi]*convert(psi,Pa) "water pressure"
V_dot_w_gpm=1 [gal/min] "water flow rate, in gpm"
V_dot_w=V_dot_w_gpm*convert(gal/min,m^3/s) "water flow rate"
p_a=1 [atm]*convert(atm,Pa) "air pressure"
N_tube=1 [-] "number of tube passes"
th_t=0.035 [inch]*convert(inch,m) "thickness of tube wall"

The average of the inlet temperatures are used to compute the properties of the water and the air.

T_a_avg=(T_w_in+T_a_in)/2 "air temperature to use for property calculations"


T_w_avg=(T_w_in+T_a_in)/2 "water temperature to use for property calculations"
The mass flow rate of air ( m a ) is guessed and used to calculate the volumetric flow rate of air is
computed according to:

m
Va = a (1)
ρa

The fan curve is used to compute the pressure rise generated by the fan (Δpf). The function
CHX_DELTAp_finned_tube is used to compute the pressure drop across the heat exchanger
(Δpa).

"Air flow rate"


m_dot_a=0.1 [kg/s] "mass flow rate of air - guess"
rho_a=density(Air,P=p_a,T=T_a_avg) "density of air"
V_dot_a=m_dot_a/rho_a "volumetric flow rate of air"
DELTAp_fan=a-b*V_dot_a "pressure rise produced by fan"
A_fr=W*H "frontal area"
Call CHX_DELTAp_finned_tube(C$, m_dot_a, A_fr,L, 'Air', T_a_in, T_a_in, p_a: DELTAp_a)
"compact heat exchanger correlation for DP"

The problem is solved and the guess values are updated. The guessed value of m a is commented
out and the pressure rise generated by the fan is set equal to the pressure drop across the heat
exchanger:

{m_dot_a=0.1 [kg/s]} "mass flow rate of air - guess"


DELTAp_fan=DELTAp_a "set pressure drop across core equal to pressure rise produced by fan"

The geometric characteristics of the heat exchanger (Do, finpitch, Dh, finthk, σ, α, and Afin/A) are
obtained using the procedure CHX_geom_finned_tube. The heat transfer coefficient on the air-side
( ha ) is calculated from the procedure CHX_h_finned_tube.

"Air-side resistance"
Call CHX_geom_finned_tube(C$: D_o, fin_pitch, D_h, fin_thk, sigma, alpha, A_fin\A)
"get geometry of core"
Call CHX_h_finned_tube(C$, m_dot_a, A_fr, 'Air',T_a_avg, p_a:h_bar_a)
"compact heat exchanger correlation for air-side heat transfer coefficient"

The fin efficiency can be estimated by treating the fin plate as an annular fin. This can be
accomplished by examining the core geometry from the Function Information Window from the
Compact Heat Exchanger library (Figure 1) which shows that the tube-to-tube distance is Ht =
25.4 mm.
Figure 1: Function Information Window for the Compact Heat Exchanger library.

The fin efficiency (ηfin) of an annular fin with an outer radius of Ht is obtained using the
eta_fin_annular_rect function.

H_t=20 [mm]*convert(mm,m) "center-to-center distance between tubes"


k_f=k_('Copper', T_a_avg) "conductivity of fin material"
eta_fin=eta_fin_annular_rect(fin_thk, D_o/2, H_t, h_bar_a, k_f) "fin efficiency estimate"

The thermal resistance of the finned surface area is:

1
R fin = (2)
Afin
ha W H L α η fin
A

The thermal resistance of the unfinned surface area is:

1
R fin = (3)
Afin
ha W H L α η fin
A

R_fin=1/(h_bar_a*W*L*H*alpha*A_fin\A*eta_fin) "finned surface area resistance"


R_unfin=1/(h_bar_a*W*L*H*alpha*(1-A_fin\A)) "unfinned surface area resistance"

The total length of tube in the heat exchanger is estimated according to:

⎛ 1 ⎞ ⎛ A ⎞
Ltotal fin pitch ⎜ − finthk ⎟ π Do = W L H α ⎜1 − fin ⎟ (4)
⎜ fin pitch ⎟ ⎝ A ⎠
⎝ ⎠
The length of tube per circuit is:

Ltotal
Ltube = (5)
N tube

The total mass flow rate of water is given by:

m = V ρ w (6)

The procedure PipeFlow is used to compute the water-side heat transfer coefficient ( hw ). The
thermal resistance to convection to the water is:

1
Rw = (7)
hw π Ltotal ( Do − 2 tht )

"Water-side resistance"
L_total*fin_pitch*(1/fin_pitch-fin_thk)*pi*D_o=W*L*H*alpha*(1-A_fin\A) "total length of tube"
L_tube=L_total/N_tube "length of each circuit"
rho_w=density(Water,p=p_w,T=T_w_avg) "density of water"
m_dot_w=rho_w*V_dot_w "mass flow of water"
call PipeFlow('Water',T_w_avg,p_w,m_dot_w/N_tube,D_o-2*th_t,L_tube,0[-]:h_bar_w, h_H ,&
DELTAP_w, Nusselt_T, f, Re) "access correlations for internal flow in a tube"
R_w=1/(L_total*pi*(D_o-2*th_t)*h_bar_w) "water-side resistance"

The fouling factor (FF) is obtained from the procedure FoulingFactor in EES. The fouling
resistance is computed according to:

FF
Rf = (8)
π Ltotal ( Do − 2 tht )

"Fouling resistance"
FF=FoulingFactor('Closed-loop treated water') "fouling factor"
R_foul=FF/(L_tube*pi*(D_o-2*th_t)) "fouling resistance"

The total resistance is:

−1
⎛ 1 1 ⎞
Rtotal = Rw + R f + ⎜ + ⎟⎟ (9)
⎜R
⎝ fin Runfin ⎠

The conductance is:

1
UA = (10)
Rtotal

"Heat exchanger calculations"


R_total=R_w+R_foul+(1/R_fin+1/R_unfin)^(-1) "total resistance"
UA=1/R_total "total conductance"

The capacitance rates of the water and air are computed:

C a = m a ca (11)

C w = m w cw (12)

The minimum capacitance rate ( C min ) is computed and used to determine the number of transfer
units:

UA
NTU =  (13)
Cmin

The effectiveness of the heat exchanger (ε) is determined using the HX function in EES.

c_a=cP(Air,T=T_a_avg) "air specific heat capacity"


c_w=cP(Water,T=T_w_avg,P=p_w) "water specific heat capacity"
C_dot_a=m_dot_a*c_a "air capacitance rate"
C_dot_w=m_dot_w*c_w "water capacitance rate"
C_dot_min=MIN(C_dot_a,C_dot_w) "minimum capacitance rate"
NTU=UA/C_dot_min "number of transfer units"
eff=HX('crossflow_both_unmixed', NTU, C_dot_a, C_dot_w, 'epsilon') "access eff-NTU solutions"

The maximum possible rate of heat transfer is:

qmax = C min (Tw,in − Ta ,in ) (14)

The actual rate of heat transfer is:

q = ε qmax (15)

The air and water outlet temperatures are computed according to:

q
Tw,out = Tw,in −  (16)
Cw

q
Ta ,out = Ta ,in +  (17)
Ca

"Energy balances to determine outlet temperatures"


q_dot_max=C_dot_min*(T_w_in-T_a_in) "maximum possible heat transfer rate"
q_dot=q_dot_max*eff "actual heat transfer rate"
T_w_out=T_w_in-q_dot/C_dot_w "water exit temperature"
T_w_out_C=converttemp(K,C,T_w_out) "in C"
T_a_out=T_a_in+q_dot/C_dot_a "air exit temperature"
T_a_out_C=converttemp(K,C,T_a_out) "in C"

b.) Plot the outlet temperature of the water as a function of the water flow rate and overlay the
manufacturer's data onto your plot.

Figure 2 illustrates the water exit temperature as a function of water flow rate. Overlaid on
Figure 2 is the manufacturer's data from Table 8.3-15.
75
70
Water exit temperature (°C)

65
60
55
50
45
40
35
manufacturer's data
30
model
25
20
0.1 1 10
Water flow rate (gal/min)
Figure 2: Water exit temperature as a function of water flow rate predicted by the model and from the
manufacturer's data.
Problem 8.3-16
An oil cooler for a small engine is a counterflow, tube-in-tube type heat exchanger that transfers
heat from hot oil leaving the engine to cold water. The water enters the annular space with
temperature Tc,in = 10ºC and mass flow rate m w = 0.02 kg/s. The oil enters the the center tube
with temperature To,in = 90ºC and mass flow rate m o = 0.004 kg/s. The outer diameter of the
outer tube is Dot,o = 0.75 inch and the outer diameter of the inner tube is Dot,i = 0.375 inch. The
length and wall thickness of both tubes are L = 48 inch and th = 0.035 inch, respectively. The
tubes are 304 stainless steel.
a.) Determine the outlet temperature of the oil as well as the heat transferred in the oil cooler
using Eq. (8-36) in the text.

The inputs are entered in EES:

$UnitSystem SI MASS RAD PA K J


$Tabstops 0.2 0.4 0.6 3.5 in

"Inputs"
m_dot_w=0.02 [kg/s] "flow rate of cooling water"
T_c_in=converttemp(C,K,10 [C]) "inlet temperature of cooling water"
m_dot_o=0.004 [kg/s] "flow rate of oil"
T_h_in_C=90 [C] "inlet temperature of oil, in C"
T_h_in=converttemp(C,K,T_h_in_C) "inlet temperature of oil"
D_ot_o=0.75 [inch]*convert(inch,m) "outer diameter of outer tube"
D_it_o=0.375 [inch]*convert(inch,m) "outer diameter of inner tube"
L_inch=48 [inch] "length of tube, in inch"
L=L_inch*convert(inch,m) "length of tube"
th=0.035 [inch]*convert(inch,m) "thickness of both tubes"

The outlet temperature of the cooling water, Tc,out, is assumed and the average cooling water
temperature is used to compute the properties ρc, cc, μc, kc, and Prc.

"Determine conductance"
"Cooling water-side"
T_c_out=converttemp(C,K,35 [C]) "guess for cooling water outlet temperature"
T_c_avg=(T_c_out+T_c_in)/2 "average cooling water temperature"
rho_c=density(Water,T=T_c_avg,P=Po#) "cooling water density"
c_c=cP(Water,T=T_c_avg,P=Po#) "cooling water specific heat capacity"
mu_c=viscosity(Water,T=T_c_avg,P=Po#) "cooling water viscosity"
k_c=conductivity(Water,T=T_c_avg,P=Po#) "cooling water conductivity"
Pr_c=mu_c*c_c/k_c "cooling water Prandtl number"

The inner diameter of the outer tube is:

Dot ,i = Dot ,o − 2 th (1)

The cross-sectional area for flow in the annulus is:

π
Ac ,c =
4
(D 2
ot ,i − Dit2,o ) (2)
and the wetted perimeter for flow in the annulus is:

perc = π ( Dot ,i + Dit ,o ) (3)

The hydraulic diameter of the annulus is:

4 Ac ,c
Dh ,c = (4)
perc

D_ot_i=D_ot_o-2*th "inner diameter of outer tube"


A_c_c=pi*(D_ot_i^2-D_it_o^2)/4 "cross-sectional area of annulus"
per_c=pi*(D_ot_i+D_it_o) "perimeter of annulus"
D_h_c=4*A_c_c/per_c "hydraulic diameter of annulus"

The mean velocity of water is:

m w
um , c = (5)
Ac ,c ρ c

The Reynolds number of the water flow is:

um ,c Dh ,c ρ c
Rec = (6)
μc

The procedure Annular_Flow_N is used to determine the average Nusselt number on the water side
( Nu c - note that the constant temperature solution is used). The heat transfer coefficient is:

Nu c kc
hc = (7)
Dh ,c

and the resistance to convection on the water side is:

1
Rconv ,c = (8)
hc π Dit ,o L

u_m_c=m_dot_w/(rho_c*A_c_c) "mean velocity"


Re_c=u_m_c*D_h_c*rho_c/mu_c "Reynolds number"
call AnnularFlow_N(Re_c, Pr_c, L/D_h_c, D_it_o/D_ot_i, 0 [-]: &
Nusselt_bar_T_c,Nusselt_bar_H_c, f_bar_c)
"access correlations for flow through an annulus"
h_bar_c=Nusselt_bar_T_c*k_c/D_h_c "average heat transfer coefficient on water side"
R_conv_c=1/(h_bar_c*pi*D_it_o*L) "resistance to convection on water side"
A similar procedure is followed on the oil side. The oil outlet temperature, Th,out, is assumed and
the properties of oil are computed using the average temperature (ρh, ch, μh, kh, and Prh - note
that oil is assumed to behave as unused engine oil).

"Oil-side"
T_h_out=converttemp(C,K,35 [C]) "guess for oil outlet temperature"
T_h_avg=(T_h_out+T_h_in)/2 "average oil temperature"
rho_h=rho_('Engine Oil-unused', T_h_avg) "oil density"
c_h=c_('Engine Oil-unused', T_h_avg) "oil specific heat capacity"
mu_h=mu_('Engine Oil-unused', T_h_avg) "oil viscosity"
k_h=k_('Engine Oil-unused', T_h_avg) "oil conductivity"
Pr_h=mu_h*c_h/k_h "oil Prandtl number"

The inner diameter of the inner tube is:

Dit ,i = Dit ,o − 2 th (9)

The cross-sectional area for flow in the inner tube is:

π
Ac ,h = Dit2,i (10)
4

The mean velocity of oil is:

m o
um , h = (11)
Ac ,h ρ h

The Reynolds number of the oil flow is:

um ,h Dit ,i ρ h
Reo = (12)
μh

The procedure PipeFlow_N is used to determine the average Nusselt number on the oil side ( Nu h -
note that the constant temperature solution is used again). The heat transfer coefficient is:

Nu h kh
hh = (13)
Dit ,i

and the resistance to convection on the oil side is:

1
Rconv ,h = (14)
hh π Dit ,i L

D_it_i=D_it_o-2*th "inner diameter of inner tube"


A_c_h=pi*D_it_i^2/4 "cross-sectional area of inner tube"
u_m_h=m_dot_o/(rho_h*A_c_h) "mean velocity"
Re_h=u_m_h*D_it_i*rho_h/mu_h "Reynolds number"
call PipeFlow_N(Re_h,Pr_h,L/D_it_i,0 [-]: Nusselt_bar_T_h,Nusselt_bar_H_h,f_bar_h)
"access correlations for flow through a tube"
h_bar_h=Nusselt_bar_T_h*k_h/D_it_i "average heat transfer coefficient on oil side"
R_conv_h=1/(h_bar_h*pi*D_it_i*L) "resistance to convection on oil side"

The area-specific fouling factors on the water and oil-sides ( Rc′′ and Rh′′ ) are determined from the
EES function FoulingFactor assuming that water behaves as city water and oil as engine
lubrication oil. The fouling resistances are:

Rc′′
R foul ,c = (15)
π Dit ,i L

Rh′′
R foul , h = (16)
π Dit ,o L

The conductivity of stainless steel (ktube) is evaluated at the average temperature within the heat
exchanger and used to compute the resistance to conduction through the tube:

⎛D ⎞
ln ⎜⎜ it ,o ⎟⎟
Dit ,i ⎠
Rcond = ⎝ (17)
2 π ktube L

"Fouling and conduction"


F_c=FoulingFactor('City or well water') "fouling factor on water side"
R_foul_c=F_c/(pi*D_it_o*L) "fouling resistance on water side"
F_h=FoulingFactor('Engine lube oil') "fouling resistance on oil side"
R_foul_h=F_h/(pi*D_it_i*L) "fouling resistance on oil side"
k_tube=k_('Stainless_AISI304', (T_h_in+T_c_in)/2) "conductivity of tube"
R_cond=ln(D_it_o/D_it_i)/(2*pi*k_tube*L) "resistance to conductivity through tube"

The total resistance is:

Rtotal = Rconv ,c + R foul ,c + Rcond + R foul , h + Rconv , h (18)

and the conductance is:

1
UA = (19)
Rtotal

The capacitance rates on both sides are:

C c = m w cc (20)
C h = m o ch (21)

"Performance"
R_total=R_conv_c+R_foul_c+R_cond+R_foul_h+R_conv_h "total resistance"
UA=1/R_total "conductance"
C_dot_h=m_dot_o*c_h "hot-side capacitance rate"
C_dot_c=m_dot_w*c_c "cold-side capacitance rate"

The guess values are updated and the assumed values of Tc,out and Th,out are commented out. The
solution derived in the text for a counterflow heat exchanger:

⎛ T −T ⎞ ⎛ 1 1 ⎞
ln ⎜ h ,out c ,in ⎟ = −UA ⎜  −  ⎟ (22)
⎜ T −T ⎟
⎝ h ,in c ,out ⎠ ⎝ Ch Cc ⎠

q = C h (Th,in − Th,out ) (23)

q = C c (Tc ,out − Tc ,in ) (24)

{T_c_out=converttemp(C,K,35 [C])} "guess for cooling water outlet temperature"


{T_h_out=converttemp(C,K,35 [C])} "guess for oil outlet temperature"
ln((T_h_out-T_c_in)/(T_h_in-T_c_out))=-UA*(1/C_dot_h-1/C_dot_c) "rate equation"
q_dot=C_dot_h*(T_h_in-T_h_out) "energy balance on hot side"
q_dot=C_dot_c*(T_c_out-T_c_in) "energy balance on cold side"
T_h_out_C=converttemp(K,C,T_h_out) "oil outlet temperature, in C"

which leads to Th,out = 67.9ºC and q = 187.8 W.

b.) Determine the performance of the heat exchanger using the ε-NTU solution and show that the
result is consistent with (a).

The minimum capacitance rate, C min , is identified using the MIN function in EES. The number of
transfer units in the heat exchanger is:

UA
NTU =  (25)
Cmin

The effectiveness, ε, is determined using the ε-NTU solutions accessed from the HX function in
EES. The maximum possible heat transfer rate is computed according to:

qmax = C min (Th ,in − Tc ,in ) (26)

and the actual heat transfer rate is:

q = ε qmax (27)
"Prediction with eff-NTU method"
C_dot_min=MIN(C_dot_c,C_dot_h) "minimum capacitance rate"
NTU=UA/C_dot_min "number of transfer units"
eff=HX('counterflow', NTU, C_dot_c, C_dot_h, 'epsilon') "effectiveness"
q_dot_max=C_dot_min*(T_h_in-T_c_in) "maximum possible heat transfer rate"
q_dot_effNTU=eff*q_dot_max "predicted performance from eff-NTU method"

which leads to q = 187.8 W, the same answer as we obtained in (a).

c.) Plot the rate of heat transfer as a function of the oil inlet temperature. If the engine rejects
200 W to the oil then what will the oil outlet temperature be?

Figure 1 illustrates the heat transfer rate as a function of the oil inlet temperature. If the engine
rejects 200 W to the oil then the oil inlet temperature will be approximately 95ºC.
350

300
Heat transfer rate (W)

250

200

150

100

50
50 75 100 125 150
Oil inlet temperature (°C)
Figure 1: Heat transfer rate as a function of the oil inlet temperature.
Problem 8.3-17
In Problem 8.3-15 (8-12 in the text) a heat exchanger with a core geometry that corresponds to
finned circular tube core 'fc_tubes_s80_38T' in the compact heat exchanger library was modeled
from first principles and the result compared to the manufacturer's data. An alternative method
of making an engineering model of the heat exchanger is to use one data point to determine the
conductance of the heat exchanger at a nominal operating condition and then model the heat
exchanger by assuming that the conductance is nominally constant (or varies in some systematic
way).

The frontal area of the core has dimensions W = 7.75 inch and H = 7.75 inch. The length of the
core is L = 1.5 inch. The core is integrated with a fan that has a head-flow curve given by:
Δp = a − bVa where Δp is the pressure rise across the fan, Va is the volumetric flow rate of air, a
= 0.3927 inH2O and b = 0.0021 inH2O/cfm are the coefficients of the fan curve. The
manufacturer has tested the heat exchanger with atmospheric air at Ta,in = 20ºC and water at Tw,in
= 75ºC, pw = 65 psia flowing through the tubes. The test data are shown in Table P8.3-15.

a.) Determine the conductance associated with the data point at Vw = 1 gpm.

The inputs are entered in EES:

$UnitSystem SI MASS RAD PA K J


$Tabstops 0.2 0.4 0.6 3.5 in

"Inputs"
W=7.75 [inch]*convert(inch,m) "width of core"
H=7.75 [inch]*convert(inch,m) "height of core"
L=1.5 [inch]*convert(inch,m) "length of core"
C$='fc_tubes_s80-38T' "identifier of core geometry"
a=0.3927 [inH2O]*convert(inH2O,Pa) "coefficient on fan curve"
b=0.0021 [inH2O-min/ft^3]*convert(inH2O-min/ft^3,Pa-s/m^3) "coefficient on fan curve"
T_w_in=converttemp(C,K,75 [C]) "inlet water temperature"
T_a_in=converttemp(C,K,20 [C]) "inlet air temperature"
p_w=65 [psi]*convert(psi,Pa) "water pressure"
V_dot_w_gpm=1 [gal/min] "water flow rate, in gpm"
V_dot_w=V_dot_w_gpm*convert(gal/min,m^3/s) "water flow rate"
p_a=1 [atm]*convert(atm,Pa) "air pressure"

The properties of air and water are computed at the average temperature within the heat
exchanger.

T_a_avg=(T_w_in+T_a_in)/2 "air temperature to use for property calculations"


T_w_avg=(T_w_in+T_a_in)/2 "water temperature to use for property calculations"

The mass flow rate of air ( m a ) is guessed and used to calculate the volumetric flow rate of air is
computed according to:

m
Va = a (1)
ρa
The fan curve is used to compute the pressure rise generated by the fan (Δpf). The function
CHX_DELTAp_finned_tube is used to compute the pressure drop across the heat exchanger
(Δpa).

"Air flow rate"


m_dot_a=0.1 [kg/s] "mass flow rate of air - guess"
rho_a=density(Air,P=p_a,T=T_a_avg) "density of air"
V_dot_a=m_dot_a/rho_a "volumetric flow rate of air"
DELTAp_fan=a-b*V_dot_a "pressure rise produced by fan"
A_fr=W*H "frontal area"
Call CHX_DELTAp_finned_tube(C$, m_dot_a, A_fr,L, 'Air', T_a_in, T_a_in, p_a: DELTAp_a)
"compact heat exchanger correlation for DP"

The problem is solved and the guess values are updated. The guessed value of m a is commented
out and the pressure rise generated by the fan is set equal to the pressure drop across the heat
exchanger:

{m_dot_a=0.1 [kg/s]} "mass flow rate of air - guess"


DELTAp_fan=DELTAp_a "set pressure drop across core equal to pressure rise produced by fan"

The specific heat capacity of the air, ca, is computed and used to determine the air-side
capacitance rate:

C a = m a ca (2)

c_a=cP(Air,T=T_a_avg) "air specific heat capacity"


C_dot_a=m_dot_a*c_a "air capacitance rate"

The density and specific heat capacity of water, ρw and cw, are computed and used to compute
the mass flow rate and capacitance rate on the water-side:

m w = ρ w Vw (3)

C w = m w cw (4)

The minimum capacitance rate, C min , is identified using the Min command.

"Water flow rate"


rho_w=density(Water,p=p_w,T=T_w_avg) "density of water"
c_w=cP(Water,T=T_w_avg,P=p_w) "water specific heat capacity"
m_dot_w=rho_w*V_dot_w "mass flow of water"
C_dot_w=m_dot_w*c_w "water capacitance rate"
C_dot_min=MIN(C_dot_a,C_dot_w) "minimum capacitance rate"

The heat transfer rate is computed based on the data:


q = C w (Tw,in − Tw,out ) (5)

where Tw,out is the measured water outlet temperature. The maximum heat transfer rate is:

qmax = C max (Tw,in − Ta ,in ) (6)

The effectiveness measured by the test is:

q
ε= (7)
qmax

The number of transfer units required to give the measured effectiveness (NTU) is obtained using
the HX function in EES. The conductance associated with the nominal operating condition is:

UAnom = NTU C min (8)

"Determine UA"
T_w_out=converttemp(C,K,66.9 [C]) "water outlet temperature"
q_dot=C_dot_w*(T_w_in-T_w_out) "heat transfer rate"
q_dot_max=C_dot_min*(T_w_in-T_a_in) "maximum possible heat transfer"
eff=q_dot/q_dot_max "effectiveness"
NTU=HX('crossflow_one_unmixed', eff, C_dot_a, C_dot_w, 'NTU') "access solution"
UA_nom=NTU*C_dot_min "conductance at nominal conditions"

which leads to UAnom = 56.2 W/K.

b.) Assume that the conductance does not change with water flow rate in order to develop a
simple model of the heat exchanger. Plot the outlet temperature of the water predicted by the
model as a function of water flow rate and compare with the data.

The code associated with computing UAnom from the data is commented out and UAnom is set:

{"Determine UA"
T_w_out=converttemp(C,K,66.9 [C]) "water outlet temperature"
q_dot=C_dot_w*(T_w_in-T_w_out) "heat transfer rate"
q_dot_max=C_dot_min*(T_w_in-T_a_in) "maximum possible heat transfer"
eff=q_dot/q_dot_max "effectiveness"
NTU=HX('crossflow_one_unmixed', eff, C_dot_a, C_dot_w, 'NTU') "access solution"
UA_nom=NTU*C_dot_min "conductance at nominal conditions"}

"assume UA is constant"
UA_nom=56.2 [W/K] "conductance at nominal conditions"

The number of transfer units is computed assuming that UA is constant:

UA
NTU =  nom (9)
Cmin
The effectiveness predicted by the simple model, ε, is determined using the HX function in EES.
The maximum heat transfer rate is:

qmax = C max (Tw,in − Ta ,in ) (10)

and the actual heat transfer rate is:

q = ε qmax (11)

The water outlet temperature is:

q
Tw,out = Tw,in − (12)

Cw

Figure 1 illustrates the predicted and measured water outlet temperature as a function of water
flow rate. Notice that the agreement is exact at Vw = 1 gpm (because UAnom was determined in
order to match this data point) but the discrepancy grows as the operating condition deviates
from the nominal condition.
75
70
Water exit temperature (°C)

65
60
55
50
45
40
35
manufacturer's data
30
model
25
20
0.1 1 10
Water flow rate (gal/min)
Figure 1: Predicted and measured water exit temperature as a function of the water flow rate.
Problem 8.3-18
A steam coil is used to provide heating to a zone of a building. The coil has geometry consistent
with finned tube fc_tubes_sCF-775-58T in the compact heat exchanger library. The coil is
installed in a square air duct with width W = 18 inch and height H = 18 inch. The coil can be
installed in modules that are placed in series. The length of each module in the flow direction is
Lmod = 6 inch. The volumetric flow rate of air entering the coil is Vin = 900 cfm. The air enters
the coil at Ta,in = -5ºC. The condensing steam in the coil is two-phase throughout and at pressure
ps = 10 psi. The temperature of the zone is Tzone = 20ºC. Air is heated in the steam coil and
enters the zone at a temperature greater than Tzone. The air leaves the zone at Tzone and therefore
provides heating to the zone. The blower that moves the air through the coil has an efficiency of
ηb = 0.55. You may neglect fouling and assume that the efficiency of the fins on the air-side is
100%. Further, you may neglect the convection resistance associated with the condensing steam
and conduction through the tubes.
a.) If only one module is installed, determine the power required by the blower, the temperature
of the air leaving the steam coil, and the heat provided to the zone.

The inputs are entered in EES:

$UnitSystem SI MASS RAD PA K J


$Tabstops 0.2 0.4 0.6 3.5 in

"Inputs"
W=18 [inch]*convert(inch,m) "width of core"
H=18 [inch]*convert(inch,m) "height of core"
L_mod=6 [inch]*convert(inch,m) "length of core module"
N_mod=1 [-] "number of modules"
V_dot_in=900 [ft^3/min]*convert(ft^3/min,m^3/s) "flow rate of air entering coil"
p_s=10 [psi]*convert(psi,Pa) "steam pressure"
T_a_in=converttemp(C,K,-5 [C]) "air temperature entering coil"
T_zone=converttemp(C,K,20[C]) "zone temperature"
eta_b=0.55 [-] "efficiency of blower"

The density and specific heat capacity of air (ρa,in and ca) are calculated at the inlet temperature.
The density is used to determine the air mass flow rate and capacitance rate:

m a = ρ a ,in Vin (1)

C a = m a ca (2)

rho_a_in=density(Air,T=T_a_in,P=Po#) "density of air entering coil"


c_a=cP(Air,T=T_a_in) "specific heat capacity of air"
m_dot_a=rho_a_in*V_dot_in "mass flow rate of air"
C_dot_a=m_dot_a*c_a "capacitance rate of air"

The geometric parameters that characterize the core are obtained using the
CHX_geom_finned_tube procedure in EES. The volume specific surface area, α, is the parameter
that will be used subsequently. The total length of the steam coil in the flow direction is
obtained according to:

Ltotal = N mod Lmod (3)

where Nmod is the number of modules installed.

Call CHX_geom_finned_tube('fc_tubes_sCF-775-58T': D_o, fin_pitch, D_h, fin_thk, sigma, alpha, A_fin\A)


"get geometric parameters that characterize core"
L_total=N_mod*L_mod "total length of heat exchanger"

The outlet air temperature, Ta,out, is assumed in order to proceed. The procedure
CHX_DELTAp_finned_tube is used to determine the pressure drop across the coil, Δpa. The blower
power is determined according to:

Vin Δpa
w b = (4)
ηb

T_a_out=converttemp(C,K,35 [C]) "guess for air outlet temperature"


Call CHX_DELTAp_finned_tube('fc_tubes_sCF-775-58T', m_dot_a, W*H,L_total, 'Air', T_a_in, &
T_a_out, Po#: DELTAp_a) "get pressure drop across core"
w_dot_b=V_dot_in*DELTAp_a/eta_b "blower power"

The average temperature of the air is determined and used in the procedure CHX_h_finned_tube to
determine the air-side heat transfer coefficient, ha . The resistance to convection on the air-side
is:

1
Rconv ,a = (5)
Ltotal W H α ha


surface area

T_a_avg=(T_a_in+T_a_out)/2 "average air temperature"


Call CHX_h_finned_tube('fc_tubes_sCF-775-58T', m_dot_a, W*H, 'Air',T_a_avg, Po#:h_bar_a)
"get air-side heat transfer coefficient"
R_conv_a=1/(L_total*H*W*alpha*h_bar_a) "resistance to air-side convection"

The total conductance is:

1
UA = (6)
Rconv ,a

and the number of transfer units is:

UA
NTU = (7)
C a
The HX function in EES is used to determine the effectiveness of the heat exchanger, ε.

UA=1/R_conv_a "conductance"
NTU=UA/C_dot_a "number of transfer units"
eff=HX('crossflow_both_unmixed', NTU, C_dot_a, 9999 [W/K], 'epsilon')
"access solution for heat exchanger"

The steam-side temperature, Ts, is obtained. The maximum possible rate of heat transfer is:

qmax = C a (Ts − Ta ,in ) (8)

The actual rate of heat transfer is:

q = ε qmax (9)

T_s=temperature(Water,p=p_s,x=1) "steam temperature"


q_dot_max=C_dot_a*(T_s-T_a_in) "maximum possible rate of heat transfer"
q_dot=eff*q_dot_max "actual rate of heat transfer"

The guess values are updated and the assumed value of Ta,out is commented out. The air outlet
temperature is calculated using an energy balance:

q
Ta ,out = Ta ,in +  (10)
Ca

{T_a_out=converttemp(C,K,35 [C])} "guess for air outlet temperature"


T_a_out=T_a_in+q_dot/C_dot_a "air outlet temperature"
T_a_out_C=converttemp(K,C,T_a_out) "in C"

The heat provided to the zone is:

q zone = C a (Ta ,out − Tzone ) (11)

q_dot_zone=C_dot_a*(T_a_out-T_zone) "rate of heat transfer delivered to zone"

which leads to w b = 40.2 W, Ta,out = 73.2ºC, and q zone = 29.8 kW.

b.) Plot the heat provided to the zone and the blower power as a function of the number of
modules. Explain the shape of your plot. How many modules would you suggest using for
the steam coil? Why?

Figure 1 illustrates the heat provided to the zone and the blower power as a function of the
number of modules. The number of transfer units associated with the cooling coil becomes very
high for 2 or more modules and therefore the thermal performance is not improved substantially
as the number of modules is increased above 2; therefore, the heat transfer to the zone
asymptotically approaches a constant value. However, the pressure drop is linear with the
number of modules and therefore the blower power increases with the number of modules. The
capital cost of the steam coil will also increase with Nmod; therefore, 1 or 2 modules should be
used.
40000 400

350
37800 heat transfer to zone
300
Heat transfer to zone (W)

Blower power (W)


250
35600
blower power
200

33400
150

100
31200
50

29000 0
1 2 3 4 5 6 7 8 9 10
Number of coil modules
Figure 1: Heat transfer to zone and blower power as a function of the number of modules.
Problem 8.3-19
A simple heat exchanger is made by soldering two tubes together, as shown in Figure P8.3-19(a).

C c = 30 W/K
Tc ,in = 300 K
hc = 500 W/m -K
2

hot side fouling factor


R′′f ,h = 0.0001 K-m /W
2

2 tubes
D = 0.005 m inner diameter
m h = 0.01kg/s L = 20 m long
Th,in = 500 K
ρ h = 1000 kg/m3
ch = 2500 J/kg-K
μh = 0.01 Pa-s
kh = 0.3 W/m-K
Figure P8.3-19(a): Heat exchanger made by soldering two tubes together.

The two tubes each have inner diameter D = 0.005 m and length L = 20 m. The resistance to
conduction through the tube walls and the solder joint can be neglected. The hot fluid has mass
flow rate m h = 0.01 kg/s and enters with Th,in = 500 K. The properties of the hot flow are density
ρh = 1000 kg/m3, specific heat capacity ch = 2500 J/kg-K, viscosity μh = 0.01 Pa-s, and
conductivity kh = 0.3 W/m-K. There is a fouling factor on the hot fluid side of R′′f ,h = 0.0001 K-
m2/W. There is no fouling on the cold side. The cold fluid enters with Tc,in = 300 K and has
capacitance rate C c = 30 W/K. The average heat transfer coefficient on the cold fluid side is hc =
500 W/m2-K. Assume that the internal surface of both tubes is smooth.

Figure 8.3-19(b) illustrates the fully developed friction factor for a smooth, round tube as a
function of Reynolds number.
0.4

Fully developed friction factor


0.1

0.01

0.005
104 102 105 106 103 107 5x107
Reynolds number
Figure 8.3-19(b): Fully developed friction factor for a smooth, round tube as a function of Reynolds number.

Figure 8.3-19(c) illustrates the fully developed Nusselt number with a constant wall temperature
boundary condition for a smooth round tube as a function of Reynolds number and various
values of the Prandtl number.

104
Fully developed Nusselt number

Pr=500
103 Pr=80
Pr=20

102
Pr=0.7

all Pr
101

100
103 104 101 105 102 106
Reynolds number
Figure 8.3-19(c): Fully developed Nusselt number at constant wall temperature for a smooth, round tube as a
function of Reynolds number and various values of the Prandtl number.

You may assume that the tubes are long enough that the fully developed friction factor and
Nusselt number represent the average friction factor and Nusselt number.
a.) Using Figure 8.3-19(b), estimate the pressure drop associated with the flow of the hot fluid
through the heat exchanger.

The cross-sectional area of the tube is:


D 2 π ( 0.005 ) m
2 2

Ac , h = π = = 1.96x10-5 m 2 (1)
4 4

The mean velocity on the hot side is:

m h 0.01 kg m3
um , h = = = 0.51 m/s (2)
Ac ,h ρ h s 1.96x10-5 m 2 1000 kg

The Reynolds number on the hot side is:

um , h D ρ h 0.51 m 0.005 m 1000 kg Pa m 2 N-s 2


Reh = = = 255 (3)
μh s m3 0.01Pa-s N kg-m

According to Figure 8.3-19(b), the average friction factor is approximately f h = 0.25. The
pressure drop is:

ρ h um2 ,h L 1000 kg ( 0.51) m 0.25 20 m Pa m 2 N-s 2


2 2

Δph = fh = = 130540 Pa (4)


2 D 2 m3 s2 0.005 m N kg-m

b.) What is the minimum possible pumping power required to move the hot fluid through the
heat exchanger?

The minimum pumping power is:

Δph m h 130540 Pa 0.01 kg m3 N W-s


w min = = = 1.31 W (5)
ρh s 1000 kg Pa-m 2 N-m

c.) Using Figure 8.3-19(c), estimate the average heat transfer coefficient on the hot side of the
heat exchanger.

According to Figure 8.3-19(c), the average Nusselt number is approximately Nu h = 3.7.


Therefore, the average heat transfer coefficient on the hot side is:

Nu h kh 3.7 0.3 W W
hh = = = 235.4 2 (6)
D m-K 0.005 m m -K

d.) Explain briefly why the Nusselt number at low Reynolds number in Figure 8.3-19(c) is
independent of both Reynolds number and Prandtl number.

The fully developed Nusselt number in the laminar region is independent of Reynolds number,
Prandtl number. This is because the heat transfer from the tube wall is resisted by conduction
through the laminar thermal boundary layer - in the fully developed region, the laminar boundary
layer is of fixed thickness that depends on the size of the duct only and does not change based on
the Reynolds number or the Prandtl number. The Nusselt number is approximately:

k Dh Dh
Nu ≈ = (7)
δt k δt

Because δt is fixed, so is the Nusselt number - it should be a number not too different from 2.

e.) Determine the hot outlet temperature. Assume that your answer from part (c) was
W
hh = 150 2 (this may or may not be the correct answer). Use Figure 8.3-19(d) to
m -K
determine the effectiveness of a counter-flow heat exchanger.
1
C R=0
C R=0.25
0.8
C R=0.5
C R=0.75
Effectiveness

0.6
C R=1.0

0.4

0.2

0
0 1 2 3 4 5 6 7
Number of transfer units
Figure 8.3-19(d): Effectiveness of a counterflow heat exchanger as a function of NTU for various values of CR.

The resistance to convection on the hot and cold sides are:

1 m 2 -K K
Rconv , h = = = 0.021 (8)
hh π D L 150 W π 0.005 m 20 m W

1 m 2 -K K
Rconv ,c = = = 0.0064 (9)
hc π D L 500 W π 0.005 m 20 m W

The resistance to fouling is:

R′′f , h
0.0001 K-m 2 K
R foul = = = 0.00032 (10)
π DL W π 0.005 m 20 m W

The total resistance is:


K K K K
Rtotal = Rconv , h + R foul + Rconv ,c = 0.021 + 0.00032 + 0.0064 = 0.028 (11)
W W W W

and therefore the conductance is:

1 W W
UA = = = 35.8 (12)
Rtotal 0.028 K K

The capacitance rate of the hot fluid is:

0.01 kg 2500 J W
C h = m h ch = = 25 (13)
s kg-K K

The minimum capacitance rate is therefore the hot side and the number of transfer units is:

UA 35.8 W K
NTU =  = = 1.43 (14)
Ch K 25 W

and the capacitance ratio is:

C 25 W K
CR =  h = = 0.833 (15)
Cc K 30 W

According to Figure 8.3-19(d), the effectiveness is approximately ε = 0.62. The maximum


possible rate of heat transfer is:

25 W ( 500 − 300 ) K
qmax = C h (Th ,in − Tc ,in ) = = 5000 W (16)
K

and the actual heat transfer is:

0.62 5000 W
q = ε qmax = = 3091 W (17)

The outlet temperature on the hot side is:

q 3091 W K
Th ,out = Th ,in − = 500 K - = 376.4 K (18)

Ch 25 W

f.) Your boss has suggested that the fouling on the hot side controls the performance of the heat
exchanger and therefore extensive testing should be carried out in order to understand this
phenomenon. Do you agree? Why or why not?
The resistance to fouling is by far the smallest. It is not very important.

g.) If the heat exchanger were re-plumbed so that it was in a parallel flow rather than a
counterflow configuration do you expect the hot outlet temperature to change much? Will it
go up or down? Justify your answers.

The performance will go down (the hot outlet temperature will go up) and the improvement will
be substantial because the heat exchanger is operating at a CR that is near one and NTU that is
relatively large.

h.) If the tube on the hot side were replaced by one that was not perfectly smooth then what
would the effect be on the hot outlet temperature and the pressure drop? Justify your
answers.

The flow on the hot side is laminar and therefore the roughness is not important. There will be
no effect on either the hot outlet temperature or the pressure drop.
Problem 8.3-20
A cross-flow heat exchanger with both fluids unmixed is used to transfer heat from a flow of air
to a flow of chilled water. The air enters with Ta,in = 90ºF and flow rate Va = 250 ft3/min. The
chilled water enters with Tw,in = 40ºF and flow rate V = 0.65 gal/min. The total conductance of
w
the heat exchanger has been estimated to be UA = 350 Btu/hr-R. Determine the air outlet
temperature.

The inputs are entered in EES:

$UnitSystem English psi F


$TabStops 0.5 3.5

"Inputs"
V_dot_a=250 [ft^3/min]*convert(ft^3/min,ft^3/hr) "volumetric flow rate of air"
V_dot_w=0.65 [gal/min]*convert(gal/min,ft^3/hr) "volumetric flow rate of cooling water"
T_a_in=90 [F] "air inlet temperature"
T_w_in=40 [F] "cooling water inlet temperature"
UA=350 [Btu/hr-R] "conductance"

The capacitance rates are computed according to:

C a = ρ a ca Va (1)

C w = ρ w cw Vw (2)

where ρ and c are the density and specific heat capacity of the streams.

"capacitance rates"
rho_a=density(Air,T=T_a_in,P=14.7 [psi]) "density of air"
c_a=cP(Air,T=T_a_in) "specific heat capacity of air"
C_dot_a=V_dot_a*rho_a*c_a "capacitance rate of air"
rho_w=density(Water,T=T_w_in,P=14.7 [psi]) "density of cooling water"
c_w=cP(Water,T=T_w_in,P=14.7 [psi]) "specific heat capacity of cooling water"
C_dot_w=V_dot_w*rho_w*c_w "capacitance rate of cooling water"

The minimum capacitance rate, C min , is determined using the Min command. The number of
transfer units is obtained from:

UA
NTU =  (3)
Cmin

The effectiveness, ε, is obtained using the HX function in EES.

"effectiveness-NTU solution"
C_dot_min=MIN(C_dot_a,C_dot_w) "minimum capacitance rate"
NTU=UA/C_dot_min "number of transfer units"
eff=HX('crossflow_both_unmixed', NTU, C_dot_a, C_dot_w, 'epsilon')
"effectiveness-NTU solution"

The rate of heat transfer is:

q = ε C min (Ta ,in − Tw,in ) (4)

The air outlet temperature is obtained from:

q = C a (Ta ,in − Ta ,out ) (5)

q_dot=eff*C_dot_min*(T_a_in-T_w_in) "rate of heat transfer"


q_dot=C_dot_a*(T_a_in-T_a_out) "air outlet temperature"

which leads to Ta,out = 61.4ºF.


Problem 8.4-1
A refrigeration cycles employs carbon dioxide as the refrigerant that enters the high temperature
cooler at pCO2 = 80 bar and TCO2,in = 70°C at steady conditions. The carbon dioxide flowrate is
m CO 2 = 0.063 kg/s and it must be cooled to TCO2,out = 30°C before being expanded. A suggestion
has been made to use this ‘waste heat’ to provide domestic hot water. The water enters at Tw,in =
5°C from the mains supply at atmospheric pressure.
a) What is the maximum temperature that the water can be heated to and what is the
corresponding flow rate of the water?

The inputs are entered in EES:

$UnitSystem SI MASS RAD PA K J


$Tabstops 0.2 0.4 0.6 3.5 in

"known information"
P_CO2=80 [bar]*convert(bar,Pa) "pressure of CO2 stream"
T_CO2_in=converttemp(C,K,70 [C]) "inlet temperature of CO2"
T_CO2_out=converttemp(C,K,25 [C]) "exit temperature of CO2"
m_dot_CO2=0.063 [kg/s] "CO2 mass flow rate"
P_w=1 [bar]*convert(bar,Pa) "pressure of water stream"
T_w_in=converttemp(C,K,5 [C]) "water inlet temperature"

The total rate of heat transfer is obtained from an energy balance on the carbon dioxide:

qtotal = m CO 2 ( iCO 2,in − iCO 2,out ) (1)

where iCO2,in and iCO2,out are the specific enthalpies of the carbon dioxide at the inlet and exit
conditions, respectively.

i_CO2_in=enthalpy(Carbondioxide,T=T_CO2_in,P=P_CO2) "specific enthalpy of inlet CO2"


i_CO2_out=enthalpy(Carbondioxide,T=T_CO2_out,P=P_CO2) "specific enthalpy of exiting CO2"
q_dot_total=m_dot_CO2*(i_CO2_in-i_CO2_out) "rate of heat transfer"

It would seem that the maximum temperature that the water could possibly attain is the inlet
temperature of the carbon dioxide, or Tw,out,wrong = TCO2,in = 70°C. An energy balance on the
water leads to:

qtotal = m w, wrong ( iw,in − iw,out , wrong ) (2)

where iw,in and iw,out,wrong are the specific enthalpies of the water at Tw,in and Tw,out,wrong.

i_w_in=enthalpy(Water,T=T_w_in,P=P_w) "specific enthalpy of inlet water"


T_w_out_max_wrong=T_CO2_in "wrong answer for the maximum possible outlet water temperature"
i_w_out_max_wrong=enthalpy(Water,T=T_w_out_max_wrong,P=P_w)
"enthalpy of water at temperature T_w_out_max"
m_dot_w_wrong*(i_w_out_max_wrong-i_w_in)=q_dot_total "mass flow rate of water"
which leads to m w, wrong = 0.0493 kg/s. However, this result is not possible because it violates the
second law of thermodynamics. The problem becomes evident when the temperatures of the
carbon dioxide and water are plotted versus the heat transfer rate. The temperature of the carbon
dioxide as a function of duty can be obtained according to:

qtotal f = m CO 2 ( iCO 2 − iCO 2,out ) (3)

where iCO2 is the specific enthalpy of the carbon dioxide at temperature TCO2 and f is the
fractional duty. Figure 1 illustrates the temperature of the carbon dioxide as a function of duty.

q_dot_total*f=m_dot_CO2*(i_CO2-i_CO2_out) "enthalpy of carbon dioxide for a given duty"


q_dot=f*q_dot_total "duty"
T_CO2=temperature(Carbondioxide,h=i_CO2,P=P_CO2)
"temperature of carbon dioxide for a given duty"
T_CO2_C=converttemp(K,C,T_CO2) "in C"

70

60

50
Temperature (°C)

carbon dioxide
40

30
water at m w ,w rong
20

10

0
0 2000 4000 6000 8000 10000 12000 14000
Duty (W)
Figure 1: Temperature of carbon dioxide as a function of duty. Also shown is the temperature of the water as
a function of duty with mass flow rate 0.049 kg/s.

The temperature of the water as a function of duty for m w can be obtained according to:

qtotal f = m w ( iw − iw,in ) (4)

where iw is the specific enthalpy of the water at temperature Tw and f is the fractional duty. The
following EES code determines the duty curve for water with m w = m w, wrong .

m_dot_w=m_dot_w_wrong "set mass flow rate to m_dot_w_wrong"


q_dot_total*f=m_dot_w*(i_w-i_w_in) "enthalpy of water for a given duty"
T_w=temperature(Water,h=i_w,P=P_w) "temperature of water for a given duty"
T_w_C=converttemp(K,C,T_w) "in C"
The temperature of the water as a function of duty is shown in Figure 1. Note that the lines
cross, indicating the second law violation. The pinchpoint temperature difference within the heat
exchanger can be identified by computing the temperature difference as a function duty:

ΔT = TCO 2 − Tw (5)

DELTAT=T_CO2-T_w "temperature difference for a given duty"

and minimizing ΔT by varying f using the Min/Max feature in EES. For m w = m w,wrong , the pinch
point temperature difference is ΔT = -8.649 K, which is physically impossible. The best possible
heat exchanger performance occurs when the pinchpoint temperature difference is 0. The value
of the water mass flow rate is commented out:

{m_dot_w=m_dot_w_wrong} "set mass flow rate to m_dot_w_wrong"

and the variables f, ΔT, and m w are placed in a parametric table. The value of m w is specified
over a range from 0.04 kg/s to 0.1 kg/s and the Min/Max Table option from the Calculate menu
is used to determine the pinchpoint temperature difference as a function of mass flow rate, as
shown in Figure 2.
15
Pinchpoint temperature difference (K)

10

-5

-10

-15

-20

-25
0.04 0.05 0.06 0.07 0.08 0.09 0.1
Water mass flow rate (kg/s)
Figure 2: Pinchpoint temperature difference as a function of water mass flow rate.

Figure 2 indicates that the best performance occurs when m w = 0.061 kg/s. The water mass flow
rate is set to this value:

m_dot_w=0.061 [kg/s] "set mass flow rate to zero pinchpoint temperature difference value"

Figure 3 illustrates the carbon dioxide and water temperatures as a function of duty; note that the
pinchpoint temperature difference is zero and that the highest temperature that the water can be
heated to is Tw,out = 57.6°C.
70

60

50

Temperature (°C)
carbon dioxide
40

30
water at 0.07 kg/s
20
water at 0.061 kg/s

10

0
0 2000 4000 6000 8000 10000 12000 14000
Duty (W)
Figure 3: Temperature of carbon dioxide as a function of duty for a pinchpoint temperature difference of 0 K
(i.e. a water flow rate of 0.061 kg/s) and a pinchpoint temperature difference of 5 K (i.e., a water flow rate of
0.07 kg/s).

b.) Repeat part (a) if a pinch point temperature difference of 5 K is required to overcome heat
transfer resistances.

Figure 2 indicates that m w = 0.07 kg/s corresponds to a pinchpoint temperature difference of 5 K.

m_dot_w=0.07 [kg/s] "set mass flow rate to zero pinchpoint temperature difference value"

Figure 3 includes the duty line for water at 0.07 kg/s and shows that the highest water outlet
temperature that can be achieved with a 5 K pinchpoint temperature difference is 50.8°C.
Problem 8.4-2 Kalina Cycle Evaporator
A Kalina cycle is a power cycle that uses a mixture of ammonia and water as the working fluid,
rather than a pure fluid such as steam. A mixture of xam = 50% ammonia by mass is introduced
to the evaporator heat exchanger at Tam-w,in = 310 K and pam-w =10 bar. The mixture exits the
evaporator at Tam-w,out = 450 K and pam-w = 10 bar. The thermal energy needed to evaporate the
mixture is provided by an air stream that enters the evaporator heat exchanger at Ta,in = 460 K
and pa = 1 bar. Determine the minimum mass flow rate of the air needed for this purpose
(corresponding to infinite heat exchanger conductance) per unit mass flow rate of the ammonia-
water mixture. Also determine the outlet air temperature. Ammonia water properties are
provided in EES with the NH3H2O external procedure. Directions for using these properties can
be found in the Function Info dialog (Options menu) by selecting External routines and
NH3H2O options and then clicking the Function Info button.

The inputs are entered in EES:

$UnitSystem SI MASS RAD PA K J


$Tabstops 0.2 0.4 0.6 3.5 in

"known information"
x_am=0.5 [-] "concentration of ammonia"
T_amw_in=310 [K] "temperature of ammonia-water solution entering"
T_amw_out=450 [K] "temperature of ammonia-water solution leaving"
P_amw=10 [bar]*convert(bar,Pa) "pressure of ammonia-water solution"
T_a_in=460 [K] "temperature of entering air"
m_dot_amw=1 [kg/s] "per unit mass flow rate of ammonia water"

A function is constructed that calls the NH3H2O procedure and returns the enthalpy given the
temperature, pressure, and ammonia concentration.

function h_amw(T,P,x)
" Inputs:
T - temperature (K)
P - pressure (Pa)
x - ammonia concentration by mass (-)

Output:
h_amw - enthalpy (J/kg)"
P_bar=P*convert(Pa,bar) "convert pressure to bar"
Call NH3H2O(123,T,P_bar,x: T, P, x, h_kJkg, s, u, v, Qu)
h_amw=h_kJkg*convert(kJ/kg,J/kg) "convert enthalpy to J/kg"
end

The minimum air flow rate corresponds to a heat exchanger conductance (and pinch point
temperature difference) of 0 K. It is not sufficient to determine the air mass flow rate and
temperature by energy balances because the pinch point may not occur at the inlet or outlet of the
heat exchanger. This problem can be solved graphically by constructing a plot of the
temperature of the ammonia water mixture versus the heat transfer rate needed to attain that
temperature.
The total rate of heat transfer required can be obtained by doing an energy balance on the
ammonia-water solution:

qtotal = m am − w ( iam− w,out − iam − w,in ) (1)

where iam-w,out and iam-w,in are the specific enthalpies of the ammonia-water solution at the outlet
and inlet temperatures, respectively.

i_amw_in=h_amw(T_amw_in,P_amw,x_am) "enthalpy of ammonia water solution entering"


i_amw_out=h_amw(T_amw_out,P_amw,x_am) "enthalpy of ammonia water solution leaving"
q_dot_total=m_dot_amw*(i_amw_out-i_amw_in) "total heat transfer rate"

The duty curve for the ammonia-water solution is generated according to:

qtotal f = m am − w ( iam − w − iam − w,in ) (2)

where f is the fractional duty and iam-w is the specific enthalpy at Tam-w.

f=0.5 [-] "


f*q_dot_total=m_dot_amw*(i_amw-i_amw_in) "enthalpy of ammonia-water solution as a function of duty"
q_dot=f*q_dot_total "duty"
i_amw=h_amw(T_amw,P_amw,x_am) "temperature of ammonia water solution at given duty"

Figure 1 illustrates the temperature of the ammonia-water solution as a function of the duty.
460

440
air
420
Temperature (K)

400
ammonia-water
380

360

340

320

300
0 0.1 0.2 0.3 0.4 0.5 0.6 0.7 0.8 0.9 1
Fractional duty
Figure 1: Temperature of the ammonia-water and air as a function of duty for an air flow rate of 50 kg/s.

The mass flow rate of air is not known before the problem is solved; the mass flow rate of air,
m a , is assumed and used to compute the outlet enthalpy of air according to:

qtotal = m a ( ia ,in − ia ,out ) (3)


The outlet temperature is obtained from the enthalpy. The duty curve for the air is obtained
according to:

qtotal f = m a ( ia − ia ,out ) (4)

where ia is the specific enthalpy of air at Ta.

i_a_in=enthalpy(Air,T=T_a_in) "inlet enthalpy of air"


m_dot_a=50 [kg/s] "mass flow rate of air - assumed"
q_dot_total=m_dot_a*(i_a_in-i_a_out) "outlet enthalpy of air"
T_a_out=temperature(Air,h=i_a_out) "outlet temperature of air"
f*q_dot_total=m_dot_a*(i_a-i_a_out) "enthalpy of air as a function of duty"
T_a=temperature(Air,h=i_a) "temperature of air at given duty"

Figure 1 shows the temperature of the air as a function of duty for the assumed value of the air
mass flow rate. The pinchpoint temperature difference is obtained by minimizing ΔT:

ΔT = Ta − Tam − w (5)

DELTAT=T_a-T_amw "temperature difference"

by varying f. Figure 2 illustrates the pinchpoint temperature difference as a function of the mass
flow rate of air and indicates that an air mass flow rate of approximately 18.5 kg/s leads to a
pinchpoint temperature difference of approximately 0 K and therefore corresponds to infinite
heat exchanger conductance.
40
Pinchpoint temperature difference (K)

20

-20

-40

-60

-80
10 15 20 25 30 35 40 45 50 55 60
Mass flow rate of air (kg/s)
Figure 2: Pinchpoint temperature difference as a function of air mass flow rate.

The air mass flow rate is specified:

{m_dot_a=50 [kg/s]} "mass flow rate of air - assumed"


m_dot_a=18.5 [kg/s] "mass flow rate of air corresponding to 0 pinchpoint"
The duty lines of the air and ammonia-water with an air mass flow rate of 18.5 kg/s is shown in
Figure 3. The exit temperature of the air is Ta,out = 337.4 K.
460

440
air
420
Temperature (K)
400 ammonia-water

380

360

340

320

300
0 0.1 0.2 0.3 0.4 0.5 0.6 0.7 0.8 0.9 1
Fractional duty
Figure 3: Temperature of the ammonia-water and air as a function of duty for an air flow rate of 18.5 kg/s.
Problem 8.5-1: Salt-steam heat exchanger in solar power plant
The power block for a solar central receiver power tower system is currently being designed.
The fluid circulated through the receiver of solar power tower is a molten salt having an average
specific heat of cs = 1.52 kJ/kg-K. At design conditions, the salt will enter the counterflow
superheater at m s = 225 kg/s and Ts,in = 480°C. The design calls for the salt to heat water at
m w = 25 kg/s from Tw,in = 225°C to Tw,in = 370°C at a pressure of pw = 100 bar. You have been
asked to determine the total heat exchanger conductance needed for this purpose and the exiting
temperature of the salt.

The inputs are entered in EES:

$UnitSystem SI MASS RAD PA K J


$Tabstops 0.2 0.4 0.6 3.5 in

T_s_in=converttemp(C,K,480 [C]) "salt inlet temperature"


m_dot_s=225 [kg/s] "salt mass flowrate"
c_s=1.52 [kJ/kg-K]*convert(kJ/kg-K,J/kg-K) "specific heat capacity of salt"
T_w_in=converttemp(C,K,225 [C]) "water inlet temperature"
p_w=100 [bar]*convert(bar,Pa) "water pressure"
T_w_out=converttemp(C,K,370 [C]) "design exit water temperature"
m_dot_w=25 [kg/s] "water flow rate"

The specific enthalpy of the water at the inlet and outlet states (iw,in and iw,out, respectively) are
obtained using EES' internal property routines. The total rate of heat transfer in the heat
exchanger is:

q = m w ( iw,out − iw,in ) (1)

The capacitance rate of the salt is:

C s = m s cs (2)

i_w_out=enthalpy(Water,T=T_w_out,P=p_w) "specific enthalpy of exiting water"


i_w_in=enthalpy(Water,T=T_w_in,P=p_w) "specific enthalpy of inlet water"
q_dot=m_dot_w*(i_w_out-i_w_in) "required heat transfer rate"
C_dot_s=m_dot_s*c_s "capacitance rate of salt"

The heat exchanger is broken into a superheat, boiler, and sub-cooler section. The temperature
of the water entering the superheat section (Tsh,in) is the saturation temperature of water. The
enthalpy of the water entering the superheat section (iw,sh,in) is the enthalpy of saturated water
vapor. The average specific heat capacity of the water vapor in the superheater is:

cw, sh =
(i
w, out − iw, sh ,in )
(3)
(Tw, out − Tw, sh ,in )

The capacitance rate of the water is:


C w, sh = m w cw, sh (4)

"Superheat section"
T_w_sh_in=T_sat(Water,P=P_w) "water superheater inlet temperature"
i_w_sh_in=enthalpy(Water,P=P_w,x=1) "specific enthalpy of water vapor entering superheater"
c_w_sh=(i_w_out-i_w_sh_in)/(T_w_out-T_w_sh_in) "specific heat of water in superheater"
C_dot_w_sh=c_w_sh*m_dot_w "capacitance rate of water in superheater"

The rate of heat transfer in the superheat section is:

qsh = m w ( iw,out − iw, sh,in ) (5)

The minimum capacitance rate in the superheat section ( C sh , min ) is identified and used to
compute the effectiveness according to:

qsh
ε sh =  (6)
C sh , min ( Ts ,in − Tw, sh ,in )

The effectiveness is used to determine the number of transfer units (NTUsh) with the HX function.
The number of transfer units is used to determine the conductance according to:

UAsh = NTU sh C sh , min (7)

An energy balance on the salt determines the salt temperature leaving the superheater:

qsh = C s (Ts ,in − Ts , sh ,out ) (8)

q_dot_sh=m_dot_w*(i_w_out-i_w_sh_in) "heat transfer rate in superheater"


C_dot_sh_min=Min(C_dot_s,C_dot_w_sh) "minimum capacitance rate in superheater"
eff_sh=q_dot_sh/(C_dot_sh_min*(T_s_in-T_w_sh_in)) "effectiveness of superheater"
NTU_sh=HX('counterflow', eff_sh, C_dot_w_sh, C_dot_s, 'NTU') "
UA_sh=NTU_sh*C_dot_sh_min "conductance in superheater section"
q_dot_sh=C_dot_s*(T_s_in-T_s_sh_out)
"energy balance determines temperature of salt exiting superheater"

The temperature of the water entering the boiler (Tw,b,in) is the same as the temperature of the
water entering the superheat region. The enthalpy of the water entering the boiler (iw,b,in) is
saturated liquid. The rate of heat transfer in the boiler is:

qb = m w ( iw, sh,in − iw,b ,in ) (9)

The effectiveness of the boiler is:


q
εb =  b
(10)
Cs (Ts , sh ,out − Tw,b ,in )

The effectiveness is used to determine the number of transfer units (NTUb) with the HX function.
The number of transfer units is used to determine the conductance according to:

UAb = NTU b C s (11)

An energy balance on the salt determines the salt temperature leaving the boiler:

qb = C s (Ts ,sh ,out − Ts ,b ,out ) (12)

"Boiler section"
T_w_b_in=T_w_sh_in "boiler inlet temperature"
i_w_b_in=enthalpy(Water,P=p_w,x=0) "specific enthalpy of saturated liquid water"
q_dot_b=m_dot_w*(i_w_sh_in-i_w_b_in) "rate of heat transfer in the boiler"
eff_b=q_dot_b/(C_dot_s*(T_s_sh_out-T_w_b_in)) "effectiveness of boiler"
NTU_b=HX('counterflow', eff_b, 1e9 [W/K], C_dot_s, 'NTU') "NTU in boiler"
UA_b=NTU_b*C_dot_s "conductance in the boiler"
q_dot_b=C_dot_s*(T_s_sh_out-T_s_b_out)
"energy balance determines the temperature of the salt exiting boiler"

The rate of heat transfer in the subcool section is:

qsc = m w ( iw,b ,in − iw,in ) (13)

The average specific heat capacity of the water vapor in the subcooler is:

cw, sc =
(iw ,b ,in − iw,in )
(14)
(T w ,b ,in − Tw,in )

The capacitance rate of the water is:

C w, sc = m w cw, sc (15)

"Subcooled section"
q_dot_sc=m_dot_w*(i_w_b_in-i_w_in) "heat transfer rate in subcooled heater section"
c_w_sc=(i_w_b_in-i_w_in)/(T_w_b_in-T_w_in) "average specific heat of subcooled water"
C_dot_w_sc=c_w_sc*m_dot_w "capacitance rate of subcooled water"

The minimum capacitance rate in the subcooling section ( C sc , min ) is identified and used to
compute the effectiveness according to:
qsh
ε sc =  (16)
C sc , min (T
s , sh , out − Tw,in )

The effectiveness is used to determine the number of transfer units (NTUsc) with the HX function.
The number of transfer units is used to determine the conductance according to:

UAsc = NTU sc C sc , min (17)

An energy balance on the salt determines the salt temperature leaving the heat exchanger:

qsc = C s (Ts ,b ,out − Ts ,out ) (18)

C_dot_sc_min=Min(C_dot_w_sc,C_dot_s) "minimum capacitance rate in the subcooler"


eff_sc=q_dot_sc/(C_dot_sc_min*(T_s_sh_out-T_w_in)) "effectiveness"
NTU_sc=HX('counterflow', eff_sc, C_dot_w_sc, C_dot_s, 'NTU') "NTU in subcooler section"
UA_sc=NTU_sc*C_dot_sc_min "conductance in subcooled section"
q_dot_sc=C_dot_s*(T_s_b_out-T_s_out) "energy balance determines the salt outlet temperature"

The total conductance is:

UA = UAsh + UAb + UAsc (19)

UA=UA_sh+UA_b+UA_sc "total conductance required"

which leads to UA = 471.2 kW/K and Ts,out = 604.8 K.


Problem 8.5-2
A single pass shell and tube heat exchanger is used to condense steam exiting from the power
turbine at ps = 0.1 bar and xs,in = 95% quality at a mass flow rate of m s = 3.8 kg/s. The steam
flows in the shell and treated cooling water from a cooling tower at Tcw,in = 24°C, pcw = 1 bar and
m cw = 325 kg/s enters the tubes. There are Ntube = 384 tubes in the shell, each having an outer
diameter of Do = 38.1 mm, an inner diameter of Di = 35.1 mm and a length of L = 8 m with a
roughness of e = 1 µm. The tubes are made of steel with a thermal conductivity of kt = 50 W/m-
K. Fouling on the inside of the tubes is expected to occur for the treated water. Based on
manufacturer’s data, the heat transfer coefficient between the condensing steam and the outer
tube wall is hs , sat = 4625 W/m2-K. Some of the tubes are submerged in the condensed steam and
the heat transfer coefficient between the condensate and the tube wall for these tubes is estimated
to be hs , sc = 150 W/m2-K. Heat losses from the shell to the surroundings are negligible.
Determine the outlet temperature of the condensate and the well-mixed temperature of exiting
cooling water.

This problem will be solved by assuming it can be analyzed as two shell and tube heat
exchangers in parallel. The top section has Ntube,sat onto which the steam is condensed. The
bottom section represents the remaining tubes that are submerged in condensate. Start by
entering known information into EES.

$UnitSystem SI MASS RAD PA K J


$Tabstops 0.2 0.4 0.6 3.5 in

"Inputs"
P_s=0.1 [bar]*convert(bar,Pa) "steam pressure in condenser"
x_s_in=0.95 [-] "quality of entering steam"
m_dot_s=3.8 [kg/s] "mass flow rate of steam"
T_cw_in=converttemp(C,K,24 [C]) "cooling water inlet temperature"
P_cw=1 [bar]*convert(bar,Pa) "cooling water inlet pressure"
m_dot_cw=325 [kg/s] "mass flow rate of cooling water"
N_tube=385 "number of tubes"
D_i=35.1 [mm]*convert(mm,m) "inner diameter of tubes"
D_o=38.1 [mm]*convert(mm,m) "outer diameter of tubes"
L=8 [m] "length of shell and tubes"
k_t=50 [W/m-K] "thermal conductivity of pipe wall"
e=1e-6 [m] "roughness of inner tube wall"
h_bar_sat=4625 [W/m^2-K]
"heat transfer coefficient between outer tube wall and condensing steam"
h_bar_sc=150 [W/m^2-K] "heat transfer coefficient between outer tube wall and condensate"

The enthalpy of the steam entering the heat exchanger (is,in) and leaving the saturated section of
the heat exchanger (is,sat,out) are computed. The rate of heat transfer in the saturated section is
computed using an energy balance on the steam:

qsat = m s ( is ,in − is , sat ,out ) (1)


The temperature of the saturated steam (Ts,sat) is computed and the specific heat capacity of the
cooling water (ccw) is determined.

"saturated section"
i_s_in=enthalpy(Water,p=p_s,x=x_s_in) "enthalpy of steam entering"
i_s_sat_out=enthalpy(Water,p=p_s,x=0 [-]) "enthalpy of saturated liquid leaving saturated section"
q_dot_sat=m_dot_s*(i_s_in-i_s_sat_out) "energy balance on steam in saturated section"
T_s_sat=temperature(Water,P=P_s,x=0 [-]) "saturation temperature on steam side"
c_cw=cP(Water,T=T_cw_in,P=P_cw)
"specific heat capacity of the cooling water in saturated section"

The fraction of the tubes in the saturated section (Fsat) is initially assumed. The number of tubes
in the saturated section is computed according to:

N tube, sat = Fsat N tube (2)

The capacitance rate of the cooling water is:

C cw, sat = Fsat m cw ccw (3)

The temperature of the cooling water leaving the saturated section is computed with an energy
balance on the cooling water:

qsat = C cw, sat (Tcw, sat ,out − Tcw,in ) (4)

F_sat=0.8 [-] "guess for fraction of the tubes involved in the saturated section"
N_tube_sat=N_tube*F_sat "number of tubes in the saturated section"
C_dot_cw_sat=m_dot_cw*F_sat*c_cw "capacitance rate of the cooling water in the saturated section"
q_dot_sat=C_dot_cw_sat*(T_cw_sat_out-T_cw_in) "energy balance on cooling water"

The effectiveness of the saturated section is:

ε sat =
(T cw , sat , out − Tcw,in )
(5)
(T s , sat − Tcw,in )

The number of transfer units in the saturated section (NTUsat) can be obtained from the
effectiveness using the ε-NTU relationships for a shell-and-tube heat exchanger, accessed using
the HX function in EES.

eff_sat=(T_cw_sat_out-T_cw_in)/(T_s_sat-T_cw_in) "effectiveness of saturated section"


NTU_sat=HX('shell&tube_1', eff_sat, C_dot_cw_sat, 1e99 [W/K], 'NTU')
"determine NTU in saturated section"

The PipeFlow function is used to determine the heat transfer coefficient on the water side ( hcw ).
The resistance to convection on the cooling water side is:
1
Rconv ,cw, sat = (6)
N tube, sat π Di hcw

The resistance to convection on the condensing steam side is:

1
Rconv , s , sat = (7)
N tube , sat π Do hsat

The total conductance is:

1
UAsat = (8)
Rconv , s , sat + Rconv ,cw, sat

R_conv_cw_sat=1/(N_tube_sat*pi*D_i*L*h_bar_cw)
"resistance to convection to the cooling water in saturated section"
R_conv_s_sat=1/(N_tube_sat*pi*D_o*L*h_bar_sat)
"resistance to convection to the condensing steam in saturated section"
UA_sat=1/(R_conv_cw_sat+R_conv_s_sat) "conductance in saturated section"

The problem is solved and the guess values are updated. The assumed value of Fsat is
commented out and the number of transfer units is recalculated according to:

UAsat
NTU sat = (9)
C cw, sat

{F_sat=0.8 [-]} "guess for fraction of the tubes involved in the saturated section"
NTU_sat=UA_sat/C_dot_cw_sat "recalculate the number of transfer units in the saturated section"

The subcooled section is addressed next. The fraction of the tubes in the subcooled section is
computed according to:

Fsc = 1 − Fsat (10)

The number of tubes in the subcooled section is computed to:

N tube , sc = Fsc N tube (11)

The capacitance rate of the cooling water is:

C cw,cs = Fsc m cw ccw (12)

The specific heat capacity of the condensate in the subcooled section (cs,sc) is computed and used
to determine the capacitance rate of the condensate:
C s ,cs = m s cs , sc (13)

The minimum capacitance rate in the subcooled section ( C cs ,min ) is identified.

"subcooled section"
F_sc=1-F_sat "fraction of tubes in the subcooled section"
N_tube_sc=F_sc*N_tube "number of tubes in the subcooled section"
C_dot_cw_sc=m_dot_cw*F_sc*c_cw "capacitance rate of the cooling water in the subcooled section"
c_s_sc=cP(Water,P=P_s,x=0 [-]) "specific heat capacity of condensate"
C_dot_s_sc=m_dot_s*c_s_sc "capacitance rate of the condensate in the subcooled section"
C_dot_sc_min=Min(C_dot_cw_sc,C_dot_s_sc) "minimum capacitance rate in the subcooled section"

The resistance to convection on the cooling water side is:

1
Rconv ,cw,cs = (14)
N tube, sc π Di hcw

The resistance to convection on the condensate side is:

1
Rconv , s , sc = (15)
N tube, sc π Do hsc

The total conductance in the subcooled section is:

1
UAsc = (16)
Rconv , s , sc + Rconv ,cw, sc

The number of transfer units in the subcooled section is:

UA
NTU sc =  sc (17)
Csc ,min

The HX function in EES is used to determine the effectiveness of the subcooled section (εsc).

R_conv_cw_sc=1/(N_tube_sc*pi*D_i*L*h_bar_cw)
"resistance to convection to the cooling water in subcooled section"
R_conv_s_sc=1/(N_tube_sc*pi*D_o*L*h_bar_sc)
"resistance to convection to the condensing steam in subcooled section"
UA_sc=1/(R_conv_cw_sc+R_conv_s_sc) "conductance in subcooled section"
NTU_sc=UA_sc/C_dot_sc_min "number of transfer units in subcooled section"
eff_sc=HX('shell&tube_1', NTU_sc, C_dot_s_sc, C_dot_sc_min, 'epsilon')
"relation between NTU and effectiveness"

The heat transfer rate in the subcooled section is:


qsc = ε sc C sc , min (Ts , sat − Tcw,in ) (18)

The temperature of the cooling water leaving the subcooled section is obtained from an energy
balance:

q
Tcw, sc ,out = Tcw,in +  sc (19)
Ccw, sc

and the temperature of the condensate leaving is:

q
Ts ,out = Ts , sat −  sc (20)
Cs , sc

The well-mixed temperature of the cooling water is the temperature that occurs after the cooling
water leaving the saturated and subcooled sections mix:

Tcw,out =
( C cw, sc Tcw, sc ,out + C cw, sat Tcw, sat ,out ) (21)
(
C cw, sc + C cw, sat )
q_dot_sc=eff_sc*C_dot_sc_min*(T_s_sat-T_cw_in) "heat transfer rate in the subcooled section"
T_cw_sc_out=T_cw_in+q_dot_sc/C_dot_cw_sc "exit temperature of the cooling water"
T_s_out=T_s_sat-q_dot_sc/C_dot_s_sc "exit temperature of the condensate"
T_s_out_C=converttemp(K,C,T_s_out) "in C"
T_cw_out=(C_dot_cw_sc*T_cw_sc_out+C_dot_cw_sat*T_cw_sat_out)/(C_dot_cw_sc+C_dot_cw_sat)
"well-mixed exit temperature of the cooling water"
T_cw_out_C=converttemp(K,C,T_cw_out) "in C"

which leads to Ts,out = 35.57ºC and Tcw,out = 30.47 ºC.


PROBLEM 8.3-1 (8-6 in text): Energy Recovery
The processing plant where you work has a process that results in a stream of hot combustion
products (assume that it has the properties of air) at moderate temperature Thg,in = 150°C with
mass flow rate m hg = 0.25 kg/s. You would like to recovery the energy associated with this flow
of air in order to reduce the cost of heating the plant and therefore you are evaluating the use of
the air-to-air heat exchanger shown in Figure 1.

Figure 1: Air-to-air heat exchanger.

The air-to-air heat exchanger is a cross-flow configuration. The length of the heat exchanger
parallel to the two flow directions is L = 10 cm. The width of the heat exchanger in the direction
perpendicular to the two flow directions is W = 20 cm. Cold air enters the heat exchanger from
outdoors at Tcg,in = -5°C with mass flow rate m cg = 0.50 kg/s and is heated by the combustion
products to Tcg,out.

The hot and cold air flows through channels that are rectangular (both sides of the heat
exchanger have the same geometry). The width of the channels is hc = 1.0 mm. There are fins
placed in the channel in order to provide structural support and also increase the surface area for
heat transfer. The fins can be assumed to run the complete length of the heat exchanger and are
100% efficient. The fins are spaced with pitch, pf = 0.5 mm and the fins are thf = 0.10 mm thick.
The thickness of the metal separating the cold channels from the hot channels is thw = 0.20 mm
and the conductivity of this metal is km = 15 W/m-K. Both the hot and cold flows are at
nominally atmospheric pressure. The fouling factor associated with the flow of combustion gas
through the heat exchanger is FF = 0.0018 K-m2/W. There is no fouling associated with the
flow of outdoor air through the heat exchanger.

a.) Compute the heat transfer coefficient between the hot air and the channel walls and the cold
gas and the channel walls. Use the inlet temperatures of the air flows to compute the
properties.

The inputs are entered in EES:

"PROBLEM 8-3-1"
$UnitSystem SI MASS RAD PA K J
$TABSTOPS 0.2 0.4 0.6 0.8 3.5 in

"Inputs"
L=10 [cm]*convert(cm,m) "size of module - parallel to flow directions"
W=20 [cm]*convert(cm,m) "size of module - perpendicular to flow directions"
th_w=0.2 [mm]*convert(mm,m) "channel wall thickness"
th_f=0.1 [mm]*convert(mm,m) "fin thickness"
h_c=1.0 [mm]*convert(mm,m) "channel height"
p_f=0.5 [mm]*convert(mm,m) "fin pitch"
k_m=15 [W/m-K] "metal conductivity"
FF=0.0018 [K-m^2/W] "combustion gas fouling factor"
m_dot_hg=0.25 [kg/s] "mass flow rate of hot gas"
T_hg_in=converttemp(C,K,150) "hot gas inlet temperature"
m_dot_cg=0.5 [kg/s] "mass flow rate of cold gas"
T_cg_in=converttemp(C,K,-5) "cold gas inlet temperature"

The properties of the hot and cold flows (chg, khg, μhg, ρhg, ccg, kcg, μcg, and ρcg) are computed
using EES’ internal property routines. The properties should be computed at the average flow
temperature. However, the outlet temperatures are not yet known, so initially, the average
temperature is set to the inlet temperature. This approximation can be corrected after the outlet
temperatures are determined and the guess values are updated.

T_hg_avg=T_hg_in "est. average hot gas temperature"


{T_hg_avg=(T_hg_in +T_hg_out)/2 "average hot gas temperature"}
c_hg=cP(Air,T=T_hg_avg) "specific heat capacity"
k_hg=conductivity(Air,T=T_hg_avg) "conductivity"
mu_hg=viscosity(Air,T=T_hg_avg) "viscosity"
rho_hg=density(Air,T=T_hg_avg,p=1 [atm]*convert(atm,Pa)) "density"

T_cg_avg=T_cg_in "est. average cold gas temperature"


{T_cg_avg=(T_cg_in+T_cg_out)/2 "average cold gas temperature"}
c_cg=cP(Air,T=T_cg_avg) "specific heat capacity"
k_cg=conductivity(Air,T=T_cg_avg) "conductivity"
mu_cg=viscosity(Air,T=T_cg_avg) "viscosity"
rho_cg=density(Air,T=T_cg_avg,p=1 [atm]*convert(atm,Pa)) "density"
The number of hot or cold channels in the heat exchanger is:

W
Nc = (1)
2 ( hc + thw )

The number of fins in each channel is:

L
Nf = (2)
pf

N_c=floor(W/(2*(h_c+th_w))) "number of channels"


N_f=floor(L/p_f) "number of fins per channel"

Note that the floor command rounds the result down to the nearest integer. The total cross-
sectional area for flow (on either side) is:

Ac = N c N f ( p f − th f ) hc (3)

and the total wetted perimeter (on either side) is:

per = N c N f 2 ( hc + p f − th f ) (4)

so the hydraulic diameter of the flow channels is:

Ac
Dh = 4 (5)
per

A_c=N_c*N_f*h_c*(p_f-th_f) "cross-sectional area for flow"


per=N_c*N_f*2*(h_c+p_f-th_f) "wetted perimeter"
D_h=4*A_c/per "hydraulic diameter"

The mean velocity of the hot gas is:

m hg
um, hg = (6)
Ac ρ hg

The Reynolds number associated with this flow is:

um, hg Dh ρ hg
Rehg = (7)
μhg

and the Prandtl number of the hot gas is:


μhg chg
Prhg = (8)
khg

u_m_hg=m_dot_hg/(A_c*rho_hg) "mean velocity"


Re_hg=u_m_hg*D_h*rho_hg/mu_hg "Reynolds number"
Pr_hg=mu_hg*c_hg/k_hg "Prandtl number"

The correlations for flow through a rectangular duct are accessed using the procedure
DuctFlow_N and used to compute the average friction factor ( f hg ) and Nusselt numbers ( Nu T ,hg
and Nu H ,hg ). The heat transfer coefficient on the hot gas side is computed:

Nu T ,hg khg
hhg = (9)
Dh

Note that the constant temperature Nusselt number is used in this case because it is a lower
bound and you would like the predictions to be conservative.

call DuctFlow_N(Re_hg,Pr_hg,L/D_h, (p_f-th_f)/h_c,0 [-]: Nusselt_T_hg, Nusselt_H_hg, f_hg)


htc_hg=Nusselt_T_hg*k_hg/D_h "heat transfer coefficient"

The calculations are repeated for the cold gas:

u_m_cg=m_dot_cg/(A_c*rho_cg) "mean velocity"


Re_cg=u_m_cg*D_h*rho_cg/mu_cg "Reynolds number"
Pr_cg=mu_cg*c_cg/k_cg "Prandtl number"
call DuctFlow_N(Re_cg,Pr_cg,L/D_h, (p_f-th_f)/h_c,0 [-]: Nusselt_T_cg, Nusselt_H_cg, f_cg)
htc_cg=Nusselt_T_cg*k_cg/D_h "heat transfer coefficient"

These calculations lead to hhg = 246 W/m2-K and hcg = 235.7 W/m2-K. When the average
(instead of the inlet) temperature is used to evaluate properties, the values are 221.2 and 207.3
W/m2-K, respectively.

b.) Compute the total conductance of the heat exchanger.

The resistance associated with convection to the hot gas is:

1
Rconv , hg = (10)
per L hhg

The resistance associated with fouling on the hot side is:

FF
R foul = (11)
per L
The resistance associated with conduction through the metal walls is:

thm
Rcond = (12)
km 2 L2 N c

The resistance associated with convection to the cold gas is:

1
Rconv ,cg = (13)
per L hcg

The total resistance between the hot and cold gas is:

Rtotal = Rconv , hg + R foul + Rcond + Rconv ,cg (14)

The conductance is:

1
UA = (15)
Rtotal

R_conv_hg=1/(per*L*htc_hg) "resistance to convection on hot gas side"


R_conv_cg=1/(per*L*htc_cg) "resistance to convection on cold gas side"
R_cond=th_w/(k_m*2*L*L*N_c) "conduction resistance"
R_foul=FF/(per*L) "fouling resistance"
R_total=R_conv_hg+R_foul+R_cond+R_conv_cg "total resistance"
UA=1/R_total "total conductance"

which leads to UA=458.2 W/K (with properties evaluated at the inlet fluid temperatures) and
415.7 W/K (with properties evaluated at the average fluid temperatures.)

c.) Determine the heat transferred in the heat exchanger and the temperature of the cold gas
leaving the heat exchanger.

The capacity rates of the hot and cold gas flows are computed:

C hg = m hg chg (16)

C cg = m cg ccg (17)

and the minimum and maximum values of the capacity rates ( C min and C max ) are determined.

C_dot_hg=m_dot_hg*c_hg "capacity rate of hot gas"


C_dot_cg=m_dot_cg*c_cg "capacity rate of cold gas"
C_dot_min=MIN(C_dot_hg,C_dot_cg) "minimum capacity rate"
C_dot_max=MAX(C_dot_hg,C_dot_cg) "maximum capacity rate"
The capacity ratio and number of transfer units are computed:

C
CR =  min (18)
Cmax

UA
NTU =  (19)
Cmin

and the function HX is used to determine the effectiveness of the heat exchanger (ε). Note that
the geometry of the passages makes this a clearly a heat exchanger where both sides are
unmixed:

C_R=C_dot_min/C_dot_max "capacity ratio"


NTU=UA/C_dot_min "number of transfer units"
epsilon=HX('crossflow_both_unmixed', NTU, C_dot_cg, C_dot_hg, 'epsilon') "access eff-NTU solution"

The maximum possible rate of heat transfer is computed:

qmax = C min (Thg ,in − Tcg ,in ) (20)

and the actual heat transfer rate is:

q = ε qmax (21)

q_dot_max=C_dot_min*(T_hg_in-T_cg_in) "maximum heat transfer rate"


q_dot=epsilon*q_dot_max "actual heat transfer rate"

which leads to q = 26.8 kW. The temperature of the cold gas leaving the heat exchanger is:

q
Tcg ,out = Tcg ,in +  (22)
Ccg

The temperature of the hot gas leaving the heat exchanger is:

q
Thg ,out = Thg ,in +  (23)
Chg

T_hg_out=T_hg_in-q_dot/C_dot_hg "hot gas exit temperature"


T_cg_out=T_cg_in+q_dot/C_dot_cg "cold gas exit temperature"

which leads to Thg,out = 313.4 K and Tcg,out =323.7 K (with properties evaluated at the inlet fluid
temperatures) and Thg,out =317.0 K and Tcg,out = 321.5 K (with properties evaluated at the average
fluid temperatures)
d.) Blowers are required to force the hot and cold flows through the heat exchanger. Assume
that you have blowers that are ηblower= 0.65 efficient. Estimate the total blower power
required to operate the energy recovery unit.

The pressure drops on the hot and cold sides of the heat exchanger are:

ρ hg um2 ,hg ⎛ L ⎞
Δphg = ⎜ f hg ⎟ (24)
2 ⎝ Dh ⎠

ρ cg um2 ,cg ⎛ L ⎞
Δpcg = ⎜ f cg ⎟ (25)
2 ⎝ Dh ⎠

The total blower power required is:

Δphg m hg Δpcg m cg
Wblower = + (26)
ρ hg ηblower ρcg ηblower

which leads to Wblower = 13.8 kW (with properties evaluated at the inlet fluid temperatures) and
13.3 kW (with properties evaluated at the average fluid temperatures).

e.) If you pay ec = 0.08$/kW-hr for electricity (to run the blowers) and 1.50$/therm for gas (to
heat the plant) then estimate the net savings associated with the energy recovery system
(neglect capital investment cost) for time = 1 year; this is the savings associated with the heat
transferred in the heat exchanger less the cost to run the blower for a year. Assume that the
plant runs continuously.

The cost to run the blowers is:

blowercost = Wblower time ec (27)

The savings associated with the heat transferred in the heat exchanger is:

heatsavings = q time hc (28)

The net savings per year is therefore:

netsavings = heatsavings − blowercost (29)

time=365 [day]*convert(day,s) "operating time"


ec=0.08 [$/kW-hr]*convert($/kW-hr,$/J) "cost of electricity"
hc=1.50 [$/therm]*convert($/therm,$/J) "cost of heat"
blowercost=ec*W_dot_blower*time "cost to run blower"
heatsavings=hc*q_dot*time "heating savings"
savingsnet=heatsavings-blowercost "net savings"
which leads to a net savings of 2,811 $/year (inlet temperatures used to evaluate properties) or
2,685$/year (average temperatures used to evaluate properties).

f.) Plot the net savings per year as a function of the mass flow rate of the cold air that is being
heated. Your plot should show a maximum; explain why this maximum exists.

Figure 2 illustrates the net savings as a function of the mass flow rate of the cold air being
heated. Also shown is the savings in heating and the cost to run the blower.

Figure 2: Net savings, blower power, and heating savings per year as a function of the mass flow rate
of cold gas.

Notice that the blower power increases dramatically with mass flow rate, particularly when the
flow becomes turbulent at approximately 0.46 kg/s. The heating savings increases with mass
flow rate as well but at a decreasing rate; this is because the heat exchanger effectiveness
decreases when the mass flow rate of air becomes very large. Note: when using the average
temperatures to evaluate properties, EES may not initially converge at low cold gas flows. A
useful trick to obtain convergence is to start the Parametric Table calculations at a cold gas flow
rate of 0.1 kg/s and do the calculations for flows up to 0.8 kg/s. Then do the calculations in the
Parametric table from the smallest flow rate to 0.1 kg/s in reverse. Be sure that the Update Guess
Values check box is checked. Alternatively evaluate properties at the inlet temperatures rather
than average temperatures.

g.) Determine the optimal values of the mass flow rate of combustion gas and cold gas ( m hg and
m cg ) that maximize the net savings per year. You should use the MIN/MAX capability in
EES to accomplish this - what is the maximum savings/year? This is the most you could
afford to pay for the blowers and heat exchanger if you wanted a 1 year pay-back.

The values of m hg and m cg are commented out:

{m_dot_hg=0.25 [kg/s] "mass flow rate of hot gas"}


{m_dot_cg=0.5 [kg/s] "mass flow rate of cold gas"}

and Min/Max is selected from the Calculate menu. The net savings is maximized by adjusting
the independent variables m_dot_hg and m_dot_cg (Figure 3).

Figure 3: Find Minimum or Maximum window.

Set the bounds on the two independent variables (Figure 4) to reasonable values.
Figure 4: Set bounds on independent variables.

The optimization routine indicates that the most optimal values of mass flow rates are m hg =
0.176 kg/s and m cg = 0.251 kg/s which leads to a net savings of $5748/year (with properties
estimated at the inlet temperatures).
Problem 8.6-1: Carbon Dioxide Cooler
This problem considers the heat rejection heat exchanger in a refrigeration cycle that uses carbon
dioxide as the refrigerant. Carbon dioxide is being considered as a replacement refrigerant for
automobile cooling and heat pump systems because it is non-flammable and it is a natural
refrigerant that does not contribute directly to ozone depletion or global warming concerns. The
critical temperature of carbon dioxide is about 304 K (31°C), which is often exceeded in an air
conditioning application. The critical pressure of carbon dioxide is about 7.4 MPa (74 atm).
Therefore, the pressure of the carbon dioxide within the heat rejection heat exchanger may range
from 80 to 120 atm; this very high pressure is one of the issues associated with using carbon
dioxide as a refrigerant.

The heat rejection heat exchanger in a carbon dioxide refrigeration cycle is called a cooler rather
than a condenser because the carbon dioxide is above its critical pressure and therefore it does
not condense, even when cooled to temperatures below the critical temperature. The specific
heat of carbon dioxide near its critical point is strongly temperature dependent and exhibits a
distinct ‘bump’ that becomes larger and sharper as the pressure approaches the critical pressure.
The carbon dioxide does not undergo a phase change and therefore the cooler cannot be modeled
using the technique discussed in Section 8.5. However, the specific heat cannot be assumed to
be constant in the cooler at a pressure of 80 or even 100 bar and therefore the ε-NTU solution for
a cross-flow heat exchanger cannot be used.

In this problem, you will develop a numerical solution where the cooler is discretized into sub-
heat exchangers that each operate over a small temperature span. If enough sub-heat exchangers
are used, then the temperature span is sufficiently small that the specific heat capacity within
each sub-heat exchanger can be assumed to be constant and therefore the performance of the
sub-heat exchangers can be predicted using the ε-NTU solution. This approach is similar to the
technique described in Section 8.6.3.

The finned, circular tube heat exchanger that is examined in EXAMPLE 8.1-1 and 8.1-2 is used
as the cooler. Clean dry air is forced to flow through the heat exchanger perpendicular to the
tubes (i.e., in cross-flow) with a volumetric flow rate VC = 0.06 m3/s. The inlet temperature of
the air is TC,in = 20°C and the air is at atmospheric pressure. Carbon dioxide enters the tubes at
TH,in = 60°C and PH,in = 80 bar with a mass flow rate m H = 0.01 kg/s.
a.) Determine the rate of heat transfer and the outlet state of the carbon dioxide.

The known information is entered into EES.

$UnitSystem SI MASS RAD PA K J


$Tabstops 0.2 0.4 0.6 3.5 in

"Inputs"
D_out=1.02 [cm]*convert(cm,m) "outer diameter of tube"
th = 0.9 [mm]*convert(mm,m) "tube wall thickness"
N_t_row=10 [-] "number of tube rows"
N_t_col=2 [-] "number of tube columns"
H=0.26 [m] "
W=0.2 [m] "width of heat exchanger face"
L=0.06 [m] "length of heat exchanger in air flow direction"
V_dot_C=0.06 [m^3/s] "volumetric flow rate of air"
P=1 [atm]*convert(atm,Pa) "atmospheric pressure"
T_C_in=convertTemp(C,K,20 [C]) "inlet air temperature"
T_H_in=convertTemp(C,K,60 [C]) "inlet carbon dioxide temperature"
m_dot_H=0.01 [kg/s] "carbon dioxide flow rate"
P_H_in=80 [bar]*convert(bar,Pa) "inlet carbon dioxide pressure"
s_v=25.4 [mm]*convert(mm,m) "vertical separation distance between tubes"
s_h=22 [mm]*convert(mm,m) "horizontal separation distance between tubes"
th_fin=0.33 [mm]*convert(mm,m) "fin thickness"
p_fin=3.18 [mm]*convert(mm,m) "fin pitch"
e=1.0 [micron]*convert(micron,m) "roughness of tube internal surface"

The important air-side parameters, including the average air-side heat transfer coefficient ( hout ),
overall surface efficiency (ηo), and air-side surface area (As,out) are copied from EXAMPLE 8.1-2
where they were calculated using the compact heat exchanger correlations.

"Air-side information (from EXAMPLE 8.1-1 and EXAMPLE 8.1-2"


h_bar_out=43.7 [W/m^2-K] "average air-side heat transfer coefficient"
eta_o=0.9713 [-] "surface efficiency of finned surface"
A_s_out=1.872 [m^2] "total surface area of finned surface"

The density and specific heat capacity of the air (ρair and cair) are determined using the inlet
temperature of the air. The capacity rate of the air flow is computed according to:

C C = VC ρ air cair (1)

"Air-side capacitance rate"


rho_air=density(Air,T=T_C_in,P=P) "air density"
c_air=cP(Air,T=T_C_in) "air specific heat capacity"
C_dot_C=c_air*rho_air*V_dot_C "air-side capacitance rate"

The total length of the heat exchanger tube is:

Ltube = N t ,row N t ,col W (2)

L_tube=N_t_row*N_t_col*W "total length of the heat exchanger tube"

The heat exchanger is divided into sub-heat exchanger sections by dividing the total length of
tube into N sections of equal length, as shown in Figure 1.

N=20 [-] "number of sub-heat exchangers"


Figure 1: Heat exchanger broken into N sections

The axial location associated with the interface between each section is:

xi =
( i − 1) L for i = 1.. ( N + 1) (3)
tube
N

duplicate i=1,(N+1)
x[i]=(i-1)*L_tube/N "axial location of the interface between each section"
end

The sections are analyzed sequentially, starting with section 1 where the carbon dioxide enters.
The temperature and pressure of the carbon dioxide entering section 1 are known:

TH 1 = TH ,in (4)

PH 1 = TH ,in (5)

and used to compute the enthalpy of the carbon dioxide entering the segment (iH,1) using the
internal property routines in EES:

"Inlet conditions to section 1"


T_H[1]=T_H_in
P_H[1]=P_H_in
i_H[1]=enthalpy(Carbondioxide,T=T_H[1],P=P_H[1]) "enthalpy entering section 1"

The process of developing the numerical model is simplified by using arrays for the properties of
each segment and a Duplicate loop to move through the heat exchanger section by section. The
inlet conditions to any section i are used to calculate the specific heat capacity that characterizes
the carbon dioxide within the section (cCO2,i). The capacitance rate of the carbon dioxide in the
section is determined according to:

C H i = cCO 2 i m H (6)

duplicate i=1,N
c_CO2[i]=cP(Carbondioxide,T=T_H[i],P=P_H[i]) "specific heat capacity of CO2 in section"
C_dot_H[i]=c_CO2[i]*m_dot_H "capacitance rate of CO2 in section"

The minimum and maximum heat capacity rates within the segment ( C min ,i and C max ,i ) are
computed.

C_dot_min[i]=MIN(C_dot_H[i],C_dot_C/N) "minimum capacitance rate in section"


C_dot_max[i]=MAX(C_dot_H[i],C_dot_C/N) "maximum capacitance rate in section"

It is not possible to solve the EES program at this point because there are not enough equations.
(Note that one strategy for developing the program would solve the equations only for the 1st
sub-heat exchanger before solving them for all of the sub-heat exchangers; this is accomplished
by replacing the N in the duplicate statement with 1 and, once terminated by an end statement,
allows each equation to be solved as it is entered). The PipeFlow_local function is used to
compute the local heat transfer coefficient associated with the flow of carbon dioxide within the
⎛ dP ⎞
section (hin,i) as well as the pressure gradient in the section ( ⎜ ⎟ ):
⎝ dx ⎠i

call PipeFlow_local('Carbondioxide',T_H[i],P_H[i],m_dot_H,D_out-2*th,(i-1/2)*L_tube/N,&
e/(D_out-2*th):h_in[i], h_H[i], dPdx[i])"evaluate local h and dPdx in the section"

Note that the position midway through the section is used to specify the location in the tube. The
total conductance within the section is computed:

1
UAi = (7)
N N
+
hout As ,out ηo Ltube π ( Dout − 2 th ) hin ,i

UA[i]=(N/(h_bar_out*A_s_out*eta_o)+N/(L_tube*pi*(D_out-2*th)*h_in[i]))^(-1)
"conductance in section"

The number of transfer units in the segment, NTUi, can be computed:

UA
NTU i =  i (8)
Cmin ,i

NTU[i]=UA[i]/C_dot_min[i] "number of transfer units in section"


and used to compute the effectiveness of the segment (εi) using the effectiveness-NTU
relationship for a cross-flow heat exchanger with both fluids unmixed, accessed using the HX
function in EES.

eff[i]=HX('crossflow_both_unmixed', NTU[i], C_dot_min[i], C_dot_max[i], 'epsilon')


"effectiveness of section"

The effectiveness is used to compute the heat transfer rate within the segment ( qi ):

qi = ε i C min ,i (TH ,i − TC ,in ) (9)

q_dot[i]=eff[i]*C_dot_min[i]*(T_H[i]-T_C_in)"heat transfer rate in section"

An energy balance on the refrigerant is used to compute the enthalpy of the carbon dioxide
leaving the segment:

qi = m H ( iH ,i − iH ,i +1 ) (10)

q_dot[i]=m_dot_H*(i_H[i]-i_H[i+1])"energy balance on CO2"

The pressure gradient in the section allows the calculation of the exit pressure:

⎛ dP ⎞ Ltube
PH i +1 = PH i + ⎜ ⎟ (11)
⎝ dx ⎠i N

P_H[i+1]=P_H[i]+dPdx[i]*L_tube/N"pressure drop across section"

The enthalpy and pressure of the carbon dioxide leaving the section can be used to calculate the
temperature:

T_H[i+1]=temperature(Carbondioxide,h=i_H[i+1],P=P_H[i+1]) "temperature of CO2 leaving section"


end

At this point it is possible to solve the equations to determine the temperature distribution in the
carbon dioxide as a function of position. The total heat transferred in the cooler is obtained by
summing the heat transferred in each section:

q_dot=sum(q_dot[i],i=1,N) "total heat transfer rate"


T_H_out=T_H[N+1] "exit temperature of the CO2"
T_H_out_C=converttemp(K,C,T_H_out) "in C"

Figure 2 illustrates the total heat transfer rate and the outlet carbon dioxide temperature as a
function of the number of heat exchanger segments; notice that significantly improved results are
achieved using just a few nodes. The heat transfer rate is about 830 W.
Figure 2: Calculated heat transfer rate and carbon dioxide outlet temperature as a function of
number of heat exchanger segments

Output from the EVAP-COND program, a detailed evaporator/condenser simulation program


that is available from the National Institute of Standards, is shown in Figure 3 using the default
heat transfer coefficient correlations provided by the program. The heat transfer rate determined
by EVAP-COND is 800 W.

Figure 3: Output from the EVAP-COND program for the carbon dioxide cooler
Problem 8.6-2 (8-13 in text): Joule Thomson Cycle
A Joule-Thomson refrigeration cycle is illustrated in Figure P8.6-2.

m = 0.01 kg/s
ph = 6.5 MPa
pc = 100 kPa Th ,in = 20°C
(h,in)
(c,out)

UA = 20 W/K

Tc,in = 150 K
(c,in)
qload (h,out)

(v,out) expansion valve


Figure P8.6-2: Joule-Thomson refrigeration cycle.

The system uses pure argon as the working fluid. High pressure argon enters a counterflow heat
exchanger with mass flow rate m = 0.01 kg/s at Th,in = 20ºC and ph = 6.5 MPa. The argon flows
through the heat exchanger where it is pre-cooled by the low pressure argon returning from the
cold end of the cycle. The high pressure argon leaving the heat exchanger enters an expansion
valve where it is expanded to pc = 100 kPa. The argon passes through a load heat exchanger
where it accepts a refrigeration load, qload , and it heated to Tc,in = 150 K. The conductance of the
heat exchanger is UA= 20 W/K. Neglect pressure loss in the heat exchanger on both the hot and
cold sides of the heat exchanger.
a.) Use the effectiveness-NTU method to estimate the effectiveness of the heat exchanger and
the rate of heat transferred from the hot to the cold stream in the heat exchanger. Calculate
the specific heat capacity of the high- and low-pressure argon using the average of the hot
and cold inlet temperatures.

The inputs are entered in EES:

$UnitSystem SI MASS RAD PA K J


$Tabstops 0.2 0.4 0.6 3.5 in

"Inputs"
m_dot=0.010 [kg/s] "mass flow rate rate"
T_h_in=converttemp(C,K,20[C]) "hot inlet temperature"
F$='Argon' "fluid"
P_h=6.5 [MPa]*convert(MPa,Pa) "high pressure"
P_c=100 [kPa]*convert(kPa,Pa) "low pressure"
T_c_in=150 [K] "cold inlet temperature"
UA=20 [W/K] "conductance"

The average temperature in the heat exchanger is computed according to:


Th ,in + Tc ,in
T = (1)
2

and used to compute the specific heat capacity of the hot and cold streams using EES' built-in
property routines (ch and cc):

"Solution based on eff-NTU method with average properties"


T_avg=(T_h_in+T_c_in)/2 "average temperature"
c_h_avg=cP(F$,P=P_h,T=T_avg) "average hot side specific heat capacity"
c_c_avg=cP(F$,P=P_c,T=T_avg) "average cold side specific heat capacity"

The capacitance rates on the hot and cold sides are computed according to:

C h = m ch (2)

C c = m cc (3)

The minimum and maximum capacitance rates ( C min and C max ) are computed using the MIN and
MAX functions.

C_dot_h_avg=c_h_avg*m_dot "hot-side capacitance rate"


C_dot_c_avg=c_c_avg*m_dot "cold-side capacitance rate"
C_dot_min=MIN(C_dot_h_avg,C_dot_c_avg) "minimum capacitance rate"
C_dot_max=MAX(C_dot_h_avg,C_dot_c_avg) "maximum capacitance rate"

The number of transfer units is computed according to:

UA
NTU =  (4)
Cmin

The effectiveness (ε) is computed using the effectiveness-NTU solution accessed by the HX
function in EES. The heat transfer rate in the heat exchanger is computed according to:

q = ε C min (Th,in − Tc ,in ) (5)

NTU=UA/C_dot_min "number of transfer units"


eff=HX('counterflow', NTU, C_dot_min, C_dot_max, 'epsilon')
"access eff-NTU solution for a counterflow heat exchanger"
q_dot=eff*C_dot_min*(T_h_in-T_c_in) "heat transfer rate"

which leads to ε = 0.877 and q = 656.5 W.

b.) Determine the refrigeration load provided by the cycle.


The enthalpy of the hot and cold inlet streams (ih,in and ic,in) are calculated using EES' internal
property routines at the inlet temperatures and pressures. The outlet enthalpies are calculated
using energy balances:

q
ih ,out = ih ,in − (6)
m

q
ic ,out = ic ,in + (7)
m

The outlet temperatures (Th,out and Tc,out) are calculated at the outlet enthalpies and pressures
using EES' internal property routines.

i_h_in=enthalpy(F$,P=P_h,T=T_h_in) "inlet hot-side enthalpy"


i_c_in=enthalpy(F$,P=P_c,T=T_c_in) "inlet cold-side enthalpy"
i_h_out=i_h_in-q_dot/m_dot "outlet hot-side enthalpy"
i_c_out=i_c_in+q_dot/m_dot "outlet cold-side enthalpy"
T_h_out=temperature(F$,h=i_h_out,P=P_h) "hot outlet temperature"
T_c_out=temperature(F$,h=i_c_out,P=P_c) "cold outlet temperature"

The valve is isenthalpic, therefore:

ih ,out = iv ,out (8)

The temperature of the fluid leaving the valve (Tv,out) is computed at the valve outlet enthalpy
and the cold-side pressure using EES' internal property functions. The refrigeration load is:

qload = m ( ic ,in − iv ,out ) (9)

i_v_out=i_h_out "expansion valve exit enthalpy"


T_v_out=temperature(F$,P=P_c,h=i_v_out) "expansion valve exit temperature"
q_dot_load=m_dot*(i_c_in-i_v_out) "load"

which leads to qload = 31.2 W.

c.) Prepare a plot of refrigeration load as a function of cold inlet temperature for 85 K < Tc,in <
290 K and various values of the conductance. A negative refrigeration load is not physically
possible (without some external cooling); therefore, terminate your plots at qload = 0 W.

Figure P8.6-2(b) illustrates the refrigeration load as a function of cold inlet temperature for
various values of the UA. Notice that when the cold inlet temperature is less than the saturation
temperature for Argon at pc (87.2 K), the refrigeration load drops to zero.
125

100

Refrigeration load (W)


30 W/K
75

25 W/K
50
20 W/K

25 15 W/K

UA = 10 W/K

0
100 125 150 175 200 225 250 275
Cold inlet temperature (K)
Figure 8.6-2(b): Refrigeration load predicted by the ε-NTU method as a function of cold inlet temperature for
various values of UA.

d.) Instead of using the effectiveness-NTU method, divide the heat exchanger into sub-heat
exchangers as discussed in Section 8.6.3. What is the heat transferred in the heat exchanger
for the conditions listed in the problem statement?

The solution using the ε-NTU technique is commented out. The sub-heat exchanger model for
the counter-flow heat exchanger is shown in Figure 8.6-2(c).

TH,1 HX 1 TH,2 HX 2 TH,3 TH,N HX N TH,N+1


m
q q q
...
TC,1 N TC,2 N TC,3 TC,N N TC,N+1
m
Figure 8.6-2(c): Sub-heat exchanger model of a counter-flow heat exchanger.

The hot-side exit temperature (TH,out) is assumed:

T_h_out= 200 [K] "guess for the hot outlet temperature"

and used to compute the total heat transfer rate in the heat exchanger:

q = m ( ih ,in − ih ,out ) (10)

where ih,in and ih,out are the specific enthalpy at the inlet and outlet states (ih,in and ih,out),
respectively, calculated using EES' internal property routine.

i_h_in=enthalpy(F$,P=P_h,T=T_h_in) "inlet hot-side enthalpy"


i_h_out=enthalpy(F$,T=T_h_out,P=P_h) "hot outlet enthalpy"
q_dot=m_dot*(i_h_in-i_h_out) "heat transfer rate"
The cold-side inlet enthalpy (ic,in) is computed at the inlet temperature and pressure. The cold-
side outlet enthalpy (ic,out) is computed according to:

q
ic ,out = ic ,in + (11)
m

The cold-side outlet temperature (Tc.out) can be obtained using EES' internal property routine:

i_c_in=enthalpy(F$,P=P_c,T=T_c_in) "inlet cold-side enthalpy"


i_c_out=i_c_in+q_dot/m_dot "cold outlet enthalpy"
T_c_out=temperature(F$,P=P_c,h=i_c_out) "cold outlet temperature"

The total heat transferred in the heat exchanger increases as you move from left-to-right in
Figure P8.6-2(c) according to:

q
qi = ( i − 1) for i = 1.. ( N + 1) (12)
N

N=10 [-] "number of sub-heat exchangers"


duplicate i=1,N
q_dot[i]=i*q_dot/N "total heat transfer rate"
end

The temperatures Th,1 and Tc,1 are the hot-side inlet and cold-side outlet fluid inlet temperatures,
respectively, for the counter-flow configuration. The enthalpies ih,1 and ic,1 are the associated
specific enthalpies associated with these states.

T_h[1]=T_h_in "hot-side inlet temperature"


T_c[1]=T_c_out "cold-side outlet temperature"
i_h[1]=i_h_in "hot_side inlet enthalpy"
i_c[1]=i_c_out "cold-side outlet enthalpy"

An energy balance on the hot-side provides the enthalpy leaving each of the sub-heat
exchangers:

q
ih ,i +1 = ih ,i − for i = 1..N (13)
N m

The temperature of the hot-side fluid leaving each sub-heat exchanger (Th,i) is obtained from the
enthalpy and pressure using EES' internal property routine:

duplicate i=2,(N+1)
i_h[i]=i_h[i-1]-q_dot/(N*m_dot) "energy balance on hot-side of each sub-heat exchanger"
T_h[i]=temperature(F$,h=i_h[i],P=p_h) "temperature leaving hot-side of each sub-heat exchanger"
end
An energy balance on the cold-side provides the enthalpy leaving each of the sub-heat
exchangers:

q
ic ,i +1 = ic ,i − for i = 1..N (14)
N m

The temperature of the cold-side fluid leaving each sub-heat exchanger (Tc,i) is obtained from the
enthalpy and pressure using EES' internal property routine:

duplicate i=2,(N+1)
i_c[i]=i_c[i-1]-q_dot/(N*m_dot) "energy balance on cold-side of each sub-heat exchanger"
T_c[i]=temperature(F$,h=i_c[i],P=p_c) "temperature leaving cold-side of each sub-heat exchanger"
end

The ε-NTU solution can be applied to each of the sub-heat exchangers. The capacitance rates on
the hot- and cold-side within each sub-heat exchanger are calculated:

C h ,i = m
(i
h ,i − ih ,i +1 )
for i = 1..N (15)
(Th ,i − Th ,i +1 )

C c ,i = m
(i
c ,i − ic ,i +1 )
for i = 1..N (16)
(Tc ,i − Tc ,i +1 )

duplicate i=1,N
C_dot_h[i]=m_dot*(i_h[i]-i_h[i+1])/(T_h[i]-T_h[i+1]) "hot-side capacitance rate"
C_dot_c[i]=m_dot*(i_c[i]-i_c[i+1])/(T_c[i]-T_c[i+1]) "cold-side capacitance rate"
end

The effectiveness of each sub-heat exchanger can be computed:

q / N
εi = for i = 1..N (8-17)
( )
MIN C c ,i , C c ,i (Th ,i − Tc ,i +1 )

The number of transfer units required by each sub-heat exchanger (NTUi) is obtained using the ε-
NTU solution for a counter-flow heat exchanger, implemented by the function HX. The
conductance required in each sub-heat exchanger is:

UAi = NTU i MIN C C ,i , C H ,i ( ) (8-18)

duplicate i=1,N
eff[i]=q_dot/(N*MIN(C_dot_h[i],C_dot_c[i])*(T_h[i]-T_c[i+1])) "effectiveness of sub-heat exchanger"
NTU[i]=HX('counterflow', eff[i], C_dot_h[i], C_dot_c[i], 'NTU') "NTU required by sub-heat exchanger"
UA[i]=NTU[i]*MIN(C_dot_h[i],C_dot_c[i]) "conductance in sub-heat exchanger"
end
The conductance for each of the sub-heat exchangers is summed to obtain the total conductance
required by the heat exchanger for the assumed inlet temperature:

N
UA = ∑ UAi (8-19)
i =1

UA_p=sum(UA[1..N]) "total conductance"

The error between the specified and predicted conductance is defined.

err=abs(UA-UA_p) "error between calculated and specified conductance"

The assumed value of the hot exit temperature is commented out:

{T_h_out= 200 [K]} "guess for the hot outlet temperature"

and the error is minimized using the Min/Max command by varying Th,out (note, the option Stop
if error occurs should not be checked so that the optimization process continues even if the value
of Th,out goes out of reasonable bounds leading to non-physical values of ε for some sub-heat
exchangers). The calculation leads to q =636.6 W, which is only 3% in error relative to the
value calculated in (a).

e.) Determine the refrigeration load associated with your prediction from (d).

The valve is isenthalpic, therefore:

ih ,out = iv ,out (20)

The temperature of the fluid leaving the valve (Tv,out) is computed at the valve outlet enthalpy
and the cold-side pressure using EES' internal property functions. The refrigeration load is:

qload = m ( ic ,in − iv ,out ) (21)

i_v_out=i_h_out "expansion valve exit enthalpy"


T_v_out=temperature(F$,P=P_c,h=i_v_out) "expansion valve exit temperature"
q_dot_load=m_dot*(i_c_in-i_v_out) "load"

which leads to qload = 11.3 W.

f.) Overlay on your plot from (c) the refrigeration load as a function of cold inlet temperature for
the same values of the conductance.

Figure P8.6-2(d) illustrates the refrigeration load as a function of Tc,in for the two models at
various values of UA. Notice that the deviation between the models is largest when the
temperatures are low and the non-ideal gas behavior (i.e., the specific heat capacity variations)
are largest.
125
sub-heat exchanger method
30 W/K
ε -NTU method
25 W/K
100

Refrigeration load (W)


75

20 W/K
50
15 W/K

25
UA = 10 W/K

0
100 125 150 175 200 225 250 275
Cold inlet temperature (K)
Figure 8.6-2(d): Refrigeration load predicted by the ε-NTU method and the sub-heat exchanger model as a
function of cold inlet temperature for various values of UA.
Problem 8.6-3 (8-14 in text)
A counter-flow heat exchanger has a total conductance of UA = 130 W/K. Air flows on the hot
side. The air enters at Th,in = 500 K with pressure ph = 1 atm and mass flow rate m h = 0.08 kg/s.
Carbon dioxide flows on the cold side. The CO2 enters at Tc,in = 300 K with pressure pc = 80 atm
and mass flow rate m c = 0.02 kg/s.
a.) Plot the specific heat capacity of air at 1 atm and carbon dioxide at 80 atm and comment on
whether the ε-NTU solution can be applied to this heat exchanger.

The problem information is entered into EES.

$UnitSystem SI MASS RAD PA K J


$Tabstops 0.2 0.4 0.6 3.5 in

"Inputs"
H$='Air_ha' "hot side fluid"
C$='CarbonDioxide' "cold side fluid"
T_h_in=500 [K] "hot inlet temperature"
p_h=1 [atm]*convert(atm,Pa) "hot fluid pressure"
m_dot_h=0.08 [kg/s] "hot fluid mass flow rate"
T_c_in=300 [K] "cold inlet temperature"
p_c_atm=80 [atm] "cold fluid pressure, in atm"
p_c=p_c_atm*convert(atm,Pa) "cold fluid pressure"
m_dot_c=0.02 [kg/s] "cold fluid mass flow rate"
UA=130 [W/K] "total conductance of heat exchanger"

To answer the question in part (a), it is necessary to prepare a Parametric table that includes the
specific heats of carbon dioxide at 80 atm and air at 1 atm evaluated over the temperature range
between 300 and 500 K.

"Generate cp data in Parametric table 1"


c_h=cp(H$,T=T,p=p_h) "specific heat of hot fluid"
c_c=cp(C$,T=T,p=p_c) "specific heat of cold fluid"

Figure 1 illustrates the specific heat capacities of air and carbon dioxide as a function of
temperature and shows that the specific heat for the carbon dioxide varies substantially with
temperature, necessitating a numerical analysis of the heat exchanger.
14000

Specific heat capacity (J/kg-K)


12000 CO2 at 80 atm

10000

8000

6000

4000

2000
air at 1 atm
0
300 350 400 450 500
Temperature (K)
Figure 1: Specific heat capacity of CO2 at 80 atm and air at 1 atm as a function of temperature.

b.) Prepare a solution to this problem by numerically integrating the governing equations using
the Euler technique, as discussed in Section 8.6.2.

The Euler analysis begins by choosing the number of nodes and specifying the dimensionless
position of each node.

xi =
( i − 1) (1)
( N − 1)
The dimensionless distance between adjacent nodes is:

1
Δx = (2)
( N − 1)
N=50 [-] "number of nodes"
Dx_bar=1/(N-1) "distance between nodes"
duplicate i=1,N
x_bar[i]=(i-1)*Dx_bar "dimensionless position"
end

An energy balance on a differential section of the hot flow leads to:

dTh −UA
=  (Th − Tc ) (3)
dx Ch

where C h is the capacitance rate of the hot fluid which is a function of the hot fluid temperature.
An energy balance on a differential section of the cold flow leads to:
dTc −UA
=  (Th − Tc ) (4)
dx Cc

where C c is the capacitance rate of the cold fluid. Equations (3) and (4) are the state equations
for the problem which will be integrated from the hot inlet side ( x = 0) to the hot outlet side ( x =
1). An estimate for the cold exit temperature, Tc,out, is assumed in order to begin the integration.
This equation will later be removed after the cold exit temperature is calculated.

T_c_out=490 [K] "assumed cold exit temperature"


T_h[1]=T_h_in "hot inlet temperature"
T_c[1]=T_c_out "assumed cold outlet temperature"

The integration is accomplished using Euler steps:

Th ,i +1 = Th,i − 
UA
Ch ,T =Th ,i
(Th,i − Tc,i ) Δx for i = 1.. ( N − 1) (5)

Tc ,i +1 = Tc ,i −

UA
Cc ,T =Tc ,i
(Th,i − Tc,i ) Δx for i = 1.. ( N − 1) (6)

duplicate i=1,(N-1)
C_dot_h[i]=cP(H$,T=T_h[i],P=p_h)*m_dot_h "capacity rate of hot fluid"
C_dot_c[i]=cP(C$,T=T_c[i],P=p_c)*m_dot_c "capacity rate of cold fluid"
T_h[i+1]=T_h[i]-UA*(T_h[i]-T_c[i])*Dx_bar/C_dot_h[i] "Euler step for hot fluid"
T_c[i+1]=T_c[i]-UA*(T_h[i]-T_c[i])*Dx_bar/C_dot_c[i] "Euler step for cold fluid"
end

The equations should be solved and the guess values updated. The assumed value of Tc,out is
commented out and an error function is defined as the absolute value between the calculated
value of Tc,in and the specified value of Tc,in:

err = Tc , N − Tc ,in (7)

{T_c_out=490 [K]} "assumed cold exit temperature"


err=abs(T_c[N]-T_c_in) "minimize err to find the value of T_c_out"

Select MinMax from the Calculate window. Minimize err by varying T_c_out as shown in Figure
2. Reasonable bounds (300 K to 500 K) must be specified for T_c_out. The optimization will
determine that Tc,out = 493.4 K.
Figure 2: Find Minimum or Maximum dialog.

c.) Using your solution from (b), plot the temperature of the carbon dioxide and air as a function
of the dimensionless axial position (x/L).

Figure 3 illustrates Tc and Th as a function of x .

Figure 3: Temperature distribution within the heat exchanger.

d.) Plot the rate of heat transfer predicted by the model as a function of the number of integration
steps.

An energy balance on the cold stream provides the heat transfer rate.

q = m c ( ic ,out − ic ,in ) (8)


where ic,out and ic,in are the specific enthalpies of the cold fluid evaluated at Tc,out and Tc,in,
respectively.

i_c_out=enthalpy(C$,p=p_c,T=T_c_out) "enthalpy of cold fluid exit"


i_c_in=enthalpy(C$,p=p_c,T=T_c_in) "enthalpy of cold fluid inlet"
q_dot=m_dot_c*(i_c_out-i_c_in) "heat transfer in heat exchanger"

Construct a Parametric table to include the variables N, q , Tc,out, and err. Use the MinMax Table
selection from the Calculate menu to minimize err for each value of N. Set the bounds on Tc,out
to 450 and 500 K to facilitate the optimization. Figure 4 illustrates the predicted value of the
total heat transfer rate as a function of the number of nodes and indiates that about 30 integration
steps are required.
7,900

7,800
Heat transfer rate (W)

7,700

7,600

7,500

7,400

7,300
0 10 20 30 40 50
Number of integration steps
Figure 4: Predicted heat transfer rate as a function of the number of integration steps.

e.) Prepare a solution to this problem by sub-dividing the heat exchanger into sub-heat
exchangers, as discussed in Section 8.6.3.

Comment out the Euler solution.

{N=50 [-] "number of nodes"


Dx_bar=1/(N-1) "distance between nodes"
duplicate i=1,N
x_bar[i]=(i-1)*Dx_bar "dimensionless position"
end

{T_c_out=490 [K]} "assumed cold exit temperature"


T_h[1]=T_h_in "hot inlet temperature"
T_c[1]=T_c_out "assumed cold outlet temperature"
duplicate i=1,(N-1)
C_dot_h[i]=cP(H$,T=T_h[i],P=p_h)*m_dot_h "capacity rate of hot fluid"
C_dot_c[i]=cP(C$,T=T_c[i],P=p_c)*m_dot_c "capacity rate of cold fluid"
T_h[i+1]=T_h[i]-UA*(T_h[i]-T_c[i])*Dx_bar/C_dot_h[i] "Euler step for hot fluid"
T_c[i+1]=T_c[i]-UA*(T_h[i]-T_c[i])*Dx_bar/C_dot_c[i] "Euler step for cold fluid"
end
err=abs(T_c[N]-T_c_in) "minimize err to find the value of T_c_out"

i_c_out=enthalpy(C$,p=p_c,T=T_c_out) "enthalpy of cold fluid exit"


i_c_in=enthalpy(C$,p=p_c,T=T_c_in) "enthalpy of cold fluid inlet"
q_dot=m_dot_c*(i_c_out-i_c_in) "heat transfer in heat exchanger"}

The sub-heat exchanger model for the counter-flow configuration is shown in Figure 5.

Figure 5: Sub-heat exchanger model of a counter-flow heat exchanger.

The number of sub-heat exchangers (N) is selected hot-side exit temperature (TH,out) is assumed

"Sub-heat exchanger solution"


N=10 [-] "number of sub-heat exchangers"
T_h_out=425 [K] "assumed hot outlet temperature"

The assumed value of TH,out is used to compute the total heat transfer rate in the heat exchanger:

q = m H ( iH ,in − iH ,out ) (9)

where iH,in and iH,out are the specific enthalpy at the inlet and outlet states, respectively, calculated
using EES' internal property routine:

i_h_in=enthalpy(H$,T=T_h_in,P=p_h) "enthalpy of hot inlet fluid"


i_h_out=enthalpy(H$,T=T_h_out,P=p_h) "enthalpy of hot outlet fluid"
q_dot=m_dot_h*(i_h_in-i_h_out) "total heat transfer rate"

The temperature TH,1 and enthalpy iH,1 are the hot inlet temperature and the associated enthalpy.

"Obtain enthalpy & temperature distribution"


T_h[1]=T_h_in "hot-side inlet temperature"
i_h[1]=i_h_in "hot_side inlet enthalpy"

An energy balance on the hot-side fluid provides the enthalpy leaving each of the sub-heat
exchangers:

q
iH ,i = iH ,i −1 − for i = 2..N + 1 (10)
N m H

The temperature of the hot-side fluid leaving each sub-heat exchanger (TH,i) is obtained from the
enthalpy and pressure using EES' internal property routine:
duplicate i=2,(N+1)
i_h[i]=i_h[i-1]-q_dot/(N*m_dot_h) "energy balance on hot-side of each sub-heat exchanger"
T_h[i]=temperature(H$,h=i_h[i],P=p_h) "temperature leaving hot-side of each sub-heat exchanger"
end

The temperature TC,N+1 and enthalpy iC,N+1 are the cold inlet temperature and the associated
enthalpy.

T_c[N+1]=T_c_in "cold side inlet temperature"


i_c[N+1]=enthalpy(C$,p=p_c,T=T_c_in) "cold side inlet enthalpy"

An energy balance on the cold-side provides the enthalpy leaving each of the sub-heat
exchangers:

q
iC ,i = iC ,i +1 + for i = 1..N (11)
N m C

The temperature of the cold-side fluid leaving each sub-heat exchanger (TC,i) is obtained from
the enthalpy and pressure using EES' internal property routine:

duplicate i=1,N
i_c[i]=i_c[i+1]+q_dot/(N*m_dot_c) "energy balance on cold-side of each sub-heat exchanger"
T_c[i]=temperature(C$,h=i_c[i],P=p_c) "temperature leaving cold-side of each sub-heat exchanger"
end

The ε-NTU solution can be applied to each of the sub-heat exchangers. The capacitance rates on
the hot- and cold-side within each sub-heat exchanger are calculated:

C H ,i = m H
(i
H ,i − iH ,i +1 )
for i = 1..N (12)
(TH ,i − TH ,i +1 )

C C ,i = m C
(i
C ,i − iC ,i +1 )
for i = 1..N (13)
(TC ,i − TC ,i +1 )

"Apply effectiveness-NTU solution"


duplicate i=1,N
C_dot_h[i]=m_dot_h*(i_h[i]-i_h[i+1])/(T_h[i]-T_h[i+1]+0.001 [K])"hot-side capacitance rate"
C_dot_c[i]=m_dot_c*(i_c[i]-i_c[i+1])/(T_c[i]-T_c[i+1]+0.001 [K]) "cold-side capacitance rate"
end

The effectiveness of each sub-heat exchanger can be computed:

q / N
εi = for i = 1..N (14)
( )
MIN CC ,i , CH ,i (TH ,i − TC ,i +1 )
 
The number of transfer units required by each sub-heat exchanger (NTUi) is obtained using the ε-
NTU solution for a counter-flow heat exchanger, implemented by the function HX. The
conductance required in each sub-heat exchanger is:

(
UAi = NTU i MIN C C ,i , C H ,i ) (15)

duplicate i=1,N
eff[i]=q_dot/(N*MIN(C_dot_h[i],C_dot_c[i])*(T_h[i]-T_c[i+1])) "effectiveness of sub-heat exchanger"
NTU[i]=HX('counterflow', eff[i], C_dot_h[i], C_dot_c[i], 'NTU') "NTU required by sub-heat exchanger"
UA[i]=NTU[i]*MIN(C_dot_h[i],C_dot_c[i]) "conductance in sub-heat exchanger"
end

The conductance of the heat exchanger is the sum of the conductances of the individual heat
exchangers:

N
UAreq = ∑ UAi (16)
i =1

The problem is solved and the guess values are updated. An error function is defined as the
difference between the predicted and specified heat exchanger conductance:

errU = UA − UAreq (17)

UA_req=sum(UA[1..N]) "total conductance of heat exchanger"


err=abs(UA-UA_req) "error"

The assumed value of Th,out is commented out and the value of errU is minimized by adjusting
Th,out using the Min/Max selection from the Calculate menu.

{T_h_out=425 [K]} "assumed hot outlet temperature"

This leads to Th,out = 403.9 K.

f.) Overlay on your plot from (c) the temperature distribution predicted by your model from (e).

Knowing the conductance within for each subsection allows the dimensionless position between
each sub-heat exchanger to be computed.

x1 = 0 (18)

UAi
xi +1 = xi + for i = 2..N (19)
UAreq

x_bar[1]=0
duplicate i=1,N
x_bar[i+1]=x_bar[i]+UA[i]/UA_req "fraction of total UA"
end

Figure 6 illustrates the temperature of the cold and hot fluid as a function of dimensionless
position predicted by integration of the state equations in (a) and by division into sub-heat
exchangers in (e).
500

475

450
Temperature (K)

425

400
integration of state equations
375 sub-heat exchanger model

350

325

300
0 0.1 0.2 0.3 0.4 0.5 0.6 0.7 0.8 0.9 1
Dimensionless position
Figure 6: Temperature distribution within the heat exchanger obtained by integration of the state equations
and by division into sub-heat exchangers.

g.) Overlay on your plot from (d) the rate of heat transfer predicted by your model from (e) as a
function of the number of sub-heat exchangers.

Figure 7 illustrates the total heat transfer predicted by the sub-heat exchanger model as a
function of the number of sub-heat exchangers overlaid onto Figure 4. Figure 7 illustrates that
the method of using sub-heat exchangers allows many fewer nodes to obtain a stable accurate
result.

7900

7800
Heat transfer rate (W)

7700

7600

7500
integration of governing equations
sub-heat exchanger model
7400

7300
0 10 20 30 40 50
Number of integration steps
Figure 7: Heat transfer rate as a function of N predicted by the models from (a) and (d).
Problem 8.7-1: Axial Conduction in a Concentric Pipe Heat Exchanger
Nitrogen gas at Tn,in = 300 K and atmospheric pressure is cooled by helium gas at Th,in = 240 K
and atmospheric pressure in a concentric tube heat exchanger that utilizes Ntube= 30 tubes in
parallel. The mass flow rate of the nitrogen is m n = 3 g/s and the mass flow rate of helium is
m h = 1.5 g/s. The heat exchanger is made of copper with pipe wall thickness of δ = 1 mm and
length L = 0.15 m.. The inner pipe has an inner diameter of Din = 0.3 cm. The outer pipe (which
is well-insulated on its outside surface) has an inner diameter of Dout = 1.0 cm. The roughness of
the pipe surfaces are e = 1 µm.. The helium flows through the annulus and the nitrogen through
the center tube in a counterflow configuration.
a) Estimate the outlet temperatures of the helium and nitrogen and the heat transfer rate
assuming there is no axial conduction.

Enter the known information into EES.

$UnitSystem SI MASS RAD PA K J


$Tabstops 0.2 0.4 0.6 3.5 in

"known information"
m_dot_n=0.003 [kg/s] "mass flow rate of the nitrogen"
m_dot_h=0.0015 [kg/s] "mass flow rate of the helium"
T_n_in=300 [K] "inlet temperature of the nitrogen"
T_h_in=240 [K] "inlet temperature of the helium"
P=101.3 [kPa]*convert(kPa,Pa) "atmospheric pressure"
D_in=0.3 [cm]*convert(cm,m) "inner diameter of inner pipe"
D_out=1.0 [cm]*convert(cm,m) "inner diameter of outer pipe"
delta=1 [mm]*convert(mm,m) "pipe wall thickness"
L=0.15 [m] "length of heat exchanger"
e=1e-6 [m] "roughness of walls"
N_tube=30 [-] "number of parallel tubes"

The outlet temperatures of the nitrogen and helium (Tn,out and Th,out, respectively) are assumed in
order to compute the average temperatures ( Tn and Th ) which are used to determine the
properties of the fluids. The specific heat capacity of the nitrogen and helium (ch and cn) are
computed and used to determine the capacitance rates:

C n = cn m n (1)

C h = ch m h (2)

The minimum capacitance rate ( C min ) is identified.

T_n_out=280 [K] "guess for outlet nitrogen temperature"


T_h_out=280 [K] "guess for outlet helium temperature"
T_n_avg=(T_n_in+T_n_out)/2 "average temperature of the nitrogen"
T_h_avg=(T_h_in+T_h_out)/2 "average temperature of the helium"
c_n=cp(N2,T=T_n_avg) "specific heat of the nitrogen"
c_h=cp(He,T=T_h_avg) "specific heat of the helium"
C_dot_n=c_n*m_dot_n "capacitance rate of the nitrogen"
C_dot_h=c_h*m_dot_h "capacitance rate of the helium"
C_dot_min=min(C_dot_n,C_dot_h) "minimum capacitance rate"

The conductivity of the copper tube (kt) is determined and used to compute the resistance to
conduction through the tube (radially):

⎛ D + 2δ ⎞
ln ⎜ in ⎟
Rcond ,t = ⎝ Din ⎠
(3)
N tube 2 π kt L

k_t=k_('Copper', T=(T_h_in+T_n_in)/2) "thermal conductivity of copper"


R_cond_t=ln((D_in+2*delta)/D_in)/(N_tube*pi*k_t*L) "resistance to conduction radially through the tube"

The PipeFlow procedure is used to compute the average heat transfer on the nitrogen side ( hn ).
The resistance to convection on the nitrogen side is:

1
Rconv ,n = (4)
hn N tube π Din L

call PipeFlow('N2',T_n_avg,P,m_dot_n/N_tube,D_in,L,e/D_in:h_bar_n, h_H_n ,DELTAP_n, &


Nusselt_T_n, f_n, Re_n) "determine h_bar_n"
R_conv_n=1/(h_bar_n*N_tube*pi*D_in*L) "resistance to convection from the nitrogen"

The AnnularFlow procedure is used to compute the average heat transfer coefficient on the helium
side ( hh ). The resistance to convection on the helium side is:

1
Rconv , h = (5)
hh N tube π ( Din + 2 δ ) L

r_in=(D_in+2*delta)/2 "inner radius of annulus"


r_out=D_out/2 "outer radius of annulus"
call AnnularFlow('He',T_h_avg,P,m_dot_h/N_tube,r_in,r_out,L,e/D_in:h_bar_h, h_H_h ,&
DELTAP_h, Nusselt_T_h, f_h, Re_h) "determine h_bar_h"
R_conv_h=1/(h_bar_h*N_tube*pi*(D_in+2*delta)*L) "resistance to convection from the helium"

The total resistance is:

Rtotal = Rconv , h + Rcond ,t + Rconv ,n (6)

The conductance is:

1
UA = (7)
Rtotal

The number of transfer units is:


UA
NTU =  (8)
Cmin

The effectiveness in the absence of axial conduction (εnac) is obtained from the HX procedure.
The rate of heat transfer is calculated according to:

q = ε nac C min (Tn ,in − Th ,in ) (9)

R_total=R_conv_n+R_conv_h+R_cond_t "sum of the resistances"


UA=1/R_total "R is the overall resistance between the nitrogen and helium"
NTU=UA/C_dot_min "definition of NTU"
eff_nac=HX('counterflow', NTU, C_dot_h, C_dot_n, 'epsilon')
"effectiveness assuming no axial conduction"
q_dot=eff_nac*C_dot_min*(T_n_in-T_h_in) "heat transfer rate mechanism equation"

The problem is solved and the guess values are updated. The assumed values of the outlet
temperatures are commented out and recalculated using energy balances:

q = C n (Tn ,in − Tn,out ) (10)

q = C h (Th ,out − Th ,in ) (11)

{T_n_out=280 [K] "guess for outlet nitrogen temperature"


T_h_out=280 [K] "guess for outlet helium temperature"}
q_dot=C_dot_n*(T_n_in-T_n_out) "energy balance on the nitrogen"
q_dot=C_dot_h*(T_h_out-T_h_in) "energy balance on the helium"

which leads to εnac = 0.4655.

b) Calculate the axial conduction parameter and determine whether axial conduction is a
concern for this heat exchanger. If so, use an approximate method to include axial
conduction in the estimates of the heat transfer rate and outlet temperatures.

The resistance to conduction along the length of the heat exchanger is:

L
Rac = (12)
N tube π ⎡( Din + 2 δ ) − Din2 ⎤ kt
2
⎣ ⎦

The axial conduction parameter is:

1
λ= (13)
Rac C min
R_ac=L/(N_tube*pi*((D_in/2+delta)^2-(D_in/2)^2)*k_t) "resistance to conduction in axial direction"
lambda=1/(R_ac*C_dot_min) "axial conduction parameter"

which leads to λ = 0.326. Therefore, axial conduction is important for this heat exchanger.

We will use all three of the approximate methods; however, the high lambda model is likely the
most appropriate and most accurate for this case. The effectiveness computed using the low
lambda model is:

ε low,λ = ε nac − λ (14)

eff_low=eff_nac-lambda "for low lambda"

which leads to εlow,λ = 0.137.

The effectiveness using the high lambda model is obtained by computing the number of transfer
units on the nitrogen and helium sides:

1
NTU n = (15)
Rconv ,n C n

1
NTU h = (16)
Rconv ,h C h

The effectiveness on the nitrogen and helium sides are computed according to:

ε n = 1 − exp ( − NTU n ) (17)

ε h = 1 − exp ( − NTU h ) (18)

The high lambda effectiveness is:

1
ε high ,λ = (19)
⎡ 1 1 ⎤
C min ⎢  +  ⎥
⎣ Cn ε n Ch ε h ⎦

NTU_n=1/(R_conv_n*C_dot_n) "NTU on nitrogen (tube) side"


NTU_h=1/(R_conv_h*C_dot_h) "NTU on helium (annulus) side"
eff_n=1-exp(-NTU_n) "effectiveness on hot side"
eff_h=1-exp(-NTU_h) "effectiveness on cold side"
eff_high=1/(C_dot_min*(1/(C_dot_n*eff_n)+1/(C_dot_h*eff_h))) "for high lambda"

which leads to εhigh,λ = 0.440.


Finally, the temperature jump model is implemented according to:

Rac C h C n Tn ,in + C h Th ,in + C n Tn ,in


Tn*,in = (20)
R C C + C + C
ac h n h h

Th*,in = Tn*,in − Rac C n (Tn ,in − Tn*,in ) (21)

ε TJ = ( ε nac +λ)
(T *
n ,in − Th*,in )
(22)
(T n ,in − Th ,in )

"Temperature jump approximation"


T_n_in_star=(R_ac*C_dot_n*C_dot_h*T_n_in+C_dot_h*T_h_in+C_dot_n*T_n_in)&
/(R_ac*C_dot_h*C_dot_n+C_dot_h+C_dot_n)
T_h_in_star=T_n_in_star-R_ac*C_dot_n*(T_n_in-T_n_in_star)
eff_tj=(eff_nac+lambda)*(T_n_in_star-T_h_in_star)/(T_n_in-T_h_in)

which leads to εTJ = 0.542.

c) Use the numerical analysis provided in the EES AxialConductionHX library to determine the
effectiveness including axial conduction. Comment on the accuracy of the approximate
method you employed in part (b).

The actual effectiveness of the heat exchanger (ε) is computed using the AxialConductionHX
function in EES with a concentration factor, γ = 2.

gamma=2 "concentration parameter for node distribution"


Call AxialConductionHX( C_dot_n, C_dot_h, NTU_n, NTU_h, lambda,gamma: eff, eff_nac2)

which leads to ε = 0.455. The accuracy of the high lambda model is pretty good, εhigh,λ = 0.440.
However, the other models are not accurate.
Problem 8.7-2
A parallel plate heat exchanger is made using thm = 3 mm thick copper plates to form 100
channels. The vertical space between any two plates is H = 6 mm. The cold fluid is argon gas at
atmospheric pressure that enters at TC,in = 90 K. The hot fluid is nitrogen at atmospheric pressure
that enters at TH,in = 300 K. The flow rates of both fluids is m H = m C = 25 g/s. Each plate in the
heat exchanger extends L = 0.7 m in the flow direction and is W = 0.35 m wide. The heat
exchanger is counterflow.
a) Estimate the convective heat transfer coefficients for the hot and cold fluids.

The input parameters are entered in EES:

$UnitSystem SI MASS RAD PA K J


$Tabstops 0.2 0.4 0.6 3.5 in

L=0.70 [m] "length of heat exchanger in flow direction"


W=0.35 [m] "width of heat exchanger"
N_ch=100 [-] "number of channels (-)"
T_H_in=300 [K] "hot-side inlet temperature"
T_C_in=90 [K] "cold-side inlet temperature"
m_dot_H=0.025 [kg/s] "hot-side mass flow rate"
m_dot_C=0.025 [kg/s] "cold-side mass flow rate"
th_m=0.3 [mm]*convert(mm,m) "thickness of plates"
P=101.3 [kPa]*convert(kPa,Pa) "pressure"
H=6 [mm]*convert(mm,m) "height of the channels"

The outlet temperatures on the hot- and cold-sides (TH,out and TC,out) are assumed. The average
temperatures on either side are used to compute the properties:

T_H_out=T_C_in "guess for hot-side outlet temperature"


T_C_out=T_H_in "guess for cold-side outlet temperature"
T_H_avg=(T_H_in+T_H_out)/2 "average temperature on hot-side"
T_C_avg=(T_C_in+T_C_out)/2 "average temperature on cold-side"

The DuctFlow procedure is used to compute the average heat transfer coefficients on either side
( hH and hC ). The resistance to convection on both sides are computed according to:

1
Rconv , H = (1)
N ch hH W L

1
Rconv ,C = (2)
N ch hC W L

The total conductance is:

1
UA = (3)
Rconv , H + Rconv ,C
RelRough=0 [-] "assume plates are smooth"
call DuctFlow('Nitrogen',T_H_avg, P, m_dot_H/(N_ch/2), H, W, L, RelRough : h_bar_H, h_H_H
,DELTAP_H,&
Nusselt_T_H, f_H, Re_H)
call DuctFlow('Argon',T_C_avg, P, m_dot_C/(N_ch/2), H, W, L, RelRough : h_bar_C, h_H_C
,DELTAP_C,&
Nusselt_T_C, f_C, Re_C)
R_conv_H=1/(N_ch*h_bar_H*W*L) "resistance to convection on hot side"
R_conv_C=1/(N_ch*h_bar_C*W*L) "resistance to convection on cold side"
UA=1/(R_conv_H+R_conv_C) "conductance"

The specific heat capacities of the two streams are determined (cH and cC) and used to compute
the capacitance rates:

C H = cH m H (4)

CC = cC m C (5)

The minimum capacitance rate ( C min ) is identified and used to compute the number of transfer
units.

UA
NTU = (6)
C min

The effectiveness of the heat exchanger in the absence of axial conduction (εnac) is obtained
using the HX function in EES.

c_H=cp(Argon,T=T_H_avg,P=P) "hot-side fluid specific heat capacity"


c_C=cp(Nitrogen,T=T_C_av,P=Pg) "cold-side fluid specific heat capacity"
C_dot_H=m_dot_H*C_H "hot fluid capacitance rate"
C_dot_C=m_dot_C*C_C "cold fluid capacitance rate"
C_dot_min=min(C_dot_H,C_dot_C) "minimum capacitance rate"
NTU=UA/C_dot_min "NTU definition"
eff_nac=HX('counterflow', Ntu, C_dot_C, C_dot_H, 'epsilon') "effectiveness for counterflow operation"

The problem is solved and the guess values are updated and the assumed values of TH,out and
TC,out are commented out. The heat transfer rate is computed according to:

q = ε nac C min (TH ,in − TC ,in ) (7)

and used to recalculate the hot and cold outlet temperatures.

q = C H (TH ,in − TH ,out ) (8)

q = C C (TC ,out − TC ,in ) (9)


{T_H_out=T_C_in "guess for hot-side outlet temperature"
T_C_out=T_H_in "guess for cold-side outlet temperature"}
q_dot=eff_nac*C_dot_min*(T_H_in-T_C_in) "rate of heat transfer"
q_dot=C_dot_H*(T_H_in-T_H_out) "energy balance on hot side"
q_dot=C_dot_C*(T_C_out-T_C_in) "energy balance on cold side"

which leads to hH = 11.33 W/m2-K, hC = 8.09 W/m2-K, and εnac = 0.9358.

c) Determine the value of the axial conduction parameter.

The resistance to axial conduction is:

L
Rac = (10)
km W thm N ch

where km is the conductivity of the metal evaluated at the average of the inlet temperatures. The
axial conduction parameter is:

1
λ= (11)
Rac C min

k_m=k_('Copper',T=(T_H_in+T_C_in)/2) "conductivity of copper"


R_ac=L/(k_m*W*N_ch*th_m) "resistance to axial conduction"
lambda=1/(R_ac*C_dot_min) "axial conduction parameter"

which leads to λ = 0.480; this is a very large number and therefore axial conduction is likely very
important.

d) Correct your result in (b) using one of the approximate methods in Section 8.7.2.

The effectiveness using the high lambda model is obtained by computing the number of transfer
units on the hot and cold sides:

1
NTU H = (12)
Rconv , H C H

1
NTU C = (13)
Rconv ,C C C

The effectiveness on the hot and cold sides are computed according to:

ε H = 1 − exp ( − NTU H ) (14)

ε C = 1 − exp ( − NTU C ) (15)


The high lambda effectiveness is then:

1
ε high ,λ = (16)
⎡ 1 1 ⎤
C min ⎢  +  ⎥
⎣ CH ε H CC ε C ⎦

NTU_H=1/(C_dot_H*R_conv_H) "NTU on hot side"


NTU_C=1/(C_dot_C*R_conv_C) "NTU on cold side"
eff_H=1-exp(-NTU_H) "effectiveness on hot side"
eff_C=1-exp(-NTU_C) "effectiveness on cold side"
eff_high=1/(C_dot_min*(1/(C_dot_H*eff_H)+1/(C_dot_C*eff_C))) "high lambda approximate model"

which leads to εhigh,λ = 0.527. The effectiveness using the temperature jump model is
implemented according to:

Rac C C C H TH ,in + C C TC ,in + C H TH ,in


TH* ,in = (17)
R C C + C + C
ac C H C H

TC*,in = TH* ,in − Rac C H (TH ,in − TH* ,in ) (18)

ε TJ = ( ε nac +λ)
(T *
H ,in − TC*,in )
(19)
(T H ,in − TC ,in )

"Temperature jump approximation"


T_H_in_star=(R_ac*C_dot_C*C_dot_H*T_H_in+C_dot_C*T_C_in+C_dot_H*T_H_in)/&
(R_ac*C_dot_C*C_dot_H+C_dot_C+C_dot_H)
T_C_in_star=T_H_in_star-R_ac*C_dot_H*(T_H_in-T_H_in_star)
eff_TJ=(eff_nac+lambda)*(T_H_in_star-T_C_in_star)/(T_H_in-T_C_in)

which leads to εTJ = 0.741.

e) Use the numerical analysis provided in the EES AxialConductionHX library to determine the
effectiveness including axial conduction. Comment on the accuracy of the approximate
method you employed in (d).

The EES AxialConductionHX procedure is used to determine the effectiveness (ε) with a
concentration factor γ = 2.

gamma=2 "concentration parameter for node distribution"


Call AxialConductionHX( C_dot_H, C_dot_C, NTU_H, NTU_C, lambda,gamma: eff, eff_nac2)

which leads to ε = 0.747. The temperature jump model does a reasonable job for this case.
Problem 8.10-1 (8-15 in text)
A solar heating system is shown in Figure P8.10-1.

heated air
′′
qsolar rock bed rock bed

collector

blower
blower

air
air
charging process discharging process
Figure P8.10-1: Solar heating system during charging and during discharging.

During the day, the solar heat is not required and therefore air is blown through a series of solar
collectors where it is heated as shown in Figure P8.10-1. The thermal energy is stored in a large
rock bed regenerator. The rock bed is L= 20 ft long in the flow direction and W = 10 ft x W = 10
ft in cross-sectional area. The bed is filled with Dp = 0.5 inch diameter rocks with density ρr =
100 lbm/ft3 and specific heat capacity cr = 0.2 Btu/lbm-°F. The charging process goes on for
tcharge =12 hr. There are Ncol = 40 solar collectors, each with area 8 ft x 4 ft. During the charging
process, atmospheric air at Tindoor = 70°F enters the collectors where it is heated by the solar
′′ = 750 W/m2. The efficiency of the collector is given by: ηcollector = 0.75 - 0.0015
irradiation qsolar
[K-1] (Tr,in - Toutdoor) where Toutdoor = 10°F and Tr,in is the temperature of the air leaving the
collector and entering the regenerator. The collector efficiency is the ratio of the energy
transferred to the air to the energy incident on the collector. During the night, the energy that
was stored in the rock bed is used to provide heating, as shown in Figure P8.10-1. Air at Tindoor =
70°F enters the rock bed where it is heated. The hot air is provided to the building. The blower
used during both the charging and discharging process has an efficiency of ηb = 0.6 and a
pressure/flow curve that goes linearly from Δpdh = 0.5 inch of water at zero flow to Vopen = 1800
cfm at zero pressure rise. Neglect the pressure drop across the collectors and assume that the
pressure drop that must be overcome by the blower is related to the flow through the rock bed.
The porosity of the rock bed is φ = 0.35 and assume that the rock bed is well-insulated.
a.) What is the temperature of the air entering the rock bed during the charging process and the
mass flow rate of air during the charging and discharging process?

The inputs are entered in EES:

$UnitSystem SI MASS RAD PA K J


$Tabstops 0.2 0.4 0.6 3.5 in

"Inputs"
D_p=0.5 [inch]*convert(inch,m) "rock diameter"
L_ft=20 [ft] "bed length, in ft"
L=L_ft*convert(ft,m) "bed length"
W=10 [ft]*convert(ft,m) "bed width & height"
rho_r=100 [lbm/ft^3]*convert(lbm/ft^3,kg/m^3) "rock density"
c_r=0.2 [Btu/lbm-F]*convert(Btu/lbm-F,J/kg-K) "rock specific heat capacity"
T_outdoor=converttemp(F,K,10 [F]) "outdoor air temperature"
T_indoor=converttemp(F,K,70 [F]) "indoor air temperature"
N_col=40 [-] "number of collectors"
A_collector=8 [ft]*4 [ft]*N_col*convert(ft^2,m^2) "installed collector area"
q``_solar=750 [W/m^2] "solar irradiation"
p_a=1 [atm]*convert(atm,Pa) "air pressure"
t_charge=12 [hr]*convert(hr,s) "charge time"
eta_b=0.6 [-] "blower efficiency"
DELTAP_dh=0.5 [inH2O]*convert(inH2O,Pa) "deadhead pressure drop for blower"
V_dot_open=1800 [ft^3/min]*convert(ft^3/min,m^3/s) "open flow rate"

The volumetric flow rate ( Va ) and the temperature leaving the collector (Tr,in) are not known;
both values are assumed and these assumptions are relaxed based on subsequent calculations.

V_dot_cfm=1200 [ft^3/min] "assumed volumetric flow rate in bed, in cfm"


V_dot=V_dot_cfm*convert(ft^3/min,m^3/s) "volumetric flow rate in bed"
T_r_in=400 [K] "assumed value of hot inlet temperature"

The density and specific heat capacity of air (ρa and ca, respectively) are calculated using EES'
internal property routines. The mass flow rate of air is computed according to:

m a = Va ρ a (1)

The collector efficiency is computed using the equation given in the problem statement.

rho_a=density(Air,T=T_indoor,P=p_a) "density of air"


c_a=cP(Air,T=T_indoor) "specific heat capacity of air"
m_dot_a=rho_a*V_dot "mass flow rate of air"
eta_collector=0.75 [-] - 0.0015 [1/K]*(T_r_in-T_outdoor) "collector efficiency"

The assumed value of the collector exit temperature is commented out and an energy balance on
the collector is used to compute Tr,in:

{T_r_in=400 [K]} "assumed value of hot inlet temperature"


T_r_in=T_indoor+q``_solar*A_collector*eta_collector/(m_dot_a*c_a)
"predicted regenerator inlet temperature during charging process"

The correlations for friction factor and heat transfer coefficient for a packed bed of spheres are
accessed using the PackedSpheres procedure in EES, which returns the average friction factor,
average heat transfer, and the number of transfer units ( f , h , and NTU). The mass flux is
computed according to:

m a
G= (2)
φW 2
The characteristic radius of the flow channels is computed according to:

φ Dp
rchar = (3)
4 (1 − φ )

The pressure drop across the bed is computed according to:

G2 f L
Δp = (4)
2 ρ a rchar

call PackedSpheres('Air',m_dot_a, D_p, W^2, L, (T_r_in+T_indoor)/2, p_a: f_bar, h_bar, NTU)


"access correlations for packed spheres"
G=m_dot_a/(phi*W^2) "mass flux"
phi=0.35 [-] "
r_char=phi*D_p/(4*(1-phi)) "characteristic radius"
DELTAP=G^2*f_bar*L/(2*rho_a*r_char) "pressure drop across bed"
DELTAP_inH2O=DELTAP*convert(Pa,inH2O) "in inH2O"

The assumed value of the volumetric flow rate is commented out and the pressure drop is
calculated using the pressure/flow curve specified by the problem statement:

{V_dot_cfm=1200 [ft^3/min]} "assumed volumetric flow rate in bed, in cfm"


DELTAP=DELTAP_dh*(1-V_dot/V_dot_open) "blower pressure rise"

The blower power is calculated according to:

ΔpV
w blower = (5)
ηb

w_dot_b=DELTAP*V_dot/eta_b "blower power"

These calculates lead to Tr,in = 389.6 K and m a = 0.519 kg/s.

b.) What is the amount of heat transfer from the rock bed to the air during the discharge process?

The volume of the regenerator material is:

Vr = W 2 L (1 − φ ) (6)

The capacitance of the matrix is:

Cmatrix = ρ r cr Vr (7)

The capacitance rate of the air is:


C a = m a ca (8)

The ε-NTU solution for a balanced, symmetric regenerator is accessed using the HX function in
order to compute the effectiveness of the heat exchanger (ε):

V_r=W^2*L*(1-phi) "volume of regenerator material"


C_dot=m_dot_a*c_a "capacitance rate of the air flow"
C_matrix=rho_r*c_r*V_r "capacitance of the matrix"
epsilon=HX('Regenerator', NTU, C_dot, C_matrix/t_charge, 'epsilon')
"effectiveness-NTU correlation for a regenerator"

The maximum possible amount of heat transfer is:

Qmax = m a ca (Tr ,in − Tindoor ) tcharge (9)

and the actual amount of heat transfer is:

Q = ε Qmax (10)

Q_max=m_dot_a*c_a*t_charge*(T_r_in-T_indoor) "maximum possible heat transfer"


Q=Q_max*epsilon "actual heat transfer in regenerator"

which leads to Q = 2.12x109 J.

c.) There are 100 heating days per year in this location. What is the total amount of heating
energy saved over a 10 year period?

The total amount of energy saved is:

Q tanalysis
Qsaved = (11)
2 tcharge

where tanalysis is the total amount of time considered in the analysis.

Nhday=100 [day] "number of heating days"


t_analysis=10 [year]*convert(year,s)*Nhday/365[day] "time of analysis"
Q_saved=Q*t_analysis/(2*t_charge) "total saved energy"

which leads to Qsaved = 2.12x1012 J.

d.) If the cost of natural gas is gc = 3.5$/therm then what is the total heating cost saved over a 10
year period (neglect the time value of money for this analyis)?

The heating cost avoided is:


ES = Qsaved gc (12)

gc=3.5 [$/therm]*convert($/therm,$/J) "cost of natural gas"


ES=Q_saved*gc "value of saved energy"

which leads to ES = $70,430.

e.) The cost of the solar collectors is cc = 45$/ft2 and the cost of the rock bed is rc = 40$/lbm.
The cost of the electrical energy required to run the blowers is ec = 0.12$/kW-hr. Determine
the net savings associated with owning the equipment over a 10 year period.

The cost to run the blowers is:

WC = w b ec tanalysis (13)

The cost of the collectors is:

ColC = Acollector cc (14)

The cost of the rock bed is:

BC = LW 2 (1 − φ ) ρ r rc (15)

The net saving is therefore:

LCS = ES − WC − BC − ColC (16)

ec=0.12 [$/kW-hr]*convert($/kW-hr,$/J) "cost of electricity"


cc=45 [$/ft^2]*convert($/ft^2,$/m^2) "collector cost"
rc=40 [$/ton]*convert($/ton,$/kg) "cost of rocks"
WC=w_dot_b*t_analysis*ec "cost of electricity to run blowers"
BC=L*W^2*(1-phi)*rho_r*rc "cost of rock bed"
ColC=A_collector*cc "cost of collectors"
LCS=ES-BC-ColC-WC "life-cycle savings associated system"

which leads to LCS = $10,100.

f.) Plot the net savings as a function of the number of solar collectors. You should see that an
optimal number of collectors exists. Explain this fact.

Figure P8.10-1(b) illustrates the net savings as a function of the number of solar collectors. The
optimal value exists because the collector efficiency decreases with the number of solar
collectors because the exit temperature increases.
12000

10000

Net savings ($)


8000

6000

4000

2000

0
0 20 40 60 80
Number of collectors
Figure P8.10-1(b): Net savings as a function of the number of collectors.

g.) Plot the net savings as a function of the length of the rock bed (with Ncol = 40). You should
see that an optimal length of the rock bed. Explain this fact.

Figure P8.10-1(c) illustrates the net savings as a function of the length of the rock bed. The
optimal value exists because the rock bed heat capacity increases to the point where it becomes
larger than the capacity of the air and thus its effectiveness approaches unity (the NTU is always
high for any reasonable value of length). As the length increases further, the blower power
increases due to increased pressure drop.
14000

12000

10000
Net savings ($)

8000

6000

4000

2000

0
0 20 40 60
Rock bed length (ft)
Figure P8.10-1(c): Net savings as a function of the rock bed length.

h.) Determine the optimal number of collectors and rock bed length.

Using the Min/Max function from the Calculate menu, the optimal number of collectors and
length is Ncol = 41 and L = 13.5 ft which leads to a net savings of $11,780.
Problem 8.10-2 (8-16 in text): Stirling Cycle Analysis
A Stirling engine is shown in Figure P8.10-2.

hot piston WH
TH = 800 K
expansion space
QH

regenerator
stainless steel screens
Dr = 10 cm
L = 20 cm
ds = 1 mm
m = 500 m-1
QC

compression space TC = 300 K


cold piston
WC
Figure P8.10-2: Stirling engine.

The mass of gas in the Stirling engine is Mgas = 0.01 kg. The gas is air and can be modeled as
being an ideal gas with gas constant Ra = 287.1 J/kg-K and specific heat capacity ratio γ = 1.4.
You may neglect the air entrained in the regenerator void volume and assume that all of the air is
either in the compression or expansion space. The Stirling engine's performance will be
estimated using a very simple model of the Stirling cycle. During the compression process, all of
the gas is contained in the compression space and the cold piston is moved up until the pressure
of the air goes from Plow = 1.0 MPa to Phigh = 1.5 MPa. This process occurs isothermally at TC =
300 K and will be modeled as being reversible. During the cold-to-hot blow process, the two
pistons move together so that the gas moves from the compression space to the expansion space.
To the extent that the regenerator is not 100% effective, the gas leaves the hot end of the
regenerator at a temperature that is below TH = 800 K and therefore a heat transfer occurs from
the hot reservoir in order to heat this gas to TH; this heat transfer is the manifestation of the
regenerator loss. During the expansion process, all of the gas is contained in the expansion space
and the hot piston is moved up until the pressure of the air goes from Phigh to Plow. This process
occurs isothermally at TH and will also be modeled as being reversible. During the hot-to-cold
blow process, the two pistons move together so that the gas moves from the expansion space to
the compression space. The cycle occurs with a frequency of f = 10 Hz and each of the four
processes take an equal amount of time.
a.) What is the efficiency of the cycle and the average power produced in the absence of any
regenerator loss?

Enter the specified information into EES.

$UnitSystem SI MASS RAD PA K J


$Tabstops 0.2 0.4 0.6 3.5 in

"Inputs"
P_low=1 [MPa]*convert(MPa,Pa) "low pressure"
P_high=1.5 [MPa]*convert(MPa,Pa) "high pressure"
M_gas=0.01 [kg] "mass of gas"
freq=10 [Hz] "frequency of operation"
T_H=800 [K] "hot temperature"
T_C=300 [K] "cold temperature"
gamma=1.4 [-] "ratio of specific heat capacities"
R_a=287.1 [J/kg-K] "gas constant"

An entropy balance on the gas in the cold space during the compression process leads to:

QC
ΔS = (1)
TC

where ΔS is the change in entropy and QC is the amount of heat rejected to the reservoir at TC.
The air is modeled as an ideal gas with constant specific heat capacity:

⎡ ⎤
⎢ ⎛ TC ⎞ ⎛ Phigh ⎞ ⎥
ΔS = M g ⎢c p , a ln ⎜ ⎟ − Ra ln ⎜ ⎟⎥ (2)
⎢  ⎝

TC ⎠

⎝ Plow ⎠ ⎥
⎢⎣ =0 ⎥⎦

Substituting Eq. (2) into Eq. (1) leads to:

⎛ Phigh ⎞
QC = TC M gas Ra ln ⎜ ⎟ (3)
⎝ Plow ⎠

An energy balance on the compression process leads to:

WC = Δ
NU + QC (4)
=0

where ΔU is the change in energy of the gas (which is zero since the process occurs
isothermally) and WC is the total work transfer to the gas.

"Ideal cycle analysis"


Q_C=-M_gas*R_a*T_C*ln(P_high/P_low) "heat transfer during compression"
W_C=Q_C "work required by compression"

An entropy and energy balance on the expansion process leads to:

⎛ Phigh ⎞
QH = TH M gas Ra ln ⎜ ⎟ (5)
⎝ Plow ⎠

WH = QH (6)

where QH is the heat transfer to the expansion space and WH is the work transfer from the
expansion space.
Q_H=M_gas*R_a*T_H*ln(P_high/P_low) "heat transfer during expansion"
W_H=Q_H "work to expander"

The net work transfer from the Stirling engine is:

Wnet = WH − WC (7)

and the efficiency of the cycle is:

Wnet
ηnoloss = (8)
QH

which is equivalent to the Carnot efficiency since no entropy production mechanisms have been
modeled. The average power is:

w net = Wnet f (9)

W_net=W_H+W_C "net work transfer"


eta_noloss=W_net/Q_H "efficiency"
eta_Carnot=(T_H-T_C)/T_H "Carnot efficiency"
power=W_net*freq "average power production"

which leads to ηnoloss = 0.625 and w net = 5820 W.

The regenerator is a cylinder filled with stainless steel screens. The regenerator diameter is Dr =
10 cm and the length is L = 20 cm. The screens have wire diameter ds = 1 mm and mesh m =
500 m-1.
b.) Estimate the regenerator loss per cycle and the efficiency including this regenerator loss.

The additional inputs are entered in EES.

"Regenerator loss"
D_r=10 [cm]*convert(cm,m) "regenerator diameter"
L=20 [cm]*convert(cm,m) "regenerator length"
d_s=1 [mm]*convert(mm,m) "screen diameter"
m=500 [1/m] "mesh"
t_B=1/(4*freq) "blow time"

The average temperature and pressure during the cycle are used to compute the properties. The
mass flow rate associated with the blow processes is:

M gas
m = (10)
tB

where tB is the blow-time, taken to be 25% of the cycle time.


T_avg=(T_H+T_C)/2 "average temperature"
p_avg=(P_low+P_high)/2 "average pressure"
m_dot=M_gas/t_b "mass flow rate of gas through the packing"

The Screens procedure is used to determine the heat transfer coefficient between the air and the
screens ( h ) and the number of transfer units (NTU). The porosity of the screen matrix is
computed according to:

π
φ = 1− m ds (11)
4

The density and specific heat capacity of the stainless steel (ρr and cr) are computed at the
average temperature.

call Screens('Air', m_dot, d_s, m, pi*D_r^2/4, L, T_avg, P_avg: f_bar, h_bar, NTU)
"access correlations for screens"
phi=1-pi*m*d_s/4 "porosity"
rho_r=rho_('Stainless_AISI304', T_avg) "density of screen material"
c_r=c_('Stainless_AISI304', T_avg) "specific heat capacity of screen material"

The total heat capacity of the matrix is:

Dr2
Cmatrix = (1 − φ ) π L ρ r cr (12)
4

The specific heat capacity of the air is computed and used to evaluate the capacitance rate of the
air that is pushed through the matrix:

C a = c p , a m (13)

The utilization is computed:

C a t B
U= (14)
Cmatrix

cP_a=cp(Air,T=T_avg) "specific heat of air"


C_dot_a=cP_a*m_dot "capacity rate of air"
U=C_dot_a*t_B/C_matrix "utilization"

The effectiveness of the balanced symmetric regenerator (ε) is obtained using the HX function in
EES. The maximum possible heat transfer during the blow processes is:

Qmax = C a t B (TH − TC ) (15)

and the actual heat transfer during the blow process is:
Q = ε Qmax (16)

The difference between Qmax and Q is heat transfer that must be provided by the hot reservoir
and is therefore a loss:

QH ,loss = Qmax − Q (17)

The efficiency including the loss is:

Wnet
ηloss = (18)
QH + QH ,loss

eff=HX('Regenerator', NTU, C_dot_a, C_matrix/t_B, 'epsilon')


"effectiveness-NTU solution for regenerator"
Q_max=C_dot_a*t_B*(T_H-T_C) "maximum heat transfer per cycle"
Q=Q_max*eff "actual heat transfer per cycle"
Q_H_loss=Q_max-Q "heat transfer required to overcome regenerator loss"
eta_wloss=W_net/(Q_H+Q_H_loss) "efficiency with loss"

which leads to QH,loss = 759.1 J and ηloss = 0.344.

c.) Plot the efficiency and average power as a function of the frequency of the Stirling engine.
Explain the shape of your plot.

Figure 2 illustrates the efficiency and average power as a function of the frequency. As the
frequency increases, the efficiency drops because the mass flow rate during the blow process
increases (i.e., you are pushing the same amount of gas through the regenerator in a shorter time)
and therefore the regenerator effectiveness drops as the NTU drops. However, the average power
increases with frequency because the cycle time drops and therefore there are more strokes each
giving nearly the same work per time.

Figure 2: Efficiency and power as a function of frequency.


Problem 8.1-1 (8-1 in text)
Dry air at Ta,in = 30°C, and atmospheric pressure is blown at ua = 1.0 m3/s through a cross-flow
heat exchanger in which refrigerant R134a is evaporating at a constant pressure of pR = 345 kPa.
The air exits the heat exchanger at Ta,out = 13°C. The tubes and fins of the heat exchanger are
both made of copper. The tubes have an outer diameter of Dout,t = 1.64 cm and tht = 1.5 mm tube
wall thickness. The fins are circular with a spacing that leads to 275 fins per meter, an outer
diameter of Dout,f = 3.1 cm and a thickness of thf = 0.25 mm. The heat transfer coefficient
between the R134a and the inner tube wall is estimated to be hR = 2,500 W/m2-K. The heat
transfer coefficient between the air and tubes and fins is estimated to be ha = 70 W/m2-K. The
total length of finned tubes is L = 110 m.
a) Determine the rate of heat transfer from the air.

Enter the known information into EES.

$UnitSystem SI MASS RAD PA K J


$Tabstops 0.2 0.4 0.6 3.5 in

"known information"
T_air_in=converttemp(C,K,30 [C]) "inlet air temperature"
P=101.3 [kPa]*convert(kPa,Pa) "atmospheric pressure"
V_dot=1.0 [m^3/s] "volumetric flowrate of air"
P_evap=345 [kPa]*convert(kPa,Pa) "pressure of evaporating R134a"
T_air_out=converttemp(C,K,10 [C]) "exit air temperature"
D_o=0.0164 [m] "outer diameter of tube"
WallThk=0.0015 [m] "pipe wall thickness"
D_fin=0.031 [m] "outer diameter of fins"
y_fin=0.00025 [m] "thickness of fins"
Fin_pitch=275 [1/m] "spacing of fins"
L=110 [m] "length of tubing"
h_bar_i=2500 [W/m^2-K] "two-phase heat transfer coefficient is high"
h_bar_o=70 [W/m^2-K] "outside heat transfer coefficient - varies with air velocity"

The mass flow rate of the air can be determined by multiplying the density of the inlet air by the
volumetric flow rate.

m = ρ air ,in V (1)

where ρair,in is the density of the air at the inlet condition. The heat transfer rate is evaluated
using an energy balance on the air:

q = m ( iair ,in − iair ,out ) (2)

where iair,in and iair,out are the specific enthalpy of the air at the inlet and exit temperatures,
respectively.

"Determine the heat transfer rate"


m_dot=density(Air,T=T_air_in,P=P)*V_dot "mass flow rate"
i_air_in=enthalpy(Air,T=T_air_in) "specific enthalpy of entering air"
i_air_out=enthalpy(Air,T=T_air_out) "specific enthalpy of exiting air"
q_dot=m_dot*(i_air_in-i_air_out) "energy balance on air"

b) Determine the value of the heat exchanger conductance for this heat exchanger.

The inner and outer radii of the tube are computed according to:

Do
ro = (3)
2

ri = ro − th (4)

The thermal resistance to convection from the inside surface of the tube is:

1
Ri = (5)
hi 2 π ri L

r_o=D_o/2 "outer radius of tube"


r_i=r_o-WallThk "inner radius of tube"
Res_i=1/(h_bar_i*2*pi*r_i*L) "resistance between evaporating fluid and tube wall"

The resistance to conduction through the tube is:

⎛r ⎞
ln ⎜ o ⎟
Rt = ⎝ i ⎠
r
(6)
2π k L

where k is the thermal conductivity of copper evaluated at the average air temperature.

T_air_avg=(T_air_in+T_air_out)/2 "average air temperature for evaluation of properties"


k=k_('Copper', (T_air_avg)/2) "thermal conductivity of copper tube"
Res_t=ln(r_o/r_i)/(2*pi*k*L) "resistance of copper wall is very small"

The fin efficiency (ηfin) is obtained using the EES function eta_fin_annular_rect. The surface area
of the fins is:

⎡⎛ D fin ⎞ 2 ⎤
Afin = π ⎢⎜ ⎟ − ro ⎥ 2 L FinPitch
2
(7)
⎢⎣⎝ 2 ⎠ ⎥⎦

and the thermal resistance of the fins is:

1
R fin = (8)
η fin Afin ho
eta=eta_fin_annular_rect(y_fin, r_o, D_fin/2, h_bar_o, k) "fin efficiency"
A_fin=pi*((D_fin/2)^2-r_o^2)*2*L*Fin_pitch "finned area"
Res_fin=1/(eta*h_bar_o*A_fin) "resistance of fins"

The area of the unfinned region is:

Aunfin = 2 π ro L (1 − FinPitch y fin ) (9)

and the thermal resistance of the unfinned area is:

1
Runfin = (10)
Aunfin ho

A_unfin=2*pi*r_o*L*(1-Fin_pitch*y_fin) "unfinned area"


Res_unfin=1/(A_unfin*h_bar_o) "resistance of unfinned surface"

The total thermal resistance is:

−1
⎡ 1 1 ⎤
R = Ri + Rt + ⎢ + ⎥ (11)
⎣⎢ R fin Runfin ⎦⎥

The conductance is:

1
UA = (12)
R

Res=Res_i+Res_t+(1/Res_fin+1/Res_unfin)^(-1) "total resistance"


UA=1/Res "conductance"

which leads to UA = 2122 W/K.


Problem 8.1-2 (8-2 in text)
The cross-flow heat exchanger described in Problem 8.1-1 (8-1 in text) has geometry similar to
that for compact heat exchanger 'fc_tubes_sCF-70-58J'. The frontal area of the heat exchanger is
Af = 0.5 m2 and the length of the heat exchanger in the flow direction is W = 0.25 m.
a.) Use the compact heat exchanger library to estimate the air-side conductance and the overall
heat exchanger conductance assuming that the heat transfer coefficient between the R134a
and the inner tube wall is hR = 2,500 W/m2-K.

Enter the known information into EES.

$UnitSystem SI MASS RAD PA K J


$Tabstops 0.2 0.4 0.6 3.5 in

"known information"
T_air_in=converttemp(C,K,30 [C]) "inlet air temperature"
P=101.3 [kPa]*convert(kPa,Pa) "atmospheric pressure"
V_dot=1.0 [m^3/s] "volumetric flowrate of air"
P_evap=345 [kPa]*convert(kPa,Pa) "pressure of evaporating R134a"
T_air_out=converttemp(C,K,10 [C]) "exit air temperature"
h_bar_i=2500 [W/m^2-K] "two-phase heat transfer coefficient is high"
A_f=0.5 [m^2] "frontal area of the heat exchanger"
W=0.25 [m] "length of the heat exchanger"
WallThk=0.0015 [m] "pipe wall thickness"

The Compact Heat Exchanger Library provides a procedure that returns the geometry
specifications for the surface area designated as ‘fc_tubes_sCF-70-58J’. The outer diameter of
the tubes (Do), fin pitch (finpitch), hydraulic diameter of the flow passages (Dh), fin thickness
(finthk), ratio of free flow to frontal area (σ), ratio of surface area to volume (α), and the ratio of
finned to unfinned surface area (Afin/A) are returned by the function.

"Use the compact heat exchanger library to determine the geometry of this heat exchanger"
Call CHX_geom_finned_tube('fc_tubes_sCF-70-58J': D_o, fin_pitch, D_h, fin_thk, sigma, alpha, A_fin\A)
"Note: sigma is the ratio of the minimum flow area to the frontal area and alpha is the surface area per
unit volume"

Air properties (μ, Pr, and c) are evaluated at the average air temperature.

"Air properties at the average temprature"


T_air_avg=(T_air_in+T_air_out)/2 "average air temperature for evaluation of properties"
mu=viscosity(Air,T=T_air_avg) "viscosity"
Pr=Prandtl(Air,T=T_air_avg) "Prandtl number"
c=cp(Air,T=T_air_avg) "specific heat"

The Compact_HX_ND procedure returns the Colburn j factor (jH) and the friction factor (f) given
the Reynolds number. The mass flow rate of the air can be determined by multiplying the
density of the inlet air by the volumetric flow rate.

m = ρ air ,in V (1)


where ρair,in is the density of the air at the inlet condition. The minimum flow area is computed
according to:

Amin = Af σ (2)

The mass flux is:

m
G= (3)
Amin

and the Reynolds number is:

G Dh
Re = (4)
μ

"Calculate the air-side heat transfer coefficient"


m_dot=density(Air,T=T_air_in,P=P)*V_dot "mass flow rate of air"
A_min=A_f*sigma "mininum free flow area"
G=m_dot/A_min "mass velocity"
Re=G*D_h/mu "Reynold's number"
Call Compact_HX_ND('fc_tubes_sCF-70-58J', Re: f,j_H) "determines j_H"

The Stanton number is computed according to:

2
jH = St Pr 3
(5)

and the heat transfer coefficient on the air side is computed according to:

ho
St = (6)
Gc

j_H=St*Pr^(2/3) "determines the Stanton number"


St=h_bar_o/(G*c) "h_bar_o is the heat transfer coefficient on the air side"

which leads to ho = 71.1 W/m2-K. The volume of the heat exchanger is:

V = Af W (7)

so the resistance on the air side (neglecting the fin efficiency) is:

1
Rair = (8)
V α ho

Vol=A_f*W "heat exchanger volume"


Res_air=1/(h_bar_o*Vol*alpha) "air-side heat transfer coefficient"

The heat transfer coefficient can be checked by calling the dimensional version of the compact
heat exchanger procedure, CHX_h_finned_tube.

Call CHX_h_finned_tube('fc_tubes_sCF-70-58J', m_dot, A_f, 'air',T_air_avg, P:h_bar_o_check)


"check result with dimensional call"

which also leads to ho = 71.1 W/m2-K.

The length of tube in the heat exchanger can be estimated based on the unfinned area of the core.
The unfinned area of the core is obtained according to:

⎛ Afin ⎞
Aunfin = V ⎜1 − ⎟ (9)
⎝ A ⎠

The unfinned area is related to the length of tube according to:

⎛ 1 ⎞
Aunfin = Ltube fin pitch ⎜ − finthk ⎟ π Do (10)
⎜ fin pitch ⎟
⎝ ⎠

which leads to Ltube = 119.3 m. The thermal resistance of convection to the flowing refrigerant
is:

1
RR134 a = (11)
hi π ( Do − th ) Ltube

where th is the wall thickness of the tube. The total conductance is:

1
UA = (12)
Rair + RR134 a

"Determine the refrigerant-side and overall heat transfer coefficients"


A_unfin=(1-A_fin\A)*Vol*alpha
"Vol*alpha is the total surface area; (1-A_fin\A) is the surface area of the tubes"
A_unfin=L_tube*fin_pitch*(1/fin_pitch-fin_thk)*pi*D_o "length of tube"
Res_R134a=1/(h_bar_i*pi*(D_o-2*WallThk)*L_tube) "R134a-side resistance"
UA=1/(Res_air+Res_R134a) "conductance"

which leads to UA = 2008 W/K.

b.) Compare the result to the value determined in Problem 8.1-1 (8-1).
The result for Problem 8.1-1, which assumed L = 110 m, is UA = 2122 W/K. If the length of the
tube in Problem 8.1-1 is increased to 119.3 m to match this problem then the conductance
increases to UA = 2302 W/K and the discrepancy increases to 14%.
Problem 8.1-3 (8-3 in text): Cross-Flow Heat Exchanger with Chilled Water
A decision has been made to use chilled water, rather than R134a in the heat exchanger
described in Problems 8.1-1 and 8.1-2 (8-1 and 8-2 in the text). The mass flow rate of chilled
water has been chosen so that the temperature rise of the water is ΔTw = 4°C as it passes through
the heat exchanger. The water-side is configured so that the chilled water flows through Nc = 10
parallel circuits.
a.) Estimate the overall heat transfer conductance and compare the result to your answers from
Problems 8.1-1 and 8.1-2 (8-1 and 8-2).

The known information into EES.

$UnitSystem SI MASS RAD PA K J


$Tabstops 0.2 0.4 0.6 3.5 in

"known information"
T_air_in=converttemp(C,K,30 [C]) "inlet air temperature"
P=101.3 [kPa]*convert(kPa,Pa) "atmospheric pressure"
V_dot=1.0 [m^3/s] "volumetric flowrate of air"
P_evap=345 [kPa]*convert(kPa,Pa) "pressure of evaporating R134a"
T_air_out=converttemp(C,K,10 [C]) "exit air temperature"
h_bar_i=2500 [W/m^2-K] "two-phase heat transfer coefficient is high"
A_f=0.5 [m^2] "frontal area of the heat exchanger"
W=0.25 [m] "length of the heat exchanger"
WallThk=0.0015 [m] "pipe wall thickness"
DELTAT_w=4 [K] "water temperature rise"

The air-side heat thermal resistance (Rair) is computed using the same compact heat exchanger
library, as discussed in Problem 8.1-2.

"Use the compact heat exchanger library to determine the geometry of this heat exchanger"
Call CHX_geom_finned_tube('fc_tubes_sCF-70-58J': D_o, fin_pitch, D_h, fin_thk, sigma, alpha, A_fin\A)
"Note: sigma is the ratio of the minimum flow area to the frontal area and alpha is the surface area per
unit volume"

"Air properties at the average temprature"


T_air_avg=(T_air_in+T_air_out)/2
"average air temperature for evaluation of properties"
mu=viscosity(Air,T=T_air_avg) "viscosity"
Pr=Prandtl(Air,T=T_air_avg) "Prandtl number"
c=cp(Air,T=T_air_avg) "specific heat"

"Calculate the air-side heat transfer coefficient"


m_dot=density(Air,T=T_air_in,P=P)*V_dot "mass flow rate of air"
A_min=A_f*sigma "mininum free flow area"
G=m_dot/A_min "mass velocity"
Re=G*D_h/mu "Reynold's number"
Call Compact_HX_ND('fc_tubes_sCF-70-58J', Re: f,j_H) "determines j_H"
j_H=St*Pr^(2/3) "determines the Stanton number"
St=h_bar_o/(G*c)
"h_bar_o is the heat transfer coefficient on the air side"
Vol=A_f*W "heat exchanger volume"
Res_air=1/(h_bar_o*Vol*alpha) "air-side heat transfer coefficient"
The length of tube in the heat exchanger can be estimated based on the unfinned area of the core.
The unfinned area of the core is obtained according to:

⎛ Afin ⎞
Aunfin = V ⎜1 − ⎟ (1)
⎝ A ⎠

The unfinned area is related to the length of tube according to:

⎛ 1 ⎞
Aunfin = Ltube fin pitch ⎜ − finthk ⎟ π Do (2)
⎜ fin pitch ⎟
⎝ ⎠

which leads to Ltube = 119.3 m.

"Determine the water-side and overall heat transfer coefficients"


A_unfin=(1-A_fin\A)*Vol*alpha
"Vol*alpha is the total surface area; (1-A_fin\A) is the surface area of the tubes"
A_unfin=L_tube*fin_pitch*(1/fin_pitch-fin_thk)*pi*D_o "length of tube"

The mass flow rate of water is calculated according to an energy balance on the heat exchanger:

m w cw ΔTw = m a ca (Ta ,in − Ta ,out ) (3)

where m a is the mass flow rate of air and cw and ca are the specific heat capacities of the water
and the air.

c_w=cP(Water,T=T_air_avg,P=1 [atm]*convert(atm,Pa)) "specific heat capacity of water"


m_dot_w*c_w*DELTAT_w=m_dot*c*(T_air_in-T_air_out) "energy balance on heat exchanger"

The inner diameter of the tube is:

Di = Do − 2 th (4)

where th is the wall thickness of the tube. The procedure PipeFlow is used to compute the heat
transfer coefficient for the flowing water ( hw , note that the mass flow rate per tube is m w / N c ).
The thermal resistance on the water side is:

1
Rw = (5)
hw π Di Ltube

The total conductance, neglecting fouling, is:


1
UA = (6)
( Rair + Rw )
D_i=D_o-2*WallThk "inner diameter of tube"
call PipeFlow('Water',T_air_avg,1 [atm]*convert(atm,Pa),m_dot_w,D_i,L_tube/N_c,0 [-]:h_bar_w, &
h_H_w ,DELTAP_w, Nusselt_T_w, f_w, Re_w)
Res_w=1/(h_bar_w*L_tube*pi*D_i) "water side resistance"
UA=1/(Res_w+Res_air) "conductance"

which leads to UA = 2358 W/K. This value is slightly higher than the conductance computed in
Problem 8.1-2 but very similar because the dominant resistance is on the air-side rather than the
water- or refrigerant-side.

b.) Estimate how much the overall heat transfer coefficient can be expected to drop over time
due to fouling of the closed chilled water loop.

The fouling factor (FF) is estimated assuming city or well water with the EES function
FoulingFactor. The resistance due to fouling is therefore:

FF
Rf = (7)
π Di Ltube

The conductance with fouling is:

1
UAf = (8)
( Rair + R f + Rw )
"with fouling"
FF=FoulingFactor('City or well water') "fouling factor"
Res_f=FF/(L_tube*pi*D_i) "resistance due to fouling"
UA_f=1/(Res_w+Res_f+Res_air) "conductance with fouling"

which leads to UAf = 2178 W/K, a decrease of approximately 8% due to fouling.


Problem 8.2-1 (8-4 in text)
In Problem 8.1-1 (8-1 in text), the inlet volumetric flowrate and the inlet and outlet temperatures
of the air were known and therefore it was possible to determine the heat transfer rate without a
heat exchanger analysis. However, you have just learned that the outlet air temperature was
measured with a thermocouple in only one location in the duct and it is not necessarily an
accurate measurement of the mixed average outlet air temperature. Use the log-mean
temperature difference method to estimate the average air outlet temperature.

The input information is entered in EES:

$UnitSystem SI MASS RAD PA K J


$Tabstops 0.2 0.4 0.6 3.5 in

"known information"
T_air_in=converttemp(C,K,30 [C]) "inlet air temperature"
P=101.3 [kPa]*convert(kPa,Pa) "atmospheric pressure"
V_dot=1.0 [m^3/s] "volumetric flowrate of air"
P_evap=345 [kPa]*convert(kPa,Pa) "pressure of evaporating R134a"

The mass flow rate of the air can be determined by multiplying the density of the inlet air by the
volumetric flow rate.

m = ρ air ,in V (1)

where ρair,in is the density of the air.

"Determine the heat transfer rate"


rho_air=density(Air,T=T_air_in,P=P) "density of air"
m_dot=rho_air*V_dot "mass flow rate"

The conductance was estimated in Problems 8.1-1 and 8.1-2. Here we will use the value
determined in Problem 8.1-1:

UA=2122 [W/K] "conductance calculated in Problem 8.1-1"

The log-mean temperature difference method is not convenient computationally. It is best to


begin with a guess for the outlet temperature, Tair,out, in order to compute the quantities that are
required to determine the heat transfer and then iterate. The measured value of Tair,out is used as
the guess value:

T_air_out=converttemp(C,K,10 [C]) "exit air temperature - measured"

The temperature of the R134a (TR134a) is obtained from the evaporating pressure:

T_R134a=temperature(R134a,P=P_evap,x=1) "temperature of evaporating R134a"

The log-mean temperature difference for a counter-flow configuration is computed according to:
ΔTlm,cf =
(Tair ,out − TR134 a ) − (Tair ,in − TR134 a )
(2)
⎡ (Tair ,out − TR134 a ) ⎤
ln ⎢ ⎥
⎢⎣ (Tair ,in − TR134 a ) ⎥⎦

The correction factor based on the fact that this is a crossflow heat exchanger with one fluid (the
refrigerant) mixed is a function of P and R, calculated according to:

P=
(T air ,in − Tair ,out )
(3)
(T air ,in − TR134 a )

C T −T
R =  air = R134 a R134 a → 0 (4)
CR134 a Tair ,in − Tair ,out

The correction factor, F, is obtained using the function LMTD_CF and the log-mean temperature
difference is computed according to:

ΔTlm = F ΔTlm,cf (5)

The heat transfer rate is specified by:

q = UA ΔTlm (6)

DELTAT_lmtd_cf=((T_air_out-T_R134a)-(T_air_in-T_R134a))/ln((T_air_out-T_R134a)/&
(T_air_in-T_R134a)) "log-mean temperature difference for a counterflow heat exchanger"
Pp=(T_air_in-T_air_out)/(T_air_in-T_R134a) "P-parameter"
Rp=0.001 [-] "R-parameter"
F=LMTD_CF('crossflow_one_unmixed',Pp,Rp) "correction factor for the LMTD method"
DELTAT_lmtd=DELTAT_lmtd_cf*F "log-mean temperature difference"
q_dot=UA*DELTAT_lmtd "heat transfer rate"

The problem is solved and the guess values are updated. The guess for Tair,out is commented out
and then Tair,out is calculated according to:

q
Tair ,out = Tair ,in − (7)
m cair

where cair is the specific heat capacity of air:

{T_air_out=converttemp(C,K,10 [C])} "exit air temperature - measured"


c_air=cP(Air,T=T_air_in) "specific heat capacity of air"
T_air_out=T_air_in-q_dot/(m_dot*c_air) "recalculate outlet air temperature"
T_air_out_C=converttemp(K,C,T_air_out) "in C"

which leads to Tair,out = 8.74ºC.


Problem 8.2-2 (8-5 in text)
Table P8.2-2 provides heat transfer data from a manufacturer’s catalog for a counterflow oil
cooler. The table provides the heat transfer rate for three different oil flow rates (expressed in
gpm, gallons per minute). The values in the table are the heat transfer rate between the oil and
water in units of Btu/min-ETD where ETD is the entering temperature difference in °F. The
density and specific heat of the oil are ρo = 830 kg/m3 and co = 2.3 kJ/kg-K, respectively. The
water enters at Vw = 35 gallons per minute at Tw,in = 180°F and atmospheric pressure. The oil
enters at To,in = 240°F.

Table P8.2-2: Heat transfer data (heat transfer rate/ETD) for different models and oil flow rates
Oil flow rate
1 gpm 3 gpm 5 gpm
Model 1 2.5 Btu/min-deg. F 4.9 Btu/min-deg. F
Model 2 2.9 Btu/min-deg. F 6.1 Btu/min-deg. F 8.1 Btu/min-deg. F
Model 3 3.1 Btu/min-deg. F 6.8 Btu/min-deg. F 9.7 Btu/min-deg. F

a.) Determine the outlet oil temperature, the log mean temperature difference, and the overall
conductance for Model 2 at oil flow rates 1, 3 and 5 gallons/min.

The input data are entered in EES:

$UnitSystem SI MASS RAD PA K J


$Tabstops 0.2 0.4 0.6 3.5 in

"Inputs"
rho_o=830 [kg/m^3] "density of oil"
c_o=2.3 [kJ/kg-K]*convert(kJ/kg-K,J/kg-K) "specific heat capacity of oil"
V_dot_w=35 [gal/min]*convert(gal/min,m^3/s) "volumetric flow rate of water"
P_w=1 [atm]*convert(atm,Pa) "water pressure"
T_w_in=converttemp(F,K,180 [F]) "inlet water temperature"
T_o_in=converttemp(F,K,240 [F]) "inlet oil temperature"

The first entry in the table for the oil flow rate and performance are entered:

V_dot_o_gpm=1 [gal/min] "oil flow rate, in gpm"


q_dot\ETD_BtuminF=2.5 [Btu/min-F] "performance, in Btu/min-F"
V_dot_o=V_dot_o_gpm*convert(gal/min,m^3/s) "oil flow rate"
q_dot\ETD=q_dot\ETD_BtuminF*convert(Btu/min-F,W/K) "performance"

The mass flow rate of water and oil are calculated according to:

m w = ρ w Vw (1)

m o = ρ o Vo (2)

rho_w=density(Water,T=T_w_in,P=P_w) "density of water"


m_dot_w=rho_w*V_dot_w "mass flow rate of water"
m_dot_o=rho_o*V_dot_o "mass flow rate of oil"
The heat transfer rate is computed from the given value of heat transfer to entering temperature
difference ratio:

q
q =
ETD
(To,in − Tw,in ) (3)

The outlet temperatures of the water and oil are computed from an energy balance:

q
Tw,out = Tw,in + (4)
m w cw

q
To ,out = To ,in − (5)
m o co

q_dot=q_dot\ETD*(T_o_in-T_w_in) "rate of heat transfer"


c_w=cP(Water,T=T_w_in,P=P_w) "specific heat capacity of water"
T_w_out=T_w_in+q_dot/(m_dot_w*c_w) "water outlet temperature"
T_o_out=T_o_in-q_dot/(m_dot_o*c_o) "oil outlet temperature"

The log-mean temperature difference is computed according to:

ΔTlm =
(To , out − Tw,in ) − (To ,in − Tw,out )
(6)
⎡ (To ,out − Tw,in ) ⎤
ln ⎢ ⎥
⎢⎣ (To ,in − Tw,out ) ⎥⎦

and the conductance is given by:

q
UA = (7)
ΔTlm

DELTAT_lmtd=((T_o_out-T_w_in)-(T_o_in-T_w_out))/ln((T_o_out-T_w_in)/(T_o_in-T_w_out))
"log-mean temperature difference for a counterflow heat exchanger"
UA=q_dot/DELTAT_lmtd "conductance"

The calculations are repeated for each of the data points for model 2 by commenting out the
values of oil flow rate and q / ETD in the equations window and setting up a parametric table
with these values set, as shown in Figure 1.
Figure 1: Parametric table.

b) Plot the conductance as a function of the oil flow rate and provide an explanation for the
observed variation.

The conductance as a function of oil flow rate is shown in Figure 2. The conductance increases
with flow rate because the increased flow likely enhances the oil-side heat transfer coefficient.
350

325

300
Conductance (W/K)

275

250

225

200

175

150
1 1.5 2 2.5 3 3.5 4 4.5 5
OIl flow rate (gpm)
Figure 2: Conductance as a function of oil flow rate.

c) Estimate the oil outlet temperature if the oil enters the heat exchanger at To,in = 225°F with a
flow rate of Vo = 4 gpm and the water enters with temperature Tw,in = 180°F at a flow rate of
V = 35 gpm.
w

Part (a) is commented out and the inputs for part (c) are entered:

"Inputs for (c)"


rho_o=830 [kg/m^3] "density of oil"
c_o=2.3 [kJ/kg-K]*convert(kJ/kg-K,J/kg-K) "specific heat capacity of oil"
V_dot_w=35 [gal/min]*convert(gal/min,m^3/s) "volumetric flow rate of water"
P_w=1 [atm]*convert(atm,Pa) "water pressure"
T_w_in=converttemp(F,K,180 [F]) "inlet water temperature"
T_o_in=converttemp(F,K,225 [F]) "inlet oil temperature"
V_dot_o_gpm=4 [gal/min] "oil flow rate, in gpm"
V_dot_o=V_dot_o_gpm*convert(gal/min,m^3/s) "oil flow rate"
According to Figure 2, the conductance of the heat exchanger with an oil flow rate of 4 gal/min
will be approximately 315 W/K.

UA=315 [W/K] "conductance for (c)"

The oil and water mass flow rates are calculated according to Eqs. (1) and (2).

rho_w=density(Water,T=T_w_in,P=P_w) "density of water"


m_dot_w=rho_w*V_dot_w "mass flow rate of water"
m_dot_o=rho_o*V_dot_o "mass flow rate of oil"

In order to use the log-mean temperature difference for a simulation type problem like this one it
is best to assume a heat transfer rate and calculate the water and oil outlet temperatures according
to Eqs. (4) and (5).

q_dot=3000 [W] "this is a guess for the heat transfer rate"


c_w=cP(Water,T=T_w_in,P=P_w) "specific heat capacity of water"
T_w_out=T_w_in+q_dot/(m_dot_w*c_w) "water outlet temperature"
T_o_out=T_o_in-q_dot/(m_dot_o*c_o) "oil outlet temperature"
T_o_out_F=converttemp(K,F,T_o_out) "in F"

The log-mean temperature difference is computed according to Eq. (6).

DELTAT_lmtd=((T_o_out-T_w_in)-(T_o_in-T_w_out))/ln((T_o_out-T_w_in)/(T_o_in-T_w_out))
"log-mean temperature difference for a counterflow heat exchanger"

The problem is solved and the guess values are updated. The assumed value for q is recalculated
according to:

q = UA ΔTlm (8)

{q_dot=3000 [W]} "this is a guess for the heat transfer rate"


q_dot=UA*DELTAT_lmtd "recalculate the heat transfer rate"

which leads to To,out = 203.6°F.


PROBLEM 8.3-1 (8-6 in text): Energy Recovery
The plant where you work includes a process that results in a stream of hot combustion products
at moderate temperature Thg,in = 150°C with mass flow rate m hg = 0.25 kg/s. The properties of
the combustion products can be assumed to be the same as those for air. You would like to
recover the energy associated with this flow in order to reduce the cost of heating the plant and
therefore you are evaluating the use of the air-to-air heat exchanger shown in Figure P8.3-1.
close-up of channel structure
top view
hc = 1 mm km = 15 W/m-K
thf = 0.1 mm

pf = 0.1 mm

hot gas
Th ,in = 150°C
cold air
m hg = 0.25 kg/s
thw = 0.2 mm Tcg ,in = −5°C
m cg = 0.5 kg/s
flow passages for cold gas
(flowing into the page) cold air, Tcg,in

flow passages for hot gas hot gas, Th,in Th,out


(flowing into the page) L = 10 cm

L = 10 cm

W = 20 cm Tcg,out

front view side view


Figure P8.3-1: Air-to-air heat exchanger.

The air-to-air heat exchanger is a cross-flow configuration. The length of the heat exchanger
parallel to the two flow directions is L = 10 cm. The width of the heat exchanger in the direction
perpendicular to the two flow directions is W = 20 cm. Cold air enters the heat exchanger from
outdoors at Tcg,in = -5°C with mass flow rate m cg = 0.50 kg/s and is heated by the combustion
products to Tcg,out. The hot and cold air flows through channels that are rectangular (both sides of
the heat exchanger have the same geometry). The width of the channels is hc = 1.0 mm. There
are fins placed in the channel in order to provide structural support and also increase the surface
area for heat transfer. The fins can be assumed to run the complete length of the heat exchanger
and are 100% efficient. The fins are spaced with pitch, pf = 0.5 mm and the fins are thf = 0.10
mm thick. The thickness of the metal separating the cold channels from the hot channels is thw =
0.20 mm and the conductivity of this metal is km = 15 W/m-K. Both the hot and cold flows are at
nominally atmospheric pressure. The fouling factor associated with the flow of combustion gas
through the heat exchanger is R′′f = 0.0018 K-m2/W. There is no fouling associated with the
flow of outdoor air through the heat exchanger.
a.) Compute the heat transfer coefficient between the hot air and the channel walls and the cold
gas and the channel walls. Use the inlet temperatures of the air flows to compute the
properties.

The inputs are entered in EES:

$UnitSystem SI MASS RAD PA K J


$TABSTOPS 0.2 0.4 0.6 0.8 3.5 in

"Inputs"
L=10 [cm]*convert(cm,m) "size of module - parallel to flow directions"
W=20 [cm]*convert(cm,m) "size of module - perpendicular to flow directions"
th_w=0.2 [mm]*convert(mm,m) "channel wall thickness"
th_f=0.1 [mm]*convert(mm,m) "fin thickness"
h_c=1.0 [mm]*convert(mm,m) "channel height"
p_f=0.5 [mm]*convert(mm,m) "fin pitch"
k_m=15 [W/m-K] "metal conductivity"
FF=0.0018 [K-m^2/W] "combustion gas fouling factor"
m_dot_hg=0.25 [kg/s] "mass flow rate of hot gas"
T_hg_in=converttemp(C,K,150 [C]) "hot gas inlet temperature"
m_dot_cg=0.5 [kg/s] "mass flow rate of cold gas"
T_cg_in=converttemp(C,K,-5 [C]) "cold gas inlet temperature"

The properties of the hot and cold flows (chg, khg, μhg, ρhg, ccg, kcg, μcg, and ρcg) are computed
using EES’ internal property routines. The properties should be computed at the average flow
temperature. However, the outlet temperatures are not yet known, so initially, the average
temperature is set to the inlet temperature. This approximation can be corrected after the outlet
temperatures are determined and the guess values are updated.

T_hg_avg=T_hg_in "estimate of the average hot gas temperature"


c_hg=cP(Air,T=T_hg_avg) "specific heat capacity"
k_hg=conductivity(Air,T=T_hg_avg) "conductivity"
mu_hg=viscosity(Air,T=T_hg_avg) "viscosity"
rho_hg=density(Air,T=T_hg_avg,p=1 [atm]*convert(atm,Pa)) "density"

T_cg_avg=T_cg_in "estimate of the average cold gas temperature"


c_cg=cP(Air,T=T_cg_avg) "specific heat capacity"
k_cg=conductivity(Air,T=T_cg_avg) "conductivity"
mu_cg=viscosity(Air,T=T_cg_avg) "viscosity"
rho_cg=density(Air,T=T_cg_avg,p=1 [atm]*convert(atm,Pa)) "density"

The number of hot or cold channels in the heat exchanger is:

W
Nc = (1)
2 ( hc + thw )

The number of fins in each channel is:

L
Nf = (2)
pf
N_c=floor(W/(2*(h_c+th_w))) "number of channels"
N_f=floor(L/p_f) "number of fins per channel"

Note that the floor command rounds the result down to the nearest integer. The total cross-
sectional area for flow (on either side) is:

Ac = N c N f ( p f − th f ) hc (3)

and the total wetted perimeter (on either side) is:

per = N c N f 2 ( hc + p f − th f ) (4)

so the hydraulic diameter of the flow channels is:

Ac
Dh = 4 (5)
per

A_c=N_c*N_f*h_c*(p_f-th_f) "cross-sectional area for flow"


per=N_c*N_f*2*(h_c+p_f-th_f) "wetted perimeter"
D_h=4*A_c/per "hydraulic diameter"

The mean velocity of the hot gas is:

m hg
um, hg = (6)
Ac ρ hg

The Reynolds number associated with this flow is:

um, hg Dh ρ hg
Rehg = (7)
μhg

and the Prandtl number of the hot gas is:

μhg chg
Prhg = (8)
khg

u_m_hg=m_dot_hg/(A_c*rho_hg) "mean velocity"


Re_hg=u_m_hg*D_h*rho_hg/mu_hg "Reynolds number"
Pr_hg=mu_hg*c_hg/k_hg "Prandtl number"

The correlations for flow through a rectangular duct are accessed using the procedure DuctFlow_N
and used to compute the average friction factor ( f hg ) and Nusselt numbers ( Nu T ,hg and Nu H , hg ).
The heat transfer coefficient on the hot gas side is computed:
Nu T ,hg khg
hhg = (9)
Dh

Note that the constant temperature Nusselt number is used in this case because it is a lower
bound and you would like the predictions to be conservative.

call DuctFlow_N(Re_hg,Pr_hg,L/D_h, (p_f-th_f)/h_c,0 [-]: Nusselt_T_hg, Nusselt_H_hg, f_hg)


htc_hg=Nusselt_T_hg*k_hg/D_h "heat transfer coefficient"

The calculations are repeated for the cold gas:

u_m_cg=m_dot_cg/(A_c*rho_cg) "mean velocity"


Re_cg=u_m_cg*D_h*rho_cg/mu_cg "Reynolds number"
Pr_cg=mu_cg*c_cg/k_cg "Prandtl number"
call DuctFlow_N(Re_cg,Pr_cg,L/D_h, (p_f-th_f)/h_c,0 [-]: Nusselt_T_cg, Nusselt_H_cg, f_cg)
htc_cg=Nusselt_T_cg*k_cg/D_h "heat transfer coefficient"

These calculations lead to hhg = 246 W/m2-K and hcg = 235.7 W/m2-K. When the average
(instead of the inlet) temperature is used to evaluate properties, the values are 221.2 and 207.3
W/m2-K, respectively.

b.) Compute the total conductance of the heat exchanger.

The resistance associated with convection to the hot gas is:

1
Rconv ,hg = (10)
per L hhg

The resistance associated with fouling on the hot side is:

FF
R foul = (11)
per L

The resistance associated with conduction through the metal walls is:

thm
Rcond = (12)
km 2 L2 N c

The resistance associated with convection to the cold gas is:

1
Rconv ,cg = (13)
per L hcg

The total resistance between the hot and cold gas is:
Rtotal = Rconv , hg + R foul + Rcond + Rconv ,cg (14)

The conductance is:

1
UA = (15)
Rtotal

R_conv_hg=1/(per*L*htc_hg) "resistance to convection on hot gas side"


R_conv_cg=1/(per*L*htc_cg) "resistance to convection on cold gas side"
R_cond=th_w/(k_m*2*L*L*N_c) "conduction resistance"
R_foul=FF/(per*L) "fouling resistance"
R_total=R_conv_hg+R_foul+R_cond+R_conv_cg "total resistance"
UA=1/R_total "total conductance"

which leads to UA = 458.2 W/K (with properties evaluated at the inlet fluid temperatures) and
415.7 W/K (with properties evaluated at the average fluid temperatures.)

c.) Determine the heat transferred in the heat exchanger and the temperature of the cold gas
leaving the heat exchanger.

The capacity rates of the hot and cold gas flows are computed:

C hg = m hg chg (16)

C cg = m cg ccg (17)

and the minimum and maximum values of the capacity rates ( C min and C max ) are determined.

C_dot_hg=m_dot_hg*c_hg "capacity rate of hot gas"


C_dot_cg=m_dot_cg*c_cg "capacity rate of cold gas"
C_dot_min=MIN(C_dot_hg,C_dot_cg) "minimum capacity rate"
C_dot_max=MAX(C_dot_hg,C_dot_cg) "maximum capacity rate"

The capacity ratio and number of transfer units are computed:

C
CR =  min (18)
Cmax

UA
NTU =  (19)
Cmin

and the function HX is used to determine the effectiveness of the heat exchanger (ε). Note that
the geometry of the passages makes this a clearly a heat exchanger where both sides are
unmixed:
C_R=C_dot_min/C_dot_max "capacity ratio"
NTU=UA/C_dot_min "number of transfer units"
epsilon=HX('crossflow_both_unmixed', NTU, C_dot_cg, C_dot_hg, 'epsilon') "access eff-NTU solution"

The maximum possible rate of heat transfer is computed:

qmax = C min (Thg ,in − Tcg ,in ) (20)

and the actual heat transfer rate is:

q = ε qmax (21)

q_dot_max=C_dot_min*(T_hg_in-T_cg_in) "maximum heat transfer rate"


q_dot=epsilon*q_dot_max "actual heat transfer rate"

which leads to q = 26.8 kW. The temperature of the cold gas leaving the heat exchanger is:

q
Tcg ,out = Tcg ,in +  (22)
Ccg

The temperature of the hot gas leaving the heat exchanger is:

q
Thg ,out = Thg ,in + (23)
C hg

T_hg_out=T_hg_in-q_dot/C_dot_hg "hot gas exit temperature"


T_cg_out=T_cg_in+q_dot/C_dot_cg "cold gas exit temperature"

The initial estimate of the average temperatures are commented out and the actual average
temperatures calculated:

{T_hg_avg=T_hg_in } "estimate of the average hot gas temperature"


T_hg_avg=(T_hg_in +T_hg_out)/2 "average hot gas temperature"
{T_cg_avg=T_cg_in} "estimate of the average cold gas temperature"
T_cg_avg=(T_cg_in+T_cg_out)/2 "average cold gas temperature"

which leads to Thg,out = 317.0 K and Tcg,out = 321.5 K.

d.) Blowers are required to force the hot and cold flows through the heat exchanger. Assume
that you have blowers that are ηblower = 0.65 efficient. Estimate the total blower power
required to operate the energy recovery unit.

The pressure drops on the hot and cold sides of the heat exchanger are:
ρ hg um2 ,hg ⎛ L ⎞
Δphg = ⎜ f hg ⎟ (24)
2 ⎝ Dh ⎠

ρ cg um2 ,cg ⎛ L ⎞
Δpcg = ⎜ f cg ⎟ (25)
2 ⎝ Dh ⎠

The total blower power required is:

Δphg m hg Δpcg m cg
Wblower = + (26)
ρ hg ηblower ρcg ηblower

DeltaP_cg=rho_cg*u_m_cg^2*(f_cg*L/D_h)/2 "cold gas pressure drop"


DeltaP_hg=rho_hg*u_m_hg^2*(f_hg*L/D_h)/2 "cold gas pressure drop"
eta_blower=0.65 [-] "blower efficiency"
W_dot_blower=DeltaP_cg*m_dot_cg/(eta_blower*rho_cg)+DeltaP_hg*m_dot_hg/(eta_blower*rho_hg)
"total blower power"

which leads to Wblower = 13.3 kW.

e.) If you pay ec = 0.08$/kW-hr for electricity (to run the blowers) and 1.50$/therm for gas (to
heat the plant) then estimate the net savings associated with the energy recovery system
(neglect capital investment cost) for time = 1 year; this is the savings associated with the heat
transferred in the heat exchanger less the cost to run the blower for a year. Assume that the
plant runs continuously.

The cost to run the blowers is:

blowercost = Wblower time ec (27)

The savings associated with the heat transferred in the heat exchanger is:

heatsavings = q time hc (28)

The net savings per year is therefore:

netsavings = heatsavings − blowercost (29)

time=365 [day]*convert(day,s) "operating time"


ec=0.08 [$/kW-hr]*convert($/kW-hr,$/J) "cost of electricity"
hc=1.50 [$/therm]*convert($/therm,$/J) "cost of heat"
blowercost=ec*W_dot_blower*time "cost to run blower"
heatsavings=hc*q_dot*time "heating savings"
savingsnet=heatsavings-blowercost "net savings"
which leads to a net savings of 2,811 $/year (inlet temperatures used to evaluate properties) or
2,685$/year (average temperatures used to evaluate properties).

f.) Plot the net savings per year as a function of the mass flow rate of the cold air that is being
heated. Your plot should show a maximum; explain why this maximum exists.

Figure 2 illustrates the net savings as a function of the mass flow rate of the cold air being
heated. Also shown is the savings in heating and the cost to run the blower.

Figure 2: Net savings, blower power, and heating savings per year as a function of the mass flow rate of cold
gas.

Notice that the blower power increases dramatically with mass flow rate, particularly when the
flow becomes turbulent at approximately 0.46 kg/s. The heating savings increases with mass
flow rate as well but at a decreasing rate; this is because the heat exchanger effectiveness
decreases when the mass flow rate of air becomes very large. Note: when using the average
temperatures to evaluate properties, EES may not initially converge at low cold gas flows. A
useful trick to obtain convergence is to start the Parametric Table calculations at a cold gas flow
rate of 0.1 kg/s and do the calculations for flows up to 0.8 kg/s. Then do the calculations in the
Parametric table from the smallest flow rate to 0.1 kg/s in reverse. Be sure that the Update Guess
Values check box is checked. Alternatively evaluate properties at the inlet temperatures rather
than average temperatures.

g.) Determine the optimal values of the mass flow rate of combustion gas and cold gas ( m hg and
m cg ) that maximize the net savings per year. You should use the MIN/MAX capability in
EES to accomplish this - what is the maximum savings/year? This is the most you could
afford to pay for the blowers and heat exchanger if you wanted a 1 year pay-back.
The values of m hg and m cg are commented out:

{m_dot_hg=0.25 [kg/s] "mass flow rate of hot gas"}


{m_dot_cg=0.5 [kg/s] "mass flow rate of cold gas"}

and Min/Max is selected from the Calculate menu. The net savings is maximized by adjusting
the independent variables m_dot_hg and m_dot_cg (Figure 3).

Figure 3: Find Minimum or Maximum window.

Set the bounds on the two independent variables (Figure 4) to reasonable values.

Figure 4: Set bounds on independent variables.


The optimization routine indicates that the most optimal values of mass flow rates are m hg =
0.176 kg/s and m cg = 0.251 kg/s which leads to a net savings of $5748/year (with properties
estimated at the inlet temperatures).
PROBLEM 8.3-2 (8-7 in text): Gas Turbine Engine
A gas turbine engine is used onboard a small ship to drive the propulsion system. The engine
consists of a compressor, turbine, and combustor as shown in Figure P8.3-2(a). Ambient air is
pulled through the gas turbine engine with a mass flow rate of m = 0.1 kg/s and enters the
compressor at T1 = 20°C and P1 = 1 atm. The exit pressure of the compressor is P2 = 3.5 atm and
T2 = 167°C. The air enters a combustor where it is heated to T3 = 810°C with very little loss of
pressure so that P3 = 3.5 atm. The hot air leaving the combustor enters a turbine where it is
expanded to P4 = 1 atm. The temperature of the air leaving the turbine is T4 = 522°C. You may
assume that the turbine and compressor are well-insulated and that the specific heat capacity of
air is constant and equal to c = 1000 J/kg-K. The difference between the power produced by the
turbine and required by the compressor is used to drive the ship.

compressor turbine

net power to ship


fuel
T4 = 522°C
P4 = 1atm
T2 = 167°C T3 = 810°C
P2 = 3.5 atm P3 = 3.5 atm
T1 = 20°C
P1 = 1 atm
m = 0.1 kg/s
Figure P8.3-2(a): Un-recuperated gas turbine engine.

a.) Determine the efficiency of the gas turbine engine (the ratio of the net power to the ship to
the heat transferred in the combustor).

The inputs are entered in EES:

$UnitSystem SI MASS RAD PA K J


$TABSTOPS 0.2 0.4 0.6 0.8 3.5 in

"Inputs"
m_dot=0.1 [kg/s] "mass flow rate of air"
T_1=converttemp(C,K,20) "inlet temperature of air"
P_1=1 [atm]*convert(atm,Pa) "inlet pressure of air"
P_2=3.5 [atm]*convert(atm,Pa) "compressor exit pressure"
T_2=converttemp(C,K,167) "compressor exit temperature"
T_3=converttemp(C,K,810) "combustor exit temperature"
P_3=3.5 [atm]*convert(atm,Pa) "combustor exit pressure"
T_4=converttemp(C,K,522) "turbine exit temperature"
P_4=1.0 [atm]*convert(atm,Pa) "turbine exit pressure"
c=1000 [J/kg-K] "specific heat capacity"

The power required by the compressor is given by:

w c = m c (T2 − T1 ) (1)
The power generated by the turbine is:

w t = m c (T3 − T4 ) (2)

The net power generated by the gas turbine engine is therefore:

w net = w t − w c (3)

The amount of heat transferred in the combustor is:

qcomb,ur = m c (T3 − T2 ) (4)

and the efficiency of the engine is:

w net
η gte ,ur = (5)
qcomb ,ur

w_dot_c=m_dot*c*(T_2-T_1) "compressor power required"


w_dot_t=m_dot*c*(T_3-T_4) "turbine power produced"
w_dot_net=w_dot_t-w_dot_c "net power produced"
q_dot_comb_ur=m_dot*c*(T_3-T_2) "combustor heat required"
eta_gte_ur=w_dot_net/q_dot_comb_ur "GTE efficiency"

which leads to ηgte,ur = 0.22.

b.) The combustor runs on fuel with a heating value of HV = 44x106 J/kg and a mission lasts t =
2 days. What is the mass of fuel that the ship must carry?

The fuel that the ship must carry is:

qcomb ,ur t
M fuel ,ur = (6)
HV

HV=44e6 [J/kg] "heating value of fuel"


time=2 [day]*convert(day,s) "mission duration"
M_fuel_ur=q_dot_comb_ur*time/HV "mass of fuel"
M_fuel_ur_lbm=M_fuel_ur*convert(kg,lbm) "in lbm"

which leads to Mfuel,ur = 253 kg (557 lbm).

In order to reduce the amount of fuel required, you have been asked to look at the option of
adding a recuperative heat exchanger to the gas turbine cycle, as shown in Figure P8.3-2(b).
You are considering the air-to-air heat exchanger that was evaluated in Problem 8.3-1 (8-6 in the
text) and is shown in Figure P8.3-1 (Figure P8-6 in the text). The air-to-air heat exchanger is a
cross-flow configuration. The length of the heat exchanger parallel to the two flow directions is
L = 10 cm. The width of the heat exchanger in the direction perpendicular to the two flow
directions is W = 20 cm but this can easily be adjusted by adding additional plates. Air enters the
heat exchanger from the compressor and is heated by the air leaving the turbine. The hot and
cold air flows through channels that are rectangular (both sides of the heat exchanger have the
same geometry). The width of the channels is hc = 1.0 mm. There are fins placed in the channel
in order to provide structural support and also increase the surface area for heat transfer. The
fins can be assumed to run the complete length of the heat exchanger and are 100% efficient.
The fins are spaced with pitch, pf = 0.5 mm and the fins are thf = 0.10 mm thick. The thickness
of the metal separating the cold channels from the hot channels is thw = 0.20 mm and the
conductivity of this metal is km = 15 W/m-K. The hot and cold flows are at nominally
atmospheric pressure and the compressor discharge pressure, respectively. The fouling factor
associated with the flow of the combustion products leaving the turbine is R′′f = 0.0018 K-m2/W.
There is no fouling associated with the flow of the air leaving the compressor.

compressor turbine

T1 = 20°C net power to ship


P1 = 1 atm fuel
m = 0.1 kg/s
(5) T3 = 810°C
T2 = 167°C P3 = 3.5 atm
P2 = 3.5 atm
T4 = 522°C
recuperative heat exchanger P4 = 1atm
Figure P8.3-2(b): Recuperated gas turbine engine.

c.) Compute the heat transfer coefficient between the hot air from the turbine and the channel
walls and the colder air from the compressor and the channel walls. You may use the inlet
temperatures of the air flows to compute the properties.

The inputs related to the heat exchanger geometry are entered in EES:

"HX Geometry"
L=10 [cm]*convert(cm,m) "size of HX - parallel to flow directions"
W=20 [cm]*convert(cm,m) "size of HX - perpendicular to flow directions"
th_w=0.2 [mm]*convert(mm,m) "channel wall thickness"
th_f=0.1 [mm]*convert(mm,m) "fin thickness"
h_c=1.0 [mm]*convert(mm,m) "channel height"
p_f=0.5 [mm]*convert(mm,m) "fin pitch"
k_m=15 [W/m-K] "stainless conductivity"
FF=0.0018 [K-m^2/W] "combustion gas fouling factor"

The properties of the hot and cold flows (ch, kh, μh, ρh, cc, kc, μc, and ρc) are computed using
EES’ internal property routines:

T_h_avg=T_4 "hot gas temperature"


k_h=conductivity(Air,T=T_h_avg) "conductivity"
mu_h=viscosity(Air,T=T_h_avg) "viscosity"
rho_h=density(Air,T=T_h_avg,P=P_4) "density"

T_c_avg=T_2 "cold gas temperature"


k_c=conductivity(Air,T=T_c_avg) "conductivity"
mu_c=viscosity(Air,T=T_c_avg) "viscosity"
rho_c=density(Air,T=T_c_avg,P=P_2) "density"

The number of hot or cold channels in the heat exchanger is:

W
Nc = (7)
2 ( hc + thw )

The number of fins in each channel is:

L
Nf = (8)
pf

N_c=floor(W/(2*(h_c+th_w))) "number of channels"


N_f=floor(L/p_f) "number of fins per channel"

Note that the floor command rounds the result down to the nearest integer. The total cross-
sectional area for flow (on either side) is:

Ac = N c N f ( p f − th f ) hc (9)

and the total wetted perimeter (on either side) is:

per = N c N f 2 ( hc + p f − th f ) (10)

so the hydraulic diameter of the flow channels is:

Ac
Dh = 4 (11)
per

A_c=N_c*N_f*h_c*(p_f-th_f) "cross-sectional area for flow"


per=N_c*N_f*2*(h_c+p_f-th_f) "wetted perimeter"
D_h=4*A_c/per "hydraulic diameter"

The mean velocity of the hot gas is:

m
um , h = (12)
Ac ρ h

The Reynolds number associated with this flow is:


um ,h Dh ρ h
Reh = (13)
μh

and the Prandtl number of the hot gas is:

μh c
Prhg = (14)
kh

u_m_h=m_dot/(A_c*rho_h) "mean velocity"


Re_h=u_m_h*D_h*rho_h/mu_h "Reynolds number"
Prandtl_h=mu_h*c/k_h "Prandtl number"

The correlations for flow through a rectangular duct are accessed using the procedure DuctFlow_N
and used to compute the average friction factor ( f h ) and Nusselt numbers ( Nu T , h and Nu H ,h ).
The heat transfer coefficient on the hot gas side is computed:

Nu T ,h kh
hh = (15)
Dh

Note that the constant temperature Nusselt number is used in this case because it is a lower
bound and I would like the predictions to be conservative.

call DuctFlow_N(Re_h,Prandtl_h,L/D_h, (p_f-th_f)/h_c,0 [-]: Nusselt_T_h, Nusselt_H_h, f_h)


htc_h=Nusselt_T_h*k_h/D_h "heat transfer coefficient"

The calculations are repeated for the cold side of the heat exchanger:

u_m_c=m_dot/(A_c*rho_c) "mean velocity"


Re_c=u_m_c*D_h*rho_c/mu_c "Reynolds number"
Prandtl_c=mu_c*c/k_c "Prandtl number"
call DuctFlow_N(Re_c,Prandtl_c,L/D_h, (p_f-th_f)/h_c,0 [-]: Nusselt_T_c, Nusselt_H_c, f_c)
htc_c=Nusselt_T_c*k_c/D_h "heat transfer coefficient"

These calculations lead to hh = 381 W/m2-K and hc = 242 W/m2-K.

d.) Compute the total conductance of the heat exchanger.

The resistance associated with convection to the hot air leaving the turbine is:

1
Rconv ,h = (16)
per L hh

The resistance associated with fouling on the hot side is:


FF
R foul = (17)
per L

The resistance associated with conduction through the metal walls is:

thm
Rcond = (18)
km 2 L2 N c

The resistance associated with convection to the air leaving the compressor is:

1
Rconv ,c = (19)
per L hc

The total resistance between the hot and cold air is:

Rtotal = Rconv ,h + R foul + Rcond + Rconv ,c (20)

The conductance is:

1
UA = (21)
Rtotal

R_conv_h=1/(per*L*htc_h)
"resistance to convection on hot gas side"
R_conv_c=1/(per*L*htc_c)
"resistance to convection on cold gas side"
R_cond=th_w/(k_m*2*L*L*N_c) "conduction resistance"
R_foul=FF/(per*L) "fouling resistance"
R_total=R_conv_h+R_foul+R_cond+R_conv_c "total resistance"
UA=1/R_total "total conductance"

which leads to UA = 541 W/K.

e.) Determine the heat transferred in the heat exchanger and the temperature of the air entering
the combustor.

The capacity rates of the hot and cold air flows are computed:

C h = m h c (22)

C c = m c c (23)

and the minimum and maximum values of the capacity rates ( C min and C max ) are determined.

C_dot_h=m_dot*c "capacity rate of hot gas"


C_dot_c=m_dot*c "capacity rate of cold gas"
C_dot_min=MIN(C_dot_h,C_dot_c) "minimum capacity rate"
C_dot_max=MAX(C_dot_h,C_dot_c) "maximum capacity rate"

The capacity ratio and number of transfer units are computed:

C
CR =  min (24)
Cmax

UA
NTU =  (25)
Cmin

and the function HX is used to determine the effectiveness of the heat exchanger (ε). Note that
the geometry of the passages makes this a clearly a heat exchanger where both sides are
unmixed:

C_R=C_dot_min/C_dot_max "capacity ratio"


NTU=UA/C_dot_min "number of transfer units"
epsilon=HX('crossflow_both_unmixed', NTU, C_dot_c, C_dot_h, 'epsilon') "access eff-NTU solution"

The maximum possible rate of heat transfer is computed:

qmax = C min (T4 − T2 ) (26)

and the actual heat transfer rate is:

q = ε qmax (27)

q_dot_max=C_dot_min*(T_4-T_2) "maximum heat transfer rate"


q_dot=epsilon*q_dot_max "actual heat transfer rate"

which leads to q = 26.8 kW. The temperature of the air entering the combustor is:

q
T5 = T2 +  (28)
Cc

T_5=T_2+q_dot/C_dot_c "cold gas exit temperature"

which leads to T5 = 708.5 K.

f.) What is the efficiency of the recuperated gas turbine engine?

The amount of heat transfer required by the combustor in the recuperated gas turbine engine is:

qcomb ,r = m c (T3 − T5 ) (29)


The efficiency associated with the recuperated gas turbine engine is:

w net
η gte,r = (30)
qcomb ,r

q_dot_comb_r=m_dot*c*(T_3-T_5)
"recuperated combustor heat transfer rate"
eta_gte_r=w_dot_net/q_dot_comb_r "recuperated GTE efficiency"

which leads to qcomb ,r = 37.5 kW and ηgte,r = 0.376.

g.) What is the mass of fuel that must be carried by the ship for the 2 day mission if it uses a
recuperated gas turbine engine?

The fuel that the ship must carry is:

qcomb ,r t
M fuel ,r = (31)
HV

M_fuel_r=q_dot_comb_r*time/HV
"Mass of fuel required by recuperated GTE"
M_fuel_r_lbm=M_fuel_r*convert(kg,lbm) "in lbm"

which leads to Mfuel,r = 147 kg (325 lbm).

h.) The density of the metal separating the cold channels from the hot channels is ρm = 8000
kg/m3 and the density of the fins is ρf = 7500 kg/m3. What is the mass of the heat exchanger?

The mass of the heat exchanger is given by:

M HX = L2 thw N c ρ m + 2 N c N f L th f hc ρ f (32)

rho_m=8000 [kg/m^3] "density of stainless steel"


rho_f=7500 [kg/m^3] "density of aluminum"
M_HX=L^2*th_w*N_c*rho_m+2*N_c*N_f*L*th_f*h_c*rho_f "mass of the heat exchanger"

which leads to MHX = 3.8 kg.

i.) What is the net savings in mass associated with using the air-to-air recuperated gas turbine
engine for the 2 day mission.

The net savings in mass if the saving in fuel less the heat exchanger mass:

M savings = M fuel ,ur − M fuel ,r − M HX (33)


M_savings=M_fuel_ur-M_fuel_r-M_HX
"net savings in mass associated with the recuperated gas turbine engine"

which leads to Msavings = 101.6 kg.

j.) Plot the net savings in mass as a function of the width of the heat exchanger (W). Your plot
should show a maximum; explain why.

Figure 2 illustrates the mass savings as a function of the width of the heat exchanger. At very
low width the mass savings is small because the heat exchanger is not very effective. At high
width the mass savings is again small because the heat exchanger mass is large and the
effectiveness is near its maximum value so the fuel savings is not increasing.

Figure 2: Mass savings as a function of the width of the recuperative heat exchanger.
Problem 8.3-4 (8-8 in text): Heat Exchanger Run-Around Loop
Buildings that have high ventilation rates can significantly reduce their heating load by
recovering energy from the exhaust air stream. One way in which this can be done is by use of a
run-around loop shown in Figure P8.3-4. As shown in the figure, a run-around loop consists of
two conventional liquid to air cross-flow heat exchangers. An ethylene glycol solution with 35%
mass percent glycol is pumped at a rate m g = 1 kg/s through both heat exchangers. The specific
heat of this glycol solution is cg = 3.58 kJ/kg-K. (Note that the properties of glycol solutions can
be determined using the brineprop2 function in EES). During winter operation, the glycol
solution is heated by the warm air exiting in the exhaust duct. The warm glycol solution is then
used to preheat cold air entering from outdoors through the ventilation duct.

ventilation duct exhaust duct


m a = 5 kg/s, T3 = 25°C
(2)
m g = 1 kg/s (3)

(5)
pump

(6)
(4)
(1)
m a = 5 kg/s, T1 = −10°C
Figure P8.3-4: Run-Around loop for energy recovery

In the present case, outdoor air is blown into the building at a rate of m a = 5 kg/s from outdoors.
The outdoor temperature is T1 = -10°C. The building is tightly constructed so the exhaust air
flow rate may be assumed to be equal to the ventilation air flow rate ( m a = 5 kg/s ). The air
leaving the building through the exhaust duct is at T3 = 25°C. The cross-flow heat exchangers in
the exhaust and ventilation streams are identical, each having a finned coil configuration and an
estimated conductance UA = 10 kW/K.
a.) Determine the effectiveness of the ventilation and exhaust heat exchangers.

Known information is entered into EES.

$UnitSystem SI MASS RAD PA K J


$TABSTOPS 0.2 0.4 0.6 0.8 3.5 in

"known information from problem statement"


P=101.3 [kPa]*convert(kPa,Pa) "atmospheric pressure"
T_3=converttemp(C,K,25 [C]) "temperature of air entering exhaust duct"
T_1=converttemp(C,K,-10 [C]) "outdoor air temperature"
m_dot_air=5.0 [kg/s] "ventilation air flowrate"
V_dot=m_dot_air*volume(Air,T=T_1,P=P) "ventilation volumetric flow - just for interest"
m_dot_g=1.0 [kg/s] "mass flow rate of glycol solution"
c_g=3.58 [kJ/kg-K]*convert(kJ/kg-K,J/kg-K) "specific heat capacity of glycol solution"
UA=10 [kW/K]*convert(kW/K,W/K) "conductance"

The capacity rates of the air and glycol are obtained from:
C air = m air cair (1)

C g = m g cg (2)

The minimum capacitance rate ( C min ) is obtained using the Min function in EES. The number of
transfer units is:

UA
NTU = (3)
C min

The effectiveness (ε) is obtained from the HX function in EES.

c_air=cp(Air,T=(T_1+T_3)/2) "ave. specific heat of air"


C_dot_air=m_dot_air*c_air "capacitance rate of the air for both heat exchangers"
C_dot_g=m_dot_g*c_g "capacitance rate of the glycol"
C_dot_min=min(C_dot_air, C_dot_g) "minimum capacitance rate"
NTU=UA/C_dot_min "definition of NTU"
epsilon_hx=HX('crossflow_one_unmixed', NTU, C_dot_air, C_dot_g, 'epsilon') "effectiveness"

which leads to ε = 0.702.

b.) Determine the temperatures of the glycol solution at states (5) and (6).

This part is tricky because it is necessary to solve the energy balances and heat exchanger
relations for the exhaust and ventilation heat exchangers simultaneously. Although it may not be
necessary, it is easier to do this calculation stepwise by guessing the temperature T5.

T_5=converttemp(C,K,10 [C]) "guess - remove this when coupling to ventilation hx"

The heat transfer for the heat exchanger in the exhaust duct is:

qex = ε C min (T3 − T5 ) (4)

The temperatures of the glycol and air leaving the exhaust duct are computed using energy
balances on the glycol and air, respectively:

q
T6 = T5 + ex (5)
Cg

q
T4 = T3 −  ex (6)
Cair

q_dot_HXex=epsilon_hx*C_dot_min*(T_3-T_5) "rate of heat transfer for exhaust hx"


T_6=T_5+q_dot_HXex/C_dot_g "temperature of glycol leaving exhaust duct"
T_4=T_3-q_dot_HXex/C_dot_air "temperature of air leaving exhaust duct"

The heat transfer rate for the heat exchanger in the ventilation duct is:

qv = ε C min (T6 − T1 ) (7)

The temperature of the air leaving the ventilation duct is obtained from an energy balance:

q
T2 = T1 +  v (8)
Cair

q_dot_HXv=epsilon_hx*C_dot_min*(T_6-T_1) "rate of heat transfer for ventilation hx"


T_2=T_1+q_dot_HXv/C_dot_air "temperature of air leaving ventilation duct"

The problem is solved and the guess values updated. The assumed value of T5 is commented out
and T5 is recalculated according to:

q
T5 = T6 − ex (9)
Cg

{T_5=converttemp(C,K,10 [C])} "guess - remove this when coupling to ventilation hx"


T_5=T_6-q_dot_HXv/C_dot_g "recalculate temperature of glycol leaving ventilation duct"

c.) Determine the overall effectiveness of the run-around loop.

The overall effectiveness of the run-around loop is defined as the ratio of the actual to the
maximum rates of heat transfer. The actual rate of heat transfer is qex (or qv , they are identical).
The maximum rate of heat transfer is the product of the minimum air capacitance rate and the
difference between T3 and T1. Since the air flow rate is the same for both heat exchangers:

qex
ε runaround =  (10)
C air ( T3 − T1 )

epsilon_runaround=q_dot_HXv/(C_dot_air*(T_3-T_1)) "definition of run-around loop effectiveness"

which leads to an overall effectiveness of 0.386.

d.) It has been suggested that the performance of the run-around loop can be improved by
optimizing the glycol flow rate. Plot the run-around loop overall effectiveness as a function
of the glycol solution flow rate for 0.1 kg/s < m g < 4 kg/s. Assume that the conductance of
the heat exchangers vary with glycol solution flow rate to the 0.4 power based on a value of
UA = 10 kW/K at m g = 1 kg/s. What flow rate do you recommend?

The conductance is specified according to:


{UA=10 [kW/K]*convert(kW/K,W/K) "conductance"}
UA=10 [kW/K]*convert(kW/K,W/K)*(m_dot_g/1 [kg/s])^0.4 "conductance as a function of glycol flow rate"

Figure 2 illustrates the overall effectiveness of the runaround loop as a function of the glycol
flow rate.

0.6
Runaround loop effectiveness

0.55
0.5
0.45
0.4
0.35
0.3
0.25
0.2
0.15
0.1
0.05
0 0.5 1 1.5 2 2.5 3 3.5 4
Mass flow rate of glycol (kg/s)
Figure 2: Runaround loop effectiveness as a function of glycol mass flow rate.
Problem 8.3-9 (8-9 in text)
A concentric tube heat exchanger is built and operated as shown in Figure P8.3-9. The hot
stream is a heat transfer fluid with specific heat capacity cH = 2.5 kJ/kg-K. The hot stream enters
at the center of the annulus at TH,in = 110°C with mass flow rate m H = 0.64 kg/s and then splits
and an equal amount flows in both directions. The cold stream has specific heat capacity cC =
4.0 kJ/kg-K. The cold enters the center pipe at TC,in = 10°C with mass flow rate m C = 0.2 kg/s .
The outlet temperature of the hot-fluid that flows towards the left is TH,out.x=0 = 45°C. The two
sections of the heat exchanger have the same conductance.

TH ,in = 110° C
m H = 0.64 kg/s
TH ,out , x = 0 = 45°C cH = 2.5 kJ/kg-K TH,out,x=L

TC ,in = 10°C
m C = 0.2 kg/s TC,out,x=L
cC = 4 kJ/kg-K TC,out,x=L/2
section 1 section 2
x
Figure P8.3-9: Concentric tube heat exchanger.

a.) Determine the temperature of the cold stream at the midpoint of the center tube and the
temperature of the cold stream leaving the heat exchanger.

The capacitance rates of the hot and cold flows are:

0.64 kg 2.5 kJ kW
C H = m H cH = = 1.6 (1)
s kg-s K

0.2 kg 4.0 kJ kW
C C = m C cC = = 0.8 (2)
s kg-s K

An overall energy balance on section 1 is:

C H
2
(TH ,in − TH ,out , x=0 ) = CC (TC ,out , x= L / 2 − TC ,in ) (3)

which can be solved for the TC,out,x=L/2:


C H
TC ,out , x = L / 2 = TC ,in +
2 C C
(TH ,in − TH ,out , x=0 )
(4)
1.6
TC ,out , x = L / 2 = 10°C + (110°C - 45°C ) = 75°C
2 ( 0.8 )

The effectiveness of section 1 is:

C C (TC ,out , x = L / 2 − TC ,in ) ( 75°C − 10°C ) = 0.65


ε1 = = (5)
(
MIN C , C / 2 (T − T
C H ) H ,in C ,in ) (110°C − 10°C )

The number of transfer units is obtained from the effectiveness-NTU solution in the book for a
counterflow heat exchanger with ε = 0.65 and CR = 1. Figure 8-14 is used to obtain NTU = 1.9.
The number of transfer units for the section 2 is the same as section 1 because the conductance
and capacitance rates are the same. The effectiveness for section 2 is obtained from Figure 8-15
with NTU = 1.9 and CR = 1, ε2 = 0.49. The heat transfer rate in section 2 is computed according
to:

( )
q2 = ε 2 MIN C C , C H / 2 (TH ,in − TC ,out , x = L / 2 )
0.49 0.8 kW (110°C − 75°C ) (6)
= = 13.7 kW
K

An energy balance on the cold fluid leads to:

q2 = C C (TC ,out , x = L − TC ,out , x = L / 2 ) (7)

which can be solved for TC,out,x=L:

q2 13.7 kW K
TC ,out , x = L = TC ,out , x = L / 2 + = 75°C+ = 92.1°C (8)

CC 0.8 kW

An energy balance on the hot fluid leads to:

C H
q2 =
2
(TH ,in − TH ,out , x= L ) (9)

which can be solved for TH,out,x=L:

q2 2 13.7 kW K
TH ,out , x = L = TH ,in − 2 = 110°C- = 92.9°C (10)
C H 1.6 kW

b.) Calculate the overall effectiveness of this heat exchanger considering both sections.
The overall effectiveness is:

C C (TC ,out , x = L − TC ,in )


ε=
(
MIN C , C (T − T
C H ) H ,in C ,in ) (11)
=
( 92.1 − 10 ) = 0.82
(110 − 10 )
c.) How will the overall effectiveness be affected if the inlet temperature is increased to 400°C.
(Assume that the properties of the heat transfer fluid are independent of temperature.)

There will be no change.

d.) Is the overall effectiveness of this heat exchanger higher, lower, or the same as a counter-
flow heat exchanger having the same inlet conditions? Justify your answer.

The overall effectiveness is less than a counter-flow heat exchanger because section 2 is parallel
flow.
Problem 8.3-12 (8-10 in text): Engine block and radiator
The power delivered to the wheels of a vehicle ( w ) as a function of vehicle speed (V) is given
⎡ hp ⎤ ⎡ hp ⎤ 2
by: w = −0.3937 [ hp ] + 0.6300 ⎢ ⎥ V + 0.01417 ⎢ 2⎥
V where power is in horsepower and
⎣ mph ⎦ ⎣ mph ⎦
velocity is in mph. The amount of heat rejected from the engine block ( qb ) is approximately
equal to the amount of power delivered to the wheel (the rest of the energy from the fuel leaves
with the exhaust gas). The heat is removed from the engine by pumping water through the
engine block with a mass flow rate of m = 0.80 kg/s. The thermal communication between the
engine block and the cooling water is very good, therefore you may assume that the water will
exit the engine block at the engine block temperature (Tb). For the purpose of this problem you
may model the water as having constant properties that are consistent with liquid water at 70°C.
The heat is rejected from the water to the surrounding air using a radiator, as shown in Figure
P8.3-12. When the car is moving, air is forced through the radiator due to the dynamic pressure
associated with the relative motion of the car with respect to the air. That is, the air is forced
through the radiator by a pressure difference that is equal to equal to ρa V2/2 where ρa is the
density of air. Assume that the temperature of the ambient air is T∞ = 35°C and model the air in
the radiator assuming that it has constant properties consistent with this temperature.

m = 0.8 kg/s
water exits at Tb

L = 10 cm qb
W = 50 cm

engine block

radiator
pf = 1.2 mm

air at T∞ = 35°C
Figure P8.3-12: Engine block and radiator.

The radiator has a plate-fin geometry. There are a series of tubes installed in closely spaced
metal plates that serve as fins. The fin pitch is pf = 1.2 mm and therefore there are W/pf plates
available for heat transfer. The heat exchanger core has overall width W = 50 cm, height H = 30
cm (into the page), and length (in the flow direction) of L = 10 cm. For the purpose of modeling
the air-side of the core you may assume that the air flow is consistent with an internal flow
through rectangular ducts with dimension H x pf. Assume that the fins are 100% efficient and
neglect the thermal resistance between the fluid and the internal surface of the tubes. Also
neglect convection from the external surfaces of the tubes as well as the reduction in the area of
the plates associated with the presence of the tubes.
a.) Using the information above, develop an EES model that will allow you to predict the engine
block temperature as a function of vehicle velocity. Prepare a plot showing Tb vs V and
explain the shape of the plot (if necessary, produce additional plots to help with your
explanation). If the maximum allowable temperature for the engine block is 100°C (in order
to prevent vaporization of the water) then what range of vehicle speeds are allowed? You
should see both a minimum and maximum limit.

The inputs are entered in EES:

$UnitSystem SI MASS RAD PA K J


$Tabstops 0.2 0.4 0.6 3.5 in

V_mph=10 [mph] "Vehicle velocity (mph)"


V=V_mph*convert(mph,m/s) "velocity in m/s"
a_0=-0.3937 [hp]*convert(hp,W) "coefficients for power curve"
a_1=0.63 [hp/mph]*convert(hp/mph,W-s/m)
a_2=0.01417 [hp/mph^2]*convert(hp/mph^2,W-s^2/m^2)
m_dot_w=0.8 [kg/s] "mass flow rate of water"
W=0.50 [m] "width of radiator (m)"
H=0.30 [m] "
L=0.10 [m] "length of radiator (m)"
p_f=0.0012 [m] "fin pitch (m)"
t_f=0.0002 [m] "fin thickness (m)"
T_infinity=converttemp(C,K,35 [C]) "ambient temperature"

The wheel power is computed according to the equation given in the problem statement and set
equal to the heat transfer.

w_dot=a_0+a_1*V+a_2*V^2 "power curve"


q_dot=w_dot "heat removed from engine"

The specific heat capacity of the water (cw) and the properties of the air (ca, ρa, μa, ka, αa, υa, and
Pra) are computed:

T_w=converttemp(C,K,70 [C]) "temperature for properties of water"


P_w=100000 [Pa] "atmospheric pressure"
c_w=cp(Water,T=T_w,P=P_w) "specific heat"

P_a=100000 [Pa] "air pressure"


c_a=cP(Air,T=T_infinity) "specific heat of air"
rho_a=density(Air,T=T_infinity,P=P_a) "density of air"
mu_a=viscosity(Air,T=T_infinity) "viscosity of air"
k_a=conductivity(Air,T=T_infinity) "thermal conductivity of air"
alpha_a=k_a/(rho_a*c_a) "thermal diffusivity of air"
nu_a=mu_a/rho_a "kinematic viscosity of air"
Pr_a=nu_a/alpha_a "Prandtl number of air"

The mass flow rate of the air ( m a ) is assumed. The hydraulic diameter and cross-sectional area
of the air passages are computed according to:

Dhyd = 2 ( p f − t f ) (1)
Ac = W H
(p f −tf ) (2)
pf

The mean velocity of the air in the duct is:

m a
ua = (3)
ρ a Ac

The Reynolds number is:

ua ρ a Dhyd
Re = (4)
μa

The DuctFlow_N procedure in EES is used to obtain the average Nusselt number, Nu , and
average friction factor, f .

m_dot_a=0.5 [kg/s] "guess for mass flow rate of air"


D_hyd=2*(p_f-t_f) "hydraulic diameter"
A_c=W*H*(p_f-t_f)/p_f "cross sectional area"
u_a=m_dot_a/(rho_a*A_c) "air velocity"
Re_a=u_a*rho_a*D_hyd/mu_a "Reynolds number for air"
call DuctFlow_N(Re_a,Pr_a,L/D_hyd, p_f/H,0 [-]: Nusselt_T_bar, Nusselt_H_bar, f_bar)

The pressure drop across the heat exchanger is computed according to:

f ρ a ua2 L
Δpa = (5)
2 Dhyd

The dynamic pressure rise created by the vehicle motion is:

ρa V 2
Δpdyn = (6)
2

DELTAP_a=0.5*f_bar*rho_a*u_a^2*(L/D_hyd) "pressure drop for air"


DELTAP_dyn=0.5*rho_a*V^2 "dynamic pressure produced by vehicle motion"

The problem is solved and the guess values are updated. The assumed value of m a is
commented out and the pressure drop across the heat exchanger is set equal to the dynamic
pressure rise.

{m_dot_a=0.5 [kg/s]} "guess for mass flow rate of air"


DELTAP_a=DELTAP_dyn "set dynamic pressure rise and pressure drop equal"
The average air-side heat transfer coefficient is computed according to:

ka
ha = Nu (7)
Dhyd

The conductance of the heat exchanger is:

W
UA = ha 2H L (8)
pf

h_bar_a=Nusselt_T_bar*k_a/D_hyd "heat transfer coefficient"


UA=h_bar_a*(W/p_f)*H*L*2 "conductance on the heat exchanger"

The capacitance rate of the air and water are computed according to:

C w = m w cw (9)

C a = m a ca (10)

The minimum capacitance rate ( C min ) is identified and the number of transfer units is computed:

UA
NTU =  (11)
Cmin

The effectiveness of the radiator (ε) is computed using the HX function. The engine block
temperature is computed according to:

q = ε C min (Tb − T∞ ) (12)

h_bar_a=Nusselt_T_bar*k_a/D_hyd "heat transfer coefficient"


UA=h_bar_a*(W/p_f)*H*L*2 "conductance on the heat exchanger"
C_dot_w=m_dot_w*c_w "capacitance rate of the water"
C_dot_a=m_dot_a*c_a "capacitance rate of the air"
C_dot_min=Min(C_dot_w,C_dot_a) "minimum capacitance rate"
NTU=UA/C_dot_min "number of transfer units"
eff=HX('crossflow_both_unmixed', NTU, C_dot_a, C_dot_w, 'epsilon') "effectiveness"
q_dot=eff*C_dot_min*(T_b-T_infinity) "solve for engine block temperature"
T_b_C=converttemp(K,C,T_b) "in C"

Figure 2 illustrates the engine block temperature as a function of vehicle speed and shows both a
lower speed limit where the dynamic pressure is insufficient to force a sufficient air flow through
the radiator as well as an upper speed limit where the heat transfer to the radiator is too high.
115
110

Engine block temperature (°C)


105
100
95
90
85
80
75
70
65
60
55
0 10 20 30 40 50 60 70 80
Vehicle velocity (mile/hr)
Figure 2: Engine block temperature as a function of vehicle velocity.

Its not easy to overcome the maximum speed limit identified (a); however, to overcome the
minimum speed limit (so that you can pull up to a stop sign without your car overheating) you
decide to add a fan. The fan can provide at most 500 cfm ( Vo - the open circuit flow) and can
produce at most 2.0 inch H2O (Δpdh - the dead-head pressure). The transition from open circuit to
⎛ V ⎞
dead-head is linear. The fan curve is given by: Δp fan = Δpdh ⎜1 − ⎟

⎝ Vo ⎠
b.) Modify your code to simulate the situation where the air is provided by the fan rather than
the vehicle motion. Overlay a plot showing Tb vs V for this configuration on the one from
(a); have you successfully overcome the lower speed limitation?

The fan characteristics are entered:

"air provided by fan"


V_dot_o_cfm=500 [cfm] "maximum velocity of fan"
V_dot_o=V_dot_o_cfm*convert(ft^3/min,m^3/s) "convert to SI"
DELTAP_dh_inH2O=2 [inH2O] "maximum pressure drop"
DELTAp_dh=DELTAP_dh_inH2O*convert(inH2O,Pa) "convert to SI"

The volumetric flow rate of air is computed:

m
Va = (13)
ρa

and the equation in the problem statement is used to compute the pressure rise caused by the fan:

V_dot_a=m_dot_a/rho_a "volumetric flow rate of air"


DELTAP_f=DELTAP_dh*(1-V_dot_a/V_dot_o) "fan curve"

The pressure drop across the heat exchanger is set equal to the pressure rise produce by the fan
rather than the dynamic pressure rise:
{DELTAP_a=DELTAP_dyn} "set dynamic pressure rise and pressure drop equal"
DELTAP_a=DELTAP_f "set equal to the pressure drop of the air"

Figure 3 illustrates the engine block temperature as a function of vehicle speed using the fan
overlaid onto Figure 2. Note that the addition of the fan maintains the engine at a reasonable
temperature even at very low speed.
115
110
Engine block temperature (°C)

105
100
95
90
85 without fan
80
75 with fan
70
65
60
55
0 10 20 30 40 50 60 70 80
Vehicle velocity (mile/hr)
Figure 3: Engine block temperature as a function of vehicle velocity with fan and without fan.
Problem 8.3-14 (8-11 in text)
A parallel-flow heat exchanger has a total conductance UA = 10 W/K. The hot fluid enters at
Th,in = 400 K and has a capacity rate C h = 10 W/K. The cold fluid enters at Tc,in = 300 K and has
a capacity rate C = 5 W/K.
c

a.) Determine the number of transfer units (NTU), effectiveness (ε), heat transfer rate ( q ), and
exit temperatures (Th,out and Tc,out) for the heat exchanger.

The number of transfer units is calculated according to:

UA
NTU = = 2.0 (1)
(
MIN C c , C h )
The capacity ratio is defined as:

CR =
(
MIN C c , C h
= 0.5
) (2)
MAX C , C ( c h )
According to Figure 8-15 in the text, the effectiveness is ε = 0.64. The maximum possible heat
transfer rate for the heat exchanger is:

( )
qmax = MIN C c , C h (Th ,in − Tc ,in ) = 500 W (3)

The actual heat transfer is therefore:

q = ε qmax = 315 W (4)

The temperatures leaving the heat exchanger are computed according to:

q
Th ,out = Th ,in −  = 369 K (5)
Ch

q
Tc ,out = Tc ,in + = 363 K (6)
C c

b.) Sketch the temperature distribution within the heat exchanger.


Figure 1: Sketch of the temperature distribution within the heat exchanger.

The capacitance rate of the hot stream is twice that of the cold stream and therefore the
temperature change of the hot stream must be half that of the cold stream. The exit temperatures
were calculated in (a); the sketch is shown in Figure 1.

c.) Sketch the temperature distribution within the heat exchanger if the conductance of the heat
exchanger is very large; that is, what is the temperature distribution in the limit that UA → ∞.

Figure 2: Sketch of the temperature distribution within the heat exchanger as UA → ∞.


In the limit that UA → ∞ the temperatures of the two streams will meet, as shown in Figure 2.
Note that the temperature change of the hot stream is still 1/2 that of the cold stream and so the
effectiveness in this limit must be 2/3.

d.) Sketch how the hot exit temperature will change as the total conductance (UA) is varied, with
all other quantities held constant at the values listed in the problem statement. Be sure to
indicate how your plot behaves as UA approaches zero and as UA approaches infinity.

Figure 3: Sketch of the temperature distribution within the heat exchanger as UA → ∞.

The requested sketch is shown in Figure 3. If UA = 0 then the hot fluid temperature will not
change and so Th,out = 400 K. In the limit that UA → ∞, the hot fluid temperature will change by
its maximum possible amount based on the capacitance ratio. Figure 2 shows that temperature of
the hot fluid must change by an amount that is half that of the temperature change of the cold
fluid and so the hot fluid exit temperature cannot drop below 366.7 K.
Problem 8.3-15 (8-12 in text)
A heat exchanger has a core geometry that corresponds to finned circular tube core
'fc_tubes_s80_38T' in the compact heat exchanger library. The frontal area of the core has
dimensions W = 7.75 inch and H = 7.75 inch. The length of the core is L = 1.5 inch. The core is
integrated with a fan that has a head-flow curve given by: Δp = a − bVa where Δp is the pressure
rise across the fan, V is the volumetric flow rate of air, a = 0.3927 inH2O and b = 0.0021
a
inH2O/cfm are the coefficients of the fan curve. The manufacturer has tested the heat exchanger
with atmospheric air at Ta,in = 20ºC and water at Tw,in = 75ºC, pw = 65 psia flowing through the
tubes. The test data are shown in Table P8.3-15. The tubes are plumbed in series (i.e., all of the
water flows through each tube) and the tube thickness is tht = 0.035 inch.

Table P8.3-15: Manufacturer's data for heat exchanger.


Water flow Water outlet
rate temperature
0.13 gpm 44.3ºC
0.25 gpm 51.1ºC
0.5 gpm 60.1ºC
1 gpm 66.9ºC
2 gpm 70.8ºC
4 gpm 72.9ºC

a.) Develop a model using the effectiveness-NTU technique that can predict the outlet
temperature of the water for a water flow rate of the water Vw .

Enter the input information from the problem statement into EES.

$UnitSystem SI MASS RAD PA K J


$Tabstops 0.2 0.4 0.6 3.5 in

"Inputs"
W=7.75 [inch]*convert(inch,m) "width of core"
H=7.75 [inch]*convert(inch,m) "height of core"
L=1.5 [inch]*convert(inch,m) "length of core"
C$='fc_tubes_s80-38T' "identifier of core geometry"
a=0.3927 [inH2O]*convert(inH2O,Pa) "coefficient on fan curve"
b=0.0021 [inH2O-min/ft^3]*convert(inH2O-min/ft^3,Pa-s/m^3) "coefficient on fan curve"
T_w_in=converttemp(C,K,75 [C]) "inlet water temperature"
T_a_in=converttemp(C,K,20 [C]) "inlet air temperature"
p_w=65 [psi]*convert(psi,Pa) "water pressure"
V_dot_w_gpm=1 [gal/min] "water flow rate, in gpm"
V_dot_w=V_dot_w_gpm*convert(gal/min,m^3/s) "water flow rate"
p_a=1 [atm]*convert(atm,Pa) "air pressure"
N_tube=1 [-] "number of tube passes"
th_t=0.035 [inch]*convert(inch,m) "thickness of tube wall"

The average of the inlet temperatures are used to compute the properties of the water and the air.

T_a_avg=(T_w_in+T_a_in)/2 "air temperature to use for property calculations"


T_w_avg=(T_w_in+T_a_in)/2 "water temperature to use for property calculations"
The mass flow rate of air ( m a ) is guessed and used to calculate the volumetric flow rate of air is
computed according to:

m
Va = a (1)
ρa

The fan curve is used to compute the pressure rise generated by the fan (Δpf). The function
CHX_DELTAp_finned_tube is used to compute the pressure drop across the heat exchanger
(Δpa).

"Air flow rate"


m_dot_a=0.1 [kg/s] "mass flow rate of air - guess"
rho_a=density(Air,P=p_a,T=T_a_avg) "density of air"
V_dot_a=m_dot_a/rho_a "volumetric flow rate of air"
DELTAp_fan=a-b*V_dot_a "pressure rise produced by fan"
A_fr=W*H "frontal area"
Call CHX_DELTAp_finned_tube(C$, m_dot_a, A_fr,L, 'Air', T_a_in, T_a_in, p_a: DELTAp_a)
"compact heat exchanger correlation for DP"

The problem is solved and the guess values are updated. The guessed value of m a is commented
out and the pressure rise generated by the fan is set equal to the pressure drop across the heat
exchanger:

{m_dot_a=0.1 [kg/s]} "mass flow rate of air - guess"


DELTAp_fan=DELTAp_a "set pressure drop across core equal to pressure rise produced by fan"

The geometric characteristics of the heat exchanger (Do, finpitch, Dh, finthk, σ, α, and Afin/A) are
obtained using the procedure CHX_geom_finned_tube. The heat transfer coefficient on the air-side
( ha ) is calculated from the procedure CHX_h_finned_tube.

"Air-side resistance"
Call CHX_geom_finned_tube(C$: D_o, fin_pitch, D_h, fin_thk, sigma, alpha, A_fin\A)
"get geometry of core"
Call CHX_h_finned_tube(C$, m_dot_a, A_fr, 'Air',T_a_avg, p_a:h_bar_a)
"compact heat exchanger correlation for air-side heat transfer coefficient"

The fin efficiency can be estimated by treating the fin plate as an annular fin. This can be
accomplished by examining the core geometry from the Function Information Window from the
Compact Heat Exchanger library (Figure 1) which shows that the tube-to-tube distance is Ht =
25.4 mm.
Figure 1: Function Information Window for the Compact Heat Exchanger library.

The fin efficiency (ηfin) of an annular fin with an outer radius of Ht is obtained using the
eta_fin_annular_rect function.

H_t=20 [mm]*convert(mm,m) "center-to-center distance between tubes"


k_f=k_('Copper', T_a_avg) "conductivity of fin material"
eta_fin=eta_fin_annular_rect(fin_thk, D_o/2, H_t, h_bar_a, k_f) "fin efficiency estimate"

The thermal resistance of the finned surface area is:

1
R fin = (2)
Afin
ha W H L α η fin
A

The thermal resistance of the unfinned surface area is:

1
R fin = (3)
Afin
ha W H L α η fin
A

R_fin=1/(h_bar_a*W*L*H*alpha*A_fin\A*eta_fin) "finned surface area resistance"


R_unfin=1/(h_bar_a*W*L*H*alpha*(1-A_fin\A)) "unfinned surface area resistance"

The total length of tube in the heat exchanger is estimated according to:

⎛ 1 ⎞ ⎛ A ⎞
Ltotal fin pitch ⎜ − finthk ⎟ π Do = W L H α ⎜1 − fin ⎟ (4)
⎜ fin pitch ⎟ ⎝ A ⎠
⎝ ⎠
The length of tube per circuit is:

Ltotal
Ltube = (5)
N tube

The total mass flow rate of water is given by:

m = V ρ w (6)

The procedure PipeFlow is used to compute the water-side heat transfer coefficient ( hw ). The
thermal resistance to convection to the water is:

1
Rw = (7)
hw π Ltotal ( Do − 2 tht )

"Water-side resistance"
L_total*fin_pitch*(1/fin_pitch-fin_thk)*pi*D_o=W*L*H*alpha*(1-A_fin\A) "total length of tube"
L_tube=L_total/N_tube "length of each circuit"
rho_w=density(Water,p=p_w,T=T_w_avg) "density of water"
m_dot_w=rho_w*V_dot_w "mass flow of water"
call PipeFlow('Water',T_w_avg,p_w,m_dot_w/N_tube,D_o-2*th_t,L_tube,0[-]:h_bar_w, h_H ,&
DELTAP_w, Nusselt_T, f, Re) "access correlations for internal flow in a tube"
R_w=1/(L_total*pi*(D_o-2*th_t)*h_bar_w) "water-side resistance"

The fouling factor (FF) is obtained from the procedure FoulingFactor in EES. The fouling
resistance is computed according to:

FF
Rf = (8)
π Ltotal ( Do − 2 tht )

"Fouling resistance"
FF=FoulingFactor('Closed-loop treated water') "fouling factor"
R_foul=FF/(L_tube*pi*(D_o-2*th_t)) "fouling resistance"

The total resistance is:

−1
⎛ 1 1 ⎞
Rtotal = Rw + R f + ⎜ + ⎟⎟ (9)
⎜R
⎝ fin Runfin ⎠

The conductance is:

1
UA = (10)
Rtotal

"Heat exchanger calculations"


R_total=R_w+R_foul+(1/R_fin+1/R_unfin)^(-1) "total resistance"
UA=1/R_total "total conductance"

The capacitance rates of the water and air are computed:

C a = m a ca (11)

C w = m w cw (12)

The minimum capacitance rate ( C min ) is computed and used to determine the number of transfer
units:

UA
NTU =  (13)
Cmin

The effectiveness of the heat exchanger (ε) is determined using the HX function in EES.

c_a=cP(Air,T=T_a_avg) "air specific heat capacity"


c_w=cP(Water,T=T_w_avg,P=p_w) "water specific heat capacity"
C_dot_a=m_dot_a*c_a "air capacitance rate"
C_dot_w=m_dot_w*c_w "water capacitance rate"
C_dot_min=MIN(C_dot_a,C_dot_w) "minimum capacitance rate"
NTU=UA/C_dot_min "number of transfer units"
eff=HX('crossflow_both_unmixed', NTU, C_dot_a, C_dot_w, 'epsilon') "access eff-NTU solutions"

The maximum possible rate of heat transfer is:

qmax = C min (Tw,in − Ta ,in ) (14)

The actual rate of heat transfer is:

q = ε qmax (15)

The air and water outlet temperatures are computed according to:

q
Tw,out = Tw,in −  (16)
Cw

q
Ta ,out = Ta ,in +  (17)
Ca

"Energy balances to determine outlet temperatures"


q_dot_max=C_dot_min*(T_w_in-T_a_in) "maximum possible heat transfer rate"
q_dot=q_dot_max*eff "actual heat transfer rate"
T_w_out=T_w_in-q_dot/C_dot_w "water exit temperature"
T_w_out_C=converttemp(K,C,T_w_out) "in C"
T_a_out=T_a_in+q_dot/C_dot_a "air exit temperature"
T_a_out_C=converttemp(K,C,T_a_out) "in C"

b.) Plot the outlet temperature of the water as a function of the water flow rate and overlay the
manufacturer's data onto your plot.

Figure 2 illustrates the water exit temperature as a function of water flow rate. Overlaid on
Figure 2 is the manufacturer's data from Table 8.3-15.
75
70
Water exit temperature (°C)

65
60
55
50
45
40
35
manufacturer's data
30
model
25
20
0.1 1 10
Water flow rate (gal/min)
Figure 2: Water exit temperature as a function of water flow rate predicted by the model and from the
manufacturer's data.
Problem 8.6-2 (8-13 in text): Joule Thomson Cycle
A Joule-Thomson refrigeration cycle is illustrated in Figure P8.6-2.

m = 0.01 kg/s
ph = 6.5 MPa
pc = 100 kPa Th ,in = 20°C
(h,in)
(c,out)

UA = 20 W/K

Tc,in = 150 K
(c,in)
qload (h,out)

(v,out) expansion valve


Figure P8.6-2: Joule-Thomson refrigeration cycle.

The system uses pure argon as the working fluid. High pressure argon enters a counterflow heat
exchanger with mass flow rate m = 0.01 kg/s at Th,in = 20ºC and ph = 6.5 MPa. The argon flows
through the heat exchanger where it is pre-cooled by the low pressure argon returning from the
cold end of the cycle. The high pressure argon leaving the heat exchanger enters an expansion
valve where it is expanded to pc = 100 kPa. The argon passes through a load heat exchanger
where it accepts a refrigeration load, qload , and it heated to Tc,in = 150 K. The conductance of the
heat exchanger is UA= 20 W/K. Neglect pressure loss in the heat exchanger on both the hot and
cold sides of the heat exchanger.
a.) Use the effectiveness-NTU method to estimate the effectiveness of the heat exchanger and
the rate of heat transferred from the hot to the cold stream in the heat exchanger. Calculate
the specific heat capacity of the high- and low-pressure argon using the average of the hot
and cold inlet temperatures.

The inputs are entered in EES:

$UnitSystem SI MASS RAD PA K J


$Tabstops 0.2 0.4 0.6 3.5 in

"Inputs"
m_dot=0.010 [kg/s] "mass flow rate rate"
T_h_in=converttemp(C,K,20[C]) "hot inlet temperature"
F$='Argon' "fluid"
P_h=6.5 [MPa]*convert(MPa,Pa) "high pressure"
P_c=100 [kPa]*convert(kPa,Pa) "low pressure"
T_c_in=150 [K] "cold inlet temperature"
UA=20 [W/K] "conductance"

The average temperature in the heat exchanger is computed according to:


Th ,in + Tc ,in
T = (1)
2

and used to compute the specific heat capacity of the hot and cold streams using EES' built-in
property routines (ch and cc):

"Solution based on eff-NTU method with average properties"


T_avg=(T_h_in+T_c_in)/2 "average temperature"
c_h_avg=cP(F$,P=P_h,T=T_avg) "average hot side specific heat capacity"
c_c_avg=cP(F$,P=P_c,T=T_avg) "average cold side specific heat capacity"

The capacitance rates on the hot and cold sides are computed according to:

C h = m ch (2)

C c = m cc (3)

The minimum and maximum capacitance rates ( C min and C max ) are computed using the MIN and
MAX functions.

C_dot_h_avg=c_h_avg*m_dot "hot-side capacitance rate"


C_dot_c_avg=c_c_avg*m_dot "cold-side capacitance rate"
C_dot_min=MIN(C_dot_h_avg,C_dot_c_avg) "minimum capacitance rate"
C_dot_max=MAX(C_dot_h_avg,C_dot_c_avg) "maximum capacitance rate"

The number of transfer units is computed according to:

UA
NTU =  (4)
Cmin

The effectiveness (ε) is computed using the effectiveness-NTU solution accessed by the HX
function in EES. The heat transfer rate in the heat exchanger is computed according to:

q = ε C min (Th,in − Tc ,in ) (5)

NTU=UA/C_dot_min "number of transfer units"


eff=HX('counterflow', NTU, C_dot_min, C_dot_max, 'epsilon')
"access eff-NTU solution for a counterflow heat exchanger"
q_dot=eff*C_dot_min*(T_h_in-T_c_in) "heat transfer rate"

which leads to ε = 0.877 and q = 656.5 W.

b.) Determine the refrigeration load provided by the cycle.


The enthalpy of the hot and cold inlet streams (ih,in and ic,in) are calculated using EES' internal
property routines at the inlet temperatures and pressures. The outlet enthalpies are calculated
using energy balances:

q
ih ,out = ih ,in − (6)
m

q
ic ,out = ic ,in + (7)
m

The outlet temperatures (Th,out and Tc,out) are calculated at the outlet enthalpies and pressures
using EES' internal property routines.

i_h_in=enthalpy(F$,P=P_h,T=T_h_in) "inlet hot-side enthalpy"


i_c_in=enthalpy(F$,P=P_c,T=T_c_in) "inlet cold-side enthalpy"
i_h_out=i_h_in-q_dot/m_dot "outlet hot-side enthalpy"
i_c_out=i_c_in+q_dot/m_dot "outlet cold-side enthalpy"
T_h_out=temperature(F$,h=i_h_out,P=P_h) "hot outlet temperature"
T_c_out=temperature(F$,h=i_c_out,P=P_c) "cold outlet temperature"

The valve is isenthalpic, therefore:

ih ,out = iv ,out (8)

The temperature of the fluid leaving the valve (Tv,out) is computed at the valve outlet enthalpy
and the cold-side pressure using EES' internal property functions. The refrigeration load is:

qload = m ( ic ,in − iv ,out ) (9)

i_v_out=i_h_out "expansion valve exit enthalpy"


T_v_out=temperature(F$,P=P_c,h=i_v_out) "expansion valve exit temperature"
q_dot_load=m_dot*(i_c_in-i_v_out) "load"

which leads to qload = 31.2 W.

c.) Prepare a plot of refrigeration load as a function of cold inlet temperature for 85 K < Tc,in <
290 K and various values of the conductance. A negative refrigeration load is not physically
possible (without some external cooling); therefore, terminate your plots at qload = 0 W.

Figure P8.6-2(b) illustrates the refrigeration load as a function of cold inlet temperature for
various values of the UA. Notice that when the cold inlet temperature is less than the saturation
temperature for Argon at pc (87.2 K), the refrigeration load drops to zero.
125

100

Refrigeration load (W)


30 W/K
75

25 W/K
50
20 W/K

25 15 W/K

UA = 10 W/K

0
100 125 150 175 200 225 250 275
Cold inlet temperature (K)
Figure 8.6-2(b): Refrigeration load predicted by the ε-NTU method as a function of cold inlet temperature for
various values of UA.

d.) Instead of using the effectiveness-NTU method, divide the heat exchanger into sub-heat
exchangers as discussed in Section 8.6.3. What is the heat transferred in the heat exchanger
for the conditions listed in the problem statement?

The solution using the ε-NTU technique is commented out. The sub-heat exchanger model for
the counter-flow heat exchanger is shown in Figure 8.6-2(c).

TH,1 HX 1 TH,2 HX 2 TH,3 TH,N HX N TH,N+1


m
q q q
...
TC,1 N TC,2 N TC,3 TC,N N TC,N+1
m
Figure 8.6-2(c): Sub-heat exchanger model of a counter-flow heat exchanger.

The hot-side exit temperature (TH,out) is assumed:

T_h_out= 200 [K] "guess for the hot outlet temperature"

and used to compute the total heat transfer rate in the heat exchanger:

q = m ( ih ,in − ih ,out ) (10)

where ih,in and ih,out are the specific enthalpy at the inlet and outlet states (ih,in and ih,out),
respectively, calculated using EES' internal property routine.

i_h_in=enthalpy(F$,P=P_h,T=T_h_in) "inlet hot-side enthalpy"


i_h_out=enthalpy(F$,T=T_h_out,P=P_h) "hot outlet enthalpy"
q_dot=m_dot*(i_h_in-i_h_out) "heat transfer rate"
The cold-side inlet enthalpy (ic,in) is computed at the inlet temperature and pressure. The cold-
side outlet enthalpy (ic,out) is computed according to:

q
ic ,out = ic ,in + (11)
m

The cold-side outlet temperature (Tc.out) can be obtained using EES' internal property routine:

i_c_in=enthalpy(F$,P=P_c,T=T_c_in) "inlet cold-side enthalpy"


i_c_out=i_c_in+q_dot/m_dot "cold outlet enthalpy"
T_c_out=temperature(F$,P=P_c,h=i_c_out) "cold outlet temperature"

The total heat transferred in the heat exchanger increases as you move from left-to-right in
Figure P8.6-2(c) according to:

q
qi = ( i − 1) for i = 1.. ( N + 1) (12)
N

N=10 [-] "number of sub-heat exchangers"


duplicate i=1,N
q_dot[i]=i*q_dot/N "total heat transfer rate"
end

The temperatures Th,1 and Tc,1 are the hot-side inlet and cold-side outlet fluid inlet temperatures,
respectively, for the counter-flow configuration. The enthalpies ih,1 and ic,1 are the associated
specific enthalpies associated with these states.

T_h[1]=T_h_in "hot-side inlet temperature"


T_c[1]=T_c_out "cold-side outlet temperature"
i_h[1]=i_h_in "hot_side inlet enthalpy"
i_c[1]=i_c_out "cold-side outlet enthalpy"

An energy balance on the hot-side provides the enthalpy leaving each of the sub-heat
exchangers:

q
ih ,i +1 = ih ,i − for i = 1..N (13)
N m

The temperature of the hot-side fluid leaving each sub-heat exchanger (Th,i) is obtained from the
enthalpy and pressure using EES' internal property routine:

duplicate i=2,(N+1)
i_h[i]=i_h[i-1]-q_dot/(N*m_dot) "energy balance on hot-side of each sub-heat exchanger"
T_h[i]=temperature(F$,h=i_h[i],P=p_h) "temperature leaving hot-side of each sub-heat exchanger"
end
An energy balance on the cold-side provides the enthalpy leaving each of the sub-heat
exchangers:

q
ic ,i +1 = ic ,i − for i = 1..N (14)
N m

The temperature of the cold-side fluid leaving each sub-heat exchanger (Tc,i) is obtained from the
enthalpy and pressure using EES' internal property routine:

duplicate i=2,(N+1)
i_c[i]=i_c[i-1]-q_dot/(N*m_dot) "energy balance on cold-side of each sub-heat exchanger"
T_c[i]=temperature(F$,h=i_c[i],P=p_c) "temperature leaving cold-side of each sub-heat exchanger"
end

The ε-NTU solution can be applied to each of the sub-heat exchangers. The capacitance rates on
the hot- and cold-side within each sub-heat exchanger are calculated:

C h ,i = m
(i
h ,i − ih ,i +1 )
for i = 1..N (15)
(Th ,i − Th ,i +1 )

C c ,i = m
(i
c ,i − ic ,i +1 )
for i = 1..N (16)
(Tc ,i − Tc ,i +1 )

duplicate i=1,N
C_dot_h[i]=m_dot*(i_h[i]-i_h[i+1])/(T_h[i]-T_h[i+1]) "hot-side capacitance rate"
C_dot_c[i]=m_dot*(i_c[i]-i_c[i+1])/(T_c[i]-T_c[i+1]) "cold-side capacitance rate"
end

The effectiveness of each sub-heat exchanger can be computed:

q / N
εi = for i = 1..N (8-17)
( )
MIN C c ,i , C c ,i (Th ,i − Tc ,i +1 )

The number of transfer units required by each sub-heat exchanger (NTUi) is obtained using the ε-
NTU solution for a counter-flow heat exchanger, implemented by the function HX. The
conductance required in each sub-heat exchanger is:

UAi = NTU i MIN C C ,i , C H ,i ( ) (8-18)

duplicate i=1,N
eff[i]=q_dot/(N*MIN(C_dot_h[i],C_dot_c[i])*(T_h[i]-T_c[i+1])) "effectiveness of sub-heat exchanger"
NTU[i]=HX('counterflow', eff[i], C_dot_h[i], C_dot_c[i], 'NTU') "NTU required by sub-heat exchanger"
UA[i]=NTU[i]*MIN(C_dot_h[i],C_dot_c[i]) "conductance in sub-heat exchanger"
end
The conductance for each of the sub-heat exchangers is summed to obtain the total conductance
required by the heat exchanger for the assumed inlet temperature:

N
UA = ∑ UAi (8-19)
i =1

UA_p=sum(UA[1..N]) "total conductance"

The error between the specified and predicted conductance is defined.

err=abs(UA-UA_p) "error between calculated and specified conductance"

The assumed value of the hot exit temperature is commented out:

{T_h_out= 200 [K]} "guess for the hot outlet temperature"

and the error is minimized using the Min/Max command by varying Th,out (note, the option Stop
if error occurs should not be checked so that the optimization process continues even if the value
of Th,out goes out of reasonable bounds leading to non-physical values of ε for some sub-heat
exchangers). The calculation leads to q =636.6 W, which is only 3% in error relative to the
value calculated in (a).

e.) Determine the refrigeration load associated with your prediction from (d).

The valve is isenthalpic, therefore:

ih ,out = iv ,out (20)

The temperature of the fluid leaving the valve (Tv,out) is computed at the valve outlet enthalpy
and the cold-side pressure using EES' internal property functions. The refrigeration load is:

qload = m ( ic ,in − iv ,out ) (21)

i_v_out=i_h_out "expansion valve exit enthalpy"


T_v_out=temperature(F$,P=P_c,h=i_v_out) "expansion valve exit temperature"
q_dot_load=m_dot*(i_c_in-i_v_out) "load"

which leads to qload = 11.3 W.

f.) Overlay on your plot from (c) the refrigeration load as a function of cold inlet temperature for
the same values of the conductance.

Figure P8.6-2(d) illustrates the refrigeration load as a function of Tc,in for the two models at
various values of UA. Notice that the deviation between the models is largest when the
temperatures are low and the non-ideal gas behavior (i.e., the specific heat capacity variations)
are largest.
125
sub-heat exchanger method
30 W/K
ε -NTU method
25 W/K
100

Refrigeration load (W)


75

20 W/K
50
15 W/K

25
UA = 10 W/K

0
100 125 150 175 200 225 250 275
Cold inlet temperature (K)
Figure 8.6-2(d): Refrigeration load predicted by the ε-NTU method and the sub-heat exchanger model as a
function of cold inlet temperature for various values of UA.
Problem 8.6-3 (8-14 in text)
A counter-flow heat exchanger has a total conductance of UA = 130 W/K. Air flows on the hot
side. The air enters at Th,in = 500 K with pressure ph = 1 atm and mass flow rate m h = 0.08 kg/s.
Carbon dioxide flows on the cold side. The CO2 enters at Tc,in = 300 K with pressure pc = 80 atm
and mass flow rate m c = 0.02 kg/s.
a.) Plot the specific heat capacity of air at 1 atm and carbon dioxide at 80 atm and comment on
whether the ε-NTU solution can be applied to this heat exchanger.

The problem information is entered into EES.

$UnitSystem SI MASS RAD PA K J


$Tabstops 0.2 0.4 0.6 3.5 in

"Inputs"
H$='Air_ha' "hot side fluid"
C$='CarbonDioxide' "cold side fluid"
T_h_in=500 [K] "hot inlet temperature"
p_h=1 [atm]*convert(atm,Pa) "hot fluid pressure"
m_dot_h=0.08 [kg/s] "hot fluid mass flow rate"
T_c_in=300 [K] "cold inlet temperature"
p_c_atm=80 [atm] "cold fluid pressure, in atm"
p_c=p_c_atm*convert(atm,Pa) "cold fluid pressure"
m_dot_c=0.02 [kg/s] "cold fluid mass flow rate"
UA=130 [W/K] "total conductance of heat exchanger"

To answer the question in part (a), it is necessary to prepare a Parametric table that includes the
specific heats of carbon dioxide at 80 atm and air at 1 atm evaluated over the temperature range
between 300 and 500 K.

"Generate cp data in Parametric table 1"


c_h=cp(H$,T=T,p=p_h) "specific heat of hot fluid"
c_c=cp(C$,T=T,p=p_c) "specific heat of cold fluid"

Figure 1 illustrates the specific heat capacities of air and carbon dioxide as a function of
temperature and shows that the specific heat for the carbon dioxide varies substantially with
temperature, necessitating a numerical analysis of the heat exchanger.
14000

Specific heat capacity (J/kg-K)


12000 CO2 at 80 atm

10000

8000

6000

4000

2000
air at 1 atm
0
300 350 400 450 500
Temperature (K)
Figure 1: Specific heat capacity of CO2 at 80 atm and air at 1 atm as a function of temperature.

b.) Prepare a solution to this problem by numerically integrating the governing equations using
the Euler technique, as discussed in Section 8.6.2.

The Euler analysis begins by choosing the number of nodes and specifying the dimensionless
position of each node.

xi =
( i − 1) (1)
( N − 1)
The dimensionless distance between adjacent nodes is:

1
Δx = (2)
( N − 1)
N=50 [-] "number of nodes"
Dx_bar=1/(N-1) "distance between nodes"
duplicate i=1,N
x_bar[i]=(i-1)*Dx_bar "dimensionless position"
end

An energy balance on a differential section of the hot flow leads to:

dTh −UA
=  (Th − Tc ) (3)
dx Ch

where C h is the capacitance rate of the hot fluid which is a function of the hot fluid temperature.
An energy balance on a differential section of the cold flow leads to:
dTc −UA
=  (Th − Tc ) (4)
dx Cc

where C c is the capacitance rate of the cold fluid. Equations (3) and (4) are the state equations
for the problem which will be integrated from the hot inlet side ( x = 0) to the hot outlet side ( x =
1). An estimate for the cold exit temperature, Tc,out, is assumed in order to begin the integration.
This equation will later be removed after the cold exit temperature is calculated.

T_c_out=490 [K] "assumed cold exit temperature"


T_h[1]=T_h_in "hot inlet temperature"
T_c[1]=T_c_out "assumed cold outlet temperature"

The integration is accomplished using Euler steps:

Th ,i +1 = Th,i − 
UA
Ch ,T =Th ,i
(Th,i − Tc,i ) Δx for i = 1.. ( N − 1) (5)

Tc ,i +1 = Tc ,i −

UA
Cc ,T =Tc ,i
(Th,i − Tc,i ) Δx for i = 1.. ( N − 1) (6)

duplicate i=1,(N-1)
C_dot_h[i]=cP(H$,T=T_h[i],P=p_h)*m_dot_h "capacity rate of hot fluid"
C_dot_c[i]=cP(C$,T=T_c[i],P=p_c)*m_dot_c "capacity rate of cold fluid"
T_h[i+1]=T_h[i]-UA*(T_h[i]-T_c[i])*Dx_bar/C_dot_h[i] "Euler step for hot fluid"
T_c[i+1]=T_c[i]-UA*(T_h[i]-T_c[i])*Dx_bar/C_dot_c[i] "Euler step for cold fluid"
end

The equations should be solved and the guess values updated. The assumed value of Tc,out is
commented out and an error function is defined as the absolute value between the calculated
value of Tc,in and the specified value of Tc,in:

err = Tc , N − Tc ,in (7)

{T_c_out=490 [K]} "assumed cold exit temperature"


err=abs(T_c[N]-T_c_in) "minimize err to find the value of T_c_out"

Select MinMax from the Calculate window. Minimize err by varying T_c_out as shown in Figure
2. Reasonable bounds (300 K to 500 K) must be specified for T_c_out. The optimization will
determine that Tc,out = 493.4 K.
Figure 2: Find Minimum or Maximum dialog.

c.) Using your solution from (b), plot the temperature of the carbon dioxide and air as a function
of the dimensionless axial position (x/L).

Figure 3 illustrates Tc and Th as a function of x .

Figure 3: Temperature distribution within the heat exchanger.

d.) Plot the rate of heat transfer predicted by the model as a function of the number of integration
steps.

An energy balance on the cold stream provides the heat transfer rate.

q = m c ( ic ,out − ic ,in ) (8)


where ic,out and ic,in are the specific enthalpies of the cold fluid evaluated at Tc,out and Tc,in,
respectively.

i_c_out=enthalpy(C$,p=p_c,T=T_c_out) "enthalpy of cold fluid exit"


i_c_in=enthalpy(C$,p=p_c,T=T_c_in) "enthalpy of cold fluid inlet"
q_dot=m_dot_c*(i_c_out-i_c_in) "heat transfer in heat exchanger"

Construct a Parametric table to include the variables N, q , Tc,out, and err. Use the MinMax Table
selection from the Calculate menu to minimize err for each value of N. Set the bounds on Tc,out
to 450 and 500 K to facilitate the optimization. Figure 4 illustrates the predicted value of the
total heat transfer rate as a function of the number of nodes and indiates that about 30 integration
steps are required.
7,900

7,800
Heat transfer rate (W)

7,700

7,600

7,500

7,400

7,300
0 10 20 30 40 50
Number of integration steps
Figure 4: Predicted heat transfer rate as a function of the number of integration steps.

e.) Prepare a solution to this problem by sub-dividing the heat exchanger into sub-heat
exchangers, as discussed in Section 8.6.3.

Comment out the Euler solution.

{N=50 [-] "number of nodes"


Dx_bar=1/(N-1) "distance between nodes"
duplicate i=1,N
x_bar[i]=(i-1)*Dx_bar "dimensionless position"
end

{T_c_out=490 [K]} "assumed cold exit temperature"


T_h[1]=T_h_in "hot inlet temperature"
T_c[1]=T_c_out "assumed cold outlet temperature"
duplicate i=1,(N-1)
C_dot_h[i]=cP(H$,T=T_h[i],P=p_h)*m_dot_h "capacity rate of hot fluid"
C_dot_c[i]=cP(C$,T=T_c[i],P=p_c)*m_dot_c "capacity rate of cold fluid"
T_h[i+1]=T_h[i]-UA*(T_h[i]-T_c[i])*Dx_bar/C_dot_h[i] "Euler step for hot fluid"
T_c[i+1]=T_c[i]-UA*(T_h[i]-T_c[i])*Dx_bar/C_dot_c[i] "Euler step for cold fluid"
end
err=abs(T_c[N]-T_c_in) "minimize err to find the value of T_c_out"

i_c_out=enthalpy(C$,p=p_c,T=T_c_out) "enthalpy of cold fluid exit"


i_c_in=enthalpy(C$,p=p_c,T=T_c_in) "enthalpy of cold fluid inlet"
q_dot=m_dot_c*(i_c_out-i_c_in) "heat transfer in heat exchanger"}

The sub-heat exchanger model for the counter-flow configuration is shown in Figure 5.

Figure 5: Sub-heat exchanger model of a counter-flow heat exchanger.

The number of sub-heat exchangers (N) is selected hot-side exit temperature (TH,out) is assumed

"Sub-heat exchanger solution"


N=10 [-] "number of sub-heat exchangers"
T_h_out=425 [K] "assumed hot outlet temperature"

The assumed value of TH,out is used to compute the total heat transfer rate in the heat exchanger:

q = m H ( iH ,in − iH ,out ) (9)

where iH,in and iH,out are the specific enthalpy at the inlet and outlet states, respectively, calculated
using EES' internal property routine:

i_h_in=enthalpy(H$,T=T_h_in,P=p_h) "enthalpy of hot inlet fluid"


i_h_out=enthalpy(H$,T=T_h_out,P=p_h) "enthalpy of hot outlet fluid"
q_dot=m_dot_h*(i_h_in-i_h_out) "total heat transfer rate"

The temperature TH,1 and enthalpy iH,1 are the hot inlet temperature and the associated enthalpy.

"Obtain enthalpy & temperature distribution"


T_h[1]=T_h_in "hot-side inlet temperature"
i_h[1]=i_h_in "hot_side inlet enthalpy"

An energy balance on the hot-side fluid provides the enthalpy leaving each of the sub-heat
exchangers:

q
iH ,i = iH ,i −1 − for i = 2..N + 1 (10)
N m H

The temperature of the hot-side fluid leaving each sub-heat exchanger (TH,i) is obtained from the
enthalpy and pressure using EES' internal property routine:
duplicate i=2,(N+1)
i_h[i]=i_h[i-1]-q_dot/(N*m_dot_h) "energy balance on hot-side of each sub-heat exchanger"
T_h[i]=temperature(H$,h=i_h[i],P=p_h) "temperature leaving hot-side of each sub-heat exchanger"
end

The temperature TC,N+1 and enthalpy iC,N+1 are the cold inlet temperature and the associated
enthalpy.

T_c[N+1]=T_c_in "cold side inlet temperature"


i_c[N+1]=enthalpy(C$,p=p_c,T=T_c_in) "cold side inlet enthalpy"

An energy balance on the cold-side provides the enthalpy leaving each of the sub-heat
exchangers:

q
iC ,i = iC ,i +1 + for i = 1..N (11)
N m C

The temperature of the cold-side fluid leaving each sub-heat exchanger (TC,i) is obtained from
the enthalpy and pressure using EES' internal property routine:

duplicate i=1,N
i_c[i]=i_c[i+1]+q_dot/(N*m_dot_c) "energy balance on cold-side of each sub-heat exchanger"
T_c[i]=temperature(C$,h=i_c[i],P=p_c) "temperature leaving cold-side of each sub-heat exchanger"
end

The ε-NTU solution can be applied to each of the sub-heat exchangers. The capacitance rates on
the hot- and cold-side within each sub-heat exchanger are calculated:

C H ,i = m H
(i
H ,i − iH ,i +1 )
for i = 1..N (12)
(TH ,i − TH ,i +1 )

C C ,i = m C
(i
C ,i − iC ,i +1 )
for i = 1..N (13)
(TC ,i − TC ,i +1 )

"Apply effectiveness-NTU solution"


duplicate i=1,N
C_dot_h[i]=m_dot_h*(i_h[i]-i_h[i+1])/(T_h[i]-T_h[i+1]+0.001 [K])"hot-side capacitance rate"
C_dot_c[i]=m_dot_c*(i_c[i]-i_c[i+1])/(T_c[i]-T_c[i+1]+0.001 [K]) "cold-side capacitance rate"
end

The effectiveness of each sub-heat exchanger can be computed:

q / N
εi = for i = 1..N (14)
( )
MIN CC ,i , CH ,i (TH ,i − TC ,i +1 )
 
The number of transfer units required by each sub-heat exchanger (NTUi) is obtained using the ε-
NTU solution for a counter-flow heat exchanger, implemented by the function HX. The
conductance required in each sub-heat exchanger is:

(
UAi = NTU i MIN C C ,i , C H ,i ) (15)

duplicate i=1,N
eff[i]=q_dot/(N*MIN(C_dot_h[i],C_dot_c[i])*(T_h[i]-T_c[i+1])) "effectiveness of sub-heat exchanger"
NTU[i]=HX('counterflow', eff[i], C_dot_h[i], C_dot_c[i], 'NTU') "NTU required by sub-heat exchanger"
UA[i]=NTU[i]*MIN(C_dot_h[i],C_dot_c[i]) "conductance in sub-heat exchanger"
end

The conductance of the heat exchanger is the sum of the conductances of the individual heat
exchangers:

N
UAreq = ∑ UAi (16)
i =1

The problem is solved and the guess values are updated. An error function is defined as the
difference between the predicted and specified heat exchanger conductance:

errU = UA − UAreq (17)

UA_req=sum(UA[1..N]) "total conductance of heat exchanger"


err=abs(UA-UA_req) "error"

The assumed value of Th,out is commented out and the value of errU is minimized by adjusting
Th,out using the Min/Max selection from the Calculate menu.

{T_h_out=425 [K]} "assumed hot outlet temperature"

This leads to Th,out = 403.9 K.

f.) Overlay on your plot from (c) the temperature distribution predicted by your model from (e).

Knowing the conductance within for each subsection allows the dimensionless position between
each sub-heat exchanger to be computed.

x1 = 0 (18)

UAi
xi +1 = xi + for i = 2..N (19)
UAreq

x_bar[1]=0
duplicate i=1,N
x_bar[i+1]=x_bar[i]+UA[i]/UA_req "fraction of total UA"
end

Figure 6 illustrates the temperature of the cold and hot fluid as a function of dimensionless
position predicted by integration of the state equations in (a) and by division into sub-heat
exchangers in (e).
500

475

450
Temperature (K)

425

400
integration of state equations
375 sub-heat exchanger model

350

325

300
0 0.1 0.2 0.3 0.4 0.5 0.6 0.7 0.8 0.9 1
Dimensionless position
Figure 6: Temperature distribution within the heat exchanger obtained by integration of the state equations
and by division into sub-heat exchangers.

g.) Overlay on your plot from (d) the rate of heat transfer predicted by your model from (e) as a
function of the number of sub-heat exchangers.

Figure 7 illustrates the total heat transfer predicted by the sub-heat exchanger model as a
function of the number of sub-heat exchangers overlaid onto Figure 4. Figure 7 illustrates that
the method of using sub-heat exchangers allows many fewer nodes to obtain a stable accurate
result.

7900

7800
Heat transfer rate (W)

7700

7600

7500
integration of governing equations
sub-heat exchanger model
7400

7300
0 10 20 30 40 50
Number of integration steps
Figure 7: Heat transfer rate as a function of N predicted by the models from (a) and (d).
Problem 8.10-1 (8-15 in text)
A solar heating system is shown in Figure P8.10-1.

heated air
′′
qsolar rock bed rock bed

collector

blower
blower

air
air
charging process discharging process
Figure P8.10-1: Solar heating system during charging and during discharging.

During the day, the solar heat is not required and therefore air is blown through a series of solar
collectors where it is heated as shown in Figure P8.10-1. The thermal energy is stored in a large
rock bed regenerator. The rock bed is L= 20 ft long in the flow direction and W = 10 ft x W = 10
ft in cross-sectional area. The bed is filled with Dp = 0.5 inch diameter rocks with density ρr =
100 lbm/ft3 and specific heat capacity cr = 0.2 Btu/lbm-°F. The charging process goes on for
tcharge =12 hr. There are Ncol = 40 solar collectors, each with area 8 ft x 4 ft. During the charging
process, atmospheric air at Tindoor = 70°F enters the collectors where it is heated by the solar
′′ = 750 W/m2. The efficiency of the collector is given by: ηcollector = 0.75 - 0.0015
irradiation qsolar
[K-1] (Tr,in - Toutdoor) where Toutdoor = 10°F and Tr,in is the temperature of the air leaving the
collector and entering the regenerator. The collector efficiency is the ratio of the energy
transferred to the air to the energy incident on the collector. During the night, the energy that
was stored in the rock bed is used to provide heating, as shown in Figure P8.10-1. Air at Tindoor =
70°F enters the rock bed where it is heated. The hot air is provided to the building. The blower
used during both the charging and discharging process has an efficiency of ηb = 0.6 and a
pressure/flow curve that goes linearly from Δpdh = 0.5 inch of water at zero flow to Vopen = 1800
cfm at zero pressure rise. Neglect the pressure drop across the collectors and assume that the
pressure drop that must be overcome by the blower is related to the flow through the rock bed.
The porosity of the rock bed is φ = 0.35 and assume that the rock bed is well-insulated.
a.) What is the temperature of the air entering the rock bed during the charging process and the
mass flow rate of air during the charging and discharging process?

The inputs are entered in EES:

$UnitSystem SI MASS RAD PA K J


$Tabstops 0.2 0.4 0.6 3.5 in

"Inputs"
D_p=0.5 [inch]*convert(inch,m) "rock diameter"
L_ft=20 [ft] "bed length, in ft"
L=L_ft*convert(ft,m) "bed length"
W=10 [ft]*convert(ft,m) "bed width & height"
rho_r=100 [lbm/ft^3]*convert(lbm/ft^3,kg/m^3) "rock density"
c_r=0.2 [Btu/lbm-F]*convert(Btu/lbm-F,J/kg-K) "rock specific heat capacity"
T_outdoor=converttemp(F,K,10 [F]) "outdoor air temperature"
T_indoor=converttemp(F,K,70 [F]) "indoor air temperature"
N_col=40 [-] "number of collectors"
A_collector=8 [ft]*4 [ft]*N_col*convert(ft^2,m^2) "installed collector area"
q``_solar=750 [W/m^2] "solar irradiation"
p_a=1 [atm]*convert(atm,Pa) "air pressure"
t_charge=12 [hr]*convert(hr,s) "charge time"
eta_b=0.6 [-] "blower efficiency"
DELTAP_dh=0.5 [inH2O]*convert(inH2O,Pa) "deadhead pressure drop for blower"
V_dot_open=1800 [ft^3/min]*convert(ft^3/min,m^3/s) "open flow rate"

The volumetric flow rate ( Va ) and the temperature leaving the collector (Tr,in) are not known;
both values are assumed and these assumptions are relaxed based on subsequent calculations.

V_dot_cfm=1200 [ft^3/min] "assumed volumetric flow rate in bed, in cfm"


V_dot=V_dot_cfm*convert(ft^3/min,m^3/s) "volumetric flow rate in bed"
T_r_in=400 [K] "assumed value of hot inlet temperature"

The density and specific heat capacity of air (ρa and ca, respectively) are calculated using EES'
internal property routines. The mass flow rate of air is computed according to:

m a = Va ρ a (1)

The collector efficiency is computed using the equation given in the problem statement.

rho_a=density(Air,T=T_indoor,P=p_a) "density of air"


c_a=cP(Air,T=T_indoor) "specific heat capacity of air"
m_dot_a=rho_a*V_dot "mass flow rate of air"
eta_collector=0.75 [-] - 0.0015 [1/K]*(T_r_in-T_outdoor) "collector efficiency"

The assumed value of the collector exit temperature is commented out and an energy balance on
the collector is used to compute Tr,in:

{T_r_in=400 [K]} "assumed value of hot inlet temperature"


T_r_in=T_indoor+q``_solar*A_collector*eta_collector/(m_dot_a*c_a)
"predicted regenerator inlet temperature during charging process"

The correlations for friction factor and heat transfer coefficient for a packed bed of spheres are
accessed using the PackedSpheres procedure in EES, which returns the average friction factor,
average heat transfer, and the number of transfer units ( f , h , and NTU). The mass flux is
computed according to:

m a
G= (2)
φW 2
The characteristic radius of the flow channels is computed according to:

φ Dp
rchar = (3)
4 (1 − φ )

The pressure drop across the bed is computed according to:

G2 f L
Δp = (4)
2 ρ a rchar

call PackedSpheres('Air',m_dot_a, D_p, W^2, L, (T_r_in+T_indoor)/2, p_a: f_bar, h_bar, NTU)


"access correlations for packed spheres"
G=m_dot_a/(phi*W^2) "mass flux"
phi=0.35 [-] "
r_char=phi*D_p/(4*(1-phi)) "characteristic radius"
DELTAP=G^2*f_bar*L/(2*rho_a*r_char) "pressure drop across bed"
DELTAP_inH2O=DELTAP*convert(Pa,inH2O) "in inH2O"

The assumed value of the volumetric flow rate is commented out and the pressure drop is
calculated using the pressure/flow curve specified by the problem statement:

{V_dot_cfm=1200 [ft^3/min]} "assumed volumetric flow rate in bed, in cfm"


DELTAP=DELTAP_dh*(1-V_dot/V_dot_open) "blower pressure rise"

The blower power is calculated according to:

ΔpV
w blower = (5)
ηb

w_dot_b=DELTAP*V_dot/eta_b "blower power"

These calculates lead to Tr,in = 389.6 K and m a = 0.519 kg/s.

b.) What is the amount of heat transfer from the rock bed to the air during the discharge process?

The volume of the regenerator material is:

Vr = W 2 L (1 − φ ) (6)

The capacitance of the matrix is:

Cmatrix = ρ r cr Vr (7)

The capacitance rate of the air is:


C a = m a ca (8)

The ε-NTU solution for a balanced, symmetric regenerator is accessed using the HX function in
order to compute the effectiveness of the heat exchanger (ε):

V_r=W^2*L*(1-phi) "volume of regenerator material"


C_dot=m_dot_a*c_a "capacitance rate of the air flow"
C_matrix=rho_r*c_r*V_r "capacitance of the matrix"
epsilon=HX('Regenerator', NTU, C_dot, C_matrix/t_charge, 'epsilon')
"effectiveness-NTU correlation for a regenerator"

The maximum possible amount of heat transfer is:

Qmax = m a ca (Tr ,in − Tindoor ) tcharge (9)

and the actual amount of heat transfer is:

Q = ε Qmax (10)

Q_max=m_dot_a*c_a*t_charge*(T_r_in-T_indoor) "maximum possible heat transfer"


Q=Q_max*epsilon "actual heat transfer in regenerator"

which leads to Q = 2.12x109 J.

c.) There are 100 heating days per year in this location. What is the total amount of heating
energy saved over a 10 year period?

The total amount of energy saved is:

Q tanalysis
Qsaved = (11)
2 tcharge

where tanalysis is the total amount of time considered in the analysis.

Nhday=100 [day] "number of heating days"


t_analysis=10 [year]*convert(year,s)*Nhday/365[day] "time of analysis"
Q_saved=Q*t_analysis/(2*t_charge) "total saved energy"

which leads to Qsaved = 2.12x1012 J.

d.) If the cost of natural gas is gc = 3.5$/therm then what is the total heating cost saved over a 10
year period (neglect the time value of money for this analyis)?

The heating cost avoided is:


ES = Qsaved gc (12)

gc=3.5 [$/therm]*convert($/therm,$/J) "cost of natural gas"


ES=Q_saved*gc "value of saved energy"

which leads to ES = $70,430.

e.) The cost of the solar collectors is cc = 45$/ft2 and the cost of the rock bed is rc = 40$/lbm.
The cost of the electrical energy required to run the blowers is ec = 0.12$/kW-hr. Determine
the net savings associated with owning the equipment over a 10 year period.

The cost to run the blowers is:

WC = w b ec tanalysis (13)

The cost of the collectors is:

ColC = Acollector cc (14)

The cost of the rock bed is:

BC = LW 2 (1 − φ ) ρ r rc (15)

The net saving is therefore:

LCS = ES − WC − BC − ColC (16)

ec=0.12 [$/kW-hr]*convert($/kW-hr,$/J) "cost of electricity"


cc=45 [$/ft^2]*convert($/ft^2,$/m^2) "collector cost"
rc=40 [$/ton]*convert($/ton,$/kg) "cost of rocks"
WC=w_dot_b*t_analysis*ec "cost of electricity to run blowers"
BC=L*W^2*(1-phi)*rho_r*rc "cost of rock bed"
ColC=A_collector*cc "cost of collectors"
LCS=ES-BC-ColC-WC "life-cycle savings associated system"

which leads to LCS = $10,100.

f.) Plot the net savings as a function of the number of solar collectors. You should see that an
optimal number of collectors exists. Explain this fact.

Figure P8.10-1(b) illustrates the net savings as a function of the number of solar collectors. The
optimal value exists because the collector efficiency decreases with the number of solar
collectors because the exit temperature increases.
12000

10000

Net savings ($)


8000

6000

4000

2000

0
0 20 40 60 80
Number of collectors
Figure P8.10-1(b): Net savings as a function of the number of collectors.

g.) Plot the net savings as a function of the length of the rock bed (with Ncol = 40). You should
see that an optimal length of the rock bed. Explain this fact.

Figure P8.10-1(c) illustrates the net savings as a function of the length of the rock bed. The
optimal value exists because the rock bed heat capacity increases to the point where it becomes
larger than the capacity of the air and thus its effectiveness approaches unity (the NTU is always
high for any reasonable value of length). As the length increases further, the blower power
increases due to increased pressure drop.
14000

12000

10000
Net savings ($)

8000

6000

4000

2000

0
0 20 40 60
Rock bed length (ft)
Figure P8.10-1(c): Net savings as a function of the rock bed length.

h.) Determine the optimal number of collectors and rock bed length.

Using the Min/Max function from the Calculate menu, the optimal number of collectors and
length is Ncol = 41 and L = 13.5 ft which leads to a net savings of $11,780.
Problem 9.1-1 (9-1 in text)

A mixture is formed mixing Mm = 0.25 kg of methane (with molar mass MWm = 16 kg/kgmol),
Me = 0.15 kg of ethane (MWe = 30 kg/kgmol) and Mn = 0.1 kg of nitrogen (MWn = 28 kg/kgmol).
The mixture is placed in a container that is maintained at T = 25°C and p = 5 bar. At these
conditions, the mixture behaves in accordance with the ideal gas law. Determine:
a.) the volume of the mixture

The known information is entered into EES. Array variables are employed to facilitate the
calculations.

$UnitSystem SI MASS RAD PA K J


$Tabstops 0.2 0.4 0.6 3.5 in

"known information"
NGases=3 [-] "number of gases in the mixture"
N$[1]='methane'; m[1]=0.25 [kg] "name and mass of gas #1, methane"
N$[2]='ethane'; m[2]=0.15 [kg] "name and mass of gas #2, methane"
N$[3]='nitrogen'; m[3]=0.1 [kg] "name and mass of gas #3, methane"
P=5 [bar]*convert(bar,Pa) "pressure"
T=converttemp(C,K,25 [C]) "temperature"

The molarmass of each constituent (MWi) is obtained using the MolarMass function. The number
of moles of each constituent is:

Mi
Ni = for i = 1..N gas (1)
MWi

where Ngas is the number of constituents. The total number of moles is:

N gas

N = ∑ Ni (2)
i =1

and the total volume is obtained using the ideal gas law:

PV = Ru N T (3)

where Ru is the universal gas constant.

"a) determine the volume of the mixture"


duplicate i=1,NGases
MW[i]=molarMass(N$[i]) "molecular weight of each gas"
N[i]=m[i]/MW[i] "number of moles of each gas"
end
N_tot=sum(N[1..NGases]) "total number of moles"
P*V=N_tot*R#*T "ideal gas law to determine volume"

which leads to V = 0.120 m3.


b.) the equivalent molecular weight of the mixture

The total mass of the mixture is:

N gas

M = ∑ Mi (4)
i =1

and the equivalent molar mass is:

M
MW = (5)
N

"b) density on mass basis"


M_tot=sum(m[1..NGases]) "total mass of the mixture"
MW=M_tot/N_tot "equivalent molar mass"

which leads to MW = 20.71 kg/kgmol.

c.) the density of the mixture on a mass basis

The density of the mixture on a mass basis is:

M
ρ= (6)
V

"c) density on molar basis"


rho=M_tot/V "mass density"

where ρ = 4.18 kg/m3.

d.) the density of the mixture on a molar basis

The density of the mixture on a molar basis is:

N
n= (7)
V

"d) density on molar basis"


molardensity=N_tot/V "molar density"

which leads to n = 0.202 kgmol/m3.

e.) the mass fractions of each species

The mass fractions of each species are computed according to:


Mi
mf i = for i = 1..N gas (8)
M

"e) mass fraction"


duplicate i=1,NGases
mf[i]=m[i]/M_tot "mass fraction"
end

which leads to mfm = 0.5, mfe = 0.3, and mfn = 0.2.

f.) the mole fractions of each species

The mole fractions of each species are computed according to:

Ni
yi = for i = 1..N gas (9)
N

"f) mole fraction"


duplicate i=1,NGases
y[i]=n[i]/N_tot "mole fraction"
end

which leads to ym = 0.646, ye = 0.207, and yn = 0.148.

g.) the mass concentrations of each species

The mass concentration of each species are computed according to:

Mi
ci = for i = 1..N gas (10)
V

"g) mass concentration"


duplicate i=1,NGases
c[i]=m[i]/V "mass concentration"
end

which leads to cm = 2.089 kg/m3, ce = 1.253 kg/m3, and cn = 0.836 kg/m3.

h.) the molar concentration of each species

The molar concentrations of each species are computed according to:

Ni
ni = (11)
V

"h) molar concentration"


duplicate i=1,NGases
nn[i]=N[i]/V "molar concentration"
end

which leads to nm = 0.130 kgmole/m3, ne = 0.0417 kgmol/m3, and nn = 0.0298 kgmol/m3.


Problem 9.1-2 (9-2 in text)
The composition of mixtures of air and water vapor are often reported in terms of the humidity
ratio. The humidity ratio, ω, is defined as the mass of water vapor per mass of dry air. The
humidity ratio is related to, but not exactly the same as the the mass fraction. In a particular
case, the humidity ratio is ω = 0.0078 at temperature T = 30°C and pressure p = 101.3 kPa.
Determine:
a.) the mass fraction of the water vapor

The inputs are entered in EES:

$UnitSystem SI MASS RAD PA K J


$Tabstops 0.2 0.4 0.6 3.5 in

omega=0.0078 [-] "humidity ratio"


T=converttemp(C,K,30 [C]) "temperature"
P=101.3 [kPa]*convert(kPa,Pa) "pressure"

The problem is carried out using an arbitrary mass of dry air (ma). The mass of vapor is
computed using the definition of the humidity ratio:

mv
ω= (1)
ma

The mass fraction of water vapor is:

mv
mf v = (2)
ma + mv

m_a=1[kg] "arbitrary mass of dry air"


omega=m_v/m_a "definition of humidity ratio"
mf_v=m_v/(m_a+m_v) "mass fraction of vapor"

which leads to mfv = 0.00774.

b.) the mole fraction of the water vapor

The number of moles of air and water vapor are computed according to:

mv
Nv = (3)
MWv

ma
Na = (4)
MWa

where MWv and MWa are the molar weights of the water vapor and air, respectively, obtained
using the MolarMass function in EES. The mole fraction of water vapor is:
Nv
yv = (5)
Nv + Na

n_v=m_v/molarMass(Water) "moles of water vapor"


n_a=m_a/molarmass(air) "moles of air"
y_v=n_v/(n_a+n_v) "mole fraction of water vapor"

which leads to yv = 0.0124.

c.) the mass concentration of the water vapor

The total volume of the mixture is obtained from the ideal gas law:

PV = Ru ( N v + N a ) T (6)

where Ru is the universal gas constant. The mass concentration of water vapor is:

mv
cv = (7)
V

P*V=(n_a+n_v)*R#*T "total volume"


c_v=m_v/V "mass concentration of water vapor"

which leads to cv = 0.00897 kg/m3.

d.) the molar concentration of the water vapor

The molar concentration of the water vapor is:

Nv
nv = (8)
V

nn_v=n_v/V "molar concentration of water vapor"

which leads to nv = 0.000498 kgmol/m3.

e.) the maximum possible value for the mole fraction of the water vapor at equilibrium.

The maximum possible mole fraction is obtained from:

yv ,max P = Pv , sat (9)

where Pv,sat is the saturation pressure of the water vapor at temperature T, obtained using the EES
function P_sat.

y_max*P=P_sat(Water,T=T) "maximum mole fraction"

which leads to yv,max = 0.042.


Problem 9.2-1 (9-3 in text): Moisture transfer rates
The air-conditioning load for a building can be broken into latent and sensible
contributions. The latent represents the energy that must be expended to remove the
water vapor from the building. Water vapor enters by infiltration as air from outdoors
leaks inside and by diffusion through the walls and ceiling. The building in question is
rectangular with outer dimensions of 40 ft by 60 ft with 8 ft ceilings. The infiltration
rate is estimated at 0.65 air changes per hour. The diffusion coefficient for water
through 3/8 inch gypsum board (without a vapor barrier) is approximately 4.5x10-5 ft2/s at
atmospheric pressure.
a.) Estimate and compare the rates of moisture transfer by infiltration and diffusion on a day in
which the outdoor conditions are 95°F and 45% relative humidity and indoor conditions are
75°F, 40% relative humidity. Is the contribution by diffusion significant? If not, then why
are people concerned with water vapor diffusion in a building?

The known information is entered in EES:

$UnitSystem SI MASS RAD PA K J


$Tabstops 0.2 0.4 0.6 3.5 in

"known information"
H=8 [ft]*convert(ft,m) "height of ceiling"
Width=40 [ft]*convert(ft,m) "width of building"
Length=60 [ft]*convert(ft,m) "length of building"
L=(3/8) [in]*convert(in,m) "width of gypsum board"
D_wg=0.000045 [ft^2/s]*convert(ft^2/s,m^2/s) "diffusion coefficient"
T_out=convertTemp(F,K,95 [F]) "outdoor temperature"
rh_out=0.45 [-] "relative humidity outddoors"
T_in=convertTemp(F,K,75 [F]) "indoor temperature"
rh_in=0.40 [-] "relative humidity indoors"
AirChangeRate=0.65 [1/hr]*convert(1/hr,1/s) "air change rate"

The total area of the building exposed to diffusion is computed:

A = Walls + Ceiling (1)

where

Walls = 2 Width H + 2 Length H (2)

and

Ceiling = Width Length (3)

The volume of the building is:

Volume = Ceiling H (4)

Ceiling=Width*Length "area of ceiling"


Walls=2*Width*H+2*Length*H "area of walls"
A=Walls+Ceiling "total area"
Volume=Ceiling*H "volume of air in building"

The saturation pressure at the outdoor temperature (psat,out) is obtained using the P_sat function in
EES. The outdoor vapor pressure is:

pv ,out = psat ,out rhout (5)

The concentration of the water vapor in the outdoor air is:

pv ,out
cv ,out = (6)
Rv Tout

where Rv is the gas constant for the vapor.

P_sat_out=p_sat(water,T=T_out) "satuation pressure outdoors"


P_v_out=rh_out*P_sat_out "definition of relative humidty"
R_v=R#/MolarMass(Water) "gas constant"
c_out=P_v_out/(R_v*T_out) "concentration of water vapor in outdoor air from ideal gas law"

The saturation pressure at the indoor temperature (psat,in) is obtained using the P_sat function in
EES. The indoor vapor pressure is:

pv ,in = psat ,in rhin (7)

The concentration of the water vapor in the indoor air is:

pv ,in
cv ,in = (8)
Rv Tin

P_sat_in=p_sat(water,T=T_in) "satuation pressure indoors"


P_v_in=rh_in*P_sat_in "definition of relative humidty"
c_in=P_v_in/(R_v*T_in) "concentration of water vapor in indoor air from ideal gas law"

The mass flow rate of vapor due to diffusion through the walls is given by Fick's law:

A Dwg
m v =
L
(c
v , out − cv ,in ) (9)

The mass flow rate of vapor due to infiltration is:

m v ,int = AirchangerateVolume ( cv ,out − cv ,in ) (10)

m_dot_v=A*D_wg*(c_out-c_in)/L "Fick's law determines diffusion rate of water vapor"


m_dot_v_int=AirChangeRate*Volume*(c_out-c_in) "mass of vapor entering due to infiltration"
which leads to m v = 0.00149 kg/s and m v ,int = 0.000898 kg/s. It is interesting that the two modes
of moisture transport are nearly the same. Diffusion is an additional concern because it affects
the properties and life of materials. This is particularly a concern in winter in which water that
diffuses into insulation reduces its insulating capability.
Problem 9.2-2 (9-4 in text): Transport of Natural Gas
Natural gas (methane) is transported at 25ºC and 100 bar over long distances through 1.2 m
diameter pipelines at a velocity of 10 m/s. The pipeline is made of steel with a wall thickness of
2.0 cm. It has been suggested that hydrogen gas could be transported in these same pipelines.
However, hydrogen is a small molecule that diffuses through most materials. The diffusion
coefficient for hydrogen in steel is about 7.9x10-9 m2/s at 25ºC.
a.) Calculate the power transported by methane (assuming it will be combusted) through the
pipeline. The lower heating value of methane is 5.002x107 J/kg.

The input information is entered in EES:

$UnitSystem SI MASS RAD PA K J


$Tabstops 0.2 0.4 0.6 3.5 in

"known information"
T=convertTemp(C,K,25 [C]) "temperature"
P=100 [bar]*convert(bar,Pa) "pressure"
D=1.2 [m] "pipeline diameter"
th=2 [cm]*convert(cm,m) "pipe wall thickness"
u_ng=10 [m/s] "velocity of natural gas"
LHV_ng=5.002e7 [J/kg] "lower heating value of methane"
D_h_steel=7.9e-9 [m^2/s] "diffusion coefficient for hydrogen through steel"

The cross-sectional area of the pipe is:

D2
Ac = π (1)
4

The density of methane (ρng) is computed and the mass flow rate of methane is:

m ng = Ac ρ ng ung (2)

and the power associated with this mass flow rate is:

Power = m ng LHVng (3)

A_c=pi*(D/2)^2 "cross-sectional area of pipeline"


rho_ng=density('Methane',T=T,P=P) "density of methane"
m_dot_ng=A_c*u_ng*rho_ng "mass flow rate of methane"
Power=LHV_ng*m_dot_ng "power associated with methane flow"
Power_MW=Power*convert(W,MW) "in MW"

which leads to Power = 42990 MW.

b.) Estimate the velocity required to move the provide the same power if hydrogen rather than
methane is transported through the pipeline at the same temperature and pressure. The lower
heating value of hydrogen is 1.200x108 J/kg.
The mass flow rate of hydrogen required is:

Power = m h LHVh (4)

The density of hydrogen (ρh) is determined and used to compute the velocity according to:

m h = Ac ρ h uh (5)

LHV_h=1.2e8 [J/kg] "lower heating value of hydrogen"


Power=LHV_h*m_dot_h "mass flow rate of hydrogen"
rho_h=density('Hydrogen',T=T,P=P) "density of hydrogen"
m_dot_h=A_c*u_h*rho_h "velocity of hydrogen"

which leads to uh = 41.3 m/s.

c.) Compare the pumping power required to transport the natural gas and hydrogen a distance of
100 km.

The PipeFlow procedure is used to determine the pressure drop associated with the flow of
methane (Δpng). The minimum pump power is:

m ng
w ng = Δpng (6)
ρ ng

L=100 [km]*convert(km,m) "length of pipeline"


call PipeFlow('Methane',T,P,m_dot_ng,D,L,0 [-]:h_bar_ng_T, h_bar_ng_H ,DELTAP_ng,&
Nusselt_bar_ng_T, f_bar_ng, Re_ng)
Power_pump_ng=m_dot_ng*DELTAP_ng/rho_ng "minimum natural gas pumping power"
Power_pump_ng_MW=Power_pump_ng*convert(W,MW) "in MW"

which leads to w ng = 22.6 MW (0.05% of the total power flow). The PipeFlow procedure is used
to determine the pressure drop associated with the flow of hydrogen (Δph). The minimum pump
power is:

m h
w h = Δph (7)
ρh

call PipeFlow('Hydrogen',T,P,m_dot_h,D,L,0 [-]:h_bar_h_T, h_bar_h_H ,DELTAP_h, Nusselt_bar_h_T,&


f_bar_h, Re_h)
Power_pump_h=m_dot_h*DELTAP_h/rho_h "minimum hydrogen pumping power"
Power_pump_h_MW=Power_pump_h*convert(W,MW) "in MW"

which leads to w h = 171 MW (0.4% of the total power flow).

d.) Estimate the rate of hydrogen loss from a 100 km pipeline. Do you believe this loss is
significant?
The concentration of the hydrogen in the pipe (cpipe) is equal to its density. The concentration of
the hydrogen outside of the pipe (camb) is zero. Fick's law is used to compute the rate of
hydrogen diffusion.

m h ,dif = Dh , steel π D L
(c pipe − camb )
(8)
th

c_pipe=rho_h "concentration (density) of hydrogen in the pipe"


c_amb=0 [kg/m^3] "no hydrogen outside of the pipe"
m_dot_h_dif=D_h_steel*pi*D*L*(c_pipe-c_amb)/th "Fick's Law"

which leads to m h ,dif = 1.141 kg/s (0.3% of the flow).


Problem 9.2-3 (9-5 in text): Helium Balloon
A balloon made of a synthetic rubber is inflated with helium to a pressure of pini = 130 kPa at
which point its diameter is Dini = 0.12 m. The mass of the balloon material is Mbal = 0.53 g and
its thickness is δ = 0.085 mm. The balloon is released in a room that is maintained at T = 25ºC
filled with air (yN2 = 79% nitrogen and yO2 = 21% oxygen) at patm = 100 kPa. Over a period of
time, helium diffuses out of the balloon and oxygen and nitrogen diffuse in. The pressure in the
balloon above atmospheric pressure is linearly proportional to the balloon volume. The diffusion
coefficients for helium, oxygen and nitrogen through the synthetic rubber are DHe,rubber = 60x10-8,
DO2,rubber = 16x10-8, and DN2,rubber = 15x10-8 cm2/s, respectively.
a.) Prepare a numerical model of the balloon deflation process. Plot the volume and pressure
within the balloon as a function of time. Plot the mole fraction of helium, oxygen, and
nitrogen in the balloon as a function of time.

The inputs are entered in EES:

$UnitSystem SI MASS RAD PA K J


$Tabstops 0.2 0.4 0.6 3.5 in

"known information"
T=converttemp(C,K,25 [C]) "temperature of balloon and air"
P_atm=100 [kPa]*convert(kPa,Pa) "atmospheric pressure"
P_ini=130 [kPa]*convert(kPa,Pa) "initial balloon pressure"
D_ini=0.12 [m] "initial balloon diameter"
D_He_rubber=60E-8 [cm^2/s]*convert(cm^2/s,m^2/s) "diffusion coefficient for helium in rubber"
D_O2_rubber=16E-8 [cm^2/s]*convert(cm^2/s,m^2/s) "diffusion coefficient for oxygen in rubber"
D_N2_rubber=15E-8 [cm^2/s]*convert(cm^2/s,m^2/s) "diffusion coefficient for nitrogen in rubber"
delta=0.085 [mm]*convert(mm,m) "balloon thickness"
m_bal=0.53 [g]*convert(g,kg) "mass of balloon material"
y_N2_atm=0.79 [-] "mole fraction of nitrogen in the atmosphere"
y_O2_atm=0.79 [-] "mole fraction of oxygen in the atmosphere"

The initial volume of the balloon is:

3
4 ⎛D ⎞
Vini = π ⎜ ini ⎟ (1)
3 ⎝ 2 ⎠

The constant of proportionality that relates the volume to the pressure elevation above ambient
pressure is determined:

pini − patm = K Vini (2)

V_ini=4/3*pi*(D_ini/2)^3 "initial volume of balloon"


P_ini-P_atm=K*V_ini "spring constant of balloon"

The number of moles of oxygen and nitrogen initially in the balloon (NO2,ini and NN2,ini) are zero.
The number of moles of helium initially in the balloon is computed from the ideal gas law:
pini Vini
N He ,ini = (3)
Ru T

N_O2_ini=0 [kgmole] "no oxygen initially in balloon"


N_N2_ini=0 [kgmole] "no nitrogen initially in balloon"
N_He_ini=P_ini*V_ini/(R#*T) "helium initially fills balloon"

The molar concentration of helium in the atmosphere (nHe,atm) is zero. The molar concentration
of oxygen and nitrogen in the atmosphere is obtained from:

patm
nO 2,atm = yO 2,atm (4)
Ru T

patm
nN 2,atm = yN 2, atm (5)
Ru T

nn_He_atm=0 [kgmol/m^3] "molar concentration of helium in the atmosphere"


nn_O2_atm=y_O2_atm*P_atm/(R#*T) "molar concentration of oxygen in atmosphere"
nn_N2_atm=y_N2_atm*P_atm/(R#*T) "molar concentration of nitrogen in atmosphere"

The problem proceeds by implementing state equations that will determine the time rate of
change of the number of moles of oxygen, nitrogen, and helium in the balloon (NO2, NN2, and
NHe, respectively). In order to develop these state equations, it is best to assume arbitrary values
of these state variables.

"arbitrary state variables"


N_O2=N_O2_ini "number of moles of oxygen"
N_N2=N_N2_ini "number of moles of nitrogen"
N_He=N_He_ini "number of moles of helium"
time=0 [s] "time"

The total number of moles in the balloon is:

N tot = N O 2 + N N 2 + N He (6)

The pressure and volume within the balloon are related by the ideal gas constant:

pV
N tot = (7)
Ru T

and the balloon constant:

p = patm + K V (8)

"state equations"
N_tot=N_O2+N_N2+N_He "total number of moles"
P*V=N_tot*R#*T "ideal gas law"
P=P_atm+K*V "pressure"

The mole fraction of oxygen, nitrogen, and helium in the balloon are obtained from:

NO 2
yO 2 = (9)
N tot

NN 2
yN 2 = (10)
N tot

N He
yHe = (11)
N tot

y_O2=N_O2/N_tot "mole fraction of oxygen"


y_N2=N_N2/N_tot "mole fraction of nitrogen"
y_He=N_He/N_tot "mole fraction of helium"

The molar concentration of oxygen, nitrogen, and helium in the balloon are calculated from:

p
nO 2 = yO 2 (12)
Ru T

p
nN 2 = y N 2 (13)
Ru T

p
nHe = yHe (14)
Ru T

nn_O2=y_O2*P/(R#*T) "molar concentration of oxygen in the balloon"


nn_N2=y_N2*P/(R#*T) "molar concentration of oxygen in the balloon"
nn_He=y_He*P/(R#*T) "molar concentration of helium in the balloon"

The diameter of the balloon is obtained from:

3
4 ⎛D⎞
V = π⎜ ⎟ (15)
3 ⎝2⎠

and the area for diffusion is:

A = 4π D2 (16)

4*pi*(D/2)^3/3=V "diameter of balloon"


A=pi*D^2 "area of balloon"

The molar flow rates due to diffusion of oxygen and nitrogen into the balloon are obtained from
Fick's law:

A DO 2, rubber
nO 2 =
δ
(nO 2, atm − nO 2 ) (17)

A DN 2, rubber
n N 2 =
δ
(n N 2, atm − nN 2 ) (18)

The molar flow rate of helium diffusing out of the balloon is:

A DHe ,rubber
n He =
δ
(n He − nHe, atm ) (19)

n_dot_O2=A*D_O2_rubber*(nn_O2_atm-nn_O2)/delta "molar flow rate of oxygen into balloon"


n_dot_N2=A*D_N2_rubber*(nn_N2_atm-nn_N2)/delta "molar flow rate of nitrogen into balloon"
n_dot_He=A*D_He_rubber*(nn_He-nn_He_atm)/delta "molar flow rate of helium leaving balloon"

Mole balances on each constituent within the balloon leads to the state equations:

dN O 2
= nO 2 (20)
dt

dN N 2
= n N 2 (21)
dt

dN He
= − nHe (22)
dt

dNO2dt=n_dot_O2 "mass balance on the oxygen"


dNN2dt=n_dot_O2 "mass balance on the nitrogen"
dNHedt=-n_dot_He "mass balance on the helium"

With the state equations successfully implemented it is possible to use the Integral command to
numerically integrate them through time. The arbitrary values of the state variables are
commented out and the state equations are integrated:

{"arbitrary state variables"


N_O2=N_O2_ini "number of moles of oxygen"
N_N2=N_N2_ini "number of moles of nitrogen"
N_He=N_He_ini "number of moles of helium"
time=0 [s] "time"}
time_sim=48 [hr]*convert(hr,s) "time to simulate"
N_O2=N_O2_ini+Integral(dNO2dt,time,0,time_sim) "number of moles of oxygen"
N_N2=N_N2_ini+Integral(dNN2dt,time,0,time_sim) "number of moles of nitrogen"
N_He=N_He_ini+Integral(dNHedt,time,0,time_sim) "number of moles of helium"
time_hr=time*convert(s,hr) "time in hr"

$IntegralTable time:100 time_hr,V, P, D,y_O2,y_N2,y_He

The balloon volume and pressure as a function of time are shown in Figure 1.
0.001 130000

125000

0.0008 120000

115000
Volume (m )
3

Pressure (Pa)
0.0006 110000
P 105000

0.0004 100000

95000

0.0002 V 90000

85000

0 80000
0 5 10 15 20 25 30 35 40 45 50
Time (hr)
Figure 1: Balloon volume and pressure as a function of time.

Figure 2 illustrates the mole fractions of nitrogen, oxygen, and helium in the balloon as a
function of time.
1

0.9

0.8

0.7 yHe
Mole fraction

0.6

0.5 yO2 and yN2


0.4

0.3

0.2

0.1

0
0 5 10 15 20 25 30 35 40 45 50
Time (hr)
Figure 2: Mole fractions of oxygen, nitrogen, and helium in the balloon as a function of time.

b.) At what time does the balloon lose its buoyancy?

The molar mass of the mixture in the balloon is computed according to:
MW = yHe MWHe + yO 2 MWO 2 + yN 2 MWN 2 (23)

where MWHe, MWN2, and MWO2 are the molar mass of helium, oxygen, and nitrogen,
respectively, obtained using the MolarMass function in EES. The density of the gas in the balloon
is:

P MW
ρ= (24)
Ru T

The density of the air in the atmosphere (ρatm) is obtained from EES' internal property function.
The buoyancy force is:

Fb = V ( ρ atm − ρ ) g (25)

The net upward force on the balloon is therefore:

F = Fb − M bal g (26)

MW=y_He*molarMass(He)+y_O2*molarMass(O2)+y_N2*molarMass(N2) "molar mass of gas mixture"


rho=P*MW/(R#*T) "density of gas mixture"
rho_atm=density(Air,T=T,P=P_atm) "density of atmospheric air"
F_b=V*(rho_atm-rho)*g# "buoyancy force"
F=F_b-M_bal*g# "net force against gravity"

$IntegralTable time:100 time_hr,V, P, D,y_O2,y_N2,y_He, F

Figure 3 illustrates the net upward force on the balloon as a function of time and shows that the
balloon loses its buoyancy after approximately 3.5 hours.
0.004
Net upward force on balloon (N)

0.002

-0.002

-0.004

-0.006
0 5 10 15 20 25 30 35 40 45 50
Time (hr)
Figure 3: Net upward force on balloon as a function of time.
Problem 9.3-1 (9-6 in text)
A janitor is about to clean a large window at one end of a corridor with an ammonia-water
solution. The corridor is 2.5 m high, 2 m wide and 3 m in length. The conditions in the corridor
are 25C, 101 kPa. The concentration of the ammonia that evaporates from the window is
estimated to be 100 ppm. Many humans can detect ammonia by smell at levels of 1 ppm.
Estimate the time required for a person standing at the other end of the corridor to detect the
ammonia after the janitor starts to wash the window.

The known information is entered in EES:

$UnitSystem SI MASS RAD PA K J


$Tabstops 0.2 0.4 0.6 3.5 in

"known information"
T=convertTemp(C,K,25 [C]) "temperature"
P=101 [kPa]*convert(kPa,Pa) "pressure"
H=2.5 [m] "height of corridor"
W=2 [m] "width of corridor"
L=3 [m] "length of corridor"
y_NH3_0=100e-6 [-] "mole fraction of ammonia at source"
y_NH3_L=1e-6 [-] "mole fraction at the other end of corridor when detected"

The gas constant for ammonia is:

Ru
RNH 3 = (1)
MWNH 3

where Ru is the universal gas constant and MWNH3 is the molecular weight, obtained using the
MolarMass function in EES. The mass concentration of ammonia at the source (x = 0) and the
detector (x = L) are computed according to:

P
cNH 3, x =0 = y NH 3, x =0 (2)
RNH 3 T

P
cNH 3, x = L = yNH 3, x = L (3)
RNH 3 T

R_NH3=R#/MolarMass('Ammonia') "density of mixture is essentially the density of air"


c_NH3_0=y_NH3_0*P/(R_NH3*T) "concentration of ammonia at source"
c_NH3_L=y_NH3_L*P/(R_NH3*T) "concentration of ammonia at end of corridor"

The diffusion coefficient for ammonia in air (DNH3,air) is estimated using the D_12_gas function
in EES. The solution to a semi-infinite body exposed to a step change in temperature (or, in this
case, concentration) at one surface is implemented in the SemiInf1 function in EES and used to
obtain the time required for the detector to register the presence of ammonia.

G1$='ammonia'
G2$='Air'
D_NH3_air=D_12_gas(G1$,G2$,T,P) "estimate of the binary diffusion coefficient"
c_NH3_L=SemiInf1(0,c_NH3_0,D_NH3_air,L,time)
"use the planewall transient library with an analogy between temperature and concentration"
time_hr=convert(s,hr)*time "time in hours for the person to detect the ammonia"

which leads to t = 8.61 hr.


Problem 9.4-1
A cylindrical tank having an internal diameter of Dt = 0.42 m and an internal height of Ht = 1.4
m was originally used to store hot water, but is no longer used. A custodian cut the two pipes
that were used to charge and discharge the tank leaving L = 0.12 m of pipe sticking up from the
top of the tank. The internal diameter of the pipes is Dp = 2.2 cm. The water remaining in the
tank is at room temperature, T = 25°C. The building in which the tank is located is at
atmospheric pressure and it is maintained at an average relative humidity of 40%.
a.) What is the rate of moisture transport to the building?

The known information is entered in EES:

$UnitSystem SI MASS RAD PA K J


$Tabstops 0.2 0.4 0.6 3.5 in
"known information"
D_t=0.42 [m] "diameter of tank"
H_t=1.4 [m] "height of tank"
D_p=2.2 [cm]*convert(cm,m) "diameter of tubing"
L=0.12 [m] "height of piping"
T=converttemp(C,K,25 [C]) "temperature"
rh=0.4 [-] "relative humidity of air"
P=101.3 [kPa]*convert(kPa,Pa) "atmospheric pressure"

The diffusion coefficient for water in air is estimated using the correlation provided by Bolz and
Tuve (1976) and presented in EXAMPLE 9.2-1:

⎡ m2 ⎤ ⎡ m2 ⎤ -10 ⎡ m
2
⎤ 2
Da , w = −2.775x10-6 ⎢ ⎥ + 4.479x10-8 ⎢ ⎥ T + 1.656x10 ⎢ 2⎥
T (1)
⎣ ⎦
s ⎣ s-K ⎦ ⎣ s-K ⎦
The density of the air-water mixture (ρ) and the humidity ratio of the air in the room (ω∞) and
the saturated air in the tank (ωsat) are obtained using EES' psychrometric functions. The mass
fraction of the air in the room and in the tank are calculated according to:

ω∞
mf ∞ = (2)
1 + ω∞

ωsat
mf sat = (3)
1 + ωsat

D_w_a=-2.775e-6 [m^2/s]+4.479e-8 [m^2/s-K]*T+1.656e-10[m^2/s-K^2]*T^2


"diffusion coefficient for air-water mixture, according to Bolz and Tuve (1976)"
rho=density(AirH2O,T=T,P=P,r=rh) "density of air"
omega_infinity=humRat(AirH2O,T=T,P=P,r=rh) "humidity ratio of water vapor in room"
omega_sat=humRat(AirH2O,T=T,P=P,r=1) "humidity ratio of saturated water vapor in tank"
mf_infinity=omega_infinity/(1+omega_infinity) "mass fraction of water vapor in room"
mf_sat=omega_sat/(1+omega_sat) "mass fraction of saturated water vapor in tank"

The cross-sectional area of the pipes is:


D p2
Ac = 2 π (4)
4

and the mass flow rate of water through the pipes is computed using:

ρ Da , w ⎡ 1 − mf ∞ ⎤
m w = ln ⎢ ⎥ (5)
L ⎣1 − mf sat ⎦

A_c=2*pi*D_p^2/4 "cross-sectional area of tubes"


m_dot_w=rho*D_w_a*A_c*ln((1-mf_infinity)/(1-mf_sat))/L "mass flow rate of water from tank to room"

which leads to m w = 2.26x10-9 kg/s.

b.) How much time is required for the level in the tank to drop by 1.0 m?

The time required for the level in the tank to change by ΔH = 1 m is:

Dt2
m w time = π ΔH ρw (6)
4

where ρw is the density of liquid water.

DH = 1 [m] "change in level of water in tank"


m_dot_w*time=pi*D_t^2*DH*density(Water,T=T,P=P)/4 "time required for level to change"
time_yr=time*convert(s,year) "in year"

which leads to time = 1942 yr. It will take a long time for the tank to empty through diffusion.
Problem 9.4-2 (9-7 in text)
In order to detect chemical threats that are being smuggled into the country within a shipping
container, the government is working on a system that samples the air inside the container on the
dock as it is being unloaded. The chance of detecting the chemical threat is strongly dependent
upon its concentration distribution at the time that the container is sampled. Therefore, you have
been asked to prepare a simple model of the migration of the threat species from its release point
within a passage formed by the space between two adjacent boxes. The problem is not a simple
diffusion problem because the threat chemical is adsorbed onto the walls of the passage. The
situation is simplified as 1-D diffusion through a duct. One end of the duct is exposed to a
constant concentration of the threat chemical that is equal to its saturation concentration, csat =
0.026 kg/m3. The duct is filled with clean air and the walls of the duct are clean (i.e., at time t =
0 there is no threat chemicals either in the air in the duct or on the walls of the duct). The
hydraulic diameter of the duct is Dh = 10 cm. The length of the duct is infinite. The diffusion
coefficient for the threat chemical in air is D = 2.2x10-5 m2/s. The mass of threat chemical per
unit area adsorbed on the wall of the container ( M w′′ ) is related to the concentration of the
chemical in the air (c) according to:
c
A
M w′′ csat
=
M w′′,m ⎛ c ⎞⎡ c ⎤
⎜1 − ⎟ ⎢1 + ( A − 1)
⎝ csat ⎠ ⎣ csat ⎦⎥
where M w′′,m = 4x10-4 kg/m2 is the mass per unit area associated with a single monolayer and A =
20 is a dimensionless constant. The total time available for diffusion between loading the
container and unloading is ttransit = 14 days. Because the length of the duct is so much larger than
the hydraulic diameter of the duct, it is reasonable to assume that the concentration distribution is
1-D. Further, because the concentration of the threat chemical is so small, it is reasonable to
neglect any bulk velocity induced by the diffusion process; that is, only mass transfer by
diffusion is considered.
a.) Prepare a 1-D transient model of the diffusion process using the ode45 solver in MATLAB.

The inputs are entered in a MATLAB script, P9p4d2:

clear all;
D=2.2e-5; % diffusion coefficient (m^2/s)
A=20; % constant for isotherm equation (-)
L=20; % length of duct (m)
D_h=0.1; % hydraulic diameter of duct (m)
c_sat=0.026; % saturation concentration (kg/m^3)
Mwm=4e-4; % mass per unit area of a monolayer (kg/m^2)
t_transit=14*24*60*60; % simulation time (s)

The computational domain is divided into N nodes. The position of each node is given by:

xi =
( i − 1) L for i = 1..N (1)
( N − 1)
where L is the duct length, selected to be sufficiently large that it does not impact the results (L =
20 m is sufficiently large, as seen in the results).

N=101; % number of length steps to use


for i=1:N
x(i)=(i-1)*L/(N-1);
end

The distance between adjacent nodes is:

L
Δx = (2)
( N − 1)
The initial concentration distribution is given by:

cini ,1 = csat (3)

cini ,i = 0 for i = 2..N (4)

% initial concentration
c_ini(1)=c_sat;
for i=2:N
c_ini(i)=0;
end

A state equation is derived for each node. The concentration at the duct inlet and exit do not
change; therefore:

dc1
=0 (5)
dt

and

dcN
=0 (6)
dt

The state equations for the internal nodes are obtained based on a mass balance. The diffusion of
mass from the two adjacent nodes ( m LHS and m LHS ) is balanced by mass storage both in the
volume of the control volume as well as due to adsorption on the walls of the control volume:

dci dM w′′
m LHS + m RHS = Ac Δx + per Δx (7)
dt dt

where Ac and per are the cross-sectional area and perimeter of the duct, respectively. The
diffusion mass transfers are written according to Fick's law:
m LHS = D
( ci −1 − ci ) A (8)
Δx
c

m RHS = D
( ci +1 − ci ) A (9)
Δx
c

Substituting Eqs. (8) and (9) into Eq. (7) leads to:

D
( ci −1 − ci ) A +D
( ci +1 − ci ) A = Ac Δx
dci
+ per Δx
dM w′′
(10)
Δx Δx
c c
dt dt

The rate of change of the mass per unit area on the walls of the duct is related to the rate of
change of concentration:

D
( ci −1 − ci ) A +D
( ci +1 − ci ) A = Ac Δx
dci
+ per Δx
dM w′′ dci
(11)
Δx Δx
c c
dt dc dt

Equation (11) is solved for the rate of change of concentration:

dci
=D
( ci −1 + ci +1 − 2 ci ) for i = 2.. ( N − 1) (12)
dt ⎛ 4 dM w′′ ⎞
Δx ⎜ 1 +
2

⎝ Dh dc ⎠

The derivative of Eq. Error! Reference source not found. is obtained using Maple:

> Mpa:=Mpam*A*(c/c_sat)/((1-c/c_sat)*(1+(A-1)*c/c_sat));
Mpam A c
Mpa :=
(A − 1) c ⎞
c_sat ⎛⎜⎜ 1 −
c ⎞⎛
⎟⎟ ⎜⎜ 1 + ⎟
⎝ c_sat ⎠ ⎝ c_sat ⎟⎠
> dMpadc:=diff(Mpa,c);
Mpam A Mpam A c
dMpadc := +
(A − 1) c ⎞
c_sat ⎛⎜⎜ 1 −
c ⎞⎛ 2
⎟⎟ ⎜⎜ 1 + ⎟⎟ c_sat 2 ⎛⎜ 1 − c ⎟⎞ ⎛⎜ 1 + ( A − 1 ) c ⎟⎞
⎝ c_sat ⎠ ⎝ c_sat ⎠ ⎜ c_sat ⎟⎠ ⎜⎝ c_sat ⎟⎠

Mpam A c ( A − 1 )
− 2
2⎛ c ⎞⎛ (A − 1) c ⎞
c_sat ⎜⎜ 1 − ⎟ ⎜1 + ⎟
⎝ c_sat ⎟⎠ ⎜⎝ c_sat ⎟⎠
> simplify(%);
Mpam A c_sat ( c_sat 2 + c 2 A − c 2 )
( −c_sat + c ) 2 ( c_sat + c A − c ) 2

A function dcdtP9p4d2 is generated to compute the state equations:


function[dcdt]=dcdtP9p4d2(time,c,D,L,D_h,Mwm,c_sat,A)

% Outputs
% dcdt - vector of the time rate of change of concentrations (kg/m^3-s)
%
% Inputs
% time - time (s)
% c - vector of concentrations (kg/m^3)
% D - diffusion coefficient (m^2/s)
% L - length of duct (m)
% D_h - hydraulic diameter (m)
% Mwm - mass per unit area associated with a monolayer (kg/m^2)
% c_sat - saturation concentration (kg/m^3)
% A - constant in the isotherm equation (-)

[N,g]=size(c);
Dx=L/(N-1);
dcdt=zeros(N,1);
dcdt(1)=0;
for i=2:(N-1)
dcdt(i)=D*(c(i+1)+c(i-1)-
2*c(i))/(Dx^2*(1+4*dMwdc(c(i),Mwm,c_sat,A)/D_h));
end
dcdt(N)=0;
end

A sub-function, dMwdc, is generated to return the derivative of the mass per unit area with
respect to concentration according to the Maple result.

function[dMwdc]=dMwdc(c,Mwm,c_sat,A)

% Outputs
% dMwdc - derivative of mass per area with respect to concentration
%
% Inputs
% c - concentration (kg/m^3)
% Mwm - mass per unit area associated with a monolayer (kg/m^2)
% c_sat - saturation concentration (kg/m^3)
% A - constant in the isotherm equation (-)
dMwdc=Mwm*A*c_sat*(c_sat^2+c^2*A-c^2)/(-c_sat+c)^2/(c_sat+c*A-c)^2;
end

The ode45 solver is used to integrate the state equations over the transit time.

OPTIONS=odeset('RelTol',1e-4);
t=linspace(0,t_transit,101);
[time,c]=ode45(@(time,c) dcdt(time,c,D,L,D_h,Mwm,c_sat,A),t,c_ini);

b.) Plot the concentration distribution within the passage at various times.

Figure P9.4-2(b) illustrates the concentration within the passage as a function of position at
various values of time.
0.03

0.025

Concentration (kg/m )
3
0.02 1 day
3 days
0.015 5 days
7 days
9 days
0.01
11 days
13 days
0.005

0
0 1 2 3 4 5 6 7 8 9 10
Position (m)
Figure P9.4-2(b): Concentration as a function of position at various values of time.

c.) Plot the concentration distribution within the passage at t = ttransit and overlay on this plot the
zero-adsorption solution to show how adsorption has retarded the migration of the threat
chemicals within the container.

Figure P9.4-2(c) illustrates the concentration of the threat vapor as a function of position at t =
ttransit. Also shown in Figure P9.4-2(c) is the concentration distribution obtained if the adsorption
is neglected (obtained by setting M w′′, m = 0). Note that the concentration penentration depth for
the zero-adsorption case is approximately consistent with 2 D t .

0.03

0.025
Concentration (kg/m )
3

0.02

without adsorption
0.015

0.01

0.005

with adsorption
0
0 2 4 6 8 10 12 14 16 18 20
Position (m)
Figure P9.4-2(c): Concentration as a function of position at t = ttransit. Also shown is the concentration at t =
ttransit if there is no adsorption.
Problem 9.4-3 (9-8 in text)
Naphthalene is an aromatic hydrocarbon with a molecular weight of MW = 128.2 kg/kgmol that
sublimes at a reasonable rate a room temperature. Naphthalene was commonly used for moth
balls, but is now considered to be a carcinogen. At T = 25°C, solid naphthalene has a density of
ρ = 1.16 g/cm3 and a vapor pressure of pv = 0.082 mm Hg. An engineer has recognized that heat
and mass transfer are analogous processes and he plans to estimate the heat transfer coefficient
for an unusual geometry by measuring how much mass of napthalene is sublimed. A review of
the literature indicates that the Schmidt number for naphthalene is Sc = 2.5. To test accuracy of
the heat /mass transfer analogy, the engineer first measures the mass of naphthalene that
sublimes from a sphere of D = 2.5 cm diameter when exposed to a stream of pure air at
temperature T = 25°C, pressure p = 101.3 kPa, and velocity u∞ = 10 m/s. The test is run for ttest =
2 hr and during this time the mass of the naphthalene sphere is reduced by Δm = 250 mg.
a.) Determine the error relative to accepted correlations for this geometry.

The input information is entered in EES:

$UnitSystem SI MASS RAD PA K J


$Tabstops 0.2 0.4 0.6 3.5 in

"known information"
D=2.50 [cm]*convert(cm,m) "diameter of sphere"
vel=10 [m/s] "velocity of air"
T=convertTemp(C,K, 25 [C]) "air temperature"
P=100 [kPa]*convert(kPa,Pa) "air pressure"
Sc=2.5 "Schmidt number"
DELTA_time=2 [hr]*convert(hr,s) "duration of experiment"
P_v=0.082 [mmHg]*convert(mmHg,Pa) "vapor pressure of napthalene at 25°C"
m_loss=250 [mg]*convert(mg,kg) "change in mass of the sphere"
MW_napthalene=128.16 [kg/kmol] "molar mass of napthalene"
rho_napthalene=1.16 [g/cm^3]*convert(g/cm^3,kg/m^3) "density of solid napthalene"

The mass flow rate of napthalene sublimation is:

Δm = m ttest (1)

The change in the diameter of the sphere is computed in order to ensure that the size of the
sphere is not altered appreciably during the test.

4 ⎡⎛ D ⎞ ⎛ D ⎞ ⎤
3 3

Δm = ρ π ⎢⎜ ⎟ − ⎜ − ΔD ⎟ ⎥ (2)
3 ⎢⎣⎝ 2 ⎠ ⎝ 2 ⎠ ⎥⎦

m_loss=m_dot*DELTA_time "mass loss assuming rate of mass is constant with time "
m_loss=rho_napthalene*4/3*pi*((D/2)^3-(D/2-DELTAd)^3)
"change in diameter over the duration of the experiment"

which leads to m = 3.47x10-8 kg/s and ΔD = 0.11 mm (0.4% of the initial diameter). The gas
constant for napthalene is obtained from:
Ru
R= (3)
MW

where Ru is the universal gas constant. The concentration of the napthalene at the surface of the
sphere is obtained from the ideal gas law:

pv
csat = (4)
RT

The concentration of napthalene in the free stream air (c∞) is zero. The mass transfer coefficient
is calculated according to:

m = hD π D 2 ( csat − c∞ ) (5)

R=R#/MW_napthalene "gas constant for napthalene"


c_sat=P_v/(R*T) "saturation concentration of napthalene at 25 C"
c_infinity=0 [kg/m^3] "concentration of napthalene is zero in free stream"
m_dot=h_bar_D*(4*pi*D^2/4)*(c_sat-c_infinity) "rate of mass loss"

which leads to hD = 0.0313 m/s. The diffusion coefficient for napthalene through air is obtained
using the Schmidt number:

μa
Sc = (6)
ρ a Dn,a

where μa and ρa are the viscosity and density, respectively, of the air. The Sherwood number is:

hD D
Sh = (7)
Dn ,a

mu=viscosity(Air,T=T) "air viscosity"


rho=density(Air,T=T,P=P) "air density"
Sc=mu/(rho*D_n_a) "Schmidt number"
Sh=(h_bar_D*D)/D_n_a "definition of Sherwood number"

which leads to Sh = 123.6.

The Sherwood number can also be computed using the correlations for heat transfer according to
the heat and mass transfer analogy:

if Nu = function of ( Re, Pr ) then Sh ≈ function of ( Re, Sc ) (8)

The Reynolds number is computed according to:


ρ a u∞ D
Re = (9)
μ

The External_Flow_Sphere_ND procedure in EES is used to access the correlation for the Nusselt
number in terms of the Reynolds number and Prandtl number; however, the Nusselt number is
replaced with the Sherwood number ( Sh c ) and the Prandtl number replaced with the Schmidt
number.

Re=rho*vel*D/mu "Reynold's number"


Call External_Flow_Sphere_ND(Re,Sc: Sh_c,C_d) "calculate Sherwood number with correlation"
error=(Sh_c-Sh)/Sh "error"

which leads to Sh c = 129.1, which is 4.4% in error relative to the measured value.
Problem 9.4-4 (9-9 in text)
Data for naphthalene at 25C are provided in problem 9.4-3. Determine the time required for
90% of the mass in a 1.0 cm sphere of naphthalene to sublime into an air stream at 25ºC and 100
kPa that is flowing at 5 m/s.

The input information is entered in EES:

$UnitSystem SI MASS RAD PA K J


$Tabstops 0.2 0.4 0.6 3.5 in

"known information"
T=298.15 [K] "temperature"
P_atm=101.3 [kPa]*convert(kPa,Pa) "atmospheric pressure"
P_naph=0.082 [torr]*convert(torr,Pa) "vapor pressure of naphthalene at 25 [C]"
rho=1.16 [g/cm^3]*convert(g/cm^3,kg/m^3) "density of naphthalene"
D_ini=1 [cm]*convert(cm,m) "initial diameter of napthalene sphere"
MW_naph=128.2 [kg/kmol] "molar mass of naphthalene"
Sc=2.5 [-] "Schmidt number for naphthalene"
Vel=5 [m/s] "frontal velocity of air"

The density and viscosity of the air (ρa and μa) are determined from the internal property
routines in EES and used to determine the diffusion coefficient based on the Schmidt number:

μa
Sc = (1)
ρ a Dn,a

The gas constant for napthalene is obtained from:

Ru
R= (2)
MW

where Ru is the universal gas constant. The concentration of napthalene at the surface of the
mothball is obtained from the ideal gas law:

pv
csat = (3)
RT

The concentration of napthalene in the free stream air (c∞) is zero.

rho_a=density(Air,T=T,P=P_atm) "density of air"


mu_a=viscosity(Air,T=T) "viscosity of air"
Sc=mu_a/(rho_a*D_n_a)
"definition of Schmidt number to determine diffusion coefficient of naphthalene in air"
R=R#/MW_Naph "gas constant for naphthalene"
c_s=P_naph/(R*T) "concentration of naphthalene at the surface"
c_infinity=0 [kg/m^3] "concentration of naphthalene in the free stream"
The problem will proceed by integrating the rate of change in the diameter of the mothball
through time; the state variables are therefore diameter (D) and time (t). Arbitrary values of the
state variables are assumed in order to implement the state equation:

"arbitrary state variables"


time=0 [s] "time"
D=D_ini "diameter"

The Reynolds number is computed according to:

ρ a u∞ D
Re = (4)
μa

The Sherwood number is computed using the correlations for heat transfer according to the heat
and mass transfer analogy:

if Nu = function of ( Re, Pr ) then Sh ≈ function of ( Re, Sc ) (5)

The correlation for the Nusselt number is accessed using the External_Flow_Sphere_ND function
in EES. The Nusselt number is replaced with the Sherwood number ( Sh c ) and the Prandtl
number replaced with the Schmidt number. The heat transfer coefficient is computed according
to:

hD D
Sh = (6)
Dn ,a

The mass flow rate of napthalene loss is:

m = hD π D 2 ( csat − c∞ ) (7)

A mass balance on the mothball provides the rate of change of the diameter:

D 2 dD
ρπ + m = 0 (8)
2 dt

Re=rho_a*Vel*D/mu_a "Reynold's number"


Call External_Flow_Sphere_ND(Re,Sc: Sh,C_d)
"determine Sherwood number as a function of Re and Sc"
Sh=h_bar_D*D/D_n_a
"definition of Sherwood number to determine mass transfer coefficient, h_bar_D"
m_dot=pi*D^2*h_bar_D*(c_s-c_infinity) "rate of naphthalene mass transfer"
(rho*pi*D^2/2)*dDdt+m_dot=0 "rate of diameter change"

The specified state variables are commented out and the Integral command in EES is used to
integrate the state equations.
{"arbitrary state variables"
time=0 [s] "time"
D=D_ini "diameter"}
time_sim=2 [day]*convert(day,s) "simulation time"
D=D_ini+Integral(dDdt,time,0,time_sim)
time_hr=time*convert(s,hr) "time in hr"
D_cm=D*convert(m,cm) "diameter in cm"
$IntegralTable time:1000 time_hr, D_cm

Figure 1 illustrates the diameter of the mothball as a function of time.

0.9

0.8
Diameter (cm)

0.7

0.6

0.5

0.4

0.3
0 5 10 15 20 25 30 35 40 45 50
Time (hr)
Figure 1: Diameter as a function of time.

The mass of the mothball is proportional to its diameter to the third power. Therefore, the
diameter at which the mothball has lost 90% of its mass corresponds to:

D90% massloss = ( 0.1) 3 Dini


1
(9)

D_final=D_ini*0.1^(1/3) "final diameter"

which leads to 0.46 cm; this diameter is reached after approximately 40 hr.
Problem 9.4-5
A square slab of dry ice (solid carbon dioxide) that is th = 1 inch thick and W = 9 inches on each
side is place on an insulated surface in a large room filled with air at T∞ = 75ºF and p = 1 atm.
Dry ice has a density of ρs = 93.6 lbm/ft3 and it sublimes with a vapor pressure of pv = 1 atm.
During this phase change, the dry ice remains at Ts = -109.4ºF. Estimate the time required for
the dry ice to disappear.

The inputs are entered in EES:

$UnitSystem SI MASS RAD PA K J


$Tabstops 0.2 0.4 0.6 3.5 in

"known information"
P = 1 [atm]*convert(atm,Pa) "atmospheric pressure"
P_s=1 [atm]*convert(atm,Pa) "vapor pressure of the carbon dioxide"
T_s=convertTemp(F,K,-109.44 [F]) "temperature of the carbon dioxide as it sublimes"
T_infinity=convertTemp(F,K,75 [F]) "air temperature"
th = 1 [inch]*convert(inch,m) "thickness of dry ice"
W = 9 [inch]*convert(inch,m) "width of dry ice"
rho_s=93.6 [lbm/ft^3]*convert(lbm/ft^3,kg/m^3) "density of solid dry ice"

The film temperature, Tfilm, is the average of the surface and air temperature. The properties of
the air (β, μ, υ, and ρ) and the diffusion coefficient (Dc,a) are computed using the film
temperature:

T_film=(T_infinity+T_s)/2 "film temperature to evaluate properties"


beta=VolExpCoef(Air,T=T_film) "volumetric coefficient of thermal expansion"
mu=viscosity(Air,T=T_film) "viscosity"
nu=mu/rho "kinematic viscosity"
rho=density(air,T=T_film,P=P) "density of the air"
D_ca=D_12_gas('air','carbondioxide',T_film,P) "diffusion coefficient for carbon dioxide in air"

The Schmidt number is computed according to:

υ
Sc = (1)
Dc ,a

The surface area and perimeter of the plate are computed according to:

As = W 2 (2)

per = 4 W (3)

The characteristic length associated with natural convection from a horizontal plate is computed,
as discussed in Section 6.2.2.
As
Lchar = (4)
per

Sc=nu/D_ca "Schmidt number"


A_s=W^2 "area of the slab"
per=4*W "perimeter of the slab"
L_char=A_s/per "significant length"

The Grashof number is analogous to the Reynolds number squared; the Grashof number is
computed according to:

g L3char β (T∞ − Ts )
Gr = (5)
υ2

The Raleigh number is the product of the Grashof number and the Prandtl number. The heat
mass transfer analogy will substitute the Schmidt number for the Prandtl number and the
Sherwood number for the Nusselt number. Therefore the Raleigh number for mass transfer is:

Ra = Gr Sc (6)

The procedure FC_plate_horizontal2_ND is used to compute the Sherwood number (Sh). The
Sherwood number is used to compute the mass transfer coefficient according to:

hD Lchar
Sh = (7)
Dc ,a

Gr=g#*L_char^3*beta*(T_infinity-T_s)/nu^2 "Grashof number"


Ra_m=Gr*Sc "Rayleigh number for mass transfer"
Call FC_plate_horizontal2_ND(Ra_m, Sc: Sh) "determine the Sherwood number for free convection"
Sh=h_bar_D*L_char/D_ca "definition of the Sherwood number"

The gas constant for carbon dioxide is computed:

Ru
R= (8)
MW

where MW is the molar mass of carbon dioxide, obtained from the MolarMass function in EES.
The concentration of carbon dioxide at the surface of the dry ice is obtained from the ideal gas
law:

ps
cs = (9)
RTs

The concentration of carbon dioxide in the atmosphere (c∞) is nearly zero. The mass flow rate of
sublimation, ignoring the blowing effect, is:
m = hD As ( cs − c∞ ) (10)

R=R#/molarMass(carbondioxide) "gas constant for carbon dioxide"


c_s=P_s/(R*T_s) "saturation density of carbon dioxide"
c_infinity=0 [kg/m^3] "density of carbon dioxide far from the slab"
m_dot=A_s*h_bar_D*(c_s-c_infinity) "rate of sublimation of carbon dioxide"

The time required to sublimate the slab is:

time m = W 2 th ρ s (11)

time*m_dot=W^2*th*rho_s "time required"


time_hr=time*convert(s,hr) "in hr"

which leads to time = 1.86 hr.

If the blowing factor is accounted for then it is calculated according to:

ln(1 + B)
BF = (12)
B

where B is:

mf ∞ − mf s
B= (13)
mf s − 1

The mass fraction of carbon dioxide in the atmosphere is zero and the mass fraction at the
surface is 1; therefore:

BF = ln(2) (14)

and the heat transfer coefficient should be multiplied by BF:

BF=ln(2) "blowing factor"


{m_dot=A_s*h_bar_D*(c_s-c_infinity) "rate of sublimation of carbon dioxide"}
m_dot=A_s*BF*h_bar_D*(c_s-c_infinity)
"rate of sublimation of carbon dioxide, considering the effect of blowing"

which leads to time = 2.69 hr.


Problem 9.5-1: Falling Raindrops
A spherical raindrop is falling through air at T∞ = 20°C, atmospheric pressure, and a relative
humidity of RH = 0.5. The diameter of the sphere is Dini = 1 mm. You may assume that the
sphere is always at its terminal velocity (i.e., the velocity at which the drag and gravitational
forces are balanced) and at the temperature where evaporation and convection are balanced.
Assume that the droplet remains spherical.
a.) Plot the diameter of the droplet as a function of time.

The known information is entered in EES:

$UnitSystem SI MASS RAD PA K J


$Tabstops 0.2 0.4 0.6 3.5 in

D_ini=1 [mm]*convert(mm,m) "initial diameter of the raindrop"


T_infinity=converttemp(C,K,20 [C]) "ambient temperature"
P=1 [atm]*convert(atm,Pa) "atmospheric pressure"
RH=0.5 [-] "relative humidity of ambient air"

Arbitrary values of diameter, D, and time, time, are assumed and used to determine the time rate
of change of the droplet diameter. The volume of the droplet is:

3
4 ⎛D⎞
V = π⎜ ⎟ (1)
3 ⎝2⎠

The mass of the droplet is:

M = ρw V (2)

where ρw is the density of liquid water.

"Arbitrary value of state variables"


D=D_ini "diameter"
time=0 [s] "time"

V=4*pi*(D/2)^3/3 "volume"
rho_w=density(Water,T=T_infinity,P=P) "density of liquid water"
M=rho_w*V "mass"

The droplet temperature (T) and velocity (u∞) are guessed; the value of these variables will be
recalculated in order to complete the problem. The film temperature is used to compute the
properties of the air in the atmosphere (ρa, μa, ka, and Pra). The Reynolds number associated
with the flow over the falling sphere is computed according to:

ρ a u∞ D
Re = (3)
μa
The External_Flow_Sphere_ND function in EES is used to compute the average Nusselt number
( Nu ) and drag coefficient (Cd).

T=converttemp(C,K,14 [C]) "guess droplet temperature"


u_infinity=15 [m/s] "guess droplet velocity"
T_film=(T+T_infinity)/2 "film temperature"
rho_a=density(Air,T=T_film,P=P) "density"
mu_a=viscosity(Air,T=T_film) "viscosity"
Pr_a=Prandtl(Air,T=T_film) "Prandtl number"
k_a=conductivity(Air,T=T_film) "conductivity"
Re=u_infinity*D*rho_a/mu_a "Reynolds number"
Call External_Flow_Sphere_ND(Re,Pr_a: Nusselt_bar,C_d) "access correlations for flow over a sphere"

The average heat transfer coefficient is computed according to:

ka Nu
h= (4)
D

The drag force is computed according to:

D2 u2
Fdrag = π Cd ρ a ∞ (5)
4 2

h_bar=Nusselt_bar*k_a/D "average heat transfer coefficient"


F_drag=(pi*D^2/4)*rho_a*C_d*u_infinity^2/2 "drag force"

The problem is solved and the guess values are updated. The guess for u∞ is removed and the
drag force is set equal to the force of gravity:

Fdrag = M g (6)

where g is the acceleration of gravity.

{u_infinity=15 [m/s]} "guess droplet velocity"


F_drag=M*g# "force balance at terminal velocity"

The diffusion coefficient for air in water is computed using the curve fit provided by Bolz and
Tuve (1976) and discussed in EXAMPLE 9.2-1.

⎡ m2 ⎤ -8 ⎡ m ⎤
2
-10 ⎡ m
2
⎤ 2
Da , w = −2.775x10 -6
⎢ s ⎥ + 4.479x10 ⎢ s-K ⎥ T + 1.656x10 ⎢ s-K 2 ⎥ T (7)
⎣ ⎦ ⎣ ⎦ ⎣ ⎦

The Schmidt number is computed according to:


μa
Sc = (8)
ρ a Da , w

The External_Flow_Sphere_ND function in EES is used to access the correlations for flow over a
sphere. The heat and mass transfer analogy requires that the Prandtl number be replaced by the
Schmidt number and therefore the function returns the average Sherwood number ( Sh ) rather
than the Nusselt number. The mass transfer coefficient is computed according to:

Sh Da , w
hD = (9)
D

D_a_w=-2.775e-6 [m^2/s]+4.479e-8 [m^2/s-K]*T+1.656e-10 [m^2/s-K^2]*T^2


"diffusion coefficient for air-water mixtures from Bolz and Tuve (1976)"
Sc=mu_a/(rho_a*D_a_w) "Schmidt number"
Call External_Flow_Sphere_ND(Re,Sc: Sh_bar,C_d2) "heat mass transfer analogy"
h_bar_D=Sh_bar*D_a_w/D "mass transfer coefficient"

The surface area of the sphere is:

As = π D 2 (10)

The gas constant for water vapor is:

Ru
R= (11)
MW

where Ru is the universal gas constant and MW is the molecular weight of water, obtained using
the MolarMass function in EES. The saturation concentration is obtained from the ideal gas law.

psat ,T =T
cs = (12)
RT

where psat,T=T is the saturation pressure of water at the droplet temperature, evaluated using EES’
built-in property routines. The concentration of water vapor in the air is obtained from:

RH psat ,T =T∞
cs = (13)
RT

The rate of mass transfer is:

m = As hD ( cs − c∞ ) (14)

The rate of change of the diameter of the droplet is:


dD
ρw π D2 = − m (15)
dt

A_s=pi*D^2 "surface area of droplet"


R=R#/MolarMass('Water') "gas constant for water"
c_s=pressure(Water,T=T,x=1 [-])/(R*T) "concentration of water at surface"
c_infinity=RH*pressure(Water,T=T_infinity,x=1 [-])/(R*T_infinity) "concentration of water in air"
m_dot=h_bar_D*A_s*(c_s-c_infinity) "mass transfer"
Di_v=enthalpy(Water,T=T,x=1[-])-enthalpy(Water,T=T,x=0[-]) "latent heat of vaporization"
m_dot*Di_v=h_bar*A_s*(T_infinity-T) "energy balance on droplet"
rho_w*4*pi*(D/2)^2*dD\dt=-m_dot "rate of change of diameter"

The values of the state variables are commented out and the Integral command.

{"Arbitrary value of state variables"


D=D_ini "diameter"
time=0 [s] "time"}
t_sim=1000 [s] "simulation time"
D=D_ini+Integral(dD\dt,time,0,t_sim) "Integrate state equations"
D_mm=D*convert(m,mm) "diameter, in mm"

$IntegralTable time, D_mm,u_infinity,T

Figure 1 illustrates the diameter of the droplet as a function of time and Figure 2 illustrates the
velocity of the droplet as a function of time.
1

0.9

0.8
Diameter (mm)

0.7

0.6

0.5

0.4

0.3

0.2
0 100 200 300 400 500 600 700 800 900 1000
Time (s)
Figure 1: Droplet diameter as a function of time.
4.5

3.5

Velocity (m/s)
2.5

1.5

0.5
0 100 200 300 400 500 600 700 800 900 1000
Time (s)
Figure 2: Droplet velocity as a function of time.
Problem 9.5-2 (9-10 in text): Design of an Air-Washer
You have asked to join the team of engineers responsible for the design of an air-washer. Your
part of this project is to prepare an analysis that will determine the diameter, velocity, and
temperature of droplets as they fall in an upward flowing air stream. You are considering a
single water droplet with an initial diameter of 1.5 mm and initial temperature of 45°C that is
released into a 25°C, 35% relative humidity, 100 kPa air stream that is flowing upward at 30
m/s.
a.) Plot the diameter, velocity and temperature of the droplet as a function of time. Assume that
the droplet remains spherical and that it can be considered to have a uniform temperature at
any time.

The known information is entered in EES:

$UnitSystem SI MASS RAD PA K J


$Tabstops 0.2 0.4 0.6 3.5 in

"known information"
D_ini=1.5 [mm]*convert(mm,m) "initial droplet diameter"
T_w_ini=convertTemp(C,K,45 [C]) "initial temperature of droplet"
RH_infinity=0.35 [-] "relative humidity of free air stream"
T_infinity=convertTemp(C,K,25 [C]) "air temperature"
P_atm=100 [kPa]*convert(kPa,Pa) "atmospheric pressure"
v_air=30 [m/s] "air velocity"
v_ini=0 [m/s] "initial velocity of droplet"
z_ini=0 [m] "initial position"

The problem has four state variables, including the droplet position (z), velocity (v), diameter
(D), and temperature (Tw). Arbitrary values of these variables and time are assumed in order to
implement a set of state equations that calculate the time rate of change of these parameters.

"Arbitrary values of the state variables"


T_w=T_w_ini "droplet temperature"
v=v_ini "absolute droplet velocity"
z=z_ini "droplet position"
D=D_ini "droplet diameter"
time = 0 [s] "time"

The time rate of change of the position is simply the velocity:

dz
=v (1)
dt

dz\dt=v "rate of change of position is absolute velocity"

The film temperature is computed and used to determine the properties of the air (ρ, μ, Pr, and
k).

T_film=(T_w+T_infinity)/2 "film temperature"


rho=density(Air,T=T_film,P=P_atm) "density of air"
mu=viscosity(Air,T=T_film) "viscosity of air"
Pr=Prandtl(Air,T=T_film) "Prandtl number"
k=conductivity(Air,T=T_film) "thermal conductivity of air"

The Reynolds number associated with the flow of air over the droplet is obtained; note that the
relative velocity between the air stream and the velocity of the droplet is used:

ρ ( vair − v ) D
Re = (2)
μ

The External_Flow_Sphere_ND function in EES is used to compute the average Nusselt number
( Nu ) and drag coefficient (Cd). The average heat transfer coefficient is computed according to:

k Nu
h= (3)
D

The drag force is computed; note that again the relative velocity is used:

(v − v)
2
D2
Fd = π Cd ρ air (4)
4 2

Re=(v_air-v)*rho*D/mu "Reynolds number associated with flow over the droplet"


Call External_Flow_Sphere_ND(Re,Pr: Nusselt_bar,C_d)
"Determine Nusselt number and drag coefficient"
h_bar=Nusselt_bar*k/D "heat transfer coefficient"
F_d=(pi*D^2/4)*C_d*rho*(v_air-v)^2/2 "drag force in the y-direction"

The volume of the droplet is:

3
4 ⎛D⎞
V = π⎜ ⎟ (5)
3 ⎝2⎠

The mass of the droplet is:

M = ρw V (6)

where ρw is the density of liquid water. A force balance on the droplet leads to the time rate of
change of the velocity:

dv
M = Fd − M g (7)
dt

Volume=4*pi*(D/2)^3/3 "volume of droplet"


rho_w=density(Water,T=T_w,P=P_atm) "density of liquid water"
M=Volume*rho_w "mass of droplet"
M*dv\dt=F_d-M*g# "force balance provides acceleration"

The diffusion coefficient for air in water is computed using the curve fit provided by Bolz and
Tuve (1976) and discussed in EXAMPLE 9.2-1.

⎡ m2 ⎤ ⎡ m2 ⎤ -10 ⎡ m
2
⎤ 2
Da , w = −2.775x10-6 ⎢ ⎥ + 4.479x10-8 ⎢ ⎥ T + 1.656x10 ⎢ 2⎥
T (8)
⎣ s ⎦ ⎣ s-K ⎦ ⎣ s-K ⎦

The Schmidt number is computed according to:

μ
Sc = (9)
ρ Da , w

The External_Flow_Sphere_ND function in EES is used to access the correlations for flow over a
sphere. The heat and mass transfer analogy requires that the Prandtl number be replaced by the
Schmidt number and therefore the function returns the average Sherwood number ( Sh ) rather
than the Nusselt number. The mass transfer coefficient is computed according to:

Sh Da , w
hD = (10)
D

D_w_a=-2.775e-6 [m^2/s]+4.479e-8 [m^2/s-K]*T_film+1.656e-10[m^2/s-K^2]*T_film^2


"diffusion coefficient for air-water mixture"
Sc=mu/(rho*D_w_a) "Schmidt number"
Call External_Flow_Sphere_ND(Re,Sc: Sh_bar,C_d2) "Determine Sherwood number"
h_bar_D=D_w_a*Sh_bar/D "mass transfer coefficient"

The surface area of the sphere is:

As = π D 2 (11)

The gas constant for water vapor is:

Ru
R= (12)
MW

where Ru is the universal gas constant and MW is the molecular weight of water, obtained using
the MolarMass function in EES. The saturation concentration is obtained from the ideal gas law.

psat ,T =T
cs = (13)
RT
where psat,T=T is the saturation pressure of water at the droplet temperature, evaluated using EES’
built-in property routines. The concentration of water vapor in the air is obtained from:

RH psat ,T =T∞
cs = (14)
RT

The rate of mass transfer is:

m = As hD ( cs − c∞ ) (15)

The rate of change of the diameter of the droplet is:

dD
ρ w As = −m (16)
dt

A_s=pi*D^2 "surface area of droplet"


R_w=R#/MolarMass(Water) "gas constant for water vapor"
c_s=P_sat(Water,T=T_w)/(R_w*T_w) "concentration of water vapor at droplet surface"
c_infinity=RH_infinity*P_sat(Water,T=T_infinity)/(R_w*T_infinity)
"concentration of water vapor in free stream"
m_dot=A_s*h_bar_D*(c_s-c_infinity) "mass flow rate"
dD\dt*A_s*rho_w=-m_dot "rate of change of droplet diameter"

An energy balance on the droplet leads to the time rate of change of the temperature of the
droplet:

dT
h As (T∞ − T ) = m Δi + cw M (17)
dt

where Δi is the latent heat of fusion and cw is the specific heat capacity of liquid water.

c_w=cP(Water,T=T_w,P=P_atm) "specific heat capacity of liquid water"


Di_vap=enthalpy(Water,T=T_w,x=1)-enthalpy(Water,T=T_w,x=0) "latent heat of fusion of water"
h_bar*A_s*(T_infinity-T_w)=m_dot*Di_vap+c_w*M*dT\dt
"energy balance to provide rate of change of temperature"

The four state equations have been derived. Therefore, the state variables are commented out
and the state equations integrated.

time_sim=3 [s] "simulation time"


"Integrate state equations"
v=v_ini+Integral(dv\dt,time,0,time_sim)
z=z_ini+Integral(dz\dt,time,0,time_sim)
D=D_ini+Integral(dD\dt,time,0,time_sim)
T_w=T_w_ini+Integral(dT\dt,time,0,time_sim)

$IntegralTable time:0.1,z,v,D,T_w
Figure 1 illustrates the droplet position and velocity as a function of time. Note that the droplet
very quickly reaches a terminal velocity that is slightly less than the velocity of the air stream.

70 35

60 30

50 25

Velocity (m/s)
Position (m)

40 v 20

30 15
z
20 10

10 5

0 0
0 0.5 1 1.5 2 2.5 3
Time (s)
Figure 1: Droplet position and velocity as a function of time.

Figure 2 illustrates the droplet temperature as a function of time and shows that the droplet very
quickly reaches the wetbulb temperature where convection and evaporation are balanced.

320

315
Droplet temperature (K)

310

305

300

295

290
0 0.5 1 1.5 2 2.5 3
Time (s)
Figure 2: Droplet temperature as a function of time.

Figure 3 illustrates the droplet diameter as a function of time for the first few seconds and shows
that the droplet diameter does not change as quickly as its temperature or velocity. Figure 4
shows the droplet diameter as a function of time for several minutes and shows the droplet
diameter drops continuously over this time.
0.0015

0.001498

0.001496

Diameter (m) 0.001494

0.001492

0.00149

0.001488
0 0.5 1 1.5 2 2.5 3
Time (s)
Figure 3: Droplet diameter as a function of time for the first 3 seconds.

0.0015

0.00145

0.0014
Diameter (m)

0.00135

0.0013

0.00125

0.0012

0.00115
0 50 100 150 200 250 300
Time (s)
Figure 4: Droplet diameter as a function of time for the first several minutes.
Problem 9.5-3 (9-11 in text)
One type of household humidifier operates by expelling water droplets into the air. The droplets
have an average diameter of 10 μm. After leaving the dehumidifier, the droplets "float" around
the room and evaporate. In a particular case, the room is maintained at 25°C, 100 kPa and 25%
relative humidity. You may assume that the droplet is at the temperature where evaporation and
convection are balanced.
a.) Plot the mass of the droplet as a function of time and determine the time required for the
droplets to completely evaporate.

The known information is entered in EES:

$UnitSystem SI MASS RAD PA K J


$Tabstops 0.2 0.4 0.6 3.5 in

"known information"
D_ini=10 [micron]*convert(micron,m) "initial droplet diameter"
T_infinity=convertTemp(C,K,25 [C]) "air temperature"
P=100 [kPa]*convert(kPa,Pa) "ambient pressure"
RH=0.25 [-] "relative humidity"

The problem has a single state variable, the droplet diameter (D). Arbitrary values of the
diameter and time are assumed in order to implement a state equations that calculates the time
rate of change of diameter.

"arbitrary values of the state variables"


D=D_ini "droplet"
time=0 [s] "time"

A value of the droplet temperature (Tw) is guessed and will be adjusted in order to balance
convection and evaporation. The film temperature is computed and used to determine the
properties of the air (ρ, μ, Pr, k, υ, and β).

T_w=290 [K] "guess for droplet temperature"


T_film=(T_w+T_infinity)/2 "film temperature"
rho=density(Air,T=T_film,P=P) "density of air"
mu=viscosity(Air,T=T_film) "viscosity of air"
Pr=Prandtl(Air,T=T_film) "Prandtl number"
k=conductivity(Air,T=T_film) "thermal conductivity of air"
nu=mu/rho "kinematic viscosity of air"
beta=1/T_film "coefficient of volumetric expansion"

The Grashof number is analogous to the Reynolds number squared and can be computed
according to:

g D 3 β (T∞ − Tw )
Gr = (1)
υ2

The Raleigh number is the product of the Grashof number and the Prandtl number:
Ra = Gr Pr (2)

The FC_sphere_ND function in EES is used to compute the average Nusselt number ( Nu ). The
average heat transfer coefficient is computed according to:

k Nu
h= (3)
D

Gr=g#*D^3*beta*abs(T_infinity-T_w)/(nu^2) "Grashof number"


Ra=Gr*Pr "Rayleigh number"
Call FC_sphere_ND(Ra, Pr: Nusselt_bar) "determine Nusselt number"
Nusselt_bar=h_bar*D/k "determine convection coefficient"

The diffusion coefficient for air in water is computed using the curve fit provided by Bolz and
Tuve (1976) and discussed in EXAMPLE 9.2-1.

⎡ m2 ⎤ ⎡ m2 ⎤ -10 ⎡ m
2
⎤ 2
Da , w = −2.775x10-6 ⎢ ⎥ + 4.479x10-8 ⎢ ⎥ T + 1.656x10 ⎢ 2⎥
T (4)
⎣ s ⎦ ⎣ s-K ⎦ ⎣ s-K ⎦

The Schmidt number is computed according to:

μ
Sc = (5)
ρ Da , w

The Raleigh number for mass transfer is calculated by replacing the Prandtl number in Eq. (2)
with the Schmidt number:

Ram = Gr Sc (6)

The FC_Sphere_ND function in EES is used to access the correlations for flow over a sphere.
The heat and mass transfer analogy requires that the Prandtl number be replaced by the Schmidt
number and therefore the function returns the average Sherwood number ( Sh ) rather than the
Nusselt number. The mass transfer coefficient is computed according to:

Sh Da , w
hD = (7)
D

D_w_a=-2.775e-6 [m^2/s]+4.479e-8 [m^2/s-K]*T_film+1.656e-10[m^2/s-K^2]*T_film^2


"diffusion coefficient for air-water mixture"
Sc=mu/(rho*D_w_a) "Schmidt number"
Ra_m=Gr*Sc "Rayleigh number for mass transfer"
Call FC_sphere_ND(Ra_m, Sc: Sh_bar) "determine Nusselt number"
Sh_bar=h_bar_D*D/D_w_a "definition of Sherwood number"
The surface area of the sphere is:

As = π D 2 (8)

The gas constant for water vapor is:

Ru
R= (9)
MW

where Ru is the universal gas constant and MW is the molecular weight of water, obtained using
the MolarMass function in EES. The saturation concentration is obtained from the ideal gas law.

psat ,T =T
cs = (10)
RT

where psat,T=T is the saturation pressure of water at the droplet temperature, evaluated using EES’
built-in property routines. The concentration of water vapor in the air is obtained from:

RH psat ,T =T∞
cs = (11)
RT

The rate of mass transfer is:

m = As hD ( cs − c∞ ) (12)

A_s=pi*D^2 "surface area of droplet"


R_w=R#/MolarMass(Water) "gas constant for water vapor"
c_s=P_sat(Water,T=T_w)/(R_w*T_w) "concentration of water vapor at droplet surface"
c_infinity=RH*P_sat(Water,T=T_infinity)/(R_w*T_infinity) "concentration of water vapor in free stream"
m_dot=A_s*h_bar_D*(c_s-c_infinity) "mass flow rate"

The problem is solved and the guess values are updated. The assumed value of Tw is
commented out and an energy balance on the droplet is enforced:

h As (T∞ − T ) = m Δi (13)

where Δi is the latent heat of fusion.

{T_w=290 [K]} "guess for droplet temperature"


Di_vap=enthalpy(Water,T=T_w,x=1)-enthalpy(Water,T=T_w,x=0) "latent heat of fusion of water"
h_bar*A_s*(T_infinity-T_w)=m_dot*Di_vap
"energy balance to provide rate of change of temperature"

The volume of the droplet is:


3
4 ⎛D⎞
V = π⎜ ⎟ (14)
3 ⎝2⎠

The mass of the droplet is:

M = ρw V (15)

where ρw is the density of liquid water. The rate of change of the diameter of the droplet is:

dD
ρ w As = −m (16)
dt

Volume=4*pi*(D/2)^3/3 "volume of droplet"


rho_w=density(Water,T=T_w,P=P) "density of liquid water"
M=Volume*rho_w "mass of droplet"
dD\dt*A_s*rho_w=-m_dot "rate of change of droplet diameter"

The state variables are commented out and the Integral command is used to integrate the state
equation through time.

{"arbitrary values of the state variables"


D=D_ini "droplet"
time=0 [s] "time"}
t_sim=0.195 [s] "simulation time"
D=D_ini+Integral(dD\dt,time,0,t_sim) "integrate state equations"
D_micron=D*convert(m,micron) "in micron"

$IntegralTable time,D_micron, M

Figure 1 illustrates the droplet diameter and mass as a function of time and shows that the droplet
will evaporate in approximately 0.2 s.
-13
10 6x 10

9
-13
5x 10
8

7 -13
4x 10
Diameter (micron)

D
6
Mass (kg)

-13
5 3x 10

4
-13
2x 10
3 M
2 -13
10
1
0
0 0x 10
0 0.04 0.08 0.12 0.16 0.2
Time (s)
Figure 1: Droplet diameter and mass as a function of time.
b.) The humidifier requires a work input to form the droplets. The work input is related to the
change in area of the water as it is transformed from one large "drop" to many smaller
droplets. Calculate the energy required to distribute 1 kg of droplets with this vaporizer and
compare it to the energy needed to vaporize one kg of water at 25°C. Comment on whether
you believe that this humidifier saves energy compared to traditional vaporization process
based on boiling water.

A unit mass of water is considered, Mw. The surface tension of water (σ) is computed. The
initial volume of the water droplet is computed according to:

3
4 ⎛D ⎞
Vini = π ⎜ ini ⎟ (17)
3 ⎝ 2 ⎠

The mass of the droplet is initially:

M ini = ρ w Vini (18)

The total number of droplets is:

Mw
N droplets = (19)
M ini

The surface area per droplet is:

As ,ini = π Dini
2
(20)

Therefore, the total surface are of the droplets is:

As ,tot = N droplets As ,ini (21)

The work required to form the droplets is:

W = σ As ,tot (22)

which can be compared to the thermal energy required to vaporize the water directly:

Q = M w Δi (23)

"Work input to make droplets"


M_w=1 [kg] "examine 1 kg of water"
sigma=surfaceTension(Water,T=T_w) "surface tension of water"
Volume_ini=4*pi*(D_ini/2)^3/3 "initial volume of droplet"
M_ini=Volume_ini*rho_w "initial mass of droplet"
N_droplets=M_w/M_ini "number of droplets"
A_s_ini=pi*D^2 "initial surface area of droplet"
A_s_tot=N_droplets*A_s_ini "total surface area of droplets"
W=sigma*A_s_tot "work to make the droplets"
Q=Di_vap*M_w "energy to vaporize the water"

which leads to W = 44.3 J and Q = 2.47x106 J. Therefore, it requires much less energy directly
to form small droplets than to vaporize them with thermal energy. However, the energy required
to evaporate the droplets comes from the air in the room which may be heated by combusting
fossil fuel; therefore, the energy required may not be very different.
Problem 9.6-1
Determine the performance of the cooling coil described in EXAMPLE 9.6-1 if R134a, rather
than chilled water is used as the coolant. The R134a enters at Tr,in = 5°C with a quality of xin =
0.35 at a flow rate of m r = 0.02 kg/s. Also, determine the outlet quality of the R134a.

The inputs are entered in EES:

$UnitSystem SI MASS RAD PA K J


$Tabstops 0.2 0.4 0.6 3.5 in

D_out=1.02 [cm]*convert(cm,m) "outer diameter of tube"


th = 0.9 [mm]*convert(mm,m) "tube wall thickness"
N_t_row=10 [-] "number of tube rows"
N_t_col=2 [-] "number of tube columns"
H=0.2 [m] "height of heat exchanger face"
W=0.2 [m] "width of heat exchanger face"
L=0.06 [m] "length of heat exchanger in air flow direction"
V_dot_a=0.06 [m^3/s] "volumetric flow rate of air"
p_a=1 [atm]*convert(atm,Pa) "atmospheric pressure"
T_a_in=convertTemp(C,K,27 [C]) "inlet air temperature"
RH_a_in=0.75 [-] "inlet air relative humidity"
m_dot_r=0.02 [kg/s] "R134a flow rate"
x_r_in=0.35 "R134a inlet quality"
T_r_in=converttemp(C,K,5 [C]) "R134a saturation temperature"
P_r=P_sat(R134a,T=T_r_in) "R134a saturation pressure"
s_v=25.4 [mm]*convert(mm,m) "vertical separation distance between tubes"
s_h=22 [mm]*convert(mm,m) "horizontal separation distance between tubes"
th_fin=0.33 [mm]*convert(mm,m) "fin thickness"
p_fin=3.18 [mm]*convert(mm,m) "fin pitch"
e=1.0 [micron]*convert(micron,m) "roughness of tube internal surface"

The inlet humidity ratio (ωa,in) and the dewpoint associated with the incoming air (Tdp, the
temperature at which the relative humidity reaches 100% and therefore water will begin to
condense) are computed:

"Properties of the inlet air"


omega_a_in=HumRat(AirH2O,P=p_a,T=T_a_in,R=RH_a_in) "humidity ratio at inlet"
T_dp=Temperature(AirH2O,P=p_a,w=omega_a_in,R=1) "dewpoint temperature of air"
T_dp_C=converttemp(K,C,T_dp) "in C"

which leads to Tdp = 295.3 K (22.19°C). Because the entering refrigerant temperature is less than
Tdp, water will condense onto the coil surface provided that the cooling coil is sufficiently large.

The refrigerant-side thermal resistance is evaluated. The inner diameter of the tube is:

Din = Dout − 2 th (1)

and the total length of tube is:


Ltube = N t ,row N t ,col W (2)

The cross-sectional area for refrigerant flow is:

D2
Ac = π (3)
4

and the mass flux of refrigerant is:

m r
G= (4)
Ac

"Heat transfer coefficient on refrigerant side"


D_in=D_out-2*th "tube inner diameter"
L_tube=N_t_row*N_t_col*W "total tube length"
A_c=pi*D_in^2/4 "cross-sectional area of the tube"
G=m_dot_r/A_c "mass velocity"

The outlet quality of the refrigerant (xout) is initially guessed; this guess will be removed later.
The total rate of heat transfer to the refrigerant is computed according to:

q = m r ( ir ,out − ir ,in ) (5)

where ir,in and ir,out are the specific enthalpies of the refrigerant at the inlet and exit conditions,
respectively. The average heat flux at the tube inner surface is:

q
q ′′ = (6)
π Din L

x_r_out=0.95 [-] "guess value of exit quality"


i_r_out=enthalpy(R134a,T=T_r_in,x=x_r_out) "specific enthalpy of exiting R134a"
i_r_in=enthalpy(R134a,T=T_r_in,x=x_r_in) "specific enthalpy of entering R134a"
q_dot=m_dot_r*(i_r_out-i_r_in) "rate of heat transfer"
q``_dot=q_dot/(pi*D_in*L) "average heat flux"

The average refrigerant-side heat transfer coefficient ( hr ) is computed using the Flow_Boiling_avg
function in EES. The resistance to heat transfer between the refrigerant and the tube inner
surface is:

1
Rin = (7)
π Din Ltube hr

h_bar_r=Flow_Boiling_avg('R134a', T_r_in, G, D_in,x_r_in, x_r_out, q``_dot, 'Horizontal')


"access correlations for internal boiling"
R_in=1/(pi*D_in*L_tube*h_bar_r) "resistance to convection on refrigerant-side"
The fouling resistance for the refrigerant flow is assumed to be negligible. The conductivity of
copper (km) is obtained at the average temperature using the internal property routine in EES and
used to compute the conduction resistance through the tube:

⎛D ⎞
ln ⎜ out ⎟
= ⎝ in ⎠
D
Rcond (8)
2 π km Ltube

"Conduction resistance"
T_avg=(T_r_in+T_a_in)/2 "average temperature"
k_m=k_('Copper',T_avg) "tube conductivity"
R_cond=ln(D_out/D_in)/(2*pi*k_m*L_tube) "tube resistance"

The mass flow rate of air is computed according to:

m a = ρ a ,in Va (9)

where ρa,in is the density of the air at the inlet. The heat exchanger geometry corresponds to
compact heat exchanger core fc_tubes_s80-38T. The compact heat exchanger function
CHX_h_finned_tube is used to evaluate the dry heat transfer coefficient on the air side ( ha ):

"Compact heat exchanger correlation"


rho_a_in=density(Air,T=T_a_in,P=P_a) "density of air at inlet"
m_dot_a=V_dot_a*rho_a_in "mass flow rate of air"
TypeHX$='fc_tubes_s80-38T' "heat exchanger identifier name"
Call CHX_h_finned_tube(TypeHX$, m_dot_a, W*H, 'Air',T_avg, P_a:h_bar_a)
"access compact heat exchanger procedure"

The total surface area of the fins is:

⎛ W ⎞⎛ π Dout
2

As , fin ,tot = 2 ⎜ ⎟ ⎜ H L − N N ⎟ (10)
⎜ p fin ⎟ ⎝ t , row t , col
4 ⎠
⎝ ⎠

and the total surface area of the unfinned tube is:

⎛ th ⎞
As ,unfin = π Dout Ltube ⎜1 − fin ⎟ (11)
⎜ p fin ⎟⎠

The total air-side surface area is:

Atot = As , fin ,tot + As ,unfin (12)

A_s_fin=2*(W/p_fin)*(H*L-N_t_row*N_t_col*pi*D_out^2/4) "total fin area"


A_s_unfin=pi*D_out*L_tube*(1-th_fin/p_fin) "total un-finned area"
A_tot=A_s_fin+A_s_unfin "total air-side surface area"

The dry coil/wet coil analysis divides the heat exchanger into a separate dry coil and wet coil. In
the dry coil, the temperature of the moist air is reduced to the dew point temperature; the exit
temperature from the dry coil (Ta,dc,out) is therefore the dew point temperature.

"wet coil/dry coil approach"

"Dry coil analysis"


T_a_dc_out=T_dp "temperature of air at exit of dry coil"

The fraction of the total heat exchange surface required by the dry coil section must be
determined. The specific heat capacity of the moist-air on a per mass of dry air basis ( ca′′ ) is
calculated using EES' internal psychrometric function. The capacitance rate of the air is:

C a , dc = m a ca′′ (13)

c``_a=cP(AirH2O,P=P_a,T=T_a_in,R=RH_a_in) "specific heat capacity of moist air in dry coil"


C_dot_a_dc=m_dot_a*c``_a "capacity rate of air"

Provided that the cooling coil is sufficiently large so that the dew point temperature is reached,
the rate of heat transfer in the dry coil section of the heat exchanger is given by:

qdc = C a ,dc (Ta ,in − Ta ,dc ,out ) (14)

The maximum possible rate of heat transfer is:

qdc , max = C a ,dc (Ta ,in − Tr ,in ) (15)

The effectiveness of the dry coil section is the ratio of the actual to the maximum possible heat
transfer rate:

qdc
ε dc = (16)
qdc ,max

q_dot_dc=C_dot_a_dc*(T_a_in-T_a_dc_out) "heat transfer rate in dry coil"


q_dot_dc_max=C_dot_a_dc*(T_a_in-T_r_in) "maximum possible heat transfer rate in dry coil"
eff_dc=q_dot_dc/q_dot_dc_max "effectiveness of dry coil"

The number of transfer units in the dry coil section (NTUdc) is obtained using the ε-NTU solution
for a cross-flow heat exchanger (the configuration does not really matter since the capacity ratio
is zero) accessed using the HX function in EES. The conductance required in the dry-coil section
is:
UAdc = NTU dc C min ,dc (17)

NTU_dc=HX('crossflow_one_unmixed', eff_dc, C_dot_a_dc, 1e99 [W/K], 'NTU')


"number of transfer units in dry coil"
UA_dc=NTU_dc*C_dot_a_dc "conductance in dry coil"

The total resistance to heat transfer in the dry coil is the inverse of the conductance:

1
Rdc = (18)
UAdc

and is made up of the resistance to convection on the refrigerant side, a resistance to conduction
through the tube, and a convection resistance on the air side (assuming that the fin efficiency is
100%):

Rin + Rcond 1
Rdc = + (19)
Fdc ha Atot Fdc

where Fdc is the fraction of the total heat exchanger required by the dry coil.

R_dc=1/UA_dc "thermal resistance in dry coil"


R_dc=(R_in+R_cond)/F_dc+1/(h_bar_a*A_tot*F_dc) "determine F_dc"

which leads to Fdc = 0.259 (i.e., 26% of the coil is dry, according to this analysis). The
remainder of the heat exchanger coil is wet:

Fwc = 1 − Fdc (20)

"Wet coil analysis"


F_wc=1-F_dc "fraction of heat exchanger that is wet"

The wet coil is analyzed using the concepts of a saturation specific heat capacity and wet coil
conductance. The saturation specific heat capacity is used to account for the latent heat
associated with the condensing water vapor and is calculated according to:

⎡ia′′ (Tdc ,out , p,RH = 1) − ia′′ (Ta ,out , p,RH = 1) ⎤⎦


ca′′,sat = ⎣ (21)
(Tdc ,out − Ta ,out )
Initially, a value for Ta,out is assumed; this value is subsequently commented out in order to close
the solution:

T_a_out=T_r_in "guess for air outlet temperature - will be removed"


c``_a_sat=(enthalpy(AirH2O,T=T_a_dc_out,R=1,P=P_a)-&
enthalpy(AirH2O,T=T_a_out,R=1,P=P_a))/(T_a_dc_out-T_a_out)
"saturation specific heat capacity - remove the guess above when uncommenting this equation"
The capacitance rate of the air in the wet coil is:

C a , wc = m a ca′′, sat (22)

The conductance of the wet coil is computed by augmenting the air side heat transfer coefficient
by the ratio ca′′, sat / ca′′ in order to account for the enhancement of the energy transfer rate due to
mass transfer. The thermal resistance of the wet coil section is:

Rin + Rcond ca′′


Rwc = + (23)
Fwc ha ca′′, sat Atot Fwc

and the conductance of the wet coil is:

1
UAwc = (24)
Rwc

The number of transfer units in the wet coil is:

UA
NTU wc =  wc (25)
Ca , wc

The effectiveness of the wet coil (εwc) is obtained using the ε-NTU solution for a cross-flow heat
exchanger, obtained using the HX function in EES:

C_dot_a_wc=m_dot_a*c``_a_sat "capacity rate of air in wet coil"


R_wc=(R_in+R_cond)/F_wc+c``_a/(A_tot*h_bar_a*c``_a_sat*F_wc)"wet coil resistance"
UA_wc=1/R_wc "wet coil conductance"
NTU_wc=UA_wc/C_dot_a_wc "number of transfer units in the wet coil"
eff_wc=HX('crossflow_one_unmixed', NTU_wc, C_dot_a_wc, 1e99 [W/K], 'epsilon')
"effectiveness of wet coil"

The heat transfer rate in the wet coil section is obtained from the effectiveness:

qwc = ε wc C a , wc (Tdc ,out − Tr ,in ) (26)

q_dot_wc=eff_wc*C_dot_a_wc*(T_a_dc_out-T_r_in) "heat transfer rate in wet coil"

The temperature of the air leaving the wet coil is obtained using an energy balance on the air-
side:

q
Ta ,out = Tdc ,out −  wc (27)
Ca , wc
Before calculating the value of Ta,out, the guess values for the problem are updated (select Update
Guesses from the Calculate menu) and the initial guess for Ta,out is commented out:

{T_a_out=T_r_in} "guess for air outlet temperature - will be removed"


T_a_out=T_a_dc_out-q_dot_wc/C_dot_a_wc "temperature of air leaving wet coil"
T_a_out_C=converttemp(K,C,T_a_out) "in C"

which leads to Ta,out = 287.0 K (13.9°C).

The wet coil model assumes that the air leaves saturated (i.e., at 100% relative humidity) and
therefore the humidity ratio of the air leaving the heat exchanger (ωa,out) can be obtained using
EES' psychrometric functions.

omega_a_out=HumRat(AirH2O,T=T_a_out,R=1,P=p_a) "humidity ratio at exit"

The total heat transfer in the cooling coil predicted by the wet coil/dry coil method is the sum of
the heat transferred in the wet and dry sections:

q = qdc + q wc (28)

The guess value of the outlet quality is commented out and the total rate of heat transfer
calculated.

{x_r_out=0.95 [-]} "guess value of exit quality"


"coil performance"
q_dot=q_dot_dc+q_dot_wc "total heat transfer rate using wet coil/dry coil method"

The mass flow rate of condensate is:

m cond = m a (ωa ,in − ωa ,out ) (29)

m_dot_cond=m_dot_a*(omega_a_in-omega_a_out) "mass flow rate of condensate"


m_dot_cond_kgph=m_dot_cond*convert(kg/s,kg/hr) "in kg/hr"

which leads to q = 2.21 kW and m cond = 0.000494 kg/s (1.78 kg/hr).


Problem 9.6-2 (9-12 in text)
Air enters a cooling coil with volumetric flow rate 20,000 cfm, temperature 90°F and 50%
relative humidity and is cooled and dehumidified by heat exchange with chilled water that enters
the cooling coil with a mass flow rate of 80,000 lbm/hr and a temperature of 45°F. The total
thermal resistance on the water-side of the heat exchanger is 4.44x10-6 hr-°F/Btu. The air-side is
finned and the total thermal resistance on the air side, including the effect of the fins, ranges
from 1.0x10-5 hr-°F/Btu when the coil is completely dry to 3.33x10-6 hr-°F/Btu when the coil is
completely wet. The coil is large and employs many rows of tubes so that a counterflow heat
transfer analysis is appropriate. Use the Dry Coil/Wet Coil analysis that is described in Section
9.6.2.
a.) Estimate the fraction of the coil that is wetted.
b.) Determine the heat transfer rate between the chilled water and the air.
c.) Determine the outlet air temperature.
d.) Determine the rate of condensate.
e.) Determine the outlet temperature of the water.

The inputs are entered in EES:

$UnitSystem SI MASS RAD PA K J


$Tabstops 0.2 0.4 0.6 3.5 in

"known:"
V_dot_a=20000 [cfm]*convert(cfm,m^3/s) "volumetric flow rate of air"
p_a=14.7 [psia]*convert(psia,Pa) "atmospheric pressure"
T_a_in=converttemp(F,K,90[F]) "inlet air temperature"
RH_a_in=0.50 [-] "inlet air relative humidity"
m_dot_w=80000 [lbm/hr]*convert(lbm/hr,kg/s) "water flow rate"
T_w_in=converttemp(F,K,45 [F]) "inlet water temperature"
p_w=p_a "pressure of the water"
R_w=4.44e-6 [hr-F/Btu]*convert(hr-F/Btu,K/W) "thermal resistance on water side"
R_a_dry=1e-5 [hr-F/Btu]*convert(hr-F/Btu,K/W) "thermal resistance on air side when dry"
R_a_wet=3.33e-6 [hr-F/Btu]*convert(hr-F/Btu,K/W) "thermal resistance on air side when wet"

The inlet humidity ratio (ωa,in), density (ρa,in), and the dewpoint associated with the incoming air
(Tdp, the temperature at which the relative humidity reaches 100% and therefore water will begin
to condense) are computed:

T_dp=DewPoint(AirH2O,P=p_a,T=T_a_in,R=RH_a_in) "dewpoint temperature of air"


omega_a_in=HumRat(AirH2O,P=p_a,T=T_a_in,R=RH_a_in) "humidity ratio at inlet"
rho_a_in=density(AirH2O,P=p_a,T=T_a_in,R=RH_a_in) "density of inlet air"

which leads to Tdp = 293.6 K. Because the entering water temperature is less than Tdp, water will
condense onto the coil surface provided that the cooling coil is sufficiently large. The mass flow
rate of air is:

m a = Va ρ a ,in (1)

m_dot_a=V_dot_a*rho_a_in "mass flow rate of dry air into coil"


The outlet temperature of the water (Tw,out) is initially guessed; this guess will be removed later.
The temperature of the air leaving the dry coil (Ta,dc,out) is the dew point temperature.

Ta ,dc ,out = Tdp (2)

The specific heat capacity of the moist-air on a per mass of dry air basis ( ca′′ ) is calculated using
EES' internal psychrometric function. The capacitance rate of the air is:

C a , dc = m a ca′′ (3)

The capacitance rate of the water is:

C w = m w cw (4)

where cw is the specific heat capacity of the water evaluated at the average temperature.

"Dry coil analysis"


T_w_out=converttemp(F,K,55[F]) "guess for the outlet water temperature - remove later"
T_a_dc_out=T_dp "temperature of air at exit of dry coil"
c``_a=cp(AirH2O,P=P_a,T=T_a_in,R=RH_a_in) "specific heat capacity of moist air in dry coil"
C_dot_a_dc=m_dot_a*c``_a "capacity rate of air"
T_w_avg=(T_w_in+T_w_out)/2 "average water temperature"
c_w=cP(Water,P=P_w,T=T_w_avg) "specific heat capacity of water"
C_dot_w=m_dot_w*c_w "capacity rate of water"

The rate of heat transfer in the dry coil section of the heat exchanger is given by:

qdc = C a ,dc (Ta ,in − Ta ,dc ,out ) (5)

The minimum capacitance rate in the dry coil ( C min ,dc ) is identified using the Min command. The
temperature of the water entering the dry coil is estimated using an energy balance:

q
Tw,dc ,in = Tw,out − dc (6)
Cw

The maximum possible rate of heat transfer in the dry coil is:

qdc ,max = C min,dc (Ta ,in − Tw, dc ,in ) (7)

and the effectiveness of the dry coil is:

qdc
ε dc = (8)
qdc ,max
q_dot_dc=C_dot_a_dc*(T_a_in-T_a_dc_out) "heat transfer rate in dry coil"
C_dot_min_dc=MIN(C_dot_a_dc,C_dot_w) "minimum heat capacity rate in dry coil"
T_w_dc_in=T_w_out-q_dot_dc/C_dot_w "temperature of water entering dry coil section"
q_dot_dc_max=C_dot_min_dc*(T_a_in-T_w_dc_in)
"maximum possible heat transfer rate in dry coil - countercurrent operation assumed"
eff_dc=q_dot_dc/q_dot_dc_max "effectiveness of dry coil"

The number of transfer units in the dry coil section (NTUdc) is obtained using the ε-NTU solution
for a counter-flow heat exchanger accessed using the HX function in EES. The conductance
required in the dry-coil section is:

UAdc = NTU dc C min ,dc (9)

The conductance is used to compute the fraction of the total heat exchanger required by the dry
coil:

Fdc
UAdc = (10)
Rw + Ra ,dry

NTU_dc=HX('counterflow', eff_dc, C_dot_a_dc, C_dot_w, 'NTU') "number of transfer units in dry coil"
UA_dc=NTU_dc*C_dot_min_dc "conductance in dry coil"
UA_dc=F_dc/(R_w+R_a_dry) "determine fraction of coil that is dry"

which leads to Fdc = 0.241 (i.e., 24% of the coil is dry, according to this analysis). The
remainder of the heat exchanger coil is wet:

Fwc = 1 − Fdc (11)

"Wet coil analysis"


F_wc=1-F_dc "fraction of heat exchanger that is wet"

The wet coil is analyzed using the concepts of a saturation specific heat capacity and wet coil
conductance. The saturation specific heat capacity is used to account for the latent heat
associated with the condensing water vapor and is calculated according to:

⎡ia′′ (Tdc ,out , p,RH = 1) − ia′′ (Ta ,out , p,RH = 1) ⎤⎦


ca′′,sat = ⎣ (12)
(Tdc ,out − Ta ,out )
Initially, a value for Ta,out is assumed; this value is subsequently commented out in order to close
the solution:

T_a_out=T_w_in "an initial assumption for the saturation specific heat capacity"
c``_a_sat=(enthalpy(AirH2O,T=T_a_dc_out,R=1,P=p_a)-&
enthalpy(AirH2O,T=T_a_out,R=1,P=p_a))/(T_a_dc_out-T_a_out)
"saturation specific heat capacity"
The capacitance rate of the air in the wet coil is:

C a , wc = m a ca′′, sat (13)

The minimum capacitance rate in the wet coil ( C min , wc ) is identified. The conductance in the wet
coil is calculated according to:

Fwc
UAwc = (14)
Rw + Ra , wet

The number of transfer units in the wet coil is:

UA
NTU wc =  wc (15)
Ca , wc

The effectiveness of the wet coil (εwc) is obtained using the ε-NTU solution for a counter-flow
heat exchanger, obtained using the HX function in EES:

C_dot_a_wc=m_dot_a*c``_a_sat "capacity rate of air in wet coil"


C_dot_min_wc=MIN(C_dot_a_wc,C_dot_w) "minimum heat capacity rate in wet coil"
UA_wc=F_wc/(R_w+R_a_wet) "wet coil conductance"
NTU_wc=UA_wc/C_dot_min_wc "number of transfer units in the wet coil"
eff_wc=HX('counterflow', NTU_wc, C_dot_a_wc, C_dot_w, 'epsilon') "effectiveness of wet coil"

The heat transfer rate in the wet coil section is obtained from the effectiveness:

qwc = ε wc C a , wc (Tdc ,out − Tr ,in ) (16)

q_dot_wc=eff_wc*C_dot_min_wc*(T_a_dc_out-T_w_in) "heat transfer rate in wet coil"

The temperature of the air leaving the wet coil is obtained using an energy balance on the air-
side:

q
Ta ,out = Tdc ,out −  wc (17)
Ca , wc

Before calculating the value of Ta,out, the guess values for the problem are updated (select Update
Guesses from the Calculate menu) and the initial guess for Ta,out is commented out:

{T_a_out=T_w_in} "an initial assumption for the saturation specific heat capacity"
T_a_out=T_a_dc_out-q_dot_wc/C_dot_a_wc
"temperature of air leaving wet coil (comment out T_a_out equation above)"
T_a_out_F=converttemp(K,F,T_a_out) "in F"
The wet coil model assumes that the air leaves saturated (i.e., at 100% relative humidity) and
therefore the humidity ratio of the air leaving the heat exchanger (ωa,out) can be obtained using
EES' psychrometric functions.

omega_a_out=HumRat(AirH2O,T=T_a_out,R=1,P=p_a) "humidity ratio at exit"

The total heat transfer in the cooling coil predicted by the wet coil/dry coil method is the sum of
the heat transferred in the wet and dry sections:

q = qdc + q wc (18)

The guess value of the water outlet temperature is commented out and recalculated using an
energy balance:

q
Tw,out = Tw,in + (19)
C w

{T_w_out=converttemp(F,K,55[F])} "guess for the outlet water temperature - remove later"

"coil performance"
q_dot=q_dot_dc+q_dot_wc "total heat transfer rate using wet coil/dry coil method"
T_w_out=T_w_in+q_dot/C_dot_w "recalculate water outlet temperature"
T_w_out_F=converttemp(K,F,T_w_out) "in F"

The mass flow rate of condensate is:

m cond = m a (ωa ,in − ωa ,out ) (20)

m_dot_cond=m_dot_a*(omega_a_in-omega_a_out) "mass flow rate of condensate"

which leads to Fwc = 0.670, q = 394.6 kW, Ta,out = 53.7°F, m cond = 0.0681 kg/s, and Tw,out =
61.8°F.
Problem 9.6-3 (9-13 in text)
Repeat Problem 9.6-2 using the enthalpy-effectiveness method described in Section 9.6.3.

The inputs are entered in EES:

$UnitSystem SI MASS RAD PA K J


$Tabstops 0.2 0.4 0.6 3.5 in

"known:"
V_dot_a=20000 [cfm]*convert(cfm,m^3/s) "volumetric flow rate of air"
p_a=14.7 [psia]*convert(psia,Pa) "atmospheric pressure"
T_a_in=converttemp(F,K,90[F]) "inlet air temperature"
RH_a_in=0.50 [-] "inlet air relative humidity"
m_dot_w=80000 [lbm/hr]*convert(lbm/hr,kg/s) "water flow rate"
T_w_in=converttemp(F,K,45 [F]) "inlet water temperature"
p_w=p_a "pressure of the water"
R_w=4.44e-6 [hr-F/Btu]*convert(hr-F/Btu,K/W) "thermal resistance on water side"
R_a_dry=1e-5 [hr-F/Btu]*convert(hr-F/Btu,K/W) "thermal resistance on air side when dry"
R_a_wet=3.33e-6 [hr-F/Btu]*convert(hr-F/Btu,K/W) "thermal resistance on air side when wet"

The inlet humidity ratio (ωa,in), density (ρa,in), and the dewpoint associated with the incoming air
(Tdp, the temperature at which the relative humidity reaches 100% and therefore water will begin
to condense) are computed:

T_dp=DewPoint(AirH2O,P=p_a,T=T_a_in,R=RH_a_in) "dewpoint temperature of air"


omega_a_in=HumRat(AirH2O,P=p_a,T=T_a_in,R=RH_a_in) "humidity ratio at inlet"
rho_a_in=density(AirH2O,P=p_a,T=T_a_in,R=RH_a_in) "density of inlet air"

which leads to Tdp = 293.6 K. Because the entering water temperature is less than Tdp, water will
condense onto the coil surface provided that the cooling coil is sufficiently large. The mass flow
rate of air is:

m a = Va ρ a ,in (1)

m_dot_a=V_dot_a*rho_a_in "mass flow rate of dry air into coil"

The outlet temperature of the water (Tw,out) is initially guessed; this guess will be removed later.
The capacitance rate of the water is:

C w = m w cw (2)

where cw is the specific heat capacity of the water evaluated at the average temperature.

T_w_out=converttemp(F,K,55[F]) "guess for the outlet water temperature - remove later"


T_w_avg=(T_w_in+T_w_out)/2 "average water temperature"
c_w=cP(Water,P=P_w,T=T_w_avg) "specific heat capacity of water"
C_dot_w=m_dot_w*c_w "capacity rate of water"

The saturation specific heat capacity of the air is computed according to:
⎡ia′′ (Ta ,in , p,RH = 1) − ia′′ (Ta ,out , p,RH = 1) ⎤⎦
ca′′,sat = ⎣ (3)
(Ta ,in − Ta ,out )
where Ta,out is the air outlet temperature; it is necessary to assume a value for Ta,out in order to
compute ca′′, sat - this assumed value will be commented out in order to complete the solution:

T_a_out=T_w_in "guess for T_a_out"


c``_a_sat=(enthalpy(AirH2O,T=T_a_in,R=1,P=p_a)-&
enthalpy(AirH2O,T=T_a_out,R=1,P=p_a))/(T_a_in-T_a_out) "saturation specific heat capacity"

The capacitance rate of the air is:

C a = ca′′, sat m a (4)

The minimum capacitance rate ( C min ) is determined using the Min function.

C_dot_a=m_dot_a*c``_a_sat "capacity rate of air in wet coil"


C_dot_min=MIN(C_dot_a,C_dot_w) "minimum heat capacity rate in wet coil"

The enthalpy-based effectiveness based effectiveness technique assumes that the total coil is wet.
Therefore, the conductance of the coil is:

1
UA = (5)
Ra , wet + Rw

The number of transfer units is:

UA
NTU = (6)
C min

and the effectiveness of the coil (ε) is computed using the ε-NTU solution for a counterflow heat
exchanger, accessed using the HX function in EES.

NTU=UA/C_dot_min "number of transfer units if all coil is wet"


eff=HX('counterflow', NTU, C_dot_a, C_dot_w, 'epsilon') "effectiveness"

The enthalpy of the inlet air ( ia′′,in ) and the minimum possible enthalpy and relative humidity of
the outlet air ( ia′′,out ,min and ωout,min) are computed using EES' psychrometric functions. The
effectiveness is used to predict the actual outlet air enthalpy ( ia′′,out ,ebe ) and relative humidity
(ωa,out,ebe) according to:
ia′′,in − ia′′,out
ε= (7)
ia′′,in − ia′′,out , min

ωa ,in − ωa ,out
ε= (8)
ωa ,in − ωa ,out ,min

i``_a_in=enthalpy(AirH2O,T=T_a_in,R=RH_a_in,P=p_a) "inlet enthalpy"


i``_a_out_min=enthalpy(AirH2O,T=T_w_in,R=1,P=p_a) "minimum possible air exit enthalpy"
omega_a_out_min=HumRat(AirH2O,T=T_w_in,R=1,p=p_a) "minimum possible air exit humidity ratio"
eff=(i``_a_in-i``_a_out)/(i``_a_in-i``_a_out_min) "enthalpy-based effectiveness"
eff=(omega_a_in-omega_a_out)/(omega_a_in-omega_a_out_min) "humidity ratio of exit air"

The outlet temperature of the air (Ta,out) can be predicted using the enthalpy and humidity ratio
with EES' psychrometric function. The guess values are updated and the initial, assumed value
for Ta,out is commented out:

{T_a_out=T_w_in} "guess for T_a_out"


T_a_out=temperature(AirH2O,h=i``_a_out,w=omega_a_out,P=p_a) "outlet temperature of air"
T_a_out_F=converttemp(K,F,T_a_out) "in F"

The rate of heat transfer in the cooling coil predicted using the enthalpy-based effectiveness
approach is:

q = m a ( ia′′,in − ia′′,out ) (9)

and the rate of condensation is:

m cond = m a (ωa ,in − ωa ,out ) (10)

q_dot=m_dot_a*(i``_a_in-i``_a_out) "heat transfer rate using enthalpy-based effectiveness method"


m_dot_cond=m_dot_a*(omega_a_in-omega_a_out)
"mass flow rate of condensate using enthalpy-based effectiveness method"

The guess value of the water inlet temperature is commented out and the water inlet temperature
is recalculated according to:

q
Tw,out = Tw,in +  (11)
Cw

{T_w_out=converttemp(F,K,55[F])} "guess for the outlet water temperature - remove later"


T_w_out=T_w_in+q_dot/C_dot_w "water outlet temperature"
T_w_out_F=converttemp(K,F,T_w_out) "in F"

which leads to q = 321.2 kW, Ta,out = 62.0°F, m cond = 0.059 kg/s, and Tw,out = 58.7°F. These
results can be compared with the answers from Problem 9.6-2 (9-12 in the text), q = 394.6 kW,
Ta,out = 53.7°F, m cond = 0.0681 kg/s, and Tw,out = 61.8°F. The difference in the predictions
obtained using the wet coil/dry coil and enthalpy based effectiveness methods are approximately
20%.
Problem 9.6-4 (9-14 in text): Cooling Tower Analysis
Cooling towers are direct-contact heat and mass exchangers. The performance of a cooling
tower can be analyzed using the enthalpy-based effectiveness method described in Section 9.6.3.
In this case, the maximum rate of heat transfer between the air and water is based on the
difference between the enthalpy of the inlet air and the enthalpy of saturated air exiting at the
inlet water temperature. The saturation specific heat should be evaluated using the enthalpies of
saturated air at the water inlet and air inlet temperatures, respectively. A steady flow of water
enters an induced draft cooling tower with a mass flow rate of 15 kg/s and a temperature of
35°C. The fans provide 4.72 m3/s of ambient air at a dry-bulb temperature of 23°C and a relative
humidity of 50%. Makeup water is supplied at 25°C. Use the enthalpy-based effectiveness
technique to analyze this cooling tower.
a.) Prepare a plot of the outlet water temperature and the rate of water loss as a function of the
number of transfer units associated with the cooling tower for NTU values between 0.5 and 5.

The inputs are entered in EES:

$UnitSystem SI MASS RAD PA K J


$Tabstops 0.2 0.4 0.6 3.5 in

"known information"
T_w_in = converttemp(C,K,35 [C]) "water inlet temperature"
m_dot_w_in=15 [kg/s] "mass flow rate of water at inlet"
T_a_in=converttemp(C,K,23 [C]) "air temperature at inlet to tower"
RH_a_in = 0.50 [-] "relative humidity of air at tower inlet"
V_dot_a = 4.72 [m^3/s] "volumetric flow of air at tower inlet"
P_atm = 101300 [Pa] "atmospheric pressure"
T_makeup=converttemp(C,K,25 [C]) "temperature of make-up water supply"
NTU=0.5 [-] "number of transfer units"

The humidity ratio, specific enthalpy, and density of the inlet air (ωa,in, ia,in, and ρa,in) are
calculated. The mass flow rate of air is computed according to:

m a = Va ρ a ,in (1)

omega_a_in = HumRat(AirH2O,T=T_a_in, P=P_atm,R=RH_a_in)"humidity ratio of the inlet air"


i_a_in= Enthalpy(AirH2O,T=T_a_in,P=P_atm,R=RH_a_in) "specific enthalpy of the inlet air"
rho_a_in=Density(AirH2O,T=T_a_in,P=P_atm,R=RH_a_in) "density of the inlet air"
m_dot_a = V_dot_a*rho_a_in "mass flow rate of dry air through tower"

The outlet air temperature (Ta,out) is assumed. The saturation specific heat capacity of the air is
computed according to:

⎡ia′′ (Ta ,in , p,RH = 1) − ia′′ (Ta ,out , p,RH = 1) ⎤⎦


ca′′,sat = ⎣ (2)
(Ta ,in − Ta ,out )
T_a_out=T_w_in "guess for the air outlet temperature"
i_a_sat_in=enthalpy(AirH2O,T=T_a_in, P=P_atm,R=1) "saturated specific enthalpy at T_air_in"
i_a_sat_out=enthalpy(AirH2O,T=T_a_out, P=P_atm,R=1) "saturated specific enthalpy at T_a_in"
c_sat = (i_a_sat_out -i_a_sat_in)/(T_a_out - T_a_in) "saturation specific heat"

The capacitance rate of the air is:

C a = ca′′, sat m a (3)

C_dot_a=c_sat*m_dot_a "capacitance rate of the air"

The capacitance rate of the water is:

C w = m w cw (4)

where cw is the specific heat capacity of the water evaluated at the inlet water temperature.

c_w = SpecHeat(Steam,T=T_w_in,P=P_atm) "specific heat of liquid water"


C_dot_w=c_w*m_dot_w_in "capacitance rate of the water"

The effectiveness of the cooling tower (ε) is obtained using the HX function in EES.

epsilon=HX('counterflow', NTU, C_dot_a, C_dot_w, 'epsilon') "epsilon from HX library"

The enthalpy and humidity ratio of saturated air at the water inlet temperature (ia,sat,Twin and
ωa,sat,Twin) are computed. The effectiveness is used to compute the enthalpy and relative humidity
of the exiting air:

ia ,out − ia ,in
ε= (5)
ia , sat ,Twin − ia ,in

ωa ,out − ωa ,in
ε= (6)
ωa , sat ,Twin − ωa ,in

i_a_sat_T_w_in=enthalpy(AirH2O,T=T_w_in, P=P_atm,R=1) "saturated specific enthalpy at T_w_in"


omega_a_sat_T_w_in=HumRat(AirH2O,T=T_w_in, P=P_atm,R=1)"saturated specific enthalpy at T_w_in"
epsilon=(i_a_out-i_a_in)/(i_a_sat_T_w_in-i_a_in) "effectiveness applied to enthalpy"
epsilon=(omega_a_out-omega_a_in)/(omega_a_sat_T_w_in-omega_a_in)
"effectiveness applied to humidity ratio"

The guess values are updated. The outlet air temperature that was assumed is commented out
and the outlet air temperature and relative humidity are computed using the enthalpy and
humidity ratio.

{T_a_out=T_w_in} "guess for the air outlet temperature"


T_a_out=Temperature(AirH2O,w=omega_a_out,h=i_a_out,P=P_atm) "get temperature of exiting air"
T_a_out_C=converttemp(K,C,T_a_out) "in C"
RH_a_out=RelHum(AirH2O,w=omega_a_out,h=i_a_out,P=P_atm) "get relative humidity of exiting air"
The specific enthalpy of the water entering the cooling tower and the makeup water (iw,in and
iw,makeup) are computed. The mass flow rate of makeup water is obtained from:

m w,makeup = m a (ωa ,out − ωa ,in ) (7)

i_w_in= Enthalpy(Water,T=T_w_in,P=P_atm) "specific enthalpy of the entering water"


i_w_makeup=Enthalpy(Water,T=T_makeup,P=P_atm) "specific enthalpy of the make-up water"
m_dot_makeup = m_dot_a*(omega_a_out - omega_a_in) "water balance"

The mass flow rate of water leaving the cooling tower is obtained with a mass balance on the
water:

m w,out = m w,in − m w, makeup (8)

The enthalpy of the water leaving the cooling tower is obtained with an energy balance:

m a ( ia ,in − ia ,out ) + m w,in iw,in − m w,out iw,out + m w,makeup iw, makeup = 0 (9)

m_dot_w_out = m_dot_w_in - m_dot_makeup "definition of make-up water flow rate"


m_dot_a*(i_a_in-i_a_out)+m_dot_w_in*i_w_in-m_dot_w_out*i_w_out+m_dot_makeup*i_w_makeup=0
"energy balance"

The outlet water temperature (Tw,out) is computed from the enthalpy.

T_w_out=temperature(Water,h=i_w_out,P=P_atm) "outlet water temperature"


T_w_out_C=converttemp(K,C,T_w_out) "in C"

Figure 1 illustrates the temperature of water leaving the cooling tower and the mass flow rate of
the makeup water as a function of the number of transfer units.
32.5 0.16
Mass flow rate of makeup water (kg/s)

32
0.14
31.5
makeup water flow rate
31 0.12
Temperature (°C)

30.5
0.1
30

29.5 0.08

29
water outlet temperature 0.06
28.5

28 0.04
0.5 1 1.5 2 2.5 3 3.5 4 4.5 5
Number of transfer units
Figure 1: Outlet water temperature and makeup water flow rate as a function of NTU.
b.) Plot the range and approach as a function of NTU. The range is the difference between the
inlet and outlet water temperatures. The approach is the difference between the outlet water
temperature and the wet bulb temperature.

The range is calculated according to:

Range = Tw,in − Tw,out (10)

and the approach is:

Approach = Tw,out − Twetbulb (11)

where Twetbulb is the wet bulb temperature, computed using the WetBulb function in EES.

Range=T_w_in-T_w_out "range of cooling tower"


T_wetbulb = WetBulb(AirH2O,T=T_a_in,P=P_atm,R=RH_a_in)
"wetbulb temperature of the air entering tower"
Approach=T_w_out-T_wetbulb "approach temperature difference"

Figure 2 illustrates the range and approach as a function of the number of transfer units.
18

16

14
Range and approach (K)

12

10

2
0.5 1 1.5 2 2.5 3 3.5 4 4.5 5
Number of transfer units
Figure 2: Range and approach as a function of NTU.

c.) Compare the rate of heat transfer associated with the cooling tower to the heat transfer rate
that would be achieved by an air-cooled dry heat exchanger with the same air flow rate and
NTU.

The rate of cooling provided by the cooling tower is:

q = C w (Tw,in − Tw,out ) (12)

q_dot=C_dot_w*(T_w_in-T_w_out) "heat transfer from the water"


The specific heat capacity of dry air (ca) is computed and used to determine the capacitance rate
of dry air:

C a ,dry = m a ca (13)

The effectiveness of the dry coil (εdry) is obtained from the HX function. The maximum possible
heat transfer rate for the dry coil is:

qdry ,max = C dry ,min (Tw,in − Ta ,in ) (14)

where C dry , min is the minimum of the dry air capacitance rate and the water capacitance rate. The
actual dry coil heat transfer rate is:

qdry = ε dry qdry ,max (15)

c_a=cp(AirH2O,T=T_a_in, P=P_atm,R=RH_a_in) "specific heat of air"


C_dot_a_dry=m_dot_a*c_a "capacitance rate of air"
epsilon_dry=HX('counterflow', NTU, C_dot_a_dry, C_dot_w, 'epsilon') "epsilon from HX library"
q_dot_max_dry=min( C_dot_a_dry, C_dot_w)*(T_w_in - T_a_in) "maximum possible rate of heat transfer"
q_dot_dry=epsilon_dry*q_dot_max_dry "heat transfer rate associated with a dry heat exchanger"

Figure 3 illustrates the heat transfer rate as a function of number of transfer units for the cooling
tower and the air cooled dry coil. The evaporation within the cooling tower leads to much higher
cooling rates.

4.5x105

4.0x105
cooling tower
3.5x105
Rate of heat transfer (W)

3.0x105

2.5x105

2.0x105

1.5x105

1.0x105 air cooled dry heat exchanger

5.0x104

0.0x100
0.5 1 1.5 2 2.5 3 3.5 4 4.5 5
Number of transfer units
Figure 3: Rate of heat transfer provided by the cooling tower and the air cooled dry heat exchanger as a
function of the number of transfer units.
Problem 9.6-5
A cooling coil with a cross-flow geometry is used to dehumidify a stream of air. The flow rate
of air is Va = 750 ft3/min and the air enters at Ta,in = 75°F and relative humidity RHa,in = 0.75.
Chilled water passes through the cooling coil with flow rate V = 5.1 gal/min and temperature
w
Tw,in = 45°F. The conductance of the coil under dry conditions is UAdry = 700 Btu/hr-R.
a.) Determine the outlet temperature of the air and the mass flow rate of condensate.

The inputs are entered in EES:

$UnitSystem English psi F


$Tabstops 0.2 0.4 0.6 3.5 in

V_dot_a_cfm=750 [ft^3/min] "volumetric flow rate of air, in ft^3/min"


V_dot_a=V_dot_a_cfm*convert(ft^3/min,ft^3/hr) "volumetric flow rate of air"
RH_a_in=0.75 [-] "relative humidity of air"
T_a_in=75 [F] "air inlet temperature"
T_w_in=45 [F] "chilled water inlet temperature"
V_dot_w=5.1 [gal/min]*convert(gal/min,ft^3/hr) "volumetric flow rate of water"
UA_dry=700 [Btu/hr-R] "dry-coil conductance"

The humidity ratio (ωa,in) and enthalpy ( ia′′,in ) are determined from the inlet temperature and
relative humidity.

omega_a_in=HumRat(AirH2O,T=T_a_in,P=14.7 [psi],R=1 [-]) "air inlet humidity ratio"


h_a_in=Enthalpy(AirH2O,T=T_a_in,P=14.7 [psi],R=RH_a_in) "inlet enthalpy"

The mass flow rate of air is computed according to:

m a = ρ a Va (1)

where ρa is the density of the air. The capacitance rate of the water is computed according to:

C w = ρ w cw Vw (2)

where ρw and cw are the density and specific heat capacity of the water, respectively.

rho_a=density(Air,T=T_a_in,P=14.7 [psi]) "density of air"


m_dot_a=rho_a*V_dot_a "mass flow rate of air"
rho_w=density(Water,T=T_w_in,P=14.7 [psi]) "density of water"
c_w=cP(Water,T=T_w_in,P=14.7 [psi]) "specific heat capacity of water"
C_dot_w=V_dot_w*rho_w*c_w "capacitance rate of water"

The saturation specific heat capacity is estimated according to:

⎡ia′′,in − ia′′,out ,min ⎤⎦


ca′′,sat ≈ ⎣ (9-3)
(Ta ,in − Ta ,out ,min )
where Ta,out,,min is the temperature of the entering water and ia′′,out ,min is the enthalpy of the air-
water mixture at Ta,out,min and 100% relative humidity.

c_a_sat=(enthalpy(AirH2O,T=T_a_in,P=14.7 [psi],R=1 [-])-&


enthalpy(AirH2O,T=T_w_in,P=14.7 [psi],R=1 [-]))/(T_a_in-T_w_in)
"saturation specific heat capacity"

The effective capacitance rate of the air is:

C a = ca′′,sat m a (9-4)

The minimum capacitance rate, C min , is identified using the Min command.

C_dot_a=c_a_sat*m_dot_a "capacitance rate of air"


C_dot_min=MIN(C_dot_a,C_dot_w) "minimum capacitance rate"

The effective conductance is computed according to:

ca′′,sat
UA = UAdry (9-5)
ca

UA=UA_dry*c_a_sat/cP(Air,T=T_a_in) "augmented conductance"

The number of transfer units is computed according to:

UA
NTU =  (9-6)
Cmin

The effectiveness, ε, is obtained using the ε-NTU solution. The minimum possible enthalpy and
humidity ratio, ia′′,out ,min and ωa,out,min, are computed. The enthalpy-based effectiveness equations:

ia′′,in − ia′′,out
ε= (9-7)
ia′′,in − ia′′,out , min

ωa ,in − ωa ,out
ε= (9-8)
ωa ,in − ωa ,out ,min

NTU=UA/C_dot_min "number of transfer units"


eff=HX('crossflow_both_unmixed', NTU, C_dot_a, C_dot_w, 'epsilon') "effectiveness"
h_a_out_min=Enthalpy(AirH2O,T=T_w_in,P=14.7 [psi],R=1 [-])
"minimum possible outlet enthalpy"
omega_a_out_min=HumRat(AirH2O,T=T_w_in,P=14.7 [psi],R=1 [-])
"minimum possible outlet humidity ratio"
eff=(h_a_in-h_a_out)/(h_a_in-h_a_out_min) "enthalpy at outlet"
eff=(omega_a_in-omega_a_out)/(omega_a_in-omega_a_out_min) "humidity ratio at outlet"

The outlet temperature, Ta,out, is computed based on the enthalpy and humidity ratio. The rate of
condensate is computed from a mass balance on the water in the air:

m c = m a (ωa ,in − ωa ,out ) (9-9)

T_a_out=temperature(AirH2O,h=h_a_out,w=omega_a_out,P=14.7 [psi]) "outlet air temperature"


m_dot_c=m_dot_a*(omega_a_in-omega_a_out) "condensate rate"

which leads to Ta,out = 49.96°F and m c = 18.68 lbm/hr.

b.) Plot the air outlet temperature and the mass flow rate of condensate as a function of the dry-
coil conductance of the coil.

Figure 1 illustrates the air outlet temperature and mass flow rate of condensate as a function of
UAdry.

53 30

Mass flow rate of condensate (lbm/hr)


52 25
Air outlet temperature (°F)

mc
51 20

50 15

49 Ta,out 10

48 5

47 0
0 250 500 750 1000 1250 1500 1750 2000
Dry coil conductance (Btu/hr-R)
Figure 1: Air outlet temperature and mass flow rate of condensate as a function of the dry-coil conductance.
Problem 9.1-1 (9-1 in text)

A mixture is formed mixing Mm = 0.25 kg of methane (with molar mass MWm = 16 kg/kgmol),
Me = 0.15 kg of ethane (MWe = 30 kg/kgmol) and Mn = 0.1 kg of nitrogen (MWn = 28 kg/kgmol).
The mixture is placed in a container that is maintained at T = 25°C and p = 5 bar. At these
conditions, the mixture behaves in accordance with the ideal gas law. Determine:
a.) the volume of the mixture

The known information is entered into EES. Array variables are employed to facilitate the
calculations.

$UnitSystem SI MASS RAD PA K J


$Tabstops 0.2 0.4 0.6 3.5 in

"known information"
NGases=3 [-] "number of gases in the mixture"
N$[1]='methane'; m[1]=0.25 [kg] "name and mass of gas #1, methane"
N$[2]='ethane'; m[2]=0.15 [kg] "name and mass of gas #2, methane"
N$[3]='nitrogen'; m[3]=0.1 [kg] "name and mass of gas #3, methane"
P=5 [bar]*convert(bar,Pa) "pressure"
T=converttemp(C,K,25 [C]) "temperature"

The molarmass of each constituent (MWi) is obtained using the MolarMass function. The number
of moles of each constituent is:

Mi
Ni = for i = 1..N gas (1)
MWi

where Ngas is the number of constituents. The total number of moles is:

N gas

N = ∑ Ni (2)
i =1

and the total volume is obtained using the ideal gas law:

PV = Ru N T (3)

where Ru is the universal gas constant.

"a) determine the volume of the mixture"


duplicate i=1,NGases
MW[i]=molarMass(N$[i]) "molecular weight of each gas"
N[i]=m[i]/MW[i] "number of moles of each gas"
end
N_tot=sum(N[1..NGases]) "total number of moles"
P*V=N_tot*R#*T "ideal gas law to determine volume"

which leads to V = 0.120 m3.


b.) the equivalent molecular weight of the mixture

The total mass of the mixture is:

N gas

M = ∑ Mi (4)
i =1

and the equivalent molar mass is:

M
MW = (5)
N

"b) density on mass basis"


M_tot=sum(m[1..NGases]) "total mass of the mixture"
MW=M_tot/N_tot "equivalent molar mass"

which leads to MW = 20.71 kg/kgmol.

c.) the density of the mixture on a mass basis

The density of the mixture on a mass basis is:

M
ρ= (6)
V

"c) density on molar basis"


rho=M_tot/V "mass density"

where ρ = 4.18 kg/m3.

d.) the density of the mixture on a molar basis

The density of the mixture on a molar basis is:

N
n= (7)
V

"d) density on molar basis"


molardensity=N_tot/V "molar density"

which leads to n = 0.202 kgmol/m3.

e.) the mass fractions of each species

The mass fractions of each species are computed according to:


Mi
mf i = for i = 1..N gas (8)
M

"e) mass fraction"


duplicate i=1,NGases
mf[i]=m[i]/M_tot "mass fraction"
end

which leads to mfm = 0.5, mfe = 0.3, and mfn = 0.2.

f.) the mole fractions of each species

The mole fractions of each species are computed according to:

Ni
yi = for i = 1..N gas (9)
N

"f) mole fraction"


duplicate i=1,NGases
y[i]=n[i]/N_tot "mole fraction"
end

which leads to ym = 0.646, ye = 0.207, and yn = 0.148.

g.) the mass concentrations of each species

The mass concentration of each species are computed according to:

Mi
ci = for i = 1..N gas (10)
V

"g) mass concentration"


duplicate i=1,NGases
c[i]=m[i]/V "mass concentration"
end

which leads to cm = 2.089 kg/m3, ce = 1.253 kg/m3, and cn = 0.836 kg/m3.

h.) the molar concentration of each species

The molar concentrations of each species are computed according to:

Ni
ni = (11)
V

"h) molar concentration"


duplicate i=1,NGases
nn[i]=N[i]/V "molar concentration"
end

which leads to nm = 0.130 kgmole/m3, ne = 0.0417 kgmol/m3, and nn = 0.0298 kgmol/m3.


Problem 9.1-2 (9-2 in text)
The composition of mixtures of air and water vapor are often reported in terms of the humidity
ratio. The humidity ratio, ω, is defined as the mass of water vapor per mass of dry air. The
humidity ratio is related to, but not exactly the same as the the mass fraction. In a particular
case, the humidity ratio is ω = 0.0078 at temperature T = 30°C and pressure p = 101.3 kPa.
Determine:
a.) the mass fraction of the water vapor

The inputs are entered in EES:

$UnitSystem SI MASS RAD PA K J


$Tabstops 0.2 0.4 0.6 3.5 in

omega=0.0078 [-] "humidity ratio"


T=converttemp(C,K,30 [C]) "temperature"
P=101.3 [kPa]*convert(kPa,Pa) "pressure"

The problem is carried out using an arbitrary mass of dry air (ma). The mass of vapor is
computed using the definition of the humidity ratio:

mv
ω= (1)
ma

The mass fraction of water vapor is:

mv
mf v = (2)
ma + mv

m_a=1[kg] "arbitrary mass of dry air"


omega=m_v/m_a "definition of humidity ratio"
mf_v=m_v/(m_a+m_v) "mass fraction of vapor"

which leads to mfv = 0.00774.

b.) the mole fraction of the water vapor

The number of moles of air and water vapor are computed according to:

mv
Nv = (3)
MWv

ma
Na = (4)
MWa

where MWv and MWa are the molar weights of the water vapor and air, respectively, obtained
using the MolarMass function in EES. The mole fraction of water vapor is:
Nv
yv = (5)
Nv + Na

n_v=m_v/molarMass(Water) "moles of water vapor"


n_a=m_a/molarmass(air) "moles of air"
y_v=n_v/(n_a+n_v) "mole fraction of water vapor"

which leads to yv = 0.0124.

c.) the mass concentration of the water vapor

The total volume of the mixture is obtained from the ideal gas law:

PV = Ru ( N v + N a ) T (6)

where Ru is the universal gas constant. The mass concentration of water vapor is:

mv
cv = (7)
V

P*V=(n_a+n_v)*R#*T "total volume"


c_v=m_v/V "mass concentration of water vapor"

which leads to cv = 0.00897 kg/m3.

d.) the molar concentration of the water vapor

The molar concentration of the water vapor is:

Nv
nv = (8)
V

nn_v=n_v/V "molar concentration of water vapor"

which leads to nv = 0.000498 kgmol/m3.

e.) the maximum possible value for the mole fraction of the water vapor at equilibrium.

The maximum possible mole fraction is obtained from:

yv ,max P = Pv , sat (9)

where Pv,sat is the saturation pressure of the water vapor at temperature T, obtained using the EES
function P_sat.

y_max*P=P_sat(Water,T=T) "maximum mole fraction"

which leads to yv,max = 0.042.


Problem 9.2-1 (9-3 in text): Moisture transfer rates
The air-conditioning load for a building can be broken into latent and sensible
contributions. The latent represents the energy that must be expended to remove the
water vapor from the building. Water vapor enters by infiltration as air from outdoors
leaks inside and by diffusion through the walls and ceiling. The building in question is
rectangular with outer dimensions of 40 ft by 60 ft with 8 ft ceilings. The infiltration
rate is estimated at 0.65 air changes per hour. The diffusion coefficient for water
through 3/8 inch gypsum board (without a vapor barrier) is approximately 4.5x10-5 ft2/s at
atmospheric pressure.
a.) Estimate and compare the rates of moisture transfer by infiltration and diffusion on a day in
which the outdoor conditions are 95°F and 45% relative humidity and indoor conditions are
75°F, 40% relative humidity. Is the contribution by diffusion significant? If not, then why
are people concerned with water vapor diffusion in a building?

The known information is entered in EES:

$UnitSystem SI MASS RAD PA K J


$Tabstops 0.2 0.4 0.6 3.5 in

"known information"
H=8 [ft]*convert(ft,m) "height of ceiling"
Width=40 [ft]*convert(ft,m) "width of building"
Length=60 [ft]*convert(ft,m) "length of building"
L=(3/8) [in]*convert(in,m) "width of gypsum board"
D_wg=0.000045 [ft^2/s]*convert(ft^2/s,m^2/s) "diffusion coefficient"
T_out=convertTemp(F,K,95 [F]) "outdoor temperature"
rh_out=0.45 [-] "relative humidity outddoors"
T_in=convertTemp(F,K,75 [F]) "indoor temperature"
rh_in=0.40 [-] "relative humidity indoors"
AirChangeRate=0.65 [1/hr]*convert(1/hr,1/s) "air change rate"

The total area of the building exposed to diffusion is computed:

A = Walls + Ceiling (1)

where

Walls = 2 Width H + 2 Length H (2)

and

Ceiling = Width Length (3)

The volume of the building is:

Volume = Ceiling H (4)

Ceiling=Width*Length "area of ceiling"


Walls=2*Width*H+2*Length*H "area of walls"
A=Walls+Ceiling "total area"
Volume=Ceiling*H "volume of air in building"

The saturation pressure at the outdoor temperature (psat,out) is obtained using the P_sat function in
EES. The outdoor vapor pressure is:

pv ,out = psat ,out rhout (5)

The concentration of the water vapor in the outdoor air is:

pv ,out
cv ,out = (6)
Rv Tout

where Rv is the gas constant for the vapor.

P_sat_out=p_sat(water,T=T_out) "satuation pressure outdoors"


P_v_out=rh_out*P_sat_out "definition of relative humidty"
R_v=R#/MolarMass(Water) "gas constant"
c_out=P_v_out/(R_v*T_out) "concentration of water vapor in outdoor air from ideal gas law"

The saturation pressure at the indoor temperature (psat,in) is obtained using the P_sat function in
EES. The indoor vapor pressure is:

pv ,in = psat ,in rhin (7)

The concentration of the water vapor in the indoor air is:

pv ,in
cv ,in = (8)
Rv Tin

P_sat_in=p_sat(water,T=T_in) "satuation pressure indoors"


P_v_in=rh_in*P_sat_in "definition of relative humidty"
c_in=P_v_in/(R_v*T_in) "concentration of water vapor in indoor air from ideal gas law"

The mass flow rate of vapor due to diffusion through the walls is given by Fick's law:

A Dwg
m v =
L
(c
v , out − cv ,in ) (9)

The mass flow rate of vapor due to infiltration is:

m v ,int = AirchangerateVolume ( cv ,out − cv ,in ) (10)

m_dot_v=A*D_wg*(c_out-c_in)/L "Fick's law determines diffusion rate of water vapor"


m_dot_v_int=AirChangeRate*Volume*(c_out-c_in) "mass of vapor entering due to infiltration"
which leads to m v = 0.00149 kg/s and m v ,int = 0.000898 kg/s. It is interesting that the two modes
of moisture transport are nearly the same. Diffusion is an additional concern because it affects
the properties and life of materials. This is particularly a concern in winter in which water that
diffuses into insulation reduces its insulating capability.
Problem 9.2-2 (9-4 in text): Transport of Natural Gas
Natural gas (methane) is transported at 25ºC and 100 bar over long distances through 1.2 m
diameter pipelines at a velocity of 10 m/s. The pipeline is made of steel with a wall thickness of
2.0 cm. It has been suggested that hydrogen gas could be transported in these same pipelines.
However, hydrogen is a small molecule that diffuses through most materials. The diffusion
coefficient for hydrogen in steel is about 7.9x10-9 m2/s at 25ºC.
a.) Calculate the power transported by methane (assuming it will be combusted) through the
pipeline. The lower heating value of methane is 5.002x107 J/kg.

The input information is entered in EES:

$UnitSystem SI MASS RAD PA K J


$Tabstops 0.2 0.4 0.6 3.5 in

"known information"
T=convertTemp(C,K,25 [C]) "temperature"
P=100 [bar]*convert(bar,Pa) "pressure"
D=1.2 [m] "pipeline diameter"
th=2 [cm]*convert(cm,m) "pipe wall thickness"
u_ng=10 [m/s] "velocity of natural gas"
LHV_ng=5.002e7 [J/kg] "lower heating value of methane"
D_h_steel=7.9e-9 [m^2/s] "diffusion coefficient for hydrogen through steel"

The cross-sectional area of the pipe is:

D2
Ac = π (1)
4

The density of methane (ρng) is computed and the mass flow rate of methane is:

m ng = Ac ρ ng ung (2)

and the power associated with this mass flow rate is:

Power = m ng LHVng (3)

A_c=pi*(D/2)^2 "cross-sectional area of pipeline"


rho_ng=density('Methane',T=T,P=P) "density of methane"
m_dot_ng=A_c*u_ng*rho_ng "mass flow rate of methane"
Power=LHV_ng*m_dot_ng "power associated with methane flow"
Power_MW=Power*convert(W,MW) "in MW"

which leads to Power = 42990 MW.

b.) Estimate the velocity required to move the provide the same power if hydrogen rather than
methane is transported through the pipeline at the same temperature and pressure. The lower
heating value of hydrogen is 1.200x108 J/kg.
The mass flow rate of hydrogen required is:

Power = m h LHVh (4)

The density of hydrogen (ρh) is determined and used to compute the velocity according to:

m h = Ac ρ h uh (5)

LHV_h=1.2e8 [J/kg] "lower heating value of hydrogen"


Power=LHV_h*m_dot_h "mass flow rate of hydrogen"
rho_h=density('Hydrogen',T=T,P=P) "density of hydrogen"
m_dot_h=A_c*u_h*rho_h "velocity of hydrogen"

which leads to uh = 41.3 m/s.

c.) Compare the pumping power required to transport the natural gas and hydrogen a distance of
100 km.

The PipeFlow procedure is used to determine the pressure drop associated with the flow of
methane (Δpng). The minimum pump power is:

m ng
w ng = Δpng (6)
ρ ng

L=100 [km]*convert(km,m) "length of pipeline"


call PipeFlow('Methane',T,P,m_dot_ng,D,L,0 [-]:h_bar_ng_T, h_bar_ng_H ,DELTAP_ng,&
Nusselt_bar_ng_T, f_bar_ng, Re_ng)
Power_pump_ng=m_dot_ng*DELTAP_ng/rho_ng "minimum natural gas pumping power"
Power_pump_ng_MW=Power_pump_ng*convert(W,MW) "in MW"

which leads to w ng = 22.6 MW (0.05% of the total power flow). The PipeFlow procedure is used
to determine the pressure drop associated with the flow of hydrogen (Δph). The minimum pump
power is:

m h
w h = Δph (7)
ρh

call PipeFlow('Hydrogen',T,P,m_dot_h,D,L,0 [-]:h_bar_h_T, h_bar_h_H ,DELTAP_h, Nusselt_bar_h_T,&


f_bar_h, Re_h)
Power_pump_h=m_dot_h*DELTAP_h/rho_h "minimum hydrogen pumping power"
Power_pump_h_MW=Power_pump_h*convert(W,MW) "in MW"

which leads to w h = 171 MW (0.4% of the total power flow).

d.) Estimate the rate of hydrogen loss from a 100 km pipeline. Do you believe this loss is
significant?
The concentration of the hydrogen in the pipe (cpipe) is equal to its density. The concentration of
the hydrogen outside of the pipe (camb) is zero. Fick's law is used to compute the rate of
hydrogen diffusion.

m h ,dif = Dh , steel π D L
(c pipe − camb )
(8)
th

c_pipe=rho_h "concentration (density) of hydrogen in the pipe"


c_amb=0 [kg/m^3] "no hydrogen outside of the pipe"
m_dot_h_dif=D_h_steel*pi*D*L*(c_pipe-c_amb)/th "Fick's Law"

which leads to m h ,dif = 1.141 kg/s (0.3% of the flow).


Problem 9.2-3 (9-5 in text): Helium Balloon
A balloon made of a synthetic rubber is inflated with helium to a pressure of pini = 130 kPa at
which point its diameter is Dini = 0.12 m. The mass of the balloon material is Mbal = 0.53 g and
its thickness is δ = 0.085 mm. The balloon is released in a room that is maintained at T = 25ºC
filled with air (yN2 = 79% nitrogen and yO2 = 21% oxygen) at patm = 100 kPa. Over a period of
time, helium diffuses out of the balloon and oxygen and nitrogen diffuse in. The pressure in the
balloon above atmospheric pressure is linearly proportional to the balloon volume. The diffusion
coefficients for helium, oxygen and nitrogen through the synthetic rubber are DHe,rubber = 60x10-8,
DO2,rubber = 16x10-8, and DN2,rubber = 15x10-8 cm2/s, respectively.
a.) Prepare a numerical model of the balloon deflation process. Plot the volume and pressure
within the balloon as a function of time. Plot the mole fraction of helium, oxygen, and
nitrogen in the balloon as a function of time.

The inputs are entered in EES:

$UnitSystem SI MASS RAD PA K J


$Tabstops 0.2 0.4 0.6 3.5 in

"known information"
T=converttemp(C,K,25 [C]) "temperature of balloon and air"
P_atm=100 [kPa]*convert(kPa,Pa) "atmospheric pressure"
P_ini=130 [kPa]*convert(kPa,Pa) "initial balloon pressure"
D_ini=0.12 [m]

Вам также может понравиться